Está en la página 1de 3734

ECU 1:

• Pregunta 1
3,3333 de 3,3333 puntos
Estudiante de medicina de la UPC de 21 años sufre de gastritis aguda ocasionada por comer en lugares poco
higiénicos. Suele consumir caramelos ( chupar ) mientras está en clase hasta la tarde. Toma gaseosas
regularmente (carbohidratos 46%, sodio 53%). También toma regular cantidad de leche (grasa 35%, lactosa 35%,
proteínas 30%), pues le calma un poco el dolor el ardor que siente por la gastritis. Incluso, cuando puede, se toma
dos vasos de agua fría para calmar las molestias. Ha decidido ir al médico para tratarse pues ya no soporta el dolor,
el cual está seguro que los síntomas se deben a una elevada producción de ácido clorhídrico en el estómago, y por
ello le ha recetado Ranitidina (antihistamínico), con lo que siente mejoría.

Al consumir leche se eleva los niveles séricos de la hormona:


Colecistoquinina (CCK)

El consumo de una pequeña cantidad de gaseosa aumentará directamente la concentración sérica de cuál de las
siguientes hormonas:
Péptido 1 similar al glucagón (GLP-1)

El uso de atropina en este paciente:

Aumentará el pH del estómago

El consumo rápido de 500 mL de gaseosa aumentará directamente la concentración sérica de cuál de las
siguientes hormonas:
gastrina
• Pregunta 2

En este paciente con gastritis aguda debida a una alta producción de ácido clorhídrico, si se le hiciera un
examen de sangre, se encontraría elevados los niveles de:

Colecistoquinina

• Pregunta 3

El consumo de dos vasos de agua seguidos agua generará indirectamente un aumento en la liberación
de:
Ácido clorhídrico

• Pregunta 4

El consumo de leche produce directamente un aumento de los niveles séricos de la hormona:


Gastrina

• Pregunta 5
3,3333 de 3,3333 puntos
Antes del uso de Ranitidina en este paciente, los valores de somatostatina en sangre están:
Aumentados

• Pregunta 6
0 de 3,3333 puntos
Este paciente tiene también la costumbre de tomar agua antes de sentarse a almorzar, la cual hace que la
producción de ácido clorhídrico del estómago:
Aumente

ECU 2

Niño de sexo masculino de 2 años de edad, sufre de estreñimiento desde el nacimiento (1 deposición cada 3-4 días).
Madre menciona que le estimula la defecación con un termómetro rectal, y continuo uso de enemas y laxantes.
Desde hace 6 meses comienza con vómitos postprandiales. Los síntomas aumentan en frecuencia y magnitud y
están en relación con los episodios de estreñimiento. No refiere fiebre, tos, diarrea ni lesiones cutáneas. Al examen
físico presenta regular estado general, luce deshidratado. Abdomen distendido, blando, depresible e indoloro. No se
palpan masas abdominales. Se permeabiliza el canal anal con termómetro rectal, encontrando cierta resistencia.
Salida de material fecal mal oliente en regular cantidad. Exámenes de laboratorio: hemograma normal. Signos
inflamatorios de fase aguda negativos. Alcalosis metabólica leve en sangre venosa. Radiografía con enema baritado
muestra recto y colon sigmoides dilatados (megacolon). Biopsia profunda: ausencia de células ganglionares en la
muestra enviada. Se realiza cirugía correctiva.

Si este paciente tuviera arcadas, entonces se debe considerar que durante la ocurrencia de dichas arcadas,
debemos encontrar contenido gástrico en:

esófago
Considerando que este paciente está sometido a estrés por el agravamiento de su enfermedad, es posible
afirmar que sus ondas lentas están:

Hiperpolarizadas
Con respecto a la defecación en este caso, marque la respuesta correcta:
:
El esfínter anal comprometido tiene inervación autónoma
Para estimular la motilidad intestinal se podría usar un análogo de:

CCK
Durante la fase faríngea de la deglución es espera que ocurra el siguiente fenómeno:
Respuesta seleccionada:
El paladar blando cierra la entrada a la nasofaringe
En la Enfermedad de Hirschprung, el patólogo debe declarar un área como sana o libre de enfermedad si es
que encuentra:

Células intersticiales en el plexo mioentérico


ECU 3
Paciente de 54 años con antecedentes de alcoholismo, gastritis crónica, tabaquismo pesado, obesidad, cálculos
biliares y cirrosis, es llevado a la emergencia por dolor abdominal en epigastrio irradiado a la espalda y trastorno del
sensorio.
Al examen físico: presión arterial 85/50 mmHg, frecuencia cardíaca 100 latidos/min, frecuencia respiratoria 18 x
minuto, temperatura axilar 36°C.
Conjuntivas pálidas, escleras ictéricas nevus arácnidos en tronco, distensión abdominal marcada, cabeza de medusa,
matidez desplazable en ambos flancos e hipogastrio, dolor a la palpación de abdomen.
Tiempo de protrombina: 24 seg (testigo: 13 seg); TPT: 38 seg, glicemia: 165 mg/dL, uremia: 20 mg/dL, ASAT: 76
UI/L, ALAT: 22 UI/L, albumina: 2,5 g/dL, bilirrubina total: 2,6 mg/dL, bilirrubina directa: 1,4 mg/dL, amilasa sérica 4000
U/L.
En todas las preguntas marque la mejor respuesta.
Considerando que el paciente sufre de gastritis, se puede decir que la secreción de ácido por la mucosa gástrica:

Involucra transporte activo de Hidrogeniones


De las siguientes sustancias secretadas por los órganos de este paciente, la más alcalina es la secreción:

Respuesta seleccionada:
Pancreática
Respuestas: Esofágica

Pancreática
Salival
Yeyunal
En este paciente, el bloqueo farmacológico de los receptores de histamina H2 en la mucosa gástrica:

Respuesta seleccionada:
No tiene efecto sobre la secreción de ácido inducida por el vago
Respuestas: Inhibe la secreción de ácido inducida por gastrina y la mediada por el vago
Evita la activación de adenil ciclasa por la gastrina
Causa un aumento en el transporte de potasio por las células parietales gástricas.
No tiene efecto sobre la secreción de ácido inducida por el vago
En esta paciente, el aumento de la Amilasa sérica se debe directamente a una lesión de:

Respuesta seleccionada:
Páncreas
Respuestas: Hígado
Vesícula y árbol biliar

Páncreas
Estómago
Si se considerara una gastrectomía total para curar la gastritis de este paciente, cuál de las siguientes sustancias ya
no se produciría:

Respuesta seleccionada:
Gastrina
Respuestas: Quimiotripsina
Gastrina
Amilasa

Pepsinógeno

Un efecto secundario en el estómago por la acción de la secretina es:

Respuesta seleccionada:
Mayor paso de bicarbonato
a la sangre periférica
Respuestas: Mayor paso de bicarbonato
a la sangre periférica
Aumento en la producción
de factor intrínseco
Menor actividad de la
pepsina
Disminución de la liberación
de pepsinógeno

ECU4:
Durante la cirugía, el cirujano observó que además la inguinaltenía divertículos en el
sigma. Se sabe que estos divertículos:

Respuesta seleccionada:
Se pueden asociar a estreñimiento crónico
Respuestas: La mayoría de veces son congénitas

Se pueden asociar a estreñimiento crónico


Son más frecuentes en personas menores de 30 años
Tienen una muscular propia engrosada
El recto recibe irrigación de tres arterias principales, la zona que recibe suministro
de una rama de la mesentérica inferior, se caracteriza por tener epitelio:

Respuesta seleccionada:
Estratificado plano no queratinizado
Respuestas: Estratificado plano no queratinizado

Simple cilíndrico
Estratificado plano queratinizado
Estratificado cilíndrico
El canal anal es irrigado por una rama de la arteria:

Respuesta seleccionada:
Mesentérica inferior
Respuestas:
Pudenda interna
Mesentérica superior
Ileocolica
Mesentérica inferior
Al realizar un corte transmural de un segmento del colon transverso, será evidente
que:

Respuesta
seleccionada: En la zona donde no hay tenias solo hay una capa
muscular
Respuestas: Se aprecia pliegues semilunares sobre la superficie
serosa
En la zona donde no hay tenias solo hay una capa
muscular
Se observa apéndices epiploicos sobre la superficie
mucosa
Hay presencia de saculaciones o haustros en la superficie
adventicia
En esta paciente, ¿cuál de las siguientes sustancias no tendrá una considerable
disminución en su absorción? (marque la mejor respuesta):

Respuesta seleccionada:
Vitamina K
Respuestas:
Calcio
Vitamina K
Vitamina B12
Triglicéridos

CI1:
Con respecto a las ondas lentas, marque la afirmación correcta:

Respuesta seleccionada:
Son contracciones rítmicas espontáneas
Respuestas: Su frecuencia disminuye por acción de la noradrenalina
Su frecuencia aumenta por acción de la acetilcolina

Son contracciones rítmicas espontáneas


Son potenciales de acción que producen la contracción del tracto gastrointestinal
Las arterias que derivan del tronco celiaco son, excepto:

Respuesta seleccionada:
Hepática derecha
Respuestas: Hepática común

Hepática derecha
Gástrica izquierda
Arteria esplénica
La arteria palatina menor irriga un área de la boca cuyo epitelio es:

Respuesta seleccionada:
Escamoso queratinizado
Respuestas:
Escamoso no queratinizado
Escamoso queratinizado
Cúbico estratificado
Cilíndrico simple
Marque la respuesta correcta:
Respuesta seleccionada:
La pared gástrica en el fondo es más delgada que en el cuerpo y antro
Respuestas: Todos los órganos del sistema digestivo tienen capa serosa
El esfínter de Oddi rodea a la papila menor duodenal

La pared gástrica en el fondo es más delgada que en el cuerpo y antro


El bronquio derecho constituye una de las estrecheces del esófago
Sustancia que inhibe la secreción y la motilidad del estómago prolongando el tiempo de digestión:

Respuesta seleccionada: Péptido insulinotrópico dependiente de la glucosa (GIP).


Respuestas: Enteroglucagon.
Péptido insulinotrópico dependiente de la glucosa (GIP).
Péptido 1 similar al glucagón (GLP-1).
Polipéptido pancreático
En qué casos los vómitos son siempre biliosos?:

Respuesta seleccionada:
Atresia yeyunal.
Respuestas: Estenosis hipertrófica del píloro.
Atresia duodenal.
Atresia esofágica.

Atresia yeyunal.
El nacimiento de la arteria mesentérica superior se puede encontrar en cuál de los cuadrantes abdominales:

Respuesta seleccionada:
Mesogastrio
Respuestas: Hipogastrio
Mesogastrio
Hipocondrio derecho

Epigastrio
El dolor periumbilical o epigástrico en el inicio de una apendicitis aguda se debe a:

Respuesta seleccionada:
Irritación del peritoneo parietal.
Respuestas: Íleo secundario.

Estímulo del sistema simpático.


Estímulo del nervio vago.
Irritación del peritoneo parietal.
¿Cuál de los siguientes líquidos corporales tiene el pH más alto?:

Respuesta seleccionada:
Bilis en la vesícula biliar.
Respuestas: Saliva.
Bilis en la vesícula biliar.

Jugo pancreático.
Jugo gástrico.
La triada sintomática: vómitos explosivos post-prandiales, movimientos peristálticos epigástricos visibles de izquierda
a derecha y nódulo palpable epigástrico subcostal derecho, pertenecen a:

Respuesta seleccionada:
Estenosis congénita hipertrófica del píloro.
Respuestas: Páncreas anular.
Tumor benigno de las glándulas submucosas.

Estenosis congénita hipertrófica del píloro.


Colon irritable.
CI2
Paciente que come entera una pizza familiar de chorizo y queso. Es posible esperar que debido a la cantidad de
alimento ingerida, las ondas lentas hayan:

Respuesta seleccionada:
Sufrido ninguna alteración en su frecuencia
Respuestas: Disminuido su frecuencia por estímulo parasimpático
Aumentado su frecuencia por estímulo simpático

Sufrido ninguna alteración en su frecuencia


Aumentado su frecuencia por estímulo parasimpático
En este caso se puede afirmar con seguridad que se presenta:

Respuesta seleccionada:
dilatación de tracto gastrointestinal afectado
Respuestas: dilatación de tracto gastrointestinal afectado

contracciones tónicas en la región ano rectal


células ganglionares sólo en el ano recto
pérdida de inervación por el nervio esplácnico menor
La arteria mesentérica superior emerge de la aorta a nivel de:

Respuesta seleccionada:
Tronco celiaco
Respuestas: Cardias
Tronco celiaco
Hilio hepático

Cabeza de páncreas
Estudiante de medicina de 20 años, se ha amanecido estudiando para su examen de Sistema Digestivo. No ha
probado alimento desde la cena, por lo que se puede afirmar que la motilidad de esta persona está siendo regulada
por:

Respuesta seleccionada:
Adrenalina
Respuestas: VIP

Motilina
CCK
Adrenalina
Al comer unas papitas fritas con mayonesa, el vaciamiento gástrico disminuye por efecto directo de la hormona:

Respuesta seleccionada:
colecistoquinina (CCK)
Respuestas: motilina
secretina
somatostatina

colecistoquinina (CCK)
Paciente con disminución del apetito marcada asociada a cáncer terminal, para promover la ingesta de alimentos se
podría usar análogos de:

Respuesta seleccionada:
Endorfinas
Respuestas: Secretina
GLP

Endorfinas
Colecistoquinina CCK
El crecimiento de un adenocarcinoma de páncreas compromete la pared gástrica por contigüidad. ¿Qué parte del
estómago se esperaría esté comprometido?

Respuesta seleccionada:
Pared posterior del antro
Respuestas: Pared anterior del píloro
Pared anterior del cardias
Pared posterior del fondo

Pared posterior del antro


Paciente de seco masculino de 43 años que es alimentado por vía intravenosa durante varias semanas. Producto de
este tipo de alimentación, se encuentra en la endoscopía atrofia de la mucosa antral. La causa más probable de esta
alteración es debido a los bajos niveles séricos de qué hormona:

Respuesta seleccionada:
Gastrina
Respuestas: Colecistocinina
GIP
Gastrina
Secretina
La forma más común de atresia esofágica contiene:

Respuesta seleccionada:
Estenosis proximal del esófago más fístula traqueoesofágica distal
Respuestas: Estenosis distal del esófago más fístula traqueoesofágica proximal
Estenosis distal del esófago más fístula traqueoesofágica distal

Estenosis proximal del esófago más fístula traqueoesofágica distal


Estenosis proximal y distal del esófago más fístula traqueoesofágica distal
La hernia fisiológica se produce dentro de:

Respuesta seleccionada:
Cordón umbilical
Respuestas: Alantoides

Cordón umbilical
Saco Amniótico
Saco Vitelino
CI3
Cuando el alimento se encuentra en el estómago, se produce la liberación de enzimas pancreáticas básicamente
debido a la acción de:

Respuesta seleccionada:
Vago
Respuestas:
Vago
Bombesina
Colecistoquinina (CCK)
Secretina
Marque lo correcto:

Respuesta seleccionada:
Ninguna es correcta
Respuestas: Ninguna es correcta
Las venas hepáticas contienen válvulas
Usualmente, son dos grandes venas suprahepáticas

Las venas sublobulillares desembocan en las venas hepáticas


En relación a la histología hepática, marque lo correcto:

Respuesta
seleccionada: La zona 3 se encuentra más cerca a la vena central lobulillar
Respuestas:
La zona 3 se encuentra más cerca a la vena central lobulillar
A zona 1 se encuentra cercana a la vena central lobulillar
La zona 1 se afecta rápidamente en estados de hipovolemia y
shock
La zona 3 se encuentra más cerca al eje menor el acino
hepático
¿Por cuál de las siguientes células es secretada principalmente la pro enzima
procarboxipeptidasa?

Respuesta seleccionada:
Acinares del páncreas
Respuestas: Centroacinares del páncreas

Acinares del páncreas


Epiteliales del duodeno
Ductales del páncreas
Un signo característico en pacientes con encefalopatía hepática es:

Respuesta seleccionada:
Asterixis
Respuestas: Onicomicosis
Reflejo de Babinsky

Asterixis
Convulsiones tónicas
En cuanto a la secreción de iones y agua en los conductos biliares es correcto lo siguiente:

Respuesta seleccionada:
Es estimulada principalmente por la colecistoquinina (CCK)
Respuestas: Es estimulada principalmente por la colecistoquinina (CCK)
Se produce reabsorción, no secreción
Es estimulado por la lipasa pancreática

Es estimulada por la secretina


La secreción de la colecistoquinina (CCK) se produce en la fase:

Respuesta seleccionada:
Intestinal
Respuestas: En las tres por igual
Cefálica

Intestinal
Gástrica
Paciente con tumor neuroendocrino productor de secretina, debido a lo cual se puede esperar
que su secreción pancreática, comparada con la de una persona sana en estado de bajo flujo,
tenga una concentración de:

Respuesta seleccionada:
sodio aumentada
Respuestas: Igual
sodio aumentada

bicarbonato aumentada
potasio disminuida
La vena central terminal desemboca en:

Respuesta seleccionada:
Vena porta
Respuestas: Espacio de Disse

Venas sublobulillares
Vena porta
Sinusoides hepáticos
Durante una endoscopía, se ingresa en la papila mayor en la segunda porción del duodeno, y
dentro de la vía biliar extrahepática, se toma una muestra de secreción biliar. Dicha muestra
debe contener ácido:

Respuesta seleccionada:
Litocólico
Respuestas: Litocólico
Litodesoxicólico

Quenodesoxicólico
Desoxicólico
CI4

El alcohol y la cafeína estimulan la fase ______________ de la producción de ácido clorhídrico:

Respuestas: intestinal

gástrica
cefálica
gástrica y cefálica
La saliva puede tener una variedad de electrolitos en su composición. Entre ellos el cloro, respecto al cual se puede
afirmar:

Respuestas: Con flujo alta, su concentración es mayor que la del plasma


Su mayor concentración se consigue con flujo bajo
Su menor concentración se alcanza con flujo alto

Su concentración no llega a ser tan alta como en el plasma


El omeprazol actúa sobre la membrana _____________ de la célula ____________

Respuestas: Basolateral / principal


basolateral / parietal

apical / parietal
Apical / principal
Respecto a la anatomía del estómago, marque lo correcto:

Respuestas: la arteria gástrica izquierda irriga la curvatura mayor del estómago


No contiene muscular de la mucosa

la arteria gástrica derecha nace de la arteria hepática común


Tiene una capacidad de 3 litros en la mayoría de personas
La atropina es el tratamiento para la intoxicación por órganos fosforados. Se administra hasta alcanzar niveles
terapéuticos. Un buen indicador que la atropina ya está en niveles terapéuticos es:

Respuestas: Humidificación de la mucosa oral y sialorrea

Sequedad de la mucosa oral


No tiene acción sobre la secreción salival
La actividad sobre la secreción salival se realiza mediante receptores nicotínicos

En el síndrome de boca seca o síndrome de Sjogren, una de las complicaciones asociadas es:
Respuestas:
Caries
Disminución de la acidez gástrica
Aumento de la producción de saliva
Aumento del pH bucal

Los músculos de la masticación que producen la retropulsión de la mandíbula son:

Respuestas: milohioideos
pterigoideos

temporales
maseteros
Con respecto a la secreción gástrica de HCl: LA HICE MAL

Respuestas: la fase intestinal es responsable del 30% de la secreción de HCl


La gastrina eleva el pH

a mayor secreción de HCl en el lumen gástrico, mayor pH en la sangre venosa gástrica


en la fase cefálica, la bombesina liberada por impulsos vagales inhibe a las células G para la
secreción de gastrina

Respecto a la motilidad del colon, marque lo correcto:


Respuestas:
Con la distención del estómago, suelen aparecer movimientos en masa
Se producen contracciones segmentarias principalmente en el colon izquierdo
Los movimientos de masa se encargarán del mezclado de las heces
Las haustras del colon contribuyen al reflejo de defecación
Para protegerse del entorno ácido, el Helicobacter pylori se autogenera un entorno de pH
menos ácido alrededor suyo, gracias a una enzima que alcaliniza su entorno local mediante la
conversión de: LA HICE MAL

Respuestas: NH3 en urea


H2O y CO2 en ácido carbónico

Urea en NH3
H2CO3 en bicarbonato

Un paciente varón de 68 años consulta por dolor y abultamiento en la región inguinal derecha, que aparece
tras la realización de un esfuerzo físico. En la exploración en bipedestación presencia de una tumoración
blanda, depresible, que aumenta con la tos. El presente caso describe una hernia:

Inguinal directa.
Umbilical.
Femoral.
Inguinal indirecta

la Grelina es sintetizada por………….. y activa las neuronas relacionadas con ……………………….en el núcleo
arqueado del hipotálamo}

El estómago/AGRP-NPY

Cuál de los siguientes órganos son intraperitoneales:


Estómago, Vesicula biliar, Ileón, Hígado

La distención gástrica por los alimentos produce incremento de secreción de HCl mediante el siguiente
mecanismo

Producción de Gastrina que desencadena su cascada de señalización en la célula parietal vía proteína Gq

La inervación de la piel del abdomen debajo del ombligo hasta la región púbica está dada por:

T11, T12, L1

Paciente de 38 años que tras riña durante partido de futbol sufre un traumatismo con arma blanca en
cuadrante inferior izquierdo del abdomen. En la tomografía de urgencias se evidencia gran hematoma de
pared y ausencia de neumoperitoneo. Desde la piel hacia al peritoneo, en ¿qué orden se atravesó la pared
abdominal?

TCSC, oblicuo externo, oblicuo interno, musculo transverso


Alrededor de 90% de los pacientes afectados por el síndrome de Zollinger-Ellison desarrollan ulcera péptica.
La causa es:

Hipersecreción ectópica de gastrina

Un hombre de 22 años sufre traumatismo en el flanco izquierdo superior del abdomen al tratar de
defenderse del robo de su vehículo. Un amigo lo traslada a un centro hospitalario. El paciente presenta
signos de hipovolemia con taquicardia e hipotensión. Se queja de dolor en el sitio de lesión que se irradia al
hombro izquierdo. Elñ mórgano probablemente lesionado es
Bazo
Aproximadamente en la semana 6 del desarrollo embrionario, el intestino medio gira 90° herniándose a nivel
del:
Cordón umbilical

La triada portal (arteria hepatica, vena portal y conducto biliar común) está contenida en el
Ligamento hepatoduodenal

En el siguiente gráfico marque la relación correcta

Plexo de Meissner – 1

¿Cuál de las siguientes sustancias es liberada por neuronas en el tracto GI, Participa en la regulación hidro
electrolíticay produce relajación del musculo liso?

VIP

La colecistoquinina inhibe
El Vaciamiento gástrico

¿La secreción de cuál de las siguientes es inhibida por un pH bajo?


Gastrina

1. ¿Cuál de las glándulas salivales es responsable del mayor porcentaje del volumen de la saliva en condiciones basales?
a. Parótida
b. Subpalatinas
c. Sublingulaes
d. Submaxilares

2. La lengua está recubierta por epitelio:


a. pseudoestratificado columnar no queratinizado
b. plano estratificado no queratinizado
c. pseudoestratificado columnar ciliado
d. plano estratificado queratinizado

3. El esfínter anal interno tiene musculatura …………… y tiene control ………………….


a. lisa / voluntario
b. lisa / involuntario
c. esquelética / simpático
d. esquelética / parasimpático

4. La arteria aorta proporciona la irrigación al tubo digestivo ¿cuál de las siguientes arterias proporciona la irrigación al ángulo
cólico derecho?
a. Mesentérica superior
b. Mesentérica inferior
c. Frénica inferior
d. Tronco celiaco

5. Paciente de 26 años que le cuenta en su historia clínica que cada vez que almuerza, a los 20 minutos tiene deseo de defecar.
Le comenta que su hijo de 1 mes le pasa lo mismo pero más intenso. Esto se explica por el reflejo …………………, el cual está
……………..en el paciente.
a. colicoileal / normal
b. colicoileal / alterado
c. gastrocolico / normal
d. gastrocolico / alterado

6. La región del estómago que se comunica con el duodeno se denomina:


a. pilórica
b. cardias
c. cuerpo
d. fórnix

7. Acude a consulta un paciente que fue diagnosticado de ulcera péptica 3 días antes. Luego de múltiples pruebas diagnósticas,
se concluye que el paciente presenta un tumor secretor de gastrina ¿Cuál de las siguientes situaciones estará incrementada?
a. Distención gástrica
b. Inhibición del vaciado gástrico
c. Secreción de ácido clorhídrico (HCl)
d. Inhibición de la secreción de pepsinógeno

8. En el sistema digestivo, el control del apetito esta dado por un complejo sistema de sustancias y órganos integradores los
cuales regulan la ingesta de alimentos. La ……………… es una sustancia oroxígena y es sintetizada por el ……………………..
a. leptina / intestino
b. grelina / intestino
c. leptina / estómago
d. grelina / estómago

9. Sobre el control autónomo del sistema digestivo, marque la alternativa correcta:


a. La inervación dada por el sistema simpático es de tipo preganglionar.
b. El sistema parasimpático usa como neurotransmisores a la acetilcolina y la noradrenalina.
c. El nervio vago (par craneal X) le da inervación simpática a la mayoría del sistema digestivo.
d. En el sistema simpático, los nervios responsables hacen una primera sinapsis en ganglios próximos al órgano a inervar.
e. En la inervación de tipo parasimpático, solo interviene el plexo submucoso, sin embargo, en la de tipo simpático intervienen
tanto el submucoso como el mientérico.

10. Con respecto a la actividad eléctrica del sistema digestivo, marque la alternativa correcta
a. Corresponden a potenciales de acción que están presentes de forma continua y le dan la capacidad de perístasis autónoma al
sistema digestivo.
b. La frecuencia de las ondas lentas no se ve influenciada por la actividad neural ni las hormas gastrointestinales.
c. En el estómago las ondas lentas se dan en una frecuencia de 6 por minuto.
d. Las ondas lentas son cambios lentos y ondulantes del potencial en reposo.
e. La frecuencia de las ondas lentas va de 6 a 12 ondas por minuto.

11. Ante una lesión del IX par craneal, el músculo……………….. se altera en su función.
a. palatogloso
b. estilofaríngeo
c. palatofaríngeo
d. constrictor superior
12. Un varón de 50 años es sometido a extirpación de duodeno y parte proximal de yeyuno. La pérdida de estímulo hormonal en
el páncreas para la secreción enzimática se explica por la pérdida de las células ……………………
a. Parietales, productoras de factor intrínseco
b. “K” productoras de factor intrínseco
c. “M” productoras de CCK
d. “I” productoras de CCK
13. Respecto al mecanismo de la defecación ¿Cuál de las siguientes afirmaciones es correcta?
a. Se produce contracción refleja del esfínter anal interno
b. Se produce contracción o relajación del esfínter anal externo por señales de la corteza cerebral
c. La presencia de materia fecal en el recto estimula la contracción del sigmoides por los nervios pélvicos simpáticos
d. En la posición de “cuclillas” el músculo puborectal se halla contraído favoreciendo la evacuación de la materia fecal

14. Un niño de tres años llega a emergencia con disfagia (dificultad para tragar), salivación y llanto. Se sospecha de ingesta de
cuerpo extraño: moneda en el esófago; al ser evaluado se constata en una radiografía presencia de cuerpo extraño a nivel de C6
y C7 (6° y 7° vértebra cervical). El cuerpo extraño estará suspendido a nivel del estrechamiento producido por el ………..
a. cayado aórtico
b. hiato esofágico
c. músculo cricofaríngeo
d. bronquio principal izquierdo

15. En el caso de un paciente con un tumor productor de gastrina, la presencia de úlceras duodenales y erosión de la mucosa
gástrica se debe principalmente a…….
a. la acción paracrina de la gastrina sobre la célula parietal
b. el exceso de HCl por estímulo de receptores CCK-B en la célula parietal
c. la sobre expresión de los receptores “G” para gastrina en la célula parietal
d. el exceso de HCl por estímulo directo de receptores “H” en la célula parietal

16. La onda peristáltica secundaria del esófago se caracteriza por ser originada ………
a. por el plexo de submucoso del esófago
b. por el plexo mientérico del esófago
c. por el reflejo de la deglución
d. durante la masticación

17. Marque lo correcto sobre las ondas lentas en el tubo digestivo


a. No son despolarizaciones
b. Son potenciales de acción subumbrales
c. Se constituyen de despolarizaciones y repolarizaciones
d. Son rítmicas y generadas por el sistema nerviosos autónomo

18. Recién nacido que presenta protrusión de contenidos abdominales los cuales no están cubiertos por peritoneo y salen de la
cavidad abdominal a través de un defecto de la pared. ¿Cómo se denomina a la afección que presenta este paciente?
a. Onfalocele
b. Atresia biliar
c. Gastrosquisis
d. Divertículo de Meckel

19. Experimentalmente se utiliza atropina (anticolinérgico) para inhibir la secreción de gastrina, sin embargo, la secreción de
esta hormona se sigue dando ante estímulos vagales. Esta situación se explica porque la atropina:
a. no bloquea la acción del péptido GRP
b. solo inhibe la acción del péptido GRP en la célula G
c. inhibe la acción de acetilcolina e histamina en la célula G
d. bloquea parcialmente la bomba de protones en la célula G

20. Niña de 4 días es llevada a la emergencia pediátrica por presentar llanto constante, la madre refiere coloración azulada de
labios al momento de lactar, acompañado de tos persistente y dificultad respiratoria así como distención abdominal. Se le coloca
sonda nasogástrica para alimentación notando que retorna a la cavidad oral en todos los intentos. ¿Cuál es la anomalía del
desarrollo en este caso?
c. Atresia esofágica proximal con fístula traqueo esofágica distal
21. Paciente varón de 36 años es traído a la emergencia luego de sufrir un accidente de tránsito, presenta traumatismos
múltiples en cabeza y tronco. Al examen físico se evidencia hematoma en hemicara izquierda, ligera protrusión y caída del lado
izquierdo del maxilar inferior, por lo que se realiza una tomografía donde se halla una fractura de la apófisis coronoides del
maxilar inferior. ¿Qué músculo está relacionada directamente con esta situación?
a. Masetero
b. Temporal

22. Un paciente refiere no percibir algunos sabores. Al examen físico constata alteración del sabor dulce y umami. ¿Cuál de los
siguientes nervios estará alterada su función?
a. Cuerda del tímpano (VII par)

23. A los pocos días de nacido, regresa a neonatología un niño con problemas de motilidad del colon; los estudios determinan
ausencia congénita de células ganglionares. Según el gráfico ¿cuál es la capa en la que se determina la ausencia de dichas
células?
c. Muscular propia - 3

24. Con respecto al control autonómico en el tracto


gastrointestinal y en relación a su fisiología. ¿Cuál es la función del sistema nervioso parasimpático en el tracto gastrointestinal?
a. Inhiben la contracción muscular y estimulan la secreción de sustancias a nivel de la submucosa
b. Estimulan la contracción muscular y estimulan la secreción de sustancias a nivel de la mucosa
c. Inhiben la contracción muscular e inhiben la secreción de sustancias a nivel de la submucosa
d. Estimulan la contracción muscular e inhiben la secreción de sustancias a nivel de la mucosa

25. Un estudiante que está preocupado por su examen parcial, no ha desayunado ni almorzado; cuando al fin ingiere alimentos,
esto le provoca el aumento de los movimientos musculares del tracto gastrointestinal y la sensación de defecar. ¿Qué reflejo se
ha activado?
a. Entero-gástrico
b. Gastro-cólico
c. Cólico-ileal
d. Ileo-ileal
26. ¿De qué par craneal es rama el nervio palatino mayor?
a. Vago
b. Hipogloso
c. Trigémino
d. Palatogloso

27. ¿En cuál de las fases de la deglución la epiglotis separa la vía respiratoria de la digestiva?
a. oral
b. laríngea
c. faríngea
d. esofágica

28. Los péptidos intestinales se pueden clasificar como sustancias endocrinas, neurocrinas y paracrinas, dentro de las paracrinas
se encuentran la somastotatina e histamina. Marque la respuesta correcta
a. La somastotatina es sintetizada por las células B de la mucosa gástrica
b. La histamina actúa estimulando su receptor tipo H1 en la mucosa gástrica
c. La histamina es sintetizada por células de tipo paracrino de las glándulas gástricas
d. La somatostatina presenta dentro de sus funciones la estimulación de la secreción de H+
29. En relación a los órganos intraabdominales y sus estructuras de fijación, elija la alternativa correcta
a. El mesenterio permite la suspensión e irrigación de los órganos retroperitoneales
b. Tanto el hígado como la vesícula biliar se encuentran ubicados a nivel del flanco derecho
c. El colon, el duodeno y el resto de intestino delgado son órganos considerados netamente como peritoneales
d. Los ligamentos que encontramos dentro de la cavidad abdominal son el esplenorenal y el gastrofrénico
e. Los omentos van desde el estómago y la segunda porción del duodeno a otras estructuras intraabdominales y existen dos: el
omento mayor y el omento menor

30. Paciente varón de 27 años es llevado por bomberos a emergencia luego de ser asaltado y, tras resistirse, es cortado con el
pico de una botella a nivel abdominal. Al examen físico usted observa que a través de la herida se puede observar la protrusión
de asas intestinales. En relación con las capas de la pared abdominal, marque la alternativa correcta.
a. La fascia de Scarpa está constituida principalmente por tejido adiposo
b. La pared abdominal está formada por piel, huesos, músculos, fascias y peritoneo parietal
c. La fascia de Camper es una estructura fibrosa que carece de grasa y su grosor es constante en toda la pared abdominal
d. El músculo oblicuo externo discurre en dirección súpero-interna y se inserta en el borde inferior de las ultimas 3 a 4 costillas
e. El músculo recto del abdomen tiene como funciones comprimir el contenido del abdomen, tensar la pared del abdomen y
flexionar la columna

1. la contracción del músculo permite la eliminación de gases sin salida de material fecal es el mismo músculo
cuya relajación sobre todo en cuclillas permite El paso del contenido fecal con menor esfuerzo durante la
defecación
puborrectal

2. paciente mujer de 54 años se presenta con náuseas vómitos y estreñimiento Y es diagnosticada de abdomen
agudo quirúrgico en la cirugía encuentra un vólvulo de Ciego es anomalía puede explicarse por
falta de fusión del mesenterio

3. paciente mujer de 23 años con faringitis aguda toma para el dolor una tableta de Paracetamol con un poco de
agua durante la deglución se relaja su esfínter esofágico interior y el fondo del estómago mientras el bolo está
aún en el esófago Qué sustancias provocará con mayor probabilidad de relajación del esfínter esofágico inferior
y el fondo del estómago en esta mujer
óxido nítrico

4. luego de 3 horas dando exámenes un alumno de Medicina comienza a sentir hambre esta situación es probable
que sea mediada por la que sintetizada por
el grelina estómago

5. Varón de 72 años con antecedente de diabetes mellitus tipo 2 que presenta enteropatia diabetica característica
por el estreñimiento este problema puede ser asociado a
deficiencia de óxido nítrico
6. Varón de 54 años con diabetes mellitus tipo 2 es el masticado de gastroparesia debido a que presenta
sensación de llenura precoz al comer y reflujo gastroesofágico está alteracion en la relajación receptiva y en el
vaciamiento gástrico lo más probable es que se deba a una alteración en el
nervio vago

7. varón de 67 años con tos y disminución de peso asociado tabaquismo pesado presenta actualmente disfagia
progresiva alimentos sólidos se considera la presencia de un carcinoma de bronquio izquierdo por esta razón
le realiza una endoscopia esofágica para descartar la posibilidad de una compresión esofágica por el tumor se
espera revisar el esofago en la estrechez que están nivel de la vértebra
tercera estreches T6

8. en un niño menor de 2 años con divertículo intestinal este divertículo tiene su origen en una falla en la
obliteración de conducto vitelino

9. mujer de 43 años sufre grave accidente de tránsito y está hospitalizada en coma es alimentada por vía
intravenosa durante varias semanas producto de ese tipo de alimentación se encuentra en la endoscopia atrofia
de la mucosa gastrointestinal la causa más probable de esta trofia son los bajos niveles séricos de la hormona
gastrina

10. una mujer de 30 años llega al consultorio porque se queja de dificultad para deglutir lo cual se agrava cada vez
más se realiza un estudio manométrico para examinar la generación de presión a lo largo del esófago esta
prueba revela que las contracciones como respuesta de la deglución está mal sincronizadas y que la presión
en el esfínter esofágico inferior permanece elevada el diagnóstico más probable es producida por niveles bajos
de
acalasia /óxido nítrico

11. paciente de dos años llega a emergencia por haber ingerido una moneda con la que estaba jugando, el lugar
mas probable donde se pudo haber quedado suspendida este objeto es a nivel del estrechamiento producido
a nivel del:
musculo cricofaringeo

12. en una apendiocectomia,al realizar la incisión de Mc burnry en la fosa iliaca derecha, es necesario cortar los
sigts músculos de afuera hacia adentro:
Oblicuo externo-Oblicuo interno-Transverso

13. Un varón de 90 años que se encuentra postrado en cama, es referido del asilo para endoscopia por dificultad
para deglutir luego de tomar un medicamento para alivia el dolor la noche anterior la endoscopia revela que la
píldora se alojó en el esófago y causó una reacción inflamatoria y lo más probable es que haya sido por la
producción de múltiples
ondas secundarias

14. Mujer de 23 años es diagnosticada de bulimia al examen físico Se observa ulceraciones en el segundo y tercer
dedo de la mano derecha esto se puede deber al uso continuo de estos dedos por inducir al vómito mediante
la estimulación del
par craneal 9

15. varón de 52 años se presenta por diarrea persistente de 6 semanas de duración en la colonoscopia Se observa
un pólipo a nivel del íleon distal el patólogo informa que se trata de un tumor neuroendocrino probablemente
originado por las células enterocromafines del intestino la sustancia que más probablemente esté produciendo
este tumor es
serotonina

16. La base oclusal de la masticación se realiza con la contracción de los músculos


masetero y temporal

17. al tomar su café en Starbucks un estudiante de Medicina sufre una quemadura de primer grado en el tercio
anterior de la superficie dorsal de la lengua información de dolores transmitida por
el nervio lingual

18. pacientes evaluados por faringitis aguda en consultorio externo el médico de familia le solicita que abra la boca
y saqué la lengua para realizar la acción de sacar la lengua es necesario que se contraiga el
musculo geniogloso

19. Paciente con síndrome de jorgen presenta boca seca disminución de la producción de saliva y caries dental
asociada a la pérdida de la función de tampón de la saliva esta desmineralización del diente puede
comprometer a las prolongaciones citoplasmáticas ubicadas en los tubos huecos de la estructura señalada con
la letra
b

20. Mujer de 32 años acude a consulta por presentar disfagia de progresión lenta reflujo gastroesofágico y vómitos
desde hace 3 meses de evolución progresiva se realiza un estudio radiológico con contraste en el que se
observa estrechamiento del esfínter esofágico inferior según sus conocimientos este paciente se beneficiaría
con el uso de
análogo de óxido nítrico

Dentro de las funciones del abdomen, se encuentra la defecación y micción, en las cuales la presión intra
abdominal debe:
Aumentar

Paciente joven es traído a emergencia con abdomen agudo quiúrgico debido a herida contusopenetrante por
verduguillo (alambre grueso con punta aguzada) recibida en una pelea después de un partido de futbol. Se
observa herida en Hipocondrio Izquierdo. El órgano que debe estar sangrando y produciendo hemoperitoneo
es (marque la mejor respuesta):
Bazo

El ligamento hepatogástrico une el _______________ con el _______________ y forma la entrada al


___________

Hígado Estómago Orificio omental

Al evaluar una tomografía abdominal, el médico asistente le pide al interno de la UPC que encuentre la imagen
con el corte a nivel de L1. El interno sabiamente busca el ________ para ubicar la vértebra L1
Cuello del páncreas

En la inspiración, la pared abdominal debe ____________ para ____________

Relajarse disminuir presión intra torácica

Paciente con vólvulo del colon sigmoides. La necrosis de este segmento del colon se produce por una
alteración en la irrigación de la arteria
Mesentérica inferior

Marque el órgano que se considera retroperitoneal:


Parte de la vía biliar

La estructura que fija órganos principalmente a la pared posterior abdominal se denomina


Mesenterio

Durante el vómito, ¿el contenido gástrico tiene que pasar necesariamente por cuál estructura para llegar al
esófago? Marque la mejor respuesta
Cardias

1. Paciente de sexo masculino de 82 años de edad ingresa a emergencia con dolor abdominal agudo y diarreas.
Se le
realiza una arteriografía en la que se observa que la arteria aorta tiene un trombo ocluyendo el 95% del flujo,
a nivel del nacimiento de la arteria mesentérica inferior. ¿Cuál de las siguientes arterias podría contribuir a la
irrigación colateral del colon descendente?
a) cólica media
2. Niño de 5 años presenta dolor esofágico y hematemesis (vómitos hemorrágicos) luego de tragarse una
espina de pescado. En la endoscopía se observa perforación del esófago distal a la cuarta estrechez esofágica.
¿Las ramas de cuál de las siguientes arterias estarán lesionada con mayor probabilidad?
a) Gástrica izquierda

3. Al ingerir una cucharada de mantequilla es muy probable que se disminuya la sensación de hambre por
medio de la activación de la vía POMC/CART (POMC=proopiomelanocortina y CART=transcripción regulada
de cocaína y anfetamina), activada directamente por la hormona:
a) colecistoquinina (CCK)

4. Al comer unas papitas fritas con mayonesa, el vaciamiento gástrico disminuye por efecto directo de la
hormona:
a) colecistoquinina (CCK)
5. Recién nacido de dos horas es diagnosticado de hernia umbilical de 1,5 cm de diámetro; el cirujano pediatra
solicita una tomografía abdominal en donde se evidencia que la hernia umbilical está ocupada por una porción
del tracto gastrointestinal. ¿Qué porción del tracto gastrointestinal estaría ocupando esta hernia con mayor
probabilidad?
a) Íleon

6. Recién nacido de 7 horas, de parto por cesárea debido a polihidramnios (aumento del volumen del líquido
amniótico), con regurgitación de la leche materna y artificial, y no ha presentado meconio. Se le realiza una
tomografía donde se evidencia aire en el estómago y una malformación del desarrollo esofágico. Con respecto
a esta malformación lo más probable es que se pueda tratar de una atresia esofágica:
a) proximal con fístula traqueoesofágica distal

7. Lactante de 6 meses de edad que es traído a consulta por presentar vómitos no biliosos a repetición y retraso
en el crecimiento. En la radiografía de abdomen simple se observa nivel hidroaéreo en estómago y en primera
porción de duodeno (doble burbuja). ¿Cuál de las siguientes alternativas puede explicar la condición del
lactante?
a) Páncreas anular

8. En ausencia o deficiencia de la secreción de la hormona motilina, se producirá:


a) sobrecrecimiento bacteriano

9. La estimulación parasimpática aumenta la motilidad intestinal, mientras que la estimulación simpática la


disminuye. ¿Sobre cuál de las siguientes alternativas el sistema nervioso autónomo actúa para el control de la
motilidad intestinal?
a) Potencial de membrana en el plexo mientérico (de Auerbach)

10. En un recién nacido con protrusión de contenidos abdominales y cubiertas por amnios o peritoneo, es cierto
que:
a) Se presenta por un defecto en el cierre de la pared

11. Lactante de 20 días con estreñimiento, distención abdominal progresiva, acompañada ocasionalmente de
vómitos biliosos. Como antecedente, el meconio lo eliminó por primera vez a las 72 horas de nacido. Su mamá
menciona que ayuda a la evacuación con ayuda de un termómetro rectal. Se sospecha de megacolon
agangliónico (Enfermedad de Hirschsprung). ¿Cuál de las siguientes alternativas explica el caso?
a) Se presenta contracciones tónicas en la región ano rectal

12. ¿Cuál de los siguientes reflejos disminuye el tránsito gastrointestinal?


a) Doloroso

13. Al ingerir una sustancia ácida como el vino (pH 3), se estimula la motilidad gástrica por acción de la
hormona:
a) motilina

14. Paciente de 24 años acude a consulta externa por presentar una fístula oronasal (comunicación entre la
cavidad oral y la cavidad nasal). Está fístula está asociada al antecedente de haber sido operada de paladar
hendido a los dos años de edad, durante una campaña gratuita extranjera de corrección de paladar fisurado.
¿Cuál de las arterias palatinas podría haberse lesionado durante esa cirugía?
a) Mayor

15. Paciente de sexo masculino de 52 años con úlcera péptica gástrica de 14 años de evolución, con cuadro
de hemorragia digestiva alta hace 4 meses, sin cicatrización de la úlcera. Entre las opciones quirúrgicas se
considera realizarle un vaguectomía troncal (sección del nervio vago) a nivel del hiato esofágico. ¿Cuál de las
siguientes complicaciones podría esperarse producto de la pérdida de inervación parasimpática?
a) Menor inervación del colon ascendente

16. Paciente de 23 años con bulimia es traída a la emergencia deshidratada, semiconsciente y con alcalosis
metabólica. Los vómitos autoinfligidos por esta paciente se producen por estimulación de receptores en la base
de la lengua que mandan información directamente al:
a) núcleo del tracto solitario

17. Niño de 3 años es traído a emergencia por madre quien manifiesta que hace 10 horas deglutió una pila
pequeña de reloj de bordes romos. El niño está asintomático. Usted la tranquiliza diciéndole es un cuerpo
extraño tan pequeño de seguro que va a seguir el tránsito intestinal como lo haría un bolo alimenticio, y que lo
más probable es que en ese momento se encuentre en:
a) colon
18. Los movimientos en masa son un tipo de movimiento muy importante, una de las consecuencias de estos
movimientos es:
a) la distensión rectal

19. Durante la deglución, al momento que el bolo alimenticio pasa por el esfínter esofágico superior, se espera
que la
presión intraesofágica:
a) disminuya en el cardias

20. Paciente con enfermedad de Chagas que presenta disfagia a sólidos. ¿Cuál de las siguientes puede ser la
causa de esta complicación?
a) Disminución de células ganglionares en el esfínter esofágico inferior
1. La explicación fisiológica de presentar somnolencia de 30 minutos a 1 hora
después de ingerir alimentos, se explica por:

e. Aumento de la alcalinidad sanguínea

2. Con respecto a la irrigación arterial del colon, a que arteria corresponde la


señalada
con la flecha
b. Cólica media

3. Si un paciente presentara dentro del punto de vista fisiológico, una disminución


de enterocinasa, entonces esto originaría una disminución de la actividad de:
b. la lipasa
c. la quimotripsina
Se valida la opción b debido a su relación con la colipasa.

4. Con respecto a la anatomía del hígado, señale a que estructura pertenece la


marcada por el número 1.
a. Ligamento falciforme
Ligamento coronario
Se valida la opción a debido a la ubicación del número 1 en donde se unen el
ligamento falciforme y ligamento coronario.

5. Se presenta un paciente, el cual presenta un antecedente de tuberculosis intestinal, por lo cual, se le resecó
80 cm de íleon distal. Desde el punto de vista fisiológico, el paciente puede presentar una de las siguientes
alteraciones:

e. Disminución de la absorción de ácido glicocólico

6. Un paciente es sometido experimentalmente a un fármaco que modifica el flujo salival, obteniéndose un


volumen
de saliva de 288 ml en 6 horas. En este caso las concentraciones de electrolitos y bicarbonato en la saliva
obtenida
varían de la siguiente manera:
a. ↑ Na+, ↓ K+, ↑ Cl-, ↑ HCO3-
b. ↓ Na+, ↓ Cl-, ↑ K+, ↓ HCO3-
Se valida la opción a debido a que se puede considerar como un aumento del flujo de saliva.

7. La siguiente imagen histológica corresponde a la glándula …………… y la estructura señalada produce


………
c. salival submaxilar / ptialina

8. Paciente varón de 65 años con antecedente de hipercolesterolemia, hipertensión arterial, fibrilación auricular
y
dos infartos al miocardio previos, aqueja de dolor abdominal intenso de inicio súbito, distensión abdominal, se
decide cirugía con resección de 1,5 metros de intestino delgado terminal y colon ascendente. Como
consecuencia
de la resección el paciente tendrá deficiencia de:
a. Vitamina C
c. Vitamina A
Se valida esta opción debido a que su absorción está relacionada al íleon.
9. Uno de los siguientes elementos debería hallarse con más probabilidad en el esófago de un paciente que
sufre de reflujo gastro esofágico…
a. Pepsina

10. Un paciente de 40 años cursa con anemia de 8g/dl, aqueja además de astenia y sensación de hormigueo
bilateral en los miembros inferiores, al examen se halla alteración de la sensibilidad a la vibración y camina con
ampliación de la base de sustentación. Uno de los siguientes procedimientos sería de ayuda para el diagnóstico
de este paciente:
b. Biopsia de la mucosa gástrica

11. Paciente de 60 años ingresa por caída hace 1 hora y pequeño hematoma en cuero cabelludo, al examen
físico ampliado se observa ictericia de piel y mucosas generalizada, abdomen blando, se palpa estructura
quística no dolorosa en hipocondrio derecho que corresponde a vesícula biliar (signo de Courvoisier), en los
exámenes de laboratorio se halla niveles bajos en la formación de estercobilinógeno y urobilinógeno en heces,
incremento de la bilirrubina conjugada en la orina, elevación de fosfatasa alcalina y gamma glutamil
transpeptidasa séricas. El presente cuadro puede ser explicado por:
c. Carcinoma de la cabeza de páncreas

12. Paciente varón de 58 años con antecedente de alcoholismo crónico es diagnosticado y recibe tratamiento
por cirrosis hepática. Hace 2 días refiere familiar que tuvo cambio de conducta y no reconoce a algunos
familiares. Al examen físico, se halla ascitis, circulación colateral en abdomen, telangiectasias, en el examen
de sistema nervioso: rigidez de extremidades, ROT incrementados, desorientación en el espacio y asterixis.
¿cuál de las siguientes circunstancias, explicaría el cuadro en este paciente?
c. Hemorragia gastrointestinal
e. Dieta normo proteica
Se valida la opción e debido al efecto sobre la encefalopatía.
Con respecto de la opción b es incorrecta debido a que hace referencia al duodeno, debería indicar colon.

13. Un recién nacido presenta vómitos biliosos poco tiempo después de cada alimento. Al preguntar a la madre
sobre antecedentes, ella recuerda que tuvo polihidramnios durante la gestación, pero un análisis de cariotipo
fue normal. Una de las siguientes es la causa más probable de estos hallazgos en el recién nacido:
e. Malrotación de la yema pancreática ventral

15. En un estudio de la secreción de hormonas gastrointestinales, sus concentraciones en la vena porta se


midieron durante perfusión luminal del intestino delgado con soluciones de diversas magnitudes de pH. ¿Qué
hormona aumentará en el plasma de la vena porta durante perfusión a través del intestino con una solución de
pH 3?
e. secretina

16. Paciente de 30 años que ingresa a causa de un traumatismo abdominal cerrado. En la exploración se
aprecia discreta palidez de piel y mucosas, auscultación pulmonar normal, taquicardia de 120 /min. Discreta
distensión abdominal y matidez en flancos; el hematocrito, que era prácticamente normal al ingreso, disminuye
a 30% a las tres horas. En la Rx de tórax se objetiva fractura de las costillas 10-11 izquierdas. La causa más
probable de la anemización en este paciente es:
c. rotura de bazo con hemoperitoneo.

17. Mujer de 65 años. Consulta por síndrome constitucional asociado a dolor abdominal epigástrico progresivo
irradiado a espalda, de dos meses de evolución. El diagnostico de sospecha de adenocarcinoma de páncreas
se confirma por biopsia. Se realiza examen de imagen de abdomen para evaluación de estructuras vasculares
próximas al tumor pancreático. ¿Cuál es el nombre de la vena señalada que está ausente, trombosada por
infiltración tumoral, condicionando circulación
colateral en la pared gástrica?
c. Esplénica

18. Revisando la angiotomografía de un hombre de 70 años en estudio por aneurisma de aorta abdominal, el
radiólogo le informa de la presencia de una oclusión completa de la arteria mesentérica inferior. El paciente se
encuentra completamente asintomático. La oclusión de la arteria mesentérica inferior cursa de manera
asintomática en muchas ocasiones ya que el territorio que irriga puede recibir flujo proveniente de la arteria:
e. cólica media
19. En las patologías de esófago es importante conocer bien la anatomía esofágica. ¿Cuál de las siguientes
afirmaciones es correcta?
c. El esófago torácico pasa por detrás del cayado aórtico

20. A pesar de que pueda haber variaciones anatómicas, lo habitual es que el ciego sea irrigado por una rama
arterial que proviene de unas de las siguientes arterias:
d. Mesentérica superior
21. Ante un paciente con una cirugía abdominal urgente, el informe operatorio señala que se ha realizado una
resección de todo el duodeno y del tercio proximal del yeyuno manteniendo íntegros el estómago y todo el
íleon, así como los dos tercios distales del yeyuno. En el seguimiento nutricional del paciente ¿Qué vitamina o
mineral presentará con menor probabilidad una disminución de su absorción?
a. Cianocobalamina

22. ¿Cuál de las siguientes alternativas detallan las venas que confluyen y forman la vena señalada?
c. esplénica, mesentérica superior y mesentérica inferior

23. ¿Cuál de las siguientes sustancias forma parte de la secreción biliar?


b. Lecitina

24. El tubo digestivo contiene diferentes tipos de epitelios y glándulas. La estructura señalada es una
………………. y
está localizada en el …………...
a. glándula de Brunner / intestino grueso
b. cripta de Lieberkuhn / colon
c. cripta de Lieberkuhn / intestino delgado Aunque las criptas de Lieberkuhn están presentes en el intestino
delgado, la microfotografía es de epitelio de colon.

25. ¿De qué musculo forma parte el ligamento inguinal?


a. Oblicuo externo del abdomen

26. Señale cuál de las siguientes afirmaciones NO se relaciona a la siguiente


glándula anexa del tubo digestivo
mostrada en la imagen:
c. Esta glándula produce una secreción mucinosa acuosa, llamada
mucoserosa, a través del conducto de Wharton.

e. Es una glándula endocrina y probablemente sea de origen pancreático


Se valida la opción e debido a que no está relacionada con la imagen.

27. ¿Cuál de las siguientes enzimas está localizada en el borde en cepillo y juega un rol en la digestión de
proteínas?
c. Enterocinasa
e. Carboxipeptidasa A.
Se valida la opción c debido a que es correcta en relación a la pregunta.

28. Una de los siguientes sustancias, NO sirve como un buen agente emulsificante:
a. Colesterol
b. Ácidos grasos
c. Sales biliares
d. Lecitina
e. Proteínas de la dieta

Se valida la opción e debido a que es correcta en relación a la pregunta.


29. La sustancia que estimula el crecimiento de la mucosa gástrica es:
a. Secretina
b. Motilina
c. Péptido estimulante de la mucosa gástrica
d. Gastrina
e. Histamina

30. ¿Cuál de las siguientes alternativas es una función de la colecistokinina?


a. Relajación de la vesícula para la salida de bilis
b. Secreción de ácidos biliares
c. Contracción del esfinter de Oddi
d. Secreción de enzimas pancreáticas
e. Contracción del duodeno
Se valida la opción b debido al efecto de la CCK sobre la vesicula biliar.

31. Con respecto a la anatomía del tronco celiaco, señale lo correcto


d. La hepática común que es una de sus ramas, participa en la irrigación del estómago.

32. Con respecto a la anatomía del duodeno, marque la respuesta correcta:


b. La 3ra porción duodenal está contenida en la pinza vascular aortomesentérica
33. El hígado está ampliamente tapizado por peritoneo, la estructura que conecta la cara diafragmática del
hígado
precisamente con el diafragma es el ligamento:

b. falciforme
c. triangular
e. coronario
Se validan la opción c y e debido a que forman parte de los ligamentos que fijan el hígado al diafragma.

34. En el íleon se absorbe aproximadamente el 95% de …………………. a través de la circulación


enterohepática.
a. agua
b. colesterol
c. sales biliares
d. hidróxicobalamina
e. factor intrínseca

35. La ……………. estimula el mecanismo paracrino de la secreción de ácido clorhídrico.


a. histamina
b. acetilcolina
c. gastrina
d. secretina
e. somatostatina

36. En la digestión de proteinas, ……………. es el principal estímulo para convertir el pepsinógeno en pepsina.
a. la gastrina
b. el pH ácido
c. la acetilcolina
d. la ptialina
e. la somatostatina
37. Con respecto a la somatostatina, marque lo correcto:
a. Es secretada por las células S del intestino
b. Induce a la producción de VIP
c. Interviene en la fase intestinal de la secreción gástrica
d. Produce acetilcolina para estimular a la célula parietal
e. No interviene en la regulación de la secreción de ácido clorhídico

38. En pecten anal, es una estructura comprendida entre:


a. la línea pectínea y los senos anales
b. la línea blanca y la apertura anal
c. el esfínter anal interno y el externo
d. la línea anocutánea y la línea pectínea
e. la línea blanca y columnas anales

39. ¿Cuál de las siguientes alternativas es una proenzima pancreática?


a. Tripsina
b. Elastasa
c. Quimotripsinógeno
d. Amilasa
e. Procarboxipepitidasa C.

40. En la segmentación hepática de Coinaud, el segmento hepático señalado con la flecha, corresponde a :
En la segmentación hepática de Coinaud, la flecha señala el segmento ………….. hepático.
a. IV
b. V
c. VI
d. VII
e. VI
1. La reabsorción de sodio y cloro en las glándulas salivales se da principalmente en el:

c) conducto estriado

2. Paciente de 35 años con dolor en hipocondrio derecho irradiado a la punta de la escápula. Se observa en la
ecografía abdominal cálculos en la vesícula biliar. Es operado de emergencia realizando una incisión a lo largo
del reborde costal derecho. La información de dolor de esta zona es inervada por las raíces nerviosas:
c) T7 – T8

3. Paciente de 23 años con dolor intenso periumbilical de inicio agudo. Es ingresado a sala de operaciones por
abdomen agudo, el cirujano encuentra sangrado a unos 60 cm proximal a la válvula ileocecal. La estructura
que
está sangrando muy probablemente es un derivado embriológico del:
d) conducto vitelino

4. Paciente de 64 años de edad con dolor en epigastrio que se distribuye en banda hacia la espalda. En la
tomografía
se encuentra tumor en cabeza de páncreas que compromete a un vaso que discurre entre la cabeza del
páncreas y el proceso uncinado. Debido a esto, este cáncer se considera irresecable porque compromete a la
arteria:
a) mesentérica superior

5. Paciente con carcinoma gástrico avanzado, en preoperatorio para gastrectotomía total. ¿Cuál de los
siguientes ganglios linfáticos recibirá primero células metastásticas con mayor probabilidad:
a) celíaco

6. Paciente de 77 años con dolor abdominal difuso de 3 horas de evolución. En los estudios de imágenes se
observa oclusión probablemente aterosclerótica de la arteria mesentérica superior; no se observa necrosis, lo
cual puede ser explicado por la irrigación sanguínea colateral. ¿Qué vasos ofrecen colaterales entre el tronco
celíaco y la arteria mesentérica superior?
d) Pancreaticoduodenal superior e inferior

7. Paciente de 62 años con vólvulo de intestino delgado e isquemia intestinal. Se realiza laparotomía
exploratoria para liberar la obstrucción. ¿Cuál estructura se utilizará como punto de referencia para determinar
la posición de la unión duodenoyeyunal?
b) Ligamento suspensorio del duodeno (de Treitz)

8. Paciente de 42 años con dolor abdominal intenso y hematemesis. En la endoscopía se observa una úlcera
duodenal posterior perforada con hemorragia intrabdominal. ¿Cuál de las siguientes arterias estará
comprometida?
d) Pancreaticoduodenal posterosuperior
9. Paciente de 51 años con antecedente de enfermedad diverticular acude a emergencia por sangrado profuso
y dolor en cuadrantes inferiores. ¿Cuál es el origen más probable de la sangre que pierde en la hemorragia?

b) Mesentérica inferior

10. Paciente de 13 días de vida con vómitos explosivos a las dos horas después de lactar. Al examen físico se
palpa la oliva pilórica. ¿Cuál es el nervió cuyos filetes dan inervación eferente a la estructura afectada?
a) Vago

11. Paciente de 80 años con dolor abdominal intenso y antecedente de estreñimiento crónico. En la
colonoscopía se
observa divertículos con áreas ulceradas difusas en colon sigmoides y descendente. El cirujano programa una
cirugía de resección y planifica que para realizar esta resección tendrá que cortar los siguientes vasos y nervios:
d) nervio esplácnico pélvico y artera cólica izquierda.

12. Paciente de 46 años ingresa a emergencia con dolor en cuadrante superior derecho e ictericia. En la
ecografía se
observa cálculos en la vesícula biliar. ¿Cuál de los siguientes nervios transmite el dolor de la colecistitis?
c) Fibras aferentes simpáticas del nervio esplácnico torácico mayor, con referencia a los dermatomas T6 a T8

13. Un niño de 8 años es alimentado por sus padres con un Cheeseburguer, papitas fritas y un vaso de Coca
Cola. ¿La presencia de lípidos, carbohidratos y proteínas en el duodeno estimulan la liberación de cuál de las
siguientes hormonas en la mucosa duodenal?
b) Péptido inhibidor gástrico (GIP)

14. La fase cefálica del control de la secreción gástrica corresponde a cerca del 30% de la secreción ácida y
es consecuencia de un reflejo. ¿Cuál de las siguientes alternativas puede eliminar totalmente la fase cefálica
de la secreción gástrica?
a) Vaguectomía

15. Recién nacido de 24 horas con llanto, vómitos y sin eliminación de meconio. Luego de los estudios auxiliares
se diagnostica la Enfermedad de Hirshprung, esta enfermedad se caracteriza por la ausencia de:
a) las células de Cajal en plexo mioentérico

16. Los complejos motores migratorios aparecen aproximadamente cada 90 minutos entre las comidas, y se
considera
que son estimulados por la hormona motilina. La ausencia de estos complejos migratorios podría producir un
aumento en:
d) las bacterias intestinales

17. ¿Cuál de las siguientes es una consecuencia probable de la resección del íleon?
b) Deficiencia de vitamina B12

18. Los movimientos en masa son importantes en la fisiología intestinal. Estos movimientos en masa ocasionan:
a) la sensación de defecar

19. La toxina colérica hace que aumente los niveles de AMPc intracelular, y este aumento hace que se
mantenga abierto un canal en las células de la cripta de Lieberkuhn. En condiciones fisiológicas, en una
persona sana ¿Qué sustancia puede promover que el canal quede abierto también?
c) Péptido intestinal vasoactivo (VIP)

20. Paciente de 64 años con tumor abdominal que comprime la cisterna del quilo. En la biopsia de duodeno
tomada
como parte del estudio, el patólogo puede observar
a) Dilatación del vaso quilífero central

21. En la enfermedad de Crohn es posible encontrar células de Paneth en el colon. Esto se puede deber a la
especial función de estas células en:
b) mantener la inmunidad innata

22. En 1967 se descubrió que la epidemia de Kuru, una enfermedad por priones, en el distrito de Okapa en
Papua Nueva Guinea, era causada por la costumbre de comer la carne de los muertos. Ahora se sabe que las
proteínas priónicas ingresan al organismo a través de:

d) las células M

23. Paciente de 48 años con alteraciones en el tránsito intestinal por diabetes mellitus tipo 2; se presenta con
esteatorrea, flatulencia y malabsorción de grasas. Las pruebas de función hepática y biliar están dentro de
rangos normales. Una causa de la disminución de sales biliares puede ser:
a) el sobrecrecimiento bacteriano

24. Al usar azúcar de mesa (sacarosa) para endulzar su café, el estudiante de medicina sabe que lo más
probable es que para su absorción tendrá utilizar el/los transportadore(s) ________________ que se
encuentran en la membrana apical de los enterocitos.
c) SGLT-1 y GLUT5

25. Una persona con la producción normal de lactasa; cada vez que toma leche, los productos de la
degradación de la
lactosa por parte de la lactasa ingresarán al enterocito usando el/los transportador(es) _____________:
a) SGLT-1

26. En una persona sana, luego de una comida basada en carnes rojas cocinadas con baja cantidad de sal, al
enterocito pueden ingresar solamente:
c) aminoácidos, dipéptidos y tripéptidos

27. Paciente con deficiencia congénita de procolipasa, sufre de esteatorrea cada vez que come comidas ricas
en grasas.En el estudio de composición de las heces, uno espera encontrar una cantidad aumentada de:
b) triglicéridos

28. Al tomar su café en Starbucks, un estudiante de medicina sufre una quemadura de primer grado en el tercio
anterior de la superficie dorsal de la lengua. La información de dolor es transmitida por el nervio:
a) cuerda del tímpano
b) glosofaríngeo
c) lingual
d) facial
29. Paciente de 32 años con esteatosis hepática no alcohólica. Se le realiza una biopsia hepática que confirma
la esteatosis; en el tejido se observan depósitos de lípidos en los hepatocitos, los cuales contienen
principalmente:
a) triglicéridos
b) colesterol
c) ácidos grasos libre
d) acil-carnitina

30. Paciente de 21 años que es estudiante de medicina, llega a consulta refiriendo que en épocas de exámenes
su piel se vuelve amarillenta. Le realizan el diagnóstico de Síndrome de Gilbert asociado a mutación del gen
UGT1A1, luego de unas semanas acude a la consulta por ictericia asociada a resfrío. Al hacerle un análisis de
sangre, usted sabe que encontrará valores elevados de:
a) bilirrubina directa
b) bilirrubina indirecta
c) bilirrubina tipo delta
d) fosfatasa alcalina

31. Se realizó un experimento en el cual se inyectó tinta china en el peritoneo de ratas de laboratorio. Al
realizarse una
biopsia hepática de dichos animales, se encontró que el tinte negro estaba depositado en:
a) las células de Ito
b) los hepatocitos
c) las células de Kupffer
d) las células de Disse

32. Paciente de 74 años de edad con shock hipovolémico asociado a deshidratación aguda severa. En este
paciente es posible encontrar hipoxia en zona …… del lobulillo hepático e infarto ……………… del intestino.
a) 1 / mucoso
b) 1 / transmural
c) 3 / transmural
d) 3 / mucoso

33. Niño de 5 años con historia de tres días de evolución caracterizado por fiebre, malestar general, odinofagia,
hiporexia, e irritabilidad. Al examen se observa lesiones ulcerativas de 4 mm de diámetro en mucosa yugal,
con fondo blanquecino y eritema periférico. El diagnóstico más probable es:
a) candidiasis oral
b) leucoplasia
c) aftas
d) herpes

34. Paciente de 52 años con enfermedad por reflujo gastroesofágico de 30 años de evolución. Se realiza
endoscopía en la que se encuentra mucosa eritematosa proximal a la línea Z. Para corroborar la presencia de
lesión preneoplásica,se toma una biopsia de esa zona, en la que se espera encontrar:
a) metaplasia gástrica
b) metaplasia intestinal
c) displasia gástrica
d) adenocarcinoma

35. Un estudiante de medicina termina su último examen final. Al abrir la boca para comer una hamburguesa
con papas fritas, la activación de los receptores muscarínicos de las células acinar y ductal estimularan un
mayor flujo de saliva, con lo cual disminuirá la concentración salival de:
a) potasio
b) bicarbonato
c) sodio
d) cloro
36. Un estudiante toma su desayuno consistente en un pan con mantequilla y queso. Antes que se puedan
digerir las grasas, es necesario que sean emulsificadas. La hormona ………………………. estimula la liberación
de las sustancias
emulsificadoras.
a) colecistoquinina (CCK)
b) secretina
c) lipasa pancreática
d) gastrina

37. Al comer un pollo a la brasa entero, con papitas fritas y ensalada, la sustancia que estimulará la liberación
de HCl
en el estómago es:
a) el neuropeptido Y
b) la secretina
c) la bombesina
d) la colecistoquinina (CCK)

38. Paciente con disminución marcada del apetito asociado a cáncer terminal, se podría utilizar análogos de
……………..para promover la ingesta de alimentos.
a) el péptido similar al glucagón (GLP)
b) la serotonina
c) la secretina
d) la endorfina

39. Un hombre de 51 años presenta de forma súbita vómitos masivos de sangre roja brillante. Tiene como
antecedente hepatitis viral B hace 23 años. En la exploración física: FC 103 latidos/min, PA 85/50 mmHg, se
palapa la punta del bazo y niega vómitos. Su hematocrito es 21%, la prueba serológica de HBsAg es positiva.
En la ecografía se observa hígado con nodulaciones. ¿Cuál es la causa más probable para la hematemesis?
a) Esófago de Barrett
b) Síndrome de Mallory Weiss
c) Varices esofágicas
d) Esofagitis por reflujo

40. Durante el reflejo del vómito, uno de los primeros sucesos es:
a) el peristaltismo inverso
b) la contracción del píloro
c) la apertura de la glotis
d) el esfínter esofágico inferior abierto

41. Paciente de 34 años que acude por diarrea desde hace 4 días asociado a comer papa rellena con ají en el
Estadio Nacional, se exacerba cuando toma lácteos o come grasas. Se acompaña de flatulencia y episodios
de tenesmo. Este cuadro de diarrea e intolerancia a la lactosa de inicio agudo se explica por la:
a) enfermedad de Crohn
b) infección por V. cholera
c) colitis ulcerativa
d) giardiasis

42. En cuanto a la secreción pancreática, mientras mayor es el flujo, mayor es la concentración de:
a) potasio
b) bicarbonato
c) cloro
d) sodio

43. Paciente de 10 años con diarrea crónica, distensión abdominal, anorexia. Se le ha encontrado anticuerpos
antigliadina y antiendomisio. Es más probable que la diarrea se correlacione con el hallazgo histológico de:
a) adelgazamiento de las criptas
b) linfocitos intramusculares
c) atrofia de las vellosidades
d) úlceras duodenales
44. La glándula parótida está inervada por el par craneal:
a) VII
b) IX
c) X
d) XII
45. Las glándulas de Brunner se encuentran en la:
a) mucosa del esófago
b) submucosa del íleon
c) mucosa del yeyuno
d) submucosa del duodeno
46. ¿Cuál de las siguientes afirmaciones describe correctamente la función de la inervación parasimpática del
tracto
gastrointestinal?
a) La norepinefrina es el principal neurotransmisor excitatorio.
b) La actividad parasimpática produce la relajación de los esfínteres
c) La actividad parasimpática excesiva puede provocar un trastorno llamado ileo paralítico (parálisis del
músculo liso intestinal)
d) El pH luminal, la osmolaridad y la distensión muscular son detectados por fibras parasimpáticas eferente

47. Al ingerir una cantidad de glucosa por vía oral, esta es interiorizada en las células del organismo más rápido
que si esa misma cantidad de glucosa hubiese sido administrada por vía endovenosa. Este fenómeno sucede
gracias a la acción de la sustancia secretada por las células:
c) K

48. A …………… secreción de HCl en el lumen gástrico, …………….. pH en la sangre venosa gástrica
a) mayor /mayor

49. ¿En cuál de las siguientes situaciones hay un menor flujo de secreción salival?
d) Sueño

50. El alcohol y la cafeína estimulan la fase ______________ de la producción de ácido clorhídrico.


b) gástrica

1. ¿Cuál de las siguientes estimula la secreción enzimática exocrina del páncreas?:


a. Secretina
b. Bicarbonato en el duodeno
c. Gastrina
d. Colecistoquinina

2. ¿Cuál de los siguientes requiere de la formación de micelas para su absorción intestinal?:


a. Vitamina D
b. Ácidos biliares
c. Cianocobalamina

3. La fase gástrica de la secreción gástrica es responsable del 60% de la producción ácido. Este
efecto sobre la secreción de ácido puede ser virtualmente abolida con el uso de::
a. Antiácidos
b. Inhibidor de la bomba de protones
c. Atropina
d. Bloqueador de histamina

4. Paciente de 42 años con diarrea desde ayer. Menciona que se automedicó con Azitromicina
500 mg por tres días, debido a una faringitis aguda. Lo más probable es que este medicamento
haya actuado como agonista del receptor de:
a. Secretina
b. Colecistoquinina
d. Motilina

5. La metoclopramida estimula el vaciamiento gástrico aumentando la fuerza de contracción de


las paredes gástricas, mediante la estimulación indirecta de neuronas liberadoras de:
a. Acetilcolina
b. Péptido intestinal vasoactivo
c. Óxido nítrico
d. Bombesina

6. En un estudio experimental para tratar el estreñimiento crónico, se analiza el efecto del


medicamento llamado elobixibat, que es un inhibidor del cotransportador de Na+
-ácidos biliares. La acción de este medicamento básicamente se producirá en:
a. Colon
b. íleon
c. Páncreas
d. Vesícula biliar

7. El estímulo de las células parietales producirá la secreción de:


a. HCl y factor intrínseco
b. HCl y pepsinógeno
c. HCl y HCO3-
d. HCO3- y factor intrínseco
8. En la motilidad intestinal, las ondas lentas se pueden definir como:
a. Potenciales de acción
b. Contracciones fásicas
c. Potenciales de reposo oscilantes
d. Liberación oscilante de colecistoquinina

9. El azúcar de mesa está constituido principalmente por el disacárido sacarosa, el cual es digerido
a dos monosacáridos que son absorbidos en el intestino delgado, y llegan a compartir un
transportador, que es el:
a. SGLT1
b. GLUT2
c. GLUT5
d. SGLT2

10. El Vibrio cholerae produce diarrea porque:


a. Aumenta los canales secretores de bicarbonato en los enterocitos
b. Previene la absorción de glucosa y causa que el agua mantenga en la luz intestinal
c. Inhibe la producción de AMPc en los enterocitos
d. Aumenta los canales secretores de cloro en la células de la cripta

11. Paciente con tumor en íleon distal, es sometido a resección de dicho segmento. Un efecto
secundario que se espera en este paciente es esteatorrea debida a disminución en la:
a. Secreción de lipasa pancreática
b. Formación de micelas en la luz intestinal
c. Formación de quilomicrones en la luz intestinal
d. Digestión de los triglicéridos de la dieta

12. ¿Cuál de los siguientes puede inhibir la relajación receptiva en el estómago:


a. Gastrina
b. Acetilcolina
c. Péptido intestinal vasoactivo
d. Atropina

13. La diarrea osmótica se produce por la disminución en la absorción de solutos, los cuales
generan un aumento de agua en la luz intestinal, como es el caso de la diarrea producida por:
a. Hidróxido de magnesio
b. Cólera
c. Hipermotilidad intestinal
d. E. coli enterotoxigénica
14. En cuanto a la secreción pancreática, mientras mayor es el flujo, mayor es la concentración de:
a. Potasio
b. Bicarbonato
c. Cloro
d. Sodio

15. Paciente de 34 años que acude por diarrea sobre todo cuando toma lácteos o come grasas,
desde hace 4 días, asociado a comer papa rellena con ají en el Estadio Nacional. Se acompaña
de flatulencia y episodios de tenesmo. Este cuadro de diarrea e intolerancia a la lactosa de
inicio agudo es sugerente de:
a. Enfermedad de Crohn
b. Infección por V. cholera
c. Colitis ulceratica
d. Giardiasis

16. Paciente de 47 años con dolor inguinal desde hace dos días. Al examen se palpa una masa por
encima del ligamento inguinal, y es programado para sala de operaciones. Durante la
operación, el cirujano encuentra un asa intestinal que pasa a través del anillo inguinal
profundo, por lo que se puede concluir que se trata de una hernia:
a. Inguinal directa
b. Inguinal indirecta

17. Paciente varón de 25 años con cuadro agudo de dolor periumbilical con dolor a la palpación. En
la tomografía se observa imágenes sugerentes de hemorragia localizada a unos 60 cm
proximales a la unión ilelocecal. Esta es una posible complicación de la falla en el cierre del
conducto:
a. Vitelino

18. Paciente mujer de 42 años sometida a colecistectomía por cálculos biliares. A los 5 días del
posoperatorio desarrolla peritonitis biliar. Considerando que el cirujano asegura que no tuvo
complicaciones durante la cirugía, la causa más probable de este problema sería una filtración
de bilis por:
b. Conductos de Lushka

19. Paciente de 38 años con pancreatitis crónica asociado a dolor intenso y difícil de controlar. Por
tal razón anestesiología lo ha programado para aplicar anestesia (bloqueo) de la siguiente
estructura:
a. Nervio vago
b. Nervio pudendo
c. Ganglio celiaco
d. Nervio esplácnico menor

20. La estimulación parasimpática de la parótida produce:


a. Disminución de la concentración de bicarbonato en la saliva
b. Aumento en el volumen de la secreción
c. Disminución del consumo de O2
d. Vasoconstricción en la parótida

21. Paciente varón de 73 años es traído a urgencias con dolor abdominal intenso. En la tomografía
se observa un aneurisma de la aorta abdominal que afecta al origen de la arteria mesentérica
superior. Basado en sus conocimientos, usted sabe que el órgano que se encontrará más
afectado será:
a. Estómago
b. Íleon
c. Colon transverso
d. Recto

22. Paciente de 36 años con dolor inguinal y una masa palpable que aumenta de tamaño al hacer
ejercicios o se pone de pie. Esta masa se ubica dentro del triángulo de Hesselbach, por lo que
hay que sospechar que la causa más probable de este tipo de hernia es un defecto a nivel de :
a. Fascia transversa alrededor del anillo inguinal profundo
b. Peritoneo alrededor del anillo inguinal profundo
c. Aponeurosis del oblicuo externo e interno del abdomen
d. Aponeurosis del músculo transverso y la fascia transversa

23. En los pacientes con colecistitis aguda no operable, una opción es la colocación de una sonda
por el cístico, procedimiento en el que se ingresa con dificultad debido a la estrechez del cístico
y a la presencia de:
a. Válvula de Herring
b. Ampolla de Vater
c. Arteria cística
d. Estenosis del hepático común

24. Para rehidratar a un lactante con diarrea, la mejor opción sería:


a. Agua destilada
b. Agua destilada con cloruro de sodio hipertónica
c. Agua destilada con cloruro de sodio y glucosa
d. Agua destilada con cloruro de sodio, glucosa y cloro

25. Paciente ha sido puesto en “Nada por vía oral” (nil per os, NPO) durante 24 horas, ¿el aumento
en el hipotálamo de cuál de las siguientes sustancias se esperaría que estimule el hambre en
este paciente?:
a. Leptina
b. Neuropéptido Y
c. Colecistoquinina
d. Péptido YY
26. La acción de la secretina sobre las células ductales del páncreas, es potenciada por:
a. Somatostatina
b. Acetilcolina
c. Óxido nítrico
d. Bombesina

27. La absorción del ácido fólico se produce gracias a:


a. Contratransporte con H+
b. Degradación de las micelas
c. La acción conjunta de lipasa/colipasa
d. Cotransporte dependiente de Na+

28. Paciente de 17 años, con diarrea crónica sin moco ni sangre, asociado a dolor abdominal, con el
único antecedente notable que desde que empezó a estudiar medicina, mastica de 6 a 10
tabletas de chicle Trident que contiene sorbitol. Este tipo de diarrea debe ser:
a. Por malabsorción
b. Osmótica
c. Secretora
d. Exudativa

29. Por lo general, el manejo de las diarreas de tipo _____________ requiere casi siempre el uso de
antibióticos:

d. Exudativa

30. En los pacientes con diarrea asociada al consumo de pastas o pan y con anticuerpos
antigliadina, la malabsorción se produce básicamente por:
a. Presencia de HLA DQ2
b. Linfocitosis intraepitelial
c. Atrofia de las vellosidades
d. Apertura de los canales de cloro

31. Paciente de 87 años, postrado, con dolor abdominal difuso de inicio súbito, diarrea exudativa y ausencia
de peristaltismo. Antecedente de dos infartos de miocardio, y fibrilación auricular sin tratamiento. Se sospecha
que el cuadro esté asociado a la presencia de múltiples émbolos arteriales, por lo que se espera que el cirujano
encuentre:
a. Infarto mucoso intestinal
b. Enfermedad de Whipple
c. Infarto transmural intestinal
d. Infarto de miocardio

32. El compromiso intestinal en Enfermedad de Crohn es clásicamente en la región:


a. Ileocólica
b. Recto/sigmoidea
c. Duodenal
d. Anal

33. El aumento de la presión intraluminal en el sigmoides de manera crónica produce


evaginaciones de la mucosa del colon en las zonas más débiles de su pared, a veces con
material fecal, denominadas:
a. Apéndices epiploicos
b. Divertículos
c. Pseudopólipos
d. Absceso críptico

34. ¿Cuál de las siguientes sustancias tiene una mayor concentración en la saliva comparado con su
concentración plasmática?:
a. Bicarbonato
b. Sodio
c. Cloro
d. Potasio

35. Al solicitar la prueba de glucosa basal en ayunas, lo que se quiere evaluar es la capacidad del
hígado de producir:
b. Glucogenólisis

36. El daño del canalículo biliar se correlaciona con el aumento de los valores sanguíneos de:
Gamma glutamil transferasa

37. El principal componente de la bilis es:


d. Ácidos biliares

38. Paciente de 2 años, llega a emergencia por haber ingerido una moneda con la que estaba
jugando. El lugar más probable donde puede haberse quedado este objeto es sobre el
estrechamiento producido a nivel del músculo:
b. cricofaríngeo
39. El alcohol y la cafeína estimulan la fase ______________ de la producción de ácido clorhídrico:
b. Gástrica

40. En los neonatos sanos, el color verde oscuro del meconio (primera deposición) se debe a:
c. Pigmentos biliares

2. La lengua está recubierta por epitelio:


a. pseudoestratificado columnar no queratinizado
b. plano estratificado no queratinizado
c. pseudoestratificado columnar ciliado
d. plano estratificado queratinizado

3. El esfínter anal interno tiene musculatura …………… y tiene control ………………….


a. lisa / voluntario
b. lisa / involuntario
c. esquelética / simpático
d. esquelética / parasimpático

4. La arteria aorta proporciona la irrigación al tubo digestivo ¿cuál de las siguientes arterias proporciona la
irrigación
al ángulo cólico derecho?
a. Mesentérica superior
b. Mesentérica inferior
c. Frénica inferior
d. Tronco celiaco

5. Paciente de 26 años que le cuenta en su historia clínica que cada vez que almuerza, a los 20 minutos tiene
deseo
de defecar. Le comenta que su hijo de 1 mes le pasa lo mismo pero más intenso. Esto se explica por el reflejo
…………………, el cual está ……………..en el paciente.
a. colicoileal / normal
b. colicoileal / alterado
c. gastrocolico / normal
d. gastrocolico / alterado

6. La región del estómago que se comunica con el duodeno se denomina:


a. pilórica
b. cardias
c. cuerpo
d. fórnix

7. Acude a consulta un paciente que fue diagnosticado de ulcera péptica 3 días antes. Luego de múltiples
pruebas
diagnósticas, se concluye que el paciente presenta un tumor secretor de gastrina ¿Cuál de las siguientes
situaciones
estará incrementada?
a. Distención gástrica
b. Inhibición del vaciado gástrico
c. Secreción de ácido clorhídrico (HCl)
d. Inhibición de la secreción de pepsinógeno

8. En el sistema digestivo, el control del apetito esta dado por un complejo sistema de sustancias y órganos
integradores los cuales regulan la ingesta de alimentos. La ……………… es una sustancia oroxígena y es
sintetizada por el ……………………..
a. leptina / intestino
b. grelina / intestino
c. leptina / estómago
d. grelina / estómago
9. Sobre el control autónomo del sistema digestivo, marque la alternativa correcta:
a. La inervación dada por el sistema simpático es de tipo preganglionar.
b. El sistema parasimpático usa como neurotransmisores a la acetilcolina y la noradrenalina.
c. El nervio vago (par craneal X) le da inervación simpática a la mayoría del sistema digestivo.
d. En el sistema simpático, los nervios responsables hacen una primera sinapsis en ganglios próximos al
órgano a inervar.
e. En la inervación de tipo parasimpático, solo interviene el plexo submucoso, sin embargo, en la de tipo
simpático intervienen tanto el submucoso como el mientérico.

10. Con respecto a la actividad eléctrica del sistema digestivo, marque la alternativa correcta
a. Corresponden a potenciales de acción que están presentes de forma continua y le dan la capacidad de
perístasis autónoma al sistema digestivo.
b. La frecuencia de las ondas lentas no se ve influenciada por la actividad neural ni las hormas
gastrointestinales.
c. En el estómago las ondas lentas se dan en una frecuencia de 6 por minuto.
d. Las ondas lentas son cambios lentos y ondulantes del potencial en reposo.
e. La frecuencia de las ondas lentas va de 6 a 12 ondas por minuto.

11. Ante una lesión del IX par craneal, el músculo……………….. se altera en su función.
a. palatogloso
b. estilofaríngeo
c. palatofaríngeo
d. constrictor superior
12. Un varón de 50 años es sometido a extirpación de duodeno y parte proximal de yeyuno. La pérdida de
estímulo hormonal en el páncreas para la secreción enzimática se explica por la pérdida de las células
……………………
a. Parietales, productoras de factor intrínseco
b. “K” productoras de factor intrínseco
c. “M” productoras de CCK
d. “I” productoras de CCK

13. Respecto al mecanismo de la defecación ¿Cuál de las siguientes afirmaciones es correcta?


a. Se produce contracción refleja del esfínter anal interno
b. Se produce contracción o relajación del esfínter anal externo por señales de la corteza cerebral
c. La presencia de materia fecal en el recto estimula la contracción del sigmoides por los nervios pélvicos
simpáticos
d. En la posición de “cuclillas” el músculo puborectal se halla contraído favoreciendo la evacuación de la
materia fecal

14. Un niño de tres años llega a emergencia con disfagia (dificultad para tragar), salivación y llanto. Se
sospecha de
ingesta de cuerpo extraño: moneda en el esófago; al ser evaluado se constata en una radiografía presencia de
cuerpo extraño a nivel de C6 y C7 (6° y 7° vértebra cervical). El cuerpo extraño estará suspendido a nivel del
estrechamiento producido por el ………..
a. cayado aórtico
b. hiato esofágico
c. músculo cricofaríngeo
d. bronquio principal izquierdo
15. En el caso de un paciente con un tumor productor de gastrina, la presencia de úlceras duodenales y erosión
de la mucosa gástrica se debe principalmente a…….
a. la acción paracrina de la gastrina sobre la célula parietal
b. el exceso de HCl por estímulo de receptores CCK-B en la célula parietal
c. la sobre expresión de los receptores “G” para gastrina en la célula parietal
d. el exceso de HCl por estímulo directo de receptores “H” en la célula parietal

16. La onda peristáltica secundaria del esófago se caracteriza por ser originada ………
a. por el plexo de submucoso del esófago
b. por el plexo mientérico del esófago
c. por el reflejo de la deglución
d. durante la masticación
17. Marque lo correcto sobre las ondas lentas en el tubo digestivo
c. Se constituyen de despolarizaciones y repolarizaciones
18. Recién nacido que presenta protrusión de contenidos abdominales los cuales no están cubiertos por
peritoneo y salen de la cavidad abdominal a través de un defecto de la pared. ¿Cómo se denomina a la afección
que presenta este paciente?
a. Onfalocele
b. Atresia biliar
c. Gastrosquisis
d. Divertículo de Meckel

19. Experimentalmente se utiliza atropina (anticolinérgico) para inhibir la secreción de gastrina, sin embargo,
la secreción de esta hormona se sigue dando ante estímulos vagales. Esta situación se explica porque la
atropina:
a. no bloquea la acción del péptido GRP
b. solo inhibe la acción del péptido GRP en la célula G
c. inhibe la acción de acetilcolina e histamina en la célula G
d. bloquea parcialmente la bomba de protones en la célula G

20. Niña de 4 días es llevada a la emergencia pediátrica por presentar llanto constante, la madre refiere
coloración azulada de labios al momento de lactar, acompañado de tos persistente y dificultad respiratoria así
como distención abdominal. Se le coloca sonda nasogástrica para alimentación notando que retorna a la
cavidad oral en todos los intentos. ¿Cuál es la anomalía del desarrollo en este caso?
a. Solo fístula traqueo esofágica
b. Fístula traqueo esofágica proximal y distal
c. Atresia esofágica proximal con fístula traqueo esofágica distal
d. Atresia esofágica distal con fístula traqueo esofágica proximal

21. Paciente varón de 36 años es traído a la emergencia luego de sufrir un accidente de tránsito, presenta
traumatismos múltiples en cabeza y tronco. Al examen físico se evidencia hematoma en hemicara izquierda,
ligera protrusión y caída del lado izquierdo del maxilar inferior, por lo que se realiza una tomografía donde se
halla una fractura de la apófisis coronoides del maxilar inferior. ¿Qué músculo está relacionada directamente
con esta situación?
a. Masetero
b. Temporal
c. Buccinador
d. Pterigoideo medial
22. Un paciente refiere no percibir algunos sabores. Al examen físico constata alteración del sabor dulce y
umami.¿Cuál de los siguientes nervios estará alterada su función?
a. Cuerda del tímpano (VII par)
b. Lingual (rama del V par)
c. Glosofaríngeo (IX par)
d. Hipogloso (XII par)
23. A los pocos días de nacido, regresa a neonatología un niño con problemas de motilidad del colon; los
estudios determinan ausencia congénita de células ganglionares. Según el gráfico ¿cuál es la capa en la que
se determina la ausencia de dichas células?
a. Mucosa - 2
b. Muscular propia - 2
c. Muscular propia - 3
d. Muscular de la mucosa – 3
24. Con respecto al control autonómico en el tracto gastrointestinal y en relación a su fisiología. ¿Cuál es la
función
del sistema nervioso parasimpático en el tracto gastrointestinal?
a. Inhiben la contracción muscular y estimulan la secreción de sustancias a nivel de la submucosa
b. Estimulan la contracción muscular y estimulan la secreción de sustancias a nivel de la mucosa
c. Inhiben la contracción muscular e inhiben la secreción de sustancias a nivel de la submucosa
d. Estimulan la contracción muscular e inhiben la secreción de sustancias a nivel de la mucosa

25. Un estudiante que está preocupado por su examen parcial, no ha desayunado ni almorzado; cuando al fin
ingiere alimentos, esto le provoca el aumento de los movimientos musculares del tracto gastrointestinal y la
sensación de defecar. ¿Qué reflejo se ha activado?
a. Entero-gástrico
b. Gastro-cólico
c. Cólico-ileal
d. Ileo-ileal

26. ¿De qué par craneal es rama el nervio palatino mayor?


a. Vago
b. Hipogloso
c. Trigémino
d. Palatogloso

27. ¿En cuál de las fases de la deglución la epiglotis separa la vía respiratoria de la digestiva?
a. oral
b. laríngea
c. faríngea
d. esofágica

28. Los péptidos intestinales se pueden clasificar como sustancias endocrinas, neurocrinas y paracrinas, dentro
de las paracrinas se encuentran la somastotatina e histamina. Marque la respuesta correcta
a. La somastotatina es sintetizada por las células B de la mucosa gástrica
b. La histamina actúa estimulando su receptor tipo H1 en la mucosa gástrica
c. La histamina es sintetizada por células de tipo paracrino de las glándulas gástricas
d. La somatostatina presenta dentro de sus funciones la estimulación de la secreción de H+

30. Paciente varón de 27 años es llevado por bomberos a emergencia luego de ser asaltado y, tras resistirse,
es cortado con el pico de una botella a nivel abdominal. Al examen físico usted observa que a través de la
herida se puede observar la protrusión de asas intestinales. En relación con las capas de la pared abdominal,
marque la alternativa correcta.
a. La fascia de Scarpa está constituida principalmente por tejido adiposo
b. La pared abdominal está formada por piel, huesos, músculos, fascias y peritoneo parietal
c. La fascia de Camper es una estructura fibrosa que carece de grasa y su grosor es constante en toda la
pared abdominal
d. El músculo oblicuo externo discurre en dirección súpero-interna y se inserta en el borde inferior de las
ultimas 3 a 4 costillas
e. El músculo recto del abdomen tiene como funciones comprimir el contenido del abdomen, tensar la
pared del abdomen y flexionar la columna


En el síndrome de Mirizzi, el paciente tiene cálculos en la vesícula biliar; pero se obstruye el conducto hepático común
debido a que un cálculo se ubica y crece de tamaño en:

Respuesta seleccionada:
la bolsa de Hartmann
Respuestas:
la bolsa de Hartmann
la válvula espiral de Heister
el colédoco
el conducto accesorio de Lushka
• Pregunta 2
0 de 2 puntos
La bilis que sale de la vesícula biliar tiene como componente principal:

Respuesta seleccionada:
al colesterol
Respuestas: al colesterol

a los ácidos biliares


a los fosfolípidos
a la bilirrubina
• Pregunta 3
2 de 2 puntos
Con respecto a la microestructura del hígado ¿Cuál de las siguientes alternativas es correcta?

Respuesta seleccionada:
Los hepatocitos están interconectados por uniones herméticas
Respuestas:
Los hepatocitos están interconectados por uniones herméticas
El sinusoide es un capilar fenestrado
La célula de Kupffer se encuentra fuera del sinusoide y fagocita células
La célula de Ito se encuentra en el espacio de Disse y reserva glucógeno
• Pregunta 4
0 de 2 puntos
¿Cuál de las siguientes alternativas es correcta sobre la estructura hepática?

Respuesta seleccionada:
Los colangiocitos producen bilis
Respuestas: El flujo sinusoidal en el lobulillo hepático es de adentro hacia afuera
En la triada portal, se encuentra la vena derivada de la suprahepática
Los colangiocitos producen bilis

El flujo biliar en el lobulillo hepático es centrífugo


• Pregunta 5
0 de 2 puntos
Las siguientes sustancias son secretadas por el páncreas, EXCEPTO:

Respuesta seleccionada:
Péptido Monitor
Respuestas: Nucleasas
Péptido Monitor
Lipasa

Tripsina
• Pregunta 6
0 de 2 puntos
La presión parcial de oxígeno en la Zona 1 del sinusoide hepático de debe ser ___________ mmHg

Respuesta seleccionada:
100
Respuestas: menor de 40
100
40

entre 95 y 45
• Pregunta 7
2 de 2 puntos
Para aumentar la cantidad de bicarbonato en el flujo pancreático es necesario:

Respuesta seleccionada:
estimular las células S
Respuestas:
estimular las células S
estimular la secreción de péptido liberador de colecistoquinina (CCK)
ingerir aminoácidos como triptófano o fenilalanina
inhibir al nervio vago
• Pregunta 8
0 de 2 puntos
En un paciente con intoxicación por órganos fosforados, la acción de la colecistoquinina (CCK) está bloqueada a
nivel de:

Respuesta seleccionada:
el sistema nervioso central
Respuestas:
el esfínter de Oddi
la célula parietal
el sistema nervioso central
la vesícula biliar
• Pregunta 9
2 de 2 puntos
El efecto de un medicamento colerético se evidencia por:

Respuesta seleccionada:
el aumento de secreción biliar
Respuestas:
el aumento de secreción biliar
el aumento de formación de micelas
la disminución de absorción de sales biliares
la mayor producción de colesterol en la bilis
• Pregunta 10
2 de 2 puntos
Marque lo correcto respecto al acino hepático:

Respuesta seleccionada:
Dos vértices del rombo acinar se constituyen por triadas portales
Respuestas: La zona 1 es la más que recibe más sangre
La zona 3 se afecta en menos en una deshidratación severa

Dos vértices del rombo acinar se constituyen por triadas portales


El eje menor lo constituye un eje imaginario entre dos venas centrolobulillares

Aquellas células que producen mayor cantidad de moco en el epitelio gástrico son las:

Respuesta seleccionada:
mucosas superficiales
Respuestas:
mucosas superficiales
enterocromafines
mucosas del fondo
mucosas del cuello
• Pregunta 2
0 de 2 puntos
Al usar un parasimpaticomimético (agonista colinérgico), usted espera que la saliva presente:

Respuesta seleccionada:
hipertonicidad
Respuestas: hipertonicidad
una menor cantidad de bicarbonato

una mayor cantidad de sodio


una menor cantidad de cloro
• Pregunta 3
2 de 2 puntos
En la producción de HCl, la acción de la somatostatina disminuye la accion de:

Respuesta seleccionada:
la gastrina
Respuestas: la acetilcolina

la gastrina
la histamina
las prostaglandinas
• Pregunta 4
2 de 2 puntos
A raíz de la infección por Helicobacter pilory, la destrucción de las células ____________ ocasiona que aumente la
producción de ácido en el estómago y por ello se produzca úlceras ____________

Respuesta seleccionada:
D / duodenales
Respuestas: G / gástricas
parietales / gástricas

D / duodenales
S / pancreáticas
• Pregunta 5
0 de 2 puntos
Cuando un paciente recibe estímulo autónomo mixto (simpático y parasimpático), el flujo de saliva:

Respuesta seleccionada:
es menor que frente a un estímulo simpático aislado
Respuestas:
aumenta en relación al basal
es menor que frente a un estímulo simpático aislado
disminuye en relación al basal
es mayor que frente a un estímulo parasimpático aislado
• Pregunta 6
2 de 2 puntos
La vena porta se forma gracias a la unión de la vena mesentérica superior con la vena:

Respuesta seleccionada:
esplénica
Respuestas:
esplénica
mesentérica inferior
celiaca
gástrica izquierda
• Pregunta 7
2 de 2 puntos
En la anemia perniciosa, con atrofia gástrica marcada, usted esperaría encontrar disminución en la absorción de
___________ a nivel de ________

Respuesta seleccionada:
vitamina B12 / íleon distal
Respuestas: hierro / duodeno
ácido fólico / yeyuno
proteínas / yeyuno

vitamina B12 / íleon distal


• Pregunta 8
0 de 2 puntos
La fase intestinal de la secreción gástrica se debe básicamente a la participación de las células:

Respuesta seleccionada:
D del estómago
Respuestas: I del yeyuno

G del duodeno
S del íleon
D del estómago
• Pregunta 9
0 de 2 puntos
La comunicación entre la irrigación gástrica y la esofágica depende de una rama de la arteria:

Respuesta seleccionada:
mesentérica superior
Respuestas: mesentérica inferior

tronco celiaco
hepática común
mesentérica superior
• Pregunta 10
2 de 2 puntos
En las glándulas salivales, el principal lugar de intercambio iónico se da a nivel del:

Respuesta seleccionada:
conducto estriado
Respuestas:
conducto estriado
acino
conducto excretor
conducto intercalar


Durante la deglución, el bolo es impedido de regresar a la cavidad oral gracias a la acción de diversos músculos, entre ellos
el músculo:

Respuesta seleccionada:
cricofaríngeo
Respuestas: salpingofaríngeo
estilofaríngeo

palatofaríngeo
cricofaríngeo
• Pregunta 2
0 de 0,8 puntos
El músculo liso gastrointestinal funciona como un sincitio debido a:

Respuesta seleccionada:
el plexo mientérico de Auerbach
Respuestas:
las uniones en hendidura
el plexo mientérico de Auerbach
las fibras musculares más largas
el calcio
• Pregunta 3
0,8 de 0,8 puntos
Durante el paso del bolo hacia la orofaringe, se desencadena una serie de contracciones musculares que
estrechan la cavidad faríngea. Estas contracciones están mediadas por el nervio craneal:

Respuesta seleccionada:
X
Respuestas: IX
XII
XI

X
• Pregunta 4
0 de 0,8 puntos
Luego de una comida rica en proteínas, se espera que los valores de secretina en sangre:

Respuesta seleccionada:
se mantenga sin cambio
Respuestas: disminuyan

aumenten
sólo se altere si hay un aumento de colecistoquinina (CCK)
se mantenga sin cambio
• Pregunta 5
0 de 0,8 puntos
Al realizar una vagotomía por úlcera péptica, usted esperaría una disminución de producción de gastrina debido a
la:

Respuesta seleccionada:
menor distensión de las paredes gástricas
Respuestas: ausencia de acetilcolina vagal
menor distensión de las paredes gástricas
ausencia de histamina

ausencia de bombesina vagal


• Pregunta 6
0,8 de 0,8 puntos
Paciente de 34 años es víctima de asalto con arma de fuego, recibiendo un impacto directo en el abdomen. En
base a la radiografía, usted puede registrar en la historia clínica que el proyectil se encuentra topográficamente
en el:

Respuesta seleccionada:
hipocondrio izquierdo
Respuestas:
hipocondrio izquierdo
mesogastrio
flanco derecho
flanco izquierdo
• Pregunta 7
0 de 0,8 puntos
Señale cuál de las estructuras que en el embrión se encuentra comunicada con el saco vitelino por medio del
conducto onfalomesentérico:

A
• Pregunta 8
0 de 0,8 puntos
Considerando sus conocimientos en embriología, la disposición del intestino delgado final tras la retracción de
asas intestinales, se distribuye de la siguiente manera:

Respuesta seleccionada:
duodeno en hipocondrio izquierdo
Respuestas:
asas ileales en cuadrante inferior derecho
duodeno en hipocondrio izquierdo
asas yeyunales en hipogastrio
asas yeyunales en cuadrante inferior derecho
• Pregunta 9
0 de 0,8 puntos
Los corpúsculos gustativos se encuentran en la lengua, pero además se les puede encontrar en:

Respuesta seleccionada:
el dorso de la lengua
Respuestas: el dorso de la lengua

el paladar blando
el paladar duro
las encías
• Pregunta 10
0,8 de 0,8 puntos
¿Cuál de las siguientes alternativas es correcta sobre la motilidad esofágica?

Respuesta seleccionada:
Las ondas secundarias son propulsoras y no van precedidas de deglución
Respuestas: Las ondas secundarias son propulsoras y siempre van precedidas de deglución
Las ondas primarias son propulsoras y pueden no ser precedidas por deglución

Las ondas secundarias son propulsoras y no van precedidas de deglución


Las ondas primarias no son propulsoras y siempre van precedidas de deglución
• Pregunta 11
0 de 0,8 puntos
¿Cuál de las siguientes alternativas estimula las ondas de motilidad gastrointestinal denominadas complejos
migratorios interdigestivos?

Respuesta seleccionada:
Metoclopramida
Respuestas: Alimentos
Metoclopramida
Colecistoquinina (CCK)

Eritromicina
• Pregunta 12
0,8 de 0,8 puntos
En un estudiante de medicina que está rindiendo un examen parcial, lo más probable es que en ese momento su
tránsito intestinal se encuentre:

Respuesta seleccionada:
muy lento
Respuestas:
muy lento
sin alteraciones
estimulado por acción de la sustancia P
muy acelerado
• Pregunta 13
0,8 de 0,8 puntos
El principal gobernante sobre todos los movimientos gastrointestinales es el sistema nervioso:

Respuesta seleccionada:
mientérico
Respuestas: parasimpático

mientérico
simpático
somático
• Pregunta 14
0 de 0,8 puntos
El peristaltismo depende que a nivel distal del bolo se secrete:

Respuesta seleccionada:
acetilcolina por las neuronas provenientes del nervio vago
Respuestas: noradrenalina secretada por las fibras del sistema simpático
acetilcolina por las neuronas provenientes del nervio vago

péptido intestinal vasoactivo por neuronas


óxido nítrico por células endoteliales locales
• Pregunta 15
0,8 de 0,8 puntos
El estímulo habitual para el movimiento peristáltico es:

Respuesta seleccionada:
distensión local
Respuestas:
distensión local
contracción de la musculatura circular Interna
estimulación vago-vagal
acción de la sustancia P
• Pregunta 16
0,8 de 0,8 puntos
El peristaltismo intestinal se produce gracias a un reflejo que:

Respuesta seleccionada:
se origina dentro de la pared intestinal
Respuestas:
se origina dentro de la pared intestinal
llega al sistema nervioso central
llega a los ganglios pre vertebrales
llega a la médula espinal
• Pregunta 17
0 de 0,8 puntos
¿Cuál de las siguientes alternativas es correcta sobre el movimiento peristáltico?

Respuesta seleccionada:
Es un reflejo largo que depende de la integración con el tronco encefálico
Respuestas: Se dirige en sentido distal siempre, nunca en sentido proximal
Es independiente del plexo mientérico
Es un reflejo largo que depende de la integración con el tronco encefálico

El contenido intestinal avanza sólo 5-10 cm


• Pregunta 18
0,8 de 0,8 puntos
Niña de 6 años se asusta por que se le ha aflojado un diente deciduo. Este fenómeno se produce por:

Respuesta seleccionada:
laxitud del ligamento periodóntico
Respuestas: desmineralización del esmalte dental
fractura del cemento

laxitud del ligamento periodóntico


aumento anómalo de la predentina
• Pregunta 19
0 de 0,8 puntos
¿Cuál de las siguientes alternativas detallan los músculos que ayudan a empujar el bolo hacia la orofaringe?
Respuesta seleccionada:
Hiogloso y geniogloso
Respuestas: Hiogloso y geniogloso

Estilogloso y palatogloso
Estilogloso y geniogloso
Geniogloso y palatofaríngeo
• Pregunta 20
0 de 0,8 puntos
Al ingresar líquidos o sólidos en la cavidad oral, un mecanismo que permite que una persona respire mientras
mastica es:

Respuesta seleccionada:
la elevación del paladar blando
Respuestas: el movimiento hacia afuera de los pliegues palatogloso y palatofaríngeo
la depresión de la parte posterior de la lengua
la elevación del paladar blando

la depresión del paladar blando


• Pregunta 21
0,8 de 0,8 puntos
¿Cuál de las siguientes alternativas es una característica de la estructura del esófago?

Respuesta seleccionada:
Contiene glándulas submucosas principalmente en su tercio distal
Respuestas: Aumenta la presión intra esofágica durante la inspiración
El esfínter esofágico inferior es un esfínter anatómico

Contiene glándulas submucosas principalmente en su tercio distal


Contiene músculo estriado en casi toda su longitud
• Pregunta 22
0,8 de 0,8 puntos
En el plexo mientérico, el origen de los impulsos eferentes está en:

Respuesta seleccionada:
las células intersticiales de Cajal
Respuestas: el plexo de Aurbach
las células intersticiales de Cajal

el plexo de Meissner
los ganglios paravertebrales
• Pregunta 23
0,8 de 0,8 puntos
¿Cuál de las siguientes alternativas es correcta sobre el control del peristaltismo?

Respuesta seleccionada:
El peristaltismo intestinal aumenta por efecto de la colecistoquinina (CCK)
Respuestas: La serotonina no tiene efecto sobre la motilidad digestiva
El peristaltismo intestinal aumenta por efecto de la colecistoquinina (CCK)
La secretina aumenta la motilidad del intestino delgado
El reflejo gastroentérico evita el avance del quimo
• Pregunta 24
0,8 de 0,8 puntos
¿Cuál de las siguientes estructuras tiene inervación somática?

Respuesta seleccionada:
Peritoneo parietal
Respuestas: Estómago

Peritoneo parietal
Peritoneo visceral
Mesosigmoides
• Pregunta 25
0,8 de 0,8 puntos
En una cirugía abierta (laparotomía), el cirujano al abrir la cavidad peritoneal por la parte anterior (línea media),
lo primero que observa es:

Respuesta seleccionada:
Epiplón mayor
Respuestas: Estomago
Colon sigmoides

Epiplón mayor
Duodeno

• Pregunta 1
2 de 2 puntos
Al seccionar el nervio facial a nivel timpánico, usted esperaría:

Respuesta seleccionada:
Disminución del gusto en la punta de la lengua
Respuestas: Imposibilidad para protruir la lengua
Imposibilidad para el cierre del istmo de las fauces.

Disminución del gusto en la punta de la lengua


Ausencia de termoalgesia en la lengua
• Pregunta 2
0 de 2 puntos
Al introducir una solución azucarada directamente al estómago mediante una gastrostomía (comunicación entre
la piel abdominal y el estómago), la sustancia que provocará que aumenten los niveles séricos de insulina es:

Respuesta seleccionada:
Glucagon
Respuestas: Glucagon
Enteroglucagon
Somatostatina
Péptido tipo glucagón 1 (GLP-1)
• Pregunta 3
0 de 2 puntos
Es un ligamento derivado del mesenterio dorsal:

Respuesta seleccionada:
Faciforme
Respuestas: Hepatogástrico

Gastrocólico
Redondo
Faciforme
• Pregunta 4
0 de 2 puntos
Al disminuir el pH duodenal por el HCl gástrico, se libera principalmente una hormona cuya célula diana es:

Respuesta seleccionada:
Células S del intestino
Respuestas: Acinos pancreáticos
Células S del intestino
Células ductales del colédoco

Célula ductal del Wirsung


• Pregunta 5
0 de 2 puntos
Paciente con apendicitis aguda, que debuta con dolor en mesogastrio. Este dolor se debe a estimulación de
receptores del dolor cuyas fibras van a viajar a la médula espinal a través de:

Respuesta seleccionada:
Plexo hipogástrico
Respuestas: Nervio esplácnico pélvico
Nervio vago

Nervios simpáticos
Plexo hipogástrico
• Pregunta 6
0 de 2 puntos
Una de las siguientes sustancias no comparte con las otras la misma acción sobre la producción de ácido gástrico:

Respuesta seleccionada:
Secretina
Respuestas:
Colecistoquinina
Somatostatina
Secretina
Péptido insulinotrópico dependiente de glucosa (GIP)
• Pregunta 7
2 de 2 puntos
En un experimento, con una sonda nasogástrica se instila por goteo en el estómago una sustancia líquida, y se
obtiene como respuesta una dramática disminución del pH del estómago. Dicha sustancia debe contener:

Respuesta seleccionada:
Aminoácidos
Respuestas:
Aminoácidos
Lípidos
Carbohidratos
Secretina
• Pregunta 8
2 de 2 puntos
Al ingerir grandes cantidades de dulces, con la subsecuente estimulación de incretinas, usted esperaría que el
apetito ______________, debido a __________________

Respuesta seleccionada:
disminuya insulina
Respuestas: aumente grelina

disminuya insulina
disminuya CCK
aumente CCK
• Pregunta 9
0 de 2 puntos
Al ingerir rápidamente un litro de agua, usted esperaría que la gastrina aumente por efecto de:

Respuesta seleccionada:
ACh por estimulación del nervio vago
Respuestas:
ACh del sistema mientérico
ACh por estimulación del nervio vago
efecto paracrino de la histamina
aumento de acidez gástrica (disminución del ph)
• Pregunta 10
2 de 2 puntos
En un paciente con shock distributivo, usted decide iniciar noradrenalina por un catéter CVC, consiguiendo
aumentar la presión arterial. ¿qué efecto sobre la motilidad intestinal esperaría encontrar?

Respuesta seleccionada:
El potencial de reposo de las fibras musculares se hace más negativo
Respuestas:
El potencial de reposo de las fibras musculares se hace más negativo
Aumentan las ondas lentas, pero disminuyen los potenciales en espiga
Aumenta la mezcla en el antro gástrico
Aumenta el peristaltismo.

• Pregunta 1
2 de 2 puntos
Dentro de las funciones del abdomen, se encuentra la defecación y micción, en las cuales la presión intra
abdominal debe:

Respuesta seleccionada:
Aumentar
Respuestas: Disminuir

Aumentar
Mantenerse igual
No tiene relación el abdomen con dichas funciones
• Pregunta 2
2 de 2 puntos
Paciente joven es traído a emergencia con abdomen agudo quiúrgico debido a herida contusopenetrante por
verduguillo (alambre grueso con punta aguzada) recibida en una pelea después de un partido de futbol. Se
observa herida en Hipocondrio Izquierdo. El órgano que debe estar sangrando y produciendo hemoperitoneo es
(marque la mejor respuesta):

Respuesta seleccionada:
Bazo
Respuestas: Colon sigmoides
Colon ascendente
Hígado

Bazo
• Pregunta 3
2 de 2 puntos
El ligamento hepatogástrico une el _______________ con el _______________ y forma la entrada al ___________:

Respuesta seleccionada:
Hígado Estómago Orificio omental
Respuestas: Hígado Estómago Orificio esofágico
Estómago Hígado Orificio anal

Hígado Estómago Orificio omental


Estómago Hígado Orificio gastrointestinal
• Pregunta 4
0 de 2 puntos
Al evaluar una tomografía abdominal, el médico asistente le pide al interno de la UPC que encuentre la imagen
con el corte a nivel de L1. El interno sabiamente busca el ________ para ubicar la vértebra L1.

Respuesta seleccionada:
Cruce entre la Aorta y la Vena Porta
Respuestas:
Cuello del páncreas
Nacimiento de la vena mesentérica superior
Cruce entre la Aorta y la Vena Porta
Nacimiento de la arteria mesentérica inferior
• Pregunta 5
0 de 2 puntos
En la inspiración, la pared abdominal debe ____________ para ____________:
Respuesta seleccionada:
Contraerse aumentar presión intra torácica
Respuestas: Relajarse aumentar presión intra abdominal

Relajarse disminuir presión intra torácica


Contraerse aumentar presión intra abdominal
Contraerse aumentar presión intra torácica
• Pregunta 6
2 de 2 puntos
Paciente con vólvulo del colon sigmoides. La necrosis de este segmento del colon se produce por una alteración
en la irrigación de la arteria:

Respuesta seleccionada:
Mesentérica inferior
Respuestas: Mesentérica superior
Pélvica

Mesentérica inferior
Tronco celíaco
• Pregunta 7
2 de 2 puntos
Marque el órgano que se considera retroperitoneal:

Respuesta seleccionada:
Parte de la vía biliar
Respuestas:
Parte de la vía biliar
Sigmoides
Vesícula biliar
Lóbulo izquierdo del hígado
• Pregunta 8
2 de 2 puntos
La estructura que fija órganos principalmente a la pared posterior abdominal se denomina:

Respuesta seleccionada:
Mesenterio
Respuestas: Fascia transversalis

Mesenterio
Omento
Ligamento
• Pregunta 9
0 de 2 puntos
Durante el vómito, ¿el contenido gástrico tiene que pasar necesariamente por cuál estructura para llegar al
esófago? Marque la mejor respuesta:

Cardias
• Pregunta 10
2 de 2 puntos
Al examinar a un paciente, usted encuentra dolor localizado en fosa iliaca derecha y diagnostica apendicitis. En
este paciente, usted puede inferir:

Respuesta seleccionada:
El peritoneo parietal regional está afectado
Respuestas:
El peritoneo parietal regional está afectado
BANCO
DIGESTIVO
2021-1
BANCO DIGESTIVO 2020-2

1) Al examinar a un paciente, usted encuentra dolor localizado en fosa iliaca derecha y


diagnostica En este paciente, usted puede inferir.

a) El peritoneo parietal regional está afectado apendicitis.


b) El peritoneo visceral regional está principalmente afectado
c) Hay inflamación de todo el peritoneo parietal (peritonitis)
d) El diagnóstico está errado por no corresponder a la región abdominal adecuada

2) La motilidad intestinal es estimulada principalmente por el:


a) Plexo de Auerbach
b) Sistema simpático
c) Sistema parasimpático
d) Sistema piramidal

3) Durante el vómito, ¿el contenido gástrico tiene que pasar necesariamente por qué
estructura para llegar al esófago? Marque la mejor respuesta:
a) Cardias
b) Papila duodenal
c) Cuerpo
d) Antro

4) Respecto a la anatomía del estómago, marque lo correcto:


a) La incisura angularis puede estar en la curvatura mayor
b) El fondo gástrico forma la curvatura mayor
c) El píloro se encuentra en el cuerpo gástrico
d) La porción más distal del estómago es el cardias

5) Marque la respuesta incorrecta:


a) En todo el tubo digestivo, se observa dos capas de muscular propia: circular interna y
longitudinal externa
b) Fuera de la cavidad abdominal, el esófago presenta capa adventicia
c) La mucosa consta de epitelio, lámina propia y muscularis mucosae
d) El colon contiene tenias

6) Paciente se queja de dolor en hipocondrio derecho, pero superficialmente. El dermatoma


relacionado es (marque la mejor respuesta)
a) T9
b) T11
c) T10
d) T12

7) Dentro de las funciones del abdomen, se encuentra la defecación y micción, en las cuales
la presión intraabdominal debe:
a) Aumentar
b) No tiene relación el abdomen con dichas funciones
c) Disminuir
d) Mantenerse igual

8) Un alumno de medicina decide hacerse un piercing en el ombligo. Al realizarle el


procedimiento, sangra ligeramente. Esta sangre proviene de la arteria (marque la mejor
respuesta)
a) Epigástrica inferior
b) Circunfleja ilíaca superficial
c) Circunfleja ilíaca profunda
d) Torácica interna

9) Señala la respuesta correcta:


a) El apéndice cecal solo tiene serosa
b) El páncreas solo tiene adventicia
c) El colon ascendente solo tiene serosa
d) El esófago solo tiene adventicia

10) Paciente mujer es traída a emergencia por sufrir una herida contuso penetrante por
cuchillo realizada por su esposo en un ataque de celos. Se observa herida en flanco izquierdo.
Está solución de continuidad ha comprometido varios músculos de la pared abdominal,
excepto:
a) oblicuo externo
b) oblicuo interno
c) recto abdominal
d) transverso

11) Marque el órgano que se considera retroperitoneal:


a) Lóbulo izquierdo del hígado
b) Vesícula biliar
c) Sigmoides
d) Parte de la vía biliar

12) La peristalsis o peristaltismo hace referencia a:


a) Motilidad para movilizar el alimento de proximidad a distancia.
b) Motilidad para mezclado de alimentos.
c) No es parte de la motilidad
d) Motilidad para fraccionamiento de alimentos.

13) Paciente tiene una úlcera sangrante en el segundo tercio del yeyuno. La arteria de la cual
proviene la sangre arterial para dicha zona es la arteria:
a) Mesentérica superior
b) Iliaca común
c) Tronco celiaco
d) Mesentérica inferior

14) La estructura que fija órganos principalmente a la pared posterior abdominal se denomina:
a) Mesenterio
b) Omento
c) Ligamento
d) Fascia transversalis
15) Al iniciar la digestión, aumenta el consumo de oxígeno por la mucosa. Esto conlleva a una
hipoxia local, lo cual hace que se libere _____________, el cual produce vasodilatación:
a) Histamina
b) Noradrenalina
c) Adenosina
d) Colecistoquinina

16) Es inervado por aferentes somáticas:


a) Peritoneo parietal
b) Mesenterio
c) Omento
d) Peritoneo visceral

17) El ligamento hepatogástrico une el __________ con el _______ y forma la entrada al


________
a) Hígado Estómago Orificio omental
b) Estomago Higado Orificio anal
c) Estómago Hígado Orificio gastrointestinal
d) Hígado Estómago Orificio esofágico

18) Al retirar completamente el mesenterio de un órgano, el mismo se vería afectado


principalmente en su:
a) Irrigación
b) No se afecta en absoluto
c) Inervación
d) Tamaño

20) Paciente con vólvulo del colon sigmoides. La necrosis de este segmento del colon se
produce por una alteración en la irrigación de la arteria:
a) Mesentérica superior
b) Tronco celíaco
c) Mesentérica inferior
d) Pélvica

21) Marque el órgano que se encuentra más distal en el tubo digestivo.


a) Íleon
b) Ciego
c) Duodeno
d) Estómago

22) Paciente de 24 años con dolor abdominal tipo cólico intenso en mesogastrio. Según sus
conocimientos de macroestructura, el origen del dolor puede ser el ___________:
a) Esófago
b) Estómago
c) Íleon
d) Colón
23) Paciente con herida por proyectil por arma de fuego, con herida de ingreso en región
paraumbilical. Entre las estructuras que usted está seguro que debe haberse lesionado es:
a) Ligamento de Treitz
b) Omento mayor
c) Mesosigmoides
d) Omento menor

24) Paciente joven es traído a emergencia con abdomen agudo quirúrgico debido a herida
contuso penetrante por verduguillo (alambre grueso con punta aguzada) recibida en una
pelea después de un partido de fútbol. Se observa herida en Hipocondrio Izquierdo. El
órgano que debe estar sangrando y produciendo hemoperitoneo es (marque la mejor
respuesta):
a) Colon ascendente
b) Hígado
c) Bazo
d) Colon sigmoides

25) Al evaluar una tomografía abdominal, el médico asistente le pide al interno de la UPC
que encuentre la imagen con el corte a nivel de L1. El interno sabiamente busca el
________ para ubicar la vértebra L1.
a) Nacimiento de la vena mesentérica superior
b) Nacimiento de la arteria mesentérica inferior
c) Cuello del páncreas
d) Cruce entre la Aorta y la Vena Porta

26) En la inspiración, la pared abdominal debe ____________ para ____________:


a) Relajarse aumentar presión intra abdominal
b) Contraerse aumentar presión intra torácica
c) Contraerse aumentar presión intra abdominal
d) Relajarse disminuir presión intra torácica

27) Cuál de las siguientes estructuras no tiene vasos sanguíneos:


a) Epitelio intestinal
b) Omento
c) Mesenterio
d) Ligamento

28) Paciente con hipoglucemia secundaria a un insulinoma (tumor neuroendocrino productor


de insulina). El órgano donde con mayor probabilidad ha crecido este tumor es:

a) Pélvico
b) Torácico
c) Retroperitoneal
d) Intraabdominal

30) Sustancia que inhibe la secreción y la motilidad del estómago prolongando el tiempo de
digestión:
- Péptido insulinotrópico dependiente de la glucosa (GIP)
31) Marque lo correcto:
- La hernia fisiológica se produce en la sexta semana y es la salida temporal de
asas intestinales a través del cordón umbilical

32) Marque la respuesta correcta en relación a la gastrina:


- Las células G son las productoras y se encuentran principalmente en el antro
gástrico

33) El consumir caramelos indirectamente activa la vía:


- POMC/CART

34) ¿En qué capa se encuentra la alteración principal en el Hirschsprung o megacolon


agangliónico?:
- Muscular propia

35) Con respecto a las ondas lentas, marque la afirmación correcta:


- Son contracciones rítmicas espontáneas

36) El uso de Ranitidina bloquea el receptor H2 de la histamina en las células parietales. La


histamina llega a estas células por:
- Difusión

37) La triada sintomática: vómitos explosivos post-prandiales, movimientos peristálticos


epigástricos visibles de izquierda a derecha y nódulo palpable epigástrico subcostal
derecho, pertenecen a:
- Estenosis congénita hipertrófica del píloro.

38) Durante una cirugía oncológica, ¿la extirpación de cuál de los siguientes órganos se
vería comprometida por la presencia de adventicia?:
- Recto

39)En cuanto a los reflejos gastrointestinales, un reflejo que estimula el tránsito intestinal es
el reflejo:
- Gastrocólico
- Cólico-ileal
- Entero-gástrico
- Vómito

40) El ligamento falciforme divide al hígado en dos lóbulos derecho e izquierdo.


Embriológicamente deriva del:
- Mesenterio ventral

41) La presencia de atresias y estenosis duodenales se deben básicamente a una:


- Falta de recanalización

42) Estudiante de medicina de 20 años, se ha amanecido estudiando para su examen de


Sistema Digestivo. No ha probado alimento desde la cena, por lo que se puede afirmar que
la motilidad de esta persona está siendo regulada por:
- Motilina

43) Paciente con disminución del apetito marcada asociada a cáncer terminal, para
promover la ingesta de alimentos se podría usar análogos de:
- Endorfinas

44) Las ondas lentas se producen por la apertura cíclica de canales de:
- Calcio

45) La forma más común de atresia esofágica contiene:


- Estenosis proximal del esófago con fístula traqueoesofágica distal

46) Al deglutir un bolo alimenticio, es lógico suponer que al pasar por el esófago haya un
mayor consumo de oxígeno en la pared del tercio:
- Proximal

47) Paciente que come entera una pizza familiar de chorizo y queso. Es posible esperar que
debido a la cantidad de alimento ingerida, las ondas lentas hayan:
- Sufrido ninguna alteración en su frecuencia

48) La hernia fisiológica se produce dentro de:


- Cordón umbilical

49) El crecimiento de un adenocarcinoma de páncreas compromete la pared gástrica por


contigüidad. ¿Qué parte del estómago se esperaría esté comprometido?
- Pared posterior del antro

50) Estimula la producción de saliva:


- Vasodilatación periglandular

51) Durante la secreción de saliva, es de esperarse que las concentraciones de ________ y


______ disminuyen al disminuir el flujo:
- Sodio Bicarbonato

52) Con respecto a la secreción gástrica de HCl:


- a mayor secreción de HCl en el lumen gástrico, mayor pH en la sangre venosa
gástrica

53) Respecto a las enfermedades del esófago, marque lo correcto:


- el diagnóstico diferencial de la acalasia es la enfermedad de Chagas esofágica

54) Con respecto a las lesiones y enfermedades de la boca, marque lo correcto:


- la eritroplasia debe ser biopsiada

55) Respecto a las glándulas salivales, marque lo incorrecto:


- la glándula sublingual tiene forma de garfio

56) El omeprazol actúa sobre la membrana _____________ de la célula ____________


- apical / parietal
57) Durante el sueño, la concentración de bicarbonato en la saliva:
- Disminuye

58) Durante el ataque con gas sarín (bloqueador de la acetilcolinesterasa) en el metro de


Tokio, en 1995, el personal de salud notó que los pacientes afectados presentaban:
- Hipersalivación

59)La célula mucosa del cuello gástrico produce:


- Moco

60) En la evaluación de una tomografía abdominal, el interno observa un aneurisma en una


arteria que se dirige al riñón derecho. Con seguridad se puede afirmar que está a nivel de la
vértebra:
- L1

61) Un un paciente con diarrea por hipermotilidad, usted sospecharía en el posible aumento
de las siguientes sustancias, excepto:
a) Péptido intestinal vasoactivo
b) Sustancia P
c) ACh
d) Motilina

62) Una de las siguientes sustancias no comparte con las otras la misma acción sobre la
producción de ácido gástrico:
a) Péptido insulinotrópico dependiente de glucosa (GIP)
b) Colecistoquinina
c) Somatostatina
d) Secretina

63) Usted encuentra músculo estriado en el siguiente segmento:


a) Esfínter anal externo
b) Duodeno
c) Tercio inferior esofágico
d) Esfinter anal interno

64) Paciente con intoxicación por organofosforados (inhibidores de acetilcolinesterasa), se


espera que el tránsito intestinal se encuentre:
a) Aumentado
b) Disminuido
c) En relación inversa al número de ondas lentas por minuto
d) No se espera ninguna alteración

65) Al disminuir el pH duodenal por el HCl gástrico, se libera principalmente una hormona
cuya célula diana es:
a) Células S del intestino
b) Acinos pancreáticos
c) Células ductales del colédoco
d) Célula ductal del wirsung
66) La hormona que tiene un efecto sinérgico con la secretina para optimizar el pH duodenal
y la digestión, es:
- CCK (colecistoquinina)

67) El ligamento falciforme del hígado proviene embriológicamente de:


- Mesenterio ventral
-

68) Todos los músculos motores de la lengua están inervados por el XII par, excepto:
- Palatogloso

69) Al ingerir grandes cantidades de dulces, con la subsecuente estimulación de incretinas,


usted esperaría que el apetito ______________, debido a __________________
- Disminuya insulina

70) Paciente obeso con Covid-19 es intubado por interno inexperto, quien al solicitar que
bombeen aire dentro del tubo endotraqueal, nota que el epigastrio se distiende. Al sospechar
que ha introducido el tubo en el estómago, también es cierto que:
- Disminuye el pH gástrico
- Aumenta el pH gástrico
- Disminuiría el tono del píloro
- Aumenta la frecuencia de ondas lentas

71) Una de las siguientes sustancias reguladoras, puede actuar de forma paracrina y como
hormona. Marque la correcta:
- Somatostatina
- Acetilcolina
- Péptido insulinotrópico dependiente de glucosa.
- GRP

72) Durante una cirugía oncológica, el cirujano observa que los órganos abdominales tienen
libre movimiento dentro de la cavidad abdominal, excepto:
- Yeyuno
- Vesícula biliar
- Colon ascendente
- Estómago

73) Al consumir un pan con mantequilla, la sensación de hambre disminuye debido a la


acción de:
- Grelina
- Somatostatina
- Colecistoquinina (CCK)
- Leptina

74) Respecto a los péptidos gastrointestinales, marque lo correcto.


- No existe sustancia neurocrina que tenga efecto en la motilidad del tubo digestivo
- Las sustancias paracrinas pueden viajar a través de vasos sanguíneos
- Las sustancias neurocrinas son péptidos que hacen su efecto en distancias cortas
- Las sustancias paracrinas atraviesan la circulación portal

75) En un paciente con gastroparesia (motilidad lenta del estómago), que presenta
distensión abdominal después de comer, usted le recomendaría que evite el consumo de
lípidos y aminoácidos para disminuir la acción de:

- Secretina
- CCK
- Somatostatina
- Gastrina

76) Paciente con apendicitis aguda, que debuta con dolor en mesogastrio. Este dolor se
debe a estimulación de receptores del dolor cuyas fibras van a viajar a la médula espinal a
través de:

● Nervio vago
● Plexo hipogástrico
● Nervios simpáticos
● Nervio esplácnico pélvico

78) Al introducir una solución azucarada directamente al estómago mediante una


gastrostomía (comunicación entre la piel abdominal y el estómago), la sustancia que
provocará que aumenten los niveles séricos de insulina es:

● Somatostatina
● Péptido tipo glucagón 1 (GLP-1)
● Glucagón
● Enteroglucagon

79) En un experimento, con una sonda nasogástrica se instila por goteo en el estómago una
sustancia líquida, y se obtiene como respuesta una dramática disminución del pH del
estómago. Dicha sustancia debe contener:

● Lípidos
● Carbohidratos
● Aminoácidos
● Secretina

80) En un paciente con hiperestimulación simpática se espera que las ondas lentas tengan
un ritmo:

● Mayor en estómago que en duodeno


● Mayor en estómago que en íleon terminal
● Menor en íleon terminal que en el duodeno
● Mayor en íleon que en duodeno

81) Se considera que el gusto puede viajar a través del nervio:

- V3
- Vago
- Lingual
- Glosofaríngeo

82) Al seccionar el nervio facial a nivel timpánico, usted esperaría:

- Disminución del gusto en la punta de la lengua


- Ausencia de termoalgesia en la lengua
- Imposibilidad para protruir la lengua
- Imposibilidad para el cierre del istmo de las fauces.

83) El ecografista sabe que para poder visualizar el nacimiento de la arteria mesentérica
superior, debe colocar el transductor sobre la piel de la siguiente región abdominal:

- Hipogastrio
- Hipocondrio derecho
- Mesogastrio
- Epigastrio

84) Marque lo correcto en relación al divertículo de Meckel.

- Se encuentra usualmente a 60 cm de la VIC


- Contiene mucosa esofágica en algunas ocasiones
- Se relaciona a un defecto en el desarrollo del intestino posterior
- Se produce en el lado mesentérico del íleon

85) Es un ligamento derivado del mesenterio dorsal:

- Gastrocólico
- Hepatogástrico
- Redondo
- Faciforme

86) Marque lo correcto respecto a la siguiente imagen:


- Produce síntomas en la gran mayoría de pacientes.
- Se produjo por giro en sentido contrario del duodeno
- Se produjo por falta de fusión de los ductos dorsal y ventral
- Como tratamiento, se podría aperturar la papila mayor (esfinterotomía).

TIPEO DE EXÁMENES
1) La arteria mesentérica superior emerge de la aorta a nivel de:
- Cabeza de páncreas
- Hilio hepático
- Tronco celiaco
- Íleon
- Cardias

2) La digestión de las proteínas se inicia en:


- Estómago
- Intestino delgado
- Boca
- Esófago
- Páncreas

3) Aproximadamente en la semana 6 del desarrollo embrionario, el intestino intermedio


gira 90º herniándose a nivel del:
- Fístula esofágica
- Borde superior del hígado
- Borde superior del pubis
- Cordón umbilical
- Borde inferior del bazo

4) Paciente de 76 años en estado de coma, con colostomía (colon abocado a la piel) por
resección parcial de colon secundaria a carcinoma de colon. Usted deja indicado que
este día se realice el cambio de la bolsa de colostomía, pero que lo hagan después
de dos horas de la nutrición enteral, debido a que quiere que se haga después de:
- La presencia de ondas lentas
- La secreción de gastrina
- El reflejo gastrocólico
- La producción de GIP
- La liberación de motilina
5) Recién nacido de 14 días, que presenta estreñimiento, distensión abdominal y
vómitos. Al examinar el orificio anal, se evidencia conducto permeable, pero al
introducir un poco el termómetro, se evidencia salida de material fecal por el ano.
Usted sospecha que el problema se deba a:
- Falta de desarrollo del tabique urogenital
- Defecto en el desarrollo del conducto vitelino
- Defecto en el desarrollo de las células de cajal
- Falta de desarrollo del seno uretral
- Falta de regresión de la membrana anal

6) La lengua está recubierta por epitelio:


- Plano estratificado no queratinizado
- Pseudoestratificado columnar no queratinizado
- Pseudoestratificado columnar ciliado
- Plano estratificado queratinizado

7) El esfínter anal interno tiene musculatura ___ y tiene control ___:


- Lisa/voluntario
- Lisa/involuntario
- Esquelética/simpático
- Esquelética/parasimpático

8) La arteria aorta proporciona la irrigación al tubo digestivo ¿cuál de las siguientes


arterias de irrigación al ángulo cólico derecho?
- Mesentérica superior
- Mesentérica inferior
- Frénica inferior
- Tronco celiaco
9) Paciente de 26 años que le cuenta en su historia clínica que cada vez que almuerza,
a los 20 minutos tiene deseo de defecar. Le comenta que su hijo de 1 mes le pasa lo
mismo pero más intenso. Esto se explica por el reflejo ____, el cual está _____ en el
paciente:
- Colicoileal/normal
- Colicoileal/alterado
- Gastrocolico/normal
- Gastrocolico/alterado

10) La región del estómago que se comunica con el duodeno se denomina


- Pilórica
- Cardias
- Cuerpo
- Fórnix
11) Acude a consulta un paciente que fue diagnosticado de úlcera péptica tres días antes.
Luego de múltiples pruebas diagnósticas, se concluye que el paciente presenta un
tumor secretor de gastrina ¿cual de las siguientes situaciones estará incrementada?
- Distensión gástrica
- Inhibición del vaciado gástrico
- Secreción de ácido clorhídrico (HCl)
- Inhibición de la secreción de pepsinógeno

12) En el sistema digestivo el control del apetito está dado por un complejo sistema de
sustancias y órganos integradores los cuales regulan la ingesta de alimentos. La ____
es una sustancia orexígena/orexígena y es sintetizada por el ____
- Leptina/intestino
- Grelina/intestino
- Leptina/estómago
- Grelina/estómago

13) Sobre el control Autónomo del sistema digestivo marque la alternativa correcta:
- La inervación dada por el sistema simpático es de tipo preganglionar
- El sistema parasimpático usa como neurotransmisores a la acetilcolina y
noradrenalina
- El nervio vago (par craneal X) le da inervación simpática a la mayoría del sistema
digestivo
- En el sistema simpático, los nervios responsables hacen una primera sinapsis en
ganglios próximos al órgano a inervar
- En la inervación de tipo parasimpático, solo interviene el plexo submucoso, sin
embargo, en la de tipo simpático intervienen tanto el submucoso como el mientérico.

14) Con respecto a la actividad eléctrica del sistema digestivo, marque la alternativa
correcta:
- Corresponden a potenciales de acción que están presentes de forma continua y le dan
capacidad de peristalsis autónoma al sistema digestivo
- La frecuencia de las ondas lentas no se ve influenciada por la actividad neural ni las
hormonas gastrointestinales
- En el estómago las ondas lentas se dan en una frecuencia de 6 por minuto
- Las ondas lentas son cambios lentos y ondulantes del potencial en reposo
- La frecuencia de las ondas lentas va de 6 a 12 ondas por minuto

15) Ante una lesión en el IX par craneal, el músculo ___ se altera en su función:
- Palatogloso
- Estilofaríngeo
- Palatofaríngeo
- Constrictor superior

16) Un varón de 50 años es sometido a extirpación de duodeno y parte proximal del


yeyuno. La pérdida de estímulo hormonal en el páncreas para la secreción enzimática
se explica por la pérdida de las células:
- Parietales, productoras del factor intrínseco
- “K” productoras del factor intrínseco
- “M” productoras de CCK
- “I” productoras de CCK

17) Respecto al mecanismo de defecación ¿cual de las siguientes afirmaciones es


correcta?
- Se produce una contracción refleja del esfínter anal interno
- Se produce contracción o relajación del esfínter anal externo por señales de la corteza
cerebral
- La presencia de materia fecal en el recto estimula la contracción del sigmoides por los
nervios pélvicos simpáticos
- En la posición de “cuclillas” el músculo puborrectal se haya contraído favoreciendo la
evacuación de la materia fecal

18) Un niño de tres años llega a emergencia con disfagia (dificultad para tragar), salivación
y llanto. Se sospecha de ingesta de cuerpo extraño (moneda en el esófago), al ser
evaluado se constata de una radiografía presencia de cuerpo extraño a nivel de C6 y
C7 (6º y 7º vértebra cervical). El cuerpo extraño estará suspendido a nivel del
estrechamiento producido por el:
- Cayado aórtico
- Hiato esofágico
- Músculo cricofaríngeo
- Bronquio principal izquierdo

19) En el caso de un paciente con un tumor productor de gastrina, la presencia de úlceras


duodenales y erosión de la mucosa gástrica se debe principalmente a:
- La acción paracrina de la gastrina sobre la célula parietal
- El exceso de HCl por estímulo de receptores CCK-B en la célula parietal
- La sobre expresión de los receptores “G” para gastrina en la célula parietal
- El exceso de HCl por estímulo directo de receptores “H” en la célula parietal

16) La onda peristáltica secundaria del esófago se caracteriza por ser originada:

- Por el plexo de submucoso del esófago


- Por el plexo mientérico del esófago
- Por el reflejo de la deglución
- Durante la masticación

17) Marque lo correcto sobre las ondas lentas en el tubo digestivo:

- No son despolarizaciones
- Son potenciales de acción subumbrales
- Se constituyen de despolarizaciones y repolarizaciones
- Son rítmicas y generadas por el sistema nervioso autónomo

18) Recién nacido presenta protrusión de contenidos abdominales los cuales no están
cubiertos por peritoneo y salen de la cavidad abdominal a través de un defecto en la pared
¿Cómo se denomina a la afección que presenta este paciente?

- Onfalocele
- Atresia biliar
- Gastrosquisis
- Divertículo de Meckel

19) Experimentalmente se utiliza atropina (anticolinérgico) para inhibir la secreción de


gastrina, sin embargo, la secreción de esta hormona se sigue dando ante estímulos vagales.
Esta situación se explica porque la atropina:

- No bloquea la acción del péptido GRP


- Solo inhibe la acción del péptido GHRP en la célula G
- Inhibe la acción de acetilcolina e histamina en la célula G
- Bloquea parcialmente la bomba de protones en la célula G

20) La niña de 4 días es llevada a la emergencia pediátrica por presentar llanto constante,
la madre refiere coloración azulada de labios al momento de lactar, acompañado de
tos persistente y dificultad respiratoria, así como distensión abdominal. Se le coloca
sonda nasogástrica para alimentación notando que retorna a la cavidad oral en todos
los intentos ¿Cuál es la anomalía del desarrollo en este caso?
- Solo fistula traqueo esofagica
- Fístula traqueo esofágica proximal y distal
- Atresia esofágica proximal con fístula traqueo esofágica distal
- Atresia esofágica distal con fístula traqueoesofágica proximal

21) Paciente varón de 36 años es traído a la emergencia luego de sufrir un accidente de


tránsito presenta traumatismos múltiples en cabeza y tronco. Al examen físico se
evidencia hematoma en hemicara izquierda, ligera protrusión y caída del lado
izquierdo del maxilar inferior por lo que se le realiza tomografía donde se haya una
fractura de apófisis coronoides del maxilar inferior. ¿Qué músculo está relacionado
directamente con esta situación?
- Masetero
- Temporal
- Buccinador
- Pterigoideo medial

22) Un paciente refiere no percibir algunos sabores. Al examen físico constata alteración
del sabor dulce y umami. ¿Cuál de los siguientes nervios estará alterada su función?
- Cuerda del tímpano (VII par)
- Lingual (rama del V par)
- Glosofaríngeo (IX par)
- Hipogloso (XII par)

23) A los pocos días de nacido, regresa a neonatología un niño con problemas de
motilidad del colon, los estudios determinan ausencia congénita de células
ganglionares. Según el gráfico cuál es la capa en la que se determina la ausencia de
dichas células?
- Mucosa - 2
- Muscular propia - 2
- Muscular propia - 3
- Muscular de la mucosa - 1

24) Con respecto al control autonómico en el tracto gastrointestinal y en relación con su


fisiología ¿Cuál es la función del sistema nervioso parasimpático y el tracto
gastrointestinal?
- Inhiben la contracción muscular y estimulan la secreción de sustancias a nivel de la
submucosa
- Estimulan la contracción muscular y estimulan la secreción de sustancias a nivel de
la mucosa
- Inhiben la contracción muscular e inhiben la secreción de sustancias a nivel de la
submucosa
- Estimulan la contracción muscular e inhiben la secreción de sustancias a nivel de la
mucosa

25) Intersticio ubicado entre el estroma del espacio portal y los hepatocitos, y por donde
migran las células cancerígenas que hacen diseminación linfática es el:
- Espacio del Mall
- Espacio de Disse
- Espacio porta
- Espacio sideral

26) La presencia de grandes cantidades de TGF - Beta estimula a las ___________ y se


deposita colágeno, formándose la cirrosis
- Células de Ito
- Células de Kupffer
- Triadas portales
- Células fibroblásticas

27) Paciente de 64 años con ICC al que se le va a realizar cirugía cardiovascular. Al


calcular el volumen sanguíneo total, se debe considerar que el hígado puede
contener un volumen de sangre de ____________ mL en un adulto sano, en este
paciente ese volumen puede llegar a ser de _____________ mL
- 450 - 1000
- 250 - 1000
- 450 - 3000
- 250 - 10000

28) El área del lobulillo que se afecta más en caso de hipoxia es la zona:
- 3
- 2
- 1
- 4

29) Paciente con carcinoma de vesícula biliar. La metástasis por continuidad afectará al
lóbulo:
- Cuadrado
- Lobulado
- Caudado
- Izquierdo

30) El ácido acetilsalicílico actúa en la membrana:


- Basolateral de la célula parietal
- Basolateral de la célula principal
- Apical de la célula principal
- Apical de la célula parietal

31) Al realizar una esplenectomía, se tiene que resecar la arteria esplénica, lo cual no es
problema para el estómago por que la arteria gastroomental izquierda se
anastomosa con la:
- Gastroomental derecha
- Gástrica derecha
- Gastroduodenal
- Pancreaticoduodenal superior

32) La arteria esplénica proviene de la aorta y la vena esplénica desemboca en la vena:


- Porta
- Gástrica izquierda
- Mesentérica superior
- Gastroduodenal

33) Paciente mujer 21 con bulimia, que luego de un episodio de vómitos presenta
hematemesis y al examen físico que se encuentra crépitos subcutáneos cervicales.
El diagnóstico más probable es:
- Sd. Boerhaave
- Sd. Mallory - Weiss
- Varices esofágicas
- Épulis

34) Paciente varón de 60 años, con antecedentes de promiscuidad sexual, tabaquismo y


alcoholismo, acude a consulta por presentar disfagia progresiva, odinofagia y al
examen se observa tumor por parte posterior de la lengua. La mejor posibilidad
diagnóstica es:
- Carcinoma escamoso
- Esófaggo de Barret
- Mucocele
- Granuloma piógeno

35) Niño de 5 años con historia de tres días de evolución caracterizado por fiebre,
malestar general, odinofagia, anorexia, e irritabilidad. Al examen de observa lesiones
ulcerativas de 4mm de diámetro en mucosa yugal, con borde blanquecino y eritema
periférico. El diagnóstico más probable es:
- Aftas orales
- Herpes simple
- Candidiasis oral
- Leucoplasia

36) Es considerada una lesión preneoplásica


- Leucoplasia
- Granuloma piógeno
- Carcinoma escamoso
- Boerhaave

37) El esófago de Barrett se considera una lesión preneoplásica que se caracteriza por
la presencia en esófago de:
- Metaplasia intestinal
- Metaplasia gástrica
- Displasia gástrica
- Neoplasia glandular

38) Paciente mujer de 23 años gestante con lesión proliferativa en mucosa oral
producida por proliferación reactiva de vasos sanguíneos. Marque la mejor respuesta
- Epulis
- Granuloma piógeno
- Leucoplaquia
- Carcinoma epidermoide
39) La glándula parótida tiene principalmente acinos de tipo:
- Seroso
- Mucoso
- Seromucoso
- Mixto

40) La reabsorción de sodio y secreción de potasio es estimulada por:


- Aldosterona
- Angiotensina II
- HAD
- Renina

41) Paciente con cirrosis hepática que tiene hipertensión portal con várices esofágica, y
actualmente presenta varices en estómago distal. Estas várices están relacionadas a
aumento en la presión de las venas:
- Gástrica derecha
- Gástrica izquierda
- Gástrica superior
- Gástrica inferior

42) La glándula parótida está inervada por el par craneal:


- IX
- X
- XII
- VII
- V

43) Paciente con cirrosis hepática que tiene hipertensión portal con varices en esofago
distal . Estas varices están relacionadas a aumento en la presión de las venas
gástricas
- Superior
- Izquierda
- Derecha
- Inferior

44) La información eferente que sale de los núcleos salivales superior e inferior a través
de los pares VII y IX hacia las glándulas salivales llevan información tipo
- Sensitivo
- Parasimpático
- Somático
- Simpático

45) Un niño de 4 años ingresa en el hospital con vómitos graves . En el estudio se


encuentra que el niño tiene un páncreas anular ¿ Cuál de las siguientes hormonas
gastrointestinales se encontrará a niveles elevados en sangre con mayor
probabilidad a raíz de esta patología
- GIP
- Gastrina
- Secretina
- VIP
46) Durante una colecistectomía laparoscópica en un hombre de 61 años ¿ Cuál de las
siguientes arterias debe pinzar para extirpar la vesícula biliar con seguridad?
- Hepática propia
- Hepática izquierda
- Hepática derecha
- Cistica

47) Un hombre de 34 años se somete a una apendicectomía de urgencia . Después de


realizar la apendicectomía satisfactoriamente , el paciente se somete a una
laparoscopia exploratoria¿Cual de las siguientes características anatómicas es más
útil para distinguir entre yeyuno e íleon?
- El yeyuno tiene menos grasa mesentérica que el íleon
- El yeyuno tiene más arcadas vasculares que el íleon
- El yeyuno tiene más folículos linfáticos bajo la mucosa que el íleon
- El yeyuno tiene menos vellosidades que el íleon

48) Una mujer de 45 años ingresa en el hospital con síntomas de obstrucción intestinal
superior .En la TC se encuentra que la tercera porción (transversa) del duodeno está
comprimida por un gran vaso ¿Cuál de los siguientes vasos causara muy
probablemente la obstrucción ?
- Arteria mesentérica inferior
- Vena mesentérica inferior
- Arteria mesentérica superior
- Vena porta

49) Durante una colecistectomía laparoscópica programada en una mujer de 47 años , el


residente pinchó accidentalmente el ligamento hepatoduodenal en vez de la arteria
cística ¿Cual de los siguientes vasos estaría muy probablemente ocluido en esta
lesión iatrogénica?
- Arteria hepática izquierda
- Arteria hepática propia
- Arteria esplénica
- Arteria mesentérica superior

50) Un hombre de 54 años ingresa en urgencias con intenso dolor abdominal superior .
La gastroscopia revela un tumor en el antro del estómago . Se pide una TC para
evaluar el drenaje linfático del estómago¿Cuál de los siguientes nódulos linfáticos
estará muy probablemente afectado en una neoplasia maligna del estómago
- Lumbar
- Mesentérico inferior
- Mesentérico superior
- Celiaco

51) Una mujer obesa de 45 años con fiebre alta acude a la consulta con náuseas y dolor
agudo e intermitente en el cuadrante superior derecho del abdomen de 2 días de
duración - Tiene una historia de ictericia de 24 horas . Tiene antecedentes de litiasis
biliar . Bilirrubina total del 10 mg/dL . Lipasa de 5 mg/mL ¿Cuál de las siguientes
estructuras está muy probablemente obstruida por un cálculo biliar ?
- Conducto colédoco
- Ampolla de Vater
- Conducto cístico
- Conducto pancreático

52) Una mujer de 45 años ingresa en urgencias con dolor abdominal intenso . La TC y
RM revelan un tumor de la cabeza del páncreas que afecta el proceso unciforme
¿Cual de los siguientes vasos es más probable que suministre irrigación a parte de
la zona afectada?
- Arterias yeyunales
- Arteria cólica media
- Arteria cólica izquierda
- Arteria ileocólica

53) Un hombre de 70 años ingresa en urgencias con diarrea intensa . La arteriografía


revela un bloqueo del 90% en el origen aórtico de la arteria mesentérica inferior
¿Cuál de las siguientes arterias proporciona muy probablemente irrigación colateral
al colon descendente?
- Arteria gastroomental izquierda
- Arteria cólica derecha
- Arteria sigmoidea
- Arteria cólica media

54) Al disminuir el pH duodenal por el HCL gástrico , se libera principalmente una


hormona cuya célula diana es
- Célula S del intestino
- Células ductales del colédoco
- Acinos pancreáticos
- Célula ductal de Wirsung

55) Una de las siguientes sustancias reguladoras , puede actuar de forma paracrina y
como hormona
- Péptido insulinotrópico dependiente de glucosa
- GRP
- Acetilcolina
- Somatostatina

56) Al ingerir grandes cantidades de dulces , con la subsecuente estimulación de


incretinas , usted esperaría que el apetito
- Aumente por grelina
- Aumente por CCK
- Disminuya por insulina
- Disminuya por CCK

57) Al seccionar el nervio facial a nivel timpánico , usted esperaría


- Disminución del gusto en la punta de la lengua
- Ausencia de termoalgesia en la lengua
- Imposibilidad para el cierre del istmo de las fauces
- Imposibilidad para protruir la lengua

58) Paciente obeso con Covid-19 es intubado por interno inexperto, quien al solicitar que
bombeen aire dentro del tubo endotraqueal, nota que el epigastrio se distiende. Al sospechar
que ha introducido el tubo en el estómago, también es cierto que:
- Aumenta pH gástricos
- Disminuye el tono del píloro
- Aumenta la frecuencia de las ondas lentas
- Disminuye el pH gástrico

59) La hormona que tiene un efecto sinérgico con la secretina para optimizar el pH
duodenal y la digestión, es:
- CCK
- Bombesina
- Gastrina
- Péptido inhibidor gástrico
60) El ecografista sabe que para poder visualizar el nacimiento de la arteria mesentérica
superior , debe colocar el transductor sobre la piel de la siguiente región abdominal
- Hipocondrio derecho
- Mesogastrio
- Hipogastrio
- Epigastrio

61) En un paciente con hiperestimulación simpática se espera que las ondas lentas
tengan un ritmo
- Mayor en estómago que en el duodeno
- Mayor en estómago que en el íleon terminal
- Menor en íleon terminal que en el duodeno
- Mayor en el íleon que en duodeno
62) Se considera que el gusto puede viajar a través del nervio
- Glosofaríngeo
- V3
- Vago
- Lingual
63) La rotación en sentido longitudinal del estómago en el desarrollo embriológico
condiciona que el nervio vago derecho quede a nivel
- Anterior
- Oblicuo
- Posterior
- Izquierdo
64) Con respecto al control autonómico en el tracto gastrointestinal y en relación con su
fisiología. ¿Cuál es la función del sistema nervioso parasimpático en el tracto
gastrointestinal?
A) Inhiben la contracción muscular y estimulan la secreción de sustancias a nivel de la
submucosa.
B) Estimulan la contracción muscular y estimulan la secreción de sustancias a nivel de la
mucosa.
C) Inhiben la contracción muscular e inhiben la secreción de sustancias a nivel de la
submucosa.
D) Estimulan la contracción muscular e inhiben la secreción de sustancias a nivel de la
mucosa.

65) Un estudiante que está preocupado por su examen parcial, no ha desayunado ni


almorzado; cuando al fin ingiere alimentos, esto le provoca el aumento de los
movimientos musculares del tracto gastrointestinal y la sensación de defecar.¿Qué
reflejo se ha activado?
A) entero - gástrico
B) gastro-cólico
C) cólico - ileal
D) ileo - ileal

66) ¿De que par craneal es rama el nervio palatino mayor?


A) Vago
B) Hipogloso
C) Trigémino
D) Palatogloso

67) ¿En cuál de las fases de deglución la epiglotis separa la vía respiratoria de la
digestiva?
A) oral
B) laringe
C) Faríngea
D) Esofágica

68) A los pocos días de nacido, regresa a neonatología un niño con problemas de
motilidad del colon; los estudios determinan ausencia congénita de células
ganglionares. Según el gráfico ¿Cuál es la capa en la que se determina la ausencia
de dichas células?
A) Mucosa - 2
B) Muscular propia - 2
C) Muscular propia - 3
D) Muscular de la mucosa - 3

69) Los péptidos intestinales se pueden clasificar como sustancias endocrinas,


paracrinas y paracrinas, dentro de las apocrinas se encuentran la somatostatina e
histamina. Marque la respuesta correcta
A) La somatostatina es sintetizada por las células B de la mucosa gástrica
B) La histamina actúa estimulando su receptor tipo H1 en la mucosa gástrica
C) La histamina es sintetizada por células de tipo paracrino de las glándulas gástricas
D) La somatostatina presenta dentro de sus funciones la estimulación de la secreción de H+

70) Paciente varón de 27 años es llevado por bomberos a emergencia luego de ser
asaltado y, tras resistirse, es cortado con el pico de una botella a nivel abdominal. Al
examen físico usted observa que a través de la herida se puede observar la
protrusión de las asas intestinales. En relación con las capas de la pared abdominal,
marque la alternativa correcta.
A) la fascia de scarpa está constituida principalmente por tejido adiposo
B) La pared abdominal está formada por piel, huesos, músculos, fascia y peritoneo parietal
C) La fascia de camper es una estructura fibrosa que carece de grasa y su grosor es
constante en toda la pared abdominal.
D) El músculo oblicuo externo discurre en dirección súpero-interna y se inserta en el borde
inferior de las últimas 3 a 4 costillas
E) El músculo recto del abdomen tiene como funciones comprimir el contenido del abdomen,
tensar la pared abdominal y flexionar la columna

71) Los músculos del tracto gastrointestinal de los segmentos propulsivo y receptor del
bolo alimenticio, responden de forma diferente al movimiento de este bolo a través
del intestino. ¿Cuál de las siguientes afirmaciones describe correctamente la
actividad del segmento propulsivo?
A) Tanto el músculo circular como el longitudinal están relajados
B) El músculo longitudinal está relajado y el circular está contraído
C) Tanto el músculo circular como el longitudinal están contraídos
D) El músculo longitudinal está contraído y el circular está relajado

72) El nervio vago inerva al músculo


A) Liso de la faringe
B) Liso de la boca
C) Estriado del esófago
D) Estriado del esófago

73) ¿Cuál de las siguientes es una característica de los ganglios mientéricos del sistema
nervioso entérico?
A) Contiene mayor número de neuronas que el plexo submucoso
B) Es también conocido como el plexo de Meissner
C) Contiene sólo neuronas motoras excitatorias del músculo liso
D) contiene neuronas sensitivas que activan a los músculos circular y longitudinal del tracto
intestinal

74) El divertículo faringoesofágico, hipofaríngeo de zenker, es una lesión muy particular


que se localiza en la cara po
75) posterolateral de la Unión de la faringe con el esófago, como una herniación de la
mucosa esofágica a través de las fibras oblicuas del músculo.
A) Salpingofaringeo
B) Constrictor inferior de la faringe
C) Constrictor superior de la faringe
D) Constrictor medio de la faringe

76) La razón por la que el potencial de acción viaja rápidamente en sentido longitudinal
por el músculo liso gastrointestinal es la presencia de uniones en hendidura,
A) Mayor cantidad de Ach
B) la presencia del plexo submucosos de meissner
C) Las fibras musculares no se disponen en haces musculares
D) varicosidades
76)Respecto a los péptidos gastrointestinales, marque lo correcto.
A) las sustancias paracrinas pueden viajar a través de vasos sanguíneos
B) no existe sustancia neurocrina que tenga efecto en la motilidad del tubo digestivo
C) las sustancias neurocrinas son péptidos que hacen su efecto en distancias cortas
D) las sustancias paracrinas atraviesan la circulación portal

77) Al ingerir rápidamente un litro de agua, usted esperaría que la gastrina


aumenta por efecto de:
A) ACh por estimulación del nervio vago
B) efecto paracrino de la histamina
C) ACh del sistema mientérico
D) aumento de acidez gástrica (disminución del ph)

78) Respecto a la anatomia del estomago,marque lo correcto:

A) El fondo gástrico forma la curvatura mayor

Pág 19 → 21
79) Los nervios esplácnicos lumbares (L1-L2) llevan información de tipo:
A) Simpática
B) Parasimpática
C) Dolorosa
D) Sensorial
E) Piramidal

80) Paciente de 32 años con herida por arma de fuego y shock hipovolémico. El intestino
delgado no se ha infartado aún a pesar de la hipoxia gracias a la liberación de:
A) Adenosina
B) Adrenalina
C) Noradrenalina
D) Péptido intestinal vasoactivo

81) Al realizarse un piercing en el ombligo,la sensación de dolor se transmite por:


A) T11
B) T8
C) T9
D) T10

82) El nivel en el que se encuentra el píloro y el páncreas se puede determinar usando el:
A) Cuadrante mayor
B) Píloro transpilórico
C) Plano subcostal
D) Ligamento inguinal
E) Anillo inguinal

83) Permite la suspensión e irrigación de los órganos peritoneales:


A) Mesenterio
B) Ligamentos
C) Arteria mesentérica superior
D) Peritoneo parietal

84) El dolor asociado a apendicitis clásicamente se ubica en:


A) Epigastrio
B) Mesogastrio
C) Hipogastrio
D) Fosa iliaca derecha
E) Hipocondrio derecho

85) El ligamento inguinales formado por la aponeurosis del:


A) Transverso
B) Recto abdominal
C) Oblicuo externo
D) Oblicuo interno
86) Los nueve cuadrantes del abdomen se delinean usando el plano subcostal,las líneas
medioclaviculares y:
A) Plano intertubercular
B) Plano transpilórico
C) Plano interapofisiario
D) Plano intercrestal
E) Plano catastral

87) Es un órgano peritoneal:


a) Páncreas
b) Hígado
c) Recto
d) Duodeno
e) Colon ascendente

88) La línea alba se encuentra:


a) Entre los rectos abdominales
b) Entre los oblicuos externos
c) Entre los oblicuos internos
d) Entre los transversos abdominales
e) Entre Huaylas y Matellini jajajaj

89) Es un órgano retroperitoneal:


a) Páncreas
b) Colon transverso
c) Colon sigmoides
d) Duodeno
e) Vesícula biliar

90) Enfermedad asociada con un error en el desarrollo de las células de Cajal


a) Enfermedad de Hirschsprung
b) Enfermedad celiaca
c) Divertículo de meckel
d) ano imperforado
e) fístula vitelina
f)
91) El divertículo de Meckel es un rezago de:
a) Conducto vitelino
b) Alantoides
c) Asas yeyunoileales
d) Cloaca
e) Conducto anorrectal

92) Aproximadamente en la semana 6 del desarrollo embrionario; el intestino medio gira 90°
orinandose a nivel del:
a) Cordón umbilical
b) Borde inferior del bazo
c) Caudal al borde hepático derecho
d) Lado derecho del abdomen
e) Suprapúbico
93) La fístula retroperitoneal es causada por una falta en el desarrollo de:
a) Tabique urorrectal
b) Membrana cloacal
c) Proctodeo
d) Membrana anal
e) Membrana urogenital

94) Paciente mujer de 54 años con náuseas,vómitos y abdomen agudo quirúrgico, se


ingresa a sala de operaciones donde se encuentra vólvulo de ciego. Esto se debe a:
a) Falta de fusión del mesenterio
b) Falta de herniación fisiológica
c) Falla de la rotación intestinal
d) Defecto en la formación de la cloaca

ANGELA-CI2 (P95-P104)
95) Al disminuir el pH duodenal por el HCl gástrico, se libera principalmente una hormona
cuya célula diana es:
Respuestas:
Célula ductal del Wirsung
Células ductales del colédoco
Células S del intestino
Acinos pancreáticos

96) Al seccionar el nervio facial a nivel timpánico, usted esperaría:

Respuestas:
Imposibilidad para el cierre del istmo de las fauces.
Ausencia de termoalgesia en la lengua
Imposibilidad para protruir la lengua
Disminución del gusto en la punta de la lengua
97) Al introducir una solución azucarada directamente al estómago mediante una
gastrostomía (comunicación entre la piel abdominal y el estómago), la sustancia que
provocará que aumenten los niveles séricos de insulina es:

Respuestas:
Péptido tipo glucagón 1 (GLP-1)
Enteroglucagon
Glucagón
Somatostatina

98) La rotación en sentido longitudinal del estómago en el desarrollo embriológico


condiciona que el nervio vago derecho quede a nivel:
Respuestas:
Oblicuo
Posterior
Izquierdo
Anterior

99) Paciente con apendicitis aguda, que debuta con dolor en mesogastrio. El dolor se debe
a estimulación de receptores del dolor cuyas fibras van a viajar a la médula espinal a través
de:
Respuestas:
a) Nervios simpáticos
b) nervio vago
c) nervio esplácnico pélvico
d) plexo hipogástrico

100) Una de las siguientes sustancias no comparte con las otras la misma acción sobre la
producción de ácido gástrico:
Respuestas:
Péptido insulinotrópico dependiente de glucosa (GIP)
Secretina
Somatostatina
Colecistoquinina

101) En un paciente con gastroparesia (motilidad lenta del estómago), que presenta
distensión abdominal después de comer, usted le recomendaría que evite el consumo de
lípidos y aminoácidos para disminuir la acción de:
Respuestas:
Gastrina
Somatostatina
CCK
Secretina

102) En un paciente con shock distributivo, usted decide iniciar noradrenalina por un catéter
CVC, consiguiendo aumentar la presión arterial. ¿qué efecto sobre la motilidad intestinal
esperaría encontrar?

Aumenta el peristaltismo.
El potencial de reposo de las fibras musculares se hace más negativo
Aumentan las ondas lentas, pero disminuyen los potenciales en espiga
Aumenta la mezcla en el antro gástrico

103) El ecografista sabe que para poder visualizar el nacimiento de la arteria mesentérica
superior, debe colocar el transductor sobre la piel de la siguiente región abdominal:

Respuestas:
Hipogastrio
Hipocondrio derecho
Epigastrio
Mesogastrio

104) De los diferentes reflejos gastrointestinales, hay uno que produce movimiento del
contenido hacia la región distal, y se llama reflejo:
Respuestas:
Entero-gástrico
Vómito
Gastrocólico
Cólico-ileal

105) La glándula submandibular recibe inervación traída por el nervio


● Cuerda del tímpano
● Petroso Mayor
● Lingual
● Mandibular
● Glosofaríngeo

106) A mayor flujo de saliva, disminuye la concentración de:


● Potasio
● Bicarbonato
● Sodio
● Cloro

107) En términos de mg/mL, el principal componente de la saliva es _____ seguido de ____:


● Proteínas - Potasio
● Potasio - Sodio
● Sodio - Cloro
● Cloro - Urea
● Urea - Proteínas

108) La reabsorción de Sodio y Cloro en las glándulas salivales se da principalmente en el:


● Conducto estriado
● Conducto intercalado
● Conducto excretor
● Acino seroso
● Acino mucinoso
109) La información eferente que sale de los núcleos salivales superior e inferior a través de
los pares VII y IX hacia las glándulas salivales llevan información de tipo:
● Parasimpático
● Simpático
● Somático
● Sensitivo

110) La glándula submaxilar le hace gancho al:


● Músculo milohioideo
● Músculo omohioideo
● Músculo geniohioideo
● Conducto de Wharton
● Conducto de Stenon

111) Los conductos salivales son ___ al agua, esa es una de las razones por las cuales la
saliva es siempre ___.
● Impermeables - hipotónica
● Permeables - hipertónica
● Permeables - isotónica
● Impermeables - isotónica

112) Los nervios esplácnicos pélvicos (S2-S4) llevan información de tipo:


● Parasimpática
● Simpática
● Dolorosa
● Sensorial
● Piramidal

113) Presenta movimientos en masa:


● Colón
● Intestino delgado
● Estómago
● Esófago
● Cavidad oral

114) La digestión de los lípidos se inicia en:


● Intestino delgado
● Colón
● Estómago
● Esófago
● Cavidad oral

115) La digestión de los carbohidratos se inicia en:


● Cavidad oral
● Esófago
● Estómago
● Intestino delgado
● Colón
116) La digestión de las proteínas se inicia en:
● Estómago
● Esófago
● Intestino delgado
● Colón
● Cavidad oral

117) El estómago recibe información simpática proveniente del:


● Ganglio celiaco
● Ganglio mesentérico superior
● Ganglio mesentérico inferior
● Nervio vago
● Nervios esplácnicos (T9-T12)

118) En el intestino delgado se absorbe los carbohidratos en forma de:


● Fructosa
● Sacarosa
● Maltosa
● Dextrosa
● Lactosa

119) Es rama de la arteria mesentérica superior:


● A. cólica media
● A. marginal
● A. cólica izquierda
● A. pancreaticoduodenal superior
● A. gastro-omental derecha

120) El “dolor de estómago” asociado a gastritis se suele ubicar en:


● Epigastrio
● Hipogastrio
● Mesogastrio
● Hipocondrio derecho
● Fosa iliaca derecha
121) El azúcar de mesa sacarosa es digerido a dos monosacáridos que comparten el
transportador:

● SGLT2
● GLUT5
● SGLT1
● GLUT 2

122) Durante la defecación se requiere:

● Liberación de acetilcolina por el nervio vago


● Contracción de tipo segmentarias
● Activación del nervio simpático esplácnico menor
● Señales inhibitorias en el nervio pudendo
123) En un paciente con fístulas intestinales y fisuras anales, con antecedente de
enfermedad inflamatoria intestinal, ud sospecharia en:

● Colitis Ulcerativa
● Anemia perniciosa
● Enfermedad de Crohn
● colitis microscópica

124) En los pacientes con Colecistitis Aguda no operable, una opción es la colocación de
una sonda por el cístico, procedimiento en el que se ingresa con dificultad debido a la
estrechez del cístico y a la presencia de:

● Ampolla de vater
● arteria cística
● válvula de Herring
● estenosis del hepático común

125) Paciente de 51 años con antecedente de enfermedad diverticular acude a emergencia


por sangrado profuso y dolor en hipocondrio izquierdo
¿Cuál es el origen más probable de la sangre que pierde el paciente ?

● Rectal superior
● rectal inferior
● Mesentérica inferior
● cólica media

126) Paciente de 42 años con dolor abdominal intenso y hematemesis. En la endoscopia se


observa una úlcera duodenal posterior perforada con hemorragia intraabdominal. ¿Cuál de
las siguientes arterias estará comprometida?

● Mesentérica superior
● Gástrica derecha
● Pancreaticoduodenal posterosuperior

127) Para que los triglicéridos sean absorbidos deben ser metabolizados a

● Colesterol y ácidos grasos


● Monoglicéridos y Ácidos grasos
● colesterol y lisolecitina
● lisolecitina y ácidos grasos

128) El vibrio cholerae produce diarrea porque:


● Previene la absorción de glucosa y causa que el agua mantenga en la luz intestinal
● aumenta los canales secretores de bicarbonato en los enterocitos
● Aumenta la producción de AMPc en los enterocitos
● inhibe los canales secretores de cloro en las células de la cripta
129) La motilidad intestinal es estimulada por
● colecistoquinina y gastrina
● Glucagón e insulina
● somatostatina y secretina
● secretina y glucagón

130) La diarrea por deficiencia de lactasa es de tipo


● Iatrogenia
● secretora
● exudativa
● osmótica

131)La metoclopramida estimula el vaciamiento gástrico aumentando la fuerza de


contracción de las paredes gástricas esto puede conseguirlo mediante la estimulación
indirecta de las neuronas liberadoras de
● Bombesina
● Acetilcolina
● Oxido nitrico
● Péptido intestinal Vasoactivo

132)Los vértices de un acino hepático están constituidos por


● una vena centrolobulillar y dos espacios porta
● Dos espacios porta y dos venas centrolobulillares
● tres venas centrolobulillares
● seis espacios porta

133)Al ingerir un pedazo de mantequilla, cuál de los siguientes tiene un efecto directo en la
reducción del vaciado gástrico:

● Colecistoquinina
● Gastrina
● Bombesina
● Secretina

134) Tras la vagotomía (resección del vago) por enfermedad úlcera péptica en un paciente
UD. esperaría encontrar:
● Aumento de síntomas de reflujo gastroesofágico
● hipertrofia de mucosa gástrica
● mayor producción de CCK
● Aumento del pH gástrico

135) Se realizó un experimento en el cual se inyectó tinta china en el peritoneo de ratas de


laboratorio. Al realizarse una biopsia hepatica de dichos animales, se encontro que el tinte
negro fue fagocitado por:
● células de disse
● Células de kupffer
● hepatocitos
● células de ito

136) En un paciente con pH gástrico muy bajo, es posible que la siguiente sustancia se
secrete en menor cantidad:
● Péptido inhibitorio gástrico
● colecistoquinina
● secretina
● Gastrina

137)La fase cefálica de la secreción gástrica responde por cerca del 30% de la respuesta
ácida a un reflejo con la _____________ se elimina la fase cefálica de la secreción gástrica

● Vaguectomía
● Histamina
● Cimetidina
● Gastrina

138)El esofago de barret se caracteriza por presentar _________ en el esofago

● Metaplasia gástrica
● metaplasia intestinal
● displasia gástrica
● adenocarcinoma

139) La presencia de orina que sale por el ombligo de un recién nacido cada vez que llora,
es posible que deba a un defecto en el desarrollo de:
● Seno urogenital
● Cloaca
● Membrana basal
● Cuerpo perineal
● Conducto vitelino

140) El conducto biliar deriva del:


● Endodermo
● Mesodermo
● Ectodermo
● Mesotelio
● Peritoneo

141) Cuál de los siguientes órganos son intraperitoneales:


● Estómago, Vesícula biliar, Y León, Hígado
● Páncreas, Colon descendente, Hígado, Vesícula biliar
● Recto, Hígado, Colon transverso, Yeyuno
● Estómago, Yeyuno, Duodeno, Páncreas
● Recto, Vesícula biliar, Y León

142) Los vasos mesentéricos superiores se hallan a nivel de:


● Cardias
● Cuello del páncreas
● Hilio hepatico
● Tronco celiaco
● Ileon

143) La colecistoquinina (CCK) inhibe:


● El vaciamiento gástrico
● La secreción pancreática de HCO3-
● La concentración de la vesícula biliar
● La relajación del esfínter de Oddi
● La secreción de amilasa

144) Un niño de 2 años es llevado a la consulta por diarrea persistente, edema de las
extremidades y falta de crecimiento en relación a su edad. Los análisis de sangre revelan
que tiene concentración plasmática baja de proteínas (hipoproteinemia). Como parte del
estudio se coloca Colecistoquinina (CCK) endovenosa y se recoge muestras del líquido
duodenal por endoscopia; el resultado del líquido confirma incapacidad para hidrolizar
proteínas a un pH neutro, esta situación mejora al añadir una pequeña cantidad de tripsina.
El paciente probablemente esté sufriendo la falta congénita de ……….
● PEPT-1
● Pepsinógeno
● Enterocinasa
● Carboxipeptidasa.

145) Paciente mujer de 35 años acude a consulta por sensación de sequedad y lesiones en
la cavidad oral. Al examen se observa atrofia de la mucosa, fisura y úlceras; nota además
sequedad e irritación de la córnea y aumento del tamaño de las glándulas parótidas. Su
diagnostico mas probable es artritis reumatoide; el hallazgo más probable en una biopsia de
glándula parótida es ….…
● Presencia de acinos anormales con hiperplasia de células ductales
● Gran infiltración de linfocitos y células plasmáticas
● Hiperplasia de acinos glandulares serosos
● Gran infiltrado de linfocitos y neutrófilos

146) Un hombre de 42 años de edad se presenta al médico con una historia clínica de 1 año
de evolución, caracterizado por dolor abdominal bajo y diarreas con crisis sanguinolentas.
Manifiesta además pérdida de peso de 8kg durante este periodo. La colonoscopia revela
lesión difusa en el colon con afectación del recto. La biopsia de estas lesiones revela
adelgazamiento de la pared, inflamación y ulceración de la mucosa y submucosa. El
diagnóstico más probable en este caso es:
● Síndrome de colon irritable
● Enfermedad de Crohn
● Colitis ulcerativa
● Sprue celiaco

Dos estudiantes deciden tomar un receso para comer una hamburguesa a la hora del
almuerzo. Antes de llegar a la cafetería, impulsos nerviosos provenientes del complejo vagal
dorsal iniciarán la secreción de ácido gástrico por la liberación de ___________ desde el
sistema nervioso entérico.
● Serotonina
● Colecistoquinina (CCK)
● Péptido inhibidor vasoactivo
● GRP (péptido liberador de gastrina)

147) Un niño de cuatro años de edad es llevado a la consulta por cuadros diarreicos
frecuentes caracterizados por heces pálidas, voluminosas y fétidas; al examen físico
presenta bajo peso y talla para la edad. Se mide la concentración de cloruro de en el sudor y
se encuentra que sus valores son muy elevados. La alteración más importante a nivel de
células ductales del páncreas tiene relación directa con la conductancia de …..
● Bicarbonato
● Potasio
● Sodio
● Cloro

148) Se evalúa los valores séricos de las siguientes sustancias a un paciente con
enfermedad hepática terminal; en este paciente se espera encontrar la combinación de la
letra …..

Glucosa Amoniaco Albúmina

a. Aumentada Disminuida Disminuida

b. Disminuida Aumentada Aumentada

c. Aumentada Aumentada Aumentada

d. Disminuida Aumentada Disminuida

149) Una mujer de 35 años de edad HIV positiva, se presenta al médico con dolor
abdominal en cuadrante superior derecho e ictericia. La paciente refiere haber tenido
múltiples episodios de ictericia durante los últimos 10 años. Los exámenes para determinar
hepatitis viral, dieron positivos para Hepatitis B, siendo catalogado el caso como hepatitis
crónica con alteración funcional. En un examen de sangre ¿Cuál de los siguientes
parámetros está disminuido?
● Albúmina
● Bilirrubina
● Fosfatasa alcalina
● Tiempo de protrombina

150) En el reflejo peristáltico del intestino delgado ¿Cuál de los siguientes eventos sucede
en la porción caudal del bolo alimenticio?
● Acción del péptido inhibidor vasoactivo (VIP) en el músculo circular
● Acción del NO (Óxido nítrico) en el músculo longitudinal
● Contraccion del musculo longitudinal interno
● Acción de la acetilcolina en el músculo circular

151) Un varón de 58 años de edad con enfermedad de Crohn severo fue sometido a una
resección ileal. Después de la cirugía esta paciente padecerá de esteatorrea esto se explica
porque ……
● Se inhibe la acción de la 7 alfa hidroxilasa
● El pool de ácidos biliares se incrementa
● Hay malabsorción de ácidos biliares
● El páncreas no secreta lipasa

152) En un experimento se inserta un balón en el estómago de un voluntario, se infla poco a


poco mientras que se vigilan las presiones intraluminales. Aunque el volumen del balón
aumenta considerablemente, las presiones permanecen constantes. Esta relación volumen-
presión se explica por la liberación local de ….
● Acetilcolina y gastrina
● Norepinefrina y oxido nitrico
● Colecistoquinina y oxido nitrico
● Óxido nítrico y péptido inhibidor vasoactivo

153) ¿Cuál de las siguientes alternativas es una característica de la secreción exocrina del
páncreas?
● Es hipotónica respecto al plasma
● Su mayor estímulo se da en la fase intestinal
● Es estimulada por la presencia de bicarbonato en el duodeno
● La secreción enzimática es estimulada principalmente por la secretina

154) Las estructuras en el hígado que permite que los productos metabólicos unidos a
proteínas tengan acceso a las membranas basolaterales de los hepatocitos, son ….
● Los canalículos
● Las células de Ito
● Las fenestras sinusoidales
● Las uniones intercelulares herméticas

155) La composición de la bilis es modificada conforme fluye por los conductillos biliares.
Durante este tránsito se espera que aumente la concentración de …..
● Ig A
● Glucosa
● Protones
● Vitamina A

156) Se mide experimentalmente el contenido gástrico de dos personas. La persona “A”


tiene alto contenido de grasa y la persona “B” tiene un contenido isotónico. ¿Cual de las
siguientes es correcta respecto al vaciamiento gástrico?
● Hay ralentización del vaciado gástrico sólo en “A”
● El vaciamiento gástrico es más rápido en ambos
● Hay ralentización del vaciado gástrico sólo en “B”
● Hay ralentización del vaciado gástrico en ambos casos
157) El examen endoscópico de un paciente con hipertensión portal grave revela venas
tortuosas que sobresalen hacia la luz del esofago. El paciente recibe tratamiento quirúrgico
mediante la colocación de una derivación que conecta la vena cava. Después de la
operación el riesgo de encefalopatía …….. y el sangrado de varices ……
● Disminuirá/Disminuirá
● Disminuirá/Aumentará
● Aumentará/Disminuirá
● Aumentará/Aumentará

158) Un bolo alimenticio grande y poco masticado se atasca en el esofago, esto ocasiona
una sensacion de dolor que es transmitida por los nervios:
● Vago
● Esplácnico
● de Latarjet
● Recurrente laríngeo

159) El peristaltismo del intestino delgado se puede intensificar debido a:


● Somatostatina
● Noradrenalina
● Secretina
● Irritación de la mucosa

160) Un paciente es diagnosticado con un tumor neuroendocrino productor de


somatostatina, esto provocará en el sistema digestivo:
● Vómitos
● Estreñimiento
● Odinofagia
● Diarrea

161) Los diferentes segmentos del tubo digestivo son susceptibles de reflejos y movimientos
según su contenido.Si colocoramos mediante una sonda un bolo alimenticio directamente en
el tercio medio del esofago:
● Se producirá la acción voluntaria del músculo estriado
● Será necesario un reflejo vago-vagal
● Se producirá ondas secundarias
● Se producirá ondas primarias

162) En el digestivo la liberación hormonal se presenta ante diversos factores o estímulos.


La hormona _________ es estimulada por la presencia de alimentos en el bulbo duodenal a
predominio de ácidos grasos y triglicéridos, por estimulación vagal y por la hormona
secretina.
● Motilina
● Colecistoquinina (CCK)
● Lipasa pancreatica
● Gastrina

163) El divertículo de Meckel es una anomalía congénita que ocurre por la persistencia del
conducto vitelino y da origen a una estructura sacular, el cual se encuentra en el:
● Duodeno
● Apéndice cecal
● Borde mesentérico
● Borde antimesentérico

164) La fase oclusal de la masticación se realiza con la contracción de los músculos


inervados por el nervio craneal:
● V
● VIII
● IX
● XII
165) En un paciente de 43 años con tumor carcinoide de páncreas productor de gastrina
(Sindrome de Zollinger-Ellison) se puede encontrar una potenciación del reflejo:
● Ileocolico
● Gastrocolico
● De relajación receptiva
● Del vómito

166) Durante la fase faríngea de la deglución ocurre el siguiente mecanismo:


● Los músculos infrahioideos hacen que se desplaza la laringe hacia abajo
● El paladar blando abre el istmo de las fauces
● La onda peristáltica lleva el alimento hacia el esofago
● El péptido inhibidor vasoactivo (VIP) actúa sobre el músculo constrictor superior

167) En los carcinomas (neoplasia benigna) es frecuente que ocurran la metástasis a través
de los vasos venosos. En el caso de un carcinoma del tercio superior del esofago, ubicado
en la cara lateral izquierda, es más probable que la metástasis viaje por la vena:
● Subcostal derecha
● Cava Inferior
● Hemiácigos accesoria
● Hemiácigos

168) Los movimientos en masa son iniciados por el reflejo:


● Del vómito
● Duodenocolico
● Colicoileal
● Duodenogástrico

169) Se evalúa los valores séricos de las siguientes sustancias de un paciente con
enfermedad hepática terminal, en este paciente se espera la combinación con la letra

Glucosa Amoníaco Albúmina

a aumentada disminuida disminuida

b disminuida aumentada aumentada

c aumentada aumentada aumentada

d disminuida aumentada disminuida


170) Una mujer de 65 años VIH positiva se presenta con dolor abdominal en el cuadrante
positivos para Hepatitis B, siendo catalogada como hepatitis crónica con alteración
funcional. En un examen de sangre ¿cuál de los siguientes parámetros está disminuido?
● Albúmina
● Bilirrubina
● Fosfatasa alcalina
● Tiempo de protrombina

171) En el reflejo peristáltico del intestino delgado¿cuál de los siguientes eventos sucede en
la porción caudal del bolo alimenticio?
● Acción del péptido inhibidor vasoactivo(VIP) en el músculo circular
● Acción del NO(óxido nítrico) en el músculo longitudinal
● Contracción del músculo longitudinal interno
● Acción de la acetilcolina en el músculo circular

172) Un varón de 58 años de edad con enfermedad de Crohn Severo fue sometido a una
resección ilegal. Después de la cirugía este paciente padecerá de esteatorrea, esto se
explica porque ….
● Se inhibe la acción de la 7 alfa hidroxilasa
● El Pool de ácidos biliares se incrementa
● Hay mala absorción de los ácidos biliares
● El páncreas no secreta lipasa

173) En un experimento se inserta un balón en el estómago de un voluntario, se infla poco a


poco mientras se vigilan las presiones intraluminales. Aunque el volumen del balón aumenta
considerablemente, las presiones permanecen constantes. Esta relación volumen presión se
explica por la liberación local de ….
● Acetilcolina y gastrina
● Norepinefrina y ácido nítrico
● Colecistoquinina y óxido nítrico
● Óxido nítrico y péptido inhibidor vasoactivo

174) ¿Cuál de las siguientes alternativas es una característica de la secreción exocrina del
páncreas?
● Es hipotónica respecto al plasma
● Su mayor estímulo se da en la fase intestinal
● Es estimulada por la presencia de bicarbonato en el duodeno
● La secreción enzima toca es estimulada principalmente por la secretina
175)Las estructuras en el hígado que permiten que los productos metabólicos unidos a
proteínas tengan acceso a membranas basolaterales de los hepatocitos son ….
● Los canalículos
● Las células de Ito
● Las de fenestras sinusoidales
● Las uniones intercelulares herméticas

176)La composición de la bilis es modificada conforme fluye por los conductillos biliares.
Durante este tránsito se espera que aumente la concentración de ….
● IgA
● Glucosa
● Protones
● Vitamina A

177)Se mide experimentalmente el contenido gástrico de dos personas. La persona A tiene


alto contenido de grasas y la persona B tiene un contenido isotónico¿ Cuál de las siguientes
es correcta respecto al vaciamiento gástrico?
● Hay ralentización del vaciado gástrico sólo en A
● El vaciamiento gástrico es más rápido en ambos
● Hay ralentización del vaciado gástrico sólo en B
● Hay ralentización del vaciado gástrico en ambos

178)El examen endoscópico de un paciente con hipertensión portal grave revela venas
tortuosas que sobresalen hacia la luz del estómago. El paciente recibe tratamiento
quirúrgico mediante la colocación de una derivación que conecta la vena porta a la vena
cava. Después de la operación el riesgo de encefalopatía …. y el riesgo de sangrado de
varices ….
● Disminuirá/ disminuirá
● Disminuirá / aumentará
● Aumentará / disminuirá
● Aumentará / aumentará

179) Un paciente varón de 18 años de edad acude al médico para sus exámenes de rutina.
Sus resultados de laboratorio muestran un valor de bilirrubina sérica de 4 mg/dl y una
bilirrubina directa de 0.3 mg/dl. Las pruebas de función hepática son normales. La alteración
que explica mejor este caso es por la deficiencia de….
● Transaminasas
● Hemo Oxigenasa
● La 7 alfa hidroxilasa
● Glucuronil transferasa

180) Un hombre de 57 años de edad es llevado a urgencias con hematemesis masiva roja
brillante, a su llegada se halla inconsciente con PA: 80/40 mm Hg y FC:124 lat/min. Luce
ictérico con presencia de arañas vasculares en el tórax anterior y extremidades, abdomen
distendido con signo de oleada positiva. Se encuentra esplenomegalia y pérdida de la masa
muscular en extremidades. La anastomosis vascular responsable del sangrado en este
paciente es ….
● Vena gastrica izquierda y vena acigos
● Arteria gástrica izquierda y arteria ácigos
● Vena paraumbilical y vena epigástrica inferior
● Vena gástrica izquierda y vena esofágica superior

190) Un estudiante de medicina está comiendo un plato de comida a base de champiñones,


espárragos y salsa de soya. El estímulo del sabor umami contenido en todos estos
alimentos viaja a través del nervio….
● Lingual
● Hipogloso
● Glosofaríngeo
● Cuerda del tímpano

191) Una paciente de 30 años de edad es sometida a una cirugía de oído medio derecho
por un problema de otoesclerosis. Luego de la cirugía refiere alteración sensitiva de la
lengua. Al evaluar el caso usted esperaría encontrar ….
● Alteración en la sensación del dolor y temperatura en el tercio posterior de la lengua
● Alteración de la sensación del dolor en los dos tercios anteriores de la lengua
● Alteración en la sensación del gusto en el tercio posterior de la lengua
● Sensación del dolor, tacto y temperatura conservadas

192) En una paciente de 45 años de edad con colestasis biliar, se encuentra una elevación
de los niveles sanguíneos de fosfatasa alcalina hasta 3 veces la cifra normal ¿Cuál de las
siguientes alternativas estará también elevada como evidencia del daño biliar?
● Tiempo de protrombina y albúmina sérica
● Transaminasas hepáticas(ALT y AST)
● Gamma glutamyl transpeptidase
● Glucoronil transferasa

193) Experimentalmente se incrementa la velocidad de la secreción salival con una


sustancia, en el análisis de la composición de esta saliva obtenida se espera encontrar….
● Disminución de la concentración de bicarbonato que supera la concentración
plasmática
● Aumento en la concentración de cloro y sodio que supera la concentración
plasmática
● Aumento de la concentración de bicarbonato que supera la concentración plasmática
● Disminución de concentración de potasio y bicarbonato

194) Lactante de 3 meses de vida es atendido por presentar diarrea, se administra una
solución glucosa y electrolitos por vía oral. La proteína de membrana apical que explica la
capacidad de esta solución para proporcionar aporte de glucosa e hidratación es ….
● CFTR
● SGLT 1
● GLUT 2
● GLUT 5
195) Paciente ha sufrido herida de bala en el abdomen, se le tenido que extirpar el
segmento medio y distal del íleon. En este caso la síntesis hepática de sales biliares estará
….
● Sin cambios en el ritmo de la síntesis
● Disminuida por inhibición de la enzima colesterol 7 alfa hidroxilasa
● Incrementada por estímulo de la enzima colesterol 7 alfa hidroxilasa
● Incrementada por inhibición de la enzima colesterol 7 alfa hidroxilasa

196) Un varón de 75 años ingresa al consultorio por presentar ictericia marcada de piel y
escleras. El estudio del paciente mostró que presentaba un tumor que obstruía la totalidad
del conducto hepático común. ¿Cual de los siguientes conductos se encontraría dilatado en
este paciente?
● De Wirsung
● De Hering
● Colédoco
● Cístico

197) Correlacione las dos columnas y marque la respuesta correcta:


● Enfermedad Hirschsprung Aganglionosis congénita
● Diarrea osmótica. Intolerancia a la lactosa
● Diarrea secretora. Canales de Cl- en las células de la cripta
● Diarrea exudativa Heces con moco y sangre
198) La fase cefálica de la secreción gástrica responde por cerca del 39% de la respuesta
ácida a un reflejo. Con la ____ se elimina la fase cefálica de la secreción gástrica
Vaguectomía
199) El esofago de Barret se caracteriza por presentar___ en el esofago
Metaplasia intestinal

198) En la regulación del apetito y la saciedad, la estimulación experimental crónica del


núcleo ventromedial del hipotálamo producirá:
● Afagia
● Obesidad
● Hiperfagia
● Activación de neuronas relacionadas a NPY

199) Paciente mujer de 25 años acude por dolor en fosa iliaca derecha que empeora al toser
o caminar,asociada a náuseas y vómitos por lo cual acude a emergencia.Dos días después
de realizarle una apendicectomía, la paciente desarrolla fiebre alta (39° C), esta hipotensa y
presenta dolor abdominal.La laparotomia exploratoria muestra un gran volumen de sangre
en la cavidad peritoneal por lesión de un vaso producida durante la apendicectomía.¿Cual
de las siguientes arterias debe ligarse para detener la hemorragia?
● Ileocólica
● cólica derecha y arteria rectal superior
● mesentérica superior
● Ileocólica y arteria cólica Media

200) ¿Cuál de los siguientes es una causa de ictericia con bilirrubina conjugada
aumentada?
● Ictericia del recién nacido
● Obstrucción del colédoco
● Anemia hemolítica
● Gran hematoma

201. Dos días después de una apendicectomía en un hombre de 45 años, ha desarrollado


fiebre alta (39), está hipotenso y presenta dolor abdominal. La laparotomía exploratoria
muestra un gran volumen de sangre en la cavidad peritoneal por lesión de un vaso
producida durante la apendicectomía. ¿Cuál de los siguientes vasos debe ligarse para
detener la hemorragia?

a. Arteria cólica derecha

b. Arteria ileocólica y arteria cólica media

c. Arteria mesentérica superior

d. Arteria ileocólica

202. Paciente de 78 años, con diabetes mellitus tipo ll y fumador, que acude a consulta
porque desde hace dos semanas tiene un dolor intenso en flanco derecho y mesogastrio ,
intenso que aparece a los 30 minutos de haber comido, y desaparece dos a tres horas
después. En estos pacientes, es muy probable que la circulación deficitaria sea
parcialmente asumida por la:

a. Arteria ileocólica

b. Arteria cólica izquierda

c. Arteria cólica media

d. Arteria yeyunales

203. Un hombre de 70 años ingresa en urgencias con diarrea intensa. La arteriografía revela
un bloqueo del 90% en el origen aórtico de la arteria mesentérica inferior. ¿Cuál de las
siguientes arterias proporciona muy probablemente irrigación colateral al colon
descendente?

a. Arteria gastro omental izquierda

b. Arteria cólica derecha

c. Arteria cólica media

204. Un niño de 4 años ingresa en el hospital con vómitos graves. La exploración radiológica
y la historias clínicas revelan que el niño tiene páncreas anular. ¿Cuál de las siguientes
estructuras es la que se encontrará dilatada con mayor probabilidad a raíz de esta
patología?

a. Tercera porción del duodeno


b. Segunda porción del duodeno

c. Píloro gástrico

d. Primera porción del duodeno

205. Un hombre de 55 años ingresó al hospital con dolor abdominal intenso. La gastroscopia
y la TC revelaron una úlcera perforada en la pared posterior del estómago. ¿Dónde se
desarrolla inicialmente con más probabilidad una peritonitis?

a. Espacio hepatorrenal (de Morison)

b. Bolsa omental (saco menor)

c. Espacio subhepático derecho

d. Espacio subfrénico derecho

206. Un hombre de 44 años ingresa en urgencias con vómitos abundantes y deshidratación.


Las imágenes radiológicas demuestran que parte del intestino está comprimido entre la
aorta abdominal y la arteria mesentérica superior. ¿Cuál de las siguientes estructuras
intestinales estará muy probablemente comprimida?

a. Segunda porción del duodeno

b. Colon transverso

c. Tercera porción del duodeno

d. Primera porción del duodeno

207. ¿Cuál de los siguientes nódulos linfáticos estará muy probablemente afectado en una
neoplasia maligna del recto?

a. Celiaco

b. Mesentérica inferior

c. mesentérica superior

d. rectal

208. Una mujer de 23 años ingresa con dolor abdominal, náuseas y vómitos. La historia
clínica muestra que el dolor es agudo y ha sido constante durante 4 años. El dolor empezó
en el epigastrio e irradiado bilateralmente alrededor del tórax hasta justo debajo de las
cápsulas. Actualmente el dolor se localiza en el hipocondrio derecho. L a TC revela cálculos
calcificados en la vesícula biliar. ¿Cuál de los siguientes nervios llevan las fibras aferentes
del dolor referido?

a. Nervios esplácnicos torácicos mayores


b. Nervio frénico

c. Nervio vago

209. Un hombre de 55 años ingresa en urgencias por severa pérdida de peso en los 6
meses previos. El examen radiológico pruebas aportan signos de un tumor que causa
hipertensión portal. Los estudios de laboratorio revelan que las deposiciones son grasas,
tiene desnutrición e hipoxia hepática. ¿Cuál de las siguientes localizaciones se encuentra
muy probablemente afectada?

a. Segmento l

b. Segmento ll

c. Segmento ll

d. Segmento IV

210. El triángulo de calot es importante reconocerlo porque sirve como reparo para
encontrar la arteria cística, y está formado por el conducto cístico, el conducto hepático
derecho y :

a. Borde hepático

b. Arteria vesical

c. Arteria hepática

211. El kernicterus se produce en recién nacidos con valores mayores de 25 m/dL en la


bilirrubina:

a. Indirecta

b. Directa

c. Delta

212. Paciente con cirrosis hepática y presión de vena cava de 15 mmhg. Lo más probable
es que el paciente presente:

a. Ascitis

b. Cabeza de medusa

c. Arañas vasculares

213. Considerando un flujo plasmático renal de 180 ml/min, y una fracción de filtración del
20%, si la concentración en sangre de la bilirrubina indirecta es de 0.6mg/dL, entonces es
correcto esperar que la carga filtrada de la bilirrubina indirecta es:

a. Menor de 1080 mg/dL

b. 2050
c. Mayor a 1080}

214. El volumen diario de bilis secretada al intestino es:

a. 500-1000 ml

b. 50-100

c. 5-10

215. La excreción del amonio se da principalmente por:

a. Orina

b. Heces

c. Pulmones

d. Piel

216. Si en una persona normal, el flujo de la arteria hepática es de 700 ml/min, entonces el
flujo de la vena porta debería ser aproximadamente:

a. 2800 mL/min

b. 3500

c. 1300

217. Paciente con Crigler-Najjar debida a mutación del gen UGT1A1, se presenta a consulta
por ictericia, usted asume que si le hiciera un análisis de sangre encontraría valores
elevados de:

a. Bilirrubina indirecta

b. Directa

c. Conjugada

218. El amoniaco corporal se forma principalmente en:

a. Colón

b. Riñón

c. Músculo

219. La zona del lobulillo hepático que se afectaría más en una intoxicación con droga
hepatotóxica es la zona:

a. 1

b. 2
c. 3

d. 4

220. El acino hepático tiene en sus aristas:

a. Vena centrolobulillar

b. Solo espacios porta.

c. Espacios porta y sinusoides

221. Paciente con esteatosis hepática no alcohólica, que en la biopsia se observa


degeneración grasa de los hepatocitos, lo cual se debe a depósitos de lípidos que
principalmente contienen:

a. Triglicéridos

b. Colesterol

c. Ácidos grasos libres superior derecho e ictericia. La paciente afirma haber tenido
múltiples episodios de ictericia durante los últimos 10 años. Los exámenes para poder
detectar hepatitis viral, dieron p

PARCIAL

222. Al evaluar la orofaringe de un paciente, el médico le solicita que abra la boca, saque la
lengua y diga a . Al hacer esta maniobra, nota que el paladar se desvía hacia la derecha, lo
cual le hace sospechar que el paciente sufre de una lesión del nervio craneal:
- X contralateral

223. Un bolo alimenticio grande y poco masticado se atasca en el esófago, esto ocasiona
una sensación de dolor que es transmitida por los nervios:
- esplácnicos

224. Para realizar el movimiento mecánico de abrir la boca, primero se necesita:


- fijar el hueso hioides

225. ¿Cuál de las siguientes alternativas se define como la protrusión directa del contenido
abdominal a la cavidad amniótica por un defecto de la pared corporal?
- Gastrosquisis

226. Un paciente requiere que se le coloque una sonda de alimentación directamente al


estómago (gastrostomía), el cirujano deberá hacer una incisión en la piel del abdomen ¿cuál
de las siguientes raíces nerviosas debe ser anestesiada para este procedimiento?
- T8
227. En un paciente de 43 años con tumor carcinoide de páncreas productor de gastrina
(Síndrome de Zollinger-Ellison) se puede esperar encontrar una potenciación del reflejo:
- gastrocólico

228. El mecanismo de la defecación incluye la participación de diversas estructuras ¿Cuál


de las siguientes alternativas es correcta?
- Puede ser mediado por un reflejo intrínseco

229. Cuando el contenido del estómago ingresa al duodeno, uno de los reflejos que inhiben
el vaciamiento gástrico es a través del:
- sistema nervioso mientérico

230. Durante la masticación, gran parte del proceso masticatorio se debe a:


- el reflejo masticatorio

231. Las glándulas salivales tienen conductos para la excreción de la saliva; las glándulas
____________ drenan en las carúnculas sublinguales.
- sublinguales

232. Los diferentes segmentos del tubo digestivo son susceptibles de reflejos y movimientos
según su contenido. Si colocamos mediante una sonda un bolo alimenticio directamente en
el tercio medio del esófago:
- se producirá ondas secundarias

233. En una persona si enfermedad se espera que el tránsito intestinal se vea disminuido
cuando se presenta el reflejo:
- doloroso

234. El divertículo de Meckel es una anomalía congénita que ocurre por la persistencia del
conducto vitelino y da origen a una estructura sacular, el cual se encuentra en el:
- borde antimesentérico

235. Si al intubar a un paciente, por error se ingresa el tubo endotraqueal en el esófago y se


insufla el manguito endotraqueal (globo TET), la dilatación de este manguito generará:
- múltiples ondas secundarias

236. El orificio omental, o hiato de Winslow, se encuentra limitado por el ligamento:


- hepatoduodenal

237. Paciente de 24 años acude a consulta externa por presentar una fístula oronasal
(comunicación entre la cavidad oral y la cavidad nasal). Está fístula es una consecuencia
tardía de la lesión de un vaso sanguíneo por el antecedente de haber sido operado de
paladar hendido en los primeros años de vida, aparentemente en una campaña gratuita de
corrección de paladar fisurado. ¿Cuál de las arterias palatinas podría haberse lesionado
durante esa cirugía?
- Mayor
238. Dentro de las anomalías congénitas se puede presentar un tejido pancreático accesorio
¿Cuál es la ubicación más común de este tejido?
- Estómago

239. Paciente con insuficiencia mitral moderada a severa, con aumento de volumen de la
aurícula izquierda; esta condición tendrá como consecuencia a nivel del sistema digestivo:
- la disfagia a sólidos

240. El inicio de la fase faríngea de la deglución se debe a estímulos sensitivos que viajan
por el nervio craneal:
- V (nervio maxilar)

241. El mesocolon transverso se origina en:


- la pared posterior del abdomen

242. Un paciente varón de 68 años consulta por dolor y abultamiento en la región inguinal
derecha, que aparece tras la realización de un esfuerzo físico. En la exploración en
bipedestación presencia de una tumoración blanda, depresible, que aumenta con la tos. El
presente caso describe una hernia:

- Inguinal directa.

243. la Grelina es sintetizada por………….. y activa las neuronas relacionadas con


……………………….en el núcleo arqueado del hipotálamo}
- El estómago/AGRP-NPY

244. Cuál de los siguientes órganos son intraperitoneales:


- Estómago, Vesícula biliar, Y León, Hígado

245. La distensión gástrica por los alimentos produce incremento de secreción de HCl
mediante el siguiente mecanismo:
- Producción de Gastrina que desencadena su cascada de señalización en la célula
parietal vía proteína Gq

246. La inervación de la piel del abdomen debajo del ombligo hasta la región púbica está
dada por:
- T11, T12, L1

247. Paciente de 38 años que tras riña durante partido de fútbol sufre un traumatismo con
arma blanca en cuadrante inferior izquierdo del abdomen. En la tomografía de urgencias se
evidencia gran hematoma de pared y ausencia de neumoperitoneo. Desde la piel hacia al
peritoneo, en ¿qué orden se atravesó la pared abdominal?

- TCSC, oblicuo externo, oblicuo interno, músculo transverso

248. Alrededor de 90% de los pacientes afectados por el síndrome de Zollinger-Ellison


desarrollan úlcera péptica. La causa es:
- Hiper Secreción ectópica de gastrina
249. Un hombre de 22 años sufre traumatismo en el flanco izquierdo superior del abdomen
al tratar de defenderse del robo de su vehículo. Un amigo lo trasladó a un centro
hospitalario. El paciente presenta signos de hipovolemia con taquicardia e hipotensión. Se
queja de dolor en el sitio de lesión que se irradia al hombro izquierdo. El órgano
probablemente lesionado es
- Bazo

250. Aproximadamente en la semana 6 del desarrollo embrionario, el intestino medio gira


90° orinandose a nivel del:
- Cordón umbilical

251. ¿Cuál de las siguientes sustancias es liberada por neuronas en el tracto GI, Participa
en la regulación hidroelectrolítica y produce relajación del músculo liso?
a) Gastrina
b) Secretina
c) CCK
d) VIP

252. La colecistoquinina inhibe:


a) El Vaciamiento gástrico
b) La Secreción pancreática de HCO3-
c) La Contracción de la vesicular biliar
d) La Relajación del esfínter de oddi

253. ¿La secreción de cuál de las siguientes es inhibida por un pH bajo?


a) Secretina
b) Gastrina
c) CCK
d) VIP

254. El conducto biliar deriva del:


a) Ectodermo.
b) Endodermo.
c) Mesodermo.
d) Mesotelio.

255. Respecto a la imagen mostrada la estructura señalada con el número 1


corresponde a:
a) Glándula parótida
b) Glándula sublingual
c) Glándula submaxilar
d) Glándula lingual
256. Los vasos mesentéricos superiores se hallan a nivel de:
a) El fondo gástrico
b) El cuello del páncreas
c) La cabeza del páncreas
d) El hilio hepático

257. La imagen mostrada representa un defecto de ͙ ͙ ͙ ͙ ͙ ͙………… y recibe el nombre


de: …………
a) La pared abdominal/onfalocele
b) La pared abdominal/divertículo de Meckel
c) Malrotación intestinal/gastrosquisis
d) La pared abdominal/gastrosquisis

PARCIAL

1) Paciente de sexo masculino de 82 años de edad ingresa a emergencia con dolor abdominal
agudo y diarreas. Se le realiza una arteriografía en la que se observa que la arteria aorta tiene
un trombo ocluyendo el 95% del flujo a nivel del nacimiento de la arteria mesentérica inferior.
¿Cuál de las siguientes arterias podría contribuir a la irrigación colateral del colon
descendente?

a) Cólica media
b) Sigmoidea
c) Rectal superior
d) Ileocólica
2) Niño de 5 años presenta dolor esofágico y hematemesis (vómitos hemorrágicos) luego de
tragarse una espina de pescado. En la endoscopía se observa perforación del esófago distal
a la cuarta estrechez esofágica. ¿Las ramas de cuál de las siguientes arterias estarán
lesionada con mayor probabilidad?
a) Gástrica izquierda
b) Bronquiales
c) Frénica inferior
d) Tiroidea inferior

3) Al ingerir una cucharada de mantequilla es muy probable que se disminuya la sensación


de hambre por medio de la activación de la vía POMC/CART (POMC=proopiomelanocortina
y CART=transcripción regulada de cocaína y anfetamina), activada directamente por la
hormona:
a) colecistoquinina (CCK)
b) insulina
c) grelina
d) secretina

4) Al comer unas papitas fritas con mayonesa, el vaciamiento gástrico disminuye por efecto
directo de la hormona:
a) colecistoquinina (CCK)
b) bombesina
c) motilina
d) gastrina

5) Recién nacido de dos horas es diagnosticado de hernia umbilical de 1,5 cm de diámetro;


el cirujano pediatra solicita una tomografía abdominal en donde se evidencia que la hernia
umbilical está ocupada por una porción del tracto gastrointestinal. ¿Qué porción del tracto
gastrointestinal estaría ocupando esta hernia con mayor probabilidad?
a) Íleon
b) Colon sigmoides
c) Duodeno
d) Colon transverso

6) Recién nacido de 7 horas, de parto por cesárea debido a polihidramnios (aumento del
volumen del líquido amniótico), con regurgitación de la leche materna y artificial, y no ha
presentado meconio. Se le realiza una tomografía donde se evidencia aire en el estómago y
una malformación del desarrollo esofágico. Con respecto a esta malformación lo más
probable es que se pueda tratar de una atresia esofágica:
a) proximal con fístula traqueoesofágica distal
b) distal con fístula traqueoesofágica proximal
c) proximal y distal
d) sin fístula

7) Lactante de 6 meses de edad que es traído a consulta por presentar vómitos no biliosos a
repetición y retraso en el crecimiento. En la radiografía de abdomen simple se observa nivel
hidroaéreo en estómago y en primera porción de duodeno (doble burbuja). ¿Cuál de las
siguientes alternativas puede explicar la condición del lactante?
a) Páncreas anular
b) Atresia duodenal en la tercera porción
c) Atresia yeyunal
d) Hipertrofia del píloro

8) En ausencia o deficiencia de la secreción de la hormona motilina, se producirá:


a) sobrecrecimiento bacteriano
b) diarrea
c) aumento del vaciamiento gástrico
d) hipertrofia del píloro

9) La estimulación parasimpática aumenta la motilidad intestinal, mientras que la


estimulación simpática la disminuye. ¿Sobre cuál de las siguientes alternativas el sistema
nervioso autónomo actúa para el control de la motilidad intestinal?

a) Potencial de membrana en el plexo mientérico (de Auerbach)


b) Frecuencia de ondas lentas
c) Secreción de secretina
d) Nivel de IP3 en el plexo submucoso (de Meissner)

10) recién nacido con protrusión de contenidos abdominales y cubiertas por amnios o
peritoneo, es cierto que:

a) Se presenta por un defecto en el cierre de la pared


b) Se acompaña de otras malformaciones congénitas
c) Se debe al no retorno de la hernia fisiológica
d) Se produce a través del ombligo

11)Lactante de 20 días con estreñimiento, distensión abdominal progresiva, acompañada


ocasionalmente de vómitos biliosos. Como antecedente, el meconio lo eliminó por primera
vez a las 72 horas de nacido. Su mamá menciona que ayuda a la evaluación con ayuda de
un termómetro rectal. Se sospecha megacolon agangliónico (Enfermedad de Hirschsprung).
¿Cuál de las siguientes alternativas explica el caso?

a) Se presenta contracciones tónicas en la región ano rectal


b) Se presenta dilatación de tracto gastrointestinal afectado
c) Las células ganglionares sólo han migrado al ano recto
d) La zona que más se afecta es inervada por fibras del nervio esplácnico menor

12) ¿Cuál de los siguientes reflejos disminuye el tránsito intestinal?


a) Doloroso
b) Gastrocólico
c) De defecación
d) Colicoileal
13) Al ingerir una sustancia ácida como el vino (ph 3), se estimula la motilidad gástrica por
acción de la hormona)
a) Motilina
b) Secretina
c) Colecistoquinina (CCK)
d) Bombesina

14) Paciente de 24 años acude a consulta externa por presentar fístula oronasal
(comunicación entre la cavidad oral y cavidad nasal). Esta fístula está asociada al
antecedente de haber sido operada del paladar hendido a los dos años de edad, durante
una campaña gratuita extranjera de corrección de paladar fisurado, ¿Cuál de las arterias
palatinas podría haberse lesionado durante esta cirugía?
a) Mayor
b) Menor
c) Ascendente
d) Rama palatina de faringea ascendente

15) Paciente de sexo masculino de 52 años con úlcera péptica gástrica de 14 años de
evolución, con cuadro de hemorragia digestiva alta hace 4 meses, sin cicatrización de la
úlcera. Entre las opciones quirúrgicas se considera vaguectomía troncal (sección del nervio
vago) a nivel del hiato esofágico. ¿Cuál de las siguientes complicaciones podría esperarse
producto de la pérdida de inervación parasimpática?
a) Menor inervación del colon ascendente
b) Se perderá el reflejo de defecación
c) Se perderá el reflejo de micción
d) Impotencia sexual

16) Paciente varón de 23 años con bulimia es traída deshidratada, semiconsciente y con
alcalosis metabólica. Los vómitos autoinfligidos por esta paciente se producen por
estimulación de receptores en la base de la lengua que mandan información directamente
al:
a) Núcleo del tracto solitario
b) Centro del vómito en el tallo encefálico
c) Zona quimiorreceptora gatillo
d) Cerebelo

17) Niño de 3 años es traído a emergencia por padre quien manifiesta que hace 10 horas
deglutió una pila pequeña de reloj de bordes romos. El niño está asintomático. Usted la
tranquiliza diciéndole es un cuerpo extraño tan pequeño que va a seguir el tránsito intestinal
como lo haría un bolo alimenticio, y que lo más probable es que en ese momento se
encuentre en:
1) colon
2) estómago
3) yeyuno
4) recto
18) Los movimientos en masa son un tipo de movimiento muy importante, una de las
consecuencias de estos movimientos es:
a) distensión rectal
b) peristaltismo del intestino delgado
c) retropulsión gástrica
d) contracción del esfínter anal interno
19) Durante la deglución, al momento que el bolo alimenticio pasa por el esfínter esofágico
superior, se espera que la presión intraesofágica:
a) disminuya en el cardias
b) disminuya en el tercio medio del esófago
c) aumente en la porción distal al bolo
d) aumente en el tercio medio del esófago
20) Paciente con enfermedad de chagas que presenta disfagia a sólidos. ¿Cuál de las
siguientes puede ser la causa de esta complicación?
a) disminución de células ganglionares en el esfínter esofágico inferior
b) aumento de la liberación de óxido nítrico en el esfínter esofágico inferior
c) disminución de las neuronas que liberan péptido intestinal vasoactivo
d) aumento de la actividad de la motilina en el esófago distal
21) El consumo de leche produce indirectamente
a) Curación de la gastritis
b) Estimulación del reflejo gastrocólico
c) Estimulación del vaciamiento gástrico
d) Inhibición del vaciamiento gástrico

22) Varón de 30 años es traído a emergencia por agresión abdominal con arma de fuego
(pistola) y es sometido a laparotomía exploratoria, observándose isquemia del colon
ascendente y parte del colon transverso ¿la lesión de cúal de las siguientes arterias
explicaría esta isquemia?
MESENTÉRICA SUPERIOR
23) Lactante masculino de 5 meses de edad producto de un primer embarazo normal
controlado . Peso de nacimiento 3.120 g y talla de 51 cm . Lactancia materna satisfactoria,
con buen incremento ponderal . Sin antecedentes patológicos hasta 5 días atrás , cuando
comienza con vómitos posprandiales no biliosos , lácteos . Los síntomas aumentan en
frecuencia y magnitud hasta hacerse explosivos después de cada alimentación . No requiere
tos , fiebre , diarrea ni lesiones cutáneas . Conserva el apetito y llora de hambre .Se espera
encontrar mayor liberación de :
a) GIP
b) Enzimas pancreáticas
c) Insulina
d) Secretina
24) Respecto a las sustancias GI que regulan la secreción pancreática; marque la
afirmación correcta:
a) La secretina, es la hormona más importante para la secreción de bicarbonato
por las células acinares del páncreas
b) La acetilcolina es capaz de estimular la secreción enzimática y de
bicarbonato del páncreas
c) La Gastrina es la hormona más importante para la secreción de enzimas
pancreáticas
d) La CCK estimula el páncreas solo para la secreción enzimática
25 ) Lactante masculino de 5 meses de edad producto de un primer embarazo normal
controlado . Peso de nacimiento 3.120 g y talla de 51 cm . Lactancia materna satisfactoria,
con buen incremento ponderal . Sin antecedentes patológicos hasta 5 días atrás , cuando
comienza con vómitos posprandiales no biliosos , lácteos . Los síntomas aumentan en
frecuencia y magnitud hasta hacerse explosivos después de cada alimentación . No requiere
tos , fiebre , diarrea ni lesiones cutáneas Una persona adulta podría tener un cuadro de
hipertrofia del píloro secundaria a :
a) Gastritis hipoclorhídrica
26) Ante una lesión del X par craneal ¿Cuál de los siguientes músculos mantiene
conservada su función?
A) Elevador del velo del paladar
B) Tensor del velo del paladar
C) Palatofaríngeo
D) Glosofaríngeo
27) Lactante masculino de 5 meses de edad producto de un primer embarazo normal
controlado . Peso de nacimiento 3.120 g y talla de 51 cm . Lactancia materna satisfactoria,
con buen incremento ponderal . Sin antecedentes patológicos hasta 5 días atrás , cuando
comienza con vómitos posprandiales no biliosos , lácteos . Los síntomas aumentan en
frecuencia y magnitud hasta hacerse explosivos después de cada alimentación . No requiere
tos , fiebre , diarrea ni lesiones cutáneas . Si el paciente tuviera 35 años de edad usted
podría pensar en un diagnóstico diferencial de la hipertrofia del píloro , la presencia de un
tumor neuroendocrino productor de la hormona
a) Secretina
b) Gastrina
c) Colecistoquinina
d) Bombesina
28) Lactante masculino de 5 meses de edad producto de un primer embarazo normal
controlado . Peso de nacimiento 3.120 g y talla de 51 cm . Lactancia materna satisfactoria,
con buen incremento ponderal . Sin antecedentes patológicos hasta 5 días atrás , cuando
comienza con vómitos posprandiales no biliosos , lácteos . Los síntomas aumentan en
frecuencia y magnitud hasta hacerse explosivos después de cada alimentación . No requiere
tos , fiebre , diarrea ni lesiones cutáneas . En relación a la estructura afectada se encuentra
a) Ligamento redondo
b) Mesenterio propiamente dicho
c) Ligamento gastroesplénico
d) Omento mayor
29) Experimentalmente se utiliza atropina (anticolinérgico) para inhibir la secreción de
gastrina, sin embargo, la secreción de esta hormona se sigue dando ante estímulos vagales.
Esta situación se explica porque la atropina:
a) Bloquea parcialmente la bomba de protones en la célula G
b) Inhibe la acción de acetilcolina e histamina en la célula G
c) Sólo inhibe la acción del péptido GRP en la célula G
d) No bloquea la acción del péptido GRP
30) Lactante masculino de 5 meses de edad producto de un primer embarazo normal
controlado . Peso de nacimiento 3.120 g y talla de 51 cm . Lactancia materna satisfactoria,
con buen incremento ponderal . Sin antecedentes patológicos hasta 5 días atrás , cuando
comienza con vómitos posprandiales no biliosos , lácteos . Los síntomas aumentan en
frecuencia y magnitud hasta hacerse explosivos después de cada alimentación . No requiere
tos , fiebre , diarrea ni lesiones cutáneas . El uso de atropina en este paciente
a) Aumenta el pH del estómago
b) Disminuye la acción del receptor CCK-B
c) Aumenta la producción de ácido clorhídrico
d) Inhibe la acción de las prostaglandinas

31) El consumo de dos vasos de agua seguidos generará indirectamente aumento en la


liberación de
a) Ácido clorhídrico
b) Enzimas pancreáticas
c) Péptido Y
d) Noradrenaline
32) Un varón de 50 años es sometido a extirpación del duodeno y parte proximal del
yeyuno. Esta situación ocasiona la pérdida de las células _____S_____, productoras de
_____ Secretina ____ que estimula la secreción de bicarbonato por el páncreas.

33) Estudiante de medicina de la UPC de 21 años sufre de gastritis aguda ocasionada por
comer en lugares poco higiénicos. Suele consumir caramelos ( chupa ) mientras está en
clase hasta la tarde. Toma gaseosa regularmente (carbohidratos 46%, sodio 53%). También
toma regular cantidad de leche (grasa 35%, lactosa 35%, proteínas 30%), pues le calma un
poco el dolor el ardor que siente por la gastritis. Incluso, cuando puede, se toma dos vasos
de agua fría para calmar las molestias. Ha decidido ir al médico para tratarse pues ya no
soporta el dolor, el cual está seguro que los síntomas se deben a una elevada producción
de ácido clorhídrico en el estómago, y por ello le ha recetado Loratadina (antihistamínico),
con lo que siente mejoría.
El uso de atropina en este paciente:
a) aumentará el pH del estómago
b) aumentará la producción de ácido clorhídrico
c) inhibirá la acción de las prostaglandinas
d) disminuirá la acción del receptor CCK-B

34) El uso de Ranitidina bloquea el receptor H2 de la histamina en las células parietales . La


histamina llega a estas células por
a) Difusión
b) Vía hematógena
c) Se produce en la misma célula parietal
d) La luz gástrica

35) Aumenta la secreción salival


a) Noradrenalina
b) Colecistoquinina
c) Secretina
d) Gastrina
36) Paciente con gastritis aguda debido a una alta producción de ácido clorhídrico si se le
hiciera un examen de sangre , se encontraría elevados los niveles de
a) Colecistoquinina

37) Recién nacido que presenta tumoración abdominal a nivel del cordón umbilical ¿cuál de
las siguientes afirmaciones es correcta respecto a este defecto en el desarrollo embriológico
del intestino?
a) Corresponde a una gastrosquisis
b) Las vísceras se hallan cubiertas por piel
c) No está asociado a otras malformaciones
d) Se asocia a malformaciones cardiacas y del tubo neural

38) El consumir caramelos eleva los niveles en sangre de una hormona cuya función es la
estimulación de las células
a) Beta del páncreas
b) Alfa del páncreas
c) G del antro
d) K del duodeno
39) La triada portal ( arteria hepática , vena portal y conducto biliar común ) está contenida
en el
a) Ligamento Gastroesplenico
b) Ligamento gastrohepatico
c) Ligamento hepatoduodenal
d) Ligamento falciforme

40) Varón de 35 años acude a la emergencia por trauma


abdominal y se decide realizar una laparoscopía
exploratoria. El cirujano observa la disposición de los
órganos abdominales como se representa en el siguiente
esquema. Esta disposición de órganos se explica por la
rotación: HORARIA DEL INTESTINO MEDIO
41) Se evalúa la expresión de la proteína Agrp en una
persona con alteración del apetito, lo correcto respecto a
esta proteína es:
a) Esta proteína es un potente anorexigénico
b) La mutación del gen que la codifica produce
adelgazamiento
c) La sobreproducción de la proteína lleva a obesidad
por agonista de receptores MC3 y MC4
d) La sobreproducción de la proteína disminuye el apetito por antagonismo de
receptores MC4.
42) Juana cae de la bicicleta y se fractura la región anterior del hueso maxilar superior con
compromiso de la fosa incisiva. Al examen físico de la región esperaría encontrar alteración
en la sensibilidad de la encía
PALATINA ANTERIOR
43) Recién nacido es atendido por el neonatólogo y luego entregado a su madre para dar de
lactar; la madre al dar de lactar observa coloración azulada de labios, acompañado de tos
persistente, dificultad respiratoria y distensión abdominal. Se le intenta colocar una sonda
nasogástrica pero esta retorna a la cavidad oral en todos los intentos ¿cual de los siguientes
anomalías el desarrollo es el más probable en este caso?
a) Estenosis esofágica proximal con fístula traqueo esofágica distal
b) Atresia esofágica proximal con fístula traqueo esofágica distal
c) Atresia esofágica distal con fístula traqueoesofágica proximal
d) Fístula traqueoesofágica proximal y distal
44) Estudiante de medicina de la UPC de 21 años sufre de gastritis aguda ocasionada por
comer en lugares poco higiénicos. Suele consumir caramelos ( chupa ) mientras está en
clase hasta la tarde. Toma gaseosa regularmente (carbohidratos 46%, sodio 53%). También
toma regular cantidad de leche (grasa 35%, lactosa 35%, proteínas 30%), pues le calma un
poco el dolor el ardor que siente por la gastritis. Incluso, cuando puede, se toma dos vasos
de agua fría para calmar las molestias. Ha decidido ir al médico para tratarse pues ya no
soporta el dolor, el cual está seguro que los síntomas se deben a una elevada producción
de ácido clorhídrico en el estómago, y por ello le ha recetado Loratadina (antihistamínico),
con lo que siente mejoría.
El consumo rápido de 500 mL de gaseosa aumentará directamente la concentración sérica
de cuál de las siguientes hormonas:
a) Colecistoquinina (CCK)
b) Neuropéptido Y
c) Secretina
d) Gastrina
45) Estudiante de medicina de la UPC de 21 años sufre de gastritis aguda ocasionada por
comer en lugares poco higiénicos. Suele consumir caramelos ( chupa ) mientras está en
clase hasta la tarde. Toma gaseosa regularmente (carbohidratos 46%, sodio 53%). También
toma regular cantidad de leche (grasa 35%, lactosa 35%, proteínas 30%), pues le calma un
poco el dolor el ardor que siente por la gastritis. Incluso, cuando puede, se toma dos vasos
de agua fría para calmar las molestias. Ha decidido ir al médico para tratarse pues ya no
soporta el dolor, el cual está seguro que los síntomas se deben a una elevada producción
de ácido clorhídrico en el estómago, y por ello le ha recetado Loratadina (antihistamínico),
con lo que siente mejoría.
Entre las sustancias cerebrales que producen ansiedad está la serotonina, la cual también
tiene acción:
a) Anorexigénica
b) Orexigénica
c) No influye en la producción de apetito
d) La acción de la serotonina en el apetito no está descrita
46) Estudiante de medicina de la UPC de 21 años sufre de gastritis aguda ocasionada por
comer en lugares poco higiénicos. Suele consumir caramelos ( chupa ) mientras está en
clase hasta la tarde. Toma gaseosa regularmente (carbohidratos 46%, sodio 53%). También
toma regular cantidad de leche (grasa 35%, lactosa 35%, proteínas 30%), pues le calma un
poco el dolor el ardor que siente por la gastritis. Incluso, cuando puede, se toma dos vasos
de agua fría para calmar las molestias. Ha decidido ir al médico para tratarse pues ya no
soporta el dolor, el cual está seguro que los síntomas se deben a una elevada producción
de ácido clorhídrico en el estómago, y por ello le ha recetado Loratadina (antihistamínico),
con lo que siente mejoría.
El consumo de leche produce directamente un aumento de los niveles séricos de la
hormona:
a) Gastrina
b) Grelina
c) Leptina
d) Péptido intestinal vasoactivo (VIP)
47) ¿En qué casos los vómitos son siempre biliosos?
a) Estenosis hipertrófica del píloro
b) Atresia duodenal
c) Atresia esofágica
d) Atresia yeyunal
48) Cuando la contracción peristáltica primaria está atraviesa el tercio superior del esófago,
el esfínter esofágico inferior estará:
a) Sin modificaciones
b) Denervado
c) Contraído
d) Relajado
49) Las arterias que derivan del tronco celíaco son, excepto:
a) Hepática derecha
b) Hepática común
c) Gástrica izquierda
d) Arteria esplénica
50) El uso de Ranitidina bloquea el receptor H2 de la histamina en las células parietales. La
histamina llega a estas células por:
a) Difusión
b) Vía hematógena
c) Se produce en la misma célula parietal
d) La luz gástrica
51) En cuanto a los reflejos gastrointestinales, un reflejo que estimula el tránsito intestinal es
el reflejo:
a) Cólico-ileal
b) Vómito
c) Gastrocólico
d) Entero-gástrico
52) Marque la respuesta correcta
a) La pared gástrica en el fondo es más delgada que en el cuerpo y antro
b) El esfínter de Oddi rodea a la papila menor duodenal
c) El bronquio derecho constituye una de las estrecheces del esofago
d) Todos los órganos del sistema digestivo tienen capa serosa
53) ¿En qué capa se encuentra la alteración principal en el Hirschsprung o megacolon
agangliónico?
a) Submucosa
b) Serosa o Adventicia
c) Muscular propia
d) Mucosa
54) Con respecto a las ondas lentas, marque la afirmación correcta:
a) Su frecuencia aumenta por acción de la acetilcolina
b) Son potenciales de acción que producen la contracción del tracto
gastrointestinal
c) Son contracciones rítmicas espontáneas
d) Su frecuencia disminuye por acción de la acetilcolina
55) En un paciente con apendicitis aguda la sensación de dolor producido por esta
inflamación es llevada por el nervio:
a) Vago
b) Pélvico
c) Esplacnico mayor
d) Esplacnico menor
56) La triada sintomática: vómitos explosivos post-prandiales, movimientos
peristálticos epigástricos visibles de izquierda a derecha y nódulo palpable
epigástrico subcostal derecho pertenece a:
a) Tumor benigno de las glándulas submucosas
b) Estenosis congénita hipertrófica del píloro
c) Colon irritable
d) Páncreas anular

EXAMEN FINAL 2019-01

1. La explicación fisiológica de presentar somnolencia de 30 minutos a 1 hora después de


ingerir alimentos, se explica por:
a. Aumento del cloro intraluminal
b. Aumento del bicarbonato intraluminal
c. Disminución de ácido carbónico en la célula parietal

d. Disminución de la actividad de la anhidrasa carbónicae. Aumento de la alcalinidad


sanguínea

2. Con respecto a la irrigación arterial del colon, a que arteria corresponde la señalada con
la flecha
a. Cólica derecha
b. Cólica media
c. Cólica izquierda
d. Ileobisecoapendículocólica
e. Arco de Riolano

3. Si un paciente presentara dentro del punto de vista fisiológico, una disminución de


enterocinasa, entonces esto originaría una disminución de la actividad de:
a. la pepsina
b. la lipasa (PUEDE SER POR SU RELACIÓN CON LA COLIPASA)
c. la quimotripsina
d. el péptido insulinotrópico dependiente de glucosa
e. la amilasa
4. Con respecto a la anatomía del hígado, señale a qué estructura pertenece la marcada por

el número 1.
a. Ligamento falciforme
b. Línea de Cantlie
c. Ligamento triangular
d. Ligamento coronario
e. Ligamento teres

5. Se presenta un paciente, el cual presenta un antecedente de tuberculosis intestinal, por lo


cual, se le resecó 80 cm de íleon distal. Desde el punto de vista fisiológico, el paciente
puede presentar una de las siguientes alteraciones:
a. Disminución de la secreción de Vitamina B12
b. Aumento indiscriminado de absorción de ácido fólico
c. Disminución de la absorción de hierro
d. Aumento de la secreción de bicarbonato
e. Disminución de la absorción de ácido glicocólico

6. Un paciente es sometido experimentalmente a un fármaco que modifica el flujo salival,


obteniéndose un volumen de saliva de 288 ml en 6 horas. En este caso las concentraciones
de electrolitos y bicarbonato en la saliva obtenida varían de la siguiente manera:
a. ↑ Na+, ↓ K+, ↑ Cl-, ↑ HCO3-
b. ↓ Na+, ↓ Cl-, ↑ K+, ↓ HCO3-
c. ↑ Na+, ↑ Cl-, ↓ K+, ↓ HCO3-
d. ↑ Na+, ↑ Cl-, ↑ K+, ↑ HCO3-
e. ↓ Na+, ↓ Cl-, ↓ K+, ↓ HCO3-

7. La siguiente imagen histológica corresponde a la glándula ............... y la estructura


señalada produce .........
a. salival sublingual / mucopolisacáridos
b. oxíntica / pepsinógeno
c. salival submaxilar / ptialina
d. salival parótida / amilasas
e. antrales / gastrina

8. Paciente varón de 65 años con antecedente de hipercolesterolemia, hipertensión arterial,


fibrilación auricular y dos infartos al miocardio previos, aqueja de dolor abdominal intenso de
inicio súbito, distensión abdominal, se decide cirugía con resección de 1,5 metros de
intestino delgado terminal y colon ascendente. Como consecuencia de la resección el
paciente tendrá deficiencia de:
a. Vitamina C
b. Tiamina
c. Vitamina A
d. Vitamina B1
e. Vitamina B6

9. Uno de los siguientes elementos debería hallarse con más probabilidad en el esófago de
un paciente que sufre de reflujo gastroesofágico...
a. Pepsina
b. Tripsina
c. Quimotripsina
d. Carboxipeptidasa
e. Ácidos biliares

10. Un paciente de 40 años cursa con anemia de 8g/dl, aqueja además de astenia y
sensación de hormigueo bilateral en los miembros inferiores, al examen se halla alteración
de la sensibilidad a la vibración y camina con ampliación de la base de sustentación. Uno de
los siguientes procedimientos sería de ayuda para el diagnóstico de este paciente:
a. Tomografía cerebral
b. Biopsia de la mucosa gástrica
c. Biopsia de hígado
d. Examen de sangre oculta en heces
e. Biopsia de Íleon proximal

11. Paciente de 60 años ingresa por caída hace 1 hora y pequeño hematoma en cuero
cabelludo, al examen físico ampliado se observa ictericia de piel y mucosas generalizada,
abdomen blando, se palpa estructura quística no dolorosa en hipocondrio derecho que
corresponde a vesícula biliar (signo de Courvoisier), en los exámenes de laboratorio se halla
niveles bajos en la formación de estercobilinógeno y urobilinógeno en heces, incremento de
la bilirrubina conjugada en la orina, elevación de fosfatasa alcalina y gamma glutamil
transpeptidasa séricas. El presente cuadro puede ser explicado por:
a. Reabsorción de hematoma
b. Litiasis vesicular
c. Carcinoma de la cabeza de páncreas
d. Carcinoma con estenosis del conducto hepático común
e. Anemia hemolítica

12. Paciente varón de 58 años con antecedente de alcoholismo crónico es diagnosticado y


recibe tratamiento por cirrosis hepática. Hace 2 días refiere familiar que tuvo cambio de
conducta y no reconoce a algunos familiares. Al examen físico, se halla ascitis, circulación
colateral en abdomen, telangiectasias, en el examen de sistema nervioso: rigidez de
extremidades, ROT incrementados, desorientación en el espacio y asterixis. ¿cuál de las
siguientes circunstancias, explicaría el cuadro en este paciente?
a. Uso de diuréticos ahorradores de potasio
b. Incremento de actividad de ureasa bacteriana duodenal
c. Hemorragia gastrointestinal
d. Disminución de la producción de NH3+ en el colon
e. Dieta normoproteica

13. Un recién nacido presenta vómitos biliosos poco tiempo después de cada alimento. Al
preguntar a la madre sobre antecedentes, ella recuerda que tuvo polihidramnios durante la
gestación, pero un análisis de cariotipo fue normal. Una de las siguientes es la causa más
probable de estos hallazgos en el recién nacido:
a. Enfermedad de Hirschsprung
b. Fístula tráqueo esofágica
c. Divertículo ileal
d. Estenosis pilórica
e. Malrotación de la yema pancreática ventral

14. Un lobulillo hepático se puede dividir en tres zonas como se muestra en el gráfico. ¿Cuál
de las siguientes afirmaciones sobre las tres zonas es verdadera?
a. La zona 1 tiene los menores depósitos de glucógeno
b. La zona 3 es la primera en afectarse en una colestasis extrahepática
c. La zona 2 es más susceptible a la injuria por isquemia que la zona periportal
d. La zona 2 tiene la mayor capacidad de regeneración
e. La zona 1 es la que tiene menos actividad metabólica.

15. En un estudio de la secreción de hormonas gastrointestinales, sus concentraciones en la


vena porta se midieron durante perfusión luminal del intestino delgado con soluciones de
diversas magnitudes de pH. ¿Qué hormona aumentará en el plasma de la vena porta
durante perfusión a través del intestino con una solución de pH 3?
a. CCK
b. gastrina
c. GIP
d. motilina
e. secretina

16. Paciente de 30 años que ingresa a causa de un traumatismo abdominal cerrado. En la


exploración se aprecia discreta palidez de piel y mucosas, auscultación pulmonar normal,
taquicardia de 120 /min. Discreta distensión abdominal y matidez en flancos; el hematocrito,
que era prácticamente normal al ingreso, disminuye un 30% a las tres horas. En la Rx de
tórax se objetiva fractura de las costillas 10-11 izquierdas. La causa más probable de la
anemización en este paciente es:
a. traumatismo renal con hemorragia retroperitoneal.
b. rotura de hígado con hemoperitoneo.
c. rotura de bazo con hemoperitoneo.
d. rotura de mesos con hemoperitoneo.
e. traumatismo pancreático con pancreatitis traumática.
17. Mujer de 65 años. Consulta por síndrome constitucional asociado a dolor abdominal
epigástrico progresivo irradiado a espalda, de dos meses de evolución. El diagnóstico de
sospecha de adenocarcinoma de páncreas se confirma por biopsia. Se realiza examen de
imagen de abdomen para evaluación de estructuras vasculares próximas al tumor
pancreático. ¿Cuál es el nombre de la vena señalada que está ausente, trombosada por
infiltración tumoral, condicionando circulación colateral en la pared gástrica?
a. Mesentérica superior
b. Coronaria estomáquica
c. Esplénica
d. Porta
e. Renal izquierda

18. Revisando la angiotomografía de un hombre de 70 años en estudio por aneurisma de


aorta abdominal, el radiólogo le informa de la presencia de una oclusión completa de la
arteria mesentérica inferior. El paciente se encuentra completamente asintomático. La
oclusión de la arteria mesentérica inferior cursa de manera asintomática en muchas
ocasiones ya que el territorio que irriga puede recibir flujo proveniente de la arteria:
a. cólica derecha
b. gastroduodenal
c. Epigástrica inferior izquierda
d. esplénica
e. cólica media

19. En las patologías de esófago es importante conocer bien la anatomía esofágica. ¿Cuál
de las siguientes afirmaciones es correcta?
a. El esófago tiene capa mucosa, muscular y serosa
b. El esófago abdominal es más largo que el cervical
c. El esófago torácico pasa por detrás del cayado aórtico
d. El epitelio esofágico normal es de tipo cilíndrico.
e. El esófago abdominal es discretamente más largo que el torácico

20. A pesar de que pueda haber variaciones anatómicas, lo habitual es que el ciego sea
irrigado por una rama arterial que proviene de unas de las siguientes arterias:
a. Iliaca derecha
b. Mesentérica inferior
c. Hepática derecha
d. Mesentérica superior
e. Iliaca izquierda

21. Ante un paciente con una cirugía abdominal urgente, el informe operatorio señala que se
ha realizado una resección de todo el duodeno y del tercio proximal del yeyuno manteniendo
íntegros el estómago y todo el íleon, así como los dos tercios distales del yeyuno. En el
seguimiento nutricional del paciente ¿Que vitamina o mineral presentará con menor
probabilidad una disminución de su absorción?
a. Cianocobalamina
b. Calcio
c. Hierro
d. Transcobalamina
e. Transferrina
22. ¿Cuál de las siguientes alternativas detallan las venas que confluyen y forman la vena
señalada?
a. mesentérica superior, gástrica izquierda y gastroepiploica izquierda
a. mesentérica inferior, gástrica izquierda y renal
b. esplénica, mesentérica superior y mesentérica inferior
c. esplénica, pancreatoduodenal y omental izquierda
d. gástrica izquierda, esplénica y hepática común

23. ¿Cuál de las siguientes sustancias forma parte de la secreción biliar?


a. Tripsina
b. Lecitina
c. Elastasa
d. Quimotripsina
e. Pepsina

24. El tubo digestivo contiene diferentes tipos de epitelios y glándulas. La estructura


señalada es una ................... y está localizada en el ...............
a. glándula de Brunner / intestino grueso
b. cripta de Lieberkühn / colon
c. cripta de Lieberkühn / intestino delgado
d. glándula oxíntica / estómago
e. célula parietal / estómago

25. ¿De qué músculo forma parte el ligamento inguinal?


a. Oblicuo externo del abdomen
b. Oblicuo interno del abdomen
c. Transverso del abdomen
d. Psoas
e. Dorsal ancho

26. Señale cuál de las siguientes afirmaciones NO se relaciona a la siguiente glándula


anexa del tubo digestivo mostrada en la imagen:
a. Es una glándula exocrina compuesta exclusivamente por acinos serosos
b. Su inervación está dada por el nervio auricular mayor (ramo posterior C2), que
inerva la vaina de la glándula así como la piel por encima de esta.
c. Esta glándula produce una secreción mucinosa acuosa, llamada mucoserosa, a
través del conducto de Wharton.
d. Su inflamación puede ser causada por un virus de los Paramyxoviridae, que
provocan una enfermedad muy frecuentemente en niños y adolescentes
e. Es una glándula endocrina y probablemente sea de origen pancreático

27. ¿Cuál de las siguientes enzimas está localizada en el borde del cepillo y juega un rol en
la digestión de proteínas?
a. Alpha dextrinase
b. Pepsina
c. Enterocinasa
d. Lactasa
e. Carboxipeptidasa A.
28. Una de los siguientes sustancias, NO sirve como un buen agente emulsificante:
a. Colesterol
b. Ácidos grasos
c. Sales biliares
d. Lecitina
e. Proteínas de la dieta

29. La sustancia que estimula el crecimiento de la mucosa gástrica es:


a. Secretina
b. Motilina
c. Péptido estimulante de la mucosa gástrica
d. Gastrina
e. Histamina

30. ¿Cuál de las siguientes alternativas es una función de la colecistocinina?


a. Relajación de la vesícula para la salida de bilis
b. Secreción de ácidos biliares
c. Contracción del esfínter de Oddi
d. Secreción de enzimas pancreáticas
e. Contracción del duodeno

31. Con respecto a la anatomía del tronco celíaco, señale lo correcto


a. El tronco celíaco se origina de la cara posterior de la aorta abdominal
b. Es una arteria delgada que tiene un calibre entre 2 y 3 mm
c. Una de sus ramas es la arteria gástrica derecha
d. La hepática común que es una de sus ramas, participa en la irrigación del estómago.

32. Con respecto a la anatomía del duodeno, marque la respuesta correcta:


a. Tiene una distribución en forma de “C”, que rodea la cola del páncreas
b. La 3ra porción duodenal está contenida en la pinza vascular aortomesentérica
c. Entre la 1ra y 2da porción se forma un ángulo, conocido como el ángulo de Treitz
d. La 4ta porción se dirige a la izquierda, hacia abajo y hacia atrás.
e. En la tercera porción desemboca el conducto colédoco.

33. El hígado está ampliamente tapizado por peritoneo, la estructura que conecta la cara
diafragmática del hígado precisamente con el diafragma es el ligamento:
a. teres
b. falciforme
c. triangular
d. hepático común
e. coronario

34. En el íleon se absorbe aproximadamente el 95% de ...................... a través de la


circulación enterohepática.
a. agua
b. colesterol
c. sales biliares
d. hidroxicobalamina
e. factor intrínseca

35. La ................ estimula el mecanismo paracrino de la secreción de ácido clorhídrico.


a. histamina
b. acetilcolina
c. gastrina
d. secretina
e. somatostatina

36. En la digestión de proteínas, ................ es el principal estímulo para convertir el


pepsinógeno en pepsina.
a. la gastrina
b. el pH ácido
c. la acetilcolina
d. la ptialina
e. la somatostatina

37. Con respecto a la somatostatina, marque lo correcto:


a. Es secretada por las células S del intestino
b. Induce a la producción de VIP
c. Interviene en la fase intestinal de la secreción gástrica
d. Produce acetilcolina para estimular a la célula parietal
e. No interviene en la regulación de la secreción de ácido clorhídrico

38. En pecten anal, es una estructura comprendida entre:


a. la línea pectínea y los senos anales
b. la línea blanca y la apertura anal
c. el esfínter anal interno y el externo
d. la línea anocutánea y la línea pectínea
e. la línea blanca y columnas anales

39. ¿Cuál de las siguientes alternativas es una proenzima pancreática?


a. Tripsina
b. Elastasa
c. Quimotripsinógeno
d. Amilasa
e. Procarboxipeptidasa C.

40. En la segmentación hepática de Couinaud, el segmento hepático señalado con la flecha,


corresponde a: En la segmentación hepática de Couinaud, la flecha señala el segmento
.............. Hepático.
a. IV
b. V
c. VI
d. VII
e. VIII

EXAMEN FINAL 2019-02


1. La reabsorción de sodio y cloro en las glándulas salivales se da principalmente en el:
a) conducto intercalado
b) conducto excretor
c) conducto estriado
d) acino glandular

2. Paciente de 35 años con dolor en hipocondrio derecho irradiado a la punta de la


escápula. Se observa en la ecografía abdominal cálculos en la vesícula biliar. Es operado de
emergencia realizando una incisión a lo largo del reborde costal derecho. La información de
dolor de esta zona es inervada por las raíces nerviosas:
a) T5 – T9
b) T6 – T7
c) T7 – T8
d) T9 – T10

3. Paciente de 23 años con dolor intenso periumbilical de inicio agudo. Es ingresado a sala
de operaciones por abdomen agudo, el cirujano encuentra sangrado a unos 60 cm proximal
a la válvula ileocecal. La estructura que está sangrando muy probablemente es un derivado
embriológico del:
a) uraco
b) ligamento Hepatoduodenal
c) alantoides
d) conducto vitelino

4. Paciente de 64 años de edad con dolor en epigastrio que se distribuye en banda hacia la
espalda. En la tomografía se encuentra tumor en cabeza de páncreas que compromete a un
vaso que discurre entre la cabeza del páncreas y el proceso uncinado. Debido a esto, este
cáncer se considera irresecable porque compromete a la arteria:
a) mesentérica superior
b) mesentérica inferior
c) tronco celíaco
d) aorta

5. Paciente con carcinoma gástrico avanzado, en preoperatorio para gastrectomía total.


¿Cuál de los siguientes ganglios linfáticos recibirá primero células metastásicas con mayor
probabilidad:
a) celíaco
b) cisterna del quilo
c) esplénico
d) gástrico posterior

6. Paciente de 77 años con dolor abdominal difuso de 3 horas de evolución. En los estudios
de imágenes se observa oclusión probablemente aterosclerótica de la arteria mesentérica
superior; no se observa necrosis, lo cual puede ser explicado por la irrigación sanguínea
colateral. ¿Qué vasos ofrecen colaterales entre el tronco celíaco y la arteria mesentérica
superior?
a) Gástrica izquierda y hepática
b) Cística y duodenal
c) Gastroomental derecha e izquierda
d) Pancreaticoduodenal superior e inferior

7. Paciente de 62 años con vólvulo de intestino delgado e isquemia intestinal. Se realiza


laparotomía exploratoria para liberar la obstrucción. ¿Cuál estructura se utilizará como punto
de referencia para determinar la posición de la unión duodenoyeyunal?
a) Vasos rectos
b) Ligamento suspensorio del duodeno (de Treitz)
c) Ligamento frenocólico
d) Nacimiento de la Arteria mesentérica superior

8. Paciente de 42 años con dolor abdominal intenso y hematemesis. En la endoscopia se


observa una úlcera duodenal posterior perforada con hemorragia intraabdominal. ¿Cuál de
las siguientes arterias estará comprometida?
a) Gástrica izquierda
b) Mesentérica superior
c) Gástrica derecha
d) Pancreaticoduodenal posterosuperior

9. Paciente de 51 años con antecedente de enfermedad diverticular acude a emergencia por


sangrado profuso y dolor en cuadrantes inferiores. ¿Cuál es el origen más probable de la
sangre que pierde en la hemorragia?
a) Cólica media
b) Mesentérica inferior
c) Rectal superior
d) Rectal inferior

10. Paciente de 13 días de vida con vómitos explosivos a las dos horas después de lactar.
Al examen físico se palpa la oliva pilórica. ¿Cuál es el nervio cuyos filetes dan inervación
eferente a la estructura afectada?
a) Vago
b) Esplácnico torácico mayor
c) Mesentérico superior
d) Esplácnico torácico menor

11. Paciente de 80 años con dolor abdominal intenso y antecedente de estreñimiento


crónico. En la colonoscopia se observa divertículos con áreas ulceradas difusas en colon
sigmoides y descendente. El cirujano programa una cirugía de resección y planifica que
para realizar esta recepción tendrá que cortar los siguientes vasos y nervios:
a) Plexo mesentérico superior y arteria rectal.
b) Nervio esplácnico torácico mayor y arteria cólica media.
ramas del nervio vago y arteria ileocólica.
d) nervio esplácnico pélvico y arteria cólica izquierda.

12. Paciente de 46 años ingresa a emergencia con dolor en cuadrante superior derecho e
ictericia. En la ecografía se observa cálculos en la vesícula biliar. ¿Cuál de los siguientes
nervios transmite el dolor de la colecistitis?
a) Filetes aferentes del nervio vago derecho, referido al ángulo inferior de la escápula
b) Filetes nerviosos de las raíces T1 a T4, con referencia al hombro izquierdo
c) Fibras aferentes simpáticas del nervio esplácnico torácico mayor, con referencia a
los dermatomas T6 a T8
d) Fibras aferentes de los ramos primarios dorsales de los nervios raquídeos T8 a T10
con referencia al epigastrio

13. Un niño de 8 años es alimentado por sus padres con un Cheese Burger, papitas fritas y
un vaso de Coca Cola. ¿La presencia de lípidos, carbohidratos y proteínas en el duodeno
estimulan la liberación de cuál de las siguientes hormonas en la mucosa duodenal?
a) Colecistoquinina (CCK)
b) Péptido inhibidor gástrico (GIP)
c) Secretina
d) Insulina

14. La fase cefálica del control de la secreción gástrica corresponde a cerca del 30% de la
secreción ácida y es consecuencia de un reflejo. ¿Cuál de las siguientes alternativas puede
eliminar totalmente la fase cefálica de la secreción gástrica?
a) Vaguectomía
b) Uso de atropina
c) Uso de bloqueador de receptor H2 de histamina
d) Uso de antiácidos

15. Recién nacido de 24 horas con llanto, vómitos y sin eliminación de meconio. Luego de
los estudios auxiliares se diagnostica la Enfermedad de Hirschsprung, esta enfermedad se
caracteriza por la ausencia de:
a) las células de Cajal en plexo mientérico
b) el plexo mientérico en el recto y colon sigmoides
c) las células mioepiteliales en el recto distal
d) el plexo de Meissner en el recto distal y colon sigmoides

16. Los complejos motores migratorios aparecen aproximadamente cada 90 minutos entre
las comidas, y se considera que son estimulados por la hormona motilina. La ausencia de
estos complejos migratorios podría producir un aumento en:
a) la motilidad duodenal
b) el vaciamiento gástrico
c) la deglución
d) las bacterias intestinales

17. ¿Cuál de las siguientes es una consecuencia probable de la resección del íleon?
a) Gastritis atrófica
b) Deficiencia de vitamina B12
c) Esteatorrea
d) Úlcera péptica

18. Los movimientos en masa son importantes en la fisiología intestinal. Estos movimientos
en masa ocasionan:
a) la sensación de defecar
b) el peristaltismo duodenal
c) la retropulsión gástrica
d) la contracción del esfínter anal interno

19. La toxina colérica hace que aumenten los niveles de AMPc intracelular, y este aumento
hace que se mantenga abierto un canal en las células de la cripta de Lieberkühn. En
condiciones fisiológicas, en una persona sana ¿Qué sustancia puede promover que el canal
quede abierto también?
a) Somatostatina
b) Óxido nítrico (NO)
c) Péptido intestinal vasoactivo (VIP)
d) Péptido similar al glucagón 1 (GLP 1)

20. Paciente de 64 años con tumor abdominal que comprime la cisterna del quilo. En la
biopsia de duodeno tomada como parte del estudio, el patólogo puede observar
a) Dilatación del vaso quilífero central
b) Contracción de las venas de las vellosidades
c) Vellosidades intestinales más largas
d) Engrosamiento de la lámina basal

21. En la enfermedad de Crohn es posible encontrar células de Paneth en el colon. Esto se


puede deber a la especial función de estas células en:
a) la activación de la inmunidad adquirida
b) mantener la inmunidad innata
c) producir hormonas
d) producir Ig A

22. En 1967 se descubrió que la epidemia de Kuru, una enfermedad por priones, en el
distrito de Okupa en Papúa Nueva Guinea, era causada por la costumbre de comer la carne
de los muertos. Ahora se sabe que las proteínas priónicas ingresan al organismo a través
de:
a) los enterocitos
b) la transmigración
c) las células de Paneth
d) las células M

23. Paciente de 48 años con alteraciones en el tránsito intestinal por diabetes mellitus tipo 2;
se presenta con esteatorrea, flatulencia y malabsorción de grasas. Las pruebas de función
hepática y biliar están dentro de rangos normales. Una causa de la disminución de sales
biliares puede ser:
a) el sobrecrecimiento bacteriano
b) la deficiencia de pepsina
c) la deficiencia de elastasa
d) la hiperestimulación del GLUT5

24. Al usar azúcar de mesa (sacarosa) para endulzar su café, el estudiante de medicina
sabe que lo más probable es que para su absorción tendrá utilizar el/los transportadoras)
________________ que se encuentran en la membrana apical de los enterocitos.
a) SGLT-1
b) GLUT2 y GLUT5
c) SGLT-1 y GLUT5
d) SGLT1 y GLUT2

25. Una persona con la producción normal de lactasa; cada vez que toma leche, los
productos de la degradación de la lactosa por parte de la lactasa ingresarán al enterocito
usando el/los transportador(es) _____________:
a) SGLT-1
b) GLUT2 y GLUT5
c) SGLT-1 y GLUT5
d) SGLT1 y GLUT2

26. En una persona sana, luego de una comida basada en carnes rojas cocinadas con baja
cantidad de sal, al enterocito pueden ingresar solamente:
a) aminoácidos
b) aminoácidos y dipéptidos
c) aminoácidos, dipéptidos y tripéptidos
d) aminoácidos, dipéptidos, tripéptidos y tetrapéptidos

27. Paciente con deficiencia congénita de procolipasa, sufre de esteatorrea cada vez que
come comidas ricas en grasas. En el estudio de composición de las heces, uno espera
encontrar una cantidad aumentada de:
a) colesterol
b) triglicéridos
c) fosfolípidos
d) lisolecitina

28. Al tomar su café en Starbucks, un estudiante de medicina sufre una quemadura de


primer grado en el tercio anterior de la superficie dorsal de la lengua. La información de
dolor es transmitida por el nervio:
a) cuerda del tímpano
b) glosofaríngeo
c) lingual
d) facial

29. Paciente de 32 años con esteatosis hepática no alcohólica. Se le realiza una biopsia
hepática que confirma la esteatosis; en el tejido se observan depósitos de lípidos en los
hepatocitos, los cuales contienen principalmente:
a) triglicéridos
b) colesterol
c) ácidos grasos libre
d) acil-carnitina

30. Paciente de 21 años que es estudiante de medicina, llega a consulta refiriendo que en
épocas de exámenes su piel se vuelve amarillenta. Le realizan el diagnóstico de Síndrome
de Gilbert asociado a mutación del gen UGT1A1, luego de unas semanas acude a la
consulta por ictericia asociada a resfriado. Al hacerle un análisis de sangre, usted sabe que
encontrará valores elevados de:
a) bilirrubina directa
b) bilirrubina indirecta
c) bilirrubina tipo delta
d) fosfatasa alcalina

31. Se realizó un experimento en el cual se inyectó tinta china en el peritoneo de ratas de


laboratorio. Al realizarse una biopsia hepática de dichos animales, se encontró que el tinte
negro estaba depositado en:
a) las células de Ito
b) los hepatocitos
c) las células de Kupffer
d) las células de Disse

32. Paciente de 74 años de edad con shock hipovolémico asociado a deshidratación aguda
severa. En este paciente es posible encontrar hipoxia en zona ...... del lobulillo hepático e
infarto .................. del intestino.
a) 1 / mucoso
b) 1 / transmural
c) 3 / transmural
d) 3 / mucoso

33. Niño de 5 años con historia de tres días de evolución caracterizado por fiebre, malestar
general, odinofagia, anorexia, e irritabilidad. Al examen se observa lesiones ulcerativas de 4
mm de diámetro en mucosa yugal, con fondo blanquecino y eritema periférico. El
diagnóstico más probable es:
a) candidiasis oral
b) leucoplasia
c) aftas
d) herpes

34. Paciente de 52 años con enfermedad por reflujo gastroesofágico de 30 años de


evolución. Se realiza endoscopía en la que se encuentra mucosa eritematosa proximal a la
línea Z. Para corroborar la presencia de lesión preneoplásica, se toma una biopsia de esa
zona, en la que se espera encontrar:
a) metaplasia gástrica
b) metaplasia intestinal
c) displasia gástrica
d) adenocarcinoma

35. Un estudiante de medicina termina su último examen final. Al abrir la boca para comer
una hamburguesa con papas fritas, la activación de los receptores muscarínicos de las
células acinar y ductal estimularan un mayor flujo de saliva, con lo cual disminuirá la
concentración salival de:
a) potasio
b) bicarbonato
c) sodio
d) cloro

36. Un estudiante toma su desayuno consistente en un pan con mantequilla y queso. Antes
que se puedan digerir las grasas, es necesario que sean emulsificadas. La hormona
............................ estimula la liberación de las sustancias emulsificadoras.
a) colecistoquinina (CCK)
b) secretina
c) lipasa pancreática
d) gastrina

37. Al comer un pollo a la brasa entero, con papitas fritas y ensalada, la sustancia que
estimulará la liberación de HCl en el estómago es:
a) el neuropéptido Y
b) la secretina
c) la bombesina
d) la colecistoquinina (CCK)

38. Paciente con disminución marcada del apetito asociado a cáncer terminal, se podría
utilizar análogos de ................. para promover la ingesta de alimentos.
a) el péptido similar al glucagón (GLP)
b) la serotonina
c) la secretina
d) la endorfina

39. Un hombre de 51 años presenta de forma súbita vómitos masivos de sangre roja
brillante. Tiene como antecedente hepatitis viral B hace 23 años. En la exploración física: FC
103 latidos/min, PA 85/50 mmHg, se palpa la punta del bazo y niega vómitos. Su
hematocrito es 21%, la prueba serológica de HBsAg es positiva. En la ecografía se observa
hígado con nodulaciones. ¿Cuál es la causa más probable para la hematemesis?
a) Esófago de Barrett
b) Síndrome de Mallory Weiss
c) Varices esofágicas
d) Esofagitis por reflujo

40. Durante el reflejo del vómito, uno de los primeros sucesos es:
a) el peristaltismo inverso
b) la contracción del píloro
c) la apertura de la glotis
d) el esfínter esofágico inferior abierto

41. Paciente de 34 años que acude por diarrea desde hace 4 días asociado a comer papa
rellena con ají en el Estadio Nacional, se exacerba cuando toma lácteos o come grasas. Se
acompaña de flatulencia y episodios de tenesmo. Este cuadro de diarrea e intolerancia a la
lactosa de inicio agudo se explica por la:
a) enfermedad de Crohn
b) infección por V. cholera
c) colitis ulcerativa
d) giardiasis

42. En cuanto a la secreción pancreática, mientras mayor es el flujo, mayor es la


concentración de:
a) potasio
b) bicarbonato
c) cloro
d) sodio
43. Paciente de 10 años con diarrea crónica, distensión abdominal, anorexia. Se le ha
encontrado anticuerpos antigliadina y antiendomisio. Es más probable que la diarrea se
correlacione con el hallazgo histológico de:
a) adelgazamiento de las criptas
b) linfocitos intramusculares
c) atrofia de las vellosidades
d) úlceras duodenales

44. La glándula parótida está inervada por el par craneal:


a) VII
b) IX
c) X
d) XII

45. Las glándulas de Brunner se encuentran en la:


a) mucosa del esófago
b) submucosa del íleon
c) mucosa del yeyuno
d) submucosa del duodeno

46. ¿Cuál de las siguientes afirmaciones describe correctamente la función de la inervación


parasimpática del tracto gastrointestinal?
a) La norepinefrina es el principal neurotransmisor excitatorio.
b) La actividad parasimpática produce la relajación de los esfínteres
c) La actividad parasimpática excesiva puede provocar un trastorno llamado íleo
paralítico (parálisis del músculo liso intestinal)
d) El pH luminal, la osmolaridad y la distensión muscular son detectados por fibras
parasimpáticas eferentes

47. Al ingerir una cantidad de glucosa por vía oral, esta es interiorizada en las células del
organismo más rápido que si esa misma cantidad de glucosa hubiese sido administrada por
vía endovenosa. Este fenómeno sucede gracias a la acción de la sustancia secretada por
las células:
a) G
b) I
c) K
d) S

48. A ............... secreción de HCl en el lumen gástrico, ................. pH en la sangre venosa


gástrica
a) mayor /mayor
b) mayor / menor
c) menor / mayor
d) mayor / igual

49. ¿En cuál de las siguientes situaciones hay un menor flujo de secreción salival?
a) Masticar goma de mascar
b) Imaginarse ser sometido a un examen dental
c) Exposición a olor nauseabundo
d) Sueño

50. El alcohol y la cafeína estimulan la fase ______________ de la producción de ácido


clorhídrico.
a) cefálica
b) gástrica
c) intestinal

¿Cuál de los siguientes mecanismos ocurren durante la defecación? En la posición de


cuclillas el músculo puborrectal se halla relajado

La estructura número 4 corresponde a ____ y está


___. La dentina/ formado por células derivadas de
la cresta neural

Un paciente luego de un accidente sufre lesión del


piso de la boca, se constata daño del nervio:
cuerda del tímpano, en este caso se esperaría
encontrar disminución de la ____ Sensación del
gusto en los 2 tercios anteriores

¿Cuál de las siguientes afirmaciones es la correcta


sobre la gastrina? Actúa en la célula diana mediante su receptor CCK tipo B

Al recibir un paciente con signos de hipovolemia y antecedentes de trauma en abdomen por


accidente de tránsito, usted identifica radiológicamente: lesión primera vértebra lumbar y
signos de lesión en páncreas; durante la cirugía se observó pobre irrigación de asas
intestinales. El vaso afectado es la arteria: mesentérica superior

Un paciente sufre daño a nivel del cuello con lesión muscular en la región de la faringe. En
el examen físico se determina dificultad para la elevación de la faringe y para el cierre del
istmo de las fauces. En este caso probablemente está afectado el músculo: palatofaríngeo

Varón de 50 años a quien le realizan la curación de la segunda molar de la arcada derecha.


En un momento determinado, el paciente acusa de intenso dolor de la pieza dentaria en
tratamiento. La vía aferente del dolor viaja a través del nervio: trigémino V2

La distensión gástrica por los alimentos produce incremento de secreción de HCL mediante
la producción de ____ que estimula a las células ___ vía proteína: gastrina/ parietal/ Gq

Un niño de 3 años llega a emergencias con disfagia (dificultad para tragar), dolor retro
esternal, salivación y llanto. Se sospecha de ingesta de cuerpo extraño (moneda) en el
esofago, al ser evaluado se constata en una radiografía presencia de cuerpo extraño a nivel
de C6. El cuerpo extraño estará suspendido a nivel del estrechamiento producido por ____
el músculo cricofaríngeo

La triada portal (arteria hepática, vena portal y conducto biliar común) está contenida en el
ligamento ____ hepatoduodenal/ mesenterio ventral
EXAMEN FINAL 2019-00

1. Un niño de 2 años es llevado a la consulta por diarrea persistente, edema de las


extremidades y falta de crecimiento en relación a su edad. Los análisis de sangre
revelan que tiene concentración plasmática baja de proteínas (hipoproteinemia). Como
parte del estudio se coloca colecistocinina (CCK) endovenosa y se recoge muestras del
líquido duodenal por endoscopia; el resultado del líquido confirma incapacidad para
hidrolizar proteínas a un ph neutro, esta situación mejora al añadir una pequeña
cantidad de tripsina. El paciente probablemente está sufriendo la falta congénita de
______

a. PEPT – 1

b. Pepsinógeno

c. Enterocinasa

d. Carboxipeptidasa

2. Paciente mujer de 35 años acude a consulta por sensación de sequedad y lesiones


en cavidad oral. Al examen se observa atrofia de la mucosa, fisura y úlcera; nota
además sequedad e irritación de la córnea y aumento del tamaño de las glándulas
parótidas. Su diagnóstico más probable es artritis reumatoide, el hallazgo más probable
en una biopsia de glándula parótida es ____

a. Presencia de acinos normales con hiperplasia de células distales

b. Gran infiltración de linfocitos y células plasmáticas

c. Hiperplasia de acinos glandulares serosos

d. Gran infiltrado de linfocitos y neutrófilos

3. Un hombre de 42 años de edad se presenta al médico con una historia de 1 año de


evolución, caracterizado por dolor abdominal bajo y diarrea con crisis sanguinolentas.
Manifiesta además pérdida de peso de 8Kg durante este periodo. La colonoscopia revela
lesión difusa en el colon con afectación del recto. La biopsia de estas lesiones revela
adelgazamiento de la pared, inflamación y ulceración de la mucosa y submucosa. El
diagnóstico más probable en este caso es:

a. Síndrome de colon irritable

b. Enfermedad de Crohn

c. Colitis ulcerativa

d. Sprue celiaco

4. Dos estudiantes deciden tomar un receso para comer una hamburguesa a la hora del
almuerzo. Antes de llegar a la cafetería, impulsos nerviosos provenientes del complejo
vagal dorsal iniciarán la secreción de ácido gástrico por liberación de _____ desde el
sistema nervioso entérico
a. Serotonina

b. Colecistoquinina

c. Péptido inhibidor vasoactivo

d. GRP (péptido liberador de gastrina)

5. Un niño de cuatro años de edad es llevado a la consulta por cuadros diarreicos


frecuentes caracterizados por heces pálidas, voluminosas y fétidas, al examen físico
presenta bajo peso y talla para la edad. Se mide la concentración de cloruro en el sudor y
se encuentra que sus valores son muy elevados. La alteración más importante a nivel de
células ductales del páncreas tiene relación directa con la conductancia de ___

a. Bicarbonato

b. Potasio

c. Sodio

d. Cloro

6. Se evalúa los valores séricos de las siguientes sustancias a un paciente con enfermedad
hepática terminal, en este paciente se espera encontrar la combinación con la letra

a. Glucosa aumentada, amoniaco disminuido, albúmina disminuida

b. Glucosa disminuida, Amoniaco aumentado, albúmina aumentada

c. Glucosa aumentada, amoniaco aumentado, albúmina aumentada

d. Glucosa disminuida, amoniaco aumentada, albúmina disminuida

7. Una mujer de 35 años de edad HIV positiva se presenta al médico con dolor abdominal
en cuadrante superior derecho e ictericia. La paciente refiere haber tenido múltiples
episodios de ictericia durante los últimos 10 años. Los exámenes para determinar
hepatitis viral dieron positivos para hepatitis B, siendo catalogado el caso como hepatitis
crónica con alteración funcional. En un examen de sangre ¿cuál de los siguientes
parámetros está disminuido?

a. Albúmina

b. Bilirrubina

c. Fosfatasa alcalina

d. Tiempo de protrombina

8. En el reflejo peristáltico del intestino delgado ¿cuál de los siguientes eventos sucede en
la porción caudal del bolo alimenticio?

a. Acción del péptido inhibidor vasoactivo (VIP) en el músculo circular


b. Acción del NO (óxido nítrico) en el músculo longitudinal)

c. Contracción del músculo longitudinal interno

d. Acción de acetilcolina en el músculo circular

9. Varón de 58 años de edad con enfermedad de Crohn severa fue sometido a una
resección ileal. Después de la cirugía este paciente padecerá de esteatorrea esto se
explica porque:

a. Se inhibe la acción de la 7 alfa hidroxilasa

b. El pool de ácidos biliares se incrementa

c. Hay malabsorción de ácidos biliares

d. El páncreas no secreta lipasa

10. En un experimento se inserta un balón en el estómago de un voluntario, se infla poco a


poco mientras que se vigilan las presiones intraluminales. Aunque el volumen del balón
aumenta considerablemente, las presiones permanecen constantes. Esta relación
volumen presión se explica por la liberación local de

a. Acetilcolina y gastrina

b. Norepinefrina y óxido nítrico

c. Colecistoquinina y óxido nítrico

d. Óxido nítrico y péptido inhibidor vasoactivo

11. ¿cuál de las siguientes alternativas es una característica de la secreción exocrina del
páncreas?

a. Es hipotónica respecto al plasma

b. Su mayor estímulo se da en la fase intestinal

c. Es estimulada por la presencia de bicarbonato en el duodeno

d. La secreción enzimática es estimulada principalmente por la secretina

12. Las estructuras en el hígado que permite que los productos metabólicos unidos a
proteínas tengan acceso a las membranas basolaterales de los hepatocitos, son:

a. Los canalículos

b. Las células de ito

c. Las fenestras sinusoidales

d. Las uniones intercelulares herméticas


13. La composición de la bilis es modificada conforme fluye por los conductillos biliares.
Durante este tránsito se espera que aumente la concentración de

a. Ig A

b. Glucosa

c. Protones

d. Vitamina A

14. Se mide experimentalmente el contenido gástrico de dos personas. La persona “A”


tiene alto contenido de grasa y la persona “B” tiene un contenido isotónico ¿Cuál de las
siguientes es correcta respecto al vaciamiento gástrico?

a. Hay ralentización del vaciado gástrico sólo en A”

b. El vaciamiento gástrico es más rápido en ambos

c. Hay ralentización del vaciado gástrico sólo en “B”

d. Hay ralentización del vaciado gástrico en ambos casos

15. El examen endoscópico de un paciente con hipertensión portal grave revela venas
tortuosas que sobresalen hacia la luz del esófago. El paciente recibe tratamiento
quirúrgico mediante la colocación de una derivación que conecta la vena porta a la vena
cava. Después de la operación el riesgo de encefalopatía _______ y el riesgo de
sangrado de várices ______

a. Disminuirá/ disminuirá

b. Disminuirá/ aumentará

c. Aumentará/ disminuirá

d. Aumentará/ aumentará

16. Un paciente varón de 18 años de edad acude al médico para sus exámenes de
rutina. Sus resultados de laboratorio muestran un valor de bilirrubina séricas de 4 mg/dl
y una bilirrubina directa de 0,3 mg/dl. Las pruebas de función hepática son normales. La
alteración que explica mejor este caso es por la deficiencia de

a. Transaminasas

b. Hemo oxigenasa

c. La 7 alfa hidroxilasa

d. Glucoronil transferasa

17. Un hombre de 57 años de edad es llevado a urgencias con hematemesis masiva


roja brillante, a su llegada se halla inconsciente con PA: 80/40 mm Hg y FC:124 lat/min.
Luce ictérico con presencia de “arañas vasculares en el tórax anterior y extremidades”,
abdomen distendido con signo de oleada positiva. Se encuentra esplenomegalia y
pérdida de la masa muscular en extremidades. La anastomosis vascular responsable del
sangrado en este paciente es:

a. Vena gástrica izquierda y vena ácigos

b. Arteria gástrica izquierda y vena ácigos

c. Vena paraumbilical y vena epigástrica inferior

d. Vena gástrica izquierda y vena esofágica superior

18. Un estudiante de medicina está comiendo un plato de comida a base de


champiñones, espárragos y salsa de soya. El estímulo del sabor umami contenido en
todos estos alimentos viaja a través del nervio:

a. Lingual

b. Hipogloso

c. Glosofaríngeo

d. Cuerda del tímpano

19. Una paciente de 30 años de edad es sometida a una cirugía en lado medio derecho
por un problema de aterosclerosis. Luego de la cirugía refiere alteración sensitiva de la
lengua. Al evaluar el caso usted esperaría encontrar

a. Alteración en la sensación del dolor y temperatura

b. Alteración en la sensación del dolor en los dos tercios anteriores de la lengua

c. Alteración en la sensación del gusto en el tercio posterior de la lengua

d. Sensación del dolor, tacto y temperatura conservadas

20) En un paciente de 45 años de edad con colestasis biliar se encuentra una elevación de
los niveles sanguíneos de fosfatasa alcalina hasta 3 veces la cifra normal ¿Cuál de las
siguientes alternativas estará también elevada como evidencia del daño de la vía biliar?

a. Tiempo de protrombina y albúmina sérica

b. Transaminasas hepáticas (ALT y AST)

c. Gamma glutamil transpeptidasa

d. Glucoronil transferasa

21) Experimentalmente se incrementa la velocidad de se incrementa la velocidad de


secreción salival con una sustancia, en el análisis de la composición de esta saliva obtenida
se espera encontrar ………..
a. Disminución de la concentración de bicarbonato que supera la concentración
plasmática
b. Aumento de la concentración de cloro y sodio que supera la concentración
plasmática
c. Aumento de la concentración de bicarbonato que supera la concentración
plasmática
d. Disminución de la concentración de potasio y bicarbonato

22) Lactante de 3 meses de vida es atendido por presentar diarrea, se administra una
solución de glucosa y electrolitos por vía oral. La proteína de membrana apical que explica
la capacidad de esta solución para proporcionar aporte de glucosa e hidratación en …..

a. CFTR

b. SGLT-1

c. GLUT-2

d. GLUT-5

23) Paciente ha sufrido herida de bala en el abdomen, se le ha tenido que extirpar el


segmento medio y distal del íleon. En este caso la síntesis hepática de sales biliares estará
……

a. Sin cambios en el ritmo de síntesis

b. Disminuida por inhibición de la enzima colesterol 7 alfa hidroxilasa

c. Incrementada por estímulo de la enzima colesterol 7 alfa hidroxilasa

d. Incrementada por inhibición de la enzima colesterol 7 alfa hidroxilasa

24) Un varón de 75 años ingresa al consultorio por presentar ictericia marcada de piel y las
escaleras. El estudio del paciente mostró que presentaba un tumor que obstruía la totalidad
del conducto hepático común. ¿Cuál de los siguientes conductos se encontrará dilatado en
este paciente

a. de Wirsung

b. de Hering

c. colédoco

d. cístico

25) Correlaciones las dos columnas y marque la fórmula correcta:

1. Enfermedad Hirschsprung ( ) heces cono moco y sangre

2. Diarrea osmótica ( ) intolerancia a lactosa

3. Diarrea secretoria ( ) aganglionosis congénita


4. Diarrea exudativa ( ) canales de Cl- en las células de la cripta

a. 4231 b. 1234 c. 2143 d. 4213

26) La vena umbilical obliterada del hígado después del nacimiento se transforma en el
ligamento

a. Cruzado

b. Redondo

c. Coronario

d. Falciforme

27) Llega a su guardia nocturna una madre que trae a su RN masculino de 2 semanas de
vida con mal estado general y resequedad de mucosas. Usted observa que lacta
ávidamente, pero a las 2 horas presenta vomito postprandial no bilioso en proyectil. Al
realizar la historia clínica descubre que el lactante recibió profilaxis con macrólidos para tos
ferina. Usted sospecha principalmente en:

a. Estenosis pilórica hipertrófica congénita

b. Fistula traqueo esofágica

c. Estenosis duodenal

d. Atresia duodenal

28) En la regulación del apetito y la saciedad, la estimulación experimental crónica del


núcleo ventro medial del hipotálamo producirá:

a. Afagia

b. Obesidad

c. Hiperfagia

d. Activación de neuronas relacionadas a NPY

29) Paciente mujer de 25 años acude por dolor en fosa iliaca derecha que empeora al toser
o caminar, asociada a náuseas y vómitos por lo cual acude a emergencia, Dos días después
de realizarle un apendicetomía, la paciente desarrolla fiebre alta (39º C), esta hipotensa y
presenta dolor abdominal. La laparotomía exploratoria muestra un gran volumen de sangre
en la cavidad peritoneal por lesión de un vaso producida durante la apendicetomía ¿Cuál de
las siguientes arterias debe ligarse para detener la hemorragia?

a. Cólica derecha y arteria rectal superior

b. Ileocólica y arteria cólica media

c. Mesentérica superior
d. Ileocólica

30) La onda peristáltica secundaria del esófago se caracteriza por ser originada

a. Por el plexo de Meissner del esófago

b. Por el plexo mesentérico del esófago

c. Por el reflejo de la deglución

d. Durante la masticación

31) ¿Cuál de los siguientes es una causa de ictericia con bilirrubina conjugada aumentada?

a. Ictericia del recién nacido

b. Obstrucción del colédoco

c. Anemia hemolítica

d. Gran hematoma

32) En relación a la absorción de nutrientes, la absorción de dipéptidos y tripéptidos a nivel


de las células epiteliales del intestino delgado, se da principalmente debido a:

a. Incremente de canales de Cl en la membrana apical

b. La gradiente de bicarbonato en la membrana basal

c. La gradiente de iones H+ en la membrana apical

d. La gradiente de Na+ en la membrana apical

33) Paciente de 20 años es traído a la emergencia por presentar diarreas desde hace 2
días. Familiar refiere que las deposiciones son liquidas y abundantes, al examen luce
deshidratado y se plantea que la diarrea es producida por una toxina que estimula la
transformación de ATP a AMPc con apertura de cales de Cl- y perdida de agua. El tipo de
diarrea más probable es:

a. Osmótica

b. Exudativa

c. Secretoria

d. Por intolerancia a la lactosa

34) Un niño fue operado por una obstrucción intestinal, observándose la presencia de
divertículo de Meckel. Según lo diferido marque lo correcto:

a. El 50% de la población lo presenta

b. Se localiza en el íleon muy cerca al yeyuno


c. Puede poseer tejido gástrico o pancreático

d. Se produce por una mala rotación de los intestinos

35) Marque la alternativa correcta respecto a la estructura marcada en el gráfico:

a. Se halla a 2 centímetros debajo de la papila duodenal mayor

b. Llega el conducto colédoco y pancreático principal

c. Llega el conducto hepático común y pancreático principal

d. Llega el conducto pancreático accesorio

36) ¿Cuál de las siguientes moléculas se encontrará aumentada en el citoplasma de las


células parietales de un paciente con un síndrome de Zollinguer Ellison?

a. Péptido liberador de gastrina (GRP)

b. Proteína G estimulante (GS)

c. Inositol Trifosfato (IP3)

d. AMO cíclico (AMPc)

37) Los fármacos inhibidores de la bomba de protones, actúan bloqueando la …………

a. Anhidrasa carbónica

b. ATPasa H+/K+ en la membrana luminal

c. ATPasa H+/K+ en la membrana basolateral

d. ATPasa Na+/K+ en la membrana basolateral

38) Un paciente fue diagnosticado de gastritis autoinmune, ¿cuál de las siguientes


alternativas es FALSA respecto a esta enfermedad

a. Afecta principalmente el fondo y cuerpo gástrico

b. Se produce hiperplasia de células G secundaria la aclorhidria

c. El propio sistema inmune destruye principalmente las células parietales


d. Se produce atrofia de la mucosa, aclorhidria, hipergastrinemia y déficit de vitamina
B6

39) Marque la correlación correcta:

1. Herpes virus ( ) en relación al abuso de antibióticos

2. Candidiasis oral ( ) lesiones vesiculares como racimo de uvas

3. Eritroplaquia ( ) Mega esófago

4. Enfermedad de Chagas ( ) Lesión precancerigena

a.- 2431 b. 1234 c. 4123 d. 2143

40) En un paciente con insuficiencia renal crónica, el déficit en la absorción de calcio a nivel
del enterocito se debe a lo siguiente:

a. No se convierte la 25 hidroxicolecalciferol a 1,25 dihidroxicolecalciferol

b. No se convierte la 1,25 dihidroxicolecalciferol a 25 hidroxicolecalciferol

c. Existe un descenso de la alfa 25 hidroxilasa renald. Se incrementa la producción


de Calbindina

41. Marque lo correcto en relación al divertículo de Meckel: Se encuentra usualmente a 60 cm de la


VIC

42. Luego de tres horas dando exámenes, un alumno de medicina comienza a sentir
hambre. Esta situación es probable que sea mediada por la _____ que es sintetizada por el
_____ grelina / estómago

43. Varón de 72 años, con antecedente de diabetes mellitus tipo 2, que presenta entropía
diabética caracterizada por estreñimiento. Este problema puede estar asociado a:
deficiencia de óxido nítrico

44. Varón de 54 años con diabetes mellitus tipo 2, es diagnosticado de gastroparesia debido
a que presenta sensación de llenura precoz al comer, y reflujo gastroesofágico. Esta
alteración en la relajación receptiva y en el vaciamiento gástrico lo más probable es que se
deba a una alteración en: el nervio vago

45. Lactante masculino de 5 meses de edad producto de un primer embarazo normal


controlado . Peso de nacimiento 3.120 g y talla de 51 cm . Lactancia materna satisfactoria,
con buen incremento ponderal . Sin antecedentes patológicos hasta 5 días atrás , cuando
comienza con vómitos posprandiales no biliosos , lácteos . Los síntomas aumentan en
frecuencia y magnitud hasta hacerse explosivos después de cada alimentación . No requiere
tos , fiebre , diarrea ni lesiones cutáneas .Es importante destacar que pese a los vómitos el
niño conserva el apetito y llora de hambre .Al examen físico presenta buen estado general.
Abdomen blando ,depresible e indoloro ,asociado a distensión de hemiabdomen superior
.Sin signos de deshidratación .No se palpan masas abdominales .Exámenes de
laboratorio:hemograma normal. Signos inflamatorios de fase aguda negativos .Alcalosis
metabólica leve en sangre venosa.

Producto de la hipertrofia pilórica uno esperaría que los niveles de gastrina se encuentren: .

Rpta}: ELEVADA

46. Lactante masculino de 5 meses de edad producto de un primer embarazo normal


controlado . Peso de nacimiento 3.120 g y talla de 51 cm . Lactancia materna satisfactoria,
con buen incremento ponderal . Sin antecedentes patológicos hasta 5 días atrás , cuando
comienza con vómitos posprandiales no biliosos , lácteos . Los síntomas aumentan en
frecuencia y magnitud hasta hacerse explosivos después de cada alimentación . No requiere
tos , fiebre , diarrea ni lesiones cutáneas .Es importante destacar que pese a los vómitos el
niño conserva el apetito y llora de hambre .Al examen físico presenta buen estado general.
Abdomen blando ,depresible e indoloro ,asociado a distensión de hemiabdomen superior
.Sin signos de deshidratación .No se palpan masas abdominales Exámenes de
laboratorio:hemograma normal. Signos inflamatorios de fase aguda negativos .Alcalosis
metabólica leve en sangre venosa.

En este paciente se puede esperar una mayor liberacion de :

Rpta: Enzimas pancreáticas

47. Lactante masculino de 5 meses de edad producto de un primer embarazo normal


controlado . Peso de nacimiento 3.120 g y talla de 51 cm . Lactancia materna satisfactoria,
con buen incremento ponderal . Sin antecedentes patológicos hasta 5 días atrás , cuando
comienza con vómitos posprandiales no biliosos , lácteos . Los síntomas aumentan en
frecuencia y magnitud hasta hacerse explosivos después de cada alimentación . No requiere
tos , fiebre , diarrea ni lesiones cutáneas .Es importante destacar que pese a los vómitos el
niño conserva el apetito y llora de hambre .Al examen físico presenta buen estado general.
Abdomen blando ,depresible e indoloro ,asociado a distensión de hemiabdomen superior
.Sin signos de deshidratación .No se palpan masas abdominales Exámenes de
laboratorio:hemograma normal. Signos inflamatorios de fase aguda negativos .Alcalosis
metabólica leve en sangre venosa.

En relación con la estructura afectada se encuentra:

Rpta: L1

48.Varón de 67 años con tos y disminución de peso asociado a tabaquismo pesado,


presenta actualmente disfagia progresiva a alimentos sólidos. Se considera la presencia de
un carcinoma de bronquio izquierdo y por esta razón le realizan una endoscopia esofágica
para descartar la posibilidad de una compresión esofágica por el tumor. Se espera revisar el
esófago en la _____ estrechez, que está a nivel de la vértebra _____.

Tercera estrechez - T6
49. En un niño menor de dos años con divertículo intestinal, este divertículo tiene su origen
en una falla en la obliteración de:

Conducto vitelino

50. Mujer de 43 años sufre un grave accidente de tránsito y está hospitalizada en coma, es
alimentada por vía intravenosa durante varias semanas. Producto de este tipo de
alimentación, se encuentra en la endoscopia atrofia de la mucosa gastrointestinal. La causa
más probable de esta atrofia son los bajos niveles séricos de la hormona:

Gastrina

51. Una mujer de 30 años llega al consultorio porque se queja de dificultades para deglutir,
la cual se agravan cada vez más. Se realiza un estudio manométrico para examinar la
generación de presión a lo largo del esófago. Esta prueba revela que las contracciones
como respuesta a la deglución están mal sincronizadas y que la presión en el esfínter
esofágico inferior permanece elevada. El diagnóstico más probable es _____ producido por
niveles bajos de _____:

acalasia / óxido nítrico

52. Paciente de 2 años, llega a emergencia por haber ingerido una moneda con la que
estaba jugando. El lugar más probable donde puede haberse quedado suspendido este
objeto es a nivel del estrechamiento producido a nivel del:

músculo cricofaríngeo

53. En una apendicectomía, al realizar la incisión de McBurney en la fosa iliaca derecha, es


necesario cortar los siguientes músculos, de afuera hacia adentro:

Oblicuo externo - Oblicuo interno - Transverso

54.Al consumir caramelos indirectamente activa la via:

Rpta: POMC/CART

55. Debido al uso de ranitidina , los valores de somatostatina en sangre:

Rpta: Disminuyen

56.En múltiples causas de la enfermedad por reflujo gastroesofágico, se puede considerar


tambien una alteración en las ___________ del esfínter esofágico inferior

Rpta: contracciones tonicas

57. Estudiante de medicina de la UPC de 21 años sufre de gastritis aguda ocasionada por
comer en lugares poco higiénicos. Suele consumir caramelos ( chupa ) mientras está en
clase hasta la tarde. Toma gaseosa regularmente (carbohidratos 46%, sodio 53%). También
toma regular cantidad de leche (grasa 35%, lactosa 35%, proteínas 30%), pues le calma un
poco el dolor el ardor que siente por la gastritis. Incluso, cuando puede, se toma dos vasos
de agua fría para calmar las molestias. Ha decidido ir al médico para tratarse pues ya no
soporta el dolor, el cual está seguro que los síntomas se deben a una elevada producción
de ácido clorhídrico en el estómago, y por ello le ha recetado Ranitidina (antihistamínico),
con lo que siente mejoría.

El consumo de una pequeña cantidad de gaseosa aumenta directamente la contracción


sérica de cual de las siguientes hormonas.

Rpta: Peptido 1 similar al glucagon(GLP-1)

58. Paciente es evaluado por faringitis aguda en consultorio externo. El médico de familia le
solicita que abra la boca y saque la lengua Para realizar la acción de sacar la lengua, es
necesario que se contraiga el músculo: geniogloso

59. en qué casos los vómitos son siempre biliosos

Rpta: atresia yeyunal

60. Durante una cirugía oncológica¿La extirpación de cuál de los siguientes órganos se
vería comprometida por la presencia de adventicia.

Rpta: recto

61. Estimulan la secreción ácido gástrica

Rpta: Proteínas

62. Cual de los líquidos corporales tiene el PH más alto

Rpta: Jugo pancreatico

63. En un paciente de 43 años con tumor carcinoide de páncreas productor de gastrina


(Sindrome de Zollinger-Ellison) se puede esperar una disminución en la:

Rpta: liberación de gastrina por la células G

64. El ligamento de Treitz característicamente:

Rpta: Suspende el ángulo de Treitz

65. Paciente de sexo masculino de 43 años que es alimentado por via iintravenosa durante
varias semanas. Producto de este tipo de alimentación se encuentra en la endoscopia
atrofia de la mucosa antral. la causa mas probable de esta alteración es debido a los bajos
niveles séricos de que hormona:

Rpta. GASTRINA

66. Con respecto al control autonómico en el tracto gastrointestinal y en relación con su fisiología
¿Cuál es la función del sistema nervioso parasimpático y el tracto gastrointestinal?

Estimulan la contracción muscular y estimulan la secreción de sustancias a nivel de la mucosa


Intersticio ubicado entre el estroma del espacio portal y los hepatocitos, y por donde migran las
células cancerígenas que hacen diseminación linfática es el:

Espacio del Mall

La presencia de grandes cantidades de TGF - Beta estimula a las ___________ y se deposita


colágeno, formándose la cirrosis

Células de Ito

Paciente de 64 años con ICC al que se le va a realizar cirugía cardiovascular. Al calcular el volumen
sanguíneo total, se debe considerar que el hígado puede contener un volumen de sangre de
____________ mL en un adulto sano, en este paciente ese volumen puede llegar a ser de
_____________ mL

450 - 1000

El área del lobulillo que se afecta más en caso de hipoxia es la zona:

Paciente con carcinoma de vesícula biliar. La metástasis por continuidad afectará al lóbulo:

Cuadrado

El ácido acetilsalicílico actúa en la membrana:

Basolateral de la célula parietal

Al realizar una esplenectomía, se tiene que resecar la arteria esplénica, lo cual no es problema para
el estómago por que la arteria gastroomental izquierda se anastomosa con la:

Gastroomental derecha

La arteria esplénica proviene de la aorta y la vena esplénica desemboca en la vena:

Porta

Paciente mujer 21 con bulimia, que luego de un episodio de vómitos presenta hematemesis y al
examen físico que se encuentra crépitos subcutáneos cervicales. El diagnóstico más probable es:

Sd. Boerhaave
Paciente varón de 60 años, con antecedentes de promiscuidad sexual, tabaquismo y alcoholismo,
acude a consulta por presentar disfagia progresiva, odinofagia y al examen se observa tumor por
parte posterior de la lengua. La mejor posibilidad diagnóstica es:

Carcinoma escamoso

Niño de 5 años con historia de tres días de evolución caracterizado por fiebre, malestar general,
odinofagia, anorexia, e irritabilidad. Al examen de observa lesiones ulcerativas de 4mm de diámetro
en mucosa yugal, con borde blanquecino y eritema periférico. El diagnóstico más probable es:

Aftas orales

Es considerada una lesión preneoplásica

Leucoplasia

El esófago de Barrett se considera una lesión preneoplásica que se caracteriza por la presencia en
esófago de:

Metaplasia intestinal

Paciente mujer de 23 años gestante con lesión proliferativa en mucosa oral producida por
proliferación reactiva de vasos sanguíneos. Marque la mejor respuesta

Epulis

La glándula parótida tiene principalmente acinos de tipo:

Seroso

La reabsorción de sodio y secreción de potasio es estimulada por:

Aldosterona

Paciente con cirrosis hepática que tiene hipertensión portal con várices esofágica, y actualmente
presenta varices en estómago distal. Estas várices están relacionadas a aumento en la presión de las
venas:

Gástrica derecha
La glándula parótida está inervada por el par craneal:

IX

Paciente con cirrosis hepática que tiene hipertensión portal con varices en esofago distal . Estas
varices están relacionadas a aumento en la presión de las venas gástricas

Inferior

La información eferente que sale de los núcleos salivales superior e inferior a través de los pares VII y
IX hacia las glándulas salivales llevan información tipo

Parasimpático

Un niño de 4 años ingresa en el hospital con vómitos graves . En el estudio se encuentra que el niño
tiene un páncreas anular ¿ Cuál de las siguientes hormonas gastrointestinales se encontrará a
niveles elevados en sangre con mayor probabilidad a raíz de esta patología

Gastrina

Durante una colecistectomía laparoscópica en un hombre de 61 años ¿ Cuál de las siguientes


arterias debe pinzar para extirpar la vesícula biliar con seguridad?

Cistica

Un hombre de 34 años se somete a una apendicectomía de urgencia . Después de realizar la


apendicectomía satisfactoriamente , el paciente se somete a una laparoscopia exploratoria¿Cual de
las siguientes características anatómicas es más útil para distinguir entre yeyuno e íleon?

El yeyuno tiene menos grasa mesentérica que el íleon

Una mujer de 45 años ingresa en el hospital con síntomas de obstrucción intestinal superior .En la TC
se encuentra que la tercera porción (transversa) del duodeno está comprimida por un gran vaso
¿Cuál de los siguientes vasos causara muy probablemente la obstrucción ?

Arteria mesentérica superior


Durante una colecistectomía laparoscópica programada en una mujer de 47 años , el residente
pinchó accidentalmente el ligamento hepatoduodenal en vez de la arteria cística ¿Cual de los
siguientes vasos estaría muy probablemente ocluido en esta lesión iatrogénica?

Arteria hepática izquierda

Un hombre de 54 años ingresa en urgencias con intenso dolor abdominal superior . La gastroscopia
revela un tumor en el antro del estómago . Se pide una TC para evaluar el drenaje linfático del
estómago¿Cuál de los siguientes nódulos linfáticos estará muy probablemente afectado en una
neoplasia maligna del estómago

Mesentérico inferior

Una mujer obesa de 45 años con fiebre alta acude a la consulta con náuseas y dolor agudo e
intermitente en el cuadrante superior derecho del abdomen de 2 días de duración - Tiene una historia
de ictericia de 24 horas. Tiene antecedentes de litiasis biliar. Bilirrubina total del 10 mg/dL. Lipasa de
5 mg/mL. ¿Cuál de las siguientes estructuras está muy probablemente obstruida por un cálculo biliar?

Ampolla de Vater

Una mujer de 45 años ingresa en urgencias con dolor abdominal intenso . La TC y RM revelan un
tumor de la cabeza del páncreas que afecta el proceso unciforme ¿Cual de los siguientes vasos es
más probable que suministre irrigación a parte de la zona afectada?

Arteria cólica media

Un hombre de 70 años ingresa en urgencias con diarrea intensa . La arteriografía revela un bloqueo
del 90% en el origen aórtico de la arteria mesentérica inferior ¿Cuál de las siguientes arterias
proporciona muy probablemente irrigación colateral al colon descendente?

Arteria sigmoidea

Al disminuir el pH duodenal por el HCL gástrico , se libera principalmente una hormona cuya célula
diana es

Célula ductal de Wirsung

Una de las siguientes sustancias reguladoras , puede actuar de forma paracrina y como hormona

Somatostatina
Al ingerir grandes cantidades de dulces , con la subsecuente estimulación de incretinas , usted
esperaría que el apetito

Disminuya por insulina

Paciente obeso con Covid-19 es intubado por interno inexperto, quien al solicitar que bombeen aire
dentro del tubo endotraqueal, nota que el epigastrio se distiende. Al sospechar que ha introducido el
tubo en el estómago, también es cierto que:

Disminuye el pH gástrico

La hormona que tiene un efecto sinérgico con la secretina para optimizar el pH duodenal y la
digestión, es:

CCK

El ecografista sabe que para poder visualizar el nacimiento de la arteria mesentérica superior , debe
colocar el transductor sobre la piel de la siguiente región abdominal

Epigastrio

En un paciente con hiperestimulación simpática se espera que las ondas lentas tengan un ritmo

Mayor en estómago que en el duodeno

Mayor en estómago que en el íleon terminal

Menor en íleon terminal que en el duodeno

Mayor en el íleon que en duodeno

Se considera que el gusto puede viajar a través del nervio

Glosofaríngeo

La rotación en sentido longitudinal del estómago en el desarrollo embriológico condiciona que el


nervio vago derecho quede a nivel

Posterior

Con respecto al control autonómico en el tracto gastrointestinal y en relación con su fisiología. ¿Cuál
es la función del sistema nervioso parasimpático en el tracto gastrointestinal?
Estimulan la contracción muscular y estimulan la secreción de sustancias a nivel de la mucosa.

Un estudiante que está preocupado por su examen parcial, no ha desayunado ni almorzado; cuando
al fin ingiere alimentos, este le provoca el aumento de los movimientos musculares del tracto
gastrointestinal y la sensación de defecar.?¿Qué reflejo se ha activado?

gastro - cólico

¿De que par craneal es rama el nervio palatino mayor?

Trigémino

¿En cuál de las fases de deglución la epiglotis separa la vía respiratoria de la digestiva?

Faríngea

Los péptidos intestinales se pueden clasificar como sustancias endocrinas, neurocrinas y paracrinas,
dentro de las paracrinas se encuentran la somatostatina e histamina. Marque la respuesta correcta

La histamina es sintetizada por células de tipo paracrino de las glándulas gástricas

Paciente varón de 27 años es llevado por bomberos a emergencia luego de ser asaltado y, tras
resistirse, es cortado con el pico de una botella a nivel abdominal. Al examen físico usted observa
que a través de la herida se puede observar la protrusión de las asas intestinales. En relación con las
capas de la pared abdominal, marque la alternativa correcta.

El músculo recto del abdomen tiene como funciones comprimir el contenido del abdomen, tensar la
pared abdominal y flexionar la columna

Los músculos del tracto gastrointestinal de los segmentos propulsivo y receptor del bolo alimenticio,
responden de forma diferente al movimiento de este bolo a través del intestino. ¿Cuál de las
siguientes afirmaciones describe correctamente la actividad del segmento propulsivo?

Tanto el músculo circular como el longitudinal están relajados

El músculo longitudinal está relajado y el circular esta contraído

Tanto el músculo circular como el longitudinal están contraídos

El músculo longitudinal está contraído y el circular está relajado

El nervio vago inerva al músculo

Estriado del esófago


¿Cuál de las siguientes es una característica de los ganglios mioentéricos del sistema nervioso
entérico?

Contiene mayor número de neuronas que el plexo submucoso

Es también conocido como el plexo de Meissner

Contiene sólo neuronas motoras excitatorias del músculo liso

Contiene neuronas sensitivas que activan a los músculos circular y longitudinal del tracto intestinal

El divertículo faringoesofágico, hipofaríngeo de zenker, es una lesión muy particular que se localiza
en la cara póstero lateral de la Unión de la faringe con el esófago, como una herniación de la mucosa
esofágica a través de las fibras oblicuas del músculo.

Constrictor inferior de la faringe

La razón por la que el potencial de acción viaja rápidamente en sentido longitudinal por el músculo
liso gastrointestinal es la presencia de uniones en hendidura,

Varicosidades

Respecto a los péptidos gastrointestinales, marque lo correcto.

las sustancias paracrinas pueden viajar a través de vasos sanguíneos

Al ingerir rápidamente un litro de agua, usted esperaría que la gastrina aumenta por efecto de:

ACh del sistema mientérico

Respecto a la anatomia del estomago,marque lo correcto:

El fondo gástrico forma la curvatura mayor

(Pág 19 → 21)

Los nervios esplácnicos lumbares (L1-L2) llevan información de tipo:

Simpática
Paciente de 32 años con herida por arma de fuego y shock hipovolémico. El intestino delgado no se
ha infartado aún a pesar de la hipoxia gracias a la liberación de:

Adenosina

Al realizarse un piercing en el ombligo,la sensación de dolor se transmite por:

T10

El nivel en el que se encuentra el píloro y el páncreas se puede determinar usando el:

Píloro transpilórico

Permite la suspensión e irrigación de los órganos peritoneales:

Mesenterio

El dolor asociado a apendicitis clásicamente se ubica en:

Fosa iliaca derecha

El ligamento inguinales formado por la aponeurosis del:

Oblicuo externo

Los nueve cuadrantes del abdomen se delinean usando el plano subcostal,las líneas
medioclaviculares y:

Plano intertubercular

Es un órgano peritoneal:

Hígado

La línea alba se encuentra:

Entre los rectos abdominales

Es un órgano retroperitoneal:

Páncreas
Enfermedad asociada con un error en el desarrollo de las células de Cajal

Enfermedad de Hirschsprung

El divertículo de Meckel es un rezago de:

Conducto vitelino

La fístula rectoperitoneal es causada por una falta en el desarrollo de:

Tabique urorrectal

95) Al disminuir el pH duodenal por el HCl gástrico, se libera principalmente una hormona cuya célula
diana es:

Célula ductal del Wirsung

Al introducir una solución azucarada directamente al estómago mediante una gastrostomía


(comunicación entre la piel abdominal y el estómago), la sustancia que provocará que aumenten los
niveles séricos de insulina es:

Péptido tipo glucagón 1 (GLP-1)

La rotación en sentido longitudinal del estómago en el desarrollo embriológico condiciona que el


nervio vago derecho quede a nivel:

Posterior

Una de las siguientes sustancias no comparte con las otras la misma acción sobre la producción de
ácido gástrico:

Colecistoquinina

En un paciente con gastroparesia (motilidad lenta del estómago), que presenta distensión abdominal
después de comer, usted le recomendaría que evite el consumo de lípidos y aminoácidos para
disminuir la acción de:

CCK
En un paciente con shock distributivo, usted decide iniciar noradrenalina por un catéter CVC,
consiguiendo aumentar la presión arterial. ¿qué efecto sobre la motilidad intestinal esperaría
encontrar?

El potencial de reposo de las fibras musculares se hace más negativo

El ecografista sabe que para poder visualizar el nacimiento de la arteria mesentérica superior, debe
colocar el transductor sobre la piel de la siguiente región abdominal:

Epigastrio

105) La glándula submandibular recibe inervación traída por el nervio

Cuerda del tímpano

A mayor flujo de saliva, disminuye la concentración de:

Potasio

107) En términos de mg/mL, el principal componente de la saliva es _____ seguido de ____:

Proteínas - Potasio

108) La reabsorción de Sodio y Cloro en las glándulas salivales se da principalmente en el:

Conducto estriado

109) La información eferente que sale de los núcleos salivales superior e inferior a través de los pares
VII y IX hacia las glándulas salivales llevan información de tipo:

Parasimpático

La glándula submaxilar le hace gancho al:

Músculo milohioideo

Los conductos salivales son ___ al agua, esa es una de las razones por las cuales la saliva es
siempre ___.

Impermeables - hipotónica
Los nervios esplácnicos pélvicos (S2-S4) llevan información de tipo:

Parasimpática

113) Presenta movimientos en masa:

Colon

114) La digestión de los lípidos se inicia en:

Intestino delgado

115) La digestión de los carbohidratos se inicia en:

Cavidad oral

117) El estómago recibe información simpática proveniente del:

Ganglio celiaco

118) En el intestino delgado se absorbe los carbohidratos en forma de:

Fructosa

119) Es rama de la arteria mesentérica superior:

A. cólica media

120) El “dolor de estómago” asociado a gastritis se suele ubicar en:

Epigastrio

121) El azúcar de mesa sacarosa es digerido a dos monosacáridos que comparten el transportador:

GLUT 2

122) Durante la defecación se requiere:

Señales inhibitorias en el nervio pudendo


123) En un paciente con fístulas intestinales y fisuras anales, con antecedente de enfermedad
inflamatoria intestinal, ud sospecharia en:

Enfermedad de Crohn

En los pacientes con Colecistitis Aguda no operable, una opción es la colocación de una sonda por el
cístico, procedimiento en el que se ingresa con dificultad debido a la estrechez del cístico y a la
presencia de:

Válvula de Herring

Paciente de 51 años con antecedente de enfermedad diverticular acude a emergencia por sangrado
profuso y dolor en hipocondrio izquierdo. ¿Cuál es el origen más probable de la sangre que pierde el
paciente ?

Mesentérica inferior

Paciente de 42 años con dolor abdominal intenso y hematemesis. En la endoscopia se observa una
úlcera duodenal posterior perforada con hemorragia intraabdominal. ¿Cuál de las siguientes arterias
estará comprometida?

Pancreaticoduodenal posterosuperior

Para que los triglicéridos sean absorbidos deben ser metabolizados a

Monoglicéridos y Ácidos grasos

El vibrio cholerae produce diarrea porque:

Aumenta la producción de AMPc en los enterocitos

La motilidad intestinal es estimulada por

Colecistoquinina y gastrina

La diarrea por deficiencia de lactasa es de tipo

osmótica
La metoclopramida estimula el vaciamiento gástrico aumentando la fuerza de contracción de las
paredes gástricas esto puede conseguirlo mediante la estimulación indirecta de las neuronas
liberadoras de

Acetilcolina

Los vértices de un acino hepático están constituidos por

Dos espacios porta y dos venas centrolobulillares

Al ingerir un pedazo de mantequilla, cuál de los siguientes tiene un efecto directo en la reducción del
vaciado gástrico:

Colecistoquinina

Tras la vagotomía (resercion del vago) por enfermedad úlcera péptica en un paciente UD. esperaría
encontrar:

Aumento del pH gástrico

Se realizó un experimento en el cual se inyectó tinta china en el peritoneo de ratas de laboratorio. Al


realizarse una biopsia hepatica de dichos animales, se encontro que el tinte negro fue fagocitado por:

Células de kupffer

En un paciente con pH gástrico muy bajo, es posible que la siguiente sustancia se secrete en menor
cantidad:

Gastrina

La fase cefálica de la secreción gástrica responde por cerca del 30% de la respuesta ácida a un
reflejo con la _____________ se elimina la fase cefálica de la secreción gástrica

Vaguectomía

El esofago de barret se caracteriza por presentar _________ en el esofago

Metaplasia intestinal
La presencia de orina que sale por el ombligo de un recien nacido cada vez que llora, es posible que
deba a un defecto en el desarrollo de:

Seno urogenital

El conducto biliar deriva del:

Endodermo

Cuál de los siguientes órganos son intraperitoneales:

Estómago, Vesícula biliar, Ileón, Hígado

Los vasos mesentéricos superiores se hallan a nivel de:

Cuello del páncreas

La colecistoquinina (CCK) inhibe:

El vaciamiento gástrico

Un niño de 2 años es llevado a la consulta por diarrea persistente, edema de las extremidades y falta
de crecimiento en relación a su edad. Los análisis de sangre revelan que tiene concentración
plasmática baja de proteínas (hipoproteinemia). Como parte del estudio se coloca Colecistoquinina
(CCK) endovenosa y se recoge muestras del líquido duodenal por endoscopia; el resultado del líquido
confirma incapacidad para hidrolizar proteínas a un pH neutro, esta situación mejora al añadir una
pequeña cantidad de tripsina. El paciente probablemente esté sufriendo la falta congénita de ……….

Enterocinasa

Paciente mujer de 35 años acude a consulta por sensación de sequedad y lesiones en la cavidad
oral. Al examen se observa atrofia de la mucosa, fisura y úlceras; nota además sequedad e irritación
de la córnea y aumento del tamaño de las glándulas parótidas. Su diagnostico mas probable es
artritis reumatoide; el hallazgo más probable en una biopsia de glándula parótida es ….…

Gran infiltración de linfocitos y células plasmáticas

Un hombre de 42 años de edad se presenta al médico con una historia clínica de 1 año de evolución,
caracterizado por dolor abdominal bajo y diarreas con crisis sanguinolentas. Manifiesta además
pérdida de peso de 8kg durante este periodo. La colonoscopia revela lesión difusa en el colon con
afectación del recto. La biopsia de estas lesiones revela adelgazamiento de la pared, inflamación y
ulceración de la mucosa y submucosa. El diagnóstico más probable en este caso es:
Colitis ulcerativa

Dos estudiantes deciden tomar un receso para comer una hamburguesa a la hora del almuerzo.
Antes de llegar a la cafetería, impulsos nerviosos provenientes del complejo vagal dorsal iniciarán la
secreción de ácido gástrico por la liberación de ___________ desde el sistema nervioso entérico.

GRP (péptido liberador de gastrina)

Un niño de cuatro años de edad es llevado a la consulta por cuadros diarreicos frecuentes
caracterizados por heces pálidas, voluminosas y fétidas; al examen físico presenta bajo peso y talla
para la edad. Se mide la concentración de cloruro de en el sudor y se encuentra que sus valores son
muy elevados. La alteración más importante a nivel de células ductales del páncreas tiene relación
directa con la conductancia de …..

Cloro
En el club del adulto mayor, a un grupo de ancianos de 65 años se les da a elegir entre un taller de
videojuegos y uno de lectura. La mayoría de ellos optó por el taller de lectura. ¿Cuál sería una
explicación para esta conducta?

La atención sostenida permanece relativamente estable a esa edad

Pablo es invitado a dar el discurso de bienvenida a los nuevos residentes del hospital. Muestra
habilidad en el discurso, planifica las ideas que quiere decir, mantiene el tema y objetivo de la
conversación. ¿Qué estructura cerebral estaría involucrada para demostrar esas características?

CPF dorsolateral

Para poder diagnosticar a un paciente de discapacidad intelectual se requiere:

Que las deficiencias adaptativas limiten el funcionamiento en una o varias actividades cotidianas

En relación a la dopamina podemos afirmar:

Esta presente dentro de la neurona presináptica

La mielinización neuronal para el desarrollo visual, motor, social y cognitivo se da en la dirección:

Cefalocaudal
En un paciente con hiperestimulación simpática se espera que las ondas lentas tengan un ritmo:

Menor en íleon terminal que en el duodeno

Al ingerir grandes cantidades de dulces, con la subsecuente estimulación de incretinas, usted


esperaría que el apetito ______________, debido a __________________

disminuya insulina

Es un ligamento derivado del mesenterio dorsal:

Gastrocólico

Una de las siguientes sustancias no comparte con las otras la misma acción sobre la producción de
ácido gástrico:

Colecistoquinina

Usted encuentra músculo estriado en el siguiente segmento:

Esfínter anal externo

Todos los músculos motores de la lengua están inervados por el XII par, excepto:

palatogloso

En un paciente con gastroparesia (motilidad lenta del estómago), que presenta distensión abdominal
después de comer, usted le recomendaría que evite el consumo de lípidos y aminoácidos para
disminuir la acción de:

CCK

En un paciente con diarrea por hipermotilidad, usted sospecharía en el posible aumento de las
siguientes sustancias, excepto:

Péptido intestinal vasoactivo

Marque lo correcto respecto a la siguiente imagen:

Se produjo por falta de fusión de los ductos dorsal y ventral

Respecto a los péptidos gastrointestinales, marque lo correcto.

las sustancias paracrinas pueden viajar a través de vasos sanguíneos


Se evalúa los valores séricos de las siguientes sustancias a un paciente con enfermedad hepática
terminal; en este paciente se espera encontrar la combinación de la letra …..

Glucosa

Amoniaco

Albúmina

a.

Aumentada

Disminuida

Disminuida

b.

Disminuida

Aumentada

Aumentada

c.

Aumentada

Aumentada

Aumentada

d.

Disminuida

Aumentada

Disminuida

Una mujer de 35 años de edad HIV positiva, se presenta al médico con dolor abdominal en cuadrante
superior derecho e ictericia. La paciente refiere haber tenido múltiples episodios de ictericia durante
los últimos 10 años. Los exámenes para determinar hepatitis viral, dieron positivos para Hepatitis B,
siendo catalogado el caso como hepatitis crónica con alteración funcional. En un examen de sangre
¿Cuál de los siguientes parámetros está disminuido?

Albúmina
En el reflejo peristáltico del intestino delgado ¿Cuál de los siguientes eventos sucede en la porción
caudal del bolo alimenticio?

Acción del péptido inhibidor vasoactivo (VIP) en el músculo circular

Un varón de 58 años de edad con enfermedad de Crohn severo fue sometido a una resección ileal.
Después de la cirugía esta paciente padecerá de esteatorrea esto se explica porque ……

Hay malabsorción de ácidos biliares

En un experimento se inserta un balón en el estómago de un voluntario, se infla poco a poco mientras


que se vigilan las presiones intraluminales. Aunque el volumen del balón aumenta
considerablemente, las presiones permanecen constantes. Esta relación volumen-presión se explica
por la liberación local de ….

Óxido nítrico y péptido inhibidor vasoactivo

¿Cuál de las siguientes alternativas es una característica de la secreción exocrina del páncreas?

Su mayor estímulo se da en la fase intestinal

Las estructuras en el hígado que permite que los productos metabólicos unidos a proteínas tengan
acceso a las membranas basolaterales de los hepatocitos, son ….

Las fenestras sinusoidales

La composición de la bilis es modificada conforme fluye por los conductillos biliares. Durante este
tránsito se espera que aumente la concentración de …..

Ig A

Se mide experimentalmente el contenido gástrico de dos personas. La persona “A” tiene alto
contenido de grasa y la persona “B” tiene un contenido isotónico. ¿Cual de las siguientes es correcta
respecto al vaciamiento gástrico?

Hay ralentización del vaciado gástrico sólo en “A”


El examen endoscópico de un paciente con hipertensión portal grave revela venas tortuosas que
sobresalen hacia la luz del esofago. El paciente recibe tratamiento quirúrgico mediante la colocación
de una derivación que conecta la vena cava. Después de la operación el riesgo de encefalopatía
…….. y el sangrado de varices ……

Aumentará/Disminuirá

Un bolo alimenticio grande y poco masticado se atasca en el esofago, esto ocasiona una sensacion
de dolor que es transmitida por los nervios:

Esplácnico

El peristaltismo del intestino delgado se puede intensificar debido a:

Irritación de la mucosa

Un paciente es diagnosticado con un tumor neuroendocrino productor de somatostatina, esto


provocará en el sistema digestivo:

Diarrea

Los diferentes segmentos del tubo digestivo son susceptibles de reflejos y movimientos según su
contenido.Si colocoramos mediante una sonda un bolo alimenticio directamente en el tercio medio del
esofago:

Se producirá ondas secundarias

En el digestivo la liberación hormonal se presenta ante diversos factores o estímulos. La hormona


_________ es estimulada por la presencia de alimentos en el bulbo duodenal a predominio de ácidos
grasos y triglicéridos, por estimulación vagal y por la hormona secretina.

Colecistoquinina (CCK)

El divertículo de Meckel es una anomalía congénita que ocurre por la persistencia del conducto
vitelino y da origen a una estructura sacular, el cual se encuentra en el:

Borde antimesentérico

La fase oclusal de la masticación se realiza con la contracción de los músculos inervados por el
nervio craneal:

V
En un paciente de 43 años con tumor carcinoide de páncreas productor de gastrina (Sindrome de
Zollinger-Ellison) se puede encontrar una potenciación del reflejo:

Gastrocolico

Durante la fase faríngea de la deglución ocurre el siguiente mecanismo:

La onda peristáltica lleva el alimento hacia el esofago

En los carcinomas (neoplasia benigna) es frecuente que ocurran la metástasis a través de los vasos
venosos. En el caso de un carcinoma del tercio superior del esofago, ubicado en la cara lateral
izquierda, es más probable que la metástasis viaje por la vena:

Hemiácigos accesoria

168) Los movimientos en masa son iniciados por el reflejo:

Duodenocolico

Una mujer de 65 años HIV positiva se presenta con dolor abdominal en el cuadrante superior derecho
e ictericia. La paciente afirma haber teñido múltiples episodios de ictericia durante los últimos 10
años. Los exámenes para poder detectar hepatitis viral, dieron positivos para Hepatitis B, siendo
catalogada como hepatitis crónica con alteración funcional. En un examen de sangre ¿cuál de los
siguientes parámetros está disminuido?

Albúmina
En el reflejo peristáltico del intestino delgado¿cuál de los siguientes eventos sucede en la porción
caudal del bolo alimenticio?

Acción del péptido inhibidor vasoactivo(VIP) en el músculo circular

Un varón de 58 años de edad con enfermedad de Crohn Severo fue sometido a una resección ilegal.
Después de la cirugía este paciente padecerá de esteatorrea, esto se explica porque ….

Hay mala absorción de los ácidos biliares

En un experimento se inserta un balón en el estómago de un voluntario, se infla poco a poco mientras


se vigilan las presiones intraluminales. Aunque el volumen del balón aumenta considerablemente, las
presiones permanecen constantes. Esta relación volumen presión se explica por la liberación local de
….

Óxido nítrico y péptido inhibidor vasoactivo

¿Cuál de las siguientes alternativas es una característica de la secreción exocrina del páncreas?

Su mayor estímulo se da en la fase intestinal

Las estructuras en el hígado que permiten que los productos metabólicos unidos a proteínas tengan
acceso a membranas basolaterales de los hepatocitos son ….

Las de fenestras sinusoidales

La composición de la bilis es modificada conforme fluye por los conductillos biliares. Durante este
tránsito se espera que aumente la concentración de ….

IgA

Se mide experimentalmente el contenido gástrico de dos personas. La persona A tiene alto contenido
de grasas y la persona B tiene un contenido isotónico ¿Cuál de las siguientes es correcta respecto al
vaciamiento gástrico?

Hay ralentización del vaciado gástrico sólo en A

El examen endoscópico de un paciente con hipertensión portal grave revela venas tortuosas que
sobresalen hacia la luz del estómago. El paciente recibe tratamiento quirúrgico mediante la
colocación de una derivación que conecta la vena porta a la vena cava. Después de la operación el
riesgo de encefalopatía …. y el riesgo de sangrado de varices ….
Aumentará / disminuirá

Un paciente varón de 18 años de edad acude al médico para sus exámenes de rutina. Sus resultados
de laboratorio muestran un valor de bilirrubina sérica de 4mg/dl y una bilirrubina directa de 0.3 mg/dl.
Las pruebas de función hepática son normales. La alteración que explica mejor este caso es por la
deficiencia de….

Glucuronil transferasa

Un hombre de 57 años de edad es llevado a urgencias con hematemesis masiva rojo brillante, a su
llegada se halla inconsciente con PA: 80/40 mm Hg y FC:124 lat/min. Luce ictérico con presencia de
arañas vasculares en el tórax anterior y extremidades, abdomen distendido con signo de oleada
positiva. Se encuentra esplenomegalia y pérdida de la masa muscular en extremidades. La
anastomosis vascular responsable del sangrado en este paciente es ….

Vena gastrica izquierda y vena acigos

Un estudiante de medicina está comiendo un plato de comida a base de champiñones, espárragos y


salsa de soya. El estímulo del sabor umami contenido en todos estos alimentos viaja a través del
nervio….

Cuerda del tímpano

Una paciente de 30 años de edad es sometida a una cirugía de oído medio derecho por un problema
de otoesclerosis. Luego de la cirugía refiere alteración sensitiva de la lengua. Al evaluar el caso usted
esperaría encontrar ….

Sensación del dolor, tacto y temperatura conservadas

En una paciente de 45 años de edad con colestasis biliar, se encuentra una elevación de los niveles
sanguíneos de fosfatasa alcalina hasta 3 veces la cifra normal ¿Cuál de las siguientes alternativas
estará también elevada como evidencia del daño biliar?

Gamma glutamil transpeptidasas

Experimentalmente se incrementa la velocidad de la secreción salival con una sustancia, en el


análisis de la composición de esta saliva obtenida se espera encontrar….

Aumento de la concentración de bicarbonato que supera la concentración plasmática


Lactante de 3 meses de vida es atendido por presentar diarrea, se administra una solución glucosa y
electrolitos por vía oral. La proteína de membrana apical que explica la capacidad de esta solución
para proporcionar aporte de glucosa e hidratación es ….

SGLT 1

Paciente ha sufrido herida de bala en el abdomen, se le tenido que extirpar el segmento medio y
distal del íleon. En este caso la síntesis hepática de sales biliares estará ….

Incrementada por estímulo de la enzima colesterol 7 alfa hidroxilasa

Un varón de 75 años ingresa al consultorio por presentar ictericia marcada de piel y escleras. El
estudio del paciente mostró que presentaba un tumor que obstruía la totalidad del conducto hepático
común. ¿Cual de los siguientes conductos se encontraría dilatado en este paciente?

De Hering

Correlacione las dos columnas y marque la respuesta correcta:

Enfermedad Hirschsprung Aganglionosis congénita

Diarrea osmótica. Intolerancia a la lactosa

Diarrea secretora. Canales de Cl- en las células de la cripta

Diarrea exudativa Heces con moco y sangre

La fase cefálica de la secreción gástrica responde por cerca del 39% de la respuesta ácida a un
reflejo. Con la ____ se elimina la fase cefálica de la secreción gástrica

Vaguectomía

El esofago de Barret se caracteriza por presentar___ en el esofago

Metaplasia intestinal

En la regulación del apetito y la saciedad, la estimulación experimental cronica del núcleo ventro
medial del hipotálamo producirá:

Afagia
Paciente mujer de 25 años acude por dolor en fosa iliaca derecha que empeora al toser o
caminar,asociada a náuseas y vómitos por lo cual acude a emergencia.Dos días después de
realizarle una apendicectomia, la paciente desarrolla fiebre alta (39° C), esta hipotensa y presenta
dolor abdominal.La laparotomia exploratoria muestra un gran volumen de sangre en la cavidad
peritoneal por lesion de un vaso producida durante la apendicectomía.¿Cual de las siguientes
arterias debe ligarse para detener la hemorragia?

Ileocolica

¿Cuál de los siguientes es una causa de ictericia con bilirrubina conjugada aumentada?

Obstrucción del colédoco

Dos días después de una apendicectomía en un hombre de 45 años, ha desarrollado fiebre alta (39),
está hipotenso y presenta dolor abdominal. La laparotomía exploratoria muestra un gran volumen de
sangre en la cavidad peritoneal por lesion de un vaso producida durante la apendectomía. ¿Cuál de
los siguientes vasos debe ligarse para detener la hemorragia?

Arteria ileocólica

Paciente de 78 años, con diabetes mellitus tipo ll y fumador, que acude a consulta porque desde hace
dos semanas tiene un dolor intenso en flanco derecho y mesogastrio , intenso que aparece a los 30
minutos de haber comido, y desaparece dos a tres horas después. En estos pacientes, es muy
probable que la circulación deficitaria sea parcialmente asumida por la:

Arteria cólica media

Un hombre de 70 años ingresa en urgencias con diarrea intensa. La arteriografía revela un blloqueo
del 90% en el origen aórtico de la arteria mesentérica inferior. ¿Cuál de las siguientes arterias
proporciona muy probablemente irrigación colateral al colon descendente?

Arteria cólica derecha

Un niño de 4 años ingresa en el hospital con vómitos graves. La exploración radiológica y la historias
clinica revelan que el niño tiene páncreas anular. ¿Cuál de las siguientes estructuras es la que se
encontrará dilatada con mayor probabilidad a raíz de esta patología?

Primera porción del duodeno

Un hombre de 55 años ingresó al hospital con dolor abdominal intenso. La gastroscopia y la TC


revelaron una úlcera perforada en la pared posterior del estómago. ¿Dónde se desarrolla
inicialmente con más probabilidad una peritonitis?

Bolsa omental (saco menor)


Un hombre de 44 años ingresa en urgencias con vómitos abundantes y deshidratación. Las imágenes
radiológicas demuestran que parte del intestino está comprimido entre la aorta abdominal y la arteria
mesentérica superior. ¿Cuál de las siguientes estructuras intestinales estará muy probablemente
comprimida?

Tercera porción del duodeno

¿Cuál de los siguientes nódulos linfáticos estará muy probablemente afectado en una neoplasia
maligna del recto?

Mesentérica inferior

Una mujer de 23 años ingresa con dolor abdominal, náuseas y vómitos. La historia Clínica muestra
que el dolor es agudo y ha sido constante durante 4 años. El dolor empezó en el epigastrio e irradió
bilateralmente alrededor del tórax hasta justo debajo de las cápsulas. Actualmente el dolor se localiza
en el hipocondrio derecho. L a TC revela cálculos calcificados en la vesícula biliar. ¿Cuál de los
siguientes nervios llevan las fibras aferentes del dolor referido?

Nervios esplácnicos torácicos mayores

Un hombre de 55 años ingresa en urgencias por severa perdida de peso en los 6 meses previos. El
examen radiológico pruebas aportan signos de un tumor que causa hipertensión portal. Los estudios
de laboratorio revelan que las deposiciones son grasas, tiene desnutrición e hipoxia hepática. ¿Cuál
de las siguientes localizaciones se encuentra muy probablemente afectada?

Segmento l

El triángulo de calot es importante reconocerlo porque sirve como reparo para encontrar la arteria
cística, y está formado por el conducto cístico, el conducto hepático derecho y :

Borde hepático

El kernícterus se produce en recién nacidos con valores mayores de 25m/dL en la bilirrubina:

Indirecta

Paciente con cirrosis hepática y presión de vena cava de 15mmHg. Lo más probable es que el
paciente presente:

Ascitis
Considerando un flujo plasmático renal de 180 ml/min, y una fracción de filtración del 20%, si la
concentración en sangre de la bilirrubina indirecta es de 0.6mg/dL, entonces es correcto esperar que
la carga filtrada de la bilirrubina indirecta es:

Menor de 1080 mg/dL

El volumen diario de bilis secretada al intestino es:

500-1000 ml

La excreción del amonio se da principalmente por:

Orina

Si en una persona normal, el flujo de la arteria hepática es de 700ml/min, entonces el flujo de la vena
porta debería ser aproximadamente:

2800mL/min

Paciente con Grigler-Najjar debida a mutación del gen UGT1A1, se presenta a consulta por ictericia,
usted asume que si le hiciera un análisis de sangre encontraría valores elevados de:

Bilirrubina indirecta

El amoniaco corporal se forma principalmente en:

Colon

La zona del lobulillo hepático que se afectaría más en una intoxicación con droga hepatotóxica es la
zona:

El acino hepático tiene en sus aristas:

Vena centrolobulillar

Paciente con esteatosis hepática no alcohólica, que en la biopsia se observa degeneración grasa de
los hepatocitos, lo cual se debe a depósitos de lípidos que principalmente contienen:
Triglicéridos

Al evaluar la orofaringe de un paciente, el médico le solicita que abra la boca, saque la lengua y diga
a . Al hacer esta maniobra, nota que el paladar se desvía hacia la derecha, lo cual le hace sospechar
que el paciente sufre de una lesión del nervio craneal:

X contralateral

Un bolo alimenticio grande y poco masticado se atasca en el esófago, esto ocasiona una sensación
de dolor que es transmitida por los nervios:

esplácnicos

Para realizar el movimiento mecánico de abrir la boca, primero se necesita:

fijar el hueso hioides

¿Cuál de las siguientes alternativas se define como la protrusión directa del contenido abdominal a la
cavidad amniótica por un defecto de la pared corporal?

Gastrosquisis

Un paciente requiere que se le coloque una sonda de alimentación directamente al estómago


(gastrostomía), el cirujano deberá hacer una incisión en la piel del abdomen ¿cuál de las siguientes
raíces nerviosas debe ser anestesiada para este procedimiento?

T8

El mecanismo de la defecación incluye la participación de diversas estructuras ¿Cuál de las


siguientes alternativas es correcta?

Puede ser mediado por un reflejo intrínseco

Cuando el contenido del estómago ingresa al duodeno, uno de los reflejos que inhiben el vaciamiento
gástrico es a través del:

sistema nervioso mientérico

Durante la masticación, gran parte del proceso masticatorio se debe a:


el reflejo masticatorio

Las glándulas salivales tienes conductos para la excreción de la saliva; las glándulas ____________
drenan en las carúnculas sublinguales.

sublinguales

En una persona si enfermedad se espera que el tránsito intestinal se vea disminuido cuando se
presenta el reflejo:

doloroso

El divertículo de Meckel es una anomalía congénita que ocurre por la persistencia del conducto
vitelino y da origen a una estructura sacular, el cual se encuentra en el:

borde antimesentérico

Si al intubar a un paciente, por error se ingresa el tubo endotraqueal en el esófago y se insufla el


manguito endotraqueal (globo TET), la dilatación de este manguito generará:

múltiples ondas secundarias

El orificio omental, o hiato de Winslow, se encuentra limitado por el ligamento:

hepatoduodenal

Paciente de 24 años acude a consulta externa por presentar una fístula oronasal (comunicación entre
la cavidad oral y la cavidad nasal). Está fístula es una consecuencia tardía de la lesión de un vaso
sanguíneo por el antecedente de haber sido operado de paladar hendido en los primeros años de
vida, aparentemente en una campaña gratuita de corrección de paladar fisurado. ¿Cuál de las
arterias palatinas podría haberse lesionado durante esa cirugía?

Mayor

Dentro de las anomalías congénitas se puede presentar un tejido pancreático accesorio ¿Cuál es la
ubicación más común de este tejido?

Estómago
Paciente con insuficiencia mitral moderada a severa, con aumento de volumen de la aurícula
izquierda; esta condición tendrá como consecuencia a nivel del sistema digestivo:

la disfagia a sólidos

El inicio de la fase faríngea de la deglución se debe a estímulos sensitivos que viajan por el nervio
craneal:

V (nervio maxilar)

El mesocolon transverso se origina en:

la pared posterior del abdomen

La contracción del músculo ………………………… permite la eliminación de gases (flatos) sin salida
de material fecal ; es el mismo músculo cuya relajación, sobretodo en cuclillas, permite el paso del
contenido fecal con menor esfuerzo durante la defecación

puborrectal

Paciente mujer de 54 años se presenta con náuseas, vómitos, estreñimiento, y es diagnosticada de


abdomen agudo quirúrgico; en la cirugía encuentran un vólvulo de ciego. Esta anomalía puede
explicarse por:

Falta de fusión del mesenterio

Paciente mujer de 23 años con faringitis aguda , toma para el dolor una tableta de paracetamol con
un poco de agua. Durante la deglución, se relaja su esfínter esofágico inferior y el fondo del
estómago, mientras el bolo está aún en el esófago. ¿Qué sustancia provocará con mayor
probabilidad la relajación del esfínter esofágico inferior y el fondo del estómago en esta mujer?

Óxido nítrico

Luego de tres horas dando exámenes, un alumno de medicina comienza a sentir hambre. Esta
situación es probable que sea mediada por la _____ que es sintetizada por el _____

grelina / estómago

Varón de 72 años, con antecedente de diabetes mellitus tipo 2, que presenta entropía diabética
caracterizada por estreñimiento. Este problema puede estar asociado a:

deficiencia de óxido nítrico


Varón de 54 años con diabetes mellitus tipo 2, es diagnosticado de gastroparesia debido a que
presenta sensación de llenura precoz al comer, y reflujo gastroesofágico. Esta alteración en la
relajación receptiva y en el vaciamiento gástrico lo más probable es que se deba a una alteración en:

el nervio vago

Varón de 67 años con tos y disminución de peso asociado a tabaquismo pesado, presenta
actualmente disfagia progresiva a alimentos sólidos. Se considera la presencia de un carcinoma de
bronquio izquierdo y por esta razón le realizan una endoscopia esofágica para descartar la posibilidad
de una compresión esofágica por el tumor. Se espera revisar el esófago en la _____ estrechez, que
está a nivel de la vértebra _____.

Tercera estrechez - T6

En un niño menor de dos años con divertículo intestinal, este divertículo tiene su origen en una falla
en la obliteración de:

Conducto vitelino

Mujer de 43 años sufre un grave accidente de tránsito y está hospitalizada en coma, es alimentada
por vía intravenosa durante varias semanas. Producto de este tipo de alimentación, se encuentra en
la endoscopia atrofia de la mucosa gastrointestinal. La causa más probable de esta atrofia son los
bajos niveles séricos de la hormona:

Gastrina

Una mujer de 30 años llega al consultorio porque se queja de dificultades para deglutir, la cual se
agravan cada vez más. Se realiza un estudio manométrico para examinar la generación de presión a
lo largo del esófago. Esta prueba revela que las contracciones como respuesta a la deglución están
mal sincronizadas y que la presión en el esfínter esofágico inferior permanece elevada. El diagnóstico
más probable es _____ producido por niveles bajos de _____:

acalasia / óxido nítrico

Paciente de 2 años, llega a emergencia por haber ingerido una moneda con la que estaba jugando. El
lugar más probable donde puede haberse quedado suspendido este objeto es a nivel del
estrechamiento producido a nivel del:

músculo cricofaríngeo

En una apendicectomía, al realizar la incisión de McBurney en la fosa iliaca derecha, es necesario


cortar los siguientes músculos, de afuera hacia adentro:

Oblicuo externo - Oblicuo interno - Transverso


Un varón de 90 años que se encuentra postrado en cama, es referido del asilo para endoscopia por
dificultad para deglutir luego de tomar un medicamento para aliviar el dolor de la noche anterior. La
endoscopia revela que la píldora se alojó en el esófago y causó una reacción inflamatoria. Lo más
probable es que esto haya sido por la producción de múltiples ondas:

secundarias

Mujer de 23 años es diagnosticada de bulimia, al examen físico se observa ulceraciones en el


segundo y tercer dedo de la mano derecha. Esto se puede deber al uso continuo de estos dedos para
inducir el vómito, mediante la estimulación del par craneal:

IX

Varón de 52 años se presenta por diarrea persistente de seis semanas de duración. En la


colonoscopia se observa un pólipo a nivel del íleon distal. El patólogo informa que se trata de un
tumor neuroendocrino, probablemente originado por las células enterocromafines del intestino. La
sustancia que más probablemente esté produciendo este tumor es:

Serotonina

La fase oclusal de la masticación se realiza con la contracción de los músculos:

masetero y temporal

Al tomar su café en Starbucks, un estudiante de medicina sufre una quemadura de primer grado en el
tercio anterior de la superficie dorsal de la lengua. La información de dolor es transmitida por el
nervio:

Lingual

Paciente es evaluado por faringitis aguda en consultorio externo. El médico de familia le solicita que
abra la boca y saque la lengua Para realizar la acción de sacar la lengua, es necesario que se
contraiga el músculo:

geniogloso

Paciente con síndrome de Sjogren, presenta “boca seca” (disminución de la producción de saliva) y
caries dental, asociada a la pérdida de la función de tampón de la saliva. Esta desmineralización del
diente puede comprometer a las prolongaciones citoplasmáticas ubicadas en los tubos huecos de la
estructura señalada con la letra:

SHADICK PARCIAL

1. Cuando el istmo de las fauces se cierra, se evita que el alimento pase hacia la orofaringe y
permite respirar mientras se mastica. Este cierre se debe a la contracción y aproximación de
los músculos:
- Palatofaríngeos
- Palatoglosos
- Estiloglosos
- Estilofaríngeos
2. Las siguientes alternativas son ciertas sobre la actividad eléctrica del músculo
gastrointestinal, EXCEPTO:
- La despolarización lenta se debe principalmente al ingreso de Na+.
- Las ondas lentas son más frecuentes en el duodeno.
- Si el potencial de membrana es más positivo, habrá mayor frecuencia de espigas.
- Las ondas lentas no son potenciales de acción.
3. Durante el paso del bolo hacia la orofaringe, se desencadena una serie de contracciones
musculares que estrechan la cavidad faringea. Estas contracciones están mediadas por el
nervio craneal:
- IX
- XII
- XI
- X
4. ¿Cuál de las siguientes alternativas detallan los músculos que ayudan a empujar el bolo
hacia la orofaringe?
- Estilogloso y palatogloso
- Estilogloso y geniogloso
- Hiogloso y geniogloso
- Geniogloso y palatofaringeo
5. En un paciente es derivado por lesión del nervio vago derecho. Al evaluar el velo del paladar,
se solicita al paciente que diga ahh, entonces se puede observar que la úvula:
- Se eleva en el centro
- Se desvía a la derecha
- Se desvía a la izquierda
- Permanece sin movimiento
6. ¿Cuál de las siguientes estructuras tiene inervación somática?
- Estómago
- Mesosigmoides
- Peritoneo parietal
- Peritoneo Visceral
7. Estas diseñando un proyecto de investigación sobre los niveles de colesterol que se
absorben luego de una comida grasosa y deseas cuantificar la cantidad de colesterol que es
absorbido por el intestino antes que el hígado lo metabolice ¿de cual de los siguientes vasos
obtendrías la muestra para tu análisis?
- Vena porta
- Vena cava superior
- Vena hemiácigos accesoria
- Conducto torácico
8. La masticación es básicamente:
- Un movimiento reflejo
- Importante para la digestión sobretodo de carnes
- Un ralentizador del vaciamiento gástrico
- Una actividad consciente
9. En la estructura dentaria, se observa qué hay una composición muy similar a la del hueso en
la capa denominada:
- Predestina
- Esmalte
- Dentina
- Cemento
10. El nervio palatino menor inerva un área del paladar que está recubierta por epitelio:
- Estratificado plano no queratinizado
- Estratificado plano queratinizado
- Simple cilíndrico
- Simple plano
11. Mujer de 30 años llega a emergencia con dolor en hipogástrico. Al examen físico presenta
una masa palpable de 10cm de diámetro en la misma región, usted sospecharía de las
siguientes condiciones, excepto:
- Tumor renal
- Cáncer de recto superior
- Embarazo
- Tumor uterino
12. Un familiar le comenta que tiene úlcera gástrica por exceso de producción de ácido; con sus
conocimientos del sistema digestivo, usted le recomendaría que reduzca el consumo de:
- Agua
- Aminoácidos
- Carbohidratos
- Vitaminas
13. Con respecto de la regulación del pH del estómago; al utilizar un bloqueador de histamina,
usted espera que el pH del estómago:
- Disminuya
- Se mantenga sin cambio
- Se neutralice por acción de bicarbonato
- Aumente
14. Las siguientes alternativas son ciertas sobre las contracciones tónicas del músculo
gastrointestinal, excepto:
- Se encuentran principalmente es esfínteres
- Tienen relación con el ingreso persistente de iones sodio
- Obedece a una mayor frecuencia de potenciales en espiga
- Tienen regulación hormonal
15. ¿Cuál de las siguientes condiciones considera que es un trastorno de la musculatura lisa
esofágica?
- Asinergia faringoesfinteriana
- Hipotonía de los constrictores faríngeos
- Acalasia
- Hipertonía del esfínter esofágico superior
16. ¿Cuál de las siguientes alternativas es correcta sobre la motilidad esofágica?
- Las ondas primarias no son propulsoras y siempre van precedidas de deglución
- Las ondas primarias son propulsoras y pueden no ser precedidas por deglución
- Las ondas secundarias son propulsoras y no van precedidas de deglución
- Las ondas secundarias son propulsoras y siempre van precedidas de deglución
17. Respecto al peristaltismo intestinal, para cumplir la ley del intestino, usted espera que a nivel
distal del quimo se libere:
- Sustancia P
- Péptido liberador de gastrina (GRP)
- Acetilcolina
- Péptido intestinal vasoactivo
18. El peristaltismo depende que a nivel distal del bolo se secrete:
- Noradrenalina secretada por las fibras del sistema simpático
- Acetilcolina por las neuronas provenientes del nervio vago
- Péptido intestinal vasoactivo por neuronas
- Óxido nítrico por células endoteliales locales
19. La relajación receptiva gástrica, se produce principalmente por la acción de:
- El péptido intestinal vasoactivo
- La sustancia P
- La acetilcolina
- La bombesina
20. El peristaltismo intestinal se produce gracias a un reflejo que:
- Llega a los ganglios pre vertebrales
- Llega a la médula espinal
- Se origina dentro de la pared intestinal
- Llega al sistema nervioso central
21. ¿Cuál de las siguientes alternativas estimula las ondas de motilidad gastrointestinal
denominadas complejos migratorios interdigestivos?
- Alimentos
- Metoclopramida
- ERITROMICINA
- Colecistoquinina (CKK)
22. ¿Cuál de las siguientes alternativas es correcta sobre el control del peristaltismo?
- El peristaltismo intestinal aumenta por efecto de la colecistoquinina (CKK)
- La secretina aumenta la motilidad del intestino delgado
- La serotonina no tiene efecto sobre la motilidad digestiva
- El reflejo gastroentérico evita el avance del quimo
23. En un paciente con shock hipovolémico, la peristaltismo intestinal se encuentra:
- Aumentada
- Sin cambios
- Disminuida
- Invertida
24. Es un derivado del mesenterio dorsal:

- D
- B
- C
- A
25. En un paciente con falla en la fusión de los conductos de las yemas central y dorsal del
páncreas, usted esperaría encontrar:
- Drenaje de la mayor parte del jugo pancreático en la papila menor
- Drenaje adecuado del jugo pancreático
- Drenaje de la mayor parte del jugo pancreático en la papila mayor
- Estenosis del duodeno
Shiro Parcial

1. La relajación receptiva gástrica, se produce principalmente por la acción de:


- La sustancia P
- El péptido intestinal vasoactivo
- La bombesina
- La acetilcolina

2. ¿Cuál de las siguientes alternativas es correcta sobre el movimiento


peristáltico?
- Es independiente del plexo mientérico
- El contenido intestinal avanza sólo 5-10 cm
- Es un reflejo largo que depende de la integración con el tronco encefálico
- Se dirige en sentido distal siempre, nunca en sentido proximal

3. Con respecto a la regulación del peristaltismo, al aplicarle atropina


(antagonista colinérgico) a un paciente, es de esperarse que el peristaltismo:
- aumente
- se mantenga sin alteración
- estimule la acción de los receptores dopaminérgicos
- disminuya

4. El estímulo habitual para el movimiento peristáltico es:


- contracción de la musculatura circular Interna
- distensión local
- acción de la sustancia P
- estimulación vago-vagal

5. El peristaltismo depende que a nivel distal del bolo se secrete:


- péptido intestinal vasoactivo por neuronas
- noradrenalina secretada por las fibras del sistema simpático
- óxido nítrico por células endoteliales locales
- acetilcolina por las neuronas provenientes del nervio vago

6. La hormona responsable de los complejos migratorios interdigestivos tiene las


siguientes características, EXCEPTO:
- es inhibida por el alimento
- cumple funciones de aumentar la motilidad y secreción gástrica e intestinal
- se produce en el estómago y el duodeno
- se libera de forma cíclica
7. Respecto al peristaltismo intestinal, para cumplir la ley del intestino , usted
espera que a nivel distal del quimo se libere:
- péptido liberador de gastrina (GRP)
- acetilcolina
- sustancia P
- péptido intestinal vasoactivo

8. La distención del yeyuno provoca que se:


- aumente la frecuencia de las ondas lentas
- disminuya el número de espigas
- despolarice el potencial de reposo de membrana
- produzca una contracción tónica

9. Un familiar le comenta que tiene úlcera gástrica por exceso de producción de


ácido; con sus conocimientos del sistema digestivo, usted le recomendaría
que reduzca el consumo de:
- carbohidratos
- agua
- aminoácidos
- vitaminas

10. El pH óptimo para la digestión a nivel duodenal es ____________ y está


regulado principalmente por la liberación de _________________
- 6-8 / secretina
- 5-6 / colecistoquinina
- 5-6 / gastrina
- 8-10 / gastrina

11. El tubo digestivo posee glándulas, las glándulas submucosas se encuentran


en el:
- íleon y esófago
- esófago y duodeno
- estómago y duodeno
- duodeno y recto

12. En una cirugía abierta (laparotomía), el cirujano al abrir la cavidad peritoneal


por la parte anterior (línea media), lo primero que observa es:
- Colon sigmoides
- Estomago
- Duodeno
- Epiplón mayor

13. Sobre el control de la peristalsis del tubo digestivo, ________________ es un


mediador neural que induce la relajación durante la peristalsis
- la acetilcolina
- el péptido intestinal vasoactivo
- la serotonina
- la somatostatina

14. ¿Cuál de las siguientes alternativas es correcta sobre el control del


peristaltismo?
- El peristaltismo intestinal aumenta por efecto de la colecistoquinina (CCK)
- El reflejo gastroentérico evita el avance del quimo
- La serotonina no tiene efecto sobre la motilidad digestiva
- La secretina aumenta la motilidad del intestino delgado

15. El frenillo de los labios se encuentra en:


- la cavidad oral
- el piso de la boca
- el dorso de la lengua
- la cavidad vestibular

16. Las siguientes alternativas son ciertas sobre la actividad eléctrica del músculo
gastrointestinal, EXCEPTO:
- Si el potencial de membrana es más positivo, habrá mayor frecuencia de
espigas
- La despolarización lenta se debe principalmente al ingreso de Na+
- Las ondas lentas no son potenciales de acción
- Las ondas lentas son más frecuentes en el duodeno

17. Durante una cena, una gestante inspira por la boca profundamente de manera
frecuente; sin embargo, el organismo evita que el aire ingrese al esófago por la
acción:
- del istmo de las fauces
- del esfínter esofágico inferior
- del músculo cricofaríngeo
- de la epiglotis

18. El músculo liso gastrointestinal funciona como un sincitio debido a:


- el plexo mientérico de Auerbach
- el calcio
- las fibras musculares más largas
- las uniones en hendidura

19. Al ingresar líquidos o sólidos en la cavidad oral, un mecanismo que permite


que una persona respire mientras mastica es:
- la depresión del paladar blando
- el movimiento hacia afuera de los pliegues palatogloso y palatofaríngeo
- la depresión de la parte posterior de la lengua
- la elevación del paladar blando

20. Al rozar agua caliente en la punta de la lengua, usted esperaría que el estímulo
viaje a través del nervio:
- cuerda del tímpano
- lingual
- glosofaríngeo
- hipogloso

21. Las siguientes alternativas son correctas sobre la deglución, EXCEPTO:


- Consta de 2 fases
- En la fase voluntaria, el bolo es impulsado hacia arriba y hacia atrás por la
lengua
- El centro de la respiración es inhibido a nivel bulbar
- Para impedir el paso del alimento a la nasofaringe, el paladar blando se eleva

22. Paciente adulto con reflujo gastroesofágico es más probable que presente:
- desfluorización de los dientes
- destrucción de ameloblastos
- remodelación del esmalte
- descalcificación del esmalte

23. Considerando sus conocimientos en embriología, la disposición del intestino


delgado final tras la retracción de asas intestinales, se distribuye de la
siguiente manera:
- asas yeyunales en hipogastrio
- asas yeyunales en cuadrante inferior derecho
- asas ileales en cuadrante inferior derecho
- duodeno en hipocondrio izquierdo

24. Señale cuál de las estructuras que en el embrión se


encuentra comunicada con el saco vitelino por medio del
conducto onfalomesentérico:
- C
- A
- D
- B

25. Con seguridad, usted puede decir que la siguiente


cicatriz postapendicectomía se encuentra en la región denominada
- fosa iliaca derecha
- flanco izquierdo
- flanco derecho
- hipocondrio derecho

Parcial - José G.
1. Niña de 6 años se asusta por que se le ha aflojado un diente deciduo. Este
fenómeno se produce por:
a. laxitud del ligamento periodóntico
b. fractura del cemento
c. desmineralización del esmalte dental
d. aumento anómalo de la predentina

2. ¿Cuál de las siguientes estructuras deriva del intestino anterior?


a. A
b. B
c. C
d. D

3. En un paciente de 3 semanas de edad, con vómitos en proyectil, y nódulo epigástrico


reptante, usted esperaría encontrar:
a. engrosamiento de la circular interna pilórica
b. distensibilidad disminuida de la región oral del estómago
c. colecistoquinina aumentada
d. vómitos biliosos e intolerancia a los ácidos grasos

4. Sobre el control de la peristalsis del tubo digestivo, ________________ es un


mediador neural que induce la relajación durante la peristalsis.
a. el péptido intestinal vasoactivo
b. la serotonina
c. la acetilcolina
d. la somatostatina

5. En el plexo mientérico, el origen de los impulsos eferentes está en:


a. el plexo de Meissner
b. los ganglios paravertebrales
c. las células intersticiales de Cajal
d. el plexo de Aurbach

6. Un familiar le comenta que tiene úlcera gástrica por exceso de producción de ácido;
con sus conocimientos del sistema digestivo, usted le recomendaría que reduzca el
consumo de:
a. aminoácidos
b. vitaminas
c. carbohidratos
d. agua

7. Las siguientes alternativas son ciertas en relación al mesenterio, EXCEPTO:


a. Los omentos cumplen una función principal de irrigación visceral
b. El omento menor deriva del mesenterio ventral
c. El ligamento esplenorrenal une al bazo con el riñón izquierdo
d. Los mesenterios cumplen la función de sostén y suspensión de órganos

8. ¿Cuál de las siguientes alternativas es una característica de la estructura del


esófago?
a. Contiene glándulas submucosas principalmente en su tercio distal
b. Aumenta la presión intra esofágica durante la inspiración
c. Contiene músculo estriado en casi toda su longitud
d. El esfínter esofágico inferior es un esfínter anatómico

9. En una cirugía abierta (laparotomía), el cirujano al abrir la cavidad peritoneal por la


parte anterior (línea media), lo primero que observa es:
a. Epiplón mayor
b. Estómago
c. Colon sigmoides
d. Duodeno

10. El tubo digestivo posee glándulas, las glándulas submucosas se encuentran en el:
a. esófago y duodeno
b. duodeno y recto
c. íleon y esófago
d. estómago y duodeno

11. ¿Cuál de las siguientes alternativas detallan los músculos que ayudan a empujar el
bolo hacia la orofaringe?
a. Estilogloso y palatogloso
b. Geniogloso y palatofaríngeo
c. Hiogloso y geniogloso
d. Estilogloso y geniogloso

12. Al ingresar líquidos o sólidos en la cavidad oral, un mecanismo que permite que una
persona respire mientras mastica es:
a. la depresión del paladar blando
b. la elevación del paladar blando
c. el movimiento hacia afuera de los pliegues palatogloso y palatofaríngeo
d. la depresión de la parte posterior de la lengua

13. Durante el paso del bolo hacia la orofaringe, se desencadena una serie de
contracciones musculares que estrechan la cavidad faríngea. Estas contracciones
están mediadas por el nervio craneal:
a. X
b. IX
c. XI
d. XII
14. Las siguientes alternativas son factores que determinan la patencia y función
adecuada del esfínter esofágico inferior, EXCEPTO:
a. Hipertrofia de la circular interna
b. Canales lentos de calcio
c. Plicatura diafragmática
d. Angulación con el estómago

15. Durante una cena, una gestante inspira por la boca profundamente de manera
frecuente; sin embargo, el organismo evita que el aire ingrese al esófago por la
acción:
a. del músculo cricofaríngeo
b. del istmo de las fauces
c. del esfínter esofágico inferior
d. de la epiglotis

16. En un estudiante de medicina que está rindiendo un examen parcial, lo más probable
es que en ese momento su tránsito intestinal se encuentre:
a. muy lento
b. estimulado por acción de la sustancia P
c. muy acelerado
d. sin alteraciones

17. El principal gobernante sobre todos los movimientos gastrointestinales es el sistema


nervioso:
a. mientérico
b. parasimpático
c. somático
d. simpático

18. ¿Cuál de las siguientes condiciones considera que es un trastorno de la musculatura


lisa esofágica?
a. Acalasia
b. Hipotonía de los constrictores faríngeos
c. Hipertonía del esfínter esofágico superior
d. Asinergia faringoesfinteriana

19. El estímulo habitual para el movimiento peristáltico es:


a. distensión local
b. acción de la sustancia P
c. contracción de la musculatura circular Interna
d. estimulación vago-vagal

20. La hormona responsable de los complejos migratorios interdigestivos tiene las


siguientes características, EXCEPTO:
a. cumple funciones de aumentar la motilidad y secreción gástrica e intestinal
b. se libera de forma cíclica
c. se produce en el estómago y el duodeno
d. es inhibida por el alimento
21. Respecto al peristaltismo intestinal, para cumplir la ley del intestino , usted espera
que a nivel distal del quimo se libere:
a. péptido intestinal vasoactivo
b. acetilcolina
c. péptido liberador de gastrina (GRP)
d. sustancia P

22. ¿Cuál de las siguientes alternativas es correcta sobre el control autónomo del
aparato gastrointestinal?
a. La estimulación simpática estimula a la muscularis mucosae
b. El plexo mientérico de Auerbach cumple funciones inhibitorias
c. Las terminaciones nerviosas simpáticas liberan mayor cantidad de adrenalina
que noradrenalina
d. Al seccionar el vago, la inervación parasimpática del colon sigmoides
disminuye

23. La distención del yeyuno provoca que se:


a. despolarice el potencial de reposo de membrana
b. produzca una contracción tónica
c. disminuya el número de espigas
d. aumente la frecuencia de las ondas lentas

24. El frenillo de los labios se encuentra en:


a. la cavidad vestibular
b. el dorso de la lengua
c. el piso de la boca
d. la cavidad oral

25. Con seguridad, usted puede decir que la siguiente cicatriz postapendicectomía se
encuentra en la región denominada:

a. flanco derecho
b. hipocondrio derecho
c. flanco izquierdo
d. fosa iliaca derecha

EXAMEN PARCIAL - PEQUE

1. La masticación es básicamente:
● un movimiento reflejo
● un ralentizador del vaciamiento gástrico
● una actividad consciente
● importante para la digestión sobretodo de carnes
2. En el plexo mientérico, el origen de los impulsos eferentes está en:
● el plexo de Meissner
● el plexo de Aurbach
● las células intersticiales de Cajal
● los ganglios paravertebrales
3. ¿Cuál de las siguientes alternativas es correcta sobre el control del peristaltismo?
● La secretina aumenta la motilidad del intestino delgado
● El reflejo gastroentérico evita el avance del quimo
● La serotonina no tiene efecto sobre la motilidad digestiva
● El peristaltismo intestinal aumenta por efecto de la colecistoquinina (CCK)
4. Señale cuál de las estructuras que en el embrión se encuentra comunicada con el
saco vitelino por medio del conducto onfalomesentérico:

● D
● A
● C
● B
5. En un paciente de 3 semanas de edad, con vómitos en proyectil, y nódulo epigástrico
reptante, usted esperaría encontrar:
● vómitos biliosos e intolerancia a los ácidos grasos
● distensibilidad disminuida de la región oral del estómago
● engrosamiento de la circular interna pilórica
● colecistoquinina aumentada
6. Durante el paso del bolo hacia la orofaringe, se desencadena una serie de
contracciones musculares que estrechan la cavidad faríngea. Estas contracciones
están mediadas por el nervio craneal:
● IX
● XII
● X
● XI
7. Cuando el istmo de las fauces se cierra, se evita que el alimento pase hacia la
orofarínge y permite respirar mientras se mastica. Este cierre se debe a la
contracción y aproximación de los músculos:
● estilofaríngeos
● palatofaríngeos
● estiloglosos
● palatoglosos
8. Las siguientes alternativas son factores que determinan la patencia y función
adecuada del esfínter esofágico inferior, EXCEPTO:
● Hipertrofia de la circular interna
● Canales lentos de calcio
● Plicatura diafragmática
● Angulación con el estómago
9. El tubo digestivo posee glándulas, las glándulas submucosas se encuentran en el:
● duodeno y recto
● esófago y duodeno
● íleon y esófago
● estómago y duodeno
10. ¿Cuál de las siguientes estructuras tiene inervación somática?
● Mesosigmoides
● Peritoneo visceral
● Estómago
● Peritoneo parietal
11. Mujer de 30 años llega a emergencia con dolor en hipogástrico. Al examen físico
presenta una masa palpable de 10 cm de diámetro a en la misma región, usted
sospecharía de las siguientes condiciones, EXCEPTO:
● Embarazo
● Tumor renal
● Cáncer de recto superior
● Tumor uterino
12. Las siguientes alternativas son ciertas en relación al mesenterio, EXCEPTO:
● Los omentos cumplen una función principal de irrigación visceral
● Los mesenterios cumplen la función de sostén y suspensión de órganos
● El omento menor deriva del mesenterio ventra
● El ligamento esplenorrenal une al bazo con el riñón izquierdo
13. Al realizar una vagotomía por úlcera péptica, usted esperaría una disminución de
producción de gastrina debido a la:
● menor distensión de las paredes gástricas
● ausencia de acetilcolina vagal
● ausencia de bombesina vagal
● ausencia de histamina
14. Al ingresar líquidos o sólidos en la cavidad oral, un mecanismo que permite que una
persona respire mientras mastica es:
● la elevación del paladar blando
● la depresión de la parte posterior de la lengu
● el movimiento hacia afuera de los pliegues palatogloso y palatofaríngeo
● la depresión del paladar blando
15. ¿Cuál de las siguientes alternativas detallan los músculos que ayudan a empujar el
bolo hacia la orofaringe?
● Estilogloso y geniogloso
● Geniogloso y palatofaríngeo
● Hiogloso y geniogloso
● Estilogloso y palatogloso
16. La información sensitiva aferente del sistema gastrointestinal pasa por las siguientes
estructuras, EXCEPTO:
● Ganglios prevertebrales
● Médula espinal
● Tronco encefálico
● Tálamo
17. Los reflejos enterogástricos son desencadenados por las siguientes circunstancias,
EXCEPTO:
● Alta carga de carbohidratos en duodeno
● Distensión de pared intestina
● Quimo con alta osmolaridad
● Disminución de pH duodenal
18. Las siguientes alternativas son ciertas sobre las contracciones tónicas del músculo
gastrointestinal, EXCEPTO:
● Obedece a una mayor frecuencia de potenciales en espiga
● Tienen regulación hormonal
● Se encuentran principalmente en esfínteres
● Tienen relación con el ingreso persistente de iones sodio
19. Respecto al peristaltismo intestinal, para cumplir la ley del intestino , usted espera
que a nivel distal del quimo se libere:
● sustancia P
● péptido intestinal vasoactivo
● péptido liberador de gastrina (GRP)
● acetilcolina
20. El principal gobernante sobre todos los movimientos gastrointestinales es el sistema
nervioso:
● mientérico
● parasimpático
● somático
● simpático
21. La relajación receptiva gástrica, se produce principalmente por la acción de:
● la bombesina
● la sustancia P
● la acetilcolina
● el péptido intestinal vasoactivo
22. Con respecto a la regulación del peristaltismo, al aplicarle atropina (antagonista
colinérgico) a un paciente, es de esperarse que el peristaltismo:
● disminuya
● se mantenga sin alteración
● aumente
● estimule la acción de los receptores dopaminérgicos
23. ¿Cuál de las siguientes alternativas es correcta sobre la motilidad esofágica?
● Las ondas primarias son propulsoras y pueden no ser precedidas por
deglució
● Las ondas secundarias son propulsoras y siempre van precedidas de
deglución
● Las ondas primarias no son propulsoras y siempre van precedidas de
deglución
● Las ondas secundarias son propulsoras y no van precedidas de deglución
24. El nervio palatino menor inerva un área del paladar que está recubierta por epitelio:
● simple plano
● estratificado plano no queratinizado
● estratificado plano queratinizado
● simple cilíndrico
25. Paciente adulto con reflujo gastroesofágico es más probable que presente:
● descalcificación del esmalte
● remodelación del esmalte
● destrucción de ameloblastos
● desfluorización de los dientes

ALEJANDRO HERRERA

1. Al rozar agua caliente en la punta de la lengua, usted esperaría que el estímulo viaje
a través del nervio:

a) Lingual

b) Cuerda del timpano

c) Glosofaríngeo

d) Hipogloso

2. Al ingresar líquidos o sólidos en la cavidad oral, un mecanismo que permite que una
persona respire mientras mastica es:

a) El movimiento hacia afuera de los pliegues palatogloso y Palatofaríngeo

b) La elevación del paladar blando


c) La depresión de la parte posterior de la lengua

d) La depresión del paladar blando

3. Paciente adulto con reflujo gastroesofágico es más probable que presente:

a) Descalcificación del esmalte

b) Destrucción de ameloblastos

c) Remodelación del esmalte

d) Desfluorización de los dientes

4. La masticación es básicamente:

a) Importante para la digestión sobretodo de carnes

b) Un ralentizador del vaciamiento gástrico

c) Una actividad consciente

d) Un movimiento reflejo

5. Paciente de 34 años es víctima de asalto con arma de fuego, recibiendo un impacto


directo en el abdomen. En base a la radiografía, usted puede registrar en la historia
clínica que el proyectil se encuentra topográficamente en el:

a) Flanco derecho

b) Mesogastrio

c) Flanco izquierdo
d) Hipocondrio izquierdo

6. Señale cuál de las estructuras que en el embrión se encuentra comunicada con el


saco vitelino por medio del conducto onfalomesentérico:

7. En un paciente de 3 semanas de edad, con vómitos en proyectil, y nódulo


epigástrico reptante, usted esperaría encontrar:

a) colecistoquinina aumentada

b) Vómitos biliosos e intolerancia a los ácidos grasos

c) Distensibilidad disminuida de la región oral del estómago

d) Engrosamiento de la circular interna pilórica

8. ¿Cuál de las siguientes estructuras tiene inervación somática?

a) Estómago

b) Peritoneo visceral
c) Mesosigmoides

d) Peritoneo parietal

9. Estas diseñando un proyecto de investigación sobre los niveles de colesterol que se


absorben luego de una comida grasosa y deseas cuantificar la cantidad de colesterol
que es absorbido por el intestino antes que el hígado lo metabolice ¿de cuál de los
siguientes vasos obtendrías la muestra para tu análisis?

a) Vena porta

b) Vena cava superior

c) Conducto torácico

d) Vena hemiácigos accesoria

10. Durante el paso del bolo hacia la orofaringe, se desencadena una serie de
contracciones musculares que estrechan la cavidad faríngea. Estas contracciones están
mediadas por el nervio craneal:

a) X

b) XI

c) XII

d) IX

11. Cuando el istmo de las fauces se cierra, se evita que el alimento pase hacia la
orofarínge y permite respirar mientras se mastica. Este cierre se debe a la contracción y
aproximación de los músculos:

a) Palatoglosos

b) Palatofaríngeos

c) Estiloglosos

d) Estilofaríngeos

12. Las siguientes alternativas son factores que determinan la patencia y función
adecuada del esfínter esofágico inferior, EXCEPTO:

a) Plicatura diafragmática
b) Hipertrofia de la circular interna

c) Angulación con el estómago

d) Canales lentos de calcio

13. Respecto al peristaltismo intestinal, para cumplir la ley del intestino, usted espera
que a nivel distal del quimo se libere:

a) Péptido liberador de gastrina (GRP)

b) Acetilcolina

c) Péptido intestinal vasoactivo

d) Sustancia P

14. El estímulo habitual para el movimiento peristáltico es:

a) Acción de la sustancia P

b) Contracción de la musculatura circular Interna

c) Estimulación vago-vagal

d) Distensión local

15. ¿Cuál de las siguientes condiciones considera que es un trastorno de la


musculatura lisa esofágica?

a) Acalasia

b) Asinergia faringoesfinteriana

c) Hipotonía de los constrictores faríngeos

d) Hipertonía del esfínter esofágico superior

16. ¿Cuál de las siguientes alternativas es correcta sobre la motilidad esofágica?

a) Las ondas primarias son propulsoras y pueden no ser precedidas por


deglución

b) Las ondas primarias no son propulsoras y siempre van precedidas de


deglución
c) Las ondas secundarias son propulsoras y siempre van precedidas de
deglución

d) Las ondas secundarias son propulsoras y no van precedidas de


deglución

17. El peristaltismo depende que a nivel distal del bolo se secrete:

a) Péptido intestinal vasoactivo por neuronas

b) Noradrenalina secretada por las fibras del sistema simpático

c) Acetilcolina por las neuronas provenientes del nervio vago

d) Óxido nítrico por células endoteliales locales

18. ¿Cuál de las siguientes alternativas es correcta sobre el movimiento peristáltico?

a) Es un reflejo largo que depende de la integración con el tronco


encefálico

b) Se dirige en sentido distal siempre, nunca en sentido proximal

c) El contenido intestinal avanza sólo 5-10 cm

d) Es independiente del plexo mientérico

19. En un estudiante de medicina que está rindiendo un examen parcial, lo más


probable es que en ese momento su tránsito intestinal se encuentre:

a) Muy acelerado

b) Estimulado por acción de la sustancia p

c) Sin alteraciones

d) Muy lento

20. Las siguientes alternativas son ciertas sobre las contracciones tónicas del músculo
gastrointestinal, EXCEPTO:

a) Tienen relación con el ingreso persistente de iones sodio

b) Se encuentran principalmente en esfínteres


c) Tienen regulación hormonal

d) Obedece a una mayor frecuencia de potenciales en espiga

21. Con respecto de la regulación del pH del estómago; al utilizar un bloqueador de


histamina, usted espera que el pH del estómago:

a) Aumente

b) Disminuya

c) Se mantenga sin cambio

d) Se neutralice por acción de bicarbonato

22. Las siguientes hormonas disminuyen el vaciamiento gástrico, EXCEPTO:

a) Gastrina

b) Péptido insulinotrópico dependiente de glucosa

c) Colecistoquinina

d) Secretina

23. El frenillo de los labios se encuentra en:

a) El piso de la boca

b) La cavidad oral

c) La cavidad vestibular

d) El dorso de la lengua

24. En un paciente con shock hipovolémico, la peristalsis intestinal se encuentra:

a) Aumentada

b) Sin cambios

c) Invertida

d) Disminuida
25. Sobre el control de la peristalsis del tubo digestivo, ________________ es un
mediador neural que induce la relajación durante la peristalsis.

a) La somatostatina

b) El péptido intestinal vasoactivo

c) La acetilcolina

d) La serotonina

26. Las siguientes funciones son inhibidas por la hormona somatostatina, EXCEPTO:
a) motilidad intestinal
b) secreción de enzimas pancreaticas
c) Secrecion gastrica de HCl
d) Liberacion de gastrina

27. ¿Cuál de las siguientes alternativas es correcta sobre el control del peristaltismo?

A) El reflejo gastroentérico evita el avance del quimo


B) La secretina aumenta la motilidad del intestino delgado
C) El peristaltismo intestinal aumenta por efecto de la colecistoquinina (CCK)
D) La serotonina no tiene efecto sobre la motilidad digestiva

28. En el plexo mientérico, el origen de los impulsos eferentes está en:

a) el plexo de Auerbach
b) los ganglios paravertebrales
c) Las células intersticiales de cajal
d) plexo de meissner

29. Los corpúsculos gustativos se encuentran en la lengua, pero además se les puede
encontrar en:

a) el dorso de la lengua
b) el paladar blando
c) el paladar duro
d) las encías

30. ¿Cuál de las siguientes condiciones considera que es un trastorno de la musculatura lisa
esofágica?

a) Hipertonía del esfínter esofágico superior


b) Acalasia
c) Hipotonía de los constrictores faríngeos
d) Asinergia faringoesfinteriana
Dayanne Cruz

1. ¿Cuál de las siguientes alternativas detallan los músculos que ayudan a empujar el
bolo hacia la orofaringe?
a. Geniogloso y palatofaríngeo
b. Estilogloso y geniogloso
c. Hiogloso y geniogloso
d. Estilogloso y palatoglo
2. Paciente con lesión del hipogloso del lado izquierdo. Para evaluarlo se le pide al
paciente que saque la lengua, la cual se espera que la punta de la lengua se dirija
hacia:
a. abajo
b. el lado izquierdo
c. adelante
d. el lado derecho
3. Cuando el istmo de las fauces se cierra, se evita que el alimento pase hacia la
orofarínge y permite respirar mientras se mastica. Este cierre se debe a la
contracción y aproximación de los músculos
a. estiloglosos
b. palatofaríngeos
c. palatoglosos
d. estilofaríngeos
4. Las siguientes alternativas son ciertas sobre la actividad eléctrica del músculo
gastrointestinal, EXCEPTO:
a. Si el potencial de membrana es más positivo, habrá mayor frecuencia de
espigas
b. Las ondas lentas son más frecuentes en el duodeno
c. La despolarización lenta se debe principalmente al ingreso de Na+
d. Las ondas lentas no son potenciales de acción
5. Durante la deglución, el bolo es impedido de regresar a la cavidad oral gracias a la
acción de diversos músculos, entre ellos el músculo:
a. estilofaríngeo
b. palatofaríngeo
c. cricofaríngeo
d. salpingofaríngeo
6. En una cirugía abierta (laparotomía), el cirujano al abrir la cavidad peritoneal por la
parte anterior (línea media), lo primero que observa es
a. Estomago
b. Duodeno
c. Colon sigmoides
d. Epiplón mayor
7. Estas diseñando un proyecto de investigación sobre los niveles de colesterol que se
absorben luego de una comida grasosa y deseas cuantificar la cantidad de colesterol
que es absorbido por el intestino antes que el hígado lo metabolice ¿de cuál de los
siguientes vasos obtendrías la muestra para tu análisis?
a. Vena cava superior
b. Conducto torácico
c. Vena hemiácigos accesoria
d. Vena porta
8. Paciente de 34 años es víctima de asalto con arma de fuego, recibiendo un impacto
directo en el abdomen. En base a la radiografía, usted puede registrar en la historia
clínica que el proyectil se encuentra topográficamente en el:

a. flanco izquierdo
b. hipocondrio izquierdo
c. flanco derecho
d. mesogastrio

9. El nervio palatino menor inerva un área del paladar que está recubierta por
epitelio:

a. estratificado plano queratinizado


b. estratificado plano no queratinizado
c. simple plano
d. simple cilíndrico

10. ¿Cuál de las siguientes estructuras deriva del intestino anterior?

a. C
b. B
c. A
d. D

11. ¿Cuál de las siguientes atresias/fístulas traqueo esofágicas considera usted que es
incompatible con la vida (de no recibir tratamiento)?

a. Tipo E
b. Tipo B
c. Tipo C
d. Tipo A

12. Las siguientes hormonas disminuyen el vaciamiento gástrico, EXCEPTO:

a. Péptido insulinotrópico dependiente de glucosa


b. Secretina
c. Colecistoquinina
d. Gastrina

13. Las siguientes alternativas son ciertas en relación al mesenterio, EXCEPTO

a. El omento menor deriva del mesenterio ventral


b. El ligamento esplenorrenal une al bazo con el riñón izquierdo
c. Los mesenterios cumplen la función de sostén y suspensión de órganos
d. Los omentos cumplen una función principal de irrigación viscera

14. ¿Cuál de las siguientes alternativas es una característica de la estructura del esófago?

a. Contiene músculo estriado en casi toda su longitud


b. El esfínter esofágico inferior es un esfínter anatómico
c. Aumenta la presión intra esofágica durante la inspiración
d. Contiene glándulas submucosas principalmente en su tercio distal

15. ¿Cuál de las siguientes alternativas es correcta sobre el control del peristaltismo?

a. La serotonina no tiene efecto sobre la motilidad digestiva


b. El reflejo gastroentérico evita el avance del quimo
c. La secretina aumenta la motilidad del intestino delgado
d. El peristaltismo intestinal aumenta por efecto de la colecistoquinina (CCK)

16. En el plexo mientérico, el origen de los impulsos eferentes está en:

a. el plexo de Meissner
b. las células intersticiales de Cajal
c. el plexo de Aurbach
d. los ganglios paravertebrales

17. El peristaltismo depende que a nivel distal del bolo se secrete:

a. noradrenalina secretada por las fibras del sistema simpático


b. óxido nítrico por células endoteliales locales
c. acetilcolina por las neuronas provenientes del nervio vago
d. péptido intestinal vasoactivo por neuronas

18. En un estudiante de medicina que está rindiendo un examen parcial, lo más probable es
que

a. muy lento
b. sin alteraciones
c. estimulado por acción de la sustancia P
d. muy acelerado en ese momento su tránsito intestinal se encuentre:

19. Los reflejos enterogástricos son desencadenados por las siguientes circunstancias,
EXCEPTO:

a. Distensión de pared intestinal


b. Disminución de pH duodenal
c. Quimo con alta osmolaridad
d. Alta carga de carbohidratos en duodeno

20. La distención del yeyuno provoca que se:


a. disminuya el número de espigas
b. despolarice el potencial de reposo de membrana
c. produzca una contracción tónica
d. aumente la frecuencia de las ondas lentas

21. Respecto al peristaltismo intestinal, para cumplir la ley del intestino , usted espera que a
nivel distal del quimo se libere:
a. péptido intestinal vasoactivo
b. sustancia P
c. péptido liberador de gastrina (GRP)
d. acetilcolina
22. ¿Cuál de las siguientes alternativas es correcta sobre el movimiento peristáltico?
a. Es un reflejo largo que depende de la integración con el tronco encefálico
b. El contenido intestinal avanza sólo 5-10 cm
c. Es independiente del plexo mientérico
d. Se dirige en sentido distal siempre, nunca en sentido proximal
23. ¿Cuál de las siguientes alternativas estimula las ondas de motilidad gastrointestinal
denominadas complejos migratorios interdigestivos?
a. Metoclopramida
b. Alimentos
c. Eritromicina
d. Colecistoquinina (CCK)
24. ¿Cuál de las siguientes condiciones considera que es un trastorno de la musculatura lisa
esofágica?
a. Hipertonía del esfínter esofágico superior
b. Asinergia faringoesfinteriana
c. Acalasia
d. Hipotonía de los constrictores faríngeos
25. En la estructura dentaria, se observa que hay una composición muy similar a la del
hueso en la capa denominada
a. dentina
b. predentina.
c. esmalte
d. cemento

1. ¿Cuál de las siguientes alternativas detallan los músculos que ayudan a empujar el bolo
hacia la orofaringe?

- Estilogloso y palatogloso
- Hiogloso y geniogloso
- Geniogloso y palatofaríngeo

2. Paciente con lesión del hipogloso del lado izquierdo. Para evaluarlo se le pide al paciente que
saque la lengua, la cual se espera que la punta de la lengua se dirija hacia
- El lado izquierdo
- Adelante
- Abajo
- El lado derecho

3. Niña de 6 años se asusta porque se le ha aflojado un diente deciduo. Este fenómeno se


produce por:
- Laxitud del ligamento periodontal
- Aumento anómalo de la predentina
- Fractura del cemento
- Desmineralización del esmalte dental

4. Sobre el control de la peristalsis del tubo digestivo, ________________ es un mediador


neutral que induce la relajación durante la peristalsis
- El péptido intestinal vasoactivo
- Somatostatina
- Serotonina
- Acetilcolina

5. En el plexo mientérico , el origen de los impulsos eferentes esta en


- El plexo de Auerbach
- Los ganglios paravertebrales
- Plexo de Meissner
- Células intersticiales de Cajal
6. Los corpúsculos gustativos se encuentran en la lengua, pero además se les puede encontrar
en:
- El dorso de la lengua
- El paladar duro
- El paladar blando
- Las encías

7. La masticación es básicamente:
- Un movimiento reflejo
- Una actividad consciente
- Un ralentizador del vaciamiento gástrico
- Importante para la digestión sobre todo de carnes

8. Con respecto de la regulación del pH del estómago; al utilizar un bloqueador de histamina,


usted espera que el pH del estómago:
- Aumente
- Disminuya
- Neutraliza por acción de bicarbonato
- Se mantenga sin cambio

9. Al realizar una vagotomía por úlcera péptica, usted esperaría una disminución de producción
de gastrina debido a la:
- Ausencia de histamina
- Menor distensión de las paredes gástricas
- ausencia de bombesina vagal
- ausencia de acetilcolina vagal

10. Mujer de 30 años llega a emergencia con dolor en hipogástrico. Al examen físico presenta
una masa palpable de 10 cm de diámetro a en la misma región, usted sospecharía de las
siguientes condiciones, EXCEPTO:
- Tumor renal
- Cáncer de recto superior
- Embarazo
- Tumor uterino

11. En un estudiante de medicina que está rindiendo un examen parcial, lo más probable es que
en ese momento su tránsito intestinal se encuentre:
- Muy lento
- Estimulado por acción de la sustancia P
- muy acelerado
- sin alteraciones

12. ¿Cuál de las siguientes alternativas estimula las ondas de motilidad gastrointestinal
denominadas complejos migratorios interdigestivos?
- Eritromicina
- Colecistoquinina
- Alimentos
- Metoclopramida
13. Respecto al peristaltismo intestinal, para cumplir la ley del intestino , usted espera que a nivel
distal del quimo se libere:
- Péptido intestinal vasoactivo
- acetilcolina
- sustancia p
- péptido liberador de gastrina
14. ¿ Cual de las siguientes alternativas es correcta sobre el control autonomo del aparato
gastrointestinal?
- La estimulación simpática estimula a la muscularis mucosae
- Las terminaciones nerviosas simpáticas liberan mayor cantidad de adrenalina que
noradrenalina
- al seccionar al vago , la inervación parasimpática del colon sigmoides disminuye
- el plexo mientérico de auerbach cumple funciones inhibitorias
15. La distensión del yeyuno provoca que
- Aumente la frecuencia de ondas lentas
- disminuya el número de espigas
- produzca contracción tónica
- despolariza el potencial de reposo de membrana
Francisco Parcial
Cuando una persona coloca en su boca una sustancia con alta concentración de carbohidratos, lo
que debería pasar es que:
- aumente las concentraciones séricas de gastrina
- disminuya la sensación de hambre
- aumenta la motilidad gástrica
- se dilate el esfínter esofágico inferior

ANGELA PARCIAL:
1. Las carnes deben su sabor especialmente delicioso debido a que presentan en su
composición:
- glutamato
2. En un paciente es derivado por lesión del nervio vago derecho. Al evaluar el velo del
paladar, se solicita al paciente que diga ahh , entonces se puede observar que la úvula:
- se desvía a la izquierda
3. Al rozar agua caliente en la punta de la lengua, usted esperaría que el estímulo viaje a
través del nervio:
- lingual
4. Considerando sus conocimientos en embriología, la disposición del intestino delgado
final tras la retracción de asas intestinales, se distribuye de la siguiente manera:
- asas ileales en cuadrante inferior derecho
5. En un paciente de 3 semanas de edad, con vómitos en proyectil, y nódulo epigástrico
reptante, usted esperaría encontrar:
- engrosamiento de la circular interna pilórica
6. La masticación es básicamente:
- un movimiento reflejo
7. Un familiar le comenta que tiene úlcera gástrica por exceso de producción de ácido;
con sus conocimientos del sistema digestivo, usted le recomendaría que reduzca el
consumo de:
- aminoácidos
8. Con respecto de la regulación del pH del estómago; al utilizar un bloqueador de
histamina, usted espera que el pH del estómago:
- aumente
9. Paciente adulto con reflujo gastroesofágico es más probable que presente:
- descalcificación del esmalte
10. Sobre el control de la peristalsis del tubo digestivo, ________________ es un mediador
neural que induce la relajación durante la peristalsis.
- el péptido intestinal vasoactivo
11. En el plexo mientérico, el origen de los impulsos eferentes está en:
- el plexo de Meissner
12. Mujer de 30 años llega a emergencia con dolor en hipogástrico. Al examen físico
presenta una masa palpable de 10 cm de diámetro a en la misma región, usted
sospecharía de las siguientes condiciones, EXCEPTO:
- Tumor renal
13. Los reflejos enterogástricos son desencadenados por las siguientes circunstancias,
EXCEPTO:
- Alta carga de carbohidratos en duodeno
14. La hormona responsable de los complejos migratorios interdigestivos tiene las
siguientes características, EXCEPTO:
- cumple funciones de aumentar la motilidad y secreción gástrica e intestinal
15. La relajación receptiva gástrica, se produce principalmente por la acción de:
- el péptido intestinal vasoactivo
16. ¿Cuál de las siguientes alternativas estimula las ondas de motilidad gastrointestinal
denominadas complejos migratorios interdigestivos?
- Eritromicina
17. En un estudiante de medicina que está rindiendo un examen parcial, lo más probable
es que en ese momento su tránsito intestinal se encuentre:
- muy lento
18. El estímulo habitual para el movimiento peristáltico es:
- distensión local
19. La distención del yeyuno provoca que se:
- despolarice el potencial de reposo de membrana
20. El principal gobernante sobre todos los movimientos gastrointestinales es el sistema
nervioso:
- mientérico
21. Estas diseñando un proyecto de investigación sobre los niveles de colesterol que se
absorben luego de una comida grasosa y deseas cuantificar la cantidad de colesterol
que es absorbido por el intestino antes que el hígado lo metabolice ¿de cuál de los
siguientes vasos obtendrías la muestra para tu análisis?
- Conducto torácico
22. El tubo digestivo posee glándulas, las glándulas submucosas se encuentran en el:
- esófago y duodeno
23. Durante la deglución, el bolo es impedido de regresar a la cavidad oral gracias a la
acción de diversos músculos, entre ellos el músculo:
- palatofaríngeo
24. Cuando el istmo de las fauces se cierra, se evita que el alimento pase hacia la
orofarínge y permite respirar mientras se mastica. Este cierre se debe a la contracción
y aproximación de los músculos:
- palatoglosos
25. El músculo liso gastrointestinal funciona como un sincitio debido a:
- las uniones en hendidura

En el esófago, el plexo mientérico o de Auerbach interviene en el _______ de los impulsos ______

a. fin / aferentes
b. fin / eferentes
c. inicio / eferentes
d. inicio / aferentes

Pregunta 1
El nervio palatino menor inerva un área del paladar que está recubierta por epitelio:
● estratificado plano no queratinizado
● simple plano
● estratificado plano queratinizado
● simple cilíndrico
Pregunta 2
¿Cuál de las siguientes atresias/fístulas traqueo esofágicas considera usted que es
incompatible con la vida (de no recibir tratamiento)?
● Tipo A
● Tipo B
● Tipo C
● Tipo E
Pregunta 3
¿Cuál de las siguientes estructuras deriva del intestino anterior?
● A
● D
● C
● B
Pregunta 4
El tubo digestivo posee glándulas, las glándulas submucosas se encuentran en el:
● esófago y duodeno
● duodeno y recto
● íleon y esófago
● estómago y duodeno
Pregunta 5
Estas diseñando un proyecto de investigación sobre los niveles de colesterol que se
absorben luego de una comida grasosa y deseas cuantificar la cantidad de colesterol que es
absorbido por el intestino antes que el hígado lo metabolice ¿de cuál de los siguientes vasos
obtendrías la muestra para tu análisis?
● Vena hemiácigos accesoria
● Vena porta
● Conducto torácico
● Vena cava superior
Pregunta 6
Un familiar le comenta que tiene úlcera gástrica por exceso de producción de ácido; con sus
conocimientos del sistema digestivo, usted le recomendaría que reduzca el consumo de:
● vitaminas
● agua
● carbohidratos
● aminoácidos
Pregunta 7
Con respecto de la regulación del pH del estómago; al utilizar un bloqueador de histamina,
usted espera que el pH del estómago:
● se neutralice por acción de bicarbonato
● se mantenga sin cambio
● disminuya
● aumente
Pregunta 8
¿Cuál de las siguientes alternativas es una característica de la estructura del esófago?
● El esfínter esofágico inferior es un esfínter anatómico
● Contiene músculo estriado en casi toda su longitud
● Contiene glándulas submucosas principalmente en su tercio distal
● Aumenta la presión intra esofágica durante la inspiración
Pregunta 9
¿Cuál de las siguientes alternativas detallan los músculos que ayudan a empujar el bolo
hacia la orofaringe?
● Hiogloso y geniogloso
● Geniogloso y palatofaríngeo
● Estilogloso y geniogloso
● Estilogloso y palatogloso
Pregunta 10
Al ingresar líquidos o sólidos en la cavidad oral, un mecanismo que permite que una
persona respire mientras mastica es:
● la depresión del paladar blando
● la elevación del paladar blando
● el movimiento hacia afuera de los pliegues palatogloso y palatofaríngeo
● la depresión de la parte posterior de la lengua
Pregunta 11
Niña de 6 años se asusta por que se le ha aflojado un diente deciduo. Este fenómeno se
produce por:
● laxitud del ligamento periodóntico
● desmineralización del esmalte dental
● aumento anómalo de la predentina
● fractura del cemento
Pregunta 12
Con seguridad, usted puede decir que la siguiente cicatriz postapendicectomía se encuentra
en la región denominada:
● flanco derecho
● hipocondrio derecho
● fosa iliaca derecha
● flanco izquierdo
Pregunta 13
Cuando el istmo de las fauces se cierra, se evita que el alimento pase hacia la orofarínge y
permite respirar mientras se mastica. Este cierre se debe a la contracción y aproximación de
los músculos:
● estiloglosos
● palatofaríngeos
● estilofaríngeos
● palatoglosos
Pregunta 14
Durante el paso del bolo hacia la orofaringe, se desencadena una serie de contracciones
musculares que estrechan la cavidad faríngea. Estas contracciones están mediadas por el
nervio craneal:
● X
● IX
● XII
● XI
Pregunta 15
Durante la deglución, el bolo es impedido de regresar a la cavidad oral gracias a la acción
de diversos músculos, entre ellos el músculo:
● palatofaríngeo
● estilofaríngeo
● salpingofaríngeo
● cricofaríngeo
Pregunta 16
En el plexo mientérico, el origen de los impulsos eferentes está en:
● el plexo de Aurbach
● el plexo de Meissner
● las células intersticiales de Cajal
● los ganglios paravertebrales
Pregunta 17
¿Cuál de las siguientes alternativas es correcta sobre el control del peristaltismo?
● La serotonina no tiene efecto sobre la motilidad digestiva
● El reflejo gastroentérico evita el avance del quimo
● La secretina aumenta la motilidad del intestino delgado
● El peristaltismo intestinal aumenta por efecto de la colecistoquinina (CCK)
Pregunta 18
En un estudiante de medicina que está rindiendo un examen parcial, lo más probable es que
en ese momento su tránsito intestinal se encuentre:
● estimulado por acción de la sustancia P
● sin alteraciones
● muy acelerado
● muy lento
Pregunta 19
El peristaltismo intestinal se produce gracias a un reflejo que:
● llega a los ganglios pre vertebrales
● llega a la médula espinal
● se origina dentro de la pared intestinal
● llega al sistema nervioso central
Pregunta 20
El estímulo habitual para el movimiento peristáltico es:
● distensión local
● acción de la sustancia P
● contracción de la musculatura circular Interna
● estimulación vago-vagal
Pregunta 21
¿Cuál de las siguientes alternativas es correcta sobre el control autónomo del aparato
gastrointestinal?
● La estimulación simpática estimula a la muscularis mucosae
● El plexo mientérico de Auerbach cumple funciones inhibitorias
● Las terminaciones nerviosas simpáticas liberan mayor cantidad de adrenalina que
noradrenalina
● Al seccionar el vago, la inervación parasimpática del colon sigmoides disminuye
Pregunta 22
El principal gobernante sobre todos los movimientos gastrointestinales es el sistema
nervioso:
● simpático
● somático
● mientérico
● parasimpático
Pregunta 23
¿Cuál de las siguientes alternativas estimula las ondas de motilidad gastrointestinal
denominadas complejos migratorios interdigestivos?
● Eritromicina
● Colecistoquinina (CCK)
● Alimentos
● Metoclopramida
Pregunta 24
Con respecto a la regulación del peristaltismo, al aplicarle atropina (antagonista colinérgico)
a un paciente, es de esperarse que el peristaltismo:
● disminuya
● estimule la acción de los receptores dopaminérgicos
● se mantenga sin alteración
● aumente
Pregunta 25
¿Cuál de las siguientes alternativas es correcta sobre el movimiento peristáltico?
● Es independiente del plexo mientérico
● Es un reflejo largo que depende de la integración con el tronco encefálico
● El contenido intestinal avanza sólo 5-10 cm
● Se dirige en sentido distal siempre, nunca en sentido proximal

¿Cuál de las siguientes alternativas es una característica de la estructura del esófago?

- Contiene glándulas submucosas principalmente en su tercio distal

- Aumenta la presión intra esofágica durante la inspiración

- El esfínter esofágico inferior es un esfínter anatómico

Las siguientes hormonas disminuyen el vaciamiento gástrico, EXCEPTO:

- Gastrina

- Péptido insulinotrópico dependiente de glucosa

- Secretina

- Colecistoquinina

Luego de una comida rica en proteínas, se espera que los valores de secretina en sangre:

- sólo se altere si hay un aumento de colecistoquinina (CCK)

- se mantenga sin cambio

- aumenten

- disminuyan

¿Cuál de las siguientes estructuras tiene inervación somática?

- Peritoneo parietal

- Mesosigmoides

- Peritoneo visceraL

- Estómago
Estas diseñando un proyecto de investigación sobre los niveles de colesterol que se
absorben luego de una comida grasosa y deseas cuantificar la cantidad de colesterol que es
absorbido por el intestino antes que el hígado lo metabolice ¿de cuál de los siguientes vasos
obtendrías la muestra para tu análisis?

- Vena porta

- Vena hemiácigos accesoria

- Vena cava superior

- Conducto torácico

Durante el paso del bolo hacia la orofaringe, se desencadena una serie de contracciones
musculares que estrechan la cavidad faríngea. Estas contracciones están mediadas por el
nervio craneal:

- X

- XII

- IX

- XI

Las siguientes alternativas son factores que determinan la patencia y función adecuada del
esfínter esofágico inferior, EXCEPTO:

- Hipertrofia de la circular interna

- Angulación con el estómago

- Canales lentos de calcio

- Canales lentos de calcio

Cuando el istmo de las fauces se cierra, se evita que el alimento pase hacia la orofarínge y
permite respirar mientras se mastica. Este cierre se debe a la contracción y aproximación de
los músculos:

- palatofaríngeos

- estilofaríngeos

- estiloglosos

- Palatogloso

En la estructura dentaria, se observa que hay una composición muy similar a la del hueso
en la capa denominada:

- dentina
- predentina

- cemento

- esmalte

El frenillo de los labios se encuentra en:

- el dorso de la lengua

- la cavidad vestibular

- el piso de la boca

- la cavidad oral

Es un derivado del mesenterio dorsal:

En un paciente con falla en la fusión de los conductos de las yemas ventral y dorsal del
páncreas, usted esperaría encontrar:

- drenaje de la mayor parte del jugo pancreático en la papila menor

- drenaje adecuado del jugo pancreático

- estenosis del duodeno

- drenaje de la mayor parte del jugo pancreático en la papila mayor

Mujer de 30 años llega a emergencia con dolor en hipogástrico. Al examen físico presenta
una masa palpable de 10 cm de diámetro a en la misma región, usted sospecharía de las
siguientes condiciones, EXCEPTO:

- Tumor renal

- Tumor uterino

- Cáncer de recto superior

- Embarazo

El principal gobernante sobre todos los movimientos gastrointestinales es el sistema


nervioso:

- Mientérico
- Simpático

- Parasimpático

- Somático

La distención del yeyuno provoca que se:

- despolarice el potencial de reposo de membrana

- disminuya el número de espigas

- produzca una contracción tónica

- aumente la frecuencia de las ondas lentas

La información sensitiva aferente del sistema gastrointestinal pasa por las siguientes
estructuras, EXCEPTO:

- Tronco encefálico

- Ganglios prevertebrales

- Médula espinal

- Tálamo

¿Cuál de las siguientes alternativas es correcta sobre la motilidad esofágica?

- Las ondas secundarias son propulsoras y no van precedidas de deglución

- Las ondas primarias no son propulsoras y siempre van precedidas de deglución

- Las ondas primarias son propulsoras y pueden no ser precedidas por deglución

- Las ondas secundarias son propulsoras y siempre van precedidas de deglución

¿Cuál de las siguientes alternativas es correcta sobre el control autónomo del aparato
gastrointestinal?

- La estimulación simpática estimula a la muscularis mucosae

- Las terminaciones nerviosas simpáticas liberan mayor cantidad de adrenalina que


noradrenalina

- Al seccionar el vago, la inervación parasimpática del colon sigmoides disminuye

- El plexo mientérico de Auerbach cumple funciones inhibitorias

El peristaltismo intestinal se produce gracias a un reflejo que:

● - llega al sistema nervioso central


● - se origina dentro de la pared intestinal
● - llega a los ganglios prevertebrales
● - llega a la médula espinal

¿Cuál de las siguientes alternativas estimula las ondas de motilidad gastrointestinal


denominadas complejos migratorios interdigestivos?

- Metoclopramida

- Colecistoquinina (CCK)

- Eritromicina

- Alimentos

El peristaltismo depende que a nivel distal del bolo se secrete:

- acetilcolina por las neuronas provenientes del nervio vago

- óxido nítrico por células endoteliales locales

- péptido intestinal vasoactivo por neuronas

- noradrenalina secretada por las fibras del sistema simpático

Sobre el control de la peristalsis del tubo digestivo, ________________ es un mediador


neural que induce la relajación durante la peristalsis.

- el péptido intestinal vasoactivo

- la somatostatina

- la serotonina

- la acetilcolina

En el esófago, el plexo mientérico o de Auerbach interviene en el _______ de los impulsos


______

- inicio / eferentes

- fin / eferentes

- inicio / aferentes

- fin / aferentes

Al rozar agua caliente en la punta de la lengua, usted esperaría que el estímulo viaje a
través del nervio:

- cuerda del tímpano

- lingual
- hipogloso

- glosofaríngeo

¿Cuál de las siguientes alternativas detallan los músculos que ayudan a empujar el bolo
hacia la orofaringe?

- Estilogloso y palatogloso

- Estilogloso y geniogloso

- Hiogloso y geniogloso

- Geniogloso y palatofarínge
CI3 - Francisco

Los pliegues gástricos gruesos son prácticamente inexistentes a nivel de:

el fondo

En un paciente con xerostomía presenta las siguientes condiciones, EXCEPTO:

Infecciones del oído a repetición

En fases iniciales, la infección por Helicobacter pylori genera disminución del pH gástrico
debido a:

la disminución de somatostatina

La fase intestinal de la secreción gástrica se debe básicamente a la participación de las


células:

G del duodeno

Los siguientes mecanismos determinan la patencia del esfínter esofágico inferior y ayudan a
impedir la enfermedad por reflujo gastroesofágico, EXCEPTO:

Longitud mayor del esófago

A raíz de la infección por Helicobacter pilory, la destrucción de las células ____________


ocasiona que aumente la producción de ácido en el estómago y por ello se produzca úlceras
____________

D / duodenales

En caso se produzca la perforación de la cara anterior del estómago, esta perforación


ocasionará _________ producto de la peritonitis química.

íleo intestinal

Al usar un parasimpaticomimético (agonista colinérgico), usted espera que la saliva


presente:

una mayor cantidad de sodio

La saliva siempre será hipotónica debido a:

la impermeabilidad de los conductos al agua

En la producción de HCl, la acción de la somatostatina disminuye la accion de:

la gastrina
CI3 DE DIGESTIVO (LES)

En una gestante de 11 semanas con antecedente de esofagitis eosinofílica, con hiperémesis


gravídica, que acude a emergencia por hematemesis leve, y presenta súbitamente disnea y
dolor torácico. Usted sospecharía de:

a) síndrome de Mallory Weiss


b) neumonía
c) esofagitis erosiva severa por reflujo
d) síndrome de Boerhaav

La fase intestinal de la secreción gástrica se debe básicamente a la participación de las


células:

a) G del duodeno
b) I del yeyuno
c) D del estómago
d) S del íleon

La vena porta se forma gracias a la unión de la vena mesentérica superior con la vena:

a) esplénica
b) mesentérica inferior
c) celiaca
d) gástrica izquierda

Aquellas células que producen mayor cantidad de moco en el epitelio gástrico son las:

a) mucosas superficiales
b) enterocromafines
c) mucosas del cuello
d) mucosas del fondo

La saliva siempre será hipotónica debido a:

a) la mayor permeabilidad al sodio en los conductos


b) el estímulo del sistema parasimpática
c) la impermeabilidad de los conductos al agua
d) la ausencia de conductor estriado

Para determinar que un paciente tiene esófago de Barrett, debemos encontrar ___________
en la biopsia de esófago.

a) células caliciformes
b) displasia
c) epitelio gástrico
d) glándulas cardiales

Paciente con acalasia es sometido a tratamiento endoscópico o quirúrgico, usted le ha


informado al paciente previamente que es posible que una complicación de este tratamiento
es que quede con cierto grado de:

a) reflujo gastroesofágico
b) úlceras gástricas
c) odinofagia
d) gastritis

¿Cuál de las siguientes alternativas es una causa de reflujo gastroesofágico de contenido


ácido?

a) Acalasia
b) Divertículo de Zenker
c) Hernia Hiatal
d) Anillo de Schatzki

En caso se produzca la perforación de la cara anterior del estómago, esta perforación


ocasionará _________ producto de la peritonitis química.

a) mayor secreción de colecistoquinina


b) íleo intestinal
c) esófago de Barret
d) mayor secreción de colecistoquinina
e) úlcera duodenal

La sangre que lleva la vena porta es tipo:

a) venosa
b) mixta
c) arterial

1. Al evaluar a un paciente con parotiditis purulenta (con absceso), usted buscaría el orificio
terminal del conducto de _____________ a la altura de _____________ → Stenon / la
segunda molar superior

2. Para bloquear la principal fuerza promotora de producción de ácido clorhídrico, lo ideal es un


medicamento que sea bloqueador de la bomba: de hidrógeno-potasio

3. En fases iniciales, la infección por Helicobacter pylori genera disminución del pH gástrico debido a →
la disminución de somatostatina
4. La comunicación entre la irrigación gástrica y la esofágica depende de una rama de la
arteria:tronco celiaco
5. Los pliegues gástricos gruesos son prácticamente inexistentes a nivel de → el fondo
6. En pacientes con hernia hiatal, la constante inflamación del esófago, puede aparecer
_______________ como complicación debida a la fibrosis por inflamación crónica. → anillo
de Schatzki
7. En un paciente con xerostomía presenta las siguientes condiciones, EXCEPTO:
Infecciones del oído a repetición

ANGELA CI-1:
1) Paciente de 24 años con dolor abdominal tipo cólico intenso en mesogastrio. Según
sus conocimientos de macroestructura, el origen del dolor puede ser el
___________:
Respuestas:
a) Colon
b) Íleon
c) Esófago
d) Estómago

2) Paciente se queja de dolor en hipocondrio derecho, pero superficialmente. El dermatoma


relacionado es (marque la mejor respuesta):
Respuestas:
a) T9
b) T11
c) T12
d) T10

3) Señale la respuesta correcta:


Respuestas:
a) El esófago sólo tiene adventicia
b) El colon ascendente sólo tiene serosa
c) El apéndice cecal sólo tiene serosa
d) El páncreas sólo tiene adventicia

4) Cuál de las siguientes estructuras no tiene vasos sanguíneos:


Respuestas:
a) Ligamento
b) Mesenterio
c) Epitelio intestinal
d) Omento

5) Paciente tiene una úlcera sangrante en el segundo tercio del Yeyuno. La arteria de la cual
proviene la sangre arterial para dicha zona es la arteria:
Respuestas:
a) Tronco celíaco
b) Mesentérica inferior

c) Mesentérica superior
d) Gástrica izquierda
e) Iliaca común

6) Al retirar completamente el mesenterio de un órgano, el mismo se vería afectado


principalmente en su:
Respuestas:
a) Inervación
b) Irrigación
c) Tamaño
d) No se afecta en absoluto

7) Respecto a la anatomía del estómago, marque lo correcto:


Respuestas:
a) El píloro se encuentra en el cuerpo gástrico
b) La incisura angularis puede estar en la curvatura mayor.
c) La porción más distal del estómago es el cardias
d) El fondo gástrico forma la curvatura mayor

8) Al examinar a un paciente, usted encuentra dolor localizado en fosa iliaca derecha y


diagnostica apendicitis. En este paciente, usted puede inferir:
Respuestas:
a) Hay inflamación de todo el peritoneo parietal (peritonitis)
b) El peritoneo parietal regional está afectado
c) El peritoneo visceral regional está principalmente afectado
d) El diagnóstico está errado por no corresponder a la región abdominal adecuada

9) Marque el órgano que se considera retroperitoneal:


Respuestas:
a) Lóbulo izquierdo del hígado
b) Vesícula biliar
c) Parte de la vía biliar
d) Sigmoides

10) La peristalsis o peristaltismo hace referencia a:


Respuestas:
a) Motilidad para mezclado de alimentos.
b) No es parte de la motilidad
c) Motilidad para fraccionamiento de alimentos.
d) Motilidad para movilizar el alimento de proximal a distal.

ANGELA CI-3:

1. Al usar atropina en una paciente, usted esperaría:


a) la disminución de gastrina
b) la disminución de secreción gástrica por bloqueo de M3
c) la hipersalivación
d) el aumento de histamina

2. Para determinar que un paciente tiene esófago de Barret, debemos encontrar


______________ en la biopsia de esófago.
a) células caliciformes
b) displasia
c) glándulas cardiales
d) epitelio gástrico

3. Paciente con acalasia es sometido a tratamiento endoscópico o quirurgico, usted le


ha informado al paciente previamente que es posible que una complicación de este
tratamiento es que quede con cierto grado de:
a) reflujo gastroesofágico
b) gastritis
c) odinofagia
d) úlceras gástricas

4. En un paciente con xerostomía presenta las siguientes condiciones, EXCEPTO:


a) Caries
b) Infecciones del oído a repetición
c) Disfagia
d) Erosiones de la mucosa oral

5. A raíz de la infección por Helicobacter pilory, la destrucción de las células


____________ ocasiona que aumente la producción de ácido en el estómago y por
eso se produzca úlceras _____________
a) S / pancreáticas
b) D / duodenales
c) parietales / gástricas
d) G / gástricas

6. Cuando un paciente recibe estímulo autónomo mixto (simpático y parasimpático), el


flujo de saliva:
a) es mayor que frente a un estímulo parasimpático aislado
b) aumenta en relación al basal
c) es menor que frente a un estímulo simpático aislado
d) disminuye en relación al basal

7. El conducto de Stenon, para entrar a la cavidad vestibular, debe atravesar el


músculo:

a) genihioideo
b) buccinador
c) masetero
d) milohioideo

8. Al evaluar a un paciente con parotiditis purulenta (con absceso), usted buscaría el


orificio terminal del conducto de _____________ a la altura de _____________

a) Stenon / la segunda molar superior


b) Stenon / las carúnculas
c) Wharton / el frenillo sublingual
d) Wharton / el piso de la boca

9. Los siguientes mecanismos determinan la patencia del esfínter esofágico inferior y


ayudan a impedir la enfermedad por reflujo gastroesofágico, EXCEPTO:

a) Canales lentos de calcio


b) Plicatura diafragmática
c) Ángulo de Hiss
d) Longitud mayor del esófago
10. La sangre que lleva la vena porta es tipo:

a) venosa
b) mixta
c) arterial
ANGELA- CI4:

1) Con respecto a la microestructura del hígado ¿Cuál de las siguientes alternativas es


correcta?

Respuestas:

a) Los hepatocitos están interconectados por uniones herméticas


b) La célula de Kupffer se encuentra fuera del sinusoide y fagocita células
c) La célula de Ito se encuentra en el espacio de Disse y reserva glucógeno
d) El sinusoide es un capilar fenestrado

2) La pancreatitis aguda puede producirse debido a una obstrucción en el flujo del conducto
de Wirsung, debido a un cálculo biliar que obstruye:

Respuestas:

a) el conducto de Wirsung
b) la ampolla de Vater
c) la papila menor
d) el tercio medio del colédoco

3) En un paciente con una estenosis severa del colédoco por una complicación quirúrgica,
usted esperaría que desarrolle:

Respuestas:

a) el aumento de transaminasas mayor que el de fosfatasa alcalina


b) hiperbilirrubinemia no conjugada
c) la cabeza de Medusa
d) el aumento del tiempo de protrombina

4) Marque lo correcto respecto al acino hepático:

Respuestas:

a) La zona 1 es la más que recibe más sangre


b) La zona 3 se afecta en menos en una deshidratación severa
c) Dos vértices del rombo acinar se constituyen por triadas portales
d) El eje menor lo constituye un eje imaginario entre dos venas centrolobulillares

5) En un paciente cirrótico con encefalopatía hepática ¿cuál de las siguientes alternativas


sustenta la reducción de carnes rojas en la dieta?

Respuestas:

a) Hasta el 80% del colesterol se transforma en sales biliares


b) El factor de crecimiento hepatocitario fomenta la regeneración del hepatocito
c) El amonio se produce principalmente en el intestino
d) La fructosa y la galactosa se convierten en glucosa en el hígado
6) La relajación del esfínter de Oddi se produce directamente por acción de:

Respuestas:

a) la acetilcolina
b) la colecistoquinina (CCK)
c) la sustancia P
d) el péptido vasoactivo intestinal (VIP)

7) Para aumentar la cantidad de bicarbonato en el flujo pancreático es necesario:

Respuestas:

a) inhibir al nervio vago


b) estimular las células S
c) ingerir aminoácidos como triptófano o fenilalanina
d) estimular la secreción de péptido liberador de colecistoquinina (CCK)

8) La bilis que sale de la vesícula biliar tiene como componente principal:

Respuestas:

a) a la bilirrubina
b) al colesterol
c) a los fosfolípidos
d) a los ácidos biliares

9) La presión parcial de oxígeno en la Zona 1 del sinusoide hepático de debe ser


___________ mmHg

Respuestas:

a) 100
b) entre 95 y 45
c) 40
d) menor de 40

10) Los hepatocitos tienen una gran capacidad regenerativa, en parte gracias a las células
madre hepáticas que se localizan en los:

Respuestas:

a) canales de Herring
b) sinusoides hepáticos
c) espacios de Mall
d) espacios de Disse
CI3 - José G

1. Para bloquear la principal fuerza promotora de producción de ácido clorhídrico, lo


ideal es un medicamento que sea bloqueador de la bomba:
a. de potasio
b. de hidrógeno-potasio
c. de cloro asociado a fibrosis quística
d. sodio-potasio

2. Al usar un parasimpaticomimético (agonista colinérgico), usted espera que la saliva


presente:
a. una menor cantidad de cloro
b. una menor cantidad de bicarbonatos
c. una mayor cantidad de sodio
d. Hipertonicidad

3. En la glándula fúndica, se evidencia células madre o stem cells a nivel de:


a. la foveola
b. el fondo
c. el istmo
d. el cuello

4. La vena porta se forma gracias a la unión de la vena mesentérica superior con la


vena:
a. mesentérica inferior
b. gástrica izquierda
c. esplénica
d. celiaca

5. En la producción de HCl, la acción de la somatostatina disminuye la accion de:


a. la gastrina
b. las prostaglandinas
c. la histamina
d. la acetilcolina

6. ¿Cuál de las siguientes alternativas es una causa de reflujo gastroesofágico de


contenido ácido?
a. Anillo de Schatzki
b. Diertículo de Zenker
c. Hernia Hiatal
d. Acalasia
7. En caso se produzca la perforación de la cara anterior del estómago, esta
perforación ocasionará _________ producto de la peritonitis química.
a. íleo intestinal
b. mayor secreción de colecistoquinina
c. esófago de Barret
d. úlcera duodenal

8. Dentro de las patologías que producen sangrado en el esófago, la que sangra más
es:
a. la esofagitis por reflujo gastroesofágico
b. el divertículo de Zenker
c. el síndrome de Mallory Weiss
d. el síndrome de Boerhaave
9. La fase intestinal de la secreción gástrica se debe básicamente a la participación de
las células:
a. I del yeyuno
b. S del íleon
c. G del duodeno
d. D del estómago

10. Las células enteroendocrinas en el estómago se localizan en la glándula oxíntica, al


mismo nivel que las células:
a. parietales
b. absortivas
c. principales
d. mucosas

CI 3: io

En las glándulas salivales, el principal lugar de intercambio iónico se da a nivel del:

A. conducto excretor
B. conducto estriado
C. conducto intercalar
D. acino

Al usar atropina en un paciente, usted esperaría:

A. la disminución de gastrina
B. la disminución de secreción gástrica por bloqueo de M3
C. el aumento de histamina
D. la hipersalivación
En un paciente con acalasia, se presentará una disminución de ___________________ en
el esfínter esofágico inferior.

A. acetilcolina
B. sustancia P
C. péptido intestinal vasoactivo
D. colecistoquinina

En cuanto a las sustancias secretadas por el estómago ¿Cuál de las siguientes sustancias estimula la
liberación de pepsinógeno?

A. Pepsinógeno
B. Secretina
C. Colecistoqunina
D. Gastrina

Las células enteroendocrinas en el estómago se localizan en la glándula oxíntica, al mismo nivel que
las células

A. parietales
B. principales
C. mucosas
D. absortivas

La fase intestinal de la secreción gástrica se debe básicamente a la participación de las


células

A. G del duodeno
B. I del yeyuno
C. S del íleon
D. D del estómago

El conducto de Stenon, para entrar a la cavidad vestibular, debe atravesar el músculo:

A. genihioideo
B. buccinador
C. milohioideo
D. masetero

Paciente con acalasia es sometido a tratamiento endoscópico o quirúrgico, usted le ha


informado al paciente previamente que es posible que una complicación de este tratamiento
es que quede con cierto grado de:

A. reflujo gastroesofágico
B. gastritis
C. odinofagia
D. úlceras gástricas

La sangre que lleva la vena porta es tipo:


A. mixta
B. arterial
C. venosa

Los pliegues gástricos gruesos son prácticamente inexistentes a nivel de:

A. el cuerpo
B. el fondo
C. la incisura angularis
D. el antro

En la producción de HCl, la acción de la somatostatina disminuye la accion de:

A. las prostaglandinas
B. la histamina
C. la acetilcolina
D. la gastrina

En el estómago se secretan las siguientes sustancias, EXCEPTO:

A. grelina
B. motilina
C. somatostatina
D. gastrina

La vena porta se forma gracias a la unión de la vena mesentérica superior con la vena:

A. mesentérica inferior
B. esplénica
C. celiaca
D. gástrica izquierda

CI3 - PEQUE

La saliva siempre será hipotónica debido a:

A. la mayor permeabilidad al sodio en los conductos


B. el estímulo del sistema parasimpático
C. la impermeabilidad de los conductos al agua
D. la ausencia de conductor estriado

En la producción de HCl, la acción de la somatostatina disminuye la acción de:

A. la histamina
B. las prostaglandinas
C. la gastrina
D. la acetilcolina

Un paciente con polimiositis posee alteración en la regulación del mecanismo de la


deglución; por eso hay que considerar la deglución de la saliva, pues a diario se produce
_______mL
A. 300
B. 500
C. 100
D. 1000

Al evaluar a un paciente con parotiditis purulenta (con absceso), usted buscaría el orificio
terminal del conducto de _____________ a la altura de _____________

A. Wharton / el piso de la boca


B. Stenon / la segunda molar superior
C. Wharton / el frenillo sublingual
D. Stenon / las carúnculas

En un paciente con acalasia, se presentará una disminución de ___________________ en


el esfínter esofágico inferior.

A. sustancia P
B. acetilcolina
C. colecistoquinina
D. péptido intestinal vasoactivo

En pacientes con hernia hiatal, la constante inflamación del esófago, puede aparecer
_______________ como complicación debida a la fibrosis por inflamación crónica.

A. anillo de Schatzki
B. esófago de Barret
C. divertículo de Zenker
D. reflujo gastroesofágico

Los pliegues gástricos gruesos son prácticamente inexistentes a nivel de:

A. el cuerpo
B. la incisura angularis
C. el fondo
D. el antro

Paciente con acalasia es sometido a tratamiento endoscópico o quirúrgico, usted le ha


informado al paciente previamente que es posible que una complicación de este tratamiento
es que quede con cierto grado de:

A. gastritis
B. úlceras gástricas
C. odinofagia
D. reflujo gastroesofágico

¿Cuál de las siguientes alternativas es una causa de reflujo gastroesofágico de contenido


ácido?

A. Acalasia
B. Diertículo de Zenke
C. Anillo de Schatzki
D. Hernia Hiatal

En las glándulas salivales, el principal lugar de intercambio iónico se da a nivel del:

A. conducto intercalar
B. conducto excretor
C. conducto estriado
D. acino

CI4 - Francisco

En un paciente con cirrosis hepática avanzada se encuentra tendencia a:

la hipoglicemia

En un paciente con infección por SARS-CoV-2 con compromiso severo e ingresado en la


unidad de cuidados intensivos (UCI), debido a la tormenta de citoquinas y a la desregulación
inmune ¿Cuál de las siguientes alternativas estará elevada en sangre al evaluar el perfil
hepático?

transaminasas

Para poder absorber al torrente sanguíneo todos los carbohidratos del azúcar común (de
mesa) es necesario usar únicamente los transportadores:

GLUT2, SGLT1, GLUT5

En la regulación de la glucosa sérica participa el hígado ¿Cuál de las siguientes alternativas


evalúa mejor esta función hepática?

Glucosa sérica en ayunas

En un paciente con intoxicación por órganos fosforados, la acción de la colecistoquinina


(CCK) está bloqueada a nivel de:

el esfínter de Oddi

En el hígado, el aumento de la resistencia vascular en los sinusoides hepáticos ocasionará:

salida de plasma hacia el intersticio

En un paciente cirrótico con encefalopatía hepática ¿cuál de las siguientes alternativas


sustenta la reducción de carnes rojas en la dieta?

El amonio se produce principalmente en el intestino

¿Cuál de las siguientes alternativas es correcta sobre la estructura hepática?

El flujo biliar en el lobulillo hepático es centrífugo

Las siguientes sustancias son secretadas por el páncreas, EXCEPTO:

Tripsina
Un paso importante para la poder absorber los lípidos es la emulsificación de las grasas.
¿Cuál de las siguientes alternativas es el principal factor encargado de dicha emulsificación?

Ácidos biliares

CI 4 : Clau A.

● Para aumentar la cantidad de bicarbonato en el flujo pancreático es necesario:

estimular las células S

ingerir aminoácidos como triptófano o fenilalanina

inhibir al nervio vago

estimular la secreción de péptido liberador de colecistoquinina (CCK)

● Marque la respuesta correcta:

El hígado recibe principalmente sangre venosa

Funcionalmente, contiene dos lóbulos.

Tiene una función endocrina pero no exocrina

Pesa aproximadamente 2,5 kg

● En un paciente con intoxicación por órganos fosforados, la acción de la


colecistoquinina (CCK) está bloqueada a nivel de:

la vesícula biliar

la célula parietal

el esfínter de Oddi

el sistema nervioso central

● En el síndrome de Mirizzi, el paciente tiene cálculos en la vesícula biliar; pero se


obstruye el conducto hepático común debido a que un cálculo se ubica y crece de
tamaño en

el colédoco

el conducto accesorio de Lushka

la válvula espiral de Heister

la bolsa de Hartmann

● ¿Cuál de las siguientes alternativas es correcta sobre la estructura hepática?

El flujo biliar en el lobulillo hepático es centrífugo

Los colangiocitos producen bilis


En la triada portal, se encuentra la vena derivada de la suprahepática

El flujo sinusoidal en el lobulillo hepático es de adentro hacia afuera

● En un recién nacido menor de 24 horas con atresia biliar, se encuentra elevación de la:

bilirrubina directa

alanina aminotransferasa (ALT)

bilirrubina indirecta

hemoglobina

● Un paso importante para la poder absorber los lípidos es la emulsificación de las


grasas. ¿Cuál de las siguientes alternativas es el principal factor encargado de dicha
emulsificación?

Fosfolípidos

Colesterol

Ácidos biliares

Bilirrubina

● En un paciente cirrótico con encefalopatía hepática ¿cuál de las siguientes alternativas


sustenta la reducción de carnes rojas en la dieta?

La fructosa y la galactosa se convierten en glucosa en el hígado

El factor de crecimiento hepatocitario fomenta la regeneración del hepatocito

El amonio se produce principalmente en el intestino

Hasta el 80% del colesterol se transforma en sales biliares

● En un paciente con cirrosis hepática la cabeza de medusa que aparece en la pared


abdominal, podría desaparecer si al paciente se le:

esclerosa las venas hemorroides internas

oblitera el ligamento redondo

administra antiandrógenos

oblitera la arteria gástrica izquierda

● Paciente con Lupus Eritematosos que desarrolla hipertensión portal debido a


trombosis portal, es probable que desarrolle várices a nivel de:

recto superior

hemorroides externas

recto inferior
canal anal

CI 4: Sifu

La presión parcial de oxígeno en la Zona 1 del sinusoide hepático de debe ser


___________ mmHg

● entre 95 y 45
● 40
● 100
● menor de 40

La bilis que sale de la vesícula biliar tiene como componente principal:

● a los ácidos biliares


● al colesterol
● a los fosfolípidos
● a la bilirrubina

En un paciente con carcinoma de páncreas, el tumor ha invadido la unión entre la


venas esplénica y mesentérica superior; eso quiere decir que estamos seguros que
el tumor se encuentra a nivel del ________ del páncreas.

● cola
● cabeza
● cuello
● cuerpo

En un paciente con una estenosis severa del colédoco por una complicación
quirúrgica, usted esperaría que desarrolle:

Respuestas:

● el aumento de transaminasas mayor que el de fosfatasa alcalina


● el aumento del tiempo de protrombina
● la cabeza de Medusa
● hiperbilirrubinemia no conjugada

Para aumentar la cantidad de bicarbonato en el flujo pancreático es necesario:

estimular las células S

ingerir aminoácidos como triptófano o fenilalanina

inhibir al nervio vago


estimular la secreción de péptido liberador de colecistoquinina (CCK)

Con respecto a la microestructura del hígado ¿Cuál de las siguientes alternativas es


correcta?

La célula de Kupffer se encuentra fuera del sinusoide y fagocita células

Los hepatocitos están interconectados por uniones hermética

El sinusoide es un capilar fenestrado

La célula de Ito se encuentra en el espacio de Disse y reserva glucógeno

En el hígado, el aumento de la resistencia vascular en los sinusoides hepáticos


ocasionará:

aumento del flujo hacia la vena porta

aumento de la presión de llenado vesicular

aumento del flujo hacia la vena cava superior

salida de plasma hacia el intersticio

En el hígado, el aumento de la resistencia vascular en los sinusoides hepáticos


ocasionará:

● salida de plasma hacia el intersticio


● aumento del flujo hacia la vena porta
● aumento de la presión de llenado vesicular
● aumento del flujo hacia la vena cava superior

En un recién nacido menor de 24 horas con atresia biliar, se encuentra elevación de


la:

● bilirrubina directa
● hemoglobina
● bilirrubina indirecta
● alanina aminotransferasa (ALT)

En un paciente con intoxicación por órganos fosforados, la acción de la


colecistoquinina (CCK) está bloqueada a nivel de:
● el esfínter de Oddi
● la célula parietal
● la vesícula biliar
● el sistema nervioso central

¿Cuál de las siguientes alternativas es una comunicación entre el tejido hepático y la


vesícula biliar?

Válvula espiral de Heister

Conducto de Lushka

Senos de Rokitansky-Aschoff

Divertículos vesiculares

CI4 - Francisco

En un paciente con cirrosis hepática avanzada se encuentra tendencia a:

● la disminución de bilirrubina hidrosoluble


● la hiperglucemia
● la hipoglucemia
● el acortamiento del tiempo de protrombina

En un paciente con infección por SARS-CoV-2 con compromiso severo e ingresado


en la unidad de cuidados intensivos (UCI), debido a la tormenta de citoquinas y a la
desregulación inmune ¿Cuál de las siguientes alternativas estará elevada en sangre
al evaluar el perfil hepático?

● transaminasas
● amilasas
● bilirrubinas
● fosfatasas

Para poder absorber al torrente sanguíneo todos los carbohidratos del azúcar común
(de mesa) es necesario usar únicamente los transportadores:

● GLUT5, SGLT1
● GLUT2, GLUT5
● SGLT1, GLUT2
● GLUT2, SGLT1, GLUT5

En la regulación de la glucosa sérica participa el hígado ¿Cuál de las siguientes


alternativas evalúa mejor esta función hepática?
● Péptido C en sangre
● Glucosa sérica en ayunas
● Glucosa en orina
● Tolerancia oral a la glucosa

En un paciente cirrótico con encefalopatía hepática ¿cuál de las siguientes


alternativas sustenta la reducción de carnes rojas en la dieta?

● La fructosa y la galactosa se convierten en glucosa en el hígado


● Hasta el 80% del colesterol se transforma en sales biliares
● El factor de crecimiento hepatocitario fomenta la regeneración del hepatocito
● El amonio se produce principalmente en el intestino

¿Cuál de las siguientes alternativas es correcta sobre la estructura hepática?

● El flujo biliar en el lobulillo hepático es centrífugo


● Los colangiocitos producen bilis
● El flujo sinusoidal en el lobulillo hepático es de adentro hacia afuera
● En la triada portal, se encuentra la vena derivada de la suprahepática

Las siguientes sustancias son secretadas por el páncreas, EXCEPTO:

● Lipasa
● Péptido Monitor
● Nucleasas
● Tripsina

Un paso importante para la poder absorber los lípidos es la emulsificación de las


grasas. ¿Cuál de las siguientes alternativas es el principal factor encargado de dicha
emulsificación?

Colesterol

Fosfolípidos

Ácidos biliares

Bilirrubina

CI4 - HEYDY
1. Un paso importante para la poder absorber los lípidos es la emulsificación de las
grasas. ¿Cuál de las siguientes alternativas es el principal factor encargado de dicha
emulsificación?
a. Ácidos biliares
b. bilirrubina
c. colesterol
d. fosfolípidos
e. Ácidos biliares
2. En un paciente con cirrosis hepática la cabeza de medusa que aparece en la pared
abdominal, podría desaparecer si al paciente se le:
a. oblitera la arteria gástrica izquierda
b. administra antiandrógenos
c. oblitera la arteria gástrica izquierda
d. oblitera el ligamento redondo
e. esclerosa las venas hemorroides internas
3. La bilis que sale de la vesícula biliar tiene como componente principal:
a. a los ácidos biliares
b. a la bilirrubina
c. al colesterol
d. a los fosfolípidos
4. Los hepatocitos tienen una gran capacidad regenerativa, en parte gracias a las células
madre hepáticas que se localizan en los:
a. espacios de Disse
b. canales de Herring
c. sinusoides hepáticos
d. espacios de mall
5. Las siguientes sustancias son secretadas por el páncreas, EXCEPTO:
a. tripsina
b. péptido monitor
c. lipasa
d. nucleasa
6. Marque lo correcto respecto al acino hepático:
a. La zona 1 es la más que recibe más sangre
b. El eje menor lo constituye un eje imaginario entre dos venas centrolobulillares
c. Dos vértices del rombo acinar se constituyen por triadas portales
d. La zona 1 es la más que recibe más sangre
e. La zona 3 se afecta en menos en una deshidratación severa
7. La vía biliar extrahepática se encuentra ubicada dentro del ligamento:
a. Hepatoduodenal
b. Falciforme
c. Hepatogástrico
d. Coronario anterior
8. En un paciente con cáncer de páncreas y que desarrolla ictericia, la localización más
probable del tumor es en:
a. el cuello del páncreas
b. la cola del páncreas
c. la cabeza del páncreas
d. la vesícula por metástasis
9. El efecto de un medicamento colerético se evidencia por:
a. el aumento de secreción biliar
b. la mayor producción de colesterol en la bilis
c. el aumento de formación de micelas
d. la disminución de absorción de sales biliares
10. La presión parcial de oxígeno en la Zona 1 del sinusoide hepático de debe ser
___________ mmHg
a. entre 95 y 45
b. 40
c. 100
d. menor de 40

En un paciente con una estenosis severa del colédoco por una complicación quirúrgica, usted
esperaría que desarrolle --- aumento de tiempo de protrombina

La vía biliar extrahepática se encuentra ubicada dentro del ligamento --- hepatoduodenal

Un recién nacido con enfermedad congénita, tiene una mutación de la proteína MRP2 encargada del
transporte de la bilirrubina conjugada hacia el interior del canalículo. Antes de las 24 horas de nacido
presenta ictericia y además presentará con mayor probabilidad ----- coluria

La bilis que sale de la vesícula biliar tiene como componente principal ---- a los ácidos biliares

Para aumentar la cantidad de bicarbonato en el flujo pancreático es necesario ----- estimular las
células s

Los hepatocitos tienen una gran capacidad regenerativa, en parte gracias a las células madre
hepáticas que se localizan en los ---- canales de Herring

En el síndrome de Mirizzi, el paciente tiene cálculos en la vesícula biliar; pero se obstruye el conducto
hepático común debido a que un cálculo se ubica y crece de tamaño en ----- La bolsa de Hartmann

¿Cuál de las siguientes alternativas es correcta sobre la estructura hepática? ---- El flujo biliar en el
lobulillo hepático es centrífugo

La relajación del esfínter de Oddi se produce directamente por acción de ----- VIP

En la anemia perniciosa, con atrofia gástrica marcada, usted esperaría encontrar disminución en la
absorción de ___________ a nivel de ________ ----- vitamina B12 / íleon distal

Dentro de las patologías que producen sangrado en el esófago, la que sangra más es ----- el
síndrome de Boerhaave

En la glándula fúndica, se evidencia células madre o stem cells a nivel de ----- el istmo

Al usar atropina en un paciente, usted esperaría --- la disminución de secreción gástrica por bloqueo
de M3

La comunicación entre la irrigación gástrica y la esofágica depende de una rama de la arteria ----
Tronco celíaco

En fases iniciales, la infección por Helicobacter pylori genera disminución del pH gástrico debido a ----
- la disminución de somatostatina

En un paciente que usa AINEs a altas dosis por artritis reumatoide, usted esperaría que presente
erosiones y úlceras gástricas debido a ----- la disminución de irrigación en la mucosa

A raíz de la infección por Helicobacter pilory, la destrucción de las células ____________ ocasiona
que aumente la producción de ácido en el estómago y por ello se produzca úlceras ____________ ---
-- D/duodenales
1. El agua se absorbe en yeyuno, íleon y colon y se excreta en las
heces. Señale en orden descendente la cantidad de agua absorbida
o excretada en los siguientes elementos.
A) Colon, yeyuno, íleon, heces.
B) Heces, colon, íleon, yeyuno.
C) Yeyuno, íleon, colon, heces.
D) Colon, íleon, yeyuno, heces.
E) Heces, yeyuno, íleon, colon.

2. Después de un desastre natural en Haití, surgió un brote de cólera


en personas desplazadas que vivían en tiendas de campamentos.
Los sujetos afectados presentaron graves síntomas diarreicos:
¿por cuál de los siguientes cambios del transporte intestinal
aparecieron?
A) Aumentan el cotransporte de sodio-potasio en el intestino
delgado.
B) Incrementan la secreción de potasio en el colon.
C) Inhiben la absorción de potasio en las criptas de Lieberkühn.
D) Aumentan la absorción de sodio en el intestino delgado.
E) Incrementan la secreción de cloruro hacia la luz intestinal.

3. Un varón de 50 años acudió a su médico y le señaló que tenía


dolor epigástrico intenso, pirosis frecuentes y pérdida identifi cada
de 9.5 kg en un lapso de 6 meses. Según él, no tuvo alivio alguno
con productos que se adquirían sin receta como antihistamínicos
contra el receptor H2. Fue referido a un gastroenterólogo y en la
endoscopia gastroduodenal se identifi caron erosiones y úlceras en
la porción proximal del duodeno y una mayor producción de
ácido gástrico con el sujeto en ayunas. El paciente muy
probablemente tiene un tumor que secreta: ¿cuáles de las
hormonas siguientes?
A) Secretina.
B) Somatostatina.
C) Motilina.
D) Gastrina.
E) Colecistocinina.

4. ¿Cuál de los siguientes tiene el pH más alto?


A) Jugo gástrico.
B) Contenido de la luz colónica.
C) Jugo pancreático.
D) Saliva.
E) Contenido de las glándulas intestinales.

5. Una mujer de 60 años es sometida a pancreatectomía total, por la


presencia de una neoplasia. De los resultados siguiente: ¿cuál no se
esperaría después de que se recuperara de la operación?
A) Esteatorrea.
B) Hiperglucemia.
C) Acidosis metabólica.
D) Incremento de peso.
E) Menor absorción de aminoácidos.

1 La absorción máxima de ácidos grasos de cadena corta producidos


por las bacterias ocurre en:
A) estómago.
B) duodeno.
C) yeyuno.
D) íleon.
E) colon.

2. Una mujer premenopáusica físicamente activa solicitó orientación


de su médico familiar en cuanto a las medidas a tomar para
asegurar la disponibilidad adecuada de calcio de alimentos, para
afianzar su salud ósea en etapas ulteriores de la vida. De los
siguientes componentes de la alimentación: ¿cuál debe
incrementar la captación de calcio?
A) Proteínas.
B) Oxalatos.
C) Hierro.
D) Vitamina D.
E) Sodio.

3. Un niño que manifiesta ausencia congénita de enterocinasa


presentaría una disminución habitual en:
A) frecuencia de pancreatitis.
B) absorción de glucosa.
C) reabsorción de ácidos biliares.
D) pH gástrico.
E) asimilación de proteínas.

4. En la enfermedad de Hartnup (un defecto del transporte de


aminoácidos neutrales), los pacientes no presentan deficiencia de
estos aminoácidos gracias a la actividad de
A) PepT1.
B) peptidasas del borde en cepillo.
C) Na, K-ATPasa.
D) regulador de la conductancia transmembrana de la fibrosis
quística (CFTR).
E) tripsina.

5. Un recién nacido es llevado al pediatra a causa de diarrea grave


que se agrava al alimentarse. Los síntomas disminuyen cuando los
nutrientes se administran por vía intravenosa. Lo más probable es
que el niño tenga una mutación en cuál de los siguientes
transportadores intestinales:
A) Na, K-ATPasa.
B) NHE3.
C) SGLT1.
D) H+, K+-ATPasa.
E) NKCC1.

1. En los lactantes, la defecación suele ocurrir después de una comida.


La causa de las contracciones colónicas en esta situación es
A) la histamina.
B) el aumento de las concentraciones de CCK en la circulación
sanguínea.
C) el reflejo gastrocólico.
D) el aumento de las concentraciones de somatostatina en la
circulación sanguínea.
E) el reflejo enterogástrico.

2. Los síntomas del síndrome de vaciamiento rápido (molestia después


de las comidas en los pacientes con derivaciones intestinales como
la anastomosis gastroyeyunal) son causados en parte por:
A) aumento de la presión arterial.
B) aumento de la secreción de glucagon.
C) aumento de la secreción de CCK.
D) hipoglucemia.
E) hiperglucemia.

3. Las presiones gástricas raras veces aumentan por arriba de los


niveles que abren el esfínter esofágico inferior, aun cuando el estómago esté lleno de comida. ¿A
cuál de los siguientes procesos
se debe este fenómeno?
A) Peristaltismo.
B) Reflejo gastroileal.
C) Segmentación.
D) Estimulación del centro del vómito.
E) Relajación receptiva.

4. ¿Cuál de las siguientes sustancias detona el complejo motor


migratorio?
A) Motilina.
B) NO.
C) CCK.
D) Somatostatina.
E) Secretina.
5. Un paciente es referido al gastroenterólogo porque tiene dificultad
persistente para la deglución. En el estudio endoscópico se
advierte que no se abre totalmente el esfínter esofágico inferior
conforme el bolo llega a él, por lo que se hace el diagnóstico de
acalasia.
En la exploración o en las biopsias estudiadas de muestras de la
región esfinteriana: ¿en cuál de los elementos siguientes cabría
esperar disminución?
A) Peristaltismo esofágico.
B) Expresión de óxido nítrico sintasa neuronal.
C) Receptores de acetilcolina.
D) Liberación de sustancia P.
E) Contracción del diafragma crural.

1. Un paciente que sufre colitis ulcerosa grave es sometido a


colectomía total con elaboración de un estoma. Después de la
recuperación completa de la operación y al comparar su estado
posoperatorio con el que tenía antes de la operación: ¿cuál de los
siguientes factores cabría esperar que disminuya?
A) Capacidad de absorber lípidos.
B) Capacidad de coagular la sangre.
C) Concentraciones circulantes de ácidos biliares conjugados.
D) Urea en orina.
E) Urobilinógeno en orina.
2. Una cirujana estudia nuevos métodos de trasplante de hígado y
realiza una hepatectomía completa en un animal de
experimentación. Antes de injertar el hígado donado cabría
esperar incremento en la concentración sanguínea de:
A) glucosa.
B) fibrinógeno.
C) 25-hidroxicolecalciferol.
D) bilirrubina conjugada.
E) estrógenos.

3. ¿Cuál de los siguientes tipos de células protege contra la septicemia


consecutiva a la translocación de bacterias intestinales?
A) Célula estrellada hepática.
B) Colangiocito.
C) Célula de Kupffer.
D) Hepatocito.
E) Célula epitelial de la vesícula biliar.

4. El citocromo P450 (CYP) se expresa de manera extraordinaria en


los hepatocitos. ¿En cuál de las siguientes no desempeña una
función importante?
A) Formación de ácidos biliares.
B) Carcinogénesis.
C) Formación de hormonas esteroideas.
D) Desintoxicación de fármacos.
E) Síntesis de glucógeno.

5. Una mujer de 40 años acude con su médico familiar y le señala que


ha tenido dolor abdominal episódico e intenso, en particular
fuerte después de ingerir una comida grasosa. Un estudio
imagenológico señala que su vesícula biliar muestra dilatación
aguda y se hace el diagnóstico de colelitiasis. ¿En qué sitio
anatómico un cálculo vesicular agravaría el riesgo de
pancreatitis?
A) Conducto hepático izquierdo.
B) Conducto hepático derecho.
C) Conducto cístico.
D) Colédoco.
E) Esfínter de Oddi.

6. La bilis de la vesícula biliar, en comparación con la que está en el


hígado, contiene una menor concentración de:
A) Ácidos biliares.
B) Iones de sodio.
C) Protones.
D) Glucosa.
E) Protones.

1. Un niño de 2 años es llevado a la consulta por diarrea persistente y edema de las


extremidades, además falta de crecimiento y desarrollo en relación a su edad. Los análisis de
sangre revelan que tiene concentración plasmática baja de proteínas (hipoproteinemia).
Durante la endoscopía duodenal, se coloca colecistokinina (CCK) endovenosa y se recoge
muestras del líquido duodenal; el resultado del líquido confirma incapacidad para hidrolizar
proteínas a un pH neutro, esta situación mejora al añadir una pequeña cantidad de tripsina.
El paciente probablemente esté sufriendo la falta congénita de

d. Enterocinasa

2. Experimentalmente se incrementa la velocidad de la secreción salival con una sustancia, el


análisis de la composición de esta saliva obtenida se espera encontrar
c. Disminución de concentración de potasio

3. Paciente varón de 46 años soltero, consulta por odinofagia y bajo de peso, tiene antecedente
de tuberculosis desde hace 3 meses y es fumador crónico (10 cigarrillos por día); al evaluar
la cavidad oral se identifica lesión blanquecina en el dorso de la lengua y paladar blando, las
lesiones se desprenden con el baja lengua dejando una base eritematosa. Esta lesión
corresponde probablemente a
b. Candidiasis oral

4. Minero de 32 años de edad, que acude a centro de salud por presentar de forma progresiva
desde hace 1 año dificultad para ingerir alimentos sólidos y luego líquidos; refiere
regurgitaciones alimentarias y marcada pérdida de peso (15 kilos). Radiografia baritada de
esófago como se muestra en la figura. El presente caso se explica por
Relajación incompleta del esfínter esofágico inferior

5. Paciente mujer de 35 años acude a consulta por sensación de sequedad y lesiones en


cavidad oral. Al examen se observa atrofia de la mucosa, fisuras y úlceras; nota además
sequedad e irritación de la córnea y aumento del tamaño de las glándulas parotídeas. Su
diagnóstico más probable es artritis reumatoide; el hallazgo más probable en una biopsia de
glándula parótida es
b. Gran infiltración de linfocitos y células plasmáticas
6. Un paciente con anemia acude con su médico quejándose de episodios frecuentes de
gastroenteritis. Un análisis de sangre revela anticuerpos circulantes dirigidos contra células
parietales gástricas. Su anemia es atribuible a la hiposecreción de
Factor intrínseco

7. Un niño de cuatro años de edad es llevado a la consulta por cuadros diarreicos frecuentes
caracterizados por heces pálidas, voluminosas y fétidas, presenta bajo peso y talla. Se mide
la concentración de cloruro en el sudor y se encuentra que sus valores son muy elevados. La
alteración más importante a nivel de células ductales del páncreas tiene relación directa con
la conductancia de
d. Cloro
8. Una mujer de 50 años de edad que sufrió durante varios años resequedad de los ojos debida
a producción inadecuada de lágrimas es enviada con un gastroenterólogo para evaluación de
pirosis crónica. El examen endoscópico revela erosiones y tejido cicatrizal en la parte distal
del esófago justo por arriba del esfínter esofágico inferior. Las lesiones pueden atribuirse a la
disminución de uno de los siguientes componentes salivales:
a. Bicarbonato
9. En el reflejo peristáltico del intestino delgado, uno de los siguientes eventos sucede en la
porción oral del bolo alimenticio
d. Acción de acetilcolina en el músculo circular
10. Experimentalmente se coloca una dosis alta de secretina en la luz intestinal duodenal; como
consecuencia de esto, en el jugo pancreático de la misma luz intestinal se observa la
disminución de la concentración de
b. Cl-
11. Un varón de 58 años de edad con enfermedad de Crohn severo fue sometido a una
resección ileal. Después de la cirugía este paciente padecerá de esteatorrea, esto se explica
porque
La micelas no pueden formarse
12. La toxina del Vibrio cholerae causa diarrea debido a
b. El Incremento de la secreción de cloro por las células de la cripta intestinal
13. ¿Cuál de las siguientes alternativas es una característica de la secreción exocrina del
páncreas?
a. Tiene una baja concentración de Cl- respecto al plasma
14. Una madre lleva a su hijo de dos años de edad a la sala de urgencias, estresada porque el
niño deglutió una moneda de 10 céntimos mientras la familia cenaba en un restaurante. El
médico observa mediante fluoroscopía que la moneda se halla en el estómago y asegura a la
madre que la moneda se eliminará con las heces. El médico recomienda utilizar la respuesta
fisiológica que permitirá la evacuación de la moneda del estómago al intestino
b. Son los movimientos de mezcla y trituración
c. Es provocada por el ayuno

15. La composición de la bilis es modificada conforme fluye por los conductillos biliares. Durante
este tránsito se espera que aumente la concentración de
a. Ig A
c. Monómeros de ácido biliar
16. Se mide experimentalmente el contenido gástrico de dos personas. La persona “A” tiene alto
contenido de grasa y la persona “B” tiene un contenido hipertónico ¿Cuál de las siguientes es
correcto respecto al vaciamiento gástrico?
Hay ralentización del vaciado gástrico en ambos casos
17. Un paciente varón de 18 años de edad acude al médico para sus exámenes de rutina. Sus
resultados de laboratorio muestran un valor de bilirrubina sérica de 4 mg/dl y una bilirrubina
directa de 0,3 mg/dl. Las pruebas de función hepática son normales. La alteración que
explica mejor este caso es por la deficiencia de
b. Glucuronil transferasa
18. Un hombre de 57 años de edad es llevado a urgencias con hematemesis masiva rojo
brillante, a su llegada se halla inconciente con PA: 80/40 mm Hg y FC: 124 lat/min. Luce
ictérico con presencia de “arañas vasculares en el tórax anterior y extremidades”, abdomen
distendido con signo de oleada positiva. Se encuentra esplenomegalia y pérdida de la masa
muscular en extremidades. La anastomosis vascular responsable del sangrado en este
paciente es
b. Vena gástrica izquierda y vena ácigos
19. Un estudiante de medicina está comiendo un plato de comida a base de champiñones,
espárrago y salsa de soya. El sabor umami contenido en todos estos alimentos actúa a nivel
de los botones gustativos estimulando
Un receptor acoplado a proteína G
20. Un hombre de 22 años de edad se presenta al médico con una historia de 1 año de
evolución caracterizado por dolor recurrente en fosa iliaca derecha y diarrea. Manifiesta
además pérdida de peso de 8 kg durante este periodo. La colonoscopía revela múltiples
lesiones en el ileon terminal y colon. La biopsia de estas lesiones revela engrosamiento,
inflamación y ulceración de la mucosa. El diagnóstico más probable en este caso es
b. Enfermedad de Crohn
21. Una de las funciones del músculo señalado es
d. Deprime la mandíbula cuando el hioides está fijo

22. Varón de 61 años que consulta por dolor retro esternal intenso desde hace 6 horas y
después de vómitos intensos y repetidos; al examen se observa disnea, cianosis, hipotensión
y signos clínicos de shock. La radiografía simple de tórax muestra neumomediastino. El
líquido en el espacio pleural aspirado tiene alta concentración de amilasa. ¿Cuál de las
siguientes alternativas puede explicar este cuadro clínico?
b. Rotura espontánea de esófago
23. La secreción del ácido en la célula parietal gástrica se lleva a cabo por una ATPasa
especifica que intercambia hidrogeniones (H+) del citosol por
d. K+
24. En condiciones normales el ingreso de 600 ml de líquido es el estómago provoca un
aumento de presión intragástrica de unos 12 cm de H2O. Después de una vagotomía (corte
del nervio vago) es de esperar que el ingreso del mismo volumen de líquido provoque lo
siguiente:
Un aumento mayor de la presión
25. Una paciente de 30 años de edad es sometida a una cirugía en oído medio derecho por un
problema de otoesclerosis. Luego de la cirugía refiere alteración en la percepción de sabores.
Al evaluar el caso usted esperaría encontrar
b. Alteración en la sensación del gusto en los dos tercios anteriores de la lengua
d. Sensación del dolor, tacto y temperatura conservada en toda la lengua

26. ¿Cuál de las siguientes alterativas es correcta?


c. Las sales biliares desconjugadas son absorbidas preferentemente en el colon
27. En un paciente de 45 años de edad con colestasis biliar, se encuentra una elevación de los
niveles sanguíneos de fosfatasa alcalina hasta 3 veces la cifra normal. ¿Cuál de las
siguientes alternativas estará también elevada como evidencia del daño de la vía biliar?
Gamma glutamil transpeptidasa
28. Revisando la angiografía de un hombre de 70 años en estudio por aneurisma de aorta
abdominal el radiólogo informa de la presencia de una oclusión completa de la arteria
mesentérica inferior. El paciente se encuentra completamente asintomático. ¿Cuál de las
siguientes arterias se anastomosa a la sistema arterial de la mesentérica inferior?
Cólica media
29. Un varón de 75 años ingresa al consultorio por presentar ictericia marcada de piel y las
escleras. El estudio del paciente mostró que presentaba un tumor que obstruía la totalidad
del conducto hepático común. ¿Cuál de las siguientes estructuras se encontrará dilatada en
este paciente?
Conductos de Hering
30. Respecto a la siguiente imagen que representa una estructura de la mucosa gástrica, la
estructura con número ……….. produce

2 / pepsinógeno
PREGUNTAS PA TIPEAR

Secreta grandes cantidades de bicarbonato y agua para neutralizar la acidez del quimo:

● Páncreas

La sacarosa se desdobla en:

● Glucosa y fructosa

Enfermedad en que no hay relajación del cardias como respuesta de deglución y hay falta de
peristaltismo en el esófago:

● Acalasia

Histológicamente la mucosa del tubo digestivo presenta las siguientes regiones excepto

● Muscular
● Epitelio
● Corion
● Lámina propia
● Muscularis mucosae

Es la porción principal y de mayor tamaño del estómago, en la cual se forma el quimo:

● Cuerpo

Entre las capas musculares se encuentran grupos de células ganglionares y haces de fibras
nerviosas amielínicas, que en conjunto representan:

● Plexo mientérico de auerbach

A nivel del íleon, se encuentran agregaciones permanentes de nódulos linfoides llamados:

● Placas de Peyer

En relación al intestino grueso, la capa mucosa está constituida por tejido conectivo laxo y mesotelio,
presenta depósitos de grasa llamados:

● Apéndices epiploicos

En que porción del tubo digestivo se sintetiza vitamina K y B

● Colon

Contienen enzimas, excepto:

● Saliva
● Bilis
● Jugo Pancreático
● Jugo intestinal
● Jugo gástrico

Producen el HCl en el estómago

● Células parietales
No es parte del duodeno:

● Descendente
● Ileal
● Ascendente
● Horizontal
● Superior

Enzima proteolítica contenida en el jugo pancreático

● Tripsina

Conducto de drenaje de la glándula parótida en la boca

● Stenon

La unidad anatomofisiológica del hígado es

● Estroma
● Hepatocito
● Lobulillo hepático
● Hilio del hígado
● Parénquima

Los ameloblastos se localizan en:

● Esmalte

Contiene glucoproteínas que permite la lubricación de la boca:

● Secreción mucosa de la saliva

El bolo alimenticio pasa hacia la faringe a través de:

● Istmo de las fauces

El conducto colédoco se forma por la convergencia de los conductos:

A. Cístico - Wirsung
B. Santorini - hepático
C. Cístico - Santorini
D. Cístico - hepático
E. Cardiaco - hepático

Las glándulas parótidas y submaxilares respectivamente presentan como conductos:

A. Rivinus y Stenon
B. Wharton y Stenon
C. Stenon y Rivinus
D. Stenon y Wharton
E. Wharton y Rivinus

Membrana epitelial que sostiene a casi todas las vísceras del abdomen

A. Mesenterio
B. Fundus
C. Peritoneo
D. Epitelio
E. Mucosa

En el mecanismo de formación de HCl intervienen los siguientes elementos químicos:

A. H2CO3
B. CO2
C. H2O
D. H+
E. T.A.

La gingivorragia se produce por deficiencia de vitamina:

A. B
B. D
C. E
D. C
E. A

Fase de secreción del HCl que se activa por saborear

A. Vagal
B. Intestinal
C. Gástrica
D. Neurógena
E. AyD

Producen colecistoquinina:

A. Células Paneth
B. Células argentafines
C. Células APUD
D. Brunner
E. N.A.

El tripsinógeno es activado por:

A. Enterocinasa
B. Quimiotripsina
C. Tripsina
D. Elastasa
E. A+B

Con respecto a la microestructura del hígado ¿Cuál de las siguientes alternativas es correcta?
- Los hepatocitos están interconectados por uniones herméticas
- La célula de Ito se encuentra en el espacio de Disse y reserva glucógeno
- El sinusoide es un capilar fenestrado
- La célula de Kupffer se encuentra fuera del sinusoide y fagocita células

En un paciente con una estenosis severa del colédoco por una complicación quirúrgica, usted esperaría
que desarrolle:

- la cabeza de Medusa
- el aumento de transaminasas mayor que el de fosfatasa alcalina
- hiperbilirrubinemia no conjugada
- el aumento del tiempo de protrombina

Marque lo correcto respecto al acino hepático:

- Dos vértices del rombo acinar se constituyen por triadas portales


- El eje menor lo constituye un eje imaginario entre dos venas centrolobulillares
- La zona 1 es la más que recibe más sangre
- La zona 3 se afecta en menos en una deshidratación severa

Los hepatocitos tienen una gran capacidad regenerativa, en parte gracias a las células madre hepáticas
que se localizan en los:

- espacios de Disse
- sinusoides hepáticos
- canales de Herring
- espacios de Mall

En un paciente con infección por SARS-CoV-2 con compromiso severo e ingresado en la unidad de
cuidados intensivos (UCI), debido a la tormenta de citoquinas y a la desregulación inmune ¿Cuál de
las siguientes alternativas estará elevada en sangre al evaluar el perfil hepático?

- transaminasas
- fosfatasas
- amilasas
- bilirrubinas

La presión parcial de oxígeno en la Zona 1 del sinusoide hepático de debe ser ___________ mmHg

- menor de 40
- 40
- 100

- entre 95 y 45

En un paciente cirrótico con encefalopatía hepática ¿cuál de las siguientes alternativas sustenta la
reducción de carnes rojas en la dieta?

- El amonio se produce principalmente en el intestino


- Hasta el 80% del colesterol se transforma en sales biliares
- La fructosa y la galactosa se convierten en glucosa en el hígado
- El factor de crecimiento hepatocitario fomenta la regeneración del hepatocito

En el hígado, el aumento de la resistencia vascular en los sinusoides hepáticos ocasionará:


- aumento del flujo hacia la vena porta
- aumento de la presión de llenado vesicular
- aumento del flujo hacia la vena cava superior
- salida de plasma hacia el intersticio

En un paciente con cirrosis hepática se desarrollará ascitis debido a los siguientes mecanismos,
EXCEPTO:

- Hipoalbuminemia
- Estrechamiento de la porta
- Hiperflujo portal
- Aumento de reabsorción renal de sodio

El efecto de un medicamento colerético se evidencia por:

- el aumento de formación de micelas


- el aumento de secreción biliar
- la mayor producción de colesterol en la bilis
- la disminución de absorción de sales biliares

CI4 - PEQUE

Paciente con Lupus Eritematosos que desarrolla hipertensión portal debido a trombosis portal, es
probable que desarrolle várices a nivel de

- canal anal
- recto inferior
- recto superior
- hemorroides externas

¿Cuál de las siguientes alternativas es una comunicación entre el tejido hepático y la vesícula biliar?

- Válvula espiral de Heister


- Conducto de Lushka
- Divertículos vesiculares
- Senos de Rokitansky-Aschoff

En un paciente con cirrosis hepática terminal, qué hormona aumenta la absorción de agua en el colon

- Disminuida
- No influye
- Aumentada
- Normal

En un paciente con cirrosis hepática avanzada se encuentra tendencia a

- la hiperglicemia
- el acortamiento del tiempo de protrombina
- la disminución de bilirrubina hidrosoluble
- la hipoglicemia

Un paso importante para la poder absorber los lípidos es la emulsificación de las grasas. ¿Cuál de las
siguientes alternativas es el principal factor encargado de dicha emulsificación?

- Ácidos biliares
- Bilirrubina
- Colesterol
- Fosfolípidos

Para aumentar la cantidad de bicarbonato en el flujo pancreático es necesario

- inhibir al nervio vago


- estimular la secreción de péptido liberador de colecistoquinina (CCK
- ingerir aminoácidos como triptófano o fenilalanina
- estimular las células S

En un paciente con cirrosis hepática se desarrollará ascitis debido a los siguientes mecanismos,
EXCEPTO:

- Hipoalbuminemia
- Aumento de reabsorción renal de sodio
- Estrechamiento de la porta
- Hiperflujo portal

En el síndrome de Mirizzi, el paciente tiene cálculos en la vesícula biliar; pero se obstruye el conducto
hepático común debido a que un cálculo se ubica y crece de tamaño en:

- la bolsa de Hartmann
- el conducto accesorio de Lushka
- el colédoco
- la válvula espiral de Heister

En un paciente con intoxicación por órganos fosforados, la acción de la colecistoquinina (CCK) está
bloqueada a nivel de:

- el sistema nervioso central


- el esfínter de Oddi
- la vesícula biliar
- la célula parietal

Con respecto a la microestructura del hígado ¿Cuál de las siguientes alternativas es correcta?

- Los hepatocitos están interconectados por uniones herméticas


- El sinusoide es un capilar fenestrado
- La célula de Ito se encuentra en el espacio de Disse y reserva glucógeno
- La célula de Kupffer se encuentra fuera del sinusoide y fagocita células

CI 4 + lo que me mandaron a tiepar - Shiro


En un paciente con cirrosis hepática la cabeza de medusa que aparece en la pared
abdominal, podría desaparecer si al paciente se le:
- esclerosa las venas hemorroides internas
- oblitera el ligamento redondo
- administra antiandrógenos
- oblitera la arteria gástrica izquierda

Para poder absorber al torrente sanguíneo todos los carbohidratos de la lactosa, es


necesario usar únicamente los transportadores:
- SGLT1, GLUT2
- GLUT5, SGLT1
- GLUT2, SGLT1, GLUT5
- GLUT2, GLUT5

Interviene en la digestión intestinal, excepto:


- jugo pancreático
- bilis
- jugo intestinal
- jugo gástrico
- tripsina

De las siguientes enzimas digestivas;


I) Ptialina
II) Pepsina
III) Tripsina
IV) Enteroquinasa
V) Aminopolipeptidasa
son producidas por el estomago:

- I, II, III
- III, IV, V
- I, III, IV
- II, IV
- Solo II

La secretina, hormona secretada por la mucosa duodenal, desempeña varias funciones,


indique la que corresponde:
- Activa la secreción gástrica
- Provoca contracción de las vesícula biliar
- Inhibe la secreción gástrica
- Activa la secreción pancreática
- Inhibe la secreción pancreática

Los sinusoides hepáticos, contienen células que pertenecen al RES con actividad fagocítica,
a estas células se les conoce con el nombre de:
- Yuxta globemerulares
- Kupffer
- Remack
- Podocitos
- Falciformes

El tubo digestivo esta formado por excepto:


- Boca
- intestino grueso
- hígado
- estómago
- intestino delgado

El plexo de Auerbach se encuentra en la capa


- Musculares mucoseae
- Subucosa
- Mucosa
- Muscular
- Serosa

Cual no es función del hígado


- hemocateresis
- secreción de bilis
- hematopoyesis
- anticoagulante
- almacenamiento principalmente de vitaminas A, D y B12

El estómago presenta epitelio


- estratificado plano sin queratina
- cúbico simple
- cilindrico simple no modificado
- cilindrico simple con ribete en cepillo
- escamoso plano

La digestion de los almidones se inicia en _______, por la enzima _______


- estomago - amilasa gastrica
- intestino delgado - amilasa pancreatica
- esofago - alfa amilasa salival
- boca - alfa amilasa salival
- intestino delgado - colon

Respecto a glándulas salivales es cierto, excepto:


- Parótida presenta el conducto de stenon
- Submaxilares secretan el 70% del volumen desaliva
- Sublinguales posee el conducto de Rivinus
- Submaxilares tienen secreción seromucosa
- Parótidas presenta secreción mucosa

El padre de rosa es alcohólico desde hace 20 años y ahora cura el médico por presentar la
piel y escleras amarillentas (ictéricas) el médico le ha dicho que su hígado se encuentra
muy dañado, Entonces qué enfermedad podría presentar
- hepatitis B
- Apendicitis
- gastritis
- cirrosis hepática
- N.A

Una de las características de la bilirrubina, es que tiene un color:


amarillento
Las placas de Peyer, se encuentran en:
- duodeno
- yeyuno
- ileon
- colón
- recto

La función de la bilis es:


- emulsificar proteínas
- emulsificar las grasas
- emulsificar carbohidratos
- facilitar la digestión de los aminoácidos
- degradar las proteínas

No es Constituyente del jugo pancreático


- tripsinógeno
- enterocinasa
- amilasa
- carboxipeptidasa
- bicarbonato

Órgano del aparato digestivo que realiza la absorción de aproximadamente el 90% de


sustancias:
- Esofago
- estómago

ALEJO

CI 1

1. La motilidad intestinal es estimulada principalmente por el:

- Sistema simpático

- Sistema piramidal

- Plexo de Auerbach

- Sistema parasimpático

2. La peristalsis o peristaltismo hace referencia a:

- Motilidad para movilizar el alimento de proximal a distal.

- No es parte de la motilidad

- Motilidad para mezclado de alimentos.

- Motilidad para fraccionamiento de alimentos.


3. Marque el órgano que se encuentra más distal en el tubo digestivo.

- Estomago

- Ciego

- Íleon

- Duodeno

4. Marque la respuesta incorrecta:

- La mucosa consta de epitelio, lámina propia y muscularis mucosae.

- En todo el tubo digestivo, se observa dos capas de muscular propia:


circular interna y longitudinal externa

- El colon contiene tenias

- Fuera de la cavidad abdominal, el esófago presenta capa adventicia.

5. Al iniciar la digestión, aumenta el consumo de oxígeno por la mucosa. Esto conlleva


a una hipoxia local, lo cual hace que se libere _____________, el cual produce
vasodilatación:

- Colecistoquinina

- Adenosina

- Histamina

- Noradrenalina

6. Cuál de las siguientes estructuras no tiene vasos sanguíneos:

- Epitelio intestinal

- Ligamento

- Omento

- Mesenterio

7. Paciente tiene una úlcera sangrante en el segundo tercio del Yeyuno. La arteria de
la cual proviene la sangre arterial para dicha zona es la arteria:

- Mesentérica superior

- Tronco celíaco

- Mesentérica inferior

- Iliaca común
- Gástrica izquierda

8. Paciente de 24 años con dolor abdominal tipo cólico intenso en mesogastrio. Según
sus conocimientos de macroestructura, el origen del dolor puede ser el ___________:

- Íleon

- Colon

- Estómago

- Esófago

9. Dentro de las funciones del abdomen, se encuentra la defecación y micción, en las


cuales la presión intra abdominal debe:

- Aumentar

- No tiene relación el abdomen con dichas funciones

- Mantenerse igual

- Disminuir

10. En la inspiración, la pared abdominal debe ____________ para ____________:

- Contraerse aumentar presión intra abdominal

- Relajarse disminuir presión intra torácica

- Relajarse aumentar presión intra abdominal

- Contraerse aumentar presión intra torácica

CI 2

1. Respecto a los péptidos gastrointestinales, marque lo correcto.

- No existe sustancia neurocrina que tenga efecto en la motilidad del tubo


digestivo

- Las sustancias paracrinas pueden viajar a través de vasos sanguíneos

- Las sustancias neurocrinas son peptidos que hacen su efecto en


distancias cortas

- Las sustancias paracrinas atraviesan la circulación portal

2. Al disminuir el pH duodenal por el HCl gástrico, se libera principalmente una


hormona cuya célula diana es:
- Acinos pancreáticos

- Células ductales del colédoco

- Célula ductal del Wirsung

- Células S del intestino

3. En un paciente con gastroparesia (motilidad lenta del estómago), que presenta


distensión abdominal después de comer, usted le recomendaría que evite el consumo
de lípidos y aminoácidos para disminuir la acción de:

- Secretina

- CCK

- Somatostatina

- Gastrina

4. La razón por la que el potencial de acción viaja rápidamente en sentido longitudinal


por el musculo liso gastrointestinal es la presencia de uniones en hendidura, además de
la presencia de:

- La presencia del plexo submucoso de Meissner

- Varicosidades

- Mayor cantidad de ACh

- Las fibras musculares no se disponen en haces musculares

5. Paciente obeso con Covid-19 es intubado por interno inexperto, quien al solicitar
que bombeen aire dentro del tubo endotraqueal, nota que el epigastrio se distiende. Al
sospechar que ha introducido el tubo en el estómago, también es cierto que:

- Disminuiría el tono del píloro

- Aumenta la frecuencia de ondas lentas

- Aumenta el pH gástrico

- Disminuye el pH gástrico

6. En un paciente con diarrea por hipermotilidad, usted sospecharía en el posible


aumento de las siguientes sustancias, excepto:

- Sustancia P

- ACh

- Péptido intestinal vasoactivo (VIP)


- Motilina

7. El ecografista sabe que para poder visualizar el nacimiento de la arteria


mesentérica superior, debe colocar el transductor sobre la piel de la siguiente región
abdominal:

- Epigastrio

- Hipocondrio derecho

- Hipogastrio

- Mesogastrio

8. Al consumir un pan con mantequilla, la sensación de hambre disminuye debido a la


acción de:

- Grelina

- Somatostatina

- Colecistoquinina (CCK)

- Leptina

9. Una de las siguientes sustancias no comparte con las otras la misma acción sobre
la producción de ácido gástrico:

- Péptido insulinotrópico dependiente de glucosa (GIP)

- Colecistoquinina

- Somatostatina

- Secretina

10. Al ingerir grandes cantidades de dulces, con la subsecuente estimulación de


incretinas, usted esperaría que el apetito ______________, debido a
__________________

- disminuya insulina

- aumente CCK

- disminuya CCK

- aumente grelina

PARCIAL

1. Al rozar agua caliente en la punta de la lengua, usted esperaría que el estímulo


viaje a través del nervio:
- lingual

- cuerda del tímpano

- glosofaríngeo

- hipogloso

2. Al ingresar líquidos o sólidos en la cavidad oral, un mecanismo que permite que una
persona respire mientras mastica es:

- el movimiento hacia afuera de los pliegues palatogloso y palatofaríngeo

- la elevación del paladar blando

- la depresión de la parte posterior de la lengua

- la depresión del paladar blando

3. Paciente adulto con reflujo gastroesofágico es más probable que presente:

- descalcificación del esmalte

- destrucción de ameloblastos

- remodelación del esmalte

- desfluorización de los dientes

4. La masticación es básicamente:

- importante para la digestión sobretodo de carnes

- un ralentizador del vaciamiento gástrico

- una actividad consciente

- un movimiento reflejo

5. Paciente de 34 años es víctima de asalto con arma de fuego, recibiendo un impacto


directo en el abdomen. En base a la radiografía, usted puede registrar en la historia
clínica que el proyectil se encuentra topográficamente en el:
- flanco derecho

- mesogastrio

- flanco izquierdo

- hipocondrio izquierdo

6. Señale cuál de las estructuras que en el embrión se encuentra comunicada con el


saco vitelino por medio del conducto onfalomesentérico:
- D

- B

- C

- A

7. En un paciente de 3 semanas de edad, con vómitos en proyectil, y nódulo


epigástrico reptante, usted esperaría encontrar:

- colecistoquinina aumentada

- vómitos biliosos e intolerancia a los ácidos grasos

- distensibilidad disminuida de la región oral del estómago

- engrosamiento de la circular interna pilórica

8. ¿Cuál de las siguientes estructuras tiene inervación somática?

- Estómago

- Peritoneo visceral

- Mesosigmoides

- Peritoneo parietal

9. Estas diseñando un proyecto de investigación sobre los niveles de colesterol que se


absorben luego de una comida grasosa y deseas cuantificar la cantidad de colesterol
que es absorbido por el intestino antes que el hígado lo metabolice ¿de cuál de los
siguientes vasos obtendrías la muestra para tu análisis?
- Vena porta

- Vena cava superior

- Conducto torácico

- Vena hemiácigos accesoria

10. Durante el paso del bolo hacia la orofaringe, se desencadena una serie de
contracciones musculares que estrechan la cavidad faríngea. Estas contracciones están
mediadas por el nervio craneal:

- X

- XI

- XII

- IX

11. Cuando el istmo de las fauces se cierra, se evita que el alimento pase hacia la
orofarínge y permite respirar mientras se mastica. Este cierre se debe a la contracción y
aproximación de los músculos:

- Palatoglosos

- Palatofaríngeos

- Estiloglosos

- Estilofaríngeos

12. Las siguientes alternativas son factores que determinan la patencia y función
adecuada del esfínter esofágico inferior, EXCEPTO:

- Plicatura diafragmática

- Hipertrofia de la circular interna

- Angulación con el estómago

- Canales lentos de calcio

13. Respecto al peristaltismo intestinal, para cumplir la ley del intestino , usted espera
que a nivel distal del quimo se libere:

- péptido liberador de gastrina (GRP)

- acetilcolina

- péptido intestinal vasoactivo

- sustancia P
14. El estímulo habitual para el movimiento peristáltico es:

- acción de la sustancia P

- contracción de la musculatura circular Interna

- estimulación vago-vagal

- distensión local

15. ¿Cuál de las siguientes condiciones considera que es un trastorno de la


musculatura lisa esofágica?

- Acalasia

- Asinergia faringoesfinteriana

- Hipotonía de los constrictores faríngeos

- Hipertonía del esfínter esofágico superior

16. ¿Cuál de las siguientes alternativas es correcta sobre la motilidad esofágica?

- Las ondas primarias son propulsoras y pueden no ser precedidas por


deglución

- Las ondas primarias no son propulsoras y siempre van precedidas de


deglución

- Las ondas secundarias son propulsoras y siempre van precedidas de


deglución

- Las ondas secundarias son propulsoras y no van precedidas de


deglución

17. El peristaltismo depende que a nivel distal del bolo se secrete:

- noradrenalina secretada por las fibras del sistema simpático

- acetilcolina por las neuronas provenientes del nervio vago

- óxido nítrico por células endoteliales locales

- péptido intestinal vasoactivo por neuronas

18. ¿Cuál de las siguientes alternativas es correcta sobre el movimiento peristáltico?

- Es un reflejo largo que depende de la integración con el tronco


encefálico

- Se dirige en sentido distal siempre, nunca en sentido proximal

- El contenido intestinal avanza sólo 5-10 cm


- Es independiente del plexo mientérico

19. En un estudiante de medicina que está rindiendo un examen parcial, lo más


probable es que en ese momento su tránsito intestinal se encuentre:

- muy lento

- muy acelerado

- estimulado por acción de la sustancia P

- sin alteraciones

20. Las siguientes alternativas son ciertas sobre las contracciones tónicas del músculo
gastrointestinal, EXCEPTO:

- Tienen relación con el ingreso persistente de iones sodio

- Se encuentran principalmente en esfínteres

- Tienen regulación hormonal

- Obedece a una mayor frecuencia de potenciales en espiga

21. Con respecto de la regulación del pH del estómago; al utilizar un bloqueador de


histamina, usted espera que el pH del estómago:

- aumente

- disminuya

- se mantenga sin cambio

- se neutralice por acción de bicarbonato

22. Las siguientes hormonas disminuyen el vaciamiento gástrico, EXCEPTO:

- Gastrina

- Péptido insulinotrópico dependiente de glucosa

- Colecistoquinina

- Secretina

23. El frenillo de los labios se encuentra en:

- el piso de la boca

- la cavidad oral

- la cavidad vestibular
- el dorso de la lengua

24. En un paciente con shock hipovolémico, la peristalsis intestinal se encuentra:

- aumentada

- sin cambios

- invertida

- disminuida

25. Sobre el control de la peristalsis del tubo digestivo, ________________ es un


mediador neural que induce la relajación durante la peristalsis.

- la somatostatina

- el péptido intestinal vasoactivo

- la acetilcolina

- la serotonina

CI 3

1. En cuanto a las sustancias secretadas por el estómago ¿Cuál de las siguientes


sustancias estimula la liberación de pepsinógeno?

- Pepsinógeno

- Secretina

- Colecistoqunina

- Gastrina

2. Las células enteroendocrinas en el estómago se localizan en la glándula oxíntica, al


mismo nivel que las células:

- parietales

- principales

- mucosas

- absortivas

3. La fase intestinal de la secreción gástrica se debe básicamente a la participación de


las células:

- G del duodeno

- I del yeyuno
- S del íleon

- D del estómago

4. El conducto de Stenon, para entrar a la cavidad vestibular, debe atravesar el


músculo:

- genihioideo

- buccinador

- milohioideo

- masetero

5. Paciente con acalasia es sometido a tratamiento endoscópico o quirúrgico, usted le


ha informado al paciente previamente que es posible que una complicación de este
tratamiento es que quede con cierto grado de:

- reflujo gastroesofágico

- gastritis

- odinofagia

- úlceras gástricas

6. La sangre que lleva la vena porta es tipo:

- mixta

- arterial

- venosa

7. Los pliegues gástricos gruesos son prácticamente inexistentes a nivel de:

- el cuerpo

- el fondo

- la incisura angularis

- el antro

8. En la producción de HCl, la acción de la somatostatina disminuye la accion de:

- las prostaglandinas

- la histamina

- la acetilcolina
- la gastrina

9. En el estómago se secretan las siguientes sustancias, EXCEPTO:

- grelina

- Correcto

- motilina

- somatostatina

- gastrina

10. La vena porta se forma gracias a la unión de la vena mesentérica superior con la
vena:

- mesentérica inferior

- esplénica

- celiaca

- gástrica izquierda

CI 4

1. En el hígado, el aumento de la resistencia vascular en los sinusoides hepáticos


ocasionará:

- aumento de la presión de llenado vesicular

- salida de plasma hacia el intersticio

- aumento del flujo hacia la vena porta

- aumento del flujo hacia la vena cava superior

2. En un paciente con carcinoma de páncreas, el tumor ha invadido la unión entre la


venas esplénica y mesentérica superior; eso quiere decir que estamos seguros que el
tumor se encuentra a nivel del ________ del páncreas.

- cuerpo

- cola

- cabeza

- cuello

3. En un paciente con cirrosis hepática la cabeza de medusa que aparece en la pared


abdominal, podría desaparecer si al paciente se le:
- esclerosa las venas hemorroides internas

- oblitera el ligamento redondo

- administra antiandrógenos

- oblitera la arteria gástrica izquierda

4. En un paciente con intoxicación por órganos fosforados, la acción de la


colecistoquinina (CCK) está bloqueada a nivel de:

- el esfínter de Oddi

- la vesícula biliar

- la célula parietal

- el sistema nervioso central

5. Con respecto a la microestructura del hígado ¿Cuál de las siguientes alternativas es


correcta?

- La célula de Ito se encuentra en el espacio de Disse y reserva glucógeno

- Los hepatocitos están interconectados por uniones herméticas

- La célula de Kupffer se encuentra fuera del sinusoide y fagocita células

- El sinusoide es un capilar fenestrado

6. Paciente con Lupus Eritematosos que desarrolla hipertensión portal debido a


trombosis portal, es probable que desarrolle várices a nivel de:

- recto inferior

- recto superior

- hemorroides externas

- canal anal

7. El efecto de un medicamento colerético se evidencia por:

- la mayor producción de colesterol en la bilis

- la disminución de absorción de sales biliares

- el aumento de secreción biliar

- el aumento de formación de micelas

8. En un paciente con cáncer de páncreas y que desarrolla ictericia, la localización


más probable del tumor es en:
- la vesícula por metástasis

- el cuello del páncreas

- la cabeza del páncreas

- la cola del páncreas

9. En un recién nacido menor de 24 horas con atresia biliar, se encuentra elevación de


la:

- bilirrubina directa

- bilirrubina indirecta

- alanina aminotransferasa (ALT)

- hemoglobina

10. ¿Cuál de las siguientes alternativas es correcta sobre la estructura hepática?

- Los colangiocitos producen bilis

- En la triada portal, se encuentra la vena derivada de la suprahepática

- El flujo sinusoidal en el lobulillo hepático es de adentro hacia afuera

- El flujo biliar en el lobulillo hepático es centrífugo

1. Después de la ligadura del cordón umbilical la vena umbilical se oblitera quedando


como vestigio fibroso, el:

a) ligamento coronario

b) ligamento falciforme

c) ligamento de Arancio

d) ligamento triangular

e) ligamento redondo

1. Unidades funcionales del hígado :

a)Lobulillo hepático

b) Lobulillo portal

c) acino hepático
d) Espacio porta

e) Espacio de Kiernan

1. Plexo nervioso del sistema nervioso vegetativo o autónomo que controla


principalmente la secreción glandular gastrointestinal:

a) Plexo de Auerbach

b) Plexo lumbar

d) Plexo cervical

c) Plexo sacro

e) Plexo de Meissner

1. La digestión de las proteinas se inicia en el estómago mediante y termina en el


intestino delgado mediante enzimas provenientes del...

a) renina - Jugo intestinal

b) pepsina - jugo intestinal

c) amilasa - Jugo pancreático

d) pepsina - jugo pancreático

e) cuajo - Jugo pancreático y bilis

1. El reflejo de defecación envia impulsos nerviosos a la medula sacra, que origina


impulsos motores que llegan al colon sigmoideo, colon descendente, recto y ano,a
través de:

a) Sistema nervioso central

b) Encéfalo

c) Sistema nervioso simpático

d) Sistema nervioso somático

e) Sistema nervioso parasimpático


1. La mucosa gástrica presenta epitelio de tipo:

a) Poliestratificado cilindrico

b) Simple cúbico modificado

c) Polimorfo

d) Simple cilindrico no modificado

e) Simple cubico no modificado

1. Extensión amplia del peritoneo que recubre y suspende al yeyuno e ileon a la pared
abdominal posterior:

a) Mesocolon transverso

b) Epiplón menor

c) Mesenterio

d) Epiplón mayor

e) Ligamento hepatoduodenal

1. En relación a las glándulas salivales; lo incorrecto es:

a) La parótida presenta conducto de Stenon.

b) La secreción salival de la submaxilar es seromucosa

c) El conducto de Rivinus corresponde a lasublingual

d) El conducto de Wharton corresponde a la submaxilar

e) La secreción salival de la parótida es serosa y secreta el 70% del total de la saliva

1. El proceso de degradación de glucógeno en glucosa, recibe el nombre de:

a) Glucogénesis

b) Gluconeogénesis

c) Glucogenólisis

d) Glucólisis

e) Glicoilsis
1. Cuando los riñones dejan de producir orina debido a obstrucción de los nefrones de
uno o ambos riñones, a esta anomalía se le denomina:

a) Enuresis

c) Oliguria

e) Disuria

b) Anuria

d) Cistitis

1. Se caracteriza por la disminución progresiva y por lo común irreversible de la


velocidad de filtración glomerular. Con efectos tales como edema, acidosis, uremia,
anemia, aumento de niveles de potasio osteomalacia Denominándose a este
trastorno:

a) Insuficiencia renal

b) Nefrosis

c) Insuficiencia renal crónica

d) Enfermedad de Britaht

e) Pielonefritis

Preguntas (cami)

1. En un paciente con cáncer de páncreas y que desarrolla ictericia, la localización más


probable del tumor es en:
- la cola del pancreas
- la cabeza del pancreas
- la vesicula por metastasis
- el cuello del pancreas

José G - CI 4

1. Marque lo correcto respecto al acino hepático:


a. Dos vértices del rombo acinar se constituyen por triadas portales
2. En un paciente con infección por SARS-CoV-2 con compromiso severo e ingresado en la
unidad de cuidados intensivos (UCI), debido a la tormenta de citoquinas y a la desregulación
inmune ¿Cuál de las siguientes alternativas estará elevada en sangre al evaluar el perfil
hepático?
a. Transaminasas
3. La relajación del esfínter de Oddi se produce directamente por acción de:
a. el péptido vasoactivo intestinal (VIP)
4. La presión parcial de oxígeno en la Zona 1 del sinusoide hepático de debe ser _____ mmHg
a. entre 95 y 45
5. En un recién nacido menor de 24 horas con atresia biliar, se encuentra elevación de la:
a. bilirrubina directa
6. En un paciente con una estenosis severa del colédoco por una complicación quirúrgica, usted
esperaría que desarrolle:
a. el aumento del tiempo de protrombina
7. En un paciente con cirrosis hepática la cabeza de medusa que aparece en la pared abdominal,
podría desaparecer si al paciente se le:
a. oblitera el ligamento redondo
8. En el síndrome de Mirizzi, el paciente tiene cálculos en la vesícula biliar; pero se obstruye el
conducto hepático común debido a que un cálculo se ubica y crece de tamaño en:
a. la bolsa de Hartmann
9. Con respecto a la microestructura del hígado ¿Cuál de las siguientes alternativas es correcta?
a. Los hepatocitos están interconectados por uniones herméticas
10. En un paciente con intoxicación por órganos fosforados, la acción de la colecistoquinina (CCK)
está bloqueada a nivel de:
a. el esfínter de Oddi

1. Dentro de las funciones del abdomen, se encuentran la defecación y micción, en las cuales la
presión intra abdominal debe:

a. aumentar

2. Paciente se queja de dolor en hipocondrio derecho, pero superficialmente. El dermatoma


relacionado es (marque la mejor respuesta):

a. T9

3. En la evaluación de una tomografía abdominal, el interno observa un aneurisma en una arteria que
se dirige al riñón derecho. Con seguridad se puede afirmar que está a nivel de la vértebra:

a. L2

4. Al evaluar una tomografía abdominal, el médico asistente le pide al interno de la UPC que
encuentre la imagen con el corte a nivel de L1. El interno sabiamente busca el ________ para
ubicar la vértebra L1.

a. cuello del páncreas

5. Paciente de 24 años con dolor abdominal tipo cólico intenso en mesogastrio. Según sus
conocimientos de macroestructura, el origen del dolor puede ser el ___________:

a. Colon
6. Al examinar a un paciente, usted encuentra dolor localizado en fosa iliaca derecha y diagnostica
apendicitis. En este paciente, usted puede inferir:

a. el peritoneo parietal regional está afectado

7. Respecto a la anatomía del estómago, marque lo correcto

a. el fondo gástrico forma la curva mayor

8. Marque el órgano que se encuentra más distal en el tubo digestivo.

a. ciego

9. La peristalsis o peristaltismo hace referencia a

a. Motilidad para movilizar el alimento de proximal a distal

10. Al retirar completamente el mesenterio de un órgano, el mismo se vería afectado principalmente


en su

a. irrigación

1.Paciente con apendicitis aguda, que debuta con dolor en mesogastrio. Este dolor se debe a
estimulación de receptores del dolor cuyas fibras van a viajar a la médula espinal a través de
a. nervios simpáticos
2. Paciente con intoxicación por órganos fosforados (inhibidores de acetilcolinesterasa), se espera
que el tránsito intestinal se encuentre
a. aumentado

3. Al seccionar el nervio facial a nivel timpánico, usted esperaría


a. disminución del gusto en la punta de la lengua

4. Al ingerir rápidamente un litro de agua, usted esperaría que la gastrina aumente por efecto de
a. ACh del sistema mientérico

5.Marque lo correcto en relación al divertículo de Meckel


a. Se encuentra usualmente 60 cm de la VIC

6. usted encuentra músculo estriado en el siguiente segmento


a. esfinter anal externo

7. Al introducir una solución azucarada directamente al estómago mediante una gastrostomía


(comunicación entre la piel abdominal y el estómago), la sustancia que provocará que aumenten los
niveles séricos de insulina es
a. Péptido tipo glucagón 1 (GLP - 1)
8. Marque lo correcto respecto a la siguiente imagen
a. se produjo por falta de fusión de los ductos dorsal y ventral

9. es un ligamento derivado del mesenterio dorsal


a. gastrocólico

10. es un paciente con diarrea por hipermotilidad, usted sospecharía en el posible aumento de las
siguientes sustancias, excepto
a. péptido intestinal vasoactivo

1. En una gestante de 11 semanas con antecedente de esofagitis eosinofílica, con hiperémesis


gravídica, que acude a emergencia por hematemesis leve, y presenta súbitamente disnea y
dolor torácico. Usted sospecharía de
a. sindrome de Mallory Weiss

2. En un paciente que usa AINEs a altas dosis por artritis reumatoide, usted esperaría que presente
erosiones y úlceras gástricas debido a
a. disminución de irrigación en la mucosa

3. la saliva siempre será hipotónica debido a


a. la impermeabilidad de los conductos al agua

4. en el estómago se secretan las siguientes sustancias, EXCEPTO


a. motilina

5. Cuando un paciente recibe estímulo autónomo mixto (simpático y parasimpático), el flujo de saliva
a. aumenta en relación al basal
6. La fase intestinal de la secreción gástrica se debe básicamente a la participación de las células
a. G del duodeno

7. la inhibición de secreción gástrica es secundaria a:


a. el reflejo enterogástrico

8. aquellas células que producen mayor cantidad de moco en el epitelio gástrico son:
a. mucosas superficiales

9. ¿cuál de las siguientes alternativas es causa de reflujo gastroesofágico de contenido ácido?


a. hernia hiatal
10. para determinar que un paciente tiene esófago de Barrett, debemos encontrar…………. en la
biopsia de esófago
a. células calciformes

1. ¿ cuál de las siguientes alternativas es la correcta sobre la estructura hepática?


a. el flujo biliar en el lobulillo hepático es centrífugo
2. en un paciente cirrótico con encefalopatía hepática ¿cuál de las siguientes alternativas sustenta la
reducción de carnes rojas en la dieta?
a. el amonio se produce principalmente en el intestino
3. para poner absorber al torrente sanguíneo todos los carbohidratos del azúcar común (de mesa) es
necesario usar únicamente los transportadores:
a. GLUT2, SGLT1, GLUT5

4. En un paciente con una estenosis severa del colédoco por una complicación quirúrgica, usted
esperaria que desarrolle
a. el aumento del tiempo de protrombina

5. en el síndrome de Mirizzi, el paciente tiene cálculos en la vesícula biliar, pero se obstruye el


conducto hepático común debido a que un cálculo se ubica y crece de tamaño en:
a. bolsa de Hartmann

6. en un paciente con carcinoma de páncreas, el tumor ha invadido la unión entre las venas esplénica
y mesentérica superior: esto quiere decir que estamos seguros que el tumor se encuentra a nivel
del….. del páncreas
a. cuello

7. las siguientes sustancias son secretadas por el páncreas, EXCEPTO:


a. tripsina

en un paciente con intoxicación por organofosforados, la acción de la colecistoquinina (CCK) está


bloqueada a nivel de:
a. el esfínter de Oddi

9. en el hígado, el aumento de la resistencia vascular de los sinusoides hepáticos ocasionará:


a. aumento del flujo hacia la vena porta
b. salida de plasma hacia el intersticio
10. en un paciente con cáncer de páncreas y que desarrolla ictericia, la localización más probable del
tumor es en:
a. la cabeza del páncreas

LORENA FERNÁNDEZ
1 . El conducto colédoco se forma por la convergencia de los conductos
Cístico - hepático

2 . Las glándulas parótidas y submaxilares respectivamente presentan como conductos


Stenon y Wharton

3 . Membrana epitelial que sostiene a casi todas las vísceras del abdomen
Peritoneo

4 . En el mecanismo de formación de HCl intervienen los siguientes elementos químicos


H2O, CO2, H, H2CO3

5 . La gingivorragia se produce por deficiencia de vitamina


C

6 . Fase de secreción de HCl que se activa para saborear


Vagal y Neurógena

7. Producen colecistoquinina
Células APUD

8. El tripsinógeno es activado por


Enterocinasa

9 . Secreta grandes cantidades de bicarbonato y agua, para neutralizar la acidez del quimo
Páncreas

10 . La sacarosa se desdobla en
Glucosa y fructosa

11. Enfermedad en que no hay relajación del cardias como respuesta de deglución y hay falta de
peristaltismo en el esofago
Acalasia

12 . Histológicamente la mucosa del tubo digestivo presenta las siguientes regiones, excepto
muscular

13 . Es la porción principal y de mayor tamaño del estómago, en la cual se forma el quimo


Cuerpo

14 . Entre las capas musculares se encuentran células ganglionares y haces de fibras nerviosas
amielínicas que en conjunto representan
Plexo mientérico de Auerbach

15 . A nivel del íleon se encuentran agregaciones permanentes de nódulos linfoides llamados


Placas de Payer

Un neonato con vómitos biliosos, persistentes y con baja ponderal, usted sospecharia:
Atresia duodenal

Paciente ha sido puesto en “nacida por vía oral” (nil per os, NPO) durante 24 horas, la sensación de
hambre en este paciente se debe la par esencia en el hipotálamo de la sustancia:
Neuropéptido Y

La absorción de ácido fólico se produce gracias a:


cotransporte dependiente de Na

Paciente con deficiencia congénita de procolipasa, sufre de esteatorrea cada vez que come comidas
ricas en grasa. Los lípidos que no son hidrolizados debido a esta deficiencia son
Triglicéridos

En paciente con hipertensión portal, la causa más probable para una hematemesis es
varices esofágicas
Varón de 73 años es traído a urgencias con dolor abdominal intenso. En la tomografía se observa un
aneurisma de la aorta abdominal que afecta al origen de la arteria mesentérica superior. Basado en
sus conocimientos, usted sabe que el órgano que se encontrará más afectado será:
Yeyuno

En los neonatos sanos, el color verde oscuro del meconio (primera deposición) se debe a
pigmentos biliares

¿cual de las siguientes alternativas es correcta sobre el control del peristaltismo?


el peristaltismo intestinal aumenta por efecto de colecistoquinina

En un paciente con falla en la fusion de los conductos de las yemas ventral y dorsal del pancreas,
usted esperaria encontrar:
drenaje de la mayor parte del jugo pancreatico en la papila menor

En un paciente alcohólico con esplenomegalia, bilirrubina elevada y tiempo de protrombina elevado,


que acude por melena, usted sospecharía que esta se debe a
Várices esofágicas

En una cirugia abierta(laparotomia), el cirujano al abrir la cavidad peritoneal por la parte anterior (linea
media), lo primero que observa es:
epiplon mayor

El tubo digestivo posee glandulas, las glandulas submucosas se encuentran en el:


esofago y duodeno

Los reflejos enterogastricos son desencadenados por las siguientes circunstancias,excepto:


alta carga de carbohidrato en duodeno

Las siguientes alternativas son ciertas en relacion al mesenterio,excepto:


los omentos cumplen una funcion principal de irrigacion visceral

Las carnes deben su sabor especialmente delicioso debido a que presentan en su composicion:
glutamato

Con respecto de la regulacion del pH del estomago; al utilizar un bloqueador de histamina, usted
espera que el pH del estomago:
aumente

Al hacer una endoscopia, usted puede inferir los segmentos del esofago, gracias a algunas
estrecheces, excepto:
bronquio derecho

Cuando el itsmo de las fauces se cierra, se evita que le alimento pase hacia la orofaring y permite
respirar mientras se mastica. Este cierre se debe a la contraccion y aproximacion de los musculos:
palatoglosos

En el plexo mienterico, el origen de los impulsos eferentes esta en:


el plexo de meissner
Las siguientes alternativas son factores que determinan la patencia y funcion adecuada del esfinter
esofagico inferior, excepto:
hipertrofia de la circular interna

La rotacion en sentido longitudinal del estomago en el desarrollo embriologico condiciona que el


nervio vago derecho quede a nivel:
posterior

Paciente de 64 años de edad con tumor en cabeza del páncreas que compromete a una arteria que
discurre entre la cabeza del páncreas y el proceso uncinado. Dicha arteria es:
Mesentérica superior

Los quilomicrones llegan a los hepatocitos por:


Sinusoides

Con respecto a la regulacion del peristaltismo, al aplicarle atropina (antagonista colinergico) a un


paciente, es de esperarse que el peristaltismo:
disminuya

En la estructura dentaria, se observa que hay una composicion muy similar a la del hueso en la capa
denominada:
cemento

En un paciente con carcinoma de pancreas, el tumor ha invadido la union entre la venas esplenica y
mesentérica superior, eso quiere decir que estamos seguros que el tumor se encuentra a nivel del ----
----pancreas:
cuello
Marque lo correcto respecto al acino hepatico:
Dos vertices del rombo acinar se contituyen por triada portales

En un paciente con intoxicacion por organos fosforados, la accion de la colecistoquinina (CCK) esta
bloqueada a nivel de:
el esfinter de Oddi

Las siguientes sustancias son secretadas por el pancreas, excepto:


tripsina

En un paciente con cirrosis hepatica se desarrollara ascitis debido a los siguientes mecanismos,
excepto:
estrechamiento de la porta

La via biliar extrahepatica se encuentra ubicada dentro del ligamento:


hepatoduodenal

En un paciente con cancer de pancreas y que desarrolla ictericia, la localizacion mas probable del
tumor es:
la cabeza del páncreas

En el sindrome de Mizzi, el paciente tiene calculos en la vesicula biliar; pero se obstruye el conducto
hepatico comun debido a que un calculo se ubica y crece de tamaño en:
la bolsa de hartmann
En un paciente con una estenosis severa del coledoco por una complicacion quirurgica, usted
esperaria que desarrolle:
el aumento del tiempo de protrombina

En la anemia perniciosa, con atrofia gastrica marcada, usted esperaria encontrar dsiminucion en la
absorcion de -----------a nivel de------------
vitamina B12 / ileon distal

En un paciente es derivado por lesion del nervio vago derecho.Al evaluar el velo del paladar, e solicita
al paciente que diga ahh, entonces se puede observar que la uvula:
se desvia a la izquierda

Paciente con lesion del hipogloso del lado izquierdo.Para evaluarlo se le pide al paciente que saque
la lengua, la cual se espera que la punta de la lengua se dirija hacia:
el lado izquierdo

En cuanto a las sustancias secretadas por el estomago. ¿cual de las siguientes sustancias estimula
la liberacion de pepsinogeno?
secretina

La sangre que lleva la vena porta es tipo:


venosa

El conducto de stennon, para entrar a la cavidad vestibular, debe atravesar el músculo:


buccinador

En la produccion de HCL, la acción de la somatostatina disminuye la accion de:


la gastrina

En un paciente con xerostomía presenta las siguientes condiciones, excepto:


infeccione del oido a repeticion

Las siguientes alternativas son correctas sobre la deglucion, excepto:


consta de 2 fases

Aquellas celulas que producen mayor cantidad de moco en el epitelio gastrico son las:
mucosas superficiales

La fase intestinal de la secrecion gastrica se debe basicamente a la participacion de las celulas:


G del duodeno

La distension del yeyuno provoca que se:


despolarice el potencial de reposo de membrana

La hormona responsable de los complejos migratorios interdigestivos tiene las siguientes


caracteristicas, excepto:
cumple funciones de aumentar la motilidad y secrecion gastrica e intestinal

El peristaltismo intestinal se produce gracias a un reflejo que:


se origina dentro de la pared intestinal
paciente de 17 años, con diarrea crónica sin moco ni sangre, asociado a dolor abdominal, con
antecedente de masticar de 6 a 10 tableta de chicle trident que contiene sorbitol.El mecanismo de
este tipo de diarrea es:
osmotica

El peristaltismo depende que a nivel distal del bolo se secrete:


peptido intestinal vasoactivo por neuronas

Las heces tienen olor característico, cuyo responsable es:


mercaptanos

Con respecto de la regulacion del pH del estomago; al utilizar un bloqueador de histamina, usted
espera que el pH del estomago:
aumente

Las siguientes alternativas son cierta en relacion al mesenterio,excepto:


los omentos cumplen una funcion principal de irrigacion viscera

En un estudiante de medicina que esta rindiendo un examne parcial, lo mas probable es que en ese
momento su transito intestinal se encuentre:
muy lento

¿cual de las siguientes alternativas es correcta sobre la motilidad esofagica?


la ondas son propulsoras y no van precedidas de deglucion

¿cual de los siguientes requiere de la formación de micelas para su absorción intestinal?


vitamina K

paciente con volvulo del colon sigmoides.La necrosis de este segmento del colon se produce por una
alteración en la irrigación de la arteria:
mesentérica inferior

Además de acidosis metabólica, las diarreas se acompañan de:


Hipokalemia

En los pacientes con diarrea asociada al consumo de pastas o pan, y con anticuerpos antigliadina, la
malabsorción se produce básicamente por:
Atrofia de las vellosidades

En un paciente con cáncer de páncreas y que desarrolla ictericia, la localización más probable del
tumor es en

Cabeza del páncreas

Paciente de 62 años con vólvulo de intestino delgado e isquemia intestinal. ¿Qué estructura se utiliza
como punto de referencia para determinar la posición de la unión duodenoyeyunal?

Ligamento suspensorio del duodeno (Treitz)

La acción de la secretina sobre las células ductales del páncreas, es potenciada por:

Acetilcolina
¿Cuál de las siguientes sustancias tiene una mayor concentración en la saliva comparado con su
concentración plasmática?

potasio

Paciente con mutación de la enzima UDP glucuronil transferasa que la hace menos activa, al hacerle
un análisi de sangre, encontraremos valores elevados de (marque la mejor respuesta):

Bilirrubina indirecta

La localización de la vesícula biliar con respecto al lóbulo cuadrado es:

Lateral

En un paciente con un cálculo en la ampolla de Váter, usted esperaría encontrar en sangre


principalmente:

Fosfatasa alcalina elevada

El conducto colédoco recibe dicho nombre a partir de

La unión del conducto cístico con el hepático común

Paciente de 2 años, llega a emergencia por haber ingerido una moneda con la que estaba jugando,.
El lugar más probable donde puede haber quedado este objeto es sobre el estrechamiento a nivel del
músculo:

Cricofaríngeo

¿Cual de las siguientes alternativas es una comunicación entre el tejido hepático y la vesícula biliar?

Conducto de Luschka

NO es una característica del proceso llamado relajación receptiva del estómago:

Se produce posteriormente al cierre del esfínter esofágico inferior

Calificación Individual 01 - 202002


1. Al examinar a un paciente, usted encuentra dolor localizado en fosa iliaca derecha y
diagnostica apendicitis. En este paciente, usted puede inferir:
a. El peritoneo visceral regional está principalmente afectado
b. El peritoneo parietal regional está afectado
c. Hay inflamación de todo el peritoneo parietal (peritonitis)
d. El diagnóstico está errado por no corresponder a la región abdominal adecuada

2. Marque el órgano que se considera retroperitoneal:


a. Lóbulo izquierdo del hígado
b. Vesícula biliar
c. Sigmoides
d. Parte de la vía biliar

3. La peristalsis o peristaltismo hace referencia a:


a. Motilidad para movilizar el alimento de proximal a distal.
b. Motilidad para mezclado de alimentos.
c. No es parte de la motilidad
d. Motilidad para fraccionamiento de alimentos.

4. La motilidad intestinal es estimulada principalmente por el:


a. Sistema parasimpático
b. Plexo de Auerbach
c. Sistema piramidal
d. Sistema simpático

5. La estructura que fija órganos principalmente a la pared posterior abdominal se denomina:


a. Omento
b. Ligamento
c. Mesenterio
d. Fascia transversalis

6. Paciente con herida por proyectil por arma de fuego, con herida de ingreso en región
paraumbilical. Entre las estructuras que usted está seguro que debe haberse lesionado es:
a. Omento menor
b. Ligamento de Treitz
c. Mesosigmoides
d. Omento mayor

7. Paciente joven es traído a emergencia con abdomen agudo quirúrgico debido a herida contuso
penetrante por verduguillo (alambre grueso con punta aguzada) recibida en una pelea después de
un partido de futbol. Se observa herida en Hipocondrio Izquierdo. El órgano que debe estar
sangrando y produciendo hemoperitoneo es (marque la mejor respuesta):
a. Colon ascendente
b. Colon sigmoides
c. Hígado
d. Bazo

8. Al evaluar una tomografía abdominal, el médico asistente le pide al interno de la UPC que
encuentre la imagen con el corte a nivel de L1. El interno sabiamente busca el ________ para
ubicar la vértebra L1.
a. Nacimiento de la vena mesentérica superior
b. Nacimiento de la arteria mesentérica inferior
c. Cuello del páncreas
d. Cruce entre la Aorta y la Vena Porta

9. El ligamento hepatogástrico une el _______________ con el _______________ y forma la


entrada al ___________:
a. Hígado Estómago Orificio esofágico
b. Hígado Estómago Orificio omental
c. Estómago Hígado Orificio anal
d. Estómago Hígado Orificio gastrointestinal

10. En la inspiración, la pared abdominal debe ____________ para ____________:


a. Contraerse aumentar presión intra torácica
b. Relajarse aumentar presión intra abdominal
c. Contraerse aumentar presión intra abdominal
d. Relajarse disminuir presión intra torácica

Calificación Individual 02 - 202002


1. Al seccionar el nervio facial a nivel timpánico, usted esperaría:
a. Imposibilidad para protruir la lengua
b. Disminución del gusto en la punta de la lengua
c. Imposibilidad para el cierre del istmo de las fauces.
d. Ausencia de termoalgesia en la lengua
2. Marque lo correcto en relación al divertículo de Meckel.
a. Se encuentra usualmente a 60 cm de la VIC
b. Contiene mucosa esofágica en algunas ocasiones
c. Se relaciona a un defecto en el desarrollo del intestino posterior
d. Se produce en el lado mesentérico del íleon

3. Paciente obeso con Covid-19 es intubado por interno inexperto, quien al solicitar que bombeen
aire dentro del tubo endotraqueal, nota que el epigastrio se distiende. Al sospechar que ha
introducido el tubo en el estómago, también es cierto que:
a. Disminuye el pH gástrico
b. Aumenta el pH gástrico
c. Disminuiría el tono del píloro
d. Aumenta la frecuencia de ondas lentas

4. En un experimento, con una sonda nasogástrica se instila por goteo en el estómago una
sustancia líquida, y se obtiene como respuesta una dramática disminución del pH del estómago.
Dicha sustancia debe contener:
a. Lípidos
b. Carbohidratos
c. Aminoácidos
d. Secretina

5. Al introducir una solución azucarada directamente al estómago mediante una gastrostomía


(comunicación entre la piel abdominal y el estómago), la sustancia que provocará que aumenten
los niveles séricos de insulina es:
a. Péptido tipo glucagón 1 (GLP-1)
b. Enteroglucagon
c. Glucagon
d. Somatostatina

6. De los diferentes reflejos gastrointestinales, hay uno que produce movimiento del contenido
hacia la región distal, y se llama reflejo:
a. Gastrocólico
b. Entero-gástrico
c. Cólico-ileal
d. Vómito

7. En un paciente con gastroparesia (motilidad lenta del estómago), que presenta distensión
abdominal después de comer, usted le recomendaría que evite el consumo de lípidos y
aminoácidos para disminuir la acción de:
a. Somatostatina
b. Secretina
c. CCK
d. Gastrina

8. Una de las siguientes sustancias reguladoras, puede actuar de forma paracrina y como
hormona. Marque la correcta:
a. Somatostatina
b. Acetilcolina
c. GRP
d. Péptido insulinotrópico dependiente de glucosa.
9. Paciente con apendicitis aguda, que debuta con dolor en mesogastrio. Este dolor se debe a
estimulación de receptores del dolor cuyas fibras van a viajar a la médula espinal a través de:
a. Nervios simpáticos
b. Plexo hipogástrico
c. Nervio esplácnico pélvico
d. Nervio vago

10. En un paciente con hiperestimulación simpática se espera que las ondas lentas tengan un
ritmo:
a. Mayor en íleon que en duodeno
b. Menor en íleon terminal que en el duodeno
c. Mayor en estómago que en duodeno
d. Mayor en estómago que en íleon terminal

Calificación Individual 03 - 202002


1. La comunicación entre la irrigación gástrica y la esofágica depende de una rama de la arteria:
a. mesentérica superior
b. hepática común
c. mesentérica inferior
d. tronco celiaco

2. En un paciente que usa AINEs a altas dosis por artritis reumatoide, usted esperaría que
presente erosiones y úlceras gástricas debido a:
a. el aumento de prostaglandinas
b. la disminución de irrigación en la mucosa
c. el aumento de histamina
d. la inhibición de secreción de secretina

3. Las células enteroendocrinas en el estómago se localizan en la glándula oxíntica, al mismo


nivel que las células:
a. Principales
b. Parietales
c. Absortivas
d. Mucosas

4. En la anemia perniciosa, con atrofia gástrica marcada, usted esperaría encontrar disminución
en la absorción de ___________ a nivel de ________
a. vitamina B12 / íleon distal
b. proteínas / yeyuno
c. hierro / duodeno
d. ácido fólico / yeyuno

5. La saliva siempre será hipotónica debido a:


a. la mayor permeabilidad al sodio en los conductos
b. el estímulo del sistema parasimpático
c. la ausencia de conductor estriado
d. la impermeabilidad de los conductos al agua

6. Un paciente con polimiositis posee alteración en la regulación del mecanismo de la deglución;


por eso hay que considerar la deglución de la saliva, pues a diario se produce _______mL
a. 500
b. 1000
c. 100
d. 300
7. La inhibición de secreción gástrica es secundaria a:
a. la acción de la bombesina
b. el reflejo enterogástrico
c. la estimulación de células G
d. la activación de la bomba de protones

8. Al usar atropina en un paciente, usted esperaría:


a. la hipersalivación
b. la disminución de gastrina
c. el aumento de histamina
d. la disminución de secreción gástrica por bloqueo de M3

9. En una gestante de 11 semanas con antecedente de esofagitis eosinofílica, con hiperémesis


gravídica, que acude a emergencia por hematemesis leve, y presenta súbitamente disnea y dolor
torácico. Usted sospecharía de:
a. Neumonía
b. síndrome de Mallory Weiss
c. esofagitis erosiva severa por reflujo
d. síndrome de Boerhaave

10. Dentro de las patologías que producen sangrado en el esófago, la que sangra más es:
a. el divertículo de Zenker
b. la esofagitis por reflujo gastroesofágico
c. el síndrome de Mallory Weiss
d. el síndrome de Boerhaave

Calificación Individual 04 - 202002


1. En un paciente con infección por SARS-CoV-2 con compromiso severo e ingresado en la
unidad de cuidados intensivos (UCI), debido a la tormenta de citoquinas y a la desregulación
inmune ¿Cuál de las siguientes alternativas estará elevada en sangre al evaluar el perfil hepático?
a. Transaminasas
b. Bilirrubinas
c. Amilasas
d. Fosfatasas

2. En el hígado, el aumento de la resistencia vascular en los sinusoides hepáticos ocasionará:


a. aumento del flujo hacia la vena cava superior
b. aumento del flujo hacia la vena porta
c. salida de plasma hacia el intersticio
d. aumento de la presión de llenado vesicular

3. En un paciente con intoxicación por órganos fosforados, la acción de la colecistoquinina (CCK)


está bloqueada a nivel de:
a. la célula parietal
b. el esfínter de Oddi
c. la vesícula biliar
d. el sistema nervioso central

4. ¿Cuál de las siguientes alternativas es correcta sobre la estructura hepática?


a. El flujo sinusoidal en el lobulillo hepático es de adentro hacia afuera
b. En la triada portal, se encuentra la vena derivada de la suprahepática
c. El flujo biliar en el lobulillo hepático es centrífugo
d. Los colangiocitos producen bilis
5. Paciente con Lupus Eritematosos que desarrolla hipertensión portal debido a trombosis portal,
es probable que desarrolle várices a nivel de:
a. recto inferior
b. recto superior
c. hemorroides externas
d. canal anal

6. La bilis que sale de la vesícula biliar tiene como componente principal:


a. a la bilirrubina
b. a los fosfolípidos
c. a los ácidos biliares
d. al colesterol

7. Marque lo correcto respecto al acino hepático:


a. Dos vértices del rombo acinar se constituyen por triadas portales
b. La zona 3 se afecta en menos en una deshidratación severa
c. El eje menor lo constituye un eje imaginario entre dos venas centrolobulillares
d. La zona 1 es la más que recibe más sangre

8. En un paciente con cirrosis hepática la cabeza de medusa que aparece en la pared abdominal,
podría desaparecer si al paciente se le:
a. administra antiandrógenos
b. oblitera el ligamento redondo
c. esclerosa las venas hemorroides internas
d. oblitera la arteria gástrica izquierda

9. Con respecto a la microestructura del hígado ¿Cuál de las siguientes alternativas es correcta?
a. La célula de Kupffer se encuentra fuera del sinusoide y fagocita células
b. El sinusoide es un capilar fenestrado
c. La célula de Ito se encuentra en el espacio de Disse y reserva glucógeno
d. Los hepatocitos están interconectados por uniones herméticas

10. En un paciente con cirrosis hepática se desarrollará ascitis debido a los siguientes
mecanismos, EXCEPTO:
a. Aumento de reabsorción renal de sodio
b. Estrechamiento de la porta
c. Hipoalbuminemia
d. Hiperflujo portal

1. Al examinar a un paciente, usted encuentra dolor localizado en fosa iliaca derecha y diagnostica
apendicitis. En este paciente, usted puede inferir:
El peritoneo parietal regional está afectado.

2. La motilidad intestinal es estimulada principalmente por el:


Plexo de Auerbach

3. Durante el vómito, ¿el contenido gástrico tiene que pasar necesariamente por cuál estructura para
llegar al esófago? Marque la mejor respuesta:
Cardias

4. Respecto a la anatomía del estómago, marque lo correcto:


El fondo gástrico forma la curvatura mayor
5. Marque la respuesta incorrecta:
En todo el tubo digestivo, se observa dos capas de muscular propia: circular interna y longitudinal
externa

6. Paciente se queja de dolor en hipocondrio derecho, pero superficialmente. El dermatoma relacionado es


(marque la mejor respuesta):
T9

7. Dentro de las funciones del abdomen, se encuentra la defecación y micción, en las cuales la presión intra
abdominal debe:
Aumentar

8. Un alumno de medicina decide hacerse un piercing en el ombligo. Al realizarle el procedimiento,


sangra ligeramente. Esta sangre proviene de la arteria (marque la mejor respuesta) Epigástrica inferior

9. Señale la respuesta correcta:


El apéndice cecal sólo tiene serosa

10. Paciente mujer es traída a emergencia por sufrir una herida contuso penetrante por cuchillo realizada
por su esposo en un ataque de celos. Se observa herida en flanco izquierdo. Esta solución de continuidad
ha comprometido varios músculos de la pared abdominal, excepto: Recto abdominal

11. Paciente con herida por proyectil por arma de fuego, con herida de ingreso en región paraumbilical.
Entre las estructuras que usted está seguro que debe haberse lesionado es:
Omento mayor

12. En la evaluación de una tomografía abdominal, el interno observa un aneurisma en una arteria que se
dirige al riñón derecho. Con seguridad se puede afirmar que está a nivel de la vértebra: L1

13. Paciente joven es traído a emergencia con abdomen agudo quirúrgico debido a herida contuso
penetrante por verduguillo (alambre grueso con punta aguzada) recibida en una pelea después de un
partido de fútbol. Se observa herida en Hipocondrio Izquierdo. El órgano que debe estar sangrando y
produciendo hemoperitoneo es (marque la mejor respuesta):
Bazo

14. Marque el órgano que se encuentra más distal en el tubo digestivo.


Ciego

15. La peristalsis o peristaltismo hace referencia a:


Motilidad para movilizar el alimento de proximal a distal.

16. Marque la respuesta incorrecta:


En todo el tubo digestivo, se observa dos capas de muscular propia: circular interna y longitudinal
externa

17. Paciente con vólvulo del colon sigmoides. La necrosis de ese segmento del colon se produce por una
alteración en la irrigación de la arteria: Mesentérica inferior

18. Marque el órgano que se considera retroperitoneal: Parte de la vía biliar

19. Un alumno de medicina decide hacerse un piercing en el ombligo. Al realizarle el procedimiento, sangra
ligeramente. Esta sangre proviene de la arteria (marque la mejor respuesta): Epigástrica inferior

20. Paciente tiene una úlcera sangrante en el segundo tercio del Yeyuno. La arteria de la cual proviene la
sangre arterial para dicha zona es la arteria: Mesentérica superior

21. Es inervado por aferentes somáticas: Peritoneo parietal


22. Paciente de 24 años con dolor abdominal tipo cólico intenso en mesogastrio. Según sus
conocimientos de macroestructura, el origen del dolor puede ser ….: Íleon

23. Paciente con hipoglucemia secundaria a un insulinoma (tumor neuroendócrino productor de


insulina). El órgano donde mayor probabilidad ha crecido este tumor es: retroperitoneal
24. Al evaluar una tomografía abdominal, el médico asistente le pide al interno de la UPC que encuentre la
imagen con el corte a nivel de L1. El interno sabiamente busca el …….. para ubicar la vértebra L1: Cuello del
páncreas

25. En la inspiración, la pared abdominal debe …. para ….. : relajarse disminuir presión intra torácica

26. Ligamento hepatogástrico une el ….. con el …… y forma la entrada al …… : Hígado Estómago Orificio
omental

27. Al retirar completamente el mesenterio de un órgano, el mismo se vería afectado principalmente en su:
Irrigación

28. La estructura que fija órganos principalmente a la pared posterior abdominal se denomina:
Mesenterio

29. Cuál de las siguientes estructuras no tiene vasos sanguíneos: Epitelio intestinal

30. Al iniciar la digestión, aumenta el consumo de oxígeno por la mucosa. Esto conlleva a una hipoxia
local, lo cual hace que se libere …., el cual produce vasodilatación: adenosina

31. Sustancia que inhibe la secreción y la motilidad del estómago prolongando el tiempo de digestión:
péptido insulinotrópico dependiente de la glucosa (GIP)

32. Marque lo correcto: La hernia fisiológica se produce en la sexta semana y es la salida temporal de asas
intestinales a través del colon umbilical

33. Marque la respuesta correcta en relación a la gastrina: Las células G son las productoras y se
encuentran principalmente en el antro gástrico.

34. El consumir caramelos indirectamente activa la vía: POMC/CART

35. ¿En qué capa se encuentra la alteración principal en el Hirschsprung o megacolon agangliónico?:
Muscular propia

36. Con respecto a las ondas lentas, marque la afirmación correcta: Son contracciones rítmicas
espontáneas

37. El uso de Ranitidina bloquea el receptor H2 de la histamina en las células parietales. La histamina llega
a estas células por: Difusión

38. La triada sintomática: vómitos explosivos post-prandiales, movimientos peristálticos epigástricos


visibles de izquierda a derecha y nódulos palpable epigástrico subcostal derecho, pertenecen a: Estenosis
congénita hipertrófica del píloro

39. Durante una cirugía oncológica, ¿la extirpación de cuál de los siguientes órganos se vería
comprometida por la presencia de adventicia?: Recto

40. En cuanto a los reflejos gastrointestinales, un reflejo que estimula el tránsito intestinal es el reflejo:
Gastrocólico

41. El ligamento falciforme divide al hígado en dos lóbulos derecho e izquierdo. Embriológicamente
deriva del: Mesenterio ventral
42. La presencia de atresias y estenosis duodenales se deben básicamente a una: Falta de recanalización
43. Estudiante de medicina de 20 años, se ha amanecido estudiando para su examen de Sistema Digestivo.
No ha probado alimento desde la cena, por lo que se puede afirmar que la motilidad de esta persona está
siendo regulada por: Motilina

44. Paciente con disminución del apetito marcada asociada a cáncer terminal, para promover la ingesta de
alimentos se podría usar análogos de: Endorfinas

45. Las ondas lentas se producen por la apertura cíclica de canales de: Calcio

46. La forma más común de atresia esofágica contiene: Estenosis proximal del esófago más fístula
traqueoesofágica distal

47. Al deglutir un bolo alimenticio, es lógico suponer que al pasar por el esofago haya un mayor
consumo de oxigeno en la pared del tercio: Proximal

48. Paciente que come entera una pizza familiar de chorizo y queso. Es posible esperar que debido a la
cantidad de alimento ingerida, las ondas lentas hayan: sufrido ninguna alteración en su frecuencia.

49. La hernia fisiológica se produce dentro de: cordón umbilical

50. El crecimiento de un adenocarcinoma de páncreas compromete la pared gástrica por contigüidad


¿Que parte del estómago se esperaría esté comprometido?: Pared posterior del antro

51. Estimula la producción de saliva: Vasodilatación periglandular

52. Durante la secreción de saliva, es de esperarse que las concentraciones de ….. y …… disminuyan al
disminuir el flujo: Sodio Bicarbonato

53. Con respecto a la secreción gástrica de HCI: a mayor secreción de HCI en el lumen gástrico, mayor pH en la
sangre venosa gástrica.

54. Respecto a las enfermedades del esófago, marque lo correcto: el diagnóstico diferencial de la
acalasia es la enfermedad de Chagas esofágica

55. Con respecto a las lesiones y enfermedades de la boca, marque lo correcto: la eritroplasia debe ser
biopsiada
56. Respecto a las glándulas salivales, marque lo incorrecto:
a. la glándula sublingual tiene forma de garfio
b. la glándula sublingual drena a través del conducto de wharton
c. la glándula parótida produce secreción serosa
d. la glandula parotida drena a través del conducto de Stenon

57. El omeprazol actúa sobre la membrana ….. de la célula ….. : apical/parietal

58. Durante el sueño, la concentración de bicarbonato en la saliva: Disminuye

59. Durante el ataque con gas sarín (bloqueador de la acetilcolinesterasa) en el metro de Tokio, en 1995, el
personal de salud noto que los pacientes afectados presentaban: Hipersalivación

60. La célula mucosa del cuello gástrico produce: Moco

Parcial 2020-01 (1)


1. Al evaluar la orofaringe de un paciente, el médico le solicita que abra la boca, saque la lengua y diga a. Al
hacer esta maniobra, nota que el paladar se desvía hacia la derecha, lo cual le hace sospechar que el paciente
sufre de una lesión del nervio craneal: X contralateral
2. Un bolo alimenticio grande y poco masticado se atasca en el esofago, esto ocasiona una sensación de
dolor que es transmitida por los nervios: esplácnicos

3. Para realizar el movimiento mecánico de abrir la boca, primero se necesita: fijar el hueso hioides

4. ¿Cual de las siguientes alternativas se define como la protrusión directa del contenido abdominal a la
cavidad amniótica por un defecto de la pared corporal?: Gastrosquisis

5. Un paciente requiere que se le coloque una sonda de alimentación directamente al estómago


(gastrostomía), el cirujano deberá hacer una incisión en la piel del abdomen ¿cuál de las siguientes
raíces nerviosas debe ser anestesiada para este procedimiento? T8

6. En un paciente de 43 años con tumor carcinoide de páncreas productor de gastrina (Síndrome de


Zollinger-Ellison) se puede esperar encontrar una potenciación del reflejo: gastrocólico

7. El mecanismo de la defecación incluye la participación de diversas estructuras ¿cuál de las siguientes


alternativas es correcta?: Puede ser mediado por un reflejo intrínseco

8. Cuando el contenido del estómago ingresa al duodeno, uno de los reflejos que inhiben el vaciamiento
gástrico es a través del: Sistema nervioso mientérico

9. Durante la masticación, gran parte del proceso masticatorio se debe a: El reflejo masticatorio

10. Las glándulas salivales tienen conductos para la excreción de la saliva, las glándulas …… drenan en las
carúnculas sublinguales. RPTA: Sublinguales

11. Los diferentes segmentos del tubo digestivo son susceptibles de reflejos y movimientos según su
contenido. Si colocamos mediante una sonda un bolo alimenticio directamente en el tercio medio del
esofago: se producira ondas secundarias

12. En una persona si enfermedad se espera que el tránsito intestinal se vea disminuido cuando se
presenta el reflejo: Doloroso

13. El divertículo de Meckel es una anomalía congénita que ocurre por la persistencia del conducto
vitelino y da origen a una estructura sacular, el cual se encuentra en el: borde antimesentérico
14. Si al intubar a un paciente, por error se ingresa el tubo endotraqueal en el esofago y se insufla el
manguito endotraqueal (globo TET), la dilatación de este manguito generará: múltiples ondas
secundarias

15. El orificio omental, o hiato de Winslow, se encuentra limitado por el ligamento: hepatoduodenal

16. Paciente de 24 años acude a consulta externa por presentar una fístula oronasal (comunicación entre la
cavidad oral y la cavidad nasal). Esta fístula es una consecuencia tardía de la lesión de un vaso sanguíneo por
el antecedente de haber sido operado de paladar hendido en los primeros años de vida, aparentemente en
una campaña gratuita de corrección de paladar fisurado. ¿Cual de las arterias palatinas podría haberse
lesionado durante esa cirugía?: Mayor

17. Dentro de las anomalías congénitas se puede presentar un tejido pancreático accesorio ¿cuál es la
ubicación más común de este tejido?: Estómago

18. Una recién nacida es evaluada por el neonatólogo y evidencia que el canal anal está completamente
cerrado. Este problema se debe probablemente a una anomalía en el desarrollo de: la membrana cloacal

19. En la digestión de los alimentos, la hormona __________ se libera frente a la presencia de


péptidos y monoglicéridos, y tiene un efecto marcado en la disminución del vaciamiento gástrico →
colecistoquinina
20. Dentro de las anomalías congénitas se puede presentar un tejido pancreático accesorio ¿ Cuál
es la ubicación más común de este tejido? → Estómago

21. Los catadores de vino tienen una habilidad increíble al momento de separar los sabores. Este aumento
de la sensibilidad gustativa debido a una mayor cantidad de papilas linguales y de corpúsculos gustativos se
conoce como: hipergeusia

22. En muchos países se usa el suplemento de flúor en el agua potable o los dentífricos, con el fin de
hacer el esmalte más resistente a la desmineralización inducida directamente por: el ácido

23. Durante el desarrollo de la región cloacal, una cuña de mesodermo ubicado entre el alantoides y el
intestino posterior vendrá a formar el: tabique urorrectal

24. En una persona sana, el momento adecuado para encontrar los mayores niveles de grelina en sangre
sería: antes de comer

25. El duodeno está constituido por el segmento terminal del intestino anterior y el segmento proximal del
intestino medio ¿Cuál de las siguientes alternativas describe mejor este lugar de unión entre los dos
intestinos?

Distal al origen de la yema hepática


26. En una persona sana, el uso de atropina producirá a nivel del estómago: Aumentará el pH del
estómago

27. En una persona sana, el consumo de leche produce indirectamente → Inhibición del vaciamiento
gástrico
28. La motilidad del colon es importante y lenta comparada con la del intestino delgado. Los
movimientos en masa ocasionan la: distensión rectal
29. Con respecto a la motilidad gástrica, los potenciales de acción disminuyen en frecuencia por efecto de:
el péptido insulinotrópico dependiente de glucosa

30. La sensación del gusto depende de la presencia de papilas gustativas en la lengua, de las cuales, algunas
de ellas tienen un surco terminal por donde drenan unas glándulas salivales linguales (llamadas glándulas de
von Ebner). Esta descripción se refiere a las papilas: circunvaladas

31. Dentro de la estructura de los dientes, la parte del diente cubierta por esmalte y que se puede
ver mediante la inspección visual de la boca se denomina → corona clínica

32. En el conducto anal se encuentra la unión entre las regiones del endodermo y el ectodermo, esta
unión se evidencia al observar: la línea pectínea

33. La sensación del gusto depende de la presencia de papilas gustativas en la lengua, las cuales tienen
corpúsculos gustativos conteniendo células neuroepiteliales sensoriales. Estas células neuroepiteliales
pueden ser dañadas fácilmente, por suerte, su tiempo de recambio es de alrededor de: 10 días

34. En un varón de 47 años con sección medular a nivel de T6 debido a un accidente automovilístico, sus
terapeutas han desarrollado un mecanismo para distender el recto e iniciar el reflejo rectoesfinteriano, lo
cual producirá la contracción de: la pared del recto

35. Durante un experimento, se insufla rápidamente dos litros de agua en un globo colocado
dentro del estómago de un voluntario. ¿cuál de las siguientes situaciones del
músculo liso será consecuencia directa de este cambio de volumen en el estómago? →
Despolarización
36. Paciente de 56 años con accidente cerebrovascular reciente. En la resonancia se observa daño de los
núcleos laterales del hipotálamo. Por este motivo es muy probable que el paciente sufra de: inanición

37. Paciente con insuficiencia mitral moderada a severa, con aumento de volumen de la aurícula
izquierda, esta condición tendrá como consecuencia a nivel del sistema digestivo: la disfagia a sólidos

38. El inicio de la fase faríngea de la deglución se debe a estímulos sensitivos que viajan por el nervio
craneal: V

39. El mesocolon transverso se origina en: la pared posterior del abdomen

40. El esofago en su microestructura tiene basicamente adventicia, a excepción de la región distal,


donde tiene serosa, específicamente a partir del nivel de: T10

41. En un paciente con arcadas, se debe considerar que durante la ocurrencia de dichas arcadas,
debemos encontrar contenido gástrico en: Tórax

42. La reabsorción de Sodio y Cloro en las glándulas salivales se da principalmente en el: Conducto
estriado

43. Para que se puedan digerir las grasas, es preferible que primero sean emulsificadas. La hormona que
estimula la liberación de las sustancias emulsificadoras es: CCK
44. Paciente de 13 días de vida con vómitos explosivos a las 2 horas después de lactar. Al examen físico se
palpa la oliva pilórica ¿cual es el nervio cuyos filetes dan inervación eferente a la estructura afectada?: Vago

45. La localización de la vesícula biliar con respecto al lóbulo cuadrado es: Lateral

46. La triada portal está constituida por el conducto biliar y la arteria hepática y una pequeña rama de la
vena: Porta

47. Al ingerir una cantidad de glucosa por vía oral, esta es interiorizada en las células del organismo más rápido
que si esa misma cantidad hubiese sido administrada por vía endovenosa. Este fenómeno sucede gracias a una
sustancia secretada por las células: K

48. Paciente de 62 años con vólvulo de intestino delgado e isquemia intestinal. ¿Qué estructura se utiliza
como punto de referencia para determinar la posición de la unión duodenoyeyunal? : Ligamento
suspensorio del duodeno (de Treitz)

49. En un paciente con Zollinger Ellison, usted esperaría encontrar: Esteatorrea 50.

La motilidad intestinal es estimulada por: Colecistoquinina y gastrina

51. La onda peristática secundaria del esofago en la deglución, es producida por: Plexo mientérico
esofágico

52. Paciente con parálisis bilateral del nervio hipogloso, el unico musculo de la lengua que conservará su
movimiento es el: palatogloso

53. ¿Cual de las siguientes sustancias tiene mayor concentración de la saliva comparado con su
concentración plasmática? Potasio

54. ¿Cual de las siguientes estimula la secreción enzimática exocrina del páncreas?: Colecistoquinina

55. Al comer un pollo a la brasa, con papas fritas y ensalada, la sustancia que estimula la liberación de HCI
en el estómago es: Bombesina

56. Marque lo correcto con respecto a Esófago de Barret: Se relaciona con reflujo gastroesofágico
ECU 1
Estudiante de medicina de la UPC de 21 años sufre de gastritis aguda ocasionada por comer en lugares poco
higiénicos. Suele consumir caramelos ( chupar ) mientras está en clase hasta la tarde. Toma gaseosas
regularmente (carbohidratos 46%, sodio 53%). También toma regular cantidad de leche (grasa 35%, lactosa
35%, proteínas 30%), pues le calma un poco el dolor el ardor que siente por la gastritis. Incluso, cuando
puede, se toma dos vasos de agua fría para calmar las molestias. Ha decidido ir al médico para tratarse pues
ya no soporta el dolor, el cual está seguro que los síntomas se deben a una elevada producción de ácido
clorhídrico en el estómago, y por ello le ha recetado Ranitidina (antihistamínico), con lo que siente mejoría.
El uso de atropina en este paciente:
- Aumentará el pH del estómago
Entre las sustancias cerebrales que producen ansiedad está la serotonina, la cual también tiene acción: -
Anorexigénica
El consumo de dos vasos de agua seguidos agua generará indirectamente un aumento en la liberación de:
- Ácido clorhídrico
En este paciente con gastritis aguda debida a una alta producción de ácido clorhídrico, sería lógico
esperar que el píloro tenga un tono muscular:
- Aumentado
El consumo de leche produce directamente un aumento de los niveles séricos de la hormona: -
Colecistoquinina (CCK)
El consumo de leche produce directamente un aumento de los niveles séricos de la hormona: -
Gastrina

ECU 2
Niño de sexo masculino de 2 años de edad, sufre de estreñimiento desde el nacimiento (1 deposición cada 3-
4 días). Madre menciona que le estimula la defecación con un termómetro rectal, y continuo uso de enemas
y laxantes. Desde hace 6 meses comienza con vómitos postprandiales. Los síntomas aumentan en frecuencia
y magnitud y están en relación con los episodios de estreñimiento. No refiere fiebre, tos, diarrea ni lesiones
cutáneas. Al examen físico presenta regular estado general, luce deshidratado. Abdomen distendido, blando,
depresible e indoloro. No se palpan masas abdominales. Se permeabiliza el canal anal con termómetro rectal,
encontrando cierta resistencia. Salida de material fecal mal oliente en regular cantidad. Exámenes de
laboratorio: hemograma normal. Signos inflamatorios de fase aguda negativos. Alcalosis metabólica leve en
sangre venosa.
Radiografía con enema baritado muestra recto y colon sigmoides dilatados (megacolon). Biopsia
profunda: ausencia de células ganglionares en la muestra enviada. Se realiza cirugía correctiva. El
contenido fecal se detiene en la zona inmediatamente proximal a la zona donde hay una menor
presencia de:
- Péptido intestinal vasoactivo
En cuanto a los reflejos gastrocólico y gastroduodenal en este paciente, indique lo correcto: - Se
pueden considerar reflejos vago-vagales
En este paciente se considera que está abolido el reflejo:
- Rectoesfinteriano
Debido al acúmulo de material fecal en todo el marco colónico, y a la irritación química asociada, el
peristaltismo del íleon distal se debe encontrar:
- Inhibido
Es un reflejo propio de la pared intestinal:
- Peristaltismo
A diferencia de las arcadas, los vómitos presentan apertura de:
- Esfínter esofágico superior

1. Un niño de 2 años es llevado a la consulta por diarrea persistente y edema de las


extremidades, además falta de crecimiento y desarrollo en relación a su edad. Los análisis
de sangre revelan que tiene concentración plasmática baja de proteínas (hipoproteinemia).
Durante la endoscopía duodenal, se coloca colecistocinina (CCK) endovenosa y se recoge
muestras del líquido duodenal; el resultado del líquido confirma incapacidad para hidrolizar
proteínas a un pH neutro, esta situación mejora al añadir una pequeña cantidad de tripsina.
El paciente probablemente esté sufriendo la falta congénita de
-Enterocinasa

2. Experimentalmente se incrementa la velocidad de la secreción salival con una


sustancia, el análisis de la composición de esta saliva obtenida se espera
encontrar…………..

-Disminución de concentración de potasio

3. Paciente varón de 46 años soltero, consulta por odinofagia y bajo de peso, tiene
antecedente de tuberculosis desde hace 3 meses y es fumador crónico (10 cigarrillos por
día); al evaluar la cavidad oral se identifica lesión blanquecina en el dorso de la lengua y
paladar blando, las lesiones se desprenden con el baja lengua dejando una base
eritematosa. Esta lesión corresponde probablemente a

……………………….…..
-Candidiasis oral

4. Paciente mujer de 35 años acude a consulta por sensación de sequedad y lesiones en


cavidad oral. Al examen se observa atrofia de la mucosa, fisuras y úlceras; nota además
sequedad e irritación de la córnea y aumento del tamaño de las glándulas parotídeas. Su
diagnóstico más probable es artritis reumatoide; el hallazgo más probable en una biopsia
de glándula parótida es……..….

-Gran infiltración de linfocitos y células plasmáticas

5. Un paciente con anemia acude con su médico quejándose de episodios frecuentes de


gastroenteritis. Un análisis de sangre revela anticuerpos circulantes dirigidos contra células
parietales gástricas. Su anemia es atribuible a la hiposecreción de
-Factor intrínseco

6. Dos estudiantes deciden tomar un receso para comer una hamburguesa a la hora del
almuerzo. Antes de llegar a la cafetería, impulsos nerviosos provenientes del
complejo vagal dorsal iniciarán la secreción de ácido gástrico por la liberación de

…………………….. desde el sistema nervioso entérico.


-GRP

7. Un niño de cuatro años de edad es llevado a la consulta por cuadros diarreicos


frecuentes caracterizados por heces pálidas, voluminosas y fétidas, presenta bajo peso y
talla. Se mide la concentración de cloruro en el sudor y se encuentra que sus valores son
muy elevados. La alteración más importante a nivel de células ductales del páncreas tiene
relación directa con la conductancia de…………

-Cloro

8. Una mujer de 50 años de edad que sufrió durante varios años resequedad de los ojos
debida a producción inadecuada de lágrimas es enviada con un gastroenterólogo para
evaluación de pirosis crónica. El examen endoscópico revela erosiones y tejido cicatrizal
en la parte distal del esófago justo por arriba del esfínter esofágico inferior. Las lesiones
pueden atribuirse a la disminución de uno de los siguientes componentes salivales:

-Bicarbonato

9. Se evalúa los valores séricos de las siguientes sustancias a un paciente con


enfermedad hepática terminal; en este paciente se espera encontrar la combinación con
la letra …………
-disminuida, aumentada, disminuida

10. Una mujer de 35 años de edad HIV positiva, se presenta al médico con dolor
abdominal en cuadrante superior derecho e ictericia. La paciente refiere haber tenido
múltiples episodios de ictericia durante los últimos 10 años. Los exámenes para
determinar hepatitis viral, dieron positivos para Hepatitis B, siendo catalogado el caso
como hepatitis crónica con alteración funcional. En un examen de sangre ¿cuál de los
siguientes parámetros está disminuido?

-Albúmina

11. En el reflejo peristáltico del intestino delgado, uno de los siguientes eventos
sucede en la porción oral del bolo alimenticio…………...

-Acción de acetilcolina en el músculo circular

12. Experimentalmente se coloca una dosis alta de secretina en la luz intestinal


duodenal; como consecuencia de esto, en el jugo pancreático de la misma luz
intestinal se observa la disminución de la concentración de …..………..

-Cl

13. Un varón de 58 años de edad con enfermedad de Crohn severo fue sometido a una
resección ileal. Después de la cirugía este paciente padecerá de esteatorrea, esto se
explica porque …..………..

- La micelas no pueden formarse


14. En un experimento se inserta un balón en el estómago de un voluntario, se infla poco
a poco mientras que se vigilan las presiones intraluminales. Aunque el volumen del balón
aumenta considerablemente, las presiones permanecen constantes. Esta relación
volumen-presión se explica por la liberación local de …………..

-Óxido nítrico y péptido inhibidor vasoactivo

15. La toxina de Vibrio cholerae causa diarrea debido a…….

-El Incremento de la secreción de cloro por las células de la cripta intestinal

16. ¿Cuál de las siguientes alternativas es una característica de la secreción exocrina del
páncreas?

-Tiene una baja concentración de Cl- respecto al plasma

17. Una madre lleva a su hijo de dos años de edad a la sala de urgencias, estresada porque
el niño deglutió una moneda de 10 céntimos mientras la familia cenaba en un restaurante.
El médico observa mediante fluoroscopía que la moneda se halla en el
estómago y asegura a la madre que la moneda se eliminará con las heces. El médico
recomienda utilizar la respuesta fisiológica que permitirá la evacuación de la moneda del
estómago al intestino ………….…..

-Son los movimientos de mezcla y trituración


-. Es provocada por el ayuno

18. Las estructuras en el hígado que permite que los productos metabólicos unidos a
proteínas tengan acceso a las membranas basolaterales de los hepatocitos, son….. - Las
fenestras sinusoidales
19. La composición de la bilis es modificada conforme fluye por los conductillos
biliares. Durante este tránsito se espera que aumente la concentración de…….

-Monómeros de ácido biliar


-Ig A

20. Se mide experimentalmente el contenido gástrico de dos personas. La persona “A”


tiene alto contenido de grasa y la persona “B” tiene un contenido hipertónico ¿Cuál de las
siguientes es correcto respecto al vaciamiento gástrico?
- Hay ralentización del vaciado gástrico en ambos casos

21. El examen endoscópico de un paciente con hipertensión portal grave revela venas
tortuosas que sobresalen hacia la luz del esófago. El paciente recibe tratamiento quirúrgico
mediante la colocación de una derivación que conecta la vena porta a la vena cava.
Después de la operación el riesgo de encefalopatía y el

riesgo de sangrado de várices ……………..


-Aumentará/disminuirá

22. Un paciente varón de 18 años de edad acude al médico para sus exámenes de rutina.
Sus resultados de laboratorio muestran un valor de bilirrubina sérica de 4 mg/dl y una
bilirrubina directa de 0,3 mg/dl. Las pruebas de función hepática son normales. La
alteración que explica mejor este caso es por la deficiencia de

………………..
-Glucuronil transferasa

23. Un hombre de 57 años de edad es llevado a urgencias con hematemesis masiva rojo
brillante, a su llegada se halla inconciente con PA: 80/40 mm Hg y FC: 124 lat/min. Luce
ictérico con presencia de “arañas vasculares en el tórax anterior y extremidades”, abdomen
distendido con signo de oleada positiva. Se encuentra esplenomegalia y pérdida de la
masa muscular en extremidades. La anastomosis vascular responsable del sangrado en
este paciente es

-Vena gástrica izquierda y vena ácigos

24. Un estudiante de medicina está comiendo un plato de comida a base de


champiñones, espárrago y salsa de soya. El sabor umami contenido en todos estos
alimentos actúa a nivel de los botones gustativos estimulando ………………..

-Un receptor acoplado a proteína G


25. Un hombre de 22 años de edad se presenta al médico con una historia de 1 año de
evolución caracterizado por dolor recurrente en fosa iliaca derecha y diarrea. Manifiesta
además pérdida de peso de 8 kg durante este periodo. La colonoscopía revela múltiples
lesiones en el ileon terminal y colon. La biopsia de estas lesiones revela engrosamiento,
inflamación y ulceración de la mucosa. El diagnóstico más probable en este caso
es…….

-Enfermedad de Crohn

26. Varón de 61 años que consulta por dolor retro esternal intenso desde hace 6 horas y
después de vómitos intensos y repetidos; al examen se observa disnea, cianosis,
hipotensión y signos clínicos de shock. La radiografía simple de tórax muestra
neumomediastino. El líquido en el espacio pleural aspirado tiene alta concentración de
amilasa. ¿Cuál de las siguientes alternativas puede explicar este cuadro clínico? –

Rotura espontánea de esófago


27. La secreción del ácido en la célula parietal gástrica se lleva a cabo por una
ATPasa especifica que intercambia hidrogeniones (H+) del citosol por…..
-K +

28. En condiciones normales el ingreso de 600 ml de líquido es el estómago provoca un


aumento de presión intragástrica de unos 12 cm de H2O. Después de una vagotomía
(corte del nervio vago) es de esperar que el ingreso del mismo volumen de líquido
provoque lo siguiente: …………………………………

-Un aumento mayor de la presión

29. Una paciente de 30 años de edad es sometida a una cirugía en oído medio derecho
por un problema de otoesclerosis. Luego de la cirugía refiere alteración en la percepción
de sabores. Al evaluar el caso usted esperaría encontrar……….

-Alteración en la sensación del gusto en los dos tercios anteriores de la


lengua
-Sensación del dolor, tacto y temperatura conservada en toda la lengua 30.

¿Cuál de las siguientes alternativas es correcta?

-Las sales biliares desconjugadas son absorbidas preferentemente en el colon

31. En un paciente de 45 años de edad con colestasis biliar, se encuentra una elevación
de los niveles sanguíneos de fosfatasa alcalina hasta 3 veces la cifra normal. ¿Cuál de
las siguientes alternativas estará también elevada como evidencia del daño de la vía
biliar?

-Gamma glutamil transpeptidasa

32. Revisando la angiografía de un hombre de 70 años en estudio por aneurisma de aorta


abdominal el radiólogo informa de la presencia de una oclusión completa de la arteria
mesentérica inferior. El paciente se encuentra completamente asintomático.

¿Cuál de las siguientes arterias se anastomosa a la sistema arterial de la mesentérica


inferior?
-Cólica media

33. Lactante de 3 meses de vida es atendido por presentar diarrea, se administra una
solución de glucosa y electrólitos por vía oral. La proteína de membrana apical que
explica la capacidad de esta solución para proporcionar aporte de glucosa e hidratación
es ………..

-SGLT-1

34. Paciente ha sufrido herida de bala en el abdomen, se le ha tenido que extirpar el


segmento medio y distal del ileon. En este caso la síntesis hepática de sales biliares estará
…..…..

-Incrementada por estímulo de la enzima colesterol 7 alfa hidroxilasa

35. Un varón de 75 años ingresa al consultorio por presentar ictericia marcada de piel y las
escleras. El estudio del paciente mostró que presentaba un tumor que obstruía la totalidad
del conducto hepático común. ¿Cuál de las siguientes estructuras se encontrará dilatada
en este paciente?
-Conductos de Hering
36. En un paciente con insuficiencia renal crónica, el déficit en la absorción de calcio a
nivel del enterocito se debe a lo siguiente:

-No se convierte la 25 hidroxicolecalciferol a 1,25 dihidroxicolecalciferol

37. Varón de 30 años es traído a emergencia por agresión abdominal con arma de fuego
(pistola) y es sometido a laparotomía exploratoria, observándose isquemia del colon
ascendente y parte del colon trasverso ¿la lesión de cuál de las siguientes arterias explicaría
esta isquemia?

d. Mesentérica superior

38. Respecto a las sustancias gastrointestinales que regulan la secreción pancreática;


marque la afirmación correcta:

b. La acetilcolina es capaz de estimular la secreción enzimática y de bicarbonato del


páncreas
39. Ante una lesión del X par craneal, ¿cuál de los siguientes músculos mantiene
conservada su función?:

b. Tensor del velo del paladar

40. Experimentalmente se utiliza atropina (anticolinérgico) para inhibir la secreción de gastrina,


sin embargo la secreción de esta hormona se sigue dando ante estímulos vagales. Esta
situación se explica porque la atropina:

d. No bloquea la acción del péptido GRP

41. Un varón de 50 años es sometido a extirpación del duodeno y parte proximal del
yeyuno. Esta situación ocasionaría la pérdida de las células ……….. , productoras de
………………… que estimula la secreción de bicarbonato por el páncreas.

“S” / secretina

42. Se evalúa la expresión de la proteína Agrp en una persona con alteración del apetito; lo
correcto respecto a esta proteína es…..

La mutación del gen que la codifica produce adelgazamiento

43. Juana cae de la bicicleta y se fractura la región anterior del hueso maxilar superior con
compromiso de la fosa incisiva. Al examen físico de la región esperaría encontrar alteración en
la sensibilidad de la encía …………………
palatina anterior

44. Recién nacido es atendido por el neonatólogo y luego entregado a su madre para dar de
lactar; la madre al dar de lactar observa coloración azulada de labios, acompañado de tos
persistente, dificultad respiratoria y distención abdominal. Se le intenta colocar una sonda
nasogástrica pero esta retorna a la cavidad oral en todos los intentos. ¿Cuál de las siguientes
anomalías del desarrollo es el más probable en este caso?

b. Atresia esofágica proximal con fístula traqueoesofágica distal l


45. ¿Cuál de los siguientes mecanismos ocurre durante la defecación? En

la posición de “cuclillas” el músculo puborrectal se halla relajado

46. Un paciente luego de un accidente sufre lesión del piso de la boca, se constata daño del
nervio “cuerda del tímpano”, en este caso se esperaría en

47. contrar disminución de la………………………….… de la lengua

Sensación del gusto en los dos tercios anteriores

48. ¿Cuál de las siguientes afirmaciones es la correcta sobre la gastrina?


Actúa en la célula diana mediante su receptor CCk tipo B

49. Al recibir un paciente con signos de hipovolemia y antecedente de trauma en abdomen por
accidente de tránsito, usted identifica radiológicamente: lesión de primera vértebra lumbar y
signos de lesión en páncreas; durante la cirugía se observó pobre irrigación de asas intestinales.
El vaso afectado es la arteria ……..

c. mesentérica inferior

50. Un paciente sufre de daño a nivel del cuello con lesión muscular en la región de la faringe.
En el examen físico se determina dificultad para la elevación de la faringe y para el cierre del
itsmo de las fauces. En este caso, probablemente esté afectado el músculo:

c. palatofaringeo

51. Varón de 50 años a quien le realizan la curación de la segunda molar de la arcada superior
derecha. En un momento determinado, el paciente acusa de intenso dolor de la pieza dentaria
en tratamiento. La vía aferente del dolor viaja a través del nervio …………
a. trigémino V2

52. La distención gástrica por los alimentos produce incremento de secreción de HCl
mediante la producción de ………….. que estimula a las célulasvía proteína

………..
a. gastrina / parietal / Gq

53. Un niño de tres años llega a emergencia con disfagia (dificultad para tragar), dolor retro
esternal, salivación y llanto. Se sospecha de ingesta de cuerpo extraño (moneda) en el
esófago; al ser evaluado se constata en una radiografía presencia de cuerpo extraño a nivel
de C6 (6° vértebra cervical). El cuerpo extraño estará suspendido a nivel del estrechamiento
producido por………..

c. el músculo cricofaríngeo

54. La triada portal (arteria hepática, vena portal y conducto biliar común) está contenida en
el ligamento …….……… y derivan embriológicamente del ……

a. hepato duodenal / mesenterio ventral


55. Un paciente refiere no percibir algunos sabores, al examen físico se constata alteración en
la percepción de sabores y del dolor en el tercio posterior de la lengua ¿Cuál de los siguientes
nervios estará alterada en su función?

c. Glosofaríngeo (IX par)

56. En el caso de un paciente con gastrinoma (tumor productor de gastrina), la presencia de


úlceras duodenales y erosión de la mucosa gástrica, se debe principalmente a……. c. el
exceso de HCl por estímulo de receptores CCK-B en la célula parietal

56. El reflejo entero gástrico se caracterizan por:


d. originarse debido a la distensión duodenal y presencia del quimo ácido

57. Mauricio tiene dificultad para deprimir el paladar y elevar la parte posterior de la
lengua. En este caso estará afectado un músculo, específicamente el músculo
…………….
b. extrínseco – palatogloso

58. En condiciones normales, el ingreso de 600 ml de líquido es el estómago provoca un


aumento de presión intragástrica de unos 12 cm de H2O. Después de una vagotomía (corte
del nervio vago) es de esperar que el ingreso del mismo volumen de líquido ocasione
………………………………… de la presión intragástrica.
c. un aumento mayor

59. La explicación fisiológica de presentar somnolencia de 30 minutos a 1 hora después de


ingerir alimentos, se explica por: a. Aumento del cloro intraluminal
e. Aumento de la alcalinidad sanguínea
60.Se presenta un paciente, el cual presenta un antecedente de tuberculosis intestinal, por lo
cual, se le resecó 80 cm de íleon distal. Desde el punto de vista fisiológico, el paciente puede
presentar una de las siguientes alteraciones: a. Disminución de la secreción de Vitamina B12
e. Disminución de la absorción de ácido glicocólico

61. Un paciente es sometido experimentalmente a un fármaco que modifica el flujo salival,


obteniéndose un volumen de saliva de 288 ml en 6 horas. En este caso las concentraciones de
electrolitos y bicarbonato en la saliva obtenida varían de la siguiente manera: a. ↑ Na+,
↓ K+, ↑ Cl-, ↑ HCO3-
1. b. ↓ Na+, ↓ Cl-, ↑ K+, ↓ HCO3-

62. Uno de los siguientes elementos debería hallarse con más probabilidad en el esófago de
un paciente que sufre de reflujo gastro esofágico…
a. Pepsina

63. Un paciente de 40 años cursa con anemia de 8g/dl, aqueja además de astenia y sensación
de hormigueo bilateral en los miembros inferiores, al examen se halla alteración de la
sensibilidad a la vibración y camina con ampliación de la base de sustentación. Uno de los
siguientes procedimientos sería de ayuda para el diagnóstico de este paciente: a. Tomografía
cerebral
b. Biopsia de la mucosa gástrica

64.Paciente de 60 años ingresa por caída hace 1 hora y pequeño hematoma en cuero
cabelludo, al examen físico ampliado se observa ictericia de piel y mucosas generalizada,
abdomen blando, se palpa estructura quística no dolorosa en hipocondrio derecho que
corresponde a vesícula biliar (signo de Courvoisier), en los exámenes de laboratorio se halla
niveles bajos en la formación de estercobilinógeno y urobilinógeno en heces, incremento de
la bilirrubina conjugada en la orina, elevación de fosfatasa alcalina y gamma glutamil
transpeptidasa séricas. El presente cuadro puede ser explicado por: a. Reabsorción de
hematoma
c.Carcinoma de la cabeza de páncreas

65. Un recién nacido presenta vómitos biliosos poco tiempo después de cada alimento. Al
preguntar a la madre sobre antecedentes, ella recuerda que tuvo polihidramnios durante la
gestación, pero un análisis de cariotipo fue normal. Una de las siguientes es la causa más
probable de estos hallazgos en el recién nacido: a. Enfermedad de Hirschprung e. Malrotación
de la yema pancreática ventral

66.En un estudio de la secreción de hormonas gastrointestinales, sus concentraciones en la


vena porta se midieron durante perfusión luminal del intestino delgado con soluciones de
diversas magnitudes de pH. ¿Qué hormona aumentará en el plasma de la vena porta durante
perfusión a través del intestino con una solución de pH 3?
a. CCK
e. secretina

67.Paciente de 30 años que ingresa a causa de un traumatismo abdominal cerrado. En la


exploración se aprecia discreta palidez de piel y mucosas, auscultación pulmonar normal,
taquicardia de 120 /min. Discreta distensión abdominal y matidez en flancos; el hematocrito,
que era prácticamente normal al ingreso, disminuye a 30% a las tres horas. En la Rx de tórax
se objetiva fractura de las costillas 10-11 izquierdas. La causa más probable de la anemización
en este paciente es: a. traumatismo renal con hemorragia retroperitoneal.
c. rotura de bazo con hemoperitoneo.
68. Revisando la angiotomografía de un hombre de 70 años en estudio por aneurisma de
aorta abdominal, el radiólogo le informa de la presencia de una oclusión completa de la
arteria mesentérica inferior. El paciente se encuentra completamente asintomático. La
oclusión de la arteria mesentérica inferior cursa de manera asintomática en muchas
ocasiones ya que el territorio que irriga puede recibir flujo proveniente de la arteria: a. cólica
derecha
e. cólica media

69. En las patologías de esófago es importante conocer bien la anatomía esofágica. ¿Cuál de
las siguientes afirmaciones es correcta? a. El esófago tiene capa mucosa, muscular y serosa
c. El esófago torácico pasa por detrás del cayado aórtico

70. A pesar de que pueda haber variaciones anatómicas, lo habitual es que el ciego sea
irrigado por una rama arterial que proviene de unas de las siguientes arterias: a. Iliaca
derecha
d. Mesentérica superior

71. Ante un paciente con una cirugía abdominal urgente, el informe operatorio señala que se
ha realizado una resección de todo el duodeno y del tercio proximal del yeyuno manteniendo
íntegros el estómago y todo el íleon, así como los dos tercios distales del yeyuno. En el
seguimiento nutricional del paciente ¿Qué vitamina o mineral presentará con menor
probabilidad una disminución de su absorción?
a. Cianocobalamina

72. ¿Cuál de las siguientes sustancias forma parte de la secreción biliar? a. Tripsina
Lecitina

73.¿De qué musculo forma parte el ligamento inguinal?


-Oblicuo externo del abdomen

74.¿Cuál de las siguientes enzimas está localizada en el borde en cepillo y juega un rol en la
digestión de proteínas?
e. Carboxipeptidasa A.

75. Una de los siguientes sustancias, NO sirve como un buen agente emulsificante: a.
Colesterol

76. La sustancia que estimula el crecimiento de la mucosa gástrica es: a.


Secretina
d. Gastrina

77.¿Cuál de las siguientes alternativas es una función de la colecistokinina? a.


Relajación de la vesícula para la salida de bilis
d. Secreción de enzimas pancreáticas

78.Con respecto a la anatomía del tronco celiaco, señale lo correcto a. El tronco celiaco se
origina de la cara posterior de la aorta abdominal
d. La hepática común que es una de sus ramas, participa en la irrigación del estómago.

79. Con respecto a la anatomía del duodeno, marque la respuesta correcta: a. Tiene una
distribución en forma de “C”, que rodea la cola del páncreas
b. La 3ra porción duodenal está contenida en la pinza vascular aortomesentérica

80. En el íleon se absorbe aproximadamente el 95% dea través de la circulación


enterohepática.
a. agua
c. sales biliares

81. La estimula el mecanismo paracrino de la secreción de ácido clorhídrico. a.


histamina

82.En la digestión de proteinas,es el principal estímulo para convertir el


pepsinógeno en pepsina. a. la gastrina
b. el pH ácido

83. Con respecto a la somatostatina, marque lo correcto:


a. Es secretada por las células S del intestino
Interviene en la fase intestinal de la secreción gástrica

84. En pecten anal, es una estructura comprendida entre: a. la línea pectínea y los senos
anales
d. la línea anocutánea y la línea pectínea

85.¿Cuál de las siguientes alternativas es una proenzima pancreática? a. Tripsina 1. b.


Elastasa
2. c. Quimotripsinógeno
3. d. Amilasa
4. e. Procarboxipeptidasa C

86. Eneste paciente, el bloqueo farmacológico de los receptores H2 en la mucosa


gástrica:
a) No tiene efecto sobre la secreción de ácido inducida por el vago b)
Evita la activación de adenilciclasa por gastrina
c) Inhibe la secreción de ácido inducida por gastrina y mediada por el vago d) Causa un
aumento en el transporte de potasio por las células parietales gástricas Se validó la A :)

87. Si se considerara una gastrectomía total para curar la gastritis del paciente, cuál de las
siguientes sustancias ya no se produciría:
a) Gastrina
b) Quimiotripsina
c) Amilasa

d) Pepsinógeno

88. Un paciente hipertenso está tomando un medicamento bloqueador de receptores alfa


1 adrenérgicos (prazosina) y como efecto secundario se queja de: d) Lo escaso que es el
medicamento
e) No tiene problemas en la salivación

c) Hiposalivación
d) Hipersalivación

89. Con respecto a las lesiones y enfermedades de la boca, marque lo correcto: f)


La leucoplasia se desprende al roce

b) la eritroplasia puede degenerar en adenocarcinoma


c) El muguet oral es una enfermedad bacteriana en inmunodeprimidos d)
la eritroplasia debe ser biopsiada
NOTA: fue validada la opción B ya que no es motivo del curso que sepamos el puto
cáncer.

90. En este paciente, se puede asumir que la pancreatitis ha sido ocasionada por una
disminución en el efecto de:
g) Amilasa
h) Lipasa

c) Inhibidor de la tripsina
d) Entercinasa

91.Un efecto secundario en el estómago por la acción de la secretina es:


i) Disminución en la liberación de pepsinógeno

b) Menor actividad de la pepsina


c) Mayor paso de bicarbonato a sangre periférica
d) Aumento en la producción de factor intrínseco
92. Dentro de los factores protectores de la mucosa gástrica se pueden mencionar
múltiples protagonistas. Uno de ellos es:
j) CCK
k) Gastrina

c) Receptor muscarínico
d) Pepsina

93. La saliva puede tener una variedad de electrolitos en su composición. Entre ellos el
cloro, respecto al cual se puede afirmar:
l) Su mayor concentración se consigue con el flujo bajo

b) Su concentración no llega a ser tan alta como en el plasma


c) Con flujo alto su concentración es mayor que la del plasma
d) Su menor concentración se alcanza con flujo alto

94. En el síndrome de boca seca o síndrome de Sjogren, una de las complicaciones


asociadas es:
a) caries
b) Disminución de la acidez gástrica
c) Aumento en de la producción de saliva
d) Aumento del pH bucal

95. Estimula la producción de saliva:


a) Vasodilatación periglandular
b) Atropina
c) Fatiga o cansancio
d) Expresión de miedo

96. El omeprazol actúa sobre la membrana de la célula


m) Basolateral/principal
n) apical/principal
o) Basolateral/parietal

d) apical/parietal

97. Para protegerse del entorno ácido, el Helicobacter pylori se autogenera un entorno de
pH menos ácido alrededor suyo, gracias a una enzima que alcaliniza su entorno local
mediante la conversión de:
a) urea en NH3
b) H2O y CO2 en ácido carbónico
c) NH3 en urea
d) H2CO3 en bicarbonato
98. La anemia perniciosa destruye las células:
p) mucosas del cuello
b) oxínticas
c) principales
d) mucosas superficiales

99. La célula mucosa del cuello gástrico produce:


a) Moco
b) ácido clorhídrico
c) pepsinógeno
d) Factor intrínseco

100.El aumento en la acidez del estómago producido principalmente por la infección de


Helicobacter pylori se debe a la disminución de:
a) Somatostatina
b) Bicarbonato por las glándulas de Brunner
c) Secretina
d) Colecistoquinina

101. De las siguientes sustancias secretadas por los órganos de este paciente, la más
alcalina es la secreción:
a) pancreática
b) Esofágica
c) Yeyunal
d) Salival

102.En cuanto a la gastritis de este paciente, se encontró que era producida por la
bacteria Helicobacter pylori. Esta bacteria sobrevive en el medio ácido del estómago
gracias a:
a) ácido clorhídrico
b) Toxina CagA

c) Ureasa
d) Jugo pancreático

103.La lengua está recubierta por epitelio:


c) pseudoestratificado columnar no queratinizado

b) plano estratificado no queratinizado


c) pseudoestratificado columnar ciliado
d) plano estratificado queratinizado

104.El esfínter anal interno tiene musculatura …….. y tiene


control ……..
d) lisa / voluntario

b) lisa / involuntario
c) esquelética / simpático
d) esquelética / parasimpático

19) La arteria Aorta proporciona la irrigación al tubo digestivo ¿cuál de las siguientes
arterias proporciona la irrigación al ángulo cólico derecho?

a) mesentérica superior
b) mesentérica inferior
c) frénica inferior
d) tronco celiaco

20) Paciente de 26 años que le cuenta en su historia clínica que cada vez que almuerza a
los 20 min tiene deseo de defecar, le comenta que su hijo de 1 mes le pasa lo mismo pero
más intenso. Esto se explica por el reflejo …….., el cual está …… en el paciente
a) colicoileal / normal
b) colicoileal / alterado
c) gastrocólico / normal
d) gastrocólico / alterado

21) La región del estómago que se comunica con el duodeno es la a)


pilórica
b) cardias
c) cuerpo
d) fórnix

22) Acude a consulta un px que fue diagnosticado de úlcera péptica 3 días antes. Luego
de múltiples pruebas diagnósticas se concluye que el paciente presenta un tumor
secretor de gastrina, ¿cual de las siguientes situaciones estará incrementada? a)
distensión gástrica
b) inhibición del vaciado gástrico
c) secrecion de acido clorhidrico
d) inhibición de la secreción de pepsinógeno
23) En el sistema digestivo, el control del apetito está dado por un complejo sistema de
sustancias y órganos integradores, los cuales regulan la ingesta de alimentos. La …… es
una sustancia orexígena y es sintetizada por el ……
a) leptina / estómago
b) felina / intestino
c) leptina / estómago

d) grelina / estómago

24) Con respecto a la actividad eléctrica del sistema digestivo, marque la alternativa
correcta
a) corresponden a potenciales de acción que están presentes de forma continua y le dan
capacidad de peristalsis autónoma al sistema digestivo

b) la frecuencia de las ondas lentas NO se ve influenciada por la actividad neural ni las


hormonas gastrointestinales
c) en el estómago las ondas lentas se dan en una frecuencia de 6 x min d) las
ondas lentas son cambios lentos y ondulantes del potencial en reposo e) la
frecuencia de las ondas lentas va de 6 a 12 ondas por minuto

25) Ante una lesión del IX pc, el músculo …… se altera en su función a)


palatogloso

b) estilofaríngeo
c) palatofaríngeo
d) constrictor superior

26) Un varón de 50 años es sometido a extirpación del duodeno y parte proximal del
yeyuno. La pérdida de estímulo hormonal en el páncreas para la secreción enzimática se
explica por la pérdida de células.

a) parietales, productoras de factor intrínseco.


b) K productoras de factor intrínseco
c) M productora de CCK

d) I productora de CCK

27) Marque la respuesta correcta:


A. El bronquio derecho constituye una de las estrecheces del esófago B.
Todos los órganos del sistema digestivo tienen capa serosa

C. La pared gástrica en el fondo es más delgada que en el cuerpo y antro D.


El esfínter de Oddi rodea a la papila menor duodenal

28) Marque la respuesta correcta en relación a la gastrina:


A. Al distenderse el estómago, se inhibe su producción.
B. Se estimula por la liberación de noradrenalina

C. Las células G son las productoras y se encuentran principalmente en el antro


gástrico
D. Las células G se encuentran principalmente en el fondo gástrico 29) Para

poder morder una manzana, es necesario usar el siguiente músculo:

A. Milohiodeo
B. Tensor del paladar C.
Orbicular de los labios D.
Buccinador
30) Sustancia que inhibe la secreción y la motilidad del estómago prolongando el
tiempo de digestión:
A. Enteroglucagon.
B. Polipéptido pancreático
C. Péptido 1 similar al glucagón (GLP-1).

D. Péptido insulinotrópico dependiente de la glucosa (GIP).

31) El nacimiento de la arteria mesentérica superior se puede encontrar en cuál de los


cuadrantes abdominales:
A. Hipocondrio derecho
B. Hipogastrio

C. Epigastrio
D. Mesogastrio

32) Entre las múltiples causas de la Enfermedad por Reflujo Gastroesofágico, se


puede considerar también a una alteración en las del esfínter esofágico inferior:

A. Ondas secundarias
B. Contracciones tónicas
C. Ondas lentas
D. Glándulas subesofágicas

33) Producto de la alimentación, se producen diversas sustancias peptídicas, cininas y


bradicininas, las cuales permiten que:
A. Se produzca neovascularización en los territorios de las arterias abdominales B.
La acción de la lipasa pancreática se vea incrementada
C. El consumo de O2 del intestino aumente ligeramente

D. El flujo sanguíneo intestinal aumente hasta 8 veces


34) El dolor periumbilical o epigástrico en el inicio de una apendicitis aguda se debe a: A.
Estímulo del nervio vago.
B. Íleo secundario.
C. Irritación del peritoneo parietal.

D. Estímulo del sistema simpático.

35) El aumento en la actividad motora de la pared gástrica genera un aumento en los


niveles locales de qué sustancia en la microvasculatura:

A.Adenosina
B. Colecistoquinina CCK
C. Endotelina
D. Gastrina

36) ¿Cuál de los siguientes péptidos inhibe el vaciamiento gástrico?

A. Colecistoquinina
B. Péptido inhibidor gástrico
C. Motilina
D. Gastrina
37) Los músculos de la masticación que producen la retropulsión de la mandíbula son:

A. temporales [mas seguro]


B. maseteros
C. milohioideos
D. pterigoideos
38) En relación a la fisiología gástrica, marque lo correcto:

A. la cimetidina actúa en la región basolateral de la célula parietal


B. la marea alcalina se debe al paso de bicarbonato través de la membrana apical de la
célula principal
C. el cloro difunde hacia el exterior por la la región basolateral de la célula parietal D.
la salida de hidrogeniones a la luz es por difusión facilitada

39) Durante el sueño, la concentración de bicarbonato en la saliva: A.


Se eleva a niveles mayores que los del plasma
B. Aumenta
C. No tiene efecto

D. Disminuye

40) La secreción de saliva es importante en la fisiología digestiva. Su concentración de


potasio llega a ser menor que la del plasma cuando su secreción tiene un flujo:

A. Intermedio
B. Nunca
C. Bajo
D. Alto

42) Respecto a las glándulas salivales, marque lo incorrecto:


A. la glándula parótida produce secreción serosa
B. la glándula sublingual drena a través de conducto de Wharton
C. La glándula parótida drena a través del conducto de Stenon

D. la glándula sublingual tiene forma de garfio

43) Con respecto a la saliva, marque la respuesta correcta:


A. será hipertónica cuando el flujo es bajo
B. a mayor flujo, menor concentración de Na
C. a mayor flujo, menor concentración de cloro

D. el sistema simpático estimula su secreción

CI 3

44) En relación a la circulación hepática, marque lo correcto:

a) Los sinusoides hepáticos transportan sangre mixta


b) La vena porta proporciona el 50% de sangre al hígado
c) La vena porta se forma a partir de la vena esplénica y la mesentérica inferior

d) La arteria hepática deriva de la mesentérica superior

45) Dentro de las funciones de las células de Ito, marque lo incorrecto: a)


Sintetizan colágeno
b) Almacenan vitamina A
c) Se les llama células estrelladas

d) Pueden fagocitar patógenos y actúan como presentadoras de antígeno

46) Paciente con tumor neuroendocrino productor de secretina, debido a lo cual se puede
esperar que su secreción pancreática, comparada con la de una persona sana de bajo
flujo, tenga una concentración de:
a) Sodio aumentada
b) Igual
c) Bicarbonato aumentada
d) Potasio disminuida
47) El GALT se localiza en:

a) Lámina propia
b) submucosa
c) borde en cepillo
d) superficie de criptas de Lieberkühn

48) En relación a la histología hepática, marque lo correcto:


a) la zona 1 se afecta rápidamente en estados de hipovolemia y shock b)
La zona 1 se encuentra cercana a la vena central lobulillar

c) La zona 3 se encuentra más cerca a la vena central lobulillar


d) La zona 3 se encuentra más cerca al eje menor del acino hepático 49)

El acino pancreático difere con el de las glándulas salivales en:

a) Contiene células centroacinares


b) No produce secreción serosa
c) El páncreas produce principalmente secreción mucosa
d) No tienen diferencias

50) Durante la digestión de las grasa, para que la lipasa actúe adecuadamente se
requiere que el pH aumento en la luz intestinal, lo cual es logrado, entre otros, por la
secreción de las células:

a) Del conducto interlobulillar


b) Centroacinares
c) Acinares
d) Alfa

51) La secreción de la colecistoquinina (CCK) se produce en la fase:

a) intestinal
b) En las 3 por igual
c) gástrica
d) Cefálica

52) ¿Por cuál de las siguientes células es secretada principalmente la pro enzima
procarboxipeptidasa?

a) Acinares del páncreas


b) Epiteliales del duodeno
c) Ductales del páncreas
d) Centro Acinares del páncreas

53) Una mujer de 43 años dolor en hipocondrio derecho e icterica. En la ecografía se


evidencia cálculos biliares. Estos cálculos lo más probable es que se encuentren
localizados en:

a) conducto colédoco
b) Conducto cístico
c) Vesícula biliar
d) Conducto pancreático secundario

ECU 1:
Estudiante de 21 años sufre de gastritis aguda ocasionada por comer en lugares poco
higenicos. Suele consumir caramelos (“chupar”) mientras esta en base hasta la tarde.
También toma regular cantidad de leche (grasa, lactora, proteinas), pues le calma el dolor y
el ardor que sitnete por la gastritis (tiene dispepsia y cuando toma la leche se le pasa).
Incluso cuando puede, se toma dos vasos de agua frita y le calma la molestia. Ha decido
ir al medico para tratarse, pues ya no soporta el dolor, el cual esta seguro que los
síntomas se producen por elevada producion de HCl en el estomago, y por ello le ha
recetado ranitidina
1.1) El consumir caramelos eleva los niveles en sangre de una hormona cuya función es
la estimulación de las células.
- Beta del páncreas por GIP el cual es una incretina y por consiguiente estimula las
células pancreáticas

1.2) Consumir caramelos indirectamente actica la via:

-POMP/ CART saciedad

1.3) Consumo de leche produce indirectamente


- CCK inhibición del vaciamiento gástrico mayor tonicidad del esfínter pilórico

1.4) Cuando el px toma dos vasos de agua, genera indirectamente un aumento en la


liberación de:
- vaso de agua distención → g astrina → secreción de HCl

1.5) El uso de ranitidina bloquea el receptor H2 de la histamina en las células


parietales, la histamina llega a estas células por:
- histamina es una hormona paracrina por → difusión

**endocrina es por via hematógena y si fuera neuroendorina es por un NTs

1.6) Aumenta la secreción salival:


- noradrenalina a través de los receptores Beta 2

1.7) En este paciente con gastritis aguda debida a una alta producción de ácido
clorhídrico, sería lógico esperar que el píloro tenga un tono muscular: - primero la
secretina
- luego CCK

**ambas reguladores del HCl, Gatritis aguda debido a una alta producción de HCL
piloro estará aumentado (por la CCK)

1.8) Debido al uso de ranitidina, los valores de somatoestina en sangre: -


ranitidina disminuye acción de gastrina se quiere secretar mas no actúan los
inhibidores como la somatoestina somatoestina disminuye

1.9) El uso de atropina en este paciente:


- Inhibirá la acción de las prostaglandinas
- Aumentará la producción de ácido clorhídrico
- Disminuirá la acción del receptor CCK-B

-Aumentará el pH del estómago


ECU 2:
Niño de sexo masculino de 2 años de edad, sufre de estreñimiento desde el nacimiento
(1 deposición cada 3-4 días). Madre menciona que le estimula la defecación con un
termómetro rectal, y continuo uso de enemas y laxantes. Desde hace 6 meses
comienza con vómitos postprandiales. Los síntomas aumentan en frecuencia y
magnitud y están en relación con los episodios de estreñimiento. No refiere fiebre, tos,
diarrea ni lesiones cutáneas. Al examen físico presenta regular
estado general, luce deshidratado. Abdomen distendido, blando, depresible e indoloro. No
se palpan masas abdominales. Se permeabiliza el canal anal con termómetro rectal,
encontrando cierta resistencia. Salida de material fecal mal oliente en regular cantidad.
Exámenes de laboratorio: hemograma normal. Signos inflamatorios de fase aguda
negativos. Alcalosis metabólica leve en sangre
venosa. Radiografía con enema baritado muestra recto y colon sigmoides dilatados
(megacolon). Biopsia profunda: ausencia de células ganglionares en la muestra
enviada. Se realiza cirugía correctiva.

2.1) Durante la fase esofágica de la deglución, para un bolo alimenticio determinado, a


medida que avanza el bolo la fuerza de la contracción se hace más: - hiperpolarizado
- fuerte
- dependiente de Ach
- debil

2.2) Cuando este paciente ingiera sus alimentos, se espera que al momento de pasar el
bolo alimenticio por el esfínter esofágico superior, la presión intraesofágica disminuya en:
- la porción proximal al bolo
- el tercio medio del esófago

-el cardias
- el lugar donde se contraiga la muscular propia

2.3) Al examinar la orofaringe del paciente, uno puede hallar fácilmente la amígdala
palatina, pues esta se encuentra inmediatamente detrás del músculo:

-Palatogloso
- Palatofaringeo
- Hiogloso
- Elevador del velo del paladar

2.4) Con respecto a la defecación en este caso, marque la respuesta correcta: - En


posición de cuclillas, el músculo puborectal genera un ángulo más agudo en el recto
- El sigmoides y el recto están inervados por el nervio vago
- La sensación de defecar sólo se da cuando el recto es ocupado por heces y
alcanzado el 80% de su capacidad

-El esfínter anal comprometido tiene inervación autónoma

2.5) En este paciente [hirschsprung] se considera que está abolido el reflejo: -


Coloileal

-Rectoesfinteriano
- Gastrocólico
- Relajación receptiva

2.6) No se espera que sea causa del vómito:

-Ayuno prolongado
- Estimulación faríngea y del glosofaríngeo
- Irritación de la mucosa gástrica
- Dolor intenso

ECU 3:
Paciente de 54 años con antecedentes de alcoholismo, gastritis crónica, tabaquismo
pesado, obesidad, cálculos biliares y cirrosis, es llevado a la emergencia por dolor
abdominal en epigastrio irradiado a la espalda y trastorno del sensorio. Al examen físico:
presión arterial 85/50 mmHg, frecuencia cardíaca 100 latidos/min, frecuencia respiratoria
18 x minuto, temperatura axilar 36°C.
Conjuntivas pálidas, escleras ictéricas nevus arácnidos en tronco, distensión
abdominal marcada, cabeza de medusa, matidez desplazable en ambos flancos e
hipogastrio, dolor a la palpación de abdomen.
Tiempo de protrombina: 24 seg (testigo: 13 seg); TPT: 38 seg, glicemia: 165 mg/dL,
uremia: 20 mg/dL, ASAT: 76 UI/L, ALAT: 22 UI/L, albumina: 2,5 g/dL, bilirrubina total: 2,6
mg/dL, bilirrubina directa: 1,4 mg/dL, amilasa sérica 4000 U/L.

3.1) En esta paciente, al aumento de la amilasa sérica, se debe directamente a una


lesión de:

a) páncreas
b) vesícula y árbol biliar
c) estómago
d) hígado

3.2) Considerando que el paciente sufre de gastritis, se puede decir que la secreción de
ácido por la mucosa gástrica

a) involucra transporte activo de hidrogeniones


b) es realizada principalmente por células principales
c) es inhibida por antihistaminas tomadas por pacientes con rinitis alérgica d)
involucra la liberación de HCl de los gránulos zimógenos

3.3) El paciente tiene hemorragia digestiva alta por várices sangrantes como
complicación. Llegando a estar en shock hipovolémico por hemorragia masiva, se
encontrara necrosis hepática en:
a) zona 1
b) no se afectan los lobulillos hepáticos en hemorragia

c) zona 3
d) zona 2

3.4) El misoprostol, análogo de las prostaglandinas está mejor indicado en: a)


cicatrización de úlcera péptica duodenal
b) erradica el helicobacter pylori
c) tratar el sind de Zollinger ellison

d) prevenir daño por AINES

3.5) De las siguientes sustancias secretadas por los órganos de este paciente, la más
alcalina es la secreción:
- Esofágica
- Salival
- Yeyunal
-Pancreática
3.6) En este paciente, se puede asumir que la pancreatitis ha sido ocasionada por una
disminución en el efecto de:
- Lipasa
- Enterocinasa
- Amilasa
-Inhibidor de la tripsina
3.7) ¿Cuál de las siguientes sustancias es segregada por el páncreas? -

Amilasa

- Pepsina
- Quimiotripsina
- Tripsina
3.8) Cada vez que este paciente toma alcohol, la acidificación de la luz del duodeno:

-Disminuye el vaciamiento gástrico


- Aumenta la contracción del esfínter de Oddi
- Aumenta la secreción del ácido gástrico
- Disminuye la secreción pancreática del bicarbonato
SISTEMA DIGESTIVO (ME 154) EXAMEN FINAL
Ciclo 2018-01
1. Un niño de 2 años es llevado a la consulta por diarrea persistente y edema de las extremidades, además
falta de crecimiento y desarrollo en relación a su edad. Los análisis de sangre revelan que tiene
concentración plasmática baja de proteínas (hipoproteinemia). Durante la endoscopía duodenal, se coloca
colecistokinina (CCK) endovenosa y se recoge muestras del líquido duodenal; el resultado del líquido
confirma incapacidad para hidrolizar proteínas a un pH neutro, esta situación mejora al añadir una pequeña
cantidad de tripsina. El paciente probablemente esté sufriendo la falta congénita de

………….
(Unidad 4, sesión 26, logro 2: Explicar la Digestión y absorción de las proteínas y sus alteraciones) a.
Pepsinógeno
b. PEPT-1
c. Carboxipeptidasas
d. Enterocinasa

2. Experimentalmente se incrementa la velocidad de la secreción salival con una sustancia, el análisis de la


composición de esta saliva obtenida se espera encontrar…………..

(Unidad 3, sesión 17, logro 5 : Explica la Influencia de la velocidad del flujo salival en la composición de la
saliva)
a. Elevación de concentración de bicarbonato, sodio y potasio
b. Elevación de concentración de cloro, sodio y potasio
c. Disminución de concentración de potasio
d. Disminución de concentración de potasio y bicarbonato

3. Paciente varón de 46 años soltero, consulta por odinofagia y bajo de peso, tiene antecedente de
tuberculosis desde hace 3 meses y es fumador crónico (10 cigarrillos por día); al evaluar la cavidad oral se
identifica lesión blanquecina en el dorso de la lengua y paladar blando, las lesiones se desprenden con el
baja lengua dejando una base eritematosa. Esta lesión corresponde probablemente a ……………………….…..

( Unidad 3, sesión18, logro 1-2 : Describe las enfermedades inflamatorias, infecciosas y proliferativas de la
cavidad oral)
a. Eritroplaquia
b. Candidiasis oral
c. Leucoplaquia vellosa
d. Fibroma en cavidad oral

4. Minero de 32 años de edad, que acude a centro de salud por presentar de forma progresiva desde hace 1
año dificultad para ingerir alimentos sólidos y luego líquidos; refiere regurgitaciones alimentarias y marcada
pérdida de peso (15 kilos). Radiografia baritada de
esófago como se muestra en la figura. El presente caso se
explica por……………….

(Unidad 2, sesión 12, logro 4: Identificar y describir la función


de los esfínteres esofágicos)
a. Contracción incompleta del esfínter esofágico inferior
b. Dificultad para el inicio de la deglución c. Relajación

incompleta del esfínter pilórico d. Relajación incompleta

del esfínter esofágico inferior

5. Paciente mujer de 35 años acude a consulta por sensación de sequedad y lesiones en cavidad oral. Al
examen se observa atrofia de la mucosa, fisuras y úlceras; nota además sequedad e irritación de la córnea y
aumento del tamaño de las glándulas parotídeas. Su diagnóstico más probable es artritis reumatoide; el
hallazgo más probable en una biopsia de glándula parótida es……..….

(Unidad 3, sesión 18, logro 3: Describe las enfermedades más frecuentes de las glándulas salivales) a.
Hiperplasia de acinos glandulares serosos
b. Gran infiltración de linfocitos y células plasmáticas
c. Gran infiltrado de linfocitos y macrófagos
d. Presencia de acinos normales con hiperplasia de células ductales
6. Un paciente con anemia acude con su médico quejándose de episodios frecuentes de gastroenteritis. Un
análisis de sangre revela anticuerpos circulantes dirigidos contra células parietales gástricas. Su anemia es
atribuible a la hiposecreción de

………………………
(Unidad 3, sesión 20, logro 5: Gastritis crónica. Tipos de gastritis)
a. Factor intrínseco
b. Proteina R (haptocorrina)
c. Pepsinógeno
d. Ácido clorhídrico

7. Dos estudiantes deciden tomar un receso para comer una hamburguesa a la hora del almuerzo. Antes de
llegar a la cafetería, impulsos nerviosos provenientes del complejo vagal dorsal iniciarán la secreción de ácido
gástrico por la liberación dedesde el sistema nervioso entérico.

(Unidad 3, sesión 20, logro 2: Regulación de la secreción gástrica: estimulación, fases de la secreción) a.
Serotonina
b. Óxido nítrico
c. GRP (péptido liberador de gastrina)
d. Péptido intestinal vaso activo

8. Un niño de cuatro años de edad es llevado a la consulta por cuadros diarreicos frecuentes
caracterizados por heces pálidas, voluminosas y fétidas, presenta bajo peso y talla. Se mide la
concentración de cloruro en el sudor y se encuentra que sus valores son muy elevados. La alteración más
importante a nivel de células ductales del páncreas tiene relación directa con la conductancia de…………

(Unidad 3, sesión 23, logro 5 Explica la Secreción pancreática: formación del jugo pancreático, influencia de la
velocidad de flujo y regulación)
a. Potasio
b. Bicarbonato
c. Sodio
d. Cloro

9. Una mujer de 50 años de edad que sufrió durante varios años resequedad de los ojos debida a
producción inadecuada de lágrimas es enviada con un gastroenterólogo para evaluación de pirosis crónica.
El examen endoscópico revela erosiones y tejido cicatrizal en la parte distal del esófago justo por arriba del
esfínter esofágico inferior. Las lesiones pueden atribuirse a la disminución de uno de los siguientes
componentes salivales:

(Unidad 3, sesión 17, logro 4: Explicar la Formación de la saliva y cuáles son sus componentes) a.
Bicarbonato
b. Lactoferrina
c. Ig A
d. Amilasa
10. Se evalúa los valores séricos de las siguientes sustancias a un paciente con enfermedad hepática
terminal; en este paciente se espera encontrar la combinación con la letra …………
hepática, encefalopatía hepática e
(Unidad 3, sesión 22, logro 5: Describe las hipertensión portal) Glucosa
Pruebas de función hepática, la Insuficiencia Amoniaco Albúmina

a. Aumentada Disminuida Disminuida b. Disminuida Aumentada Aumentada c. Aumentada


Aumentada Aumentada d. Disminuida Aumentada Disminuida

11. Una mujer de 35 años de edad HIV positiva, se presenta al médico con dolor abdominal en
cuadrante superior derecho e ictericia. La paciente refiere haber tenido múltiples episodios de ictericia
durante los últimos 10 años. Los exámenes para determinar hepatitis viral, dieron positivos para
Hepatitis B, siendo catalogado el caso como hepatitis crónica con alteración funcional. En un examen de
sangre ¿cuál de los siguientes parámetros está disminuido?
(unidad 3, sesión 22, logro 5: Pruebas de función hepática, Insuficiencia hepática, encefalopatía hepática
e hipertensión portal)
a. Fosfatasa alcalina
b. Albumina
c. Bilirrubina
d. Tiempo de protrombina

12. En el reflejo peristáltico del intestino delgado, uno de los siguientes eventos sucede en la porción
oral del bolo alimenticio…………...

(Unidad 2, sesión 13, logro 4: Explicar la Motilidad del intestino delgado: Contracciones segmentarias y
peristálticas)
a. Disminución de 5 hidroxitriptamina desde las neuronas IPAN
b. Contracción del músculo longitudinal
c. Acción del péptido intestinal vasoactivo (VIP) en el músculo circular
d. Acción de acetilcolina en el músculo circular

13. Experimentalmente se coloca una dosis alta de secretina en la luz intestinal duodenal; como
consecuencia de esto, en el jugo pancreático de la misma luz intestinal se observa la disminución de la
concentración de …..………..

(Unidad 3, sesión 23, logro 5: Explica la Secreción pancreática: formación del jugo pancreático, influencia
de la velocidad de flujo y regulación)
a. Na+
b. Cl
c. K+
d. HCO3-
14. Un varón de 58 años de edad con enfermedad de Crohn severo fue sometido a una resección ileal.
Después de la cirugía este paciente padecerá de esteatorrea, esto se explica porque …..……….. (unidad
4, sesión 26, logro 4: Explica las alteraciones en la Absorción de lípidos) a. El pool de ácidos biliares se
incrementa
b. Los quilomicrones no pueden formarse en el lumen intestinal
c. La micelas no pueden formarse
d. El páncreas no secreta lipasa

15. En un experimento se inserta un balón en el estómago de un voluntario, se infla poco a poco


mientras que se vigilan las presiones intraluminales. Aunque el volumen del balón aumenta
considerablemente, las presiones permanecen constantes. Esta relación volumen-presión se explica por
la liberación local de …………..
(Unidad 2, sesión 13, logro 1 Explica la Motilidad gástrica: relajación receptiva)
a. Acetil colina y gastrina
b. Colecistoquinina y óxido nítrico
c. Óxido nítrico y péptido inhibidor vasoactivo
d. Norepinefrina y óxido nítrico
16. La toxina del Vibrio cholerae causa diarrea debido a…….
(Unidad 4, sesión 27, logro 6: Explica el transporte hidroelectrolítico intestinal, toxina colérica)
a. La fosforilación del canal CFTR de los enterocitos de las vellosidades intestinales b. El
Incremento de la secreción de cloro por las células de la cripta intestinal
c. La inhibición de la producción de AMPc por las células epitelailes
d. El incremento de la absorción de agua y sodio a través de las uniones estrechas 17. ¿Cuál de las

siguientes alternativas es una característica de la secreción exocrina del páncreas?

(Unidad 3, sesión 23, logro 5: Secreción pancreática: formación del jugo pancreático, influencia de la
velocidad de flujo y regulación)
a. Tiene una baja concentración de Cl-respecto al plasma
b. Es estimulada por la presencia de bicarbonato en el duodeno
c. La secreción enzimática es estimulada principalmente por la gastrina
d. Es hipotónica respecto al plasma

18. Una madre lleva a su hijo de dos años de edad a la sala de urgencias, estresada porque el niño
deglutió una moneda de 10 céntimos mientras la familia cenaba en un restaurante. El médico observa
mediante fluoroscopía que la moneda se halla en el estómago y asegura a la madre que la moneda se
eliminará con las heces. El médico recomienda utilizar la respuesta fisiológica que permitirá la
evacuación de la moneda del estómago al intestino ………….…..

(Unidad 2, sesión 13, logro 2: Explica la Motilidad gástrica: mezclado y vaciamiento)


a. Es por la relajación receptiva
b. Son los movimientos de mezcla y trituración
c. Es provocada por el ayuno
d. Es por la relajación del esfínter esofágico superior

19. Las estructuras en el hígado que permite que los productos metabólicos unidos a proteínas tengan
acceso a las membranas basolaterales de los hepatocitos, son…..
(Unidad 3, sesión 21, logro 4-5: Explica la Organización micro estructural del hígado)
a. Los Canalículos
b. Las fenestras sinusoidales
c. Las uniones intercelulares herméticas
d. Las células de Ito

20. La composición de la bilis es modificada conforme fluye por los conductillos biliares. Durante este
tránsito se espera que aumente la concentración de…….

(Unidad 3, sesión 22, logro 2: Describe la Secreción biliar, visión general del sistema biliar extrahepático y
composición de la bilis)
a. Ig A
b. Glucosa
c. Monómeros de ácido biliar
d. Vitamina A

21. Se mide experimentalmente el contenido gástrico de dos personas. La persona “A” tiene alto
contenido de grasa y la persona “B” tiene un contenido hipertónico ¿Cuál de las siguientes es correcto
respecto al vaciamiento gástrico? (Unidad 2, sesión 13, logro 2: Describe la Motilidad y vaciamiento
gástrico)
a. Hay ralentización del vaciado gástrico solo en “A”
b. El vaciamiento gástrico es más rápido en ambos
c. En ambos casos hay incremento de la motilina
d. Hay ralentización del vaciado gástrico en ambos casos
22. El examen endoscópico de un paciente con hipertensión portal grave revela venas tortuosas que
sobresalen hacia la luz del esófago. El paciente recibe tratamiento quirúrgico mediante la colocación de
una derivación que conecta la vena porta a la vena cava. Después de la operación el riesgo de
encefalopatía ………………….. y el riesgo de sangrado de várices ……………..

(Unidad 3, sesión 22, logro 5: Describe la Insuficiencia hepática, encefalopatía hepática e hipertensión
portal)
a. Aumentará/disminuirá
b. Disminuirá/disminuirá

c. Aumentará/aumentará
d. Disminuirá/aumentará

23. Un paciente varón de 18 años de edad acude al médico para sus exámenes de rutina. Sus resultados
de laboratorio muestran un valor de bilirrubina sérica de 4 mg/dl y una bilirrubina directa de 0,3 mg/dl.
Las pruebas de función hepática son normales. La alteración que explica mejor este caso es por la
deficiencia de ………………..
(Unidad 3, sesión 22, logro 3: Explica la Producción y excreción de bilirrubina. Tipos de bilirrubina e
ictericia)
a. Transaminasas
b. Glucuronil transferasa
c. Hemo oxigenasa
d. La 7 alfa hidroxilasa

24. Un hombre de 57 años de edad es llevado a urgencias con hematemesis masiva rojo brillante, a su
llegada se halla inconciente con PA: 80/40 mm Hg y FC: 124 lat/min. Luce ictérico con presencia de
“arañas vasculares en el tórax anterior y extremidades”, abdomen distendido con signo de oleada
positiva. Se encuentra esplenomegalia y pérdida de la masa muscular en extremidades. La anastomosis
vascular responsable del sangrado en este paciente es ………….…..

(Unidad 3, sesión 21, logro 2: Describe las anastomosis porto sistémicas)


a. Arteria gástrica izquierda y vena ácigos
b. Vena gástrica izquierda y vena ácigos
c. Vena paraumbilical y vena epigástrica inferior
d. Vena gástrica izquierda y vena esofágica superior

25. Un estudiante de medicina está comiendo un plato de comida a base de champiñones, espárrago y
salsa de soya. El sabor umami contenido en todos estos alimentos actúa a nivel de los botones
gustativos estimulando ………………..
(Unidad 2, sesión 10, logro 5: Describe los tipos y mecanismos moleculares para la detección de los
sabores)
a. El ingreso de sodio
b. Un receptor acoplado a proteína G
c. Su receptor específico T1R3
d. El ingreso de hidrógeno

26. Un hombre de 22 años de edad se presenta al médico con una historia de 1 año de evolución
caracterizado por dolor recurrente en fosa iliaca derecha y diarrea. Manifiesta además pérdida de peso
de 8 kg durante este periodo. La colonoscopía revela múltiples lesiones en el ileon terminal y colon. La
biopsia de estas lesiones revela engrosamiento, inflamación y ulceración de la mucosa. El diagnóstico
más probable en este caso es…….
(Unidad 4, sesión 28, logro 5: Describe la Enfermedad inflamatoria intestinal. Generalidades, morfología
y características)
a. Sprue celiaco
b. Enfermedad de Crohn
c. Sindrome de colon irritable
d. Colitis ulcerativa

27. Una de las funciones del músculo señalado es:

(Unidad 2, sesión 8, logro 3: Describir el Piso de la boca: estructuras blandas que la conforman)
a. Eleva el paladar blando
b. Recibe inervación del nervio maxilar
c. Deprime el hioides cuando la mandíbula está fija
d. Deprime la mandíbula cuando el hioides está fijo

28. Varón de 61 años que consulta por dolor retro esternal intenso desde hace 6 horas y después de
vómitos intensos y repetidos; al examen se observa disnea, cianosis, hipotensión y signos clínicos de
shock. La radiografía simple de tórax muestra neumomediastino. El líquido en el espacio pleural
aspirado tiene alta concentración de amilasa. ¿Cuál de las siguientes alternativas puede explicar este
cuadro clínico?
(Unidad 3, sesión 18, logro 6: Describe algunas Enfermedades del esófago)
a. Sindrome de Mallory Weiss
b. Rotura espontánea de esófago
c. Neumotórax por probable herida penetrante
d. Perforación de ulcera gástrica de cara posterior, con complicación torácica

29. La secreción del ácido en la célula parietal gástrica se lleva a cabo por una ATPasa especifica que
intercambia hidrogeniones (H+) del citosol por…..
(Unidad 3, sesión 20, logro 1: Explica la Secreción del HCl y sustancias que la alteran)
a. Cl
b. HCO3-
c. Na +
d. K+

30. En condiciones normales el ingreso de 600 ml de líquido es el estómago provoca un aumento de


presión intragástrica de unos 12 cm de H2O. Después de una vagotomía (corte del nervio vago) es de
esperar que el ingreso del mismo volumen de líquido provoque lo siguiente: …………………………………

(Unidad 2, sesión 13, logro 1: Describe la Motilidad gástrica: relajación receptiva)


a. Un aumento igual de la presión
b. Que no aumente la presión
c. Un aumento mayor de la presión
d. Una disminución de la presión

31. Una paciente de 30 años de edad es sometida a una cirugía en oído medio derecho por un problema
de otoesclerosis. Luego de la cirugía refiere alteración en la percepción de sabores. Al evaluar el caso
usted esperaría encontrar……….
(Unidad 2, sesión 10, logro 5: Describe la Irrigación e inervación de la lengua)
a. Alteración en la sensación del dolor y temperatura en el tercio posterior de la lengua
b. Alteración en la sensación del gusto en los dos tercios anteriores de la lengua c.
Alteración en la sensación del gusto en la punta de la lengua
d. Sensación del dolor, tacto y temperatura conservada en toda la lengua

32. ¿Cuál de las siguientes alterativas es correcta?

(Unidad 4, sesión 26 : Explica la digestión y absorción de nutrientes y sus alteraciones) a. En el


borde luminal, en cepillo, del intestino delgado, la absorción de sodio únicamente se realiza
asociada a la de glucosa.
b. El lugar principal para la absorción del hierro es el ileon
c. Las sales biliares desconjugadas son absorbidas preferentemente en el colon
d. El proceso de digestión y absorción de la vitamina B12 no se altera en insuficiencia pancreática.

33. En un paciente de 45 años de edad con colestasis biliar, se encuentra una elevación de los niveles
sanguíneos de fosfatasa alcalina hasta 3 veces la cifra normal. ¿Cuál de las siguientes alternativas estará
también elevada como evidencia del daño de la vía biliar?

(Unidad 3, sesión 22, logro 5: Pruebas de función hepática, Insuficiencia hepática, encefalopatía hepática
e hipertensión portal)
a. Tiempo de protrombina y albúmina sérica
b. Transaminasas hepáticas (ALT y AST)
c. Glucoronil transferasa
d. Gamma glutamil transpeptidasa
34. Revisando la angiografía de un hombre de 70 años en estudio por aneurisma de aorta abdominal el
radiólogo informa de la presencia de una oclusión completa de la arteria mesentérica inferior. El
paciente se encuentra completamente asintomático. ¿Cuál de las siguientes arterias se anastomosa a la
sistema arterial de la mesentérica inferior?

(Unidad 4, sesión 25, logro 1: Identifica la Arteria mesentérica superior e inferior, ramas y anastomosis)
a. Ileal
b. Cólica media
c. Sigmoideas
d. Cólica izquierda

35. Lactante de 3 meses de vida es atendido por presentar diarrea, se administra una solución de
glucosa y electrólitos por vía oral. La proteína de membrana apical que explica la capacidad de esta
solución para proporcionar aporte de glucosa e hidratación es ………..
(Unidad 4, sesión 26, logro 1: Explica la Digestión y Absorción de los hidratos de carbono. Alteraciones)
a. GLUT-5
b. SGLT-1
c. CFTR
d. GLUT-2

36. Paciente ha sufrido herida de bala en el abdomen, se le ha tenido que extirpar el segmento medio y
distal del ileon. En este caso la síntesis hepática de sales biliares estará …..…..
(Unidad 3, sesión 22, logro 4: Explica la formación, función y Circulación entero hepática de lasa sales
biliares)
a. Disminuida por inhibición de la colesterol 7 alfa hidroxilasa
b. Incrementada por estímulo de la enzima colesterol 7 alfa hidroxilasa
c. Incrementada por inhibición de la colesterol 7 alfa hidroxilasa
d. Sin cambios en el ritmo de síntesis
37. Un varón de 75 años ingresa al consultorio por presentar ictericia marcada de piel y las escleras. El
estudio del paciente mostró que presentaba un tumor que obstruía la totalidad del conducto hepático
común. ¿Cuál de las siguientes estructuras se encontrará dilatada en este paciente?

(Unidad 3, sesión 21, logro 6: Describir el árbol biliar intrahepático)


a. Conducto de Wirsung
b. Conductos de Hering
c. Conducto colédoco
d. Conducto cístico

38. Correlaciones las dos columnas y marque la fórmula correcta:

(Unidad 4, sesión 28, logro 1: Diarrea: definición, mecanismos: osmótica, secretoria y exudativa)
1. Enfermedad Hirschsprung( ) heces con moco y sangre
2. Diarrea osmótica( ) intolerancia a lactosa
3. Diarrea secretoria( ) aganglionosis congénita
4. Diarrea exudativa( ) canales de Cl- en las células de la cripta

a.- 4231
b.- 1234
c.- 2143
d.- 4213

39. Respecto a la siguiente imagen que representa una estructura de la mucosa gástrica, la estructura
con número ………..

produce ……………………..
(Unidad 3, sesión 19, logro 4: La glándula fúndica. Funciones y tipos de células con sus características)
a. 3 / pepsina
b. 1 / Pepsinógeno
c. 4 / HCl y factor extrínseco
d. 2 / pepsinógeno

40. En un paciente con insuficiencia renal crónica, el déficit en la absorción de calcio a nivel del
enterocito se debe a lo siguiente:
(Unidad 4, sesión 26, logro 6: Explica la Absorción de calcio y hierro)
a. No se convierte la 25 hidroxicolecalciferol a 1,25 dihidroxicolecalciferol
b. No se convierte la 1,25 dihidroxicolecalciferol a 25 hidroxicolecalciferol
c. Se incrementa la producción de Calbindina
d. Existe un descenso de la alfa 25 hidroxilasa renal
CLAVES EXAMEN PARCIAL DE SISTEMA DIGESTIVO 2019 – 00

1. Varón de 30 años es traído a emergencia por agresión abdominal con arma de fuego (pistola) y es
sometido a laparotomía exploratoria, observándose isquemia del colon ascendente y parte del colon
trasverso ¿la lesión de cuál de las siguientes arterias explicaría esta isquemia? (unidad 1, sesión 2,
logro 6: (Describe la irrigación visceral: arterias de tronco celiaco, arteria mesentérica superior e
inferior, topografía de superficie, órganos por cuadrante) a. Celiaca
b. Colónica derecha
c. Mesentérica inferior
d. Mesentérica superior

2. Respecto a las sustancias gastrointestinales que regulan la secreción pancreática; marque la


afirmación correcta:
(unidad 1, sesión 3, logros 2 y 3: describir las hormonas gastrointestinales: estímulos y funciones) a. La
Secretina, es la hormona más importante para la secreción de bicarbonato por las células acinares del
páncreas
b. La acetilcolina es capaz de estimular la secreción enzimática y de bicarbonato del
páncreas c. La gastrina, es la hormona más importante para la secreción de enzimas
pancreáticas d. La colecistoquinina (CCK) estimula al páncreas solo para secreción
enzimática

3. Ante una lesión del X par craneal, ¿cuál de los siguientes músculos mantiene conservada su función?:

(unidad 2, sesión 08, logro 4: Paladar blando: componentes musculares)


a. Elevador del velo del paladar
b. Tensor del velo del paladar
c. Palatofaríngeo
d. Glosofaríngeo

4. Experimentalmente se utiliza atropina (anticolinérgico) para inhibir la secreción de gastrina, sin


embargo la secreción de esta hormona se sigue dando ante estímulos vagales. Esta situación se explica
porque la atropina:

(unidad 1, sesión 3, logro 3 : describir las hormonas gastrointestinales: estímulo y funciones de la


gastrina y colecistoquinina)
a. Bloquea parcialmente la bomba de protones en la célula G
b. Inhibe la acción de acetilcolina e histamina en la célula G
c. Solo inhibe la acción del péptido GRP en la célula G
d. No bloquea la acción del péptido GRP

5. Un varón de 50 años es sometido a extirpación del duodeno y parte proximal del yeyuno. Esta
situación ocasionaría la pérdida de las células ……….. , productoras deque estimula la secreción de

bicarbonato por el páncreas.


(unidad 1, sesión 3, logro 3: describir las hormonas gastrointestianles: estímulos y funciones de la
secretina y péptido insulinotrópico dependiente de glucosa)
a. “S” / secretina
b. Parietales / secretina
c. “I” / colecistoquinina
d. “S” / colecistoquinina
6. Recién nacido que presenta tumoración abdominal a nivel del cordón umbilical (fotografía). ¿cuál de
las siguientes afirmaciones es correcta respecto a este defecto en el desarrollo embriológico del
intestino?: (unidad 1, sesión 5, logro 2: identificar las anomalías del desarrollo del intestino medio)

a. Corresponde a una Gastrosquisis


b. Las vísceras se hallan cubiertas por piel
c. No está asociado a otras malformaciones
d. Se asocia a malformaciones cardiacas y del tubo
neural

7. Varón de 35 años acude a la emergencia por trauma abdominal y se decida realizar una laparoscopía
exploratoria. El cirujano observa la disposición de los órganos abdominales como se representa en el
siguiente esquema. Esta disposición de órganos se explica por la rotación(SMA=arteria mesentérica
superior)
(unidad 1, sesión 5, logro 3: identificar las anomalías del desarrollo del intestino medio: defectos de
rotación, estenosis y atresias)
a. anti horaria del intestino medio, en sólo 90°
b. incompleta del intestino medio (270°)
c. horaria del intestino medio
d. horaria del estómago

8. Se evalúa la expresión de la proteína Agrp en una persona con alteración del apetito; lo correcto
respecto a esta proteína es…..
(unidad 1, sesión 3, logro 4: Explica los mecanismos de control del apetito y saciedad )
a. Esta proteína es un potente anorexigénico
b. La mutación del gen que la codifica produce adelgazamiento
c. La sobre producción de la proteína lleva a obesidad por agonismo de receptores MC3 y MC4
d. La sobre producción de la proteína disminuye el apetito por antagonismo de receptores MC4

9. Juana cae de la bicicleta y se fractura la región anterior del hueso maxilar superior con compromiso
de la fosa incisiva. Al examen físico de la región esperaría encontrar alteración en la sensibilidad de la
encía …………………

(unidad 2, sesión 8, logro5: paladar: paladar duro y blando: irrigación e inervación)


a. bucal posterior
b. Lingual anterior
c. palatina anterior
d. palatina posterior
10. Recién nacido es atendido por el neonatólogo y luego entregado a su madre para dar de lactar; la
madre al dar de lactar observa coloración azulada de labios, acompañado de tos persistente, dificultad
respiratoria y distención abdominal. Se le intenta colocar una sonda nasogástrica pero esta retorna a la
cavidad oral en todos los intentos. ¿Cuál de las siguientes anomalías del desarrollo es el más probable
en este caso? (unidad 1, sesión 4, logro 3: identificar las anomalías en el desarrollo del esófago: atresia
y/o fístula traqueo esofágica)

a. Estenosis esofágica proximal con Fístula traqueo esofágica distal


b. Atresia esofágica proximal con fístula traqueoesofágica distal
c. Atresia esofágica distal con fístula traqueoesofágica proximal
d. Fístula traqueoesofágica proximal y distal

11. ¿Cuál de los siguientes mecanismos ocurre durante la defecación?


(unidad 2, sesión 13, logro 6: motilidad del intestino grueso: contracciones segmentarias, movimientos
en masa, defecación y reflejo gastrocólico)
a. Contracción refleja del esfínter anal interno
b. En la posición de “cuclillas” el músculo puborectal se halla relajado
c. Relajación del esfínter anal externo por efectos del VIP y óxido nítrico
d. La materia fecal en el recto estimula la contracción del sigmoides por los nervios pudendos

12. La estructura número 4 (gráfico) corresponde a


……….… y está ………..
(unidad 2, sesión 9, logro 2: Partes de un diente. Capas del diente: Esmalte: características y células que
lo producen)
a. el cemento / mineralizado en 90%
b. la dentina / formada por ameloblastos
c. el esmalte / formado por células derivadas del mesénquima
d. la dentina / formado por células derivadas de la cresta neural

13. Un paciente luego de un accidente sufre lesión del piso de la boca, se constata daño del nervio
“cuerda del tímpano”, en este caso se esperaría encontrar disminución de lade la lengua (unidad
2, sesión 10, logro 3: Irrigación e inervación de la lengua)
a. Motilidad en los dos tercios anteriores
b. Sensación del gusto en el tercio posterior
c. Sensación del gusto en los dos tercios anteriores
d. Sensibilidad al tacto en los dos tercios anteriores
14. ¿Cuál de las siguientes afirmaciones es la correcta sobre la gastrina?

(unidad 1, sesión 3, logro 1: reconocer las características de las sustancias reguladoras


gastrointestinales: hormonas, sustancias paracrinas y neurocrinas)
a. Produce atrofia de la mucosa gástrica
b. Es producida por la célula G del cuerpo gástrico
c. Es estimulada por la distensión gástrica y el Ph bajo
d. Actúa en la célula diana mediante su receptor CCk tipo B

15. Al recibir un paciente con signos de hipovolemia y antecedente de trauma en abdomen por
accidente de tránsito, usted identifica radiológicamente: lesión de primera vértebra lumbar y signos de
lesión en páncreas; durante la cirugía se observó pobre irrigación de asas intestinales. El vaso afectado
es la arteria ……..
(unidad 1, sesión 1, logro 6: reconocer las estructuras a nivel de L1, nivel de los principales vasos
sanguíneos)
a. esplénica
b. hepática común
c. mesentérica inferior
d. mesentérica superior

16. Un paciente sufre de daño a nivel del cuello con lesión muscular en la región de la faringe. En el
examen físico se determina dificultad para la elevación de la faringe y para el cierre del itsmo de las
fauces. En este caso, probablemente esté afectado el músculo:

(unidad 2, sesión 11, logro 2: Músculos de la faringe: identificación, constrictores y longitudinales)


a. palatogloso
b. estilofarinfeo
c. palatofaringeo
d. constrictor inferior

17. Varón de 50 años a quien le realizan la curación de la segunda molar de la arcada superior derecha.
En un momento determinado, el paciente acusa de intenso dolor de la pieza dentaria en tratamiento. La
vía aferente del dolor viaja a través del nervio …………

(unidad 2, sesión 9, logro 6: Inervación de los dientes)


a. trigémino V2
b. trigémino V3
c. naso palatino
d. palatino menor

18. La distención gástrica por los alimentos produce incremento de secreción de HCl mediante la
producción de

………….. que estimula a las células ……………. vía proteína ………..


(Unidad 1, sesión 3, logro 2: Describe las hormonas gastrointestinales: Estímulo y funciones de la
gastrina y colecistoquinina)
a. gastrina / parietal / Gq
b. gastrina / principal / Gs
c. acetilcolina / parietal /Gi
d. acetilcolina / principal / Gi

19. Un niño de tres años llega a emergencia con disfagia (dificultad para tragar), dolor retro esternal,
salivación y llanto. Se sospecha de ingesta de cuerpo extraño (moneda) en el esófago; al ser evaluado se
constata en una radiografía presencia de cuerpo extraño a nivel de C6 (6° vértebra cervical). El cuerpo
extraño estará suspendido a nivel del estrechamiento producido por………..

(unidad 2, sesión 11, logro4: Esófago, características anatómicas, relación con órganos vecinos y
estrecheces)
a. el cayado aórtico
b. el hiato esofágico
c. el músculo cricofaríngeo
d. el bronquio principal izquierdo

20. La triada portal (arteria hepática, vena portal y conducto biliar común) está contenida en el
ligamento

…….……… y derivan embriológicamente del ……


(Unidad 1, sesión 1, logro 4: Identifica el peritoneo, mesenterio, omento y ligamentos, retroperitoneo.)
a. hepato duodenal / mesenterio ventral

b. gastro esplénico / mesenterio dorsal


c. hepato gástrico / omento menor
d. falciforme / omento menor

21. En relación al movimiento de peristaltismo del tubo digestivo: en la flecha negra del gráfico se
produce la liberación de ……………… a nivel del músculo ………..
(unidad 2, sesión 7, logro 6: Control hormonal y tipos de movimiento)
a. noradrenalina, sustancia P y neuropéptido “ Y” / circular
b. acetilcolina y sustancia P / longitudinal
c. óxido nítrico y PIV / longitudinal
d. óxido nítrico y PIV / circular

22. Un paciente refiere no percibir algunos sabores, al examen físico se constata alteración en la
percepción de sabores y del dolor en el tercio posterior de la lengua ¿Cuál de los siguientes nervios
estará alterada en su función?
(unidad 2, sesión 10, logro 5: Sabores, tipos y mecanismos moleculares para su detección)
a. Lingual (rama del V par)
b. Cuerda del tímpano (VII par)
c. Glosofaríngeo (IX par)
d. Hipogloso (XII par)
23. El gráfico detalla la estructura de la pared del tubo digestivo
intestinal ¿Cuál de las siguientes asociaciones es correcta?
(unidad 2, sesión 7, logro 1: La pared y músculo liso gastrointestinal )
a. “1” – peristaltismo
b. “2” – secreción enzimática
c. “3” – deriva del mesodermo
d. “4” – doble hoja de tejido graso

24. En el caso de un paciente con gastrinoma (tumor productor de gastrina), la presencia de úlceras
duodenales y erosión de la mucosa gástrica, se debe principalmente a…….
(unidad 1, sesión 3, logro 2: describir las hormonas gastrointestinales: estímulo y funciones de la gastrina
y colecistoquinina)
a. la acción directa de la gastrina sobre la célula principal
b. la sobre expresión de los receptores “G” en la célula parietal
c. el exceso de HCl por estímulo de receptores CCK-B en la célula parietal
d. el exceso de HCl por estímulo directo de receptores de acetilcolina en la célula parietal

25. El reflejo entero gástrico se caracterizan por:

(unidad 2, sesión 13, logro 6: Motilidad del intestino grueso: contracciones segmentarias, movimientos
en masa defecación y reflejo gastrocólico)
a. favorecer la motilidad gástrica gracias a la CCk
b. inhibir la motilidad gástrica y estimular la secreción ácida
c. movilizar grandes volúmenes desde el estómago al duodeno
d. originarse debido a la distensión duodenal y presencia del quimo ácido

26. Mauricio tiene dificultad para deprimir el paladar y elevar la parte posterior de la lengua. En este
caso estará afectado un músculo ………………., específicamente el músculo ……………. (Unidad 2, sesión
10, logro 2: Músculos de la lengua: clasificación, identificación y sus funciones) a. intrínseco –
longitudinal inferior
b. extrínseco – palatogloso
c. extrínseco – transverso
d. extrínseco – estilogloso

27. Una de las funciones del músculo señalado es:

(Unidad 2, sesión 8, logro 3: Describir el Piso de la boca:


estructuras blandas que la conforman)
a. deprimir la lengua
b. elevar el paladar blando
c. deprimir el hioides cuando la mandíbula está fija
d. deprimir la mandíbula cuando el hioides está fijo
28. Paciente varón de 30 años es evaluado por probable enfermedad de Chagas, cursa con problemas de
motilidad del colon; los estudios de biopsia determinan ausencia de células ganglionares. Según el
gráfico
¿cuál es la capa en la que se determina la ausencia de dichas células?
(unidad 1, sesión 2, logro 1: describir las generalidades de la estructura del tubo digestivo: esófago,
estómago intestino delgado y grueso)
a. Mucosa - 1
b. Muscular propia – 1
c. Muscular de la mucosa - 2
d. Muscular propia - 3

29. Paciente varón de 32 años, que acude a centro de salud por presentar de forma progresiva desde
hace 1 año dificultad para ingerir alimentos sólidos y luego líquidos; refiere regurgitaciones alimentarias
y marcada pérdida de peso (15 kilos). Radiografía baritada (sustancia de
contraste) de esófago se muestra en la figura. El presente caso se explica
por……………….
(Unidad 2, sesión 12, logro 4: Identificar y describir la función de los esfínteres
esofágicos)
a. aumento de la peristalsis esofágica
b. relajación incompleta del esfínter pilórico
c. relajación incompleta del esfínter esofágico inferior
d. perdida de producción de PIV y óxido nítrico en el esfínter esofágico
superior

30. En condiciones normales, el ingreso de 600 ml de líquido es el estómago provoca un aumento de


presión intragástrica de unos 12 cm de H2O. Después de una vagotomía (corte del nervio vago) es de
esperar que el ingreso del mismo volumen de líquido ocasionede la presión intragástrica. (Unidad 2,
sesión 13, logro 1: Describe la Motilidad gástrica: relajación receptiva) a. la disminución
b. la no variación
c. un aumento mayor
d. un aumento similar o igual
SISTEMA DIGESTIVO (ME 154) CLAVES EXAMEN FINAL
Ciclo 201900

1. Un niño de 2 años es llevado a la consulta por diarrea persistente, edema de las extremidades y falta
de crecimiento en relación a su edad. Los análisis de sangre revelan que tiene concentración plasmática
baja de proteínas (hipoproteinemia). Como parte del estudio se coloca colecistokinina (CCK) endovenosa
y se recoge muestras del líquido duodenal por endoscopía; el resultado del líquido confirma incapacidad
para hidrolizar proteínas a un pH neutro, esta situación mejora al añadir una pequeña cantidad de
tripsina. El paciente probablemente esté sufriendo la falta congénita de …………. (Unidad 4, sesión 26,
logro 2: Explicar la Digestión y absorción de las proteínas y sus alteraciones) a. PEPT-1
b. pepsinógeno
c. enterocinasa
d. carboxipeptidasas

2. Paciente mujer de 35 años acude a consulta por sensación de sequedad y lesiones en cavidad oral. Al
examen se observa atrofia de la mucosa, fisuras y úlceras; nota además sequedad e irritación de la
córnea y aumento del tamaño de las glándulas parotídas. Su diagnóstico más probable es artritis
reumatoide; el hallazgo más probable en una biopsia de glándula parótida es……..….

(Unidad 3, sesión 18, logro 3: Describe las enfermedades más frecuentes de las glándulas salivales)
a. Presencia de acinos normales con hiperplasia de células ductales
b. Gran infiltración de linfocitos y células plasmáticas
c. Hiperplasia de acinos glandulares serosos
d. Gran infiltrado de linfocitos y neutrófilos

3. Un hombre de 42 años de edad se presenta al médico con una historia de 1 año de evolución,
caracterizado por dolor abdominal bajo y diarreas con crisis sanguinolentas. Manifiesta además pérdida
de peso de 8 kg durante este periodo. La colonoscopía revela lesión difusa en el colon con afectación del
recto. La biopsia de estas lesiones revela adelgazamiento de la pared, inflamación y ulceración de la
mucosa y sub mucosa. El diagnóstico más probable en este caso es:

(Unidad 4, sesión 28, logro 5: Describe la Enfermedad inflamatoria intestinal. Generalidades, morfología
y características)
a. sindrome de colon irritable
b. enfermedad de Crohn
c. colitis ulcerativa
d. sprue celiaco

4. Dos estudiantes deciden tomar un receso para comer una hamburguesa a la hora del almuerzo. Antes
de llegar a la cafetería, impulsos nerviosos provenientes del complejo vagal dorsal iniciarán la secreción
de ácido gástrico por la liberación dedesde el sistema nervioso entérico.

(Unidad 3, sesión 20, logro 2: Regulación de la secreción gástrica: estimulación, fases de la secreción)
a. Serotonina
b. Colecistoquinina
c. Péptido inhibidor vaso activo
d. GRP (péptido liberador de gastrina)

5. Un niño de cuatro años de edad es llevado a la consulta por cuadros diarreicos frecuentes
caracterizados por heces pálidas, voluminosas y fétidas; al examen físico presenta bajo peso y talla para
la edad. Se mide la concentración de cloruro en el sudor y se encuentra que sus valores son muy
elevados. La alteración más

importante a nivel de células ductales del páncreas tiene relación directa con la conductancia de…………
(Unidad 3, sesión 23, logro 5 Explica la Secreción pancreática: formación del jugo pancreático, influencia
de la velocidad de flujo y regulación)
a. Bicarbonato
b. Potasio
c. Sodio
d. Cloro

6. Se evalúa los valores séricos de las siguientes sustancias a un paciente con enfermedad hepática
terminal; en este paciente se espera encontrar la combinación con la letra ………… (Unidad 3,
sesión 22, logro 5: Describe las Pruebas de función hepática, la Insuficiencia hepática,
encefalopatía hepática e hipertensión portal)

Glucosa Amoniaco Albúmina


a. Aumentada Disminuida Disminuida
b. Disminuida Aumentada Aumentada
c. Aumentada Aumentada Aumentada
d. Disminuida Aumentada Disminuida

7. Una mujer de 35 años de edad HIV positiva, se presenta al médico con dolor abdominal en cuadrante
superior derecho e ictericia. La paciente refiere haber tenido múltiples episodios de ictericia durante los
últimos 10 años. Los exámenes para determinar hepatitis viral, dieron positivos para Hepatitis B, siendo
catalogado el caso como hepatitis crónica con alteración funcional. En un examen de sangre ¿cuál de los
siguientes parámetros está disminuido?
(unidad 3, sesión 22, logro 5: Pruebas de función hepática, Insuficiencia hepática, encefalopatía hepática
e hipertensión portal)
a. Albumina
b. Bilirrubina
c. Fosfatasa alcalina
d. Tiempo de protrombina

8. En el reflejo peristáltico del intestino delgado ¿Cuál de los siguientes eventos sucede en la porción
caudal del bolo alimenticio?

(Unidad 2, sesión 13, logro 4: Explicar la Motilidad del intestino delgado: Contracciones segmentarias y
peristálticas)
a. Acción del péptido inhibidor vasoactivo (VIP) en el músculo circular
b. Acción del NO (óxido nítrico) en el músculo longitudinal
c. Contracción del músculo longitudinal interno
d. Acción de acetilcolina en el músculo circular

9. Un varón de 58 años de edad con enfermedad de Crohn severo fue sometido a una resección ileal.
Después de la cirugía este paciente padecerá de esteatorrea, esto se explica porque …..………..

(unidad 4, sesión 26, logro 4: Explica las alteraciones en la Absorción de lípidos)


a. se inhibe la acción de la 7 alfa hidroxilasa
b. el pool de ácidos biliares se incrementa
c. hay mala absorción de ácidos biliares
d. el páncreas no secreta lipasa

10. En un experimento se inserta un balón en el estómago de un voluntario, se infla poco a poco


mientras que se vigilan las presiones intraluminales. Aunque el volumen del balón aumenta
considerablemente, las presiones permanecen constantes. Esta relación volumen-presión se explica por
la liberación local de …………..

(Unidad 2, sesión 13, logro 1 Explica la Motilidad gástrica: relajación receptiva)


a. acetil colina y gastrina
b. norepinefrina y óxido nítrico
c. colecistoquinina y óxido nítrico
d. óxido nítrico y péptido inhibidor vasoactivo

11. ¿Cuál de las siguientes alternativas es una característica de la secreción exocrina del páncreas?

(Unidad 3, sesión 23, logro 5: Secreción pancreática: formación del jugo pancreático, influencia de la
velocidad de flujo y regulación)
a. Es hipotónica respecto al plasma
b. Su mayor estímulo se da en la fase intestinal
c. Es estimulada por la presencia de bicarbonato en el duodeno
d. La secreción enzimática es estimulada principalmente por la secretina

12. Las estructuras en el hígado que permite que los productos metabólicos unidos a proteínas tengan
acceso a las membranas basolaterales de los hepatocitos, son…..
(Unidad 3, sesión 21, logro 4-5: Explica la Organización micro estructural del hígado)
a. los canalículos
b. las células de Ito
c. las fenestras sinusoidales
d. las uniones intercelulares herméticas

13. La composición de la bilis es modificada conforme fluye por los conductillos biliares. Durante este
tránsito se espera que aumente la concentración de…….

(Unidad 3, sesión 22, logro 2: Describe la Secreción biliar, visión general del sistema biliar extrahepático y
composición de la bilis)
a. Ig A
b. Glucosa
c. Protones
d. Vitamina A

14. Se mide experimentalmente el contenido gástrico de dos personas. La persona “A” tiene alto
contenido de grasa y la persona “B” tiene un contenido isotónico ¿Cuál de las siguientes es correcto
respecto al vaciamiento gástrico? (Unidad 2, sesión 13, logro 2: Describe la Motilidad y vaciamiento
gástrico)
a. Hay ralentización del vaciado gástrico solo en “A”
b. El vaciamiento gástrico es más rápido en ambos
c. Hay ralentización del vaciado gástrico solo en “B”
d. Hay ralentización del vaciado gástrico en ambos casos

15. El examen endoscópico de un paciente con hipertensión portal grave revela venas tortuosas que
sobresalen hacia la luz del esófago. El paciente recibe tratamiento quirúrgico mediante la colocación de
una derivación que conecta la vena porta a la vena cava. Después de la operación el riesgo de
encefalopatíay el riesgo

de sangrado de várices ……………..


(Unidad 3, sesión 22, logro 5: Describe la Insuficiencia hepática, encefalopatía hepática e hipertensión
portal)
a. disminuirá / disminuirá
b. disminuirá / aumentará
c. aumentará / disminuirá
d. aumentará / aumentará
16. Un paciente varón de 18 años de edad acude al médico para sus exámenes de rutina. Sus resultados
de laboratorio muestran un valor de bilirrubina sérica de 4 mg/dl y una bilirrubina directa de 0,3 mg/dl.
Las pruebas de función hepática son normales. La alteración que explica mejor este caso es por la
deficiencia de ………………..

(Unidad 3, sesión 22, logro 3: Explica la Producción y excreción de bilirrubina. Tipos de bilirrubina e
ictericia)
a. transaminasas
b. hemo oxigenasa
c. la 7 alfa hidroxilasa
d. glucuronil transferasa

17. Un hombre de 57 años de edad es llevado a urgencias con hematemesis masiva rojo brillante, a su
llegada se halla inconsciente con PA: 80/40 mm Hg y FC: 124 lat/min. Luce ictérico con presencia de
“arañas vasculares en el tórax anterior y extremidades”, abdomen distendido con signo de oleada
positiva. Se encuentra esplenomegalia y pérdida de la masa muscular en extremidades. La anastomosis
vascular responsable del sangrado en este paciente es ………….…..

(Unidad 3, sesión 21, logro 2: Describe las anastomosis porto sistémicas)


a. vena gástrica izquierda y vena ácigos
b. arteria gástrica izquierda y vena ácigos
c. vena paraumbilical y vena epigástrica inferior
d. vena gástrica izquierda y vena esofágica superior
18. Un estudiante de medicina está comiendo un plato de comida a base de champiñones, espárrago y
salsa de soya. El estímulo del sabor umami contenido en todos estos alimentos viaja a través del
nervio………………..

(Unidad 2, sesión 10, logro 3: Describe la irrigación e inervación de la lengua)


a. Lingual
b. Hipogloso
c. Glosofaringeo
d. Cuerda del tímpano

19. Una paciente de 30 años de edad es sometida a una cirugía en oído medio derecho por un problema
de otoesclerosis. Luego de la cirugía refiere alteración sensitiva de la lengua. Al evaluar el caso usted
esperaría encontrar……….
(Unidad 2, sesión 10, logro 5: Describe la Irrigación e inervación de la lengua)
a. Alteración en la sensación del dolor y temperatura en el tercio posterior de la lengua
b. Alteración en la sensación del dolor en los dos tercios anteriores de la lengua c.
Alteración en la sensación del gusto en el tercio posterior de la lengua
d. Sensación del dolor, tacto y temperatura conservadas

20. En un paciente de 45 años de edad con colestasis biliar, se encuentra una elevación de los niveles
sanguíneos de fosfatasa alcalina hasta 3 veces la cifra normal. ¿Cuál de las siguientes alternativas estará
también elevada como evidencia del daño de la vía biliar?

(Unidad 3, sesión 22, logro 5: Pruebas de función hepática, Insuficiencia hepática, encefalopatía hepática
e hipertensión portal)
a. Tiempo de protrombina y albúmina sérica
b. Transaminasas hepáticas (ALT y AST)
c. Gamma glutamil transpeptidasa
d. Glucoronil transferasa

21. Experimentalmente se incrementa la velocidad de la secreción salival con una sustancia, en el


análisis de la composición de esta saliva obtenida se espera encontrar…………..

(Unidad 3, sesión 17, logro 5 : Explica la Influencia de la velocidad del flujo salival en la composición de la
saliva)
a. disminución de la concentración de bicarbonato que supera la concentración plasmática
b. aumento de la concentración de cloro y sodio que supera la concentración plasmática c.
aumento de la concentración de bicarbonato que supera la concentración plasmática d.
disminución de concentración de potasio y bicarbonato

22. Lactante de 3 meses de vida es atendido por presentar diarrea, se administra una solución de
glucosa y electrólitos por vía oral. La proteína de membrana apical que explica la capacidad de esta
solución para proporcionar aporte de glucosa e hidratación es ………..

(Unidad 4, sesión 26, logro 1: Explica la Digestión y Absorción de los hidratos de carbono. Alteraciones)
a. CFTR
b. SGLT-1
c. GLUT-2
d. GLUT-5

23. Paciente ha sufrido herida de bala en el abdomen, se le ha tenido que extirpar el segmento medio y
distal del ileon. En este caso la síntesis hepática de sales biliares estará …..…..

(Unidad 3, sesión 22, logro 4: Explica la formación, función y Circulación entero hepática de lasa sales
biliares)
a. Sin cambios en el ritmo de síntesis
b. Disminuida por inhibición de la enzima colesterol 7 alfa hidroxilasa
c. Incrementada por estímulo de la enzima colesterol 7 alfa hidroxilasa
d. Incrementada por inhibición de la enzima colesterol 7 alfa hidroxilasa
24. Un varón de 75 años ingresa al consultorio por presentar ictericia marcada de piel y las escleras. El
estudio del paciente mostró que presentaba un tumor que obstruía la totalidad del conducto hepático
común. ¿Cuál de los siguientes conductos se encontrará dilatado en este paciente?

(Unidad 3, sesión 21, logro 6: Describir el árbol biliar intrahepático)


a. de Wirsung
b. de Hering
c. colédoco
d. cístico

25. Correlaciones las dos columnas y marque la fórmula correcta:


(Unidad 4, sesión 28, logro 1: Diarrea: definición, mecanismos: osmótica, secretoria y exudativa)
1. Enfermedad Hirschsprung( ) heces con moco y sangre
2. Diarrea osmótica( ) intolerancia a lactosa
3. Diarrea secretoria( ) aganglionosis congénita
4. Diarrea exudativa( ) canales de Cl- en las células de la cripta

a.- 4231
b.- 1234
c.- 2143
d.- 4213
26. La vena umbilical obliterada del hígado después del nacimiento se transforma en el ligamento:

(Unidad 3, sesión 21, logro 1: Hígado: relación con la pared abdominal, caras, lóbulos, ligamentos , hilio
hepático)
a. cruzado
b. redondo
c. coronario
d. falciforme

27. Llega a su guardia nocturna una madre que trae a su RN masculino de 2 semanas de vida con mal
estado general y sequedad de mucosas. Usted observa que lacta ávidamente, pero a las 2 horas
presenta vómito postprandial no bilioso en proyectil. Al realizar la historia clínica, descubre que el
lactante recibió profilaxis con macrólidos para tos ferina. Usted sospecha principalmente en:

(Unidad 1, sesión 4, logro 4: Desarrollo y anomalías del intestino anterior)


a. estenosis pilórica hipertrófica congénita
b. fistula traqueo esofágica
c. estenosis duodenal
d. atresia duodenal

28. En la regulación del apetito y la saciedad, la estimulación experimental crónica del núcleo ventro
medial del hipotálamo producirá:

(Unidad 1, sesión 3, logro 4: explica los mecanismos de control del apetito y saciedad)
a. afagia
b. obesidad
c. hiperfagia
d. activación de neuronas relacionadas a NPY

29. Paciente mujer de 25 años acude por dolor en fosa ilíaca derecha que empeora al toser o caminar,
asociada a náuseas y vómitos por lo cual acude a emergencia. Dos días después de realizarle una
apendicectomía, la paciente desarrolla fiebre alta (39 °C), está hipotensa y presenta dolor abdominal. La
laparotomía exploratoria muestra un gran volumen de sangre en la cavidad peritoneal por lesión de un
vaso producida durante la apendicectomía.
¿Cuál de las siguientes arterias debe ligarse para detener la hemorragia?
(Unidad 4, sesión 27, logro 4: Irrigación arterial del colon, recto y conducto anal)
a. cólica derecha y arteria rectal superior.
b. ileocólica y arteria cólica media.
c. mesentérica superior.
d. ileocólica.

30. La onda peristáltica secundaria del esófago se caracteriza por ser originada ………

(unidad 2, sesión 12, logro 3: Motilidad esofágica: fases y características)


a. por el plexo de meissner del esófago
b. por el plexo mientérico del esófago
c. por el reflejo de la deglución
d. durante la masticación

31. ¿Cuál de los siguientes es una causa de ictericia con bilirrubina conjugada aumentada?

(Unidad 3, sesión 22, logro 3: Producción y excreción de bilirrubina. Tipos de bilirrubina, ictericia)
a. Ictericia del recién nacido
b. Obstrucción del colédoco
c. Anemia hemolítica
d. Gran hematoma

32. En relación a la absorción de nutrientes, la absorción de dipéptidos y tripéptidos a nivel de las células
epiteliales del intestino delgado, se da principalmente debido a:
(Unidad 4, sesión 26, logro 2: Digestión y absorción de las proteínas. Alteraciones)
a. el incremento de los canales de Cl- en la membrana apical
b. la gradiente de bicarbonato en la membrana basal
c. la gradiente de iones H+ en la membrana apical
d. la gradiente de Na+ en la membrana apical

33. Paciente de 20 años es traído a la emergencia por presentar diarreas desde hace 2 días. Familiar
refiere que las deposiciones son líquidas y abundantes, al examen luce deshidratado y se plantea que la
diarrea es producida por una toxina que estimula la transformación de ATP a AMPc con apertura de
canales de Cl- y pérdida de agua. El tipo de diarrea más probable es:
(Unidad 4, sesión 28, logro 1: Diarrea: definición , mecanismos: osmótica, secretoria y supurativa)
a. osmótica
b. exudativa
c. secretoria
d. por intolerancia a lactosa

34. Un niño fue operado por una obstrucción intestinal, observándose la presencia de divertículo de
Meckel. Según lo referido, marque lo correcto:

(Unidad 1, sesión 5, logro 2: identifica las anomalías del desarrollo del intestino medio: onfalocele y
gastrosquisis (diferencias), Divertículo de Meckel)
a. el 50% de la población lo presenta
b. se localiza en el íleon muy cerca al yeyuno
c. puede poseer tejido gástrico o pancreático
d. se produce por una mala rotación de los intestinos

35. Marque la alternativa correcta respecto a la estructura marcada en el gráfico:


(Unidad 3, sesión 22, logro 2: Secreción biliar. Visión general del sistema biliar extrahepático y
composición de la bilis)
a. Se halla a 2 centímetros debajo de la papila duodenal mayor
b. Llega el conducto colédoco y pancreático principal
c. Llega el conducto hepático común y pancreático principal
d. Llega el conducto pancreático accesorio

36. ¿Cuál de las siguientes moléculas se encontrará aumentada en el citoplasma de las células parietales
de un paciente con sindrome de Zollinguer Ellison?

(Unidad 3, sesión 20, logro 4: Enfermedad ulcerosa péptica: úlcera gástrica, duodenal. síndrome de
Zollinger – Ellison)
a. Péptido liberador de gastrina (GRP)
b. Proteína G estimulante (GS)
c. Inositol Trifosfato (IP3)
d. AMP cíclico (AMPc)

37. Los fármacos inhibidores de la bomba de protones, actúan bloqueando la ………..……..

(Unidad 3, sesión 20, logro 3: Regulación de la secreción gástrica: inhibición, Secreción de pepsinógeno y
factor intrínseco)
a. anhidrasa carbónica
b. ATPasa H+/K+ en la membrana luminal
c. ATPasa H+/K+ en la membrana basolateral
d. ATPasa Na+/K+ en la membrana basolateral

38. Un paciente fue diagnosticado de gastritis autoinmune, ¿cuál de las siguientes alternativas es FALSA
respecto a esta enfermedad?
(Unidad 3, sesión 20, logro 5: Gastritis crónica: helicobacter pylori, autoinmune. Tipos de gastritis)
a. Afecta principalmente el fondo y cuerpo gástrico
b. Se produce hiperplasia de células G secundaria a la aclorhidria
c. El propio sistema inmune destruye principalmente las células parietales
d. Se produce atrofia de la mucosa, aclorhidria, hipergastrinemia y déficit de vitamina

B6 39. Marque la correlación correcta:

(Unidad 3, sesión:18, logros:1 y 2: Describe las enfermedades inflamatorias/infecciosas y proliferativas


de la cavidad oral)
1. Herpes virus( ) En relación al abuso de antibióticos
2. Candidiasis oral( ) Lesiones vesiculares como racimo de uvas
3. Eritroplaquia( ) Mega esófago
4. Enfermedad de Chagas( ) Lesión pre cancerígena

a.- 2431
b.- 1234
c.- 4123
d.- 2143
40. En un paciente con insuficiencia renal crónica, el déficit en la absorción de calcio a nivel del
enterocito se debe a lo siguiente:

(Unidad 4, sesión 26, logro 6: Explica la Absorción de calcio y hierro)


a. No se convierte la 25 hidroxicolecalciferol a 1,25 dihidroxicolecalciferol
b. No se convierte la 1,25 dihidroxicolecalciferol a 25 hidroxicolecalciferol
c. Existe un descenso de la alfa 25 hidroxilasa renal
d. Se incrementa la producción de Calbindina
SISTEMA DIGESTIVO (ME154) EXAMEN FINAL 2019 01

1. La explicación fisiológica de presentar somnolencia de 30 minutos a 1 hora después de ingerir


alimentos, se explica por:
a. Aumento del cloro intraluminal
b. Aumento del bicarbonato intraluminal
c. Disminución de ácido carbónico en la célula parietal
d. Disminución de la actividad de la anhidrasa carbónica
e. Aumento de la alcalinidad sanguínea

2. Con respecto a la irrigación arterial del colon, a que arteria corresponde la señalada con la flecha
a. Cólica derecha
b. Cólica media
c. Cólica izquierda
d. Ileobisecoapendículocólica
e. Arco de Riolano

3. Si un paciente presentara dentro del punto de vista fisiológico, una disminución de enterocinasa,
entonces esto originaría una disminución de la actividad de:
a. la pepsina
b.la lipasa
c. la quimotripsina
d. el peptido insulinotropo dependiente de glucosa
e. la amilasa
Se valida la opción b debido a su relación con la colipasa.

4. Con respecto a la anatomía del hígado, señale a que estructura pertenece la marcada por el número
1.
a. Ligamento falciforme
b. Línea de Cantlie
c. Ligamento triangular
d. Ligamento coronario
e. Ligamento teres
Se valida la opción a debido a la ubicación del número 1 en
donde se unen el ligamento falciforme y ligamento
coronario.
5. Se presenta un paciente, el cual presenta un antecedente de tuberculosis intestinal, por lo cual, se le
resecó 80 cm de íleon distal. Desde el punto de vista fisiológico, el paciente puede presentar una de las
siguientes alteraciones:
a. Disminución de la secreción de Vitamina B12
b. Aumento indiscriminado de absorción de ácido fólico
c. Disminución de la absorción de hierro
d. Aumento de la secreción de bicarbonato
e. Disminución de la absorción de ácido glicocólico

6. Un paciente es sometido experimentalmente a un fármaco que modifica el flujo salival, obteniéndose


un volumen de saliva de 288 ml en 6 horas. En este caso las concentraciones de electrolitos y
bicarbonato en la saliva obtenida varían de la siguiente manera:
a.↑ Na+, ↓ K+, ↑ Cl-, ↑ HCO3-
b.↓ Na+, ↓ Cl-, ↑ K+, ↓ HCO3-
c.↑ Na+, ↑ Cl-, ↓ K+, ↓ HCO3-
d.↑ Na+, ↑ Cl-, ↑ K+, ↑ HCO3-
e.↓ Na+, ↓ Cl-, ↓ K+, ↓ HCO3-
Se valida la opción a debido a que se puede considerar como un aumento del flujo de

saliva. 7. La siguiente imagen histológica corresponde a la glándula

…………… y la estructura señalada produce ………


a. salival sublingual / mucopolisacáridos
b. oxíntica / pepsinógeno
c. salival submaxilar / ptialina
d. salival parótida / amilasas
e. antrales / gastrina

8. Paciente varón de 65 años con antecedente de hipercolesterolemia, hipertensión arterial, fibrilación


auricular y dos infartos al miocardio previos, aqueja de dolor abdominal intenso de inicio súbito,
distensión abdominal, se decide cirugía con resección de 1,5 metros de intestino delgado terminal y
colon ascendente. Como consecuencia de la resección el paciente tendrá deficiencia de: a. Vitamina C
b. Tiamina
c. Vitamina A
d. Vitamina B1
e. Vitamina B6
Se valida esta opción debido a que su absorción está relacionada al íleon.

9. Uno de los siguientes elementos debería hallarse con más probabilidad en el esófago de un paciente
que sufre de
reflujo gastro esofágico…
a. Pepsina
b. Tripsina
c. Quimiotripsina
d. Carboxipeptidasa
e. Ácidos biliares

10. Un paciente de 40 años cursa con anemia de 8g/dl, aqueja además de astenia y sensación de
hormigueo bilateral en los miembros inferiores, al examen se halla alteración de la sensibilidad a la
vibración y camina con ampliación de la base de sustentación. Uno de los siguientes procedimientos
sería de ayuda para el diagnóstico de este paciente:
a. Tomografía cerebral
b. Biopsia de la mucosa gástrica
1. Un niño de 2 años es llevado a la consulta por diarrea persistente y edema de las
extremidades, además falta de crecimiento y desarrollo en relación a su edad. Los
análisis de sangre revelan que tiene concentración plasmática baja de proteínas
(hipoproteinemia). Durante la endoscopía duodenal, se coloca colecistocinina (CCK)
endovenosa y se recoge muestras del líquido duodenal; el resultado del líquido
confirma incapacidad para hidrolizar proteínas a un pH neutro, esta situación mejora
al añadir una pequeña cantidad de tripsina. El paciente probablemente esté
sufriendo la falta congénita de
​-Enterocinasa
2. Experimentalmente se incrementa la velocidad de la secreción salival con una
sustancia, el análisis de la composición de esta saliva obtenida se espera
encontrar…………..
-Disminución de concentración de potasio
3. Paciente varón de 46 años soltero, consulta por odinofagia y bajo de peso, tiene
antecedente de tuberculosis desde hace 3 meses y es fumador crónico (10
cigarrillos por día); al evaluar la cavidad oral se identifica lesión blanquecina en el
dorso de la lengua y paladar blando, las lesiones se desprenden con el baja lengua
dejando una base eritematosa. Esta lesión corresponde probablemente a
……………………….…..
-​ Candidiasis oral
4. Paciente mujer de 35 años acude a consulta por sensación de sequedad y lesiones
en cavidad oral. Al examen se observa atrofia de la mucosa, fisuras y úlceras; nota
además sequedad e irritación de la córnea y aumento del tamaño de las glándulas
parotídeas. Su diagnóstico más probable es artritis reumatoide; el hallazgo más
probable en una biopsia de glándula parótida es……..….
​-Gran infiltración de linfocitos y células plasmáticas
5. Un paciente con anemia acude con su médico quejándose de episodios frecuentes
de gastroenteritis. Un análisis de sangre revela anticuerpos circulantes dirigidos
contra células parietales gástricas. Su anemia es atribuible a la hiposecreción de
​-Factor intrínseco
6. Dos estudiantes deciden tomar un receso para comer una hamburguesa a la hora
del almuerzo. Antes de llegar a la cafetería, impulsos nerviosos provenientes del
complejo vagal dorsal iniciarán la secreción de ácido gástrico por la liberación de
…………………….. desde el sistema nervioso entérico.
-GRP
7. Un niño de cuatro años de edad es llevado a la consulta por cuadros diarreicos
frecuentes caracterizados por heces pálidas, voluminosas y fétidas, presenta bajo
peso y talla. Se mide la concentración de cloruro en el sudor y se encuentra que sus
valores son muy elevados. La alteración más importante a nivel de células ductales
del páncreas tiene relación directa con la conductancia de…………
​-Cloro
8. Una mujer de 50 años de edad que sufrió durante varios años resequedad de los
ojos debida a producción inadecuada de lágrimas es enviada con un
gastroenterólogo para evaluación de pirosis crónica. El examen endoscópico
revela erosiones y tejido cicatrizal en la parte distal del esófago justo por arriba
del esfínter esofágico inferior. Las lesiones pueden atribuirse a la disminución de
uno de los siguientes componentes salivales:
​-Bicarbonato
9. Se evalúa los valores séricos de las siguientes sustancias a un paciente con
enfermedad hepática terminal; en este paciente se espera encontrar la combinación
con la letra …………
​-disminuida, aumentada, disminuida
10. Una mujer de 35 años de edad HIV positiva, se presenta al médico con dolor
abdominal en cuadrante superior derecho e ictericia. La paciente refiere haber tenido
múltiples episodios de ictericia durante los últimos 10 años. Los exámenes para
determinar hepatitis viral, dieron positivos para Hepatitis B, siendo catalogado el caso
como hepatitis crónica con alteración funcional. En un examen de sangre ¿cuál de los
siguientes parámetros está disminuido?
​-Albúmina
11. En el reflejo peristáltico del intestino delgado, uno de los siguientes eventos sucede
en la porción oral del bolo alimenticio…………...
​-Acción de acetilcolina en el músculo circular
12. Experimentalmente se coloca una dosis alta de secretina en la luz intestinal
duodenal; como consecuencia de esto, en el jugo pancreático de la misma luz
intestinal se observa la disminución de la concentración de …..………..
​-Cl
13. Un varón de 58 años de edad con enfermedad de Crohn severo fue sometido a una
resección ileal. Después de la cirugía este paciente padecerá de esteatorrea, esto se
explica porque …..………..
- ​La micelas no pueden formarse
14. En un experimento se inserta un balón en el estómago de un voluntario, se infla poco
a poco mientras que se vigilan las presiones intraluminales. Aunque el volumen del
balón aumenta considerablemente, las presiones permanecen constantes. Esta
relación volumen-presión se explica por la liberación local de …………..
​-Óxido nítrico y péptido inhibidor vasoactivo
15. La toxina de Vibrio cholerae causa diarrea debido a…….
​-El Incremento de la secreción de cloro por las células de la cripta intestinal
16. ¿Cuál de las siguientes alternativas es una característica de la secreción exocrina
del páncreas?
​-Tiene una baja concentración de Cl- respecto al plasma
17. Una madre lleva a su hijo de dos años de edad a la sala de urgencias, estresada
porque el niño deglutió una moneda de 10 céntimos mientras la familia cenaba en un
restaurante. El médico observa mediante fluoroscopía que la moneda se halla en el
estómago y asegura a la madre que la moneda se eliminará con las heces. El médico
recomienda utilizar la respuesta fisiológica que permitirá la evacuación de la moneda
del estómago al intestino ………….…..
​-Son los movimientos de mezcla y trituración
-​ . Es provocada por el ayuno
18. Las estructuras en el hígado que permite que los productos metabólicos unidos a
proteínas tengan acceso a las membranas basolaterales de los hepatocitos, son…..
-Las fenestras sinusoidales
19. La composición de la bilis es modificada conforme fluye por los conductillos biliares.
Durante este tránsito se espera que aumente la concentración de…….
​-Monómeros de ácido biliar
-Ig A
20. Se mide experimentalmente el contenido gástrico de dos personas. La persona “A”
tiene alto contenido de grasa y la persona “B” tiene un contenido hipertónico ¿Cuál
de las siguientes es correcto respecto al vaciamiento gástrico?
- Hay ralentización del vaciado gástrico en ambos casos
21. El examen endoscópico de un paciente con hipertensión portal grave revela venas
tortuosas que sobresalen hacia la luz del esófago. El paciente recibe tratamiento
quirúrgico mediante la colocación de una derivación que conecta la vena porta a la
vena cava. Después de la operación el riesgo de encefalopatía y el
riesgo de sangrado de várices ……………..
​-Aumentará/disminuirá
22. Un paciente varón de 18 años de edad acude al médico para sus exámenes de
rutina. Sus resultados de laboratorio muestran un valor de bilirrubina sérica de 4
mg/dl y una bilirrubina directa de 0,3 mg/dl. Las pruebas de función hepática son
normales. La alteración que explica mejor este caso es por la deficiencia de
………………..
-Glucuronil transferasa
23. Un hombre de 57 años de edad es llevado a urgencias con hematemesis masiva
rojo brillante, a su llegada se halla inconciente con PA: 80/40 mm Hg y FC: 124
lat/min. Luce ictérico con presencia de “arañas vasculares en el tórax anterior y
extremidades”, abdomen distendido con signo de oleada positiva. Se encuentra
esplenomegalia y pérdida de la masa muscular en extremidades. La anastomosis
vascular responsable del sangrado en este paciente es
​-Vena gástrica izquierda y vena ácigos
24. Un estudiante de medicina está comiendo un plato de comida a base de
champiñones, espárrago y salsa de soya. El sabor umami contenido en todos estos
alimentos actúa a nivel de los botones gustativos estimulando ………………..
​-Un receptor acoplado a proteína G
25. Un hombre de 22 años de edad se presenta al médico con una historia de 1 año de
evolución caracterizado por dolor recurrente en fosa iliaca derecha y diarrea.
Manifiesta además pérdida de peso de 8 kg durante este periodo. La colonoscopía
revela múltiples lesiones en el ileon terminal y colon. La biopsia de estas lesiones
revela engrosamiento, inflamación y ulceración de la mucosa. El diagnóstico más
probable en este caso es…….
​-Enfermedad de Crohn
26. Varón de 61 años que consulta por dolor retro esternal intenso desde hace 6 horas y
después de vómitos intensos y repetidos; al examen se observa disnea, cianosis,
hipotensión y signos clínicos de shock. La radiografía simple de tórax muestra
neumomediastino. El líquido en el espacio pleural aspirado tiene alta concentración
de amilasa. ¿Cuál de las siguientes alternativas puede explicar este cuadro clínico?
-​Rotura espontánea de esófago
27. La secreción del ácido en la célula parietal gástrica se lleva a cabo por una ATPasa
especifica que intercambia hidrogeniones (H+) del citosol por…..
​-K +
28. En condiciones normales el ingreso de 600 ml de líquido es el estómago provoca un
aumento de presión intragástrica de unos 12 cm de H2O. Después de una
vagotomía (corte del nervio vago) es de esperar que el ingreso del mismo volumen
de líquido provoque lo siguiente: …………………………………
​-Un aumento mayor de la presión
29. Una paciente de 30 años de edad es sometida a una cirugía en oído medio derecho
por un problema de otoesclerosis. Luego de la cirugía refiere alteración en la
percepción de sabores. Al evaluar el caso usted esperaría encontrar……….
​-Alteración en la sensación del gusto en los dos tercios anteriores de la
l​ engua
​-Sensación del dolor, tacto y temperatura conservada en toda la lengua
30. ¿Cuál de las siguientes alternativas es correcta?
​-Las sales biliares desconjugadas son absorbidas preferentemente en el colon
31. En un paciente de 45 años de edad con colestasis biliar, se encuentra una elevación
de los niveles sanguíneos de fosfatasa alcalina hasta 3 veces la cifra normal. ¿Cuál
de las siguientes alternativas estará también elevada como evidencia del daño de la
vía biliar?
​-Gamma glutamil transpeptidasa
32. Revisando la angiografía de un hombre de 70 años en estudio por aneurisma de
aorta abdominal el radiólogo informa de la presencia de una oclusión completa de la
arteria mesentérica inferior. El paciente se encuentra completamente asintomático.
¿Cuál de las siguientes arterias se anastomosa a la sistema arterial de la
mesentérica inferior?
​-Cólica media
33. Lactante de 3 meses de vida es atendido por presentar diarrea, se administra una
solución de glucosa y electrólitos por vía oral. La proteína de membrana apical que
explica la capacidad de esta solución para proporcionar aporte de glucosa e
hidratación es ………..
​-SGLT-1
34. Paciente ha sufrido herida de bala en el abdomen, se le ha tenido que extirpar el
segmento medio y distal del ileon. En este caso la síntesis hepática de sales biliares
estará …..…..
​-Incrementada por estímulo de la enzima colesterol 7 alfa hidroxilasa
35. Un varón de 75 años ingresa al consultorio por presentar ictericia marcada de piel y
las escleras. El estudio del paciente mostró que presentaba un tumor que obstruía la
totalidad del conducto hepático común. ¿Cuál de las siguientes estructuras se
encontrará dilatada en este paciente?
​-Conductos de Hering
36. En un paciente con insuficiencia renal crónica, el déficit en la absorción de calcio a
nivel del enterocito se debe a lo siguiente:
​-No se convierte la 25 hidroxicolecalciferol a 1,25 dihidroxicolecalciferol
37. Varón de 30 años es traído a emergencia por agresión abdominal con arma de fuego
(pistola) y es sometido a laparotomía exploratoria, observándose isquemia del colon
ascendente y parte del colon trasverso ¿la lesión de cuál de las siguientes arterias
explicaría esta isquemia?
d. Mesentérica superior
38. Respecto a las sustancias gastrointestinales que regulan la secreción pancreática;
marque la afirmación correcta:
b. La acetilcolina es capaz de estimular la secreción enzimática y de bicarbonato del
páncreas
39. Ante una lesión del X par craneal, ¿cuál de los siguientes músculos mantiene
conservada su función?:
b. Tensor del velo del paladar
40. Experimentalmente se utiliza atropina (anticolinérgico) para inhibir la secreción de
gastrina, sin embargo la secreción de esta hormona se sigue dando ante estímulos
vagales. Esta situación se explica porque la atropina:
d. No bloquea la acción del péptido GRP
41. Un varón de 50 años es sometido a extirpación del duodeno y parte proximal del
yeyuno. Esta situación ocasionaría la pérdida de las células ​………​.. , productoras de
………………… ​que estimula la secreción de bicarbonato por el páncreas.
“S” / secretina
42. Se evalúa la expresión de la proteína Agrp en una persona con alteración del apetito; lo
correcto respecto a esta proteína es​…​..
La mutación del gen que la codifica produce adelgazamiento
43. Juana cae de la bicicleta y se fractura la región anterior del hueso maxilar superior con
compromiso de la fosa incisiva. Al examen físico de la región esperaría encontrar
alteración en la sensibilidad de la encía ​…………………
palatina anterior
44. Recién nacido es atendido por el neonatólogo y luego entregado a su madre para dar
de lactar; la madre al dar de lactar observa coloración azulada de labios, acompañado
de tos persistente, dificultad respiratoria y distención abdominal. Se le intenta colocar
una sonda nasogástrica pero esta retorna a la cavidad oral en todos los intentos.
¿Cuál de las siguientes anomalías del desarrollo es el más probable en este caso?
b. Atresia esofágica proximal con fístula traqueoesofágica distal l
45. ¿Cuál de los siguientes mecanismos ocurre durante la defecación?
En la posición de “cuclillas” el músculo puborrectal se halla relajado
46. Un paciente luego de un accidente sufre lesión del piso de la boca, se constata daño
del nervio “cuerda del tímpano”, en este caso se esperaría encontrar disminución de
la​…………………………​.​… ​de la lengua
Sensación del gusto en los dos tercios anteriores
47. ¿Cuál de las siguientes afirmaciones es la correcta sobre la gastrina?
Actúa en la célula diana mediante su receptor CCk tipo B
48. Al recibir un paciente con signos de hipovolemia y antecedente de trauma en
abdomen por accidente de tránsito, usted identifica radiológicamente: lesión de
primera vértebra lumbar y signos de lesión en páncreas; durante la cirugía se observó
pobre irrigación de asas intestinales. El vaso afectado es la arteria ​……​..
c. mesentérica inferior
49. Un paciente sufre de daño a nivel del cuello con lesión muscular en la región de la
faringe. En el examen físico se determina dificultad para la elevación de la faringe y
para el cierre del itsmo de las fauces. En este caso, probablemente esté afectado el
músculo:
c. palatofaringeo
50. Varón de 50 años a quien le realizan la curación de la segunda molar de la arcada
superior derecha. En un momento determinado, el paciente acusa de intenso
dolor de la pieza dentaria en tratamiento. La vía aferente del dolor viaja a través
del nervio ​…………
a. trigémino V2
51. La distención gástrica por los alimentos produce incremento de secreción de HCl
mediante la producción de ​…………​.. que estimula a las célulasvía proteína
………​..
a. gastrina / parietal / Gq
52. Un niño de tres años llega a emergencia con disfagia (dificultad para tragar), dolor
retro esternal, salivación y llanto. Se sospecha de ingesta de cuerpo extraño
(moneda) en el esófago; al ser evaluado se constata en una radiografía presencia de
cuerpo extraño a nivel de C6 (6° vértebra cervical). El cuerpo extraño estará
suspendido a nivel del estrechamiento producido por​………​..
c. el músculo cricofaríngeo
53. La triada portal (arteria hepática, vena portal y conducto biliar común) está contenida
en el ligamento ​……​.​……… ​y derivan embriológicamente del ​……
a. hepato duodenal / mesenterio ventral
54. Un paciente refiere no percibir algunos sabores, al examen físico se constata
alteración en la percepción de sabores y del dolor en el tercio posterior de la lengua
¿Cuál de los siguientes nervios estará alterada en su función?
c. Glosofaríngeo (IX par)
55. En el caso de un paciente con gastrinoma (tumor productor de gastrina), la presencia
de úlceras duodenales y erosión de la mucosa gástrica, se debe principalmente
a​……​.
c. el exceso de HCl por estímulo de receptores CCK-B en la célula parietal
56. El reflejo entero gástrico se caracterizan por:
d. originarse debido a la distensión duodenal y presencia del quimo ácido
57. Mauricio tiene dificultad para deprimir el paladar y elevar la parte posterior de la lengua.
En este caso estará afectado un músculo, específicamente el músculo
……………​.
b. extrínseco – palatogloso
58. En condiciones normales, el ingreso de 600 ml de líquido es el estómago provoca un
aumento de presión intragástrica de unos 12 cm de H2O. Después de una vagotomía
(corte del nervio vago) es de esperar que el ingreso del mismo volumen de líquido
ocasione
………………………………… ​de la presión
intragástrica.
c. un aumento mayor
59. La explicación fisiológica de presentar somnolencia de 30 minutos a 1 hora
después de ingerir alimentos, se explica por: a. Aumento del cloro intraluminal
e. Aumento de la alcalinidad sanguínea
60.Se presenta un paciente, el cual presenta un antecedente de tuberculosis intestinal,
por lo cual, se le resecó 80 cm de íleon distal. Desde el punto de vista fisiológico, el
paciente puede presentar una de las siguientes alteraciones: a. Disminución de la
secreción de Vitamina B12

e. Disminución de la absorción de ácido glicocólico


61. Un paciente es sometido experimentalmente a un fármaco que modifica el flujo salival,
obteniéndose un volumen de saliva de 288 ml en 6 horas. En este caso las concentraciones
de electrolitos y bicarbonato en la saliva obtenida varían de la siguiente manera: a. ​↑
Na+,
↓ K ​+, ​↑ ​Cl-, ​↑ ​HCO3-

1. b. ​↓ N
​ a+, ​↓ ​Cl-, ​↑ ​K+, ​↓ ​HCO3-

62. Uno de los siguientes elementos debería hallarse con más probabilidad en el esófago
de un paciente que sufre de reflujo gastro esofágico​…
a. Pepsina

63. Un paciente de 40 años cursa con anemia de 8g/dl, aqueja además de astenia y sensación
de hormigueo bilateral en los miembros inferiores, al examen se halla alteración de la
sensibilidad a la vibración y camina con ampliación de la base de sustentación. Uno de los
siguientes procedimientos sería de ayuda para el diagnóstico de este paciente:
a. Tomografía cerebral
b. Biopsia de la mucosa gástrica

64.Paciente de 60 años ingresa por caída hace 1 hora y pequeño hematoma en cuero
cabelludo, al examen físico ampliado se observa ictericia de piel y mucosas generalizada,
abdomen blando, se palpa estructura quística no dolorosa en hipocondrio derecho que
corresponde a vesícula biliar (signo de Courvoisier), en los exámenes de laboratorio se halla
niveles bajos en la formación de estercobilinógeno y urobilinógeno en heces, incremento de
la bilirrubina conjugada en la orina, elevación de fosfatasa alcalina y gamma glutamil
transpeptidasa séricas. El presente cuadro puede ser explicado por: a. Reabsorción de
hematoma
c.Carcinoma de la cabeza de páncreas
65. Un recién nacido presenta vómitos biliosos poco tiempo después de cada alimento. Al
preguntar a la madre sobre antecedentes, ella recuerda que tuvo polihidramnios durante la
gestación, pero un análisis de cariotipo fue normal. Una de las siguientes es la causa más
probable de estos hallazgos en el recién nacido: a. Enfermedad de Hirschprung
e. Malrotación de la yema pancreática ventral
66.En un estudio de la secreción de hormonas gastrointestinales, sus concentraciones en la
vena porta se midieron durante perfusión luminal del intestino delgado con soluciones de
diversas magnitudes de pH. ¿Qué hormona aumentará en el plasma de la vena porta durante
perfusión a través del intestino con una solución de pH 3?
a. CCK
e. secretina
67.​Paciente de 30 años que ingresa a causa de un traumatismo abdominal cerrado. En la
exploración se aprecia discreta palidez de piel y mucosas, auscultación pulmonar normal,
taquicardia de 120 /min. Discreta distensión abdominal y matidez en flancos; el hematocrito,
que era prácticamente normal al ingreso, disminuye a 30% a las tres horas. En la Rx de tórax
se objetiva fractura de las costillas 10-11 izquierdas. La causa más probable de la
anemización en este paciente es: a. traumatismo renal con hemorragia retroperitoneal.
c. rotura de bazo con hemoperitoneo.
68. Revisando la angiotomografía de un hombre de 70 años en estudio por aneurisma de
aorta abdominal, el radiólogo le informa de la presencia de una oclusión completa de la
arteria mesentérica inferior. El paciente se encuentra completamente asintomático. La
oclusión de la arteria mesentérica inferior cursa de manera asintomática en muchas
ocasiones ya que el territorio que irriga puede recibir flujo proveniente de la arteria:
a. cólica derecha
e. cólica media

69. En las patologías de esófago es importante conocer bien la anatomía esofágica. ¿Cuál de
las siguientes afirmaciones es correcta? a. El esófago tiene capa mucosa, muscular y serosa

c. El esófago torácico pasa por detrás del cayado aórtico

70. A pesar de que pueda haber variaciones anatómicas, lo habitual es que el ciego sea
irrigado por una rama arterial que proviene de unas de las siguientes arterias: a. Iliaca
derecha

d. Mesentérica superior

71. Ante un paciente con una cirugía abdominal urgente, el informe operatorio señala que
se ha realizado una resección de todo el duodeno y del tercio proximal del yeyuno
manteniendo íntegros el estómago y todo el íleon, así como los dos tercios distales del
yeyuno. En el seguimiento nutricional del paciente ¿Qué vitamina o mineral presentará con
menor probabilidad una disminución de su absorción?
a. Cianocobalamina

72. ¿Cuál de las siguientes sustancias forma parte de la secreción biliar? a. Tripsina
Lecitina

73.¿De qué musculo forma parte el ligamento inguinal?


-Oblicuo externo del abdomen
74.¿Cuál de las siguientes enzimas está localizada en el borde en cepillo y juega un rol en la
digestión de proteínas?
e. ​Carboxipeptidasa A.
75. Una de los siguientes sustancias, ​NO ​sirve como un buen agente emulsificante:
a. Colesterol
76. ​La sustancia que estimula el crecimiento de la mucosa gástrica es:

a. Secretina

d. Gastrina

77.¿Cuál de las siguientes alternativas es una función de la colecistokinina?

a. Relajación de la vesícula para la salida de bilis

d. Secreción de enzimas pancreáticas

78.​Con respecto a la anatomía del tronco celiaco, señale lo correcto a. El tronco celiaco se
origina de la cara posterior de la aorta abdominal
d. La hepática común que es una de sus ramas, participa en la irrigación del
estómago.
79. ​Con respecto a la anatomía del duodeno, marque la respuesta correcta: a. Tiene una
distribución en forma de “C”, que rodea la cola del páncreas
b. La 3ra porción duodenal está contenida en la pinza vascular aortomesentérica
80. ​En el íleon se absorbe aproximadamente el 95% dea través de la circulación
enterohepática.
a. agua
c. sales biliares
81. La estimula el mecanismo paracrino de la secreción de ácido clorhídrico.
a. histamina
82.​En la digestión de proteinas,es el principal estímulo para convertir el
pepsinógeno en pepsina. a. la gastrina
b. el pH ácido
83. ​Con respecto a la somatostatina, marque lo correcto:
a. Es secretada por las células S del intestino
Interviene en la fase intestinal de la secreción gástrica
84. ​En pecten anal, es una estructura comprendida entre: a. la línea pectínea y los senos
anales
d. la línea anocutánea y la línea pectínea

85.¿Cuál de las siguientes alternativas es una proenzima pancreática? a. Tripsina

1. b. Elastasa
2. c. Quimotripsinógeno
3. d. Amilasa
4. e. Procarboxipeptidasa C
86. ​En este paciente, el bloqueo farmacológico de los receptores H2 en la mucosa gástrica:
​a) No tiene efecto sobre la secreción de ácido inducida por el vago
b) Evita la activación de adenilciclasa por gastrina
​c) Inhibe la secreción de ácido inducida por gastrina y mediada por el vago
d) Causa un aumento en el transporte de potasio por las células parietales gástricas
Se validó la A :)
87. Si se considerara una gastrectomía total para curar la gastritis del paciente, cuál de
las siguientes sustancias ya no se produciría:
a) Gastrina
b) Quimiotripsina
c) Amilasa
​d) Pepsinógeno

88. Un paciente hipertenso está tomando un medicamento bloqueador de receptores alfa


1 adrenérgicos (prazosina) y como efecto secundario se queja de:
d) Lo escaso que es el medicamento
e) No tiene problemas en la salivación
​c) Hiposalivación
d) Hipersalivación

89. Con respecto a las lesiones y enfermedades de la boca, marque lo correcto:


f) La leucoplasia se desprende al roce
​b) la eritroplasia puede degenerar en adenocarcinoma
c) El muguet oral es una enfermedad bacteriana en inmunodeprimidos
​d) la eritroplasia debe ser biopsiada
​NOTA: fue validada la opción B ya que no es motivo del curso que sepamos el puto cáncer.

90. En este paciente, se puede asumir que la pancreatitis ha sido ocasionada por
una disminución en el efecto de:
g) Amilasa
h) Lipasa
​c) Inhibidor de la tripsina
d) Entercinasa
91.Un efecto secundario en el estómago por la acción de la secretina es:
i) Disminución en la liberación de pepsinógeno
​b) Menor actividad de la pepsina
c) Mayor paso de bicarbonato a sangre periférica
d) Aumento en la producción de factor intrínseco
92. Dentro de los factores protectores de la mucosa gástrica se pueden mencionar
múltiples protagonistas. Uno de ellos es:
j) CCK
k) Gastrina
c​ ) Receptor muscarínico
d) Pepsina
93. La saliva puede tener una variedad de electrolitos en su composición. Entre ellos el
cloro, respecto al cual se puede afirmar:
l) Su mayor concentración se consigue con el flujo bajo
​ ) Su concentración no llega a ser tan alta como en el plasma
b
c) Con flujo alto su concentración es mayor que la del plasma
d) Su menor concentración se alcanza con flujo alto
94. En el síndrome de boca seca o síndrome de Sjogren, una de las
complicaciones asociadas es:
​a) caries
b) Disminución de la acidez gástrica
c) Aumento en de la producción de saliva
d) Aumento del pH bucal
95. Estimula la producción de saliva:
​ ) Vasodilatación periglandular
a
b) Atropina
c) Fatiga o cansancio
d) Expresión de miedo
96. El omeprazol actúa sobre la membrana​ ​de la célula
m) Basolateral/principal
n) apical/principal
o) Basolateral/parietal
​d) apical/parietal
97. Para protegerse del entorno ácido, el Helicobacter pylori se autogenera un entorno de
pH menos ácido alrededor suyo, gracias a una enzima que alcaliniza su entorno local
mediante la conversión de:
​a) urea en NH3
b) H2O y CO2 en ácido carbónico
c) NH3 en urea
d) H2CO3 en bicarbonato
98. La anemia perniciosa destruye las células:
p) mucosas del cuello
b) oxínticas
c) principales
d) mucosas superficiales
99. La célula mucosa del cuello gástrico produce:
a) Moco
b) ácido clorhídrico
c) pepsinógeno
d) Factor intrínseco
100.El aumento en la acidez del estómago producido principalmente por la infección de
Helicobacter pylori se debe a la disminución de:
a) Somatostatina
b) Bicarbonato por las glándulas de Brunner
c) Secretina
d) Colecistoquinina
101. De las siguientes sustancias secretadas por los órganos de este paciente, la más
alcalina es la secreción:
a) pancreática
b) Esofágica
c) Yeyunal
d) Salival
102.En cuanto a la gastritis de este paciente, se encontró que era producida por la bacteria
Helicobacter pylori. Esta bacteria sobrevive en el medio ácido del estómago gracias a:
a) ácido clorhídrico
b) Toxina CagA
c) Ureasa
d) Jugo pancreático
103.La lengua está recubierta por epitelio:
c) pseudoestratificado columnar no queratinizado
b) plano estratificado no queratinizado
c) pseudoestratificado columnar ciliado
d) plano estratificado queratinizado
104.El esfínter anal interno tiene musculatura …….. y tiene control ……..
d) lisa / voluntario
b) lisa / involuntario
c) esquelética / simpático
d) esquelética / parasimpático
105.La arteria Aorta proporciona la irrigación al tubo digestivo ¿cuál de las siguientes
arterias proporciona la irrigación al ángulo cólico derecho?
a) mesentérica superior
b) mesentérica inferior
c) frénica inferior
d) tronco celiaco
106.Paciente de 26 años que le cuenta en su historia clínica que cada vez que almuerza a
los 20 min tiene deseo de defecar, le comenta que su hijo de 1 mes le pasa lo mismo pero
más intenso. Esto se explica por el reflejo …….., el cual está …… en el paciente
e) colicoileal / normal
f) colicoileal / alterado
c) gastrocólico / normal
d) gastrocólico / alterado
107.La región del estómago que se comunica con el duodeno es la
a) pilórica
b) cardias
c) cuerpo
d) fórnix
108.Acude a consulta un px que fue diagnosticado de úlcera péptica 3 días antes. Luego de
múltiples pruebas diagnósticas se concluye que el paciente presenta un tumor secretor de
gastrina, ¿cual de las siguientes situaciones estará incrementada?
g) distensión gástrica
h) inhibición del vaciado gástrico
c) secrecion de acido clorhidrico
d) inhibición de la secreción de pepsinógeno
109.En el sistema digestivo, el control del apetito está dado por un complejo sistema de
sustancias y órganos integradores, los cuales regulan la ingesta de alimentos. La …… es una
sustancia orexígena y es sintetizada por el ……
i) leptina / estómago
j) felina / intestino
k) leptina / estómago
d) grelina / estómago
110. Con respecto a la actividad eléctrica del sistema digestivo, marque la alternativa
correcta
l) corresponden a potenciales de acción que están presentes de forma continua y le
dan capacidad de peristalsis autónoma al sistema digestivo
b) la frecuencia de las ondas lentas NO se ve influenciada por la actividad neural ni las
hormonas gastrointestinales
c) en el estómago las ondas lentas se dan en una frecuencia de 6 x min
d) las ondas lentas son cambios lentos y ondulantes del potencial en reposo
e) la frecuencia de las ondas lentas va de 6 a 12 ondas por minuto
111. Ante una lesión del IX pc, el músculo …… se altera en su función
m) palatogloso
b) estilofaríngeo
c) palatofaríngeo
d) constrictor superior
112. Un varón de 50 años es sometido a extirpación del duodeno y parte proximal del
yeyuno. La pérdida de estímulo hormonal en el páncreas para la secreción enzimática se
explica por la pérdida de células
n) parietales, productoras de factor intrínseco
o) K productoras de factor intrínseco
p) M productora de CCK
d) I productora de CCK
113.Marque la respuesta correcta:
A. El bronquio derecho constituye una de las estrecheces del esófago
B. Todos los órganos del sistema digestivo tienen capa serosa
C. La pared gástrica en el fondo es más delgada que en el cuerpo y antro
D. El esfínter de Oddi rodea a la papila menor duodenal
114. Marque la respuesta correcta en relación a la gastrina:
A. Al distenderse el estómago, se inhibe su producción.
B. Se estimula por la liberación de noradrenalina
C. Las células G son las productoras y se encuentran principalmente en el antro
gástrico
D. Las células G se encuentran principalmente en el fondo gástrico
115. Para poder morder una manzana, es necesario usar el siguiente músculo:
A. Milohiodeo
B. Tensor del paladar
C. Orbicular de los labios
D. Buccinador
116. Sustancia que inhibe la secreción y la motilidad del estómago prolongando el tiempo de
digestión:
A. Enteroglucagon.
B. Polipéptido pancreático
C. Péptido 1 similar al glucagón (GLP-1).
D. Péptido insulinotrópico dependiente de la glucosa (GIP).
117. El nacimiento de la arteria mesentérica superior se puede encontrar en cuál de los
cuadrantes abdominales:
A. Hipocondrio derecho
B. Hipogastrio
C. Epigastrio
D. Mesogastrio
​ ntre las múltiples causas de la Enfermedad por Reflujo Gastroesofágico, se puede
118. E
considerar también a una alteración en las​ ​del esfínter esofágico inferior:
A. Ondas secundarias
​ . Contracciones tónicas
B
C. Ondas lentas
D. Glándulas subesofágicas
119.Producto de la alimentación, se producen diversas sustancias peptídicas, cininas y
bradicininas, las cuales permiten que:
A. Se produzca neovascularización en los territorios de las arterias abdominales
B. La acción de la lipasa pancreática se vea incrementada
C. El consumo de O2 del intestino aumente ligeramente
​D. El flujo sanguíneo intestinal aumente hasta 8 veces
120.El dolor periumbilical o epigástrico en el inicio de una apendicitis aguda se debe a:
A. Estímulo del nervio vago.
B. Íleo secundario.
C. Irritación del peritoneo parietal.
​D. Estímulo del sistema simpático.
121.El aumento en la actividad motora de la pared gástrica genera un aumento en los
niveles locales de qué sustancia en la microvasculatura:
​A.Adenosina
B. Colecistoquinina CCK
C. Endotelina
D. Gastrina
122.¿Cuál de los siguientes péptidos inhibe el vaciamiento gástrico?
​A. Colecist​oquinina
B. Péptido inhibidor gástrico
C. Motilina
D. Gastrina
123. Los músculos de la masticación que producen la retropulsión de la mandíbula son:
​A. temporales [mas seguro]
​B. maseteros
C. milohioideos
D. pterigoideos
124. En relación a la fisiología gástrica, marque lo correcto:
​A. la cimetidina actúa en la región basolateral de la célula parietal
B. la marea alcalina se debe al paso de bicarbonato través de la membrana apical de
la célula principal
C. el cloro difunde hacia el exterior por la la región basolateral de la célula parietal
D. la salida de hidrogeniones a la luz es por difusión facilitada
125. Durante el sueño, la concentración de bicarbonato en la saliva:
A. Se eleva a niveles mayores que los del plasma
B. Aumenta
C. No tiene efecto
​D. Disminuye
126. La secreción de saliva es importante en la fisiología digestiva. Su concentración
de potasio llega a ser menor que la del plasma cuando su secreción tiene un flujo:
A. Intermedio
​B. Nunca
C. Bajo
D. Alto
127. Respecto a las glándulas salivales, marque lo incorrecto:
A. la glándula parótida produce secreción serosa
B. la glándula sublingual drena a través de conducto de Wharton
C. La glándula parótida drena a través del conducto de Stenon
​D. la glándula sublingual tiene forma de garfio
128. ​Con respecto a la saliva, marque la respuesta correcta:
D. será hipertónica cuando el flujo es bajo
E. a mayor flujo, menor concentración de Na
F. a mayor flujo, menor concentración de cloro
​D. el sistema simpático estimula su secreción
CI 3
129. En relación a la circulación hepática, marque lo correcto:
​a) Los sinusoides hepáticos transportan sangre mixta
b) La vena porta proporciona el 50% de sangre al hígado
c) La vena porta se forma a partir de la vena esplénica y la mesentérica inferior
d) La arteria hepática deriva de la mesentérica superior
130. Dentro de las funciones de las células de Ito, marque lo incorrecto:
a) Sintetizan colágeno
b) Almacenan vitamina A
c) Se les llama células estrelladas
​d) Pueden fagocitar patógenos y actúan como presentadoras de antígeno
131.Paciente con tumor neuroendocrino productor de secretina, debido a lo cual se puede
esperar que su secreción pancreática, comparada con la de una persona sana de bajo flujo,
tenga una concentración de:
a) Sodio aumentada
b) Igual
​c) Bicarbonato aumentada
d) Potasio disminuida
132.El GALT se localiza en:
​a) Lámina propia
b) submucosa
c) borde en cepillo
d) superficie de criptas de Lieberkühn
133. En relación a la histología hepática, marque lo correcto:
a) la zona 1 se afecta rápidamente en estados de hipovolemia y shock
b) La zona 1 se encuentra cercana a la vena central lobulillar
​c) La zona 3 se encuentra más cerca a la vena central lobulillar
d) La zona 3 se encuentra más cerca al eje menor del acino hepático
134.El acino pancreático difere con el de las glándulas salivales en:
​ ) Contiene células centroacinares
a
b) No produce secreción serosa
c) El páncreas produce principalmente secreción mucosa
d) No tienen diferencias
135.Durante la digestión de las grasa, para que la lipasa actúe adecuadamente se requiere
que el pH aumento en la luz intestinal, lo cual es logrado, entre otros, por la secreción de las
células:
a) Del conducto interlobulillar
​ ) Centroacinares
b
c) Acinares
d) Alfa
136.La secreción de la colecistoquinina (CCK) se produce en la fase:
​ ) intestinal
a
b) En las 3 por igual
c) gástrica
d) Cefálica
137. ¿Por cuál de las siguientes células es secretada principalmente la pro
enzima procarboxipeptidasa?
​a) Acinares del páncreas
b) Epiteliales del duodeno
c) Ductales del páncreas
d) Centro Acinares del páncreas
138.Una mujer de 43 años dolor en hipocondrio derecho e icterica. En la ecografía se
evidencia cálculos biliares. Estos cálculos lo más probable es que se encuentren localizados
en:
​a) conducto colédoco
b) Conducto cístico
c) Vesícula biliar
d) Conducto pancreático secundario
139. ​Una mujer de 49 años ingresa en el hospital con dolor en epigástrico que migra hacia el lado
derecho y atrás hacia la escápula, sin ictericia. La ecografía muestra un gran cálculo biliar. ¿En cuál
de las siguientes estructuras es más probable que se localice el cálculo biliar?
→ bolsa de Hartmann

140. Una mujer de 38 años ingresa en el hospital con signos de colecistitis y cálculos biliares. Durante
la colangiografía, se inserta el catéter en la vesícula biliar con mucha dificultad. ¿Cuál de las
estructuras interfiere con más probabilidad con el paso del catéter por el conducto cístico?
→ válvula espiral (de Heister)

141. La internalización de proteínas o fragmentos de proteínas hacia el intracelular, es característico


de:
→ células M
→ se validó enterocitos

ECU 1:
Estudiante de 21 años sufre de gastritis aguda ocasionada por comer en lugares poco
higenicos. Suele consumir caramelos (“chupar”) mientras esta en base hasta la tarde.
También toma regular cantidad de leche (grasa, lactora, proteinas), pues le calma el dolor y
el ardor que sitnete por la gastritis ​(tiene dispepsia y cuando toma la leche se le pasa).
Incluso cuando puede, se toma dos vasos de agua frita y le calma la molestia. Ha decido ir
al medico para tratarse, pues ya no soporta el dolor, el cual esta seguro que los síntomas
se producen por elevada producion de HCl en el estomago, y por ello le ha recetado
ranitidina
139. El consumir caramelos eleva los niveles en sangre de una hormona cuya función es
la estimulación de las células.
- ​Beta del páncreas por GIP​ el cual es una incretina y por consiguiente estimula
las células pancreáticas
140. Consumir caramelos indirectamente actica la via:
-​ POMP/ CART saciedad
141. Consumo de leche produce indirectamente
- ​CCK​ inhibición del vaciamiento gástrico mayor tonicidad del esfínter pilórico
142.Cuando el px toma dos vasos de agua, genera indirectamente un aumento en la
liberación de:
- vaso de agua distención → ​g​ ​astrina​ → secreción de HCl
143.El uso de ranitidina bloquea el receptor H2 de la histamina en las células parietales, la
histamina llega a estas células por:
- histamina es una hormona paracrina por →​ difusión
**endocrina es por via hematógena y si fuera neuroendorina es por un NTs
144. Aumenta la secreción salival:
- ​noradrenalina​ a través de los receptores Beta 2
145.En este paciente con gastritis aguda debida a una alta producción de ácido clorhídrico,
sería lógico esperar que el píloro tenga un tono muscular:
- primero la​ secretina
- luego CCK
**ambas reguladores del HCl, Gatritis aguda debido a una alta producción de HCL piloro
estará aumentado (por la CCK)
146.Debido al uso de ranitidina, los valores de somatoestina en sangre:
- ranitidina disminuye acción de gastrina se quiere secretar mas no actúan
los inhibidores como la somatoestina somatoestina ​disminuye
147.El uso de atropina en este paciente:
- Inhibirá la acción de las prostaglandinas
- Aumentará la producción de ácido clorhídrico
- Disminuirá la acción del receptor CCK-B
​-Aumentará el pH del estómago
ECU 2:
Niño de sexo masculino de 2 años de edad, sufre de estreñimiento desde el nacimiento (1
deposición cada 3-4 días). Madre menciona que le estimula la defecación con un
termómetro rectal, y continuo uso de enemas y laxantes. Desde hace 6 meses comienza
con vómitos postprandiales. Los síntomas aumentan en frecuencia y magnitud y están en
relación con los episodios de estreñimiento. No refiere fiebre, tos, diarrea ni lesiones
cutáneas. Al examen físico presenta regular estado general, luce deshidratado. Abdomen
distendido, blando, depresible e indoloro. No se palpan masas abdominales. Se
permeabiliza el canal anal con termómetro rectal, encontrando cierta resistencia. Salida de
material fecal mal oliente en regular cantidad. Exámenes de laboratorio: hemograma
normal. Signos inflamatorios de fase aguda negativos. Alcalosis metabólica leve en sangre
venosa. Radiografía con enema baritado muestra recto y colon sigmoides dilatados
(megacolon). Biopsia profunda: ausencia de células ganglionares en la muestra enviada.
Se realiza cirugía correctiva.
148. ​Durante la fase esofágica de la deglución, para un bolo alimenticio determinado,
a medida que avanza el bolo la fuerza de la contracción se hace más:
- hiperpolarizado
- fuerte
- dependiente de Ach
- debil
149. Cuando este paciente ingiera sus alimentos, se espera que al momento de pasar el
bolo alimenticio por el esfínter esofágico superior, la presión intraesofágica disminuya en:
- la porción proximal al bolo
- el tercio medio del esófago
​-el cardias
- el lugar donde se contraiga la muscular propia
150. Al examinar la orofaringe del paciente, uno puede hallar fácilmente la amígdala
palatina, pues esta se encuentra inmediatamente detrás del músculo:
​-Palatogloso
- Palatofaringeo
- Hiogloso
- Elevador del velo del paladar
151. Con respecto a la defecación en este caso, marque la respuesta correcta:
- En posición de cuclillas, el músculo puborectal genera un ángulo más agudo en
el recto
- El sigmoides y el recto están inervados por el nervio vago
- La sensación de defecar sólo se da cuando el recto es ocupado por heces
y alcanzado el 80% de su capacidad
​-El esfínter anal comprometido tiene inervación autónoma
152. En este paciente [hirschsprung] se considera que está abolido el reflejo:
- Coloileal
​-Rectoesfinteriano
- Gastrocólico
- Relajación receptiva
153.No se espera que sea causa del vómito:
-​ Ayuno prolongado
- Estimulación faríngea y del glosofaríngeo
- Irritación de la mucosa gástrica
- Dolor intenso

ECU 3:
Paciente de 54 años con antecedentes de alcoholismo, gastritis crónica, tabaquismo
pesado, obesidad, cálculos biliares y cirrosis, es llevado a la emergencia por dolor
abdominal en epigastrio irradiado a la espalda y trastorno del sensorio.
Al examen físico: presión arterial 85/50 mmHg, frecuencia cardíaca 100 latidos/min,
frecuencia respiratoria 18 x minuto, temperatura axilar 36°C.
Conjuntivas pálidas, escleras ictéricas nevus arácnidos en tronco, distensión abdominal
marcada, cabeza de medusa, matidez desplazable en ambos flancos e hipogastrio, dolor a
la palpación de abdomen.
Tiempo de protrombina: 24 seg (testigo: 13 seg); TPT: 38 seg, glicemia: 165 mg/dL, uremia:
20 mg/dL, ASAT: 76 UI/L, ALAT: 22 UI/L, albumina: 2,5 g/dL, bilirrubina total: 2,6 mg/dL,
bilirrubina directa: 1,4 mg/dL, amilasa sérica 4000 U/L.
154. ​En esta paciente, al aumento de la amilasa sérica, se debe directamente a una lesión
de:
​a) páncreas
b) vesícula y árbol biliar
c) estómago
d) hígado
155. Considerando que el paciente sufre de gastritis, se puede decir que la secreción de
ácido por la mucosa gástrica
​a) involucra transporte activo de hidrogeniones
b) es realizada principalmente por células principales
c) es inhibida por antihistaminas tomadas por pacientes con rinitis alérgica
d) involucra la liberación de HCl de los gránulos zimógenos
156. El paciente tiene hemorragia digestiva alta por várices sangrantes como complicación.
Llegando a estar en shock hipovolémico por hemorragia masiva, se encontrara necrosis
hepática en:
a) zona 1
b) no se afectan los lobulillos hepáticos en hemorragia
​c) zona 3
d) zona 2
157. El misoprostol, análogo de las prostaglandinas está mejor indicado en:
c) cicatrización de úlcera péptica duodenal
d) erradica el helicobacter pylori
e) tratar el sind de Zollinger ellison
​d) prevenir daño por AINES
158. De las siguientes sustancias secretadas por los órganos de este paciente, la
más alcalina es la secreción:
- Esofágica
- Salival
- Yeyunal
​-Pancreática
159. En este paciente, se puede asumir que la pancreatitis ha sido ocasionada por
una disminución en el efecto de:
- Lipasa
- Enterocinasa
- Amilasa
​-Inhibidor de la tripsina
160. ¿Cuál de las siguientes sustancias es segregada por el páncreas?
​-Amilasa
-Pepsina
- Quimiotripsina
- Tripsina
161. Cada vez que este paciente toma alcohol, la acidificación de la luz del duodeno:
-​ Disminuye el vaciamiento gástrico
- Aumenta la contracción del esfínter de Oddi
- Aumenta la secreción del ácido gástrico
- Disminuye la secreción pancreática del bicarbonato

162. ​Un niño de 2 años es llevado a la consulta por diarrea persistente y edema de las extremidades, además
falta de crecimiento y desarrollo en relación a su edad. Los análisis de sangre revelan que tiene concentración
plasmática baja de proteínas (hipoproteinemia). Durante la endoscopía duodenal, se coloca colecistokinina
(CCK) endovenosa y se recoge muestras del líquido duodenal; el resultado del líquido confirma incapacidad para
hidrolizar proteínas a un pH neutro, esta situación mejora al añadir una pequeña cantidad de tripsina. El
paciente probablemente esté sufriendo la falta congénita de
………….
(Unidad 4, sesión 26, logro 2: Explicar la Digestión y absorción de las proteínas y sus alteraciones)
a. Pepsinógeno
b. PEPT-1
c. Carboxipeptidasas
d. Enterocinasa
163. ​Experimentalmente se incrementa la velocidad de la secreción salival con una sustancia, el análisis de
la composición de esta saliva obtenida se espera encontrar…………..
(Unidad 3, sesión 17, logro 5 : Explica la Influencia de la velocidad del flujo salival en la composición de la
saliva)
e. Elevación de concentración de bicarbonato, sodio y potasio
f. Elevación de concentración de cloro, sodio y potasio
g. Disminución de concentración de potasio
h. Disminución de concentración de potasio y bicarbonato
164. ​Paciente varón de 46 años soltero, consulta por odinofagia y bajo de peso, tiene antecedente de
tuberculosis desde hace 3 meses y es fumador crónico (10 cigarrillos por día); al evaluar la cavidad oral se
identifica lesión blanquecina en el dorso de la lengua y paladar blando, las lesiones se desprenden con el baja
lengua dejando una base eritematosa. Esta lesión corresponde probablemente a ……………………….…..
( Unidad 3, sesión18, logro 1-2 : Describe las enfermedades inflamatorias, infecciosas y proliferativas de la
cavidad oral)
i. Eritroplaquia
j. Candidiasis oral
k. Leucoplaquia vellosa
l. Fibroma en cavidad oral
165. Minero de 32 años de edad, que acude a centro de
salud por presentar de forma progresiva desde hace 1
año dificultad para ingerir alimentos sólidos y luego
líquidos; refiere regurgitaciones alimentarias y marcada
pérdida de peso (15 kilos). Radiografia baritada de
esófago como se muestra en la figura. El presente caso
se explica por……………….
(Unidad 2, sesión 12, logro 4: Identificar y
describir la función de los esfínteres esofágicos)
m. Contracción incompleta del esfínter esofágico inferior
n. Dificultad para el inicio de la deglución
o. Relajación incompleta del esfínter pilórico
p. Relajación incompleta del esfínter esofágico inferior
166. Paciente mujer de 35 años acude a consulta por sensación de sequedad y lesiones en cavidad oral. Al examen se observa
atrofia de la mucosa, fisuras y úlceras; nota además sequedad e irritación de la córnea y aumento del tamaño de las glándulas
parotídeas. Su diagnóstico más probable es artritis reumatoide; el hallazgo más probable en una biopsia de glándula parótida
es……..….
(Unidad 3, sesión 18, logro 3: Describe las enfermedades más frecuentes de las glándulas salivales)
q. Hiperplasia de acinos glandulares serosos
r. Gran infiltración de linfocitos y células plasmáticas
s. Gran infiltrado de linfocitos y macrófagos
t. Presencia de acinos normales con hiperplasia de células ductales

167. Un paciente con anemia acude con su médico quejándose de episodios frecuentes de gastroenteritis. Un análisis de sangre
revela anticuerpos circulantes dirigidos contra células parietales gástricas. Su anemia es atribuible a la hiposecreción de
………………………
(Unidad 3, sesión 20, logro 5: Gastritis crónica. Tipos de gastritis)
u. Factor intrínseco
v. Proteina R (haptocorrina)
w. Pepsinógeno
x. Ácido clorhídrico

168. ​Dos estudiantes deciden tomar un receso para comer una hamburguesa a la hora del almuerzo. Antes de llegar a la cafetería,
impulsos nerviosos provenientes del complejo vagal dorsal iniciarán la secreción de ácido gástrico por la liberación dedesde el
sistema nervioso entérico.
(Unidad 3, sesión 20, logro 2: Regulación de la secreción gástrica: estimulación, fases de la secreción)
y. Serotonina
z. Óxido nítrico
aa. GRP (péptido liberador de gastrina)
bb. Péptido intestinal vaso activo

169. Un niño de cuatro años de edad es llevado a la consulta por cuadros diarreicos frecuentes caracterizados por heces pálidas,
voluminosas y fétidas, presenta bajo peso y talla. Se mide la concentración de cloruro en el sudor y se encuentra que sus valores
son muy elevados. La alteración más importante a nivel de células ductales del páncreas tiene relación directa con la
conductancia de…………
(Unidad 3, sesión 23, logro 5 Explica la Secreción pancreática: formación del jugo pancreático, influencia de la velocidad de
flujo y regulación)
cc. Potasio
dd. Bicarbonato
ee. Sodio
ff. Cloro

170.Una mujer de 50 años de edad que sufrió durante varios años resequedad de los ojos debida a producción inadecuada de
lágrimas es enviada con un gastroenterólogo para evaluación de pirosis crónica. El examen endoscópico revela erosiones y tejido
cicatrizal en la parte distal del esófago justo por arriba del esfínter esofágico inferior. Las lesiones pueden atribuirse a la
disminución de uno de los siguientes componentes salivales:
(Unidad 3, sesión 17, logro 4: Explicar la Formación de la saliva y cuáles son sus componentes)
gg. Bicarbonato
hh. Lactoferrina
ii. Ig A
jj. Amilasa
171. Se evalúa los valores séricos de las siguientes sustancias a un paciente con enfermedad hepática terminal; en este paciente se
espera encontrar la combinación con la letra …………
(Unidad 3, sesión 22, logro 5: Describe las Pruebas de función hepática, la Insuficiencia hepática, encefalopatía hepática e
hipertensión portal)

Glucosa Amoniaco Albúmina


a. Aumenta Disminuida Disminuida
da
b. Disminui Aumentada Aumentada
da
c. Aumenta Aumentada Aumentada
da
d. Disminui Aumentada Disminuida
da
172. Una mujer de 35 años de edad HIV positiva, se presenta al médico con dolor abdominal en cuadrante superior derecho e
ictericia. La paciente refiere haber tenido múltiples episodios de ictericia durante los últimos 10 años. Los exámenes para
determinar hepatitis viral, dieron positivos para Hepatitis B, siendo catalogado el caso como hepatitis crónica con alteración
funcional. En un examen de sangre ¿cuál de los siguientes parámetros está disminuido?
(unidad 3, sesión 22, logro 5: Pruebas de función hepática, Insuficiencia hepática, encefalopatía hepática e hipertensión
portal)
kk. Fosfatasa alcalina
ll. Albumina
mm. Bilirrubina
nn. Tiempo de protrombina
173. En el reflejo peristáltico del intestino delgado, uno de los siguientes eventos sucede en la porción oral del bolo
alimenticio…………...
(Unidad 2, sesión 13, logro 4: Explicar la Motilidad del intestino delgado: Contracciones segmentarias y peristálticas)
oo. Disminución de 5 hidroxitriptamina desde las neuronas IPAN
pp. Contracción del músculo longitudinal
qq. Acción del péptido intestinal vasoactivo (VIP) en el músculo circular
rr. Acción de acetilcolina en el músculo circular

174. Experimentalmente se coloca una dosis alta de secretina en la luz intestinal duodenal; como consecuencia de esto, en el
jugo pancreático de la misma luz intestinal se observa la disminución de la concentración de …..………..
(Unidad 3, sesión 23, logro 5: Explica la Secreción pancreática: formación del jugo pancreático, influencia de la velocidad de
flujo y regulación)
ss. Na​+
tt. Cl​-
uu. K​+
vv. HCO3​-

175. Un varón de 58 años de edad con enfermedad de Crohn severo fue sometido a una resección ileal. Después de la cirugía
este paciente padecerá de esteatorrea, esto se explica porque …..………..
(unidad 4, sesión 26, logro 4: Explica las alteraciones en la Absorción de lípidos)
ww. El pool de ácidos biliares se incrementa
xx. Los quilomicrones no pueden formarse en el lumen intestinal
yy. La micelas no pueden formarse
zz. El páncreas no secreta lipasa

176. En un experimento se inserta un balón en el estómago de un voluntario, se infla poco a poco mientras que se vigilan las
presiones intraluminales. Aunque el volumen del balón aumenta considerablemente, las presiones permanecen constantes. Esta
relación volumen-presión se explica por la liberación local de …………..
(Unidad 2, sesión 13, logro 1 Explica la Motilidad gástrica: relajación receptiva)
aaa. Acetil colina y gastrina
bbb. Colecistoquinina y óxido nítrico
ccc. Óxido nítrico y péptido inhibidor vasoactivo
ddd. Norepinefrina y óxido nítrico
177. La toxina del Vibrio cholerae causa diarrea debido a…….
(Unidad 4, sesión 27, logro 6: Explica el transporte hidroelectrolítico intestinal, toxina colérica)
eee.La fosforilación del canal CFTR de los enterocitos de las vellosidades intestinales
fff. El Incremento de la secreción de cloro por las células de la cripta intestinal
ggg. La inhibición de la producción de AMPc por las células epitelailes
hhh. El incremento de la absorción de agua y sodio a través de las uniones estrechas

178. ¿Cuál de las siguientes alternativas es una característica de la secreción exocrina del páncreas?
(Unidad 3, sesión 23, logro 5: Secreción pancreática: formación del jugo pancreático, influencia de la velocidad de flujo y
regulación)
iii. Tiene una baja concentración de Cl​-​ respecto al plasma
jjj. Es estimulada por la presencia de bicarbonato en el duodeno
kkk. La secreción enzimática es estimulada principalmente por la gastrina
lll. Es hipotónica respecto al plasma

179. Una madre lleva a su hijo de dos años de edad a la sala de urgencias, estresada porque el niño deglutió una moneda de 10
céntimos mientras la familia cenaba en un restaurante. El médico observa mediante fluoroscopía que la moneda se halla en el
estómago y asegura a la madre que la moneda se eliminará con las heces. El médico recomienda utilizar la respuesta fisiológica
que permitirá la evacuación de la moneda del estómago al intestino ………….…..
(Unidad 2, sesión 13, logro 2: Explica la Motilidad gástrica: mezclado y vaciamiento)
mmm. Es por la relajación receptiva
nnn. Son los movimientos de mezcla y trituración
ooo. Es provocada por el ayuno
ppp. Es por la relajación del esfínter esofágico superior

180. Las estructuras en el hígado que permite que los productos metabólicos unidos a proteínas tengan acceso a las membranas
basolaterales de los hepatocitos, son…..
(Unidad 3, sesión 21, logro 4-5: Explica la Organización micro estructural del hígado)
qqq. Los Canalículos
rrr. Las fenestras sinusoidales
sss. Las uniones intercelulares herméticas
ttt. Las células de Ito

181. La composición de la bilis es modificada conforme fluye por los conductillos biliares. Durante este tránsito se espera que
aumente la concentración de…….
(Unidad 3, sesión 22, logro 2: Describe la Secreción biliar, visión general del sistema biliar extrahepático y composición de la
bilis)
uuu. Ig A
vvv. Glucosa
www. Monómeros de ácido biliar
xxx. Vitamina A

181. Se mide experimentalmente el contenido gástrico de dos personas. La persona “A” tiene alto contenido de grasa y la
persona “B” tiene un contenido hipertónico ¿Cuál de las siguientes es correcto respecto al vaciamiento gástrico? ​(Unidad
2, sesión 13, logro 2: Describe la Motilidad y vaciamiento gástrico)
yyy. Hay ralentización del vaciado gástrico solo en “A”
zzz. El vaciamiento gástrico es más rápido en ambos
aaaa. En ambos casos hay incremento de la motilina
bbbb. Hay ralentización del vaciado gástrico en ambos casos

182. El examen endoscópico de un paciente con hipertensión portal grave revela venas tortuosas que sobresalen hacia la luz del
esófago. El paciente recibe tratamiento quirúrgico mediante la colocación de una derivación que conecta la vena porta a la vena
cava. Después de la operación el riesgo de encefalopatía ………………….. y el riesgo de sangrado de várices ……………..
(Unidad 3, sesión 22, logro 5: Describe la Insuficiencia hepática, encefalopatía hepática e hipertensión portal)
cccc. Aumentará/disminuirá
dddd. Disminuirá/disminuirá
eeee. Aumentará/aumentará
ffff. Disminuirá/aumentará
183. Un paciente varón de 18 años de edad acude al médico para sus exámenes de rutina. Sus resultados de laboratorio
muestran un valor de bilirrubina sérica de 4 mg/dl y una bilirrubina directa de 0,3 mg/dl. Las pruebas de función hepática son
normales. La alteración que explica mejor este caso es por la deficiencia de ………………..
(Unidad 3, sesión 22, logro 3: Explica la Producción y excreción de bilirrubina. Tipos de bilirrubina e ictericia)
gggg. Transaminasas
hhhh. Glucuronil transferasa
iiii. Hemo oxigenasa
jjjj. La 7 alfa hidroxilasa

184. Un hombre de 57 años de edad es llevado a urgencias con hematemesis masiva rojo brillante, a su llegada se halla
inconciente con PA: 80/40 mm Hg y FC: 124 lat/min. Luce ictérico con presencia de “arañas vasculares en el tórax anterior y
extremidades”, abdomen distendido con signo de oleada positiva. Se encuentra esplenomegalia y pérdida de la masa muscular
en extremidades. La anastomosis vascular responsable del sangrado en este paciente es ………….…..
(Unidad 3, sesión 21, logro 2: Describe las anastomosis porto sistémicas)
kkkk. Arteria gástrica izquierda y vena ácigos
llll. Vena gástrica izquierda y vena ácigos
mmmm. Vena paraumbilical y vena epigástrica inferior
nnnn. Vena gástrica izquierda y vena esofágica superior

185. Un estudiante de medicina está comiendo un plato de comida a base de champiñones, espárrago y salsa de soya. El sabor
umami contenido en todos estos alimentos actúa a nivel de los botones gustativos estimulando ………………..
(Unidad 2, sesión 10, logro 5: Describe los tipos y mecanismos moleculares para la detección de los sabores)
oooo. El ingreso de sodio
pppp. Un receptor acoplado a proteína G
qqqq. Su receptor específico T1R3
rrrr. El ingreso de hidrógeno

186. Un hombre de 22 años de edad se presenta al médico con una historia de 1 año de evolución caracterizado por dolor
recurrente en fosa iliaca derecha y diarrea. Manifiesta además pérdida de peso de 8 kg durante este periodo. La
colonoscopía revela múltiples lesiones en el ileon terminal y colon. La biopsia de estas lesiones revela engrosamiento,
inflamación y ulceración de la mucosa. El diagnóstico más probable en este caso es…….
(Unidad 4, sesión 28, logro 5: Describe la Enfermedad inflamatoria intestinal. Generalidades, morfología y características)
ssss. Sprue celiaco
tttt. Enfermedad de Crohn
uuuu. Sindrome de colon irritable
vvvv. Colitis ulcerativa

187. Una de las funciones del músculo señalado es:


(Unidad 2, sesión 8, logro 3: Describir el Piso de la boca: estructuras
blandas que la conforman)
wwww. Eleva el paladar blando
xxxx. Recibe inervación del nervio maxilar
yyyy. Deprime el hioides cuando la mandíbula está fija
zzzz. Deprime la mandíbula cuando el hioides está fijo

188. Varón de 61 años que consulta por dolor retro esternal intenso desde hace 6 horas y después de vómitos intensos y
repetidos; al examen se observa disnea, cianosis, hipotensión y signos clínicos de shock. La radiografía simple de tórax
muestra neumomediastino. El líquido en el espacio pleural aspirado tiene alta concentración de amilasa. ¿Cuál de las
siguientes alternativas puede explicar este cuadro clínico?
(Unidad 3, sesión 18, logro 6: Describe algunas Enfermedades del esófago)
aaaaa. Sindrome de Mallory Weiss
bbbbb. Rotura espontánea de esófago
ccccc. Neumotórax por probable herida penetrante
ddddd. Perforación de ulcera gástrica de cara posterior, con complicación torácica
189. La secreción del ácido en la célula parietal gástrica se lleva a cabo por una ATPasa especifica que intercambia hidrogeniones
(H+) del citosol por…..
(Unidad 3, sesión 20, logro 1: Explica la Secreción del HCl y sustancias que la alteran)
eeeee. Cl-
fffff.HCO3-
ggggg. Na +
hhhhh. K+

190. ​En condiciones normales el ingreso de 600 ml de líquido es el estómago provoca un aumento de presión intragástrica de
unos 12 cm de H2O. Después de una vagotomía (corte del nervio vago) es de esperar que el ingreso del mismo volumen de
líquido provoque lo siguiente: …………………………………
(Unidad 2, sesión 13, logro 1: Describe la Motilidad gástrica: relajación receptiva)
iiiii. Un aumento igual de la presión
jjjjj. Que no aumente la presión
kkkkk. Un aumento mayor de la presión
lllll. Una disminución de la presión

191.Una paciente de 30 años de edad es sometida a una cirugía en oído medio derecho por un problema de otoesclerosis. Luego
de la cirugía refiere alteración en la percepción de sabores. Al evaluar el caso usted esperaría encontrar……….
(Unidad 2, sesión 10, logro 5: Describe la Irrigación e inervación de la lengua)
mmmmm. Alteración en la sensación del dolor y temperatura en el tercio posterior de la lengua
nnnnn. Alteración en la sensación del gusto en los dos tercios anteriores de la lengua
ooooo. Alteración en la sensación del gusto en la punta de la lengua
ppppp. Sensación del dolor, tacto y temperatura conservada en toda la lengua

192. ¿Cuál de las siguientes alterativas es correcta?


(Unidad 4, sesión 26 : Explica la digestión y absorción de nutrientes y sus alteraciones)
qqqqq. En el borde luminal, en cepillo, del intestino delgado, la absorción de sodio únicamente se realiza asociada a
la de glucosa.
rrrrr. El lugar principal para la absorción del hierro es el ileon
sssss. Las sales biliares desconjugadas son absorbidas preferentemente en el colon
ttttt. El proceso de digestión y absorción de la vitamina B12 no se altera en insuficiencia pancreática.

193. En un paciente de 45 años de edad con colestasis biliar, se encuentra una elevación de los niveles sanguíneos de fosfatasa
alcalina hasta 3 veces la cifra normal. ¿Cuál de las siguientes alternativas estará también elevada como evidencia del daño de la
vía biliar?
(Unidad 3, sesión 22, logro 5: Pruebas de función hepática, Insuficiencia hepática, encefalopatía hepática e
hipertensión portal)
uuuuu. Tiempo de protrombina y albúmina sérica
vvvvv. Transaminasas hepáticas (ALT y AST)
wwwww. Glucoronil transferasa
xxxxx. Gamma glutamil transpeptidasa

194. Revisando la angiografía de un hombre de 70 años en estudio por aneurisma de aorta abdominal el radiólogo informa de la
presencia de una oclusión completa de la arteria mesentérica inferior. El paciente se encuentra completamente asintomático.
¿Cuál de las siguientes arterias se anastomosa a la sistema arterial de la mesentérica inferior?
(Unidad 4, sesión 25, logro 1: Identifica la Arteria mesentérica superior e inferior, ramas y anastomosis)
yyyyy. Ileal
zzzzz. Cólica media
aaaaaa. Sigmoideas
bbbbbb. Cólica izquierda

195. ​Lactante de 3 meses de vida es atendido por presentar diarrea, se administra una solución de glucosa y electrólitos por vía
oral. La proteína de membrana apical que explica la capacidad de esta solución para proporcionar aporte de glucosa e
hidratación es ………..
(Unidad 4, sesión 26, logro 1: Explica la Digestión y Absorción de los hidratos de carbono. Alteraciones)
cccccc. GLUT-5
dddddd. SGLT-1
eeeeee. CFTR
ffffff.GLUT-2
196. Paciente ha sufrido herida de bala en el abdomen, se le ha tenido que extirpar el segmento medio y distal del ileon. En este
caso la síntesis hepática de sales biliares estará …..…..
(Unidad 3, sesión 22, logro 4: Explica la formación, función y Circulación entero hepática de lasa sales biliares)
gggggg. Disminuida por inhibición de la colesterol 7 alfa hidroxilasa
hhhhhh. Incrementada por estímulo de la enzima colesterol 7 alfa hidroxilasa
iiiiii. Incrementada por inhibición de la colesterol 7 alfa hidroxilasa
jjjjjj. Sin cambios en el ritmo de síntesis

197. ​Un varón de 75 años ingresa al consultorio por presentar ictericia marcada de piel y las escleras. El estudio del paciente
mostró que presentaba un tumor que obstruía la totalidad del conducto hepático común. ¿Cuál de las siguientes estructuras se
encontrará dilatada en este paciente?
(Unidad 3, sesión 21, logro 6: Describir el árbol biliar intrahepático)
kkkkkk. Conducto de Wirsung
llllll. Conductos de Hering
mmmmmm. Conducto colédoco
nnnnnn. Conducto cístico

198.Correlaciones las dos columnas y marque la fórmula correcta:


(Unidad 4, sesión 28, logro 1: Diarrea: definición, mecanismos: osmótica, secretoria y exudativa)
1. Enfermedad Hirschsprung(4) heces con moco y sangre
2. Diarrea osmótica(2) intolerancia a lactosa
3. Diarrea secretoria(1 ) aganglionosis congénita
4. Diarrea exudativa(3) canales de Cl- en las células de la cripta

a.- 4231 b.- 1234 c.- 2143 ​d.- 4213


199. Respecto a la siguiente imagen que representa una estructura de la mucosa gástrica, la estructura con número ………..
produce ……………………..
(Unidad 3, sesión 19, logro 4: La glándula fúndica. Funciones y tipos de células con sus características)
oooooo. 3 / pepsina
pppppp. 1 / Pepsinógeno
qqqqqq. 4 / HCl y factor extrínseco
rrrrrr. 2 / pepsinógeno

200. En un paciente con insuficiencia renal crónica, el déficit en la absorción de calcio a nivel del enterocito se debe a lo
siguiente:
(Unidad 4, sesión 26, logro 6: Explica la Absorción de calcio y hierro)
ssssss. No se convierte la 25 hidroxicolecalciferol a 1,25 dihidroxicolecalciferol
tttttt. No se convierte la 1,25 dihidroxicolecalciferol a 25 hidroxicolecalciferol
uuuuuu. Se incrementa la producción de Calbindina
vvvvvv. Existe un descenso de la alfa 25 hidroxilasa renal
201. Varón de 30 años es traído a emergencia por agresión abdominal con arma de fuego (pistola) y es sometido a
laparotomía exploratoria, observándose isquemia del colon ascendente y parte del colon trasverso ¿la lesión de cuál de las
siguientes arterias explicaría esta isquemia?
(unidad 1, sesión 2, logro 6: (D​escribe la irrigación visceral: arterias de tronco celiaco, arteria mesentérica
superior e inferior​, topografía de superficie, órganos por cuadrante)

a. Celiaca
b. Colónica derecha
c. Mesentérica inferior
d. Mesentérica superior
202. Respecto a las sustancias gastrointestinales que regulan la secreción pancreática; marque la afirmación correcta:
(unidad 1, sesión 3, logros 2 y 3: describir las hormonas gastrointestinales: estímulos y funciones)

a. La Secretina, es la hormona más importante para la secreción de bicarbonato por las células acinares del
páncreas
b. La acetilcolina es capaz de estimular la secreción enzimática y de bicarbonato del páncreas
c. La gastrina, es la hormona más importante para la secreción de enzimas pancreáticas
d. La colecistoquinina (CCK) estimula al páncreas solo para secreción enzimática

203. Ante una lesión del X par craneal, ¿cuál de los siguientes músculos mantiene conservada su función?:
(unidad 2, sesión 08, logro 4: Paladar blando: componentes musculares)

e. Elevador del velo del paladar


f. Tensor del velo del paladar
g. Palatofaríngeo
h. Glosofaríngeo
204. Experimentalmente se utiliza atropina (anticolinérgico) para inhibir la secreción de gastrina, sin embargo la secreción
de esta hormona se sigue dando ante estímulos vagales. Esta situación se explica porque la atropina:
(unidad 1, sesión 3, logro 3 : describir las hormonas gastrointestinales: estímulo y funciones de la gastrina y
colecistoquinina)

a. Bloquea parcialmente la bomba de protones en la célula G


b. Inhibe la acción de acetilcolina e histamina en la célula G
c. Solo inhibe la acción del péptido GRP en la célula G
d. No bloquea la acción del péptido GRP
205. ​Un varón de 50 años es sometido a extirpación del duodeno y parte proximal del yeyuno. Esta situación
ocasionaría la pérdida de las células ……….. , productoras deque estimula la secreción de
bicarbonato por el páncreas.
(unidad 1, sesión 3, logro 3: describir las hormonas gastrointestianles: estímulos y funciones de la secretina y
péptido insulinotrópico dependiente de glucosa)

a. “S” / secretina
b. Parietales / secretina
c. “I” / colecistoquinina
d. “S” / colecistoquinina

206.Recién nacido que presenta tumoración abdominal a nivel del cordón umbilical (fotografía). ¿cuál de las siguientes
afirmaciones es correcta respecto a este defecto en el desarrollo embriológico del intestino?: ​(unidad 1, sesión 5, logro 2:
identificar las anomalías del desarrollo del intestino medio)

a. Corresponde a una Gastrosquisis


b. Las vísceras se hallan cubiertas por piel
c. No está asociado a otras malformaciones
d. Se asocia a malformaciones cardiacas y
del tubo neural

207. Varón de 35 años acude a la emergencia por trauma abdominal y se decida


realizar una laparoscopía exploratoria. El cirujano observa la disposición de los
órganos abdominales como se representa en el siguiente esquema. Esta
disposición de órganos se explica por la rotación(SMA=arteria mesentérica
superior)
(unidad 1, sesión 5, logro 3: identificar las anomalías del desarrollo del
intestino medio: defectos de rotación, estenosis y atresias)

a. anti horaria del intestino medio, en sólo 90°


b. incompleta del intestino medio (270°)
c. horaria del intestino medio
d. horaria del estómago

208. Se evalúa la expresión de la proteína Agrp en una persona con alteración del apetito; lo correcto respecto a esta
proteína es…..
(unidad 1, sesión 3, logro 4: E​ xplica los mecanismos de control del apetito y saciedad ​)

a. Esta proteína es un potente anorexigénico


b. La mutación del gen que la codifica produce adelgazamiento
c. La sobre producción de la proteína lleva a obesidad por agonismo de receptores MC3 y MC4
d. La sobre producción de la proteína disminuye el apetito por antagonismo de receptores MC4

209. Juana cae de la bicicleta y se fractura la región anterior del hueso maxilar superior con compromiso de la fosa incisiva.
Al examen físico de la región esperaría encontrar alteración en la sensibilidad de la encía …………………
(unidad 2, sesión 8, logro5: paladar: paladar duro y blando: irrigación e inervación)

a. bucal posterior
b. Lingual anterior
c. palatina anterior
d. palatina posterior
210. Recién nacido es atendido por el neonatólogo y luego entregado a su madre para dar de lactar; la madre al dar de lactar
observa coloración azulada de labios, acompañado de tos persistente, dificultad respiratoria y distención abdominal. Se le intenta
colocar una sonda nasogástrica pero esta retorna a la cavidad oral en todos los intentos. ¿Cuál de las siguientes anomalías del
desarrollo es el más probable en este caso? ​(unidad 1, sesión 4, logro 3: identificar las anomalías en el desarrollo del esófago:
atresia y/o fístula traqueo esofágica)

a. Estenosis esofágica proximal con Fístula traqueo esofágica distal


b. Atresia esofágica proximal con fístula traqueoesofágica distal
c. Atresia esofágica distal con fístula traqueoesofágica proximal
d. Fístula traqueoesofágica proximal y distal

211. ¿Cuál de los siguientes mecanismos ocurre durante la defecación?


(unidad 2, sesión 13, logro 6: motilidad del intestino grueso: contracciones segmentarias, movimientos en
masa, defecación y reflejo gastrocólico)

i. Contracción refleja del esfínter anal interno


j. En la posición de “cuclillas” el músculo puborectal se halla relajado
k. Relajación del esfínter anal externo por efectos del VIP y óxido nítrico
l. La materia fecal en el recto estimula la contracción del sigmoides por los nervios pudendos

212. La estructura número 4 (gráfico) corresponde a


……….… y está ………..
(unidad 2, sesión 9, logro 2: Partes de un diente.
Capas del diente: Esmalte: características y células
que lo producen)

m. el cemento / mineralizado en 90%


n. la dentina / formada por ameloblastos
o. el esmalte / formado por células derivadas
del mesénquima
p. la dentina / formado por células derivadas de la
cresta neural

213. Un paciente luego de un accidente sufre lesión del piso de la boca, se constata daño del nervio “cuerda del
tímpano”, en este caso se esperaría encontrar disminución de lade la lengua
(unidad 2, sesión 10, logro 3: Irrigación e inervación de la lengua)

a. Motilidad en los dos tercios anteriores


b. Sensación del gusto en el tercio posterior
c. Sensación del gusto en los dos tercios anteriores
d. Sensibilidad al tacto en los dos tercios anteriores

214. ¿Cuál de las siguientes afirmaciones es la correcta sobre la gastrina?


(unidad 1, sesión 3, logro 1: reconocer las características de las sustancias reguladoras gastrointestinales:
hormonas, sustancias paracrinas y neurocrinas)

q. Produce atrofia de la mucosa gástrica


r. Es producida por la célula G del cuerpo gástrico
s. Es estimulada por la distensión gástrica y el Ph bajo
t. Actúa en la célula diana mediante su receptor CCk tipo B
215. Al recibir un paciente con signos de hipovolemia y antecedente de trauma en abdomen por accidente de tránsito, usted
identifica radiológicamente: lesión de primera vértebra lumbar y signos de lesión en páncreas; durante la cirugía se observó pobre
irrigación de asas intestinales. El vaso afectado es la arteria ……..
(unidad 1, sesión 1, logro 6: reconocer las estructuras a nivel de L1, nivel de los principales vasos sanguíneos)

a. esplénica
b. hepática común
c. mesentérica inferior
d. mesentérica superior

216. Un paciente sufre de daño a nivel del cuello con lesión muscular en la región de la faringe. En el examen físico se
determina dificultad para la elevación de la faringe y para el cierre del itsmo de las fauces. En este caso, probablemente esté
afectado el músculo:
(unidad 2, sesión 11, logro 2: Músculos de la faringe: identificación, constrictores y longitudinales)

a. palatogloso
b. estilofarinfeo
c. palatofaringeo
d. constrictor inferior

217. Varón de 50 años a quien le realizan la curación de la segunda molar de la arcada superior derecha. En un
momento determinado, el paciente acusa de intenso dolor de la pieza dentaria en tratamiento. La vía aferente del dolor
viaja a través del nervio …………
(unidad 2, sesión 9, logro 6: Inervación de los dientes)

a. trigémino V2
b. trigémino V3
c. naso palatino
d. palatino menor

218. La distención gástrica por los alimentos produce incremento de secreción de HCl mediante la producción de
………….. que estimula a las células ……………. vía proteína ………..
(Unidad 1, sesión 3, logro 2: Describe las hormonas gastrointestinales: Estímulo y funciones de la gastrina y
colecistoquinina)

u. gastrina / parietal / Gq
v. gastrina / principal / Gs
w. acetilcolina / parietal /Gi
x. acetilcolina / principal / Gi

218. Un niño de tres años llega a emergencia con disfagia (dificultad para tragar), dolor retro esternal, salivación y llanto. Se
sospecha de ingesta de cuerpo extraño (moneda) en el esófago; al ser evaluado se constata en una radiografía presencia de
cuerpo extraño a nivel de C6 (6° vértebra cervical). El cuerpo extraño estará suspendido a nivel del estrechamiento
producido por………..
(unidad 2, sesión 11, logro4: Esófago, características anatómicas, relación con órganos vecinos y estrecheces)

a. el cayado aórtico
b. el hiato esofágico
c. el músculo cricofaríngeo
d. el bronquio principal izquierdo

219. La triada portal (arteria hepática, vena portal y conducto biliar común) está contenida en el ligamento
…….……… y derivan embriológicamente del ……
(Unidad 1, sesión 1, logro 4: Identifica el peritoneo, mesenterio, omento y ligamentos, retroperitoneo.)

y. hepato duodenal / mesenterio ventral


z. gastro esplénico / mesenterio dorsal
aa. hepato gástrico / omento menor
bb. falciforme / omento menor

220. En relación al movimiento de peristaltismo del tubo digestivo:


en la flecha negra del gráfico se produce la liberación de
……………… a nivel del músculo ………..
(unidad 2, sesión 7, logro 6: Control hormonal y tipos de
movimiento)

a. noradrenalina, sustancia P y neuropéptido “ Y” / circular


b. acetilcolina y sustancia P / longitudinal
c. óxido nítrico y PIV / longitudinal
d. óxido nítrico y PIV / circular

221. Un paciente refiere no percibir algunos sabores, al examen físico se constata alteración en la percepción de sabores y
del dolor en el tercio posterior de la lengua ¿Cuál de los siguientes nervios estará alterada en su función?
(unidad 2, sesión 10, logro 5: Sabores, tipos y mecanismos moleculares para su detección)

a. Lingual (rama del V par)


b. Cuerda del tímpano (VII par)
c. Glosofaríngeo (IX par)
d. Hipogloso (XII par)

222. El gráfico detalla la estructura de la pared del tubo digestivo


intestinal ¿Cuál de las siguientes asociaciones es correcta?
(unidad 2, sesión 7, logro 1: La pared y músculo liso
gastrointestinal )

a. “1” – peristaltismo
b. “2” – secreción enzimática
c. “3” – deriva del mesodermo
d. “4” – doble hoja de tejido graso
223. En el caso de un paciente con gastrinoma (tumor productor de gastrina), la presencia de úlceras duodenales y erosión
de la mucosa gástrica, se debe principalmente a…….
(unidad 1, sesión 3, logro 2: describir las hormonas gastrointestinales: estímulo y funciones de la gastrina y
colecistoquinina)

a. la acción directa de la gastrina sobre la célula principal


b. la sobre expresión de los receptores “G” en la célula parietal
c. el exceso de HCl por estímulo de receptores CCK-B en la célula parietal
d. el exceso de HCl por estímulo directo de receptores de acetilcolina en la célula parietal

224. El reflejo entero gástrico se caracterizan por:


(unidad 2, sesión 13, logro 6: Motilidad del intestino grueso: contracciones segmentarias, movimientos en
masa defecación y reflejo gastrocólico)

cc. favorecer la motilidad gástrica gracias a la CCk


dd. inhibir la motilidad gástrica y estimular la secreción ácida
ee. movilizar grandes volúmenes desde el estómago al duodeno
ff. originarse debido a la distensión duodenal y presencia del quimo ácido
225. Mauricio tiene dificultad para deprimir el paladar y elevar la parte posterior de la lengua. En este caso estará afectado un
músculo ………………., específicamente el músculo …………….
(Unidad 2, sesión 10, logro 2: Músculos de la lengua: clasificación, identificación y sus funciones)

a. intrínseco – longitudinal inferior


b. extrínseco – palatogloso
c. extrínseco – transverso
d. extrínseco – estilogloso

226. Una de las funciones del músculo señalado es:


(Unidad 2, sesión 8, logro 3: Describir el Piso de la boca:
estructuras blandas que la conforman)

gg. deprimir la lengua


hh. elevar el paladar blando
ii. deprimir el hioides cuando la mandíbula está fija
jj. deprimir la mandíbula cuando el hioides está fijo

227. Paciente varón de 30 años es evaluado por probable enfermedad de Chagas, cursa con problemas de motilidad del
colon; los estudios de biopsia determinan ausencia de células ganglionares. Según el
gráfico
¿cuál es la capa en la que se determina la ausencia de dichas células?
(unidad 1, sesión 2, logro 1: describir las generalidades de la estructura del
tubo digestivo: esófago, estómago intestino delgado y grueso)

a. Mucosa - 1
b. Muscular propia – 1
c. Muscular de la mucosa - 2
d. Muscular propia - 3

228. ​Paciente varón de 32 años, que acude a centro de salud por presentar de forma progresiva desde hace 1 año dificultad
para ingerir alimentos sólidos y luego líquidos; refiere regurgitaciones alimentarias y marcada pérdida de peso (15 kilos).
Radiografía baritada (sustancia de contraste​) ​de esófago se muestra en la figura. El presente caso se explica
por……………….
(Unidad 2, sesión 12, logro 4: Identificar y describir la función de los esfínteres esofágicos)

a. aumento de la peristalsis esofágica


b. relajación incompleta del esfínter pilórico
c. relajación incompleta del esfínter esofágico inferior
d. perdida de producción de PIV y óxido nítrico en el esfínter
esofágico superior
229. En condiciones normales, el ingreso de 600 ml de líquido es el estómago provoca un aumento de presión
intragástrica de unos 12 cm de H​2​O. Después de una vagotomía (corte del nervio vago) es de esperar que el ingreso del
mismo volumen de líquido ocasionede la presión intragástrica.
(Unidad 2, sesión 13, logro 1: Describe la Motilidad gástrica: relajación receptiva)

a. la disminución
b. la no variación
c. un aumento mayor
d. un aumento similar o igual

230. Un niño de 2 años es llevado a la consulta por diarrea persistente, edema de las extremidades y falta de crecimiento en
relación a su edad. Los análisis de sangre revelan que tiene concentración plasmática baja de proteínas (hipoproteinemia).
Como parte del estudio se coloca colecistokinina (CCK) endovenosa y se recoge muestras del líquido duodenal por
endoscopía; el resultado del líquido confirma incapacidad para hidrolizar proteínas a un pH neutro, esta situación mejora al
añadir una pequeña cantidad de tripsina. El paciente probablemente esté sufriendo la falta congénita de ………….
(Unidad 4, sesión 26, logro 2: Explicar la Digestión y absorción de las proteínas y sus alteraciones)
a. PEPT-1
b. pepsinógeno
c. enterocinasa
d. carboxipeptidasas
231. Paciente mujer de 35 años acude a consulta por sensación de sequedad y lesiones en cavidad oral. Al examen se
observa atrofia de la mucosa, fisuras y úlceras; nota además sequedad e irritación de la córnea y aumento del tamaño de
las glándulas parotídas. Su diagnóstico más probable es artritis reumatoide; el hallazgo más probable en una biopsia de
glándula parótida es……..….
(Unidad 3, sesión 18, logro 3: Describe las enfermedades más frecuentes de las glándulas salivales)
e. Presencia de acinos normales con hiperplasia de células ductales
f. Gran infiltración de linfocitos y células plasmáticas
g. Hiperplasia de acinos glandulares serosos
h. Gran infiltrado de linfocitos y neutrófilos
232. Un hombre de 42 años de edad se presenta al médico con una historia de 1 año de evolución, caracterizado por dolor
abdominal bajo y diarreas con crisis sanguinolentas. Manifiesta además pérdida de peso de 8 kg durante este periodo. La
colonoscopía revela lesión difusa en el colon con afectación del recto. La biopsia de estas lesiones revela adelgazamiento de
la pared, inflamación y ulceración de la mucosa y sub mucosa. El diagnóstico más probable en este caso es:
(Unidad 4, sesión 28, logro 5: Describe la Enfermedad inflamatoria intestinal. Generalidades, morfología y
características)
i. sindrome de colon irritable
j. enfermedad de Crohn
k. colitis ulcerativa
l. sprue celiaco
233. ​Dos estudiantes deciden tomar un receso para comer una hamburguesa a la hora del almuerzo. Antes de llegar a la
cafetería, impulsos nerviosos provenientes del complejo vagal dorsal iniciarán la secreción de ácido gástrico por la liberación
dedesde el sistema nervioso entérico.
(Unidad 3, sesión 20, logro 2: Regulación de la secreción gástrica: estimulación, fases de la secreción)
m. Serotonina
n. Colecistoquinina
o. Péptido inhibidor vaso activo
p. GRP (péptido liberador de gastrina)

234. Un niño de cuatro años de edad es llevado a la consulta por cuadros diarreicos frecuentes caracterizados por
heces pálidas, voluminosas y fétidas; al examen físico presenta bajo peso y talla para la edad. Se mide la concentración
de cloruro en el sudor y se encuentra que sus valores son muy elevados. La alteración más importante a nivel de
células ductales del páncreas tiene relación directa con la conductancia de………… ​(Unidad 3, sesión 23, logro 5
Explica la Secreción pancreática: formación del jugo pancreático, influencia de la velocidad de flujo y regulación)
q. Bicarbonato
r. Potasio
s. Sodio
t. Cloro
235. Se evalúa los valores séricos de las siguientes sustancias a un paciente con enfermedad hepática terminal; en este
paciente se espera encontrar la combinación con la letra …………
(Unidad 3, sesión 22, logro 5: Describe las Pruebas de función hepática, la Insuficiencia hepática, encefalopatía
hepática e hipertensión portal)
Glucosa Amoniaco Albúmin
a
a. Aumenta Disminuida Disminui
da da
b. Disminui Aumentada Aumenta
da da
c. Aumenta Aumentada Aumenta
da da
d. Disminui Aumentada Disminui
da da

236. Una mujer de 35 años de edad HIV positiva, se presenta al médico con dolor abdominal en cuadrante superior
derecho e ictericia. La paciente refiere haber tenido múltiples episodios de ictericia durante los últimos 10 años. Los
exámenes para determinar hepatitis viral, dieron positivos para Hepatitis B, siendo catalogado el caso como hepatitis
crónica con alteración funcional. En un examen de sangre ¿cuál de los siguientes parámetros está disminuido?
(unidad 3, sesión 22, logro 5: Pruebas de función hepática, Insuficiencia hepática, encefalopatía hepática e
hipertensión portal)
u. Albumina
v. Bilirrubina
w. Fosfatasa alcalina
x. Tiempo de protrombina

237. En el reflejo peristáltico del intestino delgado ¿Cuál de los siguientes eventos sucede en la porción caudal del bolo
alimenticio?
(Unidad 2, sesión 13, logro 4: Explicar la Motilidad del intestino delgado: Contracciones segmentarias y peristálticas)
y. Acción del péptido inhibidor vasoactivo (VIP) en el músculo circular
z. Acción del NO (óxido nítrico) en el músculo longitudinal
aa. Contracción del músculo longitudinal interno
bb. Acción de acetilcolina en el músculo circular

238. Un varón de 58 años de edad con enfermedad de Crohn severo fue sometido a una resección ileal. Después de la
cirugía este paciente padecerá de esteatorrea, esto se explica porque …..………..
(unidad 4, sesión 26, logro 4: Explica las alteraciones en la Absorción de lípidos)
cc. se inhibe la acción de la 7 alfa hidroxilasa
dd. el pool de ácidos biliares se incrementa
ee. hay mala absorción de ácidos biliares
ff. el páncreas no secreta lipasa

239. En un experimento se inserta un balón en el estómago de un voluntario, se infla poco a poco mientras que se
vigilan las presiones intraluminales. Aunque el volumen del balón aumenta considerablemente, las presiones
permanecen constantes. Esta relación volumen-presión se explica por la liberación local de …………..
(Unidad 2, sesión 13, logro 1 Explica la Motilidad gástrica: relajación receptiva)
gg. acetil colina y gastrina
hh. norepinefrina y óxido nítrico
ii. colecistoquinina y óxido nítrico
jj. óxido nítrico y péptido inhibidor vasoactivo

240. ¿Cuál de las siguientes alternativas es una característica de la secreción exocrina del páncreas?
(Unidad 3, sesión 23, logro 5: Secreción pancreática: formación del jugo pancreático, influencia de la velocidad de
flujo y regulación)
kk. Es hipotónica respecto al plasma
ll. Su mayor estímulo se da en la fase intestinal
mm. Es estimulada por la presencia de bicarbonato en el duodeno
nn. La secreción enzimática es estimulada principalmente por la secretina
241. Las estructuras en el hígado que permite que los productos metabólicos unidos a proteínas tengan acceso a las
membranas basolaterales de los hepatocitos, son…..
(Unidad 3, sesión 21, logro 4-5: Explica la Organización micro estructural del hígado)
oo. los canalículos
pp. las células de Ito
qq. las fenestras sinusoidales
rr. las uniones intercelulares herméticas

242. La composición de la bilis es modificada conforme fluye por los conductillos biliares. Durante este tránsito se
espera que aumente la concentración de……. ​(Unidad 3, sesión 22, logro 2: Describe la Secreción biliar, visión general
del sistema biliar extrahepático y composición de la bilis)
ss. Ig A
tt. Glucosa
uu. Protones
vv. Vitamina A

243. Se mide experimentalmente el contenido gástrico de dos personas. La persona “A” tiene alto contenido de grasa y la
persona “B” tiene un contenido isotónico ¿Cuál de las siguientes es correcto respecto al vaciamiento gástrico? ​(Unidad 2,
sesión 13, logro 2: Describe la Motilidad y vaciamiento gástrico)
ww. Hay ralentización del vaciado gástrico solo en “A”
xx. El vaciamiento gástrico es más rápido en ambos
yy. Hay ralentización del vaciado gástrico solo en “B”
zz. Hay ralentización del vaciado gástrico en ambos casos

244. El examen endoscópico de un paciente con hipertensión portal grave revela venas tortuosas que sobresalen hacia la luz
del esófago. El paciente recibe tratamiento quirúrgico mediante la colocación de una derivación que conecta la vena porta a
la vena cava. Después de la operación el riesgo de encefalopatíay el riesgo
de sangrado de várices ……………..​(Unidad 3, sesión 22, logro 5: Describe la Insuficiencia hepática, encefalopatía hepática e
hipertensión portal)
aaa. disminuirá / disminuirá
bbb. disminuirá / aumentará
ccc. aumentará / disminuirá
ddd. aumentará / aumentará

245. Un paciente varón de 18 años de edad acude al médico para sus exámenes de rutina. Sus resultados de laboratorio
muestran un valor de bilirrubina sérica de 4 mg/dl y una bilirrubina directa de 0,3 mg/dl. Las pruebas de función hepática
son normales. La alteración que explica mejor este caso es por la deficiencia de ………………..
(Unidad 3, sesión 22, logro 3: Explica la Producción y excreción de bilirrubina. Tipos de bilirrubina e ictericia)
eee.transaminasas
fff. hemo oxigenasa
ggg. la 7 alfa hidroxilasa
hhh. glucuronil transferasa

246. Un hombre de 57 años de edad es llevado a urgencias con hematemesis masiva rojo brillante, a su llegada se halla
inconsciente con PA: 80/40 mm Hg y FC: 124 lat/min. Luce ictérico con presencia de “arañas vasculares en el tórax anterior y
extremidades”, abdomen distendido con signo de oleada positiva. Se encuentra esplenomegalia y pérdida de la masa
muscular en extremidades. La anastomosis vascular responsable del sangrado en este paciente es ………….…..
(Unidad 3, sesión 21, logro 2: Describe las anastomosis porto sistémicas)
iii. vena gástrica izquierda y vena ácigos
jjj. arteria gástrica izquierda y vena ácigos
kkk. vena paraumbilical y vena epigástrica inferior
lll. vena gástrica izquierda y vena esofágica superior
247. Un estudiante de medicina está comiendo un plato de comida a base de champiñones, espárrago y salsa de soya. El
estímulo del sabor umami contenido en todos estos alimentos viaja a través del nervio………………..
(Unidad 2, sesión 10, logro 3: Describe la irrigación e inervación de la lengua)
mmm. Lingual
nnn. Hipogloso
ooo. Glosofaringeo
ppp. Cuerda del tímpano
248. Una paciente de 30 años de edad es sometida a una cirugía en oído medio derecho por un problema de
otoesclerosis. Luego de la cirugía refiere alteración sensitiva de la lengua. Al evaluar el caso usted esperaría
encontrar……….
(Unidad 2, sesión 10, logro 5: Describe la Irrigación e inervación de la lengua)
qqq. Alteración en la sensación del dolor y temperatura en el tercio posterior de la lengua
rrr. Alteración en la sensación del dolor en los dos tercios anteriores de la lengua
sss. Alteración en la sensación del gusto en el tercio posterior de la lengua
ttt. Sensación del dolor, tacto y temperatura conservadas

249. En un paciente de 45 años de edad con colestasis biliar, se encuentra una elevación de los niveles sanguíneos de
fosfatasa alcalina hasta 3 veces la cifra normal. ¿Cuál de las siguientes alternativas estará también elevada como evidencia
del daño de la vía biliar?
(Unidad 3, sesión 22, logro 5: Pruebas de función hepática, Insuficiencia hepática, encefalopatía hepática e
hipertensión portal)
uuu. Tiempo de protrombina y albúmina sérica
vvv. Transaminasas hepáticas (ALT y AST)
www. Gamma glutamil transpeptidasa
xxx. Glucoronil transferasa

250. Experimentalmente se incrementa la velocidad de la secreción salival con una sustancia, en el análisis de la
composición de esta saliva obtenida se espera encontrar…………..
(Unidad 3, sesión 17, logro 5 : Explica la Influencia de la velocidad del flujo salival en la composición de la saliva)
yyy. disminución de la concentración de bicarbonato que supera la concentración plasmática
zzz. aumento de la concentración de cloro y sodio que supera la concentración plasmática
aaaa. aumento de la concentración de bicarbonato que supera la concentración plasmática
bbbb. disminución de concentración de potasio y bicarbonato

251. Lactante de 3 meses de vida es atendido por presentar diarrea, se administra una solución de glucosa y
electrólitos por vía oral. La proteína de membrana apical que explica la capacidad de esta solución para
proporcionar aporte de glucosa e hidratación es ………..
(Unidad 4, sesión 26, logro 1: Explica la Digestión y Absorción de los hidratos de carbono. Alteraciones)
cccc. CFTR
dddd. SGLT-1
eeee. GLUT-2
ffff. GLUT-5

252. Paciente ha sufrido herida de bala en el abdomen, se le ha tenido que extirpar el segmento medio y distal del
ileon. En este caso la síntesis hepática de sales biliares estará …..…..
(Unidad 3, sesión 22, logro 4: Explica la formación, función y Circulación entero hepática de lasa sales biliares)
gggg. Sin cambios en el ritmo de síntesis
hhhh. Disminuida por inhibición de la enzima colesterol 7 alfa hidroxilasa
iiii. Incrementada por estímulo de la enzima colesterol 7 alfa hidroxilasa
jjjj. Incrementada por inhibición de la enzima colesterol 7 alfa hidroxilasa

253.Un varón de 75 años ingresa al consultorio por presentar ictericia marcada de piel y las escleras. El estudio del
paciente mostró que presentaba un tumor que obstruía la totalidad del conducto hepático común. ¿Cuál de los siguientes
conductos se encontrará dilatado en este paciente?
(Unidad 3, sesión 21, logro 6: Describir el árbol biliar intrahepático)
kkkk. de Wirsung
llll. de Hering
mmmm. colédoco
nnnn. cístico
254. Correlaciones las dos columnas y marque la fórmula correcta: ​(Unidad 4, sesión 28, logro 1: Diarrea: definición, mecanismos:
osmótica, secretoria y exudativa)

1. Enfermedad Hirschsprung( ) heces con moco y sangre


2. Diarrea osmótica( ) intolerancia a lactosa
3. Diarrea secretoria( ) aganglionosis congénita
4. Diarrea exudativa( ) canales de Cl- en las células de la cripta a.- 4231b.- 1234c.- 2143​d.-

4213

255. La vena umbilical obliterada del hígado después del nacimiento se transforma en el ligamento:
(Unidad 3, sesión 21, logro 1: Hígado: relación con la pared abdominal, caras, lóbulos, ligamentos , hilio hepático)
oooo. cruzado
pppp. redondo
qqqq. coronario
rrrr. falciforme

256. Llega a su guardia nocturna una madre que trae a su RN masculino de 2 semanas de vida con mal estado general y
sequedad de mucosas. Usted observa que lacta ávidamente, pero a las 2 horas presenta vómito postprandial no bilioso en
proyectil. Al realizar la historia clínica, descubre que el lactante recibió profilaxis con macrólidos para tos ferina. Usted
sospecha principalmente en:
(Unidad 1, sesión 4, logro 4: Desarrollo y anomalías del intestino anterior)
ssss. estenosis pilórica hipertrófica congénita
tttt. fistula traqueo esofágica
uuuu. estenosis duodenal
vvvv. atresia duodenal

257. En la regulación del apetito y la saciedad, la estimulación experimental crónica del núcleo ventro medial del
hipotálamo producirá:
(Unidad 1, sesión 3, logro 4: explica los mecanismos de control del apetito y saciedad)
wwww. afagia
xxxx. obesidad
yyyy. hiperfagia
zzzz. activación de neuronas relacionadas a NPY

258. Paciente mujer de 25 años acude por dolor en fosa ilíaca derecha que empeora al toser o caminar, asociada a náuseas y
vómitos por lo cual acude a emergencia. Dos días después de realizarle una apendicectomía, la paciente desarrolla fiebre
alta (39 °C), está hipotensa y presenta dolor abdominal. La laparotomía exploratoria muestra un gran volumen de sangre en
la cavidad peritoneal por lesión de un vaso producida durante la apendicectomía.
¿Cuál de las siguientes arterias debe ligarse para detener la hemorragia?
(Unidad 4, sesión 27, logro 4: Irrigación arterial del colon, recto y conducto anal)
aaaaa. cólica derecha y arteria rectal superior.
bbbbb. ileocólica y arteria cólica media.
ccccc. mesentérica superior.
ddddd. ileocólica.

289. La onda peristáltica secundaria del esófago se caracteriza por ser originada ………
(unidad 2, sesión 12, logro 3: Motilidad esofágica: fases y características)
eeeee. por el plexo de meissner del esófago
fffff. por el plexo mientérico del esófago
ggggg. por el reflejo de la deglución
hhhhh. durante la masticación

290. ¿Cuál de los siguientes es una causa de ictericia con bilirrubina conjugada aumentada?
(Unidad 3, sesión 22, logro 3: Producción y excreción de bilirrubina. Tipos de bilirrubina, ictericia)
iiiii.Ictericia del recién nacido
jjjjj. Obstrucción del colédoco
kkkkk. Anemia hemolítica
lllll.Gran hematoma
291. En relación a la absorción de nutrientes, la absorción de dipéptidos y tripéptidos a nivel de las células epiteliales del
intestino delgado, se da principalmente debido a:
(Unidad 4, sesión 26, logro 2: Digestión y absorción de las proteínas. Alteraciones)
mmmmm. el incremento de los canales de Cl- en la membrana apical
nnnnn. la gradiente de bicarbonato en la membrana basal
ooooo. la gradiente de iones H+ en la membrana apical
ppppp. la gradiente de Na+ en la membrana apical

292. Paciente de 20 años es traído a la emergencia por presentar diarreas desde hace 2 días. Familiar refiere que las
deposiciones son líquidas y abundantes, al examen luce deshidratado y se plantea que la diarrea es producida por una
toxina que estimula la transformación de ATP a AMPc con apertura de canales de Cl- y pérdida de agua. El tipo de diarrea
más probable es:
(Unidad 4, sesión 28, logro 1: Diarrea: definición , mecanismos: osmótica, secretoria y supurativa)
qqqqq. osmótica
rrrrr. exudativa
sssss. secretoria
ttttt. por intolerancia a lactosa
293. Un niño fue operado por una obstrucción intestinal, observándose la presencia de divertículo de Meckel. Según lo
referido, marque lo correcto:
(Unidad 1, sesión 5, logro 2: identifica las anomalías del desarrollo del intestino medio: onfalocele y gastrosquisis
(diferencias), Divertículo de Meckel)
uuuuu. el 50% de la población lo presenta
vvvvv. se localiza en el íleon muy cerca al yeyuno
wwwww. puede poseer tejido gástrico o pancreático
xxxxx. se produce por una mala rotación de los intestinos

294. Marque la alternativa correcta respecto a la


estructura marcada en el gráfico:
(Unidad 3, sesión 22, logro 2: Secreción biliar.
Visión general del sistema biliar extrahepático y
composición de la bilis)

yyyyy. Se halla a 2 centímetros debajo


de la papila duodenal mayor
zzzzz. Llega el conducto colédoco y
pancreático principal
aaaaaa. Llega el conducto hepático
común y pancreático principal
bbbbbb. Llega el conducto pancreático accesorio

295. ¿Cuál de las siguientes moléculas se encontrará aumentada en el citoplasma de las células parietales de un
paciente con sindrome de Zollinguer Ellison? ​(Unidad 3, sesión 20, logro 4: Enfermedad ulcerosa péptica: úlcera
gástrica, duodenal. síndrome de Zollinger – Ellison)
cccccc. Péptido liberador de gastrina (GRP)
dddddd. Proteína G estimulante (GS)
eeeeee. Inositol Trifosfato (IP3)
ffffff. AMP cíclico (AMPc)

296. Los fármacos inhibidores de la bomba de protones, actúan bloqueando la ………..…….. ​(Unidad 3, sesión 20, logro 3:
Regulación de la secreción gástrica: inhibición, Secreción de pepsinógeno y factor intrínseco)
gggggg. anhidrasa carbónica
hhhhhh. ATPasa H+/K+ en la membrana luminal
iiiiii. ATPasa H+/K+ en la membrana basolateral
jjjjjj. ATPasa Na+/K+ en la membrana basolateral
297. Un paciente fue diagnosticado de gastritis autoinmune, ¿cuál de las siguientes alternativas es ​FALSA ​respecto a esta
enfermedad? ​(Unidad 3, sesión 20, logro 5: Gastritis crónica: helicobacter pylori, autoinmune. Tipos de gastritis)
kkkkkk. Afecta principalmente el fondo y cuerpo gástrico
llllll. Se produce hiperplasia de células G secundaria a la aclorhidria
mmmmmm. El propio sistema inmune destruye principalmente las células parietales
nnnnnn. Se produce atrofia de la mucosa, aclorhidria, hipergastrinemia y déficit de vitamina B6
298.​Marque la correlación correcta: ​(Unidad 3, sesión:18, logros:1 y 2: Describe las enfermedades inflamatorias/infecciosas y
proliferativas de la cavidad oral)

1. Herpes virus( ) En relación al abuso de antibióticos


2. Candidiasis oral( ) Lesiones vesiculares como racimo de uvas
3. Eritroplaquia( ) Mega esófago
4. Enfermedad de Chagas( ) Lesión pre cancerígena

a.- 2431b.- 1234c.- 4123 ​d.- 2143

299. En un paciente con insuficiencia renal crónica, el déficit en la absorción de calcio a nivel del enterocito se debe a lo
siguiente: ​(Unidad 4, sesión 26, logro 6: Explica la Absorción de calcio y hierro)
oooooo. No se convierte la 25 hidroxicolecalciferol a 1,25 dihidroxicolecalciferol
pppppp. No se convierte la 1,25 dihidroxicolecalciferol a 25 hidroxicolecalciferol
qqqqqq. Existe un descenso de la alfa 25 hidroxilasa renal
rrrrrr. Se incrementa la producción de Calbindina
300. ¿Cuál de las glándulas salivales es responsable del mayor porcentaje del volumen de la saliva en condiciones basales?
a. Parótida
b. Sub Palatinas c.
Sublinguales d.
Submaxilares

301. La lengua está recubierta por epitelio:


c. pseudoestratificado columnar no queratinizado
d. plano estratificado no queratinizado
e. pseudoestratificado columnar ciliado
f. plano estratificado queratinizado

302. El esfínter anal interno tiene musculatura …………… y tiene control ………………….
g. lisa / voluntario
h. lisa / involuntario
i. esquelética / simpático
j. esquelética / parasimpático

303. La arteria aorta proporciona la irrigación al tubo digestivo ¿cuál de las siguientes arterias proporciona la irrigación al ángulo
cólico derecho?
k. Mesentérica superior
l. Mesentérica inferior
m. Frénica inferior
n. Tronco celiaco

304. Paciente de 26 años que le cuenta en su historia clínica que cada vez que almuerza, a los 20 minutos tiene deseo de
defecar. Le comenta que su hijo de 1 mes le pasa lo mismo pero más intenso. Esto se explica por el reflejo
…………………, el cual estáen el paciente.
o. colicoileal / normal
p. colicoileal / alterado
q. gastrocolico / normal
r. gastrocolico / alterado
305. La región del estómago que se comunica con el duodeno se denomina:
s. pilórica
t. cardias
u. cuerpo
v. fórnix
306. Acude a consulta un paciente que fue diagnosticado de ulcera péptica 3 días antes. Luego de múltiples pruebas diagnósticas,
se concluye que el paciente presenta un tumor secretor de gastrina ¿Cuál de las siguientes situaciones estará incrementada?
w. Distención gástrica
x. Inhibición del vaciado gástrico
y. Secreción de ácido clorhídrico (HCl)
z. Inhibición de la secreción de pepsinógeno
307. En el sistema digestivo, el control del apetito esta dado por un complejo sistema de sustancias y órganos integradores los
cuales regulan la ingesta de alimentos. Laes una sustancia oroxígena y es sintetizada
por el ……………………..
aa. leptina / intestino
bb. grelina / intestino
cc. leptina / estómago
dd. grelina / estómago
308. Sobre el control autónomo del sistema digestivo, marque la alternativa correcta:
ee. La inervación dada por el sistema simpático es de tipo preganglionar.
ff. El sistema parasimpático usa como neurotransmisores a la acetilcolina y la noradrenalina.
gg. El nervio vago (par craneal X) le da inervación simpática a la mayoría del sistema digestivo.
hh. En el sistema simpático, los nervios responsables hacen una primera sinapsis en ganglios próximos al órgano a
inervar.
ii. En la inervación de tipo parasimpático, solo interviene el plexo submucoso, sin embargo, en la de tipo
simpático intervienen tanto el submucoso como el mientérico.
309. Con respecto a la actividad eléctrica del sistema digestivo, marque la alternativa correcta
jj. Corresponden a potenciales de acción que están presentes de forma continua y le dan la capacidad de
perístasis autónoma al sistema digestivo.
kk. La frecuencia de las ondas lentas no se ve influenciada por la actividad neural ni las hormas
gastrointestinales.
ll. En el estómago las ondas lentas se dan en una frecuencia de 6 por minuto.
mm. Las ondas lentas son cambios lentos y ondulantes del potencial en reposo.
nn. La frecuencia de las ondas lentas va de 6 a 12 ondas por minuto.

310. Ante una lesión del IX par craneal, el músculo…se altera en su función.
oo. palatogloso
pp. estilofaríngeo
qq. palatofaríngeo
rr. constrictor superior

311. Un varón de 50 años es sometido a extirpación de duodeno y parte proximal de yeyuno. La pérdida de estímulo hormonal
en el páncreas para la secreción enzimática se explica por la pérdida de las células ……………………
ss. Parietales, productoras de factor intrínseco
tt. “K” productoras de factor intrínseco
uu. “M” productoras de CCK
vv. “I” productoras de CCK
312. ​Respecto al mecanismo de la defecación ¿Cuál de las siguientes afirmaciones es correcta?
ww. Se produce contracción refleja del esfínter anal interno
xx. Se produce contracción o relajación del esfínter anal externo por señales de la corteza cerebral
yy. La presencia de materia fecal en el recto estimula la contracción del sigmoides por los nervios pélvicos
simpáticos
zz. En la posición de “cuclillas” el músculo puborectal se halla contraído favoreciendo la evacuación de la materia
fecal

313. Un niño de tres años llega a emergencia con disfagia (dificultad para tragar), salivación y llanto. Se sospecha de ingesta de
cuerpo extraño: moneda en el esófago; al ser evaluado se constata en una radiografía presencia de cuerpo extraño a nivel de C6 y
C7 (6° y 7° vértebra cervical). El cuerpo extraño estará suspendido a nivel del estrechamiento producido por el ………..
aaa. cayado aórtico
bbb. hiato esofágico
ccc. músculo cricofaríngeo
ddd. bronquio principal izquierdo

314. En el caso de un paciente con un tumor productor de gastrina, la presencia de úlceras duodenales y erosión de la mucosa
gástrica se debe principalmente a…….
eee.la acción paracrina de la gastrina sobre la célula parietal
fff. el exceso de HCl por estímulo de receptores CCK-B en la célula parietal
ggg. la sobre expresión de los receptores “G” para gastrina en la célula parietal
hhh. el exceso de HCl por estímulo directo de receptores “H” en la célula parietal

315. La onda peristáltica secundaria del esófago se caracteriza por ser originada ………
iii. por el plexo de submucoso del esófago
jjj. por el plexo mientérico del esófago
kkk. por el reflejo de la deglución
lll. durante la masticación

316. Marque lo correcto sobre las ondas lentas en el tubo digestivo


mmm. No son despolarizaciones
nnn. Son potenciales de acción subumbrales
ooo. Se constituyen de despolarizaciones y repolarizaciones
ppp. Son rítmicas y generadas por el sistema nerviosos autónomo

317. Recién nacido que presenta protrusión de contenidos abdominales los cuales no están cubiertos por peritoneo y salen de la
cavidad abdominal a través de un defecto de la pared. ¿Cómo se denomina a la afección que presenta este paciente?
qqq. Onfalocele
rrr. Atresia biliar
sss. Gastrosquisis
ttt. Divertículo de Meckel

318. Experimentalmente se utiliza atropina (anticolinérgico) para inhibir la secreción de gastrina, sin embargo, la secreción de
esta hormona se sigue dando ante estímulos vagales. Esta situación se explica porque la atropina:
uuu. no bloquea la acción del péptido GRP
vvv. solo inhibe la acción del péptido GRP en la célula G
www. inhibe la acción de acetilcolina e histamina en la célula G
xxx. bloquea parcialmente la bomba de protones en la célula G
319. Niña de 4 días es llevada a la emergencia pediátrica por presentar llanto constante, la madre refiere coloración azulada de
labios al momento de lactar, acompañado de tos persistente y dificultad respiratoria así como distención abdominal. Se le coloca
sonda nasogástrica para alimentación notando que retorna a la cavidad oral en todos los intentos. ¿Cuál es la anomalía del
desarrollo en este caso?
yyy. Solo fístula traqueo esofágica
zzz. Fístula traqueo esofágica proximal y distal
aaaa. Atresia esofágica proximal con fístula traqueo esofágica distal
bbbb. Atresia esofágica distal con fístula traqueo esofágica proximal
320. Paciente varón de 36 años es traído a la emergencia luego de sufrir un accidente de tránsito, presenta traumatismos múltiples
en cabeza y tronco. Al examen físico se evidencia hematoma en hemicara izquierda, ligera protrusión y caída del lado izquierdo del
maxilar inferior, por lo que se realiza una tomografía donde se halla una fractura de la apófisis coronoides del maxilar inferior.
¿Qué músculo está relacionada directamente con esta situación?
cccc. Masetero
dddd. Temporal
eeee. Buccinador
ffff. Pterigoideo medial

321. Un paciente refiere no percibir algunos sabores. Al examen físico constata alteración del sabor dulce y umami.
¿Cuál de los siguientes nervios estará alterada su función?
gggg. Cuerda del tímpano (VII par)
hhhh. Lingual (rama del V par)
iiii. Glosofaríngeo (IX par)
jjjj. Hipogloso (XII par)

322. A los pocos días de nacido, regresa a neonatología un niño con problemas de motilidad del colon; los estudios determinan
ausencia congénita de células ganglionares. Según el gráfico ¿cuál es la capa en la que se determina la ausencia de dichas células?
kkkk.Mucosa - 2
llll. Muscular propia - 2
mmmm.Muscular propia - 3
nnnn. Muscular de la mucosa - 3

323. Con respecto al control autonómico en el tracto gastrointestinal y en relación a su fisiología. ¿Cuál es la función del
sistema nervioso parasimpático en el tracto gastrointestinal?
oooo. Inhiben la contracción muscular y estimulan la secreción de sustancias a nivel de la submucosa
pppp. Estimulan la contracción muscular y estimulan la secreción de sustancias a nivel de la mucosa
qqqq. Inhiben la contracción muscular e inhiben la secreción de sustancias a nivel de la submucosa
rrrr. Estimulan la contracción muscular e inhiben la secreción de sustancias a nivel de la mucosa

324. Un estudiante que está preocupado por su examen parcial, no ha desayunado ni almorzado; cuando al fin ingiere alimentos,
esto le provoca el aumento de los movimientos musculares del tracto gastrointestinal y la sensación de defecar. ¿Qué reflejo se
ha activado?
ssss. Entero-gástrico
tttt. Gastro-cólico
uuuu. Cólico-ileal
vvvv. Ileo-ileal
325. ¿De qué par craneal es rama el nervio palatino mayor?
wwww. Vago
xxxx. Hipogloso
yyyy. Trigémino
zzzz. Palatogloso

326. ¿En cuál de las fases de la deglución la epiglotis separa la vía respiratoria de la digestiva?
aaaaa. oral
bbbbb. laríngea
ccccc. faríngea
ddddd. esofágica

327. Los péptidos intestinales se pueden clasificar como sustancias endocrinas, neurocrinas y paracrinas, dentro de las
paracrinas se encuentran la somastotatina e histamina. Marque la respuesta correcta
eeeee. La somastotatina es sintetizada por las células B de la mucosa gástrica
fffff.La histamina actúa estimulando su receptor tipo H1 en la mucosa gástrica
ggggg. La histamina es sintetizada por células de tipo paracrino de las glándulas gástricas
hhhhh. La somatostatina presenta dentro de sus funciones la estimulación de la secreción de H​+

328. En relación a los órganos intraabdominales y sus estructuras de fijación, elija la alternativa correcta a.El
mesenterio permite la suspensión e irrigación de los órganos retroperitoneales
b. Tanto el hígado como la vesícula biliar se encuentran ubicados a nivel del flanco derecho
c. El colon, el duodeno y el resto de intestino delgado son órganos considerados netamente como
peritoneales
d. Los ligamentos que encontramos dentro de la cavidad abdominal son el esplenorenal y el gastrofrénico e.Los
omentos van desde el estómago y la segunda porción del duodeno a otras estructuras
intraabdominales y existen dos: el omento mayor y el omento menor

329. Paciente varón de 27 años es llevado por bomberos a emergencia luego de ser asaltado y, tras resistirse, es cortado con el
pico de una botella a nivel abdominal. Al examen físico usted observa que a través de la herida se puede observar la protrusión
de asas intestinales. En relación con las capas de la pared abdominal, marque la alternativa correcta.
iiiii. La fascia de Scarpa está constituida principalmente por tejido adiposo
jjjjj. La pared abdominal está formada por piel, huesos, músculos, fascias y peritoneo parietal
kkkkk. La fascia de Camper es una estructura fibrosa que carece de grasa y su grosor es constante en toda la
pared abdominal
lllll. El músculo oblicuo externo discurre en dirección súpero-interna y se inserta en el borde inferior de las ultimas 3
a 4 costillas
mmmmm. El músculo recto del abdomen tiene como funciones comprimir el contenido del abdomen,
tensar la pared del abdomen y flexionar la columna

330. La explicación fisiológica de presentar somnolencia de 30 minutos a 1 hora después de ingerir alimentos, se explica
por:
a. Aumento del cloro intraluminal
b. Aumento del bicarbonato intraluminal
c. Disminución de ácido carbónico en la célula parietal
d. Disminución de la actividad de la anhidrasa carbónica
e. Aumento de la alcalinidad sanguínea

331. Con respecto a la irrigación arterial del colon, a que arteria corresponde la señalada con la
flecha
f. Cólica derecha
g. Cólica media
h. Cólica izquierda
i. Ileobisecoapendículocólica
j. Arco de Riolano

332. Si un paciente presentara dentro del punto de vista fisiológico, una disminución de
enterocinasa, entonces esto originaría una disminución de la actividad de:
k. la pepsina​ b.la lipasa
c. la quimotripsina
d. el peptido insulinotropo dependiente de glucosa
e. la amilasa

Se valida la opción b debido a su relación con la colipasa.


333. Con respecto a la anatomía del hígado, señale a que estructura pertenece
la marcada por el número 1.
l. Ligamento falciforme
m. Línea de Cantlie
n. Ligamento triangular
o. Ligamento coronario
p. Ligamento teres

Se valida la opción a debido a la ubicación del número 1 en donde se unen el


ligamento falciforme y ligamento coronario.

334. Se presenta un paciente, el cual presenta un antecedente de tuberculosis intestinal, por lo cual, se le resecó 80
cm de íleon distal. Desde el punto de vista fisiológico, el paciente puede presentar una de las siguientes alteraciones:
q. Disminución de la secreción de Vitamina B12
r. Aumento indiscriminado de absorción de ácido fólico
s. Disminución de la absorción de hierro
t. Aumento de la secreción de bicarbonato
u. Disminución de la absorción de ácido glicocólico

335. Un paciente es sometido experimentalmente a un fármaco que modifica el flujo salival, obteniéndose un volumen
de saliva de 288 ml en 6 horas. En este caso las concentraciones de electrolitos y bicarbonato en la saliva obtenida
varían de la siguiente manera:
a.↑ Na+, ↓ K+, ↑ Cl-, ↑ HCO3-
b.↓ Na+, ↓ Cl-, ↑ K+, ↓ HCO3-
c.↑ Na+, ↑ Cl-, ↓ K+, ↓ HCO3-
d.↑ Na+, ↑ Cl-, ↑ K+, ↑ HCO3-
e.↓ Na+, ↓ Cl-, ↓ K+, ↓ HCO3-

Se valida la opción a debido a que se puede considerar como un aumento del flujo de saliva.

336. La siguiente imagen histológica corresponde a la glándula


…………… y la estructura señalada produce ………
v. salival sublingual / mucopolisacáridos
w. oxíntica / pepsinógeno
x. salival submaxilar / ptialina
y. salival parótida / amilasas
z. antrales / gastrina

337. Paciente varón de 65 años con antecedente de hipercolesterolemia, hipertensión arterial, fibrilación auricular y
dos infartos al miocardio previos, aqueja de dolor abdominal intenso de inicio súbito, distensión abdominal, se decide
cirugía con resección de 1,5 metros de intestino delgado terminal y colon ascendente. Como consecuencia de la
resección el paciente tendrá deficiencia de:
aa. Vitamina C
bb. Tiamina
cc. Vitamina A
dd. Vitamina B1
ee. Vitamina B6

Se valida esta opción debido a que su absorción está relacionada al íleon.


338. Uno de los siguientes elementos debería hallarse con más probabilidad en el esófago de un paciente que sufre de
reflujo gastro esofágico…
ff. Pepsina
gg. Tripsina
hh. Quimiotripsina
ii. Carboxipeptidasa
jj. Ácidos biliares

339. Un paciente de 40 años cursa con anemia de 8g/dl, aqueja además de astenia y sensación de hormigueo bilateral
en los miembros inferiores, al examen se halla alteración de la sensibilidad a la vibración y camina con ampliación de la
base de sustentación. Uno de los siguientes procedimientos sería de ayuda para el diagnóstico de este paciente:
kk. Tomografía cerebral
ll. Biopsia de la mucosa gástrica
mm. Biopsia de hígado
nn. Examen de sangre oculta en heces
oo. Biopsia de Ileon proximal

340. Paciente de 60 años ingresa por caída hace 1 hora y pequeño hematoma en cuero cabelludo, al examen físico
ampliado se observa ictericia de piel y mucosas generalizada, abdomen blando, se palpa estructura quística no
dolorosa en hipocondrio derecho que corresponde a vesícula biliar (signo de Courvoisier), en los exámenes de
laboratorio se halla niveles bajos en la formación de estercobilinógeno y urobilinógeno en heces, incremento de la
bilirrubina conjugada en la orina, elevación de fosfatasa alcalina y gamma glutamil transpeptidasa séricas. El presente
cuadro puede ser explicado por:
pp. Reabsorción de hematoma
qq. Litiasis vesicular
rr. Carcinoma de la cabeza de páncreas
ss. Carcinoma con estenosis del conducto hepático común
tt. Anemia hemolítica

341. Paciente varón de 58 años con antecedente de alcoholismo crónico es diagnosticado y recibe tratamiento por
cirrosis hepática. Hace 2 días refiere familiar que tuvo cambio de conducta y no reconoce a algunos familiares. Al
examen físico, se halla ascitis, circulación colateral en abdomen, telangiectasias, en el examen de sistema nervioso:
rigidez de extremidades, ROT incrementados, desorientación en el espacio y asterixis. ¿cuál de las siguientes
circunstancias, explicaría el cuadro en este paciente?
uu. Uso de diuréticos ahorradores de potasio
vv. Incremento de actividad de ureasa bacteriana duodenal
ww. Hemorragia gastrointestinal
xx. Disminución de la producción de NH3+ en el colon
yy. Dieta normo proteica

Se valida la opción e debido al efecto sobre la encefalopatía.


Con respecto de la opción b es incorrecta debido a que hace referencia al duodeno, debería indicar colon.

342. Un recién nacido presenta vómitos biliosos poco tiempo después de cada alimento. Al preguntar a la madre sobre
antecedentes, ella recuerda que tuvo polihidramnios durante la gestación, pero un análisis de cariotipo fue normal. Una
de las siguientes es la causa más probable de estos hallazgos en el recién nacido:
zz. Enfermedad de Hirschprung
aaa. Fístula tráqueo esofágica
bbb. Divertículo ileal
ccc. Estenosis pilórica
ddd. Malrotación de la yema pancreática ventral

343. Un lobulillo hepático se puede dividir en tres zonas como se muestra en el gráfico. ¿Cuál de las siguientes
afirmaciones sobre las tres zonas es verdadera?
eee.La zona 1 tiene los menores depósitos de glucógeno
fff. La zona 3 es la primera en afectarse en una colestasis extra hepática
ggg. La zona 2 es más susceptible a la injuria por isquemia que la zona periportal d.La
zona 2 tiene la mayor capacidad de regeneración
e.La zona 1 es la que tiene menos actividad metabólica.

La pregunta 14 ha sido anulada, sin embargo, ningún estudiante se verá afectado negativamente en su
puntaje debido a esta anulación.

344. En un estudio de la secreción de hormonas gastrointestinales, sus concentraciones en la vena porta se midieron
durante perfusión luminal del intestino delgado con soluciones de diversas magnitudes de pH. ¿Qué hormona
aumentará en el plasma de la vena porta durante perfusión a través del intestino con una solución de pH 3?
hhh. CCK
iii. gastrina
jjj. GIP
kkk. motilina
lll. secretina

345. Paciente de 30 años que ingresa a causa de un traumatismo abdominal cerrado. En la exploración se aprecia
discreta palidez de piel y mucosas, auscultación pulmonar normal, taquicardia de 120 /min. Discreta distensión
abdominal y matidez en flancos; el hematocrito, que era prácticamente normal al ingreso, disminuye a 30% a las tres
horas. En la Rx de tórax se objetiva fractura de las costillas 10-11 izquierdas. La causa más probable de la anemización
en este paciente es:
mmm. traumatismo renal con hemorragia retroperitoneal.
nnn. rotura de hígado con hemoperitoneo.
ooo. rotura de bazo con hemoperitoneo​.
ppp. rotura de mesos con hemoperitoneo.
qqq. traumatismo pancreático con pancreatitis traumática.

346. Mujer de 65 años. Consulta por síndrome constitucional asociado a


dolor abdominal epigástrico progresivo irradiado a espalda, de dos meses de
evolución. El diagnostico de sospecha de adenocarcinoma de páncreas se
confirma por biopsia. Se realiza examen de imagen de abdomen para
evaluación de estructuras vasculares próximas al tumor pancreático. ¿Cuál
es el nombre de la vena señalada que está ausente, trombosada por
infiltración tumoral, condicionando circulación colateral en la pared
gástrica?

rrr. Mesentérica superior


sss. Coronaria estomaquica
ttt. Esplénica
uuu. Porta
vvv. Renal izquierda
347. Revisando la angiotomografía de un hombre de 70 años en estudio por aneurisma de aorta abdominal, el
radiólogo le informa de la presencia de una oclusión completa de la arteria mesentérica inferior. El paciente se
encuentra completamente asintomático. La oclusión de la arteria mesentérica inferior cursa de manera
asintomática en muchas ocasiones ya que el territorio que irriga puede recibir flujo proveniente de la arteria:
www. cólica derecha
xxx. gastroduodenal
yyy. Epigástrica inferior izquierda
zzz. esplénica
aaaa. cólica media

348. En las patologías de esófago es importante conocer bien la anatomía esofágica. ¿Cuál de las siguientes
afirmaciones es correcta?
bbbb. El esófago tiene capa mucosa, muscular y serosa
cccc. El esófago abdominal es más largo que el cervical
dddd. El esófago torácico pasa por detrás del cayado aórtico
eeee. El epitelio esofágico normal es ​de tipo ​cilíndrico.
ffff. El esófago abdominal es discretamente más largo que el torácico

349. A pesar de que pueda haber variaciones anatómicas, lo habitual es que el ciego sea irrigado por una rama arterial
que proviene de unas de las siguientes arterias:
gggg. Iliaca derecha
hhhh. Mesentérica inferior
iiii. Hepática derecha
jjjj. Mesentérica superior
kkkk. Iliaca izquierda

350. Ante un paciente con una cirugía abdominal urgente, el informe operatorio señala que se ha realizado una
resección de todo el duodeno y del tercio proximal del yeyuno manteniendo íntegros el estómago y todo el íleon, así
como los dos tercios distales del yeyuno. En el seguimiento nutricional del paciente ¿Qué vitamina o mineral
presentará con menor probabilidad una disminución de su absorción?
llll. Cianocobalamina
mmmm. Calcio
nnnn. Hierro
oooo. Transcobalamina
pppp. Transferrina

351. ¿Cuál de las siguientes alternativas detallan las venas que confluyen y forman la vena señalada?
qqqq. mesentérica superior, gástrica izquierda y
gastroepiploica izquierda
rrrr. mesentérica inferior, gástrica izquierda y renal
ssss. esplénica, mesentérica superior y mesentérica inferior
tttt. esplénica, pancreatoduodenal y omental izquierda
uuuu. gástrica izquierda, esplénica y hepática común

352. ¿Cuál de las siguientes sustancias forma parte de la secreción biliar?


vvvv. Tripsina
wwww. Lecitina
xxxx. Elastasa
yyyy. Quimotripsina
zzzz. Pepsina
353. El tubo digestivo contiene diferentes tipos de epitelios y glándulas. La estructura señalada es unay
está localizada en el …………...
aaaaa. glándula de Brunner / intestino
grueso​ b.cripta de Lieberkuhn / colon
c. cripta de Lieberkuhn / intestino delgado
d. glándula oxintica / estomago
e. célula parietal / estómago

Aunque las criptas de Lieberkuhn están


presentes en el intestino delgado, la
microfotografía es de epitelio de
colon.

354. ¿De qué musculo forma parte el ligamento inguinal?


bbbbb. Oblicuo externo del abdomen
ccccc. Oblicuo interno del abdomen
ddddd. Transverso del abdomen
eeeee. Psoas
fffff.Dorsal ancho

355. Señale cuál de las siguientes afirmaciones ​NO ​se relaciona a la siguiente glándula anexa del tubo digestivo
mostrada en la imagen:
ggggg. Es una glándula exocrina compuesta exclusivamente por acinos serosos
hhhhh. Su inervación está dada por el nervio auricular mayor (ramo posterior C2), que inerva la vaina
de la glándula así como la piel por encima de esta.
iiiii. Esta glándula produce una secreción mucinosa acuosa, llamada mucoserosa, a través del conducto de
Wharton.
jjjjj. Su inflamación puede ser causada por un virus de los Paramyxoviridae, que provocan una
enfermedad muy frecuentemente en niños y adolescentes
kkkkk. Es una glándula endocrina y probablemente sea de origen pancreático

Se valida la opción e debido a que no está relacionada


con la imagen.

356. ¿Cuál de las siguientes enzimas está localizada en el borde en cepillo y juega un rol en la digestión de proteínas?
lllll. Alfa dextrinasa
mmmmm. Pepsina
nnnnn. Enterocinasa
ooooo. Lactasa
ppppp. Carboxipeptidasa A​.

Se valida la opción c debido a que es correcta en relación a la pregunta.


357. Una de los siguientes sustancias, ​NO s​ irve como un buen agente emulsificante:
qqqqq. Colesterol
rrrrr. Ácidos grasos
sssss. Sales biliares
ttttt. Lecitina
uuuuu. Proteínas de la dieta

Se valida la opción e debido a que es correcta en relación a la pregunta.

358. La sustancia que estimula el crecimiento de la mucosa gástrica es:


vvvvv. Secretina
wwwww. Motilina
xxxxx. Péptido estimulante de la mucosa gástrica
yyyyy. Gastrina
zzzzz. Histamina

359. ¿Cuál de las siguientes alternativas es una función de la colecistokinina?


aaaaaa. Relajación de la vesícula para la salida de
bilis​ b.Secreción de ácidos biliares
c. Contracción del esfinter de Oddi
d. Secreción de enzimas pancreáticas
e. Contracción del duodeno

Se valida la opción b debido al efecto de la CCK sobre la vesicula biliar.

360. Con respecto a la anatomía del tronco celiaco, señale lo correcto


bbbbbb. El tronco celiaco se origina de la cara posterior de la aorta abdominal
cccccc. Es una arteria delgada que tiene un calibre entre 2 y 3 mm
dddddd. Una de sus ramas es la arteria gástrica derecha
eeeeee. La hepática común que es una de sus ramas, participa en la irrigación del estómago​.

361. Con respecto a la anatomía del duodeno, marque la respuesta correcta:


ffffff. Tiene una distribución en forma de “C”, que rodea la cola del páncreas
gggggg. La 3ra porción duodenal está contenida en la pinza vascular aortomesentérica
hhhhhh. Entre la 1ra y 2da porción se forma un ángulo, conocido como el ángulo de Treitz
iiiiii. La 4ta porción se dirige a la izquierda, hacia abajo y hacia atrás.
jjjjjj. En la tercera porción desemboca el conducto colédoco.

362. El hígado está ampliamente tapizado por peritoneo, la estructura que conecta la cara diafragmática del hígado
precisamente con el diafragma es el ligamento:
kkkkkk. teres
llllll. falciforme
c.triangular
d.hepático común
e.coronario

Se validan la opción c y e debido a que forman parte de los ligamentos que fijan el hígado al diafragma.

363. En el íleon se absorbe aproximadamente el 95% dea través de la circulación enterohepática.


mmmmmm. agua
nnnnnn. colesterol
oooooo. sales biliares
pppppp. hidróxicobalamina
qqqqqq. factor intrínseca
364. Laestimula el mecanismo paracrino de la secreción de ácido clorhídrico.
rrrrrr. histamina
ssssss. acetilcolina
tttttt. gastrina
uuuuuu. secretina
vvvvvv. somatostatina

365. En la digestión de proteinas,es el principal estímulo para convertir el pepsinógeno en pepsina.


wwwwww. la gastrina
xxxxxx. el pH ácido
yyyyyy. la acetilcolina
zzzzzz. la ptialina
aaaaaaa. la somatostatina

366. Con respecto a la somatostatina, marque lo correcto:


bbbbbbb. Es secretada por las células S del intestino
ccccccc. Induce a la producción de VIP
ddddddd. Interviene en la fase intestinal de la secreción gástrica
eeeeeee. Produce acetilcolina para estimular a la célula parietal
fffffff. No interviene en la regulación de la secreción de ácido clorhídico

367. En pecten anal, es una estructura comprendida entre:


ggggggg.la línea pectínea y los senos anales
hhhhhhh. la línea blanca y la apertura anal
iiiiiii. el esfínter anal interno y el externo
jjjjjjj. la línea anocutánea y la línea pectínea
kkkkkkk. la línea blanca y columnas anales

368. ¿Cuál de las siguientes alternativas es una proenzima pancreática?


lllllll. Tripsina
mmmmmmm. Elastasa
nnnnnnn. Quimotripsinógeno
ooooooo. Amilasa
ppppppp. Procarboxipepitidasa C.

369. En la segmentación hepática de Coinaud, el segmento hepático señalado con la flecha, corresponde a :
En la segmentación hepática de Coinaud, la flecha señala el segmentohepático.
qqqqqqq. IV
rrrrrrr. V
sssssss. VI
ttttttt. VII
uuuuuuu. VIII
370. La contracción del músculopermite la eliminación de gases (flatos) sin salida de material fecal;
es el mismo músculo cuya relajación, sobretodo en cuclillas, permite el paso del contenido fecal con menor
esfuerzo durante la defecación.
a) Isquirectal​ b)
Puborrectal
c) Esfínter anal externo
d) Esfínter anal interno

371. Paciente mujer de 54 años se presenta con náuseas, vómitos, estreñimiento, y es diagnosticada de abdomen
agudo quirúrgico; en la cirugía encuentran un vólvulo de ciego. Esta anomalía puede explicarse por::
b) Falta de rotación intestinal
c) Falta de fusión del mesenterio
d) Defecto en la formación de la cloaca
e) Falta de formación del omento mayor

372. Paciente mujer de 23 años con faringitis aguda, toma para el dolor una tableta de paracetamol con un poco de
agua. Durante la deglución, se relaja su esfínter esofágico inferior y el fondo del estómago, mientras el bolo está aún
en el esófago. ¿Qué sustancia provocara con mayor probabilidad la relajación del esfínter esofágico inferior y el fondo
del estómago en esta mujer?
f) Óxido nítrico
g) Sustancia P
h) Histamina
i) Motilina

373. Luego de tres horas dando exámenes, un alumno de medicina comienza a sentir hambre. Esta situación es
probable que sea mediada por la​ ​que es sintetizada por el​ ​:
j) leptina / intestino
k) leptina / estómago
l) grelina​ /​ estómago
m) grelina / tejido adiposo

374. Varón de 72 años, con antecedente de diabetes mellitus tipo 2, que presenta enteropatía diabética caracterizada
por estreñimiento. Este problema puede estar asociado a:
n) deficiencia de óxido nítrico
o) aumento del reflejo gastrocólico
p) disminución de la secreción de colecistocinina (CCK)
q) aumento de la secreción del péptido intestinal vasoactivo (PIV)

375. Varón de 54 años con Diabetes Mellitus tipo 2, es diagnosticado de gastroparesia debido a que presenta sensación
de llenura precoz al comer, y reflujo gastroesofágico. Esta alteración en la relajación receptiva y en el vaciamiento
gástrico lo más probable es que se deba a una alteración en:
r) el nervio vago
s) el ganglio celíaco
t) plexo submucoso
u) nervio hipogástrico

376. Varón de 67 años con tos y disminución de peso asociado a tabaquismo pesado, presenta actualmente disfagia
progresiva a alimentos sólidos. Se considera la presencia de un carcinoma de bronquio izquierdo y por esta razón le
realizan una endoscopía esofágica para descartar la posibilidad de una compresión esofágica por el tumor. Se espera
revisar el esófago en la​ ​estrechez, que está a nivel de la vértebra
v) Tercera estrechez -T6
w) Segunda estrechez - C6
x) Segunda estrechez - T4
y) Tercera estrechez -T10

377. Varón de 34 años con dolor abdominal agudo en flanco derecho que se irradia a fosa ilíaca derecha, es operado y
se encuentra un divertículo intestinal inflamado, ubicado a 93 cm de la válvula ileocecal. El origen de este divertículo
es una falla en la obliteración de:
z) Conducto vitelino
aa) Alantoides
bb) Cloaca
cc) Conducto anorectal
dd) Uraco

378. En un niño menor de dos años con divertículo intestinal, este divertículo tiene su origen en una falla en la
obliteración de:
a)Conducto anorectal
b) Conducto vitelino
c) Alantoides
d) Cloaca
e) Uraco

379. Mujer de 43 años sufre un grave accidente de tránsito y está hospitalizada en coma, es alimentada por vía
intravenosa durante varias semanas. Producto de este tipo de alimentación, se encuentra en la endoscopía atrofia de la
mucosa gastrointestinal. La causa más probable de esta atrofia son los bajos niveles séricos de la hormona:
ee) Colecistocinina
ff) Secretina
c)Gastrina
d) PIV

380. Una mujer de 30 años llega al consultorio porque se queja de dificultades para deglutir, la cual se agravan cada vez
más. Se realiza un estudio manométrico para examinar la generación de presión a lo largo del esófago. Esta prueba
revela que las contracciones como respuesta a la deglución están mal sincronizadas y que la presión en el esfínter
esofágico inferior permanece elevada. El diagnóstico más probable es​ ​producido por niveles bajos de
gg) acalasia / sustancia P​ b)
acalasia / óxido nítrico
c) enfermedad por reflujo gastrointestinal / acetilcolina
d) enfermedad por reflujo gastrointestinal / óxido nítrico

381. Paciente de 2 años, llega a emergencia por haber ingerido una moneda con la que estaba jugando. El lugar más
probable donde puede haberse quedado suspendido este objeto es a nivel del estrechamiento producido a nivel del:
hh) músculo milohiodeo
ii) músculo aritenoideo
jj) músculo cricofaríngeo
kk) constrictor superior de la faringe

382. En una apendicectomía, al realizar la incisión de McBurney en la fosa iliaca derecha, es necesario cortar los
siguientes músculos, de afuera hacia adentro:
ll) Recto – Oblicuo externo – Transverso
mm) Recto – Oblicuo externo – Oblicuo interno
nn) Oblicuo externo – Oblicuo interno – Recto
oo) Oblicuo externo – Oblicuo interno – Transverso

383. Un varón de 90 años que se encuentra postrado en cama, es referido del asilo para endoscopia por dificultad para
deglutir luego de tomar un medicamento para aliviar el dolor la noche anterior. La endoscopía revela que la píldora se
alojó en el esófago y causó una reacción inflamatoria. Lo más probable es que esto haya sido por la producción de
múltiples ondas:
pp) secundarias
qq) primarias
rr) lentas
ss) segmentarias

384. Mujer de 23 años es diagnosticada de bulimia, al examen físico se observa ulceraciones en el segundo y tercero
dedo de la mano derecha. Esto se puede deber al uso continuo de estos dedos para inducir el vómito, mediante la
estimulación del par craneal:
tt) V​ b)
IX
c) X
d) XI

385. Varón de 52 años se presenta por diarrea persistente de seis semanas de duración. En la colonoscopia se observa
un pólipo a nivel del íleon distal. El patólogo informa que se trata de un tumor neuroendócrino, probablemente
originado por las células enterocromafines del intestino. La sustancia que más probablemente esté produciendo este
tumor es:
uu) Serotonina
vv) Insulina
ww)CCK
xx) GIP

386. La fase oclusal de la masticación se realiza con la contracción de los músculos:


yy) digástricos
zz) masetero y temporal
aaa)orbicular y buccinador
bbb) pterigoideo lateral y digástrico

387. Al tomar su café en Starbucks, un estudiante de medicina sufre una quemadura de primer grado en el tercio
anterior de la superficie dorsal de la lengua. La información de dolor es transmitida por el nervio:
ccc) cuerda del tímpano
ddd) glosofarínge
o​ c)lingual
d) facial

388. Paciente es evaluado por faringitis aguda en consultorio externo. El médico de familia le solicita que abra la boca
y saque la lengua. Para realizar la acción de sacar la lengua, es necesario que se contraiga el músculo:
eee)estilogloso​ b)
geniogloso
c) palatogloso
d) transverso de la lengua

389. Paciente con síndrome de


Sjögren, presenta “boca seca”
(disminución de la producción de
saliva) y caries dental, asociada a la
pérdida de la función de tampón de
la saliva. Esta desminerilización del
diente puede comprometer a las
prolongaciones citoplasmáticas
ubicadas en los tubos huecos de la
estructura señalada con la letra:
fff) B
ggg)A
hhh) E
iii) C

390. Mujer de 32 años acude a


consulta por presentar disfagia de
progresión lenta, reflujo
gastroesofágico y vómitos desde hace
3 meses de evolución progresiva. Se le
realiza un estudio radiológico con
contraste en el que se observa
estrechamiento del esfínter esofágico
inferior (imagen). Según sus
conocimientos, este paciente se
beneficiaría con el uso de:
jjj) agonista beta adrenérgico
kkk) agonista alfa adrenérgico
c)análogo de óxido nítrico
d) análogo de Sustancia P
391. Paciente de sexo masculino de 82 años de edad ingresa a emergencia con dolor abdominal agudo y diarreas. Se le
realiza una arteriografía en la que se observa que la arteria aorta tiene un trombo ocluyendo el 95% del flujo, a nivel
del nacimiento de la arteria mesentérica inferior. ¿Cuál de las siguientes arterias podría contribuir a la irrigación
colateral del colon descendente?
a) cólica media
b) sigmoidea
c) rectal superior
d) ileocólica

392. Niño de 5 años presenta dolor esofágico y hematemesis (vómitos hemorrágicos) luego de tragarse una espina de
pescado. En la endoscopía se observa perforación del esófago distal a la cuarta estrechez esofágica. ¿Las ramas de cuál
de las siguientes arterias estarán lesionada con mayor probabilidad?
e) Gástrica izquierda
f) Bronquiales
g) Frénica inferior
h) Tiroidea inferior

393. Al ingerir una cucharada de mantequilla es muy probable que se disminuya la sensación de hambre por medio de
la activación de la vía POMC/CART (POMC=proopiomelanocortina y CART=transcripción regulada de cocaína y
anfetamina), activada directamente por la hormona:
i) colecistoquinina (CCK)
j) insulina
k) grelina
l) secretina

394. Al comer unas papitas fritas con mayonesa, el vaciamiento gástrico disminuye por efecto directo de la hormona:
a)colecistoquinina (CCK)
b) bombesina
c) motilina
d) gastrina

395. Recién nacido de dos horas es diagnosticado de hernia umbilical de 1,5 cm de diámetro; el cirujano pediatra
solicita una tomografía abdominal en donde se evidencia que la hernia umbilical está ocupada por una porción del
tracto gastrointestinal. ¿Qué porción del tracto gastrointestinal estaría ocupando esta hernia con mayor
probabilidad?
m) Íleon
n) Colon sigmoides
o) Duodeno
p) Colon transverso
396. Recién nacido de 7 horas, de parto por cesárea debido a polihidramnios (aumento del volumen del líquido
amniótico), con regurgitación de la leche materna y artificial, y no ha presentado meconio. Se le realiza una tomografía
donde se evidencia aire en el estómago y una malformación del desarrollo esofágico. Con respecto a esta malformación
lo más probable es que se pueda tratar de una atresia esofágica:
q) proximal con fístula traqueoesofágica distal
r) distal con fístula traqueoesofágica proximal
s) proximal y distal
t) sin fístula

397. Lactante de 6 meses de edad que es traído a consulta por presentar vómitos no biliosos a repetición y retraso en
el crecimiento. En la radiografía de abdomen simple se observa nivel hidroaéreo en estómago y en primera porción de
duodeno (doble burbuja). ¿Cuál de las siguientes alternativas puede explicar la condición del lactante?
u) Páncreas anular
v) Atresia duodenal en la tercera porción
w) Atresia yeyunal
x) Hipertrofia del píloro

398. En ausencia o deficiencia de la secreción de la hormona motilina, se producirá:


a)sobrecrecimiento bacteriano
b) diarrea
c) aumento del vaciamiento gástrico
d) hipertrofia del píloro
399. La estimulación parasimpática aumenta la motilidad intestinal, mientras que la estimulación simpática la
disminuye. ¿Sobre cuál de las siguientes alternativas el sistema nervioso autónomo actúa para el control de la
motilidad intestinal?
y) Potencial de membrana en el plexo mientérico (de Auerbach)
z) Frecuencia de ondas lentas
aa) Secreción de secretina
bb) Nivel de IP3 en el plexo submucoso (de Meissner)

400.En un recién nacido con protrusión de contenidos abdominales y cubiertas por amnios o peritoneo, es cierto que:
a)Se presenta por un defecto en el cierre de la pared
b) Se acompaña de otras malformaciones congénitas
c) Se debe al no retorno de la hernia fisiológica
d) Se produce a través del ombligo

401. Lactante de 20 días con estreñimiento, distención abdominal progresiva, acompañada ocasionalmente de vómitos
biliosos. Como antecedente, el meconio lo eliminó por primera vez a las 72 horas de nacido. Su mamá menciona que
ayuda a la evacuación con ayuda de un termómetro rectal. Se sospecha de megacolon agangliónico (Enfermedad de
Hirschsprung). ¿Cuál de las siguientes alternativas explica el caso?
cc) Se presenta contracciones tónicas en la región ano rectal
dd) Se presenta dilatación de tracto gastrointestinal afectado
ee) Las células ganglionares sólo han migrado al ano recto
ff) La zona que más se afecta es inervada por fibras del nervio esplácnico menor

402. ¿Cuál de los siguientes reflejos disminuye el tránsito gastrointestinal?


a)Doloroso
b) Gastrocólico
c) De defecación
d) Colicoileal

403. Al ingerir una sustancia ácida como el vino (pH 3), se estimula la motilidad gástrica por acción de la hormona:
a)motilina
b) secretina
c) colecistoquinina (CCK)
d) bombesina

404.Paciente de 24 años acude a consulta externa por presentar una fístula oronasal (comunicación entre la cavidad oral
y la cavidad nasal). Está fístula está asociada al antecedente de haber sido operada de paladar hendido a los dos años de
edad, durante una campaña gratuita extranjera de corrección de paladar fisurado. ¿Cuál de las arterias palatinas podría
haberse lesionado durante esa cirugía?
gg) Mayor
hh) Menor
ii) Ascendente
jj) Rama palatina de la faríngea ascendente

405.Paciente de sexo masculino de 52 años con úlcera péptica gástrica de 14 años de evolución, con cuadro de
hemorragia digestiva alta hace 4 meses, sin cicatrización de la úlcera. Entre las opciones quirúrgicas se considera
realizarle un vaguectomía troncal (sección del nervio vago) a nivel del hiato esofágico. ¿Cuál de las siguientes
complicaciones podría esperarse producto de la pérdida de inervación parasimpática?
kk) Menor inervación del colon ascendente
ll) Se perderá el reflejo de defecación
mm) Se perderá el reflejo de micción
nn) Impotencia sexual

406.Paciente de 23 años con bulimia es traída a la emergencia deshidratada, semiconsciente y con alcalosis
metabólica. Los vómitos autoinfligidos por esta paciente se producen por estimulación de receptores en la base de la
lengua que mandan información directamente al:
oo) núcleo del tracto solitario
pp) centro del vómito en el tallo encefálico
qq) zona quimiorreceptora gatillo
rr) cerebelo

407.Niño de 3 años es traído a emergencia por madre quien manifiesta que hace 10 horas deglutió una pila pequeña
de reloj de bordes romos. El niño está asintomático. Usted la tranquiliza diciéndole es un cuerpo extraño tan pequeño
de seguro que va a seguir el tránsito intestinal como lo haría un bolo alimenticio, y que lo más probable es que en ese
momento se encuentre en:
ss) colon
tt) estómago
uu) yeyuno
vv) recto

408. Los movimientos en masa son un tipo de movimiento muy importante, una de las consecuencias de estos
movimientos es:
ww)la distensión rectal
xx) el peristaltismo del intestino delgado
yy) la retropulsión gástrica
zz) la contracción del esfínter anal interno

409.Durante la deglución, al momento que el bolo alimenticio pasa por el esfínter esofágico superior, se espera que la
presión intraesofágica:
aaa)disminuya en el cardias
bbb) disminuya en el tercio medio del esófago
ccc) aumente en la porción distal al bolo
ddd) aumente en el tercio medio del esófago

410.Paciente con enfermedad de Chagas que presenta disfagia a sólidos. ¿Cuál de las siguientes puede ser la causa de
esta complicación?
eee)Disminución de células ganglionares en el esfínter esofágico inferior
fff) Aumento en la liberación de óxido nítrico en el esfínter esofágico inferior
ggg)Disminución de las neuronas que liberan péptido intestinal vasoactivo
hhh) Aumento de la actividad de la motilina en el esófago distal

QUIZIZZ
1. Los vasos mesentéricos superiores se hallan a nivel de:
a) Cuello del páncreas

2. El nivel en el que se encuentra el píloro y el páncreas se puede determinar usando el


A) plano transpilórico

3. El dolor de estómago asociado a gastritis se suele ubicar en


a) epigastrio

4. La colecistoquinina inhibe el
a) vaciamiento gástrico

5. Paciente con disfasia ( dificultad para pasar alimentos) con to y disminución de peso. Con antecedente de
tabaquismo pesado. La sospecha es que tenga una disminución del diámetro esófago a nivel de la
a) tercera estrechez

6. La digestión de las proteínas se inicia en:


a) estómago

7. El estómago recibe información simpática proveniente del:


a) ganglio celíaco

8. El reflujo gastroesofagico tiene múltiples etiologías, una de ellas tiene que ver con alteración a nivel de:
a) primera estrechez
b) segunda estrechez
c) tercera estrechez
d) cuarta estrechez

9. La fístula retroperitoneal es causada por una falla en el desarrollo de:


a) tabique urorrectal

10. El nervio vago inerva el:


a) músculo estriado del esófago

11. El divertículo de Meckel es un rezago de:


a) conducto vitelino

12. La presencia de orina que sale por el ombligo de un recién nacido casa vez que llora, es posible que se deba a
un defecto en el desarrollo del
a) seno urogenital

13. La fístula retroperitoneal es causada por una falla en el desarrollo de


a) tabique urorrectal

14. Enfermedad asociada con un error en el desarrollo de las células de Cajal:


a) enfermedad de Hirschsprung

15. Aproximadamente en la semana 6 del desarrollo embrionario, el intestino medio gira 90 herniandose a nivel
del
a) cordón umbilical

16. El conducto biliar deriva del


a) endodermo

17. Paciente mujer con 54 años con nauseas y vómitos y abdomen agudo quirúrgico, se ingresa a sala de
operaciones donde se encuentra vólvulo de ciego, esto se debe a
a) Falta de fusión del mesenterio

18. Paciente con cirrosis hepática con hipertensión portal, en el que es posible encontrar que los vasos
umbilicales están permeables dentro de
a) ligamento redondo

19. Es normal encontrar glándulas submucosas en


a) esófago medio
b) esófago proximal
c) esófago distal
d) estómago

20. Cual de los siguientes órganos son intraperitoneales


a) estómago, vesícula biliar, íleon, hígado
CASO 1

Estudiante de medicina de la UPC de 21 años sufre de gastritis aguda ocasionada por comer en lugares
poco higiénicos. Suele consumir caramelos ( chupar ) mientras está en clase hasta la tarde. Toma gaseosas
regularmente (carbohidratos 46%, sodio 53%). También toma regular cantidad de leche (grasa 35%, lactosa
35%, proteínas 30%), pues le calma un poco el dolor el ardor que siente por la gastritis. Incluso, cuando
puede, se toma dos vasos de agua fría para calmar las molestias. Ha decidido ir al médico para tratarse pues
ya no soporta el dolor, el cual está seguro que los síntomas se deben a una elevada producción de ácido
clorhídrico en el estómago, y por ello le ha recetado Ranitidina (antihistamínico), con lo que siente mejoría.

● Para reducir la secreción de HCl en esta paciente se podría usar sustancias similares a:

-​ ​Péptido insulinotrópico dependiente de la glucosa (GIP)

● Si se usara atropina en esta paciente, se esperaría que disminuya la liberación de:


- Enzimas pancreáticas

● El consumir caramelos eleva los niveles en sangre de una hormona cuya función es la
estimulación de las células:
- ​ ​Beta del páncreas

● En este paciente con gastritis aguda debida a una alta producción de ácido clorhídrico, si se le
hiciera un examen de sangre, se encontraría elevados los niveles de:
- Colecistoquinina

● El consumo de una pequeña cantidad de gaseosa aumentará directamente la concentración


sérica de cuál de las siguientes hormonas:

- Péptido 1 similar al glucagón (GLP-1)

● El consumo rápido de 500 mL de gaseosa aumentará directamente la concentración sérica de


cuál de las siguientes hormonas:

- Gastrina

● Estimulan la secreción ácida gástrica

- Proteínas

● Con respecto a las ondas lentas, marque la afirmación correcta:}

- Son contracciones rítmicas espontáneas

● El uso de Ranitidina bloquea el receptor H2 de la histamina en las células parietales. La


histamina llega a estas células por:

- ​Difusión

● El consumir caramelos indirectamente activa la vía:

- POMC/CART
● ¿Cuál de los siguientes péptidos inhibe el vaciamiento gástrico?

- Colecistoquinina

● Para poder morder una manzana, es necesario usar el siguiente músculo:

- Milohiodeo

● El crecimiento de un adenocarcinoma de cuello de páncreas puede comprometer la pared


gástrica por continuidad. ¿Qué parte del estómago estaría comprometido con mayor
probabilidad?

- Pared posterior del antro

● El nacimiento de la arteria mesentérica superior se puede encontrar en cuál de los cuadrantes


abdominales:

- Epigastrio

● En cuanto a la colecistoquinina, marque la respuesta correcta:

- Potencia la acción de buffer con bicarbonato

● El aumento en la actividad motora de la pared gástrica genera un aumento en los


niveles locales de qué sustancia en la microvasculatura:

- Adenosina

CASO 2

Niño de sexo masculino de 2 años de edad, sufre de estreñimiento desde el nacimiento (1 deposición
cada 3-4 días). Madre menciona que le estimula la defecación con un termómetro rectal, y continuo uso
de enemas y laxantes. Desde hace 6 meses comienza con vómitos postprandiales. Los síntomas
aumentan en frecuencia y magnitud y están en relación con los episodios de estreñimiento. No refiere
fiebre, tos, diarrea ni lesiones cutáneas. Al examen físico presenta regular estado general, luce
deshidratado. Abdomen distendido, blando, depresible e indoloro. No se palpan masas abdominales.
Se permeabiliza el canal anal con termómetro rectal, encontrando cierta resistencia. Salida de material
fecal mal oliente en regular cantidad. Exámenes de laboratorio: hemograma normal. Signos
inflamatorios de fase aguda negativos. Alcalosis metabólica leve en sangre venosa. Radiografía con
enema baritado muestra recto y colon sigmoides dilatados (megacolon). Biopsia profunda: ausencia de
células ganglionares en la muestra enviada. Se realiza cirugía correctiva.

● La percepción de la pirosis (sensación de dolor o quemazón en el esófago) asociado al reflujo


gastroesofágico, puede aparecer o exacerbarse debido a:

- Ejercicio
● Considerando que este paciente está sometido a estrés por el agravamiento de su enfermedad,
es posible afirmar que sus ondas lentas están:

- Hiperpolarizadas

● En cuanto a los reflejos gastrocólico y gastroduodenal en este paciente, indique lo correcto:

- Se pueden considerar reflejos vago-vagales

● diferencia de las arcadas, los vómitos presentan apertura de:


- Esfínter esofágico superior

● En la Enfermedad de Hirschprung, el patólogo debe declarar un área como sana o libre de


enfermedad si es que encuentra:
- Células intersticiales en el plexo mioentérico
● Durante la fase faríngea de la deglución es espera que ocurra el siguiente fenómeno:
- El paladar blando cierra la entrada a la nasofaringe
● En este paciente se considera que está abolido el reflejo:
- Rectoesfinteriano
● Debido al acúmulo de material fecal en todo el marco colónico, y a la irritación química
asociada, el peristaltismo del íleon distal se debe encontrar:
- Inhibido

--------------------------------------------------------------------------------------------------------------------------------------------------
● Con respecto a la defecación señale el enunciado correcto:

- Es estimulado por un llenado de la cuarta parte del volumen rectal

● El contenido fecal se detiene en la zona inmediatamente proximal a la zona donde hay una
menor presencia de:

- Péptido intestinal vasoactivo

● La presencia de atresias y estenosis duodenales se deben básicamente a una:

- Falta de recanalización

● El ligamento de Treitz característicamente:

- Suspende el ángulo de Treitz

● Al deglutir un bolo alimenticio, es lógico suponer que al pasar por el esófago haya un mayor
consumo de oxígeno en la pared del tercio:

- Proxima​l

● Estudiante de medicina de 20 años, se ha amanecido estudiando para su examen de Sistema


Digestivo. No ha probado alimento desde la cena, por lo que se puede afirmar que la motilidad
de esta persona está siendo regulada por:

- Motilina

● La hernia fisiológica se produce dentro de:

- Cordón umbilical

● El crecimiento de un adenocarcinoma de páncreas compromete la pared gástrica por


contigüidad. ¿Qué parte del estómago se esperaría esté comprometido?

- Pared posterior del antro

● Al comer unas papitas fritas con mayonesa, el vaciamiento gástrico disminuye por efecto
directo de la hormona:

- colecistoquinina (CCK)

● Paciente que come entera una pizza familiar de chorizo y queso. Es posible esperar que debido
a la cantidad de alimento ingerida, las ondas lentas hayan:

- Sufrido ninguna alteración en su frecuencia

● En este caso se puede afirmar con seguridad que se presenta:

- contracciones tónicas en la región ano rectal

● La forma más común de atresia esofágica contiene:


- Estenosis proximal del esófago más fístula traqueoesofágica distal

● Con respecto a la saliva, marque la respuesta correcta:

- el sistema simpático estimula su secreción

● Respecto a las enfermedades del esófago, marque lo correcto:

- el diagnóstico diferencial de la acalasia es la enfermedad de Chagas esofágica

● En relación a la fisilogía gástrica, marque lo correcto:

- la cimetidina actúa en la región basolateral de la célula parietal

● La célula mucosa del cuello gástrico produce:

- Moco

● La saliva puede tener una variedad de electrolitos en su composición. Entre ellos el cloro,
respecto al cual se puede afirmar:

- Su concentración no llega a ser tan alta como en el plasma

● Los músculos de la masticación que producen la retropulsión de la mandíbula son:

- temporales

● Respecto a las glándulas salivales, marque lo incorrecto:

- la glándula sublingual tiene forma de garfio

● Respecto a la anatomía del estómago, marque lo correcto:

- la arteria gástrica derecha nace de la arteria hepática común

● En cuanto a la saliva, marque lo correcto:

- La amilasa cumple función digestiva

● Durante el ataque con gas sarín (bloqueador de la acetilcolinesterasa) en el metro de Tokio, en


1995, el personal de salud notó que los pacientes afectados presentaban:

- Hipersalivación

● Para estimular la motilidad intestinal se podría usar un análogo de: ​CCK

● No se espera que sea causa del vómito: ​AYUNO PROLONGADO

● En la Enfermedad de Hirschprung, el patólogo debe declarar un área como sana o libre de enfermedad
si es que encuentra: ​CÉLULAS INTERSTICIALES EN EL PLEXO MIOENTÉRICO.

● En la zona afectada por esta enfermedad, se espera que las ondas lentas estén: ​ABOLIDAS

● Si este paciente tuviera arcadas, entonces se debe considerar que durante la ocurrencia de dichas
arcadas, debemos encontrar contenido gástrico en: ​ESÓFAGO​.

● Cuando este paciente ingiera sus alimentos, se espera que al momento de pasar el bolo alimenticio por
el esfínter esofágico superior, la presión intraesofágica disminuya en: ​EL CARDIAS.
CASO 3

Paciente de 54 años con antecedentes de alcoholismo, gastritis crónica, tabaquismo pesado,


obesidad, cálculos biliares y cirrosis, es llevado a la emergencia por dolor abdominal en
epigastrio irradiado a la espalda y trastorno del sensorio.
Al examen físico: presión arterial 85/50 mmHg, frecuencia cardíaca 100 latidos/min, frecuencia
respiratoria 18 x minuto, temperatura axilar 36°C.
Conjuntivas pálidas, escleras ictéricas nevus arácnidos en tronco, distensión abdominal
marcada, cabeza de medusa, matidez desplazable en ambos flancos e hipogastrio, dolor a la
palpación de abdomen.
Tiempo de protrombina: 24 seg (testigo: 13 seg); TPT: 38 seg, glicemia: 165 mg/dL, uremia: 20
mg/dL, ASAT: 76 UI/L, ALAT: 22 UI/L, albumina: 2,5 g/dL, bilirrubina total: 2,6 mg/dL,
bilirrubina directa: 1,4 mg/dL, amilasa sérica 4000 U/L.

● Un efecto secundario en el estómago por la acción de la secretina es:

- Menor actividad de la pepsina

● Considerando que el paciente sufre de gastritis, se puede decir que la secreción de ácido por la
mucosa gástrica:

- Involucra transporte activo de Hidrogeniones

● En cuanto a la gastritis de este paciente, se encontró que era producida por la bacteria
Helicobacter pylori. Esta bacteria sobrevive en el medio ácido del estómago gracias a:

- Ureasa

● La bilirrubina directa aumentada en cirrosis hepática se excreta en la orina debido a:

- Ser hidrosoluble

● Paciente de 42 años con adenocarcinoma ductular. La TC ha demostrado claramente que el


tumor está en el cuello del páncreas y que hay un gran vaso ocluido. ¿Cuál de los siguientes
vasos estaría más probablemente obstruido?

- Vena porta.

● En relación a la histología hepática, marque lo correcto:


- La zona 3 se encuentra más cerca a la vena central lobulillar
CASO 4

Mujer de 83 años acude a emergencia por dolor abdominal desde hace 4 días,
localizado en epigastrio, irradiado a ambos hipocondrios, nauseas, vómitos y
distensión abdominal; tiene antecedente de cardiopatía hipertensiva, diabetes
mellitus tipo II y fibrilación auricular. Refiere deposiciones diarreicas muco
sanguinolentas hace 1 día. Los exámenes iniciales muestran PA: 110/60 mmHg, FC:
110/mn, leucocitos: 17800, neutrófilos de 93%. TAC abdómino pélvica se observa
oclusión completa de arteria mesentérica superior por trombo asociado a placa
ateromatosa.
● En esta paciente, ¿cuál de las siguientes sustancias no tendrá una considerable
disminución en su absorción? (marque la mejor respuesta):

- Calcio

● El mecanismo de la diarrea muco sanguinolenta que presenta la paciente, puede mejor definido
como de tipo (marque la mejor respuesta):}

- Exudativa

● Considerando que se ha comprometido el íleon distal, entre otras áreas, la atrofia o


descamación del epitelio de superficie explicaría cuál de los signos o síntomas de la paciente
(marque la mejor respuesta):

- Diarreas mucosanguinolentas

● Producto de esta isquemia, la expresión de cuál de las siguientes enzimas se vería


notablemente disminuida. Marque la mejor respuesta:

- Enteroquinasa

● Durante la cirugía, el cirujano observó que además la paciente tenía divertículos en el sigma. Se
sabe que estos divertículos:

- Se pueden asociar a estreñimiento crónico

● Después de remover quirúrgicamente el territorio gastrointestinal afectado, estaremos seguros


que no quedará en la paciente la siguiente estructura:
- Placas de peyer
● Durante la cirugía, se observa que el colon transverso no está muy afectado por la isquemia,
esto se debe a la anastomosis de las arterias:
- Cólica media e izquierda
● Durante la cirugía, el cirujano observó que además la paciente tenía divertículos en el sigma. Se
sabe que estos divertículos:
- Se pueden asociar a estreñimiento crónico
● En la paciente, el flujo sanguíneo de cuál de las arterias pancreáticas estará afectada:
- Pancreaticoduodenal posteroinferior

● Una mujer de 43 años dolor en hipocondrio derecho e ictericia. En la ecografía se evidencia cálculos
biliares. Estos cálculos lo más probable es que se encuentren localizados en: ​CONDUCTO
COLÉDOCO.

● Si existe un fármaco de acción colerética, se asume que se refiere a que potencia o estimula la:
RECIRCULACIÓN DE SALES BILIARES.

● Paciente con tumor neutro endocrino productor de secretina, debido o cual se puede esperar que su
secreción pancreática, comparada con la de una persona sana en estado de bajo fluido, tenga una
concentración de: ​BICARBONATO AUMENTADA
● Por cuál de las siguientes células es secretada principalmente la pro enzima procarboxipeptidasa:
ACINARES DEL PÁNCREAS.

● En relación a la secreción biliar y su composición, marque lo correcto: ​LAS SALES BILIARES


CORRESPONDEN AL 50% DE SU COMPOSICIÓN .

● El triángulo hepatocístico es importante de conocer en una colecistectomía, debido que tiene:


ARTERIA CÍSTICA.

● Durante una endoscopia, se ingresa en la papila mayor en la segunda porción del duodeno, y dentro de
la vía biliar extrahepatica, se toma una muestra de secreción biliar. Dicha muestra debe contener ácido:
QUENODESOXICÓLICO​.

● La internalización de proteínas o fragmentos de proteínas hacia el intracelular, es característico de :


CÉLULAS M
● Las ramas más pequeñas del árbol biliar son: ​CANALÍCULOS BILIARES.
● Con respecto a las lesiones y enfermedades de la boca, marque lo correcto: ​LA ERITROPLASIA DEBE
SER BIOPSIADA.
● Respecto a la motilidad del colon, marque lo correcto: ​Con la distención del estómago, suelen aparecer
movimientos en masa
● En un paciente hipertenso con tratamiento a base de beta bloqueadores, la producción de saliva se
espera que: ​DISMINUYA
● La saliva puede tener una variedad de electrolitos en su composición. Entre ellos el cloro, respecto al
cual se puede afirmar: ​SU CONCENTRACIÓN NO LLEGA A SER TAN ALTA O EN EL PLASMA.
● ESTIMULA LA PRODUCCIÓN DE SALIVA: ​VASODILATACIÓN PERIGLANDULAR.
● DURANTE EL SUEÑO LA COCENTRACIÓN DE BICARBONATO EN LA SALIVA: ​DISMINUYE​ .
● Un paciente hipertenso está tomando un medicamento bloqueador de receptores alfa-1 adrenérgicos
(prazosina), y como efecto secundario se queja de: ​HIPOSALIVACIÓN​.
● La atropina es el tratamiento para la intoxicación por órganos fosforados. Se administra hasta alcanzar
niveles terapéuticos. Un buen indicador que la atropina ya está en niveles terapéuticos es: ​SEQUEDAD
DE LA MUCOSA ORAL.
● En cuanto a la saliva, marque lo correcto: ​LA AMILASA CUMPLE FUNCIÓN DIGESTIVA.
● Con respecto a la saliva: ​EL SISTEMA SIMPÁTICO ESTIMULA SU SECRECIÓN.
● La secreción de agua y bicarbonato por el páncreas exocrino se da básicamente en la fase:
- Intestinal
● En relación a la secreción biliar y su composición, marque lo correcto:
- Las sales biliares corresponden al 50% de su composición
● Si existe un fármaco con acción colerética, se asume que se refiere a que potencia o estimula la:
- Recirculación de sales biliares
● Hombre de 65 años con isquemia intestinal leve por oclusión aterosclerótica de la arteria mesentérica
superior, pero la irrigación sanguínea colateral ha retrasado el inicio de la necrosis. ¿Qué vasos
ofrecen colaterales entre el tronco celíaco y la arteria mesentérica superior?
- Pancreaticoduodenal superior e inferior.
● Una mujer de 43 años dolor en hipocondrio derecho e ictericia. En la ecografía se evidencia cálculos
biliares. Estos cálculos lo más probable es que se encuentren localizados en:
- Conducto colédoco
● Cuando el alimento se encuentra en el estómago, se produce la liberación de enzimas pancreáticas
básicamente debido a la acción de:
- Vago
● Paciente con tumor neuroendocrino productor de secretina, debido a lo cual se puede esperar que su
secreción pancreática, comparada con la de una persona sana en estado de bajo flujo, tenga una
concentración de:
- bicarbonato aumentada
● ¿Por cuál de las siguientes células es secretada principalmente la pro enzima procarboxipeptidasa?
- Acinares del páncreas
● Dentro de las funciones de las células de Ito, marque lo incorrecto:
- Pueden fagocitar patógenos y actúan como presentadoras de antígeno
● Paciente de 42 años con adenocarcinoma ductular. La TC ha demostrado claramente que el tumor está
en el cuello del páncreas y que hay un gran vaso ocluido. ¿Cuál de los siguientes vasos estaría más
probablemente obstruido?
- Vena porta.
● Respecto a la anatomía del estómago, marque lo correcto:
- la arteria gástrica derecha nace de la arteria hepática común
● Con respecto a las lesiones y enfermedades de la boca, marque lo correcto:
- a eritroplasia debe ser biopsiada
● La secreción de saliva es importante en la fisiología digestiva. Su concentración de potasio llega a ser
menor que la del plasma cuando su secreción tiene un flujo:
- Nunca
● Para protegerse del entorno ácido, el Helicobacter pylori se autogenera un entorno de pH menos ácido
alrededor s​uyo, gracias a una enzima que alcaliniza su entorno local mediante la conversión de:
- Urea en NH3
● Durante el sueño, la concentración de bicarbonato en la saliva:
- Disminuye
● Respecto a la motilidad del colon, marque lo correcto:
- Con la distensión del estómago, suelen aparecer movimientos en masa
● Respecto a las enfermedades del esófago, marque lo correcto:
- el diagnóstico diferencial de la acalasia es la enfermedad de Chagas esofágica
● El omeprazol actúa sobre la membrana _____________ de la célula ____________
- apical / parietal
● Durante el ataque con gas sarín (bloqueador de la acetilcolinesterasa) en el metro de Tokio, en 1995, el
personal de salud notó que los pacientes afectados presentaban:
- Hipersalivación
● Un paciente hipertenso está tomando un medicamento bloqueador de receptores alfa-1 adrenérgicos
(prazosina), y como efecto secundario se queja de:
- Hiposalivación
● La arteria palatina menor irriga un área de la boca cuyo epitelio es:
- Escamoso no queratinizado
● Producto de la alimentación, se producen diversas sustancias peptídicas, cininas y bradicininas, las
cuales permiten que:
- El flujo sanguíneo intestinal aumente hasta 8 veces
● En relación a los principios de motilidad, marque lo incorrecto:
- Las dos terceras partes proximales del esófago están conformadas por músculo
esquelético
● El crecimiento de un adenocarcinoma de cuello de páncreas puede comprometer la pared gástrica por
continuidad. ¿Qué parte del estómago estaría comprometido con mayor probabilidad
- Pared posterior del antro
En un paciente con apendicitis aguda, la sensación de dolor producido por esta inflamación es llevada
por el nervio:
- Esplácnico menor
Para poder morder una manzana, es necesario usar el siguiente músculo:
- Milohiodeo
El dolor periumbilical o epigástrico en el inicio de una apendicitis aguda se debe a:
- Estímulo del sistema simpático.
El nacimiento de la arteria mesentérica superior se puede encontrar en cuál de los cuadrantes
abdominales:
- Epigastrio
El uso de Ranitidina bloquea el receptor H2 de la histamina en las células parietales. La
histamina llega a estas células por:
- Difusión
Marque la respuesta correcta en relación a la gastrina:
- Las células G son las productoras y se encuentran principalmente en el antro gástrico
HOLA PUTAS YA SABEN CONTROL F
Sistema
Digestivo
1. Al examinar a un paciente, usted encuentra dolor localizado en fosa iliaca derecha y diagnostica apendicitis.
En este paciente, usted puede inferir:
El peritoneo parietal regional está afectado
2. La motilidad intestinal es estimulada principalmente por el:
Plexo de Auerbach
3. Durante el vómito, ¿el contenido gástrico tiene que pasar necesariamente por cuál estructura para llegar al
esófago? Marque la mejor respuesta:
Cardias
4. Respecto a la anatomía del estómago, marque lo correcto:
El fondo gástrico forma la curvatura mayor
5. Marque la respuesta incorrecta:
En todo el tubo digestivo, se observa dos capas de muscular propia: circular interna y longitudinal externa
6. Paciente se queja de dolor en hipocondrio derecho, pero superficialmente. El dermatoma relacionado es

A
(marque la mejor respuesta):
T9
7. Dentro de las funciones del abdomen, se encuentra la defecación y micción, en las cuales la presión intra

EK
abdominal debe:
Aumentar
8. Un alumno de medicina decide hacerse un piercing en el ombligo. Al realizarle el procedimiento, sangra
ligeramente. Esta sangre proviene de la arteria (marque la mejor respuesta)
Epigástrica inferior
OT
9. Señale la respuesta correcta:
El apéndice cecal sólo tiene serosa
10. Paciente mujer es traída a emergencia por sufrir una herida contuso penetrante por cuchillo realizada por
su esposo en un ataque de celos. Se observa herida en flanco izquierdo. Esta solución de continuidad ha
comprometido varios músculos de la pared abdominal, excepto:
NC

Recto abdominal
11. Paciente con herida por proyectil por arma de fuego, con herida de ingreso en región paraumbilical. Entre
las estructuras que usted está seguro que debe haberse lesionado es:
Omento mayor
12. En la evaluación de una tomografía abdominal, el interno observa un aneurisma en una arteria que se dirige
BA

al riñón derecho. Con seguridad se puede afirmar que está a nivel de la vértebra:
L1
13. Paciente joven es traído a emergencia con abdomen agudo quirúrgico debido a herida contuso penetrante
por verduguillo (alambre grueso con punta aguzada) recibida en una pelea después de un partido de fútbol.
Se observa herida en Hipocondrio Izquierdo. El órgano que debe estar sangrando y produciendo
hemoperitoneo es (marque la mejor respuesta):
Bazo
14. Marque el órgano que se encuentra más distal en el tubo digestivo.
Ciego
15. La peristalsis o peristaltismo hace referencia a:
Motilidad para movilizar el alimento de proximal a distal.
16. Marque la respuesta incorrecta:
En todo el tubo digestivo, se observa dos capas de muscular propia: circular interna y longitudinal externa
17. Paciente con vólvulo del colon sigmoides. La necrosis de ese segmento del colon se produce por una
alteración en la irrigación de la arteria: Mesentérica inferior
18. Marque el órgano que se considera retroperitoneal: Parte de la vía biliar
19. Un alumno de medicina decide hacerse un piercing en el ombligo. Al realizarle el procedimiento, sangra
ligeramente. Esta sangre proviene de la arteria (marque la mejor respuesta): Epigástrica inferior
20. Paciente tiene una úlcera sangrante en el segundo tercio del Yeyuno. La arteria de la cual proviene la sangre
arterial para dicha zona es la arteria: Mesentérica superior
21. Es inervado por aferentes somáticas: Peritoneo parietal
22. Paciente de 24 años con dolor abdominal tipo cólico intenso en mesogastrio. Según sus conocimientos de
macroestructura, el origen del dolor puede ser ….: Íleon
23. Paciente con hipoglucemia secundaria a un insulinoma (tumor neuroendócrino productor de insulina). El
órgano donde mayor probabilidad ha crecido este tumor es: retroperitoneal
24. Al evaluar una tomografía abdominal, el médico asistente le pide al interno de la UPC que encuentre la
imagen con el corte a nivel de L1. El interno sabiamente busca el …….. para ubicar la vértebra L1: Cuello del
páncreas
25. En la inspiración, la pared abdominal debe …. para ….. : relajarse disminuir presión intra torácica
26. Ligamento hepatogástrico une el ….. con el …… y forma la entrada al …… : Hígado Estómago Orificio omental
27. Al retirar completamente el mesenterio de un órgano, el mismo se vería afectado principalmente en su:
Irrigación
28. La estructura que fija órganos principalmente a la pared posterior abdominal se denomina: Mesenterio
29. Cuál de las siguientes estructuras no tiene vasos sanguíneos: Epitelio intestinal
30. Al iniciar la digestión, aumenta el consumo de oxígeno por la mucosa. Esto conlleva a una hipoxia local, lo
cual hace que se libere …., el cual produce vasodilatación: adenosina
31. Sustancia que inhibe la secreción y la motilidad del estómago prolongando el tiempo de digestión: péptido
insulinotrópico dependiente de la glucosa (GIP)
32. Marque lo correcto: La hernia fisiológica se produce en la sexta semana y es la salida temporal de asas
intestinales a través del colon umbilical
33. Marque la respuesta correcta en relación a la gastrina: Las células G son las productoras y se encuentran

A
principalmente en el antro gástrico.
34. El consumir caramelos indirectamente activa la vía: POMC/CART
35. ¿En qué capa se encuentra la alteración principal en el Hirschsprung o megacolon agangliónico?: Muscular

EK
propia
36. Con respecto a las ondas lentas, marque la afirmación correcta: Son contracciones rítmicas espontáneas
37. El uso de Ranitidina bloquea el receptor H2 de la histamina en las células parietales. La histamina llega a
estas células por: Difusión
OT
38. La triada sintomática: vómitos explosivos post-prandiales, movimientos peristálticos epigástricos visibles
de izquierda a derecha y nódulos palpable epigástrico subcostal derecho, pertenecen a: Estenosis congénita
hipertrófica del píloro
39. Durante una cirugía oncológica, ¿la extirpación de cuál de los siguientes órganos se vería comprometida
por la presencia de adventicia?: Recto
NC

40. En cuanto a los reflejos gastrointestinales, un reflejo que estimula el tránsito intestinal es el reflejo:
Gastrocólico
41. El ligamento falciforme divide al hígado en dos lóbulos derecho e izquierdo. Embriológicamente deriva del:
Mesenterio ventral
42. La presencia de atresias y estenosis duodenales se deben básicamente a una: Falta de recanalización
BA

43. Estudiante de medicina de 20 años, se ha amanecido estudiando para su examen de Sistema Digestivo. No
ha probado alimento desde la cena, por lo que se puede afirmar que la motilidad de esta persona está
siendo regulada por: Motilina
44. Paciente con disminución del apetito marcada asociada a cáncer terminal, para promover la ingesta de
alimentos se podría usar análogos de: Endorfinas
45. Las ondas lentas se producen por la apertura cíclica de canales de: Calcio
46. La forma más común de atresia esofágica contiene: Estenosis proximal del esófago más fístula
traqueoesofágica distal
47. Al deglutir un bolo alimenticio, es lógico suponer que al pasar por el esofago haya un mayor consumo de
oxigeno en la pared del tercio: Proximal
48. Paciente que come entera una pizza familiar de chorizo y queso. Es posible esperar que debido a la cantidad
de alimento ingerida, las ondas lentas hayan: sufrido ninguna alteración en su frecuencia.
49. La hernia fisiológica se produce dentro de: cordón umbilical
50. El crecimiento de un adenocarcinoma de páncreas compromete la pared gástrica por contigüidad ¿Que
parte del estómago se esperaría esté comprometido?: Pared posterior del antro
51. Estimula la producción de saliva: Vasodilatación periglandular
52. Durante la secreción de saliva, es de esperarse que las concentraciones de ….. y …… disminuyan al disminuir
el flujo: Sodio Bicarbonato
53. Con respecto a la secreción gástrica de HCI: a mayor secreción de HCI en el lumen gástrico, mayor pH en la
sangre venosa gástrica.
54. Respecto a las enfermedades del esófago, marque lo correcto: el diagnóstico diferencial de la acalasia es la
enfermedad de Chagas esofágica
55. Con respecto a las lesiones y enfermedades de la boca, marque lo correcto: la eritroplasia debe ser
biopsiada
56. Respecto a las glándulas salivales, marque lo incorrecto:
a. la glándula sublingual tiene forma de garfio
b. la glándula sublingual drena a través del conducto de wharton
c. la glándula parótida produce secreción serosa
d. la glandula parotida drena a través del conducto de Stenon
57. El omeprazol actúa sobre la membrana ….. de la célula ….. : apical/parietal
58. Durante el sueño, la concentración de bicarbonato en la saliva: Disminuye
59. Durante el ataque con gas sarín (bloqueador de la acetilcolinesterasa) en el metro de Tokio, en 1995, el
personal de salud noto que los pacientes afectados presentaban: Hipersalivación
60. La célula mucosa del cuello gástrico produce: Moco

Parcial 2020-01 (1)

1. Al evaluar la orofaringe de un paciente, el médico le solicita que abra la boca, saque la lengua y diga a. Al
hacer esta maniobra, nota que el paladar se desvía hacia la derecha, lo cual le hace sospechar que el
paciente sufre de una lesión del nervio craneal: X contralateral
2. Un bolo alimenticio grande y poco masticado se atasca en el esofago, esto ocasiona una sensación de

A
dolor que es transmitida por los nervios: esplácnicos
3. Para realizar el movimiento mecánico de abrir la boca, primero se necesita: fijar el hueso hioides

EK
4. ¿Cual de las siguientes alternativas se define como la protrusión directa del contenido abdominal a la
cavidad amniótica por un defecto de la pared corporal?: Gastrosquisis
5. Un paciente requiere que se le coloque una sonda de alimentación directamente al estómago
(gastrostomía), el cirujano deberá hacer una incisión en la piel del abdomen ¿cuál de las siguientes raíces
nerviosas debe ser anestesiada para este procedimiento? T8
OT
6. En un paciente de 43 años con tumor carcinoide de páncreas productor de gastrina (Síndrome de
Zollinger-Ellison) se puede esperar encontrar una potenciación del reflejo: gastrocólico
7. El mecanismo de la defecación incluye la participación de diversas estructuras ¿cuál de las siguientes
alternativas es correcta?: Puede ser mediado por un reflejo intrínseco
8. Cuando el contenido del estómago ingresa al duodeno, uno de los reflejos que inhiben el vaciamiento
NC

gástrico es a través del: Sistema nervioso mientérico


9. Durante la masticación, gran parte del proceso masticatorio se debe a: El reflejo masticatorio
10. Las glándulas salivales tienen conductos para la excreción de la saliva, las glándulas …… drenan en las
carúnculas sublinguales. RPTA: Sublinguales
11. Los diferentes segmentos del tubo digestivo son susceptibles de reflejos y movimientos según su
BA

contenido. Si colocamos mediante una sonda un bolo alimenticio directamente en el tercio medio del
esofago: se producira ondas secundarias
12. En una persona si enfermedad se espera que el tránsito intestinal se vea disminuido cuando se presenta el
reflejo: Doloroso
13. El divertículo de Meckel es una anomalía congénita que ocurre por la persistencia del conducto vitelino y
da origen a una estructura sacular, el cual se encuentra en el: borde antimesentérico
14. Si al intubar a un paciente, por error se ingresa el tubo endotraqueal en el esofago y se insufla el
manguito endotraqueal (globo TET), la dilatación de este manguito generará: múltiples ondas secundarias
15. El orificio omental, o hiato de Winslow, se encuentra limitado por el ligamento: hepatoduodenal
16. Paciente de 24 años acude a consulta externa por presentar una fístula oronasal (comunicación entre la
cavidad oral y la cavidad nasal). Esta fístula es una consecuencia tardía de la lesión de un vaso sanguíneo
por el antecedente de haber sido operado de paladar hendido en los primeros años de vida,
aparentemente en una campaña gratuita de corrección de paladar fisurado. ¿Cual de las arterias palatinas
podría haberse lesionado durante esa cirugía?: Mayor
17. Dentro de las anomalías congénitas se puede presentar un tejido pancreático accesorio ¿cuál es la
ubicación más común de este tejido?: Estómago
18. Una recién nacida es evaluada por el neonatólogo y evidencia que el canal anal está completamente
cerrado. Este problema se debe probablemente a una anomalía en el desarrollo de: la membrana cloacal
19. En la digestión de los alimentos, la hormona __________ se libera frente a la presencia de péptidos y
monoglicéridos, y tiene un efecto marcado en la disminución del vaciamiento gástrico →
colecistoquinina
20. Dentro de las anomalías congénitas se puede presentar un tejido pancreático accesorio ¿Cuál es la
ubicación más común de este tejido? → Estómago
21. Los catadores de vino tienen una habilidad increíble al momento de separar los sabores. Este aumento de
la sensibilidad gustativa debido a una mayor cantidad de papilas linguales y de corpúsculos gustativos se
conoce como: hipergeusia
22. En muchos países se usa el suplemento de flúor en el agua potable o los dentífricos, con el fin de hacer el
esmalte más resistente a la desmineralización inducida directamente por: el ácido
23. Durante el desarrollo de la región cloacal, una cuña de mesodermo ubicado entre el alantoides y el
intestino posterior vendrá a formar el: tabique urorrectal
24. En una persona sana, el momento adecuado para encontrar los mayores niveles de grelina en sangre
sería: antes de comer
25. El duodeno está constituido por el segmento terminal del intestino anterior y el segmento proximal del
intestino medio ¿Cuál de las siguientes alternativas describe mejor este lugar de unión entre los dos
intestinos?
Distal al origen de la yema hepática

A
26. En una persona sana, el uso de atropina producirá a nivel del estómago: Aumentará el pH del estómago
27. En una persona sana, el consumo de leche produce indirectamente → Inhibición del vaciamiento

EK
gástrico
28. La motilidad del colon es importante y lenta comparada con la del intestino delgado. Los movimientos en
masa ocasionan la: distensión rectal
29. Con respecto a la motilidad gástrica, los potenciales de acción disminuyen en frecuencia por efecto de:
el péptido insulinotrópico dependiente de glucosa
30. La sensación del gusto depende de la presencia de papilas gustativas en la lengua, de las cuales, algunas
OT
de ellas tienen un surco terminal por donde drenan unas glándulas salivales linguales (llamadas glándulas
de von Ebner). Esta descripción se refiere a las papilas: circunvaladas
31. Dentro de la estructura de los dientes, la parte del diente cubierta por esmalte y que se puede ver
mediante la inspección visual de la boca se denomina → corona clínica
NC

32. En el conducto anal se encuentra la unión entre las regiones del endodermo y el ectodermo, esta unión se
evidencia al observar: la línea pectínea
33. La sensación del gusto depende de la presencia de papilas gustativas en la lengua, las cuales tienen
corpúsculos gustativos conteniendo células neuroepiteliales sensoriales. Estas células neuroepiteliales
pueden ser dañadas fácilmente, por suerte, su tiempo de recambio es de alrededor de: 10 días
BA

34. En un varón de 47 años con sección medular a nivel de T6 debido a un accidente automovilístico, sus
terapeutas han desarrollado un mecanismo para distender el recto e iniciar el reflejo rectoesfinteriano, lo
cual producirá la contracción de: la pared del recto
35. Durante un experimento, se insufla rápidamente dos litros de agua en un globo colocado dentro
del estómago de un voluntario. ¿cuál de las siguientes situaciones del músculo liso será
consecuencia directa de este cambio de volumen en el estómago? → Despolarización
36. Paciente de 56 años con accidente cerebrovascular reciente. En la resonancia se observa daño de los
núcleos laterales del hipotálamo. Por este motivo es muy probable que el paciente sufra de: inanición
37. Paciente con insuficiencia mitral moderada a severa, con aumento de volumen de la aurícula izquierda,
esta condición tendrá como consecuencia a nivel del sistema digestivo: la disfagia a sólidos
38. El inicio de la fase faríngea de la deglución se debe a estímulos sensitivos que viajan por el nervio craneal:
V
39. El mesocolon transverso se origina en: la pared posterior del abdomen
40. El esofago en su microestructura tiene basicamente adventicia, a excepción de la región distal, donde
tiene serosa, específicamente a partir del nivel de: T10
41. En un paciente con arcadas, se debe considerar que durante la ocurrencia de dichas arcadas, debemos
encontrar contenido gástrico en: Tórax
42. La reabsorción de Sodio y Cloro en las glándulas salivales se da principalmente en el: Conducto estriado
43. Para que se puedan digerir las grasas, es preferible que primero sean emulsificadas. La hormona que
estimula la liberación de las sustancias emulsificadoras es: CCK
44. Paciente de 13 días de vida con vómitos explosivos a las 2 horas después de lactar. Al examen físico se
palpa la oliva pilórica ¿cual es el nervio cuyos filetes dan inervación eferente a la estructura afectada?:
Vago
45. La localización de la vesícula biliar con respecto al lóbulo cuadrado es: Lateral
46. La triada portal está constituida por el conducto biliar y la arteria hepática y una pequeña rama de la
vena: Porta
47. Al ingerir una cantidad de glucosa por vía oral, esta es interiorizada en las células del organismo más
rápido que si esa misma cantidad hubiese sido administrada por vía endovenosa. Este fenómeno sucede
gracias a una sustancia secretada por las células: K
48. Paciente de 62 años con vólvulo de intestino delgado e isquemia intestinal. ¿Qué estructura se utiliza
como punto de referencia para determinar la posición de la unión duodenoyeyunal? : Ligamento
suspensorio del duodeno (de Treitz)
49. En un paciente con Zollinger Ellison, usted esperaría encontrar: Esteatorrea
50. La motilidad intestinal es estimulada por: Colecistoquinina y gastrina
51. La onda peristática secundaria del esofago en la deglución, es producida por: Plexo mientérico esofágico
52. Paciente con parálisis bilateral del nervio hipogloso, el unico musculo de la lengua que conservará su

A
movimiento es el: palatogloso
53. ¿Cual de las siguientes sustancias tiene mayor concentración de la saliva comparado con su concentración
plasmática? Potasio

EK
54. ¿Cual de las siguientes estimula la secreción enzimática exocrina del páncreas?: Colecistoquinina
55. Al comer un pollo a la brasa, con papas fritas y ensalada, la sustancia que estimula la liberación de HCI en
el estómago es: Bombesina
56. Marque lo correcto con respecto a Esófago de Barret: Se relaciona con reflujo gastroesofágico
OT
ECU 1
Estudiante de medicina de la UPC de 21 años sufre de gastritis aguda ocasionada por comer en lugares poco
higiénicos. Suele consumir caramelos ( chupar ) mientras está en clase hasta la tarde. Toma gaseosas regularmente
(carbohidratos 46%, sodio 53%). También toma regular cantidad de leche (grasa 35%, lactosa 35%, proteínas 30%),
NC

pues le calma un poco el dolor el ardor que siente por la gastritis. Incluso, cuando puede, se toma dos vasos de
agua fría para calmar las molestias. Ha decidido ir al médico para tratarse pues ya no soporta el dolor, el cual está
seguro que los síntomas se deben a una elevada producción de ácido clorhídrico en el estómago, y por ello le ha
recetado Ranitidina (antihistamínico), con lo que siente mejoría.

El uso de atropina en este paciente:


BA

- Aumentará el pH del estómago

Entre las sustancias cerebrales que producen ansiedad está la serotonina, la cual también tiene acción:
- Anorexigénica

El consumo de dos vasos de agua seguidos agua generará indirectamente un aumento en la liberación de:
- Ácido clorhídrico

En este paciente con gastritis aguda debida a una alta producción de ácido clorhídrico, sería lógico esperar que el
píloro tenga un tono muscular:
- Aumentado

El consumo de leche produce directamente un aumento de los niveles séricos de la hormona:


- Colecistoquinina (CCK)

El consumo de leche produce directamente un aumento de los niveles séricos de la hormona:


- Gastrina

ECU 2
Niño de sexo masculino de 2 años de edad, sufre de estreñimiento desde el nacimiento (1 deposición cada 3-4
días). Madre menciona que le estimula la defecación con un termómetro rectal, y continuo uso de enemas y
laxantes. Desde hace 6 meses comienza con vómitos postprandiales. Los síntomas aumentan en frecuencia y
magnitud y están en relación con los episodios de estreñimiento. No refiere fiebre, tos, diarrea ni lesiones
cutáneas. Al examen físico presenta regular estado general, luce deshidratado. Abdomen distendido, blando,
depresible e indoloro. No se palpan masas abdominales. Se permeabiliza el canal anal con termómetro rectal,
encontrando cierta resistencia. Salida de material fecal mal oliente en regular cantidad. Exámenes de laboratorio:
hemograma normal. Signos inflamatorios de fase aguda negativos. Alcalosis metabólica leve en sangre venosa.
Radiografía con enema baritado muestra recto y colon sigmoides dilatados (megacolon). Biopsia profunda:
ausencia de células ganglionares en la muestra enviada. Se realiza cirugía correctiva.

El contenido fecal se detiene en la zona inmediatamente proximal a la zona donde hay una menor presencia de:
- Péptido intestinal vasoactivo

En cuanto a los reflejos gastrocólico y gastroduodenal en este paciente, indique lo correcto:


- Se pueden considerar reflejos vago-vagales

En este paciente se considera que está abolido el reflejo:


- Rectoesfinteriano

Debido al acúmulo de material fecal en todo el marco colónico, y a la irritación química asociada, el peristaltismo
del íleon distal se debe encontrar:
- Inhibido

Es un reflejo propio de la pared intestinal:


- Peristaltismo

A
A diferencia de las arcadas, los vómitos presentan apertura de:
- Esfínter esofágico superior

EK
1. Un niño de 2 años es llevado a la consulta por diarrea persistente y edema de las
OT
extremidades, además falta de crecimiento y desarrollo en relación a su edad. Los
análisis de sangre revelan que tiene concentración plasmática baja de proteínas
(hipoproteinemia). Durante la endoscopía duodenal, se coloca colecistocinina (CCK)
endovenosa y se recoge muestras del líquido duodenal; el resultado del líquido
NC

confirma incapacidad para hidrolizar proteínas a un pH neutro, esta situación mejora


al añadir una pequeña cantidad de tripsina. El paciente probablemente esté sufriendo
la falta congénita de
-Enterocinasa
2. Experimentalmente se incrementa la velocidad de la secreción salival con una
BA

sustancia, el análisis de la composición de esta saliva obtenida se espera


encontrar…………..
-Disminución de concentración de potasio
3. Paciente varón de 46 años soltero, consulta por odinofagia y bajo de peso, tiene
antecedente de tuberculosis desde hace 3 meses y es fumador crónico (10 cigarrillos
por día); al evaluar la cavidad oral se identifica lesión blanquecina en el dorso de la
lengua y paladar blando, las lesiones se desprenden con el baja lengua dejando una
base eritematosa. Esta lesión corresponde probablemente a
……………………….…..
-Candidiasis oral
4. Paciente mujer de 35 años acude a consulta por sensación de sequedad y lesiones
en cavidad oral. Al examen se observa atrofia de la mucosa, fisuras y úlceras; nota
además sequedad e irritación de la córnea y aumento del tamaño de las glándulas
parotídeas. Su diagnóstico más probable es artritis reumatoide; el hallazgo más
probable en una biopsia de glándula parótida es……..….
-Gran infiltración de linfocitos y células plasmáticas
5. Un paciente con anemia acude con su médico quejándose de episodios frecuentes
de gastroenteritis. Un análisis de sangre revela anticuerpos circulantes dirigidos
contra células parietales gástricas. Su anemia es atribuible a la hiposecreción de
-Factor intrínseco
6. Dos estudiantes deciden tomar un receso para comer una hamburguesa a la hora
del almuerzo. Antes de llegar a la cafetería, impulsos nerviosos provenientes del
complejo vagal dorsal iniciarán la secreción de ácido gástrico por la liberación de
…………………….. desde el sistema nervioso entérico.
-GRP
7. Un niño de cuatro años de edad es llevado a la consulta por cuadros diarreicos
frecuentes caracterizados por heces pálidas, voluminosas y fétidas, presenta bajo
peso y talla. Se mide la concentración de cloruro en el sudor y se encuentra que sus
valores son muy elevados. La alteración más importante a nivel de células ductales
del páncreas tiene relación directa con la conductancia de…………
-Cloro
8. Una mujer de 50 años de edad que sufrió durante varios años resequedad de los
ojos debida a producción inadecuada de lágrimas es enviada con un
gastroenterólogo para evaluación de pirosis crónica. El examen endoscópico
revela erosiones y tejido cicatrizal en la parte distal del esófago justo por arriba del
esfínter esofágico inferior. Las lesiones pueden atribuirse a la disminución de uno
de los siguientes componentes salivales:

A
-Bicarbonato

EK
9. Se evalúa los valores séricos de las siguientes sustancias a un paciente con
enfermedad hepática terminal; en este paciente se espera encontrar la combinación
con la letra …………
-disminuida, aumentada, disminuida
OT
10. Una mujer de 35 años de edad HIV positiva, se presenta al médico con dolor
abdominal en cuadrante superior derecho e ictericia. La paciente refiere haber tenido
múltiples episodios de ictericia durante los últimos 10 años. Los exámenes para
determinar hepatitis viral, dieron positivos para Hepatitis B, siendo catalogado el caso
como hepatitis crónica con alteración funcional. En un examen de sangre ¿cuál de los
NC

siguientes parámetros está disminuido?


-Albúmina
11. En el reflejo peristáltico del intestino delgado, uno de los siguientes eventos sucede
en la porción oral del bolo alimenticio…………...
BA

-Acción de acetilcolina en el músculo circular


12. Experimentalmente se coloca una dosis alta de secretina en la luz intestinal
duodenal; como consecuencia de esto, en el jugo pancreático de la misma luz
intestinal se observa la disminución de la concentración de …..………..
-Cl
13. Un varón de 58 años de edad con enfermedad de Crohn severo fue sometido a una
resección ileal. Después de la cirugía este paciente padecerá de esteatorrea, esto se
explica porque …..………..
- La micelas no pueden formarse
14. En un experimento se inserta un balón en el estómago de un voluntario, se infla poco a
poco mientras que se vigilan las presiones intraluminales. Aunque el volumen del
balón aumenta considerablemente, las presiones permanecen constantes. Esta
relación volumen-presión se explica por la liberación local de …………..
-Óxido nítrico y péptido inhibidor vasoactivo
15. La toxina de Vibrio cholerae causa diarrea debido a…….
-El Incremento de la secreción de cloro por las células de la cripta intestinal
16. ¿Cuál de las siguientes alternativas es una característica de la secreción exocrina
del páncreas?
-Tiene una baja concentración de Cl- respecto al plasma
17. Una madre lleva a su hijo de dos años de edad a la sala de urgencias, estresada
porque el niño deglutió una moneda de 10 céntimos mientras la familia cenaba en un
restaurante. El médico observa mediante fluoroscopía que la moneda se halla en el
estómago y asegura a la madre que la moneda se eliminará con las heces. El médico
recomienda utilizar la respuesta fisiológica que permitirá la evacuación de la moneda
del estómago al intestino ………….…..
-Son los movimientos de mezcla y trituración
-. Es provocada por el ayuno
18. Las estructuras en el hígado que permite que los productos metabólicos unidos a
proteínas tengan acceso a las membranas basolaterales de los hepatocitos, son….. -
Las fenestras sinusoidales
19. La composición de la bilis es modificada conforme fluye por los conductillos biliares.
Durante este tránsito se espera que aumente la concentración de…….
-Monómeros de ácido biliar
-Ig A
20. Se mide experimentalmente el contenido gástrico de dos personas. La persona “A”
tiene alto contenido de grasa y la persona “B” tiene un contenido hipertónico ¿Cuál de
las siguientes es correcto respecto al vaciamiento gástrico?
- Hay ralentización del vaciado gástrico en ambos casos

A
21. El examen endoscópico de un paciente con hipertensión portal grave revela venas
tortuosas que sobresalen hacia la luz del esófago. El paciente recibe tratamiento

EK
quirúrgico mediante la colocación de una derivación que conecta la vena porta a la
vena cava. Después de la operación el riesgo de encefalopatía y el
riesgo de sangrado de várices ……………..
-Aumentará/disminuirá
22. Un paciente varón de 18 años de edad acude al médico para sus exámenes de
OT
rutina. Sus resultados de laboratorio muestran un valor de bilirrubina sérica de 4
mg/dl y una bilirrubina directa de 0,3 mg/dl. Las pruebas de función hepática son
normales. La alteración que explica mejor este caso es por la deficiencia de
………………..
NC

-Glucuronil transferasa
23. Un hombre de 57 años de edad es llevado a urgencias con hematemesis masiva
rojo brillante, a su llegada se halla inconciente con PA: 80/40 mm Hg y FC: 124
lat/min. Luce ictérico con presencia de “arañas vasculares en el tórax anterior y
extremidades”, abdomen distendido con signo de oleada positiva. Se encuentra
BA

esplenomegalia y pérdida de la masa muscular en extremidades. La anastomosis


vascular responsable del sangrado en este paciente es
-Vena gástrica izquierda y vena ácigos
24. Un estudiante de medicina está comiendo un plato de comida a base de
champiñones, espárrago y salsa de soya. El sabor umami contenido en todos estos
alimentos actúa a nivel de los botones gustativos estimulando ………………..
-Un receptor acoplado a proteína G
25. Un hombre de 22 años de edad se presenta al médico con una historia de 1 año de
evolución caracterizado por dolor recurrente en fosa iliaca derecha y diarrea.
Manifiesta además pérdida de peso de 8 kg durante este periodo. La colonoscopía
revela múltiples lesiones en el ileon terminal y colon. La biopsia de estas lesiones
revela engrosamiento, inflamación y ulceración de la mucosa. El diagnóstico más
probable en este caso es…….
-Enfermedad de Crohn
26. Varón de 61 años que consulta por dolor retro esternal intenso desde hace 6 horas y
después de vómitos intensos y repetidos; al examen se observa disnea, cianosis,
hipotensión y signos clínicos de shock. La radiografía simple de tórax muestra
neumomediastino. El líquido en el espacio pleural aspirado tiene alta concentración de
amilasa. ¿Cuál de las siguientes alternativas puede explicar este cuadro clínico? -
Rotura espontánea de esófago
27. La secreción del ácido en la célula parietal gástrica se lleva a cabo por una ATPasa
especifica que intercambia hidrogeniones (H+) del citosol por…..
-K +
28. En condiciones normales el ingreso de 600 ml de líquido es el estómago provoca un
aumento de presión intragástrica de unos 12 cm de H2O. Después de una vagotomía
(corte del nervio vago) es de esperar que el ingreso del mismo volumen de líquido
provoque lo siguiente: …………………………………
-Un aumento mayor de la presión
29. Una paciente de 30 años de edad es sometida a una cirugía en oído medio derecho
por un problema de otoesclerosis. Luego de la cirugía refiere alteración en la
percepción de sabores. Al evaluar el caso usted esperaría encontrar……….
-Alteración en la sensación del gusto en los dos tercios anteriores de la
lengua
-Sensación del dolor, tacto y temperatura conservada en toda la lengua

30. ¿Cuál de las siguientes alternativas es correcta?


-Las sales biliares desconjugadas son absorbidas preferentemente en el colon

A
31. En un paciente de 45 años de edad con colestasis biliar, se encuentra una elevación
de los niveles sanguíneos de fosfatasa alcalina hasta 3 veces la cifra normal. ¿Cuál de

EK
las siguientes alternativas estará también elevada como evidencia del daño de la vía
biliar?
-Gamma glutamil transpeptidasa
32. Revisando la angiografía de un hombre de 70 años en estudio por aneurisma de aorta
abdominal el radiólogo informa de la presencia de una oclusión completa de la arteria
OT
mesentérica inferior. El paciente se encuentra completamente asintomático.
¿Cuál de las siguientes arterias se anastomosa a la sistema arterial de la
mesentérica inferior?
-Cólica media
33. Lactante de 3 meses de vida es atendido por presentar diarrea, se administra una
NC

solución de glucosa y electrólitos por vía oral. La proteína de membrana apical que
explica la capacidad de esta solución para proporcionar aporte de glucosa e
hidratación es ………..
-SGLT-1
BA

34. Paciente ha sufrido herida de bala en el abdomen, se le ha tenido que extirpar el


segmento medio y distal del ileon. En este caso la síntesis hepática de sales biliares
estará …..…..
-Incrementada por estímulo de la enzima colesterol 7 alfa hidroxilasa
35. Un varón de 75 años ingresa al consultorio por presentar ictericia marcada de piel y
las escleras. El estudio del paciente mostró que presentaba un tumor que obstruía la
totalidad del conducto hepático común. ¿Cuál de las siguientes estructuras se
encontrará dilatada en este paciente?
-Conductos de Hering
36. En un paciente con insuficiencia renal crónica, el déficit en la absorción de calcio a
nivel del enterocito se debe a lo siguiente:
-No se convierte la 25 hidroxicolecalciferol a 1,25 dihidroxicolecalciferol
37. Varón de 30 años es traído a emergencia por agresión abdominal con arma de fuego
(pistola) y es sometido a laparotomía exploratoria, observándose isquemia del colon
ascendente y parte del colon trasverso ¿la lesión de cuál de las siguientes arterias
explicaría esta isquemia?
d. Mesentérica superior
38. Respecto a las sustancias gastrointestinales que regulan la secreción pancreática;
marque la afirmación correcta:
b. La acetilcolina es capaz de estimular la secreción enzimática y de bicarbonato del
páncreas
39. Ante una lesión del X par craneal, ¿cuál de los siguientes músculos mantiene
conservada su función?:
b. Tensor del velo del paladar
40. Experimentalmente se utiliza atropina (anticolinérgico) para inhibir la secreción de
gastrina, sin embargo la secreción de esta hormona se sigue dando ante estímulos
vagales. Esta situación se explica porque la atropina:
d. No bloquea la acción del péptido GRP

41. Un varón de 50 años es sometido a extirpación del duodeno y parte proximal del
yeyuno. Esta situación ocasionaría la pérdida de las células ……….. , productoras de
………………… que estimula la secreción de bicarbonato por el páncreas.

A
“S” / secretina
42. Se evalúa la expresión de la proteína Agrp en una persona con alteración del apetito; lo

EK
correcto respecto a esta proteína es…..
La mutación del gen que la codifica produce adelgazamiento
43. Juana cae de la bicicleta y se fractura la región anterior del hueso maxilar superior con
compromiso de la fosa incisiva. Al examen físico de la región esperaría encontrar
alteración en la sensibilidad de la encía …………………
OT
palatina anterior
44. Recién nacido es atendido por el neonatólogo y luego entregado a su madre para dar
de lactar; la madre al dar de lactar observa coloración azulada de labios, acompañado
de tos persistente, dificultad respiratoria y distención abdominal. Se le intenta colocar
NC

una sonda nasogástrica pero esta retorna a la cavidad oral en todos los intentos. ¿Cuál
de las siguientes anomalías del desarrollo es el más probable en este caso?
b. Atresia esofágica proximal con fístula traqueoesofágica distal l
45. ¿Cuál de los siguientes mecanismos ocurre durante la defecación?
BA

En la posición de “cuclillas” el músculo puborrectal se halla relajado


46. Un paciente luego de un accidente sufre lesión del piso de la boca, se constata daño
del nervio “cuerda del tímpano”, en este caso se esperaría en
47. contrar disminución de la………………………….… de la lengua
Sensación del gusto en los dos tercios anteriores
48. ¿Cuál de las siguientes afirmaciones es la correcta sobre la gastrina?
Actúa en la célula diana mediante su receptor CCk tipo B
49. Al recibir un paciente con signos de hipovolemia y antecedente de trauma en
abdomen por accidente de tránsito, usted identifica radiológicamente: lesión de
primera vértebra lumbar y signos de lesión en páncreas; durante la cirugía se observó
pobre irrigación de asas intestinales. El vaso afectado es la arteria ……..
c. mesentérica inferior
50. Un paciente sufre de daño a nivel del cuello con lesión muscular en la región de la
faringe. En el examen físico se determina dificultad para la elevación de la faringe y para
el cierre del itsmo de las fauces. En este caso, probablemente esté afectado el músculo:
c. palatofaringeo
51. Varón de 50 años a quien le realizan la curación de la segunda molar de la arcada
superior derecha. En un momento determinado, el paciente acusa de intenso dolor
de la pieza dentaria en tratamiento. La vía aferente del dolor viaja a través del
nervio …………
a. trigémino V2
52. La distención gástrica por los alimentos produce incremento de secreción de HCl
mediante la producción de ………….. que estimula a las célulasvía proteína
………..
a. gastrina / parietal / Gq

53. Un niño de tres años llega a emergencia con disfagia (dificultad para tragar), dolor
retro esternal, salivación y llanto. Se sospecha de ingesta de cuerpo extraño (moneda)
en el esófago; al ser evaluado se constata en una radiografía presencia de cuerpo
extraño a nivel de C6 (6° vértebra cervical). El cuerpo extraño estará suspendido a

A
nivel del estrechamiento producido por………..
c. el músculo cricofaríngeo

EK
54. La triada portal (arteria hepática, vena portal y conducto biliar común) está contenida
en el ligamento …….……… y derivan embriológicamente del ……
a. hepato duodenal / mesenterio ventral
55. Un paciente refiere no percibir algunos sabores, al examen físico se constata
OT
alteración en la percepción de sabores y del dolor en el tercio posterior de la lengua
¿Cuál de los siguientes nervios estará alterada en su función?
c. Glosofaríngeo (IX par)
56. En el caso de un paciente con gastrinoma (tumor productor de gastrina), la presencia
NC

de úlceras duodenales y erosión de la mucosa gástrica, se debe principalmente a…….


c. el exceso de HCl por estímulo de receptores CCK-B en la célula parietal
56. El reflejo entero gástrico se caracterizan por:
d. originarse debido a la distensión duodenal y presencia del quimo ácido
BA

57. Mauricio tiene dificultad para deprimir el paladar y elevar la parte posterior de la lengua.
En este caso estará afectado un músculo, específicamente el músculo
…………….
b. extrínseco – palatogloso
58. En condiciones normales, el ingreso de 600 ml de líquido es el estómago provoca un
aumento de presión intragástrica de unos 12 cm de H2O. Después de una vagotomía (corte
del nervio vago) es de esperar que el ingreso del mismo volumen de líquido ocasione
………………………………… de la presión
intragástrica.
c. un aumento mayor
59. La explicación fisiológica de presentar somnolencia de 30 minutos a 1 hora después
de ingerir alimentos, se explica por: a. Aumento del cloro intraluminal
e. Aumento de la alcalinidad sanguínea
60.Se presenta un paciente, el cual presenta un antecedente de tuberculosis intestinal, por
lo cual, se le resecó 80 cm de íleon distal. Desde el punto de vista fisiológico, el paciente
puede presentar una de las siguientes alteraciones: a. Disminución de la secreción de
Vitamina B12

e. Disminución de la absorción de ácido glicocólico


61. Un paciente es sometido experimentalmente a un fármaco que modifica el flujo salival,
obteniéndose un volumen de saliva de 288 ml en 6 horas. En este caso las concentraciones
de electrolitos y bicarbonato en la saliva obtenida varían de la siguiente manera: a. ↑ Na+,
↓ K+, ↑ Cl-, ↑ HCO3-

1. b. ↓ Na+, ↓ Cl-, ↑ K+, ↓ HCO3-

62. Uno de los siguientes elementos debería hallarse con más probabilidad en el esófago de
un paciente que sufre de reflujo gastro esofágico…
a. Pepsina

63. Un paciente de 40 años cursa con anemia de 8g/dl, aqueja además de astenia y sensación
de hormigueo bilateral en los miembros inferiores, al examen se halla alteración de la
sensibilidad a la vibración y camina con ampliación de la base de sustentación. Uno de los
siguientes procedimientos sería de ayuda para el diagnóstico de este paciente:
a. Tomografía cerebral

A
b. Biopsia de la mucosa gástrica

EK
64.Paciente de 60 años ingresa por caída hace 1 hora y pequeño hematoma en cuero
cabelludo, al examen físico ampliado se observa ictericia de piel y mucosas generalizada,
abdomen blando, se palpa estructura quística no dolorosa en hipocondrio derecho que
corresponde a vesícula biliar (signo de Courvoisier), en los exámenes de laboratorio se halla
OT
niveles bajos en la formación de estercobilinógeno y urobilinógeno en heces, incremento de la
bilirrubina conjugada en la orina, elevación de fosfatasa alcalina y gamma glutamil
transpeptidasa séricas. El presente cuadro puede ser explicado por: a. Reabsorción de
hematoma
NC

c.Carcinoma de la cabeza de páncreas


65. Un recién nacido presenta vómitos biliosos poco tiempo después de cada alimento. Al
preguntar a la madre sobre antecedentes, ella recuerda que tuvo polihidramnios durante la
gestación, pero un análisis de cariotipo fue normal. Una de las siguientes es la causa más
BA

probable de estos hallazgos en el recién nacido: a. Enfermedad de Hirschprung


e. Malrotación de la yema pancreática ventral
66.En un estudio de la secreción de hormonas gastrointestinales, sus concentraciones en la
vena porta se midieron durante perfusión luminal del intestino delgado con soluciones de
diversas magnitudes de pH. ¿Qué hormona aumentará en el plasma de la vena porta durante
perfusión a través del intestino con una solución de pH 3?
a. CCK
e. secretina
67.Paciente de 30 años que ingresa a causa de un traumatismo abdominal cerrado. En la
exploración se aprecia discreta palidez de piel y mucosas, auscultación pulmonar normal,
taquicardia de 120 /min. Discreta distensión abdominal y matidez en flancos; el hematocrito,
que era prácticamente normal al ingreso, disminuye a 30% a las tres horas. En la Rx de tórax
se objetiva fractura de las costillas 10-11 izquierdas. La causa más probable de la anemización
en este paciente es: a. traumatismo renal con hemorragia retroperitoneal.
c. rotura de bazo con hemoperitoneo.
68. Revisando la angiotomografía de un hombre de 70 años en estudio por aneurisma de
aorta abdominal, el radiólogo le informa de la presencia de una oclusión completa de la
arteria mesentérica inferior. El paciente se encuentra completamente asintomático. La
oclusión de la arteria mesentérica inferior cursa de manera asintomática en muchas
ocasiones ya que el territorio que irriga puede recibir flujo proveniente de la arteria:
a. cólica derecha
e. cólica media

69. En las patologías de esófago es importante conocer bien la anatomía esofágica. ¿Cuál de
las siguientes afirmaciones es correcta? a. El esófago tiene capa mucosa, muscular y serosa

c. El esófago torácico pasa por detrás del cayado aórtico

70. A pesar de que pueda haber variaciones anatómicas, lo habitual es que el ciego sea
irrigado por una rama arterial que proviene de unas de las siguientes arterias: a. Iliaca
derecha

d. Mesentérica superior

71. Ante un paciente con una cirugía abdominal urgente, el informe operatorio señala que se
ha realizado una resección de todo el duodeno y del tercio proximal del yeyuno manteniendo

A
íntegros el estómago y todo el íleon, así como los dos tercios distales del yeyuno. En el
seguimiento nutricional del paciente ¿Qué vitamina o mineral presentará con menor

EK
probabilidad una disminución de su absorción?
a. Cianocobalamina

72. ¿Cuál de las siguientes sustancias forma parte de la secreción biliar? a. Tripsina
OT
Lecitina

73.¿De qué musculo forma parte el ligamento inguinal?


-Oblicuo externo del abdomen
74.¿Cuál de las siguientes enzimas está localizada en el borde en cepillo y juega un rol en la
NC

digestión de proteínas?
e. Carboxipeptidasa A.
75. Una de los siguientes sustancias, NO sirve como un buen agente emulsificante:
a. Colesterol
76. La sustancia que estimula el crecimiento de la mucosa gástrica es:
BA

a. Secretina

d. Gastrina

77.¿Cuál de las siguientes alternativas es una función de la colecistokinina?

a. Relajación de la vesícula para la salida de bilis

d. Secreción de enzimas pancreáticas

78.Con respecto a la anatomía del tronco celiaco, señale lo correcto a. El tronco celiaco se
origina de la cara posterior de la aorta abdominal
d. La hepática común que es una de sus ramas, participa en la irrigación del
estómago.
79. Con respecto a la anatomía del duodeno, marque la respuesta correcta: a. Tiene una
distribución en forma de “C”, que rodea la cola del páncreas
b. La 3ra porción duodenal está contenida en la pinza vascular aortomesentérica
80. En el íleon se absorbe aproximadamente el 95% dea través de la circulación
enterohepática.
a. agua
c. sales biliares
81. La estimula el mecanismo paracrino de la secreción de ácido clorhídrico.
a. histamina
82.En la digestión de proteinas,es el principal estímulo para convertir el
pepsinógeno en pepsina. a. la gastrina
b. el pH ácido
83. Con respecto a la somatostatina, marque lo correcto:
a. Es secretada por las células S del intestino
Interviene en la fase intestinal de la secreción gástrica
84. En pecten anal, es una estructura comprendida entre: a. la línea pectínea y los senos
anales
d. la línea anocutánea y la línea pectínea

85.¿Cuál de las siguientes alternativas es una proenzima pancreática? a. Tripsina

A
1. b. Elastasa
2. c. Quimotripsinógeno

EK
3. d. Amilasa
4. e. Procarboxipeptidasa C
86. En este paciente, el bloqueo farmacológico de los receptores H2 en la mucosa gástrica:
a) No tiene efecto sobre la secreción de ácido inducida por el vago
OT
b) Evita la activación de adenilciclasa por gastrina
c) Inhibe la secreción de ácido inducida por gastrina y mediada por el vago
d) Causa un aumento en el transporte de potasio por las células parietales gástricas
Se validó la A :)
87. Si se considerara una gastrectomía total para curar la gastritis del paciente, cuál de las
NC

siguientes sustancias ya no se produciría:


a) Gastrina
b) Quimiotripsina
c) Amilasa
BA

d) Pepsinógeno

88. Un paciente hipertenso está tomando un medicamento bloqueador de receptores alfa 1


adrenérgicos (prazosina) y como efecto secundario se queja de:
d) Lo escaso que es el medicamento
e) No tiene problemas en la salivación
c) Hiposalivación
d) Hipersalivación

89. Con respecto a las lesiones y enfermedades de la boca, marque lo correcto:


f) La leucoplasia se desprende al roce
b) la eritroplasia puede degenerar en adenocarcinoma
c) El muguet oral es una enfermedad bacteriana en inmunodeprimidos
d) la eritroplasia debe ser biopsiada
NOTA: fue validada la opción B ya que no es motivo del curso que sepamos el puto cáncer.

90. En este paciente, se puede asumir que la pancreatitis ha sido ocasionada por una
disminución en el efecto de:
g) Amilasa
h) Lipasa
c) Inhibidor de la tripsina
d) Entercinasa
91.Un efecto secundario en el estómago por la acción de la secretina es:
i) Disminución en la liberación de pepsinógeno
b) Menor actividad de la pepsina
c) Mayor paso de bicarbonato a sangre periférica
d) Aumento en la producción de factor intrínseco
92. Dentro de los factores protectores de la mucosa gástrica se pueden mencionar múltiples
protagonistas. Uno de ellos es:
j) CCK
k) Gastrina
c) Receptor muscarínico
d) Pepsina
93. La saliva puede tener una variedad de electrolitos en su composición. Entre ellos el cloro,
respecto al cual se puede afirmar:
l) Su mayor concentración se consigue con el flujo bajo
b) Su concentración no llega a ser tan alta como en el plasma
c) Con flujo alto su concentración es mayor que la del plasma

A
d) Su menor concentración se alcanza con flujo alto
94. En el síndrome de boca seca o síndrome de Sjogren, una de las complicaciones

EK
asociadas es:
a) caries
b) Disminución de la acidez gástrica
c) Aumento en de la producción de saliva
d) Aumento del pH bucal
OT
95. Estimula la producción de saliva:
a) Vasodilatación periglandular
b) Atropina
c) Fatiga o cansancio
d) Expresión de miedo
NC

96. El omeprazol actúa sobre la membrana de la célula


m) Basolateral/principal
n) apical/principal
o) Basolateral/parietal
d) apical/parietal
BA

97. Para protegerse del entorno ácido, el Helicobacter pylori se autogenera un entorno de
pH menos ácido alrededor suyo, gracias a una enzima que alcaliniza su entorno local
mediante la conversión de:
a) urea en NH3
b) H2O y CO2 en ácido carbónico
c) NH3 en urea
d) H2CO3 en bicarbonato
98. La anemia perniciosa destruye las células:
p) mucosas del cuello
b) oxínticas
c) principales
d) mucosas superficiales
99. La célula mucosa del cuello gástrico produce:
a) Moco
b) ácido clorhídrico
c) pepsinógeno
d) Factor intrínseco
100.El aumento en la acidez del estómago producido principalmente por la infección de
Helicobacter pylori se debe a la disminución de:
a) Somatostatina
b) Bicarbonato por las glándulas de Brunner
c) Secretina
d) Colecistoquinina
101. De las siguientes sustancias secretadas por los órganos de este paciente, la más
alcalina es la secreción:
a) pancreática
b) Esofágica
c) Yeyunal
d) Salival
102.En cuanto a la gastritis de este paciente, se encontró que era producida por la bacteria
Helicobacter pylori. Esta bacteria sobrevive en el medio ácido del estómago gracias a:
a) ácido clorhídrico
b) Toxina CagA
c) Ureasa

A
d) Jugo pancreático
103.La lengua está recubierta por epitelio:

EK
c) pseudoestratificado columnar no queratinizado
b) plano estratificado no queratinizado
c) pseudoestratificado columnar ciliado
d) plano estratificado queratinizado
104.El esfínter anal interno tiene musculatura …….. y tiene control ……..
OT
d) lisa / voluntario
b) lisa / involuntario
c) esquelética / simpático
d) esquelética / parasimpático
NC

19) La arteria Aorta proporciona la irrigación al tubo digestivo ¿cuál de las siguientes
arterias proporciona la irrigación al ángulo cólico derecho?
a) mesentérica superior
b) mesentérica inferior
BA

c) frénica inferior
d) tronco celiaco

20) Paciente de 26 años que le cuenta en su historia clínica que cada vez que almuerza a los
20 min tiene deseo de defecar, le comenta que su hijo de 1 mes le pasa lo mismo pero más
intenso. Esto se explica por el reflejo …….., el cual está …… en el paciente
a) colicoileal / normal
b) colicoileal / alterado
c) gastrocólico / normal
d) gastrocólico / alterado

21) La región del estómago que se comunica con el duodeno es la


a) pilórica
b) cardias
c) cuerpo
d) fórnix

22) Acude a consulta un px que fue diagnosticado de úlcera péptica 3 días antes. Luego de
múltiples pruebas diagnósticas se concluye que el paciente presenta un tumor secretor de
gastrina, ¿cual de las siguientes situaciones estará incrementada?
a) distensión gástrica
b) inhibición del vaciado gástrico
c) secrecion de acido clorhidrico
d) inhibición de la secreción de pepsinógeno

A
EK
OT
NC
BA
23) En el sistema digestivo, el control del apetito está dado por un complejo sistema de
sustancias y órganos integradores, los cuales regulan la ingesta de alimentos. La …… es
una sustancia orexígena y es sintetizada por el ……
a) leptina / estómago
b) felina / intestino
c) leptina / estómago
d) grelina / estómago

24) Con respecto a la actividad eléctrica del sistema digestivo, marque la alternativa
correcta
a) corresponden a potenciales de acción que están presentes de forma continua y le
dan capacidad de peristalsis autónoma al sistema digestivo
b) la frecuencia de las ondas lentas NO se ve influenciada por la actividad neural ni las
hormonas gastrointestinales
c) en el estómago las ondas lentas se dan en una frecuencia de 6 x min

A
d) las ondas lentas son cambios lentos y ondulantes del potencial en reposo
e) la frecuencia de las ondas lentas va de 6 a 12 ondas por minuto

EK
25) Ante una lesión del IX pc, el músculo …… se altera en su función
a) palatogloso
b) estilofaríngeo
c) palatofaríngeo
OT
d) constrictor superior

26) Un varón de 50 años es sometido a extirpación del duodeno y parte proximal del yeyuno.
La pérdida de estímulo hormonal en el páncreas para la secreción enzimática se explica
NC

por la pérdida de células


a) parietales, productoras de factor intrínseco
b) K productoras de factor intrínseco
c) M productora de CCK
d) I productora de CCK
BA

27) Marque la respuesta correcta:


A. El bronquio derecho constituye una de las estrecheces del esófago
B. Todos los órganos del sistema digestivo tienen capa serosa
C. La pared gástrica en el fondo es más delgada que en el cuerpo y antro
D. El esfínter de Oddi rodea a la papila menor duodenal

28) Marque la respuesta correcta en relación a la gastrina:


A. Al distenderse el estómago, se inhibe su producción.
B. Se estimula por la liberación de noradrenalina
C. Las células G son las productoras y se encuentran principalmente en el antro
gástrico
D. Las células G se encuentran principalmente en el fondo gástrico

29) Para poder morder una manzana, es necesario usar el siguiente músculo:
A. Milohiodeo
B. Tensor del paladar
C. Orbicular de los labios
D. Buccinador
30) Sustancia que inhibe la secreción y la motilidad del estómago prolongando el tiempo de
digestión:
A. Enteroglucagon.
B. Polipéptido pancreático
C. Péptido 1 similar al glucagón (GLP-1).
D. Péptido insulinotrópico dependiente de la glucosa (GIP).

31) El nacimiento de la arteria mesentérica superior se puede encontrar en cuál de los


cuadrantes abdominales:
A. Hipocondrio derecho
B. Hipogastrio
C. Epigastrio
D. Mesogastrio
32) Entre las múltiples causas de la Enfermedad por Reflujo Gastroesofágico, se puede
considerar también a una alteración en las del esfínter esofágico inferior:

A
A. Ondas secundarias
B. Contracciones tónicas

EK
C. Ondas lentas
D. Glándulas subesofágicas

33) Producto de la alimentación, se producen diversas sustancias peptídicas, cininas y


OT
bradicininas, las cuales permiten que:
A. Se produzca neovascularización en los territorios de las arterias abdominales
B. La acción de la lipasa pancreática se vea incrementada
C. El consumo de O2 del intestino aumente ligeramente
D. El flujo sanguíneo intestinal aumente hasta 8 veces
NC

34) El dolor periumbilical o epigástrico en el inicio de una apendicitis aguda se debe a:


A. Estímulo del nervio vago.
B. Íleo secundario.
C. Irritación del peritoneo parietal.
BA

D. Estímulo del sistema simpático.

35) El aumento en la actividad motora de la pared gástrica genera un aumento en los


niveles locales de qué sustancia en la microvasculatura:
A.Adenosina
B. Colecistoquinina CCK
C. Endotelina
D. Gastrina

36) ¿Cuál de los siguientes péptidos inhibe el vaciamiento gástrico?


A. Colecistoquinina
B. Péptido inhibidor gástrico
C. Motilina
D. Gastrina
37) Los músculos de la masticación que producen la retropulsión de la mandíbula son:
A. temporales [mas seguro]
B. maseteros
C. milohioideos
D. pterigoideos
38) En relación a la fisiología gástrica, marque lo correcto:
A. la cimetidina actúa en la región basolateral de la célula parietal
B. la marea alcalina se debe al paso de bicarbonato través de la membrana apical de la
célula principal
C. el cloro difunde hacia el exterior por la la región basolateral de la célula parietal
D. la salida de hidrogeniones a la luz es por difusión facilitada

39) Durante el sueño, la concentración de bicarbonato en la saliva:


A. Se eleva a niveles mayores que los del plasma
B. Aumenta
C. No tiene efecto
D. Disminuye

40) La secreción de saliva es importante en la fisiología digestiva. Su concentración de


potasio llega a ser menor que la del plasma cuando su secreción tiene un flujo:

A
A. Intermedio
B. Nunca

EK
C. Bajo
D. Alto

42) Respecto a las glándulas salivales, marque lo incorrecto:


A. la glándula parótida produce secreción serosa
OT
B. la glándula sublingual drena a través de conducto de Wharton
C. La glándula parótida drena a través del conducto de Stenon
D. la glándula sublingual tiene forma de garfio
NC

43) Con respecto a la saliva, marque la respuesta correcta:


A. será hipertónica cuando el flujo es bajo
B. a mayor flujo, menor concentración de Na
C. a mayor flujo, menor concentración de cloro
D. el sistema simpático estimula su secreción
BA

CI 3
44) En relación a la circulación hepática, marque lo correcto:
a) Los sinusoides hepáticos transportan sangre mixta
b) La vena porta proporciona el 50% de sangre al hígado
c) La vena porta se forma a partir de la vena esplénica y la mesentérica inferior
d) La arteria hepática deriva de la mesentérica superior

45) Dentro de las funciones de las células de Ito, marque lo incorrecto:


a) Sintetizan colágeno
b) Almacenan vitamina A
c) Se les llama células estrelladas
d) Pueden fagocitar patógenos y actúan como presentadoras de antígeno
46) Paciente con tumor neuroendocrino productor de secretina, debido a lo cual se puede
esperar que su secreción pancreática, comparada con la de una persona sana de bajo flujo,
tenga una concentración de:
a) Sodio aumentada
b) Igual
c) Bicarbonato aumentada
d) Potasio disminuida
47) El GALT se localiza en:
a) Lámina propia
b) submucosa
c) borde en cepillo
d) superficie de criptas de Lieberkühn

48) En relación a la histología hepática, marque lo correcto:


a) la zona 1 se afecta rápidamente en estados de hipovolemia y shock
b) La zona 1 se encuentra cercana a la vena central lobulillar
c) La zona 3 se encuentra más cerca a la vena central lobulillar
d) La zona 3 se encuentra más cerca al eje menor del acino hepático
49) El acino pancreático difere con el de las glándulas salivales en:
a) Contiene células centroacinares
b) No produce secreción serosa
c) El páncreas produce principalmente secreción mucosa

A
d) No tienen diferencias
50) Durante la digestión de las grasa, para que la lipasa actúe adecuadamente se requiere que el

EK
pH aumento en la luz intestinal, lo cual es logrado, entre otros, por la secreción de las
células:
a) Del conducto interlobulillar
b) Centroacinares
c) Acinares
OT
d) Alfa
51) La secreción de la colecistoquinina (CCK) se produce en la fase:
a) intestinal
b) En las 3 por igual
NC

c) gástrica
d) Cefálica
52) ¿Por cuál de las siguientes células es secretada principalmente la pro enzima
procarboxipeptidasa?
a) Acinares del páncreas
BA

b) Epiteliales del duodeno


c) Ductales del páncreas
d) Centro Acinares del páncreas
53) Una mujer de 43 años dolor en hipocondrio derecho e icterica. En la ecografía se evidencia
cálculos biliares. Estos cálculos lo más probable es que se encuentren localizados en:
a) conducto colédoco
b) Conducto cístico
c) Vesícula biliar
d) Conducto pancreático secundario
ECU 1:
Estudiante de 21 años sufre de gastritis aguda ocasionada por comer en lugares poco
higenicos. Suele consumir caramelos (“chupar”) mientras esta en base hasta la tarde.
También toma regular cantidad de leche (grasa, lactora, proteinas), pues le calma el dolor y
el ardor que sitnete por la gastritis (tiene dispepsia y cuando toma la leche se le pasa).
Incluso cuando puede, se toma dos vasos de agua frita y le calma la molestia. Ha decido ir al
medico para tratarse, pues ya no soporta el dolor, el cual esta seguro que los síntomas se
producen por elevada producion de HCl en el estomago, y por ello le ha recetado ranitidina
1.1) El consumir caramelos eleva los niveles en sangre de una hormona cuya función es la
estimulación de las células.
- Beta del páncreas por GIP el cual es una incretina y por consiguiente estimula las
células pancreáticas

1.2) Consumir caramelos indirectamente actica la via:


-POMP/ CART saciedad

1.3) Consumo de leche produce indirectamente


- CCK inhibición del vaciamiento gástrico mayor tonicidad del esfínter pilórico

1.4) Cuando el px toma dos vasos de agua, genera indirectamente un aumento en la


liberación de:
vaso de agua distención → g astrina → secreción de HCl

A
-

1.5) El uso de ranitidina bloquea el receptor H2 de la histamina en las células parietales, la

EK
histamina llega a estas células por:
- histamina es una hormona paracrina por → difusión
**endocrina es por via hematógena y si fuera neuroendorina es por un NTs
OT
1.6) Aumenta la secreción salival:
- noradrenalina a través de los receptores Beta 2
1.7) En este paciente con gastritis aguda debida a una alta producción de ácido clorhídrico, sería
lógico esperar que el píloro tenga un tono muscular:
NC

- primero la secretina
- luego CCK
**ambas reguladores del HCl, Gatritis aguda debido a una alta producción de HCL piloro
estará aumentado (por la CCK)
1.8) Debido al uso de ranitidina, los valores de somatoestina en sangre:
BA

- ranitidina disminuye acción de gastrina se quiere secretar mas no actúan los


inhibidores como la somatoestina somatoestina disminuye
1.9) El uso de atropina en este paciente:
- Inhibirá la acción de las prostaglandinas
- Aumentará la producción de ácido clorhídrico
- Disminuirá la acción del receptor CCK-B
-Aumentará el pH del estómago
ECU 2:
Niño de sexo masculino de 2 años de edad, sufre de estreñimiento desde el nacimiento (1
deposición cada 3-4 días). Madre menciona que le estimula la defecación con un
termómetro rectal, y continuo uso de enemas y laxantes. Desde hace 6 meses comienza
con vómitos postprandiales. Los síntomas aumentan en frecuencia y magnitud y están en
relación con los episodios de estreñimiento. No refiere fiebre, tos, diarrea ni lesiones
cutáneas. Al examen físico presenta regular estado general, luce deshidratado. Abdomen
distendido, blando, depresible e indoloro. No se palpan masas abdominales. Se
permeabiliza el canal anal con termómetro rectal, encontrando cierta resistencia. Salida de
material fecal mal oliente en regular cantidad. Exámenes de laboratorio: hemograma
normal. Signos inflamatorios de fase aguda negativos. Alcalosis metabólica leve en sangre
venosa. Radiografía con enema baritado muestra recto y colon sigmoides dilatados
(megacolon). Biopsia profunda: ausencia de células ganglionares en la muestra enviada. Se
realiza cirugía correctiva.

2.1) Durante la fase esofágica de la deglución, para un bolo alimenticio determinado, a


medida que avanza el bolo la fuerza de la contracción se hace más:
- hiperpolarizado
- fuerte
- dependiente de Ach
- debil
2.2) Cuando este paciente ingiera sus alimentos, se espera que al momento de pasar el bolo
alimenticio por el esfínter esofágico superior, la presión intraesofágica disminuya en:
- la porción proximal al bolo
- el tercio medio del esófago
-el cardias

A
- el lugar donde se contraiga la muscular propia
2.3) Al examinar la orofaringe del paciente, uno puede hallar fácilmente la amígdala

EK
palatina, pues esta se encuentra inmediatamente detrás del músculo:
-Palatogloso
- Palatofaringeo
- Hiogloso
- Elevador del velo del paladar
OT
2.4) Con respecto a la defecación en este caso, marque la respuesta correcta:
- En posición de cuclillas, el músculo puborectal genera un ángulo más agudo en el
recto
NC

- El sigmoides y el recto están inervados por el nervio vago


- La sensación de defecar sólo se da cuando el recto es ocupado por heces y
alcanzado el 80% de su capacidad
-El esfínter anal comprometido tiene inervación autónoma
BA

2.5) En este paciente [hirschsprung] se considera que está abolido el reflejo:


- Coloileal
-Rectoesfinteriano
- Gastrocólico
- Relajación receptiva

2.6) No se espera que sea causa del vómito:


-Ayuno prolongado
- Estimulación faríngea y del glosofaríngeo
- Irritación de la mucosa gástrica
- Dolor intenso

ECU 3:
Paciente de 54 años con antecedentes de alcoholismo, gastritis crónica, tabaquismo
pesado, obesidad, cálculos biliares y cirrosis, es llevado a la emergencia por dolor
abdominal en epigastrio irradiado a la espalda y trastorno del sensorio.
Al examen físico: presión arterial 85/50 mmHg, frecuencia cardíaca 100 latidos/min,
frecuencia respiratoria 18 x minuto, temperatura axilar 36°C.
Conjuntivas pálidas, escleras ictéricas nevus arácnidos en tronco, distensión abdominal
marcada, cabeza de medusa, matidez desplazable en ambos flancos e hipogastrio, dolor a
la palpación de abdomen.
Tiempo de protrombina: 24 seg (testigo: 13 seg); TPT: 38 seg, glicemia: 165 mg/dL, uremia:
20 mg/dL, ASAT: 76 UI/L, ALAT: 22 UI/L, albumina: 2,5 g/dL, bilirrubina total: 2,6 mg/dL,
bilirrubina directa: 1,4 mg/dL, amilasa sérica 4000 U/L.
3.1) En esta paciente, al aumento de la amilasa sérica, se debe directamente a una lesión de:
a) páncreas
b) vesícula y árbol biliar
c) estómago
d) hígado
3.2) Considerando que el paciente sufre de gastritis, se puede decir que la secreción de
ácido por la mucosa gástrica
a) involucra transporte activo de hidrogeniones
b) es realizada principalmente por células principales

A
c) es inhibida por antihistaminas tomadas por pacientes con rinitis alérgica
d) involucra la liberación de HCl de los gránulos zimógenos

EK
3.3) El paciente tiene hemorragia digestiva alta por várices sangrantes como complicación.
Llegando a estar en shock hipovolémico por hemorragia masiva, se encontrara necrosis
hepática en:
a) zona 1
b) no se afectan los lobulillos hepáticos en hemorragia
OT
c) zona 3
d) zona 2
3.4) El misoprostol, análogo de las prostaglandinas está mejor indicado en:
a) cicatrización de úlcera péptica duodenal
b) erradica el helicobacter pylori
NC

c) tratar el sind de Zollinger ellison


d) prevenir daño por AINES
3.5) De las siguientes sustancias secretadas por los órganos de este paciente, la más
alcalina es la secreción:
BA

- Esofágica
- Salival
- Yeyunal
-Pancreática
3.6) En este paciente, se puede asumir que la pancreatitis ha sido ocasionada por una
disminución en el efecto de:
- Lipasa
- Enterocinasa
- Amilasa
-Inhibidor de la tripsina
3.7) ¿Cuál de las siguientes sustancias es segregada por el páncreas?
-Amilasa
- Pepsina
- Quimiotripsina
- Tripsina
3.8) Cada vez que este paciente toma alcohol, la acidificación de la luz del duodeno:
-Disminuye el vaciamiento gástrico
- Aumenta la contracción del esfínter de Oddi
- Aumenta la secreción del ácido gástrico
- Disminuye la secreción pancreática del bicarbonato

A
EK
OT
NC
BA
SISTEMA
DIGESTIVO (ME
154) EXAMEN
FINAL
Ciclo 2018-01

1. Un niño de 2 años es llevado a la consulta por diarrea persistente y edema de las extremidades, además falta
de crecimiento y desarrollo en relación a su edad. Los análisis de sangre revelan que tiene concentración
plasmática baja de proteínas (hipoproteinemia). Durante la endoscopía duodenal, se coloca colecistokinina
(CCK) endovenosa y se recoge muestras del líquido duodenal; el resultado del líquido confirma incapacidad
para hidrolizar proteínas a un pH neutro, esta situación mejora al añadir una pequeña cantidad de tripsina.
El paciente probablemente esté sufriendo la falta congénita de
………….
(Unidad 4, sesión 26, logro 2: Explicar la Digestión y absorción de las proteínas y sus alteraciones)
a. Pepsinógeno

A
b. PEPT-1
c. Carboxipeptidasas
d. Enterocinasa

EK
2. Experimentalmente se incrementa la velocidad de la secreción salival con una sustancia, el análisis de
la composición de esta saliva obtenida se espera encontrar…………..
(Unidad 3, sesión 17, logro 5 : Explica la Influencia de la velocidad del flujo salival en la composición de la
saliva)
OT
a. Elevación de concentración de bicarbonato, sodio y potasio
b. Elevación de concentración de cloro, sodio y potasio
c. Disminución de concentración de potasio
d. Disminución de concentración de potasio y bicarbonato
NC

3. Paciente varón de 46 años soltero, consulta por odinofagia y bajo de peso, tiene antecedente de
tuberculosis desde hace 3 meses y es fumador crónico (10 cigarrillos por día); al evaluar la cavidad oral se
identifica lesión blanquecina en el dorso de la lengua y paladar blando, las lesiones se desprenden con el
baja lengua dejando una base eritematosa. Esta lesión corresponde probablemente a ……………………….…..
( Unidad 3, sesión18, logro 1-2 : Describe las enfermedades inflamatorias, infecciosas y proliferativas de la
BA

cavidad oral)
a. Eritroplaquia
b. Candidiasis oral
c. Leucoplaquia vellosa
d. Fibroma en cavidad oral

4. Minero de 32 años de edad, que acude a centro de


salud por presentar de forma progresiva desde hace
1 año dificultad para ingerir alimentos sólidos y
luego líquidos; refiere regurgitaciones alimentarias y
marcada pérdida de peso (15 kilos). Radiografia
baritada de esófago como se muestra en la figura. El
presente caso se explica por……………….
(Unidad 2, sesión 12, logro 4: Identificar y describir
la función de los esfínteres esofágicos)
a. Contracción incompleta del esfínter esofágico inferior
b. Dificultad para el inicio de la deglución
c. Relajación incompleta del esfínter pilórico
d. Relajación incompleta del esfínter esofágico inferior
5. Paciente mujer de 35 años acude a consulta por sensación de sequedad y lesiones en cavidad oral. Al
examen se observa atrofia de la mucosa, fisuras y úlceras; nota además sequedad e irritación de la
córnea y aumento del tamaño de las glándulas parotídeas. Su diagnóstico más probable es artritis
reumatoide; el hallazgo más probable en una biopsia de glándula parótida es……..….
(Unidad 3, sesión 18, logro 3: Describe las enfermedades más frecuentes de las glándulas salivales)
a. Hiperplasia de acinos glandulares serosos
b. Gran infiltración de linfocitos y células plasmáticas
c. Gran infiltrado de linfocitos y macrófagos
d. Presencia de acinos normales con hiperplasia de células ductales

6. Un paciente con anemia acude con su médico quejándose de episodios frecuentes de gastroenteritis. Un
análisis de sangre revela anticuerpos circulantes dirigidos contra células parietales gástricas. Su anemia es
atribuible a la hiposecreción de
………………………
(Unidad 3, sesión 20, logro 5: Gastritis crónica. Tipos de gastritis)
a. Factor intrínseco

A
b. Proteina R (haptocorrina)
c. Pepsinógeno
d. Ácido clorhídrico

EK
7. Dos estudiantes deciden tomar un receso para comer una hamburguesa a la hora del almuerzo. Antes de
llegar a la cafetería, impulsos nerviosos provenientes del complejo vagal dorsal iniciarán la secreción de
ácido gástrico por la liberación dedesde el sistema nervioso entérico.
(Unidad 3, sesión 20, logro 2: Regulación de la secreción gástrica: estimulación, fases de la secreción)
OT
a. Serotonina
b. Óxido nítrico
c. GRP (péptido liberador de gastrina)
d. Péptido intestinal vaso activo
NC

8. Un niño de cuatro años de edad es llevado a la consulta por cuadros diarreicos frecuentes caracterizados
por heces pálidas, voluminosas y fétidas, presenta bajo peso y talla. Se mide la concentración de cloruro en
el sudor y se encuentra que sus valores son muy elevados. La alteración más importante a nivel de células
ductales del páncreas tiene relación directa con la conductancia de…………
(Unidad 3, sesión 23, logro 5 Explica la Secreción pancreática: formación del jugo pancreático, influencia de la
BA

velocidad de flujo y regulación)


a. Potasio
b. Bicarbonato
c. Sodio
d. Cloro

9. Una mujer de 50 años de edad que sufrió durante varios años resequedad de los ojos debida a producción
inadecuada de lágrimas es enviada con un gastroenterólogo para evaluación de pirosis crónica. El examen
endoscópico revela erosiones y tejido cicatrizal en la parte distal del esófago justo por arriba del esfínter
esofágico inferior. Las lesiones pueden atribuirse a la disminución de uno de los siguientes componentes
salivales:
(Unidad 3, sesión 17, logro 4: Explicar la Formación de la saliva y cuáles son sus componentes)
a. Bicarbonato
b. Lactoferrina
c. Ig A
d. Amilasa
10. Se evalúa los valores séricos de las siguientes sustancias a un paciente con enfermedad hepática terminal;
en este paciente se espera encontrar la combinación con la letra …………
(Unidad 3, sesión 22, logro 5: Describe las Pruebas de función hepática, la Insuficiencia hepática,
encefalopatía hepática e hipertensión portal)

Glucosa Amoniaco Albúmina


a. Aumenta Disminuida Disminuida
da
b. Disminui Aumentada Aumentada
da
c. Aumenta Aumentada Aumentada
da
d. Disminui Aumentada Disminuida
da

A
11. Una mujer de 35 años de edad HIV positiva, se presenta al médico con dolor abdominal en cuadrante
superior derecho e ictericia. La paciente refiere haber tenido múltiples episodios de ictericia durante los

EK
últimos 10 años. Los exámenes para determinar hepatitis viral, dieron positivos para Hepatitis B, siendo
catalogado el caso como hepatitis crónica con alteración funcional. En un examen de sangre ¿cuál de los
siguientes parámetros está disminuido?
(unidad 3, sesión 22, logro 5: Pruebas de función hepática, Insuficiencia hepática, encefalopatía hepática
e hipertensión portal)
OT
a. Fosfatasa alcalina
b. Albumina
c. Bilirrubina
d. Tiempo de protrombina
NC

12. En el reflejo peristáltico del intestino delgado, uno de los siguientes eventos sucede en la
porción oral del bolo alimenticio…………...
(Unidad 2, sesión 13, logro 4: Explicar la Motilidad del intestino delgado: Contracciones segmentarias y
peristálticas)
a. Disminución de 5 hidroxitriptamina desde las neuronas IPAN
BA

b. Contracción del músculo longitudinal


c. Acción del péptido intestinal vasoactivo (VIP) en el músculo circular
d. Acción de acetilcolina en el músculo circular

13. Experimentalmente se coloca una dosis alta de secretina en la luz intestinal duodenal; como
consecuencia de esto, en el jugo pancreático de la misma luz intestinal se observa la disminución de la
concentración de …..………..
(Unidad 3, sesión 23, logro 5: Explica la Secreción pancreática: formación del jugo pancreático, influencia
de la velocidad de flujo y regulación)
a. Na+
b. Cl-
c. K+
d. HCO3-

14. Un varón de 58 años de edad con enfermedad de Crohn severo fue sometido a una resección ileal.
Después de la cirugía este paciente padecerá de esteatorrea, esto se explica porque …..………..
(unidad 4, sesión 26, logro 4: Explica las alteraciones en la Absorción de lípidos)
a. El pool de ácidos biliares se incrementa
b. Los quilomicrones no pueden formarse en el lumen intestinal
c. La micelas no pueden formarse
d. El páncreas no secreta lipasa

15. En un experimento se inserta un balón en el estómago de un voluntario, se infla poco a poco mientras que
se vigilan las presiones intraluminales. Aunque el volumen del balón aumenta considerablemente, las
presiones permanecen constantes. Esta relación volumen-presión se explica por la liberación local de
…………..
(Unidad 2, sesión 13, logro 1 Explica la Motilidad gástrica: relajación receptiva)
a. Acetil colina y gastrina
b. Colecistoquinina y óxido nítrico
c. Óxido nítrico y péptido inhibidor vasoactivo
d. Norepinefrina y óxido nítrico

A
EK
OT
NC
BA
16. La toxina del Vibrio cholerae causa diarrea debido a…….
(Unidad 4, sesión 27, logro 6: Explica el transporte hidroelectrolítico intestinal, toxina colérica)
a. La fosforilación del canal CFTR de los enterocitos de las vellosidades intestinales
b. El Incremento de la secreción de cloro por las células de la cripta intestinal
c. La inhibición de la producción de AMPc por las células epitelailes
d. El incremento de la absorción de agua y sodio a través de las uniones estrechas

17. ¿Cuál de las siguientes alternativas es una característica de la secreción exocrina del páncreas?
(Unidad 3, sesión 23, logro 5: Secreción pancreática: formación del jugo pancreático, influencia de la
velocidad de flujo y regulación)
a. Tiene una baja concentración de Cl- respecto al plasma
b. Es estimulada por la presencia de bicarbonato en el duodeno
c. La secreción enzimática es estimulada principalmente por la gastrina
d. Es hipotónica respecto al plasma

A
18. Una madre lleva a su hijo de dos años de edad a la sala de urgencias, estresada porque el niño deglutió una
moneda de 10 céntimos mientras la familia cenaba en un restaurante. El médico observa mediante
fluoroscopía que la moneda se halla en el estómago y asegura a la madre que la moneda se eliminará con

EK
las heces. El médico recomienda utilizar la respuesta fisiológica que permitirá la evacuación de la moneda
del estómago al intestino ………….…..
(Unidad 2, sesión 13, logro 2: Explica la Motilidad gástrica: mezclado y vaciamiento)
a. Es por la relajación receptiva
b. Son los movimientos de mezcla y trituración
c. Es provocada por el ayuno
OT
d. Es por la relajación del esfínter esofágico superior

19. Las estructuras en el hígado que permite que los productos metabólicos unidos a proteínas tengan acceso
a las membranas basolaterales de los hepatocitos, son…..
(Unidad 3, sesión 21, logro 4-5: Explica la Organización micro estructural del hígado)
NC

a. Los Canalículos
b. Las fenestras sinusoidales
c. Las uniones intercelulares herméticas
d. Las células de Ito
BA

20. La composición de la bilis es modificada conforme fluye por los conductillos biliares. Durante este
tránsito se espera que aumente la concentración de…….
(Unidad 3, sesión 22, logro 2: Describe la Secreción biliar, visión general del sistema biliar extrahepático y
composición de la bilis)
a. Ig A
b. Glucosa
c. Monómeros de ácido biliar
d. Vitamina A

21. Se mide experimentalmente el contenido gástrico de dos personas. La persona “A” tiene alto
contenido de grasa y la persona “B” tiene un contenido hipertónico ¿Cuál de las siguientes es
correcto respecto al vaciamiento gástrico? (Unidad 2, sesión 13, logro 2: Describe la Motilidad y
vaciamiento gástrico)
a. Hay ralentización del vaciado gástrico solo en “A”
b. El vaciamiento gástrico es más rápido en ambos
c. En ambos casos hay incremento de la motilina
d. Hay ralentización del vaciado gástrico en ambos casos

22. El examen endoscópico de un paciente con hipertensión portal grave revela venas tortuosas que sobresalen
hacia la luz del esófago. El paciente recibe tratamiento quirúrgico mediante la colocación de una derivación
que conecta la vena porta a la vena cava. Después de la operación el riesgo de encefalopatía …………………..
y el riesgo de sangrado de várices ……………..
(Unidad 3, sesión 22, logro 5: Describe la Insuficiencia hepática, encefalopatía hepática e hipertensión portal)
a. Aumentará/disminuirá
b. Disminuirá/disminuirá
c. Aumentará/aumentará
d. Disminuirá/aumentará

A
EK
OT
NC
BA
23. Un paciente varón de 18 años de edad acude al médico para sus exámenes de rutina. Sus resultados de
laboratorio muestran un valor de bilirrubina sérica de 4 mg/dl y una bilirrubina directa de 0,3 mg/dl. Las
pruebas de función hepática son normales. La alteración que explica mejor este caso es por la
deficiencia de ………………..
(Unidad 3, sesión 22, logro 3: Explica la Producción y excreción de bilirrubina. Tipos de bilirrubina e ictericia)
a. Transaminasas
b. Glucuronil transferasa
c. Hemo oxigenasa
d. La 7 alfa hidroxilasa

24. Un hombre de 57 años de edad es llevado a urgencias con hematemesis masiva rojo brillante, a su llegada
se halla inconciente con PA: 80/40 mm Hg y FC: 124 lat/min. Luce ictérico con presencia de “arañas
vasculares en el tórax anterior y extremidades”, abdomen distendido con signo de oleada positiva. Se
encuentra esplenomegalia y pérdida de la masa muscular en extremidades. La anastomosis vascular
responsable del sangrado en este paciente es ………….…..
(Unidad 3, sesión 21, logro 2: Describe las anastomosis porto sistémicas)

A
a. Arteria gástrica izquierda y vena ácigos
b. Vena gástrica izquierda y vena ácigos
c. Vena paraumbilical y vena epigástrica inferior

EK
d. Vena gástrica izquierda y vena esofágica superior

25. Un estudiante de medicina está comiendo un plato de comida a base de champiñones, espárrago y salsa
de soya. El sabor umami contenido en todos estos alimentos actúa a nivel de los botones gustativos
estimulando ………………..
OT
(Unidad 2, sesión 10, logro 5: Describe los tipos y mecanismos moleculares para la detección de los sabores)
a. El ingreso de sodio
b. Un receptor acoplado a proteína G
c. Su receptor específico T1R3
d. El ingreso de hidrógeno
NC

26. Un hombre de 22 años de edad se presenta al médico con una historia de 1 año de evolución
caracterizado por dolor recurrente en fosa iliaca derecha y diarrea. Manifiesta además pérdida de
peso de 8 kg durante este periodo. La colonoscopía revela múltiples lesiones en el ileon terminal y
colon. La biopsia de estas lesiones revela engrosamiento, inflamación y ulceración de la mucosa. El
diagnóstico más probable en este caso es…….
BA

(Unidad 4, sesión 28, logro 5: Describe la Enfermedad inflamatoria intestinal. Generalidades, morfología y
características)
a. Sprue celiaco
b. Enfermedad de Crohn
c. Sindrome de colon irritable
d. Colitis ulcerativa

27. Una de las funciones del músculo señalado es:


(Unidad 2, sesión 8, logro 3: Describir el Piso de la
boca: estructuras blandas que la conforman)
a. Eleva el paladar blando
b. Recibe inervación del nervio maxilar
c. Deprime el hioides cuando la mandíbula está fija
d. Deprime la mandíbula cuando el hioides está fijo

28. Varón de 61 años que consulta por dolor retro esternal intenso desde hace 6 horas y después de
vómitos intensos y repetidos; al examen se observa disnea, cianosis, hipotensión y signos clínicos de
shock. La radiografía simple de tórax muestra neumomediastino. El líquido en el espacio pleural
aspirado tiene alta concentración de amilasa. ¿Cuál de las siguientes alternativas puede explicar este
cuadro clínico?
(Unidad 3, sesión 18, logro 6: Describe algunas Enfermedades del esófago)
a. Sindrome de Mallory Weiss
b. Rotura espontánea de esófago
c. Neumotórax por probable herida penetrante
d. Perforación de ulcera gástrica de cara posterior, con complicación torácica

A
EK
OT
NC
BA
29. La secreción del ácido en la célula parietal gástrica se lleva a cabo por una ATPasa especifica que
intercambia hidrogeniones (H+) del citosol por…..
(Unidad 3, sesión 20, logro 1: Explica la Secreción del HCl y sustancias que la alteran)
a. Cl-
b. HCO3-
c. Na +
d. K+

30. En condiciones normales el ingreso de 600 ml de líquido es el estómago provoca un aumento de presión
intragástrica de unos 12 cm de H2O. Después de una vagotomía (corte del nervio vago) es de esperar que
el ingreso del mismo volumen de líquido provoque lo siguiente: …………………………………
(Unidad 2, sesión 13, logro 1: Describe la Motilidad gástrica: relajación receptiva)
a. Un aumento igual de la presión
b. Que no aumente la presión
c. Un aumento mayor de la presión
d. Una disminución de la presión

A
31. Una paciente de 30 años de edad es sometida a una cirugía en oído medio derecho por un problema de

EK
otoesclerosis. Luego de la cirugía refiere alteración en la percepción de sabores. Al evaluar el caso usted
esperaría encontrar……….
(Unidad 2, sesión 10, logro 5: Describe la Irrigación e inervación de la lengua)
a. Alteración en la sensación del dolor y temperatura en el tercio posterior de la lengua
b. Alteración en la sensación del gusto en los dos tercios anteriores de la lengua
c. Alteración en la sensación del gusto en la punta de la lengua
OT
d. Sensación del dolor, tacto y temperatura conservada en toda la lengua

32. ¿Cuál de las siguientes alterativas es correcta?


(Unidad 4, sesión 26 : Explica la digestión y absorción de nutrientes y sus alteraciones)
a. En el borde luminal, en cepillo, del intestino delgado, la absorción de sodio únicamente se realiza
NC

asociada a la de glucosa.
b. El lugar principal para la absorción del hierro es el ileon
c. Las sales biliares desconjugadas son absorbidas preferentemente en el colon
d. El proceso de digestión y absorción de la vitamina B12 no se altera en insuficiencia pancreática.

33. En un paciente de 45 años de edad con colestasis biliar, se encuentra una elevación de los niveles
BA

sanguíneos de fosfatasa alcalina hasta 3 veces la cifra normal. ¿Cuál de las siguientes alternativas estará
también elevada como evidencia del daño de la vía biliar?
(Unidad 3, sesión 22, logro 5: Pruebas de función hepática, Insuficiencia hepática, encefalopatía hepática
e hipertensión portal)
a. Tiempo de protrombina y albúmina sérica
b. Transaminasas hepáticas (ALT y AST)
c. Glucoronil transferasa
d. Gamma glutamil transpeptidasa

34. Revisando la angiografía de un hombre de 70 años en estudio por aneurisma de aorta abdominal el
radiólogo informa de la presencia de una oclusión completa de la arteria mesentérica inferior. El paciente
se encuentra completamente asintomático. ¿Cuál de las siguientes arterias se anastomosa a la sistema
arterial de la mesentérica inferior?
(Unidad 4, sesión 25, logro 1: Identifica la Arteria mesentérica superior e inferior, ramas y anastomosis)
a. Ileal
b. Cólica media
c. Sigmoideas
d. Cólica izquierda

35. Lactante de 3 meses de vida es atendido por presentar diarrea, se administra una solución de glucosa y
electrólitos por vía oral. La proteína de membrana apical que explica la capacidad de esta solución para
proporcionar aporte de glucosa e hidratación es ………..
(Unidad 4, sesión 26, logro 1: Explica la Digestión y Absorción de los hidratos de carbono. Alteraciones)
a. GLUT-5
b. SGLT-1
c. CFTR
d.
GLUT-2

BA
NC
OT
EK
A
36. Paciente ha sufrido herida de bala en el abdomen, se le ha tenido que extirpar el segmento medio y distal
del ileon. En este caso la síntesis hepática de sales biliares estará …..…..
(Unidad 3, sesión 22, logro 4: Explica la formación, función y Circulación entero hepática de lasa sales biliares)
a. Disminuida por inhibición de la colesterol 7 alfa hidroxilasa
b. Incrementada por estímulo de la enzima colesterol 7 alfa hidroxilasa
c. Incrementada por inhibición de la colesterol 7 alfa hidroxilasa
d. Sin cambios en el ritmo de síntesis

37. Un varón de 75 años ingresa al consultorio por presentar ictericia marcada de piel y las escleras. El estudio
del paciente mostró que presentaba un tumor que obstruía la totalidad del conducto hepático común.
¿Cuál de las siguientes estructuras se encontrará dilatada en este paciente?
(Unidad 3, sesión 21, logro 6: Describir el árbol biliar intrahepático)
a. Conducto de Wirsung
b. Conductos de Hering

A
c. Conducto colédoco
d. Conducto cístico

EK
38. Correlaciones las dos columnas y marque la fórmula correcta:
(Unidad 4, sesión 28, logro 1: Diarrea: definición, mecanismos: osmótica, secretoria y exudativa)
1. Enfermedad Hirschsprung( ) heces con moco y sangre
2. Diarrea osmótica( ) intolerancia a lactosa
3. Diarrea secretoria( ) aganglionosis congénita
OT
4. Diarrea exudativa( ) canales de Cl- en las células de la cripta

a.- 4231b.- 1234c.- 2143d.- 4213


NC

39. Respecto a la siguiente imagen que representa una estructura de la mucosa gástrica, la estructura con
número ………..
produce ……………………..
(Unidad 3, sesión 19, logro 4: La glándula fúndica. Funciones y tipos de células con sus características)
BA

a. 3 / pepsina
b. 1 / Pepsinógeno
c. 4 / HCl y factor extrínseco
d. 2 / pepsinógeno

2
40. En un paciente con insuficiencia renal crónica, el déficit en la absorción de calcio a nivel del
enterocito se debe a lo siguiente:
(Unidad 4, sesión 26, logro 6: Explica la Absorción de calcio y hierro)
a. No se convierte la 25 hidroxicolecalciferol a 1,25 dihidroxicolecalciferol
b. No se convierte la 1,25 dihidroxicolecalciferol a 25 hidroxicolecalciferol
c. Se incrementa la producción de Calbindina
d. Existe un descenso de la alfa 25 hidroxilasa renal

A
EK
OT
NC
BA
CLAVES EXAMEN
PARCIAL DE
SISTEMA
DIGESTIVO 2019 -
00

1. Varón de 30 años es traído a emergencia por agresión abdominal con arma de fuego (pistola)
y es sometido a laparotomía exploratoria, observándose isquemia del colon ascendente y
parte del colon trasverso ¿la lesión de cuál de las siguientes arterias explicaría esta isquemia?
(unidad 1, sesión 2, logro 6: (Describe la irrigación visceral: arterias de tronco celiaco,
arteria mesentérica superior e inferior, topografía de superficie, órganos por cuadrante)

A
a. Celiaca
b. Colónica derecha
c. Mesentérica inferior

EK
d. Mesentérica superior

2. Respecto a las sustancias gastrointestinales que regulan la secreción pancreática;


marque la afirmación correcta:
(unidad 1, sesión 3, logros 2 y 3: describir las hormonas gastrointestinales: estímulos y funciones)
OT
a. La Secretina, es la hormona más importante para la secreción de bicarbonato por las
células acinares del páncreas
b. La acetilcolina es capaz de estimular la secreción enzimática y de bicarbonato del páncreas
c. La gastrina, es la hormona más importante para la secreción de enzimas pancreáticas
d. La colecistoquinina (CCK) estimula al páncreas solo para secreción enzimática
NC

3. Ante una lesión del X par craneal, ¿cuál de los siguientes músculos mantiene conservada su
función?:
(unidad 2, sesión 08, logro 4: Paladar blando: componentes musculares)

a. Elevador del velo del paladar


BA

b. Tensor del velo del paladar


c. Palatofaríngeo
d. Glosofaríngeo

4. Experimentalmente se utiliza atropina (anticolinérgico) para inhibir la secreción de


gastrina, sin embargo la secreción de esta hormona se sigue dando ante estímulos
vagales. Esta situación se explica porque la atropina:
(unidad 1, sesión 3, logro 3 : describir las hormonas gastrointestinales: estímulo y funciones de la
gastrina y colecistoquinina)

a. Bloquea parcialmente la bomba de protones en la célula G


b. Inhibe la acción de acetilcolina e histamina en la célula G
c. Solo inhibe la acción del péptido GRP en la célula G
d. No bloquea la acción del péptido GRP

5. Un varón de 50 años es sometido a extirpación del duodeno y parte proximal del


yeyuno. Esta situación ocasionaría la pérdida de las células ……….. , productoras deque
estimula la secreción de
bicarbonato por el páncreas.
(unidad 1, sesión 3, logro 3: describir las hormonas gastrointestianles: estímulos y funciones
de la secretina y péptido insulinotrópico dependiente de glucosa)

a. “S” / secretina
b. Parietales / secretina
c. “I” / colecistoquinina
d. “S” / colecistoquinina
6. Recién nacido que presenta tumoración abdominal a nivel del cordón umbilical
(fotografía). ¿cuál de las siguientes afirmaciones es correcta respecto a este defecto en
el desarrollo embriológico del intestino?: (unidad 1, sesión 5, logro 2: identificar las
anomalías del desarrollo del intestino medio)

a. Corresponde a una Gastrosquisis


b. Las vísceras se hallan cubiertas por piel
c. No está asociado a otras malformaciones
d. Se asocia a
malformaciones cardiacas
y del tubo neural

A
7. Varón de 35 años acude a la emergencia por
trauma abdominal y se decida realizar una
laparoscopía exploratoria. El cirujano observa la
EK
OT
disposición de los órganos abdominales como se
representa en el siguiente esquema. Esta
disposición de órganos se explica por la
rotación(SMA=arteria mesentérica superior)
(unidad 1, sesión 5, logro 3: identificar las anomalías del
desarrollo del intestino medio: defectos de rotación,
NC

estenosis y atresias)

a. anti horaria del intestino medio, en sólo 90°


b. incompleta del intestino medio (270°)
c. horaria del intestino medio
BA

d. horaria del estómago

8. Se evalúa la expresión de la proteína Agrp en una persona con alteración del apetito; lo
correcto respecto a esta proteína es…..
(unidad 1, sesión 3, logro 4: Explica los mecanismos de control del apetito y saciedad )

a. Esta proteína es un potente anorexigénico


b. La mutación del gen que la codifica produce adelgazamiento
c. La sobre producción de la proteína lleva a obesidad por agonismo de receptores MC3 y MC4
d. La sobre producción de la proteína disminuye el apetito por antagonismo de receptores MC4

9. Juana cae de la bicicleta y se fractura la región anterior del hueso maxilar superior con
compromiso de la fosa incisiva. Al examen físico de la región esperaría encontrar alteración
en la sensibilidad de la encía …………………
(unidad 2, sesión 8, logro5: paladar: paladar duro y blando: irrigación e inervación)

a. bucal posterior
b. Lingual anterior
c. palatina anterior
d. palatina posterior
10. Recién nacido es atendido por el neonatólogo y luego entregado a su madre para dar de
lactar; la madre al dar de lactar observa coloración azulada de labios, acompañado de tos
persistente, dificultad respiratoria y distención abdominal. Se le intenta colocar una sonda
nasogástrica pero esta retorna a la cavidad oral en todos los intentos. ¿Cuál de las
siguientes anomalías del desarrollo es el más probable en este caso? (unidad 1, sesión 4,
logro 3: identificar las anomalías en el desarrollo del esófago: atresia y/o fístula traqueo
esofágica)

a. Estenosis esofágica proximal con Fístula traqueo esofágica distal


b. Atresia esofágica proximal con fístula traqueoesofágica distal
c. Atresia esofágica distal con fístula traqueoesofágica proximal
d. Fístula traqueoesofágica proximal y distal

11. ¿Cuál de los siguientes mecanismos ocurre durante la defecación?


(unidad 2, sesión 13, logro 6: motilidad del intestino grueso: contracciones segmentarias,
movimientos en masa, defecación y reflejo gastrocólico)

A
a. Contracción refleja del esfínter anal interno

EK
b. En la posición de “cuclillas” el músculo puborectal se halla relajado
c. Relajación del esfínter anal externo por efectos del VIP y óxido nítrico
d. La materia fecal en el recto estimula la contracción del sigmoides por los nervios pudendos

12. La estructura número 4 (gráfico) corresponde a


……….… y está ………..
OT
(unidad 2, sesión 9, logro 2: Partes
de un diente. Capas del diente:
Esmalte: características y células
que lo producen)

a. el cemento / mineralizado en 90%


NC

b. la dentina / formada por ameloblastos


c. el esmalte / formado por
células derivadas del
mesénquima
d. la dentina / formado por
BA

células derivadas de la cresta


neural

13. Un paciente luego de un accidente sufre lesión del piso de la boca, se constata daño del
nervio “cuerda del tímpano”, en este caso se esperaría encontrar disminución de lade la
lengua
(unidad 2, sesión 10, logro 3: Irrigación e inervación de la lengua)

a. Motilidad en los dos tercios anteriores


b. Sensación del gusto en el tercio posterior
c. Sensación del gusto en los dos tercios anteriores
d. Sensibilidad al tacto en los dos tercios anteriores

14. ¿Cuál de las siguientes afirmaciones es la correcta sobre la gastrina?


(unidad 1, sesión 3, logro 1: reconocer las características de las sustancias reguladoras
gastrointestinales: hormonas, sustancias paracrinas y neurocrinas)

a. Produce atrofia de la mucosa gástrica


b. Es producida por la célula G del cuerpo gástrico
c. Es estimulada por la distensión gástrica y el Ph bajo
d. Actúa en la célula diana mediante su receptor CCk tipo B
15. Al recibir un paciente con signos de hipovolemia y antecedente de trauma en abdomen por
accidente de tránsito, usted identifica radiológicamente: lesión de primera vértebra lumbar y
signos de lesión en páncreas; durante la cirugía se observó pobre irrigación de asas
intestinales. El vaso afectado es la arteria ……..
(unidad 1, sesión 1, logro 6: reconocer las estructuras a nivel de L1, nivel de los principales vasos
sanguíneos)

a. esplénica
b. hepática común
c. mesentérica inferior
d. mesentérica superior

16. Un paciente sufre de daño a nivel del cuello con lesión muscular en la región de la faringe. En
el examen físico se determina dificultad para la elevación de la faringe y para el cierre del
itsmo de las fauces. En este caso, probablemente esté afectado el músculo:
(unidad 2, sesión 11, logro 2: Músculos de la faringe: identificación, constrictores y longitudinales)

A
a. palatogloso

EK
b. estilofarinfeo
c. palatofaringeo
d. constrictor inferior

17. Varón de 50 años a quien le realizan la curación de la segunda molar de la arcada


superior derecha. En un momento determinado, el paciente acusa de intenso dolor de la
OT
pieza dentaria en tratamiento. La vía aferente del dolor viaja a través del nervio …………
(unidad 2, sesión 9, logro 6: Inervación de los dientes)

a. trigémino V2
b. trigémino V3
c. naso palatino
NC

d. palatino menor

18. La distención gástrica por los alimentos produce incremento de secreción de HCl mediante la
producción de
………….. que estimula a las células ……………. vía proteína ………..
BA

(Unidad 1, sesión 3, logro 2: Describe las hormonas gastrointestinales: Estímulo y funciones de la


gastrina y colecistoquinina)

a. gastrina / parietal / Gq
b. gastrina / principal / Gs
c. acetilcolina / parietal /Gi
d. acetilcolina / principal / Gi

19. Un niño de tres años llega a emergencia con disfagia (dificultad para tragar), dolor retro
esternal, salivación y llanto. Se sospecha de ingesta de cuerpo extraño (moneda) en el
esófago; al ser evaluado se constata en una radiografía presencia de cuerpo extraño a nivel
de C6 (6° vértebra cervical). El cuerpo extraño estará suspendido a nivel del estrechamiento
producido por………..
(unidad 2, sesión 11, logro4: Esófago, características anatómicas, relación con órganos vecinos y
estrecheces)

a. el cayado aórtico
b. el hiato esofágico
c. el músculo cricofaríngeo
d. el bronquio principal izquierdo

20. La triada portal (arteria hepática, vena portal y conducto biliar común) está contenida en el
ligamento
…….……… y derivan embriológicamente del ……
(Unidad 1, sesión 1, logro 4: Identifica el peritoneo, mesenterio, omento y ligamentos,
retroperitoneo.)

a. hepato duodenal / mesenterio ventral


b. gastro esplénico / mesenterio dorsal
c. hepato gástrico / omento menor
d. falciforme / omento menor

A
EK
OT
NC
BA
21. En relación al movimiento de
peristaltismo del tubo digestivo: en
la flecha negra del gráfico se
produce la liberación de ………………
a nivel del músculo ………..
(unidad 2, sesión 7, logro 6: Control
hormonal y tipos de movimiento)

a. noradrenalina, sustancia P y neuropéptido “ Y” / circular


b. acetilcolina y sustancia P / longitudinal
c. óxido nítrico y PIV / longitudinal
d. óxido nítrico y PIV / circular

A
EK
22. Un paciente refiere no percibir algunos sabores, al examen físico se constata alteración en
la percepción de sabores y del dolor en el tercio posterior de la lengua ¿Cuál de los
siguientes nervios estará alterada en su función?
(unidad 2, sesión 10, logro 5: Sabores, tipos y mecanismos moleculares para su detección)

a. Lingual (rama del V par)


OT
b. Cuerda del tímpano (VII par)
c. Glosofaríngeo (IX par)
d. Hipogloso (XII par)

23. El gráfico detalla la estructura de


la pared del tubo digestivo
NC

intestinal ¿Cuál de las siguientes


asociaciones es correcta?
(unidad 2, sesión 7, logro 1: La
pared y músculo liso
gastrointestinal )
BA

a. “1” – peristaltismo
b. “2” – secreción enzimática
c. “3” – deriva del mesodermo
d. “4” – doble hoja de tejido graso

24. En el caso de un paciente con gastrinoma (tumor productor de gastrina), la presencia de


úlceras duodenales y erosión de la mucosa gástrica, se debe principalmente a…….
(unidad 1, sesión 3, logro 2: describir las hormonas gastrointestinales: estímulo y funciones de la
gastrina y colecistoquinina)

a. la acción directa de la gastrina sobre la célula principal


b. la sobre expresión de los receptores “G” en la célula parietal
c. el exceso de HCl por estímulo de receptores CCK-B en la célula parietal
d. el exceso de HCl por estímulo directo de receptores de acetilcolina en la célula parietal

25. El reflejo entero gástrico se caracterizan por:


(unidad 2, sesión 13, logro 6: Motilidad del intestino grueso: contracciones segmentarias,
movimientos en masa defecación y reflejo gastrocólico)

a. favorecer la motilidad gástrica gracias a la CCk


b. inhibir la motilidad gástrica y estimular la secreción ácida
c. movilizar grandes volúmenes desde el estómago al duodeno
d. originarse debido a la distensión duodenal y presencia del quimo ácido
26. Mauricio tiene dificultad para deprimir el paladar y elevar la parte posterior de la lengua. En
este caso estará afectado un músculo ………………., específicamente el músculo …………….
(Unidad 2, sesión 10, logro 2: Músculos de la lengua: clasificación, identificación y sus funciones)

a. intrínseco – longitudinal inferior


b. extrínseco – palatogloso
c. extrínseco – transverso
d. extrínseco – estilogloso

27. Una de las funciones del músculo señalado es:


(Unidad 2, sesión 8, logro 3: Describir el Piso de la boca:
estructuras blandas que la conforman)

A
a. deprimir la lengua
b. elevar el paladar blando

EK
c. deprimir el hioides cuando la mandíbula está fija
d. deprimir la mandíbula cuando el hioides está fijo
OT
28. Paciente varón de 30 años es evaluado por probable enfermedad de Chagas, cursa con
problemas de motilidad del colon; los estudios de biopsia determinan ausencia de
células ganglionares. Según el gráfico
¿cuál es la capa en la que se determina la ausencia de dichas células?
(unidad 1, sesión 2, logro 1: describir las generalidades de la estructura del tubo digestivo: esófago,
NC

estómago intestino delgado y grueso)

a. Mucosa - 1
b. Muscular propia – 1
c. Muscular de la mucosa - 2
BA

d. Muscular propia - 3

1 2

3
29. Paciente varón de 32 años, que acude a centro de salud por presentar de forma progresiva
desde hace 1 año dificultad para ingerir alimentos sólidos y luego líquidos; refiere
regurgitaciones alimentarias y marcada pérdida de peso (15 kilos). Radiografía baritada
(sustancia de contraste) de esófago se muestra en la figura. El presente caso se explica
por……………….
(Unidad 2, sesión 12, logro 4: Identificar y describir la función de los esfínteres esofágicos)

a. aumento de la peristalsis esofágica


b. relajación incompleta del esfínter pilórico
c. relajación incompleta del esfínter esofágico inferior
d. perdida de producción de PIV y óxido

A
nítrico en el esfínter esofágico superior

EK
OT
30. En condiciones normales, el ingreso de 600 ml de líquido es el estómago provoca un
aumento de presión intragástrica de unos 12 cm de H2O. Después de una vagotomía (corte
del nervio vago) es de esperar que el ingreso del mismo volumen de líquido ocasionede la
NC

presión intragástrica.
(Unidad 2, sesión 13, logro 1: Describe la Motilidad gástrica: relajación receptiva)

a. la disminución
b. la no variación
BA

c. un aumento mayor
d. un aumento similar o igual
SIS
TEMA
DIGESTIVO
(ME 154)
CLAVES
EXAMEN
FINAL
Ciclo 201900

1. Un niño de 2 años es llevado a la consulta por diarrea persistente, edema de las extremidades y
falta de crecimiento en relación a su edad. Los análisis de sangre revelan que tiene concentración
plasmática baja de proteínas (hipoproteinemia). Como parte del estudio se coloca colecistokinina
(CCK) endovenosa y se recoge muestras del líquido duodenal por endoscopía; el resultado del
líquido confirma incapacidad para hidrolizar proteínas a un pH neutro, esta situación mejora al
añadir una pequeña cantidad de tripsina. El paciente probablemente esté sufriendo la falta

A
congénita de ………….
(Unidad 4, sesión 26, logro 2: Explicar la Digestión y absorción de las proteínas y sus alteraciones)
a. PEPT-1

EK
b. pepsinógeno
c. enterocinasa
d. carboxipeptidasas

2. Paciente mujer de 35 años acude a consulta por sensación de sequedad y lesiones en cavidad
OT
oral. Al examen se observa atrofia de la mucosa, fisuras y úlceras; nota además sequedad e
irritación de la córnea y aumento del tamaño de las glándulas parotídas. Su diagnóstico más
probable es artritis reumatoide; el hallazgo más probable en una biopsia de glándula parótida
es……..….
(Unidad 3, sesión 18, logro 3: Describe las enfermedades más frecuentes de las glándulas salivales)
a. Presencia de acinos normales con hiperplasia de células ductales
NC

b. Gran infiltración de linfocitos y células plasmáticas


c. Hiperplasia de acinos glandulares serosos
d. Gran infiltrado de linfocitos y neutrófilos

3. Un hombre de 42 años de edad se presenta al médico con una historia de 1 año de evolución,
caracterizado por dolor abdominal bajo y diarreas con crisis sanguinolentas. Manifiesta además
BA

pérdida de peso de 8 kg durante este periodo. La colonoscopía revela lesión difusa en el colon
con afectación del recto. La biopsia de estas lesiones revela adelgazamiento de la pared,
inflamación y ulceración de la mucosa y sub mucosa. El diagnóstico más probable en este caso es:
(Unidad 4, sesión 28, logro 5: Describe la Enfermedad inflamatoria intestinal. Generalidades,
morfología y características)
a. sindrome de colon irritable
b. enfermedad de Crohn
c. colitis ulcerativa
d. sprue celiaco

4. Dos estudiantes deciden tomar un receso para comer una hamburguesa a la hora del almuerzo.
Antes de llegar a la cafetería, impulsos nerviosos provenientes del complejo vagal dorsal iniciarán
la secreción de ácido gástrico por la liberación dedesde el sistema nervioso entérico.
(Unidad 3, sesión 20, logro 2: Regulación de la secreción gástrica: estimulación, fases de la secreción)
a. Serotonina
b. Colecistoquinina
c. Péptido inhibidor vaso activo
d. GRP (péptido liberador de gastrina)

5. Un niño de cuatro años de edad es llevado a la consulta por cuadros diarreicos frecuentes
caracterizados por heces pálidas, voluminosas y fétidas; al examen físico presenta bajo peso
y talla para la edad. Se mide la concentración de cloruro en el sudor y se encuentra que sus
valores son muy elevados. La alteración más
importante a nivel de células ductales del páncreas tiene relación directa con la conductancia de…………
(Unidad 3, sesión 23, logro 5 Explica la Secreción pancreática: formación del jugo
pancreático, influencia de la velocidad de flujo y regulación)
a. Bicarbonato
b. Potasio
c. Sodio
d. Cloro

A
EK
OT
NC
BA
6. Se evalúa los valores séricos de las siguientes sustancias a un paciente con enfermedad hepática
terminal; en este paciente se espera encontrar la combinación con la letra …………
(Unidad 3, sesión 22, logro 5: Describe las Pruebas de función hepática, la Insuficiencia
hepática, encefalopatía hepática e hipertensión portal)
Glucosa Amoniaco Albúmin
a
a. Aumenta Disminuida Disminui
da da
b. Disminui Aumentada Aumenta
da da
c. Aumenta Aumentada Aumenta
da da
d. Disminui Aumentada Disminui
da da

A
7. Una mujer de 35 años de edad HIV positiva, se presenta al médico con dolor abdominal en
cuadrante superior derecho e ictericia. La paciente refiere haber tenido múltiples episodios de

EK
ictericia durante los últimos 10 años. Los exámenes para determinar hepatitis viral, dieron
positivos para Hepatitis B, siendo catalogado el caso como hepatitis crónica con alteración
funcional. En un examen de sangre ¿cuál de los siguientes parámetros está disminuido?
(unidad 3, sesión 22, logro 5: Pruebas de función hepática, Insuficiencia hepática,
encefalopatía hepática e hipertensión portal)
a. Albumina
OT
b. Bilirrubina
c. Fosfatasa alcalina
d. Tiempo de protrombina

8. En el reflejo peristáltico del intestino delgado ¿Cuál de los siguientes eventos sucede en la
porción caudal del bolo alimenticio?
NC

(Unidad 2, sesión 13, logro 4: Explicar la Motilidad del intestino delgado: Contracciones
segmentarias y peristálticas)
a. Acción del péptido inhibidor vasoactivo (VIP) en el músculo circular
b. Acción del NO (óxido nítrico) en el músculo longitudinal
c. Contracción del músculo longitudinal interno
BA

d. Acción de acetilcolina en el músculo circular

9. Un varón de 58 años de edad con enfermedad de Crohn severo fue sometido a una resección
ileal. Después de la cirugía este paciente padecerá de esteatorrea, esto se explica porque
…..………..
(unidad 4, sesión 26, logro 4: Explica las alteraciones en la Absorción de lípidos)
a. se inhibe la acción de la 7 alfa hidroxilasa
b. el pool de ácidos biliares se incrementa
c. hay mala absorción de ácidos biliares
d. el páncreas no secreta lipasa

10. En un experimento se inserta un balón en el estómago de un voluntario, se infla poco a poco


mientras que se vigilan las presiones intraluminales. Aunque el volumen del balón aumenta
considerablemente, las presiones permanecen constantes. Esta relación volumen-presión se
explica por la liberación local de …………..
(Unidad 2, sesión 13, logro 1 Explica la Motilidad gástrica: relajación receptiva)
a. acetil colina y gastrina
b. norepinefrina y óxido nítrico
c. colecistoquinina y óxido nítrico
d. óxido nítrico y péptido inhibidor vasoactivo

11. ¿Cuál de las siguientes alternativas es una característica de la secreción exocrina del páncreas?
(Unidad 3, sesión 23, logro 5: Secreción pancreática: formación del jugo pancreático, influencia de la
velocidad de flujo y regulación)
a. Es hipotónica respecto al plasma
b. Su mayor estímulo se da en la fase intestinal
c. Es estimulada por la presencia de bicarbonato en el duodeno
d. La secreción enzimática es estimulada principalmente por la secretina
12. Las estructuras en el hígado que permite que los productos metabólicos unidos a proteínas
tengan acceso a las membranas basolaterales de los hepatocitos, son…..
(Unidad 3, sesión 21, logro 4-5: Explica la Organización micro estructural del hígado)
a. los canalículos
b. las células de Ito
c. las fenestras sinusoidales
d. las uniones intercelulares herméticas

13. La composición de la bilis es modificada conforme fluye por los conductillos biliares.
Durante este tránsito se espera que aumente la concentración de…….
(Unidad 3, sesión 22, logro 2: Describe la Secreción biliar, visión general del sistema biliar
extrahepático y composición de la bilis)
a. Ig A
b. Glucosa
c. Protones
d. Vitamina A

A
14. Se mide experimentalmente el contenido gástrico de dos personas. La persona “A” tiene alto
contenido de grasa y la persona “B” tiene un contenido isotónico ¿Cuál de las siguientes es
correcto respecto al vaciamiento gástrico? (Unidad 2, sesión 13, logro 2: Describe la Motilidad y

EK
vaciamiento gástrico)
a. Hay ralentización del vaciado gástrico solo en “A”
b. El vaciamiento gástrico es más rápido en ambos
c. Hay ralentización del vaciado gástrico solo en “B”
d. Hay ralentización del vaciado gástrico en ambos casos
OT
15. El examen endoscópico de un paciente con hipertensión portal grave revela venas tortuosas que
sobresalen hacia la luz del esófago. El paciente recibe tratamiento quirúrgico mediante la
colocación de una derivación que conecta la vena porta a la vena cava. Después de la operación
el riesgo de encefalopatíay el riesgo
de sangrado de várices ……………..
NC

(Unidad 3, sesión 22, logro 5: Describe la Insuficiencia hepática, encefalopatía hepática e hipertensión
portal)
a. disminuirá / disminuirá
b. disminuirá / aumentará
c. aumentará / disminuirá
BA

d. aumentará / aumentará

16. Un paciente varón de 18 años de edad acude al médico para sus exámenes de rutina. Sus
resultados de laboratorio muestran un valor de bilirrubina sérica de 4 mg/dl y una bilirrubina
directa de 0,3 mg/dl. Las pruebas de función hepática son normales. La alteración que explica
mejor este caso es por la deficiencia de ………………..
(Unidad 3, sesión 22, logro 3: Explica la Producción y excreción de bilirrubina. Tipos de bilirrubina e
ictericia)
a. transaminasas
b. hemo oxigenasa
c. la 7 alfa hidroxilasa
d. glucuronil transferasa

17. Un hombre de 57 años de edad es llevado a urgencias con hematemesis masiva rojo brillante, a
su llegada se halla inconsciente con PA: 80/40 mm Hg y FC: 124 lat/min. Luce ictérico con
presencia de “arañas vasculares en el tórax anterior y extremidades”, abdomen distendido con
signo de oleada positiva. Se encuentra esplenomegalia y pérdida de la masa muscular en
extremidades. La anastomosis vascular responsable del sangrado en este paciente es ………….…..
(Unidad 3, sesión 21, logro 2: Describe las anastomosis porto sistémicas)
a. vena gástrica izquierda y vena ácigos
b. arteria gástrica izquierda y vena ácigos
c. vena paraumbilical y vena epigástrica inferior
d. vena gástrica izquierda y vena esofágica superior
18. Un estudiante de medicina está comiendo un plato de comida a base de champiñones,
espárrago y salsa de soya. El estímulo del sabor umami contenido en todos estos alimentos viaja
a través del nervio………………..
(Unidad 2, sesión 10, logro 3: Describe la irrigación e inervación de la lengua)
a. Lingual
b. Hipogloso
c. Glosofaringeo
d. Cuerda del tímpano

A
EK
OT
NC
BA
19. Una paciente de 30 años de edad es sometida a una cirugía en oído medio derecho por un
problema de otoesclerosis. Luego de la cirugía refiere alteración sensitiva de la lengua. Al
evaluar el caso usted esperaría encontrar……….
(Unidad 2, sesión 10, logro 5: Describe la Irrigación e inervación de la lengua)
a. Alteración en la sensación del dolor y temperatura en el tercio posterior de la lengua
b. Alteración en la sensación del dolor en los dos tercios anteriores de la lengua
c. Alteración en la sensación del gusto en el tercio posterior de la lengua
d. Sensación del dolor, tacto y temperatura conservadas

20. En un paciente de 45 años de edad con colestasis biliar, se encuentra una elevación de los niveles
sanguíneos de fosfatasa alcalina hasta 3 veces la cifra normal. ¿Cuál de las siguientes alternativas
estará también elevada como evidencia del daño de la vía biliar?
(Unidad 3, sesión 22, logro 5: Pruebas de función hepática, Insuficiencia hepática,
encefalopatía hepática e hipertensión portal)
a. Tiempo de protrombina y albúmina sérica
b. Transaminasas hepáticas (ALT y AST)

A
c. Gamma glutamil transpeptidasa
d. Glucoronil transferasa

EK
21. Experimentalmente se incrementa la velocidad de la secreción salival con una sustancia,
en el análisis de la composición de esta saliva obtenida se espera encontrar…………..
(Unidad 3, sesión 17, logro 5 : Explica la Influencia de la velocidad del flujo salival en la composición de la
saliva)
a. disminución de la concentración de bicarbonato que supera la concentración plasmática
OT
b. aumento de la concentración de cloro y sodio que supera la concentración plasmática
c. aumento de la concentración de bicarbonato que supera la concentración plasmática
d. disminución de concentración de potasio y bicarbonato

22. Lactante de 3 meses de vida es atendido por presentar diarrea, se administra una
solución de glucosa y electrólitos por vía oral. La proteína de membrana apical que
NC

explica la capacidad de esta solución para proporcionar aporte de glucosa e hidratación


es ………..
(Unidad 4, sesión 26, logro 1: Explica la Digestión y Absorción de los hidratos de carbono. Alteraciones)
a. CFTR
b. SGLT-1
BA

c. GLUT-2
d. GLUT-5

23. Paciente ha sufrido herida de bala en el abdomen, se le ha tenido que extirpar el segmento
medio y distal del ileon. En este caso la síntesis hepática de sales biliares estará …..…..
(Unidad 3, sesión 22, logro 4: Explica la formación, función y Circulación entero hepática de lasa sales
biliares)
a. Sin cambios en el ritmo de síntesis
b. Disminuida por inhibición de la enzima colesterol 7 alfa hidroxilasa
c. Incrementada por estímulo de la enzima colesterol 7 alfa hidroxilasa
d. Incrementada por inhibición de la enzima colesterol 7 alfa hidroxilasa

24. Un varón de 75 años ingresa al consultorio por presentar ictericia marcada de piel y las
escleras. El estudio del paciente mostró que presentaba un tumor que obstruía la totalidad del
conducto hepático común. ¿Cuál de los siguientes conductos se encontrará dilatado en este
paciente?
(Unidad 3, sesión 21, logro 6: Describir el árbol biliar intrahepático)
a. de Wirsung
b. de Hering
c. colédoco
d. cístico
25. Correlaciones las dos columnas y marque la fórmula correcta:
(Unidad 4, sesión 28, logro 1: Diarrea: definición, mecanismos: osmótica, secretoria y exudativa)

1. Enfermedad Hirschsprung( ) heces con moco y sangre


2. Diarrea osmótica( ) intolerancia a lactosa
3. Diarrea secretoria( ) aganglionosis congénita
4. Diarrea exudativa( ) canales de Cl- en las células de la cripta a.- 4231b.-

1234c.- 2143d.- 4213

26. La vena umbilical obliterada del hígado después del nacimiento se transforma en el ligamento:
(Unidad 3, sesión 21, logro 1: Hígado: relación con la pared abdominal, caras, lóbulos, ligamentos , hilio
hepático)
a. cruzado
b. redondo
c. coronario
d. falciforme

A
27. Llega a su guardia nocturna una madre que trae a su RN masculino de 2 semanas de vida con mal
estado general y sequedad de mucosas. Usted observa que lacta ávidamente, pero a las 2 horas

EK
presenta vómito postprandial no bilioso en proyectil. Al realizar la historia clínica, descubre que el
lactante recibió profilaxis con macrólidos para tos ferina. Usted sospecha principalmente en:
(Unidad 1, sesión 4, logro 4: Desarrollo y anomalías del intestino anterior)
a. estenosis pilórica hipertrófica congénita
b. fistula traqueo esofágica
OT
c. estenosis duodenal
d. atresia duodenal

28. En la regulación del apetito y la saciedad, la estimulación experimental crónica del núcleo
ventro medial del hipotálamo producirá:
(Unidad 1, sesión 3, logro 4: explica los mecanismos de control del apetito y saciedad)
NC

a. afagia
b. obesidad
c. hiperfagia
d. activación de neuronas relacionadas a NPY

29. Paciente mujer de 25 años acude por dolor en fosa ilíaca derecha que empeora al toser o
BA

caminar, asociada a náuseas y vómitos por lo cual acude a emergencia. Dos días después de
realizarle una apendicectomía, la paciente desarrolla fiebre alta (39 °C), está hipotensa y presenta
dolor abdominal. La laparotomía exploratoria muestra un gran volumen de sangre en la cavidad
peritoneal por lesión de un vaso producida durante la apendicectomía.
¿Cuál de las siguientes arterias debe ligarse para detener la hemorragia?
(Unidad 4, sesión 27, logro 4: Irrigación arterial del colon, recto y conducto anal)
a. cólica derecha y arteria rectal superior.
b. ileocólica y arteria cólica media.
c. mesentérica superior.
d. ileocólica.

30. La onda peristáltica secundaria del esófago se caracteriza por ser originada ………
(unidad 2, sesión 12, logro 3: Motilidad esofágica: fases y características)
a. por el plexo de meissner del esófago
b. por el plexo mientérico del esófago
c. por el reflejo de la deglución
d. durante la masticación

31. ¿Cuál de los siguientes es una causa de ictericia con bilirrubina conjugada aumentada?
(Unidad 3, sesión 22, logro 3: Producción y excreción de bilirrubina. Tipos de bilirrubina, ictericia)
a. Ictericia del recién nacido
b. Obstrucción del colédoco
c. Anemia hemolítica
d. Gran hematoma
32. En relación a la absorción de nutrientes, la absorción de dipéptidos y tripéptidos a nivel de las
células epiteliales del intestino delgado, se da principalmente debido a:
(Unidad 4, sesión 26, logro 2: Digestión y absorción de las proteínas. Alteraciones)
a. el incremento de los canales de Cl- en la membrana apical
b. la gradiente de bicarbonato en la membrana basal
c. la gradiente de iones H+ en la membrana apical
d. la gradiente de Na+ en la membrana apical

33. Paciente de 20 años es traído a la emergencia por presentar diarreas desde hace 2 días. Familiar
refiere que las deposiciones son líquidas y abundantes, al examen luce deshidratado y se plantea
que la diarrea es producida por una toxina que estimula la transformación de ATP a AMPc con
apertura de canales de Cl- y pérdida de agua. El tipo de diarrea más probable es:
(Unidad 4, sesión 28, logro 1: Diarrea: definición , mecanismos: osmótica, secretoria y supurativa)
a. osmótica
b. exudativa
c. secretoria
d. por intolerancia a lactosa

A
34. Un niño fue operado por una obstrucción intestinal, observándose la presencia de divertículo de
Meckel. Según lo referido, marque lo correcto:

EK
(Unidad 1, sesión 5, logro 2: identifica las anomalías del desarrollo del intestino medio: onfalocele y
gastrosquisis (diferencias), Divertículo de Meckel)
a. el 50% de la población lo presenta
b. se localiza en el íleon muy cerca al yeyuno
c. puede poseer tejido gástrico o pancreático
OT
d. se produce por una mala rotación de los intestinos

35. Marque la alternativa


correcta respecto a la
estructura marcada en el
gráfico:
NC

(Unidad 3, sesión 22, logro 2:


Secreción biliar. Visión general
del sistema biliar extrahepático
y composición de la bilis)

a. Se halla a 2
BA

centímetros debajo
de la papila
duodenal mayor
b. Llega el conducto
colédoco y pancreático
principal
c. Llega el conducto
hepático común y
pancreático
principal
d. Llega el conducto pancreático accesorio

36. ¿Cuál de las siguientes moléculas se encontrará aumentada en el citoplasma de las células
parietales de un paciente con sindrome de Zollinguer Ellison?
(Unidad 3, sesión 20, logro 4: Enfermedad ulcerosa péptica: úlcera gástrica, duodenal.
síndrome de Zollinger – Ellison)
a. Péptido liberador de gastrina (GRP)
b. Proteína G estimulante (GS)
c. Inositol Trifosfato (IP3)
d. AMP cíclico (AMPc)

37. Los fármacos inhibidores de la bomba de protones, actúan bloqueando la ………..……..


(Unidad 3, sesión 20, logro 3: Regulación de la secreción gástrica: inhibición, Secreción de
pepsinógeno y factor intrínseco)
a. anhidrasa carbónica
b. ATPasa H+/K+ en la membrana luminal
c. ATPasa H+/K+ en la membrana basolateral
d. ATPasa Na+/K+ en la membrana basolateral

A
EK
OT
NC
BA
38. Un paciente fue diagnosticado de gastritis autoinmune, ¿cuál de las siguientes alternativas es
FALSA respecto a esta enfermedad?
(Unidad 3, sesión 20, logro 5: Gastritis crónica: helicobacter pylori, autoinmune. Tipos de gastritis)
a. Afecta principalmente el fondo y cuerpo gástrico
b. Se produce hiperplasia de células G secundaria a la aclorhidria
c. El propio sistema inmune destruye principalmente las células parietales
d. Se produce atrofia de la mucosa, aclorhidria, hipergastrinemia y déficit de vitamina B6

39. Marque la correlación correcta:


(Unidad 3, sesión:18, logros:1 y 2: Describe las enfermedades inflamatorias/infecciosas y
proliferativas de la cavidad oral)

1. Herpes virus( ) En relación al abuso de antibióticos


2. Candidiasis oral( ) Lesiones vesiculares como racimo de uvas
3. Eritroplaquia( ) Mega esófago
4. Enfermedad de Chagas( ) Lesión pre cancerígena

A
a.- 2431b.- 1234c.- 4123d.- 2143

EK
40. En un paciente con insuficiencia renal crónica, el déficit en la absorción de calcio a nivel del
enterocito se debe a lo siguiente:
(Unidad 4, sesión 26, logro 6: Explica la Absorción de calcio y hierro)
a. No se convierte la 25 hidroxicolecalciferol a 1,25 dihidroxicolecalciferol
b. No se convierte la 1,25 dihidroxicolecalciferol a 25 hidroxicolecalciferol
OT
c. Existe un descenso de la alfa 25 hidroxilasa renal
d. Se incrementa la producción de Calbindina
NC
BA
SISTEMA DIGESTIVO
(ME154) EXAMEN
PARCIAL 2019 01

Profesores : Alfaro Salazar, Herberth Romulo; Callata Caceres, Gunter; Cayo Quiñe, Alexandra Mariel; Correa
Borit, Jorge Mauricio; Cruz Cutty, Lourdes Marylin; Guzmán Calderón, Gerly Edson; Jáuregui
Farfán, Jorge Jesús; Mayor Zevallos, Otto Alberto; Montoya Suárez, José Luis; Palacios Bazan,
Enrique Elias; Robles Pino, Alexander Anibal; Wong Bravo, Juan Carlos
Sección : Todas las secciones
Duración : 50 minutos.
Indicaciones:
Lea atentamente cada pregunta antes de responder:
- Se prohíbe el uso del celular y cualquier dispositivo electrónico.

A
- Está prohibido intercambiar materiales.
- Coloque su código de alumno en la tarjeta de respuestas. Si su código contiene una letra reemplácela por
un valor numérico siguiendo la siguiente equivalencia: A=9, B=8, C=7, D=6, E=5, F=4, G=3.

EK
- Traslade sus respuestas a la tarjeta, llenando los círculos de manera completa con lapicero negro o azul.
Está prohibido el llenado con lápiz, lapicero de otro color o con lapicero de tinta borrable.
- Sea cuidadoso en el llenado de la tarjeta de respuestas, pues solo esta tiene validez para la calificación.
- Al terminar su examen avise al docente a cargo, no se levante de su sitio; debe entregar la hoja de
respuestas con la carátula del examen, este cuadernillo de preguntas se lo llevará cada estudiante.
OT
1. ¿Cuál de las glándulas salivales es responsable del mayor porcentaje del volumen de la saliva en
condiciones basales?
a. Parótida
b. Subpalatin
as c.
NC

Sublingula
es d.
Submaxilar
es

2. La lengua está recubierta por epitelio:


BA

a. pseudoestratificado columnar no queratinizado


b. plano estratificado no queratinizado
c. pseudoestratificado columnar ciliado
d. plano estratificado queratinizado

3. El esfínter anal interno tiene musculatura …………… y tiene control ………………….


a. lisa / voluntario
b. lisa / involuntario
c. esquelética / simpático
d. esquelética / parasimpático

4. La arteria aorta proporciona la irrigación al tubo digestivo ¿cuál de las siguientes arterias proporciona la
irrigación al ángulo cólico derecho?
a. Mesentérica superior
b. Mesentérica inferior
c. Frénica inferior
d. Tronco celiaco

5. Paciente de 26 años que le cuenta en su historia clínica que cada vez que almuerza, a los 20 minutos
tiene deseo de defecar. Le comenta que su hijo de 1 mes le pasa lo mismo pero más intenso. Esto se
explica por el reflejo
…………………, el cual estáen el paciente.
a. colicoileal / normal
b. colicoileal / alterado
c. gastrocolico / normal
d. gastrocolico / alterado
6. La región del estómago que se comunica con el duodeno se denomina:
a. pilórica
b. cardias
c. cuerpo
d. fórnix

7. Acude a consulta un paciente que fue diagnosticado de ulcera péptica 3 días antes. Luego de múltiples
pruebas diagnósticas, se concluye que el paciente presenta un tumor secretor de gastrina ¿Cuál de las
siguientes situaciones estará incrementada?
a. Distención gástrica
b. Inhibición del vaciado gástrico
c. Secreción de ácido clorhídrico (HCl)
d. Inhibición de la secreción de pepsinógeno

8. En el sistema digestivo, el control del apetito esta dado por un complejo sistema de sustancias y órganos

A
integradores los cuales regulan la ingesta de alimentos. Laes una sustancia oroxígena y es sintetizada
por el ……………………..
a. leptina / intestino

EK
b. grelina / intestino
c. leptina / estómago
d. grelina / estómago

9. Sobre el control autónomo del sistema digestivo, marque la alternativa correcta:


a. La inervación dada por el sistema simpático es de tipo preganglionar.
OT
b. El sistema parasimpático usa como neurotransmisores a la acetilcolina y la noradrenalina.
c. El nervio vago (par craneal X) le da inervación simpática a la mayoría del sistema digestivo.
d. En el sistema simpático, los nervios responsables hacen una primera sinapsis en ganglios
próximos al órgano a inervar.
e. En la inervación de tipo parasimpático, solo interviene el plexo submucoso, sin embargo, en la
de tipo simpático intervienen tanto el submucoso como el mientérico.
NC

10. Con respecto a la actividad eléctrica del sistema digestivo, marque la alternativa correcta
a. Corresponden a potenciales de acción que están presentes de forma continua y le dan la
capacidad de perístasis autónoma al sistema digestivo.
b. La frecuencia de las ondas lentas no se ve influenciada por la actividad neural ni las
BA

hormas gastrointestinales.
c. En el estómago las ondas lentas se dan en una frecuencia de 6 por minuto.
d. Las ondas lentas son cambios lentos y ondulantes del potencial en reposo.
e. La frecuencia de las ondas lentas va de 6 a 12 ondas por minuto.

11. Ante una lesión del IX par craneal, el músculo…se altera en su función.
a. palatogloso
b. estilofaríngeo
c. palatofaríngeo
d. constrictor superior

12. Un varón de 50 años es sometido a extirpación de duodeno y parte proximal de yeyuno. La pérdida de
estímulo hormonal en el páncreas para la secreción enzimática se explica por la pérdida de las células
……………………
a. Parietales, productoras de factor intrínseco
b. “K” productoras de factor intrínseco
c. “M” productoras de CCK
d. “I” productoras de CCK
13. Respecto al mecanismo de la defecación ¿Cuál de las siguientes afirmaciones es correcta?
a. Se produce contracción refleja del esfínter anal interno
b. Se produce contracción o relajación del esfínter anal externo por señales de la corteza cerebral
c. La presencia de materia fecal en el recto estimula la contracción del sigmoides por los nervios
pélvicos simpáticos
d. En la posición de “cuclillas” el músculo puborectal se halla contraído favoreciendo la
evacuación de la materia fecal

14. Un niño de tres años llega a emergencia con disfagia (dificultad para tragar), salivación y llanto. Se sospecha
de ingesta de cuerpo extraño: moneda en el esófago; al ser evaluado se constata en una radiografía presencia
de cuerpo extraño a nivel de C6 y C7 (6° y 7° vértebra cervical). El cuerpo extraño estará suspendido a nivel
del estrechamiento producido por el ………..
a. cayado aórtico
b. hiato esofágico
c. músculo cricofaríngeo
d. bronquio principal izquierdo

A
15. En el caso de un paciente con un tumor productor de gastrina, la presencia de úlceras duodenales y

EK
erosión de la mucosa gástrica se debe principalmente a…….
a. la acción paracrina de la gastrina sobre la célula parietal
b. el exceso de HCl por estímulo de receptores CCK-B en la célula parietal
c. la sobre expresión de los receptores “G” para gastrina en la célula parietal
d. el exceso de HCl por estímulo directo de receptores “H” en la célula parietal
OT
16. La onda peristáltica secundaria del esófago se caracteriza por ser originada ………
a. por el plexo de submucoso del esófago
b. por el plexo mientérico del esófago
c. por el reflejo de la deglución
d. durante la masticación
NC

17. Marque lo correcto sobre las ondas lentas en el tubo digestivo


a. No son despolarizaciones
b. Son potenciales de acción subumbrales
c. Se constituyen de despolarizaciones y repolarizaciones
d. Son rítmicas y generadas por el sistema nerviosos autónomo
BA

18. Recién nacido que presenta protrusión de contenidos abdominales los cuales no están cubiertos por
peritoneo y salen de la cavidad abdominal a través de un defecto de la pared. ¿Cómo se denomina a la
afección que presenta este paciente?
a. Onfalocele
b. Atresia biliar
c. Gastrosquisis
d. Divertículo de Meckel

19. Experimentalmente se utiliza atropina (anticolinérgico) para inhibir la secreción de gastrina, sin
embargo, la secreción de esta hormona se sigue dando ante estímulos vagales. Esta situación se explica
porque la atropina:
a. no bloquea la acción del péptido GRP
b. solo inhibe la acción del péptido GRP en la célula G
c. inhibe la acción de acetilcolina e histamina en la célula G
d. bloquea parcialmente la bomba de protones en la célula G

20. Niña de 4 días es llevada a la emergencia pediátrica por presentar llanto constante, la madre refiere
coloración azulada de labios al momento de lactar, acompañado de tos persistente y dificultad respiratoria
así como distención abdominal. Se le coloca sonda nasogástrica para alimentación notando que retorna a
la cavidad oral en todos los intentos. ¿Cuál es la anomalía del desarrollo en este caso?
a. Solo fístula traqueo esofágica
b. Fístula traqueo esofágica proximal y distal
c. Atresia esofágica proximal con fístula traqueo esofágica distal
d. Atresia esofágica distal con fístula traqueo esofágica proximal
21. Paciente varón de 36 años es traído a la emergencia luego de sufrir un accidente de tránsito, presenta
traumatismos múltiples en cabeza y tronco. Al examen físico se evidencia hematoma en hemicara izquierda,
ligera protrusión y caída del lado izquierdo del maxilar inferior, por lo que se realiza una tomografía donde
se halla una fractura de la apófisis coronoides del maxilar inferior. ¿Qué músculo está relacionada
directamente con esta situación?
a. Masetero
b. Temporal
c. Buccinador
d. Pterigoideo medial

22. Un paciente refiere no percibir algunos sabores. Al examen físico constata alteración del sabor dulce y
umami.
¿Cuál de los siguientes nervios estará alterada su función?
a. Cuerda del tímpano (VII par)

A
b. Lingual (rama del V par)
c. Glosofaríngeo (IX par)
d. Hipogloso (XII par)

EK
23. A los pocos días de nacido, regresa a neonatología un niño con problemas de motilidad del colon; los
estudios determinan ausencia congénita de células ganglionares. Según el gráfico ¿cuál es la capa en la que
se determina la ausencia de dichas células?
a. Mucosa - 2
b. Muscular propia - 2
OT
c. Muscular propia - 3
d. Muscular de la mucosa - 3
NC

1 2
BA

24. Con respecto al control autonómico en el tracto gastrointestinal y en relación a su fisiología. ¿Cuál es la
función del sistema nervioso parasimpático en el tracto gastrointestinal?
a. Inhiben la contracción muscular y estimulan la secreción de sustancias a nivel de la submucosa
b. Estimulan la contracción muscular y estimulan la secreción de sustancias a nivel de la mucosa
c. Inhiben la contracción muscular e inhiben la secreción de sustancias a nivel de la submucosa
d. Estimulan la contracción muscular e inhiben la secreción de sustancias a nivel de la mucosa

25. Un estudiante que está preocupado por su examen parcial, no ha desayunado ni almorzado; cuando al fin
ingiere alimentos, esto le provoca el aumento de los movimientos musculares del tracto gastrointestinal y
la sensación de defecar. ¿Qué reflejo se ha activado?
a. Entero-gástrico
b. Gastro-cólico
c. Cólico-ileal
d. Ileo-ileal
26. ¿De qué par craneal es rama el nervio palatino mayor?
a. Vago
b. Hipogloso
c. Trigémino
d. Palatogloso

27. ¿En cuál de las fases de la deglución la epiglotis separa la vía respiratoria de la digestiva?
a. oral
b. laríngea
c. faríngea
d. esofágica

28. Los péptidos intestinales se pueden clasificar como sustancias endocrinas, neurocrinas y paracrinas,
dentro de las paracrinas se encuentran la somastotatina e histamina. Marque la respuesta correcta
a. La somastotatina es sintetizada por las células B de la mucosa gástrica

A
b. La histamina actúa estimulando su receptor tipo H1 en la mucosa gástrica
c. La histamina es sintetizada por células de tipo paracrino de las glándulas gástricas
d. La somatostatina presenta dentro de sus funciones la estimulación de la secreción de H +

EK
29. En relación a los órganos intraabdominales y sus estructuras de fijación, elija la alternativa
correcta a.El mesenterio permite la suspensión e irrigación de los órganos
retroperitoneales
b. Tanto el hígado como la vesícula biliar se encuentran ubicados a nivel del flanco derecho
c. El colon, el duodeno y el resto de intestino delgado son órganos considerados
OT
netamente como peritoneales
d. Los ligamentos que encontramos dentro de la cavidad abdominal son el esplenorenal y el
gastrofrénico e.Los omentos van desde el estómago y la segunda porción del duodeno a otras
estructuras
intraabdominales y existen dos: el omento mayor y el omento menor
NC

30. Paciente varón de 27 años es llevado por bomberos a emergencia luego de ser asaltado y, tras resistirse, es
cortado con el pico de una botella a nivel abdominal. Al examen físico usted observa que a través de la herida
se puede observar la protrusión de asas intestinales. En relación con las capas de la pared abdominal, marque
la alternativa correcta.
a. La fascia de Scarpa está constituida principalmente por tejido adiposo
BA

b. La pared abdominal está formada por piel, huesos, músculos, fascias y peritoneo parietal
c. La fascia de Camper es una estructura fibrosa que carece de grasa y su grosor es constante en
toda la pared abdominal
d. El músculo oblicuo externo discurre en dirección súpero-interna y se inserta en el borde
inferior de las ultimas 3 a 4 costillas
e. El músculo recto del abdomen tiene como funciones comprimir el contenido del abdomen,
tensar la pared del abdomen y flexionar la columna
SISTEMA DIGESTIVO
(ME154) EXAMEN
FINAL 2019 01

Profesores : Alfaro Salazar, Herberth Romulo; Callata Caceres, Gunter; Cayo Quiñe, Alexandra Mariel; Correa
Borit, Jorge Mauricio; Cruz Cutty, Lourdes Marylin; Guzmán Calderón, Gerly Edson; Jáuregui
Farfán, Jorge Jesús; Mayor Zevallos, Otto Alberto; Montoya Suárez, José Luis; Palacios Bazan,
Enrique Elias; Robles Pino, Alexander Anibal; Wong Bravo, Juan Carlos
Sección : Todas las secciones
Duración : 70 minutos.
Indicaciones:
Lea atentamente cada pregunta antes de responder:
- Se prohíbe el uso del celular y cualquier dispositivo electrónico.
- Está prohibido intercambiar materiales.

A
- Coloque su código de alumno en la tarjeta de respuestas. Si su código contiene una letra reemplácela por
un valor numérico siguiendo la siguiente equivalencia: A=9, B=8, C=7, D=6, E=5, F=4, G=3.
- Traslade sus respuestas a la tarjeta, llenando los círculos de manera completa con lapicero negro o azul.

EK
Está prohibido el llenado con lápiz, lapicero de otro color o con lapicero de tinta borrable.
- Sea cuidadoso en el llenado de la tarjeta de respuestas, pues solo esta tiene validez para la calificación.
- Al terminar su examen avise al docente a cargo, no se levante de su sitio; debe entregar la hoja de
respuestas con la carátula del examen, este cuadernillo de preguntas se lo llevará cada estudiante.
OT
1. La explicación fisiológica de presentar somnolencia de 30 minutos a 1 hora después de ingerir
alimentos, se explica por:
a. Aumento del cloro intraluminal
b. Aumento del bicarbonato intraluminal
c. Disminución de ácido carbónico en la célula parietal
NC

d. Disminución de la actividad de la anhidrasa carbónica


e. Aumento de la alcalinidad sanguínea

2. Con respecto a la irrigación arterial del colon, a que arteria corresponde la


señalada con la flecha
a. Cólica derecha
BA

b. Cólica media
c. Cólica izquierda
d. Ileobisecoapendículocólica
e. Arco de Riolano

3. Si un paciente presentara dentro del punto de vista fisiológico, una


disminución de enterocinasa, entonces esto originaría una disminución de
la actividad de:
a. la
pepsina
b.la
lipasa
c. la quimotripsina
d. el peptido insulinotropo dependiente de glucosa
e. la amilasa

Se valida la opción b debido a su relación con la colipasa.


4. Con respecto a la anatomía del hígado, señale a que estructura
pertenece la marcada por el número 1.
a. Ligamento falciforme
b. Línea de Cantlie
c. Ligamento triangular
d. Ligamento coronario
e. Ligamento teres

Se valida la opción a debido a la ubicación del número 1 en donde se


unen el ligamento falciforme y ligamento coronario.

5. Se presenta un paciente, el cual presenta un antecedente de tuberculosis intestinal, por lo cual, se le


resecó 80 cm de íleon distal. Desde el punto de vista fisiológico, el paciente puede presentar una de las
siguientes alteraciones:
a. Disminución de la secreción de Vitamina B12

A
b. Aumento indiscriminado de absorción de ácido fólico
c. Disminución de la absorción de hierro

EK
d. Aumento de la secreción de bicarbonato
e. Disminución de la absorción de ácido glicocólico

6. Un paciente es sometido experimentalmente a un fármaco que modifica el flujo salival, obteniéndose un


volumen de saliva de 288 ml en 6 horas. En este caso las concentraciones de electrolitos y bicarbonato en la
saliva obtenida varían de la siguiente manera:
OT
a.↑ Na+, ↓ K+, ↑ Cl-, ↑
HCO3- b.↓ Na+, ↓ Cl-, ↑
K+, ↓ HCO3-
c.↑ Na+, ↑ Cl-, ↓ K+, ↓ HCO3-
d.↑ Na+, ↑ Cl-, ↑ K+, ↑ HCO3-
e.↓ Na+, ↓ Cl-, ↓ K+, ↓ HCO3-
NC

Se valida la opción a debido a que se puede considerar como un aumento del flujo de saliva.

7. La siguiente imagen histológica corresponde a la glándula


…………… y la estructura señalada produce ………
BA

a. salival sublingual / mucopolisacáridos


b. oxíntica / pepsinógeno
c. salival submaxilar / ptialina
d. salival parótida / amilasas
e. antrales / gastrina

8. Paciente varón de 65 años con antecedente de hipercolesterolemia, hipertensión arterial, fibrilación


auricular y dos infartos al miocardio previos, aqueja de dolor abdominal intenso de inicio súbito,
distensión abdominal, se decide cirugía con resección de 1,5 metros de intestino delgado terminal y colon
ascendente. Como consecuencia de la resección el paciente tendrá deficiencia de:
a. Vitamina C
b. Tiamina
c. Vitamina A
d. Vitamina B1
e. Vitamina B6

Se valida esta opción debido a que su absorción está relacionada al íleon.


9. Uno de los siguientes elementos debería hallarse con más probabilidad en el esófago de un paciente que
sufre de
reflujo gastro esofágico…
a. Pepsina
b. Tripsina
c. Quimiotripsina
d. Carboxipeptidasa
e. Ácidos biliares

10. Un paciente de 40 años cursa con anemia de 8g/dl, aqueja además de astenia y sensación de hormigueo
bilateral en los miembros inferiores, al examen se halla alteración de la sensibilidad a la vibración y camina
con ampliación de la base de sustentación. Uno de los siguientes procedimientos sería de ayuda para el
diagnóstico de este paciente:
a. Tomografía cerebral
b. Biopsia de la mucosa gástrica
c. Biopsia de hígado

A
d. Examen de sangre oculta en heces
e. Biopsia de Ileon proximal

EK
11. Paciente de 60 años ingresa por caída hace 1 hora y pequeño hematoma en cuero cabelludo, al examen
físico ampliado se observa ictericia de piel y mucosas generalizada, abdomen blando, se palpa estructura
quística no dolorosa en hipocondrio derecho que corresponde a vesícula biliar (signo de Courvoisier), en
los exámenes de laboratorio se halla niveles bajos en la formación de estercobilinógeno y urobilinógeno
en heces, incremento de la bilirrubina conjugada en la orina, elevación de fosfatasa alcalina y gamma
OT
glutamil transpeptidasa séricas. El presente cuadro puede ser explicado por:
a. Reabsorción de hematoma
b. Litiasis vesicular
c. Carcinoma de la cabeza de páncreas
d. Carcinoma con estenosis del conducto hepático común
e. Anemia hemolítica
NC

12. Paciente varón de 58 años con antecedente de alcoholismo crónico es diagnosticado y recibe tratamiento
por cirrosis hepática. Hace 2 días refiere familiar que tuvo cambio de conducta y no reconoce a algunos
familiares. Al examen físico, se halla ascitis, circulación colateral en abdomen, telangiectasias, en el
examen de sistema nervioso: rigidez de extremidades, ROT incrementados, desorientación en el espacio y
BA

asterixis. ¿cuál de las siguientes circunstancias, explicaría el cuadro en este paciente?


a. Uso de diuréticos ahorradores de potasio
b. Incremento de actividad de ureasa bacteriana duodenal
c. Hemorragia gastrointestinal
d. Disminución de la producción de NH3+ en el colon
e. Dieta normo proteica

Se valida la opción e debido al efecto sobre la encefalopatía.


Con respecto de la opción b es incorrecta debido a que hace referencia al duodeno, debería indicar colon.

13. Un recién nacido presenta vómitos biliosos poco tiempo después de cada alimento. Al preguntar a la
madre sobre antecedentes, ella recuerda que tuvo polihidramnios durante la gestación, pero un análisis de
cariotipo fue normal. Una de las siguientes es la causa más probable de estos hallazgos en el recién nacido:
a. Enfermedad de Hirschprung
b. Fístula tráqueo esofágica
c. Divertículo ileal
d. Estenosis pilórica
e. Malrotación de la yema pancreática ventral

14. Un lobulillo hepático se puede dividir en tres zonas como se muestra en el gráfico. ¿Cuál de las
siguientes afirmaciones sobre las tres zonas es verdadera?
a. La zona 1 tiene los menores depósitos de glucógeno
b. La zona 3 es la primera en afectarse en una colestasis extra hepática
c. La zona 2 es más susceptible a la injuria por isquemia que la zona
periportal d.La zona 2 tiene la mayor capacidad de regeneración
e.La zona 1 es la que tiene menos actividad metabólica.

A
La pregunta 14 ha sido anulada, sin embargo, ningún estudiante se verá afectado
negativamente en su puntaje debido a esta anulación.

EK
15. En un estudio de la secreción de hormonas gastrointestinales, sus concentraciones en la vena porta se
midieron durante perfusión luminal del intestino delgado con soluciones de diversas magnitudes de pH.
¿Qué hormona aumentará en el plasma de la vena porta durante perfusión a través del intestino con
una solución de pH 3?
a. CCK
OT
b. gastrina
c. GIP
d. motilina
e. secretina

16. Paciente de 30 años que ingresa a causa de un traumatismo abdominal cerrado. En la exploración se
NC

aprecia discreta palidez de piel y mucosas, auscultación pulmonar normal, taquicardia de 120 /min.
Discreta distensión abdominal y matidez en flancos; el hematocrito, que era prácticamente normal al
ingreso, disminuye a 30% a las tres horas. En la Rx de tórax se objetiva fractura de las costillas 10-11
izquierdas. La causa más probable de la anemización en este paciente es:
a. traumatismo renal con hemorragia retroperitoneal.
BA

b. rotura de hígado con hemoperitoneo.


c. rotura de bazo con hemoperitoneo.
d. rotura de mesos con hemoperitoneo.
e. traumatismo pancreático con pancreatitis traumática.

17. Mujer de 65 años. Consulta por síndrome constitucional


asociado a dolor abdominal epigástrico progresivo irradiado a
espalda, de dos meses de evolución. El diagnostico de sospecha
de adenocarcinoma de páncreas se confirma por biopsia. Se
realiza examen de imagen de abdomen para evaluación de
estructuras vasculares próximas al tumor pancreático. ¿Cuál es
el nombre de la vena señalada que está ausente, trombosada
por infiltración tumoral, condicionando circulación colateral en
la pared gástrica?

a. Mesentérica superior
b. Coronaria estomaquica
c. Esplénica
d. Porta
e. Renal izquierda
18. Revisando la angiotomografía de un hombre de 70 años en estudio por aneurisma de aorta abdominal,
el radiólogo le informa de la presencia de una oclusión completa de la arteria mesentérica inferior. El
paciente se encuentra completamente asintomático. La oclusión de la arteria mesentérica inferior
cursa de manera asintomática en muchas ocasiones ya que el territorio que irriga puede recibir flujo
proveniente de la arteria:
a. cólica derecha
b. gastroduodenal
c. Epigástrica inferior izquierda
d. esplénica
e. cólica media

19. En las patologías de esófago es importante conocer bien la anatomía esofágica. ¿Cuál de las
siguientes afirmaciones es correcta?
a. El esófago tiene capa mucosa, muscular y serosa
b. El esófago abdominal es más largo que el cervical
c. El esófago torácico pasa por detrás del cayado aórtico

A
d. El epitelio esofágico normal es de tipo cilíndrico.
e. El esófago abdominal es discretamente más largo que el torácico

EK
20. A pesar de que pueda haber variaciones anatómicas, lo habitual es que el ciego sea irrigado por una rama
arterial que proviene de unas de las siguientes arterias:
a. Iliaca derecha
b. Mesentérica inferior
c. Hepática derecha
OT
d. Mesentérica superior
e. Iliaca izquierda

21. Ante un paciente con una cirugía abdominal urgente, el informe operatorio señala que se ha realizado una
resección de todo el duodeno y del tercio proximal del yeyuno manteniendo íntegros el estómago y todo
el íleon, así como los dos tercios distales del yeyuno. En el seguimiento nutricional del paciente ¿Qué
NC

vitamina o mineral presentará con menor probabilidad una disminución de su absorción?


a. Cianocobalamina
b. Calcio
c. Hierro
d. Transcobalamina
BA

e. Transferrina

22. ¿Cuál de las siguientes alternativas detallan las venas que confluyen y forman la vena señalada?
a. mesentérica superior, gástrica izquierda y
gastroepiploica izquierda
b. mesentérica inferior, gástrica izquierda y renal
c. esplénica, mesentérica superior y mesentérica inferior
d. esplénica, pancreatoduodenal y omental izquierda
e. gástrica izquierda, esplénica y hepática común

23. ¿Cuál de las siguientes sustancias forma parte de la secreción biliar?


a. Tripsina
b. Lecitina
c. Elastasa
d. Quimotripsina
e. Pepsina
24. El tubo digestivo contiene diferentes tipos de epitelios y glándulas. La estructura señalada es unay
está localizada en el …………...
a. glándula de Brunner / intestino
grueso b.cripta de Lieberkuhn /
colon
c. cripta de Lieberkuhn / intestino delgado
d. glándula oxintica / estomago
e. célula parietal / estómago

Aunque las criptas de


Lieberkuhn están presentes en el

A
intestino delgado, la
microfotografía es de epitelio de

EK
colon.

25. ¿De qué musculo forma parte el ligamento inguinal?


a. Oblicuo externo del abdomen
b. Oblicuo interno del abdomen
OT
c. Transverso del abdomen
d. Psoas
e. Dorsal ancho

26. Señale cuál de las siguientes afirmaciones NO se relaciona a la siguiente glándula anexa del tubo
digestivo mostrada en la imagen:
NC

a. Es una glándula exocrina compuesta exclusivamente por acinos serosos


b. Su inervación está dada por el nervio auricular mayor (ramo posterior C2), que inerva la
vaina de la glándula así como la piel por encima de esta.
c. Esta glándula produce una secreción mucinosa acuosa, llamada mucoserosa, a través del
conducto de Wharton.
BA

d. Su inflamación puede ser causada por un virus de los Paramyxoviridae, que provocan una
enfermedad muy frecuentemente en niños y adolescentes
e. Es una glándula endocrina y probablemente sea de origen pancreático

Se valida la opción e debido a que no está


relacionada con la imagen.

27. ¿Cuál de las siguientes enzimas está localizada en el borde en cepillo y juega un rol en la digestión de
proteínas?
a. Alfa dextrinasa
b. Pepsina
c. Enterocinasa
d. Lactasa
e. Carboxipeptidasa A.

Se valida la opción c debido a que es correcta en relación a la pregunta.


28. Una de los siguientes sustancias, NO sirve como un buen agente emulsificante:
a. Colesterol
b. Ácidos grasos
c. Sales biliares
d. Lecitina
e. Proteínas de la dieta

Se valida la opción e debido a que es correcta en relación a la pregunta.

29. La sustancia que estimula el crecimiento de la mucosa gástrica es:


a. Secretina
b. Motilina
c. Péptido estimulante de la mucosa gástrica
d. Gastrina
e. Histamina

A
30. ¿Cuál de las siguientes alternativas es una función de la colecistokinina?
a. Relajación de la vesícula para la salida de

EK
bilis b.Secreción de ácidos biliares
c. Contracción del esfinter de Oddi
d. Secreción de enzimas pancreáticas
e. Contracción del duodeno

Se valida la opción b debido al efecto de la CCK sobre la vesicula biliar.


OT
31. Con respecto a la anatomía del tronco celiaco, señale lo correcto
a. El tronco celiaco se origina de la cara posterior de la aorta abdominal
b. Es una arteria delgada que tiene un calibre entre 2 y 3 mm
c. Una de sus ramas es la arteria gástrica derecha
d. La hepática común que es una de sus ramas, participa en la irrigación del estómago.
NC

32. Con respecto a la anatomía del duodeno, marque la respuesta correcta:


a. Tiene una distribución en forma de “C”, que rodea la cola del páncreas
b. La 3ra porción duodenal está contenida en la pinza vascular aortomesentérica
c. Entre la 1ra y 2da porción se forma un ángulo, conocido como el ángulo de Treitz
BA

d. La 4ta porción se dirige a la izquierda, hacia abajo y hacia atrás.


e. En la tercera porción desemboca el conducto colédoco.

33. El hígado está ampliamente tapizado por peritoneo, la estructura que conecta la cara diafragmática del
hígado precisamente con el diafragma es el ligamento:
a. teres
b. falcifor
me
c.triang
ular
d.hepático
común
e.coronario

Se validan la opción c y e debido a que forman parte de los ligamentos que fijan el hígado al diafragma.

34. En el íleon se absorbe aproximadamente el 95% dea través de la circulación enterohepática.


a. agua
b. colesterol
c. sales biliares
d. hidróxicobalamina
e. factor intrínseca
35. Laestimula el mecanismo paracrino de la secreción de ácido clorhídrico.
a. histamina
b. acetilcolina
c. gastrina
d. secretina
e. somatostatina

36. En la digestión de proteinas,es el principal estímulo para convertir el pepsinógeno en pepsina.


a. la gastrina
b. el pH ácido
c. la acetilcolina
d. la ptialina
e. la somatostatina

37. Con respecto a la somatostatina, marque lo correcto:


a. Es secretada por las células S del intestino

A
b. Induce a la producción de VIP
c. Interviene en la fase intestinal de la secreción gástrica

EK
d. Produce acetilcolina para estimular a la célula parietal
e. No interviene en la regulación de la secreción de ácido clorhídico

38. En pecten anal, es una estructura comprendida entre:


a. la línea pectínea y los senos anales
b. la línea blanca y la apertura anal
OT
c. el esfínter anal interno y el externo
d. la línea anocutánea y la línea pectínea
e. la línea blanca y columnas anales

39. ¿Cuál de las siguientes alternativas es una proenzima pancreática?


a. Tripsina
NC

b. Elastasa
c. Quimotripsinógeno
d. Amilasa
e. Procarboxipepitidasa C.
BA

40. En la segmentación hepática de Coinaud, el segmento hepático señalado con la flecha,


corresponde a : En la segmentación hepática de Coinaud, la flecha señala el segmentohepático.
a. IV
b. V
c. VI
d. VII
e. VIII
EXAMEN
PARCIAL SISTEMA
DIGESTIVO (ME154)
Ciclo 2019-02

Sección:Todas
Profesores:Alfaro Salazar, Herberth Romulo; Alva Muñoz, Jose Carlos; Mayor Zevallos, Otto Alberto;
Duración:30 minutos.
Indicaciones:
- Lea atentamente cada pregunta antes de responder.
- Se prohíbe el uso del celular y cualquier dispositivo electrónico.
- Está prohibido intercambiar materiales.
- Coloque su código de alumno en la tarjeta de respuestas. Si su código contiene una letra reemplácela por
un valor numérico siguiendo la siguiente equivalencia: A=9, B=8, C=7, D=6, E=5, F=4 y G=3.
- Traslade sus respuestas a la tarjeta, llenando los círculos de manera completa con lapicero negro o azul.

A
Está prohibido el llenado con lápiz, lapicero de otro color o con lapicero de tinta borrable.
- Sea cuidadoso en el llenado de la tarjeta de respuestas, pues solo esta tiene validez para la calificación.
- Al terminar su examen avise al docente a cargo, no se levante de su sitio; debe entregar la hoja de

EK
respuestas con la carátula del examen, este cuadernillo de preguntas se lo llevará cada estudiante.

1. La contracción del músculopermite la eliminación de gases (flatos) sin salida de material fecal;
es el mismo músculo cuya relajación, sobretodo en cuclillas, permite el paso del contenido fecal con
OT
menor esfuerzo durante la defecación.
a) Isquirect
al b)
Puborrec
tal
c) Esfínter anal externo
NC

d) Esfínter anal interno

2. Paciente mujer de 54 años se presenta con náuseas, vómitos, estreñimiento, y es diagnosticada de


abdomen agudo quirúrgico; en la cirugía encuentran un vólvulo de ciego. Esta anomalía puede
explicarse por::
BA

a) Falta de rotación intestinal


b) Falta de fusión del mesenterio
c) Defecto en la formación de la cloaca
d) Falta de formación del omento mayor

3. Paciente mujer de 23 años con faringitis aguda, toma para el dolor una tableta de paracetamol con un
poco de agua. Durante la deglución, se relaja su esfínter esofágico inferior y el fondo del estómago,
mientras el bolo está aún en el esófago. ¿Qué sustancia provocara con mayor probabilidad la relajación
del esfínter esofágico inferior y el fondo del estómago en esta mujer?
a) Óxido nítrico
b) Sustancia P
c) Histamina
d) Motilina

4. Luego de tres horas dando exámenes, un alumno de medicina comienza a sentir hambre. Esta
situación es probable que sea mediada por la que es sintetizada por el :
a) leptina / intestino
b) leptina / estómago
c) grelina / estómago
d) grelina / tejido adiposo

5. Varón de 72 años, con antecedente de diabetes mellitus tipo 2, que presenta enteropatía diabética
caracterizada por estreñimiento. Este problema puede estar asociado a:
a) deficiencia de óxido nítrico
b) aumento del reflejo gastrocólico
c) disminución de la secreción de colecistocinina (CCK)
d) aumento de la secreción del péptido intestinal vasoactivo (PIV)
6. Varón de 54 años con Diabetes Mellitus tipo 2, es diagnosticado de gastroparesia debido a que presenta
sensación de llenura precoz al comer, y reflujo gastroesofágico. Esta alteración en la relajación receptiva
y en el vaciamiento gástrico lo más probable es que se deba a una alteración en:
a) el nervio vago
b) el ganglio celíaco
c) plexo submucoso
d) nervio hipogástrico

7. Varón de 67 años con tos y disminución de peso asociado a tabaquismo pesado, presenta actualmente
disfagia progresiva a alimentos sólidos. Se considera la presencia de un carcinoma de bronquio
izquierdo y por esta razón le realizan una endoscopía esofágica para descartar la posibilidad de una
compresión esofágica por el tumor. Se espera revisar el esófago en la estrechez, que está a nivel de la
vértebra
a) Tercera estrechez -T6
b) Segunda estrechez - C6
c) Segunda estrechez - T4

A
d) Tercera estrechez -T10

EK
8. Varón de 34 años con dolor abdominal agudo en flanco derecho que se irradia a fosa ilíaca derecha, es
operado y se encuentra un divertículo intestinal inflamado, ubicado a 93 cm de la válvula ileocecal. El
origen de este divertículo es una falla en la obliteración de:
a) Conducto vitelino
b) Alantoides
c) Cloaca
OT
d) Conducto anorectal
e) Uraco

En un niño menor de dos años con divertículo intestinal, este divertículo tiene su origen en una
falla en la obliteración de:
NC

a)Conducto
anorectal b)
Conducto
vitelino
c) Alantoides
BA

d) Cloaca
e) Uraco

9. Mujer de 43 años sufre un grave accidente de tránsito y está hospitalizada en coma, es alimentada por vía
intravenosa durante varias semanas. Producto de este tipo de alimentación, se encuentra en la endoscopía
atrofia de la mucosa gastrointestinal. La causa más probable de esta atrofia son los bajos niveles séricos de
la hormona:
a) Colecistocinina
b) Secreti
na
c)Gast
rina
d) PIV

10. Una mujer de 30 años llega al consultorio porque se queja de dificultades para deglutir, la cual se agravan
cada vez más. Se realiza un estudio manométrico para examinar la generación de presión a lo largo del
esófago. Esta prueba revela que las contracciones como respuesta a la deglución están mal sincronizadas y
que la presión en el esfínter esofágico inferior permanece elevada. El diagnóstico más probable es
producido por niveles bajos de
a) acalasia / sustancia
P b) acalasia / óxido
nítrico
c) enfermedad por reflujo gastrointestinal / acetilcolina
d) enfermedad por reflujo gastrointestinal / óxido nítrico

11. Paciente de 2 años, llega a emergencia por haber ingerido una moneda con la que estaba jugando. El
lugar más probable donde puede haberse quedado suspendido este objeto es a nivel del estrechamiento
producido a nivel del:
a) músculo milohiodeo
b) músculo aritenoideo

A
EK
OT
NC
BA
c) músculo cricofaríngeo
d) constrictor superior de la faringe

12. En una apendicectomía, al realizar la incisión de McBurney en la fosa iliaca derecha, es necesario
cortar los siguientes músculos, de afuera hacia adentro:
a) Recto – Oblicuo externo – Transverso
b) Recto – Oblicuo externo – Oblicuo interno
c) Oblicuo externo – Oblicuo interno – Recto
d) Oblicuo externo – Oblicuo interno – Transverso

13. Un varón de 90 años que se encuentra postrado en cama, es referido del asilo para endoscopia por
dificultad para deglutir luego de tomar un medicamento para aliviar el dolor la noche anterior. La
endoscopía revela que la píldora se alojó en el esófago y causó una reacción inflamatoria. Lo más
probable es que esto haya sido por la producción de múltiples ondas:
a) secundarias
b) primarias

A
c) lentas
d) segmentarias

EK
14. Mujer de 23 años es diagnosticada de bulimia, al examen físico se observa ulceraciones en el segundo y
tercero dedo de la mano derecha. Esto se puede deber al uso continuo de estos dedos para inducir el
vómito, mediante la estimulación del par craneal:
a) V
b
OT
)
I
X
c) X
d) XI
NC

15. Varón de 52 años se presenta por diarrea persistente de seis semanas de duración. En la colonoscopia se
observa un pólipo a nivel del íleon distal. El patólogo informa que se trata de un tumor neuroendócrino,
probablemente originado por las células enterocromafines del intestino. La sustancia que más
probablemente esté produciendo este tumor es:
a) Serotonina
BA

b) Insulina
c) CCK
d) GIP

16. La fase oclusal de la masticación se realiza con la contracción de los músculos:


a) digástricos
b) masetero y temporal
c) orbicular y buccinador
d) pterigoideo lateral y digástrico

17. Al tomar su café en Starbucks, un estudiante de medicina sufre una quemadura de primer grado en
el tercio anterior de la superficie dorsal de la lengua. La información de dolor es transmitida por el
nervio:
a) cuerda del tímpano
b) glosofaríng
eo
c)lingual
d) facial

18. Paciente es evaluado por faringitis aguda en consultorio externo. El médico de familia le solicita que abra
la boca y saque la lengua. Para realizar la acción de sacar la lengua, es necesario que se contraiga el
músculo:
a) estiloglo
so b)
geniogl
oso
c) palatogloso
d) transverso de la lengua

A
EK
OT
NC
BA
19. Paciente con síndrome de Sjögren, presenta “boca seca”
(disminución de la producción de saliva) y caries dental, asociada
a la pérdida de la función de tampón de la saliva. Esta
desminerilización del diente puede comprometer a las
prolongaciones citoplasmáticas ubicadas en los tubos huecos de
la estructura señalada con la letra:
a) B
b) A
c) E
d) C

A
20. Mujer de 32 años acude a consulta por presentar disfagia de
progresión lenta, reflujo gastroesofágico y vómitos desde hace 3

EK
meses de evolución progresiva. Se le realiza un estudio radiológico
con contraste en el que se observa estrechamiento del esfínter
esofágico inferior (imagen). Según sus conocimientos, este paciente
se beneficiaría con el uso de:
a) agonista beta adrenérgico
b) agonista alfa
OT
adrenérgico c)análogo
de óxido nítrico
d) análogo de Sustancia P
NC
BA
EXAMEN
PARCIAL SISTEMA
DIGESTIVO (ME154)
Ciclo
202000

Sección:Todas
Profesores:Alva Muñoz, Jose Carlos
Duración:35 minutos.
Indicaciones:
- Lea atentamente cada pregunta antes de responder.
- Se prohíbe el uso del celular y cualquier dispositivo electrónico.
- Está prohibido intercambiar materiales.
- Coloque su código de alumno en la tarjeta de respuestas. Si su código contiene una letra reemplácela por
un valor numérico siguiendo la siguiente equivalencia: A=9, B=8, C=7, D=6, E=5, F=4 y G=3.

A
- Traslade sus respuestas a la tarjeta, llenando los círculos de manera completa con lapicero negro o azul.
Está prohibido el llenado con lápiz, lapicero de otro color o con lapicero de tinta borrable.
- Sea cuidadoso en el llenado de la tarjeta de respuestas, pues solo esta tiene validez para la calificación.

EK
- Al terminar su examen avise al docente a cargo, no se levante de su sitio; debe entregar la hoja de
respuestas con la carátula del examen, este cuadernillo de preguntas se lo llevará cada estudiante.

1. Paciente de sexo masculino de 82 años de edad ingresa a emergencia con dolor abdominal agudo y
diarreas. Se le realiza una arteriografía en la que se observa que la arteria aorta tiene un trombo
OT
ocluyendo el 95% del flujo, a nivel del nacimiento de la arteria mesentérica inferior. ¿Cuál de las siguientes
arterias podría contribuir a la irrigación colateral del colon descendente?
a) cólica media
b) sigmoidea
c) rectal superior
d) ileocólica
NC

2. Niño de 5 años presenta dolor esofágico y hematemesis (vómitos hemorrágicos) luego de tragarse una
espina de pescado. En la endoscopía se observa perforación del esófago distal a la cuarta estrechez
esofágica. ¿Las ramas de cuál de las siguientes arterias estarán lesionada con mayor probabilidad?
a) Gástrica izquierda
b) Bronquiales
BA

c) Frénica inferior
d) Tiroidea inferior

3. Al ingerir una cucharada de mantequilla es muy probable que se disminuya la sensación de hambre por
medio de la activación de la vía POMC/CART (POMC=proopiomelanocortina y CART=transcripción
regulada de cocaína y anfetamina), activada directamente por la hormona:
a) colecistoquinina (CCK)
b) insulina
c) grelina
d) secretina

4. Al comer unas papitas fritas con mayonesa, el vaciamiento gástrico disminuye por efecto directo de la
hormona: a)colecistoquinina (CCK)
b) bombesina
c) motilina
d) gastrina

5. Recién nacido de dos horas es diagnosticado de hernia umbilical de 1,5 cm de diámetro; el cirujano
pediatra solicita una tomografía abdominal en donde se evidencia que la hernia umbilical está ocupada
por una porción del tracto gastrointestinal. ¿Qué porción del tracto gastrointestinal estaría ocupando
esta hernia con mayor probabilidad?
a) Íleon
b) Colon sigmoides
c) Duodeno
d) Colon transverso
6. Recién nacido de 7 horas, de parto por cesárea debido a polihidramnios (aumento del volumen del
líquido amniótico), con regurgitación de la leche materna y artificial, y no ha presentado meconio. Se le
realiza una tomografía donde se evidencia aire en el estómago y una malformación del desarrollo
esofágico. Con respecto a esta malformación lo más probable es que se pueda tratar de una atresia
esofágica:
a) proximal con fístula traqueoesofágica distal
b) distal con fístula traqueoesofágica proximal
c) proximal y distal
d) sin fístula

7. Lactante de 6 meses de edad que es traído a consulta por presentar vómitos no biliosos a repetición y
retraso en el crecimiento. En la radiografía de abdomen simple se observa nivel hidroaéreo en estómago y
en primera porción de duodeno (doble burbuja). ¿Cuál de las siguientes alternativas puede explicar la
condición del lactante?
a) Páncreas anular
b) Atresia duodenal en la tercera porción

A
c) Atresia yeyunal
d) Hipertrofia del píloro

EK
8. En ausencia o deficiencia de la secreción de la hormona motilina, se
producirá: a)sobrecrecimiento bacteriano
b) diarrea
c) aumento del vaciamiento gástrico
d) hipertrofia del píloro
OT
9. La estimulación parasimpática aumenta la motilidad intestinal, mientras que la estimulación simpática
la disminuye. ¿Sobre cuál de las siguientes alternativas el sistema nervioso autónomo actúa para el
control de la motilidad intestinal?
a) Potencial de membrana en el plexo mientérico (de Auerbach)
b) Frecuencia de ondas lentas
NC

c) Secreción de secretina
d) Nivel de IP3 en el plexo submucoso (de Meissner)

10. En un recién nacido con protrusión de contenidos abdominales y cubiertas por amnios o peritoneo, es
cierto que: a)Se presenta por un defecto en el cierre de la pared
BA

b) Se acompaña de otras malformaciones congénitas


c) Se debe al no retorno de la hernia fisiológica
d) Se produce a través del ombligo

11. Lactante de 20 días con estreñimiento, distención abdominal progresiva, acompañada ocasionalmente de
vómitos biliosos. Como antecedente, el meconio lo eliminó por primera vez a las 72 horas de nacido. Su
mamá menciona que ayuda a la evacuación con ayuda de un termómetro rectal. Se sospecha de
megacolon agangliónico (Enfermedad de Hirschsprung). ¿Cuál de las siguientes alternativas explica el caso?
a) Se presenta contracciones tónicas en la región ano rectal
b) Se presenta dilatación de tracto gastrointestinal afectado
c) Las células ganglionares sólo han migrado al ano recto
d) La zona que más se afecta es inervada por fibras del nervio esplácnico menor

12. ¿Cuál de los siguientes reflejos disminuye el tránsito


gastrointestinal? a)Doloroso
b) Gastrocólico
c) De defecación
d) Colicoileal

13. Al ingerir una sustancia ácida como el vino (pH 3), se estimula la motilidad gástrica por acción de la
hormona: a)motilina
b) secretina
c) colecistoquinina (CCK)
d) bombesina
14. Paciente de 24 años acude a consulta externa por presentar una fístula oronasal (comunicación entre la
cavidad oral y la cavidad nasal). Está fístula está asociada al antecedente de haber sido operada de
paladar hendido a los dos años de edad, durante una campaña gratuita extranjera de corrección de
paladar fisurado. ¿Cuál de las arterias palatinas podría haberse lesionado durante esa cirugía?
a) Mayor
b) Menor
c) Ascendente
d) Rama palatina de la faríngea ascendente

15. Paciente de sexo masculino de 52 años con úlcera péptica gástrica de 14 años de evolución, con cuadro
de hemorragia digestiva alta hace 4 meses, sin cicatrización de la úlcera. Entre las opciones quirúrgicas se
considera realizarle un vaguectomía troncal (sección del nervio vago) a nivel del hiato esofágico. ¿Cuál de
las siguientes complicaciones podría esperarse producto de la pérdida de inervación parasimpática?
a) Menor inervación del colon ascendente
b) Se perderá el reflejo de defecación
c) Se perderá el reflejo de micción

A
d) Impotencia sexual

EK
16. Paciente de 23 años con bulimia es traída a la emergencia deshidratada, semiconsciente y con alcalosis
metabólica. Los vómitos autoinfligidos por esta paciente se producen por estimulación de receptores en
la base de la lengua que mandan información directamente al:
a) núcleo del tracto solitario
b) centro del vómito en el tallo encefálico
c) zona quimiorreceptora gatillo
OT
d) cerebelo

17. Niño de 3 años es traído a emergencia por madre quien manifiesta que hace 10 horas deglutió una pila
pequeña de reloj de bordes romos. El niño está asintomático. Usted la tranquiliza diciéndole es un cuerpo
extraño tan pequeño de seguro que va a seguir el tránsito intestinal como lo haría un bolo alimenticio, y
que lo más probable es que en ese momento se encuentre en:
NC

a) colon
b) estómago
c) yeyuno
d) recto
BA

18. Los movimientos en masa son un tipo de movimiento muy importante, una de las consecuencias
de estos movimientos es:
a) la distensión rectal
b) el peristaltismo del intestino delgado
c) la retropulsión gástrica
d) la contracción del esfínter anal interno

19. Durante la deglución, al momento que el bolo alimenticio pasa por el esfínter esofágico superior, se
espera que la presión intraesofágica:
a) disminuya en el cardias
b) disminuya en el tercio medio del esófago
c) aumente en la porción distal al bolo
d) aumente en el tercio medio del esófago

20. Paciente con enfermedad de Chagas que presenta disfagia a sólidos. ¿Cuál de las siguientes puede ser la
causa de esta complicación?
a) Disminución de células ganglionares en el esfínter esofágico inferior
b) Aumento en la liberación de óxido nítrico en el esfínter esofágico inferior
c) Disminución de las neuronas que liberan péptido intestinal vasoactivo
d) Aumento de la actividad de la motilina en el esófago distal
QUIZIZZ
1. Los vasos mesentéricos superiores se hallan a nivel de:
a) Cuello del páncreas

2. El nivel en el que se encuentra el píloro y el páncreas se puede determinar usando el


A) plano transpilórico

3. El dolor de estómago asociado a gastritis se suele ubicar en


a) epigastrio

4. La colecistoquinina inhibe el
a) vaciamiento gástrico

A
5. Paciente con disfasia ( dificultad para pasar alimentos) con to y disminución de peso. Con antecedente

EK
de tabaquismo pesado. La sospecha es que tenga una disminución del diámetro esófago a nivel de la
a) tercera estrechez

6. La digestión de las proteínas se inicia en:


OT
a) estómago

7. El estómago recibe información simpática proveniente del:


a) ganglio celíaco
NC

8. El reflujo gastroesofagico tiene múltiples etiologías, una de ellas tiene que ver con alteración a nivel de:
a) primera estrechez
b) segunda estrechez
BA

c) tercera estrechez
d) cuarta estrechez

9. La fístula retroperitoneal es causada por una falla en el desarrollo de:


a) tabique urorrectal

10. El nervio vago inerva el:


a) músculo estriado del esófago

11. El divertículo de Meckel es un rezago de:


a) conducto vitelino

12. La presencia de orina que sale por el ombligo de un recién nacido casa vez que llora, es posible que se
deba a un defecto en el desarrollo del
a) seno urogenital

13. La fístula retroperitoneal es causada por una falla en el desarrollo de


a) tabique urorrectal

14. Enfermedad asociada con un error en el desarrollo de las células de Cajal:


a) enfermedad de Hirschsprung

15. Aproximadamente en la semana 6 del desarrollo embrionario, el intestino medio gira 90 herniandose a
nivel del
a) cordón umbilical

16. El conducto biliar deriva del


a) endodermo

17. Paciente mujer con 54 años con nauseas y vómitos y abdomen agudo quirúrgico, se ingresa a sala de
operaciones donde se encuentra vólvulo de ciego, esto se debe a
a) Falta de fusión del mesenterio

A
18. Paciente con cirrosis hepática con hipertensión portal, en el que es posible encontrar que los vasos

EK
umbilicales están permeables dentro de
a) ligamento redondo

CUAL
OT
19. Es normal encontrar glándulas submucosas en

ES
NC

a) esófago medio
b) esófago proximal
BA

c) esófago distal
d) estómago

20. Cual de los siguientes órganos son intraperitoneales


a) estómago, vesícula biliar, íleon, hígado

Estudiante de medicina de la UPC de 21 años sufre de gastritis aguda ocasionada por comer en lugares
poco higiénicos. Suele consumir caramelos ( chupar ) mientras está en clase hasta la tarde. Toma gaseosas
regularmente (carbohidratos 46%, sodio 53%). También toma regular cantidad de leche (grasa 35%, lactosa
35%, proteínas 30%), pues le calma un poco el dolor el ardor que siente por la gastritis. Incluso, cuando
puede, se toma dos vasos de agua fría para calmar las molestias. Ha decidido ir al médico para tratarse pues
ya no soporta el dolor, el cual está seguro que los síntomas se deben a una elevada producción de ácido
clorhídrico en el estómago, y por ello le ha recetado Ranitidina (antihistamínico), con lo que siente mejoría.

● Para reducir la secreción de HCl en esta paciente se podría usar sustancias similares a:

- Péptido insulinotrópico dependiente de la glucosa (GIP)

● Si se usara atropina en esta paciente, se esperaría que disminuya la liberación de:


- Enzimas pancreáticas
● El consumir caramelos eleva los niveles en sangre de una hormona cuya función es la
estimulación de las células:
- Beta del páncreas

● En este paciente con gastritis aguda debida a una alta producción de ácido clorhídrico, si
se le hiciera un examen de sangre, se encontraría elevados los niveles de:
- Colecistoquinina

● El consumo de una pequeña cantidad de gaseosa aumentará directamente la


concentración sérica de cuál de las siguientes hormonas:

- Péptido 1 similar al glucagón (GLP-1)

● El consumo rápido de 500 mL de gaseosa aumentará directamente la concentración sérica


de cuál de las siguientes hormonas:

A
- Gastrina

EK
● Estimulan la secreción ácida gástrica

- Proteínas
OT
● Con respecto a las ondas lentas, marque la afirmación correcta:}

- Son contracciones rítmicas espontáneas

● El uso de Ranitidina bloquea el receptor H2 de la histamina en las células


NC

parietales. La histamina llega a estas células por:

- Difusión

● El consumir caramelos indirectamente activa la vía:


BA

- POMC/CART

● ¿Cuál de los siguientes péptidos inhibe el vaciamiento gástrico?

- Colecistoquinina

● Para poder morder una manzana, es necesario usar el siguiente músculo:

- Milohiodeo

● El crecimiento de un adenocarcinoma de cuello de páncreas puede comprometer la pared


gástrica por continuidad. ¿Qué parte del estómago estaría comprometido con mayor
probabilidad?

- Pared posterior del antro

● El nacimiento de la arteria mesentérica superior se puede encontrar en cuál de los


cuadrantes abdominales:

- Epigastrio

● En cuanto a la colecistoquinina, marque la respuesta correcta:

- Potencia la acción de buffer con bicarbonato

● El aumento en la actividad motora de la pared gástrica genera un aumento en los


niveles locales de qué sustancia en la microvasculatura:

- Adenosina

Niño de sexo masculino de 2 años de edad, sufre de estreñimiento desde el nacimiento (1


deposición cada 3-4 días). Madre menciona que le estimula la defecación con un termómetro
rectal, y continuo uso de enemas y laxantes. Desde hace 6 meses comienza con vómitos
postprandiales. Los síntomas aumentan en frecuencia y magnitud y están en relación con los
episodios de estreñimiento. No refiere fiebre, tos, diarrea ni lesiones cutáneas. Al examen físico
presenta regular estado general, luce deshidratado. Abdomen distendido, blando, depresible e
indoloro. No se palpan masas abdominales. Se permeabiliza el canal anal con termómetro rectal,
encontrando cierta resistencia. Salida de material fecal mal oliente en regular cantidad. Exámenes
de laboratorio: hemograma normal. Signos inflamatorios de fase aguda negativos. Alcalosis
metabólica leve en sangre venosa. Radiografía con enema baritado muestra recto y colon
sigmoides dilatados (megacolon). Biopsia profunda: ausencia de células ganglionares en la
muestra enviada. Se realiza cirugía correctiva.

A
● La percepción de la pirosis (sensación de dolor o quemazón en el esófago) asociado al
reflujo gastroesofágico, puede aparecer o exacerbarse debido a:

EK
- Ejercicio

● Considerando que este paciente está sometido a estrés por el agravamiento de su


enfermedad, es posible afirmar que sus ondas lentas están:
OT
- Hiperpolarizadas

● Debido al acúmulo de material fecal en todo el marco colónico, y a la irritación química


asociada, el peristaltismo del íleon distal se debe encontrar:

- Inhibido
NC

● En cuanto a los reflejos gastrocólico y gastroduodenal en este paciente, indique lo


correcto:

- Se pueden considerar reflejos vago-vagales


BA

● Con respecto a la defecación señale el enunciado correcto:

- Es estimulado por un llenado de la cuarta parte del volumen rectal

● El contenido fecal se detiene en la zona inmediatamente proximal a la zona donde hay una
menor presencia de:

- Péptido intestinal vasoactivo

● La presencia de atresias y estenosis duodenales se deben básicamente a una:

- Falta de recanalización

● El ligamento de Treitz característicamente:

- Suspende el ángulo de Treitz

● Al deglutir un bolo alimenticio, es lógico suponer que al pasar por el esófago haya un
mayor consumo de oxígeno en la pared del tercio:

- Proximal

● Estudiante de medicina de 20 años, se ha amanecido estudiando para su examen de


Sistema Digestivo. No ha probado alimento desde la cena, por lo que se puede afirmar que
la motilidad de esta persona está siendo regulada por:

- Motilina

● La hernia fisiológica se produce dentro de:


- Cordón umbilical

● El crecimiento de un adenocarcinoma de páncreas compromete la pared gástrica por


contigüidad. ¿Qué parte del estómago se esperaría esté comprometido?

- Pared posterior del antro

● Al comer unas papitas fritas con mayonesa, el vaciamiento gástrico disminuye por efecto
directo de la hormona:

- colecistoquinina (CCK)

● Paciente que come entera una pizza familiar de chorizo y queso. Es posible esperar que
debido a la cantidad de alimento ingerida, las ondas lentas hayan:

A
- Sufrido ninguna alteración en su frecuencia

● En este caso se puede afirmar con seguridad que se presenta:

EK
- contracciones tónicas en la región ano rectal

● La forma más común de atresia esofágica contiene:

- Estenosis proximal del esófago más fístula traqueoesofágica distal


OT
● Con respecto a la saliva, marque la respuesta correcta:

- el sistema simpático estimula su secreción

● Respecto a las enfermedades del esófago, marque lo correcto:


NC

- el diagnóstico diferencial de la acalasia es la enfermedad de Chagas esofágica

● En relación a la fisilogía gástrica, marque lo correcto:

- la cimetidina actúa en la región basolateral de la célula parietal


BA

● La célula mucosa del cuello gástrico produce:

- Moco

● La saliva puede tener una variedad de electrolitos en su composición. Entre ellos el cloro,
respecto al cual se puede afirmar:

- Su concentración no llega a ser tan alta como en el plasma

● Los músculos de la masticación que producen la retropulsión de la mandíbula son:

- temporales

● Respecto a las glándulas salivales, marque lo incorrecto:

- la glándula sublingual tiene forma de garfio

● Respecto a la anatomía del estómago, marque lo correcto:

- la arteria gástrica derecha nace de la arteria hepática común

● En cuanto a la saliva, marque lo correcto:

- La amilasa cumple función digestiva

● Durante el ataque con gas sarín (bloqueador de la acetilcolinesterasa) en el metro de


Tokio, en 1995, el personal de salud notó que los pacientes afectados presentaban:

- Hipersalivación

Paciente de 54 años con antecedentes de alcoholismo, gastritis crónica, tabaquismo


pesado, obesidad, cálculos biliares y cirrosis, es llevado a la emergencia por dolor
abdominal en epigastrio irradiado a la espalda y trastorno del sensorio.
Al examen físico: presión arterial 85/50 mmHg, frecuencia cardíaca 100 latidos/min,
frecuencia respiratoria 18 x minuto, temperatura axilar 36°C.
Conjuntivas pálidas, escleras ictéricas nevus arácnidos en tronco, distensión abdominal
marcada, cabeza de medusa, matidez desplazable en ambos flancos e hipogastrio, dolor
a la palpación de abdomen.
Tiempo de protrombina: 24 seg (testigo: 13 seg); TPT: 38 seg, glicemia: 165 mg/dL,

A
uremia: 20 mg/dL, ASAT: 76 UI/L, ALAT: 22 UI/L, albumina: 2,5 g/dL, bilirrubina total: 2,6
mg/dL, bilirrubina directa: 1,4 mg/dL, amilasa sérica 4000 U/L.

EK
● Un efecto secundario en el estómago por la acción de la secretina es:

- Menor actividad de la pepsina

● Considerando que el paciente sufre de gastritis, se puede decir que la secreción de ácido
por la mucosa gástrica:
OT
- Involucra transporte activo de Hidrogeniones

● En cuanto a la gastritis de este paciente, se encontró que era producida por la bacteria
Helicobacter pylori. Esta bacteria sobrevive en el medio ácido del estómago gracias a:
NC

- Ureasa

● La bilirrubina directa aumentada en cirrosis hepática se excreta en la orina debido a:


BA

- Ser hidrosoluble

● Paciente de 42 años con adenocarcinoma ductular. La TC ha demostrado claramente que el


tumor está en el cuello del páncreas y que hay un gran vaso ocluido. ¿Cuál de los siguientes
vasos estaría más probablemente obstruido?

- Vena porta.

● En relación a la histología hepática, marque lo correcto:


- La zona 3 se encuentra más cerca a la vena central lobulillar

Mujer de 83 años acude a emergencia por dolor abdominal desde hace 4 días,
localizado en epigastrio, irradiado a ambos hipocondrios, nauseas, vómitos y
distensión abdominal; tiene antecedente de cardiopatía hipertensiva, diabetes
mellitus tipo II y fibrilación auricular. Refiere deposiciones diarreicas muco
sanguinolentas hace 1 día. Los exámenes iniciales muestran PA: 110/60
mmHg, FC: 110/mn, leucocitos: 17800, neutrófilos de 93%. TAC abdómino
pélvica se observa oclusión completa de arteria mesentérica superior por
trombo asociado a placa ateromatosa.
● En esta paciente, ¿cuál de las siguientes sustancias no tendrá una considerable
disminución en su absorción? (marque la mejor respuesta):

- Calcio

● El mecanismo de la diarrea muco sanguinolenta que presenta la paciente, puede mejor


definido como de tipo (marque la mejor respuesta):}

- Exudativa

● Considerando que se ha comprometido el íleon distal, entre otras áreas, la atrofia o


descamación del epitelio de superficie explicaría cuál de los signos o síntomas de la
paciente (marque la mejor respuesta):

A
- Diarreas mucosanguinolentas

● Producto de esta isquemia, la expresión de cuál de las siguientes enzimas se vería

EK
notablemente disminuida. Marque la mejor respuesta:

- Enteroquinasa

● Durante la cirugía, el cirujano observó que además la paciente tenía divertículos en el


sigma. Se sabe que estos divertículos:
OT
- Se pueden asociar a estreñimiento crónico

Autoevaluación 1:
NC

Un paciente varón de 68 años consulta por dolor y abultamiento en la región inguinal derecha, que
aparece tras la realización de un esfuerzo físico. En la exploración en bipedestación presencia de una
tumoración blanda, depresible, que aumenta con la tos. El presente caso describe una hernia:
Inguinal directa.
Umbilical.
BA

Femoral.
Inguinal indirecta.

La Grelina es sintetizada por………….. y activa las neuronas relacionadas con


……………………….en el núcleo arqueado del hipotálamo}

El páncreas/AGRP-NPY
El intestino delgado/POMC-CART
El estómago/AGRP-CART
El estómago/AGRP-NPY

Cuál de los siguientes órganos son intraperitoneales:

Estómago, Vesicula biliar, Ileón, Higado


Páncreas, Colon descendente, Hígado, Vesícula biliar
Recto, Hígado, Colon transverso, Yeyuno
Estómago, Yeyuno, Duodeno, Páncreas

La distención gástrica por los alimentos produce incremento de secreción de HCl mediante el siguiente
mecanismo:

Producción de Gastrina que desencadena su cascada de señalización en la célula parietal vía proteína Gi
Producción de Acetilcolina que desencadena su cascada de señalización en la célula parietal vía proteína
Gs
Producción de Gastrina que desencadena su cascada de señalización en la célula principal vía proteína
Gs
Producción de Gastrina que desencadena su cascada de señalización en la célula parietal vía proteína Gq

La inervación de la piel del abdomen debajo del ombligo hasta la región púbica está dada por:

T7, T9
T11, T12, L1
T7, T9, L2
L2, L3, S1

Paciente de 38 años que tras riña durante partido de futbol sufre un traumatismo con arma blanca en

A
cuadrante inferior izquierdo del abdomen. En la tomografía de urgencias se evidencia gran
hematoma de pared y ausencia de neumoperitoneo. Desde la piel hacia al peritoneo, en ¿qué orden

EK
se atravesó la pared abdominal?

Oblicuo externo, oblicuo interno, musculo transverso, TCSC


TCSC, oblicuo externo, oblicuo interno, musculo transverso
TCSC, oblicuo interno, oblicuo externo, musculo transverso
OT
TCSC, musculo transverso, oblicuo externo, oblicuo interno
Oblicuo externo, musculo transverso, oblicuo interno, TCSC

Alrededor de 90% de los pacientes afectados por el síndrome de Zollinger-Ellison desarrollan ulcera
NC

péptica. La causa es:

Gastritis atrófica
Hipersecreción ectópica de gastrina
Reflujo gastroesofágico
BA

Gastropatía hipertrófica

Un hombre de 22 años sufre traumatismo en el flanco izquierdo superior del abdomen al tratar de
defenderse del robo de su vehículo. Un amigo lo traslada a un centro hospitalario. El paciente
presenta signos de hipovolemia con taquicardia e hipotensión. Se queja de dolor en el sitio de lesión
que se irradia al hombro izquierdo. Elñ mórgano probablemente lesionado es

Hígado
Bazo
Estómago
Colon

Aproximadamente en la semana 6 del desarrollo embrionario, el intestino medio gira 90° herniándose
a nivel del:

Borde inferior del bazo


Caudal al borde hepático derecho
Cordón umbilical
Lado derecho del abdomen
La triada portal (arteria hepatica, vena portal y conducto biliar común) está contenida en el:

Ligamento Gstroesplenico
Ligamento gastrohepático
Ligamento hepatoduodenal
Ligamento Falciforme

En el siguiente gráfico marque la relación correcta:

A
EK
1
4
OT
NC
BA

3 2

Omento mayor – 4
Plexo de Meissner – 1
Muscular de la mucosa – 3
Glándula de la mucosa

¿Cuál de las siguientes sustancias es liberada por neuronas en el tracto GI,


Participa en la regulación hidro electrolíticay produce relajación del musculo liso?

Gastrina
Secretina
CCK
VIP
La colecistoquinina inhibe:

El Vaciamiento gástrico
La Secrecion pancreatica de HCO3- La Contraccion de la
vesicular biliar
La Relajación del esfínter de oddi

¿La secreción de cuál de las siguientes es inhibida por un pH bajo?

Secretina
Gastrina
CCK
VIP

A
El conducto biliar deriva del:

EK
Ectodermo.
Endodermo.
Mesodermo.
Mesotelio.
OT
Respecto a la imagen mostrada la estructura señalada con el número 1 corresponde a:

Glándula parótida
NC

Glándula sublingual
Glándula submaxilar
Glándula lingual
BA

3
Los vasos mesentéricos superiores se hallan a nivel de:

El fondo gástrico
El cuello del páncreas
La cabeza del páncreas
El hilio hepático

La imagen mostrada representa un defecto de ……………… y recibe el nombre de: ……………………….

A
EK
OT
NC
BA

La pared abdominal/onfalocele
La pared abdominal/divertículo de Meckel
Malrotación intestinal/gastrosquisis
La pared abdominal/gastrosquisis

Actividad extra sem 1:

1. Existen 3 perforaciones de la membrana pleuro-parietal, denominados hiatos, causados por la vena


cava inferior, el esófago y la aorta. Mencione a que nivel torácico (vertebras torácicas) se encuentran
estos hiatos.
• VCI: T8
• ESOFAGO: T10
• AORTA: T12

2. Mencione a nivel de que dermatoma se encuentra el ombligo.


• T10
3. Mencione en orden, de adentro hacia afuera, los componentes de la pared abdominal anterior.
a) Peritoneo
b) Fascia extraperitoneal
c) Fascia transversalis
d) Músculos con sus aponeurosis
e) Fascia de Scarpa
f) Fascia de Camper
g) Grasa
h) Piel

4. ¿Cuál es el plexo nervioso que se encuentra entre la capa muscular circular interna y longitudinal
externa?
• Plexo mientérico de Auerbach
5. ¿Qué célula es considerada el marcapaso intestinal, ya que se encarga de las contracciones fácicas y
tonicas del sistema gastrointestinal?
• Célula intersticial de Cajal

A
EXAMEN PARCIAL 2019-01
1. ¿Cuál de las glándulas salivales es responsable del mayor porcentaje del volumen de la saliva en

EK
condiciones basales?
a. Parótida
b. Subpalatinas
c. Sublingulaes
d. Submaxilares
OT
2. La lengua está recubierta por epitelio:
a. pseudoestratificado columnar no queratinizado
b. plano estratificado no queratinizado
NC

c. pseudoestratificado columnar ciliado


d. plano estratificado queratinizado

3. El esfínter anal interno tiene musculatura …………… y tiene control …………………. a. lisa / voluntario
b. lisa / involuntario
BA

c. esquelética / simpático
d. esquelética / parasimpático

4. La arteria aorta proporciona la irrigación al tubo digestivo ¿cuál de las siguientes arterias
proporciona la irrigación al ángulo cólico derecho?
a. Mesentérica superior
b. Mesentérica inferior
c. Frénica inferior
d. Tronco celiaco

5. Paciente de 26 años que le cuenta en su historia clínica que cada vez que almuerza, a los 20 minutos
tiene deseo de defecar. Le comenta que su hijo de 1 mes le pasa lo mismo pero más intenso. Esto se
explica por el reflejo …………………, el cual está ……………..en el paciente.
a. colicoileal / normal
b. colicoileal / alterado
c. gastrocolico / normal
d. gastrocolico / alterado

6. La región del estómago que se comunica con el duodeno se denomina:


a. pilórica
b. cardias
c. cuerpo
d. fórnix

7. Acude a consulta un paciente que fue diagnosticado de ulcera péptica 3 días antes. Luego de
múltiples pruebas diagnósticas, se concluye que el paciente presenta un tumor secretor de gastrina
¿Cuál de las siguientes situaciones estará incrementada?
a. Distención gástrica
b. Inhibición del vaciado gástrico
c. Secreción de ácido clorhídrico (HCl)
d. Inhibición de la secreción de pepsinógeno

8. En el sistema digestivo, el control del apetito esta dado por un complejo sistema de sustancias y

A
órganos integradores los cuales regulan la ingesta de alimentos. La ……………… es una sustancia
oroxígena y es sintetizada por el ……………………..
a. leptina / intestino

EK
b. grelina / intestino
c. leptina / estómago
d. grelina / estómago
OT
9. Sobre el control autónomo del sistema digestivo, marque la alternativa correcta:
a. La inervación dada por el sistema simpático es de tipo preganglionar.
b. El sistema parasimpático usa como neurotransmisores a la acetilcolina y la noradrenalina.
c. El nervio vago (par craneal X) le da inervación simpática a la mayoría del sistema digestivo.
d. En el sistema simpático, los nervios responsables hacen una primera sinapsis en ganglios
NC

próximos al órgano a inervar.


e. En la inervación de tipo parasimpático, solo interviene el plexo submucoso, sin embargo, en
la de tipo simpático intervienen tanto el submucoso como el mientérico.
BA

10. Con respecto a la actividad eléctrica del sistema digestivo, marque la alternativa correcta
a. Corresponden a potenciales de acción que están presentes de forma continua y le dan la
capacidad de perístasis autónoma al sistema digestivo.
b. La frecuencia de las ondas lentas no se ve influenciada por la actividad neural ni las
hormas gastrointestinales.
c. En el estómago las ondas lentas se dan en una frecuencia de 6 por minuto.
d. Las ondas lentas son cambios lentos y ondulantes del potencial en reposo.
e. La frecuencia de las ondas lentas va de 6 a 12 ondas por minuto.

11. Ante una lesión del IX par craneal, el músculo……………….. se altera en su función.
a. palatogloso
b. estilofaríngeo
c. palatofaríngeo
d. constrictor superior

12. Un varón de 50 años es sometido a extirpación de duodeno y parte proximal de yeyuno. La pérdida
de estímulo hormonal en el páncreas para la secreción enzimática se explica por la pérdida de las
células …………………… a. Parietales, productoras de factor intrínseco
b. “K” productoras de factor intrínseco
c. “M” productoras de CCK
d. “I” productoras de CCK
13. Respecto al mecanismo de la defecación ¿Cuál de las siguientes afirmaciones es correcta?
a. Se produce contracción refleja del esfínter anal interno
b. Se produce contracción o relajación del esfínter anal externo por señales de la corteza
cerebral
c. La presencia de materia fecal en el recto estimula la contracción del sigmoides por los
nervios pélvicos simpáticos
d. En la posición de “cuclillas” el músculo puborectal se halla contraído favoreciendo la
evacuación de la materia fecal

14. Un niño de tres años llega a emergencia con disfagia (dificultad para tragar), salivación y llanto. Se
sospecha de ingesta de cuerpo extraño: moneda en el esófago; al ser evaluado se constata en una
radiografía presencia de cuerpo extraño a nivel de C6 y C7 (6° y 7° vértebra cervical). El cuerpo
extraño estará suspendido a nivel del estrechamiento producido por el ………..

A
a. cayado aórtico
b. hiato esofágico
c. músculo cricofaríngeo

EK
d. bronquio principal izquierdo

15. En el caso de un paciente con un tumor productor de gastrina, la presencia de úlceras duodenales y
erosión de la mucosa gástrica se debe principalmente a…….
OT
a. la acción paracrina de la gastrina sobre la célula parietal
b. el exceso de HCl por estímulo de receptores CCK-B en la célula parietal
c. la sobre expresión de los receptores “G” para gastrina en la célula parietal
d. el exceso de HCl por estímulo directo de receptores “H” en la célula parietal
NC

16. La onda peristáltica secundaria del esófago se caracteriza por ser originada ………
a. por el plexo de submucoso del esófago
b. por el plexo mientérico del esófago
c. por el reflejo de la deglución
d. durante la masticación
BA

17. Marque lo correcto sobre las ondas lentas en el tubo digestivo


a. No son despolarizaciones
b. Son potenciales de acción subumbrales
c. Se constituyen de despolarizaciones y repolarizaciones
d. Son rítmicas y generadas por el sistema nerviosos autónomo

18. Recién nacido que presenta protrusión de contenidos abdominales los cuales no están cubiertos por
peritoneo y salen de la cavidad abdominal a través de un defecto de la pared. ¿Cómo se denomina a
la afección que presenta este paciente?
a. Onfalocele
b. Atresia biliar
c. Gastrosquisis
d. Divertículo de Meckel

19. Experimentalmente se utiliza atropina (anticolinérgico) para inhibir la secreción de gastrina, sin
embargo, la secreción de esta hormona se sigue dando ante estímulos vagales. Esta situación se
explica porque la atropina:
a. no bloquea la acción del péptido GRP
b. solo inhibe la acción del péptido GRP en la célula G
c. inhibe la acción de acetilcolina e histamina en la célula G
d. bloquea parcialmente la bomba de protones en la célula G
20. Niña de 4 días es llevada a la emergencia pediátrica por presentar llanto constante, la madre refiere
coloración azulada de labios al momento de lactar, acompañado de tos persistente y dificultad
respiratoria así como distención abdominal. Se le coloca sonda nasogástrica para alimentación
notando que retorna a la cavidad oral en todos los intentos. ¿Cuál es la anomalía del desarrollo en
este caso?
a. Solo fístula traqueo esofágica
b. Fístula traqueo esofágica proximal y distal
c. Atresia esofágica proximal con fístula traqueo esofágica distal
d. Atresia esofágica distal con fístula traqueo esofágica proximal

21. Paciente varón de 36 años es traído a la emergencia luego de sufrir un accidente de tránsito,
presenta traumatismos múltiples en cabeza y tronco. Al examen físico se evidencia hematoma en

A
hemicara izquierda, ligera protrusión y caída del lado izquierdo del maxilar inferior, por lo que se
realiza una tomografía donde se halla una fractura de la apófisis coronoides del maxilar inferior.
¿Qué músculo está relacionada directamente con esta situación?

EK
a. Masetero
b. Temporal
c. Buccinador
d. Pterigoideo medial
OT
22. Un paciente refiere no percibir algunos sabores. Al examen físico constata alteración del sabor dulce
y umami. ¿Cuál de los siguientes nervios estará alterada su función?
a. Cuerda del tímpano (VII par)
b. Lingual (rama del V par)
NC

c. Glosofaríngeo (IX par)


d. Hipogloso (XII par)

23. A los pocos días de nacido, regresa a neonatología un niño con problemas de motilidad del colon; los
estudios determinan ausencia congénita de células ganglionares. Según el gráfico ¿cuál es la capa en
BA

la que se determina la ausencia de dichas células?


a. Mucosa - 2
b. Muscular propia - 2
c. Muscular propia- 3
d. Muscular de la mucosa -3

1 2

24. Con respecto al control autonómico en el tracto gastrointestinal y en relación a su fisiología. ¿Cuál
es la función del sistema nervioso parasimpático en el tracto gastrointestinal?
a. Inhiben la contracción muscular y estimulan la secreción de sustancias a nivel de la
submucosa
b. Estimulan la contracción muscular y estimulan la secreción de sustancias a nivel de la
mucosa
c. Inhiben la contracción muscular e inhiben la secreción de sustancias a nivel de la submucosa
d. Estimulan la contracción muscular e inhiben la secreción de sustancias a nivel de la mucosa

25. Un estudiante que está preocupado por su examen parcial, no ha desayunado ni almorzado; cuando
al fin ingiere alimentos, esto le provoca el aumento de los movimientos musculares del tracto
gastrointestinal y la sensación de defecar. ¿Qué reflejo se ha activado?
a. Entero-gástrico
b. Gastro-cólico
c. Cólico-ileal
d. Ileo-ileal

A
26. ¿De qué par craneal es rama el nervio palatino mayor?
a. Vago

EK
b. Hipogloso
c. Trigémino
d. Palatogloso

27. ¿En cuál de las fases de la deglución la epiglotis separa la vía respiratoria de la digestiva? a. oral
OT
b. laríngea
c. faríngea
d. esofágica

28. Los péptidos intestinales se pueden clasificar como sustancias endocrinas, neurocrinas y paracrinas,
NC

dentro de las paracrinas se encuentran la somastotatina e histamina. Marque la respuesta correcta


a. La somastotatina es sintetizada por las células B de la mucosa gástrica
b. La histamina actúa estimulando su receptor tipo H1 en la mucosa gástrica
c. La histamina es sintetizada por células de tipo paracrino de las glándulas gástricas
BA

d. La somatostatina presenta dentro de sus funciones la estimulación de la secreción de H +

29. En relación a los órganos intraabdominales y sus estructuras de fijación, elija la alternativa correcta
FFF
a. El mesenterio permite la suspensión e irrigación de los órganos retroperitoneales
b. Tanto el hígado como la vesícula biliar se encuentran ubicados a nivel del flanco derecho
c. El colon, el duodeno y el resto de intestino delgado son órganos considerados netamente
como peritoneales
d. Los ligamentos que encontramos dentro de la cavidad abdominal son el esplenorenal y el
gastrofrénico
e. Los omentos van desde el estómago y la segunda porción del duodeno a otras estructuras
intraabdominales y existen dos: el omento mayor y el omento menor

30. Paciente varón de 27 años es llevado por bomberos a emergencia luego de ser asaltado y, tras
resistirse, es cortado con el pico de una botella a nivel abdominal. Al examen físico usted observa
que a través de la herida se puede observar la protrusión de asas intestinales. En relación con las
capas de la pared abdominal, marque la alternativa correcta.
a. La fascia de Scarpa está constituida principalmente por tejido adiposo
b. La pared abdominal está formada por piel, huesos, músculos, fascias y peritoneo parietal
c. La fascia de Camper es una estructura fibrosa que carece de grasa y su grosor es constante
en toda la pared abdominal
d. El músculo oblicuo externo discurre en dirección súpero-interna y se inserta en el borde
inferior de las ultimas 3 a 4 costillas
e. El músculo recto del abdomen tiene como funciones comprimir el contenido del abdomen,
tensar la pared del abdomen y flexionar la columna

Final 2019-01

1. La explicación fisiológica de presentar somnolencia de 30 minutos a 1 hora después de ingerir


alimentos, se explica por:
a. Aumento del cloro intraluminal
b. Aumento del bicarbonato intraluminal
c. Disminución de ácido carbónico en la célula parietal
d. Disminución de la actividad de la anhidrasa carbónica

A
e. Aumento de la alcalinidad sanguínea

EK
2. Con respecto a la irrigación arterial del colon, a que arteria corresponde la
señalada con la flecha
a. Cólica derecha
b. Cólica media
c. Cólica izquierda
OT
d. Ileobisecoapendículocólica
e. Arco de Riolano

3. Si un paciente presentara dentro del punto de vista fisiológico, una disminución de enterocinasa,
NC

entonces esto originaría una disminución de la actividad de:


a. la pepsina
b. la lipasa
c. la quimotripsina
d. el peptido insulinotropo dependiente de glucosa
BA

e. la amilasa

Se valida la opción b debido a su relación con la colipasa.

Se valida la opción a debido a la ubicación del número 1 en


donde se unen el
ligamento falciforme y ligamento
coronario.
4. Con respecto a la anatomía del hígado, señale a que estructura
pertenece la marcada por el número 1.
a. Ligamento falciforme
b. Línea de Cantlie
c. Ligamento triangular
d. Ligamento coronario
e. Ligamento teres

5. Se presenta un paciente, el cual presenta un antecedente de tuberculosis intestinal, por lo cual, se


le resecó 80 cm de íleon distal. Desde el punto de vista fisiológico, el paciente puede presentar una
de las siguientes alteraciones:
a. Disminución de la secreción de Vitamina B12
b. Aumento indiscriminado de absorción de ácido fólico
c. Disminución de la absorción de hierro
d. Aumento de la secreción de bicarbonato
e. Disminución de la absorción de ácido glicocólico

6. Un paciente es sometido experimentalmente a un fármaco que modifica el flujo salival,


obteniéndose un volumen de saliva de 288 ml en 6 horas. En este caso las concentraciones de
electrolitos y bicarbonato en la saliva obtenida varían de la siguiente manera:
a. ↑ Na+, ↓ K+, ↑ Cl-, ↑ HCO3-
b. ↓ Na+, ↓ Cl-, ↑ K+, ↓ HCO3-
c. ↑ Na+, ↑ Cl-, ↓ K+, ↓ HCO3-
d. ↑ Na+, ↑ Cl-, ↑ K+, ↑ HCO3-

A
e. ↓ Na+, ↓ Cl-, ↓ K+, ↓ HCO3-

EK
Se valida la opción a debido a que se puede considerar como un aumento del flujo de saliva.

7. La siguiente imagen histológica corresponde a la


glándula
…………… y la estructura señalada produce ………
OT
a. salival sublingual / mucopolisacáridos
b. oxíntica / pepsinógeno
c. salival submaxilar / ptialina
d. salival parótida / amilasas
e. antrales / gastrina
NC
BA

8. Paciente varón de 65 años con antecedente de hipercolesterolemia, hipertensión arterial,


fibrilación auricular y dos infartos al miocardio previos, aqueja de dolor abdominal intenso de inicio
súbito, distensión abdominal, se decide cirugía con resección de 1,5 metros de intestino delgado
terminal y colon ascendente. Como consecuencia de la resección el paciente tendrá deficiencia de:
a. Vitamina C
b. Tiamina
c. Vitamina A
d. Vitamina B1
e. Vitamina B6

Se valida esta opción debido a que su absorción está relacionada al íleon.

9. Uno de los siguientes elementos debería hallarse con más probabilidad en el esófago de un
paciente que sufre de reflujo gastro esofágico…
a. Pepsina
b. Tripsina
c. Quimiotripsina
d. Carboxipeptidasa
e. Ácidos biliares
10. Un paciente de 40 años cursa con anemia de 8g/dl, aqueja además de astenia y sensación de
hormigueo bilateral en los miembros inferiores, al examen se halla alteración de la sensibilidad a la
vibración y camina con ampliación de la base de sustentación. Uno de los siguientes
procedimientos sería de ayuda para el diagnóstico de este paciente:
a. Tomografía cerebral
b. Biopsia de la mucosa gástrica
c. Biopsia de hígado
d. Examen de sangre oculta en heces
e. Biopsia de Ileon proximal

11. Paciente de 60 años ingresa por caída hace 1 hora y pequeño hematoma en cuero cabelludo, al
examen físico ampliado se observa ictericia de piel y mucosas generalizada, abdomen blando, se
palpa estructura quística no dolorosa en hipocondrio derecho que corresponde a vesícula biliar

A
(signo de Courvoisier), en los exámenes de laboratorio se halla niveles bajos en la formación de
estercobilinógeno y urobilinógeno en heces, incremento de la bilirrubina conjugada en la orina,
elevación de fosfatasa alcalina y gamma glutamil transpeptidasa séricas. El presente cuadro puede

EK
ser explicado por:
a. Reabsorción de hematoma
b. Litiasis vesicular
c. Carcinoma de la cabeza de páncreas
OT
d. Carcinoma con estenosis del conducto hepático común
e. Anemia hemolítica

12. Paciente varón de 58 años con antecedente de alcoholismo crónico es diagnosticado y recibe
tratamiento por cirrosis hepática. Hace 2 días refiere familiar que tuvo cambio de conducta y no
NC

reconoce a algunos familiares. Al examen físico, se halla ascitis, circulación colateral en abdomen,
telangiectasias, en el examen de sistema nervioso: rigidez de extremidades, ROT incrementados,
desorientación en el espacio y asterixis. ¿cuál de las siguientes circunstancias, explicaría el cuadro
en este paciente?
a. Uso de diuréticos ahorradores de potasio
BA

b. Incremento de actividad de ureasa bacteriana duodenal


c. Hemorragia gastrointestinal
d. Disminución de la producción de NH3+ en el colon
e. Dieta normo proteica

Se valida la opción e debido al efecto sobre la


encefalopatía.
Con respecto de la opción b es incorrecta debido a que hace referencia al duodeno,
debería indicar colon.

13. Un recién nacido presenta vómitos biliosos poco tiempo después de cada alimento. Al preguntar a
la madre sobre antecedentes, ella recuerda que tuvo polihidramnios durante la gestación, pero un
análisis de cariotipo fue normal. Una de las siguientes es la causa más probable de estos hallazgos
en el recién nacido:
a. Enfermedad de Hirschprung
b. Fístula tráqueo esofágica
c. Divertículo ileal
d. Estenosis pilórica
e. Malrotación de la yema pancreática ventral

14. Un lobulillo hepático se puede dividir en tres zonas como se muestra en el gráfico. ¿Cuál de las
siguientes afirmaciones sobre las tres zonas es verdadera?
a. La zona 1 tiene los menores depósitos de glucógeno
b. La zona 3 es la primera en afectarse en una colestasis extra hepática
c. La zona 2 es más susceptible a la injuria por isquemia que la zona periportal
d. La zona 2 tiene la mayor capacidad de regeneración
e. La zona 1 es la que tiene menos actividad metabólica.

La pregunta 14 ha sido anulada, sin embargo, ningún estudiante se verá afectado


negativamente en su
puntaje debido a esta
anulación.

15. En un estudio de la secreción de hormonas gastrointestinales, sus concentraciones en la vena porta


se midieron durante perfusión luminal del intestino delgado con soluciones de diversas magnitudes
de pH. ¿Qué hormona aumentará en el plasma de la vena porta durante perfusión a través del
intestino con una solución de pH 3? a. CCK
b. gastrina
c. GIP
d. motilina
e. secretina

16. Paciente de 30 años que ingresa a causa de un traumatismo abdominal cerrado. En la exploración
se aprecia discreta palidez de piel y mucosas, auscultación pulmonar normal, taquicardia de 120
/min. Discreta distensión abdominal y matidez en flancos; el hematocrito, que era prácticamente
normal al ingreso, disminuye a 30% a las tres horas. En la Rx de tórax se objetiva fractura de las
costillas 10-11 izquierdas. La causa más probable de la anemización en este paciente es:
a. traumatismo renal con hemorragia retroperitoneal.
b. rotura de hígado con hemoperitoneo.
c. rotura de bazo con hemoperitoneo.
d. rotura de mesos con hemoperitoneo.
e. traumatismo pancreático con pancreatitis traumática.

17. Mujer de 65 años. Consulta por síndrome constitucional asociado a dolor abdominal epigástrico
progresivo irradiado a espalda, de dos meses de evolución. El diagnostico de sospecha de
adenocarcinoma de páncreas se confirma por biopsia. Se realiza examen de imagen de abdomen
para evaluación de estructuras vasculares próximas al tumor pancreático. ¿Cuál es el nombre de la
vena señalada que está ausente, trombosada por infiltración
tumoral, condicionando circulación colateral en la pared
gástrica?

a. Mesentérica superior
b. Coronaria estomaquica
c. Esplénica
d. Porta
e. Renal izquierda

18. Revisando la angiotomografía de un hombre de 70 años en estudio por aneurisma de aorta


abdominal, el radiólogo le informa de la presencia de una oclusión completa de la arteria
mesentérica inferior. El paciente se encuentra completamente asintomático. La oclusión de la
arteria mesentérica inferior cursa de manera asintomática en muchas ocasiones ya que el territorio
que irriga puede recibir flujo proveniente de la arteria: a. cólica derecha
b. gastroduodenal
c. Epigástrica inferior izquierda
d. esplénica
e. cólica media

19. En las patologías de esófago es importante conocer bien la anatomía esofágica. ¿Cuál de las
siguientes afirmaciones es correcta?
a. El esófago tiene capa mucosa, muscular y serosa
b. El esófago abdominal es más largo que el cervical
c. El esófago torácico pasa por detrás del cayado aórtico
d. El epitelio esofágico normal es de tipo cilíndrico.
e. El esófago abdominal es discretamente más largo que el torácico

20. A pesar de que pueda haber variaciones anatómicas, lo habitual es que el ciego sea irrigado por una
rama arterial que proviene de unas de las siguientes arterias:
a. Iliaca derecha
b. Mesentérica inferior
c. Hepática derecha
d. Mesentérica superior
e. Iliaca izquierda

21. Ante un paciente con una cirugía abdominal urgente, el informe operatorio señala que se ha
realizado una resección de todo el duodeno y del tercio proximal del yeyuno manteniendo íntegros
el estómago y todo el íleon, así como los dos tercios distales del yeyuno. En el seguimiento
nutricional del paciente ¿Qué vitamina o mineral presentará con menor probabilidad una
disminución de su absorción?
a. Cianocobalamina
b. Calcio
c. Hierro
d. Transcobalamina
e. Transferrina
22. ¿Cuál de las siguientes alternativas detallan las venas que
confluyen y forman la vena señalada?
a. mesentérica superior, gástrica izquierda y gastroepiploica
izquierda
b. mesentérica inferior, gástrica izquierda y renal
c. esplénica, mesentérica superior y mesentérica inferior
d. esplénica, pancreatoduodenal y omental izquierda
e. gástrica izquierda, esplénica y hepática común

23. ¿Cuál de las siguientes sustancias forma parte de la secreción biliar?


a. Tripsina
b. Lecitina
c. Elastasa
d. Quimotripsina
e. Pepsina

24. El tubo digestivo contiene diferentes tipos de epitelios y glándulas. La estructura señalada es una
………………. y está localizada en el …………...
a. glándula de Brunner / intestino grueso
b. cripta de Lieberkuhn / colon
c. cripta de Lieberkuhn / intestino delgado
d. glándula oxintica / estomago
Aunque las criptas de Lieberkuhn
están
presentes en el intestino
delgado, la
microfotografía es de epitelio de
colon.
e. célula parietal / estómago

25. ¿De qué musculo forma parte el ligamento inguinal?


a. Oblicuo externo del abdomen
b. Oblicuo interno del abdomen
c. Transverso del abdomen
d. Psoas
e. Dorsal ancho

26. Señale cuál de las siguientes afirmaciones NO se relaciona a la siguiente glándula anexa del tubo
digestivo mostrada en la imagen:
a. Es una glándula exocrina compuesta exclusivamente por acinos serosos
b. Su inervación está dada por el nervio auricular mayor (ramo posterior C2), que inerva la
vaina de la glándula así como la piel por encima de esta.
c. Esta glándula produce una secreción mucinosa acuosa, llamada mucoserosa, a través del
conducto de Wharton.
d. Su inflamación puede ser causada por un virus de los Paramyxoviridae, que provocan una
enfermedad muy frecuentemente en niños y adolescentes
e. Es una glándula endocrina y probablemente sea de origen pancreático

Se valida la opción e debido a que no está


relacionada
con la
imagen.

27. ¿Cuál de las siguientes enzimas está localizada en el borde en cepillo y juega un rol en la digestión
de proteínas? a. Alfa dextrinasa
b. Pepsina
c. Enterocinasa
d. Lactasa
e. Carboxipeptidasa A.

Se valida la opción c debido a que es correcta en relación a la pregunta.

28. Una de los siguientes sustancias, NO sirve como un buen agente emulsificante: a. Colesterol
b. Ácidos grasos
c. Sales biliares
d. Lecitina
e. Proteínas de la dieta

Se valida la opción e debido a que es correcta en relación a la pregunta.

29. La sustancia que estimula el crecimiento de la mucosa gástrica es:


a. Secretina
b. Motilina
c. Péptido estimulante de la mucosa gástrica
d. Gastrina
e. Histamina

30. ¿Cuál de las siguientes alternativas es una función de la colecistokinina?


a. Relajación de la vesícula para la salida de bilis
b. Secreción de ácidos biliares
c. Contracción del esfinter de Oddi
d. Secreción de enzimas pancreáticas
e. Contracción del duodeno

Se valida la opción b debido al efecto de la CCK sobre la vesicula biliar.

31. Con respecto a la anatomía del tronco celiaco, señale lo correcto


a. El tronco celiaco se origina de la cara posterior de la aorta abdominal
b. Es una arteria delgada que tiene un calibre entre 2 y 3 mm
c. Una de sus ramas es la arteria gástrica derecha
d. La hepática común que es una de sus ramas, participa en la irrigación del estómago.

32. Con respecto a la anatomía del duodeno, marque la respuesta correcta:


a. Tiene una distribución en forma de “C”, que rodea la cola del páncreas
b. La 3ra porción duodenal está contenida en la pinza vascular aortomesentérica
c. Entre la 1ra y 2da porción se forma un ángulo, conocido como el ángulo de Treitz
d. La 4ta porción se dirige a la izquierda, hacia abajo y hacia atrás.
e. En la tercera porción desemboca el conducto colédoco.

33. El hígado está ampliamente tapizado por peritoneo, la estructura que conecta la cara diafragmática
del hígado precisamente con el diafragma es el ligamento:
a. teres
b. falciforme
c. triangular
d. hepático común
e. coronario

Se validan la opción c y e debido a que forman parte de los ligamentos que fijan el hígado al diafragma.

34. En el íleon se absorbe aproximadamente el 95% de …………………. a través de la circulación


enterohepática. a. agua
b. colesterol
c. sales biliares
d. hidróxicobalamina
e. factor intrínseca

35. La ……………. estimula el mecanismo paracrino de la secreción de ácido clorhídrico.


a. histamina
b. acetilcolina
c. gastrina
d. secretina
e. somatostatina

36. En la digestión de proteinas, ……………. es el principal estímulo para convertir el pepsinógeno en


pepsina. a. la gastrina
b. el pH ácido
c. la acetilcolina
d. la ptialina
e. la somatostatina
37. Con respecto a la somatostatina, marque lo correcto:
a. Es secretada por las células S del intestino
b. Induce a la producción de VIP
c. Interviene en la fase intestinal de la secreción gástrica
d. Produce acetilcolina para estimular a la célula parietal
e. No interviene en la regulación de la secreción de ácido clorhídico

38. En pecten anal, es una estructura comprendida entre:


a. la línea pectínea y los senos anales
b. la línea blanca y la apertura anal
c. el esfínter anal interno y el externo
d. la línea anocutánea y la línea pectínea
e. la línea blanca y columnas anales

39. ¿Cuál de las siguientes alternativas es una proenzima pancreática?


a. Tripsina
b. Elastasa
c. Quimotripsinógeno
d. Amilasa
e. Procarboxipepitidasa C.

40. En la segmentación hepática de Coinaud, el segmento hepático señalado con la flecha, corresponde
a : En la segmentación hepática de Coinaud, la flecha señala el segmento ………….. hepático.
a. IV b. V

c. VI
d. VII
e. VIII

Parcial 2019-02

1. La contracción del músculo ………………………… permite la eliminación de gases (flatos) sin salida de
material fecal; es el mismo músculo cuya relajación, sobretodo en cuclillas, permite el paso del
contenido fecal con menor esfuerzo durante la defecación.
a) Isquirectal
b) Puborrectal
c) Esfínter anal externo
d) Esfínter anal interno
2. Paciente mujer de 54 años se presenta con náuseas, vómitos, estreñimiento, y es diagnosticada de
abdomen agudo quirúrgico; en la cirugía encuentran un vólvulo de ciego. Esta anomalía puede
explicarse por:: a) Falta de rotación intestinal
b) Falta de fusión del mesenterio
c) Defecto en la formación de la cloaca
d) Falta de formación del omento mayor

3. Paciente mujer de 23 años con faringitis aguda, toma para el dolor una tableta de paracetamol con
un poco de agua. Durante la deglución, se relaja su esfínter esofágico inferior y el fondo del
estómago, mientras el bolo está aún en el esófago. ¿Qué sustancia provocara con mayor
probabilidad la relajación del esfínter esofágico inferior y el fondo del estómago en esta mujer?
a) Óxido nítrico
b) Sustancia P
c) Histamina
d) Motilina

4. Luego de tres horas dando exámenes, un alumno de medicina comienza a sentir hambre. Esta
situación es probable que sea mediada por la ______________ que es sintetizada por el
___________________: a) leptina / intestino
b) leptina / estómago
c) grelina / estómago
d) grelina / tejido adiposo

5. Varón de 72 años, con antecedente de diabetes mellitus tipo 2, que presenta enteropatía diabética
caracterizada por estreñimiento. Este problema puede estar asociado a:
a) deficiencia de óxido nítrico
b) aumento del reflejo gastrocólico
c) disminución de la secreción de colecistocinina (CCK)
d) aumento de la secreción del péptido intestinal vasoactivo (PIV)

6. Varón de 54 años con Diabetes Mellitus tipo 2, es diagnosticado de gastroparesia debido a que
presenta sensación de llenura precoz al comer, y reflujo gastroesofágico. Esta alteración en la
relajación receptiva y en el vaciamiento gástrico lo más probable es que se deba a una alteración en:
a) el nervio vago
b) el ganglio celíaco
c) plexo submucoso
d) nervio hipogástrico

7. Varón de 67 años con tos y disminución de peso asociado a tabaquismo pesado, presenta
actualmente disfagia progresiva a alimentos sólidos. Se considera la presencia de un carcinoma de
bronquio izquierdo y por esta razón le realizan una endoscopía esofágica para descartar la
posibilidad de una compresión esofágica por el tumor. Se espera revisar el esófago en la
__________________ estrechez, que está a nivel de la vértebra ______________ a) Tercera
estrechez -T6
b) Segunda estrechez - C6
c) Segunda estrechez - T4
d) Tercera estrechez -T10

8. Varón de 34 años con dolor abdominal agudo en flanco derecho que se irradia a fosa ilíaca derecha,
es operado y se encuentra un divertículo intestinal inflamado, ubicado a 93 cm de la válvula
ileocecal. El origen de este divertículo es una falla en la obliteración de:
a) Conducto vitelino
b) Alantoides
c) Cloaca
d) Conducto anorectal
e) Uraco

En un niño menor de dos años con divertículo intestinal, este divertículo tiene su origen en una falla
en la obliteración de:
a) Conducto anorectal
b) Conducto vitelino
c) Alantoides
d) Cloaca
e) Uraco

9. Mujer de 43 años sufre un grave accidente de tránsito y está hospitalizada en coma, es alimentada
por vía intravenosa durante varias semanas. Producto de este tipo de alimentación, se encuentra en
la endoscopía atrofia de la mucosa gastrointestinal. La causa más probable de esta atrofia son los
bajos niveles séricos de la hormona: a) Colecistocinina
b) Secretina
c) Gastrina
d) PIV

10. Una mujer de 30 años llega al consultorio porque se queja de dificultades para deglutir, la cual se
agravan cada vez más. Se realiza un estudio manométrico para examinar la generación de presión a
lo largo del esófago. Esta prueba revela que las contracciones como respuesta a la deglución están
mal sincronizadas y que la presión en el esfínter esofágico inferior permanece elevada. El
diagnóstico más probable es ____________________ producido por niveles bajos de
____________________ a) acalasia / sustancia P
b) acalasia / óxido nítrico
c) enfermedad por reflujo gastrointestinal / acetilcolina
d) enfermedad por reflujo gastrointestinal / óxido nítrico

11. Paciente de 2 años, llega a emergencia por haber ingerido una moneda con la que estaba jugando.
El lugar más probable donde puede haberse quedado suspendido este objeto es a nivel del
estrechamiento producido a nivel del:
a) músculo milohiodeo
b) músculo aritenoideo
c) músculo cricofaríngeo
d) constrictor superior de la faringe

12. En una apendicectomía, al realizar la incisión de McBurney en la fosa iliaca derecha, es necesario
cortar los siguientes músculos, de afuera hacia adentro:
a) Recto – Oblicuo externo – Transverso
b) Recto – Oblicuo externo – Oblicuo interno
c) Oblicuo externo – Oblicuo interno – Recto
d) Oblicuo externo – Oblicuo interno – Transverso

13. Un varón de 90 años que se encuentra postrado en cama, es referido del asilo para endoscopia por
dificultad para deglutir luego de tomar un medicamento para aliviar el dolor la noche anterior. La
endoscopía revela que la píldora se alojó en el esófago y causó una reacción inflamatoria. Lo más
probable es que esto haya sido por la producción de múltiples ondas:
a) secundarias
b) primarias
c) lentas
d) segmentarias

14. Mujer de 23 años es diagnosticada de bulimia, al examen físico se observa ulceraciones en el


segundo y tercero dedo de la mano derecha. Esto se puede deber al uso continuo de estos dedos
para inducir el vómito, mediante la estimulación del par craneal:
a) V
b) IX
c) X
d) XI

15. Varón de 52 años se presenta por diarrea persistente de seis semanas de duración. En la
colonoscopia se observa un pólipo a nivel del íleon distal. El patólogo informa que se trata de un
tumor neuroendócrino, probablemente originado por las células enterocromafines del intestino. La
sustancia que más probablemente esté produciendo este tumor es:
a) Serotonina
b) Insulina
c) CCK
d) GIP

16. La fase oclusal de la masticación se realiza con la contracción de los músculos: a) digástricos
b) masetero y temporal
c) orbicular y buccinador
d) pterigoideo lateral y digástrico

17. Al tomar su café en Starbucks, un estudiante de medicina sufre una quemadura de primer grado en
el tercio anterior de la superficie dorsal de la lengua. La información de dolor es transmitida por el
nervio: a) cuerda del tímpano
b) glosofaríngeo
c) lingual
d) facial

18. Paciente es evaluado por faringitis aguda en consultorio externo. El médico de familia le solicita que
abra la boca y saque la lengua. Para realizar la acción de sacar la lengua, es necesario que se
contraiga el músculo: a) estilogloso
b) geniogloso
c) palatogloso
d) transverso de la lengua

19. Paciente con síndrome de Sjögren, presenta “boca seca” (disminución de


la producción de saliva) y caries dental, asociada a la pérdida de la función
de tampón de la saliva. Esta desminerilización del diente puede
comprometer a las prolongaciones citoplasmáticas ubicadas en los tubos
huecos de la estructura señalada con la letra:
a) B
b) A
c) E
d) C

20. Mujer de 32 años acude a consulta por presentar disfagia de progresión


lenta, reflujo gastroesofágico y vómitos desde hace 3 meses de evolución
progresiva. Se le realiza un estudio radiológico con contraste en el que se
observa estrechamiento del esfínter esofágico inferior (imagen). Según sus
conocimientos, este paciente se beneficiaría con el uso de: a) agonista
beta adrenérgico
b) agonista alfa adrenérgico
c) análogo de óxido nítrico
d) análogo de Sustancia P

Final 2019-02

1) La reabsorción de sodio y cloro en las glándulas salivales se da principalmente en el:


a) conducto intercalado
b) conducto excretor
c) conducto estriado
d) acino glandular

2) Paciente de 35 años con dolor en hipocondrio derecho irradiado a la punta de la escápula. Se observa
en la ecografía abdominal cálculos en la vesícula biliar. Es operado de emergencia realizando una
incisión a lo largo del reborde costal derecho. La información de dolor de esta zona es inervada por
las raíces nerviosas: a) T5 – T9
b) T6 – T7
c) T7 – T8
d) T9 – T10

3) Paciente de 23 años con dolor intenso periumbilical de inicio agudo. Es ingresado a sala de
operaciones por abdomen agudo, el cirujano encuentra sangrado a unos 60 cm proximal a la válvula
ileocecal. La estructura que está sangrando muy probablemente es un derivado embriológico del:
a) uraco
b) ligamento Hepatoduodenal
c) alantoides
d) conducto vitelino

4) Paciente de 64 años de edad con dolor en epigastrio que se distribuye en banda hacia la espalda. En
la tomografía se encuentra tumor en cabeza de páncreas que compromete a un vaso que discurre
entre la cabeza del páncreas y el proceso uncinado. Debido a esto, este cáncer se considera
irresecable porque compromete a la arteria: a) mesentérica superior
b) mesentérica inferior
c) tronco celíaco
d) aorta

5) Paciente con carcinoma gástrico avanzado, en preoperatorio para gastrectotomía total. ¿Cuál de los
siguientes ganglios linfáticos recibirá primero células metastásticas con mayor probabilidad: a)
celíaco
b) cisterna del quilo
c) esplénico
d) gástrico posterior
6) Paciente de 77 años con dolor abdominal difuso de 3 horas de evolución. En los estudios de imágenes
se observa oclusión probablemente aterosclerótica de la arteria mesentérica superior; no se observa
necrosis, lo cual puede ser explicado por la irrigación sanguínea colateral. ¿Qué vasos ofrecen
colaterales entre el tronco celíaco y la arteria mesentérica superior?
a) Gástrica izquierda y hepática
b) Cística y duodenal
c) Gastroomental derecha e izquierda
d) Pancreaticoduodenal superior e inferior

7) Paciente de 62 años con vólvulo de intestino delgado e isquemia intestinal. Se realiza laparotomía
exploratoria para liberar la obstrucción. ¿Cuál estructura se utilizará como punto de referencia para
determinar la posición de la unión duodenoyeyunal?
a) Vasos rectos
b) Ligamento suspensorio del duodeno (de Treitz)
c) Ligamento frenocólico
d) Nacimiento de la Arteria mesentérica superior

8) Paciente de 42 años con dolor abdominal intenso y hematemesis. En la endoscopía se observa una
úlcera duodenal posterior perforada con hemorragia intrabdominal. ¿Cuál de las siguientes arterias
estará comprometida? a) Gástrica izquierda
b) Mesentérica superior
c) Gástrica derecha
d) Pancreaticoduodenal posterosuperior

9) Paciente de 51 años con antecedente de enfermedad diverticular acude a emergencia por sangrado
profuso y dolor en cuadrantes inferiores. ¿Cuál es el origen más probable de la sangre que pierde en
la hemorragia? a) Cólica media
b) Mesentérica inferior
c) Rectal superior
d) Rectal inferior

10) Paciente de 13 días de vida con vómitos explosivos a las dos horas después de lactar. Al examen físico
se palpa la oliva pilórica. ¿Cuál es el nervió cuyos filetes dan inervación eferente a la estructura
afectada? a) Vago
b) Esplácnico torácico mayor
c) Mesentérico superior
d) Esplácnico torácico menor

11) Paciente de 80 años con dolor abdominal intenso y antecedente de estreñimiento crónico. En la
colonoscopía se observa divertículos con áreas ulceradas difusas en colon sigmoides y descendente.
El cirujano programa una cirugía de resección y planifica que para realizar esta resección tendrá que
cortar los siguientes vasos y nervios: a) plexo mesentérico superior y arteria rectal.
b) nervio esplácnico torácico mayor y arteria cólica media.
c) ramas del nervio vago y arteria ileocólica.
d) nervio esplácnico pélvico y artera cólica izquierda.

12) Paciente de 46 años ingresa a emergencia con dolor en cuadrante superior derecho e ictericia. En la
ecografía se observa cálculos en la vesícula biliar. ¿Cuál de los siguientes nervios transmite el dolor
de la colecistitis?
a) Filetes aferentes del nervio vago derecho, referido al ángulo inferior de la escápula
b) Filetes nerviosos de las raíces T1 a T4, con referencia al hombro izquierdo
c) Fibras aferentes simpáticas del nervio esplácnico torácico mayor, con referencia a los
dermatomas T6 a T8
d) Fibras aferentes de los ramos primarios dorsales de los nervios raquídeos T8 a T10 con
referencia al epigastrio

13) Un niño de 8 años es alimentado por sus padres con un Cheeseburguer, papitas fritas y un vaso de
Coca Cola. ¿La presencia de lípidos, carbohidratos y proteínas en el duodeno estimulan la liberación
de cuál de las siguientes hormonas en la mucosa duodenal?
a) Colecistoquinina (CCK)
b) Péptido inhibidor gástrico (GIP)
c) Secretina
d) Insulina

14) La fase cefálica del control de la secreción gástrica corresponde a cerca del 30% de la secreción ácida
y es consecuencia de un reflejo. ¿Cuál de las siguientes alternativas puede eliminar totalmente la
fase cefálica de la secreción gástrica?
a) Vaguectomía
b) Uso de atropina
c) Uso de bloqueador de receptor H2 de histamina
d) Uso de antiácidos

15) Recién nacido de 24 horas con llanto, vómitos y sin eliminación de meconio. Luego de los estudios
auxiliares se diagnostica la Enfermedad de Hirshprung, esta enfermedad se caracteriza por la
ausencia de:
a) las células de Cajal en plexo mioentérico
b) el plexo mioentérico en el recto y colon sigmoides
c) las células mioepiteliales en el recto distal
d) el plexo de Meissner en el recto distal y colon sigmoides

16) Los complejos motores migratorios aparecen aproximadamente cada 90 minutos entre las comidas,
y se considera que son estimulados por la hormona motilina. La ausencia de estos complejos
migratorios podría producir un aumento en:
a) la motilidad duodenal
b) el vaciamiento gástrico
c) la deglución
d) las bacterias intestinales

17) ¿Cuál de las siguientes es una consecuencia probable de la resección del íleon? a) Gastritis atrófica
b) Deficiencia de vitamina B12
c) Esteatorrea
d) Úlcera péptica

18) Los movimientos en masa son importantes en la fisiología intestinal. Estos movimientos en masa
ocasionan: a) la sensación de defecar
b) el peristaltismo duodenal
c) la retropulsión gástrica
d) la contracción del esfínter anal interno

19) La toxina colérica hace que aumente los niveles de AMPc intracelular, y este aumento hace que se
mantenga abierto un canal en las células de la cripta de Lieberkuhn. En condiciones fisiológicas, en
una persona sana ¿Qué sustancia puede promover que el canal quede abierto también?
a) Somatostatina
b) Óxido nítrico (NO)
c) Péptido intestinal vasoactivo (VIP)
d) Péptido similar al glucagón 1 (GLP1)

20) Paciente de 64 años con tumor abdominal que comprime la cisterna del quilo. En la biopsia de
duodeno tomada como parte del estudio, el patólogo puede observar
a) Dilatación del vaso quilífero central
b) Contracción de las venas de las vellosidades
c) Vellosidades intestinales más largas
d) Engrosamiento de la lámina basal
21) En la enfermedad de Crohn es posible encontrar células de Paneth en el colon. Esto se puede deber
a la especial función de estas células en:
a) la activación de la inmunidad adquirida
b) mantener la inmunidad innata
c) producir hormonas
d) producir Ig A

22) En 1967 se descubrió que la epidemia de Kuru, una enfermedad por priones, en el distrito de Okapa
en Papua Nueva Guinea, era causada por la costumbre de comer la carne de los muertos. Ahora se
sabe que las proteínas priónicas ingresan al organismo a través de:
a) los enterocitos
b) la transmigración
c) las células de Paneth
d) las células M

23) Paciente de 48 años con alteraciones en el tránsito intestinal por diabetes mellitus tipo 2; se presenta
con esteatorrea, flatulencia y malabsorción de grasas. Las pruebas de función hepática y biliar están
dentro de rangos normales. Una causa de la disminución de sales biliares puede ser:
a) el sobrecrecimiento bacteriano
b) la deficiencia de pepsina
c) la deficiencia de elastasa
d) la hiperestimulación del GLUT5
24) Al usar azúcar de mesa (sacarosa) para endulzar su café, el estudiante de medicina sabe que lo más
probable es que para su absorción tendrá utilizar el/los transportadore(s) ________________ que se
encuentran en la membrana apical de los enterocitos.
a) SGLT-1
b) GLUT2 y GLUT5
c) SGLT-1 y GLUT5
d) SGLT-1 y GLUT2

25) Una persona con la producción normal de lactasa; cada vez que toma leche, los productos de la
degradación de la lactosa por parte de la lactasa ingresarán al enterocito usando el/los
transportador(es) _____________: a) SGLT-1
b) GLUT2 y GLUT5
c) SGLT-1 y GLUT5
d) SGLT1 y GLUT2

26) En una persona sana, luego de una comida basada en carnes rojas cocinadas con baja cantidad de
sal, al enterocito pueden ingresar solamente:
a) aminoácidos
b) aminoácidos y dipéptidos
c) aminoácidos, dipéptidos y tripéptidos
d) aminoácidos, dipéptidos, tripéptidos y tetrapéptidos

27) Paciente con deficiencia congénita de procolipasa, sufre de esteatorrea cada vez que come comidas
ricas en grasas. En el estudio de composición de las heces, uno espera encontrar una cantidad
aumentada de: a) colesterol
b) triglicéridos
c) fosfolípidos
d) lisolecitina

28) Al tomar su café en Starbucks, un estudiante de medicina sufre una quemadura de primer grado en
el tercio anterior de la superficie dorsal de la lengua. La información de dolor es transmitida por el
nervio: a) cuerda del tímpano
b) glosofaríngeo
c) lingual
d) facial
29) Paciente de 32 años con esteatosis hepática no alcohólica. Se le realiza una biopsia hepática que
confirma la esteatosis; en el tejido se observan depósitos de lípidos en los hepatocitos, los cuales
contienen principalmente: a) triglicéridos
b) colesterol
c) ácidos grasos libre
d) acil-carnitina

30) Paciente de 21 años que es estudiante de medicina, llega a consulta refiriendo que en épocas de
exámenes su piel se vuelve amarillenta. Le realizan el diagnóstico de Síndrome de Gilbert asociado a
mutación del gen UGT1A1, luego de unas semanas acude a la consulta por ictericia asociada a resfrío.
Al hacerle un análisis de sangre, usted sabe que encontrará valores elevados de:
a) bilirrubina directa
b) bilirrubina indirecta
c) bilirrubina tipo delta
d) fosfatasa alcalina
31) Se realizó un experimento en el cual se inyectó tinta china en el peritoneo de ratas de laboratorio. Al
realizarse una biopsia hepática de dichos animales, se encontró que el tinte negro estaba depositado
en: a) las células de Ito
b) los hepatocitos
c) las células de Kupffer
d) las células de Disse

32) Paciente de 74 años de edad con shock hipovolémico asociado a deshidratación aguda severa. En
este paciente es posible encontrar hipoxia en zona …… del lobulillo hepático e infarto ……………… del
intestino. a) 1 / mucoso
b) 1 / transmural
c) 3 / transmural
d) 3 / mucoso

33) Niño de 5 años con historia de tres días de evolución caracterizado por fiebre, malestar general,
odinofagia, hiporexia, e irritabilidad. Al examen se observa lesiones ulcerativas de 4 mm de diámetro
en mucosa yugal, con fondo blanquecino y eritema periférico. El diagnóstico más probable es:
a) candidiasis oral
b) leucoplasia
c) aftas
d) herpes

34) Paciente de 52 años con enfermedad por reflujo gastroesofágico de 30 años de evolución. Se realiza
endoscopía en la que se encuentra mucosa eritematosa proximal a la línea Z. Para corroborar la
presencia de lesión preneoplásica, se toma una biopsia de esa zona, en la que se espera encontrar:
a) metaplasia gástrica
b) metaplasia intestinal
c) displasia gástrica
d) adenocarcinoma

35) Un estudiante de medicina termina su último examen final. Al abrir la boca para comer una
hamburguesa con papas fritas, la activación de los receptores muscarínicos de las células acinar y
ductal estimularan un mayor flujo de saliva, con lo cual disminuirá la concentración salival de:
a) potasio
b) bicarbonato
c) sodio
d) cloro
36) Un estudiante toma su desayuno consistente en un pan con mantequilla y queso. Antes que se
puedan digerir las grasas, es necesario que sean emulsificadas. La hormona ………………………. estimula
la liberación de las sustancias emulsificadoras.
a) colecistoquinina (CCK)
b) secretina
c) lipasa pancreática
d) gastrina

37) Al comer un pollo a la brasa entero, con papitas fritas y ensalada, la sustancia que estimulará la
liberación de HCl en el estómago es:
a) el neuropeptido Y
b) la secretina
c) la bombesina
d) la colecistoquinina (CCK)
38) Paciente con disminución marcada del apetito asociado a cáncer terminal, se podría utilizar análogos
de ……………..
para promover la ingesta de alimentos.
a) el péptido similar al glucagón (GLP)
b) la serotonina
c) la secretina
d) la endorfina

39) Un hombre de 51 años presenta de forma súbita vómitos masivos de sangre roja brillante. Tiene
como antecedente hepatitis viral B hace 23 años. En la exploración física: FC 103 latidos/min, PA
85/50 mmHg, se palapa la punta del bazo y niega vómitos. Su hematocrito es 21%, la prueba
serológica de HBsAg es positiva. En la ecografía se observa hígado con nodulaciones. ¿Cuál es la causa
más probable para la hematemesis?
a) Esófago de Barrett
b) Síndrome de Mallory Weiss
c) Varices esofágicas
d) Esofagitis por reflujo

40) Durante el reflejo del vómito, uno de los primeros sucesos es:
a) el peristaltismo inverso
b) la contracción del píloro
c) la apertura de la glotis
d) el esfínter esofágico inferior abierto

41) Paciente de 34 años que acude por diarrea desde hace 4 días asociado a comer papa rellena con ají
en el Estadio Nacional, se exacerba cuando toma lácteos o come grasas. Se acompaña de flatulencia
y episodios de tenesmo. Este cuadro de diarrea e intolerancia a la lactosa de inicio agudo se explica
por la:
a) enfermedad de Crohn
b) infección por V. cholera
c) colitis ulcerativa
d) giardiasis

42) En cuanto a la secreción pancreática, mientras mayor es el flujo, mayor es la concentración de: a)
potasio
b) bicarbonato
c) cloro
d) sodio

43) Paciente de 10 años con diarrea crónica, distensión abdominal, anorexia. Se le ha encontrado
anticuerpos antigliadina y antiendomisio. Es más probable que la diarrea se correlacione con el
hallazgo histológico de: a) adelgazamiento de las criptas
b) linfocitos intramusculares
c) atrofia de las vellosidades
d) úlceras duodenales

44) La glándula parótida está inervada por el par craneal:


a) VII
b) IX
c) X
d) XII
45) Las glándulas de Brunner se encuentran en la:
a) mucosa del esófago
b) submucosa del íleon
c) mucosa del yeyuno
d) submucosa del duodeno

46) ¿Cuál de las siguientes afirmaciones describe correctamente la función de la inervación


parasimpática del tracto gastrointestinal?
a) La norepinefrina es el principal neurotransmisor excitatorio.
b) La actividad parasimpática produce la relajación de los esfínteres
c) La actividad parasimpática excesiva puede provocar un trastorno llamado ileo paralítico
(parálisis del músculo liso intestinal)
d) El pH luminal, la osmolaridad y la distensión muscular son detectados por fibras
parasimpáticas eferentes

47) Al ingerir una cantidad de glucosa por vía oral, esta es interiorizada en las células del organismo más
rápido que si esa misma cantidad de glucosa hubiese sido administrada por vía endovenosa. Este
fenómeno sucede gracias a la acción de la sustancia secretada por las células:
a) G
b) I
c) K
d) S

48) A …………… secreción de HCl en el lumen gástrico, …………….. pH en la sangre venosa gástrica a) mayor
/mayor
b) mayor / menor
c) menor / mayor
d) mayor / igual

49) ¿En cuál de las siguientes situaciones hay un menor flujo de secreción salival?
a) Masticar goma de mascar
b) Imaginarse ser sometido a un examen dental
c) Exposición a olor nauseabundo
d) Sueño
50) Al seccionar el nervio facial a nivel timpánico, usted esperaría:
A. Disminución del gusto en la punta de la lengua
B. Imposibilidad para el cierre del istmo de las fauces
C. Ausencia de la termoalgesia de la lengua
D. Imposibilidad para protruir la lengua
51) En un paciente con hiperestimulación simpática se espera que las ondas lentas tengan un ritmo:
A. Menor en íleon terminal que en el duodeno
B. Mayor en el íleon que el duodeno
C. Mayor en el estómago que el duodeno
D. Mayor en el estómago que en íleon terminal
52) Marque lo correcto en relación al divertículo de Meckel.
A. Se encuentra usualmente a 60 cm de la VIC
B. Contiene mucosa esofágica en algunas ocasiones
C. Se produce en el lado del íleon
D. Se relaciona un defecto en el desarrollo del intestino posterior
53) Al disminuir el pH duodenal por el HCl gástrico, se libera principalmente una hormona cuya célula
diana es :
A. Células S del intestino
B. Células ductales del colédoco
C. Acino pancreáticos
D. Células ductal del Wirsung
54) El ecografista sabe que para poder visualizar el nacimiento de la arteria mesentérica superior, debe
colocar el transductor sobre la piel de la siguiente región abdominal:
A. Epigastrio
B. Hipogastrio
C. Hipocondrio derecho
D. Mesogastrio
55) Durante una cirugía oncológica, el cirujano observa que los órganos abdominales tienen libre
movimiento dentro de la cavidad abdominal, excepto:
A. Vesícula biliar
B. Yeyuno
C. Estómago
D. Colon ascendente
56) Paciente obeso con Covid-19 es intubado por interno inexperto, quien al solicitar que bombeen aire
dentro del tubo endotraqueal, nota que el epigastrio se ditiende. Al sospecha que ha introducido el
tubo en el estómago, también es cierto que:
A. Aumenta la frecuencia de ondas lentas
B. Disminuye el pH gástrico
C. Disminuiría el tono del píloro
D. Aumenta el pH gástrico
57) Paciente con apendicitis aguda, que debuta con dolor en mesogastrio. ese dolor se debea
estimulación de receptores del dolor cuyas fibras van a viajar a la médula espinal a través de:
A. Nervios simpáticos
B. Plexo hipogástrico
C. Nervio esplácnico pélvico
D. Nervio vago
58) La rotación en sentido longitudinal del estómago en el desarrollo embriológico condiciona que el
nervio vago derecho quede a nivel:
A. Izquierdo
B. Oblicuo
C. Anterior
D. Posterior
59) Una de las siguientes sustancias reguladoras, puede actuar de forma paracrina y como hormona.
Marque la correcta:
A. Péptido insulinotrópico dependiente de glucosa
B. GRP
C. Acetilcolina
D. Somatostatina
60) El alcohol y la cafeína estimulan la fase ______________ de la producción de ácido clorhídrico. a)
cefálica
b) gástrica
c) intestinal
d) gástrica y cefálica
ECU 1
1) Con respecto a la saliva, marque la respuesta correcta:

Respuesta seleccionada:
el sistema simpático estimula su secreción
● Pregunta 2
2 de 2 puntos

Respecto a las enfermedades del esófago, marque lo correcto:

Respuesta
seleccionada: el diagnóstico diferencial de la acalasia es la enfermedad de Chagas
esofágica
● Pregunta 3
0 de 2 puntos

En relación a la fisilogía gástrica, marque lo correcto:

la cimetidina actúa en la región basolateral de la célula parietal


● Pregunta 4
2 de 2 puntos

La célula mucosa del cuello gástrico produce:

Respuesta seleccionada:
Moc
o
● Pregunta 5
0 de 2 puntos

La saliva puede tener una variedad de electrolitos en su composición. Entre ellos el cloro,
respecto al cual se puede afirmar:

Respuesta seleccionada:
Respuestas:

Su concentración no llega a ser tan alta como en el plasma

● Pregunta 6
2 de 2 puntos
Los músculos de la masticación que producen la retropulsión de la mandíbula son:

Respuesta seleccionada:
temporales
● Pregunta 7
0 de 2 puntos

Respecto a las glándulas salivales, marque lo incorrecto:

Respuesta seleccionada:
la glándula sublingual tiene forma de garfio

● Pregunta 8
2 de 2 puntos

Respecto a la anatomía del estómago, marque lo correcto:

Respuesta seleccionada:
la arteria gástrica derecha nace de la arteria hepática común
● Pregunta 9
2 de 2 puntos

En cuanto a la saliva, marque lo correcto:

Respuesta seleccionada:
La amilasa cumple función digestiva
● Pregunta 10
2 de 2 puntos

Durante el ataque con gas sarín (bloqueador de la acetilcolinesterasa) en el metro de Tokio, en


1995, el personal de salud notó que los pacientes afectados presentaban:

Respuesta seleccionada:
HipersalivacióN

CI1
● Pregunta 1

La presencia de atresias y estenosis duodenales se deben


básicamente a una:

Falta de recanalización


Pregunta 2

● 2 de 2 puntos

El ligamento de Treitz característicamente:

Respuesta
seleccionada: Suspende el ángulo de Treitz

Respuestas: Está adherido a la unión yeyuno-ileal

Se encuentra a nivel de hipogastrio


derecho
Suspende el ángulo de Treitz

Recibe irrigación de la arteria mesentérica


inferior


Pregunta 3

● 0 de 2 puntos

Al deglutir un bolo alimenticio, es lógico suponer que al pasar por


el esófago haya un mayor consumo de oxígeno en la pared
del tercio:

Respuesta seleccionada:
Distal

Respuestas: No hay diferencia

Medio

Proximal

Distal


Pregunta 4

● 0 de 2 puntos


Estudiante de medicina de 20 años, se ha amanecido estudiando
para su examen de Sistema Digestivo. No ha probado
alimento desde la cena, por lo que se puede afirmar que la
motilidad de esta persona está siendo regulada por:

Respuesta seleccionada:
CCK

Respuestas: VIP

Motilina

CCK

Adrenalina


Pregunta 5

● 2 de 2 puntos

La hernia fisiológica se produce dentro de:

Respuesta seleccionada:
Cordón umbilical

Respuestas: Alantoides

Saco Vitelino

Saco Amniótico

Cordón umbilical

Pregunta 6

● 0 de 2 puntos

El crecimiento de un adenocarcinoma de páncreas compromete


la pared gástrica por contigüidad. ¿Qué parte del estómago
se esperaría esté comprometido?

Respuesta seleccionada:
Pared anterior del píloro

Respuestas: Pared anterior del píloro

Pared posterior del fondo

Pared anterior del cardias

Pared posterior del antro


Pregunta 7

● 2 de 2 puntos

Al comer unas papitas fritas con mayonesa, el vaciamiento


gástrico disminuye por efecto directo de la hormona:

Respuesta seleccionada:
colecistoquinina (CCK)

Respuestas: motilina
somatostatina

secretina

colecistoquinina (CCK)


Pregunta 8

● 0 de 2 puntos

Paciente que come entera una pizza familiar de chorizo y queso.


Es posible esperar que debido a la cantidad de alimento
ingerida, las ondas lentas hayan:

Respuesta
seleccionada: Aumentado su frecuencia por estímulo
parasimpático

Respuestas: Disminuido su frecuencia por estímulo


parasimpático

Sufrido ninguna alteración en su frecuencia

Aumentado su frecuencia por estímulo


parasimpático

Aumentado su frecuencia por estímulo


simpático


Pregunta 9

● 2 de 2 puntos


En este caso se puede afirmar con seguridad que se presenta:

Respuesta
seleccionada: contracciones tónicas en la región ano
rectal

Respuestas: pérdida de inervación por el nervio


esplácnico menor

dilatación de tracto gastrointestinal


afectado

contracciones tónicas en la región ano


rectal

células ganglionares sólo en el ano recto


Pregunta 10

● 2 de 2 puntos

La forma más común de atresia esofágica contiene:

Respuesta
selecciona Estenosis proximal del esófago más fístula
da: traqueoesofágica distal

Respuestas: Estenosis distal del esófago más fístula


traqueoesofágica proximal

Estenosis proximal y distal del esófago más


fístula traqueoesofágica distal
Estenosis distal del esófago más fístula
traqueoesofágica distal

Estenosis proximal del esófago más fístula


traqueoesofágica distal


Niño de sexo masculino de 2 años de edad, sufre de
estreñimiento desde el nacimiento (1 deposición cada 3-4
días). Madre menciona que le estimula la defecación con
un termómetro rectal, y continuo uso de enemas y
laxantes. Desde hace 6 meses comienza con vómitos
postprandiales. Los síntomas aumentan en frecuencia y
magnitud y están en relación con los episodios de
estreñimiento. No refiere fiebre, tos, diarrea ni lesiones
cutáneas. Al examen físico presenta regular estado
general, luce deshidratado. Abdomen distendido, blando,
depresible e indoloro. No se palpan masas abdominales.
Se permeabiliza el canal anal con termómetro rectal,
encontrando cierta resistencia. Salida de material fecal mal
oliente en regular cantidad. Exámenes de laboratorio:
hemograma normal. Signos inflamatorios de fase aguda
negativos. Alcalosis metabólica leve en sangre venosa.
Radiografía con enema baritado muestra recto y colon
sigmoides dilatados (megacolon). Biopsia profunda:
ausencia de células ganglionares en la muestra enviada.
Se realiza cirugía correctiva.
La percepción de la pirosis (sensación de dolor o
quemazón en el esófago) asociado al reflujo
gastroesofágico, puede aparecer o exacerbarse debido
a:

Respuesta
seleccionada: Ejercicio

Respuestas: Somatostatina

Uso de antiácidos

Ejercicio

Bipedestación


Pregunta 2
● 0 de 3,3334 puntos

Niño de sexo masculino de 2 años de edad,


sufre de estreñimiento desde el nacimiento
(1 deposición cada 3-4 días). Madre
menciona que le estimula la defecación con
un termómetro rectal, y continuo uso de
enemas y laxantes. Desde hace 6 meses
comienza con vómitos postprandiales. Los
síntomas aumentan en frecuencia y
magnitud y están en relación con los
episodios de estreñimiento. No refiere fiebre,
tos, diarrea ni lesiones cutáneas. Al examen
físico presenta regular estado general, luce
deshidratado. Abdomen distendido, blando,
depresible e indoloro. No se palpan masas
abdominales. Se permeabiliza el canal anal
con termómetro rectal, encontrando cierta
resistencia. Salida de material fecal mal
oliente en regular cantidad. Exámenes de
laboratorio: hemograma normal. Signos
inflamatorios de fase aguda negativos.
Alcalosis metabólica leve en sangre venosa.
Radiografía con enema baritado muestra
recto y colon sigmoides dilatados
(megacolon). Biopsia profunda: ausencia de
células ganglionares en la muestra enviada.
Se realiza cirugía correctiva.
Considerando que este paciente está
sometido a estrés por el agravamiento de
su enfermedad, es posible afirmar que
sus ondas lentas están:

Respuesta seleccionada:
Hipopolarizadas

Respuestas: Hipopolarizadas

Hiperpolarizadas

Desmotivadas

Despolarizadas

Pregunta 3

● 3,3334 de 3,3334 puntos

Niño de sexo masculino de 2 años de edad,


sufre de estreñimiento desde el nacimiento
(1 deposición cada 3-4 días). Madre
menciona que le estimula la defecación con
un termómetro rectal, y continuo uso de
enemas y laxantes. Desde hace 6 meses
comienza con vómitos postprandiales. Los
síntomas aumentan en frecuencia y
magnitud y están en relación con los
episodios de estreñimiento. No refiere fiebre,
tos, diarrea ni lesiones cutáneas. Al examen
físico presenta regular estado general, luce
deshidratado. Abdomen distendido, blando,
depresible e indoloro. No se palpan masas
abdominales. Se permeabiliza el canal anal
con termómetro rectal, encontrando cierta
resistencia. Salida de material fecal mal
oliente en regular cantidad. Exámenes de
laboratorio: hemograma normal. Signos
inflamatorios de fase aguda negativos.
Alcalosis metabólica leve en sangre venosa.
Radiografía con enema baritado muestra
recto y colon sigmoides dilatados
(megacolon). Biopsia profunda: ausencia de
células ganglionares en la muestra enviada.
Se realiza cirugía correctiva.
Debido al acúmulo de material fecal en todo
el marco colónico, y a la irritación
química asociada, el peristaltismo del
íleon distal se debe encontrar:

Respuesta
seleccionada: Inhibido

Respuestas: Afectado por un reflejo


vago-vagal

No sufre alteraciones

Estimulado
Inhibido


Pregunta 4

● 0 de 3,3334 puntos

Niño de sexo masculino de 2 años de edad,


sufre de estreñimiento desde el nacimiento
(1 deposición cada 3-4 días). Madre
menciona que le estimula la defecación con
un termómetro rectal, y continuo uso de
enemas y laxantes. Desde hace 6 meses
comienza con vómitos postprandiales. Los
síntomas aumentan en frecuencia y
magnitud y están en relación con los
episodios de estreñimiento. No refiere fiebre,
tos, diarrea ni lesiones cutáneas. Al examen
físico presenta regular estado general, luce
deshidratado. Abdomen distendido, blando,
depresible e indoloro. No se palpan masas
abdominales. Se permeabiliza el canal anal
con termómetro rectal, encontrando cierta
resistencia. Salida de material fecal mal
oliente en regular cantidad. Exámenes de
laboratorio: hemograma normal. Signos
inflamatorios de fase aguda negativos.
Alcalosis metabólica leve en sangre venosa.
Radiografía con enema baritado muestra
recto y colon sigmoides dilatados
(megacolon). Biopsia profunda: ausencia de
células ganglionares en la muestra enviada.
Se realiza cirugía correctiva.
En cuanto a los reflejos gastrocólico y
gastroduodenal en este paciente, indique
lo correcto:
Respuesta
selecci Se dan por nervios intrínsecos
onada: del sistema entérico

Respuestas El control del nervio vago sobre


: el recto se ha abolido

El reflejo gastrocólico es más


marcado en adultos que en
niños

Se dan por nervios intrínsecos


del sistema entérico

Se pueden considerar reflejos


vago-vagales


Pregunta 5

● 3,3334 de 3,3334 puntos

Niño de sexo masculino de 2 años de edad,


sufre de estreñimiento desde el nacimiento
(1 deposición cada 3-4 días). Madre
menciona que le estimula la defecación con
un termómetro rectal, y continuo uso de
enemas y laxantes. Desde hace 6 meses
comienza con vómitos postprandiales. Los
síntomas aumentan en frecuencia y
magnitud y están en relación con los
episodios de estreñimiento. No refiere fiebre,
tos, diarrea ni lesiones cutáneas. Al examen
físico presenta regular estado general, luce
deshidratado. Abdomen distendido, blando,
depresible e indoloro. No se palpan masas
abdominales. Se permeabiliza el canal anal
con termómetro rectal, encontrando cierta
resistencia. Salida de material fecal mal
oliente en regular cantidad. Exámenes de
laboratorio: hemograma normal. Signos
inflamatorios de fase aguda negativos.
Alcalosis metabólica leve en sangre venosa.
Radiografía con enema baritado muestra
recto y colon sigmoides dilatados
(megacolon). Biopsia profunda: ausencia de
células ganglionares en la muestra enviada.
Se realiza cirugía correctiva.
Con respecto a la defecación señale el
enunciado correcto:

Respuesta
selecci Es estimulado por un llenado de
onada: la cuarta parte del volumen
rectal

Respuestas Es completamente voluntario y


: mediado por el nervio
pudendo

La aferencia parasimpática es
transmitida por vía del nervio
vago

Es un reflejo netamente local

Es estimulado por un llenado de


la cuarta parte del volumen
rectal


Pregunta 6

● 0 de 3,3334 puntos


Niño de sexo masculino de 2 años de edad,
sufre de estreñimiento desde el nacimiento
(1 deposición cada 3-4 días). Madre
menciona que le estimula la defecación con
un termómetro rectal, y continuo uso de
enemas y laxantes. Desde hace 6 meses
comienza con vómitos postprandiales. Los
síntomas aumentan en frecuencia y
magnitud y están en relación con los
episodios de estreñimiento. No refiere fiebre,
tos, diarrea ni lesiones cutáneas. Al examen
físico presenta regular estado general, luce
deshidratado. Abdomen distendido, blando,
depresible e indoloro. No se palpan masas
abdominales. Se permeabiliza el canal anal
con termómetro rectal, encontrando cierta
resistencia. Salida de material fecal mal
oliente en regular cantidad. Exámenes de
laboratorio: hemograma normal. Signos
inflamatorios de fase aguda negativos.
Alcalosis metabólica leve en sangre venosa.
Radiografía con enema baritado muestra
recto y colon sigmoides dilatados
(megacolon). Biopsia profunda: ausencia de
células ganglionares en la muestra enviada.
Se realiza cirugía correctiva.
El contenido fecal se detiene en la zona
inmediatamente proximal a la zona donde
hay una menor presencia de:

Respuesta
seleccionada: Neuropéptido Y

Respuestas: Enteroquinasa

Péptido intestinal
vasoactivo

Neuropéptido Y

Acetilcolina


Estudiante de medicina de la UPC de 21 años sufre de
gastritis aguda ocasionada por comer en lugares poco
higiénicos. Suele consumir caramelos ( chupar ) mientras
está en clase hasta la tarde. Toma gaseosas regularmente
(carbohidratos 46%, sodio 53%). También toma regular
cantidad de leche (grasa 35%, lactosa 35%, proteínas
30%), pues le calma un poco el dolor el ardor que siente
por la gastritis. Incluso, cuando puede, se toma dos vasos
de agua fría para calmar las molestias. Ha decidido ir al
médico para tratarse pues ya no soporta el dolor, el cual
está seguro que los síntomas se deben a una elevada
producción de ácido clorhídrico en el estómago, y por ello
le ha recetado Ranitidina (antihistamínico), con lo que
siente mejoría.
Para reducir la secreción de HCl en esta paciente se
podría usar sustancias similares a:

Respuesta
seleccio Péptido insulinotrópico
nada: dependiente de la glucosa
(GIP)

Respuestas: Bombesina

Péptido insulinotrópico
dependiente de la glucosa
(GIP)

Acetilcolina

Histamina


Pregunta 2

● 0 de 3,3333 puntos


Estudiante de medicina de la UPC de 21 años
sufre de gastritis aguda ocasionada por
comer en lugares poco higiénicos. Suele
consumir caramelos ( chupar ) mientras está
en clase hasta la tarde. Toma gaseosas
regularmente (carbohidratos 46%, sodio
53%). También toma regular cantidad de
leche (grasa 35%, lactosa 35%, proteínas
30%), pues le calma un poco el dolor el
ardor que siente por la gastritis. Incluso,
cuando puede, se toma dos vasos de agua
fría para calmar las molestias. Ha decidido ir
al médico para tratarse pues ya no soporta
el dolor, el cual está seguro que los
síntomas se deben a una elevada
producción de ácido clorhídrico en el
estómago, y por ello le ha recetado
Ranitidina (antihistamínico), con lo que
siente mejoría.
Si se usara atropina en esta paciente, se
esperaría que disminuya la liberación de:

Respuesta
seleccionada: Bombesina

Respuestas: GIP

Bombesina

Secretina

Enzimas pancreáticas


Pregunta 3

● 0 de 3,3333 puntos


Estudiante de medicina de la UPC de 21 años
sufre de gastritis aguda ocasionada por
comer en lugares poco higiénicos. Suele
consumir caramelos ( chupar ) mientras está
en clase hasta la tarde. Toma gaseosas
regularmente (carbohidratos 46%, sodio
53%). También toma regular cantidad de
leche (grasa 35%, lactosa 35%, proteínas
30%), pues le calma un poco el dolor el
ardor que siente por la gastritis. Incluso,
cuando puede, se toma dos vasos de agua
fría para calmar las molestias. Ha decidido ir
al médico para tratarse pues ya no soporta
el dolor, el cual está seguro que los
síntomas se deben a una elevada
producción de ácido clorhídrico en el
estómago, y por ello le ha recetado
Ranitidina (antihistamínico), con lo que
siente mejoría.
El consumir caramelos eleva los niveles en
sangre de una hormona cuya función es
la estimulación de las células:

Respuesta
seleccionada: K del duodeno

Respuestas: Alfa del páncreas

K del duodeno

Beta del páncreas

G del antro


Pregunta 4

● 0 de 3,3333 puntos


Estudiante de medicina de la UPC de 21 años
sufre de gastritis aguda ocasionada por
comer en lugares poco higiénicos. Suele
consumir caramelos ( chupar ) mientras está
en clase hasta la tarde. Toma gaseosas
regularmente (carbohidratos 46%, sodio
53%). También toma regular cantidad de
leche (grasa 35%, lactosa 35%, proteínas
30%), pues le calma un poco el dolor el
ardor que siente por la gastritis. Incluso,
cuando puede, se toma dos vasos de agua
fría para calmar las molestias. Ha decidido ir
al médico para tratarse pues ya no soporta
el dolor, el cual está seguro que los
síntomas se deben a una elevada
producción de ácido clorhídrico en el
estómago, y por ello le ha recetado
Ranitidina (antihistamínico), con lo que
siente mejoría.
En este paciente con gastritis aguda debida a
una alta producción de ácido clorhídrico,
si se le hiciera un examen de sangre, se
encontraría elevados los niveles de:

Respuesta
seleccio Histamina
nada:

Respuestas: Sustancia P

Colecistoquinina

Péptido insulinotrópico
dependiente de la glucosa
(GIP)

Histamina


Pregunta 5

● 3,3333 de 3,3333 puntos


Estudiante de medicina de la UPC de 21 años
sufre de gastritis aguda ocasionada por
comer en lugares poco higiénicos. Suele
consumir caramelos ( chupar ) mientras está
en clase hasta la tarde. Toma gaseosas
regularmente (carbohidratos 46%, sodio
53%). También toma regular cantidad de
leche (grasa 35%, lactosa 35%, proteínas
30%), pues le calma un poco el dolor el
ardor que siente por la gastritis. Incluso,
cuando puede, se toma dos vasos de agua
fría para calmar las molestias. Ha decidido ir
al médico para tratarse pues ya no soporta
el dolor, el cual está seguro que los
síntomas se deben a una elevada
producción de ácido clorhídrico en el
estómago, y por ello le ha recetado
Ranitidina (antihistamínico), con lo que
siente mejoría.
El consumo de una pequeña cantidad de
gaseosa aumentará directamente la
concentración sérica de cuál de las
siguientes hormonas:

Respuesta
seleccionad Péptido 1 similar al
a: glucagón (GLP-1)

Respuestas: Colecistoquinina (CCK)

Motilina

Secretina

Péptido 1 similar al
glucagón (GLP-1)


Pregunta 6

● 3,3333 de 3,3333 puntos


Estudiante de medicina de la UPC de 21 años
sufre de gastritis aguda ocasionada por
comer en lugares poco higiénicos. Suele
consumir caramelos ( chupar ) mientras está
en clase hasta la tarde. Toma gaseosas
regularmente (carbohidratos 46%, sodio
53%). También toma regular cantidad de
leche (grasa 35%, lactosa 35%, proteínas
30%), pues le calma un poco el dolor el
ardor que siente por la gastritis. Incluso,
cuando puede, se toma dos vasos de agua
fría para calmar las molestias. Ha decidido ir
al médico para tratarse pues ya no soporta
el dolor, el cual está seguro que los
síntomas se deben a una elevada
producción de ácido clorhídrico en el
estómago, y por ello le ha recetado
Ranitidina (antihistamínico), con lo que
siente mejoría.
El consumo rápido de 500 mL de gaseosa
aumentará directamente la concentración
sérica de cuál de las siguientes
hormonas:

Respuesta
seleccionada: Gastrina

Respuestas: Secretina

Colecistoquinina
(CCK)

Gastrina

Neuropéptido Y


Cuando el tubo digestivo está en reposo, los complejos
mioeléctricos migratorios son desencadenados por:

Respuesta
seleccionad la serotonina
a:

Respuestas: la sustancia P
la serotonina

la motilina

el péptido intestinal
vasoactivo (VIP)


Pregunta 2

● 1 de 1 puntos

El orificio omental, o hiato de Winslow, se


encuentra limitado por el ligamento:

Respuesta seleccionada:
hepatoduodenal

Respuestas:
hepatoduodenal

gastroduodenal

gastroeesplénico

esplenorrenal


Pregunta 3

● 1 de 1 puntos


En un paciente de 43 años con tumor carcinoide
de páncreas productor de gastrina
(Síndrome de Zollinger-Ellison) se puede
esperar encontrar una potenciación del
reflejo:

Respuesta
seleccionada: gastrocólico

Respuestas: del vómito

de relajación receptiva

gastrocólico

ileocólico


Pregunta 4

● 1 de 1 puntos

La mucosa irrigada por la arteria palatina mayor


está recubierta por un epitelio:

Respuesta
seleccionada: plano escamoso
queratinizado

Respuestas:
plano escamoso
queratinizado

plano simple

cilíndrico simple no ciliado

plano escamoso no
queratinizado

Pregunta 5

● 1 de 1 puntos

En un paciente con enfermedad de Hirschprung


(megacolon agangliónico) con dilatación de
los dos tercios proximales del recto, la
dilatación de este segmento se ha producido
con mayor probabilidad debido a los
movimientos:

Respuesta seleccionada:
en masa

Respuestas: peristálticos

en masa

de retropulsión

segmentarios


Pregunta 6

● 1 de 1 puntos

En el sistema digestivo la liberación hormonal se


presenta ante diversos factores o estímulos.
La hormona ____________ es estimulada
por la presencia de alimentos en el bulbo
duodenal a predominio de ácidos grasos y
triglicéridos, por estimulación vagal y por la
hormona secretina.
Respuesta
seleccionada: colecistocinina (CCK)

Respuestas: lipasa pancreática

gastrina

colecistocinina (CCK)

motilina


Pregunta 7

● 1 de 1 puntos

¿Cuál de las siguientes estructuras se la conoce


como el vigilante del abdomen , por su
capacidad de desplazarse y adherirse a
cualquier zona inflamada o envolver el
órgano para frenar la inflamación?

Respuesta
seleccionada: Omento mayor

Respuestas: Omento menor

Omento mayor

Mesogastrio anterior

Intestino delgado


Pregunta 8
● 1 de 1 puntos

¿Cuál de las siguientes sustancias disminuye la


fuerza de las contracciones de
segmentación?

Respuesta
seleccionada: Atropina

Respuestas:
Atropina

Acetilcolina

Colecistocinina (CCK)

Insulina


Pregunta 9

● 1 de 1 puntos

Durante la deglución, inmediatamente luego que


el bolo alimenticio pasa por el esfínter
esofágico superior, se espera que la presión
intraluminal:

Respuesta
seleccionad disminuya en el cardias
a:

Respuestas: aumente en el tercio medio


del esófago

disminuya en el tercio medio


del esófago
aumente en la porción distal
al bolo

disminuya en el cardias


Pregunta 10

● 1 de 1 puntos

Durante el desarrollo del tubo digestivo en la


vida intrauterina, cuando el asa intestinal
rota 90° en el sentido de las manecillas del
reloj entonces:

Respuesta
selecciona el colon pasa por detrás del
da: duodeno

Respuestas: se forma el colon izquierdo

el colon pasa por detrás del


duodeno

se realiza el plegamiento
normal de los intestinos

se desarrolla un situs inversus


Pregunta 11

● 1 de 1 puntos


La irritación del peritoneo intestinal, como la que
sucede en una peritonitis, producirá:

Respuesta
seleccion la inhibición del peristaltismo
ada:

Respuestas: la estimulación del


peristaltismo

la inhibición del peristaltismo

la destrucción de las células


intersticiales de Cajal

el aumento de las
contracciones tónicas


Pregunta 12

● 1 de 1 puntos

Si al intubar a un paciente, por error se ingresa


el tubo endotraqueal en el esófago y se
insufla el manguito endotraqueal (globo
TET), la dilatación de este manguito
generará:

Respuesta
seleccio múltiples ondas secundarias
nada:

Respuestas: múltiples ondas peristálticas


primarias

múltiples ondas secundarias

una onda peristáltica primaria


una onda peristáltica secundaria
seguida de una terciaria


Pregunta 13

● 1 de 1 puntos

Paciente de 24 años acude a consulta externa


por presentar una fístula oronasal
(comunicación entre la cavidad oral y la
cavidad nasal). Está fístula es una
consecuencia tardía de la lesión de un vaso
sanguíneo por el antecedente de haber sido
operado de paladar hendido en los primeros
años de vida, aparentemente en una
campaña gratuita de corrección de paladar
fisurado. ¿Cuál de las arterias palatinas
podría haberse lesionado durante esa
cirugía?

Respuesta
seleccionad Mayor
a:

Respuestas:
Mayor

Ascendente

Menor

Rama palatina de la faríngea


ascendente

Pregunta 14

● 0 de 1 puntos

El omento mayor deriva del mesenterio


__________ y se inserta en el ___________

Respuesta
seleccionada: medio / colon

Respuestas: ventral / duodeno

dorsal / estómago

ventral / estómago

medio / colon


Pregunta 15

● 0 de 1 puntos

Para realizar el movimiento mecánico de abrir la


boca, primero se necesita:
Respuesta
selecciona la contracción de los
da: músculos buccinadores

Respuestas:
fijar el hueso hioides

la retropulsión de la lengua

elevar la laringe

la contracción de los
músculos buccinadores


Pregunta 16

● 0 de 1 puntos

El mesocolon transverso se origina en:

Respuesta
seleccionada: la curvatura mayor del
estómago

Respuestas: el colon transverso

la curvatura mayor del


estómago

la curvatura menor del


estómago

la pared posterior del


abdomen

Pregunta 17

● 0 de 1 puntos

Paciente con insuficiencia mitral moderada a


severa, con aumento de volumen de la
aurícula izquierda; esta condición tendrá
como consecuencia a nivel del sistema
digestivo:

Respuesta
selecciona la acalasia
da:

Respuestas:
la disfagia a sólidos

alteraciones de la fase
faríngea de la deglución

la acalasia

la estenosis hipertrófica del


cardias


Pregunta 18

● 0 de 1 puntos

¿Cuál de las siguientes alternativas se define


como la herniación de las vísceras
abdominales por un anillo umbilical
agrandado?
Respuesta seleccionada:
Gastrosquisis

Respuestas: Hernia femoral

Onfalocele

Hernia inguinal

Gastrosquisis


Pregunta 19

● 1 de 1 puntos

Durante el desarrollo intrauterino del páncreas,


cuando la porción izquierda de la yema
pancreática ventral migra en dirección
opuesta, se forma el páncreas:

Respuesta seleccionada:
anular

Respuestas: ectópico

anular

divisum

derecho


Pregunta 20
● 1 de 1 puntos

Al deglutir algo, durante la fase esofágica, las


ondas lentas del esófago se:

Respuesta
seleccionada: hacen menos negativas

Respuestas: hiperpolarizan

hacen menos negativas

asocian a menor actividad


motora

estabilizan
ECU 1
Estudiante de medicina de la UPC de 21 años sufre de gastritis aguda ocasionada por comer
en lugares poco higiénicos. Suele consumir caramelos ( chupar ) mientras está en clase hasta
la tarde. Toma gaseosas regularmente (carbohidratos 46%, sodio 53%). También toma
regular cantidad de leche (grasa 35%, lactosa 35%, proteínas 30%), pues le calma un poco el
dolor el ardor que siente por la gastritis. Incluso, cuando puede, se toma dos vasos de agua
fría para calmar las molestias. Ha decidido ir al médico para tratarse pues ya no soporta el
dolor, el cual está seguro que los síntomas se deben a una elevada producción de ácido
clorhídrico en el estómago, y por ello le ha recetado Ranitidina (antihistamínico), con lo que
siente mejoría.

El uso de atropina en este paciente:


- Aumentará el pH del estómago

Entre las sustancias cerebrales que producen ansiedad está la serotonina, la cual también
tiene acción:
- Anorexigénica

El consumo de dos vasos de agua seguidos agua generará indirectamente un aumento en la


liberación de:
- Ácido clorhídrico

En este paciente con gastritis aguda debida a una alta producción de ácido clorhídrico, sería
lógico esperar que el píloro tenga un tono muscular:
- Aumentado

El consumo de leche produce directamente un aumento de los niveles séricos de la


hormona:
- Colecistoquinina (CCK)

El consumo de leche produce directamente un aumento de los niveles séricos de la


hormona:
- Gastrina

ECU 2
Niño de sexo masculino de 2 años de edad, sufre de estreñimiento desde el nacimiento (1
deposición cada 3-4 días). Madre menciona que le estimula la defecación con un
termómetro rectal, y continuo uso de enemas y laxantes. Desde hace 6 meses comienza con
vómitos postprandiales. Los síntomas aumentan en frecuencia y magnitud y están en
relación con los episodios de estreñimiento. No refiere fiebre, tos, diarrea ni lesiones
cutáneas. Al examen físico presenta regular estado general, luce deshidratado. Abdomen
distendido, blando, depresible e indoloro. No se palpan masas abdominales. Se permeabiliza
el canal anal con termómetro rectal, encontrando cierta resistencia. Salida de material fecal
mal oliente en regular cantidad. Exámenes de laboratorio: hemograma normal. Signos
inflamatorios de fase aguda negativos. Alcalosis metabólica leve en sangre venosa.
Radiografía con enema baritado muestra recto y colon sigmoides dilatados (megacolon).
Biopsia profunda: ausencia de células ganglionares en la muestra enviada. Se realiza cirugía
correctiva.

El contenido fecal se detiene en la zona inmediatamente proximal a la zona donde hay una
menor presencia de:
- Péptido intestinal vasoactivo

En cuanto a los reflejos gastrocólico y gastroduodenal en este paciente, indique lo correcto:


- Se pueden considerar reflejos vago-vagales

En este paciente se considera que está abolido el reflejo:


- Rectoesfinteriano

Debido al acúmulo de material fecal en todo el marco colónico, y a la irritación química


asociada, el peristaltismo del íleon distal se debe encontrar:
- Inhibido

Es un reflejo propio de la pared intestinal:


- Peristaltismo

A diferencia de las arcadas, los vómitos presentan apertura de:


- Esfínter esofágico superior

CI 1
Sustancia que inhibe la secreción y la motilidad del estómago prolongando el tiempo de
digestión:
- Péptido insulinotrópico dependiente de la glucosa (GIP)

Marque lo correcto:
- La hernia fisiológica se produce en la sexta semana y es la salida temporal de
asas intestinales a través del cordón umbilical

Marque la respuesta correcta en relación a la gastrina:


- Las células G son las productoras y se encuentran principalmente en el antro
gástrico
El consumir caramelos indirectamente activa la vía:
- POMC/CART

¿En qué capa se encuentra la alteración principal en el Hirschsprung o megacolon


agangliónico?:
- Muscular propia

Con respecto a las ondas lentas, marque la afirmación correcta:


- Son contracciones rítmicas espontáneas

El uso de Ranitidina bloquea el receptor H2 de la histamina en las células parietales. La


histamina llega a estas células por:
- Difusión

La triada sintomática: vómitos explosivos post-prandiales, movimientos peristálticos


epigástricos visibles de izquierda a derecha y nódulo palpable epigástrico subcostal derecho,
pertenecen a:
- Estenosis congénita hipertrófica del píloro.

Durante una cirugía oncológica, ¿la extirpación de cuál de los siguientes órganos se vería
comprometida por la presencia de adventicia?:
- Recto

En cuanto a los reflejos gastrointestinales, un reflejo que estimula el tránsito intestinal es el


reflejo:
- Gastrocólico

El ligamento falciforme divide al hígado en dos lóbulos derecho e izquierdo.


Embriológicamente deriva del:
- Mesenterio ventral

La presencia de atresias y estenosis duodenales se deben básicamente a una:


- Falta de recanalización

Estudiante de medicina de 20 años, se ha amanecido estudiando para su examen de Sistema


Digestivo. No ha probado alimento desde la cena, por lo que se puede afirmar que la
motilidad de esta persona está siendo regulada por:
- Motilina

Paciente con disminución del apetito marcada asociada a cáncer terminal, para promover la
ingesta de alimentos se podría usar análogos de:
- Endorfinas

Las ondas lentas se producen por la apertura cíclica de canales de:


- Calcio

La forma más común de atresia esofágica contiene:


- Estenosis proximal del esófago más fístula traqueoesofágica distal

Al deglutir un bolo alimenticio, es lógico suponer que al pasar por el esófago haya un mayor
consumo de oxígeno en la pared del tercio:
- Proximal

Paciente que come entera una pizza familiar de chorizo y queso. Es posible esperar que
debido a la cantidad de alimento ingerida, las ondas lentas hayan:
- Sufrido ninguna alteración en su frecuencia

La hernia fisiológica se produce dentro de:


- Cordón umbilical

El crecimiento de un adenocarcinoma de páncreas compromete la pared gástrica por


contigüidad. ¿Qué parte del estómago se esperaría esté comprometido?
- Pared posterior del antro

CI 2
Estimula la producción de saliva:
- Vasodilatación periglandular

Durante la secreción de saliva, es de esperarse que las concentraciones de ________ y


______ disminuyan al disminuir el flujo:
- Sodio Bicarbonato

Con respecto a la secreción gástrica de HCl:


- a mayor secreción de HCl en el lumen gástrico, mayor pH en la sangre venosa
gástrica

Respecto a las enfermedades del esófago, marque lo correcto:


- el diagnóstico diferencial de la acalasia es la enfermedad de Chagas esofágica

Con respecto a las lesiones y enfermedades de la boca, marque lo correcto:


- la eritroplasia debe ser biopsiada
Respecto a las glándulas salivales, marque lo incorrecto:
- la glándula sublingual tiene forma de garfio

El omeprazol actúa sobre la membrana _____________ de la célula ____________


- apical / parietal

Durante el sueño, la concentración de bicarbonato en la saliva:


- Disminuye

Durante el ataque con gas sarín (bloqueador de la acetilcolinesterasa) en el metro de Tokio,


en 1995, el personal de salud notó que los pacientes afectados presentaban:
- Hipersalivación

La célula mucosa del cuello gástrico produce:


- Moco

ECU 3
El caso:
Paciente de 54 años con antecedentes de alcoholismo, gastritis crónica, tabaquismo pesado,
obesidad, cálculos biliares y cirrosis, es llevado a la emergencia por dolor abdominal en
epigastrio irradiado a la espalda y trastorno del sensorio.
Al examen físico: presión arterial 85/50 mmHg, frecuencia cardíaca 100 latidos/min,
frecuencia respiratoria 18 x minuto, temperatura axilar 36°C.
Conjuntivas pálidas, escleras ictéricas nevus arácnidos en tronco, distensión abdominal
marcada, cabeza de medusa, matidez desplazable en ambos flancos e hipogastrio, dolor a la
palpación de abdomen.
Tiempo de protrombina: 24 seg (testigo: 13 seg); TPT: 38 seg, glicemia: 165 mg/dL, uremia:
20 mg/dL, ASAT: 76 UI/L, ALAT: 22 UI/L, albumina: 2,5 g/dL, bilirrubina total: 2,6 mg/dL,
bilirrubina directa: 1,4 mg/dL, amilasa sérica 4000 U/L.

En todas las preguntas marque la mejor respuesta.


De las siguientes sustancias secretadas por los órganos de este paciente, la más alcalina es la
secreción:
- Pancreática

En esta paciente, el aumento de la Amilasa sérica se debe directamente a una lesión de


- Páncreas

En cuanto a la gastritis de este paciente, se encontró que era producida por la bacteria
Helicobacter pylori. Esta bacteria sobrevive en el medio ácido del estómago gracias a:
- Ureasa
Para evaluar la función hepática podemos medir los niveles séricos de:
- Tiempo de protrombina

El misoprostol, análogo de las prostaglandinas, está mejor indicado en:


- Prevenir daño por AINES

El paciente llegó con trastorno del sensorio posiblemente debida a encefalopatía hepática
producida por déficit en el metabolismo del/la:
- Amoniaco

CI 3
En cuanto al urobilinógeno, se puede decir que es cierto:
- Recircula hacia el hígado

Marque lo correcto:
- Las venas sublobulillares desembocan en las venas hepáticas

En cuanto a la secreción de iones y agua en los conductos biliares es correcto lo siguiente:


- Es estimulada por la secretina

Paciente con tumor neuroendocrino productor de secretina, debido a lo cual se puede


esperar que su secreción pancreática, comparada con la de una persona sana en estado de
bajo flujo, tenga una concentración de:
- bicarbonato aumentada

El efecto colerético de las sales biliares se refiere a:


- Estimulo de secreción biliar

Paciente de 42 años con adenocarcinoma ductular. La TC ha demostrado claramente que el


tumor está en el cuello del páncreas y que hay un gran vaso ocluido. ¿Cuál de los siguientes
vasos estaría más probablemente obstruido?
- Vena porta

Las ramas más pequeñas del árbol biliar son:


- Canalículos biliares

La secreción de agua y bicarbonato por el páncreas exocrino se da básicamente en la fase:


- Intestinal
Cuando el alimento se encuentra en el estómago, se produce la liberación de enzimas
pancreáticas básicamente debido a la acción de:
- Vago

La bilirrubina directa aumentada en cirrosis hepática se excreta en la orina debido a:


- Ser hidrosoluble

ECU 4
Caso:
Mujer de 83 años acude a emergencia por dolor abdominal desde hace 4 días, localizado en
epigastrio, irradiado a ambos hipocondrios, nauseas, vómitos y distensión abdominal; tiene
antecedente de cardiopatía hipertensiva, diabetes mellitus tipo II y fibrilación auricular.
Refiere deposiciones diarreicas muco sanguinolentas hace 1 día. Los exámenes iniciales
muestran PA: 110/60 mmHg, FC: 110/mn, leucocitos: 17800, neutrófilos de 93%. TAC
abdómino pélvica se observa oclusión completa de arteria mesentérica superior por trombo
asociado a placa ateromatosa.

Considerando que se ha comprometido el íleon distal, entre otras áreas, la atrofia o


descamación del epitelio de superficie explicaría cuál de los signos o síntomas de la paciente
(marque la mejor respuesta):
- Diarreas mucosanguinolentas

Después de remover quirúrgicamente el territorio gastrointestinal afectado, estaremos


seguros que no quedará en la paciente la siguiente estructura:
- Placas de Peyer

El mecanismo de la diarrea muco sanguinolenta que presenta la paciente, puede mejor


definido como de tipo (marque la mejor respuesta):
- Exudativa

Durante la cirugía, el cirujano observó que además la paciente tenía divertículos en el sigma.
Se sabe que estos divertículos:
- Se pueden asociar a estreñimiento crónico

En esta paciente, ¿cuál de las siguientes sustancias no tendrá una considerable disminución
en su absorción? (marque la mejor respuesta):
- Calcio

PARCIAL
Al evaluar la orofaringe de un paciente, el médico le solicita que abra la boca, saque la
lengua y diga a . Al hacer esta maniobra, nota que el paladar se desvía hacia la derecha, lo
cual le hace sospechar que el paciente sufre de una lesión del nervio craneal:
- X contralateral

Un bolo alimenticio grande y poco masticado se atasca en el esófago, esto ocasiona una
sensación de dolor que es transmitida por los nervios:
- esplácnicos

Para realizar el movimiento mecánico de abrir la boca, primero se necesita:


- fijar el hueso hioides

¿Cuál de las siguientes alternativas se define como la protrusión directa del contenido
abdominal a la cavidad amniótica por un defecto de la pared corporal?
- Gastrosquisis

Un paciente requiere que se le coloque una sonda de alimentación directamente al


estómago (gastrostomía), el cirujano deberá hacer una incisión en la piel del abdomen ¿cuál
de las siguientes raíces nerviosas debe ser anestesiada para este procedimiento?
- T8

En un paciente de 43 años con tumor carcinoide de páncreas productor de gastrina


(Síndrome de Zollinger-Ellison) se puede esperar encontrar una potenciación del reflejo:
- gastrocólico

El mecanismo de la defecación incluye la participación de diversas estructuras ¿Cuál de las


siguientes alternativas es correcta?
- Puede ser mediado por un reflejo intrínseco

Cuando el contenido del estómago ingresa al duodeno, uno de los reflejos que inhiben el
vaciamiento gástrico es a través del:
- sistema nervioso mientérico

Durante la masticación, gran parte del proceso masticatorio se debe a:


- el reflejo masticatorio

Las glándulas salivales tienes conductos para la excreción de la saliva; las glándulas
____________ drenan en las carúnculas sublinguales.
- sublinguales
Los diferentes segmentos del tubo digestivo son susceptibles de reflejos y movimientos
según su contenido. Si colocáramos mediante una sonda un bolo alimenticio directamente
en el tercio medio del esófago:
- se producirá ondas secundarias

En una persona si enfermedad se espera que el tránsito intestinal se vea disminuido cuando
se presenta el reflejo:
- doloroso

El divertículo de Meckel es una anomalía congénita que ocurre por la persistencia del
conducto vitelino y da origen a una estructura sacular, el cual se encuentra en el:
- borde antimesentérico

Si al intubar a un paciente, por error se ingresa el tubo endotraqueal en el esófago y se


insufla el manguito endotraqueal (globo TET), la dilatación de este manguito generará:
- múltiples ondas secundarias

El orificio omental, o hiato de Winslow, se encuentra limitado por el ligamento:


- hepatoduodenal

Paciente de 24 años acude a consulta externa por presentar una fístula oronasal
(comunicación entre la cavidad oral y la cavidad nasal). Está fístula es una consecuencia
tardía de la lesión de un vaso sanguíneo por el antecedente de haber sido operado de
paladar hendido en los primeros años de vida, aparentemente en una campaña gratuita de
corrección de paladar fisurado. ¿Cuál de las arterias palatinas podría haberse lesionado
durante esa cirugía?
- Mayor

Dentro de las anomalías congénitas se puede presentar un tejido pancreático accesorio


¿Cuál es la ubicación más común de este tejido?
- Estómago

Paciente con insuficiencia mitral moderada a severa, con aumento de volumen de la


aurícula izquierda; esta condición tendrá como consecuencia a nivel del sistema digestivo:
- la disfagia a sólidos

El inicio de la fase faríngea de la deglución se debe a estímulos sensitivos que viajan por el
nervio craneal:
- V

El mesocolon transverso se origina en:


- la pared posterior del abdomen

En la digestión de los alimentos, la hormona __________ se libera frente a la presencia de


péptidos y monoglicéridos, y tiene un efecto marcado en la disminución del vaciamiento
gástrico.
- colecistoquinina

En un varón de 47 años con sección medular a nivel de T6 debido a un accidente


automovilístico, sus terapeutas han desarrollado un mecanismo para distender el recto e
iniciar el reflejo rectoesfinteriano, lo cual producirá la contracción de:
- la pared del recto

Los catadores de vino tienen una habilidad increíble al momento de separar los sabores.
Este aumento de la sensibilidad gustativa debido a una mayor cantidad de papilas linguales
y de corpúsculos gustativos se conoce como:
- hipergeusia

Una recién nacida es evaluada por el neonatólogo y evidencia que el canal anal está
completamente cerrado. Este problema se debe probablemente a una anomalía en el
desarrollo de:
- la membrana cloacal

Durante un experimento, se insufla rápidamente dos litros de agua en un globo colocado


dentro del estómago de un voluntario. ¿cuál de las siguientes situaciones del músculo liso
será consecuencia directa de este cambio de volumen en el estómago?
- Despolarización

En muchos países se usa el suplemento de fluor en el agua potable o los dentríficos, con el
fin de hacer el esmalte más resistente a la desmineralización inducida directamente por:
- el ácido

La sensación del gusto depende de la presencia de papilas gustativas en la lengua, las cuales
tienen corpúsculos gustativos conteniendo células neuroepiteliales sensoriales. Estas células
neuroepiteliales pueden ser dañadas fácilmente, por suerte, su tiempo de recambio es de
alrededor de:
- 10 días

Paciente de 56 años con accidente cerebrovascular reciente. En la resonancia se observa


daño de los núcleos laterales del hipotálamo. Por este motivo es muy probable que el
paciente sufra de:
- Inanición
Durante el desarrollo de la región cloacal, una cuña de mesodermo ubicado entre el
alantoides y el intestino posterior vendrá a formar el:
- tabique urorrectal

En el conducto anal se encuentra la unión entre las regiones del endodermo y el ectodermo,
esta unión se evidencia al observar:
- la línea pectínea

En una persona sana, el momento adecuado para encontrar los mayores niveles de grelina
en sangre sería:
- antes de come

Dentro de la estructura de los dientes, la parte del diente cubierta por esmalte y que se
puede ver mediante la inspección visual de la boca se denomina
- corona clínica

El esófago en su microestructura tiene básicamente adventicia, a excepción de la región


distal, donde tiene serosa, específicamente a partir del nivel de:
- T10

En un paciente con arcadas, se debe considerar que durante la ocurrencia de dichas arcadas,
debemos encontrar contenido gástrico en:
- tórax

El duodeno está constituido por el segmento terminal del intestino anterior y el segmento
proximal del intestino medio ¿Cuál de las siguientes alternativas describe mejor este lugar
de unión entre los dos intestinos?
- Distal al origen de la yema hepática

En una persona sana, el uso de atropina producirá a nivel del estómago:


- Aumentará el pH del estómago

En una persona sana, el consumo de leche produce indirectamente


- Inhibición del vaciamiento gástrico

La motilidad del colon es importante y lenta comparada con la del intestino delgado. Los
movimientos en masa ocasionan la:
- distensión rectal
Con respecto a la motilidad gástrica, los potenciales de acción disminuyen en frecuencia por
efecto de:
- el péptido insulinotrópico dependiente de glucosa

La sensación del gusto depende de la presencia de papilas gustativas en la lengua, de las


cuales, algunas de ellas tienen un surco terminal por donde drenan unas glándulas salivales
linguales (llamadas glándulas de von Ebner). Esta descripción se refiere a las papilas:
- circunvaladas
1. Un niño de 2 años es llevado a la consulta por diarrea persistente y edema de
las extremidades, además falta de crecimiento y desarrollo en relación a su
edad. Los análisis de sangre revelan que tiene concentración plasmática baja de
proteínas (hipoproteinemia). Durante la endoscopía duodenal, se coloca
colecistocinina (CCK) endovenosa y se recoge muestras del líquido duodenal; el
resultado del líquido confirma incapacidad para hidrolizar proteínas a un pH
neutro, esta situación mejora al añadir una pequeña cantidad de tripsina. El
paciente probablemente esté sufriendo la falta congénita de
-Enterocinasa
2. Experimentalmente se incrementa la velocidad de la secreción salival con
una sustancia, el análisis de la composición de esta saliva obtenida se
espera encontrar…………..
-Disminución de concentración de potasio
3. Paciente varón de 46 años soltero, consulta por odinofagia y bajo de peso, tiene
antecedente de tuberculosis desde hace 3 meses y es fumador crónico (10
cigarrillos por día); al evaluar la cavidad oral se identifica lesión blanquecina en
el dorso de la lengua y paladar blando, las lesiones se desprenden con el baja
lengua dejando una base eritematosa. Esta lesión corresponde probablemente
a
……………………….
…..
-Candidiasis oral
4. Paciente mujer de 35 años acude a consulta por sensación de sequedad y
lesiones en cavidad oral. Al examen se observa atrofia de la mucosa, fisuras y
úlceras; nota además sequedad e irritación de la córnea y aumento del tamaño
de las glándulas parotídeas. Su diagnóstico más probable es artritis reumatoide;
el hallazgo más probable en una biopsia de glándula parótida es……..….
-Gran infiltración de linfocitos y células plasmáticas
5. Un paciente con anemia acude con su médico quejándose de episodios
frecuentes de gastroenteritis. Un análisis de sangre revela anticuerpos
circulantes dirigidos contra células parietales gástricas. Su anemia es atribuible
a la hiposecreción de
-Factor intrínseco
6. Dos estudiantes deciden tomar un receso para comer una hamburguesa a la
hora del almuerzo. Antes de llegar a la cafetería, impulsos nerviosos
provenientes del complejo vagal dorsal iniciarán la secreción de ácido gástrico
por la liberación de
…………………….. desde el sistema nervioso entérico.
-GRP
7. Un niño de cuatro años de edad es llevado a la consulta por cuadros diarreicos
frecuentes caracterizados por heces pálidas, voluminosas y fétidas, presenta
bajo peso y talla. Se mide la concentración de cloruro en el sudor y se encuentra
que sus valores son muy elevados. La alteración más importante a nivel de
células ductales del páncreas tiene relación directa con la conductancia
de…………
-Cloro
8. Una mujer de 50 años de edad que sufrió durante varios años resequedad de
los ojos debida a producción inadecuada de lágrimas es enviada con un
gastroenterólogo para evaluación de pirosis crónica. El examen endoscópico
revela erosiones y tejido cicatrizal en la parte distal del esófago justo por
arriba del esfínter esofágico inferior. Las lesiones pueden atribuirse a la
disminución de uno de los siguientes componentes salivales:
-Bicarbonato

9. Se evalúa los valores séricos de las siguientes sustancias a un paciente con


enfermedad hepática terminal; en este paciente se espera encontrar la
combinación con la letra …………
-disminuida, aumentada, disminuida
10. Una mujer de 35 años de edad HIV positiva, se presenta al médico con dolor
abdominal en cuadrante superior derecho e ictericia. La paciente refiere haber
tenido múltiples episodios de ictericia durante los últimos 10 años. Los exámenes
para determinar hepatitis viral, dieron positivos para Hepatitis B, siendo
catalogado el caso como hepatitis crónica con alteración funcional. En un
examen de sangre ¿cuál de los siguientes parámetros está disminuido?
-Albúmina
11. En el reflejo peristáltico del intestino delgado, uno de los siguientes eventos
sucede en la porción oral del bolo alimenticio…………...
-Acción de acetilcolina en el músculo circular
12. Experimentalmente se coloca una dosis alta de secretina en la luz
intestinal duodenal; como consecuencia de esto, en el jugo pancreático de
la misma luz intestinal se observa la disminución de la concentración de
…..………..
-Cl
13. Un varón de 58 años de edad con enfermedad de Crohn severo fue sometido a
una resección ileal. Después de la cirugía este paciente padecerá de esteatorrea,
esto se explica porque …..………..
- La micelas no pueden formarse
14. En un experimento se inserta un balón en el estómago de un voluntario, se infla
poco a poco mientras que se vigilan las presiones intraluminales. Aunque el
volumen del balón aumenta considerablemente, las presiones permanecen
constantes. Esta relación volumen-presión se explica por la liberación local de
…………..
-Óxido nítrico y péptido inhibidor vasoactivo
15. La toxina de Vibrio cholerae causa diarrea debido a…….
-El Incremento de la secreción de cloro por las células de la cripta intestinal
16. ¿Cuál de las siguientes alternativas es una característica de la secreción
exocrina del páncreas?
-Tiene una baja concentración de Cl- respecto al plasma
17. Una madre lleva a su hijo de dos años de edad a la sala de urgencias, estresada
porque el niño deglutió una moneda de 10 céntimos mientras la familia cenaba en
un restaurante. El médico observa mediante fluoroscopía que la moneda se halla
en el estómago y asegura a la madre que la moneda se eliminará con las heces.
El médico recomienda utilizar la respuesta fisiológica que permitirá la evacuación
de la moneda del estómago al intestino ………….…..
-Son los movimientos de mezcla y trituración
-. Es provocada por el ayuno
18. Las estructuras en el hígado que permite que los productos metabólicos unidos a
proteínas tengan acceso a las membranas basolaterales de los hepatocitos,
son….. -Las fenestras sinusoidales
19. La composición de la bilis es modificada conforme fluye por los conductillos
biliares. Durante este tránsito se espera que aumente la concentración de…….
-Monómeros de ácido biliar
-Ig A
20. Se mide experimentalmente el contenido gástrico de dos personas. La persona
“A” tiene alto contenido de grasa y la persona “B” tiene un contenido hipertónico
¿Cuál de las siguientes es correcto respecto al vaciamiento gástrico?
- Hay ralentización del vaciado gástrico en ambos casos
21. El examen endoscópico de un paciente con hipertensión portal grave revela
venas tortuosas que sobresalen hacia la luz del esófago. El paciente recibe
tratamiento quirúrgico mediante la colocación de una derivación que conecta la
vena porta a la vena cava. Después de la operación el riesgo de encefalopatía y
el
riesgo de sangrado de várices ……………..
-Aumentará/disminuirá
22. Un paciente varón de 18 años de edad acude al médico para sus exámenes
de rutina. Sus resultados de laboratorio muestran un valor de bilirrubina
sérica de 4 mg/dl y una bilirrubina directa de 0,3 mg/dl. Las pruebas de
función hepática son normales. La alteración que explica mejor este caso es
por la deficiencia de
………………..
-Glucuronil transferasa
23. Un hombre de 57 años de edad es llevado a urgencias con hematemesis
masiva rojo brillante, a su llegada se halla inconciente con PA: 80/40 mm Hg y
FC: 124 lat/min. Luce ictérico con presencia de “arañas vasculares en el tórax
anterior y extremidades”, abdomen distendido con signo de oleada positiva.
Se encuentra esplenomegalia y pérdida de la masa muscular en
extremidades. La anastomosis vascular responsable del sangrado en este
paciente es
-Vena gástrica izquierda y vena ácigos
24. Un estudiante de medicina está comiendo un plato de comida a base de
champiñones, espárrago y salsa de soya. El sabor umami contenido en todos
estos alimentos actúa a nivel de los botones gustativos estimulando
………………..
-Un receptor acoplado a proteína G
25. Un hombre de 22 años de edad se presenta al médico con una historia de 1 año
de evolución caracterizado por dolor recurrente en fosa iliaca derecha y diarrea.
Manifiesta además pérdida de peso de 8 kg durante este periodo. La
colonoscopía revela múltiples lesiones en el ileon terminal y colon. La biopsia
de estas lesiones revela engrosamiento, inflamación y ulceración de la mucosa.
El diagnóstico más probable en este caso es…….
-Enfermedad de Crohn
26. Varón de 61 años que consulta por dolor retro esternal intenso desde hace 6
horas y después de vómitos intensos y repetidos; al examen se observa disnea,
cianosis, hipotensión y signos clínicos de shock. La radiografía simple de tórax
muestra neumomediastino. El líquido en el espacio pleural aspirado tiene alta
concentración de amilasa. ¿Cuál de las siguientes alternativas puede explicar
este cuadro clínico? -Rotura espontánea de esófago
27. La secreción del ácido en la célula parietal gástrica se lleva a cabo por una
ATPasa especifica que intercambia hidrogeniones (H+) del citosol por…..
-K +
28. En condiciones normales el ingreso de 600 ml de líquido es el estómago provoca
un aumento de presión intragástrica de unos 12 cm de H2O. Después de una
vagotomía (corte del nervio vago) es de esperar que el ingreso del mismo
volumen de líquido provoque lo siguiente: …………………………………
-Un aumento mayor de la presión
29. Una paciente de 30 años de edad es sometida a una cirugía en oído medio
derecho por un problema de otoesclerosis. Luego de la cirugía refiere alteración
en la percepción de sabores. Al evaluar el caso usted esperaría
encontrar……….
-Alteración en la sensación del gusto en los dos tercios anteriores de la
lengua
-Sensación del dolor, tacto y temperatura conservada en toda la lengua

30. ¿Cuál de las siguientes alternativas es correcta?


-Las sales biliares desconjugadas son absorbidas preferentemente en el
colon
31. En un paciente de 45 años de edad con colestasis biliar, se encuentra una
elevación de los niveles sanguíneos de fosfatasa alcalina hasta 3 veces la cifra
normal. ¿Cuál de las siguientes alternativas estará también elevada como
evidencia del daño de la vía biliar?
-Gamma glutamil transpeptidasa
32. Revisando la angiografía de un hombre de 70 años en estudio por aneurisma de
aorta abdominal el radiólogo informa de la presencia de una oclusión completa
de la arteria mesentérica inferior. El paciente se encuentra completamente
asintomático.
¿Cuál de las siguientes arterias se anastomosa a la sistema arterial
de la mesentérica inferior?
-Cólica media
33. Lactante de 3 meses de vida es atendido por presentar diarrea, se administra
una solución de glucosa y electrólitos por vía oral. La proteína de membrana
apical que explica la capacidad de esta solución para proporcionar aporte de
glucosa e hidratación es ………..
-SGLT-1
34. Paciente ha sufrido herida de bala en el abdomen, se le ha tenido que extirpar el
segmento medio y distal del ileon. En este caso la síntesis hepática de sales
biliares estará …..…..
-Incrementada por estímulo de la enzima colesterol 7 alfa hidroxilasa
35. Un varón de 75 años ingresa al consultorio por presentar ictericia marcada de
piel y las escleras. El estudio del paciente mostró que presentaba un tumor que
obstruía la totalidad del conducto hepático común. ¿Cuál de las siguientes
estructuras se encontrará dilatada en este paciente?
-Conductos de Hering
36. En un paciente con insuficiencia renal crónica, el déficit en la absorción de
calcio a nivel del enterocito se debe a lo siguiente:
-No se convierte la 25 hidroxicolecalciferol a 1,25 dihidroxicolecalciferol
37. Varón de 30 años es traído a emergencia por agresión abdominal con arma de
fuego (pistola) y es sometido a laparotomía exploratoria, observándose isquemia
del colon ascendente y parte del colon trasverso ¿la lesión de cuál de las
siguientes arterias explicaría esta isquemia?
d. Mesentérica superior
38. Respecto a las sustancias gastrointestinales que regulan la secreción pancreática;
marque la afirmación correcta:
b. La acetilcolina es capaz de estimular la secreción enzimática y de bicarbonato
del páncreas
39. Ante una lesión del X par craneal, ¿cuál de los siguientes músculos mantiene
conservada su función?:
b. Tensor del velo del paladar
40. Experimentalmente se utiliza atropina (anticolinérgico) para inhibir la secreción
de gastrina, sin embargo la secreción de esta hormona se sigue dando ante
estímulos vagales. Esta situación se explica porque la atropina:
d. No bloquea la acción del péptido GRP

41. Un varón de 50 años es sometido a extirpación del duodeno y parte proximal del
yeyuno. Esta situación ocasionaría la pérdida de las células ……….. ,
productoras de ………………… que estimula la secreción de bicarbonato por el
páncreas.
“S” / secretina
42. Se evalúa la expresión de la proteína Agrp en una persona con alteración del
apetito; lo correcto respecto a esta proteína es…..
La mutación del gen que la codifica produce adelgazamiento
43. Juana cae de la bicicleta y se fractura la región anterior del hueso maxilar superior
con compromiso de la fosa incisiva. Al examen físico de la región esperaría
encontrar alteración en la sensibilidad de la encía …………………
palatina anterior
44. Recién nacido es atendido por el neonatólogo y luego entregado a su madre para
dar de lactar; la madre al dar de lactar observa coloración azulada de labios,
acompañado de tos persistente, dificultad respiratoria y distención abdominal. Se
le intenta colocar una sonda nasogástrica pero esta retorna a la cavidad oral en
todos los intentos. ¿Cuál de las siguientes anomalías del desarrollo es el más
probable en este caso?
b. Atresia esofágica proximal con fístula traqueoesofágica distal l
45. ¿Cuál de los siguientes mecanismos ocurre durante la defecación?
En la posición de “cuclillas” el músculo puborrectal se halla relajado
46. Un paciente luego de un accidente sufre lesión del piso de la boca, se constata
daño del nervio “cuerda del tímpano”, en este caso se esperaría en
47. contrar disminución de la………………………….… de la lengua
Sensación del gusto en los dos tercios anteriores
48. ¿Cuál de las siguientes afirmaciones es la correcta sobre la gastrina?
Actúa en la célula diana mediante su receptor CCk tipo B
49. Al recibir un paciente con signos de hipovolemia y antecedente de trauma en
abdomen por accidente de tránsito, usted identifica radiológicamente: lesión de
primera vértebra lumbar y signos de lesión en páncreas; durante la cirugía se
observó pobre irrigación de asas intestinales. El vaso afectado es la arteria ……..
c. mesentérica inferior
50. Un paciente sufre de daño a nivel del cuello con lesión muscular en la región de la
faringe. En el examen físico se determina dificultad para la elevación de la faringe y
para el cierre del itsmo de las fauces. En este caso, probablemente esté afectado el
músculo:
c. palatofaringeo
51. Varón de 50 años a quien le realizan la curación de la segunda molar de la
arcada superior derecha. En un momento determinado, el paciente acusa de
intenso dolor de la pieza dentaria en tratamiento. La vía aferente del dolor
viaja a través del nervio …………
a. trigémino V2
52. La distención gástrica por los alimentos produce incremento de secreción de
HCl mediante la producción de ………….. que estimula a las célulasvía
proteína
………..
a. gastrina / parietal / Gq

53. Un niño de tres años llega a emergencia con disfagia (dificultad para tragar),
dolor retro esternal, salivación y llanto. Se sospecha de ingesta de cuerpo
extraño (moneda) en el esófago; al ser evaluado se constata en una radiografía
presencia de cuerpo extraño a nivel de C6 (6° vértebra cervical). El cuerpo
extraño estará suspendido a nivel del estrechamiento producido por………..
c. el músculo cricofaríngeo
54. La triada portal (arteria hepática, vena portal y conducto biliar común) está
contenida en el ligamento …….……… y derivan embriológicamente del ……
a. hepato duodenal / mesenterio ventral
55. Un paciente refiere no percibir algunos sabores, al examen físico se constata
alteración en la percepción de sabores y del dolor en el tercio posterior de la
lengua ¿Cuál de los siguientes nervios estará alterada en su función?
c. Glosofaríngeo (IX par)
56. En el caso de un paciente con gastrinoma (tumor productor de gastrina), la
presencia de úlceras duodenales y erosión de la mucosa gástrica, se debe
principalmente a…….
c. el exceso de HCl por estímulo de receptores CCK-B en la célula parietal
56. El reflejo entero gástrico se caracterizan por:
d. originarse debido a la distensión duodenal y presencia del quimo ácido
57. Mauricio tiene dificultad para deprimir el paladar y elevar la parte posterior de la
lengua. En este caso estará afectado un músculo, específicamente el músculo
…………….
b. extrínseco – palatogloso
58. En condiciones normales, el ingreso de 600 ml de líquido es el estómago provoca
un aumento de presión intragástrica de unos 12 cm de H2O. Después de una
vagotomía (corte del nervio vago) es de esperar que el ingreso del mismo volumen de
líquido ocasione
………………………………… de la presión
intragástrica.
c. un aumento mayor
59. La explicación fisiológica de presentar somnolencia de 30 minutos a 1 hora
después de ingerir alimentos, se explica por: a. Aumento del cloro intraluminal
e. Aumento de la alcalinidad sanguínea
60.Se presenta un paciente, el cual presenta un antecedente de tuberculosis
intestinal, por lo cual, se le resecó 80 cm de íleon distal. Desde el punto de vista
fisiológico, el paciente puede presentar una de las siguientes alteraciones: a.
Disminución de la secreción de Vitamina B12

e. Disminución de la absorción de ácido glicocólico


61. Un paciente es sometido experimentalmente a un fármaco que modifica el flujo
salival, obteniéndose un volumen de saliva de 288 ml en 6 horas. En este caso las
concentraciones de electrolitos y bicarbonato en la saliva obtenida varían de la
siguiente manera: a. ↑ Na+,
↓ K+, ↑ Cl-, ↑ HCO3-

1. b. ↓ Na+, ↓ Cl-, ↑ K+, ↓ HCO3-

62. Uno de los siguientes elementos debería hallarse con más probabilidad en el
esófago de un paciente que sufre de reflujo gastro esofágico…
a. Pepsina

63. Un paciente de 40 años cursa con anemia de 8g/dl, aqueja además de astenia y
sensación de hormigueo bilateral en los miembros inferiores, al examen se halla alteración
de la sensibilidad a la vibración y camina con ampliación de la base de sustentación. Uno
de los siguientes procedimientos sería de ayuda para el diagnóstico de este paciente:
a. Tomografía cerebral
b. Biopsia de la mucosa gástrica

64.Paciente de 60 años ingresa por caída hace 1 hora y pequeño hematoma en cuero
cabelludo, al examen físico ampliado se observa ictericia de piel y mucosas generalizada,
abdomen blando, se palpa estructura quística no dolorosa en hipocondrio derecho que
corresponde a vesícula biliar (signo de Courvoisier), en los exámenes de laboratorio se
halla niveles bajos en la formación de estercobilinógeno y urobilinógeno en heces,
incremento de la bilirrubina conjugada en la orina, elevación de fosfatasa alcalina y
gamma glutamil transpeptidasa séricas. El presente cuadro puede ser explicado por: a.
Reabsorción de hematoma
c.Carcinoma de la cabeza de páncreas
65. Un recién nacido presenta vómitos biliosos poco tiempo después de cada alimento.
Al preguntar a la madre sobre antecedentes, ella recuerda que tuvo polihidramnios
durante la gestación, pero un análisis de cariotipo fue normal. Una de las siguientes es la
causa más probable de estos hallazgos en el recién nacido: a. Enfermedad de
Hirschprung
e. Malrotación de la yema pancreática ventral
66.En un estudio de la secreción de hormonas gastrointestinales, sus concentraciones en
la vena porta se midieron durante perfusión luminal del intestino delgado con soluciones
de diversas magnitudes de pH. ¿Qué hormona aumentará en el plasma de la vena porta
durante perfusión a través del intestino con una solución de pH 3?
a. CCK
e. secretina
67.Paciente de 30 años que ingresa a causa de un traumatismo abdominal cerrado. En la
exploración se aprecia discreta palidez de piel y mucosas, auscultación pulmonar normal,
taquicardia de 120 /min. Discreta distensión abdominal y matidez en flancos; el
hematocrito, que era prácticamente normal al ingreso, disminuye a 30% a las tres horas.
En la Rx de tórax se objetiva fractura de las costillas 10-11 izquierdas. La causa más
probable de la anemización en este paciente es: a. traumatismo renal con hemorragia
retroperitoneal.
c. rotura de bazo con hemoperitoneo.
68. Revisando la angiotomografía de un hombre de 70 años en estudio por aneurisma
de aorta abdominal, el radiólogo le informa de la presencia de una oclusión completa
de la arteria mesentérica inferior. El paciente se encuentra completamente
asintomático. La oclusión de la arteria mesentérica inferior cursa de manera
asintomática en muchas ocasiones ya que el territorio que irriga puede recibir flujo
proveniente de la arteria:
a. cólica derecha
e. cólica media

69. En las patologías de esófago es importante conocer bien la anatomía esofágica. ¿Cuál
de las siguientes afirmaciones es correcta? a. El esófago tiene capa mucosa, muscular y
serosa

c. El esófago torácico pasa por detrás del cayado aórtico

70. A pesar de que pueda haber variaciones anatómicas, lo habitual es que el ciego
sea irrigado por una rama arterial que proviene de unas de las siguientes arterias:
a. Iliaca derecha

d. Mesentérica superior

71. Ante un paciente con una cirugía abdominal urgente, el informe operatorio señala
que se ha realizado una resección de todo el duodeno y del tercio proximal del yeyuno
manteniendo íntegros el estómago y todo el íleon, así como los dos tercios distales del
yeyuno. En el seguimiento nutricional del paciente ¿Qué vitamina o mineral presentará
con menor probabilidad una disminución de su absorción?
a. Cianocobalamina

72. ¿Cuál de las siguientes sustancias forma parte de la secreción biliar? a. Tripsina
Lecitina

73.¿De qué musculo forma parte el ligamento inguinal?


-Oblicuo externo del abdomen
74.¿Cuál de las siguientes enzimas está localizada en el borde en cepillo y juega un rol
en la digestión de proteínas?
e. Carboxipeptidasa A.
75. Una de los siguientes sustancias, NO sirve como un buen agente emulsificante:
a. Colesterol
76. La sustancia que estimula el crecimiento de la mucosa gástrica es:

a. Secretina

d. Gastrina

77.¿Cuál de las siguientes alternativas es una función de la colecistokinina?

a. Relajación de la vesícula para la salida de bilis

d. Secreción de enzimas pancreáticas

78.Con respecto a la anatomía del tronco celiaco, señale lo correcto a. El tronco celiaco
se origina de la cara posterior de la aorta abdominal
d. La hepática común que es una de sus ramas, participa en la irrigación del
estómago.
79. Con respecto a la anatomía del duodeno, marque la respuesta correcta: a. Tiene
una distribución en forma de “C”, que rodea la cola del páncreas
b. La 3ra porción duodenal está contenida en la pinza vascular aortomesentérica
80. En el íleon se absorbe aproximadamente el 95% dea través de la circulación
enterohepática.
a. agua
c. sales biliares
81. La estimula el mecanismo paracrino de la secreción de ácido clorhídrico.
a. histamina
82.En la digestión de proteinas,es el principal estímulo para convertir el
pepsinógeno en pepsina. a. la gastrina
b. el pH ácido
83. Con respecto a la somatostatina, marque lo correcto:
a. Es secretada por las células S del intestino
Interviene en la fase intestinal de la secreción gástrica
84. En pecten anal, es una estructura comprendida entre: a. la línea pectínea y los
senos anales
d. la línea anocutánea y la línea pectínea

85.¿Cuál de las siguientes alternativas es una proenzima pancreática? a. Tripsina

1. b. Elastasa
2. c. Quimotripsinógeno
3. d. Amilasa
4. e. Procarboxipeptidasa C
86. En este paciente, el bloqueo farmacológico de los receptores H2 en la mucosa gástrica:
a) No tiene efecto sobre la secreción de ácido inducida por el vago
b) Evita la activación de adenilciclasa por gastrina
c) Inhibe la secreción de ácido inducida por gastrina y mediada por el vago
d) Causa un aumento en el transporte de potasio por las células parietales gástricas
Se validó la A :)
87. Si se considerara una gastrectomía total para curar la gastritis del paciente, cuál
de las siguientes sustancias ya no se produciría:
a) Gastrina
b) Quimiotripsina
c) Amilasa
d) Pepsinógeno

88. Un paciente hipertenso está tomando un medicamento bloqueador de receptores


alfa 1 adrenérgicos (prazosina) y como efecto secundario se queja de:
d) Lo escaso que es el medicamento
e) No tiene problemas en la salivación
c) Hiposalivación
d) Hipersalivación

89. Con respecto a las lesiones y enfermedades de la boca, marque lo correcto:


f) La leucoplasia se desprende al roce
b) la eritroplasia puede degenerar en adenocarcinoma
c) El muguet oral es una enfermedad bacteriana en inmunodeprimidos
d) la eritroplasia debe ser biopsiada
NOTA: fue validada la opción B ya que no es motivo del curso que sepamos el puto
cáncer.

90. En este paciente, se puede asumir que la pancreatitis ha sido ocasionada por
una disminución en el efecto de:
g) Amilasa
h) Lipasa
c) Inhibidor de la tripsina
d) Entercinasa
91.Un efecto secundario en el estómago por la acción de la secretina es:
i) Disminución en la liberación de pepsinógeno
b) Menor actividad de la pepsina
c) Mayor paso de bicarbonato a sangre periférica
d) Aumento en la producción de factor intrínseco
92. Dentro de los factores protectores de la mucosa gástrica se pueden mencionar
múltiples protagonistas. Uno de ellos es:
j) CCK
k) Gastrina
c) Receptor muscarínico
d) Pepsina
93. La saliva puede tener una variedad de electrolitos en su composición. Entre ellos el
cloro, respecto al cual se puede afirmar:
l) Su mayor concentración se consigue con el flujo bajo
b) Su concentración no llega a ser tan alta como en el plasma
c) Con flujo alto su concentración es mayor que la del plasma
d) Su menor concentración se alcanza con flujo alto
94. En el síndrome de boca seca o síndrome de Sjogren, una de las
complicaciones asociadas es:
a) caries
b) Disminución de la acidez gástrica
c) Aumento en de la producción de saliva
d) Aumento del pH bucal
95. Estimula la producción de
saliva:
a) Vasodilatación periglandular
b) Atropina
c) Fatiga o cansancio
d) Expresión de miedo
96. El omeprazol actúa sobre la membrana de la célula
m) Basolateral/principal
n) apical/principal
o) Basolateral/parietal
d) apical/parietal
97. Para protegerse del entorno ácido, el Helicobacter pylori se autogenera un entorno
de pH menos ácido alrededor suyo, gracias a una enzima que alcaliniza su entorno
local mediante la conversión de:
a) urea en NH3
b) H2O y CO2 en ácido carbónico
c) NH3 en urea
d) H2CO3 en bicarbonato
98. La anemia perniciosa destruye las células:
p) mucosas del cuello
b) oxínticas
c) principales
d) mucosas superficiales
99. La célula mucosa del cuello gástrico produce:
a) Moco
b) ácido clorhídrico
c) pepsinógeno
d) Factor intrínseco
100.El aumento en la acidez del estómago producido principalmente por la
infección de Helicobacter pylori se debe a la disminución de:
a) Somatostatina
b) Bicarbonato por las glándulas de Brunner
c) Secretina
d) Colecistoquinina
101. De las siguientes sustancias secretadas por los órganos de este paciente, la
más alcalina es la secreción:
a) pancreática
b) Esofágica
c) Yeyunal
d) Salival
102.En cuanto a la gastritis de este paciente, se encontró que era producida por la
bacteria Helicobacter pylori. Esta bacteria sobrevive en el medio ácido del estómago
gracias a:
a) ácido clorhídrico
b) Toxina CagA
c) Ureasa
d) Jugo pancreático
103.La lengua está recubierta por epitelio:
c) pseudoestratificado columnar no queratinizado
b) plano estratificado no queratinizado
c) pseudoestratificado columnar ciliado
d) plano estratificado queratinizado
104.El esfínter anal interno tiene musculatura …….. y tiene control ……..
d) lisa / voluntario
b) lisa / involuntario
c) esquelética / simpático
d) esquelética / parasimpático

19) La arteria Aorta proporciona la irrigación al tubo digestivo ¿cuál de las


siguientes arterias proporciona la irrigación al ángulo cólico derecho?
a) mesentérica superior
b) mesentérica inferior
c) frénica inferior
d) tronco celiaco

20) Paciente de 26 años que le cuenta en su historia clínica que cada vez que almuerza a
los 20 min tiene deseo de defecar, le comenta que su hijo de 1 mes le pasa lo mismo
pero más intenso. Esto se explica por el reflejo …….., el cual está …… en el paciente
a) colicoileal / normal
b) colicoileal / alterado
c) gastrocólico / normal
d) gastrocólico / alterado

21) La región del estómago que se comunica con el duodeno es la


a) pilórica
b) cardias
c) cuerpo
d) fórnix

22) Acude a consulta un px que fue diagnosticado de úlcera péptica 3 días antes. Luego
de múltiples pruebas diagnósticas se concluye que el paciente presenta un tumor
secretor de gastrina, ¿cual de las siguientes situaciones estará incrementada?
a) distensión gástrica
b) inhibición del vaciado gástrico
c) secrecion de acido clorhidrico
d) inhibición de la secreción de pepsinógeno
23) En el sistema digestivo, el control del apetito está dado por un complejo sistema de
sustancias y órganos integradores, los cuales regulan la ingesta de alimentos. La
…… es una sustancia orexígena y es sintetizada por el ……
a) leptina / estómago
b) felina / intestino
c) leptina / estómago
d) grelina / estómago

24) Con respecto a la actividad eléctrica del sistema digestivo, marque la


alternativa correcta
a) corresponden a potenciales de acción que están presentes de forma continua
y le dan capacidad de peristalsis autónoma al sistema digestivo
b) la frecuencia de las ondas lentas NO se ve influenciada por la actividad neural ni
las
hormonas gastrointestinales
c) en el estómago las ondas lentas se dan en una frecuencia de 6 x min
d) las ondas lentas son cambios lentos y ondulantes del potencial en reposo
e) la frecuencia de las ondas lentas va de 6 a 12 ondas por minuto

25) Ante una lesión del IX pc, el músculo …… se altera en su función


a) palatogloso
b) estilofaríngeo
c) palatofaríngeo
d) constrictor superior

26) Un varón de 50 años es sometido a extirpación del duodeno y parte proximal del
yeyuno. La pérdida de estímulo hormonal en el páncreas para la secreción
enzimática se explica por la pérdida de células
a) parietales, productoras de factor intrínseco
b) K productoras de factor intrínseco
c) M productora de CCK
d) I productora de CCK

27) Marque la respuesta correcta:


A. El bronquio derecho constituye una de las estrecheces del esófago
B. Todos los órganos del sistema digestivo tienen capa serosa
C. La pared gástrica en el fondo es más delgada que en el cuerpo y antro
D. El esfínter de Oddi rodea a la papila menor duodenal

28) Marque la respuesta correcta en relación a la gastrina:


A. Al distenderse el estómago, se inhibe su producción.
B. Se estimula por la liberación de noradrenalina
C. Las células G son las productoras y se encuentran principalmente en el antro
gástrico
D. Las células G se encuentran principalmente en el fondo gástrico

29) Para poder morder una manzana, es necesario usar el siguiente músculo:
A. Milohiodeo
B. Tensor del paladar
C. Orbicular de los labios
D. Buccinador
30) Sustancia que inhibe la secreción y la motilidad del estómago prolongando el tiempo
de digestión:
A. Enteroglucagon.
B. Polipéptido pancreático
C. Péptido 1 similar al glucagón (GLP-1).
D. Péptido insulinotrópico dependiente de la glucosa (GIP).

31) El nacimiento de la arteria mesentérica superior se puede encontrar en cuál de


los cuadrantes abdominales:
A. Hipocondrio derecho
B. Hipogastrio
C. Epigastrio
D. Mesogastrio
32) Entre las múltiples causas de la Enfermedad por Reflujo Gastroesofágico, se puede
considerar también a una alteración en las del esfínter esofágico inferior:
A. Ondas secundarias
B. Contracciones tónicas
C. Ondas lentas
D. Glándulas subesofágicas

33) Producto de la alimentación, se producen diversas sustancias peptídicas, cininas


y bradicininas, las cuales permiten que:
A. Se produzca neovascularización en los territorios de las arterias abdominales
B. La acción de la lipasa pancreática se vea incrementada
C. El consumo de O2 del intestino aumente ligeramente
D. El flujo sanguíneo intestinal aumente hasta 8 veces

34) El dolor periumbilical o epigástrico en el inicio de una apendicitis aguda se debe a:


A. Estímulo del nervio vago.
B. Íleo secundario.
C. Irritación del peritoneo parietal.
D. Estímulo del sistema simpático.

35) El aumento en la actividad motora de la pared gástrica genera un aumento en


los niveles locales de qué sustancia en la microvasculatura:
A.Adenosina
B. Colecistoquinina CCK
C. Endotelina
D. Gastrina

36) ¿Cuál de los siguientes péptidos inhibe el vaciamiento gástrico?


A. Colecistoquinina
B. Péptido inhibidor gástrico
C. Motilina
D. Gastrina
37) Los músculos de la masticación que producen la retropulsión de la mandíbula son:
A. temporales [mas seguro]
B. maseteros
C. milohioideos
D. pterigoideos
38) En relación a la fisiología gástrica, marque lo correcto:
A. la cimetidina actúa en la región basolateral de la célula parietal
B. la marea alcalina se debe al paso de bicarbonato través de la membrana apical
de la célula principal
C. el cloro difunde hacia el exterior por la la región basolateral de la célula parietal
D. la salida de hidrogeniones a la luz es por difusión facilitada

39) Durante el sueño, la concentración de bicarbonato en la saliva:


A. Se eleva a niveles mayores que los del plasma
B. Aumenta
C. No tiene efecto
D. Disminuye

40) La secreción de saliva es importante en la fisiología digestiva. Su concentración


de potasio llega a ser menor que la del plasma cuando su secreción tiene un flujo:
A. Intermedio
B. Nunca
C. Bajo
D. Alto

42) Respecto a las glándulas salivales, marque lo incorrecto:


A. la glándula parótida produce secreción serosa
B. la glándula sublingual drena a través de conducto de Wharton
C. La glándula parótida drena a través del conducto de Stenon
D. la glándula sublingual tiene forma de garfio

43) Con respecto a la saliva, marque la respuesta correcta:


A. será hipertónica cuando el flujo es bajo
B. a mayor flujo, menor concentración de Na
C. a mayor flujo, menor concentración de cloro
D. el sistema simpático estimula su secreción
CI 3
44) En relación a la circulación hepática, marque lo correcto:
a) Los sinusoides hepáticos transportan sangre mixta
b) La vena porta proporciona el 50% de sangre al hígado
c) La vena porta se forma a partir de la vena esplénica y la mesentérica inferior
d) La arteria hepática deriva de la mesentérica superior

45) Dentro de las funciones de las células de Ito, marque lo incorrecto:


a) Sintetizan colágeno
b) Almacenan vitamina A
c) Se les llama células estrelladas
d) Pueden fagocitar patógenos y actúan como presentadoras de antígeno
46) Paciente con tumor neuroendocrino productor de secretina, debido a lo cual se puede
esperar que su secreción pancreática, comparada con la de una persona sana de bajo
flujo, tenga una concentración de:
a) Sodio aumentada
b) Igual
c) Bicarbonato aumentada
d) Potasio disminuida
47) El GALT se localiza en:
a) Lámina propia
b) submucosa
c) borde en cepillo
d) superficie de criptas de Lieberkühn

48) En relación a la histología hepática, marque lo correcto:


a) la zona 1 se afecta rápidamente en estados de hipovolemia y shock
b) La zona 1 se encuentra cercana a la vena central lobulillar
c) La zona 3 se encuentra más cerca a la vena central lobulillar
d) La zona 3 se encuentra más cerca al eje menor del acino hepático
49) El acino pancreático difere con el de las glándulas salivales en:
a) Contiene células centroacinares
b) No produce secreción serosa
c) El páncreas produce principalmente secreción mucosa
d) No tienen diferencias
50) Durante la digestión de las grasa, para que la lipasa actúe adecuadamente se requiere
que el pH aumento en la luz intestinal, lo cual es logrado, entre otros, por la secreción
de las células:
a) Del conducto interlobulillar
b) Centroacinares
c) Acinares
d) Alfa
51) La secreción de la colecistoquinina (CCK) se produce en la fase:
a) intestinal
b) En las 3 por igual
c) gástrica
d) Cefálica
52) ¿Por cuál de las siguientes células es secretada principalmente la pro
enzima procarboxipeptidasa?
a) Acinares del páncreas
b) Epiteliales del duodeno
c) Ductales del páncreas
d) Centro Acinares del páncreas
53) Una mujer de 43 años dolor en hipocondrio derecho e icterica. En la ecografía se
evidencia cálculos biliares. Estos cálculos lo más probable es que se encuentren
localizados en:
a) conducto colédoco
b) Conducto cístico
c) Vesícula biliar
d) Conducto pancreático secundario
ECU 1:
Estudiante de 21 años sufre de gastritis aguda ocasionada por comer en lugares poco
higenicos. Suele consumir caramelos (“chupar”) mientras esta en base hasta la tarde.
También toma regular cantidad de leche (grasa, lactora, proteinas), pues le calma el
dolor y el ardor que sitnete por la gastritis (tiene dispepsia y cuando toma la leche se le
pasa).
Incluso cuando puede, se toma dos vasos de agua frita y le calma la molestia. Ha
decido ir al medico para tratarse, pues ya no soporta el dolor, el cual esta seguro que
los síntomas se producen por elevada producion de HCl en el estomago, y por ello le ha
recetado ranitidina
1.1) El consumir caramelos eleva los niveles en sangre de una hormona cuya función es la
estimulación de las células.
- Beta del páncreas por GIP el cual es una incretina y por consiguiente
estimula las células pancreáticas

1.2) Consumir caramelos indirectamente actica la via:


-POMP/ CART saciedad

1.3) Consumo de leche produce indirectamente


- CCK inhibición del vaciamiento gástrico mayor tonicidad del esfínter pilórico

1.4) Cuando el px toma dos vasos de agua, genera indirectamente un aumento en la


liberación de:
- vaso de agua distención → g astrina → secreción de HCl

1.5) El uso de ranitidina bloquea el receptor H2 de la histamina en las células parietales, la


histamina llega a estas células por:
- histamina es una hormona paracrina por → difusión
**endocrina es por via hematógena y si fuera neuroendorina es por un NTs

1.6) Aumenta la secreción salival:


- noradrenalina a través de los receptores Beta 2
1.7) En este paciente con gastritis aguda debida a una alta producción de ácido clorhídrico,
sería lógico esperar que el píloro tenga un tono muscular:
- primero la secretina
- luego CCK
**ambas reguladores del HCl, Gatritis aguda debido a una alta producción de HCL
piloro estará aumentado (por la CCK)
1.8) Debido al uso de ranitidina, los valores de somatoestina en sangre:
- ranitidina disminuye acción de gastrina se quiere secretar mas no
actúan los inhibidores como la somatoestina somatoestina disminuye
1.9) El uso de atropina en este paciente:
- Inhibirá la acción de las prostaglandinas
- Aumentará la producción de ácido clorhídrico
- Disminuirá la acción del receptor CCK-B
-Aumentará el pH del estómago
ECU 2:
Niño de sexo masculino de 2 años de edad, sufre de estreñimiento desde el
nacimiento (1 deposición cada 3-4 días). Madre menciona que le estimula la
defecación con un termómetro rectal, y continuo uso de enemas y laxantes. Desde
hace 6 meses comienza con vómitos postprandiales. Los síntomas aumentan en
frecuencia y magnitud y están en relación con los episodios de estreñimiento. No
refiere fiebre, tos, diarrea ni lesiones cutáneas. Al examen físico presenta regular
estado general, luce deshidratado. Abdomen distendido, blando, depresible e indoloro.
No se palpan masas abdominales. Se permeabiliza el canal anal con termómetro
rectal, encontrando cierta resistencia. Salida de material fecal mal oliente en regular
cantidad. Exámenes de laboratorio: hemograma normal. Signos inflamatorios de fase
aguda negativos. Alcalosis metabólica leve en sangre
venosa. Radiografía con enema baritado muestra recto y colon sigmoides dilatados
(megacolon). Biopsia profunda: ausencia de células ganglionares en la muestra
enviada. Se realiza cirugía correctiva.

2.1) Durante la fase esofágica de la deglución, para un bolo alimenticio determinado, a


medida que avanza el bolo la fuerza de la contracción se hace más:
- hiperpolarizado
- fuerte
- dependiente de Ach
- debil
2.2) Cuando este paciente ingiera sus alimentos, se espera que al momento de pasar el
bolo alimenticio por el esfínter esofágico superior, la presión intraesofágica
disminuya en:
- la porción proximal al bolo
- el tercio medio del esófago
-el cardias
- el lugar donde se contraiga la muscular propia
2.3) Al examinar la orofaringe del paciente, uno puede hallar fácilmente la amígdala
palatina, pues esta se encuentra inmediatamente detrás del músculo:
-Palatogloso
- Palatofaringeo
- Hiogloso
- Elevador del velo del paladar

2.4) Con respecto a la defecación en este caso, marque la respuesta correcta:


- En posición de cuclillas, el músculo puborectal genera un ángulo más agudo
en el recto
- El sigmoides y el recto están inervados por el nervio vago
- La sensación de defecar sólo se da cuando el recto es ocupado por
heces y alcanzado el 80% de su capacidad
-El esfínter anal comprometido tiene inervación autónoma

2.5) En este paciente [hirschsprung] se considera que está abolido el reflejo:


- Coloileal
-Rectoesfinteriano
- Gastrocólico
- Relajación receptiva

2.6) No se espera que sea causa del vómito:


-Ayuno prolongado
- Estimulación faríngea y del glosofaríngeo
- Irritación de la mucosa gástrica
- Dolor intenso

ECU 3:
Paciente de 54 años con antecedentes de alcoholismo, gastritis crónica,
tabaquismo pesado, obesidad, cálculos biliares y cirrosis, es llevado a la
emergencia por dolor abdominal en epigastrio irradiado a la espalda y trastorno
del sensorio.
Al examen físico: presión arterial 85/50 mmHg, frecuencia cardíaca 100
latidos/min, frecuencia respiratoria 18 x minuto, temperatura axilar 36°C.
Conjuntivas pálidas, escleras ictéricas nevus arácnidos en tronco, distensión
abdominal marcada, cabeza de medusa, matidez desplazable en ambos flancos e
hipogastrio, dolor a la palpación de abdomen.
Tiempo de protrombina: 24 seg (testigo: 13 seg); TPT: 38 seg, glicemia: 165 mg/dL,
uremia: 20 mg/dL, ASAT: 76 UI/L, ALAT: 22 UI/L, albumina: 2,5 g/dL, bilirrubina total:
2,6 mg/dL, bilirrubina directa: 1,4 mg/dL, amilasa sérica 4000 U/L.
3.1) En esta paciente, al aumento de la amilasa sérica, se debe directamente a una lesión
de:
a) páncreas
b) vesícula y árbol biliar
c) estómago
d) hígado
3.2) Considerando que el paciente sufre de gastritis, se puede decir que la secreción de
ácido por la mucosa gástrica
a) involucra transporte activo de hidrogeniones
b) es realizada principalmente por células principales
c) es inhibida por antihistaminas tomadas por pacientes con rinitis alérgica
d) involucra la liberación de HCl de los gránulos zimógenos
3.3) El paciente tiene hemorragia digestiva alta por várices sangrantes como complicación.
Llegando a estar en shock hipovolémico por hemorragia masiva, se encontrara
necrosis hepática en:
a) zona 1
b) no se afectan los lobulillos hepáticos en hemorragia
c) zona 3
d) zona 2
3.4) El misoprostol, análogo de las prostaglandinas está mejor indicado en:
a) cicatrización de úlcera péptica duodenal
b) erradica el helicobacter pylori
c) tratar el sind de Zollinger ellison
d) prevenir daño por AINES
3.5) De las siguientes sustancias secretadas por los órganos de este paciente, la más
alcalina es la secreción:
- Esofágica
- Salival
- Yeyunal
-Pancreática
3.6) En este paciente, se puede asumir que la pancreatitis ha sido ocasionada por una
disminución en el efecto de:
- Lipasa
- Enterocinasa
- Amilasa
-Inhibidor de la tripsina
3.7) ¿Cuál de las siguientes sustancias es segregada por el páncreas?
-Amilasa
- Pepsina
- Quimiotripsina
- Tripsina
3.8) Cada vez que este paciente toma alcohol, la acidificación de la luz del duodeno:
-Disminuye el vaciamiento gástrico
- Aumenta la contracción del esfínter de Oddi
- Aumenta la secreción del ácido gástrico
- Disminuye la secreción pancreática del bicarbonato
SISTEMA
DIGESTIVO
(ME 154)
EXAMEN
FINAL
Ciclo 2018-01

1. Un niño de 2 años es llevado a la consulta por diarrea persistente y edema de las extremidades, además
falta de crecimiento y desarrollo en relación a su edad. Los análisis de sangre revelan que tiene
concentración plasmática baja de proteínas (hipoproteinemia). Durante la endoscopía duodenal, se
coloca colecistokinina (CCK) endovenosa y se recoge muestras del líquido duodenal; el resultado del
líquido confirma incapacidad para hidrolizar proteínas a un pH neutro, esta situación mejora al añadir
una pequeña cantidad de tripsina. El paciente probablemente esté sufriendo la falta congénita de
………….
(Unidad 4, sesión 26, logro 2: Explicar la Digestión y absorción de las proteínas y sus alteraciones)
a. Pepsinógeno
b. PEPT-1
c. Carboxipeptidasas
d. Enterocinasa

2. Experimentalmente se incrementa la velocidad de la secreción salival con una sustancia, el


análisis de la composición de esta saliva obtenida se espera encontrar…………..
(Unidad 3, sesión 17, logro 5 : Explica la Influencia de la velocidad del flujo salival en la composición de
la saliva)
a. Elevación de concentración de bicarbonato, sodio y potasio
b. Elevación de concentración de cloro, sodio y potasio
c. Disminución de concentración de potasio
d. Disminución de concentración de potasio y bicarbonato

3. Paciente varón de 46 años soltero, consulta por odinofagia y bajo de peso, tiene antecedente de
tuberculosis desde hace 3 meses y es fumador crónico (10 cigarrillos por día); al evaluar la cavidad
oral se identifica lesión blanquecina en el dorso de la lengua y paladar blando, las lesiones se
desprenden con el baja lengua dejando una base eritematosa. Esta lesión corresponde
probablemente a ……………………….…..
( Unidad 3, sesión18, logro 1-2 : Describe las enfermedades inflamatorias, infecciosas y proliferativas
de la cavidad oral)
a. Eritroplaquia
b. Candidiasis oral
c. Leucoplaquia vellosa
d. Fibroma en cavidad oral

4. Minero de 32 años de edad, que acude a


centro de salud por presentar de forma
progresiva desde hace 1 año dificultad para
ingerir alimentos sólidos y luego líquidos;
refiere regurgitaciones alimentarias y marcada
pérdida de peso (15 kilos). Radiografia
baritada de esófago como se muestra en la
figura. El presente caso se explica
por……………….
(Unidad 2, sesión 12, logro 4: Identificar y
describir la función de los esfínteres
esofágicos)
a. Contracción incompleta del esfínter esofágico inferior
b. Dificultad para el inicio de la deglución
c. Relajación incompleta del esfínter pilórico
d. Relajación incompleta del esfínter esofágico inferior
5. Paciente mujer de 35 años acude a consulta por sensación de sequedad y lesiones en cavidad
oral. Al examen se observa atrofia de la mucosa, fisuras y úlceras; nota además sequedad e
irritación de la córnea y aumento del tamaño de las glándulas parotídeas. Su diagnóstico más
probable es artritis reumatoide; el hallazgo más probable en una biopsia de glándula parótida
es……..….
(Unidad 3, sesión 18, logro 3: Describe las enfermedades más frecuentes de las glándulas salivales)
a. Hiperplasia de acinos glandulares serosos
b. Gran infiltración de linfocitos y células plasmáticas
c. Gran infiltrado de linfocitos y macrófagos
d. Presencia de acinos normales con hiperplasia de células ductales

6. Un paciente con anemia acude con su médico quejándose de episodios frecuentes de


gastroenteritis. Un análisis de sangre revela anticuerpos circulantes dirigidos contra células
parietales gástricas. Su anemia es atribuible a la hiposecreción de
………………………
(Unidad 3, sesión 20, logro 5: Gastritis crónica. Tipos de gastritis)
a. Factor intrínseco
b. Proteina R (haptocorrina)
c. Pepsinógeno
d. Ácido clorhídrico

7. Dos estudiantes deciden tomar un receso para comer una hamburguesa a la hora del almuerzo.
Antes de llegar a la cafetería, impulsos nerviosos provenientes del complejo vagal dorsal iniciarán la
secreción de ácido gástrico por la liberación dedesde el sistema nervioso entérico.
(Unidad 3, sesión 20, logro 2: Regulación de la secreción gástrica: estimulación, fases de la secreción)
a. Serotonina
b. Óxido nítrico
c. GRP (péptido liberador de gastrina)
d. Péptido intestinal vaso activo

8. Un niño de cuatro años de edad es llevado a la consulta por cuadros diarreicos frecuentes
caracterizados por heces pálidas, voluminosas y fétidas, presenta bajo peso y talla. Se mide la
concentración de cloruro en el sudor y se encuentra que sus valores son muy elevados. La alteración
más importante a nivel de células ductales del páncreas tiene relación directa con la conductancia
de…………
(Unidad 3, sesión 23, logro 5 Explica la Secreción pancreática: formación del jugo pancreático,
influencia de la velocidad de flujo y regulación)
a. Potasio
b. Bicarbonato
c. Sodio
d. Cloro

9. Una mujer de 50 años de edad que sufrió durante varios años resequedad de los ojos debida a
producción inadecuada de lágrimas es enviada con un gastroenterólogo para evaluación de pirosis
crónica. El examen endoscópico revela erosiones y tejido cicatrizal en la parte distal del esófago
justo por arriba del esfínter esofágico inferior. Las lesiones pueden atribuirse a la disminución de
uno de los siguientes componentes salivales:
(Unidad 3, sesión 17, logro 4: Explicar la Formación de la saliva y cuáles son sus componentes)
a. Bicarbonato
b. Lactoferrina
c. Ig A
d. Amilasa
10. Se evalúa los valores séricos de las siguientes sustancias a un paciente con enfermedad hepática
terminal; en este paciente se espera encontrar la combinación con la letra …………
(Unidad 3, sesión 22, logro 5: Describe las Pruebas de función hepática, la Insuficiencia hepática,
encefalopatía hepática e hipertensión portal)

Glucosa Amoniaco Albúmina


a. Aumenta Disminuida Disminuida
da
b. Disminui Aumentada Aumentada
da
c. Aumenta Aumentada Aumentada
da
d. Disminui Aumentada Disminuida
da

11. Una mujer de 35 años de edad HIV positiva, se presenta al médico con dolor abdominal en
cuadrante superior derecho e ictericia. La paciente refiere haber tenido múltiples episodios de
ictericia durante los últimos 10 años. Los exámenes para determinar hepatitis viral, dieron positivos
para Hepatitis B, siendo catalogado el caso como hepatitis crónica con alteración funcional. En un
examen de sangre ¿cuál de los siguientes parámetros está disminuido?
(unidad 3, sesión 22, logro 5: Pruebas de función hepática, Insuficiencia hepática, encefalopatía
hepática e hipertensión portal)
a. Fosfatasa alcalina
b. Albumina
c. Bilirrubina
d. Tiempo de protrombina

12. En el reflejo peristáltico del intestino delgado, uno de los siguientes eventos sucede en
la porción oral del bolo alimenticio…………...
(Unidad 2, sesión 13, logro 4: Explicar la Motilidad del intestino delgado: Contracciones segmentarias y
peristálticas)
a. Disminución de 5 hidroxitriptamina desde las neuronas IPAN
b. Contracción del músculo longitudinal
c. Acción del péptido intestinal vasoactivo (VIP) en el músculo circular
d. Acción de acetilcolina en el músculo circular

13. Experimentalmente se coloca una dosis alta de secretina en la luz intestinal duodenal; como
consecuencia de esto, en el jugo pancreático de la misma luz intestinal se observa la disminución
de la concentración de …..………..
(Unidad 3, sesión 23, logro 5: Explica la Secreción pancreática: formación del jugo pancreático,
influencia de la velocidad de flujo y regulación)
a. Na+
b. Cl-
c. K+
d. HCO3-

14. Un varón de 58 años de edad con enfermedad de Crohn severo fue sometido a una resección
ileal. Después de la cirugía este paciente padecerá de esteatorrea, esto se explica porque
…..………..
(unidad 4, sesión 26, logro 4: Explica las alteraciones en la Absorción de lípidos)
a. El pool de ácidos biliares se incrementa
b. Los quilomicrones no pueden formarse en el lumen intestinal
c. La micelas no pueden formarse
d. El páncreas no secreta lipasa

15. En un experimento se inserta un balón en el estómago de un voluntario, se infla poco a poco


mientras que se vigilan las presiones intraluminales. Aunque el volumen del balón aumenta
considerablemente, las presiones permanecen constantes. Esta relación volumen-presión se explica
por la liberación local de …………..
(Unidad 2, sesión 13, logro 1 Explica la Motilidad gástrica: relajación receptiva)
a. Acetil colina y gastrina
b. Colecistoquinina y óxido nítrico
c. Óxido nítrico y péptido inhibidor vasoactivo
d. Norepinefrina y óxido nítrico
16. La toxina del Vibrio cholerae causa diarrea debido a…….
(Unidad 4, sesión 27, logro 6: Explica el transporte hidroelectrolítico intestinal, toxina colérica)
a. La fosforilación del canal CFTR de los enterocitos de las vellosidades intestinales
b. El Incremento de la secreción de cloro por las células de la cripta intestinal
c. La inhibición de la producción de AMPc por las células epitelailes
d. El incremento de la absorción de agua y sodio a través de las uniones estrechas

17. ¿Cuál de las siguientes alternativas es una característica de la secreción exocrina del páncreas?
(Unidad 3, sesión 23, logro 5: Secreción pancreática: formación del jugo pancreático, influencia de la
velocidad de flujo y regulación)
a. Tiene una baja concentración de Cl- respecto al plasma
b. Es estimulada por la presencia de bicarbonato en el duodeno
c. La secreción enzimática es estimulada principalmente por la gastrina
d. Es hipotónica respecto al plasma

18. Una madre lleva a su hijo de dos años de edad a la sala de urgencias, estresada porque el niño
deglutió una moneda de 10 céntimos mientras la familia cenaba en un restaurante. El médico
observa mediante fluoroscopía que la moneda se halla en el estómago y asegura a la madre que la
moneda se eliminará con las heces. El médico recomienda utilizar la respuesta fisiológica que
permitirá la evacuación de la moneda del estómago al intestino ………….…..
(Unidad 2, sesión 13, logro 2: Explica la Motilidad gástrica: mezclado y vaciamiento)
a. Es por la relajación receptiva
b. Son los movimientos de mezcla y trituración
c. Es provocada por el ayuno
d. Es por la relajación del esfínter esofágico superior

19. Las estructuras en el hígado que permite que los productos metabólicos unidos a proteínas tengan
acceso a las membranas basolaterales de los hepatocitos, son…..
(Unidad 3, sesión 21, logro 4-5: Explica la Organización micro estructural del hígado)
a. Los Canalículos
b. Las fenestras sinusoidales
c. Las uniones intercelulares herméticas
d. Las células de Ito

20. La composición de la bilis es modificada conforme fluye por los conductillos biliares. Durante
este tránsito se espera que aumente la concentración de…….
(Unidad 3, sesión 22, logro 2: Describe la Secreción biliar, visión general del sistema biliar
extrahepático y composición de la bilis)
a. Ig A
b. Glucosa
c. Monómeros de ácido biliar
d. Vitamina A

21. Se mide experimentalmente el contenido gástrico de dos personas. La persona “A” tiene alto
contenido de grasa y la persona “B” tiene un contenido hipertónico ¿Cuál de las siguientes es
correcto respecto al vaciamiento gástrico? (Unidad 2, sesión 13, logro 2: Describe la Motilidad
y vaciamiento gástrico)
a. Hay ralentización del vaciado gástrico solo en “A”
b. El vaciamiento gástrico es más rápido en ambos
c. En ambos casos hay incremento de la motilina
d. Hay ralentización del vaciado gástrico en ambos casos

22. El examen endoscópico de un paciente con hipertensión portal grave revela venas tortuosas que
sobresalen hacia la luz del esófago. El paciente recibe tratamiento quirúrgico mediante la colocación
de una derivación que conecta la vena porta a la vena cava. Después de la operación el riesgo de
encefalopatía ………………….. y el riesgo de sangrado de várices ……………..
(Unidad 3, sesión 22, logro 5: Describe la Insuficiencia hepática, encefalopatía hepática e hipertensión
portal)
a. Aumentará/disminuirá
b. Disminuirá/disminuirá
c. Aumentará/aumentará
d. Disminuirá/aumentará
23. Un paciente varón de 18 años de edad acude al médico para sus exámenes de rutina. Sus
resultados de laboratorio muestran un valor de bilirrubina sérica de 4 mg/dl y una bilirrubina
directa de 0,3 mg/dl. Las pruebas de función hepática son normales. La alteración que explica
mejor este caso es por la deficiencia de ………………..
(Unidad 3, sesión 22, logro 3: Explica la Producción y excreción de bilirrubina. Tipos de bilirrubina e
ictericia)
a. Transaminasas
b. Glucuronil transferasa
c. Hemo oxigenasa
d. La 7 alfa hidroxilasa

24. Un hombre de 57 años de edad es llevado a urgencias con hematemesis masiva rojo brillante, a su
llegada se halla inconciente con PA: 80/40 mm Hg y FC: 124 lat/min. Luce ictérico con presencia de
“arañas vasculares en el tórax anterior y extremidades”, abdomen distendido con signo de oleada
positiva. Se encuentra esplenomegalia y pérdida de la masa muscular en extremidades. La
anastomosis vascular responsable del sangrado en este paciente es ………….…..
(Unidad 3, sesión 21, logro 2: Describe las anastomosis porto sistémicas)
a. Arteria gástrica izquierda y vena ácigos
b. Vena gástrica izquierda y vena ácigos
c. Vena paraumbilical y vena epigástrica inferior
d. Vena gástrica izquierda y vena esofágica superior

25. Un estudiante de medicina está comiendo un plato de comida a base de champiñones, espárrago
y salsa de soya. El sabor umami contenido en todos estos alimentos actúa a nivel de los botones
gustativos estimulando ………………..
(Unidad 2, sesión 10, logro 5: Describe los tipos y mecanismos moleculares para la detección de los
sabores)
a. El ingreso de sodio
b. Un receptor acoplado a proteína G
c. Su receptor específico T1R3
d. El ingreso de hidrógeno

26. Un hombre de 22 años de edad se presenta al médico con una historia de 1 año de evolución
caracterizado por dolor recurrente en fosa iliaca derecha y diarrea. Manifiesta además pérdida
de peso de 8 kg durante este periodo. La colonoscopía revela múltiples lesiones en el ileon
terminal y colon. La biopsia de estas lesiones revela engrosamiento, inflamación y ulceración
de la mucosa. El diagnóstico más probable en este caso es…….
(Unidad 4, sesión 28, logro 5: Describe la Enfermedad inflamatoria intestinal. Generalidades,
morfología y características)
a. Sprue celiaco
b. Enfermedad de Crohn
c. Sindrome de colon irritable
d. Colitis ulcerativa

27. Una de las funciones del músculo señalado es:


(Unidad 2, sesión 8, logro 3: Describir el Piso
de la boca: estructuras blandas que la
conforman)
a. Eleva el paladar blando
b. Recibe inervación del nervio maxilar
c. Deprime el hioides cuando la mandíbula está fija
d. Deprime la mandíbula cuando el hioides está fijo

28. Varón de 61 años que consulta por dolor retro esternal intenso desde hace 6 horas y después
de vómitos intensos y repetidos; al examen se observa disnea, cianosis, hipotensión y signos
clínicos de shock. La radiografía simple de tórax muestra neumomediastino. El líquido en el
espacio pleural aspirado tiene alta concentración de amilasa. ¿Cuál de las siguientes
alternativas puede explicar este cuadro clínico?
(Unidad 3, sesión 18, logro 6: Describe algunas Enfermedades del esófago)
a. Sindrome de Mallory Weiss
b. Rotura espontánea de esófago
c. Neumotórax por probable herida penetrante
d. Perforación de ulcera gástrica de cara posterior, con complicación torácica
29. La secreción del ácido en la célula parietal gástrica se lleva a cabo por una ATPasa especifica que
intercambia hidrogeniones (H+) del citosol por…..
(Unidad 3, sesión 20, logro 1: Explica la Secreción del HCl y sustancias que la alteran)
a. Cl-
b. HCO3-
c. Na +
d. K+

30. En condiciones normales el ingreso de 600 ml de líquido es el estómago provoca un aumento de


presión intragástrica de unos 12 cm de H2O. Después de una vagotomía (corte del nervio vago) es
de esperar que el ingreso del mismo volumen de líquido provoque lo siguiente:
…………………………………
(Unidad 2, sesión 13, logro 1: Describe la Motilidad gástrica: relajación receptiva)
a. Un aumento igual de la presión
b. Que no aumente la presión
c. Un aumento mayor de la presión
d. Una disminución de la presión

31. Una paciente de 30 años de edad es sometida a una cirugía en oído medio derecho por un problema
de otoesclerosis. Luego de la cirugía refiere alteración en la percepción de sabores. Al evaluar el caso
usted esperaría encontrar……….
(Unidad 2, sesión 10, logro 5: Describe la Irrigación e inervación de la lengua)
a. Alteración en la sensación del dolor y temperatura en el tercio posterior de la lengua
b. Alteración en la sensación del gusto en los dos tercios anteriores de la lengua
c. Alteración en la sensación del gusto en la punta de la lengua
d. Sensación del dolor, tacto y temperatura conservada en toda la lengua

32. ¿Cuál de las siguientes alterativas es correcta?


(Unidad 4, sesión 26 : Explica la digestión y absorción de nutrientes y sus alteraciones)
a. En el borde luminal, en cepillo, del intestino delgado, la absorción de sodio únicamente se
realiza asociada a la de glucosa.
b. El lugar principal para la absorción del hierro es el ileon
c. Las sales biliares desconjugadas son absorbidas preferentemente en el colon
d. El proceso de digestión y absorción de la vitamina B12 no se altera en insuficiencia
pancreática.

33. En un paciente de 45 años de edad con colestasis biliar, se encuentra una elevación de los niveles
sanguíneos de fosfatasa alcalina hasta 3 veces la cifra normal. ¿Cuál de las siguientes alternativas
estará también elevada como evidencia del daño de la vía biliar?
(Unidad 3, sesión 22, logro 5: Pruebas de función hepática, Insuficiencia hepática, encefalopatía
hepática e hipertensión portal)
a. Tiempo de protrombina y albúmina sérica
b. Transaminasas hepáticas (ALT y AST)
c. Glucoronil transferasa
d. Gamma glutamil transpeptidasa

34. Revisando la angiografía de un hombre de 70 años en estudio por aneurisma de aorta abdominal el
radiólogo informa de la presencia de una oclusión completa de la arteria mesentérica inferior. El
paciente se encuentra completamente asintomático. ¿Cuál de las siguientes arterias se anastomosa
a la sistema arterial de la mesentérica inferior?
(Unidad 4, sesión 25, logro 1: Identifica la Arteria mesentérica superior e inferior, ramas y anastomosis)
a. Ileal
b. Cólica media
c. Sigmoideas
d. Cólica izquierda

35. Lactante de 3 meses de vida es atendido por presentar diarrea, se administra una solución de
glucosa y electrólitos por vía oral. La proteína de membrana apical que explica la capacidad de esta
solución para proporcionar aporte de glucosa e hidratación es ………..
(Unidad 4, sesión 26, logro 1: Explica la Digestión y Absorción de los hidratos de carbono. Alteraciones)
a. GLUT-5
b. SGLT-1
c. CFTR
d. GLUT-2
36. Paciente ha sufrido herida de bala en el abdomen, se le ha tenido que extirpar el segmento medio y
distal del ileon. En este caso la síntesis hepática de sales biliares estará …..…..
(Unidad 3, sesión 22, logro 4: Explica la formación, función y Circulación entero hepática de lasa sales
biliares)
a. Disminuida por inhibición de la colesterol 7 alfa hidroxilasa
b. Incrementada por estímulo de la enzima colesterol 7 alfa hidroxilasa
c. Incrementada por inhibición de la colesterol 7 alfa hidroxilasa
d. Sin cambios en el ritmo de síntesis

37. Un varón de 75 años ingresa al consultorio por presentar ictericia marcada de piel y las escleras. El
estudio del paciente mostró que presentaba un tumor que obstruía la totalidad del conducto
hepático común. ¿Cuál de las siguientes estructuras se encontrará dilatada en este paciente?
(Unidad 3, sesión 21, logro 6: Describir el árbol biliar intrahepático)
a. Conducto de Wirsung
b. Conductos de Hering
c. Conducto colédoco
d. Conducto cístico

38. Correlaciones las dos columnas y marque la fórmula correcta:


(Unidad 4, sesión 28, logro 1: Diarrea: definición, mecanismos: osmótica, secretoria y exudativa)
1. Enfermedad Hirschsprung( ) heces con moco y sangre
2. Diarrea osmótica( ) intolerancia a lactosa
3. Diarrea secretoria( ) aganglionosis congénita
4. Diarrea exudativa( ) canales de Cl- en las células de la cripta

a.- 4231b.- 1234c.- 2143d.- 4213

39. Respecto a la siguiente imagen que representa una estructura de la mucosa gástrica, la estructura con
número ………..
produce ……………………..
(Unidad 3, sesión 19, logro 4: La glándula fúndica. Funciones y tipos de células con sus características)
a. 3 / pepsina
b. 1 / Pepsinógeno
c. 4 / HCl y factor extrínseco
d. 2 / pepsinógeno

2
40. En un paciente con insuficiencia renal crónica, el déficit en la absorción de calcio a nivel del
enterocito se debe a lo siguiente:
(Unidad 4, sesión 26, logro 6: Explica la Absorción de calcio y hierro)
a. No se convierte la 25 hidroxicolecalciferol a 1,25 dihidroxicolecalciferol
b. No se convierte la 1,25 dihidroxicolecalciferol a 25 hidroxicolecalciferol
c. Se incrementa la producción de Calbindina
d. Existe un descenso de la alfa 25 hidroxilasa renal
CLAVES EXAMEN PARCIAL DE SISTEMA
DIGESTIVO 2019 - 00

1. Varón de 30 años es traído a emergencia por agresión abdominal con arma de fuego
(pistola) y es sometido a laparotomía exploratoria, observándose isquemia del colon
ascendente y parte del colon trasverso ¿la lesión de cuál de las siguientes arterias
explicaría esta isquemia?
(unidad 1, sesión 2, logro 6: (Describe la irrigación visceral: arterias de tronco
celiaco, arteria mesentérica superior e inferior, topografía de superficie, órganos
por cuadrante)

a. Celiaca
b. Colónica derecha
c. Mesentérica inferior
d. Mesentérica superior

2. Respecto a las sustancias gastrointestinales que regulan la secreción pancreática;


marque la afirmación correcta:
(unidad 1, sesión 3, logros 2 y 3: describir las hormonas gastrointestinales: estímulos y
funciones)

a. La Secretina, es la hormona más importante para la secreción de bicarbonato por


las células acinares del páncreas
b. La acetilcolina es capaz de estimular la secreción enzimática y de bicarbonato del páncreas
c. La gastrina, es la hormona más importante para la secreción de enzimas pancreáticas
d. La colecistoquinina (CCK) estimula al páncreas solo para secreción enzimática

3. Ante una lesión del X par craneal, ¿cuál de los siguientes músculos mantiene conservada su
función?:
(unidad 2, sesión 08, logro 4: Paladar blando: componentes musculares)

a. Elevador del velo del paladar


b. Tensor del velo del paladar
c. Palatofaríngeo
d. Glosofaríngeo

4. Experimentalmente se utiliza atropina (anticolinérgico) para inhibir la secreción de


gastrina, sin embargo la secreción de esta hormona se sigue dando ante estímulos
vagales. Esta situación se explica porque la atropina:
(unidad 1, sesión 3, logro 3 : describir las hormonas gastrointestinales: estímulo y funciones
de la gastrina y colecistoquinina)

a. Bloquea parcialmente la bomba de protones en la célula G


b. Inhibe la acción de acetilcolina e histamina en la célula G
c. Solo inhibe la acción del péptido GRP en la célula G
d. No bloquea la acción del péptido GRP

5. Un varón de 50 años es sometido a extirpación del duodeno y parte proximal


del yeyuno. Esta situación ocasionaría la pérdida de las células ……….. ,
productoras deque estimula la secreción de
bicarbonato por el páncreas.
(unidad 1, sesión 3, logro 3: describir las hormonas gastrointestianles: estímulos y
funciones de la secretina y péptido insulinotrópico dependiente de glucosa)

a. “S” / secretina
b. Parietales / secretina
c. “I” / colecistoquinina
d. “S” / colecistoquinina
6. Recién nacido que presenta tumoración abdominal a nivel del cordón umbilical
(fotografía). ¿cuál de las siguientes afirmaciones es correcta respecto a este
defecto en el desarrollo embriológico del intestino?: (unidad 1, sesión 5, logro 2:
identificar las anomalías del desarrollo del intestino medio)

a. Corresponde a una Gastrosquisis


b. Las vísceras se hallan cubiertas por piel
c. No está asociado a otras malformaciones
d. Se asocia a
malformaciones
cardiacas y del tubo
neural

7. Varón de 35 años acude a la emergencia por


trauma abdominal y se decida realizar una
laparoscopía exploratoria. El cirujano
observa la disposición de los órganos
abdominales como se representa en el
siguiente esquema. Esta disposición de
órganos se explica por la
rotación(SMA=arteria mesentérica superior)
(unidad 1, sesión 5, logro 3: identificar las anomalías
del desarrollo del intestino medio: defectos de
rotación, estenosis y atresias)

a. anti horaria del intestino medio, en sólo 90°


b. incompleta del intestino medio (270°)
c. horaria del intestino medio
d. horaria del estómago

8. Se evalúa la expresión de la proteína Agrp en una persona con alteración del


apetito; lo correcto respecto a esta proteína es…..
(unidad 1, sesión 3, logro 4: Explica los mecanismos de control del apetito y saciedad )

a. Esta proteína es un potente anorexigénico


b. La mutación del gen que la codifica produce adelgazamiento
c. La sobre producción de la proteína lleva a obesidad por agonismo de receptores MC3 y MC4
d. La sobre producción de la proteína disminuye el apetito por antagonismo de receptores MC4

9. Juana cae de la bicicleta y se fractura la región anterior del hueso maxilar superior con
compromiso de la fosa incisiva. Al examen físico de la región esperaría encontrar
alteración en la sensibilidad de la encía …………………
(unidad 2, sesión 8, logro5: paladar: paladar duro y blando: irrigación e inervación)

a. bucal posterior
b. Lingual anterior
c. palatina anterior
d. palatina posterior
10. Recién nacido es atendido por el neonatólogo y luego entregado a su madre para
dar de lactar; la madre al dar de lactar observa coloración azulada de labios,
acompañado de tos persistente, dificultad respiratoria y distención abdominal. Se le
intenta colocar una sonda nasogástrica pero esta retorna a la cavidad oral en todos
los intentos. ¿Cuál de las siguientes anomalías del desarrollo es el más probable en
este caso? (unidad 1, sesión 4, logro 3: identificar las anomalías en el desarrollo del
esófago: atresia y/o fístula traqueo esofágica)

a. Estenosis esofágica proximal con Fístula traqueo esofágica distal


b. Atresia esofágica proximal con fístula traqueoesofágica distal
c. Atresia esofágica distal con fístula traqueoesofágica proximal
d. Fístula traqueoesofágica proximal y distal

11. ¿Cuál de los siguientes mecanismos ocurre durante la defecación?


(unidad 2, sesión 13, logro 6: motilidad del intestino grueso: contracciones
segmentarias, movimientos en masa, defecación y reflejo gastrocólico)

a. Contracción refleja del esfínter anal interno


b. En la posición de “cuclillas” el músculo puborectal se halla relajado
c. Relajación del esfínter anal externo por efectos del VIP y óxido nítrico
d. La materia fecal en el recto estimula la contracción del sigmoides por los nervios pudendos

12. La estructura número 4 (gráfico) corresponde a


……….… y está ………..
(unidad 2, sesión 9, logro 2:
Partes de un diente. Capas
del diente: Esmalte:
características y células que
lo producen)

a. el cemento / mineralizado en 90%


b. la dentina / formada por ameloblastos
c. el esmalte / formado
por células derivadas
del mesénquima
d. la dentina / formado por
células derivadas de la
cresta neural

13. Un paciente luego de un accidente sufre lesión del piso de la boca, se constata
daño del nervio “cuerda del tímpano”, en este caso se esperaría encontrar
disminución de lade la lengua
(unidad 2, sesión 10, logro 3: Irrigación e inervación de la lengua)

a. Motilidad en los dos tercios anteriores


b. Sensación del gusto en el tercio posterior
c. Sensación del gusto en los dos tercios anteriores
d. Sensibilidad al tacto en los dos tercios anteriores

14. ¿Cuál de las siguientes afirmaciones es la correcta sobre la gastrina?


(unidad 1, sesión 3, logro 1: reconocer las características de las sustancias
reguladoras gastrointestinales: hormonas, sustancias paracrinas y neurocrinas)

a. Produce atrofia de la mucosa gástrica


b. Es producida por la célula G del cuerpo gástrico
c. Es estimulada por la distensión gástrica y el Ph bajo
d. Actúa en la célula diana mediante su receptor CCk tipo B
15. Al recibir un paciente con signos de hipovolemia y antecedente de trauma en
abdomen por accidente de tránsito, usted identifica radiológicamente: lesión de
primera vértebra lumbar y signos de lesión en páncreas; durante la cirugía se observó
pobre irrigación de asas intestinales. El vaso afectado es la arteria ……..
(unidad 1, sesión 1, logro 6: reconocer las estructuras a nivel de L1, nivel de los principales
vasos sanguíneos)

a. esplénica
b. hepática común
c. mesentérica inferior
d. mesentérica superior

16. Un paciente sufre de daño a nivel del cuello con lesión muscular en la región de la
faringe. En el examen físico se determina dificultad para la elevación de la faringe y
para el cierre del itsmo de las fauces. En este caso, probablemente esté afectado el
músculo:
(unidad 2, sesión 11, logro 2: Músculos de la faringe: identificación, constrictores y
longitudinales)

a. palatogloso
b. estilofarinfeo
c. palatofaringeo
d. constrictor inferior

17. Varón de 50 años a quien le realizan la curación de la segunda molar de la arcada


superior derecha. En un momento determinado, el paciente acusa de intenso
dolor de la pieza dentaria en tratamiento. La vía aferente del dolor viaja a través
del nervio …………
(unidad 2, sesión 9, logro 6: Inervación de los dientes)

a. trigémino V2
b. trigémino V3
c. naso palatino
d. palatino menor

18. La distención gástrica por los alimentos produce incremento de secreción de HCl mediante la
producción de
………….. que estimula a las células ……………. vía proteína ………..
(Unidad 1, sesión 3, logro 2: Describe las hormonas gastrointestinales: Estímulo y funciones de
la gastrina y colecistoquinina)

a. gastrina / parietal / Gq
b. gastrina / principal / Gs
c. acetilcolina / parietal /Gi
d. acetilcolina / principal / Gi

19. Un niño de tres años llega a emergencia con disfagia (dificultad para tragar), dolor
retro esternal, salivación y llanto. Se sospecha de ingesta de cuerpo extraño
(moneda) en el esófago; al ser evaluado se constata en una radiografía presencia de
cuerpo extraño a nivel de C6 (6° vértebra cervical). El cuerpo extraño estará
suspendido a nivel del estrechamiento producido por………..
(unidad 2, sesión 11, logro4: Esófago, características anatómicas, relación con órganos
vecinos y estrecheces)

a. el cayado aórtico
b. el hiato esofágico
c. el músculo cricofaríngeo
d. el bronquio principal izquierdo

20. La triada portal (arteria hepática, vena portal y conducto biliar común) está contenida en el
ligamento
…….……… y derivan embriológicamente del ……
(Unidad 1, sesión 1, logro 4: Identifica el peritoneo, mesenterio, omento y ligamentos,
retroperitoneo.)

a. hepato duodenal / mesenterio ventral


b. gastro esplénico / mesenterio dorsal
c. hepato gástrico / omento menor
d. falciforme / omento menor
21. En relación al movimiento de
peristaltismo del tubo
digestivo: en la flecha negra
del gráfico se produce la
liberación de ……………… a
nivel del músculo ………..
(unidad 2, sesión 7, logro 6:
Control hormonal y tipos de
movimiento)

a. noradrenalina, sustancia P y neuropéptido “ Y” / circular


b. acetilcolina y sustancia P / longitudinal
c. óxido nítrico y PIV / longitudinal
d. óxido nítrico y PIV / circular

22. Un paciente refiere no percibir algunos sabores, al examen físico se constata


alteración en la percepción de sabores y del dolor en el tercio posterior de la lengua
¿Cuál de los siguientes nervios estará alterada en su función?
(unidad 2, sesión 10, logro 5: Sabores, tipos y mecanismos moleculares para su detección)

a. Lingual (rama del V par)


b. Cuerda del tímpano (VII par)
c. Glosofaríngeo (IX par)
d. Hipogloso (XII par)

23. El gráfico detalla la


estructura de la pared del
tubo digestivo intestinal
¿Cuál de las siguientes
asociaciones es correcta?
(unidad 2, sesión 7, logro 1:
La pared y músculo liso
gastrointestinal )

a. “1” – peristaltismo
b. “2” – secreción enzimática
c. “3” – deriva del mesodermo
d. “4” – doble hoja de tejido graso

24. En el caso de un paciente con gastrinoma (tumor productor de gastrina), la presencia


de úlceras duodenales y erosión de la mucosa gástrica, se debe principalmente a…….
(unidad 1, sesión 3, logro 2: describir las hormonas gastrointestinales: estímulo y funciones de
la gastrina y colecistoquinina)

a. la acción directa de la gastrina sobre la célula principal


b. la sobre expresión de los receptores “G” en la célula parietal
c. el exceso de HCl por estímulo de receptores CCK-B en la célula parietal
d. el exceso de HCl por estímulo directo de receptores de acetilcolina en la célula parietal

25. El reflejo entero gástrico se caracterizan por:


(unidad 2, sesión 13, logro 6: Motilidad del intestino grueso: contracciones
segmentarias, movimientos en masa defecación y reflejo gastrocólico)

a. favorecer la motilidad gástrica gracias a la CCk


b. inhibir la motilidad gástrica y estimular la secreción ácida
c. movilizar grandes volúmenes desde el estómago al duodeno
d. originarse debido a la distensión duodenal y presencia del quimo ácido
26. Mauricio tiene dificultad para deprimir el paladar y elevar la parte posterior de la
lengua. En este caso estará afectado un músculo ………………., específicamente el
músculo …………….
(Unidad 2, sesión 10, logro 2: Músculos de la lengua: clasificación, identificación y sus
funciones)

a. intrínseco – longitudinal inferior


b. extrínseco – palatogloso
c. extrínseco – transverso
d. extrínseco – estilogloso

27. Una de las funciones del músculo señalado es:


(Unidad 2, sesión 8, logro 3: Describir el Piso de la boca:
estructuras blandas que la conforman)

a. deprimir la lengua
b. elevar el paladar blando
c. deprimir el hioides cuando la mandíbula está fija
d. deprimir la mandíbula cuando el hioides está fijo

28. Paciente varón de 30 años es evaluado por probable enfermedad de Chagas, cursa
con problemas de motilidad del colon; los estudios de biopsia determinan
ausencia de células ganglionares. Según el gráfico
¿cuál es la capa en la que se determina la ausencia de dichas células?
(unidad 1, sesión 2, logro 1: describir las generalidades de la estructura del tubo digestivo:
esófago, estómago intestino delgado y grueso)

a. Mucosa - 1
b. Muscular propia – 1
c. Muscular de la mucosa - 2
d. Muscular propia - 3

1 2

3
29. Paciente varón de 32 años, que acude a centro de salud por presentar de forma
progresiva desde hace 1 año dificultad para ingerir alimentos sólidos y luego líquidos;
refiere regurgitaciones alimentarias y marcada pérdida de peso (15 kilos). Radiografía
baritada (sustancia de contraste) de esófago se muestra en la figura. El presente caso
se explica por……………….
(Unidad 2, sesión 12, logro 4: Identificar y describir la función de los esfínteres esofágicos)

a. aumento de la peristalsis esofágica


b. relajación incompleta del esfínter pilórico
c. relajación incompleta del esfínter esofágico inferior
d. perdida de producción de PIV y
óxido nítrico en el esfínter
esofágico superior

30. En condiciones normales, el ingreso de 600 ml de líquido es el estómago provoca un


aumento de presión intragástrica de unos 12 cm de H 2O. Después de una vagotomía
(corte del nervio vago) es de esperar que el ingreso del mismo volumen de líquido
ocasionede la presión intragástrica.
(Unidad 2, sesión 13, logro 1: Describe la Motilidad gástrica: relajación receptiva)

a. la disminución
b. la no variación
c. un aumento mayor
d. un aumento similar o igual
SISTEMA DIGESTIVO (ME 154) CLAVES
EXAMEN FINAL
Ciclo 201900

1. Un niño de 2 años es llevado a la consulta por diarrea persistente, edema de las


extremidades y falta de crecimiento en relación a su edad. Los análisis de sangre revelan que
tiene concentración plasmática baja de proteínas (hipoproteinemia). Como parte del estudio
se coloca colecistokinina (CCK) endovenosa y se recoge muestras del líquido duodenal por
endoscopía; el resultado del líquido confirma incapacidad para hidrolizar proteínas a un pH
neutro, esta situación mejora al añadir una pequeña cantidad de tripsina. El paciente
probablemente esté sufriendo la falta congénita de ………….
(Unidad 4, sesión 26, logro 2: Explicar la Digestión y absorción de las proteínas y sus alteraciones)
a. PEPT-1
b. pepsinógeno
c. enterocinasa
d. carboxipeptidasas

2. Paciente mujer de 35 años acude a consulta por sensación de sequedad y lesiones en


cavidad oral. Al examen se observa atrofia de la mucosa, fisuras y úlceras; nota además
sequedad e irritación de la córnea y aumento del tamaño de las glándulas parotídas. Su
diagnóstico más probable es artritis reumatoide; el hallazgo más probable en una biopsia
de glándula parótida es……..….
(Unidad 3, sesión 18, logro 3: Describe las enfermedades más frecuentes de las glándulas
salivales)
a. Presencia de acinos normales con hiperplasia de células ductales
b. Gran infiltración de linfocitos y células plasmáticas
c. Hiperplasia de acinos glandulares serosos
d. Gran infiltrado de linfocitos y neutrófilos

3. Un hombre de 42 años de edad se presenta al médico con una historia de 1 año de


evolución, caracterizado por dolor abdominal bajo y diarreas con crisis sanguinolentas.
Manifiesta además pérdida de peso de 8 kg durante este periodo. La colonoscopía revela
lesión difusa en el colon con afectación del recto. La biopsia de estas lesiones revela
adelgazamiento de la pared, inflamación y ulceración de la mucosa y sub mucosa. El
diagnóstico más probable en este caso es:
(Unidad 4, sesión 28, logro 5: Describe la Enfermedad inflamatoria intestinal.
Generalidades, morfología y características)
a. sindrome de colon irritable
b. enfermedad de Crohn
c. colitis ulcerativa
d. sprue celiaco

4. Dos estudiantes deciden tomar un receso para comer una hamburguesa a la hora del
almuerzo. Antes de llegar a la cafetería, impulsos nerviosos provenientes del complejo
vagal dorsal iniciarán la secreción de ácido gástrico por la liberación dedesde el sistema
nervioso entérico.
(Unidad 3, sesión 20, logro 2: Regulación de la secreción gástrica: estimulación, fases de la
secreción)
a. Serotonina
b. Colecistoquinina
c. Péptido inhibidor vaso activo
d. GRP (péptido liberador de gastrina)

5. Un niño de cuatro años de edad es llevado a la consulta por cuadros diarreicos


frecuentes caracterizados por heces pálidas, voluminosas y fétidas; al examen físico
presenta bajo peso y talla para la edad. Se mide la concentración de cloruro en el
sudor y se encuentra que sus valores son muy elevados. La alteración más
importante a nivel de células ductales del páncreas tiene relación directa con la conductancia
de…………
(Unidad 3, sesión 23, logro 5 Explica la Secreción pancreática: formación del jugo
pancreático, influencia de la velocidad de flujo y regulación)
a. Bicarbonato
b. Potasio
c. Sodio
d. Cloro
6. Se evalúa los valores séricos de las siguientes sustancias a un paciente con enfermedad
hepática terminal; en este paciente se espera encontrar la combinación con la letra …………
(Unidad 3, sesión 22, logro 5: Describe las Pruebas de función hepática, la Insuficiencia
hepática, encefalopatía hepática e hipertensión portal)
Glucosa Amoniaco Albúmin
a
a. Aumenta Disminuida Disminui
da da
b. Disminui Aumentada Aumenta
da da
c. Aumenta Aumentada Aumenta
da da
d. Disminui Aumentada Disminui
da da

7. Una mujer de 35 años de edad HIV positiva, se presenta al médico con dolor abdominal
en cuadrante superior derecho e ictericia. La paciente refiere haber tenido múltiples
episodios de ictericia durante los últimos 10 años. Los exámenes para determinar
hepatitis viral, dieron positivos para Hepatitis B, siendo catalogado el caso como
hepatitis crónica con alteración funcional. En un examen de sangre ¿cuál de los
siguientes parámetros está disminuido?
(unidad 3, sesión 22, logro 5: Pruebas de función hepática, Insuficiencia hepática,
encefalopatía hepática e hipertensión portal)
a. Albumina
b. Bilirrubina
c. Fosfatasa alcalina
d. Tiempo de protrombina

8. En el reflejo peristáltico del intestino delgado ¿Cuál de los siguientes eventos sucede en la
porción caudal del bolo alimenticio?
(Unidad 2, sesión 13, logro 4: Explicar la Motilidad del intestino delgado: Contracciones
segmentarias y peristálticas)
a. Acción del péptido inhibidor vasoactivo (VIP) en el músculo circular
b. Acción del NO (óxido nítrico) en el músculo longitudinal
c. Contracción del músculo longitudinal interno
d. Acción de acetilcolina en el músculo circular

9. Un varón de 58 años de edad con enfermedad de Crohn severo fue sometido a una
resección ileal. Después de la cirugía este paciente padecerá de esteatorrea, esto se
explica porque …..………..
(unidad 4, sesión 26, logro 4: Explica las alteraciones en la Absorción de lípidos)
a. se inhibe la acción de la 7 alfa hidroxilasa
b. el pool de ácidos biliares se incrementa
c. hay mala absorción de ácidos biliares
d. el páncreas no secreta lipasa

10. En un experimento se inserta un balón en el estómago de un voluntario, se infla poco


a poco mientras que se vigilan las presiones intraluminales. Aunque el volumen del
balón aumenta considerablemente, las presiones permanecen constantes. Esta
relación volumen-presión se explica por la liberación local de …………..
(Unidad 2, sesión 13, logro 1 Explica la Motilidad gástrica: relajación receptiva)
a. acetil colina y gastrina
b. norepinefrina y óxido nítrico
c. colecistoquinina y óxido nítrico
d. óxido nítrico y péptido inhibidor vasoactivo

11. ¿Cuál de las siguientes alternativas es una característica de la secreción exocrina del páncreas?
(Unidad 3, sesión 23, logro 5: Secreción pancreática: formación del jugo pancreático,
influencia de la velocidad de flujo y regulación)
a. Es hipotónica respecto al plasma
b. Su mayor estímulo se da en la fase intestinal
c. Es estimulada por la presencia de bicarbonato en el duodeno
d. La secreción enzimática es estimulada principalmente por la secretina
12. Las estructuras en el hígado que permite que los productos metabólicos unidos a
proteínas tengan acceso a las membranas basolaterales de los hepatocitos, son…..
(Unidad 3, sesión 21, logro 4-5: Explica la Organización micro estructural del hígado)
a. los canalículos
b. las células de Ito
c. las fenestras sinusoidales
d. las uniones intercelulares herméticas

13. La composición de la bilis es modificada conforme fluye por los conductillos biliares.
Durante este tránsito se espera que aumente la concentración de…….
(Unidad 3, sesión 22, logro 2: Describe la Secreción biliar, visión general del sistema biliar
extrahepático y composición de la bilis)
a. Ig A
b. Glucosa
c. Protones
d. Vitamina A

14. Se mide experimentalmente el contenido gástrico de dos personas. La persona “A” tiene
alto contenido de grasa y la persona “B” tiene un contenido isotónico ¿Cuál de las
siguientes es correcto respecto al vaciamiento gástrico? (Unidad 2, sesión 13, logro 2:
Describe la Motilidad y vaciamiento gástrico)
a. Hay ralentización del vaciado gástrico solo en “A”
b. El vaciamiento gástrico es más rápido en ambos
c. Hay ralentización del vaciado gástrico solo en “B”
d. Hay ralentización del vaciado gástrico en ambos casos

15. El examen endoscópico de un paciente con hipertensión portal grave revela venas
tortuosas que sobresalen hacia la luz del esófago. El paciente recibe tratamiento
quirúrgico mediante la colocación de una derivación que conecta la vena porta a la vena
cava. Después de la operación el riesgo de encefalopatíay el riesgo
de sangrado de várices ……………..
(Unidad 3, sesión 22, logro 5: Describe la Insuficiencia hepática, encefalopatía hepática e
hipertensión portal)
a. disminuirá / disminuirá
b. disminuirá / aumentará
c. aumentará / disminuirá
d. aumentará / aumentará

16. Un paciente varón de 18 años de edad acude al médico para sus exámenes de rutina. Sus
resultados de laboratorio muestran un valor de bilirrubina sérica de 4 mg/dl y una
bilirrubina directa de 0,3 mg/dl. Las pruebas de función hepática son normales. La
alteración que explica mejor este caso es por la deficiencia de ………………..
(Unidad 3, sesión 22, logro 3: Explica la Producción y excreción de bilirrubina. Tipos de bilirrubina e
ictericia)
a. transaminasas
b. hemo oxigenasa
c. la 7 alfa hidroxilasa
d. glucuronil transferasa

17. Un hombre de 57 años de edad es llevado a urgencias con hematemesis masiva rojo
brillante, a su llegada se halla inconsciente con PA: 80/40 mm Hg y FC: 124 lat/min. Luce
ictérico con presencia de “arañas vasculares en el tórax anterior y extremidades”,
abdomen distendido con signo de oleada positiva. Se encuentra esplenomegalia y pérdida
de la masa muscular en extremidades. La anastomosis vascular responsable del sangrado
en este paciente es ………….…..
(Unidad 3, sesión 21, logro 2: Describe las anastomosis porto sistémicas)
a. vena gástrica izquierda y vena ácigos
b. arteria gástrica izquierda y vena ácigos
c. vena paraumbilical y vena epigástrica inferior
d. vena gástrica izquierda y vena esofágica superior
18. Un estudiante de medicina está comiendo un plato de comida a base de champiñones,
espárrago y salsa de soya. El estímulo del sabor umami contenido en todos estos
alimentos viaja a través del nervio………………..
(Unidad 2, sesión 10, logro 3: Describe la irrigación e inervación de la lengua)
a. Lingual
b. Hipogloso
c. Glosofaringeo
d. Cuerda del tímpano
19. Una paciente de 30 años de edad es sometida a una cirugía en oído medio derecho
por un problema de otoesclerosis. Luego de la cirugía refiere alteración sensitiva de
la lengua. Al evaluar el caso usted esperaría encontrar……….
(Unidad 2, sesión 10, logro 5: Describe la Irrigación e inervación de la lengua)
a. Alteración en la sensación del dolor y temperatura en el tercio posterior de la lengua
b. Alteración en la sensación del dolor en los dos tercios anteriores de la lengua
c. Alteración en la sensación del gusto en el tercio posterior de la lengua
d. Sensación del dolor, tacto y temperatura conservadas

20. En un paciente de 45 años de edad con colestasis biliar, se encuentra una elevación de los
niveles sanguíneos de fosfatasa alcalina hasta 3 veces la cifra normal. ¿Cuál de las
siguientes alternativas estará también elevada como evidencia del daño de la vía biliar?
(Unidad 3, sesión 22, logro 5: Pruebas de función hepática, Insuficiencia hepática,
encefalopatía hepática e hipertensión portal)
a. Tiempo de protrombina y albúmina sérica
b. Transaminasas hepáticas (ALT y AST)
c. Gamma glutamil transpeptidasa
d. Glucoronil transferasa

21. Experimentalmente se incrementa la velocidad de la secreción salival con una


sustancia, en el análisis de la composición de esta saliva obtenida se espera
encontrar…………..
(Unidad 3, sesión 17, logro 5 : Explica la Influencia de la velocidad del flujo salival en la
composición de la saliva)
a. disminución de la concentración de bicarbonato que supera la concentración plasmática
b. aumento de la concentración de cloro y sodio que supera la concentración plasmática
c. aumento de la concentración de bicarbonato que supera la concentración plasmática
d. disminución de concentración de potasio y bicarbonato

22. Lactante de 3 meses de vida es atendido por presentar diarrea, se administra una
solución de glucosa y electrólitos por vía oral. La proteína de membrana apical
que explica la capacidad de esta solución para proporcionar aporte de glucosa e
hidratación es ………..
(Unidad 4, sesión 26, logro 1: Explica la Digestión y Absorción de los hidratos de carbono.
Alteraciones)
a. CFTR
b. SGLT-1
c. GLUT-2
d. GLUT-5

23. Paciente ha sufrido herida de bala en el abdomen, se le ha tenido que extirpar el


segmento medio y distal del ileon. En este caso la síntesis hepática de sales biliares
estará …..…..
(Unidad 3, sesión 22, logro 4: Explica la formación, función y Circulación entero hepática de lasa
sales biliares)
a. Sin cambios en el ritmo de síntesis
b. Disminuida por inhibición de la enzima colesterol 7 alfa hidroxilasa
c. Incrementada por estímulo de la enzima colesterol 7 alfa hidroxilasa
d. Incrementada por inhibición de la enzima colesterol 7 alfa hidroxilasa

24. Un varón de 75 años ingresa al consultorio por presentar ictericia marcada de piel y las
escleras. El estudio del paciente mostró que presentaba un tumor que obstruía la
totalidad del conducto hepático común. ¿Cuál de los siguientes conductos se encontrará
dilatado en este paciente?
(Unidad 3, sesión 21, logro 6: Describir el árbol biliar intrahepático)
a. de Wirsung
b. de Hering
c. colédoco
d. cístico
25. Correlaciones las dos columnas y marque la fórmula correcta:
(Unidad 4, sesión 28, logro 1: Diarrea: definición, mecanismos: osmótica, secretoria y exudativa)

1. Enfermedad Hirschsprung( ) heces con moco y sangre


2. Diarrea osmótica( ) intolerancia a lactosa
3. Diarrea secretoria( ) aganglionosis congénita
4. Diarrea exudativa( ) canales de Cl- en las células de la cripta a.-

4231b.- 1234c.- 2143d.- 4213

26. La vena umbilical obliterada del hígado después del nacimiento se transforma en el ligamento:
(Unidad 3, sesión 21, logro 1: Hígado: relación con la pared abdominal, caras, lóbulos, ligamentos
, hilio hepático)
a. cruzado
b. redondo
c. coronario
d. falciforme

27. Llega a su guardia nocturna una madre que trae a su RN masculino de 2 semanas de vida
con mal estado general y sequedad de mucosas. Usted observa que lacta ávidamente, pero
a las 2 horas presenta vómito postprandial no bilioso en proyectil. Al realizar la historia
clínica, descubre que el lactante recibió profilaxis con macrólidos para tos ferina. Usted
sospecha principalmente en:
(Unidad 1, sesión 4, logro 4: Desarrollo y anomalías del intestino anterior)
a. estenosis pilórica hipertrófica congénita
b. fistula traqueo esofágica
c. estenosis duodenal
d. atresia duodenal

28. En la regulación del apetito y la saciedad, la estimulación experimental crónica del


núcleo ventro medial del hipotálamo producirá:
(Unidad 1, sesión 3, logro 4: explica los mecanismos de control del apetito y saciedad)
a. afagia
b. obesidad
c. hiperfagia
d. activación de neuronas relacionadas a NPY

29. Paciente mujer de 25 años acude por dolor en fosa ilíaca derecha que empeora al toser o
caminar, asociada a náuseas y vómitos por lo cual acude a emergencia. Dos días después
de realizarle una apendicectomía, la paciente desarrolla fiebre alta (39 °C), está hipotensa y
presenta dolor abdominal. La laparotomía exploratoria muestra un gran volumen de
sangre en la cavidad peritoneal por lesión de un vaso producida durante la
apendicectomía.
¿Cuál de las siguientes arterias debe ligarse para detener la hemorragia?
(Unidad 4, sesión 27, logro 4: Irrigación arterial del colon, recto y conducto anal)
a. cólica derecha y arteria rectal superior.
b. ileocólica y arteria cólica media.
c. mesentérica superior.
d. ileocólica.

30. La onda peristáltica secundaria del esófago se caracteriza por ser originada ………
(unidad 2, sesión 12, logro 3: Motilidad esofágica: fases y características)
a. por el plexo de meissner del esófago
b. por el plexo mientérico del esófago
c. por el reflejo de la deglución
d. durante la masticación

31. ¿Cuál de los siguientes es una causa de ictericia con bilirrubina conjugada aumentada?
(Unidad 3, sesión 22, logro 3: Producción y excreción de bilirrubina. Tipos de bilirrubina, ictericia)
a. Ictericia del recién nacido
b. Obstrucción del colédoco
c. Anemia hemolítica
d. Gran hematoma
32. En relación a la absorción de nutrientes, la absorción de dipéptidos y tripéptidos a nivel
de las células epiteliales del intestino delgado, se da principalmente debido a:
(Unidad 4, sesión 26, logro 2: Digestión y absorción de las proteínas. Alteraciones)
a. el incremento de los canales de Cl- en la membrana apical
b. la gradiente de bicarbonato en la membrana basal
c. la gradiente de iones H+ en la membrana apical
d. la gradiente de Na+ en la membrana apical

33. Paciente de 20 años es traído a la emergencia por presentar diarreas desde hace 2 días.
Familiar refiere que las deposiciones son líquidas y abundantes, al examen luce
deshidratado y se plantea que la diarrea es producida por una toxina que estimula la
transformación de ATP a AMPc con apertura de canales de Cl- y pérdida de agua. El tipo de
diarrea más probable es:
(Unidad 4, sesión 28, logro 1: Diarrea: definición , mecanismos: osmótica, secretoria y supurativa)
a. osmótica
b. exudativa
c. secretoria
d. por intolerancia a lactosa

34. Un niño fue operado por una obstrucción intestinal, observándose la presencia de
divertículo de Meckel. Según lo referido, marque lo correcto:
(Unidad 1, sesión 5, logro 2: identifica las anomalías del desarrollo del intestino medio: onfalocele
y gastrosquisis (diferencias), Divertículo de Meckel)
a. el 50% de la población lo presenta
b. se localiza en el íleon muy cerca al yeyuno
c. puede poseer tejido gástrico o pancreático
d. se produce por una mala rotación de los intestinos

35. Marque la
alternativa correcta
respecto a la
estructura marcada
en el gráfico:
(Unidad 3, sesión 22,
logro 2: Secreción biliar.
Visión general del
sistema biliar
extrahepático y
composición de la bilis)

a. Se halla a 2
centímetros
debajo de la
papila
duodenal
mayor
b. Llega el conducto
colédoco y
pancreático
principal
c. Llega el
conducto
hepático
común y
pancreático
principal
d. Llega el conducto pancreático accesorio

36. ¿Cuál de las siguientes moléculas se encontrará aumentada en el citoplasma de las


células parietales de un paciente con sindrome de Zollinguer Ellison?
(Unidad 3, sesión 20, logro 4: Enfermedad ulcerosa péptica: úlcera gástrica, duodenal.
síndrome de Zollinger – Ellison)
a. Péptido liberador de gastrina (GRP)
b. Proteína G estimulante (GS)
c. Inositol Trifosfato (IP3)
d. AMP cíclico (AMPc)

37. Los fármacos inhibidores de la bomba de protones, actúan bloqueando la ………..……..


(Unidad 3, sesión 20, logro 3: Regulación de la secreción gástrica: inhibición, Secreción de
pepsinógeno y factor intrínseco)
a. anhidrasa carbónica
b. ATPasa H+/K+ en la membrana luminal
c. ATPasa H+/K+ en la membrana basolateral
d. ATPasa Na+/K+ en la membrana basolateral
38. Un paciente fue diagnosticado de gastritis autoinmune, ¿cuál de las siguientes
alternativas es FALSA respecto a esta enfermedad?
(Unidad 3, sesión 20, logro 5: Gastritis crónica: helicobacter pylori, autoinmune. Tipos de gastritis)
a. Afecta principalmente el fondo y cuerpo gástrico
b. Se produce hiperplasia de células G secundaria a la aclorhidria
c. El propio sistema inmune destruye principalmente las células parietales
d. Se produce atrofia de la mucosa, aclorhidria, hipergastrinemia y déficit de vitamina B6

39. Marque la correlación correcta:


(Unidad 3, sesión:18, logros:1 y 2: Describe las enfermedades inflamatorias/infecciosas y
proliferativas de la cavidad oral)

1. Herpes virus( ) En relación al abuso de antibióticos


2. Candidiasis oral( ) Lesiones vesiculares como racimo de uvas
3. Eritroplaquia( ) Mega esófago
4. Enfermedad de Chagas( ) Lesión pre cancerígena

a.- 2431b.- 1234c.- 4123d.- 2143

40. En un paciente con insuficiencia renal crónica, el déficit en la absorción de calcio a nivel
del enterocito se debe a lo siguiente:
(Unidad 4, sesión 26, logro 6: Explica la Absorción de calcio y hierro)
a. No se convierte la 25 hidroxicolecalciferol a 1,25 dihidroxicolecalciferol
b. No se convierte la 1,25 dihidroxicolecalciferol a 25 hidroxicolecalciferol
c. Existe un descenso de la alfa 25 hidroxilasa renal
d. Se incrementa la producción de Calbindina
SISTEMA DIGESTIVO (ME154) EXAMEN PARCIAL 2019 01

Profesores : Alfaro Salazar, Herberth Romulo; Callata Caceres, Gunter; Cayo Quiñe, Alexandra Mariel;
Correa Borit, Jorge Mauricio; Cruz Cutty, Lourdes Marylin; Guzmán Calderón, Gerly Edson;
Jáuregui Farfán, Jorge Jesús; Mayor Zevallos, Otto Alberto; Montoya Suárez, José Luis;
Palacios Bazan, Enrique Elias; Robles Pino, Alexander Anibal; Wong Bravo, Juan Carlos
Sección : Todas las secciones
Duración : 50 minutos.
Indicaciones:
Lea atentamente cada pregunta antes de responder:
- Se prohíbe el uso del celular y cualquier dispositivo electrónico.
- Está prohibido intercambiar materiales.
- Coloque su código de alumno en la tarjeta de respuestas. Si su código contiene una letra
reemplácela por un valor numérico siguiendo la siguiente equivalencia: A=9, B=8, C=7, D=6, E=5, F=4,
G=3.
- Traslade sus respuestas a la tarjeta, llenando los círculos de manera completa con lapicero negro o
azul. Está prohibido el llenado con lápiz, lapicero de otro color o con lapicero de tinta borrable.
- Sea cuidadoso en el llenado de la tarjeta de respuestas, pues solo esta tiene validez para la
calificación.
- Al terminar su examen avise al docente a cargo, no se levante de su sitio; debe entregar la hoja de
respuestas con la carátula del examen, este cuadernillo de preguntas se lo llevará cada estudiante.

1. ¿Cuál de las glándulas salivales es responsable del mayor porcentaje del volumen de la saliva en
condiciones basales?
a. Parótida
b. Sub
pala
tina
s c.
Subl
ingu
laes
d.
Sub
max
ilare
s

2. La lengua está recubierta por epitelio:


a. pseudoestratificado columnar no queratinizado
b. plano estratificado no queratinizado
c. pseudoestratificado columnar ciliado
d. plano estratificado queratinizado

3. El esfínter anal interno tiene musculatura …………… y tiene control ………………….


a. lisa / voluntario
b. lisa / involuntario
c. esquelética / simpático
d. esquelética / parasimpático

4. La arteria aorta proporciona la irrigación al tubo digestivo ¿cuál de las siguientes arterias
proporciona la irrigación al ángulo cólico derecho?
a. Mesentérica superior
b. Mesentérica inferior
c. Frénica inferior
d. Tronco celiaco

5. Paciente de 26 años que le cuenta en su historia clínica que cada vez que almuerza, a los 20
minutos tiene deseo de defecar. Le comenta que su hijo de 1 mes le pasa lo mismo pero más
intenso. Esto se explica por el reflejo
…………………, el cual estáen el paciente.
a. colicoileal / normal
b. colicoileal / alterado
c. gastrocolico / normal
d. gastrocolico / alterado
6. La región del estómago que se comunica con el duodeno se denomina:
a. pilórica
b. cardias
c. cuerpo
d. fórnix

7. Acude a consulta un paciente que fue diagnosticado de ulcera péptica 3 días antes. Luego de múltiples
pruebas diagnósticas, se concluye que el paciente presenta un tumor secretor de gastrina ¿Cuál de las
siguientes situaciones estará incrementada?
a. Distención gástrica
b. Inhibición del vaciado gástrico
c. Secreción de ácido clorhídrico (HCl)
d. Inhibición de la secreción de pepsinógeno

8. En el sistema digestivo, el control del apetito esta dado por un complejo sistema de sustancias y
órganos integradores los cuales regulan la ingesta de alimentos. Laes una sustancia oroxígena y es
sintetizada
por el ……………………..
a. leptina / intestino
b. grelina / intestino
c. leptina / estómago
d. grelina / estómago

9. Sobre el control autónomo del sistema digestivo, marque la alternativa correcta:


a. La inervación dada por el sistema simpático es de tipo preganglionar.
b. El sistema parasimpático usa como neurotransmisores a la acetilcolina y la noradrenalina.
c. El nervio vago (par craneal X) le da inervación simpática a la mayoría del sistema digestivo.
d. En el sistema simpático, los nervios responsables hacen una primera sinapsis en
ganglios próximos al órgano a inervar.
e. En la inervación de tipo parasimpático, solo interviene el plexo submucoso, sin
embargo, en la de tipo simpático intervienen tanto el submucoso como el mientérico.

10. Con respecto a la actividad eléctrica del sistema digestivo, marque la alternativa correcta
a. Corresponden a potenciales de acción que están presentes de forma continua y le dan la
capacidad de perístasis autónoma al sistema digestivo.
b. La frecuencia de las ondas lentas no se ve influenciada por la actividad neural
ni las hormas gastrointestinales.
c. En el estómago las ondas lentas se dan en una frecuencia de 6 por minuto.
d. Las ondas lentas son cambios lentos y ondulantes del potencial en reposo.
e. La frecuencia de las ondas lentas va de 6 a 12 ondas por minuto.

11. Ante una lesión del IX par craneal, el músculo…se altera en su función.
a. palatogloso
b. estilofaríngeo
c. palatofaríngeo
d. constrictor superior

12. Un varón de 50 años es sometido a extirpación de duodeno y parte proximal de yeyuno. La


pérdida de estímulo hormonal en el páncreas para la secreción enzimática se explica por la
pérdida de las células ……………………
a. Parietales, productoras de factor intrínseco
b. “K” productoras de factor intrínseco
c. “M” productoras de CCK
d. “I” productoras de CCK
13. Respecto al mecanismo de la defecación ¿Cuál de las siguientes afirmaciones es correcta?
a. Se produce contracción refleja del esfínter anal interno
b. Se produce contracción o relajación del esfínter anal externo por señales de la corteza
cerebral
c. La presencia de materia fecal en el recto estimula la contracción del sigmoides por los
nervios pélvicos simpáticos
d. En la posición de “cuclillas” el músculo puborectal se halla contraído favoreciendo la
evacuación de la materia fecal

14. Un niño de tres años llega a emergencia con disfagia (dificultad para tragar), salivación y llanto. Se
sospecha de ingesta de cuerpo extraño: moneda en el esófago; al ser evaluado se constata en una
radiografía presencia de cuerpo extraño a nivel de C6 y C7 (6° y 7° vértebra cervical). El cuerpo extraño
estará suspendido a nivel del estrechamiento producido por el ………..
a. cayado aórtico
b. hiato esofágico
c. músculo cricofaríngeo
d. bronquio principal izquierdo

15. En el caso de un paciente con un tumor productor de gastrina, la presencia de úlceras duodenales y
erosión de la mucosa gástrica se debe principalmente a…….
a. la acción paracrina de la gastrina sobre la célula parietal
b. el exceso de HCl por estímulo de receptores CCK-B en la célula parietal
c. la sobre expresión de los receptores “G” para gastrina en la célula parietal
d. el exceso de HCl por estímulo directo de receptores “H” en la célula parietal

16. La onda peristáltica secundaria del esófago se caracteriza por ser originada ………
a. por el plexo de submucoso del esófago
b. por el plexo mientérico del esófago
c. por el reflejo de la deglución
d. durante la masticación

17. Marque lo correcto sobre las ondas lentas en el tubo digestivo


a. No son despolarizaciones
b. Son potenciales de acción subumbrales
c. Se constituyen de despolarizaciones y repolarizaciones
d. Son rítmicas y generadas por el sistema nerviosos autónomo

18. Recién nacido que presenta protrusión de contenidos abdominales los cuales no están cubiertos
por peritoneo y salen de la cavidad abdominal a través de un defecto de la pared. ¿Cómo se
denomina a la afección que presenta este paciente?
a. Onfalocele
b. Atresia biliar
c. Gastrosquisis
d. Divertículo de Meckel

19. Experimentalmente se utiliza atropina (anticolinérgico) para inhibir la secreción de gastrina, sin
embargo, la secreción de esta hormona se sigue dando ante estímulos vagales. Esta situación se
explica porque la atropina:
a. no bloquea la acción del péptido GRP
b. solo inhibe la acción del péptido GRP en la célula G
c. inhibe la acción de acetilcolina e histamina en la célula G
d. bloquea parcialmente la bomba de protones en la célula G

20. Niña de 4 días es llevada a la emergencia pediátrica por presentar llanto constante, la madre refiere
coloración azulada de labios al momento de lactar, acompañado de tos persistente y dificultad
respiratoria así como distención abdominal. Se le coloca sonda nasogástrica para alimentación
notando que retorna a la cavidad oral en todos los intentos. ¿Cuál es la anomalía del desarrollo en
este caso?
a. Solo fístula traqueo esofágica
b. Fístula traqueo esofágica proximal y distal
c. Atresia esofágica proximal con fístula traqueo esofágica distal
d. Atresia esofágica distal con fístula traqueo esofágica proximal
21. Paciente varón de 36 años es traído a la emergencia luego de sufrir un accidente de tránsito, presenta
traumatismos múltiples en cabeza y tronco. Al examen físico se evidencia hematoma en hemicara
izquierda, ligera protrusión y caída del lado izquierdo del maxilar inferior, por lo que se realiza una
tomografía donde se halla una fractura de la apófisis coronoides del maxilar inferior. ¿Qué músculo
está relacionada directamente con esta situación?
a. Masetero
b. Temporal
c. Buccinador
d. Pterigoideo medial

22. Un paciente refiere no percibir algunos sabores. Al examen físico constata alteración del sabor dulce y
umami.
¿Cuál de los siguientes nervios estará alterada su función?
a. Cuerda del tímpano (VII par)
b. Lingual (rama del V par)
c. Glosofaríngeo (IX par)
d. Hipogloso (XII par)

23. A los pocos días de nacido, regresa a neonatología un niño con problemas de motilidad del colon; los
estudios determinan ausencia congénita de células ganglionares. Según el gráfico ¿cuál es la capa en
la que se determina la ausencia de dichas células?
a. Mucosa - 2
b. Muscular propia - 2
c. Muscular propia - 3
d. Muscular de la mucosa - 3

1 2

24. Con respecto al control autonómico en el tracto gastrointestinal y en relación a su fisiología. ¿Cuál
es la función del sistema nervioso parasimpático en el tracto gastrointestinal?
a. Inhiben la contracción muscular y estimulan la secreción de sustancias a nivel de la
submucosa
b. Estimulan la contracción muscular y estimulan la secreción de sustancias a nivel de la
mucosa
c. Inhiben la contracción muscular e inhiben la secreción de sustancias a nivel de la submucosa
d. Estimulan la contracción muscular e inhiben la secreción de sustancias a nivel de la mucosa

25. Un estudiante que está preocupado por su examen parcial, no ha desayunado ni almorzado; cuando
al fin ingiere alimentos, esto le provoca el aumento de los movimientos musculares del tracto
gastrointestinal y la sensación de defecar. ¿Qué reflejo se ha activado?
a. Entero-gástrico
b. Gastro-cólico
c. Cólico-ileal
d. Ileo-ileal
26. ¿De qué par craneal es rama el nervio palatino mayor?
a. Vago
b. Hipogloso
c. Trigémino
d. Palatogloso

27. ¿En cuál de las fases de la deglución la epiglotis separa la vía respiratoria de la digestiva?
a. oral
b. laríngea
c. faríngea
d. esofágica

28. Los péptidos intestinales se pueden clasificar como sustancias endocrinas, neurocrinas y paracrinas,
dentro de las paracrinas se encuentran la somastotatina e histamina. Marque la respuesta correcta
a. La somastotatina es sintetizada por las células B de la mucosa gástrica
b. La histamina actúa estimulando su receptor tipo H1 en la mucosa gástrica
c. La histamina es sintetizada por células de tipo paracrino de las glándulas gástricas
d. La somatostatina presenta dentro de sus funciones la estimulación de la secreción de H +

29. En relación a los órganos intraabdominales y sus estructuras de fijación, elija la


alternativa correcta a.El mesenterio permite la suspensión e irrigación de los
órganos retroperitoneales
b. Tanto el hígado como la vesícula biliar se encuentran ubicados a nivel del flanco derecho
c. El colon, el duodeno y el resto de intestino delgado son órganos considerados
netamente como peritoneales
d. Los ligamentos que encontramos dentro de la cavidad abdominal son el esplenorenal y el
gastrofrénico e.Los omentos van desde el estómago y la segunda porción del duodeno a otras
estructuras
intraabdominales y existen dos: el omento mayor y el omento menor

30. Paciente varón de 27 años es llevado por bomberos a emergencia luego de ser asaltado y, tras
resistirse, es cortado con el pico de una botella a nivel abdominal. Al examen físico usted observa que
a través de la herida se puede observar la protrusión de asas intestinales. En relación con las capas de
la pared abdominal, marque la alternativa correcta.
a. La fascia de Scarpa está constituida principalmente por tejido adiposo
b. La pared abdominal está formada por piel, huesos, músculos, fascias y peritoneo parietal
c. La fascia de Camper es una estructura fibrosa que carece de grasa y su grosor es
constante en toda la pared abdominal
d. El músculo oblicuo externo discurre en dirección súpero-interna y se inserta en el borde
inferior de las ultimas 3 a 4 costillas
e. El músculo recto del abdomen tiene como funciones comprimir el contenido del
abdomen, tensar la pared del abdomen y flexionar la columna
SISTEMA
DIGESTIVO
(ME154)
EXAMEN FINAL
2019 01

Profesores : Alfaro Salazar, Herberth Romulo; Callata Caceres, Gunter; Cayo Quiñe, Alexandra Mariel;
Correa Borit, Jorge Mauricio; Cruz Cutty, Lourdes Marylin; Guzmán Calderón, Gerly Edson;
Jáuregui Farfán, Jorge Jesús; Mayor Zevallos, Otto Alberto; Montoya Suárez, José Luis;
Palacios Bazan, Enrique Elias; Robles Pino, Alexander Anibal; Wong Bravo, Juan Carlos
Sección : Todas las secciones
Duración : 70 minutos.
Indicaciones:
Lea atentamente cada pregunta antes de responder:
- Se prohíbe el uso del celular y cualquier dispositivo electrónico.
- Está prohibido intercambiar materiales.
- Coloque su código de alumno en la tarjeta de respuestas. Si su código contiene una letra
reemplácela por un valor numérico siguiendo la siguiente equivalencia: A=9, B=8, C=7, D=6, E=5, F=4,
G=3.
- Traslade sus respuestas a la tarjeta, llenando los círculos de manera completa con lapicero negro o
azul. Está prohibido el llenado con lápiz, lapicero de otro color o con lapicero de tinta borrable.
- Sea cuidadoso en el llenado de la tarjeta de respuestas, pues solo esta tiene validez para la
calificación.
- Al terminar su examen avise al docente a cargo, no se levante de su sitio; debe entregar la hoja de
respuestas con la carátula del examen, este cuadernillo de preguntas se lo llevará cada estudiante.

1. La explicación fisiológica de presentar somnolencia de 30 minutos a 1 hora después de ingerir


alimentos, se explica por:
a. Aumento del cloro intraluminal
b. Aumento del bicarbonato intraluminal
c. Disminución de ácido carbónico en la célula parietal
d. Disminución de la actividad de la anhidrasa carbónica
e. Aumento de la alcalinidad sanguínea

2. Con respecto a la irrigación arterial del colon, a que arteria


corresponde la señalada con la flecha
a. Cólica derecha
b. Cólica media
c. Cólica izquierda
d. Ileobisecoapendículocólica
e. Arco de Riolano

3. Si un paciente presentara dentro del punto de vista fisiológico, una


disminución de enterocinasa, entonces esto originaría una
disminución de la actividad de:
a. L
a
p
e
p
s
i
n
a
b
.l
a
li
p
a
s
a
c. la quimotripsina
d. el peptido insulinotropo dependiente de glucosa
e. la amilasa

Se valida la opción b debido a su relación con la colipasa.


4. Con respecto a la anatomía del hígado, señale a que
estructura pertenece la marcada por el número 1.
a. Ligamento falciforme
b. Línea de Cantlie
c. Ligamento triangular
d. Ligamento coronario
e. Ligamento teres

Se valida la opción a debido a la ubicación del número 1 en


donde se unen el ligamento falciforme y ligamento coronario.

5. Se presenta un paciente, el cual presenta un antecedente de tuberculosis intestinal, por lo cual,


se le resecó 80 cm de íleon distal. Desde el punto de vista fisiológico, el paciente puede presentar
una de las siguientes alteraciones:
a. Disminución de la secreción de Vitamina B12
b. Aumento indiscriminado de absorción de ácido fólico
c. Disminución de la absorción de hierro
d. Aumento de la secreción de bicarbonato
e. Disminución de la absorción de ácido glicocólico

6. Un paciente es sometido experimentalmente a un fármaco que modifica el flujo salival, obteniéndose


un volumen de saliva de 288 ml en 6 horas. En este caso las concentraciones de electrolitos y
bicarbonato en la saliva obtenida varían de la siguiente manera:
a.↑ Na+, ↓ K+, ↑
Cl-, ↑ HCO3- b.↓
Na+, ↓ Cl-, ↑ K+, ↓
HCO3-
c.↑ Na+, ↑ Cl-, ↓ K+, ↓ HCO3-
d.↑ Na+, ↑ Cl-, ↑ K+, ↑ HCO3-
e.↓ Na+, ↓ Cl-, ↓ K+, ↓ HCO3-

Se valida la opción a debido a que se puede considerar como un aumento del flujo de saliva.

7. La siguiente imagen histológica corresponde a la glándula


…………… y la estructura señalada produce ………
a. salival sublingual / mucopolisacáridos
b. oxíntica / pepsinógeno
c. salival submaxilar / ptialina
d. salival parótida / amilasas
e. antrales / gastrina

8. Paciente varón de 65 años con antecedente de hipercolesterolemia, hipertensión arterial, fibrilación


auricular y dos infartos al miocardio previos, aqueja de dolor abdominal intenso de inicio súbito,
distensión abdominal, se decide cirugía con resección de 1,5 metros de intestino delgado terminal y
colon ascendente. Como consecuencia de la resección el paciente tendrá deficiencia de:
a. Vitamina C
b. Tiamina
c. Vitamina A
d. Vitamina B1
e. Vitamina B6

Se valida esta opción debido a que su absorción está relacionada al íleon.


9. Uno de los siguientes elementos debería hallarse con más probabilidad en el esófago de un paciente
que sufre de
reflujo gastro esofágico…
a. Pepsina
b. Tripsina
c. Quimiotripsina
d. Carboxipeptidasa
e. Ácidos biliares

10. Un paciente de 40 años cursa con anemia de 8g/dl, aqueja además de astenia y sensación de
hormigueo bilateral en los miembros inferiores, al examen se halla alteración de la sensibilidad a la
vibración y camina con ampliación de la base de sustentación. Uno de los siguientes procedimientos
sería de ayuda para el diagnóstico de este paciente:
a. Tomografía cerebral
b. Biopsia de la mucosa gástrica
c. Biopsia de hígado
d. Examen de sangre oculta en heces
e. Biopsia de Ileon proximal

11. Paciente de 60 años ingresa por caída hace 1 hora y pequeño hematoma en cuero cabelludo, al
examen físico ampliado se observa ictericia de piel y mucosas generalizada, abdomen blando, se
palpa estructura quística no dolorosa en hipocondrio derecho que corresponde a vesícula biliar
(signo de Courvoisier), en los exámenes de laboratorio se halla niveles bajos en la formación de
estercobilinógeno y urobilinógeno en heces, incremento de la bilirrubina conjugada en la orina,
elevación de fosfatasa alcalina y gamma glutamil transpeptidasa séricas. El presente cuadro puede
ser explicado por:
a. Reabsorción de hematoma
b. Litiasis vesicular
c. Carcinoma de la cabeza de páncreas
d. Carcinoma con estenosis del conducto hepático común
e. Anemia hemolítica

12. Paciente varón de 58 años con antecedente de alcoholismo crónico es diagnosticado y recibe
tratamiento por cirrosis hepática. Hace 2 días refiere familiar que tuvo cambio de conducta y no
reconoce a algunos familiares. Al examen físico, se halla ascitis, circulación colateral en abdomen,
telangiectasias, en el examen de sistema nervioso: rigidez de extremidades, ROT incrementados,
desorientación en el espacio y asterixis. ¿cuál de las siguientes circunstancias, explicaría el cuadro
en este paciente?
a. Uso de diuréticos ahorradores de potasio
b. Incremento de actividad de ureasa bacteriana duodenal
c. Hemorragia gastrointestinal
d. Disminución de la producción de NH3+ en el colon
e. Dieta normo proteica

Se valida la opción e debido al efecto sobre la encefalopatía.


Con respecto de la opción b es incorrecta debido a que hace referencia al duodeno, debería indicar colon.

13. Un recién nacido presenta vómitos biliosos poco tiempo después de cada alimento. Al preguntar a la
madre sobre antecedentes, ella recuerda que tuvo polihidramnios durante la gestación, pero un
análisis de cariotipo fue normal. Una de las siguientes es la causa más probable de estos hallazgos
en el recién nacido:
a. Enfermedad de Hirschprung
b. Fístula tráqueo esofágica
c. Divertículo ileal
d. Estenosis pilórica
e. Malrotación de la yema pancreática ventral

14. Un lobulillo hepático se puede dividir en tres zonas como se muestra en el gráfico. ¿Cuál de
las siguientes afirmaciones sobre las tres zonas es verdadera?
a. La zona 1 tiene los menores depósitos de glucógeno
b. La zona 3 es la primera en afectarse en una colestasis extra hepática
c. La zona 2 es más susceptible a la injuria por isquemia que la zona
periportal d.La zona 2 tiene la mayor capacidad de regeneración
e.La zona 1 es la que tiene menos actividad metabólica.

La pregunta 14 ha sido anulada, sin embargo, ningún estudiante se verá afectado


negativamente en su puntaje debido a esta anulación.

15. En un estudio de la secreción de hormonas gastrointestinales, sus concentraciones en la vena


porta se midieron durante perfusión luminal del intestino delgado con soluciones de diversas
magnitudes de pH. ¿Qué hormona aumentará en el plasma de la vena porta durante perfusión a
través del intestino con una solución de pH 3?
a. CCK
b. gastrina
c. GIP
d. motilina
e. secretina

16. Paciente de 30 años que ingresa a causa de un traumatismo abdominal cerrado. En la exploración
se aprecia discreta palidez de piel y mucosas, auscultación pulmonar normal, taquicardia de 120
/min. Discreta distensión abdominal y matidez en flancos; el hematocrito, que era prácticamente
normal al ingreso, disminuye a 30% a las tres horas. En la Rx de tórax se objetiva fractura de las
costillas 10-11 izquierdas. La causa más probable de la anemización en este paciente es:
a. traumatismo renal con hemorragia retroperitoneal.
b. rotura de hígado con hemoperitoneo.
c. rotura de bazo con hemoperitoneo.
d. rotura de mesos con hemoperitoneo.
e. traumatismo pancreático con pancreatitis traumática.

17. Mujer de 65 años. Consulta por síndrome constitucional


asociado a dolor abdominal epigástrico progresivo
irradiado a espalda, de dos meses de evolución. El
diagnostico de sospecha de adenocarcinoma de páncreas
se confirma por biopsia. Se realiza examen de imagen de
abdomen para evaluación de estructuras vasculares
próximas al tumor pancreático. ¿Cuál es el nombre de la
vena señalada que está ausente, trombosada por
infiltración tumoral, condicionando circulación colateral
en la pared gástrica?

a. Mesentérica superior
b. Coronaria estomaquica
c. Esplénica
d. Porta
e. Renal izquierda
18. Revisando la angiotomografía de un hombre de 70 años en estudio por aneurisma de aorta
abdominal, el radiólogo le informa de la presencia de una oclusión completa de la arteria
mesentérica inferior. El paciente se encuentra completamente asintomático. La oclusión de la
arteria mesentérica inferior cursa de manera asintomática en muchas ocasiones ya que el
territorio que irriga puede recibir flujo proveniente de la arteria:
a. cólica derecha
b. gastroduodenal
c. Epigástrica inferior izquierda
d. esplénica
e. cólica media

19. En las patologías de esófago es importante conocer bien la anatomía esofágica. ¿Cuál de
las siguientes afirmaciones es correcta?
a. El esófago tiene capa mucosa, muscular y serosa
b. El esófago abdominal es más largo que el cervical
c. El esófago torácico pasa por detrás del cayado aórtico
d. El epitelio esofágico normal es de tipo cilíndrico.
e. El esófago abdominal es discretamente más largo que el torácico

20. A pesar de que pueda haber variaciones anatómicas, lo habitual es que el ciego sea irrigado por una
rama arterial que proviene de unas de las siguientes arterias:
a. Iliaca derecha
b. Mesentérica inferior
c. Hepática derecha
d. Mesentérica superior
e. Iliaca izquierda

21. Ante un paciente con una cirugía abdominal urgente, el informe operatorio señala que se ha
realizado una resección de todo el duodeno y del tercio proximal del yeyuno manteniendo íntegros
el estómago y todo el íleon, así como los dos tercios distales del yeyuno. En el seguimiento
nutricional del paciente ¿Qué vitamina o mineral presentará con menor probabilidad una
disminución de su absorción?
a. Cianocobalamina
b. Calcio
c. Hierro
d. Transcobalamina
e. Transferrina

22. ¿Cuál de las siguientes alternativas detallan las venas que confluyen y forman la vena señalada?
a. mesentérica superior, gástrica izquierda y
gastroepiploica izquierda
b. mesentérica inferior, gástrica izquierda y renal
c. esplénica, mesentérica superior y mesentérica inferior
d. esplénica, pancreatoduodenal y omental izquierda
e. gástrica izquierda, esplénica y hepática común

23. ¿Cuál de las siguientes sustancias forma parte de la secreción biliar?


a. Tripsina
b. Lecitina
c. Elastasa
d. Quimotripsina
e. Pepsina
24. El tubo digestivo contiene diferentes tipos de epitelios y glándulas. La estructura señalada es unay
está localizada en el …………...
a. glándula de Brunner /
intestino grueso b.cripta de
Lieberkuhn / colon
c. cripta de Lieberkuhn / intestino delgado
d. glándula oxintica / estomago
e. célula parietal / estómago

Aunque las criptas de


Lieberkuhn están
presentes en el intestino
delgado, la
microfotografía es de
epitelio de colon.

25. ¿De qué musculo forma parte el ligamento inguinal?


a. Oblicuo externo del abdomen
b. Oblicuo interno del abdomen
c. Transverso del abdomen
d. Psoas
e. Dorsal ancho

26. Señale cuál de las siguientes afirmaciones NO se relaciona a la siguiente glándula anexa del
tubo digestivo mostrada en la imagen:
a. Es una glándula exocrina compuesta exclusivamente por acinos serosos
b. Su inervación está dada por el nervio auricular mayor (ramo posterior C2), que inerva
la vaina de la glándula así como la piel por encima de esta.
c. Esta glándula produce una secreción mucinosa acuosa, llamada mucoserosa, a través
del conducto de Wharton.
d. Su inflamación puede ser causada por un virus de los Paramyxoviridae, que provocan
una enfermedad muy frecuentemente en niños y adolescentes
e. Es una glándula endocrina y probablemente sea de origen pancreático

Se valida la opción e debido a que no


está relacionada con la imagen.

27. ¿Cuál de las siguientes enzimas está localizada en el borde en cepillo y juega un rol en la digestión de
proteínas?
a. Alfa dextrinasa
b. Pepsina
c. Enterocinasa
d. Lactasa
e. Carboxipeptidasa A.

Se valida la opción c debido a que es correcta en relación a la pregunta.


28. Una de los siguientes sustancias, NO sirve como un buen agente emulsificante:
a. Colesterol
b. Ácidos grasos
c. Sales biliares
d. Lecitina
e. Proteínas de la dieta

Se valida la opción e debido a que es correcta en relación a la pregunta.

29. La sustancia que estimula el crecimiento de la mucosa gástrica es:


a. Secretina
b. Motilina
c. Péptido estimulante de la mucosa gástrica
d. Gastrina
e. Histamina

30. ¿Cuál de las siguientes alternativas es una función de la colecistokinina?


a. Relajación de la vesícula para la
salida de bilis b.Secreción de ácidos
biliares
c. Contracción del esfinter de Oddi
d. Secreción de enzimas pancreáticas
e. Contracción del duodeno

Se valida la opción b debido al efecto de la CCK sobre la vesicula biliar.

31. Con respecto a la anatomía del tronco celiaco, señale lo correcto


a. El tronco celiaco se origina de la cara posterior de la aorta abdominal
b. Es una arteria delgada que tiene un calibre entre 2 y 3 mm
c. Una de sus ramas es la arteria gástrica derecha
d. La hepática común que es una de sus ramas, participa en la irrigación del estómago.

32. Con respecto a la anatomía del duodeno, marque la respuesta correcta:


a. Tiene una distribución en forma de “C”, que rodea la cola del páncreas
b. La 3ra porción duodenal está contenida en la pinza vascular aortomesentérica
c. Entre la 1ra y 2da porción se forma un ángulo, conocido como el ángulo de Treitz
d. La 4ta porción se dirige a la izquierda, hacia abajo y hacia atrás.
e. En la tercera porción desemboca el conducto colédoco.

33. El hígado está ampliamente tapizado por peritoneo, la estructura que conecta la cara
diafragmática del hígado precisamente con el diafragma es el ligamento:
a. teres
b. f
a
l
c
if
o
r
m
e
c
.t
ri
a
n
g
u
l
a
r
d.hepá
tico
común
e.coro
nario

Se validan la opción c y e debido a que forman parte de los ligamentos que fijan el hígado al diafragma.

34. En el íleon se absorbe aproximadamente el 95% dea través de la circulación enterohepática.


a. agua
b. colesterol
c. sales biliares
d. hidróxicobalamina
e. factor intrínseca
35. Laestimula el mecanismo paracrino de la secreción de ácido clorhídrico.
a. histamina
b. acetilcolina
c. gastrina
d. secretina
e. somatostatina

36. En la digestión de proteinas,es el principal estímulo para convertir el pepsinógeno en pepsina.


a. la gastrina
b. el pH ácido
c. la acetilcolina
d. la ptialina
e. la somatostatina

37. Con respecto a la somatostatina, marque lo correcto:


a. Es secretada por las células S del intestino
b. Induce a la producción de VIP
c. Interviene en la fase intestinal de la secreción gástrica
d. Produce acetilcolina para estimular a la célula parietal
e. No interviene en la regulación de la secreción de ácido clorhídico

38. En pecten anal, es una estructura comprendida entre:


a. la línea pectínea y los senos anales
b. la línea blanca y la apertura anal
c. el esfínter anal interno y el externo
d. la línea anocutánea y la línea pectínea
e. la línea blanca y columnas anales

39. ¿Cuál de las siguientes alternativas es una proenzima pancreática?


a. Tripsina
b. Elastasa
c. Quimotripsinógeno
d. Amilasa
e. Procarboxipepitidasa C.

40. En la segmentación hepática de Coinaud, el segmento hepático señalado con la flecha,


corresponde a : En la segmentación hepática de Coinaud, la flecha señala el
segmentohepático.
a. IV
b. V
c. VI
d. VII
e. VIII
EXAMEN
PARCIAL
SISTEMA
DIGESTIVO
(ME154)
Ciclo 2019-02

Sección:Todas
Profesores:Alfaro Salazar, Herberth Romulo; Alva Muñoz, Jose Carlos; Mayor Zevallos, Otto Alberto;
Duración:30 minutos.
Indicaciones:
- Lea atentamente cada pregunta antes de responder.
- Se prohíbe el uso del celular y cualquier dispositivo electrónico.
- Está prohibido intercambiar materiales.
- Coloque su código de alumno en la tarjeta de respuestas. Si su código contiene una letra
reemplácela por un valor numérico siguiendo la siguiente equivalencia: A=9, B=8, C=7, D=6, E=5, F=4
y G=3.
- Traslade sus respuestas a la tarjeta, llenando los círculos de manera completa con lapicero negro o
azul. Está prohibido el llenado con lápiz, lapicero de otro color o con lapicero de tinta borrable.
- Sea cuidadoso en el llenado de la tarjeta de respuestas, pues solo esta tiene validez para la
calificación.
- Al terminar su examen avise al docente a cargo, no se levante de su sitio; debe entregar la hoja de
respuestas con la carátula del examen, este cuadernillo de preguntas se lo llevará cada estudiante.

1. La contracción del músculo………. permite la eliminación de gases (flatos) sin salida de material fecal;
es el mismo músculo cuya relajación, sobretodo en cuclillas, permite el paso del contenido
fecal con menor esfuerzo durante la defecación.
a) Isq
uir
ec
tal
b)
Pu
bo
rre
ct
al
c) Esfínter anal externo
d) Esfínter anal interno

2. Paciente mujer de 54 años se presenta con náuseas, vómitos, estreñimiento, y es


diagnosticada de abdomen agudo quirúrgico; en la cirugía encuentran un vólvulo de ciego. Esta
anomalía puede explicarse por::
a) Falta de rotación intestinal
b) Falta de fusión del mesenterio
c) Defecto en la formación de la cloaca
d) Falta de formación del omento mayor

3. Paciente mujer de 23 años con faringitis aguda, toma para el dolor una tableta de paracetamol con
un poco de agua. Durante la deglución, se relaja su esfínter esofágico inferior y el fondo del
estómago, mientras el bolo está aún en el esófago. ¿Qué sustancia provocara con mayor
probabilidad la relajación del esfínter esofágico inferior y el fondo del estómago en esta mujer?
a) Óxido nítrico
b) Sustancia P
c) Histamina
d) Motilina

4. Luego de tres horas dando exámenes, un alumno de medicina comienza a sentir hambre.
Esta situación es probable que sea mediada por la que es sintetizada por el :
a) leptina / intestino
b) leptina / estómago
c) grelina / estómago
d) grelina / tejido adiposo

5. Varón de 72 años, con antecedente de diabetes mellitus tipo 2, que presenta enteropatía diabética
caracterizada por estreñimiento. Este problema puede estar asociado a:
a) deficiencia de óxido nítrico
b) aumento del reflejo gastrocólico
c) disminución de la secreción de colecistocinina (CCK)
d) aumento de la secreción del péptido intestinal vasoactivo (PIV)
6. Varón de 54 años con Diabetes Mellitus tipo 2, es diagnosticado de gastroparesia debido a que
presenta sensación de llenura precoz al comer, y reflujo gastroesofágico. Esta alteración en la
relajación receptiva y en el vaciamiento gástrico lo más probable es que se deba a una alteración
en:
a) el nervio vago
b) el ganglio celíaco
c) plexo submucoso
d) nervio hipogástrico

7. Varón de 67 años con tos y disminución de peso asociado a tabaquismo pesado, presenta
actualmente disfagia progresiva a alimentos sólidos. Se considera la presencia de un carcinoma
de bronquio izquierdo y por esta razón le realizan una endoscopía esofágica para descartar la
posibilidad de una compresión esofágica por el tumor. Se espera revisar el esófago en la
estrechez, que está a nivel de la vértebra
a) Tercera estrechez -T6
b) Segunda estrechez - C6
c) Segunda estrechez - T4
d) Tercera estrechez -T10

8. Varón de 34 años con dolor abdominal agudo en flanco derecho que se irradia a fosa ilíaca derecha,
es operado y se encuentra un divertículo intestinal inflamado, ubicado a 93 cm de la válvula
ileocecal. El origen de este divertículo es una falla en la obliteración de:
a) Conducto vitelino
b) Alantoides
c) Cloaca
d) Conducto anorectal
e) Uraco

En un niño menor de dos años con divertículo intestinal, este divertículo tiene su origen en
una falla en la obliteración de:
a)Conduct
o
anorectal
b)
Conducto
vitelino
c) Alantoides
d) Cloaca
e) Uraco

9. Mujer de 43 años sufre un grave accidente de tránsito y está hospitalizada en coma, es alimentada
por vía intravenosa durante varias semanas. Producto de este tipo de alimentación, se encuentra en
la endoscopía atrofia de la mucosa gastrointestinal. La causa más probable de esta atrofia son los
bajos niveles séricos de la hormona:
a) Colecistocinina
b) S
e
c
r
e
t
i
n
a
c
)
G
a
s
t
r
i
n
a
d) PIV

10. Una mujer de 30 años llega al consultorio porque se queja de dificultades para deglutir, la cual se
agravan cada vez más. Se realiza un estudio manométrico para examinar la generación de presión a
lo largo del esófago. Esta prueba revela que las contracciones como respuesta a la deglución están
mal sincronizadas y que la presión en el esfínter esofágico inferior permanece elevada. El
diagnóstico más probable es producido por niveles bajos de
a) acalasia /
sustancia P
b) acalasia /
óxido nítrico
c) enfermedad por reflujo gastrointestinal / acetilcolina
d) enfermedad por reflujo gastrointestinal / óxido nítrico

11. Paciente de 2 años, llega a emergencia por haber ingerido una moneda con la que estaba jugando.
El lugar más probable donde puede haberse quedado suspendido este objeto es a nivel del
estrechamiento producido a nivel del:
a) músculo milohiodeo
b) músculo aritenoideo
c) músculo cricofaríngeo
d) constrictor superior de la faringe

12. En una apendicectomía, al realizar la incisión de McBurney en la fosa iliaca derecha, es


necesario cortar los siguientes músculos, de afuera hacia adentro:
a) Recto – Oblicuo externo – Transverso
b) Recto – Oblicuo externo – Oblicuo interno
c) Oblicuo externo – Oblicuo interno – Recto
d) Oblicuo externo – Oblicuo interno – Transverso

13. Un varón de 90 años que se encuentra postrado en cama, es referido del asilo para endoscopia
por dificultad para deglutir luego de tomar un medicamento para aliviar el dolor la noche
anterior. La endoscopía revela que la píldora se alojó en el esófago y causó una reacción
inflamatoria. Lo más probable es que esto haya sido por la producción de múltiples ondas:
a) secundarias
b) primarias
c) lentas
d) segmentarias

14. Mujer de 23 años es diagnosticada de bulimia, al examen físico se observa ulceraciones en el


segundo y tercero dedo de la mano derecha. Esto se puede deber al uso continuo de estos dedos
para inducir el vómito, mediante la estimulación del par craneal:
a) V
b
)
I
X
c) X
d) XI

15. Varón de 52 años se presenta por diarrea persistente de seis semanas de duración. En la
colonoscopia se observa un pólipo a nivel del íleon distal. El patólogo informa que se trata de un
tumor neuroendócrino, probablemente originado por las células enterocromafines del intestino. La
sustancia que más probablemente esté produciendo este tumor es:
a) Serotonina
b) Insulina
c) CCK
d) GIP

16. La fase oclusal de la masticación se realiza con la contracción de los músculos:


a) digástricos
b) masetero y temporal
c) orbicular y buccinador
d) pterigoideo lateral y digástrico

17. Al tomar su café en Starbucks, un estudiante de medicina sufre una quemadura de primer
grado en el tercio anterior de la superficie dorsal de la lengua. La información de dolor es
transmitida por el nervio:
a) cuerda del tímpano
b) glos
ofarí
nge
o
c)lin
gual
d) facial

18. Paciente es evaluado por faringitis aguda en consultorio externo. El médico de familia le solicita
que abra la boca y saque la lengua. Para realizar la acción de sacar la lengua, es necesario que se
contraiga el músculo:
a) e
st
il
o
gl
o
s
o
b)
g
e
ni
o
gl
o
s
o
c) palatogloso
d) transverso de la lengua
19. Paciente con síndrome de Sjögren,
presenta “boca seca” (disminución de la
producción de saliva) y caries dental,
asociada a la pérdida de la función de
tampón de la saliva. Esta
desminerilización del diente puede
comprometer a las prolongaciones
citoplasmáticas ubicadas en los tubos
huecos de la estructura señalada con la
letra:
a) B
b) A
c) E
d) C

20. Mujer de 32 años acude a consulta por


presentar disfagia de progresión lenta, reflujo
gastroesofágico y vómitos desde hace 3
meses de evolución progresiva. Se le realiza
un estudio radiológico con contraste en el que
se observa estrechamiento del esfínter
esofágico inferior (imagen). Según sus
conocimientos, este paciente se beneficiaría
con el uso de:
a) agonista beta adrenérgico
b) agonista alfa
adrenérgico
c)análogo de
óxido nítrico
d) análogo de Sustancia P
EXAMEN PARCIAL SISTEMA DIGESTIVO
(ME154)
Ciclo 202000

Sección:Todas
Profesores:Alva Muñoz, Jose Carlos
Duración:35 minutos.
Indicaciones:
- Lea atentamente cada pregunta antes de responder.
- Se prohíbe el uso del celular y cualquier dispositivo electrónico.
- Está prohibido intercambiar materiales.
- Coloque su código de alumno en la tarjeta de respuestas. Si su código contiene una letra
reemplácela por un valor numérico siguiendo la siguiente equivalencia: A=9, B=8, C=7, D=6, E=5, F=4
y G=3.
- Traslade sus respuestas a la tarjeta, llenando los círculos de manera completa con lapicero negro o
azul. Está prohibido el llenado con lápiz, lapicero de otro color o con lapicero de tinta borrable.
- Sea cuidadoso en el llenado de la tarjeta de respuestas, pues solo esta tiene validez para la
calificación.
- Al terminar su examen avise al docente a cargo, no se levante de su sitio; debe entregar la hoja de
respuestas con la carátula del examen, este cuadernillo de preguntas se lo llevará cada estudiante.

1. Paciente de sexo masculino de 82 años de edad ingresa a emergencia con dolor abdominal agudo y
diarreas. Se le realiza una arteriografía en la que se observa que la arteria aorta tiene un trombo
ocluyendo el 95% del flujo, a nivel del nacimiento de la arteria mesentérica inferior. ¿Cuál de las
siguientes arterias podría contribuir a la irrigación colateral del colon descendente?
a) cólica media
b) sigmoidea
c) rectal superior
d) ileocólica

2. Niño de 5 años presenta dolor esofágico y hematemesis (vómitos hemorrágicos) luego de tragarse
una espina de pescado. En la endoscopía se observa perforación del esófago distal a la cuarta
estrechez esofágica. ¿Las ramas de cuál de las siguientes arterias estarán lesionada con mayor
probabilidad?
a) Gástrica izquierda
b) Bronquiales
c) Frénica inferior
d) Tiroidea inferior

3. Al ingerir una cucharada de mantequilla es muy probable que se disminuya la sensación de hambre
por medio de la activación de la vía POMC/CART (POMC=proopiomelanocortina y
CART=transcripción regulada de cocaína y anfetamina), activada directamente por la hormona:
a) colecistoquinina (CCK)
b) insulina
c) grelina
d) secretina

4. Al comer unas papitas fritas con mayonesa, el vaciamiento gástrico disminuye por efecto directo
de la hormona: a)colecistoquinina (CCK)
b) bombesina
c) motilina
d) gastrina

5. Recién nacido de dos horas es diagnosticado de hernia umbilical de 1,5 cm de diámetro; el


cirujano pediatra solicita una tomografía abdominal en donde se evidencia que la hernia
umbilical está ocupada por una porción del tracto gastrointestinal. ¿Qué porción del tracto
gastrointestinal estaría ocupando esta hernia con mayor probabilidad?
a) Íleon
b) Colon sigmoides
c) Duodeno
d) Colon transverso
6. Recién nacido de 7 horas, de parto por cesárea debido a polihidramnios (aumento del volumen del
líquido amniótico), con regurgitación de la leche materna y artificial, y no ha presentado meconio.
Se le realiza una tomografía donde se evidencia aire en el estómago y una malformación del
desarrollo esofágico. Con respecto a esta malformación lo más probable es que se pueda tratar de
una atresia esofágica:
a) proximal con fístula traqueoesofágica distal
b) distal con fístula traqueoesofágica proximal
c) proximal y distal
d) sin fístula

7. Lactante de 6 meses de edad que es traído a consulta por presentar vómitos no biliosos a repetición
y retraso en el crecimiento. En la radiografía de abdomen simple se observa nivel hidroaéreo en
estómago y en primera porción de duodeno (doble burbuja). ¿Cuál de las siguientes alternativas
puede explicar la condición del lactante?
a) Páncreas anular
b) Atresia duodenal en la tercera porción
c) Atresia yeyunal
d) Hipertrofia del píloro

8. En ausencia o deficiencia de la secreción de la hormona motilina,


se producirá: a)sobrecrecimiento bacteriano
b) diarrea
c) aumento del vaciamiento gástrico
d) hipertrofia del píloro

9. La estimulación parasimpática aumenta la motilidad intestinal, mientras que la estimulación


simpática la disminuye. ¿Sobre cuál de las siguientes alternativas el sistema nervioso autónomo
actúa para el control de la motilidad intestinal?
a) Potencial de membrana en el plexo mientérico (de Auerbach)
b) Frecuencia de ondas lentas
c) Secreción de secretina
d) Nivel de IP3 en el plexo submucoso (de Meissner)

10. En un recién nacido con protrusión de contenidos abdominales y cubiertas por amnios o peritoneo,
es cierto que: a)Se presenta por un defecto en el cierre de la pared
b) Se acompaña de otras malformaciones congénitas
c) Se debe al no retorno de la hernia fisiológica
d) Se produce a través del ombligo

11. Lactante de 20 días con estreñimiento, distención abdominal progresiva, acompañada


ocasionalmente de vómitos biliosos. Como antecedente, el meconio lo eliminó por primera vez a las
72 horas de nacido. Su mamá menciona que ayuda a la evacuación con ayuda de un termómetro
rectal. Se sospecha de megacolon agangliónico (Enfermedad de Hirschsprung). ¿Cuál de las
siguientes alternativas explica el caso?
a) Se presenta contracciones tónicas en la región ano rectal
b) Se presenta dilatación de tracto gastrointestinal afectado
c) Las células ganglionares sólo han migrado al ano recto
d) La zona que más se afecta es inervada por fibras del nervio esplácnico menor

12. ¿Cuál de los siguientes reflejos disminuye el tránsito


gastrointestinal?
a)Doloroso
b) Gastrocólico
c) De defecación
d) Colicoileal

13. Al ingerir una sustancia ácida como el vino (pH 3), se estimula la motilidad gástrica por acción
de la hormona:
a)motilina
b) secretina
c) colecistoquinina (CCK)
d) bombesina
14. Paciente de 24 años acude a consulta externa por presentar una fístula oronasal (comunicación
entre la cavidad oral y la cavidad nasal). Está fístula está asociada al antecedente de haber sido
operada de paladar hendido a los dos años de edad, durante una campaña gratuita extranjera de
corrección de paladar fisurado. ¿Cuál de las arterias palatinas podría haberse lesionado durante
esa cirugía?
a) Mayor
b) Menor
c) Ascendente
d) Rama palatina de la faríngea ascendente

15. Paciente de sexo masculino de 52 años con úlcera péptica gástrica de 14 años de evolución, con
cuadro de hemorragia digestiva alta hace 4 meses, sin cicatrización de la úlcera. Entre las opciones
quirúrgicas se considera realizarle un vaguectomía troncal (sección del nervio vago) a nivel del
hiato esofágico. ¿Cuál de las siguientes complicaciones podría esperarse producto de la pérdida de
inervación parasimpática?
a) Menor inervación del colon ascendente
b) Se perderá el reflejo de defecación
c) Se perderá el reflejo de micción
d) Impotencia sexual

16. Paciente de 23 años con bulimia es traída a la emergencia deshidratada, semiconsciente y con
alcalosis metabólica. Los vómitos autoinfligidos por esta paciente se producen por estimulación
de receptores en la base de la lengua que mandan información directamente al:
a) núcleo del tracto solitario
b) centro del vómito en el tallo encefálico
c) zona quimiorreceptora gatillo
d) cerebelo

17. Niño de 3 años es traído a emergencia por madre quien manifiesta que hace 10 horas deglutió una
pila pequeña de reloj de bordes romos. El niño está asintomático. Usted la tranquiliza diciéndole es
un cuerpo extraño tan pequeño de seguro que va a seguir el tránsito intestinal como lo haría un
bolo alimenticio, y que lo más probable es que en ese momento se encuentre en:
a) colon
b) estómago
c) yeyuno
d) recto

18. Los movimientos en masa son un tipo de movimiento muy importante, una de las
consecuencias de estos movimientos es:
a) la distensión rectal
b) el peristaltismo del intestino delgado
c) la retropulsión gástrica
d) la contracción del esfínter anal interno

19. Durante la deglución, al momento que el bolo alimenticio pasa por el esfínter esofágico superior, se
espera que la presión intraesofágica:
a) disminuya en el cardias
b) disminuya en el tercio medio del esófago
c) aumente en la porción distal al bolo
d) aumente en el tercio medio del esófago

20. Paciente con enfermedad de Chagas que presenta disfagia a sólidos. ¿Cuál de las siguientes puede
ser la causa de esta complicación?
a) Disminución de células ganglionares en el esfínter esofágico inferior
b) Aumento en la liberación de óxido nítrico en el esfínter esofágico inferior
c) Disminución de las neuronas que liberan péptido intestinal vasoactivo
d) Aumento de la actividad de la motilina en el esófago distal
QUIZIZZ
1. Los vasos mesentéricos superiores se hallan a nivel de:
a) Cuello del páncreas

2. El nivel en el que se encuentra el píloro y el páncreas se puede determinar usando el


A) plano transpilórico

3. El dolor de estómago asociado a gastritis se suele ubicar en


a) epigastrio

4. La colecistoquinina inhibe el
a) vaciamiento gástrico

5. Paciente con disfasia ( dificultad para pasar alimentos) con to y disminución de peso. Con
antecedente de tabaquismo pesado. La sospecha es que tenga una disminución del diámetro
esófago a nivel de la
a) tercera estrechez

6. La digestión de las proteínas se inicia en:


a) estómago

7. El estómago recibe información simpática proveniente del:


a) ganglio celíaco

8. El reflujo gastroesofagico tiene múltiples etiologías, una de ellas tiene que ver con alteración a
nivel de:
a) primera estrechez
b) segunda estrechez
c) tercera estrechez
d) cuarta estrechez

9. La fístula retroperitoneal es causada por una falla en el desarrollo de:


a) tabique urorrectal

10. El nervio vago inerva el:


a) músculo estriado del esófago

11. El divertículo de Meckel es un rezago de:


a) conducto vitelino

12. La presencia de orina que sale por el ombligo de un recién nacido casa vez que llora, es posible
que se deba a un defecto en el desarrollo del
a) seno urogenital

13. La fístula retroperitoneal es causada por una falla en el desarrollo de


a) tabique urorrectal

14. Enfermedad asociada con un error en el desarrollo de las células de Cajal:


a) enfermedad de Hirschsprung

15. Aproximadamente en la semana 6 del desarrollo embrionario, el intestino medio gira 90


herniandose a nivel del
a) cordón umbilical

16. El conducto biliar deriva del


a) endodermo

17. Paciente mujer con 54 años con nauseas y vómitos y abdomen agudo quirúrgico, se ingresa a sala
de operaciones donde se encuentra vólvulo de ciego, esto se debe a
a) Falta de fusión del mesenterio

18. Paciente con cirrosis hepática con hipertensión portal, en el que es posible encontrar que los
vasos umbilicales están permeables dentro de
a) ligamento redondo

19. Es normal encontrar glándulas submucosas en CUAL


ES
a) esófago medio
b) esófago proximal
c) esófago distal
d) estómago

20. Cual de los siguientes órganos son intraperitoneales


a) estómago, vesícula biliar, íleon, hígado

Estudiante de medicina de la UPC de 21 años sufre de gastritis aguda ocasionada por comer en lugares
poco higiénicos. Suele consumir caramelos ( chupar ) mientras está en clase hasta la tarde. Toma gaseosas
regularmente (carbohidratos 46%, sodio 53%). También toma regular cantidad de leche (grasa 35%, lactosa
35%, proteínas 30%), pues le calma un poco el dolor el ardor que siente por la gastritis. Incluso, cuando
puede, se toma dos vasos de agua fría para calmar las molestias. Ha decidido ir al médico para tratarse pues
ya no soporta el dolor, el cual está seguro que los síntomas se deben a una elevada producción de ácido
clorhídrico en el estómago, y por ello le ha recetado Ranitidina (antihistamínico), con lo que siente mejoría.

● Para reducir la secreción de HCl en esta paciente se podría usar sustancias similares a:

- Péptido insulinotrópico dependiente de la glucosa (GIP)


● Si se usara atropina en esta paciente, se esperaría que disminuya la liberación de:
- Enzimas pancreáticas

● El consumir caramelos eleva los niveles en sangre de una hormona cuya función es
la estimulación de las células:
- Beta del páncreas

● En este paciente con gastritis aguda debida a una alta producción de ácido
clorhídrico, si se le hiciera un examen de sangre, se encontraría elevados los niveles
de:
- Colecistoquinina

● El consumo de una pequeña cantidad de gaseosa aumentará directamente la


concentración sérica de cuál de las siguientes hormonas:

- Péptido 1 similar al glucagón (GLP-1)

● El consumo rápido de 500 mL de gaseosa aumentará directamente la concentración


sérica de cuál de las siguientes hormonas:

- Gastrina

● Estimulan la secreción ácida gástrica

- Proteínas

● Con respecto a las ondas lentas, marque la afirmación correcta:}

- Son contracciones rítmicas espontáneas

● El uso de Ranitidina bloquea el receptor H2 de la histamina en las células


parietales. La histamina llega a estas células por:

- Difusión

● El consumir caramelos indirectamente activa la vía:

- POMC/CART

● ¿Cuál de los siguientes péptidos inhibe el vaciamiento gástrico?

- Colecistoquinina

● Para poder morder una manzana, es necesario usar el siguiente músculo:

- Milohiodeo

● El crecimiento de un adenocarcinoma de cuello de páncreas puede comprometer la


pared gástrica por continuidad. ¿Qué parte del estómago estaría comprometido con
mayor probabilidad?

- Pared posterior del antro

● El nacimiento de la arteria mesentérica superior se puede encontrar en cuál de los


cuadrantes abdominales:

- Epigastrio
● En cuanto a la colecistoquinina, marque la respuesta correcta:

- Potencia la acción de buffer con bicarbonato

● El aumento en la actividad motora de la pared gástrica genera un aumento en


los niveles locales de qué sustancia en la microvasculatura:

- Adenosina

Niño de sexo masculino de 2 años de edad, sufre de estreñimiento desde el nacimiento (1


deposición cada 3-4 días). Madre menciona que le estimula la defecación con un termómetro
rectal, y continuo uso de enemas y laxantes. Desde hace 6 meses comienza con vómitos
postprandiales. Los síntomas aumentan en frecuencia y magnitud y están en relación con
los episodios de estreñimiento. No refiere fiebre, tos, diarrea ni lesiones cutáneas. Al
examen físico presenta regular estado general, luce deshidratado. Abdomen distendido,
blando, depresible e indoloro. No se palpan masas abdominales. Se permeabiliza el canal
anal con termómetro rectal, encontrando cierta resistencia. Salida de material fecal mal
oliente en regular cantidad. Exámenes de laboratorio: hemograma normal. Signos
inflamatorios de fase aguda negativos. Alcalosis metabólica leve en sangre venosa.
Radiografía con enema baritado muestra recto y colon sigmoides dilatados (megacolon).
Biopsia profunda: ausencia de células ganglionares en la muestra enviada. Se realiza cirugía
correctiva.

● La percepción de la pirosis (sensación de dolor o quemazón en el esófago) asociado


al reflujo gastroesofágico, puede aparecer o exacerbarse debido a:

- Ejercicio

● Considerando que este paciente está sometido a estrés por el agravamiento de su


enfermedad, es posible afirmar que sus ondas lentas están:

- Hiperpolarizadas

● Debido al acúmulo de material fecal en todo el marco colónico, y a la irritación


química asociada, el peristaltismo del íleon distal se debe encontrar:

- Inhibido

● En cuanto a los reflejos gastrocólico y gastroduodenal en este paciente, indique lo


correcto:

- Se pueden considerar reflejos vago-vagales

● Con respecto a la defecación señale el enunciado correcto:

- Es estimulado por un llenado de la cuarta parte del volumen rectal

● El contenido fecal se detiene en la zona inmediatamente proximal a la zona donde


hay una menor presencia de:

- Péptido intestinal vasoactivo

● La presencia de atresias y estenosis duodenales se deben básicamente a una:

- Falta de recanalización

● El ligamento de Treitz característicamente:

- Suspende el ángulo de Treitz

● Al deglutir un bolo alimenticio, es lógico suponer que al pasar por el esófago haya
un mayor consumo de oxígeno en la pared del tercio:

- Proximal
● Estudiante de medicina de 20 años, se ha amanecido estudiando para su examen de
Sistema Digestivo. No ha probado alimento desde la cena, por lo que se puede
afirmar que la motilidad de esta persona está siendo regulada por:

- Motilina

● La hernia fisiológica se produce dentro de:

- Cordón umbilical

● El crecimiento de un adenocarcinoma de páncreas compromete la pared gástrica por


contigüidad. ¿Qué parte del estómago se esperaría esté comprometido?

- Pared posterior del antro

● Al comer unas papitas fritas con mayonesa, el vaciamiento gástrico disminuye por
efecto directo de la hormona:

- colecistoquinina (CCK)

● Paciente que come entera una pizza familiar de chorizo y queso. Es posible esperar
que debido a la cantidad de alimento ingerida, las ondas lentas hayan:

- Sufrido ninguna alteración en su frecuencia

● En este caso se puede afirmar con seguridad que se presenta:

- contracciones tónicas en la región ano rectal

● La forma más común de atresia esofágica contiene:

- Estenosis proximal del esófago más fístula traqueoesofágica distal

● Con respecto a la saliva, marque la respuesta correcta:

- el sistema simpático estimula su secreción

● Respecto a las enfermedades del esófago, marque lo correcto:

- el diagnóstico diferencial de la acalasia es la enfermedad de Chagas esofágica

● En relación a la fisilogía gástrica, marque lo correcto:

- la cimetidina actúa en la región basolateral de la célula parietal

● La célula mucosa del cuello gástrico produce:

- Moco

● La saliva puede tener una variedad de electrolitos en su composición. Entre ellos el cloro,
respecto al cual se puede afirmar:

- Su concentración no llega a ser tan alta como en el plasma

● Los músculos de la masticación que producen la retropulsión de la mandíbula son:

- temporales

● Respecto a las glándulas salivales, marque lo incorrecto:

- la glándula sublingual tiene forma de garfio

● Respecto a la anatomía del estómago, marque lo correcto:


- la arteria gástrica derecha nace de la arteria hepática común

● En cuanto a la saliva, marque lo correcto:

- La amilasa cumple función digestiva

● Durante el ataque con gas sarín (bloqueador de la acetilcolinesterasa) en el metro de


Tokio, en 1995, el personal de salud notó que los pacientes afectados presentaban:

- Hipersalivación

Paciente de 54 años con antecedentes de alcoholismo, gastritis crónica,


tabaquismo pesado, obesidad, cálculos biliares y cirrosis, es llevado a la
emergencia por dolor abdominal en epigastrio irradiado a la espalda y trastorno del
sensorio.
Al examen físico: presión arterial 85/50 mmHg, frecuencia cardíaca 100 latidos/min,
frecuencia respiratoria 18 x minuto, temperatura axilar 36°C.
Conjuntivas pálidas, escleras ictéricas nevus arácnidos en tronco, distensión
abdominal marcada, cabeza de medusa, matidez desplazable en ambos flancos e
hipogastrio, dolor a la palpación de abdomen.
Tiempo de protrombina: 24 seg (testigo: 13 seg); TPT: 38 seg, glicemia: 165 mg/dL,
uremia: 20 mg/dL, ASAT: 76 UI/L, ALAT: 22 UI/L, albumina: 2,5 g/dL, bilirrubina total:
2,6 mg/dL, bilirrubina directa: 1,4 mg/dL, amilasa sérica 4000 U/L.

● Un efecto secundario en el estómago por la acción de la secretina es:

- Menor actividad de la pepsina

● Considerando que el paciente sufre de gastritis, se puede decir que la secreción de


ácido por la mucosa gástrica:

- Involucra transporte activo de Hidrogeniones

● En cuanto a la gastritis de este paciente, se encontró que era producida por la


bacteria Helicobacter pylori. Esta bacteria sobrevive en el medio ácido del estómago
gracias a:

- Ureasa

● La bilirrubina directa aumentada en cirrosis hepática se excreta en la orina debido a:

- Ser hidrosoluble

● Paciente de 42 años con adenocarcinoma ductular. La TC ha demostrado claramente que el


tumor está en el cuello del páncreas y que hay un gran vaso ocluido. ¿Cuál de los siguientes
vasos estaría más probablemente obstruido?

- Vena porta.

● En relación a la histología hepática, marque lo correcto:


- La zona 3 se encuentra más cerca a la vena central lobulillar
Mujer de 83 años acude a emergencia por dolor abdominal desde hace 4
días, localizado en epigastrio, irradiado a ambos hipocondrios, nauseas,
vómitos y distensión abdominal; tiene antecedente de cardiopatía
hipertensiva, diabetes mellitus tipo II y fibrilación auricular. Refiere
deposiciones diarreicas muco sanguinolentas hace 1 día. Los exámenes
iniciales muestran PA: 110/60 mmHg, FC: 110/mn, leucocitos: 17800,
neutrófilos de 93%. TAC abdómino pélvica se observa oclusión completa
de arteria mesentérica superior por trombo asociado a placa ateromatosa.

● En esta paciente, ¿cuál de las siguientes sustancias no tendrá una


considerable disminución en su absorción? (marque la mejor respuesta):

- Calcio

● El mecanismo de la diarrea muco sanguinolenta que presenta la paciente, puede


mejor definido como de tipo (marque la mejor respuesta):}

- Exudativa

● Considerando que se ha comprometido el íleon distal, entre otras áreas, la atrofia o


descamación del epitelio de superficie explicaría cuál de los signos o síntomas de la
paciente (marque la mejor respuesta):

- Diarreas mucosanguinolentas

● Producto de esta isquemia, la expresión de cuál de las siguientes enzimas se vería


notablemente disminuida. Marque la mejor respuesta:

- Enteroquinasa

● Durante la cirugía, el cirujano observó que además la paciente tenía divertículos en


el sigma. Se sabe que estos divertículos:

- Se pueden asociar a estreñimiento crónico

1) La glandula parotida tiene principalmente acinos de tipo:


Tipo seroso
2) Presenta movimientos en masa:
Colon
3) A mayor flujo de saliva, disminuye la concentración de:
potasio
4) Es rama de la arteria mesentérica superior:
Arteria cólica media
5) En el intestino delgado se absorben los carbohidratos en forma de:
Fructosa
6) El dolor asociado a apendicitis clásicamente se ubica en:
Fosa iliaca derecho
7) La glandula submandibular recibe inervación traída por el nervio:
Cuerda del tímpano
8) La línea alba se encuentra:
Entre los rectos abdominales
9) la digestión de los carbohidratos se inician en:
Cavidad oral
10) Permite la suspensión e irrigación de los órganos peritoneales:
Mesenterio
11) Los nervios esplácnicos pélvicos (S2-S4) llevan información de tipo:
Parasimpática
12) Paciente con cirrosis hepática que tiene hipertensión portal con varices esofágicas y
actualmente presenta varices en estómago distal. Estas varices están relacionadas a
aumento en la presión de las venas:
Gástrica derecha
13) Es un órgano peritoneal:
Hígado
14) Al realizar un piercing en el ombligo, la sensación de dolor se transmite por:
T10
15) A mayor flujo de saliva disminuye la concentración de:
Potasio
16) La colecistoquinina inhibe:
El vaciamiento gástrico
17) El dolor del estómago asociado a gastritis se suele ubicar en:
Epigastrio
18) Es considerada una lesión preneoplásica:
Leucoplasia
19) Los sinusoides hepáticos son en realidad capilares de tipo:
Fenestrados
20) Paciente con cirrosis hepática y presión de vena cava de 15 mmHg, lo mas probable
es que el paciente presente:
Ascitis
21) La glándula submaxilar le hace un gancho al:
Músculo milohioideo
22) La glandula parotida esta inervada por el par craneal:
IX pc
23) Los nervios esplácnicos lumbares (L1-L2) llevan información de tipo:
Simpático
24) Los 9 cuadrantes del abdomen se delinean usando el plano subcostal, las líneas
medioclaviculares y:
Plano intertubercular
25) El estómago recibe información simpática proveniente del:
Ganglio celiaco
26) Es un órgano retroperitoneal:
Páncreas
27) La digestión de los lípidos se inicia en:
Intestino delgado
28) La digestión de las proteínas se inicia en:
Estómago
29) La arteria esplénica proviene de la aorta, y la vena esplénica desemboca en la vena:
Porta
30) Enfermedad asociado con un error en el desarrollo de las células de Cajal:
Enfermedad de Hirschsprung
31) El ácido acetilsalicílico actúa en la membrana:
Basolateral de la célula parietal
32) g
33) g
34) g
35) g
36) g
37) g
38) g
39) g
40) g
41) g
42) g
43) g
44) g
45) g
46) g
47) g
48) g
49) g
50) g
51) g
52) g
SISTEMA DIGESTIVO
EXAMEN PARCIAL
201802

Sección: Todas
Profesores: Choque Chávez, Fernando Diego; Damián Bastidas, Narda Lucía; Irribarren Gamarra, Maria Patricia;
Stapleton Valdivia, Mauricio Juan Jose.
Duración: 50 minutos.
Indicaciones:
- Lea atentamente cada pregunta antes de responder.
- Se prohíbe el uso del celular y cualquier dispositivo electrónico.
- Está prohibido intercambiar materiales.
- Coloque su código de alumno en la tarjeta de respuestas. Si su código contiene una letra reemplácela por un valor
numérico siguiendo la siguiente equivalencia: A=9, B=8, C=7, D=6, E=5.
- Traslade sus respuestas a la tarjeta, llenando los círculos de manera completa con lapicero negro o azul. Está
prohibido el llenado con lápiz, lapicero de otro color o con lapicero de tinta borrable.
- Sea cuidadoso en el llenado de la tarjeta de respuestas, pues solo esta tiene validez para la calificación.
- Al terminar su examen avise al docente a cargo, no se levante de su sitio; debe entregar la hoja de respuestas con
la carátula del examen, este cuadernillo de preguntas se lo llevará cada estudiante.

Chorrillos, octubre de 2018

1. En todo el sistema gastrointestinal se encuentra diferentes tipos de músculos ¿En cuales estructuras
encontraríamos músculo estriado?
A. Faringe, esfinter esofágico inferior, colon descendente.
B. Esfínter esofágico superior, laringe, esfínter anal externo.
C. Faringe, esfínter esofágico superior, esfínter anal externo.
D. Esfínter esofágico superior, esfínter esofágico inferior, colon.

2. Durante la fase …..……………….. el paladar blando es traccionado hacia arriba.


A. oral
B. gástrica
C. faríngea
D. esofágica

3. El tubo digestivo a nivel del ………………………….. está revestido internamente por epitelio ………….
A. Esófago / plano simple
B. Estómago / cilíndrico simple
C. Intestino grueso / plano estratificado
D. Intestino delgado / plano de transición

4. Señale la estructura del tubo digestivo que presenta tres capas musculares en su pared.
A. Esófago
B. Estómago
C. Intestino grueso
D. Intestino delgado

5. Paciente de 47 años con sobrepeso, acude a consulta por presentar dolor tipo cólico en el cuadrante superior
derecho del abdomen. El dolor aumenta luego de ingesta de comidas con abundante grasa. Esta situación se explica
por el aumento en la secreción de …………………………. que estimula la contracción de la vesícula biliar.
A. Gastrina
B. Colecistoquinina
C. Péptido inhibidor gástrico
D. Péptido intestinal vasoactivo
6. Si hay un aumenta de la hormona grelina a nivel del hipotálamo ¿Cuál es la consecuencia en el organismo?
A. Hambre
B. Saciedad
C. Aumento de somatostatina
D. Disminución del tránsito intestinal

7. Señale el péptido gastrointestinal que produce relajación del músculo liso gastro-intestinal.
A. Péptido intestinal vasoactivo (PIV)
B. Gastrina vasoactiva (GV)
C. Colecistoquinina (CCK).
D. Acetilcolina (Ach)

8. ¿Cuál de las siguientes alternativas inhibiría la relajación receptiva a nivel del estómago?
A. Histamina
B. Bloqueo del nervio vago (X)
C. Péptido inhibir gástrico (GIP)
D. Péptido intestinal vasoactivo (VIP)

9. Una mujer de 28 años, con diagnóstico de Diabetes Mellitus Tipo 1, acude por presentar desde hace 10 años
estreñimiento y distensión abdominal. Se realiza estudio y se determina que la paciente presenta un retraso del
vaciamiento gástrico debido a gastroparesia diabética. ¿Cuál de los siguientes hechos aumentaría el tiempo del
vaciamiento gástrico?
A. Aumento de gastrina
B. Estimulación parasimpática
C. Ácidos grasos en el duodeno
D. Quimo isotónico en el duodeno

10. Experimentalmente, se aplica un inhibidor selectivo del péptido intestinal vasoactivo (PIV) durante la contracción
peristáltica del intestino delgado. ¿Cuál es el efecto de este inhibidor en la motilidad del intestino delgado?
A. Parálisis del movimiento anterogrado
B. Disminución del tránsito intestinal
C. Aumento del tránsito intestinal
D. Movimiento retrogrado

11. ¿Cuál de los siguientes eventos ocurre durante la defecación?


A. Relajación del esfínter anal externo
B. Relajación del músculo liso del recto
C. Contracción del esfínter anal interno
D. Disminución de la presión intrabdominal

12. Señale el péptido que cumple la función de disminuir la ingesta de alimentos.


A. Neuropéptido Y
B. Ghrelina Gastrica
C. Proteína relacionada a agouti
D. Proopiomelanocortina (POMC)

13. Indique el lugar de secreción del péptido YY


A. Estómago
B. Duodeno
C. Yeyuno
D. Ileon

14. La grelina es secretada en el estómago y estimula a las neuronas del núcleo ………………….. para la estimulación de
la secreción de……………..
A. arqueado / neuropéptido Y.
B. arqueado / melanocortinas.
C. paraventricular / neuropéptido Y.
D. lateral del hipotálamo / hormonas orexigénicas.
15. Considerando el desarrollo embriológico del intestino medio. ¿Qué evento de gran importancia se produce en la
sexta semana?
A. Retracción de asas intestinales primitivas
B. Aparición del primordio hepático y pancreático
C. Inicio de la secreción de insulina por el páncreas
D. Salida temporal de asas intestinales a través de cordón umbilical

16. En el ……………………………. se presentan los movimiento de …………………….


A. esófago / retropropulsión y mezcla
B. estómago / segmentación y ondas lentas
C. intestino grueso / propulsión y ondas en espiga
D. intestino delgado / segmentación y peristaltismo

17. Marita sufre una parálisis del músculo masetero. ¿Qué limitaciones se producirán a nivel del movimiento de la
mandíbula?
A. Lateralización
B. Propulsión
C. Elevación
D. Ninguna

18. Señale el movimiento que se produce a nivel del ciego y colon proximal que tiene como finalidad favorecer la
absorción de agua y sales.
A. De masa
B. De mezcla
C. Propulsivo
D. Peristáltico

19. La enfermedad de Hirchsprung se caracteriza por una dilatación anormal de colon y disminución de los
movimientos propulsivos que traerá como principal consecuencia …………
A. diarrea acuosa.
B. estreñimiento crónico.
C. dilatación de la válvula ileocecal.
D. disminución de la flora intestinal.

20. ¿Cuál de los siguientes factores estimulan el vaciamiento gástrico?


A. Colecistoquinina (CCK)
B. Neuropéptido Y
C. Secretina
D. Gastrina

21. La secreción de ………………………. estimula la motilidad gástrica.


A. Colecistoquinina
B. Secretina
C. Gastrina
D. Motilina

22. La hormona ……………………….. es secretada por las células “I” del …………..
A. Colecistoquinina / duodeno y yeyuno
B. Gastrina / duodeno y yeyuno
C. Colecistoquinina / estómago
D. Gastrina / estómago
GLUT2, SGLT1, GLUT5

CI 1

1. La motilidad intestinal es estimulada principalmente por el:


- Sistema simpático
- Sistema piramidal
- Plexo de Auerbach
- Sistema parasimpático
2. La peristalsis o peristaltismo hace referencia a:
- Motilidad para movilizar el alimento de proximal a distal.
- No es parte de la motilidad
- Motilidad para mezclado de alimentos.
- Motilidad para fraccionamiento de alimentos.
3. Marque el órgano que se encuentra más distal en el tubo digestivo.
- Estomago
- Ciego
- Íleon
- Duodeno
4. Marque la respuesta incorrecta:
- La mucosa consta de epitelio, lámina propia y muscularis mucosae.
- En todo el tubo digestivo, se observa dos capas de muscular propia: circular
interna y longitudinal externa
- El colon contiene tenias
- Fuera de la cavidad abdominal, el esófago presenta capa adventicia.
5. Al iniciar la digestión, aumenta el consumo de oxígeno por la mucosa. Esto conlleva a
una hipoxia local, lo cual hace que se libere _____________, el cual produce
vasodilatación:
- Colecistoquinina
- Adenosina
- Histamina
- Noradrenalina
6. Cuál de las siguientes estructuras no tiene vasos sanguíneos:
- Epitelio intestinal
- Ligamento
- Omento
- Mesenterio
7. Paciente tiene una úlcera sangrante en el segundo tercio del Yeyuno. La arteria de la
cual proviene la sangre arterial para dicha zona es la arteria:
- Mesentérica superior
- Tronco celíaco
- Mesentérica inferior
- Iliaca común
- Gástrica izquierda
8. Paciente de 24 años con dolor abdominal tipo cólico intenso en mesogastrio. Según
sus conocimientos de macroestructura, el origen del dolor puede ser el ___________:
- Íleon
- Colon
- Estómago
- Esófago
9. Dentro de las funciones del abdomen, se encuentra la defecación y micción, en las
cuales la presión intra abdominal debe:
- Aumentar
- No tiene relación el abdomen con dichas funciones
- Mantenerse igual
- Disminuir
10. En la inspiración, la pared abdominal debe ____________ para ____________:
- Contraerse aumentar presión intra abdominal
- Relajarse disminuir presión intra torácica
- Relajarse aumentar presión intra abdominal
- Contraerse aumentar presión intra torácica

CI 2

1. Respecto a los péptidos gastrointestinales, marque lo correcto.


- No existe sustancia neurocrina que tenga efecto en la motilidad del tubo
digestivo
- Las sustancias paracrinas pueden viajar a través de vasos sanguíneos
- Las sustancias neurocrinas son peptidos que hacen su efecto en distancias
cortas
- Las sustancias paracrinas atraviesan la circulación portal
2. Al disminuir el pH duodenal por el HCl gástrico, se libera principalmente una hormona
cuya célula diana es:
- Acinos pancreáticos
- Células ductales del colédoco
- Célula ductal del Wirsung
- Células S del intestino
3. En un paciente con gastroparesia (motilidad lenta del estómago), que presenta
distensión abdominal después de comer, usted le recomendaría que evite el consumo
de lípidos y aminoácidos para disminuir la acción de:
- Secretina
- CCK
- Somatostatina
- Gastrina
4. La razón por la que el potencial de acción viaja rápidamente en sentido longitudinal
por el musculo liso gastrointestinal es la presencia de uniones en hendidura, además
de la presencia de:
- La presencia del plexo submucoso de Meissner
- Varicosidades
- Mayor cantidad de ACh
- Las fibras musculares no se disponen en haces musculares
5. Paciente obeso con Covid-19 es intubado por interno inexperto, quien al solicitar que
bombeen aire dentro del tubo endotraqueal, nota que el epigastrio se distiende. Al
sospechar que ha introducido el tubo en el estómago, también es cierto que:
- Disminuiría el tono del píloro
- Aumenta la frecuencia de ondas lentas
- Aumenta el pH gástrico
- Disminuye el pH gástrico
6. En un paciente con diarrea por hipermotilidad, usted sospecharía en el posible
aumento de las siguientes sustancias, excepto:
- Sustancia P
- ACh
- Péptido intestinal vasoactivo (VIP)
- Motilina
7. El ecografista sabe que para poder visualizar el nacimiento de la arteria mesentérica
superior, debe colocar el transductor sobre la piel de la siguiente región abdominal:
- Epigastrio
- Hipocondrio derecho
- Hipogastrio
- Mesogastrio
8. Al consumir un pan con mantequilla, la sensación de hambre disminuye debido a la
acción de:
- Grelina
- Somatostatina
- Colecistoquinina (CCK)
- Leptina
9. Una de las siguientes sustancias no comparte con las otras la misma acción sobre la
producción de ácido gástrico:
- Péptido insulinotrópico dependiente de glucosa (GIP)
- Colecistoquinina
- Somatostatina
- Secretina
10. Al ingerir grandes cantidades de dulces, con la subsecuente estimulación de incretinas,
usted esperaría que el apetito ______________, debido a __________________
- disminuya insulina
- aumente CCK
- disminuya CCK
- aumente grelina

PARCIAL

1. Al rozar agua caliente en la punta de la lengua, usted esperaría que el estímulo viaje a
través del nervio:
- lingual
- cuerda del tímpano
- glosofaríngeo
- hipogloso
2. Al ingresar líquidos o sólidos en la cavidad oral, un mecanismo que permite que una
persona respire mientras mastica es:
- el movimiento hacia afuera de los pliegues palatogloso y palatofaríngeo
- la elevación del paladar blando
- la depresión de la parte posterior de la lengua
- la depresión del paladar blando
3. Paciente adulto con reflujo gastroesofágico es más probable que presente:
- descalcificación del esmalte
- destrucción de ameloblastos
- remodelación del esmalte
- desfluorización de los dientes
4. La masticación es básicamente:
- importante para la digestión sobretodo de carnes
- un ralentizador del vaciamiento gástrico
- una actividad consciente
- un movimiento reflejo
5. Paciente de 34 años es víctima de asalto con arma de fuego, recibiendo un impacto
directo en el abdomen. En base a la radiografía, usted puede registrar en la historia
clínica que el proyectil se encuentra topográficamente en el:

- flanco derecho
- mesogastrio
- flanco izquierdo
- hipocondrio izquierdo
6. Señale cuál de las estructuras que en el embrión se encuentra comunicada con el saco
vitelino por medio del conducto onfalomesentérico:
- D
- B
- C
- A
7. En un paciente de 3 semanas de edad, con vómitos en proyectil, y nódulo epigástrico
reptante, usted esperaría encontrar:
- colecistoquinina aumentada
- vómitos biliosos e intolerancia a los ácidos grasos
- distensibilidad disminuida de la región oral del estómago
- engrosamiento de la circular interna pilórica
8. ¿Cuál de las siguientes estructuras tiene inervación somática?
- Estómago
- Peritoneo visceral
- Mesosigmoides
- Peritoneo parietal
9. Estas diseñando un proyecto de investigación sobre los niveles de colesterol que se
absorben luego de una comida grasosa y deseas cuantificar la cantidad de colesterol
que es absorbido por el intestino antes que el hígado lo metabolice ¿de cuál de los
siguientes vasos obtendrías la muestra para tu análisis?
- Vena porta
- Vena cava superior
- Conducto torácico
- Vena hemiácigos accesoria
10. Durante el paso del bolo hacia la orofaringe, se desencadena una serie de
contracciones musculares que estrechan la cavidad faríngea. Estas contracciones están
mediadas por el nervio craneal:
- X
- XI
- XII
- IX
11. Cuando el istmo de las fauces se cierra, se evita que el alimento pase hacia la
orofarínge y permite respirar mientras se mastica. Este cierre se debe a la contracción
y aproximación de los músculos:
- Palatoglosos
- Palatofaríngeos
- Estiloglosos
- Estilofaríngeos
12. Las siguientes alternativas son factores que determinan la patencia y función adecuada
del esfínter esofágico inferior, EXCEPTO:
- Plicatura diafragmática
- Hipertrofia de la circular interna
- Angulación con el estómago
- Canales lentos de calcio
13. Respecto al peristaltismo intestinal, para cumplir la ley del intestino , usted espera que
a nivel distal del quimo se libere:
- péptido liberador de gastrina (GRP)
- acetilcolina
- péptido intestinal vasoactivo
- sustancia P
14. El estímulo habitual para el movimiento peristáltico es:
- acción de la sustancia P
- contracción de la musculatura circular Interna
- estimulación vago-vagal
- distensión local
15. ¿Cuál de las siguientes condiciones considera que es un trastorno de la musculatura
lisa esofágica?
- Acalasia
- Asinergia faringoesfinteriana
- Hipotonía de los constrictores faríngeos
- Hipertonía del esfínter esofágico superior
16. ¿Cuál de las siguientes alternativas es correcta sobre la motilidad esofágica?
- Las ondas primarias son propulsoras y pueden no ser precedidas por deglución
- Las ondas primarias no son propulsoras y siempre van precedidas de deglución
- Las ondas secundarias son propulsoras y siempre van precedidas de deglución
- Las ondas secundarias son propulsoras y no van precedidas de deglución
17. El peristaltismo depende que a nivel distal del bolo se secrete:
- noradrenalina secretada por las fibras del sistema simpático
- acetilcolina por las neuronas provenientes del nervio vago
- óxido nítrico por células endoteliales locales
- péptido intestinal vasoactivo por neuronas
18. ¿Cuál de las siguientes alternativas es correcta sobre el movimiento peristáltico?
- Es un reflejo largo que depende de la integración con el tronco encefálico
- Se dirige en sentido distal siempre, nunca en sentido proximal
- El contenido intestinal avanza sólo 5-10 cm
- Es independiente del plexo mientérico
19. En un estudiante de medicina que está rindiendo un examen parcial, lo más probable
es que en ese momento su tránsito intestinal se encuentre:
- muy lento
- muy acelerado
- estimulado por acción de la sustancia P
- sin alteraciones
20. Las siguientes alternativas son ciertas sobre las contracciones tónicas del músculo
gastrointestinal, EXCEPTO:
- Tienen relación con el ingreso persistente de iones sodio
- Se encuentran principalmente en esfínteres
- Tienen regulación hormonal
- Obedece a una mayor frecuencia de potenciales en espiga
21. Con respecto de la regulación del pH del estómago; al utilizar un bloqueador de
histamina, usted espera que el pH del estómago:
- aumente
- disminuya
- se mantenga sin cambio
- se neutralice por acción de bicarbonato
22. Las siguientes hormonas disminuyen el vaciamiento gástrico, EXCEPTO:
- Gastrina
- Péptido insulinotrópico dependiente de glucosa
- Colecistoquinina
- Secretina
23. El frenillo de los labios se encuentra en:
- el piso de la boca
- la cavidad oral
- la cavidad vestibular
- el dorso de la lengua
24. En un paciente con shock hipovolémico, la peristalsis intestinal se encuentra:
- aumentada
- sin cambios
- invertida
- disminuida
25. Sobre el control de la peristalsis del tubo digestivo, ________________ es un
mediador neural que induce la relajación durante la peristalsis.
- la somatostatina
- el péptido intestinal vasoactivo
- la acetilcolina
- la serotonina

CI 3

1. En cuanto a las sustancias secretadas por el estómago ¿Cuál de las siguientes


sustancias estimula la liberación de pepsinógeno?
- Pepsinógeno
- Secretina
- Colecistoqunina
- Gastrina
2. Las células enteroendocrinas en el estómago se localizan en la glándula oxíntica, al
mismo nivel que las células:
- parietales
- principales
- mucosas
- absortivas
3. La fase intestinal de la secreción gástrica se debe básicamente a la participación de las
células:
- G del duodeno
- I del yeyuno
- S del íleon
- D del estómago
4. El conducto de Stenon, para entrar a la cavidad vestibular, debe atravesar el músculo:
- genihioideo
- buccinador
- milohioideo
- masetero
5. Paciente con acalasia es sometido a tratamiento endoscópico o quirúrgico, usted le ha
informado al paciente previamente que es posible que una complicación de este
tratamiento es que quede con cierto grado de:
- reflujo gastroesofágico
- gastritis
- odinofagia
- úlceras gástricas
6. La sangre que lleva la vena porta es tipo:
- mixta
- arterial
- venosa
7. Los pliegues gástricos gruesos son prácticamente inexistentes a nivel de:
- el cuerpo
- el fondo
- la incisura angularis
- el antro
8. En la producción de HCl, la acción de la somatostatina disminuye la accion de:
- las prostaglandinas
- la histamina
- la acetilcolina
- la gastrina
9. En el estómago se secretan las siguientes sustancias, EXCEPTO:
- grelina
- Correcto
- motilina
- somatostatina
- gastrina
10. La vena porta se forma gracias a la unión de la vena mesentérica superior con la vena:
- mesentérica inferior
- esplénica
- celiaca
- gástrica izquierda

CI 4

1. En el hígado, el aumento de la resistencia vascular en los sinusoides hepáticos


ocasionará:
- aumento de la presión de llenado vesicular
- salida de plasma hacia el intersticio
- aumento del flujo hacia la vena porta
- aumento del flujo hacia la vena cava superior
2. En un paciente con carcinoma de páncreas, el tumor ha invadido la unión entre la
venas esplénica y mesentérica superior; eso quiere decir que estamos seguros que el
tumor se encuentra a nivel del ________ del páncreas.
- cuerpo
- cola
- cabeza
- cuello
3. En un paciente con cirrosis hepática la cabeza de medusa que aparece en la pared
abdominal, podría desaparecer si al paciente se le:
- esclerosa las venas hemorroides internas
- oblitera el ligamento redondo
- administra antiandrógenos
- oblitera la arteria gástrica izquierda
4. En un paciente con intoxicación por órganos fosforados, la acción de la
colecistoquinina (CCK) está bloqueada a nivel de:
- el esfínter de Oddi
- la vesícula biliar
- la célula parietal
- el sistema nervioso central
5. Con respecto a la microestructura del hígado ¿Cuál de las siguientes alternativas es
correcta?
- La célula de Ito se encuentra en el espacio de Disse y reserva glucógeno
- Los hepatocitos están interconectados por uniones herméticas
- La célula de Kupffer se encuentra fuera del sinusoide y fagocita células
- El sinusoide es un capilar fenestrado
6. Paciente con Lupus Eritematosos que desarrolla hipertensión portal debido a
trombosis portal, es probable que desarrolle várices a nivel de:
- recto inferior
- recto superior
- hemorroides externas
- canal anal
7. El efecto de un medicamento colerético se evidencia por:
- la mayor producción de colesterol en la bilis
- la disminución de absorción de sales biliares
- el aumento de secreción biliar
- el aumento de formación de micelas
8. En un paciente con cáncer de páncreas y que desarrolla ictericia, la localización más
probable del tumor es en:
- la vesícula por metástasis
- el cuello del páncreas
- la cabeza del páncreas
- la cola del páncreas
9. En un recién nacido menor de 24 horas con atresia biliar, se encuentra elevación de la:
- bilirrubina directa
- bilirrubina indirecta
- alanina aminotransferasa (ALT)
- hemoglobina
10. ¿Cuál de las siguientes alternativas es correcta sobre la estructura hepática?
- Los colangiocitos producen bilis
- En la triada portal, se encuentra la vena derivada de la suprahepática
- El flujo sinusoidal en el lobulillo hepático es de adentro hacia afuera
- El flujo biliar en el lobulillo hepático es centrífugo
EXAMEN PARCIAL 2020-02

Pregunta 1

Niña de 6 años se asusta por que se le ha aflojado un diente deciduo. Este fenómenose produce
por:

laxitud del ligamento periodóntico

fractura del cemento

desmineralización del esmalte dental

aumento anómalo de la predentina

2.- ¿Cuál de las siguientes estructuras deriva del intestino anterior?


B

3.- En un paciente de 3 semanas de edad, con vómitos en proyectil, y nódulo epigástrico reptante,
usted esperaría encontrar:

engrosamiento de la circular interna pilórica

distensibilidad disminuida de la región oral del estómago

colecistoquinina aumentada

vómitos biliosos e intolerancia a los ácidos grasos

4.- Sobre el control de la peristalsis del tubo digestivo, ________________ es un mediador neural
que induce la relajación durante la peristalsis.

el péptido intestinal vasoactivo

la serotonina

la acetilcolina

la somatostatina

5.- En el plexo mientérico, el origen de los impulsos eferentes está en:

los ganglios paravertebrales

las células intersticiales de Cajal

el plexo de Meissner

el plexo de Aurbach
6.- Un familiar le comenta que tiene úlcera gástrica por exceso de producción de ácido; con sus
conocimientos del sistema digestivo, usted le recomendaría que reduzca el consumo de:

vitaminas

carbohidratos

agua

aminoácidos

7.- Las siguientes alternativas son ciertas en relación al mesenterio, EXCEPTO:

El omento menor deriva del mesenterio ventral

El ligamento esplenorrenal une al bazo con el riñón izquierdo

Los mesenterios cumplen la función de sostén y suspensión de órganos

Los omentos cumplen una función principal de irrigación visceral

8.- ¿Cuál de las siguientes alternativas es una característica de la estructura del esófago?

Aumenta la presión intra esofágica durante la inspiración

Contiene músculo estriado en casi toda su longitud

Contiene glándulas submucosas principalmente en su tercio distal

El esfínter esofágico inferior es un esfínter anatómico

9.- En una cirugía abierta (laparotomía), el cirujano al abrir la cavidad peritoneal por la parte anterior
(línea media), lo primero que observa es:

Estomago

Colon sigmoides

Duodeno

Epiplón mayor
10.- El tubo digestivo posee glándulas, las glándulas submucosas se encuentran en el:

duodeno y recto

íleon y esófago

estómago y duodeno

esófago y duodeno

11.- ¿Cuál de las siguientes alternativas detallan los músculos que ayudan a empujar el bolo hacia
la orofaringe?

Geniogloso y palatofaríngeo

Estilogloso y palatogloso

Hiogloso y geniogloso

Estilogloso y geniogloso

12.- Al ingresar líquidos o sólidos en la cavidad oral, un mecanismo que permite que una persona
respire mientras mastica es:

la elevación del paladar blando

el movimiento hacia afuera de los pliegues palatogloso y palatofaríngeo

la depresión de la parte posterior de la lengua

la depresión del paladar blando

13.- Durante el paso del bolo hacia la orofaringe, se desencadena una serie de contracciones
musculares que estrechan la cavidad faríngea. Estas contracciones están mediadas por el nervio
craneal:

IX

XI

XII
14.- Las siguientes alternativas son factores que determinan la patencia y función adecuada del
esfínter esofágico inferior, EXCEPTO:

Canales lentos de calcio

Plicatura diafragmática

Hipertrofia de la circular interna

Angulación con el estómago

15.- Durante una cena, una gestante inspira por la boca profundamente de manera frecuente; sin
embargo, el organismo evita que el aire ingrese al esófago por la acción:

del músculo cricofaríngeo

del istmo de las fauces

del esfínter esofágico inferior

de la epiglotis

16.- En un estudiante de medicina que está rindiendo un examen parcial, lo más probable es que en
ese momento su tránsito intestinal se encuentre:

muy lento

estimulado por acción de la sustancia P

muy acelerado

sin alteraciones

17.- El principal gobernante sobre todos los movimientos gastrointestinales es el sistema nervioso:
parasimpático

somático

mientérico

simpático
18.- ¿Cuál de las siguientes condiciones considera que es un trastorno de la musculatura lisa
esofágica?

Hipotonía de los constrictores faríngeos

Hipertonía del esfínter esofágico superior

Asinergia faringoesfinteriana

Acalasia

19.- El estímulo habitual para el movimiento peristáltico es:


acción de la sustancia P

contracción de la musculatura circular Interna

estimulación vago-vagal

distensión local

20.- La hormona responsable de los complejos migratorios interdigestivos tiene las siguientes
características, EXCEPTO:

se libera de forma cíclica

se produce en el estómago y el duodeno

cumple funciones de aumentar la motilidad y secreción gástrica e intestinal

es inhibida por el alimento

21.- Respecto al peristaltismo intestinal, para cumplir la ley del intestino , usted espera que a nivel
distal del quimo se libere:

acetilcolina

péptido liberador de gastrina (GRP)

sustancia P

péptido intestinal vasoactivo


22.- ¿Cuál de las siguientes alternativas es correcta sobre el control autónomo del aparato
gastrointestinal?

El plexo mientérico de Auerbach cumple funciones inhibitorias

Las terminaciones nerviosas simpáticas liberan mayor cantidad de adrenalina que noradrenalina

La estimulación simpática estimula a la muscularis mucosae

Al seccionar el vago, la inervación parasimpática del colon sigmoides disminuye

23.- La distención del yeyuno provoca que se:

despolarice el potencial de reposo de membrana

produzca una contracción tónica

disminuya el número de espigas

aumente la frecuencia de las ondas lentas

24.- El frenillo de los labios se encuentra en:

el dorso de la lengua

el piso de la boca

la cavidad oral

la cavidad vestibular

25.- Con seguridad, usted puede decir que la siguiente cicatriz postapendicectomía se encuentra en
la región denominada:

flanco derecho

hipocondrio derecho

flanco izquierdo

fosa iliaca derecha


Banqueo Digestivo

1. Dentro de las funciones del abdomen, se encuentra la defecació n y micció n, en las cuales la
presió n intra abdominal debe:
Aumentar

2. Paciente de 24 añ os con dolor abdominal tipo có lico intenso en mesogastrio. Segú n sus
conocimientos de macroestructura, el origen del dolor puede ser el ___________ :
Ileon

3. En la evaluació n de una tomografía abdominal, el interno observa un aneurisma en una arteria


que se dirige al riñ ó n derecho. Con seguridad se puede afirmar que está a nivel de la vé rtebra:
L1

4. Es inervado por aferentes somaticas


Peritoneo parietal

5. Paciente se queja de dolor en hipocondrio derecho, pero superficialmente. El dermatoma


relacionado es (marque la mejor respuesta):
T9

6. Al iniciar la digestió n, aumenta el consumo de oxígeno por la mucosa. Esto conlleva a una
hipoxia local, lo cual hace que se libere _____________, el cual produce vasodilatació n:
Adenosina

7. Respecto a la anatomía del estó mago, marque lo correcto:


El fondo gá strico forma la curvatura mayor

8. La motilidad intestinal es estimulada principalmente por el:


Plexo de Auerbach

9. Paciente con vó lvulo del colon sigmoides. La necrosis de este segmento del colon se produce
por una alteració n en la irrigació n de la arteria:
Mesenterica inferior

10. Marque la respuesta incorrecta


En todo el tubo digestivo, se observa dos capas de muscular propia: circular interna y
longitudinal externa

11. Al disminuir el pH duodenal por el HCl gá strico, se libera principalmente una hormona cuya
cé lula diana es:
Cé lula ductal del Wirsung

12. Al consumir un pan con mantequilla, la sensació n de hambre disminuye debido a la acció n de:
Colecistoquinina (CCK)
13. La rotació n en sentido longitudinal del estó mago en el desarrollo embrioló gico condiciona que
el nervio vago derecho quede a nivel:
Posterior

14. Una de las siguientes sustancias reguladoras, puede actuar de forma paracrina y como
hormona. Marque la correcta:
Somatostatina

15. De los diferentes reflejos gastrointestinales, hay uno que produce movimiento del contenido
hacia la regió n distal, y se llama reflejo:
Gastrocó lico

16. Todos los mú sculos motores de la lengua está n inervados por el XII par, excepto:
palatogloso

17. Al ingerir rá pidamente un litro de agua, usted esperaría que la gastrina aumente por efecto de:
ACh del sistema mienté rico

18. En un paciente con hiperestimulació n simpá tica se espera que las ondas lentas tengan un
ritmo:
Menor en íleon terminal que en el duodeno

19. Paciente con apendicitis aguda, que debuta con dolor en mesogastrio. Este dolor se debe a
estimulació n de receptores del dolor cuyas fibras van a viajar a la mé dula espinal a travé s de:
Nervios simpá ticos

20. Paciente obeso con Covid-19 es intubado por interno inexperto, quien al solicitar que bombeen
aire dentro del tubo endotraqueal, nota que el epigastrio se distiende. Al sospechar que ha
introducido el tubo en el estó mago, tambié n es cierto que:
Disminuye el pH gá strico

21. Cuando una persona coloca en su boca una sustancia con alta concentració n de carbohidratos,
lo que debería pasar es que:
disminuya la sensació n de hambre

22. El pH ó ptimo para la digestió n a nivel duodenal es ____________ y está regulado principalmente
por la liberació n de _________________
6-8 / secretina

23. Los corpú sculos gustativos se encuentran en la lengua, pero ademá s se les puede encontrar en:
el paladar blando

24. Al ingresar líquidos o só lidos en la cavidad oral, un mecanismo que permite que una persona
respire mientras mastica es:
la depresió n del paladar blando

25. Al rozar agua caliente en la punta de la lengua, usted esperaría que el estímulo viaje a travé s del
nervio:
lingual

26. Sobre el control de la peristalsis del tubo digestivo, ________________ es un mediador neural que
induce la relajació n durante la peristalsis.
el pé ptido intestinal vasoactivo
27. ¿Cuá l de las siguientes alternativas es correcta sobre el control del peristaltismo?
El peristaltismo intestinal aumenta por efecto de la colecistoquinina (CCK)

28. ¿Cuá l de las siguientes alternativas es una característica de la estructura del esó fago?
Contiene glá ndulas submucosas principalmente en su tercio distal

29. En la estructura dentaria, se observa que hay una composició n muy similar a la del hueso en la
capa denominada:
cemento

30. El tubo digestivo posee glá ndulas, las glá ndulas submucosas se encuentran en el:
esó fago y duodeno

31. Estas diseñ ando un proyecto de investigació n sobre los niveles de colesterol que se absorben
luego de una comida grasosa y deseas cuantificar la cantidad de colesterol que es absorbido por
el intestino antes que el hígado lo metabolice ¿de cuá l de los siguientes vasos obtendrías la
muestra para tu aná lisis?
Conducto torá cico

32. Mujer de 30 añ os llega a emergencia con dolor en hipogá strico. Al examen físico presenta una
masa palpable de 10 cm de diá metro a en la misma regió n, usted sospecharía de las siguientes
condiciones, EXCEPTO:
Tumor renal

33. Las siguientes alternativas son factores que determinan la patencia y funció n adecuada del
esfínter esofá gico inferior, EXCEPTO:
Hipertrofia de la circular interna

34. Cuando el istmo de las fauces se cierra, se evita que el alimento pase hacia la orofarínge y
permite respirar mientras se mastica. Este cierre se debe a la contracció n y aproximació n de los
mú sculos:
palatoglosos

35. Las siguientes alternativas son ciertas sobre la actividad elé ctrica del mú sculo gastrointestinal,
EXCEPTO:
La despolarizació n lenta se debe principalmente al ingreso de Na+

36. ¿Cuá l de las siguientes alternativas es correcta sobre el movimiento peristá ltico?
El contenido intestinal avanza só lo 5-10 cm

37. La informació n sensitiva aferente del sistema gastrointestinal pasa por las siguientes
estructuras, EXCEPTO:
Tá lamo

38. Con respecto a la regulació n del peristaltismo, al aplicarle atropina (antagonista coliné rgico) a
un paciente, es de esperarse que el peristaltismo:
disminuya

39. ¿Cuá l de las siguientes alternativas es correcta sobre el control autó nomo del aparato
gastrointestinal?
La estimulació n simpá tica estimula a la muscularis mucosae
40. Respecto al peristaltismo intestinal, para cumplir la ley del intestino , usted espera que a nivel
distal del quimo se libere:
pé ptido intestinal vasoactivo

41. ¿Cuá l de las siguientes alternativas estimula las ondas de motilidad gastrointestinal
denominadas complejos migratorios interdigestivos?
Eritromicina

42. Las siguientes alternativas son ciertas sobre las contracciones tó nicas del mú sculo
gastrointestinal, EXCEPTO:
Tienen relació n con el ingreso persistente de iones sodio

43. El peristaltismo depende que a nivel distal del bolo se secrete:


pé ptido intestinal vasoactivo por neuronas

44. Señ ale cuá l de las estructuras que en el embrió n se encuentra comunicada con el saco vitelino
por medio del conducto onfalomesenté rico:
A
45. Es un derivado del mesenterio dorsal:
D

46. El ecografista sabe que pude visualizar el nacimiento de la arteria


mesenterica superior, debe colocar el transductor sobre la piel de la sgte region abdominal:
Epigastrio

47. Durante una cirugia oncologica, el cirujano observa que los organos abdominales tienen libre
movimiento dentro de la cavidad abdominal, excepto:
Colon ascendente

48. Al introducir una solucion azucarada directamente al estomago mediante una gastrometria
(comunicación entre la piel abdominal y el estomago), la sustancia que provocara que
aumente los niveles sericos de insulina es:
Peptido tipo glucagon (GLP-1)

49. Al ingerir grandes cantidades de dulces, con la subsecuente estimulacion de incretinas, usted
esperaria que el apetito ___________ debido a ___________
Disminuya / insulina

50. En un paciente con hiperestimulacion simpatica se espera que las ondas lentas tengan un
ritmo:
Menor en ileon terminal que en el duodeno

51. En un paciente con schok distributivo, usted decide iniciar noradrenalina por un cateter CVC,
consiguiendo aumentar la PA. ¿Qué efecto sobre la motilidad intestinal espera encontrar?
El potencial de reposo de las fibras musculares se hace mas negativo

52. La hormona que tiene un efecto sinergico con la secretina para optimizar el pH duodenal y la
digestion es:
CCK

53. Paciente joven que es traido a emergencia con abdomen agudo quirurgico debido a herida
contusopenetrante por verduguillo (alambre grueso con punta aguzada) recibida en una
pelea despues de un partido de futbol. Se observa herida en hipocondrio izquierdo. El organo
que debe estar sangrando y produciendo hemoperitoneo es:
Bazo

54. Paciente se queja de dolor en hipocondrio derecho, pero superficialmente. El dermatoma


relacionado es:
T9
55. Paciente de 24 años con dolor abdominal tipo colico intenso en mesogastrio. Según sus
conocimientos de macroestructura, el origen del dolor puede ser el _____
Ileon

56. Paciente tiene una ulcera sangrante en el segundo tercio del yeyuno. La arteria de la cual
proviene la sangre arterial para dicha zona es la art.:
Mesenterica superior

57. Durante el vomito, ¿el contenido gastrico tiene que pasar necesariamente por cual estructura
para llegar al esofago?
Cardias

58. La peristalsis o peristaltismo hace referencia a:


Motilidad para movilizar el alimento proximal a distal

59. Respecto a la anatomia del estomago, marque lo correcto:


El fondo gastrico forma la curvatura mayor

60. Al iniciar la digestion, aumenta el consumo del oxigeno por la mucosa. Esto conlleva a una
hipoxis local, lo cual hace que se libere ______, el cual produce:
Adenosina

61. La motilidad intestinal es estimulada parcialmente por el:


Plexo de Auerbach

62. En un paciente con falla en la fusion de los conductos de las yema ventral y dorsal del
pancreas, usted esperaria encontrar:
Drenaje de la mayor parte del jugo pancreatico en la papila menor

63. ¿Cuál de los sgtes alternaticas detallan los músculos que ayudan a empujar el bolo hacia la
orofaringe?
Estilogloso y palatogloso
64. En el esofago, el plexo mienterico o de Auerbach interviene en el _________ de los impulsos
_________
Fin / eferentes

65. Sobre el control de la peristalsis del tubo digestivo, ____________ es un indicador neural que
induce la relajacion durante la peristalsis.
El peptido intestinal vasoactivo

66. Las sgtes alternativas son correctas sobre la deglucion, EXCEPTO:


Consta de 2 fases

67. Paciente adulto con reflujo gastroesofagico es mas probable que presente:
Descalcificacion del esmalte

68. En una cirugia abierta (laparotomia), el cirujano al abrir la cavidad peritoneal por la parte
anterior (linea media), lo primero que observa es:
Epiplon mayor

69. Durante la deglucion, el bolo es impedido de ingresar a la cavidad oral gracias a la accion de
diversos músculos, entre ellos el musculo:
Palatofaringeo

70. Durante una cena, una gestante inspira por la boca profundamente de manera frecuente, sin
embargo, el organismo evita que el aire ingrese al esofago por la accion:
Del musculo cricofaringeo

71. Las sgtes alternativas son factores que determinan la patencia y funcion adecuada del esfinter
esofagico inferior, EXCEPTO:
Hipertrofia de la circular interna

72. La distencion del yeyuno provoca que se:


Despolarice el potencial de reposo de la membrana

73. El estimulo habitual para el movimiento peristaltico es:


Distencion local
74. La hormona responsable de los complejos migratorios interdigestivos tiene las siguientes
caracteristicas, EXCEPTO:
Cumple funciones de aumentar la motilidad y secrecion gastrica e intestinal

75. Con respecto a la regulación del peristaltismo, al aplicarle atropina (antagonista colinergico) a
un paciente, es de esperarse que el peristaltismo:
Disminuya
76. ¿Cuá l de las siguientes alternativas es correcta sobre la motilidad esofagica?
Las ondas secundarias son propulsoras y no van precedidas de deglucion

77. El principal gobernante sobre todos los movimientos gastrointestinales en el sistema


nervioso:
Mienterico

78. El peristaltismo intestinal se produce gracias a un reflejo que:


Se origina dentro de la pared intestinal

79. Un familiar le comenta que tiene ulcera gastrica por exceso de produccion de acido, con sus
conocimientos del sistema disgestivo, usted le recomendaria que reduzca el consumo de:
Aminoácidos

80. Un paciente con polimiositis posee alteración en la regulación del mecanismo de la deglución;
por eso hay que considerar la deglución de la saliva, pues a diario se produce _______mL
1000

81. En la producción de HCl, la acción de la somatostatina disminuye la accion de:


La gastrina

82. Al usar un parasimpaticomimético (agonista colinérgico), usted espera que la saliva presente:
Una mayor cantidad de sodio

83. Al evaluar a un paciente con parotiditis purulenta (con absceso), usted buscaría el orificio
terminal del conducto de _____________ a la altura de _____________
Stenon / la segunda molar superior

84. En cuanto a las sustancias secretadas por el estómago ¿Cuál de las siguientes sustancias
estimula la liberación de pepsinógeno?
Secretina

85. Para determinar que un paciente tiene esófago de Barrett, debemos encontrar ___________ en la
biopsia de esófago.
Células caliciformes

86. Al usar atropina en un paciente, usted esperaría:


la disminución de secreción gástrica por bloqueo de M3

87. El conducto de Stenon, para entrar a la cavidad vestibular, debe atravesar el músculo:
Buccinador

88. Las células enteroendocrinas en el estómago se localizan en la glándula oxíntica, al mismo


nivel que las células:
Principales
89. La inhibición de secreción gástrica es secundaria a:
El reflejo enterogastrico

90. En un paciente con infección por SARS-CoV-2 con compromiso severo e ingresado en la
unidad de cuidados intensivos (UCI), debido a la tormenta de citoquinas y a la desregulación
inmune ¿Cuál de las siguientes alternativas estará elevada en sangre al evaluar el perfil
hepático?
Transaminasas

91. Un recién nacido con enfermedad congénita, tiene una mutación de la proteína MRP2
encargada del transporte de la bilirrubina conjugada hacia el interior del canalículo. Antes de
las 24 horas de nacido presenta ictericia y además presentará con mayor probabilidad:
coluria

92. El efecto de un medicamento colerético se evidencia por:


el aumento de secreción biliar

1. En un paciente cirrótico con encefalopatía hepática ¿cuál de las siguientes alternativas sustenta
la reducción de carnes rojas en la dieta?
El amonio se produce principalmente en el intestino

2. En un paciente con cirrosis hepática la cabeza de medusa que aparece en la pared abdominal,
podría desaparecer si al paciente se le:
oblitera el ligamento redondo

3. En un paciente con cirrosis hepática se desarrollará ascitis debido a los siguientes


mecanismos, EXCEPTO:
Estrechamiento de la porta

4. Para poder absorber al torrente sanguíneo todos los carbohidratos de la lactosa, es necesario
usar únicamente los transportadores:
SGLT1, GLUT2

5. ¿Cuál de las siguientes alternativas es una comunicación entre el tejido hepático y la vesícula
biliar?
Conducto de Luschka

6. La bilis que sale de la vesícula biliar tiene como componente principal:


Acidos biliares

7. La relajación del esfínter de Oddi se produce directamente por acción de:


Peptido vasoactivo intestinal (VIP)

8. En un paciente con xerostomia presenta las sgtes condiciones, EXCEPTO:


Infecciones del oido a repeticion

9. Los pliegues gastricos gruesos son practicamente inexistente a nivel de:


Fonfo

10. La vena porta de forma gracias a la union de la vena mesenterica superior con la vena:
Esplenica
11. En caso se produzca la perforacion de la cara anteiror del estomago, esta perforacion
ocasionara _____ producto de la peritonitis quimica.
Ileo intestinal

12. Cuando un paciente recibe estimulo autonomo mixto (simpatico y parasimpatico), el flujo de
saliva:
Aumenta en relacion basal

13. En una gestante de 11 semanas con antecedentes de esofagitis eosinofilica, con hiperemesis
gravidica, que acude a emergencia por hematemesis leve, y presenta subitamente disnea y
dolor toracico. Usted sospecharia de:
Síndrome de Boerhaave

14. Sangre q lleva la vena porta es tipo:


Venosa

15. Aquellas cells q producen mayor cantidad de moco en el epitelio gastrico son las:
Mucosas superficiales

16. Dentro de las patologias q producen sangrado en el esofago, la que sangra mas es:
Síndrome de Mallory Weiss

17. En las glandulas salivales, el principal lugar de intercambio ionico se da a nivel del:
Conducto estriado

18. Via biliar extrahepatica se encuentra ubicada dentro del ligamento:


Hepatoduodenal

19. En un paicente con intoxicacion de organso fosforados, la accion de la colecistoquinina (CCK)


esta bloqueada a nivel de:
Esfinter de Oddi
93. En un paciente con carcinoma de pancreas, el tumor ha invadido la union entre la vena
esplenica y mesenterica superior, eso quiere decir que estamos seguros q el tumor se
encuentra a nivel del _____ del pancreas.
Cuello

94. ¿Cuál de las sgtes alternaticas es correcta sobre la estructura hepatica?


El flujo biliar en el lobulillo hepatico es centrifugo

95. En una paciente con una estenosis severe del coledoco por una complicacion quirurgica,
usted esperaria q desarrolle:
Aumento del tiempo de protrombina

96. Los hepatocitos tienen gran capacidad regenerativa, en parte gracias a las cells madre
hepaticas q se localizan en los:
Canales de Herning
97. Paciente con Lupus eritematosos q desarrolla hipertension portal debido a trombosis portal,
es probable que desarrolle varices a nivel de:
Recto superior

1. En todo el sistema gastrointestinal se encuentra diferentes tipos de músculos ¿En cuales estructuras
encontraríamos músculo estriado?
Faringe, esfínter esofágico superior, esfínter anal externo.

2. Durante la fase …..……………….. el paladar blando es traccionado hacia arriba.


faríngea

3. El tubo digestivo a nivel del ………………………….. está revestido internamente por epitelio ………….
Estómago / cilíndrico simple

4. Señale la estructura del tubo digestivo que presenta tres capas musculares en su pared.
Estómago

5. Paciente de 47 años con sobrepeso, acude a consulta por presentar dolor tipo cólico en el cuadrante
superior derecho del abdomen. El dolor aumenta luego de ingesta de comidas con abundante grasa.
Esta situación se explica por el aumento en la secreción de …………………………. que estimula la
contracción de la vesícula biliar.
Colecistoquinina

6. Si hay un aumenta de la hormona grelina a nivel del hipotálamo ¿Cuál es la consecuencia en el


organismo?
Hambre

7. Señale el péptido gastrointestinal que produce relajación del músculo liso gastro-intestinal.
Péptido intestinal vasoactivo (PIV)

8. ¿Cuál de las siguientes alternativas inhibiría la relajación receptiva a nivel del estómago?
Bloqueo del nervio vago (X)

9. Una mujer de 28 años, con diagnóstico de Diabetes Mellitus Tipo 1, acude por presentar desde hace
10 años estreñimiento y distensión abdominal. Se realiza estudio y se determina que la paciente
presenta un retraso del vaciamiento gástrico debido a gastroparesia diabética. ¿Cuál de los
siguientes hechos aumentaría el tiempo del vaciamiento gástrico?
Ácidos grasos en el duodeno

10. Experimentalmente, se aplica un inhibidor selectivo del péptido intestinal vasoactivo (PIV) durante
la contracción peristáltica del intestino delgado. ¿Cuál es el efecto de este inhibidor en la motilidad
del intestino delgado?
Disminución del tránsito intestinal
11. ¿Cuál de los siguientes eventos ocurre durante la defecación?
Relajación del esfínter anal externo

12. Señale el péptido que cumple la función de disminuir la ingesta de alimentos.


Proopiomelanocortina (POMC)

13. Indique el lugar de secreción del péptido YY


Estómago

14. La grelina es secretada en el estómago y estimula a las neuronas del núcleo ………………….. para la
estimulación de la secreción de……………..
arqueado / neuropéptido Y.

15. Considerando el desarrollo embriológico del intestino medio. ¿Qué evento de gran importancia se
produce en la sexta semana?
Salida temporal de asas intestinales a través de cordón umbilical

16. En el ……………………………. se presentan los movimiento de …………………….


intestino delgado / segmentación y peristaltismo

17. Marita sufre una parálisis del músculo masetero. ¿Qué limitaciones se producirán a nivel del
movimiento de la mandíbula?
Elevación

18. Señale el movimiento que se produce a nivel del ciego y colon proximal que tiene como finalidad
favorecer la absorción de agua y sales.
De mezcla

19. La enfermedad de Hirchsprung se caracteriza por una dilatación anormal de colon y disminución de
los movimientos propulsivos que traerá como principal consecuencia …………
estreñimiento crónico.

20. ¿Cuál de los siguientes factores estimulan el vaciamiento gástrico?


Secretina

21. La secreción de ………………………. estimula la motilidad gástrica.


Motilina

22. La hormona ……………………….. es secretada por las células “I” del …………..
Colecistoquinina / duodeno y yeyuno
23. El reflejo enterogástrico aumenta ……………………… debido a la…………………..
el tono del esfínter pilórico / distensión del duodeno.

24. ¿Cuál de las siguientes sustancias disminuye el vaciado gástrico?


Colecistocinina.

25. El plexo mientérico se encuentra ubicado en………………. y funciona controlando la ………………


entre las capas musculares / motilidad

26. ¿Cuál de las siguientes glándulas está conformada por glándulas serosas y secreta la mayor
cantidad de amilasa en la boca?
Parótidas

27. ¿Cuál de los siguientes factores es más importante para percepción mediante las papilas
gustativas?
Agua

28. Paciente obeso, fumador, consumidor de alcohol que acude por presentar molestias como disfagia,
ardor retroesternal, alteración del sueño y sensación de boca amarga en las mañanas. ¿Cuál de las
siguientes recomendaciones podría darle a este paciente?
Elevar la cabecera de la cama al dormir.

29. En la enfermedad por reflujo se encuentra alterada la función del esfínter …………………….
esofágico inferior

30. En la diarrea psicógena hay una estimulación del ……………….. que …………………….. la motilidad
gastrointestinal.
Sistema nervioso parasimpático / aumenta

Producto de la hipertrofia pilorica uno espera encontrar q los niveles de Gastrina se encuentren:

Elevados

Al realizar una pilorotomia (corte del piloro para descomprimir) ¿Q hormonas se comezara a
liberar rapidamente en respuesta a este tratamiento?

Colecistoquinina

Debido al signo de onda peristaltica, se debe asumir q como consecuencia hay un aumento en la
liberacion de _____ en la pared del estomago:
Adenosina

La oliva pilorica se logra palpar a nivel de:

L1

El uso de atropina se espera q disminuya la liberacion de:

Enzimas pancreaticas

El consumo de una pequena cantidad de gaseosa aumentara directamente la concentración serica


de cual de las sgtes hormonas:

Peptido 1 similar al glucagon (GLP-1)

La colecistoquininca inhibe: el vaciamiento gastrico

Paciente con Lupus Eritematosos que desarrolla hipertensión portal debido a trombosis portal, es
probable que desarrolle várices a nivel de: → recto superior

En el síndrome de Mirizzi, el paciente tiene cálculos en la vesícula biliar; pero se obstruye el


conducto hepático común debido a que un cálculo se ubica y crece de tamaño en: → la bolsa de
Hartmann

En un recién nacido menor de 24 horas con atresia biliar, se encuentra elevación de la: →
bilirrubina directa

En el hígado, el aumento de la resistencia vascular en los sinusoides hepáticos ocasionará → salida


de plasma hacia el intersticio

Un paso importante para la poder absorber los lípidos es la emulsificación de las grasas. ¿Cuál de
las siguientes alternativas es el principal factor encargado de dicha emulsificación? → Ácidos
biliares

La vía biliar extrahepática se encuentra ubicada dentro del ligamento: → Hepatoduodenal

¿Cuál de las siguientes alternativas es correcta sobre la estructura hepática? → El flujo biliar en el
lobulillo hepático es centrífugo

En un paciente cirrótico con encefalopatía hepática ¿cuál de las siguientes alternativas sustenta la
reducción de carnes rojas en la dieta? → El amonio se produce principalmente en el intestino
La relajación del esfínter de Oddi se produce directamente por acción de: → el péptido vasoactivo
intestinal (VIP)

C! 1
1) El dolor periumbilical o epigástrico en el inicio de una apendicitis aguda se debe a:
• - Irritación del peritoneo parietal
• - Estímulo del sistema simpático
• - Íleo secundario
• - Estímulo del nervio vago

2) ¿Cuál de los siguientes péptidos inhibe el vaciamiento gástrico?


• - Péptido inhibidor gástrico
• - motilina
• - Gastrina
• - Colecistoquinina

3) El uso de Ranitidina bloquea el receptor H2 de la histamina en las células


parietales. La histamina llega a estas células por:
• - difusión
• - vía hematógena
• - la luz gástrica

4) El consumir caramelos indirectamente activa la vía:


• - POMC/CART
• - AGRP/NPY
• - grelina
• - MCR-4

5) Entre las múltiples causas de la Enfermedad por Reflujo Gastroesofágico, se puede


considerar también a una alteración en las ______________ del esfínter esofágico
inferior:
• - ondas secundarias
• - contracciones tónica
• - ondas lentas

6) El aumento en la actividad motora de la pared gástrica genera un aumento en los


niveles locales de qué sustancia en la microvasculatura:
• - adenosina
• - CCK
• - endotelina
• - gastrina

7) Durante una cirugía oncológica, ¿la extirpación de cuál de los siguientes órganos se
vería comprometida por la presencia de adventicia?:
• - yeyuno
• - estómago
- recto
- vesícula biliar

8) En un paciente con apendicitis aguda, la sensación de dolor producido por esta


inflamación es llevada por el nervio:
• - vago
• - esplácnico menor
• - esplácnico mayor
• - pélvico

9) ¿Cuál de los siguientes líquidos corporales tiene el pH más alto?:


• - saliva
• - jugo gástrico
• - jugo pancreático
• - bilis en la vesícula biliar

10) Marque la respuesta correcta en relación a la gastrina:


• - Al distender el estómago, se inhibe su producción.
• - Las células G son las productoras y se encuentran principalmente en el antro
gástrico
• - Se estimula por la liberación de noradrenalina
• - Las células G se encuentran principalmente en el fondo gástrico

C2
1) Respecto a la anatomía del estómago, marque lo correcto:
• - la arteria gástrica derecha nace de la arteria hepática común
• - No contiene muscular de la mucosa
• - la arteria gástrica izquierda irriga la curvatura mayor del estómago
• - Tiene una capacidad de 3 litros en la mayoría de personas

2) Respecto a las enfermedades del esófago, marque lo correcto:


• - el diagnóstico diferencial de la acalasia es la enfermedad de Chagas
esofágica
• - el síndrome de Boeerhave tiene mejor pronóstico que el síndrome de Mallory
Weiss
• - el síndrome de Mallory Weiss raramente se presenta con hematemesis
• - un agente infeccioso común del esófago es el Streptococo beta hemolítico

3) Estimula la producción de saliva:


• - atropina
• - vasodilatación periglandular
• - expresión de miedo
• - fatiga o cansancio

4) Respecto a la motilidad del colon, marque lo correcto:


• - Con la distención del estómago, suelen aparecer movimientos en masa
• - Se producen contracciones segmentarias principalmente en el colon
izquierdo
• - Los movimientos de masa se encargaran del mezclado de las heces
• - Las haustras del colon contribuyen al reflejo de defecación

5) En cuanto a la saliva, marque lo correcto:


• - La amilasa cumple función digestiva
• - el sistema simpático disminuye su secreción
• - cuando hay un estímulo de mayor secreción, se vuelve hipertónica
• - se producen 200 m diarios en condiciones normales
• - La amilasa cumple función digestiva

6) La saliva puede tener una variedad de electrolitos en su composición. Entre ellos el


cloro, respecto al cual se puede afirmar:
• - Su concentración no llega a ser tan alta como en el plasma
• - Su menor concentración se alcanza con flujo alto
• - Su mayor concentración se consigue con flujo bajo
• - Con flujo alta, su concentración es mayor que la del plasma

7) En un paciente hipertenso con tratamiento a base de beta bloqueadores, la


producción de saliva se espera que:
• - disminuya
• - no se afecte
• - aumente
• - disminuya
• - sólo se afecta si los receptores de acetilcolina están activos

8) Durante la secreción de saliva, es de esperarse que las concentraciones de


________ y ______ disminuyan al disminuir el flujo:
• - potasio de cloro
• - potasio sodio
• - bicarbonato potasio
• - sodio bicarbonato

9) El omeprazol actúa sobre la membrana _____________ de la célula ____________


• - basolateral / apical
• - basolateral / parietal
• - apical / principal
- apical / parietal

10) Dentro de los factores protectores de la mucosa gástrica se pueden mencionar


múltiples protagonistas. Uno de ellos es:
• - CCK

• - gastrina

• - receptor muscarínico
• - pepsina

C3
1) Si existe un fármaco con acción colerética, se asume que se refiere a que potencia
o estimula la:
• - Secreción de enzimas biliares
• - Contracción de la vesícula biliar
• - Excreción fecal de sales biliares
• - Recirculación de sales biliares

2) Una mujer de 38 años ingresa en el hospital con signos de colecistitis y cálculos


biliares. Durante la colangiografía, se inserta el catéter en la vesícula biliar con mucha
dificultad. ¿Cuál de las estructuras interfiere con más probabilidad con el paso del
catéter por el conducto cístico?
• - Válvula espiral (de Heister)
• - Compresión de la vena porta del conducto cístico
• - Adherencias del ligamento hepatoduodenal
• - Compresión del conducto cístico por una arteria hepática.

3) El acino pancreático difiere con el de las glándulas salivales en:


• - Contiene celulares centroacinares
• - No produce secreción serosa
• - El páncreas produce principalmente secreción mucosa
• - no tiene diferencias
4) En relación a la circulación hepática, marque lo correcto:
• - Los sinusoides hepáticos transportan sangre mixta
• - La vena porta proporciona el 50% e sangre al hígado
• - La vena porta se forma a partir de la vena esplénica y la mesentérica inferior
• - La arteria hepática deriva de la mesentérica superior

5) Hombre de 65 años con isquemia intestinal leve por oclusión aterosclerótica de la


arteria mesentérica superior, pero la irrigación sanguínea colateral ha retrasado el
inicio de la necrosis. ¿Qué vasos ofrecen colaterales entre el tronco celíaco y la arteria
mesentérica superior?
• - Gastroomental derecha e izquierda
• - Gástrica izquierda y hepática.
• - Pancreaticoduodenal superior e inferior.
• - Cística y gastroduodenal

6) El GALT se localiza en:


• - borde en cepillo
• - lamina propia
• - submucosa
• - superficie de criptas de Lieberkun

7) ¿Por cuál de las siguientes células es secretada principalmente la pro enzima


procarboxipeptidasa?
• - acinares del páncreas
• - ductales del páncreas
• - centro acinares del páncreas
• - epiteliales del duodeno

8) Marque lo correcto:
• - Las venas sublobulillares desembocan en las venas hepáticas
• - Las venas hepáticas contienen válvulas
• - Usualmente, son dos grandes venas suprahepáticas
• - Ninguna es correcta

9) Una mujer de 49 años ingresa en el hospital con dolor en epigástrico que migra
hacia el lado derecho y atrás hacia la escápula, sin ictericia. La ecografía muestra un
gran cálculo biliar. ¿En cuál de las siguientes estructuras es más probable que se
localice el cálculo biliar?
• - conducto hepático derecho
• - conducto colédoco
• - bolsa de Hartmann
• - conducto hepático izquierdo

10) Cuando el alimento se encuentra en el estómago, se produce la liberación de


enzimas pancreáticas básicamente debido a la acción de:
• - vago
• - bombesina
• - CCK
• - secretina

C4
1) Paciente de 21 años con dolor intenso en arcada dentaria superior debido a
emergencia de la tercera molar. Frecuencia cardíaca 108 latidos por minuto,
PA: 140/80 mmHg, agitado, pálido, son sudoración fría, y boca con saliva
espesa. En este paciente se espera que la secreción exocrina pancreática se
encuentre:
o - disminuida
o - aumentada
o - extremadamente elevada
o - sin cambios

2) Las sales biliares son absorbidas a la sangre en :


o - íleon
o - yeyuno
o - colon proximal
o - duodeno

3) Un signo característico en pacientes con encefalopatía hepática es:


o - convulsiones tónicas
o - reflejo de Babinsky
o - Onicomicosis
o - Asterixis

4) El efecto colerético de las sales biliares se refiere a:


o - estimulo de secreción biliar
o - inhibición de secreción biliar
o - aumento de la bilirrubina indirecta
o - ninguna de las anteriores

5) Paciente de 42 años con adenocarcinoma ductular. La TC ha demostrado


claramente que el tumor está en el cuello del páncreas y que hay un gran vaso
ocluido. ¿Cuál de los siguientes vasos estaría más probablemente obstruido?
o - arteria mesentérica superior
o - arteria pancreática magna
o - vena mesentérica inferior
o - vena porta

6) La secreción de agua y bicarbonato por el páncreas exocrino se da


básicamente en la fase:
o - intestinal
o - gástrica
o - en las tres por igual
o - oral

7) En relación a la secreción biliar y su composición, marque lo correcto:


o - las sales biliares corresponde al 50% de su composición
o - se producen alrededor de 3 litros diarios
o - los fosfolípidos excretados son anfipáticos, a diferencia de las sales
biliares
o - participan principalmente en la digestión de carbohidratos

8) Paciente con tumor neuroendocrino productor de secretina, debido a lo cual se


puede esperar que su secreción pancreática, comparada con la de una
persona sana en estado de bajo flujo, tenga una concentración de:
o - sodio aumentada
o - potasio disminuida
o - bicarbonato aumentada
o - igual

9) La vena central terminal desemboca en:


o - venas sublobulillares
o - vena porta
o - sinusoides hepáticos
o - espacio de Disee

10) Las invaginaciones del epitelio de la vesícula biliar que se extiende incluso
hasta la muscular se denomina:
o - conductos de Lushka
o - senos de Rokitansky-Aschoff
o - apéndices epiploicos
o - divertículos biliares

PARCIAL 2020-02

1.- Con respecto a la regulación del peristaltismo, al aplicarle atropina (antagonista colinérgico) a un
paciente, es de esperarse que el peristaltismo:

estimule la acción de los receptores dopaminérgicos

disminuya

se mantenga sin alteración

aumente

2.- Respecto al peristaltismo intestinal, para cumplir la ley del intestino , usted espera que a nivel
distal del quimo se libere:

péptido liberador de gastrina (GRP)

sustancia P

acetilcolina

péptido intestinal vasoactivo

3.- El estímulo habitual para el movimiento peristáltico es:

acción de la sustancia P

estimulación vago-vagal

contracción de la musculatura circular Interna


distensión local

4.- ¿Cuál de las siguientes condiciones considera que es un trastorno de la musculatura


lisaesofágica?

Hipotonía de los constrictores faríngeos

Asinergia faringoesfinteriana

Hipertonía del esfínter esofágico superior

Acalasia

5.- En un estudiante de medicina que está rindiendo un examen parcial, lo más probable esque en
ese momento su tránsito intestinal se encuentre:

sin alteraciones

estimulado por acción de la sustancia P

muy lento

muy acelerado

6.- El peristaltismo intestinal se produce gracias a un reflejo que:

llega a los ganglios pre vertebrales

llega al sistema nervioso central

se origina dentro de la pared intestinal

llega a la médula espinal

7.- Las siguientes alternativas son ciertas sobre las contracciones tónicas del músculo
gastrointestinal, EXCEPTO:

Tienen relación con el ingreso persistente de iones sodio

Se encuentran principalmente en esfínteres

Obedece a una mayor frecuencia de potenciales en espiga

Tienen regulación hormonal


8.- La información sensitiva aferente del sistema gastrointestinal pasa por las siguientesestructuras,
EXCEPTO:

Tálamo

Médula espinal

Ganglios prevertebrales

Tronco encefálico

9.- Al realizar una vagotomía por úlcera péptica, usted esperaría una disminución deproducción de
gastrina debido a la:

menor distensión de las paredes gástricas

ausencia de acetilcolina vagal

ausencia de histamina

ausencia de bombesina vagal

10.- Las siguientes funciones son inhibidas por la hormona somatostatina, EXCEPTO:

Secreción de enzimas pancreáticas

Secreción gástrica de HCl

Liberación de gastrina

Motilidad intestinal

11.- En una cirugía abierta (laparotomía), el cirujano al abrir la cavidad peritoneal por la parteanterior
(línea media), lo primero que observa es:

Epiplón mayor

Estomago

Duodeno

Colon sigmoides

12.- ¿Cuál de las siguientes estructuras tiene inervación somática?

Estómago
Peritoneo parietal

Peritoneo visceral

Mesosigmoides

13.- En el plexo mientérico, el origen de los impulsos eferentes está en:

el plexo de Meissner

las células intersticiales de Cajal

los ganglios paravertebrales

el plexo de Aurbach

14.- En el esófago, el plexo mientérico o de Auerbach interviene en el _______ de losimpulsos


______

inicio / aferentes

fin / eferentes

fin / aferentes

inicio / eferentes

15.- Paciente con lesión del hipogloso del lado izquierdo. Para evaluarlo se le pide al pacienteque
saque la lengua, la cual se espera que la punta de la lengua se dirija hacia:

abajo

el lado izquierdo

adelante

el lado derecho

16.- Al ingresar líquidos o sólidos en la cavidad oral, un mecanismo que permite que unapersona
respire mientras mastica es:

la elevación del paladar blando

la depresión de la parte posterior de la lengua

el movimiento hacia afuera de los pliegues palatogloso ypalatofaríngeo


la depresión del paladar blando

17.- Paciente adulto con reflujo gastroesofágico es más probable que presente:
destrucción de ameloblastos

remodelación del esmalte

descalcificación del esmalte

desfluorización de los dientes

18.- En un paciente de 3 semanas de edad, con vómitos en proyectil, y nódulo epigástricoreptante,


usted esperaría encontrar:

engrosamiento de la circular interna pilórica

colecistoquinina aumentada

distensibilidad disminuida de la región oral del estómago

vómitos biliosos e intolerancia a los ácidos grasos

19.- ¿Cuál de las siguientes atresias/fístulas traqueo esofágicas considera usted que esincompatible
con la vida (de no recibir tratamiento)?

Tipo E

Tipo B

Tipo C

Tipo A

20.- El músculo liso gastrointestinal funciona como un sincitio debido a:


el calcio

las uniones en hendidura

las fibras musculares más largas

el plexo mientérico de Auerbach

21.- Durante el paso del bolo hacia la orofaringe, se desencadena una serie de
contraccionesmusculares que estrechan la cavidad faríngea. Estas contracciones están mediadas
porel nervio craneal:
XII

IX

XI

22.- Las siguientes alternativas son ciertas sobre la actividad eléctrica del músculo gastrointestinal,
EXCEPTO:

Si el potencial de membrana es más positivo, habrá mayorfrecuencia de espigas

Las ondas lentas no son potenciales de acción

Las ondas lentas son más frecuentes en el duodeno

La despolarización lenta se debe principalmente al ingreso de Na+

23.- Los corpúsculos gustativos se encuentran en la lengua, pero además se les puede encontrar
en:

el dorso de la lengua

las encías

el paladar blando

el paladar duro

24.- Mujer de 30 años llega a emergencia con dolor en hipogástrico. Al examen físico presentauna
masa palpable de 10 cm de diámetro a en la misma región, usted sospecharía de lassiguientes
condiciones, EXCEPTO:

Tumor renal

Embarazo

Tumor uterino

Cáncer de recto superior

25.- La masticación es básicamente:


importante para la digestión sobretodo de carnes

una actividad consciente


un movimiento reflejo

un ralentizador del vaciamiento gástrico


1. Al examinar a un paciente, usted encuentra dolor localizado en fosa iliaca derecha y diagnostica
apendicitis. En este paciente, usted puede inferir:
El peritoneo parietal regional está afectado
2. La motilidad intestinal es estimulada principalmente por el:
Plexo de Auerbach
3. Durante el vómito, ¿el contenido gástrico tiene que pasar necesariamente por cuál estructura para
llegar al esófago? Marque la mejor respuesta:
Cardias
4. Respecto a la anatomía del estómago, marque lo correcto:
El fondo gástrico forma la curvatura mayor
5. Marque la respuesta incorrecta:
En todo el tubo digestivo, se observa dos capas de muscular propia: circular interna y
longitudinal externa
6. Paciente se queja de dolor en hipocondrio derecho, pero superficialmente. El dermatoma relacionado
es (marque la mejor respuesta):
T9
7. Dentro de las funciones del abdomen, se encuentra la defecación y micción, en las cuales la presión
intra abdominal debe:
Aumentar
8. Un alumno de medicina decide hacerse un piercing en el ombligo. Al realizarle el procedimiento,
sangra ligeramente. Esta sangre proviene de la arteria (marque la mejor respuesta)
Epigástrica inferior
9. Señale la respuesta correcta:
El apéndice cecal sólo tiene serosa
10. Paciente mujer es traída a emergencia por sufrir una herida contuso penetrante por cuchillo realizada
por su esposo en un ataque de celos. Se observa herida en flanco izquierdo. Esta solución de
continuidad ha comprometido varios músculos de la pared abdominal, excepto:
Recto abdominal
11. Paciente con herida por proyectil por arma de fuego, con herida de ingreso en región paraumbilical.
Entre las estructuras que usted está seguro que debe haberse lesionado es:
Omento mayor
12. En la evaluación de una tomografía abdominal, el interno observa un aneurisma en una arteria que
se dirige al riñón derecho. Con seguridad se puede afirmar que está a nivel de la vértebra:
L1
13. Paciente joven es traído a emergencia con abdomen agudo quirúrgico debido a herida contuso
penetrante por verduguillo (alambre grueso con punta aguzada) recibida en una pelea después de un
partido de fútbol. Se observa herida en Hipocondrio Izquierdo. El órgano que debe estar sangrando y
produciendo hemoperitoneo es (marque la mejor respuesta):
Bazo
14. Marque el órgano que se encuentra más distal en el tubo digestivo.
Ciego
15. La peristalsis o peristaltismo hace referencia a:
Motilidad para movilizar el alimento de proximal a distal.
16. Marque la respuesta incorrecta:
En todo el tubo digestivo, se observa dos capas de muscular propia: circular interna y
longitudinal externa
17. Paciente con vólvulo del colon sigmoides. La necrosis de ese segmento del colon se produce por una
alteración en la irrigación de la arteria:
Mesentérica inferior
18. Marque el órgano que se considera retroperitoneal:
Parte de la vía biliar
19. Un alumno de medicina decide hacerse un piercing en el ombligo. Al realizarle el procedimiento,
sangra ligeramente. Esta sangre proviene de la arteria (marque la mejor respuesta):
Epigástrica inferior
20. Paciente tiene una úlcera sangrante en el segundo tercio del Yeyuno. La arteria de la cual proviene la
sangre arterial para dicha zona es la arteria:
Mesentérica superior
21. Es inervado por aferentes somáticas:
Peritoneo parietal
22. Paciente de 24 años con dolor abdominal tipo cólico intenso en mesogastrio. Según sus
conocimientos de macroestructura, el origen del dolor puede ser _____:
Íleon
23. Paciente con hipoglucemia secundaria a un insulinoma (tumor neuroendócrino productor de insulina).
El órgano donde mayor probabilidad ha crecido este tumor es:
retroperitoneal
24. Al evaluar una tomografía abdominal, el médico asistente le pide al interno de la UPC que encuentre
la imagen con el corte a nivel de L1. El interno sabiamente busca el para ubicar la vértebra L1:
Cuello del páncreas
25. En la inspiración, la pared abdominal debe relajarse para disminuir presión intra torácica
26. Ligamento hepatogástrico une el ….. con el …… y forma la entrada al …… :
Hígado Estómago Orificio omental
27. Al retirar completamente el mesenterio de un órgano, el mismo se vería afectado principalmente en
su:
Irrigación
28. La estructura que fija órganos principalmente a la pared posterior abdominal se denomina:
Mesenterio
29. Cuál de las siguientes estructuras no tiene vasos sanguíneos:
Epitelio intestinal
30. Al iniciar la digestión, aumenta el consumo de oxígeno por la mucosa. Esto conlleva a una hipoxia
local, lo cual hace que se libere …., el cual produce vasodilatación:
adenosina
31. Sustancia que inhibe la secreción y la motilidad del estómago prolongando el tiempo de digestión:
péptido insulinotrópico dependiente de la glucosa (GIP)
32. Marque lo correcto:
La hernia fisiológica se produce en la sexta semana y es la salida temporal de asas
intestinales a través del colon umbilical
33. Marque la respuesta correcta en relación a la gastrina:
Las células G son las productoras y se encuentran principalmente en el antro gástrico.
34. El consumir caramelos indirectamente activa la vía:
POMC/CART
35. ¿En qué capa se encuentra la alteración principal en el Hirschsprung o megacolon agangliónico?:
Muscular propia
36. Con respecto a las ondas lentas, marque la afirmación correcta:
Son contracciones rítmicas espontáneas
37. El uso de Ranitidina bloquea el receptor H2 de la histamina en las células parietales. La histamina
llega a estas células por:
Difusión
38. La triada sintomática: vómitos explosivos post-prandiales, movimientos peristálticos epigástricos
visibles de izquierda a derecha y nódulos palpable epigástrico subcostal derecho, pertenecen a:
Estenosis congénita hipertrófica del píloro
39. Durante una cirugía oncológica, ¿la extirpación de cuál de los siguientes órganos se vería
comprometida por la presencia de adventicia?:
Recto
40. En cuanto a los reflejos gastrointestinales, un reflejo que estimula el tránsito intestinal es el reflejo:
Gastrocólico
41. El ligamento falciforme divide al hígado en dos lóbulos derecho e izquierdo. Embriológicamente
deriva del:
Mesenterio ventral
42. La presencia de atresias y estenosis duodenales se deben básicamente a una:
Falta de recanalización
43. Estudiante de medicina de 20 años, se ha amanecido estudiando para su examen de Sistema
Digestivo. No ha probado alimento desde la cena, por lo que se puede afirmar que la motilidad de
esta persona está siendo regulada por:
Motilina
44. Paciente con disminución del apetito marcada asociada a cáncer terminal, para promover la ingesta
de alimentos se podría usar análogos de:
Endorfinas
45. Las ondas lentas se producen por la apertura cíclica de canales de:
Calcio
46. La forma más común de atresia esofágica contiene:
Estenosis proximal del esófago más fístula traqueoesofágica distal
47. Al deglutir un bolo alimenticio, es lógico suponer que al pasar por el esofago haya un mayor consumo
de oxigeno en la pared del tercio:
Proximal
48. Paciente que come entera una pizza familiar de chorizo y queso. Es posible esperar que debido a la
cantidad de alimento ingerida, las ondas lentas hayan:
sufrido ninguna alteración en su frecuencia.
49. La hernia fisiológica se produce dentro de:
cordón umbilical
50. El crecimiento de un adenocarcinoma de páncreas compromete la pared gástrica por contigüidad
¿Que parte del estómago se esperaría esté comprometido?:
Pared posterior del antro
51. Estimula la producción de saliva:
Vasodilatación periglandular
52. Durante la secreción de saliva, es de esperarse que las concentraciones de ….. y …… disminuyan al
disminuir el flujo:
Sodio, Bicarbonato
53. Con respecto a la secreción gástrica de HCI:
a mayor secreción de HCI en el lumen gástrico, mayor pH en la sangre venosa gástrica.
54. Respecto a las enfermedades del esófago, marque lo correcto: el diagnóstico diferencial de la
acalasia es la enfermedad de Chagas esofágica
55. Con respecto a las lesiones y enfermedades de la boca, marque lo correcto: la eritroplasia debe ser
biopsiada
56. Respecto a las glándulas salivales, marque lo incorrecto:
la glándula sublingual tiene forma de garfio
57. El omeprazol actúa sobre la membrana ….. de la célula ….. :
apical/parietal
58. Durante el sueño, la concentración de bicarbonato en la saliva:
Disminuye
59. Durante el ataque con gas sarín (bloqueador de la acetilcolinesterasa) en el metro de Tokio, en 1995,
el personal de salud noto que los pacientes afectados presentaban:
Hipersalivación
60. La célula mucosa del cuello gástrico produce:
Moco

Parcia de 2020-01
61. Al evaluar la orofaringe de un paciente, el médico le solicita que abra la boca, saque la lengua y diga
a. Al hacer esta maniobra, nota que el paladar se desvía hacia la derecha, lo cual le hace sospechar
que el paciente sufre de una lesión del nervio craneal:
X contralateral
62. Un bolo alimenticio grande y poco masticado se atasca en el esofago, esto ocasiona una sensación
de dolor que es transmitida por los nervios:
esplácnicos
63. Para realizar el movimiento mecánico de abrir la boca, primero se necesita:
fijar el hueso hioides
64. ¿Cual de las siguientes alternativas se define como la protrusión directa del contenido abdominal a la
cavidad amniótica por un defecto de la pared corporal?:
Gastrosquisis
65. Un paciente requiere que se le coloque una sonda de alimentación directamente al estómago
(gastrostomía), el cirujano deberá hacer una incisión en la piel del abdomen ¿cuál de las siguientes
raíces nerviosas debe ser anestesiada para este procedimiento?
T8
66. En un paciente de 43 años con tumor carcinoide de páncreas productor de gastrina (Síndrome de
Zollinger-Ellison) se puede esperar encontrar una potenciación del reflejo:
gastrocólico
67. El mecanismo de la defecación incluye la participación de diversas estructuras ¿cuál de las
siguientes alternativas es correcta?:
Puede ser mediado por un reflejo intrínseco
68. Cuando el contenido del estómago ingresa al duodeno, uno de los reflejos que inhiben el vaciamiento
gástrico es a través del:
Sistema nervioso mientérico
69. Durante la masticación, gran parte del proceso masticatorio se debe a:
El reflejo masticatorio
70. Las glándulas salivales tienen conductos para la excreción de la saliva, las glándulas …… drenan en
las carúnculas sublinguales.
Sublinguales
71. Los diferentes segmentos del tubo digestivo son susceptibles de reflejos y movimientos según su
contenido. Si colocamos mediante una sonda un bolo alimenticio directamente en el tercio medio del
esofago:
se producira ondas secundarias
72. En una persona si enfermedad se espera que el tránsito intestinal se vea disminuido cuando se
presenta el reflejo:
Doloroso
73. El divertículo de Meckel es una anomalía congénita que ocurre por la persistencia del conducto
vitelino y da origen a una estructura sacular, el cual se encuentra en el:
borde antimesentérico
74. Si al intubar a un paciente, por error se ingresa el tubo endotraqueal en el esofago y se insufla el
manguito endotraqueal (globo TET), la dilatación de este manguito generará:
múltiples ondas secundarias
75. El orificio omental, o hiato de Winslow, se encuentra limitado por el ligamento:
hepatoduodenal
76. Paciente de 24 años acude a consulta externa por presentar una fístula oronasal (comunicación entre
la cavidad oral y la cavidad nasal). Esta fístula es una consecuencia tardía de la lesión de un vaso
sanguíneo por el antecedente de haber sido operado de paladar hendido en los primeros años de
vida, aparentemente en una campaña gratuita de corrección de paladar fisurado. ¿Cual de las
arterias palatinas podría haberse lesionado durante esa cirugía?:
Mayor
77. Dentro de las anomalías congénitas se puede presentar un tejido pancreático accesorio ¿cuál es la
ubicación más común de este tejido?:
Estómago
78. Una recién nacida es evaluada por el neonatólogo y evidencia que el canal anal está completamente
cerrado. Este problema se debe probablemente a una anomalía en el desarrollo de:
la membrana cloacal
79. En la digestión de los alimentos, la hormona se libera frente a la presencia de péptidos y
monoglicéridos, y tiene un efecto marcado en la disminución del vaciamiento gástrico
colecistoquinina
80. Dentro de las anomalías congénitas se puede presentar un tejido pancreático accesorio ¿Cuál es la
ubicación más común de este tejido?
Estómago
81. Los catadores de vino tienen una habilidad increíble al momento de separar los sabores. Este
aumento de la sensibilidad gustativa debido a una mayor cantidad de papilas linguales y de
corpúsculos gustativos se conoce como:
hipergeusia
82. En muchos países se usa el suplemento de flúor en el agua potable o los dentífricos, con el fin de
hacer el esmalte más resistente a la desmineralización inducida directamente por:
el ácido
83. Durante el desarrollo de la región cloacal, una cuña de mesodermo ubicado entre el alantoides y el
intestino posterior vendrá a formar el:
tabique urorrectal
84. En una persona sana, el momento adecuado para encontrar los mayores niveles de grelina en
sangre sería:
antes de comer
85. El duodeno está constituido por el segmento terminal del intestino anterior y el segmento
proximal del intestino medio ¿Cuál de las siguientes alternativas describe mejor este lugar de unión
entre los dos intestinos?
Distal al origen de la yema hepática
86. En una persona sana, el uso de atropina producirá a nivel del estómago:
Aumentará el pH del estómago
87. En una persona sana, el consumo de leche produce indirectamente
Inhibición del vaciamiento gástrico
88. La motilidad del colon es importante y lenta comparada con la del intestino delgado. Los movimientos
en masa ocasionan la:
distensión rectal
89. Con respecto a la motilidad gástrica, los potenciales de acción disminuyen en frecuencia por efecto
de:
el péptido insulinotrópico dependiente de glucosa
90. La sensación del gusto depende de la presencia de papilas gustativas en la lengua, de las cuales,
algunas de ellas tienen un surco terminal por donde drenan unas glándulas salivales linguales
(llamadas glándulas de von Ebner). Esta descripción se refiere a las papilas:
circunvaladas
91. Dentro de la estructura de los dientes, la parte del diente cubierta por esmalte y que se puede ver
mediante la inspección visual de la boca se denomina
corona clínica
92. En el conducto anal se encuentra la unión entre las regiones del endodermo y el ectodermo, esta
unión se evidencia al observar:
la línea pectínea
93. La sensación del gusto depende de la presencia de papilas gustativas en la lengua, las cuales tienen
corpúsculos gustativos conteniendo células neuroepiteliales sensoriales. Estas células
neuroepiteliales pueden ser dañadas fácilmente, por suerte, su tiempo de recambio es de alrededor
de:
10 días
94. En un varón de 47 años con sección medular a nivel de T6 debido a un accidente automovilístico, sus
terapeutas han desarrollado un mecanismo para distender el recto e iniciar el reflejo
rectoesfinteriano, lo cual producirá la contracción de:
la pared del recto
95. Durante un experimento, se insufla rápidamente dos litros de agua en un globo colocado dentro del
estómago de un voluntario. ¿cuál de las siguientes situaciones del músculo liso será consecuencia
directa de este cambio de volumen en el estómago?
Despolarización
96. Paciente de 56 años con accidente cerebrovascular reciente. En la resonancia se observa daño de
los núcleos laterales del hipotálamo. Por este motivo es muy probable que el paciente sufra de:
inanición
97. Paciente con insuficiencia mitral moderada a severa, con aumento de volumen de la aurícula
izquierda, esta condición tendrá como consecuencia a nivel del sistema digestivo:
la disfagia a sólidos
98. El inicio de la fase faríngea de la deglución se debe a estímulos sensitivos que viajan por el nervio
craneal: V
99. El mesocolon transverso se origina en:
la pared posterior del abdomen
100. El esofago en su microestructura tiene basicamente adventicia, a excepción de la región distal,
donde tiene serosa, específicamente a partir del nivel de:
T10
101. En un paciente con arcadas, se debe considerar que durante la ocurrencia de dichas arcadas,
debemos encontrar contenido gástrico en:
Tórax
102. La reabsorción de Sodio y Cloro en las glándulas salivales se da principalmente en el: Conducto
estriado
103. Para que se puedan digerir las grasas, es preferible que primero sean emulsificadas. La hormona
que estimula la liberación de las sustancias emulsificadoras es:
CCK
104. Paciente de 13 días de vida con vómitos explosivos a las 2 horas después de lactar. Al examen
físico se palpa la oliva pilórica ¿cual es el nervio cuyos filetes dan inervación eferente a la estructura
afectada?:
Vago
105. La localización de la vesícula biliar con respecto al lóbulo cuadrado es:
Lateral
106. La triada portal está constituida por el conducto biliar y la arteria hepática y una pequeña rama de
la vena:
Porta
107. Al ingerir una cantidad de glucosa por vía oral, esta es interiorizada en las células del organismo
más rápido que si esa misma cantidad hubiese sido administrada por vía endovenosa. Este
fenómeno sucede gracias a una sustancia secretada por las células:
K
108. Paciente de 62 años con vólvulo de intestino delgado e isquemia intestinal. ¿Qué estructura se
utiliza como punto de referencia para determinar la posición de la unión duodenoyeyunal? :
Ligamento suspensorio del duodeno (de Treitz)
109. En un paciente con Zollinger Ellison, usted esperaría encontrar:
Esteatorrea
110. La motilidad intestinal es estimulada por:
Colecistoquinina y gastrina
111.La onda peristática secundaria del esofago en la deglución, es producida por:
Plexo mientérico esofágico
112. Paciente con parálisis bilateral del nervio hipogloso, el unico musculo de la lengua que
conservará su movimiento es el:
palatogloso
113. ¿Cual de las siguientes sustancias tiene mayor concentración de la saliva comparado con su
concentración plasmática?
Potasio
114. ¿Cual de las siguientes estimula la secreción enzimática exocrina del páncreas?:
Colecistoquinina
115. Al comer un pollo a la brasa, con papas fritas y ensalada, la sustancia que estimula la liberación
de HCI en el estómago es:
Bombesina
116. Marque lo correcto con respecto a Esófago de Barret:
Se relaciona con reflujo gastroesofágico
117. Un niño de 2 años es llevado a la consulta por diarrea persistente y edema de las extremidades,
además falta de crecimiento y desarrollo en relación a su edad. Los análisis de sangre revelan que
tiene concentración plasmática baja de proteínas (hipoproteinemia). Durante la endoscopía duodenal,
se coloca colecistocinina (CCK) endovenosa y se recoge muestras del líquido duodenal; el resultado
del líquido confirma incapacidad para hidrolizar proteínas a un pH neutro, esta situación mejora al
añadir una pequeña cantidad de tripsina. El paciente probablemente esté sufriendo la falta congénita
de
-Enterocinasa
118. Experimentalmente se incrementa la velocidad de la secreción salival con una sustancia, el
análisis de la composición de esta saliva obtenida se espera encontrar…………..
-Disminución de concentración de potasio
119. Paciente varón de 46 años soltero, consulta por odinofagia y bajo de peso, tiene antecedente de
tuberculosis desde hace 3 meses y es fumador crónico (10 cigarrillos por día); al evaluar la cavidad
oral se identifica lesión blanquecina en el dorso de la lengua y paladar blando, las lesiones se
desprenden con el baja lengua dejando una base eritematosa. Esta lesión corresponde
probablemente a ……………………….…..
-Candidiasis oral
120. Paciente mujer de 35 años acude a consulta por sensación de sequedad y lesiones en cavidad
oral. Al examen se observa atrofia de la mucosa, fisuras y úlceras; nota además sequedad e irritación
de la córnea y aumento del tamaño de las glándulas parotídeas. Su diagnóstico más probable es
artritis reumatoide; el hallazgo más probable en una biopsia de glándula parótida es……..….
-Gran infiltración de linfocitos y células plasmáticas
121. Un paciente con anemia acude con su médico quejándose de episodios frecuentes de
gastroenteritis. Un análisis de sangre revela anticuerpos circulantes dirigidos contra células parietales
gástricas. Su anemia es atribuible a la hiposecreción de
-Factor intrínseco
122. Dos estudiantes deciden tomar un receso para comer una hamburguesa a la hora del almuerzo.
Antes de llegar a la cafetería, impulsos nerviosos provenientes del complejo vagal dorsal iniciarán la
secreción de ácido gástrico por la liberación de …………………….. desde el sistema nervioso
entérico.
-GRP
123. Un niño de cuatro años de edad es llevado a la consulta por cuadros diarreicos frecuentes
caracterizados por heces pálidas, voluminosas y fétidas, presenta bajo peso y talla. Se mide la
concentración de cloruro en el sudor y se encuentra que sus valores son muy elevados. La alteración
más importante a nivel de células ductales del páncreas tiene relación directa con la conductancia
de…………
-Cloro
124. Una mujer de 50 años de edad que sufrió durante varios años resequedad de los ojos debida a
producción inadecuada de lágrimas es enviada con un gastroenterólogo para evaluación de pirosis
crónica. El examen endoscópico revela erosiones y tejido cicatrizal en la parte distal del esófago justo
por arriba del esfínter esofágico inferior. Las lesiones pueden atribuirse a la disminución de uno de
los siguientes componentes salivales:
-Bicarbonato
125. Se evalúa los valores séricos de las siguientes sustancias a un paciente con enfermedad
hepática terminal; en este paciente se espera encontrar la combinación con la letra …………
-disminuida, aumentada, disminuida
126. Una mujer de 35 años de edad HIV positiva, se presenta al médico con dolor abdominal en
cuadrante superior derecho e ictericia. La paciente refiere haber tenido múltiples episodios de
ictericia durante los últimos 10 años. Los exámenes para determinar hepatitis viral, dieron positivos
para Hepatitis B, siendo catalogado el caso como hepatitis crónica con alteración funcional. En un
examen de sangre ¿cuál de los siguientes parámetros está disminuido?
-Albúmina
127. En el reflejo peristáltico del intestino delgado, uno de los siguientes eventos sucede en la porción
oral del bolo alimenticio…………...
-Acción de acetilcolina en el músculo circular
128. Experimentalmente se coloca una dosis alta de secretina en la luz intestinal duodenal; como
consecuencia de esto, en el jugo pancreático de la misma luz intestinal se observa la disminución de
la concentración de …..………..
-Cl
129. Un varón de 58 años de edad con enfermedad de Crohn severo fue sometido a una resección
ileal. Después de la cirugía este paciente padecerá de esteatorrea, esto se explica porque
…..………..
- La micelas no pueden formarse
130. En un experimento se inserta un balón en el estómago de un voluntario, se infla poco a poco
mientras que se vigilan las presiones intraluminales. Aunque el volumen del balón aumenta
considerablemente, las presiones permanecen constantes. Esta relación volumen-presión se explica
por la liberación local de …………..
-Óxido nítrico y péptido inhibidor vasoactivo
131. La toxina de Vibrio cholerae causa diarrea debido a…….
-El Incremento de la secreción de cloro por las células de la cripta intestinal
132. ¿Cuál de las siguientes alternativas es una característica de la secreción exocrina del páncreas?
-Tiene una baja concentración de Cl- respecto al plasma
133. Una madre lleva a su hijo de dos años de edad a la sala de urgencias, estresada porque el niño
deglutió una moneda de 10 céntimos mientras la familia cenaba en un restaurante. El médico
observa mediante fluoroscopía que la moneda se halla en el estómago y asegura a la madre que la
moneda se eliminará con las heces. El médico recomienda utilizar la respuesta fisiológica que
permitirá la evacuación de la moneda del estómago al intestino ………….…..
-Son los movimientos de mezcla y trituración
-. Es provocada por el ayuno
134. Las estructuras en el hígado que permite que los productos metabólicos unidos a proteínas
tengan acceso a las membranas basolaterales de los hepatocitos, son…..
- Las fenestras sinusoidales
135. La composición de la bilis es modificada conforme fluye por los conductillos biliares. Durante este
tránsito se espera que aumente la concentración de……
-Ig A
136. Se mide experimentalmente el contenido gástrico de dos personas. La persona “A” tiene alto
contenido de grasa y la persona “B” tiene un contenido hipertónico ¿Cuál de las siguientes es
correcto respecto al vaciamiento gástrico?
- Hay ralentización del vaciado gástrico en ambos casos
137. El examen endoscópico de un paciente con hipertensión portal grave revela venas tortuosas que
sobresalen hacia la luz del esófago. El paciente recibe tratamiento quirúrgico mediante la colocación
de una derivación que conecta la vena porta a la vena cava. Después de la operación el riesgo de
encefalopatía y el riesgo de sangrado de várices ……………..
-Aumentará/disminuirá
138. Un paciente varón de 18 años de edad acude al médico para sus exámenes de rutina. Sus
resultados de laboratorio muestran un valor de bilirrubina sérica de 4 mg/dl y una bilirrubina directa
de 0,3 mg/dl. Las pruebas de función hepática son normales. La alteración que explica mejor este
caso es por la deficiencia de ………………..
-Glucuronil transferasa
139. Un hombre de 57 años de edad es llevado a urgencias con hematemesis masiva rojo brillante, a
su llegada se halla inconciente con PA: 80/40 mm Hg y FC: 124 lat/min. Luce ictérico con presencia
de “arañas vasculares en el tórax anterior y extremidades”, abdomen distendido con signo de oleada
positiva. Se encuentra esplenomegalia y pérdida de la masa muscular en extremidades. La
anastomosis vascular responsable del sangrado en este paciente es
-Vena gástrica izquierda y vena ácigos
140. Un estudiante de medicina está comiendo un plato de comida a base de champiñones, espárrago
y salsa de soya. El sabor umami contenido en todos estos alimentos actúa a nivel de los botones
gustativos estimulando ………………..
-Un receptor acoplado a proteína G
141. Un hombre de 22 años de edad se presenta al médico con una historia de 1 año de evolución
caracterizado por dolor recurrente en fosa iliaca derecha y diarrea. Manifiesta además pérdida de
peso de 8 kg durante este periodo. La colonoscopía revela múltiples lesiones en el ileon terminal y
colon. La biopsia de estas lesiones revela engrosamiento, inflamación y ulceración de la mucosa. El
diagnóstico más probable en este caso es…….
-Enfermedad de Crohn
142. Varón de 61 años que consulta por dolor retro esternal intenso desde hace 6 horas y después de
vómitos intensos y repetidos; al examen se observa disnea, cianosis, hipotensión y signos clínicos de
shock. La radiografía simple de tórax muestra neumomediastino. El líquido en el espacio pleural
aspirado tiene alta concentración de amilasa. ¿Cuál de las siguientes alternativas puede explicar este
cuadro clínico?
-Rotura espontánea de esófago
143. La secreción del ácido en la célula parietal gástrica se lleva a cabo por una ATPasa especifica
que intercambia hidrogeniones (H+) del citosol por….. -K +
144. En condiciones normales el ingreso de 600 ml de líquido es el estómago provoca un aumento de
presión intragástrica de unos 12 cm de H2O. Después de una vagotomía (corte del nervio vago) es
de esperar que el ingreso del mismo volumen de líquido provoque lo siguiente:
…………………………………
-Un aumento mayor de la presión
145. Una paciente de 30 años de edad es sometida a una cirugía en oído medio derecho por un
problema de otoesclerosis. Luego de la cirugía refiere alteración en la percepción de sabores. Al
evaluar el caso usted esperaría encontrar……….
-Alteración en la sensación del gusto en los dos tercios anteriores de la lengua
-Sensación del dolor, tacto y temperatura conservada en toda la lengua
146. ¿Cuál de las siguientes alternativas es correcta?
-Las sales biliares desconjugadas son absorbidas preferentemente en el colon
147. En un paciente de 45 años de edad con colestasis biliar, se encuentra una elevación de los
niveles sanguíneos de fosfatasa alcalina hasta 3 veces la cifra normal. ¿Cuál de las siguientes
alternativas estará también elevada como evidencia del daño de la vía biliar?
-Gamma glutamil transpeptidasa
148. Revisando la angiografía de un hombre de 70 años en estudio por aneurisma de aorta abdominal
el radiólogo informa de la presencia de una oclusión completa de la arteria mesentérica inferior. El
paciente se encuentra completamente asintomático. ¿Cuál de las siguientes arterias se anastomosa
a la sistema arterial de la mesentérica inferior?
-Cólica media
149. Lactante de 3 meses de vida es atendido por presentar diarrea, se administra una solución de
glucosa y electrólitos por vía oral. La proteína de membrana apical que explica la capacidad de esta
solución para proporcionar aporte de glucosa e hidratación es ………..
-SGLT-1
150. Paciente ha sufrido herida de bala en el abdomen, se le ha tenido que extirpar el segmento
medio y distal del ileon. En este caso la síntesis hepática de sales biliares estará …..…..
-Incrementada por estímulo de la enzima colesterol 7 alfa hidroxilasa
151. Un varón de 75 años ingresa al consultorio por presentar ictericia marcada de piel y las escleras.
El estudio del paciente mostró que presentaba un tumor que obstruía la totalidad del conducto
hepático común. ¿Cuál de las siguientes estructuras se encontrará dilatada en este paciente?
-Conductos de Hering
152. En un paciente con insuficiencia renal crónica, el déficit en la absorción de calcio a nivel del
enterocito se debe a lo siguiente:
-No se convierte la 25 hidroxicolecalciferol a 1,25 dihidroxicolecalciferol
153. Varón de 30 años es traído a emergencia por agresión abdominal con arma de fuego (pistola) y
es sometido a laparotomía exploratoria, observándose isquemia del colon ascendente y parte del
colon trasverso ¿la lesión de cuál de las siguientes arterias explicaría esta isquemia?
- Mesentérica superior
154. Respecto a las sustancias gastrointestinales que regulan la secreción pancreática; marque la
afirmación correcta:
- La acetilcolina es capaz de estimular la secreción enzimática y de bicarbonato del páncreas
155. Ante una lesión del X par craneal, ¿cuál de los siguientes músculos mantiene conservada su
función?:
- Tensor del velo del paladar
156. Experimentalmente se utiliza atropina (anticolinérgico) para inhibir la secreción de gastrina, sin
embargo la secreción de esta hormona se sigue dando ante estímulos vagales. Esta situación se
explica porque la atropina:
- No bloquea la acción del péptido GRP
157. Un varón de 50 años es sometido a extirpación del duodeno y parte proximal del yeyuno. Esta
situación ocasionaría la pérdida de las células, productoras de………………… que estimula la
secreción de bicarbonato por el páncreas.
-“S” / secretina
158. Se evalúa la expresión de la proteína Agrp en una persona con alteración del apetito; lo correcto
respecto a esta proteína es…..
- La mutación del gen que la codifica produce adelgazamiento
159. Juana cae de la bicicleta y se fractura la región anterior del hueso maxilar superior con
compromiso de la fosa incisiva. Al examen físico de la región esperaría encontrar alteración en la
sensibilidad de la encía …………………
- Palatina anterior
160. Recién nacido es atendido por el neonatólogo y luego entregado a su madre para dar de lactar;
la madre al dar de lactar observa coloración azulada de labios, acompañado de tos persistente,
dificultad respiratoria y distención abdominal. Se le intenta colocar una sonda nasogástrica pero esta
retorna a la cavidad oral en todos los intentos. ¿Cuál de las siguientes anomalías del desarrollo es el
más probable en este caso?
Atresia esofágica proximal con fístula traqueoesofágica distal
161. ¿Cuál de los siguientes mecanismos ocurre durante la defecación?
- En la posición de “cuclillas” el músculo puborrectal se halla relajado
162. Un paciente luego de un accidente sufre lesión del piso de la boca, se constata daño del nervio
“cuerda del tímpano”, en este caso se esperaría encontrar disminución de la lengua
-Sensación del gusto en los dos tercios anteriores
163. las siguientes afirmaciones es la correcta sobre la gastrina?
- Actúa en la célula diana mediante su receptor CCk tipo B
164. Al recibir un paciente con signos de hipovolemia y antecedente de trauma en abdomen por
accidente de tránsito, usted identifica radiológicamente: lesión de primera vértebra lumbar y signos
de lesión en páncreas; durante la cirugía se observó pobre irrigación de asas intestinales. El vaso
afectado es la arteria ……..
- mesentérica inferior
165. Un paciente sufre de daño a nivel del cuello con lesión muscular en la región de la faringe. En el
examen físico se determina dificultad para la elevación de la faringe y para el cierre del itsmo de las
fauces. En este caso, probablemente esté afectado el músculo:
- Palatofaringeo
166. Varón de 50 años a quien le realizan la curación de la segunda molar de la arcada superior
derecha. En un momento determinado, el paciente acusa de intenso dolor de la pieza dentaria en
tratamiento. La vía aferente del dolor viaja a través del nervio …………
- trigémino V2
167. La distención gástrica por los alimentos produce incremento de secreción de HCl mediante la
producción de que estimula a las células vía proteína
- gastrina / parietal / Gq
168. Un niño de tres años llega a emergencia con disfagia (dificultad para tragar), dolor retro esternal,
salivación y llanto. Se sospecha de ingesta de cuerpo extraño (moneda) en el esófago; al ser
evaluado se constata en una radiografía presencia de cuerpo extraño a nivel de C6 (6° vértebra
cervical). El cuerpo extraño estará suspendido a nivel del estrechamiento producido por………..
- el músculo cricofaríngeo
169. La triada portal (arteria hepática, vena portal y conducto biliar común) está contenida en el
ligamento …….……… y derivan embriológicamente del ……
- hepato duodenal / mesenterio ventral
170. Un paciente refiere no percibir algunos sabores, al examen físico se constata alteración en la
percepción de sabores y del dolor en el tercio posterior de la lengua ¿Cuál de los siguientes nervios
estará alterada en su función?
- Glosofaríngeo (IX par)
171. En el caso de un paciente con gastrinoma (tumor productor de gastrina), la presencia de úlceras
duodenales y erosión de la mucosa gástrica, se debe principalmente a…….
- El exceso de HCl por estímulo de receptores CCK-B en la célula parietal
172. El reflejo enterogástrico se caracterizan por:
- Originarse debido a la distensión duodenal y presencia del quimo ácido
173. Mauricio tiene dificultad para deprimir el paladar y elevar la parte posterior de la lengua. En este
caso estará afectado un músculo, específicamente el músculo…………….
- Extrínseco – palatogloso
174. En condiciones normales, el ingreso de 600 ml de líquido es el estómago provoca un aumento de
presión intragástrica de unos 12 cm de H2O. Después de una vagotomía (corte del nervio vago) es
de esperar que el ingreso del mismo volumen de líquido ocasione ………………………………… de la
presión intragástrica.
- un aumento mayor
175. La explicación fisiológica de presentar somnolencia de 30 minutos a 1 hora después de ingerir
alimentos, se explica por:
- Aumento de la alcalinidad sanguínea
176. Se presenta un paciente, el cual presenta un antecedente de tuberculosis intestinal, por lo cual,
se le resecó 80 cm de íleon distal. Desde el punto de vista fisiológico, el paciente puede presentar
una de las siguientes alteraciones: a. Disminución de la secreción de Vitamina B12
- Disminución de la absorción de ácido glicocólico
177. Un paciente es sometido experimentalmente a un fármaco que modifica el flujo
salival,obteniéndose un volumen de saliva de 288 ml en 6 horas. En este caso las concentraciones
de electrolitos y bicarbonato en la saliva obtenida varían de la siguiente manera:
- ↓ Na+, ↓ Cl-, ↑ K+, ↓ HCO3--
178. Uno de los siguientes elementos debería hallarse con más probabilidad en el esófago de un
paciente que sufre de reflujo gastro esofágico…
- Pepsina
179. Un paciente de 40 años cursa con anemia de 8g/dl, aqueja además de astenia y sensación de
hormigueo bilateral en los miembros inferiores, al examen se halla alteración de la sensibilidad a la
vibración y camina con ampliación de la base de sustentación. Uno de los siguientes procedimientos
sería de ayuda para el diagnóstico de este paciente:
- Biopsia de la mucosa gástrica
180. Paciente de 60 años ingresa por caída hace 1 hora y pequeño hematoma en cuero cabelludo, al
examen físico ampliado se observa ictericia de piel y mucosas generalizada, abdomen blando, se
palpa estructura quística no dolorosa en hipocondrio derecho que corresponde a vesícula biliar (signo
de Courvoisier), en los exámenes de laboratorio se halla niveles bajos en la formación de
estercobilinógeno y urobilinógeno en heces, incremento de la bilirrubina conjugada en la orina,
elevación de fosfatasa alcalina y gamma glutamil transpeptidasa séricas. El presente cuadro puede
ser explicado por
- Carcinoma de la cabeza de páncreas
181. Un recién nacido presenta vómitos biliosos poco tiempo después de cada alimento. Al preguntar
a la madre sobre antecedentes, ella recuerda que tuvo polihidramnios durante la gestación, pero un
análisis de cariotipo fue normal. Una de las siguientes es la causa más probable de estos hallazgos
en el recién nacido:
- Malrotación de la yema pancreática ventral
182. En un estudio de la secreción de hormonas gastrointestinales, sus concentraciones en la vena
porta se midieron durante perfusión luminal del intestino delgado con soluciones de diversas
magnitudes de pH. ¿Qué hormona aumentará en el plasma de la vena porta durante perfusión a
través del intestino con una solución de pH 3?
- secretina
183. Paciente de 30 años que ingresa a causa de un traumatismo abdominal cerrado. En la
exploración se aprecia discreta palidez de piel y mucosas, auscultación pulmonar normal, taquicardia
de 120 /min. Discreta distensión abdominal y matidez en flancos; el hematocrito, que era
prácticamente normal al ingreso, disminuye a 30% a las tres horas. En la Rx de tórax se objetiva
fractura de las costillas 10-11 izquierdas. La causa más probable de la anemización en este paciente
es: a. traumatismo renal con hemorragia retroperitoneal.
- rotura de bazo con hemoperitoneo.
184. Revisando la angiotomografía de un hombre de 70 años en estudio por aneurisma de aorta
abdominal, el radiólogo le informa de la presencia de una oclusión completa de la arteria mesentérica
inferior. El paciente se encuentra completamente asintomático. La oclusión de la arteria mesentérica
inferior cursa de manera asintomática en muchas ocasiones ya que el territorio que irriga puede
recibir flujo proveniente de la arteria:
- cólica media
185. En las patologías de esófago es importante conocer bien la anatomía esofágica. ¿Cuál de las
siguientes afirmaciones es correcta?
- El esófago torácico pasa por detrás del cayado aórtico
186. A pesar de que pueda haber variaciones anatómicas, lo habitual es que el ciego sea irrigado por
una rama arterial que proviene de unas de las siguientes arterias: a. Iliaca derecha
- Mesentérica superior
187. Ante un paciente con una cirugía abdominal urgente, el informe operatorio señala que se ha
realizado una resección de todo el duodeno y del tercio proximal del yeyuno manteniendo íntegros el
estómago y todo el íleon, así como los dos tercios distales del yeyuno. En el seguimiento nutricional
del paciente ¿Qué vitamina o mineral presentará con menor probabilidad una disminución de su
absorción?
- Cianocobalamina
188. ¿Cuál de las siguientes sustancias forma parte de la secreción biliar?
- Lecitina
189. ¿De qué músculo forma parte el ligamento inguinal?
-Oblicuo externo del abdomen
190. ¿Cuál de las siguientes enzimas está localizada en el borde en cepillo y juega un rol en la
digestión de proteínas?
- Carboxipeptidasa A.
191. Una de los siguientes sustancias, NO sirve como un buen agente emulsificante:
- Colesterol
192. La sustancia que estimula el crecimiento de la mucosa gástrica es:
- Gastrina
193. ¿Cuál de las siguientes alternativas es una función de la colecistokinina?
- Secreción de enzimas pancreáticas
194. Con respecto a la anatomía del tronco celiaco, señale lo correcto
- La hepática común que es una de sus ramas, participa en la irrigación del estómago.
195. Con respecto a la anatomía del duodeno, marque la respuesta correcta: a. Tiene una distribución
en forma de “C”, que rodea la cola del páncreas
-La 3ra porción duodenal está contenida en la pinza vascular aortomesentérica
196. En el íleon se absorbe aproximadamente el 95% de la circulación enterohepática.
- Sales biliares
197. La estimula el mecanismo paracrino de la secreción de ácido clorhídrico.
- histamina
198. En la digestión de proteínas,es el principal estímulo para convertir el pepsinógeno en pepsina.
- el pH ácido
199. Con respecto a la somatostatina, marque lo correcto:
- Interviene en la fase intestinal de la secreción gástrica
200. En pecten anal, es una estructura comprendida entre: a. la línea pectínea y los senos anales
- la línea anocutánea y la línea pectínea
201. Se traza un plano a nivel de la mitad de la distancia entre la horquilla esternal y la sínfisis
del pubis, a ese nivel se espera encontrar:
Cuello del páncreas
202. El mesocolon transverso se origina en:
Pared posterior del abdomen
203. El orificio omental, o hiato de Winslow, se encuentra limitado por el ligamento:
hepatoduodenal
204. Se la conoce como el vigilante del abdomen , por su capacidad de desplazarse y
adherirse a cualquier zona inflamada, y envolviendo el órgano para frenar la inflamación:
Omento mayor
205. Marque lo correcto con respecto a lo que sucede al contraerse el diafragma:
Se aplana el diafragma
206. Las glándulas submucosas se encuentran en mayor cantidad a nivel:
proximal a la unión esofagogástrica
207. En cuanto a la distribución del tubo digestivo en la cavidad abdominal, se puede afirmar
que los órganos intraabdominales están suspendidos por:
Mesenterio
208. Al palpar una masa pétrea en hipocondrio derecho, es probable que se trate de un tumor
de:
vesícula biliar
209. Al momento de espirar profundamente, ocurre lo siguiente:
aumento de la presión intraabdominal
210. La estrechez esofágica más proximal es:
cricofaríngeo
211. El peritoneo parietal está separado del peritoneo visceral por la fascia de _______ que
está fijada a la pared posterior del abdomen, y es importante porque por esta vía se pueden
diseminar las infecciones peritoneales:
Todlt
212. Marque la afirmación incorrecta:
los omentos principalmente irrigan órganos intraabdominales
213. Los dobleces de peritoneo que comunican un órgano con la superficie interna de la
pared abdominal posterior antiguamente se denominaban ligamentos , sin embargo,
actualmente se denominan:
Repliegues
214. Señale el órgano del sistema digestivo que no contiene adventicia:
Vesícula biliar
215. Es considerado un órgano sólo inicialmente intraperitoneal:
Páncreas
216. Marque el órgano que se encuentra fuera de la cavidad peritoneal:
recto
217. Para colocarle una sonda de alimentación directamente al estómago (gastrostomía), el
cirujano deberá hacer una incisión en la piel del abdomen inervada por las raíces de:
T8
218. El íleon se encuentra principalmente en:
Cuadrante inferior derecho
219. Tiene inervación somática:
Peritoneo parietal
SISTEMA DIGESTIVO
EXAMEN PARCIAL
201802

Sección: Todas
Profesores: Choque Chávez, Fernando Diego; Damián Bastidas, Narda Lucía; Irribarren Gamarra, Maria Patricia;
Stapleton Valdivia, Mauricio Juan Jose.
Duración: 50 minutos.
Indicaciones:
- Lea atentamente cada pregunta antes de responder.
- Se prohíbe el uso del celular y cualquier dispositivo electrónico.
- Está prohibido intercambiar materiales.
- Coloque su código de alumno en la tarjeta de respuestas. Si su código contiene una letra reemplácela por un valor
numérico siguiendo la siguiente equivalencia: A=9, B=8, C=7, D=6, E=5.
- Traslade sus respuestas a la tarjeta, llenando los círculos de manera completa con lapicero negro o azul. Está
prohibido el llenado con lápiz, lapicero de otro color o con lapicero de tinta borrable.
- Sea cuidadoso en el llenado de la tarjeta de respuestas, pues solo esta tiene validez para la calificación.
- Al terminar su examen avise al docente a cargo, no se levante de su sitio; debe entregar la hoja de respuestas con
la carátula del examen, este cuadernillo de preguntas se lo llevará cada estudiante.

Chorrillos, octubre de 2018

1. En todo el sistema gastrointestinal se encuentra diferentes tipos de músculos ¿En cuales estructuras
encontraríamos músculo estriado?
A. Faringe, esfinter esofágico inferior, colon descendente.
B. Esfínter esofágico superior, laringe, esfínter anal externo.
C. Faringe, esfínter esofágico superior, esfínter anal externo.
D. Esfínter esofágico superior, esfínter esofágico inferior, colon.

2. Durante la fase …..……………….. el paladar blando es traccionado hacia arriba.


A. oral
B. gástrica
C. faríngea
D. esofágica

3. El tubo digestivo a nivel del ………………………….. está revestido internamente por epitelio ………….
A. Esófago / plano simple
B. Estómago / cilíndrico simple
C. Intestino grueso / plano estratificado
D. Intestino delgado / plano de transición

4. Señale la estructura del tubo digestivo que presenta tres capas musculares en su pared.
A. Esófago
B. Estómago
C. Intestino grueso
D. Intestino delgado

5. Paciente de 47 años con sobrepeso, acude a consulta por presentar dolor tipo cólico en el cuadrante superior
derecho del abdomen. El dolor aumenta luego de ingesta de comidas con abundante grasa. Esta situación se explica
por el aumento en la secreción de …………………………. que estimula la contracción de la vesícula biliar.
A. Gastrina
B. Colecistoquinina
C. Péptido inhibidor gástrico
D. Péptido intestinal vasoactivo
6. Si hay un aumenta de la hormona grelina a nivel del hipotálamo ¿Cuál es la consecuencia en el organismo?
A. Hambre
B. Saciedad
C. Aumento de somatostatina
D. Disminución del tránsito intestinal

7. Señale el péptido gastrointestinal que produce relajación del músculo liso gastro-intestinal.
A. Péptido intestinal vasoactivo (PIV)
B. Gastrina vasoactiva (GV)
C. Colecistoquinina (CCK).
D. Acetilcolina (Ach)

8. ¿Cuál de las siguientes alternativas inhibiría la relajación receptiva a nivel del estómago?
A. Histamina
B. Bloqueo del nervio vago (X)
C. Péptido inhibir gástrico (GIP)
D. Péptido intestinal vasoactivo (VIP)

9. Una mujer de 28 años, con diagnóstico de Diabetes Mellitus Tipo 1, acude por presentar desde hace 10 años
estreñimiento y distensión abdominal. Se realiza estudio y se determina que la paciente presenta un retraso del
vaciamiento gástrico debido a gastroparesia diabética. ¿Cuál de los siguientes hechos aumentaría el tiempo del
vaciamiento gástrico?
A. Aumento de gastrina
B. Estimulación parasimpática
C. Ácidos grasos en el duodeno
D. Quimo isotónico en el duodeno

10. Experimentalmente, se aplica un inhibidor selectivo del péptido intestinal vasoactivo (PIV) durante la contracción
peristáltica del intestino delgado. ¿Cuál es el efecto de este inhibidor en la motilidad del intestino delgado?
A. Parálisis del movimiento anterogrado
B. Disminución del tránsito intestinal
C. Aumento del tránsito intestinal
D. Movimiento retrogrado

11. ¿Cuál de los siguientes eventos ocurre durante la defecación?


A. Relajación del esfínter anal externo
B. Relajación del músculo liso del recto
C. Contracción del esfínter anal interno
D. Disminución de la presión intrabdominal

12. Señale el péptido que cumple la función de disminuir la ingesta de alimentos.


A. Neuropéptido Y
B. Ghrelina Gastrica
C. Proteína relacionada a agouti
D. Proopiomelanocortina (POMC)

13. Indique el lugar de secreción del péptido YY


A. Estómago
B. Duodeno
C. Yeyuno
D. Ileon

14. La grelina es secretada en el estómago y estimula a las neuronas del núcleo ………………….. para la estimulación de
la secreción de……………..
A. arqueado / neuropéptido Y.
B. arqueado / melanocortinas.
C. paraventricular / neuropéptido Y.
D. lateral del hipotálamo / hormonas orexigénicas.
15. Considerando el desarrollo embriológico del intestino medio. ¿Qué evento de gran importancia se produce en la
sexta semana?
A. Retracción de asas intestinales primitivas
B. Aparición del primordio hepático y pancreático
C. Inicio de la secreción de insulina por el páncreas
D. Salida temporal de asas intestinales a través de cordón umbilical

16. En el ……………………………. se presentan los movimiento de …………………….


A. esófago / retropropulsión y mezcla
B. estómago / segmentación y ondas lentas
C. intestino grueso / propulsión y ondas en espiga
D. intestino delgado / segmentación y peristaltismo

17. Marita sufre una parálisis del músculo masetero. ¿Qué limitaciones se producirán a nivel del movimiento de la
mandíbula?
A. Lateralización
B. Propulsión
C. Elevación
D. Ninguna

18. Señale el movimiento que se produce a nivel del ciego y colon proximal que tiene como finalidad favorecer la
absorción de agua y sales.
A. De masa
B. De mezcla
C. Propulsivo
D. Peristáltico

19. La enfermedad de Hirchsprung se caracteriza por una dilatación anormal de colon y disminución de los
movimientos propulsivos que traerá como principal consecuencia …………
A. diarrea acuosa.
B. estreñimiento crónico.
C. dilatación de la válvula ileocecal.
D. disminución de la flora intestinal.

20. ¿Cuál de los siguientes factores estimulan el vaciamiento gástrico?


A. Colecistoquinina (CCK)
B. Neuropéptido Y
C. Secretina
D. Gastrina

21. La secreción de ………………………. estimula la motilidad gástrica.


A. Colecistoquinina
B. Secretina
C. Gastrina
D. Motilina

22. La hormona ……………………….. es secretada por las células “I” del …………..
A. Colecistoquinina / duodeno y yeyuno
B. Gastrina / duodeno y yeyuno
C. Colecistoquinina / estómago
D. Gastrina / estómago
GLUT2, SGLT1, GLUT5

CI 1

1. La motilidad intestinal es estimulada principalmente por el:


- Sistema simpático
- Sistema piramidal
- Plexo de Auerbach
- Sistema parasimpático
2. La peristalsis o peristaltismo hace referencia a:
- Motilidad para movilizar el alimento de proximal a distal.
- No es parte de la motilidad
- Motilidad para mezclado de alimentos.
- Motilidad para fraccionamiento de alimentos.
3. Marque el órgano que se encuentra más distal en el tubo digestivo.
- Estomago
- Ciego
- Íleon
- Duodeno
4. Marque la respuesta incorrecta:
- La mucosa consta de epitelio, lámina propia y muscularis mucosae.
- En todo el tubo digestivo, se observa dos capas de muscular propia: circular
interna y longitudinal externa
- El colon contiene tenias
- Fuera de la cavidad abdominal, el esófago presenta capa adventicia.
5. Al iniciar la digestión, aumenta el consumo de oxígeno por la mucosa. Esto conlleva a
una hipoxia local, lo cual hace que se libere _____________, el cual produce
vasodilatación:
- Colecistoquinina
- Adenosina
- Histamina
- Noradrenalina
6. Cuál de las siguientes estructuras no tiene vasos sanguíneos:
- Epitelio intestinal
- Ligamento
- Omento
- Mesenterio
7. Paciente tiene una úlcera sangrante en el segundo tercio del Yeyuno. La arteria de la
cual proviene la sangre arterial para dicha zona es la arteria:
- Mesentérica superior
- Tronco celíaco
- Mesentérica inferior
- Iliaca común
- Gástrica izquierda
8. Paciente de 24 años con dolor abdominal tipo cólico intenso en mesogastrio. Según
sus conocimientos de macroestructura, el origen del dolor puede ser el ___________:
- Íleon
- Colon
- Estómago
- Esófago
9. Dentro de las funciones del abdomen, se encuentra la defecación y micción, en las
cuales la presión intra abdominal debe:
- Aumentar
- No tiene relación el abdomen con dichas funciones
- Mantenerse igual
- Disminuir
10. En la inspiración, la pared abdominal debe ____________ para ____________:
- Contraerse aumentar presión intra abdominal
- Relajarse disminuir presión intra torácica
- Relajarse aumentar presión intra abdominal
- Contraerse aumentar presión intra torácica

CI 2

1. Respecto a los péptidos gastrointestinales, marque lo correcto.


- No existe sustancia neurocrina que tenga efecto en la motilidad del tubo
digestivo
- Las sustancias paracrinas pueden viajar a través de vasos sanguíneos
- Las sustancias neurocrinas son peptidos que hacen su efecto en distancias
cortas
- Las sustancias paracrinas atraviesan la circulación portal
2. Al disminuir el pH duodenal por el HCl gástrico, se libera principalmente una hormona
cuya célula diana es:
- Acinos pancreáticos
- Células ductales del colédoco
- Célula ductal del Wirsung
- Células S del intestino
3. En un paciente con gastroparesia (motilidad lenta del estómago), que presenta
distensión abdominal después de comer, usted le recomendaría que evite el consumo
de lípidos y aminoácidos para disminuir la acción de:
- Secretina
- CCK
- Somatostatina
- Gastrina
4. La razón por la que el potencial de acción viaja rápidamente en sentido longitudinal
por el musculo liso gastrointestinal es la presencia de uniones en hendidura, además
de la presencia de:
- La presencia del plexo submucoso de Meissner
- Varicosidades
- Mayor cantidad de ACh
- Las fibras musculares no se disponen en haces musculares
5. Paciente obeso con Covid-19 es intubado por interno inexperto, quien al solicitar que
bombeen aire dentro del tubo endotraqueal, nota que el epigastrio se distiende. Al
sospechar que ha introducido el tubo en el estómago, también es cierto que:
- Disminuiría el tono del píloro
- Aumenta la frecuencia de ondas lentas
- Aumenta el pH gástrico
- Disminuye el pH gástrico
6. En un paciente con diarrea por hipermotilidad, usted sospecharía en el posible
aumento de las siguientes sustancias, excepto:
- Sustancia P
- ACh
- Péptido intestinal vasoactivo (VIP)
- Motilina
7. El ecografista sabe que para poder visualizar el nacimiento de la arteria mesentérica
superior, debe colocar el transductor sobre la piel de la siguiente región abdominal:
- Epigastrio
- Hipocondrio derecho
- Hipogastrio
- Mesogastrio
8. Al consumir un pan con mantequilla, la sensación de hambre disminuye debido a la
acción de:
- Grelina
- Somatostatina
- Colecistoquinina (CCK)
- Leptina
9. Una de las siguientes sustancias no comparte con las otras la misma acción sobre la
producción de ácido gástrico:
- Péptido insulinotrópico dependiente de glucosa (GIP)
- Colecistoquinina
- Somatostatina
- Secretina
10. Al ingerir grandes cantidades de dulces, con la subsecuente estimulación de incretinas,
usted esperaría que el apetito ______________, debido a __________________
- disminuya insulina
- aumente CCK
- disminuya CCK
- aumente grelina

PARCIAL

1. Al rozar agua caliente en la punta de la lengua, usted esperaría que el estímulo viaje a
través del nervio:
- lingual
- cuerda del tímpano
- glosofaríngeo
- hipogloso
2. Al ingresar líquidos o sólidos en la cavidad oral, un mecanismo que permite que una
persona respire mientras mastica es:
- el movimiento hacia afuera de los pliegues palatogloso y palatofaríngeo
- la elevación del paladar blando
- la depresión de la parte posterior de la lengua
- la depresión del paladar blando
3. Paciente adulto con reflujo gastroesofágico es más probable que presente:
- descalcificación del esmalte
- destrucción de ameloblastos
- remodelación del esmalte
- desfluorización de los dientes
4. La masticación es básicamente:
- importante para la digestión sobretodo de carnes
- un ralentizador del vaciamiento gástrico
- una actividad consciente
- un movimiento reflejo
5. Paciente de 34 años es víctima de asalto con arma de fuego, recibiendo un impacto
directo en el abdomen. En base a la radiografía, usted puede registrar en la historia
clínica que el proyectil se encuentra topográficamente en el:

- flanco derecho
- mesogastrio
- flanco izquierdo
- hipocondrio izquierdo
6. Señale cuál de las estructuras que en el embrión se encuentra comunicada con el saco
vitelino por medio del conducto onfalomesentérico:
- D
- B
- C
- A
7. En un paciente de 3 semanas de edad, con vómitos en proyectil, y nódulo epigástrico
reptante, usted esperaría encontrar:
- colecistoquinina aumentada
- vómitos biliosos e intolerancia a los ácidos grasos
- distensibilidad disminuida de la región oral del estómago
- engrosamiento de la circular interna pilórica
8. ¿Cuál de las siguientes estructuras tiene inervación somática?
- Estómago
- Peritoneo visceral
- Mesosigmoides
- Peritoneo parietal
9. Estas diseñando un proyecto de investigación sobre los niveles de colesterol que se
absorben luego de una comida grasosa y deseas cuantificar la cantidad de colesterol
que es absorbido por el intestino antes que el hígado lo metabolice ¿de cuál de los
siguientes vasos obtendrías la muestra para tu análisis?
- Vena porta
- Vena cava superior
- Conducto torácico
- Vena hemiácigos accesoria
10. Durante el paso del bolo hacia la orofaringe, se desencadena una serie de
contracciones musculares que estrechan la cavidad faríngea. Estas contracciones están
mediadas por el nervio craneal:
- X
- XI
- XII
- IX
11. Cuando el istmo de las fauces se cierra, se evita que el alimento pase hacia la
orofarínge y permite respirar mientras se mastica. Este cierre se debe a la contracción
y aproximación de los músculos:
- Palatoglosos
- Palatofaríngeos
- Estiloglosos
- Estilofaríngeos
12. Las siguientes alternativas son factores que determinan la patencia y función adecuada
del esfínter esofágico inferior, EXCEPTO:
- Plicatura diafragmática
- Hipertrofia de la circular interna
- Angulación con el estómago
- Canales lentos de calcio
13. Respecto al peristaltismo intestinal, para cumplir la ley del intestino , usted espera que
a nivel distal del quimo se libere:
- péptido liberador de gastrina (GRP)
- acetilcolina
- péptido intestinal vasoactivo
- sustancia P
14. El estímulo habitual para el movimiento peristáltico es:
- acción de la sustancia P
- contracción de la musculatura circular Interna
- estimulación vago-vagal
- distensión local
15. ¿Cuál de las siguientes condiciones considera que es un trastorno de la musculatura
lisa esofágica?
- Acalasia
- Asinergia faringoesfinteriana
- Hipotonía de los constrictores faríngeos
- Hipertonía del esfínter esofágico superior
16. ¿Cuál de las siguientes alternativas es correcta sobre la motilidad esofágica?
- Las ondas primarias son propulsoras y pueden no ser precedidas por deglución
- Las ondas primarias no son propulsoras y siempre van precedidas de deglución
- Las ondas secundarias son propulsoras y siempre van precedidas de deglución
- Las ondas secundarias son propulsoras y no van precedidas de deglución
17. El peristaltismo depende que a nivel distal del bolo se secrete:
- noradrenalina secretada por las fibras del sistema simpático
- acetilcolina por las neuronas provenientes del nervio vago
- óxido nítrico por células endoteliales locales
- péptido intestinal vasoactivo por neuronas
18. ¿Cuál de las siguientes alternativas es correcta sobre el movimiento peristáltico?
- Es un reflejo largo que depende de la integración con el tronco encefálico
- Se dirige en sentido distal siempre, nunca en sentido proximal
- El contenido intestinal avanza sólo 5-10 cm
- Es independiente del plexo mientérico
19. En un estudiante de medicina que está rindiendo un examen parcial, lo más probable
es que en ese momento su tránsito intestinal se encuentre:
- muy lento
- muy acelerado
- estimulado por acción de la sustancia P
- sin alteraciones
20. Las siguientes alternativas son ciertas sobre las contracciones tónicas del músculo
gastrointestinal, EXCEPTO:
- Tienen relación con el ingreso persistente de iones sodio
- Se encuentran principalmente en esfínteres
- Tienen regulación hormonal
- Obedece a una mayor frecuencia de potenciales en espiga
21. Con respecto de la regulación del pH del estómago; al utilizar un bloqueador de
histamina, usted espera que el pH del estómago:
- aumente
- disminuya
- se mantenga sin cambio
- se neutralice por acción de bicarbonato
22. Las siguientes hormonas disminuyen el vaciamiento gástrico, EXCEPTO:
- Gastrina
- Péptido insulinotrópico dependiente de glucosa
- Colecistoquinina
- Secretina
23. El frenillo de los labios se encuentra en:
- el piso de la boca
- la cavidad oral
- la cavidad vestibular
- el dorso de la lengua
24. En un paciente con shock hipovolémico, la peristalsis intestinal se encuentra:
- aumentada
- sin cambios
- invertida
- disminuida
25. Sobre el control de la peristalsis del tubo digestivo, ________________ es un
mediador neural que induce la relajación durante la peristalsis.
- la somatostatina
- el péptido intestinal vasoactivo
- la acetilcolina
- la serotonina

CI 3

1. En cuanto a las sustancias secretadas por el estómago ¿Cuál de las siguientes


sustancias estimula la liberación de pepsinógeno?
- Pepsinógeno
- Secretina
- Colecistoqunina
- Gastrina
2. Las células enteroendocrinas en el estómago se localizan en la glándula oxíntica, al
mismo nivel que las células:
- parietales
- principales
- mucosas
- absortivas
3. La fase intestinal de la secreción gástrica se debe básicamente a la participación de las
células:
- G del duodeno
- I del yeyuno
- S del íleon
- D del estómago
4. El conducto de Stenon, para entrar a la cavidad vestibular, debe atravesar el músculo:
- genihioideo
- buccinador
- milohioideo
- masetero
5. Paciente con acalasia es sometido a tratamiento endoscópico o quirúrgico, usted le ha
informado al paciente previamente que es posible que una complicación de este
tratamiento es que quede con cierto grado de:
- reflujo gastroesofágico
- gastritis
- odinofagia
- úlceras gástricas
6. La sangre que lleva la vena porta es tipo:
- mixta
- arterial
- venosa
7. Los pliegues gástricos gruesos son prácticamente inexistentes a nivel de:
- el cuerpo
- el fondo
- la incisura angularis
- el antro
8. En la producción de HCl, la acción de la somatostatina disminuye la accion de:
- las prostaglandinas
- la histamina
- la acetilcolina
- la gastrina
9. En el estómago se secretan las siguientes sustancias, EXCEPTO:
- grelina
- Correcto
- motilina
- somatostatina
- gastrina
10. La vena porta se forma gracias a la unión de la vena mesentérica superior con la vena:
- mesentérica inferior
- esplénica
- celiaca
- gástrica izquierda

CI 4

1. En el hígado, el aumento de la resistencia vascular en los sinusoides hepáticos


ocasionará:
- aumento de la presión de llenado vesicular
- salida de plasma hacia el intersticio
- aumento del flujo hacia la vena porta
- aumento del flujo hacia la vena cava superior
2. En un paciente con carcinoma de páncreas, el tumor ha invadido la unión entre la
venas esplénica y mesentérica superior; eso quiere decir que estamos seguros que el
tumor se encuentra a nivel del ________ del páncreas.
- cuerpo
- cola
- cabeza
- cuello
3. En un paciente con cirrosis hepática la cabeza de medusa que aparece en la pared
abdominal, podría desaparecer si al paciente se le:
- esclerosa las venas hemorroides internas
- oblitera el ligamento redondo
- administra antiandrógenos
- oblitera la arteria gástrica izquierda
4. En un paciente con intoxicación por órganos fosforados, la acción de la
colecistoquinina (CCK) está bloqueada a nivel de:
- el esfínter de Oddi
- la vesícula biliar
- la célula parietal
- el sistema nervioso central
5. Con respecto a la microestructura del hígado ¿Cuál de las siguientes alternativas es
correcta?
- La célula de Ito se encuentra en el espacio de Disse y reserva glucógeno
- Los hepatocitos están interconectados por uniones herméticas
- La célula de Kupffer se encuentra fuera del sinusoide y fagocita células
- El sinusoide es un capilar fenestrado
6. Paciente con Lupus Eritematosos que desarrolla hipertensión portal debido a
trombosis portal, es probable que desarrolle várices a nivel de:
- recto inferior
- recto superior
- hemorroides externas
- canal anal
7. El efecto de un medicamento colerético se evidencia por:
- la mayor producción de colesterol en la bilis
- la disminución de absorción de sales biliares
- el aumento de secreción biliar
- el aumento de formación de micelas
8. En un paciente con cáncer de páncreas y que desarrolla ictericia, la localización más
probable del tumor es en:
- la vesícula por metástasis
- el cuello del páncreas
- la cabeza del páncreas
- la cola del páncreas
9. En un recién nacido menor de 24 horas con atresia biliar, se encuentra elevación de la:
- bilirrubina directa
- bilirrubina indirecta
- alanina aminotransferasa (ALT)
- hemoglobina
10. ¿Cuál de las siguientes alternativas es correcta sobre la estructura hepática?
- Los colangiocitos producen bilis
- En la triada portal, se encuentra la vena derivada de la suprahepática
- El flujo sinusoidal en el lobulillo hepático es de adentro hacia afuera
- El flujo biliar en el lobulillo hepático es centrífugo
EXAMEN PARCIAL SISTEMA DIGESTIVO
(ME154)
Ciclo 202000

Sección:Todas
Profesores:Alva Muñoz, Jose Carlos
Duración:35 minutos.
Indicaciones:
- Lea atentamente cada pregunta antes de responder.
- Se prohíbe el uso del celular y cualquier dispositivo electrónico.
- Está prohibido intercambiar materiales.
- Coloque su código de alumno en la tarjeta de respuestas. Si su código contiene una letra
reemplácela por un valor numérico siguiendo la siguiente equivalencia: A=9, B=8, C=7, D=6, E=5, F=4
y G=3.
- Traslade sus respuestas a la tarjeta, llenando los círculos de manera completa con lapicero negro o
azul. Está prohibido el llenado con lápiz, lapicero de otro color o con lapicero de tinta borrable.
- Sea cuidadoso en el llenado de la tarjeta de respuestas, pues solo esta tiene validez para la
calificación.
- Al terminar su examen avise al docente a cargo, no se levante de su sitio; debe entregar la hoja de
respuestas con la carátula del examen, este cuadernillo de preguntas se lo llevará cada estudiante.

1. Paciente de sexo masculino de 82 años de edad ingresa a emergencia con dolor abdominal agudo y
diarreas. Se le realiza una arteriografía en la que se observa que la arteria aorta tiene un trombo
ocluyendo el 95% del flujo, a nivel del nacimiento de la arteria mesentérica inferior. ¿Cuál de las
siguientes arterias podría contribuir a la irrigación colateral del colon descendente?
a) cólica media
b) sigmoidea
c) rectal superior
d) ileocólica

2. Niño de 5 años presenta dolor esofágico y hematemesis (vómitos hemorrágicos) luego de tragarse
una espina de pescado. En la endoscopía se observa perforación del esófago distal a la cuarta
estrechez esofágica. ¿Las ramas de cuál de las siguientes arterias estarán lesionada con mayor
probabilidad?
a) Gástrica izquierda
b) Bronquiales
c) Frénica inferior
d) Tiroidea inferior

3. Al ingerir una cucharada de mantequilla es muy probable que se disminuya la sensación de hambre
por medio de la activación de la vía POMC/CART (POMC=proopiomelanocortina y
CART=transcripción regulada de cocaína y anfetamina), activada directamente por la hormona:
a) colecistoquinina (CCK)
b) insulina
c) grelina
d) secretina

4. Al comer unas papitas fritas con mayonesa, el vaciamiento gástrico disminuye por efecto directo
de la hormona: a)colecistoquinina (CCK)
b) bombesina
c) motilina
d) gastrina

5. Recién nacido de dos horas es diagnosticado de hernia umbilical de 1,5 cm de diámetro; el


cirujano pediatra solicita una tomografía abdominal en donde se evidencia que la hernia
umbilical está ocupada por una porción del tracto gastrointestinal. ¿Qué porción del tracto
gastrointestinal estaría ocupando esta hernia con mayor probabilidad?
a) Íleon
b) Colon sigmoides
c) Duodeno
d) Colon transverso
6. Recién nacido de 7 horas, de parto por cesárea debido a polihidramnios (aumento del volumen del
líquido amniótico), con regurgitación de la leche materna y artificial, y no ha presentado meconio.
Se le realiza una tomografía donde se evidencia aire en el estómago y una malformación del
desarrollo esofágico. Con respecto a esta malformación lo más probable es que se pueda tratar de
una atresia esofágica:
a) proximal con fístula traqueoesofágica distal
b) distal con fístula traqueoesofágica proximal
c) proximal y distal
d) sin fístula

7. Lactante de 6 meses de edad que es traído a consulta por presentar vómitos no biliosos a repetición
y retraso en el crecimiento. En la radiografía de abdomen simple se observa nivel hidroaéreo en
estómago y en primera porción de duodeno (doble burbuja). ¿Cuál de las siguientes alternativas
puede explicar la condición del lactante?
a) Páncreas anular
b) Atresia duodenal en la tercera porción
c) Atresia yeyunal
d) Hipertrofia del píloro

8. En ausencia o deficiencia de la secreción de la hormona motilina,


se producirá: a)sobrecrecimiento bacteriano
b) diarrea
c) aumento del vaciamiento gástrico
d) hipertrofia del píloro

9. La estimulación parasimpática aumenta la motilidad intestinal, mientras que la estimulación


simpática la disminuye. ¿Sobre cuál de las siguientes alternativas el sistema nervioso autónomo
actúa para el control de la motilidad intestinal?
a) Potencial de membrana en el plexo mientérico (de Auerbach)
b) Frecuencia de ondas lentas
c) Secreción de secretina
d) Nivel de IP3 en el plexo submucoso (de Meissner)

10. En un recién nacido con protrusión de contenidos abdominales y cubiertas por amnios o peritoneo,
es cierto que: a)Se presenta por un defecto en el cierre de la pared
b) Se acompaña de otras malformaciones congénitas
c) Se debe al no retorno de la hernia fisiológica
d) Se produce a través del ombligo

11. Lactante de 20 días con estreñimiento, distención abdominal progresiva, acompañada


ocasionalmente de vómitos biliosos. Como antecedente, el meconio lo eliminó por primera vez a las
72 horas de nacido. Su mamá menciona que ayuda a la evacuación con ayuda de un termómetro
rectal. Se sospecha de megacolon agangliónico (Enfermedad de Hirschsprung). ¿Cuál de las
siguientes alternativas explica el caso?
a) Se presenta contracciones tónicas en la región ano rectal
b) Se presenta dilatación de tracto gastrointestinal afectado
c) Las células ganglionares sólo han migrado al ano recto
d) La zona que más se afecta es inervada por fibras del nervio esplácnico menor

12. ¿Cuál de los siguientes reflejos disminuye el tránsito


gastrointestinal?
a)Doloroso
b) Gastrocólico
c) De defecación
d) Colicoileal

13. Al ingerir una sustancia ácida como el vino (pH 3), se estimula la motilidad gástrica por acción
de la hormona:
a)motilina
b) secretina
c) colecistoquinina (CCK)
d) bombesina
14. Paciente de 24 años acude a consulta externa por presentar una fístula oronasal (comunicación
entre la cavidad oral y la cavidad nasal). Está fístula está asociada al antecedente de haber sido
operada de paladar hendido a los dos años de edad, durante una campaña gratuita extranjera de
corrección de paladar fisurado. ¿Cuál de las arterias palatinas podría haberse lesionado durante
esa cirugía?
a) Mayor
b) Menor
c) Ascendente
d) Rama palatina de la faríngea ascendente

15. Paciente de sexo masculino de 52 años con úlcera péptica gástrica de 14 años de evolución, con
cuadro de hemorragia digestiva alta hace 4 meses, sin cicatrización de la úlcera. Entre las opciones
quirúrgicas se considera realizarle un vaguectomía troncal (sección del nervio vago) a nivel del
hiato esofágico. ¿Cuál de las siguientes complicaciones podría esperarse producto de la pérdida de
inervación parasimpática?
a) Menor inervación del colon ascendente
b) Se perderá el reflejo de defecación
c) Se perderá el reflejo de micción
d) Impotencia sexual

16. Paciente de 23 años con bulimia es traída a la emergencia deshidratada, semiconsciente y con
alcalosis metabólica. Los vómitos autoinfligidos por esta paciente se producen por estimulación
de receptores en la base de la lengua que mandan información directamente al:
a) núcleo del tracto solitario
b) centro del vómito en el tallo encefálico
c) zona quimiorreceptora gatillo
d) cerebelo

17. Niño de 3 años es traído a emergencia por madre quien manifiesta que hace 10 horas deglutió una
pila pequeña de reloj de bordes romos. El niño está asintomático. Usted la tranquiliza diciéndole es
un cuerpo extraño tan pequeño de seguro que va a seguir el tránsito intestinal como lo haría un
bolo alimenticio, y que lo más probable es que en ese momento se encuentre en:
a) colon
b) estómago
c) yeyuno
d) recto

18. Los movimientos en masa son un tipo de movimiento muy importante, una de las
consecuencias de estos movimientos es:
a) la distensión rectal
b) el peristaltismo del intestino delgado
c) la retropulsión gástrica
d) la contracción del esfínter anal interno

19. Durante la deglución, al momento que el bolo alimenticio pasa por el esfínter esofágico superior, se
espera que la presión intraesofágica:
a) disminuya en el cardias
b) disminuya en el tercio medio del esófago
c) aumente en la porción distal al bolo
d) aumente en el tercio medio del esófago

20. Paciente con enfermedad de Chagas que presenta disfagia a sólidos. ¿Cuál de las siguientes puede
ser la causa de esta complicación?
a) Disminución de células ganglionares en el esfínter esofágico inferior
b) Aumento en la liberación de óxido nítrico en el esfínter esofágico inferior
c) Disminución de las neuronas que liberan péptido intestinal vasoactivo
d) Aumento de la actividad de la motilina en el esófago distal
EXAMEN PARCIAL 2020-02

Pregunta 1

Niña de 6 años se asusta por que se le ha aflojado un diente deciduo. Este fenómenose produce
por:

laxitud del ligamento periodóntico

fractura del cemento

desmineralización del esmalte dental

aumento anómalo de la predentina

2.- ¿Cuál de las siguientes estructuras deriva del intestino anterior?


B

3.- En un paciente de 3 semanas de edad, con vómitos en proyectil, y nódulo epigástrico reptante,
usted esperaría encontrar:

engrosamiento de la circular interna pilórica

distensibilidad disminuida de la región oral del estómago

colecistoquinina aumentada

vómitos biliosos e intolerancia a los ácidos grasos

4.- Sobre el control de la peristalsis del tubo digestivo, ________________ es un mediador neural
que induce la relajación durante la peristalsis.

el péptido intestinal vasoactivo

la serotonina

la acetilcolina

la somatostatina

5.- En el plexo mientérico, el origen de los impulsos eferentes está en:

los ganglios paravertebrales

las células intersticiales de Cajal

el plexo de Meissner

el plexo de Aurbach
6.- Un familiar le comenta que tiene úlcera gástrica por exceso de producción de ácido; con sus
conocimientos del sistema digestivo, usted le recomendaría que reduzca el consumo de:

vitaminas

carbohidratos

agua

aminoácidos

7.- Las siguientes alternativas son ciertas en relación al mesenterio, EXCEPTO:

El omento menor deriva del mesenterio ventral

El ligamento esplenorrenal une al bazo con el riñón izquierdo

Los mesenterios cumplen la función de sostén y suspensión de órganos

Los omentos cumplen una función principal de irrigación visceral

8.- ¿Cuál de las siguientes alternativas es una característica de la estructura del esófago?

Aumenta la presión intra esofágica durante la inspiración

Contiene músculo estriado en casi toda su longitud

Contiene glándulas submucosas principalmente en su tercio distal

El esfínter esofágico inferior es un esfínter anatómico

9.- En una cirugía abierta (laparotomía), el cirujano al abrir la cavidad peritoneal por la parte anterior
(línea media), lo primero que observa es:

Estomago

Colon sigmoides

Duodeno

Epiplón mayor
10.- El tubo digestivo posee glándulas, las glándulas submucosas se encuentran en el:

duodeno y recto

íleon y esófago

estómago y duodeno

esófago y duodeno

11.- ¿Cuál de las siguientes alternativas detallan los músculos que ayudan a empujar el bolo hacia
la orofaringe?

Geniogloso y palatofaríngeo

Estilogloso y palatogloso

Hiogloso y geniogloso

Estilogloso y geniogloso

12.- Al ingresar líquidos o sólidos en la cavidad oral, un mecanismo que permite que una persona
respire mientras mastica es:

la elevación del paladar blando

el movimiento hacia afuera de los pliegues palatogloso y palatofaríngeo

la depresión de la parte posterior de la lengua

la depresión del paladar blando

13.- Durante el paso del bolo hacia la orofaringe, se desencadena una serie de contracciones
musculares que estrechan la cavidad faríngea. Estas contracciones están mediadas por el nervio
craneal:

IX

XI

XII
14.- Las siguientes alternativas son factores que determinan la patencia y función adecuada del
esfínter esofágico inferior, EXCEPTO:

Canales lentos de calcio

Plicatura diafragmática

Hipertrofia de la circular interna

Angulación con el estómago

15.- Durante una cena, una gestante inspira por la boca profundamente de manera frecuente; sin
embargo, el organismo evita que el aire ingrese al esófago por la acción:

del músculo cricofaríngeo

del istmo de las fauces

del esfínter esofágico inferior

de la epiglotis

16.- En un estudiante de medicina que está rindiendo un examen parcial, lo más probable es que en
ese momento su tránsito intestinal se encuentre:

muy lento

estimulado por acción de la sustancia P

muy acelerado

sin alteraciones

17.- El principal gobernante sobre todos los movimientos gastrointestinales es el sistema nervioso:
parasimpático

somático

mientérico

simpático
18.- ¿Cuál de las siguientes condiciones considera que es un trastorno de la musculatura lisa
esofágica?

Hipotonía de los constrictores faríngeos

Hipertonía del esfínter esofágico superior

Asinergia faringoesfinteriana

Acalasia

19.- El estímulo habitual para el movimiento peristáltico es:


acción de la sustancia P

contracción de la musculatura circular Interna

estimulación vago-vagal

distensión local

20.- La hormona responsable de los complejos migratorios interdigestivos tiene las siguientes
características, EXCEPTO:

se libera de forma cíclica

se produce en el estómago y el duodeno

cumple funciones de aumentar la motilidad y secreción gástrica e intestinal

es inhibida por el alimento

21.- Respecto al peristaltismo intestinal, para cumplir la ley del intestino , usted espera que a nivel
distal del quimo se libere:

acetilcolina

péptido liberador de gastrina (GRP)

sustancia P

péptido intestinal vasoactivo


22.- ¿Cuál de las siguientes alternativas es correcta sobre el control autónomo del aparato
gastrointestinal?

El plexo mientérico de Auerbach cumple funciones inhibitorias

Las terminaciones nerviosas simpáticas liberan mayor cantidad de adrenalina que noradrenalina

La estimulación simpática estimula a la muscularis mucosae

Al seccionar el vago, la inervación parasimpática del colon sigmoides disminuye

23.- La distención del yeyuno provoca que se:

despolarice el potencial de reposo de membrana

produzca una contracción tónica

disminuya el número de espigas

aumente la frecuencia de las ondas lentas

24.- El frenillo de los labios se encuentra en:

el dorso de la lengua

el piso de la boca

la cavidad oral

la cavidad vestibular

25.- Con seguridad, usted puede decir que la siguiente cicatriz postapendicectomía se encuentra en
la región denominada:

flanco derecho

hipocondrio derecho

flanco izquierdo

fosa iliaca derecha


CLAVES EXAMEN PARCIAL DE SISTEMA
DIGESTIVO 2019 - 00

1. Varón de 30 años es traído a emergencia por agresión abdominal con arma de fuego
(pistola) y es sometido a laparotomía exploratoria, observándose isquemia del colon
ascendente y parte del colon trasverso ¿la lesión de cuál de las siguientes arterias
explicaría esta isquemia?
(unidad 1, sesión 2, logro 6: (Describe la irrigación visceral: arterias de tronco
celiaco, arteria mesentérica superior e inferior, topografía de superficie, órganos
por cuadrante)

a. Celiaca
b. Colónica derecha
c. Mesentérica inferior
d. Mesentérica superior

2. Respecto a las sustancias gastrointestinales que regulan la secreción pancreática;


marque la afirmación correcta:
(unidad 1, sesión 3, logros 2 y 3: describir las hormonas gastrointestinales: estímulos y
funciones)

a. La Secretina, es la hormona más importante para la secreción de bicarbonato por


las células acinares del páncreas
b. La acetilcolina es capaz de estimular la secreción enzimática y de bicarbonato del páncreas
c. La gastrina, es la hormona más importante para la secreción de enzimas pancreáticas
d. La colecistoquinina (CCK) estimula al páncreas solo para secreción enzimática

3. Ante una lesión del X par craneal, ¿cuál de los siguientes músculos mantiene conservada su
función?:
(unidad 2, sesión 08, logro 4: Paladar blando: componentes musculares)

a. Elevador del velo del paladar


b. Tensor del velo del paladar
c. Palatofaríngeo
d. Glosofaríngeo

4. Experimentalmente se utiliza atropina (anticolinérgico) para inhibir la secreción de


gastrina, sin embargo la secreción de esta hormona se sigue dando ante estímulos
vagales. Esta situación se explica porque la atropina:
(unidad 1, sesión 3, logro 3 : describir las hormonas gastrointestinales: estímulo y funciones
de la gastrina y colecistoquinina)

a. Bloquea parcialmente la bomba de protones en la célula G


b. Inhibe la acción de acetilcolina e histamina en la célula G
c. Solo inhibe la acción del péptido GRP en la célula G
d. No bloquea la acción del péptido GRP

5. Un varón de 50 años es sometido a extirpación del duodeno y parte proximal


del yeyuno. Esta situación ocasionaría la pérdida de las células ……….. ,
productoras deque estimula la secreción de
bicarbonato por el páncreas.
(unidad 1, sesión 3, logro 3: describir las hormonas gastrointestianles: estímulos y
funciones de la secretina y péptido insulinotrópico dependiente de glucosa)

a. “S” / secretina
b. Parietales / secretina
c. “I” / colecistoquinina
d. “S” / colecistoquinina
6. Recién nacido que presenta tumoración abdominal a nivel del cordón umbilical
(fotografía). ¿cuál de las siguientes afirmaciones es correcta respecto a este
defecto en el desarrollo embriológico del intestino?: (unidad 1, sesión 5, logro 2:
identificar las anomalías del desarrollo del intestino medio)

a. Corresponde a una Gastrosquisis


b. Las vísceras se hallan cubiertas por piel
c. No está asociado a otras malformaciones
d. Se asocia a
malformaciones
cardiacas y del tubo
neural

7. Varón de 35 años acude a la emergencia por


trauma abdominal y se decida realizar una
laparoscopía exploratoria. El cirujano
observa la disposición de los órganos
abdominales como se representa en el
siguiente esquema. Esta disposición de
órganos se explica por la
rotación(SMA=arteria mesentérica superior)
(unidad 1, sesión 5, logro 3: identificar las anomalías
del desarrollo del intestino medio: defectos de
rotación, estenosis y atresias)

a. anti horaria del intestino medio, en sólo 90°


b. incompleta del intestino medio (270°)
c. horaria del intestino medio
d. horaria del estómago

8. Se evalúa la expresión de la proteína Agrp en una persona con alteración del


apetito; lo correcto respecto a esta proteína es…..
(unidad 1, sesión 3, logro 4: Explica los mecanismos de control del apetito y saciedad )

a. Esta proteína es un potente anorexigénico


b. La mutación del gen que la codifica produce adelgazamiento
c. La sobre producción de la proteína lleva a obesidad por agonismo de receptores MC3 y MC4
d. La sobre producción de la proteína disminuye el apetito por antagonismo de receptores MC4

9. Juana cae de la bicicleta y se fractura la región anterior del hueso maxilar superior con
compromiso de la fosa incisiva. Al examen físico de la región esperaría encontrar
alteración en la sensibilidad de la encía …………………
(unidad 2, sesión 8, logro5: paladar: paladar duro y blando: irrigación e inervación)

a. bucal posterior
b. Lingual anterior
c. palatina anterior
d. palatina posterior
10. Recién nacido es atendido por el neonatólogo y luego entregado a su madre para
dar de lactar; la madre al dar de lactar observa coloración azulada de labios,
acompañado de tos persistente, dificultad respiratoria y distención abdominal. Se le
intenta colocar una sonda nasogástrica pero esta retorna a la cavidad oral en todos
los intentos. ¿Cuál de las siguientes anomalías del desarrollo es el más probable en
este caso? (unidad 1, sesión 4, logro 3: identificar las anomalías en el desarrollo del
esófago: atresia y/o fístula traqueo esofágica)

a. Estenosis esofágica proximal con Fístula traqueo esofágica distal


b. Atresia esofágica proximal con fístula traqueoesofágica distal
c. Atresia esofágica distal con fístula traqueoesofágica proximal
d. Fístula traqueoesofágica proximal y distal

11. ¿Cuál de los siguientes mecanismos ocurre durante la defecación?


(unidad 2, sesión 13, logro 6: motilidad del intestino grueso: contracciones
segmentarias, movimientos en masa, defecación y reflejo gastrocólico)

a. Contracción refleja del esfínter anal interno


b. En la posición de “cuclillas” el músculo puborectal se halla relajado
c. Relajación del esfínter anal externo por efectos del VIP y óxido nítrico
d. La materia fecal en el recto estimula la contracción del sigmoides por los nervios pudendos

12. La estructura número 4 (gráfico) corresponde a


……….… y está ………..
(unidad 2, sesión 9, logro 2:
Partes de un diente. Capas
del diente: Esmalte:
características y células que
lo producen)

a. el cemento / mineralizado en 90%


b. la dentina / formada por ameloblastos
c. el esmalte / formado
por células derivadas
del mesénquima
d. la dentina / formado por
células derivadas de la
cresta neural

13. Un paciente luego de un accidente sufre lesión del piso de la boca, se constata
daño del nervio “cuerda del tímpano”, en este caso se esperaría encontrar
disminución de lade la lengua
(unidad 2, sesión 10, logro 3: Irrigación e inervación de la lengua)

a. Motilidad en los dos tercios anteriores


b. Sensación del gusto en el tercio posterior
c. Sensación del gusto en los dos tercios anteriores
d. Sensibilidad al tacto en los dos tercios anteriores

14. ¿Cuál de las siguientes afirmaciones es la correcta sobre la gastrina?


(unidad 1, sesión 3, logro 1: reconocer las características de las sustancias
reguladoras gastrointestinales: hormonas, sustancias paracrinas y neurocrinas)

a. Produce atrofia de la mucosa gástrica


b. Es producida por la célula G del cuerpo gástrico
c. Es estimulada por la distensión gástrica y el Ph bajo
d. Actúa en la célula diana mediante su receptor CCk tipo B
15. Al recibir un paciente con signos de hipovolemia y antecedente de trauma en
abdomen por accidente de tránsito, usted identifica radiológicamente: lesión de
primera vértebra lumbar y signos de lesión en páncreas; durante la cirugía se observó
pobre irrigación de asas intestinales. El vaso afectado es la arteria ……..
(unidad 1, sesión 1, logro 6: reconocer las estructuras a nivel de L1, nivel de los principales
vasos sanguíneos)

a. esplénica
b. hepática común
c. mesentérica inferior
d. mesentérica superior

16. Un paciente sufre de daño a nivel del cuello con lesión muscular en la región de la
faringe. En el examen físico se determina dificultad para la elevación de la faringe y
para el cierre del itsmo de las fauces. En este caso, probablemente esté afectado el
músculo:
(unidad 2, sesión 11, logro 2: Músculos de la faringe: identificación, constrictores y
longitudinales)

a. palatogloso
b. estilofarinfeo
c. palatofaringeo
d. constrictor inferior

17. Varón de 50 años a quien le realizan la curación de la segunda molar de la arcada


superior derecha. En un momento determinado, el paciente acusa de intenso
dolor de la pieza dentaria en tratamiento. La vía aferente del dolor viaja a través
del nervio …………
(unidad 2, sesión 9, logro 6: Inervación de los dientes)

a. trigémino V2
b. trigémino V3
c. naso palatino
d. palatino menor

18. La distención gástrica por los alimentos produce incremento de secreción de HCl mediante la
producción de
………….. que estimula a las células ……………. vía proteína ………..
(Unidad 1, sesión 3, logro 2: Describe las hormonas gastrointestinales: Estímulo y funciones de
la gastrina y colecistoquinina)

a. gastrina / parietal / Gq
b. gastrina / principal / Gs
c. acetilcolina / parietal /Gi
d. acetilcolina / principal / Gi

19. Un niño de tres años llega a emergencia con disfagia (dificultad para tragar), dolor
retro esternal, salivación y llanto. Se sospecha de ingesta de cuerpo extraño
(moneda) en el esófago; al ser evaluado se constata en una radiografía presencia de
cuerpo extraño a nivel de C6 (6° vértebra cervical). El cuerpo extraño estará
suspendido a nivel del estrechamiento producido por………..
(unidad 2, sesión 11, logro4: Esófago, características anatómicas, relación con órganos
vecinos y estrecheces)

a. el cayado aórtico
b. el hiato esofágico
c. el músculo cricofaríngeo
d. el bronquio principal izquierdo

20. La triada portal (arteria hepática, vena portal y conducto biliar común) está contenida en el
ligamento
…….……… y derivan embriológicamente del ……
(Unidad 1, sesión 1, logro 4: Identifica el peritoneo, mesenterio, omento y ligamentos,
retroperitoneo.)

a. hepato duodenal / mesenterio ventral


b. gastro esplénico / mesenterio dorsal
c. hepato gástrico / omento menor
d. falciforme / omento menor
21. En relación al movimiento de
peristaltismo del tubo
digestivo: en la flecha negra
del gráfico se produce la
liberación de ……………… a
nivel del músculo ………..
(unidad 2, sesión 7, logro 6:
Control hormonal y tipos de
movimiento)

a. noradrenalina, sustancia P y neuropéptido “ Y” / circular


b. acetilcolina y sustancia P / longitudinal
c. óxido nítrico y PIV / longitudinal
d. óxido nítrico y PIV / circular

22. Un paciente refiere no percibir algunos sabores, al examen físico se constata


alteración en la percepción de sabores y del dolor en el tercio posterior de la lengua
¿Cuál de los siguientes nervios estará alterada en su función?
(unidad 2, sesión 10, logro 5: Sabores, tipos y mecanismos moleculares para su detección)

a. Lingual (rama del V par)


b. Cuerda del tímpano (VII par)
c. Glosofaríngeo (IX par)
d. Hipogloso (XII par)

23. El gráfico detalla la


estructura de la pared del
tubo digestivo intestinal
¿Cuál de las siguientes
asociaciones es correcta?
(unidad 2, sesión 7, logro 1:
La pared y músculo liso
gastrointestinal )

a. “1” – peristaltismo
b. “2” – secreción enzimática
c. “3” – deriva del mesodermo
d. “4” – doble hoja de tejido graso

24. En el caso de un paciente con gastrinoma (tumor productor de gastrina), la presencia


de úlceras duodenales y erosión de la mucosa gástrica, se debe principalmente a…….
(unidad 1, sesión 3, logro 2: describir las hormonas gastrointestinales: estímulo y funciones de
la gastrina y colecistoquinina)

a. la acción directa de la gastrina sobre la célula principal


b. la sobre expresión de los receptores “G” en la célula parietal
c. el exceso de HCl por estímulo de receptores CCK-B en la célula parietal
d. el exceso de HCl por estímulo directo de receptores de acetilcolina en la célula parietal

25. El reflejo entero gástrico se caracterizan por:


(unidad 2, sesión 13, logro 6: Motilidad del intestino grueso: contracciones
segmentarias, movimientos en masa defecación y reflejo gastrocólico)

a. favorecer la motilidad gástrica gracias a la CCk


b. inhibir la motilidad gástrica y estimular la secreción ácida
c. movilizar grandes volúmenes desde el estómago al duodeno
d. originarse debido a la distensión duodenal y presencia del quimo ácido
26. Mauricio tiene dificultad para deprimir el paladar y elevar la parte posterior de la
lengua. En este caso estará afectado un músculo ………………., específicamente el
músculo …………….
(Unidad 2, sesión 10, logro 2: Músculos de la lengua: clasificación, identificación y sus
funciones)

a. intrínseco – longitudinal inferior


b. extrínseco – palatogloso
c. extrínseco – transverso
d. extrínseco – estilogloso

27. Una de las funciones del músculo señalado es:


(Unidad 2, sesión 8, logro 3: Describir el Piso de la boca:
estructuras blandas que la conforman)

a. deprimir la lengua
b. elevar el paladar blando
c. deprimir el hioides cuando la mandíbula está fija
d. deprimir la mandíbula cuando el hioides está fijo

28. Paciente varón de 30 años es evaluado por probable enfermedad de Chagas, cursa
con problemas de motilidad del colon; los estudios de biopsia determinan
ausencia de células ganglionares. Según el gráfico
¿cuál es la capa en la que se determina la ausencia de dichas células?
(unidad 1, sesión 2, logro 1: describir las generalidades de la estructura del tubo digestivo:
esófago, estómago intestino delgado y grueso)

a. Mucosa - 1
b. Muscular propia – 1
c. Muscular de la mucosa - 2
d. Muscular propia - 3

1 2

3
29. Paciente varón de 32 años, que acude a centro de salud por presentar de forma
progresiva desde hace 1 año dificultad para ingerir alimentos sólidos y luego líquidos;
refiere regurgitaciones alimentarias y marcada pérdida de peso (15 kilos). Radiografía
baritada (sustancia de contraste) de esófago se muestra en la figura. El presente caso
se explica por……………….
(Unidad 2, sesión 12, logro 4: Identificar y describir la función de los esfínteres esofágicos)

a. aumento de la peristalsis esofágica


b. relajación incompleta del esfínter pilórico
c. relajación incompleta del esfínter esofágico inferior
d. perdida de producción de PIV y
óxido nítrico en el esfínter
esofágico superior

30. En condiciones normales, el ingreso de 600 ml de líquido es el estómago provoca un


aumento de presión intragástrica de unos 12 cm de H 2O. Después de una vagotomía
(corte del nervio vago) es de esperar que el ingreso del mismo volumen de líquido
ocasionede la presión intragástrica.
(Unidad 2, sesión 13, logro 1: Describe la Motilidad gástrica: relajación receptiva)

a. la disminución
b. la no variación
c. un aumento mayor
d. un aumento similar o igual
SISTEMA
DIGESTIVO
(ME154)
EXAMEN FINAL
2019 01

Profesores : Alfaro Salazar, Herberth Romulo; Callata Caceres, Gunter; Cayo Quiñe, Alexandra Mariel;
Correa Borit, Jorge Mauricio; Cruz Cutty, Lourdes Marylin; Guzmán Calderón, Gerly Edson;
Jáuregui Farfán, Jorge Jesús; Mayor Zevallos, Otto Alberto; Montoya Suárez, José Luis;
Palacios Bazan, Enrique Elias; Robles Pino, Alexander Anibal; Wong Bravo, Juan Carlos
Sección : Todas las secciones
Duración : 70 minutos.
Indicaciones:
Lea atentamente cada pregunta antes de responder:
- Se prohíbe el uso del celular y cualquier dispositivo electrónico.
- Está prohibido intercambiar materiales.
- Coloque su código de alumno en la tarjeta de respuestas. Si su código contiene una letra
reemplácela por un valor numérico siguiendo la siguiente equivalencia: A=9, B=8, C=7, D=6, E=5, F=4,
G=3.
- Traslade sus respuestas a la tarjeta, llenando los círculos de manera completa con lapicero negro o
azul. Está prohibido el llenado con lápiz, lapicero de otro color o con lapicero de tinta borrable.
- Sea cuidadoso en el llenado de la tarjeta de respuestas, pues solo esta tiene validez para la
calificación.
- Al terminar su examen avise al docente a cargo, no se levante de su sitio; debe entregar la hoja de
respuestas con la carátula del examen, este cuadernillo de preguntas se lo llevará cada estudiante.

1. La explicación fisiológica de presentar somnolencia de 30 minutos a 1 hora después de ingerir


alimentos, se explica por:
a. Aumento del cloro intraluminal
b. Aumento del bicarbonato intraluminal
c. Disminución de ácido carbónico en la célula parietal
d. Disminución de la actividad de la anhidrasa carbónica
e. Aumento de la alcalinidad sanguínea

2. Con respecto a la irrigación arterial del colon, a que arteria


corresponde la señalada con la flecha
a. Cólica derecha
b. Cólica media
c. Cólica izquierda
d. Ileobisecoapendículocólica
e. Arco de Riolano

3. Si un paciente presentara dentro del punto de vista fisiológico, una


disminución de enterocinasa, entonces esto originaría una
disminución de la actividad de:

Á
a. L
A. La
a pepsina
B. La
p lipasa
C. La
e quimiotripsina
D. El
p péptido insulinotropo dependiente de glucosa
s amilasa
E. La
i
n
a
b
.l
a
li
p
a
4. Con respecto a la anatomía del hígado, señale a que
estructura pertenece la marcada por el número 1.
a. Ligamento falciforme
b. Línea de Cantlie
c. Ligamento triangular
d. Ligamento coronario
e. Ligamento teres

jefe
Se valida la opción a debido a la ubicación del número 1 en
donde se unen el ligamento falciforme y ligamento coronario.

5. Se presenta un paciente, el cual presenta un antecedente de tuberculosis intestinal, por lo cual,


se le resecó 80 cm de íleon distal. Desde el punto de vista fisiológico, el paciente puede presentar
una de las siguientes alteraciones:
a. Disminución de la secreción de Vitamina B12
b. Aumento indiscriminado de absorción de ácido fólico
c. Disminución de la absorción de hierro
d. Aumento de la secreción de bicarbonato
e. Disminución de la absorción de ácido glicocólico

6. Un paciente es sometido experimentalmente a un fármaco que modifica el flujo salival, obteniéndose


un volumen de saliva de 288 ml en 6 horas. En este caso las concentraciones de electrolitos y
bicarbonato en la saliva obtenida varían de la siguiente manera:
a.↑ Na+, ↓ K+, ↑
Cl-, ↑ HCO3- b.↓
Na+, ↓ Cl-, ↑ K+, ↓
HCO3-
c.↑ Na+, ↑ Cl-, ↓ K+, ↓ HCO3-
d.↑ Na+, ↑ Cl-, ↑ K+, ↑ HCO3-
e.↓ Na+, ↓ Cl-, ↓ K+, ↓ HCO3-

Se valida la opción a debido a que se puede considerar como un aumento del flujo de saliva.

7. La siguiente imagen histológica corresponde a la glándula


…………… y la estructura señalada produce ………
a. salival sublingual / mucopolisacáridos
b. oxíntica / pepsinógeno
c. salival submaxilar / ptialina
d. salival parótida / amilasas
e. antrales / gastrina

8. Paciente varón de 65 años con antecedente de hipercolesterolemia, hipertensión arterial, fibrilación


auricular y dos infartos al miocardio previos, aqueja de dolor abdominal intenso de inicio súbito,
distensión abdominal, se decide cirugía con resección de 1,5 metros de intestino delgado terminal y
colon ascendente. Como consecuencia de la resección el paciente tendrá deficiencia de:
a. Vitamina C
b. Tiamina
c. Vitamina A
d. Vitamina B1
e. Vitamina B6

Se valida esta opción debido a que su absorción está relacionada al íleon.


9. Uno de los siguientes elementos debería hallarse con más probabilidad en el esófago de un paciente
que sufre de
reflujo gastro esofágico…
a. Pepsina
b. Tripsina
c. Quimiotripsina
d. Carboxipeptidasa
e. Ácidos biliares

10. Un paciente de 40 años cursa con anemia de 8g/dl, aqueja además de astenia y sensación de
hormigueo bilateral en los miembros inferiores, al examen se halla alteración de la sensibilidad a la
vibración y camina con ampliación de la base de sustentación. Uno de los siguientes procedimientos
sería de ayuda para el diagnóstico de este paciente:
a. Tomografía cerebral
b. Biopsia de la mucosa gástrica
c. Biopsia de hígado
d. Examen de sangre oculta en heces
e. Biopsia de Ileon proximal

11. Paciente de 60 años ingresa por caída hace 1 hora y pequeño hematoma en cuero cabelludo, al
examen físico ampliado se observa ictericia de piel y mucosas generalizada, abdomen blando, se
palpa estructura quística no dolorosa en hipocondrio derecho que corresponde a vesícula biliar
(signo de Courvoisier), en los exámenes de laboratorio se halla niveles bajos en la formación de
estercobilinógeno y urobilinógeno en heces, incremento de la bilirrubina conjugada en la orina,
elevación de fosfatasa alcalina y gamma glutamil transpeptidasa séricas. El presente cuadro puede
ser explicado por:
a. Reabsorción de hematoma
b. Litiasis vesicular
c. Carcinoma de la cabeza de páncreas
d. Carcinoma con estenosis del conducto hepático común
e. Anemia hemolítica

12. Paciente varón de 58 años con antecedente de alcoholismo crónico es diagnosticado y recibe
tratamiento por cirrosis hepática. Hace 2 días refiere familiar que tuvo cambio de conducta y no
reconoce a algunos familiares. Al examen físico, se halla ascitis, circulación colateral en abdomen,
telangiectasias, en el examen de sistema nervioso: rigidez de extremidades, ROT incrementados,
desorientación en el espacio y asterixis. ¿cuál de las siguientes circunstancias, explicaría el cuadro
en este paciente?
a. Uso de diuréticos ahorradores de potasio
b. Incremento de actividad de ureasa bacteriana duodenal
c. Hemorragia gastrointestinal
d. Disminución de la producción de NH3+ en el colon
e. Dieta normo proteica

Se valida la opción e debido al efecto sobre la encefalopatía.


Con respecto de la opción b es incorrecta debido a que hace referencia al duodeno, debería indicar colon.

13. Un recién nacido presenta vómitos biliosos poco tiempo después de cada alimento. Al preguntar a la
madre sobre antecedentes, ella recuerda que tuvo polihidramnios durante la gestación, pero un
análisis de cariotipo fue normal. Una de las siguientes es la causa más probable de estos hallazgos
en el recién nacido:
a. Enfermedad de Hirschprung
b. Fístula tráqueo esofágica
c. Divertículo ileal
d. Estenosis pilórica
e. Malrotación de la yema pancreática ventral

14. Un lobulillo hepático se puede dividir en tres zonas como se muestra en el gráfico. ¿Cuál de
las siguientes afirmaciones sobre las tres zonas es verdadera?
a. La zona 1 tiene los menores depósitos de glucógeno
b. La zona 3 es la primera en afectarse en una colestasis extra hepática
c. La zona 2 es más susceptible a la injuria por isquemia que la zona
periportal d.La zona 2 tiene la mayor capacidad de regeneración
e.La zona 1 es la que tiene menos actividad metabólica.

La pregunta 14 ha sido anulada, sin embargo, ningún estudiante se verá afectado


negativamente en su puntaje debido a esta anulación.

15. En un estudio de la secreción de hormonas gastrointestinales, sus concentraciones en la vena


porta se midieron durante perfusión luminal del intestino delgado con soluciones de diversas
magnitudes de pH. ¿Qué hormona aumentará en el plasma de la vena porta durante perfusión a
través del intestino con una solución de pH 3?
a. CCK
b. gastrina
c. GIP
d. motilina
e. secretina

16. Paciente de 30 años que ingresa a causa de un traumatismo abdominal cerrado. En la exploración
se aprecia discreta palidez de piel y mucosas, auscultación pulmonar normal, taquicardia de 120
/min. Discreta distensión abdominal y matidez en flancos; el hematocrito, que era prácticamente
normal al ingreso, disminuye a 30% a las tres horas. En la Rx de tórax se objetiva fractura de las
costillas 10-11 izquierdas. La causa más probable de la anemización en este paciente es:
a. traumatismo renal con hemorragia retroperitoneal.
b. rotura de hígado con hemoperitoneo.
c. rotura de bazo con hemoperitoneo.
d. rotura de mesos con hemoperitoneo.
e. traumatismo pancreático con pancreatitis traumática.

17. Mujer de 65 años. Consulta por síndrome constitucional


asociado a dolor abdominal epigástrico progresivo
irradiado a espalda, de dos meses de evolución. El
diagnostico de sospecha de adenocarcinoma de páncreas
se confirma por biopsia. Se realiza examen de imagen de
abdomen para evaluación de estructuras vasculares
próximas al tumor pancreático. ¿Cuál es el nombre de la
vena señalada que está ausente, trombosada por
infiltración tumoral, condicionando circulación colateral
en la pared gástrica?

a. Mesentérica superior
b. Coronaria estomaquica
c. Esplénica
d. Porta
e. Renal izquierda
18. Revisando la angiotomografía de un hombre de 70 años en estudio por aneurisma de aorta
abdominal, el radiólogo le informa de la presencia de una oclusión completa de la arteria
mesentérica inferior. El paciente se encuentra completamente asintomático. La oclusión de la
arteria mesentérica inferior cursa de manera asintomática en muchas ocasiones ya que el
territorio que irriga puede recibir flujo proveniente de la arteria:
a. cólica derecha
b. gastroduodenal
c. Epigástrica inferior izquierda
d. esplénica
e. cólica media

19. En las patologías de esófago es importante conocer bien la anatomía esofágica. ¿Cuál de
las siguientes afirmaciones es correcta?
a. El esófago tiene capa mucosa, muscular y serosa
b. El esófago abdominal es más largo que el cervical
c. El esófago torácico pasa por detrás del cayado aórtico
d. El epitelio esofágico normal es de tipo cilíndrico.
e. El esófago abdominal es discretamente más largo que el torácico

20. A pesar de que pueda haber variaciones anatómicas, lo habitual es que el ciego sea irrigado por una
rama arterial que proviene de unas de las siguientes arterias:
a. Iliaca derecha
b. Mesentérica inferior
c. Hepática derecha
d. Mesentérica superior
e. Iliaca izquierda

21. Ante un paciente con una cirugía abdominal urgente, el informe operatorio señala que se ha
realizado una resección de todo el duodeno y del tercio proximal del yeyuno manteniendo íntegros
el estómago y todo el íleon, así como los dos tercios distales del yeyuno. En el seguimiento
nutricional del paciente ¿Qué vitamina o mineral presentará con menor probabilidad una
disminución de su absorción?
a. Cianocobalamina
b. Calcio
c. Hierro
d. Transcobalamina
e. Transferrina

22. ¿Cuál de las siguientes alternativas detallan las venas que confluyen y forman la vena señalada?
a. mesentérica superior, gástrica izquierda y
gastroepiploica izquierda
b. mesentérica inferior, gástrica izquierda y renal
c. esplénica, mesentérica superior y mesentérica inferior
d. esplénica, pancreatoduodenal y omental izquierda
e. gástrica izquierda, esplénica y hepática común

23. ¿Cuál de las siguientes sustancias forma parte de la secreción biliar?


a. Tripsina
b. Lecitina
c. Elastasa
d. Quimotripsina
e. Pepsina
24. El tubo digestivo contiene diferentes tipos de epitelios y glándulas. La estructura señalada es unay
está localizada en el …………...
a. glándula de Brunner /
intestino grueso b.cripta de
Lieberkuhn / colon
c. cripta de Lieberkuhn / intestino delgado
d. glándula oxintica / estomago
e. célula parietal / estómago

Aunque las criptas de


Lieberkuhn están
presentes en el intestino
delgado, la
microfotografía es de
epitelio de colon.

25. ¿De qué musculo forma parte el ligamento inguinal?


a. Oblicuo externo del abdomen
b. Oblicuo interno del abdomen
c. Transverso del abdomen
d. Psoas
e. Dorsal ancho

26. Señale cuál de las siguientes afirmaciones NO se relaciona a la siguiente glándula anexa del
tubo digestivo mostrada en la imagen:
a. Es una glándula exocrina compuesta exclusivamente por acinos serosos
b. Su inervación está dada por el nervio auricular mayor (ramo posterior C2), que inerva
la vaina de la glándula así como la piel por encima de esta.
c. Esta glándula produce una secreción mucinosa acuosa, llamada mucoserosa, a través
del conducto de Wharton.
d. Su inflamación puede ser causada por un virus de los Paramyxoviridae, que provocan
una enfermedad muy frecuentemente en niños y adolescentes
e. Es una glándula endocrina y probablemente sea de origen pancreático

Se valida la opción e debido a que no


está relacionada con la imagen.

27. ¿Cuál de las siguientes enzimas está localizada en el borde en cepillo y juega un rol en la digestión de
proteínas?
a. Alfa dextrinasa
b. Pepsina
c. Enterocinasa
d. Lactasa
e. Carboxipeptidasa A.

Se valida la opción c debido a que es correcta en relación a la pregunta.


e. Dorsal ancho

26. Señale cuál de las siguientes afirmaciones NO se relaciona a la siguiente glándula anexa del tubo
digestivo mostrada en la imagen:
a. Es una glándula exocrina compuesta exclusivamente por acinos serosos
b. Su inervación está dada por el nervio auricular mayor (ramo posterior C2), que inerva la
vaina de la glándula así como la piel por encima de esta.
c. Esta glándula produce una secreción mucinosa acuosa, llamada mucoserosa, a través del
conducto de Wharton.
d. Su inflamación puede ser causada por un virus de los Paramyxoviridae, que provocan una
enfermedad muy frecuentemente en niños y adolescentes
e. Es una glándula endocrina y probablemente sea de origen pancreático

Se valida la opción e debido a que no está


relacionada
con la
imagen.

27. ¿Cuál de las siguientes enzimas está localizada en el borde en cepillo y juega un rol en la digestión
de proteínas? a. Alfa dextrinasa
b. Pepsina
c. Enterocinasa
d. Lactasa
e. Carboxipeptidasa A.

Se valida la opción c debido a que es correcta en relación a la pregunta.

28. Una de los siguientes sustancias, NO sirve como un buen agente emulsificante: a. Colesterol
b. Ácidos grasos
c. Sales biliares
d. Lecitina
e. Proteínas de la dieta

Se valida la opción e debido a que es correcta en relación a la pregunta.

29. La sustancia que estimula el crecimiento de la mucosa gástrica es:


a. Secretina
b. Motilina
c. Péptido estimulante de la mucosa gástrica
d. Gastrina
e. Histamina

30. ¿Cuál de las siguientes alternativas es una función de la colecistokinina?


a. Relajación de la vesícula para la salida de bilis
b. Secreción de ácidos biliares
c. Contracción del esfinter de Oddi
d. Secreción de enzimas pancreáticas
e. Contracción del duodeno

Se valida la opción b debido al efecto de la CCK sobre la vesicula biliar.

31. Con respecto a la anatomía del tronco celiaco, señale lo correcto


a. El tronco celiaco se origina de la cara posterior de la aorta abdominal
b. Es una arteria delgada que tiene un calibre entre 2 y 3 mm
c. Una de sus ramas es la arteria gástrica derecha
d. La hepática común que es una de sus ramas, participa en la irrigación del estómago.

32. Con respecto a la anatomía del duodeno, marque la respuesta correcta:


a. Tiene una distribución en forma de “C”, que rodea la cola del páncreas
b. La 3ra porción duodenal está contenida en la pinza vascular aortomesentérica
c. Entre la 1ra y 2da porción se forma un ángulo, conocido como el ángulo de Treitz
d. La 4ta porción se dirige a la izquierda, hacia abajo y hacia atrás.
e. En la tercera porción desemboca el conducto colédoco.

33. El hígado está ampliamente tapizado por peritoneo, la estructura que conecta la cara diafragmática
del hígado precisamente con el diafragma es el ligamento:
a. teres
b. falciforme
c. triangular
d. hepático común
e. coronario

Se validan la opción c y e debido a que forman parte de los ligamentos que fijan el hígado al diafragma.

34. En el íleon se absorbe aproximadamente el 95% de …………………. a través de la circulación


enterohepática. a. agua
b. colesterol
c. sales biliares
d. hidróxicobalamina
e. factor intrínseca

35. La ……………. estimula el mecanismo paracrino de la secreción de ácido clorhídrico.


a. histamina
b. acetilcolina
c. gastrina
d. secretina
e. somatostatina

36. En la digestión de proteinas, ……………. es el principal estímulo para convertir el pepsinógeno en


pepsina. a. la gastrina
b. el pH ácido
c. la acetilcolina
d. la ptialina
e. la somatostatina
37. Con respecto a la somatostatina, marque lo correcto:
a. Es secretada por las células S del intestino
b. Induce a la producción de VIP
c. Interviene en la fase intestinal de la secreción gástrica
d. Produce acetilcolina para estimular a la célula parietal
e. No interviene en la regulación de la secreción de ácido clorhídico

38. En pecten anal, es una estructura comprendida entre:


a. la línea pectínea y los senos anales
b. la línea blanca y la apertura anal
c. el esfínter anal interno y el externo
d. la línea anocutánea y la línea pectínea
e. la línea blanca y columnas anales

39. ¿Cuál de las siguientes alternativas es una proenzima pancreática?


a. Tripsina
b. Elastasa
c. Quimotripsinógeno
d. Amilasa
e. Procarboxipepitidasa C.

40. En la segmentación hepática de Coinaud, el segmento hepático señalado con la flecha, corresponde
a : En la segmentación hepática de Coinaud, la flecha señala el segmento ………….. hepático.
a. IV b. V

c. VI
d. VII
e. VIII

Parcial 2019-02

1. La contracción del músculo ………………………… permite la eliminación de gases (flatos) sin salida de
material fecal; es el mismo músculo cuya relajación, sobretodo en cuclillas, permite el paso del
contenido fecal con menor esfuerzo durante la defecación.
a) Isquirectal
b) Puborrectal
c) Esfínter anal externo
d) Esfínter anal interno
2. Paciente mujer de 54 años se presenta con náuseas, vómitos, estreñimiento, y es diagnosticada de
abdomen agudo quirúrgico; en la cirugía encuentran un vólvulo de ciego. Esta anomalía puede
explicarse por:: a) Falta de rotación intestinal
b) Falta de fusión del mesenterio
c) Defecto en la formación de la cloaca
d) Falta de formación del omento mayor

3. Paciente mujer de 23 años con faringitis aguda, toma para el dolor una tableta de paracetamol con
un poco de agua. Durante la deglución, se relaja su esfínter esofágico inferior y el fondo del
estómago, mientras el bolo está aún en el esófago. ¿Qué sustancia provocara con mayor
probabilidad la relajación del esfínter esofágico inferior y el fondo del estómago en esta mujer?
a) Óxido nítrico
b) Sustancia P
c) Histamina
d) Motilina

4. Luego de tres horas dando exámenes, un alumno de medicina comienza a sentir hambre. Esta
situación es probable que sea mediada por la ______________ que es sintetizada por el
___________________: a) leptina / intestino
b) leptina / estómago
c) grelina / estómago
d) grelina / tejido adiposo

5. Varón de 72 años, con antecedente de diabetes mellitus tipo 2, que presenta enteropatía diabética
caracterizada por estreñimiento. Este problema puede estar asociado a:
a) deficiencia de óxido nítrico
b) aumento del reflejo gastrocólico
c) disminución de la secreción de colecistocinina (CCK)
d) aumento de la secreción del péptido intestinal vasoactivo (PIV)

6. Varón de 54 años con Diabetes Mellitus tipo 2, es diagnosticado de gastroparesia debido a que
presenta sensación de llenura precoz al comer, y reflujo gastroesofágico. Esta alteración en la
relajación receptiva y en el vaciamiento gástrico lo más probable es que se deba a una alteración en:
a) el nervio vago
b) el ganglio celíaco
c) plexo submucoso
d) nervio hipogástrico

7. Varón de 67 años con tos y disminución de peso asociado a tabaquismo pesado, presenta
actualmente disfagia progresiva a alimentos sólidos. Se considera la presencia de un carcinoma de
bronquio izquierdo y por esta razón le realizan una endoscopía esofágica para descartar la
posibilidad de una compresión esofágica por el tumor. Se espera revisar el esófago en la
__________________ estrechez, que está a nivel de la vértebra ______________ a) Tercera
estrechez -T6
b) Segunda estrechez - C6
c) Segunda estrechez - T4
d) Tercera estrechez -T10

8. Varón de 34 años con dolor abdominal agudo en flanco derecho que se irradia a fosa ilíaca derecha,
es operado y se encuentra un divertículo intestinal inflamado, ubicado a 93 cm de la válvula
ileocecal. El origen de este divertículo es una falla en la obliteración de:
a) Conducto vitelino
b) Alantoides
c) Cloaca
d) Conducto anorectal
e) Uraco

En un niño menor de dos años con divertículo intestinal, este divertículo tiene su origen en una falla
en la obliteración de:
a) Conducto anorectal
b) Conducto vitelino
c) Alantoides
d) Cloaca
e) Uraco

9. Mujer de 43 años sufre un grave accidente de tránsito y está hospitalizada en coma, es alimentada
por vía intravenosa durante varias semanas. Producto de este tipo de alimentación, se encuentra en
la endoscopía atrofia de la mucosa gastrointestinal. La causa más probable de esta atrofia son los
bajos niveles séricos de la hormona: a) Colecistocinina
b) Secretina
c) Gastrina
d) PIV

10. Una mujer de 30 años llega al consultorio porque se queja de dificultades para deglutir, la cual se
agravan cada vez más. Se realiza un estudio manométrico para examinar la generación de presión a
lo largo del esófago. Esta prueba revela que las contracciones como respuesta a la deglución están
mal sincronizadas y que la presión en el esfínter esofágico inferior permanece elevada. El
diagnóstico más probable es ____________________ producido por niveles bajos de
____________________ a) acalasia / sustancia P
b) acalasia / óxido nítrico
c) enfermedad por reflujo gastrointestinal / acetilcolina
d) enfermedad por reflujo gastrointestinal / óxido nítrico

11. Paciente de 2 años, llega a emergencia por haber ingerido una moneda con la que estaba jugando.
El lugar más probable donde puede haberse quedado suspendido este objeto es a nivel del
estrechamiento producido a nivel del:
a) músculo milohiodeo
b) músculo aritenoideo
c) músculo cricofaríngeo
d) constrictor superior de la faringe

12. En una apendicectomía, al realizar la incisión de McBurney en la fosa iliaca derecha, es necesario
cortar los siguientes músculos, de afuera hacia adentro:
a) Recto – Oblicuo externo – Transverso
b) Recto – Oblicuo externo – Oblicuo interno
c) Oblicuo externo – Oblicuo interno – Recto
d) Oblicuo externo – Oblicuo interno – Transverso

13. Un varón de 90 años que se encuentra postrado en cama, es referido del asilo para endoscopia por
dificultad para deglutir luego de tomar un medicamento para aliviar el dolor la noche anterior. La
endoscopía revela que la píldora se alojó en el esófago y causó una reacción inflamatoria. Lo más
probable es que esto haya sido por la producción de múltiples ondas:
a) secundarias
b) primarias
c) lentas
d) segmentarias

14. Mujer de 23 años es diagnosticada de bulimia, al examen físico se observa ulceraciones en el


segundo y tercero dedo de la mano derecha. Esto se puede deber al uso continuo de estos dedos
para inducir el vómito, mediante la estimulación del par craneal:
a) V
b) IX
c) X
d) XI

15. Varón de 52 años se presenta por diarrea persistente de seis semanas de duración. En la
colonoscopia se observa un pólipo a nivel del íleon distal. El patólogo informa que se trata de un
tumor neuroendócrino, probablemente originado por las células enterocromafines del intestino. La
sustancia que más probablemente esté produciendo este tumor es:
a) Serotonina
b) Insulina
c) CCK
d) GIP

16. La fase oclusal de la masticación se realiza con la contracción de los músculos: a) digástricos
b) masetero y temporal
c) orbicular y buccinador
d) pterigoideo lateral y digástrico

17. Al tomar su café en Starbucks, un estudiante de medicina sufre una quemadura de primer grado en
el tercio anterior de la superficie dorsal de la lengua. La información de dolor es transmitida por el
nervio: a) cuerda del tímpano
b) glosofaríngeo
c) lingual
d) facial

18. Paciente es evaluado por faringitis aguda en consultorio externo. El médico de familia le solicita que
abra la boca y saque la lengua. Para realizar la acción de sacar la lengua, es necesario que se
contraiga el músculo: a) estilogloso
b) geniogloso
c) palatogloso
d) transverso de la lengua

19. Paciente con síndrome de Sjögren, presenta “boca seca” (disminución de


la producción de saliva) y caries dental, asociada a la pérdida de la función
de tampón de la saliva. Esta desminerilización del diente puede
comprometer a las prolongaciones citoplasmáticas ubicadas en los tubos
huecos de la estructura señalada con la letra:
a) B
b) A
c) E
d) C

20. Mujer de 32 años acude a consulta por presentar disfagia de progresión


lenta, reflujo gastroesofágico y vómitos desde hace 3 meses de evolución
progresiva. Se le realiza un estudio radiológico con contraste en el que se
observa estrechamiento del esfínter esofágico inferior (imagen). Según sus
conocimientos, este paciente se beneficiaría con el uso de: a) agonista
beta adrenérgico
b) agonista alfa adrenérgico
c) análogo de óxido nítrico
d) análogo de Sustancia P

Final 2019-02

1) La reabsorción de sodio y cloro en las glándulas salivales se da principalmente en el:


a) conducto intercalado
b) conducto excretor
c) conducto estriado
d) acino glandular

2) Paciente de 35 años con dolor en hipocondrio derecho irradiado a la punta de la escápula. Se observa
en la ecografía abdominal cálculos en la vesícula biliar. Es operado de emergencia realizando una
incisión a lo largo del reborde costal derecho. La información de dolor de esta zona es inervada por
las raíces nerviosas: a) T5 – T9
b) T6 – T7
c) T7 – T8
d) T9 – T10

3) Paciente de 23 años con dolor intenso periumbilical de inicio agudo. Es ingresado a sala de
operaciones por abdomen agudo, el cirujano encuentra sangrado a unos 60 cm proximal a la válvula
ileocecal. La estructura que está sangrando muy probablemente es un derivado embriológico del:
a) uraco
b) ligamento Hepatoduodenal
c) alantoides
d) conducto vitelino

4) Paciente de 64 años de edad con dolor en epigastrio que se distribuye en banda hacia la espalda. En
la tomografía se encuentra tumor en cabeza de páncreas que compromete a un vaso que discurre
entre la cabeza del páncreas y el proceso uncinado. Debido a esto, este cáncer se considera
irresecable porque compromete a la arteria: a) mesentérica superior
b) mesentérica inferior
c) tronco celíaco
d) aorta

5) Paciente con carcinoma gástrico avanzado, en preoperatorio para gastrectotomía total. ¿Cuál de los
siguientes ganglios linfáticos recibirá primero células metastásticas con mayor probabilidad: a)
celíaco
b) cisterna del quilo
c) esplénico
d) gástrico posterior
6) Paciente de 77 años con dolor abdominal difuso de 3 horas de evolución. En los estudios de imágenes
se observa oclusión probablemente aterosclerótica de la arteria mesentérica superior; no se observa
necrosis, lo cual puede ser explicado por la irrigación sanguínea colateral. ¿Qué vasos ofrecen
colaterales entre el tronco celíaco y la arteria mesentérica superior?
a) Gástrica izquierda y hepática
b) Cística y duodenal
c) Gastroomental derecha e izquierda
d) Pancreaticoduodenal superior e inferior

7) Paciente de 62 años con vólvulo de intestino delgado e isquemia intestinal. Se realiza laparotomía
exploratoria para liberar la obstrucción. ¿Cuál estructura se utilizará como punto de referencia para
determinar la posición de la unión duodenoyeyunal?
a) Vasos rectos
b) Ligamento suspensorio del duodeno (de Treitz)
c) Ligamento frenocólico
d) Nacimiento de la Arteria mesentérica superior

8) Paciente de 42 años con dolor abdominal intenso y hematemesis. En la endoscopía se observa una
úlcera duodenal posterior perforada con hemorragia intrabdominal. ¿Cuál de las siguientes arterias
estará comprometida? a) Gástrica izquierda
b) Mesentérica superior
c) Gástrica derecha
d) Pancreaticoduodenal posterosuperior

9) Paciente de 51 años con antecedente de enfermedad diverticular acude a emergencia por sangrado
profuso y dolor en cuadrantes inferiores. ¿Cuál es el origen más probable de la sangre que pierde en
la hemorragia? a) Cólica media
b) Mesentérica inferior
c) Rectal superior
d) Rectal inferior

10) Paciente de 13 días de vida con vómitos explosivos a las dos horas después de lactar. Al examen físico
se palpa la oliva pilórica. ¿Cuál es el nervió cuyos filetes dan inervación eferente a la estructura
afectada? a) Vago
b) Esplácnico torácico mayor
c) Mesentérico superior
d) Esplácnico torácico menor

11) Paciente de 80 años con dolor abdominal intenso y antecedente de estreñimiento crónico. En la
colonoscopía se observa divertículos con áreas ulceradas difusas en colon sigmoides y descendente.
El cirujano programa una cirugía de resección y planifica que para realizar esta resección tendrá que
cortar los siguientes vasos y nervios: a) plexo mesentérico superior y arteria rectal.
b) nervio esplácnico torácico mayor y arteria cólica media.
c) ramas del nervio vago y arteria ileocólica.
d) nervio esplácnico pélvico y artera cólica izquierda.

12) Paciente de 46 años ingresa a emergencia con dolor en cuadrante superior derecho e ictericia. En la
ecografía se observa cálculos en la vesícula biliar. ¿Cuál de los siguientes nervios transmite el dolor
de la colecistitis?
a) Filetes aferentes del nervio vago derecho, referido al ángulo inferior de la escápula
b) Filetes nerviosos de las raíces T1 a T4, con referencia al hombro izquierdo
c) Fibras aferentes simpáticas del nervio esplácnico torácico mayor, con referencia a los
dermatomas T6 a T8
d) Fibras aferentes de los ramos primarios dorsales de los nervios raquídeos T8 a T10 con
referencia al epigastrio

13) Un niño de 8 años es alimentado por sus padres con un Cheeseburguer, papitas fritas y un vaso de
Coca Cola. ¿La presencia de lípidos, carbohidratos y proteínas en el duodeno estimulan la liberación
de cuál de las siguientes hormonas en la mucosa duodenal?
a) Colecistoquinina (CCK)
b) Péptido inhibidor gástrico (GIP)
c) Secretina
d) Insulina

14) La fase cefálica del control de la secreción gástrica corresponde a cerca del 30% de la secreción ácida
y es consecuencia de un reflejo. ¿Cuál de las siguientes alternativas puede eliminar totalmente la
fase cefálica de la secreción gástrica?
a) Vaguectomía
b) Uso de atropina
c) Uso de bloqueador de receptor H2 de histamina
d) Uso de antiácidos

15) Recién nacido de 24 horas con llanto, vómitos y sin eliminación de meconio. Luego de los estudios
auxiliares se diagnostica la Enfermedad de Hirshprung, esta enfermedad se caracteriza por la
ausencia de:
a) las células de Cajal en plexo mioentérico
b) el plexo mioentérico en el recto y colon sigmoides
c) las células mioepiteliales en el recto distal
d) el plexo de Meissner en el recto distal y colon sigmoides

16) Los complejos motores migratorios aparecen aproximadamente cada 90 minutos entre las comidas,
y se considera que son estimulados por la hormona motilina. La ausencia de estos complejos
migratorios podría producir un aumento en:
a) la motilidad duodenal
b) el vaciamiento gástrico
c) la deglución
d) las bacterias intestinales

17) ¿Cuál de las siguientes es una consecuencia probable de la resección del íleon? a) Gastritis atrófica
b) Deficiencia de vitamina B12
c) Esteatorrea
d) Úlcera péptica

18) Los movimientos en masa son importantes en la fisiología intestinal. Estos movimientos en masa
ocasionan: a) la sensación de defecar
b) el peristaltismo duodenal
c) la retropulsión gástrica
d) la contracción del esfínter anal interno

19) La toxina colérica hace que aumente los niveles de AMPc intracelular, y este aumento hace que se
mantenga abierto un canal en las células de la cripta de Lieberkuhn. En condiciones fisiológicas, en
una persona sana ¿Qué sustancia puede promover que el canal quede abierto también?
a) Somatostatina
b) Óxido nítrico (NO)
c) Péptido intestinal vasoactivo (VIP)
d) Péptido similar al glucagón 1 (GLP1)

20) Paciente de 64 años con tumor abdominal que comprime la cisterna del quilo. En la biopsia de
duodeno tomada como parte del estudio, el patólogo puede observar
a) Dilatación del vaso quilífero central
b) Contracción de las venas de las vellosidades
c) Vellosidades intestinales más largas
d) Engrosamiento de la lámina basal
21) En la enfermedad de Crohn es posible encontrar células de Paneth en el colon. Esto se puede deber
a la especial función de estas células en:
a) la activación de la inmunidad adquirida
b) mantener la inmunidad innata
c) producir hormonas
d) producir Ig A

22) En 1967 se descubrió que la epidemia de Kuru, una enfermedad por priones, en el distrito de Okapa
en Papua Nueva Guinea, era causada por la costumbre de comer la carne de los muertos. Ahora se
sabe que las proteínas priónicas ingresan al organismo a través de:
a) los enterocitos
b) la transmigración
c) las células de Paneth
d) las células M

23) Paciente de 48 años con alteraciones en el tránsito intestinal por diabetes mellitus tipo 2; se presenta
con esteatorrea, flatulencia y malabsorción de grasas. Las pruebas de función hepática y biliar están
dentro de rangos normales. Una causa de la disminución de sales biliares puede ser:
a) el sobrecrecimiento bacteriano
b) la deficiencia de pepsina
c) la deficiencia de elastasa
d) la hiperestimulación del GLUT5
24) Al usar azúcar de mesa (sacarosa) para endulzar su café, el estudiante de medicina sabe que lo más
probable es que para su absorción tendrá utilizar el/los transportadore(s) ________________ que se
encuentran en la membrana apical de los enterocitos.
a) SGLT-1
b) GLUT2 y GLUT5
c) SGLT-1 y GLUT5
d) SGLT-1 y GLUT2

25) Una persona con la producción normal de lactasa; cada vez que toma leche, los productos de la
degradación de la lactosa por parte de la lactasa ingresarán al enterocito usando el/los
transportador(es) _____________: a) SGLT-1
b) GLUT2 y GLUT5
c) SGLT-1 y GLUT5
d) SGLT1 y GLUT2

26) En una persona sana, luego de una comida basada en carnes rojas cocinadas con baja cantidad de
sal, al enterocito pueden ingresar solamente:
a) aminoácidos
b) aminoácidos y dipéptidos
c) aminoácidos, dipéptidos y tripéptidos
d) aminoácidos, dipéptidos, tripéptidos y tetrapéptidos

27) Paciente con deficiencia congénita de procolipasa, sufre de esteatorrea cada vez que come comidas
ricas en grasas. En el estudio de composición de las heces, uno espera encontrar una cantidad
aumentada de: a) colesterol
b) triglicéridos
c) fosfolípidos
d) lisolecitina

28) Al tomar su café en Starbucks, un estudiante de medicina sufre una quemadura de primer grado en
el tercio anterior de la superficie dorsal de la lengua. La información de dolor es transmitida por el
nervio: a) cuerda del tímpano
b) glosofaríngeo
c) lingual
d) facial
29) Paciente de 32 años con esteatosis hepática no alcohólica. Se le realiza una biopsia hepática que
confirma la esteatosis; en el tejido se observan depósitos de lípidos en los hepatocitos, los cuales
contienen principalmente: a) triglicéridos
b) colesterol
c) ácidos grasos libre
d) acil-carnitina

30) Paciente de 21 años que es estudiante de medicina, llega a consulta refiriendo que en épocas de
exámenes su piel se vuelve amarillenta. Le realizan el diagnóstico de Síndrome de Gilbert asociado a
mutación del gen UGT1A1, luego de unas semanas acude a la consulta por ictericia asociada a resfrío.
Al hacerle un análisis de sangre, usted sabe que encontrará valores elevados de:
a) bilirrubina directa
b) bilirrubina indirecta
c) bilirrubina tipo delta
d) fosfatasa alcalina
31) Se realizó un experimento en el cual se inyectó tinta china en el peritoneo de ratas de laboratorio. Al
realizarse una biopsia hepática de dichos animales, se encontró que el tinte negro estaba depositado
en: a) las células de Ito
b) los hepatocitos
c) las células de Kupffer
d) las células de Disse

32) Paciente de 74 años de edad con shock hipovolémico asociado a deshidratación aguda severa. En
este paciente es posible encontrar hipoxia en zona …… del lobulillo hepático e infarto ……………… del
intestino. a) 1 / mucoso
b) 1 / transmural
c) 3 / transmural
d) 3 / mucoso

33) Niño de 5 años con historia de tres días de evolución caracterizado por fiebre, malestar general,
odinofagia, hiporexia, e irritabilidad. Al examen se observa lesiones ulcerativas de 4 mm de diámetro
en mucosa yugal, con fondo blanquecino y eritema periférico. El diagnóstico más probable es:
a) candidiasis oral
b) leucoplasia
c) aftas
d) herpes

34) Paciente de 52 años con enfermedad por reflujo gastroesofágico de 30 años de evolución. Se realiza
endoscopía en la que se encuentra mucosa eritematosa proximal a la línea Z. Para corroborar la
presencia de lesión preneoplásica, se toma una biopsia de esa zona, en la que se espera encontrar:
a) metaplasia gástrica
b) metaplasia intestinal
c) displasia gástrica
d) adenocarcinoma

35) Un estudiante de medicina termina su último examen final. Al abrir la boca para comer una
hamburguesa con papas fritas, la activación de los receptores muscarínicos de las células acinar y
ductal estimularan un mayor flujo de saliva, con lo cual disminuirá la concentración salival de:
a) potasio
b) bicarbonato
c) sodio
d) cloro
36) Un estudiante toma su desayuno consistente en un pan con mantequilla y queso. Antes que se
puedan digerir las grasas, es necesario que sean emulsificadas. La hormona ………………………. estimula
la liberación de las sustancias emulsificadoras.
a) colecistoquinina (CCK)
b) secretina
c) lipasa pancreática
d) gastrina

37) Al comer un pollo a la brasa entero, con papitas fritas y ensalada, la sustancia que estimulará la
liberación de HCl en el estómago es:
a) el neuropeptido Y
b) la secretina
c) la bombesina
d) la colecistoquinina (CCK)
38) Paciente con disminución marcada del apetito asociado a cáncer terminal, se podría utilizar análogos
de ……………..
para promover la ingesta de alimentos.
a) el péptido similar al glucagón (GLP)
b) la serotonina
c) la secretina
d) la endorfina

39) Un hombre de 51 años presenta de forma súbita vómitos masivos de sangre roja brillante. Tiene
como antecedente hepatitis viral B hace 23 años. En la exploración física: FC 103 latidos/min, PA
85/50 mmHg, se palapa la punta del bazo y niega vómitos. Su hematocrito es 21%, la prueba
serológica de HBsAg es positiva. En la ecografía se observa hígado con nodulaciones. ¿Cuál es la causa
más probable para la hematemesis?
a) Esófago de Barrett
b) Síndrome de Mallory Weiss
c) Varices esofágicas
d) Esofagitis por reflujo

40) Durante el reflejo del vómito, uno de los primeros sucesos es:
a) el peristaltismo inverso
b) la contracción del píloro
c) la apertura de la glotis
d) el esfínter esofágico inferior abierto

41) Paciente de 34 años que acude por diarrea desde hace 4 días asociado a comer papa rellena con ají
en el Estadio Nacional, se exacerba cuando toma lácteos o come grasas. Se acompaña de flatulencia
y episodios de tenesmo. Este cuadro de diarrea e intolerancia a la lactosa de inicio agudo se explica
por la:
a) enfermedad de Crohn
b) infección por V. cholera
c) colitis ulcerativa
d) giardiasis

42) En cuanto a la secreción pancreática, mientras mayor es el flujo, mayor es la concentración de: a)
potasio
b) bicarbonato
c) cloro
d) sodio

43) Paciente de 10 años con diarrea crónica, distensión abdominal, anorexia. Se le ha encontrado
anticuerpos antigliadina y antiendomisio. Es más probable que la diarrea se correlacione con el
hallazgo histológico de: a) adelgazamiento de las criptas
b) linfocitos intramusculares
c) atrofia de las vellosidades
d) úlceras duodenales

44) La glándula parótida está inervada por el par craneal:


a) VII
b) IX
c) X
d) XII
45) Las glándulas de Brunner se encuentran en la:
a) mucosa del esófago
b) submucosa del íleon
c) mucosa del yeyuno
d) submucosa del duodeno

46) ¿Cuál de las siguientes afirmaciones describe correctamente la función de la inervación


parasimpática del tracto gastrointestinal?
a) La norepinefrina es el principal neurotransmisor excitatorio.
b) La actividad parasimpática produce la relajación de los esfínteres
c) La actividad parasimpática excesiva puede provocar un trastorno llamado ileo paralítico
(parálisis del músculo liso intestinal)
d) El pH luminal, la osmolaridad y la distensión muscular son detectados por fibras
parasimpáticas eferentes

47) Al ingerir una cantidad de glucosa por vía oral, esta es interiorizada en las células del organismo más
rápido que si esa misma cantidad de glucosa hubiese sido administrada por vía endovenosa. Este
fenómeno sucede gracias a la acción de la sustancia secretada por las células:
a) G
b) I
c) K
d) S

48) A …………… secreción de HCl en el lumen gástrico, …………….. pH en la sangre venosa gástrica a) mayor
/mayor
b) mayor / menor
c) menor / mayor
d) mayor / igual

49) ¿En cuál de las siguientes situaciones hay un menor flujo de secreción salival?
a) Masticar goma de mascar
b) Imaginarse ser sometido a un examen dental
c) Exposición a olor nauseabundo
d) Sueño
50) Al seccionar el nervio facial a nivel timpánico, usted esperaría:
A. Disminución del gusto en la punta de la lengua
B. Imposibilidad para el cierre del istmo de las fauces
C. Ausencia de la termoalgesia de la lengua
D. Imposibilidad para protruir la lengua
51) En un paciente con hiperestimulación simpática se espera que las ondas lentas tengan un ritmo:
A. Menor en íleon terminal que en el duodeno
B. Mayor en el íleon que el duodeno
C. Mayor en el estómago que el duodeno
D. Mayor en el estómago que en íleon terminal
52) Marque lo correcto en relación al divertículo de Meckel.
A. Se encuentra usualmente a 60 cm de la VIC
B. Contiene mucosa esofágica en algunas ocasiones
C. Se produce en el lado del íleon
D. Se relaciona un defecto en el desarrollo del intestino posterior
53) Al disminuir el pH duodenal por el HCl gástrico, se libera principalmente una hormona cuya célula
diana es :
A. Células S del intestino
B. Células ductales del colédoco
C. Acino pancreáticos
D. Células ductal del Wirsung
54) El ecografista sabe que para poder visualizar el nacimiento de la arteria mesentérica superior, debe
colocar el transductor sobre la piel de la siguiente región abdominal:
A. Epigastrio
B. Hipogastrio
C. Hipocondrio derecho
D. Mesogastrio
55) Durante una cirugía oncológica, el cirujano observa que los órganos abdominales tienen libre
movimiento dentro de la cavidad abdominal, excepto:
A. Vesícula biliar
B. Yeyuno
C. Estómago
D. Colon ascendente
56) Paciente obeso con Covid-19 es intubado por interno inexperto, quien al solicitar que bombeen aire
dentro del tubo endotraqueal, nota que el epigastrio se ditiende. Al sospecha que ha introducido el
tubo en el estómago, también es cierto que:
A. Aumenta la frecuencia de ondas lentas
B. Disminuye el pH gástrico
C. Disminuiría el tono del píloro
D. Aumenta el pH gástrico
57) Paciente con apendicitis aguda, que debuta con dolor en mesogastrio. ese dolor se debea
estimulación de receptores del dolor cuyas fibras van a viajar a la médula espinal a través de:
A. Nervios simpáticos
B. Plexo hipogástrico
C. Nervio esplácnico pélvico
D. Nervio vago
58) La rotación en sentido longitudinal del estómago en el desarrollo embriológico condiciona que el
nervio vago derecho quede a nivel:
A. Izquierdo
B. Oblicuo
C. Anterior
D. Posterior
59) Una de las siguientes sustancias reguladoras, puede actuar de forma paracrina y como hormona.
Marque la correcta:
A. Péptido insulinotrópico dependiente de glucosa
B. GRP
C. Acetilcolina
D. Somatostatina
60) El alcohol y la cafeína estimulan la fase ______________ de la producción de ácido clorhídrico. a)
cefálica
b) gástrica
c) intestinal
d) gástrica y cefálica
ECU 1
Estudiante de medicina de la UPC de 21 años sufre de gastritis aguda ocasionada por comer
en lugares poco higiénicos. Suele consumir caramelos ( chupar ) mientras está en clase hasta
la tarde. Toma gaseosas regularmente (carbohidratos 46%, sodio 53%). También toma
regular cantidad de leche (grasa 35%, lactosa 35%, proteínas 30%), pues le calma un poco el
dolor el ardor que siente por la gastritis. Incluso, cuando puede, se toma dos vasos de agua
fría para calmar las molestias. Ha decidido ir al médico para tratarse pues ya no soporta el
dolor, el cual está seguro que los síntomas se deben a una elevada producción de ácido
clorhídrico en el estómago, y por ello le ha recetado Ranitidina (antihistamínico), con lo que
siente mejoría.

El uso de atropina en este paciente:


- Aumentará el pH del estómago

Entre las sustancias cerebrales que producen ansiedad está la serotonina, la cual también
tiene acción:
- Anorexigénica

El consumo de dos vasos de agua seguidos agua generará indirectamente un aumento en la


liberación de:
- Ácido clorhídrico

En este paciente con gastritis aguda debida a una alta producción de ácido clorhídrico, sería
lógico esperar que el píloro tenga un tono muscular:
- Aumentado

El consumo de leche produce directamente un aumento de los niveles séricos de la


hormona:
- Colecistoquinina (CCK)

El consumo de leche produce directamente un aumento de los niveles séricos de la


hormona:
- Gastrina

ECU 2
Niño de sexo masculino de 2 años de edad, sufre de estreñimiento desde el nacimiento (1
deposición cada 3-4 días). Madre menciona que le estimula la defecación con un
termómetro rectal, y continuo uso de enemas y laxantes. Desde hace 6 meses comienza con
vómitos postprandiales. Los síntomas aumentan en frecuencia y magnitud y están en
relación con los episodios de estreñimiento. No refiere fiebre, tos, diarrea ni lesiones
cutáneas. Al examen físico presenta regular estado general, luce deshidratado. Abdomen
distendido, blando, depresible e indoloro. No se palpan masas abdominales. Se permeabiliza
el canal anal con termómetro rectal, encontrando cierta resistencia. Salida de material fecal
mal oliente en regular cantidad. Exámenes de laboratorio: hemograma normal. Signos
inflamatorios de fase aguda negativos. Alcalosis metabólica leve en sangre venosa.
Radiografía con enema baritado muestra recto y colon sigmoides dilatados (megacolon).
Biopsia profunda: ausencia de células ganglionares en la muestra enviada. Se realiza cirugía
correctiva.

El contenido fecal se detiene en la zona inmediatamente proximal a la zona donde hay una
menor presencia de:
- Péptido intestinal vasoactivo

En cuanto a los reflejos gastrocólico y gastroduodenal en este paciente, indique lo correcto:


- Se pueden considerar reflejos vago-vagales

En este paciente se considera que está abolido el reflejo:


- Rectoesfinteriano

Debido al acúmulo de material fecal en todo el marco colónico, y a la irritación química


asociada, el peristaltismo del íleon distal se debe encontrar:
- Inhibido

Es un reflejo propio de la pared intestinal:


- Peristaltismo

A diferencia de las arcadas, los vómitos presentan apertura de:


- Esfínter esofágico superior

CI 1
Sustancia que inhibe la secreción y la motilidad del estómago prolongando el tiempo de
digestión:
- Péptido insulinotrópico dependiente de la glucosa (GIP)

Marque lo correcto:
- La hernia fisiológica se produce en la sexta semana y es la salida temporal de
asas intestinales a través del cordón umbilical

Marque la respuesta correcta en relación a la gastrina:


- Las células G son las productoras y se encuentran principalmente en el antro
gástrico
El consumir caramelos indirectamente activa la vía:
- POMC/CART

¿En qué capa se encuentra la alteración principal en el Hirschsprung o megacolon


agangliónico?:
- Muscular propia

Con respecto a las ondas lentas, marque la afirmación correcta:


- Son contracciones rítmicas espontáneas

El uso de Ranitidina bloquea el receptor H2 de la histamina en las células parietales. La


histamina llega a estas células por:
- Difusión

La triada sintomática: vómitos explosivos post-prandiales, movimientos peristálticos


epigástricos visibles de izquierda a derecha y nódulo palpable epigástrico subcostal derecho,
pertenecen a:
- Estenosis congénita hipertrófica del píloro.

Durante una cirugía oncológica, ¿la extirpación de cuál de los siguientes órganos se vería
comprometida por la presencia de adventicia?:
- Recto

En cuanto a los reflejos gastrointestinales, un reflejo que estimula el tránsito intestinal es el


reflejo:
- Gastrocólico

El ligamento falciforme divide al hígado en dos lóbulos derecho e izquierdo.


Embriológicamente deriva del:
- Mesenterio ventral

La presencia de atresias y estenosis duodenales se deben básicamente a una:


- Falta de recanalización

Estudiante de medicina de 20 años, se ha amanecido estudiando para su examen de Sistema


Digestivo. No ha probado alimento desde la cena, por lo que se puede afirmar que la
motilidad de esta persona está siendo regulada por:
- Motilina

Paciente con disminución del apetito marcada asociada a cáncer terminal, para promover la
ingesta de alimentos se podría usar análogos de:
- Endorfinas

Las ondas lentas se producen por la apertura cíclica de canales de:


- Calcio

La forma más común de atresia esofágica contiene:


- Estenosis proximal del esófago más fístula traqueoesofágica distal

Al deglutir un bolo alimenticio, es lógico suponer que al pasar por el esófago haya un mayor
consumo de oxígeno en la pared del tercio:
- Proximal

Paciente que come entera una pizza familiar de chorizo y queso. Es posible esperar que
debido a la cantidad de alimento ingerida, las ondas lentas hayan:
- Sufrido ninguna alteración en su frecuencia

La hernia fisiológica se produce dentro de:


- Cordón umbilical

El crecimiento de un adenocarcinoma de páncreas compromete la pared gástrica por


contigüidad. ¿Qué parte del estómago se esperaría esté comprometido?
- Pared posterior del antro

CI 2
Estimula la producción de saliva:
- Vasodilatación periglandular

Durante la secreción de saliva, es de esperarse que las concentraciones de ________ y


______ disminuyan al disminuir el flujo:
- Sodio Bicarbonato

Con respecto a la secreción gástrica de HCl:


- a mayor secreción de HCl en el lumen gástrico, mayor pH en la sangre venosa
gástrica

Respecto a las enfermedades del esófago, marque lo correcto:


- el diagnóstico diferencial de la acalasia es la enfermedad de Chagas esofágica

Con respecto a las lesiones y enfermedades de la boca, marque lo correcto:


- la eritroplasia debe ser biopsiada
Respecto a las glándulas salivales, marque lo incorrecto:
- la glándula sublingual tiene forma de garfio

El omeprazol actúa sobre la membrana _____________ de la célula ____________


- apical / parietal

Durante el sueño, la concentración de bicarbonato en la saliva:


- Disminuye

Durante el ataque con gas sarín (bloqueador de la acetilcolinesterasa) en el metro de Tokio,


en 1995, el personal de salud notó que los pacientes afectados presentaban:
- Hipersalivación

La célula mucosa del cuello gástrico produce:


- Moco

ECU 3
El caso:
Paciente de 54 años con antecedentes de alcoholismo, gastritis crónica, tabaquismo pesado,
obesidad, cálculos biliares y cirrosis, es llevado a la emergencia por dolor abdominal en
epigastrio irradiado a la espalda y trastorno del sensorio.
Al examen físico: presión arterial 85/50 mmHg, frecuencia cardíaca 100 latidos/min,
frecuencia respiratoria 18 x minuto, temperatura axilar 36°C.
Conjuntivas pálidas, escleras ictéricas nevus arácnidos en tronco, distensión abdominal
marcada, cabeza de medusa, matidez desplazable en ambos flancos e hipogastrio, dolor a la
palpación de abdomen.
Tiempo de protrombina: 24 seg (testigo: 13 seg); TPT: 38 seg, glicemia: 165 mg/dL, uremia:
20 mg/dL, ASAT: 76 UI/L, ALAT: 22 UI/L, albumina: 2,5 g/dL, bilirrubina total: 2,6 mg/dL,
bilirrubina directa: 1,4 mg/dL, amilasa sérica 4000 U/L.

En todas las preguntas marque la mejor respuesta.


De las siguientes sustancias secretadas por los órganos de este paciente, la más alcalina es la
secreción:
- Pancreática

En esta paciente, el aumento de la Amilasa sérica se debe directamente a una lesión de


- Páncreas

En cuanto a la gastritis de este paciente, se encontró que era producida por la bacteria
Helicobacter pylori. Esta bacteria sobrevive en el medio ácido del estómago gracias a:
- Ureasa
Para evaluar la función hepática podemos medir los niveles séricos de:
- Tiempo de protrombina

El misoprostol, análogo de las prostaglandinas, está mejor indicado en:


- Prevenir daño por AINES

El paciente llegó con trastorno del sensorio posiblemente debida a encefalopatía hepática
producida por déficit en el metabolismo del/la:
- Amoniaco

CI 3
En cuanto al urobilinógeno, se puede decir que es cierto:
- Recircula hacia el hígado

Marque lo correcto:
- Las venas sublobulillares desembocan en las venas hepáticas

En cuanto a la secreción de iones y agua en los conductos biliares es correcto lo siguiente:


- Es estimulada por la secretina

Paciente con tumor neuroendocrino productor de secretina, debido a lo cual se puede


esperar que su secreción pancreática, comparada con la de una persona sana en estado de
bajo flujo, tenga una concentración de:
- bicarbonato aumentada

El efecto colerético de las sales biliares se refiere a:


- Estimulo de secreción biliar

Paciente de 42 años con adenocarcinoma ductular. La TC ha demostrado claramente que el


tumor está en el cuello del páncreas y que hay un gran vaso ocluido. ¿Cuál de los siguientes
vasos estaría más probablemente obstruido?
- Vena porta

Las ramas más pequeñas del árbol biliar son:


- Canalículos biliares

La secreción de agua y bicarbonato por el páncreas exocrino se da básicamente en la fase:


- Intestinal
Cuando el alimento se encuentra en el estómago, se produce la liberación de enzimas
pancreáticas básicamente debido a la acción de:
- Vago

La bilirrubina directa aumentada en cirrosis hepática se excreta en la orina debido a:


- Ser hidrosoluble

ECU 4
Caso:
Mujer de 83 años acude a emergencia por dolor abdominal desde hace 4 días, localizado en
epigastrio, irradiado a ambos hipocondrios, nauseas, vómitos y distensión abdominal; tiene
antecedente de cardiopatía hipertensiva, diabetes mellitus tipo II y fibrilación auricular.
Refiere deposiciones diarreicas muco sanguinolentas hace 1 día. Los exámenes iniciales
muestran PA: 110/60 mmHg, FC: 110/mn, leucocitos: 17800, neutrófilos de 93%. TAC
abdómino pélvica se observa oclusión completa de arteria mesentérica superior por trombo
asociado a placa ateromatosa.

Considerando que se ha comprometido el íleon distal, entre otras áreas, la atrofia o


descamación del epitelio de superficie explicaría cuál de los signos o síntomas de la paciente
(marque la mejor respuesta):
- Diarreas mucosanguinolentas

Después de remover quirúrgicamente el territorio gastrointestinal afectado, estaremos


seguros que no quedará en la paciente la siguiente estructura:
- Placas de Peyer

El mecanismo de la diarrea muco sanguinolenta que presenta la paciente, puede mejor


definido como de tipo (marque la mejor respuesta):
- Exudativa

Durante la cirugía, el cirujano observó que además la paciente tenía divertículos en el sigma.
Se sabe que estos divertículos:
- Se pueden asociar a estreñimiento crónico

En esta paciente, ¿cuál de las siguientes sustancias no tendrá una considerable disminución
en su absorción? (marque la mejor respuesta):
- Calcio

PARCIAL
Al evaluar la orofaringe de un paciente, el médico le solicita que abra la boca, saque la
lengua y diga a . Al hacer esta maniobra, nota que el paladar se desvía hacia la derecha, lo
cual le hace sospechar que el paciente sufre de una lesión del nervio craneal:
- X contralateral

Un bolo alimenticio grande y poco masticado se atasca en el esófago, esto ocasiona una
sensación de dolor que es transmitida por los nervios:
- esplácnicos

Para realizar el movimiento mecánico de abrir la boca, primero se necesita:


- fijar el hueso hioides

¿Cuál de las siguientes alternativas se define como la protrusión directa del contenido
abdominal a la cavidad amniótica por un defecto de la pared corporal?
- Gastrosquisis

Un paciente requiere que se le coloque una sonda de alimentación directamente al


estómago (gastrostomía), el cirujano deberá hacer una incisión en la piel del abdomen ¿cuál
de las siguientes raíces nerviosas debe ser anestesiada para este procedimiento?
- T8

En un paciente de 43 años con tumor carcinoide de páncreas productor de gastrina


(Síndrome de Zollinger-Ellison) se puede esperar encontrar una potenciación del reflejo:
- gastrocólico

El mecanismo de la defecación incluye la participación de diversas estructuras ¿Cuál de las


siguientes alternativas es correcta?
- Puede ser mediado por un reflejo intrínseco

Cuando el contenido del estómago ingresa al duodeno, uno de los reflejos que inhiben el
vaciamiento gástrico es a través del:
- sistema nervioso mientérico

Durante la masticación, gran parte del proceso masticatorio se debe a:


- el reflejo masticatorio

Las glándulas salivales tienes conductos para la excreción de la saliva; las glándulas
____________ drenan en las carúnculas sublinguales.
- sublinguales
Los diferentes segmentos del tubo digestivo son susceptibles de reflejos y movimientos
según su contenido. Si colocáramos mediante una sonda un bolo alimenticio directamente
en el tercio medio del esófago:
- se producirá ondas secundarias

En una persona si enfermedad se espera que el tránsito intestinal se vea disminuido cuando
se presenta el reflejo:
- doloroso

El divertículo de Meckel es una anomalía congénita que ocurre por la persistencia del
conducto vitelino y da origen a una estructura sacular, el cual se encuentra en el:
- borde antimesentérico

Si al intubar a un paciente, por error se ingresa el tubo endotraqueal en el esófago y se


insufla el manguito endotraqueal (globo TET), la dilatación de este manguito generará:
- múltiples ondas secundarias

El orificio omental, o hiato de Winslow, se encuentra limitado por el ligamento:


- hepatoduodenal

Paciente de 24 años acude a consulta externa por presentar una fístula oronasal
(comunicación entre la cavidad oral y la cavidad nasal). Está fístula es una consecuencia
tardía de la lesión de un vaso sanguíneo por el antecedente de haber sido operado de
paladar hendido en los primeros años de vida, aparentemente en una campaña gratuita de
corrección de paladar fisurado. ¿Cuál de las arterias palatinas podría haberse lesionado
durante esa cirugía?
- Mayor

Dentro de las anomalías congénitas se puede presentar un tejido pancreático accesorio


¿Cuál es la ubicación más común de este tejido?
- Estómago

Paciente con insuficiencia mitral moderada a severa, con aumento de volumen de la


aurícula izquierda; esta condición tendrá como consecuencia a nivel del sistema digestivo:
- la disfagia a sólidos

El inicio de la fase faríngea de la deglución se debe a estímulos sensitivos que viajan por el
nervio craneal:
- V

El mesocolon transverso se origina en:


- la pared posterior del abdomen

En la digestión de los alimentos, la hormona __________ se libera frente a la presencia de


péptidos y monoglicéridos, y tiene un efecto marcado en la disminución del vaciamiento
gástrico.
- colecistoquinina

En un varón de 47 años con sección medular a nivel de T6 debido a un accidente


automovilístico, sus terapeutas han desarrollado un mecanismo para distender el recto e
iniciar el reflejo rectoesfinteriano, lo cual producirá la contracción de:
- la pared del recto

Los catadores de vino tienen una habilidad increíble al momento de separar los sabores.
Este aumento de la sensibilidad gustativa debido a una mayor cantidad de papilas linguales
y de corpúsculos gustativos se conoce como:
- hipergeusia

Una recién nacida es evaluada por el neonatólogo y evidencia que el canal anal está
completamente cerrado. Este problema se debe probablemente a una anomalía en el
desarrollo de:
- la membrana cloacal

Durante un experimento, se insufla rápidamente dos litros de agua en un globo colocado


dentro del estómago de un voluntario. ¿cuál de las siguientes situaciones del músculo liso
será consecuencia directa de este cambio de volumen en el estómago?
- Despolarización

En muchos países se usa el suplemento de fluor en el agua potable o los dentríficos, con el
fin de hacer el esmalte más resistente a la desmineralización inducida directamente por:
- el ácido

La sensación del gusto depende de la presencia de papilas gustativas en la lengua, las cuales
tienen corpúsculos gustativos conteniendo células neuroepiteliales sensoriales. Estas células
neuroepiteliales pueden ser dañadas fácilmente, por suerte, su tiempo de recambio es de
alrededor de:
- 10 días

Paciente de 56 años con accidente cerebrovascular reciente. En la resonancia se observa


daño de los núcleos laterales del hipotálamo. Por este motivo es muy probable que el
paciente sufra de:
- Inanición
Durante el desarrollo de la región cloacal, una cuña de mesodermo ubicado entre el
alantoides y el intestino posterior vendrá a formar el:
- tabique urorrectal

En el conducto anal se encuentra la unión entre las regiones del endodermo y el ectodermo,
esta unión se evidencia al observar:
- la línea pectínea

En una persona sana, el momento adecuado para encontrar los mayores niveles de grelina
en sangre sería:
- antes de come

Dentro de la estructura de los dientes, la parte del diente cubierta por esmalte y que se
puede ver mediante la inspección visual de la boca se denomina
- corona clínica

El esófago en su microestructura tiene básicamente adventicia, a excepción de la región


distal, donde tiene serosa, específicamente a partir del nivel de:
- T10

En un paciente con arcadas, se debe considerar que durante la ocurrencia de dichas arcadas,
debemos encontrar contenido gástrico en:
- tórax

El duodeno está constituido por el segmento terminal del intestino anterior y el segmento
proximal del intestino medio ¿Cuál de las siguientes alternativas describe mejor este lugar
de unión entre los dos intestinos?
- Distal al origen de la yema hepática

En una persona sana, el uso de atropina producirá a nivel del estómago:


- Aumentará el pH del estómago

En una persona sana, el consumo de leche produce indirectamente


- Inhibición del vaciamiento gástrico

La motilidad del colon es importante y lenta comparada con la del intestino delgado. Los
movimientos en masa ocasionan la:
- distensión rectal
Con respecto a la motilidad gástrica, los potenciales de acción disminuyen en frecuencia por
efecto de:
- el péptido insulinotrópico dependiente de glucosa

La sensación del gusto depende de la presencia de papilas gustativas en la lengua, de las


cuales, algunas de ellas tienen un surco terminal por donde drenan unas glándulas salivales
linguales (llamadas glándulas de von Ebner). Esta descripción se refiere a las papilas:
- circunvaladas
1. Un niño de 2 años es llevado a la consulta por diarrea persistente y edema de
las extremidades, además falta de crecimiento y desarrollo en relación a su
edad. Los análisis de sangre revelan que tiene concentración plasmática baja de
proteínas (hipoproteinemia). Durante la endoscopía duodenal, se coloca
colecistocinina (CCK) endovenosa y se recoge muestras del líquido duodenal; el
resultado del líquido confirma incapacidad para hidrolizar proteínas a un pH
neutro, esta situación mejora al añadir una pequeña cantidad de tripsina. El
paciente probablemente esté sufriendo la falta congénita de
-Enterocinasa
2. Experimentalmente se incrementa la velocidad de la secreción salival con
una sustancia, el análisis de la composición de esta saliva obtenida se
espera encontrar…………..
-Disminución de concentración de potasio
3. Paciente varón de 46 años soltero, consulta por odinofagia y bajo de peso, tiene
antecedente de tuberculosis desde hace 3 meses y es fumador crónico (10
cigarrillos por día); al evaluar la cavidad oral se identifica lesión blanquecina en
el dorso de la lengua y paladar blando, las lesiones se desprenden con el baja
lengua dejando una base eritematosa. Esta lesión corresponde probablemente
a
……………………….
…..
-Candidiasis oral
4. Paciente mujer de 35 años acude a consulta por sensación de sequedad y
lesiones en cavidad oral. Al examen se observa atrofia de la mucosa, fisuras y
úlceras; nota además sequedad e irritación de la córnea y aumento del tamaño
de las glándulas parotídeas. Su diagnóstico más probable es artritis reumatoide;
el hallazgo más probable en una biopsia de glándula parótida es……..….
-Gran infiltración de linfocitos y células plasmáticas
5. Un paciente con anemia acude con su médico quejándose de episodios
frecuentes de gastroenteritis. Un análisis de sangre revela anticuerpos
circulantes dirigidos contra células parietales gástricas. Su anemia es atribuible
a la hiposecreción de
-Factor intrínseco
6. Dos estudiantes deciden tomar un receso para comer una hamburguesa a la
hora del almuerzo. Antes de llegar a la cafetería, impulsos nerviosos
provenientes del complejo vagal dorsal iniciarán la secreción de ácido gástrico
por la liberación de
…………………….. desde el sistema nervioso entérico.
-GRP
7. Un niño de cuatro años de edad es llevado a la consulta por cuadros diarreicos
frecuentes caracterizados por heces pálidas, voluminosas y fétidas, presenta
bajo peso y talla. Se mide la concentración de cloruro en el sudor y se encuentra
que sus valores son muy elevados. La alteración más importante a nivel de
células ductales del páncreas tiene relación directa con la conductancia
de…………
-Cloro
8. Una mujer de 50 años de edad que sufrió durante varios años resequedad de
los ojos debida a producción inadecuada de lágrimas es enviada con un
gastroenterólogo para evaluación de pirosis crónica. El examen endoscópico
revela erosiones y tejido cicatrizal en la parte distal del esófago justo por
arriba del esfínter esofágico inferior. Las lesiones pueden atribuirse a la
disminución de uno de los siguientes componentes salivales:
-Bicarbonato

9. Se evalúa los valores séricos de las siguientes sustancias a un paciente con


enfermedad hepática terminal; en este paciente se espera encontrar la
combinación con la letra …………
-disminuida, aumentada, disminuida
10. Una mujer de 35 años de edad HIV positiva, se presenta al médico con dolor
abdominal en cuadrante superior derecho e ictericia. La paciente refiere haber
tenido múltiples episodios de ictericia durante los últimos 10 años. Los exámenes
para determinar hepatitis viral, dieron positivos para Hepatitis B, siendo
catalogado el caso como hepatitis crónica con alteración funcional. En un
examen de sangre ¿cuál de los siguientes parámetros está disminuido?
-Albúmina
11. En el reflejo peristáltico del intestino delgado, uno de los siguientes eventos
sucede en la porción oral del bolo alimenticio…………...
-Acción de acetilcolina en el músculo circular
12. Experimentalmente se coloca una dosis alta de secretina en la luz
intestinal duodenal; como consecuencia de esto, en el jugo pancreático de
la misma luz intestinal se observa la disminución de la concentración de
…..………..
-Cl
13. Un varón de 58 años de edad con enfermedad de Crohn severo fue sometido a
una resección ileal. Después de la cirugía este paciente padecerá de esteatorrea,
esto se explica porque …..………..
- La micelas no pueden formarse
14. En un experimento se inserta un balón en el estómago de un voluntario, se infla
poco a poco mientras que se vigilan las presiones intraluminales. Aunque el
volumen del balón aumenta considerablemente, las presiones permanecen
constantes. Esta relación volumen-presión se explica por la liberación local de
…………..
-Óxido nítrico y péptido inhibidor vasoactivo
15. La toxina de Vibrio cholerae causa diarrea debido a…….
-El Incremento de la secreción de cloro por las células de la cripta intestinal
16. ¿Cuál de las siguientes alternativas es una característica de la secreción
exocrina del páncreas?
-Tiene una baja concentración de Cl- respecto al plasma
17. Una madre lleva a su hijo de dos años de edad a la sala de urgencias, estresada
porque el niño deglutió una moneda de 10 céntimos mientras la familia cenaba en
un restaurante. El médico observa mediante fluoroscopía que la moneda se halla
en el estómago y asegura a la madre que la moneda se eliminará con las heces.
El médico recomienda utilizar la respuesta fisiológica que permitirá la evacuación
de la moneda del estómago al intestino ………….…..
-Son los movimientos de mezcla y trituración
-. Es provocada por el ayuno
18. Las estructuras en el hígado que permite que los productos metabólicos unidos a
proteínas tengan acceso a las membranas basolaterales de los hepatocitos,
son….. -Las fenestras sinusoidales
19. La composición de la bilis es modificada conforme fluye por los conductillos
biliares. Durante este tránsito se espera que aumente la concentración de…….
-Monómeros de ácido biliar
-Ig A
20. Se mide experimentalmente el contenido gástrico de dos personas. La persona
“A” tiene alto contenido de grasa y la persona “B” tiene un contenido hipertónico
¿Cuál de las siguientes es correcto respecto al vaciamiento gástrico?
- Hay ralentización del vaciado gástrico en ambos casos
21. El examen endoscópico de un paciente con hipertensión portal grave revela
venas tortuosas que sobresalen hacia la luz del esófago. El paciente recibe
tratamiento quirúrgico mediante la colocación de una derivación que conecta la
vena porta a la vena cava. Después de la operación el riesgo de encefalopatía y
el
riesgo de sangrado de várices ……………..
-Aumentará/disminuirá
22. Un paciente varón de 18 años de edad acude al médico para sus exámenes
de rutina. Sus resultados de laboratorio muestran un valor de bilirrubina
sérica de 4 mg/dl y una bilirrubina directa de 0,3 mg/dl. Las pruebas de
función hepática son normales. La alteración que explica mejor este caso es
por la deficiencia de
………………..
-Glucuronil transferasa
23. Un hombre de 57 años de edad es llevado a urgencias con hematemesis
masiva rojo brillante, a su llegada se halla inconciente con PA: 80/40 mm Hg y
FC: 124 lat/min. Luce ictérico con presencia de “arañas vasculares en el tórax
anterior y extremidades”, abdomen distendido con signo de oleada positiva.
Se encuentra esplenomegalia y pérdida de la masa muscular en
extremidades. La anastomosis vascular responsable del sangrado en este
paciente es
-Vena gástrica izquierda y vena ácigos
24. Un estudiante de medicina está comiendo un plato de comida a base de
champiñones, espárrago y salsa de soya. El sabor umami contenido en todos
estos alimentos actúa a nivel de los botones gustativos estimulando
………………..
-Un receptor acoplado a proteína G
25. Un hombre de 22 años de edad se presenta al médico con una historia de 1 año
de evolución caracterizado por dolor recurrente en fosa iliaca derecha y diarrea.
Manifiesta además pérdida de peso de 8 kg durante este periodo. La
colonoscopía revela múltiples lesiones en el ileon terminal y colon. La biopsia
de estas lesiones revela engrosamiento, inflamación y ulceración de la mucosa.
El diagnóstico más probable en este caso es…….
-Enfermedad de Crohn
26. Varón de 61 años que consulta por dolor retro esternal intenso desde hace 6
horas y después de vómitos intensos y repetidos; al examen se observa disnea,
cianosis, hipotensión y signos clínicos de shock. La radiografía simple de tórax
muestra neumomediastino. El líquido en el espacio pleural aspirado tiene alta
concentración de amilasa. ¿Cuál de las siguientes alternativas puede explicar
este cuadro clínico? -Rotura espontánea de esófago
27. La secreción del ácido en la célula parietal gástrica se lleva a cabo por una
ATPasa especifica que intercambia hidrogeniones (H+) del citosol por…..
-K +
28. En condiciones normales el ingreso de 600 ml de líquido es el estómago provoca
un aumento de presión intragástrica de unos 12 cm de H2O. Después de una
vagotomía (corte del nervio vago) es de esperar que el ingreso del mismo
volumen de líquido provoque lo siguiente: …………………………………
-Un aumento mayor de la presión
29. Una paciente de 30 años de edad es sometida a una cirugía en oído medio
derecho por un problema de otoesclerosis. Luego de la cirugía refiere alteración
en la percepción de sabores. Al evaluar el caso usted esperaría
encontrar……….
-Alteración en la sensación del gusto en los dos tercios anteriores de la
lengua
-Sensación del dolor, tacto y temperatura conservada en toda la lengua

30. ¿Cuál de las siguientes alternativas es correcta?


-Las sales biliares desconjugadas son absorbidas preferentemente en el
colon
31. En un paciente de 45 años de edad con colestasis biliar, se encuentra una
elevación de los niveles sanguíneos de fosfatasa alcalina hasta 3 veces la cifra
normal. ¿Cuál de las siguientes alternativas estará también elevada como
evidencia del daño de la vía biliar?
-Gamma glutamil transpeptidasa
32. Revisando la angiografía de un hombre de 70 años en estudio por aneurisma de
aorta abdominal el radiólogo informa de la presencia de una oclusión completa
de la arteria mesentérica inferior. El paciente se encuentra completamente
asintomático.
¿Cuál de las siguientes arterias se anastomosa a la sistema arterial
de la mesentérica inferior?
-Cólica media
33. Lactante de 3 meses de vida es atendido por presentar diarrea, se administra
una solución de glucosa y electrólitos por vía oral. La proteína de membrana
apical que explica la capacidad de esta solución para proporcionar aporte de
glucosa e hidratación es ………..
-SGLT-1
34. Paciente ha sufrido herida de bala en el abdomen, se le ha tenido que extirpar el
segmento medio y distal del ileon. En este caso la síntesis hepática de sales
biliares estará …..…..
-Incrementada por estímulo de la enzima colesterol 7 alfa hidroxilasa
35. Un varón de 75 años ingresa al consultorio por presentar ictericia marcada de
piel y las escleras. El estudio del paciente mostró que presentaba un tumor que
obstruía la totalidad del conducto hepático común. ¿Cuál de las siguientes
estructuras se encontrará dilatada en este paciente?
-Conductos de Hering
36. En un paciente con insuficiencia renal crónica, el déficit en la absorción de
calcio a nivel del enterocito se debe a lo siguiente:
-No se convierte la 25 hidroxicolecalciferol a 1,25 dihidroxicolecalciferol
37. Varón de 30 años es traído a emergencia por agresión abdominal con arma de
fuego (pistola) y es sometido a laparotomía exploratoria, observándose isquemia
del colon ascendente y parte del colon trasverso ¿la lesión de cuál de las
siguientes arterias explicaría esta isquemia?
d. Mesentérica superior
38. Respecto a las sustancias gastrointestinales que regulan la secreción pancreática;
marque la afirmación correcta:
b. La acetilcolina es capaz de estimular la secreción enzimática y de bicarbonato
del páncreas
39. Ante una lesión del X par craneal, ¿cuál de los siguientes músculos mantiene
conservada su función?:
b. Tensor del velo del paladar
40. Experimentalmente se utiliza atropina (anticolinérgico) para inhibir la secreción
de gastrina, sin embargo la secreción de esta hormona se sigue dando ante
estímulos vagales. Esta situación se explica porque la atropina:
d. No bloquea la acción del péptido GRP

41. Un varón de 50 años es sometido a extirpación del duodeno y parte proximal del
yeyuno. Esta situación ocasionaría la pérdida de las células ……….. ,
productoras de ………………… que estimula la secreción de bicarbonato por el
páncreas.
“S” / secretina
42. Se evalúa la expresión de la proteína Agrp en una persona con alteración del
apetito; lo correcto respecto a esta proteína es…..
La mutación del gen que la codifica produce adelgazamiento
43. Juana cae de la bicicleta y se fractura la región anterior del hueso maxilar superior
con compromiso de la fosa incisiva. Al examen físico de la región esperaría
encontrar alteración en la sensibilidad de la encía …………………
palatina anterior
44. Recién nacido es atendido por el neonatólogo y luego entregado a su madre para
dar de lactar; la madre al dar de lactar observa coloración azulada de labios,
acompañado de tos persistente, dificultad respiratoria y distención abdominal. Se
le intenta colocar una sonda nasogástrica pero esta retorna a la cavidad oral en
todos los intentos. ¿Cuál de las siguientes anomalías del desarrollo es el más
probable en este caso?
b. Atresia esofágica proximal con fístula traqueoesofágica distal l
45. ¿Cuál de los siguientes mecanismos ocurre durante la defecación?
En la posición de “cuclillas” el músculo puborrectal se halla relajado
46. Un paciente luego de un accidente sufre lesión del piso de la boca, se constata
daño del nervio “cuerda del tímpano”, en este caso se esperaría en
47. contrar disminución de la………………………….… de la lengua
Sensación del gusto en los dos tercios anteriores
48. ¿Cuál de las siguientes afirmaciones es la correcta sobre la gastrina?
Actúa en la célula diana mediante su receptor CCk tipo B
49. Al recibir un paciente con signos de hipovolemia y antecedente de trauma en
abdomen por accidente de tránsito, usted identifica radiológicamente: lesión de
primera vértebra lumbar y signos de lesión en páncreas; durante la cirugía se
observó pobre irrigación de asas intestinales. El vaso afectado es la arteria ……..
c. mesentérica inferior
50. Un paciente sufre de daño a nivel del cuello con lesión muscular en la región de la
faringe. En el examen físico se determina dificultad para la elevación de la faringe y
para el cierre del itsmo de las fauces. En este caso, probablemente esté afectado el
músculo:
c. palatofaringeo
51. Varón de 50 años a quien le realizan la curación de la segunda molar de la
arcada superior derecha. En un momento determinado, el paciente acusa de
intenso dolor de la pieza dentaria en tratamiento. La vía aferente del dolor
viaja a través del nervio …………
a. trigémino V2
52. La distención gástrica por los alimentos produce incremento de secreción de
HCl mediante la producción de ………….. que estimula a las célulasvía
proteína
………..
a. gastrina / parietal / Gq

53. Un niño de tres años llega a emergencia con disfagia (dificultad para tragar),
dolor retro esternal, salivación y llanto. Se sospecha de ingesta de cuerpo
extraño (moneda) en el esófago; al ser evaluado se constata en una radiografía
presencia de cuerpo extraño a nivel de C6 (6° vértebra cervical). El cuerpo
extraño estará suspendido a nivel del estrechamiento producido por………..
c. el músculo cricofaríngeo
54. La triada portal (arteria hepática, vena portal y conducto biliar común) está
contenida en el ligamento …….……… y derivan embriológicamente del ……
a. hepato duodenal / mesenterio ventral
55. Un paciente refiere no percibir algunos sabores, al examen físico se constata
alteración en la percepción de sabores y del dolor en el tercio posterior de la
lengua ¿Cuál de los siguientes nervios estará alterada en su función?
c. Glosofaríngeo (IX par)
56. En el caso de un paciente con gastrinoma (tumor productor de gastrina), la
presencia de úlceras duodenales y erosión de la mucosa gástrica, se debe
principalmente a…….
c. el exceso de HCl por estímulo de receptores CCK-B en la célula parietal
56. El reflejo entero gástrico se caracterizan por:
d. originarse debido a la distensión duodenal y presencia del quimo ácido
57. Mauricio tiene dificultad para deprimir el paladar y elevar la parte posterior de la
lengua. En este caso estará afectado un músculo, específicamente el músculo
…………….
b. extrínseco – palatogloso
58. En condiciones normales, el ingreso de 600 ml de líquido es el estómago provoca
un aumento de presión intragástrica de unos 12 cm de H2O. Después de una
vagotomía (corte del nervio vago) es de esperar que el ingreso del mismo volumen de
líquido ocasione
………………………………… de la presión
intragástrica.
c. un aumento mayor
59. La explicación fisiológica de presentar somnolencia de 30 minutos a 1 hora
después de ingerir alimentos, se explica por: a. Aumento del cloro intraluminal
e. Aumento de la alcalinidad sanguínea
60.Se presenta un paciente, el cual presenta un antecedente de tuberculosis
intestinal, por lo cual, se le resecó 80 cm de íleon distal. Desde el punto de vista
fisiológico, el paciente puede presentar una de las siguientes alteraciones: a.
Disminución de la secreción de Vitamina B12

e. Disminución de la absorción de ácido glicocólico


61. Un paciente es sometido experimentalmente a un fármaco que modifica el flujo
salival, obteniéndose un volumen de saliva de 288 ml en 6 horas. En este caso las
concentraciones de electrolitos y bicarbonato en la saliva obtenida varían de la
siguiente manera: a. ↑ Na+,
↓ K+, ↑ Cl-, ↑ HCO3-

1. b. ↓ Na+, ↓ Cl-, ↑ K+, ↓ HCO3-

62. Uno de los siguientes elementos debería hallarse con más probabilidad en el
esófago de un paciente que sufre de reflujo gastro esofágico…
a. Pepsina

63. Un paciente de 40 años cursa con anemia de 8g/dl, aqueja además de astenia y
sensación de hormigueo bilateral en los miembros inferiores, al examen se halla alteración
de la sensibilidad a la vibración y camina con ampliación de la base de sustentación. Uno
de los siguientes procedimientos sería de ayuda para el diagnóstico de este paciente:
a. Tomografía cerebral
b. Biopsia de la mucosa gástrica

64.Paciente de 60 años ingresa por caída hace 1 hora y pequeño hematoma en cuero
cabelludo, al examen físico ampliado se observa ictericia de piel y mucosas generalizada,
abdomen blando, se palpa estructura quística no dolorosa en hipocondrio derecho que
corresponde a vesícula biliar (signo de Courvoisier), en los exámenes de laboratorio se
halla niveles bajos en la formación de estercobilinógeno y urobilinógeno en heces,
incremento de la bilirrubina conjugada en la orina, elevación de fosfatasa alcalina y
gamma glutamil transpeptidasa séricas. El presente cuadro puede ser explicado por: a.
Reabsorción de hematoma
c.Carcinoma de la cabeza de páncreas
65. Un recién nacido presenta vómitos biliosos poco tiempo después de cada alimento.
Al preguntar a la madre sobre antecedentes, ella recuerda que tuvo polihidramnios
durante la gestación, pero un análisis de cariotipo fue normal. Una de las siguientes es la
causa más probable de estos hallazgos en el recién nacido: a. Enfermedad de
Hirschprung
e. Malrotación de la yema pancreática ventral
66.En un estudio de la secreción de hormonas gastrointestinales, sus concentraciones en
la vena porta se midieron durante perfusión luminal del intestino delgado con soluciones
de diversas magnitudes de pH. ¿Qué hormona aumentará en el plasma de la vena porta
durante perfusión a través del intestino con una solución de pH 3?
a. CCK
e. secretina
67.Paciente de 30 años que ingresa a causa de un traumatismo abdominal cerrado. En la
exploración se aprecia discreta palidez de piel y mucosas, auscultación pulmonar normal,
taquicardia de 120 /min. Discreta distensión abdominal y matidez en flancos; el
hematocrito, que era prácticamente normal al ingreso, disminuye a 30% a las tres horas.
En la Rx de tórax se objetiva fractura de las costillas 10-11 izquierdas. La causa más
probable de la anemización en este paciente es: a. traumatismo renal con hemorragia
retroperitoneal.
c. rotura de bazo con hemoperitoneo.
68. Revisando la angiotomografía de un hombre de 70 años en estudio por aneurisma
de aorta abdominal, el radiólogo le informa de la presencia de una oclusión completa
de la arteria mesentérica inferior. El paciente se encuentra completamente
asintomático. La oclusión de la arteria mesentérica inferior cursa de manera
asintomática en muchas ocasiones ya que el territorio que irriga puede recibir flujo
proveniente de la arteria:
a. cólica derecha
e. cólica media

69. En las patologías de esófago es importante conocer bien la anatomía esofágica. ¿Cuál
de las siguientes afirmaciones es correcta? a. El esófago tiene capa mucosa, muscular y
serosa

c. El esófago torácico pasa por detrás del cayado aórtico

70. A pesar de que pueda haber variaciones anatómicas, lo habitual es que el ciego
sea irrigado por una rama arterial que proviene de unas de las siguientes arterias:
a. Iliaca derecha

d. Mesentérica superior

71. Ante un paciente con una cirugía abdominal urgente, el informe operatorio señala
que se ha realizado una resección de todo el duodeno y del tercio proximal del yeyuno
manteniendo íntegros el estómago y todo el íleon, así como los dos tercios distales del
yeyuno. En el seguimiento nutricional del paciente ¿Qué vitamina o mineral presentará
con menor probabilidad una disminución de su absorción?
a. Cianocobalamina

72. ¿Cuál de las siguientes sustancias forma parte de la secreción biliar? a. Tripsina
Lecitina

73.¿De qué musculo forma parte el ligamento inguinal?


-Oblicuo externo del abdomen
74.¿Cuál de las siguientes enzimas está localizada en el borde en cepillo y juega un rol
en la digestión de proteínas?
e. Carboxipeptidasa A.
75. Una de los siguientes sustancias, NO sirve como un buen agente emulsificante:
a. Colesterol
76. La sustancia que estimula el crecimiento de la mucosa gástrica es:

a. Secretina

d. Gastrina

77.¿Cuál de las siguientes alternativas es una función de la colecistokinina?

a. Relajación de la vesícula para la salida de bilis

d. Secreción de enzimas pancreáticas

78.Con respecto a la anatomía del tronco celiaco, señale lo correcto a. El tronco celiaco
se origina de la cara posterior de la aorta abdominal
d. La hepática común que es una de sus ramas, participa en la irrigación del
estómago.
79. Con respecto a la anatomía del duodeno, marque la respuesta correcta: a. Tiene
una distribución en forma de “C”, que rodea la cola del páncreas
b. La 3ra porción duodenal está contenida en la pinza vascular aortomesentérica
80. En el íleon se absorbe aproximadamente el 95% dea través de la circulación
enterohepática.
a. agua
c. sales biliares
81. La estimula el mecanismo paracrino de la secreción de ácido clorhídrico.
a. histamina
82.En la digestión de proteinas,es el principal estímulo para convertir el
pepsinógeno en pepsina. a. la gastrina
b. el pH ácido
83. Con respecto a la somatostatina, marque lo correcto:
a. Es secretada por las células S del intestino
Interviene en la fase intestinal de la secreción gástrica
84. En pecten anal, es una estructura comprendida entre: a. la línea pectínea y los
senos anales
d. la línea anocutánea y la línea pectínea

85.¿Cuál de las siguientes alternativas es una proenzima pancreática? a. Tripsina

1. b. Elastasa
2. c. Quimotripsinógeno
3. d. Amilasa
4. e. Procarboxipeptidasa C
86. En este paciente, el bloqueo farmacológico de los receptores H2 en la mucosa gástrica:
a) No tiene efecto sobre la secreción de ácido inducida por el vago
b) Evita la activación de adenilciclasa por gastrina
c) Inhibe la secreción de ácido inducida por gastrina y mediada por el vago
d) Causa un aumento en el transporte de potasio por las células parietales gástricas
Se validó la A :)
87. Si se considerara una gastrectomía total para curar la gastritis del paciente, cuál
de las siguientes sustancias ya no se produciría:
a) Gastrina
b) Quimiotripsina
c) Amilasa
d) Pepsinógeno

88. Un paciente hipertenso está tomando un medicamento bloqueador de receptores


alfa 1 adrenérgicos (prazosina) y como efecto secundario se queja de:
d) Lo escaso que es el medicamento
e) No tiene problemas en la salivación
c) Hiposalivación
d) Hipersalivación

89. Con respecto a las lesiones y enfermedades de la boca, marque lo correcto:


f) La leucoplasia se desprende al roce
b) la eritroplasia puede degenerar en adenocarcinoma
c) El muguet oral es una enfermedad bacteriana en inmunodeprimidos
d) la eritroplasia debe ser biopsiada
NOTA: fue validada la opción B ya que no es motivo del curso que sepamos el puto
cáncer.

90. En este paciente, se puede asumir que la pancreatitis ha sido ocasionada por
una disminución en el efecto de:
g) Amilasa
h) Lipasa
c) Inhibidor de la tripsina
d) Entercinasa
91.Un efecto secundario en el estómago por la acción de la secretina es:
i) Disminución en la liberación de pepsinógeno
b) Menor actividad de la pepsina
c) Mayor paso de bicarbonato a sangre periférica
d) Aumento en la producción de factor intrínseco
92. Dentro de los factores protectores de la mucosa gástrica se pueden mencionar
múltiples protagonistas. Uno de ellos es:
j) CCK
k) Gastrina
c) Receptor muscarínico
d) Pepsina
93. La saliva puede tener una variedad de electrolitos en su composición. Entre ellos el
cloro, respecto al cual se puede afirmar:
l) Su mayor concentración se consigue con el flujo bajo
b) Su concentración no llega a ser tan alta como en el plasma
c) Con flujo alto su concentración es mayor que la del plasma
d) Su menor concentración se alcanza con flujo alto
94. En el síndrome de boca seca o síndrome de Sjogren, una de las
complicaciones asociadas es:
a) caries
b) Disminución de la acidez gástrica
c) Aumento en de la producción de saliva
d) Aumento del pH bucal
95. Estimula la producción de
saliva:
a) Vasodilatación periglandular
b) Atropina
c) Fatiga o cansancio
d) Expresión de miedo
96. El omeprazol actúa sobre la membrana de la célula
m) Basolateral/principal
n) apical/principal
o) Basolateral/parietal
d) apical/parietal
97. Para protegerse del entorno ácido, el Helicobacter pylori se autogenera un entorno
de pH menos ácido alrededor suyo, gracias a una enzima que alcaliniza su entorno
local mediante la conversión de:
a) urea en NH3
b) H2O y CO2 en ácido carbónico
c) NH3 en urea
d) H2CO3 en bicarbonato
98. La anemia perniciosa destruye las células:
p) mucosas del cuello
b) oxínticas
c) principales
d) mucosas superficiales
99. La célula mucosa del cuello gástrico produce:
a) Moco
b) ácido clorhídrico
c) pepsinógeno
d) Factor intrínseco
100.El aumento en la acidez del estómago producido principalmente por la
infección de Helicobacter pylori se debe a la disminución de:
a) Somatostatina
b) Bicarbonato por las glándulas de Brunner
c) Secretina
d) Colecistoquinina
101. De las siguientes sustancias secretadas por los órganos de este paciente, la
más alcalina es la secreción:
a) pancreática
b) Esofágica
c) Yeyunal
d) Salival
102.En cuanto a la gastritis de este paciente, se encontró que era producida por la
bacteria Helicobacter pylori. Esta bacteria sobrevive en el medio ácido del estómago
gracias a:
a) ácido clorhídrico
b) Toxina CagA
c) Ureasa
d) Jugo pancreático
103.La lengua está recubierta por epitelio:
c) pseudoestratificado columnar no queratinizado
b) plano estratificado no queratinizado
c) pseudoestratificado columnar ciliado
d) plano estratificado queratinizado
104.El esfínter anal interno tiene musculatura …….. y tiene control ……..
d) lisa / voluntario
b) lisa / involuntario
c) esquelética / simpático
d) esquelética / parasimpático

19) La arteria Aorta proporciona la irrigación al tubo digestivo ¿cuál de las


siguientes arterias proporciona la irrigación al ángulo cólico derecho?
a) mesentérica superior
b) mesentérica inferior
c) frénica inferior
d) tronco celiaco

20) Paciente de 26 años que le cuenta en su historia clínica que cada vez que almuerza a
los 20 min tiene deseo de defecar, le comenta que su hijo de 1 mes le pasa lo mismo
pero más intenso. Esto se explica por el reflejo …….., el cual está …… en el paciente
a) colicoileal / normal
b) colicoileal / alterado
c) gastrocólico / normal
d) gastrocólico / alterado

21) La región del estómago que se comunica con el duodeno es la


a) pilórica
b) cardias
c) cuerpo
d) fórnix

22) Acude a consulta un px que fue diagnosticado de úlcera péptica 3 días antes. Luego
de múltiples pruebas diagnósticas se concluye que el paciente presenta un tumor
secretor de gastrina, ¿cual de las siguientes situaciones estará incrementada?
a) distensión gástrica
b) inhibición del vaciado gástrico
c) secrecion de acido clorhidrico
d) inhibición de la secreción de pepsinógeno
23) En el sistema digestivo, el control del apetito está dado por un complejo sistema de
sustancias y órganos integradores, los cuales regulan la ingesta de alimentos. La
…… es una sustancia orexígena y es sintetizada por el ……
a) leptina / estómago
b) felina / intestino
c) leptina / estómago
d) grelina / estómago

24) Con respecto a la actividad eléctrica del sistema digestivo, marque la


alternativa correcta
a) corresponden a potenciales de acción que están presentes de forma continua
y le dan capacidad de peristalsis autónoma al sistema digestivo
b) la frecuencia de las ondas lentas NO se ve influenciada por la actividad neural ni
las
hormonas gastrointestinales
c) en el estómago las ondas lentas se dan en una frecuencia de 6 x min
d) las ondas lentas son cambios lentos y ondulantes del potencial en reposo
e) la frecuencia de las ondas lentas va de 6 a 12 ondas por minuto

25) Ante una lesión del IX pc, el músculo …… se altera en su función


a) palatogloso
b) estilofaríngeo
c) palatofaríngeo
d) constrictor superior

26) Un varón de 50 años es sometido a extirpación del duodeno y parte proximal del
yeyuno. La pérdida de estímulo hormonal en el páncreas para la secreción
enzimática se explica por la pérdida de células
a) parietales, productoras de factor intrínseco
b) K productoras de factor intrínseco
c) M productora de CCK
d) I productora de CCK

27) Marque la respuesta correcta:


A. El bronquio derecho constituye una de las estrecheces del esófago
B. Todos los órganos del sistema digestivo tienen capa serosa
C. La pared gástrica en el fondo es más delgada que en el cuerpo y antro
D. El esfínter de Oddi rodea a la papila menor duodenal

28) Marque la respuesta correcta en relación a la gastrina:


A. Al distenderse el estómago, se inhibe su producción.
B. Se estimula por la liberación de noradrenalina
C. Las células G son las productoras y se encuentran principalmente en el antro
gástrico
D. Las células G se encuentran principalmente en el fondo gástrico

29) Para poder morder una manzana, es necesario usar el siguiente músculo:
A. Milohiodeo
B. Tensor del paladar
C. Orbicular de los labios
D. Buccinador
30) Sustancia que inhibe la secreción y la motilidad del estómago prolongando el tiempo
de digestión:
A. Enteroglucagon.
B. Polipéptido pancreático
C. Péptido 1 similar al glucagón (GLP-1).
D. Péptido insulinotrópico dependiente de la glucosa (GIP).

31) El nacimiento de la arteria mesentérica superior se puede encontrar en cuál de


los cuadrantes abdominales:
A. Hipocondrio derecho
B. Hipogastrio
C. Epigastrio
D. Mesogastrio
32) Entre las múltiples causas de la Enfermedad por Reflujo Gastroesofágico, se puede
considerar también a una alteración en las del esfínter esofágico inferior:
A. Ondas secundarias
B. Contracciones tónicas
C. Ondas lentas
D. Glándulas subesofágicas

33) Producto de la alimentación, se producen diversas sustancias peptídicas, cininas


y bradicininas, las cuales permiten que:
A. Se produzca neovascularización en los territorios de las arterias abdominales
B. La acción de la lipasa pancreática se vea incrementada
C. El consumo de O2 del intestino aumente ligeramente
D. El flujo sanguíneo intestinal aumente hasta 8 veces

34) El dolor periumbilical o epigástrico en el inicio de una apendicitis aguda se debe a:


A. Estímulo del nervio vago.
B. Íleo secundario.
C. Irritación del peritoneo parietal.
D. Estímulo del sistema simpático.

35) El aumento en la actividad motora de la pared gástrica genera un aumento en


los niveles locales de qué sustancia en la microvasculatura:
A.Adenosina
B. Colecistoquinina CCK
C. Endotelina
D. Gastrina

36) ¿Cuál de los siguientes péptidos inhibe el vaciamiento gástrico?


A. Colecistoquinina
B. Péptido inhibidor gástrico
C. Motilina
D. Gastrina
37) Los músculos de la masticación que producen la retropulsión de la mandíbula son:
A. temporales [mas seguro]
B. maseteros
C. milohioideos
D. pterigoideos
38) En relación a la fisiología gástrica, marque lo correcto:
A. la cimetidina actúa en la región basolateral de la célula parietal
B. la marea alcalina se debe al paso de bicarbonato través de la membrana apical
de la célula principal
C. el cloro difunde hacia el exterior por la la región basolateral de la célula parietal
D. la salida de hidrogeniones a la luz es por difusión facilitada

39) Durante el sueño, la concentración de bicarbonato en la saliva:


A. Se eleva a niveles mayores que los del plasma
B. Aumenta
C. No tiene efecto
D. Disminuye

40) La secreción de saliva es importante en la fisiología digestiva. Su concentración


de potasio llega a ser menor que la del plasma cuando su secreción tiene un flujo:
A. Intermedio
B. Nunca
C. Bajo
D. Alto

42) Respecto a las glándulas salivales, marque lo incorrecto:


A. la glándula parótida produce secreción serosa
B. la glándula sublingual drena a través de conducto de Wharton
C. La glándula parótida drena a través del conducto de Stenon
D. la glándula sublingual tiene forma de garfio

43) Con respecto a la saliva, marque la respuesta correcta:


A. será hipertónica cuando el flujo es bajo
B. a mayor flujo, menor concentración de Na
C. a mayor flujo, menor concentración de cloro
D. el sistema simpático estimula su secreción
CI 3
44) En relación a la circulación hepática, marque lo correcto:
a) Los sinusoides hepáticos transportan sangre mixta
b) La vena porta proporciona el 50% de sangre al hígado
c) La vena porta se forma a partir de la vena esplénica y la mesentérica inferior
d) La arteria hepática deriva de la mesentérica superior

45) Dentro de las funciones de las células de Ito, marque lo incorrecto:


a) Sintetizan colágeno
b) Almacenan vitamina A
c) Se les llama células estrelladas
d) Pueden fagocitar patógenos y actúan como presentadoras de antígeno
46) Paciente con tumor neuroendocrino productor de secretina, debido a lo cual se puede
esperar que su secreción pancreática, comparada con la de una persona sana de bajo
flujo, tenga una concentración de:
a) Sodio aumentada
b) Igual
c) Bicarbonato aumentada
d) Potasio disminuida
47) El GALT se localiza en:
a) Lámina propia
b) submucosa
c) borde en cepillo
d) superficie de criptas de Lieberkühn

48) En relación a la histología hepática, marque lo correcto:


a) la zona 1 se afecta rápidamente en estados de hipovolemia y shock
b) La zona 1 se encuentra cercana a la vena central lobulillar
c) La zona 3 se encuentra más cerca a la vena central lobulillar
d) La zona 3 se encuentra más cerca al eje menor del acino hepático
49) El acino pancreático difere con el de las glándulas salivales en:
a) Contiene células centroacinares
b) No produce secreción serosa
c) El páncreas produce principalmente secreción mucosa
d) No tienen diferencias
50) Durante la digestión de las grasa, para que la lipasa actúe adecuadamente se requiere
que el pH aumento en la luz intestinal, lo cual es logrado, entre otros, por la secreción
de las células:
a) Del conducto interlobulillar
b) Centroacinares
c) Acinares
d) Alfa
51) La secreción de la colecistoquinina (CCK) se produce en la fase:
a) intestinal
b) En las 3 por igual
c) gástrica
d) Cefálica
52) ¿Por cuál de las siguientes células es secretada principalmente la pro
enzima procarboxipeptidasa?
a) Acinares del páncreas
b) Epiteliales del duodeno
c) Ductales del páncreas
d) Centro Acinares del páncreas
53) Una mujer de 43 años dolor en hipocondrio derecho e icterica. En la ecografía se
evidencia cálculos biliares. Estos cálculos lo más probable es que se encuentren
localizados en:
a) conducto colédoco
b) Conducto cístico
c) Vesícula biliar
d) Conducto pancreático secundario
ECU 1:
Estudiante de 21 años sufre de gastritis aguda ocasionada por comer en lugares poco
higenicos. Suele consumir caramelos (“chupar”) mientras esta en base hasta la tarde.
También toma regular cantidad de leche (grasa, lactora, proteinas), pues le calma el
dolor y el ardor que sitnete por la gastritis (tiene dispepsia y cuando toma la leche se le
pasa).
Incluso cuando puede, se toma dos vasos de agua frita y le calma la molestia. Ha
decido ir al medico para tratarse, pues ya no soporta el dolor, el cual esta seguro que
los síntomas se producen por elevada producion de HCl en el estomago, y por ello le ha
recetado ranitidina
1.1) El consumir caramelos eleva los niveles en sangre de una hormona cuya función es la
estimulación de las células.
- Beta del páncreas por GIP el cual es una incretina y por consiguiente
estimula las células pancreáticas

1.2) Consumir caramelos indirectamente actica la via:


-POMP/ CART saciedad

1.3) Consumo de leche produce indirectamente


- CCK inhibición del vaciamiento gástrico mayor tonicidad del esfínter pilórico

1.4) Cuando el px toma dos vasos de agua, genera indirectamente un aumento en la


liberación de:
- vaso de agua distención → g astrina → secreción de HCl

1.5) El uso de ranitidina bloquea el receptor H2 de la histamina en las células parietales, la


histamina llega a estas células por:
- histamina es una hormona paracrina por → difusión
**endocrina es por via hematógena y si fuera neuroendorina es por un NTs

1.6) Aumenta la secreción salival:


- noradrenalina a través de los receptores Beta 2
1.7) En este paciente con gastritis aguda debida a una alta producción de ácido clorhídrico,
sería lógico esperar que el píloro tenga un tono muscular:
- primero la secretina
- luego CCK
**ambas reguladores del HCl, Gatritis aguda debido a una alta producción de HCL
piloro estará aumentado (por la CCK)
1.8) Debido al uso de ranitidina, los valores de somatoestina en sangre:
- ranitidina disminuye acción de gastrina se quiere secretar mas no
actúan los inhibidores como la somatoestina somatoestina disminuye
1.9) El uso de atropina en este paciente:
- Inhibirá la acción de las prostaglandinas
- Aumentará la producción de ácido clorhídrico
- Disminuirá la acción del receptor CCK-B
-Aumentará el pH del estómago
ECU 2:
Niño de sexo masculino de 2 años de edad, sufre de estreñimiento desde el
nacimiento (1 deposición cada 3-4 días). Madre menciona que le estimula la
defecación con un termómetro rectal, y continuo uso de enemas y laxantes. Desde
hace 6 meses comienza con vómitos postprandiales. Los síntomas aumentan en
frecuencia y magnitud y están en relación con los episodios de estreñimiento. No
refiere fiebre, tos, diarrea ni lesiones cutáneas. Al examen físico presenta regular
estado general, luce deshidratado. Abdomen distendido, blando, depresible e indoloro.
No se palpan masas abdominales. Se permeabiliza el canal anal con termómetro
rectal, encontrando cierta resistencia. Salida de material fecal mal oliente en regular
cantidad. Exámenes de laboratorio: hemograma normal. Signos inflamatorios de fase
aguda negativos. Alcalosis metabólica leve en sangre
venosa. Radiografía con enema baritado muestra recto y colon sigmoides dilatados
(megacolon). Biopsia profunda: ausencia de células ganglionares en la muestra
enviada. Se realiza cirugía correctiva.

2.1) Durante la fase esofágica de la deglución, para un bolo alimenticio determinado, a


medida que avanza el bolo la fuerza de la contracción se hace más:
- hiperpolarizado
- fuerte
- dependiente de Ach
- debil
2.2) Cuando este paciente ingiera sus alimentos, se espera que al momento de pasar el
bolo alimenticio por el esfínter esofágico superior, la presión intraesofágica
disminuya en:
- la porción proximal al bolo
- el tercio medio del esófago
-el cardias
- el lugar donde se contraiga la muscular propia
2.3) Al examinar la orofaringe del paciente, uno puede hallar fácilmente la amígdala
palatina, pues esta se encuentra inmediatamente detrás del músculo:
-Palatogloso
- Palatofaringeo
- Hiogloso
- Elevador del velo del paladar

2.4) Con respecto a la defecación en este caso, marque la respuesta correcta:


- En posición de cuclillas, el músculo puborectal genera un ángulo más agudo
en el recto
- El sigmoides y el recto están inervados por el nervio vago
- La sensación de defecar sólo se da cuando el recto es ocupado por
heces y alcanzado el 80% de su capacidad
-El esfínter anal comprometido tiene inervación autónoma

2.5) En este paciente [hirschsprung] se considera que está abolido el reflejo:


- Coloileal
-Rectoesfinteriano
- Gastrocólico
- Relajación receptiva

2.6) No se espera que sea causa del vómito:


-Ayuno prolongado
- Estimulación faríngea y del glosofaríngeo
- Irritación de la mucosa gástrica
- Dolor intenso

ECU 3:
Paciente de 54 años con antecedentes de alcoholismo, gastritis crónica,
tabaquismo pesado, obesidad, cálculos biliares y cirrosis, es llevado a la
emergencia por dolor abdominal en epigastrio irradiado a la espalda y trastorno
del sensorio.
Al examen físico: presión arterial 85/50 mmHg, frecuencia cardíaca 100
latidos/min, frecuencia respiratoria 18 x minuto, temperatura axilar 36°C.
Conjuntivas pálidas, escleras ictéricas nevus arácnidos en tronco, distensión
abdominal marcada, cabeza de medusa, matidez desplazable en ambos flancos e
hipogastrio, dolor a la palpación de abdomen.
Tiempo de protrombina: 24 seg (testigo: 13 seg); TPT: 38 seg, glicemia: 165 mg/dL,
uremia: 20 mg/dL, ASAT: 76 UI/L, ALAT: 22 UI/L, albumina: 2,5 g/dL, bilirrubina total:
2,6 mg/dL, bilirrubina directa: 1,4 mg/dL, amilasa sérica 4000 U/L.
3.1) En esta paciente, al aumento de la amilasa sérica, se debe directamente a una lesión
de:
a) páncreas
b) vesícula y árbol biliar
c) estómago
d) hígado
3.2) Considerando que el paciente sufre de gastritis, se puede decir que la secreción de
ácido por la mucosa gástrica
a) involucra transporte activo de hidrogeniones
b) es realizada principalmente por células principales
c) es inhibida por antihistaminas tomadas por pacientes con rinitis alérgica
d) involucra la liberación de HCl de los gránulos zimógenos
3.3) El paciente tiene hemorragia digestiva alta por várices sangrantes como complicación.
Llegando a estar en shock hipovolémico por hemorragia masiva, se encontrara
necrosis hepática en:
a) zona 1
b) no se afectan los lobulillos hepáticos en hemorragia
c) zona 3
d) zona 2
3.4) El misoprostol, análogo de las prostaglandinas está mejor indicado en:
a) cicatrización de úlcera péptica duodenal
b) erradica el helicobacter pylori
c) tratar el sind de Zollinger ellison
d) prevenir daño por AINES
3.5) De las siguientes sustancias secretadas por los órganos de este paciente, la más
alcalina es la secreción:
- Esofágica
- Salival
- Yeyunal
-Pancreática
3.6) En este paciente, se puede asumir que la pancreatitis ha sido ocasionada por una
disminución en el efecto de:
- Lipasa
- Enterocinasa
- Amilasa
-Inhibidor de la tripsina
3.7) ¿Cuál de las siguientes sustancias es segregada por el páncreas?
-Amilasa
- Pepsina
- Quimiotripsina
- Tripsina
3.8) Cada vez que este paciente toma alcohol, la acidificación de la luz del duodeno:
-Disminuye el vaciamiento gástrico
- Aumenta la contracción del esfínter de Oddi
- Aumenta la secreción del ácido gástrico
- Disminuye la secreción pancreática del bicarbonato
SISTEMA
DIGESTIVO
(ME 154)
EXAMEN
FINAL
Ciclo 2018-01

1. Un niño de 2 años es llevado a la consulta por diarrea persistente y edema de las extremidades, además
falta de crecimiento y desarrollo en relación a su edad. Los análisis de sangre revelan que tiene
concentración plasmática baja de proteínas (hipoproteinemia). Durante la endoscopía duodenal, se
coloca colecistokinina (CCK) endovenosa y se recoge muestras del líquido duodenal; el resultado del
líquido confirma incapacidad para hidrolizar proteínas a un pH neutro, esta situación mejora al añadir
una pequeña cantidad de tripsina. El paciente probablemente esté sufriendo la falta congénita de
………….
(Unidad 4, sesión 26, logro 2: Explicar la Digestión y absorción de las proteínas y sus alteraciones)
a. Pepsinógeno
b. PEPT-1
c. Carboxipeptidasas
d. Enterocinasa

2. Experimentalmente se incrementa la velocidad de la secreción salival con una sustancia, el


análisis de la composición de esta saliva obtenida se espera encontrar…………..
(Unidad 3, sesión 17, logro 5 : Explica la Influencia de la velocidad del flujo salival en la composición de
la saliva)
a. Elevación de concentración de bicarbonato, sodio y potasio
b. Elevación de concentración de cloro, sodio y potasio
c. Disminución de concentración de potasio
d. Disminución de concentración de potasio y bicarbonato

3. Paciente varón de 46 años soltero, consulta por odinofagia y bajo de peso, tiene antecedente de
tuberculosis desde hace 3 meses y es fumador crónico (10 cigarrillos por día); al evaluar la cavidad
oral se identifica lesión blanquecina en el dorso de la lengua y paladar blando, las lesiones se
desprenden con el baja lengua dejando una base eritematosa. Esta lesión corresponde
probablemente a ……………………….…..
( Unidad 3, sesión18, logro 1-2 : Describe las enfermedades inflamatorias, infecciosas y proliferativas
de la cavidad oral)
a. Eritroplaquia
b. Candidiasis oral
c. Leucoplaquia vellosa
d. Fibroma en cavidad oral

4. Minero de 32 años de edad, que acude a


centro de salud por presentar de forma
progresiva desde hace 1 año dificultad para
ingerir alimentos sólidos y luego líquidos;
refiere regurgitaciones alimentarias y marcada
pérdida de peso (15 kilos). Radiografia
baritada de esófago como se muestra en la
figura. El presente caso se explica
por……………….
(Unidad 2, sesión 12, logro 4: Identificar y
describir la función de los esfínteres
esofágicos)
a. Contracción incompleta del esfínter esofágico inferior
b. Dificultad para el inicio de la deglución
c. Relajación incompleta del esfínter pilórico
d. Relajación incompleta del esfínter esofágico inferior
5. Paciente mujer de 35 años acude a consulta por sensación de sequedad y lesiones en cavidad
oral. Al examen se observa atrofia de la mucosa, fisuras y úlceras; nota además sequedad e
irritación de la córnea y aumento del tamaño de las glándulas parotídeas. Su diagnóstico más
probable es artritis reumatoide; el hallazgo más probable en una biopsia de glándula parótida
es……..….
(Unidad 3, sesión 18, logro 3: Describe las enfermedades más frecuentes de las glándulas salivales)
a. Hiperplasia de acinos glandulares serosos
b. Gran infiltración de linfocitos y células plasmáticas
c. Gran infiltrado de linfocitos y macrófagos
d. Presencia de acinos normales con hiperplasia de células ductales

6. Un paciente con anemia acude con su médico quejándose de episodios frecuentes de


gastroenteritis. Un análisis de sangre revela anticuerpos circulantes dirigidos contra células
parietales gástricas. Su anemia es atribuible a la hiposecreción de
………………………
(Unidad 3, sesión 20, logro 5: Gastritis crónica. Tipos de gastritis)
a. Factor intrínseco
b. Proteina R (haptocorrina)
c. Pepsinógeno
d. Ácido clorhídrico

7. Dos estudiantes deciden tomar un receso para comer una hamburguesa a la hora del almuerzo.
Antes de llegar a la cafetería, impulsos nerviosos provenientes del complejo vagal dorsal iniciarán la
secreción de ácido gástrico por la liberación dedesde el sistema nervioso entérico.
(Unidad 3, sesión 20, logro 2: Regulación de la secreción gástrica: estimulación, fases de la secreción)
a. Serotonina
b. Óxido nítrico
c. GRP (péptido liberador de gastrina)
d. Péptido intestinal vaso activo

8. Un niño de cuatro años de edad es llevado a la consulta por cuadros diarreicos frecuentes
caracterizados por heces pálidas, voluminosas y fétidas, presenta bajo peso y talla. Se mide la
concentración de cloruro en el sudor y se encuentra que sus valores son muy elevados. La alteración
más importante a nivel de células ductales del páncreas tiene relación directa con la conductancia
de…………
(Unidad 3, sesión 23, logro 5 Explica la Secreción pancreática: formación del jugo pancreático,
influencia de la velocidad de flujo y regulación)
a. Potasio
b. Bicarbonato
c. Sodio
d. Cloro

9. Una mujer de 50 años de edad que sufrió durante varios años resequedad de los ojos debida a
producción inadecuada de lágrimas es enviada con un gastroenterólogo para evaluación de pirosis
crónica. El examen endoscópico revela erosiones y tejido cicatrizal en la parte distal del esófago
justo por arriba del esfínter esofágico inferior. Las lesiones pueden atribuirse a la disminución de
uno de los siguientes componentes salivales:
(Unidad 3, sesión 17, logro 4: Explicar la Formación de la saliva y cuáles son sus componentes)
a. Bicarbonato
b. Lactoferrina
c. Ig A
d. Amilasa
10. Se evalúa los valores séricos de las siguientes sustancias a un paciente con enfermedad hepática
terminal; en este paciente se espera encontrar la combinación con la letra …………
(Unidad 3, sesión 22, logro 5: Describe las Pruebas de función hepática, la Insuficiencia hepática,
encefalopatía hepática e hipertensión portal)

Glucosa Amoniaco Albúmina


a. Aumenta Disminuida Disminuida
da
b. Disminui Aumentada Aumentada
da
c. Aumenta Aumentada Aumentada
da
d. Disminui Aumentada Disminuida
da

11. Una mujer de 35 años de edad HIV positiva, se presenta al médico con dolor abdominal en
cuadrante superior derecho e ictericia. La paciente refiere haber tenido múltiples episodios de
ictericia durante los últimos 10 años. Los exámenes para determinar hepatitis viral, dieron positivos
para Hepatitis B, siendo catalogado el caso como hepatitis crónica con alteración funcional. En un
examen de sangre ¿cuál de los siguientes parámetros está disminuido?
(unidad 3, sesión 22, logro 5: Pruebas de función hepática, Insuficiencia hepática, encefalopatía
hepática e hipertensión portal)
a. Fosfatasa alcalina
b. Albumina
c. Bilirrubina
d. Tiempo de protrombina

12. En el reflejo peristáltico del intestino delgado, uno de los siguientes eventos sucede en
la porción oral del bolo alimenticio…………...
(Unidad 2, sesión 13, logro 4: Explicar la Motilidad del intestino delgado: Contracciones segmentarias y
peristálticas)
a. Disminución de 5 hidroxitriptamina desde las neuronas IPAN
b. Contracción del músculo longitudinal
c. Acción del péptido intestinal vasoactivo (VIP) en el músculo circular
d. Acción de acetilcolina en el músculo circular

13. Experimentalmente se coloca una dosis alta de secretina en la luz intestinal duodenal; como
consecuencia de esto, en el jugo pancreático de la misma luz intestinal se observa la disminución
de la concentración de …..………..
(Unidad 3, sesión 23, logro 5: Explica la Secreción pancreática: formación del jugo pancreático,
influencia de la velocidad de flujo y regulación)
a. Na+
b. Cl-
c. K+
d. HCO3-

14. Un varón de 58 años de edad con enfermedad de Crohn severo fue sometido a una resección
ileal. Después de la cirugía este paciente padecerá de esteatorrea, esto se explica porque
…..………..
(unidad 4, sesión 26, logro 4: Explica las alteraciones en la Absorción de lípidos)
a. El pool de ácidos biliares se incrementa
b. Los quilomicrones no pueden formarse en el lumen intestinal
c. La micelas no pueden formarse
d. El páncreas no secreta lipasa

15. En un experimento se inserta un balón en el estómago de un voluntario, se infla poco a poco


mientras que se vigilan las presiones intraluminales. Aunque el volumen del balón aumenta
considerablemente, las presiones permanecen constantes. Esta relación volumen-presión se explica
por la liberación local de …………..
(Unidad 2, sesión 13, logro 1 Explica la Motilidad gástrica: relajación receptiva)
a. Acetil colina y gastrina
b. Colecistoquinina y óxido nítrico
c. Óxido nítrico y péptido inhibidor vasoactivo
d. Norepinefrina y óxido nítrico
16. La toxina del Vibrio cholerae causa diarrea debido a…….
(Unidad 4, sesión 27, logro 6: Explica el transporte hidroelectrolítico intestinal, toxina colérica)
a. La fosforilación del canal CFTR de los enterocitos de las vellosidades intestinales
b. El Incremento de la secreción de cloro por las células de la cripta intestinal
c. La inhibición de la producción de AMPc por las células epitelailes
d. El incremento de la absorción de agua y sodio a través de las uniones estrechas

17. ¿Cuál de las siguientes alternativas es una característica de la secreción exocrina del páncreas?
(Unidad 3, sesión 23, logro 5: Secreción pancreática: formación del jugo pancreático, influencia de la
velocidad de flujo y regulación)
a. Tiene una baja concentración de Cl- respecto al plasma
b. Es estimulada por la presencia de bicarbonato en el duodeno
c. La secreción enzimática es estimulada principalmente por la gastrina
d. Es hipotónica respecto al plasma

18. Una madre lleva a su hijo de dos años de edad a la sala de urgencias, estresada porque el niño
deglutió una moneda de 10 céntimos mientras la familia cenaba en un restaurante. El médico
observa mediante fluoroscopía que la moneda se halla en el estómago y asegura a la madre que la
moneda se eliminará con las heces. El médico recomienda utilizar la respuesta fisiológica que
permitirá la evacuación de la moneda del estómago al intestino ………….…..
(Unidad 2, sesión 13, logro 2: Explica la Motilidad gástrica: mezclado y vaciamiento)
a. Es por la relajación receptiva
b. Son los movimientos de mezcla y trituración
c. Es provocada por el ayuno
d. Es por la relajación del esfínter esofágico superior

19. Las estructuras en el hígado que permite que los productos metabólicos unidos a proteínas tengan
acceso a las membranas basolaterales de los hepatocitos, son…..
(Unidad 3, sesión 21, logro 4-5: Explica la Organización micro estructural del hígado)
a. Los Canalículos
b. Las fenestras sinusoidales
c. Las uniones intercelulares herméticas
d. Las células de Ito

20. La composición de la bilis es modificada conforme fluye por los conductillos biliares. Durante
este tránsito se espera que aumente la concentración de…….
(Unidad 3, sesión 22, logro 2: Describe la Secreción biliar, visión general del sistema biliar
extrahepático y composición de la bilis)
a. Ig A
b. Glucosa
c. Monómeros de ácido biliar
d. Vitamina A

21. Se mide experimentalmente el contenido gástrico de dos personas. La persona “A” tiene alto
contenido de grasa y la persona “B” tiene un contenido hipertónico ¿Cuál de las siguientes es
correcto respecto al vaciamiento gástrico? (Unidad 2, sesión 13, logro 2: Describe la Motilidad
y vaciamiento gástrico)
a. Hay ralentización del vaciado gástrico solo en “A”
b. El vaciamiento gástrico es más rápido en ambos
c. En ambos casos hay incremento de la motilina
d. Hay ralentización del vaciado gástrico en ambos casos

22. El examen endoscópico de un paciente con hipertensión portal grave revela venas tortuosas que
sobresalen hacia la luz del esófago. El paciente recibe tratamiento quirúrgico mediante la colocación
de una derivación que conecta la vena porta a la vena cava. Después de la operación el riesgo de
encefalopatía ………………….. y el riesgo de sangrado de várices ……………..
(Unidad 3, sesión 22, logro 5: Describe la Insuficiencia hepática, encefalopatía hepática e hipertensión
portal)
a. Aumentará/disminuirá
b. Disminuirá/disminuirá
c. Aumentará/aumentará
d. Disminuirá/aumentará
23. Un paciente varón de 18 años de edad acude al médico para sus exámenes de rutina. Sus
resultados de laboratorio muestran un valor de bilirrubina sérica de 4 mg/dl y una bilirrubina
directa de 0,3 mg/dl. Las pruebas de función hepática son normales. La alteración que explica
mejor este caso es por la deficiencia de ………………..
(Unidad 3, sesión 22, logro 3: Explica la Producción y excreción de bilirrubina. Tipos de bilirrubina e
ictericia)
a. Transaminasas
b. Glucuronil transferasa
c. Hemo oxigenasa
d. La 7 alfa hidroxilasa

24. Un hombre de 57 años de edad es llevado a urgencias con hematemesis masiva rojo brillante, a su
llegada se halla inconciente con PA: 80/40 mm Hg y FC: 124 lat/min. Luce ictérico con presencia de
“arañas vasculares en el tórax anterior y extremidades”, abdomen distendido con signo de oleada
positiva. Se encuentra esplenomegalia y pérdida de la masa muscular en extremidades. La
anastomosis vascular responsable del sangrado en este paciente es ………….…..
(Unidad 3, sesión 21, logro 2: Describe las anastomosis porto sistémicas)
a. Arteria gástrica izquierda y vena ácigos
b. Vena gástrica izquierda y vena ácigos
c. Vena paraumbilical y vena epigástrica inferior
d. Vena gástrica izquierda y vena esofágica superior

25. Un estudiante de medicina está comiendo un plato de comida a base de champiñones, espárrago
y salsa de soya. El sabor umami contenido en todos estos alimentos actúa a nivel de los botones
gustativos estimulando ………………..
(Unidad 2, sesión 10, logro 5: Describe los tipos y mecanismos moleculares para la detección de los
sabores)
a. El ingreso de sodio
b. Un receptor acoplado a proteína G
c. Su receptor específico T1R3
d. El ingreso de hidrógeno

26. Un hombre de 22 años de edad se presenta al médico con una historia de 1 año de evolución
caracterizado por dolor recurrente en fosa iliaca derecha y diarrea. Manifiesta además pérdida
de peso de 8 kg durante este periodo. La colonoscopía revela múltiples lesiones en el ileon
terminal y colon. La biopsia de estas lesiones revela engrosamiento, inflamación y ulceración
de la mucosa. El diagnóstico más probable en este caso es…….
(Unidad 4, sesión 28, logro 5: Describe la Enfermedad inflamatoria intestinal. Generalidades,
morfología y características)
a. Sprue celiaco
b. Enfermedad de Crohn
c. Sindrome de colon irritable
d. Colitis ulcerativa

27. Una de las funciones del músculo señalado es:


(Unidad 2, sesión 8, logro 3: Describir el Piso
de la boca: estructuras blandas que la
conforman)
a. Eleva el paladar blando
b. Recibe inervación del nervio maxilar
c. Deprime el hioides cuando la mandíbula está fija
d. Deprime la mandíbula cuando el hioides está fijo

28. Varón de 61 años que consulta por dolor retro esternal intenso desde hace 6 horas y después
de vómitos intensos y repetidos; al examen se observa disnea, cianosis, hipotensión y signos
clínicos de shock. La radiografía simple de tórax muestra neumomediastino. El líquido en el
espacio pleural aspirado tiene alta concentración de amilasa. ¿Cuál de las siguientes
alternativas puede explicar este cuadro clínico?
(Unidad 3, sesión 18, logro 6: Describe algunas Enfermedades del esófago)
a. Sindrome de Mallory Weiss
b. Rotura espontánea de esófago
c. Neumotórax por probable herida penetrante
d. Perforación de ulcera gástrica de cara posterior, con complicación torácica
29. La secreción del ácido en la célula parietal gástrica se lleva a cabo por una ATPasa especifica que
intercambia hidrogeniones (H+) del citosol por…..
(Unidad 3, sesión 20, logro 1: Explica la Secreción del HCl y sustancias que la alteran)
a. Cl-
b. HCO3-
c. Na +
d. K+

30. En condiciones normales el ingreso de 600 ml de líquido es el estómago provoca un aumento de


presión intragástrica de unos 12 cm de H2O. Después de una vagotomía (corte del nervio vago) es
de esperar que el ingreso del mismo volumen de líquido provoque lo siguiente:
…………………………………
(Unidad 2, sesión 13, logro 1: Describe la Motilidad gástrica: relajación receptiva)
a. Un aumento igual de la presión
b. Que no aumente la presión
c. Un aumento mayor de la presión
d. Una disminución de la presión

31. Una paciente de 30 años de edad es sometida a una cirugía en oído medio derecho por un problema
de otoesclerosis. Luego de la cirugía refiere alteración en la percepción de sabores. Al evaluar el caso
usted esperaría encontrar……….
(Unidad 2, sesión 10, logro 5: Describe la Irrigación e inervación de la lengua)
a. Alteración en la sensación del dolor y temperatura en el tercio posterior de la lengua
b. Alteración en la sensación del gusto en los dos tercios anteriores de la lengua
c. Alteración en la sensación del gusto en la punta de la lengua
d. Sensación del dolor, tacto y temperatura conservada en toda la lengua

32. ¿Cuál de las siguientes alterativas es correcta?


(Unidad 4, sesión 26 : Explica la digestión y absorción de nutrientes y sus alteraciones)
a. En el borde luminal, en cepillo, del intestino delgado, la absorción de sodio únicamente se
realiza asociada a la de glucosa.
b. El lugar principal para la absorción del hierro es el ileon
c. Las sales biliares desconjugadas son absorbidas preferentemente en el colon
d. El proceso de digestión y absorción de la vitamina B12 no se altera en insuficiencia
pancreática.

33. En un paciente de 45 años de edad con colestasis biliar, se encuentra una elevación de los niveles
sanguíneos de fosfatasa alcalina hasta 3 veces la cifra normal. ¿Cuál de las siguientes alternativas
estará también elevada como evidencia del daño de la vía biliar?
(Unidad 3, sesión 22, logro 5: Pruebas de función hepática, Insuficiencia hepática, encefalopatía
hepática e hipertensión portal)
a. Tiempo de protrombina y albúmina sérica
b. Transaminasas hepáticas (ALT y AST)
c. Glucoronil transferasa
d. Gamma glutamil transpeptidasa

34. Revisando la angiografía de un hombre de 70 años en estudio por aneurisma de aorta abdominal el
radiólogo informa de la presencia de una oclusión completa de la arteria mesentérica inferior. El
paciente se encuentra completamente asintomático. ¿Cuál de las siguientes arterias se anastomosa
a la sistema arterial de la mesentérica inferior?
(Unidad 4, sesión 25, logro 1: Identifica la Arteria mesentérica superior e inferior, ramas y anastomosis)
a. Ileal
b. Cólica media
c. Sigmoideas
d. Cólica izquierda

35. Lactante de 3 meses de vida es atendido por presentar diarrea, se administra una solución de
glucosa y electrólitos por vía oral. La proteína de membrana apical que explica la capacidad de esta
solución para proporcionar aporte de glucosa e hidratación es ………..
(Unidad 4, sesión 26, logro 1: Explica la Digestión y Absorción de los hidratos de carbono. Alteraciones)
a. GLUT-5
b. SGLT-1
c. CFTR
d. GLUT-2
36. Paciente ha sufrido herida de bala en el abdomen, se le ha tenido que extirpar el segmento medio y
distal del ileon. En este caso la síntesis hepática de sales biliares estará …..…..
(Unidad 3, sesión 22, logro 4: Explica la formación, función y Circulación entero hepática de lasa sales
biliares)
a. Disminuida por inhibición de la colesterol 7 alfa hidroxilasa
b. Incrementada por estímulo de la enzima colesterol 7 alfa hidroxilasa
c. Incrementada por inhibición de la colesterol 7 alfa hidroxilasa
d. Sin cambios en el ritmo de síntesis

37. Un varón de 75 años ingresa al consultorio por presentar ictericia marcada de piel y las escleras. El
estudio del paciente mostró que presentaba un tumor que obstruía la totalidad del conducto
hepático común. ¿Cuál de las siguientes estructuras se encontrará dilatada en este paciente?
(Unidad 3, sesión 21, logro 6: Describir el árbol biliar intrahepático)
a. Conducto de Wirsung
b. Conductos de Hering
c. Conducto colédoco
d. Conducto cístico

38. Correlaciones las dos columnas y marque la fórmula correcta:


(Unidad 4, sesión 28, logro 1: Diarrea: definición, mecanismos: osmótica, secretoria y exudativa)
1. Enfermedad Hirschsprung( ) heces con moco y sangre
2. Diarrea osmótica( ) intolerancia a lactosa
3. Diarrea secretoria( ) aganglionosis congénita
4. Diarrea exudativa( ) canales de Cl- en las células de la cripta

a.- 4231b.- 1234c.- 2143d.- 4213

39. Respecto a la siguiente imagen que representa una estructura de la mucosa gástrica, la estructura con
número ………..
produce ……………………..
(Unidad 3, sesión 19, logro 4: La glándula fúndica. Funciones y tipos de células con sus características)
a. 3 / pepsina
b. 1 / Pepsinógeno
c. 4 / HCl y factor extrínseco
d. 2 / pepsinógeno

2
40. En un paciente con insuficiencia renal crónica, el déficit en la absorción de calcio a nivel del
enterocito se debe a lo siguiente:
(Unidad 4, sesión 26, logro 6: Explica la Absorción de calcio y hierro)
a. No se convierte la 25 hidroxicolecalciferol a 1,25 dihidroxicolecalciferol
b. No se convierte la 1,25 dihidroxicolecalciferol a 25 hidroxicolecalciferol
c. Se incrementa la producción de Calbindina
d. Existe un descenso de la alfa 25 hidroxilasa renal
SISTEMA DIGESTIVO (ME 154) CLAVES
EXAMEN FINAL
Ciclo 201900

1. Un niño de 2 años es llevado a la consulta por diarrea persistente, edema de las


extremidades y falta de crecimiento en relación a su edad. Los análisis de sangre revelan que
tiene concentración plasmática baja de proteínas (hipoproteinemia). Como parte del estudio
se coloca colecistokinina (CCK) endovenosa y se recoge muestras del líquido duodenal por
endoscopía; el resultado del líquido confirma incapacidad para hidrolizar proteínas a un pH
neutro, esta situación mejora al añadir una pequeña cantidad de tripsina. El paciente
probablemente esté sufriendo la falta congénita de ………….
(Unidad 4, sesión 26, logro 2: Explicar la Digestión y absorción de las proteínas y sus alteraciones)
a. PEPT-1
b. pepsinógeno
c. enterocinasa
d. carboxipeptidasas

2. Paciente mujer de 35 años acude a consulta por sensación de sequedad y lesiones en


cavidad oral. Al examen se observa atrofia de la mucosa, fisuras y úlceras; nota además
sequedad e irritación de la córnea y aumento del tamaño de las glándulas parotídas. Su
diagnóstico más probable es artritis reumatoide; el hallazgo más probable en una biopsia
de glándula parótida es……..….
(Unidad 3, sesión 18, logro 3: Describe las enfermedades más frecuentes de las glándulas
salivales)
a. Presencia de acinos normales con hiperplasia de células ductales
b. Gran infiltración de linfocitos y células plasmáticas
c. Hiperplasia de acinos glandulares serosos
d. Gran infiltrado de linfocitos y neutrófilos

3. Un hombre de 42 años de edad se presenta al médico con una historia de 1 año de


evolución, caracterizado por dolor abdominal bajo y diarreas con crisis sanguinolentas.
Manifiesta además pérdida de peso de 8 kg durante este periodo. La colonoscopía revela
lesión difusa en el colon con afectación del recto. La biopsia de estas lesiones revela
adelgazamiento de la pared, inflamación y ulceración de la mucosa y sub mucosa. El
diagnóstico más probable en este caso es:
(Unidad 4, sesión 28, logro 5: Describe la Enfermedad inflamatoria intestinal.
Generalidades, morfología y características)
a. sindrome de colon irritable
b. enfermedad de Crohn
c. colitis ulcerativa
d. sprue celiaco

4. Dos estudiantes deciden tomar un receso para comer una hamburguesa a la hora del
almuerzo. Antes de llegar a la cafetería, impulsos nerviosos provenientes del complejo
vagal dorsal iniciarán la secreción de ácido gástrico por la liberación dedesde el sistema
nervioso entérico.
(Unidad 3, sesión 20, logro 2: Regulación de la secreción gástrica: estimulación, fases de la
secreción)
a. Serotonina
b. Colecistoquinina
c. Péptido inhibidor vaso activo
d. GRP (péptido liberador de gastrina)

5. Un niño de cuatro años de edad es llevado a la consulta por cuadros diarreicos


frecuentes caracterizados por heces pálidas, voluminosas y fétidas; al examen físico
presenta bajo peso y talla para la edad. Se mide la concentración de cloruro en el
sudor y se encuentra que sus valores son muy elevados. La alteración más
importante a nivel de células ductales del páncreas tiene relación directa con la conductancia
de…………
(Unidad 3, sesión 23, logro 5 Explica la Secreción pancreática: formación del jugo
pancreático, influencia de la velocidad de flujo y regulación)
a. Bicarbonato
b. Potasio
c. Sodio
d. Cloro
6. Se evalúa los valores séricos de las siguientes sustancias a un paciente con enfermedad
hepática terminal; en este paciente se espera encontrar la combinación con la letra …………
(Unidad 3, sesión 22, logro 5: Describe las Pruebas de función hepática, la Insuficiencia
hepática, encefalopatía hepática e hipertensión portal)
Glucosa Amoniaco Albúmin
a
a. Aumenta Disminuida Disminui
da da
b. Disminui Aumentada Aumenta
da da
c. Aumenta Aumentada Aumenta
da da
d. Disminui Aumentada Disminui
da da

7. Una mujer de 35 años de edad HIV positiva, se presenta al médico con dolor abdominal
en cuadrante superior derecho e ictericia. La paciente refiere haber tenido múltiples
episodios de ictericia durante los últimos 10 años. Los exámenes para determinar
hepatitis viral, dieron positivos para Hepatitis B, siendo catalogado el caso como
hepatitis crónica con alteración funcional. En un examen de sangre ¿cuál de los
siguientes parámetros está disminuido?
(unidad 3, sesión 22, logro 5: Pruebas de función hepática, Insuficiencia hepática,
encefalopatía hepática e hipertensión portal)
a. Albumina
b. Bilirrubina
c. Fosfatasa alcalina
d. Tiempo de protrombina

8. En el reflejo peristáltico del intestino delgado ¿Cuál de los siguientes eventos sucede en la
porción caudal del bolo alimenticio?
(Unidad 2, sesión 13, logro 4: Explicar la Motilidad del intestino delgado: Contracciones
segmentarias y peristálticas)
a. Acción del péptido inhibidor vasoactivo (VIP) en el músculo circular
b. Acción del NO (óxido nítrico) en el músculo longitudinal
c. Contracción del músculo longitudinal interno
d. Acción de acetilcolina en el músculo circular

9. Un varón de 58 años de edad con enfermedad de Crohn severo fue sometido a una
resección ileal. Después de la cirugía este paciente padecerá de esteatorrea, esto se
explica porque …..………..
(unidad 4, sesión 26, logro 4: Explica las alteraciones en la Absorción de lípidos)
a. se inhibe la acción de la 7 alfa hidroxilasa
b. el pool de ácidos biliares se incrementa
c. hay mala absorción de ácidos biliares
d. el páncreas no secreta lipasa

10. En un experimento se inserta un balón en el estómago de un voluntario, se infla poco


a poco mientras que se vigilan las presiones intraluminales. Aunque el volumen del
balón aumenta considerablemente, las presiones permanecen constantes. Esta
relación volumen-presión se explica por la liberación local de …………..
(Unidad 2, sesión 13, logro 1 Explica la Motilidad gástrica: relajación receptiva)
a. acetil colina y gastrina
b. norepinefrina y óxido nítrico
c. colecistoquinina y óxido nítrico
d. óxido nítrico y péptido inhibidor vasoactivo

11. ¿Cuál de las siguientes alternativas es una característica de la secreción exocrina del páncreas?
(Unidad 3, sesión 23, logro 5: Secreción pancreática: formación del jugo pancreático,
influencia de la velocidad de flujo y regulación)
a. Es hipotónica respecto al plasma
b. Su mayor estímulo se da en la fase intestinal
c. Es estimulada por la presencia de bicarbonato en el duodeno
d. La secreción enzimática es estimulada principalmente por la secretina
12. Las estructuras en el hígado que permite que los productos metabólicos unidos a
proteínas tengan acceso a las membranas basolaterales de los hepatocitos, son…..
(Unidad 3, sesión 21, logro 4-5: Explica la Organización micro estructural del hígado)
a. los canalículos
b. las células de Ito
c. las fenestras sinusoidales
d. las uniones intercelulares herméticas

13. La composición de la bilis es modificada conforme fluye por los conductillos biliares.
Durante este tránsito se espera que aumente la concentración de…….
(Unidad 3, sesión 22, logro 2: Describe la Secreción biliar, visión general del sistema biliar
extrahepático y composición de la bilis)
a. Ig A
b. Glucosa
c. Protones
d. Vitamina A

14. Se mide experimentalmente el contenido gástrico de dos personas. La persona “A” tiene
alto contenido de grasa y la persona “B” tiene un contenido isotónico ¿Cuál de las
siguientes es correcto respecto al vaciamiento gástrico? (Unidad 2, sesión 13, logro 2:
Describe la Motilidad y vaciamiento gástrico)
a. Hay ralentización del vaciado gástrico solo en “A”
b. El vaciamiento gástrico es más rápido en ambos
c. Hay ralentización del vaciado gástrico solo en “B”
d. Hay ralentización del vaciado gástrico en ambos casos

15. El examen endoscópico de un paciente con hipertensión portal grave revela venas
tortuosas que sobresalen hacia la luz del esófago. El paciente recibe tratamiento
quirúrgico mediante la colocación de una derivación que conecta la vena porta a la vena
cava. Después de la operación el riesgo de encefalopatíay el riesgo
de sangrado de várices ……………..
(Unidad 3, sesión 22, logro 5: Describe la Insuficiencia hepática, encefalopatía hepática e
hipertensión portal)
a. disminuirá / disminuirá
b. disminuirá / aumentará
c. aumentará / disminuirá
d. aumentará / aumentará

16. Un paciente varón de 18 años de edad acude al médico para sus exámenes de rutina. Sus
resultados de laboratorio muestran un valor de bilirrubina sérica de 4 mg/dl y una
bilirrubina directa de 0,3 mg/dl. Las pruebas de función hepática son normales. La
alteración que explica mejor este caso es por la deficiencia de ………………..
(Unidad 3, sesión 22, logro 3: Explica la Producción y excreción de bilirrubina. Tipos de bilirrubina e
ictericia)
a. transaminasas
b. hemo oxigenasa
c. la 7 alfa hidroxilasa
d. glucuronil transferasa

17. Un hombre de 57 años de edad es llevado a urgencias con hematemesis masiva rojo
brillante, a su llegada se halla inconsciente con PA: 80/40 mm Hg y FC: 124 lat/min. Luce
ictérico con presencia de “arañas vasculares en el tórax anterior y extremidades”,
abdomen distendido con signo de oleada positiva. Se encuentra esplenomegalia y pérdida
de la masa muscular en extremidades. La anastomosis vascular responsable del sangrado
en este paciente es ………….…..
(Unidad 3, sesión 21, logro 2: Describe las anastomosis porto sistémicas)
a. vena gástrica izquierda y vena ácigos
b. arteria gástrica izquierda y vena ácigos
c. vena paraumbilical y vena epigástrica inferior
d. vena gástrica izquierda y vena esofágica superior
18. Un estudiante de medicina está comiendo un plato de comida a base de champiñones,
espárrago y salsa de soya. El estímulo del sabor umami contenido en todos estos
alimentos viaja a través del nervio………………..
(Unidad 2, sesión 10, logro 3: Describe la irrigación e inervación de la lengua)
a. Lingual
b. Hipogloso
c. Glosofaringeo
d. Cuerda del tímpano
19. Una paciente de 30 años de edad es sometida a una cirugía en oído medio derecho
por un problema de otoesclerosis. Luego de la cirugía refiere alteración sensitiva de
la lengua. Al evaluar el caso usted esperaría encontrar……….
(Unidad 2, sesión 10, logro 5: Describe la Irrigación e inervación de la lengua)
a. Alteración en la sensación del dolor y temperatura en el tercio posterior de la lengua
b. Alteración en la sensación del dolor en los dos tercios anteriores de la lengua
c. Alteración en la sensación del gusto en el tercio posterior de la lengua
d. Sensación del dolor, tacto y temperatura conservadas

20. En un paciente de 45 años de edad con colestasis biliar, se encuentra una elevación de los
niveles sanguíneos de fosfatasa alcalina hasta 3 veces la cifra normal. ¿Cuál de las
siguientes alternativas estará también elevada como evidencia del daño de la vía biliar?
(Unidad 3, sesión 22, logro 5: Pruebas de función hepática, Insuficiencia hepática,
encefalopatía hepática e hipertensión portal)
a. Tiempo de protrombina y albúmina sérica
b. Transaminasas hepáticas (ALT y AST)
c. Gamma glutamil transpeptidasa
d. Glucoronil transferasa

21. Experimentalmente se incrementa la velocidad de la secreción salival con una


sustancia, en el análisis de la composición de esta saliva obtenida se espera
encontrar…………..
(Unidad 3, sesión 17, logro 5 : Explica la Influencia de la velocidad del flujo salival en la
composición de la saliva)
a. disminución de la concentración de bicarbonato que supera la concentración plasmática
b. aumento de la concentración de cloro y sodio que supera la concentración plasmática
c. aumento de la concentración de bicarbonato que supera la concentración plasmática
d. disminución de concentración de potasio y bicarbonato

22. Lactante de 3 meses de vida es atendido por presentar diarrea, se administra una
solución de glucosa y electrólitos por vía oral. La proteína de membrana apical
que explica la capacidad de esta solución para proporcionar aporte de glucosa e
hidratación es ………..
(Unidad 4, sesión 26, logro 1: Explica la Digestión y Absorción de los hidratos de carbono.
Alteraciones)
a. CFTR
b. SGLT-1
c. GLUT-2
d. GLUT-5

23. Paciente ha sufrido herida de bala en el abdomen, se le ha tenido que extirpar el


segmento medio y distal del ileon. En este caso la síntesis hepática de sales biliares
estará …..…..
(Unidad 3, sesión 22, logro 4: Explica la formación, función y Circulación entero hepática de lasa
sales biliares)
a. Sin cambios en el ritmo de síntesis
b. Disminuida por inhibición de la enzima colesterol 7 alfa hidroxilasa
c. Incrementada por estímulo de la enzima colesterol 7 alfa hidroxilasa
d. Incrementada por inhibición de la enzima colesterol 7 alfa hidroxilasa

24. Un varón de 75 años ingresa al consultorio por presentar ictericia marcada de piel y las
escleras. El estudio del paciente mostró que presentaba un tumor que obstruía la
totalidad del conducto hepático común. ¿Cuál de los siguientes conductos se encontrará
dilatado en este paciente?
(Unidad 3, sesión 21, logro 6: Describir el árbol biliar intrahepático)
a. de Wirsung
b. de Hering
c. colédoco
d. cístico
25. Correlaciones las dos columnas y marque la fórmula correcta:
(Unidad 4, sesión 28, logro 1: Diarrea: definición, mecanismos: osmótica, secretoria y exudativa)

1. Enfermedad Hirschsprung( ) heces con moco y sangre


2. Diarrea osmótica( ) intolerancia a lactosa
3. Diarrea secretoria( ) aganglionosis congénita
4. Diarrea exudativa( ) canales de Cl- en las células de la cripta a.-

4231b.- 1234c.- 2143d.- 4213

26. La vena umbilical obliterada del hígado después del nacimiento se transforma en el ligamento:
(Unidad 3, sesión 21, logro 1: Hígado: relación con la pared abdominal, caras, lóbulos, ligamentos
, hilio hepático)
a. cruzado
b. redondo
c. coronario
d. falciforme

27. Llega a su guardia nocturna una madre que trae a su RN masculino de 2 semanas de vida
con mal estado general y sequedad de mucosas. Usted observa que lacta ávidamente, pero
a las 2 horas presenta vómito postprandial no bilioso en proyectil. Al realizar la historia
clínica, descubre que el lactante recibió profilaxis con macrólidos para tos ferina. Usted
sospecha principalmente en:
(Unidad 1, sesión 4, logro 4: Desarrollo y anomalías del intestino anterior)
a. estenosis pilórica hipertrófica congénita
b. fistula traqueo esofágica
c. estenosis duodenal
d. atresia duodenal

28. En la regulación del apetito y la saciedad, la estimulación experimental crónica del


núcleo ventro medial del hipotálamo producirá:
(Unidad 1, sesión 3, logro 4: explica los mecanismos de control del apetito y saciedad)
a. afagia
b. obesidad
c. hiperfagia
d. activación de neuronas relacionadas a NPY

29. Paciente mujer de 25 años acude por dolor en fosa ilíaca derecha que empeora al toser o
caminar, asociada a náuseas y vómitos por lo cual acude a emergencia. Dos días después
de realizarle una apendicectomía, la paciente desarrolla fiebre alta (39 °C), está hipotensa y
presenta dolor abdominal. La laparotomía exploratoria muestra un gran volumen de
sangre en la cavidad peritoneal por lesión de un vaso producida durante la
apendicectomía.
¿Cuál de las siguientes arterias debe ligarse para detener la hemorragia?
(Unidad 4, sesión 27, logro 4: Irrigación arterial del colon, recto y conducto anal)
a. cólica derecha y arteria rectal superior.
b. ileocólica y arteria cólica media.
c. mesentérica superior.
d. ileocólica.

30. La onda peristáltica secundaria del esófago se caracteriza por ser originada ………
(unidad 2, sesión 12, logro 3: Motilidad esofágica: fases y características)
a. por el plexo de meissner del esófago
b. por el plexo mientérico del esófago
c. por el reflejo de la deglución
d. durante la masticación

31. ¿Cuál de los siguientes es una causa de ictericia con bilirrubina conjugada aumentada?
(Unidad 3, sesión 22, logro 3: Producción y excreción de bilirrubina. Tipos de bilirrubina, ictericia)
a. Ictericia del recién nacido
b. Obstrucción del colédoco
c. Anemia hemolítica
d. Gran hematoma
32. En relación a la absorción de nutrientes, la absorción de dipéptidos y tripéptidos a nivel
de las células epiteliales del intestino delgado, se da principalmente debido a:
(Unidad 4, sesión 26, logro 2: Digestión y absorción de las proteínas. Alteraciones)
a. el incremento de los canales de Cl- en la membrana apical
b. la gradiente de bicarbonato en la membrana basal
c. la gradiente de iones H+ en la membrana apical
d. la gradiente de Na+ en la membrana apical

33. Paciente de 20 años es traído a la emergencia por presentar diarreas desde hace 2 días.
Familiar refiere que las deposiciones son líquidas y abundantes, al examen luce
deshidratado y se plantea que la diarrea es producida por una toxina que estimula la
transformación de ATP a AMPc con apertura de canales de Cl- y pérdida de agua. El tipo de
diarrea más probable es:
(Unidad 4, sesión 28, logro 1: Diarrea: definición , mecanismos: osmótica, secretoria y supurativa)
a. osmótica
b. exudativa
c. secretoria
d. por intolerancia a lactosa

34. Un niño fue operado por una obstrucción intestinal, observándose la presencia de
divertículo de Meckel. Según lo referido, marque lo correcto:
(Unidad 1, sesión 5, logro 2: identifica las anomalías del desarrollo del intestino medio: onfalocele
y gastrosquisis (diferencias), Divertículo de Meckel)
a. el 50% de la población lo presenta
b. se localiza en el íleon muy cerca al yeyuno
c. puede poseer tejido gástrico o pancreático
d. se produce por una mala rotación de los intestinos

35. Marque la
alternativa correcta
respecto a la
estructura marcada
en el gráfico:
(Unidad 3, sesión 22,
logro 2: Secreción biliar.
Visión general del
sistema biliar
extrahepático y
composición de la bilis)

a. Se halla a 2
centímetros
debajo de la
papila
duodenal
mayor
b. Llega el conducto
colédoco y
pancreático
principal
c. Llega el
conducto
hepático
común y
pancreático
principal
d. Llega el conducto pancreático accesorio

36. ¿Cuál de las siguientes moléculas se encontrará aumentada en el citoplasma de las


células parietales de un paciente con sindrome de Zollinguer Ellison?
(Unidad 3, sesión 20, logro 4: Enfermedad ulcerosa péptica: úlcera gástrica, duodenal.
síndrome de Zollinger – Ellison)
a. Péptido liberador de gastrina (GRP)
b. Proteína G estimulante (GS)
c. Inositol Trifosfato (IP3)
d. AMP cíclico (AMPc)

37. Los fármacos inhibidores de la bomba de protones, actúan bloqueando la ………..……..


(Unidad 3, sesión 20, logro 3: Regulación de la secreción gástrica: inhibición, Secreción de
pepsinógeno y factor intrínseco)
a. anhidrasa carbónica
b. ATPasa H+/K+ en la membrana luminal
c. ATPasa H+/K+ en la membrana basolateral
d. ATPasa Na+/K+ en la membrana basolateral
38. Un paciente fue diagnosticado de gastritis autoinmune, ¿cuál de las siguientes
alternativas es FALSA respecto a esta enfermedad?
(Unidad 3, sesión 20, logro 5: Gastritis crónica: helicobacter pylori, autoinmune. Tipos de gastritis)
a. Afecta principalmente el fondo y cuerpo gástrico
b. Se produce hiperplasia de células G secundaria a la aclorhidria
c. El propio sistema inmune destruye principalmente las células parietales
d. Se produce atrofia de la mucosa, aclorhidria, hipergastrinemia y déficit de vitamina B6

39. Marque la correlación correcta:


(Unidad 3, sesión:18, logros:1 y 2: Describe las enfermedades inflamatorias/infecciosas y
proliferativas de la cavidad oral)

1. Herpes virus( ) En relación al abuso de antibióticos


2. Candidiasis oral( ) Lesiones vesiculares como racimo de uvas
3. Eritroplaquia( ) Mega esófago
4. Enfermedad de Chagas( ) Lesión pre cancerígena

a.- 2431b.- 1234c.- 4123d.- 2143

40. En un paciente con insuficiencia renal crónica, el déficit en la absorción de calcio a nivel
del enterocito se debe a lo siguiente:
(Unidad 4, sesión 26, logro 6: Explica la Absorción de calcio y hierro)
a. No se convierte la 25 hidroxicolecalciferol a 1,25 dihidroxicolecalciferol
b. No se convierte la 1,25 dihidroxicolecalciferol a 25 hidroxicolecalciferol
c. Existe un descenso de la alfa 25 hidroxilasa renal
d. Se incrementa la producción de Calbindina
QUIZIZZ
1. Los vasos mesentéricos superiores se hallan a nivel de:
a) Cuello del páncreas

2. El nivel en el que se encuentra el píloro y el páncreas se puede determinar usando el


A) plano transpilórico

3. El dolor de estómago asociado a gastritis se suele ubicar en


a) epigastrio

4. La colecistoquinina inhibe el
a) vaciamiento gástrico

5. Paciente con disfasia ( dificultad para pasar alimentos) con to y disminución de peso. Con
antecedente de tabaquismo pesado. La sospecha es que tenga una disminución del diámetro
esófago a nivel de la
a) tercera estrechez

6. La digestión de las proteínas se inicia en:


a) estómago

7. El estómago recibe información simpática proveniente del:


a) ganglio celíaco

8. El reflujo gastroesofagico tiene múltiples etiologías, una de ellas tiene que ver con alteración a
nivel de:
a) primera estrechez
b) segunda estrechez
c) tercera estrechez
d) cuarta estrechez

9. La fístula retroperitoneal es causada por una falla en el desarrollo de:


a) tabique urorrectal

10. El nervio vago inerva el:


a) músculo estriado del esófago

11. El divertículo de Meckel es un rezago de:


a) conducto vitelino

12. La presencia de orina que sale por el ombligo de un recién nacido casa vez que llora, es posible
que se deba a un defecto en el desarrollo del
a) seno urogenital

13. La fístula retroperitoneal es causada por una falla en el desarrollo de


a) tabique urorrectal

14. Enfermedad asociada con un error en el desarrollo de las células de Cajal:


a) enfermedad de Hirschsprung

15. Aproximadamente en la semana 6 del desarrollo embrionario, el intestino medio gira 90


herniandose a nivel del
a) cordón umbilical

16. El conducto biliar deriva del


a) endodermo

17. Paciente mujer con 54 años con nauseas y vómitos y abdomen agudo quirúrgico, se ingresa a sala
de operaciones donde se encuentra vólvulo de ciego, esto se debe a
a) Falta de fusión del mesenterio

18. Paciente con cirrosis hepática con hipertensión portal, en el que es posible encontrar que los
vasos umbilicales están permeables dentro de
a) ligamento redondo

19. Es normal encontrar glándulas submucosas en CUAL


ES
a) esófago medio
b) esófago proximal
c) esófago distal
d) estómago

20. Cual de los siguientes órganos son intraperitoneales


a) estómago, vesícula biliar, íleon, hígado

Estudiante de medicina de la UPC de 21 años sufre de gastritis aguda ocasionada por comer en lugares
poco higiénicos. Suele consumir caramelos ( chupar ) mientras está en clase hasta la tarde. Toma gaseosas
regularmente (carbohidratos 46%, sodio 53%). También toma regular cantidad de leche (grasa 35%, lactosa
35%, proteínas 30%), pues le calma un poco el dolor el ardor que siente por la gastritis. Incluso, cuando
puede, se toma dos vasos de agua fría para calmar las molestias. Ha decidido ir al médico para tratarse pues
ya no soporta el dolor, el cual está seguro que los síntomas se deben a una elevada producción de ácido
clorhídrico en el estómago, y por ello le ha recetado Ranitidina (antihistamínico), con lo que siente mejoría.

● Para reducir la secreción de HCl en esta paciente se podría usar sustancias similares a:

- Péptido insulinotrópico dependiente de la glucosa (GIP)


● Si se usara atropina en esta paciente, se esperaría que disminuya la liberación de:
- Enzimas pancreáticas

● El consumir caramelos eleva los niveles en sangre de una hormona cuya función es
la estimulación de las células:
- Beta del páncreas

● En este paciente con gastritis aguda debida a una alta producción de ácido
clorhídrico, si se le hiciera un examen de sangre, se encontraría elevados los niveles
de:
- Colecistoquinina

● El consumo de una pequeña cantidad de gaseosa aumentará directamente la


concentración sérica de cuál de las siguientes hormonas:

- Péptido 1 similar al glucagón (GLP-1)

● El consumo rápido de 500 mL de gaseosa aumentará directamente la concentración


sérica de cuál de las siguientes hormonas:

- Gastrina

● Estimulan la secreción ácida gástrica

- Proteínas

● Con respecto a las ondas lentas, marque la afirmación correcta:}

- Son contracciones rítmicas espontáneas

● El uso de Ranitidina bloquea el receptor H2 de la histamina en las células


parietales. La histamina llega a estas células por:

- Difusión

● El consumir caramelos indirectamente activa la vía:

- POMC/CART

● ¿Cuál de los siguientes péptidos inhibe el vaciamiento gástrico?

- Colecistoquinina

● Para poder morder una manzana, es necesario usar el siguiente músculo:

- Milohiodeo

● El crecimiento de un adenocarcinoma de cuello de páncreas puede comprometer la


pared gástrica por continuidad. ¿Qué parte del estómago estaría comprometido con
mayor probabilidad?

- Pared posterior del antro

● El nacimiento de la arteria mesentérica superior se puede encontrar en cuál de los


cuadrantes abdominales:

- Epigastrio
● En cuanto a la colecistoquinina, marque la respuesta correcta:

- Potencia la acción de buffer con bicarbonato

● El aumento en la actividad motora de la pared gástrica genera un aumento en


los niveles locales de qué sustancia en la microvasculatura:

- Adenosina

Niño de sexo masculino de 2 años de edad, sufre de estreñimiento desde el nacimiento (1


deposición cada 3-4 días). Madre menciona que le estimula la defecación con un termómetro
rectal, y continuo uso de enemas y laxantes. Desde hace 6 meses comienza con vómitos
postprandiales. Los síntomas aumentan en frecuencia y magnitud y están en relación con
los episodios de estreñimiento. No refiere fiebre, tos, diarrea ni lesiones cutáneas. Al
examen físico presenta regular estado general, luce deshidratado. Abdomen distendido,
blando, depresible e indoloro. No se palpan masas abdominales. Se permeabiliza el canal
anal con termómetro rectal, encontrando cierta resistencia. Salida de material fecal mal
oliente en regular cantidad. Exámenes de laboratorio: hemograma normal. Signos
inflamatorios de fase aguda negativos. Alcalosis metabólica leve en sangre venosa.
Radiografía con enema baritado muestra recto y colon sigmoides dilatados (megacolon).
Biopsia profunda: ausencia de células ganglionares en la muestra enviada. Se realiza cirugía
correctiva.

● La percepción de la pirosis (sensación de dolor o quemazón en el esófago) asociado


al reflujo gastroesofágico, puede aparecer o exacerbarse debido a:

- Ejercicio

● Considerando que este paciente está sometido a estrés por el agravamiento de su


enfermedad, es posible afirmar que sus ondas lentas están:

- Hiperpolarizadas

● Debido al acúmulo de material fecal en todo el marco colónico, y a la irritación


química asociada, el peristaltismo del íleon distal se debe encontrar:

- Inhibido

● En cuanto a los reflejos gastrocólico y gastroduodenal en este paciente, indique lo


correcto:

- Se pueden considerar reflejos vago-vagales

● Con respecto a la defecación señale el enunciado correcto:

- Es estimulado por un llenado de la cuarta parte del volumen rectal

● El contenido fecal se detiene en la zona inmediatamente proximal a la zona donde


hay una menor presencia de:

- Péptido intestinal vasoactivo

● La presencia de atresias y estenosis duodenales se deben básicamente a una:

- Falta de recanalización

● El ligamento de Treitz característicamente:

- Suspende el ángulo de Treitz

● Al deglutir un bolo alimenticio, es lógico suponer que al pasar por el esófago haya
un mayor consumo de oxígeno en la pared del tercio:

- Proximal
● Estudiante de medicina de 20 años, se ha amanecido estudiando para su examen de
Sistema Digestivo. No ha probado alimento desde la cena, por lo que se puede
afirmar que la motilidad de esta persona está siendo regulada por:

- Motilina

● La hernia fisiológica se produce dentro de:

- Cordón umbilical

● El crecimiento de un adenocarcinoma de páncreas compromete la pared gástrica por


contigüidad. ¿Qué parte del estómago se esperaría esté comprometido?

- Pared posterior del antro

● Al comer unas papitas fritas con mayonesa, el vaciamiento gástrico disminuye por
efecto directo de la hormona:

- colecistoquinina (CCK)

● Paciente que come entera una pizza familiar de chorizo y queso. Es posible esperar
que debido a la cantidad de alimento ingerida, las ondas lentas hayan:

- Sufrido ninguna alteración en su frecuencia

● En este caso se puede afirmar con seguridad que se presenta:

- contracciones tónicas en la región ano rectal

● La forma más común de atresia esofágica contiene:

- Estenosis proximal del esófago más fístula traqueoesofágica distal

● Con respecto a la saliva, marque la respuesta correcta:

- el sistema simpático estimula su secreción

● Respecto a las enfermedades del esófago, marque lo correcto:

- el diagnóstico diferencial de la acalasia es la enfermedad de Chagas esofágica

● En relación a la fisilogía gástrica, marque lo correcto:

- la cimetidina actúa en la región basolateral de la célula parietal

● La célula mucosa del cuello gástrico produce:

- Moco

● La saliva puede tener una variedad de electrolitos en su composición. Entre ellos el cloro,
respecto al cual se puede afirmar:

- Su concentración no llega a ser tan alta como en el plasma

● Los músculos de la masticación que producen la retropulsión de la mandíbula son:

- temporales

● Respecto a las glándulas salivales, marque lo incorrecto:

- la glándula sublingual tiene forma de garfio

● Respecto a la anatomía del estómago, marque lo correcto:


- la arteria gástrica derecha nace de la arteria hepática común

● En cuanto a la saliva, marque lo correcto:

- La amilasa cumple función digestiva

● Durante el ataque con gas sarín (bloqueador de la acetilcolinesterasa) en el metro de


Tokio, en 1995, el personal de salud notó que los pacientes afectados presentaban:

- Hipersalivación

Paciente de 54 años con antecedentes de alcoholismo, gastritis crónica,


tabaquismo pesado, obesidad, cálculos biliares y cirrosis, es llevado a la
emergencia por dolor abdominal en epigastrio irradiado a la espalda y trastorno del
sensorio.
Al examen físico: presión arterial 85/50 mmHg, frecuencia cardíaca 100 latidos/min,
frecuencia respiratoria 18 x minuto, temperatura axilar 36°C.
Conjuntivas pálidas, escleras ictéricas nevus arácnidos en tronco, distensión
abdominal marcada, cabeza de medusa, matidez desplazable en ambos flancos e
hipogastrio, dolor a la palpación de abdomen.
Tiempo de protrombina: 24 seg (testigo: 13 seg); TPT: 38 seg, glicemia: 165 mg/dL,
uremia: 20 mg/dL, ASAT: 76 UI/L, ALAT: 22 UI/L, albumina: 2,5 g/dL, bilirrubina total:
2,6 mg/dL, bilirrubina directa: 1,4 mg/dL, amilasa sérica 4000 U/L.

● Un efecto secundario en el estómago por la acción de la secretina es:

- Menor actividad de la pepsina

● Considerando que el paciente sufre de gastritis, se puede decir que la secreción de


ácido por la mucosa gástrica:

- Involucra transporte activo de Hidrogeniones

● En cuanto a la gastritis de este paciente, se encontró que era producida por la


bacteria Helicobacter pylori. Esta bacteria sobrevive en el medio ácido del estómago
gracias a:

- Ureasa

● La bilirrubina directa aumentada en cirrosis hepática se excreta en la orina debido a:

- Ser hidrosoluble

● Paciente de 42 años con adenocarcinoma ductular. La TC ha demostrado claramente que el


tumor está en el cuello del páncreas y que hay un gran vaso ocluido. ¿Cuál de los siguientes
vasos estaría más probablemente obstruido?

- Vena porta.

● En relación a la histología hepática, marque lo correcto:


- La zona 3 se encuentra más cerca a la vena central lobulillar
Mujer de 83 años acude a emergencia por dolor abdominal desde hace 4
días, localizado en epigastrio, irradiado a ambos hipocondrios, nauseas,
vómitos y distensión abdominal; tiene antecedente de cardiopatía
hipertensiva, diabetes mellitus tipo II y fibrilación auricular. Refiere
deposiciones diarreicas muco sanguinolentas hace 1 día. Los exámenes
iniciales muestran PA: 110/60 mmHg, FC: 110/mn, leucocitos: 17800,
neutrófilos de 93%. TAC abdómino pélvica se observa oclusión completa
de arteria mesentérica superior por trombo asociado a placa ateromatosa.

● En esta paciente, ¿cuál de las siguientes sustancias no tendrá una


considerable disminución en su absorción? (marque la mejor respuesta):

- Calcio

● El mecanismo de la diarrea muco sanguinolenta que presenta la paciente, puede


mejor definido como de tipo (marque la mejor respuesta):}

- Exudativa

● Considerando que se ha comprometido el íleon distal, entre otras áreas, la atrofia o


descamación del epitelio de superficie explicaría cuál de los signos o síntomas de la
paciente (marque la mejor respuesta):

- Diarreas mucosanguinolentas

● Producto de esta isquemia, la expresión de cuál de las siguientes enzimas se vería


notablemente disminuida. Marque la mejor respuesta:

- Enteroquinasa

● Durante la cirugía, el cirujano observó que además la paciente tenía divertículos en


el sigma. Se sabe que estos divertículos:

- Se pueden asociar a estreñimiento crónico

1) La glandula parotida tiene principalmente acinos de tipo:


Tipo seroso
2) Presenta movimientos en masa:
Colon
3) A mayor flujo de saliva, disminuye la concentración de:
potasio
4) Es rama de la arteria mesentérica superior:
Arteria cólica media
5) En el intestino delgado se absorben los carbohidratos en forma de:
Fructosa
6) El dolor asociado a apendicitis clásicamente se ubica en:
Fosa iliaca derecho
7) La glandula submandibular recibe inervación traída por el nervio:
Cuerda del tímpano
8) La línea alba se encuentra:
Entre los rectos abdominales
9) la digestión de los carbohidratos se inician en:
Cavidad oral
10) Permite la suspensión e irrigación de los órganos peritoneales:
Mesenterio
11) Los nervios esplácnicos pélvicos (S2-S4) llevan información de tipo:
Parasimpática
12) Paciente con cirrosis hepática que tiene hipertensión portal con varices esofágicas y
actualmente presenta varices en estómago distal. Estas varices están relacionadas a
aumento en la presión de las venas:
Gástrica derecha
13) Es un órgano peritoneal:
Hígado
14) Al realizar un piercing en el ombligo, la sensación de dolor se transmite por:
T10
15) A mayor flujo de saliva disminuye la concentración de:
Potasio
16) La colecistoquinina inhibe:
El vaciamiento gástrico
17) El dolor del estómago asociado a gastritis se suele ubicar en:
Epigastrio
18) Es considerada una lesión preneoplásica:
Leucoplasia
19) Los sinusoides hepáticos son en realidad capilares de tipo:
Fenestrados
20) Paciente con cirrosis hepática y presión de vena cava de 15 mmHg, lo mas probable
es que el paciente presente:
Ascitis
21) La glándula submaxilar le hace un gancho al:
Músculo milohioideo
22) La glandula parotida esta inervada por el par craneal:
IX pc
23) Los nervios esplácnicos lumbares (L1-L2) llevan información de tipo:
Simpático
24) Los 9 cuadrantes del abdomen se delinean usando el plano subcostal, las líneas
medioclaviculares y:
Plano intertubercular
25) El estómago recibe información simpática proveniente del:
Ganglio celiaco
26) Es un órgano retroperitoneal:
Páncreas
27) La digestión de los lípidos se inicia en:
Intestino delgado
28) La digestión de las proteínas se inicia en:
Estómago
29) La arteria esplénica proviene de la aorta, y la vena esplénica desemboca en la vena:
Porta
30) Enfermedad asociado con un error en el desarrollo de las células de Cajal:
Enfermedad de Hirschsprung
31) El ácido acetilsalicílico actúa en la membrana:
Basolateral de la célula parietal
32) g
33) g
34) g
35) g
36) g
37) g
38) g
39) g
40) g
41) g
42) g
43) g
44) g
45) g
46) g
47) g
48) g
49) g
50) g
51) g
52) g
SISTEMA DIGESTIVO
EXAMEN PARCIAL
201802

Sección: Todas
Profesores: Choque Chávez, Fernando Diego; Damián Bastidas, Narda Lucía; Irribarren Gamarra, Maria Patricia;
Stapleton Valdivia, Mauricio Juan Jose.
Duración: 50 minutos.
Indicaciones:
- Lea atentamente cada pregunta antes de responder.
- Se prohíbe el uso del celular y cualquier dispositivo electrónico.
- Está prohibido intercambiar materiales.
- Coloque su código de alumno en la tarjeta de respuestas. Si su código contiene una letra reemplácela por un valor
numérico siguiendo la siguiente equivalencia: A=9, B=8, C=7, D=6, E=5.
- Traslade sus respuestas a la tarjeta, llenando los círculos de manera completa con lapicero negro o azul. Está
prohibido el llenado con lápiz, lapicero de otro color o con lapicero de tinta borrable.
- Sea cuidadoso en el llenado de la tarjeta de respuestas, pues solo esta tiene validez para la calificación.
- Al terminar su examen avise al docente a cargo, no se levante de su sitio; debe entregar la hoja de respuestas con
la carátula del examen, este cuadernillo de preguntas se lo llevará cada estudiante.

Chorrillos, octubre de 2018

1. En todo el sistema gastrointestinal se encuentra diferentes tipos de músculos ¿En cuales estructuras
encontraríamos músculo estriado?
A. Faringe, esfinter esofágico inferior, colon descendente.
B. Esfínter esofágico superior, laringe, esfínter anal externo.
C. Faringe, esfínter esofágico superior, esfínter anal externo.
D. Esfínter esofágico superior, esfínter esofágico inferior, colon.

2. Durante la fase …..……………….. el paladar blando es traccionado hacia arriba.


A. oral
B. gástrica
C. faríngea
D. esofágica

3. El tubo digestivo a nivel del ………………………….. está revestido internamente por epitelio ………….
A. Esófago / plano simple
B. Estómago / cilíndrico simple
C. Intestino grueso / plano estratificado
D. Intestino delgado / plano de transición

4. Señale la estructura del tubo digestivo que presenta tres capas musculares en su pared.
A. Esófago
B. Estómago
C. Intestino grueso
D. Intestino delgado

5. Paciente de 47 años con sobrepeso, acude a consulta por presentar dolor tipo cólico en el cuadrante superior
derecho del abdomen. El dolor aumenta luego de ingesta de comidas con abundante grasa. Esta situación se explica
por el aumento en la secreción de …………………………. que estimula la contracción de la vesícula biliar.
A. Gastrina
B. Colecistoquinina
C. Péptido inhibidor gástrico
D. Péptido intestinal vasoactivo
6. Si hay un aumenta de la hormona grelina a nivel del hipotálamo ¿Cuál es la consecuencia en el organismo?
A. Hambre
B. Saciedad
C. Aumento de somatostatina
D. Disminución del tránsito intestinal

7. Señale el péptido gastrointestinal que produce relajación del músculo liso gastro-intestinal.
A. Péptido intestinal vasoactivo (PIV)
B. Gastrina vasoactiva (GV)
C. Colecistoquinina (CCK).
D. Acetilcolina (Ach)

8. ¿Cuál de las siguientes alternativas inhibiría la relajación receptiva a nivel del estómago?
A. Histamina
B. Bloqueo del nervio vago (X)
C. Péptido inhibir gástrico (GIP)
D. Péptido intestinal vasoactivo (VIP)

9. Una mujer de 28 años, con diagnóstico de Diabetes Mellitus Tipo 1, acude por presentar desde hace 10 años
estreñimiento y distensión abdominal. Se realiza estudio y se determina que la paciente presenta un retraso del
vaciamiento gástrico debido a gastroparesia diabética. ¿Cuál de los siguientes hechos aumentaría el tiempo del
vaciamiento gástrico?
A. Aumento de gastrina
B. Estimulación parasimpática
C. Ácidos grasos en el duodeno
D. Quimo isotónico en el duodeno

10. Experimentalmente, se aplica un inhibidor selectivo del péptido intestinal vasoactivo (PIV) durante la contracción
peristáltica del intestino delgado. ¿Cuál es el efecto de este inhibidor en la motilidad del intestino delgado?
A. Parálisis del movimiento anterogrado
B. Disminución del tránsito intestinal
C. Aumento del tránsito intestinal
D. Movimiento retrogrado

11. ¿Cuál de los siguientes eventos ocurre durante la defecación?


A. Relajación del esfínter anal externo
B. Relajación del músculo liso del recto
C. Contracción del esfínter anal interno
D. Disminución de la presión intrabdominal

12. Señale el péptido que cumple la función de disminuir la ingesta de alimentos.


A. Neuropéptido Y
B. Ghrelina Gastrica
C. Proteína relacionada a agouti
D. Proopiomelanocortina (POMC)

13. Indique el lugar de secreción del péptido YY


A. Estómago
B. Duodeno
C. Yeyuno
D. Ileon

14. La grelina es secretada en el estómago y estimula a las neuronas del núcleo ………………….. para la estimulación de
la secreción de……………..
A. arqueado / neuropéptido Y.
B. arqueado / melanocortinas.
C. paraventricular / neuropéptido Y.
D. lateral del hipotálamo / hormonas orexigénicas.
15. Considerando el desarrollo embriológico del intestino medio. ¿Qué evento de gran importancia se produce en la
sexta semana?
A. Retracción de asas intestinales primitivas
B. Aparición del primordio hepático y pancreático
C. Inicio de la secreción de insulina por el páncreas
D. Salida temporal de asas intestinales a través de cordón umbilical

16. En el ……………………………. se presentan los movimiento de …………………….


A. esófago / retropropulsión y mezcla
B. estómago / segmentación y ondas lentas
C. intestino grueso / propulsión y ondas en espiga
D. intestino delgado / segmentación y peristaltismo

17. Marita sufre una parálisis del músculo masetero. ¿Qué limitaciones se producirán a nivel del movimiento de la
mandíbula?
A. Lateralización
B. Propulsión
C. Elevación
D. Ninguna

18. Señale el movimiento que se produce a nivel del ciego y colon proximal que tiene como finalidad favorecer la
absorción de agua y sales.
A. De masa
B. De mezcla
C. Propulsivo
D. Peristáltico

19. La enfermedad de Hirchsprung se caracteriza por una dilatación anormal de colon y disminución de los
movimientos propulsivos que traerá como principal consecuencia …………
A. diarrea acuosa.
B. estreñimiento crónico.
C. dilatación de la válvula ileocecal.
D. disminución de la flora intestinal.

20. ¿Cuál de los siguientes factores estimulan el vaciamiento gástrico?


A. Colecistoquinina (CCK)
B. Neuropéptido Y
C. Secretina
D. Gastrina

21. La secreción de ………………………. estimula la motilidad gástrica.


A. Colecistoquinina
B. Secretina
C. Gastrina
D. Motilina

22. La hormona ……………………….. es secretada por las células “I” del …………..
A. Colecistoquinina / duodeno y yeyuno
B. Gastrina / duodeno y yeyuno
C. Colecistoquinina / estómago
D. Gastrina / estómago
GLUT2, SGLT1, GLUT5

CI 1

1. La motilidad intestinal es estimulada principalmente por el:


- Sistema simpático
- Sistema piramidal
- Plexo de Auerbach
- Sistema parasimpático
2. La peristalsis o peristaltismo hace referencia a:
- Motilidad para movilizar el alimento de proximal a distal.
- No es parte de la motilidad
- Motilidad para mezclado de alimentos.
- Motilidad para fraccionamiento de alimentos.
3. Marque el órgano que se encuentra más distal en el tubo digestivo.
- Estomago
- Ciego
- Íleon
- Duodeno
4. Marque la respuesta incorrecta:
- La mucosa consta de epitelio, lámina propia y muscularis mucosae.
- En todo el tubo digestivo, se observa dos capas de muscular propia: circular
interna y longitudinal externa
- El colon contiene tenias
- Fuera de la cavidad abdominal, el esófago presenta capa adventicia.
5. Al iniciar la digestión, aumenta el consumo de oxígeno por la mucosa. Esto conlleva a
una hipoxia local, lo cual hace que se libere _____________, el cual produce
vasodilatación:
- Colecistoquinina
- Adenosina
- Histamina
- Noradrenalina
6. Cuál de las siguientes estructuras no tiene vasos sanguíneos:
- Epitelio intestinal
- Ligamento
- Omento
- Mesenterio
7. Paciente tiene una úlcera sangrante en el segundo tercio del Yeyuno. La arteria de la
cual proviene la sangre arterial para dicha zona es la arteria:
- Mesentérica superior
- Tronco celíaco
- Mesentérica inferior
- Iliaca común
- Gástrica izquierda
8. Paciente de 24 años con dolor abdominal tipo cólico intenso en mesogastrio. Según
sus conocimientos de macroestructura, el origen del dolor puede ser el ___________:
- Íleon
- Colon
- Estómago
- Esófago
9. Dentro de las funciones del abdomen, se encuentra la defecación y micción, en las
cuales la presión intra abdominal debe:
- Aumentar
- No tiene relación el abdomen con dichas funciones
- Mantenerse igual
- Disminuir
10. En la inspiración, la pared abdominal debe ____________ para ____________:
- Contraerse aumentar presión intra abdominal
- Relajarse disminuir presión intra torácica
- Relajarse aumentar presión intra abdominal
- Contraerse aumentar presión intra torácica

CI 2

1. Respecto a los péptidos gastrointestinales, marque lo correcto.


- No existe sustancia neurocrina que tenga efecto en la motilidad del tubo
digestivo
- Las sustancias paracrinas pueden viajar a través de vasos sanguíneos
- Las sustancias neurocrinas son peptidos que hacen su efecto en distancias
cortas
- Las sustancias paracrinas atraviesan la circulación portal
2. Al disminuir el pH duodenal por el HCl gástrico, se libera principalmente una hormona
cuya célula diana es:
- Acinos pancreáticos
- Células ductales del colédoco
- Célula ductal del Wirsung
- Células S del intestino
3. En un paciente con gastroparesia (motilidad lenta del estómago), que presenta
distensión abdominal después de comer, usted le recomendaría que evite el consumo
de lípidos y aminoácidos para disminuir la acción de:
- Secretina
- CCK
- Somatostatina
- Gastrina
4. La razón por la que el potencial de acción viaja rápidamente en sentido longitudinal
por el musculo liso gastrointestinal es la presencia de uniones en hendidura, además
de la presencia de:
- La presencia del plexo submucoso de Meissner
- Varicosidades
- Mayor cantidad de ACh
- Las fibras musculares no se disponen en haces musculares
5. Paciente obeso con Covid-19 es intubado por interno inexperto, quien al solicitar que
bombeen aire dentro del tubo endotraqueal, nota que el epigastrio se distiende. Al
sospechar que ha introducido el tubo en el estómago, también es cierto que:
- Disminuiría el tono del píloro
- Aumenta la frecuencia de ondas lentas
- Aumenta el pH gástrico
- Disminuye el pH gástrico
6. En un paciente con diarrea por hipermotilidad, usted sospecharía en el posible
aumento de las siguientes sustancias, excepto:
- Sustancia P
- ACh
- Péptido intestinal vasoactivo (VIP)
- Motilina
7. El ecografista sabe que para poder visualizar el nacimiento de la arteria mesentérica
superior, debe colocar el transductor sobre la piel de la siguiente región abdominal:
- Epigastrio
- Hipocondrio derecho
- Hipogastrio
- Mesogastrio
8. Al consumir un pan con mantequilla, la sensación de hambre disminuye debido a la
acción de:
- Grelina
- Somatostatina
- Colecistoquinina (CCK)
- Leptina
9. Una de las siguientes sustancias no comparte con las otras la misma acción sobre la
producción de ácido gástrico:
- Péptido insulinotrópico dependiente de glucosa (GIP)
- Colecistoquinina
- Somatostatina
- Secretina
10. Al ingerir grandes cantidades de dulces, con la subsecuente estimulación de incretinas,
usted esperaría que el apetito ______________, debido a __________________
- disminuya insulina
- aumente CCK
- disminuya CCK
- aumente grelina

PARCIAL

1. Al rozar agua caliente en la punta de la lengua, usted esperaría que el estímulo viaje a
través del nervio:
- lingual
- cuerda del tímpano
- glosofaríngeo
- hipogloso
2. Al ingresar líquidos o sólidos en la cavidad oral, un mecanismo que permite que una
persona respire mientras mastica es:
- el movimiento hacia afuera de los pliegues palatogloso y palatofaríngeo
- la elevación del paladar blando
- la depresión de la parte posterior de la lengua
- la depresión del paladar blando
3. Paciente adulto con reflujo gastroesofágico es más probable que presente:
- descalcificación del esmalte
- destrucción de ameloblastos
- remodelación del esmalte
- desfluorización de los dientes
4. La masticación es básicamente:
- importante para la digestión sobretodo de carnes
- un ralentizador del vaciamiento gástrico
- una actividad consciente
- un movimiento reflejo
5. Paciente de 34 años es víctima de asalto con arma de fuego, recibiendo un impacto
directo en el abdomen. En base a la radiografía, usted puede registrar en la historia
clínica que el proyectil se encuentra topográficamente en el:

- flanco derecho
- mesogastrio
- flanco izquierdo
- hipocondrio izquierdo
6. Señale cuál de las estructuras que en el embrión se encuentra comunicada con el saco
vitelino por medio del conducto onfalomesentérico:
- D
- B
- C
- A
7. En un paciente de 3 semanas de edad, con vómitos en proyectil, y nódulo epigástrico
reptante, usted esperaría encontrar:
- colecistoquinina aumentada
- vómitos biliosos e intolerancia a los ácidos grasos
- distensibilidad disminuida de la región oral del estómago
- engrosamiento de la circular interna pilórica
8. ¿Cuál de las siguientes estructuras tiene inervación somática?
- Estómago
- Peritoneo visceral
- Mesosigmoides
- Peritoneo parietal
9. Estas diseñando un proyecto de investigación sobre los niveles de colesterol que se
absorben luego de una comida grasosa y deseas cuantificar la cantidad de colesterol
que es absorbido por el intestino antes que el hígado lo metabolice ¿de cuál de los
siguientes vasos obtendrías la muestra para tu análisis?
- Vena porta
- Vena cava superior
- Conducto torácico
- Vena hemiácigos accesoria
10. Durante el paso del bolo hacia la orofaringe, se desencadena una serie de
contracciones musculares que estrechan la cavidad faríngea. Estas contracciones están
mediadas por el nervio craneal:
- X
- XI
- XII
- IX
11. Cuando el istmo de las fauces se cierra, se evita que el alimento pase hacia la
orofarínge y permite respirar mientras se mastica. Este cierre se debe a la contracción
y aproximación de los músculos:
- Palatoglosos
- Palatofaríngeos
- Estiloglosos
- Estilofaríngeos
12. Las siguientes alternativas son factores que determinan la patencia y función adecuada
del esfínter esofágico inferior, EXCEPTO:
- Plicatura diafragmática
- Hipertrofia de la circular interna
- Angulación con el estómago
- Canales lentos de calcio
13. Respecto al peristaltismo intestinal, para cumplir la ley del intestino , usted espera que
a nivel distal del quimo se libere:
- péptido liberador de gastrina (GRP)
- acetilcolina
- péptido intestinal vasoactivo
- sustancia P
14. El estímulo habitual para el movimiento peristáltico es:
- acción de la sustancia P
- contracción de la musculatura circular Interna
- estimulación vago-vagal
- distensión local
15. ¿Cuál de las siguientes condiciones considera que es un trastorno de la musculatura
lisa esofágica?
- Acalasia
- Asinergia faringoesfinteriana
- Hipotonía de los constrictores faríngeos
- Hipertonía del esfínter esofágico superior
16. ¿Cuál de las siguientes alternativas es correcta sobre la motilidad esofágica?
- Las ondas primarias son propulsoras y pueden no ser precedidas por deglución
- Las ondas primarias no son propulsoras y siempre van precedidas de deglución
- Las ondas secundarias son propulsoras y siempre van precedidas de deglución
- Las ondas secundarias son propulsoras y no van precedidas de deglución
17. El peristaltismo depende que a nivel distal del bolo se secrete:
- noradrenalina secretada por las fibras del sistema simpático
- acetilcolina por las neuronas provenientes del nervio vago
- óxido nítrico por células endoteliales locales
- péptido intestinal vasoactivo por neuronas
18. ¿Cuál de las siguientes alternativas es correcta sobre el movimiento peristáltico?
- Es un reflejo largo que depende de la integración con el tronco encefálico
- Se dirige en sentido distal siempre, nunca en sentido proximal
- El contenido intestinal avanza sólo 5-10 cm
- Es independiente del plexo mientérico
19. En un estudiante de medicina que está rindiendo un examen parcial, lo más probable
es que en ese momento su tránsito intestinal se encuentre:
- muy lento
- muy acelerado
- estimulado por acción de la sustancia P
- sin alteraciones
20. Las siguientes alternativas son ciertas sobre las contracciones tónicas del músculo
gastrointestinal, EXCEPTO:
- Tienen relación con el ingreso persistente de iones sodio
- Se encuentran principalmente en esfínteres
- Tienen regulación hormonal
- Obedece a una mayor frecuencia de potenciales en espiga
21. Con respecto de la regulación del pH del estómago; al utilizar un bloqueador de
histamina, usted espera que el pH del estómago:
- aumente
- disminuya
- se mantenga sin cambio
- se neutralice por acción de bicarbonato
22. Las siguientes hormonas disminuyen el vaciamiento gástrico, EXCEPTO:
- Gastrina
- Péptido insulinotrópico dependiente de glucosa
- Colecistoquinina
- Secretina
23. El frenillo de los labios se encuentra en:
- el piso de la boca
- la cavidad oral
- la cavidad vestibular
- el dorso de la lengua
24. En un paciente con shock hipovolémico, la peristalsis intestinal se encuentra:
- aumentada
- sin cambios
- invertida
- disminuida
25. Sobre el control de la peristalsis del tubo digestivo, ________________ es un
mediador neural que induce la relajación durante la peristalsis.
- la somatostatina
- el péptido intestinal vasoactivo
- la acetilcolina
- la serotonina

CI 3

1. En cuanto a las sustancias secretadas por el estómago ¿Cuál de las siguientes


sustancias estimula la liberación de pepsinógeno?
- Pepsinógeno
- Secretina
- Colecistoqunina
- Gastrina
2. Las células enteroendocrinas en el estómago se localizan en la glándula oxíntica, al
mismo nivel que las células:
- parietales
- principales
- mucosas
- absortivas
3. La fase intestinal de la secreción gástrica se debe básicamente a la participación de las
células:
- G del duodeno
- I del yeyuno
- S del íleon
- D del estómago
4. El conducto de Stenon, para entrar a la cavidad vestibular, debe atravesar el músculo:
- genihioideo
- buccinador
- milohioideo
- masetero
5. Paciente con acalasia es sometido a tratamiento endoscópico o quirúrgico, usted le ha
informado al paciente previamente que es posible que una complicación de este
tratamiento es que quede con cierto grado de:
- reflujo gastroesofágico
- gastritis
- odinofagia
- úlceras gástricas
6. La sangre que lleva la vena porta es tipo:
- mixta
- arterial
- venosa
7. Los pliegues gástricos gruesos son prácticamente inexistentes a nivel de:
- el cuerpo
- el fondo
- la incisura angularis
- el antro
8. En la producción de HCl, la acción de la somatostatina disminuye la accion de:
- las prostaglandinas
- la histamina
- la acetilcolina
- la gastrina
9. En el estómago se secretan las siguientes sustancias, EXCEPTO:
- grelina
- Correcto
- motilina
- somatostatina
- gastrina
10. La vena porta se forma gracias a la unión de la vena mesentérica superior con la vena:
- mesentérica inferior
- esplénica
- celiaca
- gástrica izquierda

CI 4

1. En el hígado, el aumento de la resistencia vascular en los sinusoides hepáticos


ocasionará:
- aumento de la presión de llenado vesicular
- salida de plasma hacia el intersticio
- aumento del flujo hacia la vena porta
- aumento del flujo hacia la vena cava superior
2. En un paciente con carcinoma de páncreas, el tumor ha invadido la unión entre la
venas esplénica y mesentérica superior; eso quiere decir que estamos seguros que el
tumor se encuentra a nivel del ________ del páncreas.
- cuerpo
- cola
- cabeza
- cuello
3. En un paciente con cirrosis hepática la cabeza de medusa que aparece en la pared
abdominal, podría desaparecer si al paciente se le:
- esclerosa las venas hemorroides internas
- oblitera el ligamento redondo
- administra antiandrógenos
- oblitera la arteria gástrica izquierda
4. En un paciente con intoxicación por órganos fosforados, la acción de la
colecistoquinina (CCK) está bloqueada a nivel de:
- el esfínter de Oddi
- la vesícula biliar
- la célula parietal
- el sistema nervioso central
5. Con respecto a la microestructura del hígado ¿Cuál de las siguientes alternativas es
correcta?
- La célula de Ito se encuentra en el espacio de Disse y reserva glucógeno
- Los hepatocitos están interconectados por uniones herméticas
- La célula de Kupffer se encuentra fuera del sinusoide y fagocita células
- El sinusoide es un capilar fenestrado
6. Paciente con Lupus Eritematosos que desarrolla hipertensión portal debido a
trombosis portal, es probable que desarrolle várices a nivel de:
- recto inferior
- recto superior
- hemorroides externas
- canal anal
7. El efecto de un medicamento colerético se evidencia por:
- la mayor producción de colesterol en la bilis
- la disminución de absorción de sales biliares
- el aumento de secreción biliar
- el aumento de formación de micelas
8. En un paciente con cáncer de páncreas y que desarrolla ictericia, la localización más
probable del tumor es en:
- la vesícula por metástasis
- el cuello del páncreas
- la cabeza del páncreas
- la cola del páncreas
9. En un recién nacido menor de 24 horas con atresia biliar, se encuentra elevación de la:
- bilirrubina directa
- bilirrubina indirecta
- alanina aminotransferasa (ALT)
- hemoglobina
10. ¿Cuál de las siguientes alternativas es correcta sobre la estructura hepática?
- Los colangiocitos producen bilis
- En la triada portal, se encuentra la vena derivada de la suprahepática
- El flujo sinusoidal en el lobulillo hepático es de adentro hacia afuera
- El flujo biliar en el lobulillo hepático es centrífugo
Banqueo Digestivo

1. Dentro de las funciones del abdomen, se encuentra la defecació n y micció n, en las cuales la
presió n intra abdominal debe:
Aumentar

2. Paciente de 24 añ os con dolor abdominal tipo có lico intenso en mesogastrio. Segú n sus
conocimientos de macroestructura, el origen del dolor puede ser el ___________ :
Ileon

3. En la evaluació n de una tomografía abdominal, el interno observa un aneurisma en una arteria


que se dirige al riñ ó n derecho. Con seguridad se puede afirmar que está a nivel de la vé rtebra:
L1

4. Es inervado por aferentes somaticas


Peritoneo parietal

5. Paciente se queja de dolor en hipocondrio derecho, pero superficialmente. El dermatoma


relacionado es (marque la mejor respuesta):
T9

6. Al iniciar la digestió n, aumenta el consumo de oxígeno por la mucosa. Esto conlleva a una
hipoxia local, lo cual hace que se libere _____________, el cual produce vasodilatació n:
Adenosina

7. Respecto a la anatomía del estó mago, marque lo correcto:


El fondo gá strico forma la curvatura mayor

8. La motilidad intestinal es estimulada principalmente por el:


Plexo de Auerbach

9. Paciente con vó lvulo del colon sigmoides. La necrosis de este segmento del colon se produce
por una alteració n en la irrigació n de la arteria:
Mesenterica inferior

10. Marque la respuesta incorrecta


En todo el tubo digestivo, se observa dos capas de muscular propia: circular interna y
longitudinal externa

11. Al disminuir el pH duodenal por el HCl gá strico, se libera principalmente una hormona cuya
cé lula diana es:
Cé lula ductal del Wirsung

12. Al consumir un pan con mantequilla, la sensació n de hambre disminuye debido a la acció n de:
Colecistoquinina (CCK)
13. La rotació n en sentido longitudinal del estó mago en el desarrollo embrioló gico condiciona que
el nervio vago derecho quede a nivel:
Posterior

14. Una de las siguientes sustancias reguladoras, puede actuar de forma paracrina y como
hormona. Marque la correcta:
Somatostatina

15. De los diferentes reflejos gastrointestinales, hay uno que produce movimiento del contenido
hacia la regió n distal, y se llama reflejo:
Gastrocó lico

16. Todos los mú sculos motores de la lengua está n inervados por el XII par, excepto:
palatogloso

17. Al ingerir rá pidamente un litro de agua, usted esperaría que la gastrina aumente por efecto de:
ACh del sistema mienté rico

18. En un paciente con hiperestimulació n simpá tica se espera que las ondas lentas tengan un
ritmo:
Menor en íleon terminal que en el duodeno

19. Paciente con apendicitis aguda, que debuta con dolor en mesogastrio. Este dolor se debe a
estimulació n de receptores del dolor cuyas fibras van a viajar a la mé dula espinal a travé s de:
Nervios simpá ticos

20. Paciente obeso con Covid-19 es intubado por interno inexperto, quien al solicitar que bombeen
aire dentro del tubo endotraqueal, nota que el epigastrio se distiende. Al sospechar que ha
introducido el tubo en el estó mago, tambié n es cierto que:
Disminuye el pH gá strico

21. Cuando una persona coloca en su boca una sustancia con alta concentració n de carbohidratos,
lo que debería pasar es que:
disminuya la sensació n de hambre

22. El pH ó ptimo para la digestió n a nivel duodenal es ____________ y está regulado principalmente
por la liberació n de _________________
6-8 / secretina

23. Los corpú sculos gustativos se encuentran en la lengua, pero ademá s se les puede encontrar en:
el paladar blando

24. Al ingresar líquidos o só lidos en la cavidad oral, un mecanismo que permite que una persona
respire mientras mastica es:
la depresió n del paladar blando

25. Al rozar agua caliente en la punta de la lengua, usted esperaría que el estímulo viaje a travé s del
nervio:
lingual

26. Sobre el control de la peristalsis del tubo digestivo, ________________ es un mediador neural que
induce la relajació n durante la peristalsis.
el pé ptido intestinal vasoactivo
27. ¿Cuá l de las siguientes alternativas es correcta sobre el control del peristaltismo?
El peristaltismo intestinal aumenta por efecto de la colecistoquinina (CCK)

28. ¿Cuá l de las siguientes alternativas es una característica de la estructura del esó fago?
Contiene glá ndulas submucosas principalmente en su tercio distal

29. En la estructura dentaria, se observa que hay una composició n muy similar a la del hueso en la
capa denominada:
cemento

30. El tubo digestivo posee glá ndulas, las glá ndulas submucosas se encuentran en el:
esó fago y duodeno

31. Estas diseñ ando un proyecto de investigació n sobre los niveles de colesterol que se absorben
luego de una comida grasosa y deseas cuantificar la cantidad de colesterol que es absorbido por
el intestino antes que el hígado lo metabolice ¿de cuá l de los siguientes vasos obtendrías la
muestra para tu aná lisis?
Conducto torá cico

32. Mujer de 30 añ os llega a emergencia con dolor en hipogá strico. Al examen físico presenta una
masa palpable de 10 cm de diá metro a en la misma regió n, usted sospecharía de las siguientes
condiciones, EXCEPTO:
Tumor renal

33. Las siguientes alternativas son factores que determinan la patencia y funció n adecuada del
esfínter esofá gico inferior, EXCEPTO:
Hipertrofia de la circular interna

34. Cuando el istmo de las fauces se cierra, se evita que el alimento pase hacia la orofarínge y
permite respirar mientras se mastica. Este cierre se debe a la contracció n y aproximació n de los
mú sculos:
palatoglosos

35. Las siguientes alternativas son ciertas sobre la actividad elé ctrica del mú sculo gastrointestinal,
EXCEPTO:
La despolarizació n lenta se debe principalmente al ingreso de Na+

36. ¿Cuá l de las siguientes alternativas es correcta sobre el movimiento peristá ltico?
El contenido intestinal avanza só lo 5-10 cm

37. La informació n sensitiva aferente del sistema gastrointestinal pasa por las siguientes
estructuras, EXCEPTO:
Tá lamo

38. Con respecto a la regulació n del peristaltismo, al aplicarle atropina (antagonista coliné rgico) a
un paciente, es de esperarse que el peristaltismo:
disminuya

39. ¿Cuá l de las siguientes alternativas es correcta sobre el control autó nomo del aparato
gastrointestinal?
La estimulació n simpá tica estimula a la muscularis mucosae
40. Respecto al peristaltismo intestinal, para cumplir la ley del intestino , usted espera que a nivel
distal del quimo se libere:
pé ptido intestinal vasoactivo

41. ¿Cuá l de las siguientes alternativas estimula las ondas de motilidad gastrointestinal
denominadas complejos migratorios interdigestivos?
Eritromicina

42. Las siguientes alternativas son ciertas sobre las contracciones tó nicas del mú sculo
gastrointestinal, EXCEPTO:
Tienen relació n con el ingreso persistente de iones sodio

43. El peristaltismo depende que a nivel distal del bolo se secrete:


pé ptido intestinal vasoactivo por neuronas

44. Señ ale cuá l de las estructuras que en el embrió n se encuentra comunicada con el saco vitelino
por medio del conducto onfalomesenté rico:
A
45. Es un derivado del mesenterio dorsal:
D

46. El ecografista sabe que pude visualizar el nacimiento de la arteria


mesenterica superior, debe colocar el transductor sobre la piel de la sgte region abdominal:
Epigastrio

47. Durante una cirugia oncologica, el cirujano observa que los organos abdominales tienen libre
movimiento dentro de la cavidad abdominal, excepto:
Colon ascendente

48. Al introducir una solucion azucarada directamente al estomago mediante una gastrometria
(comunicación entre la piel abdominal y el estomago), la sustancia que provocara que
aumente los niveles sericos de insulina es:
Peptido tipo glucagon (GLP-1)

49. Al ingerir grandes cantidades de dulces, con la subsecuente estimulacion de incretinas, usted
esperaria que el apetito ___________ debido a ___________
Disminuya / insulina

50. En un paciente con hiperestimulacion simpatica se espera que las ondas lentas tengan un
ritmo:
Menor en ileon terminal que en el duodeno

51. En un paciente con schok distributivo, usted decide iniciar noradrenalina por un cateter CVC,
consiguiendo aumentar la PA. ¿Qué efecto sobre la motilidad intestinal espera encontrar?
El potencial de reposo de las fibras musculares se hace mas negativo

52. La hormona que tiene un efecto sinergico con la secretina para optimizar el pH duodenal y la
digestion es:
CCK

53. Paciente joven que es traido a emergencia con abdomen agudo quirurgico debido a herida
contusopenetrante por verduguillo (alambre grueso con punta aguzada) recibida en una
pelea despues de un partido de futbol. Se observa herida en hipocondrio izquierdo. El organo
que debe estar sangrando y produciendo hemoperitoneo es:
Bazo

54. Paciente se queja de dolor en hipocondrio derecho, pero superficialmente. El dermatoma


relacionado es:
T9
55. Paciente de 24 años con dolor abdominal tipo colico intenso en mesogastrio. Según sus
conocimientos de macroestructura, el origen del dolor puede ser el _____
Ileon

56. Paciente tiene una ulcera sangrante en el segundo tercio del yeyuno. La arteria de la cual
proviene la sangre arterial para dicha zona es la art.:
Mesenterica superior

57. Durante el vomito, ¿el contenido gastrico tiene que pasar necesariamente por cual estructura
para llegar al esofago?
Cardias

58. La peristalsis o peristaltismo hace referencia a:


Motilidad para movilizar el alimento proximal a distal

59. Respecto a la anatomia del estomago, marque lo correcto:


El fondo gastrico forma la curvatura mayor

60. Al iniciar la digestion, aumenta el consumo del oxigeno por la mucosa. Esto conlleva a una
hipoxis local, lo cual hace que se libere ______, el cual produce:
Adenosina

61. La motilidad intestinal es estimulada parcialmente por el:


Plexo de Auerbach

62. En un paciente con falla en la fusion de los conductos de las yema ventral y dorsal del
pancreas, usted esperaria encontrar:
Drenaje de la mayor parte del jugo pancreatico en la papila menor

63. ¿Cuál de los sgtes alternaticas detallan los músculos que ayudan a empujar el bolo hacia la
orofaringe?
Estilogloso y palatogloso
64. En el esofago, el plexo mienterico o de Auerbach interviene en el _________ de los impulsos
_________
Fin / eferentes

65. Sobre el control de la peristalsis del tubo digestivo, ____________ es un indicador neural que
induce la relajacion durante la peristalsis.
El peptido intestinal vasoactivo

66. Las sgtes alternativas son correctas sobre la deglucion, EXCEPTO:


Consta de 2 fases

67. Paciente adulto con reflujo gastroesofagico es mas probable que presente:
Descalcificacion del esmalte

68. En una cirugia abierta (laparotomia), el cirujano al abrir la cavidad peritoneal por la parte
anterior (linea media), lo primero que observa es:
Epiplon mayor

69. Durante la deglucion, el bolo es impedido de ingresar a la cavidad oral gracias a la accion de
diversos músculos, entre ellos el musculo:
Palatofaringeo

70. Durante una cena, una gestante inspira por la boca profundamente de manera frecuente, sin
embargo, el organismo evita que el aire ingrese al esofago por la accion:
Del musculo cricofaringeo

71. Las sgtes alternativas son factores que determinan la patencia y funcion adecuada del esfinter
esofagico inferior, EXCEPTO:
Hipertrofia de la circular interna

72. La distencion del yeyuno provoca que se:


Despolarice el potencial de reposo de la membrana

73. El estimulo habitual para el movimiento peristaltico es:


Distencion local
74. La hormona responsable de los complejos migratorios interdigestivos tiene las siguientes
caracteristicas, EXCEPTO:
Cumple funciones de aumentar la motilidad y secrecion gastrica e intestinal

75. Con respecto a la regulación del peristaltismo, al aplicarle atropina (antagonista colinergico) a
un paciente, es de esperarse que el peristaltismo:
Disminuya
76. ¿Cuá l de las siguientes alternativas es correcta sobre la motilidad esofagica?
Las ondas secundarias son propulsoras y no van precedidas de deglucion

77. El principal gobernante sobre todos los movimientos gastrointestinales en el sistema


nervioso:
Mienterico

78. El peristaltismo intestinal se produce gracias a un reflejo que:


Se origina dentro de la pared intestinal

79. Un familiar le comenta que tiene ulcera gastrica por exceso de produccion de acido, con sus
conocimientos del sistema disgestivo, usted le recomendaria que reduzca el consumo de:
Aminoácidos

80. Un paciente con polimiositis posee alteración en la regulación del mecanismo de la deglución;
por eso hay que considerar la deglución de la saliva, pues a diario se produce _______mL
1000

81. En la producción de HCl, la acción de la somatostatina disminuye la accion de:


La gastrina

82. Al usar un parasimpaticomimético (agonista colinérgico), usted espera que la saliva presente:
Una mayor cantidad de sodio

83. Al evaluar a un paciente con parotiditis purulenta (con absceso), usted buscaría el orificio
terminal del conducto de _____________ a la altura de _____________
Stenon / la segunda molar superior

84. En cuanto a las sustancias secretadas por el estómago ¿Cuál de las siguientes sustancias
estimula la liberación de pepsinógeno?
Secretina

85. Para determinar que un paciente tiene esófago de Barrett, debemos encontrar ___________ en la
biopsia de esófago.
Células caliciformes

86. Al usar atropina en un paciente, usted esperaría:


la disminución de secreción gástrica por bloqueo de M3

87. El conducto de Stenon, para entrar a la cavidad vestibular, debe atravesar el músculo:
Buccinador

88. Las células enteroendocrinas en el estómago se localizan en la glándula oxíntica, al mismo


nivel que las células:
Principales
89. La inhibición de secreción gástrica es secundaria a:
El reflejo enterogastrico

90. En un paciente con infección por SARS-CoV-2 con compromiso severo e ingresado en la
unidad de cuidados intensivos (UCI), debido a la tormenta de citoquinas y a la desregulación
inmune ¿Cuál de las siguientes alternativas estará elevada en sangre al evaluar el perfil
hepático?
Transaminasas

91. Un recién nacido con enfermedad congénita, tiene una mutación de la proteína MRP2
encargada del transporte de la bilirrubina conjugada hacia el interior del canalículo. Antes de
las 24 horas de nacido presenta ictericia y además presentará con mayor probabilidad:
coluria

92. El efecto de un medicamento colerético se evidencia por:


el aumento de secreción biliar

1. En un paciente cirrótico con encefalopatía hepática ¿cuál de las siguientes alternativas sustenta
la reducción de carnes rojas en la dieta?
El amonio se produce principalmente en el intestino

2. En un paciente con cirrosis hepática la cabeza de medusa que aparece en la pared abdominal,
podría desaparecer si al paciente se le:
oblitera el ligamento redondo

3. En un paciente con cirrosis hepática se desarrollará ascitis debido a los siguientes


mecanismos, EXCEPTO:
Estrechamiento de la porta

4. Para poder absorber al torrente sanguíneo todos los carbohidratos de la lactosa, es necesario
usar únicamente los transportadores:
SGLT1, GLUT2

5. ¿Cuál de las siguientes alternativas es una comunicación entre el tejido hepático y la vesícula
biliar?
Conducto de Luschka

6. La bilis que sale de la vesícula biliar tiene como componente principal:


Acidos biliares

7. La relajación del esfínter de Oddi se produce directamente por acción de:


Peptido vasoactivo intestinal (VIP)

8. En un paciente con xerostomia presenta las sgtes condiciones, EXCEPTO:


Infecciones del oido a repeticion

9. Los pliegues gastricos gruesos son practicamente inexistente a nivel de:


Fonfo

10. La vena porta de forma gracias a la union de la vena mesenterica superior con la vena:
Esplenica
11. En caso se produzca la perforacion de la cara anteiror del estomago, esta perforacion
ocasionara _____ producto de la peritonitis quimica.
Ileo intestinal

12. Cuando un paciente recibe estimulo autonomo mixto (simpatico y parasimpatico), el flujo de
saliva:
Aumenta en relacion basal

13. En una gestante de 11 semanas con antecedentes de esofagitis eosinofilica, con hiperemesis
gravidica, que acude a emergencia por hematemesis leve, y presenta subitamente disnea y
dolor toracico. Usted sospecharia de:
Síndrome de Boerhaave

14. Sangre q lleva la vena porta es tipo:


Venosa

15. Aquellas cells q producen mayor cantidad de moco en el epitelio gastrico son las:
Mucosas superficiales

16. Dentro de las patologias q producen sangrado en el esofago, la que sangra mas es:
Síndrome de Mallory Weiss

17. En las glandulas salivales, el principal lugar de intercambio ionico se da a nivel del:
Conducto estriado

18. Via biliar extrahepatica se encuentra ubicada dentro del ligamento:


Hepatoduodenal

19. En un paicente con intoxicacion de organso fosforados, la accion de la colecistoquinina (CCK)


esta bloqueada a nivel de:
Esfinter de Oddi
93. En un paciente con carcinoma de pancreas, el tumor ha invadido la union entre la vena
esplenica y mesenterica superior, eso quiere decir que estamos seguros q el tumor se
encuentra a nivel del _____ del pancreas.
Cuello

94. ¿Cuál de las sgtes alternaticas es correcta sobre la estructura hepatica?


El flujo biliar en el lobulillo hepatico es centrifugo

95. En una paciente con una estenosis severe del coledoco por una complicacion quirurgica,
usted esperaria q desarrolle:
Aumento del tiempo de protrombina

96. Los hepatocitos tienen gran capacidad regenerativa, en parte gracias a las cells madre
hepaticas q se localizan en los:
Canales de Herning
97. Paciente con Lupus eritematosos q desarrolla hipertension portal debido a trombosis portal,
es probable que desarrolle varices a nivel de:
Recto superior

1. En todo el sistema gastrointestinal se encuentra diferentes tipos de músculos ¿En cuales estructuras
encontraríamos músculo estriado?
Faringe, esfínter esofágico superior, esfínter anal externo.

2. Durante la fase …..……………….. el paladar blando es traccionado hacia arriba.


faríngea

3. El tubo digestivo a nivel del ………………………….. está revestido internamente por epitelio ………….
Estómago / cilíndrico simple

4. Señale la estructura del tubo digestivo que presenta tres capas musculares en su pared.
Estómago

5. Paciente de 47 años con sobrepeso, acude a consulta por presentar dolor tipo cólico en el cuadrante
superior derecho del abdomen. El dolor aumenta luego de ingesta de comidas con abundante grasa.
Esta situación se explica por el aumento en la secreción de …………………………. que estimula la
contracción de la vesícula biliar.
Colecistoquinina

6. Si hay un aumenta de la hormona grelina a nivel del hipotálamo ¿Cuál es la consecuencia en el


organismo?
Hambre

7. Señale el péptido gastrointestinal que produce relajación del músculo liso gastro-intestinal.
Péptido intestinal vasoactivo (PIV)

8. ¿Cuál de las siguientes alternativas inhibiría la relajación receptiva a nivel del estómago?
Bloqueo del nervio vago (X)

9. Una mujer de 28 años, con diagnóstico de Diabetes Mellitus Tipo 1, acude por presentar desde hace
10 años estreñimiento y distensión abdominal. Se realiza estudio y se determina que la paciente
presenta un retraso del vaciamiento gástrico debido a gastroparesia diabética. ¿Cuál de los
siguientes hechos aumentaría el tiempo del vaciamiento gástrico?
Ácidos grasos en el duodeno

10. Experimentalmente, se aplica un inhibidor selectivo del péptido intestinal vasoactivo (PIV) durante
la contracción peristáltica del intestino delgado. ¿Cuál es el efecto de este inhibidor en la motilidad
del intestino delgado?
Disminución del tránsito intestinal
11. ¿Cuál de los siguientes eventos ocurre durante la defecación?
Relajación del esfínter anal externo

12. Señale el péptido que cumple la función de disminuir la ingesta de alimentos.


Proopiomelanocortina (POMC)

13. Indique el lugar de secreción del péptido YY


Estómago

14. La grelina es secretada en el estómago y estimula a las neuronas del núcleo ………………….. para la
estimulación de la secreción de……………..
arqueado / neuropéptido Y.

15. Considerando el desarrollo embriológico del intestino medio. ¿Qué evento de gran importancia se
produce en la sexta semana?
Salida temporal de asas intestinales a través de cordón umbilical

16. En el ……………………………. se presentan los movimiento de …………………….


intestino delgado / segmentación y peristaltismo

17. Marita sufre una parálisis del músculo masetero. ¿Qué limitaciones se producirán a nivel del
movimiento de la mandíbula?
Elevación

18. Señale el movimiento que se produce a nivel del ciego y colon proximal que tiene como finalidad
favorecer la absorción de agua y sales.
De mezcla

19. La enfermedad de Hirchsprung se caracteriza por una dilatación anormal de colon y disminución de
los movimientos propulsivos que traerá como principal consecuencia …………
estreñimiento crónico.

20. ¿Cuál de los siguientes factores estimulan el vaciamiento gástrico?


Secretina

21. La secreción de ………………………. estimula la motilidad gástrica.


Motilina

22. La hormona ……………………….. es secretada por las células “I” del …………..
Colecistoquinina / duodeno y yeyuno
23. El reflejo enterogástrico aumenta ……………………… debido a la…………………..
el tono del esfínter pilórico / distensión del duodeno.

24. ¿Cuál de las siguientes sustancias disminuye el vaciado gástrico?


Colecistocinina.

25. El plexo mientérico se encuentra ubicado en………………. y funciona controlando la ………………


entre las capas musculares / motilidad

26. ¿Cuál de las siguientes glándulas está conformada por glándulas serosas y secreta la mayor
cantidad de amilasa en la boca?
Parótidas

27. ¿Cuál de los siguientes factores es más importante para percepción mediante las papilas
gustativas?
Agua

28. Paciente obeso, fumador, consumidor de alcohol que acude por presentar molestias como disfagia,
ardor retroesternal, alteración del sueño y sensación de boca amarga en las mañanas. ¿Cuál de las
siguientes recomendaciones podría darle a este paciente?
Elevar la cabecera de la cama al dormir.

29. En la enfermedad por reflujo se encuentra alterada la función del esfínter …………………….
esofágico inferior

30. En la diarrea psicógena hay una estimulación del ……………….. que …………………….. la motilidad
gastrointestinal.
Sistema nervioso parasimpático / aumenta

Producto de la hipertrofia pilorica uno espera encontrar q los niveles de Gastrina se encuentren:

Elevados

Al realizar una pilorotomia (corte del piloro para descomprimir) ¿Q hormonas se comezara a
liberar rapidamente en respuesta a este tratamiento?

Colecistoquinina

Debido al signo de onda peristaltica, se debe asumir q como consecuencia hay un aumento en la
liberacion de _____ en la pared del estomago:
Adenosina

La oliva pilorica se logra palpar a nivel de:

L1

El uso de atropina se espera q disminuya la liberacion de:

Enzimas pancreaticas

El consumo de una pequena cantidad de gaseosa aumentara directamente la concentración serica


de cual de las sgtes hormonas:

Peptido 1 similar al glucagon (GLP-1)

La colecistoquininca inhibe: el vaciamiento gastrico

Paciente con Lupus Eritematosos que desarrolla hipertensión portal debido a trombosis portal, es
probable que desarrolle várices a nivel de: → recto superior

En el síndrome de Mirizzi, el paciente tiene cálculos en la vesícula biliar; pero se obstruye el


conducto hepático común debido a que un cálculo se ubica y crece de tamaño en: → la bolsa de
Hartmann

En un recién nacido menor de 24 horas con atresia biliar, se encuentra elevación de la: →
bilirrubina directa

En el hígado, el aumento de la resistencia vascular en los sinusoides hepáticos ocasionará → salida


de plasma hacia el intersticio

Un paso importante para la poder absorber los lípidos es la emulsificación de las grasas. ¿Cuál de
las siguientes alternativas es el principal factor encargado de dicha emulsificación? → Ácidos
biliares

La vía biliar extrahepática se encuentra ubicada dentro del ligamento: → Hepatoduodenal

¿Cuál de las siguientes alternativas es correcta sobre la estructura hepática? → El flujo biliar en el
lobulillo hepático es centrífugo

En un paciente cirrótico con encefalopatía hepática ¿cuál de las siguientes alternativas sustenta la
reducción de carnes rojas en la dieta? → El amonio se produce principalmente en el intestino
La relajación del esfínter de Oddi se produce directamente por acción de: → el péptido vasoactivo
intestinal (VIP)

C! 1
1) El dolor periumbilical o epigástrico en el inicio de una apendicitis aguda se debe a:
• - Irritación del peritoneo parietal
• - Estímulo del sistema simpático
• - Íleo secundario
• - Estímulo del nervio vago

2) ¿Cuál de los siguientes péptidos inhibe el vaciamiento gástrico?


• - Péptido inhibidor gástrico
• - motilina
• - Gastrina
• - Colecistoquinina

3) El uso de Ranitidina bloquea el receptor H2 de la histamina en las células


parietales. La histamina llega a estas células por:
• - difusión
• - vía hematógena
• - la luz gástrica

4) El consumir caramelos indirectamente activa la vía:


• - POMC/CART
• - AGRP/NPY
• - grelina
• - MCR-4

5) Entre las múltiples causas de la Enfermedad por Reflujo Gastroesofágico, se puede


considerar también a una alteración en las ______________ del esfínter esofágico
inferior:
• - ondas secundarias
• - contracciones tónica
• - ondas lentas

6) El aumento en la actividad motora de la pared gástrica genera un aumento en los


niveles locales de qué sustancia en la microvasculatura:
• - adenosina
• - CCK
• - endotelina
• - gastrina

7) Durante una cirugía oncológica, ¿la extirpación de cuál de los siguientes órganos se
vería comprometida por la presencia de adventicia?:
• - yeyuno
• - estómago
- recto
- vesícula biliar

8) En un paciente con apendicitis aguda, la sensación de dolor producido por esta


inflamación es llevada por el nervio:
• - vago
• - esplácnico menor
• - esplácnico mayor
• - pélvico

9) ¿Cuál de los siguientes líquidos corporales tiene el pH más alto?:


• - saliva
• - jugo gástrico
• - jugo pancreático
• - bilis en la vesícula biliar

10) Marque la respuesta correcta en relación a la gastrina:


• - Al distender el estómago, se inhibe su producción.
• - Las células G son las productoras y se encuentran principalmente en el antro
gástrico
• - Se estimula por la liberación de noradrenalina
• - Las células G se encuentran principalmente en el fondo gástrico

C2
1) Respecto a la anatomía del estómago, marque lo correcto:
• - la arteria gástrica derecha nace de la arteria hepática común
• - No contiene muscular de la mucosa
• - la arteria gástrica izquierda irriga la curvatura mayor del estómago
• - Tiene una capacidad de 3 litros en la mayoría de personas

2) Respecto a las enfermedades del esófago, marque lo correcto:


• - el diagnóstico diferencial de la acalasia es la enfermedad de Chagas
esofágica
• - el síndrome de Boeerhave tiene mejor pronóstico que el síndrome de Mallory
Weiss
• - el síndrome de Mallory Weiss raramente se presenta con hematemesis
• - un agente infeccioso común del esófago es el Streptococo beta hemolítico

3) Estimula la producción de saliva:


• - atropina
• - vasodilatación periglandular
• - expresión de miedo
• - fatiga o cansancio

4) Respecto a la motilidad del colon, marque lo correcto:


• - Con la distención del estómago, suelen aparecer movimientos en masa
• - Se producen contracciones segmentarias principalmente en el colon
izquierdo
• - Los movimientos de masa se encargaran del mezclado de las heces
• - Las haustras del colon contribuyen al reflejo de defecación

5) En cuanto a la saliva, marque lo correcto:


• - La amilasa cumple función digestiva
• - el sistema simpático disminuye su secreción
• - cuando hay un estímulo de mayor secreción, se vuelve hipertónica
• - se producen 200 m diarios en condiciones normales
• - La amilasa cumple función digestiva

6) La saliva puede tener una variedad de electrolitos en su composición. Entre ellos el


cloro, respecto al cual se puede afirmar:
• - Su concentración no llega a ser tan alta como en el plasma
• - Su menor concentración se alcanza con flujo alto
• - Su mayor concentración se consigue con flujo bajo
• - Con flujo alta, su concentración es mayor que la del plasma

7) En un paciente hipertenso con tratamiento a base de beta bloqueadores, la


producción de saliva se espera que:
• - disminuya
• - no se afecte
• - aumente
• - disminuya
• - sólo se afecta si los receptores de acetilcolina están activos

8) Durante la secreción de saliva, es de esperarse que las concentraciones de


________ y ______ disminuyan al disminuir el flujo:
• - potasio de cloro
• - potasio sodio
• - bicarbonato potasio
• - sodio bicarbonato

9) El omeprazol actúa sobre la membrana _____________ de la célula ____________


• - basolateral / apical
• - basolateral / parietal
• - apical / principal
- apical / parietal

10) Dentro de los factores protectores de la mucosa gástrica se pueden mencionar


múltiples protagonistas. Uno de ellos es:
• - CCK

• - gastrina

• - receptor muscarínico
• - pepsina

C3
1) Si existe un fármaco con acción colerética, se asume que se refiere a que potencia
o estimula la:
• - Secreción de enzimas biliares
• - Contracción de la vesícula biliar
• - Excreción fecal de sales biliares
• - Recirculación de sales biliares

2) Una mujer de 38 años ingresa en el hospital con signos de colecistitis y cálculos


biliares. Durante la colangiografía, se inserta el catéter en la vesícula biliar con mucha
dificultad. ¿Cuál de las estructuras interfiere con más probabilidad con el paso del
catéter por el conducto cístico?
• - Válvula espiral (de Heister)
• - Compresión de la vena porta del conducto cístico
• - Adherencias del ligamento hepatoduodenal
• - Compresión del conducto cístico por una arteria hepática.

3) El acino pancreático difiere con el de las glándulas salivales en:


• - Contiene celulares centroacinares
• - No produce secreción serosa
• - El páncreas produce principalmente secreción mucosa
• - no tiene diferencias
4) En relación a la circulación hepática, marque lo correcto:
• - Los sinusoides hepáticos transportan sangre mixta
• - La vena porta proporciona el 50% e sangre al hígado
• - La vena porta se forma a partir de la vena esplénica y la mesentérica inferior
• - La arteria hepática deriva de la mesentérica superior

5) Hombre de 65 años con isquemia intestinal leve por oclusión aterosclerótica de la


arteria mesentérica superior, pero la irrigación sanguínea colateral ha retrasado el
inicio de la necrosis. ¿Qué vasos ofrecen colaterales entre el tronco celíaco y la arteria
mesentérica superior?
• - Gastroomental derecha e izquierda
• - Gástrica izquierda y hepática.
• - Pancreaticoduodenal superior e inferior.
• - Cística y gastroduodenal

6) El GALT se localiza en:


• - borde en cepillo
• - lamina propia
• - submucosa
• - superficie de criptas de Lieberkun

7) ¿Por cuál de las siguientes células es secretada principalmente la pro enzima


procarboxipeptidasa?
• - acinares del páncreas
• - ductales del páncreas
• - centro acinares del páncreas
• - epiteliales del duodeno

8) Marque lo correcto:
• - Las venas sublobulillares desembocan en las venas hepáticas
• - Las venas hepáticas contienen válvulas
• - Usualmente, son dos grandes venas suprahepáticas
• - Ninguna es correcta

9) Una mujer de 49 años ingresa en el hospital con dolor en epigástrico que migra
hacia el lado derecho y atrás hacia la escápula, sin ictericia. La ecografía muestra un
gran cálculo biliar. ¿En cuál de las siguientes estructuras es más probable que se
localice el cálculo biliar?
• - conducto hepático derecho
• - conducto colédoco
• - bolsa de Hartmann
• - conducto hepático izquierdo

10) Cuando el alimento se encuentra en el estómago, se produce la liberación de


enzimas pancreáticas básicamente debido a la acción de:
• - vago
• - bombesina
• - CCK
• - secretina

C4
1) Paciente de 21 años con dolor intenso en arcada dentaria superior debido a
emergencia de la tercera molar. Frecuencia cardíaca 108 latidos por minuto,
PA: 140/80 mmHg, agitado, pálido, son sudoración fría, y boca con saliva
espesa. En este paciente se espera que la secreción exocrina pancreática se
encuentre:
o - disminuida
o - aumentada
o - extremadamente elevada
o - sin cambios

2) Las sales biliares son absorbidas a la sangre en :


o - íleon
o - yeyuno
o - colon proximal
o - duodeno

3) Un signo característico en pacientes con encefalopatía hepática es:


o - convulsiones tónicas
o - reflejo de Babinsky
o - Onicomicosis
o - Asterixis

4) El efecto colerético de las sales biliares se refiere a:


o - estimulo de secreción biliar
o - inhibición de secreción biliar
o - aumento de la bilirrubina indirecta
o - ninguna de las anteriores

5) Paciente de 42 años con adenocarcinoma ductular. La TC ha demostrado


claramente que el tumor está en el cuello del páncreas y que hay un gran vaso
ocluido. ¿Cuál de los siguientes vasos estaría más probablemente obstruido?
o - arteria mesentérica superior
o - arteria pancreática magna
o - vena mesentérica inferior
o - vena porta

6) La secreción de agua y bicarbonato por el páncreas exocrino se da


básicamente en la fase:
o - intestinal
o - gástrica
o - en las tres por igual
o - oral

7) En relación a la secreción biliar y su composición, marque lo correcto:


o - las sales biliares corresponde al 50% de su composición
o - se producen alrededor de 3 litros diarios
o - los fosfolípidos excretados son anfipáticos, a diferencia de las sales
biliares
o - participan principalmente en la digestión de carbohidratos

8) Paciente con tumor neuroendocrino productor de secretina, debido a lo cual se


puede esperar que su secreción pancreática, comparada con la de una
persona sana en estado de bajo flujo, tenga una concentración de:
o - sodio aumentada
o - potasio disminuida
o - bicarbonato aumentada
o - igual

9) La vena central terminal desemboca en:


o - venas sublobulillares
o - vena porta
o - sinusoides hepáticos
o - espacio de Disee

10) Las invaginaciones del epitelio de la vesícula biliar que se extiende incluso
hasta la muscular se denomina:
o - conductos de Lushka
o - senos de Rokitansky-Aschoff
o - apéndices epiploicos
o - divertículos biliares

PARCIAL 2020-02

1.- Con respecto a la regulación del peristaltismo, al aplicarle atropina (antagonista colinérgico) a un
paciente, es de esperarse que el peristaltismo:

estimule la acción de los receptores dopaminérgicos

disminuya

se mantenga sin alteración

aumente

2.- Respecto al peristaltismo intestinal, para cumplir la ley del intestino , usted espera que a nivel
distal del quimo se libere:

péptido liberador de gastrina (GRP)

sustancia P

acetilcolina

péptido intestinal vasoactivo

3.- El estímulo habitual para el movimiento peristáltico es:

acción de la sustancia P

estimulación vago-vagal

contracción de la musculatura circular Interna


distensión local

4.- ¿Cuál de las siguientes condiciones considera que es un trastorno de la musculatura


lisaesofágica?

Hipotonía de los constrictores faríngeos

Asinergia faringoesfinteriana

Hipertonía del esfínter esofágico superior

Acalasia

5.- En un estudiante de medicina que está rindiendo un examen parcial, lo más probable esque en
ese momento su tránsito intestinal se encuentre:

sin alteraciones

estimulado por acción de la sustancia P

muy lento

muy acelerado

6.- El peristaltismo intestinal se produce gracias a un reflejo que:

llega a los ganglios pre vertebrales

llega al sistema nervioso central

se origina dentro de la pared intestinal

llega a la médula espinal

7.- Las siguientes alternativas son ciertas sobre las contracciones tónicas del músculo
gastrointestinal, EXCEPTO:

Tienen relación con el ingreso persistente de iones sodio

Se encuentran principalmente en esfínteres

Obedece a una mayor frecuencia de potenciales en espiga

Tienen regulación hormonal


8.- La información sensitiva aferente del sistema gastrointestinal pasa por las siguientesestructuras,
EXCEPTO:

Tálamo

Médula espinal

Ganglios prevertebrales

Tronco encefálico

9.- Al realizar una vagotomía por úlcera péptica, usted esperaría una disminución deproducción de
gastrina debido a la:

menor distensión de las paredes gástricas

ausencia de acetilcolina vagal

ausencia de histamina

ausencia de bombesina vagal

10.- Las siguientes funciones son inhibidas por la hormona somatostatina, EXCEPTO:

Secreción de enzimas pancreáticas

Secreción gástrica de HCl

Liberación de gastrina

Motilidad intestinal

11.- En una cirugía abierta (laparotomía), el cirujano al abrir la cavidad peritoneal por la parteanterior
(línea media), lo primero que observa es:

Epiplón mayor

Estomago

Duodeno

Colon sigmoides

12.- ¿Cuál de las siguientes estructuras tiene inervación somática?

Estómago
Peritoneo parietal

Peritoneo visceral

Mesosigmoides

13.- En el plexo mientérico, el origen de los impulsos eferentes está en:

el plexo de Meissner

las células intersticiales de Cajal

los ganglios paravertebrales

el plexo de Aurbach

14.- En el esófago, el plexo mientérico o de Auerbach interviene en el _______ de losimpulsos


______

inicio / aferentes

fin / eferentes

fin / aferentes

inicio / eferentes

15.- Paciente con lesión del hipogloso del lado izquierdo. Para evaluarlo se le pide al pacienteque
saque la lengua, la cual se espera que la punta de la lengua se dirija hacia:

abajo

el lado izquierdo

adelante

el lado derecho

16.- Al ingresar líquidos o sólidos en la cavidad oral, un mecanismo que permite que unapersona
respire mientras mastica es:

la elevación del paladar blando

la depresión de la parte posterior de la lengua

el movimiento hacia afuera de los pliegues palatogloso ypalatofaríngeo


la depresión del paladar blando

17.- Paciente adulto con reflujo gastroesofágico es más probable que presente:
destrucción de ameloblastos

remodelación del esmalte

descalcificación del esmalte

desfluorización de los dientes

18.- En un paciente de 3 semanas de edad, con vómitos en proyectil, y nódulo epigástricoreptante,


usted esperaría encontrar:

engrosamiento de la circular interna pilórica

colecistoquinina aumentada

distensibilidad disminuida de la región oral del estómago

vómitos biliosos e intolerancia a los ácidos grasos

19.- ¿Cuál de las siguientes atresias/fístulas traqueo esofágicas considera usted que esincompatible
con la vida (de no recibir tratamiento)?

Tipo E

Tipo B

Tipo C

Tipo A

20.- El músculo liso gastrointestinal funciona como un sincitio debido a:


el calcio

las uniones en hendidura

las fibras musculares más largas

el plexo mientérico de Auerbach

21.- Durante el paso del bolo hacia la orofaringe, se desencadena una serie de
contraccionesmusculares que estrechan la cavidad faríngea. Estas contracciones están mediadas
porel nervio craneal:
XII

IX

XI

22.- Las siguientes alternativas son ciertas sobre la actividad eléctrica del músculo gastrointestinal,
EXCEPTO:

Si el potencial de membrana es más positivo, habrá mayorfrecuencia de espigas

Las ondas lentas no son potenciales de acción

Las ondas lentas son más frecuentes en el duodeno

La despolarización lenta se debe principalmente al ingreso de Na+

23.- Los corpúsculos gustativos se encuentran en la lengua, pero además se les puede encontrar
en:

el dorso de la lengua

las encías

el paladar blando

el paladar duro

24.- Mujer de 30 años llega a emergencia con dolor en hipogástrico. Al examen físico presentauna
masa palpable de 10 cm de diámetro a en la misma región, usted sospecharía de lassiguientes
condiciones, EXCEPTO:

Tumor renal

Embarazo

Tumor uterino

Cáncer de recto superior

25.- La masticación es básicamente:


importante para la digestión sobretodo de carnes

una actividad consciente


un movimiento reflejo

un ralentizador del vaciamiento gástrico


-2mA
ECU 1
1) Con respecto a la saliva, marque la respuesta correcta:

Respuesta seleccionada:
el sistema simpático estimula su secreción
● Pregunta 2
2 de 2 puntos

Respecto a las enfermedades del esófago, marque lo correcto:

Respuesta
seleccionada: el diagnóstico diferencial de la acalasia es la enfermedad de Chagas
esofágica
● Pregunta 3
0 de 2 puntos

En relación a la fisilogía gástrica, marque lo correcto:

la cimetidina actúa en la región basolateral de la célula parietal


● Pregunta 4
2 de 2 puntos

La célula mucosa del cuello gástrico produce:

Respuesta seleccionada:
Moc
o
● Pregunta 5
0 de 2 puntos

La saliva puede tener una variedad de electrolitos en su composición. Entre ellos el cloro,
respecto al cual se puede afirmar:

Respuesta seleccionada:
Respuestas:

Su concentración no llega a ser tan alta como en el plasma

● Pregunta 6
2 de 2 puntos
Los músculos de la masticación que producen la retropulsión de la mandíbula son:

Respuesta seleccionada:
temporales
● Pregunta 7
0 de 2 puntos

Respecto a las glándulas salivales, marque lo incorrecto:

Respuesta seleccionada:
la glándula sublingual tiene forma de garfio

● Pregunta 8
2 de 2 puntos

Respecto a la anatomía del estómago, marque lo correcto:

Respuesta seleccionada:
la arteria gástrica derecha nace de la arteria hepática común
● Pregunta 9
2 de 2 puntos

En cuanto a la saliva, marque lo correcto:

Respuesta seleccionada:
La amilasa cumple función digestiva
● Pregunta 10
2 de 2 puntos

Durante el ataque con gas sarín (bloqueador de la acetilcolinesterasa) en el metro de Tokio, en


1995, el personal de salud notó que los pacientes afectados presentaban:

Respuesta seleccionada:
HipersalivacióN

CI1
● Pregunta 1

La presencia de atresias y estenosis duodenales se deben


básicamente a una:

Falta de recanalización


Pregunta 2

● 2 de 2 puntos

El ligamento de Treitz característicamente:

Respuesta
seleccionada: Suspende el ángulo de Treitz

Respuestas: Está adherido a la unión yeyuno-ileal

Se encuentra a nivel de hipogastrio


derecho
Suspende el ángulo de Treitz

Recibe irrigación de la arteria mesentérica


inferior


Pregunta 3

● 0 de 2 puntos

Al deglutir un bolo alimenticio, es lógico suponer que al pasar por


el esófago haya un mayor consumo de oxígeno en la pared
del tercio:

Respuesta seleccionada:
Distal

Respuestas: No hay diferencia

Medio

Proximal

Distal


Pregunta 4

● 0 de 2 puntos


Estudiante de medicina de 20 años, se ha amanecido estudiando
para su examen de Sistema Digestivo. No ha probado
alimento desde la cena, por lo que se puede afirmar que la
motilidad de esta persona está siendo regulada por:

Respuesta seleccionada:
CCK

Respuestas: VIP

Motilina

CCK

Adrenalina


Pregunta 5

● 2 de 2 puntos

La hernia fisiológica se produce dentro de:

Respuesta seleccionada:
Cordón umbilical

Respuestas: Alantoides

Saco Vitelino

Saco Amniótico

Cordón umbilical

Pregunta 6

● 0 de 2 puntos

El crecimiento de un adenocarcinoma de páncreas compromete


la pared gástrica por contigüidad. ¿Qué parte del estómago
se esperaría esté comprometido?

Respuesta seleccionada:
Pared anterior del píloro

Respuestas: Pared anterior del píloro

Pared posterior del fondo

Pared anterior del cardias

Pared posterior del antro


Pregunta 7

● 2 de 2 puntos

Al comer unas papitas fritas con mayonesa, el vaciamiento


gástrico disminuye por efecto directo de la hormona:

Respuesta seleccionada:
colecistoquinina (CCK)

Respuestas: motilina
somatostatina

secretina

colecistoquinina (CCK)


Pregunta 8

● 0 de 2 puntos

Paciente que come entera una pizza familiar de chorizo y queso.


Es posible esperar que debido a la cantidad de alimento
ingerida, las ondas lentas hayan:

Respuesta
seleccionada: Aumentado su frecuencia por estímulo
parasimpático

Respuestas: Disminuido su frecuencia por estímulo


parasimpático

Sufrido ninguna alteración en su frecuencia

Aumentado su frecuencia por estímulo


parasimpático

Aumentado su frecuencia por estímulo


simpático


Pregunta 9

● 2 de 2 puntos


En este caso se puede afirmar con seguridad que se presenta:

Respuesta
seleccionada: contracciones tónicas en la región ano
rectal

Respuestas: pérdida de inervación por el nervio


esplácnico menor

dilatación de tracto gastrointestinal


afectado

contracciones tónicas en la región ano


rectal

células ganglionares sólo en el ano recto


Pregunta 10

● 2 de 2 puntos

La forma más común de atresia esofágica contiene:

Respuesta
selecciona Estenosis proximal del esófago más fístula
da: traqueoesofágica distal

Respuestas: Estenosis distal del esófago más fístula


traqueoesofágica proximal

Estenosis proximal y distal del esófago más


fístula traqueoesofágica distal
Estenosis distal del esófago más fístula
traqueoesofágica distal

Estenosis proximal del esófago más fístula


traqueoesofágica distal


Niño de sexo masculino de 2 años de edad, sufre de
estreñimiento desde el nacimiento (1 deposición cada 3-4
días). Madre menciona que le estimula la defecación con
un termómetro rectal, y continuo uso de enemas y
laxantes. Desde hace 6 meses comienza con vómitos
postprandiales. Los síntomas aumentan en frecuencia y
magnitud y están en relación con los episodios de
estreñimiento. No refiere fiebre, tos, diarrea ni lesiones
cutáneas. Al examen físico presenta regular estado
general, luce deshidratado. Abdomen distendido, blando,
depresible e indoloro. No se palpan masas abdominales.
Se permeabiliza el canal anal con termómetro rectal,
encontrando cierta resistencia. Salida de material fecal mal
oliente en regular cantidad. Exámenes de laboratorio:
hemograma normal. Signos inflamatorios de fase aguda
negativos. Alcalosis metabólica leve en sangre venosa.
Radiografía con enema baritado muestra recto y colon
sigmoides dilatados (megacolon). Biopsia profunda:
ausencia de células ganglionares en la muestra enviada.
Se realiza cirugía correctiva.
La percepción de la pirosis (sensación de dolor o
quemazón en el esófago) asociado al reflujo
gastroesofágico, puede aparecer o exacerbarse debido
a:

Respuesta
seleccionada: Ejercicio

Respuestas: Somatostatina

Uso de antiácidos

Ejercicio

Bipedestación


Pregunta 2
● 0 de 3,3334 puntos

Niño de sexo masculino de 2 años de edad,


sufre de estreñimiento desde el nacimiento
(1 deposición cada 3-4 días). Madre
menciona que le estimula la defecación con
un termómetro rectal, y continuo uso de
enemas y laxantes. Desde hace 6 meses
comienza con vómitos postprandiales. Los
síntomas aumentan en frecuencia y
magnitud y están en relación con los
episodios de estreñimiento. No refiere fiebre,
tos, diarrea ni lesiones cutáneas. Al examen
físico presenta regular estado general, luce
deshidratado. Abdomen distendido, blando,
depresible e indoloro. No se palpan masas
abdominales. Se permeabiliza el canal anal
con termómetro rectal, encontrando cierta
resistencia. Salida de material fecal mal
oliente en regular cantidad. Exámenes de
laboratorio: hemograma normal. Signos
inflamatorios de fase aguda negativos.
Alcalosis metabólica leve en sangre venosa.
Radiografía con enema baritado muestra
recto y colon sigmoides dilatados
(megacolon). Biopsia profunda: ausencia de
células ganglionares en la muestra enviada.
Se realiza cirugía correctiva.
Considerando que este paciente está
sometido a estrés por el agravamiento de
su enfermedad, es posible afirmar que
sus ondas lentas están:

Respuesta seleccionada:
Hipopolarizadas

Respuestas: Hipopolarizadas

Hiperpolarizadas

Desmotivadas

Despolarizadas

Pregunta 3

● 3,3334 de 3,3334 puntos

Niño de sexo masculino de 2 años de edad,


sufre de estreñimiento desde el nacimiento
(1 deposición cada 3-4 días). Madre
menciona que le estimula la defecación con
un termómetro rectal, y continuo uso de
enemas y laxantes. Desde hace 6 meses
comienza con vómitos postprandiales. Los
síntomas aumentan en frecuencia y
magnitud y están en relación con los
episodios de estreñimiento. No refiere fiebre,
tos, diarrea ni lesiones cutáneas. Al examen
físico presenta regular estado general, luce
deshidratado. Abdomen distendido, blando,
depresible e indoloro. No se palpan masas
abdominales. Se permeabiliza el canal anal
con termómetro rectal, encontrando cierta
resistencia. Salida de material fecal mal
oliente en regular cantidad. Exámenes de
laboratorio: hemograma normal. Signos
inflamatorios de fase aguda negativos.
Alcalosis metabólica leve en sangre venosa.
Radiografía con enema baritado muestra
recto y colon sigmoides dilatados
(megacolon). Biopsia profunda: ausencia de
células ganglionares en la muestra enviada.
Se realiza cirugía correctiva.
Debido al acúmulo de material fecal en todo
el marco colónico, y a la irritación
química asociada, el peristaltismo del
íleon distal se debe encontrar:

Respuesta
seleccionada: Inhibido

Respuestas: Afectado por un reflejo


vago-vagal

No sufre alteraciones

Estimulado
Inhibido


Pregunta 4

● 0 de 3,3334 puntos

Niño de sexo masculino de 2 años de edad,


sufre de estreñimiento desde el nacimiento
(1 deposición cada 3-4 días). Madre
menciona que le estimula la defecación con
un termómetro rectal, y continuo uso de
enemas y laxantes. Desde hace 6 meses
comienza con vómitos postprandiales. Los
síntomas aumentan en frecuencia y
magnitud y están en relación con los
episodios de estreñimiento. No refiere fiebre,
tos, diarrea ni lesiones cutáneas. Al examen
físico presenta regular estado general, luce
deshidratado. Abdomen distendido, blando,
depresible e indoloro. No se palpan masas
abdominales. Se permeabiliza el canal anal
con termómetro rectal, encontrando cierta
resistencia. Salida de material fecal mal
oliente en regular cantidad. Exámenes de
laboratorio: hemograma normal. Signos
inflamatorios de fase aguda negativos.
Alcalosis metabólica leve en sangre venosa.
Radiografía con enema baritado muestra
recto y colon sigmoides dilatados
(megacolon). Biopsia profunda: ausencia de
células ganglionares en la muestra enviada.
Se realiza cirugía correctiva.
En cuanto a los reflejos gastrocólico y
gastroduodenal en este paciente, indique
lo correcto:
Respuesta
selecci Se dan por nervios intrínsecos
onada: del sistema entérico

Respuestas El control del nervio vago sobre


: el recto se ha abolido

El reflejo gastrocólico es más


marcado en adultos que en
niños

Se dan por nervios intrínsecos


del sistema entérico

Se pueden considerar reflejos


vago-vagales


Pregunta 5

● 3,3334 de 3,3334 puntos

Niño de sexo masculino de 2 años de edad,


sufre de estreñimiento desde el nacimiento
(1 deposición cada 3-4 días). Madre
menciona que le estimula la defecación con
un termómetro rectal, y continuo uso de
enemas y laxantes. Desde hace 6 meses
comienza con vómitos postprandiales. Los
síntomas aumentan en frecuencia y
magnitud y están en relación con los
episodios de estreñimiento. No refiere fiebre,
tos, diarrea ni lesiones cutáneas. Al examen
físico presenta regular estado general, luce
deshidratado. Abdomen distendido, blando,
depresible e indoloro. No se palpan masas
abdominales. Se permeabiliza el canal anal
con termómetro rectal, encontrando cierta
resistencia. Salida de material fecal mal
oliente en regular cantidad. Exámenes de
laboratorio: hemograma normal. Signos
inflamatorios de fase aguda negativos.
Alcalosis metabólica leve en sangre venosa.
Radiografía con enema baritado muestra
recto y colon sigmoides dilatados
(megacolon). Biopsia profunda: ausencia de
células ganglionares en la muestra enviada.
Se realiza cirugía correctiva.
Con respecto a la defecación señale el
enunciado correcto:

Respuesta
selecci Es estimulado por un llenado de
onada: la cuarta parte del volumen
rectal

Respuestas Es completamente voluntario y


: mediado por el nervio
pudendo

La aferencia parasimpática es
transmitida por vía del nervio
vago

Es un reflejo netamente local

Es estimulado por un llenado de


la cuarta parte del volumen
rectal


Pregunta 6

● 0 de 3,3334 puntos


Niño de sexo masculino de 2 años de edad,
sufre de estreñimiento desde el nacimiento
(1 deposición cada 3-4 días). Madre
menciona que le estimula la defecación con
un termómetro rectal, y continuo uso de
enemas y laxantes. Desde hace 6 meses
comienza con vómitos postprandiales. Los
síntomas aumentan en frecuencia y
magnitud y están en relación con los
episodios de estreñimiento. No refiere fiebre,
tos, diarrea ni lesiones cutáneas. Al examen
físico presenta regular estado general, luce
deshidratado. Abdomen distendido, blando,
depresible e indoloro. No se palpan masas
abdominales. Se permeabiliza el canal anal
con termómetro rectal, encontrando cierta
resistencia. Salida de material fecal mal
oliente en regular cantidad. Exámenes de
laboratorio: hemograma normal. Signos
inflamatorios de fase aguda negativos.
Alcalosis metabólica leve en sangre venosa.
Radiografía con enema baritado muestra
recto y colon sigmoides dilatados
(megacolon). Biopsia profunda: ausencia de
células ganglionares en la muestra enviada.
Se realiza cirugía correctiva.
El contenido fecal se detiene en la zona
inmediatamente proximal a la zona donde
hay una menor presencia de:

Respuesta
seleccionada: Neuropéptido Y

Respuestas: Enteroquinasa

Péptido intestinal
vasoactivo

Neuropéptido Y

Acetilcolina


Estudiante de medicina de la UPC de 21 años sufre de
gastritis aguda ocasionada por comer en lugares poco
higiénicos. Suele consumir caramelos ( chupar ) mientras
está en clase hasta la tarde. Toma gaseosas regularmente
(carbohidratos 46%, sodio 53%). También toma regular
cantidad de leche (grasa 35%, lactosa 35%, proteínas
30%), pues le calma un poco el dolor el ardor que siente
por la gastritis. Incluso, cuando puede, se toma dos vasos
de agua fría para calmar las molestias. Ha decidido ir al
médico para tratarse pues ya no soporta el dolor, el cual
está seguro que los síntomas se deben a una elevada
producción de ácido clorhídrico en el estómago, y por ello
le ha recetado Ranitidina (antihistamínico), con lo que
siente mejoría.
Para reducir la secreción de HCl en esta paciente se
podría usar sustancias similares a:

Respuesta
seleccio Péptido insulinotrópico
nada: dependiente de la glucosa
(GIP)

Respuestas: Bombesina

Péptido insulinotrópico
dependiente de la glucosa
(GIP)

Acetilcolina

Histamina


Pregunta 2

● 0 de 3,3333 puntos


Estudiante de medicina de la UPC de 21 años
sufre de gastritis aguda ocasionada por
comer en lugares poco higiénicos. Suele
consumir caramelos ( chupar ) mientras está
en clase hasta la tarde. Toma gaseosas
regularmente (carbohidratos 46%, sodio
53%). También toma regular cantidad de
leche (grasa 35%, lactosa 35%, proteínas
30%), pues le calma un poco el dolor el
ardor que siente por la gastritis. Incluso,
cuando puede, se toma dos vasos de agua
fría para calmar las molestias. Ha decidido ir
al médico para tratarse pues ya no soporta
el dolor, el cual está seguro que los
síntomas se deben a una elevada
producción de ácido clorhídrico en el
estómago, y por ello le ha recetado
Ranitidina (antihistamínico), con lo que
siente mejoría.
Si se usara atropina en esta paciente, se
esperaría que disminuya la liberación de:

Respuesta
seleccionada: Bombesina

Respuestas: GIP

Bombesina

Secretina

Enzimas pancreáticas


Pregunta 3

● 0 de 3,3333 puntos


Estudiante de medicina de la UPC de 21 años
sufre de gastritis aguda ocasionada por
comer en lugares poco higiénicos. Suele
consumir caramelos ( chupar ) mientras está
en clase hasta la tarde. Toma gaseosas
regularmente (carbohidratos 46%, sodio
53%). También toma regular cantidad de
leche (grasa 35%, lactosa 35%, proteínas
30%), pues le calma un poco el dolor el
ardor que siente por la gastritis. Incluso,
cuando puede, se toma dos vasos de agua
fría para calmar las molestias. Ha decidido ir
al médico para tratarse pues ya no soporta
el dolor, el cual está seguro que los
síntomas se deben a una elevada
producción de ácido clorhídrico en el
estómago, y por ello le ha recetado
Ranitidina (antihistamínico), con lo que
siente mejoría.
El consumir caramelos eleva los niveles en
sangre de una hormona cuya función es
la estimulación de las células:

Respuesta
seleccionada: K del duodeno

Respuestas: Alfa del páncreas

K del duodeno

Beta del páncreas

G del antro


Pregunta 4

● 0 de 3,3333 puntos


Estudiante de medicina de la UPC de 21 años
sufre de gastritis aguda ocasionada por
comer en lugares poco higiénicos. Suele
consumir caramelos ( chupar ) mientras está
en clase hasta la tarde. Toma gaseosas
regularmente (carbohidratos 46%, sodio
53%). También toma regular cantidad de
leche (grasa 35%, lactosa 35%, proteínas
30%), pues le calma un poco el dolor el
ardor que siente por la gastritis. Incluso,
cuando puede, se toma dos vasos de agua
fría para calmar las molestias. Ha decidido ir
al médico para tratarse pues ya no soporta
el dolor, el cual está seguro que los
síntomas se deben a una elevada
producción de ácido clorhídrico en el
estómago, y por ello le ha recetado
Ranitidina (antihistamínico), con lo que
siente mejoría.
En este paciente con gastritis aguda debida a
una alta producción de ácido clorhídrico,
si se le hiciera un examen de sangre, se
encontraría elevados los niveles de:

Respuesta
seleccio Histamina
nada:

Respuestas: Sustancia P

Colecistoquinina

Péptido insulinotrópico
dependiente de la glucosa
(GIP)

Histamina


Pregunta 5

● 3,3333 de 3,3333 puntos


Estudiante de medicina de la UPC de 21 años
sufre de gastritis aguda ocasionada por
comer en lugares poco higiénicos. Suele
consumir caramelos ( chupar ) mientras está
en clase hasta la tarde. Toma gaseosas
regularmente (carbohidratos 46%, sodio
53%). También toma regular cantidad de
leche (grasa 35%, lactosa 35%, proteínas
30%), pues le calma un poco el dolor el
ardor que siente por la gastritis. Incluso,
cuando puede, se toma dos vasos de agua
fría para calmar las molestias. Ha decidido ir
al médico para tratarse pues ya no soporta
el dolor, el cual está seguro que los
síntomas se deben a una elevada
producción de ácido clorhídrico en el
estómago, y por ello le ha recetado
Ranitidina (antihistamínico), con lo que
siente mejoría.
El consumo de una pequeña cantidad de
gaseosa aumentará directamente la
concentración sérica de cuál de las
siguientes hormonas:

Respuesta
seleccionad Péptido 1 similar al
a: glucagón (GLP-1)

Respuestas: Colecistoquinina (CCK)

Motilina

Secretina

Péptido 1 similar al
glucagón (GLP-1)


Pregunta 6

● 3,3333 de 3,3333 puntos


Estudiante de medicina de la UPC de 21 años
sufre de gastritis aguda ocasionada por
comer en lugares poco higiénicos. Suele
consumir caramelos ( chupar ) mientras está
en clase hasta la tarde. Toma gaseosas
regularmente (carbohidratos 46%, sodio
53%). También toma regular cantidad de
leche (grasa 35%, lactosa 35%, proteínas
30%), pues le calma un poco el dolor el
ardor que siente por la gastritis. Incluso,
cuando puede, se toma dos vasos de agua
fría para calmar las molestias. Ha decidido ir
al médico para tratarse pues ya no soporta
el dolor, el cual está seguro que los
síntomas se deben a una elevada
producción de ácido clorhídrico en el
estómago, y por ello le ha recetado
Ranitidina (antihistamínico), con lo que
siente mejoría.
El consumo rápido de 500 mL de gaseosa
aumentará directamente la concentración
sérica de cuál de las siguientes
hormonas:

Respuesta
seleccionada: Gastrina

Respuestas: Secretina

Colecistoquinina
(CCK)

Gastrina

Neuropéptido Y


Cuando el tubo digestivo está en reposo, los complejos
mioeléctricos migratorios son desencadenados por:

Respuesta
seleccionad la serotonina
a:

Respuestas: la sustancia P
la serotonina

la motilina

el péptido intestinal
vasoactivo (VIP)


Pregunta 2

● 1 de 1 puntos

El orificio omental, o hiato de Winslow, se


encuentra limitado por el ligamento:

Respuesta seleccionada:
hepatoduodenal

Respuestas:
hepatoduodenal

gastroduodenal

gastroeesplénico

esplenorrenal


Pregunta 3

● 1 de 1 puntos


En un paciente de 43 años con tumor carcinoide
de páncreas productor de gastrina
(Síndrome de Zollinger-Ellison) se puede
esperar encontrar una potenciación del
reflejo:

Respuesta
seleccionada: gastrocólico

Respuestas: del vómito

de relajación receptiva

gastrocólico

ileocólico


Pregunta 4

● 1 de 1 puntos

La mucosa irrigada por la arteria palatina mayor


está recubierta por un epitelio:

Respuesta
seleccionada: plano escamoso
queratinizado

Respuestas:
plano escamoso
queratinizado

plano simple

cilíndrico simple no ciliado

plano escamoso no
queratinizado

Pregunta 5

● 1 de 1 puntos

En un paciente con enfermedad de Hirschprung


(megacolon agangliónico) con dilatación de
los dos tercios proximales del recto, la
dilatación de este segmento se ha producido
con mayor probabilidad debido a los
movimientos:

Respuesta seleccionada:
en masa

Respuestas: peristálticos

en masa

de retropulsión

segmentarios


Pregunta 6

● 1 de 1 puntos

En el sistema digestivo la liberación hormonal se


presenta ante diversos factores o estímulos.
La hormona ____________ es estimulada
por la presencia de alimentos en el bulbo
duodenal a predominio de ácidos grasos y
triglicéridos, por estimulación vagal y por la
hormona secretina.
Respuesta
seleccionada: colecistocinina (CCK)

Respuestas: lipasa pancreática

gastrina

colecistocinina (CCK)

motilina


Pregunta 7

● 1 de 1 puntos

¿Cuál de las siguientes estructuras se la conoce


como el vigilante del abdomen , por su
capacidad de desplazarse y adherirse a
cualquier zona inflamada o envolver el
órgano para frenar la inflamación?

Respuesta
seleccionada: Omento mayor

Respuestas: Omento menor

Omento mayor

Mesogastrio anterior

Intestino delgado


Pregunta 8
● 1 de 1 puntos

¿Cuál de las siguientes sustancias disminuye la


fuerza de las contracciones de
segmentación?

Respuesta
seleccionada: Atropina

Respuestas:
Atropina

Acetilcolina

Colecistocinina (CCK)

Insulina


Pregunta 9

● 1 de 1 puntos

Durante la deglución, inmediatamente luego que


el bolo alimenticio pasa por el esfínter
esofágico superior, se espera que la presión
intraluminal:

Respuesta
seleccionad disminuya en el cardias
a:

Respuestas: aumente en el tercio medio


del esófago

disminuya en el tercio medio


del esófago
aumente en la porción distal
al bolo

disminuya en el cardias


Pregunta 10

● 1 de 1 puntos

Durante el desarrollo del tubo digestivo en la


vida intrauterina, cuando el asa intestinal
rota 90° en el sentido de las manecillas del
reloj entonces:

Respuesta
selecciona el colon pasa por detrás del
da: duodeno

Respuestas: se forma el colon izquierdo

el colon pasa por detrás del


duodeno

se realiza el plegamiento
normal de los intestinos

se desarrolla un situs inversus


Pregunta 11

● 1 de 1 puntos


La irritación del peritoneo intestinal, como la que
sucede en una peritonitis, producirá:

Respuesta
seleccion la inhibición del peristaltismo
ada:

Respuestas: la estimulación del


peristaltismo

la inhibición del peristaltismo

la destrucción de las células


intersticiales de Cajal

el aumento de las
contracciones tónicas


Pregunta 12

● 1 de 1 puntos

Si al intubar a un paciente, por error se ingresa


el tubo endotraqueal en el esófago y se
insufla el manguito endotraqueal (globo
TET), la dilatación de este manguito
generará:

Respuesta
seleccio múltiples ondas secundarias
nada:

Respuestas: múltiples ondas peristálticas


primarias

múltiples ondas secundarias

una onda peristáltica primaria


una onda peristáltica secundaria
seguida de una terciaria


Pregunta 13

● 1 de 1 puntos

Paciente de 24 años acude a consulta externa


por presentar una fístula oronasal
(comunicación entre la cavidad oral y la
cavidad nasal). Está fístula es una
consecuencia tardía de la lesión de un vaso
sanguíneo por el antecedente de haber sido
operado de paladar hendido en los primeros
años de vida, aparentemente en una
campaña gratuita de corrección de paladar
fisurado. ¿Cuál de las arterias palatinas
podría haberse lesionado durante esa
cirugía?

Respuesta
seleccionad Mayor
a:

Respuestas:
Mayor

Ascendente

Menor

Rama palatina de la faríngea


ascendente

Pregunta 14

● 0 de 1 puntos

El omento mayor deriva del mesenterio


__________ y se inserta en el ___________

Respuesta
seleccionada: medio / colon

Respuestas: ventral / duodeno

dorsal / estómago

ventral / estómago

medio / colon


Pregunta 15

● 0 de 1 puntos

Para realizar el movimiento mecánico de abrir la


boca, primero se necesita:
Respuesta
selecciona la contracción de los
da: músculos buccinadores

Respuestas:
fijar el hueso hioides

la retropulsión de la lengua

elevar la laringe

la contracción de los
músculos buccinadores


Pregunta 16

● 0 de 1 puntos

El mesocolon transverso se origina en:

Respuesta
seleccionada: la curvatura mayor del
estómago

Respuestas: el colon transverso

la curvatura mayor del


estómago

la curvatura menor del


estómago

la pared posterior del


abdomen

Pregunta 17

● 0 de 1 puntos

Paciente con insuficiencia mitral moderada a


severa, con aumento de volumen de la
aurícula izquierda; esta condición tendrá
como consecuencia a nivel del sistema
digestivo:

Respuesta
selecciona la acalasia
da:

Respuestas:
la disfagia a sólidos

alteraciones de la fase
faríngea de la deglución

la acalasia

la estenosis hipertrófica del


cardias


Pregunta 18

● 0 de 1 puntos

¿Cuál de las siguientes alternativas se define


como la herniación de las vísceras
abdominales por un anillo umbilical
agrandado?
Respuesta seleccionada:
Gastrosquisis

Respuestas: Hernia femoral

Onfalocele

Hernia inguinal

Gastrosquisis


Pregunta 19

● 1 de 1 puntos

Durante el desarrollo intrauterino del páncreas,


cuando la porción izquierda de la yema
pancreática ventral migra en dirección
opuesta, se forma el páncreas:

Respuesta seleccionada:
anular

Respuestas: ectópico

anular

divisum

derecho


Pregunta 20
● 1 de 1 puntos

Al deglutir algo, durante la fase esofágica, las


ondas lentas del esófago se:

Respuesta
seleccionada: hacen menos negativas

Respuestas: hiperpolarizan

hacen menos negativas



^ asocian a menor actividad
motora

estabilizan
EXAMEN FINAL (RUMBO AL 20)
subrayen su rpta (AMARILLO)

1. El segmento que se encuentra entre la foveola gástrica y la glándula se llama ________ y


contiene ______________: ​itsmo /células madre
2. A un paciente con cáncer avanzado se le administra morfina para calmar el dolor. Ahora ya
no tiene dolor, pero ha comenzado a vomitar, probablemente por activación de → ​zona gatillo
3. Dentro de las sustancias que aumentan la producción de ácido gástrico se encuentra: →
colecitoquinina
4. Marque lo correcto respecto a los sinusoides hepáticos → contiene sangre mixta
5. La división entre duodeno y yeyuno, que determina la diferenciación de hemorragia digestiva
alta, está dada por → ligamento de treitz
6. ¿Cuál de las siguientes sustancias aumenta la secreción de saliva? --- ​bradicinina
7. Mujer de 83 años con antecedente de aterosclerosis y fibrilación auricular, postrada, con
dolor abdominal agudo en epigastrio y mesogastrio, náuseas, vómitos, distensión abdominal,
y diarreas mucosanguinolentas. Usted espera encontrar en esta paciente un infarto intestinal:
→​ transmural
8. En los pacientes con diabetes mellitus tipo II que desarrollan insuficiencia hepática severa, es
posible encontrar dentro de las complicaciones → empeoramiento de la hiperglicemia
9. La glándula parótida es inervada por el nervio --- ​glosofaríngeo
10. Para diagnosticar a un paciente con esófago de Barrett, es necesario encontrar en la biopsia
esofágica:----​ cel caliciformes
11. Paciente de 72 años, sufre de constipación desde hace 15 años, y ahora cursa con dolor
crónico en fosa iliaca izquierda, y que ocasionalmente presenta deposiciones con moco y un
poco de sangre. Un diagnóstico posible es →
12. Una paciente de 17 años, con antecedente de bulimia, ingresa a emergencia por
hematemesis masiva posterior a episodios de vómitos persistentes provocados. Su sospecha
clínica es: ---​- sindrome de mallory weis
13. Durante un experimento en el que se perfundía Coca Cola (pH 3) en el duodeno de ratas, se
midieron en la vena porta las concentraciones de diferentes sustancias. ¿Cuál de las
siguientes sustancias se espera esté aumentada? ---​ secretina
14. En la enteropatía por gluten, el principal mecanismo por el que se produce diarrea es por:
malaabsorición
15. Al ingerir una gran cantidad de lípidos, usted esperaría que se produzca lo siguiente:>
disminución del hambre
16. ¿Cuál de las siguientes sustancias aumenta la secreción de saliva? ​Bradicinina
17. En un paciente con diverticulitis de un divertículo falso o adquirido, es más probable
que se presente dolor en: ​fosa iliaca izquierda
18. En el epitelio del estómago, las células que secretan moco en mayor cantidad y más denso
son las células: ​Mucosas superficiales
19. Para estimular la secreción de gastrina por células G a través de la sustancia neurocrina
acetilcolina, es necesario que haya: ​estímulo vagal en fase cefálica
20. Al hacer una incisión mediana de epigastrio a mesogastrio durante una laparotomía, al
momento de abrir el peritoneo parietal lo que observará el cirujano será: ---- omento
mayor
21. Es la principal hormona en la regulación del vaciamiento gástrico: colecistoquinina
22. Las células mioepiteliales de las glándulas salivales cumplen la función de:xpulsión de
contenido al conducto excreto
23. Malformación intestinal ______________ es producida por la falta de retorno del intestino
medio a la cavidad abdominal. --- onfalocele
24.
25. Niño de 4 años de edad es llevado a emergencia por ingesta de cuerpo extraño (anzuelo
para pescar) y presentar de manera brusca disfagia, saialorrea, dificultad respiratoria.
Considerando el lugar donde debe haber quedado atrapado el cuerpo extraño, se debe
solicitar una radiografía de:
26. Para diagnosticar a un paciente con esófago de Barrett, es necesario encontrar en la biopsia
esofágica: → celulas caliciformes
27. En el epitelio del estómago, las células que secretan moco en mayor cantidad y más denso
son las célula >mucosas superficiales
En el intestino delgado se absorben distintas sustancias durante la alimentación. De las siguientes,
una de ellas no es absorbida como tal: fosfolipidos

Para estimular la secreción de gastrina por células G a través de la sustancia neurocrina acetilcolina,
es necesario que haya: distensión de la pared gástrica

La encefalopatía hepática que presentan los pacientes con cirrosis se debe principalmente a la
concentración sérica aumentada de amoniaco. Para evitar llegar a dicho nivel sérico de esta
sustancia, se le debe recomendar al paciente una dieta: baja en proteinas

Recién nacido desarrolla ictericia marcada dentro de las primeras 24 horas, se le diagnostica atresia
biliar intrahepática. Una característica que se presenta en este caso con mayor probabilidad es: ---
coluria

28. Un paciente varón, obeso, con diabetes, de 62 años, ingresa a emergencia por hematemesis.
Se realiza una ecografía, encontrándose una porta dilatada, y una esplenomegalia marcada,
así como escaso líquido intraperitoneal. Usted sospecharía que el sangrado es causado por
---- varices esofágicas
29. El hígado tiene una microestructura peculiar en la que --- el flujo sinusoidal es centrípeto
30. El espacio de Mall se encuentra en la región ---- periportal
31. Las siguientes alternativas son estimulantes de la liberación de gastrina, EXCEPTO ----
lipidos
32. Las glándulas gástricas que colaboran con la patencia del epitelio esofágico frente al reflujo
ácido gástrico son --- glandulas cardiales
33.
Respecto a los conductos excretores de las glándulas salivales, marque lo correcto:

el conducto estriado es llamado así por repliegues en su


membrana
l evaluar a un niño con diarrea crónica, la madre le indica que la
diarrea se detiene con el ayuno. Usted sospecharía de una
diarrea de tipo: osmotica

El peristaltismo es un fenómeno que se da por un reflejo de tipo:


Neonato de 4 días de vida, desde el día de hoy presenta vómitos que son muy fuertes, no biliosos,
que preocupan a la madre. Hay una gran probabilidad que el problema comprometa a la
muscular>circular interna

vago vagal

Al haber estimulación vagal, el esfínter de Oddi se apertura gracias a: ​colecistoquinina

Para diagnosticar a un paciente con esófago de Barrett, es necesario encontrar en la biopsia


esofágica: celulas caliciformes

Al hacer una incisión mediana de epigastrio a mesogastrio durante una laparotomía, al momento de
abrir el peritoneo parietal lo que observará el cirujano será: Omento mayor

Respecto a los conductos excretores de las glándulas salivales, marque lo correcto: conducto de
Stenon debe atravesar el masetero para entrar a la cavidad ora

Al palpar una masa pétrea en hipocondrio izquierdo, es probable que se trate de un tumor: cola del
pancreas

¿Cuál de las siguientes estructuras deriva del mesenterio ventral? omento menor

En la enfermedad celiaca se observa que la esteatorrea está compuesta en su mayoría por ácidos
grasos más que de grasas neutras no digeridas, lo cual es una forma de demostrar que el problema
se presenta principalmente en la​: formación activa de la lipasa
En la enteropatía por gluten, el principal mecanismo por el que se produce diarrea es por:
malabsorción

Las siguientes alternativas son estimulantes de la liberación de gastrina, EXCEPTO:​LIPIDOS

Al haber estimulación vagal, el esfínter de Oddi se apertura gracias a: ​la colecistoquinina

En un paciente con diverticulitis de un divertículo falso o adquirido, es más probable que se presente
dolor en:​la fosa ilíaca izquierda

Las células encargadas de mantener la flora intestinal de manera adecuada son las células --- ​células
de panteh

En la pancreatitis crónica usted esperaría encontrar: ictericia

Se debe tener claro, con respecto al músculo liso del tracto digestivo, que: ​funciona como un sincitio
EXAMEN FINAL
SISTEMA DIGESTIVO (ME154)
Ciclo 2019-02

Sección: Todas
Profesores: Alfaro Salazar, Herberth Romulo; Alva Muñoz, Jose Carlos; Mayor Zevallos, Otto Alberto;
Duración: 90 minutos.
Indicaciones:
- Lea atentamente cada pregunta antes de responder.
- Se prohíbe el uso del celular y cualquier dispositivo electrónico.
- Está prohibido intercambiar materiales.
- Coloque su código de alumno en la tarjeta de respuestas. Si su código contiene una letra reemplácela por un valor
numérico siguiendo la siguiente equivalencia: A=9, B=8, C=7, D=6, E=5, F=4 y G=3.
- Traslade sus respuestas a la tarjeta, llenando los círculos de manera completa con lapicero negro o azul. Está
prohibido el llenado con lápiz, lapicero de otro color o con lapicero de tinta borrable.
- Sea cuidadoso en el llenado de la tarjeta de respuestas, pues solo esta tiene validez para la calificación.
- Al terminar su examen avise al docente a cargo, no se levante de su sitio; debe entregar la hoja de respuestas con
la carátula del examen, este cuadernillo de preguntas se lo llevará cada estudiante.

1. La reabsorción de sodio y cloro en las glándulas salivales se da principalmente en el:


a) conducto intercalado
b) conducto excretor
c) conducto estriado
d) acino glandular

2. Paciente de 35 años con dolor en hipocondrio derecho irradiado a la punta de la escápula. Se observa en la ecografía
abdominal cálculos en la vesícula biliar. Es operado de emergencia realizando una incisión a lo largo del reborde
costal derecho. La información de dolor de esta zona es inervada por las raíces nerviosas:
a) T5 – T9
b) T6 – T7
c) T7 – T8
d) T9 – T10

3. Paciente de 23 años con dolor intenso periumbilical de inicio agudo. Es ingresado a sala de operaciones por
abdomen agudo, el cirujano encuentra sangrado a unos 60 cm proximal a la válvula ileocecal. La estructura que
está sangrando muy probablemente es un derivado embriológico del:
a) uraco
b) ligamento Hepatoduodenal
c) alantoides
d) conducto vitelino

4. Paciente de 64 años de edad con dolor en epigastrio que se distribuye en banda hacia la espalda. En la tomografía
se encuentra tumor en cabeza de páncreas que compromete a un vaso que discurre entre la cabeza del páncreas y
el proceso uncinado. Debido a esto, este cáncer se considera irresecable porque compromete a la arteria:
a) mesentérica superior
b) mesentérica inferior
c) tronco celíaco
d) aorta

5. Paciente con carcinoma gástrico avanzado, en preoperatorio para gastrectotomía total. ¿Cuál de los siguientes
ganglios linfáticos recibirá primero células metastásticas con mayor probabilidad:
a) celíaco
b) cisterna del quilo
c) esplénico
d) gástrico posterior
6. Paciente de 77 años con dolor abdominal difuso de 3 horas de evolución. En los estudios de imágenes se observa
oclusión probablemente aterosclerótica de la arteria mesentérica superior; no se observa necrosis, lo cual puede
ser explicado por la irrigación sanguínea colateral. ¿Qué vasos ofrecen colaterales entre el tronco celíaco y la arteria
mesentérica superior?
a) Gástrica izquierda y hepática
b) Cística y duodenal
c) Gastroomental derecha e izquierda
d) Pancreaticoduodenal superior e inferior

7. Paciente de 62 años con vólvulo de intestino delgado e isquemia intestinal. Se realiza laparotomía exploratoria para
liberar la obstrucción. ¿Cuál estructura se utilizará como punto de referencia para determinar la posición de la unión
duodenoyeyunal?
a) Vasos rectos
b) Ligamento suspensorio del duodeno (de Treitz)
c) Ligamento frenocólico
d) Nacimiento de la Arteria mesentérica superior

8. Paciente de 42 años con dolor abdominal intenso y hematemesis. En la endoscopía se observa una úlcera duodenal
posterior perforada con hemorragia intrabdominal. ¿Cuál de las siguientes arterias estará comprometida?
a) Gástrica izquierda
b) Mesentérica superior
c) Gástrica derecha
d) Pancreaticoduodenal posterosuperior

9. Paciente de 51 años con antecedente de enfermedad diverticular acude a emergencia por sangrado profuso y dolor
en cuadrantes inferiores. ¿Cuál es el origen más probable de la sangre que pierde en la hemorragia?
a) Cólica media
b) Mesentérica inferior
c) Rectal superior
d) Rectal inferior

10. Paciente de 13 días de vida con vómitos explosivos a las dos horas después de lactar. Al examen físico se palpa la
oliva pilórica. ¿Cuál es el nervió cuyos filetes dan inervación eferente a la estructura afectada?
a) Vago
b) Esplácnico torácico mayor
c) Mesentérico superior
d) Esplácnico torácico menor

11. Paciente de 80 años con dolor abdominal intenso y antecedente de estreñimiento crónico. En la colonoscopía se
observa divertículos con áreas ulceradas difusas en colon sigmoides y descendente. El cirujano programa una cirugía
de resección y planifica que para realizar esta resección tendrá que cortar los siguientes vasos y nervios:
a) plexo mesentérico superior y arteria rectal.
b) nervio esplácnico torácico mayor y arteria cólica media.
c) ramas del nervio vago y arteria ileocólica.
d) nervio esplácnico pélvico y artera cólica izquierda.

12. Paciente de 46 años ingresa a emergencia con dolor en cuadrante superior derecho e ictericia. En la ecografía se
observa cálculos en la vesícula biliar. ¿Cuál de los siguientes nervios transmite el dolor de la colecistitis?
a) Filetes aferentes del nervio vago derecho, referido al ángulo inferior de la escápula
b) Filetes nerviosos de las raíces T1 a T4, con referencia al hombro izquierdo
c) Fibras aferentes simpáticas del nervio esplácnico torácico mayor, con referencia a los dermatomas T6 a T8
d) Fibras aferentes de los ramos primarios dorsales de los nervios raquídeos T8 a T10 con referencia al
epigastrio
13. Un niño de 8 años es alimentado por sus padres con un Cheeseburguer, papitas fritas y un vaso de Coca Cola. ¿La
presencia de lípidos, carbohidratos y proteínas en el duodeno estimulan la liberación de cuál de las siguientes
hormonas en la mucosa duodenal?
a) Colecistoquinina (CCK)
b) Péptido inhibidor gástrico (GIP)
c) Secretina
d) Insulina

14. La fase cefálica del control de la secreción gástrica corresponde a cerca del 30% de la secreción ácida y es
consecuencia de un reflejo. ¿Cuál de las siguientes alternativas puede eliminar totalmente la fase cefálica de la
secreción gástrica?
a) Vaguectomía
b) Uso de atropina
c) Uso de bloqueador de receptor H2 de histamina
d) Uso de antiácidos

15. Recién nacido de 24 horas con llanto, vómitos y sin eliminación de meconio. Luego de los estudios auxiliares se
diagnostica la Enfermedad de Hirshprung, esta enfermedad se caracteriza por la ausencia de:
a) las células de Cajal en plexo mioentérico
b) el plexo mioentérico en el recto y colon sigmoides
c) las células mioepiteliales en el recto distal
d) el plexo de Meissner en el recto distal y colon sigmoides

16. Los complejos motores migratorios aparecen aproximadamente cada 90 minutos entre las comidas, y se considera
que son estimulados por la hormona motilina. La ausencia de estos complejos migratorios podría producir un
aumento en:
a) la motilidad duodenal
b) el vaciamiento gástrico
c) la deglución
d) las bacterias intestinales

17. ¿Cuál de las siguientes es una consecuencia probable de la resección del íleon?
a) Gastritis atrófica
b) Deficiencia de vitamina B12
c) Esteatorrea
d) Úlcera péptica

18. Los movimientos en masa son importantes en la fisiología intestinal. Estos movimientos en masa ocasionan:
a) la sensación de defecar
b) el peristaltismo duodenal
c) la retropulsión gástrica
d) la contracción del esfínter anal interno

19. La toxina colérica hace que aumente los niveles de AMPc intracelular, y este aumento hace que se mantenga abierto
un canal en las células de la cripta de Lieberkuhn. En condiciones fisiológicas, en una persona sana ¿Qué sustancia
puede promover que el canal quede abierto también?
a) Somatostatina
b) Óxido nítrico (NO)
c) Péptido intestinal vasoactivo (VIP)
d) Péptido similar al glucagón 1 (GLP1)

20. Paciente de 64 años con tumor abdominal que comprime la cisterna del quilo. En la biopsia de duodeno tomada
como parte del estudio, el patólogo puede observar
a) Dilatación del vaso quilífero central
b) Contracción de las venas de las vellosidades
c) Vellosidades intestinales más largas
d) Engrosamiento de la lámina basal
21. En la enfermedad de Crohn es posible encontrar células de Paneth en el colon. Esto se puede deber a la especial
función de estas células en:
a) la activación de la inmunidad adquirida
b) mantener la inmunidad innata
c) producir hormonas
d) producir Ig A

22. En 1967 se descubrió que la epidemia de Kuru, una enfermedad por priones, en el distrito de Okapa en Papua Nueva
Guinea, era causada por la costumbre de comer la carne de los muertos. Ahora se sabe que las proteínas priónicas
ingresan al organismo a través de:
a) los enterocitos
b) la transmigración
c) las células de Paneth
d) las células M

23. Paciente de 48 años con alteraciones en el tránsito intestinal por diabetes mellitus tipo 2; se presenta con
esteatorrea, flatulencia y malabsorción de grasas. Las pruebas de función hepática y biliar están dentro de rangos
normales. Una causa de la disminución de sales biliares puede ser:
a) el sobrecrecimiento bacteriano
b) la deficiencia de pepsina
c) la deficiencia de elastasa
d) la hiperestimulación del GLUT5

24. Al usar azúcar de mesa (sacarosa) para endulzar su café, el estudiante de medicina sabe que lo más probable es
que para su absorción tendrá utilizar el/los transportadore(s) ________________ que se encuentran en la
membrana apical de los enterocitos.
a) SGLT-1
b) GLUT2 y GLUT5
c) SGLT-1 y GLUT5
d) SGLT-1 y GLUT2

25. Una persona con la producción normal de lactasa; cada vez que toma leche, los productos de la degradación de la
lactosa por parte de la lactasa ingresarán al enterocito usando el/los transportador(es) _____________:
a) SGLT-1
b) GLUT2 y GLUT5
c) SGLT-1 y GLUT5
d) SGLT1 y GLUT2

26. En una persona sana, luego de una comida basada en carnes rojas cocinadas con baja cantidad de sal, al enterocito
pueden ingresar solamente:
a) aminoácidos
b) aminoácidos y dipéptidos
c) aminoácidos, dipéptidos y tripéptidos
d) aminoácidos, dipéptidos, tripéptidos y tetrapéptidos

27. Paciente con deficiencia congénita de procolipasa, sufre de esteatorrea cada vez que come comidas ricas en grasas.
En el estudio de composición de las heces, uno espera encontrar una cantidad aumentada de:
a) colesterol
b) triglicéridos
c) fosfolípidos
d) lisolecitina

28. Al tomar su café en Starbucks, un estudiante de medicina sufre una quemadura de primer grado en el tercio
anterior de la superficie dorsal de la lengua. La información de dolor es transmitida por el nervio:
a) cuerda del tímpano
b) glosofaríngeo
c) lingual
d) facial
29. Paciente de 32 años con esteatosis hepática no alcohólica. Se le realiza una biopsia hepática que confirma la
esteatosis; en el tejido se observan depósitos de lípidos en los hepatocitos, los cuales contienen principalmente:
a) triglicéridos
b) colesterol
c) ácidos grasos libre
d) acil-carnitina

30. Paciente de 21 años que es estudiante de medicina, llega a consulta refiriendo que en épocas de exámenes su piel
se vuelve amarillenta. Le realizan el diagnóstico de Síndrome de Gilbert asociado a mutación del gen UGT1A1, luego
de unas semanas acude a la consulta por ictericia asociada a resfrío. Al hacerle un análisis de sangre, usted sabe
que encontrará valores elevados de:
a) bilirrubina directa
b) bilirrubina indirecta
c) bilirrubina tipo delta
d) fosfatasa alcalina

31. Se realizó un experimento en el cual se inyectó tinta china en el peritoneo de ratas de laboratorio. Al realizarse una
biopsia hepática de dichos animales, se encontró que el tinte negro estaba depositado en:
a) las células de Ito
b) los hepatocitos
c) las células de Kupffer
d) las células de Disse

32. Paciente de 74 años de edad con shock hipovolémico asociado a deshidratación aguda severa. En este paciente es
posible encontrar hipoxia en zona …… del lobulillo hepático e infarto ……………… del intestino.
a) 1 / mucoso
b) 1 / transmural
c) 3 / transmural
d) 3 / mucoso

33. Niño de 5 años con historia de tres días de evolución caracterizado por fiebre, malestar general, odinofagia,
hiporexia, e irritabilidad. Al examen se observa lesiones ulcerativas de 4 mm de diámetro en mucosa yugal, con
fondo blanquecino y eritema periférico. El diagnóstico más probable es:
a) candidiasis oral
b) leucoplasia
c) aftas
d) herpes

34. Paciente de 52 años con enfermedad por reflujo gastroesofágico de 30 años de evolución. Se realiza endoscopía en
la que se encuentra mucosa eritematosa proximal a la línea Z. Para corroborar la presencia de lesión preneoplásica,
se toma una biopsia de esa zona, en la que se espera encontrar:
a) metaplasia gástrica
b) metaplasia intestinal
c) displasia gástrica
d) adenocarcinoma

35. Un estudiante de medicina termina su último examen final. Al abrir la boca para comer una hamburguesa con papas
fritas, la activación de los receptores muscarínicos de las células acinar y ductal estimularan un mayor flujo de saliva,
con lo cual disminuirá la concentración salival de:
a) potasio
b) bicarbonato
c) sodio
d) cloro
36. Un estudiante toma su desayuno consistente en un pan con mantequilla y queso. Antes que se puedan digerir las
grasas, es necesario que sean emulsificadas. La hormona ………………………. estimula la liberación de las sustancias
emulsificadoras.
a) colecistoquinina (CCK)
b) secretina
c) lipasa pancreática
d) gastrina

37. Al comer un pollo a la brasa entero, con papitas fritas y ensalada, la sustancia que estimulará la liberación de HCl
en el estómago es:
a) el neuropeptido Y
b) la secretina
c) la bombesina
d) la colecistoquinina (CCK)

38. Paciente con disminución marcada del apetito asociado a cáncer terminal, se podría utilizar análogos de ……………..
para promover la ingesta de alimentos.
a) el péptido similar al glucagón (GLP)
b) la serotonina
c) la secretina
d) la endorfina

39. Un hombre de 51 años presenta de forma súbita vómitos masivos de sangre roja brillante. Tiene como antecedente
hepatitis viral B hace 23 años. En la exploración física: FC 103 latidos/min, PA 85/50 mmHg, se palapa la punta del
bazo y niega vómitos. Su hematocrito es 21%, la prueba serológica de HBsAg es positiva. En la ecografía se observa
hígado con nodulaciones. ¿Cuál es la causa más probable para la hematemesis?
a) Esófago de Barrett
b) Síndrome de Mallory Weiss
c) Varices esofágicas
d) Esofagitis por reflujo

40. Durante el reflejo del vómito, uno de los primeros sucesos es:
a) el peristaltismo inverso
b) la contracción del píloro
c) la apertura de la glotis
d) el esfínter esofágico inferior abierto

41. Paciente de 34 años que acude por diarrea desde hace 4 días asociado a comer papa rellena con ají en el Estadio
Nacional, se exacerba cuando toma lácteos o come grasas. Se acompaña de flatulencia y episodios de tenesmo.
Este cuadro de diarrea e intolerancia a la lactosa de inicio agudo se explica por la:
a) enfermedad de Crohn
b) infección por V. cholera
c) colitis ulcerativa
d) giardiasis

42. En cuanto a la secreción pancreática, mientras mayor es el flujo, mayor es la concentración de:
a) potasio
b) bicarbonato
c) cloro
d) sodio

43. Paciente de 10 años con diarrea crónica, distensión abdominal, anorexia. Se le ha encontrado anticuerpos
antigliadina y antiendomisio. Es más probable que la diarrea se correlacione con el hallazgo histológico de:
a) adelgazamiento de las criptas
b) linfocitos intramusculares
c) atrofia de las vellosidades
d) úlceras duodenales

44. La glándula parótida está inervada por el par craneal:


a) VII
b) IX
c) X
d) XII
45. Las glándulas de Brunner se encuentran en la:
a) mucosa del esófago
b) submucosa del íleon
c) mucosa del yeyuno
d) submucosa del duodeno

46. ¿Cuál de las siguientes afirmaciones describe correctamente la función de la inervación parasimpática del tracto
gastrointestinal?
a) La norepinefrina es el principal neurotransmisor excitatorio.
b) La actividad parasimpática produce la relajación de los esfínteres
c) La actividad parasimpática excesiva puede provocar un trastorno llamado ileo paralítico (parálisis del
músculo liso intestinal)
d) El pH luminal, la osmolaridad y la distensión muscular son detectados por fibras parasimpáticas eferentes

47. Al ingerir una cantidad de glucosa por vía oral, esta es interiorizada en las células del organismo más rápido que si
esa misma cantidad de glucosa hubiese sido administrada por vía endovenosa. Este fenómeno sucede gracias a la
acción de la sustancia secretada por las células:
a) G
b) I
c) K
d) S

48. A …………… secreción de HCl en el lumen gástrico, …………….. pH en la sangre venosa gástrica
a) mayor /mayor
b) mayor / menor
c) menor / mayor
d) mayor / igual

49. ¿En cuál de las siguientes situaciones hay un menor flujo de secreción salival?
a) Masticar goma de mascar
b) Imaginarse ser sometido a un examen dental
c) Exposición a olor nauseabundo
d) Sueño

50. El alcohol y la cafeína estimulan la fase ______________ de la producción de ácido clorhídrico.


a) cefálica
b) gástrica
c) intestinal
d) gástrica y cefálica
BANCO DIGESTIVO

1. Paciente se queja de dolor en hipocondrio derecho, pero superficialmente. El dermatoma relacionado es


(marque la mejor respuesta)
a. T9

2. Dentro de las funciones del abdomen, se encuentra la defecación y micción, en las cuales la presión
intraabdominal debe:
a. Aumentar

3. Al examinar a un paciente, usted encuentra dolor localizado en fosa iliaca derecha y diagnostica
apendicitis. En este paciente, usted puede inferir.
a. El peritoneo parietal regional está afectado

4. La motilidad intestinal es estimulada principalmente por el:


a. Plexo de Auerbach

5. Durante el vómito, ¿el contenido gástrico tiene que pasar necesariamente por qué estructura para llegar
al esófago? Marque la mejor respuesta:
a. Cardias

6. Respecto a la anatomía del estómago, marque lo correcto:


a. El fondo gástrico forma la curvatura mayor

7. Marque la respuesta incorrecta:


a. En todo el tubo digestivo, se observa dos capas de muscular propia: circular interna y
longitudinal externa

8. Un alumno de medicina decide hacerse un piercing en el ombligo. Al realizarle el procedimiento, sangra


ligeramente. Esta sangre proviene de la arteria (marque la mejor respuesta)
a. Epigástrica inferior

9. Señala la respuesta correcta:


a. El apéndice cecal solo tiene serosa

10. Paciente mujer es traída a emergencia por sufrir una herida contuso penetrante por cuchillo realizada por
su esposo en un ataque de celos. Se observa herida en flanco izquierdo. Está solución de continuidad ha
comprometido varios músculos de la pared abdominal, excepto:
a. Recto abdominal

11. Marque el órgano que se considera retroperitoneal:


a. Parte de la vía biliar

12. La peristalsis o peristaltismo hace referencia a:


a. Motilidad para movilizar el alimento de proximal a distal.

13. Paciente tiene una úlcera sangrante en el segundo tercio del yeyuno. La arteria de la cual proviene la
sangre arterial para dicha zona es la arteria:
a. Mesentérica superior

14. La estructura que fija órganos principalmente a la pared posterior abdominal se denomina:
a. Mesenterio

15. Al iniciar la digestión, aumenta el consumo de oxígeno por la mucosa. Esto conlleva a una hipoxia local,
lo cual hace que se libere _____________, el cual produce vasodilatación:
a. Adenosina
16. Es inervado por aferentes somáticas:
a. Peritoneo parietal

17. El ligamento hepatogástrico une el __________ con el _______ y forma la entrada al ________
a. Hígado Estómago Orificio omental

18. Paciente con vólvulo del colon sigmoides. La necrosis de este segmento del colon se produce por una
alteración en la irrigación de la arteria:
a. Mesentérica inferior

19. Al retirar completamente el mesenterio de un órgano, el mismo se vería afectado principalmente en su:
a. Irrigación

20. Marque el órgano que se encuentra más distal en el tubo digestivo.


a. Ciego

21. Paciente de 24 años con dolor abdominal tipo cólico intenso en mesogastrio. Según sus conocimientos
de macroestructura, el origen del dolor puede ser el ___________:
a. Íleon

22. Paciente con herida por proyectil por arma de fuego, con herida de ingreso en región paraumbilical. Entre
las estructuras que usted está seguro que debe haberse lesionado es:
a. Omento mayor

23. Paciente joven es traído a emergencia con abdomen agudo quirúrgico debido a herida contuso
penetrante por verduguillo (alambre grueso con punta aguzada) recibida en una pelea después de un
partido de fútbol. Se observa herida en Hipocondrio Izquierdo. El órgano que debe estar sangrando y
produciendo hemoperitoneo es (marque la mejor respuesta):
a. Bazo

24. Al evaluar una tomografía abdominal, el médico asistente le pide al interno de la UPC que encuentre la
imagen con el corte a nivel de L1. El interno sabiamente busca el ________ para ubicar la vértebra L1.
a. Cuello del páncreas

25. En la inspiración, la pared abdominal debe ____________ para ____________:


a. Relajarse disminuir presión intra torácica

26. Sustancia que inhibe la secreción y la motilidad del estómago prolongando el tiempo de digestión:
a. Péptido insulinotrópico dependiente de la glucosa (GIP)

27. Marque lo correcto:


a. La hernia fisiológica se produce en la sexta semana y es la salida temporal de asas intestinales
a través del cordón umbilical

28. Marque la respuesta correcta en relación a la gastrina:


a. Las células G son las productoras y se encuentran principalmente en el antro gástrico

29. El consumir caramelos indirectamente activa la vía:


a. POMC/CART

30. Cuál de las siguientes estructuras no tiene vasos sanguíneos:


a. Epitelio intestinal

31. Paciente con hipoglucemia secundaria a un insulinoma (tumor neuroendocrino productor de insulina). El
órgano donde con mayor probabilidad ha crecido este tumor es:
a. Retroperitoneal
32. ¿En qué capa se encuentra la alteración principal en el Hirschsprung o megacolon agangliónico?:
a. Muscular propia

33. Con respecto a las ondas lentas, marque la afirmación correcta:


a. Son contracciones rítmicas espontáneas

34. El uso de Ranitidina bloquea el receptor H2 de la histamina en las células parietales. La histamina llega a
estas células por:
a. Difusión

35. La triada sintomática: vómitos explosivos post-prandiales, movimientos peristálticos epigástricos visibles
de izquierda a derecha y nódulo palpable epigástrico subcostal derecho, pertenecen a:
a. Estenosis congénita hipertrófica del píloro

36. Durante una cirugía oncológica, ¿la extirpación de cuál de los siguientes órganos se vería comprometida
por la presencia de adventicia?:
a. Recto

37. En cuanto a los reflejos gastrointestinales, un reflejo que estimula el tránsito intestinal es el reflejo:
a. Gastrocólico

38. El ligamento falciforme divide al hígado en dos lóbulos derecho e izquierdo. Embriológicamente deriva
del:
a. Mesenterio ventral

39. La presencia de atresias y estenosis duodenales se deben básicamente a una:


a. Falta de recanalización

40. Estudiante de medicina de 20 años, se ha amanecido estudiando para su examen de Sistema Digestivo.
No ha probado alimento desde la cena, por lo que se puede afirmar que la motilidad de esta persona está
siendo regulada por:
a. Motilina

41. Paciente con disminución del apetito marcada asociada a cáncer terminal, para promover la ingesta de
alimentos se podría usar análogos de:
a. Endorfinas

42. Las ondas lentas se producen por la apertura cíclica de canales de:
a. Calcio

43. La forma más común de atresia esofágica contiene:


a. Estenosis proximal del esófago con fístula traqueoesofágica distal

44. Al deglutir un bolo alimenticio, es lógico suponer que al pasar por el esófago haya un mayor consumo de
oxígeno en la pared del tercio:
a. Proximal

45. Paciente que come entera una pizza familiar de chorizo y queso. Es posible esperar que debido a la
cantidad de alimento ingerida, las ondas lentas hayan:
a. Sufrido ninguna alteración en su frecuencia

46. La hernia fisiológica se produce dentro de:


a. Cordón umbilical

47. El crecimiento de un adenocarcinoma de páncreas compromete la pared gástrica por contigüidad. ¿Qué
parte del estómago se esperaría esté comprometido?
a. Pared posterior del antro

48. Estimula la producción de saliva:


a. Vasodilatación periglandular

49. Durante la secreción de saliva, es de esperarse que las concentraciones de ________ y ______
disminuyen al disminuir el flujo:
a. Sodio Bicarbonato

50. Con respecto a la secreción gástrica de HCl:


a. A mayor secreción de HCl en el lumen gástrico, mayor pH en la sangre venosa gástrica

51. Respecto a las enfermedades del esófago, marque lo correcto:


a. El diagnóstico diferencial de la acalasia es la enfermedad de Chagas esofágica

52. Con respecto a las lesiones y enfermedades de la boca, marque lo correcto:


a. La eritroplasia debe ser biopsiada

53. Respecto a las glándulas salivales, marque lo incorrecto:


a. La glándula sublingual tiene forma de garfio

54. El omeprazol actúa sobre la membrana _____________ de la célula ____________


a. Apical / parietal

55. Durante el sueño, la concentración de bicarbonato en la saliva:


a. Disminuye

56. Durante el ataque con gas sarín (bloqueador de la acetilcolinesterasa) en el metro de Tokio, en 1995, el
personal de salud notó que los pacientes afectados presentaban:
a. Hipersalivación

57. La célula mucosa del cuello gástrico produce:


a. Moco

58. Un paciente con diarrea por hipermotilidad, usted sospecharía en el posible aumento de las siguientes
sustancias, excepto:
a. Péptido intestinal vasoactivo

59. En la evaluación de una tomografía abdominal, el interno observa un aneurisma en una arteria que se
dirige al riñón derecho. Con seguridad se puede afirmar que está a nivel de la vértebra:
a. L1

60. Una de las siguientes sustancias no comparte con las otras la misma acción sobre la producción de ácido
gástrico:
a. Colecistoquinina

61. Usted encuentra músculo estriado en el siguiente segmento:


a. Esfínter anal externo

62. Paciente con intoxicación por organofosforados (inhibidores de acetilcolinesterasa), se espera que el
tránsito intestinal se encuentre:
a. Aumentado

63. Al disminuir el pH duodenal por el HCl gástrico, se libera principalmente una hormona cuya célula diana
es:
a. Célula ductal del wirsung
64. La hormona que tiene un efecto sinérgico con la secretina para optimizar el pH duodenal y la digestión,
es:
a. CCK (colecistoquinina)
65. El ligamento falciforme del hígado proviene embriológicamente de:
a. Mesenterio ventral

66. Todos los músculos motores de la lengua están inervados por el XII par, excepto:
a. Palatogloso

67. Al ingerir grandes cantidades de dulces, con la subsecuente estimulación de incretinas, usted esperaría
que el apetito ______________, debido a __________________
a. Disminuya insulina

68. Paciente obeso con Covid-19 es intubado por interno inexperto, quien al solicitar que bombeen aire
dentro del tubo endotraqueal, nota que el epigastrio se distiende. Al sospechar que ha introducido el tubo
en el estómago, también es cierto que:
a. Disminuye el pH gástrico

69. Una de las siguientes sustancias reguladoras, puede actuar de forma paracrina y como hormona. Marque
la correcta:
a. Somatostatina

70. Durante una cirugía oncológica, el cirujano observa que los órganos abdominales tienen libre movimiento
dentro de la cavidad abdominal, excepto:
a. Colon ascendente

71. Al consumir un pan con mantequilla, la sensación de hambre disminuye debido a la acción de:
a. Colecistoquinina (CCK)

72. Respecto a los péptidos gastrointestinales, marque lo correcto.


a. Las sustancias paracrinas pueden viajar a través de vasos sanguíneos

73. En un paciente con gastroparesia (motilidad lenta del estómago), que presenta distensión abdominal
después de comer, usted le recomendaría que evite el consumo de lípidos y aminoácidos para disminuir
la acción de:
a. CCK

74. Paciente con apendicitis aguda, que debuta con dolor en mesogastrio. Este dolor se debe a estimulación
de receptores del dolor cuyas fibras van a viajar a la médula espinal a través de:
a. Nervios simpáticos

75. Todos los músculos motores de la lengua están inervados por el XII par, excepto:
a. Palatogloso

76. Al introducir una solución azucarada directamente al estómago mediante una gastrostomía (comunicación
entre la piel abdominal y el estómago), la sustancia que provocará que aumenten los niveles séricos de
insulina es:
a. Péptido tipo glucagón 1 (GLP-1)

77. En un experimento, con una sonda nasogástrica se instila por goteo en el estómago una sustancia
líquida, y se obtiene como respuesta una dramática disminución del pH del estómago. Dicha sustancia
debe contener:
a. Aminoácidos

78. En un paciente con hiperestimulación simpática se espera que las ondas lentas tengan un ritmo:
a. Menor en íleon terminal que en el duodeno

79. Se considera que el gusto puede viajar a través del nervio:


a. Glosofaríngeo

80. El ecografista sabe que para poder visualizar el nacimiento de la arteria mesentérica superior, debe
colocar el transductor sobre la piel de la siguiente región abdominal:
a. Epigastrio

81. Al seccionar el nervio facial a nivel timpánico, usted esperaría:


a. Disminución del gusto en la punta de la lengua

82. Marque lo correcto en relación al divertículo de Meckel.


a. Se encuentra usualmente a 60 cm de la VIC

83. Es un ligamento derivado del mesenterio dorsal:


a. Gastrocólico

84. Marque lo correcto respecto a la siguiente imagen:

a. Se produjo por falta de fusión de los ductos dorsal y ventral

85. La arteria mesentérica superior emerge de la aorta a nivel de:


a. Cabeza de páncreas

86. La digestión de las proteínas se inicia en:


a. Estómago

87. Aproximadamente en la semana 6 del desarrollo embrionario, el intestino intermedio gira 90º herniándose
a nivel del:
a. Cordón umbilical

88. Paciente de 76 años en estado de coma, con colostomía (colon abocado a la piel) por resección parcial
de colon secundaria a carcinoma de colon. Usted deja indicado que este día se realice el cambio de la
bolsa de colostomía, pero que lo hagan después de dos horas de la nutrición enteral, debido a que quiere
que se haga después de:
a. El reflejo gastrocólico

89. Recién nacido de 14 días, que presenta estreñimiento, distensión abdominal y vómitos. Al examinar el
orificio anal, se evidencia conducto permeable, pero al introducir un poco el termómetro, se evidencia
salida de material fecal por el ano. Usted sospecha que el problema se deba a:
a. Falta de desarrollo del tabique urogenital
b. Defecto en el desarrollo del conducto vitelino
c. Defecto en el desarrollo de las células de cajal
d. Falta de desarrollo del seno uretral
e. Falta de regresión de la membrana anal
90. La lengua está recubierta por epitelio:
a. Plano estratificado no queratinizado

91. El esfínter anal interno tiene musculatura ___ y tiene control ___:
a. Lisa/involuntario

92. La arteria aorta proporciona la irrigación al tubo digestivo ¿cuál de las siguientes arterias de irrigación al
ángulo cólico derecho?
a. Mesentérica superior

93. Paciente de 26 años que le cuenta en su historia clínica que cada vez que almuerza, a los 20 minutos
tiene deseo de defecar. Le comenta que su hijo de 1 mes le pasa lo mismo pero más intenso. Esto se
explica por el reflejo ____, el cual está _____ en el paciente:
a. Gastrocolico/normal

94. La región del estómago que se comunica con el duodeno se denomina


a. Pilórica

95. Acude a consulta un paciente que fue diagnosticado de úlcera péptica tres días antes. Luego de múltiples
pruebas diagnósticas, se concluye que el paciente presenta un tumor secretor de gastrina ¿cual de las
siguientes situaciones estará incrementada?
a. Secreción de ácido clorhídrico (HCl)

96. En el sistema digestivo el control del apetito estado por un complejo sistema de sustancias y órganos
integradores los cuales regulan le dijiste alimentos. La ____ es una sustancia orexígena/orexígena y es
sintetizada por el ____
a. Grelina/estómago

97. Sobre el control Autónomo del sistema digestivo marque la alternativa correcta:
a. En el sistema simpático, los nervios responsables hacen una primera sinapsis en ganglios
próximos al órgano a inervar

98. Con respecto a la actividad eléctrica del sistema digestivo, marque la alternativa correcta:
a. La frecuencia de las ondas lentas no se ve influenciada por la actividad neural ni las hormonas
gastrointestinales

99. Ante una lesión en el IX par craneal, el músculo ___ se altera en su función:
a. Estilofaríngeo

100.Un varón de 50 años es sometido a extirpación de duodeno y parte proximal del yeyuno. La pérdida de
estímulo hormonal en el páncreas para la secreción enzimática se explica por la pérdida de las células:
a. “I” productoras de CCK

101.Respecto al mecanismo de defecación ¿cual de las siguientes afirmaciones es correcta?


a. Se produce contracción o relajación del esfínter anal externo por señales de la corteza cerebral

102.Un niño de tres años llega a emergencia con disfagia (dificultad para tragar), salivación y llanto. Se
sospecha de ingesta de cuerpo extraño (moneda en el esófago), al ser evaluado se constata de una
radiografía presencia de cuerpo extraño a nivel de C6 y C7 (6º y 7º vértebra cervical). El cuerpo extraño
estará suspendido a nivel del estrechamiento producido por el:
a. Músculo cricofaríngeo

103.En el caso de un paciente con un tumor productor de gastrina, la presencia de úlceras duodenales y
erosión de la mucosa gástrica se debe principalmente a:
a. El exceso de HCl por estímulo de receptores CCK-B en la célula parietal

104.La onda peristáltica secundaria del esófago se caracteriza por ser originada:
a. Por el plexo mientérico del esófago

105.Marque lo correcto sobre las ondas lentas en el tubo digestivo:


a. Se constituyen de despolarizaciones y repolarizaciones
106.Recién nacido presenta protrusión de contenidos abdominales los cuales no están cubiertos por peritoneo
y salen de la cavidad abdominal a través de un defecto en la pared ¿Cómo se denomina a la afección
que presenta este paciente?
a. Gastrosquisis

107.Experimentalmente se utiliza atropina (anticolinérgico) para inhibir la secreción de gastrina, sin embargo,
la secreción de esta hormona se sigue dando ante estímulos vagales. Esta situación se explica porque la
atropina:
a. No bloquea la acción del péptido GRP

108.La niña de 4 días es llevada a la emergencia pediátrica por presentar llanto constante, la madre refiere
coloración azulada de labios al momento de lactar, acompañado de tos persistente y dificultad
respiratoria, así como distensión abdominal. Se le coloca sonda nasogástrica para alimentación notando
que retorna a la cavidad oral en todos los intentos ¿Cuál es la anomalía del desarrollo en este caso?
a. Atresia esofágica proximal con fístula traqueo esofágica distal

109.Paciente varón de 36 años es traído a la emergencia luego de sufrir un accidente de tránsito presenta
traumatismos múltiples en cabeza y tronco. Al examen físico se evidencia hematoma en hemicara
izquierda, ligera protrusión y caída del lado izquierdo del maxilar inferior por lo que se le realiza
tomografía donde se haya una fractura de apófisis coronoides del maxilar inferior. ¿Qué músculo está
relacionado directamente con esta situación?
a. Temporal

110.Un paciente refiere no percibir algunos sabores. Al examen físico constata alteración del sabor dulce y
umami. ¿Cuál de los siguientes nervios estará alterada su función?
a. Cuerda del tímpano (VII par)

111.A los pocos días de nacido, regresa a neonatología un niño con problemas de motilidad del colon, los
estudios determinan ausencia congénita de células ganglionares. Según el gráfico cuál es la capa en la
que se determina la ausencia de dichas células?

a. Muscular propia - 3

112.Con respecto al control autonómico en el tracto gastrointestinal y en relación con su fisiología ¿Cuál es la
función del sistema nervioso parasimpático y el tracto gastrointestinal?
a. Estimulan la contracción muscular y estimulan la secreción de sustancias a nivel de la mucosa

113.Intersticio ubicado entre el estroma del espacio portal y los hepatocitos, y por donde migran las células
cancerígenas que hacen diseminación linfática es el:
a. Espacio del Mall

114.La presencia de grandes cantidades de TGF - Beta estimula a las ___________ y se deposita colágeno,
formándose la cirrosis
a. Células de Ito
115.Paciente de 64 años con ICC al que se le va a realizar cirugía cardiovascular. Al calcular el volumen
sanguíneo total, se debe considerar que el hígado puede contener un volumen de sangre de
____________ mL en un adulto sano, en este paciente ese volumen puede llegar a ser de
_____________ mL
a. 450 - 1000

116.El área del lobulillo que se afecta más en caso de hipoxia es la zona:
a. 3

117.Paciente con carcinoma de vesícula biliar. La metástasis por continuidad afectará al lóbulo:
a. Cuadrado

118.El ácido acetilsalicílico actúa en la membrana:


a. Basolateral de la célula parietal

119.Al realizar una esplenectomía, se tiene que resecar la arteria esplénica, lo cual no es problema para el
estómago por que la arteria gastroomental izquierda se anastomosa con la:
a. Gastroomental derecha

120.La arteria esplénica proviene de la aorta y la vena esplénica desemboca en la vena:


a. Porta

121.Paciente mujer 21 con bulimia, que luego de un episodio de vómitos presenta hematemesis y al examen
físico que se encuentra crépitos subcutáneos cervicales. El diagnóstico más probable es:
a. Sd. Boerhaave

122.Paciente varón de 60 años, con antecedentes de promiscuidad sexual, tabaquismo y alcoholismo, acude
a consulta por presentar disfagia progresiva, odinofagia y al examen se observa tumor por parte posterior
de la lengua. La mejor posibilidad diagnóstica es:
a. Carcinoma escamoso

123.Niño de 5 años con historia de tres días de evolución caracterizado por fiebre, malestar general,
odinofagia, anorexia, e irritabilidad. Al examen de observa lesiones ulcerativas de 4mm de diámetro en
mucosa yugal, con borde blanquecino y eritema periférico. El diagnóstico más probable es:
a. Aftas orales

124.Es considerada una lesión preneoplásica


a. Leucoplasia

125.El esófago de Barrett se considera una lesión preneoplásica que se caracteriza por la presencia en
esófago de:
a. Metaplasia intestinal

126.Paciente mujer de 23 años gestante con lesión proliferativa en mucosa oral producida por proliferación
reactiva de vasos sanguíneos. Marque la mejor respuesta
a. Epulis

127.La glándula parótida tiene principalmente acinos de tipo:


a. Seroso

128.La reabsorción de sodio y secreción de potasio es estimulada por:


a. Aldosterona

129.Paciente con cirrosis hepática que tiene hipertensión portal con várices esofágica, y actualmente presenta
varices en estómago distal. Estas várices están relacionadas a aumento en la presión de las venas:
a. Gástrica derecha

130.La glándula parótida está inervada por el par craneal:


a. IX

131.Paciente con cirrosis hepática que tiene hipertensión portal con varices en esofago distal . Estas varices
están relacionadas a aumento en la presión de las venas gástricas
a. Inferior

132.La información eferente que sale de los núcleos salivales superior e inferior a través de los pares VII y IX
hacia las glándulas salivales llevan información tipo
a. Parasimpático

133.Un niño de 4 años ingresa en el hospital con vómitos graves . En el estudio se encuentra que el niño tiene
un páncreas anular ¿ Cuál de las siguientes hormonas gastrointestinales se encontrará a niveles
elevados en sangre con mayor probabilidad a raíz de esta patología
a. Gastrina

134.Durante una colecistectomía laparoscópica en un hombre de 61 años ¿ Cuál de las siguientes arterias
debe pinzar para extirpar la vesícula biliar con seguridad?
a. Cistica

135.Un hombre de 34 años se somete a una apendicectomía de urgencia . Después de realizar la


apendicectomía satisfactoriamente , el paciente se somete a una laparoscopia exploratoria¿Cual de las
siguientes características anatómicas es más útil para distinguir entre yeyuno e íleon?
a. El yeyuno tiene menos grasa mesentérica que el íleon

136.Una mujer de 45 años ingresa en el hospital con síntomas de obstrucción intestinal superior .En la TC se
encuentra que la tercera porción (transversa) del duodeno está comprimida por un gran vaso ¿Cuál de
los siguientes vasos causara muy probablemente la obstrucción ?
a. Arteria mesentérica superior

137.Durante una colecistectomía laparoscópica programada en una mujer de 47 años , el residente pinchó
accidentalmente el ligamento hepatoduodenal en vez de la arteria cística ¿Cual de los siguientes vasos
estaría muy probablemente ocluido en esta lesión iatrogénica?
a. Arteria hepática izquierda

138.Un hombre de 54 años ingresa en urgencias con intenso dolor abdominal superior . La gastroscopia
revela un tumor en el antro del estómago . Se pide una TC para evaluar el drenaje linfático del
estómago¿Cuál de los siguientes nódulos linfáticos estará muy probablemente afectado en una neoplasia
maligna del estómago
a. Mesentérico inferior

139.Una mujer obesa de 45 años con fiebre alta acude a la consulta con náuseas y dolor agudo e intermitente
en el cuadrante superior derecho del abdomen de 2 días de duración - Tiene una historia de ictericia de
24 horas. Tiene antecedentes de litiasis biliar. Bilirrubina total del 10 mg/dL. Lipasa de 5 mg/mL. ¿Cuál de
las siguientes estructuras está muy probablemente obstruida por un cálculo biliar?
a. Ampolla de Vater

140.Una mujer de 45 años ingresa en urgencias con dolor abdominal intenso . La TC y RM revelan un tumor
de la cabeza del páncreas que afecta el proceso unciforme ¿Cual de los siguientes vasos es más
probable que suministre irrigación a parte de la zona afectada?
a. Arteria cólica media

141.Un hombre de 70 años ingresa en urgencias con diarrea intensa . La arteriografía revela un bloqueo del
90% en el origen aórtico de la arteria mesentérica inferior ¿Cuál de las siguientes arterias proporciona
muy probablemente irrigación colateral al colon descendente?
a. Arteria sigmoidea

142.Al disminuir el pH duodenal por el HCL gástrico , se libera principalmente una hormona cuya célula diana
es
a. Célula ductal de Wirsung

143.Una de las siguientes sustancias reguladoras , puede actuar de forma paracrina y como hormona
a. Somatostatina

144.Al ingerir grandes cantidades de dulces , con la subsecuente estimulación de incretinas , usted esperaría
que el apetito
a. Disminuya por insulina

145.Paciente obeso con Covid-19 es intubado por interno inexperto, quien al solicitar que bombeen aire
dentro del tubo endotraqueal, nota que el epigastrio se distiende. Al sospechar que ha introducido el tubo
en el estómago, también es cierto que:
a. Disminuye el pH gástrico

146.La hormona que tiene un efecto sinérgico con la secretina para optimizar el pH duodenal y la digestión,
es:
a. CCK

147.El ecografista sabe que para poder visualizar el nacimiento de la arteria mesentérica superior , debe
colocar el transductor sobre la piel de la siguiente región abdominal
a. Epigastrio

148.En un paciente con hiperestimulación simpática se espera que las ondas lentas tengan un ritmo
a. Mayor en estómago que en el duodeno
b. Mayor en estómago que en el íleon terminal
c. Menor en íleon terminal que en el duodeno
d. Mayor en el íleon que en duodeno

149.Se considera que el gusto puede viajar a través del nervio


a. Glosofaríngeo

150.La rotación en sentido longitudinal del estómago en el desarrollo embriológico condiciona que el nervio
vago derecho quede a nivel
a. Posterior

151.Con respecto al control autonómico en el tracto gastrointestinal y en relación con su fisiología. ¿Cuál es la
función del sistema nervioso parasimpático en el tracto gastrointestinal?
a. Estimulan la contracción muscular y estimulan la secreción de sustancias a nivel de la mucosa.

152.Un estudiante que está preocupado por su examen parcial, no ha desayunado ni almorzado; cuando al fin
ingiere alimentos, este le provoca el aumento de los movimientos musculares del tracto gastrointestinal y
la sensación de defecar.?¿Qué reflejo se ha activado?
a. gastro - cólico

153.¿De que par craneal es rama el nervio palatino mayor?


a. Trigémino

154.¿En cuál de las fases de deglución la epiglotis separa la vía respiratoria de la digestiva?
a. Faríngea

155.Los péptidos intestinales se pueden clasificar como sustancias endocrinas, neurocrinas y paracrinas,
dentro de las paracrinas se encuentran la somatostatina e histamina. Marque la respuesta correcta
a. La histamina es sintetizada por células de tipo paracrino de las glándulas gástricas
156.Paciente varón de 27 años es llevado por bomberos a emergencia luego de ser asaltado y, tras resistirse,
es cortado con el pico de una botella a nivel abdominal. Al examen físico usted observa que a través de la
herida se puede observar la protrusión de las asas intestinales. En relación con las capas de la pared
abdominal, marque la alternativa correcta.
a. El músculo recto del abdomen tiene como funciones comprimir el contenido del abdomen, tensar
la pared abdominal y flexionar la columna

157.Los músculos del tracto gastrointestinal de los segmentos propulsivo y receptor del bolo alimenticio,
responden de forma diferente al movimiento de este bolo a través del intestino. ¿Cuál de las siguientes
afirmaciones describe correctamente la actividad del segmento propulsivo?
a. Tanto el músculo circular como el longitudinal están relajados
b. El músculo longitudinal está relajado y el circular esta contraído
c. Tanto el músculo circular como el longitudinal están contraídos
d. El músculo longitudinal está contraído y el circular está relajado

158.El nervio vago inerva al músculo


a. Estriado del esófago

159.¿Cuál de las siguientes es una característica de los ganglios mioentéricos del sistema nervioso entérico?
a. Contiene mayor número de neuronas que el plexo submucoso
b. Es también conocido como el plexo de Meissner
c. Contiene sólo neuronas motoras excitatorias del músculo liso
d. Contiene neuronas sensitivas que activan a los músculos circular y longitudinal del tracto
intestinal

160.El divertículo faringoesofágico, hipofaríngeo de zenker, es una lesión muy particular que se localiza en la
cara póstero lateral de la Unión de la faringe con el esófago, como una herniación de la mucosa
esofágica a través de las fibras oblicuas del músculo.
a. Constrictor inferior de la faringe

161.La razón por la que el potencial de acción viaja rápidamente en sentido longitudinal por el músculo liso
gastrointestinal es la presencia de uniones en hendidura,
a. Varicosidades

162.Respecto a los péptidos gastrointestinales, marque lo correcto.


a. las sustancias paracrinas pueden viajar a través de vasos sanguíneos

163.Al ingerir rápidamente un litro de agua, usted esperaría que la gastrina aumenta por efecto de:
a. ACh del sistema mientérico

164.Respecto a la anatomia del estomago,marque lo correcto:


a. El fondo gástrico forma la curvatura mayor
(Pág 19 → 21)

165.Los nervios esplácnicos lumbares (L1-L2) llevan información de tipo:


a. Simpática

166.Paciente de 32 años con herida por arma de fuego y shock hipovolémico. El intestino delgado no se ha
infartado aún a pesar de la hipoxia gracias a la liberación de:
a. Adenosina

167.Al realizarse un piercing en el ombligo,la sensación de dolor se transmite por:


a. T10
168.El nivel en el que se encuentra el píloro y el páncreas se puede determinar usando el:
a. Píloro transpilórico

169.Permite la suspensión e irrigación de los órganos peritoneales:


a. Mesenterio
170.El dolor asociado a apendicitis clásicamente se ubica en:
a. Fosa iliaca derecha

171.El ligamento inguinales formado por la aponeurosis del:


a. Oblicuo externo

172.Los nueve cuadrantes del abdomen se delinean usando el plano subcostal,las líneas medioclaviculares y:
a. Plano intertubercular

173.Es un órgano peritoneal:


a. Hígado

174.La línea alba se encuentra:


a. Entre los rectos abdominales

175.Es un órgano retroperitoneal:


a. Páncreas

176.Enfermedad asociada con un error en el desarrollo de las células de Cajal


a. Enfermedad de Hirschsprung

177.El divertículo de Meckel es un rezago de:


a. Conducto vitelino

178.La fístula rectoperitoneal es causada por una falta en el desarrollo de:


a. Tabique urorrectal

179.95) Al disminuir el pH duodenal por el HCl gástrico, se libera principalmente una hormona cuya célula
diana es:
a. Célula ductal del Wirsung

180.Al introducir una solución azucarada directamente al estómago mediante una gastrostomía (comunicación
entre la piel abdominal y el estómago), la sustancia que provocará que aumenten los niveles séricos de
insulina es:
a. Péptido tipo glucagón 1 (GLP-1)

181.La rotación en sentido longitudinal del estómago en el desarrollo embriológico condiciona que el nervio
vago derecho quede a nivel:
a. Posterior

182.Una de las siguientes sustancias no comparte con las otras la misma acción sobre la producción de ácido
gástrico:
a. Colecistoquinina

183.En un paciente con gastroparesia (motilidad lenta del estómago), que presenta distensión abdominal
después de comer, usted le recomendaría que evite el consumo de lípidos y aminoácidos para disminuir
la acción de:
a. CCK

184.En un paciente con shock distributivo, usted decide iniciar noradrenalina por un catéter CVC,
consiguiendo aumentar la presión arterial. ¿qué efecto sobre la motilidad intestinal esperaría encontrar?
a. El potencial de reposo de las fibras musculares se hace más negativo

185.El ecografista sabe que para poder visualizar el nacimiento de la arteria mesentérica superior, debe
colocar el transductor sobre la piel de la siguiente región abdominal:
a. Epigastrio
186.La glándula submandibular recibe inervación traída por el nervio
a. Cuerda del tímpano

187.A mayor flujo de saliva, disminuye la concentración de:


a. Potasio

188.En términos de mg/mL, el principal componente de la saliva es _____ seguido de ____:


a. Proteínas - Potasio

189.La reabsorción de Sodio y Cloro en las glándulas salivales se da principalmente en el:


a. Conducto estriado

190.La información eferente que sale de los núcleos salivales superior e inferior a través de los pares VII y IX
hacia las glándulas salivales llevan información de tipo:
a. Parasimpático

191.La glándula submaxilar le hace gancho al:


a. Músculo milohioideo

192.Los conductos salivales son ___ al agua, esa es una de las razones por las cuales la saliva es siempre
___.
a. Impermeables - hipotónica

193.Los nervios esplácnicos pélvicos (S2-S4) llevan información de tipo:


a. Parasimpática

194.113) Presenta movimientos en masa:


a. Colon

195.114) La digestión de los lípidos se inicia en:


a. Intestino delgado

196.115) La digestión de los carbohidratos se inicia en:


a. Cavidad oral

197.117) El estómago recibe información simpática proveniente del:


a. Ganglio celiaco

198.118) En el intestino delgado se absorbe los carbohidratos en forma de:


a. Fructosa

199.119) Es rama de la arteria mesentérica superior:


a. A. cólica media

200.120) El “dolor de estómago” asociado a gastritis se suele ubicar en:


a. Epigastrio

201.121) El azúcar de mesa sacarosa es digerido a dos monosacáridos que comparten el transportador:
a. GLUT 2

202.122) Durante la defecación se requiere:


a. Señales inhibitorias en el nervio pudendo

203.123) En un paciente con fístulas intestinales y fisuras anales, con antecedente de enfermedad
inflamatoria intestinal, ud sospecharia en:
a. Enfermedad de Crohn
204.En los pacientes con Colecistitis Aguda no operable, una opción es la colocación de una sonda por el
cístico, procedimiento en el que se ingresa con dificultad debido a la estrechez del cístico y a la presencia
de:
a. Válvula de Herring

205.Paciente de 51 años con antecedente de enfermedad diverticular acude a emergencia por sangrado
profuso y dolor en hipocondrio izquierdo. ¿Cuál es el origen más probable de la sangre que pierde el
paciente ?
a. Mesentérica inferior

206.Paciente de 42 años con dolor abdominal intenso y hematemesis. En la endoscopia se observa una
úlcera duodenal posterior perforada con hemorragia intraabdominal. ¿Cuál de las siguientes arterias
estará comprometida?
a. Pancreaticoduodenal posterosuperior

207.Para que los triglicéridos sean absorbidos deben ser metabolizados a


a. Monoglicéridos y Ácidos grasos

208.El vibrio cholerae produce diarrea porque:


a. Aumenta la producción de AMPc en los enterocitos

209.La motilidad intestinal es estimulada por


a. Colecistoquinina y gastrina

210.La diarrea por deficiencia de lactasa es de tipo


a. osmótica

211.La metoclopramida estimula el vaciamiento gástrico aumentando la fuerza de contracción de las paredes
gástricas esto puede conseguirlo mediante la estimulación indirecta de las neuronas liberadoras de
a. Acetilcolina

212.Los vértices de un acino hepático están constituidos por


a. Dos espacios porta y dos venas centrolobulillares

213.Al ingerir un pedazo de mantequilla, cuál de los siguientes tiene un efecto directo en la reducción del
vaciado gástrico:
a. Colecistoquinina

214.Tras la vagotomía (resercion del vago) por enfermedad úlcera péptica en un paciente UD. esperaría
encontrar:
a. Aumento del pH gástrico

215.Se realizó un experimento en el cual se inyectó tinta china en el peritoneo de ratas de laboratorio. Al
realizarse una biopsia hepatica de dichos animales, se encontro que el tinte negro fue fagocitado por:
a. Células de kupffer

216.En un paciente con pH gástrico muy bajo, es posible que la siguiente sustancia se secrete en menor
cantidad:
a. Gastrina

217.La fase cefálica de la secreción gástrica responde por cerca del 30% de la respuesta ácida a un reflejo
con la _____________ se elimina la fase cefálica de la secreción gástrica
a. Vaguectomía

218.El esofago de barret se caracteriza por presentar _________ en el esofago


a. Metaplasia intestinal
219.La presencia de orina que sale por el ombligo de un recien nacido cada vez que llora, es posible que
deba a un defecto en el desarrollo de:
a. Seno urogenital

220.El conducto biliar deriva del:


a. Endodermo

221.Cuál de los siguientes órganos son intraperitoneales:


a. Estómago, Vesícula biliar, Ileón, Hígado

222.Los vasos mesentéricos superiores se hallan a nivel de:


a. Cuello del páncreas

223.La colecistoquinina (CCK) inhibe:


a. El vaciamiento gástrico

224.Un niño de 2 años es llevado a la consulta por diarrea persistente, edema de las extremidades y falta de
crecimiento en relación a su edad. Los análisis de sangre revelan que tiene concentración plasmática
baja de proteínas (hipoproteinemia). Como parte del estudio se coloca Colecistoquinina (CCK)
endovenosa y se recoge muestras del líquido duodenal por endoscopia; el resultado del líquido confirma
incapacidad para hidrolizar proteínas a un pH neutro, esta situación mejora al añadir una pequeña
cantidad de tripsina. El paciente probablemente esté sufriendo la falta congénita de ……….
a. Enterocinasa

225.Paciente mujer de 35 años acude a consulta por sensación de sequedad y lesiones en la cavidad oral. Al
examen se observa atrofia de la mucosa, fisura y úlceras; nota además sequedad e irritación de la córnea
y aumento del tamaño de las glándulas parótidas. Su diagnostico mas probable es artritis reumatoide; el
hallazgo más probable en una biopsia de glándula parótida es ….…
a. Gran infiltración de linfocitos y células plasmáticas

226.Un hombre de 42 años de edad se presenta al médico con una historia clínica de 1 año de evolución,
caracterizado por dolor abdominal bajo y diarreas con crisis sanguinolentas. Manifiesta además pérdida
de peso de 8kg durante este periodo. La colonoscopia revela lesión difusa en el colon con afectación del
recto. La biopsia de estas lesiones revela adelgazamiento de la pared, inflamación y ulceración de la
mucosa y submucosa. El diagnóstico más probable en este caso es:
a. Colitis ulcerativa

227.Dos estudiantes deciden tomar un receso para comer una hamburguesa a la hora del almuerzo. Antes de
llegar a la cafetería, impulsos nerviosos provenientes del complejo vagal dorsal iniciarán la secreción de
ácido gástrico por la liberación de ___________ desde el sistema nervioso entérico.
a. GRP (péptido liberador de gastrina)

228.Un niño de cuatro años de edad es llevado a la consulta por cuadros diarreicos frecuentes caracterizados
por heces pálidas, voluminosas y fétidas; al examen físico presenta bajo peso y talla para la edad. Se
mide la concentración de cloruro de en el sudor y se encuentra que sus valores son muy elevados. La
alteración más importante a nivel de células ductales del páncreas tiene relación directa con la
conductancia de …..
a. Cloro
229.Se evalúa los valores séricos de las siguientes sustancias a un paciente con enfermedad hepática
terminal; en este paciente se espera encontrar la combinación de la letra …..

Glucosa Amoniaco Albúmina

a. Aumentada Disminuida Disminuida

b. Disminuida Aumentada Aumentada

c. Aumentada Aumentada Aumentada

d. Disminuida Aumentada Disminuida

230.Una mujer de 35 años de edad HIV positiva, se presenta al médico con dolor abdominal en cuadrante
superior derecho e ictericia. La paciente refiere haber tenido múltiples episodios de ictericia durante los
últimos 10 años. Los exámenes para determinar hepatitis viral, dieron positivos para Hepatitis B, siendo
catalogado el caso como hepatitis crónica con alteración funcional. En un examen de sangre ¿Cuál de los
siguientes parámetros está disminuido?
a. Albúmina

231.En el reflejo peristáltico del intestino delgado ¿Cuál de los siguientes eventos sucede en la porción caudal
del bolo alimenticio?
a. Acción del péptido inhibidor vasoactivo (VIP) en el músculo circular

232.Un varón de 58 años de edad con enfermedad de Crohn severo fue sometido a una resección ileal.
Después de la cirugía esta paciente padecerá de esteatorrea esto se explica porque ……
a. Hay malabsorción de ácidos biliares

233.En un experimento se inserta un balón en el estómago de un voluntario, se infla poco a poco mientras
que se vigilan las presiones intraluminales. Aunque el volumen del balón aumenta considerablemente, las
presiones permanecen constantes. Esta relación volumen-presión se explica por la liberación local de ….
a. Óxido nítrico y péptido inhibidor vasoactivo

234.¿Cuál de las siguientes alternativas es una característica de la secreción exocrina del páncreas?
a. Su mayor estímulo se da en la fase intestinal

235.Las estructuras en el hígado que permite que los productos metabólicos unidos a proteínas tengan
acceso a las membranas basolaterales de los hepatocitos, son ….
a. Las fenestras sinusoidales

236.La composición de la bilis es modificada conforme fluye por los conductillos biliares. Durante este tránsito
se espera que aumente la concentración de …..
a. Ig A

237.Se mide experimentalmente el contenido gástrico de dos personas. La persona “A” tiene alto contenido
de grasa y la persona “B” tiene un contenido isotónico. ¿Cual de las siguientes es correcta respecto al
vaciamiento gástrico?
a. Hay ralentización del vaciado gástrico sólo en “A”
238.El examen endoscópico de un paciente con hipertensión portal grave revela venas tortuosas que
sobresalen hacia la luz del esofago. El paciente recibe tratamiento quirúrgico mediante la colocación de
una derivación que conecta la vena cava. Después de la operación el riesgo de encefalopatía …….. y el
sangrado de varices ……
a. Aumentará/Disminuirá

239.Un bolo alimenticio grande y poco masticado se atasca en el esofago, esto ocasiona una sensacion de
dolor que es transmitida por los nervios:
a. Esplácnico

240.El peristaltismo del intestino delgado se puede intensificar debido a:


a. Irritación de la mucosa

241.Un paciente es diagnosticado con un tumor neuroendocrino productor de somatostatina, esto provocará
en el sistema digestivo:
a. Diarrea

242.Los diferentes segmentos del tubo digestivo son susceptibles de reflejos y movimientos según su
contenido.Si colocoramos mediante una sonda un bolo alimenticio directamente en el tercio medio del
esofago:
a. Se producirá ondas secundarias

243.En el digestivo la liberación hormonal se presenta ante diversos factores o estímulos. La hormona
_________ es estimulada por la presencia de alimentos en el bulbo duodenal a predominio de ácidos
grasos y triglicéridos, por estimulación vagal y por la hormona secretina.
a. Colecistoquinina (CCK)

244.El divertículo de Meckel es una anomalía congénita que ocurre por la persistencia del conducto vitelino y
da origen a una estructura sacular, el cual se encuentra en el:
a. Borde antimesentérico

245.La fase oclusal de la masticación se realiza con la contracción de los músculos inervados por el nervio
craneal:
a. V

246.En un paciente de 43 años con tumor carcinoide de páncreas productor de gastrina (Sindrome de
Zollinger-Ellison) se puede encontrar una potenciación del reflejo:
a. Gastrocolico

247.Durante la fase faríngea de la deglución ocurre el siguiente mecanismo:


a. La onda peristáltica lleva el alimento hacia el esofago

248.En los carcinomas (neoplasia benigna) es frecuente que ocurran la metástasis a través de los vasos
venosos. En el caso de un carcinoma del tercio superior del esofago, ubicado en la cara lateral izquierda,
es más probable que la metástasis viaje por la vena:
a. Hemiácigos accesoria

249.168) Los movimientos en masa son iniciados por el reflejo:


a. Duodenocolico
250.Una mujer de 65 años HIV positiva se presenta con dolor abdominal en el cuadrante superior derecho e
ictericia. La paciente afirma haber teñido múltiples episodios de ictericia durante los últimos 10 años. Los
exámenes para poder detectar hepatitis viral, dieron positivos para Hepatitis B, siendo catalogada como
hepatitis crónica con alteración funcional. En un examen de sangre ¿cuál de los siguientes parámetros
está disminuido?
a. Albúmina

251.En el reflejo peristáltico del intestino delgado¿cuál de los siguientes eventos sucede en la porción caudal
del bolo alimenticio?
a. Acción del péptido inhibidor vasoactivo(VIP) en el músculo circular

252.Un varón de 58 años de edad con enfermedad de Crohn Severo fue sometido a una resección ilegal.
Después de la cirugía este paciente padecerá de esteatorrea, esto se explica porque ….
a. Hay mala absorción de los ácidos biliares

253.En un experimento se inserta un balón en el estómago de un voluntario, se infla poco a poco mientras se
vigilan las presiones intraluminales. Aunque el volumen del balón aumenta considerablemente, las
presiones permanecen constantes. Esta relación volumen presión se explica por la liberación local de ….
a. Óxido nítrico y péptido inhibidor vasoactivo

254.¿Cuál de las siguientes alternativas es una característica de la secreción exocrina del páncreas?
a. Su mayor estímulo se da en la fase intestinal

255.Las estructuras en el hígado que permiten que los productos metabólicos unidos a proteínas tengan
acceso a membranas basolaterales de los hepatocitos son ….
a. Las de fenestras sinusoidales

256.La composición de la bilis es modificada conforme fluye por los conductillos biliares. Durante este tránsito
se espera que aumente la concentración de ….
a. IgA

257.Se mide experimentalmente el contenido gástrico de dos personas. La persona A tiene alto contenido de
grasas y la persona B tiene un contenido isotónico ¿Cuál de las siguientes es correcta respecto al
vaciamiento gástrico?
a. Hay ralentización del vaciado gástrico sólo en A

258.El examen endoscópico de un paciente con hipertensión portal grave revela venas tortuosas que
sobresalen hacia la luz del estómago. El paciente recibe tratamiento quirúrgico mediante la colocación de
una derivación que conecta la vena porta a la vena cava. Después de la operación el riesgo de
encefalopatía …. y el riesgo de sangrado de varices ….
a. Aumentará / disminuirá

259.Un paciente varón de 18 años de edad acude al médico para sus exámenes de rutina. Sus resultados de
laboratorio muestran un valor de bilirrubina sérica de 4mg/dl y una bilirrubina directa de 0.3 mg/dl. Las
pruebas de función hepática son normales. La alteración que explica mejor este caso es por la deficiencia
de….
a. Glucuronil transferasa

260.Un hombre de 57 años de edad es llevado a urgencias con hematemesis masiva rojo brillante, a su
llegada se halla inconsciente con PA: 80/40 mm Hg y FC:124 lat/min. Luce ictérico con presencia de
arañas vasculares en el tórax anterior y extremidades, abdomen distendido con signo de oleada positiva.
Se encuentra esplenomegalia y pérdida de la masa muscular en extremidades. La anastomosis vascular
responsable del sangrado en este paciente es ….
a. Vena gastrica izquierda y vena acigos
261.Un estudiante de medicina está comiendo un plato de comida a base de champiñones, espárragos y
salsa de soya. El estímulo del sabor umami contenido en todos estos alimentos viaja a través del
nervio….
a. Cuerda del tímpano

262.Una paciente de 30 años de edad es sometida a una cirugía de oído medio derecho por un problema de
otoesclerosis. Luego de la cirugía refiere alteración sensitiva de la lengua. Al evaluar el caso usted
esperaría encontrar ….
a. Sensación del dolor, tacto y temperatura conservadas

263.En una paciente de 45 años de edad con colestasis biliar, se encuentra una elevación de los niveles
sanguíneos de fosfatasa alcalina hasta 3 veces la cifra normal ¿Cuál de las siguientes alternativas estará
también elevada como evidencia del daño biliar?
a. Gamma glutamil transpeptidasas

264.Experimentalmente se incrementa la velocidad de la secreción salival con una sustancia, en el análisis de


la composición de esta saliva obtenida se espera encontrar….
a. Aumento de la concentración de bicarbonato que supera la concentración plasmática

265.Lactante de 3 meses de vida es atendido por presentar diarrea, se administra una solución glucosa y
electrolitos por vía oral. La proteína de membrana apical que explica la capacidad de esta solución para
proporcionar aporte de glucosa e hidratación es ….
a. SGLT 1

266.Paciente ha sufrido herida de bala en el abdomen, se le tenido que extirpar el segmento medio y distal del
íleon. En este caso la síntesis hepática de sales biliares estará ….
a. Incrementada por estímulo de la enzima colesterol 7 alfa hidroxilasa

267.Un varón de 75 años ingresa al consultorio por presentar ictericia marcada de piel y escleras. El estudio
del paciente mostró que presentaba un tumor que obstruía la totalidad del conducto hepático común.
¿Cual de los siguientes conductos se encontraría dilatado en este paciente?
a. De Hering

268.Correlacione las dos columnas y marque la respuesta correcta:


a. Enfermedad Hirschsprung Aganglionosis congénita
b. Diarrea osmótica. Intolerancia a la lactosa
c. Diarrea secretora. Canales de Cl- en las células de la cripta
d. Diarrea exudativa Heces con moco y sangre

269.La fase cefálica de la secreción gástrica responde por cerca del 39% de la respuesta ácida a un reflejo.
Con la ____ se elimina la fase cefálica de la secreción gástrica
a. Vaguectomía

270.El esofago de Barret se caracteriza por presentar___ en el esofago


a. Metaplasia intestinal

271.En la regulación del apetito y la saciedad, la estimulación experimental cronica del núcleo ventro medial
del hipotálamo producirá:
a. Afagia

272.Paciente mujer de 25 años acude por dolor en fosa iliaca derecha que empeora al toser o
caminar,asociada a náuseas y vómitos por lo cual acude a emergencia.Dos días después de realizarle
una apendicectomia, la paciente desarrolla fiebre alta (39° C), esta hipotensa y presenta dolor
abdominal.La laparotomia exploratoria muestra un gran volumen de sangre en la cavidad peritoneal por
lesion de un vaso producida durante la apendicectomía.¿Cual de las siguientes arterias debe ligarse
para detener la hemorragia?
a. Ileocolica
273.¿Cuál de los siguientes es una causa de ictericia con bilirrubina conjugada aumentada?
a. Obstrucción del colédoco

274.Dos días después de una apendicectomía en un hombre de 45 años, ha desarrollado fiebre alta (39), está
hipotenso y presenta dolor abdominal. La laparotomía exploratoria muestra un gran volumen de sangre
en la cavidad peritoneal por lesion de un vaso producida durante la apendectomía. ¿Cuál de los
siguientes vasos debe ligarse para detener la hemorragia?
a. Arteria ileocólica

275.Paciente de 78 años, con diabetes mellitus tipo ll y fumador, que acude a consulta porque desde hace
dos semanas tiene un dolor intenso en flanco derecho y mesogastrio , intenso que aparece a los 30
minutos de haber comido, y desaparece dos a tres horas después. En estos pacientes, es muy probable
que la circulación deficitaria sea parcialmente asumida por la:
a. Arteria cólica media

276.Un hombre de 70 años ingresa en urgencias con diarrea intensa. La arteriografía revela un blloqueo del
90% en el origen aórtico de la arteria mesentérica inferior. ¿Cuál de las siguientes arterias proporciona
muy probablemente irrigación colateral al colon descendente?
a. Arteria cólica derecha

277.Un niño de 4 años ingresa en el hospital con vómitos graves. La exploración radiológica y la historias
clinica revelan que el niño tiene páncreas anular. ¿Cuál de las siguientes estructuras es la que se
encontrará dilatada con mayor probabilidad a raíz de esta patología?
a. Primera porción del duodeno

278.Un hombre de 55 años ingresó al hospital con dolor abdominal intenso. La gastroscopia y la TC revelaron
una úlcera perforada en la pared posterior del estómago. ¿Dónde se desarrolla inicialmente con más
probabilidad una peritonitis?
a. Bolsa omental (saco menor)

279.Un hombre de 44 años ingresa en urgencias con vómitos abundantes y deshidratación. Las imágenes
radiológicas demuestran que parte del intestino está comprimido entre la aorta abdominal y la arteria
mesentérica superior. ¿Cuál de las siguientes estructuras intestinales estará muy probablemente
comprimida?
a. Tercera porción del duodeno

280.¿Cuál de los siguientes nódulos linfáticos estará muy probablemente afectado en una neoplasia maligna
del recto?
a. Mesentérica inferior

281.Una mujer de 23 años ingresa con dolor abdominal, náuseas y vómitos. La historia Clínica muestra que el
dolor es agudo y ha sido constante durante 4 años. El dolor empezó en el epigastrio e irradió
bilateralmente alrededor del tórax hasta justo debajo de las cápsulas. Actualmente el dolor se localiza en
el hipocondrio derecho. L a TC revela cálculos calcificados en la vesícula biliar. ¿Cuál de los siguientes
nervios llevan las fibras aferentes del dolor referido?
a. Nervios esplácnicos torácicos mayores

282.Un hombre de 55 años ingresa en urgencias por severa perdida de peso en los 6 meses previos. El
examen radiológico pruebas aportan signos de un tumor que causa hipertensión portal. Los estudios de
laboratorio revelan que las deposiciones son grasas, tiene desnutrición e hipoxia hepática. ¿Cuál de las
siguientes localizaciones se encuentra muy probablemente afectada?
a. Segmento l

283.El triángulo de calot es importante reconocerlo porque sirve como reparo para encontrar la arteria cística,
y está formado por el conducto cístico, el conducto hepático derecho y :
a. Borde hepático
284.El kernícterus se produce en recién nacidos con valores mayores de 25m/dL en la bilirrubina:
a. Indirecta

285.Paciente con cirrosis hepática y presión de vena cava de 15mmHg. Lo más probable es que el paciente
presente:
a. Ascitis

286.Considerando un flujo plasmático renal de 180 ml/min, y una fracción de filtración del 20%, si la
concentración en sangre de la bilirrubina indirecta es de 0.6mg/dL, entonces es correcto esperar que la
carga filtrada de la bilirrubina indirecta es:
a. Menor de 1080 mg/dL

287.El volumen diario de bilis secretada al intestino es:


a. 500-1000 ml

288.La excreción del amonio se da principalmente por:


a. Orina

289.Si en una persona normal, el flujo de la arteria hepática es de 700ml/min, entonces el flujo de la vena
porta debería ser aproximadamente:
a. 2800mL/min

290.Paciente con Grigler-Najjar debida a mutación del gen UGT1A1, se presenta a consulta por ictericia,
usted asume que si le hiciera un análisis de sangre encontraría valores elevados de:
a. Bilirrubina indirecta

291.El amoniaco corporal se forma principalmente en:


a. Colon

292.La zona del lobulillo hepático que se afectaría más en una intoxicación con droga hepatotóxica es la
zona:
a. 1

293.El acino hepático tiene en sus aristas:


a. Vena centrolobulillar

294.Paciente con esteatosis hepática no alcohólica, que en la biopsia se observa degeneración grasa de los
hepatocitos, lo cual se debe a depósitos de lípidos que principalmente contienen:
a. Triglicéridos

295.Al evaluar la orofaringe de un paciente, el médico le solicita que abra la boca, saque la lengua y diga a .
Al hacer esta maniobra, nota que el paladar se desvía hacia la derecha, lo cual le hace sospechar que el
paciente sufre de una lesión del nervio craneal:
a. X contralateral

296.Un bolo alimenticio grande y poco masticado se atasca en el esófago, esto ocasiona una sensación de
dolor que es transmitida por los nervios:
a. esplácnicos

297.Para realizar el movimiento mecánico de abrir la boca, primero se necesita:


a. fijar el hueso hioides

298.¿Cuál de las siguientes alternativas se define como la protrusión directa del contenido abdominal a la
cavidad amniótica por un defecto de la pared corporal?
a. Gastrosquisis
299.Un paciente requiere que se le coloque una sonda de alimentación directamente al estómago
(gastrostomía), el cirujano deberá hacer una incisión en la piel del abdomen ¿cuál de las siguientes
raíces nerviosas debe ser anestesiada para este procedimiento?
a. T8

300.El mecanismo de la defecación incluye la participación de diversas estructuras ¿Cuál de las siguientes
alternativas es correcta?
a. Puede ser mediado por un reflejo intrínseco

301.Cuando el contenido del estómago ingresa al duodeno, uno de los reflejos que inhiben el vaciamiento
gástrico es a través del:
a. sistema nervioso mientérico

302.Durante la masticación, gran parte del proceso masticatorio se debe a:


a. el reflejo masticatorio

303.Las glándulas salivales tienes conductos para la excreción de la saliva; las glándulas ____________
drenan en las carúnculas sublinguales.
a. sublinguales

304.En una persona si enfermedad se espera que el tránsito intestinal se vea disminuido cuando se presenta
el reflejo:
a. doloroso

305.El divertículo de Meckel es una anomalía congénita que ocurre por la persistencia del conducto vitelino y
da origen a una estructura sacular, el cual se encuentra en el:
a. borde antimesentérico

306.Si al intubar a un paciente, por error se ingresa el tubo endotraqueal en el esófago y se insufla el
manguito endotraqueal (globo TET), la dilatación de este manguito generará:
a. múltiples ondas secundarias

307.El orificio omental, o hiato de Winslow, se encuentra limitado por el ligamento:


a. hepatoduodenal

308.Paciente de 24 años acude a consulta externa por presentar una fístula oronasal (comunicación entre la
cavidad oral y la cavidad nasal). Está fístula es una consecuencia tardía de la lesión de un vaso
sanguíneo por el antecedente de haber sido operado de paladar hendido en los primeros años de vida,
aparentemente en una campaña gratuita de corrección de paladar fisurado. ¿Cuál de las arterias
palatinas podría haberse lesionado durante esa cirugía?
a. Mayor

309.Dentro de las anomalías congénitas se puede presentar un tejido pancreático accesorio ¿Cuál es la
ubicación más común de este tejido?
a. Estómago

310.Paciente con insuficiencia mitral moderada a severa, con aumento de volumen de la aurícula izquierda;
esta condición tendrá como consecuencia a nivel del sistema digestivo:
a. la disfagia a sólidos

311.El inicio de la fase faríngea de la deglución se debe a estímulos sensitivos que viajan por el nervio
craneal:
a. V (nervio maxilar)

312.El mesocolon transverso se origina en:


a. la pared posterior del abdomen
313. La contracción del músculo ………………………… permite la eliminación de gases (flatos) sin salida de
material fecal ; es el mismo músculo cuya relajación, sobretodo en cuclillas, permite el paso del contenido
fecal con menor esfuerzo durante la defecación
a. puborrectal
314. Paciente mujer de 54 años se presenta con náuseas, vómitos, estreñimiento, y es diagnosticada de
abdomen agudo quirúrgico; en la cirugía encuentran un vólvulo de ciego. Esta anomalía puede explicarse
por:
a. Falta de fusión del mesenterio
315.Paciente mujer de 23 años con faringitis aguda , toma para el dolor una tableta de paracetamol con un
poco de agua. Durante la deglución, se relaja su esfínter esofágico inferior y el fondo del estómago,
mientras el bolo está aún en el esófago. ¿Qué sustancia provocará con mayor probabilidad la relajación
del esfínter esofágico inferior y el fondo del estómago en esta mujer?
a. Óxido nítrico

316.Luego de tres horas dando exámenes, un alumno de medicina comienza a sentir hambre. Esta situación
es probable que sea mediada por la _____ que es sintetizada por el _____
a. grelina / estómago

317.Varón de 72 años, con antecedente de diabetes mellitus tipo 2, que presenta entropía diabética
caracterizada por estreñimiento. Este problema puede estar asociado a:
a. deficiencia de óxido nítrico

318.Varón de 54 años con diabetes mellitus tipo 2, es diagnosticado de gastroparesia debido a que presenta
sensación de llenura precoz al comer, y reflujo gastroesofágico. Esta alteración en la relajación receptiva
y en el vaciamiento gástrico lo más probable es que se deba a una alteración en:
a. el nervio vago

319.Varón de 67 años con tos y disminución de peso asociado a tabaquismo pesado, presenta actualmente
disfagia progresiva a alimentos sólidos. Se considera la presencia de un carcinoma de bronquio izquierdo
y por esta razón le realizan una endoscopia esofágica para descartar la posibilidad de una compresión
esofágica por el tumor. Se espera revisar el esófago en la _____ estrechez, que está a nivel de la
vértebra _____.
a. Tercera estrechez - T6

320.En un niño menor de dos años con divertículo intestinal, este divertículo tiene su origen en una falla en la
obliteración de:
a. Conducto vitelino

321.Mujer de 43 años sufre un grave accidente de tránsito y está hospitalizada en coma, es alimentada por
vía intravenosa durante varias semanas. Producto de este tipo de alimentación, se encuentra en la
endoscopia atrofia de la mucosa gastrointestinal. La causa más probable de esta atrofia son los bajos
niveles séricos de la hormona:
a. Gastrina

322.Una mujer de 30 años llega al consultorio porque se queja de dificultades para deglutir, la cual se agravan
cada vez más. Se realiza un estudio manométrico para examinar la generación de presión a lo largo del
esófago. Esta prueba revela que las contracciones como respuesta a la deglución están mal
sincronizadas y que la presión en el esfínter esofágico inferior permanece elevada. El diagnóstico más
probable es _____ producido por niveles bajos de _____:
a. acalasia / óxido nítrico
323.Paciente de 2 años, llega a emergencia por haber ingerido una moneda con la que estaba jugando. El
lugar más probable donde puede haberse quedado suspendido este objeto es a nivel del estrechamiento
producido a nivel del:
a. músculo cricofaríngeo

324.En una apendicectomía, al realizar la incisión de McBurney en la fosa iliaca derecha, es necesario cortar
los siguientes músculos, de afuera hacia adentro:
a. Oblicuo externo - Oblicuo interno - Transverso
325. Un varón de 90 años que se encuentra postrado en cama, es referido del asilo para endoscopia por
dificultad para deglutir luego de tomar un medicamento para aliviar el dolor de la noche anterior. La
endoscopia revela que la píldora se alojó en el esófago y causó una reacción inflamatoria. Lo más
probable es que esto haya sido por la producción de múltiples ondas:
a. secundarias
326.Mujer de 23 años es diagnosticada de bulimia, al examen físico se observa ulceraciones en el segundo y
tercer dedo de la mano derecha. Esto se puede deber al uso continuo de estos dedos para inducir el
vómito, mediante la estimulación del par craneal:
a. IX

327.Varón de 52 años se presenta por diarrea persistente de seis semanas de duración. En la colonoscopia
se observa un pólipo a nivel del íleon distal. El patólogo informa que se trata de un tumor neuroendocrino,
probablemente originado por las células enterocromafines del intestino. La sustancia que más
probablemente esté produciendo este tumor es:
a. Serotonina

328.La fase oclusal de la masticación se realiza con la contracción de los músculos:


a. masetero y temporal

329.Al tomar su café en Starbucks, un estudiante de medicina sufre una quemadura de primer grado en el
tercio anterior de la superficie dorsal de la lengua. La información de dolor es transmitida por el nervio:
a. Lingual

330.Paciente es evaluado por faringitis aguda en consultorio externo. El médico de familia le solicita que abra
la boca y saque la lengua Para realizar la acción de sacar la lengua, es necesario que se contraiga el
músculo:
a. geniogloso

331.Paciente con síndrome de Sjogren, presenta “boca seca” (disminución de la producción de saliva) y
caries dental, asociada a la pérdida de la función de tampón de la saliva. Esta desmineralización del
diente puede comprometer a las prolongaciones citoplasmáticas ubicadas en los tubos huecos de la
estructura señalada con la letra:
a. B

332.Una mujer de 32 años acude a consulta por presentar disfagia de progresión lenta, reflujo
gastroesofágico y vómitos desde hace 3 meses de evolución progresiva. Se le realiza un estudio
radiológico con contraste en el que se observa estrechamiento del esfínter esofágico inferior (imagen).
Según sus conocimientos este paciente se beneficiaría con el uso de:
a. análogo de óxido nítrico

333. ¿Cuál de las glándulas salivales


1. Al examinar a un paciente, usted encuentra dolor localizado en fosa iliaca derecha y diagnostica apendicitis.
En este paciente, usted puede inferir:
El peritoneo parietal regional está afectado
2. La motilidad intestinal es estimulada principalmente por el:
Plexo de Auerbach
3. Durante el vómito, ¿el contenido gástrico tiene que pasar necesariamente por cuál estructura para llegar al
esófago? Marque la mejor respuesta:
Cardias
4. Respecto a la anatomía del estómago, marque lo correcto:
El fondo gástrico forma la curvatura mayor
5. Marque la respuesta incorrecta:
En todo el tubo digestivo, se observa dos capas de muscular propia: circular interna y longitudinal externa
6. Paciente se queja de dolor en hipocondrio derecho, pero superficialmente. El dermatoma relacionado es
(marque la mejor respuesta):
T9
7. Dentro de las funciones del abdomen, se encuentra la defecación y micción, en las cuales la presión intra
abdominal debe:
Aumentar
8. Un alumno de medicina decide hacerse un piercing en el ombligo. Al realizarle el procedimiento, sangra
ligeramente. Esta sangre proviene de la arteria (marque la mejor respuesta)
Epigástrica inferior
9. Señale la respuesta correcta:
El apéndice cecal sólo tiene serosa
10. Paciente mujer es traída a emergencia por sufrir una herida contuso penetrante por cuchillo realizada por
su esposo en un ataque de celos. Se observa herida en flanco izquierdo. Esta solución de continuidad ha
comprometido varios músculos de la pared abdominal, excepto:
Recto abdominal
11. Paciente con herida por proyectil por arma de fuego, con herida de ingreso en región paraumbilical. Entre
las estructuras que usted está seguro que debe haberse lesionado es:
Omento mayor
12. En la evaluación de una tomografía abdominal, el interno observa un aneurisma en una arteria que se dirige
al riñón derecho. Con seguridad se puede afirmar que está a nivel de la vértebra:
L1
13. Paciente joven es traído a emergencia con abdomen agudo quirúrgico debido a herida contuso penetrante
por verduguillo (alambre grueso con punta aguzada) recibida en una pelea después de un partido de fútbol.
Se observa herida en Hipocondrio Izquierdo. El órgano que debe estar sangrando y produciendo
hemoperitoneo es (marque la mejor respuesta):
Bazo
14. Marque el órgano que se encuentra más distal en el tubo digestivo.
Ciego
15. La peristalsis o peristaltismo hace referencia a:
Motilidad para movilizar el alimento de proximal a distal.
16. Marque la respuesta incorrecta:
En todo el tubo digestivo, se observa dos capas de muscular propia: circular interna y longitudinal externa
17. Paciente con vólvulo del colon sigmoides. La necrosis de ese segmento del colon se produce por una
alteración en la irrigación de la arteria: Mesentérica inferior
18. Marque el órgano que se considera retroperitoneal: Parte de la vía biliar
19. Un alumno de medicina decide hacerse un piercing en el ombligo. Al realizarle el procedimiento, sangra
ligeramente. Esta sangre proviene de la arteria (marque la mejor respuesta): Epigástrica inferior
20. Paciente tiene una úlcera sangrante en el segundo tercio del Yeyuno. La arteria de la cual proviene la sangre
arterial para dicha zona es la arteria: Mesentérica superior
21. Es inervado por aferentes somáticas: Peritoneo parietal
22. Paciente de 24 años con dolor abdominal tipo cólico intenso en mesogastrio. Según sus conocimientos de
macroestructura, el origen del dolor puede ser ….: Íleon
23. Paciente con hipoglucemia secundaria a un insulinoma (tumor neuroendócrino productor de insulina). El
órgano donde mayor probabilidad ha crecido este tumor es: retroperitoneal
24. Al evaluar una tomografía abdominal, el médico asistente le pide al interno de la UPC que encuentre la
imagen con el corte a nivel de L1. El interno sabiamente busca el …….. para ubicar la vértebra L1: Cuello del
páncreas
25. En la inspiración, la pared abdominal debe …. para ….. : relajarse disminuir presión intra torácica
26. Ligamento hepatogástrico une el ….. con el …… y forma la entrada al …… : Hígado Estómago Orificio omental
27. Al retirar completamente el mesenterio de un órgano, el mismo se vería afectado principalmente en su:
Irrigación
28. La estructura que fija órganos principalmente a la pared posterior abdominal se denomina: Mesenterio
29. Cuál de las siguientes estructuras no tiene vasos sanguíneos: Epitelio intestinal
30. Al iniciar la digestión, aumenta el consumo de oxígeno por la mucosa. Esto conlleva a una hipoxia local, lo
cual hace que se libere …., el cual produce vasodilatación: adenosina
31. Sustancia que inhibe la secreción y la motilidad del estómago prolongando el tiempo de digestión: péptido
insulinotrópico dependiente de la glucosa (GIP)
32. Marque lo correcto: La hernia fisiológica se produce en la sexta semana y es la salida temporal de asas
intestinales a través del colon umbilical
33. Marque la respuesta correcta en relación a la gastrina: Las células G son las productoras y se encuentran
principalmente en el antro gástrico.
34. El consumir caramelos indirectamente activa la vía: POMC/CART
35. ¿En qué capa se encuentra la alteración principal en el Hirschsprung o megacolon agangliónico?: Muscular
propia
36. Con respecto a las ondas lentas, marque la afirmación correcta: Son contracciones rítmicas espontáneas
37. El uso de Ranitidina bloquea el receptor H2 de la histamina en las células parietales. La histamina llega a
estas células por: Difusión
38. La triada sintomática: vómitos explosivos post-prandiales, movimientos peristálticos epigástricos visibles
de izquierda a derecha y nódulos palpable epigástrico subcostal derecho, pertenecen a: Estenosis congénita
hipertrófica del píloro
39. Durante una cirugía oncológica, ¿la extirpación de cuál de los siguientes órganos se vería comprometida
por la presencia de adventicia?: Recto
40. En cuanto a los reflejos gastrointestinales, un reflejo que estimula el tránsito intestinal es el reflejo:
Gastrocólico
41. El ligamento falciforme divide al hígado en dos lóbulos derecho e izquierdo. Embriológicamente deriva del:
Mesenterio ventral
42. La presencia de atresias y estenosis duodenales se deben básicamente a una: Falta de recanalización
43. Estudiante de medicina de 20 años, se ha amanecido estudiando para su examen de Sistema Digestivo. No
ha probado alimento desde la cena, por lo que se puede afirmar que la motilidad de esta persona está
siendo regulada por: Motilina
44. Paciente con disminución del apetito marcada asociada a cáncer terminal, para promover la ingesta de
alimentos se podría usar análogos de: Endorfinas
45. Las ondas lentas se producen por la apertura cíclica de canales de: Calcio
46. La forma más común de atresia esofágica contiene: Estenosis proximal del esófago más fístula
traqueoesofágica distal
47. Al deglutir un bolo alimenticio, es lógico suponer que al pasar por el esofago haya un mayor consumo de
oxigeno en la pared del tercio: Proximal
48. Paciente que come entera una pizza familiar de chorizo y queso. Es posible esperar que debido a la cantidad
de alimento ingerida, las ondas lentas hayan: sufrido ninguna alteración en su frecuencia.
49. La hernia fisiológica se produce dentro de: cordón umbilical
50. El crecimiento de un adenocarcinoma de páncreas compromete la pared gástrica por contigüidad ¿Que
parte del estómago se esperaría esté comprometido?: Pared posterior del antro
51. Estimula la producción de saliva: Vasodilatación periglandular
52. Durante la secreción de saliva, es de esperarse que las concentraciones de ….. y …… disminuyan al disminuir
el flujo: Sodio Bicarbonato
53. Con respecto a la secreción gástrica de HCI: a mayor secreción de HCI en el lumen gástrico, mayor pH en la
sangre venosa gástrica.
54. Respecto a las enfermedades del esófago, marque lo correcto: el diagnóstico diferencial de la acalasia es la
enfermedad de Chagas esofágica
55. Con respecto a las lesiones y enfermedades de la boca, marque lo correcto: la eritroplasia debe ser
biopsiada
56. Respecto a las glándulas salivales, marque lo incorrecto:
a. la glándula sublingual tiene forma de garfio
b. la glándula sublingual drena a través del conducto de wharton
c. la glándula parótida produce secreción serosa
d. la glandula parotida drena a través del conducto de Stenon
57. El omeprazol actúa sobre la membrana ….. de la célula ….. : apical/parietal
58. Durante el sueño, la concentración de bicarbonato en la saliva: Disminuye
59. Durante el ataque con gas sarín (bloqueador de la acetilcolinesterasa) en el metro de Tokio, en 1995, el
personal de salud noto que los pacientes afectados presentaban: Hipersalivación
60. La célula mucosa del cuello gástrico produce: Moco

Parcial 2020-01 (1)

1. Al evaluar la orofaringe de un paciente, el médico le solicita que abra la boca, saque la lengua y diga a. Al
hacer esta maniobra, nota que el paladar se desvía hacia la derecha, lo cual le hace sospechar que el
paciente sufre de una lesión del nervio craneal: X contralateral
2. Un bolo alimenticio grande y poco masticado se atasca en el esofago, esto ocasiona una sensación de
dolor que es transmitida por los nervios: esplácnicos
3. Para realizar el movimiento mecánico de abrir la boca, primero se necesita: fijar el hueso hioides
4. ¿Cual de las siguientes alternativas se define como la protrusión directa del contenido abdominal a la
cavidad amniótica por un defecto de la pared corporal?: Gastrosquisis
5. Un paciente requiere que se le coloque una sonda de alimentación directamente al estómago
(gastrostomía), el cirujano deberá hacer una incisión en la piel del abdomen ¿cuál de las siguientes raíces
nerviosas debe ser anestesiada para este procedimiento? T8
6. En un paciente de 43 años con tumor carcinoide de páncreas productor de gastrina (Síndrome de
Zollinger-Ellison) se puede esperar encontrar una potenciación del reflejo: gastrocólico
7. El mecanismo de la defecación incluye la participación de diversas estructuras ¿cuál de las siguientes
alternativas es correcta?: Puede ser mediado por un reflejo intrínseco
8. Cuando el contenido del estómago ingresa al duodeno, uno de los reflejos que inhiben el vaciamiento
gástrico es a través del: Sistema nervioso mientérico
9. Durante la masticación, gran parte del proceso masticatorio se debe a: El reflejo masticatorio
10. Las glándulas salivales tienen conductos para la excreción de la saliva, las glándulas …… drenan en las
carúnculas sublinguales. RPTA: Sublinguales
11. Los diferentes segmentos del tubo digestivo son susceptibles de reflejos y movimientos según su
contenido. Si colocamos mediante una sonda un bolo alimenticio directamente en el tercio medio del
esofago: se producira ondas secundarias
12. En una persona si enfermedad se espera que el tránsito intestinal se vea disminuido cuando se presenta el
reflejo: Doloroso
13. El divertículo de Meckel es una anomalía congénita que ocurre por la persistencia del conducto vitelino y
da origen a una estructura sacular, el cual se encuentra en el: borde antimesentérico
14. Si al intubar a un paciente, por error se ingresa el tubo endotraqueal en el esofago y se insufla el
manguito endotraqueal (globo TET), la dilatación de este manguito generará: múltiples ondas secundarias
15. El orificio omental, o hiato de Winslow, se encuentra limitado por el ligamento: hepatoduodenal
16. Paciente de 24 años acude a consulta externa por presentar una fístula oronasal (comunicación entre la
cavidad oral y la cavidad nasal). Esta fístula es una consecuencia tardía de la lesión de un vaso sanguíneo
por el antecedente de haber sido operado de paladar hendido en los primeros años de vida,
aparentemente en una campaña gratuita de corrección de paladar fisurado. ¿Cual de las arterias palatinas
podría haberse lesionado durante esa cirugía?: Mayor
17. Dentro de las anomalías congénitas se puede presentar un tejido pancreático accesorio ¿cuál es la
ubicación más común de este tejido?: Estómago
18. Una recién nacida es evaluada por el neonatólogo y evidencia que el canal anal está completamente
cerrado. Este problema se debe probablemente a una anomalía en el desarrollo de: la membrana cloacal
19. En la digestión de los alimentos, la hormona __________ se libera frente a la presencia de péptidos y
monoglicéridos, y tiene un efecto marcado en la disminución del vaciamiento gástrico →
colecistoquinina
20. Dentro de las anomalías congénitas se puede presentar un tejido pancreático accesorio ¿Cuál es la
ubicación más común de este tejido? → Estómago
21. Los catadores de vino tienen una habilidad increíble al momento de separar los sabores. Este aumento de
la sensibilidad gustativa debido a una mayor cantidad de papilas linguales y de corpúsculos gustativos se
conoce como: hipergeusia
22. En muchos países se usa el suplemento de flúor en el agua potable o los dentífricos, con el fin de hacer el
esmalte más resistente a la desmineralización inducida directamente por: el ácido
23. Durante el desarrollo de la región cloacal, una cuña de mesodermo ubicado entre el alantoides y el
intestino posterior vendrá a formar el: tabique urorrectal
24. En una persona sana, el momento adecuado para encontrar los mayores niveles de grelina en sangre
sería: antes de comer
25. El duodeno está constituido por el segmento terminal del intestino anterior y el segmento proximal del
intestino medio ¿Cuál de las siguientes alternativas describe mejor este lugar de unión entre los dos
intestinos?
Distal al origen de la yema hepática
26. En una persona sana, el uso de atropina producirá a nivel del estómago: Aumentará el pH del estómago
27. En una persona sana, el consumo de leche produce indirectamente → Inhibición del vaciamiento
gástrico
28. La motilidad del colon es importante y lenta comparada con la del intestino delgado. Los movimientos en
masa ocasionan la: distensión rectal
29. Con respecto a la motilidad gástrica, los potenciales de acción disminuyen en frecuencia por efecto de:
el péptido insulinotrópico dependiente de glucosa
30. La sensación del gusto depende de la presencia de papilas gustativas en la lengua, de las cuales, algunas
de ellas tienen un surco terminal por donde drenan unas glándulas salivales linguales (llamadas glándulas
de von Ebner). Esta descripción se refiere a las papilas: circunvaladas
31. Dentro de la estructura de los dientes, la parte del diente cubierta por esmalte y que se puede ver
mediante la inspección visual de la boca se denomina → corona clínica
32. En el conducto anal se encuentra la unión entre las regiones del endodermo y el ectodermo, esta unión se
evidencia al observar: la línea pectínea
33. La sensación del gusto depende de la presencia de papilas gustativas en la lengua, las cuales tienen
corpúsculos gustativos conteniendo células neuroepiteliales sensoriales. Estas células neuroepiteliales
pueden ser dañadas fácilmente, por suerte, su tiempo de recambio es de alrededor de: 10 días
34. En un varón de 47 años con sección medular a nivel de T6 debido a un accidente automovilístico, sus
terapeutas han desarrollado un mecanismo para distender el recto e iniciar el reflejo rectoesfinteriano, lo
cual producirá la contracción de: la pared del recto
35. Durante un experimento, se insufla rápidamente dos litros de agua en un globo colocado dentro
del estómago de un voluntario. ¿cuál de las siguientes situaciones del músculo liso será
consecuencia directa de este cambio de volumen en el estómago? → Despolarización
36. Paciente de 56 años con accidente cerebrovascular reciente. En la resonancia se observa daño de los
núcleos laterales del hipotálamo. Por este motivo es muy probable que el paciente sufra de: inanición
37. Paciente con insuficiencia mitral moderada a severa, con aumento de volumen de la aurícula izquierda,
esta condición tendrá como consecuencia a nivel del sistema digestivo: la disfagia a sólidos
38. El inicio de la fase faríngea de la deglución se debe a estímulos sensitivos que viajan por el nervio craneal:
V
39. El mesocolon transverso se origina en: la pared posterior del abdomen
40. El esofago en su microestructura tiene basicamente adventicia, a excepción de la región distal, donde
tiene serosa, específicamente a partir del nivel de: T10
41. En un paciente con arcadas, se debe considerar que durante la ocurrencia de dichas arcadas, debemos
encontrar contenido gástrico en: Tórax
42. La reabsorción de Sodio y Cloro en las glándulas salivales se da principalmente en el: Conducto estriado
43. Para que se puedan digerir las grasas, es preferible que primero sean emulsificadas. La hormona que
estimula la liberación de las sustancias emulsificadoras es: CCK
44. Paciente de 13 días de vida con vómitos explosivos a las 2 horas después de lactar. Al examen físico se
palpa la oliva pilórica ¿cual es el nervio cuyos filetes dan inervación eferente a la estructura afectada?:
Vago
45. La localización de la vesícula biliar con respecto al lóbulo cuadrado es: Lateral
46. La triada portal está constituida por el conducto biliar y la arteria hepática y una pequeña rama de la
vena: Porta
47. Al ingerir una cantidad de glucosa por vía oral, esta es interiorizada en las células del organismo más
rápido que si esa misma cantidad hubiese sido administrada por vía endovenosa. Este fenómeno sucede
gracias a una sustancia secretada por las células: K
48. Paciente de 62 años con vólvulo de intestino delgado e isquemia intestinal. ¿Qué estructura se utiliza
como punto de referencia para determinar la posición de la unión duodenoyeyunal? : Ligamento
suspensorio del duodeno (de Treitz)
49. En un paciente con Zollinger Ellison, usted esperaría encontrar: Esteatorrea
50. La motilidad intestinal es estimulada por: Colecistoquinina y gastrina
51. La onda peristática secundaria del esofago en la deglución, es producida por: Plexo mientérico esofágico
52. Paciente con parálisis bilateral del nervio hipogloso, el unico musculo de la lengua que conservará su
movimiento es el: palatogloso
53. ¿Cual de las siguientes sustancias tiene mayor concentración de la saliva comparado con su concentración
plasmática? Potasio
54. ¿Cual de las siguientes estimula la secreción enzimática exocrina del páncreas?: Colecistoquinina
55. Al comer un pollo a la brasa, con papas fritas y ensalada, la sustancia que estimula la liberación de HCI en
el estómago es: Bombesina
56. Marque lo correcto con respecto a Esófago de Barret: Se relaciona con reflujo gastroesofágico

ECU 1
Estudiante de medicina de la UPC de 21 años sufre de gastritis aguda ocasionada por comer en lugares poco
higiénicos. Suele consumir caramelos ( chupar ) mientras está en clase hasta la tarde. Toma gaseosas regularmente
(carbohidratos 46%, sodio 53%). También toma regular cantidad de leche (grasa 35%, lactosa 35%, proteínas 30%),
pues le calma un poco el dolor el ardor que siente por la gastritis. Incluso, cuando puede, se toma dos vasos de
agua fría para calmar las molestias. Ha decidido ir al médico para tratarse pues ya no soporta el dolor, el cual está
seguro que los síntomas se deben a una elevada producción de ácido clorhídrico en el estómago, y por ello le ha
recetado Ranitidina (antihistamínico), con lo que siente mejoría.

El uso de atropina en este paciente:


- Aumentará el pH del estómago

Entre las sustancias cerebrales que producen ansiedad está la serotonina, la cual también tiene acción:
- Anorexigénica

El consumo de dos vasos de agua seguidos agua generará indirectamente un aumento en la liberación de:
- Ácido clorhídrico

En este paciente con gastritis aguda debida a una alta producción de ácido clorhídrico, sería lógico esperar que el
píloro tenga un tono muscular:
- Aumentado

El consumo de leche produce directamente un aumento de los niveles séricos de la hormona:


- Colecistoquinina (CCK)

El consumo de leche produce directamente un aumento de los niveles séricos de la hormona:


- Gastrina

ECU 2
Niño de sexo masculino de 2 años de edad, sufre de estreñimiento desde el nacimiento (1 deposición cada 3-4
días). Madre menciona que le estimula la defecación con un termómetro rectal, y continuo uso de enemas y
laxantes. Desde hace 6 meses comienza con vómitos postprandiales. Los síntomas aumentan en frecuencia y
magnitud y están en relación con los episodios de estreñimiento. No refiere fiebre, tos, diarrea ni lesiones
cutáneas. Al examen físico presenta regular estado general, luce deshidratado. Abdomen distendido, blando,
depresible e indoloro. No se palpan masas abdominales. Se permeabiliza el canal anal con termómetro rectal,
encontrando cierta resistencia. Salida de material fecal mal oliente en regular cantidad. Exámenes de laboratorio:
hemograma normal. Signos inflamatorios de fase aguda negativos. Alcalosis metabólica leve en sangre venosa.
Radiografía con enema baritado muestra recto y colon sigmoides dilatados (megacolon). Biopsia profunda:
ausencia de células ganglionares en la muestra enviada. Se realiza cirugía correctiva.

El contenido fecal se detiene en la zona inmediatamente proximal a la zona donde hay una menor presencia de:
- Péptido intestinal vasoactivo

En cuanto a los reflejos gastrocólico y gastroduodenal en este paciente, indique lo correcto:


- Se pueden considerar reflejos vago-vagales

En este paciente se considera que está abolido el reflejo:


- Rectoesfinteriano

Debido al acúmulo de material fecal en todo el marco colónico, y a la irritación química asociada, el peristaltismo
del íleon distal se debe encontrar:
- Inhibido

Es un reflejo propio de la pared intestinal:


- Peristaltismo

A diferencia de las arcadas, los vómitos presentan apertura de:


- Esfínter esofágico superior

1. Un niño de 2 años es llevado a la consulta por diarrea persistente y edema de las


extremidades, además falta de crecimiento y desarrollo en relación a su edad. Los
análisis de sangre revelan que tiene concentración plasmática baja de proteínas
(hipoproteinemia). Durante la endoscopía duodenal, se coloca colecistocinina (CCK)
endovenosa y se recoge muestras del líquido duodenal; el resultado del líquido
confirma incapacidad para hidrolizar proteínas a un pH neutro, esta situación mejora
al añadir una pequeña cantidad de tripsina. El paciente probablemente esté sufriendo
la falta congénita de
-Enterocinasa
2. Experimentalmente se incrementa la velocidad de la secreción salival con una
sustancia, el análisis de la composición de esta saliva obtenida se espera
encontrar…………..
-Disminución de concentración de potasio
3. Paciente varón de 46 años soltero, consulta por odinofagia y bajo de peso, tiene
antecedente de tuberculosis desde hace 3 meses y es fumador crónico (10 cigarrillos
por día); al evaluar la cavidad oral se identifica lesión blanquecina en el dorso de la
lengua y paladar blando, las lesiones se desprenden con el baja lengua dejando una
base eritematosa. Esta lesión corresponde probablemente a
……………………….…..
-Candidiasis oral
4. Paciente mujer de 35 años acude a consulta por sensación de sequedad y lesiones
en cavidad oral. Al examen se observa atrofia de la mucosa, fisuras y úlceras; nota
además sequedad e irritación de la córnea y aumento del tamaño de las glándulas
parotídeas. Su diagnóstico más probable es artritis reumatoide; el hallazgo más
probable en una biopsia de glándula parótida es……..….
-Gran infiltración de linfocitos y células plasmáticas
5. Un paciente con anemia acude con su médico quejándose de episodios frecuentes
de gastroenteritis. Un análisis de sangre revela anticuerpos circulantes dirigidos
contra células parietales gástricas. Su anemia es atribuible a la hiposecreción de
-Factor intrínseco
6. Dos estudiantes deciden tomar un receso para comer una hamburguesa a la hora
del almuerzo. Antes de llegar a la cafetería, impulsos nerviosos provenientes del
complejo vagal dorsal iniciarán la secreción de ácido gástrico por la liberación de
…………………….. desde el sistema nervioso entérico.
-GRP
7. Un niño de cuatro años de edad es llevado a la consulta por cuadros diarreicos
frecuentes caracterizados por heces pálidas, voluminosas y fétidas, presenta bajo
peso y talla. Se mide la concentración de cloruro en el sudor y se encuentra que sus
valores son muy elevados. La alteración más importante a nivel de células ductales
del páncreas tiene relación directa con la conductancia de…………
-Cloro
8. Una mujer de 50 años de edad que sufrió durante varios años resequedad de los
ojos debida a producción inadecuada de lágrimas es enviada con un
gastroenterólogo para evaluación de pirosis crónica. El examen endoscópico
revela erosiones y tejido cicatrizal en la parte distal del esófago justo por arriba del
esfínter esofágico inferior. Las lesiones pueden atribuirse a la disminución de uno
de los siguientes componentes salivales:
-Bicarbonato

9. Se evalúa los valores séricos de las siguientes sustancias a un paciente con


enfermedad hepática terminal; en este paciente se espera encontrar la combinación
con la letra …………
-disminuida, aumentada, disminuida
10. Una mujer de 35 años de edad HIV positiva, se presenta al médico con dolor
abdominal en cuadrante superior derecho e ictericia. La paciente refiere haber tenido
múltiples episodios de ictericia durante los últimos 10 años. Los exámenes para
determinar hepatitis viral, dieron positivos para Hepatitis B, siendo catalogado el caso
como hepatitis crónica con alteración funcional. En un examen de sangre ¿cuál de los
siguientes parámetros está disminuido?
-Albúmina
11. En el reflejo peristáltico del intestino delgado, uno de los siguientes eventos sucede
en la porción oral del bolo alimenticio…………...
-Acción de acetilcolina en el músculo circular
12. Experimentalmente se coloca una dosis alta de secretina en la luz intestinal
duodenal; como consecuencia de esto, en el jugo pancreático de la misma luz
intestinal se observa la disminución de la concentración de …..………..
-Cl
13. Un varón de 58 años de edad con enfermedad de Crohn severo fue sometido a una
resección ileal. Después de la cirugía este paciente padecerá de esteatorrea, esto se
explica porque …..………..
- La micelas no pueden formarse
14. En un experimento se inserta un balón en el estómago de un voluntario, se infla poco a
poco mientras que se vigilan las presiones intraluminales. Aunque el volumen del
balón aumenta considerablemente, las presiones permanecen constantes. Esta
relación volumen-presión se explica por la liberación local de …………..
-Óxido nítrico y péptido inhibidor vasoactivo
15. La toxina de Vibrio cholerae causa diarrea debido a…….
-El Incremento de la secreción de cloro por las células de la cripta intestinal
16. ¿Cuál de las siguientes alternativas es una característica de la secreción exocrina
del páncreas?
-Tiene una baja concentración de Cl- respecto al plasma
17. Una madre lleva a su hijo de dos años de edad a la sala de urgencias, estresada
porque el niño deglutió una moneda de 10 céntimos mientras la familia cenaba en un
restaurante. El médico observa mediante fluoroscopía que la moneda se halla en el
estómago y asegura a la madre que la moneda se eliminará con las heces. El médico
recomienda utilizar la respuesta fisiológica que permitirá la evacuación de la moneda
del estómago al intestino ………….…..
-Son los movimientos de mezcla y trituración
-. Es provocada por el ayuno
18. Las estructuras en el hígado que permite que los productos metabólicos unidos a
proteínas tengan acceso a las membranas basolaterales de los hepatocitos, son….. -
Las fenestras sinusoidales
19. La composición de la bilis es modificada conforme fluye por los conductillos biliares.
Durante este tránsito se espera que aumente la concentración de…….
-Monómeros de ácido biliar
-Ig A
20. Se mide experimentalmente el contenido gástrico de dos personas. La persona “A”
tiene alto contenido de grasa y la persona “B” tiene un contenido hipertónico ¿Cuál de
las siguientes es correcto respecto al vaciamiento gástrico?
- Hay ralentización del vaciado gástrico en ambos casos
21. El examen endoscópico de un paciente con hipertensión portal grave revela venas
tortuosas que sobresalen hacia la luz del esófago. El paciente recibe tratamiento
quirúrgico mediante la colocación de una derivación que conecta la vena porta a la
vena cava. Después de la operación el riesgo de encefalopatía y el
riesgo de sangrado de várices ……………..
-Aumentará/disminuirá
22. Un paciente varón de 18 años de edad acude al médico para sus exámenes de
rutina. Sus resultados de laboratorio muestran un valor de bilirrubina sérica de 4
mg/dl y una bilirrubina directa de 0,3 mg/dl. Las pruebas de función hepática son
normales. La alteración que explica mejor este caso es por la deficiencia de
………………..
-Glucuronil transferasa
23. Un hombre de 57 años de edad es llevado a urgencias con hematemesis masiva
rojo brillante, a su llegada se halla inconciente con PA: 80/40 mm Hg y FC: 124
lat/min. Luce ictérico con presencia de “arañas vasculares en el tórax anterior y
extremidades”, abdomen distendido con signo de oleada positiva. Se encuentra
esplenomegalia y pérdida de la masa muscular en extremidades. La anastomosis
vascular responsable del sangrado en este paciente es
-Vena gástrica izquierda y vena ácigos
24. Un estudiante de medicina está comiendo un plato de comida a base de
champiñones, espárrago y salsa de soya. El sabor umami contenido en todos estos
alimentos actúa a nivel de los botones gustativos estimulando ………………..
-Un receptor acoplado a proteína G
25. Un hombre de 22 años de edad se presenta al médico con una historia de 1 año de
evolución caracterizado por dolor recurrente en fosa iliaca derecha y diarrea.
Manifiesta además pérdida de peso de 8 kg durante este periodo. La colonoscopía
revela múltiples lesiones en el ileon terminal y colon. La biopsia de estas lesiones
revela engrosamiento, inflamación y ulceración de la mucosa. El diagnóstico más
probable en este caso es…….
-Enfermedad de Crohn
26. Varón de 61 años que consulta por dolor retro esternal intenso desde hace 6 horas y
después de vómitos intensos y repetidos; al examen se observa disnea, cianosis,
hipotensión y signos clínicos de shock. La radiografía simple de tórax muestra
neumomediastino. El líquido en el espacio pleural aspirado tiene alta concentración de
amilasa. ¿Cuál de las siguientes alternativas puede explicar este cuadro clínico? -
Rotura espontánea de esófago
27. La secreción del ácido en la célula parietal gástrica se lleva a cabo por una ATPasa
especifica que intercambia hidrogeniones (H+) del citosol por…..
-K +
28. En condiciones normales el ingreso de 600 ml de líquido es el estómago provoca un
aumento de presión intragástrica de unos 12 cm de H2O. Después de una vagotomía
(corte del nervio vago) es de esperar que el ingreso del mismo volumen de líquido
provoque lo siguiente: …………………………………
-Un aumento mayor de la presión
29. Una paciente de 30 años de edad es sometida a una cirugía en oído medio derecho
por un problema de otoesclerosis. Luego de la cirugía refiere alteración en la
percepción de sabores. Al evaluar el caso usted esperaría encontrar……….
-Alteración en la sensación del gusto en los dos tercios anteriores de la
lengua
-Sensación del dolor, tacto y temperatura conservada en toda la lengua

30. ¿Cuál de las siguientes alternativas es correcta?


-Las sales biliares desconjugadas son absorbidas preferentemente en el colon
31. En un paciente de 45 años de edad con colestasis biliar, se encuentra una elevación
de los niveles sanguíneos de fosfatasa alcalina hasta 3 veces la cifra normal. ¿Cuál de
las siguientes alternativas estará también elevada como evidencia del daño de la vía
biliar?
-Gamma glutamil transpeptidasa
32. Revisando la angiografía de un hombre de 70 años en estudio por aneurisma de aorta
abdominal el radiólogo informa de la presencia de una oclusión completa de la arteria
mesentérica inferior. El paciente se encuentra completamente asintomático.
¿Cuál de las siguientes arterias se anastomosa a la sistema arterial de la
mesentérica inferior?
-Cólica media
33. Lactante de 3 meses de vida es atendido por presentar diarrea, se administra una
solución de glucosa y electrólitos por vía oral. La proteína de membrana apical que
explica la capacidad de esta solución para proporcionar aporte de glucosa e
hidratación es ………..
-SGLT-1
34. Paciente ha sufrido herida de bala en el abdomen, se le ha tenido que extirpar el
segmento medio y distal del ileon. En este caso la síntesis hepática de sales biliares
estará …..…..
-Incrementada por estímulo de la enzima colesterol 7 alfa hidroxilasa
35. Un varón de 75 años ingresa al consultorio por presentar ictericia marcada de piel y
las escleras. El estudio del paciente mostró que presentaba un tumor que obstruía la
totalidad del conducto hepático común. ¿Cuál de las siguientes estructuras se
encontrará dilatada en este paciente?
-Conductos de Hering
36. En un paciente con insuficiencia renal crónica, el déficit en la absorción de calcio a
nivel del enterocito se debe a lo siguiente:
-No se convierte la 25 hidroxicolecalciferol a 1,25 dihidroxicolecalciferol
37. Varón de 30 años es traído a emergencia por agresión abdominal con arma de fuego
(pistola) y es sometido a laparotomía exploratoria, observándose isquemia del colon
ascendente y parte del colon trasverso ¿la lesión de cuál de las siguientes arterias
explicaría esta isquemia?
d. Mesentérica superior
38. Respecto a las sustancias gastrointestinales que regulan la secreción pancreática;
marque la afirmación correcta:
b. La acetilcolina es capaz de estimular la secreción enzimática y de bicarbonato del
páncreas
39. Ante una lesión del X par craneal, ¿cuál de los siguientes músculos mantiene
conservada su función?:
b. Tensor del velo del paladar
40. Experimentalmente se utiliza atropina (anticolinérgico) para inhibir la secreción de
gastrina, sin embargo la secreción de esta hormona se sigue dando ante estímulos
vagales. Esta situación se explica porque la atropina:
d. No bloquea la acción del péptido GRP

41. Un varón de 50 años es sometido a extirpación del duodeno y parte proximal del
yeyuno. Esta situación ocasionaría la pérdida de las células ……….. , productoras de
………………… que estimula la secreción de bicarbonato por el páncreas.
“S” / secretina
42. Se evalúa la expresión de la proteína Agrp en una persona con alteración del apetito; lo
correcto respecto a esta proteína es…..
La mutación del gen que la codifica produce adelgazamiento
43. Juana cae de la bicicleta y se fractura la región anterior del hueso maxilar superior con
compromiso de la fosa incisiva. Al examen físico de la región esperaría encontrar
alteración en la sensibilidad de la encía …………………
palatina anterior
44. Recién nacido es atendido por el neonatólogo y luego entregado a su madre para dar
de lactar; la madre al dar de lactar observa coloración azulada de labios, acompañado
de tos persistente, dificultad respiratoria y distención abdominal. Se le intenta colocar
una sonda nasogástrica pero esta retorna a la cavidad oral en todos los intentos. ¿Cuál
de las siguientes anomalías del desarrollo es el más probable en este caso?
b. Atresia esofágica proximal con fístula traqueoesofágica distal l
45. ¿Cuál de los siguientes mecanismos ocurre durante la defecación?
En la posición de “cuclillas” el músculo puborrectal se halla relajado
46. Un paciente luego de un accidente sufre lesión del piso de la boca, se constata daño
del nervio “cuerda del tímpano”, en este caso se esperaría en
47. contrar disminución de la………………………….… de la lengua
Sensación del gusto en los dos tercios anteriores
48. ¿Cuál de las siguientes afirmaciones es la correcta sobre la gastrina?
Actúa en la célula diana mediante su receptor CCk tipo B
49. Al recibir un paciente con signos de hipovolemia y antecedente de trauma en
abdomen por accidente de tránsito, usted identifica radiológicamente: lesión de
primera vértebra lumbar y signos de lesión en páncreas; durante la cirugía se observó
pobre irrigación de asas intestinales. El vaso afectado es la arteria ……..
c. mesentérica inferior
50. Un paciente sufre de daño a nivel del cuello con lesión muscular en la región de la
faringe. En el examen físico se determina dificultad para la elevación de la faringe y para
el cierre del itsmo de las fauces. En este caso, probablemente esté afectado el músculo:
c. palatofaringeo
51. Varón de 50 años a quien le realizan la curación de la segunda molar de la arcada
superior derecha. En un momento determinado, el paciente acusa de intenso dolor
de la pieza dentaria en tratamiento. La vía aferente del dolor viaja a través del
nervio …………
a. trigémino V2
52. La distención gástrica por los alimentos produce incremento de secreción de HCl
mediante la producción de ………….. que estimula a las célulasvía proteína
………..
a. gastrina / parietal / Gq

53. Un niño de tres años llega a emergencia con disfagia (dificultad para tragar), dolor
retro esternal, salivación y llanto. Se sospecha de ingesta de cuerpo extraño (moneda)
en el esófago; al ser evaluado se constata en una radiografía presencia de cuerpo
extraño a nivel de C6 (6° vértebra cervical). El cuerpo extraño estará suspendido a
nivel del estrechamiento producido por………..
c. el músculo cricofaríngeo
54. La triada portal (arteria hepática, vena portal y conducto biliar común) está contenida
en el ligamento …….……… y derivan embriológicamente del ……
a. hepato duodenal / mesenterio ventral
55. Un paciente refiere no percibir algunos sabores, al examen físico se constata
alteración en la percepción de sabores y del dolor en el tercio posterior de la lengua
¿Cuál de los siguientes nervios estará alterada en su función?
c. Glosofaríngeo (IX par)
56. En el caso de un paciente con gastrinoma (tumor productor de gastrina), la presencia
de úlceras duodenales y erosión de la mucosa gástrica, se debe principalmente a…….
c. el exceso de HCl por estímulo de receptores CCK-B en la célula parietal
56. El reflejo entero gástrico se caracterizan por:
d. originarse debido a la distensión duodenal y presencia del quimo ácido
57. Mauricio tiene dificultad para deprimir el paladar y elevar la parte posterior de la lengua.
En este caso estará afectado un músculo, específicamente el músculo
…………….
b. extrínseco – palatogloso
58. En condiciones normales, el ingreso de 600 ml de líquido es el estómago provoca un
aumento de presión intragástrica de unos 12 cm de H2O. Después de una vagotomía (corte
del nervio vago) es de esperar que el ingreso del mismo volumen de líquido ocasione
………………………………… de la presión
intragástrica.
c. un aumento mayor
59. La explicación fisiológica de presentar somnolencia de 30 minutos a 1 hora después
de ingerir alimentos, se explica por: a. Aumento del cloro intraluminal
e. Aumento de la alcalinidad sanguínea
60.Se presenta un paciente, el cual presenta un antecedente de tuberculosis intestinal, por
lo cual, se le resecó 80 cm de íleon distal. Desde el punto de vista fisiológico, el paciente
puede presentar una de las siguientes alteraciones: a. Disminución de la secreción de
Vitamina B12

e. Disminución de la absorción de ácido glicocólico


61. Un paciente es sometido experimentalmente a un fármaco que modifica el flujo salival,
obteniéndose un volumen de saliva de 288 ml en 6 horas. En este caso las concentraciones
de electrolitos y bicarbonato en la saliva obtenida varían de la siguiente manera: a. ↑ Na+,
↓ K+, ↑ Cl-, ↑ HCO3-

1. b. ↓ Na+, ↓ Cl-, ↑ K+, ↓ HCO3-

62. Uno de los siguientes elementos debería hallarse con más probabilidad en el esófago de
un paciente que sufre de reflujo gastro esofágico…
a. Pepsina

63. Un paciente de 40 años cursa con anemia de 8g/dl, aqueja además de astenia y sensación
de hormigueo bilateral en los miembros inferiores, al examen se halla alteración de la
sensibilidad a la vibración y camina con ampliación de la base de sustentación. Uno de los
siguientes procedimientos sería de ayuda para el diagnóstico de este paciente:
a. Tomografía cerebral
b. Biopsia de la mucosa gástrica

64.Paciente de 60 años ingresa por caída hace 1 hora y pequeño hematoma en cuero
cabelludo, al examen físico ampliado se observa ictericia de piel y mucosas generalizada,
abdomen blando, se palpa estructura quística no dolorosa en hipocondrio derecho que
corresponde a vesícula biliar (signo de Courvoisier), en los exámenes de laboratorio se halla
niveles bajos en la formación de estercobilinógeno y urobilinógeno en heces, incremento de la
bilirrubina conjugada en la orina, elevación de fosfatasa alcalina y gamma glutamil
transpeptidasa séricas. El presente cuadro puede ser explicado por: a. Reabsorción de
hematoma
c.Carcinoma de la cabeza de páncreas
65. Un recién nacido presenta vómitos biliosos poco tiempo después de cada alimento. Al
preguntar a la madre sobre antecedentes, ella recuerda que tuvo polihidramnios durante la
gestación, pero un análisis de cariotipo fue normal. Una de las siguientes es la causa más
probable de estos hallazgos en el recién nacido: a. Enfermedad de Hirschprung
e. Malrotación de la yema pancreática ventral
66.En un estudio de la secreción de hormonas gastrointestinales, sus concentraciones en la
vena porta se midieron durante perfusión luminal del intestino delgado con soluciones de
diversas magnitudes de pH. ¿Qué hormona aumentará en el plasma de la vena porta durante
perfusión a través del intestino con una solución de pH 3?
a. CCK
e. secretina
67.Paciente de 30 años que ingresa a causa de un traumatismo abdominal cerrado. En la
exploración se aprecia discreta palidez de piel y mucosas, auscultación pulmonar normal,
taquicardia de 120 /min. Discreta distensión abdominal y matidez en flancos; el hematocrito,
que era prácticamente normal al ingreso, disminuye a 30% a las tres horas. En la Rx de tórax
se objetiva fractura de las costillas 10-11 izquierdas. La causa más probable de la anemización
en este paciente es: a. traumatismo renal con hemorragia retroperitoneal.
c. rotura de bazo con hemoperitoneo.
68. Revisando la angiotomografía de un hombre de 70 años en estudio por aneurisma de
aorta abdominal, el radiólogo le informa de la presencia de una oclusión completa de la
arteria mesentérica inferior. El paciente se encuentra completamente asintomático. La
oclusión de la arteria mesentérica inferior cursa de manera asintomática en muchas
ocasiones ya que el territorio que irriga puede recibir flujo proveniente de la arteria:
a. cólica derecha
e. cólica media

69. En las patologías de esófago es importante conocer bien la anatomía esofágica. ¿Cuál de
las siguientes afirmaciones es correcta? a. El esófago tiene capa mucosa, muscular y serosa

c. El esófago torácico pasa por detrás del cayado aórtico

70. A pesar de que pueda haber variaciones anatómicas, lo habitual es que el ciego sea
irrigado por una rama arterial que proviene de unas de las siguientes arterias: a. Iliaca
derecha

d. Mesentérica superior

71. Ante un paciente con una cirugía abdominal urgente, el informe operatorio señala que se
ha realizado una resección de todo el duodeno y del tercio proximal del yeyuno manteniendo
íntegros el estómago y todo el íleon, así como los dos tercios distales del yeyuno. En el
seguimiento nutricional del paciente ¿Qué vitamina o mineral presentará con menor
probabilidad una disminución de su absorción?
a. Cianocobalamina

72. ¿Cuál de las siguientes sustancias forma parte de la secreción biliar? a. Tripsina
Lecitina

73.¿De qué musculo forma parte el ligamento inguinal?


-Oblicuo externo del abdomen
74.¿Cuál de las siguientes enzimas está localizada en el borde en cepillo y juega un rol en la
digestión de proteínas?
e. Carboxipeptidasa A.
75. Una de los siguientes sustancias, NO sirve como un buen agente emulsificante:
a. Colesterol
76. La sustancia que estimula el crecimiento de la mucosa gástrica es:

a. Secretina

d. Gastrina

77.¿Cuál de las siguientes alternativas es una función de la colecistokinina?

a. Relajación de la vesícula para la salida de bilis

d. Secreción de enzimas pancreáticas

78.Con respecto a la anatomía del tronco celiaco, señale lo correcto a. El tronco celiaco se
origina de la cara posterior de la aorta abdominal
d. La hepática común que es una de sus ramas, participa en la irrigación del
estómago.
79. Con respecto a la anatomía del duodeno, marque la respuesta correcta: a. Tiene una
distribución en forma de “C”, que rodea la cola del páncreas
b. La 3ra porción duodenal está contenida en la pinza vascular aortomesentérica
80. En el íleon se absorbe aproximadamente el 95% dea través de la circulación
enterohepática.
a. agua
c. sales biliares
81. La estimula el mecanismo paracrino de la secreción de ácido clorhídrico.
a. histamina
82.En la digestión de proteinas,es el principal estímulo para convertir el
pepsinógeno en pepsina. a. la gastrina
b. el pH ácido
83. Con respecto a la somatostatina, marque lo correcto:
a. Es secretada por las células S del intestino
Interviene en la fase intestinal de la secreción gástrica
84. En pecten anal, es una estructura comprendida entre: a. la línea pectínea y los senos
anales
d. la línea anocutánea y la línea pectínea

85.¿Cuál de las siguientes alternativas es una proenzima pancreática? a. Tripsina

1. b. Elastasa
2. c. Quimotripsinógeno
3. d. Amilasa
4. e. Procarboxipeptidasa C
86. En este paciente, el bloqueo farmacológico de los receptores H2 en la mucosa gástrica:
a) No tiene efecto sobre la secreción de ácido inducida por el vago
b) Evita la activación de adenilciclasa por gastrina
c) Inhibe la secreción de ácido inducida por gastrina y mediada por el vago
d) Causa un aumento en el transporte de potasio por las células parietales gástricas
Se validó la A :)
87. Si se considerara una gastrectomía total para curar la gastritis del paciente, cuál de las
siguientes sustancias ya no se produciría:
a) Gastrina
b) Quimiotripsina
c) Amilasa
d) Pepsinógeno

88. Un paciente hipertenso está tomando un medicamento bloqueador de receptores alfa 1


adrenérgicos (prazosina) y como efecto secundario se queja de:
d) Lo escaso que es el medicamento
e) No tiene problemas en la salivación
c) Hiposalivación
d) Hipersalivación

89. Con respecto a las lesiones y enfermedades de la boca, marque lo correcto:


f) La leucoplasia se desprende al roce
b) la eritroplasia puede degenerar en adenocarcinoma
c) El muguet oral es una enfermedad bacteriana en inmunodeprimidos
d) la eritroplasia debe ser biopsiada
NOTA: fue validada la opción B ya que no es motivo del curso que sepamos el puto cáncer.

90. En este paciente, se puede asumir que la pancreatitis ha sido ocasionada por una
disminución en el efecto de:
g) Amilasa
h) Lipasa
c) Inhibidor de la tripsina
d) Entercinasa
91.Un efecto secundario en el estómago por la acción de la secretina es:
i) Disminución en la liberación de pepsinógeno
b) Menor actividad de la pepsina
c) Mayor paso de bicarbonato a sangre periférica
d) Aumento en la producción de factor intrínseco
92. Dentro de los factores protectores de la mucosa gástrica se pueden mencionar múltiples
protagonistas. Uno de ellos es:
j) CCK
k) Gastrina
c) Receptor muscarínico
d) Pepsina
93. La saliva puede tener una variedad de electrolitos en su composición. Entre ellos el cloro,
respecto al cual se puede afirmar:
l) Su mayor concentración se consigue con el flujo bajo
b) Su concentración no llega a ser tan alta como en el plasma
c) Con flujo alto su concentración es mayor que la del plasma
d) Su menor concentración se alcanza con flujo alto
94. En el síndrome de boca seca o síndrome de Sjogren, una de las complicaciones
asociadas es:
a) caries
b) Disminución de la acidez gástrica
c) Aumento en de la producción de saliva
d) Aumento del pH bucal
95. Estimula la producción de saliva:
a) Vasodilatación periglandular
b) Atropina
c) Fatiga o cansancio
d) Expresión de miedo
96. El omeprazol actúa sobre la membrana de la célula
m) Basolateral/principal
n) apical/principal
o) Basolateral/parietal
d) apical/parietal
97. Para protegerse del entorno ácido, el Helicobacter pylori se autogenera un entorno de
pH menos ácido alrededor suyo, gracias a una enzima que alcaliniza su entorno local
mediante la conversión de:
a) urea en NH3
b) H2O y CO2 en ácido carbónico
c) NH3 en urea
d) H2CO3 en bicarbonato
98. La anemia perniciosa destruye las células:
p) mucosas del cuello
b) oxínticas
c) principales
d) mucosas superficiales
99. La célula mucosa del cuello gástrico produce:
a) Moco
b) ácido clorhídrico
c) pepsinógeno
d) Factor intrínseco
100.El aumento en la acidez del estómago producido principalmente por la infección de
Helicobacter pylori se debe a la disminución de:
a) Somatostatina
b) Bicarbonato por las glándulas de Brunner
c) Secretina
d) Colecistoquinina
101. De las siguientes sustancias secretadas por los órganos de este paciente, la más
alcalina es la secreción:
a) pancreática
b) Esofágica
c) Yeyunal
d) Salival
102.En cuanto a la gastritis de este paciente, se encontró que era producida por la bacteria
Helicobacter pylori. Esta bacteria sobrevive en el medio ácido del estómago gracias a:
a) ácido clorhídrico
b) Toxina CagA
c) Ureasa
d) Jugo pancreático
103.La lengua está recubierta por epitelio:
c) pseudoestratificado columnar no queratinizado
b) plano estratificado no queratinizado
c) pseudoestratificado columnar ciliado
d) plano estratificado queratinizado
104.El esfínter anal interno tiene musculatura …….. y tiene control ……..
d) lisa / voluntario
b) lisa / involuntario
c) esquelética / simpático
d) esquelética / parasimpático

19) La arteria Aorta proporciona la irrigación al tubo digestivo ¿cuál de las siguientes
arterias proporciona la irrigación al ángulo cólico derecho?
a) mesentérica superior
b) mesentérica inferior
c) frénica inferior
d) tronco celiaco

20) Paciente de 26 años que le cuenta en su historia clínica que cada vez que almuerza a los
20 min tiene deseo de defecar, le comenta que su hijo de 1 mes le pasa lo mismo pero más
intenso. Esto se explica por el reflejo …….., el cual está …… en el paciente
a) colicoileal / normal
b) colicoileal / alterado
c) gastrocólico / normal
d) gastrocólico / alterado

21) La región del estómago que se comunica con el duodeno es la


a) pilórica
b) cardias
c) cuerpo
d) fórnix

22) Acude a consulta un px que fue diagnosticado de úlcera péptica 3 días antes. Luego de
múltiples pruebas diagnósticas se concluye que el paciente presenta un tumor secretor de
gastrina, ¿cual de las siguientes situaciones estará incrementada?
a) distensión gástrica
b) inhibición del vaciado gástrico
c) secrecion de acido clorhidrico
d) inhibición de la secreción de pepsinógeno
23) En el sistema digestivo, el control del apetito está dado por un complejo sistema de
sustancias y órganos integradores, los cuales regulan la ingesta de alimentos. La …… es
una sustancia orexígena y es sintetizada por el ……
a) leptina / estómago
b) felina / intestino
c) leptina / estómago
d) grelina / estómago

24) Con respecto a la actividad eléctrica del sistema digestivo, marque la alternativa
correcta
a) corresponden a potenciales de acción que están presentes de forma continua y le
dan capacidad de peristalsis autónoma al sistema digestivo
b) la frecuencia de las ondas lentas NO se ve influenciada por la actividad neural ni las
hormonas gastrointestinales
c) en el estómago las ondas lentas se dan en una frecuencia de 6 x min
d) las ondas lentas son cambios lentos y ondulantes del potencial en reposo
e) la frecuencia de las ondas lentas va de 6 a 12 ondas por minuto

25) Ante una lesión del IX pc, el músculo …… se altera en su función


a) palatogloso
b) estilofaríngeo
c) palatofaríngeo
d) constrictor superior

26) Un varón de 50 años es sometido a extirpación del duodeno y parte proximal del yeyuno.
La pérdida de estímulo hormonal en el páncreas para la secreción enzimática se explica
por la pérdida de células
a) parietales, productoras de factor intrínseco
b) K productoras de factor intrínseco
c) M productora de CCK
d) I productora de CCK

27) Marque la respuesta correcta:


A. El bronquio derecho constituye una de las estrecheces del esófago
B. Todos los órganos del sistema digestivo tienen capa serosa
C. La pared gástrica en el fondo es más delgada que en el cuerpo y antro
D. El esfínter de Oddi rodea a la papila menor duodenal

28) Marque la respuesta correcta en relación a la gastrina:


A. Al distenderse el estómago, se inhibe su producción.
B. Se estimula por la liberación de noradrenalina
C. Las células G son las productoras y se encuentran principalmente en el antro
gástrico
D. Las células G se encuentran principalmente en el fondo gástrico

29) Para poder morder una manzana, es necesario usar el siguiente músculo:
A. Milohiodeo
B. Tensor del paladar
C. Orbicular de los labios
D. Buccinador
30) Sustancia que inhibe la secreción y la motilidad del estómago prolongando el tiempo de
digestión:
A. Enteroglucagon.
B. Polipéptido pancreático
C. Péptido 1 similar al glucagón (GLP-1).
D. Péptido insulinotrópico dependiente de la glucosa (GIP).

31) El nacimiento de la arteria mesentérica superior se puede encontrar en cuál de los


cuadrantes abdominales:
A. Hipocondrio derecho
B. Hipogastrio
C. Epigastrio
D. Mesogastrio
32) Entre las múltiples causas de la Enfermedad por Reflujo Gastroesofágico, se puede
considerar también a una alteración en las del esfínter esofágico inferior:
A. Ondas secundarias
B. Contracciones tónicas
C. Ondas lentas
D. Glándulas subesofágicas

33) Producto de la alimentación, se producen diversas sustancias peptídicas, cininas y


bradicininas, las cuales permiten que:
A. Se produzca neovascularización en los territorios de las arterias abdominales
B. La acción de la lipasa pancreática se vea incrementada
C. El consumo de O2 del intestino aumente ligeramente
D. El flujo sanguíneo intestinal aumente hasta 8 veces

34) El dolor periumbilical o epigástrico en el inicio de una apendicitis aguda se debe a:


A. Estímulo del nervio vago.
B. Íleo secundario.
C. Irritación del peritoneo parietal.
D. Estímulo del sistema simpático.

35) El aumento en la actividad motora de la pared gástrica genera un aumento en los


niveles locales de qué sustancia en la microvasculatura:
A.Adenosina
B. Colecistoquinina CCK
C. Endotelina
D. Gastrina

36) ¿Cuál de los siguientes péptidos inhibe el vaciamiento gástrico?


A. Colecistoquinina
B. Péptido inhibidor gástrico
C. Motilina
D. Gastrina
37) Los músculos de la masticación que producen la retropulsión de la mandíbula son:
A. temporales [mas seguro]
B. maseteros
C. milohioideos
D. pterigoideos
38) En relación a la fisiología gástrica, marque lo correcto:
A. la cimetidina actúa en la región basolateral de la célula parietal
B. la marea alcalina se debe al paso de bicarbonato través de la membrana apical de la
célula principal
C. el cloro difunde hacia el exterior por la la región basolateral de la célula parietal
D. la salida de hidrogeniones a la luz es por difusión facilitada

39) Durante el sueño, la concentración de bicarbonato en la saliva:


A. Se eleva a niveles mayores que los del plasma
B. Aumenta
C. No tiene efecto
D. Disminuye

40) La secreción de saliva es importante en la fisiología digestiva. Su concentración de


potasio llega a ser menor que la del plasma cuando su secreción tiene un flujo:
A. Intermedio
B. Nunca
C. Bajo
D. Alto

42) Respecto a las glándulas salivales, marque lo incorrecto:


A. la glándula parótida produce secreción serosa
B. la glándula sublingual drena a través de conducto de Wharton
C. La glándula parótida drena a través del conducto de Stenon
D. la glándula sublingual tiene forma de garfio

43) Con respecto a la saliva, marque la respuesta correcta:


A. será hipertónica cuando el flujo es bajo
B. a mayor flujo, menor concentración de Na
C. a mayor flujo, menor concentración de cloro
D. el sistema simpático estimula su secreción
CI 3
44) En relación a la circulación hepática, marque lo correcto:
a) Los sinusoides hepáticos transportan sangre mixta
b) La vena porta proporciona el 50% de sangre al hígado
c) La vena porta se forma a partir de la vena esplénica y la mesentérica inferior
d) La arteria hepática deriva de la mesentérica superior

45) Dentro de las funciones de las células de Ito, marque lo incorrecto:


a) Sintetizan colágeno
b) Almacenan vitamina A
c) Se les llama células estrelladas
d) Pueden fagocitar patógenos y actúan como presentadoras de antígeno
46) Paciente con tumor neuroendocrino productor de secretina, debido a lo cual se puede
esperar que su secreción pancreática, comparada con la de una persona sana de bajo flujo,
tenga una concentración de:
a) Sodio aumentada
b) Igual
c) Bicarbonato aumentada
d) Potasio disminuida
47) El GALT se localiza en:
a) Lámina propia
b) submucosa
c) borde en cepillo
d) superficie de criptas de Lieberkühn

48) En relación a la histología hepática, marque lo correcto:


a) la zona 1 se afecta rápidamente en estados de hipovolemia y shock
b) La zona 1 se encuentra cercana a la vena central lobulillar
c) La zona 3 se encuentra más cerca a la vena central lobulillar
d) La zona 3 se encuentra más cerca al eje menor del acino hepático
49) El acino pancreático difere con el de las glándulas salivales en:
a) Contiene células centroacinares
b) No produce secreción serosa
c) El páncreas produce principalmente secreción mucosa
d) No tienen diferencias
50) Durante la digestión de las grasa, para que la lipasa actúe adecuadamente se requiere que el
pH aumento en la luz intestinal, lo cual es logrado, entre otros, por la secreción de las
células:
a) Del conducto interlobulillar
b) Centroacinares
c) Acinares
d) Alfa
51) La secreción de la colecistoquinina (CCK) se produce en la fase:
a) intestinal
b) En las 3 por igual
c) gástrica
d) Cefálica
52) ¿Por cuál de las siguientes células es secretada principalmente la pro enzima
procarboxipeptidasa?
a) Acinares del páncreas
b) Epiteliales del duodeno
c) Ductales del páncreas
d) Centro Acinares del páncreas
53) Una mujer de 43 años dolor en hipocondrio derecho e icterica. En la ecografía se evidencia
cálculos biliares. Estos cálculos lo más probable es que se encuentren localizados en:
a) conducto colédoco
b) Conducto cístico
c) Vesícula biliar
d) Conducto pancreático secundario
ECU 1:
Estudiante de 21 años sufre de gastritis aguda ocasionada por comer en lugares poco
higenicos. Suele consumir caramelos (“chupar”) mientras esta en base hasta la tarde.
También toma regular cantidad de leche (grasa, lactora, proteinas), pues le calma el dolor y
el ardor que sitnete por la gastritis (tiene dispepsia y cuando toma la leche se le pasa).
Incluso cuando puede, se toma dos vasos de agua frita y le calma la molestia. Ha decido ir al
medico para tratarse, pues ya no soporta el dolor, el cual esta seguro que los síntomas se
producen por elevada producion de HCl en el estomago, y por ello le ha recetado ranitidina
1.1) El consumir caramelos eleva los niveles en sangre de una hormona cuya función es la
estimulación de las células.
- Beta del páncreas por GIP el cual es una incretina y por consiguiente estimula las
células pancreáticas

1.2) Consumir caramelos indirectamente actica la via:


-POMP/ CART saciedad

1.3) Consumo de leche produce indirectamente


- CCK inhibición del vaciamiento gástrico mayor tonicidad del esfínter pilórico

1.4) Cuando el px toma dos vasos de agua, genera indirectamente un aumento en la


liberación de:
- vaso de agua distención → g astrina → secreción de HCl

1.5) El uso de ranitidina bloquea el receptor H2 de la histamina en las células parietales, la


histamina llega a estas células por:
- histamina es una hormona paracrina por → difusión
**endocrina es por via hematógena y si fuera neuroendorina es por un NTs

1.6) Aumenta la secreción salival:


- noradrenalina a través de los receptores Beta 2
1.7) En este paciente con gastritis aguda debida a una alta producción de ácido clorhídrico, sería
lógico esperar que el píloro tenga un tono muscular:
- primero la secretina
- luego CCK
**ambas reguladores del HCl, Gatritis aguda debido a una alta producción de HCL piloro
estará aumentado (por la CCK)
1.8) Debido al uso de ranitidina, los valores de somatoestina en sangre:
- ranitidina disminuye acción de gastrina se quiere secretar mas no actúan los
inhibidores como la somatoestina somatoestina disminuye
1.9) El uso de atropina en este paciente:
- Inhibirá la acción de las prostaglandinas
- Aumentará la producción de ácido clorhídrico
- Disminuirá la acción del receptor CCK-B
-Aumentará el pH del estómago
ECU 2:
Niño de sexo masculino de 2 años de edad, sufre de estreñimiento desde el nacimiento (1
deposición cada 3-4 días). Madre menciona que le estimula la defecación con un
termómetro rectal, y continuo uso de enemas y laxantes. Desde hace 6 meses comienza
con vómitos postprandiales. Los síntomas aumentan en frecuencia y magnitud y están en
relación con los episodios de estreñimiento. No refiere fiebre, tos, diarrea ni lesiones
cutáneas. Al examen físico presenta regular estado general, luce deshidratado. Abdomen
distendido, blando, depresible e indoloro. No se palpan masas abdominales. Se
permeabiliza el canal anal con termómetro rectal, encontrando cierta resistencia. Salida de
material fecal mal oliente en regular cantidad. Exámenes de laboratorio: hemograma
normal. Signos inflamatorios de fase aguda negativos. Alcalosis metabólica leve en sangre
venosa. Radiografía con enema baritado muestra recto y colon sigmoides dilatados
(megacolon). Biopsia profunda: ausencia de células ganglionares en la muestra enviada. Se
realiza cirugía correctiva.

2.1) Durante la fase esofágica de la deglución, para un bolo alimenticio determinado, a


medida que avanza el bolo la fuerza de la contracción se hace más:
- hiperpolarizado
- fuerte
- dependiente de Ach
- debil
2.2) Cuando este paciente ingiera sus alimentos, se espera que al momento de pasar el bolo
alimenticio por el esfínter esofágico superior, la presión intraesofágica disminuya en:
- la porción proximal al bolo
- el tercio medio del esófago
-el cardias
- el lugar donde se contraiga la muscular propia
2.3) Al examinar la orofaringe del paciente, uno puede hallar fácilmente la amígdala
palatina, pues esta se encuentra inmediatamente detrás del músculo:
-Palatogloso
- Palatofaringeo
- Hiogloso
- Elevador del velo del paladar

2.4) Con respecto a la defecación en este caso, marque la respuesta correcta:


- En posición de cuclillas, el músculo puborectal genera un ángulo más agudo en el
recto
- El sigmoides y el recto están inervados por el nervio vago
- La sensación de defecar sólo se da cuando el recto es ocupado por heces y
alcanzado el 80% de su capacidad
-El esfínter anal comprometido tiene inervación autónoma

2.5) En este paciente [hirschsprung] se considera que está abolido el reflejo:


- Coloileal
-Rectoesfinteriano
- Gastrocólico
- Relajación receptiva

2.6) No se espera que sea causa del vómito:


-Ayuno prolongado
- Estimulación faríngea y del glosofaríngeo
- Irritación de la mucosa gástrica
- Dolor intenso

ECU 3:
Paciente de 54 años con antecedentes de alcoholismo, gastritis crónica, tabaquismo
pesado, obesidad, cálculos biliares y cirrosis, es llevado a la emergencia por dolor
abdominal en epigastrio irradiado a la espalda y trastorno del sensorio.
Al examen físico: presión arterial 85/50 mmHg, frecuencia cardíaca 100 latidos/min,
frecuencia respiratoria 18 x minuto, temperatura axilar 36°C.
Conjuntivas pálidas, escleras ictéricas nevus arácnidos en tronco, distensión abdominal
marcada, cabeza de medusa, matidez desplazable en ambos flancos e hipogastrio, dolor a
la palpación de abdomen.
Tiempo de protrombina: 24 seg (testigo: 13 seg); TPT: 38 seg, glicemia: 165 mg/dL, uremia:
20 mg/dL, ASAT: 76 UI/L, ALAT: 22 UI/L, albumina: 2,5 g/dL, bilirrubina total: 2,6 mg/dL,
bilirrubina directa: 1,4 mg/dL, amilasa sérica 4000 U/L.
3.1) En esta paciente, al aumento de la amilasa sérica, se debe directamente a una lesión de:
a) páncreas
b) vesícula y árbol biliar
c) estómago
d) hígado
3.2) Considerando que el paciente sufre de gastritis, se puede decir que la secreción de
ácido por la mucosa gástrica
a) involucra transporte activo de hidrogeniones
b) es realizada principalmente por células principales
c) es inhibida por antihistaminas tomadas por pacientes con rinitis alérgica
d) involucra la liberación de HCl de los gránulos zimógenos
3.3) El paciente tiene hemorragia digestiva alta por várices sangrantes como complicación.
Llegando a estar en shock hipovolémico por hemorragia masiva, se encontrara necrosis
hepática en:
a) zona 1
b) no se afectan los lobulillos hepáticos en hemorragia
c) zona 3
d) zona 2
3.4) El misoprostol, análogo de las prostaglandinas está mejor indicado en:
a) cicatrización de úlcera péptica duodenal
b) erradica el helicobacter pylori
c) tratar el sind de Zollinger ellison
d) prevenir daño por AINES
3.5) De las siguientes sustancias secretadas por los órganos de este paciente, la más
alcalina es la secreción:
- Esofágica
- Salival
- Yeyunal
-Pancreática
3.6) En este paciente, se puede asumir que la pancreatitis ha sido ocasionada por una
disminución en el efecto de:
- Lipasa
- Enterocinasa
- Amilasa
-Inhibidor de la tripsina
3.7) ¿Cuál de las siguientes sustancias es segregada por el páncreas?
-Amilasa
- Pepsina
- Quimiotripsina
- Tripsina
3.8) Cada vez que este paciente toma alcohol, la acidificación de la luz del duodeno:
-Disminuye el vaciamiento gástrico
- Aumenta la contracción del esfínter de Oddi
- Aumenta la secreción del ácido gástrico
- Disminuye la secreción pancreática del bicarbonato
SISTEMA
DIGESTIVO (ME
154) EXAMEN
FINAL
Ciclo 2018-01

1. Un niño de 2 años es llevado a la consulta por diarrea persistente y edema de las extremidades, además falta
de crecimiento y desarrollo en relación a su edad. Los análisis de sangre revelan que tiene concentración
plasmática baja de proteínas (hipoproteinemia). Durante la endoscopía duodenal, se coloca colecistokinina
(CCK) endovenosa y se recoge muestras del líquido duodenal; el resultado del líquido confirma incapacidad
para hidrolizar proteínas a un pH neutro, esta situación mejora al añadir una pequeña cantidad de tripsina.
El paciente probablemente esté sufriendo la falta congénita de
………….
(Unidad 4, sesión 26, logro 2: Explicar la Digestión y absorción de las proteínas y sus alteraciones)
a. Pepsinógeno
b. PEPT-1
c. Carboxipeptidasas
d. Enterocinasa

2. Experimentalmente se incrementa la velocidad de la secreción salival con una sustancia, el análisis de


la composición de esta saliva obtenida se espera encontrar…………..
(Unidad 3, sesión 17, logro 5 : Explica la Influencia de la velocidad del flujo salival en la composición de la
saliva)
a. Elevación de concentración de bicarbonato, sodio y potasio
b. Elevación de concentración de cloro, sodio y potasio
c. Disminución de concentración de potasio
d. Disminución de concentración de potasio y bicarbonato

3. Paciente varón de 46 años soltero, consulta por odinofagia y bajo de peso, tiene antecedente de
tuberculosis desde hace 3 meses y es fumador crónico (10 cigarrillos por día); al evaluar la cavidad oral se
identifica lesión blanquecina en el dorso de la lengua y paladar blando, las lesiones se desprenden con el
baja lengua dejando una base eritematosa. Esta lesión corresponde probablemente a ……………………….…..
( Unidad 3, sesión18, logro 1-2 : Describe las enfermedades inflamatorias, infecciosas y proliferativas de la
cavidad oral)
a. Eritroplaquia
b. Candidiasis oral
c. Leucoplaquia vellosa
d. Fibroma en cavidad oral

4. Minero de 32 años de edad, que acude a centro de


salud por presentar de forma progresiva desde hace
1 año dificultad para ingerir alimentos sólidos y
luego líquidos; refiere regurgitaciones alimentarias y
marcada pérdida de peso (15 kilos). Radiografia
baritada de esófago como se muestra en la figura. El
presente caso se explica por……………….
(Unidad 2, sesión 12, logro 4: Identificar y describir
la función de los esfínteres esofágicos)
a. Contracción incompleta del esfínter esofágico inferior
b. Dificultad para el inicio de la deglución
c. Relajación incompleta del esfínter pilórico
d. Relajación incompleta del esfínter esofágico inferior
5. Paciente mujer de 35 años acude a consulta por sensación de sequedad y lesiones en cavidad oral. Al
examen se observa atrofia de la mucosa, fisuras y úlceras; nota además sequedad e irritación de la
córnea y aumento del tamaño de las glándulas parotídeas. Su diagnóstico más probable es artritis
reumatoide; el hallazgo más probable en una biopsia de glándula parótida es……..….
(Unidad 3, sesión 18, logro 3: Describe las enfermedades más frecuentes de las glándulas salivales)
a. Hiperplasia de acinos glandulares serosos
b. Gran infiltración de linfocitos y células plasmáticas
c. Gran infiltrado de linfocitos y macrófagos
d. Presencia de acinos normales con hiperplasia de células ductales

6. Un paciente con anemia acude con su médico quejándose de episodios frecuentes de gastroenteritis. Un
análisis de sangre revela anticuerpos circulantes dirigidos contra células parietales gástricas. Su anemia es
atribuible a la hiposecreción de
………………………
(Unidad 3, sesión 20, logro 5: Gastritis crónica. Tipos de gastritis)
a. Factor intrínseco
b. Proteina R (haptocorrina)
c. Pepsinógeno
d. Ácido clorhídrico

7. Dos estudiantes deciden tomar un receso para comer una hamburguesa a la hora del almuerzo. Antes de
llegar a la cafetería, impulsos nerviosos provenientes del complejo vagal dorsal iniciarán la secreción de
ácido gástrico por la liberación dedesde el sistema nervioso entérico.
(Unidad 3, sesión 20, logro 2: Regulación de la secreción gástrica: estimulación, fases de la secreción)
a. Serotonina
b. Óxido nítrico
c. GRP (péptido liberador de gastrina)
d. Péptido intestinal vaso activo

8. Un niño de cuatro años de edad es llevado a la consulta por cuadros diarreicos frecuentes caracterizados
por heces pálidas, voluminosas y fétidas, presenta bajo peso y talla. Se mide la concentración de cloruro en
el sudor y se encuentra que sus valores son muy elevados. La alteración más importante a nivel de células
ductales del páncreas tiene relación directa con la conductancia de…………
(Unidad 3, sesión 23, logro 5 Explica la Secreción pancreática: formación del jugo pancreático, influencia de la
velocidad de flujo y regulación)
a. Potasio
b. Bicarbonato
c. Sodio
d. Cloro

9. Una mujer de 50 años de edad que sufrió durante varios años resequedad de los ojos debida a producción
inadecuada de lágrimas es enviada con un gastroenterólogo para evaluación de pirosis crónica. El examen
endoscópico revela erosiones y tejido cicatrizal en la parte distal del esófago justo por arriba del esfínter
esofágico inferior. Las lesiones pueden atribuirse a la disminución de uno de los siguientes componentes
salivales:
(Unidad 3, sesión 17, logro 4: Explicar la Formación de la saliva y cuáles son sus componentes)
a. Bicarbonato
b. Lactoferrina
c. Ig A
d. Amilasa
10. Se evalúa los valores séricos de las siguientes sustancias a un paciente con enfermedad hepática terminal;
en este paciente se espera encontrar la combinación con la letra …………
(Unidad 3, sesión 22, logro 5: Describe las Pruebas de función hepática, la Insuficiencia hepática,
encefalopatía hepática e hipertensión portal)

Glucosa Amoniaco Albúmina


a. Aumenta Disminuida Disminuida
da
b. Disminui Aumentada Aumentada
da
c. Aumenta Aumentada Aumentada
da
d. Disminui Aumentada Disminuida
da

11. Una mujer de 35 años de edad HIV positiva, se presenta al médico con dolor abdominal en cuadrante
superior derecho e ictericia. La paciente refiere haber tenido múltiples episodios de ictericia durante los
últimos 10 años. Los exámenes para determinar hepatitis viral, dieron positivos para Hepatitis B, siendo
catalogado el caso como hepatitis crónica con alteración funcional. En un examen de sangre ¿cuál de los
siguientes parámetros está disminuido?
(unidad 3, sesión 22, logro 5: Pruebas de función hepática, Insuficiencia hepática, encefalopatía hepática
e hipertensión portal)
a. Fosfatasa alcalina
b. Albumina
c. Bilirrubina
d. Tiempo de protrombina

12. En el reflejo peristáltico del intestino delgado, uno de los siguientes eventos sucede en la
porción oral del bolo alimenticio…………...
(Unidad 2, sesión 13, logro 4: Explicar la Motilidad del intestino delgado: Contracciones segmentarias y
peristálticas)
a. Disminución de 5 hidroxitriptamina desde las neuronas IPAN
b. Contracción del músculo longitudinal
c. Acción del péptido intestinal vasoactivo (VIP) en el músculo circular
d. Acción de acetilcolina en el músculo circular

13. Experimentalmente se coloca una dosis alta de secretina en la luz intestinal duodenal; como
consecuencia de esto, en el jugo pancreático de la misma luz intestinal se observa la disminución de la
concentración de …..………..
(Unidad 3, sesión 23, logro 5: Explica la Secreción pancreática: formación del jugo pancreático, influencia
de la velocidad de flujo y regulación)
a. Na+
b. Cl-
c. K+
d. HCO3-

14. Un varón de 58 años de edad con enfermedad de Crohn severo fue sometido a una resección ileal.
Después de la cirugía este paciente padecerá de esteatorrea, esto se explica porque …..………..
(unidad 4, sesión 26, logro 4: Explica las alteraciones en la Absorción de lípidos)
a. El pool de ácidos biliares se incrementa
b. Los quilomicrones no pueden formarse en el lumen intestinal
c. La micelas no pueden formarse
d. El páncreas no secreta lipasa

15. En un experimento se inserta un balón en el estómago de un voluntario, se infla poco a poco mientras que
se vigilan las presiones intraluminales. Aunque el volumen del balón aumenta considerablemente, las
presiones permanecen constantes. Esta relación volumen-presión se explica por la liberación local de
…………..
(Unidad 2, sesión 13, logro 1 Explica la Motilidad gástrica: relajación receptiva)
a. Acetil colina y gastrina
b. Colecistoquinina y óxido nítrico
c. Óxido nítrico y péptido inhibidor vasoactivo
d. Norepinefrina y óxido nítrico
16. La toxina del Vibrio cholerae causa diarrea debido a…….
(Unidad 4, sesión 27, logro 6: Explica el transporte hidroelectrolítico intestinal, toxina colérica)
a. La fosforilación del canal CFTR de los enterocitos de las vellosidades intestinales
b. El Incremento de la secreción de cloro por las células de la cripta intestinal
c. La inhibición de la producción de AMPc por las células epitelailes
d. El incremento de la absorción de agua y sodio a través de las uniones estrechas

17. ¿Cuál de las siguientes alternativas es una característica de la secreción exocrina del páncreas?
(Unidad 3, sesión 23, logro 5: Secreción pancreática: formación del jugo pancreático, influencia de la
velocidad de flujo y regulación)
a. Tiene una baja concentración de Cl- respecto al plasma
b. Es estimulada por la presencia de bicarbonato en el duodeno
c. La secreción enzimática es estimulada principalmente por la gastrina
d. Es hipotónica respecto al plasma

18. Una madre lleva a su hijo de dos años de edad a la sala de urgencias, estresada porque el niño deglutió una
moneda de 10 céntimos mientras la familia cenaba en un restaurante. El médico observa mediante
fluoroscopía que la moneda se halla en el estómago y asegura a la madre que la moneda se eliminará con
las heces. El médico recomienda utilizar la respuesta fisiológica que permitirá la evacuación de la moneda
del estómago al intestino ………….…..
(Unidad 2, sesión 13, logro 2: Explica la Motilidad gástrica: mezclado y vaciamiento)
a. Es por la relajación receptiva
b. Son los movimientos de mezcla y trituración
c. Es provocada por el ayuno
d. Es por la relajación del esfínter esofágico superior

19. Las estructuras en el hígado que permite que los productos metabólicos unidos a proteínas tengan acceso
a las membranas basolaterales de los hepatocitos, son…..
(Unidad 3, sesión 21, logro 4-5: Explica la Organización micro estructural del hígado)
a. Los Canalículos
b. Las fenestras sinusoidales
c. Las uniones intercelulares herméticas
d. Las células de Ito

20. La composición de la bilis es modificada conforme fluye por los conductillos biliares. Durante este
tránsito se espera que aumente la concentración de…….
(Unidad 3, sesión 22, logro 2: Describe la Secreción biliar, visión general del sistema biliar extrahepático y
composición de la bilis)
a. Ig A
b. Glucosa
c. Monómeros de ácido biliar
d. Vitamina A

21. Se mide experimentalmente el contenido gástrico de dos personas. La persona “A” tiene alto
contenido de grasa y la persona “B” tiene un contenido hipertónico ¿Cuál de las siguientes es
correcto respecto al vaciamiento gástrico? (Unidad 2, sesión 13, logro 2: Describe la Motilidad y
vaciamiento gástrico)
a. Hay ralentización del vaciado gástrico solo en “A”
b. El vaciamiento gástrico es más rápido en ambos
c. En ambos casos hay incremento de la motilina
d. Hay ralentización del vaciado gástrico en ambos casos

22. El examen endoscópico de un paciente con hipertensión portal grave revela venas tortuosas que sobresalen
hacia la luz del esófago. El paciente recibe tratamiento quirúrgico mediante la colocación de una derivación
que conecta la vena porta a la vena cava. Después de la operación el riesgo de encefalopatía …………………..
y el riesgo de sangrado de várices ……………..
(Unidad 3, sesión 22, logro 5: Describe la Insuficiencia hepática, encefalopatía hepática e hipertensión portal)
a. Aumentará/disminuirá
b. Disminuirá/disminuirá
c. Aumentará/aumentará
d. Disminuirá/aumentará
23. Un paciente varón de 18 años de edad acude al médico para sus exámenes de rutina. Sus resultados de
laboratorio muestran un valor de bilirrubina sérica de 4 mg/dl y una bilirrubina directa de 0,3 mg/dl. Las
pruebas de función hepática son normales. La alteración que explica mejor este caso es por la
deficiencia de ………………..
(Unidad 3, sesión 22, logro 3: Explica la Producción y excreción de bilirrubina. Tipos de bilirrubina e ictericia)
a. Transaminasas
b. Glucuronil transferasa
c. Hemo oxigenasa
d. La 7 alfa hidroxilasa

24. Un hombre de 57 años de edad es llevado a urgencias con hematemesis masiva rojo brillante, a su llegada
se halla inconciente con PA: 80/40 mm Hg y FC: 124 lat/min. Luce ictérico con presencia de “arañas
vasculares en el tórax anterior y extremidades”, abdomen distendido con signo de oleada positiva. Se
encuentra esplenomegalia y pérdida de la masa muscular en extremidades. La anastomosis vascular
responsable del sangrado en este paciente es ………….…..
(Unidad 3, sesión 21, logro 2: Describe las anastomosis porto sistémicas)
a. Arteria gástrica izquierda y vena ácigos
b. Vena gástrica izquierda y vena ácigos
c. Vena paraumbilical y vena epigástrica inferior
d. Vena gástrica izquierda y vena esofágica superior

25. Un estudiante de medicina está comiendo un plato de comida a base de champiñones, espárrago y salsa
de soya. El sabor umami contenido en todos estos alimentos actúa a nivel de los botones gustativos
estimulando ………………..
(Unidad 2, sesión 10, logro 5: Describe los tipos y mecanismos moleculares para la detección de los sabores)
a. El ingreso de sodio
b. Un receptor acoplado a proteína G
c. Su receptor específico T1R3
d. El ingreso de hidrógeno

26. Un hombre de 22 años de edad se presenta al médico con una historia de 1 año de evolución
caracterizado por dolor recurrente en fosa iliaca derecha y diarrea. Manifiesta además pérdida de
peso de 8 kg durante este periodo. La colonoscopía revela múltiples lesiones en el ileon terminal y
colon. La biopsia de estas lesiones revela engrosamiento, inflamación y ulceración de la mucosa. El
diagnóstico más probable en este caso es…….
(Unidad 4, sesión 28, logro 5: Describe la Enfermedad inflamatoria intestinal. Generalidades, morfología y
características)
a. Sprue celiaco
b. Enfermedad de Crohn
c. Sindrome de colon irritable
d. Colitis ulcerativa

27. Una de las funciones del músculo señalado es:


(Unidad 2, sesión 8, logro 3: Describir el Piso de la
boca: estructuras blandas que la conforman)
a. Eleva el paladar blando
b. Recibe inervación del nervio maxilar
c. Deprime el hioides cuando la mandíbula está fija
d. Deprime la mandíbula cuando el hioides está fijo

28. Varón de 61 años que consulta por dolor retro esternal intenso desde hace 6 horas y después de
vómitos intensos y repetidos; al examen se observa disnea, cianosis, hipotensión y signos clínicos de
shock. La radiografía simple de tórax muestra neumomediastino. El líquido en el espacio pleural
aspirado tiene alta concentración de amilasa. ¿Cuál de las siguientes alternativas puede explicar este
cuadro clínico?
(Unidad 3, sesión 18, logro 6: Describe algunas Enfermedades del esófago)
a. Sindrome de Mallory Weiss
b. Rotura espontánea de esófago
c. Neumotórax por probable herida penetrante
d. Perforación de ulcera gástrica de cara posterior, con complicación torácica
29. La secreción del ácido en la célula parietal gástrica se lleva a cabo por una ATPasa especifica que
intercambia hidrogeniones (H+) del citosol por…..
(Unidad 3, sesión 20, logro 1: Explica la Secreción del HCl y sustancias que la alteran)
a. Cl-
b. HCO3-
c. Na +
d. K+

30. En condiciones normales el ingreso de 600 ml de líquido es el estómago provoca un aumento de presión
intragástrica de unos 12 cm de H2O. Después de una vagotomía (corte del nervio vago) es de esperar que
el ingreso del mismo volumen de líquido provoque lo siguiente: …………………………………
(Unidad 2, sesión 13, logro 1: Describe la Motilidad gástrica: relajación receptiva)
a. Un aumento igual de la presión
b. Que no aumente la presión
c. Un aumento mayor de la presión
d. Una disminución de la presión

31. Una paciente de 30 años de edad es sometida a una cirugía en oído medio derecho por un problema de
otoesclerosis. Luego de la cirugía refiere alteración en la percepción de sabores. Al evaluar el caso usted
esperaría encontrar……….
(Unidad 2, sesión 10, logro 5: Describe la Irrigación e inervación de la lengua)
a. Alteración en la sensación del dolor y temperatura en el tercio posterior de la lengua
b. Alteración en la sensación del gusto en los dos tercios anteriores de la lengua
c. Alteración en la sensación del gusto en la punta de la lengua
d. Sensación del dolor, tacto y temperatura conservada en toda la lengua

32. ¿Cuál de las siguientes alterativas es correcta?


(Unidad 4, sesión 26 : Explica la digestión y absorción de nutrientes y sus alteraciones)
a. En el borde luminal, en cepillo, del intestino delgado, la absorción de sodio únicamente se realiza
asociada a la de glucosa.
b. El lugar principal para la absorción del hierro es el ileon
c. Las sales biliares desconjugadas son absorbidas preferentemente en el colon
d. El proceso de digestión y absorción de la vitamina B12 no se altera en insuficiencia pancreática.

33. En un paciente de 45 años de edad con colestasis biliar, se encuentra una elevación de los niveles
sanguíneos de fosfatasa alcalina hasta 3 veces la cifra normal. ¿Cuál de las siguientes alternativas estará
también elevada como evidencia del daño de la vía biliar?
(Unidad 3, sesión 22, logro 5: Pruebas de función hepática, Insuficiencia hepática, encefalopatía hepática
e hipertensión portal)
a. Tiempo de protrombina y albúmina sérica
b. Transaminasas hepáticas (ALT y AST)
c. Glucoronil transferasa
d. Gamma glutamil transpeptidasa

34. Revisando la angiografía de un hombre de 70 años en estudio por aneurisma de aorta abdominal el
radiólogo informa de la presencia de una oclusión completa de la arteria mesentérica inferior. El paciente
se encuentra completamente asintomático. ¿Cuál de las siguientes arterias se anastomosa a la sistema
arterial de la mesentérica inferior?
(Unidad 4, sesión 25, logro 1: Identifica la Arteria mesentérica superior e inferior, ramas y anastomosis)
a. Ileal
b. Cólica media
c. Sigmoideas
d. Cólica izquierda

35. Lactante de 3 meses de vida es atendido por presentar diarrea, se administra una solución de glucosa y
electrólitos por vía oral. La proteína de membrana apical que explica la capacidad de esta solución para
proporcionar aporte de glucosa e hidratación es ………..
(Unidad 4, sesión 26, logro 1: Explica la Digestión y Absorción de los hidratos de carbono. Alteraciones)
a. GLUT-5
b. SGLT-1
c. CFTR
d. GLUT-2
36. Paciente ha sufrido herida de bala en el abdomen, se le ha tenido que extirpar el segmento medio y distal
del ileon. En este caso la síntesis hepática de sales biliares estará …..…..
(Unidad 3, sesión 22, logro 4: Explica la formación, función y Circulación entero hepática de lasa sales biliares)
a. Disminuida por inhibición de la colesterol 7 alfa hidroxilasa
b. Incrementada por estímulo de la enzima colesterol 7 alfa hidroxilasa
c. Incrementada por inhibición de la colesterol 7 alfa hidroxilasa
d. Sin cambios en el ritmo de síntesis

37. Un varón de 75 años ingresa al consultorio por presentar ictericia marcada de piel y las escleras. El estudio
del paciente mostró que presentaba un tumor que obstruía la totalidad del conducto hepático común.
¿Cuál de las siguientes estructuras se encontrará dilatada en este paciente?
(Unidad 3, sesión 21, logro 6: Describir el árbol biliar intrahepático)
a. Conducto de Wirsung
b. Conductos de Hering
c. Conducto colédoco
d. Conducto cístico

38. Correlaciones las dos columnas y marque la fórmula correcta:


(Unidad 4, sesión 28, logro 1: Diarrea: definición, mecanismos: osmótica, secretoria y exudativa)
1. Enfermedad Hirschsprung( ) heces con moco y sangre
2. Diarrea osmótica( ) intolerancia a lactosa
3. Diarrea secretoria( ) aganglionosis congénita
4. Diarrea exudativa( ) canales de Cl- en las células de la cripta

a.- 4231b.- 1234c.- 2143d.- 4213

39. Respecto a la siguiente imagen que representa una estructura de la mucosa gástrica, la estructura con
número ………..
produce ……………………..
(Unidad 3, sesión 19, logro 4: La glándula fúndica. Funciones y tipos de células con sus características)
a. 3 / pepsina
b. 1 / Pepsinógeno
c. 4 / HCl y factor extrínseco
d. 2 / pepsinógeno

2
40. En un paciente con insuficiencia renal crónica, el déficit en la absorción de calcio a nivel del
enterocito se debe a lo siguiente:
(Unidad 4, sesión 26, logro 6: Explica la Absorción de calcio y hierro)
a. No se convierte la 25 hidroxicolecalciferol a 1,25 dihidroxicolecalciferol
b. No se convierte la 1,25 dihidroxicolecalciferol a 25 hidroxicolecalciferol
c. Se incrementa la producción de Calbindina
d. Existe un descenso de la alfa 25 hidroxilasa renal
CLAVES EXAMEN
PARCIAL DE
SISTEMA
DIGESTIVO 2019 -
00

1. Varón de 30 años es traído a emergencia por agresión abdominal con arma de fuego (pistola)
y es sometido a laparotomía exploratoria, observándose isquemia del colon ascendente y
parte del colon trasverso ¿la lesión de cuál de las siguientes arterias explicaría esta isquemia?
(unidad 1, sesión 2, logro 6: (Describe la irrigación visceral: arterias de tronco celiaco,
arteria mesentérica superior e inferior, topografía de superficie, órganos por cuadrante)

a. Celiaca
b. Colónica derecha
c. Mesentérica inferior
d. Mesentérica superior

2. Respecto a las sustancias gastrointestinales que regulan la secreción pancreática;


marque la afirmación correcta:
(unidad 1, sesión 3, logros 2 y 3: describir las hormonas gastrointestinales: estímulos y funciones)

a. La Secretina, es la hormona más importante para la secreción de bicarbonato por las


células acinares del páncreas
b. La acetilcolina es capaz de estimular la secreción enzimática y de bicarbonato del páncreas
c. La gastrina, es la hormona más importante para la secreción de enzimas pancreáticas
d. La colecistoquinina (CCK) estimula al páncreas solo para secreción enzimática

3. Ante una lesión del X par craneal, ¿cuál de los siguientes músculos mantiene conservada su
función?:
(unidad 2, sesión 08, logro 4: Paladar blando: componentes musculares)

a. Elevador del velo del paladar


b. Tensor del velo del paladar
c. Palatofaríngeo
d. Glosofaríngeo

4. Experimentalmente se utiliza atropina (anticolinérgico) para inhibir la secreción de


gastrina, sin embargo la secreción de esta hormona se sigue dando ante estímulos
vagales. Esta situación se explica porque la atropina:
(unidad 1, sesión 3, logro 3 : describir las hormonas gastrointestinales: estímulo y funciones de la
gastrina y colecistoquinina)

a. Bloquea parcialmente la bomba de protones en la célula G


b. Inhibe la acción de acetilcolina e histamina en la célula G
c. Solo inhibe la acción del péptido GRP en la célula G
d. No bloquea la acción del péptido GRP

5. Un varón de 50 años es sometido a extirpación del duodeno y parte proximal del


yeyuno. Esta situación ocasionaría la pérdida de las células ……….. , productoras deque
estimula la secreción de
bicarbonato por el páncreas.
(unidad 1, sesión 3, logro 3: describir las hormonas gastrointestianles: estímulos y funciones
de la secretina y péptido insulinotrópico dependiente de glucosa)

a. “S” / secretina
b. Parietales / secretina
c. “I” / colecistoquinina
d. “S” / colecistoquinina
6. Recién nacido que presenta tumoración abdominal a nivel del cordón umbilical
(fotografía). ¿cuál de las siguientes afirmaciones es correcta respecto a este defecto en
el desarrollo embriológico del intestino?: (unidad 1, sesión 5, logro 2: identificar las
anomalías del desarrollo del intestino medio)

a. Corresponde a una Gastrosquisis


b. Las vísceras se hallan cubiertas por piel
c. No está asociado a otras malformaciones
d. Se asocia a
malformaciones cardiacas
y del tubo neural

7. Varón de 35 años acude a la emergencia por


trauma abdominal y se decida realizar una
laparoscopía exploratoria. El cirujano observa la
disposición de los órganos abdominales como se
representa en el siguiente esquema. Esta
disposición de órganos se explica por la
rotación(SMA=arteria mesentérica superior)
(unidad 1, sesión 5, logro 3: identificar las anomalías del
desarrollo del intestino medio: defectos de rotación,
estenosis y atresias)

a. anti horaria del intestino medio, en sólo 90°


b. incompleta del intestino medio (270°)
c. horaria del intestino medio
d. horaria del estómago

8. Se evalúa la expresión de la proteína Agrp en una persona con alteración del apetito; lo
correcto respecto a esta proteína es…..
(unidad 1, sesión 3, logro 4: Explica los mecanismos de control del apetito y saciedad )

a. Esta proteína es un potente anorexigénico


b. La mutación del gen que la codifica produce adelgazamiento
c. La sobre producción de la proteína lleva a obesidad por agonismo de receptores MC3 y MC4
d. La sobre producción de la proteína disminuye el apetito por antagonismo de receptores MC4

9. Juana cae de la bicicleta y se fractura la región anterior del hueso maxilar superior con
compromiso de la fosa incisiva. Al examen físico de la región esperaría encontrar alteración
en la sensibilidad de la encía …………………
(unidad 2, sesión 8, logro5: paladar: paladar duro y blando: irrigación e inervación)

a. bucal posterior
b. Lingual anterior
c. palatina anterior
d. palatina posterior
10. Recién nacido es atendido por el neonatólogo y luego entregado a su madre para dar de
lactar; la madre al dar de lactar observa coloración azulada de labios, acompañado de tos
persistente, dificultad respiratoria y distención abdominal. Se le intenta colocar una sonda
nasogástrica pero esta retorna a la cavidad oral en todos los intentos. ¿Cuál de las
siguientes anomalías del desarrollo es el más probable en este caso? (unidad 1, sesión 4,
logro 3: identificar las anomalías en el desarrollo del esófago: atresia y/o fístula traqueo
esofágica)

a. Estenosis esofágica proximal con Fístula traqueo esofágica distal


b. Atresia esofágica proximal con fístula traqueoesofágica distal
c. Atresia esofágica distal con fístula traqueoesofágica proximal
d. Fístula traqueoesofágica proximal y distal

11. ¿Cuál de los siguientes mecanismos ocurre durante la defecación?


(unidad 2, sesión 13, logro 6: motilidad del intestino grueso: contracciones segmentarias,
movimientos en masa, defecación y reflejo gastrocólico)

a. Contracción refleja del esfínter anal interno


b. En la posición de “cuclillas” el músculo puborectal se halla relajado
c. Relajación del esfínter anal externo por efectos del VIP y óxido nítrico
d. La materia fecal en el recto estimula la contracción del sigmoides por los nervios pudendos

12. La estructura número 4 (gráfico) corresponde a


……….… y está ………..
(unidad 2, sesión 9, logro 2: Partes
de un diente. Capas del diente:
Esmalte: características y células
que lo producen)

a. el cemento / mineralizado en 90%


b. la dentina / formada por ameloblastos
c. el esmalte / formado por
células derivadas del
mesénquima
d. la dentina / formado por
células derivadas de la cresta
neural

13. Un paciente luego de un accidente sufre lesión del piso de la boca, se constata daño del
nervio “cuerda del tímpano”, en este caso se esperaría encontrar disminución de lade la
lengua
(unidad 2, sesión 10, logro 3: Irrigación e inervación de la lengua)

a. Motilidad en los dos tercios anteriores


b. Sensación del gusto en el tercio posterior
c. Sensación del gusto en los dos tercios anteriores
d. Sensibilidad al tacto en los dos tercios anteriores

14. ¿Cuál de las siguientes afirmaciones es la correcta sobre la gastrina?


(unidad 1, sesión 3, logro 1: reconocer las características de las sustancias reguladoras
gastrointestinales: hormonas, sustancias paracrinas y neurocrinas)

a. Produce atrofia de la mucosa gástrica


b. Es producida por la célula G del cuerpo gástrico
c. Es estimulada por la distensión gástrica y el Ph bajo
d. Actúa en la célula diana mediante su receptor CCk tipo B
15. Al recibir un paciente con signos de hipovolemia y antecedente de trauma en abdomen por
accidente de tránsito, usted identifica radiológicamente: lesión de primera vértebra lumbar y
signos de lesión en páncreas; durante la cirugía se observó pobre irrigación de asas
intestinales. El vaso afectado es la arteria ……..
(unidad 1, sesión 1, logro 6: reconocer las estructuras a nivel de L1, nivel de los principales vasos
sanguíneos)

a. esplénica
b. hepática común
c. mesentérica inferior
d. mesentérica superior

16. Un paciente sufre de daño a nivel del cuello con lesión muscular en la región de la faringe. En
el examen físico se determina dificultad para la elevación de la faringe y para el cierre del
itsmo de las fauces. En este caso, probablemente esté afectado el músculo:
(unidad 2, sesión 11, logro 2: Músculos de la faringe: identificación, constrictores y longitudinales)

a. palatogloso
b. estilofarinfeo
c. palatofaringeo
d. constrictor inferior

17. Varón de 50 años a quien le realizan la curación de la segunda molar de la arcada


superior derecha. En un momento determinado, el paciente acusa de intenso dolor de la
pieza dentaria en tratamiento. La vía aferente del dolor viaja a través del nervio …………
(unidad 2, sesión 9, logro 6: Inervación de los dientes)

a. trigémino V2
b. trigémino V3
c. naso palatino
d. palatino menor

18. La distención gástrica por los alimentos produce incremento de secreción de HCl mediante la
producción de
………….. que estimula a las células ……………. vía proteína ………..
(Unidad 1, sesión 3, logro 2: Describe las hormonas gastrointestinales: Estímulo y funciones de la
gastrina y colecistoquinina)

a. gastrina / parietal / Gq
b. gastrina / principal / Gs
c. acetilcolina / parietal /Gi
d. acetilcolina / principal / Gi

19. Un niño de tres años llega a emergencia con disfagia (dificultad para tragar), dolor retro
esternal, salivación y llanto. Se sospecha de ingesta de cuerpo extraño (moneda) en el
esófago; al ser evaluado se constata en una radiografía presencia de cuerpo extraño a nivel
de C6 (6° vértebra cervical). El cuerpo extraño estará suspendido a nivel del estrechamiento
producido por………..
(unidad 2, sesión 11, logro4: Esófago, características anatómicas, relación con órganos vecinos y
estrecheces)

a. el cayado aórtico
b. el hiato esofágico
c. el músculo cricofaríngeo
d. el bronquio principal izquierdo

20. La triada portal (arteria hepática, vena portal y conducto biliar común) está contenida en el
ligamento
…….……… y derivan embriológicamente del ……
(Unidad 1, sesión 1, logro 4: Identifica el peritoneo, mesenterio, omento y ligamentos,
retroperitoneo.)

a. hepato duodenal / mesenterio ventral


b. gastro esplénico / mesenterio dorsal
c. hepato gástrico / omento menor
d. falciforme / omento menor
21. En relación al movimiento de
peristaltismo del tubo digestivo: en
la flecha negra del gráfico se
produce la liberación de ………………
a nivel del músculo ………..
(unidad 2, sesión 7, logro 6: Control
hormonal y tipos de movimiento)

a. noradrenalina, sustancia P y neuropéptido “ Y” / circular


b. acetilcolina y sustancia P / longitudinal
c. óxido nítrico y PIV / longitudinal
d. óxido nítrico y PIV / circular

22. Un paciente refiere no percibir algunos sabores, al examen físico se constata alteración en
la percepción de sabores y del dolor en el tercio posterior de la lengua ¿Cuál de los
siguientes nervios estará alterada en su función?
(unidad 2, sesión 10, logro 5: Sabores, tipos y mecanismos moleculares para su detección)

a. Lingual (rama del V par)


b. Cuerda del tímpano (VII par)
c. Glosofaríngeo (IX par)
d. Hipogloso (XII par)

23. El gráfico detalla la estructura de


la pared del tubo digestivo
intestinal ¿Cuál de las siguientes
asociaciones es correcta?
(unidad 2, sesión 7, logro 1: La
pared y músculo liso
gastrointestinal )

a. “1” – peristaltismo
b. “2” – secreción enzimática
c. “3” – deriva del mesodermo
d. “4” – doble hoja de tejido graso

24. En el caso de un paciente con gastrinoma (tumor productor de gastrina), la presencia de


úlceras duodenales y erosión de la mucosa gástrica, se debe principalmente a…….
(unidad 1, sesión 3, logro 2: describir las hormonas gastrointestinales: estímulo y funciones de la
gastrina y colecistoquinina)

a. la acción directa de la gastrina sobre la célula principal


b. la sobre expresión de los receptores “G” en la célula parietal
c. el exceso de HCl por estímulo de receptores CCK-B en la célula parietal
d. el exceso de HCl por estímulo directo de receptores de acetilcolina en la célula parietal

25. El reflejo entero gástrico se caracterizan por:


(unidad 2, sesión 13, logro 6: Motilidad del intestino grueso: contracciones segmentarias,
movimientos en masa defecación y reflejo gastrocólico)

a. favorecer la motilidad gástrica gracias a la CCk


b. inhibir la motilidad gástrica y estimular la secreción ácida
c. movilizar grandes volúmenes desde el estómago al duodeno
d. originarse debido a la distensión duodenal y presencia del quimo ácido
26. Mauricio tiene dificultad para deprimir el paladar y elevar la parte posterior de la lengua. En
este caso estará afectado un músculo ………………., específicamente el músculo …………….
(Unidad 2, sesión 10, logro 2: Músculos de la lengua: clasificación, identificación y sus funciones)

a. intrínseco – longitudinal inferior


b. extrínseco – palatogloso
c. extrínseco – transverso
d. extrínseco – estilogloso

27. Una de las funciones del músculo señalado es:


(Unidad 2, sesión 8, logro 3: Describir el Piso de la boca:
estructuras blandas que la conforman)

a. deprimir la lengua
b. elevar el paladar blando
c. deprimir el hioides cuando la mandíbula está fija
d. deprimir la mandíbula cuando el hioides está fijo

28. Paciente varón de 30 años es evaluado por probable enfermedad de Chagas, cursa con
problemas de motilidad del colon; los estudios de biopsia determinan ausencia de
células ganglionares. Según el gráfico
¿cuál es la capa en la que se determina la ausencia de dichas células?
(unidad 1, sesión 2, logro 1: describir las generalidades de la estructura del tubo digestivo: esófago,
estómago intestino delgado y grueso)

a. Mucosa - 1
b. Muscular propia – 1
c. Muscular de la mucosa - 2
d. Muscular propia - 3

1 2

3
29. Paciente varón de 32 años, que acude a centro de salud por presentar de forma progresiva
desde hace 1 año dificultad para ingerir alimentos sólidos y luego líquidos; refiere
regurgitaciones alimentarias y marcada pérdida de peso (15 kilos). Radiografía baritada
(sustancia de contraste) de esófago se muestra en la figura. El presente caso se explica
por……………….
(Unidad 2, sesión 12, logro 4: Identificar y describir la función de los esfínteres esofágicos)

a. aumento de la peristalsis esofágica


b. relajación incompleta del esfínter pilórico
c. relajación incompleta del esfínter esofágico inferior
d. perdida de producción de PIV y óxido
nítrico en el esfínter esofágico superior

30. En condiciones normales, el ingreso de 600 ml de líquido es el estómago provoca un


aumento de presión intragástrica de unos 12 cm de H2O. Después de una vagotomía (corte
del nervio vago) es de esperar que el ingreso del mismo volumen de líquido ocasionede la
presión intragástrica.
(Unidad 2, sesión 13, logro 1: Describe la Motilidad gástrica: relajación receptiva)

a. la disminución
b. la no variación
c. un aumento mayor
d. un aumento similar o igual
SIS
TEMA
DIGESTIVO
(ME 154)
CLAVES
EXAMEN
FINAL
Ciclo 201900

1. Un niño de 2 años es llevado a la consulta por diarrea persistente, edema de las extremidades y
falta de crecimiento en relación a su edad. Los análisis de sangre revelan que tiene concentración
plasmática baja de proteínas (hipoproteinemia). Como parte del estudio se coloca colecistokinina
(CCK) endovenosa y se recoge muestras del líquido duodenal por endoscopía; el resultado del
líquido confirma incapacidad para hidrolizar proteínas a un pH neutro, esta situación mejora al
añadir una pequeña cantidad de tripsina. El paciente probablemente esté sufriendo la falta
congénita de ………….
(Unidad 4, sesión 26, logro 2: Explicar la Digestión y absorción de las proteínas y sus alteraciones)
a. PEPT-1
b. pepsinógeno
c. enterocinasa
d. carboxipeptidasas

2. Paciente mujer de 35 años acude a consulta por sensación de sequedad y lesiones en cavidad
oral. Al examen se observa atrofia de la mucosa, fisuras y úlceras; nota además sequedad e
irritación de la córnea y aumento del tamaño de las glándulas parotídas. Su diagnóstico más
probable es artritis reumatoide; el hallazgo más probable en una biopsia de glándula parótida
es……..….
(Unidad 3, sesión 18, logro 3: Describe las enfermedades más frecuentes de las glándulas salivales)
a. Presencia de acinos normales con hiperplasia de células ductales
b. Gran infiltración de linfocitos y células plasmáticas
c. Hiperplasia de acinos glandulares serosos
d. Gran infiltrado de linfocitos y neutrófilos

3. Un hombre de 42 años de edad se presenta al médico con una historia de 1 año de evolución,
caracterizado por dolor abdominal bajo y diarreas con crisis sanguinolentas. Manifiesta además
pérdida de peso de 8 kg durante este periodo. La colonoscopía revela lesión difusa en el colon
con afectación del recto. La biopsia de estas lesiones revela adelgazamiento de la pared,
inflamación y ulceración de la mucosa y sub mucosa. El diagnóstico más probable en este caso es:
(Unidad 4, sesión 28, logro 5: Describe la Enfermedad inflamatoria intestinal. Generalidades,
morfología y características)
a. sindrome de colon irritable
b. enfermedad de Crohn
c. colitis ulcerativa
d. sprue celiaco

4. Dos estudiantes deciden tomar un receso para comer una hamburguesa a la hora del almuerzo.
Antes de llegar a la cafetería, impulsos nerviosos provenientes del complejo vagal dorsal iniciarán
la secreción de ácido gástrico por la liberación dedesde el sistema nervioso entérico.
(Unidad 3, sesión 20, logro 2: Regulación de la secreción gástrica: estimulación, fases de la secreción)
a. Serotonina
b. Colecistoquinina
c. Péptido inhibidor vaso activo
d. GRP (péptido liberador de gastrina)

5. Un niño de cuatro años de edad es llevado a la consulta por cuadros diarreicos frecuentes
caracterizados por heces pálidas, voluminosas y fétidas; al examen físico presenta bajo peso
y talla para la edad. Se mide la concentración de cloruro en el sudor y se encuentra que sus
valores son muy elevados. La alteración más
importante a nivel de células ductales del páncreas tiene relación directa con la conductancia de…………
(Unidad 3, sesión 23, logro 5 Explica la Secreción pancreática: formación del jugo
pancreático, influencia de la velocidad de flujo y regulación)
a. Bicarbonato
b. Potasio
c. Sodio
d. Cloro
6. Se evalúa los valores séricos de las siguientes sustancias a un paciente con enfermedad hepática
terminal; en este paciente se espera encontrar la combinación con la letra …………
(Unidad 3, sesión 22, logro 5: Describe las Pruebas de función hepática, la Insuficiencia
hepática, encefalopatía hepática e hipertensión portal)
Glucosa Amoniaco Albúmin
a
a. Aumenta Disminuida Disminui
da da
b. Disminui Aumentada Aumenta
da da
c. Aumenta Aumentada Aumenta
da da
d. Disminui Aumentada Disminui
da da

7. Una mujer de 35 años de edad HIV positiva, se presenta al médico con dolor abdominal en
cuadrante superior derecho e ictericia. La paciente refiere haber tenido múltiples episodios de
ictericia durante los últimos 10 años. Los exámenes para determinar hepatitis viral, dieron
positivos para Hepatitis B, siendo catalogado el caso como hepatitis crónica con alteración
funcional. En un examen de sangre ¿cuál de los siguientes parámetros está disminuido?
(unidad 3, sesión 22, logro 5: Pruebas de función hepática, Insuficiencia hepática,
encefalopatía hepática e hipertensión portal)
a. Albumina
b. Bilirrubina
c. Fosfatasa alcalina
d. Tiempo de protrombina

8. En el reflejo peristáltico del intestino delgado ¿Cuál de los siguientes eventos sucede en la
porción caudal del bolo alimenticio?
(Unidad 2, sesión 13, logro 4: Explicar la Motilidad del intestino delgado: Contracciones
segmentarias y peristálticas)
a. Acción del péptido inhibidor vasoactivo (VIP) en el músculo circular
b. Acción del NO (óxido nítrico) en el músculo longitudinal
c. Contracción del músculo longitudinal interno
d. Acción de acetilcolina en el músculo circular

9. Un varón de 58 años de edad con enfermedad de Crohn severo fue sometido a una resección
ileal. Después de la cirugía este paciente padecerá de esteatorrea, esto se explica porque
…..………..
(unidad 4, sesión 26, logro 4: Explica las alteraciones en la Absorción de lípidos)
a. se inhibe la acción de la 7 alfa hidroxilasa
b. el pool de ácidos biliares se incrementa
c. hay mala absorción de ácidos biliares
d. el páncreas no secreta lipasa

10. En un experimento se inserta un balón en el estómago de un voluntario, se infla poco a poco


mientras que se vigilan las presiones intraluminales. Aunque el volumen del balón aumenta
considerablemente, las presiones permanecen constantes. Esta relación volumen-presión se
explica por la liberación local de …………..
(Unidad 2, sesión 13, logro 1 Explica la Motilidad gástrica: relajación receptiva)
a. acetil colina y gastrina
b. norepinefrina y óxido nítrico
c. colecistoquinina y óxido nítrico
d. óxido nítrico y péptido inhibidor vasoactivo

11. ¿Cuál de las siguientes alternativas es una característica de la secreción exocrina del páncreas?
(Unidad 3, sesión 23, logro 5: Secreción pancreática: formación del jugo pancreático, influencia de la
velocidad de flujo y regulación)
a. Es hipotónica respecto al plasma
b. Su mayor estímulo se da en la fase intestinal
c. Es estimulada por la presencia de bicarbonato en el duodeno
d. La secreción enzimática es estimulada principalmente por la secretina
12. Las estructuras en el hígado que permite que los productos metabólicos unidos a proteínas
tengan acceso a las membranas basolaterales de los hepatocitos, son…..
(Unidad 3, sesión 21, logro 4-5: Explica la Organización micro estructural del hígado)
a. los canalículos
b. las células de Ito
c. las fenestras sinusoidales
d. las uniones intercelulares herméticas

13. La composición de la bilis es modificada conforme fluye por los conductillos biliares.
Durante este tránsito se espera que aumente la concentración de…….
(Unidad 3, sesión 22, logro 2: Describe la Secreción biliar, visión general del sistema biliar
extrahepático y composición de la bilis)
a. Ig A
b. Glucosa
c. Protones
d. Vitamina A

14. Se mide experimentalmente el contenido gástrico de dos personas. La persona “A” tiene alto
contenido de grasa y la persona “B” tiene un contenido isotónico ¿Cuál de las siguientes es
correcto respecto al vaciamiento gástrico? (Unidad 2, sesión 13, logro 2: Describe la Motilidad y
vaciamiento gástrico)
a. Hay ralentización del vaciado gástrico solo en “A”
b. El vaciamiento gástrico es más rápido en ambos
c. Hay ralentización del vaciado gástrico solo en “B”
d. Hay ralentización del vaciado gástrico en ambos casos

15. El examen endoscópico de un paciente con hipertensión portal grave revela venas tortuosas que
sobresalen hacia la luz del esófago. El paciente recibe tratamiento quirúrgico mediante la
colocación de una derivación que conecta la vena porta a la vena cava. Después de la operación
el riesgo de encefalopatíay el riesgo
de sangrado de várices ……………..
(Unidad 3, sesión 22, logro 5: Describe la Insuficiencia hepática, encefalopatía hepática e hipertensión
portal)
a. disminuirá / disminuirá
b. disminuirá / aumentará
c. aumentará / disminuirá
d. aumentará / aumentará

16. Un paciente varón de 18 años de edad acude al médico para sus exámenes de rutina. Sus
resultados de laboratorio muestran un valor de bilirrubina sérica de 4 mg/dl y una bilirrubina
directa de 0,3 mg/dl. Las pruebas de función hepática son normales. La alteración que explica
mejor este caso es por la deficiencia de ………………..
(Unidad 3, sesión 22, logro 3: Explica la Producción y excreción de bilirrubina. Tipos de bilirrubina e
ictericia)
a. transaminasas
b. hemo oxigenasa
c. la 7 alfa hidroxilasa
d. glucuronil transferasa

17. Un hombre de 57 años de edad es llevado a urgencias con hematemesis masiva rojo brillante, a
su llegada se halla inconsciente con PA: 80/40 mm Hg y FC: 124 lat/min. Luce ictérico con
presencia de “arañas vasculares en el tórax anterior y extremidades”, abdomen distendido con
signo de oleada positiva. Se encuentra esplenomegalia y pérdida de la masa muscular en
extremidades. La anastomosis vascular responsable del sangrado en este paciente es ………….…..
(Unidad 3, sesión 21, logro 2: Describe las anastomosis porto sistémicas)
a. vena gástrica izquierda y vena ácigos
b. arteria gástrica izquierda y vena ácigos
c. vena paraumbilical y vena epigástrica inferior
d. vena gástrica izquierda y vena esofágica superior
18. Un estudiante de medicina está comiendo un plato de comida a base de champiñones,
espárrago y salsa de soya. El estímulo del sabor umami contenido en todos estos alimentos viaja
a través del nervio………………..
(Unidad 2, sesión 10, logro 3: Describe la irrigación e inervación de la lengua)
a. Lingual
b. Hipogloso
c. Glosofaringeo
d. Cuerda del tímpano
19. Una paciente de 30 años de edad es sometida a una cirugía en oído medio derecho por un
problema de otoesclerosis. Luego de la cirugía refiere alteración sensitiva de la lengua. Al
evaluar el caso usted esperaría encontrar……….
(Unidad 2, sesión 10, logro 5: Describe la Irrigación e inervación de la lengua)
a. Alteración en la sensación del dolor y temperatura en el tercio posterior de la lengua
b. Alteración en la sensación del dolor en los dos tercios anteriores de la lengua
c. Alteración en la sensación del gusto en el tercio posterior de la lengua
d. Sensación del dolor, tacto y temperatura conservadas

20. En un paciente de 45 años de edad con colestasis biliar, se encuentra una elevación de los niveles
sanguíneos de fosfatasa alcalina hasta 3 veces la cifra normal. ¿Cuál de las siguientes alternativas
estará también elevada como evidencia del daño de la vía biliar?
(Unidad 3, sesión 22, logro 5: Pruebas de función hepática, Insuficiencia hepática,
encefalopatía hepática e hipertensión portal)
a. Tiempo de protrombina y albúmina sérica
b. Transaminasas hepáticas (ALT y AST)
c. Gamma glutamil transpeptidasa
d. Glucoronil transferasa

21. Experimentalmente se incrementa la velocidad de la secreción salival con una sustancia,


en el análisis de la composición de esta saliva obtenida se espera encontrar…………..
(Unidad 3, sesión 17, logro 5 : Explica la Influencia de la velocidad del flujo salival en la composición de la
saliva)
a. disminución de la concentración de bicarbonato que supera la concentración plasmática
b. aumento de la concentración de cloro y sodio que supera la concentración plasmática
c. aumento de la concentración de bicarbonato que supera la concentración plasmática
d. disminución de concentración de potasio y bicarbonato

22. Lactante de 3 meses de vida es atendido por presentar diarrea, se administra una
solución de glucosa y electrólitos por vía oral. La proteína de membrana apical que
explica la capacidad de esta solución para proporcionar aporte de glucosa e hidratación
es ………..
(Unidad 4, sesión 26, logro 1: Explica la Digestión y Absorción de los hidratos de carbono. Alteraciones)
a. CFTR
b. SGLT-1
c. GLUT-2
d. GLUT-5

23. Paciente ha sufrido herida de bala en el abdomen, se le ha tenido que extirpar el segmento
medio y distal del ileon. En este caso la síntesis hepática de sales biliares estará …..…..
(Unidad 3, sesión 22, logro 4: Explica la formación, función y Circulación entero hepática de lasa sales
biliares)
a. Sin cambios en el ritmo de síntesis
b. Disminuida por inhibición de la enzima colesterol 7 alfa hidroxilasa
c. Incrementada por estímulo de la enzima colesterol 7 alfa hidroxilasa
d. Incrementada por inhibición de la enzima colesterol 7 alfa hidroxilasa

24. Un varón de 75 años ingresa al consultorio por presentar ictericia marcada de piel y las
escleras. El estudio del paciente mostró que presentaba un tumor que obstruía la totalidad del
conducto hepático común. ¿Cuál de los siguientes conductos se encontrará dilatado en este
paciente?
(Unidad 3, sesión 21, logro 6: Describir el árbol biliar intrahepático)
a. de Wirsung
b. de Hering
c. colédoco
d. cístico
25. Correlaciones las dos columnas y marque la fórmula correcta:
(Unidad 4, sesión 28, logro 1: Diarrea: definición, mecanismos: osmótica, secretoria y exudativa)

1. Enfermedad Hirschsprung( ) heces con moco y sangre


2. Diarrea osmótica( ) intolerancia a lactosa
3. Diarrea secretoria( ) aganglionosis congénita
4. Diarrea exudativa( ) canales de Cl- en las células de la cripta a.- 4231b.-

1234c.- 2143d.- 4213

26. La vena umbilical obliterada del hígado después del nacimiento se transforma en el ligamento:
(Unidad 3, sesión 21, logro 1: Hígado: relación con la pared abdominal, caras, lóbulos, ligamentos , hilio
hepático)
a. cruzado
b. redondo
c. coronario
d. falciforme

27. Llega a su guardia nocturna una madre que trae a su RN masculino de 2 semanas de vida con mal
estado general y sequedad de mucosas. Usted observa que lacta ávidamente, pero a las 2 horas
presenta vómito postprandial no bilioso en proyectil. Al realizar la historia clínica, descubre que el
lactante recibió profilaxis con macrólidos para tos ferina. Usted sospecha principalmente en:
(Unidad 1, sesión 4, logro 4: Desarrollo y anomalías del intestino anterior)
a. estenosis pilórica hipertrófica congénita
b. fistula traqueo esofágica
c. estenosis duodenal
d. atresia duodenal

28. En la regulación del apetito y la saciedad, la estimulación experimental crónica del núcleo
ventro medial del hipotálamo producirá:
(Unidad 1, sesión 3, logro 4: explica los mecanismos de control del apetito y saciedad)
a. afagia
b. obesidad
c. hiperfagia
d. activación de neuronas relacionadas a NPY

29. Paciente mujer de 25 años acude por dolor en fosa ilíaca derecha que empeora al toser o
caminar, asociada a náuseas y vómitos por lo cual acude a emergencia. Dos días después de
realizarle una apendicectomía, la paciente desarrolla fiebre alta (39 °C), está hipotensa y presenta
dolor abdominal. La laparotomía exploratoria muestra un gran volumen de sangre en la cavidad
peritoneal por lesión de un vaso producida durante la apendicectomía.
¿Cuál de las siguientes arterias debe ligarse para detener la hemorragia?
(Unidad 4, sesión 27, logro 4: Irrigación arterial del colon, recto y conducto anal)
a. cólica derecha y arteria rectal superior.
b. ileocólica y arteria cólica media.
c. mesentérica superior.
d. ileocólica.

30. La onda peristáltica secundaria del esófago se caracteriza por ser originada ………
(unidad 2, sesión 12, logro 3: Motilidad esofágica: fases y características)
a. por el plexo de meissner del esófago
b. por el plexo mientérico del esófago
c. por el reflejo de la deglución
d. durante la masticación

31. ¿Cuál de los siguientes es una causa de ictericia con bilirrubina conjugada aumentada?
(Unidad 3, sesión 22, logro 3: Producción y excreción de bilirrubina. Tipos de bilirrubina, ictericia)
a. Ictericia del recién nacido
b. Obstrucción del colédoco
c. Anemia hemolítica
d. Gran hematoma
32. En relación a la absorción de nutrientes, la absorción de dipéptidos y tripéptidos a nivel de las
células epiteliales del intestino delgado, se da principalmente debido a:
(Unidad 4, sesión 26, logro 2: Digestión y absorción de las proteínas. Alteraciones)
a. el incremento de los canales de Cl- en la membrana apical
b. la gradiente de bicarbonato en la membrana basal
c. la gradiente de iones H+ en la membrana apical
d. la gradiente de Na+ en la membrana apical

33. Paciente de 20 años es traído a la emergencia por presentar diarreas desde hace 2 días. Familiar
refiere que las deposiciones son líquidas y abundantes, al examen luce deshidratado y se plantea
que la diarrea es producida por una toxina que estimula la transformación de ATP a AMPc con
apertura de canales de Cl- y pérdida de agua. El tipo de diarrea más probable es:
(Unidad 4, sesión 28, logro 1: Diarrea: definición , mecanismos: osmótica, secretoria y supurativa)
a. osmótica
b. exudativa
c. secretoria
d. por intolerancia a lactosa

34. Un niño fue operado por una obstrucción intestinal, observándose la presencia de divertículo de
Meckel. Según lo referido, marque lo correcto:
(Unidad 1, sesión 5, logro 2: identifica las anomalías del desarrollo del intestino medio: onfalocele y
gastrosquisis (diferencias), Divertículo de Meckel)
a. el 50% de la población lo presenta
b. se localiza en el íleon muy cerca al yeyuno
c. puede poseer tejido gástrico o pancreático
d. se produce por una mala rotación de los intestinos

35. Marque la alternativa


correcta respecto a la
estructura marcada en el
gráfico:
(Unidad 3, sesión 22, logro 2:
Secreción biliar. Visión general
del sistema biliar extrahepático
y composición de la bilis)

a. Se halla a 2
centímetros debajo
de la papila
duodenal mayor
b. Llega el conducto
colédoco y pancreático
principal
c. Llega el conducto
hepático común y
pancreático
principal
d. Llega el conducto pancreático accesorio

36. ¿Cuál de las siguientes moléculas se encontrará aumentada en el citoplasma de las células
parietales de un paciente con sindrome de Zollinguer Ellison?
(Unidad 3, sesión 20, logro 4: Enfermedad ulcerosa péptica: úlcera gástrica, duodenal.
síndrome de Zollinger – Ellison)
a. Péptido liberador de gastrina (GRP)
b. Proteína G estimulante (GS)
c. Inositol Trifosfato (IP3)
d. AMP cíclico (AMPc)

37. Los fármacos inhibidores de la bomba de protones, actúan bloqueando la ………..……..


(Unidad 3, sesión 20, logro 3: Regulación de la secreción gástrica: inhibición, Secreción de
pepsinógeno y factor intrínseco)
a. anhidrasa carbónica
b. ATPasa H+/K+ en la membrana luminal
c. ATPasa H+/K+ en la membrana basolateral
d. ATPasa Na+/K+ en la membrana basolateral
38. Un paciente fue diagnosticado de gastritis autoinmune, ¿cuál de las siguientes alternativas es
FALSA respecto a esta enfermedad?
(Unidad 3, sesión 20, logro 5: Gastritis crónica: helicobacter pylori, autoinmune. Tipos de gastritis)
a. Afecta principalmente el fondo y cuerpo gástrico
b. Se produce hiperplasia de células G secundaria a la aclorhidria
c. El propio sistema inmune destruye principalmente las células parietales
d. Se produce atrofia de la mucosa, aclorhidria, hipergastrinemia y déficit de vitamina B6

39. Marque la correlación correcta:


(Unidad 3, sesión:18, logros:1 y 2: Describe las enfermedades inflamatorias/infecciosas y
proliferativas de la cavidad oral)

1. Herpes virus( ) En relación al abuso de antibióticos


2. Candidiasis oral( ) Lesiones vesiculares como racimo de uvas
3. Eritroplaquia( ) Mega esófago
4. Enfermedad de Chagas( ) Lesión pre cancerígena

a.- 2431b.- 1234c.- 4123d.- 2143

40. En un paciente con insuficiencia renal crónica, el déficit en la absorción de calcio a nivel del
enterocito se debe a lo siguiente:
(Unidad 4, sesión 26, logro 6: Explica la Absorción de calcio y hierro)
a. No se convierte la 25 hidroxicolecalciferol a 1,25 dihidroxicolecalciferol
b. No se convierte la 1,25 dihidroxicolecalciferol a 25 hidroxicolecalciferol
c. Existe un descenso de la alfa 25 hidroxilasa renal
d. Se incrementa la producción de Calbindina
SISTEMA DIGESTIVO
(ME154) EXAMEN
PARCIAL 2019 01

Profesores : Alfaro Salazar, Herberth Romulo; Callata Caceres, Gunter; Cayo Quiñe, Alexandra Mariel; Correa
Borit, Jorge Mauricio; Cruz Cutty, Lourdes Marylin; Guzmán Calderón, Gerly Edson; Jáuregui
Farfán, Jorge Jesús; Mayor Zevallos, Otto Alberto; Montoya Suárez, José Luis; Palacios Bazan,
Enrique Elias; Robles Pino, Alexander Anibal; Wong Bravo, Juan Carlos
Sección : Todas las secciones
Duración : 50 minutos.
Indicaciones:
Lea atentamente cada pregunta antes de responder:
- Se prohíbe el uso del celular y cualquier dispositivo electrónico.
- Está prohibido intercambiar materiales.
- Coloque su código de alumno en la tarjeta de respuestas. Si su código contiene una letra reemplácela por
un valor numérico siguiendo la siguiente equivalencia: A=9, B=8, C=7, D=6, E=5, F=4, G=3.
- Traslade sus respuestas a la tarjeta, llenando los círculos de manera completa con lapicero negro o azul.
Está prohibido el llenado con lápiz, lapicero de otro color o con lapicero de tinta borrable.
- Sea cuidadoso en el llenado de la tarjeta de respuestas, pues solo esta tiene validez para la calificación.
- Al terminar su examen avise al docente a cargo, no se levante de su sitio; debe entregar la hoja de
respuestas con la carátula del examen, este cuadernillo de preguntas se lo llevará cada estudiante.

1. ¿Cuál de las glándulas salivales es responsable del mayor porcentaje del volumen de la saliva en
condiciones basales?
a. Parótida
b. Subpalatin
as c.
Sublingula
es d.
Submaxilar
es

2. La lengua está recubierta por epitelio:


a. pseudoestratificado columnar no queratinizado
b. plano estratificado no queratinizado
c. pseudoestratificado columnar ciliado
d. plano estratificado queratinizado

3. El esfínter anal interno tiene musculatura …………… y tiene control ………………….


a. lisa / voluntario
b. lisa / involuntario
c. esquelética / simpático
d. esquelética / parasimpático

4. La arteria aorta proporciona la irrigación al tubo digestivo ¿cuál de las siguientes arterias proporciona la
irrigación al ángulo cólico derecho?
a. Mesentérica superior
b. Mesentérica inferior
c. Frénica inferior
d. Tronco celiaco

5. Paciente de 26 años que le cuenta en su historia clínica que cada vez que almuerza, a los 20 minutos
tiene deseo de defecar. Le comenta que su hijo de 1 mes le pasa lo mismo pero más intenso. Esto se
explica por el reflejo
…………………, el cual estáen el paciente.
a. colicoileal / normal
b. colicoileal / alterado
c. gastrocolico / normal
d. gastrocolico / alterado
6. La región del estómago que se comunica con el duodeno se denomina:
a. pilórica
b. cardias
c. cuerpo
d. fórnix

7. Acude a consulta un paciente que fue diagnosticado de ulcera péptica 3 días antes. Luego de múltiples
pruebas diagnósticas, se concluye que el paciente presenta un tumor secretor de gastrina ¿Cuál de las
siguientes situaciones estará incrementada?
a. Distención gástrica
b. Inhibición del vaciado gástrico
c. Secreción de ácido clorhídrico (HCl)
d. Inhibición de la secreción de pepsinógeno

8. En el sistema digestivo, el control del apetito esta dado por un complejo sistema de sustancias y órganos
integradores los cuales regulan la ingesta de alimentos. Laes una sustancia oroxígena y es sintetizada
por el ……………………..
a. leptina / intestino
b. grelina / intestino
c. leptina / estómago
d. grelina / estómago

9. Sobre el control autónomo del sistema digestivo, marque la alternativa correcta:


a. La inervación dada por el sistema simpático es de tipo preganglionar.
b. El sistema parasimpático usa como neurotransmisores a la acetilcolina y la noradrenalina.
c. El nervio vago (par craneal X) le da inervación simpática a la mayoría del sistema digestivo.
d. En el sistema simpático, los nervios responsables hacen una primera sinapsis en ganglios
próximos al órgano a inervar.
e. En la inervación de tipo parasimpático, solo interviene el plexo submucoso, sin embargo, en la
de tipo simpático intervienen tanto el submucoso como el mientérico.

10. Con respecto a la actividad eléctrica del sistema digestivo, marque la alternativa correcta
a. Corresponden a potenciales de acción que están presentes de forma continua y le dan la
capacidad de perístasis autónoma al sistema digestivo.
b. La frecuencia de las ondas lentas no se ve influenciada por la actividad neural ni las
hormas gastrointestinales.
c. En el estómago las ondas lentas se dan en una frecuencia de 6 por minuto.
d. Las ondas lentas son cambios lentos y ondulantes del potencial en reposo.
e. La frecuencia de las ondas lentas va de 6 a 12 ondas por minuto.

11. Ante una lesión del IX par craneal, el músculo…se altera en su función.
a. palatogloso
b. estilofaríngeo
c. palatofaríngeo
d. constrictor superior

12. Un varón de 50 años es sometido a extirpación de duodeno y parte proximal de yeyuno. La pérdida de
estímulo hormonal en el páncreas para la secreción enzimática se explica por la pérdida de las células
……………………
a. Parietales, productoras de factor intrínseco
b. “K” productoras de factor intrínseco
c. “M” productoras de CCK
d. “I” productoras de CCK
13. Respecto al mecanismo de la defecación ¿Cuál de las siguientes afirmaciones es correcta?
a. Se produce contracción refleja del esfínter anal interno
b. Se produce contracción o relajación del esfínter anal externo por señales de la corteza cerebral
c. La presencia de materia fecal en el recto estimula la contracción del sigmoides por los nervios
pélvicos simpáticos
d. En la posición de “cuclillas” el músculo puborectal se halla contraído favoreciendo la
evacuación de la materia fecal

14. Un niño de tres años llega a emergencia con disfagia (dificultad para tragar), salivación y llanto. Se sospecha
de ingesta de cuerpo extraño: moneda en el esófago; al ser evaluado se constata en una radiografía presencia
de cuerpo extraño a nivel de C6 y C7 (6° y 7° vértebra cervical). El cuerpo extraño estará suspendido a nivel
del estrechamiento producido por el ………..
a. cayado aórtico
b. hiato esofágico
c. músculo cricofaríngeo
d. bronquio principal izquierdo

15. En el caso de un paciente con un tumor productor de gastrina, la presencia de úlceras duodenales y
erosión de la mucosa gástrica se debe principalmente a…….
a. la acción paracrina de la gastrina sobre la célula parietal
b. el exceso de HCl por estímulo de receptores CCK-B en la célula parietal
c. la sobre expresión de los receptores “G” para gastrina en la célula parietal
d. el exceso de HCl por estímulo directo de receptores “H” en la célula parietal

16. La onda peristáltica secundaria del esófago se caracteriza por ser originada ………
a. por el plexo de submucoso del esófago
b. por el plexo mientérico del esófago
c. por el reflejo de la deglución
d. durante la masticación

17. Marque lo correcto sobre las ondas lentas en el tubo digestivo


a. No son despolarizaciones
b. Son potenciales de acción subumbrales
c. Se constituyen de despolarizaciones y repolarizaciones
d. Son rítmicas y generadas por el sistema nerviosos autónomo

18. Recién nacido que presenta protrusión de contenidos abdominales los cuales no están cubiertos por
peritoneo y salen de la cavidad abdominal a través de un defecto de la pared. ¿Cómo se denomina a la
afección que presenta este paciente?
a. Onfalocele
b. Atresia biliar
c. Gastrosquisis
d. Divertículo de Meckel

19. Experimentalmente se utiliza atropina (anticolinérgico) para inhibir la secreción de gastrina, sin
embargo, la secreción de esta hormona se sigue dando ante estímulos vagales. Esta situación se explica
porque la atropina:
a. no bloquea la acción del péptido GRP
b. solo inhibe la acción del péptido GRP en la célula G
c. inhibe la acción de acetilcolina e histamina en la célula G
d. bloquea parcialmente la bomba de protones en la célula G

20. Niña de 4 días es llevada a la emergencia pediátrica por presentar llanto constante, la madre refiere
coloración azulada de labios al momento de lactar, acompañado de tos persistente y dificultad respiratoria
así como distención abdominal. Se le coloca sonda nasogástrica para alimentación notando que retorna a
la cavidad oral en todos los intentos. ¿Cuál es la anomalía del desarrollo en este caso?
a. Solo fístula traqueo esofágica
b. Fístula traqueo esofágica proximal y distal
c. Atresia esofágica proximal con fístula traqueo esofágica distal
d. Atresia esofágica distal con fístula traqueo esofágica proximal
21. Paciente varón de 36 años es traído a la emergencia luego de sufrir un accidente de tránsito, presenta
traumatismos múltiples en cabeza y tronco. Al examen físico se evidencia hematoma en hemicara izquierda,
ligera protrusión y caída del lado izquierdo del maxilar inferior, por lo que se realiza una tomografía donde
se halla una fractura de la apófisis coronoides del maxilar inferior. ¿Qué músculo está relacionada
directamente con esta situación?
a. Masetero
b. Temporal
c. Buccinador
d. Pterigoideo medial

22. Un paciente refiere no percibir algunos sabores. Al examen físico constata alteración del sabor dulce y
umami.
¿Cuál de los siguientes nervios estará alterada su función?
a. Cuerda del tímpano (VII par)
b. Lingual (rama del V par)
c. Glosofaríngeo (IX par)
d. Hipogloso (XII par)

23. A los pocos días de nacido, regresa a neonatología un niño con problemas de motilidad del colon; los
estudios determinan ausencia congénita de células ganglionares. Según el gráfico ¿cuál es la capa en la que
se determina la ausencia de dichas células?
a. Mucosa - 2
b. Muscular propia - 2
c. Muscular propia - 3
d. Muscular de la mucosa - 3

1 2

24. Con respecto al control autonómico en el tracto gastrointestinal y en relación a su fisiología. ¿Cuál es la
función del sistema nervioso parasimpático en el tracto gastrointestinal?
a. Inhiben la contracción muscular y estimulan la secreción de sustancias a nivel de la submucosa
b. Estimulan la contracción muscular y estimulan la secreción de sustancias a nivel de la mucosa
c. Inhiben la contracción muscular e inhiben la secreción de sustancias a nivel de la submucosa
d. Estimulan la contracción muscular e inhiben la secreción de sustancias a nivel de la mucosa

25. Un estudiante que está preocupado por su examen parcial, no ha desayunado ni almorzado; cuando al fin
ingiere alimentos, esto le provoca el aumento de los movimientos musculares del tracto gastrointestinal y
la sensación de defecar. ¿Qué reflejo se ha activado?
a. Entero-gástrico
b. Gastro-cólico
c. Cólico-ileal
d. Ileo-ileal
26. ¿De qué par craneal es rama el nervio palatino mayor?
a. Vago
b. Hipogloso
c. Trigémino
d. Palatogloso

27. ¿En cuál de las fases de la deglución la epiglotis separa la vía respiratoria de la digestiva?
a. oral
b. laríngea
c. faríngea
d. esofágica

28. Los péptidos intestinales se pueden clasificar como sustancias endocrinas, neurocrinas y paracrinas,
dentro de las paracrinas se encuentran la somastotatina e histamina. Marque la respuesta correcta
a. La somastotatina es sintetizada por las células B de la mucosa gástrica
b. La histamina actúa estimulando su receptor tipo H1 en la mucosa gástrica
c. La histamina es sintetizada por células de tipo paracrino de las glándulas gástricas
d. La somatostatina presenta dentro de sus funciones la estimulación de la secreción de H +

29. En relación a los órganos intraabdominales y sus estructuras de fijación, elija la alternativa
correcta a.El mesenterio permite la suspensión e irrigación de los órganos
retroperitoneales
b. Tanto el hígado como la vesícula biliar se encuentran ubicados a nivel del flanco derecho
c. El colon, el duodeno y el resto de intestino delgado son órganos considerados
netamente como peritoneales
d. Los ligamentos que encontramos dentro de la cavidad abdominal son el esplenorenal y el
gastrofrénico e.Los omentos van desde el estómago y la segunda porción del duodeno a otras
estructuras
intraabdominales y existen dos: el omento mayor y el omento menor

30. Paciente varón de 27 años es llevado por bomberos a emergencia luego de ser asaltado y, tras resistirse, es
cortado con el pico de una botella a nivel abdominal. Al examen físico usted observa que a través de la herida
se puede observar la protrusión de asas intestinales. En relación con las capas de la pared abdominal, marque
la alternativa correcta.
a. La fascia de Scarpa está constituida principalmente por tejido adiposo
b. La pared abdominal está formada por piel, huesos, músculos, fascias y peritoneo parietal
c. La fascia de Camper es una estructura fibrosa que carece de grasa y su grosor es constante en
toda la pared abdominal
d. El músculo oblicuo externo discurre en dirección súpero-interna y se inserta en el borde
inferior de las ultimas 3 a 4 costillas
e. El músculo recto del abdomen tiene como funciones comprimir el contenido del abdomen,
tensar la pared del abdomen y flexionar la columna
SISTEMA DIGESTIVO
(ME154) EXAMEN
FINAL 2019 01

Profesores : Alfaro Salazar, Herberth Romulo; Callata Caceres, Gunter; Cayo Quiñe, Alexandra Mariel; Correa
Borit, Jorge Mauricio; Cruz Cutty, Lourdes Marylin; Guzmán Calderón, Gerly Edson; Jáuregui
Farfán, Jorge Jesús; Mayor Zevallos, Otto Alberto; Montoya Suárez, José Luis; Palacios Bazan,
Enrique Elias; Robles Pino, Alexander Anibal; Wong Bravo, Juan Carlos
Sección : Todas las secciones
Duración : 70 minutos.
Indicaciones:
Lea atentamente cada pregunta antes de responder:
- Se prohíbe el uso del celular y cualquier dispositivo electrónico.
- Está prohibido intercambiar materiales.
- Coloque su código de alumno en la tarjeta de respuestas. Si su código contiene una letra reemplácela por
un valor numérico siguiendo la siguiente equivalencia: A=9, B=8, C=7, D=6, E=5, F=4, G=3.
- Traslade sus respuestas a la tarjeta, llenando los círculos de manera completa con lapicero negro o azul.
Está prohibido el llenado con lápiz, lapicero de otro color o con lapicero de tinta borrable.
- Sea cuidadoso en el llenado de la tarjeta de respuestas, pues solo esta tiene validez para la calificación.
- Al terminar su examen avise al docente a cargo, no se levante de su sitio; debe entregar la hoja de
respuestas con la carátula del examen, este cuadernillo de preguntas se lo llevará cada estudiante.

1. La explicación fisiológica de presentar somnolencia de 30 minutos a 1 hora después de ingerir


alimentos, se explica por:
a. Aumento del cloro intraluminal
b. Aumento del bicarbonato intraluminal
c. Disminución de ácido carbónico en la célula parietal
d. Disminución de la actividad de la anhidrasa carbónica
e. Aumento de la alcalinidad sanguínea

2. Con respecto a la irrigación arterial del colon, a que arteria corresponde la


señalada con la flecha
a. Cólica derecha
b. Cólica media
c. Cólica izquierda
d. Ileobisecoapendículocólica
e. Arco de Riolano

3. Si un paciente presentara dentro del punto de vista fisiológico, una


disminución de enterocinasa, entonces esto originaría una disminución de
la actividad de:
a. la
pepsina
b.la
lipasa
c. la quimotripsina
d. el peptido insulinotropo dependiente de glucosa
e. la amilasa

Se valida la opción b debido a su relación con la colipasa.


4. Con respecto a la anatomía del hígado, señale a que estructura
pertenece la marcada por el número 1.
a. Ligamento falciforme
b. Línea de Cantlie
c. Ligamento triangular
d. Ligamento coronario
e. Ligamento teres

Se valida la opción a debido a la ubicación del número 1 en donde se


unen el ligamento falciforme y ligamento coronario.

5. Se presenta un paciente, el cual presenta un antecedente de tuberculosis intestinal, por lo cual, se le


resecó 80 cm de íleon distal. Desde el punto de vista fisiológico, el paciente puede presentar una de las
siguientes alteraciones:
a. Disminución de la secreción de Vitamina B12
b. Aumento indiscriminado de absorción de ácido fólico
c. Disminución de la absorción de hierro
d. Aumento de la secreción de bicarbonato
e. Disminución de la absorción de ácido glicocólico

6. Un paciente es sometido experimentalmente a un fármaco que modifica el flujo salival, obteniéndose un


volumen de saliva de 288 ml en 6 horas. En este caso las concentraciones de electrolitos y bicarbonato en la
saliva obtenida varían de la siguiente manera:
a.↑ Na+, ↓ K+, ↑ Cl-, ↑
HCO3- b.↓ Na+, ↓ Cl-, ↑
K+, ↓ HCO3-
c.↑ Na+, ↑ Cl-, ↓ K+, ↓ HCO3-
d.↑ Na+, ↑ Cl-, ↑ K+, ↑ HCO3-
e.↓ Na+, ↓ Cl-, ↓ K+, ↓ HCO3-

Se valida la opción a debido a que se puede considerar como un aumento del flujo de saliva.

7. La siguiente imagen histológica corresponde a la glándula


…………… y la estructura señalada produce ………
a. salival sublingual / mucopolisacáridos
b. oxíntica / pepsinógeno
c. salival submaxilar / ptialina
d. salival parótida / amilasas
e. antrales / gastrina

8. Paciente varón de 65 años con antecedente de hipercolesterolemia, hipertensión arterial, fibrilación


auricular y dos infartos al miocardio previos, aqueja de dolor abdominal intenso de inicio súbito,
distensión abdominal, se decide cirugía con resección de 1,5 metros de intestino delgado terminal y colon
ascendente. Como consecuencia de la resección el paciente tendrá deficiencia de:
a. Vitamina C
b. Tiamina
c. Vitamina A
d. Vitamina B1
e. Vitamina B6

Se valida esta opción debido a que su absorción está relacionada al íleon.


9. Uno de los siguientes elementos debería hallarse con más probabilidad en el esófago de un paciente que
sufre de
reflujo gastro esofágico…
a. Pepsina
b. Tripsina
c. Quimiotripsina
d. Carboxipeptidasa
e. Ácidos biliares

10. Un paciente de 40 años cursa con anemia de 8g/dl, aqueja además de astenia y sensación de hormigueo
bilateral en los miembros inferiores, al examen se halla alteración de la sensibilidad a la vibración y camina
con ampliación de la base de sustentación. Uno de los siguientes procedimientos sería de ayuda para el
diagnóstico de este paciente:
a. Tomografía cerebral
b. Biopsia de la mucosa gástrica
c. Biopsia de hígado
d. Examen de sangre oculta en heces
e. Biopsia de Ileon proximal

11. Paciente de 60 años ingresa por caída hace 1 hora y pequeño hematoma en cuero cabelludo, al examen
físico ampliado se observa ictericia de piel y mucosas generalizada, abdomen blando, se palpa estructura
quística no dolorosa en hipocondrio derecho que corresponde a vesícula biliar (signo de Courvoisier), en
los exámenes de laboratorio se halla niveles bajos en la formación de estercobilinógeno y urobilinógeno
en heces, incremento de la bilirrubina conjugada en la orina, elevación de fosfatasa alcalina y gamma
glutamil transpeptidasa séricas. El presente cuadro puede ser explicado por:
a. Reabsorción de hematoma
b. Litiasis vesicular
c. Carcinoma de la cabeza de páncreas
d. Carcinoma con estenosis del conducto hepático común
e. Anemia hemolítica

12. Paciente varón de 58 años con antecedente de alcoholismo crónico es diagnosticado y recibe tratamiento
por cirrosis hepática. Hace 2 días refiere familiar que tuvo cambio de conducta y no reconoce a algunos
familiares. Al examen físico, se halla ascitis, circulación colateral en abdomen, telangiectasias, en el
examen de sistema nervioso: rigidez de extremidades, ROT incrementados, desorientación en el espacio y
asterixis. ¿cuál de las siguientes circunstancias, explicaría el cuadro en este paciente?
a. Uso de diuréticos ahorradores de potasio
b. Incremento de actividad de ureasa bacteriana duodenal
c. Hemorragia gastrointestinal
d. Disminución de la producción de NH3+ en el colon
e. Dieta normo proteica

Se valida la opción e debido al efecto sobre la encefalopatía.


Con respecto de la opción b es incorrecta debido a que hace referencia al duodeno, debería indicar colon.

13. Un recién nacido presenta vómitos biliosos poco tiempo después de cada alimento. Al preguntar a la
madre sobre antecedentes, ella recuerda que tuvo polihidramnios durante la gestación, pero un análisis de
cariotipo fue normal. Una de las siguientes es la causa más probable de estos hallazgos en el recién nacido:
a. Enfermedad de Hirschprung
b. Fístula tráqueo esofágica
c. Divertículo ileal
d. Estenosis pilórica
e. Malrotación de la yema pancreática ventral

14. Un lobulillo hepático se puede dividir en tres zonas como se muestra en el gráfico. ¿Cuál de las
siguientes afirmaciones sobre las tres zonas es verdadera?
a. La zona 1 tiene los menores depósitos de glucógeno
b. La zona 3 es la primera en afectarse en una colestasis extra hepática
c. La zona 2 es más susceptible a la injuria por isquemia que la zona
periportal d.La zona 2 tiene la mayor capacidad de regeneración
e.La zona 1 es la que tiene menos actividad metabólica.

La pregunta 14 ha sido anulada, sin embargo, ningún estudiante se verá afectado


negativamente en su puntaje debido a esta anulación.

15. En un estudio de la secreción de hormonas gastrointestinales, sus concentraciones en la vena porta se


midieron durante perfusión luminal del intestino delgado con soluciones de diversas magnitudes de pH.
¿Qué hormona aumentará en el plasma de la vena porta durante perfusión a través del intestino con
una solución de pH 3?
a. CCK
b. gastrina
c. GIP
d. motilina
e. secretina

16. Paciente de 30 años que ingresa a causa de un traumatismo abdominal cerrado. En la exploración se
aprecia discreta palidez de piel y mucosas, auscultación pulmonar normal, taquicardia de 120 /min.
Discreta distensión abdominal y matidez en flancos; el hematocrito, que era prácticamente normal al
ingreso, disminuye a 30% a las tres horas. En la Rx de tórax se objetiva fractura de las costillas 10-11
izquierdas. La causa más probable de la anemización en este paciente es:
a. traumatismo renal con hemorragia retroperitoneal.
b. rotura de hígado con hemoperitoneo.
c. rotura de bazo con hemoperitoneo.
d. rotura de mesos con hemoperitoneo.
e. traumatismo pancreático con pancreatitis traumática.

17. Mujer de 65 años. Consulta por síndrome constitucional


asociado a dolor abdominal epigástrico progresivo irradiado a
espalda, de dos meses de evolución. El diagnostico de sospecha
de adenocarcinoma de páncreas se confirma por biopsia. Se
realiza examen de imagen de abdomen para evaluación de
estructuras vasculares próximas al tumor pancreático. ¿Cuál es
el nombre de la vena señalada que está ausente, trombosada
por infiltración tumoral, condicionando circulación colateral en
la pared gástrica?

a. Mesentérica superior
b. Coronaria estomaquica
c. Esplénica
d. Porta
e. Renal izquierda
18. Revisando la angiotomografía de un hombre de 70 años en estudio por aneurisma de aorta abdominal,
el radiólogo le informa de la presencia de una oclusión completa de la arteria mesentérica inferior. El
paciente se encuentra completamente asintomático. La oclusión de la arteria mesentérica inferior
cursa de manera asintomática en muchas ocasiones ya que el territorio que irriga puede recibir flujo
proveniente de la arteria:
a. cólica derecha
b. gastroduodenal
c. Epigástrica inferior izquierda
d. esplénica
e. cólica media

19. En las patologías de esófago es importante conocer bien la anatomía esofágica. ¿Cuál de las
siguientes afirmaciones es correcta?
a. El esófago tiene capa mucosa, muscular y serosa
b. El esófago abdominal es más largo que el cervical
c. El esófago torácico pasa por detrás del cayado aórtico
d. El epitelio esofágico normal es de tipo cilíndrico.
e. El esófago abdominal es discretamente más largo que el torácico

20. A pesar de que pueda haber variaciones anatómicas, lo habitual es que el ciego sea irrigado por una rama
arterial que proviene de unas de las siguientes arterias:
a. Iliaca derecha
b. Mesentérica inferior
c. Hepática derecha
d. Mesentérica superior
e. Iliaca izquierda

21. Ante un paciente con una cirugía abdominal urgente, el informe operatorio señala que se ha realizado una
resección de todo el duodeno y del tercio proximal del yeyuno manteniendo íntegros el estómago y todo
el íleon, así como los dos tercios distales del yeyuno. En el seguimiento nutricional del paciente ¿Qué
vitamina o mineral presentará con menor probabilidad una disminución de su absorción?
a. Cianocobalamina
b. Calcio
c. Hierro
d. Transcobalamina
e. Transferrina

22. ¿Cuál de las siguientes alternativas detallan las venas que confluyen y forman la vena señalada?
a. mesentérica superior, gástrica izquierda y
gastroepiploica izquierda
b. mesentérica inferior, gástrica izquierda y renal
c. esplénica, mesentérica superior y mesentérica inferior
d. esplénica, pancreatoduodenal y omental izquierda
e. gástrica izquierda, esplénica y hepática común

23. ¿Cuál de las siguientes sustancias forma parte de la secreción biliar?


a. Tripsina
b. Lecitina
c. Elastasa
d. Quimotripsina
e. Pepsina
24. El tubo digestivo contiene diferentes tipos de epitelios y glándulas. La estructura señalada es unay
está localizada en el …………...
a. glándula de Brunner / intestino
grueso b.cripta de Lieberkuhn /
colon
c. cripta de Lieberkuhn / intestino delgado
d. glándula oxintica / estomago
e. célula parietal / estómago

Aunque las criptas de


Lieberkuhn están presentes en el
intestino delgado, la
microfotografía es de epitelio de
colon.

25. ¿De qué musculo forma parte el ligamento inguinal?


a. Oblicuo externo del abdomen
b. Oblicuo interno del abdomen
c. Transverso del abdomen
d. Psoas
e. Dorsal ancho

26. Señale cuál de las siguientes afirmaciones NO se relaciona a la siguiente glándula anexa del tubo
digestivo mostrada en la imagen:
a. Es una glándula exocrina compuesta exclusivamente por acinos serosos
b. Su inervación está dada por el nervio auricular mayor (ramo posterior C2), que inerva la
vaina de la glándula así como la piel por encima de esta.
c. Esta glándula produce una secreción mucinosa acuosa, llamada mucoserosa, a través del
conducto de Wharton.
d. Su inflamación puede ser causada por un virus de los Paramyxoviridae, que provocan una
enfermedad muy frecuentemente en niños y adolescentes
e. Es una glándula endocrina y probablemente sea de origen pancreático

Se valida la opción e debido a que no está


relacionada con la imagen.

27. ¿Cuál de las siguientes enzimas está localizada en el borde en cepillo y juega un rol en la digestión de
proteínas?
a. Alfa dextrinasa
b. Pepsina
c. Enterocinasa
d. Lactasa
e. Carboxipeptidasa A.

Se valida la opción c debido a que es correcta en relación a la pregunta.


28. Una de los siguientes sustancias, NO sirve como un buen agente emulsificante:
a. Colesterol
b. Ácidos grasos
c. Sales biliares
d. Lecitina
e. Proteínas de la dieta

Se valida la opción e debido a que es correcta en relación a la pregunta.

29. La sustancia que estimula el crecimiento de la mucosa gástrica es:


a. Secretina
b. Motilina
c. Péptido estimulante de la mucosa gástrica
d. Gastrina
e. Histamina

30. ¿Cuál de las siguientes alternativas es una función de la colecistokinina?


a. Relajación de la vesícula para la salida de
bilis b.Secreción de ácidos biliares
c. Contracción del esfinter de Oddi
d. Secreción de enzimas pancreáticas
e. Contracción del duodeno

Se valida la opción b debido al efecto de la CCK sobre la vesicula biliar.

31. Con respecto a la anatomía del tronco celiaco, señale lo correcto


a. El tronco celiaco se origina de la cara posterior de la aorta abdominal
b. Es una arteria delgada que tiene un calibre entre 2 y 3 mm
c. Una de sus ramas es la arteria gástrica derecha
d. La hepática común que es una de sus ramas, participa en la irrigación del estómago.

32. Con respecto a la anatomía del duodeno, marque la respuesta correcta:


a. Tiene una distribución en forma de “C”, que rodea la cola del páncreas
b. La 3ra porción duodenal está contenida en la pinza vascular aortomesentérica
c. Entre la 1ra y 2da porción se forma un ángulo, conocido como el ángulo de Treitz
d. La 4ta porción se dirige a la izquierda, hacia abajo y hacia atrás.
e. En la tercera porción desemboca el conducto colédoco.

33. El hígado está ampliamente tapizado por peritoneo, la estructura que conecta la cara diafragmática del
hígado precisamente con el diafragma es el ligamento:
a. teres
b. falcifor
me
c.triang
ular
d.hepático
común
e.coronario

Se validan la opción c y e debido a que forman parte de los ligamentos que fijan el hígado al diafragma.

34. En el íleon se absorbe aproximadamente el 95% dea través de la circulación enterohepática.


a. agua
b. colesterol
c. sales biliares
d. hidróxicobalamina
e. factor intrínseca
35. Laestimula el mecanismo paracrino de la secreción de ácido clorhídrico.
a. histamina
b. acetilcolina
c. gastrina
d. secretina
e. somatostatina

36. En la digestión de proteinas,es el principal estímulo para convertir el pepsinógeno en pepsina.


a. la gastrina
b. el pH ácido
c. la acetilcolina
d. la ptialina
e. la somatostatina

37. Con respecto a la somatostatina, marque lo correcto:


a. Es secretada por las células S del intestino
b. Induce a la producción de VIP
c. Interviene en la fase intestinal de la secreción gástrica
d. Produce acetilcolina para estimular a la célula parietal
e. No interviene en la regulación de la secreción de ácido clorhídico

38. En pecten anal, es una estructura comprendida entre:


a. la línea pectínea y los senos anales
b. la línea blanca y la apertura anal
c. el esfínter anal interno y el externo
d. la línea anocutánea y la línea pectínea
e. la línea blanca y columnas anales

39. ¿Cuál de las siguientes alternativas es una proenzima pancreática?


a. Tripsina
b. Elastasa
c. Quimotripsinógeno
d. Amilasa
e. Procarboxipepitidasa C.

40. En la segmentación hepática de Coinaud, el segmento hepático señalado con la flecha,


corresponde a : En la segmentación hepática de Coinaud, la flecha señala el segmentohepático.
a. IV
b. V
c. VI
d. VII
e. VIII
SISTEMA DIGESTIVO (ME 154)
CLAVES EXAMEN FINAL
Ciclo 201900

1. Un niño de 2 años es llevado a la consulta por diarrea persistente, edema de las extremidades y falta de crecimiento
en relación a su edad. Los análisis de sangre revelan que tiene concentración plasmática baja de proteínas
(hipoproteinemia). Como parte del estudio se coloca colecistokinina (CCK) endovenosa y se recoge muestras del
líquido duodenal por endoscopía; el resultado del líquido confirma incapacidad para hidrolizar proteínas a un pH
neutro, esta situación mejora al añadir una pequeña cantidad de tripsina. El paciente probablemente esté sufriendo
la falta congénita de ………….
(Unidad 4, sesión 26, logro 2: Explicar la Digestión y absorción de las proteínas y sus alteraciones)
a. PEPT-1
b. pepsinógeno
c. enterocinasa
d. carboxipeptidasas

2. Paciente mujer de 35 años acude a consulta por sensación de sequedad y lesiones en cavidad oral. Al examen se
observa atrofia de la mucosa, fisuras y úlceras; nota además sequedad e irritación de la córnea y aumento del
tamaño de las glándulas parotídas. Su diagnóstico más probable es artritis reumatoide; el hallazgo más probable
en una biopsia de glándula parótida es……..….
(Unidad 3, sesión 18, logro 3: Describe las enfermedades más frecuentes de las glándulas salivales)
a. Presencia de acinos normales con hiperplasia de células ductales
b. Gran infiltración de linfocitos y células plasmáticas
c. Hiperplasia de acinos glandulares serosos
d. Gran infiltrado de linfocitos y neutrófilos

3. Un hombre de 42 años de edad se presenta al médico con una historia de 1 año de evolución, caracterizado por
dolor abdominal bajo y diarreas con crisis sanguinolentas. Manifiesta además pérdida de peso de 8 kg durante
este periodo. La colonoscopía revela lesión difusa en el colon con afectación del recto. La biopsia de estas lesiones
revela adelgazamiento de la pared, inflamación y ulceración de la mucosa y sub mucosa. El diagnóstico más
probable en este caso es:
(Unidad 4, sesión 28, logro 5: Describe la Enfermedad inflamatoria intestinal. Generalidades, morfología y
características)
a. sindrome de colon irritable
b. enfermedad de Crohn
c. colitis ulcerativa
d. sprue celiaco

4. Dos estudiantes deciden tomar un receso para comer una hamburguesa a la hora del almuerzo. Antes de llegar a
la cafetería, impulsos nerviosos provenientes del complejo vagal dorsal iniciarán la secreción de ácido gástrico por
la liberación de …………………….. desde el sistema nervioso entérico.
(Unidad 3, sesión 20, logro 2: Regulación de la secreción gástrica: estimulación, fases de la secreción)
a. Serotonina
b. Colecistoquinina
c. Péptido inhibidor vaso activo
d. GRP (péptido liberador de gastrina)

5. Un niño de cuatro años de edad es llevado a la consulta por cuadros diarreicos frecuentes caracterizados por
heces pálidas, voluminosas y fétidas; al examen físico presenta bajo peso y talla para la edad. Se mide la
concentración de cloruro en el sudor y se encuentra que sus valores son muy elevados. La alteración más
importante a nivel de células ductales del páncreas tiene relación directa con la conductancia de…………
(Unidad 3, sesión 23, logro 5 Explica la Secreción pancreática: formación del jugo pancreático, influencia de la
velocidad de flujo y regulación)
a. Bicarbonato
b. Potasio
c. Sodio
d. Cloro
6. Se evalúa los valores séricos de las siguientes sustancias a un paciente con enfermedad hepática terminal; en este
paciente se espera encontrar la combinación con la letra …………
(Unidad 3, sesión 22, logro 5: Describe las Pruebas de función hepática, la Insuficiencia hepática, encefalopatía
hepática e hipertensión portal)
Glucosa Amoniaco Albúmina

a. Aumentada Disminuida Disminuida

b. Disminuida Aumentada Aumentada

c. Aumentada Aumentada Aumentada

d. Disminuida Aumentada Disminuida

7. Una mujer de 35 años de edad HIV positiva, se presenta al médico con dolor abdominal en cuadrante superior
derecho e ictericia. La paciente refiere haber tenido múltiples episodios de ictericia durante los últimos 10 años.
Los exámenes para determinar hepatitis viral, dieron positivos para Hepatitis B, siendo catalogado el caso como
hepatitis crónica con alteración funcional. En un examen de sangre ¿cuál de los siguientes parámetros está
disminuido?
(unidad 3, sesión 22, logro 5: Pruebas de función hepática, Insuficiencia hepática, encefalopatía hepática e
hipertensión portal)
a. Albumina
b. Bilirrubina
c. Fosfatasa alcalina
d. Tiempo de protrombina

8. En el reflejo peristáltico del intestino delgado ¿Cuál de los siguientes eventos sucede en la porción caudal del bolo
alimenticio?
(Unidad 2, sesión 13, logro 4: Explicar la Motilidad del intestino delgado: Contracciones segmentarias y
peristálticas)
a. Acción del péptido inhibidor vasoactivo (VIP) en el músculo circular
b. Acción del NO (óxido nítrico) en el músculo longitudinal
c. Contracción del músculo longitudinal interno
d. Acción de acetilcolina en el músculo circular

9. Un varón de 58 años de edad con enfermedad de Crohn severo fue sometido a una resección ileal. Después de la
cirugía este paciente padecerá de esteatorrea, esto se explica porque …..………..
(unidad 4, sesión 26, logro 4: Explica las alteraciones en la Absorción de lípidos)
a. se inhibe la acción de la 7 alfa hidroxilasa
b. el pool de ácidos biliares se incrementa
c. hay mala absorción de ácidos biliares
d. el páncreas no secreta lipasa

10. En un experimento se inserta un balón en el estómago de un voluntario, se infla poco a poco mientras que se
vigilan las presiones intraluminales. Aunque el volumen del balón aumenta considerablemente, las presiones
permanecen constantes. Esta relación volumen-presión se explica por la liberación local de …………..
(Unidad 2, sesión 13, logro 1 Explica la Motilidad gástrica: relajación receptiva)
a. acetil colina y gastrina
b. norepinefrina y óxido nítrico
c. colecistoquinina y óxido nítrico
d. óxido nítrico y péptido inhibidor vasoactivo

11. ¿Cuál de las siguientes alternativas es una característica de la secreción exocrina del páncreas?
(Unidad 3, sesión 23, logro 5: Secreción pancreática: formación del jugo pancreático, influencia de la velocidad de
flujo y regulación)
a. Es hipotónica respecto al plasma
b. Su mayor estímulo se da en la fase intestinal
c. Es estimulada por la presencia de bicarbonato en el duodeno
d. La secreción enzimática es estimulada principalmente por la secretina
12. Las estructuras en el hígado que permite que los productos metabólicos unidos a proteínas tengan acceso a las
membranas basolaterales de los hepatocitos, son…..
(Unidad 3, sesión 21, logro 4-5: Explica la Organización micro estructural del hígado)
a. los canalículos
b. las células de Ito
c. las fenestras sinusoidales
d. las uniones intercelulares herméticas

13. La composición de la bilis es modificada conforme fluye por los conductillos biliares. Durante este tránsito se
espera que aumente la concentración de…….
(Unidad 3, sesión 22, logro 2: Describe la Secreción biliar, visión general del sistema biliar extrahepático y
composición de la bilis)
a. Ig A
b. Glucosa
c. Protones
d. Vitamina A

14. Se mide experimentalmente el contenido gástrico de dos personas. La persona “A” tiene alto contenido de grasa y
la persona “B” tiene un contenido isotónico ¿Cuál de las siguientes es correcto respecto al vaciamiento gástrico?
(Unidad 2, sesión 13, logro 2: Describe la Motilidad y vaciamiento gástrico)
a. Hay ralentización del vaciado gástrico solo en “A”
b. El vaciamiento gástrico es más rápido en ambos
c. Hay ralentización del vaciado gástrico solo en “B”
d. Hay ralentización del vaciado gástrico en ambos casos

15. El examen endoscópico de un paciente con hipertensión portal grave revela venas tortuosas que sobresalen hacia
la luz del esófago. El paciente recibe tratamiento quirúrgico mediante la colocación de una derivación que
conecta la vena porta a la vena cava. Después de la operación el riesgo de encefalopatía ………………….. y el riesgo
de sangrado de várices ……………..
(Unidad 3, sesión 22, logro 5: Describe la Insuficiencia hepática, encefalopatía hepática e hipertensión portal)
a. disminuirá / disminuirá
b. disminuirá / aumentará
c. aumentará / disminuirá
d. aumentará / aumentará

16. Un paciente varón de 18 años de edad acude al médico para sus exámenes de rutina. Sus resultados de
laboratorio muestran un valor de bilirrubina sérica de 4 mg/dl y una bilirrubina directa de 0,3 mg/dl. Las pruebas
de función hepática son normales. La alteración que explica mejor este caso es por la deficiencia de ………………..
(Unidad 3, sesión 22, logro 3: Explica la Producción y excreción de bilirrubina. Tipos de bilirrubina e ictericia)
a. transaminasas
b. hemo oxigenasa
c. la 7 alfa hidroxilasa
d. glucuronil transferasa

17. Un hombre de 57 años de edad es llevado a urgencias con hematemesis masiva rojo brillante, a su llegada se halla
inconsciente con PA: 80/40 mm Hg y FC: 124 lat/min. Luce ictérico con presencia de “arañas vasculares en el tórax
anterior y extremidades”, abdomen distendido con signo de oleada positiva. Se encuentra esplenomegalia y
pérdida de la masa muscular en extremidades. La anastomosis vascular responsable del sangrado en este
paciente es ………….…..
(Unidad 3, sesión 21, logro 2: Describe las anastomosis porto sistémicas)
a. vena gástrica izquierda y vena ácigos
b. arteria gástrica izquierda y vena ácigos
c. vena paraumbilical y vena epigástrica inferior
d. vena gástrica izquierda y vena esofágica superior
18. Un estudiante de medicina está comiendo un plato de comida a base de champiñones, espárrago y salsa de soya.
El estímulo del sabor umami contenido en todos estos alimentos viaja a través del nervio………………..
(Unidad 2, sesión 10, logro 3: Describe la irrigación e inervación de la lengua)
a. Lingual
b. Hipogloso
c. Glosofaringeo
d. Cuerda del tímpano
19. Una paciente de 30 años de edad es sometida a una cirugía en oído medio derecho por un problema de
otoesclerosis. Luego de la cirugía refiere alteración sensitiva de la lengua. Al evaluar el caso usted esperaría
encontrar……….
(Unidad 2, sesión 10, logro 5: Describe la Irrigación e inervación de la lengua)
a. Alteración en la sensación del dolor y temperatura en el tercio posterior de la lengua
b. Alteración en la sensación del dolor en los dos tercios anteriores de la lengua
c. Alteración en la sensación del gusto en el tercio posterior de la lengua
d. Sensación del dolor, tacto y temperatura conservadas

20. En un paciente de 45 años de edad con colestasis biliar, se encuentra una elevación de los niveles sanguíneos de
fosfatasa alcalina hasta 3 veces la cifra normal. ¿Cuál de las siguientes alternativas estará también elevada como
evidencia del daño de la vía biliar?
(Unidad 3, sesión 22, logro 5: Pruebas de función hepática, Insuficiencia hepática, encefalopatía hepática e
hipertensión portal)
a. Tiempo de protrombina y albúmina sérica
b. Transaminasas hepáticas (ALT y AST)
c. Gamma glutamil transpeptidasa
d. Glucoronil transferasa

21. Experimentalmente se incrementa la velocidad de la secreción salival con una sustancia, en el análisis de la
composición de esta saliva obtenida se espera encontrar…………..
(Unidad 3, sesión 17, logro 5 : Explica la Influencia de la velocidad del flujo salival en la composición de la saliva)
a. disminución de la concentración de bicarbonato que supera la concentración plasmática
b. aumento de la concentración de cloro y sodio que supera la concentración plasmática
c. aumento de la concentración de bicarbonato que supera la concentración plasmática
d. disminución de concentración de potasio y bicarbonato

22. Lactante de 3 meses de vida es atendido por presentar diarrea, se administra una solución de glucosa y
electrólitos por vía oral. La proteína de membrana apical que explica la capacidad de esta solución para
proporcionar aporte de glucosa e hidratación es ………..
(Unidad 4, sesión 26, logro 1: Explica la Digestión y Absorción de los hidratos de carbono. Alteraciones)
a. CFTR
b. SGLT-1
c. GLUT-2
d. GLUT-5

23. Paciente ha sufrido herida de bala en el abdomen, se le ha tenido que extirpar el segmento medio y distal del
ileon. En este caso la síntesis hepática de sales biliares estará …..…..
(Unidad 3, sesión 22, logro 4: Explica la formación, función y Circulación entero hepática de lasa sales biliares)
a. Sin cambios en el ritmo de síntesis
b. Disminuida por inhibición de la enzima colesterol 7 alfa hidroxilasa
c. Incrementada por estímulo de la enzima colesterol 7 alfa hidroxilasa
d. Incrementada por inhibición de la enzima colesterol 7 alfa hidroxilasa

24. Un varón de 75 años ingresa al consultorio por presentar ictericia marcada de piel y las escleras. El estudio del
paciente mostró que presentaba un tumor que obstruía la totalidad del conducto hepático común. ¿Cuál de los
siguientes conductos se encontrará dilatado en este paciente?
(Unidad 3, sesión 21, logro 6: Describir el árbol biliar intrahepático)
a. de Wirsung
b. de Hering
c. colédoco
d. cístico
25. Correlaciones las dos columnas y marque la fórmula correcta:
(Unidad 4, sesión 28, logro 1: Diarrea: definición, mecanismos: osmótica, secretoria y exudativa)

1. Enfermedad Hirschsprung ( ) heces con moco y sangre


2. Diarrea osmótica ( ) intolerancia a lactosa
3. Diarrea secretoria ( ) aganglionosis congénita
4. Diarrea exudativa ( ) canales de Cl- en las células de la cripta

a.- 4231 b.- 1234 c.- 2143 d.- 4213

26. La vena umbilical obliterada del hígado después del nacimiento se transforma en el ligamento:
(Unidad 3, sesión 21, logro 1: Hígado: relación con la pared abdominal, caras, lóbulos, ligamentos , hilio hepático)
a. cruzado
b. redondo
c. coronario
d. falciforme

27. Llega a su guardia nocturna una madre que trae a su RN masculino de 2 semanas de vida con mal estado general y
sequedad de mucosas. Usted observa que lacta ávidamente, pero a las 2 horas presenta vómito postprandial no
bilioso en proyectil. Al realizar la historia clínica, descubre que el lactante recibió profilaxis con macrólidos para
tos ferina. Usted sospecha principalmente en:
(Unidad 1, sesión 4, logro 4: Desarrollo y anomalías del intestino anterior)
a. estenosis pilórica hipertrófica congénita
b. fistula traqueo esofágica
c. estenosis duodenal
d. atresia duodenal

28. En la regulación del apetito y la saciedad, la estimulación experimental crónica del núcleo ventro medial del
hipotálamo producirá:
(Unidad 1, sesión 3, logro 4: explica los mecanismos de control del apetito y saciedad)
a. afagia
b. obesidad
c. hiperfagia
d. activación de neuronas relacionadas a NPY

29. Paciente mujer de 25 años acude por dolor en fosa ilíaca derecha que empeora al toser o caminar, asociada a
náuseas y vómitos por lo cual acude a emergencia. Dos días después de realizarle una apendicectomía, la paciente
desarrolla fiebre alta (39 °C), está hipotensa y presenta dolor abdominal. La laparotomía exploratoria muestra un
gran volumen de sangre en la cavidad peritoneal por lesión de un vaso producida durante la apendicectomía.
¿Cuál de las siguientes arterias debe ligarse para detener la hemorragia?
(Unidad 4, sesión 27, logro 4: Irrigación arterial del colon, recto y conducto anal)
a. cólica derecha y arteria rectal superior.
b. ileocólica y arteria cólica media.
c. mesentérica superior.
d. ileocólica.

30. La onda peristáltica secundaria del esófago se caracteriza por ser originada ………
(unidad 2, sesión 12, logro 3: Motilidad esofágica: fases y características)
a. por el plexo de meissner del esófago
b. por el plexo mientérico del esófago
c. por el reflejo de la deglución
d. durante la masticación

31. ¿Cuál de los siguientes es una causa de ictericia con bilirrubina conjugada aumentada?
(Unidad 3, sesión 22, logro 3: Producción y excreción de bilirrubina. Tipos de bilirrubina, ictericia)
a. Ictericia del recién nacido
b. Obstrucción del colédoco
c. Anemia hemolítica
d. Gran hematoma
32. En relación a la absorción de nutrientes, la absorción de dipéptidos y tripéptidos a nivel de las células epiteliales
del intestino delgado, se da principalmente debido a:
(Unidad 4, sesión 26, logro 2: Digestión y absorción de las proteínas. Alteraciones)
a. el incremento de los canales de Cl- en la membrana apical
b. la gradiente de bicarbonato en la membrana basal
c. la gradiente de iones H+ en la membrana apical
d. la gradiente de Na+ en la membrana apical

33. Paciente de 20 años es traído a la emergencia por presentar diarreas desde hace 2 días. Familiar refiere que las
deposiciones son líquidas y abundantes, al examen luce deshidratado y se plantea que la diarrea es producida por
una toxina que estimula la transformación de ATP a AMPc con apertura de canales de Cl- y pérdida de agua. El
tipo de diarrea más probable es:
(Unidad 4, sesión 28, logro 1: Diarrea: definición , mecanismos: osmótica, secretoria y supurativa)
a. osmótica
b. exudativa
c. secretoria
d. por intolerancia a lactosa

34. Un niño fue operado por una obstrucción intestinal, observándose la presencia de divertículo de Meckel. Según lo
referido, marque lo correcto:
(Unidad 1, sesión 5, logro 2: identifica las anomalías del desarrollo del intestino medio: onfalocele y gastrosquisis
(diferencias), Divertículo de Meckel)
a. el 50% de la población lo presenta
b. se localiza en el íleon muy cerca al yeyuno
c. puede poseer tejido gástrico o pancreático
d. se produce por una mala rotación de los intestinos

35. Marque la alternativa correcta respecto a la


estructura marcada en el gráfico:
(Unidad 3, sesión 22, logro 2: Secreción biliar.
Visión general del sistema biliar extrahepático y
composición de la bilis)

a. Se halla a 2 centímetros debajo de la


papila duodenal mayor
b. Llega el conducto colédoco y pancreático
principal
c. Llega el conducto hepático común y
pancreático principal
d. Llega el conducto pancreático accesorio

36. ¿Cuál de las siguientes moléculas se encontrará aumentada en el citoplasma de las células parietales de un
paciente con sindrome de Zollinguer Ellison?
(Unidad 3, sesión 20, logro 4: Enfermedad ulcerosa péptica: úlcera gástrica, duodenal. síndrome de Zollinger –
Ellison)
a. Péptido liberador de gastrina (GRP)
b. Proteína G estimulante (GS)
c. Inositol Trifosfato (IP3)
d. AMP cíclico (AMPc)

37. Los fármacos inhibidores de la bomba de protones, actúan bloqueando la ………..……..


(Unidad 3, sesión 20, logro 3: Regulación de la secreción gástrica: inhibición, Secreción de pepsinógeno y factor
intrínseco)
a. anhidrasa carbónica
b. ATPasa H+/K+ en la membrana luminal
c. ATPasa H+/K+ en la membrana basolateral
d. ATPasa Na+/K+ en la membrana basolateral
38. Un paciente fue diagnosticado de gastritis autoinmune, ¿cuál de las siguientes alternativas es FALSA respecto a
esta enfermedad?
(Unidad 3, sesión 20, logro 5: Gastritis crónica: helicobacter pylori, autoinmune. Tipos de gastritis)
a. Afecta principalmente el fondo y cuerpo gástrico
b. Se produce hiperplasia de células G secundaria a la aclorhidria
c. El propio sistema inmune destruye principalmente las células parietales
d. Se produce atrofia de la mucosa, aclorhidria, hipergastrinemia y déficit de vitamina B6

39. Marque la correlación correcta:


(Unidad 3, sesión:18, logros:1 y 2: Describe las enfermedades inflamatorias/infecciosas y proliferativas de la
cavidad oral)

1. Herpes virus ( ) En relación al abuso de antibióticos


2. Candidiasis oral ( ) Lesiones vesiculares como racimo de uvas
3. Eritroplaquia ( ) Mega esófago
4. Enfermedad de Chagas ( ) Lesión pre cancerígena

a.- 2431 b.- 1234 c.- 4123 d.- 2143

40. En un paciente con insuficiencia renal crónica, el déficit en la absorción de calcio a nivel del enterocito se debe a
lo siguiente:
(Unidad 4, sesión 26, logro 6: Explica la Absorción de calcio y hierro)
a. No se convierte la 25 hidroxicolecalciferol a 1,25 dihidroxicolecalciferol
b. No se convierte la 1,25 dihidroxicolecalciferol a 25 hidroxicolecalciferol
c. Existe un descenso de la alfa 25 hidroxilasa renal
d. Se incrementa la producción de Calbindina
SISTEMA DIGESTIVO
EXAMEN PARCIAL
201802

Sección: Todas
Profesores: Choque Chávez, Fernando Diego; Damián Bastidas, Narda Lucía; Irribarren Gamarra, Maria Patricia;
Stapleton Valdivia, Mauricio Juan Jose.
Duración: 50 minutos.
Indicaciones:
- Lea atentamente cada pregunta antes de responder.
- Se prohíbe el uso del celular y cualquier dispositivo electrónico.
- Está prohibido intercambiar materiales.
- Coloque su código de alumno en la tarjeta de respuestas. Si su código contiene una letra reemplácela por un valor
numérico siguiendo la siguiente equivalencia: A=9, B=8, C=7, D=6, E=5.
- Traslade sus respuestas a la tarjeta, llenando los círculos de manera completa con lapicero negro o azul. Está
prohibido el llenado con lápiz, lapicero de otro color o con lapicero de tinta borrable.
- Sea cuidadoso en el llenado de la tarjeta de respuestas, pues solo esta tiene validez para la calificación.
- Al terminar su examen avise al docente a cargo, no se levante de su sitio; debe entregar la hoja de respuestas con
la carátula del examen, este cuadernillo de preguntas se lo llevará cada estudiante.

Chorrillos, octubre de 2018

1. En todo el sistema gastrointestinal se encuentra diferentes tipos de músculos ¿En cuales estructuras
encontraríamos músculo estriado?
A. Faringe, esfinter esofágico inferior, colon descendente.
B. Esfínter esofágico superior, laringe, esfínter anal externo.
C. Faringe, esfínter esofágico superior, esfínter anal externo.
D. Esfínter esofágico superior, esfínter esofágico inferior, colon.

2. Durante la fase …..……………….. el paladar blando es traccionado hacia arriba.


A. oral
B. gástrica
C. faríngea
D. esofágica

3. El tubo digestivo a nivel del ………………………….. está revestido internamente por epitelio ………….
A. Esófago / plano simple
B. Estómago / cilíndrico simple
C. Intestino grueso / plano estratificado
D. Intestino delgado / plano de transición

4. Señale la estructura del tubo digestivo que presenta tres capas musculares en su pared.
A. Esófago
B. Estómago
C. Intestino grueso
D. Intestino delgado

5. Paciente de 47 años con sobrepeso, acude a consulta por presentar dolor tipo cólico en el cuadrante superior
derecho del abdomen. El dolor aumenta luego de ingesta de comidas con abundante grasa. Esta situación se explica
por el aumento en la secreción de …………………………. que estimula la contracción de la vesícula biliar.
A. Gastrina
B. Colecistoquinina
C. Péptido inhibidor gástrico
D. Péptido intestinal vasoactivo
6. Si hay un aumenta de la hormona grelina a nivel del hipotálamo ¿Cuál es la consecuencia en el organismo?
A. Hambre
B. Saciedad
C. Aumento de somatostatina
D. Disminución del tránsito intestinal

7. Señale el péptido gastrointestinal que produce relajación del músculo liso gastro-intestinal.
A. Péptido intestinal vasoactivo (PIV)
B. Gastrina vasoactiva (GV)
C. Colecistoquinina (CCK).
D. Acetilcolina (Ach)

8. ¿Cuál de las siguientes alternativas inhibiría la relajación receptiva a nivel del estómago?
A. Histamina
B. Bloqueo del nervio vago (X)
C. Péptido inhibir gástrico (GIP)
D. Péptido intestinal vasoactivo (VIP)

9. Una mujer de 28 años, con diagnóstico de Diabetes Mellitus Tipo 1, acude por presentar desde hace 10 años
estreñimiento y distensión abdominal. Se realiza estudio y se determina que la paciente presenta un retraso del
vaciamiento gástrico debido a gastroparesia diabética. ¿Cuál de los siguientes hechos aumentaría el tiempo del
vaciamiento gástrico?
A. Aumento de gastrina
B. Estimulación parasimpática
C. Ácidos grasos en el duodeno
D. Quimo isotónico en el duodeno

10. Experimentalmente, se aplica un inhibidor selectivo del péptido intestinal vasoactivo (PIV) durante la contracción
peristáltica del intestino delgado. ¿Cuál es el efecto de este inhibidor en la motilidad del intestino delgado?
A. Parálisis del movimiento anterogrado
B. Disminución del tránsito intestinal
C. Aumento del tránsito intestinal
D. Movimiento retrogrado

11. ¿Cuál de los siguientes eventos ocurre durante la defecación?


A. Relajación del esfínter anal externo
B. Relajación del músculo liso del recto
C. Contracción del esfínter anal interno
D. Disminución de la presión intrabdominal

12. Señale el péptido que cumple la función de disminuir la ingesta de alimentos.


A. Neuropéptido Y
B. Ghrelina Gastrica
C. Proteína relacionada a agouti
D. Proopiomelanocortina (POMC)

13. Indique el lugar de secreción del péptido YY


A. Estómago
B. Duodeno
C. Yeyuno
D. Ileon

14. La grelina es secretada en el estómago y estimula a las neuronas del núcleo ………………….. para la estimulación de
la secreción de……………..
A. arqueado / neuropéptido Y.
B. arqueado / melanocortinas.
C. paraventricular / neuropéptido Y.
D. lateral del hipotálamo / hormonas orexigénicas.
15. Considerando el desarrollo embriológico del intestino medio. ¿Qué evento de gran importancia se produce en la
sexta semana?
A. Retracción de asas intestinales primitivas
B. Aparición del primordio hepático y pancreático
C. Inicio de la secreción de insulina por el páncreas
D. Salida temporal de asas intestinales a través de cordón umbilical

16. En el ……………………………. se presentan los movimiento de …………………….


A. esófago / retropropulsión y mezcla
B. estómago / segmentación y ondas lentas
C. intestino grueso / propulsión y ondas en espiga
D. intestino delgado / segmentación y peristaltismo

17. Marita sufre una parálisis del músculo masetero. ¿Qué limitaciones se producirán a nivel del movimiento de la
mandíbula?
A. Lateralización
B. Propulsión
C. Elevación
D. Ninguna

18. Señale el movimiento que se produce a nivel del ciego y colon proximal que tiene como finalidad favorecer la
absorción de agua y sales.
A. De masa
B. De mezcla
C. Propulsivo
D. Peristáltico

19. La enfermedad de Hirchsprung se caracteriza por una dilatación anormal de colon y disminución de los
movimientos propulsivos que traerá como principal consecuencia …………
A. diarrea acuosa.
B. estreñimiento crónico.
C. dilatación de la válvula ileocecal.
D. disminución de la flora intestinal.

20. ¿Cuál de los siguientes factores estimulan el vaciamiento gástrico?


A. Colecistoquinina (CCK)
B. Neuropéptido Y
C. Secretina
D. Gastrina

21. La secreción de ………………………. estimula la motilidad gástrica.


A. Colecistoquinina
B. Secretina
C. Gastrina
D. Motilina

22. La hormona ……………………….. es secretada por las células “I” del …………..
A. Colecistoquinina / duodeno y yeyuno
B. Gastrina / duodeno y yeyuno
C. Colecistoquinina / estómago
D. Gastrina / estómago
23. El reflejo enterogástrico aumenta ……………………… debido a la…………………..
A. el tono de la válvula ileocecal /distención del colon.
B. el tono del esfínter pilórico / distensión del duodeno.
C. la motilidad del intestino delgado / distensión del ciego.
D. la motilidad del intestino delgado / distensión del estómago.

24. ¿Cuál de las siguientes sustancias disminuye el vaciado gástrico?


A. Motilina.
B. Noradrenalina.
C. Colecistocinina.
D. Neuropéptico Y.

25. El plexo mientérico se encuentra ubicado en………………. y funciona controlando la ………………


A. submucosa / secreción
B. submucosa / motilidad
C. entre las capas musculares / motilidad
D. entre las capas musculares / secreción

26. ¿Cuál de las siguientes glándulas está conformada por glándulas serosas y secreta la mayor cantidad de amilasa
en la boca?
A. Linguales
B. Parótidas
C. Sublinguales
D. Submaxilares

27. ¿Cuál de los siguientes factores es más importante para percepción mediante las papilas gustativas?
A. Agua
B. Enzimas
C. Ion fierro
D. Bicarbonato

28. Paciente obeso, fumador, consumidor de alcohol que acude por presentar molestias como disfagia, ardor
retroesternal, alteración del sueño y sensación de boca amarga en las mañanas. ¿Cuál de las siguientes
recomendaciones podría darle a este paciente?
A. Una dieta rica en lípidos y grasas.
B. Elevar la cabecera de la cama al dormir.
C. Disminuir la frecuencia de comidas y aumentar el volumen.
D. Una dieta hiperproteica y líquida en las noches antes de dormir.

29. En la enfermedad por reflujo se encuentra alterada la función del esfínter …………………….
A. esofágico superior
B. esofágico inferior
C. de Oddi
D. pilórico

30. En la diarrea psicógena hay una estimulación del ……………….. que …………………….. la motilidad gastrointestinal.
A. Sistema nervioso parasimpático / aumenta
B. Sistema nervioso simpático / inhibe
C. Plexo submucoso / estimula
D. Plexo mientérico / inhibe
Ecu 1
Caso 1
Lactante masculino de 5 semanas de edad, producto de un primer embarazo normal controlado.
Peso de nacimiento 3.120 g y talla de 51 cm. Lactancia materna satisfactoria, con buen incremento
ponderal. Sin antecedentes patológicos hasta 5 días atrás, cuando comienza con vómitos
postprandiales no biliosos, lácteos. Los síntomas aumentan en frecuencia y magnitud hasta hacerse
explosivos después de cada alimentación. No refiere fiebre, tos, diarrea ni lesiones cutáneas. Es
importante destacar que, pese a los vómitos, el niño conserva el apetito y llora de hambre. Al
examen físico presenta buen estado general. Abdomen blando, depresible e indoloro, asociado a
distensión del hemiabdomen superior. Sin signos de deshidratación. No se palpan masas
abdominales. Exámenes de laboratorio: hemograma normal. Signos inflamatorios de fase aguda
negativos. Alcalosis metabólica leve en sangre venosa.
Preguntas:
1. Debido al signo de la onda peristáltica que se evidencia en este paciente, se debe
asumir que como consecuencia hay un aumento en la liberación de ___________ en la
pared del estómago:
a) Secretina
b) Histamina
c) Gip
d) Adenosina
2. Al momento de realizar la pilorotomía (corte del músculo del píloro para descomprimir)
¿Qué hormona se comenzará a liberar rápidamente en respuesta a este tratamiento?
a) Secretina
b) Bombesina
c) Colecistoquinina
d) Gastrina
3. La oliva pilórica se logra palpar a nivel de:
a) Epigastrio parte superior
b) Hipocondrio derecho
c) Entre epigastrio y mesogastrio
d) L1
4. Producto de la hipertrofia pilórica uno esperaría que los niveles de Gastrina se
encuentren:
a) Elevado
b) Disminuido
c) Depende del alimento
d) Normales
5. Si el paciente tuviera 35 años, uno podría pensar como diagnóstico diferencial de la
hipertrofia de píloro, la presencia de un tumor neuroendocrino productor de hormona:
a) Gastrina
b) Bombesina
c) Colecistoquinina
d) Secretina
6. En este paciente se puede esperar una mayor liberación de:
a) Enzimas pancreáticas
b) Gip
c) Secretina
d) Insulina
7. En relación con la estructura afectada se encuentra:
a) Mesenterio propiamente dicho
b) Ligamento gastroesplénico
c) Omento mayor
d) Ligamento redondo
Caso 2
Estudiante de medicina de la UPC de 21 años sufre de gastritis aguda ocasionada por comer en
lugares poco higiénicos. Suele consumir caramelos ( chupar ) mientras está en clase hasta la tarde.
Toma gaseosas regularmente (carbohidratos 46%, sodio 53%). También toma regular cantidad de
leche (grasa 35%, lactosa 35%, proteínas 30%), pues le calma un poco el dolor el ardor que siente
por la gastritis. Incluso, cuando puede, se toma dos vasos de agua fría para calmar las molestias. Ha
decidido ir al médico para tratarse pues ya no soporta el dolor, el cual está seguro que los síntomas
se deben a una elevada producción de ácido clorhídrico en el estómago, y por ello le ha recetado
Ranitidina (antihistamínico), con lo que siente mejoría.
Preguntas:
1. Antes del uso de Ranitidina en este paciente, los valores de somatostatina en sangre
están:
a) Aumentados
b) Disminuidos
c) En valores normales
d) No hay somatostatina en sangre
2. En este paciente con gastritis aguda debida a una alta producción de ácido clorhídrico,
si se le hiciera un examen de sangre, se encontraría elevados los niveles de:
a) GIP
b) Sustancia P
c) Histamina
d) CCK
3. El consumo de leche produce directamente un aumento de los niveles séricos de la
hormona:
a) Gastrina
b) Leptina
c) Grelina
d) VIP
4. Entre las sustancias cerebrales que producen ansiedad esta la serotonina, la cual
también tiene acción;
a) Anoxigénica
b) Orexigenica
c) La acción de la serotonina en el apetito no está descrita
5. El uso de atropina en este paciente
a) Aumentará la producción de ácido clorhídrico
b) Aumentara el pH del estomago
c) Inhibirá la acción de las prostaglandinas
d) Disminuirá la acción del receptor CCK-B
6. El consumo rápido de 500 mL de gaseosa aumentara directamente la concentración
sérica de cual de las siguientes hormonas:
a) Cck
b) Secretina
c) Gastrina
d) Neuropéptido Y
7. El consumo de una pequeña cantidad de gaseosa aumentara directamente la
concentración sérica de cual de las siguientes hormonas
a) Péptido 1 similar al glucagón (GLP-1)
b) CCK
c) Secretina
d) Motilina
8. En este paciente con gastritis agua hiperclorhídrica, y debido a esta gran producción
de ácido, teóricamente podría tener como complicación:
a) Aumento del vaciamiento gástrico
b) Hipertrofia del píloro
c) Hemorroides
d) Ictericia
9. Este paciente tiene también la costumbre de tomar agua antes de sentarse a almorzar,
la cual hace que la producción de ácido clorhídrico del estomago
a) Aumente
b) Disminuya
c) Regrese a lo normal
d) No varié
10. El consumo de dos vasos de agua seguidos generara indirectamente un aumento en la
liberación de:
a) Péptido Y
b) Noradrenalina
c) Acido clorhídrico
d) Enzimas pancreáticas

ECU 2
Niño de sexo masculino de 2 años de edad, sufre de estreñimiento desde el nacimiento (1 deposición cada 3-4
días). Madre menciona que le estimula la defecación con un termómetro rectal, y continuo uso de enemas y
laxantes. Desde hace 6 meses comienza con vómitos postprandiales. Los síntomas aumentan en frecuencia y
magnitud y están en relación con los episodios de estreñimiento. No refiere fiebre, tos, diarrea ni lesiones
cutáneas. Al examen físico presenta regular estado general, luce deshidratado. Abdomen distendido, blando,
depresible e indoloro. No se palpan masas abdominales. Se permeabiliza el canal anal con termómetro rectal,
encontrando cierta resistencia. Salida de material fecal mal oliente en regular cantidad. Exámenes de
laboratorio: hemograma normal. Signos inflamatorios de fase aguda negativos. Alcalosis metabólica leve en
sangre venosa. Radiografía con enema baritado muestra recto y colon sigmoides dilatados (megacolon).
Biopsia profunda: ausencia de células ganglionares en la muestra enviada. Se realiza cirugía correctiva.

Preguntas:

1. En la zona afectada por esta enfermedad, se espera que las ondas lentas esten:
a) Disminuidas
b) Ocurran de manera normal
c) Abolidas
d) Aumentadas
2. Considerando la época que vivimos y como el paciente esta programado para sala de
operaciones, para lo cual se le toma una muestra de hisopado para descartar infección por
coronavirus. Al realizar el hisopado, el personal toca la pared de la orofaringe, la cual envía una
señal sensitiva a través del par craneal.
a) IX
b) X
c) V3
d) V2
3. No se espera que sea causa del vomito
a) Ayuno prolongado
b) Estimulación faríngea y del glosofaríngeo
c) Irritación de la mucosa gástrica
d) Dolor intenso
4. Con respecto a la defecación señale el enunciado correcto:
a) Es estimulado por llenado de la cuarta parte del volumen rectal
b) Es un reflejo netamente local
c) Es completamente voluntario y mediado por el nervio pudendo
d) La aferencia parasimpática es transmitida por vía del nervio vago
5. Al examinar la orofaringe del paciente, uno puede hallar fácilmente la amígdala
palatina, pues esta se encuentra inmediatamente detrás del musculo:
a) Palatogloso
b) Palatofaringeo
c) Hiogloso
d) Elevador del velo del paladar
6. En este paciente se considera que esta abolido el reflejo:
a) Gastrocolico
b) Relajación receptiva
c) Coloileal
d) Rectoesfintereano
7. Es un reflejo propio de la pared intestinal:
a) Peristaltismo
b) Doloroso
c) Defecación
d) Cólicoileal
8. Durante la fase esofágica de la deglución, para un bolo alimenticio determinado, a
medida que avanza el bolo la fuerza de la contracción se hace mas:
a) Débil
b) Dependiente de acetilcolina
c) Fuerte
d) Hiperpolarizado
9. El contenido fecal se detiene en la zona inmediatamente proximal a la zona donde hay
una menor presencia de:
a) Péptido intestinal vasoactivo
b) Neuropéptido Y
c) Enteroquinasa
d) Acetilcolina
10. La percepción de la pirosis (sensación de dolor o quemazón en el esófago) asociado a
reflujo gastroesofágico, puede aparecer o exacerbarse debido a:
a) Ejercicio
b) Bipedestación
c) Uso de antiácidos
d) Somatostina
11. En cuanto a los reflejos gastrocólico y gastroduodenal en este paciente, indique lo
correcto:
a) Se puede considerar reflejos vago – vágales
b) El control del nervio vago sobre el recto se ha abolido
c) El reflejo Gastrocolico es mas marcado en adultos que en niños
d) Se dan por nervios intrínsecos del sistema entérico
12. Cuando este paciente ingiera sus alumentos, se espera que al momento de pasar el
bolo aluimenticio por el esfínter esofágico superior, la presión intraesofagica
disminuya en:
a) El cadias
b) Tercio medio del esofago
c) El lugar donde se contraiga la muscular propia
d) Porcion proximal al bolo
13. Respecto a la motilidad del colon, marque lo correcto
a) Con la distencion del estomago, suelen aparecer movimientos en masa
b) Se producen contracciones segmentarias principalmente en el colon izquierdo
c) Los movimientos de masa se encargan del mezclado de las heces
d) Las haustras del colon contribuyen al reflejo de defecación
14. Durante la mezcla y digestion, la fuerza de la contracción puede aumentar gracias a la
acción de la:
a) Gastrina
b) Estimulacion simpática
c) Secretina
d) VIP
15. Para estimular la motilidad intestinal se podría usar un analago de:
a) Secretina
b) CCK
c) Glucagon
d) Noradrenalina
16. Debido al acumulo de material fecal en todo el marco colonico, y a la irritación química
asociada, el peristaltismo del ileon distal se debe encontrar:
a) Inhibido
b) Estimulado
c) No sufre alteraciones
d) Afectado por un reflejo vago – vagal
17. Durante la fase faríngea de la deglución se espera que ocurra el siguiente fenómeno:
a) El paladar blando cierra la entrada a la nasofaringe
b) Se libera acetilcolina en musculo cricofaringeo
c) La onda peristáltica primaria lleva el alimento hacia el esófago
d) Inicialmente se desplaza el hueso hioides hacia abajo
18. Considerando que este paciente esta sometido a estrés por el agravamiento de su
enfermedad, es posible afirmar que sus ondas lentas están:
a) Desmotivadas
b) Hipopolarizadas
c) Despolarizadas
d) Hiperpolarizadas

Ci 1
1. Durante una cirugía oncológica, ¿La extirpación de cual de los siguientes órganos se
vería comprometida por la presencia de adventicia?
a) Vesícula biliar
b) Estomago
c) Recto
d) Yeyuno
2. Maque la respuesta correcta en relación a la gastrina:
a) Las células G son las productoras y se encuentran principalmente en el antro gástrico
b) Las células G se encuentran principalmente en el fondo gástrico
c) Se estimula por la liberación de noradrenalina
d) Al distenderse el estómago, se inhibe su producción
3. En relación con los principios de motilidad, marque lo incorrecto
a) El musculo liso del tubo digestivo funciona como un sincilio
b) Es regulado por calcio
c) Las dos terceras partes proximales del esófago están conformadas por musculo
esquelético
d) Se produce mayor fuerza de contracción que en el musculo esquelético
4. Para poder morder una manzana, es necesario el siguiente musculo:
a) Milohioideo
b) Buccinador
c) Tensor del paladar
d) Orbicular de los labios
5. El consumir caramelos indirectamente activa la vía:
a) POMC/CART
b) AGRP/NPY
c) MCR-4
d) Grelina
6. ¿En que casos los vómitos son siembre biliosos?
a) Estenosis
b) Atresia esofágica
c) Atresia yeyunal
d) Atresia duodenal
7. Las arterias que derivan del tronco celiaco son, excepto:
a) Gástrica izquierda
b) Hepática derecha
c) Arteria esplénica
d) Hepática común
8. Entre las múltiples causas de la enfermedad por reflujo gastroesofágico, se puede
considerar también a una alteración en las ________ del esfínter esofágico inferior.
a) Contracciones tónicas
b) Ondas secundarias
c) Ondas lentas
d) Glándulas subesofagicas
9. Con respecto a las ondas lentas, maque la afirmación correcta:
a) Su frecuencia aumenta por acción de la acetilcolina
b) Su frecuencia disminuye por acción de la noradrenalina
c) Son potenciales de acción que producen la contracción del tracto gastrointestinal
d) Son contracciones rítmicas espontaneas
10. Producto de la alimentación, se producen diversas sustancias peptídicas, cininas y
bradicininas, las cuales permiten que:
a) El flujo sanguíneo intestinal aumente hasta 8 veces
b) La acción de la lipasa pancreática se vea incrementada
c) Se produzca neovascularización en los territorios de las arterias abdominales
d) El consumo de O2 del intestino aumente ligeramente
11. Cual de los siguientes péptidos inhibe el vaciamiento gástrico
a) Gastrina
b) Péptido inhibidor gástrico
c) CCK
d) Motilina
12. Sustancia que inhibe la secreción y la motilidad del estomago prolongando el tiempo
de digestión
a) GIP
b) Polipéptido pancreático
c) GLP-1
d) Enteroglucagón
13. En cuanto a los reflejos gastrointestinales, un reflejo que estimula el transito intestinal
es el reflejo:
a) Gastro cólico
b) Entero-gástrico
c) Vomito
d) Cólico-ileal
14. ¿Cuál de los siguientes líquidos corporales tiene el pH más alto?
a) Bilis de la vesícula biliar
b) Jugo pancreático
c) Saliva
d) Jugo gástrico
15. El crecimiento de un adenoma de cuello de páncreas puede comprometer la pared
gástrica por continuidad, ¿Qué parte del estómago estaría comprometida con mayor
probabilidad?
a) Pared anterior del píloro
b) Pared posterior del cuerpo
c) Pared anterior del cardias
d) Pared posterior del antro
16. Marque la respuesta correcta
a) La pared gástrica en el fondo es mas delgada que en el cuerpo y antro
b) Todos los órganos del sistema digestivo tienen capa serosa
c) El bronquio derecho constituye una de las estrecheces del esófago
d) El esfínter de Oddi rodea la papila menor duodenal
17. Un paciente con apendicitis agua, la sensación de dolor por esta inflamación es llevada
por el nervio:
a) Pélvico
b) Esplácnico menor
c) Vago
d) Esplácnico mayor
18. Estimulan la secreción gástrica
a) Proteínas
b) Soluciones inertes
c) Soluciones hipertónicas de glucosa
d) Grasas
19. El dolor periumbilical o epigástrico en el inicio de una apendicitis aguda se debe a
a) Estimulación del nervio vago
b) Irritación del peritoneo parietal
c) Estimulo del sistema simpático
d) Íleo secundario
20. El uso de ranitidina bloquea el receptor H2 de la histamina en las células parietales. La
histamina llega a estas células por
a) Vía hematógena
b) Difusión
c) Se produce en la misma célula parietal
d) La luz gástrica
21. El nacimiento de la arteria mesentérica superior se puede encontrar en cual de los
cuadrantes abdominales:
a) Mesogastrio
b) Hipocondrio derecho
c) Hipogastrio
d) Epigastrio
22. Con respecto las ondas lentas, una estimulación adrenérgica producirá:
a) Hiperpolarización
b) Mas espigas
c) Una contracción más sostenida
d) Despolarización
23. Al delgutir el bolo alimenticio, es lógico suponer que al pasar por el esofago haya un
mayor consumo de oxigfeno en la pared del tercio:
a) No hay diferencia
b) Medio
c) Proximal
d) Distal
24. La arteria mesentérica superior emerge de la aorta a nivel de
a) Cabeza de pancreas
b) Hilio hepático
c) Cardias
d) Tronco celiaco
25. La hernia fisiológica se produce dentro de:
a) Cordon umbilical
b) Saco amniotico
c) Saco vitelino
d) Alantoides
26. El ligamento falciforme divide al hígado en dos lobulos derecho e izquierdo.
Embriologicamente derivan de:
a) Mesentrio ventral
b) Fascia de Todd
c) Mesogastrio dorsal
d) Vena umbilical
27. Paciente de sexo masculino de 43 años que es alimentado por via intravenosa durante
varias semanas. Producto de este tipo de alimentación, se encuentra en la endoscopia
atrofia de la mucosa antral. La causa mas probable de esta alteración es debido a los
bajos niveles de:
a) Gastrina
b) Secretina
c) GIP
d) CCK
28. Aproximadamente en la semana 6 del desarrollo embrionario, el intestino intermedio
gira 90° herniándose a nivel del:
a) Cordón umbilical
b) Borde inferior del bazo
c) Borde superior del hígado
d) Borde superior del pubis
29. Al comer unas papitas fritas con mayonesa, el vaciamiento gástrico disminuye por
efecto directo de la hormona:
a) CCK
b) Somatostatina
c) Secretina
d) Motilina
30. El crecimiento de un adenocarcinoma de pancreas compromete la pared gástrica por
contigüidad. ¿Qué parte del estomago se esperaría este comprometido?
a) Pared posterior del antro
b) Pared posterior del fondo
c) Pared posterior del píloro
d) Pared posterior del cardias
31. Al deglutir un bolo alimenticio, es lógico suponer que al pasar por el esofago haya un
mayor consumo de oxigeno en la pared del tercio:
a) Proximal
b) No hay diferencia
c) Distal
d) Medio
32. Estudiante de medicina de 20 años, se ha amanecido estudiando para su examen. No
ha probado alumento desde la cena, por lo que se puede afirmar que la motilidad de
esta persona esta siendo regulado por:
a) VIP
b) CCK
c) Motilina
d) Adrenalina
33. Paciente con disminución del apetito marcada asociada a cancer terminal, para
promover la ingesta de alimentos se podría usar analogos de:
a) Endorfinas
b) CCK
c) Secretina
d) GLP
34. En un paciente con tumor carcinoide de pancreas productor de gastrina (síndrome de
Zollinger- Ellison) se puede esperar una disminución en la:
a) Secreción de enteroglucagón
b) Liberacion de acido clorhidrico
c) Liberacion de gastrina por las células G
d) Secrecion de secretina
35. Con respecto a las contracciones de segmentación, es cierto que
a) No produce movimiento anterógrado
b) Impiden la mezcla del alimento
c) La relajación del segmento se debe a la acción de la actilcolina
d) Se produce solo en el colon
36. Al comer unas papitas fritas con mayonesa, el vaciamiento gastrico diminuye por
efecto directo de la hormona:
a) CCK
b) Motilina
c) Somatostatina
d) Secretina
37. Paciente que como una pizza entera de chorizo y queso. Es posible esperar que debido
a la cantidad de alimento ingerido, las ondas lentas hayan:
a) Disminuido su frecuencia por estimulo parasimpatico
b) Aumentado su frecuencia por estimulo simpatico
c) Aumentado su frecuencia por estimulo simpatico
d) Sufrido ninguna alteración en su frecuencia

PARCIAL DE SISTEMA DIGESTIVO 2019 - 00


1. Varón de 30 años es traído a emergencia por agresión abdominal con arma de fuego (pistola) y es
sometido a laparotomía exploratoria, observándose isquemia del colon ascendente y parte del
colon trasverso ¿la lesión de cuál de las siguientes arterias explicaría esta isquemia?
(unidad 1, sesión 2, logro 6: (Describe la irrigación visceral: arterias de tronco celiaco, arteria
mesentérica superior e inferior, topografía de superficie, órganos por cuadrante)
a) Celiaca
b) Colónica derecha
c) Mesentérica inferior
d) Mesentérica superior

2. Respecto a las sustancias gastrointestinales que regulan la secreción pancreática; marque la


afirmación correcta:
(unidad 1, sesión 3, logros 2 y 3: describir las hormonas gastrointestinales: estímulos y funciones)

a) La Secretina, es la hormona más importante para la secreción de bicarbonato por las células
acinares del páncreas
b) La acetilcolina es capaz de estimular la secreción enzimática y de bicarbonato del páncreas
c) La gastrina, es la hormona más importante para la secreción de enzimas pancreáticas
d) La colecistoquinina (CCK) estimula al páncreas solo para secreción enzimática
3. Ante una lesión del X par craneal, ¿cuál de los siguientes músculos mantiene conservada su
función?:
(unidad 2, sesión 08, logro 4: Paladar blando: componentes musculares)

a) Elevador del velo del paladar


b) Tensor del velo del paladar
c) Palatofaríngeo
d) Glosofaríngeo

4. Experimentalmente se utiliza atropina (anticolinérgico) para inhibir la secreción de gastrina, sin


embargo, la secreción de esta hormona se sigue dando ante estímulos vágales. Esta situación se
explica porque la atropina:
(unidad 1, sesión 3, logro 3: describir las hormonas gastrointestinales: estímulo y funciones de la gastrina
y colecistoquinina)

a. Bloquea parcialmente la bomba de protones en la célula G


b. Inhibe la acción de acetilcolina e histamina en la célula G
c. Solo inhibe la acción del péptido GRP en la célula G
d. No bloquea la acción del péptido GRP

5. Un varón de 50 años es sometido a extirpación del duodeno y parte proximal del yeyuno. Esta
situación ocasionaría la pérdida de las células ………. , productoras deque estimula la secreción de
bicarbonato por el páncreas.
(unidad 1, sesión 3, logro 3: describir las hormonas gastrointestianles: estímulos y funciones de la
secretina y péptido insulinotrópico dependiente de glucosa)

a) “S” / secretina
b) Parietales / secretina
c) “I” / colecistoquinina
d) “S” / colecistoquinina

6. Recién nacido que presenta tumoración abdominal a nivel del cordón umbilical (fotografía).
¿cuál de las siguientes afirmaciones es correcta respecto a este defecto en el desarrollo
embriológico del intestino?: (unidad 1, sesión 5, logro 2: identificar las anomalías del desarrollo
del intestino medio)
a) Corresponde a una Gastrosquisis
b) Las vísceras se hallan cubiertas por piel
c) No está asociado a otras malformaciones
d) Se asocia a malformaciones
cardiacas y del tubo neural
7. Varón de 35 años acude a la emergencia por trauma
abdominal y se decida realizar una laparoscopía exploratoria.
El cirujano observa la disposición de los órganos abdominales
como se representa en el siguiente esquema. Esta disposición
de órganos se explica por la rotación (SMA=arteria
mesentérica superior)
(unidad 1, sesión 5, logro 3: identificar las anomalías del desarrollo del
intestino medio: defectos de rotación, estenosis y atresias)
a) antihoraria del intestino medio, en sólo 90°
b) incompleta del intestino medio (270°)
c) horaria del intestino medio
d) horaria del estómago

8. Se evalúa la expresión de la proteína Agrp en una persona con alteración del apetito; lo
correcto respecto a esta proteína es…..
(unidad 1, sesión 3, logro 4: Explica los mecanismos de control del apetito y saciedad )
a) Esta proteína es un potente anorexigénico
b) La mutación del gen que la codifica produce adelgazamiento
c) La sobre producción de la proteína lleva a obesidad por agonismo de receptores MC3 y MC4
d) La sobre producción de la proteína disminuye el apetito por antagonismo de receptores MC4

9. Juana cae de la bicicleta y se fractura la región anterior del hueso maxilar superior con
compromiso de la fosa incisiva. Al examen físico de la región esperaría encontrar alteración en la sensibilidad
de la encía …………………
(unidad 2, sesión 8, logro5: paladar: paladar duro y blando: irrigación e inervación)

a) bucal posterior
b) Lingual anterior
c) palatina anterior
d) palatina posterior

10. Recién nacido es atendido por el neonatólogo y luego entregado a su madre para dar de lactar; la
madre al dar de lactar observa coloración azulada de labios, acompañado de tos persistente,
dificultad respiratoria y distención abdominal. Se le intenta colocar una sonda nasogástrica, pero
esta retorna a la cavidad oral en todos los intentos. ¿Cuál de las siguientes anomalías del
desarrollo es el más probable en este caso? (unidad 1, sesión 4, logro 3: identificar las anomalías en
el desarrollo del esófago: atresia y/o fístula traqueo esofágica)

a) Estenosis esofágica proximal con Fístula traqueo esofágica distal


b) Atresia esofágica proximal con fístula traqueoesofágica distal
c) Atresia esofágica distal con fístula traqueoesofágica proximal
d) Fístula traqueoesofágica proximal y distal

11. ¿Cuál de los siguientes mecanismos ocurre durante la defecación?


(unidad 2, sesión 13, logro 6: motilidad del intestino grueso: contracciones segmentarias, movimientos
en masa, defecación y reflejo gastrocólico)

a) Contracción refleja del esfínter anal interno


b) En la posición de “cuclillas” el músculo puborectal se halla relajado
c) Relajación del esfínter anal externo por efectos del VIP y óxido nítrico
d) La materia fecal en el recto estimula la contracción del sigmoides por los nervios pudendos

12. La estructura número 4 (gráfico) corresponde a


……….… y está ………..
(unidad 2, sesión 9, logro 2: Partes de un diente.
Capas del diente: Esmalte: características y células
que lo producen)

a) el cemento / mineralizado en 90%


b) la dentina / formada por ameloblastos
c) el esmalte / formado por células
derivadas de la mesénquima
d) la dentina / formado por células
derivadas de la cresta neural

13. Un paciente luego de un accidente sufre lesión del piso de la boca, se constata daño del nervio “cuerda
del tímpano”, en este caso se esperaría encontrar disminución de la ....................de la lengua
(unidad 2, sesión 10, logro 3: Irrigación e inervación de la lengua)

a) Motilidad en los dos tercios anteriores


b) Sensación del gusto en el tercio posterior
c) Sensación del gusto en los dos tercios anteriores
d) Sensibilidad al tacto en los dos tercios anteriores

14. ¿Cuál de las siguientes afirmaciones es la correcta sobre la gastrina?


(unidad 1, sesión 3, logro 1: reconocer las características de las sustancias reguladoras
gastrointestinales: hormonas, sustancias paracrinas y neurocrinas)
a) Produce atrofia de la mucosa gástrica
b) Es producida por la célula G del cuerpo gástrico
c) Es estimulada por la distensión gástrica y el Ph bajo
d) Actúa en la célula diana mediante su receptor CCk tipo B
15. Al recibir un paciente con signos de hipovolemia y antecedente de trauma en abdomen por
accidente de tránsito, usted identifica radiológicamente: lesión de primera vértebra lumbar y
signos de lesión en páncreas; durante la cirugía se observó pobre irrigación de asas intestinales. El
vaso afectado es la arteria …………….
a) esplénica
b) hepática común
c) mesentérica inferior
d) mesentérica superior
16. Un paciente sufre de daño a nivel del cuello con lesión muscular en la región de la faringe. En el
examen físico se determina dificultad para la elevación de la faringe y para el cierre del itsmo de
las fauces. En este caso, probablemente esté afectado el músculo:
(unidad 2, sesión 11, logro 2: Músculos de la faringe: identificación, constrictores y longitudinales)
a) palatogloso
b) estilofarinfeo
c) palatofaringeo
d) constrictor inferior

17. Varón de 50 años a quien le realizan la curación de la segunda molar de la arcada superior
derecha. En un momento determinado, el paciente acusa de intenso dolor de la pieza dentaria
en tratamiento. La vía aferente del dolor viaja a través del nervio …………
(unidad 2, sesión 9, logro 6: Inervación de los dientes)
a) trigémino V2
b) trigémino V3
c) naso palatino
d) palatino menor

18. La distención gástrica por los alimentos produce incremento de secreción de HCl mediante la
producción de …………. que estimula a las células ……………. vía proteína ……….
(Unidad 1, sesión 3, logro 2: Describe las hormonas gastrointestinales: Estímulo y funciones de la
gastrina y colecistoquinina)
a) gastrina / parietal / Gq
b) gastrina / principal / Gs
c) acetilcolina / parietal /Gi
d) acetilcolina / principal / Gi

19. Un niño de tres años llega a emergencia con disfagia (dificultad para tragar), dolor retro esternal,
salivación y llanto. Se sospecha de ingesta de cuerpo extraño (moneda) en el esófago; al ser
evaluado se constata en una radiografía presencia de cuerpo extraño a nivel de C6 (6° vértebra
cervical). El cuerpo extraño estará suspendido a nivel del estrechamiento producido por………..
(unidad 2, sesión 11, logro4: Esófago, características anatómicas, relación con órganos vecinos y
estrecheces)
a) el cayado aórtico
b) el hiato esofágico
c) el músculo cricofaríngeo
d) el bronquio principal izquierdo

20. La triada portal (arteria hepática, vena portal y conducto biliar común) está contenida en el
ligamento
…….……… y derivan embriológicamente del ……
(Unidad 1, sesión 1, logro 4: Identifica el peritoneo, mesenterio, omento y ligamentos, retroperitoneo.)

a) hepato duodenal / mesenterio ventral


b) gastro esplénico / mesenterio dorsal
c) hepato gástrico / omento menor
d) falciforme / omento menor
21. En relación al movimiento de peristaltismo del
tubo digestivo: en la flecha negra del gráfico se
produce la liberación de ……………… a nivel del
músculo ………..
(unidad 2, sesión 7, logro 6: Control hormonal y tipos de
movimiento)
a) noradrenalina, sustancia P y neuropéptido “ Y” / circular
b) acetilcolina y sustancia P / longitudinal
c) óxido nítrico y PIV / longitudinal
d) óxido nítrico y PIV / circular
22. Un paciente refiere no percibir algunos sabores, al examen físico se constata alteración en la
percepción de sabores y del dolor en el tercio posterior de la lengua ¿Cuál de los siguientes
nervios estará alterada en su función?
(unidad 2, sesión 10, logro 5: Sabores, tipos y mecanismos moleculares para su detección)

a) Lingual (rama del V par)


b) Cuerda del tímpano (VII par)
c) Glosofaríngeo (IX par)
d) Hipogloso (XII par)

23. El gráfico detalla la estructura de la pared del


tubo digestivo intestinal ¿Cuál de las siguientes
asociaciones es correcta?
(unidad 2, sesión 7, logro 1: La pared y músculo
liso gastrointestinal )

a) “1” – peristaltismo
b) “2” – secreción enzimática
c) “3” – deriva del mesodermo
d) “4” – doble hoja de tejido graso

24. En el caso de un paciente con gastrinoma (tumor productor de gastrina), la presencia de úlceras
duodenales y erosión de la mucosa gástrica, se debe principalmente a…….
(unidad 1, sesión 3, logro 2: describir las hormonas gastrointestinales: estímulo y funciones de la gastrina
y colecistoquinina)

a) la acción directa de la gastrina sobre la célula principal


b) la sobre expresión de los receptores “G” en la célula parietal
c) el exceso de HCl por estímulo de receptores CCK-B en la célula parietal
d) el exceso de HCl por estímulo directo de receptores de acetilcolina en la célula parietal

25. El reflejo entero gástrico se caracteriza por:


(unidad 2, sesión 13, logro 6: Motilidad del intestino grueso: contracciones segmentarias, movimientos
en masa defecación y reflejo gastrocólico)

a) favorecer la motilidad gástrica gracias a la CCk


b) inhibir la motilidad gástrica y estimular la secreción ácida
c) movilizar grandes volúmenes desde el estómago al duodeno
d) originarse debido a la distensión duodenal y presencia del quimo ácido
26. Mauricio tiene dificultad para deprimir el paladar y elevar la parte posterior de la lengua. En este
caso estará afectado un músculo ………………., específicamente el músculo …………….
(Unidad 2, sesión 10, logro 2: Músculos de la lengua: clasificación, identificación y sus funciones)

a) intrínseco – longitudinal inferior


b) extrínseco – palatogloso
c) extrínseco – transverso
d) extrínseco – estilogloso

27. Una de las funciones del músculo señalado es:


(Unidad 2, sesión 8, logro 3: Describir el Piso de la boca:
estructuras blandas que la conforman)

a) deprimir la lengua
b) elevar el paladar blando
c) deprimir el hioides cuando la mandíbula está fija
d) deprimir la mandíbula cuando el hioides está fijo

28. Paciente varón de 30 años es evaluado por probable enfermedad de Chagas, cursa con
problemas de motilidad del colon; los estudios de biopsia determinan ausencia de células
ganglionares. Según el gráfico
¿cuál es la capa en la que se determina la ausencia de dichas células?
(unidad 1, sesión 2, logro 1: describir las generalidades de la estructura del tubo digestivo: esófago,
estómago intestino delgado y grueso)

a) Mucosa - 1
b) Muscular propia – 1
c) Muscular de la mucosa - 2
d) Muscular propia - 3

1 2

29. Paciente varón de 32 años, que acude a centro de salud por presentar de forma progresiva desde
hace 1 año dificultad para ingerir alimentos sólidos y luego líquidos; refiere regurgitaciones
alimentarias y marcada pérdida de peso (15 kilos). Radiografía baritada (sustancia de contraste)
de esófago se muestra en la figura. El presente caso se explica por……………….
(Unidad 2, sesión 12, logro 4: Identificar y describir la función de los esfínteres esofágicos)

a) aumento de la peristalsis esofágica


b) relajación incompleta del esfínter pilórico
c) relajación incompleta del esfínter esofágico inferior
d) perdida de producción de PIV y óxido nítrico en el
esfínter esofágico superior

30. En condiciones normales, el ingreso de 600 ml de líquido es el estómago provoca un aumento de


presión intragástrica de unos 12 cm de H2O. Después de una vagotomía (corte del nervio vago)
es de esperar que el ingreso del mismo volumen de líquido ocasione de la presión intragástrica.
(Unidad 2, sesión 13, logro 1: Describe la Motilidad gástrica: relajación receptiva)

a) la disminución
b) la no variación
c) un aumento mayor
d) un aumento similar o igual

Examen parcial 2019-2


1. La contracción del músculo ....................... permite la eliminación de gases (flatos) sin salida de material
fecal;
es el mismo músculo cuya relajación, sobretodo en cuclillas, permite el paso del contenido
fecal con menor esfuerzo durante la defecación.
a) Isquirectal
b) Puborrectal
c) Esfínter anal externo
d) Esfínter anal interno
2. Paciente mujer de 54 años se presenta con náuseas, vómitos, estreñimiento, y es diagnosticada
de abdomen agudo quirúrgico; en la cirugía encuentran un vólvulo de ciego. Esta anomalía
puede explicarse por:
a) Falta de rotación intestinal
b) Falta de fusión del mesenterio
c) Defecto en la formación de la cloaca
d) Falta de formación del omento mayor

3. Paciente mujer de 23 años con faringitis aguda, toma para el dolor una tableta de paracetamol
con un poco de agua. Durante la deglución, se relaja su esfínter esofágico inferior y el fondo del
estómago, mientras el bolo está aún en el esófago. ¿Qué sustancia provocara con mayor
probabilidad la relajación del esfínter esofágico inferior y el fondo del estómago en esta mujer?
a) Óxido nítrico
b) Sustancia P
c) Histamina
d) Motilina
4. Luego de tres horas dando exámenes, un alumno de medicina comienza a sentir hambre.
Esta situación es probable que sea mediada por la que
es sintetizada por el :
a) leptina / intestino
b) leptina / estómago
c) grelina / estómago
d) grelina / tejido adiposo

5. Varón de 72 años, con antecedente de diabetes mellitus tipo 2, que presenta enteropatía diabética
caracterizada por estreñimiento. Este problema puede estar asociado a:
a) deficiencia de óxido nítrico
b) aumento del reflejo gastrocólico
c) disminución de la secreción de colecistocinina (CCK)
d) aumento de la secreción del péptido intestinal vasoactivo (PIV)

6. Varón de 54 años con Diabetes Mellitus tipo 2, es diagnosticado de gastroparesia debido a que
presenta sensación de llenura precoz al comer, y reflujo gastroesofágico. Esta alteración en la
relajación receptiva y en el vaciamiento gástrico lo más probable es que se deba a una alteración
en:
a) el nervio vago
b) el ganglio celíaco
c) plexo submucoso
d) nervio hipogástrico
7. Varón de 67 años con tos y disminución de peso asociado a tabaquismo pesado, presenta
actualmente disfagia progresiva a alimentos sólidos. Se considera la presencia de un carcinoma de
bronquio izquierdo y por esta razón le realizan una endoscopía esofágica para descartar la
posibilidad de una compresión esofágica por el tumor. Se espera revisar el esófago en la
estrechez, que está a nivel de la vértebra
a) Tercera estrechez -T6
b) Segunda estrechez - C6
c) Segunda estrechez - T4
d) Tercera estrechez -T10

8. Varón de 34 años con dolor abdominal agudo en flanco derecho que se irradia a fosa ilíaca
derecha, es operado y se encuentra un divertículo intestinal inflamado, ubicado a 93 cm de la
válvula ileocecal. El origen de este divertículo es una falla en la obliteración de:
a) Conducto vitelino
b) Alantoides
c) Cloaca
d) Conducto anorectal
e) Uraco
9. En un niño menor de dos años con divertículo intestinal, este divertículo tiene su origen en una falla
en la obliteración de:
a) Conductoanorectal
b) Conducto vitelino
c) Alantoides
d) Cloaca
e) Uraco

10. Mujer de 43 años sufre un grave accidente de tránsito y está hospitalizada en coma, es alimentada
por vía intravenosa durante varias semanas. Producto de este tipo de alimentación, se encuentra en
la endoscopía atrofia de la mucosa gastrointestinal. La causa más probable de esta atrofia son los
bajos niveles séricos de la hormona:
a) Colecistocinina
b) Secretina
c) Gastrina
d) PIV
11. Una mujer de 30 años llega al consultorio porque se queja de dificultades para deglutir, la cual se
agravan cada vez más. Se realiza un estudio manométrico para examinar la generación de presión
a lo largo del esófago. Esta prueba revela que las contracciones como respuesta a la deglución
están mal sincronizadas y que la presión en el esfínter esofágico inferior permanece elevada. El
diagnóstico más probable es producido por niveles bajos de
a) acalasia / sustancia P
b) acalasia / óxido nítrico
c) enfermedad por reflujo gastrointestinal / acetilcolina
d) enfermedad por reflujo gastrointestinal / óxido nítrico

12. Paciente de 2 años, llega a emergencia por haber ingerido una moneda con la que estaba
jugando. El lugar más probable donde puede haberse quedado suspendido este objeto es a nivel
del estrechamiento producido a nivel del:
a) músculo milohiodeo
b) músculo aritenoideo
c) músculo cricofaríngeo
d) constrictor superior de la faringe

13. En una apendicectomía, al realizar la incisión de McBurney en la fosa iliaca derecha, es


necesario cortar los siguientes músculos, de afuera hacia adentro:
a) Recto – Oblicuo externo – Transverso
b) Recto – Oblicuo externo – Oblicuo interno
c) Oblicuo externo – Oblicuo interno – Recto
d) Oblicuo externo – Oblicuo interno – Transverso

14. Un varón de 90 años que se encuentra postrado en cama, es referido del asilo para endoscopia por
dificultad para deglutir luego de tomar un medicamento para aliviar el dolor la noche anterior. La
endoscopía revela que la píldora se alojó en el esófago y causó una reacción inflamatoria. Lo más
probable es que esto haya sido por la producción de múltiples ondas:
a) secundarias
b) primarias
c) lentas
d) segmentarias

15. Mujer de 23 años es diagnosticada de bulimia, al examen físico se observa ulceraciones en el


segundo y tercero dedo de la mano derecha. Esto se puede deber al uso continuo de estos dedos
para inducir el vómito, mediante la estimulación del par craneal:
a) V
b) IX
c) X
d) XI

16. Varón de 52 años se presenta por diarrea persistente de seis semanas de duración. En la
colonoscopia se observa un pólipo a nivel del íleon distal. El patólogo informa que se trata de un
tumor neuroendócrino, probablemente originado por las células enterocromafines del intestino.
La sustancia que más probablemente esté produciendo este tumor es:
a) Serotonina
b) Insulina
c) CCK
d) GIP

17. La fase oclusal de la masticación se realiza con la contracción de los músculos:


a) digástricos
b) masetero y temporal
c) orbicular y buccinador
d) pterigoideo lateral y digástrico

18. Al tomar su café en Starbucks, un estudiante de medicina sufre una quemadura de primer
grado en el tercio anterior de la superficie dorsal de la lengua. La información de dolor es
transmitida por el nervio:
a) cuerda del tímpano
b) glosofaríngeo
c) lingual
d) facial

19. Paciente es evaluado por faringitis aguda en consultorio externo. El médico de familia le solicita
que abra la boca y saque la lengua. Para realizar la acción de sacar la lengua, es necesario que se
contraiga el músculo:
a) Estilogloso
b) Geniogloso
c) Palatogloso
d) transverso de la lengua
20. Paciente con síndrome de Sjögren, presenta “boca seca”
(disminución de la producción de saliva) y caries dental,
asociada a la pérdida de la función de tampón de la saliva. Esta
desminerilización del diente puede comprometer a las
prolongaciones citoplasmáticas ubicadas en los tubos huecos de
la estructura señalada con la letra:
a) B
b) A
c) E
d) C

21. Mujer de 32 años acude a consulta por presentar disfagia de


progresión lenta, reflujo gastroesofágico y vómitos desde hace 3
meses de evolución progresiva. Se le realiza un estudio
radiológico con contraste en el que se observa estrechamiento
del esfínter esofágico inferior (imagen). Según sus conocimientos,
este paciente se beneficiaría con el uso de:
a) agonista beta adrenérgico
b) agonista alfa adrenérgico
c) análogo de óxido nítrico
d) análogo de Sustancia P
Parcial 2020 – 1
1. ¿Cuál de las siguientes alternativas se define como la protrusión directa del contenido abdominal
a la cavidad amniótico por un defecto de la pared corporal?
a) Gastrosquisis
b) Onfalocele
c) Hernia inguinal
d) Hernia femoral
2. Los movimientos en masa son iniciados por el reflejo:
a) Duodenocólico
b) Cólicoileal
c) Del vomito
d) Duodenogástrico
3. En una persona si enfermedad se espera que el tránsito intestinal se vea disminuido cuando se
presenta el reflejo:
a) Cólicoileal
b) De defecación
c) Doloroso
d) Gastrocólico
4. El peristaltismo del íleon es intensificado gracias al reflejo:
a) Cólicoileal
b) Gastroileal
c) Duodenogástrico
d) Gastrocolico
5. Un paciente requiere que se le coloque una sonda de alimentación directamente al estómago
(gastrostomía), el cirujano deberá hacer una incisión en la piel del abdomen ¿Cuál de las
siguientes raíces nerviosas debe ser anestesiada para este procedimiento?
a) T8
b) T12
c) T10
d) L1
6. Las glándulas salivales tienes conductos para la excreción de la saliva; las glándulas _______
drenan en las carúnculas sublinguales.
a) Menores
b) Parótidas
c) Submaxilares
d) Sublinguales
7. Durante el desarrollo intrauterino del páncreas, cuando la porción izquierda de la yema
pancreática ventral migra en dirección opuesta, se forma el páncreas:
a) Anular
b) Derecho
c) Ectópico
d) Divisum
8. ¿Cuál de las siguientes alternativas se define como la herniación de las visceras abdominales por
un anillo umbilical agrandado?
a) Gastrosquisis
b) Hernia femoral
c) Hernia inguinal
d) Onfalocele
9. Durante la fase faríngea de la deglución ocurre el siguiente mecanismo:
a) Los músculos infrahioideos hacen que se desplaza la laringe hacia abajo
b) La onda peristáltica lleva el alimento hacia el esófago
c) El péptido inhibidor vasoactivo (VIP) actúa sobre el musculo constrictor superior
d) El paladar blando abre el istmo de las fauces
10. El peristaltismo del intestino delgado se puede intensificar debido a:
a) Somatostatina
b) Irritación de la mucosa
c) Secretina
d) Noradrenalina
11. Al evaluar la orofaringe de un paciente, el medico solicita que abra la boca, saque la lengua y diga
a. Al hacer la maniobra, nota que el paladar se desvía hacia la derecha, lo cual le hace sospechar
que el paciente sufre de una lesión del nervio craneal:
a) XII contralateral
b) X contralateral
c) V3 contralateral
d) VII ipsilateral
12. El divertículo de Meckel es una anomalía congénita que ocurre por la persistencia del conducto
vitelino y da origen a una estructura sacular, el cual se encuentra en el
a) Duodeno
b) Apéndice cecal
c) Borde antimesentérico
d) Borde mesentérico
13. En un paciente con enfermedad de Hirschprung (megacolon aganlionico) con dilatación de los dos
tercios proximales del recto, la dilatación de este segmento se ha producido con mayor
probabilidad debido a los movimientos:
a) Peristálticos
b) En masa
c) De retropulsión
d) Segmentario
14. Cuando el tubo digestivo esta en reposo, los complejos mioeléctrico migratorios son
desencadenados por:
a) La sustancia P
b) La serotonina
c) La motilina
d) El péptido
15. Paciente con insuficiencia mitral moderada a severa, con aumento de la aurícula izquierda; esta
condición tendrá como consecuencia a nivel del sistema digestivo:
a) La estenosis hipertrófica de los cardias
b) La disfagia a solidos
c) La acalasia
d) Alteración de la fase faríngea de la deglución
16. Los diferentes segmentos del tubo digestivo son susceptibles de reflejos y movimientos según su
contenido. Si colocáramos mediante una sonda un bolo alimenticio directamente en el tercio
medio del esófago:
a) Se producirá ondas primarias
b) Se producirá ondas secundarias
c) Sera necesario un reflejo vago – vagal para estimular la deglución
d) Se producirá la acción voluntaria del musculo estriado
17. Durante el desarrollo del tubo digestivo en la vida intrauterina, cuando el asa intestinal rota 90° en
el sentido de las manecillas del reloj entonces:
a) Se desarrolla un situs inversus
b) El colon pasa por detrás del duodeno
c) Se forma el colon izquierdo
d) Se realiza el plegamiento normal de los intestinos
18. En un paciente con tumor carcinoide de páncreas productor de gastrina (Sindrome de Zollinger –
Ellison) se puede esperar que ocurra un aumento en la:
a) Liberación de gastrina por las células G
b) Secreción de bombesina
c) Absorción de los lípidos de la dieta
d) Secreción de secretina
19. La mucosa irrigada por la arteria palatina mayor esta recubierta por un epitelio:
a) Plano escamoso no queratinizado
b) Cilíndrico simple no ciliado
c) Plano simple
d) Plano escamoso queratinizado
20. La irritación del peritoneo intestinal, como la que sucede en una peritonitis, producirá:
a) La estimulación del peristaltismo
b) La destrucción de las células intersticiales de Cajal
c) El aumento de las contracciones tónicas
d) La inhibición del peristaltismo
21. En un paciente de 43 años con tumor carcinoide páncreas productor de gastrina (Síndrome de
Zollinger – Ellison) se puede esperar encontrar una potenciación del reflejo:
a) De relajación receptiva
b) Gastrocolico
c) Del vomito
d) Ileocólico
22. Durante el desarrollo intrauterino del páncreas, el segmento proximal del conducto pancreatico
dorsal se convertirá en:
a) Papila mayor
b) La ampolla de Vater
c) El segmento proximal del conducto pancreático principal
d) El conducto pancreático accesorio
23. Cada segmento del tubo digestivo tiene características diferentes respecto de su motilidad. ¿Cuál
de las siguientes alternativas es una característica de la motilidad del colon?
a) Se producen contracciones segmentarias principalmente en el colon izquierdo
b) Los movimientos de masa se encargarán del mezclado de las heces
c) Al distenderse el estómago, usualmente de producen movimientos segmentarios
d) Las haustras del colon contribuyen a las contracciones segmentarias
24. Durante la deglución, inmediatamente luego que el bolo alimenticio pasa por el esfínter esofágico
superior, se espera que la presión intraluminal
a) Aumente en el tercio medio del esófago
b) Aumente en la porción distal al bolo
c) Disminuya en los cardias
d) Disminuya en el tercio medio del esófago
25. La fase oclusal de la masticación se realiza con la contracción de los músculos inervados pro el
nervio craneal:
a) XII
b) VII
c) V
d) IX
26. ¿Cuál de las siguientes sustancias disminuye la fuerza de las contracciones de segmentación
a) Atropina
b) Insulina
c) CCK
d) Acetilcolina
27. Para realizar el movimiento mecánico de abrir la boca, primero se necesita:
a) La contracción de los músculos buccinadores
b) La retropulsión de la lengua
c) Elevar la laringe
d) Fijar el hueso hioides
28. El omento mayor deriva del mesenterio ____ y se inserta en el _____
a) Dorsal / estomago
b) Ventral / duodeno
c) Ventral / estomago
d) Medio / colon
29. Paciente de 24 años acude a consulta externa por presentar una fistula oronasal (comunicación
entre la cavidad oral y la cavidad nasal). Esta fistula es una consecuencia tardía de la lesión de un
vaso sanguíneo por el antecedente de haber sido aperado de paladar hendido en los primeros
años de vida, aparentemente en una campaña gratuita de corrección de paladar fisurado. ¿Cuál de
las arterias palatinas podría haberse lesionado durante esa cirugía?
a) Menor
b) Mayor
c) Rama palatina de la faríngea ascendente
d) Ascendente
30. El mecanismo de la defecación incluye la participación de diversas estructuras ¿Cuál de las
siguientes alternativas es correcta?
a) Puede ser mediado por un reflejo intrínseco
b) Se acompaña de elevación del piso de la pelvis
c) El esfínter anal interno es relajado por la acción del nervio vago
d) El esfínter anal externo es involuntario
31. Un paciente es diagnosticado con un tumor neuroendocrino productor de serotonina, esto
provocara en el sistema digestivo:
a) Diarrea
b) Vómitos
c) Odinofagia
d) Estreñimiento
32. Durante la masticación, gran parte del proceso masticatorio se debe a:
a) La actividad voluntaria
b) Reflejo masticatorio
c) La acción del nervio vago
d) La acción del VII par craneal
33. El proceso de la deglución tiene fases y la participación de muchas estructuras ¿Cuál de los
siguientes eventos ocurrirá si una persona esta hablando e inconscientemente intenta deglutir?
a) El centro respiratorio inhibirá al centro de la deglución
b) Se inhibirá la deglución hasta que deje de inspirar
c) Se interrumpirá la respiración
d) La acción del VII par inhibirá la deglución
BANCO DIGESTIVO 2020-2

1) Al examinar a un paciente, usted encuentra dolor localizado en fosa iliaca derecha y


diagnostica En este paciente, usted puede inferir.

a) El peritoneo parietal regional está afectado apendicitis.


b) El peritoneo visceral regional está principalmente afectado
c) Hay inflamación de todo el peritoneo parietal (peritonitis)
d) El diagnóstico está errado por no corresponder a la región abdominal adecuada

2) La motilidad intestinal es estimulada principalmente por el:


a) Plexo de Auerbach
b) Sistema simpático
c) Sistema parasimpático
d) Sistema piramidal

3) Durante el vómito, ¿el contenido gástrico tiene que pasar necesariamente por qué
estructura para llegar al esófago? Marque la mejor respuesta:
a) Cardias
b) Papila duodenal
c) Cuerpo
d) Antro

4) Respecto a la anatomía del estómago, marque lo correcto:


a) La incisura angularis puede estar en la curvatura mayor
b) El fondo gástrico forma la curvatura mayor
c) El píloro se encuentra en el cuerpo gástrico
d) La porción más distal del estómago es el cardias

5) Marque la respuesta incorrecta:


a) En todo el tubo digestivo, se observa dos capas de muscular propia: circular interna y
longitudinal externa
b) Fuera de la cavidad abdominal, el esófago presenta capa adventicia
c) La mucosa consta de epitelio, lámina propia y muscularis mucosae
d) El colon contiene tenias

6) Paciente se queja de dolor en hipocondrio derecho, pero superficialmente. El dermatoma


relacionado es (marque la mejor respuesta)
a) T9
b) T11
c) T10
d) T12

7) Dentro de las funciones del abdomen, se encuentra la defecación y micción, en las cuales
la presión intraabdominal debe:
a) Aumentar
b) No tiene relación el abdomen con dichas funciones
c) Disminuir
d) Mantenerse igual

8) Un alumno de medicina decide hacerse un piercing en el ombligo. Al realizarle el


procedimiento, sangra ligeramente. Esta sangre proviene de la arteria (marque la mejor
respuesta)
a) Epigástrica inferior
b) Circunfleja ilíaca superficial
c) Circunfleja ilíaca profunda
d) Torácica interna

9) Señala la respuesta correcta:


a) El apéndice cecal solo tiene serosa
b) El páncreas solo tiene adventicia
c) El colon ascendente solo tiene serosa
d) El esófago solo tiene adventicia

10) Paciente mujer es traída a emergencia por sufrir una herida contuso penetrante por
cuchillo realizada por su esposo en un ataque de celos. Se observa herida en flanco izquierdo.
Está solución de continuidad ha comprometido varios músculos de la pared abdominal,
excepto:
a) oblicuo externo
b) oblicuo interno
c) recto abdominal
d) transverso

11) Marque el órgano que se considera retroperitoneal:


a) Lóbulo izquierdo del hígado
b) Vesícula biliar
c) Sigmoides
d) Parte de la vía biliar

12) La peristalsis o peristaltismo hace referencia a:


a) Motilidad para movilizar el alimento de proximidad a distancia.
b) Motilidad para mezclado de alimentos.
c) No es parte de la motilidad
d) Motilidad para fraccionamiento de alimentos.

13) Paciente tiene una úlcera sangrante en el segundo tercio del yeyuno. La arteria de la cual
proviene la sangre arterial para dicha zona es la arteria:
a) Mesentérica superior
b) Iliaca común
c) Tronco celiaco
d) Mesentérica inferior

14) La estructura que fija órganos principalmente a la pared posterior abdominal se denomina:
a) Mesenterio
b) Omento
c) Ligamento
d) Fascia transversalis

15) Al iniciar la digestión, aumenta el consumo de oxígeno por la mucosa. Esto conlleva a una
hipoxia local, lo cual hace que se libere _____________, el cual produce vasodilatación:
a) Histamina
b) Noradrenalina
c) Adenosina
d) Colecistoquinina

16) Es inervado por aferentes somáticas:


a) Peritoneo parietal
b) Mesenterio
c) Omento
d) Peritoneo visceral

17) El ligamento hepatogástrico une el __________ con el _______ y forma la entrada al


________
a) Hígado Estómago Orificio omental
b) Estomago Higado Orificio anal
c) Estómago Hígado Orificio gastrointestinal
d) Hígado Estómago Orificio esofágico

18) Al retirar completamente el mesenterio de un órgano, el mismo se vería afectado


principalmente en su:
a) Irrigación
b) No se afecta en absoluto
c) Inervación
d) Tamaño

20) Paciente con vólvulo del colon sigmoides. La necrosis de este segmento del colon se
produce por una alteración en la irrigación de la arteria:
a) Mesentérica superior
b) Tronco celíaco
c) Mesentérica inferior
d) Pélvica

21) Marque el órgano que se encuentra más distal en el tubo digestivo.


a) Íleon
b) Ciego
c) Duodeno
d) Estómago

22) Paciente de 24 años con dolor abdominal tipo cólico intenso en mesogastrio. Según sus
conocimientos de macroestructura, el origen del dolor puede ser el ___________:
a) Esófago
b) Estómago
c) Íleon
d) Colón

23) Paciente con herida por proyectil por arma de fuego, con herida de ingreso en región
paraumbilical. Entre las estructuras que usted está seguro que debe haberse lesionado es:
a) Ligamento de Treitz
b) Omento mayor
c) Mesosigmoides
d) Omento menor

24) Paciente joven es traído a emergencia con abdomen agudo quirúrgico debido a herida
contuso penetrante por verduguillo (alambre grueso con punta aguzada) recibida en una
pelea después de un partido de fútbol. Se observa herida en Hipocondrio Izquierdo. El
órgano que debe estar sangrando y produciendo hemoperitoneo es (marque la mejor
respuesta):
a) Colon ascendente
b) Hígado
c) Bazo
d) Colon sigmoides

25) Al evaluar una tomografía abdominal, el médico asistente le pide al interno de la UPC
que encuentre la imagen con el corte a nivel de L1. El interno sabiamente busca el
________ para ubicar la vértebra L1.
a) Nacimiento de la vena mesentérica superior
b) Nacimiento de la arteria mesentérica inferior
c) Cuello del páncreas
d) Cruce entre la Aorta y la Vena Porta

26) En la inspiración, la pared abdominal debe ____________ para ____________:


a) Relajarse aumentar presión intra abdominal
b) Contraerse aumentar presión intra torácica
c) Contraerse aumentar presión intra abdominal
d) Relajarse disminuir presión intra torácica

27) Cuál de las siguientes estructuras no tiene vasos sanguíneos:


a) Epitelio intestinal
b) Omento
c) Mesenterio
d) Ligamento

28) Paciente con hipoglucemia secundaria a un insulinoma (tumor neuroendocrino productor


de insulina). El órgano donde con mayor probabilidad ha crecido este tumor es:

a) Pélvico
b) Torácico
c) Retroperitoneal
d) Intraabdominal
30) Sustancia que inhibe la secreción y la motilidad del estómago prolongando el tiempo de
digestión:
- Péptido insulinotrópico dependiente de la glucosa (GIP)

31) Marque lo correcto:


- La hernia fisiológica se produce en la sexta semana y es la salida temporal de
asas intestinales a través del cordón umbilical

32) Marque la respuesta correcta en relación a la gastrina:


- Las células G son las productoras y se encuentran principalmente en el antro
gástrico

33) El consumir caramelos indirectamente activa la vía:


- POMC/CART

34) ¿En qué capa se encuentra la alteración principal en el Hirschsprung o megacolon


agangliónico?:
- Muscular propia

35) Con respecto a las ondas lentas, marque la afirmación correcta:


- Son contracciones rítmicas espontáneas

36) El uso de Ranitidina bloquea el receptor H2 de la histamina en las células parietales. La


histamina llega a estas células por:
- Difusión

37) La triada sintomática: vómitos explosivos post-prandiales, movimientos peristálticos


epigástricos visibles de izquierda a derecha y nódulo palpable epigástrico subcostal
derecho, pertenecen a:
- Estenosis congénita hipertrófica del píloro.

38) Durante una cirugía oncológica, ¿la extirpación de cuál de los siguientes órganos se
vería comprometida por la presencia de adventicia?:
- Recto

39)En cuanto a los reflejos gastrointestinales, un reflejo que estimula el tránsito intestinal es
el reflejo:
- Gastrocólico
- Cólico-ileal
- Entero-gástrico
- Vómito

40) El ligamento falciforme divide al hígado en dos lóbulos derecho e izquierdo.


Embriológicamente deriva del:
- Mesenterio ventral

41) La presencia de atresias y estenosis duodenales se deben básicamente a una:


- Falta de recanalización
42) Estudiante de medicina de 20 años, se ha amanecido estudiando para su examen de
Sistema Digestivo. No ha probado alimento desde la cena, por lo que se puede afirmar que
la motilidad de esta persona está siendo regulada por:
- Motilina

43) Paciente con disminución del apetito marcada asociada a cáncer terminal, para
promover la ingesta de alimentos se podría usar análogos de:
- Endorfinas

44) Las ondas lentas se producen por la apertura cíclica de canales de:
- Calcio

45) La forma más común de atresia esofágica contiene:


- Estenosis proximal del esófago con fístula traqueoesofágica distal

46) Al deglutir un bolo alimenticio, es lógico suponer que al pasar por el esófago haya un
mayor consumo de oxígeno en la pared del tercio:
- Proximal

47) Paciente que come entera una pizza familiar de chorizo y queso. Es posible esperar que
debido a la cantidad de alimento ingerida, las ondas lentas hayan:
- Sufrido ninguna alteración en su frecuencia

48) La hernia fisiológica se produce dentro de:


- Cordón umbilical

49) El crecimiento de un adenocarcinoma de páncreas compromete la pared gástrica por


contigüidad. ¿Qué parte del estómago se esperaría esté comprometido?
- Pared posterior del antro

50) Estimula la producción de saliva:


- Vasodilatación periglandular

51) Durante la secreción de saliva, es de esperarse que las concentraciones de ________ y


______ disminuyen al disminuir el flujo:
- Sodio Bicarbonato

52) Con respecto a la secreción gástrica de HCl:


- a mayor secreción de HCl en el lumen gástrico, mayor pH en la sangre venosa
gástrica

53) Respecto a las enfermedades del esófago, marque lo correcto:


- el diagnóstico diferencial de la acalasia es la enfermedad de Chagas esofágica

54) Con respecto a las lesiones y enfermedades de la boca, marque lo correcto:


- la eritroplasia debe ser biopsiada
55) Respecto a las glándulas salivales, marque lo incorrecto:
- la glándula sublingual tiene forma de garfio

56) El omeprazol actúa sobre la membrana _____________ de la célula ____________


- apical / parietal

57) Durante el sueño, la concentración de bicarbonato en la saliva:


- Disminuye

58) Durante el ataque con gas sarín (bloqueador de la acetilcolinesterasa) en el metro de


Tokio, en 1995, el personal de salud notó que los pacientes afectados presentaban:
- Hipersalivación

59)La célula mucosa del cuello gástrico produce:


- Moco

60) En la evaluación de una tomografía abdominal, el interno observa un aneurisma en una


arteria que se dirige al riñón derecho. Con seguridad se puede afirmar que está a nivel de la
vértebra:
- L1

61) Un un paciente con diarrea por hipermotilidad, usted sospecharía en el posible aumento
de las siguientes sustancias, excepto:
a) Péptido intestinal vasoactivo
b) Sustancia P
c) ACh
d) Motilina

62) Una de las siguientes sustancias no comparte con las otras la misma acción sobre la
producción de ácido gástrico:
a) Péptido insulinotrópico dependiente de glucosa (GIP)
b) Colecistoquinina
c) Somatostatina
d) Secretina

63) Usted encuentra músculo estriado en el siguiente segmento:


a) Esfínter anal externo
b) Duodeno
c) Tercio inferior esofágico
d) Esfinter anal interno

64) Paciente con intoxicación por organofosforados (inhibidores de acetilcolinesterasa), se


espera que el tránsito intestinal se encuentre:
a) Aumentado
b) Disminuido
c) En relación inversa al número de ondas lentas por minuto
d) No se espera ninguna alteración
65) Al disminuir el pH duodenal por el HCl gástrico, se libera principalmente una hormona
cuya célula diana es:
a) Células S del intestino
b) Acinos pancreáticos
c) Células ductales del colédoco
d) Célula ductal del wirsung

66) La hormona que tiene un efecto sinérgico con la secretina para optimizar el pH duodenal
y la digestión, es:
- CCK (colecistoquinina)

67) El ligamento falciforme del hígado proviene embriológicamente de:


- Mesenterio ventral
-

68) Todos los músculos motores de la lengua están inervados por el XII par, excepto:
- Palatogloso

69) Al ingerir grandes cantidades de dulces, con la subsecuente estimulación de incretinas,


usted esperaría que el apetito ______________, debido a __________________
- Disminuya insulina

70) Paciente obeso con Covid-19 es intubado por interno inexperto, quien al solicitar que
bombeen aire dentro del tubo endotraqueal, nota que el epigastrio se distiende. Al sospechar
que ha introducido el tubo en el estómago, también es cierto que:
- Disminuye el pH gástrico
- Aumenta el pH gástrico
- Disminuiría el tono del píloro
- Aumenta la frecuencia de ondas lentas

71) Una de las siguientes sustancias reguladoras, puede actuar de forma paracrina y como
hormona. Marque la correcta:
- Somatostatina
- Acetilcolina
- Péptido insulinotrópico dependiente de glucosa.
- GRP

72) Durante una cirugía oncológica, el cirujano observa que los órganos abdominales tienen
libre movimiento dentro de la cavidad abdominal, excepto:
- Yeyuno
- Vesícula biliar
- Colon ascendente
- Estómago

73) Al consumir un pan con mantequilla, la sensación de hambre disminuye debido a la


acción de:
- Grelina
- Somatostatina
- Colecistoquinina (CCK)
- Leptina

74) Respecto a los péptidos gastrointestinales, marque lo correcto.


- No existe sustancia neurocrina que tenga efecto en la motilidad del tubo digestivo
- Las sustancias paracrinas pueden viajar a través de vasos sanguíneos
- Las sustancias neurocrinas son péptidos que hacen su efecto en distancias cortas
- Las sustancias paracrinas atraviesan la circulación portal

75) En un paciente con gastroparesia (motilidad lenta del estómago), que presenta
distensión abdominal después de comer, usted le recomendaría que evite el consumo de
lípidos y aminoácidos para disminuir la acción de:

- Secretina
- CCK
- Somatostatina
- Gastrina

76) Paciente con apendicitis aguda, que debuta con dolor en mesogastrio. Este dolor se
debe a estimulación de receptores del dolor cuyas fibras van a viajar a la médula espinal a
través de:

● Nervio vago
● Plexo hipogástrico
● Nervios simpáticos
● Nervio esplácnico pélvico

78) Al introducir una solución azucarada directamente al estómago mediante una


gastrostomía (comunicación entre la piel abdominal y el estómago), la sustancia que
provocará que aumenten los niveles séricos de insulina es:

● Somatostatina
● Péptido tipo glucagón 1 (GLP-1)
● Glucagón
● Enteroglucagon

79) En un experimento, con una sonda nasogástrica se instila por goteo en el estómago una
sustancia líquida, y se obtiene como respuesta una dramática disminución del pH del
estómago. Dicha sustancia debe contener:

● Lípidos
● Carbohidratos
● Aminoácidos
● Secretina

80) En un paciente con hiperestimulación simpática se espera que las ondas lentas tengan
un ritmo:
● Mayor en estómago que en duodeno
● Mayor en estómago que en íleon terminal
● Menor en íleon terminal que en el duodeno
● Mayor en íleon que en duodeno

81) Se considera que el gusto puede viajar a través del nervio:

- V3
- Vago
- Lingual
- Glosofaríngeo

82) Al seccionar el nervio facial a nivel timpánico, usted esperaría:

- Disminución del gusto en la punta de la lengua


- Ausencia de termoalgesia en la lengua
- Imposibilidad para protruir la lengua
- Imposibilidad para el cierre del istmo de las fauces.

83) El ecografista sabe que para poder visualizar el nacimiento de la arteria mesentérica
superior, debe colocar el transductor sobre la piel de la siguiente región abdominal:

- Hipogastrio
- Hipocondrio derecho
- Mesogastrio
- Epigastrio

84) Marque lo correcto en relación al divertículo de Meckel.

- Se encuentra usualmente a 60 cm de la VIC


- Contiene mucosa esofágica en algunas ocasiones
- Se relaciona a un defecto en el desarrollo del intestino posterior
- Se produce en el lado mesentérico del íleon

85) Es un ligamento derivado del mesenterio dorsal:

- Gastrocólico
- Hepatogástrico
- Redondo
- Faciforme

86) Marque lo correcto respecto a la siguiente imagen:


- Produce síntomas en la gran mayoría de pacientes.
- Se produjo por giro en sentido contrario del duodeno
- Se produjo por falta de fusión de los ductos dorsal y ventral
- Como tratamiento, se podría aperturar la papila mayor (esfinterotomía).

TIPEO DE EXÁMENES
1) La arteria mesentérica superior emerge de la aorta a nivel de:
- Cabeza de páncreas
- Hilio hepático
- Tronco celiaco
- Íleon
- Cardias

2) La digestión de las proteínas se inicia en:


- Estómago
- Intestino delgado
- Boca
- Esófago
- Páncreas

3) Aproximadamente en la semana 6 del desarrollo embrionario, el intestino intermedio


gira 90º herniándose a nivel del:
- Fístula esofágica
- Borde superior del hígado
- Borde superior del pubis
- Cordón umbilical
- Borde inferior del bazo

4) Paciente de 76 años en estado de coma, con colostomía (colon abocado a la piel) por
resección parcial de colon secundaria a carcinoma de colon. Usted deja indicado que
este día se realice el cambio de la bolsa de colostomía, pero que lo hagan después
de dos horas de la nutrición enteral, debido a que quiere que se haga después de:
- La presencia de ondas lentas
- La secreción de gastrina
- El reflejo gastrocólico
- La producción de GIP
- La liberación de motilina

5) Recién nacido de 14 días, que presenta estreñimiento, distensión abdominal y


vómitos. Al examinar el orificio anal, se evidencia conducto permeable, pero al
introducir un poco el termómetro, se evidencia salida de material fecal por el ano.
Usted sospecha que el problema se deba a:
- Falta de desarrollo del tabique urogenital
- Defecto en el desarrollo del conducto vitelino
- Defecto en el desarrollo de las células de cajal
- Falta de desarrollo del seno uretral
- Falta de regresión de la membrana anal

6) La lengua está recubierta por epitelio:


- Plano estratificado no queratinizado
- Pseudoestratificado columnar no queratinizado
- Pseudoestratificado columnar ciliado
- Plano estratificado queratinizado

7) El esfínter anal interno tiene musculatura ___ y tiene control ___:


- Lisa/voluntario
- Lisa/involuntario
- Esquelética/simpático
- Esquelética/parasimpático
8) La arteria aorta proporciona la irrigación al tubo digestivo ¿cuál de las siguientes
arterias de irrigación al ángulo cólico derecho?
- Mesentérica superior
- Mesentérica inferior
- Frénica inferior
- Tronco celiaco
9) Paciente de 26 años que le cuenta en su historia clínica que cada vez que almuerza,
a los 20 minutos tiene deseo de defecar. Le comenta que su hijo de 1 mes le pasa lo
mismo pero más intenso. Esto se explica por el reflejo ____, el cual está _____ en el
paciente:
- Colicoileal/normal
- Colicoileal/alterado
- Gastrocolico/normal
- Gastrocolico/alterado

10) La región del estómago que se comunica con el duodeno se denomina


- Pilórica
- Cardias
- Cuerpo
- Fórnix

11) Acude a consulta un paciente que fue diagnosticado de úlcera péptica tres días antes.
Luego de múltiples pruebas diagnósticas, se concluye que el paciente presenta un
tumor secretor de gastrina ¿cual de las siguientes situaciones estará incrementada?
- Distensión gástrica
- Inhibición del vaciado gástrico
- Secreción de ácido clorhídrico (HCl)
- Inhibición de la secreción de pepsinógeno

12) En el sistema digestivo el control del apetito está dado por un complejo sistema de
sustancias y órganos integradores los cuales regulan la ingesta de alimentos. La ____
es una sustancia orexígena/orexígena y es sintetizada por el ____
- Leptina/intestino
- Grelina/intestino
- Leptina/estómago
- Grelina/estómago

13) Sobre el control Autónomo del sistema digestivo marque la alternativa correcta:
- La inervación dada por el sistema simpático es de tipo preganglionar
- El sistema parasimpático usa como neurotransmisores a la acetilcolina y
noradrenalina
- El nervio vago (par craneal X) le da inervación simpática a la mayoría del sistema
digestivo
- En el sistema simpático, los nervios responsables hacen una primera sinapsis en
ganglios próximos al órgano a inervar
- En la inervación de tipo parasimpático, solo interviene el plexo submucoso, sin
embargo, en la de tipo simpático intervienen tanto el submucoso como el mientérico.

14) Con respecto a la actividad eléctrica del sistema digestivo, marque la alternativa
correcta:
- Corresponden a potenciales de acción que están presentes de forma continua y le dan
capacidad de peristalsis autónoma al sistema digestivo
- La frecuencia de las ondas lentas no se ve influenciada por la actividad neural ni las
hormonas gastrointestinales
- En el estómago las ondas lentas se dan en una frecuencia de 6 por minuto
- Las ondas lentas son cambios lentos y ondulantes del potencial en reposo
- La frecuencia de las ondas lentas va de 6 a 12 ondas por minuto

15) Ante una lesión en el IX par craneal, el músculo ___ se altera en su función:
- Palatogloso
- Estilofaríngeo
- Palatofaríngeo
- Constrictor superior

16) Un varón de 50 años es sometido a extirpación de duodeno y parte proximal del


yeyuno. La pérdida de estímulo hormonal en el páncreas para la secreción enzimática
se explica por la pérdida de las células:
- Parietales, productoras del factor intrínseco
- “K” productoras del factor intrínseco
- “M” productoras de CCK
- “I” productoras de CCK

17) Respecto al mecanismo de defecación ¿cual de las siguientes afirmaciones es


correcta?
- Se produce una contracción refleja del esfínter anal interno
- Se produce contracción o relajación del esfínter anal externo por señales de la corteza
cerebral
- La presencia de materia fecal en el recto estimula la contracción del sigmoides por los
nervios pélvicos simpáticos
- En la posición de “cuclillas” el músculo puborrectal se haya contraído favoreciendo la
evacuación de la materia fecal
18) Un niño de tres años llega a emergencia con disfagia (dificultad para tragar), salivación
y llanto. Se sospecha de ingesta de cuerpo extraño (moneda en el esófago), al ser
evaluado se constata de una radiografía presencia de cuerpo extraño a nivel de C6 y
C7 (6º y 7º vértebra cervical). El cuerpo extraño estará suspendido a nivel del
estrechamiento producido por el:
- Cayado aórtico
- Hiato esofágico
- Músculo cricofaríngeo
- Bronquio principal izquierdo

19) En el caso de un paciente con un tumor productor de gastrina, la presencia de úlceras


duodenales y erosión de la mucosa gástrica se debe principalmente a:
- La acción paracrina de la gastrina sobre la célula parietal
- El exceso de HCl por estímulo de receptores CCK-B en la célula parietal
- La sobre expresión de los receptores “G” para gastrina en la célula parietal
- El exceso de HCl por estímulo directo de receptores “H” en la célula parietal

16) La onda peristáltica secundaria del esófago se caracteriza por ser originada:

- Por el plexo de submucoso del esófago


- Por el plexo mientérico del esófago
- Por el reflejo de la deglución
- Durante la masticación

17) Marque lo correcto sobre las ondas lentas en el tubo digestivo:

- No son despolarizaciones
- Son potenciales de acción subumbrales
- Se constituyen de despolarizaciones y repolarizaciones
- Son rítmicas y generadas por el sistema nervioso autónomo

18) Recién nacido presenta protrusión de contenidos abdominales los cuales no están
cubiertos por peritoneo y salen de la cavidad abdominal a través de un defecto en la pared
¿Cómo se denomina a la afección que presenta este paciente?

- Onfalocele
- Atresia biliar
- Gastrosquisis
- Divertículo de Meckel
19) Experimentalmente se utiliza atropina (anticolinérgico) para inhibir la secreción de
gastrina, sin embargo, la secreción de esta hormona se sigue dando ante estímulos vagales.
Esta situación se explica porque la atropina:

- No bloquea la acción del péptido GRP


- Solo inhibe la acción del péptido GHRP en la célula G
- Inhibe la acción de acetilcolina e histamina en la célula G
- Bloquea parcialmente la bomba de protones en la célula G

20) La niña de 4 días es llevada a la emergencia pediátrica por presentar llanto constante,
la madre refiere coloración azulada de labios al momento de lactar, acompañado de
tos persistente y dificultad respiratoria, así como distensión abdominal. Se le coloca
sonda nasogástrica para alimentación notando que retorna a la cavidad oral en todos
los intentos ¿Cuál es la anomalía del desarrollo en este caso?
- Solo fistula traqueo esofagica
- Fístula traqueo esofágica proximal y distal
- Atresia esofágica proximal con fístula traqueo esofágica distal
- Atresia esofágica distal con fístula traqueoesofágica proximal

21) Paciente varón de 36 años es traído a la emergencia luego de sufrir un accidente de


tránsito presenta traumatismos múltiples en cabeza y tronco. Al examen físico se
evidencia hematoma en hemicara izquierda, ligera protrusión y caída del lado
izquierdo del maxilar inferior por lo que se le realiza tomografía donde se haya una
fractura de apófisis coronoides del maxilar inferior. ¿Qué músculo está relacionado
directamente con esta situación?
- Masetero
- Temporal
- Buccinador
- Pterigoideo medial

22) Un paciente refiere no percibir algunos sabores. Al examen físico constata alteración
del sabor dulce y umami. ¿Cuál de los siguientes nervios estará alterada su función?
- Cuerda del tímpano (VII par)
- Lingual (rama del V par)
- Glosofaríngeo (IX par)
- Hipogloso (XII par)

23) A los pocos días de nacido, regresa a neonatología un niño con problemas de
motilidad del colon, los estudios determinan ausencia congénita de células
ganglionares. Según el gráfico cuál es la capa en la que se determina la ausencia de
dichas células?
- Mucosa - 2
- Muscular propia - 2
- Muscular propia - 3
- Muscular de la mucosa - 1

24) Con respecto al control autonómico en el tracto gastrointestinal y en relación con su


fisiología ¿Cuál es la función del sistema nervioso parasimpático y el tracto
gastrointestinal?
- Inhiben la contracción muscular y estimulan la secreción de sustancias a nivel de la
submucosa
- Estimulan la contracción muscular y estimulan la secreción de sustancias a nivel de
la mucosa
- Inhiben la contracción muscular e inhiben la secreción de sustancias a nivel de la
submucosa
- Estimulan la contracción muscular e inhiben la secreción de sustancias a nivel de la
mucosa

25) Intersticio ubicado entre el estroma del espacio portal y los hepatocitos, y por donde
migran las células cancerígenas que hacen diseminación linfática es el:
- Espacio del Mall
- Espacio de Disse
- Espacio porta
- Espacio sideral

26) La presencia de grandes cantidades de TGF - Beta estimula a las ___________ y se


deposita colágeno, formándose la cirrosis
- Células de Ito
- Células de Kupffer
- Triadas portales
- Células fibroblásticas

27) Paciente de 64 años con ICC al que se le va a realizar cirugía cardiovascular. Al


calcular el volumen sanguíneo total, se debe considerar que el hígado puede
contener un volumen de sangre de ____________ mL en un adulto sano, en este
paciente ese volumen puede llegar a ser de _____________ mL
- 450 - 1000
- 250 - 1000
- 450 - 3000
- 250 - 10000
28) El área del lobulillo que se afecta más en caso de hipoxia es la zona:
- 3
- 2
- 1
- 4

29) Paciente con carcinoma de vesícula biliar. La metástasis por continuidad afectará al
lóbulo:
- Cuadrado
- Lobulado
- Caudado
- Izquierdo

30) El ácido acetilsalicílico actúa en la membrana:


- Basolateral de la célula parietal
- Basolateral de la célula principal
- Apical de la célula principal
- Apical de la célula parietal

31) Al realizar una esplenectomía, se tiene que resecar la arteria esplénica, lo cual no es
problema para el estómago por que la arteria gastroomental izquierda se
anastomosa con la:
- Gastroomental derecha
- Gástrica derecha
- Gastroduodenal
- Pancreaticoduodenal superior

32) La arteria esplénica proviene de la aorta y la vena esplénica desemboca en la vena:


- Porta
- Gástrica izquierda
- Mesentérica superior
- Gastroduodenal

33) Paciente mujer 21 con bulimia, que luego de un episodio de vómitos presenta
hematemesis y al examen físico que se encuentra crépitos subcutáneos cervicales.
El diagnóstico más probable es:
- Sd. Boerhaave
- Sd. Mallory - Weiss
- Varices esofágicas
- Épulis

34) Paciente varón de 60 años, con antecedentes de promiscuidad sexual, tabaquismo y


alcoholismo, acude a consulta por presentar disfagia progresiva, odinofagia y al
examen se observa tumor por parte posterior de la lengua. La mejor posibilidad
diagnóstica es:
- Carcinoma escamoso
- Esófaggo de Barret
- Mucocele
- Granuloma piógeno

35) Niño de 5 años con historia de tres días de evolución caracterizado por fiebre,
malestar general, odinofagia, anorexia, e irritabilidad. Al examen de observa lesiones
ulcerativas de 4mm de diámetro en mucosa yugal, con borde blanquecino y eritema
periférico. El diagnóstico más probable es:
- Aftas orales
- Herpes simple
- Candidiasis oral
- Leucoplasia

36) Es considerada una lesión preneoplásica


- Leucoplasia
- Granuloma piógeno
- Carcinoma escamoso
- Boerhaave

37) El esófago de Barrett se considera una lesión preneoplásica que se caracteriza por
la presencia en esófago de:
- Metaplasia intestinal
- Metaplasia gástrica
- Displasia gástrica
- Neoplasia glandular

38) Paciente mujer de 23 años gestante con lesión proliferativa en mucosa oral
producida por proliferación reactiva de vasos sanguíneos. Marque la mejor respuesta
- Epulis
- Granuloma piógeno
- Leucoplaquia
- Carcinoma epidermoide

39) La glándula parótida tiene principalmente acinos de tipo:


- Seroso
- Mucoso
- Seromucoso
- Mixto

40) La reabsorción de sodio y secreción de potasio es estimulada por:


- Aldosterona
- Angiotensina II
- HAD
- Renina

41) Paciente con cirrosis hepática que tiene hipertensión portal con várices esofágica, y
actualmente presenta varices en estómago distal. Estas várices están relacionadas a
aumento en la presión de las venas:
- Gástrica derecha
- Gástrica izquierda
- Gástrica superior
- Gástrica inferior

42) La glándula parótida está inervada por el par craneal:


- IX
- X
- XII
- VII
- V

43) Paciente con cirrosis hepática que tiene hipertensión portal con varices en esofago
distal . Estas varices están relacionadas a aumento en la presión de las venas
gástricas
- Superior
- Izquierda
- Derecha
- Inferior

44) La información eferente que sale de los núcleos salivales superior e inferior a través
de los pares VII y IX hacia las glándulas salivales llevan información tipo
- Sensitivo
- Parasimpático
- Somático
- Simpático

45) Un niño de 4 años ingresa en el hospital con vómitos graves . En el estudio se


encuentra que el niño tiene un páncreas anular ¿ Cuál de las siguientes hormonas
gastrointestinales se encontrará a niveles elevados en sangre con mayor
probabilidad a raíz de esta patología
- GIP
- Gastrina
- Secretina
- VIP
46) Durante una colecistectomía laparoscópica en un hombre de 61 años ¿ Cuál de las
siguientes arterias debe pinzar para extirpar la vesícula biliar con seguridad?
- Hepática propia
- Hepática izquierda
- Hepática derecha
- Cistica

47) Un hombre de 34 años se somete a una apendicectomía de urgencia . Después de


realizar la apendicectomía satisfactoriamente , el paciente se somete a una
laparoscopia exploratoria¿Cual de las siguientes características anatómicas es más
útil para distinguir entre yeyuno e íleon?
- El yeyuno tiene menos grasa mesentérica que el íleon
- El yeyuno tiene más arcadas vasculares que el íleon
- El yeyuno tiene más folículos linfáticos bajo la mucosa que el íleon
- El yeyuno tiene menos vellosidades que el íleon
48) Una mujer de 45 años ingresa en el hospital con síntomas de obstrucción intestinal
superior .En la TC se encuentra que la tercera porción (transversa) del duodeno está
comprimida por un gran vaso ¿Cuál de los siguientes vasos causara muy
probablemente la obstrucción ?
- Arteria mesentérica inferior
- Vena mesentérica inferior
- Arteria mesentérica superior
- Vena porta

49) Durante una colecistectomía laparoscópica programada en una mujer de 47 años , el


residente pinchó accidentalmente el ligamento hepatoduodenal en vez de la arteria
cística ¿Cual de los siguientes vasos estaría muy probablemente ocluido en esta
lesión iatrogénica?
- Arteria hepática izquierda
- Arteria hepática propia
- Arteria esplénica
- Arteria mesentérica superior

50) Un hombre de 54 años ingresa en urgencias con intenso dolor abdominal superior .
La gastroscopia revela un tumor en el antro del estómago . Se pide una TC para
evaluar el drenaje linfático del estómago¿Cuál de los siguientes nódulos linfáticos
estará muy probablemente afectado en una neoplasia maligna del estómago
- Lumbar
- Mesentérico inferior
- Mesentérico superior
- Celiaco

51) Una mujer obesa de 45 años con fiebre alta acude a la consulta con náuseas y dolor
agudo e intermitente en el cuadrante superior derecho del abdomen de 2 días de
duración - Tiene una historia de ictericia de 24 horas . Tiene antecedentes de litiasis
biliar . Bilirrubina total del 10 mg/dL . Lipasa de 5 mg/mL ¿Cuál de las siguientes
estructuras está muy probablemente obstruida por un cálculo biliar ?
- Conducto colédoco
- Ampolla de Vater
- Conducto cístico
- Conducto pancreático

52) Una mujer de 45 años ingresa en urgencias con dolor abdominal intenso . La TC y
RM revelan un tumor de la cabeza del páncreas que afecta el proceso unciforme
¿Cual de los siguientes vasos es más probable que suministre irrigación a parte de
la zona afectada?
- Arterias yeyunales
- Arteria cólica media
- Arteria cólica izquierda
- Arteria ileocólica

53) Un hombre de 70 años ingresa en urgencias con diarrea intensa . La arteriografía


revela un bloqueo del 90% en el origen aórtico de la arteria mesentérica inferior
¿Cuál de las siguientes arterias proporciona muy probablemente irrigación colateral
al colon descendente?
- Arteria gastroomental izquierda
- Arteria cólica derecha
- Arteria sigmoidea
- Arteria cólica media

54) Al disminuir el pH duodenal por el HCL gástrico , se libera principalmente una


hormona cuya célula diana es
- Célula S del intestino
- Células ductales del colédoco
- Acinos pancreáticos
- Célula ductal de Wirsung

55) Una de las siguientes sustancias reguladoras , puede actuar de forma paracrina y
como hormona
- Péptido insulinotrópico dependiente de glucosa
- GRP
- Acetilcolina
- Somatostatina

56) Al ingerir grandes cantidades de dulces , con la subsecuente estimulación de


incretinas , usted esperaría que el apetito
- Aumente por grelina
- Aumente por CCK
- Disminuya por insulina
- Disminuya por CCK

57) Al seccionar el nervio facial a nivel timpánico , usted esperaría


- Disminución del gusto en la punta de la lengua
- Ausencia de termoalgesia en la lengua
- Imposibilidad para el cierre del istmo de las fauces
- Imposibilidad para protruir la lengua

58) Paciente obeso con Covid-19 es intubado por interno inexperto, quien al solicitar que
bombeen aire dentro del tubo endotraqueal, nota que el epigastrio se distiende. Al sospechar
que ha introducido el tubo en el estómago, también es cierto que:
- Aumenta pH gástricos
- Disminuye el tono del píloro
- Aumenta la frecuencia de las ondas lentas
- Disminuye el pH gástrico
59) La hormona que tiene un efecto sinérgico con la secretina para optimizar el pH
duodenal y la digestión, es:
- CCK
- Bombesina
- Gastrina
- Péptido inhibidor gástrico
60) El ecografista sabe que para poder visualizar el nacimiento de la arteria mesentérica
superior , debe colocar el transductor sobre la piel de la siguiente región abdominal
- Hipocondrio derecho
- Mesogastrio
- Hipogastrio
- Epigastrio

61) En un paciente con hiperestimulación simpática se espera que las ondas lentas
tengan un ritmo
- Mayor en estómago que en el duodeno
- Mayor en estómago que en el íleon terminal
- Menor en íleon terminal que en el duodeno
- Mayor en el íleon que en duodeno
62) Se considera que el gusto puede viajar a través del nervio
- Glosofaríngeo
- V3
- Vago
- Lingual
63) La rotación en sentido longitudinal del estómago en el desarrollo embriológico
condiciona que el nervio vago derecho quede a nivel
- Anterior
- Oblicuo
- Posterior
- Izquierdo
64) Con respecto al control autonómico en el tracto gastrointestinal y en relación con su
fisiología. ¿Cuál es la función del sistema nervioso parasimpático en el tracto
gastrointestinal?
A) Inhiben la contracción muscular y estimulan la secreción de sustancias a nivel de la
submucosa.
B) Estimulan la contracción muscular y estimulan la secreción de sustancias a nivel de la
mucosa.
C) Inhiben la contracción muscular e inhiben la secreción de sustancias a nivel de la
submucosa.
D) Estimulan la contracción muscular e inhiben la secreción de sustancias a nivel de la
mucosa.

65) Un estudiante que está preocupado por su examen parcial, no ha desayunado ni


almorzado; cuando al fin ingiere alimentos, esto le provoca el aumento de los
movimientos musculares del tracto gastrointestinal y la sensación de defecar.¿Qué
reflejo se ha activado?
A) entero - gástrico
B) gastro-cólico
C) cólico - ileal
D) ileo - ileal

66) ¿De que par craneal es rama el nervio palatino mayor?


A) Vago
B) Hipogloso
C) Trigémino
D) Palatogloso

67) ¿En cuál de las fases de deglución la epiglotis separa la vía respiratoria de la
digestiva?
A) oral
B) laringe
C) Faríngea
D) Esofágica

68) A los pocos días de nacido, regresa a neonatología un niño con problemas de
motilidad del colon; los estudios determinan ausencia congénita de células
ganglionares. Según el gráfico ¿Cuál es la capa en la que se determina la ausencia
de dichas células?
A) Mucosa - 2
B) Muscular propia - 2
C) Muscular propia - 3
D) Muscular de la mucosa - 3

69) Los péptidos intestinales se pueden clasificar como sustancias endocrinas,


paracrinas y paracrinas, dentro de las apocrinas se encuentran la somatostatina e
histamina. Marque la respuesta correcta
A) La somatostatina es sintetizada por las células B de la mucosa gástrica
B) La histamina actúa estimulando su receptor tipo H1 en la mucosa gástrica
C) La histamina es sintetizada por células de tipo paracrino de las glándulas gástricas
D) La somatostatina presenta dentro de sus funciones la estimulación de la secreción de H+

70) Paciente varón de 27 años es llevado por bomberos a emergencia luego de ser
asaltado y, tras resistirse, es cortado con el pico de una botella a nivel abdominal. Al
examen físico usted observa que a través de la herida se puede observar la
protrusión de las asas intestinales. En relación con las capas de la pared abdominal,
marque la alternativa correcta.
A) la fascia de scarpa está constituida principalmente por tejido adiposo
B) La pared abdominal está formada por piel, huesos, músculos, fascia y peritoneo parietal
C) La fascia de camper es una estructura fibrosa que carece de grasa y su grosor es
constante en toda la pared abdominal.
D) El músculo oblicuo externo discurre en dirección súpero-interna y se inserta en el borde
inferior de las últimas 3 a 4 costillas
E) El músculo recto del abdomen tiene como funciones comprimir el contenido del abdomen,
tensar la pared abdominal y flexionar la columna
71) Los músculos del tracto gastrointestinal de los segmentos propulsivo y receptor del
bolo alimenticio, responden de forma diferente al movimiento de este bolo a través
del intestino. ¿Cuál de las siguientes afirmaciones describe correctamente la
actividad del segmento propulsivo?
A) Tanto el músculo circular como el longitudinal están relajados
B) El músculo longitudinal está relajado y el circular está contraído
C) Tanto el músculo circular como el longitudinal están contraídos
D) El músculo longitudinal está contraído y el circular está relajado

72) El nervio vago inerva al músculo


A) Liso de la faringe
B) Liso de la boca
C) Estriado del esófago
D) Estriado del esófago

73) ¿Cuál de las siguientes es una característica de los ganglios mientéricos del sistema
nervioso entérico?
A) Contiene mayor número de neuronas que el plexo submucoso
B) Es también conocido como el plexo de Meissner
C) Contiene sólo neuronas motoras excitatorias del músculo liso
D) contiene neuronas sensitivas que activan a los músculos circular y longitudinal del tracto
intestinal

74) El divertículo faringoesofágico, hipofaríngeo de zenker, es una lesión muy particular


que se localiza en la cara po
75) posterolateral de la Unión de la faringe con el esófago, como una herniación de la
mucosa esofágica a través de las fibras oblicuas del músculo.
A) Salpingofaringeo
B) Constrictor inferior de la faringe
C) Constrictor superior de la faringe
D) Constrictor medio de la faringe

76) La razón por la que el potencial de acción viaja rápidamente en sentido longitudinal
por el músculo liso gastrointestinal es la presencia de uniones en hendidura,
A) Mayor cantidad de Ach
B) la presencia del plexo submucosos de meissner
C) Las fibras musculares no se disponen en haces musculares
D) varicosidades

76)Respecto a los péptidos gastrointestinales, marque lo correcto.


A) las sustancias paracrinas pueden viajar a través de vasos sanguíneos
B) no existe sustancia neurocrina que tenga efecto en la motilidad del tubo digestivo
C) las sustancias neurocrinas son péptidos que hacen su efecto en distancias cortas
D) las sustancias paracrinas atraviesan la circulación portal
77) Al ingerir rápidamente un litro de agua, usted esperaría que la gastrina
aumenta por efecto de:
A) ACh por estimulación del nervio vago
B) efecto paracrino de la histamina
C) ACh del sistema mientérico
D) aumento de acidez gástrica (disminución del ph)

78) Respecto a la anatomia del estomago,marque lo correcto:

A) El fondo gástrico forma la curvatura mayor

Pág 19 → 21
79) Los nervios esplácnicos lumbares (L1-L2) llevan información de tipo:
A) Simpática
B) Parasimpática
C) Dolorosa
D) Sensorial
E) Piramidal

80) Paciente de 32 años con herida por arma de fuego y shock hipovolémico. El intestino
delgado no se ha infartado aún a pesar de la hipoxia gracias a la liberación de:
A) Adenosina
B) Adrenalina
C) Noradrenalina
D) Péptido intestinal vasoactivo

81) Al realizarse un piercing en el ombligo,la sensación de dolor se transmite por:


A) T11
B) T8
C) T9
D) T10

82) El nivel en el que se encuentra el píloro y el páncreas se puede determinar usando el:
A) Cuadrante mayor
B) Píloro transpilórico
C) Plano subcostal
D) Ligamento inguinal
E) Anillo inguinal

83) Permite la suspensión e irrigación de los órganos peritoneales:


A) Mesenterio
B) Ligamentos
C) Arteria mesentérica superior
D) Peritoneo parietal

84) El dolor asociado a apendicitis clásicamente se ubica en:


A) Epigastrio
B) Mesogastrio
C) Hipogastrio
D) Fosa iliaca derecha
E) Hipocondrio derecho

85) El ligamento inguinales formado por la aponeurosis del:


A) Transverso
B) Recto abdominal
C) Oblicuo externo
D) Oblicuo interno
86) Los nueve cuadrantes del abdomen se delinean usando el plano subcostal,las líneas
medioclaviculares y:
A) Plano intertubercular
B) Plano transpilórico
C) Plano interapofisiario
D) Plano intercrestal
E) Plano catastral

87) Es un órgano peritoneal:


a) Páncreas
b) Hígado
c) Recto
d) Duodeno
e) Colon ascendente

88) La línea alba se encuentra:


a) Entre los rectos abdominales
b) Entre los oblicuos externos
c) Entre los oblicuos internos
d) Entre los transversos abdominales
e) Entre Huaylas y Matellini jajajaj

89) Es un órgano retroperitoneal:


a) Páncreas
b) Colon transverso
c) Colon sigmoides
d) Duodeno
e) Vesícula biliar

90) Enfermedad asociada con un error en el desarrollo de las células de Cajal


a) Enfermedad de Hirschsprung
b) Enfermedad celiaca
c) Divertículo de meckel
d) ano imperforado
e) fístula vitelina
f)
91) El divertículo de Meckel es un rezago de:
a) Conducto vitelino
b) Alantoides
c) Asas yeyunoileales
d) Cloaca
e) Conducto anorrectal

92) Aproximadamente en la semana 6 del desarrollo embrionario; el intestino medio gira 90°
orinandose a nivel del:
a) Cordón umbilical
b) Borde inferior del bazo
c) Caudal al borde hepático derecho
d) Lado derecho del abdomen
e) Suprapúbico
93) La fístula retroperitoneal es causada por una falta en el desarrollo de:
a) Tabique urorrectal
b) Membrana cloacal
c) Proctodeo
d) Membrana anal
e) Membrana urogenital

94) Paciente mujer de 54 años con náuseas,vómitos y abdomen agudo quirúrgico, se


ingresa a sala de operaciones donde se encuentra vólvulo de ciego. Esto se debe a:
a) Falta de fusión del mesenterio
b) Falta de herniación fisiológica
c) Falla de la rotación intestinal
d) Defecto en la formación de la cloaca

ANGELA-CI2 (P95-P104)
95) Al disminuir el pH duodenal por el HCl gástrico, se libera principalmente una hormona
cuya célula diana es:
Respuestas:
Célula ductal del Wirsung
Células ductales del colédoco
Células S del intestino
Acinos pancreáticos

96) Al seccionar el nervio facial a nivel timpánico, usted esperaría:

Respuestas:
Imposibilidad para el cierre del istmo de las fauces.
Ausencia de termoalgesia en la lengua
Imposibilidad para protruir la lengua
Disminución del gusto en la punta de la lengua

97) Al introducir una solución azucarada directamente al estómago mediante una


gastrostomía (comunicación entre la piel abdominal y el estómago), la sustancia que
provocará que aumenten los niveles séricos de insulina es:

Respuestas:
Péptido tipo glucagón 1 (GLP-1)
Enteroglucagon
Glucagón
Somatostatina

98) La rotación en sentido longitudinal del estómago en el desarrollo embriológico


condiciona que el nervio vago derecho quede a nivel:
Respuestas:
Oblicuo
Posterior
Izquierdo
Anterior

99) Paciente con apendicitis aguda, que debuta con dolor en mesogastrio. El dolor se debe
a estimulación de receptores del dolor cuyas fibras van a viajar a la médula espinal a través
de:
Respuestas:
a) Nervios simpáticos
b) nervio vago
c) nervio esplácnico pélvico
d) plexo hipogástrico

100) Una de las siguientes sustancias no comparte con las otras la misma acción sobre la
producción de ácido gástrico:
Respuestas:
Péptido insulinotrópico dependiente de glucosa (GIP)
Secretina
Somatostatina
Colecistoquinina

101) En un paciente con gastroparesia (motilidad lenta del estómago), que presenta
distensión abdominal después de comer, usted le recomendaría que evite el consumo de
lípidos y aminoácidos para disminuir la acción de:
Respuestas:
Gastrina
Somatostatina
CCK
Secretina

102) En un paciente con shock distributivo, usted decide iniciar noradrenalina por un catéter
CVC, consiguiendo aumentar la presión arterial. ¿qué efecto sobre la motilidad intestinal
esperaría encontrar?

Aumenta el peristaltismo.
El potencial de reposo de las fibras musculares se hace más negativo
Aumentan las ondas lentas, pero disminuyen los potenciales en espiga
Aumenta la mezcla en el antro gástrico
103) El ecografista sabe que para poder visualizar el nacimiento de la arteria mesentérica
superior, debe colocar el transductor sobre la piel de la siguiente región abdominal:

Respuestas:
Hipogastrio
Hipocondrio derecho
Epigastrio
Mesogastrio

104) De los diferentes reflejos gastrointestinales, hay uno que produce movimiento del
contenido hacia la región distal, y se llama reflejo:
Respuestas:
Entero-gástrico
Vómito
Gastrocólico
Cólico-ileal

105) La glándula submandibular recibe inervación traída por el nervio


● Cuerda del tímpano
● Petroso Mayor
● Lingual
● Mandibular
● Glosofaríngeo

106) A mayor flujo de saliva, disminuye la concentración de:


● Potasio
● Bicarbonato
● Sodio
● Cloro

107) En términos de mg/mL, el principal componente de la saliva es _____ seguido de ____:


● Proteínas - Potasio
● Potasio - Sodio
● Sodio - Cloro
● Cloro - Urea
● Urea - Proteínas

108) La reabsorción de Sodio y Cloro en las glándulas salivales se da principalmente en el:


● Conducto estriado
● Conducto intercalado
● Conducto excretor
● Acino seroso
● Acino mucinoso

109) La información eferente que sale de los núcleos salivales superior e inferior a través de
los pares VII y IX hacia las glándulas salivales llevan información de tipo:
● Parasimpático
● Simpático
● Somático
● Sensitivo

110) La glándula submaxilar le hace gancho al:


● Músculo milohioideo
● Músculo omohioideo
● Músculo geniohioideo
● Conducto de Wharton
● Conducto de Stenon

111) Los conductos salivales son ___ al agua, esa es una de las razones por las cuales la
saliva es siempre ___.
● Impermeables - hipotónica
● Permeables - hipertónica
● Permeables - isotónica
● Impermeables - isotónica

112) Los nervios esplácnicos pélvicos (S2-S4) llevan información de tipo:


● Parasimpática
● Simpática
● Dolorosa
● Sensorial
● Piramidal

113) Presenta movimientos en masa:


● Colón
● Intestino delgado
● Estómago
● Esófago
● Cavidad oral

114) La digestión de los lípidos se inicia en:


● Intestino delgado
● Colón
● Estómago
● Esófago
● Cavidad oral

115) La digestión de los carbohidratos se inicia en:


● Cavidad oral
● Esófago
● Estómago
● Intestino delgado
● Colón

116) La digestión de las proteínas se inicia en:


● Estómago
● Esófago
● Intestino delgado
● Colón
● Cavidad oral

117) El estómago recibe información simpática proveniente del:


● Ganglio celiaco
● Ganglio mesentérico superior
● Ganglio mesentérico inferior
● Nervio vago
● Nervios esplácnicos (T9-T12)

118) En el intestino delgado se absorbe los carbohidratos en forma de:


● Fructosa
● Sacarosa
● Maltosa
● Dextrosa
● Lactosa

119) Es rama de la arteria mesentérica superior:


● A. cólica media
● A. marginal
● A. cólica izquierda
● A. pancreaticoduodenal superior
● A. gastro-omental derecha

120) El “dolor de estómago” asociado a gastritis se suele ubicar en:


● Epigastrio
● Hipogastrio
● Mesogastrio
● Hipocondrio derecho
● Fosa iliaca derecha
121) El azúcar de mesa sacarosa es digerido a dos monosacáridos que comparten el
transportador:

● SGLT2
● GLUT5
● SGLT1
● GLUT 2

122) Durante la defecación se requiere:

● Liberación de acetilcolina por el nervio vago


● Contracción de tipo segmentarias
● Activación del nervio simpático esplácnico menor
● Señales inhibitorias en el nervio pudendo

123) En un paciente con fístulas intestinales y fisuras anales, con antecedente de


enfermedad inflamatoria intestinal, ud sospecharia en:
● Colitis Ulcerativa
● Anemia perniciosa
● Enfermedad de Crohn
● colitis microscópica

124) En los pacientes con Colecistitis Aguda no operable, una opción es la colocación de
una sonda por el cístico, procedimiento en el que se ingresa con dificultad debido a la
estrechez del cístico y a la presencia de:

● Ampolla de vater
● arteria cística
● válvula de Herring
● estenosis del hepático común

125) Paciente de 51 años con antecedente de enfermedad diverticular acude a emergencia


por sangrado profuso y dolor en hipocondrio izquierdo
¿Cuál es el origen más probable de la sangre que pierde el paciente ?

● Rectal superior
● rectal inferior
● Mesentérica inferior
● cólica media

126) Paciente de 42 años con dolor abdominal intenso y hematemesis. En la endoscopia se


observa una úlcera duodenal posterior perforada con hemorragia intraabdominal. ¿Cuál de
las siguientes arterias estará comprometida?

● Mesentérica superior
● Gástrica derecha
● Pancreaticoduodenal posterosuperior

127) Para que los triglicéridos sean absorbidos deben ser metabolizados a

● Colesterol y ácidos grasos


● Monoglicéridos y Ácidos grasos
● colesterol y lisolecitina
● lisolecitina y ácidos grasos

128) El vibrio cholerae produce diarrea porque:


● Previene la absorción de glucosa y causa que el agua mantenga en la luz intestinal
● aumenta los canales secretores de bicarbonato en los enterocitos
● Aumenta la producción de AMPc en los enterocitos
● inhibe los canales secretores de cloro en las células de la cripta

129) La motilidad intestinal es estimulada por


● colecistoquinina y gastrina
● Glucagón e insulina
● somatostatina y secretina
● secretina y glucagón

130) La diarrea por deficiencia de lactasa es de tipo


● Iatrogenia
● secretora
● exudativa
● osmótica

131)La metoclopramida estimula el vaciamiento gástrico aumentando la fuerza de


contracción de las paredes gástricas esto puede conseguirlo mediante la estimulación
indirecta de las neuronas liberadoras de
● Bombesina
● Acetilcolina
● Oxido nitrico
● Péptido intestinal Vasoactivo

132)Los vértices de un acino hepático están constituidos por


● una vena centrolobulillar y dos espacios porta
● Dos espacios porta y dos venas centrolobulillares
● tres venas centrolobulillares
● seis espacios porta

133)Al ingerir un pedazo de mantequilla, cuál de los siguientes tiene un efecto directo en la
reducción del vaciado gástrico:

● Colecistoquinina
● Gastrina
● Bombesina
● Secretina

134) Tras la vagotomía (resección del vago) por enfermedad úlcera péptica en un paciente
UD. esperaría encontrar:
● Aumento de síntomas de reflujo gastroesofágico
● hipertrofia de mucosa gástrica
● mayor producción de CCK
● Aumento del pH gástrico

135) Se realizó un experimento en el cual se inyectó tinta china en el peritoneo de ratas de


laboratorio. Al realizarse una biopsia hepatica de dichos animales, se encontro que el tinte
negro fue fagocitado por:
● células de disse
● Células de kupffer
● hepatocitos
● células de ito

136) En un paciente con pH gástrico muy bajo, es posible que la siguiente sustancia se
secrete en menor cantidad:
● Péptido inhibitorio gástrico
● colecistoquinina
● secretina
● Gastrina

137)La fase cefálica de la secreción gástrica responde por cerca del 30% de la respuesta
ácida a un reflejo con la _____________ se elimina la fase cefálica de la secreción gástrica

● Vaguectomía
● Histamina
● Cimetidina
● Gastrina

138)El esofago de barret se caracteriza por presentar _________ en el esofago

● Metaplasia gástrica
● metaplasia intestinal
● displasia gástrica
● adenocarcinoma

139) La presencia de orina que sale por el ombligo de un recién nacido cada vez que llora,
es posible que deba a un defecto en el desarrollo de:
● Seno urogenital
● Cloaca
● Membrana basal
● Cuerpo perineal
● Conducto vitelino

140) El conducto biliar deriva del:


● Endodermo
● Mesodermo
● Ectodermo
● Mesotelio
● Peritoneo

141) Cuál de los siguientes órganos son intraperitoneales:


● Estómago, Vesícula biliar, Y León, Hígado
● Páncreas, Colon descendente, Hígado, Vesícula biliar
● Recto, Hígado, Colon transverso, Yeyuno
● Estómago, Yeyuno, Duodeno, Páncreas
● Recto, Vesícula biliar, Y León

142) Los vasos mesentéricos superiores se hallan a nivel de:


● Cardias
● Cuello del páncreas
● Hilio hepatico
● Tronco celiaco
● Ileon

143) La colecistoquinina (CCK) inhibe:


● El vaciamiento gástrico
● La secreción pancreática de HCO3-
● La concentración de la vesícula biliar
● La relajación del esfínter de Oddi
● La secreción de amilasa

144) Un niño de 2 años es llevado a la consulta por diarrea persistente, edema de las
extremidades y falta de crecimiento en relación a su edad. Los análisis de sangre revelan
que tiene concentración plasmática baja de proteínas (hipoproteinemia). Como parte del
estudio se coloca Colecistoquinina (CCK) endovenosa y se recoge muestras del líquido
duodenal por endoscopia; el resultado del líquido confirma incapacidad para hidrolizar
proteínas a un pH neutro, esta situación mejora al añadir una pequeña cantidad de tripsina.
El paciente probablemente esté sufriendo la falta congénita de ……….
● PEPT-1
● Pepsinógeno
● Enterocinasa
● Carboxipeptidasa.

145) Paciente mujer de 35 años acude a consulta por sensación de sequedad y lesiones en
la cavidad oral. Al examen se observa atrofia de la mucosa, fisura y úlceras; nota además
sequedad e irritación de la córnea y aumento del tamaño de las glándulas parótidas. Su
diagnostico mas probable es artritis reumatoide; el hallazgo más probable en una biopsia de
glándula parótida es ….…
● Presencia de acinos anormales con hiperplasia de células ductales
● Gran infiltración de linfocitos y células plasmáticas
● Hiperplasia de acinos glandulares serosos
● Gran infiltrado de linfocitos y neutrófilos

146) Un hombre de 42 años de edad se presenta al médico con una historia clínica de 1 año
de evolución, caracterizado por dolor abdominal bajo y diarreas con crisis sanguinolentas.
Manifiesta además pérdida de peso de 8kg durante este periodo. La colonoscopia revela
lesión difusa en el colon con afectación del recto. La biopsia de estas lesiones revela
adelgazamiento de la pared, inflamación y ulceración de la mucosa y submucosa. El
diagnóstico más probable en este caso es:
● Síndrome de colon irritable
● Enfermedad de Crohn
● Colitis ulcerativa
● Sprue celiaco

Dos estudiantes deciden tomar un receso para comer una hamburguesa a la hora del
almuerzo. Antes de llegar a la cafetería, impulsos nerviosos provenientes del complejo vagal
dorsal iniciarán la secreción de ácido gástrico por la liberación de ___________ desde el
sistema nervioso entérico.
● Serotonina
● Colecistoquinina (CCK)
● Péptido inhibidor vasoactivo
● GRP (péptido liberador de gastrina)

147) Un niño de cuatro años de edad es llevado a la consulta por cuadros diarreicos
frecuentes caracterizados por heces pálidas, voluminosas y fétidas; al examen físico
presenta bajo peso y talla para la edad. Se mide la concentración de cloruro de en el sudor y
se encuentra que sus valores son muy elevados. La alteración más importante a nivel de
células ductales del páncreas tiene relación directa con la conductancia de …..
● Bicarbonato
● Potasio
● Sodio
● Cloro

148) Se evalúa los valores séricos de las siguientes sustancias a un paciente con
enfermedad hepática terminal; en este paciente se espera encontrar la combinación de la
letra …..

Glucosa Amoniaco Albúmina

a. Aumentada Disminuida Disminuida

b. Disminuida Aumentada Aumentada

c. Aumentada Aumentada Aumentada

d. Disminuida Aumentada Disminuida

149) Una mujer de 35 años de edad HIV positiva, se presenta al médico con dolor
abdominal en cuadrante superior derecho e ictericia. La paciente refiere haber tenido
múltiples episodios de ictericia durante los últimos 10 años. Los exámenes para determinar
hepatitis viral, dieron positivos para Hepatitis B, siendo catalogado el caso como hepatitis
crónica con alteración funcional. En un examen de sangre ¿Cuál de los siguientes
parámetros está disminuido?
● Albúmina
● Bilirrubina
● Fosfatasa alcalina
● Tiempo de protrombina
150) En el reflejo peristáltico del intestino delgado ¿Cuál de los siguientes eventos sucede
en la porción caudal del bolo alimenticio?
● Acción del péptido inhibidor vasoactivo (VIP) en el músculo circular
● Acción del NO (Óxido nítrico) en el músculo longitudinal
● Contraccion del musculo longitudinal interno
● Acción de la acetilcolina en el músculo circular

151) Un varón de 58 años de edad con enfermedad de Crohn severo fue sometido a una
resección ileal. Después de la cirugía esta paciente padecerá de esteatorrea esto se explica
porque ……
● Se inhibe la acción de la 7 alfa hidroxilasa
● El pool de ácidos biliares se incrementa
● Hay malabsorción de ácidos biliares
● El páncreas no secreta lipasa

152) En un experimento se inserta un balón en el estómago de un voluntario, se infla poco a


poco mientras que se vigilan las presiones intraluminales. Aunque el volumen del balón
aumenta considerablemente, las presiones permanecen constantes. Esta relación volumen-
presión se explica por la liberación local de ….
● Acetilcolina y gastrina
● Norepinefrina y oxido nitrico
● Colecistoquinina y oxido nitrico
● Óxido nítrico y péptido inhibidor vasoactivo

153) ¿Cuál de las siguientes alternativas es una característica de la secreción exocrina del
páncreas?
● Es hipotónica respecto al plasma
● Su mayor estímulo se da en la fase intestinal
● Es estimulada por la presencia de bicarbonato en el duodeno
● La secreción enzimática es estimulada principalmente por la secretina

154) Las estructuras en el hígado que permite que los productos metabólicos unidos a
proteínas tengan acceso a las membranas basolaterales de los hepatocitos, son ….
● Los canalículos
● Las células de Ito
● Las fenestras sinusoidales
● Las uniones intercelulares herméticas

155) La composición de la bilis es modificada conforme fluye por los conductillos biliares.
Durante este tránsito se espera que aumente la concentración de …..
● Ig A
● Glucosa
● Protones
● Vitamina A

156) Se mide experimentalmente el contenido gástrico de dos personas. La persona “A”


tiene alto contenido de grasa y la persona “B” tiene un contenido isotónico. ¿Cual de las
siguientes es correcta respecto al vaciamiento gástrico?
● Hay ralentización del vaciado gástrico sólo en “A”
● El vaciamiento gástrico es más rápido en ambos
● Hay ralentización del vaciado gástrico sólo en “B”
● Hay ralentización del vaciado gástrico en ambos casos

157) El examen endoscópico de un paciente con hipertensión portal grave revela venas
tortuosas que sobresalen hacia la luz del esofago. El paciente recibe tratamiento quirúrgico
mediante la colocación de una derivación que conecta la vena cava. Después de la
operación el riesgo de encefalopatía …….. y el sangrado de varices ……
● Disminuirá/Disminuirá
● Disminuirá/Aumentará
● Aumentará/Disminuirá
● Aumentará/Aumentará

158) Un bolo alimenticio grande y poco masticado se atasca en el esofago, esto ocasiona
una sensacion de dolor que es transmitida por los nervios:
● Vago
● Esplácnico
● de Latarjet
● Recurrente laríngeo

159) El peristaltismo del intestino delgado se puede intensificar debido a:


● Somatostatina
● Noradrenalina
● Secretina
● Irritación de la mucosa

160) Un paciente es diagnosticado con un tumor neuroendocrino productor de


somatostatina, esto provocará en el sistema digestivo:
● Vómitos
● Estreñimiento
● Odinofagia
● Diarrea

161) Los diferentes segmentos del tubo digestivo son susceptibles de reflejos y movimientos
según su contenido.Si colocoramos mediante una sonda un bolo alimenticio directamente en
el tercio medio del esofago:
● Se producirá la acción voluntaria del músculo estriado
● Será necesario un reflejo vago-vagal
● Se producirá ondas secundarias
● Se producirá ondas primarias

162) En el digestivo la liberación hormonal se presenta ante diversos factores o estímulos.


La hormona _________ es estimulada por la presencia de alimentos en el bulbo duodenal a
predominio de ácidos grasos y triglicéridos, por estimulación vagal y por la hormona
secretina.
● Motilina
● Colecistoquinina (CCK)
● Lipasa pancreatica
● Gastrina

163) El divertículo de Meckel es una anomalía congénita que ocurre por la persistencia del
conducto vitelino y da origen a una estructura sacular, el cual se encuentra en el:
● Duodeno
● Apéndice cecal
● Borde mesentérico
● Borde antimesentérico

164) La fase oclusal de la masticación se realiza con la contracción de los músculos


inervados por el nervio craneal:
● V
● VIII
● IX
● XII
165) En un paciente de 43 años con tumor carcinoide de páncreas productor de gastrina
(Sindrome de Zollinger-Ellison) se puede encontrar una potenciación del reflejo:
● Ileocolico
● Gastrocolico
● De relajación receptiva
● Del vómito

166) Durante la fase faríngea de la deglución ocurre el siguiente mecanismo:


● Los músculos infrahioideos hacen que se desplaza la laringe hacia abajo
● El paladar blando abre el istmo de las fauces
● La onda peristáltica lleva el alimento hacia el esofago
● El péptido inhibidor vasoactivo (VIP) actúa sobre el músculo constrictor superior

167) En los carcinomas (neoplasia benigna) es frecuente que ocurran la metástasis a través
de los vasos venosos. En el caso de un carcinoma del tercio superior del esofago, ubicado
en la cara lateral izquierda, es más probable que la metástasis viaje por la vena:
● Subcostal derecha
● Cava Inferior
● Hemiácigos accesoria
● Hemiácigos

168) Los movimientos en masa son iniciados por el reflejo:


● Del vómito
● Duodenocolico
● Colicoileal
● Duodenogástrico

169) Se evalúa los valores séricos de las siguientes sustancias de un paciente con
enfermedad hepática terminal, en este paciente se espera la combinación con la letra

Glucosa Amoníaco Albúmina


a aumentada disminuida disminuida

b disminuida aumentada aumentada

c aumentada aumentada aumentada

d disminuida aumentada disminuida

170) Una mujer de 65 años VIH positiva se presenta con dolor abdominal en el cuadrante
positivos para Hepatitis B, siendo catalogada como hepatitis crónica con alteración
funcional. En un examen de sangre ¿cuál de los siguientes parámetros está disminuido?
● Albúmina
● Bilirrubina
● Fosfatasa alcalina
● Tiempo de protrombina

171) En el reflejo peristáltico del intestino delgado¿cuál de los siguientes eventos sucede en
la porción caudal del bolo alimenticio?
● Acción del péptido inhibidor vasoactivo(VIP) en el músculo circular
● Acción del NO(óxido nítrico) en el músculo longitudinal
● Contracción del músculo longitudinal interno
● Acción de la acetilcolina en el músculo circular

172) Un varón de 58 años de edad con enfermedad de Crohn Severo fue sometido a una
resección ilegal. Después de la cirugía este paciente padecerá de esteatorrea, esto se
explica porque ….
● Se inhibe la acción de la 7 alfa hidroxilasa
● El Pool de ácidos biliares se incrementa
● Hay mala absorción de los ácidos biliares
● El páncreas no secreta lipasa

173) En un experimento se inserta un balón en el estómago de un voluntario, se infla poco a


poco mientras se vigilan las presiones intraluminales. Aunque el volumen del balón aumenta
considerablemente, las presiones permanecen constantes. Esta relación volumen presión se
explica por la liberación local de ….
● Acetilcolina y gastrina
● Norepinefrina y ácido nítrico
● Colecistoquinina y óxido nítrico
● Óxido nítrico y péptido inhibidor vasoactivo

174) ¿Cuál de las siguientes alternativas es una característica de la secreción exocrina del
páncreas?
● Es hipotónica respecto al plasma
● Su mayor estímulo se da en la fase intestinal
● Es estimulada por la presencia de bicarbonato en el duodeno
● La secreción enzima toca es estimulada principalmente por la secretina

175)Las estructuras en el hígado que permiten que los productos metabólicos unidos a
proteínas tengan acceso a membranas basolaterales de los hepatocitos son ….
● Los canalículos
● Las células de Ito
● Las de fenestras sinusoidales
● Las uniones intercelulares herméticas

176)La composición de la bilis es modificada conforme fluye por los conductillos biliares.
Durante este tránsito se espera que aumente la concentración de ….
● IgA
● Glucosa
● Protones
● Vitamina A

177)Se mide experimentalmente el contenido gástrico de dos personas. La persona A tiene


alto contenido de grasas y la persona B tiene un contenido isotónico¿ Cuál de las siguientes
es correcta respecto al vaciamiento gástrico?
● Hay ralentización del vaciado gástrico sólo en A
● El vaciamiento gástrico es más rápido en ambos
● Hay ralentización del vaciado gástrico sólo en B
● Hay ralentización del vaciado gástrico en ambos

178)El examen endoscópico de un paciente con hipertensión portal grave revela venas
tortuosas que sobresalen hacia la luz del estómago. El paciente recibe tratamiento
quirúrgico mediante la colocación de una derivación que conecta la vena porta a la vena
cava. Después de la operación el riesgo de encefalopatía …. y el riesgo de sangrado de
varices ….
● Disminuirá/ disminuirá
● Disminuirá / aumentará
● Aumentará / disminuirá
● Aumentará / aumentará

179) Un paciente varón de 18 años de edad acude al médico para sus exámenes de rutina.
Sus resultados de laboratorio muestran un valor de bilirrubina sérica de 4 mg/dl y una
bilirrubina directa de 0.3 mg/dl. Las pruebas de función hepática son normales. La alteración
que explica mejor este caso es por la deficiencia de….
● Transaminasas
● Hemo Oxigenasa
● La 7 alfa hidroxilasa
● Glucuronil transferasa

180) Un hombre de 57 años de edad es llevado a urgencias con hematemesis masiva roja
brillante, a su llegada se halla inconsciente con PA: 80/40 mm Hg y FC:124 lat/min. Luce
ictérico con presencia de arañas vasculares en el tórax anterior y extremidades, abdomen
distendido con signo de oleada positiva. Se encuentra esplenomegalia y pérdida de la masa
muscular en extremidades. La anastomosis vascular responsable del sangrado en este
paciente es ….
● Vena gastrica izquierda y vena acigos
● Arteria gástrica izquierda y arteria ácigos
● Vena paraumbilical y vena epigástrica inferior
● Vena gástrica izquierda y vena esofágica superior

190) Un estudiante de medicina está comiendo un plato de comida a base de champiñones,


espárragos y salsa de soya. El estímulo del sabor umami contenido en todos estos
alimentos viaja a través del nervio….
● Lingual
● Hipogloso
● Glosofaríngeo
● Cuerda del tímpano

191) Una paciente de 30 años de edad es sometida a una cirugía de oído medio derecho
por un problema de otoesclerosis. Luego de la cirugía refiere alteración sensitiva de la
lengua. Al evaluar el caso usted esperaría encontrar ….
● Alteración en la sensación del dolor y temperatura en el tercio posterior de la lengua
● Alteración de la sensación del dolor en los dos tercios anteriores de la lengua
● Alteración en la sensación del gusto en el tercio posterior de la lengua
● Sensación del dolor, tacto y temperatura conservadas

192) En una paciente de 45 años de edad con colestasis biliar, se encuentra una elevación
de los niveles sanguíneos de fosfatasa alcalina hasta 3 veces la cifra normal ¿Cuál de las
siguientes alternativas estará también elevada como evidencia del daño biliar?
● Tiempo de protrombina y albúmina sérica
● Transaminasas hepáticas(ALT y AST)
● Gamma glutamyl transpeptidase
● Glucoronil transferasa

193) Experimentalmente se incrementa la velocidad de la secreción salival con una


sustancia, en el análisis de la composición de esta saliva obtenida se espera encontrar….
● Disminución de la concentración de bicarbonato que supera la concentración
plasmática
● Aumento en la concentración de cloro y sodio que supera la concentración
plasmática
● Aumento de la concentración de bicarbonato que supera la concentración plasmática
● Disminución de concentración de potasio y bicarbonato

194) Lactante de 3 meses de vida es atendido por presentar diarrea, se administra una
solución glucosa y electrolitos por vía oral. La proteína de membrana apical que explica la
capacidad de esta solución para proporcionar aporte de glucosa e hidratación es ….
● CFTR
● SGLT 1
● GLUT 2
● GLUT 5

195) Paciente ha sufrido herida de bala en el abdomen, se le tenido que extirpar el


segmento medio y distal del íleon. En este caso la síntesis hepática de sales biliares estará
….
● Sin cambios en el ritmo de la síntesis
● Disminuida por inhibición de la enzima colesterol 7 alfa hidroxilasa
● Incrementada por estímulo de la enzima colesterol 7 alfa hidroxilasa
● Incrementada por inhibición de la enzima colesterol 7 alfa hidroxilasa

196) Un varón de 75 años ingresa al consultorio por presentar ictericia marcada de piel y
escleras. El estudio del paciente mostró que presentaba un tumor que obstruía la totalidad
del conducto hepático común. ¿Cual de los siguientes conductos se encontraría dilatado en
este paciente?
● De Wirsung
● De Hering
● Colédoco
● Cístico

197) Correlacione las dos columnas y marque la respuesta correcta:


● Enfermedad Hirschsprung Aganglionosis congénita
● Diarrea osmótica. Intolerancia a la lactosa
● Diarrea secretora. Canales de Cl- en las células de la cripta
● Diarrea exudativa Heces con moco y sangre
198) La fase cefálica de la secreción gástrica responde por cerca del 39% de la respuesta
ácida a un reflejo. Con la ____ se elimina la fase cefálica de la secreción gástrica
Vaguectomía
199) El esofago de Barret se caracteriza por presentar___ en el esofago
Metaplasia intestinal

198) En la regulación del apetito y la saciedad, la estimulación experimental crónica del


núcleo ventromedial del hipotálamo producirá:
● Afagia
● Obesidad
● Hiperfagia
● Activación de neuronas relacionadas a NPY

199) Paciente mujer de 25 años acude por dolor en fosa iliaca derecha que empeora al toser
o caminar,asociada a náuseas y vómitos por lo cual acude a emergencia.Dos días después
de realizarle una apendicectomía, la paciente desarrolla fiebre alta (39° C), esta hipotensa y
presenta dolor abdominal.La laparotomia exploratoria muestra un gran volumen de sangre
en la cavidad peritoneal por lesión de un vaso producida durante la apendicectomía.¿Cual
de las siguientes arterias debe ligarse para detener la hemorragia?
● Ileocólica
● cólica derecha y arteria rectal superior
● mesentérica superior
● Ileocólica y arteria cólica Media

200) ¿Cuál de los siguientes es una causa de ictericia con bilirrubina conjugada
aumentada?
● Ictericia del recién nacido
● Obstrucción del colédoco
● Anemia hemolítica
● Gran hematoma

201. Dos días después de una apendicectomía en un hombre de 45 años, ha desarrollado


fiebre alta (39), está hipotenso y presenta dolor abdominal. La laparotomía exploratoria
muestra un gran volumen de sangre en la cavidad peritoneal por lesión de un vaso
producida durante la apendicectomía. ¿Cuál de los siguientes vasos debe ligarse para
detener la hemorragia?

a. Arteria cólica derecha

b. Arteria ileocólica y arteria cólica media

c. Arteria mesentérica superior

d. Arteria ileocólica

202. Paciente de 78 años, con diabetes mellitus tipo ll y fumador, que acude a consulta
porque desde hace dos semanas tiene un dolor intenso en flanco derecho y mesogastrio ,
intenso que aparece a los 30 minutos de haber comido, y desaparece dos a tres horas
después. En estos pacientes, es muy probable que la circulación deficitaria sea
parcialmente asumida por la:

a. Arteria ileocólica

b. Arteria cólica izquierda

c. Arteria cólica media

d. Arteria yeyunales

203. Un hombre de 70 años ingresa en urgencias con diarrea intensa. La arteriografía revela
un bloqueo del 90% en el origen aórtico de la arteria mesentérica inferior. ¿Cuál de las
siguientes arterias proporciona muy probablemente irrigación colateral al colon
descendente?
a. Arteria gastro omental izquierda

b. Arteria cólica derecha

c. Arteria cólica media

204. Un niño de 4 años ingresa en el hospital con vómitos graves. La exploración radiológica
y la historias clínicas revelan que el niño tiene páncreas anular. ¿Cuál de las siguientes
estructuras es la que se encontrará dilatada con mayor probabilidad a raíz de esta
patología?

a. Tercera porción del duodeno

b. Segunda porción del duodeno

c. Píloro gástrico

d. Primera porción del duodeno

205. Un hombre de 55 años ingresó al hospital con dolor abdominal intenso. La gastroscopia
y la TC revelaron una úlcera perforada en la pared posterior del estómago. ¿Dónde se
desarrolla inicialmente con más probabilidad una peritonitis?

a. Espacio hepatorrenal (de Morison)

b. Bolsa omental (saco menor)

c. Espacio subhepático derecho

d. Espacio subfrénico derecho

206. Un hombre de 44 años ingresa en urgencias con vómitos abundantes y deshidratación.


Las imágenes radiológicas demuestran que parte del intestino está comprimido entre la
aorta abdominal y la arteria mesentérica superior. ¿Cuál de las siguientes estructuras
intestinales estará muy probablemente comprimida?

a. Segunda porción del duodeno

b. Colon transverso

c. Tercera porción del duodeno

d. Primera porción del duodeno

207. ¿Cuál de los siguientes nódulos linfáticos estará muy probablemente afectado en una
neoplasia maligna del recto?

a. Celiaco
b. Mesentérica inferior

c. mesentérica superior

d. rectal

208. Una mujer de 23 años ingresa con dolor abdominal, náuseas y vómitos. La historia
clínica muestra que el dolor es agudo y ha sido constante durante 4 años. El dolor empezó
en el epigastrio e irradiado bilateralmente alrededor del tórax hasta justo debajo de las
cápsulas. Actualmente el dolor se localiza en el hipocondrio derecho. L a TC revela cálculos
calcificados en la vesícula biliar. ¿Cuál de los siguientes nervios llevan las fibras aferentes
del dolor referido?

a. Nervios esplácnicos torácicos mayores

b. Nervio frénico

c. Nervio vago

209. Un hombre de 55 años ingresa en urgencias por severa pérdida de peso en los 6
meses previos. El examen radiológico pruebas aportan signos de un tumor que causa
hipertensión portal. Los estudios de laboratorio revelan que las deposiciones son grasas,
tiene desnutrición e hipoxia hepática. ¿Cuál de las siguientes localizaciones se encuentra
muy probablemente afectada?

a. Segmento l

b. Segmento ll

c. Segmento ll

d. Segmento IV

210. El triángulo de calot es importante reconocerlo porque sirve como reparo para
encontrar la arteria cística, y está formado por el conducto cístico, el conducto hepático
derecho y :

a. Borde hepático

b. Arteria vesical

c. Arteria hepática

211. El kernicterus se produce en recién nacidos con valores mayores de 25 m/dL en la


bilirrubina:

a. Indirecta

b. Directa

c. Delta
212. Paciente con cirrosis hepática y presión de vena cava de 15 mmhg. Lo más probable
es que el paciente presente:

a. Ascitis

b. Cabeza de medusa

c. Arañas vasculares

213. Considerando un flujo plasmático renal de 180 ml/min, y una fracción de filtración del
20%, si la concentración en sangre de la bilirrubina indirecta es de 0.6mg/dL, entonces es
correcto esperar que la carga filtrada de la bilirrubina indirecta es:

a. Menor de 1080 mg/dL

b. 2050

c. Mayor a 1080}

214. El volumen diario de bilis secretada al intestino es:

a. 500-1000 ml

b. 50-100

c. 5-10

215. La excreción del amonio se da principalmente por:

a. Orina

b. Heces

c. Pulmones

d. Piel

216. Si en una persona normal, el flujo de la arteria hepática es de 700 ml/min, entonces el
flujo de la vena porta debería ser aproximadamente:

a. 2800 mL/min

b. 3500

c. 1300

217. Paciente con Crigler-Najjar debida a mutación del gen UGT1A1, se presenta a consulta
por ictericia, usted asume que si le hiciera un análisis de sangre encontraría valores
elevados de:

a. Bilirrubina indirecta
b. Directa

c. Conjugada

218. El amoniaco corporal se forma principalmente en:

a. Colón

b. Riñón

c. Músculo

219. La zona del lobulillo hepático que se afectaría más en una intoxicación con droga
hepatotóxica es la zona:

a. 1

b. 2

c. 3

d. 4

220. El acino hepático tiene en sus aristas:

a. Vena centrolobulillar

b. Solo espacios porta.

c. Espacios porta y sinusoides

221. Paciente con esteatosis hepática no alcohólica, que en la biopsia se observa


degeneración grasa de los hepatocitos, lo cual se debe a depósitos de lípidos que
principalmente contienen:

a. Triglicéridos

b. Colesterol

c. Ácidos grasos libres superior derecho e ictericia. La paciente afirma haber tenido
múltiples episodios de ictericia durante los últimos 10 años. Los exámenes para poder
detectar hepatitis viral, dieron p

PARCIAL
222. Al evaluar la orofaringe de un paciente, el médico le solicita que abra la boca, saque la
lengua y diga a . Al hacer esta maniobra, nota que el paladar se desvía hacia la derecha, lo
cual le hace sospechar que el paciente sufre de una lesión del nervio craneal:
- X contralateral

223. Un bolo alimenticio grande y poco masticado se atasca en el esófago, esto ocasiona
una sensación de dolor que es transmitida por los nervios:
- esplácnicos

224. Para realizar el movimiento mecánico de abrir la boca, primero se necesita:


- fijar el hueso hioides

225. ¿Cuál de las siguientes alternativas se define como la protrusión directa del contenido
abdominal a la cavidad amniótica por un defecto de la pared corporal?
- Gastrosquisis

226. Un paciente requiere que se le coloque una sonda de alimentación directamente al


estómago (gastrostomía), el cirujano deberá hacer una incisión en la piel del abdomen ¿cuál
de las siguientes raíces nerviosas debe ser anestesiada para este procedimiento?
- T8

227. En un paciente de 43 años con tumor carcinoide de páncreas productor de gastrina


(Síndrome de Zollinger-Ellison) se puede esperar encontrar una potenciación del reflejo:
- gastrocólico

228. El mecanismo de la defecación incluye la participación de diversas estructuras ¿Cuál


de las siguientes alternativas es correcta?
- Puede ser mediado por un reflejo intrínseco

229. Cuando el contenido del estómago ingresa al duodeno, uno de los reflejos que inhiben
el vaciamiento gástrico es a través del:
- sistema nervioso mientérico

230. Durante la masticación, gran parte del proceso masticatorio se debe a:


- el reflejo masticatorio

231. Las glándulas salivales tienen conductos para la excreción de la saliva; las glándulas
____________ drenan en las carúnculas sublinguales.
- sublinguales

232. Los diferentes segmentos del tubo digestivo son susceptibles de reflejos y movimientos
según su contenido. Si colocamos mediante una sonda un bolo alimenticio directamente en
el tercio medio del esófago:
- se producirá ondas secundarias

233. En una persona si enfermedad se espera que el tránsito intestinal se vea disminuido
cuando se presenta el reflejo:
- doloroso
234. El divertículo de Meckel es una anomalía congénita que ocurre por la persistencia del
conducto vitelino y da origen a una estructura sacular, el cual se encuentra en el:
- borde antimesentérico

235. Si al intubar a un paciente, por error se ingresa el tubo endotraqueal en el esófago y se


insufla el manguito endotraqueal (globo TET), la dilatación de este manguito generará:
- múltiples ondas secundarias

236. El orificio omental, o hiato de Winslow, se encuentra limitado por el ligamento:


- hepatoduodenal

237. Paciente de 24 años acude a consulta externa por presentar una fístula oronasal
(comunicación entre la cavidad oral y la cavidad nasal). Está fístula es una consecuencia
tardía de la lesión de un vaso sanguíneo por el antecedente de haber sido operado de
paladar hendido en los primeros años de vida, aparentemente en una campaña gratuita de
corrección de paladar fisurado. ¿Cuál de las arterias palatinas podría haberse lesionado
durante esa cirugía?
- Mayor

238. Dentro de las anomalías congénitas se puede presentar un tejido pancreático accesorio
¿Cuál es la ubicación más común de este tejido?
- Estómago

239. Paciente con insuficiencia mitral moderada a severa, con aumento de volumen de la
aurícula izquierda; esta condición tendrá como consecuencia a nivel del sistema digestivo:
- la disfagia a sólidos

240. El inicio de la fase faríngea de la deglución se debe a estímulos sensitivos que viajan
por el nervio craneal:
- V (nervio maxilar)

241. El mesocolon transverso se origina en:


- la pared posterior del abdomen

242. Un paciente varón de 68 años consulta por dolor y abultamiento en la región inguinal
derecha, que aparece tras la realización de un esfuerzo físico. En la exploración en
bipedestación presencia de una tumoración blanda, depresible, que aumenta con la tos. El
presente caso describe una hernia:

- Inguinal directa.

243. la Grelina es sintetizada por………….. y activa las neuronas relacionadas con


……………………….en el núcleo arqueado del hipotálamo}
- El estómago/AGRP-NPY

244. Cuál de los siguientes órganos son intraperitoneales:


- Estómago, Vesícula biliar, Y León, Hígado
245. La distensión gástrica por los alimentos produce incremento de secreción de HCl
mediante el siguiente mecanismo:
- Producción de Gastrina que desencadena su cascada de señalización en la célula
parietal vía proteína Gq

246. La inervación de la piel del abdomen debajo del ombligo hasta la región púbica está
dada por:
- T11, T12, L1

247. Paciente de 38 años que tras riña durante partido de fútbol sufre un traumatismo con
arma blanca en cuadrante inferior izquierdo del abdomen. En la tomografía de urgencias se
evidencia gran hematoma de pared y ausencia de neumoperitoneo. Desde la piel hacia al
peritoneo, en ¿qué orden se atravesó la pared abdominal?

- TCSC, oblicuo externo, oblicuo interno, músculo transverso

248. Alrededor de 90% de los pacientes afectados por el síndrome de Zollinger-Ellison


desarrollan úlcera péptica. La causa es:
- Hiper Secreción ectópica de gastrina

249. Un hombre de 22 años sufre traumatismo en el flanco izquierdo superior del abdomen
al tratar de defenderse del robo de su vehículo. Un amigo lo trasladó a un centro
hospitalario. El paciente presenta signos de hipovolemia con taquicardia e hipotensión. Se
queja de dolor en el sitio de lesión que se irradia al hombro izquierdo. El órgano
probablemente lesionado es
- Bazo

250. Aproximadamente en la semana 6 del desarrollo embrionario, el intestino medio gira


90° orinandose a nivel del:
- Cordón umbilical

251. ¿Cuál de las siguientes sustancias es liberada por neuronas en el tracto GI, Participa
en la regulación hidroelectrolítica y produce relajación del músculo liso?
a) Gastrina
b) Secretina
c) CCK
d) VIP

252. La colecistoquinina inhibe:


a) El Vaciamiento gástrico
b) La Secreción pancreática de HCO3-
c) La Contracción de la vesicular biliar
d) La Relajación del esfínter de oddi

253. ¿La secreción de cuál de las siguientes es inhibida por un pH bajo?


a) Secretina
b) Gastrina
c) CCK
d) VIP

254. El conducto biliar deriva del:


a) Ectodermo.
b) Endodermo.
c) Mesodermo.
d) Mesotelio.

255. Respecto a la imagen mostrada la estructura señalada con el número 1


corresponde a:
a) Glándula parótida
b) Glándula sublingual
c) Glándula submaxilar
d) Glándula lingual

256. Los vasos mesentéricos superiores se hallan a nivel de:


a) El fondo gástrico
b) El cuello del páncreas
c) La cabeza del páncreas
d) El hilio hepático

257. La imagen mostrada representa un defecto de ͙ ͙ ͙ ͙ ͙ …


͙ ……… y recibe el nombre
de: …………
a) La pared abdominal/onfalocele
b) La pared abdominal/divertículo de Meckel
c) Malrotación intestinal/gastrosquisis
d) La pared abdominal/gastrosquisis
PARCIAL

1) Paciente de sexo masculino de 82 años de edad ingresa a emergencia con dolor abdominal
agudo y diarreas. Se le realiza una arteriografía en la que se observa que la arteria aorta tiene
un trombo ocluyendo el 95% del flujo a nivel del nacimiento de la arteria mesentérica inferior.
¿Cuál de las siguientes arterias podría contribuir a la irrigación colateral del colon
descendente?

a) Cólica media
b) Sigmoidea
c) Rectal superior
d) Ileocólica
2) Niño de 5 años presenta dolor esofágico y hematemesis (vómitos hemorrágicos) luego de
tragarse una espina de pescado. En la endoscopía se observa perforación del esófago distal
a la cuarta estrechez esofágica. ¿Las ramas de cuál de las siguientes arterias estarán
lesionada con mayor probabilidad?
a) Gástrica izquierda
b) Bronquiales
c) Frénica inferior
d) Tiroidea inferior

3) Al ingerir una cucharada de mantequilla es muy probable que se disminuya la sensación


de hambre por medio de la activación de la vía POMC/CART (POMC=proopiomelanocortina
y CART=transcripción regulada de cocaína y anfetamina), activada directamente por la
hormona:
a) colecistoquinina (CCK)
b) insulina
c) grelina
d) secretina

4) Al comer unas papitas fritas con mayonesa, el vaciamiento gástrico disminuye por efecto
directo de la hormona:
a) colecistoquinina (CCK)
b) bombesina
c) motilina
d) gastrina

5) Recién nacido de dos horas es diagnosticado de hernia umbilical de 1,5 cm de diámetro;


el cirujano pediatra solicita una tomografía abdominal en donde se evidencia que la hernia
umbilical está ocupada por una porción del tracto gastrointestinal. ¿Qué porción del tracto
gastrointestinal estaría ocupando esta hernia con mayor probabilidad?
a) Íleon
b) Colon sigmoides
c) Duodeno
d) Colon transverso

6) Recién nacido de 7 horas, de parto por cesárea debido a polihidramnios (aumento del
volumen del líquido amniótico), con regurgitación de la leche materna y artificial, y no ha
presentado meconio. Se le realiza una tomografía donde se evidencia aire en el estómago y
una malformación del desarrollo esofágico. Con respecto a esta malformación lo más
probable es que se pueda tratar de una atresia esofágica:
a) proximal con fístula traqueoesofágica distal
b) distal con fístula traqueoesofágica proximal
c) proximal y distal
d) sin fístula

7) Lactante de 6 meses de edad que es traído a consulta por presentar vómitos no biliosos a
repetición y retraso en el crecimiento. En la radiografía de abdomen simple se observa nivel
hidroaéreo en estómago y en primera porción de duodeno (doble burbuja). ¿Cuál de las
siguientes alternativas puede explicar la condición del lactante?
a) Páncreas anular
b) Atresia duodenal en la tercera porción
c) Atresia yeyunal
d) Hipertrofia del píloro

8) En ausencia o deficiencia de la secreción de la hormona motilina, se producirá:


a) sobrecrecimiento bacteriano
b) diarrea
c) aumento del vaciamiento gástrico
d) hipertrofia del píloro

9) La estimulación parasimpática aumenta la motilidad intestinal, mientras que la


estimulación simpática la disminuye. ¿Sobre cuál de las siguientes alternativas el sistema
nervioso autónomo actúa para el control de la motilidad intestinal?
a) Potencial de membrana en el plexo mientérico (de Auerbach)
b) Frecuencia de ondas lentas
c) Secreción de secretina
d) Nivel de IP3 en el plexo submucoso (de Meissner)

10) recién nacido con protrusión de contenidos abdominales y cubiertas por amnios o
peritoneo, es cierto que:

a) Se presenta por un defecto en el cierre de la pared


b) Se acompaña de otras malformaciones congénitas
c) Se debe al no retorno de la hernia fisiológica
d) Se produce a través del ombligo

11)Lactante de 20 días con estreñimiento, distensión abdominal progresiva, acompañada


ocasionalmente de vómitos biliosos. Como antecedente, el meconio lo eliminó por primera
vez a las 72 horas de nacido. Su mamá menciona que ayuda a la evaluación con ayuda de
un termómetro rectal. Se sospecha megacolon agangliónico (Enfermedad de Hirschsprung).
¿Cuál de las siguientes alternativas explica el caso?

a) Se presenta contracciones tónicas en la región ano rectal


b) Se presenta dilatación de tracto gastrointestinal afectado
c) Las células ganglionares sólo han migrado al ano recto
d) La zona que más se afecta es inervada por fibras del nervio esplácnico menor

12) ¿Cuál de los siguientes reflejos disminuye el tránsito intestinal?


a) Doloroso
b) Gastrocólico
c) De defecación
d) Colicoileal
13) Al ingerir una sustancia ácida como el vino (ph 3), se estimula la motilidad gástrica por
acción de la hormona)
a) Motilina
b) Secretina
c) Colecistoquinina (CCK)
d) Bombesina

14) Paciente de 24 años acude a consulta externa por presentar fístula oronasal
(comunicación entre la cavidad oral y cavidad nasal). Esta fístula está asociada al
antecedente de haber sido operada del paladar hendido a los dos años de edad, durante
una campaña gratuita extranjera de corrección de paladar fisurado, ¿Cuál de las arterias
palatinas podría haberse lesionado durante esta cirugía?
a) Mayor
b) Menor
c) Ascendente
d) Rama palatina de faringea ascendente

15) Paciente de sexo masculino de 52 años con úlcera péptica gástrica de 14 años de
evolución, con cuadro de hemorragia digestiva alta hace 4 meses, sin cicatrización de la
úlcera. Entre las opciones quirúrgicas se considera vaguectomía troncal (sección del nervio
vago) a nivel del hiato esofágico. ¿Cuál de las siguientes complicaciones podría esperarse
producto de la pérdida de inervación parasimpática?
a) Menor inervación del colon ascendente
b) Se perderá el reflejo de defecación
c) Se perderá el reflejo de micción
d) Impotencia sexual

16) Paciente varón de 23 años con bulimia es traída deshidratada, semiconsciente y con
alcalosis metabólica. Los vómitos autoinfligidos por esta paciente se producen por
estimulación de receptores en la base de la lengua que mandan información directamente
al:
a) Núcleo del tracto solitario
b) Centro del vómito en el tallo encefálico
c) Zona quimiorreceptora gatillo
d) Cerebelo

17) Niño de 3 años es traído a emergencia por padre quien manifiesta que hace 10 horas
deglutió una pila pequeña de reloj de bordes romos. El niño está asintomático. Usted la
tranquiliza diciéndole es un cuerpo extraño tan pequeño que va a seguir el tránsito intestinal
como lo haría un bolo alimenticio, y que lo más probable es que en ese momento se
encuentre en:
1) colon
2) estómago
3) yeyuno
4) recto
18) Los movimientos en masa son un tipo de movimiento muy importante, una de las
consecuencias de estos movimientos es:
a) distensión rectal
b) peristaltismo del intestino delgado
c) retropulsión gástrica
d) contracción del esfínter anal interno
19) Durante la deglución, al momento que el bolo alimenticio pasa por el esfínter esofágico
superior, se espera que la presión intraesofágica:
a) disminuya en el cardias
b) disminuya en el tercio medio del esófago
c) aumente en la porción distal al bolo
d) aumente en el tercio medio del esófago
20) Paciente con enfermedad de chagas que presenta disfagia a sólidos. ¿Cuál de las
siguientes puede ser la causa de esta complicación?
a) disminución de células ganglionares en el esfínter esofágico inferior
b) aumento de la liberación de óxido nítrico en el esfínter esofágico inferior
c) disminución de las neuronas que liberan péptido intestinal vasoactivo
d) aumento de la actividad de la motilina en el esófago distal
21) El consumo de leche produce indirectamente
a) Curación de la gastritis
b) Estimulación del reflejo gastrocólico
c) Estimulación del vaciamiento gástrico
d) Inhibición del vaciamiento gástrico

22) Varón de 30 años es traído a emergencia por agresión abdominal con arma de fuego
(pistola) y es sometido a laparotomía exploratoria, observándose isquemia del colon
ascendente y parte del colon transverso ¿la lesión de cúal de las siguientes arterias
explicaría esta isquemia?
MESENTÉRICA SUPERIOR
23) Lactante masculino de 5 meses de edad producto de un primer embarazo normal
controlado . Peso de nacimiento 3.120 g y talla de 51 cm . Lactancia materna satisfactoria,
con buen incremento ponderal . Sin antecedentes patológicos hasta 5 días atrás , cuando
comienza con vómitos posprandiales no biliosos , lácteos . Los síntomas aumentan en
frecuencia y magnitud hasta hacerse explosivos después de cada alimentación . No requiere
tos , fiebre , diarrea ni lesiones cutáneas . Conserva el apetito y llora de hambre .Se espera
encontrar mayor liberación de :
a) GIP
b) Enzimas pancreáticas
c) Insulina
d) Secretina
24) Respecto a las sustancias GI que regulan la secreción pancreática; marque la
afirmación correcta:
a) La secretina, es la hormona más importante para la secreción de bicarbonato
por las células acinares del páncreas
b) La acetilcolina es capaz de estimular la secreción enzimática y de
bicarbonato del páncreas
c) La Gastrina es la hormona más importante para la secreción de enzimas
pancreáticas
d) La CCK estimula el páncreas solo para la secreción enzimática
25 ) Lactante masculino de 5 meses de edad producto de un primer embarazo normal
controlado . Peso de nacimiento 3.120 g y talla de 51 cm . Lactancia materna satisfactoria,
con buen incremento ponderal . Sin antecedentes patológicos hasta 5 días atrás , cuando
comienza con vómitos posprandiales no biliosos , lácteos . Los síntomas aumentan en
frecuencia y magnitud hasta hacerse explosivos después de cada alimentación . No requiere
tos , fiebre , diarrea ni lesiones cutáneas Una persona adulta podría tener un cuadro de
hipertrofia del píloro secundaria a :
a) Gastritis hipoclorhídrica
26) Ante una lesión del X par craneal ¿Cuál de los siguientes músculos mantiene
conservada su función?
A) Elevador del velo del paladar
B) Tensor del velo del paladar
C) Palatofaríngeo
D) Glosofaríngeo
27) Lactante masculino de 5 meses de edad producto de un primer embarazo normal
controlado . Peso de nacimiento 3.120 g y talla de 51 cm . Lactancia materna satisfactoria,
con buen incremento ponderal . Sin antecedentes patológicos hasta 5 días atrás , cuando
comienza con vómitos posprandiales no biliosos , lácteos . Los síntomas aumentan en
frecuencia y magnitud hasta hacerse explosivos después de cada alimentación . No requiere
tos , fiebre , diarrea ni lesiones cutáneas . Si el paciente tuviera 35 años de edad usted
podría pensar en un diagnóstico diferencial de la hipertrofia del píloro , la presencia de un
tumor neuroendocrino productor de la hormona
a) Secretina
b) Gastrina
c) Colecistoquinina
d) Bombesina
28) Lactante masculino de 5 meses de edad producto de un primer embarazo normal
controlado . Peso de nacimiento 3.120 g y talla de 51 cm . Lactancia materna satisfactoria,
con buen incremento ponderal . Sin antecedentes patológicos hasta 5 días atrás , cuando
comienza con vómitos posprandiales no biliosos , lácteos . Los síntomas aumentan en
frecuencia y magnitud hasta hacerse explosivos después de cada alimentación . No requiere
tos , fiebre , diarrea ni lesiones cutáneas . En relación a la estructura afectada se encuentra
a) Ligamento redondo
b) Mesenterio propiamente dicho
c) Ligamento gastroesplénico
d) Omento mayor
29) Experimentalmente se utiliza atropina (anticolinérgico) para inhibir la secreción de
gastrina, sin embargo, la secreción de esta hormona se sigue dando ante estímulos vagales.
Esta situación se explica porque la atropina:
a) Bloquea parcialmente la bomba de protones en la célula G
b) Inhibe la acción de acetilcolina e histamina en la célula G
c) Sólo inhibe la acción del péptido GRP en la célula G
d) No bloquea la acción del péptido GRP
30) Lactante masculino de 5 meses de edad producto de un primer embarazo normal
controlado . Peso de nacimiento 3.120 g y talla de 51 cm . Lactancia materna satisfactoria,
con buen incremento ponderal . Sin antecedentes patológicos hasta 5 días atrás , cuando
comienza con vómitos posprandiales no biliosos , lácteos . Los síntomas aumentan en
frecuencia y magnitud hasta hacerse explosivos después de cada alimentación . No requiere
tos , fiebre , diarrea ni lesiones cutáneas . El uso de atropina en este paciente
a) Aumenta el pH del estómago
b) Disminuye la acción del receptor CCK-B
c) Aumenta la producción de ácido clorhídrico
d) Inhibe la acción de las prostaglandinas

31) El consumo de dos vasos de agua seguidos generará indirectamente aumento en la


liberación de
a) Ácido clorhídrico
b) Enzimas pancreáticas
c) Péptido Y
d) Noradrenaline
32) Un varón de 50 años es sometido a extirpación del duodeno y parte proximal del
yeyuno. Esta situación ocasiona la pérdida de las células _____S_____, productoras de
_____ Secretina ____ que estimula la secreción de bicarbonato por el páncreas.

33) Estudiante de medicina de la UPC de 21 años sufre de gastritis aguda ocasionada por
comer en lugares poco higiénicos. Suele consumir caramelos ( chupa ) mientras está en
clase hasta la tarde. Toma gaseosa regularmente (carbohidratos 46%, sodio 53%). También
toma regular cantidad de leche (grasa 35%, lactosa 35%, proteínas 30%), pues le calma un
poco el dolor el ardor que siente por la gastritis. Incluso, cuando puede, se toma dos vasos
de agua fría para calmar las molestias. Ha decidido ir al médico para tratarse pues ya no
soporta el dolor, el cual está seguro que los síntomas se deben a una elevada producción
de ácido clorhídrico en el estómago, y por ello le ha recetado Loratadina (antihistamínico),
con lo que siente mejoría.
El uso de atropina en este paciente:
a) aumentará el pH del estómago
b) aumentará la producción de ácido clorhídrico
c) inhibirá la acción de las prostaglandinas
d) disminuirá la acción del receptor CCK-B

34) El uso de Ranitidina bloquea el receptor H2 de la histamina en las células parietales . La


histamina llega a estas células por
a) Difusión
b) Vía hematógena
c) Se produce en la misma célula parietal
d) La luz gástrica

35) Aumenta la secreción salival


a) Noradrenalina
b) Colecistoquinina
c) Secretina
d) Gastrina
36) Paciente con gastritis aguda debido a una alta producción de ácido clorhídrico si se le
hiciera un examen de sangre , se encontraría elevados los niveles de
a) Colecistoquinina

37) Recién nacido que presenta tumoración abdominal a nivel del cordón umbilical ¿cuál de
las siguientes afirmaciones es correcta respecto a este defecto en el desarrollo embriológico
del intestino?
a) Corresponde a una gastrosquisis
b) Las vísceras se hallan cubiertas por piel
c) No está asociado a otras malformaciones
d) Se asocia a malformaciones cardiacas y del tubo neural

38) El consumir caramelos eleva los niveles en sangre de una hormona cuya función es la
estimulación de las células
a) Beta del páncreas
b) Alfa del páncreas
c) G del antro
d) K del duodeno
39) La triada portal ( arteria hepática , vena portal y conducto biliar común ) está contenida
en el
a) Ligamento Gastroesplenico
b) Ligamento gastrohepatico
c) Ligamento hepatoduodenal
d) Ligamento falciforme

40) Varón de 35 años acude a la emergencia por trauma


abdominal y se decide realizar una laparoscopía
exploratoria. El cirujano observa la disposición de los
órganos abdominales como se representa en el siguiente
esquema. Esta disposición de órganos se explica por la
rotación: HORARIA DEL INTESTINO MEDIO
41) Se evalúa la expresión de la proteína Agrp en una
persona con alteración del apetito, lo correcto respecto a
esta proteína es:
a) Esta proteína es un potente anorexigénico
b) La mutación del gen que la codifica produce
adelgazamiento
c) La sobreproducción de la proteína lleva a obesidad
por agonista de receptores MC3 y MC4
d) La sobreproducción de la proteína disminuye el apetito por antagonismo de
receptores MC4.
42) Juana cae de la bicicleta y se fractura la región anterior del hueso maxilar superior con
compromiso de la fosa incisiva. Al examen físico de la región esperaría encontrar alteración
en la sensibilidad de la encía
PALATINA ANTERIOR
43) Recién nacido es atendido por el neonatólogo y luego entregado a su madre para dar de
lactar; la madre al dar de lactar observa coloración azulada de labios, acompañado de tos
persistente, dificultad respiratoria y distensión abdominal. Se le intenta colocar una sonda
nasogástrica pero esta retorna a la cavidad oral en todos los intentos ¿cual de los siguientes
anomalías el desarrollo es el más probable en este caso?
a) Estenosis esofágica proximal con fístula traqueo esofágica distal
b) Atresia esofágica proximal con fístula traqueo esofágica distal
c) Atresia esofágica distal con fístula traqueoesofágica proximal
d) Fístula traqueoesofágica proximal y distal
44) Estudiante de medicina de la UPC de 21 años sufre de gastritis aguda ocasionada por
comer en lugares poco higiénicos. Suele consumir caramelos ( chupa ) mientras está en
clase hasta la tarde. Toma gaseosa regularmente (carbohidratos 46%, sodio 53%). También
toma regular cantidad de leche (grasa 35%, lactosa 35%, proteínas 30%), pues le calma un
poco el dolor el ardor que siente por la gastritis. Incluso, cuando puede, se toma dos vasos
de agua fría para calmar las molestias. Ha decidido ir al médico para tratarse pues ya no
soporta el dolor, el cual está seguro que los síntomas se deben a una elevada producción
de ácido clorhídrico en el estómago, y por ello le ha recetado Loratadina (antihistamínico),
con lo que siente mejoría.
El consumo rápido de 500 mL de gaseosa aumentará directamente la concentración sérica
de cuál de las siguientes hormonas:
a) Colecistoquinina (CCK)
b) Neuropéptido Y
c) Secretina
d) Gastrina
45) Estudiante de medicina de la UPC de 21 años sufre de gastritis aguda ocasionada por
comer en lugares poco higiénicos. Suele consumir caramelos ( chupa ) mientras está en
clase hasta la tarde. Toma gaseosa regularmente (carbohidratos 46%, sodio 53%). También
toma regular cantidad de leche (grasa 35%, lactosa 35%, proteínas 30%), pues le calma un
poco el dolor el ardor que siente por la gastritis. Incluso, cuando puede, se toma dos vasos
de agua fría para calmar las molestias. Ha decidido ir al médico para tratarse pues ya no
soporta el dolor, el cual está seguro que los síntomas se deben a una elevada producción
de ácido clorhídrico en el estómago, y por ello le ha recetado Loratadina (antihistamínico),
con lo que siente mejoría.
Entre las sustancias cerebrales que producen ansiedad está la serotonina, la cual también
tiene acción:
a) Anorexigénica
b) Orexigénica
c) No influye en la producción de apetito
d) La acción de la serotonina en el apetito no está descrita
46) Estudiante de medicina de la UPC de 21 años sufre de gastritis aguda ocasionada por
comer en lugares poco higiénicos. Suele consumir caramelos ( chupa ) mientras está en
clase hasta la tarde. Toma gaseosa regularmente (carbohidratos 46%, sodio 53%). También
toma regular cantidad de leche (grasa 35%, lactosa 35%, proteínas 30%), pues le calma un
poco el dolor el ardor que siente por la gastritis. Incluso, cuando puede, se toma dos vasos
de agua fría para calmar las molestias. Ha decidido ir al médico para tratarse pues ya no
soporta el dolor, el cual está seguro que los síntomas se deben a una elevada producción
de ácido clorhídrico en el estómago, y por ello le ha recetado Loratadina (antihistamínico),
con lo que siente mejoría.
El consumo de leche produce directamente un aumento de los niveles séricos de la
hormona:
a) Gastrina
b) Grelina
c) Leptina
d) Péptido intestinal vasoactivo (VIP)
47) ¿En qué casos los vómitos son siempre biliosos?
a) Estenosis hipertrófica del píloro
b) Atresia duodenal
c) Atresia esofágica
d) Atresia yeyunal
48) Cuando la contracción peristáltica primaria está atraviesa el tercio superior del esófago,
el esfínter esofágico inferior estará:
a) Sin modificaciones
b) Denervado
c) Contraído
d) Relajado
49) Las arterias que derivan del tronco celíaco son, excepto:
a) Hepática derecha
b) Hepática común
c) Gástrica izquierda
d) Arteria esplénica
50) El uso de Ranitidina bloquea el receptor H2 de la histamina en las células parietales. La
histamina llega a estas células por:
a) Difusión
b) Vía hematógena
c) Se produce en la misma célula parietal
d) La luz gástrica
51) En cuanto a los reflejos gastrointestinales, un reflejo que estimula el tránsito intestinal es
el reflejo:
a) Cólico-ileal
b) Vómito
c) Gastrocólico
d) Entero-gástrico
52) Marque la respuesta correcta
a) La pared gástrica en el fondo es más delgada que en el cuerpo y antro
b) El esfínter de Oddi rodea a la papila menor duodenal
c) El bronquio derecho constituye una de las estrecheces del esofago
d) Todos los órganos del sistema digestivo tienen capa serosa
53) ¿En qué capa se encuentra la alteración principal en el Hirschsprung o megacolon
agangliónico?
a) Submucosa
b) Serosa o Adventicia
c) Muscular propia
d) Mucosa
54) Con respecto a las ondas lentas, marque la afirmación correcta:
a) Su frecuencia aumenta por acción de la acetilcolina
b) Son potenciales de acción que producen la contracción del tracto
gastrointestinal
c) Son contracciones rítmicas espontáneas
d) Su frecuencia disminuye por acción de la acetilcolina
55) En un paciente con apendicitis aguda la sensación de dolor producido por esta
inflamación es llevada por el nervio:
a) Vago
b) Pélvico
c) Esplacnico mayor
d) Esplacnico menor
56) La triada sintomática: vómitos explosivos post-prandiales, movimientos
peristálticos epigástricos visibles de izquierda a derecha y nódulo palpable
epigástrico subcostal derecho pertenece a:
a) Tumor benigno de las glándulas submucosas
b) Estenosis congénita hipertrófica del píloro
c) Colon irritable
d) Páncreas anular

EXAMEN FINAL 2019-01

1. La explicación fisiológica de presentar somnolencia de 30 minutos a 1 hora después de


ingerir alimentos, se explica por:
a. Aumento del cloro intraluminal
b. Aumento del bicarbonato intraluminal
c. Disminución de ácido carbónico en la célula parietal

d. Disminución de la actividad de la anhidrasa carbónicae. Aumento de la alcalinidad


sanguínea

2. Con respecto a la irrigación arterial del colon, a que arteria corresponde la señalada con
la flecha
a. Cólica derecha
b. Cólica media
c. Cólica izquierda
d. Ileobisecoapendículocólica
e. Arco de Riolano

3. Si un paciente presentara dentro del punto de vista fisiológico, una disminución de


enterocinasa, entonces esto originaría una disminución de la actividad de:
a. la pepsina
b. la lipasa (PUEDE SER POR SU RELACIÓN CON LA COLIPASA)
c. la quimotripsina
d. el péptido insulinotrópico dependiente de glucosa
e. la amilasa

4. Con respecto a la anatomía del hígado, señale a qué estructura pertenece la marcada por

el número 1.
a. Ligamento falciforme
b. Línea de Cantlie
c. Ligamento triangular
d. Ligamento coronario
e. Ligamento teres
5. Se presenta un paciente, el cual presenta un antecedente de tuberculosis intestinal, por lo
cual, se le resecó 80 cm de íleon distal. Desde el punto de vista fisiológico, el paciente
puede presentar una de las siguientes alteraciones:
a. Disminución de la secreción de Vitamina B12
b. Aumento indiscriminado de absorción de ácido fólico
c. Disminución de la absorción de hierro
d. Aumento de la secreción de bicarbonato
e. Disminución de la absorción de ácido glicocólico

6. Un paciente es sometido experimentalmente a un fármaco que modifica el flujo salival,


obteniéndose un volumen de saliva de 288 ml en 6 horas. En este caso las concentraciones
de electrolitos y bicarbonato en la saliva obtenida varían de la siguiente manera:
a. ↑ Na+, ↓ K+, ↑ Cl-, ↑ HCO3-
b. ↓ Na+, ↓ Cl-, ↑ K+, ↓ HCO3-
c. ↑ Na+, ↑ Cl-, ↓ K+, ↓ HCO3-
d. ↑ Na+, ↑ Cl-, ↑ K+, ↑ HCO3-
e. ↓ Na+, ↓ Cl-, ↓ K+, ↓ HCO3-

7. La siguiente imagen histológica corresponde a la glándula ............... y la estructura


señalada produce .........
a. salival sublingual / mucopolisacáridos
b. oxíntica / pepsinógeno
c. salival submaxilar / ptialina
d. salival parótida / amilasas
e. antrales / gastrina

8. Paciente varón de 65 años con antecedente de hipercolesterolemia, hipertensión arterial,


fibrilación auricular y dos infartos al miocardio previos, aqueja de dolor abdominal intenso de
inicio súbito, distensión abdominal, se decide cirugía con resección de 1,5 metros de
intestino delgado terminal y colon ascendente. Como consecuencia de la resección el
paciente tendrá deficiencia de:
a. Vitamina C
b. Tiamina
c. Vitamina A
d. Vitamina B1
e. Vitamina B6

9. Uno de los siguientes elementos debería hallarse con más probabilidad en el esófago de
un paciente que sufre de reflujo gastroesofágico...
a. Pepsina
b. Tripsina
c. Quimotripsina
d. Carboxipeptidasa
e. Ácidos biliares

10. Un paciente de 40 años cursa con anemia de 8g/dl, aqueja además de astenia y
sensación de hormigueo bilateral en los miembros inferiores, al examen se halla alteración
de la sensibilidad a la vibración y camina con ampliación de la base de sustentación. Uno de
los siguientes procedimientos sería de ayuda para el diagnóstico de este paciente:
a. Tomografía cerebral
b. Biopsia de la mucosa gástrica
c. Biopsia de hígado
d. Examen de sangre oculta en heces
e. Biopsia de Íleon proximal

11. Paciente de 60 años ingresa por caída hace 1 hora y pequeño hematoma en cuero
cabelludo, al examen físico ampliado se observa ictericia de piel y mucosas generalizada,
abdomen blando, se palpa estructura quística no dolorosa en hipocondrio derecho que
corresponde a vesícula biliar (signo de Courvoisier), en los exámenes de laboratorio se halla
niveles bajos en la formación de estercobilinógeno y urobilinógeno en heces, incremento de
la bilirrubina conjugada en la orina, elevación de fosfatasa alcalina y gamma glutamil
transpeptidasa séricas. El presente cuadro puede ser explicado por:
a. Reabsorción de hematoma
b. Litiasis vesicular
c. Carcinoma de la cabeza de páncreas
d. Carcinoma con estenosis del conducto hepático común
e. Anemia hemolítica

12. Paciente varón de 58 años con antecedente de alcoholismo crónico es diagnosticado y


recibe tratamiento por cirrosis hepática. Hace 2 días refiere familiar que tuvo cambio de
conducta y no reconoce a algunos familiares. Al examen físico, se halla ascitis, circulación
colateral en abdomen, telangiectasias, en el examen de sistema nervioso: rigidez de
extremidades, ROT incrementados, desorientación en el espacio y asterixis. ¿cuál de las
siguientes circunstancias, explicaría el cuadro en este paciente?
a. Uso de diuréticos ahorradores de potasio
b. Incremento de actividad de ureasa bacteriana duodenal
c. Hemorragia gastrointestinal
d. Disminución de la producción de NH3+ en el colon
e. Dieta normoproteica

13. Un recién nacido presenta vómitos biliosos poco tiempo después de cada alimento. Al
preguntar a la madre sobre antecedentes, ella recuerda que tuvo polihidramnios durante la
gestación, pero un análisis de cariotipo fue normal. Una de las siguientes es la causa más
probable de estos hallazgos en el recién nacido:
a. Enfermedad de Hirschsprung
b. Fístula tráqueo esofágica
c. Divertículo ileal
d. Estenosis pilórica
e. Malrotación de la yema pancreática ventral

14. Un lobulillo hepático se puede dividir en tres zonas como se muestra en el gráfico. ¿Cuál
de las siguientes afirmaciones sobre las tres zonas es verdadera?
a. La zona 1 tiene los menores depósitos de glucógeno
b. La zona 3 es la primera en afectarse en una colestasis extrahepática
c. La zona 2 es más susceptible a la injuria por isquemia que la zona periportal
d. La zona 2 tiene la mayor capacidad de regeneración
e. La zona 1 es la que tiene menos actividad metabólica.

15. En un estudio de la secreción de hormonas gastrointestinales, sus concentraciones en la


vena porta se midieron durante perfusión luminal del intestino delgado con soluciones de
diversas magnitudes de pH. ¿Qué hormona aumentará en el plasma de la vena porta
durante perfusión a través del intestino con una solución de pH 3?
a. CCK
b. gastrina
c. GIP
d. motilina
e. secretina

16. Paciente de 30 años que ingresa a causa de un traumatismo abdominal cerrado. En la


exploración se aprecia discreta palidez de piel y mucosas, auscultación pulmonar normal,
taquicardia de 120 /min. Discreta distensión abdominal y matidez en flancos; el hematocrito,
que era prácticamente normal al ingreso, disminuye un 30% a las tres horas. En la Rx de
tórax se objetiva fractura de las costillas 10-11 izquierdas. La causa más probable de la
anemización en este paciente es:
a. traumatismo renal con hemorragia retroperitoneal.
b. rotura de hígado con hemoperitoneo.
c. rotura de bazo con hemoperitoneo.
d. rotura de mesos con hemoperitoneo.
e. traumatismo pancreático con pancreatitis traumática.

17. Mujer de 65 años. Consulta por síndrome constitucional asociado a dolor abdominal
epigástrico progresivo irradiado a espalda, de dos meses de evolución. El diagnóstico de
sospecha de adenocarcinoma de páncreas se confirma por biopsia. Se realiza examen de
imagen de abdomen para evaluación de estructuras vasculares próximas al tumor
pancreático. ¿Cuál es el nombre de la vena señalada que está ausente, trombosada por
infiltración tumoral, condicionando circulación colateral en la pared gástrica?
a. Mesentérica superior
b. Coronaria estomáquica
c. Esplénica
d. Porta
e. Renal izquierda

18. Revisando la angiotomografía de un hombre de 70 años en estudio por aneurisma de


aorta abdominal, el radiólogo le informa de la presencia de una oclusión completa de la
arteria mesentérica inferior. El paciente se encuentra completamente asintomático. La
oclusión de la arteria mesentérica inferior cursa de manera asintomática en muchas
ocasiones ya que el territorio que irriga puede recibir flujo proveniente de la arteria:
a. cólica derecha
b. gastroduodenal
c. Epigástrica inferior izquierda
d. esplénica
e. cólica media
19. En las patologías de esófago es importante conocer bien la anatomía esofágica. ¿Cuál
de las siguientes afirmaciones es correcta?
a. El esófago tiene capa mucosa, muscular y serosa
b. El esófago abdominal es más largo que el cervical
c. El esófago torácico pasa por detrás del cayado aórtico
d. El epitelio esofágico normal es de tipo cilíndrico.
e. El esófago abdominal es discretamente más largo que el torácico

20. A pesar de que pueda haber variaciones anatómicas, lo habitual es que el ciego sea
irrigado por una rama arterial que proviene de unas de las siguientes arterias:
a. Iliaca derecha
b. Mesentérica inferior
c. Hepática derecha
d. Mesentérica superior
e. Iliaca izquierda

21. Ante un paciente con una cirugía abdominal urgente, el informe operatorio señala que se
ha realizado una resección de todo el duodeno y del tercio proximal del yeyuno manteniendo
íntegros el estómago y todo el íleon, así como los dos tercios distales del yeyuno. En el
seguimiento nutricional del paciente ¿Que vitamina o mineral presentará con menor
probabilidad una disminución de su absorción?
a. Cianocobalamina
b. Calcio
c. Hierro
d. Transcobalamina
e. Transferrina

22. ¿Cuál de las siguientes alternativas detallan las venas que confluyen y forman la vena
señalada?
a. mesentérica superior, gástrica izquierda y gastroepiploica izquierda
a. mesentérica inferior, gástrica izquierda y renal
b. esplénica, mesentérica superior y mesentérica inferior
c. esplénica, pancreatoduodenal y omental izquierda
d. gástrica izquierda, esplénica y hepática común

23. ¿Cuál de las siguientes sustancias forma parte de la secreción biliar?


a. Tripsina
b. Lecitina
c. Elastasa
d. Quimotripsina
e. Pepsina

24. El tubo digestivo contiene diferentes tipos de epitelios y glándulas. La estructura


señalada es una ................... y está localizada en el ...............
a. glándula de Brunner / intestino grueso
b. cripta de Lieberkühn / colon
c. cripta de Lieberkühn / intestino delgado
d. glándula oxíntica / estómago
e. célula parietal / estómago

25. ¿De qué músculo forma parte el ligamento inguinal?


a. Oblicuo externo del abdomen
b. Oblicuo interno del abdomen
c. Transverso del abdomen
d. Psoas
e. Dorsal ancho

26. Señale cuál de las siguientes afirmaciones NO se relaciona a la siguiente glándula


anexa del tubo digestivo mostrada en la imagen:
a. Es una glándula exocrina compuesta exclusivamente por acinos serosos
b. Su inervación está dada por el nervio auricular mayor (ramo posterior C2), que
inerva la vaina de la glándula así como la piel por encima de esta.
c. Esta glándula produce una secreción mucinosa acuosa, llamada mucoserosa, a
través del conducto de Wharton.
d. Su inflamación puede ser causada por un virus de los Paramyxoviridae, que
provocan una enfermedad muy frecuentemente en niños y adolescentes
e. Es una glándula endocrina y probablemente sea de origen pancreático

27. ¿Cuál de las siguientes enzimas está localizada en el borde del cepillo y juega un rol en
la digestión de proteínas?
a. Alpha dextrinase
b. Pepsina
c. Enterocinasa
d. Lactasa
e. Carboxipeptidasa A.

28. Una de los siguientes sustancias, NO sirve como un buen agente emulsificante:
a. Colesterol
b. Ácidos grasos
c. Sales biliares
d. Lecitina
e. Proteínas de la dieta

29. La sustancia que estimula el crecimiento de la mucosa gástrica es:


a. Secretina
b. Motilina
c. Péptido estimulante de la mucosa gástrica
d. Gastrina
e. Histamina

30. ¿Cuál de las siguientes alternativas es una función de la colecistocinina?


a. Relajación de la vesícula para la salida de bilis
b. Secreción de ácidos biliares
c. Contracción del esfínter de Oddi
d. Secreción de enzimas pancreáticas
e. Contracción del duodeno
31. Con respecto a la anatomía del tronco celíaco, señale lo correcto
a. El tronco celíaco se origina de la cara posterior de la aorta abdominal
b. Es una arteria delgada que tiene un calibre entre 2 y 3 mm
c. Una de sus ramas es la arteria gástrica derecha
d. La hepática común que es una de sus ramas, participa en la irrigación del estómago.

32. Con respecto a la anatomía del duodeno, marque la respuesta correcta:


a. Tiene una distribución en forma de “C”, que rodea la cola del páncreas
b. La 3ra porción duodenal está contenida en la pinza vascular aortomesentérica
c. Entre la 1ra y 2da porción se forma un ángulo, conocido como el ángulo de Treitz
d. La 4ta porción se dirige a la izquierda, hacia abajo y hacia atrás.
e. En la tercera porción desemboca el conducto colédoco.

33. El hígado está ampliamente tapizado por peritoneo, la estructura que conecta la cara
diafragmática del hígado precisamente con el diafragma es el ligamento:
a. teres
b. falciforme
c. triangular
d. hepático común
e. coronario

34. En el íleon se absorbe aproximadamente el 95% de ...................... a través de la


circulación enterohepática.
a. agua
b. colesterol
c. sales biliares
d. hidroxicobalamina
e. factor intrínseca

35. La ................ estimula el mecanismo paracrino de la secreción de ácido clorhídrico.


a. histamina
b. acetilcolina
c. gastrina
d. secretina
e. somatostatina

36. En la digestión de proteínas, ................ es el principal estímulo para convertir el


pepsinógeno en pepsina.
a. la gastrina
b. el pH ácido
c. la acetilcolina
d. la ptialina
e. la somatostatina

37. Con respecto a la somatostatina, marque lo correcto:


a. Es secretada por las células S del intestino
b. Induce a la producción de VIP
c. Interviene en la fase intestinal de la secreción gástrica
d. Produce acetilcolina para estimular a la célula parietal
e. No interviene en la regulación de la secreción de ácido clorhídrico

38. En pecten anal, es una estructura comprendida entre:


a. la línea pectínea y los senos anales
b. la línea blanca y la apertura anal
c. el esfínter anal interno y el externo
d. la línea anocutánea y la línea pectínea
e. la línea blanca y columnas anales

39. ¿Cuál de las siguientes alternativas es una proenzima pancreática?


a. Tripsina
b. Elastasa
c. Quimotripsinógeno
d. Amilasa
e. Procarboxipeptidasa C.

40. En la segmentación hepática de Couinaud, el segmento hepático señalado con la flecha,


corresponde a: En la segmentación hepática de Couinaud, la flecha señala el segmento
.............. Hepático.
a. IV
b. V
c. VI
d. VII
e. VIII

EXAMEN FINAL 2019-02

1. La reabsorción de sodio y cloro en las glándulas salivales se da principalmente en el:


a) conducto intercalado
b) conducto excretor
c) conducto estriado
d) acino glandular

2. Paciente de 35 años con dolor en hipocondrio derecho irradiado a la punta de la


escápula. Se observa en la ecografía abdominal cálculos en la vesícula biliar. Es operado de
emergencia realizando una incisión a lo largo del reborde costal derecho. La información de
dolor de esta zona es inervada por las raíces nerviosas:
a) T5 – T9
b) T6 – T7
c) T7 – T8
d) T9 – T10

3. Paciente de 23 años con dolor intenso periumbilical de inicio agudo. Es ingresado a sala
de operaciones por abdomen agudo, el cirujano encuentra sangrado a unos 60 cm proximal
a la válvula ileocecal. La estructura que está sangrando muy probablemente es un derivado
embriológico del:
a) uraco
b) ligamento Hepatoduodenal
c) alantoides
d) conducto vitelino

4. Paciente de 64 años de edad con dolor en epigastrio que se distribuye en banda hacia la
espalda. En la tomografía se encuentra tumor en cabeza de páncreas que compromete a un
vaso que discurre entre la cabeza del páncreas y el proceso uncinado. Debido a esto, este
cáncer se considera irresecable porque compromete a la arteria:
a) mesentérica superior
b) mesentérica inferior
c) tronco celíaco
d) aorta

5. Paciente con carcinoma gástrico avanzado, en preoperatorio para gastrectomía total.


¿Cuál de los siguientes ganglios linfáticos recibirá primero células metastásicas con mayor
probabilidad:
a) celíaco
b) cisterna del quilo
c) esplénico
d) gástrico posterior

6. Paciente de 77 años con dolor abdominal difuso de 3 horas de evolución. En los estudios
de imágenes se observa oclusión probablemente aterosclerótica de la arteria mesentérica
superior; no se observa necrosis, lo cual puede ser explicado por la irrigación sanguínea
colateral. ¿Qué vasos ofrecen colaterales entre el tronco celíaco y la arteria mesentérica
superior?
a) Gástrica izquierda y hepática
b) Cística y duodenal
c) Gastroomental derecha e izquierda
d) Pancreaticoduodenal superior e inferior

7. Paciente de 62 años con vólvulo de intestino delgado e isquemia intestinal. Se realiza


laparotomía exploratoria para liberar la obstrucción. ¿Cuál estructura se utilizará como punto
de referencia para determinar la posición de la unión duodenoyeyunal?
a) Vasos rectos
b) Ligamento suspensorio del duodeno (de Treitz)
c) Ligamento frenocólico
d) Nacimiento de la Arteria mesentérica superior

8. Paciente de 42 años con dolor abdominal intenso y hematemesis. En la endoscopia se


observa una úlcera duodenal posterior perforada con hemorragia intraabdominal. ¿Cuál de
las siguientes arterias estará comprometida?
a) Gástrica izquierda
b) Mesentérica superior
c) Gástrica derecha
d) Pancreaticoduodenal posterosuperior
9. Paciente de 51 años con antecedente de enfermedad diverticular acude a emergencia por
sangrado profuso y dolor en cuadrantes inferiores. ¿Cuál es el origen más probable de la
sangre que pierde en la hemorragia?
a) Cólica media
b) Mesentérica inferior
c) Rectal superior
d) Rectal inferior

10. Paciente de 13 días de vida con vómitos explosivos a las dos horas después de lactar.
Al examen físico se palpa la oliva pilórica. ¿Cuál es el nervio cuyos filetes dan inervación
eferente a la estructura afectada?
a) Vago
b) Esplácnico torácico mayor
c) Mesentérico superior
d) Esplácnico torácico menor

11. Paciente de 80 años con dolor abdominal intenso y antecedente de estreñimiento


crónico. En la colonoscopia se observa divertículos con áreas ulceradas difusas en colon
sigmoides y descendente. El cirujano programa una cirugía de resección y planifica que
para realizar esta recepción tendrá que cortar los siguientes vasos y nervios:
a) Plexo mesentérico superior y arteria rectal.
b) Nervio esplácnico torácico mayor y arteria cólica media.
ramas del nervio vago y arteria ileocólica.
d) nervio esplácnico pélvico y arteria cólica izquierda.

12. Paciente de 46 años ingresa a emergencia con dolor en cuadrante superior derecho e
ictericia. En la ecografía se observa cálculos en la vesícula biliar. ¿Cuál de los siguientes
nervios transmite el dolor de la colecistitis?
a) Filetes aferentes del nervio vago derecho, referido al ángulo inferior de la escápula
b) Filetes nerviosos de las raíces T1 a T4, con referencia al hombro izquierdo
c) Fibras aferentes simpáticas del nervio esplácnico torácico mayor, con referencia a
los dermatomas T6 a T8
d) Fibras aferentes de los ramos primarios dorsales de los nervios raquídeos T8 a T10
con referencia al epigastrio

13. Un niño de 8 años es alimentado por sus padres con un Cheese Burger, papitas fritas y
un vaso de Coca Cola. ¿La presencia de lípidos, carbohidratos y proteínas en el duodeno
estimulan la liberación de cuál de las siguientes hormonas en la mucosa duodenal?
a) Colecistoquinina (CCK)
b) Péptido inhibidor gástrico (GIP)
c) Secretina
d) Insulina

14. La fase cefálica del control de la secreción gástrica corresponde a cerca del 30% de la
secreción ácida y es consecuencia de un reflejo. ¿Cuál de las siguientes alternativas puede
eliminar totalmente la fase cefálica de la secreción gástrica?
a) Vaguectomía
b) Uso de atropina
c) Uso de bloqueador de receptor H2 de histamina
d) Uso de antiácidos

15. Recién nacido de 24 horas con llanto, vómitos y sin eliminación de meconio. Luego de
los estudios auxiliares se diagnostica la Enfermedad de Hirschsprung, esta enfermedad se
caracteriza por la ausencia de:
a) las células de Cajal en plexo mientérico
b) el plexo mientérico en el recto y colon sigmoides
c) las células mioepiteliales en el recto distal
d) el plexo de Meissner en el recto distal y colon sigmoides

16. Los complejos motores migratorios aparecen aproximadamente cada 90 minutos entre
las comidas, y se considera que son estimulados por la hormona motilina. La ausencia de
estos complejos migratorios podría producir un aumento en:
a) la motilidad duodenal
b) el vaciamiento gástrico
c) la deglución
d) las bacterias intestinales

17. ¿Cuál de las siguientes es una consecuencia probable de la resección del íleon?
a) Gastritis atrófica
b) Deficiencia de vitamina B12
c) Esteatorrea
d) Úlcera péptica

18. Los movimientos en masa son importantes en la fisiología intestinal. Estos movimientos
en masa ocasionan:
a) la sensación de defecar
b) el peristaltismo duodenal
c) la retropulsión gástrica
d) la contracción del esfínter anal interno

19. La toxina colérica hace que aumenten los niveles de AMPc intracelular, y este aumento
hace que se mantenga abierto un canal en las células de la cripta de Lieberkühn. En
condiciones fisiológicas, en una persona sana ¿Qué sustancia puede promover que el canal
quede abierto también?
a) Somatostatina
b) Óxido nítrico (NO)
c) Péptido intestinal vasoactivo (VIP)
d) Péptido similar al glucagón 1 (GLP 1)

20. Paciente de 64 años con tumor abdominal que comprime la cisterna del quilo. En la
biopsia de duodeno tomada como parte del estudio, el patólogo puede observar
a) Dilatación del vaso quilífero central
b) Contracción de las venas de las vellosidades
c) Vellosidades intestinales más largas
d) Engrosamiento de la lámina basal
21. En la enfermedad de Crohn es posible encontrar células de Paneth en el colon. Esto se
puede deber a la especial función de estas células en:
a) la activación de la inmunidad adquirida
b) mantener la inmunidad innata
c) producir hormonas
d) producir Ig A

22. En 1967 se descubrió que la epidemia de Kuru, una enfermedad por priones, en el
distrito de Okupa en Papúa Nueva Guinea, era causada por la costumbre de comer la carne
de los muertos. Ahora se sabe que las proteínas priónicas ingresan al organismo a través
de:
a) los enterocitos
b) la transmigración
c) las células de Paneth
d) las células M

23. Paciente de 48 años con alteraciones en el tránsito intestinal por diabetes mellitus tipo 2;
se presenta con esteatorrea, flatulencia y malabsorción de grasas. Las pruebas de función
hepática y biliar están dentro de rangos normales. Una causa de la disminución de sales
biliares puede ser:
a) el sobrecrecimiento bacteriano
b) la deficiencia de pepsina
c) la deficiencia de elastasa
d) la hiperestimulación del GLUT5

24. Al usar azúcar de mesa (sacarosa) para endulzar su café, el estudiante de medicina
sabe que lo más probable es que para su absorción tendrá utilizar el/los transportadoras)
________________ que se encuentran en la membrana apical de los enterocitos.
a) SGLT-1
b) GLUT2 y GLUT5
c) SGLT-1 y GLUT5
d) SGLT1 y GLUT2

25. Una persona con la producción normal de lactasa; cada vez que toma leche, los
productos de la degradación de la lactosa por parte de la lactasa ingresarán al enterocito
usando el/los transportador(es) _____________:
a) SGLT-1
b) GLUT2 y GLUT5
c) SGLT-1 y GLUT5
d) SGLT1 y GLUT2

26. En una persona sana, luego de una comida basada en carnes rojas cocinadas con baja
cantidad de sal, al enterocito pueden ingresar solamente:
a) aminoácidos
b) aminoácidos y dipéptidos
c) aminoácidos, dipéptidos y tripéptidos
d) aminoácidos, dipéptidos, tripéptidos y tetrapéptidos
27. Paciente con deficiencia congénita de procolipasa, sufre de esteatorrea cada vez que
come comidas ricas en grasas. En el estudio de composición de las heces, uno espera
encontrar una cantidad aumentada de:
a) colesterol
b) triglicéridos
c) fosfolípidos
d) lisolecitina

28. Al tomar su café en Starbucks, un estudiante de medicina sufre una quemadura de


primer grado en el tercio anterior de la superficie dorsal de la lengua. La información de
dolor es transmitida por el nervio:
a) cuerda del tímpano
b) glosofaríngeo
c) lingual
d) facial

29. Paciente de 32 años con esteatosis hepática no alcohólica. Se le realiza una biopsia
hepática que confirma la esteatosis; en el tejido se observan depósitos de lípidos en los
hepatocitos, los cuales contienen principalmente:
a) triglicéridos
b) colesterol
c) ácidos grasos libre
d) acil-carnitina

30. Paciente de 21 años que es estudiante de medicina, llega a consulta refiriendo que en
épocas de exámenes su piel se vuelve amarillenta. Le realizan el diagnóstico de Síndrome
de Gilbert asociado a mutación del gen UGT1A1, luego de unas semanas acude a la
consulta por ictericia asociada a resfriado. Al hacerle un análisis de sangre, usted sabe que
encontrará valores elevados de:
a) bilirrubina directa
b) bilirrubina indirecta
c) bilirrubina tipo delta
d) fosfatasa alcalina

31. Se realizó un experimento en el cual se inyectó tinta china en el peritoneo de ratas de


laboratorio. Al realizarse una biopsia hepática de dichos animales, se encontró que el tinte
negro estaba depositado en:
a) las células de Ito
b) los hepatocitos
c) las células de Kupffer
d) las células de Disse

32. Paciente de 74 años de edad con shock hipovolémico asociado a deshidratación aguda
severa. En este paciente es posible encontrar hipoxia en zona ...... del lobulillo hepático e
infarto .................. del intestino.
a) 1 / mucoso
b) 1 / transmural
c) 3 / transmural
d) 3 / mucoso

33. Niño de 5 años con historia de tres días de evolución caracterizado por fiebre, malestar
general, odinofagia, anorexia, e irritabilidad. Al examen se observa lesiones ulcerativas de 4
mm de diámetro en mucosa yugal, con fondo blanquecino y eritema periférico. El
diagnóstico más probable es:
a) candidiasis oral
b) leucoplasia
c) aftas
d) herpes

34. Paciente de 52 años con enfermedad por reflujo gastroesofágico de 30 años de


evolución. Se realiza endoscopía en la que se encuentra mucosa eritematosa proximal a la
línea Z. Para corroborar la presencia de lesión preneoplásica, se toma una biopsia de esa
zona, en la que se espera encontrar:
a) metaplasia gástrica
b) metaplasia intestinal
c) displasia gástrica
d) adenocarcinoma

35. Un estudiante de medicina termina su último examen final. Al abrir la boca para comer
una hamburguesa con papas fritas, la activación de los receptores muscarínicos de las
células acinar y ductal estimularan un mayor flujo de saliva, con lo cual disminuirá la
concentración salival de:
a) potasio
b) bicarbonato
c) sodio
d) cloro

36. Un estudiante toma su desayuno consistente en un pan con mantequilla y queso. Antes
que se puedan digerir las grasas, es necesario que sean emulsificadas. La hormona
............................ estimula la liberación de las sustancias emulsificadoras.
a) colecistoquinina (CCK)
b) secretina
c) lipasa pancreática
d) gastrina

37. Al comer un pollo a la brasa entero, con papitas fritas y ensalada, la sustancia que
estimulará la liberación de HCl en el estómago es:
a) el neuropéptido Y
b) la secretina
c) la bombesina
d) la colecistoquinina (CCK)

38. Paciente con disminución marcada del apetito asociado a cáncer terminal, se podría
utilizar análogos de ................. para promover la ingesta de alimentos.
a) el péptido similar al glucagón (GLP)
b) la serotonina
c) la secretina
d) la endorfina

39. Un hombre de 51 años presenta de forma súbita vómitos masivos de sangre roja
brillante. Tiene como antecedente hepatitis viral B hace 23 años. En la exploración física: FC
103 latidos/min, PA 85/50 mmHg, se palpa la punta del bazo y niega vómitos. Su
hematocrito es 21%, la prueba serológica de HBsAg es positiva. En la ecografía se observa
hígado con nodulaciones. ¿Cuál es la causa más probable para la hematemesis?
a) Esófago de Barrett
b) Síndrome de Mallory Weiss
c) Varices esofágicas
d) Esofagitis por reflujo

40. Durante el reflejo del vómito, uno de los primeros sucesos es:
a) el peristaltismo inverso
b) la contracción del píloro
c) la apertura de la glotis
d) el esfínter esofágico inferior abierto

41. Paciente de 34 años que acude por diarrea desde hace 4 días asociado a comer papa
rellena con ají en el Estadio Nacional, se exacerba cuando toma lácteos o come grasas. Se
acompaña de flatulencia y episodios de tenesmo. Este cuadro de diarrea e intolerancia a la
lactosa de inicio agudo se explica por la:
a) enfermedad de Crohn
b) infección por V. cholera
c) colitis ulcerativa
d) giardiasis

42. En cuanto a la secreción pancreática, mientras mayor es el flujo, mayor es la


concentración de:
a) potasio
b) bicarbonato
c) cloro
d) sodio

43. Paciente de 10 años con diarrea crónica, distensión abdominal, anorexia. Se le ha


encontrado anticuerpos antigliadina y antiendomisio. Es más probable que la diarrea se
correlacione con el hallazgo histológico de:
a) adelgazamiento de las criptas
b) linfocitos intramusculares
c) atrofia de las vellosidades
d) úlceras duodenales

44. La glándula parótida está inervada por el par craneal:


a) VII
b) IX
c) X
d) XII

45. Las glándulas de Brunner se encuentran en la:


a) mucosa del esófago
b) submucosa del íleon
c) mucosa del yeyuno
d) submucosa del duodeno

46. ¿Cuál de las siguientes afirmaciones describe correctamente la función de la inervación


parasimpática del tracto gastrointestinal?
a) La norepinefrina es el principal neurotransmisor excitatorio.
b) La actividad parasimpática produce la relajación de los esfínteres
c) La actividad parasimpática excesiva puede provocar un trastorno llamado íleo
paralítico (parálisis del músculo liso intestinal)
d) El pH luminal, la osmolaridad y la distensión muscular son detectados por fibras
parasimpáticas eferentes

47. Al ingerir una cantidad de glucosa por vía oral, esta es interiorizada en las células del
organismo más rápido que si esa misma cantidad de glucosa hubiese sido administrada por
vía endovenosa. Este fenómeno sucede gracias a la acción de la sustancia secretada por
las células:
a) G
b) I
c) K
d) S

48. A ............... secreción de HCl en el lumen gástrico, ................. pH en la sangre venosa


gástrica
a) mayor /mayor
b) mayor / menor
c) menor / mayor
d) mayor / igual

49. ¿En cuál de las siguientes situaciones hay un menor flujo de secreción salival?
a) Masticar goma de mascar
b) Imaginarse ser sometido a un examen dental
c) Exposición a olor nauseabundo
d) Sueño

50. El alcohol y la cafeína estimulan la fase ______________ de la producción de ácido


clorhídrico.
a) cefálica
b) gástrica
c) intestinal
¿Cuál de los siguientes mecanismos ocurren durante la defecación? En la posición de
cuclillas el músculo puborrectal se halla relajado

La estructura número 4 corresponde a ____ y está


___. La dentina/ formado por células derivadas de
la cresta neural

Un paciente luego de un accidente sufre lesión del


piso de la boca, se constata daño del nervio:
cuerda del tímpano, en este caso se esperaría
encontrar disminución de la ____ Sensación del
gusto en los 2 tercios anteriores

¿Cuál de las siguientes afirmaciones es la correcta


sobre la gastrina? Actúa en la célula diana
mediante su receptor CCK tipo B

Al recibir un paciente con signos de hipovolemia y antecedentes de trauma en abdomen por


accidente de tránsito, usted identifica radiológicamente: lesión primera vértebra lumbar y
signos de lesión en páncreas; durante la cirugía se observó pobre irrigación de asas
intestinales. El vaso afectado es la arteria: mesentérica superior

Un paciente sufre daño a nivel del cuello con lesión muscular en la región de la faringe. En
el examen físico se determina dificultad para la elevación de la faringe y para el cierre del
istmo de las fauces. En este caso probablemente está afectado el músculo: palatofaríngeo

Varón de 50 años a quien le realizan la curación de la segunda molar de la arcada derecha.


En un momento determinado, el paciente acusa de intenso dolor de la pieza dentaria en
tratamiento. La vía aferente del dolor viaja a través del nervio: trigémino V2

La distensión gástrica por los alimentos produce incremento de secreción de HCL mediante
la producción de ____ que estimula a las células ___ vía proteína: gastrina/ parietal/ Gq

Un niño de 3 años llega a emergencias con disfagia (dificultad para tragar), dolor retro
esternal, salivación y llanto. Se sospecha de ingesta de cuerpo extraño (moneda) en el
esofago, al ser evaluado se constata en una radiografía presencia de cuerpo extraño a nivel
de C6. El cuerpo extraño estará suspendido a nivel del estrechamiento producido por ____
el músculo cricofaríngeo

La triada portal (arteria hepática, vena portal y conducto biliar común) está contenida en el
ligamento ____ hepatoduodenal/ mesenterio ventral

EXAMEN FINAL 2019-00

1. Un niño de 2 años es llevado a la consulta por diarrea persistente, edema de las


extremidades y falta de crecimiento en relación a su edad. Los análisis de sangre
revelan que tiene concentración plasmática baja de proteínas (hipoproteinemia). Como
parte del estudio se coloca colecistocinina (CCK) endovenosa y se recoge muestras del
líquido duodenal por endoscopia; el resultado del líquido confirma incapacidad para
hidrolizar proteínas a un ph neutro, esta situación mejora al añadir una pequeña
cantidad de tripsina. El paciente probablemente está sufriendo la falta congénita de
______

a. PEPT – 1

b. Pepsinógeno

c. Enterocinasa

d. Carboxipeptidasa

2. Paciente mujer de 35 años acude a consulta por sensación de sequedad y lesiones


en cavidad oral. Al examen se observa atrofia de la mucosa, fisura y úlcera; nota
además sequedad e irritación de la córnea y aumento del tamaño de las glándulas
parótidas. Su diagnóstico más probable es artritis reumatoide, el hallazgo más probable
en una biopsia de glándula parótida es ____

a. Presencia de acinos normales con hiperplasia de células distales

b. Gran infiltración de linfocitos y células plasmáticas

c. Hiperplasia de acinos glandulares serosos

d. Gran infiltrado de linfocitos y neutrófilos

3. Un hombre de 42 años de edad se presenta al médico con una historia de 1 año de


evolución, caracterizado por dolor abdominal bajo y diarrea con crisis sanguinolentas.
Manifiesta además pérdida de peso de 8Kg durante este periodo. La colonoscopia revela
lesión difusa en el colon con afectación del recto. La biopsia de estas lesiones revela
adelgazamiento de la pared, inflamación y ulceración de la mucosa y submucosa. El
diagnóstico más probable en este caso es:

a. Síndrome de colon irritable

b. Enfermedad de Crohn

c. Colitis ulcerativa

d. Sprue celiaco

4. Dos estudiantes deciden tomar un receso para comer una hamburguesa a la hora del
almuerzo. Antes de llegar a la cafetería, impulsos nerviosos provenientes del complejo
vagal dorsal iniciarán la secreción de ácido gástrico por liberación de _____ desde el
sistema nervioso entérico

a. Serotonina

b. Colecistoquinina
c. Péptido inhibidor vasoactivo

d. GRP (péptido liberador de gastrina)

5. Un niño de cuatro años de edad es llevado a la consulta por cuadros diarreicos


frecuentes caracterizados por heces pálidas, voluminosas y fétidas, al examen físico
presenta bajo peso y talla para la edad. Se mide la concentración de cloruro en el sudor y
se encuentra que sus valores son muy elevados. La alteración más importante a nivel de
células ductales del páncreas tiene relación directa con la conductancia de ___

a. Bicarbonato

b. Potasio

c. Sodio

d. Cloro

6. Se evalúa los valores séricos de las siguientes sustancias a un paciente con enfermedad
hepática terminal, en este paciente se espera encontrar la combinación con la letra

a. Glucosa aumentada, amoniaco disminuido, albúmina disminuida

b. Glucosa disminuida, Amoniaco aumentado, albúmina aumentada

c. Glucosa aumentada, amoniaco aumentado, albúmina aumentada

d. Glucosa disminuida, amoniaco aumentada, albúmina disminuida

7. Una mujer de 35 años de edad HIV positiva se presenta al médico con dolor abdominal
en cuadrante superior derecho e ictericia. La paciente refiere haber tenido múltiples
episodios de ictericia durante los últimos 10 años. Los exámenes para determinar
hepatitis viral dieron positivos para hepatitis B, siendo catalogado el caso como hepatitis
crónica con alteración funcional. En un examen de sangre ¿cuál de los siguientes
parámetros está disminuido?

a. Albúmina

b. Bilirrubina

c. Fosfatasa alcalina

d. Tiempo de protrombina

8. En el reflejo peristáltico del intestino delgado ¿cuál de los siguientes eventos sucede en
la porción caudal del bolo alimenticio?

a. Acción del péptido inhibidor vasoactivo (VIP) en el músculo circular

b. Acción del NO (óxido nítrico) en el músculo longitudinal)


c. Contracción del músculo longitudinal interno

d. Acción de acetilcolina en el músculo circular

9. Varón de 58 años de edad con enfermedad de Crohn severa fue sometido a una
resección ileal. Después de la cirugía este paciente padecerá de esteatorrea esto se
explica porque:

a. Se inhibe la acción de la 7 alfa hidroxilasa

b. El pool de ácidos biliares se incrementa

c. Hay malabsorción de ácidos biliares

d. El páncreas no secreta lipasa

10. En un experimento se inserta un balón en el estómago de un voluntario, se infla poco a


poco mientras que se vigilan las presiones intraluminales. Aunque el volumen del balón
aumenta considerablemente, las presiones permanecen constantes. Esta relación
volumen presión se explica por la liberación local de

a. Acetilcolina y gastrina

b. Norepinefrina y óxido nítrico

c. Colecistoquinina y óxido nítrico

d. Óxido nítrico y péptido inhibidor vasoactivo

11. ¿cuál de las siguientes alternativas es una característica de la secreción exocrina del
páncreas?

a. Es hipotónica respecto al plasma

b. Su mayor estímulo se da en la fase intestinal

c. Es estimulada por la presencia de bicarbonato en el duodeno

d. La secreción enzimática es estimulada principalmente por la secretina

12. Las estructuras en el hígado que permite que los productos metabólicos unidos a
proteínas tengan acceso a las membranas basolaterales de los hepatocitos, son:

a. Los canalículos

b. Las células de ito

c. Las fenestras sinusoidales

d. Las uniones intercelulares herméticas


13. La composición de la bilis es modificada conforme fluye por los conductillos biliares.
Durante este tránsito se espera que aumente la concentración de

a. Ig A

b. Glucosa

c. Protones

d. Vitamina A

14. Se mide experimentalmente el contenido gástrico de dos personas. La persona “A”


tiene alto contenido de grasa y la persona “B” tiene un contenido isotónico ¿Cuál de las
siguientes es correcta respecto al vaciamiento gástrico?

a. Hay ralentización del vaciado gástrico sólo en A”

b. El vaciamiento gástrico es más rápido en ambos

c. Hay ralentización del vaciado gástrico sólo en “B”

d. Hay ralentización del vaciado gástrico en ambos casos

15. El examen endoscópico de un paciente con hipertensión portal grave revela venas
tortuosas que sobresalen hacia la luz del esófago. El paciente recibe tratamiento
quirúrgico mediante la colocación de una derivación que conecta la vena porta a la vena
cava. Después de la operación el riesgo de encefalopatía _______ y el riesgo de
sangrado de várices ______

a. Disminuirá/ disminuirá

b. Disminuirá/ aumentará

c. Aumentará/ disminuirá

d. Aumentará/ aumentará

16. Un paciente varón de 18 años de edad acude al médico para sus exámenes de
rutina. Sus resultados de laboratorio muestran un valor de bilirrubina séricas de 4 mg/dl
y una bilirrubina directa de 0,3 mg/dl. Las pruebas de función hepática son normales. La
alteración que explica mejor este caso es por la deficiencia de

a. Transaminasas

b. Hemo oxigenasa

c. La 7 alfa hidroxilasa

d. Glucoronil transferasa
17. Un hombre de 57 años de edad es llevado a urgencias con hematemesis masiva
roja brillante, a su llegada se halla inconsciente con PA: 80/40 mm Hg y FC:124 lat/min.
Luce ictérico con presencia de “arañas vasculares en el tórax anterior y extremidades”,
abdomen distendido con signo de oleada positiva. Se encuentra esplenomegalia y
pérdida de la masa muscular en extremidades. La anastomosis vascular responsable del
sangrado en este paciente es:

a. Vena gástrica izquierda y vena ácigos

b. Arteria gástrica izquierda y vena ácigos

c. Vena paraumbilical y vena epigástrica inferior

d. Vena gástrica izquierda y vena esofágica superior

18. Un estudiante de medicina está comiendo un plato de comida a base de


champiñones, espárragos y salsa de soya. El estímulo del sabor umami contenido en
todos estos alimentos viaja a través del nervio:

a. Lingual

b. Hipogloso

c. Glosofaríngeo

d. Cuerda del tímpano

19. Una paciente de 30 años de edad es sometida a una cirugía en lado medio derecho
por un problema de aterosclerosis. Luego de la cirugía refiere alteración sensitiva de la
lengua. Al evaluar el caso usted esperaría encontrar

a. Alteración en la sensación del dolor y temperatura

b. Alteración en la sensación del dolor en los dos tercios anteriores de la lengua

c. Alteración en la sensación del gusto en el tercio posterior de la lengua

d. Sensación del dolor, tacto y temperatura conservadas

20) En un paciente de 45 años de edad con colestasis biliar se encuentra una elevación de
los niveles sanguíneos de fosfatasa alcalina hasta 3 veces la cifra normal ¿Cuál de las
siguientes alternativas estará también elevada como evidencia del daño de la vía biliar?

a. Tiempo de protrombina y albúmina sérica

b. Transaminasas hepáticas (ALT y AST)

c. Gamma glutamil transpeptidasa

d. Glucoronil transferasa
21) Experimentalmente se incrementa la velocidad de se incrementa la velocidad de
secreción salival con una sustancia, en el análisis de la composición de esta saliva obtenida
se espera encontrar ………..

a. Disminución de la concentración de bicarbonato que supera la concentración


plasmática
b. Aumento de la concentración de cloro y sodio que supera la concentración
plasmática
c. Aumento de la concentración de bicarbonato que supera la concentración
plasmática
d. Disminución de la concentración de potasio y bicarbonato

22) Lactante de 3 meses de vida es atendido por presentar diarrea, se administra una
solución de glucosa y electrolitos por vía oral. La proteína de membrana apical que explica
la capacidad de esta solución para proporcionar aporte de glucosa e hidratación en …..

a. CFTR

b. SGLT-1

c. GLUT-2

d. GLUT-5

23) Paciente ha sufrido herida de bala en el abdomen, se le ha tenido que extirpar el


segmento medio y distal del íleon. En este caso la síntesis hepática de sales biliares estará
……

a. Sin cambios en el ritmo de síntesis

b. Disminuida por inhibición de la enzima colesterol 7 alfa hidroxilasa

c. Incrementada por estímulo de la enzima colesterol 7 alfa hidroxilasa

d. Incrementada por inhibición de la enzima colesterol 7 alfa hidroxilasa

24) Un varón de 75 años ingresa al consultorio por presentar ictericia marcada de piel y las
escaleras. El estudio del paciente mostró que presentaba un tumor que obstruía la totalidad
del conducto hepático común. ¿Cuál de los siguientes conductos se encontrará dilatado en
este paciente

a. de Wirsung

b. de Hering

c. colédoco

d. cístico

25) Correlaciones las dos columnas y marque la fórmula correcta:

1. Enfermedad Hirschsprung ( ) heces cono moco y sangre


2. Diarrea osmótica ( ) intolerancia a lactosa

3. Diarrea secretoria ( ) aganglionosis congénita

4. Diarrea exudativa ( ) canales de Cl- en las células de la cripta

a. 4231 b. 1234 c. 2143 d. 4213

26) La vena umbilical obliterada del hígado después del nacimiento se transforma en el
ligamento

a. Cruzado

b. Redondo

c. Coronario

d. Falciforme

27) Llega a su guardia nocturna una madre que trae a su RN masculino de 2 semanas de
vida con mal estado general y resequedad de mucosas. Usted observa que lacta
ávidamente, pero a las 2 horas presenta vomito postprandial no bilioso en proyectil. Al
realizar la historia clínica descubre que el lactante recibió profilaxis con macrólidos para tos
ferina. Usted sospecha principalmente en:

a. Estenosis pilórica hipertrófica congénita

b. Fistula traqueo esofágica

c. Estenosis duodenal

d. Atresia duodenal

28) En la regulación del apetito y la saciedad, la estimulación experimental crónica del


núcleo ventro medial del hipotálamo producirá:

a. Afagia

b. Obesidad

c. Hiperfagia

d. Activación de neuronas relacionadas a NPY

29) Paciente mujer de 25 años acude por dolor en fosa iliaca derecha que empeora al toser
o caminar, asociada a náuseas y vómitos por lo cual acude a emergencia, Dos días después
de realizarle un apendicetomía, la paciente desarrolla fiebre alta (39º C), esta hipotensa y
presenta dolor abdominal. La laparotomía exploratoria muestra un gran volumen de sangre
en la cavidad peritoneal por lesión de un vaso producida durante la apendicetomía ¿Cuál de
las siguientes arterias debe ligarse para detener la hemorragia?
a. Cólica derecha y arteria rectal superior

b. Ileocólica y arteria cólica media

c. Mesentérica superior

d. Ileocólica

30) La onda peristáltica secundaria del esófago se caracteriza por ser originada

a. Por el plexo de Meissner del esófago

b. Por el plexo mesentérico del esófago

c. Por el reflejo de la deglución

d. Durante la masticación

31) ¿Cuál de los siguientes es una causa de ictericia con bilirrubina conjugada aumentada?

a. Ictericia del recién nacido

b. Obstrucción del colédoco

c. Anemia hemolítica

d. Gran hematoma

32) En relación a la absorción de nutrientes, la absorción de dipéptidos y tripéptidos a nivel


de las células epiteliales del intestino delgado, se da principalmente debido a:

a. Incremente de canales de Cl en la membrana apical

b. La gradiente de bicarbonato en la membrana basal

c. La gradiente de iones H+ en la membrana apical

d. La gradiente de Na+ en la membrana apical

33) Paciente de 20 años es traído a la emergencia por presentar diarreas desde hace 2
días. Familiar refiere que las deposiciones son liquidas y abundantes, al examen luce
deshidratado y se plantea que la diarrea es producida por una toxina que estimula la
transformación de ATP a AMPc con apertura de cales de Cl- y perdida de agua. El tipo de
diarrea más probable es:

a. Osmótica

b. Exudativa

c. Secretoria

d. Por intolerancia a la lactosa


34) Un niño fue operado por una obstrucción intestinal, observándose la presencia de
divertículo de Meckel. Según lo diferido marque lo correcto:

a. El 50% de la población lo presenta

b. Se localiza en el íleon muy cerca al yeyuno

c. Puede poseer tejido gástrico o pancreático

d. Se produce por una mala rotación de los intestinos

35) Marque la alternativa correcta respecto a la estructura marcada en el gráfico:

a. Se halla a 2 centímetros debajo de la papila duodenal mayor

b. Llega el conducto colédoco y pancreático principal

c. Llega el conducto hepático común y pancreático principal

d. Llega el conducto pancreático accesorio

36) ¿Cuál de las siguientes moléculas se encontrará aumentada en el citoplasma de las


células parietales de un paciente con un síndrome de Zollinguer Ellison?

a. Péptido liberador de gastrina (GRP)

b. Proteína G estimulante (GS)

c. Inositol Trifosfato (IP3)

d. AMO cíclico (AMPc)

37) Los fármacos inhibidores de la bomba de protones, actúan bloqueando la …………

a. Anhidrasa carbónica

b. ATPasa H+/K+ en la membrana luminal

c. ATPasa H+/K+ en la membrana basolateral

d. ATPasa Na+/K+ en la membrana basolateral


38) Un paciente fue diagnosticado de gastritis autoinmune, ¿cuál de las siguientes
alternativas es FALSA respecto a esta enfermedad

a. Afecta principalmente el fondo y cuerpo gástrico

b. Se produce hiperplasia de células G secundaria la aclorhidria

c. El propio sistema inmune destruye principalmente las células parietales

d. Se produce atrofia de la mucosa, aclorhidria, hipergastrinemia y déficit de vitamina


B6

39) Marque la correlación correcta:

1. Herpes virus ( ) en relación al abuso de antibióticos

2. Candidiasis oral ( ) lesiones vesiculares como racimo de uvas

3. Eritroplaquia ( ) Mega esófago

4. Enfermedad de Chagas ( ) Lesión precancerigena

a.- 2431 b. 1234 c. 4123 d. 2143

40) En un paciente con insuficiencia renal crónica, el déficit en la absorción de calcio a nivel
del enterocito se debe a lo siguiente:

a. No se convierte la 25 hidroxicolecalciferol a 1,25 dihidroxicolecalciferol

b. No se convierte la 1,25 dihidroxicolecalciferol a 25 hidroxicolecalciferol

c. Existe un descenso de la alfa 25 hidroxilasa renald. Se incrementa la producción


de Calbindina

41. Marque lo correcto en relación al divertículo de Meckel: Se encuentra usualmente a 60 cm de la


VIC

42. Luego de tres horas dando exámenes, un alumno de medicina comienza a sentir
hambre. Esta situación es probable que sea mediada por la _____ que es sintetizada por el
_____ grelina / estómago

43. Varón de 72 años, con antecedente de diabetes mellitus tipo 2, que presenta entropía
diabética caracterizada por estreñimiento. Este problema puede estar asociado a:
deficiencia de óxido nítrico

44. Varón de 54 años con diabetes mellitus tipo 2, es diagnosticado de gastroparesia debido
a que presenta sensación de llenura precoz al comer, y reflujo gastroesofágico. Esta
alteración en la relajación receptiva y en el vaciamiento gástrico lo más probable es que se
deba a una alteración en: el nervio vago

45. Lactante masculino de 5 meses de edad producto de un primer embarazo normal


controlado . Peso de nacimiento 3.120 g y talla de 51 cm . Lactancia materna satisfactoria,
con buen incremento ponderal . Sin antecedentes patológicos hasta 5 días atrás , cuando
comienza con vómitos posprandiales no biliosos , lácteos . Los síntomas aumentan en
frecuencia y magnitud hasta hacerse explosivos después de cada alimentación . No requiere
tos , fiebre , diarrea ni lesiones cutáneas .Es importante destacar que pese a los vómitos el
niño conserva el apetito y llora de hambre .Al examen físico presenta buen estado general.
Abdomen blando ,depresible e indoloro ,asociado a distensión de hemiabdomen superior
.Sin signos de deshidratación .No se palpan masas abdominales .Exámenes de
laboratorio:hemograma normal. Signos inflamatorios de fase aguda negativos .Alcalosis
metabólica leve en sangre venosa.

Producto de la hipertrofia pilórica uno esperaría que los niveles de gastrina se encuentren: .

Rpta}: ELEVADA

46. Lactante masculino de 5 meses de edad producto de un primer embarazo normal


controlado . Peso de nacimiento 3.120 g y talla de 51 cm . Lactancia materna satisfactoria,
con buen incremento ponderal . Sin antecedentes patológicos hasta 5 días atrás , cuando
comienza con vómitos posprandiales no biliosos , lácteos . Los síntomas aumentan en
frecuencia y magnitud hasta hacerse explosivos después de cada alimentación . No requiere
tos , fiebre , diarrea ni lesiones cutáneas .Es importante destacar que pese a los vómitos el
niño conserva el apetito y llora de hambre .Al examen físico presenta buen estado general.
Abdomen blando ,depresible e indoloro ,asociado a distensión de hemiabdomen superior
.Sin signos de deshidratación .No se palpan masas abdominales Exámenes de
laboratorio:hemograma normal. Signos inflamatorios de fase aguda negativos .Alcalosis
metabólica leve en sangre venosa.

En este paciente se puede esperar una mayor liberacion de :

Rpta: Enzimas pancreáticas

47. Lactante masculino de 5 meses de edad producto de un primer embarazo normal


controlado . Peso de nacimiento 3.120 g y talla de 51 cm . Lactancia materna satisfactoria,
con buen incremento ponderal . Sin antecedentes patológicos hasta 5 días atrás , cuando
comienza con vómitos posprandiales no biliosos , lácteos . Los síntomas aumentan en
frecuencia y magnitud hasta hacerse explosivos después de cada alimentación . No requiere
tos , fiebre , diarrea ni lesiones cutáneas .Es importante destacar que pese a los vómitos el
niño conserva el apetito y llora de hambre .Al examen físico presenta buen estado general.
Abdomen blando ,depresible e indoloro ,asociado a distensión de hemiabdomen superior
.Sin signos de deshidratación .No se palpan masas abdominales Exámenes de
laboratorio:hemograma normal. Signos inflamatorios de fase aguda negativos .Alcalosis
metabólica leve en sangre venosa.

En relación con la estructura afectada se encuentra:


Rpta: L1

48.Varón de 67 años con tos y disminución de peso asociado a tabaquismo pesado,


presenta actualmente disfagia progresiva a alimentos sólidos. Se considera la presencia de
un carcinoma de bronquio izquierdo y por esta razón le realizan una endoscopia esofágica
para descartar la posibilidad de una compresión esofágica por el tumor. Se espera revisar el
esófago en la _____ estrechez, que está a nivel de la vértebra _____.

Tercera estrechez - T6

49. En un niño menor de dos años con divertículo intestinal, este divertículo tiene su origen
en una falla en la obliteración de:

Conducto vitelino

50. Mujer de 43 años sufre un grave accidente de tránsito y está hospitalizada en coma, es
alimentada por vía intravenosa durante varias semanas. Producto de este tipo de
alimentación, se encuentra en la endoscopia atrofia de la mucosa gastrointestinal. La causa
más probable de esta atrofia son los bajos niveles séricos de la hormona:

Gastrina

51. Una mujer de 30 años llega al consultorio porque se queja de dificultades para deglutir,
la cual se agravan cada vez más. Se realiza un estudio manométrico para examinar la
generación de presión a lo largo del esófago. Esta prueba revela que las contracciones
como respuesta a la deglución están mal sincronizadas y que la presión en el esfínter
esofágico inferior permanece elevada. El diagnóstico más probable es _____ producido por
niveles bajos de _____:

acalasia / óxido nítrico

52. Paciente de 2 años, llega a emergencia por haber ingerido una moneda con la que
estaba jugando. El lugar más probable donde puede haberse quedado suspendido este
objeto es a nivel del estrechamiento producido a nivel del:

músculo cricofaríngeo

53. En una apendicectomía, al realizar la incisión de McBurney en la fosa iliaca derecha, es


necesario cortar los siguientes músculos, de afuera hacia adentro:

Oblicuo externo - Oblicuo interno - Transverso

54.Al consumir caramelos indirectamente activa la via:

Rpta: POMC/CART

55. Debido al uso de ranitidina , los valores de somatostatina en sangre:

Rpta: Disminuyen
56.En múltiples causas de la enfermedad por reflujo gastroesofágico, se puede considerar
tambien una alteración en las ___________ del esfínter esofágico inferior

Rpta: contracciones tonicas

57. Estudiante de medicina de la UPC de 21 años sufre de gastritis aguda ocasionada por
comer en lugares poco higiénicos. Suele consumir caramelos ( chupa ) mientras está en
clase hasta la tarde. Toma gaseosa regularmente (carbohidratos 46%, sodio 53%). También
toma regular cantidad de leche (grasa 35%, lactosa 35%, proteínas 30%), pues le calma un
poco el dolor el ardor que siente por la gastritis. Incluso, cuando puede, se toma dos vasos
de agua fría para calmar las molestias. Ha decidido ir al médico para tratarse pues ya no
soporta el dolor, el cual está seguro que los síntomas se deben a una elevada producción
de ácido clorhídrico en el estómago, y por ello le ha recetado Ranitidina (antihistamínico),
con lo que siente mejoría.

El consumo de una pequeña cantidad de gaseosa aumenta directamente la contracción


sérica de cual de las siguientes hormonas.

Rpta: Peptido 1 similar al glucagon(GLP-1)

58. Paciente es evaluado por faringitis aguda en consultorio externo. El médico de familia le
solicita que abra la boca y saque la lengua Para realizar la acción de sacar la lengua, es
necesario que se contraiga el músculo: geniogloso

59. en qué casos los vómitos son siempre biliosos

Rpta: atresia yeyunal

60. Durante una cirugía oncológica¿La extirpación de cuál de los siguientes órganos se
vería comprometida por la presencia de adventicia.

Rpta: recto

61. Estimulan la secreción ácido gástrica

Rpta: Proteínas

62. Cual de los líquidos corporales tiene el PH más alto

Rpta: Jugo pancreatico

63. En un paciente de 43 años con tumor carcinoide de páncreas productor de gastrina


(Sindrome de Zollinger-Ellison) se puede esperar una disminución en la:

Rpta: liberación de gastrina por la células G

64. El ligamento de Treitz característicamente:

Rpta: Suspende el ángulo de Treitz


65. Paciente de sexo masculino de 43 años que es alimentado por via iintravenosa durante
varias semanas. Producto de este tipo de alimentación se encuentra en la endoscopia
atrofia de la mucosa antral. la causa mas probable de esta alteración es debido a los bajos
niveles séricos de que hormona:

Rpta. GASTRINA

66. Con respecto al control autonómico en el tracto gastrointestinal y en relación con su fisiología
¿Cuál es la función del sistema nervioso parasimpático y el tracto gastrointestinal?

Estimulan la contracción muscular y estimulan la secreción de sustancias a nivel de la mucosa

Intersticio ubicado entre el estroma del espacio portal y los hepatocitos, y por donde migran las
células cancerígenas que hacen diseminación linfática es el:

Espacio del Mall

La presencia de grandes cantidades de TGF - Beta estimula a las ___________ y se deposita


colágeno, formándose la cirrosis

Células de Ito

Paciente de 64 años con ICC al que se le va a realizar cirugía cardiovascular. Al calcular el volumen
sanguíneo total, se debe considerar que el hígado puede contener un volumen de sangre de
____________ mL en un adulto sano, en este paciente ese volumen puede llegar a ser de
_____________ mL

450 - 1000

El área del lobulillo que se afecta más en caso de hipoxia es la zona:

Paciente con carcinoma de vesícula biliar. La metástasis por continuidad afectará al lóbulo:

Cuadrado

El ácido acetilsalicílico actúa en la membrana:

Basolateral de la célula parietal


Al realizar una esplenectomía, se tiene que resecar la arteria esplénica, lo cual no es problema para
el estómago por que la arteria gastroomental izquierda se anastomosa con la:

Gastroomental derecha

La arteria esplénica proviene de la aorta y la vena esplénica desemboca en la vena:

Porta

Paciente mujer 21 con bulimia, que luego de un episodio de vómitos presenta hematemesis y al
examen físico que se encuentra crépitos subcutáneos cervicales. El diagnóstico más probable es:

Sd. Boerhaave

Paciente varón de 60 años, con antecedentes de promiscuidad sexual, tabaquismo y alcoholismo,


acude a consulta por presentar disfagia progresiva, odinofagia y al examen se observa tumor por
parte posterior de la lengua. La mejor posibilidad diagnóstica es:

Carcinoma escamoso

Niño de 5 años con historia de tres días de evolución caracterizado por fiebre, malestar general,
odinofagia, anorexia, e irritabilidad. Al examen de observa lesiones ulcerativas de 4mm de diámetro
en mucosa yugal, con borde blanquecino y eritema periférico. El diagnóstico más probable es:

Aftas orales

Es considerada una lesión preneoplásica

Leucoplasia

El esófago de Barrett se considera una lesión preneoplásica que se caracteriza por la presencia en
esófago de:

Metaplasia intestinal

Paciente mujer de 23 años gestante con lesión proliferativa en mucosa oral producida por
proliferación reactiva de vasos sanguíneos. Marque la mejor respuesta

Epulis
La glándula parótida tiene principalmente acinos de tipo:

Seroso

La reabsorción de sodio y secreción de potasio es estimulada por:

Aldosterona

Paciente con cirrosis hepática que tiene hipertensión portal con várices esofágica, y actualmente
presenta varices en estómago distal. Estas várices están relacionadas a aumento en la presión de las
venas:

Gástrica derecha

La glándula parótida está inervada por el par craneal:

IX

Paciente con cirrosis hepática que tiene hipertensión portal con varices en esofago distal . Estas
varices están relacionadas a aumento en la presión de las venas gástricas

Inferior

La información eferente que sale de los núcleos salivales superior e inferior a través de los pares VII y
IX hacia las glándulas salivales llevan información tipo

Parasimpático

Un niño de 4 años ingresa en el hospital con vómitos graves . En el estudio se encuentra que el niño
tiene un páncreas anular ¿ Cuál de las siguientes hormonas gastrointestinales se encontrará a
niveles elevados en sangre con mayor probabilidad a raíz de esta patología

Gastrina

Durante una colecistectomía laparoscópica en un hombre de 61 años ¿ Cuál de las siguientes


arterias debe pinzar para extirpar la vesícula biliar con seguridad?

Cistica
Un hombre de 34 años se somete a una apendicectomía de urgencia . Después de realizar la
apendicectomía satisfactoriamente , el paciente se somete a una laparoscopia exploratoria¿Cual de
las siguientes características anatómicas es más útil para distinguir entre yeyuno e íleon?

El yeyuno tiene menos grasa mesentérica que el íleon

Una mujer de 45 años ingresa en el hospital con síntomas de obstrucción intestinal superior .En la TC
se encuentra que la tercera porción (transversa) del duodeno está comprimida por un gran vaso
¿Cuál de los siguientes vasos causara muy probablemente la obstrucción ?

Arteria mesentérica superior

Durante una colecistectomía laparoscópica programada en una mujer de 47 años , el residente


pinchó accidentalmente el ligamento hepatoduodenal en vez de la arteria cística ¿Cual de los
siguientes vasos estaría muy probablemente ocluido en esta lesión iatrogénica?

Arteria hepática izquierda

Un hombre de 54 años ingresa en urgencias con intenso dolor abdominal superior . La gastroscopia
revela un tumor en el antro del estómago . Se pide una TC para evaluar el drenaje linfático del
estómago¿Cuál de los siguientes nódulos linfáticos estará muy probablemente afectado en una
neoplasia maligna del estómago

Mesentérico inferior

Una mujer obesa de 45 años con fiebre alta acude a la consulta con náuseas y dolor agudo e
intermitente en el cuadrante superior derecho del abdomen de 2 días de duración - Tiene una historia
de ictericia de 24 horas. Tiene antecedentes de litiasis biliar. Bilirrubina total del 10 mg/dL. Lipasa de
5 mg/mL. ¿Cuál de las siguientes estructuras está muy probablemente obstruida por un cálculo biliar?

Ampolla de Vater

Una mujer de 45 años ingresa en urgencias con dolor abdominal intenso . La TC y RM revelan un
tumor de la cabeza del páncreas que afecta el proceso unciforme ¿Cual de los siguientes vasos es
más probable que suministre irrigación a parte de la zona afectada?

Arteria cólica media


Un hombre de 70 años ingresa en urgencias con diarrea intensa . La arteriografía revela un bloqueo
del 90% en el origen aórtico de la arteria mesentérica inferior ¿Cuál de las siguientes arterias
proporciona muy probablemente irrigación colateral al colon descendente?

Arteria sigmoidea

Al disminuir el pH duodenal por el HCL gástrico , se libera principalmente una hormona cuya célula
diana es

Célula ductal de Wirsung

Una de las siguientes sustancias reguladoras , puede actuar de forma paracrina y como hormona

Somatostatina

Al ingerir grandes cantidades de dulces , con la subsecuente estimulación de incretinas , usted


esperaría que el apetito

Disminuya por insulina

Paciente obeso con Covid-19 es intubado por interno inexperto, quien al solicitar que bombeen aire
dentro del tubo endotraqueal, nota que el epigastrio se distiende. Al sospechar que ha introducido el
tubo en el estómago, también es cierto que:

Disminuye el pH gástrico

La hormona que tiene un efecto sinérgico con la secretina para optimizar el pH duodenal y la
digestión, es:

CCK

El ecografista sabe que para poder visualizar el nacimiento de la arteria mesentérica superior , debe
colocar el transductor sobre la piel de la siguiente región abdominal

Epigastrio

En un paciente con hiperestimulación simpática se espera que las ondas lentas tengan un ritmo

Mayor en estómago que en el duodeno

Mayor en estómago que en el íleon terminal


Menor en íleon terminal que en el duodeno

Mayor en el íleon que en duodeno

Se considera que el gusto puede viajar a través del nervio

Glosofaríngeo

La rotación en sentido longitudinal del estómago en el desarrollo embriológico condiciona que el


nervio vago derecho quede a nivel

Posterior

Con respecto al control autonómico en el tracto gastrointestinal y en relación con su fisiología. ¿Cuál
es la función del sistema nervioso parasimpático en el tracto gastrointestinal?

Estimulan la contracción muscular y estimulan la secreción de sustancias a nivel de la mucosa.

Un estudiante que está preocupado por su examen parcial, no ha desayunado ni almorzado; cuando
al fin ingiere alimentos, este le provoca el aumento de los movimientos musculares del tracto
gastrointestinal y la sensación de defecar.?¿Qué reflejo se ha activado?

gastro - cólico

¿De que par craneal es rama el nervio palatino mayor?

Trigémino

¿En cuál de las fases de deglución la epiglotis separa la vía respiratoria de la digestiva?

Faríngea

Los péptidos intestinales se pueden clasificar como sustancias endocrinas, neurocrinas y paracrinas,
dentro de las paracrinas se encuentran la somatostatina e histamina. Marque la respuesta correcta

La histamina es sintetizada por células de tipo paracrino de las glándulas gástricas

Paciente varón de 27 años es llevado por bomberos a emergencia luego de ser asaltado y, tras
resistirse, es cortado con el pico de una botella a nivel abdominal. Al examen físico usted observa
que a través de la herida se puede observar la protrusión de las asas intestinales. En relación con las
capas de la pared abdominal, marque la alternativa correcta.
El músculo recto del abdomen tiene como funciones comprimir el contenido del abdomen, tensar la
pared abdominal y flexionar la columna

Los músculos del tracto gastrointestinal de los segmentos propulsivo y receptor del bolo alimenticio,
responden de forma diferente al movimiento de este bolo a través del intestino. ¿Cuál de las
siguientes afirmaciones describe correctamente la actividad del segmento propulsivo?

Tanto el músculo circular como el longitudinal están relajados

El músculo longitudinal está relajado y el circular esta contraído

Tanto el músculo circular como el longitudinal están contraídos

El músculo longitudinal está contraído y el circular está relajado

El nervio vago inerva al músculo

Estriado del esófago

¿Cuál de las siguientes es una característica de los ganglios mioentéricos del sistema nervioso
entérico?

Contiene mayor número de neuronas que el plexo submucoso

Es también conocido como el plexo de Meissner

Contiene sólo neuronas motoras excitatorias del músculo liso

Contiene neuronas sensitivas que activan a los músculos circular y longitudinal del tracto intestinal

El divertículo faringoesofágico, hipofaríngeo de zenker, es una lesión muy particular que se localiza
en la cara póstero lateral de la Unión de la faringe con el esófago, como una herniación de la mucosa
esofágica a través de las fibras oblicuas del músculo.

Constrictor inferior de la faringe

La razón por la que el potencial de acción viaja rápidamente en sentido longitudinal por el músculo
liso gastrointestinal es la presencia de uniones en hendidura,

Varicosidades

Respecto a los péptidos gastrointestinales, marque lo correcto.


las sustancias paracrinas pueden viajar a través de vasos sanguíneos

Al ingerir rápidamente un litro de agua, usted esperaría que la gastrina aumenta por efecto de:

ACh del sistema mientérico

Respecto a la anatomia del estomago,marque lo correcto:

El fondo gástrico forma la curvatura mayor

(Pág 19 → 21)

Los nervios esplácnicos lumbares (L1-L2) llevan información de tipo:

Simpática

Paciente de 32 años con herida por arma de fuego y shock hipovolémico. El intestino delgado no se
ha infartado aún a pesar de la hipoxia gracias a la liberación de:

Adenosina

Al realizarse un piercing en el ombligo,la sensación de dolor se transmite por:

T10

El nivel en el que se encuentra el píloro y el páncreas se puede determinar usando el:

Píloro transpilórico

Permite la suspensión e irrigación de los órganos peritoneales:

Mesenterio

El dolor asociado a apendicitis clásicamente se ubica en:

Fosa iliaca derecha

El ligamento inguinales formado por la aponeurosis del:

Oblicuo externo
Los nueve cuadrantes del abdomen se delinean usando el plano subcostal,las líneas
medioclaviculares y:

Plano intertubercular

Es un órgano peritoneal:

Hígado

La línea alba se encuentra:

Entre los rectos abdominales

Es un órgano retroperitoneal:

Páncreas

Enfermedad asociada con un error en el desarrollo de las células de Cajal

Enfermedad de Hirschsprung

El divertículo de Meckel es un rezago de:

Conducto vitelino

La fístula rectoperitoneal es causada por una falta en el desarrollo de:

Tabique urorrectal

95) Al disminuir el pH duodenal por el HCl gástrico, se libera principalmente una hormona cuya célula
diana es:

Célula ductal del Wirsung


Al introducir una solución azucarada directamente al estómago mediante una gastrostomía
(comunicación entre la piel abdominal y el estómago), la sustancia que provocará que aumenten los
niveles séricos de insulina es:

Péptido tipo glucagón 1 (GLP-1)

La rotación en sentido longitudinal del estómago en el desarrollo embriológico condiciona que el


nervio vago derecho quede a nivel:

Posterior

Una de las siguientes sustancias no comparte con las otras la misma acción sobre la producción de
ácido gástrico:

Colecistoquinina

En un paciente con gastroparesia (motilidad lenta del estómago), que presenta distensión abdominal
después de comer, usted le recomendaría que evite el consumo de lípidos y aminoácidos para
disminuir la acción de:

CCK

En un paciente con shock distributivo, usted decide iniciar noradrenalina por un catéter CVC,
consiguiendo aumentar la presión arterial. ¿qué efecto sobre la motilidad intestinal esperaría
encontrar?

El potencial de reposo de las fibras musculares se hace más negativo

El ecografista sabe que para poder visualizar el nacimiento de la arteria mesentérica superior, debe
colocar el transductor sobre la piel de la siguiente región abdominal:

Epigastrio

105) La glándula submandibular recibe inervación traída por el nervio

Cuerda del tímpano

A mayor flujo de saliva, disminuye la concentración de:

Potasio
107) En términos de mg/mL, el principal componente de la saliva es _____ seguido de ____:

Proteínas - Potasio

108) La reabsorción de Sodio y Cloro en las glándulas salivales se da principalmente en el:

Conducto estriado

109) La información eferente que sale de los núcleos salivales superior e inferior a través de los pares
VII y IX hacia las glándulas salivales llevan información de tipo:

Parasimpático

La glándula submaxilar le hace gancho al:

Músculo milohioideo

Los conductos salivales son ___ al agua, esa es una de las razones por las cuales la saliva es
siempre ___.

Impermeables - hipotónica

Los nervios esplácnicos pélvicos (S2-S4) llevan información de tipo:

Parasimpática

113) Presenta movimientos en masa:

Colon

114) La digestión de los lípidos se inicia en:

Intestino delgado

115) La digestión de los carbohidratos se inicia en:

Cavidad oral
117) El estómago recibe información simpática proveniente del:

Ganglio celiaco

118) En el intestino delgado se absorbe los carbohidratos en forma de:

Fructosa

119) Es rama de la arteria mesentérica superior:

A. cólica media

120) El “dolor de estómago” asociado a gastritis se suele ubicar en:

Epigastrio

121) El azúcar de mesa sacarosa es digerido a dos monosacáridos que comparten el transportador:

GLUT 2

122) Durante la defecación se requiere:

Señales inhibitorias en el nervio pudendo

123) En un paciente con fístulas intestinales y fisuras anales, con antecedente de enfermedad
inflamatoria intestinal, ud sospecharia en:

Enfermedad de Crohn

En los pacientes con Colecistitis Aguda no operable, una opción es la colocación de una sonda por el
cístico, procedimiento en el que se ingresa con dificultad debido a la estrechez del cístico y a la
presencia de:

Válvula de Herring
Paciente de 51 años con antecedente de enfermedad diverticular acude a emergencia por sangrado
profuso y dolor en hipocondrio izquierdo. ¿Cuál es el origen más probable de la sangre que pierde el
paciente ?

Mesentérica inferior

Paciente de 42 años con dolor abdominal intenso y hematemesis. En la endoscopia se observa una
úlcera duodenal posterior perforada con hemorragia intraabdominal. ¿Cuál de las siguientes arterias
estará comprometida?

Pancreaticoduodenal posterosuperior

Para que los triglicéridos sean absorbidos deben ser metabolizados a

Monoglicéridos y Ácidos grasos

El vibrio cholerae produce diarrea porque:

Aumenta la producción de AMPc en los enterocitos

La motilidad intestinal es estimulada por

Colecistoquinina y gastrina

La diarrea por deficiencia de lactasa es de tipo

osmótica

La metoclopramida estimula el vaciamiento gástrico aumentando la fuerza de contracción de las


paredes gástricas esto puede conseguirlo mediante la estimulación indirecta de las neuronas
liberadoras de

Acetilcolina

Los vértices de un acino hepático están constituidos por

Dos espacios porta y dos venas centrolobulillares


Al ingerir un pedazo de mantequilla, cuál de los siguientes tiene un efecto directo en la reducción del
vaciado gástrico:

Colecistoquinina

Tras la vagotomía (resercion del vago) por enfermedad úlcera péptica en un paciente UD. esperaría
encontrar:

Aumento del pH gástrico

Se realizó un experimento en el cual se inyectó tinta china en el peritoneo de ratas de laboratorio. Al


realizarse una biopsia hepatica de dichos animales, se encontro que el tinte negro fue fagocitado por:

Células de kupffer

En un paciente con pH gástrico muy bajo, es posible que la siguiente sustancia se secrete en menor
cantidad:

Gastrina

La fase cefálica de la secreción gástrica responde por cerca del 30% de la respuesta ácida a un
reflejo con la _____________ se elimina la fase cefálica de la secreción gástrica

Vaguectomía

El esofago de barret se caracteriza por presentar _________ en el esofago

Metaplasia intestinal

La presencia de orina que sale por el ombligo de un recien nacido cada vez que llora, es posible que
deba a un defecto en el desarrollo de:

Seno urogenital

El conducto biliar deriva del:

Endodermo

Cuál de los siguientes órganos son intraperitoneales:


Estómago, Vesícula biliar, Ileón, Hígado

Los vasos mesentéricos superiores se hallan a nivel de:

Cuello del páncreas

La colecistoquinina (CCK) inhibe:

El vaciamiento gástrico

Un niño de 2 años es llevado a la consulta por diarrea persistente, edema de las extremidades y falta
de crecimiento en relación a su edad. Los análisis de sangre revelan que tiene concentración
plasmática baja de proteínas (hipoproteinemia). Como parte del estudio se coloca Colecistoquinina
(CCK) endovenosa y se recoge muestras del líquido duodenal por endoscopia; el resultado del líquido
confirma incapacidad para hidrolizar proteínas a un pH neutro, esta situación mejora al añadir una
pequeña cantidad de tripsina. El paciente probablemente esté sufriendo la falta congénita de ……….

Enterocinasa

Paciente mujer de 35 años acude a consulta por sensación de sequedad y lesiones en la cavidad
oral. Al examen se observa atrofia de la mucosa, fisura y úlceras; nota además sequedad e irritación
de la córnea y aumento del tamaño de las glándulas parótidas. Su diagnostico mas probable es
artritis reumatoide; el hallazgo más probable en una biopsia de glándula parótida es ….…

Gran infiltración de linfocitos y células plasmáticas

Un hombre de 42 años de edad se presenta al médico con una historia clínica de 1 año de evolución,
caracterizado por dolor abdominal bajo y diarreas con crisis sanguinolentas. Manifiesta además
pérdida de peso de 8kg durante este periodo. La colonoscopia revela lesión difusa en el colon con
afectación del recto. La biopsia de estas lesiones revela adelgazamiento de la pared, inflamación y
ulceración de la mucosa y submucosa. El diagnóstico más probable en este caso es:

Colitis ulcerativa

Dos estudiantes deciden tomar un receso para comer una hamburguesa a la hora del almuerzo.
Antes de llegar a la cafetería, impulsos nerviosos provenientes del complejo vagal dorsal iniciarán la
secreción de ácido gástrico por la liberación de ___________ desde el sistema nervioso entérico.

GRP (péptido liberador de gastrina)


Un niño de cuatro años de edad es llevado a la consulta por cuadros diarreicos frecuentes
caracterizados por heces pálidas, voluminosas y fétidas; al examen físico presenta bajo peso y talla
para la edad. Se mide la concentración de cloruro de en el sudor y se encuentra que sus valores son
muy elevados. La alteración más importante a nivel de células ductales del páncreas tiene relación
directa con la conductancia de …..

Cloro
En el club del adulto mayor, a un grupo de ancianos de 65 años se les da a elegir entre un taller de
videojuegos y uno de lectura. La mayoría de ellos optó por el taller de lectura. ¿Cuál sería una
explicación para esta conducta?

La atención sostenida permanece relativamente estable a esa edad

Pablo es invitado a dar el discurso de bienvenida a los nuevos residentes del hospital. Muestra
habilidad en el discurso, planifica las ideas que quiere decir, mantiene el tema y objetivo de la
conversación. ¿Qué estructura cerebral estaría involucrada para demostrar esas características?

CPF dorsolateral

Para poder diagnosticar a un paciente de discapacidad intelectual se requiere:

Que las deficiencias adaptativas limiten el funcionamiento en una o varias actividades cotidianas

En relación a la dopamina podemos afirmar:

Esta presente dentro de la neurona presináptica

La mielinización neuronal para el desarrollo visual, motor, social y cognitivo se da en la dirección:

Cefalocaudal
En un paciente con hiperestimulación simpática se espera que las ondas lentas tengan un ritmo:

Menor en íleon terminal que en el duodeno

Al ingerir grandes cantidades de dulces, con la subsecuente estimulación de incretinas, usted


esperaría que el apetito ______________, debido a __________________

disminuya insulina

Es un ligamento derivado del mesenterio dorsal:

Gastrocólico

Una de las siguientes sustancias no comparte con las otras la misma acción sobre la producción de
ácido gástrico:

Colecistoquinina

Usted encuentra músculo estriado en el siguiente segmento:

Esfínter anal externo

Todos los músculos motores de la lengua están inervados por el XII par, excepto:

palatogloso

En un paciente con gastroparesia (motilidad lenta del estómago), que presenta distensión abdominal
después de comer, usted le recomendaría que evite el consumo de lípidos y aminoácidos para
disminuir la acción de:

CCK

En un paciente con diarrea por hipermotilidad, usted sospecharía en el posible aumento de las
siguientes sustancias, excepto:

Péptido intestinal vasoactivo

Marque lo correcto respecto a la siguiente imagen:

Se produjo por falta de fusión de los ductos dorsal y ventral

Respecto a los péptidos gastrointestinales, marque lo correcto.

las sustancias paracrinas pueden viajar a través de vasos sanguíneos


Se evalúa los valores séricos de las siguientes sustancias a un paciente con enfermedad hepática
terminal; en este paciente se espera encontrar la combinación de la letra …..

Glucosa

Amoniaco

Albúmina

a.

Aumentada

Disminuida

Disminuida

b.

Disminuida

Aumentada

Aumentada

c.

Aumentada

Aumentada

Aumentada

d.

Disminuida

Aumentada

Disminuida

Una mujer de 35 años de edad HIV positiva, se presenta al médico con dolor abdominal en cuadrante
superior derecho e ictericia. La paciente refiere haber tenido múltiples episodios de ictericia durante
los últimos 10 años. Los exámenes para determinar hepatitis viral, dieron positivos para Hepatitis B,
siendo catalogado el caso como hepatitis crónica con alteración funcional. En un examen de sangre
¿Cuál de los siguientes parámetros está disminuido?

Albúmina
En el reflejo peristáltico del intestino delgado ¿Cuál de los siguientes eventos sucede en la porción
caudal del bolo alimenticio?

Acción del péptido inhibidor vasoactivo (VIP) en el músculo circular

Un varón de 58 años de edad con enfermedad de Crohn severo fue sometido a una resección ileal.
Después de la cirugía esta paciente padecerá de esteatorrea esto se explica porque ……

Hay malabsorción de ácidos biliares

En un experimento se inserta un balón en el estómago de un voluntario, se infla poco a poco mientras


que se vigilan las presiones intraluminales. Aunque el volumen del balón aumenta
considerablemente, las presiones permanecen constantes. Esta relación volumen-presión se explica
por la liberación local de ….

Óxido nítrico y péptido inhibidor vasoactivo

¿Cuál de las siguientes alternativas es una característica de la secreción exocrina del páncreas?

Su mayor estímulo se da en la fase intestinal

Las estructuras en el hígado que permite que los productos metabólicos unidos a proteínas tengan
acceso a las membranas basolaterales de los hepatocitos, son ….

Las fenestras sinusoidales

La composición de la bilis es modificada conforme fluye por los conductillos biliares. Durante este
tránsito se espera que aumente la concentración de …..

Ig A

Se mide experimentalmente el contenido gástrico de dos personas. La persona “A” tiene alto
contenido de grasa y la persona “B” tiene un contenido isotónico. ¿Cual de las siguientes es correcta
respecto al vaciamiento gástrico?

Hay ralentización del vaciado gástrico sólo en “A”


El examen endoscópico de un paciente con hipertensión portal grave revela venas tortuosas que
sobresalen hacia la luz del esofago. El paciente recibe tratamiento quirúrgico mediante la colocación
de una derivación que conecta la vena cava. Después de la operación el riesgo de encefalopatía
…….. y el sangrado de varices ……

Aumentará/Disminuirá

Un bolo alimenticio grande y poco masticado se atasca en el esofago, esto ocasiona una sensacion
de dolor que es transmitida por los nervios:

Esplácnico

El peristaltismo del intestino delgado se puede intensificar debido a:

Irritación de la mucosa

Un paciente es diagnosticado con un tumor neuroendocrino productor de somatostatina, esto


provocará en el sistema digestivo:

Diarrea

Los diferentes segmentos del tubo digestivo son susceptibles de reflejos y movimientos según su
contenido.Si colocoramos mediante una sonda un bolo alimenticio directamente en el tercio medio del
esofago:

Se producirá ondas secundarias

En el digestivo la liberación hormonal se presenta ante diversos factores o estímulos. La hormona


_________ es estimulada por la presencia de alimentos en el bulbo duodenal a predominio de ácidos
grasos y triglicéridos, por estimulación vagal y por la hormona secretina.

Colecistoquinina (CCK)

El divertículo de Meckel es una anomalía congénita que ocurre por la persistencia del conducto
vitelino y da origen a una estructura sacular, el cual se encuentra en el:

Borde antimesentérico
La fase oclusal de la masticación se realiza con la contracción de los músculos inervados por el
nervio craneal:

En un paciente de 43 años con tumor carcinoide de páncreas productor de gastrina (Sindrome de


Zollinger-Ellison) se puede encontrar una potenciación del reflejo:

Gastrocolico

Durante la fase faríngea de la deglución ocurre el siguiente mecanismo:

La onda peristáltica lleva el alimento hacia el esofago

En los carcinomas (neoplasia benigna) es frecuente que ocurran la metástasis a través de los vasos
venosos. En el caso de un carcinoma del tercio superior del esofago, ubicado en la cara lateral
izquierda, es más probable que la metástasis viaje por la vena:

Hemiácigos accesoria

168) Los movimientos en masa son iniciados por el reflejo:

Duodenocolico

Una mujer de 65 años HIV positiva se presenta con dolor abdominal en el cuadrante superior derecho
e ictericia. La paciente afirma haber teñido múltiples episodios de ictericia durante los últimos 10
años. Los exámenes para poder detectar hepatitis viral, dieron positivos para Hepatitis B, siendo
catalogada como hepatitis crónica con alteración funcional. En un examen de sangre ¿cuál de los
siguientes parámetros está disminuido?
Albúmina

En el reflejo peristáltico del intestino delgado¿cuál de los siguientes eventos sucede en la porción
caudal del bolo alimenticio?

Acción del péptido inhibidor vasoactivo(VIP) en el músculo circular

Un varón de 58 años de edad con enfermedad de Crohn Severo fue sometido a una resección ilegal.
Después de la cirugía este paciente padecerá de esteatorrea, esto se explica porque ….

Hay mala absorción de los ácidos biliares

En un experimento se inserta un balón en el estómago de un voluntario, se infla poco a poco mientras


se vigilan las presiones intraluminales. Aunque el volumen del balón aumenta considerablemente, las
presiones permanecen constantes. Esta relación volumen presión se explica por la liberación local de
….

Óxido nítrico y péptido inhibidor vasoactivo

¿Cuál de las siguientes alternativas es una característica de la secreción exocrina del páncreas?

Su mayor estímulo se da en la fase intestinal

Las estructuras en el hígado que permiten que los productos metabólicos unidos a proteínas tengan
acceso a membranas basolaterales de los hepatocitos son ….

Las de fenestras sinusoidales

La composición de la bilis es modificada conforme fluye por los conductillos biliares. Durante este
tránsito se espera que aumente la concentración de ….

IgA

Se mide experimentalmente el contenido gástrico de dos personas. La persona A tiene alto contenido
de grasas y la persona B tiene un contenido isotónico ¿Cuál de las siguientes es correcta respecto al
vaciamiento gástrico?

Hay ralentización del vaciado gástrico sólo en A


El examen endoscópico de un paciente con hipertensión portal grave revela venas tortuosas que
sobresalen hacia la luz del estómago. El paciente recibe tratamiento quirúrgico mediante la
colocación de una derivación que conecta la vena porta a la vena cava. Después de la operación el
riesgo de encefalopatía …. y el riesgo de sangrado de varices ….

Aumentará / disminuirá

Un paciente varón de 18 años de edad acude al médico para sus exámenes de rutina. Sus resultados
de laboratorio muestran un valor de bilirrubina sérica de 4mg/dl y una bilirrubina directa de 0.3 mg/dl.
Las pruebas de función hepática son normales. La alteración que explica mejor este caso es por la
deficiencia de….

Glucuronil transferasa

Un hombre de 57 años de edad es llevado a urgencias con hematemesis masiva rojo brillante, a su
llegada se halla inconsciente con PA: 80/40 mm Hg y FC:124 lat/min. Luce ictérico con presencia de
arañas vasculares en el tórax anterior y extremidades, abdomen distendido con signo de oleada
positiva. Se encuentra esplenomegalia y pérdida de la masa muscular en extremidades. La
anastomosis vascular responsable del sangrado en este paciente es ….

Vena gastrica izquierda y vena acigos

Un estudiante de medicina está comiendo un plato de comida a base de champiñones, espárragos y


salsa de soya. El estímulo del sabor umami contenido en todos estos alimentos viaja a través del
nervio….

Cuerda del tímpano

Una paciente de 30 años de edad es sometida a una cirugía de oído medio derecho por un problema
de otoesclerosis. Luego de la cirugía refiere alteración sensitiva de la lengua. Al evaluar el caso usted
esperaría encontrar ….

Sensación del dolor, tacto y temperatura conservadas

En una paciente de 45 años de edad con colestasis biliar, se encuentra una elevación de los niveles
sanguíneos de fosfatasa alcalina hasta 3 veces la cifra normal ¿Cuál de las siguientes alternativas
estará también elevada como evidencia del daño biliar?

Gamma glutamil transpeptidasas


Experimentalmente se incrementa la velocidad de la secreción salival con una sustancia, en el
análisis de la composición de esta saliva obtenida se espera encontrar….

Aumento de la concentración de bicarbonato que supera la concentración plasmática

Lactante de 3 meses de vida es atendido por presentar diarrea, se administra una solución glucosa y
electrolitos por vía oral. La proteína de membrana apical que explica la capacidad de esta solución
para proporcionar aporte de glucosa e hidratación es ….

SGLT 1

Paciente ha sufrido herida de bala en el abdomen, se le tenido que extirpar el segmento medio y
distal del íleon. En este caso la síntesis hepática de sales biliares estará ….

Incrementada por estímulo de la enzima colesterol 7 alfa hidroxilasa

Un varón de 75 años ingresa al consultorio por presentar ictericia marcada de piel y escleras. El
estudio del paciente mostró que presentaba un tumor que obstruía la totalidad del conducto hepático
común. ¿Cual de los siguientes conductos se encontraría dilatado en este paciente?

De Hering

Correlacione las dos columnas y marque la respuesta correcta:

Enfermedad Hirschsprung Aganglionosis congénita

Diarrea osmótica. Intolerancia a la lactosa

Diarrea secretora. Canales de Cl- en las células de la cripta

Diarrea exudativa Heces con moco y sangre

La fase cefálica de la secreción gástrica responde por cerca del 39% de la respuesta ácida a un
reflejo. Con la ____ se elimina la fase cefálica de la secreción gástrica

Vaguectomía

El esofago de Barret se caracteriza por presentar___ en el esofago

Metaplasia intestinal
En la regulación del apetito y la saciedad, la estimulación experimental cronica del núcleo ventro
medial del hipotálamo producirá:

Afagia

Paciente mujer de 25 años acude por dolor en fosa iliaca derecha que empeora al toser o
caminar,asociada a náuseas y vómitos por lo cual acude a emergencia.Dos días después de
realizarle una apendicectomia, la paciente desarrolla fiebre alta (39° C), esta hipotensa y presenta
dolor abdominal.La laparotomia exploratoria muestra un gran volumen de sangre en la cavidad
peritoneal por lesion de un vaso producida durante la apendicectomía.¿Cual de las siguientes
arterias debe ligarse para detener la hemorragia?

Ileocolica

¿Cuál de los siguientes es una causa de ictericia con bilirrubina conjugada aumentada?

Obstrucción del colédoco

Dos días después de una apendicectomía en un hombre de 45 años, ha desarrollado fiebre alta (39),
está hipotenso y presenta dolor abdominal. La laparotomía exploratoria muestra un gran volumen de
sangre en la cavidad peritoneal por lesion de un vaso producida durante la apendectomía. ¿Cuál de
los siguientes vasos debe ligarse para detener la hemorragia?

Arteria ileocólica

Paciente de 78 años, con diabetes mellitus tipo ll y fumador, que acude a consulta porque desde hace
dos semanas tiene un dolor intenso en flanco derecho y mesogastrio , intenso que aparece a los 30
minutos de haber comido, y desaparece dos a tres horas después. En estos pacientes, es muy
probable que la circulación deficitaria sea parcialmente asumida por la:

Arteria cólica media

Un hombre de 70 años ingresa en urgencias con diarrea intensa. La arteriografía revela un blloqueo
del 90% en el origen aórtico de la arteria mesentérica inferior. ¿Cuál de las siguientes arterias
proporciona muy probablemente irrigación colateral al colon descendente?

Arteria cólica derecha

Un niño de 4 años ingresa en el hospital con vómitos graves. La exploración radiológica y la historias
clinica revelan que el niño tiene páncreas anular. ¿Cuál de las siguientes estructuras es la que se
encontrará dilatada con mayor probabilidad a raíz de esta patología?

Primera porción del duodeno


Un hombre de 55 años ingresó al hospital con dolor abdominal intenso. La gastroscopia y la TC
revelaron una úlcera perforada en la pared posterior del estómago. ¿Dónde se desarrolla
inicialmente con más probabilidad una peritonitis?

Bolsa omental (saco menor)

Un hombre de 44 años ingresa en urgencias con vómitos abundantes y deshidratación. Las imágenes
radiológicas demuestran que parte del intestino está comprimido entre la aorta abdominal y la arteria
mesentérica superior. ¿Cuál de las siguientes estructuras intestinales estará muy probablemente
comprimida?

Tercera porción del duodeno

¿Cuál de los siguientes nódulos linfáticos estará muy probablemente afectado en una neoplasia
maligna del recto?

Mesentérica inferior

Una mujer de 23 años ingresa con dolor abdominal, náuseas y vómitos. La historia Clínica muestra
que el dolor es agudo y ha sido constante durante 4 años. El dolor empezó en el epigastrio e irradió
bilateralmente alrededor del tórax hasta justo debajo de las cápsulas. Actualmente el dolor se localiza
en el hipocondrio derecho. L a TC revela cálculos calcificados en la vesícula biliar. ¿Cuál de los
siguientes nervios llevan las fibras aferentes del dolor referido?

Nervios esplácnicos torácicos mayores

Un hombre de 55 años ingresa en urgencias por severa perdida de peso en los 6 meses previos. El
examen radiológico pruebas aportan signos de un tumor que causa hipertensión portal. Los estudios
de laboratorio revelan que las deposiciones son grasas, tiene desnutrición e hipoxia hepática. ¿Cuál
de las siguientes localizaciones se encuentra muy probablemente afectada?

Segmento l

El triángulo de calot es importante reconocerlo porque sirve como reparo para encontrar la arteria
cística, y está formado por el conducto cístico, el conducto hepático derecho y :

Borde hepático
El kernícterus se produce en recién nacidos con valores mayores de 25m/dL en la bilirrubina:

Indirecta

Paciente con cirrosis hepática y presión de vena cava de 15mmHg. Lo más probable es que el
paciente presente:

Ascitis

Considerando un flujo plasmático renal de 180 ml/min, y una fracción de filtración del 20%, si la
concentración en sangre de la bilirrubina indirecta es de 0.6mg/dL, entonces es correcto esperar que
la carga filtrada de la bilirrubina indirecta es:

Menor de 1080 mg/dL

El volumen diario de bilis secretada al intestino es:

500-1000 ml

La excreción del amonio se da principalmente por:

Orina

Si en una persona normal, el flujo de la arteria hepática es de 700ml/min, entonces el flujo de la vena
porta debería ser aproximadamente:

2800mL/min

Paciente con Grigler-Najjar debida a mutación del gen UGT1A1, se presenta a consulta por ictericia,
usted asume que si le hiciera un análisis de sangre encontraría valores elevados de:

Bilirrubina indirecta

El amoniaco corporal se forma principalmente en:

Colon

La zona del lobulillo hepático que se afectaría más en una intoxicación con droga hepatotóxica es la
zona:
1

El acino hepático tiene en sus aristas:

Vena centrolobulillar

Paciente con esteatosis hepática no alcohólica, que en la biopsia se observa degeneración grasa de
los hepatocitos, lo cual se debe a depósitos de lípidos que principalmente contienen:

Triglicéridos

Al evaluar la orofaringe de un paciente, el médico le solicita que abra la boca, saque la lengua y diga
a . Al hacer esta maniobra, nota que el paladar se desvía hacia la derecha, lo cual le hace sospechar
que el paciente sufre de una lesión del nervio craneal:

X contralateral

Un bolo alimenticio grande y poco masticado se atasca en el esófago, esto ocasiona una sensación
de dolor que es transmitida por los nervios:

esplácnicos

Para realizar el movimiento mecánico de abrir la boca, primero se necesita:

fijar el hueso hioides

¿Cuál de las siguientes alternativas se define como la protrusión directa del contenido abdominal a la
cavidad amniótica por un defecto de la pared corporal?

Gastrosquisis

Un paciente requiere que se le coloque una sonda de alimentación directamente al estómago


(gastrostomía), el cirujano deberá hacer una incisión en la piel del abdomen ¿cuál de las siguientes
raíces nerviosas debe ser anestesiada para este procedimiento?

T8

El mecanismo de la defecación incluye la participación de diversas estructuras ¿Cuál de las


siguientes alternativas es correcta?
Puede ser mediado por un reflejo intrínseco

Cuando el contenido del estómago ingresa al duodeno, uno de los reflejos que inhiben el vaciamiento
gástrico es a través del:

sistema nervioso mientérico

Durante la masticación, gran parte del proceso masticatorio se debe a:

el reflejo masticatorio

Las glándulas salivales tienes conductos para la excreción de la saliva; las glándulas ____________
drenan en las carúnculas sublinguales.

sublinguales

En una persona si enfermedad se espera que el tránsito intestinal se vea disminuido cuando se
presenta el reflejo:

doloroso

El divertículo de Meckel es una anomalía congénita que ocurre por la persistencia del conducto
vitelino y da origen a una estructura sacular, el cual se encuentra en el:

borde antimesentérico

Si al intubar a un paciente, por error se ingresa el tubo endotraqueal en el esófago y se insufla el


manguito endotraqueal (globo TET), la dilatación de este manguito generará:

múltiples ondas secundarias

El orificio omental, o hiato de Winslow, se encuentra limitado por el ligamento:

hepatoduodenal

Paciente de 24 años acude a consulta externa por presentar una fístula oronasal (comunicación entre
la cavidad oral y la cavidad nasal). Está fístula es una consecuencia tardía de la lesión de un vaso
sanguíneo por el antecedente de haber sido operado de paladar hendido en los primeros años de
vida, aparentemente en una campaña gratuita de corrección de paladar fisurado. ¿Cuál de las
arterias palatinas podría haberse lesionado durante esa cirugía?

Mayor

Dentro de las anomalías congénitas se puede presentar un tejido pancreático accesorio ¿Cuál es la
ubicación más común de este tejido?

Estómago

Paciente con insuficiencia mitral moderada a severa, con aumento de volumen de la aurícula
izquierda; esta condición tendrá como consecuencia a nivel del sistema digestivo:

la disfagia a sólidos

El inicio de la fase faríngea de la deglución se debe a estímulos sensitivos que viajan por el nervio
craneal:

V (nervio maxilar)

El mesocolon transverso se origina en:

la pared posterior del abdomen

La contracción del músculo ………………………… permite la eliminación de gases (flatos) sin salida
de material fecal ; es el mismo músculo cuya relajación, sobretodo en cuclillas, permite el paso del
contenido fecal con menor esfuerzo durante la defecación

puborrectal

Paciente mujer de 54 años se presenta con náuseas, vómitos, estreñimiento, y es diagnosticada de


abdomen agudo quirúrgico; en la cirugía encuentran un vólvulo de ciego. Esta anomalía puede
explicarse por:

Falta de fusión del mesenterio

Paciente mujer de 23 años con faringitis aguda , toma para el dolor una tableta de paracetamol con
un poco de agua. Durante la deglución, se relaja su esfínter esofágico inferior y el fondo del
estómago, mientras el bolo está aún en el esófago. ¿Qué sustancia provocará con mayor
probabilidad la relajación del esfínter esofágico inferior y el fondo del estómago en esta mujer?

Óxido nítrico
Luego de tres horas dando exámenes, un alumno de medicina comienza a sentir hambre. Esta
situación es probable que sea mediada por la _____ que es sintetizada por el _____

grelina / estómago

Varón de 72 años, con antecedente de diabetes mellitus tipo 2, que presenta entropía diabética
caracterizada por estreñimiento. Este problema puede estar asociado a:

deficiencia de óxido nítrico

Varón de 54 años con diabetes mellitus tipo 2, es diagnosticado de gastroparesia debido a que
presenta sensación de llenura precoz al comer, y reflujo gastroesofágico. Esta alteración en la
relajación receptiva y en el vaciamiento gástrico lo más probable es que se deba a una alteración en:

el nervio vago

Varón de 67 años con tos y disminución de peso asociado a tabaquismo pesado, presenta
actualmente disfagia progresiva a alimentos sólidos. Se considera la presencia de un carcinoma de
bronquio izquierdo y por esta razón le realizan una endoscopia esofágica para descartar la posibilidad
de una compresión esofágica por el tumor. Se espera revisar el esófago en la _____ estrechez, que
está a nivel de la vértebra _____.

Tercera estrechez - T6

En un niño menor de dos años con divertículo intestinal, este divertículo tiene su origen en una falla
en la obliteración de:

Conducto vitelino

Mujer de 43 años sufre un grave accidente de tránsito y está hospitalizada en coma, es alimentada
por vía intravenosa durante varias semanas. Producto de este tipo de alimentación, se encuentra en
la endoscopia atrofia de la mucosa gastrointestinal. La causa más probable de esta atrofia son los
bajos niveles séricos de la hormona:

Gastrina

Una mujer de 30 años llega al consultorio porque se queja de dificultades para deglutir, la cual se
agravan cada vez más. Se realiza un estudio manométrico para examinar la generación de presión a
lo largo del esófago. Esta prueba revela que las contracciones como respuesta a la deglución están
mal sincronizadas y que la presión en el esfínter esofágico inferior permanece elevada. El diagnóstico
más probable es _____ producido por niveles bajos de _____:
acalasia / óxido nítrico

Paciente de 2 años, llega a emergencia por haber ingerido una moneda con la que estaba jugando. El
lugar más probable donde puede haberse quedado suspendido este objeto es a nivel del
estrechamiento producido a nivel del:

músculo cricofaríngeo

En una apendicectomía, al realizar la incisión de McBurney en la fosa iliaca derecha, es necesario


cortar los siguientes músculos, de afuera hacia adentro:

Oblicuo externo - Oblicuo interno - Transverso

Un varón de 90 años que se encuentra postrado en cama, es referido del asilo para endoscopia por
dificultad para deglutir luego de tomar un medicamento para aliviar el dolor de la noche anterior. La
endoscopia revela que la píldora se alojó en el esófago y causó una reacción inflamatoria. Lo más
probable es que esto haya sido por la producción de múltiples ondas:

secundarias

Mujer de 23 años es diagnosticada de bulimia, al examen físico se observa ulceraciones en el


segundo y tercer dedo de la mano derecha. Esto se puede deber al uso continuo de estos dedos para
inducir el vómito, mediante la estimulación del par craneal:

IX

Varón de 52 años se presenta por diarrea persistente de seis semanas de duración. En la


colonoscopia se observa un pólipo a nivel del íleon distal. El patólogo informa que se trata de un
tumor neuroendocrino, probablemente originado por las células enterocromafines del intestino. La
sustancia que más probablemente esté produciendo este tumor es:

Serotonina

La fase oclusal de la masticación se realiza con la contracción de los músculos:

masetero y temporal

Al tomar su café en Starbucks, un estudiante de medicina sufre una quemadura de primer grado en el
tercio anterior de la superficie dorsal de la lengua. La información de dolor es transmitida por el
nervio:

Lingual

Paciente es evaluado por faringitis aguda en consultorio externo. El médico de familia le solicita que
abra la boca y saque la lengua Para realizar la acción de sacar la lengua, es necesario que se
contraiga el músculo:

geniogloso
Paciente con síndrome de Sjogren, presenta “boca seca” (disminución de la producción de saliva) y
caries dental, asociada a la pérdida de la función de tampón de la saliva. Esta desmineralización del
diente puede comprometer a las prolongaciones citoplasmáticas ubicadas en los tubos huecos de la
estructura señalada con la letra:

SHADICK PARCIAL

1. Cuando el istmo de las fauces se cierra, se evita que el alimento pase hacia la orofaringe y
permite respirar mientras se mastica. Este cierre se debe a la contracción y aproximación de
los músculos:
- Palatofaríngeos
- Palatoglosos
- Estiloglosos
- Estilofaríngeos
2. Las siguientes alternativas son ciertas sobre la actividad eléctrica del músculo
gastrointestinal, EXCEPTO:
- La despolarización lenta se debe principalmente al ingreso de Na+.
- Las ondas lentas son más frecuentes en el duodeno.
- Si el potencial de membrana es más positivo, habrá mayor frecuencia de espigas.
- Las ondas lentas no son potenciales de acción.
3. Durante el paso del bolo hacia la orofaringe, se desencadena una serie de contracciones
musculares que estrechan la cavidad faringea. Estas contracciones están mediadas por el
nervio craneal:
- IX
- XII
- XI
- X
4. ¿Cuál de las siguientes alternativas detallan los músculos que ayudan a empujar el bolo
hacia la orofaringe?
- Estilogloso y palatogloso
- Estilogloso y geniogloso
- Hiogloso y geniogloso
- Geniogloso y palatofaringeo
5. En un paciente es derivado por lesión del nervio vago derecho. Al evaluar el velo del paladar,
se solicita al paciente que diga ahh, entonces se puede observar que la úvula:
- Se eleva en el centro
- Se desvía a la derecha
- Se desvía a la izquierda
- Permanece sin movimiento
6. ¿Cuál de las siguientes estructuras tiene inervación somática?
- Estómago
- Mesosigmoides
- Peritoneo parietal
- Peritoneo Visceral
7. Estas diseñando un proyecto de investigación sobre los niveles de colesterol que se
absorben luego de una comida grasosa y deseas cuantificar la cantidad de colesterol que es
absorbido por el intestino antes que el hígado lo metabolice ¿de cual de los siguientes vasos
obtendrías la muestra para tu análisis?
- Vena porta
- Vena cava superior
- Vena hemiácigos accesoria
- Conducto torácico
8. La masticación es básicamente:
- Un movimiento reflejo
- Importante para la digestión sobretodo de carnes
- Un ralentizador del vaciamiento gástrico
- Una actividad consciente
9. En la estructura dentaria, se observa qué hay una composición muy similar a la del hueso en
la capa denominada:
- Predestina
- Esmalte
- Dentina
- Cemento
10. El nervio palatino menor inerva un área del paladar que está recubierta por epitelio:
- Estratificado plano no queratinizado
- Estratificado plano queratinizado
- Simple cilíndrico
- Simple plano
11. Mujer de 30 años llega a emergencia con dolor en hipogástrico. Al examen físico presenta
una masa palpable de 10cm de diámetro en la misma región, usted sospecharía de las
siguientes condiciones, excepto:
- Tumor renal
- Cáncer de recto superior
- Embarazo
- Tumor uterino
12. Un familiar le comenta que tiene úlcera gástrica por exceso de producción de ácido; con sus
conocimientos del sistema digestivo, usted le recomendaría que reduzca el consumo de:
- Agua
- Aminoácidos
- Carbohidratos
- Vitaminas
13. Con respecto de la regulación del pH del estómago; al utilizar un bloqueador de histamina,
usted espera que el pH del estómago:
- Disminuya
- Se mantenga sin cambio
- Se neutralice por acción de bicarbonato
- Aumente
14. Las siguientes alternativas son ciertas sobre las contracciones tónicas del músculo
gastrointestinal, excepto:
- Se encuentran principalmente es esfínteres
- Tienen relación con el ingreso persistente de iones sodio
- Obedece a una mayor frecuencia de potenciales en espiga
- Tienen regulación hormonal
15. ¿Cuál de las siguientes condiciones considera que es un trastorno de la musculatura lisa
esofágica?
- Asinergia faringoesfinteriana
- Hipotonía de los constrictores faríngeos
- Acalasia
- Hipertonía del esfínter esofágico superior
16. ¿Cuál de las siguientes alternativas es correcta sobre la motilidad esofágica?
- Las ondas primarias no son propulsoras y siempre van precedidas de deglución
- Las ondas primarias son propulsoras y pueden no ser precedidas por deglución
- Las ondas secundarias son propulsoras y no van precedidas de deglución
- Las ondas secundarias son propulsoras y siempre van precedidas de deglución
17. Respecto al peristaltismo intestinal, para cumplir la ley del intestino, usted espera que a nivel
distal del quimo se libere:
- Sustancia P
- Péptido liberador de gastrina (GRP)
- Acetilcolina
- Péptido intestinal vasoactivo
18. El peristaltismo depende que a nivel distal del bolo se secrete:
- Noradrenalina secretada por las fibras del sistema simpático
- Acetilcolina por las neuronas provenientes del nervio vago
- Péptido intestinal vasoactivo por neuronas
- Óxido nítrico por células endoteliales locales
19. La relajación receptiva gástrica, se produce principalmente por la acción de:
- El péptido intestinal vasoactivo
- La sustancia P
- La acetilcolina
- La bombesina
20. El peristaltismo intestinal se produce gracias a un reflejo que:
- Llega a los ganglios pre vertebrales
- Llega a la médula espinal
- Se origina dentro de la pared intestinal
- Llega al sistema nervioso central
21. ¿Cuál de las siguientes alternativas estimula las ondas de motilidad gastrointestinal
denominadas complejos migratorios interdigestivos?
- Alimentos
- Metoclopramida
- ERITROMICINA
- Colecistoquinina (CKK)
22. ¿Cuál de las siguientes alternativas es correcta sobre el control del peristaltismo?
- El peristaltismo intestinal aumenta por efecto de la colecistoquinina (CKK)
- La secretina aumenta la motilidad del intestino delgado
- La serotonina no tiene efecto sobre la motilidad digestiva
- El reflejo gastroentérico evita el avance del quimo
23. En un paciente con shock hipovolémico, la peristaltismo intestinal se encuentra:
- Aumentada
- Sin cambios
- Disminuida
- Invertida
24. Es un derivado del mesenterio dorsal:
- D
- B
- C
- A
25. En un paciente con falla en la fusión de los conductos de las yemas central y dorsal del
páncreas, usted esperaría encontrar:
- Drenaje de la mayor parte del jugo pancreático en la papila menor
- Drenaje adecuado del jugo pancreático
- Drenaje de la mayor parte del jugo pancreático en la papila mayor
- Estenosis del duodeno

Shiro Parcial

1. La relajación receptiva gástrica, se produce principalmente por la acción de:


- La sustancia P
- El péptido intestinal vasoactivo
- La bombesina
- La acetilcolina

2. ¿Cuál de las siguientes alternativas es correcta sobre el movimiento


peristáltico?
- Es independiente del plexo mientérico
- El contenido intestinal avanza sólo 5-10 cm
- Es un reflejo largo que depende de la integración con el tronco encefálico
- Se dirige en sentido distal siempre, nunca en sentido proximal

3. Con respecto a la regulación del peristaltismo, al aplicarle atropina


(antagonista colinérgico) a un paciente, es de esperarse que el peristaltismo:
- aumente
- se mantenga sin alteración
- estimule la acción de los receptores dopaminérgicos
- disminuya

4. El estímulo habitual para el movimiento peristáltico es:


- contracción de la musculatura circular Interna
- distensión local
- acción de la sustancia P
- estimulación vago-vagal

5. El peristaltismo depende que a nivel distal del bolo se secrete:


- péptido intestinal vasoactivo por neuronas
- noradrenalina secretada por las fibras del sistema simpático
- óxido nítrico por células endoteliales locales
- acetilcolina por las neuronas provenientes del nervio vago

6. La hormona responsable de los complejos migratorios interdigestivos tiene las


siguientes características, EXCEPTO:
- es inhibida por el alimento
- cumple funciones de aumentar la motilidad y secreción gástrica e intestinal
- se produce en el estómago y el duodeno
- se libera de forma cíclica
7. Respecto al peristaltismo intestinal, para cumplir la ley del intestino , usted
espera que a nivel distal del quimo se libere:
- péptido liberador de gastrina (GRP)
- acetilcolina
- sustancia P
- péptido intestinal vasoactivo

8. La distención del yeyuno provoca que se:


- aumente la frecuencia de las ondas lentas
- disminuya el número de espigas
- despolarice el potencial de reposo de membrana
- produzca una contracción tónica

9. Un familiar le comenta que tiene úlcera gástrica por exceso de producción de


ácido; con sus conocimientos del sistema digestivo, usted le recomendaría
que reduzca el consumo de:
- carbohidratos
- agua
- aminoácidos
- vitaminas

10. El pH óptimo para la digestión a nivel duodenal es ____________ y está


regulado principalmente por la liberación de _________________
- 6-8 / secretina
- 5-6 / colecistoquinina
- 5-6 / gastrina
- 8-10 / gastrina

11. El tubo digestivo posee glándulas, las glándulas submucosas se encuentran


en el:
- íleon y esófago
- esófago y duodeno
- estómago y duodeno
- duodeno y recto

12. En una cirugía abierta (laparotomía), el cirujano al abrir la cavidad peritoneal


por la parte anterior (línea media), lo primero que observa es:
- Colon sigmoides
- Estomago
- Duodeno
- Epiplón mayor

13. Sobre el control de la peristalsis del tubo digestivo, ________________ es un


mediador neural que induce la relajación durante la peristalsis
- la acetilcolina
- el péptido intestinal vasoactivo
- la serotonina
- la somatostatina

14. ¿Cuál de las siguientes alternativas es correcta sobre el control del


peristaltismo?
- El peristaltismo intestinal aumenta por efecto de la colecistoquinina (CCK)
- El reflejo gastroentérico evita el avance del quimo
- La serotonina no tiene efecto sobre la motilidad digestiva
- La secretina aumenta la motilidad del intestino delgado

15. El frenillo de los labios se encuentra en:


- la cavidad oral
- el piso de la boca
- el dorso de la lengua
- la cavidad vestibular

16. Las siguientes alternativas son ciertas sobre la actividad eléctrica del músculo
gastrointestinal, EXCEPTO:
- Si el potencial de membrana es más positivo, habrá mayor frecuencia de
espigas
- La despolarización lenta se debe principalmente al ingreso de Na+
- Las ondas lentas no son potenciales de acción
- Las ondas lentas son más frecuentes en el duodeno

17. Durante una cena, una gestante inspira por la boca profundamente de manera
frecuente; sin embargo, el organismo evita que el aire ingrese al esófago por la
acción:
- del istmo de las fauces
- del esfínter esofágico inferior
- del músculo cricofaríngeo
- de la epiglotis

18. El músculo liso gastrointestinal funciona como un sincitio debido a:


- el plexo mientérico de Auerbach
- el calcio
- las fibras musculares más largas
- las uniones en hendidura

19. Al ingresar líquidos o sólidos en la cavidad oral, un mecanismo que permite


que una persona respire mientras mastica es:
- la depresión del paladar blando
- el movimiento hacia afuera de los pliegues palatogloso y palatofaríngeo
- la depresión de la parte posterior de la lengua
- la elevación del paladar blando

20. Al rozar agua caliente en la punta de la lengua, usted esperaría que el estímulo
viaje a través del nervio:
- cuerda del tímpano
- lingual
- glosofaríngeo
- hipogloso

21. Las siguientes alternativas son correctas sobre la deglución, EXCEPTO:


- Consta de 2 fases
- En la fase voluntaria, el bolo es impulsado hacia arriba y hacia atrás por la
lengua
- El centro de la respiración es inhibido a nivel bulbar
- Para impedir el paso del alimento a la nasofaringe, el paladar blando se eleva

22. Paciente adulto con reflujo gastroesofágico es más probable que presente:
- desfluorización de los dientes
- destrucción de ameloblastos
- remodelación del esmalte
- descalcificación del esmalte

23. Considerando sus conocimientos en embriología, la disposición del intestino


delgado final tras la retracción de asas intestinales, se distribuye de la
siguiente manera:
- asas yeyunales en hipogastrio
- asas yeyunales en cuadrante inferior derecho
- asas ileales en cuadrante inferior derecho
- duodeno en hipocondrio izquierdo

24. Señale cuál de las estructuras que en el embrión se


encuentra comunicada con el saco vitelino por medio del
conducto onfalomesentérico:
- C
- A
- D
- B

25. Con seguridad, usted puede decir que la siguiente


cicatriz postapendicectomía se encuentra en la región denominada
- fosa iliaca derecha
- flanco izquierdo
- flanco derecho
- hipocondrio derecho

José G. Parcial
1. Niña de 6 años se asusta por que se le ha aflojado un diente deciduo. Este
fenómeno se produce por:
a. laxitud del ligamento periodóntico
b. fractura del cemento
c. desmineralización del esmalte dental
d. aumento anómalo de la predentina

2. ¿Cuál de las siguientes estructuras deriva del intestino anterior?

a. A
b. B
c. C
d. D
3. En un paciente de 3 semanas de edad, con vómitos en proyectil, y nódulo epigástrico
reptante, usted esperaría encontrar:
a. engrosamiento de la circular interna pilórica
b. distensibilidad disminuida de la región oral del estómago
c. colecistoquinina aumentada
d. vómitos biliosos e intolerancia a los ácidos grasos

4. Sobre el control de la peristalsis del tubo digestivo, ________________ es un


mediador neural que induce la relajación durante la peristalsis.
a. el péptido intestinal vasoactivo
b. la serotonina
c. la acetilcolina
d. la somatostatina

5. En el plexo mientérico, el origen de los impulsos eferentes está en:


a. el plexo de Meissner
b. los ganglios paravertebrales
c. las células intersticiales de Cajal
d. el plexo de Aurbach

6. Un familiar le comenta que tiene úlcera gástrica por exceso de producción de ácido;
con sus conocimientos del sistema digestivo, usted le recomendaría que reduzca el
consumo de:
a. aminoácidos
b. vitaminas
c. carbohidratos
d. agua

7. Las siguientes alternativas son ciertas en relación al mesenterio, EXCEPTO:


a. Los omentos cumplen una función principal de irrigación visceral
b. El omento menor deriva del mesenterio ventral
c. El ligamento esplenorrenal une al bazo con el riñón izquierdo
d. Los mesenterios cumplen la función de sostén y suspensión de órganos

8. ¿Cuál de las siguientes alternativas es una característica de la estructura del


esófago?
a. Contiene glándulas submucosas principalmente en su tercio distal
b. Aumenta la presión intra esofágica durante la inspiración
c. Contiene músculo estriado en casi toda su longitud
d. El esfínter esofágico inferior es un esfínter anatómico

9. En una cirugía abierta (laparotomía), el cirujano al abrir la cavidad peritoneal por la


parte anterior (línea media), lo primero que observa es:
a. Epiplón mayor
b. Estómago
c. Colon sigmoides
d. Duodeno
10. El tubo digestivo posee glándulas, las glándulas submucosas se encuentran en el:
a. esófago y duodeno
b. duodeno y recto
c. íleon y esófago
d. estómago y duodeno

11. ¿Cuál de las siguientes alternativas detallan los músculos que ayudan a empujar el
bolo hacia la orofaringe?
a. Estilogloso y palatogloso
b. Geniogloso y palatofaríngeo
c. Hiogloso y geniogloso
d. Estilogloso y geniogloso

12. Al ingresar líquidos o sólidos en la cavidad oral, un mecanismo que permite que una
persona respire mientras mastica es:
a. la depresión del paladar blando
b. la elevación del paladar blando
c. el movimiento hacia afuera de los pliegues palatogloso y palatofaríngeo
d. la depresión de la parte posterior de la lengua

13. Durante el paso del bolo hacia la orofaringe, se desencadena una serie de
contracciones musculares que estrechan la cavidad faríngea. Estas contracciones
están mediadas por el nervio craneal:
a. X
b. IX
c. XI
d. XII

14. Las siguientes alternativas son factores que determinan la patencia y función
adecuada del esfínter esofágico inferior, EXCEPTO:
a. Hipertrofia de la circular interna
b. Canales lentos de calcio
c. Plicatura diafragmática
d. Angulación con el estómago

15. Durante una cena, una gestante inspira por la boca profundamente de manera
frecuente; sin embargo, el organismo evita que el aire ingrese al esófago por la
acción:
a. del músculo cricofaríngeo
b. del istmo de las fauces
c. del esfínter esofágico inferior
d. de la epiglotis
16. En un estudiante de medicina que está rindiendo un examen parcial, lo más probable
es que en ese momento su tránsito intestinal se encuentre:
a. muy lento
b. estimulado por acción de la sustancia P
c. muy acelerado
d. sin alteraciones

17. El principal gobernante sobre todos los movimientos gastrointestinales es el sistema


nervioso:
a. mientérico
b. parasimpático
c. somático
d. simpático

18. ¿Cuál de las siguientes condiciones considera que es un trastorno de la musculatura


lisa esofágica?
a. Acalasia
b. Hipotonía de los constrictores faríngeos
c. Hipertonía del esfínter esofágico superior
d. Asinergia faringoesfinteriana

19. El estímulo habitual para el movimiento peristáltico es:


a. distensión local
b. acción de la sustancia P
c. contracción de la musculatura circular Interna
d. estimulación vago-vagal

20. La hormona responsable de los complejos migratorios interdigestivos tiene las


siguientes características, EXCEPTO:
a. cumple funciones de aumentar la motilidad y secreción gástrica e intestinal
b. se libera de forma cíclica
c. se produce en el estómago y el duodeno
d. es inhibida por el alimento

21. Respecto al peristaltismo intestinal, para cumplir la ley del intestino , usted espera
que a nivel distal del quimo se libere:
a. péptido intestinal vasoactivo
b. acetilcolina
c. péptido liberador de gastrina (GRP)
d. sustancia P

22. ¿Cuál de las siguientes alternativas es correcta sobre el control autónomo del
aparato gastrointestinal?
a. La estimulación simpática estimula a la muscularis mucosae
b. El plexo mientérico de Auerbach cumple funciones inhibitorias
c. Las terminaciones nerviosas simpáticas liberan mayor cantidad de adrenalina
que noradrenalina
d. Al seccionar el vago, la inervación parasimpática del colon sigmoides
disminuye

23. La distención del yeyuno provoca que se:


a. despolarice el potencial de reposo de membrana
b. produzca una contracción tónica
c. disminuya el número de espigas
d. aumente la frecuencia de las ondas lentas

24. El frenillo de los labios se encuentra en:


a. la cavidad vestibular
b. el dorso de la lengua
c. el piso de la boca
d. la cavidad oral

25. Con seguridad, usted puede decir que la siguiente cicatriz postapendicectomía se
encuentra en la región denominada:

a. flanco derecho
b. hipocondrio derecho
c. flanco izquierdo
d. fosa iliaca derecha

EXAMEN PARCIAL - PEQUE

1. La masticación es básicamente:
● un movimiento reflejo
● un ralentizador del vaciamiento gástrico
● una actividad consciente
● importante para la digestión sobretodo de carnes
2. En el plexo mientérico, el origen de los impulsos eferentes está en:
● el plexo de Meissner
● el plexo de Aurbach
● las células intersticiales de Cajal
● los ganglios paravertebrales
3. ¿Cuál de las siguientes alternativas es correcta sobre el control del peristaltismo?
● La secretina aumenta la motilidad del intestino delgado
● El reflejo gastroentérico evita el avance del quimo
● La serotonina no tiene efecto sobre la motilidad digestiva
● El peristaltismo intestinal aumenta por efecto de la colecistoquinina (CCK)
4. Señale cuál de las estructuras que en el embrión se encuentra comunicada con el
saco vitelino por medio del conducto onfalomesentérico:

● D
● A
● C
● B
5. En un paciente de 3 semanas de edad, con vómitos en proyectil, y nódulo epigástrico
reptante, usted esperaría encontrar:
● vómitos biliosos e intolerancia a los ácidos grasos
● distensibilidad disminuida de la región oral del estómago
● engrosamiento de la circular interna pilórica
● colecistoquinina aumentada
6. Durante el paso del bolo hacia la orofaringe, se desencadena una serie de
contracciones musculares que estrechan la cavidad faríngea. Estas contracciones
están mediadas por el nervio craneal:
● IX
● XII
● X
● XI
7. Cuando el istmo de las fauces se cierra, se evita que el alimento pase hacia la
orofarínge y permite respirar mientras se mastica. Este cierre se debe a la
contracción y aproximación de los músculos:
● estilofaríngeos
● palatofaríngeos
● estiloglosos
● palatoglosos
8. Las siguientes alternativas son factores que determinan la patencia y función
adecuada del esfínter esofágico inferior, EXCEPTO:
● Hipertrofia de la circular interna
● Canales lentos de calcio
● Plicatura diafragmática
● Angulación con el estómago
9. El tubo digestivo posee glándulas, las glándulas submucosas se encuentran en el:
● duodeno y recto
● esófago y duodeno
● íleon y esófago
● estómago y duodeno
10. ¿Cuál de las siguientes estructuras tiene inervación somática?
● Mesosigmoides
● Peritoneo visceral
● Estómago
● Peritoneo parietal
11. Mujer de 30 años llega a emergencia con dolor en hipogástrico. Al examen físico
presenta una masa palpable de 10 cm de diámetro a en la misma región, usted
sospecharía de las siguientes condiciones, EXCEPTO:
● Embarazo
● Tumor renal
● Cáncer de recto superior
● Tumor uterino
12. Las siguientes alternativas son ciertas en relación al mesenterio, EXCEPTO:
● Los omentos cumplen una función principal de irrigación visceral
● Los mesenterios cumplen la función de sostén y suspensión de órganos
● El omento menor deriva del mesenterio ventra
● El ligamento esplenorrenal une al bazo con el riñón izquierdo
13. Al realizar una vagotomía por úlcera péptica, usted esperaría una disminución de
producción de gastrina debido a la:
● menor distensión de las paredes gástricas
● ausencia de acetilcolina vagal
● ausencia de bombesina vagal
● ausencia de histamina
14. Al ingresar líquidos o sólidos en la cavidad oral, un mecanismo que permite que una
persona respire mientras mastica es:
● la elevación del paladar blando
● la depresión de la parte posterior de la lengu
● el movimiento hacia afuera de los pliegues palatogloso y palatofaríngeo
● la depresión del paladar blando
15. ¿Cuál de las siguientes alternativas detallan los músculos que ayudan a empujar el
bolo hacia la orofaringe?
● Estilogloso y geniogloso
● Geniogloso y palatofaríngeo
● Hiogloso y geniogloso
● Estilogloso y palatogloso
16. La información sensitiva aferente del sistema gastrointestinal pasa por las siguientes
estructuras, EXCEPTO:
● Ganglios prevertebrales
● Médula espinal
● Tronco encefálico
● Tálamo
17. Los reflejos enterogástricos son desencadenados por las siguientes circunstancias,
EXCEPTO:
● Alta carga de carbohidratos en duodeno
● Distensión de pared intestina
● Quimo con alta osmolaridad
● Disminución de pH duodenal
18. Las siguientes alternativas son ciertas sobre las contracciones tónicas del músculo
gastrointestinal, EXCEPTO:
● Obedece a una mayor frecuencia de potenciales en espiga
● Tienen regulación hormonal
● Se encuentran principalmente en esfínteres
● Tienen relación con el ingreso persistente de iones sodio
19. Respecto al peristaltismo intestinal, para cumplir la ley del intestino , usted espera
que a nivel distal del quimo se libere:
● sustancia P
● péptido intestinal vasoactivo
● péptido liberador de gastrina (GRP)
● acetilcolina
20. El principal gobernante sobre todos los movimientos gastrointestinales es el sistema
nervioso:
● mientérico
● parasimpático
● somático
● simpático
21. La relajación receptiva gástrica, se produce principalmente por la acción de:
● la bombesina
● la sustancia P
● la acetilcolina
● el péptido intestinal vasoactivo
22. Con respecto a la regulación del peristaltismo, al aplicarle atropina (antagonista
colinérgico) a un paciente, es de esperarse que el peristaltismo:
● disminuya
● se mantenga sin alteración
● aumente
● estimule la acción de los receptores dopaminérgicos
23. ¿Cuál de las siguientes alternativas es correcta sobre la motilidad esofágica?
● Las ondas primarias son propulsoras y pueden no ser precedidas por
deglució
● Las ondas secundarias son propulsoras y siempre van precedidas de
deglución
● Las ondas primarias no son propulsoras y siempre van precedidas de
deglución
● Las ondas secundarias son propulsoras y no van precedidas de deglución
24. El nervio palatino menor inerva un área del paladar que está recubierta por epitelio:
● simple plano
● estratificado plano no queratinizado
● estratificado plano queratinizado
● simple cilíndrico
25. Paciente adulto con reflujo gastroesofágico es más probable que presente:
● descalcificación del esmalte
● remodelación del esmalte
● destrucción de ameloblastos
● desfluorización de los dientes

ALEJANDRO HERRERA

1. Al rozar agua caliente en la punta de la lengua, usted esperaría que el estímulo viaje
a través del nervio:

a) Lingual

b) Cuerda del timpano

c) Glosofaríngeo

d) Hipogloso

2. Al ingresar líquidos o sólidos en la cavidad oral, un mecanismo que permite que una
persona respire mientras mastica es:

a) El movimiento hacia afuera de los pliegues palatogloso y Palatofaríngeo

b) La elevación del paladar blando

c) La depresión de la parte posterior de la lengua

d) La depresión del paladar blando


3. Paciente adulto con reflujo gastroesofágico es más probable que presente:

a) Descalcificación del esmalte

b) Destrucción de ameloblastos

c) Remodelación del esmalte

d) Desfluorización de los dientes

4. La masticación es básicamente:

a) Importante para la digestión sobretodo de carnes

b) Un ralentizador del vaciamiento gástrico

c) Una actividad consciente

d) Un movimiento reflejo

5. Paciente de 34 años es víctima de asalto con arma de fuego, recibiendo un impacto


directo en el abdomen. En base a la radiografía, usted puede registrar en la historia
clínica que el proyectil se encuentra topográficamente en el:

a) Flanco derecho

b) Mesogastrio

c) Flanco izquierdo

d) Hipocondrio izquierdo
6. Señale cuál de las estructuras que en el embrión se encuentra comunicada con el
saco vitelino por medio del conducto onfalomesentérico:

7. En un paciente de 3 semanas de edad, con vómitos en proyectil, y nódulo


epigástrico reptante, usted esperaría encontrar:

a) colecistoquinina aumentada

b) Vómitos biliosos e intolerancia a los ácidos grasos

c) Distensibilidad disminuida de la región oral del estómago

d) Engrosamiento de la circular interna pilórica

8. ¿Cuál de las siguientes estructuras tiene inervación somática?

a) Estómago

b) Peritoneo visceral
c) Mesosigmoides

d) Peritoneo parietal

9. Estas diseñando un proyecto de investigación sobre los niveles de colesterol que se


absorben luego de una comida grasosa y deseas cuantificar la cantidad de colesterol
que es absorbido por el intestino antes que el hígado lo metabolice ¿de cuál de los
siguientes vasos obtendrías la muestra para tu análisis?

a) Vena porta

b) Vena cava superior

c) Conducto torácico

d) Vena hemiácigos accesoria

10. Durante el paso del bolo hacia la orofaringe, se desencadena una serie de
contracciones musculares que estrechan la cavidad faríngea. Estas contracciones están
mediadas por el nervio craneal:

a) X

b) XI

c) XII

d) IX

11. Cuando el istmo de las fauces se cierra, se evita que el alimento pase hacia la
orofarínge y permite respirar mientras se mastica. Este cierre se debe a la contracción y
aproximación de los músculos:

a) Palatoglosos

b) Palatofaríngeos

c) Estiloglosos

d) Estilofaríngeos

12. Las siguientes alternativas son factores que determinan la patencia y función
adecuada del esfínter esofágico inferior, EXCEPTO:
a) Plicatura diafragmática

b) Hipertrofia de la circular interna

c) Angulación con el estómago

d) Canales lentos de calcio

13. Respecto al peristaltismo intestinal, para cumplir la ley del intestino, usted espera
que a nivel distal del quimo se libere:

a) Péptido liberador de gastrina (GRP)

b) Acetilcolina

c) Péptido intestinal vasoactivo

d) Sustancia P

14. El estímulo habitual para el movimiento peristáltico es:

a) Acción de la sustancia P

b) Contracción de la musculatura circular Interna

c) Estimulación vago-vagal

d) Distensión local

15. ¿Cuál de las siguientes condiciones considera que es un trastorno de la


musculatura lisa esofágica?

a) Acalasia

b) Asinergia faringoesfinteriana

c) Hipotonía de los constrictores faríngeos

d) Hipertonía del esfínter esofágico superior

16. ¿Cuál de las siguientes alternativas es correcta sobre la motilidad esofágica?

a) Las ondas primarias son propulsoras y pueden no ser precedidas por


deglución
b) Las ondas primarias no son propulsoras y siempre van precedidas de
deglución

c) Las ondas secundarias son propulsoras y siempre van precedidas de


deglución

d) Las ondas secundarias son propulsoras y no van precedidas de


deglución

17. El peristaltismo depende que a nivel distal del bolo se secrete:

a) Péptido intestinal vasoactivo por neuronas

b) Noradrenalina secretada por las fibras del sistema simpático

c) Acetilcolina por las neuronas provenientes del nervio vago

d) Óxido nítrico por células endoteliales locales

18. ¿Cuál de las siguientes alternativas es correcta sobre el movimiento peristáltico?

a) Es un reflejo largo que depende de la integración con el tronco


encefálico

b) Se dirige en sentido distal siempre, nunca en sentido proximal

c) El contenido intestinal avanza sólo 5-10 cm

d) Es independiente del plexo mientérico

19. En un estudiante de medicina que está rindiendo un examen parcial, lo más


probable es que en ese momento su tránsito intestinal se encuentre:

a) Muy acelerado

b) Estimulado por acción de la sustancia p

c) Sin alteraciones

d) Muy lento

20. Las siguientes alternativas son ciertas sobre las contracciones tónicas del músculo
gastrointestinal, EXCEPTO:
a) Tienen relación con el ingreso persistente de iones sodio

b) Se encuentran principalmente en esfínteres

c) Tienen regulación hormonal

d) Obedece a una mayor frecuencia de potenciales en espiga

21. Con respecto de la regulación del pH del estómago; al utilizar un bloqueador de


histamina, usted espera que el pH del estómago:

a) Aumente

b) Disminuya

c) Se mantenga sin cambio

d) Se neutralice por acción de bicarbonato

22. Las siguientes hormonas disminuyen el vaciamiento gástrico, EXCEPTO:

a) Gastrina

b) Péptido insulinotrópico dependiente de glucosa

c) Colecistoquinina

d) Secretina

23. El frenillo de los labios se encuentra en:

a) El piso de la boca

b) La cavidad oral

c) La cavidad vestibular

d) El dorso de la lengua

24. En un paciente con shock hipovolémico, la peristalsis intestinal se encuentra:

a) Aumentada

b) Sin cambios
c) Invertida

d) Disminuida

25. Sobre el control de la peristalsis del tubo digestivo, ________________ es un


mediador neural que induce la relajación durante la peristalsis.

a) La somatostatina

b) El péptido intestinal vasoactivo

c) La acetilcolina

d) La serotonina

26. Las siguientes funciones son inhibidas por la hormona somatostatina, EXCEPTO:
a) motilidad intestinal
b) secreción de enzimas pancreaticas
c) Secrecion gastrica de HCl
d) Liberacion de gastrina

27. ¿Cuál de las siguientes alternativas es correcta sobre el control del peristaltismo?

A) El reflejo gastroentérico evita el avance del quimo


B) La secretina aumenta la motilidad del intestino delgado
C) El peristaltismo intestinal aumenta por efecto de la colecistoquinina (CCK)
D) La serotonina no tiene efecto sobre la motilidad digestiva

28. En el plexo mientérico, el origen de los impulsos eferentes está en:

a) el plexo de Auerbach
b) los ganglios paravertebrales
c) Las células intersticiales de cajal
d) plexo de meissner

29. Los corpúsculos gustativos se encuentran en la lengua, pero además se les puede
encontrar en:

a) el dorso de la lengua
b) el paladar blando
c) el paladar duro
d) las encías

30. ¿Cuál de las siguientes condiciones considera que es un trastorno de la musculatura lisa
esofágica?
a) Hipertonía del esfínter esofágico superior
b) Acalasia
c) Hipotonía de los constrictores faríngeos
d) Asinergia faringoesfinteriana

Dayanne Cruz

1. ¿Cuál de las siguientes alternativas detallan los músculos que ayudan a empujar el
bolo hacia la orofaringe?
a. Geniogloso y palatofaríngeo
b. Estilogloso y geniogloso
c. Hiogloso y geniogloso
d. Estilogloso y palatoglo
2. Paciente con lesión del hipogloso del lado izquierdo. Para evaluarlo se le pide al
paciente que saque la lengua, la cual se espera que la punta de la lengua se dirija
hacia:
a. abajo
b. el lado izquierdo
c. adelante
d. el lado derecho
3. Cuando el istmo de las fauces se cierra, se evita que el alimento pase hacia la
orofarínge y permite respirar mientras se mastica. Este cierre se debe a la
contracción y aproximación de los músculos
a. estiloglosos
b. palatofaríngeos
c. palatoglosos
d. estilofaríngeos
4. Las siguientes alternativas son ciertas sobre la actividad eléctrica del músculo
gastrointestinal, EXCEPTO:
a. Si el potencial de membrana es más positivo, habrá mayor frecuencia de
espigas
b. Las ondas lentas son más frecuentes en el duodeno
c. La despolarización lenta se debe principalmente al ingreso de Na+
d. Las ondas lentas no son potenciales de acción
5. Durante la deglución, el bolo es impedido de regresar a la cavidad oral gracias a la
acción de diversos músculos, entre ellos el músculo:
a. estilofaríngeo
b. palatofaríngeo
c. cricofaríngeo
d. salpingofaríngeo
6. En una cirugía abierta (laparotomía), el cirujano al abrir la cavidad peritoneal por la
parte anterior (línea media), lo primero que observa es
a. Estomago
b. Duodeno
c. Colon sigmoides
d. Epiplón mayor
7. Estas diseñando un proyecto de investigación sobre los niveles de colesterol que se
absorben luego de una comida grasosa y deseas cuantificar la cantidad de colesterol
que es absorbido por el intestino antes que el hígado lo metabolice ¿de cuál de los
siguientes vasos obtendrías la muestra para tu análisis?
a. Vena cava superior
b. Conducto torácico
c. Vena hemiácigos accesoria
d. Vena porta
8. Paciente de 34 años es víctima de asalto con arma de fuego, recibiendo un impacto
directo en el abdomen. En base a la radiografía, usted puede registrar en la historia
clínica que el proyectil se encuentra topográficamente en el:

a. flanco izquierdo
b. hipocondrio izquierdo
c. flanco derecho
d. mesogastrio

9. El nervio palatino menor inerva un área del paladar que está recubierta por
epitelio:

a. estratificado plano queratinizado


b. estratificado plano no queratinizado
c. simple plano
d. simple cilíndrico

10. ¿Cuál de las siguientes estructuras deriva del intestino anterior?


a. C
b. B
c. A
d. D

11. ¿Cuál de las siguientes atresias/fístulas traqueo esofágicas considera usted que es
incompatible con la vida (de no recibir tratamiento)?

a. Tipo E
b. Tipo B
c. Tipo C
d. Tipo A

12. Las siguientes hormonas disminuyen el vaciamiento gástrico, EXCEPTO:

a. Péptido insulinotrópico dependiente de glucosa


b. Secretina
c. Colecistoquinina
d. Gastrina

13. Las siguientes alternativas son ciertas en relación al mesenterio, EXCEPTO

a. El omento menor deriva del mesenterio ventral


b. El ligamento esplenorrenal une al bazo con el riñón izquierdo
c. Los mesenterios cumplen la función de sostén y suspensión de órganos
d. Los omentos cumplen una función principal de irrigación viscera

14. ¿Cuál de las siguientes alternativas es una característica de la estructura del esófago?

a. Contiene músculo estriado en casi toda su longitud


b. El esfínter esofágico inferior es un esfínter anatómico
c. Aumenta la presión intra esofágica durante la inspiración
d. Contiene glándulas submucosas principalmente en su tercio distal

15. ¿Cuál de las siguientes alternativas es correcta sobre el control del peristaltismo?
a. La serotonina no tiene efecto sobre la motilidad digestiva
b. El reflejo gastroentérico evita el avance del quimo
c. La secretina aumenta la motilidad del intestino delgado
d. El peristaltismo intestinal aumenta por efecto de la colecistoquinina (CCK)

16. En el plexo mientérico, el origen de los impulsos eferentes está en:

a. el plexo de Meissner
b. las células intersticiales de Cajal
c. el plexo de Aurbach
d. los ganglios paravertebrales

17. El peristaltismo depende que a nivel distal del bolo se secrete:

a. noradrenalina secretada por las fibras del sistema simpático


b. óxido nítrico por células endoteliales locales
c. acetilcolina por las neuronas provenientes del nervio vago
d. péptido intestinal vasoactivo por neuronas

18. En un estudiante de medicina que está rindiendo un examen parcial, lo más probable es
que

a. muy lento
b. sin alteraciones
c. estimulado por acción de la sustancia P
d. muy acelerado en ese momento su tránsito intestinal se encuentre:

19. Los reflejos enterogástricos son desencadenados por las siguientes circunstancias,
EXCEPTO:

a. Distensión de pared intestinal


b. Disminución de pH duodenal
c. Quimo con alta osmolaridad
d. Alta carga de carbohidratos en duodeno

20. La distención del yeyuno provoca que se:


a. disminuya el número de espigas
b. despolarice el potencial de reposo de membrana
c. produzca una contracción tónica
d. aumente la frecuencia de las ondas lentas

21. Respecto al peristaltismo intestinal, para cumplir la ley del intestino , usted espera que a
nivel distal del quimo se libere:
a. péptido intestinal vasoactivo
b. sustancia P
c. péptido liberador de gastrina (GRP)
d. acetilcolina
22. ¿Cuál de las siguientes alternativas es correcta sobre el movimiento peristáltico?
a. Es un reflejo largo que depende de la integración con el tronco encefálico
b. El contenido intestinal avanza sólo 5-10 cm
c. Es independiente del plexo mientérico
d. Se dirige en sentido distal siempre, nunca en sentido proximal
23. ¿Cuál de las siguientes alternativas estimula las ondas de motilidad gastrointestinal
denominadas complejos migratorios interdigestivos?
a. Metoclopramida
b. Alimentos
c. Eritromicina
d. Colecistoquinina (CCK)
24. ¿Cuál de las siguientes condiciones considera que es un trastorno de la musculatura lisa
esofágica?
a. Hipertonía del esfínter esofágico superior
b. Asinergia faringoesfinteriana
c. Acalasia
d. Hipotonía de los constrictores faríngeos
25. En la estructura dentaria, se observa que hay una composición muy similar a la del
hueso en la capa denominada
a. dentina
b. predentina.
c. esmalte
d. cemento

1. ¿Cuál de las siguientes alternativas detallan los músculos que ayudan a empujar el bolo
hacia la orofaringe?

- Estilogloso y palatogloso
- Hiogloso y geniogloso
- Geniogloso y palatofaríngeo

2. Paciente con lesión del hipogloso del lado izquierdo. Para evaluarlo se le pide al paciente que
saque la lengua, la cual se espera que la punta de la lengua se dirija hacia
- El lado izquierdo
- Adelante
- Abajo
- El lado derecho
3. Niña de 6 años se asusta porque se le ha aflojado un diente deciduo. Este fenómeno se
produce por:
- Laxitud del ligamento periodontal
- Aumento anómalo de la predentina
- Fractura del cemento
- Desmineralización del esmalte dental

4. Sobre el control de la peristalsis del tubo digestivo, ________________ es un mediador


neutral que induce la relajación durante la peristalsis
- El péptido intestinal vasoactivo
- Somatostatina
- Serotonina
- Acetilcolina

5. En el plexo mientérico , el origen de los impulsos eferentes esta en


- El plexo de Auerbach
- Los ganglios paravertebrales
- Plexo de Meissner
- Células intersticiales de Cajal

6. Los corpúsculos gustativos se encuentran en la lengua, pero además se les puede encontrar
en:
- El dorso de la lengua
- El paladar duro
- El paladar blando
- Las encías

7. La masticación es básicamente:
- Un movimiento reflejo
- Una actividad consciente
- Un ralentizador del vaciamiento gástrico
- Importante para la digestión sobre todo de carnes

8. Con respecto de la regulación del pH del estómago; al utilizar un bloqueador de histamina,


usted espera que el pH del estómago:
- Aumente
- Disminuya
- Neutraliza por acción de bicarbonato
- Se mantenga sin cambio

9. Al realizar una vagotomía por úlcera péptica, usted esperaría una disminución de producción
de gastrina debido a la:
- Ausencia de histamina
- Menor distensión de las paredes gástricas
- ausencia de bombesina vagal
- ausencia de acetilcolina vagal

10. Mujer de 30 años llega a emergencia con dolor en hipogástrico. Al examen físico presenta
una masa palpable de 10 cm de diámetro a en la misma región, usted sospecharía de las
siguientes condiciones, EXCEPTO:
- Tumor renal
- Cáncer de recto superior
- Embarazo
- Tumor uterino

11. En un estudiante de medicina que está rindiendo un examen parcial, lo más probable es que
en ese momento su tránsito intestinal se encuentre:
- Muy lento
- Estimulado por acción de la sustancia P
- muy acelerado
- sin alteraciones

12. ¿Cuál de las siguientes alternativas estimula las ondas de motilidad gastrointestinal
denominadas complejos migratorios interdigestivos?
- Eritromicina
- Colecistoquinina
- Alimentos
- Metoclopramida
13. Respecto al peristaltismo intestinal, para cumplir la ley del intestino , usted espera que a nivel
distal del quimo se libere:
- Péptido intestinal vasoactivo
- acetilcolina
- sustancia p
- péptido liberador de gastrina
14. ¿ Cual de las siguientes alternativas es correcta sobre el control autonomo del aparato
gastrointestinal?
- La estimulación simpática estimula a la muscularis mucosae
- Las terminaciones nerviosas simpáticas liberan mayor cantidad de adrenalina que
noradrenalina
- al seccionar al vago , la inervación parasimpática del colon sigmoides disminuye
- el plexo mientérico de auerbach cumple funciones inhibitorias
15. La distensión del yeyuno provoca que
- Aumente la frecuencia de ondas lentas
- disminuya el número de espigas
- produzca contracción tónica
- despolariza el potencial de reposo de membrana
Francisco Parcial
Cuando una persona coloca en su boca una sustancia con alta concentración de carbohidratos, lo
que debería pasar es que:
- aumente las concentraciones séricas de gastrina
- disminuya la sensación de hambre
- aumenta la motilidad gástrica
- se dilate el esfínter esofágico inferior

ANGELA PARCIAL:
1. Las carnes deben su sabor especialmente delicioso debido a que presentan en su
composición:
- glutamato
2. En un paciente es derivado por lesión del nervio vago derecho. Al evaluar el velo del
paladar, se solicita al paciente que diga ahh , entonces se puede observar que la úvula:
- se desvía a la izquierda
3. Al rozar agua caliente en la punta de la lengua, usted esperaría que el estímulo viaje a
través del nervio:
- lingual
4. Considerando sus conocimientos en embriología, la disposición del intestino delgado
final tras la retracción de asas intestinales, se distribuye de la siguiente manera:
- asas ileales en cuadrante inferior derecho
5. En un paciente de 3 semanas de edad, con vómitos en proyectil, y nódulo epigástrico
reptante, usted esperaría encontrar:
- engrosamiento de la circular interna pilórica
6. La masticación es básicamente:
- un movimiento reflejo
7. Un familiar le comenta que tiene úlcera gástrica por exceso de producción de ácido;
con sus conocimientos del sistema digestivo, usted le recomendaría que reduzca el
consumo de:
- aminoácidos
8. Con respecto de la regulación del pH del estómago; al utilizar un bloqueador de
histamina, usted espera que el pH del estómago:
- aumente
9. Paciente adulto con reflujo gastroesofágico es más probable que presente:
- descalcificación del esmalte
10. Sobre el control de la peristalsis del tubo digestivo, ________________ es un mediador
neural que induce la relajación durante la peristalsis.
- el péptido intestinal vasoactivo
11. En el plexo mientérico, el origen de los impulsos eferentes está en:
- el plexo de Meissner
12. Mujer de 30 años llega a emergencia con dolor en hipogástrico. Al examen físico
presenta una masa palpable de 10 cm de diámetro a en la misma región, usted
sospecharía de las siguientes condiciones, EXCEPTO:
- Tumor renal
13. Los reflejos enterogástricos son desencadenados por las siguientes circunstancias,
EXCEPTO:
- Alta carga de carbohidratos en duodeno
14. La hormona responsable de los complejos migratorios interdigestivos tiene las
siguientes características, EXCEPTO:
- cumple funciones de aumentar la motilidad y secreción gástrica e intestinal
15. La relajación receptiva gástrica, se produce principalmente por la acción de:
- el péptido intestinal vasoactivo
16. ¿Cuál de las siguientes alternativas estimula las ondas de motilidad gastrointestinal
denominadas complejos migratorios interdigestivos?
- Eritromicina
17. En un estudiante de medicina que está rindiendo un examen parcial, lo más probable
es que en ese momento su tránsito intestinal se encuentre:
- muy lento
18. El estímulo habitual para el movimiento peristáltico es:
- distensión local
19. La distención del yeyuno provoca que se:
- despolarice el potencial de reposo de membrana
20. El principal gobernante sobre todos los movimientos gastrointestinales es el sistema
nervioso:
- mientérico
21. Estas diseñando un proyecto de investigación sobre los niveles de colesterol que se
absorben luego de una comida grasosa y deseas cuantificar la cantidad de colesterol
que es absorbido por el intestino antes que el hígado lo metabolice ¿de cuál de los
siguientes vasos obtendrías la muestra para tu análisis?
- Conducto torácico
22. El tubo digestivo posee glándulas, las glándulas submucosas se encuentran en el:
- esófago y duodeno
23. Durante la deglución, el bolo es impedido de regresar a la cavidad oral gracias a la
acción de diversos músculos, entre ellos el músculo:
- palatofaríngeo
24. Cuando el istmo de las fauces se cierra, se evita que el alimento pase hacia la
orofarínge y permite respirar mientras se mastica. Este cierre se debe a la contracción
y aproximación de los músculos:
- palatoglosos
25. El músculo liso gastrointestinal funciona como un sincitio debido a:
- las uniones en hendidura

En el esófago, el plexo mientérico o de Auerbach interviene en el _______ de los impulsos ______

a. fin / aferentes
b. fin / eferentes
c. inicio / eferentes
d. inicio / aferentes

Pregunta 1
El nervio palatino menor inerva un área del paladar que está recubierta por epitelio:
● estratificado plano no queratinizado
● simple plano
● estratificado plano queratinizado
● simple cilíndrico
Pregunta 2
¿Cuál de las siguientes atresias/fístulas traqueo esofágicas considera usted que es
incompatible con la vida (de no recibir tratamiento)?
● Tipo A
● Tipo B
● Tipo C
● Tipo E
Pregunta 3
¿Cuál de las siguientes estructuras deriva del intestino anterior?

● A
● D
● C
● B
Pregunta 4
El tubo digestivo posee glándulas, las glándulas submucosas se encuentran en el:
● esófago y duodeno
● duodeno y recto
● íleon y esófago
● estómago y duodeno
Pregunta 5
Estas diseñando un proyecto de investigación sobre los niveles de colesterol que se
absorben luego de una comida grasosa y deseas cuantificar la cantidad de colesterol que es
absorbido por el intestino antes que el hígado lo metabolice ¿de cuál de los siguientes vasos
obtendrías la muestra para tu análisis?
● Vena hemiácigos accesoria
● Vena porta
● Conducto torácico
● Vena cava superior
Pregunta 6
Un familiar le comenta que tiene úlcera gástrica por exceso de producción de ácido; con sus
conocimientos del sistema digestivo, usted le recomendaría que reduzca el consumo de:
● vitaminas
● agua
● carbohidratos
● aminoácidos
Pregunta 7
Con respecto de la regulación del pH del estómago; al utilizar un bloqueador de histamina,
usted espera que el pH del estómago:
● se neutralice por acción de bicarbonato
● se mantenga sin cambio
● disminuya
● aumente
Pregunta 8
¿Cuál de las siguientes alternativas es una característica de la estructura del esófago?
● El esfínter esofágico inferior es un esfínter anatómico
● Contiene músculo estriado en casi toda su longitud
● Contiene glándulas submucosas principalmente en su tercio distal
● Aumenta la presión intra esofágica durante la inspiración
Pregunta 9
¿Cuál de las siguientes alternativas detallan los músculos que ayudan a empujar el bolo
hacia la orofaringe?
● Hiogloso y geniogloso
● Geniogloso y palatofaríngeo
● Estilogloso y geniogloso
● Estilogloso y palatogloso
Pregunta 10
Al ingresar líquidos o sólidos en la cavidad oral, un mecanismo que permite que una
persona respire mientras mastica es:
● la depresión del paladar blando
● la elevación del paladar blando
● el movimiento hacia afuera de los pliegues palatogloso y palatofaríngeo
● la depresión de la parte posterior de la lengua
Pregunta 11
Niña de 6 años se asusta por que se le ha aflojado un diente deciduo. Este fenómeno se
produce por:
● laxitud del ligamento periodóntico
● desmineralización del esmalte dental
● aumento anómalo de la predentina
● fractura del cemento
Pregunta 12
Con seguridad, usted puede decir que la siguiente cicatriz postapendicectomía se encuentra
en la región denominada:
● flanco derecho
● hipocondrio derecho
● fosa iliaca derecha
● flanco izquierdo
Pregunta 13
Cuando el istmo de las fauces se cierra, se evita que el alimento pase hacia la orofarínge y
permite respirar mientras se mastica. Este cierre se debe a la contracción y aproximación de
los músculos:
● estiloglosos
● palatofaríngeos
● estilofaríngeos
● palatoglosos
Pregunta 14
Durante el paso del bolo hacia la orofaringe, se desencadena una serie de contracciones
musculares que estrechan la cavidad faríngea. Estas contracciones están mediadas por el
nervio craneal:
● X
● IX
● XII
● XI
Pregunta 15
Durante la deglución, el bolo es impedido de regresar a la cavidad oral gracias a la acción
de diversos músculos, entre ellos el músculo:
● palatofaríngeo
● estilofaríngeo
● salpingofaríngeo
● cricofaríngeo
Pregunta 16
En el plexo mientérico, el origen de los impulsos eferentes está en:
● el plexo de Aurbach
● el plexo de Meissner
● las células intersticiales de Cajal
● los ganglios paravertebrales
Pregunta 17
¿Cuál de las siguientes alternativas es correcta sobre el control del peristaltismo?
● La serotonina no tiene efecto sobre la motilidad digestiva
● El reflejo gastroentérico evita el avance del quimo
● La secretina aumenta la motilidad del intestino delgado
● El peristaltismo intestinal aumenta por efecto de la colecistoquinina (CCK)
Pregunta 18
En un estudiante de medicina que está rindiendo un examen parcial, lo más probable es que
en ese momento su tránsito intestinal se encuentre:
● estimulado por acción de la sustancia P
● sin alteraciones
● muy acelerado
● muy lento
Pregunta 19
El peristaltismo intestinal se produce gracias a un reflejo que:
● llega a los ganglios pre vertebrales
● llega a la médula espinal
● se origina dentro de la pared intestinal
● llega al sistema nervioso central
Pregunta 20
El estímulo habitual para el movimiento peristáltico es:
● distensión local
● acción de la sustancia P
● contracción de la musculatura circular Interna
● estimulación vago-vagal
Pregunta 21
¿Cuál de las siguientes alternativas es correcta sobre el control autónomo del aparato
gastrointestinal?
● La estimulación simpática estimula a la muscularis mucosae
● El plexo mientérico de Auerbach cumple funciones inhibitorias
● Las terminaciones nerviosas simpáticas liberan mayor cantidad de adrenalina que
noradrenalina
● Al seccionar el vago, la inervación parasimpática del colon sigmoides disminuye
Pregunta 22
El principal gobernante sobre todos los movimientos gastrointestinales es el sistema
nervioso:
● simpático
● somático
● mientérico
● parasimpático
Pregunta 23
¿Cuál de las siguientes alternativas estimula las ondas de motilidad gastrointestinal
denominadas complejos migratorios interdigestivos?
● Eritromicina
● Colecistoquinina (CCK)
● Alimentos
● Metoclopramida
Pregunta 24
Con respecto a la regulación del peristaltismo, al aplicarle atropina (antagonista colinérgico)
a un paciente, es de esperarse que el peristaltismo:
● disminuya
● estimule la acción de los receptores dopaminérgicos
● se mantenga sin alteración
● aumente
Pregunta 25
¿Cuál de las siguientes alternativas es correcta sobre el movimiento peristáltico?
● Es independiente del plexo mientérico
● Es un reflejo largo que depende de la integración con el tronco encefálico
● El contenido intestinal avanza sólo 5-10 cm
● Se dirige en sentido distal siempre, nunca en sentido proximal

¿Cuál de las siguientes alternativas es una característica de la estructura del esófago?

- Contiene glándulas submucosas principalmente en su tercio distal

- Aumenta la presión intra esofágica durante la inspiración

- El esfínter esofágico inferior es un esfínter anatómico

Las siguientes hormonas disminuyen el vaciamiento gástrico, EXCEPTO:

- Gastrina

- Péptido insulinotrópico dependiente de glucosa

- Secretina

- Colecistoquinina

Luego de una comida rica en proteínas, se espera que los valores de secretina en sangre:

- sólo se altere si hay un aumento de colecistoquinina (CCK)

- se mantenga sin cambio

- aumenten

- disminuyan

¿Cuál de las siguientes estructuras tiene inervación somática?

- Peritoneo parietal
- Mesosigmoides

- Peritoneo visceraL

- Estómago

Estas diseñando un proyecto de investigación sobre los niveles de colesterol que se


absorben luego de una comida grasosa y deseas cuantificar la cantidad de colesterol que es
absorbido por el intestino antes que el hígado lo metabolice ¿de cuál de los siguientes vasos
obtendrías la muestra para tu análisis?

- Vena porta

- Vena hemiácigos accesoria

- Vena cava superior

- Conducto torácico

Durante el paso del bolo hacia la orofaringe, se desencadena una serie de contracciones
musculares que estrechan la cavidad faríngea. Estas contracciones están mediadas por el
nervio craneal:

- X

- XII

- IX

- XI

Las siguientes alternativas son factores que determinan la patencia y función adecuada del
esfínter esofágico inferior, EXCEPTO:

- Hipertrofia de la circular interna

- Angulación con el estómago

- Canales lentos de calcio

- Canales lentos de calcio

Cuando el istmo de las fauces se cierra, se evita que el alimento pase hacia la orofarínge y
permite respirar mientras se mastica. Este cierre se debe a la contracción y aproximación de
los músculos:

- palatofaríngeos

- estilofaríngeos

- estiloglosos
- Palatogloso

En la estructura dentaria, se observa que hay una composición muy similar a la del hueso
en la capa denominada:

- dentina

- predentina

- cemento

- esmalte

El frenillo de los labios se encuentra en:

- el dorso de la lengua

- la cavidad vestibular

- el piso de la boca

- la cavidad oral

Es un derivado del mesenterio dorsal:

En un paciente con falla en la fusión de los conductos de las yemas ventral y dorsal del
páncreas, usted esperaría encontrar:

- drenaje de la mayor parte del jugo pancreático en la papila menor

- drenaje adecuado del jugo pancreático

- estenosis del duodeno

- drenaje de la mayor parte del jugo pancreático en la papila mayor

Mujer de 30 años llega a emergencia con dolor en hipogástrico. Al examen físico presenta
una masa palpable de 10 cm de diámetro a en la misma región, usted sospecharía de las
siguientes condiciones, EXCEPTO:

- Tumor renal

- Tumor uterino
- Cáncer de recto superior

- Embarazo

El principal gobernante sobre todos los movimientos gastrointestinales es el sistema


nervioso:

- Mientérico

- Simpático

- Parasimpático

- Somático

La distención del yeyuno provoca que se:

- despolarice el potencial de reposo de membrana

- disminuya el número de espigas

- produzca una contracción tónica

- aumente la frecuencia de las ondas lentas

La información sensitiva aferente del sistema gastrointestinal pasa por las siguientes
estructuras, EXCEPTO:

- Tronco encefálico

- Ganglios prevertebrales

- Médula espinal

- Tálamo

¿Cuál de las siguientes alternativas es correcta sobre la motilidad esofágica?

- Las ondas secundarias son propulsoras y no van precedidas de deglución

- Las ondas primarias no son propulsoras y siempre van precedidas de deglución

- Las ondas primarias son propulsoras y pueden no ser precedidas por deglución

- Las ondas secundarias son propulsoras y siempre van precedidas de deglución

¿Cuál de las siguientes alternativas es correcta sobre el control autónomo del aparato
gastrointestinal?

- La estimulación simpática estimula a la muscularis mucosae


- Las terminaciones nerviosas simpáticas liberan mayor cantidad de adrenalina que
noradrenalina

- Al seccionar el vago, la inervación parasimpática del colon sigmoides disminuye

- El plexo mientérico de Auerbach cumple funciones inhibitorias

El peristaltismo intestinal se produce gracias a un reflejo que:

● - llega al sistema nervioso central


● - se origina dentro de la pared intestinal
● - llega a los ganglios prevertebrales
● - llega a la médula espinal

¿Cuál de las siguientes alternativas estimula las ondas de motilidad gastrointestinal


denominadas complejos migratorios interdigestivos?

- Metoclopramida

- Colecistoquinina (CCK)

- Eritromicina

- Alimentos

El peristaltismo depende que a nivel distal del bolo se secrete:

- acetilcolina por las neuronas provenientes del nervio vago

- óxido nítrico por células endoteliales locales

- péptido intestinal vasoactivo por neuronas

- noradrenalina secretada por las fibras del sistema simpático

Sobre el control de la peristalsis del tubo digestivo, ________________ es un mediador


neural que induce la relajación durante la peristalsis.

- el péptido intestinal vasoactivo

- la somatostatina

- la serotonina

- la acetilcolina

En el esófago, el plexo mientérico o de Auerbach interviene en el _______ de los impulsos


______

- inicio / eferentes
- fin / eferentes

- inicio / aferentes

- fin / aferentes

Al rozar agua caliente en la punta de la lengua, usted esperaría que el estímulo viaje a
través del nervio:

- cuerda del tímpano

- lingual

- hipogloso

- glosofaríngeo

¿Cuál de las siguientes alternativas detallan los músculos que ayudan a empujar el bolo
hacia la orofaringe?

- Estilogloso y palatogloso

- Estilogloso y geniogloso

- Hiogloso y geniogloso

- Geniogloso y palatofarínge
CI3 - Francisco

Los pliegues gástricos gruesos son prácticamente inexistentes a nivel de:

el fondo

En un paciente con xerostomía presenta las siguientes condiciones, EXCEPTO:

Infecciones del oído a repetición

En fases iniciales, la infección por Helicobacter pylori genera disminución del pH gástrico
debido a:

la disminución de somatostatina

La fase intestinal de la secreción gástrica se debe básicamente a la participación de las


células:

G del duodeno

Los siguientes mecanismos determinan la patencia del esfínter esofágico inferior y ayudan a
impedir la enfermedad por reflujo gastroesofágico, EXCEPTO:

Longitud mayor del esófago

A raíz de la infección por Helicobacter pilory, la destrucción de las células ____________


ocasiona que aumente la producción de ácido en el estómago y por ello se produzca úlceras
____________

D / duodenales

En caso se produzca la perforación de la cara anterior del estómago, esta perforación


ocasionará _________ producto de la peritonitis química.

íleo intestinal

Al usar un parasimpaticomimético (agonista colinérgico), usted espera que la saliva


presente:

una mayor cantidad de sodio

La saliva siempre será hipotónica debido a:

la impermeabilidad de los conductos al agua

En la producción de HCl, la acción de la somatostatina disminuye la accion de:

la gastrina
CI3 DE DIGESTIVO (LES)

En una gestante de 11 semanas con antecedente de esofagitis eosinofílica, con hiperémesis


gravídica, que acude a emergencia por hematemesis leve, y presenta súbitamente disnea y
dolor torácico. Usted sospecharía de:

a) síndrome de Mallory Weiss


b) neumonía
c) esofagitis erosiva severa por reflujo
d) síndrome de Boerhaav

La fase intestinal de la secreción gástrica se debe básicamente a la participación de las


células:

a) G del duodeno
b) I del yeyuno
c) D del estómago
d) S del íleon

La vena porta se forma gracias a la unión de la vena mesentérica superior con la vena:

a) esplénica
b) mesentérica inferior
c) celiaca
d) gástrica izquierda

Aquellas células que producen mayor cantidad de moco en el epitelio gástrico son las:

a) mucosas superficiales
b) enterocromafines
c) mucosas del cuello
d) mucosas del fondo

La saliva siempre será hipotónica debido a:

a) la mayor permeabilidad al sodio en los conductos


b) el estímulo del sistema parasimpática
c) la impermeabilidad de los conductos al agua
d) la ausencia de conductor estriado

Para determinar que un paciente tiene esófago de Barrett, debemos encontrar ___________
en la biopsia de esófago.
a) células caliciformes
b) displasia
c) epitelio gástrico
d) glándulas cardiales

Paciente con acalasia es sometido a tratamiento endoscópico o quirúrgico, usted le ha


informado al paciente previamente que es posible que una complicación de este tratamiento
es que quede con cierto grado de:

a) reflujo gastroesofágico
b) úlceras gástricas
c) odinofagia
d) gastritis

¿Cuál de las siguientes alternativas es una causa de reflujo gastroesofágico de contenido


ácido?

a) Acalasia
b) Divertículo de Zenker
c) Hernia Hiatal
d) Anillo de Schatzki

En caso se produzca la perforación de la cara anterior del estómago, esta perforación


ocasionará _________ producto de la peritonitis química.

a) mayor secreción de colecistoquinina


b) íleo intestinal
c) esófago de Barret
d) mayor secreción de colecistoquinina
e) úlcera duodenal

La sangre que lleva la vena porta es tipo:

a) venosa
b) mixta
c) arterial

1. Al evaluar a un paciente con parotiditis purulenta (con absceso), usted buscaría el orificio
terminal del conducto de _____________ a la altura de _____________ → Stenon / la
segunda molar superior

2. Para bloquear la principal fuerza promotora de producción de ácido clorhídrico, lo ideal es un


medicamento que sea bloqueador de la bomba: de hidrógeno-potasio

3. En fases iniciales, la infección por Helicobacter pylori genera disminución del pH gástrico
debido a → la disminución de somatostatina
4. La comunicación entre la irrigación gástrica y la esofágica depende de una rama de la
arteria:tronco celiaco
5. Los pliegues gástricos gruesos son prácticamente inexistentes a nivel de → el fondo

6. En pacientes con hernia hiatal, la constante inflamación del esófago, puede aparecer
_______________ como complicación debida a la fibrosis por inflamación crónica. → anillo
de Schatzki
7. En un paciente con xerostomía presenta las siguientes condiciones, EXCEPTO:
Infecciones del oído a repetición

ANGELA CI-1:
1) Paciente de 24 años con dolor abdominal tipo cólico intenso en mesogastrio. Según
sus conocimientos de macroestructura, el origen del dolor puede ser el
___________:
Respuestas:
a) Colon
b) Íleon
c) Esófago
d) Estómago

2) Paciente se queja de dolor en hipocondrio derecho, pero superficialmente. El dermatoma


relacionado es (marque la mejor respuesta):
Respuestas:
a) T9
b) T11
c) T12
d) T10

3) Señale la respuesta correcta:


Respuestas:
a) El esófago sólo tiene adventicia
b) El colon ascendente sólo tiene serosa
c) El apéndice cecal sólo tiene serosa
d) El páncreas sólo tiene adventicia

4) Cuál de las siguientes estructuras no tiene vasos sanguíneos:


Respuestas:
a) Ligamento
b) Mesenterio
c) Epitelio intestinal
d) Omento

5) Paciente tiene una úlcera sangrante en el segundo tercio del Yeyuno. La arteria de la cual
proviene la sangre arterial para dicha zona es la arteria:
Respuestas:
a) Tronco celíaco
b) Mesentérica inferior

c) Mesentérica superior
d) Gástrica izquierda
e) Iliaca común
6) Al retirar completamente el mesenterio de un órgano, el mismo se vería afectado
principalmente en su:
Respuestas:
a) Inervación
b) Irrigación
c) Tamaño
d) No se afecta en absoluto

7) Respecto a la anatomía del estómago, marque lo correcto:


Respuestas:
a) El píloro se encuentra en el cuerpo gástrico
b) La incisura angularis puede estar en la curvatura mayor.
c) La porción más distal del estómago es el cardias
d) El fondo gástrico forma la curvatura mayor

8) Al examinar a un paciente, usted encuentra dolor localizado en fosa iliaca derecha y


diagnostica apendicitis. En este paciente, usted puede inferir:
Respuestas:
a) Hay inflamación de todo el peritoneo parietal (peritonitis)
b) El peritoneo parietal regional está afectado
c) El peritoneo visceral regional está principalmente afectado
d) El diagnóstico está errado por no corresponder a la región abdominal adecuada

9) Marque el órgano que se considera retroperitoneal:


Respuestas:
a) Lóbulo izquierdo del hígado
b) Vesícula biliar
c) Parte de la vía biliar
d) Sigmoides

10) La peristalsis o peristaltismo hace referencia a:


Respuestas:
a) Motilidad para mezclado de alimentos.
b) No es parte de la motilidad
c) Motilidad para fraccionamiento de alimentos.
d) Motilidad para movilizar el alimento de proximal a distal.

ANGELA CI-3:

1. Al usar atropina en una paciente, usted esperaría:


a) la disminución de gastrina
b) la disminución de secreción gástrica por bloqueo de M3
c) la hipersalivación
d) el aumento de histamina

2. Para determinar que un paciente tiene esófago de Barret, debemos encontrar


______________ en la biopsia de esófago.
a) células caliciformes
b) displasia
c) glándulas cardiales
d) epitelio gástrico

3. Paciente con acalasia es sometido a tratamiento endoscópico o quirurgico, usted le


ha informado al paciente previamente que es posible que una complicación de este
tratamiento es que quede con cierto grado de:
a) reflujo gastroesofágico
b) gastritis
c) odinofagia
d) úlceras gástricas

4. En un paciente con xerostomía presenta las siguientes condiciones, EXCEPTO:


a) Caries
b) Infecciones del oído a repetición
c) Disfagia
d) Erosiones de la mucosa oral

5. A raíz de la infección por Helicobacter pilory, la destrucción de las células


____________ ocasiona que aumente la producción de ácido en el estómago y por
eso se produzca úlceras _____________
a) S / pancreáticas
b) D / duodenales
c) parietales / gástricas
d) G / gástricas

6. Cuando un paciente recibe estímulo autónomo mixto (simpático y parasimpático), el


flujo de saliva:
a) es mayor que frente a un estímulo parasimpático aislado
b) aumenta en relación al basal
c) es menor que frente a un estímulo simpático aislado
d) disminuye en relación al basal

7. El conducto de Stenon, para entrar a la cavidad vestibular, debe atravesar el


músculo:

a) genihioideo
b) buccinador
c) masetero
d) milohioideo

8. Al evaluar a un paciente con parotiditis purulenta (con absceso), usted buscaría el


orificio terminal del conducto de _____________ a la altura de _____________

a) Stenon / la segunda molar superior


b) Stenon / las carúnculas
c) Wharton / el frenillo sublingual
d) Wharton / el piso de la boca

9. Los siguientes mecanismos determinan la patencia del esfínter esofágico inferior y


ayudan a impedir la enfermedad por reflujo gastroesofágico, EXCEPTO:

a) Canales lentos de calcio


b) Plicatura diafragmática
c) Ángulo de Hiss
d) Longitud mayor del esófago
10. La sangre que lleva la vena porta es tipo:

a) venosa
b) mixta
c) arterial

ANGELA- CI4:

1) Con respecto a la microestructura del hígado ¿Cuál de las siguientes alternativas es


correcta?

Respuestas:

a) Los hepatocitos están interconectados por uniones herméticas


b) La célula de Kupffer se encuentra fuera del sinusoide y fagocita células
c) La célula de Ito se encuentra en el espacio de Disse y reserva glucógeno
d) El sinusoide es un capilar fenestrado

2) La pancreatitis aguda puede producirse debido a una obstrucción en el flujo del conducto
de Wirsung, debido a un cálculo biliar que obstruye:

Respuestas:

a) el conducto de Wirsung
b) la ampolla de Vater
c) la papila menor
d) el tercio medio del colédoco

3) En un paciente con una estenosis severa del colédoco por una complicación quirúrgica,
usted esperaría que desarrolle:

Respuestas:

a) el aumento de transaminasas mayor que el de fosfatasa alcalina


b) hiperbilirrubinemia no conjugada
c) la cabeza de Medusa
d) el aumento del tiempo de protrombina

4) Marque lo correcto respecto al acino hepático:

Respuestas:

a) La zona 1 es la más que recibe más sangre


b) La zona 3 se afecta en menos en una deshidratación severa
c) Dos vértices del rombo acinar se constituyen por triadas portales
d) El eje menor lo constituye un eje imaginario entre dos venas centrolobulillares
5) En un paciente cirrótico con encefalopatía hepática ¿cuál de las siguientes alternativas
sustenta la reducción de carnes rojas en la dieta?

Respuestas:

a) Hasta el 80% del colesterol se transforma en sales biliares


b) El factor de crecimiento hepatocitario fomenta la regeneración del hepatocito
c) El amonio se produce principalmente en el intestino
d) La fructosa y la galactosa se convierten en glucosa en el hígado

6) La relajación del esfínter de Oddi se produce directamente por acción de:

Respuestas:

a) la acetilcolina
b) la colecistoquinina (CCK)
c) la sustancia P
d) el péptido vasoactivo intestinal (VIP)

7) Para aumentar la cantidad de bicarbonato en el flujo pancreático es necesario:

Respuestas:

a) inhibir al nervio vago


b) estimular las células S
c) ingerir aminoácidos como triptófano o fenilalanina
d) estimular la secreción de péptido liberador de colecistoquinina (CCK)

8) La bilis que sale de la vesícula biliar tiene como componente principal:

Respuestas:

a) a la bilirrubina
b) al colesterol
c) a los fosfolípidos
d) a los ácidos biliares

9) La presión parcial de oxígeno en la Zona 1 del sinusoide hepático de debe ser


___________ mmHg

Respuestas:

a) 100
b) entre 95 y 45
c) 40
d) menor de 40

10) Los hepatocitos tienen una gran capacidad regenerativa, en parte gracias a las células
madre hepáticas que se localizan en los:
Respuestas:

a) canales de Herring
b) sinusoides hepáticos
c) espacios de Mall
d) espacios de Disse

CI3 - José G

1. Para bloquear la principal fuerza promotora de producción de ácido clorhídrico, lo


ideal es un medicamento que sea bloqueador de la bomba:
a. de potasio
b. de hidrógeno-potasio
c. de cloro asociado a fibrosis quística
d. sodio-potasio

2. Al usar un parasimpaticomimético (agonista colinérgico), usted espera que la saliva


presente:
a. una menor cantidad de cloro
b. una menor cantidad de bicarbonatos
c. una mayor cantidad de sodio
d. Hipertonicidad

3. En la glándula fúndica, se evidencia células madre o stem cells a nivel de:


a. la foveola
b. el fondo
c. el istmo
d. el cuello

4. La vena porta se forma gracias a la unión de la vena mesentérica superior con la


vena:
a. mesentérica inferior
b. gástrica izquierda
c. esplénica
d. celiaca

5. En la producción de HCl, la acción de la somatostatina disminuye la accion de:


a. la gastrina
b. las prostaglandinas
c. la histamina
d. la acetilcolina

6. ¿Cuál de las siguientes alternativas es una causa de reflujo gastroesofágico de


contenido ácido?
a. Anillo de Schatzki
b. Diertículo de Zenker
c. Hernia Hiatal
d. Acalasia

7. En caso se produzca la perforación de la cara anterior del estómago, esta


perforación ocasionará _________ producto de la peritonitis química.
a. íleo intestinal
b. mayor secreción de colecistoquinina
c. esófago de Barret
d. úlcera duodenal

8. Dentro de las patologías que producen sangrado en el esófago, la que sangra más
es:
a. la esofagitis por reflujo gastroesofágico
b. el divertículo de Zenker
c. el síndrome de Mallory Weiss
d. el síndrome de Boerhaave
9. La fase intestinal de la secreción gástrica se debe básicamente a la participación de
las células:
a. I del yeyuno
b. S del íleon
c. G del duodeno
d. D del estómago

10. Las células enteroendocrinas en el estómago se localizan en la glándula oxíntica, al


mismo nivel que las células:
a. parietales
b. absortivas
c. principales
d. mucosas
CI 3: io

En las glándulas salivales, el principal lugar de intercambio iónico se da a nivel del:

A. conducto excretor
B. conducto estriado
C. conducto intercalar
D. acino

Al usar atropina en un paciente, usted esperaría:

A. la disminución de gastrina
B. la disminución de secreción gástrica por bloqueo de M3
C. el aumento de histamina
D. la hipersalivación

En un paciente con acalasia, se presentará una disminución de ___________________ en


el esfínter esofágico inferior.

A. acetilcolina
B. sustancia P
C. péptido intestinal vasoactivo
D. colecistoquinina

En cuanto a las sustancias secretadas por el estómago ¿Cuál de las siguientes sustancias estimula la
liberación de pepsinógeno?

A. Pepsinógeno
B. Secretina
C. Colecistoqunina
D. Gastrina

Las células enteroendocrinas en el estómago se localizan en la glándula oxíntica, al mismo nivel que
las células

A. parietales
B. principales
C. mucosas
D. absortivas

La fase intestinal de la secreción gástrica se debe básicamente a la participación de las


células

A. G del duodeno
B. I del yeyuno
C. S del íleon
D. D del estómago
El conducto de Stenon, para entrar a la cavidad vestibular, debe atravesar el músculo:

A. genihioideo
B. buccinador
C. milohioideo
D. masetero

Paciente con acalasia es sometido a tratamiento endoscópico o quirúrgico, usted le ha


informado al paciente previamente que es posible que una complicación de este tratamiento
es que quede con cierto grado de:

A. reflujo gastroesofágico
B. gastritis
C. odinofagia
D. úlceras gástricas

La sangre que lleva la vena porta es tipo:

A. mixta
B. arterial
C. venosa

Los pliegues gástricos gruesos son prácticamente inexistentes a nivel de:

A. el cuerpo
B. el fondo
C. la incisura angularis
D. el antro

En la producción de HCl, la acción de la somatostatina disminuye la accion de:

A. las prostaglandinas
B. la histamina
C. la acetilcolina
D. la gastrina

En el estómago se secretan las siguientes sustancias, EXCEPTO:

A. grelina
B. motilina
C. somatostatina
D. gastrina

La vena porta se forma gracias a la unión de la vena mesentérica superior con la vena:

A. mesentérica inferior
B. esplénica
C. celiaca
D. gástrica izquierda
CI3 - PEQUE

La saliva siempre será hipotónica debido a:

A. la mayor permeabilidad al sodio en los conductos


B. el estímulo del sistema parasimpático
C. la impermeabilidad de los conductos al agua
D. la ausencia de conductor estriado

En la producción de HCl, la acción de la somatostatina disminuye la acción de:

A. la histamina
B. las prostaglandinas
C. la gastrina
D. la acetilcolina

Un paciente con polimiositis posee alteración en la regulación del mecanismo de la


deglución; por eso hay que considerar la deglución de la saliva, pues a diario se produce
_______mL

A. 300
B. 500
C. 100
D. 1000

Al evaluar a un paciente con parotiditis purulenta (con absceso), usted buscaría el orificio
terminal del conducto de _____________ a la altura de _____________

A. Wharton / el piso de la boca


B. Stenon / la segunda molar superior
C. Wharton / el frenillo sublingual
D. Stenon / las carúnculas

En un paciente con acalasia, se presentará una disminución de ___________________ en


el esfínter esofágico inferior.

A. sustancia P
B. acetilcolina
C. colecistoquinina
D. péptido intestinal vasoactivo

En pacientes con hernia hiatal, la constante inflamación del esófago, puede aparecer
_______________ como complicación debida a la fibrosis por inflamación crónica.

A. anillo de Schatzki
B. esófago de Barret
C. divertículo de Zenker
D. reflujo gastroesofágico

Los pliegues gástricos gruesos son prácticamente inexistentes a nivel de:


A. el cuerpo
B. la incisura angularis
C. el fondo
D. el antro

Paciente con acalasia es sometido a tratamiento endoscópico o quirúrgico, usted le ha


informado al paciente previamente que es posible que una complicación de este tratamiento
es que quede con cierto grado de:

A. gastritis
B. úlceras gástricas
C. odinofagia
D. reflujo gastroesofágico

¿Cuál de las siguientes alternativas es una causa de reflujo gastroesofágico de contenido


ácido?

A. Acalasia
B. Diertículo de Zenke
C. Anillo de Schatzki
D. Hernia Hiatal

En las glándulas salivales, el principal lugar de intercambio iónico se da a nivel del:

A. conducto intercalar
B. conducto excretor
C. conducto estriado
D. acino

CI4 - Francisco

En un paciente con cirrosis hepática avanzada se encuentra tendencia a:

la hipoglicemia

En un paciente con infección por SARS-CoV-2 con compromiso severo e ingresado en la


unidad de cuidados intensivos (UCI), debido a la tormenta de citoquinas y a la desregulación
inmune ¿Cuál de las siguientes alternativas estará elevada en sangre al evaluar el perfil
hepático?

transaminasas

Para poder absorber al torrente sanguíneo todos los carbohidratos del azúcar común (de
mesa) es necesario usar únicamente los transportadores:

GLUT2, SGLT1, GLUT5

En la regulación de la glucosa sérica participa el hígado ¿Cuál de las siguientes alternativas


evalúa mejor esta función hepática?
Glucosa sérica en ayunas

En un paciente con intoxicación por órganos fosforados, la acción de la colecistoquinina


(CCK) está bloqueada a nivel de:

el esfínter de Oddi

En el hígado, el aumento de la resistencia vascular en los sinusoides hepáticos ocasionará:

salida de plasma hacia el intersticio

En un paciente cirrótico con encefalopatía hepática ¿cuál de las siguientes alternativas


sustenta la reducción de carnes rojas en la dieta?

El amonio se produce principalmente en el intestino

¿Cuál de las siguientes alternativas es correcta sobre la estructura hepática?

El flujo biliar en el lobulillo hepático es centrífugo

Las siguientes sustancias son secretadas por el páncreas, EXCEPTO:

Tripsina

Un paso importante para la poder absorber los lípidos es la emulsificación de las grasas.
¿Cuál de las siguientes alternativas es el principal factor encargado de dicha emulsificación?

Ácidos biliares

CI 4 : Clau A.

CI 4: Sifu

La presión parcial de oxígeno en la Zona 1 del sinusoide hepático de debe ser


___________ mmHg

● entre 95 y 45
● 40
● 100
● menor de 40

La bilis que sale de la vesícula biliar tiene como componente principal:

● a los ácidos biliares


● al colesterol
● a los fosfolípidos
● a la bilirrubina
En un paciente con carcinoma de páncreas, el tumor ha invadido la unión entre la
venas esplénica y mesentérica superior; eso quiere decir que estamos seguros que
el tumor se encuentra a nivel del ________ del páncreas.

● cola
● cabeza
● cuello
● cuerpo

En un paciente con una estenosis severa del colédoco por una complicación
quirúrgica, usted esperaría que desarrolle:

Respuestas:

● el aumento de transaminasas mayor que el de fosfatasa alcalina


● el aumento del tiempo de protrombina
● la cabeza de Medusa
● hiperbilirrubinemia no conjugada

Para aumentar la cantidad de bicarbonato en el flujo pancreático es necesario:

estimular las células S

ingerir aminoácidos como triptófano o fenilalanina

inhibir al nervio vago

estimular la secreción de péptido liberador de colecistoquinina (CCK)

Con respecto a la microestructura del hígado ¿Cuál de las siguientes alternativas es


correcta?

La célula de Kupffer se encuentra fuera del sinusoide y fagocita células

Los hepatocitos están interconectados por uniones hermética

El sinusoide es un capilar fenestrado

La célula de Ito se encuentra en el espacio de Disse y reserva glucógeno


En el hígado, el aumento de la resistencia vascular en los sinusoides hepáticos
ocasionará:

aumento del flujo hacia la vena porta

aumento de la presión de llenado vesicular

aumento del flujo hacia la vena cava superior

salida de plasma hacia el intersticio

En el hígado, el aumento de la resistencia vascular en los sinusoides hepáticos


ocasionará:

● salida de plasma hacia el intersticio


● aumento del flujo hacia la vena porta
● aumento de la presión de llenado vesicular
● aumento del flujo hacia la vena cava superior

En un recién nacido menor de 24 horas con atresia biliar, se encuentra elevación de


la:

● bilirrubina directa
● hemoglobina
● bilirrubina indirecta
● alanina aminotransferasa (ALT)

En un paciente con intoxicación por órganos fosforados, la acción de la


colecistoquinina (CCK) está bloqueada a nivel de:

● el esfínter de Oddi
● la célula parietal
● la vesícula biliar
● el sistema nervioso central

¿Cuál de las siguientes alternativas es una comunicación entre el tejido hepático y la


vesícula biliar?

Válvula espiral de Heister

Conducto de Lushka

Senos de Rokitansky-Aschoff
Divertículos vesiculares

CI4 - Francisco

En un paciente con cirrosis hepática avanzada se encuentra tendencia a:

● la disminución de bilirrubina hidrosoluble


● la hiperglucemia
● la hipoglucemia
● el acortamiento del tiempo de protrombina

En un paciente con infección por SARS-CoV-2 con compromiso severo e ingresado


en la unidad de cuidados intensivos (UCI), debido a la tormenta de citoquinas y a la
desregulación inmune ¿Cuál de las siguientes alternativas estará elevada en sangre
al evaluar el perfil hepático?

● transaminasas
● amilasas
● bilirrubinas
● fosfatasas

Para poder absorber al torrente sanguíneo todos los carbohidratos del azúcar común
(de mesa) es necesario usar únicamente los transportadores:

● GLUT5, SGLT1
● GLUT2, GLUT5
● SGLT1, GLUT2
● GLUT2, SGLT1, GLUT5

En la regulación de la glucosa sérica participa el hígado ¿Cuál de las siguientes


alternativas evalúa mejor esta función hepática?

● Péptido C en sangre
● Glucosa sérica en ayunas
● Glucosa en orina
● Tolerancia oral a la glucosa

En un paciente cirrótico con encefalopatía hepática ¿cuál de las siguientes


alternativas sustenta la reducción de carnes rojas en la dieta?

● La fructosa y la galactosa se convierten en glucosa en el hígado


● Hasta el 80% del colesterol se transforma en sales biliares
● El factor de crecimiento hepatocitario fomenta la regeneración del hepatocito
● El amonio se produce principalmente en el intestino

¿Cuál de las siguientes alternativas es correcta sobre la estructura hepática?

● El flujo biliar en el lobulillo hepático es centrífugo


● Los colangiocitos producen bilis
● El flujo sinusoidal en el lobulillo hepático es de adentro hacia afuera
● En la triada portal, se encuentra la vena derivada de la suprahepática

Las siguientes sustancias son secretadas por el páncreas, EXCEPTO:

● Lipasa
● Péptido Monitor
● Nucleasas
● Tripsina

Un paso importante para la poder absorber los lípidos es la emulsificación de las


grasas. ¿Cuál de las siguientes alternativas es el principal factor encargado de dicha
emulsificación?

Colesterol

Fosfolípidos

Ácidos biliares

Bilirrubina

En un paciente con una estenosis severa del colédoco por una complicación quirúrgica, usted
esperaría que desarrolle --- aumento de tiempo de protrombina

La vía biliar extrahepática se encuentra ubicada dentro del ligamento --- hepatoduodenal

Un recién nacido con enfermedad congénita, tiene una mutación de la proteína MRP2 encargada del
transporte de la bilirrubina conjugada hacia el interior del canalículo. Antes de las 24 horas de nacido
presenta ictericia y además presentará con mayor probabilidad ----- coluria

La bilis que sale de la vesícula biliar tiene como componente principal ---- a los ácidos biliares

Para aumentar la cantidad de bicarbonato en el flujo pancreático es necesario ----- estimular las
células s

Los hepatocitos tienen una gran capacidad regenerativa, en parte gracias a las células madre
hepáticas que se localizan en los ---- canales de Herring
En el síndrome de Mirizzi, el paciente tiene cálculos en la vesícula biliar; pero se obstruye el conducto
hepático común debido a que un cálculo se ubica y crece de tamaño en ----- La bolsa de Hartmann

¿Cuál de las siguientes alternativas es correcta sobre la estructura hepática? ---- El flujo biliar en el
lobulillo hepático es centrífugo

La relajación del esfínter de Oddi se produce directamente por acción de ----- VIP

En la anemia perniciosa, con atrofia gástrica marcada, usted esperaría encontrar disminución en la
absorción de ___________ a nivel de ________ ----- vitamina B12 / íleon distal

Dentro de las patologías que producen sangrado en el esófago, la que sangra más es ----- el
síndrome de Boerhaave

En la glándula fúndica, se evidencia células madre o stem cells a nivel de ----- el istmo

Al usar atropina en un paciente, usted esperaría --- la disminución de secreción gástrica por bloqueo
de M3

La comunicación entre la irrigación gástrica y la esofágica depende de una rama de la arteria ----
Tronco celíaco

En fases iniciales, la infección por Helicobacter pylori genera disminución del pH gástrico debido a ----
- la disminución de somatostatina

En un paciente que usa AINEs a altas dosis por artritis reumatoide, usted esperaría que presente
erosiones y úlceras gástricas debido a ----- la disminución de irrigación en la mucosa

A raíz de la infección por Helicobacter pilory, la destrucción de las células ____________ ocasiona
que aumente la producción de ácido en el estómago y por ello se produzca úlceras ____________ ---
-- D/duodenales

1. El agua se absorbe en yeyuno, íleon y colon y se excreta en las


heces. Señale en orden descendente la cantidad de agua absorbida
o excretada en los siguientes elementos.
A) Colon, yeyuno, íleon, heces.
B) Heces, colon, íleon, yeyuno.
C) Yeyuno, íleon, colon, heces.
D) Colon, íleon, yeyuno, heces.
E) Heces, yeyuno, íleon, colon.

2. Después de un desastre natural en Haití, surgió un brote de cólera


en personas desplazadas que vivían en tiendas de campamentos.
Los sujetos afectados presentaron graves síntomas diarreicos:
¿por cuál de los siguientes cambios del transporte intestinal
aparecieron?
A) Aumentan el cotransporte de sodio-potasio en el intestino
delgado.
B) Incrementan la secreción de potasio en el colon.
C) Inhiben la absorción de potasio en las criptas de Lieberkühn.
D) Aumentan la absorción de sodio en el intestino delgado.
E) Incrementan la secreción de cloruro hacia la luz intestinal.
3. Un varón de 50 años acudió a su médico y le señaló que tenía
dolor epigástrico intenso, pirosis frecuentes y pérdida identifi cada
de 9.5 kg en un lapso de 6 meses. Según él, no tuvo alivio alguno
con productos que se adquirían sin receta como antihistamínicos
contra el receptor H2. Fue referido a un gastroenterólogo y en la
endoscopia gastroduodenal se identifi caron erosiones y úlceras en
la porción proximal del duodeno y una mayor producción de
ácido gástrico con el sujeto en ayunas. El paciente muy
probablemente tiene un tumor que secreta: ¿cuáles de las
hormonas siguientes?
A) Secretina.
B) Somatostatina.
C) Motilina.
D) Gastrina.
E) Colecistocinina.

4. ¿Cuál de los siguientes tiene el pH más alto?


A) Jugo gástrico.
B) Contenido de la luz colónica.
C) Jugo pancreático.
D) Saliva.
E) Contenido de las glándulas intestinales.

5. Una mujer de 60 años es sometida a pancreatectomía total, por la


presencia de una neoplasia. De los resultados siguiente: ¿cuál no se
esperaría después de que se recuperara de la operación?
A) Esteatorrea.
B) Hiperglucemia.
C) Acidosis metabólica.
D) Incremento de peso.
E) Menor absorción de aminoácidos.

1 La absorción máxima de ácidos grasos de cadena corta producidos


por las bacterias ocurre en:
A) estómago.
B) duodeno.
C) yeyuno.
D) íleon.
E) colon.

2. Una mujer premenopáusica físicamente activa solicitó orientación


de su médico familiar en cuanto a las medidas a tomar para
asegurar la disponibilidad adecuada de calcio de alimentos, para
afianzar su salud ósea en etapas ulteriores de la vida. De los
siguientes componentes de la alimentación: ¿cuál debe
incrementar la captación de calcio?
A) Proteínas.
B) Oxalatos.
C) Hierro.
D) Vitamina D.
E) Sodio.
3. Un niño que manifiesta ausencia congénita de enterocinasa
presentaría una disminución habitual en:
A) frecuencia de pancreatitis.
B) absorción de glucosa.
C) reabsorción de ácidos biliares.
D) pH gástrico.
E) asimilación de proteínas.

4. En la enfermedad de Hartnup (un defecto del transporte de


aminoácidos neutrales), los pacientes no presentan deficiencia de
estos aminoácidos gracias a la actividad de
A) PepT1.
B) peptidasas del borde en cepillo.
C) Na, K-ATPasa.
D) regulador de la conductancia transmembrana de la fibrosis
quística (CFTR).
E) tripsina.

5. Un recién nacido es llevado al pediatra a causa de diarrea grave


que se agrava al alimentarse. Los síntomas disminuyen cuando los
nutrientes se administran por vía intravenosa. Lo más probable es
que el niño tenga una mutación en cuál de los siguientes
transportadores intestinales:
A) Na, K-ATPasa.
B) NHE3.
C) SGLT1.
D) H+, K+-ATPasa.
E) NKCC1.

1. En los lactantes, la defecación suele ocurrir después de una comida.


La causa de las contracciones colónicas en esta situación es
A) la histamina.
B) el aumento de las concentraciones de CCK en la circulación
sanguínea.
C) el reflejo gastrocólico.
D) el aumento de las concentraciones de somatostatina en la
circulación sanguínea.
E) el reflejo enterogástrico.

2. Los síntomas del síndrome de vaciamiento rápido (molestia después


de las comidas en los pacientes con derivaciones intestinales como
la anastomosis gastroyeyunal) son causados en parte por:
A) aumento de la presión arterial.
B) aumento de la secreción de glucagon.
C) aumento de la secreción de CCK.
D) hipoglucemia.
E) hiperglucemia.

3. Las presiones gástricas raras veces aumentan por arriba de los


niveles que abren el esfínter esofágico inferior, aun cuando el estómago esté lleno de comida. ¿A
cuál de los siguientes procesos
se debe este fenómeno?
A) Peristaltismo.
B) Reflejo gastroileal.
C) Segmentación.
D) Estimulación del centro del vómito.
E) Relajación receptiva.

4. ¿Cuál de las siguientes sustancias detona el complejo motor


migratorio?
A) Motilina.
B) NO.
C) CCK.
D) Somatostatina.
E) Secretina.
5. Un paciente es referido al gastroenterólogo porque tiene dificultad
persistente para la deglución. En el estudio endoscópico se
advierte que no se abre totalmente el esfínter esofágico inferior
conforme el bolo llega a él, por lo que se hace el diagnóstico de
acalasia.
En la exploración o en las biopsias estudiadas de muestras de la
región esfinteriana: ¿en cuál de los elementos siguientes cabría
esperar disminución?
A) Peristaltismo esofágico.
B) Expresión de óxido nítrico sintasa neuronal.
C) Receptores de acetilcolina.
D) Liberación de sustancia P.
E) Contracción del diafragma crural.

1. Un paciente que sufre colitis ulcerosa grave es sometido a


colectomía total con elaboración de un estoma. Después de la
recuperación completa de la operación y al comparar su estado
posoperatorio con el que tenía antes de la operación: ¿cuál de los
siguientes factores cabría esperar que disminuya?
A) Capacidad de absorber lípidos.
B) Capacidad de coagular la sangre.
C) Concentraciones circulantes de ácidos biliares conjugados.
D) Urea en orina.
E) Urobilinógeno en orina.

2. Una cirujana estudia nuevos métodos de trasplante de hígado y


realiza una hepatectomía completa en un animal de
experimentación. Antes de injertar el hígado donado cabría
esperar incremento en la concentración sanguínea de:
A) glucosa.
B) fibrinógeno.
C) 25-hidroxicolecalciferol.
D) bilirrubina conjugada.
E) estrógenos.

3. ¿Cuál de los siguientes tipos de células protege contra la septicemia


consecutiva a la translocación de bacterias intestinales?
A) Célula estrellada hepática.
B) Colangiocito.
C) Célula de Kupffer.
D) Hepatocito.
E) Célula epitelial de la vesícula biliar.

4. El citocromo P450 (CYP) se expresa de manera extraordinaria en


los hepatocitos. ¿En cuál de las siguientes no desempeña una
función importante?
A) Formación de ácidos biliares.
B) Carcinogénesis.
C) Formación de hormonas esteroideas.
D) Desintoxicación de fármacos.
E) Síntesis de glucógeno.

5. Una mujer de 40 años acude con su médico familiar y le señala que


ha tenido dolor abdominal episódico e intenso, en particular
fuerte después de ingerir una comida grasosa. Un estudio
imagenológico señala que su vesícula biliar muestra dilatación
aguda y se hace el diagnóstico de colelitiasis. ¿En qué sitio
anatómico un cálculo vesicular agravaría el riesgo de
pancreatitis?
A) Conducto hepático izquierdo.
B) Conducto hepático derecho.
C) Conducto cístico.
D) Colédoco.
E) Esfínter de Oddi.

6. La bilis de la vesícula biliar, en comparación con la que está en el


hígado, contiene una menor concentración de:
A) Ácidos biliares.
B) Iones de sodio.
C) Protones.
D) Glucosa.
E) Protones.

1. Un niño de 2 años es llevado a la consulta por diarrea persistente y edema de las


extremidades, además falta de crecimiento y desarrollo en relación a su edad. Los análisis de
sangre revelan que tiene concentración plasmática baja de proteínas (hipoproteinemia).
Durante la endoscopía duodenal, se coloca colecistokinina (CCK) endovenosa y se recoge
muestras del líquido duodenal; el resultado del líquido confirma incapacidad para hidrolizar
proteínas a un pH neutro, esta situación mejora al añadir una pequeña cantidad de tripsina.
El paciente probablemente esté sufriendo la falta congénita de

d. Enterocinasa

2. Experimentalmente se incrementa la velocidad de la secreción salival con una sustancia, el


análisis de la composición de esta saliva obtenida se espera encontrar
c. Disminución de concentración de potasio

3. Paciente varón de 46 años soltero, consulta por odinofagia y bajo de peso, tiene antecedente
de tuberculosis desde hace 3 meses y es fumador crónico (10 cigarrillos por día); al evaluar
la cavidad oral se identifica lesión blanquecina en el dorso de la lengua y paladar blando, las
lesiones se desprenden con el baja lengua dejando una base eritematosa. Esta lesión
corresponde probablemente a
b. Candidiasis oral

4. Minero de 32 años de edad, que acude a centro de salud por presentar de forma progresiva
desde hace 1 año dificultad para ingerir alimentos sólidos y luego líquidos; refiere
regurgitaciones alimentarias y marcada pérdida de peso (15 kilos). Radiografia baritada de
esófago como se muestra en la figura. El presente caso se explica por
Relajación incompleta del esfínter esofágico inferior

5. Paciente mujer de 35 años acude a consulta por sensación de sequedad y lesiones en


cavidad oral. Al examen se observa atrofia de la mucosa, fisuras y úlceras; nota además
sequedad e irritación de la córnea y aumento del tamaño de las glándulas parotídeas. Su
diagnóstico más probable es artritis reumatoide; el hallazgo más probable en una biopsia de
glándula parótida es
b. Gran infiltración de linfocitos y células plasmáticas

6. Un paciente con anemia acude con su médico quejándose de episodios frecuentes de


gastroenteritis. Un análisis de sangre revela anticuerpos circulantes dirigidos contra células
parietales gástricas. Su anemia es atribuible a la hiposecreción de
Factor intrínseco

7. Un niño de cuatro años de edad es llevado a la consulta por cuadros diarreicos frecuentes
caracterizados por heces pálidas, voluminosas y fétidas, presenta bajo peso y talla. Se mide
la concentración de cloruro en el sudor y se encuentra que sus valores son muy elevados. La
alteración más importante a nivel de células ductales del páncreas tiene relación directa con
la conductancia de
d. Cloro
8. Una mujer de 50 años de edad que sufrió durante varios años resequedad de los ojos debida
a producción inadecuada de lágrimas es enviada con un gastroenterólogo para evaluación de
pirosis crónica. El examen endoscópico revela erosiones y tejido cicatrizal en la parte distal
del esófago justo por arriba del esfínter esofágico inferior. Las lesiones pueden atribuirse a la
disminución de uno de los siguientes componentes salivales:
a. Bicarbonato
9. En el reflejo peristáltico del intestino delgado, uno de los siguientes eventos sucede en la
porción oral del bolo alimenticio
d. Acción de acetilcolina en el músculo circular
10. Experimentalmente se coloca una dosis alta de secretina en la luz intestinal duodenal; como
consecuencia de esto, en el jugo pancreático de la misma luz intestinal se observa la
disminución de la concentración de
b. Cl-
11. Un varón de 58 años de edad con enfermedad de Crohn severo fue sometido a una
resección ileal. Después de la cirugía este paciente padecerá de esteatorrea, esto se explica
porque
La micelas no pueden formarse
12. La toxina del Vibrio cholerae causa diarrea debido a
b. El Incremento de la secreción de cloro por las células de la cripta intestinal
13. ¿Cuál de las siguientes alternativas es una característica de la secreción exocrina del
páncreas?
a. Tiene una baja concentración de Cl- respecto al plasma
14. Una madre lleva a su hijo de dos años de edad a la sala de urgencias, estresada porque el
niño deglutió una moneda de 10 céntimos mientras la familia cenaba en un restaurante. El
médico observa mediante fluoroscopía que la moneda se halla en el estómago y asegura a la
madre que la moneda se eliminará con las heces. El médico recomienda utilizar la respuesta
fisiológica que permitirá la evacuación de la moneda del estómago al intestino
b. Son los movimientos de mezcla y trituración
c. Es provocada por el ayuno

15. La composición de la bilis es modificada conforme fluye por los conductillos biliares. Durante
este tránsito se espera que aumente la concentración de
a. Ig A
c. Monómeros de ácido biliar
16. Se mide experimentalmente el contenido gástrico de dos personas. La persona “A” tiene alto
contenido de grasa y la persona “B” tiene un contenido hipertónico ¿Cuál de las siguientes es
correcto respecto al vaciamiento gástrico?
Hay ralentización del vaciado gástrico en ambos casos
17. Un paciente varón de 18 años de edad acude al médico para sus exámenes de rutina. Sus
resultados de laboratorio muestran un valor de bilirrubina sérica de 4 mg/dl y una bilirrubina
directa de 0,3 mg/dl. Las pruebas de función hepática son normales. La alteración que
explica mejor este caso es por la deficiencia de
b. Glucuronil transferasa
18. Un hombre de 57 años de edad es llevado a urgencias con hematemesis masiva rojo
brillante, a su llegada se halla inconciente con PA: 80/40 mm Hg y FC: 124 lat/min. Luce
ictérico con presencia de “arañas vasculares en el tórax anterior y extremidades”, abdomen
distendido con signo de oleada positiva. Se encuentra esplenomegalia y pérdida de la masa
muscular en extremidades. La anastomosis vascular responsable del sangrado en este
paciente es
b. Vena gástrica izquierda y vena ácigos
19. Un estudiante de medicina está comiendo un plato de comida a base de champiñones,
espárrago y salsa de soya. El sabor umami contenido en todos estos alimentos actúa a nivel
de los botones gustativos estimulando
Un receptor acoplado a proteína G
20. Un hombre de 22 años de edad se presenta al médico con una historia de 1 año de
evolución caracterizado por dolor recurrente en fosa iliaca derecha y diarrea. Manifiesta
además pérdida de peso de 8 kg durante este periodo. La colonoscopía revela múltiples
lesiones en el ileon terminal y colon. La biopsia de estas lesiones revela engrosamiento,
inflamación y ulceración de la mucosa. El diagnóstico más probable en este caso es
b. Enfermedad de Crohn
21. Una de las funciones del músculo señalado es
d. Deprime la mandíbula cuando el hioides está fijo

22. Varón de 61 años que consulta por dolor retro esternal intenso desde hace 6 horas y
después de vómitos intensos y repetidos; al examen se observa disnea, cianosis, hipotensión
y signos clínicos de shock. La radiografía simple de tórax muestra neumomediastino. El
líquido en el espacio pleural aspirado tiene alta concentración de amilasa. ¿Cuál de las
siguientes alternativas puede explicar este cuadro clínico?
b. Rotura espontánea de esófago
23. La secreción del ácido en la célula parietal gástrica se lleva a cabo por una ATPasa
especifica que intercambia hidrogeniones (H+) del citosol por
d. K+
24. En condiciones normales el ingreso de 600 ml de líquido es el estómago provoca un
aumento de presión intragástrica de unos 12 cm de H2O. Después de una vagotomía (corte
del nervio vago) es de esperar que el ingreso del mismo volumen de líquido provoque lo
siguiente:
Un aumento mayor de la presión
25. Una paciente de 30 años de edad es sometida a una cirugía en oído medio derecho por un
problema de otoesclerosis. Luego de la cirugía refiere alteración en la percepción de sabores.
Al evaluar el caso usted esperaría encontrar
b. Alteración en la sensación del gusto en los dos tercios anteriores de la lengua
d. Sensación del dolor, tacto y temperatura conservada en toda la lengua

26. ¿Cuál de las siguientes alterativas es correcta?


c. Las sales biliares desconjugadas son absorbidas preferentemente en el colon
27. En un paciente de 45 años de edad con colestasis biliar, se encuentra una elevación de los
niveles sanguíneos de fosfatasa alcalina hasta 3 veces la cifra normal. ¿Cuál de las
siguientes alternativas estará también elevada como evidencia del daño de la vía biliar?
Gamma glutamil transpeptidasa
28. Revisando la angiografía de un hombre de 70 años en estudio por aneurisma de aorta
abdominal el radiólogo informa de la presencia de una oclusión completa de la arteria
mesentérica inferior. El paciente se encuentra completamente asintomático. ¿Cuál de las
siguientes arterias se anastomosa a la sistema arterial de la mesentérica inferior?
Cólica media
29. Un varón de 75 años ingresa al consultorio por presentar ictericia marcada de piel y las
escleras. El estudio del paciente mostró que presentaba un tumor que obstruía la totalidad
del conducto hepático común. ¿Cuál de las siguientes estructuras se encontrará dilatada en
este paciente?
Conductos de Hering
30. Respecto a la siguiente imagen que representa una estructura de la mucosa gástrica, la
estructura con número ……….. produce

2 / pepsinógeno

PREGUNTAS PA TIPEAR

Secreta grandes cantidades de bicarbonato y agua para neutralizar la acidez del quimo:

● Páncreas

La sacarosa se desdobla en:

● Glucosa y fructosa
Enfermedad en que no hay relajación del cardias como respuesta de deglución y hay falta de
peristaltismo en el esófago:

● Acalasia

Histológicamente la mucosa del tubo digestivo presenta las siguientes regiones excepto

● Muscular
● Epitelio
● Corion
● Lámina propia
● Muscularis mucosae

Es la porción principal y de mayor tamaño del estómago, en la cual se forma el quimo:

● Cuerpo

Entre las capas musculares se encuentran grupos de células ganglionares y haces de fibras
nerviosas amielínicas, que en conjunto representan:

● Plexo mientérico de auerbach

A nivel del íleon, se encuentran agregaciones permanentes de nódulos linfoides llamados:

● Placas de Peyer

En relación al intestino grueso, la capa mucosa está constituida por tejido conectivo laxo y mesotelio,
presenta depósitos de grasa llamados:

● Apéndices epiploicos

En que porción del tubo digestivo se sintetiza vitamina K y B

● Colon

Contienen enzimas, excepto:

● Saliva
● Bilis
● Jugo Pancreático
● Jugo intestinal
● Jugo gástrico

Producen el HCl en el estómago

● Células parietales

No es parte del duodeno:

● Descendente
● Ileal
● Ascendente
● Horizontal
● Superior
Enzima proteolítica contenida en el jugo pancreático

● Tripsina

Conducto de drenaje de la glándula parótida en la boca

● Stenon

La unidad anatomofisiológica del hígado es

● Estroma
● Hepatocito
● Lobulillo hepático
● Hilio del hígado
● Parénquima

Los ameloblastos se localizan en:

● Esmalte

Contiene glucoproteínas que permite la lubricación de la boca:

● Secreción mucosa de la saliva

El bolo alimenticio pasa hacia la faringe a través de:

● Istmo de las fauces

El conducto colédoco se forma por la convergencia de los conductos:

A. Cístico - Wirsung
B. Santorini - hepático
C. Cístico - Santorini
D. Cístico - hepático
E. Cardiaco - hepático

Las glándulas parótidas y submaxilares respectivamente presentan como conductos:

A. Rivinus y Stenon
B. Wharton y Stenon
C. Stenon y Rivinus
D. Stenon y Wharton
E. Wharton y Rivinus

Membrana epitelial que sostiene a casi todas las vísceras del abdomen
A. Mesenterio
B. Fundus
C. Peritoneo
D. Epitelio
E. Mucosa

En el mecanismo de formación de HCl intervienen los siguientes elementos químicos:

A. H2CO3
B. CO2
C. H2O
D. H+
E. T.A.

La gingivorragia se produce por deficiencia de vitamina:

A. B
B. D
C. E
D. C
E. A

Fase de secreción del HCl que se activa por saborear

A. Vagal
B. Intestinal
C. Gástrica
D. Neurógena
E. AyD

Producen colecistoquinina:

A. Células Paneth
B. Células argentafines
C. Células APUD
D. Brunner
E. N.A.

El tripsinógeno es activado por:

A. Enterocinasa
B. Quimiotripsina
C. Tripsina
D. Elastasa
E. A+B

Con respecto a la microestructura del hígado ¿Cuál de las siguientes alternativas es correcta?

- Los hepatocitos están interconectados por uniones herméticas


- La célula de Ito se encuentra en el espacio de Disse y reserva glucógeno
- El sinusoide es un capilar fenestrado
- La célula de Kupffer se encuentra fuera del sinusoide y fagocita células
En un paciente con una estenosis severa del colédoco por una complicación quirúrgica, usted esperaría
que desarrolle:

- la cabeza de Medusa
- el aumento de transaminasas mayor que el de fosfatasa alcalina
- hiperbilirrubinemia no conjugada
- el aumento del tiempo de protrombina

Marque lo correcto respecto al acino hepático:

- Dos vértices del rombo acinar se constituyen por triadas portales


- El eje menor lo constituye un eje imaginario entre dos venas centrolobulillares
- La zona 1 es la más que recibe más sangre
- La zona 3 se afecta en menos en una deshidratación severa

Los hepatocitos tienen una gran capacidad regenerativa, en parte gracias a las células madre hepáticas
que se localizan en los:

- espacios de Disse
- sinusoides hepáticos
- canales de Herring
- espacios de Mall

En un paciente con infección por SARS-CoV-2 con compromiso severo e ingresado en la unidad de
cuidados intensivos (UCI), debido a la tormenta de citoquinas y a la desregulación inmune ¿Cuál de
las siguientes alternativas estará elevada en sangre al evaluar el perfil hepático?

- transaminasas
- fosfatasas
- amilasas
- bilirrubinas

La presión parcial de oxígeno en la Zona 1 del sinusoide hepático de debe ser ___________ mmHg

- menor de 40
- 40
- 100

- entre 95 y 45

En un paciente cirrótico con encefalopatía hepática ¿cuál de las siguientes alternativas sustenta la
reducción de carnes rojas en la dieta?

- El amonio se produce principalmente en el intestino


- Hasta el 80% del colesterol se transforma en sales biliares
- La fructosa y la galactosa se convierten en glucosa en el hígado
- El factor de crecimiento hepatocitario fomenta la regeneración del hepatocito

En el hígado, el aumento de la resistencia vascular en los sinusoides hepáticos ocasionará:

- aumento del flujo hacia la vena porta


- aumento de la presión de llenado vesicular
- aumento del flujo hacia la vena cava superior
- salida de plasma hacia el intersticio
En un paciente con cirrosis hepática se desarrollará ascitis debido a los siguientes mecanismos,
EXCEPTO:

- Hipoalbuminemia
- Estrechamiento de la porta
- Hiperflujo portal
- Aumento de reabsorción renal de sodio

El efecto de un medicamento colerético se evidencia por:

- el aumento de formación de micelas


- el aumento de secreción biliar
- la mayor producción de colesterol en la bilis
- la disminución de absorción de sales biliares

CI4 - PEQUE

Paciente con Lupus Eritematosos que desarrolla hipertensión portal debido a trombosis portal, es
probable que desarrolle várices a nivel de

- canal anal
- recto inferior
- recto superior
- hemorroides externas

¿Cuál de las siguientes alternativas es una comunicación entre el tejido hepático y la vesícula biliar?

- Válvula espiral de Heister


- Conducto de Lushka
- Divertículos vesiculares
- Senos de Rokitansky-Aschoff

En un paciente con cirrosis hepática terminal, qué hormona aumenta la absorción de agua en el colon

- Disminuida
- No influye
- Aumentada
- Normal

En un paciente con cirrosis hepática avanzada se encuentra tendencia a

- la hiperglicemia
- el acortamiento del tiempo de protrombina
- la disminución de bilirrubina hidrosoluble
- la hipoglicemia

Un paso importante para la poder absorber los lípidos es la emulsificación de las grasas. ¿Cuál de las
siguientes alternativas es el principal factor encargado de dicha emulsificación?

- Ácidos biliares
- Bilirrubina
- Colesterol
- Fosfolípidos
Para aumentar la cantidad de bicarbonato en el flujo pancreático es necesario

- inhibir al nervio vago


- estimular la secreción de péptido liberador de colecistoquinina (CCK
- ingerir aminoácidos como triptófano o fenilalanina
- estimular las células S

En un paciente con cirrosis hepática se desarrollará ascitis debido a los siguientes mecanismos,
EXCEPTO:

- Hipoalbuminemia
- Aumento de reabsorción renal de sodio
- Estrechamiento de la porta
- Hiperflujo portal

En el síndrome de Mirizzi, el paciente tiene cálculos en la vesícula biliar; pero se obstruye el conducto
hepático común debido a que un cálculo se ubica y crece de tamaño en:

- la bolsa de Hartmann
- el conducto accesorio de Lushka
- el colédoco
- la válvula espiral de Heister

En un paciente con intoxicación por órganos fosforados, la acción de la colecistoquinina (CCK) está
bloqueada a nivel de:

- el sistema nervioso central


- el esfínter de Oddi
- la vesícula biliar
- la célula parietal

Con respecto a la microestructura del hígado ¿Cuál de las siguientes alternativas es correcta?

- Los hepatocitos están interconectados por uniones herméticas


- El sinusoide es un capilar fenestrado
- La célula de Ito se encuentra en el espacio de Disse y reserva glucógeno
- La célula de Kupffer se encuentra fuera del sinusoide y fagocita células

CI 4 + lo que me mandaron a tiepar - Shiro


En un paciente con cirrosis hepática la cabeza de medusa que aparece en la pared
abdominal, podría desaparecer si al paciente se le:
- esclerosa las venas hemorroides internas
- oblitera el ligamento redondo
- administra antiandrógenos
- oblitera la arteria gástrica izquierda

Para poder absorber al torrente sanguíneo todos los carbohidratos de la lactosa, es


necesario usar únicamente los transportadores:
- SGLT1, GLUT2
- GLUT5, SGLT1
- GLUT2, SGLT1, GLUT5
- GLUT2, GLUT5
Interviene en la digestión intestinal, excepto:
- jugo pancreático
- bilis
- jugo intestinal
- jugo gástrico
- tripsina

De las siguientes enzimas digestivas;


I) Ptialina
II) Pepsina
III) Tripsina
IV) Enteroquinasa
V) Aminopolipeptidasa
son producidas por el estomago:

- I, II, III
- III, IV, V
- I, III, IV
- II, IV
- Solo II

La secretina, hormona secretada por la mucosa duodenal, desempeña varias funciones,


indique la que corresponde:
- Activa la secreción gástrica
- Provoca contracción de las vesícula biliar
- Inhibe la secreción gástrica
- Activa la secreción pancreática
- Inhibe la secreción pancreática

Los sinusoides hepáticos, contienen células que pertenecen al RES con actividad fagocítica,
a estas células se les conoce con el nombre de:
- Yuxta globemerulares
- Kupffer
- Remack
- Podocitos
- Falciformes

El tubo digestivo esta formado por excepto:


- Boca
- intestino grueso
- hígado
- estómago
- intestino delgado

El plexo de Auerbach se encuentra en la capa


- Musculares mucoseae
- Subucosa
- Mucosa
- Muscular
- Serosa

Cual no es función del hígado


- hemocateresis
- secreción de bilis
- hematopoyesis
- anticoagulante
- almacenamiento principalmente de vitaminas A, D y B12

El estómago presenta epitelio


- estratificado plano sin queratina
- cúbico simple
- cilindrico simple no modificado
- cilindrico simple con ribete en cepillo
- escamoso plano

La digestion de los almidones se inicia en _______, por la enzima _______


- estomago - amilasa gastrica
- intestino delgado - amilasa pancreatica
- esofago - alfa amilasa salival
- boca - alfa amilasa salival
- intestino delgado - colon

Respecto a glándulas salivales es cierto, excepto:


- Parótida presenta el conducto de stenon
- Submaxilares secretan el 70% del volumen desaliva
- Sublinguales posee el conducto de Rivinus
- Submaxilares tienen secreción seromucosa
- Parótidas presenta secreción mucosa

El padre de rosa es alcohólico desde hace 20 años y ahora cura el médico por presentar la
piel y escleras amarillentas (ictéricas) el médico le ha dicho que su hígado se encuentra
muy dañado, Entonces qué enfermedad podría presentar
- hepatitis B
- Apendicitis
- gastritis
- cirrosis hepática
- N.A

Las placas de Peyer, se encuentran en:


- duodeno
- yeyuno
- ileon
- colón
- recto
La función de la bilis es:
- emulsificar proteínas
- emulsificar las grasas
- emulsificar carbohidratos
- facilitar la digestión de los aminoácidos
- degradar las proteínas

No es Constituyente del jugo pancreático


- tripsinógeno
- enterocinasa
- amilasa
- carboxipeptidasa
- bicarbonato

Órgano del aparato digestivo que realiza la absorción de aproximadamente el 90% de


sustancias:
- Esofago
- estómago
-
SISTEMA DIGESTIVO (ME154)
EXAMEN PARCIAL 2019 01

Profesores : Alfaro Salazar, Herberth Romulo; Callata Caceres, Gunter; Cayo Quiñe, Alexandra Mariel; Correa Borit,
Jorge Mauricio; Cruz Cutty, Lourdes Marylin; Guzmán Calderón, Gerly Edson; Jáuregui Farfán, Jorge
Jesús; Mayor Zevallos, Otto Alberto; Montoya Suárez, José Luis; Palacios Bazan, Enrique Elias; Robles
Pino, Alexander Anibal; Wong Bravo, Juan Carlos
Sección : Todas las secciones
Duración : 50 minutos.
Indicaciones:
Lea atentamente cada pregunta antes de responder:
- Se prohíbe el uso del celular y cualquier dispositivo electrónico.
- Está prohibido intercambiar materiales.
- Coloque su código de alumno en la tarjeta de respuestas. Si su código contiene una letra reemplácela por un valor
numérico siguiendo la siguiente equivalencia: A=9, B=8, C=7, D=6, E=5, F=4, G=3.
- Traslade sus respuestas a la tarjeta, llenando los círculos de manera completa con lapicero negro o azul. Está
prohibido el llenado con lápiz, lapicero de otro color o con lapicero de tinta borrable.
- Sea cuidadoso en el llenado de la tarjeta de respuestas, pues solo esta tiene validez para la calificación.
- Al terminar su examen avise al docente a cargo, no se levante de su sitio; debe entregar la hoja de respuestas con
la carátula del examen, este cuadernillo de preguntas se lo llevará cada estudiante.

1. ¿Cuál de las glándulas salivales es responsable del mayor porcentaje del volumen de la saliva en condiciones
basales?
a. Parótida
b. Subpalatinas
c. Sublingulaes
d. Submaxilares

2. La lengua está recubierta por epitelio:


a. pseudoestratificado columnar no queratinizado
b. plano estratificado no queratinizado
c. pseudoestratificado columnar ciliado
d. plano estratificado queratinizado

3. El esfínter anal interno tiene musculatura …………… y tiene control ………………….


a. lisa / voluntario
b. lisa / involuntario
c. esquelética / simpático
d. esquelética / parasimpático

4. La arteria aorta proporciona la irrigación al tubo digestivo ¿cuál de las siguientes arterias proporciona la irrigación
al ángulo cólico derecho?
a. Mesentérica superior
b. Mesentérica inferior
c. Frénica inferior
d. Tronco celiaco

5. Paciente de 26 años que le cuenta en su historia clínica que cada vez que almuerza, a los 20 minutos tiene deseo
de defecar. Le comenta que su hijo de 1 mes le pasa lo mismo pero más intenso. Esto se explica por el reflejo
…………………, el cual está ……………..en el paciente.
a. colicoileal / normal
b. colicoileal / alterado
c. gastrocolico / normal
d. gastrocolico / alterado
6. La región del estómago que se comunica con el duodeno se denomina:
a. pilórica
b. cardias
c. cuerpo
d. fórnix

7. Acude a consulta un paciente que fue diagnosticado de ulcera péptica 3 días antes. Luego de múltiples pruebas
diagnósticas, se concluye que el paciente presenta un tumor secretor de gastrina ¿Cuál de las siguientes situaciones
estará incrementada?
a. Distención gástrica
b. Inhibición del vaciado gástrico
c. Secreción de ácido clorhídrico (HCl)
d. Inhibición de la secreción de pepsinógeno

8. En el sistema digestivo, el control del apetito esta dado por un complejo sistema de sustancias y órganos
integradores los cuales regulan la ingesta de alimentos. La ……………… es una sustancia oroxígena y es sintetizada
por el ……………………..
a. leptina / intestino
b. grelina / intestino
c. leptina / estómago
d. grelina / estómago

9. Sobre el control autónomo del sistema digestivo, marque la alternativa correcta:


a. La inervación dada por el sistema simpático es de tipo preganglionar.
b. El sistema parasimpático usa como neurotransmisores a la acetilcolina y la noradrenalina.
c. El nervio vago (par craneal X) le da inervación simpática a la mayoría del sistema digestivo.
d. En el sistema simpático, los nervios responsables hacen una primera sinapsis en ganglios próximos al
órgano a inervar.
e. En la inervación de tipo parasimpático, solo interviene el plexo submucoso, sin embargo, en la de tipo
simpático intervienen tanto el submucoso como el mientérico.

10. Con respecto a la actividad eléctrica del sistema digestivo, marque la alternativa correcta
a. Corresponden a potenciales de acción que están presentes de forma continua y le dan la capacidad de
perístasis autónoma al sistema digestivo.
b. La frecuencia de las ondas lentas no se ve influenciada por la actividad neural ni las hormas
gastrointestinales.
c. En el estómago las ondas lentas se dan en una frecuencia de 6 por minuto.
d. Las ondas lentas son cambios lentos y ondulantes del potencial en reposo.
e. La frecuencia de las ondas lentas va de 6 a 12 ondas por minuto.

11. Ante una lesión del IX par craneal, el músculo……………….. se altera en su función.
a. palatogloso
b. estilofaríngeo
c. palatofaríngeo
d. constrictor superior

12. Un varón de 50 años es sometido a extirpación de duodeno y parte proximal de yeyuno. La pérdida de estímulo
hormonal en el páncreas para la secreción enzimática se explica por la pérdida de las células ……………………
a. Parietales, productoras de factor intrínseco
b. “K” productoras de factor intrínseco
c. “M” productoras de CCK
d. “I” productoras de CCK
13. Respecto al mecanismo de la defecación ¿Cuál de las siguientes afirmaciones es correcta?
a. Se produce contracción refleja del esfínter anal interno
b. Se produce contracción o relajación del esfínter anal externo por señales de la corteza cerebral
c. La presencia de materia fecal en el recto estimula la contracción del sigmoides por los nervios pélvicos
simpáticos
d. En la posición de “cuclillas” el músculo puborectal se halla contraído favoreciendo la evacuación de la
materia fecal

14. Un niño de tres años llega a emergencia con disfagia (dificultad para tragar), salivación y llanto. Se sospecha de
ingesta de cuerpo extraño: moneda en el esófago; al ser evaluado se constata en una radiografía presencia de
cuerpo extraño a nivel de C6 y C7 (6° y 7° vértebra cervical). El cuerpo extraño estará suspendido a nivel del
estrechamiento producido por el ………..
a. cayado aórtico
b. hiato esofágico
c. músculo cricofaríngeo
d. bronquio principal izquierdo

15. En el caso de un paciente con un tumor productor de gastrina, la presencia de úlceras duodenales y erosión de la
mucosa gástrica se debe principalmente a…….
a. la acción paracrina de la gastrina sobre la célula parietal
b. el exceso de HCl por estímulo de receptores CCK-B en la célula parietal
c. la sobre expresión de los receptores “G” para gastrina en la célula parietal
d. el exceso de HCl por estímulo directo de receptores “H” en la célula parietal

16. La onda peristáltica secundaria del esófago se caracteriza por ser originada ………
a. por el plexo de submucoso del esófago
b. por el plexo mientérico del esófago
c. por el reflejo de la deglución
d. durante la masticación

17. Marque lo correcto sobre las ondas lentas en el tubo digestivo


a. No son despolarizaciones
b. Son potenciales de acción subumbrales
c. Se constituyen de despolarizaciones y repolarizaciones
d. Son rítmicas y generadas por el sistema nerviosos autónomo

18. Recién nacido que presenta protrusión de contenidos abdominales los cuales no están cubiertos por peritoneo y
salen de la cavidad abdominal a través de un defecto de la pared. ¿Cómo se denomina a la afección que presenta
este paciente?
a. Onfalocele
b. Atresia biliar
c. Gastrosquisis
d. Divertículo de Meckel

19. Experimentalmente se utiliza atropina (anticolinérgico) para inhibir la secreción de gastrina, sin embargo, la
secreción de esta hormona se sigue dando ante estímulos vagales. Esta situación se explica porque la atropina:
a. no bloquea la acción del péptido GRP
b. solo inhibe la acción del péptido GRP en la célula G
c. inhibe la acción de acetilcolina e histamina en la célula G
d. bloquea parcialmente la bomba de protones en la célula G

20. Niña de 4 días es llevada a la emergencia pediátrica por presentar llanto constante, la madre refiere coloración
azulada de labios al momento de lactar, acompañado de tos persistente y dificultad respiratoria así como
distención abdominal. Se le coloca sonda nasogástrica para alimentación notando que retorna a la cavidad oral en
todos los intentos. ¿Cuál es la anomalía del desarrollo en este caso?
a. Solo fístula traqueo esofágica
b. Fístula traqueo esofágica proximal y distal
c. Atresia esofágica proximal con fístula traqueo esofágica distal
d. Atresia esofágica distal con fístula traqueo esofágica proximal
21. Paciente varón de 36 años es traído a la emergencia luego de sufrir un accidente de tránsito, presenta traumatismos
múltiples en cabeza y tronco. Al examen físico se evidencia hematoma en hemicara izquierda, ligera protrusión y
caída del lado izquierdo del maxilar inferior, por lo que se realiza una tomografía donde se halla una fractura de la
apófisis coronoides del maxilar inferior. ¿Qué músculo está relacionada directamente con esta situación?
a. Masetero
b. Temporal
c. Buccinador
d. Pterigoideo medial

22. Un paciente refiere no percibir algunos sabores. Al examen físico constata alteración del sabor dulce y umami.
¿Cuál de los siguientes nervios estará alterada su función?
a. Cuerda del tímpano (VII par)
b. Lingual (rama del V par)
c. Glosofaríngeo (IX par)
d. Hipogloso (XII par)

23. A los pocos días de nacido, regresa a neonatología un niño con problemas de motilidad del colon; los estudios
determinan ausencia congénita de células ganglionares. Según el gráfico ¿cuál es la capa en la que se determina la
ausencia de dichas células?
a. Mucosa - 2
b. Muscular propia - 2
c. Muscular propia - 3
d. Muscular de la mucosa - 3

1 2

24. Con respecto al control autonómico en el tracto gastrointestinal y en relación a su fisiología. ¿Cuál es la función
del sistema nervioso parasimpático en el tracto gastrointestinal?
a. Inhiben la contracción muscular y estimulan la secreción de sustancias a nivel de la submucosa
b. Estimulan la contracción muscular y estimulan la secreción de sustancias a nivel de la mucosa
c. Inhiben la contracción muscular e inhiben la secreción de sustancias a nivel de la submucosa
d. Estimulan la contracción muscular e inhiben la secreción de sustancias a nivel de la mucosa

25. Un estudiante que está preocupado por su examen parcial, no ha desayunado ni almorzado; cuando al fin ingiere
alimentos, esto le provoca el aumento de los movimientos musculares del tracto gastrointestinal y la sensación de
defecar. ¿Qué reflejo se ha activado?
a. Entero-gástrico
b. Gastro-cólico
c. Cólico-ileal
d. Ileo-ileal
26. ¿De qué par craneal es rama el nervio palatino mayor?
a. Vago
b. Hipogloso
c. Trigémino
d. Palatogloso

27. ¿En cuál de las fases de la deglución la epiglotis separa la vía respiratoria de la digestiva?
a. oral
b. laríngea
c. faríngea
d. esofágica

28. Los péptidos intestinales se pueden clasificar como sustancias endocrinas, neurocrinas y paracrinas, dentro de las
paracrinas se encuentran la somastotatina e histamina. Marque la respuesta correcta
a. La somastotatina es sintetizada por las células B de la mucosa gástrica
b. La histamina actúa estimulando su receptor tipo H1 en la mucosa gástrica
c. La histamina es sintetizada por células de tipo paracrino de las glándulas gástricas
d. La somatostatina presenta dentro de sus funciones la estimulación de la secreción de H+

29. En relación a los órganos intraabdominales y sus estructuras de fijación, elija la alternativa correcta
a. El mesenterio permite la suspensión e irrigación de los órganos retroperitoneales
b. Tanto el hígado como la vesícula biliar se encuentran ubicados a nivel del flanco derecho
c. El colon, el duodeno y el resto de intestino delgado son órganos considerados netamente como
peritoneales
d. Los ligamentos que encontramos dentro de la cavidad abdominal son el esplenorenal y el gastrofrénico
e. Los omentos van desde el estómago y la segunda porción del duodeno a otras estructuras
intraabdominales y existen dos: el omento mayor y el omento menor

30. Paciente varón de 27 años es llevado por bomberos a emergencia luego de ser asaltado y, tras resistirse, es cortado
con el pico de una botella a nivel abdominal. Al examen físico usted observa que a través de la herida se puede
observar la protrusión de asas intestinales. En relación con las capas de la pared abdominal, marque la alternativa
correcta.
a. La fascia de Scarpa está constituida principalmente por tejido adiposo
b. La pared abdominal está formada por piel, huesos, músculos, fascias y peritoneo parietal
c. La fascia de Camper es una estructura fibrosa que carece de grasa y su grosor es constante en toda la
pared abdominal
d. El músculo oblicuo externo discurre en dirección súpero-interna y se inserta en el borde inferior de las
ultimas 3 a 4 costillas
e. El músculo recto del abdomen tiene como funciones comprimir el contenido del abdomen, tensar la
pared del abdomen y flexionar la columna
BANCO DIGESTIVO

1. Paciente se queja de dolor en hipocondrio derecho, pero superficialmente. El dermatoma relacionado es


(marque la mejor respuesta)
a. T9

2. Dentro de las funciones del abdomen, se encuentra la defecación y micción, en las cuales la presión
intraabdominal debe:
a. Aumentar

3. Al examinar a un paciente, usted encuentra dolor localizado en fosa iliaca derecha y diagnostica
apendicitis. En este paciente, usted puede inferir.
a. El peritoneo parietal regional está afectado

4. La motilidad intestinal es estimulada principalmente por el:


a. Plexo de Auerbach

5. Durante el vómito, ¿el contenido gástrico tiene que pasar necesariamente por qué estructura para llegar
al esófago? Marque la mejor respuesta:
a. Cardias

6. Respecto a la anatomía del estómago, marque lo correcto:


a. El fondo gástrico forma la curvatura mayor

7. Marque la respuesta incorrecta:


a. En todo el tubo digestivo, se observa dos capas de muscular propia: circular interna y
longitudinal externa

8. Un alumno de medicina decide hacerse un piercing en el ombligo. Al realizarle el procedimiento, sangra


ligeramente. Esta sangre proviene de la arteria (marque la mejor respuesta)
a. Epigástrica inferior

9. Señala la respuesta correcta:


a. El apéndice cecal solo tiene serosa

10. Paciente mujer es traída a emergencia por sufrir una herida contuso penetrante por cuchillo realizada por
su esposo en un ataque de celos. Se observa herida en flanco izquierdo. Está solución de continuidad ha
comprometido varios músculos de la pared abdominal, excepto:
a. Recto abdominal

11. Marque el órgano que se considera retroperitoneal:


a. Parte de la vía biliar

12. La peristalsis o peristaltismo hace referencia a:


a. Motilidad para movilizar el alimento de proximal a distal.

13. Paciente tiene una úlcera sangrante en el segundo tercio del yeyuno. La arteria de la cual proviene la
sangre arterial para dicha zona es la arteria:
a. Mesentérica superior

14. La estructura que fija órganos principalmente a la pared posterior abdominal se denomina:
a. Mesenterio

15. Al iniciar la digestión, aumenta el consumo de oxígeno por la mucosa. Esto conlleva a una hipoxia local,
lo cual hace que se libere _____________, el cual produce vasodilatación:
a. Adenosina
16. Es inervado por aferentes somáticas:
a. Peritoneo parietal

17. El ligamento hepatogástrico une el __________ con el _______ y forma la entrada al ________
a. Hígado Estómago Orificio omental

18. Paciente con vólvulo del colon sigmoides. La necrosis de este segmento del colon se produce por una
alteración en la irrigación de la arteria:
a. Mesentérica inferior

19. Al retirar completamente el mesenterio de un órgano, el mismo se vería afectado principalmente en su:
a. Irrigación

20. Marque el órgano que se encuentra más distal en el tubo digestivo.


a. Ciego

21. Paciente de 24 años con dolor abdominal tipo cólico intenso en mesogastrio. Según sus conocimientos
de macroestructura, el origen del dolor puede ser el ___________:
a. Íleon

22. Paciente con herida por proyectil por arma de fuego, con herida de ingreso en región paraumbilical. Entre
las estructuras que usted está seguro que debe haberse lesionado es:
a. Omento mayor

23. Paciente joven es traído a emergencia con abdomen agudo quirúrgico debido a herida contuso
penetrante por verduguillo (alambre grueso con punta aguzada) recibida en una pelea después de un
partido de fútbol. Se observa herida en Hipocondrio Izquierdo. El órgano que debe estar sangrando y
produciendo hemoperitoneo es (marque la mejor respuesta):
a. Bazo

24. Al evaluar una tomografía abdominal, el médico asistente le pide al interno de la UPC que encuentre la
imagen con el corte a nivel de L1. El interno sabiamente busca el ________ para ubicar la vértebra L1.
a. Cuello del páncreas

25. En la inspiración, la pared abdominal debe ____________ para ____________:


a. Relajarse disminuir presión intra torácica

26. Sustancia que inhibe la secreción y la motilidad del estómago prolongando el tiempo de digestión:
a. Péptido insulinotrópico dependiente de la glucosa (GIP)

27. Marque lo correcto:


a. La hernia fisiológica se produce en la sexta semana y es la salida temporal de asas intestinales
a través del cordón umbilical

28. Marque la respuesta correcta en relación a la gastrina:


a. Las células G son las productoras y se encuentran principalmente en el antro gástrico

29. El consumir caramelos indirectamente activa la vía:


a. POMC/CART

30. Cuál de las siguientes estructuras no tiene vasos sanguíneos:


a. Epitelio intestinal

31. Paciente con hipoglucemia secundaria a un insulinoma (tumor neuroendocrino productor de insulina). El
órgano donde con mayor probabilidad ha crecido este tumor es:
a. Retroperitoneal
32. ¿En qué capa se encuentra la alteración principal en el Hirschsprung o megacolon agangliónico?:
a. Muscular propia

33. Con respecto a las ondas lentas, marque la afirmación correcta:


a. Son contracciones rítmicas espontáneas

34. El uso de Ranitidina bloquea el receptor H2 de la histamina en las células parietales. La histamina llega a
estas células por:
a. Difusión

35. La triada sintomática: vómitos explosivos post-prandiales, movimientos peristálticos epigástricos visibles
de izquierda a derecha y nódulo palpable epigástrico subcostal derecho, pertenecen a:
a. Estenosis congénita hipertrófica del píloro

36. Durante una cirugía oncológica, ¿la extirpación de cuál de los siguientes órganos se vería comprometida
por la presencia de adventicia?:
a. Recto

37. En cuanto a los reflejos gastrointestinales, un reflejo que estimula el tránsito intestinal es el reflejo:
a. Gastrocólico

38. El ligamento falciforme divide al hígado en dos lóbulos derecho e izquierdo. Embriológicamente deriva
del:
a. Mesenterio ventral

39. La presencia de atresias y estenosis duodenales se deben básicamente a una:


a. Falta de recanalización

40. Estudiante de medicina de 20 años, se ha amanecido estudiando para su examen de Sistema Digestivo.
No ha probado alimento desde la cena, por lo que se puede afirmar que la motilidad de esta persona está
siendo regulada por:
a. Motilina

41. Paciente con disminución del apetito marcada asociada a cáncer terminal, para promover la ingesta de
alimentos se podría usar análogos de:
a. Endorfinas

42. Las ondas lentas se producen por la apertura cíclica de canales de:
a. Calcio

43. La forma más común de atresia esofágica contiene:


a. Estenosis proximal del esófago con fístula traqueoesofágica distal

44. Al deglutir un bolo alimenticio, es lógico suponer que al pasar por el esófago haya un mayor consumo de
oxígeno en la pared del tercio:
a. Proximal

45. Paciente que come entera una pizza familiar de chorizo y queso. Es posible esperar que debido a la
cantidad de alimento ingerida, las ondas lentas hayan:
a. Sufrido ninguna alteración en su frecuencia

46. La hernia fisiológica se produce dentro de:


a. Cordón umbilical

47. El crecimiento de un adenocarcinoma de páncreas compromete la pared gástrica por contigüidad. ¿Qué
parte del estómago se esperaría esté comprometido?
a. Pared posterior del antro

48. Estimula la producción de saliva:


a. Vasodilatación periglandular

49. Durante la secreción de saliva, es de esperarse que las concentraciones de ________ y ______
disminuyen al disminuir el flujo:
a. Sodio Bicarbonato

50. Con respecto a la secreción gástrica de HCl:


a. A mayor secreción de HCl en el lumen gástrico, mayor pH en la sangre venosa gástrica

51. Respecto a las enfermedades del esófago, marque lo correcto:


a. El diagnóstico diferencial de la acalasia es la enfermedad de Chagas esofágica

52. Con respecto a las lesiones y enfermedades de la boca, marque lo correcto:


a. La eritroplasia debe ser biopsiada

53. Respecto a las glándulas salivales, marque lo incorrecto:


a. La glándula sublingual tiene forma de garfio

54. El omeprazol actúa sobre la membrana _____________ de la célula ____________


a. Apical / parietal

55. Durante el sueño, la concentración de bicarbonato en la saliva:


a. Disminuye

56. Durante el ataque con gas sarín (bloqueador de la acetilcolinesterasa) en el metro de Tokio, en 1995, el
personal de salud notó que los pacientes afectados presentaban:
a. Hipersalivación

57. La célula mucosa del cuello gástrico produce:


a. Moco

58. Un paciente con diarrea por hipermotilidad, usted sospecharía en el posible aumento de las siguientes
sustancias, excepto:
a. Péptido intestinal vasoactivo

59. En la evaluación de una tomografía abdominal, el interno observa un aneurisma en una arteria que se
dirige al riñón derecho. Con seguridad se puede afirmar que está a nivel de la vértebra:
a. L1

60. Una de las siguientes sustancias no comparte con las otras la misma acción sobre la producción de ácido
gástrico:
a. Colecistoquinina

61. Usted encuentra músculo estriado en el siguiente segmento:


a. Esfínter anal externo

62. Paciente con intoxicación por organofosforados (inhibidores de acetilcolinesterasa), se espera que el
tránsito intestinal se encuentre:
a. Aumentado

63. Al disminuir el pH duodenal por el HCl gástrico, se libera principalmente una hormona cuya célula diana
es:
a. Célula ductal del wirsung
64. La hormona que tiene un efecto sinérgico con la secretina para optimizar el pH duodenal y la digestión,
es:
a. CCK (colecistoquinina)
65. El ligamento falciforme del hígado proviene embriológicamente de:
a. Mesenterio ventral

66. Todos los músculos motores de la lengua están inervados por el XII par, excepto:
a. Palatogloso

67. Al ingerir grandes cantidades de dulces, con la subsecuente estimulación de incretinas, usted esperaría
que el apetito ______________, debido a __________________
a. Disminuya insulina

68. Paciente obeso con Covid-19 es intubado por interno inexperto, quien al solicitar que bombeen aire
dentro del tubo endotraqueal, nota que el epigastrio se distiende. Al sospechar que ha introducido el tubo
en el estómago, también es cierto que:
a. Disminuye el pH gástrico

69. Una de las siguientes sustancias reguladoras, puede actuar de forma paracrina y como hormona. Marque
la correcta:
a. Somatostatina

70. Durante una cirugía oncológica, el cirujano observa que los órganos abdominales tienen libre movimiento
dentro de la cavidad abdominal, excepto:
a. Colon ascendente

71. Al consumir un pan con mantequilla, la sensación de hambre disminuye debido a la acción de:
a. Colecistoquinina (CCK)

72. Respecto a los péptidos gastrointestinales, marque lo correcto.


a. Las sustancias paracrinas pueden viajar a través de vasos sanguíneos

73. En un paciente con gastroparesia (motilidad lenta del estómago), que presenta distensión abdominal
después de comer, usted le recomendaría que evite el consumo de lípidos y aminoácidos para disminuir
la acción de:
a. CCK

74. Paciente con apendicitis aguda, que debuta con dolor en mesogastrio. Este dolor se debe a estimulación
de receptores del dolor cuyas fibras van a viajar a la médula espinal a través de:
a. Nervios simpáticos

75. Todos los músculos motores de la lengua están inervados por el XII par, excepto:
a. Palatogloso

76. Al introducir una solución azucarada directamente al estómago mediante una gastrostomía (comunicación
entre la piel abdominal y el estómago), la sustancia que provocará que aumenten los niveles séricos de
insulina es:
a. Péptido tipo glucagón 1 (GLP-1)

77. En un experimento, con una sonda nasogástrica se instila por goteo en el estómago una sustancia
líquida, y se obtiene como respuesta una dramática disminución del pH del estómago. Dicha sustancia
debe contener:
a. Aminoácidos

78. En un paciente con hiperestimulación simpática se espera que las ondas lentas tengan un ritmo:
a. Menor en íleon terminal que en el duodeno

79. Se considera que el gusto puede viajar a través del nervio:


a. Glosofaríngeo

80. El ecografista sabe que para poder visualizar el nacimiento de la arteria mesentérica superior, debe
colocar el transductor sobre la piel de la siguiente región abdominal:
a. Epigastrio

81. Al seccionar el nervio facial a nivel timpánico, usted esperaría:


a. Disminución del gusto en la punta de la lengua

82. Marque lo correcto en relación al divertículo de Meckel.


a. Se encuentra usualmente a 60 cm de la VIC

83. Es un ligamento derivado del mesenterio dorsal:


a. Gastrocólico

84. Marque lo correcto respecto a la siguiente imagen:

a. Se produjo por falta de fusión de los ductos dorsal y ventral

85. La arteria mesentérica superior emerge de la aorta a nivel de:


a. Cabeza de páncreas

86. La digestión de las proteínas se inicia en:


a. Estómago

87. Aproximadamente en la semana 6 del desarrollo embrionario, el intestino intermedio gira 90º herniándose
a nivel del:
a. Cordón umbilical

88. Paciente de 76 años en estado de coma, con colostomía (colon abocado a la piel) por resección parcial
de colon secundaria a carcinoma de colon. Usted deja indicado que este día se realice el cambio de la
bolsa de colostomía, pero que lo hagan después de dos horas de la nutrición enteral, debido a que quiere
que se haga después de:
a. El reflejo gastrocólico

89. Recién nacido de 14 días, que presenta estreñimiento, distensión abdominal y vómitos. Al examinar el
orificio anal, se evidencia conducto permeable, pero al introducir un poco el termómetro, se evidencia
salida de material fecal por el ano. Usted sospecha que el problema se deba a:
a. Falta de desarrollo del tabique urogenital
b. Defecto en el desarrollo del conducto vitelino
c. Defecto en el desarrollo de las células de cajal
d. Falta de desarrollo del seno uretral
e. Falta de regresión de la membrana anal
90. La lengua está recubierta por epitelio:
a. Plano estratificado no queratinizado

91. El esfínter anal interno tiene musculatura ___ y tiene control ___:
a. Lisa/involuntario

92. La arteria aorta proporciona la irrigación al tubo digestivo ¿cuál de las siguientes arterias de irrigación al
ángulo cólico derecho?
a. Mesentérica superior

93. Paciente de 26 años que le cuenta en su historia clínica que cada vez que almuerza, a los 20 minutos
tiene deseo de defecar. Le comenta que su hijo de 1 mes le pasa lo mismo pero más intenso. Esto se
explica por el reflejo ____, el cual está _____ en el paciente:
a. Gastrocolico/normal

94. La región del estómago que se comunica con el duodeno se denomina


a. Pilórica

95. Acude a consulta un paciente que fue diagnosticado de úlcera péptica tres días antes. Luego de múltiples
pruebas diagnósticas, se concluye que el paciente presenta un tumor secretor de gastrina ¿cual de las
siguientes situaciones estará incrementada?
a. Secreción de ácido clorhídrico (HCl)

96. En el sistema digestivo el control del apetito estado por un complejo sistema de sustancias y órganos
integradores los cuales regulan le dijiste alimentos. La ____ es una sustancia orexígena/orexígena y es
sintetizada por el ____
a. Grelina/estómago

97. Sobre el control Autónomo del sistema digestivo marque la alternativa correcta:
a. En el sistema simpático, los nervios responsables hacen una primera sinapsis en ganglios
próximos al órgano a inervar

98. Con respecto a la actividad eléctrica del sistema digestivo, marque la alternativa correcta:
a. La frecuencia de las ondas lentas no se ve influenciada por la actividad neural ni las hormonas
gastrointestinales

99. Ante una lesión en el IX par craneal, el músculo ___ se altera en su función:
a. Estilofaríngeo

100.Un varón de 50 años es sometido a extirpación de duodeno y parte proximal del yeyuno. La pérdida de
estímulo hormonal en el páncreas para la secreción enzimática se explica por la pérdida de las células:
a. “I” productoras de CCK

101.Respecto al mecanismo de defecación ¿cual de las siguientes afirmaciones es correcta?


a. Se produce contracción o relajación del esfínter anal externo por señales de la corteza cerebral

102.Un niño de tres años llega a emergencia con disfagia (dificultad para tragar), salivación y llanto. Se
sospecha de ingesta de cuerpo extraño (moneda en el esófago), al ser evaluado se constata de una
radiografía presencia de cuerpo extraño a nivel de C6 y C7 (6º y 7º vértebra cervical). El cuerpo extraño
estará suspendido a nivel del estrechamiento producido por el:
a. Músculo cricofaríngeo

103.En el caso de un paciente con un tumor productor de gastrina, la presencia de úlceras duodenales y
erosión de la mucosa gástrica se debe principalmente a:
a. El exceso de HCl por estímulo de receptores CCK-B en la célula parietal

104.La onda peristáltica secundaria del esófago se caracteriza por ser originada:
a. Por el plexo mientérico del esófago

105.Marque lo correcto sobre las ondas lentas en el tubo digestivo:


a. Se constituyen de despolarizaciones y repolarizaciones
106.Recién nacido presenta protrusión de contenidos abdominales los cuales no están cubiertos por peritoneo
y salen de la cavidad abdominal a través de un defecto en la pared ¿Cómo se denomina a la afección
que presenta este paciente?
a. Gastrosquisis

107.Experimentalmente se utiliza atropina (anticolinérgico) para inhibir la secreción de gastrina, sin embargo,
la secreción de esta hormona se sigue dando ante estímulos vagales. Esta situación se explica porque la
atropina:
a. No bloquea la acción del péptido GRP

108.La niña de 4 días es llevada a la emergencia pediátrica por presentar llanto constante, la madre refiere
coloración azulada de labios al momento de lactar, acompañado de tos persistente y dificultad
respiratoria, así como distensión abdominal. Se le coloca sonda nasogástrica para alimentación notando
que retorna a la cavidad oral en todos los intentos ¿Cuál es la anomalía del desarrollo en este caso?
a. Atresia esofágica proximal con fístula traqueo esofágica distal

109.Paciente varón de 36 años es traído a la emergencia luego de sufrir un accidente de tránsito presenta
traumatismos múltiples en cabeza y tronco. Al examen físico se evidencia hematoma en hemicara
izquierda, ligera protrusión y caída del lado izquierdo del maxilar inferior por lo que se le realiza
tomografía donde se haya una fractura de apófisis coronoides del maxilar inferior. ¿Qué músculo está
relacionado directamente con esta situación?
a. Temporal

110.Un paciente refiere no percibir algunos sabores. Al examen físico constata alteración del sabor dulce y
umami. ¿Cuál de los siguientes nervios estará alterada su función?
a. Cuerda del tímpano (VII par)

111.A los pocos días de nacido, regresa a neonatología un niño con problemas de motilidad del colon, los
estudios determinan ausencia congénita de células ganglionares. Según el gráfico cuál es la capa en la
que se determina la ausencia de dichas células?

a. Muscular propia - 3

112.Con respecto al control autonómico en el tracto gastrointestinal y en relación con su fisiología ¿Cuál es la
función del sistema nervioso parasimpático y el tracto gastrointestinal?
a. Estimulan la contracción muscular y estimulan la secreción de sustancias a nivel de la mucosa

113.Intersticio ubicado entre el estroma del espacio portal y los hepatocitos, y por donde migran las células
cancerígenas que hacen diseminación linfática es el:
a. Espacio del Mall

114.La presencia de grandes cantidades de TGF - Beta estimula a las ___________ y se deposita colágeno,
formándose la cirrosis
a. Células de Ito
115.Paciente de 64 años con ICC al que se le va a realizar cirugía cardiovascular. Al calcular el volumen
sanguíneo total, se debe considerar que el hígado puede contener un volumen de sangre de
____________ mL en un adulto sano, en este paciente ese volumen puede llegar a ser de
_____________ mL
a. 450 - 1000

116.El área del lobulillo que se afecta más en caso de hipoxia es la zona:
a. 3

117.Paciente con carcinoma de vesícula biliar. La metástasis por continuidad afectará al lóbulo:
a. Cuadrado

118.El ácido acetilsalicílico actúa en la membrana:


a. Basolateral de la célula parietal

119.Al realizar una esplenectomía, se tiene que resecar la arteria esplénica, lo cual no es problema para el
estómago por que la arteria gastroomental izquierda se anastomosa con la:
a. Gastroomental derecha

120.La arteria esplénica proviene de la aorta y la vena esplénica desemboca en la vena:


a. Porta

121.Paciente mujer 21 con bulimia, que luego de un episodio de vómitos presenta hematemesis y al examen
físico que se encuentra crépitos subcutáneos cervicales. El diagnóstico más probable es:
a. Sd. Boerhaave

122.Paciente varón de 60 años, con antecedentes de promiscuidad sexual, tabaquismo y alcoholismo, acude
a consulta por presentar disfagia progresiva, odinofagia y al examen se observa tumor por parte posterior
de la lengua. La mejor posibilidad diagnóstica es:
a. Carcinoma escamoso

123.Niño de 5 años con historia de tres días de evolución caracterizado por fiebre, malestar general,
odinofagia, anorexia, e irritabilidad. Al examen de observa lesiones ulcerativas de 4mm de diámetro en
mucosa yugal, con borde blanquecino y eritema periférico. El diagnóstico más probable es:
a. Aftas orales

124.Es considerada una lesión preneoplásica


a. Leucoplasia

125.El esófago de Barrett se considera una lesión preneoplásica que se caracteriza por la presencia en
esófago de:
a. Metaplasia intestinal

126.Paciente mujer de 23 años gestante con lesión proliferativa en mucosa oral producida por proliferación
reactiva de vasos sanguíneos. Marque la mejor respuesta
a. Epulis

127.La glándula parótida tiene principalmente acinos de tipo:


a. Seroso

128.La reabsorción de sodio y secreción de potasio es estimulada por:


a. Aldosterona

129.Paciente con cirrosis hepática que tiene hipertensión portal con várices esofágica, y actualmente presenta
varices en estómago distal. Estas várices están relacionadas a aumento en la presión de las venas:
a. Gástrica derecha

130.La glándula parótida está inervada por el par craneal:


a. IX

131.Paciente con cirrosis hepática que tiene hipertensión portal con varices en esofago distal . Estas varices
están relacionadas a aumento en la presión de las venas gástricas
a. Inferior

132.La información eferente que sale de los núcleos salivales superior e inferior a través de los pares VII y IX
hacia las glándulas salivales llevan información tipo
a. Parasimpático

133.Un niño de 4 años ingresa en el hospital con vómitos graves . En el estudio se encuentra que el niño tiene
un páncreas anular ¿ Cuál de las siguientes hormonas gastrointestinales se encontrará a niveles
elevados en sangre con mayor probabilidad a raíz de esta patología
a. Gastrina

134.Durante una colecistectomía laparoscópica en un hombre de 61 años ¿ Cuál de las siguientes arterias
debe pinzar para extirpar la vesícula biliar con seguridad?
a. Cistica

135.Un hombre de 34 años se somete a una apendicectomía de urgencia . Después de realizar la


apendicectomía satisfactoriamente , el paciente se somete a una laparoscopia exploratoria¿Cual de las
siguientes características anatómicas es más útil para distinguir entre yeyuno e íleon?
a. El yeyuno tiene menos grasa mesentérica que el íleon

136.Una mujer de 45 años ingresa en el hospital con síntomas de obstrucción intestinal superior .En la TC se
encuentra que la tercera porción (transversa) del duodeno está comprimida por un gran vaso ¿Cuál de
los siguientes vasos causara muy probablemente la obstrucción ?
a. Arteria mesentérica superior

137.Durante una colecistectomía laparoscópica programada en una mujer de 47 años , el residente pinchó
accidentalmente el ligamento hepatoduodenal en vez de la arteria cística ¿Cual de los siguientes vasos
estaría muy probablemente ocluido en esta lesión iatrogénica?
a. Arteria hepática izquierda

138.Un hombre de 54 años ingresa en urgencias con intenso dolor abdominal superior . La gastroscopia
revela un tumor en el antro del estómago . Se pide una TC para evaluar el drenaje linfático del
estómago¿Cuál de los siguientes nódulos linfáticos estará muy probablemente afectado en una neoplasia
maligna del estómago
a. Mesentérico inferior

139.Una mujer obesa de 45 años con fiebre alta acude a la consulta con náuseas y dolor agudo e intermitente
en el cuadrante superior derecho del abdomen de 2 días de duración - Tiene una historia de ictericia de
24 horas. Tiene antecedentes de litiasis biliar. Bilirrubina total del 10 mg/dL. Lipasa de 5 mg/mL. ¿Cuál de
las siguientes estructuras está muy probablemente obstruida por un cálculo biliar?
a. Ampolla de Vater

140.Una mujer de 45 años ingresa en urgencias con dolor abdominal intenso . La TC y RM revelan un tumor
de la cabeza del páncreas que afecta el proceso unciforme ¿Cual de los siguientes vasos es más
probable que suministre irrigación a parte de la zona afectada?
a. Arteria cólica media

141.Un hombre de 70 años ingresa en urgencias con diarrea intensa . La arteriografía revela un bloqueo del
90% en el origen aórtico de la arteria mesentérica inferior ¿Cuál de las siguientes arterias proporciona
muy probablemente irrigación colateral al colon descendente?
a. Arteria sigmoidea

142.Al disminuir el pH duodenal por el HCL gástrico , se libera principalmente una hormona cuya célula diana
es
a. Célula ductal de Wirsung

143.Una de las siguientes sustancias reguladoras , puede actuar de forma paracrina y como hormona
a. Somatostatina

144.Al ingerir grandes cantidades de dulces , con la subsecuente estimulación de incretinas , usted esperaría
que el apetito
a. Disminuya por insulina

145.Paciente obeso con Covid-19 es intubado por interno inexperto, quien al solicitar que bombeen aire
dentro del tubo endotraqueal, nota que el epigastrio se distiende. Al sospechar que ha introducido el tubo
en el estómago, también es cierto que:
a. Disminuye el pH gástrico

146.La hormona que tiene un efecto sinérgico con la secretina para optimizar el pH duodenal y la digestión,
es:
a. CCK

147.El ecografista sabe que para poder visualizar el nacimiento de la arteria mesentérica superior , debe
colocar el transductor sobre la piel de la siguiente región abdominal
a. Epigastrio

148.En un paciente con hiperestimulación simpática se espera que las ondas lentas tengan un ritmo
a. Mayor en estómago que en el duodeno
b. Mayor en estómago que en el íleon terminal
c. Menor en íleon terminal que en el duodeno
d. Mayor en el íleon que en duodeno

149.Se considera que el gusto puede viajar a través del nervio


a. Glosofaríngeo

150.La rotación en sentido longitudinal del estómago en el desarrollo embriológico condiciona que el nervio
vago derecho quede a nivel
a. Posterior

151.Con respecto al control autonómico en el tracto gastrointestinal y en relación con su fisiología. ¿Cuál es la
función del sistema nervioso parasimpático en el tracto gastrointestinal?
a. Estimulan la contracción muscular y estimulan la secreción de sustancias a nivel de la mucosa.

152.Un estudiante que está preocupado por su examen parcial, no ha desayunado ni almorzado; cuando al fin
ingiere alimentos, este le provoca el aumento de los movimientos musculares del tracto gastrointestinal y
la sensación de defecar.?¿Qué reflejo se ha activado?
a. gastro - cólico

153.¿De que par craneal es rama el nervio palatino mayor?


a. Trigémino

154.¿En cuál de las fases de deglución la epiglotis separa la vía respiratoria de la digestiva?
a. Faríngea

155.Los péptidos intestinales se pueden clasificar como sustancias endocrinas, neurocrinas y paracrinas,
dentro de las paracrinas se encuentran la somatostatina e histamina. Marque la respuesta correcta
a. La histamina es sintetizada por células de tipo paracrino de las glándulas gástricas
156.Paciente varón de 27 años es llevado por bomberos a emergencia luego de ser asaltado y, tras resistirse,
es cortado con el pico de una botella a nivel abdominal. Al examen físico usted observa que a través de la
herida se puede observar la protrusión de las asas intestinales. En relación con las capas de la pared
abdominal, marque la alternativa correcta.
a. El músculo recto del abdomen tiene como funciones comprimir el contenido del abdomen, tensar
la pared abdominal y flexionar la columna

157.Los músculos del tracto gastrointestinal de los segmentos propulsivo y receptor del bolo alimenticio,
responden de forma diferente al movimiento de este bolo a través del intestino. ¿Cuál de las siguientes
afirmaciones describe correctamente la actividad del segmento propulsivo?
a. Tanto el músculo circular como el longitudinal están relajados
b. El músculo longitudinal está relajado y el circular esta contraído
c. Tanto el músculo circular como el longitudinal están contraídos
d. El músculo longitudinal está contraído y el circular está relajado

158.El nervio vago inerva al músculo


a. Estriado del esófago

159.¿Cuál de las siguientes es una característica de los ganglios mioentéricos del sistema nervioso entérico?
a. Contiene mayor número de neuronas que el plexo submucoso
b. Es también conocido como el plexo de Meissner
c. Contiene sólo neuronas motoras excitatorias del músculo liso
d. Contiene neuronas sensitivas que activan a los músculos circular y longitudinal del tracto
intestinal

160.El divertículo faringoesofágico, hipofaríngeo de zenker, es una lesión muy particular que se localiza en la
cara póstero lateral de la Unión de la faringe con el esófago, como una herniación de la mucosa
esofágica a través de las fibras oblicuas del músculo.
a. Constrictor inferior de la faringe

161.La razón por la que el potencial de acción viaja rápidamente en sentido longitudinal por el músculo liso
gastrointestinal es la presencia de uniones en hendidura,
a. Varicosidades

162.Respecto a los péptidos gastrointestinales, marque lo correcto.


a. las sustancias paracrinas pueden viajar a través de vasos sanguíneos

163.Al ingerir rápidamente un litro de agua, usted esperaría que la gastrina aumenta por efecto de:
a. ACh del sistema mientérico

164.Respecto a la anatomia del estomago,marque lo correcto:


a. El fondo gástrico forma la curvatura mayor
(Pág 19 → 21)

165.Los nervios esplácnicos lumbares (L1-L2) llevan información de tipo:


a. Simpática

166.Paciente de 32 años con herida por arma de fuego y shock hipovolémico. El intestino delgado no se ha
infartado aún a pesar de la hipoxia gracias a la liberación de:
a. Adenosina

167.Al realizarse un piercing en el ombligo,la sensación de dolor se transmite por:


a. T10
168.El nivel en el que se encuentra el píloro y el páncreas se puede determinar usando el:
a. Píloro transpilórico

169.Permite la suspensión e irrigación de los órganos peritoneales:


a. Mesenterio
170.El dolor asociado a apendicitis clásicamente se ubica en:
a. Fosa iliaca derecha

171.El ligamento inguinales formado por la aponeurosis del:


a. Oblicuo externo

172.Los nueve cuadrantes del abdomen se delinean usando el plano subcostal,las líneas medioclaviculares y:
a. Plano intertubercular

173.Es un órgano peritoneal:


a. Hígado

174.La línea alba se encuentra:


a. Entre los rectos abdominales

175.Es un órgano retroperitoneal:


a. Páncreas

176.Enfermedad asociada con un error en el desarrollo de las células de Cajal


a. Enfermedad de Hirschsprung

177.El divertículo de Meckel es un rezago de:


a. Conducto vitelino

178.La fístula rectoperitoneal es causada por una falta en el desarrollo de:


a. Tabique urorrectal

179.95) Al disminuir el pH duodenal por el HCl gástrico, se libera principalmente una hormona cuya célula
diana es:
a. Célula ductal del Wirsung

180.Al introducir una solución azucarada directamente al estómago mediante una gastrostomía (comunicación
entre la piel abdominal y el estómago), la sustancia que provocará que aumenten los niveles séricos de
insulina es:
a. Péptido tipo glucagón 1 (GLP-1)

181.La rotación en sentido longitudinal del estómago en el desarrollo embriológico condiciona que el nervio
vago derecho quede a nivel:
a. Posterior

182.Una de las siguientes sustancias no comparte con las otras la misma acción sobre la producción de ácido
gástrico:
a. Colecistoquinina

183.En un paciente con gastroparesia (motilidad lenta del estómago), que presenta distensión abdominal
después de comer, usted le recomendaría que evite el consumo de lípidos y aminoácidos para disminuir
la acción de:
a. CCK

184.En un paciente con shock distributivo, usted decide iniciar noradrenalina por un catéter CVC,
consiguiendo aumentar la presión arterial. ¿qué efecto sobre la motilidad intestinal esperaría encontrar?
a. El potencial de reposo de las fibras musculares se hace más negativo

185.El ecografista sabe que para poder visualizar el nacimiento de la arteria mesentérica superior, debe
colocar el transductor sobre la piel de la siguiente región abdominal:
a. Epigastrio
186.La glándula submandibular recibe inervación traída por el nervio
a. Cuerda del tímpano

187.A mayor flujo de saliva, disminuye la concentración de:


a. Potasio

188.En términos de mg/mL, el principal componente de la saliva es _____ seguido de ____:


a. Proteínas - Potasio

189.La reabsorción de Sodio y Cloro en las glándulas salivales se da principalmente en el:


a. Conducto estriado

190.La información eferente que sale de los núcleos salivales superior e inferior a través de los pares VII y IX
hacia las glándulas salivales llevan información de tipo:
a. Parasimpático

191.La glándula submaxilar le hace gancho al:


a. Músculo milohioideo

192.Los conductos salivales son ___ al agua, esa es una de las razones por las cuales la saliva es siempre
___.
a. Impermeables - hipotónica

193.Los nervios esplácnicos pélvicos (S2-S4) llevan información de tipo:


a. Parasimpática

194.113) Presenta movimientos en masa:


a. Colon

195.114) La digestión de los lípidos se inicia en:


a. Intestino delgado

196.115) La digestión de los carbohidratos se inicia en:


a. Cavidad oral

197.117) El estómago recibe información simpática proveniente del:


a. Ganglio celiaco

198.118) En el intestino delgado se absorbe los carbohidratos en forma de:


a. Fructosa

199.119) Es rama de la arteria mesentérica superior:


a. A. cólica media

200.120) El “dolor de estómago” asociado a gastritis se suele ubicar en:


a. Epigastrio

201.121) El azúcar de mesa sacarosa es digerido a dos monosacáridos que comparten el transportador:
a. GLUT 2

202.122) Durante la defecación se requiere:


a. Señales inhibitorias en el nervio pudendo

203.123) En un paciente con fístulas intestinales y fisuras anales, con antecedente de enfermedad
inflamatoria intestinal, ud sospecharia en:
a. Enfermedad de Crohn
204.En los pacientes con Colecistitis Aguda no operable, una opción es la colocación de una sonda por el
cístico, procedimiento en el que se ingresa con dificultad debido a la estrechez del cístico y a la presencia
de:
a. Válvula de Herring

205.Paciente de 51 años con antecedente de enfermedad diverticular acude a emergencia por sangrado
profuso y dolor en hipocondrio izquierdo. ¿Cuál es el origen más probable de la sangre que pierde el
paciente ?
a. Mesentérica inferior

206.Paciente de 42 años con dolor abdominal intenso y hematemesis. En la endoscopia se observa una
úlcera duodenal posterior perforada con hemorragia intraabdominal. ¿Cuál de las siguientes arterias
estará comprometida?
a. Pancreaticoduodenal posterosuperior

207.Para que los triglicéridos sean absorbidos deben ser metabolizados a


a. Monoglicéridos y Ácidos grasos

208.El vibrio cholerae produce diarrea porque:


a. Aumenta la producción de AMPc en los enterocitos

209.La motilidad intestinal es estimulada por


a. Colecistoquinina y gastrina

210.La diarrea por deficiencia de lactasa es de tipo


a. osmótica

211.La metoclopramida estimula el vaciamiento gástrico aumentando la fuerza de contracción de las paredes
gástricas esto puede conseguirlo mediante la estimulación indirecta de las neuronas liberadoras de
a. Acetilcolina

212.Los vértices de un acino hepático están constituidos por


a. Dos espacios porta y dos venas centrolobulillares

213.Al ingerir un pedazo de mantequilla, cuál de los siguientes tiene un efecto directo en la reducción del
vaciado gástrico:
a. Colecistoquinina

214.Tras la vagotomía (resercion del vago) por enfermedad úlcera péptica en un paciente UD. esperaría
encontrar:
a. Aumento del pH gástrico

215.Se realizó un experimento en el cual se inyectó tinta china en el peritoneo de ratas de laboratorio. Al
realizarse una biopsia hepatica de dichos animales, se encontro que el tinte negro fue fagocitado por:
a. Células de kupffer

216.En un paciente con pH gástrico muy bajo, es posible que la siguiente sustancia se secrete en menor
cantidad:
a. Gastrina

217.La fase cefálica de la secreción gástrica responde por cerca del 30% de la respuesta ácida a un reflejo
con la _____________ se elimina la fase cefálica de la secreción gástrica
a. Vaguectomía

218.El esofago de barret se caracteriza por presentar _________ en el esofago


a. Metaplasia intestinal
219.La presencia de orina que sale por el ombligo de un recien nacido cada vez que llora, es posible que
deba a un defecto en el desarrollo de:
a. Seno urogenital

220.El conducto biliar deriva del:


a. Endodermo

221.Cuál de los siguientes órganos son intraperitoneales:


a. Estómago, Vesícula biliar, Ileón, Hígado

222.Los vasos mesentéricos superiores se hallan a nivel de:


a. Cuello del páncreas

223.La colecistoquinina (CCK) inhibe:


a. El vaciamiento gástrico

224.Un niño de 2 años es llevado a la consulta por diarrea persistente, edema de las extremidades y falta de
crecimiento en relación a su edad. Los análisis de sangre revelan que tiene concentración plasmática
baja de proteínas (hipoproteinemia). Como parte del estudio se coloca Colecistoquinina (CCK)
endovenosa y se recoge muestras del líquido duodenal por endoscopia; el resultado del líquido confirma
incapacidad para hidrolizar proteínas a un pH neutro, esta situación mejora al añadir una pequeña
cantidad de tripsina. El paciente probablemente esté sufriendo la falta congénita de ……….
a. Enterocinasa

225.Paciente mujer de 35 años acude a consulta por sensación de sequedad y lesiones en la cavidad oral. Al
examen se observa atrofia de la mucosa, fisura y úlceras; nota además sequedad e irritación de la córnea
y aumento del tamaño de las glándulas parótidas. Su diagnostico mas probable es artritis reumatoide; el
hallazgo más probable en una biopsia de glándula parótida es ….…
a. Gran infiltración de linfocitos y células plasmáticas

226.Un hombre de 42 años de edad se presenta al médico con una historia clínica de 1 año de evolución,
caracterizado por dolor abdominal bajo y diarreas con crisis sanguinolentas. Manifiesta además pérdida
de peso de 8kg durante este periodo. La colonoscopia revela lesión difusa en el colon con afectación del
recto. La biopsia de estas lesiones revela adelgazamiento de la pared, inflamación y ulceración de la
mucosa y submucosa. El diagnóstico más probable en este caso es:
a. Colitis ulcerativa

227.Dos estudiantes deciden tomar un receso para comer una hamburguesa a la hora del almuerzo. Antes de
llegar a la cafetería, impulsos nerviosos provenientes del complejo vagal dorsal iniciarán la secreción de
ácido gástrico por la liberación de ___________ desde el sistema nervioso entérico.
a. GRP (péptido liberador de gastrina)

228.Un niño de cuatro años de edad es llevado a la consulta por cuadros diarreicos frecuentes caracterizados
por heces pálidas, voluminosas y fétidas; al examen físico presenta bajo peso y talla para la edad. Se
mide la concentración de cloruro de en el sudor y se encuentra que sus valores son muy elevados. La
alteración más importante a nivel de células ductales del páncreas tiene relación directa con la
conductancia de …..
a. Cloro
229.Se evalúa los valores séricos de las siguientes sustancias a un paciente con enfermedad hepática
terminal; en este paciente se espera encontrar la combinación de la letra …..

Glucosa Amoniaco Albúmina

a. Aumentada Disminuida Disminuida

b. Disminuida Aumentada Aumentada

c. Aumentada Aumentada Aumentada

d. Disminuida Aumentada Disminuida

230.Una mujer de 35 años de edad HIV positiva, se presenta al médico con dolor abdominal en cuadrante
superior derecho e ictericia. La paciente refiere haber tenido múltiples episodios de ictericia durante los
últimos 10 años. Los exámenes para determinar hepatitis viral, dieron positivos para Hepatitis B, siendo
catalogado el caso como hepatitis crónica con alteración funcional. En un examen de sangre ¿Cuál de los
siguientes parámetros está disminuido?
a. Albúmina

231.En el reflejo peristáltico del intestino delgado ¿Cuál de los siguientes eventos sucede en la porción caudal
del bolo alimenticio?
a. Acción del péptido inhibidor vasoactivo (VIP) en el músculo circular

232.Un varón de 58 años de edad con enfermedad de Crohn severo fue sometido a una resección ileal.
Después de la cirugía esta paciente padecerá de esteatorrea esto se explica porque ……
a. Hay malabsorción de ácidos biliares

233.En un experimento se inserta un balón en el estómago de un voluntario, se infla poco a poco mientras
que se vigilan las presiones intraluminales. Aunque el volumen del balón aumenta considerablemente, las
presiones permanecen constantes. Esta relación volumen-presión se explica por la liberación local de ….
a. Óxido nítrico y péptido inhibidor vasoactivo

234.¿Cuál de las siguientes alternativas es una característica de la secreción exocrina del páncreas?
a. Su mayor estímulo se da en la fase intestinal

235.Las estructuras en el hígado que permite que los productos metabólicos unidos a proteínas tengan
acceso a las membranas basolaterales de los hepatocitos, son ….
a. Las fenestras sinusoidales

236.La composición de la bilis es modificada conforme fluye por los conductillos biliares. Durante este tránsito
se espera que aumente la concentración de …..
a. Ig A

237.Se mide experimentalmente el contenido gástrico de dos personas. La persona “A” tiene alto contenido
de grasa y la persona “B” tiene un contenido isotónico. ¿Cual de las siguientes es correcta respecto al
vaciamiento gástrico?
a. Hay ralentización del vaciado gástrico sólo en “A”
238.El examen endoscópico de un paciente con hipertensión portal grave revela venas tortuosas que
sobresalen hacia la luz del esofago. El paciente recibe tratamiento quirúrgico mediante la colocación de
una derivación que conecta la vena cava. Después de la operación el riesgo de encefalopatía …….. y el
sangrado de varices ……
a. Aumentará/Disminuirá

239.Un bolo alimenticio grande y poco masticado se atasca en el esofago, esto ocasiona una sensacion de
dolor que es transmitida por los nervios:
a. Esplácnico

240.El peristaltismo del intestino delgado se puede intensificar debido a:


a. Irritación de la mucosa

241.Un paciente es diagnosticado con un tumor neuroendocrino productor de somatostatina, esto provocará
en el sistema digestivo:
a. Diarrea

242.Los diferentes segmentos del tubo digestivo son susceptibles de reflejos y movimientos según su
contenido.Si colocoramos mediante una sonda un bolo alimenticio directamente en el tercio medio del
esofago:
a. Se producirá ondas secundarias

243.En el digestivo la liberación hormonal se presenta ante diversos factores o estímulos. La hormona
_________ es estimulada por la presencia de alimentos en el bulbo duodenal a predominio de ácidos
grasos y triglicéridos, por estimulación vagal y por la hormona secretina.
a. Colecistoquinina (CCK)

244.El divertículo de Meckel es una anomalía congénita que ocurre por la persistencia del conducto vitelino y
da origen a una estructura sacular, el cual se encuentra en el:
a. Borde antimesentérico

245.La fase oclusal de la masticación se realiza con la contracción de los músculos inervados por el nervio
craneal:
a. V

246.En un paciente de 43 años con tumor carcinoide de páncreas productor de gastrina (Sindrome de
Zollinger-Ellison) se puede encontrar una potenciación del reflejo:
a. Gastrocolico

247.Durante la fase faríngea de la deglución ocurre el siguiente mecanismo:


a. La onda peristáltica lleva el alimento hacia el esofago

248.En los carcinomas (neoplasia benigna) es frecuente que ocurran la metástasis a través de los vasos
venosos. En el caso de un carcinoma del tercio superior del esofago, ubicado en la cara lateral izquierda,
es más probable que la metástasis viaje por la vena:
a. Hemiácigos accesoria

249.168) Los movimientos en masa son iniciados por el reflejo:


a. Duodenocolico
250.Una mujer de 65 años HIV positiva se presenta con dolor abdominal en el cuadrante superior derecho e
ictericia. La paciente afirma haber teñido múltiples episodios de ictericia durante los últimos 10 años. Los
exámenes para poder detectar hepatitis viral, dieron positivos para Hepatitis B, siendo catalogada como
hepatitis crónica con alteración funcional. En un examen de sangre ¿cuál de los siguientes parámetros
está disminuido?
a. Albúmina

251.En el reflejo peristáltico del intestino delgado¿cuál de los siguientes eventos sucede en la porción caudal
del bolo alimenticio?
a. Acción del péptido inhibidor vasoactivo(VIP) en el músculo circular

252.Un varón de 58 años de edad con enfermedad de Crohn Severo fue sometido a una resección ilegal.
Después de la cirugía este paciente padecerá de esteatorrea, esto se explica porque ….
a. Hay mala absorción de los ácidos biliares

253.En un experimento se inserta un balón en el estómago de un voluntario, se infla poco a poco mientras se
vigilan las presiones intraluminales. Aunque el volumen del balón aumenta considerablemente, las
presiones permanecen constantes. Esta relación volumen presión se explica por la liberación local de ….
a. Óxido nítrico y péptido inhibidor vasoactivo

254.¿Cuál de las siguientes alternativas es una característica de la secreción exocrina del páncreas?
a. Su mayor estímulo se da en la fase intestinal

255.Las estructuras en el hígado que permiten que los productos metabólicos unidos a proteínas tengan
acceso a membranas basolaterales de los hepatocitos son ….
a. Las de fenestras sinusoidales

256.La composición de la bilis es modificada conforme fluye por los conductillos biliares. Durante este tránsito
se espera que aumente la concentración de ….
a. IgA

257.Se mide experimentalmente el contenido gástrico de dos personas. La persona A tiene alto contenido de
grasas y la persona B tiene un contenido isotónico ¿Cuál de las siguientes es correcta respecto al
vaciamiento gástrico?
a. Hay ralentización del vaciado gástrico sólo en A

258.El examen endoscópico de un paciente con hipertensión portal grave revela venas tortuosas que
sobresalen hacia la luz del estómago. El paciente recibe tratamiento quirúrgico mediante la colocación de
una derivación que conecta la vena porta a la vena cava. Después de la operación el riesgo de
encefalopatía …. y el riesgo de sangrado de varices ….
a. Aumentará / disminuirá

259.Un paciente varón de 18 años de edad acude al médico para sus exámenes de rutina. Sus resultados de
laboratorio muestran un valor de bilirrubina sérica de 4mg/dl y una bilirrubina directa de 0.3 mg/dl. Las
pruebas de función hepática son normales. La alteración que explica mejor este caso es por la deficiencia
de….
a. Glucuronil transferasa

260.Un hombre de 57 años de edad es llevado a urgencias con hematemesis masiva rojo brillante, a su
llegada se halla inconsciente con PA: 80/40 mm Hg y FC:124 lat/min. Luce ictérico con presencia de
arañas vasculares en el tórax anterior y extremidades, abdomen distendido con signo de oleada positiva.
Se encuentra esplenomegalia y pérdida de la masa muscular en extremidades. La anastomosis vascular
responsable del sangrado en este paciente es ….
a. Vena gastrica izquierda y vena acigos
261.Un estudiante de medicina está comiendo un plato de comida a base de champiñones, espárragos y
salsa de soya. El estímulo del sabor umami contenido en todos estos alimentos viaja a través del
nervio….
a. Cuerda del tímpano

262.Una paciente de 30 años de edad es sometida a una cirugía de oído medio derecho por un problema de
otoesclerosis. Luego de la cirugía refiere alteración sensitiva de la lengua. Al evaluar el caso usted
esperaría encontrar ….
a. Sensación del dolor, tacto y temperatura conservadas

263.En una paciente de 45 años de edad con colestasis biliar, se encuentra una elevación de los niveles
sanguíneos de fosfatasa alcalina hasta 3 veces la cifra normal ¿Cuál de las siguientes alternativas estará
también elevada como evidencia del daño biliar?
a. Gamma glutamil transpeptidasas

264.Experimentalmente se incrementa la velocidad de la secreción salival con una sustancia, en el análisis de


la composición de esta saliva obtenida se espera encontrar….
a. Aumento de la concentración de bicarbonato que supera la concentración plasmática

265.Lactante de 3 meses de vida es atendido por presentar diarrea, se administra una solución glucosa y
electrolitos por vía oral. La proteína de membrana apical que explica la capacidad de esta solución para
proporcionar aporte de glucosa e hidratación es ….
a. SGLT 1

266.Paciente ha sufrido herida de bala en el abdomen, se le tenido que extirpar el segmento medio y distal del
íleon. En este caso la síntesis hepática de sales biliares estará ….
a. Incrementada por estímulo de la enzima colesterol 7 alfa hidroxilasa

267.Un varón de 75 años ingresa al consultorio por presentar ictericia marcada de piel y escleras. El estudio
del paciente mostró que presentaba un tumor que obstruía la totalidad del conducto hepático común.
¿Cual de los siguientes conductos se encontraría dilatado en este paciente?
a. De Hering

268.Correlacione las dos columnas y marque la respuesta correcta:


a. Enfermedad Hirschsprung Aganglionosis congénita
b. Diarrea osmótica. Intolerancia a la lactosa
c. Diarrea secretora. Canales de Cl- en las células de la cripta
d. Diarrea exudativa Heces con moco y sangre

269.La fase cefálica de la secreción gástrica responde por cerca del 39% de la respuesta ácida a un reflejo.
Con la ____ se elimina la fase cefálica de la secreción gástrica
a. Vaguectomía

270.El esofago de Barret se caracteriza por presentar___ en el esofago


a. Metaplasia intestinal

271.En la regulación del apetito y la saciedad, la estimulación experimental cronica del núcleo ventro medial
del hipotálamo producirá:
a. Afagia

272.Paciente mujer de 25 años acude por dolor en fosa iliaca derecha que empeora al toser o
caminar,asociada a náuseas y vómitos por lo cual acude a emergencia.Dos días después de realizarle
una apendicectomia, la paciente desarrolla fiebre alta (39° C), esta hipotensa y presenta dolor
abdominal.La laparotomia exploratoria muestra un gran volumen de sangre en la cavidad peritoneal por
lesion de un vaso producida durante la apendicectomía.¿Cual de las siguientes arterias debe ligarse
para detener la hemorragia?
a. Ileocolica
273.¿Cuál de los siguientes es una causa de ictericia con bilirrubina conjugada aumentada?
a. Obstrucción del colédoco

274.Dos días después de una apendicectomía en un hombre de 45 años, ha desarrollado fiebre alta (39), está
hipotenso y presenta dolor abdominal. La laparotomía exploratoria muestra un gran volumen de sangre
en la cavidad peritoneal por lesion de un vaso producida durante la apendectomía. ¿Cuál de los
siguientes vasos debe ligarse para detener la hemorragia?
a. Arteria ileocólica

275.Paciente de 78 años, con diabetes mellitus tipo ll y fumador, que acude a consulta porque desde hace
dos semanas tiene un dolor intenso en flanco derecho y mesogastrio , intenso que aparece a los 30
minutos de haber comido, y desaparece dos a tres horas después. En estos pacientes, es muy probable
que la circulación deficitaria sea parcialmente asumida por la:
a. Arteria cólica media

276.Un hombre de 70 años ingresa en urgencias con diarrea intensa. La arteriografía revela un blloqueo del
90% en el origen aórtico de la arteria mesentérica inferior. ¿Cuál de las siguientes arterias proporciona
muy probablemente irrigación colateral al colon descendente?
a. Arteria cólica derecha

277.Un niño de 4 años ingresa en el hospital con vómitos graves. La exploración radiológica y la historias
clinica revelan que el niño tiene páncreas anular. ¿Cuál de las siguientes estructuras es la que se
encontrará dilatada con mayor probabilidad a raíz de esta patología?
a. Primera porción del duodeno

278.Un hombre de 55 años ingresó al hospital con dolor abdominal intenso. La gastroscopia y la TC revelaron
una úlcera perforada en la pared posterior del estómago. ¿Dónde se desarrolla inicialmente con más
probabilidad una peritonitis?
a. Bolsa omental (saco menor)

279.Un hombre de 44 años ingresa en urgencias con vómitos abundantes y deshidratación. Las imágenes
radiológicas demuestran que parte del intestino está comprimido entre la aorta abdominal y la arteria
mesentérica superior. ¿Cuál de las siguientes estructuras intestinales estará muy probablemente
comprimida?
a. Tercera porción del duodeno

280.¿Cuál de los siguientes nódulos linfáticos estará muy probablemente afectado en una neoplasia maligna
del recto?
a. Mesentérica inferior

281.Una mujer de 23 años ingresa con dolor abdominal, náuseas y vómitos. La historia Clínica muestra que el
dolor es agudo y ha sido constante durante 4 años. El dolor empezó en el epigastrio e irradió
bilateralmente alrededor del tórax hasta justo debajo de las cápsulas. Actualmente el dolor se localiza en
el hipocondrio derecho. L a TC revela cálculos calcificados en la vesícula biliar. ¿Cuál de los siguientes
nervios llevan las fibras aferentes del dolor referido?
a. Nervios esplácnicos torácicos mayores

282.Un hombre de 55 años ingresa en urgencias por severa perdida de peso en los 6 meses previos. El
examen radiológico pruebas aportan signos de un tumor que causa hipertensión portal. Los estudios de
laboratorio revelan que las deposiciones son grasas, tiene desnutrición e hipoxia hepática. ¿Cuál de las
siguientes localizaciones se encuentra muy probablemente afectada?
a. Segmento l

283.El triángulo de calot es importante reconocerlo porque sirve como reparo para encontrar la arteria cística,
y está formado por el conducto cístico, el conducto hepático derecho y :
a. Borde hepático
284.El kernícterus se produce en recién nacidos con valores mayores de 25m/dL en la bilirrubina:
a. Indirecta

285.Paciente con cirrosis hepática y presión de vena cava de 15mmHg. Lo más probable es que el paciente
presente:
a. Ascitis

286.Considerando un flujo plasmático renal de 180 ml/min, y una fracción de filtración del 20%, si la
concentración en sangre de la bilirrubina indirecta es de 0.6mg/dL, entonces es correcto esperar que la
carga filtrada de la bilirrubina indirecta es:
a. Menor de 1080 mg/dL

287.El volumen diario de bilis secretada al intestino es:


a. 500-1000 ml

288.La excreción del amonio se da principalmente por:


a. Orina

289.Si en una persona normal, el flujo de la arteria hepática es de 700ml/min, entonces el flujo de la vena
porta debería ser aproximadamente:
a. 2800mL/min

290.Paciente con Grigler-Najjar debida a mutación del gen UGT1A1, se presenta a consulta por ictericia,
usted asume que si le hiciera un análisis de sangre encontraría valores elevados de:
a. Bilirrubina indirecta

291.El amoniaco corporal se forma principalmente en:


a. Colon

292.La zona del lobulillo hepático que se afectaría más en una intoxicación con droga hepatotóxica es la
zona:
a. 1

293.El acino hepático tiene en sus aristas:


a. Vena centrolobulillar

294.Paciente con esteatosis hepática no alcohólica, que en la biopsia se observa degeneración grasa de los
hepatocitos, lo cual se debe a depósitos de lípidos que principalmente contienen:
a. Triglicéridos

295.Al evaluar la orofaringe de un paciente, el médico le solicita que abra la boca, saque la lengua y diga a .
Al hacer esta maniobra, nota que el paladar se desvía hacia la derecha, lo cual le hace sospechar que el
paciente sufre de una lesión del nervio craneal:
a. X contralateral

296.Un bolo alimenticio grande y poco masticado se atasca en el esófago, esto ocasiona una sensación de
dolor que es transmitida por los nervios:
a. esplácnicos

297.Para realizar el movimiento mecánico de abrir la boca, primero se necesita:


a. fijar el hueso hioides

298.¿Cuál de las siguientes alternativas se define como la protrusión directa del contenido abdominal a la
cavidad amniótica por un defecto de la pared corporal?
a. Gastrosquisis
299.Un paciente requiere que se le coloque una sonda de alimentación directamente al estómago
(gastrostomía), el cirujano deberá hacer una incisión en la piel del abdomen ¿cuál de las siguientes
raíces nerviosas debe ser anestesiada para este procedimiento?
a. T8

300.El mecanismo de la defecación incluye la participación de diversas estructuras ¿Cuál de las siguientes
alternativas es correcta?
a. Puede ser mediado por un reflejo intrínseco

301.Cuando el contenido del estómago ingresa al duodeno, uno de los reflejos que inhiben el vaciamiento
gástrico es a través del:
a. sistema nervioso mientérico

302.Durante la masticación, gran parte del proceso masticatorio se debe a:


a. el reflejo masticatorio

303.Las glándulas salivales tienes conductos para la excreción de la saliva; las glándulas ____________
drenan en las carúnculas sublinguales.
a. sublinguales

304.En una persona si enfermedad se espera que el tránsito intestinal se vea disminuido cuando se presenta
el reflejo:
a. doloroso

305.El divertículo de Meckel es una anomalía congénita que ocurre por la persistencia del conducto vitelino y
da origen a una estructura sacular, el cual se encuentra en el:
a. borde antimesentérico

306.Si al intubar a un paciente, por error se ingresa el tubo endotraqueal en el esófago y se insufla el
manguito endotraqueal (globo TET), la dilatación de este manguito generará:
a. múltiples ondas secundarias

307.El orificio omental, o hiato de Winslow, se encuentra limitado por el ligamento:


a. hepatoduodenal

308.Paciente de 24 años acude a consulta externa por presentar una fístula oronasal (comunicación entre la
cavidad oral y la cavidad nasal). Está fístula es una consecuencia tardía de la lesión de un vaso
sanguíneo por el antecedente de haber sido operado de paladar hendido en los primeros años de vida,
aparentemente en una campaña gratuita de corrección de paladar fisurado. ¿Cuál de las arterias
palatinas podría haberse lesionado durante esa cirugía?
a. Mayor

309.Dentro de las anomalías congénitas se puede presentar un tejido pancreático accesorio ¿Cuál es la
ubicación más común de este tejido?
a. Estómago

310.Paciente con insuficiencia mitral moderada a severa, con aumento de volumen de la aurícula izquierda;
esta condición tendrá como consecuencia a nivel del sistema digestivo:
a. la disfagia a sólidos

311.El inicio de la fase faríngea de la deglución se debe a estímulos sensitivos que viajan por el nervio
craneal:
a. V (nervio maxilar)

312.El mesocolon transverso se origina en:


a. la pared posterior del abdomen
313. La contracción del músculo ………………………… permite la eliminación de gases (flatos) sin salida de
material fecal ; es el mismo músculo cuya relajación, sobretodo en cuclillas, permite el paso del contenido
fecal con menor esfuerzo durante la defecación
a. puborrectal
314. Paciente mujer de 54 años se presenta con náuseas, vómitos, estreñimiento, y es diagnosticada de
abdomen agudo quirúrgico; en la cirugía encuentran un vólvulo de ciego. Esta anomalía puede explicarse
por:
a. Falta de fusión del mesenterio
315.Paciente mujer de 23 años con faringitis aguda , toma para el dolor una tableta de paracetamol con un
poco de agua. Durante la deglución, se relaja su esfínter esofágico inferior y el fondo del estómago,
mientras el bolo está aún en el esófago. ¿Qué sustancia provocará con mayor probabilidad la relajación
del esfínter esofágico inferior y el fondo del estómago en esta mujer?
a. Óxido nítrico

316.Luego de tres horas dando exámenes, un alumno de medicina comienza a sentir hambre. Esta situación
es probable que sea mediada por la _____ que es sintetizada por el _____
a. grelina / estómago

317.Varón de 72 años, con antecedente de diabetes mellitus tipo 2, que presenta entropía diabética
caracterizada por estreñimiento. Este problema puede estar asociado a:
a. deficiencia de óxido nítrico

318.Varón de 54 años con diabetes mellitus tipo 2, es diagnosticado de gastroparesia debido a que presenta
sensación de llenura precoz al comer, y reflujo gastroesofágico. Esta alteración en la relajación receptiva
y en el vaciamiento gástrico lo más probable es que se deba a una alteración en:
a. el nervio vago

319.Varón de 67 años con tos y disminución de peso asociado a tabaquismo pesado, presenta actualmente
disfagia progresiva a alimentos sólidos. Se considera la presencia de un carcinoma de bronquio izquierdo
y por esta razón le realizan una endoscopia esofágica para descartar la posibilidad de una compresión
esofágica por el tumor. Se espera revisar el esófago en la _____ estrechez, que está a nivel de la
vértebra _____.
a. Tercera estrechez - T6

320.En un niño menor de dos años con divertículo intestinal, este divertículo tiene su origen en una falla en la
obliteración de:
a. Conducto vitelino

321.Mujer de 43 años sufre un grave accidente de tránsito y está hospitalizada en coma, es alimentada por
vía intravenosa durante varias semanas. Producto de este tipo de alimentación, se encuentra en la
endoscopia atrofia de la mucosa gastrointestinal. La causa más probable de esta atrofia son los bajos
niveles séricos de la hormona:
a. Gastrina

322.Una mujer de 30 años llega al consultorio porque se queja de dificultades para deglutir, la cual se agravan
cada vez más. Se realiza un estudio manométrico para examinar la generación de presión a lo largo del
esófago. Esta prueba revela que las contracciones como respuesta a la deglución están mal
sincronizadas y que la presión en el esfínter esofágico inferior permanece elevada. El diagnóstico más
probable es _____ producido por niveles bajos de _____:
a. acalasia / óxido nítrico
323.Paciente de 2 años, llega a emergencia por haber ingerido una moneda con la que estaba jugando. El
lugar más probable donde puede haberse quedado suspendido este objeto es a nivel del estrechamiento
producido a nivel del:
a. músculo cricofaríngeo

324.En una apendicectomía, al realizar la incisión de McBurney en la fosa iliaca derecha, es necesario cortar
los siguientes músculos, de afuera hacia adentro:
a. Oblicuo externo - Oblicuo interno - Transverso
325. Un varón de 90 años que se encuentra postrado en cama, es referido del asilo para endoscopia por
dificultad para deglutir luego de tomar un medicamento para aliviar el dolor de la noche anterior. La
endoscopia revela que la píldora se alojó en el esófago y causó una reacción inflamatoria. Lo más
probable es que esto haya sido por la producción de múltiples ondas:
a. secundarias
326.Mujer de 23 años es diagnosticada de bulimia, al examen físico se observa ulceraciones en el segundo y
tercer dedo de la mano derecha. Esto se puede deber al uso continuo de estos dedos para inducir el
vómito, mediante la estimulación del par craneal:
a. IX

327.Varón de 52 años se presenta por diarrea persistente de seis semanas de duración. En la colonoscopia
se observa un pólipo a nivel del íleon distal. El patólogo informa que se trata de un tumor neuroendocrino,
probablemente originado por las células enterocromafines del intestino. La sustancia que más
probablemente esté produciendo este tumor es:
a. Serotonina

328.La fase oclusal de la masticación se realiza con la contracción de los músculos:


a. masetero y temporal

329.Al tomar su café en Starbucks, un estudiante de medicina sufre una quemadura de primer grado en el
tercio anterior de la superficie dorsal de la lengua. La información de dolor es transmitida por el nervio:
a. Lingual

330.Paciente es evaluado por faringitis aguda en consultorio externo. El médico de familia le solicita que abra
la boca y saque la lengua Para realizar la acción de sacar la lengua, es necesario que se contraiga el
músculo:
a. geniogloso

331.Paciente con síndrome de Sjogren, presenta “boca seca” (disminución de la producción de saliva) y
caries dental, asociada a la pérdida de la función de tampón de la saliva. Esta desmineralización del
diente puede comprometer a las prolongaciones citoplasmáticas ubicadas en los tubos huecos de la
estructura señalada con la letra:
a. B

332.Una mujer de 32 años acude a consulta por presentar disfagia de progresión lenta, reflujo
gastroesofágico y vómitos desde hace 3 meses de evolución progresiva. Se le realiza un estudio
radiológico con contraste en el que se observa estrechamiento del esfínter esofágico inferior (imagen).
Según sus conocimientos este paciente se beneficiaría con el uso de:
a. análogo de óxido nítrico

333. ¿Cuál de las glándulas salivales


1. Al examinar a un paciente, usted encuentra dolor localizado en fosa iliaca derecha y diagnostica apendicitis.
En este paciente, usted puede inferir:
El peritoneo parietal regional está afectado
2. La motilidad intestinal es estimulada principalmente por el:
Plexo de Auerbach
3. Durante el vómito, ¿el contenido gástrico tiene que pasar necesariamente por cuál estructura para llegar al
esófago? Marque la mejor respuesta:
Cardias
4. Respecto a la anatomía del estómago, marque lo correcto:
El fondo gástrico forma la curvatura mayor
5. Marque la respuesta incorrecta:
En todo el tubo digestivo, se observa dos capas de muscular propia: circular interna y longitudinal externa
6. Paciente se queja de dolor en hipocondrio derecho, pero superficialmente. El dermatoma relacionado es
(marque la mejor respuesta):
T9
7. Dentro de las funciones del abdomen, se encuentra la defecación y micción, en las cuales la presión intra
abdominal debe:
Aumentar
8. Un alumno de medicina decide hacerse un piercing en el ombligo. Al realizarle el procedimiento, sangra
ligeramente. Esta sangre proviene de la arteria (marque la mejor respuesta)
Epigástrica inferior
9. Señale la respuesta correcta:
El apéndice cecal sólo tiene serosa
10. Paciente mujer es traída a emergencia por sufrir una herida contuso penetrante por cuchillo realizada por
su esposo en un ataque de celos. Se observa herida en flanco izquierdo. Esta solución de continuidad ha
comprometido varios músculos de la pared abdominal, excepto:
Recto abdominal
11. Paciente con herida por proyectil por arma de fuego, con herida de ingreso en región paraumbilical. Entre
las estructuras que usted está seguro que debe haberse lesionado es:
Omento mayor
12. En la evaluación de una tomografía abdominal, el interno observa un aneurisma en una arteria que se dirige
al riñón derecho. Con seguridad se puede afirmar que está a nivel de la vértebra:
L1
13. Paciente joven es traído a emergencia con abdomen agudo quirúrgico debido a herida contuso penetrante
por verduguillo (alambre grueso con punta aguzada) recibida en una pelea después de un partido de fútbol.
Se observa herida en Hipocondrio Izquierdo. El órgano que debe estar sangrando y produciendo
hemoperitoneo es (marque la mejor respuesta):
Bazo
14. Marque el órgano que se encuentra más distal en el tubo digestivo.
Ciego
15. La peristalsis o peristaltismo hace referencia a:
Motilidad para movilizar el alimento de proximal a distal.
16. Marque la respuesta incorrecta:
En todo el tubo digestivo, se observa dos capas de muscular propia: circular interna y longitudinal externa
17. Paciente con vólvulo del colon sigmoides. La necrosis de ese segmento del colon se produce por una
alteración en la irrigación de la arteria: Mesentérica inferior
18. Marque el órgano que se considera retroperitoneal: Parte de la vía biliar
19. Un alumno de medicina decide hacerse un piercing en el ombligo. Al realizarle el procedimiento, sangra
ligeramente. Esta sangre proviene de la arteria (marque la mejor respuesta): Epigástrica inferior
20. Paciente tiene una úlcera sangrante en el segundo tercio del Yeyuno. La arteria de la cual proviene la sangre
arterial para dicha zona es la arteria: Mesentérica superior
21. Es inervado por aferentes somáticas: Peritoneo parietal
22. Paciente de 24 años con dolor abdominal tipo cólico intenso en mesogastrio. Según sus conocimientos de
macroestructura, el origen del dolor puede ser ….: Íleon
23. Paciente con hipoglucemia secundaria a un insulinoma (tumor neuroendócrino productor de insulina). El
órgano donde mayor probabilidad ha crecido este tumor es: retroperitoneal
24. Al evaluar una tomografía abdominal, el médico asistente le pide al interno de la UPC que encuentre la
imagen con el corte a nivel de L1. El interno sabiamente busca el …….. para ubicar la vértebra L1: Cuello del
páncreas
25. En la inspiración, la pared abdominal debe …. para ….. : relajarse disminuir presión intra torácica
26. Ligamento hepatogástrico une el ….. con el …… y forma la entrada al …… : Hígado Estómago Orificio omental
27. Al retirar completamente el mesenterio de un órgano, el mismo se vería afectado principalmente en su:
Irrigación
28. La estructura que fija órganos principalmente a la pared posterior abdominal se denomina: Mesenterio
29. Cuál de las siguientes estructuras no tiene vasos sanguíneos: Epitelio intestinal
30. Al iniciar la digestión, aumenta el consumo de oxígeno por la mucosa. Esto conlleva a una hipoxia local, lo
cual hace que se libere …., el cual produce vasodilatación: adenosina
31. Sustancia que inhibe la secreción y la motilidad del estómago prolongando el tiempo de digestión: péptido
insulinotrópico dependiente de la glucosa (GIP)
32. Marque lo correcto: La hernia fisiológica se produce en la sexta semana y es la salida temporal de asas
intestinales a través del colon umbilical
33. Marque la respuesta correcta en relación a la gastrina: Las células G son las productoras y se encuentran
principalmente en el antro gástrico.
34. El consumir caramelos indirectamente activa la vía: POMC/CART
35. ¿En qué capa se encuentra la alteración principal en el Hirschsprung o megacolon agangliónico?: Muscular
propia
36. Con respecto a las ondas lentas, marque la afirmación correcta: Son contracciones rítmicas espontáneas
37. El uso de Ranitidina bloquea el receptor H2 de la histamina en las células parietales. La histamina llega a
estas células por: Difusión
38. La triada sintomática: vómitos explosivos post-prandiales, movimientos peristálticos epigástricos visibles
de izquierda a derecha y nódulos palpable epigástrico subcostal derecho, pertenecen a: Estenosis congénita
hipertrófica del píloro
39. Durante una cirugía oncológica, ¿la extirpación de cuál de los siguientes órganos se vería comprometida
por la presencia de adventicia?: Recto
40. En cuanto a los reflejos gastrointestinales, un reflejo que estimula el tránsito intestinal es el reflejo:
Gastrocólico
41. El ligamento falciforme divide al hígado en dos lóbulos derecho e izquierdo. Embriológicamente deriva del:
Mesenterio ventral
42. La presencia de atresias y estenosis duodenales se deben básicamente a una: Falta de recanalización
43. Estudiante de medicina de 20 años, se ha amanecido estudiando para su examen de Sistema Digestivo. No
ha probado alimento desde la cena, por lo que se puede afirmar que la motilidad de esta persona está
siendo regulada por: Motilina
44. Paciente con disminución del apetito marcada asociada a cáncer terminal, para promover la ingesta de
alimentos se podría usar análogos de: Endorfinas
45. Las ondas lentas se producen por la apertura cíclica de canales de: Calcio
46. La forma más común de atresia esofágica contiene: Estenosis proximal del esófago más fístula
traqueoesofágica distal
47. Al deglutir un bolo alimenticio, es lógico suponer que al pasar por el esofago haya un mayor consumo de
oxigeno en la pared del tercio: Proximal
48. Paciente que come entera una pizza familiar de chorizo y queso. Es posible esperar que debido a la cantidad
de alimento ingerida, las ondas lentas hayan: sufrido ninguna alteración en su frecuencia.
49. La hernia fisiológica se produce dentro de: cordón umbilical
50. El crecimiento de un adenocarcinoma de páncreas compromete la pared gástrica por contigüidad ¿Que
parte del estómago se esperaría esté comprometido?: Pared posterior del antro
51. Estimula la producción de saliva: Vasodilatación periglandular
52. Durante la secreción de saliva, es de esperarse que las concentraciones de ….. y …… disminuyan al disminuir
el flujo: Sodio Bicarbonato
53. Con respecto a la secreción gástrica de HCI: a mayor secreción de HCI en el lumen gástrico, mayor pH en la
sangre venosa gástrica.
54. Respecto a las enfermedades del esófago, marque lo correcto: el diagnóstico diferencial de la acalasia es la
enfermedad de Chagas esofágica
55. Con respecto a las lesiones y enfermedades de la boca, marque lo correcto: la eritroplasia debe ser
biopsiada
56. Respecto a las glándulas salivales, marque lo incorrecto:
a. la glándula sublingual tiene forma de garfio
b. la glándula sublingual drena a través del conducto de wharton
c. la glándula parótida produce secreción serosa
d. la glandula parotida drena a través del conducto de Stenon
57. El omeprazol actúa sobre la membrana ….. de la célula ….. : apical/parietal
58. Durante el sueño, la concentración de bicarbonato en la saliva: Disminuye
59. Durante el ataque con gas sarín (bloqueador de la acetilcolinesterasa) en el metro de Tokio, en 1995, el
personal de salud noto que los pacientes afectados presentaban: Hipersalivación
60. La célula mucosa del cuello gástrico produce: Moco

Parcial 2020-01 (1)

1. Al evaluar la orofaringe de un paciente, el médico le solicita que abra la boca, saque la lengua y diga a. Al
hacer esta maniobra, nota que el paladar se desvía hacia la derecha, lo cual le hace sospechar que el
paciente sufre de una lesión del nervio craneal: X contralateral
2. Un bolo alimenticio grande y poco masticado se atasca en el esofago, esto ocasiona una sensación de
dolor que es transmitida por los nervios: esplácnicos
3. Para realizar el movimiento mecánico de abrir la boca, primero se necesita: fijar el hueso hioides
4. ¿Cual de las siguientes alternativas se define como la protrusión directa del contenido abdominal a la
cavidad amniótica por un defecto de la pared corporal?: Gastrosquisis
5. Un paciente requiere que se le coloque una sonda de alimentación directamente al estómago
(gastrostomía), el cirujano deberá hacer una incisión en la piel del abdomen ¿cuál de las siguientes raíces
nerviosas debe ser anestesiada para este procedimiento? T8
6. En un paciente de 43 años con tumor carcinoide de páncreas productor de gastrina (Síndrome de
Zollinger-Ellison) se puede esperar encontrar una potenciación del reflejo: gastrocólico
7. El mecanismo de la defecación incluye la participación de diversas estructuras ¿cuál de las siguientes
alternativas es correcta?: Puede ser mediado por un reflejo intrínseco
8. Cuando el contenido del estómago ingresa al duodeno, uno de los reflejos que inhiben el vaciamiento
gástrico es a través del: Sistema nervioso mientérico
9. Durante la masticación, gran parte del proceso masticatorio se debe a: El reflejo masticatorio
10. Las glándulas salivales tienen conductos para la excreción de la saliva, las glándulas …… drenan en las
carúnculas sublinguales. RPTA: Sublinguales
11. Los diferentes segmentos del tubo digestivo son susceptibles de reflejos y movimientos según su
contenido. Si colocamos mediante una sonda un bolo alimenticio directamente en el tercio medio del
esofago: se producira ondas secundarias
12. En una persona si enfermedad se espera que el tránsito intestinal se vea disminuido cuando se presenta el
reflejo: Doloroso
13. El divertículo de Meckel es una anomalía congénita que ocurre por la persistencia del conducto vitelino y
da origen a una estructura sacular, el cual se encuentra en el: borde antimesentérico
14. Si al intubar a un paciente, por error se ingresa el tubo endotraqueal en el esofago y se insufla el
manguito endotraqueal (globo TET), la dilatación de este manguito generará: múltiples ondas secundarias
15. El orificio omental, o hiato de Winslow, se encuentra limitado por el ligamento: hepatoduodenal
16. Paciente de 24 años acude a consulta externa por presentar una fístula oronasal (comunicación entre la
cavidad oral y la cavidad nasal). Esta fístula es una consecuencia tardía de la lesión de un vaso sanguíneo
por el antecedente de haber sido operado de paladar hendido en los primeros años de vida,
aparentemente en una campaña gratuita de corrección de paladar fisurado. ¿Cual de las arterias palatinas
podría haberse lesionado durante esa cirugía?: Mayor
17. Dentro de las anomalías congénitas se puede presentar un tejido pancreático accesorio ¿cuál es la
ubicación más común de este tejido?: Estómago
18. Una recién nacida es evaluada por el neonatólogo y evidencia que el canal anal está completamente
cerrado. Este problema se debe probablemente a una anomalía en el desarrollo de: la membrana cloacal
19. En la digestión de los alimentos, la hormona __________ se libera frente a la presencia de péptidos y
monoglicéridos, y tiene un efecto marcado en la disminución del vaciamiento gástrico →
colecistoquinina
20. Dentro de las anomalías congénitas se puede presentar un tejido pancreático accesorio ¿Cuál es la
ubicación más común de este tejido? → Estómago
21. Los catadores de vino tienen una habilidad increíble al momento de separar los sabores. Este aumento de
la sensibilidad gustativa debido a una mayor cantidad de papilas linguales y de corpúsculos gustativos se
conoce como: hipergeusia
22. En muchos países se usa el suplemento de flúor en el agua potable o los dentífricos, con el fin de hacer el
esmalte más resistente a la desmineralización inducida directamente por: el ácido
23. Durante el desarrollo de la región cloacal, una cuña de mesodermo ubicado entre el alantoides y el
intestino posterior vendrá a formar el: tabique urorrectal
24. En una persona sana, el momento adecuado para encontrar los mayores niveles de grelina en sangre
sería: antes de comer
25. El duodeno está constituido por el segmento terminal del intestino anterior y el segmento proximal del
intestino medio ¿Cuál de las siguientes alternativas describe mejor este lugar de unión entre los dos
intestinos?
Distal al origen de la yema hepática
26. En una persona sana, el uso de atropina producirá a nivel del estómago: Aumentará el pH del estómago
27. En una persona sana, el consumo de leche produce indirectamente → Inhibición del vaciamiento
gástrico
28. La motilidad del colon es importante y lenta comparada con la del intestino delgado. Los movimientos en
masa ocasionan la: distensión rectal
29. Con respecto a la motilidad gástrica, los potenciales de acción disminuyen en frecuencia por efecto de:
el péptido insulinotrópico dependiente de glucosa
30. La sensación del gusto depende de la presencia de papilas gustativas en la lengua, de las cuales, algunas
de ellas tienen un surco terminal por donde drenan unas glándulas salivales linguales (llamadas glándulas
de von Ebner). Esta descripción se refiere a las papilas: circunvaladas
31. Dentro de la estructura de los dientes, la parte del diente cubierta por esmalte y que se puede ver
mediante la inspección visual de la boca se denomina → corona clínica
32. En el conducto anal se encuentra la unión entre las regiones del endodermo y el ectodermo, esta unión se
evidencia al observar: la línea pectínea
33. La sensación del gusto depende de la presencia de papilas gustativas en la lengua, las cuales tienen
corpúsculos gustativos conteniendo células neuroepiteliales sensoriales. Estas células neuroepiteliales
pueden ser dañadas fácilmente, por suerte, su tiempo de recambio es de alrededor de: 10 días
34. En un varón de 47 años con sección medular a nivel de T6 debido a un accidente automovilístico, sus
terapeutas han desarrollado un mecanismo para distender el recto e iniciar el reflejo rectoesfinteriano, lo
cual producirá la contracción de: la pared del recto
35. Durante un experimento, se insufla rápidamente dos litros de agua en un globo colocado dentro
del estómago de un voluntario. ¿cuál de las siguientes situaciones del músculo liso será
consecuencia directa de este cambio de volumen en el estómago? → Despolarización
36. Paciente de 56 años con accidente cerebrovascular reciente. En la resonancia se observa daño de los
núcleos laterales del hipotálamo. Por este motivo es muy probable que el paciente sufra de: inanición
37. Paciente con insuficiencia mitral moderada a severa, con aumento de volumen de la aurícula izquierda,
esta condición tendrá como consecuencia a nivel del sistema digestivo: la disfagia a sólidos
38. El inicio de la fase faríngea de la deglución se debe a estímulos sensitivos que viajan por el nervio craneal:
V
39. El mesocolon transverso se origina en: la pared posterior del abdomen
40. El esofago en su microestructura tiene basicamente adventicia, a excepción de la región distal, donde
tiene serosa, específicamente a partir del nivel de: T10
41. En un paciente con arcadas, se debe considerar que durante la ocurrencia de dichas arcadas, debemos
encontrar contenido gástrico en: Tórax
42. La reabsorción de Sodio y Cloro en las glándulas salivales se da principalmente en el: Conducto estriado
43. Para que se puedan digerir las grasas, es preferible que primero sean emulsificadas. La hormona que
estimula la liberación de las sustancias emulsificadoras es: CCK
44. Paciente de 13 días de vida con vómitos explosivos a las 2 horas después de lactar. Al examen físico se
palpa la oliva pilórica ¿cual es el nervio cuyos filetes dan inervación eferente a la estructura afectada?:
Vago
45. La localización de la vesícula biliar con respecto al lóbulo cuadrado es: Lateral
46. La triada portal está constituida por el conducto biliar y la arteria hepática y una pequeña rama de la
vena: Porta
47. Al ingerir una cantidad de glucosa por vía oral, esta es interiorizada en las células del organismo más
rápido que si esa misma cantidad hubiese sido administrada por vía endovenosa. Este fenómeno sucede
gracias a una sustancia secretada por las células: K
48. Paciente de 62 años con vólvulo de intestino delgado e isquemia intestinal. ¿Qué estructura se utiliza
como punto de referencia para determinar la posición de la unión duodenoyeyunal? : Ligamento
suspensorio del duodeno (de Treitz)
49. En un paciente con Zollinger Ellison, usted esperaría encontrar: Esteatorrea
50. La motilidad intestinal es estimulada por: Colecistoquinina y gastrina
51. La onda peristática secundaria del esofago en la deglución, es producida por: Plexo mientérico esofágico
52. Paciente con parálisis bilateral del nervio hipogloso, el unico musculo de la lengua que conservará su
movimiento es el: palatogloso
53. ¿Cual de las siguientes sustancias tiene mayor concentración de la saliva comparado con su concentración
plasmática? Potasio
54. ¿Cual de las siguientes estimula la secreción enzimática exocrina del páncreas?: Colecistoquinina
55. Al comer un pollo a la brasa, con papas fritas y ensalada, la sustancia que estimula la liberación de HCI en
el estómago es: Bombesina
56. Marque lo correcto con respecto a Esófago de Barret: Se relaciona con reflujo gastroesofágico

ECU 1
Estudiante de medicina de la UPC de 21 años sufre de gastritis aguda ocasionada por comer en lugares poco
higiénicos. Suele consumir caramelos ( chupar ) mientras está en clase hasta la tarde. Toma gaseosas regularmente
(carbohidratos 46%, sodio 53%). También toma regular cantidad de leche (grasa 35%, lactosa 35%, proteínas 30%),
pues le calma un poco el dolor el ardor que siente por la gastritis. Incluso, cuando puede, se toma dos vasos de
agua fría para calmar las molestias. Ha decidido ir al médico para tratarse pues ya no soporta el dolor, el cual está
seguro que los síntomas se deben a una elevada producción de ácido clorhídrico en el estómago, y por ello le ha
recetado Ranitidina (antihistamínico), con lo que siente mejoría.

El uso de atropina en este paciente:


- Aumentará el pH del estómago

Entre las sustancias cerebrales que producen ansiedad está la serotonina, la cual también tiene acción:
- Anorexigénica

El consumo de dos vasos de agua seguidos agua generará indirectamente un aumento en la liberación de:
- Ácido clorhídrico

En este paciente con gastritis aguda debida a una alta producción de ácido clorhídrico, sería lógico esperar que el
píloro tenga un tono muscular:
- Aumentado

El consumo de leche produce directamente un aumento de los niveles séricos de la hormona:


- Colecistoquinina (CCK)

El consumo de leche produce directamente un aumento de los niveles séricos de la hormona:


- Gastrina

ECU 2
Niño de sexo masculino de 2 años de edad, sufre de estreñimiento desde el nacimiento (1 deposición cada 3-4
días). Madre menciona que le estimula la defecación con un termómetro rectal, y continuo uso de enemas y
laxantes. Desde hace 6 meses comienza con vómitos postprandiales. Los síntomas aumentan en frecuencia y
magnitud y están en relación con los episodios de estreñimiento. No refiere fiebre, tos, diarrea ni lesiones
cutáneas. Al examen físico presenta regular estado general, luce deshidratado. Abdomen distendido, blando,
depresible e indoloro. No se palpan masas abdominales. Se permeabiliza el canal anal con termómetro rectal,
encontrando cierta resistencia. Salida de material fecal mal oliente en regular cantidad. Exámenes de laboratorio:
hemograma normal. Signos inflamatorios de fase aguda negativos. Alcalosis metabólica leve en sangre venosa.
Radiografía con enema baritado muestra recto y colon sigmoides dilatados (megacolon). Biopsia profunda:
ausencia de células ganglionares en la muestra enviada. Se realiza cirugía correctiva.

El contenido fecal se detiene en la zona inmediatamente proximal a la zona donde hay una menor presencia de:
- Péptido intestinal vasoactivo

En cuanto a los reflejos gastrocólico y gastroduodenal en este paciente, indique lo correcto:


- Se pueden considerar reflejos vago-vagales

En este paciente se considera que está abolido el reflejo:


- Rectoesfinteriano

Debido al acúmulo de material fecal en todo el marco colónico, y a la irritación química asociada, el peristaltismo
del íleon distal se debe encontrar:
- Inhibido

Es un reflejo propio de la pared intestinal:


- Peristaltismo

A diferencia de las arcadas, los vómitos presentan apertura de:


- Esfínter esofágico superior

1. Un niño de 2 años es llevado a la consulta por diarrea persistente y edema de las


extremidades, además falta de crecimiento y desarrollo en relación a su edad. Los
análisis de sangre revelan que tiene concentración plasmática baja de proteínas
(hipoproteinemia). Durante la endoscopía duodenal, se coloca colecistocinina (CCK)
endovenosa y se recoge muestras del líquido duodenal; el resultado del líquido
confirma incapacidad para hidrolizar proteínas a un pH neutro, esta situación mejora
al añadir una pequeña cantidad de tripsina. El paciente probablemente esté sufriendo
la falta congénita de
-Enterocinasa
2. Experimentalmente se incrementa la velocidad de la secreción salival con una
sustancia, el análisis de la composición de esta saliva obtenida se espera
encontrar…………..
-Disminución de concentración de potasio
3. Paciente varón de 46 años soltero, consulta por odinofagia y bajo de peso, tiene
antecedente de tuberculosis desde hace 3 meses y es fumador crónico (10 cigarrillos
por día); al evaluar la cavidad oral se identifica lesión blanquecina en el dorso de la
lengua y paladar blando, las lesiones se desprenden con el baja lengua dejando una
base eritematosa. Esta lesión corresponde probablemente a
……………………….…..
-Candidiasis oral
4. Paciente mujer de 35 años acude a consulta por sensación de sequedad y lesiones
en cavidad oral. Al examen se observa atrofia de la mucosa, fisuras y úlceras; nota
además sequedad e irritación de la córnea y aumento del tamaño de las glándulas
parotídeas. Su diagnóstico más probable es artritis reumatoide; el hallazgo más
probable en una biopsia de glándula parótida es……..….
-Gran infiltración de linfocitos y células plasmáticas
5. Un paciente con anemia acude con su médico quejándose de episodios frecuentes
de gastroenteritis. Un análisis de sangre revela anticuerpos circulantes dirigidos
contra células parietales gástricas. Su anemia es atribuible a la hiposecreción de
-Factor intrínseco
6. Dos estudiantes deciden tomar un receso para comer una hamburguesa a la hora
del almuerzo. Antes de llegar a la cafetería, impulsos nerviosos provenientes del
complejo vagal dorsal iniciarán la secreción de ácido gástrico por la liberación de
…………………….. desde el sistema nervioso entérico.
-GRP
7. Un niño de cuatro años de edad es llevado a la consulta por cuadros diarreicos
frecuentes caracterizados por heces pálidas, voluminosas y fétidas, presenta bajo
peso y talla. Se mide la concentración de cloruro en el sudor y se encuentra que sus
valores son muy elevados. La alteración más importante a nivel de células ductales
del páncreas tiene relación directa con la conductancia de…………
-Cloro
8. Una mujer de 50 años de edad que sufrió durante varios años resequedad de los
ojos debida a producción inadecuada de lágrimas es enviada con un
gastroenterólogo para evaluación de pirosis crónica. El examen endoscópico
revela erosiones y tejido cicatrizal en la parte distal del esófago justo por arriba del
esfínter esofágico inferior. Las lesiones pueden atribuirse a la disminución de uno
de los siguientes componentes salivales:
-Bicarbonato

9. Se evalúa los valores séricos de las siguientes sustancias a un paciente con


enfermedad hepática terminal; en este paciente se espera encontrar la combinación
con la letra …………
-disminuida, aumentada, disminuida
10. Una mujer de 35 años de edad HIV positiva, se presenta al médico con dolor
abdominal en cuadrante superior derecho e ictericia. La paciente refiere haber tenido
múltiples episodios de ictericia durante los últimos 10 años. Los exámenes para
determinar hepatitis viral, dieron positivos para Hepatitis B, siendo catalogado el caso
como hepatitis crónica con alteración funcional. En un examen de sangre ¿cuál de los
siguientes parámetros está disminuido?
-Albúmina
11. En el reflejo peristáltico del intestino delgado, uno de los siguientes eventos sucede
en la porción oral del bolo alimenticio…………...
-Acción de acetilcolina en el músculo circular
12. Experimentalmente se coloca una dosis alta de secretina en la luz intestinal
duodenal; como consecuencia de esto, en el jugo pancreático de la misma luz
intestinal se observa la disminución de la concentración de …..………..
-Cl
13. Un varón de 58 años de edad con enfermedad de Crohn severo fue sometido a una
resección ileal. Después de la cirugía este paciente padecerá de esteatorrea, esto se
explica porque …..………..
- La micelas no pueden formarse
14. En un experimento se inserta un balón en el estómago de un voluntario, se infla poco a
poco mientras que se vigilan las presiones intraluminales. Aunque el volumen del
balón aumenta considerablemente, las presiones permanecen constantes. Esta
relación volumen-presión se explica por la liberación local de …………..
-Óxido nítrico y péptido inhibidor vasoactivo
15. La toxina de Vibrio cholerae causa diarrea debido a…….
-El Incremento de la secreción de cloro por las células de la cripta intestinal
16. ¿Cuál de las siguientes alternativas es una característica de la secreción exocrina
del páncreas?
-Tiene una baja concentración de Cl- respecto al plasma
17. Una madre lleva a su hijo de dos años de edad a la sala de urgencias, estresada
porque el niño deglutió una moneda de 10 céntimos mientras la familia cenaba en un
restaurante. El médico observa mediante fluoroscopía que la moneda se halla en el
estómago y asegura a la madre que la moneda se eliminará con las heces. El médico
recomienda utilizar la respuesta fisiológica que permitirá la evacuación de la moneda
del estómago al intestino ………….…..
-Son los movimientos de mezcla y trituración
-. Es provocada por el ayuno
18. Las estructuras en el hígado que permite que los productos metabólicos unidos a
proteínas tengan acceso a las membranas basolaterales de los hepatocitos, son….. -
Las fenestras sinusoidales
19. La composición de la bilis es modificada conforme fluye por los conductillos biliares.
Durante este tránsito se espera que aumente la concentración de…….
-Monómeros de ácido biliar
-Ig A
20. Se mide experimentalmente el contenido gástrico de dos personas. La persona “A”
tiene alto contenido de grasa y la persona “B” tiene un contenido hipertónico ¿Cuál de
las siguientes es correcto respecto al vaciamiento gástrico?
- Hay ralentización del vaciado gástrico en ambos casos
21. El examen endoscópico de un paciente con hipertensión portal grave revela venas
tortuosas que sobresalen hacia la luz del esófago. El paciente recibe tratamiento
quirúrgico mediante la colocación de una derivación que conecta la vena porta a la
vena cava. Después de la operación el riesgo de encefalopatía y el
riesgo de sangrado de várices ……………..
-Aumentará/disminuirá
22. Un paciente varón de 18 años de edad acude al médico para sus exámenes de
rutina. Sus resultados de laboratorio muestran un valor de bilirrubina sérica de 4
mg/dl y una bilirrubina directa de 0,3 mg/dl. Las pruebas de función hepática son
normales. La alteración que explica mejor este caso es por la deficiencia de
………………..
-Glucuronil transferasa
23. Un hombre de 57 años de edad es llevado a urgencias con hematemesis masiva
rojo brillante, a su llegada se halla inconciente con PA: 80/40 mm Hg y FC: 124
lat/min. Luce ictérico con presencia de “arañas vasculares en el tórax anterior y
extremidades”, abdomen distendido con signo de oleada positiva. Se encuentra
esplenomegalia y pérdida de la masa muscular en extremidades. La anastomosis
vascular responsable del sangrado en este paciente es
-Vena gástrica izquierda y vena ácigos
24. Un estudiante de medicina está comiendo un plato de comida a base de
champiñones, espárrago y salsa de soya. El sabor umami contenido en todos estos
alimentos actúa a nivel de los botones gustativos estimulando ………………..
-Un receptor acoplado a proteína G
25. Un hombre de 22 años de edad se presenta al médico con una historia de 1 año de
evolución caracterizado por dolor recurrente en fosa iliaca derecha y diarrea.
Manifiesta además pérdida de peso de 8 kg durante este periodo. La colonoscopía
revela múltiples lesiones en el ileon terminal y colon. La biopsia de estas lesiones
revela engrosamiento, inflamación y ulceración de la mucosa. El diagnóstico más
probable en este caso es…….
-Enfermedad de Crohn
26. Varón de 61 años que consulta por dolor retro esternal intenso desde hace 6 horas y
después de vómitos intensos y repetidos; al examen se observa disnea, cianosis,
hipotensión y signos clínicos de shock. La radiografía simple de tórax muestra
neumomediastino. El líquido en el espacio pleural aspirado tiene alta concentración de
amilasa. ¿Cuál de las siguientes alternativas puede explicar este cuadro clínico? -
Rotura espontánea de esófago
27. La secreción del ácido en la célula parietal gástrica se lleva a cabo por una ATPasa
especifica que intercambia hidrogeniones (H+) del citosol por…..
-K +
28. En condiciones normales el ingreso de 600 ml de líquido es el estómago provoca un
aumento de presión intragástrica de unos 12 cm de H2O. Después de una vagotomía
(corte del nervio vago) es de esperar que el ingreso del mismo volumen de líquido
provoque lo siguiente: …………………………………
-Un aumento mayor de la presión
29. Una paciente de 30 años de edad es sometida a una cirugía en oído medio derecho
por un problema de otoesclerosis. Luego de la cirugía refiere alteración en la
percepción de sabores. Al evaluar el caso usted esperaría encontrar……….
-Alteración en la sensación del gusto en los dos tercios anteriores de la
lengua
-Sensación del dolor, tacto y temperatura conservada en toda la lengua

30. ¿Cuál de las siguientes alternativas es correcta?


-Las sales biliares desconjugadas son absorbidas preferentemente en el colon
31. En un paciente de 45 años de edad con colestasis biliar, se encuentra una elevación
de los niveles sanguíneos de fosfatasa alcalina hasta 3 veces la cifra normal. ¿Cuál de
las siguientes alternativas estará también elevada como evidencia del daño de la vía
biliar?
-Gamma glutamil transpeptidasa
32. Revisando la angiografía de un hombre de 70 años en estudio por aneurisma de aorta
abdominal el radiólogo informa de la presencia de una oclusión completa de la arteria
mesentérica inferior. El paciente se encuentra completamente asintomático.
¿Cuál de las siguientes arterias se anastomosa a la sistema arterial de la
mesentérica inferior?
-Cólica media
33. Lactante de 3 meses de vida es atendido por presentar diarrea, se administra una
solución de glucosa y electrólitos por vía oral. La proteína de membrana apical que
explica la capacidad de esta solución para proporcionar aporte de glucosa e
hidratación es ………..
-SGLT-1
34. Paciente ha sufrido herida de bala en el abdomen, se le ha tenido que extirpar el
segmento medio y distal del ileon. En este caso la síntesis hepática de sales biliares
estará …..…..
-Incrementada por estímulo de la enzima colesterol 7 alfa hidroxilasa
35. Un varón de 75 años ingresa al consultorio por presentar ictericia marcada de piel y
las escleras. El estudio del paciente mostró que presentaba un tumor que obstruía la
totalidad del conducto hepático común. ¿Cuál de las siguientes estructuras se
encontrará dilatada en este paciente?
-Conductos de Hering
36. En un paciente con insuficiencia renal crónica, el déficit en la absorción de calcio a
nivel del enterocito se debe a lo siguiente:
-No se convierte la 25 hidroxicolecalciferol a 1,25 dihidroxicolecalciferol
37. Varón de 30 años es traído a emergencia por agresión abdominal con arma de fuego
(pistola) y es sometido a laparotomía exploratoria, observándose isquemia del colon
ascendente y parte del colon trasverso ¿la lesión de cuál de las siguientes arterias
explicaría esta isquemia?
d. Mesentérica superior
38. Respecto a las sustancias gastrointestinales que regulan la secreción pancreática;
marque la afirmación correcta:
b. La acetilcolina es capaz de estimular la secreción enzimática y de bicarbonato del
páncreas
39. Ante una lesión del X par craneal, ¿cuál de los siguientes músculos mantiene
conservada su función?:
b. Tensor del velo del paladar
40. Experimentalmente se utiliza atropina (anticolinérgico) para inhibir la secreción de
gastrina, sin embargo la secreción de esta hormona se sigue dando ante estímulos
vagales. Esta situación se explica porque la atropina:
d. No bloquea la acción del péptido GRP

41. Un varón de 50 años es sometido a extirpación del duodeno y parte proximal del
yeyuno. Esta situación ocasionaría la pérdida de las células ……….. , productoras de
………………… que estimula la secreción de bicarbonato por el páncreas.
“S” / secretina
42. Se evalúa la expresión de la proteína Agrp en una persona con alteración del apetito; lo
correcto respecto a esta proteína es…..
La mutación del gen que la codifica produce adelgazamiento
43. Juana cae de la bicicleta y se fractura la región anterior del hueso maxilar superior con
compromiso de la fosa incisiva. Al examen físico de la región esperaría encontrar
alteración en la sensibilidad de la encía …………………
palatina anterior
44. Recién nacido es atendido por el neonatólogo y luego entregado a su madre para dar
de lactar; la madre al dar de lactar observa coloración azulada de labios, acompañado
de tos persistente, dificultad respiratoria y distención abdominal. Se le intenta colocar
una sonda nasogástrica pero esta retorna a la cavidad oral en todos los intentos. ¿Cuál
de las siguientes anomalías del desarrollo es el más probable en este caso?
b. Atresia esofágica proximal con fístula traqueoesofágica distal l
45. ¿Cuál de los siguientes mecanismos ocurre durante la defecación?
En la posición de “cuclillas” el músculo puborrectal se halla relajado
46. Un paciente luego de un accidente sufre lesión del piso de la boca, se constata daño
del nervio “cuerda del tímpano”, en este caso se esperaría en
47. contrar disminución de la………………………….… de la lengua
Sensación del gusto en los dos tercios anteriores
48. ¿Cuál de las siguientes afirmaciones es la correcta sobre la gastrina?
Actúa en la célula diana mediante su receptor CCk tipo B
49. Al recibir un paciente con signos de hipovolemia y antecedente de trauma en
abdomen por accidente de tránsito, usted identifica radiológicamente: lesión de
primera vértebra lumbar y signos de lesión en páncreas; durante la cirugía se observó
pobre irrigación de asas intestinales. El vaso afectado es la arteria ……..
c. mesentérica inferior
50. Un paciente sufre de daño a nivel del cuello con lesión muscular en la región de la
faringe. En el examen físico se determina dificultad para la elevación de la faringe y para
el cierre del itsmo de las fauces. En este caso, probablemente esté afectado el músculo:
c. palatofaringeo
51. Varón de 50 años a quien le realizan la curación de la segunda molar de la arcada
superior derecha. En un momento determinado, el paciente acusa de intenso dolor
de la pieza dentaria en tratamiento. La vía aferente del dolor viaja a través del
nervio …………
a. trigémino V2
52. La distención gástrica por los alimentos produce incremento de secreción de HCl
mediante la producción de ………….. que estimula a las célulasvía proteína
………..
a. gastrina / parietal / Gq

53. Un niño de tres años llega a emergencia con disfagia (dificultad para tragar), dolor
retro esternal, salivación y llanto. Se sospecha de ingesta de cuerpo extraño (moneda)
en el esófago; al ser evaluado se constata en una radiografía presencia de cuerpo
extraño a nivel de C6 (6° vértebra cervical). El cuerpo extraño estará suspendido a
nivel del estrechamiento producido por………..
c. el músculo cricofaríngeo
54. La triada portal (arteria hepática, vena portal y conducto biliar común) está contenida
en el ligamento …….……… y derivan embriológicamente del ……
a. hepato duodenal / mesenterio ventral
55. Un paciente refiere no percibir algunos sabores, al examen físico se constata
alteración en la percepción de sabores y del dolor en el tercio posterior de la lengua
¿Cuál de los siguientes nervios estará alterada en su función?
c. Glosofaríngeo (IX par)
56. En el caso de un paciente con gastrinoma (tumor productor de gastrina), la presencia
de úlceras duodenales y erosión de la mucosa gástrica, se debe principalmente a…….
c. el exceso de HCl por estímulo de receptores CCK-B en la célula parietal
56. El reflejo entero gástrico se caracterizan por:
d. originarse debido a la distensión duodenal y presencia del quimo ácido
57. Mauricio tiene dificultad para deprimir el paladar y elevar la parte posterior de la lengua.
En este caso estará afectado un músculo, específicamente el músculo
…………….
b. extrínseco – palatogloso
58. En condiciones normales, el ingreso de 600 ml de líquido es el estómago provoca un
aumento de presión intragástrica de unos 12 cm de H2O. Después de una vagotomía (corte
del nervio vago) es de esperar que el ingreso del mismo volumen de líquido ocasione
………………………………… de la presión
intragástrica.
c. un aumento mayor
59. La explicación fisiológica de presentar somnolencia de 30 minutos a 1 hora después
de ingerir alimentos, se explica por: a. Aumento del cloro intraluminal
e. Aumento de la alcalinidad sanguínea
60.Se presenta un paciente, el cual presenta un antecedente de tuberculosis intestinal, por
lo cual, se le resecó 80 cm de íleon distal. Desde el punto de vista fisiológico, el paciente
puede presentar una de las siguientes alteraciones: a. Disminución de la secreción de
Vitamina B12

e. Disminución de la absorción de ácido glicocólico


61. Un paciente es sometido experimentalmente a un fármaco que modifica el flujo salival,
obteniéndose un volumen de saliva de 288 ml en 6 horas. En este caso las concentraciones
de electrolitos y bicarbonato en la saliva obtenida varían de la siguiente manera: a. ↑ Na+,
↓ K+, ↑ Cl-, ↑ HCO3-

1. b. ↓ Na+, ↓ Cl-, ↑ K+, ↓ HCO3-

62. Uno de los siguientes elementos debería hallarse con más probabilidad en el esófago de
un paciente que sufre de reflujo gastro esofágico…
a. Pepsina

63. Un paciente de 40 años cursa con anemia de 8g/dl, aqueja además de astenia y sensación
de hormigueo bilateral en los miembros inferiores, al examen se halla alteración de la
sensibilidad a la vibración y camina con ampliación de la base de sustentación. Uno de los
siguientes procedimientos sería de ayuda para el diagnóstico de este paciente:
a. Tomografía cerebral
b. Biopsia de la mucosa gástrica

64.Paciente de 60 años ingresa por caída hace 1 hora y pequeño hematoma en cuero
cabelludo, al examen físico ampliado se observa ictericia de piel y mucosas generalizada,
abdomen blando, se palpa estructura quística no dolorosa en hipocondrio derecho que
corresponde a vesícula biliar (signo de Courvoisier), en los exámenes de laboratorio se halla
niveles bajos en la formación de estercobilinógeno y urobilinógeno en heces, incremento de la
bilirrubina conjugada en la orina, elevación de fosfatasa alcalina y gamma glutamil
transpeptidasa séricas. El presente cuadro puede ser explicado por: a. Reabsorción de
hematoma
c.Carcinoma de la cabeza de páncreas
65. Un recién nacido presenta vómitos biliosos poco tiempo después de cada alimento. Al
preguntar a la madre sobre antecedentes, ella recuerda que tuvo polihidramnios durante la
gestación, pero un análisis de cariotipo fue normal. Una de las siguientes es la causa más
probable de estos hallazgos en el recién nacido: a. Enfermedad de Hirschprung
e. Malrotación de la yema pancreática ventral
66.En un estudio de la secreción de hormonas gastrointestinales, sus concentraciones en la
vena porta se midieron durante perfusión luminal del intestino delgado con soluciones de
diversas magnitudes de pH. ¿Qué hormona aumentará en el plasma de la vena porta durante
perfusión a través del intestino con una solución de pH 3?
a. CCK
e. secretina
67.Paciente de 30 años que ingresa a causa de un traumatismo abdominal cerrado. En la
exploración se aprecia discreta palidez de piel y mucosas, auscultación pulmonar normal,
taquicardia de 120 /min. Discreta distensión abdominal y matidez en flancos; el hematocrito,
que era prácticamente normal al ingreso, disminuye a 30% a las tres horas. En la Rx de tórax
se objetiva fractura de las costillas 10-11 izquierdas. La causa más probable de la anemización
en este paciente es: a. traumatismo renal con hemorragia retroperitoneal.
c. rotura de bazo con hemoperitoneo.
68. Revisando la angiotomografía de un hombre de 70 años en estudio por aneurisma de
aorta abdominal, el radiólogo le informa de la presencia de una oclusión completa de la
arteria mesentérica inferior. El paciente se encuentra completamente asintomático. La
oclusión de la arteria mesentérica inferior cursa de manera asintomática en muchas
ocasiones ya que el territorio que irriga puede recibir flujo proveniente de la arteria:
a. cólica derecha
e. cólica media

69. En las patologías de esófago es importante conocer bien la anatomía esofágica. ¿Cuál de
las siguientes afirmaciones es correcta? a. El esófago tiene capa mucosa, muscular y serosa

c. El esófago torácico pasa por detrás del cayado aórtico

70. A pesar de que pueda haber variaciones anatómicas, lo habitual es que el ciego sea
irrigado por una rama arterial que proviene de unas de las siguientes arterias: a. Iliaca
derecha

d. Mesentérica superior

71. Ante un paciente con una cirugía abdominal urgente, el informe operatorio señala que se
ha realizado una resección de todo el duodeno y del tercio proximal del yeyuno manteniendo
íntegros el estómago y todo el íleon, así como los dos tercios distales del yeyuno. En el
seguimiento nutricional del paciente ¿Qué vitamina o mineral presentará con menor
probabilidad una disminución de su absorción?
a. Cianocobalamina

72. ¿Cuál de las siguientes sustancias forma parte de la secreción biliar? a. Tripsina
Lecitina

73.¿De qué musculo forma parte el ligamento inguinal?


-Oblicuo externo del abdomen
74.¿Cuál de las siguientes enzimas está localizada en el borde en cepillo y juega un rol en la
digestión de proteínas?
e. Carboxipeptidasa A.
75. Una de los siguientes sustancias, NO sirve como un buen agente emulsificante:
a. Colesterol
76. La sustancia que estimula el crecimiento de la mucosa gástrica es:

a. Secretina

d. Gastrina

77.¿Cuál de las siguientes alternativas es una función de la colecistokinina?

a. Relajación de la vesícula para la salida de bilis

d. Secreción de enzimas pancreáticas

78.Con respecto a la anatomía del tronco celiaco, señale lo correcto a. El tronco celiaco se
origina de la cara posterior de la aorta abdominal
d. La hepática común que es una de sus ramas, participa en la irrigación del
estómago.
79. Con respecto a la anatomía del duodeno, marque la respuesta correcta: a. Tiene una
distribución en forma de “C”, que rodea la cola del páncreas
b. La 3ra porción duodenal está contenida en la pinza vascular aortomesentérica
80. En el íleon se absorbe aproximadamente el 95% dea través de la circulación
enterohepática.
a. agua
c. sales biliares
81. La estimula el mecanismo paracrino de la secreción de ácido clorhídrico.
a. histamina
82.En la digestión de proteinas,es el principal estímulo para convertir el
pepsinógeno en pepsina. a. la gastrina
b. el pH ácido
83. Con respecto a la somatostatina, marque lo correcto:
a. Es secretada por las células S del intestino
Interviene en la fase intestinal de la secreción gástrica
84. En pecten anal, es una estructura comprendida entre: a. la línea pectínea y los senos
anales
d. la línea anocutánea y la línea pectínea

85.¿Cuál de las siguientes alternativas es una proenzima pancreática? a. Tripsina

1. b. Elastasa
2. c. Quimotripsinógeno
3. d. Amilasa
4. e. Procarboxipeptidasa C
86. En este paciente, el bloqueo farmacológico de los receptores H2 en la mucosa gástrica:
a) No tiene efecto sobre la secreción de ácido inducida por el vago
b) Evita la activación de adenilciclasa por gastrina
c) Inhibe la secreción de ácido inducida por gastrina y mediada por el vago
d) Causa un aumento en el transporte de potasio por las células parietales gástricas
Se validó la A :)
87. Si se considerara una gastrectomía total para curar la gastritis del paciente, cuál de las
siguientes sustancias ya no se produciría:
a) Gastrina
b) Quimiotripsina
c) Amilasa
d) Pepsinógeno

88. Un paciente hipertenso está tomando un medicamento bloqueador de receptores alfa 1


adrenérgicos (prazosina) y como efecto secundario se queja de:
d) Lo escaso que es el medicamento
e) No tiene problemas en la salivación
c) Hiposalivación
d) Hipersalivación

89. Con respecto a las lesiones y enfermedades de la boca, marque lo correcto:


f) La leucoplasia se desprende al roce
b) la eritroplasia puede degenerar en adenocarcinoma
c) El muguet oral es una enfermedad bacteriana en inmunodeprimidos
d) la eritroplasia debe ser biopsiada
NOTA: fue validada la opción B ya que no es motivo del curso que sepamos el puto cáncer.

90. En este paciente, se puede asumir que la pancreatitis ha sido ocasionada por una
disminución en el efecto de:
g) Amilasa
h) Lipasa
c) Inhibidor de la tripsina
d) Entercinasa
91.Un efecto secundario en el estómago por la acción de la secretina es:
i) Disminución en la liberación de pepsinógeno
b) Menor actividad de la pepsina
c) Mayor paso de bicarbonato a sangre periférica
d) Aumento en la producción de factor intrínseco
92. Dentro de los factores protectores de la mucosa gástrica se pueden mencionar múltiples
protagonistas. Uno de ellos es:
j) CCK
k) Gastrina
c) Receptor muscarínico
d) Pepsina
93. La saliva puede tener una variedad de electrolitos en su composición. Entre ellos el cloro,
respecto al cual se puede afirmar:
l) Su mayor concentración se consigue con el flujo bajo
b) Su concentración no llega a ser tan alta como en el plasma
c) Con flujo alto su concentración es mayor que la del plasma
d) Su menor concentración se alcanza con flujo alto
94. En el síndrome de boca seca o síndrome de Sjogren, una de las complicaciones
asociadas es:
a) caries
b) Disminución de la acidez gástrica
c) Aumento en de la producción de saliva
d) Aumento del pH bucal
95. Estimula la producción de saliva:
a) Vasodilatación periglandular
b) Atropina
c) Fatiga o cansancio
d) Expresión de miedo
96. El omeprazol actúa sobre la membrana de la célula
m) Basolateral/principal
n) apical/principal
o) Basolateral/parietal
d) apical/parietal
97. Para protegerse del entorno ácido, el Helicobacter pylori se autogenera un entorno de
pH menos ácido alrededor suyo, gracias a una enzima que alcaliniza su entorno local
mediante la conversión de:
a) urea en NH3
b) H2O y CO2 en ácido carbónico
c) NH3 en urea
d) H2CO3 en bicarbonato
98. La anemia perniciosa destruye las células:
p) mucosas del cuello
b) oxínticas
c) principales
d) mucosas superficiales
99. La célula mucosa del cuello gástrico produce:
a) Moco
b) ácido clorhídrico
c) pepsinógeno
d) Factor intrínseco
100.El aumento en la acidez del estómago producido principalmente por la infección de
Helicobacter pylori se debe a la disminución de:
a) Somatostatina
b) Bicarbonato por las glándulas de Brunner
c) Secretina
d) Colecistoquinina
101. De las siguientes sustancias secretadas por los órganos de este paciente, la más
alcalina es la secreción:
a) pancreática
b) Esofágica
c) Yeyunal
d) Salival
102.En cuanto a la gastritis de este paciente, se encontró que era producida por la bacteria
Helicobacter pylori. Esta bacteria sobrevive en el medio ácido del estómago gracias a:
a) ácido clorhídrico
b) Toxina CagA
c) Ureasa
d) Jugo pancreático
103.La lengua está recubierta por epitelio:
c) pseudoestratificado columnar no queratinizado
b) plano estratificado no queratinizado
c) pseudoestratificado columnar ciliado
d) plano estratificado queratinizado
104.El esfínter anal interno tiene musculatura …….. y tiene control ……..
d) lisa / voluntario
b) lisa / involuntario
c) esquelética / simpático
d) esquelética / parasimpático

19) La arteria Aorta proporciona la irrigación al tubo digestivo ¿cuál de las siguientes
arterias proporciona la irrigación al ángulo cólico derecho?
a) mesentérica superior
b) mesentérica inferior
c) frénica inferior
d) tronco celiaco

20) Paciente de 26 años que le cuenta en su historia clínica que cada vez que almuerza a los
20 min tiene deseo de defecar, le comenta que su hijo de 1 mes le pasa lo mismo pero más
intenso. Esto se explica por el reflejo …….., el cual está …… en el paciente
a) colicoileal / normal
b) colicoileal / alterado
c) gastrocólico / normal
d) gastrocólico / alterado

21) La región del estómago que se comunica con el duodeno es la


a) pilórica
b) cardias
c) cuerpo
d) fórnix

22) Acude a consulta un px que fue diagnosticado de úlcera péptica 3 días antes. Luego de
múltiples pruebas diagnósticas se concluye que el paciente presenta un tumor secretor de
gastrina, ¿cual de las siguientes situaciones estará incrementada?
a) distensión gástrica
b) inhibición del vaciado gástrico
c) secrecion de acido clorhidrico
d) inhibición de la secreción de pepsinógeno
23) En el sistema digestivo, el control del apetito está dado por un complejo sistema de
sustancias y órganos integradores, los cuales regulan la ingesta de alimentos. La …… es
una sustancia orexígena y es sintetizada por el ……
a) leptina / estómago
b) felina / intestino
c) leptina / estómago
d) grelina / estómago

24) Con respecto a la actividad eléctrica del sistema digestivo, marque la alternativa
correcta
a) corresponden a potenciales de acción que están presentes de forma continua y le
dan capacidad de peristalsis autónoma al sistema digestivo
b) la frecuencia de las ondas lentas NO se ve influenciada por la actividad neural ni las
hormonas gastrointestinales
c) en el estómago las ondas lentas se dan en una frecuencia de 6 x min
d) las ondas lentas son cambios lentos y ondulantes del potencial en reposo
e) la frecuencia de las ondas lentas va de 6 a 12 ondas por minuto

25) Ante una lesión del IX pc, el músculo …… se altera en su función


a) palatogloso
b) estilofaríngeo
c) palatofaríngeo
d) constrictor superior

26) Un varón de 50 años es sometido a extirpación del duodeno y parte proximal del yeyuno.
La pérdida de estímulo hormonal en el páncreas para la secreción enzimática se explica
por la pérdida de células
a) parietales, productoras de factor intrínseco
b) K productoras de factor intrínseco
c) M productora de CCK
d) I productora de CCK

27) Marque la respuesta correcta:


A. El bronquio derecho constituye una de las estrecheces del esófago
B. Todos los órganos del sistema digestivo tienen capa serosa
C. La pared gástrica en el fondo es más delgada que en el cuerpo y antro
D. El esfínter de Oddi rodea a la papila menor duodenal

28) Marque la respuesta correcta en relación a la gastrina:


A. Al distenderse el estómago, se inhibe su producción.
B. Se estimula por la liberación de noradrenalina
C. Las células G son las productoras y se encuentran principalmente en el antro
gástrico
D. Las células G se encuentran principalmente en el fondo gástrico

29) Para poder morder una manzana, es necesario usar el siguiente músculo:
A. Milohiodeo
B. Tensor del paladar
C. Orbicular de los labios
D. Buccinador
30) Sustancia que inhibe la secreción y la motilidad del estómago prolongando el tiempo de
digestión:
A. Enteroglucagon.
B. Polipéptido pancreático
C. Péptido 1 similar al glucagón (GLP-1).
D. Péptido insulinotrópico dependiente de la glucosa (GIP).

31) El nacimiento de la arteria mesentérica superior se puede encontrar en cuál de los


cuadrantes abdominales:
A. Hipocondrio derecho
B. Hipogastrio
C. Epigastrio
D. Mesogastrio
32) Entre las múltiples causas de la Enfermedad por Reflujo Gastroesofágico, se puede
considerar también a una alteración en las del esfínter esofágico inferior:
A. Ondas secundarias
B. Contracciones tónicas
C. Ondas lentas
D. Glándulas subesofágicas

33) Producto de la alimentación, se producen diversas sustancias peptídicas, cininas y


bradicininas, las cuales permiten que:
A. Se produzca neovascularización en los territorios de las arterias abdominales
B. La acción de la lipasa pancreática se vea incrementada
C. El consumo de O2 del intestino aumente ligeramente
D. El flujo sanguíneo intestinal aumente hasta 8 veces

34) El dolor periumbilical o epigástrico en el inicio de una apendicitis aguda se debe a:


A. Estímulo del nervio vago.
B. Íleo secundario.
C. Irritación del peritoneo parietal.
D. Estímulo del sistema simpático.

35) El aumento en la actividad motora de la pared gástrica genera un aumento en los


niveles locales de qué sustancia en la microvasculatura:
A.Adenosina
B. Colecistoquinina CCK
C. Endotelina
D. Gastrina

36) ¿Cuál de los siguientes péptidos inhibe el vaciamiento gástrico?


A. Colecistoquinina
B. Péptido inhibidor gástrico
C. Motilina
D. Gastrina
37) Los músculos de la masticación que producen la retropulsión de la mandíbula son:
A. temporales [mas seguro]
B. maseteros
C. milohioideos
D. pterigoideos
38) En relación a la fisiología gástrica, marque lo correcto:
A. la cimetidina actúa en la región basolateral de la célula parietal
B. la marea alcalina se debe al paso de bicarbonato través de la membrana apical de la
célula principal
C. el cloro difunde hacia el exterior por la la región basolateral de la célula parietal
D. la salida de hidrogeniones a la luz es por difusión facilitada

39) Durante el sueño, la concentración de bicarbonato en la saliva:


A. Se eleva a niveles mayores que los del plasma
B. Aumenta
C. No tiene efecto
D. Disminuye

40) La secreción de saliva es importante en la fisiología digestiva. Su concentración de


potasio llega a ser menor que la del plasma cuando su secreción tiene un flujo:
A. Intermedio
B. Nunca
C. Bajo
D. Alto

42) Respecto a las glándulas salivales, marque lo incorrecto:


A. la glándula parótida produce secreción serosa
B. la glándula sublingual drena a través de conducto de Wharton
C. La glándula parótida drena a través del conducto de Stenon
D. la glándula sublingual tiene forma de garfio

43) Con respecto a la saliva, marque la respuesta correcta:


A. será hipertónica cuando el flujo es bajo
B. a mayor flujo, menor concentración de Na
C. a mayor flujo, menor concentración de cloro
D. el sistema simpático estimula su secreción
CI 3
44) En relación a la circulación hepática, marque lo correcto:
a) Los sinusoides hepáticos transportan sangre mixta
b) La vena porta proporciona el 50% de sangre al hígado
c) La vena porta se forma a partir de la vena esplénica y la mesentérica inferior
d) La arteria hepática deriva de la mesentérica superior

45) Dentro de las funciones de las células de Ito, marque lo incorrecto:


a) Sintetizan colágeno
b) Almacenan vitamina A
c) Se les llama células estrelladas
d) Pueden fagocitar patógenos y actúan como presentadoras de antígeno
46) Paciente con tumor neuroendocrino productor de secretina, debido a lo cual se puede
esperar que su secreción pancreática, comparada con la de una persona sana de bajo flujo,
tenga una concentración de:
a) Sodio aumentada
b) Igual
c) Bicarbonato aumentada
d) Potasio disminuida
47) El GALT se localiza en:
a) Lámina propia
b) submucosa
c) borde en cepillo
d) superficie de criptas de Lieberkühn

48) En relación a la histología hepática, marque lo correcto:


a) la zona 1 se afecta rápidamente en estados de hipovolemia y shock
b) La zona 1 se encuentra cercana a la vena central lobulillar
c) La zona 3 se encuentra más cerca a la vena central lobulillar
d) La zona 3 se encuentra más cerca al eje menor del acino hepático
49) El acino pancreático difere con el de las glándulas salivales en:
a) Contiene células centroacinares
b) No produce secreción serosa
c) El páncreas produce principalmente secreción mucosa
d) No tienen diferencias
50) Durante la digestión de las grasa, para que la lipasa actúe adecuadamente se requiere que el
pH aumento en la luz intestinal, lo cual es logrado, entre otros, por la secreción de las
células:
a) Del conducto interlobulillar
b) Centroacinares
c) Acinares
d) Alfa
51) La secreción de la colecistoquinina (CCK) se produce en la fase:
a) intestinal
b) En las 3 por igual
c) gástrica
d) Cefálica
52) ¿Por cuál de las siguientes células es secretada principalmente la pro enzima
procarboxipeptidasa?
a) Acinares del páncreas
b) Epiteliales del duodeno
c) Ductales del páncreas
d) Centro Acinares del páncreas
53) Una mujer de 43 años dolor en hipocondrio derecho e icterica. En la ecografía se evidencia
cálculos biliares. Estos cálculos lo más probable es que se encuentren localizados en:
a) conducto colédoco
b) Conducto cístico
c) Vesícula biliar
d) Conducto pancreático secundario
ECU 1:
Estudiante de 21 años sufre de gastritis aguda ocasionada por comer en lugares poco
higenicos. Suele consumir caramelos (“chupar”) mientras esta en base hasta la tarde.
También toma regular cantidad de leche (grasa, lactora, proteinas), pues le calma el dolor y
el ardor que sitnete por la gastritis (tiene dispepsia y cuando toma la leche se le pasa).
Incluso cuando puede, se toma dos vasos de agua frita y le calma la molestia. Ha decido ir al
medico para tratarse, pues ya no soporta el dolor, el cual esta seguro que los síntomas se
producen por elevada producion de HCl en el estomago, y por ello le ha recetado ranitidina
1.1) El consumir caramelos eleva los niveles en sangre de una hormona cuya función es la
estimulación de las células.
- Beta del páncreas por GIP el cual es una incretina y por consiguiente estimula las
células pancreáticas

1.2) Consumir caramelos indirectamente actica la via:


-POMP/ CART saciedad

1.3) Consumo de leche produce indirectamente


- CCK inhibición del vaciamiento gástrico mayor tonicidad del esfínter pilórico

1.4) Cuando el px toma dos vasos de agua, genera indirectamente un aumento en la


liberación de:
- vaso de agua distención → g astrina → secreción de HCl

1.5) El uso de ranitidina bloquea el receptor H2 de la histamina en las células parietales, la


histamina llega a estas células por:
- histamina es una hormona paracrina por → difusión
**endocrina es por via hematógena y si fuera neuroendorina es por un NTs

1.6) Aumenta la secreción salival:


- noradrenalina a través de los receptores Beta 2
1.7) En este paciente con gastritis aguda debida a una alta producción de ácido clorhídrico, sería
lógico esperar que el píloro tenga un tono muscular:
- primero la secretina
- luego CCK
**ambas reguladores del HCl, Gatritis aguda debido a una alta producción de HCL piloro
estará aumentado (por la CCK)
1.8) Debido al uso de ranitidina, los valores de somatoestina en sangre:
- ranitidina disminuye acción de gastrina se quiere secretar mas no actúan los
inhibidores como la somatoestina somatoestina disminuye
1.9) El uso de atropina en este paciente:
- Inhibirá la acción de las prostaglandinas
- Aumentará la producción de ácido clorhídrico
- Disminuirá la acción del receptor CCK-B
-Aumentará el pH del estómago
ECU 2:
Niño de sexo masculino de 2 años de edad, sufre de estreñimiento desde el nacimiento (1
deposición cada 3-4 días). Madre menciona que le estimula la defecación con un
termómetro rectal, y continuo uso de enemas y laxantes. Desde hace 6 meses comienza
con vómitos postprandiales. Los síntomas aumentan en frecuencia y magnitud y están en
relación con los episodios de estreñimiento. No refiere fiebre, tos, diarrea ni lesiones
cutáneas. Al examen físico presenta regular estado general, luce deshidratado. Abdomen
distendido, blando, depresible e indoloro. No se palpan masas abdominales. Se
permeabiliza el canal anal con termómetro rectal, encontrando cierta resistencia. Salida de
material fecal mal oliente en regular cantidad. Exámenes de laboratorio: hemograma
normal. Signos inflamatorios de fase aguda negativos. Alcalosis metabólica leve en sangre
venosa. Radiografía con enema baritado muestra recto y colon sigmoides dilatados
(megacolon). Biopsia profunda: ausencia de células ganglionares en la muestra enviada. Se
realiza cirugía correctiva.

2.1) Durante la fase esofágica de la deglución, para un bolo alimenticio determinado, a


medida que avanza el bolo la fuerza de la contracción se hace más:
- hiperpolarizado
- fuerte
- dependiente de Ach
- debil
2.2) Cuando este paciente ingiera sus alimentos, se espera que al momento de pasar el bolo
alimenticio por el esfínter esofágico superior, la presión intraesofágica disminuya en:
- la porción proximal al bolo
- el tercio medio del esófago
-el cardias
- el lugar donde se contraiga la muscular propia
2.3) Al examinar la orofaringe del paciente, uno puede hallar fácilmente la amígdala
palatina, pues esta se encuentra inmediatamente detrás del músculo:
-Palatogloso
- Palatofaringeo
- Hiogloso
- Elevador del velo del paladar

2.4) Con respecto a la defecación en este caso, marque la respuesta correcta:


- En posición de cuclillas, el músculo puborectal genera un ángulo más agudo en el
recto
- El sigmoides y el recto están inervados por el nervio vago
- La sensación de defecar sólo se da cuando el recto es ocupado por heces y
alcanzado el 80% de su capacidad
-El esfínter anal comprometido tiene inervación autónoma

2.5) En este paciente [hirschsprung] se considera que está abolido el reflejo:


- Coloileal
-Rectoesfinteriano
- Gastrocólico
- Relajación receptiva

2.6) No se espera que sea causa del vómito:


-Ayuno prolongado
- Estimulación faríngea y del glosofaríngeo
- Irritación de la mucosa gástrica
- Dolor intenso

ECU 3:
Paciente de 54 años con antecedentes de alcoholismo, gastritis crónica, tabaquismo
pesado, obesidad, cálculos biliares y cirrosis, es llevado a la emergencia por dolor
abdominal en epigastrio irradiado a la espalda y trastorno del sensorio.
Al examen físico: presión arterial 85/50 mmHg, frecuencia cardíaca 100 latidos/min,
frecuencia respiratoria 18 x minuto, temperatura axilar 36°C.
Conjuntivas pálidas, escleras ictéricas nevus arácnidos en tronco, distensión abdominal
marcada, cabeza de medusa, matidez desplazable en ambos flancos e hipogastrio, dolor a
la palpación de abdomen.
Tiempo de protrombina: 24 seg (testigo: 13 seg); TPT: 38 seg, glicemia: 165 mg/dL, uremia:
20 mg/dL, ASAT: 76 UI/L, ALAT: 22 UI/L, albumina: 2,5 g/dL, bilirrubina total: 2,6 mg/dL,
bilirrubina directa: 1,4 mg/dL, amilasa sérica 4000 U/L.
3.1) En esta paciente, al aumento de la amilasa sérica, se debe directamente a una lesión de:
a) páncreas
b) vesícula y árbol biliar
c) estómago
d) hígado
3.2) Considerando que el paciente sufre de gastritis, se puede decir que la secreción de
ácido por la mucosa gástrica
a) involucra transporte activo de hidrogeniones
b) es realizada principalmente por células principales
c) es inhibida por antihistaminas tomadas por pacientes con rinitis alérgica
d) involucra la liberación de HCl de los gránulos zimógenos
3.3) El paciente tiene hemorragia digestiva alta por várices sangrantes como complicación.
Llegando a estar en shock hipovolémico por hemorragia masiva, se encontrara necrosis
hepática en:
a) zona 1
b) no se afectan los lobulillos hepáticos en hemorragia
c) zona 3
d) zona 2
3.4) El misoprostol, análogo de las prostaglandinas está mejor indicado en:
a) cicatrización de úlcera péptica duodenal
b) erradica el helicobacter pylori
c) tratar el sind de Zollinger ellison
d) prevenir daño por AINES
3.5) De las siguientes sustancias secretadas por los órganos de este paciente, la más
alcalina es la secreción:
- Esofágica
- Salival
- Yeyunal
-Pancreática
3.6) En este paciente, se puede asumir que la pancreatitis ha sido ocasionada por una
disminución en el efecto de:
- Lipasa
- Enterocinasa
- Amilasa
-Inhibidor de la tripsina
3.7) ¿Cuál de las siguientes sustancias es segregada por el páncreas?
-Amilasa
- Pepsina
- Quimiotripsina
- Tripsina
3.8) Cada vez que este paciente toma alcohol, la acidificación de la luz del duodeno:
-Disminuye el vaciamiento gástrico
- Aumenta la contracción del esfínter de Oddi
- Aumenta la secreción del ácido gástrico
- Disminuye la secreción pancreática del bicarbonato
SISTEMA
DIGESTIVO (ME
154) EXAMEN
FINAL
Ciclo 2018-01

1. Un niño de 2 años es llevado a la consulta por diarrea persistente y edema de las extremidades, además falta
de crecimiento y desarrollo en relación a su edad. Los análisis de sangre revelan que tiene concentración
plasmática baja de proteínas (hipoproteinemia). Durante la endoscopía duodenal, se coloca colecistokinina
(CCK) endovenosa y se recoge muestras del líquido duodenal; el resultado del líquido confirma incapacidad
para hidrolizar proteínas a un pH neutro, esta situación mejora al añadir una pequeña cantidad de tripsina.
El paciente probablemente esté sufriendo la falta congénita de
………….
(Unidad 4, sesión 26, logro 2: Explicar la Digestión y absorción de las proteínas y sus alteraciones)
a. Pepsinógeno
b. PEPT-1
c. Carboxipeptidasas
d. Enterocinasa

2. Experimentalmente se incrementa la velocidad de la secreción salival con una sustancia, el análisis de


la composición de esta saliva obtenida se espera encontrar…………..
(Unidad 3, sesión 17, logro 5 : Explica la Influencia de la velocidad del flujo salival en la composición de la
saliva)
a. Elevación de concentración de bicarbonato, sodio y potasio
b. Elevación de concentración de cloro, sodio y potasio
c. Disminución de concentración de potasio
d. Disminución de concentración de potasio y bicarbonato

3. Paciente varón de 46 años soltero, consulta por odinofagia y bajo de peso, tiene antecedente de
tuberculosis desde hace 3 meses y es fumador crónico (10 cigarrillos por día); al evaluar la cavidad oral se
identifica lesión blanquecina en el dorso de la lengua y paladar blando, las lesiones se desprenden con el
baja lengua dejando una base eritematosa. Esta lesión corresponde probablemente a ……………………….…..
( Unidad 3, sesión18, logro 1-2 : Describe las enfermedades inflamatorias, infecciosas y proliferativas de la
cavidad oral)
a. Eritroplaquia
b. Candidiasis oral
c. Leucoplaquia vellosa
d. Fibroma en cavidad oral

4. Minero de 32 años de edad, que acude a centro de


salud por presentar de forma progresiva desde hace
1 año dificultad para ingerir alimentos sólidos y
luego líquidos; refiere regurgitaciones alimentarias y
marcada pérdida de peso (15 kilos). Radiografia
baritada de esófago como se muestra en la figura. El
presente caso se explica por……………….
(Unidad 2, sesión 12, logro 4: Identificar y describir
la función de los esfínteres esofágicos)
a. Contracción incompleta del esfínter esofágico inferior
b. Dificultad para el inicio de la deglución
c. Relajación incompleta del esfínter pilórico
d. Relajación incompleta del esfínter esofágico inferior
5. Paciente mujer de 35 años acude a consulta por sensación de sequedad y lesiones en cavidad oral. Al
examen se observa atrofia de la mucosa, fisuras y úlceras; nota además sequedad e irritación de la
córnea y aumento del tamaño de las glándulas parotídeas. Su diagnóstico más probable es artritis
reumatoide; el hallazgo más probable en una biopsia de glándula parótida es……..….
(Unidad 3, sesión 18, logro 3: Describe las enfermedades más frecuentes de las glándulas salivales)
a. Hiperplasia de acinos glandulares serosos
b. Gran infiltración de linfocitos y células plasmáticas
c. Gran infiltrado de linfocitos y macrófagos
d. Presencia de acinos normales con hiperplasia de células ductales

6. Un paciente con anemia acude con su médico quejándose de episodios frecuentes de gastroenteritis. Un
análisis de sangre revela anticuerpos circulantes dirigidos contra células parietales gástricas. Su anemia es
atribuible a la hiposecreción de
………………………
(Unidad 3, sesión 20, logro 5: Gastritis crónica. Tipos de gastritis)
a. Factor intrínseco
b. Proteina R (haptocorrina)
c. Pepsinógeno
d. Ácido clorhídrico

7. Dos estudiantes deciden tomar un receso para comer una hamburguesa a la hora del almuerzo. Antes de
llegar a la cafetería, impulsos nerviosos provenientes del complejo vagal dorsal iniciarán la secreción de
ácido gástrico por la liberación dedesde el sistema nervioso entérico.
(Unidad 3, sesión 20, logro 2: Regulación de la secreción gástrica: estimulación, fases de la secreción)
a. Serotonina
b. Óxido nítrico
c. GRP (péptido liberador de gastrina)
d. Péptido intestinal vaso activo

8. Un niño de cuatro años de edad es llevado a la consulta por cuadros diarreicos frecuentes caracterizados
por heces pálidas, voluminosas y fétidas, presenta bajo peso y talla. Se mide la concentración de cloruro en
el sudor y se encuentra que sus valores son muy elevados. La alteración más importante a nivel de células
ductales del páncreas tiene relación directa con la conductancia de…………
(Unidad 3, sesión 23, logro 5 Explica la Secreción pancreática: formación del jugo pancreático, influencia de la
velocidad de flujo y regulación)
a. Potasio
b. Bicarbonato
c. Sodio
d. Cloro

9. Una mujer de 50 años de edad que sufrió durante varios años resequedad de los ojos debida a producción
inadecuada de lágrimas es enviada con un gastroenterólogo para evaluación de pirosis crónica. El examen
endoscópico revela erosiones y tejido cicatrizal en la parte distal del esófago justo por arriba del esfínter
esofágico inferior. Las lesiones pueden atribuirse a la disminución de uno de los siguientes componentes
salivales:
(Unidad 3, sesión 17, logro 4: Explicar la Formación de la saliva y cuáles son sus componentes)
a. Bicarbonato
b. Lactoferrina
c. Ig A
d. Amilasa
10. Se evalúa los valores séricos de las siguientes sustancias a un paciente con enfermedad hepática terminal;
en este paciente se espera encontrar la combinación con la letra …………
(Unidad 3, sesión 22, logro 5: Describe las Pruebas de función hepática, la Insuficiencia hepática,
encefalopatía hepática e hipertensión portal)

Glucosa Amoniaco Albúmina


a. Aumenta Disminuida Disminuida
da
b. Disminui Aumentada Aumentada
da
c. Aumenta Aumentada Aumentada
da
d. Disminui Aumentada Disminuida
da

11. Una mujer de 35 años de edad HIV positiva, se presenta al médico con dolor abdominal en cuadrante
superior derecho e ictericia. La paciente refiere haber tenido múltiples episodios de ictericia durante los
últimos 10 años. Los exámenes para determinar hepatitis viral, dieron positivos para Hepatitis B, siendo
catalogado el caso como hepatitis crónica con alteración funcional. En un examen de sangre ¿cuál de los
siguientes parámetros está disminuido?
(unidad 3, sesión 22, logro 5: Pruebas de función hepática, Insuficiencia hepática, encefalopatía hepática
e hipertensión portal)
a. Fosfatasa alcalina
b. Albumina
c. Bilirrubina
d. Tiempo de protrombina

12. En el reflejo peristáltico del intestino delgado, uno de los siguientes eventos sucede en la
porción oral del bolo alimenticio…………...
(Unidad 2, sesión 13, logro 4: Explicar la Motilidad del intestino delgado: Contracciones segmentarias y
peristálticas)
a. Disminución de 5 hidroxitriptamina desde las neuronas IPAN
b. Contracción del músculo longitudinal
c. Acción del péptido intestinal vasoactivo (VIP) en el músculo circular
d. Acción de acetilcolina en el músculo circular

13. Experimentalmente se coloca una dosis alta de secretina en la luz intestinal duodenal; como
consecuencia de esto, en el jugo pancreático de la misma luz intestinal se observa la disminución de la
concentración de …..………..
(Unidad 3, sesión 23, logro 5: Explica la Secreción pancreática: formación del jugo pancreático, influencia
de la velocidad de flujo y regulación)
a. Na+
b. Cl-
c. K+
d. HCO3-

14. Un varón de 58 años de edad con enfermedad de Crohn severo fue sometido a una resección ileal.
Después de la cirugía este paciente padecerá de esteatorrea, esto se explica porque …..………..
(unidad 4, sesión 26, logro 4: Explica las alteraciones en la Absorción de lípidos)
a. El pool de ácidos biliares se incrementa
b. Los quilomicrones no pueden formarse en el lumen intestinal
c. La micelas no pueden formarse
d. El páncreas no secreta lipasa

15. En un experimento se inserta un balón en el estómago de un voluntario, se infla poco a poco mientras que
se vigilan las presiones intraluminales. Aunque el volumen del balón aumenta considerablemente, las
presiones permanecen constantes. Esta relación volumen-presión se explica por la liberación local de
…………..
(Unidad 2, sesión 13, logro 1 Explica la Motilidad gástrica: relajación receptiva)
a. Acetil colina y gastrina
b. Colecistoquinina y óxido nítrico
c. Óxido nítrico y péptido inhibidor vasoactivo
d. Norepinefrina y óxido nítrico
16. La toxina del Vibrio cholerae causa diarrea debido a…….
(Unidad 4, sesión 27, logro 6: Explica el transporte hidroelectrolítico intestinal, toxina colérica)
a. La fosforilación del canal CFTR de los enterocitos de las vellosidades intestinales
b. El Incremento de la secreción de cloro por las células de la cripta intestinal
c. La inhibición de la producción de AMPc por las células epitelailes
d. El incremento de la absorción de agua y sodio a través de las uniones estrechas

17. ¿Cuál de las siguientes alternativas es una característica de la secreción exocrina del páncreas?
(Unidad 3, sesión 23, logro 5: Secreción pancreática: formación del jugo pancreático, influencia de la
velocidad de flujo y regulación)
a. Tiene una baja concentración de Cl- respecto al plasma
b. Es estimulada por la presencia de bicarbonato en el duodeno
c. La secreción enzimática es estimulada principalmente por la gastrina
d. Es hipotónica respecto al plasma

18. Una madre lleva a su hijo de dos años de edad a la sala de urgencias, estresada porque el niño deglutió una
moneda de 10 céntimos mientras la familia cenaba en un restaurante. El médico observa mediante
fluoroscopía que la moneda se halla en el estómago y asegura a la madre que la moneda se eliminará con
las heces. El médico recomienda utilizar la respuesta fisiológica que permitirá la evacuación de la moneda
del estómago al intestino ………….…..
(Unidad 2, sesión 13, logro 2: Explica la Motilidad gástrica: mezclado y vaciamiento)
a. Es por la relajación receptiva
b. Son los movimientos de mezcla y trituración
c. Es provocada por el ayuno
d. Es por la relajación del esfínter esofágico superior

19. Las estructuras en el hígado que permite que los productos metabólicos unidos a proteínas tengan acceso
a las membranas basolaterales de los hepatocitos, son…..
(Unidad 3, sesión 21, logro 4-5: Explica la Organización micro estructural del hígado)
a. Los Canalículos
b. Las fenestras sinusoidales
c. Las uniones intercelulares herméticas
d. Las células de Ito

20. La composición de la bilis es modificada conforme fluye por los conductillos biliares. Durante este
tránsito se espera que aumente la concentración de…….
(Unidad 3, sesión 22, logro 2: Describe la Secreción biliar, visión general del sistema biliar extrahepático y
composición de la bilis)
a. Ig A
b. Glucosa
c. Monómeros de ácido biliar
d. Vitamina A

21. Se mide experimentalmente el contenido gástrico de dos personas. La persona “A” tiene alto
contenido de grasa y la persona “B” tiene un contenido hipertónico ¿Cuál de las siguientes es
correcto respecto al vaciamiento gástrico? (Unidad 2, sesión 13, logro 2: Describe la Motilidad y
vaciamiento gástrico)
a. Hay ralentización del vaciado gástrico solo en “A”
b. El vaciamiento gástrico es más rápido en ambos
c. En ambos casos hay incremento de la motilina
d. Hay ralentización del vaciado gástrico en ambos casos

22. El examen endoscópico de un paciente con hipertensión portal grave revela venas tortuosas que sobresalen
hacia la luz del esófago. El paciente recibe tratamiento quirúrgico mediante la colocación de una derivación
que conecta la vena porta a la vena cava. Después de la operación el riesgo de encefalopatía …………………..
y el riesgo de sangrado de várices ……………..
(Unidad 3, sesión 22, logro 5: Describe la Insuficiencia hepática, encefalopatía hepática e hipertensión portal)
a. Aumentará/disminuirá
b. Disminuirá/disminuirá
c. Aumentará/aumentará
d. Disminuirá/aumentará
23. Un paciente varón de 18 años de edad acude al médico para sus exámenes de rutina. Sus resultados de
laboratorio muestran un valor de bilirrubina sérica de 4 mg/dl y una bilirrubina directa de 0,3 mg/dl. Las
pruebas de función hepática son normales. La alteración que explica mejor este caso es por la
deficiencia de ………………..
(Unidad 3, sesión 22, logro 3: Explica la Producción y excreción de bilirrubina. Tipos de bilirrubina e ictericia)
a. Transaminasas
b. Glucuronil transferasa
c. Hemo oxigenasa
d. La 7 alfa hidroxilasa

24. Un hombre de 57 años de edad es llevado a urgencias con hematemesis masiva rojo brillante, a su llegada
se halla inconciente con PA: 80/40 mm Hg y FC: 124 lat/min. Luce ictérico con presencia de “arañas
vasculares en el tórax anterior y extremidades”, abdomen distendido con signo de oleada positiva. Se
encuentra esplenomegalia y pérdida de la masa muscular en extremidades. La anastomosis vascular
responsable del sangrado en este paciente es ………….…..
(Unidad 3, sesión 21, logro 2: Describe las anastomosis porto sistémicas)
a. Arteria gástrica izquierda y vena ácigos
b. Vena gástrica izquierda y vena ácigos
c. Vena paraumbilical y vena epigástrica inferior
d. Vena gástrica izquierda y vena esofágica superior

25. Un estudiante de medicina está comiendo un plato de comida a base de champiñones, espárrago y salsa
de soya. El sabor umami contenido en todos estos alimentos actúa a nivel de los botones gustativos
estimulando ………………..
(Unidad 2, sesión 10, logro 5: Describe los tipos y mecanismos moleculares para la detección de los sabores)
a. El ingreso de sodio
b. Un receptor acoplado a proteína G
c. Su receptor específico T1R3
d. El ingreso de hidrógeno

26. Un hombre de 22 años de edad se presenta al médico con una historia de 1 año de evolución
caracterizado por dolor recurrente en fosa iliaca derecha y diarrea. Manifiesta además pérdida de
peso de 8 kg durante este periodo. La colonoscopía revela múltiples lesiones en el ileon terminal y
colon. La biopsia de estas lesiones revela engrosamiento, inflamación y ulceración de la mucosa. El
diagnóstico más probable en este caso es…….
(Unidad 4, sesión 28, logro 5: Describe la Enfermedad inflamatoria intestinal. Generalidades, morfología y
características)
a. Sprue celiaco
b. Enfermedad de Crohn
c. Sindrome de colon irritable
d. Colitis ulcerativa

27. Una de las funciones del músculo señalado es:


(Unidad 2, sesión 8, logro 3: Describir el Piso de la
boca: estructuras blandas que la conforman)
a. Eleva el paladar blando
b. Recibe inervación del nervio maxilar
c. Deprime el hioides cuando la mandíbula está fija
d. Deprime la mandíbula cuando el hioides está fijo

28. Varón de 61 años que consulta por dolor retro esternal intenso desde hace 6 horas y después de
vómitos intensos y repetidos; al examen se observa disnea, cianosis, hipotensión y signos clínicos de
shock. La radiografía simple de tórax muestra neumomediastino. El líquido en el espacio pleural
aspirado tiene alta concentración de amilasa. ¿Cuál de las siguientes alternativas puede explicar este
cuadro clínico?
(Unidad 3, sesión 18, logro 6: Describe algunas Enfermedades del esófago)
a. Sindrome de Mallory Weiss
b. Rotura espontánea de esófago
c. Neumotórax por probable herida penetrante
d. Perforación de ulcera gástrica de cara posterior, con complicación torácica
29. La secreción del ácido en la célula parietal gástrica se lleva a cabo por una ATPasa especifica que
intercambia hidrogeniones (H+) del citosol por…..
(Unidad 3, sesión 20, logro 1: Explica la Secreción del HCl y sustancias que la alteran)
a. Cl-
b. HCO3-
c. Na +
d. K+

30. En condiciones normales el ingreso de 600 ml de líquido es el estómago provoca un aumento de presión
intragástrica de unos 12 cm de H2O. Después de una vagotomía (corte del nervio vago) es de esperar que
el ingreso del mismo volumen de líquido provoque lo siguiente: …………………………………
(Unidad 2, sesión 13, logro 1: Describe la Motilidad gástrica: relajación receptiva)
a. Un aumento igual de la presión
b. Que no aumente la presión
c. Un aumento mayor de la presión
d. Una disminución de la presión

31. Una paciente de 30 años de edad es sometida a una cirugía en oído medio derecho por un problema de
otoesclerosis. Luego de la cirugía refiere alteración en la percepción de sabores. Al evaluar el caso usted
esperaría encontrar……….
(Unidad 2, sesión 10, logro 5: Describe la Irrigación e inervación de la lengua)
a. Alteración en la sensación del dolor y temperatura en el tercio posterior de la lengua
b. Alteración en la sensación del gusto en los dos tercios anteriores de la lengua
c. Alteración en la sensación del gusto en la punta de la lengua
d. Sensación del dolor, tacto y temperatura conservada en toda la lengua

32. ¿Cuál de las siguientes alterativas es correcta?


(Unidad 4, sesión 26 : Explica la digestión y absorción de nutrientes y sus alteraciones)
a. En el borde luminal, en cepillo, del intestino delgado, la absorción de sodio únicamente se realiza
asociada a la de glucosa.
b. El lugar principal para la absorción del hierro es el ileon
c. Las sales biliares desconjugadas son absorbidas preferentemente en el colon
d. El proceso de digestión y absorción de la vitamina B12 no se altera en insuficiencia pancreática.

33. En un paciente de 45 años de edad con colestasis biliar, se encuentra una elevación de los niveles
sanguíneos de fosfatasa alcalina hasta 3 veces la cifra normal. ¿Cuál de las siguientes alternativas estará
también elevada como evidencia del daño de la vía biliar?
(Unidad 3, sesión 22, logro 5: Pruebas de función hepática, Insuficiencia hepática, encefalopatía hepática
e hipertensión portal)
a. Tiempo de protrombina y albúmina sérica
b. Transaminasas hepáticas (ALT y AST)
c. Glucoronil transferasa
d. Gamma glutamil transpeptidasa

34. Revisando la angiografía de un hombre de 70 años en estudio por aneurisma de aorta abdominal el
radiólogo informa de la presencia de una oclusión completa de la arteria mesentérica inferior. El paciente
se encuentra completamente asintomático. ¿Cuál de las siguientes arterias se anastomosa a la sistema
arterial de la mesentérica inferior?
(Unidad 4, sesión 25, logro 1: Identifica la Arteria mesentérica superior e inferior, ramas y anastomosis)
a. Ileal
b. Cólica media
c. Sigmoideas
d. Cólica izquierda

35. Lactante de 3 meses de vida es atendido por presentar diarrea, se administra una solución de glucosa y
electrólitos por vía oral. La proteína de membrana apical que explica la capacidad de esta solución para
proporcionar aporte de glucosa e hidratación es ………..
(Unidad 4, sesión 26, logro 1: Explica la Digestión y Absorción de los hidratos de carbono. Alteraciones)
a. GLUT-5
b. SGLT-1
c. CFTR
d. GLUT-2
36. Paciente ha sufrido herida de bala en el abdomen, se le ha tenido que extirpar el segmento medio y distal
del ileon. En este caso la síntesis hepática de sales biliares estará …..…..
(Unidad 3, sesión 22, logro 4: Explica la formación, función y Circulación entero hepática de lasa sales biliares)
a. Disminuida por inhibición de la colesterol 7 alfa hidroxilasa
b. Incrementada por estímulo de la enzima colesterol 7 alfa hidroxilasa
c. Incrementada por inhibición de la colesterol 7 alfa hidroxilasa
d. Sin cambios en el ritmo de síntesis

37. Un varón de 75 años ingresa al consultorio por presentar ictericia marcada de piel y las escleras. El estudio
del paciente mostró que presentaba un tumor que obstruía la totalidad del conducto hepático común.
¿Cuál de las siguientes estructuras se encontrará dilatada en este paciente?
(Unidad 3, sesión 21, logro 6: Describir el árbol biliar intrahepático)
a. Conducto de Wirsung
b. Conductos de Hering
c. Conducto colédoco
d. Conducto cístico

38. Correlaciones las dos columnas y marque la fórmula correcta:


(Unidad 4, sesión 28, logro 1: Diarrea: definición, mecanismos: osmótica, secretoria y exudativa)
1. Enfermedad Hirschsprung( ) heces con moco y sangre
2. Diarrea osmótica( ) intolerancia a lactosa
3. Diarrea secretoria( ) aganglionosis congénita
4. Diarrea exudativa( ) canales de Cl- en las células de la cripta

a.- 4231b.- 1234c.- 2143d.- 4213

39. Respecto a la siguiente imagen que representa una estructura de la mucosa gástrica, la estructura con
número ………..
produce ……………………..
(Unidad 3, sesión 19, logro 4: La glándula fúndica. Funciones y tipos de células con sus características)
a. 3 / pepsina
b. 1 / Pepsinógeno
c. 4 / HCl y factor extrínseco
d. 2 / pepsinógeno

2
40. En un paciente con insuficiencia renal crónica, el déficit en la absorción de calcio a nivel del
enterocito se debe a lo siguiente:
(Unidad 4, sesión 26, logro 6: Explica la Absorción de calcio y hierro)
a. No se convierte la 25 hidroxicolecalciferol a 1,25 dihidroxicolecalciferol
b. No se convierte la 1,25 dihidroxicolecalciferol a 25 hidroxicolecalciferol
c. Se incrementa la producción de Calbindina
d. Existe un descenso de la alfa 25 hidroxilasa renal
CLAVES EXAMEN
PARCIAL DE
SISTEMA
DIGESTIVO 2019 -
00

1. Varón de 30 años es traído a emergencia por agresión abdominal con arma de fuego (pistola)
y es sometido a laparotomía exploratoria, observándose isquemia del colon ascendente y
parte del colon trasverso ¿la lesión de cuál de las siguientes arterias explicaría esta isquemia?
(unidad 1, sesión 2, logro 6: (Describe la irrigación visceral: arterias de tronco celiaco,
arteria mesentérica superior e inferior, topografía de superficie, órganos por cuadrante)

a. Celiaca
b. Colónica derecha
c. Mesentérica inferior
d. Mesentérica superior

2. Respecto a las sustancias gastrointestinales que regulan la secreción pancreática;


marque la afirmación correcta:
(unidad 1, sesión 3, logros 2 y 3: describir las hormonas gastrointestinales: estímulos y funciones)

a. La Secretina, es la hormona más importante para la secreción de bicarbonato por las


células acinares del páncreas
b. La acetilcolina es capaz de estimular la secreción enzimática y de bicarbonato del páncreas
c. La gastrina, es la hormona más importante para la secreción de enzimas pancreáticas
d. La colecistoquinina (CCK) estimula al páncreas solo para secreción enzimática

3. Ante una lesión del X par craneal, ¿cuál de los siguientes músculos mantiene conservada su
función?:
(unidad 2, sesión 08, logro 4: Paladar blando: componentes musculares)

a. Elevador del velo del paladar


b. Tensor del velo del paladar
c. Palatofaríngeo
d. Glosofaríngeo

4. Experimentalmente se utiliza atropina (anticolinérgico) para inhibir la secreción de


gastrina, sin embargo la secreción de esta hormona se sigue dando ante estímulos
vagales. Esta situación se explica porque la atropina:
(unidad 1, sesión 3, logro 3 : describir las hormonas gastrointestinales: estímulo y funciones de la
gastrina y colecistoquinina)

a. Bloquea parcialmente la bomba de protones en la célula G


b. Inhibe la acción de acetilcolina e histamina en la célula G
c. Solo inhibe la acción del péptido GRP en la célula G
d. No bloquea la acción del péptido GRP

5. Un varón de 50 años es sometido a extirpación del duodeno y parte proximal del


yeyuno. Esta situación ocasionaría la pérdida de las células ……….. , productoras deque
estimula la secreción de
bicarbonato por el páncreas.
(unidad 1, sesión 3, logro 3: describir las hormonas gastrointestianles: estímulos y funciones
de la secretina y péptido insulinotrópico dependiente de glucosa)

a. “S” / secretina
b. Parietales / secretina
c. “I” / colecistoquinina
d. “S” / colecistoquinina
6. Recién nacido que presenta tumoración abdominal a nivel del cordón umbilical
(fotografía). ¿cuál de las siguientes afirmaciones es correcta respecto a este defecto en
el desarrollo embriológico del intestino?: (unidad 1, sesión 5, logro 2: identificar las
anomalías del desarrollo del intestino medio)

a. Corresponde a una Gastrosquisis


b. Las vísceras se hallan cubiertas por piel
c. No está asociado a otras malformaciones
d. Se asocia a
malformaciones cardiacas
y del tubo neural

7. Varón de 35 años acude a la emergencia por


trauma abdominal y se decida realizar una
laparoscopía exploratoria. El cirujano observa la
disposición de los órganos abdominales como se
representa en el siguiente esquema. Esta
disposición de órganos se explica por la
rotación(SMA=arteria mesentérica superior)
(unidad 1, sesión 5, logro 3: identificar las anomalías del
desarrollo del intestino medio: defectos de rotación,
estenosis y atresias)

a. anti horaria del intestino medio, en sólo 90°


b. incompleta del intestino medio (270°)
c. horaria del intestino medio
d. horaria del estómago

8. Se evalúa la expresión de la proteína Agrp en una persona con alteración del apetito; lo
correcto respecto a esta proteína es…..
(unidad 1, sesión 3, logro 4: Explica los mecanismos de control del apetito y saciedad )

a. Esta proteína es un potente anorexigénico


b. La mutación del gen que la codifica produce adelgazamiento
c. La sobre producción de la proteína lleva a obesidad por agonismo de receptores MC3 y MC4
d. La sobre producción de la proteína disminuye el apetito por antagonismo de receptores MC4

9. Juana cae de la bicicleta y se fractura la región anterior del hueso maxilar superior con
compromiso de la fosa incisiva. Al examen físico de la región esperaría encontrar alteración
en la sensibilidad de la encía …………………
(unidad 2, sesión 8, logro5: paladar: paladar duro y blando: irrigación e inervación)

a. bucal posterior
b. Lingual anterior
c. palatina anterior
d. palatina posterior
10. Recién nacido es atendido por el neonatólogo y luego entregado a su madre para dar de
lactar; la madre al dar de lactar observa coloración azulada de labios, acompañado de tos
persistente, dificultad respiratoria y distención abdominal. Se le intenta colocar una sonda
nasogástrica pero esta retorna a la cavidad oral en todos los intentos. ¿Cuál de las
siguientes anomalías del desarrollo es el más probable en este caso? (unidad 1, sesión 4,
logro 3: identificar las anomalías en el desarrollo del esófago: atresia y/o fístula traqueo
esofágica)

a. Estenosis esofágica proximal con Fístula traqueo esofágica distal


b. Atresia esofágica proximal con fístula traqueoesofágica distal
c. Atresia esofágica distal con fístula traqueoesofágica proximal
d. Fístula traqueoesofágica proximal y distal

11. ¿Cuál de los siguientes mecanismos ocurre durante la defecación?


(unidad 2, sesión 13, logro 6: motilidad del intestino grueso: contracciones segmentarias,
movimientos en masa, defecación y reflejo gastrocólico)

a. Contracción refleja del esfínter anal interno


b. En la posición de “cuclillas” el músculo puborectal se halla relajado
c. Relajación del esfínter anal externo por efectos del VIP y óxido nítrico
d. La materia fecal en el recto estimula la contracción del sigmoides por los nervios pudendos

12. La estructura número 4 (gráfico) corresponde a


……….… y está ………..
(unidad 2, sesión 9, logro 2: Partes
de un diente. Capas del diente:
Esmalte: características y células
que lo producen)

a. el cemento / mineralizado en 90%


b. la dentina / formada por ameloblastos
c. el esmalte / formado por
células derivadas del
mesénquima
d. la dentina / formado por
células derivadas de la cresta
neural

13. Un paciente luego de un accidente sufre lesión del piso de la boca, se constata daño del
nervio “cuerda del tímpano”, en este caso se esperaría encontrar disminución de lade la
lengua
(unidad 2, sesión 10, logro 3: Irrigación e inervación de la lengua)

a. Motilidad en los dos tercios anteriores


b. Sensación del gusto en el tercio posterior
c. Sensación del gusto en los dos tercios anteriores
d. Sensibilidad al tacto en los dos tercios anteriores

14. ¿Cuál de las siguientes afirmaciones es la correcta sobre la gastrina?


(unidad 1, sesión 3, logro 1: reconocer las características de las sustancias reguladoras
gastrointestinales: hormonas, sustancias paracrinas y neurocrinas)

a. Produce atrofia de la mucosa gástrica


b. Es producida por la célula G del cuerpo gástrico
c. Es estimulada por la distensión gástrica y el Ph bajo
d. Actúa en la célula diana mediante su receptor CCk tipo B
15. Al recibir un paciente con signos de hipovolemia y antecedente de trauma en abdomen por
accidente de tránsito, usted identifica radiológicamente: lesión de primera vértebra lumbar y
signos de lesión en páncreas; durante la cirugía se observó pobre irrigación de asas
intestinales. El vaso afectado es la arteria ……..
(unidad 1, sesión 1, logro 6: reconocer las estructuras a nivel de L1, nivel de los principales vasos
sanguíneos)

a. esplénica
b. hepática común
c. mesentérica inferior
d. mesentérica superior

16. Un paciente sufre de daño a nivel del cuello con lesión muscular en la región de la faringe. En
el examen físico se determina dificultad para la elevación de la faringe y para el cierre del
itsmo de las fauces. En este caso, probablemente esté afectado el músculo:
(unidad 2, sesión 11, logro 2: Músculos de la faringe: identificación, constrictores y longitudinales)

a. palatogloso
b. estilofarinfeo
c. palatofaringeo
d. constrictor inferior

17. Varón de 50 años a quien le realizan la curación de la segunda molar de la arcada


superior derecha. En un momento determinado, el paciente acusa de intenso dolor de la
pieza dentaria en tratamiento. La vía aferente del dolor viaja a través del nervio …………
(unidad 2, sesión 9, logro 6: Inervación de los dientes)

a. trigémino V2
b. trigémino V3
c. naso palatino
d. palatino menor

18. La distención gástrica por los alimentos produce incremento de secreción de HCl mediante la
producción de
………….. que estimula a las células ……………. vía proteína ………..
(Unidad 1, sesión 3, logro 2: Describe las hormonas gastrointestinales: Estímulo y funciones de la
gastrina y colecistoquinina)

a. gastrina / parietal / Gq
b. gastrina / principal / Gs
c. acetilcolina / parietal /Gi
d. acetilcolina / principal / Gi

19. Un niño de tres años llega a emergencia con disfagia (dificultad para tragar), dolor retro
esternal, salivación y llanto. Se sospecha de ingesta de cuerpo extraño (moneda) en el
esófago; al ser evaluado se constata en una radiografía presencia de cuerpo extraño a nivel
de C6 (6° vértebra cervical). El cuerpo extraño estará suspendido a nivel del estrechamiento
producido por………..
(unidad 2, sesión 11, logro4: Esófago, características anatómicas, relación con órganos vecinos y
estrecheces)

a. el cayado aórtico
b. el hiato esofágico
c. el músculo cricofaríngeo
d. el bronquio principal izquierdo

20. La triada portal (arteria hepática, vena portal y conducto biliar común) está contenida en el
ligamento
…….……… y derivan embriológicamente del ……
(Unidad 1, sesión 1, logro 4: Identifica el peritoneo, mesenterio, omento y ligamentos,
retroperitoneo.)

a. hepato duodenal / mesenterio ventral


b. gastro esplénico / mesenterio dorsal
c. hepato gástrico / omento menor
d. falciforme / omento menor
21. En relación al movimiento de
peristaltismo del tubo digestivo: en
la flecha negra del gráfico se
produce la liberación de ………………
a nivel del músculo ………..
(unidad 2, sesión 7, logro 6: Control
hormonal y tipos de movimiento)

a. noradrenalina, sustancia P y neuropéptido “ Y” / circular


b. acetilcolina y sustancia P / longitudinal
c. óxido nítrico y PIV / longitudinal
d. óxido nítrico y PIV / circular

22. Un paciente refiere no percibir algunos sabores, al examen físico se constata alteración en
la percepción de sabores y del dolor en el tercio posterior de la lengua ¿Cuál de los
siguientes nervios estará alterada en su función?
(unidad 2, sesión 10, logro 5: Sabores, tipos y mecanismos moleculares para su detección)

a. Lingual (rama del V par)


b. Cuerda del tímpano (VII par)
c. Glosofaríngeo (IX par)
d. Hipogloso (XII par)

23. El gráfico detalla la estructura de


la pared del tubo digestivo
intestinal ¿Cuál de las siguientes
asociaciones es correcta?
(unidad 2, sesión 7, logro 1: La
pared y músculo liso
gastrointestinal )

a. “1” – peristaltismo
b. “2” – secreción enzimática
c. “3” – deriva del mesodermo
d. “4” – doble hoja de tejido graso

24. En el caso de un paciente con gastrinoma (tumor productor de gastrina), la presencia de


úlceras duodenales y erosión de la mucosa gástrica, se debe principalmente a…….
(unidad 1, sesión 3, logro 2: describir las hormonas gastrointestinales: estímulo y funciones de la
gastrina y colecistoquinina)

a. la acción directa de la gastrina sobre la célula principal


b. la sobre expresión de los receptores “G” en la célula parietal
c. el exceso de HCl por estímulo de receptores CCK-B en la célula parietal
d. el exceso de HCl por estímulo directo de receptores de acetilcolina en la célula parietal

25. El reflejo entero gástrico se caracterizan por:


(unidad 2, sesión 13, logro 6: Motilidad del intestino grueso: contracciones segmentarias,
movimientos en masa defecación y reflejo gastrocólico)

a. favorecer la motilidad gástrica gracias a la CCk


b. inhibir la motilidad gástrica y estimular la secreción ácida
c. movilizar grandes volúmenes desde el estómago al duodeno
d. originarse debido a la distensión duodenal y presencia del quimo ácido
26. Mauricio tiene dificultad para deprimir el paladar y elevar la parte posterior de la lengua. En
este caso estará afectado un músculo ………………., específicamente el músculo …………….
(Unidad 2, sesión 10, logro 2: Músculos de la lengua: clasificación, identificación y sus funciones)

a. intrínseco – longitudinal inferior


b. extrínseco – palatogloso
c. extrínseco – transverso
d. extrínseco – estilogloso

27. Una de las funciones del músculo señalado es:


(Unidad 2, sesión 8, logro 3: Describir el Piso de la boca:
estructuras blandas que la conforman)

a. deprimir la lengua
b. elevar el paladar blando
c. deprimir el hioides cuando la mandíbula está fija
d. deprimir la mandíbula cuando el hioides está fijo

28. Paciente varón de 30 años es evaluado por probable enfermedad de Chagas, cursa con
problemas de motilidad del colon; los estudios de biopsia determinan ausencia de
células ganglionares. Según el gráfico
¿cuál es la capa en la que se determina la ausencia de dichas células?
(unidad 1, sesión 2, logro 1: describir las generalidades de la estructura del tubo digestivo: esófago,
estómago intestino delgado y grueso)

a. Mucosa - 1
b. Muscular propia – 1
c. Muscular de la mucosa - 2
d. Muscular propia - 3

1 2

3
29. Paciente varón de 32 años, que acude a centro de salud por presentar de forma progresiva
desde hace 1 año dificultad para ingerir alimentos sólidos y luego líquidos; refiere
regurgitaciones alimentarias y marcada pérdida de peso (15 kilos). Radiografía baritada
(sustancia de contraste) de esófago se muestra en la figura. El presente caso se explica
por……………….
(Unidad 2, sesión 12, logro 4: Identificar y describir la función de los esfínteres esofágicos)

a. aumento de la peristalsis esofágica


b. relajación incompleta del esfínter pilórico
c. relajación incompleta del esfínter esofágico inferior
d. perdida de producción de PIV y óxido
nítrico en el esfínter esofágico superior

30. En condiciones normales, el ingreso de 600 ml de líquido es el estómago provoca un


aumento de presión intragástrica de unos 12 cm de H2O. Después de una vagotomía (corte
del nervio vago) es de esperar que el ingreso del mismo volumen de líquido ocasionede la
presión intragástrica.
(Unidad 2, sesión 13, logro 1: Describe la Motilidad gástrica: relajación receptiva)

a. la disminución
b. la no variación
c. un aumento mayor
d. un aumento similar o igual
SIS
TEMA
DIGESTIVO
(ME 154)
CLAVES
EXAMEN
FINAL
Ciclo 201900

1. Un niño de 2 años es llevado a la consulta por diarrea persistente, edema de las extremidades y
falta de crecimiento en relación a su edad. Los análisis de sangre revelan que tiene concentración
plasmática baja de proteínas (hipoproteinemia). Como parte del estudio se coloca colecistokinina
(CCK) endovenosa y se recoge muestras del líquido duodenal por endoscopía; el resultado del
líquido confirma incapacidad para hidrolizar proteínas a un pH neutro, esta situación mejora al
añadir una pequeña cantidad de tripsina. El paciente probablemente esté sufriendo la falta
congénita de ………….
(Unidad 4, sesión 26, logro 2: Explicar la Digestión y absorción de las proteínas y sus alteraciones)
a. PEPT-1
b. pepsinógeno
c. enterocinasa
d. carboxipeptidasas

2. Paciente mujer de 35 años acude a consulta por sensación de sequedad y lesiones en cavidad
oral. Al examen se observa atrofia de la mucosa, fisuras y úlceras; nota además sequedad e
irritación de la córnea y aumento del tamaño de las glándulas parotídas. Su diagnóstico más
probable es artritis reumatoide; el hallazgo más probable en una biopsia de glándula parótida
es……..….
(Unidad 3, sesión 18, logro 3: Describe las enfermedades más frecuentes de las glándulas salivales)
a. Presencia de acinos normales con hiperplasia de células ductales
b. Gran infiltración de linfocitos y células plasmáticas
c. Hiperplasia de acinos glandulares serosos
d. Gran infiltrado de linfocitos y neutrófilos

3. Un hombre de 42 años de edad se presenta al médico con una historia de 1 año de evolución,
caracterizado por dolor abdominal bajo y diarreas con crisis sanguinolentas. Manifiesta además
pérdida de peso de 8 kg durante este periodo. La colonoscopía revela lesión difusa en el colon
con afectación del recto. La biopsia de estas lesiones revela adelgazamiento de la pared,
inflamación y ulceración de la mucosa y sub mucosa. El diagnóstico más probable en este caso es:
(Unidad 4, sesión 28, logro 5: Describe la Enfermedad inflamatoria intestinal. Generalidades,
morfología y características)
a. sindrome de colon irritable
b. enfermedad de Crohn
c. colitis ulcerativa
d. sprue celiaco

4. Dos estudiantes deciden tomar un receso para comer una hamburguesa a la hora del almuerzo.
Antes de llegar a la cafetería, impulsos nerviosos provenientes del complejo vagal dorsal iniciarán
la secreción de ácido gástrico por la liberación dedesde el sistema nervioso entérico.
(Unidad 3, sesión 20, logro 2: Regulación de la secreción gástrica: estimulación, fases de la secreción)
a. Serotonina
b. Colecistoquinina
c. Péptido inhibidor vaso activo
d. GRP (péptido liberador de gastrina)

5. Un niño de cuatro años de edad es llevado a la consulta por cuadros diarreicos frecuentes
caracterizados por heces pálidas, voluminosas y fétidas; al examen físico presenta bajo peso
y talla para la edad. Se mide la concentración de cloruro en el sudor y se encuentra que sus
valores son muy elevados. La alteración más
importante a nivel de células ductales del páncreas tiene relación directa con la conductancia de…………
(Unidad 3, sesión 23, logro 5 Explica la Secreción pancreática: formación del jugo
pancreático, influencia de la velocidad de flujo y regulación)
a. Bicarbonato
b. Potasio
c. Sodio
d. Cloro
6. Se evalúa los valores séricos de las siguientes sustancias a un paciente con enfermedad hepática
terminal; en este paciente se espera encontrar la combinación con la letra …………
(Unidad 3, sesión 22, logro 5: Describe las Pruebas de función hepática, la Insuficiencia
hepática, encefalopatía hepática e hipertensión portal)
Glucosa Amoniaco Albúmin
a
a. Aumenta Disminuida Disminui
da da
b. Disminui Aumentada Aumenta
da da
c. Aumenta Aumentada Aumenta
da da
d. Disminui Aumentada Disminui
da da

7. Una mujer de 35 años de edad HIV positiva, se presenta al médico con dolor abdominal en
cuadrante superior derecho e ictericia. La paciente refiere haber tenido múltiples episodios de
ictericia durante los últimos 10 años. Los exámenes para determinar hepatitis viral, dieron
positivos para Hepatitis B, siendo catalogado el caso como hepatitis crónica con alteración
funcional. En un examen de sangre ¿cuál de los siguientes parámetros está disminuido?
(unidad 3, sesión 22, logro 5: Pruebas de función hepática, Insuficiencia hepática,
encefalopatía hepática e hipertensión portal)
a. Albumina
b. Bilirrubina
c. Fosfatasa alcalina
d. Tiempo de protrombina

8. En el reflejo peristáltico del intestino delgado ¿Cuál de los siguientes eventos sucede en la
porción caudal del bolo alimenticio?
(Unidad 2, sesión 13, logro 4: Explicar la Motilidad del intestino delgado: Contracciones
segmentarias y peristálticas)
a. Acción del péptido inhibidor vasoactivo (VIP) en el músculo circular
b. Acción del NO (óxido nítrico) en el músculo longitudinal
c. Contracción del músculo longitudinal interno
d. Acción de acetilcolina en el músculo circular

9. Un varón de 58 años de edad con enfermedad de Crohn severo fue sometido a una resección
ileal. Después de la cirugía este paciente padecerá de esteatorrea, esto se explica porque
…..………..
(unidad 4, sesión 26, logro 4: Explica las alteraciones en la Absorción de lípidos)
a. se inhibe la acción de la 7 alfa hidroxilasa
b. el pool de ácidos biliares se incrementa
c. hay mala absorción de ácidos biliares
d. el páncreas no secreta lipasa

10. En un experimento se inserta un balón en el estómago de un voluntario, se infla poco a poco


mientras que se vigilan las presiones intraluminales. Aunque el volumen del balón aumenta
considerablemente, las presiones permanecen constantes. Esta relación volumen-presión se
explica por la liberación local de …………..
(Unidad 2, sesión 13, logro 1 Explica la Motilidad gástrica: relajación receptiva)
a. acetil colina y gastrina
b. norepinefrina y óxido nítrico
c. colecistoquinina y óxido nítrico
d. óxido nítrico y péptido inhibidor vasoactivo

11. ¿Cuál de las siguientes alternativas es una característica de la secreción exocrina del páncreas?
(Unidad 3, sesión 23, logro 5: Secreción pancreática: formación del jugo pancreático, influencia de la
velocidad de flujo y regulación)
a. Es hipotónica respecto al plasma
b. Su mayor estímulo se da en la fase intestinal
c. Es estimulada por la presencia de bicarbonato en el duodeno
d. La secreción enzimática es estimulada principalmente por la secretina
12. Las estructuras en el hígado que permite que los productos metabólicos unidos a proteínas
tengan acceso a las membranas basolaterales de los hepatocitos, son…..
(Unidad 3, sesión 21, logro 4-5: Explica la Organización micro estructural del hígado)
a. los canalículos
b. las células de Ito
c. las fenestras sinusoidales
d. las uniones intercelulares herméticas

13. La composición de la bilis es modificada conforme fluye por los conductillos biliares.
Durante este tránsito se espera que aumente la concentración de…….
(Unidad 3, sesión 22, logro 2: Describe la Secreción biliar, visión general del sistema biliar
extrahepático y composición de la bilis)
a. Ig A
b. Glucosa
c. Protones
d. Vitamina A

14. Se mide experimentalmente el contenido gástrico de dos personas. La persona “A” tiene alto
contenido de grasa y la persona “B” tiene un contenido isotónico ¿Cuál de las siguientes es
correcto respecto al vaciamiento gástrico? (Unidad 2, sesión 13, logro 2: Describe la Motilidad y
vaciamiento gástrico)
a. Hay ralentización del vaciado gástrico solo en “A”
b. El vaciamiento gástrico es más rápido en ambos
c. Hay ralentización del vaciado gástrico solo en “B”
d. Hay ralentización del vaciado gástrico en ambos casos

15. El examen endoscópico de un paciente con hipertensión portal grave revela venas tortuosas que
sobresalen hacia la luz del esófago. El paciente recibe tratamiento quirúrgico mediante la
colocación de una derivación que conecta la vena porta a la vena cava. Después de la operación
el riesgo de encefalopatíay el riesgo
de sangrado de várices ……………..
(Unidad 3, sesión 22, logro 5: Describe la Insuficiencia hepática, encefalopatía hepática e hipertensión
portal)
a. disminuirá / disminuirá
b. disminuirá / aumentará
c. aumentará / disminuirá
d. aumentará / aumentará

16. Un paciente varón de 18 años de edad acude al médico para sus exámenes de rutina. Sus
resultados de laboratorio muestran un valor de bilirrubina sérica de 4 mg/dl y una bilirrubina
directa de 0,3 mg/dl. Las pruebas de función hepática son normales. La alteración que explica
mejor este caso es por la deficiencia de ………………..
(Unidad 3, sesión 22, logro 3: Explica la Producción y excreción de bilirrubina. Tipos de bilirrubina e
ictericia)
a. transaminasas
b. hemo oxigenasa
c. la 7 alfa hidroxilasa
d. glucuronil transferasa

17. Un hombre de 57 años de edad es llevado a urgencias con hematemesis masiva rojo brillante, a
su llegada se halla inconsciente con PA: 80/40 mm Hg y FC: 124 lat/min. Luce ictérico con
presencia de “arañas vasculares en el tórax anterior y extremidades”, abdomen distendido con
signo de oleada positiva. Se encuentra esplenomegalia y pérdida de la masa muscular en
extremidades. La anastomosis vascular responsable del sangrado en este paciente es ………….…..
(Unidad 3, sesión 21, logro 2: Describe las anastomosis porto sistémicas)
a. vena gástrica izquierda y vena ácigos
b. arteria gástrica izquierda y vena ácigos
c. vena paraumbilical y vena epigástrica inferior
d. vena gástrica izquierda y vena esofágica superior
18. Un estudiante de medicina está comiendo un plato de comida a base de champiñones,
espárrago y salsa de soya. El estímulo del sabor umami contenido en todos estos alimentos viaja
a través del nervio………………..
(Unidad 2, sesión 10, logro 3: Describe la irrigación e inervación de la lengua)
a. Lingual
b. Hipogloso
c. Glosofaringeo
d. Cuerda del tímpano
19. Una paciente de 30 años de edad es sometida a una cirugía en oído medio derecho por un
problema de otoesclerosis. Luego de la cirugía refiere alteración sensitiva de la lengua. Al
evaluar el caso usted esperaría encontrar……….
(Unidad 2, sesión 10, logro 5: Describe la Irrigación e inervación de la lengua)
a. Alteración en la sensación del dolor y temperatura en el tercio posterior de la lengua
b. Alteración en la sensación del dolor en los dos tercios anteriores de la lengua
c. Alteración en la sensación del gusto en el tercio posterior de la lengua
d. Sensación del dolor, tacto y temperatura conservadas

20. En un paciente de 45 años de edad con colestasis biliar, se encuentra una elevación de los niveles
sanguíneos de fosfatasa alcalina hasta 3 veces la cifra normal. ¿Cuál de las siguientes alternativas
estará también elevada como evidencia del daño de la vía biliar?
(Unidad 3, sesión 22, logro 5: Pruebas de función hepática, Insuficiencia hepática,
encefalopatía hepática e hipertensión portal)
a. Tiempo de protrombina y albúmina sérica
b. Transaminasas hepáticas (ALT y AST)
c. Gamma glutamil transpeptidasa
d. Glucoronil transferasa

21. Experimentalmente se incrementa la velocidad de la secreción salival con una sustancia,


en el análisis de la composición de esta saliva obtenida se espera encontrar…………..
(Unidad 3, sesión 17, logro 5 : Explica la Influencia de la velocidad del flujo salival en la composición de la
saliva)
a. disminución de la concentración de bicarbonato que supera la concentración plasmática
b. aumento de la concentración de cloro y sodio que supera la concentración plasmática
c. aumento de la concentración de bicarbonato que supera la concentración plasmática
d. disminución de concentración de potasio y bicarbonato

22. Lactante de 3 meses de vida es atendido por presentar diarrea, se administra una
solución de glucosa y electrólitos por vía oral. La proteína de membrana apical que
explica la capacidad de esta solución para proporcionar aporte de glucosa e hidratación
es ………..
(Unidad 4, sesión 26, logro 1: Explica la Digestión y Absorción de los hidratos de carbono. Alteraciones)
a. CFTR
b. SGLT-1
c. GLUT-2
d. GLUT-5

23. Paciente ha sufrido herida de bala en el abdomen, se le ha tenido que extirpar el segmento
medio y distal del ileon. En este caso la síntesis hepática de sales biliares estará …..…..
(Unidad 3, sesión 22, logro 4: Explica la formación, función y Circulación entero hepática de lasa sales
biliares)
a. Sin cambios en el ritmo de síntesis
b. Disminuida por inhibición de la enzima colesterol 7 alfa hidroxilasa
c. Incrementada por estímulo de la enzima colesterol 7 alfa hidroxilasa
d. Incrementada por inhibición de la enzima colesterol 7 alfa hidroxilasa

24. Un varón de 75 años ingresa al consultorio por presentar ictericia marcada de piel y las
escleras. El estudio del paciente mostró que presentaba un tumor que obstruía la totalidad del
conducto hepático común. ¿Cuál de los siguientes conductos se encontrará dilatado en este
paciente?
(Unidad 3, sesión 21, logro 6: Describir el árbol biliar intrahepático)
a. de Wirsung
b. de Hering
c. colédoco
d. cístico
25. Correlaciones las dos columnas y marque la fórmula correcta:
(Unidad 4, sesión 28, logro 1: Diarrea: definición, mecanismos: osmótica, secretoria y exudativa)

1. Enfermedad Hirschsprung( ) heces con moco y sangre


2. Diarrea osmótica( ) intolerancia a lactosa
3. Diarrea secretoria( ) aganglionosis congénita
4. Diarrea exudativa( ) canales de Cl- en las células de la cripta a.- 4231b.-

1234c.- 2143d.- 4213

26. La vena umbilical obliterada del hígado después del nacimiento se transforma en el ligamento:
(Unidad 3, sesión 21, logro 1: Hígado: relación con la pared abdominal, caras, lóbulos, ligamentos , hilio
hepático)
a. cruzado
b. redondo
c. coronario
d. falciforme

27. Llega a su guardia nocturna una madre que trae a su RN masculino de 2 semanas de vida con mal
estado general y sequedad de mucosas. Usted observa que lacta ávidamente, pero a las 2 horas
presenta vómito postprandial no bilioso en proyectil. Al realizar la historia clínica, descubre que el
lactante recibió profilaxis con macrólidos para tos ferina. Usted sospecha principalmente en:
(Unidad 1, sesión 4, logro 4: Desarrollo y anomalías del intestino anterior)
a. estenosis pilórica hipertrófica congénita
b. fistula traqueo esofágica
c. estenosis duodenal
d. atresia duodenal

28. En la regulación del apetito y la saciedad, la estimulación experimental crónica del núcleo
ventro medial del hipotálamo producirá:
(Unidad 1, sesión 3, logro 4: explica los mecanismos de control del apetito y saciedad)
a. afagia
b. obesidad
c. hiperfagia
d. activación de neuronas relacionadas a NPY

29. Paciente mujer de 25 años acude por dolor en fosa ilíaca derecha que empeora al toser o
caminar, asociada a náuseas y vómitos por lo cual acude a emergencia. Dos días después de
realizarle una apendicectomía, la paciente desarrolla fiebre alta (39 °C), está hipotensa y presenta
dolor abdominal. La laparotomía exploratoria muestra un gran volumen de sangre en la cavidad
peritoneal por lesión de un vaso producida durante la apendicectomía.
¿Cuál de las siguientes arterias debe ligarse para detener la hemorragia?
(Unidad 4, sesión 27, logro 4: Irrigación arterial del colon, recto y conducto anal)
a. cólica derecha y arteria rectal superior.
b. ileocólica y arteria cólica media.
c. mesentérica superior.
d. ileocólica.

30. La onda peristáltica secundaria del esófago se caracteriza por ser originada ………
(unidad 2, sesión 12, logro 3: Motilidad esofágica: fases y características)
a. por el plexo de meissner del esófago
b. por el plexo mientérico del esófago
c. por el reflejo de la deglución
d. durante la masticación

31. ¿Cuál de los siguientes es una causa de ictericia con bilirrubina conjugada aumentada?
(Unidad 3, sesión 22, logro 3: Producción y excreción de bilirrubina. Tipos de bilirrubina, ictericia)
a. Ictericia del recién nacido
b. Obstrucción del colédoco
c. Anemia hemolítica
d. Gran hematoma
32. En relación a la absorción de nutrientes, la absorción de dipéptidos y tripéptidos a nivel de las
células epiteliales del intestino delgado, se da principalmente debido a:
(Unidad 4, sesión 26, logro 2: Digestión y absorción de las proteínas. Alteraciones)
a. el incremento de los canales de Cl- en la membrana apical
b. la gradiente de bicarbonato en la membrana basal
c. la gradiente de iones H+ en la membrana apical
d. la gradiente de Na+ en la membrana apical

33. Paciente de 20 años es traído a la emergencia por presentar diarreas desde hace 2 días. Familiar
refiere que las deposiciones son líquidas y abundantes, al examen luce deshidratado y se plantea
que la diarrea es producida por una toxina que estimula la transformación de ATP a AMPc con
apertura de canales de Cl- y pérdida de agua. El tipo de diarrea más probable es:
(Unidad 4, sesión 28, logro 1: Diarrea: definición , mecanismos: osmótica, secretoria y supurativa)
a. osmótica
b. exudativa
c. secretoria
d. por intolerancia a lactosa

34. Un niño fue operado por una obstrucción intestinal, observándose la presencia de divertículo de
Meckel. Según lo referido, marque lo correcto:
(Unidad 1, sesión 5, logro 2: identifica las anomalías del desarrollo del intestino medio: onfalocele y
gastrosquisis (diferencias), Divertículo de Meckel)
a. el 50% de la población lo presenta
b. se localiza en el íleon muy cerca al yeyuno
c. puede poseer tejido gástrico o pancreático
d. se produce por una mala rotación de los intestinos

35. Marque la alternativa


correcta respecto a la
estructura marcada en el
gráfico:
(Unidad 3, sesión 22, logro 2:
Secreción biliar. Visión general
del sistema biliar extrahepático
y composición de la bilis)

a. Se halla a 2
centímetros debajo
de la papila
duodenal mayor
b. Llega el conducto
colédoco y pancreático
principal
c. Llega el conducto
hepático común y
pancreático
principal
d. Llega el conducto pancreático accesorio

36. ¿Cuál de las siguientes moléculas se encontrará aumentada en el citoplasma de las células
parietales de un paciente con sindrome de Zollinguer Ellison?
(Unidad 3, sesión 20, logro 4: Enfermedad ulcerosa péptica: úlcera gástrica, duodenal.
síndrome de Zollinger – Ellison)
a. Péptido liberador de gastrina (GRP)
b. Proteína G estimulante (GS)
c. Inositol Trifosfato (IP3)
d. AMP cíclico (AMPc)

37. Los fármacos inhibidores de la bomba de protones, actúan bloqueando la ………..……..


(Unidad 3, sesión 20, logro 3: Regulación de la secreción gástrica: inhibición, Secreción de
pepsinógeno y factor intrínseco)
a. anhidrasa carbónica
b. ATPasa H+/K+ en la membrana luminal
c. ATPasa H+/K+ en la membrana basolateral
d. ATPasa Na+/K+ en la membrana basolateral
38. Un paciente fue diagnosticado de gastritis autoinmune, ¿cuál de las siguientes alternativas es
FALSA respecto a esta enfermedad?
(Unidad 3, sesión 20, logro 5: Gastritis crónica: helicobacter pylori, autoinmune. Tipos de gastritis)
a. Afecta principalmente el fondo y cuerpo gástrico
b. Se produce hiperplasia de células G secundaria a la aclorhidria
c. El propio sistema inmune destruye principalmente las células parietales
d. Se produce atrofia de la mucosa, aclorhidria, hipergastrinemia y déficit de vitamina B6

39. Marque la correlación correcta:


(Unidad 3, sesión:18, logros:1 y 2: Describe las enfermedades inflamatorias/infecciosas y
proliferativas de la cavidad oral)

1. Herpes virus( ) En relación al abuso de antibióticos


2. Candidiasis oral( ) Lesiones vesiculares como racimo de uvas
3. Eritroplaquia( ) Mega esófago
4. Enfermedad de Chagas( ) Lesión pre cancerígena

a.- 2431b.- 1234c.- 4123d.- 2143

40. En un paciente con insuficiencia renal crónica, el déficit en la absorción de calcio a nivel del
enterocito se debe a lo siguiente:
(Unidad 4, sesión 26, logro 6: Explica la Absorción de calcio y hierro)
a. No se convierte la 25 hidroxicolecalciferol a 1,25 dihidroxicolecalciferol
b. No se convierte la 1,25 dihidroxicolecalciferol a 25 hidroxicolecalciferol
c. Existe un descenso de la alfa 25 hidroxilasa renal
d. Se incrementa la producción de Calbindina
SISTEMA DIGESTIVO
(ME154) EXAMEN
PARCIAL 2019 01

Profesores : Alfaro Salazar, Herberth Romulo; Callata Caceres, Gunter; Cayo Quiñe, Alexandra Mariel; Correa
Borit, Jorge Mauricio; Cruz Cutty, Lourdes Marylin; Guzmán Calderón, Gerly Edson; Jáuregui
Farfán, Jorge Jesús; Mayor Zevallos, Otto Alberto; Montoya Suárez, José Luis; Palacios Bazan,
Enrique Elias; Robles Pino, Alexander Anibal; Wong Bravo, Juan Carlos
Sección : Todas las secciones
Duración : 50 minutos.
Indicaciones:
Lea atentamente cada pregunta antes de responder:
- Se prohíbe el uso del celular y cualquier dispositivo electrónico.
- Está prohibido intercambiar materiales.
- Coloque su código de alumno en la tarjeta de respuestas. Si su código contiene una letra reemplácela por
un valor numérico siguiendo la siguiente equivalencia: A=9, B=8, C=7, D=6, E=5, F=4, G=3.
- Traslade sus respuestas a la tarjeta, llenando los círculos de manera completa con lapicero negro o azul.
Está prohibido el llenado con lápiz, lapicero de otro color o con lapicero de tinta borrable.
- Sea cuidadoso en el llenado de la tarjeta de respuestas, pues solo esta tiene validez para la calificación.
- Al terminar su examen avise al docente a cargo, no se levante de su sitio; debe entregar la hoja de
respuestas con la carátula del examen, este cuadernillo de preguntas se lo llevará cada estudiante.

1. ¿Cuál de las glándulas salivales es responsable del mayor porcentaje del volumen de la saliva en
condiciones basales?
a. Parótida
b. Subpalatin
as c.
Sublingula
es d.
Submaxilar
es

2. La lengua está recubierta por epitelio:


a. pseudoestratificado columnar no queratinizado
b. plano estratificado no queratinizado
c. pseudoestratificado columnar ciliado
d. plano estratificado queratinizado

3. El esfínter anal interno tiene musculatura …………… y tiene control ………………….


a. lisa / voluntario
b. lisa / involuntario
c. esquelética / simpático
d. esquelética / parasimpático

4. La arteria aorta proporciona la irrigación al tubo digestivo ¿cuál de las siguientes arterias proporciona la
irrigación al ángulo cólico derecho?
a. Mesentérica superior
b. Mesentérica inferior
c. Frénica inferior
d. Tronco celiaco

5. Paciente de 26 años que le cuenta en su historia clínica que cada vez que almuerza, a los 20 minutos
tiene deseo de defecar. Le comenta que su hijo de 1 mes le pasa lo mismo pero más intenso. Esto se
explica por el reflejo
…………………, el cual estáen el paciente.
a. colicoileal / normal
b. colicoileal / alterado
c. gastrocolico / normal
d. gastrocolico / alterado
6. La región del estómago que se comunica con el duodeno se denomina:
a. pilórica
b. cardias
c. cuerpo
d. fórnix

7. Acude a consulta un paciente que fue diagnosticado de ulcera péptica 3 días antes. Luego de múltiples
pruebas diagnósticas, se concluye que el paciente presenta un tumor secretor de gastrina ¿Cuál de las
siguientes situaciones estará incrementada?
a. Distención gástrica
b. Inhibición del vaciado gástrico
c. Secreción de ácido clorhídrico (HCl)
d. Inhibición de la secreción de pepsinógeno

8. En el sistema digestivo, el control del apetito esta dado por un complejo sistema de sustancias y órganos
integradores los cuales regulan la ingesta de alimentos. Laes una sustancia oroxígena y es sintetizada
por el ……………………..
a. leptina / intestino
b. grelina / intestino
c. leptina / estómago
d. grelina / estómago

9. Sobre el control autónomo del sistema digestivo, marque la alternativa correcta:


a. La inervación dada por el sistema simpático es de tipo preganglionar.
b. El sistema parasimpático usa como neurotransmisores a la acetilcolina y la noradrenalina.
c. El nervio vago (par craneal X) le da inervación simpática a la mayoría del sistema digestivo.
d. En el sistema simpático, los nervios responsables hacen una primera sinapsis en ganglios
próximos al órgano a inervar.
e. En la inervación de tipo parasimpático, solo interviene el plexo submucoso, sin embargo, en la
de tipo simpático intervienen tanto el submucoso como el mientérico.

10. Con respecto a la actividad eléctrica del sistema digestivo, marque la alternativa correcta
a. Corresponden a potenciales de acción que están presentes de forma continua y le dan la
capacidad de perístasis autónoma al sistema digestivo.
b. La frecuencia de las ondas lentas no se ve influenciada por la actividad neural ni las
hormas gastrointestinales.
c. En el estómago las ondas lentas se dan en una frecuencia de 6 por minuto.
d. Las ondas lentas son cambios lentos y ondulantes del potencial en reposo.
e. La frecuencia de las ondas lentas va de 6 a 12 ondas por minuto.

11. Ante una lesión del IX par craneal, el músculo…se altera en su función.
a. palatogloso
b. estilofaríngeo
c. palatofaríngeo
d. constrictor superior

12. Un varón de 50 años es sometido a extirpación de duodeno y parte proximal de yeyuno. La pérdida de
estímulo hormonal en el páncreas para la secreción enzimática se explica por la pérdida de las células
……………………
a. Parietales, productoras de factor intrínseco
b. “K” productoras de factor intrínseco
c. “M” productoras de CCK
d. “I” productoras de CCK
13. Respecto al mecanismo de la defecación ¿Cuál de las siguientes afirmaciones es correcta?
a. Se produce contracción refleja del esfínter anal interno
b. Se produce contracción o relajación del esfínter anal externo por señales de la corteza cerebral
c. La presencia de materia fecal en el recto estimula la contracción del sigmoides por los nervios
pélvicos simpáticos
d. En la posición de “cuclillas” el músculo puborectal se halla contraído favoreciendo la
evacuación de la materia fecal

14. Un niño de tres años llega a emergencia con disfagia (dificultad para tragar), salivación y llanto. Se sospecha
de ingesta de cuerpo extraño: moneda en el esófago; al ser evaluado se constata en una radiografía presencia
de cuerpo extraño a nivel de C6 y C7 (6° y 7° vértebra cervical). El cuerpo extraño estará suspendido a nivel
del estrechamiento producido por el ………..
a. cayado aórtico
b. hiato esofágico
c. músculo cricofaríngeo
d. bronquio principal izquierdo

15. En el caso de un paciente con un tumor productor de gastrina, la presencia de úlceras duodenales y
erosión de la mucosa gástrica se debe principalmente a…….
a. la acción paracrina de la gastrina sobre la célula parietal
b. el exceso de HCl por estímulo de receptores CCK-B en la célula parietal
c. la sobre expresión de los receptores “G” para gastrina en la célula parietal
d. el exceso de HCl por estímulo directo de receptores “H” en la célula parietal

16. La onda peristáltica secundaria del esófago se caracteriza por ser originada ………
a. por el plexo de submucoso del esófago
b. por el plexo mientérico del esófago
c. por el reflejo de la deglución
d. durante la masticación

17. Marque lo correcto sobre las ondas lentas en el tubo digestivo


a. No son despolarizaciones
b. Son potenciales de acción subumbrales
c. Se constituyen de despolarizaciones y repolarizaciones
d. Son rítmicas y generadas por el sistema nerviosos autónomo

18. Recién nacido que presenta protrusión de contenidos abdominales los cuales no están cubiertos por
peritoneo y salen de la cavidad abdominal a través de un defecto de la pared. ¿Cómo se denomina a la
afección que presenta este paciente?
a. Onfalocele
b. Atresia biliar
c. Gastrosquisis
d. Divertículo de Meckel

19. Experimentalmente se utiliza atropina (anticolinérgico) para inhibir la secreción de gastrina, sin
embargo, la secreción de esta hormona se sigue dando ante estímulos vagales. Esta situación se explica
porque la atropina:
a. no bloquea la acción del péptido GRP
b. solo inhibe la acción del péptido GRP en la célula G
c. inhibe la acción de acetilcolina e histamina en la célula G
d. bloquea parcialmente la bomba de protones en la célula G

20. Niña de 4 días es llevada a la emergencia pediátrica por presentar llanto constante, la madre refiere
coloración azulada de labios al momento de lactar, acompañado de tos persistente y dificultad respiratoria
así como distención abdominal. Se le coloca sonda nasogástrica para alimentación notando que retorna a
la cavidad oral en todos los intentos. ¿Cuál es la anomalía del desarrollo en este caso?
a. Solo fístula traqueo esofágica
b. Fístula traqueo esofágica proximal y distal
c. Atresia esofágica proximal con fístula traqueo esofágica distal
d. Atresia esofágica distal con fístula traqueo esofágica proximal
21. Paciente varón de 36 años es traído a la emergencia luego de sufrir un accidente de tránsito, presenta
traumatismos múltiples en cabeza y tronco. Al examen físico se evidencia hematoma en hemicara izquierda,
ligera protrusión y caída del lado izquierdo del maxilar inferior, por lo que se realiza una tomografía donde
se halla una fractura de la apófisis coronoides del maxilar inferior. ¿Qué músculo está relacionada
directamente con esta situación?
a. Masetero
b. Temporal
c. Buccinador
d. Pterigoideo medial

22. Un paciente refiere no percibir algunos sabores. Al examen físico constata alteración del sabor dulce y
umami.
¿Cuál de los siguientes nervios estará alterada su función?
a. Cuerda del tímpano (VII par)
b. Lingual (rama del V par)
c. Glosofaríngeo (IX par)
d. Hipogloso (XII par)

23. A los pocos días de nacido, regresa a neonatología un niño con problemas de motilidad del colon; los
estudios determinan ausencia congénita de células ganglionares. Según el gráfico ¿cuál es la capa en la que
se determina la ausencia de dichas células?
a. Mucosa - 2
b. Muscular propia - 2
c. Muscular propia - 3
d. Muscular de la mucosa - 3

1 2

24. Con respecto al control autonómico en el tracto gastrointestinal y en relación a su fisiología. ¿Cuál es la
función del sistema nervioso parasimpático en el tracto gastrointestinal?
a. Inhiben la contracción muscular y estimulan la secreción de sustancias a nivel de la submucosa
b. Estimulan la contracción muscular y estimulan la secreción de sustancias a nivel de la mucosa
c. Inhiben la contracción muscular e inhiben la secreción de sustancias a nivel de la submucosa
d. Estimulan la contracción muscular e inhiben la secreción de sustancias a nivel de la mucosa

25. Un estudiante que está preocupado por su examen parcial, no ha desayunado ni almorzado; cuando al fin
ingiere alimentos, esto le provoca el aumento de los movimientos musculares del tracto gastrointestinal y
la sensación de defecar. ¿Qué reflejo se ha activado?
a. Entero-gástrico
b. Gastro-cólico
c. Cólico-ileal
d. Ileo-ileal
26. ¿De qué par craneal es rama el nervio palatino mayor?
a. Vago
b. Hipogloso
c. Trigémino
d. Palatogloso

27. ¿En cuál de las fases de la deglución la epiglotis separa la vía respiratoria de la digestiva?
a. oral
b. laríngea
c. faríngea
d. esofágica

28. Los péptidos intestinales se pueden clasificar como sustancias endocrinas, neurocrinas y paracrinas,
dentro de las paracrinas se encuentran la somastotatina e histamina. Marque la respuesta correcta
a. La somastotatina es sintetizada por las células B de la mucosa gástrica
b. La histamina actúa estimulando su receptor tipo H1 en la mucosa gástrica
c. La histamina es sintetizada por células de tipo paracrino de las glándulas gástricas
d. La somatostatina presenta dentro de sus funciones la estimulación de la secreción de H +

29. En relación a los órganos intraabdominales y sus estructuras de fijación, elija la alternativa
correcta a.El mesenterio permite la suspensión e irrigación de los órganos
retroperitoneales
b. Tanto el hígado como la vesícula biliar se encuentran ubicados a nivel del flanco derecho
c. El colon, el duodeno y el resto de intestino delgado son órganos considerados
netamente como peritoneales
d. Los ligamentos que encontramos dentro de la cavidad abdominal son el esplenorenal y el
gastrofrénico e.Los omentos van desde el estómago y la segunda porción del duodeno a otras
estructuras
intraabdominales y existen dos: el omento mayor y el omento menor

30. Paciente varón de 27 años es llevado por bomberos a emergencia luego de ser asaltado y, tras resistirse, es
cortado con el pico de una botella a nivel abdominal. Al examen físico usted observa que a través de la herida
se puede observar la protrusión de asas intestinales. En relación con las capas de la pared abdominal, marque
la alternativa correcta.
a. La fascia de Scarpa está constituida principalmente por tejido adiposo
b. La pared abdominal está formada por piel, huesos, músculos, fascias y peritoneo parietal
c. La fascia de Camper es una estructura fibrosa que carece de grasa y su grosor es constante en
toda la pared abdominal
d. El músculo oblicuo externo discurre en dirección súpero-interna y se inserta en el borde
inferior de las ultimas 3 a 4 costillas
e. El músculo recto del abdomen tiene como funciones comprimir el contenido del abdomen,
tensar la pared del abdomen y flexionar la columna
SISTEMA DIGESTIVO
(ME154) EXAMEN
FINAL 2019 01

Profesores : Alfaro Salazar, Herberth Romulo; Callata Caceres, Gunter; Cayo Quiñe, Alexandra Mariel; Correa
Borit, Jorge Mauricio; Cruz Cutty, Lourdes Marylin; Guzmán Calderón, Gerly Edson; Jáuregui
Farfán, Jorge Jesús; Mayor Zevallos, Otto Alberto; Montoya Suárez, José Luis; Palacios Bazan,
Enrique Elias; Robles Pino, Alexander Anibal; Wong Bravo, Juan Carlos
Sección : Todas las secciones
Duración : 70 minutos.
Indicaciones:
Lea atentamente cada pregunta antes de responder:
- Se prohíbe el uso del celular y cualquier dispositivo electrónico.
- Está prohibido intercambiar materiales.
- Coloque su código de alumno en la tarjeta de respuestas. Si su código contiene una letra reemplácela por
un valor numérico siguiendo la siguiente equivalencia: A=9, B=8, C=7, D=6, E=5, F=4, G=3.
- Traslade sus respuestas a la tarjeta, llenando los círculos de manera completa con lapicero negro o azul.
Está prohibido el llenado con lápiz, lapicero de otro color o con lapicero de tinta borrable.
- Sea cuidadoso en el llenado de la tarjeta de respuestas, pues solo esta tiene validez para la calificación.
- Al terminar su examen avise al docente a cargo, no se levante de su sitio; debe entregar la hoja de
respuestas con la carátula del examen, este cuadernillo de preguntas se lo llevará cada estudiante.

1. La explicación fisiológica de presentar somnolencia de 30 minutos a 1 hora después de ingerir


alimentos, se explica por:
a. Aumento del cloro intraluminal
b. Aumento del bicarbonato intraluminal
c. Disminución de ácido carbónico en la célula parietal
d. Disminución de la actividad de la anhidrasa carbónica
e. Aumento de la alcalinidad sanguínea

2. Con respecto a la irrigación arterial del colon, a que arteria corresponde la


señalada con la flecha
a. Cólica derecha
b. Cólica media
c. Cólica izquierda
d. Ileobisecoapendículocólica
e. Arco de Riolano

3. Si un paciente presentara dentro del punto de vista fisiológico, una


disminución de enterocinasa, entonces esto originaría una disminución de
la actividad de:
a. la
pepsina
b.la
lipasa
c. la quimotripsina
d. el peptido insulinotropo dependiente de glucosa
e. la amilasa

Se valida la opción b debido a su relación con la colipasa.


4. Con respecto a la anatomía del hígado, señale a que estructura
pertenece la marcada por el número 1.
a. Ligamento falciforme
b. Línea de Cantlie
c. Ligamento triangular
d. Ligamento coronario
e. Ligamento teres

Se valida la opción a debido a la ubicación del número 1 en donde se


unen el ligamento falciforme y ligamento coronario.

5. Se presenta un paciente, el cual presenta un antecedente de tuberculosis intestinal, por lo cual, se le


resecó 80 cm de íleon distal. Desde el punto de vista fisiológico, el paciente puede presentar una de las
siguientes alteraciones:
a. Disminución de la secreción de Vitamina B12
b. Aumento indiscriminado de absorción de ácido fólico
c. Disminución de la absorción de hierro
d. Aumento de la secreción de bicarbonato
e. Disminución de la absorción de ácido glicocólico

6. Un paciente es sometido experimentalmente a un fármaco que modifica el flujo salival, obteniéndose un


volumen de saliva de 288 ml en 6 horas. En este caso las concentraciones de electrolitos y bicarbonato en la
saliva obtenida varían de la siguiente manera:
a.↑ Na+, ↓ K+, ↑ Cl-, ↑
HCO3- b.↓ Na+, ↓ Cl-, ↑
K+, ↓ HCO3-
c.↑ Na+, ↑ Cl-, ↓ K+, ↓ HCO3-
d.↑ Na+, ↑ Cl-, ↑ K+, ↑ HCO3-
e.↓ Na+, ↓ Cl-, ↓ K+, ↓ HCO3-

Se valida la opción a debido a que se puede considerar como un aumento del flujo de saliva.

7. La siguiente imagen histológica corresponde a la glándula


…………… y la estructura señalada produce ………
a. salival sublingual / mucopolisacáridos
b. oxíntica / pepsinógeno
c. salival submaxilar / ptialina
d. salival parótida / amilasas
e. antrales / gastrina

8. Paciente varón de 65 años con antecedente de hipercolesterolemia, hipertensión arterial, fibrilación


auricular y dos infartos al miocardio previos, aqueja de dolor abdominal intenso de inicio súbito,
distensión abdominal, se decide cirugía con resección de 1,5 metros de intestino delgado terminal y colon
ascendente. Como consecuencia de la resección el paciente tendrá deficiencia de:
a. Vitamina C
b. Tiamina
c. Vitamina A
d. Vitamina B1
e. Vitamina B6

Se valida esta opción debido a que su absorción está relacionada al íleon.


9. Uno de los siguientes elementos debería hallarse con más probabilidad en el esófago de un paciente que
sufre de
reflujo gastro esofágico…
a. Pepsina
b. Tripsina
c. Quimiotripsina
d. Carboxipeptidasa
e. Ácidos biliares

10. Un paciente de 40 años cursa con anemia de 8g/dl, aqueja además de astenia y sensación de hormigueo
bilateral en los miembros inferiores, al examen se halla alteración de la sensibilidad a la vibración y camina
con ampliación de la base de sustentación. Uno de los siguientes procedimientos sería de ayuda para el
diagnóstico de este paciente:
a. Tomografía cerebral
b. Biopsia de la mucosa gástrica
c. Biopsia de hígado
d. Examen de sangre oculta en heces
e. Biopsia de Ileon proximal

11. Paciente de 60 años ingresa por caída hace 1 hora y pequeño hematoma en cuero cabelludo, al examen
físico ampliado se observa ictericia de piel y mucosas generalizada, abdomen blando, se palpa estructura
quística no dolorosa en hipocondrio derecho que corresponde a vesícula biliar (signo de Courvoisier), en
los exámenes de laboratorio se halla niveles bajos en la formación de estercobilinógeno y urobilinógeno
en heces, incremento de la bilirrubina conjugada en la orina, elevación de fosfatasa alcalina y gamma
glutamil transpeptidasa séricas. El presente cuadro puede ser explicado por:
a. Reabsorción de hematoma
b. Litiasis vesicular
c. Carcinoma de la cabeza de páncreas
d. Carcinoma con estenosis del conducto hepático común
e. Anemia hemolítica

12. Paciente varón de 58 años con antecedente de alcoholismo crónico es diagnosticado y recibe tratamiento
por cirrosis hepática. Hace 2 días refiere familiar que tuvo cambio de conducta y no reconoce a algunos
familiares. Al examen físico, se halla ascitis, circulación colateral en abdomen, telangiectasias, en el
examen de sistema nervioso: rigidez de extremidades, ROT incrementados, desorientación en el espacio y
asterixis. ¿cuál de las siguientes circunstancias, explicaría el cuadro en este paciente?
a. Uso de diuréticos ahorradores de potasio
b. Incremento de actividad de ureasa bacteriana duodenal
c. Hemorragia gastrointestinal
d. Disminución de la producción de NH3+ en el colon
e. Dieta normo proteica

Se valida la opción e debido al efecto sobre la encefalopatía.


Con respecto de la opción b es incorrecta debido a que hace referencia al duodeno, debería indicar colon.

13. Un recién nacido presenta vómitos biliosos poco tiempo después de cada alimento. Al preguntar a la
madre sobre antecedentes, ella recuerda que tuvo polihidramnios durante la gestación, pero un análisis de
cariotipo fue normal. Una de las siguientes es la causa más probable de estos hallazgos en el recién nacido:
a. Enfermedad de Hirschprung
b. Fístula tráqueo esofágica
c. Divertículo ileal
d. Estenosis pilórica
e. Malrotación de la yema pancreática ventral

14. Un lobulillo hepático se puede dividir en tres zonas como se muestra en el gráfico. ¿Cuál de las
siguientes afirmaciones sobre las tres zonas es verdadera?
a. La zona 1 tiene los menores depósitos de glucógeno
b. La zona 3 es la primera en afectarse en una colestasis extra hepática
c. La zona 2 es más susceptible a la injuria por isquemia que la zona
periportal d.La zona 2 tiene la mayor capacidad de regeneración
e.La zona 1 es la que tiene menos actividad metabólica.

La pregunta 14 ha sido anulada, sin embargo, ningún estudiante se verá afectado


negativamente en su puntaje debido a esta anulación.

15. En un estudio de la secreción de hormonas gastrointestinales, sus concentraciones en la vena porta se


midieron durante perfusión luminal del intestino delgado con soluciones de diversas magnitudes de pH.
¿Qué hormona aumentará en el plasma de la vena porta durante perfusión a través del intestino con
una solución de pH 3?
a. CCK
b. gastrina
c. GIP
d. motilina
e. secretina

16. Paciente de 30 años que ingresa a causa de un traumatismo abdominal cerrado. En la exploración se
aprecia discreta palidez de piel y mucosas, auscultación pulmonar normal, taquicardia de 120 /min.
Discreta distensión abdominal y matidez en flancos; el hematocrito, que era prácticamente normal al
ingreso, disminuye a 30% a las tres horas. En la Rx de tórax se objetiva fractura de las costillas 10-11
izquierdas. La causa más probable de la anemización en este paciente es:
a. traumatismo renal con hemorragia retroperitoneal.
b. rotura de hígado con hemoperitoneo.
c. rotura de bazo con hemoperitoneo.
d. rotura de mesos con hemoperitoneo.
e. traumatismo pancreático con pancreatitis traumática.

17. Mujer de 65 años. Consulta por síndrome constitucional


asociado a dolor abdominal epigástrico progresivo irradiado a
espalda, de dos meses de evolución. El diagnostico de sospecha
de adenocarcinoma de páncreas se confirma por biopsia. Se
realiza examen de imagen de abdomen para evaluación de
estructuras vasculares próximas al tumor pancreático. ¿Cuál es
el nombre de la vena señalada que está ausente, trombosada
por infiltración tumoral, condicionando circulación colateral en
la pared gástrica?

a. Mesentérica superior
b. Coronaria estomaquica
c. Esplénica
d. Porta
e. Renal izquierda
18. Revisando la angiotomografía de un hombre de 70 años en estudio por aneurisma de aorta abdominal,
el radiólogo le informa de la presencia de una oclusión completa de la arteria mesentérica inferior. El
paciente se encuentra completamente asintomático. La oclusión de la arteria mesentérica inferior
cursa de manera asintomática en muchas ocasiones ya que el territorio que irriga puede recibir flujo
proveniente de la arteria:
a. cólica derecha
b. gastroduodenal
c. Epigástrica inferior izquierda
d. esplénica
e. cólica media

19. En las patologías de esófago es importante conocer bien la anatomía esofágica. ¿Cuál de las
siguientes afirmaciones es correcta?
a. El esófago tiene capa mucosa, muscular y serosa
b. El esófago abdominal es más largo que el cervical
c. El esófago torácico pasa por detrás del cayado aórtico
d. El epitelio esofágico normal es de tipo cilíndrico.
e. El esófago abdominal es discretamente más largo que el torácico

20. A pesar de que pueda haber variaciones anatómicas, lo habitual es que el ciego sea irrigado por una rama
arterial que proviene de unas de las siguientes arterias:
a. Iliaca derecha
b. Mesentérica inferior
c. Hepática derecha
d. Mesentérica superior
e. Iliaca izquierda

21. Ante un paciente con una cirugía abdominal urgente, el informe operatorio señala que se ha realizado una
resección de todo el duodeno y del tercio proximal del yeyuno manteniendo íntegros el estómago y todo
el íleon, así como los dos tercios distales del yeyuno. En el seguimiento nutricional del paciente ¿Qué
vitamina o mineral presentará con menor probabilidad una disminución de su absorción?
a. Cianocobalamina
b. Calcio
c. Hierro
d. Transcobalamina
e. Transferrina

22. ¿Cuál de las siguientes alternativas detallan las venas que confluyen y forman la vena señalada?
a. mesentérica superior, gástrica izquierda y
gastroepiploica izquierda
b. mesentérica inferior, gástrica izquierda y renal
c. esplénica, mesentérica superior y mesentérica inferior
d. esplénica, pancreatoduodenal y omental izquierda
e. gástrica izquierda, esplénica y hepática común

23. ¿Cuál de las siguientes sustancias forma parte de la secreción biliar?


a. Tripsina
b. Lecitina
c. Elastasa
d. Quimotripsina
e. Pepsina
24. El tubo digestivo contiene diferentes tipos de epitelios y glándulas. La estructura señalada es unay
está localizada en el …………...
a. glándula de Brunner / intestino
grueso b.cripta de Lieberkuhn /
colon
c. cripta de Lieberkuhn / intestino delgado
d. glándula oxintica / estomago
e. célula parietal / estómago

Aunque las criptas de


Lieberkuhn están presentes en el
intestino delgado, la
microfotografía es de epitelio de
colon.

25. ¿De qué musculo forma parte el ligamento inguinal?


a. Oblicuo externo del abdomen
b. Oblicuo interno del abdomen
c. Transverso del abdomen
d. Psoas
e. Dorsal ancho

26. Señale cuál de las siguientes afirmaciones NO se relaciona a la siguiente glándula anexa del tubo
digestivo mostrada en la imagen:
a. Es una glándula exocrina compuesta exclusivamente por acinos serosos
b. Su inervación está dada por el nervio auricular mayor (ramo posterior C2), que inerva la
vaina de la glándula así como la piel por encima de esta.
c. Esta glándula produce una secreción mucinosa acuosa, llamada mucoserosa, a través del
conducto de Wharton.
d. Su inflamación puede ser causada por un virus de los Paramyxoviridae, que provocan una
enfermedad muy frecuentemente en niños y adolescentes
e. Es una glándula endocrina y probablemente sea de origen pancreático

Se valida la opción e debido a que no está


relacionada con la imagen.

27. ¿Cuál de las siguientes enzimas está localizada en el borde en cepillo y juega un rol en la digestión de
proteínas?
a. Alfa dextrinasa
b. Pepsina
c. Enterocinasa
d. Lactasa
e. Carboxipeptidasa A.

Se valida la opción c debido a que es correcta en relación a la pregunta.


28. Una de los siguientes sustancias, NO sirve como un buen agente emulsificante:
a. Colesterol
b. Ácidos grasos
c. Sales biliares
d. Lecitina
e. Proteínas de la dieta

Se valida la opción e debido a que es correcta en relación a la pregunta.

29. La sustancia que estimula el crecimiento de la mucosa gástrica es:


a. Secretina
b. Motilina
c. Péptido estimulante de la mucosa gástrica
d. Gastrina
e. Histamina

30. ¿Cuál de las siguientes alternativas es una función de la colecistokinina?


a. Relajación de la vesícula para la salida de
bilis b.Secreción de ácidos biliares
c. Contracción del esfinter de Oddi
d. Secreción de enzimas pancreáticas
e. Contracción del duodeno

Se valida la opción b debido al efecto de la CCK sobre la vesicula biliar.

31. Con respecto a la anatomía del tronco celiaco, señale lo correcto


a. El tronco celiaco se origina de la cara posterior de la aorta abdominal
b. Es una arteria delgada que tiene un calibre entre 2 y 3 mm
c. Una de sus ramas es la arteria gástrica derecha
d. La hepática común que es una de sus ramas, participa en la irrigación del estómago.

32. Con respecto a la anatomía del duodeno, marque la respuesta correcta:


a. Tiene una distribución en forma de “C”, que rodea la cola del páncreas
b. La 3ra porción duodenal está contenida en la pinza vascular aortomesentérica
c. Entre la 1ra y 2da porción se forma un ángulo, conocido como el ángulo de Treitz
d. La 4ta porción se dirige a la izquierda, hacia abajo y hacia atrás.
e. En la tercera porción desemboca el conducto colédoco.

33. El hígado está ampliamente tapizado por peritoneo, la estructura que conecta la cara diafragmática del
hígado precisamente con el diafragma es el ligamento:
a. teres
b. falcifor
me
c.triang
ular
d.hepático
común
e.coronario

Se validan la opción c y e debido a que forman parte de los ligamentos que fijan el hígado al diafragma.

34. En el íleon se absorbe aproximadamente el 95% dea través de la circulación enterohepática.


a. agua
b. colesterol
c. sales biliares
d. hidróxicobalamina
e. factor intrínseca
35. Laestimula el mecanismo paracrino de la secreción de ácido clorhídrico.
a. histamina
b. acetilcolina
c. gastrina
d. secretina
e. somatostatina

36. En la digestión de proteinas,es el principal estímulo para convertir el pepsinógeno en pepsina.


a. la gastrina
b. el pH ácido
c. la acetilcolina
d. la ptialina
e. la somatostatina

37. Con respecto a la somatostatina, marque lo correcto:


a. Es secretada por las células S del intestino
b. Induce a la producción de VIP
c. Interviene en la fase intestinal de la secreción gástrica
d. Produce acetilcolina para estimular a la célula parietal
e. No interviene en la regulación de la secreción de ácido clorhídico

38. En pecten anal, es una estructura comprendida entre:


a. la línea pectínea y los senos anales
b. la línea blanca y la apertura anal
c. el esfínter anal interno y el externo
d. la línea anocutánea y la línea pectínea
e. la línea blanca y columnas anales

39. ¿Cuál de las siguientes alternativas es una proenzima pancreática?


a. Tripsina
b. Elastasa
c. Quimotripsinógeno
d. Amilasa
e. Procarboxipepitidasa C.

40. En la segmentación hepática de Coinaud, el segmento hepático señalado con la flecha,


corresponde a : En la segmentación hepática de Coinaud, la flecha señala el segmentohepático.
a. IV
b. V
c. VI
d. VII
e. VIII
SISTEMA DIGESTIVO (ME 154)
CLAVES EXAMEN FINAL
Ciclo 201900

1. Un niño de 2 años es llevado a la consulta por diarrea persistente, edema de las extremidades y falta de crecimiento
en relación a su edad. Los análisis de sangre revelan que tiene concentración plasmática baja de proteínas
(hipoproteinemia). Como parte del estudio se coloca colecistokinina (CCK) endovenosa y se recoge muestras del
líquido duodenal por endoscopía; el resultado del líquido confirma incapacidad para hidrolizar proteínas a un pH
neutro, esta situación mejora al añadir una pequeña cantidad de tripsina. El paciente probablemente esté sufriendo
la falta congénita de ………….
(Unidad 4, sesión 26, logro 2: Explicar la Digestión y absorción de las proteínas y sus alteraciones)
a. PEPT-1
b. pepsinógeno
c. enterocinasa
d. carboxipeptidasas

2. Paciente mujer de 35 años acude a consulta por sensación de sequedad y lesiones en cavidad oral. Al examen se
observa atrofia de la mucosa, fisuras y úlceras; nota además sequedad e irritación de la córnea y aumento del
tamaño de las glándulas parotídas. Su diagnóstico más probable es artritis reumatoide; el hallazgo más probable
en una biopsia de glándula parótida es……..….
(Unidad 3, sesión 18, logro 3: Describe las enfermedades más frecuentes de las glándulas salivales)
a. Presencia de acinos normales con hiperplasia de células ductales
b. Gran infiltración de linfocitos y células plasmáticas
c. Hiperplasia de acinos glandulares serosos
d. Gran infiltrado de linfocitos y neutrófilos

3. Un hombre de 42 años de edad se presenta al médico con una historia de 1 año de evolución, caracterizado por
dolor abdominal bajo y diarreas con crisis sanguinolentas. Manifiesta además pérdida de peso de 8 kg durante
este periodo. La colonoscopía revela lesión difusa en el colon con afectación del recto. La biopsia de estas lesiones
revela adelgazamiento de la pared, inflamación y ulceración de la mucosa y sub mucosa. El diagnóstico más
probable en este caso es:
(Unidad 4, sesión 28, logro 5: Describe la Enfermedad inflamatoria intestinal. Generalidades, morfología y
características)
a. sindrome de colon irritable
b. enfermedad de Crohn
c. colitis ulcerativa
d. sprue celiaco

4. Dos estudiantes deciden tomar un receso para comer una hamburguesa a la hora del almuerzo. Antes de llegar a
la cafetería, impulsos nerviosos provenientes del complejo vagal dorsal iniciarán la secreción de ácido gástrico por
la liberación de …………………….. desde el sistema nervioso entérico.
(Unidad 3, sesión 20, logro 2: Regulación de la secreción gástrica: estimulación, fases de la secreción)
a. Serotonina
b. Colecistoquinina
c. Péptido inhibidor vaso activo
d. GRP (péptido liberador de gastrina)

5. Un niño de cuatro años de edad es llevado a la consulta por cuadros diarreicos frecuentes caracterizados por
heces pálidas, voluminosas y fétidas; al examen físico presenta bajo peso y talla para la edad. Se mide la
concentración de cloruro en el sudor y se encuentra que sus valores son muy elevados. La alteración más
importante a nivel de células ductales del páncreas tiene relación directa con la conductancia de…………
(Unidad 3, sesión 23, logro 5 Explica la Secreción pancreática: formación del jugo pancreático, influencia de la
velocidad de flujo y regulación)
a. Bicarbonato
b. Potasio
c. Sodio
d. Cloro
6. Se evalúa los valores séricos de las siguientes sustancias a un paciente con enfermedad hepática terminal; en este
paciente se espera encontrar la combinación con la letra …………
(Unidad 3, sesión 22, logro 5: Describe las Pruebas de función hepática, la Insuficiencia hepática, encefalopatía
hepática e hipertensión portal)
Glucosa Amoniaco Albúmina

a. Aumentada Disminuida Disminuida

b. Disminuida Aumentada Aumentada

c. Aumentada Aumentada Aumentada

d. Disminuida Aumentada Disminuida

7. Una mujer de 35 años de edad HIV positiva, se presenta al médico con dolor abdominal en cuadrante superior
derecho e ictericia. La paciente refiere haber tenido múltiples episodios de ictericia durante los últimos 10 años.
Los exámenes para determinar hepatitis viral, dieron positivos para Hepatitis B, siendo catalogado el caso como
hepatitis crónica con alteración funcional. En un examen de sangre ¿cuál de los siguientes parámetros está
disminuido?
(unidad 3, sesión 22, logro 5: Pruebas de función hepática, Insuficiencia hepática, encefalopatía hepática e
hipertensión portal)
a. Albumina
b. Bilirrubina
c. Fosfatasa alcalina
d. Tiempo de protrombina

8. En el reflejo peristáltico del intestino delgado ¿Cuál de los siguientes eventos sucede en la porción caudal del bolo
alimenticio?
(Unidad 2, sesión 13, logro 4: Explicar la Motilidad del intestino delgado: Contracciones segmentarias y
peristálticas)
a. Acción del péptido inhibidor vasoactivo (VIP) en el músculo circular
b. Acción del NO (óxido nítrico) en el músculo longitudinal
c. Contracción del músculo longitudinal interno
d. Acción de acetilcolina en el músculo circular

9. Un varón de 58 años de edad con enfermedad de Crohn severo fue sometido a una resección ileal. Después de la
cirugía este paciente padecerá de esteatorrea, esto se explica porque …..………..
(unidad 4, sesión 26, logro 4: Explica las alteraciones en la Absorción de lípidos)
a. se inhibe la acción de la 7 alfa hidroxilasa
b. el pool de ácidos biliares se incrementa
c. hay mala absorción de ácidos biliares
d. el páncreas no secreta lipasa

10. En un experimento se inserta un balón en el estómago de un voluntario, se infla poco a poco mientras que se
vigilan las presiones intraluminales. Aunque el volumen del balón aumenta considerablemente, las presiones
permanecen constantes. Esta relación volumen-presión se explica por la liberación local de …………..
(Unidad 2, sesión 13, logro 1 Explica la Motilidad gástrica: relajación receptiva)
a. acetil colina y gastrina
b. norepinefrina y óxido nítrico
c. colecistoquinina y óxido nítrico
d. óxido nítrico y péptido inhibidor vasoactivo

11. ¿Cuál de las siguientes alternativas es una característica de la secreción exocrina del páncreas?
(Unidad 3, sesión 23, logro 5: Secreción pancreática: formación del jugo pancreático, influencia de la velocidad de
flujo y regulación)
a. Es hipotónica respecto al plasma
b. Su mayor estímulo se da en la fase intestinal
c. Es estimulada por la presencia de bicarbonato en el duodeno
d. La secreción enzimática es estimulada principalmente por la secretina
12. Las estructuras en el hígado que permite que los productos metabólicos unidos a proteínas tengan acceso a las
membranas basolaterales de los hepatocitos, son…..
(Unidad 3, sesión 21, logro 4-5: Explica la Organización micro estructural del hígado)
a. los canalículos
b. las células de Ito
c. las fenestras sinusoidales
d. las uniones intercelulares herméticas

13. La composición de la bilis es modificada conforme fluye por los conductillos biliares. Durante este tránsito se
espera que aumente la concentración de…….
(Unidad 3, sesión 22, logro 2: Describe la Secreción biliar, visión general del sistema biliar extrahepático y
composición de la bilis)
a. Ig A
b. Glucosa
c. Protones
d. Vitamina A

14. Se mide experimentalmente el contenido gástrico de dos personas. La persona “A” tiene alto contenido de grasa y
la persona “B” tiene un contenido isotónico ¿Cuál de las siguientes es correcto respecto al vaciamiento gástrico?
(Unidad 2, sesión 13, logro 2: Describe la Motilidad y vaciamiento gástrico)
a. Hay ralentización del vaciado gástrico solo en “A”
b. El vaciamiento gástrico es más rápido en ambos
c. Hay ralentización del vaciado gástrico solo en “B”
d. Hay ralentización del vaciado gástrico en ambos casos

15. El examen endoscópico de un paciente con hipertensión portal grave revela venas tortuosas que sobresalen hacia
la luz del esófago. El paciente recibe tratamiento quirúrgico mediante la colocación de una derivación que
conecta la vena porta a la vena cava. Después de la operación el riesgo de encefalopatía ………………….. y el riesgo
de sangrado de várices ……………..
(Unidad 3, sesión 22, logro 5: Describe la Insuficiencia hepática, encefalopatía hepática e hipertensión portal)
a. disminuirá / disminuirá
b. disminuirá / aumentará
c. aumentará / disminuirá
d. aumentará / aumentará

16. Un paciente varón de 18 años de edad acude al médico para sus exámenes de rutina. Sus resultados de
laboratorio muestran un valor de bilirrubina sérica de 4 mg/dl y una bilirrubina directa de 0,3 mg/dl. Las pruebas
de función hepática son normales. La alteración que explica mejor este caso es por la deficiencia de ………………..
(Unidad 3, sesión 22, logro 3: Explica la Producción y excreción de bilirrubina. Tipos de bilirrubina e ictericia)
a. transaminasas
b. hemo oxigenasa
c. la 7 alfa hidroxilasa
d. glucuronil transferasa

17. Un hombre de 57 años de edad es llevado a urgencias con hematemesis masiva rojo brillante, a su llegada se halla
inconsciente con PA: 80/40 mm Hg y FC: 124 lat/min. Luce ictérico con presencia de “arañas vasculares en el tórax
anterior y extremidades”, abdomen distendido con signo de oleada positiva. Se encuentra esplenomegalia y
pérdida de la masa muscular en extremidades. La anastomosis vascular responsable del sangrado en este
paciente es ………….…..
(Unidad 3, sesión 21, logro 2: Describe las anastomosis porto sistémicas)
a. vena gástrica izquierda y vena ácigos
b. arteria gástrica izquierda y vena ácigos
c. vena paraumbilical y vena epigástrica inferior
d. vena gástrica izquierda y vena esofágica superior
18. Un estudiante de medicina está comiendo un plato de comida a base de champiñones, espárrago y salsa de soya.
El estímulo del sabor umami contenido en todos estos alimentos viaja a través del nervio………………..
(Unidad 2, sesión 10, logro 3: Describe la irrigación e inervación de la lengua)
a. Lingual
b. Hipogloso
c. Glosofaringeo
d. Cuerda del tímpano
19. Una paciente de 30 años de edad es sometida a una cirugía en oído medio derecho por un problema de
otoesclerosis. Luego de la cirugía refiere alteración sensitiva de la lengua. Al evaluar el caso usted esperaría
encontrar……….
(Unidad 2, sesión 10, logro 5: Describe la Irrigación e inervación de la lengua)
a. Alteración en la sensación del dolor y temperatura en el tercio posterior de la lengua
b. Alteración en la sensación del dolor en los dos tercios anteriores de la lengua
c. Alteración en la sensación del gusto en el tercio posterior de la lengua
d. Sensación del dolor, tacto y temperatura conservadas

20. En un paciente de 45 años de edad con colestasis biliar, se encuentra una elevación de los niveles sanguíneos de
fosfatasa alcalina hasta 3 veces la cifra normal. ¿Cuál de las siguientes alternativas estará también elevada como
evidencia del daño de la vía biliar?
(Unidad 3, sesión 22, logro 5: Pruebas de función hepática, Insuficiencia hepática, encefalopatía hepática e
hipertensión portal)
a. Tiempo de protrombina y albúmina sérica
b. Transaminasas hepáticas (ALT y AST)
c. Gamma glutamil transpeptidasa
d. Glucoronil transferasa

21. Experimentalmente se incrementa la velocidad de la secreción salival con una sustancia, en el análisis de la
composición de esta saliva obtenida se espera encontrar…………..
(Unidad 3, sesión 17, logro 5 : Explica la Influencia de la velocidad del flujo salival en la composición de la saliva)
a. disminución de la concentración de bicarbonato que supera la concentración plasmática
b. aumento de la concentración de cloro y sodio que supera la concentración plasmática
c. aumento de la concentración de bicarbonato que supera la concentración plasmática
d. disminución de concentración de potasio y bicarbonato

22. Lactante de 3 meses de vida es atendido por presentar diarrea, se administra una solución de glucosa y
electrólitos por vía oral. La proteína de membrana apical que explica la capacidad de esta solución para
proporcionar aporte de glucosa e hidratación es ………..
(Unidad 4, sesión 26, logro 1: Explica la Digestión y Absorción de los hidratos de carbono. Alteraciones)
a. CFTR
b. SGLT-1
c. GLUT-2
d. GLUT-5

23. Paciente ha sufrido herida de bala en el abdomen, se le ha tenido que extirpar el segmento medio y distal del
ileon. En este caso la síntesis hepática de sales biliares estará …..…..
(Unidad 3, sesión 22, logro 4: Explica la formación, función y Circulación entero hepática de lasa sales biliares)
a. Sin cambios en el ritmo de síntesis
b. Disminuida por inhibición de la enzima colesterol 7 alfa hidroxilasa
c. Incrementada por estímulo de la enzima colesterol 7 alfa hidroxilasa
d. Incrementada por inhibición de la enzima colesterol 7 alfa hidroxilasa

24. Un varón de 75 años ingresa al consultorio por presentar ictericia marcada de piel y las escleras. El estudio del
paciente mostró que presentaba un tumor que obstruía la totalidad del conducto hepático común. ¿Cuál de los
siguientes conductos se encontrará dilatado en este paciente?
(Unidad 3, sesión 21, logro 6: Describir el árbol biliar intrahepático)
a. de Wirsung
b. de Hering
c. colédoco
d. cístico
25. Correlaciones las dos columnas y marque la fórmula correcta:
(Unidad 4, sesión 28, logro 1: Diarrea: definición, mecanismos: osmótica, secretoria y exudativa)

1. Enfermedad Hirschsprung ( ) heces con moco y sangre


2. Diarrea osmótica ( ) intolerancia a lactosa
3. Diarrea secretoria ( ) aganglionosis congénita
4. Diarrea exudativa ( ) canales de Cl- en las células de la cripta

a.- 4231 b.- 1234 c.- 2143 d.- 4213

26. La vena umbilical obliterada del hígado después del nacimiento se transforma en el ligamento:
(Unidad 3, sesión 21, logro 1: Hígado: relación con la pared abdominal, caras, lóbulos, ligamentos , hilio hepático)
a. cruzado
b. redondo
c. coronario
d. falciforme

27. Llega a su guardia nocturna una madre que trae a su RN masculino de 2 semanas de vida con mal estado general y
sequedad de mucosas. Usted observa que lacta ávidamente, pero a las 2 horas presenta vómito postprandial no
bilioso en proyectil. Al realizar la historia clínica, descubre que el lactante recibió profilaxis con macrólidos para
tos ferina. Usted sospecha principalmente en:
(Unidad 1, sesión 4, logro 4: Desarrollo y anomalías del intestino anterior)
a. estenosis pilórica hipertrófica congénita
b. fistula traqueo esofágica
c. estenosis duodenal
d. atresia duodenal

28. En la regulación del apetito y la saciedad, la estimulación experimental crónica del núcleo ventro medial del
hipotálamo producirá:
(Unidad 1, sesión 3, logro 4: explica los mecanismos de control del apetito y saciedad)
a. afagia
b. obesidad
c. hiperfagia
d. activación de neuronas relacionadas a NPY

29. Paciente mujer de 25 años acude por dolor en fosa ilíaca derecha que empeora al toser o caminar, asociada a
náuseas y vómitos por lo cual acude a emergencia. Dos días después de realizarle una apendicectomía, la paciente
desarrolla fiebre alta (39 °C), está hipotensa y presenta dolor abdominal. La laparotomía exploratoria muestra un
gran volumen de sangre en la cavidad peritoneal por lesión de un vaso producida durante la apendicectomía.
¿Cuál de las siguientes arterias debe ligarse para detener la hemorragia?
(Unidad 4, sesión 27, logro 4: Irrigación arterial del colon, recto y conducto anal)
a. cólica derecha y arteria rectal superior.
b. ileocólica y arteria cólica media.
c. mesentérica superior.
d. ileocólica.

30. La onda peristáltica secundaria del esófago se caracteriza por ser originada ………
(unidad 2, sesión 12, logro 3: Motilidad esofágica: fases y características)
a. por el plexo de meissner del esófago
b. por el plexo mientérico del esófago
c. por el reflejo de la deglución
d. durante la masticación

31. ¿Cuál de los siguientes es una causa de ictericia con bilirrubina conjugada aumentada?
(Unidad 3, sesión 22, logro 3: Producción y excreción de bilirrubina. Tipos de bilirrubina, ictericia)
a. Ictericia del recién nacido
b. Obstrucción del colédoco
c. Anemia hemolítica
d. Gran hematoma
32. En relación a la absorción de nutrientes, la absorción de dipéptidos y tripéptidos a nivel de las células epiteliales
del intestino delgado, se da principalmente debido a:
(Unidad 4, sesión 26, logro 2: Digestión y absorción de las proteínas. Alteraciones)
a. el incremento de los canales de Cl- en la membrana apical
b. la gradiente de bicarbonato en la membrana basal
c. la gradiente de iones H+ en la membrana apical
d. la gradiente de Na+ en la membrana apical

33. Paciente de 20 años es traído a la emergencia por presentar diarreas desde hace 2 días. Familiar refiere que las
deposiciones son líquidas y abundantes, al examen luce deshidratado y se plantea que la diarrea es producida por
una toxina que estimula la transformación de ATP a AMPc con apertura de canales de Cl- y pérdida de agua. El
tipo de diarrea más probable es:
(Unidad 4, sesión 28, logro 1: Diarrea: definición , mecanismos: osmótica, secretoria y supurativa)
a. osmótica
b. exudativa
c. secretoria
d. por intolerancia a lactosa

34. Un niño fue operado por una obstrucción intestinal, observándose la presencia de divertículo de Meckel. Según lo
referido, marque lo correcto:
(Unidad 1, sesión 5, logro 2: identifica las anomalías del desarrollo del intestino medio: onfalocele y gastrosquisis
(diferencias), Divertículo de Meckel)
a. el 50% de la población lo presenta
b. se localiza en el íleon muy cerca al yeyuno
c. puede poseer tejido gástrico o pancreático
d. se produce por una mala rotación de los intestinos

35. Marque la alternativa correcta respecto a la


estructura marcada en el gráfico:
(Unidad 3, sesión 22, logro 2: Secreción biliar.
Visión general del sistema biliar extrahepático y
composición de la bilis)

a. Se halla a 2 centímetros debajo de la


papila duodenal mayor
b. Llega el conducto colédoco y pancreático
principal
c. Llega el conducto hepático común y
pancreático principal
d. Llega el conducto pancreático accesorio

36. ¿Cuál de las siguientes moléculas se encontrará aumentada en el citoplasma de las células parietales de un
paciente con sindrome de Zollinguer Ellison?
(Unidad 3, sesión 20, logro 4: Enfermedad ulcerosa péptica: úlcera gástrica, duodenal. síndrome de Zollinger –
Ellison)
a. Péptido liberador de gastrina (GRP)
b. Proteína G estimulante (GS)
c. Inositol Trifosfato (IP3)
d. AMP cíclico (AMPc)

37. Los fármacos inhibidores de la bomba de protones, actúan bloqueando la ………..……..


(Unidad 3, sesión 20, logro 3: Regulación de la secreción gástrica: inhibición, Secreción de pepsinógeno y factor
intrínseco)
a. anhidrasa carbónica
b. ATPasa H+/K+ en la membrana luminal
c. ATPasa H+/K+ en la membrana basolateral
d. ATPasa Na+/K+ en la membrana basolateral
38. Un paciente fue diagnosticado de gastritis autoinmune, ¿cuál de las siguientes alternativas es FALSA respecto a
esta enfermedad?
(Unidad 3, sesión 20, logro 5: Gastritis crónica: helicobacter pylori, autoinmune. Tipos de gastritis)
a. Afecta principalmente el fondo y cuerpo gástrico
b. Se produce hiperplasia de células G secundaria a la aclorhidria
c. El propio sistema inmune destruye principalmente las células parietales
d. Se produce atrofia de la mucosa, aclorhidria, hipergastrinemia y déficit de vitamina B6

39. Marque la correlación correcta:


(Unidad 3, sesión:18, logros:1 y 2: Describe las enfermedades inflamatorias/infecciosas y proliferativas de la
cavidad oral)

1. Herpes virus ( ) En relación al abuso de antibióticos


2. Candidiasis oral ( ) Lesiones vesiculares como racimo de uvas
3. Eritroplaquia ( ) Mega esófago
4. Enfermedad de Chagas ( ) Lesión pre cancerígena

a.- 2431 b.- 1234 c.- 4123 d.- 2143

40. En un paciente con insuficiencia renal crónica, el déficit en la absorción de calcio a nivel del enterocito se debe a
lo siguiente:
(Unidad 4, sesión 26, logro 6: Explica la Absorción de calcio y hierro)
a. No se convierte la 25 hidroxicolecalciferol a 1,25 dihidroxicolecalciferol
b. No se convierte la 1,25 dihidroxicolecalciferol a 25 hidroxicolecalciferol
c. Existe un descenso de la alfa 25 hidroxilasa renal
d. Se incrementa la producción de Calbindina
SISTEMA DIGESTIVO
EXAMEN PARCIAL
201802

Sección: Todas
Profesores: Choque Chávez, Fernando Diego; Damián Bastidas, Narda Lucía; Irribarren Gamarra, Maria Patricia;
Stapleton Valdivia, Mauricio Juan Jose.
Duración: 50 minutos.
Indicaciones:
- Lea atentamente cada pregunta antes de responder.
- Se prohíbe el uso del celular y cualquier dispositivo electrónico.
- Está prohibido intercambiar materiales.
- Coloque su código de alumno en la tarjeta de respuestas. Si su código contiene una letra reemplácela por un valor
numérico siguiendo la siguiente equivalencia: A=9, B=8, C=7, D=6, E=5.
- Traslade sus respuestas a la tarjeta, llenando los círculos de manera completa con lapicero negro o azul. Está
prohibido el llenado con lápiz, lapicero de otro color o con lapicero de tinta borrable.
- Sea cuidadoso en el llenado de la tarjeta de respuestas, pues solo esta tiene validez para la calificación.
- Al terminar su examen avise al docente a cargo, no se levante de su sitio; debe entregar la hoja de respuestas con
la carátula del examen, este cuadernillo de preguntas se lo llevará cada estudiante.

Chorrillos, octubre de 2018

1. En todo el sistema gastrointestinal se encuentra diferentes tipos de músculos ¿En cuales estructuras
encontraríamos músculo estriado?
A. Faringe, esfinter esofágico inferior, colon descendente.
B. Esfínter esofágico superior, laringe, esfínter anal externo.
C. Faringe, esfínter esofágico superior, esfínter anal externo.
D. Esfínter esofágico superior, esfínter esofágico inferior, colon.

2. Durante la fase …..……………….. el paladar blando es traccionado hacia arriba.


A. oral
B. gástrica
C. faríngea
D. esofágica

3. El tubo digestivo a nivel del ………………………….. está revestido internamente por epitelio ………….
A. Esófago / plano simple
B. Estómago / cilíndrico simple
C. Intestino grueso / plano estratificado
D. Intestino delgado / plano de transición

4. Señale la estructura del tubo digestivo que presenta tres capas musculares en su pared.
A. Esófago
B. Estómago
C. Intestino grueso
D. Intestino delgado

5. Paciente de 47 años con sobrepeso, acude a consulta por presentar dolor tipo cólico en el cuadrante superior
derecho del abdomen. El dolor aumenta luego de ingesta de comidas con abundante grasa. Esta situación se explica
por el aumento en la secreción de …………………………. que estimula la contracción de la vesícula biliar.
A. Gastrina
B. Colecistoquinina
C. Péptido inhibidor gástrico
D. Péptido intestinal vasoactivo
6. Si hay un aumenta de la hormona grelina a nivel del hipotálamo ¿Cuál es la consecuencia en el organismo?
A. Hambre
B. Saciedad
C. Aumento de somatostatina
D. Disminución del tránsito intestinal

7. Señale el péptido gastrointestinal que produce relajación del músculo liso gastro-intestinal.
A. Péptido intestinal vasoactivo (PIV)
B. Gastrina vasoactiva (GV)
C. Colecistoquinina (CCK).
D. Acetilcolina (Ach)

8. ¿Cuál de las siguientes alternativas inhibiría la relajación receptiva a nivel del estómago?
A. Histamina
B. Bloqueo del nervio vago (X)
C. Péptido inhibir gástrico (GIP)
D. Péptido intestinal vasoactivo (VIP)

9. Una mujer de 28 años, con diagnóstico de Diabetes Mellitus Tipo 1, acude por presentar desde hace 10 años
estreñimiento y distensión abdominal. Se realiza estudio y se determina que la paciente presenta un retraso del
vaciamiento gástrico debido a gastroparesia diabética. ¿Cuál de los siguientes hechos aumentaría el tiempo del
vaciamiento gástrico?
A. Aumento de gastrina
B. Estimulación parasimpática
C. Ácidos grasos en el duodeno
D. Quimo isotónico en el duodeno

10. Experimentalmente, se aplica un inhibidor selectivo del péptido intestinal vasoactivo (PIV) durante la contracción
peristáltica del intestino delgado. ¿Cuál es el efecto de este inhibidor en la motilidad del intestino delgado?
A. Parálisis del movimiento anterogrado
B. Disminución del tránsito intestinal
C. Aumento del tránsito intestinal
D. Movimiento retrogrado

11. ¿Cuál de los siguientes eventos ocurre durante la defecación?


A. Relajación del esfínter anal externo
B. Relajación del músculo liso del recto
C. Contracción del esfínter anal interno
D. Disminución de la presión intrabdominal

12. Señale el péptido que cumple la función de disminuir la ingesta de alimentos.


A. Neuropéptido Y
B. Ghrelina Gastrica
C. Proteína relacionada a agouti
D. Proopiomelanocortina (POMC)

13. Indique el lugar de secreción del péptido YY


A. Estómago
B. Duodeno
C. Yeyuno
D. Ileon

14. La grelina es secretada en el estómago y estimula a las neuronas del núcleo ………………….. para la estimulación de
la secreción de……………..
A. arqueado / neuropéptido Y.
B. arqueado / melanocortinas.
C. paraventricular / neuropéptido Y.
D. lateral del hipotálamo / hormonas orexigénicas.
15. Considerando el desarrollo embriológico del intestino medio. ¿Qué evento de gran importancia se produce en la
sexta semana?
A. Retracción de asas intestinales primitivas
B. Aparición del primordio hepático y pancreático
C. Inicio de la secreción de insulina por el páncreas
D. Salida temporal de asas intestinales a través de cordón umbilical

16. En el ……………………………. se presentan los movimiento de …………………….


A. esófago / retropropulsión y mezcla
B. estómago / segmentación y ondas lentas
C. intestino grueso / propulsión y ondas en espiga
D. intestino delgado / segmentación y peristaltismo

17. Marita sufre una parálisis del músculo masetero. ¿Qué limitaciones se producirán a nivel del movimiento de la
mandíbula?
A. Lateralización
B. Propulsión
C. Elevación
D. Ninguna

18. Señale el movimiento que se produce a nivel del ciego y colon proximal que tiene como finalidad favorecer la
absorción de agua y sales.
A. De masa
B. De mezcla
C. Propulsivo
D. Peristáltico

19. La enfermedad de Hirchsprung se caracteriza por una dilatación anormal de colon y disminución de los
movimientos propulsivos que traerá como principal consecuencia …………
A. diarrea acuosa.
B. estreñimiento crónico.
C. dilatación de la válvula ileocecal.
D. disminución de la flora intestinal.

20. ¿Cuál de los siguientes factores estimulan el vaciamiento gástrico?


A. Colecistoquinina (CCK)
B. Neuropéptido Y
C. Secretina
D. Gastrina

21. La secreción de ………………………. estimula la motilidad gástrica.


A. Colecistoquinina
B. Secretina
C. Gastrina
D. Motilina

22. La hormona ……………………….. es secretada por las células “I” del …………..
A. Colecistoquinina / duodeno y yeyuno
B. Gastrina / duodeno y yeyuno
C. Colecistoquinina / estómago
D. Gastrina / estómago
23. El reflejo enterogástrico aumenta ……………………… debido a la…………………..
A. el tono de la válvula ileocecal /distención del colon.
B. el tono del esfínter pilórico / distensión del duodeno.
C. la motilidad del intestino delgado / distensión del ciego.
D. la motilidad del intestino delgado / distensión del estómago.

24. ¿Cuál de las siguientes sustancias disminuye el vaciado gástrico?


A. Motilina.
B. Noradrenalina.
C. Colecistocinina.
D. Neuropéptico Y.

25. El plexo mientérico se encuentra ubicado en………………. y funciona controlando la ………………


A. submucosa / secreción
B. submucosa / motilidad
C. entre las capas musculares / motilidad
D. entre las capas musculares / secreción

26. ¿Cuál de las siguientes glándulas está conformada por glándulas serosas y secreta la mayor cantidad de amilasa
en la boca?
A. Linguales
B. Parótidas
C. Sublinguales
D. Submaxilares

27. ¿Cuál de los siguientes factores es más importante para percepción mediante las papilas gustativas?
A. Agua
B. Enzimas
C. Ion fierro
D. Bicarbonato

28. Paciente obeso, fumador, consumidor de alcohol que acude por presentar molestias como disfagia, ardor
retroesternal, alteración del sueño y sensación de boca amarga en las mañanas. ¿Cuál de las siguientes
recomendaciones podría darle a este paciente?
A. Una dieta rica en lípidos y grasas.
B. Elevar la cabecera de la cama al dormir.
C. Disminuir la frecuencia de comidas y aumentar el volumen.
D. Una dieta hiperproteica y líquida en las noches antes de dormir.

29. En la enfermedad por reflujo se encuentra alterada la función del esfínter …………………….
A. esofágico superior
B. esofágico inferior
C. de Oddi
D. pilórico

30. En la diarrea psicógena hay una estimulación del ……………….. que …………………….. la motilidad gastrointestinal.
A. Sistema nervioso parasimpático / aumenta
B. Sistema nervioso simpático / inhibe
C. Plexo submucoso / estimula
D. Plexo mientérico / inhibe
1)​La hernia fisiológica es la protrusión del intestino medio y posterior por fuera del abdomen,
teniendo como eje principal a: ​Arteria mesentérica superior
2)​marque la afirmación incorrecta: l​ os omentos principalmente irrigan órganos
intraabdominales
3)​Al momento de espirar profundamente, ocurre lo siguiente:​ aumento de la presión
intraabdominal
4)​Las glándulas submucosas se encuentran en mayor cantidad a nivel:​ proximal a la unión
esofagogástrica
5)​Tiene inervación somática: ​Peritoneo parietal
6)​La estrechez esofágica más proximal es:​ cricofaríngeo
7)​Es considerado un órgano retroperitoneal secundario: ​Páncreas
8)En relación a la formación de los mesenterios, marque la que no corresponde al grupo:
Omento menor
9)​El hígado es un gran órgano de la cavidad abdominal, cuyas células principales son los
hepatocitos, los cuales derivan embriológicamente de:​ Endodermo
10)​Marque el órgano que se encuentra fuera de la cavidad peritoneal:r​ecto
11) ​El sistema parasimpático utiliza casi siempre la misma sustancia neurócrina para transmitir
información. En el caso de la estimulación vagal de la secreción de gastrina se lleva a cabo a
través de:​ bombesina
12) ​Con sus conocimientos de fisiología digestiva, a un paciente con úlcera gástrica por exceso
de producción de ácido, usted le recomendaría que reduzca el consumo de: ​Aminoácidos
13)​Al disminuir el pH gástrico se eleva las concentraciones en sangre de:​Somatostatina
14) ​Paciente con terapia preventiva de infarto con ácido acetil salicílico (inhibidores de
prostaglandinas) 500 mg/24 h. Usted sabe que este paciente presenta un aumento de secreción
de:​ ​ácido clorhídrico
15)​En un paciente con falla en la fusión de los conductos de las yemas ventral y dorsal
del páncreas, usted esperaría encontrar: embriología: ​Drenaje de la mayor parte del
jugo pancreático en la papila menor
16)​En cuanto a la distribución del tubo digestivo en la cavidad abdominal, se puede afirmar que
los órganos intraabdominales están suspendidos por:​Mesenterio
17) ​Paciente adulto con reflujo gastroesofágico es más probable que presente:​ Descalcificación
del esmalte
18) ​f un bolo que produce dilatación de las paredes en el intestino delgado producirá en
respuesta un reflejo: L
​ ocal
19)​El hígado es un gran órgano de la cavidad abdominal, cuyas células principales son los
hepatocitos, los cuales derivan embriológicamente de:​Endodermo
20) ​Paciente con Covid-19 con sintomatología leve, es evaluado por médico general que le indica
observación en su domicilia, uso de ivermectina (antiparasitario) y azitromicina (macrólido). Un
efecto secundario esperable por el uso de esta última medicina es:​ ​Diarrea
21)​Al palpar una masa pétrea en hipocondrio derecho, es probable que se trate de un tumor
de:​vesícula biliar
22) ​Paciente con dolor crónico debido a carcinoma de hígado, se le receta Petidina (análogo de
morfina) para el manejo del dolor. En cuanto a la función digestiva, se espera que se presente:
Estreñimiento
23)​Malformación intestinal producida por la falta de retorno del intestino medio a la cavidad
abdominal:​Onfalocele
24)​El componente más importante del tejido más duro del cuerpo humano es:​hidroxiapatita
25)​En un paciente de 23 años de edad que sufre odontalgia debido a una caries, es verdadero
que el dolor se debe a la afectación de la siguiente parte del diente:​Dentina
26)​Paciente de 21 años en estado de ebriedad es traído a emergencia debido a intoxicación
alcohólica. Durante la observación, el familiar le proporcionó un sándwich sin permiso del
personal médico. Casi inmediatamente después de comerlo, el paciente presentó deposiciones
tipo diarrea. Este reflejo secundario a la ingesta de alimentos, involucra:​Ganglios simpáticos
27)​Al probar comida muy caliente, usted retirará la lengua debido al estímulo aferente del
nervio:​trigémino
28)​Los músculos genihioideo y milohioideo deprimen la mandíbula cuando: ​el hioides está fijo
29)​Paciente de 56 años con shock séptico, es internado en UCI donde le administran
noradrenalina para mantener la presión arterial adecuada. En este paciente, usted esperaría
encontrar el peristaltismo:​ disminuido
30)​Lactante de 6 meses de edad que recibe sus primeras comidas, es traído a consulta por su
madre quien preocupada consulta si es malo que el bebé haga deposiciones cada vez que come.
Usted tranquiliza a la madre pues sabe que esto es una consecuencia de los movimientos:
En masa
31)​La contracción intestinal es más prolongada que otras debido a la presencia de músculo liso
y en especial a la acción del:​calcio
32)L​a parte del diente que se parece más al tejido óseo, es producida por:​Cementoblastos
33)​Al haber una frecuencia de espigas muy alta, usted esperaría que se genere:​una contracción
tónica
34)​En los lactantes se evidencia un reflejo característico que es el de defecar inmediatamente
después de lactar o ingerir papillas. Este reflejo se inicia en el estómago y termina en el colon
donde produce: M
​ ovimientos en masa

35) ​Paciente acude a consulta por presentar una placa blanquecina en la mucosa oral, a nivel del
modiolo. Como antecedente refiere que siempre se muerde esa zona, sobre todo cuando está
nerviosa . El médico retira la lesión y la envía a patología donde le informan que la mucosa está
revestida de un epitelio: ​Escamoso estratificado queratinizado
36) ​La artritis reumatoide afecta a las articulaciones con sinovia. En la cavidad oral, la
articulación que se puede afectar es la: ​Temporomaxilar
37) ​Durante un viaje a la sierra de Perú, un estudiante de medicina percibe un zumbido
persistente y disminución de la audición. Por sus conocimientos sabe que se debe a una
diferencia de presiones entre el medio ambiente y el oído medio, por lo que procede a hacer
maniobras para contraer el músculo __________ con el fin de abrir el/la ___________: T
​ ensor del
velo del paladar / trompa de Eustaquio
38) ​Paciente con herida por arma blanca en hipogastrio superior a la sínfisis del pubis, acude a
emergencia. Usted sospecha que la lesión involucra: ​ íleon
39)​Las carnes tienen un sabor especialmente delicioso debido a que presentan en su
composición:​ G
​ lutamato
40) ​Un paciente con un tumor neuroendocrino productor de serotonina presentará: ​Diarrea
41)​Durante una cirugía, usted por error secciona los nervios vagos a nivel del diafragma. Por ello
esperaría que haya denervación a nivel de: e
​ stómago
42)​Los músculos que producen la oclusión de los dientes son inervados por ramas motoras del
nervio:​Trigémino
43)​Entre los factores predisponentes a la presencia de reflujo gastroesofágico está:​ Sobrepeso
44) ​El fenómeno de relajación receptiva del esfínter esofágico inferior se inicia cuando el bolo
atraviesa el: ​Esfínter esofágico superior
45)​El deglutir alimentos sin masticarlos lo suficiente puede producir:​Excoriaciones en la mucosa
gastrointestinal
46) ​La afectación del centro de la deglución en el tronco encefálico abolirá la aparición de:
Ondas primarias
47)​El fenómeno de relajación receptiva del esfínter esofágico inferior se inicia cuando el bolo
atraviesa el:​Esfínter esofágico superior
48) ​Durante la fase esofágica de la deglución, el movimiento peristáltico primario se inicia en:​ La
faringe
49)​Al ingresar líquidos o sólidos en la cavidad oral, un mecanismo que permite que una persona
respire mientras mastica es:​Depresión del paladar blando
50) ​La mayor fuerza de oclusión durante la masticación, sucede en la región de los dientes:
Molares
51)​La afectación del centro de la deglución en el tronco encefálico abolirá la aparición de:
Ondas primarias

52)​Al ingresar líquidos o sólidos en la cavidad oral, un mecanismo que permite que una persona
respire mientras mastica es: ​Depresión del paladar blando
53) ​Al realizar una endoscopia, usted encuentra un abombamiento anterior en tercio distal del
esófago, que tiene movimiento. Usted relacionaría este hallazgo con: ​aurícula izquierda

54)Una persona anciana con pérdida de casi todos los dientes, tendrá un deterioro en la
digestión de los alimentos, principalmente de: V
​ ERDURAS

55)​Al intubar a un paciente, por error se ingresa el tubo endotraqueal en el esófago y se insufla
el manguito endotraqueal (globo TET), la dilatación de este manguito dentro del esófago
generará: ​Múltiples ondas secundarias
56)​El anillo de contracción característico del movimiento peristáltico depende la liberación
de:​Sustancia P en la circular interna proximal al bolo
57)​Durante el desarrollo embrionario del páncreas, el segmento proximal del conducto
pancreático dorsal se puede convertirá en el conducto:​ Pancreático accesorio
58)​Respecto a la motilidad del colon, marque lo correcto: ​Las haustras del colon contribuyen a
las contracciones segmentarias
59) ​El paso de contenido del íleon al colon produce un reflejo que como respuesta produce una:
Contracción tónica
60)​Para aumentar la frecuencia de complejos mioeléctricos interdigestivos y así mejorar el
tránsito intestinal en pacientes con estreñimiento, se puede utilizar: E
​ ritromicina
61)​Paciente con dolor crónico debido a carcinoma de hígado, se le receta Petidina (análogo de
morfina) para el manejo del dolor. En cuanto a la función digestiva, se espera que se presente:
Estreñimiento
61)Al palpar una masa pétrea en hipocondrio derecho, es probable que se trate de un tumor de:
vesícula biliar
62)La estrechez esofágica más proximal es: ​cricofaríngeo
63)El íleon se encuentra principalmente en:​Cuadrante inferior derecho
64))Paciente con intoxicación por órganos fosforados, es traído a la emergencia donde se le
administra atropina hasta revertir los efectos de la intoxicación. Como efecto secundario del uso
de atropina se evidencia: D
​ isminución de los ruidos hidroaéreos
65) El Dr. Javier Okumura le realiza una endoscopía alta a un paciente de 25 años de edad, por
sospecha de gastritis, y observa una oclusión parcial de la luz de la segunda porción del
duodeno. La causa de este problema es una: ​migración anómala de la yema pancreática dorsal
66)La hernia fisiológica es la protrusión del intestino medio y posterior por fuera del abdomen,
​ rteria mesentérica superior
teniendo como eje principal a:​ A
67) En cuanto a la distribución del tubo digestivo en la cavidad abdominal, se puede afirmar que
los órganos intraabdominales están suspendidos por:​ Mesenterio
68)Tras la retracción de asas (habiendo girado el intestino en sentido horario), en la etapa
embriológica, usted esperaría encontrar: e
​ l colon transverso por delante del duodeno
69) En un estudio hecho en cobayos se encontró que la inyección endovenosa de PYY (que
fisiológicamente se libera en el íleon y colon) tendrá un efecto marcado sobre el hipotálamo
produciendo disminución de:​ período interprandial
70)Los dobleces de peritoneo que comunican un órgano con la superficie interna de la pared
abdominal posterior antiguamente se denominaban ligamentos , sin embargo, actualmente se
​ epliegues
denominan:​ R
71) Al tomar grandes cantidades de agua de golpe por sed intensa, se podría elevar el nivel
sérico de: ​gastrina
​ perturar la papila menor
72)En el páncreas divisum, usted trataría de:​ a
73) Malformación intestinal producida por la falta de retorno del intestino medio a la cavidad
abdominal: O
​ nfalocele
​ escalcificación
74)Paciente adulto con reflujo gastroesofágico es más probable que presente:​ D
del esmalte
75) E
​ n un paciente con falla en la fusión de los conductos de las yemas ventral y dorsal
del páncreas, usted esperaría encontrar: embriología ​Drenaje de la mayor parte del
jugo pancreático en la papila menor
76)Durante un viaje a la sierra de Perú, un estudiante de medicina percibe un zumbido
persistente y disminución de la audición. Por sus conocimientos sabe que se debe a una
diferencia de presiones entre el medio ambiente y el oído medio, por lo que procede a hacer
maniobras para contraer el músculo __________ con el fin de abrir el/la ___________: T
​ ensor del
velo del paladar / trompa de Eustaquio
77) El íleon se encuentra principalmente en: Cuadrante inferior derecho
​ mento menor
78) Del mesenterio ventral deriva la siguiente estructura: ​ o
79)La amígdala palatina se encuentra fácilmente detrás del músculo: P
​ alatogloso
80)La estructura que delimita la cavidad oral de la orofaringe es: ​Istmo de las fauces
81)Durante el desarrollo embrionario del páncreas, el segmento proximal del conducto
pancreático dorsal se puede convertirá en el conducto: P
​ ancreático accesorio
83)En un paciente con tumor carcinoide de páncreas productor de gastrina (Síndrome de
​ ecreción de secretina
Zollinger-Ellison) se puede esperar un aumento en la:​ S
84) Debido a su origen embriológico similar, las neoplasias de los acinos glandulares salivares, su
comportamiento y respuesta a tratamiento es similar a las de las neoplasias de: ​Mucosa del tubo
intestinal
85)Paciente con terapia preventiva de infarto con ácido acetil salicílico (inhibidores de
prostaglandinas) 500 mg/24 h. Usted sabe que este paciente presenta un aumento de secreción
de: ​ácido clorhídrico
86) Marque lo correcto en relación al control autónomo de la motilidad: l​ a muscularis mucosae
se contrae al recibir estimulación simpática
87)Paciente de 23 años con bulimia es traída a la emergencia deshidratada, semiconsciente y
con alcalosis metabólica. Los vómitos autoinfligidos por esta paciente se producen por
estimulación de receptores en la base de la lengua que mandan información directamente al:
núcleo del tracto solitario
88) Un tumor productor neuroendocrino productor de serotonina, se manifestará con: diarrea
89)Paciente con herida por arma blanca en hipogastrio superior a la sínfisis del pubis, acude a
emergencia. Usted sospecha que la lesión involucra: ​íleon
90) El componente más importante del tejido más duro del cuerpo humano es: h
​ idroxiapatita
91) E
​ l principal musculo encargado de la propulsión de la mandíbula es: ​pterigoideo
lateral
92) La parte del diente que se parece más al tejido óseo, es producida por: Cementoblastos
93) Al haber una frecuencia de espigas muy alta, usted esperaría que se genere: u
​ na contracción
tónica
94) Paciente de 56 años con shock séptico, es internado en UCI donde le administran
noradrenalina para mantener la presión arterial adecuada. En este paciente, usted esperaría
encontrar el peristaltismo: ​disminuido
95) Los músculos genihioideo y milohioideo deprimen la mandíbula cuando: el hioides esta fijo
96) Los dientes que contienen una raíz única son los siguientes, excepto: ​molares
97) Es falso que cuando una persona coloca en su boca una sustancia con alta concentración de
carbohidratos, suceda lo siguiente: s​ e dilata el esfínter esofágico inferior
98)Al realizar una vagotomía por ulcera péptica, usted esperaría una disminución de producción
de gastrina debido a: a
​ usencia de la bombesina vagal
99) Paciente consulta con usted por dolor epigástrico tipo “ardor” persistente, principalmente
cuando toma café o alcohol. Usted sospecharía:​ úlcera duodenal
100) respecto a los péptidos gastrointestinales, marque lo correcto​:​ la somatostatina actúa como
hormona y sustancia paracrina
101)Al haber obstrucción del píloro por cuerpo extraño, con una distensión abdominal
importante, usted esperaría encontrar: ​aumento de secreción de ácido clorhídrico
102)en un paciente postrado, con constipación crónica, usted evitaria el uso de: ​opioides
103) debido a la migración de las yemas pancreáticas, es posible que aparezca un tejido
pancreático accesorio. La localización más común de esta anomalía es: ​estomago
104)En un ensayo clínico se usa Zaldaride (antagonista de la calmodulina) para tratar osteopenia.
Basado en sus conocimientos de motilidad intestinal, usted sabe que, uno de los efectos
secundarios que se presenta es:​ estreñimiento
105)a un paciente de 34 años con colon irritable se le receta el antimuscarínico Hiosciamina
(hioscina) para tratar un síndrome de colon irritable. Uno de los efectos que se espera con esta
medicación directamente en la pared intestinal es:​ disminución del potencial de reposo de
membrana
106)Paciente con migraña va a ser sometido a terapia con propanolol (betabloqueador). Al
momento de darle consejería de los posibles efectos secundarios de esta medicación, usted
tiene claro que en la pared intestinal se va a presentar: a
​ umento transitorio del potencial de
membrana
107)Paciente con vejiga hiperactiva es medicado con Solifenacina( antagonista M3) por lo cual se
espera que presente: ​estreñimiento
108) Con respecto a los complejos mioeléctricos interdigestivos, marque lo correcto: L​a motilina
se secreta en el duodeno
109) Durante la masticación, es factible evitar que la comida se acumule en la cavidad vestibular
gracias al músculo: ​buccinador
110) Durante la evaluación de la faringe de un paciente, se le solicita que abra la boca y diga “A”.
Esto se puede realizar gracias a los músculos (marque la mejor respuesta): ​Genihioideo
111) La mucosa irrigada por la arteria palatina mayor esta cubierta por un epitelio: P
​ lano
escamoso queratinizado
112) La lesión del nervio lingual debido a un tumor de la base de la lengua, producirá una
pérdida de la: ​Sensibilidad y sensorialidad ipsilateral
113)Los corpúsculos gustativos se encuentran en la lengua, pero además se les puede encontrar
en : p
​ aladar blando
114) Al momento de tomar un café caliente, se ha producido una quemadura de primer grado
muy focal en la lengua, sin pérdida de la sensación del gusto. ¿Cual de las siguientes papilas son
las que estamos seguros que están afectadas? f​ iliforme
115)Durante el reflejo de la masticación, la presencia de un bolo alimenticio en la boca
desencadena el primer evento del reflejo que es: ​Inhibición de los músculos masticatorios
116)La fase voluntaria de la deglución depende de los músculos inervados por el nervio:​ vago
117) Paciente con polimiositis (miopatía inflamatoria que causa debilidad de los músculos
estriados) activa, que ingresa a hospitalización para tratamiento, usted asumirá que tiene
compromiso de la:​faringe
118) Un paciente no hace caso a las indicaciones de su médico, y toma una pastilla de Dorixina
(clonixilato de lisina) sin agua. Acude a emergencia porque siente dolor retroesternal y sensación
de que la pastilla “se ha estancado”. Es verdadero afirmar que el paciente presenta: m
​ últiples
ondas secundarias
119)Paciente con accidente cerebrovascular en tronco encefálico, que compromete el centro de
la deglución, es seguro colocar la sonda de alimentación en: e
​ sófago
120) Una complicación de la disfagia faríngea es la aparición de: Neumonía aspirativa
121)Paciente con acalasia leve, usted le recomendaría evitar el consumo de: A
​ nálogos de
acetilcolina
122)La aparición de un divertículo de Zenker, tiene que ver con un aumento en el tono muscular
del músculo: c​ ricofaríngeo
123) El gastroenterólogo evidencia durante una endoscopia, que hay un alfiler casi
completamente enterrado en la pared del esófago posterior a la cuarta estrechez. Su mayor
preocupación, es que probablemente se haya afectado el/la: S
​ erosa
124)
125)
126)
127)
128)
129)
130)
131)
Ecu 1
Caso 1
Lactante masculino de 5 semanas de edad, producto de un primer embarazo normal controlado.
Peso de nacimiento 3.120 g y talla de 51 cm. Lactancia materna satisfactoria, con buen incremento
ponderal. Sin antecedentes patológicos hasta 5 días atrás, cuando comienza con vómitos
postprandiales no biliosos, lácteos. Los síntomas aumentan en frecuencia y magnitud hasta hacerse
explosivos después de cada alimentación. No refiere fiebre, tos, diarrea ni lesiones cutáneas. Es
importante destacar que, pese a los vómitos, el niño conserva el apetito y llora de hambre. Al
examen físico presenta buen estado general. Abdomen blando, depresible e indoloro, asociado a
distensión del hemiabdomen superior. Sin signos de deshidratación. No se palpan masas
abdominales. Exámenes de laboratorio: hemograma normal. Signos inflamatorios de fase aguda
negativos. Alcalosis metabólica leve en sangre venosa.
Preguntas:
1. Debido al signo de la onda peristáltica que se evidencia en este paciente, se debe
asumir que como consecuencia hay un aumento en la liberación de ___________ en la
pared del estómago:
a) Secretina
b) Histamina
c) Gip
d) Adenosina
2. Al momento de realizar la pilorotomía (corte del músculo del píloro para descomprimir)
¿Qué hormona se comenzará a liberar rápidamente en respuesta a este tratamiento?
a) Secretina
b) Bombesina
c) Colecistoquinina
d) Gastrina
3. La oliva pilórica se logra palpar a nivel de:
a) Epigastrio parte superior
b) Hipocondrio derecho
c) Entre epigastrio y mesogastrio
d) L1
4. Producto de la hipertrofia pilórica uno esperaría que los niveles de Gastrina se
encuentren:
a) Elevado
b) Disminuido
c) Depende del alimento
d) Normales
5. Si el paciente tuviera 35 años, uno podría pensar como diagnóstico diferencial de la
hipertrofia de píloro, la presencia de un tumor neuroendocrino productor de hormona:
a) Gastrina
b) Bombesina
c) Colecistoquinina
d) Secretina
6. En este paciente se puede esperar una mayor liberación de:
a) Enzimas pancreáticas
b) Gip
c) Secretina
d) Insulina
7. En relación con la estructura afectada se encuentra:
a) Mesenterio propiamente dicho
b) Ligamento gastroesplénico
c) Omento mayor
d) Ligamento redondo
Caso 2
Estudiante de medicina de la UPC de 21 años sufre de gastritis aguda ocasionada por comer en
lugares poco higiénicos. Suele consumir caramelos ( chupar ) mientras está en clase hasta la tarde.
Toma gaseosas regularmente (carbohidratos 46%, sodio 53%). También toma regular cantidad de
leche (grasa 35%, lactosa 35%, proteínas 30%), pues le calma un poco el dolor el ardor que siente
por la gastritis. Incluso, cuando puede, se toma dos vasos de agua fría para calmar las molestias. Ha
decidido ir al médico para tratarse pues ya no soporta el dolor, el cual está seguro que los síntomas
se deben a una elevada producción de ácido clorhídrico en el estómago, y por ello le ha recetado
Ranitidina (antihistamínico), con lo que siente mejoría.
Preguntas:
1. Antes del uso de Ranitidina en este paciente, los valores de somatostatina en sangre
están:
a) Aumentados
b) Disminuidos
c) En valores normales
d) No hay somatostatina en sangre
2. En este paciente con gastritis aguda debida a una alta producción de ácido clorhídrico,
si se le hiciera un examen de sangre, se encontraría elevados los niveles de:
a) GIP
b) Sustancia P
c) Histamina
d) CCK
3. El consumo de leche produce directamente un aumento de los niveles séricos de la
hormona:
a) Gastrina
b) Leptina
c) Grelina
d) VIP
4. Entre las sustancias cerebrales que producen ansiedad esta la serotonina, la cual
también tiene acción;
a) Anoxigénica
b) Orexigenica
c) La acción de la serotonina en el apetito no está descrita
5. El uso de atropina en este paciente
a) Aumentará la producción de ácido clorhídrico
b) Aumentara el pH del estomago
c) Inhibirá la acción de las prostaglandinas
d) Disminuirá la acción del receptor CCK-B
6. El consumo rápido de 500 mL de gaseosa aumentara directamente la concentración
sérica de cual de las siguientes hormonas:
a) Cck
b) Secretina
c) Gastrina
d) Neuropéptido Y
7. El consumo de una pequeña cantidad de gaseosa aumentara directamente la
concentración sérica de cual de las siguientes hormonas
a) Péptido 1 similar al glucagón (GLP-1)
b) CCK
c) Secretina
d) Motilina
8. En este paciente con gastritis agua hiperclorhídrica, y debido a esta gran producción
de ácido, teóricamente podría tener como complicación:
a) Aumento del vaciamiento gástrico
b) Hipertrofia del píloro
c) Hemorroides
d) Ictericia
9. Este paciente tiene también la costumbre de tomar agua antes de sentarse a almorzar,
la cual hace que la producción de ácido clorhídrico del estomago
a) Aumente
b) Disminuya
c) Regrese a lo normal
d) No varié
10. El consumo de dos vasos de agua seguidos generara indirectamente un aumento en la
liberación de:
a) Péptido Y
b) Noradrenalina
c) Acido clorhídrico
d) Enzimas pancreáticas

ECU 2
Niño de sexo masculino de 2 años de edad, sufre de estreñimiento desde el nacimiento (1 deposición cada 3-4
días). Madre menciona que le estimula la defecación con un termómetro rectal, y continuo uso de enemas y
laxantes. Desde hace 6 meses comienza con vómitos postprandiales. Los síntomas aumentan en frecuencia y
magnitud y están en relación con los episodios de estreñimiento. No refiere fiebre, tos, diarrea ni lesiones
cutáneas. Al examen físico presenta regular estado general, luce deshidratado. Abdomen distendido, blando,
depresible e indoloro. No se palpan masas abdominales. Se permeabiliza el canal anal con termómetro rectal,
encontrando cierta resistencia. Salida de material fecal mal oliente en regular cantidad. Exámenes de
laboratorio: hemograma normal. Signos inflamatorios de fase aguda negativos. Alcalosis metabólica leve en
sangre venosa. Radiografía con enema baritado muestra recto y colon sigmoides dilatados (megacolon).
Biopsia profunda: ausencia de células ganglionares en la muestra enviada. Se realiza cirugía correctiva.

Preguntas:

1. En la zona afectada por esta enfermedad, se espera que las ondas lentas esten:
a) Disminuidas
b) Ocurran de manera normal
c) Abolidas
d) Aumentadas
2. Considerando la época que vivimos y como el paciente esta programado para sala de
operaciones, para lo cual se le toma una muestra de hisopado para descartar infección por
coronavirus. Al realizar el hisopado, el personal toca la pared de la orofaringe, la cual envía una
señal sensitiva a través del par craneal.
a) IX
b) X
c) V3
d) V2
3. No se espera que sea causa del vomito
a) Ayuno prolongado
b) Estimulación faríngea y del glosofaríngeo
c) Irritación de la mucosa gástrica
d) Dolor intenso
4. Con respecto a la defecación señale el enunciado correcto:
a) Es estimulado por llenado de la cuarta parte del volumen rectal
b) Es un reflejo netamente local
c) Es completamente voluntario y mediado por el nervio pudendo
d) La aferencia parasimpática es transmitida por vía del nervio vago
5. Al examinar la orofaringe del paciente, uno puede hallar fácilmente la amígdala
palatina, pues esta se encuentra inmediatamente detrás del musculo:
a) Palatogloso
b) Palatofaringeo
c) Hiogloso
d) Elevador del velo del paladar
6. En este paciente se considera que esta abolido el reflejo:
a) Gastrocolico
b) Relajación receptiva
c) Coloileal
d) Rectoesfintereano
7. Es un reflejo propio de la pared intestinal:
a) Peristaltismo
b) Doloroso
c) Defecación
d) Cólicoileal
8. Durante la fase esofágica de la deglución, para un bolo alimenticio determinado, a
medida que avanza el bolo la fuerza de la contracción se hace mas:
a) Débil
b) Dependiente de acetilcolina
c) Fuerte
d) Hiperpolarizado
9. El contenido fecal se detiene en la zona inmediatamente proximal a la zona donde hay
una menor presencia de:
a) Péptido intestinal vasoactivo
b) Neuropéptido Y
c) Enteroquinasa
d) Acetilcolina
10. La percepción de la pirosis (sensación de dolor o quemazón en el esófago) asociado a
reflujo gastroesofágico, puede aparecer o exacerbarse debido a:
a) Ejercicio
b) Bipedestación
c) Uso de antiácidos
d) Somatostina
11. En cuanto a los reflejos gastrocólico y gastroduodenal en este paciente, indique lo
correcto:
a) Se puede considerar reflejos vago – vágales
b) El control del nervio vago sobre el recto se ha abolido
c) El reflejo Gastrocolico es mas marcado en adultos que en niños
d) Se dan por nervios intrínsecos del sistema entérico
12. Cuando este paciente ingiera sus alumentos, se espera que al momento de pasar el
bolo aluimenticio por el esfínter esofágico superior, la presión intraesofagica
disminuya en:
a) El cadias
b) Tercio medio del esofago
c) El lugar donde se contraiga la muscular propia
d) Porcion proximal al bolo
13. Respecto a la motilidad del colon, marque lo correcto
a) Con la distencion del estomago, suelen aparecer movimientos en masa
b) Se producen contracciones segmentarias principalmente en el colon izquierdo
c) Los movimientos de masa se encargan del mezclado de las heces
d) Las haustras del colon contribuyen al reflejo de defecación
14. Durante la mezcla y digestion, la fuerza de la contracción puede aumentar gracias a la
acción de la:
a) Gastrina
b) Estimulacion simpática
c) Secretina
d) VIP
15. Para estimular la motilidad intestinal se podría usar un analago de:
a) Secretina
b) CCK
c) Glucagon
d) Noradrenalina
16. Debido al acumulo de material fecal en todo el marco colonico, y a la irritación química
asociada, el peristaltismo del ileon distal se debe encontrar:
a) Inhibido
b) Estimulado
c) No sufre alteraciones
d) Afectado por un reflejo vago – vagal
17. Durante la fase faríngea de la deglución se espera que ocurra el siguiente fenómeno:
a) El paladar blando cierra la entrada a la nasofaringe
b) Se libera acetilcolina en musculo cricofaringeo
c) La onda peristáltica primaria lleva el alimento hacia el esófago
d) Inicialmente se desplaza el hueso hioides hacia abajo
18. Considerando que este paciente esta sometido a estrés por el agravamiento de su
enfermedad, es posible afirmar que sus ondas lentas están:
a) Desmotivadas
b) Hipopolarizadas
c) Despolarizadas
d) Hiperpolarizadas

Ci 1
1. Durante una cirugía oncológica, ¿La extirpación de cual de los siguientes órganos se
vería comprometida por la presencia de adventicia?
a) Vesícula biliar
b) Estomago
c) Recto
d) Yeyuno
2. Maque la respuesta correcta en relación a la gastrina:
a) Las células G son las productoras y se encuentran principalmente en el antro gástrico
b) Las células G se encuentran principalmente en el fondo gástrico
c) Se estimula por la liberación de noradrenalina
d) Al distenderse el estómago, se inhibe su producción
3. En relación con los principios de motilidad, marque lo incorrecto
a) El musculo liso del tubo digestivo funciona como un sincilio
b) Es regulado por calcio
c) Las dos terceras partes proximales del esófago están conformadas por musculo
esquelético
d) Se produce mayor fuerza de contracción que en el musculo esquelético
4. Para poder morder una manzana, es necesario el siguiente musculo:
a) Milohioideo
b) Buccinador
c) Tensor del paladar
d) Orbicular de los labios
5. El consumir caramelos indirectamente activa la vía:
a) POMC/CART
b) AGRP/NPY
c) MCR-4
d) Grelina
6. ¿En que casos los vómitos son siembre biliosos?
a) Estenosis
b) Atresia esofágica
c) Atresia yeyunal
d) Atresia duodenal
7. Las arterias que derivan del tronco celiaco son, excepto:
a) Gástrica izquierda
b) Hepática derecha
c) Arteria esplénica
d) Hepática común
8. Entre las múltiples causas de la enfermedad por reflujo gastroesofágico, se puede
considerar también a una alteración en las ________ del esfínter esofágico inferior.
a) Contracciones tónicas
b) Ondas secundarias
c) Ondas lentas
d) Glándulas subesofagicas
9. Con respecto a las ondas lentas, maque la afirmación correcta:
a) Su frecuencia aumenta por acción de la acetilcolina
b) Su frecuencia disminuye por acción de la noradrenalina
c) Son potenciales de acción que producen la contracción del tracto gastrointestinal
d) Son contracciones rítmicas espontaneas
10. Producto de la alimentación, se producen diversas sustancias peptídicas, cininas y
bradicininas, las cuales permiten que:
a) El flujo sanguíneo intestinal aumente hasta 8 veces
b) La acción de la lipasa pancreática se vea incrementada
c) Se produzca neovascularización en los territorios de las arterias abdominales
d) El consumo de O2 del intestino aumente ligeramente
11. Cual de los siguientes péptidos inhibe el vaciamiento gástrico
a) Gastrina
b) Péptido inhibidor gástrico
c) CCK
d) Motilina
12. Sustancia que inhibe la secreción y la motilidad del estomago prolongando el tiempo
de digestión
a) GIP
b) Polipéptido pancreático
c) GLP-1
d) Enteroglucagón
13. En cuanto a los reflejos gastrointestinales, un reflejo que estimula el transito intestinal
es el reflejo:
a) Gastro cólico
b) Entero-gástrico
c) Vomito
d) Cólico-ileal
14. ¿Cuál de los siguientes líquidos corporales tiene el pH más alto?
a) Bilis de la vesícula biliar
b) Jugo pancreático
c) Saliva
d) Jugo gástrico
15. El crecimiento de un adenoma de cuello de páncreas puede comprometer la pared
gástrica por continuidad, ¿Qué parte del estómago estaría comprometida con mayor
probabilidad?
a) Pared anterior del píloro
b) Pared posterior del cuerpo
c) Pared anterior del cardias
d) Pared posterior del antro
16. Marque la respuesta correcta
a) La pared gástrica en el fondo es mas delgada que en el cuerpo y antro
b) Todos los órganos del sistema digestivo tienen capa serosa
c) El bronquio derecho constituye una de las estrecheces del esófago
d) El esfínter de Oddi rodea la papila menor duodenal
17. Un paciente con apendicitis agua, la sensación de dolor por esta inflamación es llevada
por el nervio:
a) Pélvico
b) Esplácnico menor
c) Vago
d) Esplácnico mayor
18. Estimulan la secreción gástrica
a) Proteínas
b) Soluciones inertes
c) Soluciones hipertónicas de glucosa
d) Grasas
19. El dolor periumbilical o epigástrico en el inicio de una apendicitis aguda se debe a
a) Estimulación del nervio vago
b) Irritación del peritoneo parietal
c) Estimulo del sistema simpático
d) Íleo secundario
20. El uso de ranitidina bloquea el receptor H2 de la histamina en las células parietales. La
histamina llega a estas células por
a) Vía hematógena
b) Difusión
c) Se produce en la misma célula parietal
d) La luz gástrica
21. El nacimiento de la arteria mesentérica superior se puede encontrar en cual de los
cuadrantes abdominales:
a) Mesogastrio
b) Hipocondrio derecho
c) Hipogastrio
d) Epigastrio
22. Con respecto las ondas lentas, una estimulación adrenérgica producirá:
a) Hiperpolarización
b) Mas espigas
c) Una contracción más sostenida
d) Despolarización
23. Al delgutir el bolo alimenticio, es lógico suponer que al pasar por el esofago haya un
mayor consumo de oxigfeno en la pared del tercio:
a) No hay diferencia
b) Medio
c) Proximal
d) Distal
24. La arteria mesentérica superior emerge de la aorta a nivel de
a) Cabeza de pancreas
b) Hilio hepático
c) Cardias
d) Tronco celiaco
25. La hernia fisiológica se produce dentro de:
a) Cordon umbilical
b) Saco amniotico
c) Saco vitelino
d) Alantoides
26. El ligamento falciforme divide al hígado en dos lobulos derecho e izquierdo.
Embriologicamente derivan de:
a) Mesentrio ventral
b) Fascia de Todd
c) Mesogastrio dorsal
d) Vena umbilical
27. Paciente de sexo masculino de 43 años que es alimentado por via intravenosa durante
varias semanas. Producto de este tipo de alimentación, se encuentra en la endoscopia
atrofia de la mucosa antral. La causa mas probable de esta alteración es debido a los
bajos niveles de:
a) Gastrina
b) Secretina
c) GIP
d) CCK
28. Aproximadamente en la semana 6 del desarrollo embrionario, el intestino intermedio
gira 90° herniándose a nivel del:
a) Cordón umbilical
b) Borde inferior del bazo
c) Borde superior del hígado
d) Borde superior del pubis
29. Al comer unas papitas fritas con mayonesa, el vaciamiento gástrico disminuye por
efecto directo de la hormona:
a) CCK
b) Somatostatina
c) Secretina
d) Motilina
30. El crecimiento de un adenocarcinoma de pancreas compromete la pared gástrica por
contigüidad. ¿Qué parte del estomago se esperaría este comprometido?
a) Pared posterior del antro
b) Pared posterior del fondo
c) Pared posterior del píloro
d) Pared posterior del cardias

PARCIAL DE SISTEMA DIGESTIVO 2019 - 00


1. Varón de 30 años es traído a emergencia por agresión abdominal con arma de fuego (pistola) y es
sometido a laparotomía exploratoria, observándose isquemia del colon ascendente y parte del
colon trasverso ¿la lesión de cuál de las siguientes arterias explicaría esta isquemia?
(unidad 1, sesión 2, logro 6: (Describe la irrigación visceral: arterias de tronco celiaco, arteria
mesentérica superior e inferior, topografía de superficie, órganos por cuadrante)
a) Celiaca
b) Colónica derecha
c) Mesentérica inferior
d) Mesentérica superior

2. Respecto a las sustancias gastrointestinales que regulan la secreción pancreática; marque la


afirmación correcta:
(unidad 1, sesión 3, logros 2 y 3: describir las hormonas gastrointestinales: estímulos y funciones)

a) La Secretina, es la hormona más importante para la secreción de bicarbonato por las células
acinares del páncreas
b) La acetilcolina es capaz de estimular la secreción enzimática y de bicarbonato del páncreas
c) La gastrina, es la hormona más importante para la secreción de enzimas pancreáticas
d) La colecistoquinina (CCK) estimula al páncreas solo para secreción enzimática

3. Ante una lesión del X par craneal, ¿cuál de los siguientes músculos mantiene conservada su
función?:
(unidad 2, sesión 08, logro 4: Paladar blando: componentes musculares)

a) Elevador del velo del paladar


b) Tensor del velo del paladar
c) Palatofaríngeo
d) Glosofaríngeo

4. Experimentalmente se utiliza atropina (anticolinérgico) para inhibir la secreción de gastrina, sin


embargo, la secreción de esta hormona se sigue dando ante estímulos vágales. Esta situación se
explica porque la atropina:
(unidad 1, sesión 3, logro 3: describir las hormonas gastrointestinales: estímulo y funciones de la gastrina
y colecistoquinina)

a. Bloquea parcialmente la bomba de protones en la célula G


b. Inhibe la acción de acetilcolina e histamina en la célula G
c. Solo inhibe la acción del péptido GRP en la célula G
d. No bloquea la acción del péptido GRP

5. Un varón de 50 años es sometido a extirpación del duodeno y parte proximal del yeyuno. Esta
situación ocasionaría la pérdida de las células ………. , productoras deque estimula la secreción de
bicarbonato por el páncreas.
(unidad 1, sesión 3, logro 3: describir las hormonas gastrointestianles: estímulos y funciones de la
secretina y péptido insulinotrópico dependiente de glucosa)

a) “S” / secretina
b) Parietales / secretina
c) “I” / colecistoquinina
d) “S” / colecistoquinina

6. Recién nacido que presenta tumoración abdominal a nivel del cordón umbilical (fotografía).
¿cuál de las siguientes afirmaciones es correcta respecto a este defecto en el desarrollo
embriológico del intestino?: (unidad 1, sesión 5, logro 2: identificar las anomalías del desarrollo
del intestino medio)
a) Corresponde a una Gastrosquisis
b) Las vísceras se hallan cubiertas por piel
c) No está asociado a otras malformaciones
d) Se asocia a malformaciones
cardiacas y del tubo neural
7. Varón de 35 años acude a la emergencia por trauma
abdominal y se decida realizar una laparoscopía exploratoria.
El cirujano observa la disposición de los órganos abdominales
como se representa en el siguiente esquema. Esta disposición
de órganos se explica por la rotación (SMA=arteria
mesentérica superior)
(unidad 1, sesión 5, logro 3: identificar las anomalías del desarrollo del
intestino medio: defectos de rotación, estenosis y atresias)
a) antihoraria del intestino medio, en sólo 90°
b) incompleta del intestino medio (270°)
c) horaria del intestino medio
d) horaria del estómago

8. Se evalúa la expresión de la proteína Agrp en una persona con alteración del apetito; lo correcto
respecto a esta proteína es…..
(unidad 1, sesión 3, logro 4: Explica los mecanismos de control del apetito y saciedad )

a) Esta proteína es un potente anorexigénico


b) La mutación del gen que la codifica produce adelgazamiento
c) La sobre producción de la proteína lleva a obesidad por agonismo de receptores MC3 y MC4
d) La sobre producción de la proteína disminuye el apetito por antagonismo de receptores MC4

9. Juana cae de la bicicleta y se fractura la región anterior del hueso maxilar superior con
compromiso de la fosa incisiva. Al examen físico de la región esperaría encontrar alteración en la sensibilidad
de la encía …………………
(unidad 2, sesión 8, logro5: paladar: paladar duro y blando: irrigación e inervación)

a) bucal posterior
b) Lingual anterior
c) palatina anterior
d) palatina posterior

10. Recién nacido es atendido por el neonatólogo y luego entregado a su madre para dar de lactar; la
madre al dar de lactar observa coloración azulada de labios, acompañado de tos persistente,
dificultad respiratoria y distención abdominal. Se le intenta colocar una sonda nasogástrica pero
esta retorna a la cavidad oral en todos los intentos. ¿Cuál de las siguientes anomalías del
desarrollo es el más probable en este caso? (unidad 1, sesión 4, logro 3: identificar las anomalías en
el desarrollo del esófago: atresia y/o fístula traqueo esofágica)

a) Estenosis esofágica proximal con Fístula traqueo esofágica distal


b) Atresia esofágica proximal con fístula traqueoesofágica distal
c) Atresia esofágica distal con fístula traqueoesofágica proximal
d) Fístula traqueoesofágica proximal y distal

11. ¿Cuál de los siguientes mecanismos ocurre durante la defecación?


(unidad 2, sesión 13, logro 6: motilidad del intestino grueso: contracciones segmentarias, movimientos
en masa, defecación y reflejo gastrocólico)

a) Contracción refleja del esfínter anal interno


b) En la posición de “cuclillas” el músculo puborectal se halla relajado
c) Relajación del esfínter anal externo por efectos del VIP y óxido nítrico
d) La materia fecal en el recto estimula la contracción del sigmoides por los nervios pudendos

12. La estructura número 4 (gráfico) corresponde a


……….… y está ………..
(unidad 2, sesión 9, logro 2: Partes de un diente.
Capas del diente: Esmalte: características y células
que lo producen)

a) el cemento / mineralizado en 90%


b) la dentina / formada por ameloblastos
c) el esmalte / formado por células
derivadas de la mesénquima
d) la dentina / formado por células
derivadas de la cresta neural

13. Un paciente luego de un accidente sufre lesión del piso de la boca, se constata daño del nervio “cuerda
del tímpano”, en este caso se esperaría encontrar disminución de la ....................de la lengua
(unidad 2, sesión 10, logro 3: Irrigación e inervación de la lengua)

a) Motilidad en los dos tercios anteriores


b) Sensación del gusto en el tercio posterior
c) Sensación del gusto en los dos tercios anteriores
d) Sensibilidad al tacto en los dos tercios anteriores

14. ¿Cuál de las siguientes afirmaciones es la correcta sobre la gastrina?


(unidad 1, sesión 3, logro 1: reconocer las características de las sustancias reguladoras
gastrointestinales: hormonas, sustancias paracrinas y neurocrinas)
a) Produce atrofia de la mucosa gástrica
b) Es producida por la célula G del cuerpo gástrico
c) Es estimulada por la distensión gástrica y el Ph bajo
d) Actúa en la célula diana mediante su receptor CCk tipo B
15. Al recibir un paciente con signos de hipovolemia y antecedente de trauma en abdomen por
accidente de tránsito, usted identifica radiológicamente: lesión de primera vértebra lumbar y
signos de lesión en páncreas; durante la cirugía se observó pobre irrigación de asas intestinales. El
vaso afectado es la arteria …………….
a) esplénica
b) hepática común
c) mesentérica inferior
d) mesentérica superior
16. Un paciente sufre de daño a nivel del cuello con lesión muscular en la región de la faringe. En el
examen físico se determina dificultad para la elevación de la faringe y para el cierre del itsmo de
las fauces. En este caso, probablemente esté afectado el músculo:
(unidad 2, sesión 11, logro 2: Músculos de la faringe: identificación, constrictores y longitudinales)
a) palatogloso
b) estilofarinfeo
c) palatofaringeo
d) constrictor inferior

17. Varón de 50 años a quien le realizan la curación de la segunda molar de la arcada superior
derecha. En un momento determinado, el paciente acusa de intenso dolor de la pieza dentaria
en tratamiento. La vía aferente del dolor viaja a través del nervio …………
(unidad 2, sesión 9, logro 6: Inervación de los dientes)
a) trigémino V2
b) trigémino V3
c) naso palatino
d) palatino menor

18. La distención gástrica por los alimentos produce incremento de secreción de HCl mediante la
producción de …………. que estimula a las células ……………. vía proteína ……….
(Unidad 1, sesión 3, logro 2: Describe las hormonas gastrointestinales: Estímulo y funciones de la
gastrina y colecistoquinina)
a) gastrina / parietal / Gq
b) gastrina / principal / Gs
c) acetilcolina / parietal /Gi
d) acetilcolina / principal / Gi

19. Un niño de tres años llega a emergencia con disfagia (dificultad para tragar), dolor retro esternal,
salivación y llanto. Se sospecha de ingesta de cuerpo extraño (moneda) en el esófago; al ser
evaluado se constata en una radiografía presencia de cuerpo extraño a nivel de C6 (6° vértebra
cervical). El cuerpo extraño estará suspendido a nivel del estrechamiento producido por………..
(unidad 2, sesión 11, logro4: Esófago, características anatómicas, relación con órganos vecinos y
estrecheces)
a) el cayado aórtico
b) el hiato esofágico
c) el músculo cricofaríngeo
d) el bronquio principal izquierdo

20. La triada portal (arteria hepática, vena portal y conducto biliar común) está contenida en el
ligamento
…….……… y derivan embriológicamente del ……
(Unidad 1, sesión 1, logro 4: Identifica el peritoneo, mesenterio, omento y ligamentos, retroperitoneo.)

a) hepato duodenal / mesenterio ventral


b) gastro esplénico / mesenterio dorsal
c) hepato gástrico / omento menor
d) falciforme / omento menor
21. En relación al movimiento de peristaltismo del
tubo digestivo: en la flecha negra del gráfico se
produce la liberación de ……………… a nivel del
músculo ………..
(unidad 2, sesión 7, logro 6: Control hormonal y tipos de
movimiento)
a) noradrenalina, sustancia P y neuropéptido “ Y” / circular
b) acetilcolina y sustancia P / longitudinal
c) óxido nítrico y PIV / longitudinal
d) óxido nítrico y PIV / circular
22. Un paciente refiere no percibir algunos sabores, al examen físico se constata alteración en la
percepción de sabores y del dolor en el tercio posterior de la lengua ¿Cuál de los siguientes
nervios estará alterada en su función?
(unidad 2, sesión 10, logro 5: Sabores, tipos y mecanismos moleculares para su detección)

a) Lingual (rama del V par)


b) Cuerda del tímpano (VII par)
c) Glosofaríngeo (IX par)
d) Hipogloso (XII par)

23. El gráfico detalla la estructura de la pared del


tubo digestivo intestinal ¿Cuál de las siguientes
asociaciones es correcta?
(unidad 2, sesión 7, logro 1: La pared y músculo
liso gastrointestinal )

a) “1” – peristaltismo
b) “2” – secreción enzimática
c) “3” – deriva del mesodermo
d) “4” – doble hoja de tejido graso
24. En el caso de un paciente con gastrinoma (tumor productor de gastrina), la presencia de úlceras
duodenales y erosión de la mucosa gástrica, se debe principalmente a…….
(unidad 1, sesión 3, logro 2: describir las hormonas gastrointestinales: estímulo y funciones de la gastrina
y colecistoquinina)

a) la acción directa de la gastrina sobre la célula principal


b) la sobre expresión de los receptores “G” en la célula parietal
c) el exceso de HCl por estímulo de receptores CCK-B en la célula parietal
d) el exceso de HCl por estímulo directo de receptores de acetilcolina en la célula parietal

25. El reflejo entero gástrico se caracteriza por:


(unidad 2, sesión 13, logro 6: Motilidad del intestino grueso: contracciones segmentarias, movimientos
en masa defecación y reflejo gastrocólico)

a) favorecer la motilidad gástrica gracias a la CCk


b) inhibir la motilidad gástrica y estimular la secreción ácida
c) movilizar grandes volúmenes desde el estómago al duodeno
d) originarse debido a la distensión duodenal y presencia del quimo ácido
26. Mauricio tiene dificultad para deprimir el paladar y elevar la parte posterior de la lengua. En este
caso estará afectado un músculo ………………., específicamente el músculo …………….
(Unidad 2, sesión 10, logro 2: Músculos de la lengua: clasificación, identificación y sus funciones)

a) intrínseco – longitudinal inferior


b) extrínseco – palatogloso
c) extrínseco – transverso
d) extrínseco – estilogloso

27. Una de las funciones del músculo señalado es:


(Unidad 2, sesión 8, logro 3: Describir el Piso de la boca:
estructuras blandas que la conforman)

a) deprimir la lengua
b) elevar el paladar blando
c) deprimir el hioides cuando la mandíbula está fija
d) deprimir la mandíbula cuando el hioides está fijo

28. Paciente varón de 30 años es evaluado por probable enfermedad de Chagas, cursa con
problemas de motilidad del colon; los estudios de biopsia determinan ausencia de células
ganglionares. Según el gráfico
¿cuál es la capa en la que se determina la ausencia de dichas células?
(unidad 1, sesión 2, logro 1: describir las generalidades de la estructura del tubo digestivo: esófago,
estómago intestino delgado y grueso)

a) Mucosa - 1
b) Muscular propia – 1
c) Muscular de la mucosa - 2
d) Muscular propia - 3
1 2

29. Paciente varón de 32 años, que acude a centro de salud por presentar de forma progresiva desde
hace 1 año dificultad para ingerir alimentos sólidos y luego líquidos; refiere regurgitaciones
alimentarias y marcada pérdida de peso (15 kilos). Radiografía baritada (sustancia de contraste)
de esófago se muestra en la figura. El presente caso se explica por……………….
(Unidad 2, sesión 12, logro 4: Identificar y describir la función de los esfínteres esofágicos)

a) aumento de la peristalsis esofágica


b) relajación incompleta del esfínter pilórico
c) relajación incompleta del esfínter esofágico inferior
d) perdida de producción de PIV y óxido nítrico en el
esfínter esofágico superior

30. En condiciones normales, el ingreso de 600 ml de líquido es el estómago provoca un aumento de


presión intragástrica de unos 12 cm de H2O. Después de una vagotomía (corte del nervio vago)
es de esperar que el ingreso del mismo volumen de líquido ocasione de la presión intragástrica.
(Unidad 2, sesión 13, logro 1: Describe la Motilidad gástrica: relajación receptiva)

a) la disminución
b) la no variación
c) un aumento mayor
d) un aumento similar o igual

Examen parcial 2019-2


1. La contracción del músculo ....................... permite la eliminación de gases (flatos) sin salida de material
fecal;
es el mismo músculo cuya relajación, sobretodo en cuclillas, permite el paso del contenido
fecal con menor esfuerzo durante la defecación.
a) Isquirectal
b) Puborrectal
c) Esfínter anal externo
d) Esfínter anal interno
2. Paciente mujer de 54 años se presenta con náuseas, vómitos, estreñimiento, y es diagnosticada
de abdomen agudo quirúrgico; en la cirugía encuentran un vólvulo de ciego. Esta anomalía
puede explicarse por:
a) Falta de rotación intestinal
b) Falta de fusión del mesenterio
c) Defecto en la formación de la cloaca
d) Falta de formación del omento mayor

3. Paciente mujer de 23 años con faringitis aguda, toma para el dolor una tableta de paracetamol
con un poco de agua. Durante la deglución, se relaja su esfínter esofágico inferior y el fondo del
estómago, mientras el bolo está aún en el esófago. ¿Qué sustancia provocara con mayor
probabilidad la relajación del esfínter esofágico inferior y el fondo del estómago en esta mujer?
a) Óxido nítrico
b) Sustancia P
c) Histamina
d) Motilina

4. Luego de tres horas dando exámenes, un alumno de medicina comienza a sentir hambre.
Esta situación es probable que sea mediada por la que
es sintetizada por el :
a) leptina / intestino
b) leptina / estómago
c) grelina / estómago
d) grelina / tejido adiposo

5. Varón de 72 años, con antecedente de diabetes mellitus tipo 2, que presenta enteropatía diabética
caracterizada por estreñimiento. Este problema puede estar asociado a:
a) deficiencia de óxido nítrico
b) aumento del reflejo gastrocólico
c) disminución de la secreción de colecistocinina (CCK)
d) aumento de la secreción del péptido intestinal vasoactivo (PIV)

6. Varón de 54 años con Diabetes Mellitus tipo 2, es diagnosticado de gastroparesia debido a que
presenta sensación de llenura precoz al comer, y reflujo gastroesofágico. Esta alteración en la
relajación receptiva y en el vaciamiento gástrico lo más probable es que se deba a una alteración
en:
a) el nervio vago
b) el ganglio celíaco
c) plexo submucoso
d) nervio hipogástrico
7. Varón de 67 años con tos y disminución de peso asociado a tabaquismo pesado, presenta
actualmente disfagia progresiva a alimentos sólidos. Se considera la presencia de un carcinoma de
bronquio izquierdo y por esta razón le realizan una endoscopía esofágica para descartar la
posibilidad de una compresión esofágica por el tumor. Se espera revisar el esófago en la
estrechez, que está a nivel de la vértebra
a) Tercera estrechez -T6
b) Segunda estrechez - C6
c) Segunda estrechez - T4
d) Tercera estrechez -T10

8. Varón de 34 años con dolor abdominal agudo en flanco derecho que se irradia a fosa ilíaca
derecha, es operado y se encuentra un divertículo intestinal inflamado, ubicado a 93 cm de la
válvula ileocecal. El origen de este divertículo es una falla en la obliteración de:
a) Conducto vitelino
b) Alantoides
c) Cloaca
d) Conducto anorectal
e) Uraco
9. En un niño menor de dos años con divertículo intestinal, este divertículo tiene su origen en una falla
en la obliteración de:
a) Conductoanorectal
b) Conducto vitelino
c) Alantoides
d) Cloaca
e) Uraco

10. Mujer de 43 años sufre un grave accidente de tránsito y está hospitalizada en coma, es alimentada
por vía intravenosa durante varias semanas. Producto de este tipo de alimentación, se encuentra en
la endoscopía atrofia de la mucosa gastrointestinal. La causa más probable de esta atrofia son los
bajos niveles séricos de la hormona:
a) Colecistocinina
b) Secretina
c) Gastrina
d) PIV

11. Una mujer de 30 años llega al consultorio porque se queja de dificultades para deglutir, la cual se
agravan cada vez más. Se realiza un estudio manométrico para examinar la generación de presión
a lo largo del esófago. Esta prueba revela que las contracciones como respuesta a la deglución
están mal sincronizadas y que la presión en el esfínter esofágico inferior permanece elevada. El
diagnóstico más probable es producido por niveles bajos de
a) acalasia / sustancia P
b) acalasia / óxido nítrico
c) enfermedad por reflujo gastrointestinal / acetilcolina
d) enfermedad por reflujo gastrointestinal / óxido nítrico

12. Paciente de 2 años, llega a emergencia por haber ingerido una moneda con la que estaba
jugando. El lugar más probable donde puede haberse quedado suspendido este objeto es a nivel
del estrechamiento producido a nivel del:
a) músculo milohiodeo
b) músculo aritenoideo
c) músculo cricofaríngeo
d) constrictor superior de la faringe

13. En una apendicectomía, al realizar la incisión de McBurney en la fosa iliaca derecha, es


necesario cortar los siguientes músculos, de afuera hacia adentro:
a) Recto – Oblicuo externo – Transverso
b) Recto – Oblicuo externo – Oblicuo interno
c) Oblicuo externo – Oblicuo interno – Recto
d) Oblicuo externo – Oblicuo interno – Transverso

14. Un varón de 90 años que se encuentra postrado en cama, es referido del asilo para endoscopia por
dificultad para deglutir luego de tomar un medicamento para aliviar el dolor la noche anterior. La
endoscopía revela que la píldora se alojó en el esófago y causó una reacción inflamatoria. Lo más
probable es que esto haya sido por la producción de múltiples ondas:
a) secundarias
b) primarias
c) lentas
d) segmentarias

15. Mujer de 23 años es diagnosticada de bulimia, al examen físico se observa ulceraciones en el


segundo y tercero dedo de la mano derecha. Esto se puede deber al uso continuo de estos dedos
para inducir el vómito, mediante la estimulación del par craneal:
a) V
b) IX
c) X
d) XI

16. Varón de 52 años se presenta por diarrea persistente de seis semanas de duración. En la
colonoscopia se observa un pólipo a nivel del íleon distal. El patólogo informa que se trata de un
tumor neuroendócrino, probablemente originado por las células enterocromafines del intestino.
La sustancia que más probablemente esté produciendo este tumor es:
a) Serotonina
b) Insulina
c) CCK
d) GIP

17. La fase oclusal de la masticación se realiza con la contracción de los músculos:


a) digástricos
b) masetero y temporal
c) orbicular y buccinador
d) pterigoideo lateral y digástrico

18. Al tomar su café en Starbucks, un estudiante de medicina sufre una quemadura de primer
grado en el tercio anterior de la superficie dorsal de la lengua. La información de dolor es
transmitida por el nervio:
a) cuerda del tímpano
b) glosofaríngeo
c) lingual
d) facial

19. Paciente es evaluado por faringitis aguda en consultorio externo. El médico de familia le solicita
que abra la boca y saque la lengua. Para realizar la acción de sacar la lengua, es necesario que se
contraiga el músculo:
a) Estilogloso
b) Geniogloso
c) Palatogloso
d) transverso de la lengua
20. Paciente con síndrome de Sjögren, presenta “boca seca”
(disminución de la producción de saliva) y caries dental,
asociada a la pérdida de la función de tampón de la saliva. Esta
desminerilización del diente puede comprometer a las
prolongaciones citoplasmáticas ubicadas en los tubos huecos de
la estructura señalada con la letra:
a) B
b) A
c) E
d) C

21. Mujer de 32 años acude a consulta por presentar disfagia de


progresión lenta, reflujo gastroesofágico y vómitos desde hace 3
meses de evolución progresiva. Se le realiza un estudio
radiológico con contraste en el que se observa estrechamiento
del esfínter esofágico inferior (imagen). Según sus conocimientos,
este paciente se beneficiaría con el uso de:
a) agonista beta adrenérgico
b) agonista alfa adrenérgico
c) análogo de óxido nítrico
d) análogo de Sustancia P
1. Un niño de 2 años es llevado a la consulta por diarrea persistente y edema de
las extremidades, además falta de crecimiento y desarrollo en relación a su
edad. Los análisis de sangre revelan que tiene concentración plasmática baja
de proteínas (hipoproteinemia). Durante la endoscopía duodenal, se coloca
colecistocinina (CCK) endovenosa y se recoge muestras del líquido duodenal;
el resultado del líquido confirma incapacidad para hidrolizar proteínas a un pH
neutro, esta situación mejora al añadir una pequeña cantidad de tripsina. El
paciente probablemente esté sufriendo la falta congénita de
- Enterocinasa

2. Experimentalmente se incrementa la velocidad de la secreción salival con


una sustancia, el análisis de la composición de esta saliva obtenida se
espera encontrar…………..
- Disminución de concentración de potasio
3. Paciente varón de 46 años soltero, consulta por odinofagia y bajo de peso,
tiene antecedente de tuberculosis desde hace 3 meses y es fumador crónico
(10 cigarrillos por día); al evaluar la cavidad oral se identifica lesión
blanquecina en el dorso de la lengua y paladar blando, las lesiones se
desprenden con el baja lengua dejando una base eritematosa. Esta lesión
corresponde probablemente a
……………………….…..
- Candidiasis oral
4. Paciente mujer de 35 años acude a consulta por sensación de sequedad y
lesiones en cavidad oral. Al examen se observa atrofia de la mucosa, fisuras y
úlceras; nota además sequedad e irritación de la córnea y aumento del tamaño
de las glándulas parotídeas. Su diagnóstico más probable es artritis
reumatoide; el hallazgo más probable en una biopsia de glándula parótida
es……..….
- Gran infiltración de linfocitos y células plasmáticas
5. Un paciente con anemia acude con su médico quejándose de episodios
frecuentes de gastroenteritis. Un análisis de sangre revela anticuerpos
circulantes dirigidos contra células parietales gástricas. Su anemia es
atribuible a la hiposecreción de
- Factor intrínseco
6. Dos estudiantes deciden tomar un receso para comer una hamburguesa a la
hora del almuerzo. Antes de llegar a la cafetería, impulsos nerviosos
provenientes del complejo vagal dorsal iniciarán la secreción de ácido
gástrico por la liberación de
…………………….. desde el sistema nervioso entérico.
- GRP
7. Un niño de cuatro años de edad es llevado a la consulta por cuadros diarreicos
frecuentes caracterizados por heces pálidas, voluminosas y fétidas, presenta
bajo peso y talla. Se mide la concentración de cloruro en el sudor y se
encuentra que sus valores son muy elevados. La alteración más importante a
nivel de células ductales del páncreas tiene relación directa con la conductancia
de…………
- Cloro
8. Una mujer de 50 años de edad que sufrió durante varios años resequedad
de los ojos debida a producción inadecuada de lágrimas es enviada con un
gastroenterólogo para evaluación de pirosis crónica. El examen endoscópico
revela erosiones y tejido cicatrizal en la parte distal del esófago justo por arriba
del esfínter esofágico inferior. Las lesiones pueden atribuirse a la disminución
de uno de los siguientes componentes salivales:
- Bicarbonato
9. Se evalúa los valores séricos de las siguientes sustancias a un paciente con
enfermedad hepática terminal; en este paciente se espera encontrar la
combinación con la letra …………
- disminuida, aumentada, disminuida
10. Una mujer de 35 años de edad HIV positiva, se presenta al médico con dolor
abdominal en cuadrante superior derecho e ictericia. La paciente refiere haber
tenido múltiples episodios de ictericia durante los últimos 10 años. Los
exámenes para determinar hepatitis viral, dieron positivos para Hepatitis B,
siendo catalogado el caso como hepatitis crónica con alteración funcional. En un
examen de sangre ¿cuál de los siguientes parámetros está disminuido?
- Albúmina
11. En el reflejo peristáltico del intestino delgado, uno de los siguientes eventos
sucede en la porción oral del bolo alimenticio…………...
- Acción de acetilcolina en el músculo circular
12. Experimentalmente se coloca una dosis alta de secretina en la luz
intestinal duodenal; como consecuencia de esto, en el jugo pancreático
de la misma luz intestinal se observa la disminución de la concentración
de …..………..
- Cl
13. Un varón de 58 años de edad con enfermedad de Crohn severo fue sometido a
una resección ileal. Después de la cirugía este paciente padecerá de
esteatorrea, esto se explica porque …..………..
- La micelas no pueden formarse
14. En un experimento se inserta un balón en el estómago de un voluntario, se infla
poco a poco mientras que se vigilan las presiones intraluminales. Aunque el
volumen del balón aumenta considerablemente, las presiones permanecen
constantes. Esta relación volumen-presión se explica por la liberación local de
…………..
- Óxido nítrico y péptido inhibidor vasoactivo
15. La toxina de Vibrio cholerae causa diarrea debido a…….
- El Incremento de la secreción de cloro por las células de la cripta intestinal
16. ¿Cuál de las siguientes alternativas es una característica de la secreción
exocrina del páncreas?
- Tiene una baja concentración de Cl- respecto al plasma

17. Una madre lleva a su hijo de dos años de edad a la sala de urgencias, estresada
porque el niño deglutió una moneda de 10 céntimos mientras la familia cenaba
en un restaurante. El médico observa mediante fluoroscopía que la moneda se
halla en el estómago y asegura a la madre que la moneda se eliminará con las
heces. El médico recomienda utilizar la respuesta fisiológica que permitirá la
evacuación de la moneda del estómago al intestino ………….…..
- Son los movimientos de mezcla y trituración
- . Es provocada por el ayuno
18. Las estructuras en el hígado que permite que los productos metabólicos unidos
a proteínas tengan acceso a las membranas basolaterales de los hepatocitos,
son…..
- Las fenestras sinusoidales
19. La composición de la bilis es modificada conforme fluye por los conductillos
biliares. Durante este tránsito se espera que aumente la concentración de…….
- Monómeros de ácido biliar
- Ig A
20. Se mide experimentalmente el contenido gástrico de dos personas. La persona
“A” tiene alto contenido de grasa y la persona “B” tiene un contenido hipertónico
¿Cuál de las siguientes es correcto respecto al vaciamiento gástrico?
- Hay ralentización del vaciado gástrico en ambos casos
21. El examen endoscópico de un paciente con hipertensión portal grave revela
venas tortuosas que sobresalen hacia la luz del esófago. El paciente recibe
tratamiento quirúrgico mediante la colocación de una derivación que conecta la
vena porta a la vena cava. Después de la operación el riesgo de
encefalopatía
y el
riesgo de sangrado de várices ……………..
- Aumentará/disminuirá
22. Un paciente varón de 18 años de edad acude al médico para sus exámenes
de rutina. Sus resultados de laboratorio muestran un valor de bilirrubina
sérica de 4 mg/dl y una bilirrubina directa de 0,3 mg/dl. Las pruebas de
función hepática son normales. La alteración que explica mejor este caso es
por la deficiencia de
………………..
- Glucuronil transferasa
23. Un hombre de 57 años de edad es llevado a urgencias con hematemesis
masiva rojo brillante, a su llegada se halla inconciente con PA: 80/40 mm Hg
y FC: 124 lat/min. Luce ictérico con presencia de “arañas vasculares en el
tórax anterior y extremidades”, abdomen distendido con signo de oleada
positiva. Se encuentra esplenomegalia y pérdida de la masa muscular en
extremidades. La anastomosis vascular responsable del sangrado en este
paciente es ………….…..
- Vena gástrica izquierda y vena ácigos
24. Un estudiante de medicina está comiendo un plato de comida a base de
champiñones, espárrago y salsa de soya. El sabor umami contenido en todos
estos alimentos actúa a nivel de los botones gustativos estimulando
………………..
- Un receptor acoplado a proteína G
25. Un hombre de 22 años de edad se presenta al médico con una historia de 1
año de evolución caracterizado por dolor recurrente en fosa iliaca derecha y
diarrea. Manifiesta además pérdida de peso de 8 kg durante este periodo. La
colonoscopía revela múltiples lesiones en el ileon terminal y colon. La biopsia
de estas lesiones revela engrosamiento, inflamación y ulceración de la
mucosa. El diagnóstico más probable en este caso es…….
- Enfermedad de Crohn
26. Varón de 61 años que consulta por dolor retro esternal intenso desde hace 6
horas y después de vómitos intensos y repetidos; al examen se observa disnea,
cianosis, hipotensión y signos clínicos de shock. La radiografía simple de tórax
muestra neumomediastino. El líquido en el espacio pleural aspirado tiene alta
concentración de amilasa. ¿Cuál de las siguientes alternativas puede explicar
este cuadro clínico
- Rotura espontánea de esófago
27. La secreción del ácido en la célula parietal gástrica se lleva a cabo por una
ATPasa especifica que intercambia hidrogeniones (H+) del citosol por…..
- K+
28. En condiciones normales el ingreso de 600 ml de líquido es el estómago
provoca un aumento de presión intragástrica de unos 12 cm de H2O. Después
de una vagotomía (corte del nervio vago) es de esperar que el ingreso del
mismo volumen de líquido provoque lo siguiente: …………………………………
- Un aumento mayor de la presión
29. Una paciente de 30 años de edad es sometida a una cirugía en oído medio
derecho por un problema de otoesclerosis. Luego de la cirugía refiere
alteración en la percepción de sabores. Al evaluar el caso usted esperaría
encontrar……….
- Alteración en la sensación del gusto en los dos tercios anteriores de la
lengua
- Sensación del dolor, tacto y temperatura conservada en toda la lengua
30. ¿Cuál de las siguientes alternativas es correcta?
- Las sales biliares desconjugadas son absorbidas preferentemente en el
colon
31. En un paciente de 45 años de edad con colestasis biliar, se encuentra una
elevación de los niveles sanguíneos de fosfatasa alcalina hasta 3 veces la cifra
normal. ¿Cuál de las siguientes alternativas estará también elevada como
evidencia del daño de la vía biliar?
- Gamma glutamil transpeptidasa
32. Revisando la angiografía de un hombre de 70 años en estudio por aneurisma
de aorta abdominal el radiólogo informa de la presencia de una oclusión
completa de la arteria mesentérica inferior. El paciente se encuentra
completamente asintomático.
¿Cuál de las siguientes arterias se anastomosa a la sistema arterial
de la mesentérica inferior?
- Cólica media
33. Lactante de 3 meses de vida es atendido por presentar diarrea, se administra
una solución de glucosa y electrólitos por vía oral. La proteína de membrana
apical que explica la capacidad de esta solución para proporcionar aporte de
glucosa e hidratación es ………..
- SGLT-1
34. Paciente ha sufrido herida de bala en el abdomen, se le ha tenido que extirpar
el segmento medio y distal del ileon. En este caso la síntesis hepática de sales
biliares estará …..…..
- Incrementada por estímulo de la enzima colesterol 7 alfa hidroxilasa
35. Un varón de 75 años ingresa al consultorio por presentar ictericia marcada de
piel y las escleras. El estudio del paciente mostró que presentaba un tumor que
obstruía la totalidad del conducto hepático común. ¿Cuál de las siguientes
estructuras se encontrará dilatada en este paciente?
- Conductos de Hering
36. En un paciente con insuficiencia renal crónica, el déficit en la absorción de
calcio a nivel del enterocito se debe a lo siguiente:
- No se convierte la 25 hidroxicolecalciferol a 1,25 dihidroxicolecalciferol
1. Varón de 30 años es traído a emergencia por agresión abdominal con arma de fuego
(pistola) y es sometido a laparotomía exploratoria, observándose isquemia del colon
ascendente y parte del colon trasverso ¿la lesión de cuál de las siguientes arterias
explicaría esta isquemia?
(unidad 1, sesión 2, logro 6: (Describe la irrigación visceral: arterias de tronco celiaco,
arteria mesentérica superior e inferior, topografía de superficie, órganos por cuadrante)
d. Mesentérica superior

2. Respecto a las sustancias gastrointestinales que regulan la secreción pancreática;


marque la afirmación correcta:

(unidad 1, sesión 3, logros 2 y 3: describir las hormonas gastrointestinales:


estímulos y funciones)
b. La acetilcolina es capaz de estimular la secreción enzimática y de bicarbonato del
páncreas

3. Ante una lesión del X par craneal, ¿cuál de los siguientes músculos mantiene
conservada su función?:
(unidad 2, sesión 08, logro 4: Paladar blando: componentes musculares)
b. Tensor del velo del paladar

4. Experimentalmente se utiliza atropina (anticolinérgico) para inhibir la secreción de


gastrina, sin embargo la secreción de esta hormona se sigue dando ante estímulos
vagales. Esta situación se explica porque la atropina:
(unidad 1, sesión 3, logro 3 : describir las hormonas gastrointestinales: estímulo y
funciones de la gastrina y colecistoquinina)

d. No bloquea la acción del péptido GRP

5. Un varón de 50 años es sometido a extirpación del duodeno y parte proximal del


yeyuno. Esta situación ocasionaría la pérdida de las células ……….. , productoras de
…………………
que estimula la secreción de bicarbonato por el páncreas.
(unidad 1, sesión 3, logro 3: describir las hormonas gastrointestianles: estímulos y
funciones de la secretina y péptido insulinotrópico dependiente de glucosa)
a. “S” / secretina

8. Se evalúa la expresión de la proteína Agrp en una persona con alteración del


apetito; lo correcto respecto a esta proteína es…..
(unidad 1, sesión 3, logro 4: Explica los mecanismos de control del apetito y saciedad )
b. La mutación del gen que la codifica produce adelgazamiento
9. Juana cae de la bicicleta y se fractura la región anterior del hueso maxilar superior
con compromiso de la fosa incisiva. Al examen físico de la región esperaría encontrar
alteración en la sensibilidad de la encía …………………
(unidad 2, sesión 8, logro5: paladar: paladar duro y blando: irrigación e inervación)
c. palatina anterior

10. Recién nacido es atendido por el neonatólogo y luego entregado a su madre para
dar de lactar; la madre al dar de lactar observa coloración azulada de labios,
acompañado de tos persistente, dificultad respiratoria y distención abdominal. Se le
intenta colocar una sonda nasogástrica pero esta retorna a la cavidad oral en todos los
intentos. ¿Cuál de las siguientes anomalías del desarrollo es el más probable en este
caso?
(unidad 1, sesión 4, logro 3: identificar las anomalías en el desarrollo del esófago:
atresia y/o fístula traqueo esofágica)
b. Atresia esofágica proximal con fístula traqueoesofágica distal l

11. ¿Cuál de los siguientes mecanismos ocurre durante la defecación?


(unidad 2, sesión 13, logro 6: motilidad del intestino grueso: contracciones
segmentarias, movimientos en masa, defecación y reflejo gastrocólico)
b. En la posición de “cuclillas” el músculo puborrectal se halla relajado

13. Un paciente luego de un accidente sufre lesión del piso de la boca, se constata
daño del nervio “cuerda del tímpano”, en este caso se esperaría encontrar disminución
de la
………………………….… de la lengua
(unidad 2, sesión 10, logro 3: Irrigación e inervación de la lengua)
c. Sensación del gusto en los dos tercios anteriores

14. ¿Cuál de las siguientes afirmaciones es la correcta sobre la gastrina?


(unidad 1, sesión 3, logro 1: reconocer las características de las sustancias
reguladoras gastrointestinales: hormonas, sustancias paracrinas y neurocrinas)
d. Actúa en la célula diana mediante su receptor CCk tipo B

15. Al recibir un paciente con signos de hipovolemia y antecedente de trauma en


abdomen por accidente de tránsito, usted identifica radiológicamente: lesión de
primera vértebra lumbar y signos de lesión en páncreas; durante la cirugía se observó
pobre irrigación de asas intestinales. El vaso afectado es la arteria ……..
(unidad 1, sesión 1, logro 6: reconocer las estructuras a nivel de L1, nivel de los
principales vasos sanguíneos)
c. mesentérica inferior
16. Un paciente sufre de daño a nivel del cuello con lesión muscular en la región de la
faringe. En el examen físico se determina dificultad para la elevación de la faringe y
para el cierre del itsmo de las fauces. En este caso, probablemente esté afectado el
músculo: (unidad 2, sesión 11, logro 2: Músculos de la faringe: identificación,
constrictores y longitudinales)

c. palatofaringeo

17. Varón de 50 años a quien le realizan la curación de la segunda molar de la arcada


superior derecha. En un momento determinado, el paciente acusa de intenso dolor
de la pieza dentaria en tratamiento. La vía aferente del dolor viaja a través del
nervio …………
(unidad 2, sesión 9, logro 6: Inervación de los dientes)
a. trigémino V2

18. La distención gástrica por los alimentos produce incremento de secreción de


HCl mediante la producción de ………….. que estimula a las células vía
proteína
………..
a. gastrina / parietal / Gq

19. Un niño de tres años llega a emergencia con disfagia (dificultad para tragar), dolor
retro esternal, salivación y llanto. Se sospecha de ingesta de cuerpo extraño (moneda)
en el esófago; al ser evaluado se constata en una radiografía presencia de cuerpo
extraño a nivel de C6 (6° vértebra cervical). El cuerpo extraño estará suspendido a nivel
del estrechamiento producido por………..
c. el músculo cricofaríngeo

20. La triada portal (arteria hepática, vena portal y conducto biliar común) está
contenida en el ligamento …….……… y derivan embriológicamente del ……
a. hepato duodenal / mesenterio ventral

22. Un paciente refiere no percibir algunos sabores, al examen físico se constata


alteración en la percepción de sabores y del dolor en el tercio posterior de la lengua
¿Cuál de los siguientes nervios estará alterada en su función?
c. Glosofaríngeo (IX par)

24. En el caso de un paciente con gastrinoma (tumor productor de gastrina), la


presencia de úlceras duodenales y erosión de la mucosa gástrica, se debe
principalmente a…….
c. el exceso de HCl por estímulo de receptores CCK-B en la célula parietal
25. El reflejo entero gástrico se caracterizan por:
d. originarse debido a la distensión duodenal y presencia del quimo ácido

26. Mauricio tiene dificultad para deprimir el paladar y elevar la parte posterior de la
lengua. En este caso estará afectado un músculo , específicamente el músculo
…………….
b. extrínseco – palatogloso
30. En condiciones normales, el ingreso de 600 ml de líquido es el estómago provoca
un aumento de presión intragástrica de unos 12 cm de H2O. Después de una
vagotomía (corte del nervio vago) es de esperar que el ingreso del mismo volumen de
líquido ocasione
………………………………… de la presión
intragástrica. c. un aumento mayor

La explicación fisiológica de presentar somnolencia de 30 minutos a 1 hora


después de ingerir alimentos, se explica por: a. Aumento del cloro intraluminal

e. Aumento de la alcalinidad sanguínea

5. Se presenta un paciente, el cual presenta un antecedente de tuberculosis intestinal,


por lo cual, se le resecó 80 cm de íleon distal. Desde el punto de vista fisiológico, el
paciente puede presentar una de las siguientes alteraciones: a. Disminución de la
secreción de Vitamina B12

e. Disminución de la absorción de ácido glicocólico

Un paciente es sometido experimentalmente a un fármaco que modifica el flujo salival,


obteniéndose un volumen de saliva de 288 ml en 6 horas. En este caso las
concentraciones de electrolitos y bicarbonato en la saliva obtenida varían de la
siguiente manera: a. ↑ Na+,
↓ K+, ↑ Cl-, ↑ HCO3-

1. b. ↓ Na+, ↓ Cl-, ↑ K+, ↓ HCO3-

9. Uno de los siguientes elementos debería hallarse con más probabilidad en el


esófago de un paciente que sufre de reflujo gastro esofágico… a. Pepsina

- PEPSINA

10. Un paciente de 40 años cursa con anemia de 8g/dl, aqueja además de astenia y
sensación de hormigueo bilateral en los miembros inferiores, al examen se halla
alteración de la sensibilidad a la vibración y camina con ampliación de la base de
sustentación. Uno de los siguientes procedimientos sería de ayuda para el diagnóstico
de este paciente: a. Tomografía cerebral
b. Biopsia de la mucosa gástrica

11. Paciente de 60 años ingresa por caída hace 1 hora y pequeño hematoma en cuero
cabelludo, al examen físico ampliado se observa ictericia de piel y mucosas
generalizada, abdomen blando, se palpa estructura quística no dolorosa en hipocondrio
derecho que corresponde a vesícula biliar (signo de Courvoisier), en los exámenes de
laboratorio se halla niveles bajos en la formación de estercobilinógeno y urobilinógeno
en heces, incremento de la bilirrubina conjugada en la orina, elevación de fosfatasa
alcalina y gamma glutamil transpeptidasa séricas. El presente cuadro puede ser
explicado por: a. Reabsorción de hematoma

c. Carcinoma de la cabeza de páncreas

13. Un recién nacido presenta vómitos biliosos poco tiempo después de cada alimento.
Al preguntar a la madre sobre antecedentes, ella recuerda que tuvo polihidramnios
durante la gestación, pero un análisis de cariotipo fue normal. Una de las siguientes es
la causa más probable de estos hallazgos en el recién nacido: a. Enfermedad de
Hirschprung

e. Malrotación de la yema pancreática ventral

15. En un estudio de la secreción de hormonas gastrointestinales, sus concentraciones


en la vena porta se midieron durante perfusión luminal del intestino delgado con
soluciones de diversas magnitudes de pH. ¿Qué hormona aumentará en el plasma de la
vena porta durante perfusión a través del intestino con una solución de pH 3? a. CCK

- e. secretina

16. Paciente de 30 años que ingresa a causa de un traumatismo abdominal cerrado. En


la exploración se aprecia discreta palidez de piel y mucosas, auscultación pulmonar
normal, taquicardia de 120 /min. Discreta distensión abdominal y matidez en flancos; el
hematocrito, que era prácticamente normal al ingreso, disminuye a 30% a las tres
horas. En la Rx de tórax se objetiva fractura de las costillas 10-11 izquierdas. La causa
más probable de la anemización en este paciente es: a. traumatismo renal con
hemorragia retroperitoneal.

1. c. rotura de bazo con hemoperitoneo.

18. Revisando la angiotomografía de un hombre de 70 años en estudio por


aneurisma de aorta abdominal, el radiólogo le informa de la presencia de una
oclusión completa de la arteria mesentérica inferior. El paciente se encuentra
completamente asintomático. La oclusión de la arteria mesentérica inferior cursa de
manera asintomática en muchas ocasiones ya que el territorio que irriga puede
recibir flujo proveniente de la arteria: a. cólica derecha
e. cólica media

19. En las patologías de esófago es importante conocer bien la anatomía esofágica.


¿Cuál de las siguientes afirmaciones es correcta? a. El esófago tiene capa mucosa,
muscular y serosa

1. c. El esófago torácico pasa por detrás del cayado aórtico

20. A pesar de que pueda haber variaciones anatómicas, lo habitual es que el


ciego sea irrigado por una rama arterial que proviene de unas de las siguientes
arterias: a. Iliaca derecha

1. d. Mesentérica superior

21. Ante un paciente con una cirugía abdominal urgente, el informe operatorio señala
que se ha realizado una resección de todo el duodeno y del tercio proximal del yeyuno
manteniendo íntegros el estómago y todo el íleon, así como los dos tercios distales del
yeyuno. En el seguimiento nutricional del paciente ¿Qué vitamina o mineral presentará
con menor probabilidad una disminución de su absorción?

- a. Cianocobalamina

23. ¿Cuál de las siguientes sustancias forma parte de la secreción biliar? a. Tripsina

- Lecitina

25. ¿De qué musculo forma parte el ligamento inguinal?

- Oblicuo externo del abdomen

27. ¿Cuál de las siguientes enzimas está localizada en el borde en cepillo y juega un rol
en la digestión de proteínas?

e. Carboxipeptidasa A.

28. Una de los siguientes sustancias, NO sirve como un buen agente emulsificante:

- a. Colesterol

29. La sustancia que estimula el crecimiento de la mucosa gástrica es: a.

Secretina d. Gastrina

30. ¿Cuál de las siguientes alternativas es una función de la colecistokinina? a.


Relajación de la vesícula para la salida de bilis
d. Secreción de enzimas pancreáticas

31. Con respecto a la anatomía del tronco celiaco, señale lo correcto a. El tronco
celiaco se origina de la cara posterior de la aorta abdominal

- d. La hepática común que es una de sus ramas, participa en la


irrigación del estómago.

32. Con respecto a la anatomía del duodeno, marque la respuesta correcta: a. Tiene
una distribución en forma de “C”, que rodea la cola del páncreas

- b. La 3ra porción duodenal está contenida en la pinza vascular aortomesentérica

En el íleon se absorbe aproximadamente el 95% de a través de la


circulación
enterohepática. a. agua

c. sales biliares

35. La estimula el mecanismo paracrino de la secreción de ácido clorhídrico.

- a. histamina

36. En la digestión de proteinas, es el principal estímulo para convertir el


pepsinógeno en pepsina. a. la gastrina

- b. el pH ácido

37. Con respecto a la somatostatina, marque lo correcto: a. Es secretada por las células
S del intestino

- . Interviene en la fase intestinal de la secreción gástrica

38. En pecten anal, es una estructura comprendida entre: a. la línea pectínea y los
senos anales

- d. la línea anocutánea y la línea pectínea

39. ¿Cuál de las siguientes alternativas es una proenzima pancreática? a. Tripsina

1. b. Elastasa
2. c. Quimotripsinógeno
3. d. Amilasa
4. e. Procarboxipepitidasa C.
1) En este paciente, el bloqueo farmacológico de los receptores H2 en la mucosa
gástrica:
a) No tiene efecto sobre la secreción de ácido inducida por el vago
b) Evita la activación de adenilciclasa por gastrina
c) Inhibe la secreción de ácido inducida por gastrina y mediada por el vago
d) Causa un aumento en el transporte de potasio por las células parietales gástricas
Se validó la A :)
2) Si se considerara una gastrectomía total para curar la gastritis del paciente, cuál
de las siguientes sustancias ya no se produciría:
a) Gastrina
b) Quimiotripsina
c) Amilasa
d) Pepsinógeno

3) Un paciente hipertenso está tomando un medicamento bloqueador de receptores


alfa 1 adrenérgicos (prazosina) y como efecto secundario se queja de:
a) Lo escaso que es el medicamento
b) No tiene problemas en la salivación
c) Hiposalivación
d) Hipersalivación

4) Con respecto a las lesiones y enfermedades de la boca, marque lo correcto:


a) La leucoplasia se desprende al roce
b) la eritroplasia puede degenerar en adenocarcinoma
c) El muguet oral es una enfermedad bacteriana en inmunodeprimidos
d) la eritroplasia debe ser biopsiada
NOTA: fue validada la opción B ya que no es motivo del curso que sepamos el puto
cáncer.

5) En este paciente, se puede asumir que la pancreatitis ha sido ocasionada por


una disminución en el efecto de:
a) Amilasa
b) Lipasa
c) Inhibidor de la tripsina
d) Entercinasa

6) Un efecto secundario en el estómago por la acción de la secretina es:


a) Disminución en la liberación de pepsinógeno
b) Menor actividad de la pepsina
c) Mayor paso de bicarbonato a sangre periférica
d) Aumento en la producción de factor intrínseco

7) Dentro de los factores protectores de la mucosa gástrica se pueden mencionar


múltiples protagonistas. Uno de ellos es:
a) CCK
b) Gastrina
c) Receptor muscarínico
d) Pepsina

8) La saliva puede tener una variedad de electrolitos en su composición. Entre ellos el


cloro, respecto al cual se puede afirmar:
a) Su mayor concentración se consigue con el flujo bajo
b) Su concentración no llega a ser tan alta como en el plasma
c) Con flujo alto su concentración es mayor que la del plasma
d) Su menor concentración se alcanza con flujo alto
9) En el síndrome de boca seca o síndrome de Sjogren, una de las
complicaciones asociadas es:
a) caries
b) Disminución de la acidez gástrica
c) Aumento en de la producción de saliva
d) Aumento del pH bucal

10) Estimula la producción de


saliva:
a) Vasodilatación periglandular
b) Atropina
c) Fatiga o cansancio
d) Expresión de miedo

11) El omeprazol actúa sobre la membrana de la célula


a) Basolateral/principal
b) apical/principal
c) Basolateral/parietal
d) apical/parietal

12) Para protegerse del entorno ácido, el Helicobacter pylori se autogenera un


entorno de pH menos ácido alrededor suyo, gracias a una enzima que alcaliniza su
entorno local mediante la conversión de:
a) urea en NH3
b) H2O y CO2 en ácido carbónico
c) NH3 en urea
d) H2CO3 en bicarbonato

13) La anemia perniciosa destruye las células:


a) mucosas del cuello
b) oxínticas
c) principales
d) mucosas superficiales

14) La célula mucosa del cuello gástrico produce:


a) Moco
b) ácido clorhídrico
c) pepsinógeno
d) Factor intrínseco

15) El aumento en la acidez del estómago producido principalmente por la


infección de Helicobacter pylori se debe a la disminución de:
a) Somatostatina
b) Bicarbonato por las glándulas de Brunner
c) Secretina
d) Colecistoquinina

16) De las siguientes sustancias secretadas por los órganos de este paciente, la
más alcalina es la secreción:
a) pancreática
b) Esofágica
c) Yeyunal
d) Salival

18) En cuanto a la gastritis de este paciente, se encontró que era producida por la
bacteria Helicobacter pylori. Esta bacteria sobrevive en el medio ácido del estómago
gracias a:
a) ácido clorhídrico
b) Toxina CagA
c) Ureasa
d) Jugo pancreático

19) La lengua está recubierta por epitelio:


a) pseudoestratificado columnar no queratinizado
b) plano estratificado no queratinizado
c) pseudoestratificado columnar ciliado
d) plano estratificado queratinizado

20) El esfínter anal interno tiene musculatura …….. y tiene control ……..
a) lisa / voluntario
b) lisa / involuntario
c) esquelética / simpático
d) esquelética / parasimpático

21) La arteria Aorta proporciona la irrigación al tubo digestivo ¿cuál de las


siguientes arterias proporciona la irrigación al ángulo cólico derecho?
a) mesentérica superior
b) mesentérica inferior
c) frénica inferior
d) tronco celiaco

22) Paciente de 26 años que le cuenta en su historia clínica que cada vez que almuerza
a los 20 min tiene deseo de defecar, le comenta que su hijo de 1 mes le pasa lo mismo
pero más intenso. Esto se explica por el reflejo …….., el cual está …… en el paciente
a) colicoileal / normal
b) colicoileal / alterado
c) gastrocólico / normal
d) gastrocólico / alterado

23) La región del estómago que se comunica con el duodeno es la


a) pilórica
b) cardias
c) cuerpo
d) fórnix

24) Acude a consulta un px que fue diagnosticado de úlcera péptica 3 días antes.
Luego de múltiples pruebas diagnósticas se concluye que el paciente presenta un
tumor secretor de gastrina, ¿cual de las siguientes situaciones estará incrementada?
a) distensión gástrica
b) inhibición del vaciado gástrico
c) secrecion de acido clorhidrico
d) inhibición de la secreción de pepsinógeno
25) En el sistema digestivo, el control del apetito está dado por un complejo sistema
de sustancias y órganos integradores, los cuales regulan la ingesta de alimentos. La
…… es una sustancia orexígena y es sintetizada por el ……
a) leptina / estómago
b) felina / intestino
c) leptina / estómago
d) grelina / estómago

26) Con respecto a la actividad eléctrica del sistema digestivo, marque la


alternativa correcta
a) corresponden a potenciales de acción que están presentes de forma continua
y le dan capacidad de peristalsis autónoma al sistema digestivo
b) la frecuencia de las ondas lentas NO se ve influenciada por la actividad neural ni
las
hormonas gastrointestinales
c) en el estómago las ondas lentas se dan en una frecuencia de 6 x min
d) las ondas lentas son cambios lentos y ondulantes del potencial en reposo
e) la frecuencia de las ondas lentas va de 6 a 12 ondas por minuto

27) Ante una lesión del IX pc, el músculo …… se altera en su función


a) palatogloso
b) estilofaríngeo
c) palatofaríngeo
d) constrictor superior

28) Un varón de 50 años es sometido a extirpación del duodeno y parte proximal del
yeyuno. La pérdida de estímulo hormonal en el páncreas para la secreción
enzimática se explica por la pérdida de células
a) parietales, productoras de factor intrínseco
b) K productoras de factor intrínseco
c) M productora de CCK
d) I productora de CCK

29) Marque la respuesta correcta:


A. El bronquio derecho constituye una de las estrecheces del esófago
B. Todos los órganos del sistema digestivo tienen capa serosa
C. La pared gástrica en el fondo es más delgada que en el cuerpo y antro
D. El esfínter de Oddi rodea a la papila menor duodenal

30) Marque la respuesta correcta en relación a la gastrina:


A. Al distenderse el estómago, se inhibe su producción.
B. Se estimula por la liberación de noradrenalina
C. Las células G son las productoras y se encuentran principalmente en el antro
gástrico
D. Las células G se encuentran principalmente en el fondo gástrico

31) Para poder morder una manzana, es necesario usar el siguiente músculo:
A. Milohiodeo
B. Tensor del paladar
C. Orbicular de los labios
D. Buccinador
32) Sustancia que inhibe la secreción y la motilidad del estómago prolongando el
tiempo de digestión:
A. Enteroglucagon.
B. Polipéptido pancreático
C. Péptido 1 similar al glucagón (GLP-1).
D. Péptido insulinotrópico dependiente de la glucosa (GIP).

33) El nacimiento de la arteria mesentérica superior se puede encontrar en cuál


de los cuadrantes abdominales:
A. Hipocondrio derecho
B. Hipogastrio
C. Epigastrio
D. Mesogastrio
34) Entre las múltiples causas de la Enfermedad por Reflujo Gastroesofágico, se
puede considerar también a una alteración en las del esfínter esofágico
inferior:
A. Ondas secundarias
B. Contracciones tónicas
C. Ondas lentas
D. Glándulas subesofágicas

35) Producto de la alimentación, se producen diversas sustancias peptídicas,


cininas y bradicininas, las cuales permiten que:
A. Se produzca neovascularización en los territorios de las arterias abdominales
B. La acción de la lipasa pancreática se vea incrementada
C. El consumo de O2 del intestino aumente ligeramente
D. El flujo sanguíneo intestinal aumente hasta 8 veces

36) El dolor periumbilical o epigástrico en el inicio de una apendicitis aguda se debe a:


A. Estímulo del nervio vago.
B. Íleo secundario.
C. Irritación del peritoneo parietal.
D. Estímulo del sistema simpático.

37) El aumento en la actividad motora de la pared gástrica genera un aumento


en los niveles locales de qué sustancia en la microvasculatura:
A. Adenosina
B. Colecistoquinina CCK
C. Endotelina
D. Gastrina

38) ¿Cuál de los siguientes péptidos inhibe el vaciamiento gástrico?


A. Colecistoquinina
B. Péptido inhibidor gástrico
C. Motilina
D. Gastrina

39) Los músculos de la masticación que producen la retropulsión de la mandíbula son:


A. temporales [mas seguro]
B. maseteros
C. milohioideos
D. pterigoideos
40) En relación a la fisiología gástrica, marque lo correcto:
A. la cimetidina actúa en la región basolateral de la célula parietal
B. la marea alcalina se debe al paso de bicarbonato través de la membrana apical
de la célula principal
C. el cloro difunde hacia el exterior por la la región basolateral de la célula parietal
D. la salida de hidrogeniones a la luz es por difusión facilitada

41) Durante el sueño, la concentración de bicarbonato en la saliva:


A. Se eleva a niveles mayores que los del plasma
B. Aumenta
C. No tiene efecto
D. Disminuye

42) La secreción de saliva es importante en la fisiología digestiva. Su


concentración de potasio llega a ser menor que la del plasma cuando su secreción
tiene un flujo:
A. Intermedio
B. Nunca
C. Bajo
D. Alto

42) Respecto a las glándulas salivales, marque lo incorrecto:


A. la glándula parótida produce secreción serosa
B. la glándula sublingual drena a través de conducto de Wharton
C. La glándula parótida drena a través del conducto de Stenon
D. la glándula sublingual tiene forma de garfio

43) Con respecto a la saliva, marque la respuesta correcta:


A. será hipertónica cuando el flujo es bajo
B. a mayor flujo, menor concentración de Na
C. a mayor flujo, menor concentración de cloro
D. el sistema simpático estimula su secreción
CI 3
44) En relación a la circulación hepática, marque lo correcto:
a) Los sinusoides hepáticos transportan sangre mixta
b) La vena porta proporciona el 50% de sangre al hígado
c) La vena porta se forma a partir de la vena esplénica y la mesentérica inferior
d) La arteria hepática deriva de la mesentérica superior

45) Dentro de las funciones de las células de Ito, marque lo incorrecto:


a) Sintetizan colágeno
b) Almacenan vitamina A
c) Se les llama células estrelladas
d) Pueden fagocitar patógenos y actúan como presentadoras de antígeno

46) Paciente con tumor neuroendocrino productor de secretina, debido a lo cual se


puede esperar que su secreción pancreática, comparada con la de una persona sana
de bajo flujo, tenga una concentración de:
a) Sodio aumentada
b) Igual
c) Bicarbonato aumentada
d) Potasio disminuida
47) El GALT se localiza en:
a) Lámina propia
b) submucosa
c) borde en cepillo
d) superficie de criptas de Lieberkühn

48) En relación a la histología hepática, marque lo correcto:


a) la zona 1 se afecta rápidamente en estados de hipovolemia y shock
b) La zona 1 se encuentra cercana a la vena central lobulillar
c) La zona 3 se encuentra más cerca a la vena central lobulillar
d) La zona 3 se encuentra más cerca al eje menor del acino hepático

49) El acino pancreático difere con el de las glándulas salivales en:


a) Contiene células centroacinares
b) No produce secreción serosa
c) El páncreas produce principalmente secreción mucosa
d) No tienen diferencias

50) Durante la digestión de las grasa, para que la lipasa actúe adecuadamente se
requiere que el pH aumento en la luz intestinal, lo cual es logrado, entre otros, por la
secreción de las células:
a) Del conducto interlobulillar
b) Centroacinares
c) Acinares
d) Alfa
51) La secreción de la colecistoquinina (CCK) se produce en la fase:
a) intestinal
b) En las 3 por igual
c) gástrica
d) Cefálica
52) ¿Por cuál de las siguientes células es secretada principalmente la pro
enzima procarboxipeptidasa?
a) Acinares del páncreas
b) Epiteliales del duodeno
c) Ductales del páncreas
d) Centro Acinares del páncreas
53) Una mujer de 43 años dolor en hipocondrio derecho e icterica. En la ecografía se
evidencia cálculos biliares. Estos cálculos lo más probable es que se encuentren
localizados en:
a) conducto colédoco
b) Conducto cístico
c) Vesícula biliar
d) Conducto pancreático secundario

ECU 1:
Estudiante de 21 años sufre de gastritis aguda ocasionada por comer en lugares poco
higenicos. Suele consumir caramelos (“chupar”) mientras esta en base hasta la tarde.
También toma regular cantidad de leche (grasa, lactora, proteinas), pues le calma el
dolor y el ardor que sitnete por la gastritis (tiene dispepsia y cuando toma la leche se le
pasa).
Incluso cuando puede, se toma dos vasos de agua frita y le calma la molestia. Ha
decido ir al medico para tratarse, pues ya no soporta el dolor, el cual esta seguro que
los síntomas se producen por elevada producion de HCl en el estomago, y por ello le ha
recetado ranitidina
1.1) El consumir caramelos eleva los niveles en sangre de una hormona cuya función
es la estimulación de las células.
- Beta del páncreas por GIP el cual es una incretina y por consiguiente
estimula las células pancreáticas

1.2) Consumir caramelos indirectamente actica la via:


- POMP/ CART saciedad

1.3) Consumo de leche produce indirectamente


- CCK inhibición del vaciamiento gástrico mayor tonicidad del esfínter pilórico

1.4) Cuando el px toma dos vasos de agua, genera indirectamente un aumento en la


liberación de:
- vaso de agua distención → g astrina → secreción de HCl

1.5) El uso de ranitidina bloquea el receptor H2 de la histamina en las células parietales, la


histamina llega a estas células por:
- histamina es una hormona paracrina por → difusión
**endocrina es por via hematógena y si fuera neuroendorina es por un NTs

1.6) Aumenta la secreción salival:


- noradrenalina a través de los receptores Beta 2

1.7) En este paciente con gastritis aguda debida a una alta producción de ácido clorhídrico,
sería lógico esperar que el píloro tenga un tono muscular:
- primero la secretina
- luego CCK
**ambas reguladores del HCl, Gatritis aguda debido a una alta producción de HCL
piloro estará aumentado (por la CCK)

1.8) Debido al uso de ranitidina, los valores de somatoestina en sangre:


- ranitidina disminuye acción de gastrina se quiere secretar mas no
actúan los inhibidores como la somatoestina somatoestina disminuye

1.9) El uso de atropina en este paciente:


- Inhibirá la acción de las prostaglandinas
- Aumentará la producción de ácido clorhídrico
- Disminuirá la acción del receptor CCK-B
- Aumentará el pH del estómago
ECU 2:
Niño de sexo masculino de 2 años de edad, sufre de estreñimiento desde el
nacimiento (1 deposición cada 3-4 días). Madre menciona que le estimula la
defecación con un termómetro rectal, y continuo uso de enemas y laxantes. Desde
hace 6 meses comienza con vómitos postprandiales. Los síntomas aumentan en
frecuencia y magnitud y están en relación con los episodios de estreñimiento. No
refiere fiebre, tos, diarrea ni lesiones cutáneas. Al examen físico presenta regular
estado general, luce deshidratado. Abdomen distendido, blando, depresible e indoloro.
No se palpan masas abdominales. Se permeabiliza el canal anal con termómetro
rectal, encontrando cierta resistencia. Salida de material fecal mal oliente en regular
cantidad. Exámenes de laboratorio: hemograma normal. Signos inflamatorios de fase
aguda negativos. Alcalosis metabólica leve en sangre venosa. Radiografía con enema
baritado muestra recto y colon sigmoides dilatados (megacolon). Biopsia profunda:
ausencia de células ganglionares en la muestra enviada. Se realiza cirugía correctiva.

2.1) Durante la fase esofágica de la deglución, para un bolo alimenticio


determinado, a medida que avanza el bolo la fuerza de la contracción se hace
más:
- hiperpolarizado
- fuerte
- dependiente de Ach
- debil
2.2) Cuando este paciente ingiera sus alimentos, se espera que al momento de
pasar el bolo alimenticio por el esfínter esofágico superior, la presión intraesofágica
disminuya en:
- la porción proximal al bolo
- el tercio medio del esófago
- el cardias
- el lugar donde se contraiga la muscular propia
2.3) Al examinar la orofaringe del paciente, uno puede hallar fácilmente la
amígdala palatina, pues esta se encuentra inmediatamente detrás del músculo:
- Palatogloso
- Palatofaringeo
- Hiogloso
- Elevador del velo del paladar

2.4) Con respecto a la defecación en este caso, marque la respuesta correcta:


- En posición de cuclillas, el músculo puborectal genera un ángulo más agudo
en el recto
- El sigmoides y el recto están inervados por el nervio vago
- La sensación de defecar sólo se da cuando el recto es ocupado por
heces y alcanzado el 80% de su capacidad
- El esfínter anal comprometido tiene inervación autónoma

2.5) En este paciente [hirschsprung] se considera que está abolido el reflejo:


- Coloileal
- Rectoesfinteriano
- Gastrocólico
- Relajación receptiva
2.6) No se espera que sea causa del vómito:
- Ayuno prolongado
- Estimulación faríngea y del glosofaríngeo
- Irritación de la mucosa gástrica
- Dolor intenso
ECU 3:
Paciente de 54 años con antecedentes de alcoholismo, gastritis crónica,
tabaquismo pesado, obesidad, cálculos biliares y cirrosis, es llevado a la
emergencia por dolor abdominal en epigastrio irradiado a la espalda y trastorno
del sensorio.
Al examen físico: presión arterial 85/50 mmHg, frecuencia cardíaca 100
latidos/min, frecuencia respiratoria 18 x minuto, temperatura axilar 36°C.
Conjuntivas pálidas, escleras ictéricas nevus arácnidos en tronco, distensión
abdominal marcada, cabeza de medusa, matidez desplazable en ambos
flancos e hipogastrio, dolor a la palpación de abdomen.
Tiempo de protrombina: 24 seg (testigo: 13 seg); TPT: 38 seg, glicemia: 165 mg/dL,
uremia: 20 mg/dL, ASAT: 76 UI/L, ALAT: 22 UI/L, albumina: 2,5 g/dL, bilirrubina total:
2,6 mg/dL, bilirrubina directa: 1,4 mg/dL, amilasa sérica 4000 U/L.

3.1) En esta paciente, al aumento de la amilasa sérica, se debe directamente a una


lesión de:
a) páncreas
b) vesícula y árbol biliar
c) estómago
d) hígado

3.2) Considerando que el paciente sufre de gastritis, se puede decir que la


secreción de ácido por la mucosa gástrica
a) involucra transporte activo de hidrogeniones
b) es realizada principalmente por células principales
c) es inhibida por antihistaminas tomadas por pacientes con rinitis alérgica
d) involucra la liberación de HCl de los gránulos zimógenos

3.3) El paciente tiene hemorragia digestiva alta por várices sangrantes como
complicación. Llegando a estar en shock hipovolémico por hemorragia masiva, se
encontrara necrosis hepática en:
a) zona 1
b) no se afectan los lobulillos hepáticos en hemorragia
c) zona 3
d) zona 2

3.4) El misoprostol, análogo de las prostaglandinas está mejor indicado en:


a) cicatrización de úlcera péptica duodenal
b) erradica el helicobacter pylori
c) tratar el sind de Zollinger ellison
d) prevenir daño por AINES

3.5) De las siguientes sustancias secretadas por los órganos de este paciente, la
más alcalina es la secreción:
- Esofágica
- Salival
- Yeyunal
- Pancreática

3.6) En este paciente, se puede asumir que la pancreatitis ha sido ocasionada por
una disminución en el efecto de:
- Lipasa
- Enterocinasa
- Amilasa
- Inhibidor de la tripsina

3.7) ¿Cuál de las siguientes sustancias es segregada por el páncreas?


- Amilasa
- Pepsina
- Quimiotripsina
- Tripsina
3.8) Cada vez que este paciente toma alcohol, la acidificación de la luz del duodeno:
- Disminuye el vaciamiento gástrico
- Aumenta la contracción del esfínter de Oddi
- Aumenta la secreción del ácido gástrico
- Disminuye la secreción pancreática del bicarbonato
SISTEMA
DIGESTIVO
(ME 154)
EXAMEN
FINAL
Ciclo 2018-01

1. Un niño de 2 años es llevado a la consulta por diarrea persistente y edema de las extremidades, además
falta de crecimiento y desarrollo en relación a su edad. Los análisis de sangre revelan que tiene
concentración plasmática baja de proteínas (hipoproteinemia). Durante la endoscopía duodenal, se
coloca colecistokinina (CCK) endovenosa y se recoge muestras del líquido duodenal; el resultado del
líquido confirma incapacidad para hidrolizar proteínas a un pH neutro, esta situación mejora al añadir
una pequeña cantidad de tripsina. El paciente probablemente esté sufriendo la falta congénita de
………….
(Unidad 4, sesión 26, logro 2: Explicar la Digestión y absorción de las proteínas y sus alteraciones)
a. Pepsinógeno
b. PEPT-1
c. Carboxipeptidasas
d. Enterocinasa

2. Experimentalmente se incrementa la velocidad de la secreción salival con una sustancia, el


análisis de la composición de esta saliva obtenida se espera encontrar…………..
(Unidad 3, sesión 17, logro 5 : Explica la Influencia de la velocidad del flujo salival en la composición de
la saliva)
a. Elevación de concentración de bicarbonato, sodio y potasio
b. Elevación de concentración de cloro, sodio y potasio
c. Disminución de concentración de potasio
d. Disminución de concentración de potasio y bicarbonato

3. Paciente varón de 46 años soltero, consulta por odinofagia y bajo de peso, tiene antecedente de
tuberculosis desde hace 3 meses y es fumador crónico (10 cigarrillos por día); al evaluar la cavidad
oral se identifica lesión blanquecina en el dorso de la lengua y paladar blando, las lesiones se
desprenden con el baja lengua dejando una base eritematosa. Esta lesión corresponde
probablemente a ……………………….…..
( Unidad 3, sesión18, logro 1-2 : Describe las enfermedades inflamatorias, infecciosas y proliferativas
de la cavidad oral)
a. Eritroplaquia
b. Candidiasis oral
c. Leucoplaquia vellosa
d. Fibroma en cavidad oral

4. Minero de 32 años de edad, que acude a


centro de salud por presentar de forma
progresiva desde hace 1 año dificultad para
ingerir alimentos sólidos y luego líquidos;
refiere regurgitaciones alimentarias y marcada
pérdida de peso (15 kilos). Radiografia
baritada de esófago como se muestra en la
figura. El presente caso se explica
por……………….
(Unidad 2, sesión 12, logro 4: Identificar y
describir la función de los esfínteres
esofágicos)
a. Contracción incompleta del esfínter esofágico inferior
b. Dificultad para el inicio de la deglución
c. Relajación incompleta del esfínter pilórico
d. Relajación incompleta del esfínter esofágico inferior
5. Paciente mujer de 35 años acude a consulta por sensación de sequedad y lesiones en cavidad
oral. Al examen se observa atrofia de la mucosa, fisuras y úlceras; nota además sequedad e
irritación de la córnea y aumento del tamaño de las glándulas parotídeas. Su diagnóstico más
probable es artritis reumatoide; el hallazgo más probable en una biopsia de glándula parótida
es……..….
(Unidad 3, sesión 18, logro 3: Describe las enfermedades más frecuentes de las glándulas salivales)
a. Hiperplasia de acinos glandulares serosos
b. Gran infiltración de linfocitos y células plasmáticas
c. Gran infiltrado de linfocitos y macrófagos
d. Presencia de acinos normales con hiperplasia de células ductales

6. Un paciente con anemia acude con su médico quejándose de episodios frecuentes de


gastroenteritis. Un análisis de sangre revela anticuerpos circulantes dirigidos contra células
parietales gástricas. Su anemia es atribuible a la hiposecreción de
………………………
(Unidad 3, sesión 20, logro 5: Gastritis crónica. Tipos de gastritis)
a. Factor intrínseco
b. Proteina R (haptocorrina)
c. Pepsinógeno
d. Ácido clorhídrico

7. Dos estudiantes deciden tomar un receso para comer una hamburguesa a la hora del almuerzo.
Antes de llegar a la cafetería, impulsos nerviosos provenientes del complejo vagal dorsal iniciarán la
secreción de ácido gástrico por la liberación de desde el sistema nervioso entérico.
(Unidad 3, sesión 20, logro 2: Regulación de la secreción gástrica: estimulación, fases de la secreción)
a. Serotonina
b. Óxido nítrico
c. GRP (péptido liberador de gastrina)
d. Péptido intestinal vaso activo

8. Un niño de cuatro años de edad es llevado a la consulta por cuadros diarreicos frecuentes
caracterizados por heces pálidas, voluminosas y fétidas, presenta bajo peso y talla. Se mide la
concentración de cloruro en el sudor y se encuentra que sus valores son muy elevados. La alteración
más importante a nivel de células ductales del páncreas tiene relación directa con la conductancia
de…………
(Unidad 3, sesión 23, logro 5 Explica la Secreción pancreática: formación del jugo pancreático,
influencia de la velocidad de flujo y regulación)
a. Potasio
b. Bicarbonato
c. Sodio
d. Cloro

9. Una mujer de 50 años de edad que sufrió durante varios años resequedad de los ojos debida a
producción inadecuada de lágrimas es enviada con un gastroenterólogo para evaluación de pirosis
crónica. El examen endoscópico revela erosiones y tejido cicatrizal en la parte distal del esófago
justo por arriba del esfínter esofágico inferior. Las lesiones pueden atribuirse a la disminución de
uno de los siguientes componentes salivales:
(Unidad 3, sesión 17, logro 4: Explicar la Formación de la saliva y cuáles son sus componentes)
a. Bicarbonato
b. Lactoferrina
c. Ig A
d. Amilasa
10. Se evalúa los valores séricos de las siguientes sustancias a un paciente con enfermedad hepática
terminal; en este paciente se espera encontrar la combinación con la letra …………
(Unidad 3, sesión 22, logro 5: Describe las Pruebas de función hepática, la Insuficiencia hepática,
encefalopatía hepática e hipertensión portal)

Glucosa Amoniaco Albúmina


a. Aumenta Disminuida Disminuida
da
b. Disminui Aumentada Aumentada
da
c. Aumenta Aumentada Aumentada
da
d. Disminui Aumentada Disminuida
da

11. Una mujer de 35 años de edad HIV positiva, se presenta al médico con dolor abdominal en
cuadrante superior derecho e ictericia. La paciente refiere haber tenido múltiples episodios de
ictericia durante los últimos 10 años. Los exámenes para determinar hepatitis viral, dieron positivos
para Hepatitis B, siendo catalogado el caso como hepatitis crónica con alteración funcional. En un
examen de sangre ¿cuál de los siguientes parámetros está disminuido?
(unidad 3, sesión 22, logro 5: Pruebas de función hepática, Insuficiencia hepática, encefalopatía
hepática e hipertensión portal)
a. Fosfatasa alcalina
b. Albumina
c. Bilirrubina
d. Tiempo de protrombina

12. En el reflejo peristáltico del intestino delgado, uno de los siguientes eventos sucede en
la porción oral del bolo alimenticio…………...
(Unidad 2, sesión 13, logro 4: Explicar la Motilidad del intestino delgado: Contracciones segmentarias y
peristálticas)
a. Disminución de 5 hidroxitriptamina desde las neuronas IPAN
b. Contracción del músculo longitudinal
c. Acción del péptido intestinal vasoactivo (VIP) en el músculo circular
d. Acción de acetilcolina en el músculo circular

13. Experimentalmente se coloca una dosis alta de secretina en la luz intestinal duodenal; como
consecuencia de esto, en el jugo pancreático de la misma luz intestinal se observa la disminución
de la concentración de …..………..
(Unidad 3, sesión 23, logro 5: Explica la Secreción pancreática: formación del jugo pancreático,
influencia de la velocidad de flujo y regulación)
a. Na+
b. Cl-
c. K+
d. HCO3-

14. Un varón de 58 años de edad con enfermedad de Crohn severo fue sometido a una resección
ileal. Después de la cirugía este paciente padecerá de esteatorrea, esto se explica porque
…..………..
(unidad 4, sesión 26, logro 4: Explica las alteraciones en la Absorción de lípidos)
a. El pool de ácidos biliares se incrementa
b. Los quilomicrones no pueden formarse en el lumen intestinal
c. La micelas no pueden formarse
d. El páncreas no secreta lipasa

15. En un experimento se inserta un balón en el estómago de un voluntario, se infla poco a poco


mientras que se vigilan las presiones intraluminales. Aunque el volumen del balón aumenta
considerablemente, las presiones permanecen constantes. Esta relación volumen-presión se explica
por la liberación local de …………..
(Unidad 2, sesión 13, logro 1 Explica la Motilidad gástrica: relajación receptiva)
a. Acetil colina y gastrina
b. Colecistoquinina y óxido nítrico
c. Óxido nítrico y péptido inhibidor vasoactivo
d. Norepinefrina y óxido nítrico
16. La toxina del Vibrio cholerae causa diarrea debido a…….
(Unidad 4, sesión 27, logro 6: Explica el transporte hidroelectrolítico intestinal, toxina colérica)
a. La fosforilación del canal CFTR de los enterocitos de las vellosidades intestinales
b. El Incremento de la secreción de cloro por las células de la cripta intestinal
c. La inhibición de la producción de AMPc por las células epitelailes
d. El incremento de la absorción de agua y sodio a través de las uniones estrechas

17. ¿Cuál de las siguientes alternativas es una característica de la secreción exocrina del páncreas?
(Unidad 3, sesión 23, logro 5: Secreción pancreática: formación del jugo pancreático, influencia de la
velocidad de flujo y regulación)
a. Tiene una baja concentración de Cl- respecto al plasma
b. Es estimulada por la presencia de bicarbonato en el duodeno
c. La secreción enzimática es estimulada principalmente por la gastrina
d. Es hipotónica respecto al plasma

18. Una madre lleva a su hijo de dos años de edad a la sala de urgencias, estresada porque el niño
deglutió una moneda de 10 céntimos mientras la familia cenaba en un restaurante. El médico
observa mediante fluoroscopía que la moneda se halla en el estómago y asegura a la madre que la
moneda se eliminará con las heces. El médico recomienda utilizar la respuesta fisiológica que
permitirá la evacuación de la moneda del estómago al intestino ………….…..
(Unidad 2, sesión 13, logro 2: Explica la Motilidad gástrica: mezclado y vaciamiento)
a. Es por la relajación receptiva
b. Son los movimientos de mezcla y trituración
c. Es provocada por el ayuno
d. Es por la relajación del esfínter esofágico superior

19. Las estructuras en el hígado que permite que los productos metabólicos unidos a proteínas tengan
acceso a las membranas basolaterales de los hepatocitos, son…..
(Unidad 3, sesión 21, logro 4-5: Explica la Organización micro estructural del hígado)
a. Los Canalículos
b. Las fenestras sinusoidales
c. Las uniones intercelulares herméticas
d. Las células de Ito

20. La composición de la bilis es modificada conforme fluye por los conductillos biliares. Durante
este tránsito se espera que aumente la concentración de…….
(Unidad 3, sesión 22, logro 2: Describe la Secreción biliar, visión general del sistema biliar
extrahepático y composición de la bilis)
a. Ig A
b. Glucosa
c. Monómeros de ácido biliar
d. Vitamina A

21. Se mide experimentalmente el contenido gástrico de dos personas. La persona “A” tiene alto
contenido de grasa y la persona “B” tiene un contenido hipertónico ¿Cuál de las siguientes es
correcto respecto al vaciamiento gástrico? (Unidad 2, sesión 13, logro 2: Describe la Motilidad
y vaciamiento gástrico)
a. Hay ralentización del vaciado gástrico solo en “A”
b. El vaciamiento gástrico es más rápido en ambos
c. En ambos casos hay incremento de la motilina
d. Hay ralentización del vaciado gástrico en ambos casos

22. El examen endoscópico de un paciente con hipertensión portal grave revela venas tortuosas que
sobresalen hacia la luz del esófago. El paciente recibe tratamiento quirúrgico mediante la colocación
de una derivación que conecta la vena porta a la vena cava. Después de la operación el riesgo de
encefalopatía ………………….. y el riesgo de sangrado de várices ……………..
(Unidad 3, sesión 22, logro 5: Describe la Insuficiencia hepática, encefalopatía hepática e hipertensión
portal)
a. Aumentará/disminuirá
b. Disminuirá/disminuirá
c. Aumentará/aumentará
d. Disminuirá/aumentará
23. Un paciente varón de 18 años de edad acude al médico para sus exámenes de rutina. Sus
resultados de laboratorio muestran un valor de bilirrubina sérica de 4 mg/dl y una bilirrubina
directa de 0,3 mg/dl. Las pruebas de función hepática son normales. La alteración que explica
mejor este caso es por la deficiencia de ………………..
(Unidad 3, sesión 22, logro 3: Explica la Producción y excreción de bilirrubina. Tipos de bilirrubina e
ictericia)
a. Transaminasas
b. Glucuronil transferasa
c. Hemo oxigenasa
d. La 7 alfa hidroxilasa

24. Un hombre de 57 años de edad es llevado a urgencias con hematemesis masiva rojo brillante, a su
llegada se halla inconciente con PA: 80/40 mm Hg y FC: 124 lat/min. Luce ictérico con presencia de
“arañas vasculares en el tórax anterior y extremidades”, abdomen distendido con signo de oleada
positiva. Se encuentra esplenomegalia y pérdida de la masa muscular en extremidades. La
anastomosis vascular responsable del sangrado en este paciente es ………….…..
(Unidad 3, sesión 21, logro 2: Describe las anastomosis porto sistémicas)
a. Arteria gástrica izquierda y vena ácigos
b. Vena gástrica izquierda y vena ácigos
c. Vena paraumbilical y vena epigástrica inferior
d. Vena gástrica izquierda y vena esofágica superior

25. Un estudiante de medicina está comiendo un plato de comida a base de champiñones, espárrago
y salsa de soya. El sabor umami contenido en todos estos alimentos actúa a nivel de los botones
gustativos estimulando ………………..
(Unidad 2, sesión 10, logro 5: Describe los tipos y mecanismos moleculares para la detección de los
sabores)
a. El ingreso de sodio
b. Un receptor acoplado a proteína G
c. Su receptor específico T1R3
d. El ingreso de hidrógeno

26. Un hombre de 22 años de edad se presenta al médico con una historia de 1 año de evolución
caracterizado por dolor recurrente en fosa iliaca derecha y diarrea. Manifiesta además pérdida
de peso de 8 kg durante este periodo. La colonoscopía revela múltiples lesiones en el ileon
terminal y colon. La biopsia de estas lesiones revela engrosamiento, inflamación y ulceración
de la mucosa. El diagnóstico más probable en este caso es…….
(Unidad 4, sesión 28, logro 5: Describe la Enfermedad inflamatoria intestinal. Generalidades,
morfología y características)
a. Sprue celiaco
b. Enfermedad de Crohn
c. Sindrome de colon irritable
d. Colitis ulcerativa

27. Una de las funciones del músculo señalado es:


(Unidad 2, sesión 8, logro 3: Describir el Piso
de la boca: estructuras blandas que la
conforman)
a. Eleva el paladar blando
b. Recibe inervación del nervio maxilar
c. Deprime el hioides cuando la mandíbula está fija
d. Deprime la mandíbula cuando el hioides está fijo

28. Varón de 61 años que consulta por dolor retro esternal intenso desde hace 6 horas y después
de vómitos intensos y repetidos; al examen se observa disnea, cianosis, hipotensión y signos
clínicos de shock. La radiografía simple de tórax muestra neumomediastino. El líquido en el
espacio pleural aspirado tiene alta concentración de amilasa. ¿Cuál de las siguientes
alternativas puede explicar este cuadro clínico?
(Unidad 3, sesión 18, logro 6: Describe algunas Enfermedades del esófago)
a. Sindrome de Mallory Weiss
b. Rotura espontánea de esófago
c. Neumotórax por probable herida penetrante
d. Perforación de ulcera gástrica de cara posterior, con complicación torácica
29. La secreción del ácido en la célula parietal gástrica se lleva a cabo por una ATPasa especifica que
intercambia hidrogeniones (H+) del citosol por…..
(Unidad 3, sesión 20, logro 1: Explica la Secreción del HCl y sustancias que la alteran)
a. Cl-
b. HCO3-
c. Na +
d. K+

30. En condiciones normales el ingreso de 600 ml de líquido es el estómago provoca un aumento de


presión intragástrica de unos 12 cm de H2O. Después de una vagotomía (corte del nervio vago) es
de esperar que el ingreso del mismo volumen de líquido provoque lo siguiente:
…………………………………
(Unidad 2, sesión 13, logro 1: Describe la Motilidad gástrica: relajación receptiva)
a. Un aumento igual de la presión
b. Que no aumente la presión
c. Un aumento mayor de la presión
d. Una disminución de la presión

31. Una paciente de 30 años de edad es sometida a una cirugía en oído medio derecho por un problema
de otoesclerosis. Luego de la cirugía refiere alteración en la percepción de sabores. Al evaluar el caso
usted esperaría encontrar……….
(Unidad 2, sesión 10, logro 5: Describe la Irrigación e inervación de la lengua)
a. Alteración en la sensación del dolor y temperatura en el tercio posterior de la lengua
b. Alteración en la sensación del gusto en los dos tercios anteriores de la lengua
c. Alteración en la sensación del gusto en la punta de la lengua
d. Sensación del dolor, tacto y temperatura conservada en toda la lengua

32. ¿Cuál de las siguientes alterativas es correcta?


(Unidad 4, sesión 26 : Explica la digestión y absorción de nutrientes y sus alteraciones)
a. En el borde luminal, en cepillo, del intestino delgado, la absorción de sodio únicamente se
realiza asociada a la de glucosa.
b. El lugar principal para la absorción del hierro es el ileon
c. Las sales biliares desconjugadas son absorbidas preferentemente en el colon
d. El proceso de digestión y absorción de la vitamina B12 no se altera en insuficiencia
pancreática.

33. En un paciente de 45 años de edad con colestasis biliar, se encuentra una elevación de los niveles
sanguíneos de fosfatasa alcalina hasta 3 veces la cifra normal. ¿Cuál de las siguientes alternativas
estará también elevada como evidencia del daño de la vía biliar?
(Unidad 3, sesión 22, logro 5: Pruebas de función hepática, Insuficiencia hepática, encefalopatía
hepática e hipertensión portal)
a. Tiempo de protrombina y albúmina sérica
b. Transaminasas hepáticas (ALT y AST)
c. Glucoronil transferasa
d. Gamma glutamil transpeptidasa

34. Revisando la angiografía de un hombre de 70 años en estudio por aneurisma de aorta abdominal el
radiólogo informa de la presencia de una oclusión completa de la arteria mesentérica inferior. El
paciente se encuentra completamente asintomático. ¿Cuál de las siguientes arterias se anastomosa
a la sistema arterial de la mesentérica inferior?
(Unidad 4, sesión 25, logro 1: Identifica la Arteria mesentérica superior e inferior, ramas y anastomosis)
a. Ileal
b. Cólica media
c. Sigmoideas
d. Cólica izquierda

35. Lactante de 3 meses de vida es atendido por presentar diarrea, se administra una solución de
glucosa y electrólitos por vía oral. La proteína de membrana apical que explica la capacidad de esta
solución para proporcionar aporte de glucosa e hidratación es ………..
(Unidad 4, sesión 26, logro 1: Explica la Digestión y Absorción de los hidratos de carbono. Alteraciones)
a. GLUT-5
b. SGLT-1
c. CFTR
d. GLUT-2
36. Paciente ha sufrido herida de bala en el abdomen, se le ha tenido que extirpar el segmento medio y
distal del ileon. En este caso la síntesis hepática de sales biliares estará …..…..
(Unidad 3, sesión 22, logro 4: Explica la formación, función y Circulación entero hepática de lasa sales
biliares)
a. Disminuida por inhibición de la colesterol 7 alfa hidroxilasa
b. Incrementada por estímulo de la enzima colesterol 7 alfa hidroxilasa
c. Incrementada por inhibición de la colesterol 7 alfa hidroxilasa
d. Sin cambios en el ritmo de síntesis

37. Un varón de 75 años ingresa al consultorio por presentar ictericia marcada de piel y las escleras. El
estudio del paciente mostró que presentaba un tumor que obstruía la totalidad del conducto
hepático común. ¿Cuál de las siguientes estructuras se encontrará dilatada en este paciente?
(Unidad 3, sesión 21, logro 6: Describir el árbol biliar intrahepático)
a. Conducto de Wirsung
b. Conductos de Hering
c. Conducto colédoco
d. Conducto cístico

38. Correlaciones las dos columnas y marque la fórmula correcta:


(Unidad 4, sesión 28, logro 1: Diarrea: definición, mecanismos: osmótica, secretoria y exudativa)
1. Enfermedad Hirschsprung ( ) heces con moco y sangre
2. Diarrea osmótica ( ) intolerancia a lactosa
3. Diarrea secretoria ( ) aganglionosis congénita
4. Diarrea exudativa ( ) canales de Cl- en las células de la cripta

a.- 4231 b.- 1234 c.- 2143 d.- 4213

39. Respecto a la siguiente imagen que representa una estructura de la mucosa gástrica, la estructura con
número ………..
produce ……………………..
(Unidad 3, sesión 19, logro 4: La glándula fúndica. Funciones y tipos de células con sus características)
a. 3 / pepsina
b. 1 / Pepsinógeno
c. 4 / HCl y factor extrínseco
d. 2 / pepsinógeno

2
40. En un paciente con insuficiencia renal crónica, el déficit en la absorción de calcio a nivel del
enterocito se debe a lo siguiente:
(Unidad 4, sesión 26, logro 6: Explica la Absorción de calcio y hierro)
a. No se convierte la 25 hidroxicolecalciferol a 1,25 dihidroxicolecalciferol
b. No se convierte la 1,25 dihidroxicolecalciferol a 25 hidroxicolecalciferol
c. Se incrementa la producción de Calbindina
d. Existe un descenso de la alfa 25 hidroxilasa renal
CLAVES
EXAMEN
PARCIAL DE
SISTEMA
DIGESTIVO
2019 - 00

1. Varón de 30 años es traído a emergencia por agresión abdominal con arma de fuego
(pistola) y es sometido a laparotomía exploratoria, observándose isquemia del colon
ascendente y parte del colon trasverso ¿la lesión de cuál de las siguientes arterias
explicaría esta isquemia?
(unidad 1, sesión 2, logro 6: (Describe la irrigación visceral: arterias de tronco
celiaco, arteria mesentérica superior e inferior, topografía de superficie, órganos
por cuadrante)

a. Celiaca
b. Colónica derecha
c. Mesentérica inferior
d. Mesentérica superior

2. Respecto a las sustancias gastrointestinales que regulan la secreción pancreática;


marque la afirmación correcta:
(unidad 1, sesión 3, logros 2 y 3: describir las hormonas gastrointestinales: estímulos y
funciones)

a. La Secretina, es la hormona más importante para la secreción de bicarbonato por


las células acinares del páncreas
b. La acetilcolina es capaz de estimular la secreción enzimática y de bicarbonato del páncreas
c. La gastrina, es la hormona más importante para la secreción de enzimas pancreáticas
d. La colecistoquinina (CCK) estimula al páncreas solo para secreción enzimática

3. Ante una lesión del X par craneal, ¿cuál de los siguientes músculos mantiene conservada su
función?:
(unidad 2, sesión 08, logro 4: Paladar blando: componentes musculares)

a. Elevador del velo del paladar


b. Tensor del velo del paladar
c. Palatofaríngeo
d. Glosofaríngeo

4. Experimentalmente se utiliza atropina (anticolinérgico) para inhibir la secreción de


gastrina, sin embargo la secreción de esta hormona se sigue dando ante estímulos
vagales. Esta situación se explica porque la atropina:
(unidad 1, sesión 3, logro 3 : describir las hormonas gastrointestinales: estímulo y funciones
de la gastrina y colecistoquinina)

a. Bloquea parcialmente la bomba de protones en la célula G


b. Inhibe la acción de acetilcolina e histamina en la célula G
c. Solo inhibe la acción del péptido GRP en la célula G
d. No bloquea la acción del péptido GRP

5. Un varón de 50 años es sometido a extirpación del duodeno y parte proximal


del yeyuno. Esta situación ocasionaría la pérdida de las células ……….. ,
productoras de que
estimula la secreción de
bicarbonato por el páncreas.
(unidad 1, sesión 3, logro 3: describir las hormonas gastrointestianles: estímulos y
funciones de la secretina y péptido insulinotrópico dependiente de glucosa)
a. “S” / secretina
b. Parietales / secretina
c. “I” / colecistoquinina
d. “S” / colecistoquinina
6. Recién nacido que presenta tumoración abdominal a nivel del cordón umbilical
(fotografía). ¿cuál de las siguientes afirmaciones es correcta respecto a este
defecto en el desarrollo embriológico del intestino?: (unidad 1, sesión 5, logro 2:
identificar las anomalías del desarrollo del intestino medio)

a. Corresponde a una Gastrosquisis


b. Las vísceras se hallan cubiertas por piel
c. No está asociado a otras malformaciones
d. Se asocia a
malformaciones
cardiacas y del tubo
neural

7. Varón de 35 años acude a la emergencia por


trauma abdominal y se decida realizar una
laparoscopía exploratoria. El cirujano
observa la disposición de los órganos
abdominales como se representa en el
siguiente esquema. Esta disposición de
órganos se explica por la rotación
(SMA=arteria mesentérica superior)
(unidad 1, sesión 5, logro 3: identificar las anomalías
del desarrollo del intestino medio: defectos de
rotación, estenosis y atresias)

a. anti horaria del intestino medio, en sólo 90°


b. incompleta del intestino medio (270°)
c. horaria del intestino medio
d. horaria del estómago

8. Se evalúa la expresión de la proteína Agrp en una persona con alteración del


apetito; lo correcto respecto a esta proteína es…..
(unidad 1, sesión 3, logro 4: Explica los mecanismos de control del apetito y saciedad )

a. Esta proteína es un potente anorexigénico


b. La mutación del gen que la codifica produce adelgazamiento
c. La sobre producción de la proteína lleva a obesidad por agonismo de receptores MC3 y MC4
d. La sobre producción de la proteína disminuye el apetito por antagonismo de receptores MC4

9. Juana cae de la bicicleta y se fractura la región anterior del hueso maxilar superior con
compromiso de la fosa incisiva. Al examen físico de la región esperaría encontrar
alteración en la sensibilidad de la encía …………………
(unidad 2, sesión 8, logro5: paladar: paladar duro y blando: irrigación e inervación)

a. bucal posterior
b. Lingual anterior
c. palatina anterior
d. palatina posterior
10. Recién nacido es atendido por el neonatólogo y luego entregado a su madre para
dar de lactar; la madre al dar de lactar observa coloración azulada de labios,
acompañado de tos persistente, dificultad respiratoria y distención abdominal. Se le
intenta colocar una sonda nasogástrica pero esta retorna a la cavidad oral en todos
los intentos. ¿Cuál de las siguientes anomalías del desarrollo es el más probable en
este caso? (unidad 1, sesión 4, logro 3: identificar las anomalías en el desarrollo del
esófago: atresia y/o fístula traqueo esofágica)

a. Estenosis esofágica proximal con Fístula traqueo esofágica distal


b. Atresia esofágica proximal con fístula traqueoesofágica distal
c. Atresia esofágica distal con fístula traqueoesofágica proximal
d. Fístula traqueoesofágica proximal y distal

11. ¿Cuál de los siguientes mecanismos ocurre durante la defecación?


(unidad 2, sesión 13, logro 6: motilidad del intestino grueso: contracciones
segmentarias, movimientos en masa, defecación y reflejo gastrocólico)

a. Contracción refleja del esfínter anal interno


b. En la posición de “cuclillas” el músculo puborectal se halla relajado
c. Relajación del esfínter anal externo por efectos del VIP y óxido nítrico
d. La materia fecal en el recto estimula la contracción del sigmoides por los nervios pudendos

12. La estructura número 4 (gráfico) corresponde a


……….… y está ………..
(unidad 2, sesión 9, logro 2:
Partes de un diente. Capas
del diente: Esmalte:
características y células que
lo producen)

a. el cemento / mineralizado en 90%


b. la dentina / formada por ameloblastos
c. el esmalte / formado
por células derivadas
del mesénquima
d. la dentina / formado por
células derivadas de la
cresta neural

13. Un paciente luego de un accidente sufre lesión del piso de la boca, se constata
daño del nervio “cuerda del tímpano”, en este caso se esperaría encontrar
disminución de la de la
lengua
(unidad 2, sesión 10, logro 3: Irrigación e inervación de la lengua)

a. Motilidad en los dos tercios anteriores


b. Sensación del gusto en el tercio posterior
c. Sensación del gusto en los dos tercios anteriores
d. Sensibilidad al tacto en los dos tercios anteriores

14. ¿Cuál de las siguientes afirmaciones es la correcta sobre la gastrina?


(unidad 1, sesión 3, logro 1: reconocer las características de las sustancias
reguladoras gastrointestinales: hormonas, sustancias paracrinas y neurocrinas)

a. Produce atrofia de la mucosa gástrica


b. Es producida por la célula G del cuerpo gástrico
c. Es estimulada por la distensión gástrica y el Ph bajo
d. Actúa en la célula diana mediante su receptor CCk tipo B
15. Al recibir un paciente con signos de hipovolemia y antecedente de trauma en
abdomen por accidente de tránsito, usted identifica radiológicamente: lesión de
primera vértebra lumbar y signos de lesión en páncreas; durante la cirugía se observó
pobre irrigación de asas intestinales. El vaso afectado es la arteria ……..
(unidad 1, sesión 1, logro 6: reconocer las estructuras a nivel de L1, nivel de los principales
vasos sanguíneos)

a. esplénica
b. hepática común
c. mesentérica inferior
d. mesentérica superior

16. Un paciente sufre de daño a nivel del cuello con lesión muscular en la región de la
faringe. En el examen físico se determina dificultad para la elevación de la faringe y
para el cierre del itsmo de las fauces. En este caso, probablemente esté afectado el
músculo:
(unidad 2, sesión 11, logro 2: Músculos de la faringe: identificación, constrictores y
longitudinales)

a. palatogloso
b. estilofarinfeo
c. palatofaringeo
d. constrictor inferior

17. Varón de 50 años a quien le realizan la curación de la segunda molar de la arcada


superior derecha. En un momento determinado, el paciente acusa de intenso
dolor de la pieza dentaria en tratamiento. La vía aferente del dolor viaja a través
del nervio …………
(unidad 2, sesión 9, logro 6: Inervación de los dientes)

a. trigémino V2
b. trigémino V3
c. naso palatino
d. palatino menor

18. La distención gástrica por los alimentos produce incremento de secreción de HCl mediante la
producción de
………….. que estimula a las células ……………. vía proteína ………..
(Unidad 1, sesión 3, logro 2: Describe las hormonas gastrointestinales: Estímulo y funciones de
la gastrina y colecistoquinina)

a. gastrina / parietal / Gq
b. gastrina / principal / Gs
c. acetilcolina / parietal /Gi
d. acetilcolina / principal / Gi

19. Un niño de tres años llega a emergencia con disfagia (dificultad para tragar), dolor
retro esternal, salivación y llanto. Se sospecha de ingesta de cuerpo extraño
(moneda) en el esófago; al ser evaluado se constata en una radiografía presencia de
cuerpo extraño a nivel de C6 (6° vértebra cervical). El cuerpo extraño estará
suspendido a nivel del estrechamiento producido por………..
(unidad 2, sesión 11, logro4: Esófago, características anatómicas, relación con órganos
vecinos y estrecheces)

a. el cayado aórtico
b. el hiato esofágico
c. el músculo cricofaríngeo
d. el bronquio principal izquierdo

20. La triada portal (arteria hepática, vena portal y conducto biliar común) está contenida en el
ligamento
…….……… y derivan embriológicamente del ……
(Unidad 1, sesión 1, logro 4: Identifica el peritoneo, mesenterio, omento y ligamentos,
retroperitoneo.)

a. hepato duodenal / mesenterio ventral


b. gastro esplénico / mesenterio dorsal
c. hepato gástrico / omento menor
d. falciforme / omento menor
21. En relación al movimiento de
peristaltismo del tubo
digestivo: en la flecha negra
del gráfico se produce la
liberación de ……………… a
nivel del músculo ………..
(unidad 2, sesión 7, logro 6:
Control hormonal y tipos de
movimiento)

a. noradrenalina, sustancia P y neuropéptido “ Y” / circular


b. acetilcolina y sustancia P / longitudinal
c. óxido nítrico y PIV / longitudinal
d. óxido nítrico y PIV / circular

22. Un paciente refiere no percibir algunos sabores, al examen físico se constata


alteración en la percepción de sabores y del dolor en el tercio posterior de la lengua
¿Cuál de los siguientes nervios estará alterada en su función?
(unidad 2, sesión 10, logro 5: Sabores, tipos y mecanismos moleculares para su detección)

a. Lingual (rama del V par)


b. Cuerda del tímpano (VII par)
c. Glosofaríngeo (IX par)
d. Hipogloso (XII par)

23. El gráfico detalla la


estructura de la pared del
tubo digestivo intestinal
¿Cuál de las siguientes
asociaciones es correcta?
(unidad 2, sesión 7, logro 1:
La pared y músculo liso
gastrointestinal )

a. “1” – peristaltismo
b. “2” – secreción enzimática
c. “3” – deriva del mesodermo
d. “4” – doble hoja de tejido graso

24. En el caso de un paciente con gastrinoma (tumor productor de gastrina), la presencia


de úlceras duodenales y erosión de la mucosa gástrica, se debe principalmente a…….
(unidad 1, sesión 3, logro 2: describir las hormonas gastrointestinales: estímulo y funciones de
la gastrina y colecistoquinina)

a. la acción directa de la gastrina sobre la célula principal


b. la sobre expresión de los receptores “G” en la célula parietal
c. el exceso de HCl por estímulo de receptores CCK-B en la célula parietal
d. el exceso de HCl por estímulo directo de receptores de acetilcolina en la célula parietal

25. El reflejo entero gástrico se caracterizan por:


(unidad 2, sesión 13, logro 6: Motilidad del intestino grueso: contracciones
segmentarias, movimientos en masa defecación y reflejo gastrocólico)

a. favorecer la motilidad gástrica gracias a la CCk


b. inhibir la motilidad gástrica y estimular la secreción ácida
c. movilizar grandes volúmenes desde el estómago al duodeno
d. originarse debido a la distensión duodenal y presencia del quimo ácido
26. Mauricio tiene dificultad para deprimir el paladar y elevar la parte posterior de la
lengua. En este caso estará afectado un músculo ………………., específicamente el
músculo …………….
(Unidad 2, sesión 10, logro 2: Músculos de la lengua: clasificación, identificación y sus
funciones)

a. intrínseco – longitudinal inferior


b. extrínseco – palatogloso
c. extrínseco – transverso
d. extrínseco – estilogloso

27. Una de las funciones del músculo señalado es:


(Unidad 2, sesión 8, logro 3: Describir el Piso de la boca:
estructuras blandas que la conforman)

a. deprimir la lengua
b. elevar el paladar blando
c. deprimir el hioides cuando la mandíbula está fija
d. deprimir la mandíbula cuando el hioides está fijo

28. Paciente varón de 30 años es evaluado por probable enfermedad de Chagas, cursa
con problemas de motilidad del colon; los estudios de biopsia determinan
ausencia de células ganglionares. Según el gráfico
¿cuál es la capa en la que se determina la ausencia de dichas células?
(unidad 1, sesión 2, logro 1: describir las generalidades de la estructura del tubo digestivo:
esófago, estómago intestino delgado y grueso)

a. Mucosa - 1
b. Muscular propia – 1
c. Muscular de la mucosa - 2
d. Muscular propia - 3

1 2

3
29. Paciente varón de 32 años, que acude a centro de salud por presentar de forma
progresiva desde hace 1 año dificultad para ingerir alimentos sólidos y luego líquidos;
refiere regurgitaciones alimentarias y marcada pérdida de peso (15 kilos). Radiografía
baritada (sustancia de contraste) de esófago se muestra en la figura. El presente caso
se explica por……………….
(Unidad 2, sesión 12, logro 4: Identificar y describir la función de los esfínteres esofágicos)

a. aumento de la peristalsis esofágica


b. relajación incompleta del esfínter pilórico
c. relajación incompleta del esfínter esofágico inferior
d. perdida de producción de PIV y
óxido nítrico en el esfínter
esofágico superior

30. En condiciones normales, el ingreso de 600 ml de líquido es el estómago provoca un


aumento de presión intragástrica de unos 12 cm de H2O. Después de una vagotomía
(corte del nervio vago) es de esperar que el ingreso del mismo volumen de líquido
ocasione de la presión
intragástrica.
(Unidad 2, sesión 13, logro 1: Describe la Motilidad gástrica: relajación receptiva)

a. la disminución
b. la no variación
c. un aumento mayor
d. un aumento similar o igual
SISTEM
A
DIGEST
IVO
(ME
154)
CLAVES
EXAME
N
FINAL
Ciclo
201900

1. Un niño de 2 años es llevado a la consulta por diarrea persistente, edema de las


extremidades y falta de crecimiento en relación a su edad. Los análisis de sangre revelan que
tiene concentración plasmática baja de proteínas (hipoproteinemia). Como parte del estudio
se coloca colecistokinina (CCK) endovenosa y se recoge muestras del líquido duodenal por
endoscopía; el resultado del líquido confirma incapacidad para hidrolizar proteínas a un pH
neutro, esta situación mejora al añadir una pequeña cantidad de tripsina. El paciente
probablemente esté sufriendo la falta congénita de ………….
(Unidad 4, sesión 26, logro 2: Explicar la Digestión y absorción de las proteínas y sus alteraciones)
a. PEPT-1
b. pepsinógeno
c. enterocinasa
d. carboxipeptidasas

2. Paciente mujer de 35 años acude a consulta por sensación de sequedad y lesiones en


cavidad oral. Al examen se observa atrofia de la mucosa, fisuras y úlceras; nota además
sequedad e irritación de la córnea y aumento del tamaño de las glándulas parotídas. Su
diagnóstico más probable es artritis reumatoide; el hallazgo más probable en una biopsia
de glándula parótida es……..….
(Unidad 3, sesión 18, logro 3: Describe las enfermedades más frecuentes de las glándulas
salivales)
a. Presencia de acinos normales con hiperplasia de células ductales
b. Gran infiltración de linfocitos y células plasmáticas
c. Hiperplasia de acinos glandulares serosos
d. Gran infiltrado de linfocitos y neutrófilos

3. Un hombre de 42 años de edad se presenta al médico con una historia de 1 año de


evolución, caracterizado por dolor abdominal bajo y diarreas con crisis sanguinolentas.
Manifiesta además pérdida de peso de 8 kg durante este periodo. La colonoscopía revela
lesión difusa en el colon con afectación del recto. La biopsia de estas lesiones revela
adelgazamiento de la pared, inflamación y ulceración de la mucosa y sub mucosa. El
diagnóstico más probable en este caso es:
(Unidad 4, sesión 28, logro 5: Describe la Enfermedad inflamatoria intestinal.
Generalidades, morfología y características)
a. sindrome de colon irritable
b. enfermedad de Crohn
c. colitis ulcerativa
d. sprue celiaco

4. Dos estudiantes deciden tomar un receso para comer una hamburguesa a la hora del
almuerzo. Antes de llegar a la cafetería, impulsos nerviosos provenientes del complejo
vagal dorsal iniciarán la secreción de ácido gástrico por la liberación de desde el
sistema nervioso entérico.
(Unidad 3, sesión 20, logro 2: Regulación de la secreción gástrica: estimulación, fases de la
secreción)
a. Serotonina
b. Colecistoquinina
c. Péptido inhibidor vaso activo
d. GRP (péptido liberador de gastrina)
5. Un niño de cuatro años de edad es llevado a la consulta por cuadros diarreicos
frecuentes caracterizados por heces pálidas, voluminosas y fétidas; al examen físico
presenta bajo peso y talla para la edad. Se mide la concentración de cloruro en el
sudor y se encuentra que sus valores son muy elevados. La alteración más
importante a nivel de células ductales del páncreas tiene relación directa con la conductancia
de…………
(Unidad 3, sesión 23, logro 5 Explica la Secreción pancreática: formación del jugo
pancreático, influencia de la velocidad de flujo y regulación)
a. Bicarbonato
b. Potasio
c. Sodio
d. Cloro
6. Se evalúa los valores séricos de las siguientes sustancias a un paciente con enfermedad
hepática terminal; en este paciente se espera encontrar la combinación con la letra …………
(Unidad 3, sesión 22, logro 5: Describe las Pruebas de función hepática, la Insuficiencia
hepática, encefalopatía hepática e hipertensión portal)
Glucosa Amoniaco Albúmin
a
a. Aumenta Disminuida Disminui
da da
b. Disminui Aumentada Aumenta
da da
c. Aumenta Aumentada Aumenta
da da
d. Disminui Aumentada Disminui
da da

7. Una mujer de 35 años de edad HIV positiva, se presenta al médico con dolor abdominal
en cuadrante superior derecho e ictericia. La paciente refiere haber tenido múltiples
episodios de ictericia durante los últimos 10 años. Los exámenes para determinar
hepatitis viral, dieron positivos para Hepatitis B, siendo catalogado el caso como
hepatitis crónica con alteración funcional. En un examen de sangre ¿cuál de los
siguientes parámetros está disminuido?
(unidad 3, sesión 22, logro 5: Pruebas de función hepática, Insuficiencia hepática,
encefalopatía hepática e hipertensión portal)
a. Albumina
b. Bilirrubina
c. Fosfatasa alcalina
d. Tiempo de protrombina

8. En el reflejo peristáltico del intestino delgado ¿Cuál de los siguientes eventos sucede en la
porción caudal del bolo alimenticio?
(Unidad 2, sesión 13, logro 4: Explicar la Motilidad del intestino delgado: Contracciones
segmentarias y peristálticas)
a. Acción del péptido inhibidor vasoactivo (VIP) en el músculo circular
b. Acción del NO (óxido nítrico) en el músculo longitudinal
c. Contracción del músculo longitudinal interno
d. Acción de acetilcolina en el músculo circular

9. Un varón de 58 años de edad con enfermedad de Crohn severo fue sometido a una
resección ileal. Después de la cirugía este paciente padecerá de esteatorrea, esto se
explica porque …..………..
(unidad 4, sesión 26, logro 4: Explica las alteraciones en la Absorción de lípidos)
a. se inhibe la acción de la 7 alfa hidroxilasa
b. el pool de ácidos biliares se incrementa
c. hay mala absorción de ácidos biliares
d. el páncreas no secreta lipasa

10. En un experimento se inserta un balón en el estómago de un voluntario, se infla poco


a poco mientras que se vigilan las presiones intraluminales. Aunque el volumen del
balón aumenta considerablemente, las presiones permanecen constantes. Esta
relación volumen-presión se explica por la liberación local de …………..
(Unidad 2, sesión 13, logro 1 Explica la Motilidad gástrica: relajación receptiva)
a. acetil colina y gastrina
b. norepinefrina y óxido nítrico
c. colecistoquinina y óxido nítrico
d. óxido nítrico y péptido inhibidor vasoactivo

11. ¿Cuál de las siguientes alternativas es una característica de la secreción exocrina del páncreas?
(Unidad 3, sesión 23, logro 5: Secreción pancreática: formación del jugo pancreático,
influencia de la velocidad de flujo y regulación)
a. Es hipotónica respecto al plasma
b. Su mayor estímulo se da en la fase intestinal
c. Es estimulada por la presencia de bicarbonato en el duodeno
d. La secreción enzimática es estimulada principalmente por la secretina
12. Las estructuras en el hígado que permite que los productos metabólicos unidos a
proteínas tengan acceso a las membranas basolaterales de los hepatocitos, son…..
(Unidad 3, sesión 21, logro 4-5: Explica la Organización micro estructural del hígado)
a. los canalículos
b. las células de Ito
c. las fenestras sinusoidales
d. las uniones intercelulares herméticas

13. La composición de la bilis es modificada conforme fluye por los conductillos biliares.
Durante este tránsito se espera que aumente la concentración de…….
(Unidad 3, sesión 22, logro 2: Describe la Secreción biliar, visión general del sistema biliar
extrahepático y composición de la bilis)
a. Ig A
b. Glucosa
c. Protones
d. Vitamina A

14. Se mide experimentalmente el contenido gástrico de dos personas. La persona “A” tiene
alto contenido de grasa y la persona “B” tiene un contenido isotónico ¿Cuál de las
siguientes es correcto respecto al vaciamiento gástrico? (Unidad 2, sesión 13, logro 2:
Describe la Motilidad y vaciamiento gástrico)
a. Hay ralentización del vaciado gástrico solo en “A”
b. El vaciamiento gástrico es más rápido en ambos
c. Hay ralentización del vaciado gástrico solo en “B”
d. Hay ralentización del vaciado gástrico en ambos casos

15. El examen endoscópico de un paciente con hipertensión portal grave revela venas
tortuosas que sobresalen hacia la luz del esófago. El paciente recibe tratamiento
quirúrgico mediante la colocación de una derivación que conecta la vena porta a la vena
cava. Después de la operación el riesgo de
encefalopatía
y el riesgo
de sangrado de várices ……………..
(Unidad 3, sesión 22, logro 5: Describe la Insuficiencia hepática, encefalopatía hepática e
hipertensión portal)
a. disminuirá / disminuirá
b. disminuirá / aumentará
c. aumentará / disminuirá
d. aumentará / aumentará

16. Un paciente varón de 18 años de edad acude al médico para sus exámenes de rutina. Sus
resultados de laboratorio muestran un valor de bilirrubina sérica de 4 mg/dl y una
bilirrubina directa de 0,3 mg/dl. Las pruebas de función hepática son normales. La
alteración que explica mejor este caso es por la deficiencia de ………………..
(Unidad 3, sesión 22, logro 3: Explica la Producción y excreción de bilirrubina. Tipos de bilirrubina e
ictericia)
a. transaminasas
b. hemo oxigenasa
c. la 7 alfa hidroxilasa
d. glucuronil transferasa

17. Un hombre de 57 años de edad es llevado a urgencias con hematemesis masiva rojo
brillante, a su llegada se halla inconsciente con PA: 80/40 mm Hg y FC: 124 lat/min. Luce
ictérico con presencia de “arañas vasculares en el tórax anterior y extremidades”,
abdomen distendido con signo de oleada positiva. Se encuentra esplenomegalia y pérdida
de la masa muscular en extremidades. La anastomosis vascular responsable del sangrado
en este paciente es ………….…..
(Unidad 3, sesión 21, logro 2: Describe las anastomosis porto sistémicas)
a. vena gástrica izquierda y vena ácigos
b. arteria gástrica izquierda y vena ácigos
c. vena paraumbilical y vena epigástrica inferior
d. vena gástrica izquierda y vena esofágica superior
18. Un estudiante de medicina está comiendo un plato de comida a base de champiñones,
espárrago y salsa de soya. El estímulo del sabor umami contenido en todos estos
alimentos viaja a través del nervio………………..
(Unidad 2, sesión 10, logro 3: Describe la irrigación e inervación de la lengua)
a. Lingual
b. Hipogloso
c. Glosofaringeo
d. Cuerda del tímpano
19. Una paciente de 30 años de edad es sometida a una cirugía en oído medio derecho
por un problema de otoesclerosis. Luego de la cirugía refiere alteración sensitiva de
la lengua. Al evaluar el caso usted esperaría encontrar……….
(Unidad 2, sesión 10, logro 5: Describe la Irrigación e inervación de la lengua)
a. Alteración en la sensación del dolor y temperatura en el tercio posterior de la lengua
b. Alteración en la sensación del dolor en los dos tercios anteriores de la lengua
c. Alteración en la sensación del gusto en el tercio posterior de la lengua
d. Sensación del dolor, tacto y temperatura conservadas

20. En un paciente de 45 años de edad con colestasis biliar, se encuentra una elevación de los
niveles sanguíneos de fosfatasa alcalina hasta 3 veces la cifra normal. ¿Cuál de las
siguientes alternativas estará también elevada como evidencia del daño de la vía biliar?
(Unidad 3, sesión 22, logro 5: Pruebas de función hepática, Insuficiencia hepática,
encefalopatía hepática e hipertensión portal)
a. Tiempo de protrombina y albúmina sérica
b. Transaminasas hepáticas (ALT y AST)
c. Gamma glutamil transpeptidasa
d. Glucoronil transferasa

21. Experimentalmente se incrementa la velocidad de la secreción salival con una


sustancia, en el análisis de la composición de esta saliva obtenida se espera
encontrar…………..
(Unidad 3, sesión 17, logro 5 : Explica la Influencia de la velocidad del flujo salival en la
composición de la saliva)
a. disminución de la concentración de bicarbonato que supera la concentración plasmática
b. aumento de la concentración de cloro y sodio que supera la concentración plasmática
c. aumento de la concentración de bicarbonato que supera la concentración plasmática
d. disminución de concentración de potasio y bicarbonato

22. Lactante de 3 meses de vida es atendido por presentar diarrea, se administra una
solución de glucosa y electrólitos por vía oral. La proteína de membrana apical
que explica la capacidad de esta solución para proporcionar aporte de glucosa e
hidratación es ………..
(Unidad 4, sesión 26, logro 1: Explica la Digestión y Absorción de los hidratos de carbono.
Alteraciones)
a. CFTR
b. SGLT-1
c. GLUT-2
d. GLUT-5

23. Paciente ha sufrido herida de bala en el abdomen, se le ha tenido que extirpar el


segmento medio y distal del ileon. En este caso la síntesis hepática de sales biliares
estará …..…..
(Unidad 3, sesión 22, logro 4: Explica la formación, función y Circulación entero hepática de lasa
sales biliares)
a. Sin cambios en el ritmo de síntesis
b. Disminuida por inhibición de la enzima colesterol 7 alfa hidroxilasa
c. Incrementada por estímulo de la enzima colesterol 7 alfa hidroxilasa
d. Incrementada por inhibición de la enzima colesterol 7 alfa hidroxilasa

24. Un varón de 75 años ingresa al consultorio por presentar ictericia marcada de piel y las
escleras. El estudio del paciente mostró que presentaba un tumor que obstruía la
totalidad del conducto hepático común. ¿Cuál de los siguientes conductos se encontrará
dilatado en este paciente?
(Unidad 3, sesión 21, logro 6: Describir el árbol biliar intrahepático)
a. de Wirsung
b. de Hering
c. colédoco
d. cístico
25. Correlaciones las dos columnas y marque la fórmula correcta:
(Unidad 4, sesión 28, logro 1: Diarrea: definición, mecanismos: osmótica, secretoria y exudativa)

1. Enfermedad Hirschsprung ( ) heces con moco y sangre


2. Diarrea osmótica ( ) intolerancia a lactosa
3. Diarrea secretoria ( ) aganglionosis congénita
4. Diarrea exudativa ( ) canales de Cl- en

las células de la cripta a.- 4231 b.- 1234 c.-

2143 d.- 4213

26. La vena umbilical obliterada del hígado después del nacimiento se transforma en el ligamento:
(Unidad 3, sesión 21, logro 1: Hígado: relación con la pared abdominal, caras, lóbulos, ligamentos
, hilio hepático)
a. cruzado
b. redondo
c. coronario
d. falciforme

27. Llega a su guardia nocturna una madre que trae a su RN masculino de 2 semanas de vida
con mal estado general y sequedad de mucosas. Usted observa que lacta ávidamente, pero
a las 2 horas presenta vómito postprandial no bilioso en proyectil. Al realizar la historia
clínica, descubre que el lactante recibió profilaxis con macrólidos para tos ferina. Usted
sospecha principalmente en:
(Unidad 1, sesión 4, logro 4: Desarrollo y anomalías del intestino anterior)
a. estenosis pilórica hipertrófica congénita
b. fistula traqueo esofágica
c. estenosis duodenal
d. atresia duodenal

28. En la regulación del apetito y la saciedad, la estimulación experimental crónica del


núcleo ventro medial del hipotálamo producirá:
(Unidad 1, sesión 3, logro 4: explica los mecanismos de control del apetito y saciedad)
a. afagia
b. obesidad
c. hiperfagia
d. activación de neuronas relacionadas a NPY

29. Paciente mujer de 25 años acude por dolor en fosa ilíaca derecha que empeora al toser o
caminar, asociada a náuseas y vómitos por lo cual acude a emergencia. Dos días después
de realizarle una apendicectomía, la paciente desarrolla fiebre alta (39 °C), está hipotensa y
presenta dolor abdominal. La laparotomía exploratoria muestra un gran volumen de
sangre en la cavidad peritoneal por lesión de un vaso producida durante la
apendicectomía.
¿Cuál de las siguientes arterias debe ligarse para detener la hemorragia?
(Unidad 4, sesión 27, logro 4: Irrigación arterial del colon, recto y conducto anal)
a. cólica derecha y arteria rectal superior.
b. ileocólica y arteria cólica media.
c. mesentérica superior.
d. ileocólica.

30. La onda peristáltica secundaria del esófago se caracteriza por ser originada ………
(unidad 2, sesión 12, logro 3: Motilidad esofágica: fases y características)
a. por el plexo de meissner del esófago
b. por el plexo mientérico del esófago
c. por el reflejo de la deglución
d. durante la masticación

31. ¿Cuál de los siguientes es una causa de ictericia con bilirrubina conjugada aumentada?
(Unidad 3, sesión 22, logro 3: Producción y excreción de bilirrubina. Tipos de bilirrubina, ictericia)
a. Ictericia del recién nacido
b. Obstrucción del colédoco
c. Anemia hemolítica
d. Gran hematoma
32. En relación a la absorción de nutrientes, la absorción de dipéptidos y tripéptidos a nivel
de las células epiteliales del intestino delgado, se da principalmente debido a:
(Unidad 4, sesión 26, logro 2: Digestión y absorción de las proteínas. Alteraciones)
a. el incremento de los canales de Cl- en la membrana apical
b. la gradiente de bicarbonato en la membrana basal
c. la gradiente de iones H+ en la membrana apical
d. la gradiente de Na+ en la membrana apical

33. Paciente de 20 años es traído a la emergencia por presentar diarreas desde hace 2 días.
Familiar refiere que las deposiciones son líquidas y abundantes, al examen luce
deshidratado y se plantea que la diarrea es producida por una toxina que estimula la
transformación de ATP a AMPc con apertura de canales de Cl- y pérdida de agua. El tipo de
diarrea más probable es:
(Unidad 4, sesión 28, logro 1: Diarrea: definición , mecanismos: osmótica, secretoria y supurativa)
a. osmótica
b. exudativa
c. secretoria
d. por intolerancia a lactosa

34. Un niño fue operado por una obstrucción intestinal, observándose la presencia de
divertículo de Meckel. Según lo referido, marque lo correcto:
(Unidad 1, sesión 5, logro 2: identifica las anomalías del desarrollo del intestino medio: onfalocele
y gastrosquisis (diferencias), Divertículo de Meckel)
a. el 50% de la población lo presenta
b. se localiza en el íleon muy cerca al yeyuno
c. puede poseer tejido gástrico o pancreático
d. se produce por una mala rotación de los intestinos

35. Marque la
alternativa correcta
respecto a la
estructura marcada
en el gráfico:
(Unidad 3, sesión 22,
logro 2: Secreción biliar.
Visión general del
sistema biliar
extrahepático y
composición de la bilis)

a. Se halla a 2
centímetros
debajo de la
papila
duodenal
mayor
b. Llega el conducto
colédoco y
pancreático
principal
c. Llega el
conducto
hepático
común y
pancreático
principal
d. Llega el conducto pancreático accesorio

36. ¿Cuál de las siguientes moléculas se encontrará aumentada en el citoplasma de las


células parietales de un paciente con sindrome de Zollinguer Ellison?
(Unidad 3, sesión 20, logro 4: Enfermedad ulcerosa péptica: úlcera gástrica, duodenal.
síndrome de Zollinger – Ellison)
a. Péptido liberador de gastrina (GRP)
b. Proteína G estimulante (GS)
c. Inositol Trifosfato (IP3)
d. AMP cíclico (AMPc)

37. Los fármacos inhibidores de la bomba de protones, actúan bloqueando la ………..……..


(Unidad 3, sesión 20, logro 3: Regulación de la secreción gástrica: inhibición, Secreción de
pepsinógeno y factor intrínseco)
a. anhidrasa carbónica
b. ATPasa H+/K+ en la membrana luminal
c. ATPasa H+/K+ en la membrana basolateral
d. ATPasa Na+/K+ en la membrana basolateral
38. Un paciente fue diagnosticado de gastritis autoinmune, ¿cuál de las siguientes
alternativas es FALSA respecto a esta enfermedad?
(Unidad 3, sesión 20, logro 5: Gastritis crónica: helicobacter pylori, autoinmune. Tipos de gastritis)
a. Afecta principalmente el fondo y cuerpo gástrico
b. Se produce hiperplasia de células G secundaria a la aclorhidria
c. El propio sistema inmune destruye principalmente las células parietales
d. Se produce atrofia de la mucosa, aclorhidria, hipergastrinemia y déficit de vitamina B6

39. Marque la correlación correcta:


(Unidad 3, sesión:18, logros:1 y 2: Describe las enfermedades inflamatorias/infecciosas y
proliferativas de la cavidad oral)

1. Herpes virus ( ) En relación al abuso de antibióticos


2. Candidiasis oral ( ) Lesiones vesiculares como racimo de uvas
3. Eritroplaquia ( ) Mega esófago
4. Enfermedad de Chagas ( ) Lesión pre cancerígena

a.- 2431 b.- 1234 c.- 4123 d.- 2143

40. En un paciente con insuficiencia renal crónica, el déficit en la absorción de calcio a nivel
del enterocito se debe a lo siguiente:
(Unidad 4, sesión 26, logro 6: Explica la Absorción de calcio y hierro)
a. No se convierte la 25 hidroxicolecalciferol a 1,25 dihidroxicolecalciferol
b. No se convierte la 1,25 dihidroxicolecalciferol a 25 hidroxicolecalciferol
c. Existe un descenso de la alfa 25 hidroxilasa renal
d. Se incrementa la producción de Calbindina
SISTEMA
DIGESTIVO
(ME154)
EXAMEN
PARCIAL 2019 01

1. La lengua está recubierta por epitelio:


a. pseudoestratificado columnar no queratinizado
b. plano estratificado no queratinizado
c. pseudoestratificado columnar ciliado
d. plano estratificado queratinizado

2. El esfínter anal interno tiene musculatura …………… y tiene control ………………….


a. lisa / voluntario
b. lisa / involuntario
c. esquelética / simpático
d. esquelética / parasimpático

3. La arteria aorta proporciona la irrigación al tubo digestivo ¿cuál de las siguientes arterias
proporciona la irrigación al ángulo cólico derecho?
a. Mesentérica superior
b. Mesentérica inferior
c. Frénica inferior
d. Tronco celiaco

4. Paciente de 26 años que le cuenta en su historia clínica que cada vez que almuerza, a los 20
minutos tiene deseo de defecar. Le comenta que su hijo de 1 mes le pasa lo mismo pero más
intenso. Esto se explica por el reflejo
…………………, el cual está en el paciente.
a. colicoileal / normal
b. colicoileal / alterado
c. gastrocolico / normal
d. gastrocolico / alterado
5. La región del estómago que se comunica con el duodeno se denomina:
a. pilórica
b. cardias
c. cuerpo
d. fórnix

6. Acude a consulta un paciente que fue diagnosticado de ulcera péptica 3 días antes. Luego de múltiples
pruebas diagnósticas, se concluye que el paciente presenta un tumor secretor de gastrina ¿Cuál de las
siguientes situaciones estará incrementada?
a. Distención gástrica
b. Inhibición del vaciado gástrico
c. Secreción de ácido clorhídrico (HCl)
d. Inhibición de la secreción de pepsinógeno

7. En el sistema digestivo, el control del apetito esta dado por un complejo sistema de sustancias y
órganos integradores los cuales regulan la ingesta de alimentos. La es una sustancia oroxígena y
es sintetizada
por el ……………………..
a. leptina / intestino
b. grelina / intestino
c. leptina / estómago
d. grelina / estómago

8. Sobre el control autónomo del sistema digestivo, marque la alternativa correcta:


a. La inervación dada por el sistema simpático es de tipo preganglionar.
b. El sistema parasimpático usa como neurotransmisores a la acetilcolina y la noradrenalina.
c. El nervio vago (par craneal X) le da inervación simpática a la mayoría del sistema digestivo.
d. En el sistema simpático, los nervios responsables hacen una primera sinapsis en
ganglios próximos al órgano a inervar.
e. En la inervación de tipo parasimpático, solo interviene el plexo submucoso, sin
embargo, en la de tipo simpático intervienen tanto el submucoso como el mientérico.

9. Con respecto a la actividad eléctrica del sistema digestivo, marque la alternativa correcta
a. Corresponden a potenciales de acción que están presentes de forma continua y le dan la
capacidad de perístasis autónoma al sistema digestivo.
b. La frecuencia de las ondas lentas no se ve influenciada por la actividad neural
ni las hormas gastrointestinales.
c. En el estómago las ondas lentas se dan en una frecuencia de 6 por minuto.
d. Las ondas lentas son cambios lentos y ondulantes del potencial en reposo.
e. La frecuencia de las ondas lentas va de 6 a 12 ondas por minuto.

10. Ante una lesión del IX par craneal, el músculo… se altera en su función.
a. palatogloso
b. estilofaríngeo
c. palatofaríngeo
d. constrictor superior

11. Un varón de 50 años es sometido a extirpación de duodeno y parte proximal de yeyuno. La


pérdida de estímulo hormonal en el páncreas para la secreción enzimática se explica por la
pérdida de las células ……………………
a. Parietales, productoras de factor intrínseco
b. “K” productoras de factor intrínseco
c. “M” productoras de CCK
d. “I” productoras de CCK
12. Respecto al mecanismo de la defecación ¿Cuál de las siguientes afirmaciones es correcta?
a. Se produce contracción refleja del esfínter anal interno
b. Se produce contracción o relajación del esfínter anal externo por señales de la corteza
cerebral
c. La presencia de materia fecal en el recto estimula la contracción del sigmoides por los
nervios pélvicos simpáticos
d. En la posición de “cuclillas” el músculo puborectal se halla contraído favoreciendo la
evacuación de la materia fecal

13. Un niño de tres años llega a emergencia con disfagia (dificultad para tragar), salivación y llanto. Se
sospecha de ingesta de cuerpo extraño: moneda en el esófago; al ser evaluado se constata en una
radiografía presencia de cuerpo extraño a nivel de C6 y C7 (6° y 7° vértebra cervical). El cuerpo extraño
estará suspendido a nivel del estrechamiento producido por el ………..
a. cayado aórtico
b. hiato esofágico
c. músculo cricofaríngeo
d. bronquio principal izquierdo

14. En el caso de un paciente con un tumor productor de gastrina, la presencia de úlceras duodenales y
erosión de la mucosa gástrica se debe principalmente a…….
a. la acción paracrina de la gastrina sobre la célula parietal
b. el exceso de HCl por estímulo de receptores CCK-B en la célula parietal
c. la sobre expresión de los receptores “G” para gastrina en la célula parietal
d. el exceso de HCl por estímulo directo de receptores “H” en la célula parietal

15. La onda peristáltica secundaria del esófago se caracteriza por ser originada ………
a. por el plexo de submucoso del esófago
b. por el plexo mientérico del esófago
c. por el reflejo de la deglución
d. durante la masticación

16. Marque lo correcto sobre las ondas lentas en el tubo digestivo


a. No son despolarizaciones
b. Son potenciales de acción subumbrales
c. Se constituyen de despolarizaciones y repolarizaciones
d. Son rítmicas y generadas por el sistema nerviosos autónomo

17. Recién nacido que presenta protrusión de contenidos abdominales los cuales no están cubiertos
por peritoneo y salen de la cavidad abdominal a través de un defecto de la pared. ¿Cómo se
denomina a la afección que presenta este paciente?
a. Onfalocele
b. Atresia biliar
c. Gastrosquisis
d. Divertículo de Meckel

18. Experimentalmente se utiliza atropina (anticolinérgico) para inhibir la secreción de gastrina, sin
embargo, la secreción de esta hormona se sigue dando ante estímulos vagales. Esta situación se
explica porque la atropina:
a. no bloquea la acción del péptido GRP
b. solo inhibe la acción del péptido GRP en la célula G
c. inhibe la acción de acetilcolina e histamina en la célula G
d. bloquea parcialmente la bomba de protones en la célula G

19. Niña de 4 días es llevada a la emergencia pediátrica por presentar llanto constante, la madre refiere
coloración azulada de labios al momento de lactar, acompañado de tos persistente y dificultad
respiratoria así como distención abdominal. Se le coloca sonda nasogástrica para alimentación
notando que retorna a la cavidad oral en todos los intentos. ¿Cuál es la anomalía del desarrollo en
este caso?
a. Solo fístula traqueo esofágica
b. Fístula traqueo esofágica proximal y distal
c. Atresia esofágica proximal con fístula traqueo esofágica distal
d. Atresia esofágica distal con fístula traqueo esofágica proximal
20. Paciente varón de 36 años es traído a la emergencia luego de sufrir un accidente de tránsito, presenta
traumatismos múltiples en cabeza y tronco. Al examen físico se evidencia hematoma en hemicara
izquierda, ligera protrusión y caída del lado izquierdo del maxilar inferior, por lo que se realiza una
tomografía donde se halla una fractura de la apófisis coronoides del maxilar inferior. ¿Qué músculo
está relacionada directamente con esta situación?
a. Masetero
b. Temporal
c. Buccinador
d. Pterigoideo medial

21. Un paciente refiere no percibir algunos sabores. Al examen físico constata alteración del sabor dulce y
umami.
¿Cuál de los siguientes nervios estará alterada su función?
a. Cuerda del tímpano (VII par)
b. Lingual (rama del V par)
c. Glosofaríngeo (IX par)
d. Hipogloso (XII par)

22. A los pocos días de nacido, regresa a neonatología un niño con problemas de motilidad del colon; los
estudios determinan ausencia congénita de células ganglionares. Según el gráfico ¿cuál es la capa en
la que se determina la ausencia de dichas células?
a. Mucosa - 2
b. Muscular propia - 2
c. Muscular propia - 3
d. Muscular de la mucosa - 3

1 2

23. Con respecto al control autonómico en el tracto gastrointestinal y en relación a su fisiología. ¿Cuál
es la función del sistema nervioso parasimpático en el tracto gastrointestinal?
a. Inhiben la contracción muscular y estimulan la secreción de sustancias a nivel de la
submucosa
b. Estimulan la contracción muscular y estimulan la secreción de sustancias a nivel de la
mucosa
c. Inhiben la contracción muscular e inhiben la secreción de sustancias a nivel de la submucosa
d. Estimulan la contracción muscular e inhiben la secreción de sustancias a nivel de la mucosa

24. Un estudiante que está preocupado por su examen parcial, no ha desayunado ni almorzado; cuando
al fin ingiere alimentos, esto le provoca el aumento de los movimientos musculares del tracto
gastrointestinal y la sensación de defecar. ¿Qué reflejo se ha activado?
a. Entero-gástrico
b. Gastro-cólico
c. Cólico-ileal
d. Ileo-ileal
25. ¿De qué par craneal es rama el nervio palatino mayor?
a. Vago
b. Hipogloso
c. Trigémino
d. Palatogloso

26. ¿En cuál de las fases de la deglución la epiglotis separa la vía respiratoria de la digestiva?
a. oral
b. laríngea
c. faríngea
d. esofágica

27. Los péptidos intestinales se pueden clasificar como sustancias endocrinas, neurocrinas y paracrinas,
dentro de las paracrinas se encuentran la somastotatina e histamina. Marque la respuesta correcta
a. La somastotatina es sintetizada por las células B de la mucosa gástrica
b. La histamina actúa estimulando su receptor tipo H1 en la mucosa gástrica
c. La histamina es sintetizada por células de tipo paracrino de las glándulas gástricas
d. La somatostatina presenta dentro de sus funciones la estimulación de la secreción de H +

28. Paciente varón de 27 años es llevado por bomberos a emergencia luego de ser asaltado y, tras
resistirse, es cortado con el pico de una botella a nivel abdominal. Al examen físico usted observa que
a través de la herida se puede observar la protrusión de asas intestinales. En relación con las capas de
la pared abdominal, marque la alternativa correcta.
a. La fascia de Scarpa está constituida principalmente por tejido adiposo
b. La pared abdominal está formada por piel, huesos, músculos, fascias y peritoneo parietal
c. La fascia de Camper es una estructura fibrosa que carece de grasa y su grosor es
constante en toda la pared abdominal
d. El músculo oblicuo externo discurre en dirección súpero-interna y se inserta en el borde
inferior de las ultimas 3 a 4 costillas
e. El músculo recto del abdomen tiene como funciones comprimir el contenido del
abdomen, tensar la pared del abdomen y flexionar la columna
SISTEMA
DIGESTIVO
(ME154)
EXAMEN FINAL
2019 01

Profesores : Alfaro Salazar, Herberth Romulo; Callata Caceres, Gunter; Cayo Quiñe, Alexandra Mariel;
Correa Borit, Jorge Mauricio; Cruz Cutty, Lourdes Marylin; Guzmán Calderón, Gerly Edson;
Jáuregui Farfán, Jorge Jesús; Mayor Zevallos, Otto Alberto; Montoya Suárez, José Luis;
Palacios Bazan, Enrique Elias; Robles Pino, Alexander Anibal; Wong Bravo, Juan Carlos
Sección : Todas las secciones
Duración : 70 minutos.
Indicaciones:
Lea atentamente cada pregunta antes de responder:
- Se prohíbe el uso del celular y cualquier dispositivo electrónico.
- Está prohibido intercambiar materiales.
- Coloque su código de alumno en la tarjeta de respuestas. Si su código contiene una letra
reemplácela por un valor numérico siguiendo la siguiente equivalencia: A=9, B=8, C=7, D=6, E=5, F=4,
G=3.
- Traslade sus respuestas a la tarjeta, llenando los círculos de manera completa con lapicero negro o
azul. Está prohibido el llenado con lápiz, lapicero de otro color o con lapicero de tinta borrable.
- Sea cuidadoso en el llenado de la tarjeta de respuestas, pues solo esta tiene validez para la
calificación.
- Al terminar su examen avise al docente a cargo, no se levante de su sitio; debe entregar la hoja de
respuestas con la carátula del examen, este cuadernillo de preguntas se lo llevará cada estudiante.

1. La explicación fisiológica de presentar somnolencia de 30 minutos a 1 hora después de ingerir


alimentos, se explica por:
a. Aumento del cloro intraluminal
b. Aumento del bicarbonato intraluminal
c. Disminución de ácido carbónico en la célula parietal
d. Disminución de la actividad de la anhidrasa carbónica
e. Aumento de la alcalinidad sanguínea

2. Con respecto a la irrigación arterial del colon, a que arteria


corresponde la señalada con la flecha
a. Cólica derecha
b. Cólica media
c. Cólica izquierda
d. Ileobisecoapendículocólica
e. Arco de Riolano

3. Si un paciente presentara dentro del punto de vista fisiológico, una


disminución de enterocinasa, entonces esto originaría una
disminución de la actividad de:
a. l
a
p
e
p
s
i
n
a
b
. la lipasa
c. la quimotripsina
d. el peptido insulinotropo dependiente de glucosa
e. la amilasa
Se valida la opción b debido a su relación con la colipasa.
4. Con respecto a la anatomía del hígado, señale a que
estructura pertenece la marcada por el número 1.
a. Ligamento falciforme
b. Línea de Cantlie
c. Ligamento triangular
d. Ligamento coronario
e. Ligamento teres

Se valida la opción a debido a la ubicación del número 1 en


donde se unen el ligamento falciforme y ligamento coronario.

5. Se presenta un paciente, el cual presenta un antecedente de tuberculosis intestinal, por lo cual,


se le resecó 80 cm de íleon distal. Desde el punto de vista fisiológico, el paciente puede presentar
una de las siguientes alteraciones:
a. Disminución de la secreción de Vitamina B12
b. Aumento indiscriminado de absorción de ácido fólico
c. Disminución de la absorción de hierro
d. Aumento de la secreción de bicarbonato
e. Disminución de la absorción de ácido glicocólico

6. Un paciente es sometido experimentalmente a un fármaco que modifica el flujo salival, obteniéndose


un volumen de saliva de 288 ml en 6 horas. En este caso las concentraciones de electrolitos y
bicarbonato en la saliva obtenida varían de la siguiente manera:
a. ↑ Na+, ↓ K+, ↑
Cl-, ↑ HCO3- b.↓
Na+, ↓ Cl-, ↑ K+, ↓
HCO3-
c. ↑ Na+, ↑ Cl-, ↓ K+, ↓ HCO3-
d. ↑ Na+, ↑ Cl-, ↑ K+, ↑ HCO3-
e. ↓ Na+, ↓ Cl-, ↓ K+, ↓ HCO3-

Se valida la opción a debido a que se puede considerar como un aumento del flujo de saliva.

7. La siguiente imagen histológica corresponde a la glándula


…………… y la estructura señalada produce ………
a. salival sublingual / mucopolisacáridos
b. oxíntica / pepsinógeno
c. salival submaxilar / ptialina
d. salival parótida / amilasas
e. antrales / gastrina

8. Paciente varón de 65 años con antecedente de hipercolesterolemia, hipertensión arterial, fibrilación


auricular y dos infartos al miocardio previos, aqueja de dolor abdominal intenso de inicio súbito,
distensión abdominal, se decide cirugía con resección de 1,5 metros de intestino delgado terminal y
colon ascendente. Como consecuencia de la resección el paciente tendrá deficiencia de:
a. Vitamina C
b. Tiamina
c. Vitamina A
d. Vitamina B1
e. Vitamina B6

Se valida esta opción debido a que su absorción está relacionada al íleon.


9. Uno de los siguientes elementos debería hallarse con más probabilidad en el esófago de un paciente
que sufre de
reflujo gastro esofágico…
a. Pepsina
b. Tripsina
c. Quimiotripsina
d. Carboxipeptidasa
e. Ácidos biliares

10. Un paciente de 40 años cursa con anemia de 8g/dl, aqueja además de astenia y sensación de
hormigueo bilateral en los miembros inferiores, al examen se halla alteración de la sensibilidad a la
vibración y camina con ampliación de la base de sustentación. Uno de los siguientes procedimientos
sería de ayuda para el diagnóstico de este paciente:
a. Tomografía cerebral
b. Biopsia de la mucosa gástrica
c. Biopsia de hígado
d. Examen de sangre oculta en heces
e. Biopsia de Ileon proximal

11. Paciente de 60 años ingresa por caída hace 1 hora y pequeño hematoma en cuero cabelludo, al
examen físico ampliado se observa ictericia de piel y mucosas generalizada, abdomen blando, se
palpa estructura quística no dolorosa en hipocondrio derecho que corresponde a vesícula biliar
(signo de Courvoisier), en los exámenes de laboratorio se halla niveles bajos en la formación de
estercobilinógeno y urobilinógeno en heces, incremento de la bilirrubina conjugada en la orina,
elevación de fosfatasa alcalina y gamma glutamil transpeptidasa séricas. El presente cuadro puede
ser explicado por:
a. Reabsorción de hematoma
b. Litiasis vesicular
c. Carcinoma de la cabeza de páncreas
d. Carcinoma con estenosis del conducto hepático común
e. Anemia hemolítica

12. Paciente varón de 58 años con antecedente de alcoholismo crónico es diagnosticado y recibe
tratamiento por cirrosis hepática. Hace 2 días refiere familiar que tuvo cambio de conducta y no
reconoce a algunos familiares. Al examen físico, se halla ascitis, circulación colateral en abdomen,
telangiectasias, en el examen de sistema nervioso: rigidez de extremidades, ROT incrementados,
desorientación en el espacio y asterixis. ¿cuál de las siguientes circunstancias, explicaría el cuadro
en este paciente?
a. Uso de diuréticos ahorradores de potasio
b. Incremento de actividad de ureasa bacteriana duodenal
c. Hemorragia gastrointestinal
d. Disminución de la producción de NH3+ en el colon
e. Dieta normo proteica

Se valida la opción e debido al efecto sobre la encefalopatía.


Con respecto de la opción b es incorrecta debido a que hace referencia al duodeno, debería indicar colon.

13. Un recién nacido presenta vómitos biliosos poco tiempo después de cada alimento. Al preguntar a la
madre sobre antecedentes, ella recuerda que tuvo polihidramnios durante la gestación, pero un
análisis de cariotipo fue normal. Una de las siguientes es la causa más probable de estos hallazgos
en el recién nacido:
a. Enfermedad de Hirschprung
b. Fístula tráqueo esofágica
c. Divertículo ileal
d. Estenosis pilórica
e. Malrotación de la yema pancreática ventral

14. En un estudio de la secreción de hormonas gastrointestinales, sus concentraciones en la vena


porta se midieron durante perfusión luminal del intestino delgado con soluciones de diversas
magnitudes de pH. ¿Qué hormona aumentará en el plasma de la vena porta durante perfusión a
través del intestino con una solución de pH 3?
a. CCK
b. gastrina
c. GIP
d. motilina
e. secretina

15. Paciente de 30 años que ingresa a causa de un traumatismo abdominal cerrado. En la exploración
se aprecia discreta palidez de piel y mucosas, auscultación pulmonar normal, taquicardia de 120
/min. Discreta distensión abdominal y matidez en flancos; el hematocrito, que era prácticamente
normal al ingreso, disminuye a 30% a las tres horas. En la Rx de tórax se objetiva fractura de las
costillas 10-11 izquierdas. La causa más probable de la anemización en este paciente es:
a. traumatismo renal con hemorragia retroperitoneal.
b. rotura de hígado con hemoperitoneo.
c. rotura de bazo con hemoperitoneo.
d. rotura de mesos con hemoperitoneo.
e. traumatismo pancreático con pancreatitis traumática.

16. Mujer de 65 años. Consulta por síndrome constitucional


asociado a dolor abdominal epigástrico progresivo
irradiado a espalda, de dos meses de evolución. El
diagnostico de sospecha de adenocarcinoma de páncreas
se confirma por biopsia. Se realiza examen de imagen de
abdomen para evaluación de estructuras vasculares
próximas al tumor pancreático. ¿Cuál es el nombre de la
vena señalada que está ausente, trombosada por
infiltración tumoral, condicionando circulación colateral
en la pared gástrica?

a. Mesentérica superior
b. Coronaria estomaquica
c. Esplénica
d. Porta
e. Renal izquierda
17. Revisando la angiotomografía de un hombre de 70 años en estudio por aneurisma de aorta
abdominal, el radiólogo le informa de la presencia de una oclusión completa de la arteria
mesentérica inferior. El paciente se encuentra completamente asintomático. La oclusión de la
arteria mesentérica inferior cursa de manera asintomática en muchas ocasiones ya que el
territorio que irriga puede recibir flujo proveniente de la arteria:
a. cólica derecha
b. gastroduodenal
c. Epigástrica inferior izquierda
d. esplénica
e. cólica media

18. En las patologías de esófago es importante conocer bien la anatomía esofágica. ¿Cuál de
las siguientes afirmaciones es correcta?
a. El esófago tiene capa mucosa, muscular y serosa
b. El esófago abdominal es más largo que el cervical
c. El esófago torácico pasa por detrás del cayado aórtico
d. El epitelio esofágico normal es de tipo cilíndrico.
e. El esófago abdominal es discretamente más largo que el torácico

19. A pesar de que pueda haber variaciones anatómicas, lo habitual es que el ciego sea irrigado por una
rama arterial que proviene de unas de las siguientes arterias:
a. Iliaca derecha
b. Mesentérica inferior
c. Hepática derecha
d. Mesentérica superior
e. Iliaca izquierda

20. Ante un paciente con una cirugía abdominal urgente, el informe operatorio señala que se ha
realizado una resección de todo el duodeno y del tercio proximal del yeyuno manteniendo íntegros
el estómago y todo el íleon, así como los dos tercios distales del yeyuno. En el seguimiento
nutricional del paciente ¿Qué vitamina o mineral presentará con menor probabilidad una
disminución de su absorción?
a. Cianocobalamina
b. Calcio
c. Hierro
d. Transcobalamina
e. Transferrina

21. ¿Cuál de las siguientes alternativas detallan las venas que confluyen y forman la vena señalada?
a. mesentérica superior, gástrica izquierda y
gastroepiploica izquierda
b. mesentérica inferior, gástrica izquierda y renal
c. esplénica, mesentérica superior y mesentérica inferior
d. esplénica, pancreatoduodenal y omental izquierda
e. gástrica izquierda, esplénica y hepática común

22. ¿Cuál de las siguientes sustancias forma parte de la secreción biliar?


a. Tripsina
b. Lecitina
c. Elastasa
d. Quimotripsina
e. Pepsina
23. El tubo digestivo contiene diferentes tipos de epitelios y glándulas. La estructura señalada es
una
y
está localizada en el …………...
a. glándula de Brunner /
intestino grueso b. cripta de
Lieberkuhn / colon
c. cripta de Lieberkuhn / intestino delgado
d. glándula oxintica / estomago
e. célula parietal / estómago

Aunque las criptas de


Lieberkuhn están
presentes en el intestino
delgado, la
microfotografía es de
epitelio de colon.

24. ¿De qué musculo forma parte el ligamento inguinal?


a. Oblicuo externo del abdomen
b. Oblicuo interno del abdomen
c. Transverso del abdomen
d. Psoas
e. Dorsal ancho

25. Señale cuál de las siguientes afirmaciones NO se relaciona a la siguiente glándula anexa del
tubo digestivo mostrada en la imagen:
a. Es una glándula exocrina compuesta exclusivamente por acinos serosos
b. Su inervación está dada por el nervio auricular mayor (ramo posterior C2), que inerva
la vaina de la glándula así como la piel por encima de esta.
c. Esta glándula produce una secreción mucinosa acuosa, llamada mucoserosa, a través
del conducto de Wharton.
d. Su inflamación puede ser causada por un virus de los Paramyxoviridae, que provocan
una enfermedad muy frecuentemente en niños y adolescentes
e. Es una glándula endocrina y probablemente sea de origen pancreático

Se valida la opción e debido a que no


está relacionada con la imagen.

26. ¿Cuál de las siguientes enzimas está localizada en el borde en cepillo y juega un rol en la digestión de
proteínas?
a. Alfa dextrinasa
b. Pepsina
c. Enterocinasa
d. Lactasa
e. Carboxipeptidasa A.
Se valida la opción c debido a que es correcta en relación a la pregunta.
27. Una de los siguientes sustancias, NO sirve como un buen agente emulsificante:
a. Colesterol
b. Ácidos grasos
c. Sales biliares
d. Lecitina
e. Proteínas de la dieta

Se valida la opción e debido a que es correcta en relación a la pregunta.

28. La sustancia que estimula el crecimiento de la mucosa gástrica es:


a. Secretina
b. Motilina
c. Péptido estimulante de la mucosa gástrica
d. Gastrina
e. Histamina

29. ¿Cuál de las siguientes alternativas es una función de la colecistokinina?


a. Relajación de la vesícula para la
salida de bilis b. Secreción
de ácidos biliares
c. Contracción del esfinter de Oddi
d. Secreción de enzimas pancreáticas
e. Contracción del duodeno

Se valida la opción b debido al efecto de la CCK sobre la vesicula biliar.

30. Con respecto a la anatomía del tronco celiaco, señale lo correcto


a. El tronco celiaco se origina de la cara posterior de la aorta abdominal
b. Es una arteria delgada que tiene un calibre entre 2 y 3 mm
c. Una de sus ramas es la arteria gástrica derecha
d. La hepática común que es una de sus ramas, participa en la irrigación del estómago.

31. Con respecto a la anatomía del duodeno, marque la respuesta correcta:


a. Tiene una distribución en forma de “C”, que rodea la cola del páncreas
b. La 3ra porción duodenal está contenida en la pinza vascular aortomesentérica
c. Entre la 1ra y 2da porción se forma un ángulo, conocido como el ángulo de Treitz
d. La 4ta porción se dirige a la izquierda, hacia abajo y hacia atrás.
e. En la tercera porción desemboca el conducto colédoco.

32. El hígado está ampliamente tapizado por peritoneo, la estructura que conecta la cara
diafragmática del hígado precisamente con el diafragma es el ligamento:
a. teres
b. f
a
l
c
if
o
r
m
e
c
. triangular
d.
he
pático
común
e.
co
ronario

Se validan la opción c y e debido a que forman parte de los ligamentos que fijan el hígado al diafragma.
33. En el íleon se absorbe aproximadamente el 95% de a través de la circulación
enterohepática.
a. agua
b. colesterol
c. sales biliares
d. hidróxicobalamina
e. factor intrínseca
34. La estimula el mecanismo paracrino de la secreción de ácido clorhídrico.
a. histamina
b. acetilcolina
c. gastrina
d. secretina
e. somatostatina

35. En la digestión de proteinas, es el principal estímulo para convertir el pepsinógeno en


pepsina.
a. la gastrina
b. el pH ácido
c. la acetilcolina
d. la ptialina
e. la somatostatina

36. Con respecto a la somatostatina, marque lo correcto:


a. Es secretada por las células S del intestino
b. Induce a la producción de VIP
c. Interviene en la fase intestinal de la secreción gástrica
d. Produce acetilcolina para estimular a la célula parietal
e. No interviene en la regulación de la secreción de ácido clorhídico

37. En pecten anal, es una estructura comprendida entre:


a. la línea pectínea y los senos anales
b. la línea blanca y la apertura anal
c. el esfínter anal interno y el externo
d. la línea anocutánea y la línea pectínea
e. la línea blanca y columnas anales

38. ¿Cuál de las siguientes alternativas es una proenzima pancreática?


a. Tripsina
b. Elastasa
c. Quimotripsinógeno
d. Amilasa
e. Procarboxipepitidasa C.

39. En la segmentación hepática de Coinaud, el segmento hepático señalado con la flecha,


corresponde a : En la segmentación hepática de Coinaud, la flecha señala el segmento
hepático.
a. IV
b. V
c. VI
d. VII
e. VIII
EXAMEN
PARCIAL
SISTEMA
DIGESTIVO
(ME154)
Ciclo 2019-02

Sección: Todas
Profesores: Alfaro Salazar, Herberth Romulo; Alva Muñoz, Jose Carlos; Mayor Zevallos, Otto Alberto;
Duración: 30 minutos.
Indicaciones:
- Lea atentamente cada pregunta antes de responder.
- Se prohíbe el uso del celular y cualquier dispositivo electrónico.
- Está prohibido intercambiar materiales.
- Coloque su código de alumno en la tarjeta de respuestas. Si su código contiene una letra
reemplácela por un valor numérico siguiendo la siguiente equivalencia: A=9, B=8, C=7, D=6, E=5, F=4
y G=3.
- Traslade sus respuestas a la tarjeta, llenando los círculos de manera completa con lapicero negro o
azul. Está prohibido el llenado con lápiz, lapicero de otro color o con lapicero de tinta borrable.
- Sea cuidadoso en el llenado de la tarjeta de respuestas, pues solo esta tiene validez para la
calificación.
- Al terminar su examen avise al docente a cargo, no se levante de su sitio; debe entregar la hoja de
respuestas con la carátula del examen, este cuadernillo de preguntas se lo llevará cada estudiante.

1. La contracción del músculo permite la eliminación de gases (flatos) sin salida de


material fecal;
es el mismo músculo cuya relajación, sobretodo en cuclillas, permite el paso del contenido
fecal con menor esfuerzo durante la defecación.
a)
Isq
ui
or
re
ct
al
b)
Pu
bo
rre
ct
al
c) Esfínter anal externo
d) Esfínter anal interno

2. Paciente mujer de 54 años se presenta con náuseas, vómitos, estreñimiento, y es


diagnosticada de abdomen agudo quirúrgico; en la cirugía encuentran un vólvulo de ciego. Esta
anomalía puede explicarse por::
a) Falta de rotación intestinal
b) Falta de fusión del mesenterio
c) Defecto en la formación de la cloaca
d) Falta de formación del omento mayor

3. Paciente mujer de 23 años con faringitis aguda, toma para el dolor una tableta de paracetamol con
un poco de agua. Durante la deglución, se relaja su esfínter esofágico inferior y el fondo del
estómago, mientras el bolo está aún en el esófago. ¿Qué sustancia provocara con mayor
probabilidad la relajación del esfínter esofágico inferior y el fondo del estómago en esta mujer?
a) Óxido nítrico
b) Sustancia P
c) Histamina
d) Motilina
4. Luego de tres horas dando exámenes, un alumno de medicina comienza a sentir hambre.
Esta situación es probable que sea mediada por la que es sintetizada por el :
a) leptina / intestino
b) leptina / estómago
c) grelina / estómago
d) grelina / tejido adiposo

5. Varón de 72 años, con antecedente de diabetes mellitus tipo 2, que presenta enteropatía diabética
caracterizada por estreñimiento. Este problema puede estar asociado a:
a) deficiencia de óxido nítrico
b) aumento del reflejo gastrocólico
c) disminución de la secreción de colecistocinina (CCK)
d) aumento de la secreción del péptido intestinal vasoactivo (PIV)
6. Varón de 54 años con Diabetes Mellitus tipo 2, es diagnosticado de gastroparesia debido a que
presenta sensación de llenura precoz al comer, y reflujo gastroesofágico. Esta alteración en la
relajación receptiva y en el vaciamiento gástrico lo más probable es que se deba a una alteración
en:
a) el nervio vago
b) el ganglio celíaco
c) plexo submucoso
d) nervio hipogástrico

7. Varón de 67 años con tos y disminución de peso asociado a tabaquismo pesado, presenta
actualmente disfagia progresiva a alimentos sólidos. Se considera la presencia de un carcinoma
de bronquio izquierdo y por esta razón le realizan una endoscopía esofágica para descartar la
posibilidad de una compresión esofágica por el tumor. Se espera revisar el esófago en la
estrechez, que está a nivel de la vértebra
a) Tercera estrechez -T6
b) Segunda estrechez - C6
c) Segunda estrechez - T4
d) Tercera estrechez -T10

8. Varón de 34 años con dolor abdominal agudo en flanco derecho que se irradia a fosa ilíaca derecha,
es operado y se encuentra un divertículo intestinal inflamado, ubicado a 93 cm de la válvula
ileocecal. El origen de este divertículo es una falla en la obliteración de:
a) Conducto vitelino
b) Alantoides
c) Cloaca
d) Conducto anorectal
e) Uraco

En un niño menor de dos años con divertículo intestinal, este divertículo tiene su origen en
una falla en la obliteración de:
a)
Condu
cto
anorectal
b)
Conducto
vitelino
c) Alantoides
d) Cloaca
e) Uraco

9. Mujer de 43 años sufre un grave accidente de tránsito y está hospitalizada en coma, es alimentada
por vía intravenosa durante varias semanas. Producto de este tipo de alimentación, se encuentra en
la endoscopía atrofia de la mucosa gastrointestinal. La causa más probable de esta atrofia son los
bajos niveles séricos de la hormona:
a) Colecistocinina
b) S
e
c
r
e
t
i
n
a
c
)
Gastrina
d) PIV

10. Una mujer de 30 años llega al consultorio porque se queja de dificultades para deglutir, la cual se
agravan cada vez más. Se realiza un estudio manométrico para examinar la generación de presión a
lo largo del esófago. Esta prueba revela que las contracciones como respuesta a la deglución están
mal sincronizadas y que la presión en el esfínter esofágico inferior permanece elevada. El
diagnóstico más probable es producido por niveles bajos de

a). acalasia /
sustancia P
b) acalasia /
óxido nítrico
c) enfermedad por reflujo gastrointestinal / acetilcolina
d) enfermedad por reflujo gastrointestinal / óxido nítrico

11. Paciente de 2 años, llega a emergencia por haber ingerido una moneda con la que estaba jugando.
El lugar más probable donde puede haberse quedado suspendido este objeto es a nivel del
estrechamiento producido a nivel del:
a) músculo milohiodeo
b) músculo aritenoideo
c) músculo cricofaríngeo
d) constrictor superior de la faringe

12. En una apendicectomía, al realizar la incisión de McBurney en la fosa iliaca derecha, es


necesario cortar los siguientes músculos, de afuera hacia adentro:
a) Recto – Oblicuo externo – Transverso
b) Recto – Oblicuo externo – Oblicuo interno
c) Oblicuo externo – Oblicuo interno – Recto
d) Oblicuo externo – Oblicuo interno – Transverso

13. Un varón de 90 años que se encuentra postrado en cama, es referido del asilo para endoscopia
por dificultad para deglutir luego de tomar un medicamento para aliviar el dolor la noche
anterior. La endoscopía revela que la píldora se alojó en el esófago y causó una reacción
inflamatoria. Lo más probable es que esto haya sido por la producción de múltiples ondas:
a) secundarias
b) primarias
c) lentas
d) segmentarias

14. Mujer de 23 años es diagnosticada de bulimia, al examen físico se observa ulceraciones en el


segundo y tercero dedo de la mano derecha. Esto se puede deber al uso continuo de estos dedos
para inducir el vómito, mediante la estimulación del par craneal:
a) V
b
)
I
X
c) X
d) XI

15. Varón de 52 años se presenta por diarrea persistente de seis semanas de duración. En la
colonoscopia se observa un pólipo a nivel del íleon distal. El patólogo informa que se trata de un
tumor neuroendócrino, probablemente originado por las células enterocromafines del intestino. La
sustancia que más probablemente esté produciendo este tumor es:
a) Serotonina
b) Insulina
c) CCK
d) GIP

16. La fase oclusal de la masticación se realiza con la contracción de los músculos:


a) digástricos
b) masetero y temporal
c) orbicular y buccinador
d) pterigoideo lateral y digástrico

17. Al tomar su café en Starbucks, un estudiante de medicina sufre una quemadura de primer
grado en el tercio anterior de la superficie dorsal de la lengua. La información de dolor es
transmitida por el nervio:
a) cuerda del tímpano
b) g
loso
farín
geo
c)

ling
ual
d) facial

18. Paciente es evaluado por faringitis aguda en consultorio externo. El médico de familia le solicita
que abra la boca y saque la lengua. Para realizar la acción de sacar la lengua, es necesario que se
contraiga el músculo:
a) e
st
il
o
gl
o
s
o
b)
g
e
ni
o
gl
o
s
o
c) palatogloso
d) transverso de la lengua
19. Paciente con síndrome de Sjögren, presenta “boca seca”
(disminución de la producción de saliva) y caries dental,
asociada a la pérdida de la función de tampón de la saliva.
Esta desminerilización del diente puede comprometer a las
prolongaciones citoplasmáticas ubicadas en los tubos
huecos de la estructura señalada con la letra:
a) B
b) A
c) E
d) C

20. Mujer de 32 años acude a consulta por presentar disfagia de


progresión lenta, reflujo gastroesofágico y vómitos desde
hace 3 meses de evolución progresiva. Se le realiza un
estudio radiológico con contraste en el que se observa
estrechamiento del esfínter esofágico inferior (imagen).
Según sus conocimientos, este paciente se beneficiaría con el
uso de:
a) agonista beta adrenérgico
b) agonista
alfa adrenérgico
c) análogo de
óxido nítrico
d) análogo de Sustancia P
EXAMEN
PARCIAL
SISTEMA
DIGESTIVO
(ME154)
C
ic
lo
2
0
2
0
0
0

Sección: Todas
Profesores: Alva Muñoz, Jose Carlos
Duración: 35 minutos.
Indicaciones:
- Lea atentamente cada pregunta antes de responder.
- Se prohíbe el uso del celular y cualquier dispositivo electrónico.
- Está prohibido intercambiar materiales.
- Coloque su código de alumno en la tarjeta de respuestas. Si su código contiene una letra
reemplácela por un valor numérico siguiendo la siguiente equivalencia: A=9, B=8, C=7, D=6, E=5, F=4
y G=3.
- Traslade sus respuestas a la tarjeta, llenando los círculos de manera completa con lapicero negro o
azul. Está prohibido el llenado con lápiz, lapicero de otro color o con lapicero de tinta borrable.
- Sea cuidadoso en el llenado de la tarjeta de respuestas, pues solo esta tiene validez para la
calificación.
- Al terminar su examen avise al docente a cargo, no se levante de su sitio; debe entregar la hoja de
respuestas con la carátula del examen, este cuadernillo de preguntas se lo llevará cada estudiante.

1. Paciente de sexo masculino de 82 años de edad ingresa a emergencia con dolor abdominal agudo y
diarreas. Se le realiza una arteriografía en la que se observa que la arteria aorta tiene un trombo
ocluyendo el 95% del flujo, a nivel del nacimiento de la arteria mesentérica inferior. ¿Cuál de las
siguientes arterias podría contribuir a la irrigación colateral del colon descendente?
a) cólica media
b) sigmoidea
c) rectal superior
d) ileocólica

2. Niño de 5 años presenta dolor esofágico y hematemesis (vómitos hemorrágicos) luego de tragarse
una espina de pescado. En la endoscopía se observa perforación del esófago distal a la cuarta
estrechez esofágica. ¿Las ramas de cuál de las siguientes arterias estarán lesionada con mayor
probabilidad?
a) Gástrica izquierda
b) Bronquiales
c) Frénica inferior
d) Tiroidea inferior

3. Al ingerir una cucharada de mantequilla es muy probable que se disminuya la sensación de hambre
por medio de la activación de la vía POMC/CART (POMC=proopiomelanocortina y
CART=transcripción regulada de cocaína y anfetamina), activada directamente por la hormona:
a) colecistoquinina (CCK)
b) insulina
c) grelina
d) secretina

4. Al comer unas papitas fritas con mayonesa, el vaciamiento gástrico disminuye por efecto directo
de la hormona: a) colecistoquinina (CCK)
b) bombesina
c) motilina
d) gastrina
5. Recién nacido de dos horas es diagnosticado de hernia umbilical de 1,5 cm de diámetro; el
cirujano pediatra solicita una tomografía abdominal en donde se evidencia que la hernia
umbilical está ocupada por una porción del tracto gastrointestinal. ¿Qué porción del tracto
gastrointestinal estaría ocupando esta hernia con mayor probabilidad?
a) Íleon
b) Colon sigmoides
c) Duodeno
d) Colon transverso
6. Recién nacido de 7 horas, de parto por cesárea debido a polihidramnios (aumento del volumen del
líquido amniótico), con regurgitación de la leche materna y artificial, y no ha presentado meconio.
Se le realiza una tomografía donde se evidencia aire en el estómago y una malformación del
desarrollo esofágico. Con respecto a esta malformación lo más probable es que se pueda tratar de
una atresia esofágica:
a) proximal con fístula traqueoesofágica distal
b) distal con fístula traqueoesofágica proximal
c) proximal y distal
d) sin fístula

7. Lactante de 6 meses de edad que es traído a consulta por presentar vómitos no biliosos a repetición
y retraso en el crecimiento. En la radiografía de abdomen simple se observa nivel hidroaéreo en
estómago y en primera porción de duodeno (doble burbuja). ¿Cuál de las siguientes alternativas
puede explicar la condición del lactante?
a) Páncreas anular
b) Atresia duodenal en la tercera porción
c) Atresia yeyunal
d) Hipertrofia del píloro

8. En ausencia o deficiencia de la secreción de la hormona motilina,


se producirá:
a)sobrecrecimiento bacteriano
b) diarrea
c) aumento del vaciamiento gástrico
d) hipertrofia del píloro

9. La estimulación parasimpática aumenta la motilidad intestinal, mientras que la estimulación


simpática la disminuye. ¿Sobre cuál de las siguientes alternativas el sistema nervioso autónomo
actúa para el control de la motilidad intestinal?
a) Potencial de membrana en el plexo mientérico (de Auerbach)
b) Frecuencia de ondas lentas
c) Secreción de secretina
d) Nivel de IP3 en el plexo submucoso (de Meissner)

10. En un recién nacido con protrusión de contenidos abdominales y cubiertas por amnios o peritoneo,
es cierto que: a) Se presenta por un defecto en el cierre de la pared
b) Se acompaña de otras malformaciones congénitas
c) Se debe al no retorno de la hernia fisiológica
d) Se produce a través del ombligo

11. Lactante de 20 días con estreñimiento, distención abdominal progresiva, acompañada


ocasionalmente de vómitos biliosos. Como antecedente, el meconio lo eliminó por primera vez a las
72 horas de nacido. Su mamá menciona que ayuda a la evacuación con ayuda de un termómetro
rectal. Se sospecha de megacolon agangliónico (Enfermedad de Hirschsprung). ¿Cuál de las
siguientes alternativas explica el caso?
a) Se presenta contracciones tónicas en la región ano rectal
b) Se presenta dilatación de tracto gastrointestinal afectado
c) Las células ganglionares sólo han migrado al ano recto
d) La zona que más se afecta es inervada por fibras del nervio esplácnico menor

12. ¿Cuál de los siguientes reflejos disminuye el tránsito


gastrointestinal? a) Doloroso
b) Gastrocólico
c) De defecación
d) Colicoileal

13. Al ingerir una sustancia ácida como el vino (pH 3), se estimula la motilidad gástrica por acción
de la hormona: a) motilina
b) secretina
c) colecistoquinina (CCK)
d) bombesina
14. Paciente de 24 años acude a consulta externa por presentar una fístula oronasal
(comunicación entre la cavidad oral y la cavidad nasal). Está fístula está asociada al
antecedente de haber sido operada de paladar hendido a los dos años de edad, durante una
campaña gratuita extranjera de corrección de paladar fisurado. ¿Cuál de las arterias palatinas
podría haberse lesionado durante esa cirugía?
a) Mayor
b) Menor
c) Ascendente
d) Rama palatina de la faríngea ascendente

15. Paciente de sexo masculino de 52 años con úlcera péptica gástrica de 14 años de evolución,
con cuadro de hemorragia digestiva alta hace 4 meses, sin cicatrización de la úlcera. Entre las
opciones quirúrgicas se considera realizarle un vaguectomía troncal (sección del nervio vago) a
nivel del hiato esofágico. ¿Cuál de las siguientes complicaciones podría esperarse producto de
la pérdida de inervación parasimpática?
a) Menor inervación del colon ascendente
b) Se perderá el reflejo de defecación
c) Se perderá el reflejo de micción
d) Impotencia sexual

16. Paciente de 23 años con bulimia es traída a la emergencia deshidratada, semiconsciente y con
alcalosis metabólica. Los vómitos autoinfligidos por esta paciente se producen por
estimulación de receptores en la base de la lengua que mandan información directamente al:
a) núcleo del tracto solitario
b) centro del vómito en el tallo encefálico
c) zona quimiorreceptora gatillo
d) cerebelo

17. Niño de 3 años es traído a emergencia por madre quien manifiesta que hace 10 horas deglutió
una pila pequeña de reloj de bordes romos. El niño está asintomático. Usted la tranquiliza
diciéndole es un cuerpo extraño tan pequeño de seguro que va a seguir el tránsito intestinal
como lo haría un bolo alimenticio, y que lo más probable es que en ese momento se encuentre
en:
a) colon
b) estómago
c) yeyuno
d) recto

18. Los movimientos en masa son un tipo de movimiento muy importante, una de las
consecuencias de estos movimientos es:
a) la distensión rectal
b) el peristaltismo del intestino delgado
c) la retropulsión gástrica
d) la contracción del esfínter anal interno

19. Durante la deglución, al momento que el bolo alimenticio pasa por el esfínter esofágico
superior, se espera que la presión intraesofágica:
a) disminuya en el cardias
b) disminuya en el tercio medio del esófago
c) aumente en la porción distal al bolo
d) aumente en el tercio medio del esófago

20. Paciente con enfermedad de Chagas que presenta disfagia a sólidos. ¿Cuál de las siguientes
puede ser la causa de esta complicación?
a) Disminución de células ganglionares en el esfínter esofágico inferior
b) Aumento en la liberación de óxido nítrico en el esfínter esofágico inferior
c) Disminución de las neuronas que liberan péptido intestinal vasoactivo
d) Aumento de la actividad de la motilina en el esófago distal

La triada sintomática vómitos explosivos post-prandiales, movimientos peristálticos


epigástricos visibles de izquierda derecha y nódulo palpable epigástrico subcostal
derecho pertenecen a:
- Estenosis congénita hipertrófica de píloro

El uso de ranitidina bloquea el receptor H2 de la histamina en células parietales. La


histamina llega a estas células por:
- Difusión

Cuando la contracción peristáltica primaria esta atraviesa el tercio superior del


esofago, el esfinter esofágico inferior estará:
- Relajado

¿En qué casos los vómitos siempre son brillosos?


- Atresia yeyunal
El nacimiento de la arteria
1. Un niño de 2 años es llevado a la consulta por diarrea persistente y edema de las
extremidades, además falta de crecimiento y desarrollo en relación a su edad. Los
análisis de sangre revelan que tiene concentración plasmática baja de proteínas
(hipoproteinemia). Durante la endoscopía duodenal, se coloca colecistocinina
(CCK) endovenosa y se recoge muestras del líquido duodenal; el resultado del
líquido confirma incapacidad para hidrolizar proteínas a un pH neutro, esta
situación mejora al añadir una pequeña cantidad de tripsina. El paciente
probablemente esté sufriendo la falta congénita de
-Enterocinasa
2. Experimentalmente se incrementa la velocidad de la secreción salival con
una sustancia, el análisis de la composición de esta saliva obtenida se
espera encontrar…………..
-Disminución de concentración de potasio
3. Paciente varón de 46 años soltero, consulta por odinofagia y bajo de peso, tiene
antecedente de tuberculosis desde hace 3 meses y es fumador crónico (10
cigarrillos por día); al evaluar la cavidad oral se identifica lesión blanquecina en
el dorso de la lengua y paladar blando, las lesiones se desprenden con el baja
lengua dejando una base eritematosa. Esta lesión corresponde probablemente a
……………………….
…..
-Candidiasis oral
4. Paciente mujer de 35 años acude a consulta por sensación de sequedad y
lesiones en cavidad oral. Al examen se observa atrofia de la mucosa, fisuras y
úlceras; nota además sequedad e irritación de la córnea y aumento del tamaño
de las glándulas parotídeas. Su diagnóstico más probable es artritis reumatoide;
el hallazgo más probable en una biopsia de glándula parótida es……..….
-Gran infiltración de linfocitos y células plasmáticas
5. Un paciente con anemia acude con su médico quejándose de episodios
frecuentes de gastroenteritis. Un análisis de sangre revela anticuerpos
circulantes dirigidos contra células parietales gástricas. Su anemia es atribuible
a la hiposecreción de
-Factor intrínseco
6. Dos estudiantes deciden tomar un receso para comer una hamburguesa a la
hora del almuerzo. Antes de llegar a la cafetería, impulsos nerviosos
provenientes del complejo vagal dorsal iniciarán la secreción de ácido gástrico
por la liberación de
…………………….. desde el sistema nervioso entérico.
-GRP
7. Un niño de cuatro años de edad es llevado a la consulta por cuadros diarreicos
frecuentes caracterizados por heces pálidas, voluminosas y fétidas, presenta bajo
peso y talla. Se mide la concentración de cloruro en el sudor y se encuentra que
sus valores son muy elevados. La alteración más importante a nivel de células
ductales del páncreas tiene relación directa con la conductancia de…………
-Cloro
8. Una mujer de 50 años de edad que sufrió durante varios años resequedad de
los ojos debida a producción inadecuada de lágrimas es enviada con un
gastroenterólogo para evaluación de pirosis crónica. El examen endoscópico
revela erosiones y tejido cicatrizal en la parte distal del esófago justo por arriba
del esfínter esofágico inferior. Las lesiones pueden atribuirse a la disminución
de uno de los siguientes componentes salivales:
-Bicarbonato

9. Se evalúa los valores séricos de las siguientes sustancias a un paciente con


enfermedad hepática terminal; en este paciente se espera encontrar la
combinación con la letra …………
-disminuida, aumentada, disminuida
10. Una mujer de 35 años de edad HIV positiva, se presenta al médico con dolor
abdominal en cuadrante superior derecho e ictericia. La paciente refiere haber
tenido múltiples episodios de ictericia durante los últimos 10 años. Los exámenes
para determinar hepatitis viral, dieron positivos para Hepatitis B, siendo catalogado
el caso como hepatitis crónica con alteración funcional. En un examen de sangre
¿cuál de los siguientes parámetros está disminuido?
-Albúmina
11. En el reflejo peristáltico del intestino delgado, uno de los siguientes eventos
sucede en la porción oral del bolo alimenticio…………...
-Acción de acetilcolina en el músculo circular
12. Experimentalmente se coloca una dosis alta de secretina en la luz intestinal
duodenal; como consecuencia de esto, en el jugo pancreático de la misma
luz intestinal se observa la disminución de la concentración de …..………..
-Cl
13. Un varón de 58 años de edad con enfermedad de Crohn severo fue sometido a
una resección ileal. Después de la cirugía este paciente padecerá de esteatorrea,
esto se explica porque …..………..
- La micelas no pueden formarse
14. En un experimento se inserta un balón en el estómago de un voluntario, se infla
poco a poco mientras que se vigilan las presiones intraluminales. Aunque el
volumen del balón aumenta considerablemente, las presiones permanecen
constantes. Esta relación volumen-presión se explica por la liberación local de
…………..
-Óxido nítrico y péptido inhibidor vasoactivo
15. La toxina de Vibrio cholerae causa diarrea debido a…….
-El Incremento de la secreción de cloro por las células de la cripta intestinal
16. ¿Cuál de las siguientes alternativas es una característica de la secreción
exocrina del páncreas?
-Tiene una baja concentración de Cl- respecto al plasma
17. Una madre lleva a su hijo de dos años de edad a la sala de urgencias, estresada
porque el niño deglutió una moneda de 10 céntimos mientras la familia cenaba en
un restaurante. El médico observa mediante fluoroscopía que la moneda se halla
en el estómago y asegura a la madre que la moneda se eliminará con las heces.
El médico recomienda utilizar la respuesta fisiológica que permitirá la evacuación
de la moneda del estómago al intestino ………….…..
-Son los movimientos de mezcla y trituración
-. Es provocada por el ayuno
18. Las estructuras en el hígado que permite que los productos metabólicos unidos a
proteínas tengan acceso a las membranas basolaterales de los hepatocitos, son…..
-Las fenestras sinusoidales
19. La composición de la bilis es modificada conforme fluye por los conductillos
biliares. Durante este tránsito se espera que aumente la concentración de…….
-Monómeros de ácido biliar
-Ig A
20. Se mide experimentalmente el contenido gástrico de dos personas. La persona
“A” tiene alto contenido de grasa y la persona “B” tiene un contenido hipertónico
¿Cuál de las siguientes es correcto respecto al vaciamiento gástrico?
- Hay ralentización del vaciado gástrico en ambos casos
21. El examen endoscópico de un paciente con hipertensión portal grave revela venas
tortuosas que sobresalen hacia la luz del esófago. El paciente recibe tratamiento
quirúrgico mediante la colocación de una derivación que conecta la vena porta a
la vena cava. Después de la operación el riesgo de encefalopatía y el
riesgo de sangrado de várices ……………..
-Aumentará/disminuirá
22. Un paciente varón de 18 años de edad acude al médico para sus exámenes
de rutina. Sus resultados de laboratorio muestran un valor de bilirrubina sérica
de 4 mg/dl y una bilirrubina directa de 0,3 mg/dl. Las pruebas de función
hepática son normales. La alteración que explica mejor este caso es por la
deficiencia de
………………..
-Glucuronil transferasa
23. Un hombre de 57 años de edad es llevado a urgencias con hematemesis
masiva rojo brillante, a su llegada se halla inconciente con PA: 80/40 mm Hg y
FC: 124 lat/min. Luce ictérico con presencia de “arañas vasculares en el tórax
anterior y extremidades”, abdomen distendido con signo de oleada positiva. Se
encuentra esplenomegalia y pérdida de la masa muscular en extremidades. La
anastomosis vascular responsable del sangrado en este paciente es
-Vena gástrica izquierda y vena ácigos
24. Un estudiante de medicina está comiendo un plato de comida a base de
champiñones, espárrago y salsa de soya. El sabor umami contenido en todos
estos alimentos actúa a nivel de los botones gustativos estimulando
………………..
-Un receptor acoplado a proteína G
25. Un hombre de 22 años de edad se presenta al médico con una historia de 1 año
de evolución caracterizado por dolor recurrente en fosa iliaca derecha y diarrea.
Manifiesta además pérdida de peso de 8 kg durante este periodo. La
colonoscopía revela múltiples lesiones en el ileon terminal y colon. La biopsia de
estas lesiones revela engrosamiento, inflamación y ulceración de la mucosa. El
diagnóstico más probable en este caso es…….
-Enfermedad de Crohn
26. Varón de 61 años que consulta por dolor retro esternal intenso desde hace 6 horas
y después de vómitos intensos y repetidos; al examen se observa disnea, cianosis,
hipotensión y signos clínicos de shock. La radiografía simple de tórax muestra
neumomediastino. El líquido en el espacio pleural aspirado tiene alta
concentración de amilasa. ¿Cuál de las siguientes alternativas puede explicar este
cuadro clínico? -Rotura espontánea de esófago
27. La secreción del ácido en la célula parietal gástrica se lleva a cabo por una
ATPasa especifica que intercambia hidrogeniones (H+) del citosol por…..
-K +
28. En condiciones normales el ingreso de 600 ml de líquido es el estómago provoca
un aumento de presión intragástrica de unos 12 cm de H2O. Después de una
vagotomía (corte del nervio vago) es de esperar que el ingreso del mismo volumen
de líquido provoque lo siguiente: …………………………………
-Un aumento mayor de la presión
29. Una paciente de 30 años de edad es sometida a una cirugía en oído medio
derecho por un problema de otoesclerosis. Luego de la cirugía refiere alteración
en la percepción de sabores. Al evaluar el caso usted esperaría encontrar……….
-Alteración en la sensación del gusto en los dos tercios anteriores de la
lengua
-Sensación del dolor, tacto y temperatura conservada en toda la lengua
30. ¿Cuál de las siguientes alternativas es correcta?
-Las sales biliares desconjugadas son absorbidas preferentemente en el colon
31. En un paciente de 45 años de edad con colestasis biliar, se encuentra una
elevación de los niveles sanguíneos de fosfatasa alcalina hasta 3 veces la cifra
normal. ¿Cuál de las siguientes alternativas estará también elevada como
evidencia del daño de la vía biliar?
-Gamma glutamil transpeptidasa
32. Revisando la angiografía de un hombre de 70 años en estudio por aneurisma de
aorta abdominal el radiólogo informa de la presencia de una oclusión completa de
la arteria mesentérica inferior. El paciente se encuentra completamente
asintomático.
¿Cuál de las siguientes arterias se anastomosa a la sistema arterial
de la mesentérica inferior?
-Cólica media
33. Lactante de 3 meses de vida es atendido por presentar diarrea, se administra
una solución de glucosa y electrólitos por vía oral. La proteína de membrana
apical que explica la capacidad de esta solución para proporcionar aporte de
glucosa e hidratación es ………..
-SGLT-1
34. Paciente ha sufrido herida de bala en el abdomen, se le ha tenido que extirpar el
segmento medio y distal del ileon. En este caso la síntesis hepática de sales
biliares estará …..…..
-Incrementada por estímulo de la enzima colesterol 7 alfa hidroxilasa
35. Un varón de 75 años ingresa al consultorio por presentar ictericia marcada de piel
y las escleras. El estudio del paciente mostró que presentaba un tumor que
obstruía la totalidad del conducto hepático común. ¿Cuál de las siguientes
estructuras se encontrará dilatada en este paciente?
-Conductos de Hering
36. En un paciente con insuficiencia renal crónica, el déficit en la absorción de calcio
a nivel del enterocito se debe a lo siguiente:
-No se convierte la 25 hidroxicolecalciferol a 1,25 dihidroxicolecalciferol
37. Varón de 30 años es traído a emergencia por agresión abdominal con arma de
fuego (pistola) y es sometido a laparotomía exploratoria, observándose isquemia
del colon ascendente y parte del colon trasverso ¿la lesión de cuál de las siguientes
arterias explicaría esta isquemia?
d. Mesentérica superior
38. Respecto a las sustancias gastrointestinales que regulan la secreción pancreática;
marque la afirmación correcta:
b. La acetilcolina es capaz de estimular la secreción enzimática y de bicarbonato del
páncreas
39. Ante una lesión del X par craneal, ¿cuál de los siguientes músculos mantiene
conservada su función?:
b. Tensor del velo del paladar
40. Experimentalmente se utiliza atropina (anticolinérgico) para inhibir la secreción
de gastrina, sin embargo la secreción de esta hormona se sigue dando ante
estímulos vagales. Esta situación se explica porque la atropina:
d. No bloquea la acción del péptido GRP
41. Un varón de 50 años es sometido a extirpación del duodeno y parte proximal del
yeyuno. Esta situación ocasionaría la pérdida de las células ……….. , productoras
de ………………… que estimula la secreción de bicarbonato por el páncreas.
“S” / secretina
42. Se evalúa la expresión de la proteína Agrp en una persona con alteración del apetito;
lo correcto respecto a esta proteína es…..
La mutación del gen que la codifica produce adelgazamiento
43. Juana cae de la bicicleta y se fractura la región anterior del hueso maxilar superior
con compromiso de la fosa incisiva. Al examen físico de la región esperaría encontrar
alteración en la sensibilidad de la encía …………………
palatina anterior
44. Recién nacido es atendido por el neonatólogo y luego entregado a su madre para
dar de lactar; la madre al dar de lactar observa coloración azulada de labios,
acompañado de tos persistente, dificultad respiratoria y distención abdominal. Se
le intenta colocar una sonda nasogástrica pero esta retorna a la cavidad oral en
todos los intentos. ¿Cuál de las siguientes anomalías del desarrollo es el más
probable en este caso?
b. Atresia esofágica proximal con fístula traqueoesofágica distal l
45. ¿Cuál de los siguientes mecanismos ocurre durante la defecación?
En la posición de “cuclillas” el músculo puborrectal se halla relajado
46. Un paciente luego de un accidente sufre lesión del piso de la boca, se constata
daño del nervio “cuerda del tímpano”, en este caso se esperaría en
47. contrar disminución de la………………………….… de la lengua
Sensación del gusto en los dos tercios anteriores
48. ¿Cuál de las siguientes afirmaciones es la correcta sobre la gastrina?
Actúa en la célula diana mediante su receptor CCk tipo B
49. Al recibir un paciente con signos de hipovolemia y antecedente de trauma en
abdomen por accidente de tránsito, usted identifica radiológicamente: lesión de
primera vértebra lumbar y signos de lesión en páncreas; durante la cirugía se
observó pobre irrigación de asas intestinales. El vaso afectado es la arteria ……..
c. mesentérica inferior
50. Un paciente sufre de daño a nivel del cuello con lesión muscular en la región de la
faringe. En el examen físico se determina dificultad para la elevación de la faringe y
para el cierre del itsmo de las fauces. En este caso, probablemente esté afectado el
músculo:
c. palatofaringeo
51. Varón de 50 años a quien le realizan la curación de la segunda molar de la
arcada superior derecha. En un momento determinado, el paciente acusa de
intenso dolor de la pieza dentaria en tratamiento. La vía aferente del dolor viaja
a través del nervio …………
a. trigémino V2
52. La distención gástrica por los alimentos produce incremento de secreción de
HCl mediante la producción de ………….. que estimula a las célulasvía
proteína
………..
a. gastrina / parietal / Gq
53. Un niño de tres años llega a emergencia con disfagia (dificultad para tragar), dolor
retro esternal, salivación y llanto. Se sospecha de ingesta de cuerpo extraño
(moneda) en el esófago; al ser evaluado se constata en una radiografía presencia
de cuerpo extraño a nivel de C6 (6° vértebra cervical). El cuerpo extraño estará
suspendido a nivel del estrechamiento producido por………..
c. el músculo cricofaríngeo
54. La triada portal (arteria hepática, vena portal y conducto biliar común) está
contenida en el ligamento …….……… y derivan embriológicamente del ……
a. hepato duodenal / mesenterio ventral
55. Un paciente refiere no percibir algunos sabores, al examen físico se constata
alteración en la percepción de sabores y del dolor en el tercio posterior de la
lengua ¿Cuál de los siguientes nervios estará alterada en su función?
c. Glosofaríngeo (IX par)
56. En el caso de un paciente con gastrinoma (tumor productor de gastrina), la
presencia de úlceras duodenales y erosión de la mucosa gástrica, se debe
principalmente a…….
c. el exceso de HCl por estímulo de receptores CCK-B en la célula parietal
56. El reflejo entero gástrico se caracterizan por:
d. originarse debido a la distensión duodenal y presencia del quimo ácido
57. Mauricio tiene dificultad para deprimir el paladar y elevar la parte posterior de la
lengua. En este caso estará afectado un músculo, específicamente el músculo
…………….
b. extrínseco – palatogloso
58. En condiciones normales, el ingreso de 600 ml de líquido es el estómago provoca un
aumento de presión intragástrica de unos 12 cm de H2O. Después de una vagotomía
(corte del nervio vago) es de esperar que el ingreso del mismo volumen de líquido
ocasione
………………………………… de la presión
intragástrica.
c. un aumento mayor
59. La explicación fisiológica de presentar somnolencia de 30 minutos a 1 hora
después de ingerir alimentos, se explica por: a. Aumento del cloro intraluminal
e. Aumento de la alcalinidad sanguínea
60.Se presenta un paciente, el cual presenta un antecedente de tuberculosis intestinal,
por lo cual, se le resecó 80 cm de íleon distal. Desde el punto de vista fisiológico, el
paciente puede presentar una de las siguientes alteraciones: a. Disminución de la
secreción de Vitamina B12

e. Disminución de la absorción de ácido glicocólico


61. Un paciente es sometido experimentalmente a un fármaco que modifica el flujo
salival, obteniéndose un volumen de saliva de 288 ml en 6 horas. En este caso las
concentraciones de electrolitos y bicarbonato en la saliva obtenida varían de la siguiente
manera: a. ↑ Na+,
↓ K+, ↑ Cl-, ↑ HCO3-

1. b. ↓ Na+, ↓ Cl-, ↑ K+, ↓ HCO3-


62. Uno de los siguientes elementos debería hallarse con más probabilidad en el esófago
de un paciente que sufre de reflujo gastro esofágico…
a. Pepsina

63. Un paciente de 40 años cursa con anemia de 8g/dl, aqueja además de astenia y
sensación de hormigueo bilateral en los miembros inferiores, al examen se halla alteración
de la sensibilidad a la vibración y camina con ampliación de la base de sustentación. Uno de
los siguientes procedimientos sería de ayuda para el diagnóstico de este paciente:
a. Tomografía cerebral
b. Biopsia de la mucosa gástrica

64.Paciente de 60 años ingresa por caída hace 1 hora y pequeño hematoma en cuero
cabelludo, al examen físico ampliado se observa ictericia de piel y mucosas generalizada,
abdomen blando, se palpa estructura quística no dolorosa en hipocondrio derecho que
corresponde a vesícula biliar (signo de Courvoisier), en los exámenes de laboratorio se
halla niveles bajos en la formación de estercobilinógeno y urobilinógeno en heces,
incremento de la bilirrubina conjugada en la orina, elevación de fosfatasa alcalina y
gamma glutamil transpeptidasa séricas. El presente cuadro puede ser explicado por: a.
Reabsorción de hematoma
c.Carcinoma de la cabeza de páncreas
65. Un recién nacido presenta vómitos biliosos poco tiempo después de cada alimento.
Al preguntar a la madre sobre antecedentes, ella recuerda que tuvo polihidramnios
durante la gestación, pero un análisis de cariotipo fue normal. Una de las siguientes es la
causa más probable de estos hallazgos en el recién nacido: a. Enfermedad de Hirschprung
e. Malrotación de la yema pancreática ventral
66.En un estudio de la secreción de hormonas gastrointestinales, sus concentraciones en la
vena porta se midieron durante perfusión luminal del intestino delgado con soluciones de
diversas magnitudes de pH. ¿Qué hormona aumentará en el plasma de la vena porta
durante perfusión a través del intestino con una solución de pH 3?
a. CCK
e. secretina
67.Paciente de 30 años que ingresa a causa de un traumatismo abdominal cerrado. En la
exploración se aprecia discreta palidez de piel y mucosas, auscultación pulmonar normal,
taquicardia de 120 /min. Discreta distensión abdominal y matidez en flancos; el
hematocrito, que era prácticamente normal al ingreso, disminuye a 30% a las tres horas.
En la Rx de tórax se objetiva fractura de las costillas 10-11 izquierdas. La causa más
probable de la anemización en este paciente es: a. traumatismo renal con hemorragia
retroperitoneal.
c. rotura de bazo con hemoperitoneo.
68. Revisando la angiotomografía de un hombre de 70 años en estudio por aneurisma
de aorta abdominal, el radiólogo le informa de la presencia de una oclusión completa
de la arteria mesentérica inferior. El paciente se encuentra completamente
asintomático. La oclusión de la arteria mesentérica inferior cursa de manera
asintomática en muchas ocasiones ya que el territorio que irriga puede recibir flujo
proveniente de la arteria:
a. cólica derecha
e. cólica media

69. En las patologías de esófago es importante conocer bien la anatomía esofágica. ¿Cuál
de las siguientes afirmaciones es correcta? a. El esófago tiene capa mucosa, muscular y
serosa

c. El esófago torácico pasa por detrás del cayado aórtico

70. A pesar de que pueda haber variaciones anatómicas, lo habitual es que el ciego
sea irrigado por una rama arterial que proviene de unas de las siguientes arterias: a.
Iliaca derecha

d. Mesentérica superior

71. Ante un paciente con una cirugía abdominal urgente, el informe operatorio señala
que se ha realizado una resección de todo el duodeno y del tercio proximal del yeyuno
manteniendo íntegros el estómago y todo el íleon, así como los dos tercios distales del
yeyuno. En el seguimiento nutricional del paciente ¿Qué vitamina o mineral presentará
con menor probabilidad una disminución de su absorción?
a. Cianocobalamina

72. ¿Cuál de las siguientes sustancias forma parte de la secreción biliar? a. Tripsina
Lecitina

73.¿De qué musculo forma parte el ligamento inguinal?


-Oblicuo externo del abdomen
74.¿Cuál de las siguientes enzimas está localizada en el borde en cepillo y juega un rol en
la digestión de proteínas?
e. Carboxipeptidasa A.
75. Una de los siguientes sustancias, NO sirve como un buen agente emulsificante:
a. Colesterol
76. La sustancia que estimula el crecimiento de la mucosa gástrica es:

a. Secretina

d. Gastrina

77.¿Cuál de las siguientes alternativas es una función de la colecistokinina?

a. Relajación de la vesícula para la salida de bilis

d. Secreción de enzimas pancreáticas

78.Con respecto a la anatomía del tronco celiaco, señale lo correcto a. El tronco celiaco
se origina de la cara posterior de la aorta abdominal
d. La hepática común que es una de sus ramas, participa en la irrigación del
estómago.
79. Con respecto a la anatomía del duodeno, marque la respuesta correcta: a. Tiene
una distribución en forma de “C”, que rodea la cola del páncreas
b. La 3ra porción duodenal está contenida en la pinza vascular aortomesentérica
80. En el íleon se absorbe aproximadamente el 95% dea través de la circulación
enterohepática.
a. agua
c. sales biliares
81. La estimula el mecanismo paracrino de la secreción de ácido clorhídrico.
a. histamina
82.En la digestión de proteinas,es el principal estímulo para convertir el
pepsinógeno en pepsina. a. la gastrina
b. el pH ácido
83. Con respecto a la somatostatina, marque lo correcto:
a. Es secretada por las células S del intestino
Interviene en la fase intestinal de la secreción gástrica
84. En pecten anal, es una estructura comprendida entre: a. la línea pectínea y los
senos anales
d. la línea anocutánea y la línea pectínea

85.¿Cuál de las siguientes alternativas es una proenzima pancreática? a. Tripsina

1. b. Elastasa
2. c. Quimotripsinógeno
3. d. Amilasa
4. e. Procarboxipeptidasa C
86. En este paciente, el bloqueo farmacológico de los receptores H2 en la mucosa gástrica:
a) No tiene efecto sobre la secreción de ácido inducida por el vago
b) Evita la activación de adenilciclasa por gastrina
c) Inhibe la secreción de ácido inducida por gastrina y mediada por el vago
d) Causa un aumento en el transporte de potasio por las células parietales gástricas
Se validó la A :)
87. Si se considerara una gastrectomía total para curar la gastritis del paciente, cuál de
las siguientes sustancias ya no se produciría:
a) Gastrina
b) Quimiotripsina
c) Amilasa
d) Pepsinógeno

88. Un paciente hipertenso está tomando un medicamento bloqueador de receptores


alfa 1 adrenérgicos (prazosina) y como efecto secundario se queja de:
d) Lo escaso que es el medicamento
e) No tiene problemas en la salivación
c) Hiposalivación
d) Hipersalivación

89. Con respecto a las lesiones y enfermedades de la boca, marque lo correcto:


f) La leucoplasia se desprende al roce
b) la eritroplasia puede degenerar en adenocarcinoma
c) El muguet oral es una enfermedad bacteriana en inmunodeprimidos
d) la eritroplasia debe ser biopsiada
NOTA: fue validada la opción B ya que no es motivo del curso que sepamos el puto cáncer.

90. En este paciente, se puede asumir que la pancreatitis ha sido ocasionada por
una disminución en el efecto de:
g) Amilasa
h) Lipasa
c) Inhibidor de la tripsina
d) Entercinasa
91.Un efecto secundario en el estómago por la acción de la secretina es:
i) Disminución en la liberación de pepsinógeno
b) Menor actividad de la pepsina
c) Mayor paso de bicarbonato a sangre periférica
d) Aumento en la producción de factor intrínseco
92. Dentro de los factores protectores de la mucosa gástrica se pueden mencionar
múltiples protagonistas. Uno de ellos es:
j) CCK
k) Gastrina
c) Receptor muscarínico
d) Pepsina
93. La saliva puede tener una variedad de electrolitos en su composición. Entre ellos el
cloro, respecto al cual se puede afirmar:
l) Su mayor concentración se consigue con el flujo bajo
b) Su concentración no llega a ser tan alta como en el plasma
c) Con flujo alto su concentración es mayor que la del plasma
d) Su menor concentración se alcanza con flujo alto
94. En el síndrome de boca seca o síndrome de Sjogren, una de las
complicaciones asociadas es:
a) caries
b) Disminución de la acidez gástrica
c) Aumento en de la producción de saliva
d) Aumento del pH bucal
95. Estimula la producción de
saliva:
a) Vasodilatación periglandular
b) Atropina
c) Fatiga o cansancio
d) Expresión de miedo
96. El omeprazol actúa sobre la membrana de la célula
m) Basolateral/principal
n) apical/principal
o) Basolateral/parietal
d) apical/parietal
97. Para protegerse del entorno ácido, el Helicobacter pylori se autogenera un entorno
de pH menos ácido alrededor suyo, gracias a una enzima que alcaliniza su entorno local
mediante la conversión de:
a) urea en NH3
b) H2O y CO2 en ácido carbónico
c) NH3 en urea
d) H2CO3 en bicarbonato
98. La anemia perniciosa destruye las células:
p) mucosas del cuello
b) oxínticas
c) principales
d) mucosas superficiales
99. La célula mucosa del cuello gástrico produce:
a) Moco
b) ácido clorhídrico
c) pepsinógeno
d) Factor intrínseco
100.El aumento en la acidez del estómago producido principalmente por la infección
de Helicobacter pylori se debe a la disminución de:
a) Somatostatina
b) Bicarbonato por las glándulas de Brunner
c) Secretina
d) Colecistoquinina
101. De las siguientes sustancias secretadas por los órganos de este paciente, la
más alcalina es la secreción:
a) pancreática
b) Esofágica
c) Yeyunal
d) Salival
102.En cuanto a la gastritis de este paciente, se encontró que era producida por la
bacteria Helicobacter pylori. Esta bacteria sobrevive en el medio ácido del estómago
gracias a:
a) ácido clorhídrico
b) Toxina CagA
c) Ureasa
d) Jugo pancreático
103.La lengua está recubierta por epitelio:
c) pseudoestratificado columnar no queratinizado
b) plano estratificado no queratinizado
c) pseudoestratificado columnar ciliado
d) plano estratificado queratinizado
104.El esfínter anal interno tiene musculatura …….. y tiene control ……..
d) lisa / voluntario
b) lisa / involuntario
c) esquelética / simpático
d) esquelética / parasimpático

19) La arteria Aorta proporciona la irrigación al tubo digestivo ¿cuál de las siguientes
arterias proporciona la irrigación al ángulo cólico derecho?
a) mesentérica superior
b) mesentérica inferior
c) frénica inferior
d) tronco celiaco

20) Paciente de 26 años que le cuenta en su historia clínica que cada vez que almuerza a
los 20 min tiene deseo de defecar, le comenta que su hijo de 1 mes le pasa lo mismo
pero más intenso. Esto se explica por el reflejo …….., el cual está …… en el paciente
a) colicoileal / normal
b) colicoileal / alterado
c) gastrocólico / normal
d) gastrocólico / alterado

21) La región del estómago que se comunica con el duodeno es la


a) pilórica
b) cardias
c) cuerpo
d) fórnix

22) Acude a consulta un px que fue diagnosticado de úlcera péptica 3 días antes. Luego de
múltiples pruebas diagnósticas se concluye que el paciente presenta un tumor secretor
de gastrina, ¿cual de las siguientes situaciones estará incrementada?
a) distensión gástrica
b) inhibición del vaciado gástrico
c) secrecion de acido clorhidrico
d) inhibición de la secreción de pepsinógeno
23) En el sistema digestivo, el control del apetito está dado por un complejo sistema de
sustancias y órganos integradores, los cuales regulan la ingesta de alimentos. La ……
es una sustancia orexígena y es sintetizada por el ……
a) leptina / estómago
b) felina / intestino
c) leptina / estómago
d) grelina / estómago

24) Con respecto a la actividad eléctrica del sistema digestivo, marque la alternativa
correcta
a) corresponden a potenciales de acción que están presentes de forma continua
y le dan capacidad de peristalsis autónoma al sistema digestivo
b) la frecuencia de las ondas lentas NO se ve influenciada por la actividad neural ni
las
hormonas gastrointestinales
c) en el estómago las ondas lentas se dan en una frecuencia de 6 x min
d) las ondas lentas son cambios lentos y ondulantes del potencial en reposo
e) la frecuencia de las ondas lentas va de 6 a 12 ondas por minuto

25) Ante una lesión del IX pc, el músculo …… se altera en su función


a) palatogloso
b) estilofaríngeo
c) palatofaríngeo
d) constrictor superior

26) Un varón de 50 años es sometido a extirpación del duodeno y parte proximal del
yeyuno. La pérdida de estímulo hormonal en el páncreas para la secreción enzimática
se explica por la pérdida de células
a) parietales, productoras de factor intrínseco
b) K productoras de factor intrínseco
c) M productora de CCK
d) I productora de CCK

27) Marque la respuesta correcta:


A. El bronquio derecho constituye una de las estrecheces del esófago
B. Todos los órganos del sistema digestivo tienen capa serosa
C. La pared gástrica en el fondo es más delgada que en el cuerpo y antro
D. El esfínter de Oddi rodea a la papila menor duodenal

28) Marque la respuesta correcta en relación a la gastrina:


A. Al distenderse el estómago, se inhibe su producción.
B. Se estimula por la liberación de noradrenalina
C. Las células G son las productoras y se encuentran principalmente en el antro
gástrico
D. Las células G se encuentran principalmente en el fondo gástrico

29) Para poder morder una manzana, es necesario usar el siguiente músculo:
A. Milohiodeo
B. Tensor del paladar
C. Orbicular de los labios
D. Buccinador
30) Sustancia que inhibe la secreción y la motilidad del estómago prolongando el tiempo de
digestión:
A. Enteroglucagon.
B. Polipéptido pancreático
C. Péptido 1 similar al glucagón (GLP-1).
D. Péptido insulinotrópico dependiente de la glucosa (GIP).

31) El nacimiento de la arteria mesentérica superior se puede encontrar en cuál de los


cuadrantes abdominales:
A. Hipocondrio derecho
B. Hipogastrio
C. Epigastrio
D. Mesogastrio
32) Entre las múltiples causas de la Enfermedad por Reflujo Gastroesofágico, se puede
considerar también a una alteración en las del esfínter esofágico inferior:
A. Ondas secundarias
B. Contracciones tónicas
C. Ondas lentas
D. Glándulas subesofágicas

33) Producto de la alimentación, se producen diversas sustancias peptídicas, cininas y


bradicininas, las cuales permiten que:
A. Se produzca neovascularización en los territorios de las arterias abdominales
B. La acción de la lipasa pancreática se vea incrementada
C. El consumo de O2 del intestino aumente ligeramente
D. El flujo sanguíneo intestinal aumente hasta 8 veces

34) El dolor periumbilical o epigástrico en el inicio de una apendicitis aguda se debe a:


A. Estímulo del nervio vago.
B. Íleo secundario.
C. Irritación del peritoneo parietal.
D. Estímulo del sistema simpático.

35) El aumento en la actividad motora de la pared gástrica genera un aumento en los


niveles locales de qué sustancia en la microvasculatura:
A.Adenosina
B. Colecistoquinina CCK
C. Endotelina
D. Gastrina

36) ¿Cuál de los siguientes péptidos inhibe el vaciamiento gástrico?


A. Colecistoquinina
B. Péptido inhibidor gástrico
C. Motilina
D. Gastrina
37) Los músculos de la masticación que producen la retropulsión de la mandíbula son:
A. temporales [mas seguro]
B. maseteros
C. milohioideos
D. pterigoideos
38) En relación a la fisiología gástrica, marque lo correcto:
A. la cimetidina actúa en la región basolateral de la célula parietal
B. la marea alcalina se debe al paso de bicarbonato través de la membrana apical
de la célula principal
C. el cloro difunde hacia el exterior por la la región basolateral de la célula parietal
D. la salida de hidrogeniones a la luz es por difusión facilitada

39) Durante el sueño, la concentración de bicarbonato en la saliva:


A. Se eleva a niveles mayores que los del plasma
B. Aumenta
C. No tiene efecto
D. Disminuye

40) La secreción de saliva es importante en la fisiología digestiva. Su concentración de


potasio llega a ser menor que la del plasma cuando su secreción tiene un flujo:
A. Intermedio
B. Nunca
C. Bajo
D. Alto

42) Respecto a las glándulas salivales, marque lo incorrecto:


A. la glándula parótida produce secreción serosa
B. la glándula sublingual drena a través de conducto de Wharton
C. La glándula parótida drena a través del conducto de Stenon
D. la glándula sublingual tiene forma de garfio

43) Con respecto a la saliva, marque la respuesta correcta:


A. será hipertónica cuando el flujo es bajo
B. a mayor flujo, menor concentración de Na
C. a mayor flujo, menor concentración de cloro
D. el sistema simpático estimula su secreción
CI 3
44) En relación a la circulación hepática, marque lo correcto:
a) Los sinusoides hepáticos transportan sangre mixta
b) La vena porta proporciona el 50% de sangre al hígado
c) La vena porta se forma a partir de la vena esplénica y la mesentérica inferior
d) La arteria hepática deriva de la mesentérica superior

45) Dentro de las funciones de las células de Ito, marque lo incorrecto:


a) Sintetizan colágeno
b) Almacenan vitamina A
c) Se les llama células estrelladas
d) Pueden fagocitar patógenos y actúan como presentadoras de antígeno
46) Paciente con tumor neuroendocrino productor de secretina, debido a lo cual se puede
esperar que su secreción pancreática, comparada con la de una persona sana de bajo
flujo, tenga una concentración de:
a) Sodio aumentada
b) Igual
c) Bicarbonato aumentada
d) Potasio disminuida
47) El GALT se localiza en:
a) Lámina propia
b) submucosa
c) borde en cepillo
d) superficie de criptas de Lieberkühn

48) En relación a la histología hepática, marque lo correcto:


a) la zona 1 se afecta rápidamente en estados de hipovolemia y shock
b) La zona 1 se encuentra cercana a la vena central lobulillar
c) La zona 3 se encuentra más cerca a la vena central lobulillar
d) La zona 3 se encuentra más cerca al eje menor del acino hepático
49) El acino pancreático difere con el de las glándulas salivales en:
a) Contiene células centroacinares
b) No produce secreción serosa
c) El páncreas produce principalmente secreción mucosa
d) No tienen diferencias
50) Durante la digestión de las grasa, para que la lipasa actúe adecuadamente se requiere
que el pH aumento en la luz intestinal, lo cual es logrado, entre otros, por la secreción de
las células:
a) Del conducto interlobulillar
b) Centroacinares
c) Acinares
d) Alfa
51) La secreción de la colecistoquinina (CCK) se produce en la fase:
a) intestinal
b) En las 3 por igual
c) gástrica
d) Cefálica
52) ¿Por cuál de las siguientes células es secretada principalmente la pro enzima
procarboxipeptidasa?
a) Acinares del páncreas
b) Epiteliales del duodeno
c) Ductales del páncreas
d) Centro Acinares del páncreas
53) Una mujer de 43 años dolor en hipocondrio derecho e icterica. En la ecografía se
evidencia cálculos biliares. Estos cálculos lo más probable es que se encuentren
localizados en:
a) conducto colédoco
b) Conducto cístico
c) Vesícula biliar
d) Conducto pancreático secundario
ECU 1:
Estudiante de 21 años sufre de gastritis aguda ocasionada por comer en lugares poco
higenicos. Suele consumir caramelos (“chupar”) mientras esta en base hasta la tarde.
También toma regular cantidad de leche (grasa, lactora, proteinas), pues le calma el
dolor y el ardor que sitnete por la gastritis (tiene dispepsia y cuando toma la leche se le
pasa).
Incluso cuando puede, se toma dos vasos de agua frita y le calma la molestia. Ha decido
ir al medico para tratarse, pues ya no soporta el dolor, el cual esta seguro que los
síntomas se producen por elevada producion de HCl en el estomago, y por ello le ha
recetado ranitidina
1.1) El consumir caramelos eleva los niveles en sangre de una hormona cuya función es la
estimulación de las células.
- Beta del páncreas por GIP el cual es una incretina y por consiguiente estimula
las células pancreáticas

1.2) Consumir caramelos indirectamente actica la via:


-POMP/ CART saciedad

1.3) Consumo de leche produce indirectamente


- CCK inhibición del vaciamiento gástrico mayor tonicidad del esfínter pilórico

1.4) Cuando el px toma dos vasos de agua, genera indirectamente un aumento en la


liberación de:
- vaso de agua distención → g astrina → secreción de HCl

1.5) El uso de ranitidina bloquea el receptor H2 de la histamina en las células parietales, la


histamina llega a estas células por:
- histamina es una hormona paracrina por → difusión
**endocrina es por via hematógena y si fuera neuroendorina es por un NTs

1.6) Aumenta la secreción salival:


- noradrenalina a través de los receptores Beta 2
1.7) En este paciente con gastritis aguda debida a una alta producción de ácido clorhídrico,
sería lógico esperar que el píloro tenga un tono muscular:
- primero la secretina
- luego CCK
**ambas reguladores del HCl, Gatritis aguda debido a una alta producción de HCL
piloro estará aumentado (por la CCK)
1.8) Debido al uso de ranitidina, los valores de somatoestina en sangre:
- ranitidina disminuye acción de gastrina se quiere secretar mas no actúan
los inhibidores como la somatoestina somatoestina disminuye
1.9) El uso de atropina en este paciente:
- Inhibirá la acción de las prostaglandinas
- Aumentará la producción de ácido clorhídrico
- Disminuirá la acción del receptor CCK-B
-Aumentará el pH del estómago
ECU 2:
Niño de sexo masculino de 2 años de edad, sufre de estreñimiento desde el nacimiento
(1 deposición cada 3-4 días). Madre menciona que le estimula la defecación con un
termómetro rectal, y continuo uso de enemas y laxantes. Desde hace 6 meses comienza
con vómitos postprandiales. Los síntomas aumentan en frecuencia y magnitud y están
en relación con los episodios de estreñimiento. No refiere fiebre, tos, diarrea ni lesiones
cutáneas. Al examen físico presenta regular estado general, luce deshidratado.
Abdomen distendido, blando, depresible e indoloro. No se palpan masas abdominales.
Se permeabiliza el canal anal con termómetro rectal, encontrando cierta resistencia.
Salida de material fecal mal oliente en regular cantidad. Exámenes de laboratorio:
hemograma normal. Signos inflamatorios de fase aguda negativos. Alcalosis metabólica
leve en sangre
venosa. Radiografía con enema baritado muestra recto y colon sigmoides dilatados
(megacolon). Biopsia profunda: ausencia de células ganglionares en la muestra enviada.
Se realiza cirugía correctiva.

2.1) Durante la fase esofágica de la deglución, para un bolo alimenticio determinado, a


medida que avanza el bolo la fuerza de la contracción se hace más:
- hiperpolarizado
- fuerte
- dependiente de Ach
- debil
2.2) Cuando este paciente ingiera sus alimentos, se espera que al momento de pasar el
bolo alimenticio por el esfínter esofágico superior, la presión intraesofágica disminuya
en:
- la porción proximal al bolo
- el tercio medio del esófago
-el cardias
- el lugar donde se contraiga la muscular propia
2.3) Al examinar la orofaringe del paciente, uno puede hallar fácilmente la amígdala
palatina, pues esta se encuentra inmediatamente detrás del músculo:
-Palatogloso
- Palatofaringeo
- Hiogloso
- Elevador del velo del paladar

2.4) Con respecto a la defecación en este caso, marque la respuesta correcta:


- En posición de cuclillas, el músculo puborectal genera un ángulo más agudo
en el recto
- El sigmoides y el recto están inervados por el nervio vago
- La sensación de defecar sólo se da cuando el recto es ocupado por
heces y alcanzado el 80% de su capacidad
-El esfínter anal comprometido tiene inervación autónoma

2.5) En este paciente [hirschsprung] se considera que está abolido el reflejo:


- Coloileal
-Rectoesfinteriano
- Gastrocólico
- Relajación receptiva

2.6) No se espera que sea causa del vómito:


-Ayuno prolongado
- Estimulación faríngea y del glosofaríngeo
- Irritación de la mucosa gástrica
- Dolor intenso

ECU 3:
Paciente de 54 años con antecedentes de alcoholismo, gastritis crónica,
tabaquismo pesado, obesidad, cálculos biliares y cirrosis, es llevado a la
emergencia por dolor abdominal en epigastrio irradiado a la espalda y trastorno
del sensorio.
Al examen físico: presión arterial 85/50 mmHg, frecuencia cardíaca 100
latidos/min, frecuencia respiratoria 18 x minuto, temperatura axilar 36°C.
Conjuntivas pálidas, escleras ictéricas nevus arácnidos en tronco, distensión abdominal
marcada, cabeza de medusa, matidez desplazable en ambos flancos e hipogastrio,
dolor a la palpación de abdomen.
Tiempo de protrombina: 24 seg (testigo: 13 seg); TPT: 38 seg, glicemia: 165 mg/dL,
uremia: 20 mg/dL, ASAT: 76 UI/L, ALAT: 22 UI/L, albumina: 2,5 g/dL, bilirrubina total: 2,6
mg/dL, bilirrubina directa: 1,4 mg/dL, amilasa sérica 4000 U/L.
3.1) En esta paciente, al aumento de la amilasa sérica, se debe directamente a una lesión
de:
a) páncreas
b) vesícula y árbol biliar
c) estómago
d) hígado
3.2) Considerando que el paciente sufre de gastritis, se puede decir que la secreción de
ácido por la mucosa gástrica
a) involucra transporte activo de hidrogeniones
b) es realizada principalmente por células principales
c) es inhibida por antihistaminas tomadas por pacientes con rinitis alérgica
d) involucra la liberación de HCl de los gránulos zimógenos
3.3) El paciente tiene hemorragia digestiva alta por várices sangrantes como complicación.
Llegando a estar en shock hipovolémico por hemorragia masiva, se encontrara necrosis
hepática en:
a) zona 1
b) no se afectan los lobulillos hepáticos en hemorragia
c) zona 3
d) zona 2
3.4) El misoprostol, análogo de las prostaglandinas está mejor indicado en:
a) cicatrización de úlcera péptica duodenal
b) erradica el helicobacter pylori
c) tratar el sind de Zollinger ellison
d) prevenir daño por AINES
3.5) De las siguientes sustancias secretadas por los órganos de este paciente, la más
alcalina es la secreción:
- Esofágica
- Salival
- Yeyunal
-Pancreática
3.6) En este paciente, se puede asumir que la pancreatitis ha sido ocasionada por una
disminución en el efecto de:
- Lipasa
- Enterocinasa
- Amilasa
-Inhibidor de la tripsina
3.7) ¿Cuál de las siguientes sustancias es segregada por el páncreas?
-Amilasa
- Pepsina
- Quimiotripsina
- Tripsina
3.8) Cada vez que este paciente toma alcohol, la acidificación de la luz del duodeno:
-Disminuye el vaciamiento gástrico
- Aumenta la contracción del esfínter de Oddi
- Aumenta la secreción del ácido gástrico
- Disminuye la secreción pancreática del bicarbonato
SISTEMA
DIGESTIVO
(ME 154)
EXAMEN
FINAL
Ciclo 2018-01

1. Un niño de 2 años es llevado a la consulta por diarrea persistente y edema de las extremidades, además
falta de crecimiento y desarrollo en relación a su edad. Los análisis de sangre revelan que tiene
concentración plasmática baja de proteínas (hipoproteinemia). Durante la endoscopía duodenal, se
coloca colecistokinina (CCK) endovenosa y se recoge muestras del líquido duodenal; el resultado del
líquido confirma incapacidad para hidrolizar proteínas a un pH neutro, esta situación mejora al añadir
una pequeña cantidad de tripsina. El paciente probablemente esté sufriendo la falta congénita de
………….
(Unidad 4, sesión 26, logro 2: Explicar la Digestión y absorción de las proteínas y sus alteraciones)
a. Pepsinógeno
b. PEPT-1
c. Carboxipeptidasas
d. Enterocinasa

2. Experimentalmente se incrementa la velocidad de la secreción salival con una sustancia, el análisis


de la composición de esta saliva obtenida se espera encontrar…………..
(Unidad 3, sesión 17, logro 5 : Explica la Influencia de la velocidad del flujo salival en la composición de
la saliva)
a. Elevación de concentración de bicarbonato, sodio y potasio
b. Elevación de concentración de cloro, sodio y potasio
c. Disminución de concentración de potasio
d. Disminución de concentración de potasio y bicarbonato

3. Paciente varón de 46 años soltero, consulta por odinofagia y bajo de peso, tiene antecedente de
tuberculosis desde hace 3 meses y es fumador crónico (10 cigarrillos por día); al evaluar la cavidad
oral se identifica lesión blanquecina en el dorso de la lengua y paladar blando, las lesiones se
desprenden con el baja lengua dejando una base eritematosa. Esta lesión corresponde
probablemente a ……………………….…..
( Unidad 3, sesión18, logro 1-2 : Describe las enfermedades inflamatorias, infecciosas y proliferativas de
la cavidad oral)
a. Eritroplaquia
b. Candidiasis oral
c. Leucoplaquia vellosa
d. Fibroma en cavidad oral

4. Minero de 32 años de edad, que acude a centro


de salud por presentar de forma progresiva
desde hace 1 año dificultad para ingerir
alimentos sólidos y luego líquidos; refiere
regurgitaciones alimentarias y marcada
pérdida de peso (15 kilos). Radiografia baritada
de esófago como se muestra en la figura. El
presente caso se explica por……………….
(Unidad 2, sesión 12, logro 4: Identificar y
describir la función de los esfínteres
esofágicos)
a. Contracción incompleta del esfínter esofágico inferior
b. Dificultad para el inicio de la deglución
c. Relajación incompleta del esfínter pilórico
d. Relajación incompleta del esfínter esofágico inferior
5. Paciente mujer de 35 años acude a consulta por sensación de sequedad y lesiones en cavidad oral.
Al examen se observa atrofia de la mucosa, fisuras y úlceras; nota además sequedad e irritación de
la córnea y aumento del tamaño de las glándulas parotídeas. Su diagnóstico más probable es artritis
reumatoide; el hallazgo más probable en una biopsia de glándula parótida es……..….
(Unidad 3, sesión 18, logro 3: Describe las enfermedades más frecuentes de las glándulas salivales)
a. Hiperplasia de acinos glandulares serosos
b. Gran infiltración de linfocitos y células plasmáticas
c. Gran infiltrado de linfocitos y macrófagos
d. Presencia de acinos normales con hiperplasia de células ductales

6. Un paciente con anemia acude con su médico quejándose de episodios frecuentes de gastroenteritis.
Un análisis de sangre revela anticuerpos circulantes dirigidos contra células parietales gástricas. Su
anemia es atribuible a la hiposecreción de
………………………
(Unidad 3, sesión 20, logro 5: Gastritis crónica. Tipos de gastritis)
a. Factor intrínseco
b. Proteina R (haptocorrina)
c. Pepsinógeno
d. Ácido clorhídrico

7. Dos estudiantes deciden tomar un receso para comer una hamburguesa a la hora del almuerzo. Antes
de llegar a la cafetería, impulsos nerviosos provenientes del complejo vagal dorsal iniciarán la secreción
de ácido gástrico por la liberación dedesde el sistema nervioso entérico.
(Unidad 3, sesión 20, logro 2: Regulación de la secreción gástrica: estimulación, fases de la secreción)
a. Serotonina
b. Óxido nítrico
c. GRP (péptido liberador de gastrina)
d. Péptido intestinal vaso activo

8. Un niño de cuatro años de edad es llevado a la consulta por cuadros diarreicos frecuentes
caracterizados por heces pálidas, voluminosas y fétidas, presenta bajo peso y talla. Se mide la
concentración de cloruro en el sudor y se encuentra que sus valores son muy elevados. La alteración
más importante a nivel de células ductales del páncreas tiene relación directa con la conductancia
de…………
(Unidad 3, sesión 23, logro 5 Explica la Secreción pancreática: formación del jugo pancreático, influencia
de la velocidad de flujo y regulación)
a. Potasio
b. Bicarbonato
c. Sodio
d. Cloro

9. Una mujer de 50 años de edad que sufrió durante varios años resequedad de los ojos debida a
producción inadecuada de lágrimas es enviada con un gastroenterólogo para evaluación de pirosis
crónica. El examen endoscópico revela erosiones y tejido cicatrizal en la parte distal del esófago justo
por arriba del esfínter esofágico inferior. Las lesiones pueden atribuirse a la disminución de uno de los
siguientes componentes salivales:
(Unidad 3, sesión 17, logro 4: Explicar la Formación de la saliva y cuáles son sus componentes)
a. Bicarbonato
b. Lactoferrina
c. Ig A
d. Amilasa
10. Se evalúa los valores séricos de las siguientes sustancias a un paciente con enfermedad hepática
terminal; en este paciente se espera encontrar la combinación con la letra …………
(Unidad 3, sesión 22, logro 5: Describe las Pruebas de función hepática, la Insuficiencia hepática,
encefalopatía hepática e hipertensión portal)

Glucosa Amoniaco Albúmina


a. Aumentada Disminuida Disminuida
b. Disminuida Aumentada Aumentada
c. Aumentada Aumentada Aumentada
d. Disminuida Aumentada Disminuida

11. Una mujer de 35 años de edad HIV positiva, se presenta al médico con dolor abdominal en cuadrante
superior derecho e ictericia. La paciente refiere haber tenido múltiples episodios de ictericia durante
los últimos 10 años. Los exámenes para determinar hepatitis viral, dieron positivos para Hepatitis B,
siendo catalogado el caso como hepatitis crónica con alteración funcional. En un examen de sangre
¿cuál de los siguientes parámetros está disminuido?
(unidad 3, sesión 22, logro 5: Pruebas de función hepática, Insuficiencia hepática, encefalopatía
hepática e hipertensión portal)
a. Fosfatasa alcalina
b. Albumina
c. Bilirrubina
d. Tiempo de protrombina

12. En el reflejo peristáltico del intestino delgado, uno de los siguientes eventos sucede en
la porción oral del bolo alimenticio…………...
(Unidad 2, sesión 13, logro 4: Explicar la Motilidad del intestino delgado: Contracciones segmentarias y
peristálticas)
a. Disminución de 5 hidroxitriptamina desde las neuronas IPAN
b. Contracción del músculo longitudinal
c. Acción del péptido intestinal vasoactivo (VIP) en el músculo circular
d. Acción de acetilcolina en el músculo circular

13. Experimentalmente se coloca una dosis alta de secretina en la luz intestinal duodenal; como
consecuencia de esto, en el jugo pancreático de la misma luz intestinal se observa la disminución
de la concentración de …..………..
(Unidad 3, sesión 23, logro 5: Explica la Secreción pancreática: formación del jugo pancreático,
influencia de la velocidad de flujo y regulación)
a. Na+
b. Cl-
c. K+
d. HCO3-

14. Un varón de 58 años de edad con enfermedad de Crohn severo fue sometido a una resección ileal.
Después de la cirugía este paciente padecerá de esteatorrea, esto se explica porque …..………..
(unidad 4, sesión 26, logro 4: Explica las alteraciones en la Absorción de lípidos)
a. El pool de ácidos biliares se incrementa
b. Los quilomicrones no pueden formarse en el lumen intestinal
c. La micelas no pueden formarse
d. El páncreas no secreta lipasa

15. En un experimento se inserta un balón en el estómago de un voluntario, se infla poco a poco mientras
que se vigilan las presiones intraluminales. Aunque el volumen del balón aumenta considerablemente,
las presiones permanecen constantes. Esta relación volumen-presión se explica por la liberación local
de …………..
(Unidad 2, sesión 13, logro 1 Explica la Motilidad gástrica: relajación receptiva)
a. Acetil colina y gastrina
b. Colecistoquinina y óxido nítrico
c. Óxido nítrico y péptido inhibidor vasoactivo
d. Norepinefrina y óxido nítrico
16. La toxina del Vibrio cholerae causa diarrea debido a…….
(Unidad 4, sesión 27, logro 6: Explica el transporte hidroelectrolítico intestinal, toxina colérica)
a. La fosforilación del canal CFTR de los enterocitos de las vellosidades intestinales
b. El Incremento de la secreción de cloro por las células de la cripta intestinal
c. La inhibición de la producción de AMPc por las células epitelailes
d. El incremento de la absorción de agua y sodio a través de las uniones estrechas

17. ¿Cuál de las siguientes alternativas es una característica de la secreción exocrina del páncreas?
(Unidad 3, sesión 23, logro 5: Secreción pancreática: formación del jugo pancreático, influencia de la
velocidad de flujo y regulación)
a. Tiene una baja concentración de Cl- respecto al plasma
b. Es estimulada por la presencia de bicarbonato en el duodeno
c. La secreción enzimática es estimulada principalmente por la gastrina
d. Es hipotónica respecto al plasma

18. Una madre lleva a su hijo de dos años de edad a la sala de urgencias, estresada porque el niño deglutió
una moneda de 10 céntimos mientras la familia cenaba en un restaurante. El médico observa
mediante fluoroscopía que la moneda se halla en el estómago y asegura a la madre que la moneda se
eliminará con las heces. El médico recomienda utilizar la respuesta fisiológica que permitirá la
evacuación de la moneda del estómago al intestino ………….…..
(Unidad 2, sesión 13, logro 2: Explica la Motilidad gástrica: mezclado y vaciamiento)
a. Es por la relajación receptiva
b. Son los movimientos de mezcla y trituración
c. Es provocada por el ayuno
d. Es por la relajación del esfínter esofágico superior

19. Las estructuras en el hígado que permite que los productos metabólicos unidos a proteínas tengan
acceso a las membranas basolaterales de los hepatocitos, son…..
(Unidad 3, sesión 21, logro 4-5: Explica la Organización micro estructural del hígado)
a. Los Canalículos
b. Las fenestras sinusoidales
c. Las uniones intercelulares herméticas
d. Las células de Ito

20. La composición de la bilis es modificada conforme fluye por los conductillos biliares. Durante este
tránsito se espera que aumente la concentración de…….
(Unidad 3, sesión 22, logro 2: Describe la Secreción biliar, visión general del sistema biliar
extrahepático y composición de la bilis)
a. Ig A
b. Glucosa
c. Monómeros de ácido biliar
d. Vitamina A

21. Se mide experimentalmente el contenido gástrico de dos personas. La persona “A” tiene alto
contenido de grasa y la persona “B” tiene un contenido hipertónico ¿Cuál de las siguientes es
correcto respecto al vaciamiento gástrico? (Unidad 2, sesión 13, logro 2: Describe la Motilidad
y vaciamiento gástrico)
a. Hay ralentización del vaciado gástrico solo en “A”
b. El vaciamiento gástrico es más rápido en ambos
c. En ambos casos hay incremento de la motilina
d. Hay ralentización del vaciado gástrico en ambos casos

22. El examen endoscópico de un paciente con hipertensión portal grave revela venas tortuosas que
sobresalen hacia la luz del esófago. El paciente recibe tratamiento quirúrgico mediante la colocación
de una derivación que conecta la vena porta a la vena cava. Después de la operación el riesgo de
encefalopatía ………………….. y el riesgo de sangrado de várices ……………..
(Unidad 3, sesión 22, logro 5: Describe la Insuficiencia hepática, encefalopatía hepática e hipertensión
portal)
a. Aumentará/disminuirá
b. Disminuirá/disminuirá
c. Aumentará/aumentará
d. Disminuirá/aumentará
23. Un paciente varón de 18 años de edad acude al médico para sus exámenes de rutina. Sus resultados
de laboratorio muestran un valor de bilirrubina sérica de 4 mg/dl y una bilirrubina directa de 0,3
mg/dl. Las pruebas de función hepática son normales. La alteración que explica mejor este caso es
por la deficiencia de ………………..
(Unidad 3, sesión 22, logro 3: Explica la Producción y excreción de bilirrubina. Tipos de bilirrubina e
ictericia)
a. Transaminasas
b. Glucuronil transferasa
c. Hemo oxigenasa
d. La 7 alfa hidroxilasa

24. Un hombre de 57 años de edad es llevado a urgencias con hematemesis masiva rojo brillante, a su
llegada se halla inconciente con PA: 80/40 mm Hg y FC: 124 lat/min. Luce ictérico con presencia de
“arañas vasculares en el tórax anterior y extremidades”, abdomen distendido con signo de oleada
positiva. Se encuentra esplenomegalia y pérdida de la masa muscular en extremidades. La
anastomosis vascular responsable del sangrado en este paciente es ………….…..
(Unidad 3, sesión 21, logro 2: Describe las anastomosis porto sistémicas)
a. Arteria gástrica izquierda y vena ácigos
b. Vena gástrica izquierda y vena ácigos
c. Vena paraumbilical y vena epigástrica inferior
d. Vena gástrica izquierda y vena esofágica superior

25. Un estudiante de medicina está comiendo un plato de comida a base de champiñones, espárrago y
salsa de soya. El sabor umami contenido en todos estos alimentos actúa a nivel de los botones
gustativos estimulando ………………..
(Unidad 2, sesión 10, logro 5: Describe los tipos y mecanismos moleculares para la detección de los
sabores)
a. El ingreso de sodio
b. Un receptor acoplado a proteína G
c. Su receptor específico T1R3
d. El ingreso de hidrógeno

26. Un hombre de 22 años de edad se presenta al médico con una historia de 1 año de evolución
caracterizado por dolor recurrente en fosa iliaca derecha y diarrea. Manifiesta además pérdida
de peso de 8 kg durante este periodo. La colonoscopía revela múltiples lesiones en el ileon
terminal y colon. La biopsia de estas lesiones revela engrosamiento, inflamación y ulceración de
la mucosa. El diagnóstico más probable en este caso es…….
(Unidad 4, sesión 28, logro 5: Describe la Enfermedad inflamatoria intestinal. Generalidades, morfología
y características)
a. Sprue celiaco
b. Enfermedad de Crohn
c. Sindrome de colon irritable
d. Colitis ulcerativa

27. Una de las funciones del músculo señalado es:


(Unidad 2, sesión 8, logro 3: Describir el Piso
de la boca: estructuras blandas que la
conforman)
a. Eleva el paladar blando
b. Recibe inervación del nervio maxilar
c. Deprime el hioides cuando la mandíbula está fija
d. Deprime la mandíbula cuando el hioides está fijo

28. Varón de 61 años que consulta por dolor retro esternal intenso desde hace 6 horas y después
de vómitos intensos y repetidos; al examen se observa disnea, cianosis, hipotensión y signos
clínicos de shock. La radiografía simple de tórax muestra neumomediastino. El líquido en el
espacio pleural aspirado tiene alta concentración de amilasa. ¿Cuál de las siguientes alternativas
puede explicar este cuadro clínico?
(Unidad 3, sesión 18, logro 6: Describe algunas Enfermedades del esófago)
a. Sindrome de Mallory Weiss
b. Rotura espontánea de esófago
c. Neumotórax por probable herida penetrante
d. Perforación de ulcera gástrica de cara posterior, con complicación torácica
29. La secreción del ácido en la célula parietal gástrica se lleva a cabo por una ATPasa especifica que
intercambia hidrogeniones (H+) del citosol por…..
(Unidad 3, sesión 20, logro 1: Explica la Secreción del HCl y sustancias que la alteran)
a. Cl-
b. HCO3-
c. Na +
d. K+

30. En condiciones normales el ingreso de 600 ml de líquido es el estómago provoca un aumento de


presión intragástrica de unos 12 cm de H2O. Después de una vagotomía (corte del nervio vago) es de
esperar que el ingreso del mismo volumen de líquido provoque lo siguiente: …………………………………
(Unidad 2, sesión 13, logro 1: Describe la Motilidad gástrica: relajación receptiva)
a. Un aumento igual de la presión
b. Que no aumente la presión
c. Un aumento mayor de la presión
d. Una disminución de la presión

31. Una paciente de 30 años de edad es sometida a una cirugía en oído medio derecho por un problema
de otoesclerosis. Luego de la cirugía refiere alteración en la percepción de sabores. Al evaluar el caso
usted esperaría encontrar……….
(Unidad 2, sesión 10, logro 5: Describe la Irrigación e inervación de la lengua)
a. Alteración en la sensación del dolor y temperatura en el tercio posterior de la lengua
b. Alteración en la sensación del gusto en los dos tercios anteriores de la lengua
c. Alteración en la sensación del gusto en la punta de la lengua
d. Sensación del dolor, tacto y temperatura conservada en toda la lengua

32. ¿Cuál de las siguientes alterativas es correcta?


(Unidad 4, sesión 26 : Explica la digestión y absorción de nutrientes y sus alteraciones)
a. En el borde luminal, en cepillo, del intestino delgado, la absorción de sodio únicamente se
realiza asociada a la de glucosa.
b. El lugar principal para la absorción del hierro es el ileon
c. Las sales biliares desconjugadas son absorbidas preferentemente en el colon
d. El proceso de digestión y absorción de la vitamina B12 no se altera en insuficiencia pancreática.

33. En un paciente de 45 años de edad con colestasis biliar, se encuentra una elevación de los niveles
sanguíneos de fosfatasa alcalina hasta 3 veces la cifra normal. ¿Cuál de las siguientes alternativas
estará también elevada como evidencia del daño de la vía biliar?
(Unidad 3, sesión 22, logro 5: Pruebas de función hepática, Insuficiencia hepática, encefalopatía
hepática e hipertensión portal)
a. Tiempo de protrombina y albúmina sérica
b. Transaminasas hepáticas (ALT y AST)
c. Glucoronil transferasa
d. Gamma glutamil transpeptidasa

34. Revisando la angiografía de un hombre de 70 años en estudio por aneurisma de aorta abdominal el
radiólogo informa de la presencia de una oclusión completa de la arteria mesentérica inferior. El
paciente se encuentra completamente asintomático. ¿Cuál de las siguientes arterias se anastomosa
a la sistema arterial de la mesentérica inferior?
(Unidad 4, sesión 25, logro 1: Identifica la Arteria mesentérica superior e inferior, ramas y anastomosis)
a. Ileal
b. Cólica media
c. Sigmoideas
d. Cólica izquierda

35. Lactante de 3 meses de vida es atendido por presentar diarrea, se administra una solución de glucosa
y electrólitos por vía oral. La proteína de membrana apical que explica la capacidad de esta solución
para proporcionar aporte de glucosa e hidratación es ………..
(Unidad 4, sesión 26, logro 1: Explica la Digestión y Absorción de los hidratos de carbono. Alteraciones)
a. GLUT-5
b. SGLT-1
c. CFTR
d. GLUT-2
36. Paciente ha sufrido herida de bala en el abdomen, se le ha tenido que extirpar el segmento medio y
distal del ileon. En este caso la síntesis hepática de sales biliares estará …..…..
(Unidad 3, sesión 22, logro 4: Explica la formación, función y Circulación entero hepática de lasa sales
biliares)
a. Disminuida por inhibición de la colesterol 7 alfa hidroxilasa
b. Incrementada por estímulo de la enzima colesterol 7 alfa hidroxilasa
c. Incrementada por inhibición de la colesterol 7 alfa hidroxilasa
d. Sin cambios en el ritmo de síntesis

37. Un varón de 75 años ingresa al consultorio por presentar ictericia marcada de piel y las escleras. El
estudio del paciente mostró que presentaba un tumor que obstruía la totalidad del conducto hepático
común. ¿Cuál de las siguientes estructuras se encontrará dilatada en este paciente?
(Unidad 3, sesión 21, logro 6: Describir el árbol biliar intrahepático)
a. Conducto de Wirsung
b. Conductos de Hering
c. Conducto colédoco
d. Conducto cístico

38. Correlaciones las dos columnas y marque la fórmula correcta:


(Unidad 4, sesión 28, logro 1: Diarrea: definición, mecanismos: osmótica, secretoria y exudativa)
1. Enfermedad Hirschsprung( ) heces con moco y sangre
2. Diarrea osmótica( ) intolerancia a lactosa
3. Diarrea secretoria( ) aganglionosis congénita
4. Diarrea exudativa( ) canales de Cl- en las células de la cripta

a.- 4231b.- 1234c.- 2143d.- 4213

39. Respecto a la siguiente imagen que representa una estructura de la mucosa gástrica, la estructura con
número ………..
produce ……………………..
(Unidad 3, sesión 19, logro 4: La glándula fúndica. Funciones y tipos de células con sus características)
a. 3 / pepsina
b. 1 / Pepsinógeno
c. 4 / HCl y factor extrínseco
d. 2 / pepsinógeno

2
40. En un paciente con insuficiencia renal crónica, el déficit en la absorción de calcio a nivel del
enterocito se debe a lo siguiente:
(Unidad 4, sesión 26, logro 6: Explica la Absorción de calcio y hierro)
a. No se convierte la 25 hidroxicolecalciferol a 1,25 dihidroxicolecalciferol
b. No se convierte la 1,25 dihidroxicolecalciferol a 25 hidroxicolecalciferol
c. Se incrementa la producción de Calbindina
d. Existe un descenso de la alfa 25 hidroxilasa renal
CLAVES
EXAMEN
PARCIAL DE
SISTEMA
DIGESTIVO 2019
- 00

1. Varón de 30 años es traído a emergencia por agresión abdominal con arma de fuego
(pistola) y es sometido a laparotomía exploratoria, observándose isquemia del colon
ascendente y parte del colon trasverso ¿la lesión de cuál de las siguientes arterias
explicaría esta isquemia?
(unidad 1, sesión 2, logro 6: (Describe la irrigación visceral: arterias de tronco
celiaco, arteria mesentérica superior e inferior, topografía de superficie, órganos
por cuadrante)

a. Celiaca
b. Colónica derecha
c. Mesentérica inferior
d. Mesentérica superior

2. Respecto a las sustancias gastrointestinales que regulan la secreción pancreática;


marque la afirmación correcta:
(unidad 1, sesión 3, logros 2 y 3: describir las hormonas gastrointestinales: estímulos y
funciones)

a. La Secretina, es la hormona más importante para la secreción de bicarbonato por


las células acinares del páncreas
b. La acetilcolina es capaz de estimular la secreción enzimática y de bicarbonato del páncreas
c. La gastrina, es la hormona más importante para la secreción de enzimas pancreáticas
d. La colecistoquinina (CCK) estimula al páncreas solo para secreción enzimática

3. Ante una lesión del X par craneal, ¿cuál de los siguientes músculos mantiene conservada su
función?:
(unidad 2, sesión 08, logro 4: Paladar blando: componentes musculares)

a. Elevador del velo del paladar


b. Tensor del velo del paladar
c. Palatofaríngeo
d. Glosofaríngeo

4. Experimentalmente se utiliza atropina (anticolinérgico) para inhibir la secreción de


gastrina, sin embargo la secreción de esta hormona se sigue dando ante estímulos
vagales. Esta situación se explica porque la atropina:
(unidad 1, sesión 3, logro 3 : describir las hormonas gastrointestinales: estímulo y funciones de
la gastrina y colecistoquinina)

a. Bloquea parcialmente la bomba de protones en la célula G


b. Inhibe la acción de acetilcolina e histamina en la célula G
c. Solo inhibe la acción del péptido GRP en la célula G
d. No bloquea la acción del péptido GRP

5. Un varón de 50 años es sometido a extirpación del duodeno y parte proximal del


yeyuno. Esta situación ocasionaría la pérdida de las células ……….. , productoras
deque estimula la secreción de
bicarbonato por el páncreas.
(unidad 1, sesión 3, logro 3: describir las hormonas gastrointestianles: estímulos y
funciones de la secretina y péptido insulinotrópico dependiente de glucosa)

a. “S” / secretina
b. Parietales / secretina
c. “I” / colecistoquinina
d. “S” / colecistoquinina
6. Recién nacido que presenta tumoración abdominal a nivel del cordón umbilical
(fotografía). ¿cuál de las siguientes afirmaciones es correcta respecto a este
defecto en el desarrollo embriológico del intestino?: (unidad 1, sesión 5, logro 2:
identificar las anomalías del desarrollo del intestino medio)

a. Corresponde a una Gastrosquisis


b. Las vísceras se hallan cubiertas por piel
c. No está asociado a otras malformaciones
d. Se asocia a
malformaciones
cardiacas y del tubo
neural

7. Varón de 35 años acude a la emergencia por


trauma abdominal y se decida realizar una
laparoscopía exploratoria. El cirujano observa
la disposición de los órganos abdominales
como se representa en el siguiente esquema.
Esta disposición de órganos se explica por la
rotación(SMA=arteria mesentérica superior)
(unidad 1, sesión 5, logro 3: identificar las anomalías
del desarrollo del intestino medio: defectos de
rotación, estenosis y atresias)

a. anti horaria del intestino medio, en sólo 90°


b. incompleta del intestino medio (270°)
c. horaria del intestino medio
d. horaria del estómago

8. Se evalúa la expresión de la proteína Agrp en una persona con alteración del apetito;
lo correcto respecto a esta proteína es…..
(unidad 1, sesión 3, logro 4: Explica los mecanismos de control del apetito y saciedad )

a. Esta proteína es un potente anorexigénico


b. La mutación del gen que la codifica produce adelgazamiento
c. La sobre producción de la proteína lleva a obesidad por agonismo de receptores MC3 y MC4
d. La sobre producción de la proteína disminuye el apetito por antagonismo de receptores MC4

9. Juana cae de la bicicleta y se fractura la región anterior del hueso maxilar superior con
compromiso de la fosa incisiva. Al examen físico de la región esperaría encontrar
alteración en la sensibilidad de la encía …………………
(unidad 2, sesión 8, logro5: paladar: paladar duro y blando: irrigación e inervación)

a. bucal posterior
b. Lingual anterior
c. palatina anterior
d. palatina posterior
10. Recién nacido es atendido por el neonatólogo y luego entregado a su madre para dar
de lactar; la madre al dar de lactar observa coloración azulada de labios, acompañado
de tos persistente, dificultad respiratoria y distención abdominal. Se le intenta colocar
una sonda nasogástrica pero esta retorna a la cavidad oral en todos los intentos. ¿Cuál
de las siguientes anomalías del desarrollo es el más probable en este caso? (unidad 1,
sesión 4, logro 3: identificar las anomalías en el desarrollo del esófago: atresia y/o
fístula traqueo esofágica)

a. Estenosis esofágica proximal con Fístula traqueo esofágica distal


b. Atresia esofágica proximal con fístula traqueoesofágica distal
c. Atresia esofágica distal con fístula traqueoesofágica proximal
d. Fístula traqueoesofágica proximal y distal

11. ¿Cuál de los siguientes mecanismos ocurre durante la defecación?


(unidad 2, sesión 13, logro 6: motilidad del intestino grueso: contracciones
segmentarias, movimientos en masa, defecación y reflejo gastrocólico)

a. Contracción refleja del esfínter anal interno


b. En la posición de “cuclillas” el músculo puborectal se halla relajado
c. Relajación del esfínter anal externo por efectos del VIP y óxido nítrico
d. La materia fecal en el recto estimula la contracción del sigmoides por los nervios pudendos

12. La estructura número 4 (gráfico) corresponde a


……….… y está ………..
(unidad 2, sesión 9, logro 2:
Partes de un diente. Capas
del diente: Esmalte:
características y células que
lo producen)

a. el cemento / mineralizado en 90%


b. la dentina / formada por ameloblastos
c. el esmalte / formado
por células derivadas
del mesénquima
d. la dentina / formado por
células derivadas de la
cresta neural

13. Un paciente luego de un accidente sufre lesión del piso de la boca, se constata daño
del nervio “cuerda del tímpano”, en este caso se esperaría encontrar disminución de
lade la lengua
(unidad 2, sesión 10, logro 3: Irrigación e inervación de la lengua)

a. Motilidad en los dos tercios anteriores


b. Sensación del gusto en el tercio posterior
c. Sensación del gusto en los dos tercios anteriores
d. Sensibilidad al tacto en los dos tercios anteriores

14. ¿Cuál de las siguientes afirmaciones es la correcta sobre la gastrina?


(unidad 1, sesión 3, logro 1: reconocer las características de las sustancias reguladoras
gastrointestinales: hormonas, sustancias paracrinas y neurocrinas)

a. Produce atrofia de la mucosa gástrica


b. Es producida por la célula G del cuerpo gástrico
c. Es estimulada por la distensión gástrica y el Ph bajo
d. Actúa en la célula diana mediante su receptor CCk tipo B
15. Al recibir un paciente con signos de hipovolemia y antecedente de trauma en abdomen
por accidente de tránsito, usted identifica radiológicamente: lesión de primera vértebra
lumbar y signos de lesión en páncreas; durante la cirugía se observó pobre irrigación de
asas intestinales. El vaso afectado es la arteria ……..
(unidad 1, sesión 1, logro 6: reconocer las estructuras a nivel de L1, nivel de los principales vasos
sanguíneos)

a. esplénica
b. hepática común
c. mesentérica inferior
d. mesentérica superior

16. Un paciente sufre de daño a nivel del cuello con lesión muscular en la región de la
faringe. En el examen físico se determina dificultad para la elevación de la faringe y para
el cierre del itsmo de las fauces. En este caso, probablemente esté afectado el músculo:
(unidad 2, sesión 11, logro 2: Músculos de la faringe: identificación, constrictores y
longitudinales)

a. palatogloso
b. estilofarinfeo
c. palatofaringeo
d. constrictor inferior

17. Varón de 50 años a quien le realizan la curación de la segunda molar de la arcada


superior derecha. En un momento determinado, el paciente acusa de intenso dolor
de la pieza dentaria en tratamiento. La vía aferente del dolor viaja a través del
nervio …………
(unidad 2, sesión 9, logro 6: Inervación de los dientes)

a. trigémino V2
b. trigémino V3
c. naso palatino
d. palatino menor

18. La distención gástrica por los alimentos produce incremento de secreción de HCl mediante la
producción de
………….. que estimula a las células ……………. vía proteína ………..
(Unidad 1, sesión 3, logro 2: Describe las hormonas gastrointestinales: Estímulo y funciones de
la gastrina y colecistoquinina)

a. gastrina / parietal / Gq
b. gastrina / principal / Gs
c. acetilcolina / parietal /Gi
d. acetilcolina / principal / Gi

19. Un niño de tres años llega a emergencia con disfagia (dificultad para tragar), dolor retro
esternal, salivación y llanto. Se sospecha de ingesta de cuerpo extraño (moneda) en el
esófago; al ser evaluado se constata en una radiografía presencia de cuerpo extraño a
nivel de C6 (6° vértebra cervical). El cuerpo extraño estará suspendido a nivel del
estrechamiento producido por………..
(unidad 2, sesión 11, logro4: Esófago, características anatómicas, relación con órganos vecinos
y estrecheces)

a. el cayado aórtico
b. el hiato esofágico
c. el músculo cricofaríngeo
d. el bronquio principal izquierdo

20. La triada portal (arteria hepática, vena portal y conducto biliar común) está contenida en el
ligamento
…….……… y derivan embriológicamente del ……
(Unidad 1, sesión 1, logro 4: Identifica el peritoneo, mesenterio, omento y ligamentos,
retroperitoneo.)
a. hepato duodenal / mesenterio ventral
b. gastro esplénico / mesenterio dorsal
c. hepato gástrico / omento menor
d. falciforme / omento menor
21. En relación al movimiento de
peristaltismo del tubo
digestivo: en la flecha negra
del gráfico se produce la
liberación de ……………… a
nivel del músculo ………..
(unidad 2, sesión 7, logro 6:
Control hormonal y tipos de
movimiento)

a. noradrenalina, sustancia P y neuropéptido “ Y” / circular


b. acetilcolina y sustancia P / longitudinal
c. óxido nítrico y PIV / longitudinal
d. óxido nítrico y PIV / circular

22. Un paciente refiere no percibir algunos sabores, al examen físico se constata


alteración en la percepción de sabores y del dolor en el tercio posterior de la lengua
¿Cuál de los siguientes nervios estará alterada en su función?
(unidad 2, sesión 10, logro 5: Sabores, tipos y mecanismos moleculares para su detección)

a. Lingual (rama del V par)


b. Cuerda del tímpano (VII par)
c. Glosofaríngeo (IX par)
d. Hipogloso (XII par)

23. El gráfico detalla la


estructura de la pared del
tubo digestivo intestinal
¿Cuál de las siguientes
asociaciones es correcta?
(unidad 2, sesión 7, logro 1:
La pared y músculo liso
gastrointestinal )

a. “1” – peristaltismo
b. “2” – secreción enzimática
c. “3” – deriva del mesodermo
d. “4” – doble hoja de tejido graso

24. En el caso de un paciente con gastrinoma (tumor productor de gastrina), la presencia


de úlceras duodenales y erosión de la mucosa gástrica, se debe principalmente a…….
(unidad 1, sesión 3, logro 2: describir las hormonas gastrointestinales: estímulo y funciones de
la gastrina y colecistoquinina)

a. la acción directa de la gastrina sobre la célula principal


b. la sobre expresión de los receptores “G” en la célula parietal
c. el exceso de HCl por estímulo de receptores CCK-B en la célula parietal
d. el exceso de HCl por estímulo directo de receptores de acetilcolina en la célula parietal

25. El reflejo entero gástrico se caracterizan por:


(unidad 2, sesión 13, logro 6: Motilidad del intestino grueso: contracciones
segmentarias, movimientos en masa defecación y reflejo gastrocólico)

a. favorecer la motilidad gástrica gracias a la CCk


b. inhibir la motilidad gástrica y estimular la secreción ácida
c. movilizar grandes volúmenes desde el estómago al duodeno
d. originarse debido a la distensión duodenal y presencia del quimo ácido
26. Mauricio tiene dificultad para deprimir el paladar y elevar la parte posterior de la
lengua. En este caso estará afectado un músculo ………………., específicamente el
músculo …………….
(Unidad 2, sesión 10, logro 2: Músculos de la lengua: clasificación, identificación y sus
funciones)

a. intrínseco – longitudinal inferior


b. extrínseco – palatogloso
c. extrínseco – transverso
d. extrínseco – estilogloso

27. Una de las funciones del músculo señalado es:


(Unidad 2, sesión 8, logro 3: Describir el Piso de la boca:
estructuras blandas que la conforman)

a. deprimir la lengua
b. elevar el paladar blando
c. deprimir el hioides cuando la mandíbula está fija
d. deprimir la mandíbula cuando el hioides está fijo

28. Paciente varón de 30 años es evaluado por probable enfermedad de Chagas, cursa
con problemas de motilidad del colon; los estudios de biopsia determinan ausencia
de células ganglionares. Según el gráfico
¿cuál es la capa en la que se determina la ausencia de dichas células?
(unidad 1, sesión 2, logro 1: describir las generalidades de la estructura del tubo digestivo:
esófago, estómago intestino delgado y grueso)

a. Mucosa - 1
b. Muscular propia – 1
c. Muscular de la mucosa - 2
d. Muscular propia - 3

1 2

3
29. Paciente varón de 32 años, que acude a centro de salud por presentar de forma
progresiva desde hace 1 año dificultad para ingerir alimentos sólidos y luego líquidos;
refiere regurgitaciones alimentarias y marcada pérdida de peso (15 kilos). Radiografía
baritada (sustancia de contraste) de esófago se muestra en la figura. El presente caso
se explica por……………….
(Unidad 2, sesión 12, logro 4: Identificar y describir la función de los esfínteres esofágicos)

a. aumento de la peristalsis esofágica


b. relajación incompleta del esfínter pilórico
c. relajación incompleta del esfínter esofágico inferior
d. perdida de producción de PIV y
óxido nítrico en el esfínter
esofágico superior

30. En condiciones normales, el ingreso de 600 ml de líquido es el estómago provoca un


aumento de presión intragástrica de unos 12 cm de H2O. Después de una vagotomía
(corte del nervio vago) es de esperar que el ingreso del mismo volumen de líquido
ocasionede la presión intragástrica.
(Unidad 2, sesión 13, logro 1: Describe la Motilidad gástrica: relajación receptiva)

a. la disminución
b. la no variación
c. un aumento mayor
d. un aumento similar o igual
S
ISTEM
A
DIGEST
IVO
(ME
154)
CLAVES
EXAME
N
FINAL
Ciclo
201900

1. Un niño de 2 años es llevado a la consulta por diarrea persistente, edema de las


extremidades y falta de crecimiento en relación a su edad. Los análisis de sangre revelan que
tiene concentración plasmática baja de proteínas (hipoproteinemia). Como parte del estudio
se coloca colecistokinina (CCK) endovenosa y se recoge muestras del líquido duodenal por
endoscopía; el resultado del líquido confirma incapacidad para hidrolizar proteínas a un pH
neutro, esta situación mejora al añadir una pequeña cantidad de tripsina. El paciente
probablemente esté sufriendo la falta congénita de ………….
(Unidad 4, sesión 26, logro 2: Explicar la Digestión y absorción de las proteínas y sus alteraciones)
a. PEPT-1
b. pepsinógeno
c. enterocinasa
d. carboxipeptidasas

2. Paciente mujer de 35 años acude a consulta por sensación de sequedad y lesiones en


cavidad oral. Al examen se observa atrofia de la mucosa, fisuras y úlceras; nota además
sequedad e irritación de la córnea y aumento del tamaño de las glándulas parotídas. Su
diagnóstico más probable es artritis reumatoide; el hallazgo más probable en una biopsia
de glándula parótida es……..….
(Unidad 3, sesión 18, logro 3: Describe las enfermedades más frecuentes de las glándulas salivales)
a. Presencia de acinos normales con hiperplasia de células ductales
b. Gran infiltración de linfocitos y células plasmáticas
c. Hiperplasia de acinos glandulares serosos
d. Gran infiltrado de linfocitos y neutrófilos

3. Un hombre de 42 años de edad se presenta al médico con una historia de 1 año de evolución,
caracterizado por dolor abdominal bajo y diarreas con crisis sanguinolentas. Manifiesta
además pérdida de peso de 8 kg durante este periodo. La colonoscopía revela lesión difusa
en el colon con afectación del recto. La biopsia de estas lesiones revela adelgazamiento de
la pared, inflamación y ulceración de la mucosa y sub mucosa. El diagnóstico más probable
en este caso es:
(Unidad 4, sesión 28, logro 5: Describe la Enfermedad inflamatoria intestinal.
Generalidades, morfología y características)
a. sindrome de colon irritable
b. enfermedad de Crohn
c. colitis ulcerativa
d. sprue celiaco

4. Dos estudiantes deciden tomar un receso para comer una hamburguesa a la hora del
almuerzo. Antes de llegar a la cafetería, impulsos nerviosos provenientes del complejo vagal
dorsal iniciarán la secreción de ácido gástrico por la liberación dedesde el sistema nervioso
entérico.
(Unidad 3, sesión 20, logro 2: Regulación de la secreción gástrica: estimulación, fases de la
secreción)
a. Serotonina
b. Colecistoquinina
c. Péptido inhibidor vaso activo
d. GRP (péptido liberador de gastrina)
5. Un niño de cuatro años de edad es llevado a la consulta por cuadros diarreicos
frecuentes caracterizados por heces pálidas, voluminosas y fétidas; al examen físico
presenta bajo peso y talla para la edad. Se mide la concentración de cloruro en el sudor
y se encuentra que sus valores son muy elevados. La alteración más
importante a nivel de células ductales del páncreas tiene relación directa con la conductancia
de…………
(Unidad 3, sesión 23, logro 5 Explica la Secreción pancreática: formación del jugo
pancreático, influencia de la velocidad de flujo y regulación)
a. Bicarbonato
b. Potasio
c. Sodio
d. Cloro
6. Se evalúa los valores séricos de las siguientes sustancias a un paciente con enfermedad
hepática terminal; en este paciente se espera encontrar la combinación con la letra …………
(Unidad 3, sesión 22, logro 5: Describe las Pruebas de función hepática, la Insuficiencia
hepática, encefalopatía hepática e hipertensión portal)
Glucosa Amoniaco Albúmina

a. Aumentada Disminuida Disminuida

b. Disminuida Aumentada Aumentada

c. Aumentada Aumentada Aumentada

d. Disminuida Aumentada Disminuida

7. Una mujer de 35 años de edad HIV positiva, se presenta al médico con dolor abdominal
en cuadrante superior derecho e ictericia. La paciente refiere haber tenido múltiples
episodios de ictericia durante los últimos 10 años. Los exámenes para determinar hepatitis
viral, dieron positivos para Hepatitis B, siendo catalogado el caso como hepatitis crónica
con alteración funcional. En un examen de sangre ¿cuál de los siguientes parámetros está
disminuido?
(unidad 3, sesión 22, logro 5: Pruebas de función hepática, Insuficiencia hepática,
encefalopatía hepática e hipertensión portal)
a. Albumina
b. Bilirrubina
c. Fosfatasa alcalina
d. Tiempo de protrombina

8. En el reflejo peristáltico del intestino delgado ¿Cuál de los siguientes eventos sucede en la
porción caudal del bolo alimenticio?
(Unidad 2, sesión 13, logro 4: Explicar la Motilidad del intestino delgado: Contracciones
segmentarias y peristálticas)
a. Acción del péptido inhibidor vasoactivo (VIP) en el músculo circular
b. Acción del NO (óxido nítrico) en el músculo longitudinal
c. Contracción del músculo longitudinal interno
d. Acción de acetilcolina en el músculo circular

9. Un varón de 58 años de edad con enfermedad de Crohn severo fue sometido a una
resección ileal. Después de la cirugía este paciente padecerá de esteatorrea, esto se explica
porque …..………..
(unidad 4, sesión 26, logro 4: Explica las alteraciones en la Absorción de lípidos)
a. se inhibe la acción de la 7 alfa hidroxilasa
b. el pool de ácidos biliares se incrementa
c. hay mala absorción de ácidos biliares
d. el páncreas no secreta lipasa

10. En un experimento se inserta un balón en el estómago de un voluntario, se infla poco


a poco mientras que se vigilan las presiones intraluminales. Aunque el volumen del
balón aumenta considerablemente, las presiones permanecen constantes. Esta
relación volumen-presión se explica por la liberación local de …………..
(Unidad 2, sesión 13, logro 1 Explica la Motilidad gástrica: relajación receptiva)
a. acetil colina y gastrina
b. norepinefrina y óxido nítrico
c. colecistoquinina y óxido nítrico
d. óxido nítrico y péptido inhibidor vasoactivo

11. ¿Cuál de las siguientes alternativas es una característica de la secreción exocrina del páncreas?
(Unidad 3, sesión 23, logro 5: Secreción pancreática: formación del jugo pancreático, influencia
de la velocidad de flujo y regulación)
a. Es hipotónica respecto al plasma
b. Su mayor estímulo se da en la fase intestinal
c. Es estimulada por la presencia de bicarbonato en el duodeno
d. La secreción enzimática es estimulada principalmente por la secretina
12. Las estructuras en el hígado que permite que los productos metabólicos unidos a
proteínas tengan acceso a las membranas basolaterales de los hepatocitos, son…..
(Unidad 3, sesión 21, logro 4-5: Explica la Organización micro estructural del hígado)
a. los canalículos
b. las células de Ito
c. las fenestras sinusoidales
d. las uniones intercelulares herméticas

13. La composición de la bilis es modificada conforme fluye por los conductillos biliares.
Durante este tránsito se espera que aumente la concentración de…….
(Unidad 3, sesión 22, logro 2: Describe la Secreción biliar, visión general del sistema biliar
extrahepático y composición de la bilis)
a. Ig A
b. Glucosa
c. Protones
d. Vitamina A

14. Se mide experimentalmente el contenido gástrico de dos personas. La persona “A” tiene alto
contenido de grasa y la persona “B” tiene un contenido isotónico ¿Cuál de las siguientes es
correcto respecto al vaciamiento gástrico? (Unidad 2, sesión 13, logro 2: Describe la
Motilidad y vaciamiento gástrico)
a. Hay ralentización del vaciado gástrico solo en “A”
b. El vaciamiento gástrico es más rápido en ambos
c. Hay ralentización del vaciado gástrico solo en “B”
d. Hay ralentización del vaciado gástrico en ambos casos

15. El examen endoscópico de un paciente con hipertensión portal grave revela venas tortuosas
que sobresalen hacia la luz del esófago. El paciente recibe tratamiento quirúrgico mediante
la colocación de una derivación que conecta la vena porta a la vena cava. Después de la
operación el riesgo de encefalopatíay el riesgo
de sangrado de várices ……………..
(Unidad 3, sesión 22, logro 5: Describe la Insuficiencia hepática, encefalopatía hepática e
hipertensión portal)
a. disminuirá / disminuirá
b. disminuirá / aumentará
c. aumentará / disminuirá
d. aumentará / aumentará

16. Un paciente varón de 18 años de edad acude al médico para sus exámenes de rutina. Sus
resultados de laboratorio muestran un valor de bilirrubina sérica de 4 mg/dl y una
bilirrubina directa de 0,3 mg/dl. Las pruebas de función hepática son normales. La
alteración que explica mejor este caso es por la deficiencia de ………………..
(Unidad 3, sesión 22, logro 3: Explica la Producción y excreción de bilirrubina. Tipos de bilirrubina e
ictericia)
a. transaminasas
b. hemo oxigenasa
c. la 7 alfa hidroxilasa
d. glucuronil transferasa

17. Un hombre de 57 años de edad es llevado a urgencias con hematemesis masiva rojo
brillante, a su llegada se halla inconsciente con PA: 80/40 mm Hg y FC: 124 lat/min. Luce
ictérico con presencia de “arañas vasculares en el tórax anterior y extremidades”, abdomen
distendido con signo de oleada positiva. Se encuentra esplenomegalia y pérdida de la masa
muscular en extremidades. La anastomosis vascular responsable del sangrado en este
paciente es ………….…..
(Unidad 3, sesión 21, logro 2: Describe las anastomosis porto sistémicas)
a. vena gástrica izquierda y vena ácigos
b. arteria gástrica izquierda y vena ácigos
c. vena paraumbilical y vena epigástrica inferior
d. vena gástrica izquierda y vena esofágica superior
18. Un estudiante de medicina está comiendo un plato de comida a base de champiñones,
espárrago y salsa de soya. El estímulo del sabor umami contenido en todos estos alimentos
viaja a través del nervio………………..
(Unidad 2, sesión 10, logro 3: Describe la irrigación e inervación de la lengua)
a. Lingual
b. Hipogloso
c. Glosofaringeo
d. Cuerda del tímpano
19. Una paciente de 30 años de edad es sometida a una cirugía en oído medio derecho
por un problema de otoesclerosis. Luego de la cirugía refiere alteración sensitiva de
la lengua. Al evaluar el caso usted esperaría encontrar……….
(Unidad 2, sesión 10, logro 5: Describe la Irrigación e inervación de la lengua)
a. Alteración en la sensación del dolor y temperatura en el tercio posterior de la lengua
b. Alteración en la sensación del dolor en los dos tercios anteriores de la lengua
c. Alteración en la sensación del gusto en el tercio posterior de la lengua
d. Sensación del dolor, tacto y temperatura conservadas

20. En un paciente de 45 años de edad con colestasis biliar, se encuentra una elevación de los
niveles sanguíneos de fosfatasa alcalina hasta 3 veces la cifra normal. ¿Cuál de las
siguientes alternativas estará también elevada como evidencia del daño de la vía biliar?
(Unidad 3, sesión 22, logro 5: Pruebas de función hepática, Insuficiencia hepática,
encefalopatía hepática e hipertensión portal)
a. Tiempo de protrombina y albúmina sérica
b. Transaminasas hepáticas (ALT y AST)
c. Gamma glutamil transpeptidasa
d. Glucoronil transferasa

21. Experimentalmente se incrementa la velocidad de la secreción salival con una


sustancia, en el análisis de la composición de esta saliva obtenida se espera
encontrar…………..
(Unidad 3, sesión 17, logro 5 : Explica la Influencia de la velocidad del flujo salival en la composición
de la saliva)
a. disminución de la concentración de bicarbonato que supera la concentración plasmática
b. aumento de la concentración de cloro y sodio que supera la concentración plasmática
c. aumento de la concentración de bicarbonato que supera la concentración plasmática
d. disminución de concentración de potasio y bicarbonato

22. Lactante de 3 meses de vida es atendido por presentar diarrea, se administra una
solución de glucosa y electrólitos por vía oral. La proteína de membrana apical
que explica la capacidad de esta solución para proporcionar aporte de glucosa e
hidratación es ………..
(Unidad 4, sesión 26, logro 1: Explica la Digestión y Absorción de los hidratos de carbono.
Alteraciones)
a. CFTR
b. SGLT-1
c. GLUT-2
d. GLUT-5

23. Paciente ha sufrido herida de bala en el abdomen, se le ha tenido que extirpar el


segmento medio y distal del ileon. En este caso la síntesis hepática de sales biliares
estará …..…..
(Unidad 3, sesión 22, logro 4: Explica la formación, función y Circulación entero hepática de lasa
sales biliares)
a. Sin cambios en el ritmo de síntesis
b. Disminuida por inhibición de la enzima colesterol 7 alfa hidroxilasa
c. Incrementada por estímulo de la enzima colesterol 7 alfa hidroxilasa
d. Incrementada por inhibición de la enzima colesterol 7 alfa hidroxilasa

24. Un varón de 75 años ingresa al consultorio por presentar ictericia marcada de piel y las
escleras. El estudio del paciente mostró que presentaba un tumor que obstruía la
totalidad del conducto hepático común. ¿Cuál de los siguientes conductos se encontrará
dilatado en este paciente?
(Unidad 3, sesión 21, logro 6: Describir el árbol biliar intrahepático)
a. de Wirsung
b. de Hering
c. colédoco
d. cístico
25. Correlaciones las dos columnas y marque la fórmula correcta:
(Unidad 4, sesión 28, logro 1: Diarrea: definición, mecanismos: osmótica, secretoria y exudativa)

1. Enfermedad Hirschsprung( ) heces con moco y sangre


2. Diarrea osmótica( ) intolerancia a lactosa
3. Diarrea secretoria( ) aganglionosis congénita
4. Diarrea exudativa( ) canales de Cl- en las células de la cripta a.-

4231b.- 1234c.- 2143d.- 4213

26. La vena umbilical obliterada del hígado después del nacimiento se transforma en el ligamento:
(Unidad 3, sesión 21, logro 1: Hígado: relación con la pared abdominal, caras, lóbulos, ligamentos ,
hilio hepático)
a. cruzado
b. redondo
c. coronario
d. falciforme

27. Llega a su guardia nocturna una madre que trae a su RN masculino de 2 semanas de vida
con mal estado general y sequedad de mucosas. Usted observa que lacta ávidamente, pero
a las 2 horas presenta vómito postprandial no bilioso en proyectil. Al realizar la historia
clínica, descubre que el lactante recibió profilaxis con macrólidos para tos ferina. Usted
sospecha principalmente en:
(Unidad 1, sesión 4, logro 4: Desarrollo y anomalías del intestino anterior)
a. estenosis pilórica hipertrófica congénita
b. fistula traqueo esofágica
c. estenosis duodenal
d. atresia duodenal

28. En la regulación del apetito y la saciedad, la estimulación experimental crónica del


núcleo ventro medial del hipotálamo producirá:
(Unidad 1, sesión 3, logro 4: explica los mecanismos de control del apetito y saciedad)
a. afagia
b. obesidad
c. hiperfagia
d. activación de neuronas relacionadas a NPY

29. Paciente mujer de 25 años acude por dolor en fosa ilíaca derecha que empeora al toser o
caminar, asociada a náuseas y vómitos por lo cual acude a emergencia. Dos días después de
realizarle una apendicectomía, la paciente desarrolla fiebre alta (39 °C), está hipotensa y
presenta dolor abdominal. La laparotomía exploratoria muestra un gran volumen de sangre
en la cavidad peritoneal por lesión de un vaso producida durante la apendicectomía.
¿Cuál de las siguientes arterias debe ligarse para detener la hemorragia?
(Unidad 4, sesión 27, logro 4: Irrigación arterial del colon, recto y conducto anal)
a. cólica derecha y arteria rectal superior.
b. ileocólica y arteria cólica media.
c. mesentérica superior.
d. ileocólica.

30. La onda peristáltica secundaria del esófago se caracteriza por ser originada ………
(unidad 2, sesión 12, logro 3: Motilidad esofágica: fases y características)
a. por el plexo de meissner del esófago
b. por el plexo mientérico del esófago
c. por el reflejo de la deglución
d. durante la masticación

31. ¿Cuál de los siguientes es una causa de ictericia con bilirrubina conjugada aumentada?
(Unidad 3, sesión 22, logro 3: Producción y excreción de bilirrubina. Tipos de bilirrubina, ictericia)
a. Ictericia del recién nacido
b. Obstrucción del colédoco
c. Anemia hemolítica
d. Gran hematoma
32. En relación a la absorción de nutrientes, la absorción de dipéptidos y tripéptidos a nivel
de las células epiteliales del intestino delgado, se da principalmente debido a:
(Unidad 4, sesión 26, logro 2: Digestión y absorción de las proteínas. Alteraciones)
a. el incremento de los canales de Cl- en la membrana apical
b. la gradiente de bicarbonato en la membrana basal
c. la gradiente de iones H+ en la membrana apical
d. la gradiente de Na+ en la membrana apical

33. Paciente de 20 años es traído a la emergencia por presentar diarreas desde hace 2 días.
Familiar refiere que las deposiciones son líquidas y abundantes, al examen luce
deshidratado y se plantea que la diarrea es producida por una toxina que estimula la
transformación de ATP a AMPc con apertura de canales de Cl- y pérdida de agua. El tipo de
diarrea más probable es:
(Unidad 4, sesión 28, logro 1: Diarrea: definición , mecanismos: osmótica, secretoria y supurativa)
a. osmótica
b. exudativa
c. secretoria
d. por intolerancia a lactosa

34. Un niño fue operado por una obstrucción intestinal, observándose la presencia de
divertículo de Meckel. Según lo referido, marque lo correcto:
(Unidad 1, sesión 5, logro 2: identifica las anomalías del desarrollo del intestino medio: onfalocele
y gastrosquisis (diferencias), Divertículo de Meckel)
a. el 50% de la población lo presenta
b. se localiza en el íleon muy cerca al yeyuno
c. puede poseer tejido gástrico o pancreático
d. se produce por una mala rotación de los intestinos

35. Marque la
alternativa correcta
respecto a la
estructura marcada
en el gráfico:
(Unidad 3, sesión 22,
logro 2: Secreción biliar.
Visión general del
sistema biliar
extrahepático y
composición de la bilis)

a. Se halla a 2
centímetros
debajo de la
papila
duodenal
mayor
b. Llega el conducto
colédoco y
pancreático
principal
c. Llega el
conducto
hepático
común y
pancreático
principal
d. Llega el conducto pancreático accesorio

36. ¿Cuál de las siguientes moléculas se encontrará aumentada en el citoplasma de las


células parietales de un paciente con sindrome de Zollinguer Ellison?
(Unidad 3, sesión 20, logro 4: Enfermedad ulcerosa péptica: úlcera gástrica, duodenal.
síndrome de Zollinger – Ellison)
a. Péptido liberador de gastrina (GRP)
b. Proteína G estimulante (GS)
c. Inositol Trifosfato (IP3)
d. AMP cíclico (AMPc)

37. Los fármacos inhibidores de la bomba de protones, actúan bloqueando la ………..……..


(Unidad 3, sesión 20, logro 3: Regulación de la secreción gástrica: inhibición, Secreción de
pepsinógeno y factor intrínseco)
a. anhidrasa carbónica
b. ATPasa H+/K+ en la membrana luminal
c. ATPasa H+/K+ en la membrana basolateral
d. ATPasa Na+/K+ en la membrana basolateral
38. Un paciente fue diagnosticado de gastritis autoinmune, ¿cuál de las siguientes
alternativas es FALSA respecto a esta enfermedad?
(Unidad 3, sesión 20, logro 5: Gastritis crónica: helicobacter pylori, autoinmune. Tipos de gastritis)
a. Afecta principalmente el fondo y cuerpo gástrico
b. Se produce hiperplasia de células G secundaria a la aclorhidria
c. El propio sistema inmune destruye principalmente las células parietales
d. Se produce atrofia de la mucosa, aclorhidria, hipergastrinemia y déficit de vitamina B6

39. Marque la correlación correcta:


(Unidad 3, sesión:18, logros:1 y 2: Describe las enfermedades inflamatorias/infecciosas y
proliferativas de la cavidad oral)

1. Herpes virus( ) En relación al abuso de antibióticos


2. Candidiasis oral( ) Lesiones vesiculares como racimo de uvas
3. Eritroplaquia( ) Mega esófago
4. Enfermedad de Chagas( ) Lesión pre cancerígena

a.- 2431b.- 1234c.- 4123d.- 2143

40. En un paciente con insuficiencia renal crónica, el déficit en la absorción de calcio a nivel
del enterocito se debe a lo siguiente:
(Unidad 4, sesión 26, logro 6: Explica la Absorción de calcio y hierro)
a. No se convierte la 25 hidroxicolecalciferol a 1,25 dihidroxicolecalciferol
b. No se convierte la 1,25 dihidroxicolecalciferol a 25 hidroxicolecalciferol
c. Existe un descenso de la alfa 25 hidroxilasa renal
d. Se incrementa la producción de Calbindina
SISTEMA
DIGESTIVO
(ME154) EXAMEN
PARCIAL 2019 01

Profesores : Alfaro Salazar, Herberth Romulo; Callata Caceres, Gunter; Cayo Quiñe, Alexandra Mariel;
Correa Borit, Jorge Mauricio; Cruz Cutty, Lourdes Marylin; Guzmán Calderón, Gerly Edson;
Jáuregui Farfán, Jorge Jesús; Mayor Zevallos, Otto Alberto; Montoya Suárez, José Luis;
Palacios Bazan, Enrique Elias; Robles Pino, Alexander Anibal; Wong Bravo, Juan Carlos
Sección : Todas las secciones
Duración : 50 minutos.
Indicaciones:
Lea atentamente cada pregunta antes de responder:
- Se prohíbe el uso del celular y cualquier dispositivo electrónico.
- Está prohibido intercambiar materiales.
- Coloque su código de alumno en la tarjeta de respuestas. Si su código contiene una letra reemplácela
por un valor numérico siguiendo la siguiente equivalencia: A=9, B=8, C=7, D=6, E=5, F=4, G=3.
- Traslade sus respuestas a la tarjeta, llenando los círculos de manera completa con lapicero negro o
azul. Está prohibido el llenado con lápiz, lapicero de otro color o con lapicero de tinta borrable.
- Sea cuidadoso en el llenado de la tarjeta de respuestas, pues solo esta tiene validez para la calificación.
- Al terminar su examen avise al docente a cargo, no se levante de su sitio; debe entregar la hoja de
respuestas con la carátula del examen, este cuadernillo de preguntas se lo llevará cada estudiante.

1. ¿Cuál de las glándulas salivales es responsable del mayor porcentaje del volumen de la saliva en
condiciones basales?
a. Parótida
b. Sub
pala
tina
s c.
Subl
ingu
laes
d.
Sub
max
ilare
s

2. La lengua está recubierta por epitelio:


a. pseudoestratificado columnar no queratinizado
b. plano estratificado no queratinizado
c. pseudoestratificado columnar ciliado
d. plano estratificado queratinizado

3. El esfínter anal interno tiene musculatura …………… y tiene control ………………….


a. lisa / voluntario
b. lisa / involuntario
c. esquelética / simpático
d. esquelética / parasimpático

4. La arteria aorta proporciona la irrigación al tubo digestivo ¿cuál de las siguientes arterias proporciona
la irrigación al ángulo cólico derecho?
a. Mesentérica superior
b. Mesentérica inferior
c. Frénica inferior
d. Tronco celiaco

5. Paciente de 26 años que le cuenta en su historia clínica que cada vez que almuerza, a los 20 minutos
tiene deseo de defecar. Le comenta que su hijo de 1 mes le pasa lo mismo pero más intenso. Esto se
explica por el reflejo
…………………, el cual estáen el paciente.
a. colicoileal / normal
b. colicoileal / alterado
c. gastrocolico / normal
d. gastrocolico / alterado
6. La región del estómago que se comunica con el duodeno se denomina:
a. pilórica
b. cardias
c. cuerpo
d. fórnix

7. Acude a consulta un paciente que fue diagnosticado de ulcera péptica 3 días antes. Luego de múltiples
pruebas diagnósticas, se concluye que el paciente presenta un tumor secretor de gastrina ¿Cuál de las
siguientes situaciones estará incrementada?
a. Distención gástrica
b. Inhibición del vaciado gástrico
c. Secreción de ácido clorhídrico (HCl)
d. Inhibición de la secreción de pepsinógeno

8. En el sistema digestivo, el control del apetito esta dado por un complejo sistema de sustancias y
órganos integradores los cuales regulan la ingesta de alimentos. Laes una sustancia oroxígena y es
sintetizada
por el ……………………..
a. leptina / intestino
b. grelina / intestino
c. leptina / estómago
d. grelina / estómago

9. Sobre el control autónomo del sistema digestivo, marque la alternativa correcta:


a. La inervación dada por el sistema simpático es de tipo preganglionar.
b. El sistema parasimpático usa como neurotransmisores a la acetilcolina y la noradrenalina.
c. El nervio vago (par craneal X) le da inervación simpática a la mayoría del sistema digestivo.
d. En el sistema simpático, los nervios responsables hacen una primera sinapsis en ganglios
próximos al órgano a inervar.
e. En la inervación de tipo parasimpático, solo interviene el plexo submucoso, sin embargo,
en la de tipo simpático intervienen tanto el submucoso como el mientérico.

10. Con respecto a la actividad eléctrica del sistema digestivo, marque la alternativa correcta
a. Corresponden a potenciales de acción que están presentes de forma continua y le dan la
capacidad de perístasis autónoma al sistema digestivo.
b. La frecuencia de las ondas lentas no se ve influenciada por la actividad neural
ni las hormas gastrointestinales.
c. En el estómago las ondas lentas se dan en una frecuencia de 6 por minuto.
d. Las ondas lentas son cambios lentos y ondulantes del potencial en reposo.
e. La frecuencia de las ondas lentas va de 6 a 12 ondas por minuto.

11. Ante una lesión del IX par craneal, el músculo…se altera en su función.
a. palatogloso
b. estilofaríngeo
c. palatofaríngeo
d. constrictor superior

12. Un varón de 50 años es sometido a extirpación de duodeno y parte proximal de yeyuno. La pérdida
de estímulo hormonal en el páncreas para la secreción enzimática se explica por la pérdida de las
células ……………………
a. Parietales, productoras de factor intrínseco
b. “K” productoras de factor intrínseco
c. “M” productoras de CCK
d. “I” productoras de CCK
13. Respecto al mecanismo de la defecación ¿Cuál de las siguientes afirmaciones es correcta?
a. Se produce contracción refleja del esfínter anal interno
b. Se produce contracción o relajación del esfínter anal externo por señales de la corteza
cerebral
c. La presencia de materia fecal en el recto estimula la contracción del sigmoides por los
nervios pélvicos simpáticos
d. En la posición de “cuclillas” el músculo puborectal se halla contraído favoreciendo la
evacuación de la materia fecal

14. Un niño de tres años llega a emergencia con disfagia (dificultad para tragar), salivación y llanto. Se
sospecha de ingesta de cuerpo extraño: moneda en el esófago; al ser evaluado se constata en una
radiografía presencia de cuerpo extraño a nivel de C6 y C7 (6° y 7° vértebra cervical). El cuerpo extraño
estará suspendido a nivel del estrechamiento producido por el ………..
a. cayado aórtico
b. hiato esofágico
c. músculo cricofaríngeo
d. bronquio principal izquierdo

15. En el caso de un paciente con un tumor productor de gastrina, la presencia de úlceras duodenales y
erosión de la mucosa gástrica se debe principalmente a…….
a. la acción paracrina de la gastrina sobre la célula parietal
b. el exceso de HCl por estímulo de receptores CCK-B en la célula parietal
c. la sobre expresión de los receptores “G” para gastrina en la célula parietal
d. el exceso de HCl por estímulo directo de receptores “H” en la célula parietal

16. La onda peristáltica secundaria del esófago se caracteriza por ser originada ………
a. por el plexo de submucoso del esófago
b. por el plexo mientérico del esófago
c. por el reflejo de la deglución
d. durante la masticación

17. Marque lo correcto sobre las ondas lentas en el tubo digestivo


a. No son despolarizaciones
b. Son potenciales de acción subumbrales
c. Se constituyen de despolarizaciones y repolarizaciones
d. Son rítmicas y generadas por el sistema nerviosos autónomo

18. Recién nacido que presenta protrusión de contenidos abdominales los cuales no están cubiertos por
peritoneo y salen de la cavidad abdominal a través de un defecto de la pared. ¿Cómo se denomina a
la afección que presenta este paciente?
a. Onfalocele
b. Atresia biliar
c. Gastrosquisis
d. Divertículo de Meckel

19. Experimentalmente se utiliza atropina (anticolinérgico) para inhibir la secreción de gastrina, sin
embargo, la secreción de esta hormona se sigue dando ante estímulos vagales. Esta situación se
explica porque la atropina:
a. no bloquea la acción del péptido GRP
b. solo inhibe la acción del péptido GRP en la célula G
c. inhibe la acción de acetilcolina e histamina en la célula G
d. bloquea parcialmente la bomba de protones en la célula G

20. Niña de 4 días es llevada a la emergencia pediátrica por presentar llanto constante, la madre refiere
coloración azulada de labios al momento de lactar, acompañado de tos persistente y dificultad
respiratoria así como distención abdominal. Se le coloca sonda nasogástrica para alimentación
notando que retorna a la cavidad oral en todos los intentos. ¿Cuál es la anomalía del desarrollo en
este caso?
a. Solo fístula traqueo esofágica
b. Fístula traqueo esofágica proximal y distal
c. Atresia esofágica proximal con fístula traqueo esofágica distal
d. Atresia esofágica distal con fístula traqueo esofágica proximal
21. Paciente varón de 36 años es traído a la emergencia luego de sufrir un accidente de tránsito, presenta
traumatismos múltiples en cabeza y tronco. Al examen físico se evidencia hematoma en hemicara
izquierda, ligera protrusión y caída del lado izquierdo del maxilar inferior, por lo que se realiza una
tomografía donde se halla una fractura de la apófisis coronoides del maxilar inferior. ¿Qué músculo
está relacionada directamente con esta situación?
a. Masetero
b. Temporal
c. Buccinador
d. Pterigoideo medial

22. Un paciente refiere no percibir algunos sabores. Al examen físico constata alteración del sabor dulce y
umami.
¿Cuál de los siguientes nervios estará alterada su función?
a. Cuerda del tímpano (VII par)
b. Lingual (rama del V par)
c. Glosofaríngeo (IX par)
d. Hipogloso (XII par)

23. A los pocos días de nacido, regresa a neonatología un niño con problemas de motilidad del colon; los
estudios determinan ausencia congénita de células ganglionares. Según el gráfico ¿cuál es la capa en
la que se determina la ausencia de dichas células?
a. Mucosa - 2
b. Muscular propia - 2
c. Muscular propia - 3
d. Muscular de la mucosa - 3

1 2

24. Con respecto al control autonómico en el tracto gastrointestinal y en relación a su fisiología. ¿Cuál
es la función del sistema nervioso parasimpático en el tracto gastrointestinal?
a. Inhiben la contracción muscular y estimulan la secreción de sustancias a nivel de la submucosa
b. Estimulan la contracción muscular y estimulan la secreción de sustancias a nivel de la mucosa
c. Inhiben la contracción muscular e inhiben la secreción de sustancias a nivel de la submucosa
d. Estimulan la contracción muscular e inhiben la secreción de sustancias a nivel de la mucosa

25. Un estudiante que está preocupado por su examen parcial, no ha desayunado ni almorzado; cuando
al fin ingiere alimentos, esto le provoca el aumento de los movimientos musculares del tracto
gastrointestinal y la sensación de defecar. ¿Qué reflejo se ha activado?
a. Entero-gástrico
b. Gastro-cólico
c. Cólico-ileal
d. Ileo-ileal
26. ¿De qué par craneal es rama el nervio palatino mayor?
a. Vago
b. Hipogloso
c. Trigémino
d. Palatogloso

27. ¿En cuál de las fases de la deglución la epiglotis separa la vía respiratoria de la digestiva?
a. oral
b. laríngea
c. faríngea
d. esofágica

28. Los péptidos intestinales se pueden clasificar como sustancias endocrinas, neurocrinas y paracrinas,
dentro de las paracrinas se encuentran la somastotatina e histamina. Marque la respuesta correcta
a. La somastotatina es sintetizada por las células B de la mucosa gástrica
b. La histamina actúa estimulando su receptor tipo H1 en la mucosa gástrica
c. La histamina es sintetizada por células de tipo paracrino de las glándulas gástricas
d. La somatostatina presenta dentro de sus funciones la estimulación de la secreción de H+

29. En relación a los órganos intraabdominales y sus estructuras de fijación, elija la


alternativa correcta a.El mesenterio permite la suspensión e irrigación de los
órganos retroperitoneales
b. Tanto el hígado como la vesícula biliar se encuentran ubicados a nivel del flanco derecho
c. El colon, el duodeno y el resto de intestino delgado son órganos considerados
netamente como peritoneales
d. Los ligamentos que encontramos dentro de la cavidad abdominal son el esplenorenal y el
gastrofrénico e.Los omentos van desde el estómago y la segunda porción del duodeno a otras
estructuras
intraabdominales y existen dos: el omento mayor y el omento menor

30. Paciente varón de 27 años es llevado por bomberos a emergencia luego de ser asaltado y, tras
resistirse, es cortado con el pico de una botella a nivel abdominal. Al examen físico usted observa que
a través de la herida se puede observar la protrusión de asas intestinales. En relación con las capas de
la pared abdominal, marque la alternativa correcta.
a. La fascia de Scarpa está constituida principalmente por tejido adiposo
b. La pared abdominal está formada por piel, huesos, músculos, fascias y peritoneo parietal
c. La fascia de Camper es una estructura fibrosa que carece de grasa y su grosor es
constante en toda la pared abdominal
d. El músculo oblicuo externo discurre en dirección súpero-interna y se inserta en el borde
inferior de las ultimas 3 a 4 costillas
e. El músculo recto del abdomen tiene como funciones comprimir el contenido del
abdomen, tensar la pared del abdomen y flexionar la columna
SISTEMA
DIGESTIVO
(ME154)
EXAMEN FINAL
2019 01

Profesores : Alfaro Salazar, Herberth Romulo; Callata Caceres, Gunter; Cayo Quiñe, Alexandra Mariel;
Correa Borit, Jorge Mauricio; Cruz Cutty, Lourdes Marylin; Guzmán Calderón, Gerly Edson;
Jáuregui Farfán, Jorge Jesús; Mayor Zevallos, Otto Alberto; Montoya Suárez, José Luis;
Palacios Bazan, Enrique Elias; Robles Pino, Alexander Anibal; Wong Bravo, Juan Carlos
Sección : Todas las secciones
Duración : 70 minutos.
Indicaciones:
Lea atentamente cada pregunta antes de responder:
- Se prohíbe el uso del celular y cualquier dispositivo electrónico.
- Está prohibido intercambiar materiales.
- Coloque su código de alumno en la tarjeta de respuestas. Si su código contiene una letra reemplácela
por un valor numérico siguiendo la siguiente equivalencia: A=9, B=8, C=7, D=6, E=5, F=4, G=3.
- Traslade sus respuestas a la tarjeta, llenando los círculos de manera completa con lapicero negro o
azul. Está prohibido el llenado con lápiz, lapicero de otro color o con lapicero de tinta borrable.
- Sea cuidadoso en el llenado de la tarjeta de respuestas, pues solo esta tiene validez para la calificación.
- Al terminar su examen avise al docente a cargo, no se levante de su sitio; debe entregar la hoja de
respuestas con la carátula del examen, este cuadernillo de preguntas se lo llevará cada estudiante.

1. La explicación fisiológica de presentar somnolencia de 30 minutos a 1 hora después de ingerir


alimentos, se explica por:
a. Aumento del cloro intraluminal
b. Aumento del bicarbonato intraluminal
c. Disminución de ácido carbónico en la célula parietal
d. Disminución de la actividad de la anhidrasa carbónica
e. Aumento de la alcalinidad sanguínea

2. Con respecto a la irrigación arterial del colon, a que arteria corresponde


la señalada con la flecha
a. Cólica derecha
b. Cólica media
c. Cólica izquierda
d. Ileobisecoapendículocólica
e. Arco de Riolano

3. Si un paciente presentara dentro del punto de vista fisiológico, una


disminución de enterocinasa, entonces esto originaría una
disminución de la actividad de:
a. l
a
p
e
p
s
i
n
a
b
.l
a
li
p
a
s
a
c. la quimotripsina
d. el peptido insulinotropo dependiente de glucosa
e. la amilasa

Se valida la opción b debido a su relación con la colipasa.


4. Con respecto a la anatomía del hígado, señale a que
estructura pertenece la marcada por el número 1.
a. Ligamento falciforme
b. Línea de Cantlie
c. Ligamento triangular
d. Ligamento coronario
e. Ligamento teres

Se valida la opción a debido a la ubicación del número 1 en


donde se unen el ligamento falciforme y ligamento coronario.

5. Se presenta un paciente, el cual presenta un antecedente de tuberculosis intestinal, por lo cual, se


le resecó 80 cm de íleon distal. Desde el punto de vista fisiológico, el paciente puede presentar una
de las siguientes alteraciones:
a. Disminución de la secreción de Vitamina B12
b. Aumento indiscriminado de absorción de ácido fólico
c. Disminución de la absorción de hierro
d. Aumento de la secreción de bicarbonato
e. Disminución de la absorción de ácido glicocólico

6. Un paciente es sometido experimentalmente a un fármaco que modifica el flujo salival, obteniéndose


un volumen de saliva de 288 ml en 6 horas. En este caso las concentraciones de electrolitos y
bicarbonato en la saliva obtenida varían de la siguiente manera:
a.↑ Na+, ↓ K+, ↑ Cl-
, ↑ HCO3- b.↓ Na+,
↓ Cl-, ↑ K+, ↓
HCO3-
c.↑ Na+, ↑ Cl-, ↓ K+, ↓ HCO3-
d.↑ Na+, ↑ Cl-, ↑ K+, ↑ HCO3-
e.↓ Na+, ↓ Cl-, ↓ K+, ↓ HCO3-

Se valida la opción a debido a que se puede considerar como un aumento del flujo de saliva.

7. La siguiente imagen histológica corresponde a la glándula


…………… y la estructura señalada produce ………
a. salival sublingual / mucopolisacáridos
b. oxíntica / pepsinógeno
c. salival submaxilar / ptialina
d. salival parótida / amilasas
e. antrales / gastrina

8. Paciente varón de 65 años con antecedente de hipercolesterolemia, hipertensión arterial, fibrilación


auricular y dos infartos al miocardio previos, aqueja de dolor abdominal intenso de inicio súbito,
distensión abdominal, se decide cirugía con resección de 1,5 metros de intestino delgado terminal y
colon ascendente. Como consecuencia de la resección el paciente tendrá deficiencia de:
a. Vitamina C
b. Tiamina
c. Vitamina A
d. Vitamina B1
e. Vitamina B6

Se valida esta opción debido a que su absorción está relacionada al íleon.


9. Uno de los siguientes elementos debería hallarse con más probabilidad en el esófago de un paciente
que sufre de
reflujo gastro esofágico…
a. Pepsina
b. Tripsina
c. Quimiotripsina
d. Carboxipeptidasa
e. Ácidos biliares

10. Un paciente de 40 años cursa con anemia de 8g/dl, aqueja además de astenia y sensación de
hormigueo bilateral en los miembros inferiores, al examen se halla alteración de la sensibilidad a la
vibración y camina con ampliación de la base de sustentación. Uno de los siguientes procedimientos
sería de ayuda para el diagnóstico de este paciente:
a. Tomografía cerebral
b. Biopsia de la mucosa gástrica
c. Biopsia de hígado
d. Examen de sangre oculta en heces
e. Biopsia de Ileon proximal

11. Paciente de 60 años ingresa por caída hace 1 hora y pequeño hematoma en cuero cabelludo, al
examen físico ampliado se observa ictericia de piel y mucosas generalizada, abdomen blando, se
palpa estructura quística no dolorosa en hipocondrio derecho que corresponde a vesícula biliar
(signo de Courvoisier), en los exámenes de laboratorio se halla niveles bajos en la formación de
estercobilinógeno y urobilinógeno en heces, incremento de la bilirrubina conjugada en la orina,
elevación de fosfatasa alcalina y gamma glutamil transpeptidasa séricas. El presente cuadro puede
ser explicado por:
a. Reabsorción de hematoma
b. Litiasis vesicular
c. Carcinoma de la cabeza de páncreas
d. Carcinoma con estenosis del conducto hepático común
e. Anemia hemolítica

12. Paciente varón de 58 años con antecedente de alcoholismo crónico es diagnosticado y recibe
tratamiento por cirrosis hepática. Hace 2 días refiere familiar que tuvo cambio de conducta y no
reconoce a algunos familiares. Al examen físico, se halla ascitis, circulación colateral en abdomen,
telangiectasias, en el examen de sistema nervioso: rigidez de extremidades, ROT incrementados,
desorientación en el espacio y asterixis. ¿cuál de las siguientes circunstancias, explicaría el cuadro
en este paciente?
a. Uso de diuréticos ahorradores de potasio
b. Incremento de actividad de ureasa bacteriana duodenal
c. Hemorragia gastrointestinal
d. Disminución de la producción de NH3+ en el colon
e. Dieta normo proteica

Se valida la opción e debido al efecto sobre la encefalopatía.


Con respecto de la opción b es incorrecta debido a que hace referencia al duodeno, debería indicar colon.

13. Un recién nacido presenta vómitos biliosos poco tiempo después de cada alimento. Al preguntar a la
madre sobre antecedentes, ella recuerda que tuvo polihidramnios durante la gestación, pero un
análisis de cariotipo fue normal. Una de las siguientes es la causa más probable de estos hallazgos en
el recién nacido:
a. Enfermedad de Hirschprung
b. Fístula tráqueo esofágica
c. Divertículo ileal
d. Estenosis pilórica
e. Malrotación de la yema pancreática ventral

14. Un lobulillo hepático se puede dividir en tres zonas como se muestra en el gráfico. ¿Cuál de
las siguientes afirmaciones sobre las tres zonas es verdadera?
a. La zona 1 tiene los menores depósitos de glucógeno
b. La zona 3 es la primera en afectarse en una colestasis extra hepática
c. La zona 2 es más susceptible a la injuria por isquemia que la zona
periportal d.La zona 2 tiene la mayor capacidad de regeneración
e.La zona 1 es la que tiene menos actividad metabólica.

La pregunta 14 ha sido anulada, sin embargo, ningún estudiante se verá afectado


negativamente en su puntaje debido a esta anulación.

15. En un estudio de la secreción de hormonas gastrointestinales, sus concentraciones en la vena porta


se midieron durante perfusión luminal del intestino delgado con soluciones de diversas magnitudes
de pH. ¿Qué hormona aumentará en el plasma de la vena porta durante perfusión a través del
intestino con una solución de pH 3?
a. CCK
b. gastrina
c. GIP
d. motilina
e. secretina

16. Paciente de 30 años que ingresa a causa de un traumatismo abdominal cerrado. En la exploración
se aprecia discreta palidez de piel y mucosas, auscultación pulmonar normal, taquicardia de 120
/min. Discreta distensión abdominal y matidez en flancos; el hematocrito, que era prácticamente
normal al ingreso, disminuye a 30% a las tres horas. En la Rx de tórax se objetiva fractura de las
costillas 10-11 izquierdas. La causa más probable de la anemización en este paciente es:
a. traumatismo renal con hemorragia retroperitoneal.
b. rotura de hígado con hemoperitoneo.
c. rotura de bazo con hemoperitoneo.
d. rotura de mesos con hemoperitoneo.
e. traumatismo pancreático con pancreatitis traumática.

17. Mujer de 65 años. Consulta por síndrome constitucional


asociado a dolor abdominal epigástrico progresivo
irradiado a espalda, de dos meses de evolución. El
diagnostico de sospecha de adenocarcinoma de páncreas
se confirma por biopsia. Se realiza examen de imagen de
abdomen para evaluación de estructuras vasculares
próximas al tumor pancreático. ¿Cuál es el nombre de la
vena señalada que está ausente, trombosada por
infiltración tumoral, condicionando circulación colateral
en la pared gástrica?

a. Mesentérica superior
b. Coronaria estomaquica
c. Esplénica
d. Porta
e. Renal izquierda
18. Revisando la angiotomografía de un hombre de 70 años en estudio por aneurisma de aorta
abdominal, el radiólogo le informa de la presencia de una oclusión completa de la arteria
mesentérica inferior. El paciente se encuentra completamente asintomático. La oclusión de la
arteria mesentérica inferior cursa de manera asintomática en muchas ocasiones ya que el
territorio que irriga puede recibir flujo proveniente de la arteria:
a. cólica derecha
b. gastroduodenal
c. Epigástrica inferior izquierda
d. esplénica
e. cólica media

19. En las patologías de esófago es importante conocer bien la anatomía esofágica. ¿Cuál de
las siguientes afirmaciones es correcta?
a. El esófago tiene capa mucosa, muscular y serosa
b. El esófago abdominal es más largo que el cervical
c. El esófago torácico pasa por detrás del cayado aórtico
d. El epitelio esofágico normal es de tipo cilíndrico.
e. El esófago abdominal es discretamente más largo que el torácico

20. A pesar de que pueda haber variaciones anatómicas, lo habitual es que el ciego sea irrigado por una
rama arterial que proviene de unas de las siguientes arterias:
a. Iliaca derecha
b. Mesentérica inferior
c. Hepática derecha
d. Mesentérica superior
e. Iliaca izquierda

21. Ante un paciente con una cirugía abdominal urgente, el informe operatorio señala que se ha realizado
una resección de todo el duodeno y del tercio proximal del yeyuno manteniendo íntegros el
estómago y todo el íleon, así como los dos tercios distales del yeyuno. En el seguimiento nutricional
del paciente ¿Qué vitamina o mineral presentará con menor probabilidad una disminución de su
absorción?
a. Cianocobalamina
b. Calcio
c. Hierro
d. Transcobalamina
e. Transferrina

22. ¿Cuál de las siguientes alternativas detallan las venas que confluyen y forman la vena señalada?
a. mesentérica superior, gástrica izquierda y
gastroepiploica izquierda
b. mesentérica inferior, gástrica izquierda y renal
c. esplénica, mesentérica superior y mesentérica inferior
d. esplénica, pancreatoduodenal y omental izquierda
e. gástrica izquierda, esplénica y hepática común

23. ¿Cuál de las siguientes sustancias forma parte de la secreción biliar?


a. Tripsina
b. Lecitina
c. Elastasa
d. Quimotripsina
e. Pepsina
24. El tubo digestivo contiene diferentes tipos de epitelios y glándulas. La estructura señalada es unay
está localizada en el …………...
a. glándula de Brunner /
intestino grueso b.cripta de
Lieberkuhn / colon
c. cripta de Lieberkuhn / intestino delgado
d. glándula oxintica / estomago
e. célula parietal / estómago

Aunque las criptas de


Lieberkuhn están
presentes en el intestino
delgado, la
microfotografía es de
epitelio de colon.

25. ¿De qué musculo forma parte el ligamento inguinal?


a. Oblicuo externo del abdomen
b. Oblicuo interno del abdomen
c. Transverso del abdomen
d. Psoas
e. Dorsal ancho

26. Señale cuál de las siguientes afirmaciones NO se relaciona a la siguiente glándula anexa del
tubo digestivo mostrada en la imagen:
a. Es una glándula exocrina compuesta exclusivamente por acinos serosos
b. Su inervación está dada por el nervio auricular mayor (ramo posterior C2), que inerva
la vaina de la glándula así como la piel por encima de esta.
c. Esta glándula produce una secreción mucinosa acuosa, llamada mucoserosa, a través del
conducto de Wharton.
d. Su inflamación puede ser causada por un virus de los Paramyxoviridae, que provocan una
enfermedad muy frecuentemente en niños y adolescentes
e. Es una glándula endocrina y probablemente sea de origen pancreático

Se valida la opción e debido a que no está


relacionada con la imagen.

27. ¿Cuál de las siguientes enzimas está localizada en el borde en cepillo y juega un rol en la digestión de
proteínas?
a. Alfa dextrinasa
b. Pepsina
c. Enterocinasa
d. Lactasa
e. Carboxipeptidasa A.

Se valida la opción c debido a que es correcta en relación a la pregunta.


28. Una de los siguientes sustancias, NO sirve como un buen agente emulsificante:
a. Colesterol
b. Ácidos grasos
c. Sales biliares
d. Lecitina
e. Proteínas de la dieta

Se valida la opción e debido a que es correcta en relación a la pregunta.

29. La sustancia que estimula el crecimiento de la mucosa gástrica es:


a. Secretina
b. Motilina
c. Péptido estimulante de la mucosa gástrica
d. Gastrina
e. Histamina

30. ¿Cuál de las siguientes alternativas es una función de la colecistokinina?


a. Relajación de la vesícula para la
salida de bilis b.Secreción de ácidos
biliares
c. Contracción del esfinter de Oddi
d. Secreción de enzimas pancreáticas
e. Contracción del duodeno

Se valida la opción b debido al efecto de la CCK sobre la vesicula biliar.

31. Con respecto a la anatomía del tronco celiaco, señale lo correcto


a. El tronco celiaco se origina de la cara posterior de la aorta abdominal
b. Es una arteria delgada que tiene un calibre entre 2 y 3 mm
c. Una de sus ramas es la arteria gástrica derecha
d. La hepática común que es una de sus ramas, participa en la irrigación del estómago.

32. Con respecto a la anatomía del duodeno, marque la respuesta correcta:


a. Tiene una distribución en forma de “C”, que rodea la cola del páncreas
b. La 3ra porción duodenal está contenida en la pinza vascular aortomesentérica
c. Entre la 1ra y 2da porción se forma un ángulo, conocido como el ángulo de Treitz
d. La 4ta porción se dirige a la izquierda, hacia abajo y hacia atrás.
e. En la tercera porción desemboca el conducto colédoco.

33. El hígado está ampliamente tapizado por peritoneo, la estructura que conecta la cara
diafragmática del hígado precisamente con el diafragma es el ligamento:
a. teres
b. f
a
l
c
if
o
r
m
e
c
.t
ri
a
n
g
u
l
a
r
d.hepá
tico
común
e.coro
nario

Se validan la opción c y e debido a que forman parte de los ligamentos que fijan el hígado al diafragma.

34. En el íleon se absorbe aproximadamente el 95% dea través de la circulación enterohepática.


a. agua
b. colesterol
c. sales biliares
d. hidróxicobalamina
e. factor intrínseca
35. Laestimula el mecanismo paracrino de la secreción de ácido clorhídrico.
a. histamina
b. acetilcolina
c. gastrina
d. secretina
e. somatostatina

36. En la digestión de proteinas,es el principal estímulo para convertir el pepsinógeno en pepsina.


a. la gastrina
b. el pH ácido
c. la acetilcolina
d. la ptialina
e. la somatostatina

37. Con respecto a la somatostatina, marque lo correcto:


a. Es secretada por las células S del intestino
b. Induce a la producción de VIP
c. Interviene en la fase intestinal de la secreción gástrica
d. Produce acetilcolina para estimular a la célula parietal
e. No interviene en la regulación de la secreción de ácido clorhídico

38. En pecten anal, es una estructura comprendida entre:


a. la línea pectínea y los senos anales
b. la línea blanca y la apertura anal
c. el esfínter anal interno y el externo
d. la línea anocutánea y la línea pectínea
e. la línea blanca y columnas anales

39. ¿Cuál de las siguientes alternativas es una proenzima pancreática?


a. Tripsina
b. Elastasa
c. Quimotripsinógeno
d. Amilasa
e. Procarboxipepitidasa C.

40. En la segmentación hepática de Coinaud, el segmento hepático señalado con la flecha,


corresponde a : En la segmentación hepática de Coinaud, la flecha señala el
segmentohepático.
a. IV
b. V
c. VI
d. VII
e. VIII
EXAMEN
PARCIAL
SISTEMA
DIGESTIVO
(ME154)
Ciclo 2019-02

Sección:Todas
Profesores:Alfaro Salazar, Herberth Romulo; Alva Muñoz, Jose Carlos; Mayor Zevallos, Otto Alberto;
Duración:30 minutos.
Indicaciones:
- Lea atentamente cada pregunta antes de responder.
- Se prohíbe el uso del celular y cualquier dispositivo electrónico.
- Está prohibido intercambiar materiales.
- Coloque su código de alumno en la tarjeta de respuestas. Si su código contiene una letra reemplácela
por un valor numérico siguiendo la siguiente equivalencia: A=9, B=8, C=7, D=6, E=5, F=4 y G=3.
- Traslade sus respuestas a la tarjeta, llenando los círculos de manera completa con lapicero negro o
azul. Está prohibido el llenado con lápiz, lapicero de otro color o con lapicero de tinta borrable.
- Sea cuidadoso en el llenado de la tarjeta de respuestas, pues solo esta tiene validez para la calificación.
- Al terminar su examen avise al docente a cargo, no se levante de su sitio; debe entregar la hoja de
respuestas con la carátula del examen, este cuadernillo de preguntas se lo llevará cada estudiante.

1. La contracción del músculopermite la eliminación de gases (flatos) sin salida de material fecal;
es el mismo músculo cuya relajación, sobretodo en cuclillas, permite el paso del contenido fecal
con menor esfuerzo durante la defecación.
a) Isq
uir
ec
tal
b)
Pu
bo
rre
ct
al
c) Esfínter anal externo
d) Esfínter anal interno

2. Paciente mujer de 54 años se presenta con náuseas, vómitos, estreñimiento, y es diagnosticada


de abdomen agudo quirúrgico; en la cirugía encuentran un vólvulo de ciego. Esta anomalía
puede explicarse por::
a) Falta de rotación intestinal
b) Falta de fusión del mesenterio
c) Defecto en la formación de la cloaca
d) Falta de formación del omento mayor

3. Paciente mujer de 23 años con faringitis aguda, toma para el dolor una tableta de paracetamol con
un poco de agua. Durante la deglución, se relaja su esfínter esofágico inferior y el fondo del estómago,
mientras el bolo está aún en el esófago. ¿Qué sustancia provocara con mayor probabilidad la
relajación del esfínter esofágico inferior y el fondo del estómago en esta mujer?
a) Óxido nítrico
b) Sustancia P
c) Histamina
d) Motilina

4. Luego de tres horas dando exámenes, un alumno de medicina comienza a sentir hambre. Esta
situación es probable que sea mediada por la que es sintetizada por el :
a) leptina / intestino
b) leptina / estómago
c) grelina / estómago
d) grelina / tejido adiposo
5. Varón de 72 años, con antecedente de diabetes mellitus tipo 2, que presenta enteropatía diabética
caracterizada por estreñimiento. Este problema puede estar asociado a:
a) deficiencia de óxido nítrico
b) aumento del reflejo gastrocólico
c) disminución de la secreción de colecistocinina (CCK)
d) aumento de la secreción del péptido intestinal vasoactivo (PIV)
6. Varón de 54 años con Diabetes Mellitus tipo 2, es diagnosticado de gastroparesia debido a que
presenta sensación de llenura precoz al comer, y reflujo gastroesofágico. Esta alteración en la
relajación receptiva y en el vaciamiento gástrico lo más probable es que se deba a una alteración
en:
a) el nervio vago
b) el ganglio celíaco
c) plexo submucoso
d) nervio hipogástrico

7. Varón de 67 años con tos y disminución de peso asociado a tabaquismo pesado, presenta
actualmente disfagia progresiva a alimentos sólidos. Se considera la presencia de un carcinoma
de bronquio izquierdo y por esta razón le realizan una endoscopía esofágica para descartar la
posibilidad de una compresión esofágica por el tumor. Se espera revisar el esófago en la
estrechez, que está a nivel de la vértebra
a) Tercera estrechez -T6
b) Segunda estrechez - C6
c) Segunda estrechez - T4
d) Tercera estrechez -T10

8. Varón de 34 años con dolor abdominal agudo en flanco derecho que se irradia a fosa ilíaca derecha,
es operado y se encuentra un divertículo intestinal inflamado, ubicado a 93 cm de la válvula ileocecal.
El origen de este divertículo es una falla en la obliteración de:
a) Conducto vitelino
b) Alantoides
c) Cloaca
d) Conducto anorectal
e) Uraco

En un niño menor de dos años con divertículo intestinal, este divertículo tiene su origen en
una falla en la obliteración de:
a)Conduct
o
anorectal
b)
Conducto
vitelino
c) Alantoides
d) Cloaca
e) Uraco

9. Mujer de 43 años sufre un grave accidente de tránsito y está hospitalizada en coma, es alimentada
por vía intravenosa durante varias semanas. Producto de este tipo de alimentación, se encuentra en
la endoscopía atrofia de la mucosa gastrointestinal. La causa más probable de esta atrofia son los bajos
niveles séricos de la hormona:
a) Colecistocinina
b) S
e
c
r
e
t
i
n
a
c
)
G
a
s
t
r
i
n
a
d) PIV

10. Una mujer de 30 años llega al consultorio porque se queja de dificultades para deglutir, la cual se
agravan cada vez más. Se realiza un estudio manométrico para examinar la generación de presión a
lo largo del esófago. Esta prueba revela que las contracciones como respuesta a la deglución están
mal sincronizadas y que la presión en el esfínter esofágico inferior permanece elevada. El diagnóstico
más probable es producido por niveles bajos de
a) acalasia /
sustancia P b)
acalasia /
óxido nítrico
c) enfermedad por reflujo gastrointestinal / acetilcolina
d) enfermedad por reflujo gastrointestinal / óxido nítrico

11. Paciente de 2 años, llega a emergencia por haber ingerido una moneda con la que estaba jugando.
El lugar más probable donde puede haberse quedado suspendido este objeto es a nivel del
estrechamiento producido a nivel del:
a) músculo milohiodeo
b) músculo aritenoideo
c) músculo cricofaríngeo
d) constrictor superior de la faringe

12. En una apendicectomía, al realizar la incisión de McBurney en la fosa iliaca derecha, es


necesario cortar los siguientes músculos, de afuera hacia adentro:
a) Recto – Oblicuo externo – Transverso
b) Recto – Oblicuo externo – Oblicuo interno
c) Oblicuo externo – Oblicuo interno – Recto
d) Oblicuo externo – Oblicuo interno – Transverso

13. Un varón de 90 años que se encuentra postrado en cama, es referido del asilo para endoscopia
por dificultad para deglutir luego de tomar un medicamento para aliviar el dolor la noche anterior.
La endoscopía revela que la píldora se alojó en el esófago y causó una reacción inflamatoria. Lo
más probable es que esto haya sido por la producción de múltiples ondas:
a) secundarias
b) primarias
c) lentas
d) segmentarias

14. Mujer de 23 años es diagnosticada de bulimia, al examen físico se observa ulceraciones en el segundo
y tercero dedo de la mano derecha. Esto se puede deber al uso continuo de estos dedos para inducir
el vómito, mediante la estimulación del par craneal:
a) V
b
)
I
X
c) X
d) XI

15. Varón de 52 años se presenta por diarrea persistente de seis semanas de duración. En la colonoscopia
se observa un pólipo a nivel del íleon distal. El patólogo informa que se trata de un tumor
neuroendócrino, probablemente originado por las células enterocromafines del intestino. La
sustancia que más probablemente esté produciendo este tumor es:
a) Serotonina
b) Insulina
c) CCK
d) GIP

16. La fase oclusal de la masticación se realiza con la contracción de los músculos:


a) digástricos
b) masetero y temporal
c) orbicular y buccinador
d) pterigoideo lateral y digástrico

17. Al tomar su café en Starbucks, un estudiante de medicina sufre una quemadura de primer grado
en el tercio anterior de la superficie dorsal de la lengua. La información de dolor es transmitida
por el nervio:
a) cuerda del tímpano
b) glos
ofarí
nge
o
c)lin
gual
d) facial

18. Paciente es evaluado por faringitis aguda en consultorio externo. El médico de familia le solicita que
abra la boca y saque la lengua. Para realizar la acción de sacar la lengua, es necesario que se contraiga
el músculo:
a) e
st
il
o
gl
o
s
o
b)
g
e
ni
o
gl
o
s
o
c) palatogloso
d) transverso de la lengua
19. Paciente con síndrome de Sjögren, presenta “boca seca”
(disminución de la producción de saliva) y caries dental,
asociada a la pérdida de la función de tampón de la saliva.
Esta desminerilización del diente puede comprometer a las
prolongaciones citoplasmáticas ubicadas en los tubos
huecos de la estructura señalada con la letra:
a) B
b) A
c) E
d) C

20. Mujer de 32 años acude a consulta por presentar disfagia de


progresión lenta, reflujo gastroesofágico y vómitos desde
hace 3 meses de evolución progresiva. Se le realiza un estudio
radiológico con contraste en el que se observa
estrechamiento del esfínter esofágico inferior (imagen).
Según sus conocimientos, este paciente se beneficiaría con el
uso de:
a) agonista beta adrenérgico
b) agonista alfa
adrenérgico
c)análogo de
óxido nítrico
d) análogo de Sustancia P
EXAMEN
PARCIAL
SISTEMA
DIGESTIVO
(ME154)
C
ic
lo
2
0
2
0
0
0

Sección:Todas
Profesores:Alva Muñoz, Jose Carlos
Duración:35 minutos.
Indicaciones:
- Lea atentamente cada pregunta antes de responder.
- Se prohíbe el uso del celular y cualquier dispositivo electrónico.
- Está prohibido intercambiar materiales.
- Coloque su código de alumno en la tarjeta de respuestas. Si su código contiene una letra reemplácela
por un valor numérico siguiendo la siguiente equivalencia: A=9, B=8, C=7, D=6, E=5, F=4 y G=3.
- Traslade sus respuestas a la tarjeta, llenando los círculos de manera completa con lapicero negro o
azul. Está prohibido el llenado con lápiz, lapicero de otro color o con lapicero de tinta borrable.
- Sea cuidadoso en el llenado de la tarjeta de respuestas, pues solo esta tiene validez para la calificación.
- Al terminar su examen avise al docente a cargo, no se levante de su sitio; debe entregar la hoja de
respuestas con la carátula del examen, este cuadernillo de preguntas se lo llevará cada estudiante.

1. Paciente de sexo masculino de 82 años de edad ingresa a emergencia con dolor abdominal agudo y
diarreas. Se le realiza una arteriografía en la que se observa que la arteria aorta tiene un trombo
ocluyendo el 95% del flujo, a nivel del nacimiento de la arteria mesentérica inferior. ¿Cuál de las
siguientes arterias podría contribuir a la irrigación colateral del colon descendente?
a) cólica media
b) sigmoidea
c) rectal superior
d) ileocólica

2. Niño de 5 años presenta dolor esofágico y hematemesis (vómitos hemorrágicos) luego de tragarse
una espina de pescado. En la endoscopía se observa perforación del esófago distal a la cuarta
estrechez esofágica. ¿Las ramas de cuál de las siguientes arterias estarán lesionada con mayor
probabilidad?
a) Gástrica izquierda
b) Bronquiales
c) Frénica inferior
d) Tiroidea inferior

3. Al ingerir una cucharada de mantequilla es muy probable que se disminuya la sensación de hambre
por medio de la activación de la vía POMC/CART (POMC=proopiomelanocortina y CART=transcripción
regulada de cocaína y anfetamina), activada directamente por la hormona:
a) colecistoquinina (CCK)
b) insulina
c) grelina
d) secretina

4. Al comer unas papitas fritas con mayonesa, el vaciamiento gástrico disminuye por efecto directo de
la hormona: a)colecistoquinina (CCK)
b) bombesina
c) motilina
d) gastrina

5. Recién nacido de dos horas es diagnosticado de hernia umbilical de 1,5 cm de diámetro; el cirujano
pediatra solicita una tomografía abdominal en donde se evidencia que la hernia umbilical está
ocupada por una porción del tracto gastrointestinal. ¿Qué porción del tracto gastrointestinal
estaría ocupando esta hernia con mayor probabilidad?
a) Íleon
b) Colon sigmoides
c) Duodeno
d) Colon transverso
6. Recién nacido de 7 horas, de parto por cesárea debido a polihidramnios (aumento del volumen del
líquido amniótico), con regurgitación de la leche materna y artificial, y no ha presentado meconio.
Se le realiza una tomografía donde se evidencia aire en el estómago y una malformación del
desarrollo esofágico. Con respecto a esta malformación lo más probable es que se pueda tratar de
una atresia esofágica:
a) proximal con fístula traqueoesofágica distal
b) distal con fístula traqueoesofágica proximal
c) proximal y distal
d) sin fístula

7. Lactante de 6 meses de edad que es traído a consulta por presentar vómitos no biliosos a repetición
y retraso en el crecimiento. En la radiografía de abdomen simple se observa nivel hidroaéreo en
estómago y en primera porción de duodeno (doble burbuja). ¿Cuál de las siguientes alternativas
puede explicar la condición del lactante?
a) Páncreas anular
b) Atresia duodenal en la tercera porción
c) Atresia yeyunal
d) Hipertrofia del píloro

8. En ausencia o deficiencia de la secreción de la hormona motilina,


se producirá: a)sobrecrecimiento bacteriano
b) diarrea
c) aumento del vaciamiento gástrico
d) hipertrofia del píloro

9. La estimulación parasimpática aumenta la motilidad intestinal, mientras que la estimulación


simpática la disminuye. ¿Sobre cuál de las siguientes alternativas el sistema nervioso autónomo
actúa para el control de la motilidad intestinal?
a) Potencial de membrana en el plexo mientérico (de Auerbach)
b) Frecuencia de ondas lentas
c) Secreción de secretina
d) Nivel de IP3 en el plexo submucoso (de Meissner)

10. En un recién nacido con protrusión de contenidos abdominales y cubiertas por amnios o peritoneo,
es cierto que: a)Se presenta por un defecto en el cierre de la pared
b) Se acompaña de otras malformaciones congénitas
c) Se debe al no retorno de la hernia fisiológica
d) Se produce a través del ombligo

11. Lactante de 20 días con estreñimiento, distención abdominal progresiva, acompañada


ocasionalmente de vómitos biliosos. Como antecedente, el meconio lo eliminó por primera vez a las
72 horas de nacido. Su mamá menciona que ayuda a la evacuación con ayuda de un termómetro
rectal. Se sospecha de megacolon agangliónico (Enfermedad de Hirschsprung). ¿Cuál de las siguientes
alternativas explica el caso?
a) Se presenta contracciones tónicas en la región ano rectal
b) Se presenta dilatación de tracto gastrointestinal afectado
c) Las células ganglionares sólo han migrado al ano recto
d) La zona que más se afecta es inervada por fibras del nervio esplácnico menor

12. ¿Cuál de los siguientes reflejos disminuye el tránsito


gastrointestinal? a)Doloroso
b) Gastrocólico
c) De defecación
d) Colicoileal

13. Al ingerir una sustancia ácida como el vino (pH 3), se estimula la motilidad gástrica por acción de
la hormona: a)motilina
b) secretina
c) colecistoquinina (CCK)
d) bombesina
14. Paciente de 24 años acude a consulta externa por presentar una fístula oronasal (comunicación
entre la cavidad oral y la cavidad nasal). Está fístula está asociada al antecedente de haber sido
operada de paladar hendido a los dos años de edad, durante una campaña gratuita extranjera
de corrección de paladar fisurado. ¿Cuál de las arterias palatinas podría haberse lesionado
durante esa cirugía?
a) Mayor
b) Menor
c) Ascendente
d) Rama palatina de la faríngea ascendente

15. Paciente de sexo masculino de 52 años con úlcera péptica gástrica de 14 años de evolución, con
cuadro de hemorragia digestiva alta hace 4 meses, sin cicatrización de la úlcera. Entre las
opciones quirúrgicas se considera realizarle un vaguectomía troncal (sección del nervio vago) a
nivel del hiato esofágico. ¿Cuál de las siguientes complicaciones podría esperarse producto de
la pérdida de inervación parasimpática?
a) Menor inervación del colon ascendente
b) Se perderá el reflejo de defecación
c) Se perderá el reflejo de micción
d) Impotencia sexual

16. Paciente de 23 años con bulimia es traída a la emergencia deshidratada, semiconsciente y con
alcalosis metabólica. Los vómitos autoinfligidos por esta paciente se producen por estimulación
de receptores en la base de la lengua que mandan información directamente al:
a) núcleo del tracto solitario
b) centro del vómito en el tallo encefálico
c) zona quimiorreceptora gatillo
d) cerebelo

17. Niño de 3 años es traído a emergencia por madre quien manifiesta que hace 10 horas deglutió
una pila pequeña de reloj de bordes romos. El niño está asintomático. Usted la tranquiliza
diciéndole es un cuerpo extraño tan pequeño de seguro que va a seguir el tránsito intestinal
como lo haría un bolo alimenticio, y que lo más probable es que en ese momento se encuentre
en:
a) colon
b) estómago
c) yeyuno
d) recto

18. Los movimientos en masa son un tipo de movimiento muy importante, una de las
consecuencias de estos movimientos es:
a) la distensión rectal
b) el peristaltismo del intestino delgado
c) la retropulsión gástrica
d) la contracción del esfínter anal interno

19. Durante la deglución, al momento que el bolo alimenticio pasa por el esfínter esofágico superior,
se espera que la presión intraesofágica:
a) disminuya en el cardias
b) disminuya en el tercio medio del esófago
c) aumente en la porción distal al bolo
d) aumente en el tercio medio del esófago

20. Paciente con enfermedad de Chagas que presenta disfagia a sólidos. ¿Cuál de las siguientes
puede ser la causa de esta complicación?
a) Disminución de células ganglionares en el esfínter esofágico inferior
b) Aumento en la liberación de óxido nítrico en el esfínter esofágico inferior
c) Disminución de las neuronas que liberan péptido intestinal vasoactivo
d) Aumento de la actividad de la motilina en el esófago distal

QUIZIZZ
1. Los vasos mesentéricos superiores se hallan a nivel de:
a) Cuello del páncreas

2. El nivel en el que se encuentra el píloro y el páncreas se puede determinar usando el


A) plano transpilórico

3. El dolor de estómago asociado a gastritis se suele ubicar en


a) epigastrio

4. La colecistoquinina inhibe el
a) vaciamiento gástrico

5. Paciente con disfasia ( dificultad para pasar alimentos) con to y disminución de peso. Con
antecedente de tabaquismo pesado. La sospecha es que tenga una disminución del diámetro
esófago a nivel de la
a) tercera estrechez

6. La digestión de las proteínas se inicia en:


a) estómago

7. El estómago recibe información simpática proveniente del:


a) ganglio celíaco

8. El reflujo gastroesofagico tiene múltiples etiologías, una de ellas tiene que ver con alteración a
nivel de:
a) primera estrechez
b) segunda estrechez
c) tercera estrechez
d) cuarta estrechez

9. La fístula retroperitoneal es causada por una falla en el desarrollo de:


a) tabique urorrectal

10. El nervio vago inerva el:


a) músculo estriado del esófago

11. El divertículo de Meckel es un rezago de:


a) conducto vitelino

12. La presencia de orina que sale por el ombligo de un recién nacido casa vez que llora, es posible
que se deba a un defecto en el desarrollo del
a) seno urogenital

13. La fístula retroperitoneal es causada por una falla en el desarrollo de


a) tabique urorrectal

14. Enfermedad asociada con un error en el desarrollo de las células de Cajal:


a) enfermedad de Hirschsprung

15. Aproximadamente en la semana 6 del desarrollo embrionario, el intestino medio gira 90


herniandose a nivel del
a) cordón umbilical

16. El conducto biliar deriva del


a) endodermo

17. Paciente mujer con 54 años con nauseas y vómitos y abdomen agudo quirúrgico, se ingresa a
sala de operaciones donde se encuentra vólvulo de ciego, esto se debe a
a) Falta de fusión del mesenterio

18. Paciente con cirrosis hepática con hipertensión portal, en el que es posible encontrar que los
vasos umbilicales están permeables dentro de
a) ligamento redondo

19. Es normal encontrar glándulas submucosas en CUAL


ES
a) esófago medio
b) esófago proximal
c) esófago distal
d) estómago
20. Cual de los siguientes órganos son intraperitoneales
a) estómago, vesícula biliar, íleon, hígado

Estudiante de medicina de la UPC de 21 años sufre de gastritis aguda ocasionada por comer en lugares poco
higiénicos. Suele consumir caramelos ( chupar ) mientras está en clase hasta la tarde. Toma gaseosas
regularmente (carbohidratos 46%, sodio 53%). También toma regular cantidad de leche (grasa 35%, lactosa
35%, proteínas 30%), pues le calma un poco el dolor el ardor que siente por la gastritis. Incluso, cuando puede,
se toma dos vasos de agua fría para calmar las molestias. Ha decidido ir al médico para tratarse pues ya no
soporta el dolor, el cual está seguro que los síntomas se deben a una elevada producción de ácido clorhídrico
en el estómago, y por ello le ha recetado Ranitidina (antihistamínico), con lo que siente mejoría.

● Para reducir la secreción de HCl en esta paciente se podría usar sustancias similares a:

- Péptido insulinotrópico dependiente de la glucosa (GIP)

● Si se usara atropina en esta paciente, se esperaría que disminuya la liberación de:


- Enzimas pancreáticas

● El consumir caramelos eleva los niveles en sangre de una hormona cuya función
es la estimulación de las células:
- Beta del páncreas

● En este paciente con gastritis aguda debida a una alta producción de ácido
clorhídrico, si se le hiciera un examen de sangre, se encontraría elevados los
niveles de:
- Colecistoquinina

● El consumo de una pequeña cantidad de gaseosa aumentará directamente la


concentración sérica de cuál de las siguientes hormonas:

- Péptido 1 similar al glucagón (GLP-1)

● El consumo rápido de 500 mL de gaseosa aumentará directamente la


concentración sérica de cuál de las siguientes hormonas:

- Gastrina

● Estimulan la secreción ácida gástrica

- Proteínas

● Con respecto a las ondas lentas, marque la afirmación correcta:}

- Son contracciones rítmicas espontáneas

● El uso de Ranitidina bloquea el receptor H2 de la histamina en las células


parietales. La histamina llega a estas células por:
- Difusión

● El consumir caramelos indirectamente activa la vía:

- POMC/CART

● ¿Cuál de los siguientes péptidos inhibe el vaciamiento gástrico?

- Colecistoquinina

● Para poder morder una manzana, es necesario usar el siguiente músculo:

- Milohiodeo

● El crecimiento de un adenocarcinoma de cuello de páncreas puede comprometer


la pared gástrica por continuidad. ¿Qué parte del estómago estaría comprometido
con mayor probabilidad?

- Pared posterior del antro

● El nacimiento de la arteria mesentérica superior se puede encontrar en cuál de los


cuadrantes abdominales:

- Epigastrio

● En cuanto a la colecistoquinina, marque la respuesta correcta:

- Potencia la acción de buffer con bicarbonato

● El aumento en la actividad motora de la pared gástrica genera un aumento


en los niveles locales de qué sustancia en la microvasculatura:

- Adenosina

Niño de sexo masculino de 2 años de edad, sufre de estreñimiento desde el nacimiento (1


deposición cada 3-4 días). Madre menciona que le estimula la defecación con un
termómetro rectal, y continuo uso de enemas y laxantes. Desde hace 6 meses comienza
con vómitos postprandiales. Los síntomas aumentan en frecuencia y magnitud y están en
relación con los episodios de estreñimiento. No refiere fiebre, tos, diarrea ni lesiones
cutáneas. Al examen físico presenta regular estado general, luce deshidratado. Abdomen
distendido, blando, depresible e indoloro. No se palpan masas abdominales. Se
permeabiliza el canal anal con termómetro rectal, encontrando cierta resistencia. Salida
de material fecal mal oliente en regular cantidad. Exámenes de laboratorio: hemograma
normal. Signos inflamatorios de fase aguda negativos. Alcalosis metabólica leve en
sangre venosa. Radiografía con enema baritado muestra recto y colon sigmoides
dilatados (megacolon). Biopsia profunda: ausencia de células ganglionares en la muestra
enviada. Se realiza cirugía correctiva.

● La percepción de la pirosis (sensación de dolor o quemazón en el esófago)


asociado al reflujo gastroesofágico, puede aparecer o exacerbarse debido a:

- Ejercicio

● Considerando que este paciente está sometido a estrés por el agravamiento de su


enfermedad, es posible afirmar que sus ondas lentas están:

- Hiperpolarizadas
● Debido al acúmulo de material fecal en todo el marco colónico, y a la irritación
química asociada, el peristaltismo del íleon distal se debe encontrar:

- Inhibido

● En cuanto a los reflejos gastrocólico y gastroduodenal en este paciente, indique


lo correcto:

- Se pueden considerar reflejos vago-vagales

● Con respecto a la defecación señale el enunciado correcto:

- Es estimulado por un llenado de la cuarta parte del volumen rectal

● El contenido fecal se detiene en la zona inmediatamente proximal a la zona donde


hay una menor presencia de:

- Péptido intestinal vasoactivo

● La presencia de atresias y estenosis duodenales se deben básicamente a una:

- Falta de recanalización

● El ligamento de Treitz característicamente:

- Suspende el ángulo de Treitz

● Al deglutir un bolo alimenticio, es lógico suponer que al pasar por el esófago haya
un mayor consumo de oxígeno en la pared del tercio:

- Proximal

● Estudiante de medicina de 20 años, se ha amanecido estudiando para su examen


de Sistema Digestivo. No ha probado alimento desde la cena, por lo que se puede
afirmar que la motilidad de esta persona está siendo regulada por:

- Motilina

● La hernia fisiológica se produce dentro de:

- Cordón umbilical

● El crecimiento de un adenocarcinoma de páncreas compromete la pared gástrica


por contigüidad. ¿Qué parte del estómago se esperaría esté comprometido?

- Pared posterior del antro

● Al comer unas papitas fritas con mayonesa, el vaciamiento gástrico disminuye por
efecto directo de la hormona:

- colecistoquinina (CCK)

● Paciente que come entera una pizza familiar de chorizo y queso. Es posible esperar
que debido a la cantidad de alimento ingerida, las ondas lentas hayan:

- Sufrido ninguna alteración en su frecuencia

● En este caso se puede afirmar con seguridad que se presenta:


- contracciones tónicas en la región ano rectal

● La forma más común de atresia esofágica contiene:

- Estenosis proximal del esófago más fístula traqueoesofágica distal

● Con respecto a la saliva, marque la respuesta correcta:

- el sistema simpático estimula su secreción

● Respecto a las enfermedades del esófago, marque lo correcto:

- el diagnóstico diferencial de la acalasia es la enfermedad de Chagas esofágica

● En relación a la fisilogía gástrica, marque lo correcto:

- la cimetidina actúa en la región basolateral de la célula parietal

● La célula mucosa del cuello gástrico produce:

- Moco

● La saliva puede tener una variedad de electrolitos en su composición. Entre ellos el cloro,
respecto al cual se puede afirmar:

- Su concentración no llega a ser tan alta como en el plasma

● Los músculos de la masticación que producen la retropulsión de la mandíbula son:

- temporales

● Respecto a las glándulas salivales, marque lo incorrecto:

- la glándula sublingual tiene forma de garfio

● Respecto a la anatomía del estómago, marque lo correcto:

- la arteria gástrica derecha nace de la arteria hepática común

● En cuanto a la saliva, marque lo correcto:

- La amilasa cumple función digestiva

● Durante el ataque con gas sarín (bloqueador de la acetilcolinesterasa) en el metro


de Tokio, en 1995, el personal de salud notó que los pacientes afectados
presentaban:

- Hipersalivación

Paciente de 54 años con antecedentes de alcoholismo, gastritis crónica,


tabaquismo pesado, obesidad, cálculos biliares y cirrosis, es llevado a la
emergencia por dolor abdominal en epigastrio irradiado a la espalda y trastorno
del sensorio.
Al examen físico: presión arterial 85/50 mmHg, frecuencia cardíaca 100
latidos/min, frecuencia respiratoria 18 x minuto, temperatura axilar 36°C.
Conjuntivas pálidas, escleras ictéricas nevus arácnidos en tronco, distensión
abdominal marcada, cabeza de medusa, matidez desplazable en ambos flancos e
hipogastrio, dolor a la palpación de abdomen.
Tiempo de protrombina: 24 seg (testigo: 13 seg); TPT: 38 seg, glicemia: 165 mg/dL,
uremia: 20 mg/dL, ASAT: 76 UI/L, ALAT: 22 UI/L, albumina: 2,5 g/dL, bilirrubina
total: 2,6 mg/dL, bilirrubina directa: 1,4 mg/dL, amilasa sérica 4000 U/L.

● Un efecto secundario en el estómago por la acción de la secretina es:

- Menor actividad de la pepsina

● Considerando que el paciente sufre de gastritis, se puede decir que la secreción


de ácido por la mucosa gástrica:

- Involucra transporte activo de Hidrogeniones

● En cuanto a la gastritis de este paciente, se encontró que era producida por la


bacteria Helicobacter pylori. Esta bacteria sobrevive en el medio ácido del
estómago gracias a:

- Ureasa

● La bilirrubina directa aumentada en cirrosis hepática se excreta en la orina debido a:

- Ser hidrosoluble

● Paciente de 42 años con adenocarcinoma ductular. La TC ha demostrado claramente que el


tumor está en el cuello del páncreas y que hay un gran vaso ocluido. ¿Cuál de los siguientes
vasos estaría más probablemente obstruido?

- Vena porta.

● En relación a la histología hepática, marque lo correcto:


- La zona 3 se encuentra más cerca a la vena central lobulillar

Mujer de 83 años acude a emergencia por dolor abdominal desde hace


4 días, localizado en epigastrio, irradiado a ambos hipocondrios,
nauseas, vómitos y distensión abdominal; tiene antecedente de
cardiopatía hipertensiva, diabetes mellitus tipo II y fibrilación auricular.
Refiere deposiciones diarreicas muco sanguinolentas hace 1 día. Los
exámenes iniciales muestran PA: 110/60 mmHg, FC: 110/mn, leucocitos:
17800, neutrófilos de 93%. TAC abdómino pélvica se observa oclusión
completa de arteria mesentérica superior por trombo asociado a placa
ateromatosa.
● En esta paciente, ¿cuál de las siguientes sustancias no tendrá una
considerable disminución en su absorción? (marque la mejor respuesta):

- Calcio

● El mecanismo de la diarrea muco sanguinolenta que presenta la paciente, puede


mejor definido como de tipo (marque la mejor respuesta):}

- Exudativa

● Considerando que se ha comprometido el íleon distal, entre otras áreas, la atrofia


o descamación del epitelio de superficie explicaría cuál de los signos o síntomas
de la paciente (marque la mejor respuesta):

- Diarreas mucosanguinolentas

● Producto de esta isquemia, la expresión de cuál de las siguientes enzimas se vería


notablemente disminuida. Marque la mejor respuesta:

- Enteroquinasa

● Durante la cirugía, el cirujano observó que además la paciente tenía divertículos


en el sigma. Se sabe que estos divertículos:

- Se pueden asociar a estreñimiento crónico


PREGUNTAS PARCIAL:
1)Los diferentes segmentos del tubo digestivo son susceptibles de reflejos y movimientos
según su contenido. Si colocáramos mediante una sonda un bolo alimenticio directamente
en el tercio medio del esófago:​ se producirá ondas secundarias.
2) El peristaltismo del intestino delgado se puede intensificar debido a: ​irritación de la
mucosa
3) Un bolo alimenticio grande y poco masticado se atasca en el esófago, esto ocasiona una
sensación de dolor que se transmita por los nervios: ​esplácnicos
4) Un paciente es diagnosticado con un tumor neuroendocrino productor de la serotonina,
este provocará en el sistema digestivo:​ diarrea
5) La fase oclusal de la masticación se realiza con la contracción de los músculos inervados
por el nervio craneal: ​V
6) El divertículo de Meckel es una anomalía congénita que ocurre por la persistencia del
conducto vitelino y da origen a una estructura sacular, el cual se encuentra en el: ​borde
antimesentérico
7) En el sistema digestivo la liberación hormonal se presenta ante diversos factores o
estímulos. La hormona ______ es estimulada por la presencia de alimentos en el bulbo
duodenal a predominio de ácidos grasos y triglicéridos, por estimulación vagal y por la
hormona secretina: ​colecistocinina (CCK)
8)Durante la fase faríngea de la deglución ocurre el siguiente mecanismo:​ la onda
peristáltica lleva el alimento hacia el esofago
9) En un paciente de 43 años con tumor carcinoide de páncreas productor de gastrina
(Sídrome de Zollinger-Ellison) se puede esperar encontrar una potenciación del reflejo:
gastrocólico
10) El mecanismo de la defecaciòn incluye la participaciòn de diversas estructuras ¿Cuál de
las siguientes alternativas es correcta?: ​puede ser mediado por un reflejo intrínseco
11) Si al intubar a un paciente, por error se ingresa el tubo endotraqueal en el esófago y se
insufla el manguito endotraqueal (globo TET), la dilatación de este manguito generará:
múltiples ondas secundarias
12)En los carcinomas (neoplasia maligna) es frecuente que ocurran la metástasis a través
de los vasos venosos. En caso de un carcinoma del tercio posterior del esófago, ubicado en
la cara lateral izquierda, es más probable que la metástasis viaje por la vena: ​hemiácigos
accesoria
13) Los movimientos en masa son iniciados por el reflejo: ​duodenocólico
14)El inicio de la fase faríngea de la deglusiòn se debe a los estímulos sensitivos que viajan
al nervio craneal: ​V
15) El peristaltismo del íleon es intensificado gracias al reflejo: ​gastroileal
16) En una persona si enfermedad se espera que el tránsito intestinal se vea disminuido
cuando se presenta el reflejo: ​doloroso
17) El orificio omental, o hiato de Winslow, se encuentran limitados por el ligamento:
hepatoduodenal
18) Paciente con insuficiencia mitral moderada a severa, con aumento de volumen de la
aurícula izquierda; esta condición tendrá como consecuencia a nivel del sistema digestivo:
la disfagia a sólidos
19) El omento mayor deriva del mesenterio ______ y se inserta en el ____​:
dorsal/Estómago
20)¿ Cuál de las siguientes sustancias disminuye la fuerza de las contracciones de
segmentación? :​ atropina
PREGUNTAS:
1) Sustancia que inhibe la secreción y la motilidad del estómago prolongando el tiempo
de digestión:​ Péptido insulinotrópico dependiente de la glucosa (GIP)
2)El dolor periumbilical o epigástrico en el inicio de una apendicitis aguda se debe a:
estimulo del sistema simpático
3) Cuando la contracción peristáltica primaria esta atraviesa el tercio superior del esofago, el
esfinter esofagico inferior estara: ​relajado
4)El nacimiento de la arteria mesentérica superior se puede encontrar en cúal de los
cuadrantes abdominales: ​epigastrio
5) ¿Cuál de los siguientes péptidos inhibe el vaciamiento gástrico? ​colecistoquinina
6) Durante una cirugía oncológica: ¿la extirpación de cual de los siguientes órganos se vería
comprometida por la presencia de adventicia?: ​recto
7) Marque lo correcto:​ La hernia fisiológica se produce en la sexta semana y es la salida
temporal de asas intestinales a través del cordón umbilical
8) Estimula la secrecion acida gastrica: ​proteínas
9)Marque la respuesta correcta en relación a la gastrina:​ Las células G son las productoras
y se encuentran en el antro gástrico.
10) Con respecto a las ondas lentas, marque la afirmación correcta:​ son contracciones
rítmicas espontáneas
11)Estimula la producción de saliva: ​vasodilatación periglandular
12)En el esofago, el cambio de adventicia a serosa se produce a nivel de la: ​cuarta
estrechez
13) En el síndrome de la boca seca o síndrome de Sjogren, una de las complicaciones
asociada es: ​caries
14) Un paciente hipertenso está tomando un medicamento bloqueador de receptores alfa 1
adrenérgicos (prazosina) y como efecto secundario se queja de: ​Hiposalivación
15)En cuanto a la saliva, marque lo correcto: ​La amilasa cumple función digestiva
16) Durante el ataque con gas sarín (bloqueador de la acetilcolinesterasa) en el metro de
Tokio, en 1995, el personal de salud notó que los pacientes afectados presentaban:
Hipersalivación
17)Respecto a la anatomia del estomago, marque lo correcto: ​la arteria gástrica derecha
nace de la arteria hepática común
18)Respecto a la glándulas salivales, marque lo incorrecto: ​la glándula sublingual tiene
forma de garfio
19) En relación a la fisiologia gastrica, marque lo correcto: ​la cimetidina actúa en la región
basolateral de la célula parietal
20) El alcohol y la cafeína estimulan la fase _______ de la producción de ácido clorhídrico:
gástrica
21) El efecto colerético de las sales biliares se refiere a: ​estímulo de secreción biliar
22)Paciente de 21 años con dolor intenso en arcada dentaria superior debido a emergencia
de la tercera molar. FC: 136lpm, PA (140/80mmHg), agitado, pálido, son sudoración fría y
boca con saliva espesa. En este paciente se espera que la secreción exocrina pancreática
se encuentre: ​disminuida
23)Hombre de 65 años con isquemia intestinal leve por oclusión aterosclerotica de la arteria
mesenterica superior, pero la irrigación sanguinea colateral ha retrasado el inico de la
necrosis. ¿ Qué vasos ofrecen colaterales entre el tronco celíaco y la arteria mesentérica
superior? ​Pancreaticoduodenal superior e inferior
24) Durante una endoscopia, se ingresa en la papila mayor en la segunda porción del
duodeno, y dentro de la vía biliar extrahepática, se toma una muestra de secreción biliar.
Dicha muestra debe contener ácido: ​quenodesoxicólico
25) Paciente de 42 años con adenocarcinoma ductular. La TC ha demostrado claramente
que el tumor está en el cuello del páncreas y que hay un gran vaso ocluido. ¿Cuál de los
siguientes vasos estaría más probablemente obstruido?: ​vena porta
26)Las ramas más pequeñas del árbol biliar son: ​Canalículos biliares
27) La bilirrubina directa aumentada en cirrosis hepática se excreta en la orina debido a: ​ser
hidrosoluble
28) En relación a la secreción biliar y su composición, marque lo correcto: ​las sales biliares
corresponden al 50% de su composición
29) Las sales biliares son absorbidas a la sangre en: íleon
30) Dentro de las funciones de las células de Ito, marque lo incorrecto: ​pueden fagocitar
patógenos y actúan como presentadoras de antígeno

31)Un efecto secundario en estómago por la acción de la secretina es: ​menor actividad de
la pepsina
32)Considerando la gastritis que sufre el paciente, en cuanto a la barrera de su mucosa
gástrica ¿cual de las siguientes afirmaciones es correcta? ​Se vería alterada por una
vagotomía
33) En este paciente con cirrosis, producto del compromiso del árbol biliar, es posible
encontrar elevados los niveles sanguíneos de: ​gammaglutamiltranspeptidasa
34) La producción de saliva en este paciente tiene como característica: incluye ptialina
35)¿Cuál de las siguientes sustancias es segregada en el páncreas?:​ tripsina
36) A pesar de la gastritis del paciente, se puede decir que las células parietales se
caracterizan por:​ sintetizar factor intrínseco

37) El recto recibe irrigación de tres arterias principales, la zona que recibe suministro de
una rama de la mesentérica inferior se caracteriza por tener epitelio: ​simple cilíndrico
38) Considerando que se ha comprometido el íleon distal, entre otras áreas, la atrofia o
descamación del epitelio de superficie explicaría cuál de los signos o síntomas de la
paciente (marque la mejor respuesta): ​Diarreas mucosanguinolentas
39)Al realizar un corte transmural de un segmento del colon transverso, será evidente que:
En la zona donde no hay tenias solo hay una capa muscular
40) En este caso, se puede encontrar zonas de penumbra en: ​colon transverso
41) En la paciente, el flujo sanguíneo de cúal de las arterias pancreáticas estará afectada:
pancreaticoduodenal posteroinferior
42)Paciente de 23 años con dolor intenso periumbilical de inicio agudo. Durante, la cirugia,
el cirujano encuentra una estructura sacular perforada en el intestino delgado a unos 60 cm
proximal a la válvula ileocecal. Dicha estructura es un derivado embriológico del:​ conducto
vitelino
43) En un neonato con vómitos biliosos, persistentes y con baja ponderal, usted
sospecharía: ​atresia duodenal
44) Tras una vagotomía (resección del vago) por enfermedad ulcero péptica en un paciente,
usted esperaría encontrar: ​aumento de ph gástrico
45) Marque lo correcto con respecto a Esófago de Barret: ​se relaciona con reflujo
gastroesofágico
46) La metoclopramida estimula el vaciamiento gástrico aumentando la fuerza de
contracción de las paredes gástricas, esto puede conseguirlo mediante la estimulación
indirecta de neuronas liberadoras de: ​acetilcolina
47) El esofago de Barret se carateriza por presentar ______ en el esofago: ​metaplasia
intestinal
48)El conducto colédoco recibe dicho nombre partir de: ​la unión del conducto cístico con el
hepático común
49)En la anemia perniciosa, con anticuerpos anti células parietales presentes, usted
esperaría encontrar: ​Deficiencia de vitamina b12
50) El estímulo de las células parietales producirá la secreción de: ​HCl y factor intrínseco
51) La acción de la secretina sobre las células ductales del páncreas, es potenciada por:
Acetilcolina
52) Con respecto a la secreción gástrica de HCl: ​La bombesina estimula las células g para
la secreción de gastrina
53)¿Cuál de las siguientes estimula la secreción enzimática exocrina del páncreas? :
Colecistoquinina
54) Los monosacáridos que conforman la lactosa ingresan al enterocito usando el/los
transportador (es)______ : ​SGLT-1
55) Los quilomicrones llegan al hepatocitos por: ​sinusoides
56) El vibrio cholerae produce diarrea porque:​ aumenta la producción de AMPc en los
enterocitos
57) El azúcar de mesa (sacarosa) es digerido a dos monosacáridos que comparten el
transportador ______ para su absorción: ​GLUT2
58) La absorción de ácido fólico se produce gracias a: ​Cotransporte dependiente de Na+
59) Paciente de 62 años con vólvulo de intestino delgado e isquemia intestinal. ¿Qué
estructura se utiliza como punto de referencia para determinar la posición de la unión
duodenoyeyunal?: ​Ligamento suspensorio del duodeno (de Treitz)
60) El compromiso intestinal en Enfermedad de Crohn es clásicamente en la región:
ileocólica
61) Una manifestación clínica del sobrecrecimiento bacteriano es: ​esteatorrea
62) En la enfermedad de Crohn es posible encontrar células de Paneth en el colon. Esto se
puede deber a que una función de estas células es: ​Mantener la inmunidad innata
63) En los pacientes con colecistitis aguda no operable, una opción es la colocación de una
sonda por el cístico, procedimiento en el que se ingresa con dificultad debido a la estrechez
del cístico y a la presencia de: ​Válvula de Herring
64) Paciente varón de 73 años es traído a urgencias con dolor abdominal intenso. En la
tomografía se observa un aneurisma de la aorta abdominal que afecta al origen de la arteria
mesentérica superior. Basado en sus conocimientos, usted sabe que el órgano que se
encontrará más afectado será:​ yeyuno
65) Durante la defecación se requiere: ​señales inhibitorias en el nervio pudendo
66) En los neonatos sanos, el color verde oscuro del meconio (primera deposición) se debe
a: ​pigmentos biliares
67) En la enfermedad de Gilbert hay una reducción en la concentración intracelular de
glucuroniltranferasa en el hepatocito, lo cual conlleva a un aumento plasmático de:
bilirrubina indirecta
68) Paciente con deficiencia congénita de procolipasa, sufre de esteatorrea cada vez que
come comidas ricas en grasas. Los lípidos que no son hidrolizados debido a esta deficiencia
son: ​triglicéridos
69) En el metabolismo hepático de las toxinas, las que son liposolubles son metabolizadas
por la reacción de _____ que son catalizadas por las enzimas del ______ : ​Fase 1
citocromo p450
70) El principal componente de las heces es: ​Agua
71) El aumento de la presión intraluminal en el sigmoides de manera crónica produce
evaginaciones de la mucosa del colon en las zona más débiles de su pared, a veces con
materia fecal, denominadas: ​divertículos
72) En un paciente alcohólico con esplenomegalia, bilirrubina elevada u tiempo de
protrombina elevado, que acude por melena, usted sospecharía que esta se debe a: ​varices
esofágicas
73) Cuando se consume maltosa, la absorción de los monosacáridos requiere
necesariamente la presencia de: ​SGLT1
74) Paciente de 74 años de edad con shock hipovolémico asociado a deshidratación aguda
severa. ¿Cuales de las siguientes complicaciones es posible encontrar en este paciente:
Hipoxia en zona 3 del lobulillo hepático infarto mucoso del intestino
75) La fase de despolarizante de las ondas lentas se debe a apertura cìclica de canales de
calcio
76)La onda peristaltica secundria al esofago en la deglucion, es producida por: ​plexo
mienterico esofagico
77) Paciente de 2 años, llega a emergencia por haber ingerido una moneda con la que
estaba jugando. El lugar más probable donde puede haber quedado este objeto es sobre el
estrechamiento a nivel del músculo: ​Cricofaríngeo
78)¿Cuál de los siguientes puede inhibir la relajación receptiva en el estómago?:​ atropina
79) Al ingerir una cantidad de glucosa por vìa oral, esta es interiorizada en las cèlulas del
organismo más rápido que si esa misma cantidad hubiese sido administrada por vía
endovenosa. Este fenómeno sucede gracias a una sustancia secretada por las células​: K
80) Paciente ha sido puesto en “Nada por vía oral” (nil per os, NPO) durante 24 horas, la
sensación de hambre en este paciente se debe la par esencia en el hipotálamo de la
sustancia: ​Neuropéptido Y
CLAVES PARCIAL DIGESTIVO
2018 - 1

1. Experimentalmente se utiliza atropina (anticolinérgico) para inhibir la secreción


de gastrina, sin embargo la secreción de esta hormona se sigue dando ante
estímulos vagales. Esta situación se explica porque la atropina……………:

(unidad 1, sesión 3, logro 3 : describir las hormonas gastrointestinales: estímulo y


funciones de la gastrina y colecistoquinina)

a. Inhibe la acción de acetilcolina e histamina en la célula G


b. No bloquea la acción del péptido GRP
c. Solo inhibe la acción del péptido GRP en la célula G
d. Bloquea parcialmente la bomba de protones en la célula G

2. Ante una lesión del X par craneal, ¿cuál de los siguientes músculos mantiene
conservada su función?:

(unidad 2, sesión 11, logro 2: Músculos de la faringe: identificación, constrictores y


longitudinales)

a. Elevador del velo del paladar


b. Palatofaríngeo
c. Estilofaríngeo
d. Constrictor superior de la faringe

3. Paciente varón de 30 años, adelgazado, víctima de agresión con arma blanca


(cuchillo), presenta herida punzo penetrante en región subcostal derecha, en el
acto operatorio se encontró lesión de órgano sólido. ¿Cuál de los siguientes
órganos estará comprometido?:

(unidad 1, sesión 1, logro 5: graficar la topografía de superficie, órganos por


cuadrante)

a. Hígado
b. Vesícula biliar
c. Ángulo hepático del colon
d. Páncreas
4. Recién nacido que presenta tumoración abdominal a nivel del cordón umbilical
(como se observa en la fotografía). ¿cuál de las siguientes afirmaciones es
correcta respecto a este defecto en el desarrollo embriológico del intestino?:

(unidad 1, sesión 5, logro 2: identificar las anomalías del desarrollo del intestino
medio)

a. Corresponde a una hernia


inguinal
b. No está asociado a otras
malformaciones
c. Se asocia a malformaciones
cardiacas y del tubo neural
d. Corresponde a una Gastrosquisis

5. Al recibir un paciente con signos de hipovolemia y antecedente de lesión


contusa en abdomen por accidente de tránsito, usted identifica
radiológicamente: lesión de primera vértebra lumbar y signos de lesión
pancreática; durante la cirugía se observó pobre irrigación de asas intestinales.
El vaso arterial afectado es….:

(unidad 1, sesión 1, logro 6: reconocer las estructuras a nivel de L1, nivel de los
principales vasos sanguíneos)

a. Arteria Mesentérica inferior


b. Arteria esplénica
c. Arteria mesentérica superior
d. Arteria hepática común

6. Respecto a las sustancias gastrointestinales que regulan la secreción


pancreática; marque la afirmación correcta:

(unidad 1, sesión 3, logros 2 y 3: describir las hormonas gastrointestinales:


estímulos y funciones)

a. La colecistoquinina (CCK), es la hormona más importante para la secreción de


bicarbonato pancreático
b. La Secretina, es la hormona más importante para la secreción enzimática del
páncreas
c. La acetilcolina estimula la secreción enzimática y de bicarbonato del páncreas
d. La colecistoquinina (CCK) estimula al páncreas solo para secreción enzimática
7. Un varón de 50 años es sometido a extirpación de duodeno y parte proximal de
yeyuno. La pérdida de estímulo hormonal en el páncreas para la secreción de
bicarbonato se explica por la pérdida de las células ……………………

(unidad 1, sesión 3, logro 3: describir las hormonas gastrointestianles: estímulos y


funciones de la secretina y péptido insulinotrópico dependiente de glucosa)

a. “I” duodenales productoras de secretina


b. “M” duodenales productoras de CCK
c. Parietales, productoras de Factor intrínseco
d. “S” productoras de secretina

8. Varón de 35 años acude a la emergencia por dolor abdominal difuso irradiado al


cuadrante inferior izquierdo, asociado a vómitos y fiebre. Cursa con distensión
abdominal; siendo evaluado por cirugía, se evalúa con enema baritado
(sustancia de contraste - imagen). Se decide operar, encontrándose apendicitis
aguda izquierda. Según lo descrito, diga usted ¿cuál es la anomalía en el
desarrollo del tubo digestivo en este caso?:

(unidad 1, sesión 5, logro 3: identificar las anomalías del desarrollo del intestino
medio: defectos de rotación, estenosis y atresias)

a. Defecto por malrotación del intestino


anterior
b. Defecto por malrotación del intestino
medio
c. Defecto por atresia y estenosis en el colon
d. Persistencia de conducto vitelino

9. Experimentalmente se aplica leptina a nivel del hipotálamo en ratas de


laboratorio. Esto ocasionaría un incremento ………

(unidad 1, sesión 3, logro 5: explicar la influencia de algunas sustancias en el


control del apeptito: leptina, grelina, peptido similar a glucagon y péptido yy)

a. Del neuropétido CART


b. Del neuropéptido NPY
c. Del apetito
d. De sustancias orexigénicas
10. Juana cae de la bicicleta y se fractura la región anterior del hueso maxilar
superior con compromiso de la fosa incisiva. En este caso se esperaría encontrar
alteración en la …………………

(unidad 2, sesión 8, logro5: paladar: paladar duro y blando: irrigación e


inervación)

a. Sensibilidad de la encía palatina anterior


b. Sensibilidad de la encía bucal anterior
c. Sensibilidad de la encía lingual
d. Sensación del gusto en la encía palatina incisiva

11. Niña de 4 días de nacida, es llevada a la emergencia pediátrica por presentar


llanto constante, la madre refiere coloración azulada de labios al momento de
lactar, acompañado de tos persistente y dificultad respiratoria así como
distención abdominal. Se le coloca sonda nasogástrica para alimentación
notando que esta es retornada a la cavidad oral en todos los intentos. ¿Cuál es la
anomalía del desarrollo en este caso?

(unidad 1, sesión 4, logro 3: identificar las anomalías en el desarrollo del esófago:


atresia y/o fístula traqueo esofágica)

a. Fístula traqueo esofágica proximal y distal


b. Atresia esofágica proximal con fístula traqueo esofágica distal
c. Atresia esofágica distal con fístula traqueo esofágica proximal
d. Solo fístula traqueo esofágica

12. Paciente varón de 36 años, es traído a la emergencia luego de sufrir un accidente


de tránsito. Presenta traumatismos múltiples en cabeza y tronco, al examen
físico se evidencia hematoma en hemicara izquierda ligera protrusión y caída del
lado izquierdo del maxilar inferior por lo que se realiza una tomografía donde se
halla una fractura de la apófisis coronoides del maxilar inferior. ¿qué músculo
está relacionada directamente con esta situación?

(unidad 2, sesión 8, logro 2: Cavidad oral: estructuras óseas (maxilar superior,


inferior y palatinos) y articulación témporo mandibular)

a. Masetero
b. Buccinador
c. Pterigoideo medial
d. Temporal
13. Respecto al mecanismo de la defecación, ¿cuál de las siguientes afirmaciones es
correcta?

(unidad 2, sesión 13, logro 6: motilidad del intestino grueso: contracciones


segmentarias, movimientos en masa, defecación y reflejo gastrocólico)

a. Se produce contracción refleja del esfínter anal interno


b. Se produce relajación del esfínter anal externo por efectos del VIP y óxido
nítrico
c. En la posición de “cuclillas” el músculo puborectal se halla relajado
favoreciendo la evacuación de la materia fecal
d. La presencia de materia fecal en el recto estimula la contracción del sigmoides
por los nervios pudendos

14. Un paciente luego de un evento traumático sufre lesión del piso de la boca, se
constata daño solo del nervio lingual, se esperaría encontrar en este caso
disminución de la ………

(unidad 2, sesión 10, logro 3: Irrigación e inervación de la lengua)

a. Motilidad de la lengua en sus dos tercios anteriores


b. Sensibilidad al tacto en los dos tercios anteriores de la
lengua
c. Sensibilidad al dolor en el tercio posterior de la lengua
d. Sensación del gusto en tercio posterior de la lengua

15. La estructura número 3 mostrada en el gráfico corresponde a……….…Y


está………..

(unidad 2, sesión 9, logro 2: Partes de un diente. Capas del diente: Esmalte:


características y células que lo producen)

a. La Dentida – formada por


ameloblastos
b. El Cemento – mineralizado en
90%
c. El Esmalte – mineralizado en
98%
d. El Esmalte – formado por
odontoblastos
16. Varón de 50 años, se halla en un procedimiento odontológico : curación dental
de la segunda molar de la arcada inferior derecha. En un momento determinado
paciente acusa de intenso dolor en la pieza dentaria en tratamiento. La vía
aferente del dolor viaja a través del nervio …………

(unidad 2, sesión 9, logro 6: Inervación de los dientes)

a. Nervio alveolar superior posterior


b. Nervio alveolar inferior
c. Nervio palatino menor
d. Nervio lingual

17. ¿Cuál de las siguientes sería el correcto orden del recorrido de la gastrina desde
su secreción hasta su llegada al órgano diana?:

(unidad 1, sesión 3, logro 1: reconocer las características de las sustancias


reguladoras gastrointestinales: hormonas, sustancias paracrinas y neurocrinas)

a. Secreción, circulación sistémica, hígado, circulación portal y órgano diana


b. Secreción, circulación portal, hígado, circulación sistémica y órgano diana
c. Hígado, circulación sistémica, circulación portal y órgano diana
d. Secreción, circulación portal, hígado, vía biliar y órgano diana

18. Un paciente sufre de daño a nivel del cuello con lesión de algunos músculos de la
faringe, se evalúa el caso, determinando dificultad para la elevación de la
faringe y para el cierre del itsmo de las fauces. El músculo probablemente
afectado es el…….
(unidad 2, sesión 11, logro 2: Músculos de la faringe: identificación, constrictores y
longitudinales)
a. Músculo palatogloso
b. Músculo estilofarinfeo
c. Músculo constrictor medio e inferior
d. Músculo palatofaringeo

19. El paso del quimo al intestino delgado provocará un aumento de la hormona


……………… debido al cambio en el pH del medio.

(unidad 1, sesión 3, logro 3: describir las hormonas gastrointestianles: estímulos y


funciones de la secretina y péptido insulinotrópico dependiente de glucosa)

a. Ghrelina
b. gastrina
c. motilina
d. Secretina
20. Un ratón es sometido a procedimientos de ingeniería genética para evitar la
formación de los receptores CCK tipo B a nivel del tubo gastrointestinal. ¿Cuál de
las siguientes hormonas se acumulará en sangre como consecuencia de esta
modificación genética?

(unidad 1, sesión 3, logro2: describir las hormonas gastrointestinales: estímulo y


funciones de la gastrina y colecistoquinina)

a. gastrina
b. secretina
c. Péptido inhibidor gástrico
d. Insulina

21. Un niño de tres años llega a emergencia con disfagia (dificultad para tragar),
dolor retro esternal, salivación y llanto. Se sospecha de ingesta de cuerpo
extraño: moneda en el esófago; al ser evaluado se constata en una radiografía
presencia de cuerpo extraño a nivel de C6 (6° vértebra cervical). El cuerpo
extraño estará suspendido a nivel del estrechamiento producido por………..

(unidad 2, sesión 11, logro4: Esófago, características anatómicas, relación con


órganos vecinos y estrecheces)

a. El cayado aórtico
b. El hiato esofágico
c. El bronquio principal izquierdo
d. El músculo cricofaríngeo

22. En relación al movimiento de peristaltismo del tubo digestivo, en el siguiente


gráfico, ¿cuál de los eventos señalados es correcto a nivel de la flecha negra?

(unidad 2, sesión 7, logro 6: Control hormonal y tipos de movimiento)

a. A nivel del músculo longitudinal se


libera óxido nítrico y acetilcolina
b. A nivel del músculo circular se liberan
Acetilcolina, Sustancia P y
neuropéptido “ Y”
c. A nivel del músculo longitudinal se
libera Acetilcolina y sustancia P
d. A nivel del músculo circular se libera
óxido nítrico y PIV
23. Un paciente refiere no percibir algunos sabores, al examen físico constata
alteración del sabor dulce y umami. ¿Cuál de los siguientes nervios estará
alterada en su función?

(unidad 2, sesión 10, logro 5: Sabores, tipos y mecanismos moleculares para su


detección)

a. Glosofaríngeo (IX par)


b. Lingual (rama del V par)
c. Hipogloso (XII par)
d. Cuerda del tímpano (VII par)

24. Según la imagen, ¿Cuál de las estructuras señaladas facilita el pasaje de


alimentos sólidos y líquidos hacia el esófago?

(unidad 2, sesión 11, logro 3: Características más importantes de la nasofaringe, la


orofaringe y la hipofaringe)

a. Receso piriforme - 1
b. Epiglotis - 2
c. Receso piriforme - 2
d. Receso faríngeo - 3 1
3

25. En el caso de un paciente con gastrinoma (tumor productor de gastrina), la


presencia de úlceras duodenales y erosión de la mucosa gástrica, se debe
principalmente a…….
(unidad 1, sesión 3, logro 2: describir las hormonas gastrointestinales: estímulo y
funciones de la gastrina y colecistoquinina)

a. La acción directa de la gastrina sobre la célula principal


b. La sobre expresión de los receptores de somatostatina en la célula parietal
c. El Exceso de HCl por estímulo directo de receptores de acetilcolina en la
célula parietal
d. El Exceso de HCl por estímulo de receptores CCK-B en la célula parietal
26. Neonato varón de 20 días de nacido que acude a la emergencia; madre refiere
presenta vómitos no biliosos, explosivos, llanto persistente, luce con sequedad
de mucosas y bajo peso. Al examen se observa una masa abdominal “reptante”.
La alteración más probable que explica el caso sería:

(unidad 1, sesión 4, logro 4: explicar el desarrollo del intestino anterior)

a. Fístula traqueo esofágica


b. Reflujo gastro esofágico
c. Hipertensión endocraneana
d. Estenosis hipertrófica del píloro

27. Los movimientos en masa se caracterizan por:

(unidad 2, sesión 13, logro 6: Motilidad del intestino grueso: contracciones


segmentarias, movimientos en masa defecación y reflejo gastrocólico)

a. Movilizar grandes volúmenes en el intestino delgado


b. Transportar las heces hasta el recto
c. Tener una frecuencia de 1 a 3 contracciones por minuto
d. Transportar las heces hasta el ciego

28. Un joven obeso acude por presentar dolor de tipo ardor retro esternal, tos y
nauseas, se diagnostica reflujo gastroesofágico. ¿qué acciones recomendaría a
su paciente?:

(unidad 2, sesión 12, logro 5: Reflujo gastroesofágico: concepto y factores que lo


predisponen)

a. Dormir sin almohadas


b. Bajar de peso
c. Beber café para mejorar la digestión
d. Descansar con las piernas elevadas

29. La onda peristáltica primaria del esófago se caracteriza por ser originada ………

(unidad 2, sesión 12, logro 3: Motilidad esofágica: fases y características)

a. Por el plexo de meissner del esófago


b. Por el plexo mientérico del esófago
c. Por el reflejo de la deglución
d. Durante la masticación
30. A los pocos días de nacido, regresa a neonatología un niño con problemas de
motilidad del colon; los estudios determinan ausencia congénita de células
ganglionares. Según el gráfico ¿cuál es la capa en la que se determina la ausencia
de dichas células?

(unidad 1, sesión 2, logro 1: describir las generalidades de la estructura del tubo


digestivo: esófago, estómago intestino delgado y grueso)

a. Mucosa - 2
b. Muscular de la mucosa - 3
c. Muscular propia - 3
d. Muscular propia – 1

1 2

3
Un paciente varón de 68 años consulta por dolor y abultamiento en la región
inguinal derecha, que aparece tras la realización de un esfuerzo físico. En la
exploración en bipedestación presencia de una tumoración blanda, depresible,
que aumenta con la tos. El presente caso describe una hernia:
Inguinal directa.
Umbilical.
Femoral.
Inguinal indirecta.

la Grelina es sintetizada por………….. y activa las neuronas relacionadas con


……………………….en el núcleo arqueado del hipotálamo}

El páncreas/AGRP-NPY
El intestino delgado/POMC-CART
El estómago/AGRP-CART
El estómago/AGRP-NPY

Cuál de los siguientes órganos son intraperitoneales:

Estómago, Vesicula biliar, Ileón, Higado


Páncreas, Colon descendente, Hígado, Vesícula biliar
Recto, Hígado, Colon transverso, Yeyuno
Estómago, Yeyuno, Duodeno, Páncreas

La distención gástrica por los alimentos produce incremento de secreción de HCl


mediante el siguiente mecanismo:

Producción de Gastrina que desencadena su cascada de señalización en la célula


parietal vía proteína Gi
Producción de Acetilcolina que desencadena su cascada de señalización en la célula
parietal vía proteína Gs
Producción de Gastrina que desencadena su cascada de señalización en la célula
principal vía proteína Gs
Producción de Gastrina que desencadena su cascada de señalización en la célula
parietal vía proteína Gq

La inervación de la piel del abdomen debajo del ombligo hasta la región púbica
está dada por:

T7, T9
T11, T12, L1
T7, T9, L2
L2, L3, S1
Paciente de 38 años que tras riña durante partido de futbol sufre un traumatismo
con arma blanca en cuadrante inferior izquierdo del abdomen. En la tomografía
de urgencias se evidencia gran hematoma de pared y ausencia de
neumoperitoneo. Desde la piel hacia al peritoneo, en ¿qué orden se atravesó la
pared abdominal?

Oblicuo externo, oblicuo interno, musculo transverso, TCSC


TCSC, oblicuo externo, oblicuo interno, musculo transverso
TCSC, oblicuo interno, oblicuo externo, musculo transverso
TCSC, musculo transverso, oblicuo externo, oblicuo interno
Oblicuo externo, musculo transverso, oblicuo interno, TCSC

Alrededor de 90% de los pacientes afectados por el síndrome de Zollinger-Ellison


desarrollan ulcera péptica. La causa es:

Gastritis atrófica
Hipersecreción ectópica de gastrina
Reflujo gastroesofágico
Gastropatía hipertrófica

Un hombre de 22 años sufre traumatismo en el flanco izquierdo superior del


abdomen al tratar de defenderse del robo de su vehículo. Un amigo lo traslada a
un centro hospitalario. El paciente presenta signos de hipovolemia con
taquicardia e hipotensión. Se queja de dolor en el sitio de lesión que se irradia al
hombro izquierdo. Elñ mórgano probablemente lesionado es

Hígado
Bazo
Estómago
Colon

Aproximadamente en la semana 6 del desarrollo embrionario, el intestino medio


gira 90° herniándose a nivel del:

Borde inferior del bazo


Caudal al borde hepático derecho
Cordón umbilical
Lado derecho del abdomen
La triada portal (arteria hepatica, vena portal y conducto biliar común) está
contenida en el:

Ligamento Gstroesplenico
Ligamento gastrohepático
Ligamento hepatoduodenal
Ligamento Falciforme

En el siguiente gráfico marque la relación correcta:

1
4

3 2

Omento mayor – 4
Plexo de Meissner – 1
Muscular de la mucosa – 3
Glándula de la mucosa
¿Cuál de las siguientes sustancias es liberada por neuronas en el tracto GI,
Participa en la regulación hidro electrolíticay produce relajación del musculo liso?

Gastrina
Secretina
CCK
VIP

La colecistoquinina inhibe:

El Vaciamiento gástrico
La Secrecion pancreatica de HCO3-
La Contraccion de la vesicular biliar
La Relajación del esfínter de oddi

¿La secreción de cuál de las siguientes es inhibida por un pH bajo?

Secretina
Gastrina
CCK
VIP

El conducto biliar deriva del:

Ectodermo.
Endodermo.
Mesodermo.
Mesotelio.

Respecto a la imagen mostrada la estructura señalada con el número 1


corresponde a:

Glándula parótida
Glándula sublingual
Glándula submaxilar
Glándula lingual
2

Los vasos mesentéricos superiores se hallan a nivel de:

El fondo gástrico
El cuello del páncreas
La cabeza del páncreas
El hilio hepático
La imagen mostrada representa un defecto de ……………… y recibe el nombre
de:……………………….

La pared abdominal/onfalocele
La pared abdominal/divertículo de Meckel
Malrotación intestinal/gastrosquisis
La pared abdominal/gastrosquisis
Ecu 1
Caso 1
Lactante masculino de 5 semanas de edad, producto de un primer embarazo normal controlado.
Peso de nacimiento 3.120 g y talla de 51 cm. Lactancia materna satisfactoria, con buen incremento
ponderal. Sin antecedentes patológicos hasta 5 días atrás, cuando comienza con vómitos
postprandiales no biliosos, lácteos. Los síntomas aumentan en frecuencia y magnitud hasta hacerse
explosivos después de cada alimentación. No refiere fiebre, tos, diarrea ni lesiones cutáneas. Es
importante destacar que, pese a los vómitos, el niño conserva el apetito y llora de hambre. Al
examen físico presenta buen estado general. Abdomen blando, depresible e indoloro, asociado a
distensión del hemiabdomen superior. Sin signos de deshidratación. No se palpan masas
abdominales. Exámenes de laboratorio: hemograma normal. Signos inflamatorios de fase aguda
negativos. Alcalosis metabólica leve en sangre venosa.
Preguntas:
1. Debido al signo de la onda peristáltica que se evidencia en este paciente, se debe
asumir que como consecuencia hay un aumento en la liberación de ___________ en la
pared del estómago:
a) Secretina
b) Histamina
c) Gip
d) Adenosina
2. Al momento de realizar la pilorotomía (corte del músculo del píloro para descomprimir)
¿Qué hormona se comenzará a liberar rápidamente en respuesta a este tratamiento?
a) Secretina
b) Bombesina
c) Colecistoquinina
d) Gastrina
3. La oliva pilórica se logra palpar a nivel de:
a) Epigastrio parte superior
b) Hipocondrio derecho
c) Entre epigastrio y mesogastrio
d) L1
4. Producto de la hipertrofia pilórica uno esperaría que los niveles de Gastrina se
encuentren:
a) Elevado
b) Disminuido
c) Depende del alimento
d) Normales
5. Si el paciente tuviera 35 años, uno podría pensar como diagnóstico diferencial de la
hipertrofia de píloro, la presencia de un tumor neuroendocrino productor de hormona:
a) Gastrina
b) Bombesina
c) Colecistoquinina
d) Secretina
6. En este paciente se puede esperar una mayor liberación de:
a) Enzimas pancreáticas
b) Gip
c) Secretina
d) Insulina
7. En relación con la estructura afectada se encuentra:
a) Mesenterio propiamente dicho
b) Ligamento gastroesplénico
c) Omento mayor
d) Ligamento redondo
Caso 2
Estudiante de medicina de la UPC de 21 años sufre de gastritis aguda ocasionada por comer en
lugares poco higiénicos. Suele consumir caramelos ( chupar ) mientras está en clase hasta la tarde.
Toma gaseosas regularmente (carbohidratos 46%, sodio 53%). También toma regular cantidad de
leche (grasa 35%, lactosa 35%, proteínas 30%), pues le calma un poco el dolor el ardor que siente
por la gastritis. Incluso, cuando puede, se toma dos vasos de agua fría para calmar las molestias. Ha
decidido ir al médico para tratarse pues ya no soporta el dolor, el cual está seguro que los síntomas
se deben a una elevada producción de ácido clorhídrico en el estómago, y por ello le ha recetado
Ranitidina (antihistamínico), con lo que siente mejoría.
Preguntas:
1. Antes del uso de Ranitidina en este paciente, los valores de somatostatina en sangre
están:
a) Aumentados
b) Disminuidos
c) En valores normales
d) No hay somatostatina en sangre
2. En este paciente con gastritis aguda debida a una alta producción de ácido clorhídrico,
si se le hiciera un examen de sangre, se encontraría elevados los niveles de:
a) GIP
b) Sustancia P
c) Histamina
d) CCK
3. El consumo de leche produce directamente un aumento de los niveles séricos de la
hormona:
a) Gastrina
b) Leptina
c) Grelina
d) VIP
4. Entre las sustancias cerebrales que producen ansiedad esta la serotonina, la cual
también tiene acción;
a) Anoxigénica
b) Orexigenica
c) La acción de la serotonina en el apetito no está descrita
5. El uso de atropina en este paciente
a) Aumentará la producción de ácido clorhídrico
b) Aumentara el pH del estomago
c) Inhibirá la acción de las prostaglandinas
d) Disminuirá la acción del receptor CCK-B
6. El consumo rápido de 500 mL de gaseosa aumentara directamente la concentración
sérica de cual de las siguientes hormonas:
a) Cck
b) Secretina
c) Gastrina
d) Neuropéptido Y
7. El consumo de una pequeña cantidad de gaseosa aumentara directamente la
concentración sérica de cual de las siguientes hormonas
a) Péptido 1 similar al glucagón (GLP-1)
b) CCK
c) Secretina
d) Motilina
8. En este paciente con gastritis agua hiperclorhídrica, y debido a esta gran producción
de ácido, teóricamente podría tener como complicación:
a) Aumento del vaciamiento gástrico
b) Hipertrofia del píloro
c) Hemorroides
d) Ictericia
9. Este paciente tiene también la costumbre de tomar agua antes de sentarse a almorzar,
la cual hace que la producción de ácido clorhídrico del estomago
a) Aumente
b) Disminuya
c) Regrese a lo normal
d) No varié
10. El consumo de dos vasos de agua seguidos generara indirectamente un aumento​ en la
liberación de:
a) Péptido Y
b) Noradrenalina
c) Acido clorhídrico
d) Enzimas pancreáticas

ECU 2
Niño de sexo masculino de 2 años de edad, sufre de estreñimiento desde el nacimiento (1 deposición cada 3-4
días). Madre menciona que le estimula la defecación con un termómetro rectal, y continuo uso de enemas y
laxantes. Desde hace 6 meses comienza con vómitos postprandiales. Los síntomas aumentan en frecuencia y
magnitud y están en relación con los episodios de estreñimiento. No refiere fiebre, tos, diarrea ni lesiones
cutáneas. Al examen físico presenta regular estado general, luce deshidratado. Abdomen distendido, blando,
depresible e indoloro. No se palpan masas abdominales. Se permeabiliza el canal anal con termómetro rectal,
encontrando cierta resistencia. Salida de material fecal mal oliente en regular cantidad. Exámenes de
laboratorio: hemograma normal. Signos inflamatorios de fase aguda negativos. Alcalosis metabólica leve en
sangre venosa. Radiografía con enema baritado muestra recto y colon sigmoides dilatados (megacolon).
Biopsia profunda: ausencia de células ganglionares en la muestra enviada. Se realiza cirugía correctiva.

Preguntas:

1. En la zona afectada por esta enfermedad, se espera que las ondas lentas esten:
a) Disminuidas
b) Ocurran de manera normal
c) Abolidas
d) Aumentadas
2. Considerando la época que vivimos y como el paciente esta programado para sala de
operaciones, para lo cual se le toma una muestra de hisopado para descartar infección por
coronavirus. Al realizar el hisopado, el personal toca la pared de la ​orofaringe​, la cual envía una
señal sensitiva a través del par craneal.
a) IX
b) X
c) V3
d) V2
3. No se espera que sea causa del vomito
a) Ayuno prolongado
b) Estimulación faríngea y del glosofaríngeo
c) Irritación de la mucosa gástrica
d) Dolor intenso
4. Con respecto a la defecación señale el enunciado correcto:
a) Es estimulado por llenado de la cuarta parte del volumen rectal
b) Es un reflejo netamente local
c) Es completamente voluntario y mediado por el nervio pudendo
d) La aferencia parasimpática es transmitida por vía del nervio vago
5. Al examinar la orofaringe del paciente, uno puede hallar fácilmente la amígdala
palatina, pues esta se encuentra inmediatamente detrás del musculo:
a) Palatogloso
b) Palatofaringeo
c) Hiogloso
d) Elevador del velo del paladar
6. En este paciente se considera que esta abolido el reflejo:
a) Gastrocolico
b) Relajación receptiva
c) Coloileal
d) Rectoesfintereano
7. Es un reflejo propio de la pared intestinal:
a) Peristaltismo
b) Doloroso
c) Defecación
d) Cólicoileal
8. Durante la fase esofágica de la deglución, para un bolo alimenticio determinado, a
medida que avanza el bolo la fuerza de la contracción se hace mas:
a) Débil
b) Dependiente de acetilcolina
c) Fuerte
d) Hiperpolarizado
9. El contenido fecal se detiene en la zona inmediatamente proximal a la zona donde hay
una menor presencia de:
a) Péptido intestinal vasoactivo
b) Neuropéptido Y
c) Enteroquinasa
d) Acetilcolina
10. La percepción de la pirosis (sensación de dolor o quemazón en el esófago) asociado a
reflujo gastroesofágico, puede aparecer o exacerbarse debido a:
a) Ejercicio
b) Bipedestación
c) Uso de antiácidos
d) Somatostina
11. En cuanto a los reflejos gastrocólico y gastroduodenal en este paciente, indique lo
correcto:
a) Se puede considerar reflejos vago – vágales
b) El control del nervio vago sobre el recto se ha abolido
c) El reflejo Gastrocolico es mas marcado en adultos que en niños
d) Se dan por nervios intrínsecos del sistema entérico
12. Cuando este paciente ingiera sus alumentos, se espera que al momento de pasar el
bolo aluimenticio por el esfínter esofágico superior, la presión intraesofagica
disminuya en:
a) El cadias
b) Tercio medio del esofago
c) El lugar donde se contraiga la muscular propia
d) Porcion proximal al bolo
13. Respecto a la motilidad del colon, marque lo correcto
a) Con la distencion del estomago, suelen aparecer movimientos en masa
b) Se producen contracciones segmentarias principalmente en el colon izquierdo
c) Los movimientos de masa se encargan del mezclado de las heces
d) Las haustras del colon contribuyen al reflejo de defecación
14. Durante la mezcla y digestion, la fuerza de la contracción puede aumentar gracias a la
acción de la:
a) Gastrina
b) Estimulacion simpática
c) Secretina
d) VIP
15. Para estimular la motilidad intestinal se podría usar un analago de:
a) Secretina
b) CCK
c) Glucagon
d) Noradrenalina
16. Debido al acumulo de material fecal en todo el marco colonico, y a la irritación química
asociada, el peristaltismo del ileon distal se debe encontrar:
a) Inhibido
b) Estimulado
c) No sufre alteraciones
d) Afectado por un reflejo vago – vagal
17. Durante la fase faríngea de la deglución se espera que ocurra el siguiente fenómeno:
a) El paladar blando cierra la entrada a la nasofaringe
b) Se libera acetilcolina en musculo cricofaringeo
c) La onda peristáltica primaria lleva el alimento hacia el esófago
d) Inicialmente se desplaza el hueso hioides hacia abajo
18. Considerando que este paciente esta sometido a estrés por el agravamiento de su
enfermedad, es posible afirmar que sus ondas lentas están:
a) Desmotivadas
b) Hipopolarizadas
c) Despolarizadas
d) Hiperpolarizadas

Ci 1
1. Durante una cirugía oncológica, ¿La extirpación de cual de los siguientes órganos se
vería comprometida por la presencia de adventicia?
a) Vesícula biliar
b) Estomago
c) Recto
d) Yeyuno
2. Maque la respuesta correcta en relación a la gastrina:
a) Las células G son las productoras y se encuentran principalmente en el antro gástrico
b) Las células G se encuentran principalmente en el fondo gástrico
c) Se estimula por la liberación de noradrenalina
d) Al distenderse el estómago, se inhibe su producción
3. En relación con los principios de motilidad, marque lo incorrecto
a) El musculo liso del tubo digestivo funciona como un sincilio
b) Es regulado por calcio
c) Las dos terceras partes proximales del esófago están conformadas por musculo
esquelético
d) Se produce mayor fuerza de contracción que en el musculo esquelético
4. Para poder morder una manzana, es necesario el siguiente musculo:
a) Milohioideo
b) Buccinador
c) Tensor del paladar
d) Orbicular de los labios
5. El consumir caramelos indirectamente activa la vía:
a) POMC/CART
b) AGRP/NPY
c) MCR-4
d) Grelina
6. ¿En que casos los vómitos son siembre biliosos?
a) Estenosis
b) Atresia esofágica
c) Atresia yeyunal
d) Atresia duodenal
7. Las arterias que derivan del tronco celiaco son, excepto:
a) Gástrica izquierda
b) Hepática derecha
c) Arteria esplénica
d) Hepática común
8. Entre las múltiples causas de la enfermedad por reflujo gastroesofágico, se puede
considerar también a una alteración en las ________ del esfínter esofágico inferior.
a) Contracciones tónicas
b) Ondas secundarias
c) Ondas lentas
d) Glándulas subesofagicas
9. Con respecto a las ondas lentas, maque la afirmación correcta:
a) Su frecuencia aumenta por acción de la acetilcolina
b) Su frecuencia disminuye por acción de la noradrenalina
c) Son potenciales de acción que producen la contracción del tracto gastrointestinal
d) Son contracciones rítmicas espontaneas
10. Producto de la alimentación, se producen diversas sustancias peptídicas, cininas y
bradicininas, las cuales permiten que:
a) El flujo sanguíneo intestinal aumente hasta 8 veces
b) La acción de la lipasa pancreática se vea incrementada
c) Se produzca neovascularización en los territorios de las arterias abdominales
d) El consumo de O2 del intestino aumente ligeramente
11. Cual de los siguientes péptidos inhibe el vaciamiento gástrico
a) Gastrina
b) Péptido inhibidor gástrico
c) CCK
d) Motilina
12. Sustancia que inhibe la secreción y la motilidad del estomago prolongando el tiempo
de digestión
a) GIP
b) Polipéptido pancreático
c) GLP-1
d) Enteroglucagón
13. En cuanto a los reflejos gastrointestinales, un reflejo que estimula el transito intestinal
es el reflejo:
a) Gastro cólico
b) Entero-gástrico
c) Vomito
d) Cólico-ileal
14. ¿Cuál de los siguientes líquidos corporales tiene el pH más alto?
a) Bilis de la vesícula biliar
b) Jugo pancreático
c) Saliva
d) Jugo gástrico
15. El crecimiento de un adenoma de cuello de páncreas puede comprometer la pared
gástrica por continuidad, ¿Qué parte del estómago estaría comprometida con mayor
probabilidad?
a) Pared anterior del píloro
b) Pared posterior del cuerpo
c) Pared anterior del cardias
d) Pared posterior del antro
16. Marque la respuesta correcta
a) La pared gástrica en el fondo es mas delgada que en el cuerpo y antro
b) Todos los órganos del sistema digestivo tienen capa serosa
c) El bronquio derecho constituye una de las estrecheces del esófago
d) El esfínter de Oddi rodea la papila menor duodenal
17. Un paciente con apendicitis agua, la sensación de dolor por esta inflamación es llevada
por el nervio:
a) Pélvico
b) Esplácnico menor
c) Vago
d) Esplácnico mayor
18. Estimulan la secreción gástrica
a) Proteínas
b) Soluciones inertes
c) Soluciones hipertónicas de glucosa
d) Grasas
19. El dolor periumbilical o epigástrico en el inicio de una apendicitis aguda se debe a
a) Estimulación del nervio vago
b) Irritación del peritoneo parietal
c) Estimulo del sistema simpático
d) Íleo secundario
20. El uso de ranitidina bloquea el receptor H2 de la histamina en las células parietales. La
histamina llega a estas células por
a) Vía hematógena
b) Difusión
c) Se produce en la misma célula parietal
d) La luz gástrica
21. El nacimiento de la arteria mesentérica superior se puede encontrar en cual de los
cuadrantes abdominales:
a) Mesogastrio
b) Hipocondrio derecho
c) Hipogastrio
d) Epigastrio
22. Con respecto las ondas lentas, una estimulación adrenérgica producirá:
a) Hiperpolarización
b) Mas espigas
c) Una contracción más sostenida
d) Despolarización
23. Al delgutir el bolo alimenticio, es lógico suponer que al pasar por el esofago haya un
mayor consumo de oxigfeno en la pared del tercio:
a) No hay diferencia
b) Medio
c) Proximal
d) Distal
24. La arteria mesentérica superior emerge de la aorta a nivel de
a) Cabeza de pancreas
b) Hilio hepático
c) Cardias
d) Tronco celiaco
25. La hernia fisiológica se produce dentro de:
a) Cordon umbilical
b) Saco amniotico
c) Saco vitelino
d) Alantoides
26. El ligamento falciforme divide al hígado en dos lobulos derecho e izquierdo.
Embriologicamente derivan de:
a) Mesentrio ventral
b) Fascia de Todd
c) Mesogastrio dorsal
d) Vena umbilical
27. Paciente de sexo masculino de 43 años que es alimentado por via intravenosa durante
varias semanas. Producto de este tipo de alimentación, se encuentra en la endoscopia
atrofia de la mucosa antral. La causa mas probable de esta alteración es debido a los
bajos niveles de:
a) Gastrina
b) Secretina
c) GIP
d) CCK
28. Aproximadamente en la semana 6 del desarrollo embrionario, el intestino intermedio
gira 90° herniándose a nivel del:
a) Cordón umbilical
b) Borde inferior del bazo
c) Borde superior del hígado
d) Borde superior del pubis
29. Al comer unas papitas fritas con mayonesa, el vaciamiento gástrico disminuye por
efecto directo de la hormona:
a) CCK
b) Somatostatina
c) Secretina
d) Motilina
30. El crecimiento de un adenocarcinoma de pancreas compromete la pared gástrica por
contigüidad. ¿Qué parte del estomago se esperaría este comprometido?
a) Pared posterior del antro
b) Pared posterior del fondo
c) Pared posterior del píloro
d) Pared posterior del cardias

PARCIAL DE SISTEMA DIGESTIVO 2019 - 00


1. Varón de 30 años es traído a emergencia por agresión abdominal con arma de fuego (pistola) y es
sometido a laparotomía exploratoria, observándose isquemia del colon ascendente y parte del
colon trasverso ¿la lesión de cuál de las siguientes arterias explicaría esta isquemia?
(unidad 1, sesión 2, logro 6: (D​escribe la irrigación visceral: arterias de tronco celiaco, arteria
mesentérica superior e inferior​, topografía de superficie, órganos por cuadrante)
a) Celiaca
b) Colónica derecha
c) Mesentérica inferior
d) Mesentérica superior

2. Respecto a las sustancias gastrointestinales que regulan la secreción pancreática; marque la


afirmación correcta:
(unidad 1, sesión 3, logros 2 y 3: describir las hormonas gastrointestinales: estímulos y funciones)

a) La Secretina, es la hormona más importante para la secreción de bicarbonato por las células
acinares del páncreas
b) La acetilcolina es capaz de estimular la secreción enzimática y de bicarbonato del páncreas
c) La gastrina, es la hormona más importante para la secreción de enzimas pancreáticas
d) La colecistoquinina (CCK) estimula al páncreas solo para secreción enzimática

3. Ante una lesión del X par craneal, ¿cuál de los siguientes músculos mantiene conservada su
función?:
(unidad 2, sesión 08, logro 4: Paladar blando: componentes musculares)

a) Elevador del velo del paladar


b) Tensor del velo del paladar
c) Palatofaríngeo
d) Glosofaríngeo

4. Experimentalmente se utiliza atropina (anticolinérgico) para inhibir la secreción de gastrina, sin


embargo, la secreción de esta hormona se sigue dando ante estímulos vágales. Esta situación se
explica porque la atropina:
(unidad 1, sesión 3, logro 3: describir las hormonas gastrointestinales: estímulo y funciones de la gastrina
y colecistoquinina)

a. Bloquea parcialmente la bomba de protones en la célula G


b. Inhibe la acción de acetilcolina e histamina en la célula G
c. Solo inhibe la acción del péptido GRP en la célula G
d. No bloquea la acción del péptido GRP

5. Un varón de 50 años es sometido a extirpación del duodeno y parte proximal del yeyuno. Esta
situación ocasionaría la pérdida de las células ………. , productoras de que estimula la
secreción de
bicarbonato por el páncreas.
(unidad 1, sesión 3, logro 3: describir las hormonas gastrointestianles: estímulos y funciones de la
secretina y péptido insulinotrópico dependiente de glucosa)

a) “S” / secretina
b) Parietales / secretina
c) “I” / colecistoquinina
d) “S” / colecistoquinina

6. Recién nacido que presenta tumoración abdominal a nivel del cordón umbilical (fotografía).
¿cuál de las siguientes afirmaciones es correcta respecto a este defecto en el desarrollo
embriológico del intestino?:​ ​(unidad 1, sesión 5, logro 2: identificar las anomalías del desarrollo
del intestino medio)
a) Corresponde a una Gastrosquisis
b) Las vísceras se hallan cubiertas por piel
c) No está asociado a otras malformaciones
d) Se asocia a malformaciones
cardiacas y del tubo neural
7. Varón de 35 años acude a la emergencia por trauma
abdominal y se decida realizar una laparoscopía exploratoria.
El cirujano observa la disposición de los órganos abdominales
como se representa en el siguiente esquema. Esta disposición
de órganos se explica por la rotación (SMA=arteria
mesentérica superior)
(unidad 1, sesión 5, logro 3: identificar las anomalías del desarrollo del
intestino medio: defectos de rotación, estenosis y atresias)
a) antihoraria del intestino medio, en sólo 90°
b) incompleta del intestino medio (270°)
c) horaria del intestino medio
d) horaria del estómago

8. Se evalúa la expresión de la proteína Agrp en una persona con alteración del apetito; lo correcto
respecto a esta proteína es…..
(unidad 1, sesión 3, logro 4: ​Explica los mecanismos de control del apetito y saciedad ​)

a) Esta proteína es un potente anorexigénico


b) La mutación del gen que la codifica produce adelgazamiento
c) La sobre producción de la proteína lleva a obesidad por agonismo de receptores MC3 y MC4
d) La sobre producción de la proteína disminuye el apetito por antagonismo de receptores MC4

9. Juana cae de la bicicleta y se fractura la región anterior del hueso maxilar superior con
compromiso de la fosa incisiva. Al examen físico de la región esperaría encontrar alteración en la
sensibilidad de la encía …………………
(unidad 2, sesión 8, logro5: paladar: paladar duro y blando: irrigación e inervación)

a) bucal posterior
b) Lingual anterior
c) palatina anterior
d) palatina posterior

10. Recién nacido es atendido por el neonatólogo y luego entregado a su madre para dar de lactar; la
madre al dar de lactar observa coloración azulada de labios, acompañado de tos persistente,
dificultad respiratoria y distención abdominal. Se le intenta colocar una sonda nasogástrica pero
esta retorna a la cavidad oral en todos los intentos. ¿Cuál de las siguientes anomalías del
desarrollo es el más probable en este caso​? ​(unidad 1, sesión 4, logro 3: identificar las anomalías en
el desarrollo del esófago: atresia y/o fístula traqueo esofágica)

a) Estenosis esofágica proximal con Fístula traqueo esofágica distal


b) Atresia esofágica proximal con fístula traqueoesofágica distal
c) Atresia esofágica distal con fístula traqueoesofágica proximal
d) Fístula traqueoesofágica proximal y distal

11. ¿Cuál de los siguientes mecanismos ocurre durante la defecación?


(unidad 2, sesión 13, logro 6: motilidad del intestino grueso: contracciones segmentarias, movimientos
en masa, defecación y reflejo gastrocólico)

a) Contracción refleja del esfínter anal interno


b) En la posición de “cuclillas” el músculo puborectal se halla relajado
c) Relajación del esfínter anal externo por efectos del VIP y óxido nítrico
d) La materia fecal en el recto estimula la contracción del sigmoides por los nervios pudendos

12. La estructura número 4 (gráfico) corresponde a


……….… y está ………..
(unidad 2, sesión 9, logro 2: Partes de un diente.
Capas del diente: Esmalte: características y células
que lo producen)

a) el cemento / mineralizado en 90%


b) la dentina / formada por ameloblastos
c) el esmalte / formado por células
derivadas de la mesénquima
d) la dentina / formado por células
derivadas de la cresta neural

13. Un paciente luego de un accidente sufre lesión del piso de la boca, se constata daño del nervio
“cuerda del tímpano”, en este caso se esperaría encontrar disminución de la de la lengua
(unidad 2, sesión 10, logro 3: Irrigación e inervación de la lengua)

a) Motilidad en los dos tercios anteriores


b) Sensación del gusto en el tercio posterior
c) Sensación del gusto en los dos tercios anteriores
d) Sensibilidad al tacto en los dos tercios anteriores

14. ¿​Cuál de las siguientes afirmaciones es la correcta sobre la gastrina?


(unidad 1, sesión 3, logro 1: reconocer las características de las sustancias reguladoras
gastrointestinales: hormonas, sustancias paracrinas y neurocrinas)
a) Produce atrofia de la mucosa gástrica
b) Es producida por la célula G del cuerpo gástrico
c) Es estimulada por la distensión gástrica y el Ph bajo
d) Actúa en la célula diana mediante su receptor CCk tipo B
15. Al recibir un paciente con signos de hipovolemia y antecedente de trauma en abdomen por
accidente de tránsito, usted identifica radiológicamente: lesión de primera vértebra lumbar y
signos de lesión en páncreas; durante la cirugía se observó pobre irrigación de asas intestinales. El
vaso afectado es la arteria​ …………….
a) esplénica
b) hepática común
c) mesentérica inferior
d) mesentérica superior
16. Un paciente sufre de daño a nivel del cuello con lesión muscular en la región de la faringe. En el
examen físico se determina dificultad para la elevación de la faringe y para el cierre del itsmo de
las fauces. En este caso, probablemente esté afectado el músculo:
(unidad 2, sesión 11, logro 2: Músculos de la faringe: identificación, constrictores y longitudinales)
a) palatogloso
b) estilofarinfeo
c) palatofaringeo
d) constrictor inferior

17. Varón de 50 años a quien le realizan la curación de la segunda molar de la arcada superior
derecha. En un momento determinado, el paciente acusa de intenso dolor de la pieza dentaria
en tratamiento. La vía aferente del dolor viaja a través del nervio …………
(unidad 2, sesión 9, logro 6: Inervación de los dientes)
a) trigémino V2
b) trigémino V3
c) naso palatino
d) palatino menor

18. La distención gástrica por los alimentos produce incremento de secreción de HCl mediante la
producción de …………. que estimula a las células ……………. vía proteína ……….
(Unidad 1, sesión 3, logro 2: Describe las hormonas gastrointestinales: Estímulo y funciones de la
gastrina y colecistoquinina)
a) gastrina / parietal / Gq
b) gastrina / principal / Gs
c) acetilcolina / parietal /Gi
d) acetilcolina / principal / Gi

19. Un niño de tres años llega a emergencia con disfagia (dificultad para tragar), dolor retro esternal,
salivación y llanto. Se sospecha de ingesta de cuerpo extraño (moneda) en el esófago; al ser
evaluado se constata en una radiografía presencia de cuerpo extraño a nivel de C6 (6° vértebra
cervical). El cuerpo extraño estará suspendido a nivel del estrechamiento producido por………..
(unidad 2, sesión 11, logro4: Esófago, características anatómicas, relación con órganos vecinos y
estrecheces)
a) el cayado aórtico
b) el hiato esofágico
c) el músculo cricofaríngeo
d) el bronquio principal izquierdo

20. La triada portal (arteria hepática, vena portal y conducto biliar común) está contenida en el
ligamento
…….……… y derivan embriológicamente del ……
(Unidad 1, sesión 1, logro 4: Identifica el peritoneo, mesenterio, omento y ligamentos, retroperitoneo.)

a) hepato duodenal / mesenterio ventral


b) gastro esplénico / mesenterio dorsal
c) hepato gástrico / omento menor
d) falciforme / omento menor
21. En relación al movimiento de peristaltismo del
tubo digestivo: en la flecha negra del gráfico se
produce la liberación de ……………… a nivel del
músculo ………..
(unidad 2, sesión 7, logro 6: Control hormonal y tipos de
movimiento)
a) noradrenalina, sustancia P y neuropéptido “ Y” / circular
b) acetilcolina y sustancia P / longitudinal
c) óxido nítrico y PIV / longitudinal
d) óxido nítrico y PIV / circular
22. Un paciente refiere no percibir algunos sabores, al examen físico se constata alteración en la
percepción de sabores y del dolor en el tercio posterior de la lengua ¿Cuál de los siguientes
nervios estará alterada en su función?
(unidad 2, sesión 10, logro 5: Sabores, tipos y mecanismos moleculares para su detección)

a) Lingual (rama del V par)


b) Cuerda del tímpano (VII par)
c) Glosofaríngeo (IX par)
d) Hipogloso (XII par)

23. El gráfico detalla la estructura de la pared del


tubo digestivo intestinal ¿Cuál de las siguientes
asociaciones es correcta?
(unidad 2, sesión 7, logro 1: La pared y músculo
liso gastrointestinal )

a) “1” – peristaltismo
b) “2” – secreción enzimática
c) “3” – deriva del mesodermo
d) “4” – doble hoja de tejido graso

24. En el caso de un paciente con gastrinoma (tumor productor de gastrina), la presencia de úlceras
duodenales y erosión de la mucosa gástrica, se debe principalmente a…….
(unidad 1, sesión 3, logro 2: describir las hormonas gastrointestinales: estímulo y funciones de la gastrina
y colecistoquinina)
a) la acción directa de la gastrina sobre la célula principal
b) la sobre expresión de los receptores “G” en la célula parietal
c) el exceso de HCl por estímulo de receptores CCK-B en la célula parietal
d) el exceso de HCl por estímulo directo de receptores de acetilcolina en la célula parietal

25. El reflejo entero gástrico se caracteriza por:


(unidad 2, sesión 13, logro 6: Motilidad del intestino grueso: contracciones segmentarias, movimientos
en masa defecación y reflejo gastrocólico)

a) favorecer la motilidad gástrica gracias a la CCk


b) inhibir la motilidad gástrica y estimular la secreción ácida
c) movilizar grandes volúmenes desde el estómago al duodeno
d) originarse debido a la distensión duodenal y presencia del quimo ácido
26. Mauricio tiene dificultad para deprimir el paladar y elevar la parte posterior de la lengua. En este
caso estará afectado un músculo ………………., específicamente el músculo …………….
(Unidad 2, sesión 10, logro 2: Músculos de la lengua: clasificación, identificación y sus funciones)

a) intrínseco – longitudinal inferior


b) extrínseco – palatogloso
c) extrínseco – transverso
d) extrínseco – estilogloso

27. Una de las funciones del músculo señalado es:


(Unidad 2, sesión 8, logro 3: Describir el Piso de la boca:
estructuras blandas que la conforman)

a) deprimir la lengua
b) elevar el paladar blando
c) deprimir el hioides cuando la mandíbula está fija
d) deprimir la mandíbula cuando el hioides está fijo

28. Paciente varón de 30 años es evaluado por probable enfermedad de Chagas, cursa con
problemas de motilidad del colon; los estudios de biopsia determinan ausencia de células
ganglionares. Según el gráfico
¿cuál es la capa en la que se determina la ausencia de dichas células?
(unidad 1, sesión 2, logro 1: describir las generalidades de la estructura del tubo digestivo: esófago,
estómago intestino delgado y grueso)

a) Mucosa - 1
b) Muscular propia – 1
c) Muscular de la mucosa - 2
d) Muscular propia - 3
29. Paciente varón de 32 años, que acude a centro de salud por presentar de forma progresiva desde
hace 1 año dificultad para ingerir alimentos sólidos y luego líquidos; refiere regurgitaciones
alimentarias y marcada pérdida de peso (15 kilos). Radiografía baritada (sustancia de contraste​)
de esófago se muestra en la figura. El presente caso se explica por……………….
(Unidad 2, sesión 12, logro 4: Identificar y describir la función de los esfínteres esofágicos)

a) aumento de la peristalsis esofágica


b) relajación incompleta del esfínter pilórico
c) relajación incompleta del esfínter esofágico inferior
d) perdida de producción de PIV y óxido nítrico en el
esfínter esofágico superior

30. En condiciones normales, el ingreso de 600 ml de líquido es el estómago provoca un aumento de


presión intragástrica de unos 12 cm de H2O. Después de una vagotomía (corte del nervio vago)
es de esperar que el ingreso del mismo volumen de líquido ocasione de la presión
intragástrica.
(Unidad 2, sesión 13, logro 1: Describe la Motilidad gástrica: relajación receptiva)

a) la disminución
b) la no variación
c) un aumento mayor
d) un aumento similar o igual

Examen parcial 2019-2


1. La contracción del músculo permite la eliminación de gases (flatos) sin salida de material
fecal;
es el mismo músculo cuya relajación, sobretodo en cuclillas, permite el paso del contenido
fecal con menor esfuerzo durante la defecación.
a) Isquirectal
b) Puborrectal
c) Esfínter anal externo
d) Esfínter anal interno
2. Paciente mujer de 54 años se presenta con náuseas, vómitos, estreñimiento, y es diagnosticada
de abdomen agudo quirúrgico; en la cirugía encuentran un vólvulo de ciego. Esta anomalía
puede explicarse por:
a) Falta de rotación intestinal
b) Falta de fusión del mesenterio
c) Defecto en la formación de la cloaca
d) Falta de formación del omento mayor

3. Paciente mujer de 23 años con faringitis aguda, toma para el dolor una tableta de paracetamol
con un poco de agua. Durante la deglución, se relaja su esfínter esofágico inferior y el fondo del
estómago, mientras el bolo está aún en el esófago. ¿Qué sustancia provocara con mayor
probabilidad la relajación del esfínter esofágico inferior y el fondo del estómago en esta mujer?
a) Óxido nítrico
b) Sustancia P
c) Histamina
d) Motilina

4. Luego de tres horas dando exámenes, un alumno de medicina comienza a sentir hambre.
Esta situación es probable que sea mediada por la que
es sintetizada por el :
a) leptina / intestino
b) leptina / estómago
c) grelina​ /​ estómago
d) grelina / tejido adiposo

5. Varón de 72 años, con antecedente de diabetes mellitus tipo 2, que presenta enteropatía diabética
caracterizada por estreñimiento. Este problema puede estar asociado a:
a) deficiencia de óxido nítrico
b) aumento del reflejo gastrocólico
c) disminución de la secreción de colecistocinina (CCK)
d) aumento de la secreción del péptido intestinal vasoactivo (PIV)

6. Varón de 54 años con Diabetes Mellitus tipo 2, es diagnosticado de gastroparesia debido a que
presenta sensación de llenura precoz al comer, y reflujo gastroesofágico. Esta alteración en la
relajación receptiva y en el vaciamiento gástrico lo más probable es que se deba a una alteración
en​:
a) el nervio vago
b) el ganglio celíaco
c) plexo submucoso
d) nervio hipogástrico
7. Varón de 67 años con tos y disminución de peso asociado a tabaquismo pesado, presenta
actualmente disfagia progresiva a alimentos sólidos. Se considera la presencia de un carcinoma de
bronquio izquierdo y por esta razón le realizan una endoscopía esofágica para descartar la
posibilidad de una compresión esofágica por el tumor. Se espera revisar el esófago en la
estrechez, que está a nivel de la vértebra
a) Tercera estrechez -T6
b) Segunda estrechez - C6
c) Segunda estrechez - T4
d) Tercera estrechez -T10

8. Varón de 34 años con dolor abdominal agudo en flanco derecho que se irradia a fosa ilíaca
derecha, es operado y se encuentra un divertículo intestinal inflamado, ubicado a 93 cm de la
válvula ileocecal. El origen de este divertículo es una falla en la obliteración de:
a) Conducto vitelino
b) Alantoides
c) Cloaca
d) Conducto anorectal
e) Uraco
9. En un niño menor de dos años con divertículo intestinal, este divertículo tiene su origen en una falla
en la obliteración de:
a) Conductoanorectal
b) Conducto vitelino
c) Alantoides
d) Cloaca
e) Uraco

10. Mujer de 43 años sufre un grave accidente de tránsito y está hospitalizada en coma, es alimentada
por vía intravenosa durante varias semanas. Producto de este tipo de alimentación, se encuentra
en la endoscopía atrofia de la mucosa gastrointestinal. La causa más probable de esta atrofia son
los bajos niveles séricos de la hormona:
a) Colecistocinina
b) Secretina
c) Gastrina
d) PIV

11. Una mujer de 30 años llega al consultorio porque se queja de dificultades para deglutir, la cual se
agravan cada vez más. Se realiza un estudio manométrico para examinar la generación de presión
a lo largo del esófago. Esta prueba revela que las contracciones como respuesta a la deglución
están mal sincronizadas y que la presión en el esfínter esofágico inferior permanece elevada. El
diagnóstico más probable es producido por niveles bajos de
a) acalasia / sustancia P
b) acalasia / óxido nítrico
c) enfermedad por reflujo gastrointestinal / acetilcolina
d) enfermedad por reflujo gastrointestinal / óxido nítrico

12. Paciente de 2 años, llega a emergencia por haber ingerido una moneda con la que estaba
jugando. El lugar más probable donde puede haberse quedado suspendido este objeto es a nivel
del estrechamiento producido a nivel del:
a) músculo milohiodeo
b) músculo aritenoideo
c) músculo cricofaríngeo
d) constrictor superior de la faringe

13. En una apendicectomía, al realizar la incisión de McBurney en la fosa iliaca derecha, es


necesario cortar los siguientes músculos, de afuera hacia adentro:
a) Recto – Oblicuo externo – Transverso
b) Recto – Oblicuo externo – Oblicuo interno
c) Oblicuo externo – Oblicuo interno – Recto
d) Oblicuo externo – Oblicuo interno – Transverso

14. Un varón de 90 años que se encuentra postrado en cama, es referido del asilo para endoscopia por
dificultad para deglutir luego de tomar un medicamento para aliviar el dolor la noche anterior. La
endoscopía revela que la píldora se alojó en el esófago y causó una reacción inflamatoria. Lo más
probable es que esto haya sido por la producción de múltiples ondas​:
a) secundarias
b) primarias
c) lentas
d) segmentarias

15. Mujer de 23 años es diagnosticada de bulimia, al examen físico se observa ulceraciones en el


segundo y tercero dedo de la mano derecha. Esto se puede deber al uso continuo de estos dedos
para inducir el vómito, mediante la estimulación del par craneal:
a) V
b) IX
c) X
d) XI

16. Varón de 52 años se presenta por diarrea persistente de seis semanas de duración. En la
colonoscopia se observa un pólipo a nivel del íleon distal. El patólogo informa que se trata de un
tumor neuroendócrino, probablemente originado por las células enterocromafines del intestino.
La sustancia que más probablemente esté produciendo este tumor es:
a) Serotonina
b) Insulina
c) CCK
d) GIP

17. La fase oclusal de la masticación se realiza con la contracción de los músculos:


a) digástricos
b) masetero y temporal
c) orbicular y buccinador
d) pterigoideo lateral y digástrico

18. Al tomar su café en Starbucks, un estudiante de medicina sufre una quemadura de primer
grado en el tercio anterior de la superficie dorsal de la lengua. La información de dolor es
transmitida por el nervio:
a) cuerda del tímpano
b) glosofaríngeo
c) lingual
d) facial

19. Paciente es evaluado por faringitis aguda en consultorio externo. El médico de familia le solicita
que abra la boca y saque la lengua. Para realizar la acción de sacar la lengua, es necesario que se
contraiga el músculo:
a) Estilogloso
b) Geniogloso
c) Palatogloso
d) transverso de la lengua
20. Paciente con síndrome de Sjögren, presenta “boca seca”
(disminución de la producción de saliva) y caries dental,
asociada a la pérdida de la función de tampón de la saliva. Esta
desminerilización del diente puede comprometer a las
prolongaciones citoplasmáticas ubicadas en los tubos huecos de
la estructura señalada con la letra:
a) B
b) A
c) E
d) C

21. Mujer de 32 años acude a consulta por presentar disfagia de


progresión lenta, reflujo gastroesofágico y vómitos desde hace 3
meses de evolución progresiva. Se le realiza un estudio
radiológico con contraste en el que se observa estrechamiento
del esfínter esofágico inferior (imagen). Según sus
conocimientos, este paciente se beneficiaría con el uso de:
a) agonista beta adrenérgico
b) agonista alfa adrenérgico
c) análogo de óxido nítrico
d) análogo de Sustancia P
1. Un niño de 2 años es llevado a la consulta por diarrea persistente y edema de las
extremidades, además falta de crecimiento y desarrollo en relación a su edad. Los
análisis de sangre revelan que tiene concentración plasmática baja de proteínas
(hipoproteinemia). Durante la endoscopía duodenal, se coloca colecistocinina (CCK)
endovenosa y se recoge muestras del líquido duodenal; el resultado del líquido
confirma incapacidad para hidrolizar proteínas a un pH neutro, esta situación mejora
al añadir una pequeña cantidad de tripsina. El paciente probablemente esté
sufriendo la falta congénita de
​-Enterocinasa
2. Experimentalmente se incrementa la velocidad de la secreción salival con una
sustancia, el análisis de la composición de esta saliva obtenida se espera
encontrar…………..
-Disminución de concentración de potasio
3. Paciente varón de 46 años soltero, consulta por odinofagia y bajo de peso, tiene
antecedente de tuberculosis desde hace 3 meses y es fumador crónico (10
cigarrillos por día); al evaluar la cavidad oral se identifica lesión blanquecina en el
dorso de la lengua y paladar blando, las lesiones se desprenden con el baja lengua
dejando una base eritematosa. Esta lesión corresponde probablemente a
……………………….…..
-​ Candidiasis oral
4. Paciente mujer de 35 años acude a consulta por sensación de sequedad y lesiones
en cavidad oral. Al examen se observa atrofia de la mucosa, fisuras y úlceras; nota
además sequedad e irritación de la córnea y aumento del tamaño de las glándulas
parotídeas. Su diagnóstico más probable es artritis reumatoide; el hallazgo más
probable en una biopsia de glándula parótida es……..….
​-Gran infiltración de linfocitos y células plasmáticas
5. Un paciente con anemia acude con su médico quejándose de episodios frecuentes
de gastroenteritis. Un análisis de sangre revela anticuerpos circulantes dirigidos
contra células parietales gástricas. Su anemia es atribuible a la hiposecreción de
​-Factor intrínseco
6. Dos estudiantes deciden tomar un receso para comer una hamburguesa a la hora
del almuerzo. Antes de llegar a la cafetería, impulsos nerviosos provenientes del
complejo vagal dorsal iniciarán la secreción de ácido gástrico por la liberación de
…………………….. desde el sistema nervioso entérico.
-GRP
7. Un niño de cuatro años de edad es llevado a la consulta por cuadros diarreicos
frecuentes caracterizados por heces pálidas, voluminosas y fétidas, presenta bajo
peso y talla. Se mide la concentración de cloruro en el sudor y se encuentra que sus
valores son muy elevados. La alteración más importante a nivel de células ductales
del páncreas tiene relación directa con la conductancia de…………
​-Cloro
8. Una mujer de 50 años de edad que sufrió durante varios años resequedad de los
ojos debida a producción inadecuada de lágrimas es enviada con un
gastroenterólogo para evaluación de pirosis crónica. El examen endoscópico
revela erosiones y tejido cicatrizal en la parte distal del esófago justo por arriba
del esfínter esofágico inferior. Las lesiones pueden atribuirse a la disminución de
uno de los siguientes componentes salivales:
​-Bicarbonato
9. Se evalúa los valores séricos de las siguientes sustancias a un paciente con
enfermedad hepática terminal; en este paciente se espera encontrar la combinación
con la letra …………
​-disminuida, aumentada, disminuida
10. Una mujer de 35 años de edad HIV positiva, se presenta al médico con dolor
abdominal en cuadrante superior derecho e ictericia. La paciente refiere haber tenido
múltiples episodios de ictericia durante los últimos 10 años. Los exámenes para
determinar hepatitis viral, dieron positivos para Hepatitis B, siendo catalogado el caso
como hepatitis crónica con alteración funcional. En un examen de sangre ¿cuál de los
siguientes parámetros está disminuido?
​-Albúmina
11. En el reflejo peristáltico del intestino delgado, uno de los siguientes eventos sucede
en la porción oral del bolo alimenticio…………...
​-Acción de acetilcolina en el músculo circular
12. Experimentalmente se coloca una dosis alta de secretina en la luz intestinal
duodenal; como consecuencia de esto, en el jugo pancreático de la misma luz
intestinal se observa la disminución de la concentración de …..………..
​-Cl
13. Un varón de 58 años de edad con enfermedad de Crohn severo fue sometido a una
resección ileal. Después de la cirugía este paciente padecerá de esteatorrea, esto se
explica porque …..………..
- ​La micelas no pueden formarse
14. En un experimento se inserta un balón en el estómago de un voluntario, se infla poco
a poco mientras que se vigilan las presiones intraluminales. Aunque el volumen del
balón aumenta considerablemente, las presiones permanecen constantes. Esta
relación volumen-presión se explica por la liberación local de …………..
​-Óxido nítrico y péptido inhibidor vasoactivo
15. La toxina de Vibrio cholerae causa diarrea debido a…….
​-El Incremento de la secreción de cloro por las células de la cripta intestinal
16. ¿Cuál de las siguientes alternativas es una característica de la secreción exocrina
del páncreas?
​-Tiene una baja concentración de Cl- respecto al plasma
17. Una madre lleva a su hijo de dos años de edad a la sala de urgencias, estresada
porque el niño deglutió una moneda de 10 céntimos mientras la familia cenaba en un
restaurante. El médico observa mediante fluoroscopía que la moneda se halla en el
estómago y asegura a la madre que la moneda se eliminará con las heces. El médico
recomienda utilizar la respuesta fisiológica que permitirá la evacuación de la moneda
del estómago al intestino ………….…..
​-Son los movimientos de mezcla y trituración
-​ . Es provocada por el ayuno
18. Las estructuras en el hígado que permite que los productos metabólicos unidos a
proteínas tengan acceso a las membranas basolaterales de los hepatocitos, son…..
-Las fenestras sinusoidales
19. La composición de la bilis es modificada conforme fluye por los conductillos biliares.
Durante este tránsito se espera que aumente la concentración de…….
​-Monómeros de ácido biliar
-Ig A
20. Se mide experimentalmente el contenido gástrico de dos personas. La persona “A”
tiene alto contenido de grasa y la persona “B” tiene un contenido hipertónico ¿Cuál
de las siguientes es correcto respecto al vaciamiento gástrico?
- Hay ralentización del vaciado gástrico en ambos casos
21. El examen endoscópico de un paciente con hipertensión portal grave revela venas
tortuosas que sobresalen hacia la luz del esófago. El paciente recibe tratamiento
quirúrgico mediante la colocación de una derivación que conecta la vena porta a la
vena cava. Después de la operación el riesgo de encefalopatía y el
riesgo de sangrado de várices ……………..
​-Aumentará/disminuirá
22. Un paciente varón de 18 años de edad acude al médico para sus exámenes de
rutina. Sus resultados de laboratorio muestran un valor de bilirrubina sérica de 4
mg/dl y una bilirrubina directa de 0,3 mg/dl. Las pruebas de función hepática son
normales. La alteración que explica mejor este caso es por la deficiencia de
………………..
-Glucuronil transferasa
23. Un hombre de 57 años de edad es llevado a urgencias con hematemesis masiva
rojo brillante, a su llegada se halla inconciente con PA: 80/40 mm Hg y FC: 124
lat/min. Luce ictérico con presencia de “arañas vasculares en el tórax anterior y
extremidades”, abdomen distendido con signo de oleada positiva. Se encuentra
esplenomegalia y pérdida de la masa muscular en extremidades. La anastomosis
vascular responsable del sangrado en este paciente es
​-Vena gástrica izquierda y vena ácigos
24. Un estudiante de medicina está comiendo un plato de comida a base de
champiñones, espárrago y salsa de soya. El sabor umami contenido en todos estos
alimentos actúa a nivel de los botones gustativos estimulando ………………..
​-Un receptor acoplado a proteína G
25. Un hombre de 22 años de edad se presenta al médico con una historia de 1 año de
evolución caracterizado por dolor recurrente en fosa iliaca derecha y diarrea.
Manifiesta además pérdida de peso de 8 kg durante este periodo. La colonoscopía
revela múltiples lesiones en el ileon terminal y colon. La biopsia de estas lesiones
revela engrosamiento, inflamación y ulceración de la mucosa. El diagnóstico más
probable en este caso es…….
​-Enfermedad de Crohn
26. Varón de 61 años que consulta por dolor retro esternal intenso desde hace 6 horas y
después de vómitos intensos y repetidos; al examen se observa disnea, cianosis,
hipotensión y signos clínicos de shock. La radiografía simple de tórax muestra
neumomediastino. El líquido en el espacio pleural aspirado tiene alta concentración
de amilasa. ¿Cuál de las siguientes alternativas puede explicar este cuadro clínico?
-​Rotura espontánea de esófago
27. La secreción del ácido en la célula parietal gástrica se lleva a cabo por una ATPasa
especifica que intercambia hidrogeniones (H+) del citosol por…..
​-K +
28. En condiciones normales el ingreso de 600 ml de líquido es el estómago provoca un
aumento de presión intragástrica de unos 12 cm de H2O. Después de una
vagotomía (corte del nervio vago) es de esperar que el ingreso del mismo volumen
de líquido provoque lo siguiente: …………………………………
​-Un aumento mayor de la presión
29. Una paciente de 30 años de edad es sometida a una cirugía en oído medio derecho
por un problema de otoesclerosis. Luego de la cirugía refiere alteración en la
percepción de sabores. Al evaluar el caso usted esperaría encontrar……….
​-Alteración en la sensación del gusto en los dos tercios anteriores de la
l​ engua
​-Sensación del dolor, tacto y temperatura conservada en toda la lengua
30. ¿Cuál de las siguientes alternativas es correcta?
​-Las sales biliares desconjugadas son absorbidas preferentemente en el colon
31. En un paciente de 45 años de edad con colestasis biliar, se encuentra una elevación
de los niveles sanguíneos de fosfatasa alcalina hasta 3 veces la cifra normal. ¿Cuál
de las siguientes alternativas estará también elevada como evidencia del daño de la
vía biliar?
​-Gamma glutamil transpeptidasa
32. Revisando la angiografía de un hombre de 70 años en estudio por aneurisma de
aorta abdominal el radiólogo informa de la presencia de una oclusión completa de la
arteria mesentérica inferior. El paciente se encuentra completamente asintomático.
¿Cuál de las siguientes arterias se anastomosa a la sistema arterial de la
mesentérica inferior?
​-Cólica media
33. Lactante de 3 meses de vida es atendido por presentar diarrea, se administra una
solución de glucosa y electrólitos por vía oral. La proteína de membrana apical que
explica la capacidad de esta solución para proporcionar aporte de glucosa e
hidratación es ………..
​-SGLT-1
34. Paciente ha sufrido herida de bala en el abdomen, se le ha tenido que extirpar el
segmento medio y distal del ileon. En este caso la síntesis hepática de sales biliares
estará …..…..
​-Incrementada por estímulo de la enzima colesterol 7 alfa hidroxilasa
35. Un varón de 75 años ingresa al consultorio por presentar ictericia marcada de piel y
las escleras. El estudio del paciente mostró que presentaba un tumor que obstruía la
totalidad del conducto hepático común. ¿Cuál de las siguientes estructuras se
encontrará dilatada en este paciente?
​-Conductos de Hering
36. En un paciente con insuficiencia renal crónica, el déficit en la absorción de calcio a
nivel del enterocito se debe a lo siguiente:
​-No se convierte la 25 hidroxicolecalciferol a 1,25 dihidroxicolecalciferol
37. Varón de 30 años es traído a emergencia por agresión abdominal con arma de fuego
(pistola) y es sometido a laparotomía exploratoria, observándose isquemia del colon
ascendente y parte del colon trasverso ¿la lesión de cuál de las siguientes arterias
explicaría esta isquemia?
d. Mesentérica superior
38. Respecto a las sustancias gastrointestinales que regulan la secreción pancreática;
marque la afirmación correcta:
b. La acetilcolina es capaz de estimular la secreción enzimática y de bicarbonato del
páncreas
39. Ante una lesión del X par craneal, ¿cuál de los siguientes músculos mantiene
conservada su función?:
b. Tensor del velo del paladar
40. Experimentalmente se utiliza atropina (anticolinérgico) para inhibir la secreción de
gastrina, sin embargo la secreción de esta hormona se sigue dando ante estímulos
vagales. Esta situación se explica porque la atropina:
d. No bloquea la acción del péptido GRP
41. Un varón de 50 años es sometido a extirpación del duodeno y parte proximal del
yeyuno. Esta situación ocasionaría la pérdida de las células ​………​.. , productoras de
………………… ​que estimula la secreción de bicarbonato por el páncreas.
“S” / secretina
42. Se evalúa la expresión de la proteína Agrp en una persona con alteración del apetito; lo
correcto respecto a esta proteína es​…​..
La mutación del gen que la codifica produce adelgazamiento
43. Juana cae de la bicicleta y se fractura la región anterior del hueso maxilar superior con
compromiso de la fosa incisiva. Al examen físico de la región esperaría encontrar
alteración en la sensibilidad de la encía ​…………………
palatina anterior
44. Recién nacido es atendido por el neonatólogo y luego entregado a su madre para dar
de lactar; la madre al dar de lactar observa coloración azulada de labios, acompañado
de tos persistente, dificultad respiratoria y distención abdominal. Se le intenta colocar
una sonda nasogástrica pero esta retorna a la cavidad oral en todos los intentos.
¿Cuál de las siguientes anomalías del desarrollo es el más probable en este caso?
b. Atresia esofágica proximal con fístula traqueoesofágica distal l
45. ¿Cuál de los siguientes mecanismos ocurre durante la defecación?
En la posición de “cuclillas” el músculo puborrectal se halla relajado
46. Un paciente luego de un accidente sufre lesión del piso de la boca, se constata daño
del nervio “cuerda del tímpano”, en este caso se esperaría encontrar disminución de
la​…………………………​.​… ​de la lengua
Sensación del gusto en los dos tercios anteriores
47. ¿Cuál de las siguientes afirmaciones es la correcta sobre la gastrina?
Actúa en la célula diana mediante su receptor CCk tipo B
48. Al recibir un paciente con signos de hipovolemia y antecedente de trauma en
abdomen por accidente de tránsito, usted identifica radiológicamente: lesión de
primera vértebra lumbar y signos de lesión en páncreas; durante la cirugía se observó
pobre irrigación de asas intestinales. El vaso afectado es la arteria ​……​..
c. mesentérica inferior
49. Un paciente sufre de daño a nivel del cuello con lesión muscular en la región de la
faringe. En el examen físico se determina dificultad para la elevación de la faringe y
para el cierre del itsmo de las fauces. En este caso, probablemente esté afectado el
músculo:
c. palatofaringeo
50. Varón de 50 años a quien le realizan la curación de la segunda molar de la arcada
superior derecha. En un momento determinado, el paciente acusa de intenso
dolor de la pieza dentaria en tratamiento. La vía aferente del dolor viaja a través
del nervio ​…………
a. trigémino V2
51. La distención gástrica por los alimentos produce incremento de secreción de HCl
mediante la producción de ​…………​.. que estimula a las célulasvía proteína
………​..
a. gastrina / parietal / Gq
52. Un niño de tres años llega a emergencia con disfagia (dificultad para tragar), dolor
retro esternal, salivación y llanto. Se sospecha de ingesta de cuerpo extraño
(moneda) en el esófago; al ser evaluado se constata en una radiografía presencia de
cuerpo extraño a nivel de C6 (6° vértebra cervical). El cuerpo extraño estará
suspendido a nivel del estrechamiento producido por​………​..
c. el músculo cricofaríngeo
53. La triada portal (arteria hepática, vena portal y conducto biliar común) está contenida
en el ligamento ​……​.​……… ​y derivan embriológicamente del ​……
a. hepato duodenal / mesenterio ventral
54. Un paciente refiere no percibir algunos sabores, al examen físico se constata
alteración en la percepción de sabores y del dolor en el tercio posterior de la lengua
¿Cuál de los siguientes nervios estará alterada en su función?
c. Glosofaríngeo (IX par)
55. En el caso de un paciente con gastrinoma (tumor productor de gastrina), la presencia
de úlceras duodenales y erosión de la mucosa gástrica, se debe principalmente
a​……​.
c. el exceso de HCl por estímulo de receptores CCK-B en la célula parietal
56. El reflejo entero gástrico se caracterizan por:
d. originarse debido a la distensión duodenal y presencia del quimo ácido
57. Mauricio tiene dificultad para deprimir el paladar y elevar la parte posterior de la lengua.
En este caso estará afectado un músculo, específicamente el músculo
……………​.
b. extrínseco – palatogloso
58. En condiciones normales, el ingreso de 600 ml de líquido es el estómago provoca un
aumento de presión intragástrica de unos 12 cm de H2O. Después de una vagotomía
(corte del nervio vago) es de esperar que el ingreso del mismo volumen de líquido
ocasione
………………………………… ​de la presión
intragástrica.
c. un aumento mayor
59. La explicación fisiológica de presentar somnolencia de 30 minutos a 1 hora
después de ingerir alimentos, se explica por: a. Aumento del cloro intraluminal
e. Aumento de la alcalinidad sanguínea
60.Se presenta un paciente, el cual presenta un antecedente de tuberculosis intestinal,
por lo cual, se le resecó 80 cm de íleon distal. Desde el punto de vista fisiológico, el
paciente puede presentar una de las siguientes alteraciones: a. Disminución de la
secreción de Vitamina B12

e. Disminución de la absorción de ácido glicocólico


61. Un paciente es sometido experimentalmente a un fármaco que modifica el flujo salival,
obteniéndose un volumen de saliva de 288 ml en 6 horas. En este caso las concentraciones
de electrolitos y bicarbonato en la saliva obtenida varían de la siguiente manera: a. ​↑
Na+,
↓ ​ K+, ​↑ ​Cl-, ​↑ ​HCO3-

1. b. ​↓ ​
Na+, ​↓ ​Cl-, ​↑ ​K+, ​↓ ​HCO3-

62. Uno de los siguientes elementos debería hallarse con más probabilidad en el esófago
de un paciente que sufre de reflujo gastro esofágico​…
a. Pepsina

63. Un paciente de 40 años cursa con anemia de 8g/dl, aqueja además de astenia y sensación
de hormigueo bilateral en los miembros inferiores, al examen se halla alteración de la
sensibilidad a la vibración y camina con ampliación de la base de sustentación. Uno de los
siguientes procedimientos sería de ayuda para el diagnóstico de este paciente:
a. Tomografía cerebral
b. Biopsia de la mucosa gástrica

64.Paciente de 60 años ingresa por caída hace 1 hora y pequeño hematoma en cuero
cabelludo, al examen físico ampliado se observa ictericia de piel y mucosas generalizada,
abdomen blando, se palpa estructura quística no dolorosa en hipocondrio derecho que
corresponde a vesícula biliar (signo de Courvoisier), en los exámenes de laboratorio se halla
niveles bajos en la formación de estercobilinógeno y urobilinógeno en heces, incremento de
la bilirrubina conjugada en la orina, elevación de fosfatasa alcalina y gamma glutamil
transpeptidasa séricas. El presente cuadro puede ser explicado por: a. Reabsorción de
hematoma
c.Carcinoma de la cabeza de páncreas
65. Un recién nacido presenta vómitos biliosos poco tiempo después de cada alimento. Al
preguntar a la madre sobre antecedentes, ella recuerda que tuvo polihidramnios durante la
gestación, pero un análisis de cariotipo fue normal. Una de las siguientes es la causa más
probable de estos hallazgos en el recién nacido: a. Enfermedad de Hirschprung
e. Malrotación de la yema pancreática ventral
66.En un estudio de la secreción de hormonas gastrointestinales, sus concentraciones en la
vena porta se midieron durante perfusión luminal del intestino delgado con soluciones de
diversas magnitudes de pH. ¿Qué hormona aumentará en el plasma de la vena porta durante
perfusión a través del intestino con una solución de pH 3?
a. CCK
e. secretina
67.​Paciente de 30 años que ingresa a causa de un traumatismo abdominal cerrado. En la
exploración se aprecia discreta palidez de piel y mucosas, auscultación pulmonar normal,
taquicardia de 120 /min. Discreta distensión abdominal y matidez en flancos; el hematocrito,
que era prácticamente normal al ingreso, disminuye a 30% a las tres horas. En la Rx de tórax
se objetiva fractura de las costillas 10-11 izquierdas. La causa más probable de la
anemización en este paciente es: a. traumatismo renal con hemorragia retroperitoneal.
c. rotura de bazo con hemoperitoneo.
68. Revisando la angiotomografía de un hombre de 70 años en estudio por aneurisma de
aorta abdominal, el radiólogo le informa de la presencia de una oclusión completa de la
arteria mesentérica inferior. El paciente se encuentra completamente asintomático. La
oclusión de la arteria mesentérica inferior cursa de manera asintomática en muchas
ocasiones ya que el territorio que irriga puede recibir flujo proveniente de la arteria:
a. cólica derecha
e. cólica media

69. En las patologías de esófago es importante conocer bien la anatomía esofágica. ¿Cuál de
las siguientes afirmaciones es correcta? a. El esófago tiene capa mucosa, muscular y serosa

c. El esófago torácico pasa por detrás del cayado aórtico

70. A pesar de que pueda haber variaciones anatómicas, lo habitual es que el ciego sea
irrigado por una rama arterial que proviene de unas de las siguientes arterias: a. Iliaca
derecha

d. Mesentérica superior

71. Ante un paciente con una cirugía abdominal urgente, el informe operatorio señala que
se ha realizado una resección de todo el duodeno y del tercio proximal del yeyuno
manteniendo íntegros el estómago y todo el íleon, así como los dos tercios distales del
yeyuno. En el seguimiento nutricional del paciente ¿Qué vitamina o mineral presentará con
menor probabilidad una disminución de su absorción?
a. Cianocobalamina

72. ¿Cuál de las siguientes sustancias forma parte de la secreción biliar? a. Tripsina
Lecitina

73.¿De qué musculo forma parte el ligamento inguinal?


-Oblicuo externo del abdomen
74.¿Cuál de las siguientes enzimas está localizada en el borde en cepillo y juega un rol en la
digestión de proteínas?
e. ​Carboxipeptidasa A.
75. Una de los siguientes sustancias, ​NO ​sirve como un buen agente emulsificante:
a. Colesterol
76. ​La sustancia que estimula el crecimiento de la mucosa gástrica es:

a. Secretina

d. Gastrina

77.¿Cuál de las siguientes alternativas es una función de la colecistokinina?

a. Relajación de la vesícula para la salida de bilis

d. Secreción de enzimas pancreáticas

78.​Con respecto a la anatomía del tronco celiaco, señale lo correcto a. El tronco celiaco se
origina de la cara posterior de la aorta abdominal
d. La hepática común que es una de sus ramas, participa en la irrigación del
estómago.
79. ​Con respecto a la anatomía del duodeno, marque la respuesta correcta: a. Tiene una
distribución en forma de “C”, que rodea la cola del páncreas
b. La 3ra porción duodenal está contenida en la pinza vascular aortomesentérica
80. ​En el íleon se absorbe aproximadamente el 95% dea través de la circulación
enterohepática.
a. agua
c. sales biliares
81. La estimula el mecanismo paracrino de la secreción de ácido clorhídrico.
a. histamina
82.​En la digestión de proteinas,es el principal estímulo para convertir el
pepsinógeno en pepsina. a. la gastrina
b. el pH ácido
83. ​Con respecto a la somatostatina, marque lo correcto:
a. Es secretada por las células S del intestino
Interviene en la fase intestinal de la secreción gástrica
84. ​En pecten anal, es una estructura comprendida entre: a. la línea pectínea y los senos
anales
d. la línea anocutánea y la línea pectínea

85.¿Cuál de las siguientes alternativas es una proenzima pancreática? a. Tripsina

1. b. Elastasa
2. c. Quimotripsinógeno
3. d. Amilasa
4. e. Procarboxipeptidasa C
86. ​En este paciente, el bloqueo farmacológico de los receptores H2 en la mucosa gástrica:
​a) No tiene efecto sobre la secreción de ácido inducida por el vago
b) Evita la activación de adenilciclasa por gastrina
​c) Inhibe la secreción de ácido inducida por gastrina y mediada por el vago
d) Causa un aumento en el transporte de potasio por las células parietales gástricas
Se validó la A :)
87. Si se considerara una gastrectomía total para curar la gastritis del paciente, cuál de
las siguientes sustancias ya no se produciría:
a) Gastrina
b) Quimiotripsina
c) Amilasa
​d) Pepsinógeno

88. Un paciente hipertenso está tomando un medicamento bloqueador de receptores alfa


1 adrenérgicos (prazosina) y como efecto secundario se queja de:
d) Lo escaso que es el medicamento
e) No tiene problemas en la salivación
​c) Hiposalivación
d) Hipersalivación

89. Con respecto a las lesiones y enfermedades de la boca, marque lo correcto:


f) La leucoplasia se desprende al roce
​b) la eritroplasia puede degenerar en adenocarcinoma
c) El muguet oral es una enfermedad bacteriana en inmunodeprimidos
​d) la eritroplasia debe ser biopsiada
​NOTA: fue validada la opción B ya que no es motivo del curso que sepamos el puto cáncer.

90. En este paciente, se puede asumir que la pancreatitis ha sido ocasionada por
una disminución en el efecto de:
g) Amilasa
h) Lipasa
​c) Inhibidor de la tripsina
d) Entercinasa
91.Un efecto secundario en el estómago por la acción de la secretina es:
i) Disminución en la liberación de pepsinógeno
​b) Menor actividad de la pepsina
c) Mayor paso de bicarbonato a sangre periférica
d) Aumento en la producción de factor intrínseco
92. Dentro de los factores protectores de la mucosa gástrica se pueden mencionar
múltiples protagonistas. Uno de ellos es:
j) CCK
k) Gastrina
c​ ) Receptor muscarínico
d) Pepsina
93. La saliva puede tener una variedad de electrolitos en su composición. Entre ellos el
cloro, respecto al cual se puede afirmar:
l) Su mayor concentración se consigue con el flujo bajo
​ ) Su concentración no llega a ser tan alta como en el plasma
b
c) Con flujo alto su concentración es mayor que la del plasma
d) Su menor concentración se alcanza con flujo alto
94. En el síndrome de boca seca o síndrome de Sjogren, una de las
complicaciones asociadas es:
​a) caries
b) Disminución de la acidez gástrica
c) Aumento en de la producción de saliva
d) Aumento del pH bucal
95. Estimula la producción de saliva:
​ ) Vasodilatación periglandular
a
b) Atropina
c) Fatiga o cansancio
d) Expresión de miedo
96. El omeprazol actúa sobre la membrana​ ​de la célula
m) Basolateral/principal
n) apical/principal
o) Basolateral/parietal
​d) apical/parietal
97. Para protegerse del entorno ácido, el Helicobacter pylori se autogenera un entorno de
pH menos ácido alrededor suyo, gracias a una enzima que alcaliniza su entorno local
mediante la conversión de:
​a) urea en NH3
b) H2O y CO2 en ácido carbónico
c) NH3 en urea
d) H2CO3 en bicarbonato
98. La anemia perniciosa destruye las células:
p) mucosas del cuello
b) oxínticas
c) principales
d) mucosas superficiales
99. La célula mucosa del cuello gástrico produce:
a) Moco
b) ácido clorhídrico
c) pepsinógeno
d) Factor intrínseco
100.El aumento en la acidez del estómago producido principalmente por la infección de
Helicobacter pylori se debe a la disminución de:
a) Somatostatina
b) Bicarbonato por las glándulas de Brunner
c) Secretina
d) Colecistoquinina
101. De las siguientes sustancias secretadas por los órganos de este paciente, la más
alcalina es la secreción:
a) pancreática
b) Esofágica
c) Yeyunal
d) Salival
102.En cuanto a la gastritis de este paciente, se encontró que era producida por la bacteria
Helicobacter pylori. Esta bacteria sobrevive en el medio ácido del estómago gracias a:
a) ácido clorhídrico
b) Toxina CagA
c) Ureasa
d) Jugo pancreático
103.La lengua está recubierta por epitelio:
c) pseudoestratificado columnar no queratinizado
b) plano estratificado no queratinizado
c) pseudoestratificado columnar ciliado
d) plano estratificado queratinizado
104.El esfínter anal interno tiene musculatura …….. y tiene control ……..
d) lisa / voluntario
b) lisa / involuntario
c) esquelética / simpático
d) esquelética / parasimpático
105.La arteria Aorta proporciona la irrigación al tubo digestivo ¿cuál de las siguientes
arterias proporciona la irrigación al ángulo cólico derecho?
a) mesentérica superior
b) mesentérica inferior
c) frénica inferior
d) tronco celiaco
106.Paciente de 26 años que le cuenta en su historia clínica que cada vez que almuerza a
los 20 min tiene deseo de defecar, le comenta que su hijo de 1 mes le pasa lo mismo pero
más intenso. Esto se explica por el reflejo …….., el cual está …… en el paciente
e) colicoileal / normal
f) colicoileal / alterado
c) gastrocólico / normal
d) gastrocólico / alterado
107.La región del estómago que se comunica con el duodeno es la
a) pilórica
b) cardias
c) cuerpo
d) fórnix
108.Acude a consulta un px que fue diagnosticado de úlcera péptica 3 días antes. Luego de
múltiples pruebas diagnósticas se concluye que el paciente presenta un tumor secretor de
gastrina, ¿cual de las siguientes situaciones estará incrementada?
g) distensión gástrica
h) inhibición del vaciado gástrico
c) secrecion de acido clorhidrico
d) inhibición de la secreción de pepsinógeno
109.En el sistema digestivo, el control del apetito está dado por un complejo sistema de
sustancias y órganos integradores, los cuales regulan la ingesta de alimentos. La …… es una
sustancia orexígena y es sintetizada por el ……
i) leptina / estómago
j) felina / intestino
k) leptina / estómago
d) grelina / estómago
110. Con respecto a la actividad eléctrica del sistema digestivo, marque la alternativa
correcta
l) corresponden a potenciales de acción que están presentes de forma continua y le
dan capacidad de peristalsis autónoma al sistema digestivo
b) la frecuencia de las ondas lentas NO se ve influenciada por la actividad neural ni las
hormonas gastrointestinales
c) en el estómago las ondas lentas se dan en una frecuencia de 6 x min
d) las ondas lentas son cambios lentos y ondulantes del potencial en reposo
e) la frecuencia de las ondas lentas va de 6 a 12 ondas por minuto
111. Ante una lesión del IX pc, el músculo …… se altera en su función
m) palatogloso
b) estilofaríngeo
c) palatofaríngeo
d) constrictor superior
112. Un varón de 50 años es sometido a extirpación del duodeno y parte proximal del
yeyuno. La pérdida de estímulo hormonal en el páncreas para la secreción enzimática se
explica por la pérdida de células
n) parietales, productoras de factor intrínseco
o) K productoras de factor intrínseco
p) M productora de CCK
d) I productora de CCK
113.Marque la respuesta correcta:
A. El bronquio derecho constituye una de las estrecheces del esófago
B. Todos los órganos del sistema digestivo tienen capa serosa
C. La pared gástrica en el fondo es más delgada que en el cuerpo y antro
D. El esfínter de Oddi rodea a la papila menor duodenal
114. Marque la respuesta correcta en relación a la gastrina:
A. Al distenderse el estómago, se inhibe su producción.
B. Se estimula por la liberación de noradrenalina
C. Las células G son las productoras y se encuentran principalmente en el antro
gástrico
D. Las células G se encuentran principalmente en el fondo gástrico
115. Para poder morder una manzana, es necesario usar el siguiente músculo:
A. Milohiodeo
B. Tensor del paladar
C. Orbicular de los labios
D. Buccinador
116. Sustancia que inhibe la secreción y la motilidad del estómago prolongando el tiempo de
digestión:
A. Enteroglucagon.
B. Polipéptido pancreático
C. Péptido 1 similar al glucagón (GLP-1).
D. Péptido insulinotrópico dependiente de la glucosa (GIP).
117. El nacimiento de la arteria mesentérica superior se puede encontrar en cuál de los
cuadrantes abdominales:
A. Hipocondrio derecho
B. Hipogastrio
C. Epigastrio
D. Mesogastrio
​ ntre las múltiples causas de la Enfermedad por Reflujo Gastroesofágico, se puede
118. E
considerar también a una alteración en las​ ​del esfínter esofágico inferior:
A. Ondas secundarias
​ . Contracciones tónicas
B
C. Ondas lentas
D. Glándulas subesofágicas
119.Producto de la alimentación, se producen diversas sustancias peptídicas, cininas y
bradicininas, las cuales permiten que:
A. Se produzca neovascularización en los territorios de las arterias abdominales
B. La acción de la lipasa pancreática se vea incrementada
C. El consumo de O2 del intestino aumente ligeramente
​D. El flujo sanguíneo intestinal aumente hasta 8 veces
120.El dolor periumbilical o epigástrico en el inicio de una apendicitis aguda se debe a:
A. Estímulo del nervio vago.
B. Íleo secundario.
C. Irritación del peritoneo parietal.
​D. Estímulo del sistema simpático.
121.El aumento en la actividad motora de la pared gástrica genera un aumento en los
niveles locales de qué sustancia en la microvasculatura:
​A.Adenosina
B. Colecistoquinina CCK
C. Endotelina
D. Gastrina
122.¿Cuál de los siguientes péptidos inhibe el vaciamiento gástrico?
​A. Colecist​oquinina
B. Péptido inhibidor gástrico
C. Motilina
D. Gastrina
123. Los músculos de la masticación que producen la retropulsión de la mandíbula son:
​A. temporales [mas seguro]
​B. maseteros
C. milohioideos
D. pterigoideos
124. En relación a la fisiología gástrica, marque lo correcto:
​A. la cimetidina actúa en la región basolateral de la célula parietal
B. la marea alcalina se debe al paso de bicarbonato través de la membrana apical de
la célula principal
C. el cloro difunde hacia el exterior por la la región basolateral de la célula parietal
D. la salida de hidrogeniones a la luz es por difusión facilitada
125. Durante el sueño, la concentración de bicarbonato en la saliva:
A. Se eleva a niveles mayores que los del plasma
B. Aumenta
C. No tiene efecto
​D. Disminuye
126. La secreción de saliva es importante en la fisiología digestiva. Su concentración
de potasio llega a ser menor que la del plasma cuando su secreción tiene un flujo:
A. Intermedio
​B. Nunca
C. Bajo
D. Alto
127. Respecto a las glándulas salivales, marque lo incorrecto:
A. la glándula parótida produce secreción serosa
B. la glándula sublingual drena a través de conducto de Wharton
C. La glándula parótida drena a través del conducto de Stenon
​D. la glándula sublingual tiene forma de garfio
128. ​Con respecto a la saliva, marque la respuesta correcta:
D. será hipertónica cuando el flujo es bajo
E. a mayor flujo, menor concentración de Na
F. a mayor flujo, menor concentración de cloro
​D. el sistema simpático estimula su secreción
CI 3
129. En relación a la circulación hepática, marque lo correcto:
​a) Los sinusoides hepáticos transportan sangre mixta
b) La vena porta proporciona el 50% de sangre al hígado
c) La vena porta se forma a partir de la vena esplénica y la mesentérica inferior
d) La arteria hepática deriva de la mesentérica superior
130. Dentro de las funciones de las células de Ito, marque lo incorrecto:
a) Sintetizan colágeno
b) Almacenan vitamina A
c) Se les llama células estrelladas
​d) Pueden fagocitar patógenos y actúan como presentadoras de antígeno
131.Paciente con tumor neuroendocrino productor de secretina, debido a lo cual se puede
esperar que su secreción pancreática, comparada con la de una persona sana de bajo flujo,
tenga una concentración de:
a) Sodio aumentada
b) Igual
​c) Bicarbonato aumentada
d) Potasio disminuida
132.El GALT se localiza en:
​a) Lámina propia
b) submucosa
c) borde en cepillo
d) superficie de criptas de Lieberkühn
133. En relación a la histología hepática, marque lo correcto:
a) la zona 1 se afecta rápidamente en estados de hipovolemia y shock
b) La zona 1 se encuentra cercana a la vena central lobulillar
​c) La zona 3 se encuentra más cerca a la vena central lobulillar
d) La zona 3 se encuentra más cerca al eje menor del acino hepático
134.El acino pancreático difere con el de las glándulas salivales en:
​ ) Contiene células centroacinares
a
b) No produce secreción serosa
c) El páncreas produce principalmente secreción mucosa
d) No tienen diferencias
135.Durante la digestión de las grasa, para que la lipasa actúe adecuadamente se requiere
que el pH aumento en la luz intestinal, lo cual es logrado, entre otros, por la secreción de las
células:
a) Del conducto interlobulillar
​ ) Centroacinares
b
c) Acinares
d) Alfa
136.La secreción de la colecistoquinina (CCK) se produce en la fase:
​ ) intestinal
a
b) En las 3 por igual
c) gástrica
d) Cefálica
137. ¿Por cuál de las siguientes células es secretada principalmente la pro
enzima procarboxipeptidasa?
​a) Acinares del páncreas
b) Epiteliales del duodeno
c) Ductales del páncreas
d) Centro Acinares del páncreas
138.Una mujer de 43 años dolor en hipocondrio derecho e icterica. En la ecografía se
evidencia cálculos biliares. Estos cálculos lo más probable es que se encuentren localizados
en:
​a) conducto colédoco
b) Conducto cístico
c) Vesícula biliar
d) Conducto pancreático secundario
139. ​Una mujer de 49 años ingresa en el hospital con dolor en epigástrico que migra hacia el lado
derecho y atrás hacia la escápula, sin ictericia. La ecografía muestra un gran cálculo biliar. ¿En cuál
de las siguientes estructuras es más probable que se localice el cálculo biliar?
→ bolsa de Hartmann

140. Una mujer de 38 años ingresa en el hospital con signos de colecistitis y cálculos biliares. Durante
la colangiografía, se inserta el catéter en la vesícula biliar con mucha dificultad. ¿Cuál de las
estructuras interfiere con más probabilidad con el paso del catéter por el conducto cístico?
→ válvula espiral (de Heister)

141. La internalización de proteínas o fragmentos de proteínas hacia el intracelular, es característico


de:
→ células M
→ se validó enterocitos

ECU 1:
Estudiante de 21 años sufre de gastritis aguda ocasionada por comer en lugares poco
higenicos. Suele consumir caramelos (“chupar”) mientras esta en base hasta la tarde.
También toma regular cantidad de leche (grasa, lactora, proteinas), pues le calma el dolor y
el ardor que sitnete por la gastritis ​(tiene dispepsia y cuando toma la leche se le pasa).
Incluso cuando puede, se toma dos vasos de agua frita y le calma la molestia. Ha decido ir
al medico para tratarse, pues ya no soporta el dolor, el cual esta seguro que los síntomas
se producen por elevada producion de HCl en el estomago, y por ello le ha recetado
ranitidina
139. El consumir caramelos eleva los niveles en sangre de una hormona cuya función es
la estimulación de las células.
- ​Beta del páncreas por GIP​ el cual es una incretina y por consiguiente estimula
las células pancreáticas
140. Consumir caramelos indirectamente actica la via:
-​ POMP/ CART saciedad
141. Consumo de leche produce indirectamente
- ​CCK​ inhibición del vaciamiento gástrico mayor tonicidad del esfínter pilórico
142.Cuando el px toma dos vasos de agua, genera indirectamente un aumento en la
liberación de:
- vaso de agua distención → ​g​ ​astrina​ → secreción de HCl
143.El uso de ranitidina bloquea el receptor H2 de la histamina en las células parietales, la
histamina llega a estas células por:
- histamina es una hormona paracrina por →​ difusión
**endocrina es por via hematógena y si fuera neuroendorina es por un NTs
144. Aumenta la secreción salival:
- ​noradrenalina​ a través de los receptores Beta 2
145.En este paciente con gastritis aguda debida a una alta producción de ácido clorhídrico,
sería lógico esperar que el píloro tenga un tono muscular:
- primero la​ secretina
- luego CCK
**ambas reguladores del HCl, Gatritis aguda debido a una alta producción de HCL piloro
estará aumentado (por la CCK)
146.Debido al uso de ranitidina, los valores de somatoestina en sangre:
- ranitidina disminuye acción de gastrina se quiere secretar mas no actúan
los inhibidores como la somatoestina somatoestina ​disminuye
147.El uso de atropina en este paciente:
- Inhibirá la acción de las prostaglandinas
- Aumentará la producción de ácido clorhídrico
- Disminuirá la acción del receptor CCK-B
​-Aumentará el pH del estómago
ECU 2:
Niño de sexo masculino de 2 años de edad, sufre de estreñimiento desde el nacimiento (1
deposición cada 3-4 días). Madre menciona que le estimula la defecación con un
termómetro rectal, y continuo uso de enemas y laxantes. Desde hace 6 meses comienza
con vómitos postprandiales. Los síntomas aumentan en frecuencia y magnitud y están en
relación con los episodios de estreñimiento. No refiere fiebre, tos, diarrea ni lesiones
cutáneas. Al examen físico presenta regular estado general, luce deshidratado. Abdomen
distendido, blando, depresible e indoloro. No se palpan masas abdominales. Se
permeabiliza el canal anal con termómetro rectal, encontrando cierta resistencia. Salida de
material fecal mal oliente en regular cantidad. Exámenes de laboratorio: hemograma
normal. Signos inflamatorios de fase aguda negativos. Alcalosis metabólica leve en sangre
venosa. Radiografía con enema baritado muestra recto y colon sigmoides dilatados
(megacolon). Biopsia profunda: ausencia de células ganglionares en la muestra enviada.
Se realiza cirugía correctiva.
148. ​Durante la fase esofágica de la deglución, para un bolo alimenticio determinado,
a medida que avanza el bolo la fuerza de la contracción se hace más:
- hiperpolarizado
- fuerte
- dependiente de Ach
- debil
149. Cuando este paciente ingiera sus alimentos, se espera que al momento de pasar el
bolo alimenticio por el esfínter esofágico superior, la presión intraesofágica disminuya en:
- la porción proximal al bolo
- el tercio medio del esófago
​-el cardias
- el lugar donde se contraiga la muscular propia
150. Al examinar la orofaringe del paciente, uno puede hallar fácilmente la amígdala
palatina, pues esta se encuentra inmediatamente detrás del músculo:
​-Palatogloso
- Palatofaringeo
- Hiogloso
- Elevador del velo del paladar
151. Con respecto a la defecación en este caso, marque la respuesta correcta:
- En posición de cuclillas, el músculo puborectal genera un ángulo más agudo en
el recto
- El sigmoides y el recto están inervados por el nervio vago
- La sensación de defecar sólo se da cuando el recto es ocupado por heces
y alcanzado el 80% de su capacidad
​-El esfínter anal comprometido tiene inervación autónoma
152. En este paciente [hirschsprung] se considera que está abolido el reflejo:
- Coloileal
​-Rectoesfinteriano
- Gastrocólico
- Relajación receptiva
153.No se espera que sea causa del vómito:
-​ Ayuno prolongado
- Estimulación faríngea y del glosofaríngeo
- Irritación de la mucosa gástrica
- Dolor intenso

ECU 3:
Paciente de 54 años con antecedentes de alcoholismo, gastritis crónica, tabaquismo
pesado, obesidad, cálculos biliares y cirrosis, es llevado a la emergencia por dolor
abdominal en epigastrio irradiado a la espalda y trastorno del sensorio.
Al examen físico: presión arterial 85/50 mmHg, frecuencia cardíaca 100 latidos/min,
frecuencia respiratoria 18 x minuto, temperatura axilar 36°C.
Conjuntivas pálidas, escleras ictéricas nevus arácnidos en tronco, distensión abdominal
marcada, cabeza de medusa, matidez desplazable en ambos flancos e hipogastrio, dolor a
la palpación de abdomen.
Tiempo de protrombina: 24 seg (testigo: 13 seg); TPT: 38 seg, glicemia: 165 mg/dL, uremia:
20 mg/dL, ASAT: 76 UI/L, ALAT: 22 UI/L, albumina: 2,5 g/dL, bilirrubina total: 2,6 mg/dL,
bilirrubina directa: 1,4 mg/dL, amilasa sérica 4000 U/L.
154. ​En esta paciente, al aumento de la amilasa sérica, se debe directamente a una lesión
de:
​a) páncreas
b) vesícula y árbol biliar
c) estómago
d) hígado
155. Considerando que el paciente sufre de gastritis, se puede decir que la secreción de
ácido por la mucosa gástrica
​a) involucra transporte activo de hidrogeniones
b) es realizada principalmente por células principales
c) es inhibida por antihistaminas tomadas por pacientes con rinitis alérgica
d) involucra la liberación de HCl de los gránulos zimógenos
156. El paciente tiene hemorragia digestiva alta por várices sangrantes como complicación.
Llegando a estar en shock hipovolémico por hemorragia masiva, se encontrara necrosis
hepática en:
a) zona 1
b) no se afectan los lobulillos hepáticos en hemorragia
​c) zona 3
d) zona 2
157. El misoprostol, análogo de las prostaglandinas está mejor indicado en:
c) cicatrización de úlcera péptica duodenal
d) erradica el helicobacter pylori
e) tratar el sind de Zollinger ellison
​d) prevenir daño por AINES
158. De las siguientes sustancias secretadas por los órganos de este paciente, la
más alcalina es la secreción:
- Esofágica
- Salival
- Yeyunal
​-Pancreática
159. En este paciente, se puede asumir que la pancreatitis ha sido ocasionada por
una disminución en el efecto de:
- Lipasa
- Enterocinasa
- Amilasa
​-Inhibidor de la tripsina
160. ¿Cuál de las siguientes sustancias es segregada por el páncreas?
​-Amilasa
-Pepsina
- Quimiotripsina
- Tripsina
161. Cada vez que este paciente toma alcohol, la acidificación de la luz del duodeno:
-​ Disminuye el vaciamiento gástrico
- Aumenta la contracción del esfínter de Oddi
- Aumenta la secreción del ácido gástrico
- Disminuye la secreción pancreática del bicarbonato

162. ​Un niño de 2 años es llevado a la consulta por diarrea persistente y edema de las extremidades, además
falta de crecimiento y desarrollo en relación a su edad. Los análisis de sangre revelan que tiene concentración
plasmática baja de proteínas (hipoproteinemia). Durante la endoscopía duodenal, se coloca colecistokinina
(CCK) endovenosa y se recoge muestras del líquido duodenal; el resultado del líquido confirma incapacidad para
hidrolizar proteínas a un pH neutro, esta situación mejora al añadir una pequeña cantidad de tripsina. El
paciente probablemente esté sufriendo la falta congénita de
………….
(Unidad 4, sesión 26, logro 2: Explicar la Digestión y absorción de las proteínas y sus alteraciones)
a. Pepsinógeno
b. PEPT-1
c. Carboxipeptidasas
d. Enterocinasa
163. ​Experimentalmente se incrementa la velocidad de la secreción salival con una sustancia, el análisis de
la composición de esta saliva obtenida se espera encontrar…………..
(Unidad 3, sesión 17, logro 5 : Explica la Influencia de la velocidad del flujo salival en la composición de la
saliva)
e. Elevación de concentración de bicarbonato, sodio y potasio
f. Elevación de concentración de cloro, sodio y potasio
g. Disminución de concentración de potasio
h. Disminución de concentración de potasio y bicarbonato
164. ​Paciente varón de 46 años soltero, consulta por odinofagia y bajo de peso, tiene antecedente de
tuberculosis desde hace 3 meses y es fumador crónico (10 cigarrillos por día); al evaluar la cavidad oral se
identifica lesión blanquecina en el dorso de la lengua y paladar blando, las lesiones se desprenden con el baja
lengua dejando una base eritematosa. Esta lesión corresponde probablemente a ……………………….…..
( Unidad 3, sesión18, logro 1-2 : Describe las enfermedades inflamatorias, infecciosas y proliferativas de la
cavidad oral)
i. Eritroplaquia
j. Candidiasis oral
k. Leucoplaquia vellosa
l. Fibroma en cavidad oral
165. Minero de 32 años de edad, que acude a centro de
salud por presentar de forma progresiva desde hace 1
año dificultad para ingerir alimentos sólidos y luego
líquidos; refiere regurgitaciones alimentarias y marcada
pérdida de peso (15 kilos). Radiografia baritada de
esófago como se muestra en la figura. El presente caso
se explica por……………….
(Unidad 2, sesión 12, logro 4: Identificar y
describir la función de los esfínteres esofágicos)
m. Contracción incompleta del esfínter esofágico inferior
n. Dificultad para el inicio de la deglución
o. Relajación incompleta del esfínter pilórico
p. Relajación incompleta del esfínter esofágico inferior
166. Paciente mujer de 35 años acude a consulta por sensación de sequedad y lesiones en cavidad oral. Al examen se observa
atrofia de la mucosa, fisuras y úlceras; nota además sequedad e irritación de la córnea y aumento del tamaño de las glándulas
parotídeas. Su diagnóstico más probable es artritis reumatoide; el hallazgo más probable en una biopsia de glándula parótida
es……..….
(Unidad 3, sesión 18, logro 3: Describe las enfermedades más frecuentes de las glándulas salivales)
q. Hiperplasia de acinos glandulares serosos
r. Gran infiltración de linfocitos y células plasmáticas
s. Gran infiltrado de linfocitos y macrófagos
t. Presencia de acinos normales con hiperplasia de células ductales

167. Un paciente con anemia acude con su médico quejándose de episodios frecuentes de gastroenteritis. Un análisis de sangre
revela anticuerpos circulantes dirigidos contra células parietales gástricas. Su anemia es atribuible a la hiposecreción de
………………………
(Unidad 3, sesión 20, logro 5: Gastritis crónica. Tipos de gastritis)
u. Factor intrínseco
v. Proteina R (haptocorrina)
w. Pepsinógeno
x. Ácido clorhídrico

168. ​Dos estudiantes deciden tomar un receso para comer una hamburguesa a la hora del almuerzo. Antes de llegar a la cafetería,
impulsos nerviosos provenientes del complejo vagal dorsal iniciarán la secreción de ácido gástrico por la liberación dedesde el
sistema nervioso entérico.
(Unidad 3, sesión 20, logro 2: Regulación de la secreción gástrica: estimulación, fases de la secreción)
y. Serotonina
z. Óxido nítrico
aa. GRP (péptido liberador de gastrina)
bb. Péptido intestinal vaso activo

169. Un niño de cuatro años de edad es llevado a la consulta por cuadros diarreicos frecuentes caracterizados por heces pálidas,
voluminosas y fétidas, presenta bajo peso y talla. Se mide la concentración de cloruro en el sudor y se encuentra que sus valores
son muy elevados. La alteración más importante a nivel de células ductales del páncreas tiene relación directa con la
conductancia de…………
(Unidad 3, sesión 23, logro 5 Explica la Secreción pancreática: formación del jugo pancreático, influencia de la velocidad de
flujo y regulación)
cc. Potasio
dd. Bicarbonato
ee. Sodio
ff. Cloro

170.Una mujer de 50 años de edad que sufrió durante varios años resequedad de los ojos debida a producción inadecuada de
lágrimas es enviada con un gastroenterólogo para evaluación de pirosis crónica. El examen endoscópico revela erosiones y tejido
cicatrizal en la parte distal del esófago justo por arriba del esfínter esofágico inferior. Las lesiones pueden atribuirse a la
disminución de uno de los siguientes componentes salivales:
(Unidad 3, sesión 17, logro 4: Explicar la Formación de la saliva y cuáles son sus componentes)
gg. Bicarbonato
hh. Lactoferrina
ii. Ig A
jj. Amilasa
171. Se evalúa los valores séricos de las siguientes sustancias a un paciente con enfermedad hepática terminal; en este paciente se
espera encontrar la combinación con la letra …………
(Unidad 3, sesión 22, logro 5: Describe las Pruebas de función hepática, la Insuficiencia hepática, encefalopatía hepática e
hipertensión portal)

Glucosa Amoniaco Albúmina


a. Aumenta Disminuida Disminuida
da
b. Disminui Aumentada Aumentada
da
c. Aumenta Aumentada Aumentada
da
d. Disminui Aumentada Disminuida
da
172. Una mujer de 35 años de edad HIV positiva, se presenta al médico con dolor abdominal en cuadrante superior derecho e
ictericia. La paciente refiere haber tenido múltiples episodios de ictericia durante los últimos 10 años. Los exámenes para
determinar hepatitis viral, dieron positivos para Hepatitis B, siendo catalogado el caso como hepatitis crónica con alteración
funcional. En un examen de sangre ¿cuál de los siguientes parámetros está disminuido?
(unidad 3, sesión 22, logro 5: Pruebas de función hepática, Insuficiencia hepática, encefalopatía hepática e hipertensión
portal)
kk. Fosfatasa alcalina
ll. Albumina
mm. Bilirrubina
nn. Tiempo de protrombina
173. En el reflejo peristáltico del intestino delgado, uno de los siguientes eventos sucede en la porción oral del bolo
alimenticio…………...
(Unidad 2, sesión 13, logro 4: Explicar la Motilidad del intestino delgado: Contracciones segmentarias y peristálticas)
oo. Disminución de 5 hidroxitriptamina desde las neuronas IPAN
pp. Contracción del músculo longitudinal
qq. Acción del péptido intestinal vasoactivo (VIP) en el músculo circular
rr. Acción de acetilcolina en el músculo circular

174. Experimentalmente se coloca una dosis alta de secretina en la luz intestinal duodenal; como consecuencia de esto, en el
jugo pancreático de la misma luz intestinal se observa la disminución de la concentración de …..………..
(Unidad 3, sesión 23, logro 5: Explica la Secreción pancreática: formación del jugo pancreático, influencia de la velocidad de
flujo y regulación)
ss. Na​+
tt. Cl​-
uu. K​+
vv. HCO3​-

175. Un varón de 58 años de edad con enfermedad de Crohn severo fue sometido a una resección ileal. Después de la cirugía
este paciente padecerá de esteatorrea, esto se explica porque …..………..
(unidad 4, sesión 26, logro 4: Explica las alteraciones en la Absorción de lípidos)
ww. El pool de ácidos biliares se incrementa
xx. Los quilomicrones no pueden formarse en el lumen intestinal
yy. La micelas no pueden formarse
zz. El páncreas no secreta lipasa

176. En un experimento se inserta un balón en el estómago de un voluntario, se infla poco a poco mientras que se vigilan las
presiones intraluminales. Aunque el volumen del balón aumenta considerablemente, las presiones permanecen constantes. Esta
relación volumen-presión se explica por la liberación local de …………..
(Unidad 2, sesión 13, logro 1 Explica la Motilidad gástrica: relajación receptiva)
aaa. Acetil colina y gastrina
bbb. Colecistoquinina y óxido nítrico
ccc. Óxido nítrico y péptido inhibidor vasoactivo
ddd. Norepinefrina y óxido nítrico
177. La toxina del Vibrio cholerae causa diarrea debido a…….
(Unidad 4, sesión 27, logro 6: Explica el transporte hidroelectrolítico intestinal, toxina colérica)
eee.La fosforilación del canal CFTR de los enterocitos de las vellosidades intestinales
fff. El Incremento de la secreción de cloro por las células de la cripta intestinal
ggg. La inhibición de la producción de AMPc por las células epitelailes
hhh. El incremento de la absorción de agua y sodio a través de las uniones estrechas

178. ¿Cuál de las siguientes alternativas es una característica de la secreción exocrina del páncreas?
(Unidad 3, sesión 23, logro 5: Secreción pancreática: formación del jugo pancreático, influencia de la velocidad de flujo y
regulación)
iii. Tiene una baja concentración de Cl​-​ respecto al plasma
jjj. Es estimulada por la presencia de bicarbonato en el duodeno
kkk. La secreción enzimática es estimulada principalmente por la gastrina
lll. Es hipotónica respecto al plasma

179. Una madre lleva a su hijo de dos años de edad a la sala de urgencias, estresada porque el niño deglutió una moneda de 10
céntimos mientras la familia cenaba en un restaurante. El médico observa mediante fluoroscopía que la moneda se halla en el
estómago y asegura a la madre que la moneda se eliminará con las heces. El médico recomienda utilizar la respuesta fisiológica
que permitirá la evacuación de la moneda del estómago al intestino ………….…..
(Unidad 2, sesión 13, logro 2: Explica la Motilidad gástrica: mezclado y vaciamiento)
mmm. Es por la relajación receptiva
nnn. Son los movimientos de mezcla y trituración
ooo. Es provocada por el ayuno
ppp. Es por la relajación del esfínter esofágico superior

180. Las estructuras en el hígado que permite que los productos metabólicos unidos a proteínas tengan acceso a las membranas
basolaterales de los hepatocitos, son…..
(Unidad 3, sesión 21, logro 4-5: Explica la Organización micro estructural del hígado)
qqq. Los Canalículos
rrr. Las fenestras sinusoidales
sss. Las uniones intercelulares herméticas
ttt. Las células de Ito

181. La composición de la bilis es modificada conforme fluye por los conductillos biliares. Durante este tránsito se espera que
aumente la concentración de…….
(Unidad 3, sesión 22, logro 2: Describe la Secreción biliar, visión general del sistema biliar extrahepático y composición de la
bilis)
uuu. Ig A
vvv. Glucosa
www. Monómeros de ácido biliar
xxx. Vitamina A

181. Se mide experimentalmente el contenido gástrico de dos personas. La persona “A” tiene alto contenido de grasa y la
persona “B” tiene un contenido hipertónico ¿Cuál de las siguientes es correcto respecto al vaciamiento gástrico? ​(Unidad
2, sesión 13, logro 2: Describe la Motilidad y vaciamiento gástrico)
yyy. Hay ralentización del vaciado gástrico solo en “A”
zzz. El vaciamiento gástrico es más rápido en ambos
aaaa. En ambos casos hay incremento de la motilina
bbbb. Hay ralentización del vaciado gástrico en ambos casos

182. El examen endoscópico de un paciente con hipertensión portal grave revela venas tortuosas que sobresalen hacia la luz del
esófago. El paciente recibe tratamiento quirúrgico mediante la colocación de una derivación que conecta la vena porta a la vena
cava. Después de la operación el riesgo de encefalopatía ………………….. y el riesgo de sangrado de várices ……………..
(Unidad 3, sesión 22, logro 5: Describe la Insuficiencia hepática, encefalopatía hepática e hipertensión portal)
cccc. Aumentará/disminuirá
dddd. Disminuirá/disminuirá
eeee. Aumentará/aumentará
ffff. Disminuirá/aumentará
183. Un paciente varón de 18 años de edad acude al médico para sus exámenes de rutina. Sus resultados de laboratorio
muestran un valor de bilirrubina sérica de 4 mg/dl y una bilirrubina directa de 0,3 mg/dl. Las pruebas de función hepática son
normales. La alteración que explica mejor este caso es por la deficiencia de ………………..
(Unidad 3, sesión 22, logro 3: Explica la Producción y excreción de bilirrubina. Tipos de bilirrubina e ictericia)
gggg. Transaminasas
hhhh. Glucuronil transferasa
iiii. Hemo oxigenasa
jjjj. La 7 alfa hidroxilasa

184. Un hombre de 57 años de edad es llevado a urgencias con hematemesis masiva rojo brillante, a su llegada se halla
inconciente con PA: 80/40 mm Hg y FC: 124 lat/min. Luce ictérico con presencia de “arañas vasculares en el tórax anterior y
extremidades”, abdomen distendido con signo de oleada positiva. Se encuentra esplenomegalia y pérdida de la masa muscular
en extremidades. La anastomosis vascular responsable del sangrado en este paciente es ………….…..
(Unidad 3, sesión 21, logro 2: Describe las anastomosis porto sistémicas)
kkkk. Arteria gástrica izquierda y vena ácigos
llll. Vena gástrica izquierda y vena ácigos
mmmm. Vena paraumbilical y vena epigástrica inferior
nnnn. Vena gástrica izquierda y vena esofágica superior

185. Un estudiante de medicina está comiendo un plato de comida a base de champiñones, espárrago y salsa de soya. El sabor
umami contenido en todos estos alimentos actúa a nivel de los botones gustativos estimulando ………………..
(Unidad 2, sesión 10, logro 5: Describe los tipos y mecanismos moleculares para la detección de los sabores)
oooo. El ingreso de sodio
pppp. Un receptor acoplado a proteína G
qqqq. Su receptor específico T1R3
rrrr. El ingreso de hidrógeno

186. Un hombre de 22 años de edad se presenta al médico con una historia de 1 año de evolución caracterizado por dolor
recurrente en fosa iliaca derecha y diarrea. Manifiesta además pérdida de peso de 8 kg durante este periodo. La
colonoscopía revela múltiples lesiones en el ileon terminal y colon. La biopsia de estas lesiones revela engrosamiento,
inflamación y ulceración de la mucosa. El diagnóstico más probable en este caso es…….
(Unidad 4, sesión 28, logro 5: Describe la Enfermedad inflamatoria intestinal. Generalidades, morfología y características)
ssss. Sprue celiaco
tttt. Enfermedad de Crohn
uuuu. Sindrome de colon irritable
vvvv. Colitis ulcerativa

187. Una de las funciones del músculo señalado es:


(Unidad 2, sesión 8, logro 3: Describir el Piso de la boca: estructuras
blandas que la conforman)
wwww. Eleva el paladar blando
xxxx. Recibe inervación del nervio maxilar
yyyy. Deprime el hioides cuando la mandíbula está fija
zzzz. Deprime la mandíbula cuando el hioides está fijo

188. Varón de 61 años que consulta por dolor retro esternal intenso desde hace 6 horas y después de vómitos intensos y
repetidos; al examen se observa disnea, cianosis, hipotensión y signos clínicos de shock. La radiografía simple de tórax
muestra neumomediastino. El líquido en el espacio pleural aspirado tiene alta concentración de amilasa. ¿Cuál de las
siguientes alternativas puede explicar este cuadro clínico?
(Unidad 3, sesión 18, logro 6: Describe algunas Enfermedades del esófago)
aaaaa. Sindrome de Mallory Weiss
bbbbb. Rotura espontánea de esófago
ccccc. Neumotórax por probable herida penetrante
ddddd. Perforación de ulcera gástrica de cara posterior, con complicación torácica
189. La secreción del ácido en la célula parietal gástrica se lleva a cabo por una ATPasa especifica que intercambia hidrogeniones
(H+) del citosol por…..
(Unidad 3, sesión 20, logro 1: Explica la Secreción del HCl y sustancias que la alteran)
eeeee. Cl-
fffff.HCO3-
ggggg. Na +
hhhhh. K+

190. ​En condiciones normales el ingreso de 600 ml de líquido es el estómago provoca un aumento de presión intragástrica de
unos 12 cm de H2O. Después de una vagotomía (corte del nervio vago) es de esperar que el ingreso del mismo volumen de
líquido provoque lo siguiente: …………………………………
(Unidad 2, sesión 13, logro 1: Describe la Motilidad gástrica: relajación receptiva)
iiiii. Un aumento igual de la presión
jjjjj. Que no aumente la presión
kkkkk. Un aumento mayor de la presión
lllll. Una disminución de la presión

191.Una paciente de 30 años de edad es sometida a una cirugía en oído medio derecho por un problema de otoesclerosis. Luego
de la cirugía refiere alteración en la percepción de sabores. Al evaluar el caso usted esperaría encontrar……….
(Unidad 2, sesión 10, logro 5: Describe la Irrigación e inervación de la lengua)
mmmmm. Alteración en la sensación del dolor y temperatura en el tercio posterior de la lengua
nnnnn. Alteración en la sensación del gusto en los dos tercios anteriores de la lengua
ooooo. Alteración en la sensación del gusto en la punta de la lengua
ppppp. Sensación del dolor, tacto y temperatura conservada en toda la lengua

192. ¿Cuál de las siguientes alterativas es correcta?


(Unidad 4, sesión 26 : Explica la digestión y absorción de nutrientes y sus alteraciones)
qqqqq. En el borde luminal, en cepillo, del intestino delgado, la absorción de sodio únicamente se realiza asociada a
la de glucosa.
rrrrr. El lugar principal para la absorción del hierro es el ileon
sssss. Las sales biliares desconjugadas son absorbidas preferentemente en el colon
ttttt. El proceso de digestión y absorción de la vitamina B12 no se altera en insuficiencia pancreática.

193. En un paciente de 45 años de edad con colestasis biliar, se encuentra una elevación de los niveles sanguíneos de fosfatasa
alcalina hasta 3 veces la cifra normal. ¿Cuál de las siguientes alternativas estará también elevada como evidencia del daño de la
vía biliar?
(Unidad 3, sesión 22, logro 5: Pruebas de función hepática, Insuficiencia hepática, encefalopatía hepática e
hipertensión portal)
uuuuu. Tiempo de protrombina y albúmina sérica
vvvvv. Transaminasas hepáticas (ALT y AST)
wwwww. Glucoronil transferasa
xxxxx. Gamma glutamil transpeptidasa

194. Revisando la angiografía de un hombre de 70 años en estudio por aneurisma de aorta abdominal el radiólogo informa de la
presencia de una oclusión completa de la arteria mesentérica inferior. El paciente se encuentra completamente asintomático.
¿Cuál de las siguientes arterias se anastomosa a la sistema arterial de la mesentérica inferior?
(Unidad 4, sesión 25, logro 1: Identifica la Arteria mesentérica superior e inferior, ramas y anastomosis)
yyyyy. Ileal
zzzzz. Cólica media
aaaaaa. Sigmoideas
bbbbbb. Cólica izquierda

195. ​Lactante de 3 meses de vida es atendido por presentar diarrea, se administra una solución de glucosa y electrólitos por vía
oral. La proteína de membrana apical que explica la capacidad de esta solución para proporcionar aporte de glucosa e
hidratación es ………..
(Unidad 4, sesión 26, logro 1: Explica la Digestión y Absorción de los hidratos de carbono. Alteraciones)
cccccc. GLUT-5
dddddd. SGLT-1
eeeeee. CFTR
ffffff.GLUT-2
196. Paciente ha sufrido herida de bala en el abdomen, se le ha tenido que extirpar el segmento medio y distal del ileon. En este
caso la síntesis hepática de sales biliares estará …..…..
(Unidad 3, sesión 22, logro 4: Explica la formación, función y Circulación entero hepática de lasa sales biliares)
gggggg. Disminuida por inhibición de la colesterol 7 alfa hidroxilasa
hhhhhh. Incrementada por estímulo de la enzima colesterol 7 alfa hidroxilasa
iiiiii. Incrementada por inhibición de la colesterol 7 alfa hidroxilasa
jjjjjj. Sin cambios en el ritmo de síntesis

197. ​Un varón de 75 años ingresa al consultorio por presentar ictericia marcada de piel y las escleras. El estudio del paciente
mostró que presentaba un tumor que obstruía la totalidad del conducto hepático común. ¿Cuál de las siguientes estructuras se
encontrará dilatada en este paciente?
(Unidad 3, sesión 21, logro 6: Describir el árbol biliar intrahepático)
kkkkkk. Conducto de Wirsung
llllll. Conductos de Hering
mmmmmm. Conducto colédoco
nnnnnn. Conducto cístico

198.Correlaciones las dos columnas y marque la fórmula correcta:


(Unidad 4, sesión 28, logro 1: Diarrea: definición, mecanismos: osmótica, secretoria y exudativa)
1. Enfermedad Hirschsprung(4) heces con moco y sangre
2. Diarrea osmótica(2) intolerancia a lactosa
3. Diarrea secretoria(1 ) aganglionosis congénita
4. Diarrea exudativa(3) canales de Cl- en las células de la cripta

a.- 4231 b.- 1234 c.- 2143 ​d.- 4213


199. Respecto a la siguiente imagen que representa una estructura de la mucosa gástrica, la estructura con número ………..
produce ……………………..
(Unidad 3, sesión 19, logro 4: La glándula fúndica. Funciones y tipos de células con sus características)
oooooo. 3 / pepsina
pppppp. 1 / Pepsinógeno
qqqqqq. 4 / HCl y factor extrínseco
rrrrrr. 2 / pepsinógeno

200. En un paciente con insuficiencia renal crónica, el déficit en la absorción de calcio a nivel del enterocito se debe a lo
siguiente:
(Unidad 4, sesión 26, logro 6: Explica la Absorción de calcio y hierro)
ssssss. No se convierte la 25 hidroxicolecalciferol a 1,25 dihidroxicolecalciferol
tttttt. No se convierte la 1,25 dihidroxicolecalciferol a 25 hidroxicolecalciferol
uuuuuu. Se incrementa la producción de Calbindina
vvvvvv. Existe un descenso de la alfa 25 hidroxilasa renal
201. Varón de 30 años es traído a emergencia por agresión abdominal con arma de fuego (pistola) y es sometido a
laparotomía exploratoria, observándose isquemia del colon ascendente y parte del colon trasverso ¿la lesión de cuál de las
siguientes arterias explicaría esta isquemia?
(unidad 1, sesión 2, logro 6: (D​escribe la irrigación visceral: arterias de tronco celiaco, arteria mesentérica
superior e inferior​, topografía de superficie, órganos por cuadrante)

a. Celiaca
b. Colónica derecha
c. Mesentérica inferior
d. Mesentérica superior
202. Respecto a las sustancias gastrointestinales que regulan la secreción pancreática; marque la afirmación correcta:
(unidad 1, sesión 3, logros 2 y 3: describir las hormonas gastrointestinales: estímulos y funciones)

a. La Secretina, es la hormona más importante para la secreción de bicarbonato por las células acinares del
páncreas
b. La acetilcolina es capaz de estimular la secreción enzimática y de bicarbonato del páncreas
c. La gastrina, es la hormona más importante para la secreción de enzimas pancreáticas
d. La colecistoquinina (CCK) estimula al páncreas solo para secreción enzimática

203. Ante una lesión del X par craneal, ¿cuál de los siguientes músculos mantiene conservada su función?:
(unidad 2, sesión 08, logro 4: Paladar blando: componentes musculares)

e. Elevador del velo del paladar


f. Tensor del velo del paladar
g. Palatofaríngeo
h. Glosofaríngeo
204. Experimentalmente se utiliza atropina (anticolinérgico) para inhibir la secreción de gastrina, sin embargo la secreción
de esta hormona se sigue dando ante estímulos vagales. Esta situación se explica porque la atropina:
(unidad 1, sesión 3, logro 3 : describir las hormonas gastrointestinales: estímulo y funciones de la gastrina y
colecistoquinina)

a. Bloquea parcialmente la bomba de protones en la célula G


b. Inhibe la acción de acetilcolina e histamina en la célula G
c. Solo inhibe la acción del péptido GRP en la célula G
d. No bloquea la acción del péptido GRP
205. ​Un varón de 50 años es sometido a extirpación del duodeno y parte proximal del yeyuno. Esta situación
ocasionaría la pérdida de las células ……….. , productoras deque estimula la secreción de
bicarbonato por el páncreas.
(unidad 1, sesión 3, logro 3: describir las hormonas gastrointestianles: estímulos y funciones de la secretina y
péptido insulinotrópico dependiente de glucosa)

a. “S” / secretina
b. Parietales / secretina
c. “I” / colecistoquinina
d. “S” / colecistoquinina

206.Recién nacido que presenta tumoración abdominal a nivel del cordón umbilical (fotografía). ¿cuál de las siguientes
afirmaciones es correcta respecto a este defecto en el desarrollo embriológico del intestino?: ​(unidad 1, sesión 5, logro 2:
identificar las anomalías del desarrollo del intestino medio)

a. Corresponde a una Gastrosquisis


b. Las vísceras se hallan cubiertas por piel
c. No está asociado a otras malformaciones
d. Se asocia a malformaciones cardiacas y
del tubo neural

207. Varón de 35 años acude a la emergencia por trauma abdominal y se decida


realizar una laparoscopía exploratoria. El cirujano observa la disposición de los
órganos abdominales como se representa en el siguiente esquema. Esta
disposición de órganos se explica por la rotación(SMA=arteria mesentérica
superior)
(unidad 1, sesión 5, logro 3: identificar las anomalías del desarrollo del
intestino medio: defectos de rotación, estenosis y atresias)

a. anti horaria del intestino medio, en sólo 90°


b. incompleta del intestino medio (270°)
c. horaria del intestino medio
d. horaria del estómago

208. Se evalúa la expresión de la proteína Agrp en una persona con alteración del apetito; lo correcto respecto a esta
proteína es…..
(unidad 1, sesión 3, logro 4: E​ xplica los mecanismos de control del apetito y saciedad ​)

a. Esta proteína es un potente anorexigénico


b. La mutación del gen que la codifica produce adelgazamiento
c. La sobre producción de la proteína lleva a obesidad por agonismo de receptores MC3 y MC4
d. La sobre producción de la proteína disminuye el apetito por antagonismo de receptores MC4

209. Juana cae de la bicicleta y se fractura la región anterior del hueso maxilar superior con compromiso de la fosa incisiva.
Al examen físico de la región esperaría encontrar alteración en la sensibilidad de la encía …………………
(unidad 2, sesión 8, logro5: paladar: paladar duro y blando: irrigación e inervación)

a. bucal posterior
b. Lingual anterior
c. palatina anterior
d. palatina posterior
210. Recién nacido es atendido por el neonatólogo y luego entregado a su madre para dar de lactar; la madre al dar de lactar
observa coloración azulada de labios, acompañado de tos persistente, dificultad respiratoria y distención abdominal. Se le intenta
colocar una sonda nasogástrica pero esta retorna a la cavidad oral en todos los intentos. ¿Cuál de las siguientes anomalías del
desarrollo es el más probable en este caso? ​(unidad 1, sesión 4, logro 3: identificar las anomalías en el desarrollo del esófago:
atresia y/o fístula traqueo esofágica)

a. Estenosis esofágica proximal con Fístula traqueo esofágica distal


b. Atresia esofágica proximal con fístula traqueoesofágica distal
c. Atresia esofágica distal con fístula traqueoesofágica proximal
d. Fístula traqueoesofágica proximal y distal

211. ¿Cuál de los siguientes mecanismos ocurre durante la defecación?


(unidad 2, sesión 13, logro 6: motilidad del intestino grueso: contracciones segmentarias, movimientos en
masa, defecación y reflejo gastrocólico)

i. Contracción refleja del esfínter anal interno


j. En la posición de “cuclillas” el músculo puborectal se halla relajado
k. Relajación del esfínter anal externo por efectos del VIP y óxido nítrico
l. La materia fecal en el recto estimula la contracción del sigmoides por los nervios pudendos

212. La estructura número 4 (gráfico) corresponde a


……….… y está ………..
(unidad 2, sesión 9, logro 2: Partes de un diente.
Capas del diente: Esmalte: características y células
que lo producen)

m. el cemento / mineralizado en 90%


n. la dentina / formada por ameloblastos
o. el esmalte / formado por células derivadas
del mesénquima
p. la dentina / formado por células derivadas de la
cresta neural

213. Un paciente luego de un accidente sufre lesión del piso de la boca, se constata daño del nervio “cuerda del
tímpano”, en este caso se esperaría encontrar disminución de lade la lengua
(unidad 2, sesión 10, logro 3: Irrigación e inervación de la lengua)

a. Motilidad en los dos tercios anteriores


b. Sensación del gusto en el tercio posterior
c. Sensación del gusto en los dos tercios anteriores
d. Sensibilidad al tacto en los dos tercios anteriores

214. ¿Cuál de las siguientes afirmaciones es la correcta sobre la gastrina?


(unidad 1, sesión 3, logro 1: reconocer las características de las sustancias reguladoras gastrointestinales:
hormonas, sustancias paracrinas y neurocrinas)

q. Produce atrofia de la mucosa gástrica


r. Es producida por la célula G del cuerpo gástrico
s. Es estimulada por la distensión gástrica y el Ph bajo
t. Actúa en la célula diana mediante su receptor CCk tipo B
215. Al recibir un paciente con signos de hipovolemia y antecedente de trauma en abdomen por accidente de tránsito, usted
identifica radiológicamente: lesión de primera vértebra lumbar y signos de lesión en páncreas; durante la cirugía se observó pobre
irrigación de asas intestinales. El vaso afectado es la arteria ……..
(unidad 1, sesión 1, logro 6: reconocer las estructuras a nivel de L1, nivel de los principales vasos sanguíneos)

a. esplénica
b. hepática común
c. mesentérica inferior
d. mesentérica superior

216. Un paciente sufre de daño a nivel del cuello con lesión muscular en la región de la faringe. En el examen físico se
determina dificultad para la elevación de la faringe y para el cierre del itsmo de las fauces. En este caso, probablemente esté
afectado el músculo:
(unidad 2, sesión 11, logro 2: Músculos de la faringe: identificación, constrictores y longitudinales)

a. palatogloso
b. estilofarinfeo
c. palatofaringeo
d. constrictor inferior

217. Varón de 50 años a quien le realizan la curación de la segunda molar de la arcada superior derecha. En un
momento determinado, el paciente acusa de intenso dolor de la pieza dentaria en tratamiento. La vía aferente del dolor
viaja a través del nervio …………
(unidad 2, sesión 9, logro 6: Inervación de los dientes)

a. trigémino V2
b. trigémino V3
c. naso palatino
d. palatino menor

218. La distención gástrica por los alimentos produce incremento de secreción de HCl mediante la producción de
………….. que estimula a las células ……………. vía proteína ………..
(Unidad 1, sesión 3, logro 2: Describe las hormonas gastrointestinales: Estímulo y funciones de la gastrina y
colecistoquinina)

u. gastrina / parietal / Gq
v. gastrina / principal / Gs
w. acetilcolina / parietal /Gi
x. acetilcolina / principal / Gi

218. Un niño de tres años llega a emergencia con disfagia (dificultad para tragar), dolor retro esternal, salivación y llanto. Se
sospecha de ingesta de cuerpo extraño (moneda) en el esófago; al ser evaluado se constata en una radiografía presencia de
cuerpo extraño a nivel de C6 (6° vértebra cervical). El cuerpo extraño estará suspendido a nivel del estrechamiento
producido por………..
(unidad 2, sesión 11, logro4: Esófago, características anatómicas, relación con órganos vecinos y estrecheces)

a. el cayado aórtico
b. el hiato esofágico
c. el músculo cricofaríngeo
d. el bronquio principal izquierdo

219. La triada portal (arteria hepática, vena portal y conducto biliar común) está contenida en el ligamento
…….……… y derivan embriológicamente del ……
(Unidad 1, sesión 1, logro 4: Identifica el peritoneo, mesenterio, omento y ligamentos, retroperitoneo.)

y. hepato duodenal / mesenterio ventral


z. gastro esplénico / mesenterio dorsal
aa. hepato gástrico / omento menor
bb. falciforme / omento menor

220. En relación al movimiento de peristaltismo del tubo digestivo:


en la flecha negra del gráfico se produce la liberación de
……………… a nivel del músculo ………..
(unidad 2, sesión 7, logro 6: Control hormonal y tipos de
movimiento)

a. noradrenalina, sustancia P y neuropéptido “ Y” / circular


b. acetilcolina y sustancia P / longitudinal
c. óxido nítrico y PIV / longitudinal
d. óxido nítrico y PIV / circular

221. Un paciente refiere no percibir algunos sabores, al examen físico se constata alteración en la percepción de sabores y
del dolor en el tercio posterior de la lengua ¿Cuál de los siguientes nervios estará alterada en su función?
(unidad 2, sesión 10, logro 5: Sabores, tipos y mecanismos moleculares para su detección)

a. Lingual (rama del V par)


b. Cuerda del tímpano (VII par)
c. Glosofaríngeo (IX par)
d. Hipogloso (XII par)

222. El gráfico detalla la estructura de la pared del tubo digestivo


intestinal ¿Cuál de las siguientes asociaciones es correcta?
(unidad 2, sesión 7, logro 1: La pared y músculo liso
gastrointestinal )

a. “1” – peristaltismo
b. “2” – secreción enzimática
c. “3” – deriva del mesodermo
d. “4” – doble hoja de tejido graso
223. En el caso de un paciente con gastrinoma (tumor productor de gastrina), la presencia de úlceras duodenales y erosión
de la mucosa gástrica, se debe principalmente a…….
(unidad 1, sesión 3, logro 2: describir las hormonas gastrointestinales: estímulo y funciones de la gastrina y
colecistoquinina)

a. la acción directa de la gastrina sobre la célula principal


b. la sobre expresión de los receptores “G” en la célula parietal
c. el exceso de HCl por estímulo de receptores CCK-B en la célula parietal
d. el exceso de HCl por estímulo directo de receptores de acetilcolina en la célula parietal

224. El reflejo entero gástrico se caracterizan por:


(unidad 2, sesión 13, logro 6: Motilidad del intestino grueso: contracciones segmentarias, movimientos en
masa defecación y reflejo gastrocólico)

cc. favorecer la motilidad gástrica gracias a la CCk


dd. inhibir la motilidad gástrica y estimular la secreción ácida
ee. movilizar grandes volúmenes desde el estómago al duodeno
ff. originarse debido a la distensión duodenal y presencia del quimo ácido
225. Mauricio tiene dificultad para deprimir el paladar y elevar la parte posterior de la lengua. En este caso estará afectado un
músculo ………………., específicamente el músculo …………….
(Unidad 2, sesión 10, logro 2: Músculos de la lengua: clasificación, identificación y sus funciones)

a. intrínseco – longitudinal inferior


b. extrínseco – palatogloso
c. extrínseco – transverso
d. extrínseco – estilogloso

226. Una de las funciones del músculo señalado es:


(Unidad 2, sesión 8, logro 3: Describir el Piso de la boca:
estructuras blandas que la conforman)

gg. deprimir la lengua


hh. elevar el paladar blando
ii. deprimir el hioides cuando la mandíbula está fija
jj. deprimir la mandíbula cuando el hioides está fijo

227. Paciente varón de 30 años es evaluado por probable enfermedad de Chagas, cursa con problemas de motilidad del
colon; los estudios de biopsia determinan ausencia de células ganglionares. Según el
gráfico
¿cuál es la capa en la que se determina la ausencia de dichas células?
(unidad 1, sesión 2, logro 1: describir las generalidades de la estructura del
tubo digestivo: esófago, estómago intestino delgado y grueso)

a. Mucosa - 1
b. Muscular propia – 1
c. Muscular de la mucosa - 2
d. Muscular propia - 3

228. ​Paciente varón de 32 años, que acude a centro de salud por presentar de forma progresiva desde hace 1 año dificultad
para ingerir alimentos sólidos y luego líquidos; refiere regurgitaciones alimentarias y marcada pérdida de peso (15 kilos).
Radiografía baritada (sustancia de contraste​) d​ e esófago se muestra en la figura. El presente caso se explica
por……………….
(Unidad 2, sesión 12, logro 4: Identificar y describir la función de los esfínteres esofágicos)

a. aumento de la peristalsis esofágica


b. relajación incompleta del esfínter pilórico
c. relajación incompleta del esfínter esofágico inferior
d. perdida de producción de PIV y óxido nítrico en el esfínter
esofágico superior
229. En condiciones normales, el ingreso de 600 ml de líquido es el estómago provoca un aumento de presión
intragástrica de unos 12 cm de H​2​O. Después de una vagotomía (corte del nervio vago) es de esperar que el ingreso del
mismo volumen de líquido ocasionede la presión intragástrica.
(Unidad 2, sesión 13, logro 1: Describe la Motilidad gástrica: relajación receptiva)

a. la disminución
b. la no variación
c. un aumento mayor
d. un aumento similar o igual

230. Un niño de 2 años es llevado a la consulta por diarrea persistente, edema de las extremidades y falta de crecimiento en
relación a su edad. Los análisis de sangre revelan que tiene concentración plasmática baja de proteínas (hipoproteinemia).
Como parte del estudio se coloca colecistokinina (CCK) endovenosa y se recoge muestras del líquido duodenal por
endoscopía; el resultado del líquido confirma incapacidad para hidrolizar proteínas a un pH neutro, esta situación mejora al
añadir una pequeña cantidad de tripsina. El paciente probablemente esté sufriendo la falta congénita de ………….
(Unidad 4, sesión 26, logro 2: Explicar la Digestión y absorción de las proteínas y sus alteraciones)
a. PEPT-1
b. pepsinógeno
c. enterocinasa
d. carboxipeptidasas
231. Paciente mujer de 35 años acude a consulta por sensación de sequedad y lesiones en cavidad oral. Al examen se
observa atrofia de la mucosa, fisuras y úlceras; nota además sequedad e irritación de la córnea y aumento del tamaño de
las glándulas parotídas. Su diagnóstico más probable es artritis reumatoide; el hallazgo más probable en una biopsia de
glándula parótida es……..….
(Unidad 3, sesión 18, logro 3: Describe las enfermedades más frecuentes de las glándulas salivales)
e. Presencia de acinos normales con hiperplasia de células ductales
f. Gran infiltración de linfocitos y células plasmáticas
g. Hiperplasia de acinos glandulares serosos
h. Gran infiltrado de linfocitos y neutrófilos
232. Un hombre de 42 años de edad se presenta al médico con una historia de 1 año de evolución, caracterizado por dolor
abdominal bajo y diarreas con crisis sanguinolentas. Manifiesta además pérdida de peso de 8 kg durante este periodo. La
colonoscopía revela lesión difusa en el colon con afectación del recto. La biopsia de estas lesiones revela adelgazamiento de
la pared, inflamación y ulceración de la mucosa y sub mucosa. El diagnóstico más probable en este caso es:
(Unidad 4, sesión 28, logro 5: Describe la Enfermedad inflamatoria intestinal. Generalidades, morfología y
características)
i. sindrome de colon irritable
j. enfermedad de Crohn
k. colitis ulcerativa
l. sprue celiaco
233. ​Dos estudiantes deciden tomar un receso para comer una hamburguesa a la hora del almuerzo. Antes de llegar a la
cafetería, impulsos nerviosos provenientes del complejo vagal dorsal iniciarán la secreción de ácido gástrico por la liberación
dedesde el sistema nervioso entérico.
(Unidad 3, sesión 20, logro 2: Regulación de la secreción gástrica: estimulación, fases de la secreción)
m. Serotonina
n. Colecistoquinina
o. Péptido inhibidor vaso activo
p. GRP (péptido liberador de gastrina)

234. Un niño de cuatro años de edad es llevado a la consulta por cuadros diarreicos frecuentes caracterizados por
heces pálidas, voluminosas y fétidas; al examen físico presenta bajo peso y talla para la edad. Se mide la concentración
de cloruro en el sudor y se encuentra que sus valores son muy elevados. La alteración más importante a nivel de
células ductales del páncreas tiene relación directa con la conductancia de………… ​(Unidad 3, sesión 23, logro 5
Explica la Secreción pancreática: formación del jugo pancreático, influencia de la velocidad de flujo y regulación)
q. Bicarbonato
r. Potasio
s. Sodio
t. Cloro
235. Se evalúa los valores séricos de las siguientes sustancias a un paciente con enfermedad hepática terminal; en este
paciente se espera encontrar la combinación con la letra …………
(Unidad 3, sesión 22, logro 5: Describe las Pruebas de función hepática, la Insuficiencia hepática, encefalopatía
hepática e hipertensión portal)
Glucosa Amoniaco Albúmin
a
a. Aumenta Disminuida Disminui
da da
b. Disminui Aumentada Aumenta
da da
c. Aumenta Aumentada Aumenta
da da
d. Disminui Aumentada Disminui
da da

236. Una mujer de 35 años de edad HIV positiva, se presenta al médico con dolor abdominal en cuadrante superior
derecho e ictericia. La paciente refiere haber tenido múltiples episodios de ictericia durante los últimos 10 años. Los
exámenes para determinar hepatitis viral, dieron positivos para Hepatitis B, siendo catalogado el caso como hepatitis
crónica con alteración funcional. En un examen de sangre ¿cuál de los siguientes parámetros está disminuido?
(unidad 3, sesión 22, logro 5: Pruebas de función hepática, Insuficiencia hepática, encefalopatía hepática e
hipertensión portal)
u. Albumina
v. Bilirrubina
w. Fosfatasa alcalina
x. Tiempo de protrombina

237. En el reflejo peristáltico del intestino delgado ¿Cuál de los siguientes eventos sucede en la porción caudal del bolo
alimenticio?
(Unidad 2, sesión 13, logro 4: Explicar la Motilidad del intestino delgado: Contracciones segmentarias y peristálticas)
y. Acción del péptido inhibidor vasoactivo (VIP) en el músculo circular
z. Acción del NO (óxido nítrico) en el músculo longitudinal
aa. Contracción del músculo longitudinal interno
bb. Acción de acetilcolina en el músculo circular

238. Un varón de 58 años de edad con enfermedad de Crohn severo fue sometido a una resección ileal. Después de la
cirugía este paciente padecerá de esteatorrea, esto se explica porque …..………..
(unidad 4, sesión 26, logro 4: Explica las alteraciones en la Absorción de lípidos)
cc. se inhibe la acción de la 7 alfa hidroxilasa
dd. el pool de ácidos biliares se incrementa
ee. hay mala absorción de ácidos biliares
ff. el páncreas no secreta lipasa

239. En un experimento se inserta un balón en el estómago de un voluntario, se infla poco a poco mientras que se
vigilan las presiones intraluminales. Aunque el volumen del balón aumenta considerablemente, las presiones
permanecen constantes. Esta relación volumen-presión se explica por la liberación local de …………..
(Unidad 2, sesión 13, logro 1 Explica la Motilidad gástrica: relajación receptiva)
gg. acetil colina y gastrina
hh. norepinefrina y óxido nítrico
ii. colecistoquinina y óxido nítrico
jj. óxido nítrico y péptido inhibidor vasoactivo

240. ¿Cuál de las siguientes alternativas es una característica de la secreción exocrina del páncreas?
(Unidad 3, sesión 23, logro 5: Secreción pancreática: formación del jugo pancreático, influencia de la velocidad de
flujo y regulación)
kk. Es hipotónica respecto al plasma
ll. Su mayor estímulo se da en la fase intestinal
mm. Es estimulada por la presencia de bicarbonato en el duodeno
nn. La secreción enzimática es estimulada principalmente por la secretina
241. Las estructuras en el hígado que permite que los productos metabólicos unidos a proteínas tengan acceso a las
membranas basolaterales de los hepatocitos, son…..
(Unidad 3, sesión 21, logro 4-5: Explica la Organización micro estructural del hígado)
oo. los canalículos
pp. las células de Ito
qq. las fenestras sinusoidales
rr. las uniones intercelulares herméticas

242. La composición de la bilis es modificada conforme fluye por los conductillos biliares. Durante este tránsito se
espera que aumente la concentración de……. ​(Unidad 3, sesión 22, logro 2: Describe la Secreción biliar, visión general
del sistema biliar extrahepático y composición de la bilis)
ss. Ig A
tt. Glucosa
uu. Protones
vv. Vitamina A

243. Se mide experimentalmente el contenido gástrico de dos personas. La persona “A” tiene alto contenido de grasa y la
persona “B” tiene un contenido isotónico ¿Cuál de las siguientes es correcto respecto al vaciamiento gástrico? ​(Unidad 2,
sesión 13, logro 2: Describe la Motilidad y vaciamiento gástrico)
ww. Hay ralentización del vaciado gástrico solo en “A”
xx. El vaciamiento gástrico es más rápido en ambos
yy. Hay ralentización del vaciado gástrico solo en “B”
zz. Hay ralentización del vaciado gástrico en ambos casos

244. El examen endoscópico de un paciente con hipertensión portal grave revela venas tortuosas que sobresalen hacia la luz
del esófago. El paciente recibe tratamiento quirúrgico mediante la colocación de una derivación que conecta la vena porta a
la vena cava. Después de la operación el riesgo de encefalopatíay el riesgo
de sangrado de várices ……………..​(Unidad 3, sesión 22, logro 5: Describe la Insuficiencia hepática, encefalopatía hepática e
hipertensión portal)
aaa. disminuirá / disminuirá
bbb. disminuirá / aumentará
ccc. aumentará / disminuirá
ddd. aumentará / aumentará

245. Un paciente varón de 18 años de edad acude al médico para sus exámenes de rutina. Sus resultados de laboratorio
muestran un valor de bilirrubina sérica de 4 mg/dl y una bilirrubina directa de 0,3 mg/dl. Las pruebas de función hepática
son normales. La alteración que explica mejor este caso es por la deficiencia de ………………..
(Unidad 3, sesión 22, logro 3: Explica la Producción y excreción de bilirrubina. Tipos de bilirrubina e ictericia)
eee.transaminasas
fff. hemo oxigenasa
ggg. la 7 alfa hidroxilasa
hhh. glucuronil transferasa

246. Un hombre de 57 años de edad es llevado a urgencias con hematemesis masiva rojo brillante, a su llegada se halla
inconsciente con PA: 80/40 mm Hg y FC: 124 lat/min. Luce ictérico con presencia de “arañas vasculares en el tórax anterior y
extremidades”, abdomen distendido con signo de oleada positiva. Se encuentra esplenomegalia y pérdida de la masa
muscular en extremidades. La anastomosis vascular responsable del sangrado en este paciente es ………….…..
(Unidad 3, sesión 21, logro 2: Describe las anastomosis porto sistémicas)
iii. vena gástrica izquierda y vena ácigos
jjj. arteria gástrica izquierda y vena ácigos
kkk. vena paraumbilical y vena epigástrica inferior
lll. vena gástrica izquierda y vena esofágica superior
247. Un estudiante de medicina está comiendo un plato de comida a base de champiñones, espárrago y salsa de soya. El
estímulo del sabor umami contenido en todos estos alimentos viaja a través del nervio………………..
(Unidad 2, sesión 10, logro 3: Describe la irrigación e inervación de la lengua)
mmm. Lingual
nnn. Hipogloso
ooo. Glosofaringeo
ppp. Cuerda del tímpano
248. Una paciente de 30 años de edad es sometida a una cirugía en oído medio derecho por un problema de
otoesclerosis. Luego de la cirugía refiere alteración sensitiva de la lengua. Al evaluar el caso usted esperaría
encontrar……….
(Unidad 2, sesión 10, logro 5: Describe la Irrigación e inervación de la lengua)
qqq. Alteración en la sensación del dolor y temperatura en el tercio posterior de la lengua
rrr. Alteración en la sensación del dolor en los dos tercios anteriores de la lengua
sss. Alteración en la sensación del gusto en el tercio posterior de la lengua
ttt. Sensación del dolor, tacto y temperatura conservadas

249. En un paciente de 45 años de edad con colestasis biliar, se encuentra una elevación de los niveles sanguíneos de
fosfatasa alcalina hasta 3 veces la cifra normal. ¿Cuál de las siguientes alternativas estará también elevada como evidencia
del daño de la vía biliar?
(Unidad 3, sesión 22, logro 5: Pruebas de función hepática, Insuficiencia hepática, encefalopatía hepática e
hipertensión portal)
uuu. Tiempo de protrombina y albúmina sérica
vvv. Transaminasas hepáticas (ALT y AST)
www. Gamma glutamil transpeptidasa
xxx. Glucoronil transferasa

250. Experimentalmente se incrementa la velocidad de la secreción salival con una sustancia, en el análisis de la
composición de esta saliva obtenida se espera encontrar…………..
(Unidad 3, sesión 17, logro 5 : Explica la Influencia de la velocidad del flujo salival en la composición de la saliva)
yyy. disminución de la concentración de bicarbonato que supera la concentración plasmática
zzz. aumento de la concentración de cloro y sodio que supera la concentración plasmática
aaaa. aumento de la concentración de bicarbonato que supera la concentración plasmática
bbbb. disminución de concentración de potasio y bicarbonato

251. Lactante de 3 meses de vida es atendido por presentar diarrea, se administra una solución de glucosa y
electrólitos por vía oral. La proteína de membrana apical que explica la capacidad de esta solución para
proporcionar aporte de glucosa e hidratación es ………..
(Unidad 4, sesión 26, logro 1: Explica la Digestión y Absorción de los hidratos de carbono. Alteraciones)
cccc. CFTR
dddd. SGLT-1
eeee. GLUT-2
ffff. GLUT-5

252. Paciente ha sufrido herida de bala en el abdomen, se le ha tenido que extirpar el segmento medio y distal del
ileon. En este caso la síntesis hepática de sales biliares estará …..…..
(Unidad 3, sesión 22, logro 4: Explica la formación, función y Circulación entero hepática de lasa sales biliares)
gggg. Sin cambios en el ritmo de síntesis
hhhh. Disminuida por inhibición de la enzima colesterol 7 alfa hidroxilasa
iiii. Incrementada por estímulo de la enzima colesterol 7 alfa hidroxilasa
jjjj. Incrementada por inhibición de la enzima colesterol 7 alfa hidroxilasa

253.Un varón de 75 años ingresa al consultorio por presentar ictericia marcada de piel y las escleras. El estudio del
paciente mostró que presentaba un tumor que obstruía la totalidad del conducto hepático común. ¿Cuál de los siguientes
conductos se encontrará dilatado en este paciente?
(Unidad 3, sesión 21, logro 6: Describir el árbol biliar intrahepático)
kkkk. de Wirsung
llll. de Hering
mmmm. colédoco
nnnn. cístico
254. Correlaciones las dos columnas y marque la fórmula correcta: ​(Unidad 4, sesión 28, logro 1: Diarrea: definición, mecanismos:
osmótica, secretoria y exudativa)

1. Enfermedad Hirschsprung( ) heces con moco y sangre


2. Diarrea osmótica( ) intolerancia a lactosa
3. Diarrea secretoria( ) aganglionosis congénita
4. Diarrea exudativa( ) canales de Cl- en las células de la cripta a.- 4231b.- 1234c.- 2143​d.-

4213

255. La vena umbilical obliterada del hígado después del nacimiento se transforma en el ligamento:
(Unidad 3, sesión 21, logro 1: Hígado: relación con la pared abdominal, caras, lóbulos, ligamentos , hilio hepático)
oooo. cruzado
pppp. redondo
qqqq. coronario
rrrr. falciforme

256. Llega a su guardia nocturna una madre que trae a su RN masculino de 2 semanas de vida con mal estado general y
sequedad de mucosas. Usted observa que lacta ávidamente, pero a las 2 horas presenta vómito postprandial no bilioso en
proyectil. Al realizar la historia clínica, descubre que el lactante recibió profilaxis con macrólidos para tos ferina. Usted
sospecha principalmente en:
(Unidad 1, sesión 4, logro 4: Desarrollo y anomalías del intestino anterior)
ssss. estenosis pilórica hipertrófica congénita
tttt. fistula traqueo esofágica
uuuu. estenosis duodenal
vvvv. atresia duodenal

257. En la regulación del apetito y la saciedad, la estimulación experimental crónica del núcleo ventro medial del
hipotálamo producirá:
(Unidad 1, sesión 3, logro 4: explica los mecanismos de control del apetito y saciedad)
wwww. afagia
xxxx. obesidad
yyyy. hiperfagia
zzzz. activación de neuronas relacionadas a NPY

258. Paciente mujer de 25 años acude por dolor en fosa ilíaca derecha que empeora al toser o caminar, asociada a náuseas y
vómitos por lo cual acude a emergencia. Dos días después de realizarle una apendicectomía, la paciente desarrolla fiebre
alta (39 °C), está hipotensa y presenta dolor abdominal. La laparotomía exploratoria muestra un gran volumen de sangre en
la cavidad peritoneal por lesión de un vaso producida durante la apendicectomía.
¿Cuál de las siguientes arterias debe ligarse para detener la hemorragia?
(Unidad 4, sesión 27, logro 4: Irrigación arterial del colon, recto y conducto anal)
aaaaa. cólica derecha y arteria rectal superior.
bbbbb. ileocólica y arteria cólica media.
ccccc. mesentérica superior.
ddddd. ileocólica.

289. La onda peristáltica secundaria del esófago se caracteriza por ser originada ………
(unidad 2, sesión 12, logro 3: Motilidad esofágica: fases y características)
eeeee. por el plexo de meissner del esófago
fffff. por el plexo mientérico del esófago
ggggg. por el reflejo de la deglución
hhhhh. durante la masticación

290. ¿Cuál de los siguientes es una causa de ictericia con bilirrubina conjugada aumentada?
(Unidad 3, sesión 22, logro 3: Producción y excreción de bilirrubina. Tipos de bilirrubina, ictericia)
iiiii.Ictericia del recién nacido
jjjjj. Obstrucción del colédoco
kkkkk. Anemia hemolítica
lllll.Gran hematoma
291. En relación a la absorción de nutrientes, la absorción de dipéptidos y tripéptidos a nivel de las células epiteliales del
intestino delgado, se da principalmente debido a:
(Unidad 4, sesión 26, logro 2: Digestión y absorción de las proteínas. Alteraciones)
mmmmm. el incremento de los canales de Cl- en la membrana apical
nnnnn. la gradiente de bicarbonato en la membrana basal
ooooo. la gradiente de iones H+ en la membrana apical
ppppp. la gradiente de Na+ en la membrana apical

292. Paciente de 20 años es traído a la emergencia por presentar diarreas desde hace 2 días. Familiar refiere que las
deposiciones son líquidas y abundantes, al examen luce deshidratado y se plantea que la diarrea es producida por una
toxina que estimula la transformación de ATP a AMPc con apertura de canales de Cl- y pérdida de agua. El tipo de diarrea
más probable es:
(Unidad 4, sesión 28, logro 1: Diarrea: definición , mecanismos: osmótica, secretoria y supurativa)
qqqqq. osmótica
rrrrr. exudativa
sssss. secretoria
ttttt. por intolerancia a lactosa
293. Un niño fue operado por una obstrucción intestinal, observándose la presencia de divertículo de Meckel. Según lo
referido, marque lo correcto:
(Unidad 1, sesión 5, logro 2: identifica las anomalías del desarrollo del intestino medio: onfalocele y gastrosquisis
(diferencias), Divertículo de Meckel)
uuuuu. el 50% de la población lo presenta
vvvvv. se localiza en el íleon muy cerca al yeyuno
wwwww. puede poseer tejido gástrico o pancreático
xxxxx. se produce por una mala rotación de los intestinos

294. Marque la alternativa correcta respecto a la


estructura marcada en el gráfico:
(Unidad 3, sesión 22, logro 2: Secreción biliar.
Visión general del sistema biliar extrahepático y
composición de la bilis)

yyyyy. Se halla a 2 centímetros debajo


de la papila duodenal mayor
zzzzz. Llega el conducto colédoco y
pancreático principal
aaaaaa. Llega el conducto hepático
común y pancreático principal
bbbbbb. Llega el conducto pancreático accesorio

295. ¿Cuál de las siguientes moléculas se encontrará aumentada en el citoplasma de las células parietales de un
paciente con sindrome de Zollinguer Ellison? ​(Unidad 3, sesión 20, logro 4: Enfermedad ulcerosa péptica: úlcera
gástrica, duodenal. síndrome de Zollinger – Ellison)
cccccc. Péptido liberador de gastrina (GRP)
dddddd. Proteína G estimulante (GS)
eeeeee. Inositol Trifosfato (IP3)
ffffff. AMP cíclico (AMPc)

296. Los fármacos inhibidores de la bomba de protones, actúan bloqueando la ………..…….. ​(Unidad 3, sesión 20, logro 3:
Regulación de la secreción gástrica: inhibición, Secreción de pepsinógeno y factor intrínseco)
gggggg. anhidrasa carbónica
hhhhhh. ATPasa H+/K+ en la membrana luminal
iiiiii. ATPasa H+/K+ en la membrana basolateral
jjjjjj. ATPasa Na+/K+ en la membrana basolateral
297. Un paciente fue diagnosticado de gastritis autoinmune, ¿cuál de las siguientes alternativas es ​FALSA ​respecto a esta
enfermedad? ​(Unidad 3, sesión 20, logro 5: Gastritis crónica: helicobacter pylori, autoinmune. Tipos de gastritis)
kkkkkk. Afecta principalmente el fondo y cuerpo gástrico
llllll. Se produce hiperplasia de células G secundaria a la aclorhidria
mmmmmm. El propio sistema inmune destruye principalmente las células parietales
nnnnnn. Se produce atrofia de la mucosa, aclorhidria, hipergastrinemia y déficit de vitamina B6
298.​Marque la correlación correcta: ​(Unidad 3, sesión:18, logros:1 y 2: Describe las enfermedades inflamatorias/infecciosas y
proliferativas de la cavidad oral)

1. Herpes virus( ) En relación al abuso de antibióticos


2. Candidiasis oral( ) Lesiones vesiculares como racimo de uvas
3. Eritroplaquia( ) Mega esófago
4. Enfermedad de Chagas( ) Lesión pre cancerígena

a.- 2431b.- 1234c.- 4123 ​d.- 2143

299. En un paciente con insuficiencia renal crónica, el déficit en la absorción de calcio a nivel del enterocito se debe a lo
siguiente: ​(Unidad 4, sesión 26, logro 6: Explica la Absorción de calcio y hierro)
oooooo. No se convierte la 25 hidroxicolecalciferol a 1,25 dihidroxicolecalciferol
pppppp. No se convierte la 1,25 dihidroxicolecalciferol a 25 hidroxicolecalciferol
qqqqqq. Existe un descenso de la alfa 25 hidroxilasa renal
rrrrrr. Se incrementa la producción de Calbindina
300. ¿Cuál de las glándulas salivales es responsable del mayor porcentaje del volumen de la saliva en condiciones basales?
a. Parótida
b. Sub Palatinas c.
Sublinguales d.
Submaxilares

301. La lengua está recubierta por epitelio:


c. pseudoestratificado columnar no queratinizado
d. plano estratificado no queratinizado
e. pseudoestratificado columnar ciliado
f. plano estratificado queratinizado

302. El esfínter anal interno tiene musculatura …………… y tiene control ………………….
g. lisa / voluntario
h. lisa / involuntario
i. esquelética / simpático
j. esquelética / parasimpático

303. La arteria aorta proporciona la irrigación al tubo digestivo ¿cuál de las siguientes arterias proporciona la irrigación al ángulo
cólico derecho?
k. Mesentérica superior
l. Mesentérica inferior
m. Frénica inferior
n. Tronco celiaco

304. Paciente de 26 años que le cuenta en su historia clínica que cada vez que almuerza, a los 20 minutos tiene deseo de
defecar. Le comenta que su hijo de 1 mes le pasa lo mismo pero más intenso. Esto se explica por el reflejo
…………………, el cual estáen el paciente.
o. colicoileal / normal
p. colicoileal / alterado
q. gastrocolico / normal
r. gastrocolico / alterado
305. La región del estómago que se comunica con el duodeno se denomina:
s. pilórica
t. cardias
u. cuerpo
v. fórnix
306. Acude a consulta un paciente que fue diagnosticado de ulcera péptica 3 días antes. Luego de múltiples pruebas diagnósticas,
se concluye que el paciente presenta un tumor secretor de gastrina ¿Cuál de las siguientes situaciones estará incrementada?
w. Distención gástrica
x. Inhibición del vaciado gástrico
y. Secreción de ácido clorhídrico (HCl)
z. Inhibición de la secreción de pepsinógeno
307. En el sistema digestivo, el control del apetito esta dado por un complejo sistema de sustancias y órganos integradores los
cuales regulan la ingesta de alimentos. Laes una sustancia oroxígena y es sintetizada
por el ……………………..
aa. leptina / intestino
bb. grelina / intestino
cc. leptina / estómago
dd. grelina / estómago
308. Sobre el control autónomo del sistema digestivo, marque la alternativa correcta:
ee. La inervación dada por el sistema simpático es de tipo preganglionar.
ff. El sistema parasimpático usa como neurotransmisores a la acetilcolina y la noradrenalina.
gg. El nervio vago (par craneal X) le da inervación simpática a la mayoría del sistema digestivo.
hh. En el sistema simpático, los nervios responsables hacen una primera sinapsis en ganglios próximos al órgano a
inervar.
ii. En la inervación de tipo parasimpático, solo interviene el plexo submucoso, sin embargo, en la de tipo
simpático intervienen tanto el submucoso como el mientérico.
309. Con respecto a la actividad eléctrica del sistema digestivo, marque la alternativa correcta
jj. Corresponden a potenciales de acción que están presentes de forma continua y le dan la capacidad de
perístasis autónoma al sistema digestivo.
kk. La frecuencia de las ondas lentas no se ve influenciada por la actividad neural ni las hormas
gastrointestinales.
ll. En el estómago las ondas lentas se dan en una frecuencia de 6 por minuto.
mm. Las ondas lentas son cambios lentos y ondulantes del potencial en reposo.
nn. La frecuencia de las ondas lentas va de 6 a 12 ondas por minuto.

310. Ante una lesión del IX par craneal, el músculo…se altera en su función.
oo. palatogloso
pp. estilofaríngeo
qq. palatofaríngeo
rr. constrictor superior

311. Un varón de 50 años es sometido a extirpación de duodeno y parte proximal de yeyuno. La pérdida de estímulo hormonal
en el páncreas para la secreción enzimática se explica por la pérdida de las células ……………………
ss. Parietales, productoras de factor intrínseco
tt. “K” productoras de factor intrínseco
uu. “M” productoras de CCK
vv. “I” productoras de CCK
312. ​Respecto al mecanismo de la defecación ¿Cuál de las siguientes afirmaciones es correcta?
ww. Se produce contracción refleja del esfínter anal interno
xx. Se produce contracción o relajación del esfínter anal externo por señales de la corteza cerebral
yy. La presencia de materia fecal en el recto estimula la contracción del sigmoides por los nervios pélvicos
simpáticos
zz. En la posición de “cuclillas” el músculo puborectal se halla contraído favoreciendo la evacuación de la materia
fecal

313. Un niño de tres años llega a emergencia con disfagia (dificultad para tragar), salivación y llanto. Se sospecha de ingesta de
cuerpo extraño: moneda en el esófago; al ser evaluado se constata en una radiografía presencia de cuerpo extraño a nivel de C6 y
C7 (6° y 7° vértebra cervical). El cuerpo extraño estará suspendido a nivel del estrechamiento producido por el ………..
aaa. cayado aórtico
bbb. hiato esofágico
ccc. músculo cricofaríngeo
ddd. bronquio principal izquierdo

314. En el caso de un paciente con un tumor productor de gastrina, la presencia de úlceras duodenales y erosión de la mucosa
gástrica se debe principalmente a…….
eee.la acción paracrina de la gastrina sobre la célula parietal
fff. el exceso de HCl por estímulo de receptores CCK-B en la célula parietal
ggg. la sobre expresión de los receptores “G” para gastrina en la célula parietal
hhh. el exceso de HCl por estímulo directo de receptores “H” en la célula parietal

315. La onda peristáltica secundaria del esófago se caracteriza por ser originada ………
iii. por el plexo de submucoso del esófago
jjj. por el plexo mientérico del esófago
kkk. por el reflejo de la deglución
lll. durante la masticación

316. Marque lo correcto sobre las ondas lentas en el tubo digestivo


mmm. No son despolarizaciones
nnn. Son potenciales de acción subumbrales
ooo. Se constituyen de despolarizaciones y repolarizaciones
ppp. Son rítmicas y generadas por el sistema nerviosos autónomo

317. Recién nacido que presenta protrusión de contenidos abdominales los cuales no están cubiertos por peritoneo y salen de la
cavidad abdominal a través de un defecto de la pared. ¿Cómo se denomina a la afección que presenta este paciente?
qqq. Onfalocele
rrr. Atresia biliar
sss. Gastrosquisis
ttt. Divertículo de Meckel

318. Experimentalmente se utiliza atropina (anticolinérgico) para inhibir la secreción de gastrina, sin embargo, la secreción de
esta hormona se sigue dando ante estímulos vagales. Esta situación se explica porque la atropina:
uuu. no bloquea la acción del péptido GRP
vvv. solo inhibe la acción del péptido GRP en la célula G
www. inhibe la acción de acetilcolina e histamina en la célula G
xxx. bloquea parcialmente la bomba de protones en la célula G
319. Niña de 4 días es llevada a la emergencia pediátrica por presentar llanto constante, la madre refiere coloración azulada de
labios al momento de lactar, acompañado de tos persistente y dificultad respiratoria así como distención abdominal. Se le coloca
sonda nasogástrica para alimentación notando que retorna a la cavidad oral en todos los intentos. ¿Cuál es la anomalía del
desarrollo en este caso?
yyy. Solo fístula traqueo esofágica
zzz. Fístula traqueo esofágica proximal y distal
aaaa. Atresia esofágica proximal con fístula traqueo esofágica distal
bbbb. Atresia esofágica distal con fístula traqueo esofágica proximal
320. Paciente varón de 36 años es traído a la emergencia luego de sufrir un accidente de tránsito, presenta traumatismos múltiples
en cabeza y tronco. Al examen físico se evidencia hematoma en hemicara izquierda, ligera protrusión y caída del lado izquierdo del
maxilar inferior, por lo que se realiza una tomografía donde se halla una fractura de la apófisis coronoides del maxilar inferior.
¿Qué músculo está relacionada directamente con esta situación?
cccc. Masetero
dddd. Temporal
eeee. Buccinador
ffff. Pterigoideo medial

321. Un paciente refiere no percibir algunos sabores. Al examen físico constata alteración del sabor dulce y umami.
¿Cuál de los siguientes nervios estará alterada su función?
gggg. Cuerda del tímpano (VII par)
hhhh. Lingual (rama del V par)
iiii. Glosofaríngeo (IX par)
jjjj. Hipogloso (XII par)

322. A los pocos días de nacido, regresa a neonatología un niño con problemas de motilidad del colon; los estudios determinan
ausencia congénita de células ganglionares. Según el gráfico ¿cuál es la capa en la que se determina la ausencia de dichas células?
kkkk.Mucosa - 2
llll. Muscular propia - 2
mmmm.Muscular propia - 3
nnnn. Muscular de la mucosa - 3

323. Con respecto al control autonómico en el tracto gastrointestinal y en relación a su fisiología. ¿Cuál es la función del
sistema nervioso parasimpático en el tracto gastrointestinal?
oooo. Inhiben la contracción muscular y estimulan la secreción de sustancias a nivel de la submucosa
pppp. Estimulan la contracción muscular y estimulan la secreción de sustancias a nivel de la mucosa
qqqq. Inhiben la contracción muscular e inhiben la secreción de sustancias a nivel de la submucosa
rrrr. Estimulan la contracción muscular e inhiben la secreción de sustancias a nivel de la mucosa

324. Un estudiante que está preocupado por su examen parcial, no ha desayunado ni almorzado; cuando al fin ingiere alimentos,
esto le provoca el aumento de los movimientos musculares del tracto gastrointestinal y la sensación de defecar. ¿Qué reflejo se
ha activado?
ssss. Entero-gástrico
tttt. Gastro-cólico
uuuu. Cólico-ileal
vvvv. Ileo-ileal
325. ¿De qué par craneal es rama el nervio palatino mayor?
wwww. Vago
xxxx. Hipogloso
yyyy. Trigémino
zzzz. Palatogloso

326. ¿En cuál de las fases de la deglución la epiglotis separa la vía respiratoria de la digestiva?
aaaaa. oral
bbbbb. laríngea
ccccc. faríngea
ddddd. esofágica

327. Los péptidos intestinales se pueden clasificar como sustancias endocrinas, neurocrinas y paracrinas, dentro de las
paracrinas se encuentran la somastotatina e histamina. Marque la respuesta correcta
eeeee. La somastotatina es sintetizada por las células B de la mucosa gástrica
fffff.La histamina actúa estimulando su receptor tipo H1 en la mucosa gástrica
ggggg. La histamina es sintetizada por células de tipo paracrino de las glándulas gástricas
hhhhh. La somatostatina presenta dentro de sus funciones la estimulación de la secreción de H​+

328. En relación a los órganos intraabdominales y sus estructuras de fijación, elija la alternativa correcta a.El
mesenterio permite la suspensión e irrigación de los órganos retroperitoneales
b. Tanto el hígado como la vesícula biliar se encuentran ubicados a nivel del flanco derecho
c. El colon, el duodeno y el resto de intestino delgado son órganos considerados netamente como
peritoneales
d. Los ligamentos que encontramos dentro de la cavidad abdominal son el esplenorenal y el gastrofrénico e.Los
omentos van desde el estómago y la segunda porción del duodeno a otras estructuras
intraabdominales y existen dos: el omento mayor y el omento menor

329. Paciente varón de 27 años es llevado por bomberos a emergencia luego de ser asaltado y, tras resistirse, es cortado con el
pico de una botella a nivel abdominal. Al examen físico usted observa que a través de la herida se puede observar la protrusión
de asas intestinales. En relación con las capas de la pared abdominal, marque la alternativa correcta.
iiiii. La fascia de Scarpa está constituida principalmente por tejido adiposo
jjjjj. La pared abdominal está formada por piel, huesos, músculos, fascias y peritoneo parietal
kkkkk. La fascia de Camper es una estructura fibrosa que carece de grasa y su grosor es constante en toda la
pared abdominal
lllll. El músculo oblicuo externo discurre en dirección súpero-interna y se inserta en el borde inferior de las ultimas 3
a 4 costillas
mmmmm. El músculo recto del abdomen tiene como funciones comprimir el contenido del abdomen,
tensar la pared del abdomen y flexionar la columna

330. La explicación fisiológica de presentar somnolencia de 30 minutos a 1 hora después de ingerir alimentos, se explica
por:
a. Aumento del cloro intraluminal
b. Aumento del bicarbonato intraluminal
c. Disminución de ácido carbónico en la célula parietal
d. Disminución de la actividad de la anhidrasa carbónica
e. Aumento de la alcalinidad sanguínea

331. Con respecto a la irrigación arterial del colon, a que arteria corresponde la señalada con la
flecha
f. Cólica derecha
g. Cólica media
h. Cólica izquierda
i. Ileobisecoapendículocólica
j. Arco de Riolano

332. Si un paciente presentara dentro del punto de vista fisiológico, una disminución de
enterocinasa, entonces esto originaría una disminución de la actividad de:
k. la pepsina​ b.la lipasa
c. la quimotripsina
d. el peptido insulinotropo dependiente de glucosa
e. la amilasa

Se valida la opción b debido a su relación con la colipasa.


333. Con respecto a la anatomía del hígado, señale a que estructura pertenece
la marcada por el número 1.
l. Ligamento falciforme
m. Línea de Cantlie
n. Ligamento triangular
o. Ligamento coronario
p. Ligamento teres

Se valida la opción a debido a la ubicación del número 1 en donde se unen el


ligamento falciforme y ligamento coronario.

334. Se presenta un paciente, el cual presenta un antecedente de tuberculosis intestinal, por lo cual, se le resecó 80
cm de íleon distal. Desde el punto de vista fisiológico, el paciente puede presentar una de las siguientes alteraciones:
q. Disminución de la secreción de Vitamina B12
r. Aumento indiscriminado de absorción de ácido fólico
s. Disminución de la absorción de hierro
t. Aumento de la secreción de bicarbonato
u. Disminución de la absorción de ácido glicocólico

335. Un paciente es sometido experimentalmente a un fármaco que modifica el flujo salival, obteniéndose un volumen
de saliva de 288 ml en 6 horas. En este caso las concentraciones de electrolitos y bicarbonato en la saliva obtenida
varían de la siguiente manera:
a.↑ Na+, ↓ K+, ↑ Cl-, ↑ HCO3-
b.↓ Na+, ↓ Cl-, ↑ K+, ↓ HCO3-
c.↑ Na+, ↑ Cl-, ↓ K+, ↓ HCO3-
d.↑ Na+, ↑ Cl-, ↑ K+, ↑ HCO3-
e.↓ Na+, ↓ Cl-, ↓ K+, ↓ HCO3-

Se valida la opción a debido a que se puede considerar como un aumento del flujo de saliva.

336. La siguiente imagen histológica corresponde a la glándula


…………… y la estructura señalada produce ………
v. salival sublingual / mucopolisacáridos
w. oxíntica / pepsinógeno
x. salival submaxilar / ptialina
y. salival parótida / amilasas
z. antrales / gastrina

337. Paciente varón de 65 años con antecedente de hipercolesterolemia, hipertensión arterial, fibrilación auricular y
dos infartos al miocardio previos, aqueja de dolor abdominal intenso de inicio súbito, distensión abdominal, se decide
cirugía con resección de 1,5 metros de intestino delgado terminal y colon ascendente. Como consecuencia de la
resección el paciente tendrá deficiencia de:
aa. Vitamina C
bb. Tiamina
cc. Vitamina A
dd. Vitamina B1
ee. Vitamina B6

Se valida esta opción debido a que su absorción está relacionada al íleon.


338. Uno de los siguientes elementos debería hallarse con más probabilidad en el esófago de un paciente que sufre de
reflujo gastro esofágico…
ff. Pepsina
gg. Tripsina
hh. Quimiotripsina
ii. Carboxipeptidasa
jj. Ácidos biliares

339. Un paciente de 40 años cursa con anemia de 8g/dl, aqueja además de astenia y sensación de hormigueo bilateral
en los miembros inferiores, al examen se halla alteración de la sensibilidad a la vibración y camina con ampliación de la
base de sustentación. Uno de los siguientes procedimientos sería de ayuda para el diagnóstico de este paciente:
kk. Tomografía cerebral
ll. Biopsia de la mucosa gástrica
mm. Biopsia de hígado
nn. Examen de sangre oculta en heces
oo. Biopsia de Ileon proximal

340. Paciente de 60 años ingresa por caída hace 1 hora y pequeño hematoma en cuero cabelludo, al examen físico
ampliado se observa ictericia de piel y mucosas generalizada, abdomen blando, se palpa estructura quística no
dolorosa en hipocondrio derecho que corresponde a vesícula biliar (signo de Courvoisier), en los exámenes de
laboratorio se halla niveles bajos en la formación de estercobilinógeno y urobilinógeno en heces, incremento de la
bilirrubina conjugada en la orina, elevación de fosfatasa alcalina y gamma glutamil transpeptidasa séricas. El presente
cuadro puede ser explicado por:
pp. Reabsorción de hematoma
qq. Litiasis vesicular
rr. Carcinoma de la cabeza de páncreas
ss. Carcinoma con estenosis del conducto hepático común
tt. Anemia hemolítica

341. Paciente varón de 58 años con antecedente de alcoholismo crónico es diagnosticado y recibe tratamiento por
cirrosis hepática. Hace 2 días refiere familiar que tuvo cambio de conducta y no reconoce a algunos familiares. Al
examen físico, se halla ascitis, circulación colateral en abdomen, telangiectasias, en el examen de sistema nervioso:
rigidez de extremidades, ROT incrementados, desorientación en el espacio y asterixis. ¿cuál de las siguientes
circunstancias, explicaría el cuadro en este paciente?
uu. Uso de diuréticos ahorradores de potasio
vv. Incremento de actividad de ureasa bacteriana duodenal
ww. Hemorragia gastrointestinal
xx. Disminución de la producción de NH3+ en el colon
yy. Dieta normo proteica

Se valida la opción e debido al efecto sobre la encefalopatía.


Con respecto de la opción b es incorrecta debido a que hace referencia al duodeno, debería indicar colon.

342. Un recién nacido presenta vómitos biliosos poco tiempo después de cada alimento. Al preguntar a la madre sobre
antecedentes, ella recuerda que tuvo polihidramnios durante la gestación, pero un análisis de cariotipo fue normal. Una
de las siguientes es la causa más probable de estos hallazgos en el recién nacido:
zz. Enfermedad de Hirschprung
aaa. Fístula tráqueo esofágica
bbb. Divertículo ileal
ccc. Estenosis pilórica
ddd. Malrotación de la yema pancreática ventral

343. Un lobulillo hepático se puede dividir en tres zonas como se muestra en el gráfico. ¿Cuál de las siguientes
afirmaciones sobre las tres zonas es verdadera?
eee.La zona 1 tiene los menores depósitos de glucógeno
fff. La zona 3 es la primera en afectarse en una colestasis extra hepática
ggg. La zona 2 es más susceptible a la injuria por isquemia que la zona periportal d.La
zona 2 tiene la mayor capacidad de regeneración
e.La zona 1 es la que tiene menos actividad metabólica.

La pregunta 14 ha sido anulada, sin embargo, ningún estudiante se verá afectado negativamente en su
puntaje debido a esta anulación.

344. En un estudio de la secreción de hormonas gastrointestinales, sus concentraciones en la vena porta se midieron
durante perfusión luminal del intestino delgado con soluciones de diversas magnitudes de pH. ¿Qué hormona
aumentará en el plasma de la vena porta durante perfusión a través del intestino con una solución de pH 3?
hhh. CCK
iii. gastrina
jjj. GIP
kkk. motilina
lll. secretina

345. Paciente de 30 años que ingresa a causa de un traumatismo abdominal cerrado. En la exploración se aprecia
discreta palidez de piel y mucosas, auscultación pulmonar normal, taquicardia de 120 /min. Discreta distensión
abdominal y matidez en flancos; el hematocrito, que era prácticamente normal al ingreso, disminuye a 30% a las tres
horas. En la Rx de tórax se objetiva fractura de las costillas 10-11 izquierdas. La causa más probable de la anemización
en este paciente es:
mmm. traumatismo renal con hemorragia retroperitoneal.
nnn. rotura de hígado con hemoperitoneo.
ooo. rotura de bazo con hemoperitoneo​.
ppp. rotura de mesos con hemoperitoneo.
qqq. traumatismo pancreático con pancreatitis traumática.

346. Mujer de 65 años. Consulta por síndrome constitucional asociado a


dolor abdominal epigástrico progresivo irradiado a espalda, de dos meses de
evolución. El diagnostico de sospecha de adenocarcinoma de páncreas se
confirma por biopsia. Se realiza examen de imagen de abdomen para
evaluación de estructuras vasculares próximas al tumor pancreático. ¿Cuál
es el nombre de la vena señalada que está ausente, trombosada por
infiltración tumoral, condicionando circulación colateral en la pared
gástrica?

rrr. Mesentérica superior


sss. Coronaria estomaquica
ttt. Esplénica
uuu. Porta
vvv. Renal izquierda
347. Revisando la angiotomografía de un hombre de 70 años en estudio por aneurisma de aorta abdominal, el
radiólogo le informa de la presencia de una oclusión completa de la arteria mesentérica inferior. El paciente se
encuentra completamente asintomático. La oclusión de la arteria mesentérica inferior cursa de manera
asintomática en muchas ocasiones ya que el territorio que irriga puede recibir flujo proveniente de la arteria:
www. cólica derecha
xxx. gastroduodenal
yyy. Epigástrica inferior izquierda
zzz. esplénica
aaaa. cólica media

348. En las patologías de esófago es importante conocer bien la anatomía esofágica. ¿Cuál de las siguientes
afirmaciones es correcta?
bbbb. El esófago tiene capa mucosa, muscular y serosa
cccc. El esófago abdominal es más largo que el cervical
dddd. El esófago torácico pasa por detrás del cayado aórtico
eeee. El epitelio esofágico normal es ​de tipo ​cilíndrico.
ffff. El esófago abdominal es discretamente más largo que el torácico

349. A pesar de que pueda haber variaciones anatómicas, lo habitual es que el ciego sea irrigado por una rama arterial
que proviene de unas de las siguientes arterias:
gggg. Iliaca derecha
hhhh. Mesentérica inferior
iiii. Hepática derecha
jjjj. Mesentérica superior
kkkk. Iliaca izquierda

350. Ante un paciente con una cirugía abdominal urgente, el informe operatorio señala que se ha realizado una
resección de todo el duodeno y del tercio proximal del yeyuno manteniendo íntegros el estómago y todo el íleon, así
como los dos tercios distales del yeyuno. En el seguimiento nutricional del paciente ¿Qué vitamina o mineral
presentará con menor probabilidad una disminución de su absorción?
llll. Cianocobalamina
mmmm. Calcio
nnnn. Hierro
oooo. Transcobalamina
pppp. Transferrina

351. ¿Cuál de las siguientes alternativas detallan las venas que confluyen y forman la vena señalada?
qqqq. mesentérica superior, gástrica izquierda y
gastroepiploica izquierda
rrrr. mesentérica inferior, gástrica izquierda y renal
ssss. esplénica, mesentérica superior y mesentérica inferior
tttt. esplénica, pancreatoduodenal y omental izquierda
uuuu. gástrica izquierda, esplénica y hepática común

352. ¿Cuál de las siguientes sustancias forma parte de la secreción biliar?


vvvv. Tripsina
wwww. Lecitina
xxxx. Elastasa
yyyy. Quimotripsina
zzzz. Pepsina
353. El tubo digestivo contiene diferentes tipos de epitelios y glándulas. La estructura señalada es unay
está localizada en el …………...
aaaaa. glándula de Brunner / intestino
grueso​ b.cripta de Lieberkuhn / colon
c. cripta de Lieberkuhn / intestino delgado
d. glándula oxintica / estomago
e. célula parietal / estómago

Aunque las criptas de Lieberkuhn están


presentes en el intestino delgado, la
microfotografía es de epitelio de
colon.

354. ¿De qué musculo forma parte el ligamento inguinal?


bbbbb. Oblicuo externo del abdomen
ccccc. Oblicuo interno del abdomen
ddddd. Transverso del abdomen
eeeee. Psoas
fffff.Dorsal ancho

355. Señale cuál de las siguientes afirmaciones ​NO ​se relaciona a la siguiente glándula anexa del tubo digestivo
mostrada en la imagen:
ggggg. Es una glándula exocrina compuesta exclusivamente por acinos serosos
hhhhh. Su inervación está dada por el nervio auricular mayor (ramo posterior C2), que inerva la vaina
de la glándula así como la piel por encima de esta.
iiiii. Esta glándula produce una secreción mucinosa acuosa, llamada mucoserosa, a través del conducto de
Wharton.
jjjjj. Su inflamación puede ser causada por un virus de los Paramyxoviridae, que provocan una
enfermedad muy frecuentemente en niños y adolescentes
kkkkk. Es una glándula endocrina y probablemente sea de origen pancreático

Se valida la opción e debido a que no está relacionada


con la imagen.

356. ¿Cuál de las siguientes enzimas está localizada en el borde en cepillo y juega un rol en la digestión de proteínas?
lllll. Alfa dextrinasa
mmmmm. Pepsina
nnnnn. Enterocinasa
ooooo. Lactasa
ppppp. Carboxipeptidasa A​.

Se valida la opción c debido a que es correcta en relación a la pregunta.


357. Una de los siguientes sustancias, ​NO s​ irve como un buen agente emulsificante:
qqqqq. Colesterol
rrrrr. Ácidos grasos
sssss. Sales biliares
ttttt. Lecitina
uuuuu. Proteínas de la dieta

Se valida la opción e debido a que es correcta en relación a la pregunta.

358. La sustancia que estimula el crecimiento de la mucosa gástrica es:


vvvvv. Secretina
wwwww. Motilina
xxxxx. Péptido estimulante de la mucosa gástrica
yyyyy. Gastrina
zzzzz. Histamina

359. ¿Cuál de las siguientes alternativas es una función de la colecistokinina?


aaaaaa. Relajación de la vesícula para la salida de
bilis​ b.Secreción de ácidos biliares
c. Contracción del esfinter de Oddi
d. Secreción de enzimas pancreáticas
e. Contracción del duodeno

Se valida la opción b debido al efecto de la CCK sobre la vesicula biliar.

360. Con respecto a la anatomía del tronco celiaco, señale lo correcto


bbbbbb. El tronco celiaco se origina de la cara posterior de la aorta abdominal
cccccc. Es una arteria delgada que tiene un calibre entre 2 y 3 mm
dddddd. Una de sus ramas es la arteria gástrica derecha
eeeeee. La hepática común que es una de sus ramas, participa en la irrigación del estómago​.

361. Con respecto a la anatomía del duodeno, marque la respuesta correcta:


ffffff. Tiene una distribución en forma de “C”, que rodea la cola del páncreas
gggggg. La 3ra porción duodenal está contenida en la pinza vascular aortomesentérica
hhhhhh. Entre la 1ra y 2da porción se forma un ángulo, conocido como el ángulo de Treitz
iiiiii. La 4ta porción se dirige a la izquierda, hacia abajo y hacia atrás.
jjjjjj. En la tercera porción desemboca el conducto colédoco.

362. El hígado está ampliamente tapizado por peritoneo, la estructura que conecta la cara diafragmática del hígado
precisamente con el diafragma es el ligamento:
kkkkkk. teres
llllll. falciforme
c.triangular
d.hepático común
e.coronario

Se validan la opción c y e debido a que forman parte de los ligamentos que fijan el hígado al diafragma.

363. En el íleon se absorbe aproximadamente el 95% dea través de la circulación enterohepática.


mmmmmm. agua
nnnnnn. colesterol
oooooo. sales biliares
pppppp. hidróxicobalamina
qqqqqq. factor intrínseca
364. Laestimula el mecanismo paracrino de la secreción de ácido clorhídrico.
rrrrrr. histamina
ssssss. acetilcolina
tttttt. gastrina
uuuuuu. secretina
vvvvvv. somatostatina

365. En la digestión de proteinas,es el principal estímulo para convertir el pepsinógeno en pepsina.


wwwwww. la gastrina
xxxxxx. el pH ácido
yyyyyy. la acetilcolina
zzzzzz. la ptialina
aaaaaaa. la somatostatina

366. Con respecto a la somatostatina, marque lo correcto:


bbbbbbb. Es secretada por las células S del intestino
ccccccc. Induce a la producción de VIP
ddddddd. Interviene en la fase intestinal de la secreción gástrica
eeeeeee. Produce acetilcolina para estimular a la célula parietal
fffffff. No interviene en la regulación de la secreción de ácido clorhídico

367. En pecten anal, es una estructura comprendida entre:


ggggggg.la línea pectínea y los senos anales
hhhhhhh. la línea blanca y la apertura anal
iiiiiii. el esfínter anal interno y el externo
jjjjjjj. la línea anocutánea y la línea pectínea
kkkkkkk. la línea blanca y columnas anales

368. ¿Cuál de las siguientes alternativas es una proenzima pancreática?


lllllll. Tripsina
mmmmmmm. Elastasa
nnnnnnn. Quimotripsinógeno
ooooooo. Amilasa
ppppppp. Procarboxipepitidasa C.

369. En la segmentación hepática de Coinaud, el segmento hepático señalado con la flecha, corresponde a :
En la segmentación hepática de Coinaud, la flecha señala el segmentohepático.
qqqqqqq. IV
rrrrrrr. V
sssssss. VI
ttttttt. VII
uuuuuuu. VIII
370. La contracción del músculopermite la eliminación de gases (flatos) sin salida de material fecal;
es el mismo músculo cuya relajación, sobretodo en cuclillas, permite el paso del contenido fecal con menor
esfuerzo durante la defecación.
a) Isquirectal​ b)
Puborrectal
c) Esfínter anal externo
d) Esfínter anal interno

371. Paciente mujer de 54 años se presenta con náuseas, vómitos, estreñimiento, y es diagnosticada de abdomen
agudo quirúrgico; en la cirugía encuentran un vólvulo de ciego. Esta anomalía puede explicarse por::
b) Falta de rotación intestinal
c) Falta de fusión del mesenterio
d) Defecto en la formación de la cloaca
e) Falta de formación del omento mayor

372. Paciente mujer de 23 años con faringitis aguda, toma para el dolor una tableta de paracetamol con un poco de
agua. Durante la deglución, se relaja su esfínter esofágico inferior y el fondo del estómago, mientras el bolo está aún
en el esófago. ¿Qué sustancia provocara con mayor probabilidad la relajación del esfínter esofágico inferior y el fondo
del estómago en esta mujer?
f) Óxido nítrico
g) Sustancia P
h) Histamina
i) Motilina

373. Luego de tres horas dando exámenes, un alumno de medicina comienza a sentir hambre. Esta situación es
probable que sea mediada por la​ ​que es sintetizada por el​ ​:
j) leptina / intestino
k) leptina / estómago
l) grelina​ /​ estómago
m) grelina / tejido adiposo

374. Varón de 72 años, con antecedente de diabetes mellitus tipo 2, que presenta enteropatía diabética caracterizada
por estreñimiento. Este problema puede estar asociado a:
n) deficiencia de óxido nítrico
o) aumento del reflejo gastrocólico
p) disminución de la secreción de colecistocinina (CCK)
q) aumento de la secreción del péptido intestinal vasoactivo (PIV)

375. Varón de 54 años con Diabetes Mellitus tipo 2, es diagnosticado de gastroparesia debido a que presenta sensación
de llenura precoz al comer, y reflujo gastroesofágico. Esta alteración en la relajación receptiva y en el vaciamiento
gástrico lo más probable es que se deba a una alteración en:
r) el nervio vago
s) el ganglio celíaco
t) plexo submucoso
u) nervio hipogástrico

376. Varón de 67 años con tos y disminución de peso asociado a tabaquismo pesado, presenta actualmente disfagia
progresiva a alimentos sólidos. Se considera la presencia de un carcinoma de bronquio izquierdo y por esta razón le
realizan una endoscopía esofágica para descartar la posibilidad de una compresión esofágica por el tumor. Se espera
revisar el esófago en la​ ​estrechez, que está a nivel de la vértebra
v) Tercera estrechez -T6
w) Segunda estrechez - C6
x) Segunda estrechez - T4
y) Tercera estrechez -T10

377. Varón de 34 años con dolor abdominal agudo en flanco derecho que se irradia a fosa ilíaca derecha, es operado y
se encuentra un divertículo intestinal inflamado, ubicado a 93 cm de la válvula ileocecal. El origen de este divertículo
es una falla en la obliteración de:
z) Conducto vitelino
aa) Alantoides
bb) Cloaca
cc) Conducto anorectal
dd) Uraco

378. En un niño menor de dos años con divertículo intestinal, este divertículo tiene su origen en una falla en la
obliteración de:
a)Conducto anorectal
b) Conducto vitelino
c) Alantoides
d) Cloaca
e) Uraco

379. Mujer de 43 años sufre un grave accidente de tránsito y está hospitalizada en coma, es alimentada por vía
intravenosa durante varias semanas. Producto de este tipo de alimentación, se encuentra en la endoscopía atrofia de la
mucosa gastrointestinal. La causa más probable de esta atrofia son los bajos niveles séricos de la hormona:
ee) Colecistocinina
ff) Secretina
c)Gastrina
d) PIV

380. Una mujer de 30 años llega al consultorio porque se queja de dificultades para deglutir, la cual se agravan cada vez
más. Se realiza un estudio manométrico para examinar la generación de presión a lo largo del esófago. Esta prueba
revela que las contracciones como respuesta a la deglución están mal sincronizadas y que la presión en el esfínter
esofágico inferior permanece elevada. El diagnóstico más probable es​ ​producido por niveles bajos de
gg) acalasia / sustancia P​ b)
acalasia / óxido nítrico
c) enfermedad por reflujo gastrointestinal / acetilcolina
d) enfermedad por reflujo gastrointestinal / óxido nítrico

381. Paciente de 2 años, llega a emergencia por haber ingerido una moneda con la que estaba jugando. El lugar más
probable donde puede haberse quedado suspendido este objeto es a nivel del estrechamiento producido a nivel del:
hh) músculo milohiodeo
ii) músculo aritenoideo
jj) músculo cricofaríngeo
kk) constrictor superior de la faringe

382. En una apendicectomía, al realizar la incisión de McBurney en la fosa iliaca derecha, es necesario cortar los
siguientes músculos, de afuera hacia adentro:
ll) Recto – Oblicuo externo – Transverso
mm) Recto – Oblicuo externo – Oblicuo interno
nn) Oblicuo externo – Oblicuo interno – Recto
oo) Oblicuo externo – Oblicuo interno – Transverso

383. Un varón de 90 años que se encuentra postrado en cama, es referido del asilo para endoscopia por dificultad para
deglutir luego de tomar un medicamento para aliviar el dolor la noche anterior. La endoscopía revela que la píldora se
alojó en el esófago y causó una reacción inflamatoria. Lo más probable es que esto haya sido por la producción de
múltiples ondas:
pp) secundarias
qq) primarias
rr) lentas
ss) segmentarias

384. Mujer de 23 años es diagnosticada de bulimia, al examen físico se observa ulceraciones en el segundo y tercero
dedo de la mano derecha. Esto se puede deber al uso continuo de estos dedos para inducir el vómito, mediante la
estimulación del par craneal:
tt) V​ b)
IX
c) X
d) XI

385. Varón de 52 años se presenta por diarrea persistente de seis semanas de duración. En la colonoscopia se observa
un pólipo a nivel del íleon distal. El patólogo informa que se trata de un tumor neuroendócrino, probablemente
originado por las células enterocromafines del intestino. La sustancia que más probablemente esté produciendo este
tumor es:
uu) Serotonina
vv) Insulina
ww)CCK
xx) GIP

386. La fase oclusal de la masticación se realiza con la contracción de los músculos:


yy) digástricos
zz) masetero y temporal
aaa)orbicular y buccinador
bbb) pterigoideo lateral y digástrico

387. Al tomar su café en Starbucks, un estudiante de medicina sufre una quemadura de primer grado en el tercio
anterior de la superficie dorsal de la lengua. La información de dolor es transmitida por el nervio:
ccc) cuerda del tímpano
ddd) glosofarínge
o​ c)lingual
d) facial

388. Paciente es evaluado por faringitis aguda en consultorio externo. El médico de familia le solicita que abra la boca
y saque la lengua. Para realizar la acción de sacar la lengua, es necesario que se contraiga el músculo:
eee)estilogloso​ b)
geniogloso
c) palatogloso
d) transverso de la lengua

389. Paciente con síndrome de


Sjögren, presenta “boca seca”
(disminución de la producción de
saliva) y caries dental, asociada a la
pérdida de la función de tampón de
la saliva. Esta desminerilización del
diente puede comprometer a las
prolongaciones citoplasmáticas
ubicadas en los tubos huecos de la
estructura señalada con la letra:
fff) B
ggg)A
hhh) E
iii) C

390. Mujer de 32 años acude a


consulta por presentar disfagia de
progresión lenta, reflujo
gastroesofágico y vómitos desde hace
3 meses de evolución progresiva. Se le
realiza un estudio radiológico con
contraste en el que se observa
estrechamiento del esfínter esofágico
inferior (imagen). Según sus
conocimientos, este paciente se
beneficiaría con el uso de:
jjj) agonista beta adrenérgico
kkk) agonista alfa adrenérgico
c)análogo de óxido nítrico
d) análogo de Sustancia P
391. Paciente de sexo masculino de 82 años de edad ingresa a emergencia con dolor abdominal agudo y diarreas. Se le
realiza una arteriografía en la que se observa que la arteria aorta tiene un trombo ocluyendo el 95% del flujo, a nivel
del nacimiento de la arteria mesentérica inferior. ¿Cuál de las siguientes arterias podría contribuir a la irrigación
colateral del colon descendente?
a) cólica media
b) sigmoidea
c) rectal superior
d) ileocólica

392. Niño de 5 años presenta dolor esofágico y hematemesis (vómitos hemorrágicos) luego de tragarse una espina de
pescado. En la endoscopía se observa perforación del esófago distal a la cuarta estrechez esofágica. ¿Las ramas de cuál
de las siguientes arterias estarán lesionada con mayor probabilidad?
e) Gástrica izquierda
f) Bronquiales
g) Frénica inferior
h) Tiroidea inferior

393. Al ingerir una cucharada de mantequilla es muy probable que se disminuya la sensación de hambre por medio de
la activación de la vía POMC/CART (POMC=proopiomelanocortina y CART=transcripción regulada de cocaína y
anfetamina), activada directamente por la hormona:
i) colecistoquinina (CCK)
j) insulina
k) grelina
l) secretina

394. Al comer unas papitas fritas con mayonesa, el vaciamiento gástrico disminuye por efecto directo de la hormona:
a)colecistoquinina (CCK)
b) bombesina
c) motilina
d) gastrina

395. Recién nacido de dos horas es diagnosticado de hernia umbilical de 1,5 cm de diámetro; el cirujano pediatra
solicita una tomografía abdominal en donde se evidencia que la hernia umbilical está ocupada por una porción del
tracto gastrointestinal. ¿Qué porción del tracto gastrointestinal estaría ocupando esta hernia con mayor
probabilidad?
m) Íleon
n) Colon sigmoides
o) Duodeno
p) Colon transverso
396. Recién nacido de 7 horas, de parto por cesárea debido a polihidramnios (aumento del volumen del líquido
amniótico), con regurgitación de la leche materna y artificial, y no ha presentado meconio. Se le realiza una tomografía
donde se evidencia aire en el estómago y una malformación del desarrollo esofágico. Con respecto a esta malformación
lo más probable es que se pueda tratar de una atresia esofágica:
q) proximal con fístula traqueoesofágica distal
r) distal con fístula traqueoesofágica proximal
s) proximal y distal
t) sin fístula

397. Lactante de 6 meses de edad que es traído a consulta por presentar vómitos no biliosos a repetición y retraso en
el crecimiento. En la radiografía de abdomen simple se observa nivel hidroaéreo en estómago y en primera porción de
duodeno (doble burbuja). ¿Cuál de las siguientes alternativas puede explicar la condición del lactante?
u) Páncreas anular
v) Atresia duodenal en la tercera porción
w) Atresia yeyunal
x) Hipertrofia del píloro

398. En ausencia o deficiencia de la secreción de la hormona motilina, se producirá:


a)sobrecrecimiento bacteriano
b) diarrea
c) aumento del vaciamiento gástrico
d) hipertrofia del píloro
399. La estimulación parasimpática aumenta la motilidad intestinal, mientras que la estimulación simpática la
disminuye. ¿Sobre cuál de las siguientes alternativas el sistema nervioso autónomo actúa para el control de la
motilidad intestinal?
y) Potencial de membrana en el plexo mientérico (de Auerbach)
z) Frecuencia de ondas lentas
aa) Secreción de secretina
bb) Nivel de IP3 en el plexo submucoso (de Meissner)

400.En un recién nacido con protrusión de contenidos abdominales y cubiertas por amnios o peritoneo, es cierto que:
a)Se presenta por un defecto en el cierre de la pared
b) Se acompaña de otras malformaciones congénitas
c) Se debe al no retorno de la hernia fisiológica
d) Se produce a través del ombligo

401. Lactante de 20 días con estreñimiento, distención abdominal progresiva, acompañada ocasionalmente de vómitos
biliosos. Como antecedente, el meconio lo eliminó por primera vez a las 72 horas de nacido. Su mamá menciona que
ayuda a la evacuación con ayuda de un termómetro rectal. Se sospecha de megacolon agangliónico (Enfermedad de
Hirschsprung). ¿Cuál de las siguientes alternativas explica el caso?
cc) Se presenta contracciones tónicas en la región ano rectal
dd) Se presenta dilatación de tracto gastrointestinal afectado
ee) Las células ganglionares sólo han migrado al ano recto
ff) La zona que más se afecta es inervada por fibras del nervio esplácnico menor

402. ¿Cuál de los siguientes reflejos disminuye el tránsito gastrointestinal?


a)Doloroso
b) Gastrocólico
c) De defecación
d) Colicoileal

403. Al ingerir una sustancia ácida como el vino (pH 3), se estimula la motilidad gástrica por acción de la hormona:
a)motilina
b) secretina
c) colecistoquinina (CCK)
d) bombesina

404.Paciente de 24 años acude a consulta externa por presentar una fístula oronasal (comunicación entre la cavidad oral
y la cavidad nasal). Está fístula está asociada al antecedente de haber sido operada de paladar hendido a los dos años de
edad, durante una campaña gratuita extranjera de corrección de paladar fisurado. ¿Cuál de las arterias palatinas podría
haberse lesionado durante esa cirugía?
gg) Mayor
hh) Menor
ii) Ascendente
jj) Rama palatina de la faríngea ascendente

405.Paciente de sexo masculino de 52 años con úlcera péptica gástrica de 14 años de evolución, con cuadro de
hemorragia digestiva alta hace 4 meses, sin cicatrización de la úlcera. Entre las opciones quirúrgicas se considera
realizarle un vaguectomía troncal (sección del nervio vago) a nivel del hiato esofágico. ¿Cuál de las siguientes
complicaciones podría esperarse producto de la pérdida de inervación parasimpática?
kk) Menor inervación del colon ascendente
ll) Se perderá el reflejo de defecación
mm) Se perderá el reflejo de micción
nn) Impotencia sexual

406.Paciente de 23 años con bulimia es traída a la emergencia deshidratada, semiconsciente y con alcalosis
metabólica. Los vómitos autoinfligidos por esta paciente se producen por estimulación de receptores en la base de la
lengua que mandan información directamente al:
oo) núcleo del tracto solitario
pp) centro del vómito en el tallo encefálico
qq) zona quimiorreceptora gatillo
rr) cerebelo

407.Niño de 3 años es traído a emergencia por madre quien manifiesta que hace 10 horas deglutió una pila pequeña
de reloj de bordes romos. El niño está asintomático. Usted la tranquiliza diciéndole es un cuerpo extraño tan pequeño
de seguro que va a seguir el tránsito intestinal como lo haría un bolo alimenticio, y que lo más probable es que en ese
momento se encuentre en:
ss) colon
tt) estómago
uu) yeyuno
vv) recto

408. Los movimientos en masa son un tipo de movimiento muy importante, una de las consecuencias de estos
movimientos es:
ww)la distensión rectal
xx) el peristaltismo del intestino delgado
yy) la retropulsión gástrica
zz) la contracción del esfínter anal interno

409.Durante la deglución, al momento que el bolo alimenticio pasa por el esfínter esofágico superior, se espera que la
presión intraesofágica:
aaa)disminuya en el cardias
bbb) disminuya en el tercio medio del esófago
ccc) aumente en la porción distal al bolo
ddd) aumente en el tercio medio del esófago

410.Paciente con enfermedad de Chagas que presenta disfagia a sólidos. ¿Cuál de las siguientes puede ser la causa de
esta complicación?
eee)Disminución de células ganglionares en el esfínter esofágico inferior
fff) Aumento en la liberación de óxido nítrico en el esfínter esofágico inferior
ggg)Disminución de las neuronas que liberan péptido intestinal vasoactivo
hhh) Aumento de la actividad de la motilina en el esófago distal

QUIZIZZ
1. Los vasos mesentéricos superiores se hallan a nivel de:
a) Cuello del páncreas

2. El nivel en el que se encuentra el píloro y el páncreas se puede determinar usando el


A) plano transpilórico

3. El dolor de estómago asociado a gastritis se suele ubicar en


a) epigastrio

4. La colecistoquinina inhibe el
a) vaciamiento gástrico

5. Paciente con disfasia ( dificultad para pasar alimentos) con to y disminución de peso. Con antecedente de
tabaquismo pesado. La sospecha es que tenga una disminución del diámetro esófago a nivel de la
a) tercera estrechez

6. La digestión de las proteínas se inicia en:


a) estómago

7. El estómago recibe información simpática proveniente del:


a) ganglio celíaco

8. El reflujo gastroesofagico tiene múltiples etiologías, una de ellas tiene que ver con alteración a nivel de:
a) primera estrechez
b) segunda estrechez
c) tercera estrechez
d) cuarta estrechez

9. La fístula retroperitoneal es causada por una falla en el desarrollo de:


a) tabique urorrectal

10. El nervio vago inerva el:


a) músculo estriado del esófago

11. El divertículo de Meckel es un rezago de:


a) conducto vitelino

12. La presencia de orina que sale por el ombligo de un recién nacido casa vez que llora, es posible que se deba a
un defecto en el desarrollo del
a) seno urogenital

13. La fístula retroperitoneal es causada por una falla en el desarrollo de


a) tabique urorrectal

14. Enfermedad asociada con un error en el desarrollo de las células de Cajal:


a) enfermedad de Hirschsprung

15. Aproximadamente en la semana 6 del desarrollo embrionario, el intestino medio gira 90 herniandose a nivel
del
a) cordón umbilical

16. El conducto biliar deriva del


a) endodermo

17. Paciente mujer con 54 años con nauseas y vómitos y abdomen agudo quirúrgico, se ingresa a sala de
operaciones donde se encuentra vólvulo de ciego, esto se debe a
a) Falta de fusión del mesenterio

18. Paciente con cirrosis hepática con hipertensión portal, en el que es posible encontrar que los vasos
umbilicales están permeables dentro de
a) ligamento redondo

19. Es normal encontrar glándulas submucosas en


a) esófago medio
b) esófago proximal
c) esófago distal
d) estómago

20. Cual de los siguientes órganos son intraperitoneales


a) estómago, vesícula biliar, íleon, hígado
CASO 1

Estudiante de medicina de la UPC de 21 años sufre de gastritis aguda ocasionada por comer en lugares
poco higiénicos. Suele consumir caramelos ( chupar ) mientras está en clase hasta la tarde. Toma gaseosas
regularmente (carbohidratos 46%, sodio 53%). También toma regular cantidad de leche (grasa 35%, lactosa
35%, proteínas 30%), pues le calma un poco el dolor el ardor que siente por la gastritis. Incluso, cuando
puede, se toma dos vasos de agua fría para calmar las molestias. Ha decidido ir al médico para tratarse pues
ya no soporta el dolor, el cual está seguro que los síntomas se deben a una elevada producción de ácido
clorhídrico en el estómago, y por ello le ha recetado Ranitidina (antihistamínico), con lo que siente mejoría.

● Para reducir la secreción de HCl en esta paciente se podría usar sustancias similares a:

-​ ​Péptido insulinotrópico dependiente de la glucosa (GIP)

● Si se usara atropina en esta paciente, se esperaría que disminuya la liberación de:


- Enzimas pancreáticas

● El consumir caramelos eleva los niveles en sangre de una hormona cuya función es la
estimulación de las células:
- ​ ​Beta del páncreas

● En este paciente con gastritis aguda debida a una alta producción de ácido clorhídrico, si se le
hiciera un examen de sangre, se encontraría elevados los niveles de:
- Colecistoquinina

● El consumo de una pequeña cantidad de gaseosa aumentará directamente la concentración


sérica de cuál de las siguientes hormonas:

- Péptido 1 similar al glucagón (GLP-1)

● El consumo rápido de 500 mL de gaseosa aumentará directamente la concentración sérica de


cuál de las siguientes hormonas:

- Gastrina

● Estimulan la secreción ácida gástrica

- Proteínas

● Con respecto a las ondas lentas, marque la afirmación correcta:}

- Son contracciones rítmicas espontáneas

● El uso de Ranitidina bloquea el receptor H2 de la histamina en las células parietales. La


histamina llega a estas células por:

- ​Difusión

● El consumir caramelos indirectamente activa la vía:

- POMC/CART
● ¿Cuál de los siguientes péptidos inhibe el vaciamiento gástrico?

- Colecistoquinina

● Para poder morder una manzana, es necesario usar el siguiente músculo:

- Milohiodeo

● El crecimiento de un adenocarcinoma de cuello de páncreas puede comprometer la pared


gástrica por continuidad. ¿Qué parte del estómago estaría comprometido con mayor
probabilidad?

- Pared posterior del antro

● El nacimiento de la arteria mesentérica superior se puede encontrar en cuál de los cuadrantes


abdominales:

- Epigastrio

● En cuanto a la colecistoquinina, marque la respuesta correcta:

- Potencia la acción de buffer con bicarbonato

● El aumento en la actividad motora de la pared gástrica genera un aumento en los


niveles locales de qué sustancia en la microvasculatura:

- Adenosina

CASO 2

Niño de sexo masculino de 2 años de edad, sufre de estreñimiento desde el nacimiento (1 deposición
cada 3-4 días). Madre menciona que le estimula la defecación con un termómetro rectal, y continuo uso
de enemas y laxantes. Desde hace 6 meses comienza con vómitos postprandiales. Los síntomas
aumentan en frecuencia y magnitud y están en relación con los episodios de estreñimiento. No refiere
fiebre, tos, diarrea ni lesiones cutáneas. Al examen físico presenta regular estado general, luce
deshidratado. Abdomen distendido, blando, depresible e indoloro. No se palpan masas abdominales.
Se permeabiliza el canal anal con termómetro rectal, encontrando cierta resistencia. Salida de material
fecal mal oliente en regular cantidad. Exámenes de laboratorio: hemograma normal. Signos
inflamatorios de fase aguda negativos. Alcalosis metabólica leve en sangre venosa. Radiografía con
enema baritado muestra recto y colon sigmoides dilatados (megacolon). Biopsia profunda: ausencia de
células ganglionares en la muestra enviada. Se realiza cirugía correctiva.

● La percepción de la pirosis (sensación de dolor o quemazón en el esófago) asociado al reflujo


gastroesofágico, puede aparecer o exacerbarse debido a:

- Ejercicio
● Considerando que este paciente está sometido a estrés por el agravamiento de su enfermedad,
es posible afirmar que sus ondas lentas están:

- Hiperpolarizadas

● En cuanto a los reflejos gastrocólico y gastroduodenal en este paciente, indique lo correcto:

- Se pueden considerar reflejos vago-vagales

● diferencia de las arcadas, los vómitos presentan apertura de:


- Esfínter esofágico superior

● En la Enfermedad de Hirschprung, el patólogo debe declarar un área como sana o libre de


enfermedad si es que encuentra:
- Células intersticiales en el plexo mioentérico
● Durante la fase faríngea de la deglución es espera que ocurra el siguiente fenómeno:
- El paladar blando cierra la entrada a la nasofaringe
● En este paciente se considera que está abolido el reflejo:
- Rectoesfinteriano
● Debido al acúmulo de material fecal en todo el marco colónico, y a la irritación química
asociada, el peristaltismo del íleon distal se debe encontrar:
- Inhibido

--------------------------------------------------------------------------------------------------------------------------------------------------
● Con respecto a la defecación señale el enunciado correcto:

- Es estimulado por un llenado de la cuarta parte del volumen rectal

● El contenido fecal se detiene en la zona inmediatamente proximal a la zona donde hay una
menor presencia de:

- Péptido intestinal vasoactivo

● La presencia de atresias y estenosis duodenales se deben básicamente a una:

- Falta de recanalización

● El ligamento de Treitz característicamente:

- Suspende el ángulo de Treitz

● Al deglutir un bolo alimenticio, es lógico suponer que al pasar por el esófago haya un mayor
consumo de oxígeno en la pared del tercio:

- Proxima​l

● Estudiante de medicina de 20 años, se ha amanecido estudiando para su examen de Sistema


Digestivo. No ha probado alimento desde la cena, por lo que se puede afirmar que la motilidad
de esta persona está siendo regulada por:

- Motilina

● La hernia fisiológica se produce dentro de:

- Cordón umbilical

● El crecimiento de un adenocarcinoma de páncreas compromete la pared gástrica por


contigüidad. ¿Qué parte del estómago se esperaría esté comprometido?

- Pared posterior del antro

● Al comer unas papitas fritas con mayonesa, el vaciamiento gástrico disminuye por efecto
directo de la hormona:

- colecistoquinina (CCK)

● Paciente que come entera una pizza familiar de chorizo y queso. Es posible esperar que debido
a la cantidad de alimento ingerida, las ondas lentas hayan:

- Sufrido ninguna alteración en su frecuencia

● En este caso se puede afirmar con seguridad que se presenta:

- contracciones tónicas en la región ano rectal

● La forma más común de atresia esofágica contiene:


- Estenosis proximal del esófago más fístula traqueoesofágica distal

● Con respecto a la saliva, marque la respuesta correcta:

- el sistema simpático estimula su secreción

● Respecto a las enfermedades del esófago, marque lo correcto:

- el diagnóstico diferencial de la acalasia es la enfermedad de Chagas esofágica

● En relación a la fisilogía gástrica, marque lo correcto:

- la cimetidina actúa en la región basolateral de la célula parietal

● La célula mucosa del cuello gástrico produce:

- Moco

● La saliva puede tener una variedad de electrolitos en su composición. Entre ellos el cloro,
respecto al cual se puede afirmar:

- Su concentración no llega a ser tan alta como en el plasma

● Los músculos de la masticación que producen la retropulsión de la mandíbula son:

- temporales

● Respecto a las glándulas salivales, marque lo incorrecto:

- la glándula sublingual tiene forma de garfio

● Respecto a la anatomía del estómago, marque lo correcto:

- la arteria gástrica derecha nace de la arteria hepática común

● En cuanto a la saliva, marque lo correcto:

- La amilasa cumple función digestiva

● Durante el ataque con gas sarín (bloqueador de la acetilcolinesterasa) en el metro de Tokio, en


1995, el personal de salud notó que los pacientes afectados presentaban:

- Hipersalivación

● Para estimular la motilidad intestinal se podría usar un análogo de: ​CCK

● No se espera que sea causa del vómito: ​AYUNO PROLONGADO

● En la Enfermedad de Hirschprung, el patólogo debe declarar un área como sana o libre de enfermedad
si es que encuentra: ​CÉLULAS INTERSTICIALES EN EL PLEXO MIOENTÉRICO.

● En la zona afectada por esta enfermedad, se espera que las ondas lentas estén: ​ABOLIDAS

● Si este paciente tuviera arcadas, entonces se debe considerar que durante la ocurrencia de dichas
arcadas, debemos encontrar contenido gástrico en: ​ESÓFAGO​.

● Cuando este paciente ingiera sus alimentos, se espera que al momento de pasar el bolo alimenticio por
el esfínter esofágico superior, la presión intraesofágica disminuya en: ​EL CARDIAS.
CASO 3

Paciente de 54 años con antecedentes de alcoholismo, gastritis crónica, tabaquismo pesado,


obesidad, cálculos biliares y cirrosis, es llevado a la emergencia por dolor abdominal en
epigastrio irradiado a la espalda y trastorno del sensorio.
Al examen físico: presión arterial 85/50 mmHg, frecuencia cardíaca 100 latidos/min, frecuencia
respiratoria 18 x minuto, temperatura axilar 36°C.
Conjuntivas pálidas, escleras ictéricas nevus arácnidos en tronco, distensión abdominal
marcada, cabeza de medusa, matidez desplazable en ambos flancos e hipogastrio, dolor a la
palpación de abdomen.
Tiempo de protrombina: 24 seg (testigo: 13 seg); TPT: 38 seg, glicemia: 165 mg/dL, uremia: 20
mg/dL, ASAT: 76 UI/L, ALAT: 22 UI/L, albumina: 2,5 g/dL, bilirrubina total: 2,6 mg/dL,
bilirrubina directa: 1,4 mg/dL, amilasa sérica 4000 U/L.

● Un efecto secundario en el estómago por la acción de la secretina es:

- Menor actividad de la pepsina

● Considerando que el paciente sufre de gastritis, se puede decir que la secreción de ácido por la
mucosa gástrica:

- Involucra transporte activo de Hidrogeniones

● En cuanto a la gastritis de este paciente, se encontró que era producida por la bacteria
Helicobacter pylori. Esta bacteria sobrevive en el medio ácido del estómago gracias a:

- Ureasa

● La bilirrubina directa aumentada en cirrosis hepática se excreta en la orina debido a:

- Ser hidrosoluble

● Paciente de 42 años con adenocarcinoma ductular. La TC ha demostrado claramente que el


tumor está en el cuello del páncreas y que hay un gran vaso ocluido. ¿Cuál de los siguientes
vasos estaría más probablemente obstruido?

- Vena porta.

● En relación a la histología hepática, marque lo correcto:


- La zona 3 se encuentra más cerca a la vena central lobulillar
CASO 4

Mujer de 83 años acude a emergencia por dolor abdominal desde hace 4 días,
localizado en epigastrio, irradiado a ambos hipocondrios, nauseas, vómitos y
distensión abdominal; tiene antecedente de cardiopatía hipertensiva, diabetes
mellitus tipo II y fibrilación auricular. Refiere deposiciones diarreicas muco
sanguinolentas hace 1 día. Los exámenes iniciales muestran PA: 110/60 mmHg, FC:
110/mn, leucocitos: 17800, neutrófilos de 93%. TAC abdómino pélvica se observa
oclusión completa de arteria mesentérica superior por trombo asociado a placa
ateromatosa.
● En esta paciente, ¿cuál de las siguientes sustancias no tendrá una considerable
disminución en su absorción? (marque la mejor respuesta):

- Calcio

● El mecanismo de la diarrea muco sanguinolenta que presenta la paciente, puede mejor definido
como de tipo (marque la mejor respuesta):}

- Exudativa

● Considerando que se ha comprometido el íleon distal, entre otras áreas, la atrofia o


descamación del epitelio de superficie explicaría cuál de los signos o síntomas de la paciente
(marque la mejor respuesta):

- Diarreas mucosanguinolentas

● Producto de esta isquemia, la expresión de cuál de las siguientes enzimas se vería


notablemente disminuida. Marque la mejor respuesta:

- Enteroquinasa

● Durante la cirugía, el cirujano observó que además la paciente tenía divertículos en el sigma. Se
sabe que estos divertículos:

- Se pueden asociar a estreñimiento crónico

● Después de remover quirúrgicamente el territorio gastrointestinal afectado, estaremos seguros


que no quedará en la paciente la siguiente estructura:
- Placas de peyer
● Durante la cirugía, se observa que el colon transverso no está muy afectado por la isquemia,
esto se debe a la anastomosis de las arterias:
- Cólica media e izquierda
● Durante la cirugía, el cirujano observó que además la paciente tenía divertículos en el sigma. Se
sabe que estos divertículos:
- Se pueden asociar a estreñimiento crónico
● En la paciente, el flujo sanguíneo de cuál de las arterias pancreáticas estará afectada:
- Pancreaticoduodenal posteroinferior

● Una mujer de 43 años dolor en hipocondrio derecho e ictericia. En la ecografía se evidencia cálculos
biliares. Estos cálculos lo más probable es que se encuentren localizados en: ​CONDUCTO
COLÉDOCO.

● Si existe un fármaco de acción colerética, se asume que se refiere a que potencia o estimula la:
RECIRCULACIÓN DE SALES BILIARES.

● Paciente con tumor neutro endocrino productor de secretina, debido o cual se puede esperar que su
secreción pancreática, comparada con la de una persona sana en estado de bajo fluido, tenga una
concentración de: ​BICARBONATO AUMENTADA
● Por cuál de las siguientes células es secretada principalmente la pro enzima procarboxipeptidasa:
ACINARES DEL PÁNCREAS.

● En relación a la secreción biliar y su composición, marque lo correcto: ​LAS SALES BILIARES


CORRESPONDEN AL 50% DE SU COMPOSICIÓN .

● El triángulo hepatocístico es importante de conocer en una colecistectomía, debido que tiene:


ARTERIA CÍSTICA.

● Durante una endoscopia, se ingresa en la papila mayor en la segunda porción del duodeno, y dentro de
la vía biliar extrahepatica, se toma una muestra de secreción biliar. Dicha muestra debe contener ácido:
QUENODESOXICÓLICO​.

● La internalización de proteínas o fragmentos de proteínas hacia el intracelular, es característico de :


CÉLULAS M
● Las ramas más pequeñas del árbol biliar son: ​CANALÍCULOS BILIARES.
● Con respecto a las lesiones y enfermedades de la boca, marque lo correcto: ​LA ERITROPLASIA DEBE
SER BIOPSIADA.
● Respecto a la motilidad del colon, marque lo correcto: ​Con la distención del estómago, suelen aparecer
movimientos en masa
● En un paciente hipertenso con tratamiento a base de beta bloqueadores, la producción de saliva se
espera que: ​DISMINUYA
● La saliva puede tener una variedad de electrolitos en su composición. Entre ellos el cloro, respecto al
cual se puede afirmar: ​SU CONCENTRACIÓN NO LLEGA A SER TAN ALTA O EN EL PLASMA.
● ESTIMULA LA PRODUCCIÓN DE SALIVA: ​VASODILATACIÓN PERIGLANDULAR.
● DURANTE EL SUEÑO LA COCENTRACIÓN DE BICARBONATO EN LA SALIVA: ​DISMINUYE​ .
● Un paciente hipertenso está tomando un medicamento bloqueador de receptores alfa-1 adrenérgicos
(prazosina), y como efecto secundario se queja de: ​HIPOSALIVACIÓN​.
● La atropina es el tratamiento para la intoxicación por órganos fosforados. Se administra hasta alcanzar
niveles terapéuticos. Un buen indicador que la atropina ya está en niveles terapéuticos es: ​SEQUEDAD
DE LA MUCOSA ORAL.
● En cuanto a la saliva, marque lo correcto: ​LA AMILASA CUMPLE FUNCIÓN DIGESTIVA.
● Con respecto a la saliva: ​EL SISTEMA SIMPÁTICO ESTIMULA SU SECRECIÓN.
● La secreción de agua y bicarbonato por el páncreas exocrino se da básicamente en la fase:
- Intestinal
● En relación a la secreción biliar y su composición, marque lo correcto:
- Las sales biliares corresponden al 50% de su composición
● Si existe un fármaco con acción colerética, se asume que se refiere a que potencia o estimula la:
- Recirculación de sales biliares
● Hombre de 65 años con isquemia intestinal leve por oclusión aterosclerótica de la arteria mesentérica
superior, pero la irrigación sanguínea colateral ha retrasado el inicio de la necrosis. ¿Qué vasos
ofrecen colaterales entre el tronco celíaco y la arteria mesentérica superior?
- Pancreaticoduodenal superior e inferior.
● Una mujer de 43 años dolor en hipocondrio derecho e ictericia. En la ecografía se evidencia cálculos
biliares. Estos cálculos lo más probable es que se encuentren localizados en:
- Conducto colédoco
● Cuando el alimento se encuentra en el estómago, se produce la liberación de enzimas pancreáticas
básicamente debido a la acción de:
- Vago
● Paciente con tumor neuroendocrino productor de secretina, debido a lo cual se puede esperar que su
secreción pancreática, comparada con la de una persona sana en estado de bajo flujo, tenga una
concentración de:
- bicarbonato aumentada
● ¿Por cuál de las siguientes células es secretada principalmente la pro enzima procarboxipeptidasa?
- Acinares del páncreas
● Dentro de las funciones de las células de Ito, marque lo incorrecto:
- Pueden fagocitar patógenos y actúan como presentadoras de antígeno
● Paciente de 42 años con adenocarcinoma ductular. La TC ha demostrado claramente que el tumor está
en el cuello del páncreas y que hay un gran vaso ocluido. ¿Cuál de los siguientes vasos estaría más
probablemente obstruido?
- Vena porta.
● Respecto a la anatomía del estómago, marque lo correcto:
- la arteria gástrica derecha nace de la arteria hepática común
● Con respecto a las lesiones y enfermedades de la boca, marque lo correcto:
- a eritroplasia debe ser biopsiada
● La secreción de saliva es importante en la fisiología digestiva. Su concentración de potasio llega a ser
menor que la del plasma cuando su secreción tiene un flujo:
- Nunca
● Para protegerse del entorno ácido, el Helicobacter pylori se autogenera un entorno de pH menos ácido
alrededor s​uyo, gracias a una enzima que alcaliniza su entorno local mediante la conversión de:
- Urea en NH3
● Durante el sueño, la concentración de bicarbonato en la saliva:
- Disminuye
● Respecto a la motilidad del colon, marque lo correcto:
- Con la distensión del estómago, suelen aparecer movimientos en masa
● Respecto a las enfermedades del esófago, marque lo correcto:
- el diagnóstico diferencial de la acalasia es la enfermedad de Chagas esofágica
● El omeprazol actúa sobre la membrana _____________ de la célula ____________
- apical / parietal
● Durante el ataque con gas sarín (bloqueador de la acetilcolinesterasa) en el metro de Tokio, en 1995, el
personal de salud notó que los pacientes afectados presentaban:
- Hipersalivación
● Un paciente hipertenso está tomando un medicamento bloqueador de receptores alfa-1 adrenérgicos
(prazosina), y como efecto secundario se queja de:
- Hiposalivación
● La arteria palatina menor irriga un área de la boca cuyo epitelio es:
- Escamoso no queratinizado
● Producto de la alimentación, se producen diversas sustancias peptídicas, cininas y bradicininas, las
cuales permiten que:
- El flujo sanguíneo intestinal aumente hasta 8 veces
● En relación a los principios de motilidad, marque lo incorrecto:
- Las dos terceras partes proximales del esófago están conformadas por músculo
esquelético
● El crecimiento de un adenocarcinoma de cuello de páncreas puede comprometer la pared gástrica por
continuidad. ¿Qué parte del estómago estaría comprometido con mayor probabilidad
- Pared posterior del antro
En un paciente con apendicitis aguda, la sensación de dolor producido por esta inflamación es llevada
por el nervio:
- Esplácnico menor
Para poder morder una manzana, es necesario usar el siguiente músculo:
- Milohiodeo
El dolor periumbilical o epigástrico en el inicio de una apendicitis aguda se debe a:
- Estímulo del sistema simpático.
El nacimiento de la arteria mesentérica superior se puede encontrar en cuál de los cuadrantes
abdominales:
- Epigastrio
El uso de Ranitidina bloquea el receptor H2 de la histamina en las células parietales. La
histamina llega a estas células por:
- Difusión
Marque la respuesta correcta en relación a la gastrina:
- Las células G son las productoras y se encuentran principalmente en el antro gástrico
HOLA PUTAS YA SABEN CONTROL F
Autoevaluación 01

Preguntas Respuestas

En cuanto a la distribución del tubo digestivo en Mesenterio


la cavidad abdominal, se puede afirmar que los
órganos intraabdominales están suspendidos
por: (marque la mejor respuesta)

Marque el órgano que se encuentra fuera de la Recto


cavidad peritoneal:

El íleon se encuentra principalmente en: Cuadrante inferior derecho

Al palpar una masa pétrea en hipocondrio Vesícula biliar


derecho, es probable que se trate de un tumor
de:

Al momento de espirar profundamente, ocurre Aumento de la presión intraabdominal


lo siguiente:

El mesocolon transverso se origina en: Pared posterior del abdomen

Se la conoce como el vigilante del abdomen , Omento mayor


por su capacidad de desplazarse y adherirse a
cualquier zona inflamada, y envolviendo el
órgano para frenar la inflamación:

Las glándulas submucosas se encuentran en Proximal a la unión esofagogástrica


mayor cantidad a nivel:

Señale el órgano del sistema digestivo que no Vesícula biliar


tiene adventicia:

Marque lo correcto con respecto a lo que Se aplana el diafragma


sucede al contraerse el diafragma:

Para colocarle una sonda de alimentación T8


directamente al estómago (gastrostomía), el
cirujano deberá hacer una incisión en la piel del
abdomen inervada por las raíces de:

La estrechez esofágica más proximal es: cricofaringeo

El orificio omental, o hiato de Winslow, se Hepatoduodenal


encuentra limitado por el ligamento:

peritoneo parietal

Estructura que tiene inervación somátic

s considerado un órgano inicialmente pancreas


intraperitoneal:
Al atorarse el bolo alimenticio grande y Nervios esplácnicos
poco masticado en el esófago, se puede
despertar una sensación de dolor, que es
transmitida por:

Los dobleces de peritoneo que comunican Repliegues


un órgano con la superficie interna de la
pared abdominal posterior antiguamente se
denominaban ligamentos, sin embargo,
actualmente se denominan:

Se traza un plano a nivel de la mitad de la


distancia entre la horquilla esternal y la
sínfisis del pubis, a ese nivel se espera
encontrar: Cuello del páncreas

El peritoneo parietal está separado del


peritoneo visceral por la fascia de _______
que está fijada a la pared posterior del
abdomen, y es importante porque por esta
vía se pueden diseminar las infecciones
peritoneales:
Todlt

Autoevaluación 2

Producto de la alimentación, se producen


diversas sustancias peptídicas, cininas y El flujo sanguíneo intestinal aumente hasta 8
bradicininas, las cuales permiten que: veces

El crecimiento de un adenocarcinoma de cuello Pared posterior del antro


de páncreas puede comprometer la pared
gástrica por continuidad. ¿Qué parte del
estómago estaría comprometido con mayor
probabilidad?

El nacimiento de la arteria mesentérica Epigastrio


superior se puede encontrar en cuál de los
cuadrantes abdominales

Marque lo correcto:
La hernia fisiológica se produce en la sexta
semana y es la salida temporal de asas
intestinales a través del cordón umbilical
Estimulan la secreción ácida gástrica: Proteínas

¿Cuál de los siguientes líquidos corporales Jugo pancreático


tiene el pH más alto?:

Paciente ha sido puesto en Nada por vía neuropeptido Y


oral (nil per os, NPO) durante 24 horas, la
sensación de hambre en este paciente se
debe la presencia en el hipotálamo de la
sustancia:

Marque la respuesta correcta en relación a Las células G son las productoras y se


la gastrina: encuentran principalmente en el antro
gástrico

En cuanto a la colecistoquinina, marque la Potencia la alcalinización del duodeno


respuesta correcta:

Las arterias que derivan del tronco celiaco son, Hepática derecha
excepto:

La triada sintomática: vómitos explosivos Estenosis congénita hipertrófica del píloro


post-prandiales, movimientos peristálticos
epigástricos visibles de izquierda a derecha y
nódulo palpable epigástrico subcostal derecho,
pertenecen a:

Cuál de los siguientes péptidos inhibe el colecistoquinina


vaciamiento gástrico?

¿En qué casos los vómitos son siempre Atresia yeyunal


biliosos?:

Al ingerir una cantidad de glucosa por vía oral, K


ésta es interiorizada en las células del
organismo más rápido que si esa misma
cantidad hubiese sido administrada por vía
endovenosa. Este fenómeno sucede gracias a
una sustancia secretada por las células:

El dolor periumbilical o epigástrico en el inicio


de una apendicitis aguda se debe a: Estímulo del sistema simpático.

Durante una cirugía oncológica, ¿la extirpación Recto


de cuál de los siguientes órganos se vería
comprometida por la presencia de adventicia?:

Paciente con disminución del apetito Endorfinas


marcada asociada a cáncer terminal, para
promover la ingesta de alimentos se podría
usar análogos de:
Sustancia que inhibe la secreción y Péptido insulinotrópico dependiente de
motilidad del estómago prolongando el glucosa (GIP)
tiempo de digestión:

En un neonato con vómitos biliosos, Atresia duodenal


persistentes y con baja ponderal, usted
sospecharía:

Paciente de 13 días de vida con vómitos Vago


explosivos a las dos horas después de lactar. Al
examen físico se palpa la oliva pilórica. ¿Cuál
es el nervió cuyos filetes dan inervación
eferente a la estructura afectada?

¿En qué capa se encuentra la alteración Muscular propia


principal en el Hirschprung o megacolon
aganglliónico?

Paciente de 23 años con dolor intenso conducto vitelino


periumbilical de inicio agudo. Durante la
cirugía, el cirujano encuentra una estructura
sacular perforada en intestino delgado, a
unos 60 cm proximal a la válvula ileocecal.
Dicha estructura es un derivado
embriológico del:

Autoevaluación 03

¿Cuál de los siguientes péptidos inhibe el Colecistoquinina


vaciamiento gástrico? (marque la mejor
respuesta

Al ingerir rápidamente un litro de agua, usted ACh del sistema mienterico


esperaría que la gastrina aumente por efecto
directo de:

En un paciente con diarrea por hipermotilidad, Peptido intestinal vasoactivo


usted sospecharía en el posible aumento de las
siguientes sustancias, excepto:

En un paciente con gastroparesia (motilidad CCK


lenta del estómago), que presenta distensión
abdominal después de comer, usted le
recomendaría que evite el consumo de lípidos y
aminoácidos para disminuir la acción de:
Paciente obeso con Covid-19 es intubado por Disminuye el Ph gástrico
interno inexperto, quien al solicitar que
bombeen aire dentro del tubo endotraqueal,
nota que el epigastrio se distiende. Al
sospechar que ha introducido el tubo en el
estómago, también es ci

Una de las siguientes sustancias reguladoras, SOMATOSTATINA


puede actuar de forma paracrina y como
hormona. Marque la correcta:

Péptido regulador cuya liberación es inhibida GASTRINA


por la disminución del pH gástrico

Al introducir una solución azucarada PÉPTIDO TIPO GLUCAGÓN 1


directamente al estómago mediante una
gastrostomía (comunicación entre la piel
abdominal y el estómago), la sustancia que
provocará que aumenten los niveles séricos de
insulina es:

Dos estudiantes de medicina después de rendir BOMBESINA


su ECU, deciden celebrar su buena nota
conseguida, yendo a comer tomar un café y
comer un sándwich en el Starbucks de la
Facultad. Antes de llegar a la cafetería, los
impulsos nerviosos del complejo vagal dorsal
activan la secreción de ácido gástrico por la
inducción de la liberación del siguiente
neurotransmisor del sistema nervioso entérico:

Paciente obeso con Covid-19 es intubado por DISMINUYE EL pH GÁSTRICO


interno inexperto, quien al solicitar que
bombeen aire dentro del tubo endotraqueal,
nota que el epigastrio se distiende. Al
sospechar que ha introducido el tubo en el
estómago, también es cierto que:

Sustancia proteica que inhibe la secreción y la GIP


motilidad del estómago prolongando el tiempo
de digestión:

Paciente de 43 años con tumor neuroendócrino El vaciamiento gástrico


bronquial productor de CCK. En este paciente
estamos seguros que se encontrará inhibido:

Al disminuir el pH duodenal por el HCl gástrico, célula ductal de Wirsung


se libera principalmente una hormona cuya
célula diana es:

¿Cuál de los siguientes péptidos inhibe el colecistoquinina


vaciamiento gástrico? (marque la mejor
respuesta)

Al introducir una solución azucarada Péptido tipo glucagón 1


directamente al estómago mediante una
gastrostomía (comunicación entre la piel
abdominal y el estómago), la sustancia que
provocará que aumenten los niveles séricos de
insulina es:

El omeprazol actúa sobre la membrana apical / parietal


_____________ de la célula ____________

Al ingerir grandes cantidades de dulces, con la Disminuye insulina


subsecuente estimulación de incretinas, usted
esperaría que el apetito .., debido a

La hormona que tiene un efecto sinérgico con la CCK


secretina para optimizar el pH duodenal y la
digestión, es

En un paciente con tumor carcinoide de páncreas productor de


gastrina (Síndrome de Zollinger-Ellison) se puede esperar un Secreción de secretina
aumento en la:

Un paciente con un tumor neuroendocrino productor de Diarrea


serotonina presentará:

La Ranitidina (antagonista de receptor H2) bloquea los efectos H2 pero también M3


sobre el receptor:

Estimulan directamente la secreción ácida gástrica: Proteínas

Una de las siguientes sustancias no comparte con las otras la


misma acción sobre la producción de ácido gástrico Colecistoquinina

Al consumir un pan con mantequilla, la sensación de hambre Colecistoquinina


disminuye debido a la acción de:

AUTOEVALUACIÓN 04

Todos los músculos motores de la lengua están inervados Patalagoso


por el XII par, excepto:

Las carnes tienen un sabor especialmente delicioso debido Glutamato


a que presentan en su composición

Músculos que ayudan a empujar el bolo hacia la orofaringe: Estilogloso y palatogloso

En un paciente con diarrea por hipermotilidad, usted Péptido intestinal vasoactivo


sospecharía en el posible aumento de las siguientes
sustancias, excepto
En un paciente con lesión del nervio vago derecho, al Izquierda x2
evaluarle el velo del paladar, se solicita al paciente que diga
ahh . Entonces se puede observar una desviación de la
úvula hacia:

Al ingresar líquidos o sólidos en la cavidad oral, un Depresión del paladar blando


mecanismo que permite que una persona respire mientras
mastica es

El nervio palatino menor inerva un área del paladar que está Estratificado plano no queratinizado
recubierta por epitelio

Al ingresar líquidos o sólidos en la cavidad oral, un Depresión del paladar blando


mecanismo que permite que una persona respire mientras
mastica es:

Al rozar agua caliente en la punta de la lengua, usted Lingual x2


esperaría la estimulación del nervio:

Niña de 6 años se asusta por que se le ha aflojado un diente


deciduo. Este fenómeno se produce por: Laxitud del ligamento periodóntico
x2

Son músculos de la masticación que producen la milohioideos x2


retropulsión de la mandíbula, excepto:

El frenillo de los labios se encuentra en: Cavidad vestibular x2

Paciente con lesión del hipoglosó del lado izquierdo. Para El lado izquierdo x2
evaluarlo se le pide al paciente que saque la lengua, la cual
se espera que la apunta de la lengua se dirija hacia:

Se considera que el gusto puede viajar a través del nervio Glosofaríngeo

Los corpúsculos gustativos se encuentran en la lengua, pero Paladar blando


además se les puede encontrar en:

Al seccionar el nervio facial a nivel timpánico, usted Disminución del gusto en la punta
esperaría: de la lengua x2

PREGUNTAS DE LA CI 1:

1. Paciente mujer de 22 años con dolor abdominal en fosa iliaca derecha. Marque la opción que
no consideraría como diagnóstico:Calculo renal derecho
2. Un paciente recibe una bala que ingresa por la pared anterior abdominal de forma transversal
y daña la vértebra L1. ¿Qué órgano NO esperaría que se encuentre dañado?:Cola del
páncreas
3. En la atención de un recién nacido se observa que rechaza la alimentación oral, regurgita y
tiene tos cada vez que lacta, se intenta colocar una sonda nasogástrica, la cual es imposible
de hacer que llegue al estómago. Esta anomalía congénita se debe a: Desviación del tabique
traqueoesofágico
4. Durante la alimentación se secreta bicarbonato en el duodeno con el fin de amortiguar la
acidez proveniente del estómago. Esta función es estimulada por una sustancia liberada por
las células: S
5. Similar a la hemodiálisis, existe un procedimiento de filtración sanguínea colocando un tubo
en la cavidad peritoneal a través de la pared abdominal. En esta diálisis peritoneal, la
membrana de intercambio es un epitelio: Plano simple
6. Un paciente con rigidez de la pared abdominal anterior por dolor abdominal, realiza una
inspiración forzada. Usted esperaría que la presión intra abdominal: Aumente
7. En un paciente con hipomotilidad intestinal y colónica por diabetes de larga evolución, usted
esperaría el siguiente efecto al darle encefalinas: Constipación severa
8. Al realizar una sección del vago a nivel de L1, usted esperaría que el siguiente órgano siga
funcionando con normalidad: Sigmoides
9. Si un cirujano clampa (obstruye) el nacimiento de la arteria mesentérica superior, usted
esperaría que NO se necrose el siguiente órgano: Sigmoides
10. Paciente con gastrinoma el cual produce una hormona que tiene ciertos efectos sobre el
estómago. Producto de ello se espera encontrar elevados los valores sanguíneos de:
Secretina
11. Paciente de 3 años ingiere accidentalmente una moneda. En la emergencia se le toma la
siguiente radiografía, confirmándose que la moneda se encuentra en: Ciego

12. Luego de 12 horas de tener un dolor en epigastrio, un niño de 12 años menciona que el
ahora el dolor se ubica en fosa iliaca derecha. Usted sabe que este cambio en la ubicación
del dolor se debe a que ahora se encuentra comprometido: Peritoneo parietal
13. En un paciente con hipomotilidad intestinal y colónica por diabetes de larga evolución, usted
esperaría el siguiente efecto al darle encefalinas: Constipación severa
14. Un paciente recibe una bala que ingresa por la pared anterior abdominal de forma transversal
y daña la vértebra L1. ¿Qué órgano NO esperaría que se encuentre dañado?Cola del
páncreas
15. Paciente de 3 años ingiere accidentalmente una moneda. En la emergencia se le toma la
siguiente radiografía, confirmándose que la moneda se encuentra a nivel de: Hipocondrio
derecho

16. Si un cirujano clampa (obstruye) el nacimiento de la arteria mesentérica superior, usted


esperaría que NO se necrose el siguiente órgano: Sigmoides
17. Al visualizar en qué capa se encuentran las glándulas marcadas con el puntero, es evidente
que esta microfotografía corresponde al: Duodeno (vellosidades) y esófago (glándulas de

Bruner, submucosa)

18. En la siguiente imagen, usted pensaría que el cuerpo extraño se encuentra en: Yeyuno

19. Marque la respuesta correcta: Los omentos contienen vasos sanguíneos grandes que irrigan
sólo a su propio tejido
20. Paciente mujer de 22 años con dolor abdominal en fosa iliaca derecha. Marque la opción que
no consideraría como diagnóstico: Calculo renal derecho
21. Un paciente recibe una bala que ingresa por la pared anterior abdominal de forma transversal
y daña la vértebra L1. ¿Qué órgano NO esperaría que se encuentre dañado?: Cola del
páncreas
22. En la región señalada con una flecha roja en la siguiente radiografía baritada de abdomen, se
produce una hormona cuya secreción es inhibida por: Ácido clorhídrico

23. Una neoplasia maligna de la pared posterior del estómago ha perforado la serosa y ha hecho
diseminación por la cavidad adyacente. Esta diseminación ha llevado a implantes de tumor
en otros órganos, excepto: (marque la mejor respuesta): Íleon
24. Al comer pescado, una persona se tragó una de las espinas, de mediano tamaño, la cual se
ha incrustado en la mucosa cecal. El epitelio que ha sido perforado es de tipo: Cilíndrico
simple
25. Si un cirujano clampa (obstruye) el nacimiento de la arteria mesentérica superior, usted
esperaría que NO se necrose el siguiente órgano: Sigmoides
26. Para la cuarta semana de desarrollo embriológico, se produce una malformación
vascular en el borde dorsal del estómago primitivo. Durante una endoscopía gástrica
a los 23 años de vida, se encuentra dicha malformación en ______________ del
estómago: Curvatura Mayor
27. Un paciente con rigidez de la pared abdominal anterior por dolor abdominal, realiza
una inspiración forzada. Usted esperaría que la presión intra abdominal: Aumente
28. Durante la alimentacion se secreta bicarbonato en el duodeno con el fin de
amortiguar la acidez proveniente del estómago. Esta función es estimulada por una
sustancia liberada por las células: S
29. Durante la exploración abdominal, casualmente se descubrió un tejido ectópico (señalado
con flecha roja). Este ectópico está ubicado en: Estómago

30. Respecto a la bolsa Omental, marque la respuesta incorrecta:Se encuentra por detrás del
hígado y del colon transverso
31. El leiomioma de la muscular propia es un tumor benigno de músculo liso, el cual
puede aparecer casionalmente en el esófago_______: Distal
32. En la atención de un recién nacido se observa que rechaza la alimentación oral,
regurgita y tiene tos cada vez que lacta, se intenta colocar una sonda nasogástrica,
la cual es imposible de hacer que llegue al estómago. Esta anomalía congénita se
debe a: Desviación del tabique traqueoesofágico
33. El principal defecto en un paciente con páncreas divisum, se da por la falta de: Fusión de
conductos de yema ventral y dorsal
34. Durante una comida rica en grasas, el contenido de la vesícula biliar es expulsado hacia el
intestino delgado debido a la secreción de las células __________ que se encuentran en el
___________: I duodeno
35. La secreción de las células G promueve la liberación de una sustancia que a su vez estimula
la secreción de las células: S
36. Al hacer una endoscopia e insuflar aire dentro del estómago y distenderlo, usted esperaría
que se secreta gastrina debido a la siguiente sustancia neurocrina: Acetilcolina
37. Marque el órgano que contenga capa adventicia: (marque la mejor respuesta): Colon
38. El esófago contiene músculo estriado derivado de: Arcos faríngeos
39. La célula diana de la secretina se encuentra en: Páncreas
Preguntas de la CI2 :
1. El sabor umami es muy particular, y tiene como característica que: Su percepción se
lleva por el nervio craneal VII
2. Paciente con enfermedad por reflujo gastroesofágico por 10 años, se sabe que una
de las causas de esta enfermedad es un defecto en:
a. El rodete del esfínter
b. Plicatura diafragmática
3. Los procesos palatinos pertenecen al hueso: Maxilar superior
4. En el tratamiento de la intoxicación por órganos fosforados (inhibidor de la
colinesterasa) se debe usar atropina para revertir sus efectos. Uno de los signos que
me indicará que se está logrando el efecto terapéutico deseado es: Sequedad de la
boca
5. Paciente de 5 años de edad, con fiebre e hinchazón de la mejilla izquierda. Al
examinar el vestíbulo oral, el pediatra diagnostica parotiditis purulenta. El conducto
salival comprometido atraviesa el: Buccinador
6. Un paciente que usa brackets presenta una lesión blanquecina al lado del carrillo
dental superior, sobre la mucosa de la mejilla. Esto podría deberse a:Queratinización
de la mucosa
7. El esófago cuenta con dos esfínteres. Uno de ellos es funcional. Marque la
característica que NO le proporciona dicha función: Engrosamiento de la capa
muscular
8. La producción de la saliva es regulada por el sistema nervioso autónomo de forma
que:El sistema simpático estimula su secreción
9. La saliva tiene diferentes funciones, entre ellas: La amilasa digiere carbohidratos
10. Paciente de 74 años con una neurinoma del nervio craneal XII, el cual le produce
una parálisis de los músculos inervados por dicho par craneal. En el examen clínico,
se evidencia que la lengua aún puede realizar el movimiento hacia: Arriba y Atrás
11. En la comunidad agrícola de Acotabamba se ha reportado un brote de casos de
intoxicación, probablemente por un insecticida de tipo carbamato (anticolinesterasa).
Dentro de la información clínica enviada, usted busca signos que confirmen esta
sospecha, por ejemplo: Hipersalivación
12. Un paciente con perforación del esófago a nivel de la unión esofagogástrica,
presentará: Neumoperitoneo
13. Al prescribir neostigmina (inhibidor de la colinesterasa) a un paciente para un
trastorno de la motilidad intestinal, se puede predecir un aumento en la
concentración salival de: Bicarbonato
14. La producción de saliva es estimulada por: Vasodilatación periglandular
15. Con respecto a la lengua, marque la respuesta incorrecta: En su tercio posterior, la
inervación sensitiva es por el nervio vago
16. Al seccionar el nervio facial a nivel del tímpano, se producirá: Disminución del gusto
en la punto de la lengua
17. Recién nacido con labio y paladar hendido severo, usted NO espera encontrar el
siguiente síntoma/signo: Distensión gástrica
18. Con la contracción de los músculos del piso de la boca, estando el hueso hioides
fijo, la siguiente glándula verterá su contenido hacia la cavidad oral: Submandibular
19. En una persona que sufre de trastorno de ansiedad y presenta síndrome de boca
seca desde hace un año, una de las complicaciones que se puede encontrar es:
Caries
20. No es una característica del paladar blando: Cierra el istmo de las fauces, evitando
así el ingreso del bolo a la nasogofaringe
21. Paciente con intoxicación por órganos fosforados (Inihibidores de la colinestereasa)
es traído a la emergencia con sialorrea. En esa condición: La concentración salival
de potasio llega a ser mayor que la del plasma
22. Paciente de 63 años con deshidratación severa por golpe de calor, se espera que la
saliva esté: Hipotónica
23. La saliva tiene diferentes funciones, entre ellas: La amilasa digiere carbohidratos
24. Paciente con historia de reflujo gastroesofágico por 15 años, se toma biopsia de
esófago durante una endoscopía de control. Para confirmar la presencia de un
Esófago de Barret, el patólogo debe encontrar en la mucosa esofágica: Células
caliciformes
25. La saliva puede tener una variedad de electrolitos en su composición. Entre ellos el
cloro, respecto al cual se puede afirmar que: Su concentración no llega a ser tan alta
como en el plasma
26. La cavidad oral es una estructura que tiene diversas particularidades, entre ellas no
se encuentra: El paladar blando tiene tejido muscular y óseo
27. En el esófago, el cambio de adventicia a serosa se produce a nivel de: Cuarta
estrechez

REPASO DE DIGESTIVO
● Respecto a la motilidad del colon, marque lo correcto
○ Con la distensión del estómago, suelen aparecer movimientos en masa
○ Se producen contracciones segmentarias principalmente en colon izquierdo
○ Los movimientos de masa se encargaran del mezclado de las heces
○ Las haustras del colon contribuyen al reflejo de defecación
● Paciente de 3 años ingiere accidentalmente una moneda. En la emergencia se le
toma la siguiente radiografía confirmándose que la moneda se encuentra en:
○ Colon ascendente
○ íleon proximal
○ yeyuno
○ ciego

● Paciente con tos y disminución de peso, fumador, presenta disfagia mecánica, el


tumor está a nivel de:
○ Segunda estrechez -T4
○ Cuarta estrechez - T12
○ Tercera estrechez - T6
○ Tercera estrechez - T10

● Durante la exploración tomográfica abdominal casualmente se descubrió un tejido


ectópico (señalado con flecha roja). Este ectópico está ubicado en
○ Vesícula biliar
○ Duodeno
○ Estómago
○ Páncreas
● Paciente con disfagia mecánica, y presión elevada en EEI. El diagnóstico es ____
con niveles bajos de ___
○ Enfermedad por reflujo gastroesofágico – Acetilcolina
○ Acalasia – Sustancia P
○ Enfermedad por reflujo gastrointestinal – Óxido nítrico
○ Acalasia – Óxido nítrico
Si pregunta con niveles altos es la Ach o sustancia P
● Paciente de 3 años ingiere accidentalmente una moneda. En la emergencia se le
toma la siguiente radiografía confirmándose que la:
○ Hipocondrio derecho
● Al visualizar en qué capa se encuentran las glándulas marcadas con el puntero a es
evidente que esta microfotografía corresponde al:
○ Esófagp → aca tambien hay glándulas
○ Yeyuno
○ Duodeno → las glándulas de brunner
○ Ileon
Se diferencia por el epitelio y por las vellosidades (duodeno)
● Paciente con parálisis bilateral del nervio hipogloso, la única función que aún podrá
hacer la lengua será:
○ Pronuncia la “R” del idioma español
○ Sacar la lengua de la boca
○ Deprimir la lengua
○ Deglutir
● Durante la secreción de saliva es de esperarse que las concentraciones de
________ y ______ disminuyan al disminuir el flujo:
○ Bicarbonato - potasio
○ Potasio - cloro
○ Sodio - bicarbonato
○ Potasio - sodio
○ Sodio - cloro

PARCIAL:
1. Alrededor de los 6 años comienza el cambio de la dentadura decidua por la dentición
permanente. Este proceso se inicia con una remodelación de: Ligamentos
periodontales
2. Al examinar a un paciente, el otorrinolaringólogo puede encontrar en la cavidad
faríngea la siguiente estructura: Amígdala lingual
3. El siguiente conducto drena en la cavidad vestibular oral: Stenon
4. La acción de los músculos longitudinales de la faringe es importante porque permite
que se movilice el bolo y se cierre la vía aérea. En base a ello, marque la opción
correcta: Palatofaríngeo N. Vago
5. Paciente de 47 años con tumor neuroendócrino productor de péptido intestinal
vasoactivo, debido a lo cual presenta: Mayor secreción intestinal
6. El punto crítico en el que la deglución pasa de ser un proceso voluntario, a ser un
proceso completamente involuntario, está marcado por el paso del bolo por: Itsmo
de las fauces
7. El fenómeno de relajación receptiva del esfínter esofágico inferior es llevado a cabo
por un reflejo a larga distancia, llevado por el Nervio Vago. La sustancia neurocrina
involucrada en esta relajación es: Péptido intestinal vasoactivo
8. En relación a la macroestructura de las glándulas salivales, marque lo incorrecto: El
conducto de Stenon perfora el masetero
9. El proceso de alimentación debe ser iniciado con un proceso que incluye una la
contracción de los músculos del piso de la boca, y que: El hueso hioides esté fijo
10. Paciente de 22 años ingresa a emergencia con shock hipovolémico. Se espera
encontrar: Motilidad intestinal disminuida
11. Paciente con hiposecreción salival postradioterapia, NO presentará el siguiente
problema: Menor digestión de los lípidos
12. Se puede lograr que aparezcan contracciones tónicas mediante: Aumento de la
frecuencia de espigas
13. Marque la afirmación incorrecta: Las ramas esofágicas de la aorta emergen de las
arterias intercostales posteriores
14. En relación a la cavidad peritoneal: El mesenterio suspende al intestino delgado
15. ¿Cuál de los siguientes no inhibe la secreción de gastrina?: Engrosamiento de la
pared gástrica
16. Al ingerir alimentos grasos, se espera que se presente posteriormente disminución
del apetito por acción de: Colescistoquinina
17. La siguiente hormona no genera ralentización del vaciamiento gástrico: Gastrina
18. En la cultura occidental, el beso en la boca es un gesto de intimidad entre dos
personas, y se procede mediante la aposición de las superficies de la boca cuya
característica es que: No tiene glándulas salivales
19. Con respecto a los péptidos gastrointestinales, marque lo correcto: Las sustancias
paracrinas pueden llegar a sus células diana por vía sistémica también
20. Al realizar la sección de los nervios vagos a nivel torácico, se espera que el siguiente
órgano se mantenga indemne: Sigmoides
21. Paciente de 56 años con cirrosis hepática ingresa a emergencia por hemorragia
digestiva alta, asociada a várices esofágicas. Se puede afirmar que la sangre
proviene de la vena: (marque la mejor respuesta) : GÁSTRICA IZQUIERDA
22. Marque el órgano cuyo drenaje venoso no se relaciona con la circulación esplácnica:
RIÑÓN IZQUIERDO
23. La única hormona que puede ser secretada por estímulo de cualquiera de los tres
principales macronutrientes es:PÉPTIDO INHIBIDOR GÁSTRICO
24. La rotación de los intestinos fuera del abdomen se da alrededor de: ARTERIA
MESENTÉRICA SUPERIOR
25. Paciente de 3 semanas de vida con vómitos no biliosos y deshidratación moderada,
y se palpa un nódulo en epigastrio. La sospecha diagnóstica más adecuada
es:ESTENOSIS HIPERTRÓFICA DEL PÍLORO
26. Fernando de 17 años comete intento de suicidio mediante ingesta de ácido
clorhídrico. Considerando el tiempo de exposición al ácido, cuál de las siguientes
zonas será la más afectada por acción del ácido:LARINGOFARINGE DISTAL
27. Respecto a las glándulas del sistema digestivo, marque lo incorrecto:EXISTEN
GLÁNDULAS COMPLEJAS COMO LAS FÚNDICAS
28. Persona de 21 años en antesala para entrevista laboral por primera vez en su vida,
con miedo intenso. Presenta diaforesis, palidez y taquicardia. Con ese cuadro se
espera que la tonicidad de la saliva: DISMINUYA
29. En los conductos de las glándulas salivales tiene diferentes características
distintivas. Una de ellas es que las células madre se ubican generalmente a nivel del
conducto:INTERCALAR
30. El esfínter esofágico inferior se dilata durante:EL PASO DEL BOLO POR EL
CRICOFARÍNGEO
31. La desmineralización de los dientes es un proceso que lleva a la producción de
caries, y es directamente relacionado con:pH ÁCIDO
32. En una evaluación manonétrica del tubo digestivo normal, siempre se va a encontrar
que la mayor presión se encuentra a nivel de: C7
33. Las ondas lentas son producidas por una fluctuación del potencial de membrana,
producida principalmente por:SODIO
34. Paciente de 32 años con pirosis, sobre todo cuando se echa a dormir, o cuando
toma café, sometido a estudios radiológicos donde se evidencia una hernia hiatal
tipo I, por lo que se diagnostica Enfermedad por Reflujo Gastroesfágica, la cual se
debe debe a: (Marque la mejor respuesta): PÉRDIDA DE LA PLICATURA
DIAFRAGMÁTICA
35. La apertura de la boca incluye la contracción de los músculos del piso de la boca.
Dicha contracción puede asociarse directamente a la compresión la glándula:
(marque la mejor respuesta): Submandibular
36. Los nódulos linfáticos hiperplásicos señalados con flechas producen sensación de
dolor que viaja por el nervio:IX
37. Durante la formación del sistema digestivo, marque lo correcto: i. De la yema ventral
del páncreas, emergerá la papila mayor. ii. La herniación fisiológica debe dar un giro
completo de 360 grados. iii. Al retraerse las asas intestinales, el ciego se localiza
inicialmente en fosa ilíaca derecha. iv. El omento menor proviene del mesenterio
ventral → VFFV
38. El área señalada está constituida por un músculo inervado por el nervio: X

39. Paciente de 57 años con cáncer de páncreas, es sometido a


pancreatoduodenectomía donde se encuentra que el conducto pancreático de
Wirsung drena completamente en la papila menor. El diagnóstico más adecuado es:
Páncreas divisum
40. Una de las principales diferencias entre el epitelio de la mucosa masticatoria y la de
revestimiento es la: Queratinización
41. En relación a la macroestructura de las glándulas salivales, maque lo incorrecto: El conducto
de Stenon perfora el masetero
42. Respecto a las glándulas del sistema digestivo, marque lo incorrecto: Existen glándulas
complejas como las fúndicas
43. Ocasionalmente se puede presentar pneumoperitoneo debido a una solución de continuidad
transmural en la pared del esofágo, siempre que la lesión se produzca en: D
44. Paciente de 64 años acude a consulta quejándose de disfagia esofágica de inicio
insidioso. En los últimos tres días se ha agregado dolor a nivel esofágico inferior
asociado a la ingesta de alimentos. Este dolor viaja por los nervios: Esplácnicos
45. Paciente con síndrome de Sjögren avanzado, se podrá encontrar todos los
siguientes problemas, excepto: Caries dental y disminución de la digestión de los
lípidos
46. En la siguiente radiografía, indique a qué nivel se encontrará la cuarta estrechez del
esófago:LETRA D

47. El tipo de epitelio ubicado inmediatamente proximal a la línea Z es: Plano


estratificado no queratinizado
48. Es el músculo que más ayuda en prevenir la aspiración pulmonar: Cricofaríngeo
49. Durante la deglución, el paso del bolo alimenticio hacia la nasofaringe es
directamente evitado por varios músculos, entre ellos el: Tensor del velo del paladar
50. Debido a la retracción de las asas intestinales, la mayor parte del íleon se localiza
en: Cuadrante inferior derecho
51. Una de las principales funciones del páncreas tiene que ver con la digestión. Esta
función es estimulada por: Colecistoquinina
52. Con respecto a la cavidad peritoneal, marque la respuesta incorrecta: Los órganos
retroperitoneales no tienen serosa
53. Respecto a la distribución de vísceras intraabdominales, marque la respuesta
correcta: FVVV
- El páncreas es completamente retroperitoneal
- Los órganos retroperitoneales pueden tener serosa parcialmente
- El recto contiene adventicia
- La bolsa omental pertenece a la cavidad peritoneal
54. Paciente de 23 años se presenta a la emergencia con síndrome de Boerhaave. Cuál
de las siguientes anomalías pudo estar relacionada con la presentación de esta
rotura esofágica: Cricofaríngeo hipertónico
55. Ocasionalmente se puede presentar pneumoperitoneo debido a una solución de
continuidad transmural en la pared del esofágo, siempre que la lesión se produzca
en: D

56. Paciente varón de 31 años acude a emergencia por dolor en fosa iliaca derecha de
seis horas. Usted NO sospecharía del siguiente diagnóstico:
- De la yema ventral del páncreas, emergerá la papila mayor
- La herniación fisiológica debe dar un giro completo de 360 grados
- Al retraerse las asas intestinales, el ciego se localiza inicialmente en fosa
iliaca derecha.
- El omento menor proviene del mesenterio ventral.
- Torsión de ovario derecho
57. Durante la deglución, uno de los músculos longitudinales de la faringe es el
palatofaríngeo, cuya función es:
- Subir la faringe y cerrar el itsmo orofaríngeo
- Elevar la faringe y llevar la lengua hacia arriba y atrás
- Elevar la faringe y estrechar la cavidad faríngea
- Estrechar la cavidad faríngea y elevar la faríngea
58. Paciente de 57 años con cáncer de pancreas, es sometido a pancreatoduodenoctomía donde
se encuentra que el conducto pancreático de Wirsung drena completamente en la papila
menor. El diagnóstico más adecuado es: Páncreas divisum

AUTOEVALUACIÓN 9

PREGUNTAS RESPUESTA

La relajación receptiva gástrica es: Estimulada por la distensión gástrica

Uno de los personajes de la novela El Código Da Vinci Íleo intestinal


sufre una perforación gástrica por proyectil de arma de
fuego, como complicación se produce una peritonitis
química, ello hace que se presente ____________:

Los reflejos enterogástricos son desencadenados por Alta carga de carbohidratos en duodeno
diversas circunstancias, EXCEPTO:

Durante una cirugía, usted por error secciona los nervios estómago
vagos a nivel del diafragma. Por ello esperaría que haya
denervación a nivel de:

Paciente de 13 días de vida con vómitos explosivos a las vago


dos horas después de lactar. Al examen físico se palpa la
oliva pilórica. ¿Cuál es el nervió cuyos filetes dan
inervación eferente a la estructura afectada?

Estudiante de medicina vomita minutos antes de rendir su Centro del vómito en el tallo encefálico
examen final, debido al estrés que le produce rendir
exámenes. Este reflejo del vómito fue iniciado por
activación del núcleo:

Paciente de 63 años con shock hipovolémico al que se le Menor secreción de HCL


inicia tratamiento con noradrenalina endovenosa para
mejorar la presión arterial, usted sabe que se presentará:

¿Cuál de los siguientes puede inhibir la relajación receptiva Atropina


en el estómago:

En la motilidad gástrica sucede que la frecuencia de Gastrina / Secretina


potenciales y la fuerza de contracción son estimuladas por
la ____________ y reducidas por acción de la
____________:

La regulación extrínseca de la motilidad gástrica se integra Ganglio celiaco


en el:

El esfínter esofágico inferior se contrae al pasar el alimento Estómago


al:

Para que se realice la defecación se requiere: (Marque la Movimientos en masa


mejor respuesta

¿Cuál de las siguientes sustancias tendrá un vaciamiento Agua con azúcar


gástrico más rápido?

Mejoran la digestión de las grasas, excepto: Motilina

Al ingerir un pedazo de mantequilla, cuál de los siguientes CCK


tiene un efecto directo en la reducción del vaciado gástrico:

Al llegar el bolo al intestino, se genera un reflejo llamado Péptido intestinal vasoactivo


enterogástrico, el cuál utiliza como sustancia neurocrina:

Para que se realice la defecación se requiere: (Marque la Movimientos en masa

mejor respuesta)

AUTOEVALUACIÓN 10

PREGUNTAS RESPUESTA

La vena central terminal desemboca en venas sublobulillares

En paciente con hipertensión portal, la causa Varices esofágicas


más probable para una hematemesis es

En relación a la fisiología gástrica, marque lo la cimetidina actúa en la región basolateral de la


correcto célula parietal

El alcohol y la cafeína estimulan la fase gástrica


______________ de la producción de ácido
clorhídrico:

Para protegerse del entorno ácido, el urea en NH3


Helicobacter pylori genera un entorno de pH
menos ácido alrededor suyo, gracias a una
enzima que alcaliniza su entorno local mediante
la conversión de:

Paciente de 55 años con hipertensión portal Dilatación de venas en el ligamento falciforme,


grave. ¿Cuál de las siguientes es una que se anastomosan con venas del
característica de esta patología? mesogastrio.

Dentro de los factores protectores de la mucosa Receptor muscarínico


gástrica se pueden mencionar múltiples
protagonistas. Uno de ellos es:

En relación a la circulación hepática, marque lo Los sinusoides hepáticos transportan sangre


correcto: mixta

La bilirrubina directa está aumentada en la Ser hidrosoluble


cirrosis hepática, y se excreta en la orina debido
a:
Con respecto a la secreción gástrica de HCl: A mayor secreción de HCl en el lumen gástrico,
mayor pH en la sangre venosa gástrica

Se realizó un experimento en el cual se inyectó células de kupffer


tinta china en el peritoneo de ratas de
laboratorio. Al realizarse una biopsia hepática
de dichos animales, se encontró que el tinte
negro fue fagocitado por

En relación a la histología hepática, marque lo La zona 3 se encuentra más cerca a la vena


correcto central lobulillar

En la anemia perniciosa, los anticuerpos oxinticas


destruyen las células:

Marque lo correcto: Las venas sublobulillares desembocan en las


venas hepáticas

Paciente de 74 años de edad con shock Hipoxia en zona 3 del lobulillo hepático Infarto
mucoso del intestino
hipovolémico asociado a deshidratación aguda
severa. Cuáles de las siguientes
complicaciones es posible encontrar en este
paciente:

Dentro de las funciones de las células esteladas Pueden fagocitar patógenos y actúan como
o de Ito, marque lo incorrecto: presentadoras de antígeno

Las ramas más pequeñas del árbol biliar son: Canalículos biliares

El omeprazol actúa sobre la membrana Apical/Parietal


_____________ de la célula
____________

La célula mucosa del cuello gástrico moco


produce:

Autoevaluación 11

Paciente con mutación de la enzima UDP bilirrubina indirecta


glucoronil transferasa que la hace menos activa,
al hacerle un análisis de sangre, encontraremos
valores elevados de (marque la mejor
respuesta):

En relación a la secreción biliar y su Las sales biliares corresponden al 50% de su


composición, marque lo correcto: composición

Se realizó un experimento en el cual se inyectó Células de Kupffer


tinta china en el peritoneo de ratas de
laboratorio. Al realizarse una biopsia hepática
de dichos animales, se encontró que el tinte
negro fue fagocitado por:
La bilirrubina directa aumentada en cirrosis ser hidrosoluble
hepática se excreta en la orina debido a:

En paciente con hipertensión portal, la causa varices esofágicas


más probable para una hematemesis es

En cuanto a la secreción de iones y agua en los Es estimulada por la secretina


conductos biliares es correcto lo siguiente:

Si existe un fármaco con acción colerética, se Recirculación de sales biliares


asume que se refiere a que potencia o estimula
la:

En un hombre de 55 años con hipertensión Dilatación de venas en el ligamento falciforme,


portal grave. ¿Cuál de las siguientes es una que se anastomosan con venas del
característica de una hipertensión portal grave? mesogastrio.

En cuanto al urobilinógeno, se puede decir que Recircula hacia el hígado


es cierto:

Debido a su alto pKa, los ácidos biliares absorción


producidos por la desconjugación por las
bacterias intestinales se encuentran en su
forma no ionizada, lo cual facilita su:

Las sales biliares son absorbidas a la sangre en Íleon

El efecto colerético de las sales biliares se Estímulo de secreción biliar


refiere a:

Paciente de 74 años de edad con shock Hipoxia en zona 3 del lobulillo hepático Infarto
mucoso del intestino
hipovolémico asociado a deshidratación aguda
severa. Cuáles de las siguientes
complicaciones es posible encontrar en este
paciente:

Durante una endoscopía, se ingresa en la Quenodesoxicólico


papila mayor en la segunda porción del
duodeno, y dentro de la vía biliar extrahepática,
se toma una muestra de secreción biliar. Dicha
muestra debe contener ácido:
Un signo característico en pacientes con Asterixis
encefalopatía hepática es:

Autoevaluación 12

Paciente de 42 años con adenocarcinoma Vena porta


ductular. La TC ha demostrado claramente que
el tumor está en el cuello del páncreas y que
hay un gran vaso ocluido. ¿Cuál de los
siguientes vasos estaría más probablemente
obstruido?

El triángulo hepátocistico es importante de Arteria cística


conocer en una colecistectomía, debido que
contiene:

Las invaginaciones del epitelio de la vesícula Senos de Rokitansky-Aschoff


biliar que se extiende incluso hasta la muscular
se denomina:

Una mujer de 43 años dolor en hipocondrio Conducto colédoco


derecho e ictericia. En la ecografía se evidencia
cálculos biliares. Estos cálculos lo más probable
es que se encuentren localizados en:

El acino pancreático difiere con el de las Contiene celulares centroacinares


glándulas salivales en:

Una mujer de 38 años ingresa en el hospital con


signos de colecistitis y cálculos biliares. Durante Válvula espiral (de Heister)
la colangiografía, se inserta el catéter en la
vesícula biliar con mucha dificultad. ¿Cuál de
las estructuras interfiere con más probabilidad
con el paso del catéter por el conducto cístico?

Hombre de 65 años con isquemia intestinal leve Pancreaticoduodenal superior e inferior


por oclusión aterosclerótica de la arteria
mesentérica superior, pero la irrigación
sanguínea colateral ha retrasado el inicio de la
necrosis. ¿Qué vasos ofrecen colaterales entre
el tronco celíaco y la arteria mesentérica
superior?

Paciente de 21 años con dolor intenso en Disminuida


arcada dentaria superior debido a emergencia
de la tercera molar. Frecuencia cardíaca 108
latidos por minuto, PA: 140/80 mmHg, agitado,
pálido, son sudoración fría, y boca con saliva
espesa. En este paciente se espera que la
secreción exocrina pancreática se encuentre:

La secreción de la colecistoquinina (CCK) se Intestinal


produce en la fase:

Una mujer de 49 años ingresa en el hospital con Bolsa de Hartmann


dolor en epigástrico que migra hacia el lado
derecho y atrás hacia la escápula, sin ictericia.
La ecografía muestra un gran cálculo biliar. ¿En
cuál de las siguientes estructuras es más
probable que se localice el cálculo biliar?

Paciente con tumor neuroendocrino productor bicarbonato aumentada


de secretina, debido a lo cual se puede esperar
que su secreción pancreática, comparada con
la de una persona sana en estado de bajo flujo,
tenga una concentración de:

La secreción acuosa pancreática inicial es Ductales


isotónica, está constituida por sodio, agua, entre
otros, y es producida por las células:

La secreción de agua y bicarbonato por el Intestinal


páncreas exocrino se da básicamente en la
fase:

La estructura afectada en la fibrosis quística es: CFTR

¿Por cuál de las siguientes células es secretada Acinares del pancreas


principalmente la pro enzima
procarboxipeptidasa?

Durante la digestión de las grasas, para que la Centroacinares


lipasa actúe adecuadamente se requiere que el
pH aumento en la luz intestinal, lo cual es
logrado, entre otros, por la secreción de las
células:

Cuando el alimento se encuentra en el VAGO


estómago, se produce la liberación de enzimas
pancreáticas básicamente debido a la acción
de:

CI3
1. Marque la relación correcta: VFFF
i. El amonio se produce principalmente en el intestino
ii. Hasta el 80% del colesterol se transforma en sales biliares
iii. La fructosa y la galactosa se convierten en glucosa en el hígado
iv. El factor de crecimiento hepatocitario fomenta la regeneración del hepatocito
2. Marque cuál de las siguientes sustancias puede ser usada como
proquinético:Gastrina
3. La relajación receptiva de la región oral del estómago empieza desde: El fenómeno
de la deglución
4. Para disminuir la hipertensión portal en un paciente con sangrado variceal
persistente, se decidió operarlo y crearle una comunicación porto cava funcionante.
Con sus conocimientos en fisiología hepática, Usted sospecha que, producto de este
tratamiento, el paciente podría presentar la siguiente complicación:Encefalopatía
hepática
5. Paciente de 72 años con ictericia por bilirrubina directa, asociada a obstrucción en el
colédoco. Se sabe que, en este caso, la bilirrubina conjugada pasará hacia la
circulación sistémica a través del canalículo biliar hacia:Vena centrolobulillar → Vena
suprahepática
6. Paciente de 23 años con gastritis crónica caracterizada por dolor en epigastrio al
consumir café o alcohol. Dicha sensación de dolor es llevada por los nervios
___________ y se integran en la médula a nivel de _________: Simpáticos/T8
7. Paciente con insuficiencia hepática crónica presenta sobrecarga de hierro debido
a:Disminución de hepcidina
8. Dentro del tratamiento integral de un paciente con cirrosis, se desea eliminar la
cabeza de medusa que presenta en la pared abdominal. Para ello se
requiere:Obliterar el ligamento redondo
9. En un paciente con hemorragia digestiva masiva por úlcera péptica, con hipotensión
refractaria, se evidencia hipertransaminasemia marcada. ¿Qué zona del acino cree
usted que se encuentra más afectado? ZONA 3
10. Paciente de 56 años con en un paciente con gastritis atrófica, metaplasia intestinal e
inflamación de toda la mucosa, como se observa en la microfotografía. Por todo ello,
se espera encontrar, además: Hipergastrinemia
11. Paciente con disfagia esofágica es evaluado con una resonancia, en la sospecha
que tenga una estenosis a nivel de la unión gastroesofágica, para lo cual se asume
que la imagen que contiene esta estructura está a nivel de la vértebra:T11

12. El urobilinógeno: Recircula hacia el hígado


13. Recién nacido con mutación de la proteína MRP2 del hepatocito, presenta ictericia
ictericia y ______: Coliuria
14. Paciente de 67 años con cirrosis es traído a emergencia en estado soporoso. En este evento
agudo, usted examina al paciente buscando la presencia de ___________ y confirmaría su
diagnóstico con un dosaje de ___________ en sangre: Asterixis / Amonio
15. En un recién nacido con atresia de vías biliares al nacimiento, usted espera
encontrar lo siguiente en el perfil hepático: Bilirrubina directa elevada
16. Paciente de 74 años con úlceras pépticas por gastrinoma irresecable, es sometido a
vagotomía total. Una de los efectos secundarios de dicha intervención es la
desaparición de la: Relajación receptiva
17. Durante la guardia nocturna en el Hospital Edgardo Rebagliati Martins, el residente de
segundo año tiene varios pacientes con dispepsia en la programación de endoscopias de
urgencia, según sus conocimientos, cuál de los siguientes pondría usted primero en la lista:
Hombre de 78 años con melena
18. Paciente de 18 años con vómitos persistentes es ingresada a la emergencia por
presentar síndrome de Mallory-Weiss. Es evidente que debe presentar
deshidratación moderada a severa, y también _______: Hipopotasemia
19. Paciente de 17 años con gastritis aguda por Helicobacter pylori presenta un estado
de hipersecreción ácida debido a: Disminución de somatostatina
20. La acción del hígado sobre el metabolismo de los carbohidratos y el mantenimiento
de la glicemia, se ve reflejada en la prueba de laboratorio denominada: Glucosa
sérica en ayunas
21. Paciente de 27 años con Covid-19 severo asociado a falla de múltiples órganos. En
el perfil hepático se espera encontrar elevado los valores de: Transaminasas
22. La relajación receptiva de la región oral del estómago empieza desde: El fenómeno
de la deglución
23. Respecto a la macroestructura del hígado, marque lo incorrecto: A través de los
sinusoides hepáticos, fluye sangre venosa
24. Paciente de 64 años con úlcera péptica perforada, con peritonitis química. En la
siguiente radiografía baritada superior se señala la úlcera con una flecha. Según sus
conocimientos en macroestructura, usted sabe que por la ubicación en la que se
encuentra la úlcera, la capa de la pared gástrica que no está comprometida es la:
Muscular oblicua
25. Paciente de 35 años con gastritis aguda, es sometido a endoscopía alta donde se
encuentra eritema en toda la mucosa. En la biopsia se encuentra una bacteria,
según se muestra en la microfotografía. Usted sabe que debido al entorno requerido
por esta bacteria, el mejor lugar para encontrarla es en: Antro

26. Llega a emergencia un paciente con hematemesis franca de dos horas de evolución.
Al examen físico, le encuentra dilatación de venas periumbilicales, plaquetopenia y
con albumina ligeramente disminuida. Usted sospecha que la alteración que explica
el cuadro se encuentra en: Vena gástrica izquierda
27. Paciente con insuficiencia hepática crónica presenta sobrecarga de hierro debido a:
disminución de la hepcidina
28. Paciente mujer de 48 años, previamente sana, con trombosis portal aguda, se espera
encontrar: tiempo de protrombina alargado
29. Paciente con hiperbilirrubinemia directa transitoria (Síndrome de Gilbert) es evaluado previo
a una cirugía por apendicitis aguda. Se espera encontrar: Coluria
30. Paciente de 27 años con Covid-19 severo asociado a falla de múltiples órganos. En el perfil
hepático se esperea encontrar elevado los valores de: Transaminasas
Autoevaluación 13

Paciente de 48 años con enlentecimiento del Sobrecrecimiento bacteriano


tránsito intestinal por diabetes mellitus tipo II, y
presenta esteatorrea, flatulencia y malabsorción
de grasas. Una causa de disminución de sales
biliares puede ser:

Paciente es sometido a resección de íleon Formación de micelas en la luz intestinal

distal. Se espera que este paciente presente


esteatorrea debida a disminución en la:

Para absorber el azúcar de mesa (sacarosa) SGLT-1 y GLUT5

usado para endulzar el café, se tiene que usar


utilizar el(los) transportador(es)
________________ de la membrana apical de
los enterocitos:

En la irrigación de los órganos abdominales


Pancreaticoduodenal superior e inferior
¿qué vasos ofrecen colaterales entre el tronco
celíaco y la arteria mesentérica superior?:

Paciente de 62 años con vólvulo de intestino Ligamento suspensorio del duodeno (de Treitz)

delgado e isquemia intestinal. ¿Qué estructura


se utiliza como punto de referencia para
determinar la posición de la unión
duodenoyeyunal?

En el yeyuno hay una absorción neta de NaCl / HCO3-

___________ mientras que en íleon hay una


absorción neta de ________________:
Para que se puedan digerir las grasas, es CCK

preferible que primero sean emulsificadas. La


hormona que estimula la liberación de las
sustancias emulsificadoras es:

Paciente de 42 años con dolor abdominal Pancreaticoduodenal posterosuperior


intenso y hematemesis. En la endoscopía se
observa una úlcera duodenal posterior
perforada con hemorragia intrabdominal. ¿Cuál
de las siguientes arterias estará comprometida?

Los monosacáridos que conforman la lactosa SGLT-1

ingresan al enterocito usando el(los)


transportador(es) _____________:

¿Cuál de los siguientes requiere de la formación Vitamina K

de micelas para su absorción intestinal?:

Paciente con deficiencia congénita de Triglicéridos


procolipasa, sufre de esteatorrea cada vez que
come comidas ricas en grasas.Los lípidos que
no son hidrolizados debido a esta deficiencia
son:

Paciente de 10 años con diarrea crónica, Atrofia de las vellosidades


distensión abdominal, anorexia. Se le ha
encontrado anticuerpos antigliadina y
antiendomisio. Es probable que la diarrea se
correlacione con el hallazgo histológico de:
El azúcar de mesa (sacarosa) es digerido a dos GLUT2
monosacáridos que comparten el transportador
___________ para su absorción:

A nivel de macroestructura, ¿cuál de las El yeyuno tiene menos grasa mesentérica que
siguientes características anatómicas es más el íleon.
útil para distinguir entre yeyuno e íleon?

Una mujer de 45 años ingresa en el hospital con Arteria mesentérica superior.


síntomas de obstrucción intestinal superior. En
la TC se encuentra que la tercera porción
(transversa) del duodeno está comprimida por
un gran vaso. ¿Un aneurisma gigante de cuál
de los siguientes vasos causará muy
probablemente la obstrucción?

Autoevaluación 14

Agua
El principal componente de las heces es:

Paciente de 17 años, con diarrea crónica sin Osmótica


moco ni sangre, asociado a dolor abdominal,
con antecedente de masticar de 6 a 10 tabletas
de chicle Trident que contiene sorbitol. El
mecanismo de este tipo de diarrea es:

Durante la defecación se requiere: Señales inhibitorias en el nervio pudendo

Paciente de 51 años con antecedente de


Mesentérica inferior
enfermedad diverticular acude a emergencia
por sangrado profuso y dolor en hipocondrio
izquierdo. ¿Cuál es el origen más probable de
la sangre que pierde el paciente?
En condiciones fisiológicas, en una persona VIP

sana, hay una sustancia que aumenta los


niveles de AMPc intracelular en las células
epiteliales de las criptas intestinales, logrando
que el canal de cloro CFTR quede abierto.
Dicha sustancia es:

Las heces tienen un olor característico, cuyo Mercaptanos

responsable es:

Un tumor neuroendocrino productor de péptido Secretora

vasoactivo intestinal (VIP), el cual mantiene


abierto los canales de cloro de la cripta
colónica, se presentará con una diarrea de tipo:

El aumento de la presión intraluminal en el Divertículos

sigmoides de manera crónica produce


evaginaciones de la mucosa del colon en las
zonas más débiles de su pared, a veces con
material fecal, denominadas:

En los pacientes con diarrea asociada al Atrofia de las vellosidades

consumo de pastas o pan, y con anticuerpos


antigliadina, la malabsorción se produce
básicamente por:

El compromiso intestinal en Enfermedad de Ileocólica

Crohn es clásicamente en la región:

Los movimientos en masa son importantes en la Deseo de defecar

fisiología intestinal. Estos en movimientos en


masa causan cuál de los siguientes eventos:
El Vibrio Cholerae produce diarrea porque: Aumenta la producción de AMPc en los
enterocitos

en los neonatos sanos, el color verde oscuro del pigmentos biliares

meconio(primera deposicion) se debe a:

Paciente de 87 años, postrado, con dolor infarto transmural intestinal

abdominal y diarrea exudativa. Antecedente de


infartos de miocardio, y fibrilación auricular sin
tratamiento. Se sospecha la presencia de
múltiples émbolos arteriales, asociados a

En la enfermedad de Crohn es posible Mantener la inmunidad innata


encontrar células de Paneth en el colon. Esto se
puede deber a que una función de estas células
es:

En un paciente con diarrea se puede encontrar: Acidosis metabólica con hipocloremia

CI4
1. En una biopsia de intestino delgado se ha realizado una inmunotinción para poder
ver mejor las células de Paneth. Al microscopio, se observa que se ha teñido muy
bien: Base de la cripta
2. Paciente de 78 años postrado con fibrilación auricular, sufre diarreas y se sospecha
que tiene una isquemia intestinal crónica. Se realiza una biopsia de íleon distal, en
donde se espera encontrar por lo menos isquemia en: (marque la mejor
respuesta):Vellosidades
3. El intestino delgado se divide en tres porciones. Marque lo correcto en relación a su
anatomía: El ileon tiene menos válvulas conniventes
4. El consumo de alcohol etílico (bebidas alcohólicas) está asociado a ciertas
características especiales en su metabolismo. Marque la respuesta correcta: VFFV
- El daño hepático está asociado a acetaldehído
- La vía del sistema microsómico de oxidación del etanol lleva a la producción
de acetato
- El etanol induce a las células esteladas que pueden estimular la
transformación maligna de los hepatocitos
- El etanol induce a las células esteladas para que produzcan colágeno
5. En los movimientos en masa del colon, las haustras del mismo desaparecen en
sentido: Distal al bolo fecal
6. Con respecto a la estructura señalada con una letra X en esta colangioresonancia
(vías biliares), es cierto que: En sus paredes se reabsorbe sodio, cloro y agua

7. Se realiza una resonancia magnética en un paciente con dolor en epigastrio en


forma de cinturón, y se observa un tumor en el páncreas (flecha). Basado en la
localización, se puede afirmar que el tumor: El tumor no compromete el conducto de
Santorini

8. El primer evento que se produce en el fenómeno de la defecación es: Reflejo


rectoesfinteriano
9. Se le realiza una colangioresonancia (vías biliares) a un paciente, en la cual se
encuentra una obstrucción (cálculo biliar) en la zona marcada con una X . Este
hallazgo se correlaciona con la presencia de: Hiperbilirrubinemia directa
10. El sangrado de las hemorroides internas es principalmente de tipo arterial, debido a
que es sangre proveniente de la arteria: Mesentérica inferior
11. Paciente con deficiencia congénita de procolipasa, sufre de esteatorrea cada vez
que come comidas ricas en grasas. Los lípidos que no son hidrolizados debido a
esta deficiencia son: Triglicéridos
12. La secreción de agua en el intestino delgado se da principalmente por vía:
Paracelular
13. Para realizar una hemicolectomía derecha por cáncer de colon en colon ascendente,
el cirujano sabe que debe resecar el íleon distal debido a la irrigación de la arteria:
Ileocólica
14. Paciente de 82 años con trombosis de la arteria mesentérica superior, se espera que
no esté necrosado el: Duodeno
15. Durante una colonoscopía, se insufla CO2 en el colon para distender las paredes y
mejorar la visibilidad, lo puede comprometer las uniones estrechas entre los
enterocitos y producir: Traslocación bacteriana
16. Durante la digestión de los lípidos, se requiere la acción de la lipasa sobre las
micelas, para lo cual es imprescindible la acción de la: Colipasa
17. La secreción intestinal se produce principalmente a través de unos
canales:Aperturados por AMPc
18. En la enfermedad de Rotor se puede encontrar lo siguiente: Pruebas de función
hepática normales
19. Durante la absorción de nutrientes, ¿cuál de los siguientes no es absorbido como tal
en el intestino delgado? Triglicéridos
20. Paciente de 47 años con cirrosis severa, presenta somnolencia y desorientación.
Debido a ello, se espera encontrar en sangre una elevación de: Amoniaco
21. Paciente mujer de 48 años con incontinencia fecal, se le indica realizar ejercicios de
Kegel para reforzar el musculo puborectal. Este musculo se en carga de generar:
Convexidad perineal rectal
22. Paciente con una mutación parcial del canal de cloro CFTR presenta deficiencia de
la secreción intestinal y estreñimiento. Para aliviar este problema, se puede usar un
análogo de: Péptido intestinal vasoactivo
23. Paciente mujer de 32 años con dolor en hipocondrio derecho, e ictericia. Se realiza
exámenes de laboratorio. Se espera encontrar elevadas en sangre las enzimas
relacionadas con colestasis, como la: Gamma Glutamil Transpeptidasa
24. En un paciente con megacolon congénito, la constipación se produce por: Ausencia
de motilidad
25. Los ____________ son absorbidos por los enterocitos, pero no pasan por la
circulación portal: Lípidos
26. Paciente de 72 años con cirrosis hepática e hipertensión portal es evaluado por
control de enfermedad crónica, en donde se espera encontrar valores séricos
elevados de: Aldosterona
27. La acolia es la ausencia de pigmentación de las heces, y puede presentarse
en:Coledocolitiasis
28. El recto está íntimamente rodeado principalmente de: tejido graso
BANCO
DIGESTIVO
2021-1
BANCO DIGESTIVO 2020-2

1) Al examinar a un paciente, usted encuentra dolor localizado en fosa iliaca derecha y


diagnostica En este paciente, usted puede inferir.

a) El peritoneo parietal regional está afectado apendicitis.


b) El peritoneo visceral regional está principalmente afectado
c) Hay inflamación de todo el peritoneo parietal (peritonitis)
d) El diagnóstico está errado por no corresponder a la región abdominal adecuada

2) La motilidad intestinal es estimulada principalmente por el:


a) Plexo de Auerbach
b) Sistema simpático
c) Sistema parasimpático
d) Sistema piramidal

3) Durante el vómito, ¿el contenido gástrico tiene que pasar necesariamente por qué
estructura para llegar al esófago? Marque la mejor respuesta:
a) Cardias
b) Papila duodenal
c) Cuerpo
d) Antro

4) Respecto a la anatomía del estómago, marque lo correcto:


a) La incisura angularis puede estar en la curvatura mayor
b) El fondo gástrico forma la curvatura mayor
c) El píloro se encuentra en el cuerpo gástrico
d) La porción más distal del estómago es el cardias

5) Marque la respuesta incorrecta:


a) En todo el tubo digestivo, se observa dos capas de muscular propia: circular interna y
longitudinal externa
b) Fuera de la cavidad abdominal, el esófago presenta capa adventicia
c) La mucosa consta de epitelio, lámina propia y muscularis mucosae
d) El colon contiene tenias

6) Paciente se queja de dolor en hipocondrio derecho, pero superficialmente. El dermatoma


relacionado es (marque la mejor respuesta)
a) T9
b) T11
c) T10
d) T12

7) Dentro de las funciones del abdomen, se encuentra la defecación y micción, en las cuales
la presión intraabdominal debe:
a) Aumentar
b) No tiene relación el abdomen con dichas funciones
c) Disminuir
d) Mantenerse igual

8) Un alumno de medicina decide hacerse un piercing en el ombligo. Al realizarle el


procedimiento, sangra ligeramente. Esta sangre proviene de la arteria (marque la mejor
respuesta)
a) Epigástrica inferior
b) Circunfleja ilíaca superficial
c) Circunfleja ilíaca profunda
d) Torácica interna

9) Señala la respuesta correcta:


a) El apéndice cecal solo tiene serosa
b) El páncreas solo tiene adventicia
c) El colon ascendente solo tiene serosa
d) El esófago solo tiene adventicia

10) Paciente mujer es traída a emergencia por sufrir una herida contuso penetrante por
cuchillo realizada por su esposo en un ataque de celos. Se observa herida en flanco izquierdo.
Está solución de continuidad ha comprometido varios músculos de la pared abdominal,
excepto:
a) oblicuo externo
b) oblicuo interno
c) recto abdominal
d) transverso

11) Marque el órgano que se considera retroperitoneal:


a) Lóbulo izquierdo del hígado
b) Vesícula biliar
c) Sigmoides
d) Parte de la vía biliar

12) La peristalsis o peristaltismo hace referencia a:


a) Motilidad para movilizar el alimento de proximidad a distancia.
b) Motilidad para mezclado de alimentos.
c) No es parte de la motilidad
d) Motilidad para fraccionamiento de alimentos.

13) Paciente tiene una úlcera sangrante en el segundo tercio del yeyuno. La arteria de la cual
proviene la sangre arterial para dicha zona es la arteria:
a) Mesentérica superior
b) Iliaca común
c) Tronco celiaco
d) Mesentérica inferior

14) La estructura que fija órganos principalmente a la pared posterior abdominal se denomina:
a) Mesenterio
b) Omento
c) Ligamento
d) Fascia transversalis
15) Al iniciar la digestión, aumenta el consumo de oxígeno por la mucosa. Esto conlleva a una
hipoxia local, lo cual hace que se libere _____________, el cual produce vasodilatación:
a) Histamina
b) Noradrenalina
c) Adenosina
d) Colecistoquinina

16) Es inervado por aferentes somáticas:


a) Peritoneo parietal
b) Mesenterio
c) Omento
d) Peritoneo visceral

17) El ligamento hepatogástrico une el __________ con el _______ y forma la entrada al


________
a) Hígado Estómago Orificio omental
b) Estomago Higado Orificio anal
c) Estómago Hígado Orificio gastrointestinal
d) Hígado Estómago Orificio esofágico

18) Al retirar completamente el mesenterio de un órgano, el mismo se vería afectado


principalmente en su:
a) Irrigación
b) No se afecta en absoluto
c) Inervación
d) Tamaño

20) Paciente con vólvulo del colon sigmoides. La necrosis de este segmento del colon se
produce por una alteración en la irrigación de la arteria:
a) Mesentérica superior
b) Tronco celíaco
c) Mesentérica inferior
d) Pélvica

21) Marque el órgano que se encuentra más distal en el tubo digestivo.


a) Íleon
b) Ciego
c) Duodeno
d) Estómago

22) Paciente de 24 años con dolor abdominal tipo cólico intenso en mesogastrio. Según sus
conocimientos de macroestructura, el origen del dolor puede ser el ___________:
a) Esófago
b) Estómago
c) Íleon
d) Colón
23) Paciente con herida por proyectil por arma de fuego, con herida de ingreso en región
paraumbilical. Entre las estructuras que usted está seguro que debe haberse lesionado es:
a) Ligamento de Treitz
b) Omento mayor
c) Mesosigmoides
d) Omento menor

24) Paciente joven es traído a emergencia con abdomen agudo quirúrgico debido a herida
contuso penetrante por verduguillo (alambre grueso con punta aguzada) recibida en una
pelea después de un partido de fútbol. Se observa herida en Hipocondrio Izquierdo. El
órgano que debe estar sangrando y produciendo hemoperitoneo es (marque la mejor
respuesta):
a) Colon ascendente
b) Hígado
c) Bazo
d) Colon sigmoides

25) Al evaluar una tomografía abdominal, el médico asistente le pide al interno de la UPC
que encuentre la imagen con el corte a nivel de L1. El interno sabiamente busca el
________ para ubicar la vértebra L1.
a) Nacimiento de la vena mesentérica superior
b) Nacimiento de la arteria mesentérica inferior
c) Cuello del páncreas
d) Cruce entre la Aorta y la Vena Porta

26) En la inspiración, la pared abdominal debe ____________ para ____________:


a) Relajarse aumentar presión intra abdominal
b) Contraerse aumentar presión intra torácica
c) Contraerse aumentar presión intra abdominal
d) Relajarse disminuir presión intra torácica

27) Cuál de las siguientes estructuras no tiene vasos sanguíneos:


a) Epitelio intestinal
b) Omento
c) Mesenterio
d) Ligamento

28) Paciente con hipoglucemia secundaria a un insulinoma (tumor neuroendocrino productor


de insulina). El órgano donde con mayor probabilidad ha crecido este tumor es:

a) Pélvico
b) Torácico
c) Retroperitoneal
d) Intraabdominal

30) Sustancia que inhibe la secreción y la motilidad del estómago prolongando el tiempo de
digestión:
- Péptido insulinotrópico dependiente de la glucosa (GIP)
31) Marque lo correcto:
- La hernia fisiológica se produce en la sexta semana y es la salida temporal de
asas intestinales a través del cordón umbilical

32) Marque la respuesta correcta en relación a la gastrina:


- Las células G son las productoras y se encuentran principalmente en el antro
gástrico

33) El consumir caramelos indirectamente activa la vía:


- POMC/CART

34) ¿En qué capa se encuentra la alteración principal en el Hirschsprung o megacolon


agangliónico?:
- Muscular propia

35) Con respecto a las ondas lentas, marque la afirmación correcta:


- Son contracciones rítmicas espontáneas

36) El uso de Ranitidina bloquea el receptor H2 de la histamina en las células parietales. La


histamina llega a estas células por:
- Difusión

37) La triada sintomática: vómitos explosivos post-prandiales, movimientos peristálticos


epigástricos visibles de izquierda a derecha y nódulo palpable epigástrico subcostal
derecho, pertenecen a:
- Estenosis congénita hipertrófica del píloro.

38) Durante una cirugía oncológica, ¿la extirpación de cuál de los siguientes órganos se
vería comprometida por la presencia de adventicia?:
- Recto

39)En cuanto a los reflejos gastrointestinales, un reflejo que estimula el tránsito intestinal es
el reflejo:
- Gastrocólico
- Cólico-ileal
- Entero-gástrico
- Vómito

40) El ligamento falciforme divide al hígado en dos lóbulos derecho e izquierdo.


Embriológicamente deriva del:
- Mesenterio ventral

41) La presencia de atresias y estenosis duodenales se deben básicamente a una:


- Falta de recanalización

42) Estudiante de medicina de 20 años, se ha amanecido estudiando para su examen de


Sistema Digestivo. No ha probado alimento desde la cena, por lo que se puede afirmar que
la motilidad de esta persona está siendo regulada por:
- Motilina

43) Paciente con disminución del apetito marcada asociada a cáncer terminal, para
promover la ingesta de alimentos se podría usar análogos de:
- Endorfinas

44) Las ondas lentas se producen por la apertura cíclica de canales de:
- Calcio

45) La forma más común de atresia esofágica contiene:


- Estenosis proximal del esófago con fístula traqueoesofágica distal

46) Al deglutir un bolo alimenticio, es lógico suponer que al pasar por el esófago haya un
mayor consumo de oxígeno en la pared del tercio:
- Proximal

47) Paciente que come entera una pizza familiar de chorizo y queso. Es posible esperar que
debido a la cantidad de alimento ingerida, las ondas lentas hayan:
- Sufrido ninguna alteración en su frecuencia

48) La hernia fisiológica se produce dentro de:


- Cordón umbilical

49) El crecimiento de un adenocarcinoma de páncreas compromete la pared gástrica por


contigüidad. ¿Qué parte del estómago se esperaría esté comprometido?
- Pared posterior del antro

50) Estimula la producción de saliva:


- Vasodilatación periglandular

51) Durante la secreción de saliva, es de esperarse que las concentraciones de ________ y


______ disminuyen al disminuir el flujo:
- Sodio Bicarbonato

52) Con respecto a la secreción gástrica de HCl:


- a mayor secreción de HCl en el lumen gástrico, mayor pH en la sangre venosa
gástrica

53) Respecto a las enfermedades del esófago, marque lo correcto:


- el diagnóstico diferencial de la acalasia es la enfermedad de Chagas esofágica

54) Con respecto a las lesiones y enfermedades de la boca, marque lo correcto:


- la eritroplasia debe ser biopsiada

55) Respecto a las glándulas salivales, marque lo incorrecto:


- la glándula sublingual tiene forma de garfio

56) El omeprazol actúa sobre la membrana _____________ de la célula ____________


- apical / parietal
57) Durante el sueño, la concentración de bicarbonato en la saliva:
- Disminuye

58) Durante el ataque con gas sarín (bloqueador de la acetilcolinesterasa) en el metro de


Tokio, en 1995, el personal de salud notó que los pacientes afectados presentaban:
- Hipersalivación

59)La célula mucosa del cuello gástrico produce:


- Moco

60) En la evaluación de una tomografía abdominal, el interno observa un aneurisma en una


arteria que se dirige al riñón derecho. Con seguridad se puede afirmar que está a nivel de la
vértebra:
- L1

61) Un un paciente con diarrea por hipermotilidad, usted sospecharía en el posible aumento
de las siguientes sustancias, excepto:
a) Péptido intestinal vasoactivo
b) Sustancia P
c) ACh
d) Motilina

62) Una de las siguientes sustancias no comparte con las otras la misma acción sobre la
producción de ácido gástrico:
a) Péptido insulinotrópico dependiente de glucosa (GIP)
b) Colecistoquinina
c) Somatostatina
d) Secretina

63) Usted encuentra músculo estriado en el siguiente segmento:


a) Esfínter anal externo
b) Duodeno
c) Tercio inferior esofágico
d) Esfinter anal interno

64) Paciente con intoxicación por organofosforados (inhibidores de acetilcolinesterasa), se


espera que el tránsito intestinal se encuentre:
a) Aumentado
b) Disminuido
c) En relación inversa al número de ondas lentas por minuto
d) No se espera ninguna alteración

65) Al disminuir el pH duodenal por el HCl gástrico, se libera principalmente una hormona
cuya célula diana es:
a) Células S del intestino
b) Acinos pancreáticos
c) Células ductales del colédoco
d) Célula ductal del wirsung
66) La hormona que tiene un efecto sinérgico con la secretina para optimizar el pH duodenal
y la digestión, es:
- CCK (colecistoquinina)

67) El ligamento falciforme del hígado proviene embriológicamente de:


- Mesenterio ventral
-

68) Todos los músculos motores de la lengua están inervados por el XII par, excepto:
- Palatogloso

69) Al ingerir grandes cantidades de dulces, con la subsecuente estimulación de incretinas,


usted esperaría que el apetito ______________, debido a __________________
- Disminuya insulina

70) Paciente obeso con Covid-19 es intubado por interno inexperto, quien al solicitar que
bombeen aire dentro del tubo endotraqueal, nota que el epigastrio se distiende. Al sospechar
que ha introducido el tubo en el estómago, también es cierto que:
- Disminuye el pH gástrico
- Aumenta el pH gástrico
- Disminuiría el tono del píloro
- Aumenta la frecuencia de ondas lentas

71) Una de las siguientes sustancias reguladoras, puede actuar de forma paracrina y como
hormona. Marque la correcta:
- Somatostatina
- Acetilcolina
- Péptido insulinotrópico dependiente de glucosa.
- GRP

72) Durante una cirugía oncológica, el cirujano observa que los órganos abdominales tienen
libre movimiento dentro de la cavidad abdominal, excepto:
- Yeyuno
- Vesícula biliar
- Colon ascendente
- Estómago

73) Al consumir un pan con mantequilla, la sensación de hambre disminuye debido a la


acción de:
- Grelina
- Somatostatina
- Colecistoquinina (CCK)
- Leptina

74) Respecto a los péptidos gastrointestinales, marque lo correcto.


- No existe sustancia neurocrina que tenga efecto en la motilidad del tubo digestivo
- Las sustancias paracrinas pueden viajar a través de vasos sanguíneos
- Las sustancias neurocrinas son péptidos que hacen su efecto en distancias cortas
- Las sustancias paracrinas atraviesan la circulación portal

75) En un paciente con gastroparesia (motilidad lenta del estómago), que presenta
distensión abdominal después de comer, usted le recomendaría que evite el consumo de
lípidos y aminoácidos para disminuir la acción de:

- Secretina
- CCK
- Somatostatina
- Gastrina

76) Paciente con apendicitis aguda, que debuta con dolor en mesogastrio. Este dolor se
debe a estimulación de receptores del dolor cuyas fibras van a viajar a la médula espinal a
través de:

● Nervio vago
● Plexo hipogástrico
● Nervios simpáticos
● Nervio esplácnico pélvico

78) Al introducir una solución azucarada directamente al estómago mediante una


gastrostomía (comunicación entre la piel abdominal y el estómago), la sustancia que
provocará que aumenten los niveles séricos de insulina es:

● Somatostatina
● Péptido tipo glucagón 1 (GLP-1)
● Glucagón
● Enteroglucagon

79) En un experimento, con una sonda nasogástrica se instila por goteo en el estómago una
sustancia líquida, y se obtiene como respuesta una dramática disminución del pH del
estómago. Dicha sustancia debe contener:

● Lípidos
● Carbohidratos
● Aminoácidos
● Secretina

80) En un paciente con hiperestimulación simpática se espera que las ondas lentas tengan
un ritmo:

● Mayor en estómago que en duodeno


● Mayor en estómago que en íleon terminal
● Menor en íleon terminal que en el duodeno
● Mayor en íleon que en duodeno

81) Se considera que el gusto puede viajar a través del nervio:

- V3
- Vago
- Lingual
- Glosofaríngeo

82) Al seccionar el nervio facial a nivel timpánico, usted esperaría:

- Disminución del gusto en la punta de la lengua


- Ausencia de termoalgesia en la lengua
- Imposibilidad para protruir la lengua
- Imposibilidad para el cierre del istmo de las fauces.

83) El ecografista sabe que para poder visualizar el nacimiento de la arteria mesentérica
superior, debe colocar el transductor sobre la piel de la siguiente región abdominal:

- Hipogastrio
- Hipocondrio derecho
- Mesogastrio
- Epigastrio

84) Marque lo correcto en relación al divertículo de Meckel.

- Se encuentra usualmente a 60 cm de la VIC


- Contiene mucosa esofágica en algunas ocasiones
- Se relaciona a un defecto en el desarrollo del intestino posterior
- Se produce en el lado mesentérico del íleon

85) Es un ligamento derivado del mesenterio dorsal:

- Gastrocólico
- Hepatogástrico
- Redondo
- Faciforme

86) Marque lo correcto respecto a la siguiente imagen:


- Produce síntomas en la gran mayoría de pacientes.
- Se produjo por giro en sentido contrario del duodeno
- Se produjo por falta de fusión de los ductos dorsal y ventral
- Como tratamiento, se podría aperturar la papila mayor (esfinterotomía).

TIPEO DE EXÁMENES
1) La arteria mesentérica superior emerge de la aorta a nivel de:
- Cabeza de páncreas
- Hilio hepático
- Tronco celiaco
- Íleon
- Cardias

2) La digestión de las proteínas se inicia en:


- Estómago
- Intestino delgado
- Boca
- Esófago
- Páncreas

3) Aproximadamente en la semana 6 del desarrollo embrionario, el intestino intermedio


gira 90º herniándose a nivel del:
- Fístula esofágica
- Borde superior del hígado
- Borde superior del pubis
- Cordón umbilical
- Borde inferior del bazo

4) Paciente de 76 años en estado de coma, con colostomía (colon abocado a la piel) por
resección parcial de colon secundaria a carcinoma de colon. Usted deja indicado que
este día se realice el cambio de la bolsa de colostomía, pero que lo hagan después
de dos horas de la nutrición enteral, debido a que quiere que se haga después de:
- La presencia de ondas lentas
- La secreción de gastrina
- El reflejo gastrocólico
- La producción de GIP
- La liberación de motilina
5) Recién nacido de 14 días, que presenta estreñimiento, distensión abdominal y
vómitos. Al examinar el orificio anal, se evidencia conducto permeable, pero al
introducir un poco el termómetro, se evidencia salida de material fecal por el ano.
Usted sospecha que el problema se deba a:
- Falta de desarrollo del tabique urogenital
- Defecto en el desarrollo del conducto vitelino
- Defecto en el desarrollo de las células de cajal
- Falta de desarrollo del seno uretral
- Falta de regresión de la membrana anal

6) La lengua está recubierta por epitelio:


- Plano estratificado no queratinizado
- Pseudoestratificado columnar no queratinizado
- Pseudoestratificado columnar ciliado
- Plano estratificado queratinizado

7) El esfínter anal interno tiene musculatura ___ y tiene control ___:


- Lisa/voluntario
- Lisa/involuntario
- Esquelética/simpático
- Esquelética/parasimpático

8) La arteria aorta proporciona la irrigación al tubo digestivo ¿cuál de las siguientes


arterias de irrigación al ángulo cólico derecho?
- Mesentérica superior
- Mesentérica inferior
- Frénica inferior
- Tronco celiaco
9) Paciente de 26 años que le cuenta en su historia clínica que cada vez que almuerza,
a los 20 minutos tiene deseo de defecar. Le comenta que su hijo de 1 mes le pasa lo
mismo pero más intenso. Esto se explica por el reflejo ____, el cual está _____ en el
paciente:
- Colicoileal/normal
- Colicoileal/alterado
- Gastrocolico/normal
- Gastrocolico/alterado

10) La región del estómago que se comunica con el duodeno se denomina


- Pilórica
- Cardias
- Cuerpo
- Fórnix
11) Acude a consulta un paciente que fue diagnosticado de úlcera péptica tres días antes.
Luego de múltiples pruebas diagnósticas, se concluye que el paciente presenta un
tumor secretor de gastrina ¿cual de las siguientes situaciones estará incrementada?
- Distensión gástrica
- Inhibición del vaciado gástrico
- Secreción de ácido clorhídrico (HCl)
- Inhibición de la secreción de pepsinógeno

12) En el sistema digestivo el control del apetito está dado por un complejo sistema de
sustancias y órganos integradores los cuales regulan la ingesta de alimentos. La ____
es una sustancia orexígena/orexígena y es sintetizada por el ____
- Leptina/intestino
- Grelina/intestino
- Leptina/estómago
- Grelina/estómago

13) Sobre el control Autónomo del sistema digestivo marque la alternativa correcta:
- La inervación dada por el sistema simpático es de tipo preganglionar
- El sistema parasimpático usa como neurotransmisores a la acetilcolina y
noradrenalina
- El nervio vago (par craneal X) le da inervación simpática a la mayoría del sistema
digestivo
- En el sistema simpático, los nervios responsables hacen una primera sinapsis en
ganglios próximos al órgano a inervar
- En la inervación de tipo parasimpático, solo interviene el plexo submucoso, sin
embargo, en la de tipo simpático intervienen tanto el submucoso como el mientérico.

14) Con respecto a la actividad eléctrica del sistema digestivo, marque la alternativa
correcta:
- Corresponden a potenciales de acción que están presentes de forma continua y le dan
capacidad de peristalsis autónoma al sistema digestivo
- La frecuencia de las ondas lentas no se ve influenciada por la actividad neural ni las
hormonas gastrointestinales
- En el estómago las ondas lentas se dan en una frecuencia de 6 por minuto
- Las ondas lentas son cambios lentos y ondulantes del potencial en reposo
- La frecuencia de las ondas lentas va de 6 a 12 ondas por minuto

15) Ante una lesión en el IX par craneal, el músculo ___ se altera en su función:
- Palatogloso
- Estilofaríngeo
- Palatofaríngeo
- Constrictor superior

16) Un varón de 50 años es sometido a extirpación de duodeno y parte proximal del


yeyuno. La pérdida de estímulo hormonal en el páncreas para la secreción enzimática
se explica por la pérdida de las células:
- Parietales, productoras del factor intrínseco
- “K” productoras del factor intrínseco
- “M” productoras de CCK
- “I” productoras de CCK

17) Respecto al mecanismo de defecación ¿cual de las siguientes afirmaciones es


correcta?
- Se produce una contracción refleja del esfínter anal interno
- Se produce contracción o relajación del esfínter anal externo por señales de la corteza
cerebral
- La presencia de materia fecal en el recto estimula la contracción del sigmoides por los
nervios pélvicos simpáticos
- En la posición de “cuclillas” el músculo puborrectal se haya contraído favoreciendo la
evacuación de la materia fecal

18) Un niño de tres años llega a emergencia con disfagia (dificultad para tragar), salivación
y llanto. Se sospecha de ingesta de cuerpo extraño (moneda en el esófago), al ser
evaluado se constata de una radiografía presencia de cuerpo extraño a nivel de C6 y
C7 (6º y 7º vértebra cervical). El cuerpo extraño estará suspendido a nivel del
estrechamiento producido por el:
- Cayado aórtico
- Hiato esofágico
- Músculo cricofaríngeo
- Bronquio principal izquierdo

19) En el caso de un paciente con un tumor productor de gastrina, la presencia de úlceras


duodenales y erosión de la mucosa gástrica se debe principalmente a:
- La acción paracrina de la gastrina sobre la célula parietal
- El exceso de HCl por estímulo de receptores CCK-B en la célula parietal
- La sobre expresión de los receptores “G” para gastrina en la célula parietal
- El exceso de HCl por estímulo directo de receptores “H” en la célula parietal

16) La onda peristáltica secundaria del esófago se caracteriza por ser originada:

- Por el plexo de submucoso del esófago


- Por el plexo mientérico del esófago
- Por el reflejo de la deglución
- Durante la masticación

17) Marque lo correcto sobre las ondas lentas en el tubo digestivo:

- No son despolarizaciones
- Son potenciales de acción subumbrales
- Se constituyen de despolarizaciones y repolarizaciones
- Son rítmicas y generadas por el sistema nervioso autónomo

18) Recién nacido presenta protrusión de contenidos abdominales los cuales no están
cubiertos por peritoneo y salen de la cavidad abdominal a través de un defecto en la pared
¿Cómo se denomina a la afección que presenta este paciente?

- Onfalocele
- Atresia biliar
- Gastrosquisis
- Divertículo de Meckel

19) Experimentalmente se utiliza atropina (anticolinérgico) para inhibir la secreción de


gastrina, sin embargo, la secreción de esta hormona se sigue dando ante estímulos vagales.
Esta situación se explica porque la atropina:

- No bloquea la acción del péptido GRP


- Solo inhibe la acción del péptido GHRP en la célula G
- Inhibe la acción de acetilcolina e histamina en la célula G
- Bloquea parcialmente la bomba de protones en la célula G

20) La niña de 4 días es llevada a la emergencia pediátrica por presentar llanto constante,
la madre refiere coloración azulada de labios al momento de lactar, acompañado de
tos persistente y dificultad respiratoria, así como distensión abdominal. Se le coloca
sonda nasogástrica para alimentación notando que retorna a la cavidad oral en todos
los intentos ¿Cuál es la anomalía del desarrollo en este caso?
- Solo fistula traqueo esofagica
- Fístula traqueo esofágica proximal y distal
- Atresia esofágica proximal con fístula traqueo esofágica distal
- Atresia esofágica distal con fístula traqueoesofágica proximal

21) Paciente varón de 36 años es traído a la emergencia luego de sufrir un accidente de


tránsito presenta traumatismos múltiples en cabeza y tronco. Al examen físico se
evidencia hematoma en hemicara izquierda, ligera protrusión y caída del lado
izquierdo del maxilar inferior por lo que se le realiza tomografía donde se haya una
fractura de apófisis coronoides del maxilar inferior. ¿Qué músculo está relacionado
directamente con esta situación?
- Masetero
- Temporal
- Buccinador
- Pterigoideo medial

22) Un paciente refiere no percibir algunos sabores. Al examen físico constata alteración
del sabor dulce y umami. ¿Cuál de los siguientes nervios estará alterada su función?
- Cuerda del tímpano (VII par)
- Lingual (rama del V par)
- Glosofaríngeo (IX par)
- Hipogloso (XII par)

23) A los pocos días de nacido, regresa a neonatología un niño con problemas de
motilidad del colon, los estudios determinan ausencia congénita de células
ganglionares. Según el gráfico cuál es la capa en la que se determina la ausencia de
dichas células?
- Mucosa - 2
- Muscular propia - 2
- Muscular propia - 3
- Muscular de la mucosa - 1

24) Con respecto al control autonómico en el tracto gastrointestinal y en relación con su


fisiología ¿Cuál es la función del sistema nervioso parasimpático y el tracto
gastrointestinal?
- Inhiben la contracción muscular y estimulan la secreción de sustancias a nivel de la
submucosa
- Estimulan la contracción muscular y estimulan la secreción de sustancias a nivel de
la mucosa
- Inhiben la contracción muscular e inhiben la secreción de sustancias a nivel de la
submucosa
- Estimulan la contracción muscular e inhiben la secreción de sustancias a nivel de la
mucosa

25) Intersticio ubicado entre el estroma del espacio portal y los hepatocitos, y por donde
migran las células cancerígenas que hacen diseminación linfática es el:
- Espacio del Mall
- Espacio de Disse
- Espacio porta
- Espacio sideral

26) La presencia de grandes cantidades de TGF - Beta estimula a las ___________ y se


deposita colágeno, formándose la cirrosis
- Células de Ito
- Células de Kupffer
- Triadas portales
- Células fibroblásticas

27) Paciente de 64 años con ICC al que se le va a realizar cirugía cardiovascular. Al


calcular el volumen sanguíneo total, se debe considerar que el hígado puede
contener un volumen de sangre de ____________ mL en un adulto sano, en este
paciente ese volumen puede llegar a ser de _____________ mL
- 450 - 1000
- 250 - 1000
- 450 - 3000
- 250 - 10000

28) El área del lobulillo que se afecta más en caso de hipoxia es la zona:
- 3
- 2
- 1
- 4

29) Paciente con carcinoma de vesícula biliar. La metástasis por continuidad afectará al
lóbulo:
- Cuadrado
- Lobulado
- Caudado
- Izquierdo

30) El ácido acetilsalicílico actúa en la membrana:


- Basolateral de la célula parietal
- Basolateral de la célula principal
- Apical de la célula principal
- Apical de la célula parietal

31) Al realizar una esplenectomía, se tiene que resecar la arteria esplénica, lo cual no es
problema para el estómago por que la arteria gastroomental izquierda se
anastomosa con la:
- Gastroomental derecha
- Gástrica derecha
- Gastroduodenal
- Pancreaticoduodenal superior

32) La arteria esplénica proviene de la aorta y la vena esplénica desemboca en la vena:


- Porta
- Gástrica izquierda
- Mesentérica superior
- Gastroduodenal

33) Paciente mujer 21 con bulimia, que luego de un episodio de vómitos presenta
hematemesis y al examen físico que se encuentra crépitos subcutáneos cervicales.
El diagnóstico más probable es:
- Sd. Boerhaave
- Sd. Mallory - Weiss
- Varices esofágicas
- Épulis

34) Paciente varón de 60 años, con antecedentes de promiscuidad sexual, tabaquismo y


alcoholismo, acude a consulta por presentar disfagia progresiva, odinofagia y al
examen se observa tumor por parte posterior de la lengua. La mejor posibilidad
diagnóstica es:
- Carcinoma escamoso
- Esófaggo de Barret
- Mucocele
- Granuloma piógeno

35) Niño de 5 años con historia de tres días de evolución caracterizado por fiebre,
malestar general, odinofagia, anorexia, e irritabilidad. Al examen de observa lesiones
ulcerativas de 4mm de diámetro en mucosa yugal, con borde blanquecino y eritema
periférico. El diagnóstico más probable es:
- Aftas orales
- Herpes simple
- Candidiasis oral
- Leucoplasia

36) Es considerada una lesión preneoplásica


- Leucoplasia
- Granuloma piógeno
- Carcinoma escamoso
- Boerhaave

37) El esófago de Barrett se considera una lesión preneoplásica que se caracteriza por
la presencia en esófago de:
- Metaplasia intestinal
- Metaplasia gástrica
- Displasia gástrica
- Neoplasia glandular

38) Paciente mujer de 23 años gestante con lesión proliferativa en mucosa oral
producida por proliferación reactiva de vasos sanguíneos. Marque la mejor respuesta
- Epulis
- Granuloma piógeno
- Leucoplaquia
- Carcinoma epidermoide
39) La glándula parótida tiene principalmente acinos de tipo:
- Seroso
- Mucoso
- Seromucoso
- Mixto

40) La reabsorción de sodio y secreción de potasio es estimulada por:


- Aldosterona
- Angiotensina II
- HAD
- Renina

41) Paciente con cirrosis hepática que tiene hipertensión portal con várices esofágica, y
actualmente presenta varices en estómago distal. Estas várices están relacionadas a
aumento en la presión de las venas:
- Gástrica derecha
- Gástrica izquierda
- Gástrica superior
- Gástrica inferior

42) La glándula parótida está inervada por el par craneal:


- IX
- X
- XII
- VII
- V

43) Paciente con cirrosis hepática que tiene hipertensión portal con varices en esofago
distal . Estas varices están relacionadas a aumento en la presión de las venas
gástricas
- Superior
- Izquierda
- Derecha
- Inferior

44) La información eferente que sale de los núcleos salivales superior e inferior a través
de los pares VII y IX hacia las glándulas salivales llevan información tipo
- Sensitivo
- Parasimpático
- Somático
- Simpático

45) Un niño de 4 años ingresa en el hospital con vómitos graves . En el estudio se


encuentra que el niño tiene un páncreas anular ¿ Cuál de las siguientes hormonas
gastrointestinales se encontrará a niveles elevados en sangre con mayor
probabilidad a raíz de esta patología
- GIP
- Gastrina
- Secretina
- VIP
46) Durante una colecistectomía laparoscópica en un hombre de 61 años ¿ Cuál de las
siguientes arterias debe pinzar para extirpar la vesícula biliar con seguridad?
- Hepática propia
- Hepática izquierda
- Hepática derecha
- Cistica

47) Un hombre de 34 años se somete a una apendicectomía de urgencia . Después de


realizar la apendicectomía satisfactoriamente , el paciente se somete a una
laparoscopia exploratoria¿Cual de las siguientes características anatómicas es más
útil para distinguir entre yeyuno e íleon?
- El yeyuno tiene menos grasa mesentérica que el íleon
- El yeyuno tiene más arcadas vasculares que el íleon
- El yeyuno tiene más folículos linfáticos bajo la mucosa que el íleon
- El yeyuno tiene menos vellosidades que el íleon

48) Una mujer de 45 años ingresa en el hospital con síntomas de obstrucción intestinal
superior .En la TC se encuentra que la tercera porción (transversa) del duodeno está
comprimida por un gran vaso ¿Cuál de los siguientes vasos causara muy
probablemente la obstrucción ?
- Arteria mesentérica inferior
- Vena mesentérica inferior
- Arteria mesentérica superior
- Vena porta

49) Durante una colecistectomía laparoscópica programada en una mujer de 47 años , el


residente pinchó accidentalmente el ligamento hepatoduodenal en vez de la arteria
cística ¿Cual de los siguientes vasos estaría muy probablemente ocluido en esta
lesión iatrogénica?
- Arteria hepática izquierda
- Arteria hepática propia
- Arteria esplénica
- Arteria mesentérica superior

50) Un hombre de 54 años ingresa en urgencias con intenso dolor abdominal superior .
La gastroscopia revela un tumor en el antro del estómago . Se pide una TC para
evaluar el drenaje linfático del estómago¿Cuál de los siguientes nódulos linfáticos
estará muy probablemente afectado en una neoplasia maligna del estómago
- Lumbar
- Mesentérico inferior
- Mesentérico superior
- Celiaco

51) Una mujer obesa de 45 años con fiebre alta acude a la consulta con náuseas y dolor
agudo e intermitente en el cuadrante superior derecho del abdomen de 2 días de
duración - Tiene una historia de ictericia de 24 horas . Tiene antecedentes de litiasis
biliar . Bilirrubina total del 10 mg/dL . Lipasa de 5 mg/mL ¿Cuál de las siguientes
estructuras está muy probablemente obstruida por un cálculo biliar ?
- Conducto colédoco
- Ampolla de Vater
- Conducto cístico
- Conducto pancreático

52) Una mujer de 45 años ingresa en urgencias con dolor abdominal intenso . La TC y
RM revelan un tumor de la cabeza del páncreas que afecta el proceso unciforme
¿Cual de los siguientes vasos es más probable que suministre irrigación a parte de
la zona afectada?
- Arterias yeyunales
- Arteria cólica media
- Arteria cólica izquierda
- Arteria ileocólica

53) Un hombre de 70 años ingresa en urgencias con diarrea intensa . La arteriografía


revela un bloqueo del 90% en el origen aórtico de la arteria mesentérica inferior
¿Cuál de las siguientes arterias proporciona muy probablemente irrigación colateral
al colon descendente?
- Arteria gastroomental izquierda
- Arteria cólica derecha
- Arteria sigmoidea
- Arteria cólica media

54) Al disminuir el pH duodenal por el HCL gástrico , se libera principalmente una


hormona cuya célula diana es
- Célula S del intestino
- Células ductales del colédoco
- Acinos pancreáticos
- Célula ductal de Wirsung

55) Una de las siguientes sustancias reguladoras , puede actuar de forma paracrina y
como hormona
- Péptido insulinotrópico dependiente de glucosa
- GRP
- Acetilcolina
- Somatostatina

56) Al ingerir grandes cantidades de dulces , con la subsecuente estimulación de


incretinas , usted esperaría que el apetito
- Aumente por grelina
- Aumente por CCK
- Disminuya por insulina
- Disminuya por CCK

57) Al seccionar el nervio facial a nivel timpánico , usted esperaría


- Disminución del gusto en la punta de la lengua
- Ausencia de termoalgesia en la lengua
- Imposibilidad para el cierre del istmo de las fauces
- Imposibilidad para protruir la lengua

58) Paciente obeso con Covid-19 es intubado por interno inexperto, quien al solicitar que
bombeen aire dentro del tubo endotraqueal, nota que el epigastrio se distiende. Al sospechar
que ha introducido el tubo en el estómago, también es cierto que:
- Aumenta pH gástricos
- Disminuye el tono del píloro
- Aumenta la frecuencia de las ondas lentas
- Disminuye el pH gástrico

59) La hormona que tiene un efecto sinérgico con la secretina para optimizar el pH
duodenal y la digestión, es:
- CCK
- Bombesina
- Gastrina
- Péptido inhibidor gástrico
60) El ecografista sabe que para poder visualizar el nacimiento de la arteria mesentérica
superior , debe colocar el transductor sobre la piel de la siguiente región abdominal
- Hipocondrio derecho
- Mesogastrio
- Hipogastrio
- Epigastrio

61) En un paciente con hiperestimulación simpática se espera que las ondas lentas
tengan un ritmo
- Mayor en estómago que en el duodeno
- Mayor en estómago que en el íleon terminal
- Menor en íleon terminal que en el duodeno
- Mayor en el íleon que en duodeno
62) Se considera que el gusto puede viajar a través del nervio
- Glosofaríngeo
- V3
- Vago
- Lingual
63) La rotación en sentido longitudinal del estómago en el desarrollo embriológico
condiciona que el nervio vago derecho quede a nivel
- Anterior
- Oblicuo
- Posterior
- Izquierdo
64) Con respecto al control autonómico en el tracto gastrointestinal y en relación con su
fisiología. ¿Cuál es la función del sistema nervioso parasimpático en el tracto
gastrointestinal?
A) Inhiben la contracción muscular y estimulan la secreción de sustancias a nivel de la
submucosa.
B) Estimulan la contracción muscular y estimulan la secreción de sustancias a nivel de la
mucosa.
C) Inhiben la contracción muscular e inhiben la secreción de sustancias a nivel de la
submucosa.
D) Estimulan la contracción muscular e inhiben la secreción de sustancias a nivel de la
mucosa.

65) Un estudiante que está preocupado por su examen parcial, no ha desayunado ni


almorzado; cuando al fin ingiere alimentos, esto le provoca el aumento de los
movimientos musculares del tracto gastrointestinal y la sensación de defecar.¿Qué
reflejo se ha activado?
A) entero - gástrico
B) gastro-cólico
C) cólico - ileal
D) ileo - ileal

66) ¿De que par craneal es rama el nervio palatino mayor?


A) Vago
B) Hipogloso
C) Trigémino
D) Palatogloso

67) ¿En cuál de las fases de deglución la epiglotis separa la vía respiratoria de la
digestiva?
A) oral
B) laringe
C) Faríngea
D) Esofágica

68) A los pocos días de nacido, regresa a neonatología un niño con problemas de
motilidad del colon; los estudios determinan ausencia congénita de células
ganglionares. Según el gráfico ¿Cuál es la capa en la que se determina la ausencia
de dichas células?
A) Mucosa - 2
B) Muscular propia - 2
C) Muscular propia - 3
D) Muscular de la mucosa - 3

69) Los péptidos intestinales se pueden clasificar como sustancias endocrinas,


paracrinas y paracrinas, dentro de las apocrinas se encuentran la somatostatina e
histamina. Marque la respuesta correcta
A) La somatostatina es sintetizada por las células B de la mucosa gástrica
B) La histamina actúa estimulando su receptor tipo H1 en la mucosa gástrica
C) La histamina es sintetizada por células de tipo paracrino de las glándulas gástricas
D) La somatostatina presenta dentro de sus funciones la estimulación de la secreción de H+

70) Paciente varón de 27 años es llevado por bomberos a emergencia luego de ser
asaltado y, tras resistirse, es cortado con el pico de una botella a nivel abdominal. Al
examen físico usted observa que a través de la herida se puede observar la
protrusión de las asas intestinales. En relación con las capas de la pared abdominal,
marque la alternativa correcta.
A) la fascia de scarpa está constituida principalmente por tejido adiposo
B) La pared abdominal está formada por piel, huesos, músculos, fascia y peritoneo parietal
C) La fascia de camper es una estructura fibrosa que carece de grasa y su grosor es
constante en toda la pared abdominal.
D) El músculo oblicuo externo discurre en dirección súpero-interna y se inserta en el borde
inferior de las últimas 3 a 4 costillas
E) El músculo recto del abdomen tiene como funciones comprimir el contenido del abdomen,
tensar la pared abdominal y flexionar la columna

71) Los músculos del tracto gastrointestinal de los segmentos propulsivo y receptor del
bolo alimenticio, responden de forma diferente al movimiento de este bolo a través
del intestino. ¿Cuál de las siguientes afirmaciones describe correctamente la
actividad del segmento propulsivo?
A) Tanto el músculo circular como el longitudinal están relajados
B) El músculo longitudinal está relajado y el circular está contraído
C) Tanto el músculo circular como el longitudinal están contraídos
D) El músculo longitudinal está contraído y el circular está relajado

72) El nervio vago inerva al músculo


A) Liso de la faringe
B) Liso de la boca
C) Estriado del esófago
D) Estriado del esófago

73) ¿Cuál de las siguientes es una característica de los ganglios mientéricos del sistema
nervioso entérico?
A) Contiene mayor número de neuronas que el plexo submucoso
B) Es también conocido como el plexo de Meissner
C) Contiene sólo neuronas motoras excitatorias del músculo liso
D) contiene neuronas sensitivas que activan a los músculos circular y longitudinal del tracto
intestinal

74) El divertículo faringoesofágico, hipofaríngeo de zenker, es una lesión muy particular


que se localiza en la cara po
75) posterolateral de la Unión de la faringe con el esófago, como una herniación de la
mucosa esofágica a través de las fibras oblicuas del músculo.
A) Salpingofaringeo
B) Constrictor inferior de la faringe
C) Constrictor superior de la faringe
D) Constrictor medio de la faringe

76) La razón por la que el potencial de acción viaja rápidamente en sentido longitudinal
por el músculo liso gastrointestinal es la presencia de uniones en hendidura,
A) Mayor cantidad de Ach
B) la presencia del plexo submucosos de meissner
C) Las fibras musculares no se disponen en haces musculares
D) varicosidades
76)Respecto a los péptidos gastrointestinales, marque lo correcto.
A) las sustancias paracrinas pueden viajar a través de vasos sanguíneos
B) no existe sustancia neurocrina que tenga efecto en la motilidad del tubo digestivo
C) las sustancias neurocrinas son péptidos que hacen su efecto en distancias cortas
D) las sustancias paracrinas atraviesan la circulación portal

77) Al ingerir rápidamente un litro de agua, usted esperaría que la gastrina


aumenta por efecto de:
A) ACh por estimulación del nervio vago
B) efecto paracrino de la histamina
C) ACh del sistema mientérico
D) aumento de acidez gástrica (disminución del ph)

78) Respecto a la anatomia del estomago,marque lo correcto:

A) El fondo gástrico forma la curvatura mayor

Pág 19 → 21
79) Los nervios esplácnicos lumbares (L1-L2) llevan información de tipo:
A) Simpática
B) Parasimpática
C) Dolorosa
D) Sensorial
E) Piramidal

80) Paciente de 32 años con herida por arma de fuego y shock hipovolémico. El intestino
delgado no se ha infartado aún a pesar de la hipoxia gracias a la liberación de:
A) Adenosina
B) Adrenalina
C) Noradrenalina
D) Péptido intestinal vasoactivo

81) Al realizarse un piercing en el ombligo,la sensación de dolor se transmite por:


A) T11
B) T8
C) T9
D) T10

82) El nivel en el que se encuentra el píloro y el páncreas se puede determinar usando el:
A) Cuadrante mayor
B) Píloro transpilórico
C) Plano subcostal
D) Ligamento inguinal
E) Anillo inguinal

83) Permite la suspensión e irrigación de los órganos peritoneales:


A) Mesenterio
B) Ligamentos
C) Arteria mesentérica superior
D) Peritoneo parietal

84) El dolor asociado a apendicitis clásicamente se ubica en:


A) Epigastrio
B) Mesogastrio
C) Hipogastrio
D) Fosa iliaca derecha
E) Hipocondrio derecho

85) El ligamento inguinales formado por la aponeurosis del:


A) Transverso
B) Recto abdominal
C) Oblicuo externo
D) Oblicuo interno
86) Los nueve cuadrantes del abdomen se delinean usando el plano subcostal,las líneas
medioclaviculares y:
A) Plano intertubercular
B) Plano transpilórico
C) Plano interapofisiario
D) Plano intercrestal
E) Plano catastral

87) Es un órgano peritoneal:


a) Páncreas
b) Hígado
c) Recto
d) Duodeno
e) Colon ascendente

88) La línea alba se encuentra:


a) Entre los rectos abdominales
b) Entre los oblicuos externos
c) Entre los oblicuos internos
d) Entre los transversos abdominales
e) Entre Huaylas y Matellini jajajaj

89) Es un órgano retroperitoneal:


a) Páncreas
b) Colon transverso
c) Colon sigmoides
d) Duodeno
e) Vesícula biliar

90) Enfermedad asociada con un error en el desarrollo de las células de Cajal


a) Enfermedad de Hirschsprung
b) Enfermedad celiaca
c) Divertículo de meckel
d) ano imperforado
e) fístula vitelina
f)
91) El divertículo de Meckel es un rezago de:
a) Conducto vitelino
b) Alantoides
c) Asas yeyunoileales
d) Cloaca
e) Conducto anorrectal

92) Aproximadamente en la semana 6 del desarrollo embrionario; el intestino medio gira 90°
orinandose a nivel del:
a) Cordón umbilical
b) Borde inferior del bazo
c) Caudal al borde hepático derecho
d) Lado derecho del abdomen
e) Suprapúbico
93) La fístula retroperitoneal es causada por una falta en el desarrollo de:
a) Tabique urorrectal
b) Membrana cloacal
c) Proctodeo
d) Membrana anal
e) Membrana urogenital

94) Paciente mujer de 54 años con náuseas,vómitos y abdomen agudo quirúrgico, se


ingresa a sala de operaciones donde se encuentra vólvulo de ciego. Esto se debe a:
a) Falta de fusión del mesenterio
b) Falta de herniación fisiológica
c) Falla de la rotación intestinal
d) Defecto en la formación de la cloaca

ANGELA-CI2 (P95-P104)
95) Al disminuir el pH duodenal por el HCl gástrico, se libera principalmente una hormona
cuya célula diana es:
Respuestas:
Célula ductal del Wirsung
Células ductales del colédoco
Células S del intestino
Acinos pancreáticos

96) Al seccionar el nervio facial a nivel timpánico, usted esperaría:

Respuestas:
Imposibilidad para el cierre del istmo de las fauces.
Ausencia de termoalgesia en la lengua
Imposibilidad para protruir la lengua
Disminución del gusto en la punta de la lengua
97) Al introducir una solución azucarada directamente al estómago mediante una
gastrostomía (comunicación entre la piel abdominal y el estómago), la sustancia que
provocará que aumenten los niveles séricos de insulina es:

Respuestas:
Péptido tipo glucagón 1 (GLP-1)
Enteroglucagon
Glucagón
Somatostatina

98) La rotación en sentido longitudinal del estómago en el desarrollo embriológico


condiciona que el nervio vago derecho quede a nivel:
Respuestas:
Oblicuo
Posterior
Izquierdo
Anterior

99) Paciente con apendicitis aguda, que debuta con dolor en mesogastrio. El dolor se debe
a estimulación de receptores del dolor cuyas fibras van a viajar a la médula espinal a través
de:
Respuestas:
a) Nervios simpáticos
b) nervio vago
c) nervio esplácnico pélvico
d) plexo hipogástrico

100) Una de las siguientes sustancias no comparte con las otras la misma acción sobre la
producción de ácido gástrico:
Respuestas:
Péptido insulinotrópico dependiente de glucosa (GIP)
Secretina
Somatostatina
Colecistoquinina

101) En un paciente con gastroparesia (motilidad lenta del estómago), que presenta
distensión abdominal después de comer, usted le recomendaría que evite el consumo de
lípidos y aminoácidos para disminuir la acción de:
Respuestas:
Gastrina
Somatostatina
CCK
Secretina

102) En un paciente con shock distributivo, usted decide iniciar noradrenalina por un catéter
CVC, consiguiendo aumentar la presión arterial. ¿qué efecto sobre la motilidad intestinal
esperaría encontrar?

Aumenta el peristaltismo.
El potencial de reposo de las fibras musculares se hace más negativo
Aumentan las ondas lentas, pero disminuyen los potenciales en espiga
Aumenta la mezcla en el antro gástrico

103) El ecografista sabe que para poder visualizar el nacimiento de la arteria mesentérica
superior, debe colocar el transductor sobre la piel de la siguiente región abdominal:

Respuestas:
Hipogastrio
Hipocondrio derecho
Epigastrio
Mesogastrio

104) De los diferentes reflejos gastrointestinales, hay uno que produce movimiento del
contenido hacia la región distal, y se llama reflejo:
Respuestas:
Entero-gástrico
Vómito
Gastrocólico
Cólico-ileal

105) La glándula submandibular recibe inervación traída por el nervio


● Cuerda del tímpano
● Petroso Mayor
● Lingual
● Mandibular
● Glosofaríngeo

106) A mayor flujo de saliva, disminuye la concentración de:


● Potasio
● Bicarbonato
● Sodio
● Cloro

107) En términos de mg/mL, el principal componente de la saliva es _____ seguido de ____:


● Proteínas - Potasio
● Potasio - Sodio
● Sodio - Cloro
● Cloro - Urea
● Urea - Proteínas

108) La reabsorción de Sodio y Cloro en las glándulas salivales se da principalmente en el:


● Conducto estriado
● Conducto intercalado
● Conducto excretor
● Acino seroso
● Acino mucinoso
109) La información eferente que sale de los núcleos salivales superior e inferior a través de
los pares VII y IX hacia las glándulas salivales llevan información de tipo:
● Parasimpático
● Simpático
● Somático
● Sensitivo

110) La glándula submaxilar le hace gancho al:


● Músculo milohioideo
● Músculo omohioideo
● Músculo geniohioideo
● Conducto de Wharton
● Conducto de Stenon

111) Los conductos salivales son ___ al agua, esa es una de las razones por las cuales la
saliva es siempre ___.
● Impermeables - hipotónica
● Permeables - hipertónica
● Permeables - isotónica
● Impermeables - isotónica

112) Los nervios esplácnicos pélvicos (S2-S4) llevan información de tipo:


● Parasimpática
● Simpática
● Dolorosa
● Sensorial
● Piramidal

113) Presenta movimientos en masa:


● Colón
● Intestino delgado
● Estómago
● Esófago
● Cavidad oral

114) La digestión de los lípidos se inicia en:


● Intestino delgado
● Colón
● Estómago
● Esófago
● Cavidad oral

115) La digestión de los carbohidratos se inicia en:


● Cavidad oral
● Esófago
● Estómago
● Intestino delgado
● Colón
116) La digestión de las proteínas se inicia en:
● Estómago
● Esófago
● Intestino delgado
● Colón
● Cavidad oral

117) El estómago recibe información simpática proveniente del:


● Ganglio celiaco
● Ganglio mesentérico superior
● Ganglio mesentérico inferior
● Nervio vago
● Nervios esplácnicos (T9-T12)

118) En el intestino delgado se absorbe los carbohidratos en forma de:


● Fructosa
● Sacarosa
● Maltosa
● Dextrosa
● Lactosa

119) Es rama de la arteria mesentérica superior:


● A. cólica media
● A. marginal
● A. cólica izquierda
● A. pancreaticoduodenal superior
● A. gastro-omental derecha

120) El “dolor de estómago” asociado a gastritis se suele ubicar en:


● Epigastrio
● Hipogastrio
● Mesogastrio
● Hipocondrio derecho
● Fosa iliaca derecha
121) El azúcar de mesa sacarosa es digerido a dos monosacáridos que comparten el
transportador:

● SGLT2
● GLUT5
● SGLT1
● GLUT 2

122) Durante la defecación se requiere:

● Liberación de acetilcolina por el nervio vago


● Contracción de tipo segmentarias
● Activación del nervio simpático esplácnico menor
● Señales inhibitorias en el nervio pudendo
123) En un paciente con fístulas intestinales y fisuras anales, con antecedente de
enfermedad inflamatoria intestinal, ud sospecharia en:

● Colitis Ulcerativa
● Anemia perniciosa
● Enfermedad de Crohn
● colitis microscópica

124) En los pacientes con Colecistitis Aguda no operable, una opción es la colocación de
una sonda por el cístico, procedimiento en el que se ingresa con dificultad debido a la
estrechez del cístico y a la presencia de:

● Ampolla de vater
● arteria cística
● válvula de Herring
● estenosis del hepático común

125) Paciente de 51 años con antecedente de enfermedad diverticular acude a emergencia


por sangrado profuso y dolor en hipocondrio izquierdo
¿Cuál es el origen más probable de la sangre que pierde el paciente ?

● Rectal superior
● rectal inferior
● Mesentérica inferior
● cólica media

126) Paciente de 42 años con dolor abdominal intenso y hematemesis. En la endoscopia se


observa una úlcera duodenal posterior perforada con hemorragia intraabdominal. ¿Cuál de
las siguientes arterias estará comprometida?

● Mesentérica superior
● Gástrica derecha
● Pancreaticoduodenal posterosuperior

127) Para que los triglicéridos sean absorbidos deben ser metabolizados a

● Colesterol y ácidos grasos


● Monoglicéridos y Ácidos grasos
● colesterol y lisolecitina
● lisolecitina y ácidos grasos

128) El vibrio cholerae produce diarrea porque:


● Previene la absorción de glucosa y causa que el agua mantenga en la luz intestinal
● aumenta los canales secretores de bicarbonato en los enterocitos
● Aumenta la producción de AMPc en los enterocitos
● inhibe los canales secretores de cloro en las células de la cripta
129) La motilidad intestinal es estimulada por
● colecistoquinina y gastrina
● Glucagón e insulina
● somatostatina y secretina
● secretina y glucagón

130) La diarrea por deficiencia de lactasa es de tipo


● Iatrogenia
● secretora
● exudativa
● osmótica

131)La metoclopramida estimula el vaciamiento gástrico aumentando la fuerza de


contracción de las paredes gástricas esto puede conseguirlo mediante la estimulación
indirecta de las neuronas liberadoras de
● Bombesina
● Acetilcolina
● Oxido nitrico
● Péptido intestinal Vasoactivo

132)Los vértices de un acino hepático están constituidos por


● una vena centrolobulillar y dos espacios porta
● Dos espacios porta y dos venas centrolobulillares
● tres venas centrolobulillares
● seis espacios porta

133)Al ingerir un pedazo de mantequilla, cuál de los siguientes tiene un efecto directo en la
reducción del vaciado gástrico:

● Colecistoquinina
● Gastrina
● Bombesina
● Secretina

134) Tras la vagotomía (resección del vago) por enfermedad úlcera péptica en un paciente
UD. esperaría encontrar:
● Aumento de síntomas de reflujo gastroesofágico
● hipertrofia de mucosa gástrica
● mayor producción de CCK
● Aumento del pH gástrico

135) Se realizó un experimento en el cual se inyectó tinta china en el peritoneo de ratas de


laboratorio. Al realizarse una biopsia hepatica de dichos animales, se encontro que el tinte
negro fue fagocitado por:
● células de disse
● Células de kupffer
● hepatocitos
● células de ito

136) En un paciente con pH gástrico muy bajo, es posible que la siguiente sustancia se
secrete en menor cantidad:
● Péptido inhibitorio gástrico
● colecistoquinina
● secretina
● Gastrina

137)La fase cefálica de la secreción gástrica responde por cerca del 30% de la respuesta
ácida a un reflejo con la _____________ se elimina la fase cefálica de la secreción gástrica

● Vaguectomía
● Histamina
● Cimetidina
● Gastrina

138)El esofago de barret se caracteriza por presentar _________ en el esofago

● Metaplasia gástrica
● metaplasia intestinal
● displasia gástrica
● adenocarcinoma

139) La presencia de orina que sale por el ombligo de un recién nacido cada vez que llora,
es posible que deba a un defecto en el desarrollo de:
● Seno urogenital
● Cloaca
● Membrana basal
● Cuerpo perineal
● Conducto vitelino

140) El conducto biliar deriva del:


● Endodermo
● Mesodermo
● Ectodermo
● Mesotelio
● Peritoneo

141) Cuál de los siguientes órganos son intraperitoneales:


● Estómago, Vesícula biliar, Y León, Hígado
● Páncreas, Colon descendente, Hígado, Vesícula biliar
● Recto, Hígado, Colon transverso, Yeyuno
● Estómago, Yeyuno, Duodeno, Páncreas
● Recto, Vesícula biliar, Y León

142) Los vasos mesentéricos superiores se hallan a nivel de:


● Cardias
● Cuello del páncreas
● Hilio hepatico
● Tronco celiaco
● Ileon

143) La colecistoquinina (CCK) inhibe:


● El vaciamiento gástrico
● La secreción pancreática de HCO3-
● La concentración de la vesícula biliar
● La relajación del esfínter de Oddi
● La secreción de amilasa

144) Un niño de 2 años es llevado a la consulta por diarrea persistente, edema de las
extremidades y falta de crecimiento en relación a su edad. Los análisis de sangre revelan
que tiene concentración plasmática baja de proteínas (hipoproteinemia). Como parte del
estudio se coloca Colecistoquinina (CCK) endovenosa y se recoge muestras del líquido
duodenal por endoscopia; el resultado del líquido confirma incapacidad para hidrolizar
proteínas a un pH neutro, esta situación mejora al añadir una pequeña cantidad de tripsina.
El paciente probablemente esté sufriendo la falta congénita de ……….
● PEPT-1
● Pepsinógeno
● Enterocinasa
● Carboxipeptidasa.

145) Paciente mujer de 35 años acude a consulta por sensación de sequedad y lesiones en
la cavidad oral. Al examen se observa atrofia de la mucosa, fisura y úlceras; nota además
sequedad e irritación de la córnea y aumento del tamaño de las glándulas parótidas. Su
diagnostico mas probable es artritis reumatoide; el hallazgo más probable en una biopsia de
glándula parótida es ….…
● Presencia de acinos anormales con hiperplasia de células ductales
● Gran infiltración de linfocitos y células plasmáticas
● Hiperplasia de acinos glandulares serosos
● Gran infiltrado de linfocitos y neutrófilos

146) Un hombre de 42 años de edad se presenta al médico con una historia clínica de 1 año
de evolución, caracterizado por dolor abdominal bajo y diarreas con crisis sanguinolentas.
Manifiesta además pérdida de peso de 8kg durante este periodo. La colonoscopia revela
lesión difusa en el colon con afectación del recto. La biopsia de estas lesiones revela
adelgazamiento de la pared, inflamación y ulceración de la mucosa y submucosa. El
diagnóstico más probable en este caso es:
● Síndrome de colon irritable
● Enfermedad de Crohn
● Colitis ulcerativa
● Sprue celiaco

Dos estudiantes deciden tomar un receso para comer una hamburguesa a la hora del
almuerzo. Antes de llegar a la cafetería, impulsos nerviosos provenientes del complejo vagal
dorsal iniciarán la secreción de ácido gástrico por la liberación de ___________ desde el
sistema nervioso entérico.
● Serotonina
● Colecistoquinina (CCK)
● Péptido inhibidor vasoactivo
● GRP (péptido liberador de gastrina)

147) Un niño de cuatro años de edad es llevado a la consulta por cuadros diarreicos
frecuentes caracterizados por heces pálidas, voluminosas y fétidas; al examen físico
presenta bajo peso y talla para la edad. Se mide la concentración de cloruro de en el sudor y
se encuentra que sus valores son muy elevados. La alteración más importante a nivel de
células ductales del páncreas tiene relación directa con la conductancia de …..
● Bicarbonato
● Potasio
● Sodio
● Cloro

148) Se evalúa los valores séricos de las siguientes sustancias a un paciente con
enfermedad hepática terminal; en este paciente se espera encontrar la combinación de la
letra …..

Glucosa Amoniaco Albúmina

a. Aumentada Disminuida Disminuida

b. Disminuida Aumentada Aumentada

c. Aumentada Aumentada Aumentada

d. Disminuida Aumentada Disminuida

149) Una mujer de 35 años de edad HIV positiva, se presenta al médico con dolor
abdominal en cuadrante superior derecho e ictericia. La paciente refiere haber tenido
múltiples episodios de ictericia durante los últimos 10 años. Los exámenes para determinar
hepatitis viral, dieron positivos para Hepatitis B, siendo catalogado el caso como hepatitis
crónica con alteración funcional. En un examen de sangre ¿Cuál de los siguientes
parámetros está disminuido?
● Albúmina
● Bilirrubina
● Fosfatasa alcalina
● Tiempo de protrombina

150) En el reflejo peristáltico del intestino delgado ¿Cuál de los siguientes eventos sucede
en la porción caudal del bolo alimenticio?
● Acción del péptido inhibidor vasoactivo (VIP) en el músculo circular
● Acción del NO (Óxido nítrico) en el músculo longitudinal
● Contraccion del musculo longitudinal interno
● Acción de la acetilcolina en el músculo circular

151) Un varón de 58 años de edad con enfermedad de Crohn severo fue sometido a una
resección ileal. Después de la cirugía esta paciente padecerá de esteatorrea esto se explica
porque ……
● Se inhibe la acción de la 7 alfa hidroxilasa
● El pool de ácidos biliares se incrementa
● Hay malabsorción de ácidos biliares
● El páncreas no secreta lipasa

152) En un experimento se inserta un balón en el estómago de un voluntario, se infla poco a


poco mientras que se vigilan las presiones intraluminales. Aunque el volumen del balón
aumenta considerablemente, las presiones permanecen constantes. Esta relación volumen-
presión se explica por la liberación local de ….
● Acetilcolina y gastrina
● Norepinefrina y oxido nitrico
● Colecistoquinina y oxido nitrico
● Óxido nítrico y péptido inhibidor vasoactivo

153) ¿Cuál de las siguientes alternativas es una característica de la secreción exocrina del
páncreas?
● Es hipotónica respecto al plasma
● Su mayor estímulo se da en la fase intestinal
● Es estimulada por la presencia de bicarbonato en el duodeno
● La secreción enzimática es estimulada principalmente por la secretina

154) Las estructuras en el hígado que permite que los productos metabólicos unidos a
proteínas tengan acceso a las membranas basolaterales de los hepatocitos, son ….
● Los canalículos
● Las células de Ito
● Las fenestras sinusoidales
● Las uniones intercelulares herméticas

155) La composición de la bilis es modificada conforme fluye por los conductillos biliares.
Durante este tránsito se espera que aumente la concentración de …..
● Ig A
● Glucosa
● Protones
● Vitamina A

156) Se mide experimentalmente el contenido gástrico de dos personas. La persona “A”


tiene alto contenido de grasa y la persona “B” tiene un contenido isotónico. ¿Cual de las
siguientes es correcta respecto al vaciamiento gástrico?
● Hay ralentización del vaciado gástrico sólo en “A”
● El vaciamiento gástrico es más rápido en ambos
● Hay ralentización del vaciado gástrico sólo en “B”
● Hay ralentización del vaciado gástrico en ambos casos
157) El examen endoscópico de un paciente con hipertensión portal grave revela venas
tortuosas que sobresalen hacia la luz del esofago. El paciente recibe tratamiento quirúrgico
mediante la colocación de una derivación que conecta la vena cava. Después de la
operación el riesgo de encefalopatía …….. y el sangrado de varices ……
● Disminuirá/Disminuirá
● Disminuirá/Aumentará
● Aumentará/Disminuirá
● Aumentará/Aumentará

158) Un bolo alimenticio grande y poco masticado se atasca en el esofago, esto ocasiona
una sensacion de dolor que es transmitida por los nervios:
● Vago
● Esplácnico
● de Latarjet
● Recurrente laríngeo

159) El peristaltismo del intestino delgado se puede intensificar debido a:


● Somatostatina
● Noradrenalina
● Secretina
● Irritación de la mucosa

160) Un paciente es diagnosticado con un tumor neuroendocrino productor de


somatostatina, esto provocará en el sistema digestivo:
● Vómitos
● Estreñimiento
● Odinofagia
● Diarrea

161) Los diferentes segmentos del tubo digestivo son susceptibles de reflejos y movimientos
según su contenido.Si colocoramos mediante una sonda un bolo alimenticio directamente en
el tercio medio del esofago:
● Se producirá la acción voluntaria del músculo estriado
● Será necesario un reflejo vago-vagal
● Se producirá ondas secundarias
● Se producirá ondas primarias

162) En el digestivo la liberación hormonal se presenta ante diversos factores o estímulos.


La hormona _________ es estimulada por la presencia de alimentos en el bulbo duodenal a
predominio de ácidos grasos y triglicéridos, por estimulación vagal y por la hormona
secretina.
● Motilina
● Colecistoquinina (CCK)
● Lipasa pancreatica
● Gastrina

163) El divertículo de Meckel es una anomalía congénita que ocurre por la persistencia del
conducto vitelino y da origen a una estructura sacular, el cual se encuentra en el:
● Duodeno
● Apéndice cecal
● Borde mesentérico
● Borde antimesentérico

164) La fase oclusal de la masticación se realiza con la contracción de los músculos


inervados por el nervio craneal:
● V
● VIII
● IX
● XII
165) En un paciente de 43 años con tumor carcinoide de páncreas productor de gastrina
(Sindrome de Zollinger-Ellison) se puede encontrar una potenciación del reflejo:
● Ileocolico
● Gastrocolico
● De relajación receptiva
● Del vómito

166) Durante la fase faríngea de la deglución ocurre el siguiente mecanismo:


● Los músculos infrahioideos hacen que se desplaza la laringe hacia abajo
● El paladar blando abre el istmo de las fauces
● La onda peristáltica lleva el alimento hacia el esofago
● El péptido inhibidor vasoactivo (VIP) actúa sobre el músculo constrictor superior

167) En los carcinomas (neoplasia benigna) es frecuente que ocurran la metástasis a través
de los vasos venosos. En el caso de un carcinoma del tercio superior del esofago, ubicado
en la cara lateral izquierda, es más probable que la metástasis viaje por la vena:
● Subcostal derecha
● Cava Inferior
● Hemiácigos accesoria
● Hemiácigos

168) Los movimientos en masa son iniciados por el reflejo:


● Del vómito
● Duodenocolico
● Colicoileal
● Duodenogástrico

169) Se evalúa los valores séricos de las siguientes sustancias de un paciente con
enfermedad hepática terminal, en este paciente se espera la combinación con la letra

Glucosa Amoníaco Albúmina

a aumentada disminuida disminuida

b disminuida aumentada aumentada

c aumentada aumentada aumentada

d disminuida aumentada disminuida


170) Una mujer de 65 años VIH positiva se presenta con dolor abdominal en el cuadrante
positivos para Hepatitis B, siendo catalogada como hepatitis crónica con alteración
funcional. En un examen de sangre ¿cuál de los siguientes parámetros está disminuido?
● Albúmina
● Bilirrubina
● Fosfatasa alcalina
● Tiempo de protrombina

171) En el reflejo peristáltico del intestino delgado¿cuál de los siguientes eventos sucede en
la porción caudal del bolo alimenticio?
● Acción del péptido inhibidor vasoactivo(VIP) en el músculo circular
● Acción del NO(óxido nítrico) en el músculo longitudinal
● Contracción del músculo longitudinal interno
● Acción de la acetilcolina en el músculo circular

172) Un varón de 58 años de edad con enfermedad de Crohn Severo fue sometido a una
resección ilegal. Después de la cirugía este paciente padecerá de esteatorrea, esto se
explica porque ….
● Se inhibe la acción de la 7 alfa hidroxilasa
● El Pool de ácidos biliares se incrementa
● Hay mala absorción de los ácidos biliares
● El páncreas no secreta lipasa

173) En un experimento se inserta un balón en el estómago de un voluntario, se infla poco a


poco mientras se vigilan las presiones intraluminales. Aunque el volumen del balón aumenta
considerablemente, las presiones permanecen constantes. Esta relación volumen presión se
explica por la liberación local de ….
● Acetilcolina y gastrina
● Norepinefrina y ácido nítrico
● Colecistoquinina y óxido nítrico
● Óxido nítrico y péptido inhibidor vasoactivo

174) ¿Cuál de las siguientes alternativas es una característica de la secreción exocrina del
páncreas?
● Es hipotónica respecto al plasma
● Su mayor estímulo se da en la fase intestinal
● Es estimulada por la presencia de bicarbonato en el duodeno
● La secreción enzima toca es estimulada principalmente por la secretina
175)Las estructuras en el hígado que permiten que los productos metabólicos unidos a
proteínas tengan acceso a membranas basolaterales de los hepatocitos son ….
● Los canalículos
● Las células de Ito
● Las de fenestras sinusoidales
● Las uniones intercelulares herméticas

176)La composición de la bilis es modificada conforme fluye por los conductillos biliares.
Durante este tránsito se espera que aumente la concentración de ….
● IgA
● Glucosa
● Protones
● Vitamina A

177)Se mide experimentalmente el contenido gástrico de dos personas. La persona A tiene


alto contenido de grasas y la persona B tiene un contenido isotónico¿ Cuál de las siguientes
es correcta respecto al vaciamiento gástrico?
● Hay ralentización del vaciado gástrico sólo en A
● El vaciamiento gástrico es más rápido en ambos
● Hay ralentización del vaciado gástrico sólo en B
● Hay ralentización del vaciado gástrico en ambos

178)El examen endoscópico de un paciente con hipertensión portal grave revela venas
tortuosas que sobresalen hacia la luz del estómago. El paciente recibe tratamiento
quirúrgico mediante la colocación de una derivación que conecta la vena porta a la vena
cava. Después de la operación el riesgo de encefalopatía …. y el riesgo de sangrado de
varices ….
● Disminuirá/ disminuirá
● Disminuirá / aumentará
● Aumentará / disminuirá
● Aumentará / aumentará

179) Un paciente varón de 18 años de edad acude al médico para sus exámenes de rutina.
Sus resultados de laboratorio muestran un valor de bilirrubina sérica de 4 mg/dl y una
bilirrubina directa de 0.3 mg/dl. Las pruebas de función hepática son normales. La alteración
que explica mejor este caso es por la deficiencia de….
● Transaminasas
● Hemo Oxigenasa
● La 7 alfa hidroxilasa
● Glucuronil transferasa

180) Un hombre de 57 años de edad es llevado a urgencias con hematemesis masiva roja
brillante, a su llegada se halla inconsciente con PA: 80/40 mm Hg y FC:124 lat/min. Luce
ictérico con presencia de arañas vasculares en el tórax anterior y extremidades, abdomen
distendido con signo de oleada positiva. Se encuentra esplenomegalia y pérdida de la masa
muscular en extremidades. La anastomosis vascular responsable del sangrado en este
paciente es ….
● Vena gastrica izquierda y vena acigos
● Arteria gástrica izquierda y arteria ácigos
● Vena paraumbilical y vena epigástrica inferior
● Vena gástrica izquierda y vena esofágica superior

190) Un estudiante de medicina está comiendo un plato de comida a base de champiñones,


espárragos y salsa de soya. El estímulo del sabor umami contenido en todos estos
alimentos viaja a través del nervio….
● Lingual
● Hipogloso
● Glosofaríngeo
● Cuerda del tímpano

191) Una paciente de 30 años de edad es sometida a una cirugía de oído medio derecho
por un problema de otoesclerosis. Luego de la cirugía refiere alteración sensitiva de la
lengua. Al evaluar el caso usted esperaría encontrar ….
● Alteración en la sensación del dolor y temperatura en el tercio posterior de la lengua
● Alteración de la sensación del dolor en los dos tercios anteriores de la lengua
● Alteración en la sensación del gusto en el tercio posterior de la lengua
● Sensación del dolor, tacto y temperatura conservadas

192) En una paciente de 45 años de edad con colestasis biliar, se encuentra una elevación
de los niveles sanguíneos de fosfatasa alcalina hasta 3 veces la cifra normal ¿Cuál de las
siguientes alternativas estará también elevada como evidencia del daño biliar?
● Tiempo de protrombina y albúmina sérica
● Transaminasas hepáticas(ALT y AST)
● Gamma glutamyl transpeptidase
● Glucoronil transferasa

193) Experimentalmente se incrementa la velocidad de la secreción salival con una


sustancia, en el análisis de la composición de esta saliva obtenida se espera encontrar….
● Disminución de la concentración de bicarbonato que supera la concentración
plasmática
● Aumento en la concentración de cloro y sodio que supera la concentración
plasmática
● Aumento de la concentración de bicarbonato que supera la concentración plasmática
● Disminución de concentración de potasio y bicarbonato

194) Lactante de 3 meses de vida es atendido por presentar diarrea, se administra una
solución glucosa y electrolitos por vía oral. La proteína de membrana apical que explica la
capacidad de esta solución para proporcionar aporte de glucosa e hidratación es ….
● CFTR
● SGLT 1
● GLUT 2
● GLUT 5
195) Paciente ha sufrido herida de bala en el abdomen, se le tenido que extirpar el
segmento medio y distal del íleon. En este caso la síntesis hepática de sales biliares estará
….
● Sin cambios en el ritmo de la síntesis
● Disminuida por inhibición de la enzima colesterol 7 alfa hidroxilasa
● Incrementada por estímulo de la enzima colesterol 7 alfa hidroxilasa
● Incrementada por inhibición de la enzima colesterol 7 alfa hidroxilasa

196) Un varón de 75 años ingresa al consultorio por presentar ictericia marcada de piel y
escleras. El estudio del paciente mostró que presentaba un tumor que obstruía la totalidad
del conducto hepático común. ¿Cual de los siguientes conductos se encontraría dilatado en
este paciente?
● De Wirsung
● De Hering
● Colédoco
● Cístico

197) Correlacione las dos columnas y marque la respuesta correcta:


● Enfermedad Hirschsprung Aganglionosis congénita
● Diarrea osmótica. Intolerancia a la lactosa
● Diarrea secretora. Canales de Cl- en las células de la cripta
● Diarrea exudativa Heces con moco y sangre
198) La fase cefálica de la secreción gástrica responde por cerca del 39% de la respuesta
ácida a un reflejo. Con la ____ se elimina la fase cefálica de la secreción gástrica
Vaguectomía
199) El esofago de Barret se caracteriza por presentar___ en el esofago
Metaplasia intestinal

198) En la regulación del apetito y la saciedad, la estimulación experimental crónica del


núcleo ventromedial del hipotálamo producirá:
● Afagia
● Obesidad
● Hiperfagia
● Activación de neuronas relacionadas a NPY

199) Paciente mujer de 25 años acude por dolor en fosa iliaca derecha que empeora al toser
o caminar,asociada a náuseas y vómitos por lo cual acude a emergencia.Dos días después
de realizarle una apendicectomía, la paciente desarrolla fiebre alta (39° C), esta hipotensa y
presenta dolor abdominal.La laparotomia exploratoria muestra un gran volumen de sangre
en la cavidad peritoneal por lesión de un vaso producida durante la apendicectomía.¿Cual
de las siguientes arterias debe ligarse para detener la hemorragia?
● Ileocólica
● cólica derecha y arteria rectal superior
● mesentérica superior
● Ileocólica y arteria cólica Media

200) ¿Cuál de los siguientes es una causa de ictericia con bilirrubina conjugada
aumentada?
● Ictericia del recién nacido
● Obstrucción del colédoco
● Anemia hemolítica
● Gran hematoma

201. Dos días después de una apendicectomía en un hombre de 45 años, ha desarrollado


fiebre alta (39), está hipotenso y presenta dolor abdominal. La laparotomía exploratoria
muestra un gran volumen de sangre en la cavidad peritoneal por lesión de un vaso
producida durante la apendicectomía. ¿Cuál de los siguientes vasos debe ligarse para
detener la hemorragia?

a. Arteria cólica derecha

b. Arteria ileocólica y arteria cólica media

c. Arteria mesentérica superior

d. Arteria ileocólica

202. Paciente de 78 años, con diabetes mellitus tipo ll y fumador, que acude a consulta
porque desde hace dos semanas tiene un dolor intenso en flanco derecho y mesogastrio ,
intenso que aparece a los 30 minutos de haber comido, y desaparece dos a tres horas
después. En estos pacientes, es muy probable que la circulación deficitaria sea
parcialmente asumida por la:

a. Arteria ileocólica

b. Arteria cólica izquierda

c. Arteria cólica media

d. Arteria yeyunales

203. Un hombre de 70 años ingresa en urgencias con diarrea intensa. La arteriografía revela
un bloqueo del 90% en el origen aórtico de la arteria mesentérica inferior. ¿Cuál de las
siguientes arterias proporciona muy probablemente irrigación colateral al colon
descendente?

a. Arteria gastro omental izquierda

b. Arteria cólica derecha

c. Arteria cólica media

204. Un niño de 4 años ingresa en el hospital con vómitos graves. La exploración radiológica
y la historias clínicas revelan que el niño tiene páncreas anular. ¿Cuál de las siguientes
estructuras es la que se encontrará dilatada con mayor probabilidad a raíz de esta
patología?

a. Tercera porción del duodeno


b. Segunda porción del duodeno

c. Píloro gástrico

d. Primera porción del duodeno

205. Un hombre de 55 años ingresó al hospital con dolor abdominal intenso. La gastroscopia
y la TC revelaron una úlcera perforada en la pared posterior del estómago. ¿Dónde se
desarrolla inicialmente con más probabilidad una peritonitis?

a. Espacio hepatorrenal (de Morison)

b. Bolsa omental (saco menor)

c. Espacio subhepático derecho

d. Espacio subfrénico derecho

206. Un hombre de 44 años ingresa en urgencias con vómitos abundantes y deshidratación.


Las imágenes radiológicas demuestran que parte del intestino está comprimido entre la
aorta abdominal y la arteria mesentérica superior. ¿Cuál de las siguientes estructuras
intestinales estará muy probablemente comprimida?

a. Segunda porción del duodeno

b. Colon transverso

c. Tercera porción del duodeno

d. Primera porción del duodeno

207. ¿Cuál de los siguientes nódulos linfáticos estará muy probablemente afectado en una
neoplasia maligna del recto?

a. Celiaco

b. Mesentérica inferior

c. mesentérica superior

d. rectal

208. Una mujer de 23 años ingresa con dolor abdominal, náuseas y vómitos. La historia
clínica muestra que el dolor es agudo y ha sido constante durante 4 años. El dolor empezó
en el epigastrio e irradiado bilateralmente alrededor del tórax hasta justo debajo de las
cápsulas. Actualmente el dolor se localiza en el hipocondrio derecho. L a TC revela cálculos
calcificados en la vesícula biliar. ¿Cuál de los siguientes nervios llevan las fibras aferentes
del dolor referido?

a. Nervios esplácnicos torácicos mayores


b. Nervio frénico

c. Nervio vago

209. Un hombre de 55 años ingresa en urgencias por severa pérdida de peso en los 6
meses previos. El examen radiológico pruebas aportan signos de un tumor que causa
hipertensión portal. Los estudios de laboratorio revelan que las deposiciones son grasas,
tiene desnutrición e hipoxia hepática. ¿Cuál de las siguientes localizaciones se encuentra
muy probablemente afectada?

a. Segmento l

b. Segmento ll

c. Segmento ll

d. Segmento IV

210. El triángulo de calot es importante reconocerlo porque sirve como reparo para
encontrar la arteria cística, y está formado por el conducto cístico, el conducto hepático
derecho y :

a. Borde hepático

b. Arteria vesical

c. Arteria hepática

211. El kernicterus se produce en recién nacidos con valores mayores de 25 m/dL en la


bilirrubina:

a. Indirecta

b. Directa

c. Delta

212. Paciente con cirrosis hepática y presión de vena cava de 15 mmhg. Lo más probable
es que el paciente presente:

a. Ascitis

b. Cabeza de medusa

c. Arañas vasculares

213. Considerando un flujo plasmático renal de 180 ml/min, y una fracción de filtración del
20%, si la concentración en sangre de la bilirrubina indirecta es de 0.6mg/dL, entonces es
correcto esperar que la carga filtrada de la bilirrubina indirecta es:

a. Menor de 1080 mg/dL

b. 2050
c. Mayor a 1080}

214. El volumen diario de bilis secretada al intestino es:

a. 500-1000 ml

b. 50-100

c. 5-10

215. La excreción del amonio se da principalmente por:

a. Orina

b. Heces

c. Pulmones

d. Piel

216. Si en una persona normal, el flujo de la arteria hepática es de 700 ml/min, entonces el
flujo de la vena porta debería ser aproximadamente:

a. 2800 mL/min

b. 3500

c. 1300

217. Paciente con Crigler-Najjar debida a mutación del gen UGT1A1, se presenta a consulta
por ictericia, usted asume que si le hiciera un análisis de sangre encontraría valores
elevados de:

a. Bilirrubina indirecta

b. Directa

c. Conjugada

218. El amoniaco corporal se forma principalmente en:

a. Colón

b. Riñón

c. Músculo

219. La zona del lobulillo hepático que se afectaría más en una intoxicación con droga
hepatotóxica es la zona:

a. 1

b. 2
c. 3

d. 4

220. El acino hepático tiene en sus aristas:

a. Vena centrolobulillar

b. Solo espacios porta.

c. Espacios porta y sinusoides

221. Paciente con esteatosis hepática no alcohólica, que en la biopsia se observa


degeneración grasa de los hepatocitos, lo cual se debe a depósitos de lípidos que
principalmente contienen:

a. Triglicéridos

b. Colesterol

c. Ácidos grasos libres superior derecho e ictericia. La paciente afirma haber tenido
múltiples episodios de ictericia durante los últimos 10 años. Los exámenes para poder
detectar hepatitis viral, dieron p

PARCIAL

222. Al evaluar la orofaringe de un paciente, el médico le solicita que abra la boca, saque la
lengua y diga a . Al hacer esta maniobra, nota que el paladar se desvía hacia la derecha, lo
cual le hace sospechar que el paciente sufre de una lesión del nervio craneal:
- X contralateral

223. Un bolo alimenticio grande y poco masticado se atasca en el esófago, esto ocasiona
una sensación de dolor que es transmitida por los nervios:
- esplácnicos

224. Para realizar el movimiento mecánico de abrir la boca, primero se necesita:


- fijar el hueso hioides

225. ¿Cuál de las siguientes alternativas se define como la protrusión directa del contenido
abdominal a la cavidad amniótica por un defecto de la pared corporal?
- Gastrosquisis

226. Un paciente requiere que se le coloque una sonda de alimentación directamente al


estómago (gastrostomía), el cirujano deberá hacer una incisión en la piel del abdomen ¿cuál
de las siguientes raíces nerviosas debe ser anestesiada para este procedimiento?
- T8
227. En un paciente de 43 años con tumor carcinoide de páncreas productor de gastrina
(Síndrome de Zollinger-Ellison) se puede esperar encontrar una potenciación del reflejo:
- gastrocólico

228. El mecanismo de la defecación incluye la participación de diversas estructuras ¿Cuál


de las siguientes alternativas es correcta?
- Puede ser mediado por un reflejo intrínseco

229. Cuando el contenido del estómago ingresa al duodeno, uno de los reflejos que inhiben
el vaciamiento gástrico es a través del:
- sistema nervioso mientérico

230. Durante la masticación, gran parte del proceso masticatorio se debe a:


- el reflejo masticatorio

231. Las glándulas salivales tienen conductos para la excreción de la saliva; las glándulas
____________ drenan en las carúnculas sublinguales.
- sublinguales

232. Los diferentes segmentos del tubo digestivo son susceptibles de reflejos y movimientos
según su contenido. Si colocamos mediante una sonda un bolo alimenticio directamente en
el tercio medio del esófago:
- se producirá ondas secundarias

233. En una persona si enfermedad se espera que el tránsito intestinal se vea disminuido
cuando se presenta el reflejo:
- doloroso

234. El divertículo de Meckel es una anomalía congénita que ocurre por la persistencia del
conducto vitelino y da origen a una estructura sacular, el cual se encuentra en el:
- borde antimesentérico

235. Si al intubar a un paciente, por error se ingresa el tubo endotraqueal en el esófago y se


insufla el manguito endotraqueal (globo TET), la dilatación de este manguito generará:
- múltiples ondas secundarias

236. El orificio omental, o hiato de Winslow, se encuentra limitado por el ligamento:


- hepatoduodenal

237. Paciente de 24 años acude a consulta externa por presentar una fístula oronasal
(comunicación entre la cavidad oral y la cavidad nasal). Está fístula es una consecuencia
tardía de la lesión de un vaso sanguíneo por el antecedente de haber sido operado de
paladar hendido en los primeros años de vida, aparentemente en una campaña gratuita de
corrección de paladar fisurado. ¿Cuál de las arterias palatinas podría haberse lesionado
durante esa cirugía?
- Mayor
238. Dentro de las anomalías congénitas se puede presentar un tejido pancreático accesorio
¿Cuál es la ubicación más común de este tejido?
- Estómago

239. Paciente con insuficiencia mitral moderada a severa, con aumento de volumen de la
aurícula izquierda; esta condición tendrá como consecuencia a nivel del sistema digestivo:
- la disfagia a sólidos

240. El inicio de la fase faríngea de la deglución se debe a estímulos sensitivos que viajan
por el nervio craneal:
- V (nervio maxilar)

241. El mesocolon transverso se origina en:


- la pared posterior del abdomen

242. Un paciente varón de 68 años consulta por dolor y abultamiento en la región inguinal
derecha, que aparece tras la realización de un esfuerzo físico. En la exploración en
bipedestación presencia de una tumoración blanda, depresible, que aumenta con la tos. El
presente caso describe una hernia:

- Inguinal directa.

243. la Grelina es sintetizada por………….. y activa las neuronas relacionadas con


……………………….en el núcleo arqueado del hipotálamo}
- El estómago/AGRP-NPY

244. Cuál de los siguientes órganos son intraperitoneales:


- Estómago, Vesícula biliar, Y León, Hígado

245. La distensión gástrica por los alimentos produce incremento de secreción de HCl
mediante el siguiente mecanismo:
- Producción de Gastrina que desencadena su cascada de señalización en la célula
parietal vía proteína Gq

246. La inervación de la piel del abdomen debajo del ombligo hasta la región púbica está
dada por:
- T11, T12, L1

247. Paciente de 38 años que tras riña durante partido de fútbol sufre un traumatismo con
arma blanca en cuadrante inferior izquierdo del abdomen. En la tomografía de urgencias se
evidencia gran hematoma de pared y ausencia de neumoperitoneo. Desde la piel hacia al
peritoneo, en ¿qué orden se atravesó la pared abdominal?

- TCSC, oblicuo externo, oblicuo interno, músculo transverso

248. Alrededor de 90% de los pacientes afectados por el síndrome de Zollinger-Ellison


desarrollan úlcera péptica. La causa es:
- Hiper Secreción ectópica de gastrina
249. Un hombre de 22 años sufre traumatismo en el flanco izquierdo superior del abdomen
al tratar de defenderse del robo de su vehículo. Un amigo lo trasladó a un centro
hospitalario. El paciente presenta signos de hipovolemia con taquicardia e hipotensión. Se
queja de dolor en el sitio de lesión que se irradia al hombro izquierdo. El órgano
probablemente lesionado es
- Bazo

250. Aproximadamente en la semana 6 del desarrollo embrionario, el intestino medio gira


90° orinandose a nivel del:
- Cordón umbilical

251. ¿Cuál de las siguientes sustancias es liberada por neuronas en el tracto GI, Participa
en la regulación hidroelectrolítica y produce relajación del músculo liso?
a) Gastrina
b) Secretina
c) CCK
d) VIP

252. La colecistoquinina inhibe:


a) El Vaciamiento gástrico
b) La Secreción pancreática de HCO3-
c) La Contracción de la vesicular biliar
d) La Relajación del esfínter de oddi

253. ¿La secreción de cuál de las siguientes es inhibida por un pH bajo?


a) Secretina
b) Gastrina
c) CCK
d) VIP

254. El conducto biliar deriva del:


a) Ectodermo.
b) Endodermo.
c) Mesodermo.
d) Mesotelio.

255. Respecto a la imagen mostrada la estructura señalada con el número 1


corresponde a:
a) Glándula parótida
b) Glándula sublingual
c) Glándula submaxilar
d) Glándula lingual
256. Los vasos mesentéricos superiores se hallan a nivel de:
a) El fondo gástrico
b) El cuello del páncreas
c) La cabeza del páncreas
d) El hilio hepático

257. La imagen mostrada representa un defecto de ͙ ͙ ͙ ͙ ͙ ͙………… y recibe el nombre


de: …………
a) La pared abdominal/onfalocele
b) La pared abdominal/divertículo de Meckel
c) Malrotación intestinal/gastrosquisis
d) La pared abdominal/gastrosquisis

PARCIAL

1) Paciente de sexo masculino de 82 años de edad ingresa a emergencia con dolor abdominal
agudo y diarreas. Se le realiza una arteriografía en la que se observa que la arteria aorta tiene
un trombo ocluyendo el 95% del flujo a nivel del nacimiento de la arteria mesentérica inferior.
¿Cuál de las siguientes arterias podría contribuir a la irrigación colateral del colon
descendente?

a) Cólica media
b) Sigmoidea
c) Rectal superior
d) Ileocólica
2) Niño de 5 años presenta dolor esofágico y hematemesis (vómitos hemorrágicos) luego de
tragarse una espina de pescado. En la endoscopía se observa perforación del esófago distal
a la cuarta estrechez esofágica. ¿Las ramas de cuál de las siguientes arterias estarán
lesionada con mayor probabilidad?
a) Gástrica izquierda
b) Bronquiales
c) Frénica inferior
d) Tiroidea inferior

3) Al ingerir una cucharada de mantequilla es muy probable que se disminuya la sensación


de hambre por medio de la activación de la vía POMC/CART (POMC=proopiomelanocortina
y CART=transcripción regulada de cocaína y anfetamina), activada directamente por la
hormona:
a) colecistoquinina (CCK)
b) insulina
c) grelina
d) secretina

4) Al comer unas papitas fritas con mayonesa, el vaciamiento gástrico disminuye por efecto
directo de la hormona:
a) colecistoquinina (CCK)
b) bombesina
c) motilina
d) gastrina

5) Recién nacido de dos horas es diagnosticado de hernia umbilical de 1,5 cm de diámetro;


el cirujano pediatra solicita una tomografía abdominal en donde se evidencia que la hernia
umbilical está ocupada por una porción del tracto gastrointestinal. ¿Qué porción del tracto
gastrointestinal estaría ocupando esta hernia con mayor probabilidad?
a) Íleon
b) Colon sigmoides
c) Duodeno
d) Colon transverso

6) Recién nacido de 7 horas, de parto por cesárea debido a polihidramnios (aumento del
volumen del líquido amniótico), con regurgitación de la leche materna y artificial, y no ha
presentado meconio. Se le realiza una tomografía donde se evidencia aire en el estómago y
una malformación del desarrollo esofágico. Con respecto a esta malformación lo más
probable es que se pueda tratar de una atresia esofágica:
a) proximal con fístula traqueoesofágica distal
b) distal con fístula traqueoesofágica proximal
c) proximal y distal
d) sin fístula

7) Lactante de 6 meses de edad que es traído a consulta por presentar vómitos no biliosos a
repetición y retraso en el crecimiento. En la radiografía de abdomen simple se observa nivel
hidroaéreo en estómago y en primera porción de duodeno (doble burbuja). ¿Cuál de las
siguientes alternativas puede explicar la condición del lactante?
a) Páncreas anular
b) Atresia duodenal en la tercera porción
c) Atresia yeyunal
d) Hipertrofia del píloro

8) En ausencia o deficiencia de la secreción de la hormona motilina, se producirá:


a) sobrecrecimiento bacteriano
b) diarrea
c) aumento del vaciamiento gástrico
d) hipertrofia del píloro

9) La estimulación parasimpática aumenta la motilidad intestinal, mientras que la


estimulación simpática la disminuye. ¿Sobre cuál de las siguientes alternativas el sistema
nervioso autónomo actúa para el control de la motilidad intestinal?

a) Potencial de membrana en el plexo mientérico (de Auerbach)


b) Frecuencia de ondas lentas
c) Secreción de secretina
d) Nivel de IP3 en el plexo submucoso (de Meissner)

10) recién nacido con protrusión de contenidos abdominales y cubiertas por amnios o
peritoneo, es cierto que:

a) Se presenta por un defecto en el cierre de la pared


b) Se acompaña de otras malformaciones congénitas
c) Se debe al no retorno de la hernia fisiológica
d) Se produce a través del ombligo

11)Lactante de 20 días con estreñimiento, distensión abdominal progresiva, acompañada


ocasionalmente de vómitos biliosos. Como antecedente, el meconio lo eliminó por primera
vez a las 72 horas de nacido. Su mamá menciona que ayuda a la evaluación con ayuda de
un termómetro rectal. Se sospecha megacolon agangliónico (Enfermedad de Hirschsprung).
¿Cuál de las siguientes alternativas explica el caso?

a) Se presenta contracciones tónicas en la región ano rectal


b) Se presenta dilatación de tracto gastrointestinal afectado
c) Las células ganglionares sólo han migrado al ano recto
d) La zona que más se afecta es inervada por fibras del nervio esplácnico menor

12) ¿Cuál de los siguientes reflejos disminuye el tránsito intestinal?


a) Doloroso
b) Gastrocólico
c) De defecación
d) Colicoileal
13) Al ingerir una sustancia ácida como el vino (ph 3), se estimula la motilidad gástrica por
acción de la hormona)
a) Motilina
b) Secretina
c) Colecistoquinina (CCK)
d) Bombesina

14) Paciente de 24 años acude a consulta externa por presentar fístula oronasal
(comunicación entre la cavidad oral y cavidad nasal). Esta fístula está asociada al
antecedente de haber sido operada del paladar hendido a los dos años de edad, durante
una campaña gratuita extranjera de corrección de paladar fisurado, ¿Cuál de las arterias
palatinas podría haberse lesionado durante esta cirugía?
a) Mayor
b) Menor
c) Ascendente
d) Rama palatina de faringea ascendente

15) Paciente de sexo masculino de 52 años con úlcera péptica gástrica de 14 años de
evolución, con cuadro de hemorragia digestiva alta hace 4 meses, sin cicatrización de la
úlcera. Entre las opciones quirúrgicas se considera vaguectomía troncal (sección del nervio
vago) a nivel del hiato esofágico. ¿Cuál de las siguientes complicaciones podría esperarse
producto de la pérdida de inervación parasimpática?
a) Menor inervación del colon ascendente
b) Se perderá el reflejo de defecación
c) Se perderá el reflejo de micción
d) Impotencia sexual

16) Paciente varón de 23 años con bulimia es traída deshidratada, semiconsciente y con
alcalosis metabólica. Los vómitos autoinfligidos por esta paciente se producen por
estimulación de receptores en la base de la lengua que mandan información directamente
al:
a) Núcleo del tracto solitario
b) Centro del vómito en el tallo encefálico
c) Zona quimiorreceptora gatillo
d) Cerebelo

17) Niño de 3 años es traído a emergencia por padre quien manifiesta que hace 10 horas
deglutió una pila pequeña de reloj de bordes romos. El niño está asintomático. Usted la
tranquiliza diciéndole es un cuerpo extraño tan pequeño que va a seguir el tránsito intestinal
como lo haría un bolo alimenticio, y que lo más probable es que en ese momento se
encuentre en:
1) colon
2) estómago
3) yeyuno
4) recto
18) Los movimientos en masa son un tipo de movimiento muy importante, una de las
consecuencias de estos movimientos es:
a) distensión rectal
b) peristaltismo del intestino delgado
c) retropulsión gástrica
d) contracción del esfínter anal interno
19) Durante la deglución, al momento que el bolo alimenticio pasa por el esfínter esofágico
superior, se espera que la presión intraesofágica:
a) disminuya en el cardias
b) disminuya en el tercio medio del esófago
c) aumente en la porción distal al bolo
d) aumente en el tercio medio del esófago
20) Paciente con enfermedad de chagas que presenta disfagia a sólidos. ¿Cuál de las
siguientes puede ser la causa de esta complicación?
a) disminución de células ganglionares en el esfínter esofágico inferior
b) aumento de la liberación de óxido nítrico en el esfínter esofágico inferior
c) disminución de las neuronas que liberan péptido intestinal vasoactivo
d) aumento de la actividad de la motilina en el esófago distal
21) El consumo de leche produce indirectamente
a) Curación de la gastritis
b) Estimulación del reflejo gastrocólico
c) Estimulación del vaciamiento gástrico
d) Inhibición del vaciamiento gástrico

22) Varón de 30 años es traído a emergencia por agresión abdominal con arma de fuego
(pistola) y es sometido a laparotomía exploratoria, observándose isquemia del colon
ascendente y parte del colon transverso ¿la lesión de cúal de las siguientes arterias
explicaría esta isquemia?
MESENTÉRICA SUPERIOR
23) Lactante masculino de 5 meses de edad producto de un primer embarazo normal
controlado . Peso de nacimiento 3.120 g y talla de 51 cm . Lactancia materna satisfactoria,
con buen incremento ponderal . Sin antecedentes patológicos hasta 5 días atrás , cuando
comienza con vómitos posprandiales no biliosos , lácteos . Los síntomas aumentan en
frecuencia y magnitud hasta hacerse explosivos después de cada alimentación . No requiere
tos , fiebre , diarrea ni lesiones cutáneas . Conserva el apetito y llora de hambre .Se espera
encontrar mayor liberación de :
a) GIP
b) Enzimas pancreáticas
c) Insulina
d) Secretina
24) Respecto a las sustancias GI que regulan la secreción pancreática; marque la
afirmación correcta:
a) La secretina, es la hormona más importante para la secreción de bicarbonato
por las células acinares del páncreas
b) La acetilcolina es capaz de estimular la secreción enzimática y de
bicarbonato del páncreas
c) La Gastrina es la hormona más importante para la secreción de enzimas
pancreáticas
d) La CCK estimula el páncreas solo para la secreción enzimática
25 ) Lactante masculino de 5 meses de edad producto de un primer embarazo normal
controlado . Peso de nacimiento 3.120 g y talla de 51 cm . Lactancia materna satisfactoria,
con buen incremento ponderal . Sin antecedentes patológicos hasta 5 días atrás , cuando
comienza con vómitos posprandiales no biliosos , lácteos . Los síntomas aumentan en
frecuencia y magnitud hasta hacerse explosivos después de cada alimentación . No requiere
tos , fiebre , diarrea ni lesiones cutáneas Una persona adulta podría tener un cuadro de
hipertrofia del píloro secundaria a :
a) Gastritis hipoclorhídrica
26) Ante una lesión del X par craneal ¿Cuál de los siguientes músculos mantiene
conservada su función?
A) Elevador del velo del paladar
B) Tensor del velo del paladar
C) Palatofaríngeo
D) Glosofaríngeo
27) Lactante masculino de 5 meses de edad producto de un primer embarazo normal
controlado . Peso de nacimiento 3.120 g y talla de 51 cm . Lactancia materna satisfactoria,
con buen incremento ponderal . Sin antecedentes patológicos hasta 5 días atrás , cuando
comienza con vómitos posprandiales no biliosos , lácteos . Los síntomas aumentan en
frecuencia y magnitud hasta hacerse explosivos después de cada alimentación . No requiere
tos , fiebre , diarrea ni lesiones cutáneas . Si el paciente tuviera 35 años de edad usted
podría pensar en un diagnóstico diferencial de la hipertrofia del píloro , la presencia de un
tumor neuroendocrino productor de la hormona
a) Secretina
b) Gastrina
c) Colecistoquinina
d) Bombesina
28) Lactante masculino de 5 meses de edad producto de un primer embarazo normal
controlado . Peso de nacimiento 3.120 g y talla de 51 cm . Lactancia materna satisfactoria,
con buen incremento ponderal . Sin antecedentes patológicos hasta 5 días atrás , cuando
comienza con vómitos posprandiales no biliosos , lácteos . Los síntomas aumentan en
frecuencia y magnitud hasta hacerse explosivos después de cada alimentación . No requiere
tos , fiebre , diarrea ni lesiones cutáneas . En relación a la estructura afectada se encuentra
a) Ligamento redondo
b) Mesenterio propiamente dicho
c) Ligamento gastroesplénico
d) Omento mayor
29) Experimentalmente se utiliza atropina (anticolinérgico) para inhibir la secreción de
gastrina, sin embargo, la secreción de esta hormona se sigue dando ante estímulos vagales.
Esta situación se explica porque la atropina:
a) Bloquea parcialmente la bomba de protones en la célula G
b) Inhibe la acción de acetilcolina e histamina en la célula G
c) Sólo inhibe la acción del péptido GRP en la célula G
d) No bloquea la acción del péptido GRP
30) Lactante masculino de 5 meses de edad producto de un primer embarazo normal
controlado . Peso de nacimiento 3.120 g y talla de 51 cm . Lactancia materna satisfactoria,
con buen incremento ponderal . Sin antecedentes patológicos hasta 5 días atrás , cuando
comienza con vómitos posprandiales no biliosos , lácteos . Los síntomas aumentan en
frecuencia y magnitud hasta hacerse explosivos después de cada alimentación . No requiere
tos , fiebre , diarrea ni lesiones cutáneas . El uso de atropina en este paciente
a) Aumenta el pH del estómago
b) Disminuye la acción del receptor CCK-B
c) Aumenta la producción de ácido clorhídrico
d) Inhibe la acción de las prostaglandinas

31) El consumo de dos vasos de agua seguidos generará indirectamente aumento en la


liberación de
a) Ácido clorhídrico
b) Enzimas pancreáticas
c) Péptido Y
d) Noradrenaline
32) Un varón de 50 años es sometido a extirpación del duodeno y parte proximal del
yeyuno. Esta situación ocasiona la pérdida de las células _____S_____, productoras de
_____ Secretina ____ que estimula la secreción de bicarbonato por el páncreas.

33) Estudiante de medicina de la UPC de 21 años sufre de gastritis aguda ocasionada por
comer en lugares poco higiénicos. Suele consumir caramelos ( chupa ) mientras está en
clase hasta la tarde. Toma gaseosa regularmente (carbohidratos 46%, sodio 53%). También
toma regular cantidad de leche (grasa 35%, lactosa 35%, proteínas 30%), pues le calma un
poco el dolor el ardor que siente por la gastritis. Incluso, cuando puede, se toma dos vasos
de agua fría para calmar las molestias. Ha decidido ir al médico para tratarse pues ya no
soporta el dolor, el cual está seguro que los síntomas se deben a una elevada producción
de ácido clorhídrico en el estómago, y por ello le ha recetado Loratadina (antihistamínico),
con lo que siente mejoría.
El uso de atropina en este paciente:
a) aumentará el pH del estómago
b) aumentará la producción de ácido clorhídrico
c) inhibirá la acción de las prostaglandinas
d) disminuirá la acción del receptor CCK-B

34) El uso de Ranitidina bloquea el receptor H2 de la histamina en las células parietales . La


histamina llega a estas células por
a) Difusión
b) Vía hematógena
c) Se produce en la misma célula parietal
d) La luz gástrica

35) Aumenta la secreción salival


a) Noradrenalina
b) Colecistoquinina
c) Secretina
d) Gastrina
36) Paciente con gastritis aguda debido a una alta producción de ácido clorhídrico si se le
hiciera un examen de sangre , se encontraría elevados los niveles de
a) Colecistoquinina

37) Recién nacido que presenta tumoración abdominal a nivel del cordón umbilical ¿cuál de
las siguientes afirmaciones es correcta respecto a este defecto en el desarrollo embriológico
del intestino?
a) Corresponde a una gastrosquisis
b) Las vísceras se hallan cubiertas por piel
c) No está asociado a otras malformaciones
d) Se asocia a malformaciones cardiacas y del tubo neural

38) El consumir caramelos eleva los niveles en sangre de una hormona cuya función es la
estimulación de las células
a) Beta del páncreas
b) Alfa del páncreas
c) G del antro
d) K del duodeno
39) La triada portal ( arteria hepática , vena portal y conducto biliar común ) está contenida
en el
a) Ligamento Gastroesplenico
b) Ligamento gastrohepatico
c) Ligamento hepatoduodenal
d) Ligamento falciforme

40) Varón de 35 años acude a la emergencia por trauma


abdominal y se decide realizar una laparoscopía
exploratoria. El cirujano observa la disposición de los
órganos abdominales como se representa en el siguiente
esquema. Esta disposición de órganos se explica por la
rotación: HORARIA DEL INTESTINO MEDIO
41) Se evalúa la expresión de la proteína Agrp en una
persona con alteración del apetito, lo correcto respecto a
esta proteína es:
a) Esta proteína es un potente anorexigénico
b) La mutación del gen que la codifica produce
adelgazamiento
c) La sobreproducción de la proteína lleva a obesidad
por agonista de receptores MC3 y MC4
d) La sobreproducción de la proteína disminuye el apetito por antagonismo de
receptores MC4.
42) Juana cae de la bicicleta y se fractura la región anterior del hueso maxilar superior con
compromiso de la fosa incisiva. Al examen físico de la región esperaría encontrar alteración
en la sensibilidad de la encía
PALATINA ANTERIOR
43) Recién nacido es atendido por el neonatólogo y luego entregado a su madre para dar de
lactar; la madre al dar de lactar observa coloración azulada de labios, acompañado de tos
persistente, dificultad respiratoria y distensión abdominal. Se le intenta colocar una sonda
nasogástrica pero esta retorna a la cavidad oral en todos los intentos ¿cual de los siguientes
anomalías el desarrollo es el más probable en este caso?
a) Estenosis esofágica proximal con fístula traqueo esofágica distal
b) Atresia esofágica proximal con fístula traqueo esofágica distal
c) Atresia esofágica distal con fístula traqueoesofágica proximal
d) Fístula traqueoesofágica proximal y distal
44) Estudiante de medicina de la UPC de 21 años sufre de gastritis aguda ocasionada por
comer en lugares poco higiénicos. Suele consumir caramelos ( chupa ) mientras está en
clase hasta la tarde. Toma gaseosa regularmente (carbohidratos 46%, sodio 53%). También
toma regular cantidad de leche (grasa 35%, lactosa 35%, proteínas 30%), pues le calma un
poco el dolor el ardor que siente por la gastritis. Incluso, cuando puede, se toma dos vasos
de agua fría para calmar las molestias. Ha decidido ir al médico para tratarse pues ya no
soporta el dolor, el cual está seguro que los síntomas se deben a una elevada producción
de ácido clorhídrico en el estómago, y por ello le ha recetado Loratadina (antihistamínico),
con lo que siente mejoría.
El consumo rápido de 500 mL de gaseosa aumentará directamente la concentración sérica
de cuál de las siguientes hormonas:
a) Colecistoquinina (CCK)
b) Neuropéptido Y
c) Secretina
d) Gastrina
45) Estudiante de medicina de la UPC de 21 años sufre de gastritis aguda ocasionada por
comer en lugares poco higiénicos. Suele consumir caramelos ( chupa ) mientras está en
clase hasta la tarde. Toma gaseosa regularmente (carbohidratos 46%, sodio 53%). También
toma regular cantidad de leche (grasa 35%, lactosa 35%, proteínas 30%), pues le calma un
poco el dolor el ardor que siente por la gastritis. Incluso, cuando puede, se toma dos vasos
de agua fría para calmar las molestias. Ha decidido ir al médico para tratarse pues ya no
soporta el dolor, el cual está seguro que los síntomas se deben a una elevada producción
de ácido clorhídrico en el estómago, y por ello le ha recetado Loratadina (antihistamínico),
con lo que siente mejoría.
Entre las sustancias cerebrales que producen ansiedad está la serotonina, la cual también
tiene acción:
a) Anorexigénica
b) Orexigénica
c) No influye en la producción de apetito
d) La acción de la serotonina en el apetito no está descrita
46) Estudiante de medicina de la UPC de 21 años sufre de gastritis aguda ocasionada por
comer en lugares poco higiénicos. Suele consumir caramelos ( chupa ) mientras está en
clase hasta la tarde. Toma gaseosa regularmente (carbohidratos 46%, sodio 53%). También
toma regular cantidad de leche (grasa 35%, lactosa 35%, proteínas 30%), pues le calma un
poco el dolor el ardor que siente por la gastritis. Incluso, cuando puede, se toma dos vasos
de agua fría para calmar las molestias. Ha decidido ir al médico para tratarse pues ya no
soporta el dolor, el cual está seguro que los síntomas se deben a una elevada producción
de ácido clorhídrico en el estómago, y por ello le ha recetado Loratadina (antihistamínico),
con lo que siente mejoría.
El consumo de leche produce directamente un aumento de los niveles séricos de la
hormona:
a) Gastrina
b) Grelina
c) Leptina
d) Péptido intestinal vasoactivo (VIP)
47) ¿En qué casos los vómitos son siempre biliosos?
a) Estenosis hipertrófica del píloro
b) Atresia duodenal
c) Atresia esofágica
d) Atresia yeyunal
48) Cuando la contracción peristáltica primaria está atraviesa el tercio superior del esófago,
el esfínter esofágico inferior estará:
a) Sin modificaciones
b) Denervado
c) Contraído
d) Relajado
49) Las arterias que derivan del tronco celíaco son, excepto:
a) Hepática derecha
b) Hepática común
c) Gástrica izquierda
d) Arteria esplénica
50) El uso de Ranitidina bloquea el receptor H2 de la histamina en las células parietales. La
histamina llega a estas células por:
a) Difusión
b) Vía hematógena
c) Se produce en la misma célula parietal
d) La luz gástrica
51) En cuanto a los reflejos gastrointestinales, un reflejo que estimula el tránsito intestinal es
el reflejo:
a) Cólico-ileal
b) Vómito
c) Gastrocólico
d) Entero-gástrico
52) Marque la respuesta correcta
a) La pared gástrica en el fondo es más delgada que en el cuerpo y antro
b) El esfínter de Oddi rodea a la papila menor duodenal
c) El bronquio derecho constituye una de las estrecheces del esofago
d) Todos los órganos del sistema digestivo tienen capa serosa
53) ¿En qué capa se encuentra la alteración principal en el Hirschsprung o megacolon
agangliónico?
a) Submucosa
b) Serosa o Adventicia
c) Muscular propia
d) Mucosa
54) Con respecto a las ondas lentas, marque la afirmación correcta:
a) Su frecuencia aumenta por acción de la acetilcolina
b) Son potenciales de acción que producen la contracción del tracto
gastrointestinal
c) Son contracciones rítmicas espontáneas
d) Su frecuencia disminuye por acción de la acetilcolina
55) En un paciente con apendicitis aguda la sensación de dolor producido por esta
inflamación es llevada por el nervio:
a) Vago
b) Pélvico
c) Esplacnico mayor
d) Esplacnico menor
56) La triada sintomática: vómitos explosivos post-prandiales, movimientos
peristálticos epigástricos visibles de izquierda a derecha y nódulo palpable
epigástrico subcostal derecho pertenece a:
a) Tumor benigno de las glándulas submucosas
b) Estenosis congénita hipertrófica del píloro
c) Colon irritable
d) Páncreas anular

EXAMEN FINAL 2019-01

1. La explicación fisiológica de presentar somnolencia de 30 minutos a 1 hora después de


ingerir alimentos, se explica por:
a. Aumento del cloro intraluminal
b. Aumento del bicarbonato intraluminal
c. Disminución de ácido carbónico en la célula parietal

d. Disminución de la actividad de la anhidrasa carbónicae. Aumento de la alcalinidad


sanguínea

2. Con respecto a la irrigación arterial del colon, a que arteria corresponde la señalada con
la flecha
a. Cólica derecha
b. Cólica media
c. Cólica izquierda
d. Ileobisecoapendículocólica
e. Arco de Riolano

3. Si un paciente presentara dentro del punto de vista fisiológico, una disminución de


enterocinasa, entonces esto originaría una disminución de la actividad de:
a. la pepsina
b. la lipasa (PUEDE SER POR SU RELACIÓN CON LA COLIPASA)
c. la quimotripsina
d. el péptido insulinotrópico dependiente de glucosa
e. la amilasa
4. Con respecto a la anatomía del hígado, señale a qué estructura pertenece la marcada por

el número 1.
a. Ligamento falciforme
b. Línea de Cantlie
c. Ligamento triangular
d. Ligamento coronario
e. Ligamento teres

5. Se presenta un paciente, el cual presenta un antecedente de tuberculosis intestinal, por lo


cual, se le resecó 80 cm de íleon distal. Desde el punto de vista fisiológico, el paciente
puede presentar una de las siguientes alteraciones:
a. Disminución de la secreción de Vitamina B12
b. Aumento indiscriminado de absorción de ácido fólico
c. Disminución de la absorción de hierro
d. Aumento de la secreción de bicarbonato
e. Disminución de la absorción de ácido glicocólico

6. Un paciente es sometido experimentalmente a un fármaco que modifica el flujo salival,


obteniéndose un volumen de saliva de 288 ml en 6 horas. En este caso las concentraciones
de electrolitos y bicarbonato en la saliva obtenida varían de la siguiente manera:
a. ↑ Na+, ↓ K+, ↑ Cl-, ↑ HCO3-
b. ↓ Na+, ↓ Cl-, ↑ K+, ↓ HCO3-
c. ↑ Na+, ↑ Cl-, ↓ K+, ↓ HCO3-
d. ↑ Na+, ↑ Cl-, ↑ K+, ↑ HCO3-
e. ↓ Na+, ↓ Cl-, ↓ K+, ↓ HCO3-

7. La siguiente imagen histológica corresponde a la glándula ............... y la estructura


señalada produce .........
a. salival sublingual / mucopolisacáridos
b. oxíntica / pepsinógeno
c. salival submaxilar / ptialina
d. salival parótida / amilasas
e. antrales / gastrina

8. Paciente varón de 65 años con antecedente de hipercolesterolemia, hipertensión arterial,


fibrilación auricular y dos infartos al miocardio previos, aqueja de dolor abdominal intenso de
inicio súbito, distensión abdominal, se decide cirugía con resección de 1,5 metros de
intestino delgado terminal y colon ascendente. Como consecuencia de la resección el
paciente tendrá deficiencia de:
a. Vitamina C
b. Tiamina
c. Vitamina A
d. Vitamina B1
e. Vitamina B6

9. Uno de los siguientes elementos debería hallarse con más probabilidad en el esófago de
un paciente que sufre de reflujo gastroesofágico...
a. Pepsina
b. Tripsina
c. Quimotripsina
d. Carboxipeptidasa
e. Ácidos biliares

10. Un paciente de 40 años cursa con anemia de 8g/dl, aqueja además de astenia y
sensación de hormigueo bilateral en los miembros inferiores, al examen se halla alteración
de la sensibilidad a la vibración y camina con ampliación de la base de sustentación. Uno de
los siguientes procedimientos sería de ayuda para el diagnóstico de este paciente:
a. Tomografía cerebral
b. Biopsia de la mucosa gástrica
c. Biopsia de hígado
d. Examen de sangre oculta en heces
e. Biopsia de Íleon proximal

11. Paciente de 60 años ingresa por caída hace 1 hora y pequeño hematoma en cuero
cabelludo, al examen físico ampliado se observa ictericia de piel y mucosas generalizada,
abdomen blando, se palpa estructura quística no dolorosa en hipocondrio derecho que
corresponde a vesícula biliar (signo de Courvoisier), en los exámenes de laboratorio se halla
niveles bajos en la formación de estercobilinógeno y urobilinógeno en heces, incremento de
la bilirrubina conjugada en la orina, elevación de fosfatasa alcalina y gamma glutamil
transpeptidasa séricas. El presente cuadro puede ser explicado por:
a. Reabsorción de hematoma
b. Litiasis vesicular
c. Carcinoma de la cabeza de páncreas
d. Carcinoma con estenosis del conducto hepático común
e. Anemia hemolítica

12. Paciente varón de 58 años con antecedente de alcoholismo crónico es diagnosticado y


recibe tratamiento por cirrosis hepática. Hace 2 días refiere familiar que tuvo cambio de
conducta y no reconoce a algunos familiares. Al examen físico, se halla ascitis, circulación
colateral en abdomen, telangiectasias, en el examen de sistema nervioso: rigidez de
extremidades, ROT incrementados, desorientación en el espacio y asterixis. ¿cuál de las
siguientes circunstancias, explicaría el cuadro en este paciente?
a. Uso de diuréticos ahorradores de potasio
b. Incremento de actividad de ureasa bacteriana duodenal
c. Hemorragia gastrointestinal
d. Disminución de la producción de NH3+ en el colon
e. Dieta normoproteica

13. Un recién nacido presenta vómitos biliosos poco tiempo después de cada alimento. Al
preguntar a la madre sobre antecedentes, ella recuerda que tuvo polihidramnios durante la
gestación, pero un análisis de cariotipo fue normal. Una de las siguientes es la causa más
probable de estos hallazgos en el recién nacido:
a. Enfermedad de Hirschsprung
b. Fístula tráqueo esofágica
c. Divertículo ileal
d. Estenosis pilórica
e. Malrotación de la yema pancreática ventral

14. Un lobulillo hepático se puede dividir en tres zonas como se muestra en el gráfico. ¿Cuál
de las siguientes afirmaciones sobre las tres zonas es verdadera?
a. La zona 1 tiene los menores depósitos de glucógeno
b. La zona 3 es la primera en afectarse en una colestasis extrahepática
c. La zona 2 es más susceptible a la injuria por isquemia que la zona periportal
d. La zona 2 tiene la mayor capacidad de regeneración
e. La zona 1 es la que tiene menos actividad metabólica.

15. En un estudio de la secreción de hormonas gastrointestinales, sus concentraciones en la


vena porta se midieron durante perfusión luminal del intestino delgado con soluciones de
diversas magnitudes de pH. ¿Qué hormona aumentará en el plasma de la vena porta
durante perfusión a través del intestino con una solución de pH 3?
a. CCK
b. gastrina
c. GIP
d. motilina
e. secretina

16. Paciente de 30 años que ingresa a causa de un traumatismo abdominal cerrado. En la


exploración se aprecia discreta palidez de piel y mucosas, auscultación pulmonar normal,
taquicardia de 120 /min. Discreta distensión abdominal y matidez en flancos; el hematocrito,
que era prácticamente normal al ingreso, disminuye un 30% a las tres horas. En la Rx de
tórax se objetiva fractura de las costillas 10-11 izquierdas. La causa más probable de la
anemización en este paciente es:
a. traumatismo renal con hemorragia retroperitoneal.
b. rotura de hígado con hemoperitoneo.
c. rotura de bazo con hemoperitoneo.
d. rotura de mesos con hemoperitoneo.
e. traumatismo pancreático con pancreatitis traumática.
17. Mujer de 65 años. Consulta por síndrome constitucional asociado a dolor abdominal
epigástrico progresivo irradiado a espalda, de dos meses de evolución. El diagnóstico de
sospecha de adenocarcinoma de páncreas se confirma por biopsia. Se realiza examen de
imagen de abdomen para evaluación de estructuras vasculares próximas al tumor
pancreático. ¿Cuál es el nombre de la vena señalada que está ausente, trombosada por
infiltración tumoral, condicionando circulación colateral en la pared gástrica?
a. Mesentérica superior
b. Coronaria estomáquica
c. Esplénica
d. Porta
e. Renal izquierda

18. Revisando la angiotomografía de un hombre de 70 años en estudio por aneurisma de


aorta abdominal, el radiólogo le informa de la presencia de una oclusión completa de la
arteria mesentérica inferior. El paciente se encuentra completamente asintomático. La
oclusión de la arteria mesentérica inferior cursa de manera asintomática en muchas
ocasiones ya que el territorio que irriga puede recibir flujo proveniente de la arteria:
a. cólica derecha
b. gastroduodenal
c. Epigástrica inferior izquierda
d. esplénica
e. cólica media

19. En las patologías de esófago es importante conocer bien la anatomía esofágica. ¿Cuál
de las siguientes afirmaciones es correcta?
a. El esófago tiene capa mucosa, muscular y serosa
b. El esófago abdominal es más largo que el cervical
c. El esófago torácico pasa por detrás del cayado aórtico
d. El epitelio esofágico normal es de tipo cilíndrico.
e. El esófago abdominal es discretamente más largo que el torácico

20. A pesar de que pueda haber variaciones anatómicas, lo habitual es que el ciego sea
irrigado por una rama arterial que proviene de unas de las siguientes arterias:
a. Iliaca derecha
b. Mesentérica inferior
c. Hepática derecha
d. Mesentérica superior
e. Iliaca izquierda

21. Ante un paciente con una cirugía abdominal urgente, el informe operatorio señala que se
ha realizado una resección de todo el duodeno y del tercio proximal del yeyuno manteniendo
íntegros el estómago y todo el íleon, así como los dos tercios distales del yeyuno. En el
seguimiento nutricional del paciente ¿Que vitamina o mineral presentará con menor
probabilidad una disminución de su absorción?
a. Cianocobalamina
b. Calcio
c. Hierro
d. Transcobalamina
e. Transferrina
22. ¿Cuál de las siguientes alternativas detallan las venas que confluyen y forman la vena
señalada?
a. mesentérica superior, gástrica izquierda y gastroepiploica izquierda
a. mesentérica inferior, gástrica izquierda y renal
b. esplénica, mesentérica superior y mesentérica inferior
c. esplénica, pancreatoduodenal y omental izquierda
d. gástrica izquierda, esplénica y hepática común

23. ¿Cuál de las siguientes sustancias forma parte de la secreción biliar?


a. Tripsina
b. Lecitina
c. Elastasa
d. Quimotripsina
e. Pepsina

24. El tubo digestivo contiene diferentes tipos de epitelios y glándulas. La estructura


señalada es una ................... y está localizada en el ...............
a. glándula de Brunner / intestino grueso
b. cripta de Lieberkühn / colon
c. cripta de Lieberkühn / intestino delgado
d. glándula oxíntica / estómago
e. célula parietal / estómago

25. ¿De qué músculo forma parte el ligamento inguinal?


a. Oblicuo externo del abdomen
b. Oblicuo interno del abdomen
c. Transverso del abdomen
d. Psoas
e. Dorsal ancho

26. Señale cuál de las siguientes afirmaciones NO se relaciona a la siguiente glándula


anexa del tubo digestivo mostrada en la imagen:
a. Es una glándula exocrina compuesta exclusivamente por acinos serosos
b. Su inervación está dada por el nervio auricular mayor (ramo posterior C2), que
inerva la vaina de la glándula así como la piel por encima de esta.
c. Esta glándula produce una secreción mucinosa acuosa, llamada mucoserosa, a
través del conducto de Wharton.
d. Su inflamación puede ser causada por un virus de los Paramyxoviridae, que
provocan una enfermedad muy frecuentemente en niños y adolescentes
e. Es una glándula endocrina y probablemente sea de origen pancreático

27. ¿Cuál de las siguientes enzimas está localizada en el borde del cepillo y juega un rol en
la digestión de proteínas?
a. Alpha dextrinase
b. Pepsina
c. Enterocinasa
d. Lactasa
e. Carboxipeptidasa A.
28. Una de los siguientes sustancias, NO sirve como un buen agente emulsificante:
a. Colesterol
b. Ácidos grasos
c. Sales biliares
d. Lecitina
e. Proteínas de la dieta

29. La sustancia que estimula el crecimiento de la mucosa gástrica es:


a. Secretina
b. Motilina
c. Péptido estimulante de la mucosa gástrica
d. Gastrina
e. Histamina

30. ¿Cuál de las siguientes alternativas es una función de la colecistocinina?


a. Relajación de la vesícula para la salida de bilis
b. Secreción de ácidos biliares
c. Contracción del esfínter de Oddi
d. Secreción de enzimas pancreáticas
e. Contracción del duodeno

31. Con respecto a la anatomía del tronco celíaco, señale lo correcto


a. El tronco celíaco se origina de la cara posterior de la aorta abdominal
b. Es una arteria delgada que tiene un calibre entre 2 y 3 mm
c. Una de sus ramas es la arteria gástrica derecha
d. La hepática común que es una de sus ramas, participa en la irrigación del estómago.

32. Con respecto a la anatomía del duodeno, marque la respuesta correcta:


a. Tiene una distribución en forma de “C”, que rodea la cola del páncreas
b. La 3ra porción duodenal está contenida en la pinza vascular aortomesentérica
c. Entre la 1ra y 2da porción se forma un ángulo, conocido como el ángulo de Treitz
d. La 4ta porción se dirige a la izquierda, hacia abajo y hacia atrás.
e. En la tercera porción desemboca el conducto colédoco.

33. El hígado está ampliamente tapizado por peritoneo, la estructura que conecta la cara
diafragmática del hígado precisamente con el diafragma es el ligamento:
a. teres
b. falciforme
c. triangular
d. hepático común
e. coronario

34. En el íleon se absorbe aproximadamente el 95% de ...................... a través de la


circulación enterohepática.
a. agua
b. colesterol
c. sales biliares
d. hidroxicobalamina
e. factor intrínseca

35. La ................ estimula el mecanismo paracrino de la secreción de ácido clorhídrico.


a. histamina
b. acetilcolina
c. gastrina
d. secretina
e. somatostatina

36. En la digestión de proteínas, ................ es el principal estímulo para convertir el


pepsinógeno en pepsina.
a. la gastrina
b. el pH ácido
c. la acetilcolina
d. la ptialina
e. la somatostatina

37. Con respecto a la somatostatina, marque lo correcto:


a. Es secretada por las células S del intestino
b. Induce a la producción de VIP
c. Interviene en la fase intestinal de la secreción gástrica
d. Produce acetilcolina para estimular a la célula parietal
e. No interviene en la regulación de la secreción de ácido clorhídrico

38. En pecten anal, es una estructura comprendida entre:


a. la línea pectínea y los senos anales
b. la línea blanca y la apertura anal
c. el esfínter anal interno y el externo
d. la línea anocutánea y la línea pectínea
e. la línea blanca y columnas anales

39. ¿Cuál de las siguientes alternativas es una proenzima pancreática?


a. Tripsina
b. Elastasa
c. Quimotripsinógeno
d. Amilasa
e. Procarboxipeptidasa C.

40. En la segmentación hepática de Couinaud, el segmento hepático señalado con la flecha,


corresponde a: En la segmentación hepática de Couinaud, la flecha señala el segmento
.............. Hepático.
a. IV
b. V
c. VI
d. VII
e. VIII

EXAMEN FINAL 2019-02


1. La reabsorción de sodio y cloro en las glándulas salivales se da principalmente en el:
a) conducto intercalado
b) conducto excretor
c) conducto estriado
d) acino glandular

2. Paciente de 35 años con dolor en hipocondrio derecho irradiado a la punta de la


escápula. Se observa en la ecografía abdominal cálculos en la vesícula biliar. Es operado de
emergencia realizando una incisión a lo largo del reborde costal derecho. La información de
dolor de esta zona es inervada por las raíces nerviosas:
a) T5 – T9
b) T6 – T7
c) T7 – T8
d) T9 – T10

3. Paciente de 23 años con dolor intenso periumbilical de inicio agudo. Es ingresado a sala
de operaciones por abdomen agudo, el cirujano encuentra sangrado a unos 60 cm proximal
a la válvula ileocecal. La estructura que está sangrando muy probablemente es un derivado
embriológico del:
a) uraco
b) ligamento Hepatoduodenal
c) alantoides
d) conducto vitelino

4. Paciente de 64 años de edad con dolor en epigastrio que se distribuye en banda hacia la
espalda. En la tomografía se encuentra tumor en cabeza de páncreas que compromete a un
vaso que discurre entre la cabeza del páncreas y el proceso uncinado. Debido a esto, este
cáncer se considera irresecable porque compromete a la arteria:
a) mesentérica superior
b) mesentérica inferior
c) tronco celíaco
d) aorta

5. Paciente con carcinoma gástrico avanzado, en preoperatorio para gastrectomía total.


¿Cuál de los siguientes ganglios linfáticos recibirá primero células metastásicas con mayor
probabilidad:
a) celíaco
b) cisterna del quilo
c) esplénico
d) gástrico posterior

6. Paciente de 77 años con dolor abdominal difuso de 3 horas de evolución. En los estudios
de imágenes se observa oclusión probablemente aterosclerótica de la arteria mesentérica
superior; no se observa necrosis, lo cual puede ser explicado por la irrigación sanguínea
colateral. ¿Qué vasos ofrecen colaterales entre el tronco celíaco y la arteria mesentérica
superior?
a) Gástrica izquierda y hepática
b) Cística y duodenal
c) Gastroomental derecha e izquierda
d) Pancreaticoduodenal superior e inferior

7. Paciente de 62 años con vólvulo de intestino delgado e isquemia intestinal. Se realiza


laparotomía exploratoria para liberar la obstrucción. ¿Cuál estructura se utilizará como punto
de referencia para determinar la posición de la unión duodenoyeyunal?
a) Vasos rectos
b) Ligamento suspensorio del duodeno (de Treitz)
c) Ligamento frenocólico
d) Nacimiento de la Arteria mesentérica superior

8. Paciente de 42 años con dolor abdominal intenso y hematemesis. En la endoscopia se


observa una úlcera duodenal posterior perforada con hemorragia intraabdominal. ¿Cuál de
las siguientes arterias estará comprometida?
a) Gástrica izquierda
b) Mesentérica superior
c) Gástrica derecha
d) Pancreaticoduodenal posterosuperior

9. Paciente de 51 años con antecedente de enfermedad diverticular acude a emergencia por


sangrado profuso y dolor en cuadrantes inferiores. ¿Cuál es el origen más probable de la
sangre que pierde en la hemorragia?
a) Cólica media
b) Mesentérica inferior
c) Rectal superior
d) Rectal inferior

10. Paciente de 13 días de vida con vómitos explosivos a las dos horas después de lactar.
Al examen físico se palpa la oliva pilórica. ¿Cuál es el nervio cuyos filetes dan inervación
eferente a la estructura afectada?
a) Vago
b) Esplácnico torácico mayor
c) Mesentérico superior
d) Esplácnico torácico menor

11. Paciente de 80 años con dolor abdominal intenso y antecedente de estreñimiento


crónico. En la colonoscopia se observa divertículos con áreas ulceradas difusas en colon
sigmoides y descendente. El cirujano programa una cirugía de resección y planifica que
para realizar esta recepción tendrá que cortar los siguientes vasos y nervios:
a) Plexo mesentérico superior y arteria rectal.
b) Nervio esplácnico torácico mayor y arteria cólica media.
ramas del nervio vago y arteria ileocólica.
d) nervio esplácnico pélvico y arteria cólica izquierda.

12. Paciente de 46 años ingresa a emergencia con dolor en cuadrante superior derecho e
ictericia. En la ecografía se observa cálculos en la vesícula biliar. ¿Cuál de los siguientes
nervios transmite el dolor de la colecistitis?
a) Filetes aferentes del nervio vago derecho, referido al ángulo inferior de la escápula
b) Filetes nerviosos de las raíces T1 a T4, con referencia al hombro izquierdo
c) Fibras aferentes simpáticas del nervio esplácnico torácico mayor, con referencia a
los dermatomas T6 a T8
d) Fibras aferentes de los ramos primarios dorsales de los nervios raquídeos T8 a T10
con referencia al epigastrio

13. Un niño de 8 años es alimentado por sus padres con un Cheese Burger, papitas fritas y
un vaso de Coca Cola. ¿La presencia de lípidos, carbohidratos y proteínas en el duodeno
estimulan la liberación de cuál de las siguientes hormonas en la mucosa duodenal?
a) Colecistoquinina (CCK)
b) Péptido inhibidor gástrico (GIP)
c) Secretina
d) Insulina

14. La fase cefálica del control de la secreción gástrica corresponde a cerca del 30% de la
secreción ácida y es consecuencia de un reflejo. ¿Cuál de las siguientes alternativas puede
eliminar totalmente la fase cefálica de la secreción gástrica?
a) Vaguectomía
b) Uso de atropina
c) Uso de bloqueador de receptor H2 de histamina
d) Uso de antiácidos

15. Recién nacido de 24 horas con llanto, vómitos y sin eliminación de meconio. Luego de
los estudios auxiliares se diagnostica la Enfermedad de Hirschsprung, esta enfermedad se
caracteriza por la ausencia de:
a) las células de Cajal en plexo mientérico
b) el plexo mientérico en el recto y colon sigmoides
c) las células mioepiteliales en el recto distal
d) el plexo de Meissner en el recto distal y colon sigmoides

16. Los complejos motores migratorios aparecen aproximadamente cada 90 minutos entre
las comidas, y se considera que son estimulados por la hormona motilina. La ausencia de
estos complejos migratorios podría producir un aumento en:
a) la motilidad duodenal
b) el vaciamiento gástrico
c) la deglución
d) las bacterias intestinales

17. ¿Cuál de las siguientes es una consecuencia probable de la resección del íleon?
a) Gastritis atrófica
b) Deficiencia de vitamina B12
c) Esteatorrea
d) Úlcera péptica

18. Los movimientos en masa son importantes en la fisiología intestinal. Estos movimientos
en masa ocasionan:
a) la sensación de defecar
b) el peristaltismo duodenal
c) la retropulsión gástrica
d) la contracción del esfínter anal interno

19. La toxina colérica hace que aumenten los niveles de AMPc intracelular, y este aumento
hace que se mantenga abierto un canal en las células de la cripta de Lieberkühn. En
condiciones fisiológicas, en una persona sana ¿Qué sustancia puede promover que el canal
quede abierto también?
a) Somatostatina
b) Óxido nítrico (NO)
c) Péptido intestinal vasoactivo (VIP)
d) Péptido similar al glucagón 1 (GLP 1)

20. Paciente de 64 años con tumor abdominal que comprime la cisterna del quilo. En la
biopsia de duodeno tomada como parte del estudio, el patólogo puede observar
a) Dilatación del vaso quilífero central
b) Contracción de las venas de las vellosidades
c) Vellosidades intestinales más largas
d) Engrosamiento de la lámina basal

21. En la enfermedad de Crohn es posible encontrar células de Paneth en el colon. Esto se


puede deber a la especial función de estas células en:
a) la activación de la inmunidad adquirida
b) mantener la inmunidad innata
c) producir hormonas
d) producir Ig A

22. En 1967 se descubrió que la epidemia de Kuru, una enfermedad por priones, en el
distrito de Okupa en Papúa Nueva Guinea, era causada por la costumbre de comer la carne
de los muertos. Ahora se sabe que las proteínas priónicas ingresan al organismo a través
de:
a) los enterocitos
b) la transmigración
c) las células de Paneth
d) las células M

23. Paciente de 48 años con alteraciones en el tránsito intestinal por diabetes mellitus tipo 2;
se presenta con esteatorrea, flatulencia y malabsorción de grasas. Las pruebas de función
hepática y biliar están dentro de rangos normales. Una causa de la disminución de sales
biliares puede ser:
a) el sobrecrecimiento bacteriano
b) la deficiencia de pepsina
c) la deficiencia de elastasa
d) la hiperestimulación del GLUT5

24. Al usar azúcar de mesa (sacarosa) para endulzar su café, el estudiante de medicina
sabe que lo más probable es que para su absorción tendrá utilizar el/los transportadoras)
________________ que se encuentran en la membrana apical de los enterocitos.
a) SGLT-1
b) GLUT2 y GLUT5
c) SGLT-1 y GLUT5
d) SGLT1 y GLUT2

25. Una persona con la producción normal de lactasa; cada vez que toma leche, los
productos de la degradación de la lactosa por parte de la lactasa ingresarán al enterocito
usando el/los transportador(es) _____________:
a) SGLT-1
b) GLUT2 y GLUT5
c) SGLT-1 y GLUT5
d) SGLT1 y GLUT2

26. En una persona sana, luego de una comida basada en carnes rojas cocinadas con baja
cantidad de sal, al enterocito pueden ingresar solamente:
a) aminoácidos
b) aminoácidos y dipéptidos
c) aminoácidos, dipéptidos y tripéptidos
d) aminoácidos, dipéptidos, tripéptidos y tetrapéptidos

27. Paciente con deficiencia congénita de procolipasa, sufre de esteatorrea cada vez que
come comidas ricas en grasas. En el estudio de composición de las heces, uno espera
encontrar una cantidad aumentada de:
a) colesterol
b) triglicéridos
c) fosfolípidos
d) lisolecitina

28. Al tomar su café en Starbucks, un estudiante de medicina sufre una quemadura de


primer grado en el tercio anterior de la superficie dorsal de la lengua. La información de
dolor es transmitida por el nervio:
a) cuerda del tímpano
b) glosofaríngeo
c) lingual
d) facial

29. Paciente de 32 años con esteatosis hepática no alcohólica. Se le realiza una biopsia
hepática que confirma la esteatosis; en el tejido se observan depósitos de lípidos en los
hepatocitos, los cuales contienen principalmente:
a) triglicéridos
b) colesterol
c) ácidos grasos libre
d) acil-carnitina

30. Paciente de 21 años que es estudiante de medicina, llega a consulta refiriendo que en
épocas de exámenes su piel se vuelve amarillenta. Le realizan el diagnóstico de Síndrome
de Gilbert asociado a mutación del gen UGT1A1, luego de unas semanas acude a la
consulta por ictericia asociada a resfriado. Al hacerle un análisis de sangre, usted sabe que
encontrará valores elevados de:
a) bilirrubina directa
b) bilirrubina indirecta
c) bilirrubina tipo delta
d) fosfatasa alcalina

31. Se realizó un experimento en el cual se inyectó tinta china en el peritoneo de ratas de


laboratorio. Al realizarse una biopsia hepática de dichos animales, se encontró que el tinte
negro estaba depositado en:
a) las células de Ito
b) los hepatocitos
c) las células de Kupffer
d) las células de Disse

32. Paciente de 74 años de edad con shock hipovolémico asociado a deshidratación aguda
severa. En este paciente es posible encontrar hipoxia en zona ...... del lobulillo hepático e
infarto .................. del intestino.
a) 1 / mucoso
b) 1 / transmural
c) 3 / transmural
d) 3 / mucoso

33. Niño de 5 años con historia de tres días de evolución caracterizado por fiebre, malestar
general, odinofagia, anorexia, e irritabilidad. Al examen se observa lesiones ulcerativas de 4
mm de diámetro en mucosa yugal, con fondo blanquecino y eritema periférico. El
diagnóstico más probable es:
a) candidiasis oral
b) leucoplasia
c) aftas
d) herpes

34. Paciente de 52 años con enfermedad por reflujo gastroesofágico de 30 años de


evolución. Se realiza endoscopía en la que se encuentra mucosa eritematosa proximal a la
línea Z. Para corroborar la presencia de lesión preneoplásica, se toma una biopsia de esa
zona, en la que se espera encontrar:
a) metaplasia gástrica
b) metaplasia intestinal
c) displasia gástrica
d) adenocarcinoma

35. Un estudiante de medicina termina su último examen final. Al abrir la boca para comer
una hamburguesa con papas fritas, la activación de los receptores muscarínicos de las
células acinar y ductal estimularan un mayor flujo de saliva, con lo cual disminuirá la
concentración salival de:
a) potasio
b) bicarbonato
c) sodio
d) cloro

36. Un estudiante toma su desayuno consistente en un pan con mantequilla y queso. Antes
que se puedan digerir las grasas, es necesario que sean emulsificadas. La hormona
............................ estimula la liberación de las sustancias emulsificadoras.
a) colecistoquinina (CCK)
b) secretina
c) lipasa pancreática
d) gastrina

37. Al comer un pollo a la brasa entero, con papitas fritas y ensalada, la sustancia que
estimulará la liberación de HCl en el estómago es:
a) el neuropéptido Y
b) la secretina
c) la bombesina
d) la colecistoquinina (CCK)

38. Paciente con disminución marcada del apetito asociado a cáncer terminal, se podría
utilizar análogos de ................. para promover la ingesta de alimentos.
a) el péptido similar al glucagón (GLP)
b) la serotonina
c) la secretina
d) la endorfina

39. Un hombre de 51 años presenta de forma súbita vómitos masivos de sangre roja
brillante. Tiene como antecedente hepatitis viral B hace 23 años. En la exploración física: FC
103 latidos/min, PA 85/50 mmHg, se palpa la punta del bazo y niega vómitos. Su
hematocrito es 21%, la prueba serológica de HBsAg es positiva. En la ecografía se observa
hígado con nodulaciones. ¿Cuál es la causa más probable para la hematemesis?
a) Esófago de Barrett
b) Síndrome de Mallory Weiss
c) Varices esofágicas
d) Esofagitis por reflujo

40. Durante el reflejo del vómito, uno de los primeros sucesos es:
a) el peristaltismo inverso
b) la contracción del píloro
c) la apertura de la glotis
d) el esfínter esofágico inferior abierto

41. Paciente de 34 años que acude por diarrea desde hace 4 días asociado a comer papa
rellena con ají en el Estadio Nacional, se exacerba cuando toma lácteos o come grasas. Se
acompaña de flatulencia y episodios de tenesmo. Este cuadro de diarrea e intolerancia a la
lactosa de inicio agudo se explica por la:
a) enfermedad de Crohn
b) infección por V. cholera
c) colitis ulcerativa
d) giardiasis

42. En cuanto a la secreción pancreática, mientras mayor es el flujo, mayor es la


concentración de:
a) potasio
b) bicarbonato
c) cloro
d) sodio
43. Paciente de 10 años con diarrea crónica, distensión abdominal, anorexia. Se le ha
encontrado anticuerpos antigliadina y antiendomisio. Es más probable que la diarrea se
correlacione con el hallazgo histológico de:
a) adelgazamiento de las criptas
b) linfocitos intramusculares
c) atrofia de las vellosidades
d) úlceras duodenales

44. La glándula parótida está inervada por el par craneal:


a) VII
b) IX
c) X
d) XII

45. Las glándulas de Brunner se encuentran en la:


a) mucosa del esófago
b) submucosa del íleon
c) mucosa del yeyuno
d) submucosa del duodeno

46. ¿Cuál de las siguientes afirmaciones describe correctamente la función de la inervación


parasimpática del tracto gastrointestinal?
a) La norepinefrina es el principal neurotransmisor excitatorio.
b) La actividad parasimpática produce la relajación de los esfínteres
c) La actividad parasimpática excesiva puede provocar un trastorno llamado íleo
paralítico (parálisis del músculo liso intestinal)
d) El pH luminal, la osmolaridad y la distensión muscular son detectados por fibras
parasimpáticas eferentes

47. Al ingerir una cantidad de glucosa por vía oral, esta es interiorizada en las células del
organismo más rápido que si esa misma cantidad de glucosa hubiese sido administrada por
vía endovenosa. Este fenómeno sucede gracias a la acción de la sustancia secretada por
las células:
a) G
b) I
c) K
d) S

48. A ............... secreción de HCl en el lumen gástrico, ................. pH en la sangre venosa


gástrica
a) mayor /mayor
b) mayor / menor
c) menor / mayor
d) mayor / igual

49. ¿En cuál de las siguientes situaciones hay un menor flujo de secreción salival?
a) Masticar goma de mascar
b) Imaginarse ser sometido a un examen dental
c) Exposición a olor nauseabundo
d) Sueño

50. El alcohol y la cafeína estimulan la fase ______________ de la producción de ácido


clorhídrico.
a) cefálica
b) gástrica
c) intestinal

¿Cuál de los siguientes mecanismos ocurren durante la defecación? En la posición de


cuclillas el músculo puborrectal se halla relajado

La estructura número 4 corresponde a ____ y está


___. La dentina/ formado por células derivadas de
la cresta neural

Un paciente luego de un accidente sufre lesión del


piso de la boca, se constata daño del nervio:
cuerda del tímpano, en este caso se esperaría
encontrar disminución de la ____ Sensación del
gusto en los 2 tercios anteriores

¿Cuál de las siguientes afirmaciones es la correcta


sobre la gastrina? Actúa en la célula diana mediante su receptor CCK tipo B

Al recibir un paciente con signos de hipovolemia y antecedentes de trauma en abdomen por


accidente de tránsito, usted identifica radiológicamente: lesión primera vértebra lumbar y
signos de lesión en páncreas; durante la cirugía se observó pobre irrigación de asas
intestinales. El vaso afectado es la arteria: mesentérica superior

Un paciente sufre daño a nivel del cuello con lesión muscular en la región de la faringe. En
el examen físico se determina dificultad para la elevación de la faringe y para el cierre del
istmo de las fauces. En este caso probablemente está afectado el músculo: palatofaríngeo

Varón de 50 años a quien le realizan la curación de la segunda molar de la arcada derecha.


En un momento determinado, el paciente acusa de intenso dolor de la pieza dentaria en
tratamiento. La vía aferente del dolor viaja a través del nervio: trigémino V2

La distensión gástrica por los alimentos produce incremento de secreción de HCL mediante
la producción de ____ que estimula a las células ___ vía proteína: gastrina/ parietal/ Gq

Un niño de 3 años llega a emergencias con disfagia (dificultad para tragar), dolor retro
esternal, salivación y llanto. Se sospecha de ingesta de cuerpo extraño (moneda) en el
esofago, al ser evaluado se constata en una radiografía presencia de cuerpo extraño a nivel
de C6. El cuerpo extraño estará suspendido a nivel del estrechamiento producido por ____
el músculo cricofaríngeo

La triada portal (arteria hepática, vena portal y conducto biliar común) está contenida en el
ligamento ____ hepatoduodenal/ mesenterio ventral
EXAMEN FINAL 2019-00

1. Un niño de 2 años es llevado a la consulta por diarrea persistente, edema de las


extremidades y falta de crecimiento en relación a su edad. Los análisis de sangre
revelan que tiene concentración plasmática baja de proteínas (hipoproteinemia). Como
parte del estudio se coloca colecistocinina (CCK) endovenosa y se recoge muestras del
líquido duodenal por endoscopia; el resultado del líquido confirma incapacidad para
hidrolizar proteínas a un ph neutro, esta situación mejora al añadir una pequeña
cantidad de tripsina. El paciente probablemente está sufriendo la falta congénita de
______

a. PEPT – 1

b. Pepsinógeno

c. Enterocinasa

d. Carboxipeptidasa

2. Paciente mujer de 35 años acude a consulta por sensación de sequedad y lesiones


en cavidad oral. Al examen se observa atrofia de la mucosa, fisura y úlcera; nota
además sequedad e irritación de la córnea y aumento del tamaño de las glándulas
parótidas. Su diagnóstico más probable es artritis reumatoide, el hallazgo más probable
en una biopsia de glándula parótida es ____

a. Presencia de acinos normales con hiperplasia de células distales

b. Gran infiltración de linfocitos y células plasmáticas

c. Hiperplasia de acinos glandulares serosos

d. Gran infiltrado de linfocitos y neutrófilos

3. Un hombre de 42 años de edad se presenta al médico con una historia de 1 año de


evolución, caracterizado por dolor abdominal bajo y diarrea con crisis sanguinolentas.
Manifiesta además pérdida de peso de 8Kg durante este periodo. La colonoscopia revela
lesión difusa en el colon con afectación del recto. La biopsia de estas lesiones revela
adelgazamiento de la pared, inflamación y ulceración de la mucosa y submucosa. El
diagnóstico más probable en este caso es:

a. Síndrome de colon irritable

b. Enfermedad de Crohn

c. Colitis ulcerativa

d. Sprue celiaco

4. Dos estudiantes deciden tomar un receso para comer una hamburguesa a la hora del
almuerzo. Antes de llegar a la cafetería, impulsos nerviosos provenientes del complejo
vagal dorsal iniciarán la secreción de ácido gástrico por liberación de _____ desde el
sistema nervioso entérico
a. Serotonina

b. Colecistoquinina

c. Péptido inhibidor vasoactivo

d. GRP (péptido liberador de gastrina)

5. Un niño de cuatro años de edad es llevado a la consulta por cuadros diarreicos


frecuentes caracterizados por heces pálidas, voluminosas y fétidas, al examen físico
presenta bajo peso y talla para la edad. Se mide la concentración de cloruro en el sudor y
se encuentra que sus valores son muy elevados. La alteración más importante a nivel de
células ductales del páncreas tiene relación directa con la conductancia de ___

a. Bicarbonato

b. Potasio

c. Sodio

d. Cloro

6. Se evalúa los valores séricos de las siguientes sustancias a un paciente con enfermedad
hepática terminal, en este paciente se espera encontrar la combinación con la letra

a. Glucosa aumentada, amoniaco disminuido, albúmina disminuida

b. Glucosa disminuida, Amoniaco aumentado, albúmina aumentada

c. Glucosa aumentada, amoniaco aumentado, albúmina aumentada

d. Glucosa disminuida, amoniaco aumentada, albúmina disminuida

7. Una mujer de 35 años de edad HIV positiva se presenta al médico con dolor abdominal
en cuadrante superior derecho e ictericia. La paciente refiere haber tenido múltiples
episodios de ictericia durante los últimos 10 años. Los exámenes para determinar
hepatitis viral dieron positivos para hepatitis B, siendo catalogado el caso como hepatitis
crónica con alteración funcional. En un examen de sangre ¿cuál de los siguientes
parámetros está disminuido?

a. Albúmina

b. Bilirrubina

c. Fosfatasa alcalina

d. Tiempo de protrombina

8. En el reflejo peristáltico del intestino delgado ¿cuál de los siguientes eventos sucede en
la porción caudal del bolo alimenticio?

a. Acción del péptido inhibidor vasoactivo (VIP) en el músculo circular


b. Acción del NO (óxido nítrico) en el músculo longitudinal)

c. Contracción del músculo longitudinal interno

d. Acción de acetilcolina en el músculo circular

9. Varón de 58 años de edad con enfermedad de Crohn severa fue sometido a una
resección ileal. Después de la cirugía este paciente padecerá de esteatorrea esto se
explica porque:

a. Se inhibe la acción de la 7 alfa hidroxilasa

b. El pool de ácidos biliares se incrementa

c. Hay malabsorción de ácidos biliares

d. El páncreas no secreta lipasa

10. En un experimento se inserta un balón en el estómago de un voluntario, se infla poco a


poco mientras que se vigilan las presiones intraluminales. Aunque el volumen del balón
aumenta considerablemente, las presiones permanecen constantes. Esta relación
volumen presión se explica por la liberación local de

a. Acetilcolina y gastrina

b. Norepinefrina y óxido nítrico

c. Colecistoquinina y óxido nítrico

d. Óxido nítrico y péptido inhibidor vasoactivo

11. ¿cuál de las siguientes alternativas es una característica de la secreción exocrina del
páncreas?

a. Es hipotónica respecto al plasma

b. Su mayor estímulo se da en la fase intestinal

c. Es estimulada por la presencia de bicarbonato en el duodeno

d. La secreción enzimática es estimulada principalmente por la secretina

12. Las estructuras en el hígado que permite que los productos metabólicos unidos a
proteínas tengan acceso a las membranas basolaterales de los hepatocitos, son:

a. Los canalículos

b. Las células de ito

c. Las fenestras sinusoidales

d. Las uniones intercelulares herméticas


13. La composición de la bilis es modificada conforme fluye por los conductillos biliares.
Durante este tránsito se espera que aumente la concentración de

a. Ig A

b. Glucosa

c. Protones

d. Vitamina A

14. Se mide experimentalmente el contenido gástrico de dos personas. La persona “A”


tiene alto contenido de grasa y la persona “B” tiene un contenido isotónico ¿Cuál de las
siguientes es correcta respecto al vaciamiento gástrico?

a. Hay ralentización del vaciado gástrico sólo en A”

b. El vaciamiento gástrico es más rápido en ambos

c. Hay ralentización del vaciado gástrico sólo en “B”

d. Hay ralentización del vaciado gástrico en ambos casos

15. El examen endoscópico de un paciente con hipertensión portal grave revela venas
tortuosas que sobresalen hacia la luz del esófago. El paciente recibe tratamiento
quirúrgico mediante la colocación de una derivación que conecta la vena porta a la vena
cava. Después de la operación el riesgo de encefalopatía _______ y el riesgo de
sangrado de várices ______

a. Disminuirá/ disminuirá

b. Disminuirá/ aumentará

c. Aumentará/ disminuirá

d. Aumentará/ aumentará

16. Un paciente varón de 18 años de edad acude al médico para sus exámenes de
rutina. Sus resultados de laboratorio muestran un valor de bilirrubina séricas de 4 mg/dl
y una bilirrubina directa de 0,3 mg/dl. Las pruebas de función hepática son normales. La
alteración que explica mejor este caso es por la deficiencia de

a. Transaminasas

b. Hemo oxigenasa

c. La 7 alfa hidroxilasa

d. Glucoronil transferasa

17. Un hombre de 57 años de edad es llevado a urgencias con hematemesis masiva


roja brillante, a su llegada se halla inconsciente con PA: 80/40 mm Hg y FC:124 lat/min.
Luce ictérico con presencia de “arañas vasculares en el tórax anterior y extremidades”,
abdomen distendido con signo de oleada positiva. Se encuentra esplenomegalia y
pérdida de la masa muscular en extremidades. La anastomosis vascular responsable del
sangrado en este paciente es:

a. Vena gástrica izquierda y vena ácigos

b. Arteria gástrica izquierda y vena ácigos

c. Vena paraumbilical y vena epigástrica inferior

d. Vena gástrica izquierda y vena esofágica superior

18. Un estudiante de medicina está comiendo un plato de comida a base de


champiñones, espárragos y salsa de soya. El estímulo del sabor umami contenido en
todos estos alimentos viaja a través del nervio:

a. Lingual

b. Hipogloso

c. Glosofaríngeo

d. Cuerda del tímpano

19. Una paciente de 30 años de edad es sometida a una cirugía en lado medio derecho
por un problema de aterosclerosis. Luego de la cirugía refiere alteración sensitiva de la
lengua. Al evaluar el caso usted esperaría encontrar

a. Alteración en la sensación del dolor y temperatura

b. Alteración en la sensación del dolor en los dos tercios anteriores de la lengua

c. Alteración en la sensación del gusto en el tercio posterior de la lengua

d. Sensación del dolor, tacto y temperatura conservadas

20) En un paciente de 45 años de edad con colestasis biliar se encuentra una elevación de
los niveles sanguíneos de fosfatasa alcalina hasta 3 veces la cifra normal ¿Cuál de las
siguientes alternativas estará también elevada como evidencia del daño de la vía biliar?

a. Tiempo de protrombina y albúmina sérica

b. Transaminasas hepáticas (ALT y AST)

c. Gamma glutamil transpeptidasa

d. Glucoronil transferasa

21) Experimentalmente se incrementa la velocidad de se incrementa la velocidad de


secreción salival con una sustancia, en el análisis de la composición de esta saliva obtenida
se espera encontrar ………..
a. Disminución de la concentración de bicarbonato que supera la concentración
plasmática
b. Aumento de la concentración de cloro y sodio que supera la concentración
plasmática
c. Aumento de la concentración de bicarbonato que supera la concentración
plasmática
d. Disminución de la concentración de potasio y bicarbonato

22) Lactante de 3 meses de vida es atendido por presentar diarrea, se administra una
solución de glucosa y electrolitos por vía oral. La proteína de membrana apical que explica
la capacidad de esta solución para proporcionar aporte de glucosa e hidratación en …..

a. CFTR

b. SGLT-1

c. GLUT-2

d. GLUT-5

23) Paciente ha sufrido herida de bala en el abdomen, se le ha tenido que extirpar el


segmento medio y distal del íleon. En este caso la síntesis hepática de sales biliares estará
……

a. Sin cambios en el ritmo de síntesis

b. Disminuida por inhibición de la enzima colesterol 7 alfa hidroxilasa

c. Incrementada por estímulo de la enzima colesterol 7 alfa hidroxilasa

d. Incrementada por inhibición de la enzima colesterol 7 alfa hidroxilasa

24) Un varón de 75 años ingresa al consultorio por presentar ictericia marcada de piel y las
escaleras. El estudio del paciente mostró que presentaba un tumor que obstruía la totalidad
del conducto hepático común. ¿Cuál de los siguientes conductos se encontrará dilatado en
este paciente

a. de Wirsung

b. de Hering

c. colédoco

d. cístico

25) Correlaciones las dos columnas y marque la fórmula correcta:

1. Enfermedad Hirschsprung ( ) heces cono moco y sangre

2. Diarrea osmótica ( ) intolerancia a lactosa

3. Diarrea secretoria ( ) aganglionosis congénita


4. Diarrea exudativa ( ) canales de Cl- en las células de la cripta

a. 4231 b. 1234 c. 2143 d. 4213

26) La vena umbilical obliterada del hígado después del nacimiento se transforma en el
ligamento

a. Cruzado

b. Redondo

c. Coronario

d. Falciforme

27) Llega a su guardia nocturna una madre que trae a su RN masculino de 2 semanas de
vida con mal estado general y resequedad de mucosas. Usted observa que lacta
ávidamente, pero a las 2 horas presenta vomito postprandial no bilioso en proyectil. Al
realizar la historia clínica descubre que el lactante recibió profilaxis con macrólidos para tos
ferina. Usted sospecha principalmente en:

a. Estenosis pilórica hipertrófica congénita

b. Fistula traqueo esofágica

c. Estenosis duodenal

d. Atresia duodenal

28) En la regulación del apetito y la saciedad, la estimulación experimental crónica del


núcleo ventro medial del hipotálamo producirá:

a. Afagia

b. Obesidad

c. Hiperfagia

d. Activación de neuronas relacionadas a NPY

29) Paciente mujer de 25 años acude por dolor en fosa iliaca derecha que empeora al toser
o caminar, asociada a náuseas y vómitos por lo cual acude a emergencia, Dos días después
de realizarle un apendicetomía, la paciente desarrolla fiebre alta (39º C), esta hipotensa y
presenta dolor abdominal. La laparotomía exploratoria muestra un gran volumen de sangre
en la cavidad peritoneal por lesión de un vaso producida durante la apendicetomía ¿Cuál de
las siguientes arterias debe ligarse para detener la hemorragia?

a. Cólica derecha y arteria rectal superior

b. Ileocólica y arteria cólica media

c. Mesentérica superior
d. Ileocólica

30) La onda peristáltica secundaria del esófago se caracteriza por ser originada

a. Por el plexo de Meissner del esófago

b. Por el plexo mesentérico del esófago

c. Por el reflejo de la deglución

d. Durante la masticación

31) ¿Cuál de los siguientes es una causa de ictericia con bilirrubina conjugada aumentada?

a. Ictericia del recién nacido

b. Obstrucción del colédoco

c. Anemia hemolítica

d. Gran hematoma

32) En relación a la absorción de nutrientes, la absorción de dipéptidos y tripéptidos a nivel


de las células epiteliales del intestino delgado, se da principalmente debido a:

a. Incremente de canales de Cl en la membrana apical

b. La gradiente de bicarbonato en la membrana basal

c. La gradiente de iones H+ en la membrana apical

d. La gradiente de Na+ en la membrana apical

33) Paciente de 20 años es traído a la emergencia por presentar diarreas desde hace 2
días. Familiar refiere que las deposiciones son liquidas y abundantes, al examen luce
deshidratado y se plantea que la diarrea es producida por una toxina que estimula la
transformación de ATP a AMPc con apertura de cales de Cl- y perdida de agua. El tipo de
diarrea más probable es:

a. Osmótica

b. Exudativa

c. Secretoria

d. Por intolerancia a la lactosa

34) Un niño fue operado por una obstrucción intestinal, observándose la presencia de
divertículo de Meckel. Según lo diferido marque lo correcto:

a. El 50% de la población lo presenta

b. Se localiza en el íleon muy cerca al yeyuno


c. Puede poseer tejido gástrico o pancreático

d. Se produce por una mala rotación de los intestinos

35) Marque la alternativa correcta respecto a la estructura marcada en el gráfico:

a. Se halla a 2 centímetros debajo de la papila duodenal mayor

b. Llega el conducto colédoco y pancreático principal

c. Llega el conducto hepático común y pancreático principal

d. Llega el conducto pancreático accesorio

36) ¿Cuál de las siguientes moléculas se encontrará aumentada en el citoplasma de las


células parietales de un paciente con un síndrome de Zollinguer Ellison?

a. Péptido liberador de gastrina (GRP)

b. Proteína G estimulante (GS)

c. Inositol Trifosfato (IP3)

d. AMO cíclico (AMPc)

37) Los fármacos inhibidores de la bomba de protones, actúan bloqueando la …………

a. Anhidrasa carbónica

b. ATPasa H+/K+ en la membrana luminal

c. ATPasa H+/K+ en la membrana basolateral

d. ATPasa Na+/K+ en la membrana basolateral

38) Un paciente fue diagnosticado de gastritis autoinmune, ¿cuál de las siguientes


alternativas es FALSA respecto a esta enfermedad

a. Afecta principalmente el fondo y cuerpo gástrico

b. Se produce hiperplasia de células G secundaria la aclorhidria

c. El propio sistema inmune destruye principalmente las células parietales


d. Se produce atrofia de la mucosa, aclorhidria, hipergastrinemia y déficit de vitamina
B6

39) Marque la correlación correcta:

1. Herpes virus ( ) en relación al abuso de antibióticos

2. Candidiasis oral ( ) lesiones vesiculares como racimo de uvas

3. Eritroplaquia ( ) Mega esófago

4. Enfermedad de Chagas ( ) Lesión precancerigena

a.- 2431 b. 1234 c. 4123 d. 2143

40) En un paciente con insuficiencia renal crónica, el déficit en la absorción de calcio a nivel
del enterocito se debe a lo siguiente:

a. No se convierte la 25 hidroxicolecalciferol a 1,25 dihidroxicolecalciferol

b. No se convierte la 1,25 dihidroxicolecalciferol a 25 hidroxicolecalciferol

c. Existe un descenso de la alfa 25 hidroxilasa renald. Se incrementa la producción


de Calbindina

41. Marque lo correcto en relación al divertículo de Meckel: Se encuentra usualmente a 60 cm de la


VIC

42. Luego de tres horas dando exámenes, un alumno de medicina comienza a sentir
hambre. Esta situación es probable que sea mediada por la _____ que es sintetizada por el
_____ grelina / estómago

43. Varón de 72 años, con antecedente de diabetes mellitus tipo 2, que presenta entropía
diabética caracterizada por estreñimiento. Este problema puede estar asociado a:
deficiencia de óxido nítrico

44. Varón de 54 años con diabetes mellitus tipo 2, es diagnosticado de gastroparesia debido
a que presenta sensación de llenura precoz al comer, y reflujo gastroesofágico. Esta
alteración en la relajación receptiva y en el vaciamiento gástrico lo más probable es que se
deba a una alteración en: el nervio vago

45. Lactante masculino de 5 meses de edad producto de un primer embarazo normal


controlado . Peso de nacimiento 3.120 g y talla de 51 cm . Lactancia materna satisfactoria,
con buen incremento ponderal . Sin antecedentes patológicos hasta 5 días atrás , cuando
comienza con vómitos posprandiales no biliosos , lácteos . Los síntomas aumentan en
frecuencia y magnitud hasta hacerse explosivos después de cada alimentación . No requiere
tos , fiebre , diarrea ni lesiones cutáneas .Es importante destacar que pese a los vómitos el
niño conserva el apetito y llora de hambre .Al examen físico presenta buen estado general.
Abdomen blando ,depresible e indoloro ,asociado a distensión de hemiabdomen superior
.Sin signos de deshidratación .No se palpan masas abdominales .Exámenes de
laboratorio:hemograma normal. Signos inflamatorios de fase aguda negativos .Alcalosis
metabólica leve en sangre venosa.

Producto de la hipertrofia pilórica uno esperaría que los niveles de gastrina se encuentren: .

Rpta}: ELEVADA

46. Lactante masculino de 5 meses de edad producto de un primer embarazo normal


controlado . Peso de nacimiento 3.120 g y talla de 51 cm . Lactancia materna satisfactoria,
con buen incremento ponderal . Sin antecedentes patológicos hasta 5 días atrás , cuando
comienza con vómitos posprandiales no biliosos , lácteos . Los síntomas aumentan en
frecuencia y magnitud hasta hacerse explosivos después de cada alimentación . No requiere
tos , fiebre , diarrea ni lesiones cutáneas .Es importante destacar que pese a los vómitos el
niño conserva el apetito y llora de hambre .Al examen físico presenta buen estado general.
Abdomen blando ,depresible e indoloro ,asociado a distensión de hemiabdomen superior
.Sin signos de deshidratación .No se palpan masas abdominales Exámenes de
laboratorio:hemograma normal. Signos inflamatorios de fase aguda negativos .Alcalosis
metabólica leve en sangre venosa.

En este paciente se puede esperar una mayor liberacion de :

Rpta: Enzimas pancreáticas

47. Lactante masculino de 5 meses de edad producto de un primer embarazo normal


controlado . Peso de nacimiento 3.120 g y talla de 51 cm . Lactancia materna satisfactoria,
con buen incremento ponderal . Sin antecedentes patológicos hasta 5 días atrás , cuando
comienza con vómitos posprandiales no biliosos , lácteos . Los síntomas aumentan en
frecuencia y magnitud hasta hacerse explosivos después de cada alimentación . No requiere
tos , fiebre , diarrea ni lesiones cutáneas .Es importante destacar que pese a los vómitos el
niño conserva el apetito y llora de hambre .Al examen físico presenta buen estado general.
Abdomen blando ,depresible e indoloro ,asociado a distensión de hemiabdomen superior
.Sin signos de deshidratación .No se palpan masas abdominales Exámenes de
laboratorio:hemograma normal. Signos inflamatorios de fase aguda negativos .Alcalosis
metabólica leve en sangre venosa.

En relación con la estructura afectada se encuentra:

Rpta: L1

48.Varón de 67 años con tos y disminución de peso asociado a tabaquismo pesado,


presenta actualmente disfagia progresiva a alimentos sólidos. Se considera la presencia de
un carcinoma de bronquio izquierdo y por esta razón le realizan una endoscopia esofágica
para descartar la posibilidad de una compresión esofágica por el tumor. Se espera revisar el
esófago en la _____ estrechez, que está a nivel de la vértebra _____.

Tercera estrechez - T6
49. En un niño menor de dos años con divertículo intestinal, este divertículo tiene su origen
en una falla en la obliteración de:

Conducto vitelino

50. Mujer de 43 años sufre un grave accidente de tránsito y está hospitalizada en coma, es
alimentada por vía intravenosa durante varias semanas. Producto de este tipo de
alimentación, se encuentra en la endoscopia atrofia de la mucosa gastrointestinal. La causa
más probable de esta atrofia son los bajos niveles séricos de la hormona:

Gastrina

51. Una mujer de 30 años llega al consultorio porque se queja de dificultades para deglutir,
la cual se agravan cada vez más. Se realiza un estudio manométrico para examinar la
generación de presión a lo largo del esófago. Esta prueba revela que las contracciones
como respuesta a la deglución están mal sincronizadas y que la presión en el esfínter
esofágico inferior permanece elevada. El diagnóstico más probable es _____ producido por
niveles bajos de _____:

acalasia / óxido nítrico

52. Paciente de 2 años, llega a emergencia por haber ingerido una moneda con la que
estaba jugando. El lugar más probable donde puede haberse quedado suspendido este
objeto es a nivel del estrechamiento producido a nivel del:

músculo cricofaríngeo

53. En una apendicectomía, al realizar la incisión de McBurney en la fosa iliaca derecha, es


necesario cortar los siguientes músculos, de afuera hacia adentro:

Oblicuo externo - Oblicuo interno - Transverso

54.Al consumir caramelos indirectamente activa la via:

Rpta: POMC/CART

55. Debido al uso de ranitidina , los valores de somatostatina en sangre:

Rpta: Disminuyen

56.En múltiples causas de la enfermedad por reflujo gastroesofágico, se puede considerar


tambien una alteración en las ___________ del esfínter esofágico inferior

Rpta: contracciones tonicas

57. Estudiante de medicina de la UPC de 21 años sufre de gastritis aguda ocasionada por
comer en lugares poco higiénicos. Suele consumir caramelos ( chupa ) mientras está en
clase hasta la tarde. Toma gaseosa regularmente (carbohidratos 46%, sodio 53%). También
toma regular cantidad de leche (grasa 35%, lactosa 35%, proteínas 30%), pues le calma un
poco el dolor el ardor que siente por la gastritis. Incluso, cuando puede, se toma dos vasos
de agua fría para calmar las molestias. Ha decidido ir al médico para tratarse pues ya no
soporta el dolor, el cual está seguro que los síntomas se deben a una elevada producción
de ácido clorhídrico en el estómago, y por ello le ha recetado Ranitidina (antihistamínico),
con lo que siente mejoría.

El consumo de una pequeña cantidad de gaseosa aumenta directamente la contracción


sérica de cual de las siguientes hormonas.

Rpta: Peptido 1 similar al glucagon(GLP-1)

58. Paciente es evaluado por faringitis aguda en consultorio externo. El médico de familia le
solicita que abra la boca y saque la lengua Para realizar la acción de sacar la lengua, es
necesario que se contraiga el músculo: geniogloso

59. en qué casos los vómitos son siempre biliosos

Rpta: atresia yeyunal

60. Durante una cirugía oncológica¿La extirpación de cuál de los siguientes órganos se
vería comprometida por la presencia de adventicia.

Rpta: recto

61. Estimulan la secreción ácido gástrica

Rpta: Proteínas

62. Cual de los líquidos corporales tiene el PH más alto

Rpta: Jugo pancreatico

63. En un paciente de 43 años con tumor carcinoide de páncreas productor de gastrina


(Sindrome de Zollinger-Ellison) se puede esperar una disminución en la:

Rpta: liberación de gastrina por la células G

64. El ligamento de Treitz característicamente:

Rpta: Suspende el ángulo de Treitz

65. Paciente de sexo masculino de 43 años que es alimentado por via iintravenosa durante
varias semanas. Producto de este tipo de alimentación se encuentra en la endoscopia
atrofia de la mucosa antral. la causa mas probable de esta alteración es debido a los bajos
niveles séricos de que hormona:

Rpta. GASTRINA

66. Con respecto al control autonómico en el tracto gastrointestinal y en relación con su fisiología
¿Cuál es la función del sistema nervioso parasimpático y el tracto gastrointestinal?

Estimulan la contracción muscular y estimulan la secreción de sustancias a nivel de la mucosa


Intersticio ubicado entre el estroma del espacio portal y los hepatocitos, y por donde migran las
células cancerígenas que hacen diseminación linfática es el:

Espacio del Mall

La presencia de grandes cantidades de TGF - Beta estimula a las ___________ y se deposita


colágeno, formándose la cirrosis

Células de Ito

Paciente de 64 años con ICC al que se le va a realizar cirugía cardiovascular. Al calcular el volumen
sanguíneo total, se debe considerar que el hígado puede contener un volumen de sangre de
____________ mL en un adulto sano, en este paciente ese volumen puede llegar a ser de
_____________ mL

450 - 1000

El área del lobulillo que se afecta más en caso de hipoxia es la zona:

Paciente con carcinoma de vesícula biliar. La metástasis por continuidad afectará al lóbulo:

Cuadrado

El ácido acetilsalicílico actúa en la membrana:

Basolateral de la célula parietal

Al realizar una esplenectomía, se tiene que resecar la arteria esplénica, lo cual no es problema para
el estómago por que la arteria gastroomental izquierda se anastomosa con la:

Gastroomental derecha

La arteria esplénica proviene de la aorta y la vena esplénica desemboca en la vena:

Porta

Paciente mujer 21 con bulimia, que luego de un episodio de vómitos presenta hematemesis y al
examen físico que se encuentra crépitos subcutáneos cervicales. El diagnóstico más probable es:

Sd. Boerhaave
Paciente varón de 60 años, con antecedentes de promiscuidad sexual, tabaquismo y alcoholismo,
acude a consulta por presentar disfagia progresiva, odinofagia y al examen se observa tumor por
parte posterior de la lengua. La mejor posibilidad diagnóstica es:

Carcinoma escamoso

Niño de 5 años con historia de tres días de evolución caracterizado por fiebre, malestar general,
odinofagia, anorexia, e irritabilidad. Al examen de observa lesiones ulcerativas de 4mm de diámetro
en mucosa yugal, con borde blanquecino y eritema periférico. El diagnóstico más probable es:

Aftas orales

Es considerada una lesión preneoplásica

Leucoplasia

El esófago de Barrett se considera una lesión preneoplásica que se caracteriza por la presencia en
esófago de:

Metaplasia intestinal

Paciente mujer de 23 años gestante con lesión proliferativa en mucosa oral producida por
proliferación reactiva de vasos sanguíneos. Marque la mejor respuesta

Epulis

La glándula parótida tiene principalmente acinos de tipo:

Seroso

La reabsorción de sodio y secreción de potasio es estimulada por:

Aldosterona

Paciente con cirrosis hepática que tiene hipertensión portal con várices esofágica, y actualmente
presenta varices en estómago distal. Estas várices están relacionadas a aumento en la presión de las
venas:

Gástrica derecha
La glándula parótida está inervada por el par craneal:

IX

Paciente con cirrosis hepática que tiene hipertensión portal con varices en esofago distal . Estas
varices están relacionadas a aumento en la presión de las venas gástricas

Inferior

La información eferente que sale de los núcleos salivales superior e inferior a través de los pares VII y
IX hacia las glándulas salivales llevan información tipo

Parasimpático

Un niño de 4 años ingresa en el hospital con vómitos graves . En el estudio se encuentra que el niño
tiene un páncreas anular ¿ Cuál de las siguientes hormonas gastrointestinales se encontrará a
niveles elevados en sangre con mayor probabilidad a raíz de esta patología

Gastrina

Durante una colecistectomía laparoscópica en un hombre de 61 años ¿ Cuál de las siguientes


arterias debe pinzar para extirpar la vesícula biliar con seguridad?

Cistica

Un hombre de 34 años se somete a una apendicectomía de urgencia . Después de realizar la


apendicectomía satisfactoriamente , el paciente se somete a una laparoscopia exploratoria¿Cual de
las siguientes características anatómicas es más útil para distinguir entre yeyuno e íleon?

El yeyuno tiene menos grasa mesentérica que el íleon

Una mujer de 45 años ingresa en el hospital con síntomas de obstrucción intestinal superior .En la TC
se encuentra que la tercera porción (transversa) del duodeno está comprimida por un gran vaso
¿Cuál de los siguientes vasos causara muy probablemente la obstrucción ?

Arteria mesentérica superior


Durante una colecistectomía laparoscópica programada en una mujer de 47 años , el residente
pinchó accidentalmente el ligamento hepatoduodenal en vez de la arteria cística ¿Cual de los
siguientes vasos estaría muy probablemente ocluido en esta lesión iatrogénica?

Arteria hepática izquierda

Un hombre de 54 años ingresa en urgencias con intenso dolor abdominal superior . La gastroscopia
revela un tumor en el antro del estómago . Se pide una TC para evaluar el drenaje linfático del
estómago¿Cuál de los siguientes nódulos linfáticos estará muy probablemente afectado en una
neoplasia maligna del estómago

Mesentérico inferior

Una mujer obesa de 45 años con fiebre alta acude a la consulta con náuseas y dolor agudo e
intermitente en el cuadrante superior derecho del abdomen de 2 días de duración - Tiene una historia
de ictericia de 24 horas. Tiene antecedentes de litiasis biliar. Bilirrubina total del 10 mg/dL. Lipasa de
5 mg/mL. ¿Cuál de las siguientes estructuras está muy probablemente obstruida por un cálculo biliar?

Ampolla de Vater

Una mujer de 45 años ingresa en urgencias con dolor abdominal intenso . La TC y RM revelan un
tumor de la cabeza del páncreas que afecta el proceso unciforme ¿Cual de los siguientes vasos es
más probable que suministre irrigación a parte de la zona afectada?

Arteria cólica media

Un hombre de 70 años ingresa en urgencias con diarrea intensa . La arteriografía revela un bloqueo
del 90% en el origen aórtico de la arteria mesentérica inferior ¿Cuál de las siguientes arterias
proporciona muy probablemente irrigación colateral al colon descendente?

Arteria sigmoidea

Al disminuir el pH duodenal por el HCL gástrico , se libera principalmente una hormona cuya célula
diana es

Célula ductal de Wirsung

Una de las siguientes sustancias reguladoras , puede actuar de forma paracrina y como hormona

Somatostatina
Al ingerir grandes cantidades de dulces , con la subsecuente estimulación de incretinas , usted
esperaría que el apetito

Disminuya por insulina

Paciente obeso con Covid-19 es intubado por interno inexperto, quien al solicitar que bombeen aire
dentro del tubo endotraqueal, nota que el epigastrio se distiende. Al sospechar que ha introducido el
tubo en el estómago, también es cierto que:

Disminuye el pH gástrico

La hormona que tiene un efecto sinérgico con la secretina para optimizar el pH duodenal y la
digestión, es:

CCK

El ecografista sabe que para poder visualizar el nacimiento de la arteria mesentérica superior , debe
colocar el transductor sobre la piel de la siguiente región abdominal

Epigastrio

En un paciente con hiperestimulación simpática se espera que las ondas lentas tengan un ritmo

Mayor en estómago que en el duodeno

Mayor en estómago que en el íleon terminal

Menor en íleon terminal que en el duodeno

Mayor en el íleon que en duodeno

Se considera que el gusto puede viajar a través del nervio

Glosofaríngeo

La rotación en sentido longitudinal del estómago en el desarrollo embriológico condiciona que el


nervio vago derecho quede a nivel

Posterior

Con respecto al control autonómico en el tracto gastrointestinal y en relación con su fisiología. ¿Cuál
es la función del sistema nervioso parasimpático en el tracto gastrointestinal?
Estimulan la contracción muscular y estimulan la secreción de sustancias a nivel de la mucosa.

Un estudiante que está preocupado por su examen parcial, no ha desayunado ni almorzado; cuando
al fin ingiere alimentos, este le provoca el aumento de los movimientos musculares del tracto
gastrointestinal y la sensación de defecar.?¿Qué reflejo se ha activado?

gastro - cólico

¿De que par craneal es rama el nervio palatino mayor?

Trigémino

¿En cuál de las fases de deglución la epiglotis separa la vía respiratoria de la digestiva?

Faríngea

Los péptidos intestinales se pueden clasificar como sustancias endocrinas, neurocrinas y paracrinas,
dentro de las paracrinas se encuentran la somatostatina e histamina. Marque la respuesta correcta

La histamina es sintetizada por células de tipo paracrino de las glándulas gástricas

Paciente varón de 27 años es llevado por bomberos a emergencia luego de ser asaltado y, tras
resistirse, es cortado con el pico de una botella a nivel abdominal. Al examen físico usted observa
que a través de la herida se puede observar la protrusión de las asas intestinales. En relación con las
capas de la pared abdominal, marque la alternativa correcta.

El músculo recto del abdomen tiene como funciones comprimir el contenido del abdomen, tensar la
pared abdominal y flexionar la columna

Los músculos del tracto gastrointestinal de los segmentos propulsivo y receptor del bolo alimenticio,
responden de forma diferente al movimiento de este bolo a través del intestino. ¿Cuál de las
siguientes afirmaciones describe correctamente la actividad del segmento propulsivo?

Tanto el músculo circular como el longitudinal están relajados

El músculo longitudinal está relajado y el circular esta contraído

Tanto el músculo circular como el longitudinal están contraídos

El músculo longitudinal está contraído y el circular está relajado

El nervio vago inerva al músculo

Estriado del esófago


¿Cuál de las siguientes es una característica de los ganglios mioentéricos del sistema nervioso
entérico?

Contiene mayor número de neuronas que el plexo submucoso

Es también conocido como el plexo de Meissner

Contiene sólo neuronas motoras excitatorias del músculo liso

Contiene neuronas sensitivas que activan a los músculos circular y longitudinal del tracto intestinal

El divertículo faringoesofágico, hipofaríngeo de zenker, es una lesión muy particular que se localiza
en la cara póstero lateral de la Unión de la faringe con el esófago, como una herniación de la mucosa
esofágica a través de las fibras oblicuas del músculo.

Constrictor inferior de la faringe

La razón por la que el potencial de acción viaja rápidamente en sentido longitudinal por el músculo
liso gastrointestinal es la presencia de uniones en hendidura,

Varicosidades

Respecto a los péptidos gastrointestinales, marque lo correcto.

las sustancias paracrinas pueden viajar a través de vasos sanguíneos

Al ingerir rápidamente un litro de agua, usted esperaría que la gastrina aumenta por efecto de:

ACh del sistema mientérico

Respecto a la anatomia del estomago,marque lo correcto:

El fondo gástrico forma la curvatura mayor

(Pág 19 → 21)

Los nervios esplácnicos lumbares (L1-L2) llevan información de tipo:

Simpática
Paciente de 32 años con herida por arma de fuego y shock hipovolémico. El intestino delgado no se
ha infartado aún a pesar de la hipoxia gracias a la liberación de:

Adenosina

Al realizarse un piercing en el ombligo,la sensación de dolor se transmite por:

T10

El nivel en el que se encuentra el píloro y el páncreas se puede determinar usando el:

Píloro transpilórico

Permite la suspensión e irrigación de los órganos peritoneales:

Mesenterio

El dolor asociado a apendicitis clásicamente se ubica en:

Fosa iliaca derecha

El ligamento inguinales formado por la aponeurosis del:

Oblicuo externo

Los nueve cuadrantes del abdomen se delinean usando el plano subcostal,las líneas
medioclaviculares y:

Plano intertubercular

Es un órgano peritoneal:

Hígado

La línea alba se encuentra:

Entre los rectos abdominales

Es un órgano retroperitoneal:

Páncreas
Enfermedad asociada con un error en el desarrollo de las células de Cajal

Enfermedad de Hirschsprung

El divertículo de Meckel es un rezago de:

Conducto vitelino

La fístula rectoperitoneal es causada por una falta en el desarrollo de:

Tabique urorrectal

95) Al disminuir el pH duodenal por el HCl gástrico, se libera principalmente una hormona cuya célula
diana es:

Célula ductal del Wirsung

Al introducir una solución azucarada directamente al estómago mediante una gastrostomía


(comunicación entre la piel abdominal y el estómago), la sustancia que provocará que aumenten los
niveles séricos de insulina es:

Péptido tipo glucagón 1 (GLP-1)

La rotación en sentido longitudinal del estómago en el desarrollo embriológico condiciona que el


nervio vago derecho quede a nivel:

Posterior

Una de las siguientes sustancias no comparte con las otras la misma acción sobre la producción de
ácido gástrico:

Colecistoquinina

En un paciente con gastroparesia (motilidad lenta del estómago), que presenta distensión abdominal
después de comer, usted le recomendaría que evite el consumo de lípidos y aminoácidos para
disminuir la acción de:

CCK
En un paciente con shock distributivo, usted decide iniciar noradrenalina por un catéter CVC,
consiguiendo aumentar la presión arterial. ¿qué efecto sobre la motilidad intestinal esperaría
encontrar?

El potencial de reposo de las fibras musculares se hace más negativo

El ecografista sabe que para poder visualizar el nacimiento de la arteria mesentérica superior, debe
colocar el transductor sobre la piel de la siguiente región abdominal:

Epigastrio

105) La glándula submandibular recibe inervación traída por el nervio

Cuerda del tímpano

A mayor flujo de saliva, disminuye la concentración de:

Potasio

107) En términos de mg/mL, el principal componente de la saliva es _____ seguido de ____:

Proteínas - Potasio

108) La reabsorción de Sodio y Cloro en las glándulas salivales se da principalmente en el:

Conducto estriado

109) La información eferente que sale de los núcleos salivales superior e inferior a través de los pares
VII y IX hacia las glándulas salivales llevan información de tipo:

Parasimpático

La glándula submaxilar le hace gancho al:

Músculo milohioideo

Los conductos salivales son ___ al agua, esa es una de las razones por las cuales la saliva es
siempre ___.

Impermeables - hipotónica
Los nervios esplácnicos pélvicos (S2-S4) llevan información de tipo:

Parasimpática

113) Presenta movimientos en masa:

Colon

114) La digestión de los lípidos se inicia en:

Intestino delgado

115) La digestión de los carbohidratos se inicia en:

Cavidad oral

117) El estómago recibe información simpática proveniente del:

Ganglio celiaco

118) En el intestino delgado se absorbe los carbohidratos en forma de:

Fructosa

119) Es rama de la arteria mesentérica superior:

A. cólica media

120) El “dolor de estómago” asociado a gastritis se suele ubicar en:

Epigastrio

121) El azúcar de mesa sacarosa es digerido a dos monosacáridos que comparten el transportador:

GLUT 2

122) Durante la defecación se requiere:

Señales inhibitorias en el nervio pudendo


123) En un paciente con fístulas intestinales y fisuras anales, con antecedente de enfermedad
inflamatoria intestinal, ud sospecharia en:

Enfermedad de Crohn

En los pacientes con Colecistitis Aguda no operable, una opción es la colocación de una sonda por el
cístico, procedimiento en el que se ingresa con dificultad debido a la estrechez del cístico y a la
presencia de:

Válvula de Herring

Paciente de 51 años con antecedente de enfermedad diverticular acude a emergencia por sangrado
profuso y dolor en hipocondrio izquierdo. ¿Cuál es el origen más probable de la sangre que pierde el
paciente ?

Mesentérica inferior

Paciente de 42 años con dolor abdominal intenso y hematemesis. En la endoscopia se observa una
úlcera duodenal posterior perforada con hemorragia intraabdominal. ¿Cuál de las siguientes arterias
estará comprometida?

Pancreaticoduodenal posterosuperior

Para que los triglicéridos sean absorbidos deben ser metabolizados a

Monoglicéridos y Ácidos grasos

El vibrio cholerae produce diarrea porque:

Aumenta la producción de AMPc en los enterocitos

La motilidad intestinal es estimulada por

Colecistoquinina y gastrina

La diarrea por deficiencia de lactasa es de tipo

osmótica
La metoclopramida estimula el vaciamiento gástrico aumentando la fuerza de contracción de las
paredes gástricas esto puede conseguirlo mediante la estimulación indirecta de las neuronas
liberadoras de

Acetilcolina

Los vértices de un acino hepático están constituidos por

Dos espacios porta y dos venas centrolobulillares

Al ingerir un pedazo de mantequilla, cuál de los siguientes tiene un efecto directo en la reducción del
vaciado gástrico:

Colecistoquinina

Tras la vagotomía (resercion del vago) por enfermedad úlcera péptica en un paciente UD. esperaría
encontrar:

Aumento del pH gástrico

Se realizó un experimento en el cual se inyectó tinta china en el peritoneo de ratas de laboratorio. Al


realizarse una biopsia hepatica de dichos animales, se encontro que el tinte negro fue fagocitado por:

Células de kupffer

En un paciente con pH gástrico muy bajo, es posible que la siguiente sustancia se secrete en menor
cantidad:

Gastrina

La fase cefálica de la secreción gástrica responde por cerca del 30% de la respuesta ácida a un
reflejo con la _____________ se elimina la fase cefálica de la secreción gástrica

Vaguectomía

El esofago de barret se caracteriza por presentar _________ en el esofago

Metaplasia intestinal
La presencia de orina que sale por el ombligo de un recien nacido cada vez que llora, es posible que
deba a un defecto en el desarrollo de:

Seno urogenital

El conducto biliar deriva del:

Endodermo

Cuál de los siguientes órganos son intraperitoneales:

Estómago, Vesícula biliar, Ileón, Hígado

Los vasos mesentéricos superiores se hallan a nivel de:

Cuello del páncreas

La colecistoquinina (CCK) inhibe:

El vaciamiento gástrico

Un niño de 2 años es llevado a la consulta por diarrea persistente, edema de las extremidades y falta
de crecimiento en relación a su edad. Los análisis de sangre revelan que tiene concentración
plasmática baja de proteínas (hipoproteinemia). Como parte del estudio se coloca Colecistoquinina
(CCK) endovenosa y se recoge muestras del líquido duodenal por endoscopia; el resultado del líquido
confirma incapacidad para hidrolizar proteínas a un pH neutro, esta situación mejora al añadir una
pequeña cantidad de tripsina. El paciente probablemente esté sufriendo la falta congénita de ……….

Enterocinasa

Paciente mujer de 35 años acude a consulta por sensación de sequedad y lesiones en la cavidad
oral. Al examen se observa atrofia de la mucosa, fisura y úlceras; nota además sequedad e irritación
de la córnea y aumento del tamaño de las glándulas parótidas. Su diagnostico mas probable es
artritis reumatoide; el hallazgo más probable en una biopsia de glándula parótida es ….…

Gran infiltración de linfocitos y células plasmáticas

Un hombre de 42 años de edad se presenta al médico con una historia clínica de 1 año de evolución,
caracterizado por dolor abdominal bajo y diarreas con crisis sanguinolentas. Manifiesta además
pérdida de peso de 8kg durante este periodo. La colonoscopia revela lesión difusa en el colon con
afectación del recto. La biopsia de estas lesiones revela adelgazamiento de la pared, inflamación y
ulceración de la mucosa y submucosa. El diagnóstico más probable en este caso es:
Colitis ulcerativa

Dos estudiantes deciden tomar un receso para comer una hamburguesa a la hora del almuerzo.
Antes de llegar a la cafetería, impulsos nerviosos provenientes del complejo vagal dorsal iniciarán la
secreción de ácido gástrico por la liberación de ___________ desde el sistema nervioso entérico.

GRP (péptido liberador de gastrina)

Un niño de cuatro años de edad es llevado a la consulta por cuadros diarreicos frecuentes
caracterizados por heces pálidas, voluminosas y fétidas; al examen físico presenta bajo peso y talla
para la edad. Se mide la concentración de cloruro de en el sudor y se encuentra que sus valores son
muy elevados. La alteración más importante a nivel de células ductales del páncreas tiene relación
directa con la conductancia de …..

Cloro
En el club del adulto mayor, a un grupo de ancianos de 65 años se les da a elegir entre un taller de
videojuegos y uno de lectura. La mayoría de ellos optó por el taller de lectura. ¿Cuál sería una
explicación para esta conducta?

La atención sostenida permanece relativamente estable a esa edad

Pablo es invitado a dar el discurso de bienvenida a los nuevos residentes del hospital. Muestra
habilidad en el discurso, planifica las ideas que quiere decir, mantiene el tema y objetivo de la
conversación. ¿Qué estructura cerebral estaría involucrada para demostrar esas características?

CPF dorsolateral

Para poder diagnosticar a un paciente de discapacidad intelectual se requiere:

Que las deficiencias adaptativas limiten el funcionamiento en una o varias actividades cotidianas

En relación a la dopamina podemos afirmar:

Esta presente dentro de la neurona presináptica

La mielinización neuronal para el desarrollo visual, motor, social y cognitivo se da en la dirección:

Cefalocaudal
En un paciente con hiperestimulación simpática se espera que las ondas lentas tengan un ritmo:

Menor en íleon terminal que en el duodeno

Al ingerir grandes cantidades de dulces, con la subsecuente estimulación de incretinas, usted


esperaría que el apetito ______________, debido a __________________

disminuya insulina

Es un ligamento derivado del mesenterio dorsal:

Gastrocólico

Una de las siguientes sustancias no comparte con las otras la misma acción sobre la producción de
ácido gástrico:

Colecistoquinina

Usted encuentra músculo estriado en el siguiente segmento:

Esfínter anal externo

Todos los músculos motores de la lengua están inervados por el XII par, excepto:

palatogloso

En un paciente con gastroparesia (motilidad lenta del estómago), que presenta distensión abdominal
después de comer, usted le recomendaría que evite el consumo de lípidos y aminoácidos para
disminuir la acción de:

CCK

En un paciente con diarrea por hipermotilidad, usted sospecharía en el posible aumento de las
siguientes sustancias, excepto:

Péptido intestinal vasoactivo

Marque lo correcto respecto a la siguiente imagen:

Se produjo por falta de fusión de los ductos dorsal y ventral

Respecto a los péptidos gastrointestinales, marque lo correcto.

las sustancias paracrinas pueden viajar a través de vasos sanguíneos


Se evalúa los valores séricos de las siguientes sustancias a un paciente con enfermedad hepática
terminal; en este paciente se espera encontrar la combinación de la letra …..

Glucosa

Amoniaco

Albúmina

a.

Aumentada

Disminuida

Disminuida

b.

Disminuida

Aumentada

Aumentada

c.

Aumentada

Aumentada

Aumentada

d.

Disminuida

Aumentada

Disminuida

Una mujer de 35 años de edad HIV positiva, se presenta al médico con dolor abdominal en cuadrante
superior derecho e ictericia. La paciente refiere haber tenido múltiples episodios de ictericia durante
los últimos 10 años. Los exámenes para determinar hepatitis viral, dieron positivos para Hepatitis B,
siendo catalogado el caso como hepatitis crónica con alteración funcional. En un examen de sangre
¿Cuál de los siguientes parámetros está disminuido?

Albúmina
En el reflejo peristáltico del intestino delgado ¿Cuál de los siguientes eventos sucede en la porción
caudal del bolo alimenticio?

Acción del péptido inhibidor vasoactivo (VIP) en el músculo circular

Un varón de 58 años de edad con enfermedad de Crohn severo fue sometido a una resección ileal.
Después de la cirugía esta paciente padecerá de esteatorrea esto se explica porque ……

Hay malabsorción de ácidos biliares

En un experimento se inserta un balón en el estómago de un voluntario, se infla poco a poco mientras


que se vigilan las presiones intraluminales. Aunque el volumen del balón aumenta
considerablemente, las presiones permanecen constantes. Esta relación volumen-presión se explica
por la liberación local de ….

Óxido nítrico y péptido inhibidor vasoactivo

¿Cuál de las siguientes alternativas es una característica de la secreción exocrina del páncreas?

Su mayor estímulo se da en la fase intestinal

Las estructuras en el hígado que permite que los productos metabólicos unidos a proteínas tengan
acceso a las membranas basolaterales de los hepatocitos, son ….

Las fenestras sinusoidales

La composición de la bilis es modificada conforme fluye por los conductillos biliares. Durante este
tránsito se espera que aumente la concentración de …..

Ig A

Se mide experimentalmente el contenido gástrico de dos personas. La persona “A” tiene alto
contenido de grasa y la persona “B” tiene un contenido isotónico. ¿Cual de las siguientes es correcta
respecto al vaciamiento gástrico?

Hay ralentización del vaciado gástrico sólo en “A”


El examen endoscópico de un paciente con hipertensión portal grave revela venas tortuosas que
sobresalen hacia la luz del esofago. El paciente recibe tratamiento quirúrgico mediante la colocación
de una derivación que conecta la vena cava. Después de la operación el riesgo de encefalopatía
…….. y el sangrado de varices ……

Aumentará/Disminuirá

Un bolo alimenticio grande y poco masticado se atasca en el esofago, esto ocasiona una sensacion
de dolor que es transmitida por los nervios:

Esplácnico

El peristaltismo del intestino delgado se puede intensificar debido a:

Irritación de la mucosa

Un paciente es diagnosticado con un tumor neuroendocrino productor de somatostatina, esto


provocará en el sistema digestivo:

Diarrea

Los diferentes segmentos del tubo digestivo son susceptibles de reflejos y movimientos según su
contenido.Si colocoramos mediante una sonda un bolo alimenticio directamente en el tercio medio del
esofago:

Se producirá ondas secundarias

En el digestivo la liberación hormonal se presenta ante diversos factores o estímulos. La hormona


_________ es estimulada por la presencia de alimentos en el bulbo duodenal a predominio de ácidos
grasos y triglicéridos, por estimulación vagal y por la hormona secretina.

Colecistoquinina (CCK)

El divertículo de Meckel es una anomalía congénita que ocurre por la persistencia del conducto
vitelino y da origen a una estructura sacular, el cual se encuentra en el:

Borde antimesentérico

La fase oclusal de la masticación se realiza con la contracción de los músculos inervados por el
nervio craneal:

V
En un paciente de 43 años con tumor carcinoide de páncreas productor de gastrina (Sindrome de
Zollinger-Ellison) se puede encontrar una potenciación del reflejo:

Gastrocolico

Durante la fase faríngea de la deglución ocurre el siguiente mecanismo:

La onda peristáltica lleva el alimento hacia el esofago

En los carcinomas (neoplasia benigna) es frecuente que ocurran la metástasis a través de los vasos
venosos. En el caso de un carcinoma del tercio superior del esofago, ubicado en la cara lateral
izquierda, es más probable que la metástasis viaje por la vena:

Hemiácigos accesoria

168) Los movimientos en masa son iniciados por el reflejo:

Duodenocolico

Una mujer de 65 años HIV positiva se presenta con dolor abdominal en el cuadrante superior derecho
e ictericia. La paciente afirma haber teñido múltiples episodios de ictericia durante los últimos 10
años. Los exámenes para poder detectar hepatitis viral, dieron positivos para Hepatitis B, siendo
catalogada como hepatitis crónica con alteración funcional. En un examen de sangre ¿cuál de los
siguientes parámetros está disminuido?

Albúmina
En el reflejo peristáltico del intestino delgado¿cuál de los siguientes eventos sucede en la porción
caudal del bolo alimenticio?

Acción del péptido inhibidor vasoactivo(VIP) en el músculo circular

Un varón de 58 años de edad con enfermedad de Crohn Severo fue sometido a una resección ilegal.
Después de la cirugía este paciente padecerá de esteatorrea, esto se explica porque ….

Hay mala absorción de los ácidos biliares

En un experimento se inserta un balón en el estómago de un voluntario, se infla poco a poco mientras


se vigilan las presiones intraluminales. Aunque el volumen del balón aumenta considerablemente, las
presiones permanecen constantes. Esta relación volumen presión se explica por la liberación local de
….

Óxido nítrico y péptido inhibidor vasoactivo

¿Cuál de las siguientes alternativas es una característica de la secreción exocrina del páncreas?

Su mayor estímulo se da en la fase intestinal

Las estructuras en el hígado que permiten que los productos metabólicos unidos a proteínas tengan
acceso a membranas basolaterales de los hepatocitos son ….

Las de fenestras sinusoidales

La composición de la bilis es modificada conforme fluye por los conductillos biliares. Durante este
tránsito se espera que aumente la concentración de ….

IgA

Se mide experimentalmente el contenido gástrico de dos personas. La persona A tiene alto contenido
de grasas y la persona B tiene un contenido isotónico ¿Cuál de las siguientes es correcta respecto al
vaciamiento gástrico?

Hay ralentización del vaciado gástrico sólo en A

El examen endoscópico de un paciente con hipertensión portal grave revela venas tortuosas que
sobresalen hacia la luz del estómago. El paciente recibe tratamiento quirúrgico mediante la
colocación de una derivación que conecta la vena porta a la vena cava. Después de la operación el
riesgo de encefalopatía …. y el riesgo de sangrado de varices ….
Aumentará / disminuirá

Un paciente varón de 18 años de edad acude al médico para sus exámenes de rutina. Sus resultados
de laboratorio muestran un valor de bilirrubina sérica de 4mg/dl y una bilirrubina directa de 0.3 mg/dl.
Las pruebas de función hepática son normales. La alteración que explica mejor este caso es por la
deficiencia de….

Glucuronil transferasa

Un hombre de 57 años de edad es llevado a urgencias con hematemesis masiva rojo brillante, a su
llegada se halla inconsciente con PA: 80/40 mm Hg y FC:124 lat/min. Luce ictérico con presencia de
arañas vasculares en el tórax anterior y extremidades, abdomen distendido con signo de oleada
positiva. Se encuentra esplenomegalia y pérdida de la masa muscular en extremidades. La
anastomosis vascular responsable del sangrado en este paciente es ….

Vena gastrica izquierda y vena acigos

Un estudiante de medicina está comiendo un plato de comida a base de champiñones, espárragos y


salsa de soya. El estímulo del sabor umami contenido en todos estos alimentos viaja a través del
nervio….

Cuerda del tímpano

Una paciente de 30 años de edad es sometida a una cirugía de oído medio derecho por un problema
de otoesclerosis. Luego de la cirugía refiere alteración sensitiva de la lengua. Al evaluar el caso usted
esperaría encontrar ….

Sensación del dolor, tacto y temperatura conservadas

En una paciente de 45 años de edad con colestasis biliar, se encuentra una elevación de los niveles
sanguíneos de fosfatasa alcalina hasta 3 veces la cifra normal ¿Cuál de las siguientes alternativas
estará también elevada como evidencia del daño biliar?

Gamma glutamil transpeptidasas

Experimentalmente se incrementa la velocidad de la secreción salival con una sustancia, en el


análisis de la composición de esta saliva obtenida se espera encontrar….

Aumento de la concentración de bicarbonato que supera la concentración plasmática


Lactante de 3 meses de vida es atendido por presentar diarrea, se administra una solución glucosa y
electrolitos por vía oral. La proteína de membrana apical que explica la capacidad de esta solución
para proporcionar aporte de glucosa e hidratación es ….

SGLT 1

Paciente ha sufrido herida de bala en el abdomen, se le tenido que extirpar el segmento medio y
distal del íleon. En este caso la síntesis hepática de sales biliares estará ….

Incrementada por estímulo de la enzima colesterol 7 alfa hidroxilasa

Un varón de 75 años ingresa al consultorio por presentar ictericia marcada de piel y escleras. El
estudio del paciente mostró que presentaba un tumor que obstruía la totalidad del conducto hepático
común. ¿Cual de los siguientes conductos se encontraría dilatado en este paciente?

De Hering

Correlacione las dos columnas y marque la respuesta correcta:

Enfermedad Hirschsprung Aganglionosis congénita

Diarrea osmótica. Intolerancia a la lactosa

Diarrea secretora. Canales de Cl- en las células de la cripta

Diarrea exudativa Heces con moco y sangre

La fase cefálica de la secreción gástrica responde por cerca del 39% de la respuesta ácida a un
reflejo. Con la ____ se elimina la fase cefálica de la secreción gástrica

Vaguectomía

El esofago de Barret se caracteriza por presentar___ en el esofago

Metaplasia intestinal

En la regulación del apetito y la saciedad, la estimulación experimental cronica del núcleo ventro
medial del hipotálamo producirá:

Afagia
Paciente mujer de 25 años acude por dolor en fosa iliaca derecha que empeora al toser o
caminar,asociada a náuseas y vómitos por lo cual acude a emergencia.Dos días después de
realizarle una apendicectomia, la paciente desarrolla fiebre alta (39° C), esta hipotensa y presenta
dolor abdominal.La laparotomia exploratoria muestra un gran volumen de sangre en la cavidad
peritoneal por lesion de un vaso producida durante la apendicectomía.¿Cual de las siguientes
arterias debe ligarse para detener la hemorragia?

Ileocolica

¿Cuál de los siguientes es una causa de ictericia con bilirrubina conjugada aumentada?

Obstrucción del colédoco

Dos días después de una apendicectomía en un hombre de 45 años, ha desarrollado fiebre alta (39),
está hipotenso y presenta dolor abdominal. La laparotomía exploratoria muestra un gran volumen de
sangre en la cavidad peritoneal por lesion de un vaso producida durante la apendectomía. ¿Cuál de
los siguientes vasos debe ligarse para detener la hemorragia?

Arteria ileocólica

Paciente de 78 años, con diabetes mellitus tipo ll y fumador, que acude a consulta porque desde hace
dos semanas tiene un dolor intenso en flanco derecho y mesogastrio , intenso que aparece a los 30
minutos de haber comido, y desaparece dos a tres horas después. En estos pacientes, es muy
probable que la circulación deficitaria sea parcialmente asumida por la:

Arteria cólica media

Un hombre de 70 años ingresa en urgencias con diarrea intensa. La arteriografía revela un blloqueo
del 90% en el origen aórtico de la arteria mesentérica inferior. ¿Cuál de las siguientes arterias
proporciona muy probablemente irrigación colateral al colon descendente?

Arteria cólica derecha

Un niño de 4 años ingresa en el hospital con vómitos graves. La exploración radiológica y la historias
clinica revelan que el niño tiene páncreas anular. ¿Cuál de las siguientes estructuras es la que se
encontrará dilatada con mayor probabilidad a raíz de esta patología?

Primera porción del duodeno

Un hombre de 55 años ingresó al hospital con dolor abdominal intenso. La gastroscopia y la TC


revelaron una úlcera perforada en la pared posterior del estómago. ¿Dónde se desarrolla
inicialmente con más probabilidad una peritonitis?

Bolsa omental (saco menor)


Un hombre de 44 años ingresa en urgencias con vómitos abundantes y deshidratación. Las imágenes
radiológicas demuestran que parte del intestino está comprimido entre la aorta abdominal y la arteria
mesentérica superior. ¿Cuál de las siguientes estructuras intestinales estará muy probablemente
comprimida?

Tercera porción del duodeno

¿Cuál de los siguientes nódulos linfáticos estará muy probablemente afectado en una neoplasia
maligna del recto?

Mesentérica inferior

Una mujer de 23 años ingresa con dolor abdominal, náuseas y vómitos. La historia Clínica muestra
que el dolor es agudo y ha sido constante durante 4 años. El dolor empezó en el epigastrio e irradió
bilateralmente alrededor del tórax hasta justo debajo de las cápsulas. Actualmente el dolor se localiza
en el hipocondrio derecho. L a TC revela cálculos calcificados en la vesícula biliar. ¿Cuál de los
siguientes nervios llevan las fibras aferentes del dolor referido?

Nervios esplácnicos torácicos mayores

Un hombre de 55 años ingresa en urgencias por severa perdida de peso en los 6 meses previos. El
examen radiológico pruebas aportan signos de un tumor que causa hipertensión portal. Los estudios
de laboratorio revelan que las deposiciones son grasas, tiene desnutrición e hipoxia hepática. ¿Cuál
de las siguientes localizaciones se encuentra muy probablemente afectada?

Segmento l

El triángulo de calot es importante reconocerlo porque sirve como reparo para encontrar la arteria
cística, y está formado por el conducto cístico, el conducto hepático derecho y :

Borde hepático

El kernícterus se produce en recién nacidos con valores mayores de 25m/dL en la bilirrubina:

Indirecta

Paciente con cirrosis hepática y presión de vena cava de 15mmHg. Lo más probable es que el
paciente presente:

Ascitis
Considerando un flujo plasmático renal de 180 ml/min, y una fracción de filtración del 20%, si la
concentración en sangre de la bilirrubina indirecta es de 0.6mg/dL, entonces es correcto esperar que
la carga filtrada de la bilirrubina indirecta es:

Menor de 1080 mg/dL

El volumen diario de bilis secretada al intestino es:

500-1000 ml

La excreción del amonio se da principalmente por:

Orina

Si en una persona normal, el flujo de la arteria hepática es de 700ml/min, entonces el flujo de la vena
porta debería ser aproximadamente:

2800mL/min

Paciente con Grigler-Najjar debida a mutación del gen UGT1A1, se presenta a consulta por ictericia,
usted asume que si le hiciera un análisis de sangre encontraría valores elevados de:

Bilirrubina indirecta

El amoniaco corporal se forma principalmente en:

Colon

La zona del lobulillo hepático que se afectaría más en una intoxicación con droga hepatotóxica es la
zona:

El acino hepático tiene en sus aristas:

Vena centrolobulillar

Paciente con esteatosis hepática no alcohólica, que en la biopsia se observa degeneración grasa de
los hepatocitos, lo cual se debe a depósitos de lípidos que principalmente contienen:
Triglicéridos

Al evaluar la orofaringe de un paciente, el médico le solicita que abra la boca, saque la lengua y diga
a . Al hacer esta maniobra, nota que el paladar se desvía hacia la derecha, lo cual le hace sospechar
que el paciente sufre de una lesión del nervio craneal:

X contralateral

Un bolo alimenticio grande y poco masticado se atasca en el esófago, esto ocasiona una sensación
de dolor que es transmitida por los nervios:

esplácnicos

Para realizar el movimiento mecánico de abrir la boca, primero se necesita:

fijar el hueso hioides

¿Cuál de las siguientes alternativas se define como la protrusión directa del contenido abdominal a la
cavidad amniótica por un defecto de la pared corporal?

Gastrosquisis

Un paciente requiere que se le coloque una sonda de alimentación directamente al estómago


(gastrostomía), el cirujano deberá hacer una incisión en la piel del abdomen ¿cuál de las siguientes
raíces nerviosas debe ser anestesiada para este procedimiento?

T8

El mecanismo de la defecación incluye la participación de diversas estructuras ¿Cuál de las


siguientes alternativas es correcta?

Puede ser mediado por un reflejo intrínseco

Cuando el contenido del estómago ingresa al duodeno, uno de los reflejos que inhiben el vaciamiento
gástrico es a través del:

sistema nervioso mientérico

Durante la masticación, gran parte del proceso masticatorio se debe a:


el reflejo masticatorio

Las glándulas salivales tienes conductos para la excreción de la saliva; las glándulas ____________
drenan en las carúnculas sublinguales.

sublinguales

En una persona si enfermedad se espera que el tránsito intestinal se vea disminuido cuando se
presenta el reflejo:

doloroso

El divertículo de Meckel es una anomalía congénita que ocurre por la persistencia del conducto
vitelino y da origen a una estructura sacular, el cual se encuentra en el:

borde antimesentérico

Si al intubar a un paciente, por error se ingresa el tubo endotraqueal en el esófago y se insufla el


manguito endotraqueal (globo TET), la dilatación de este manguito generará:

múltiples ondas secundarias

El orificio omental, o hiato de Winslow, se encuentra limitado por el ligamento:

hepatoduodenal

Paciente de 24 años acude a consulta externa por presentar una fístula oronasal (comunicación entre
la cavidad oral y la cavidad nasal). Está fístula es una consecuencia tardía de la lesión de un vaso
sanguíneo por el antecedente de haber sido operado de paladar hendido en los primeros años de
vida, aparentemente en una campaña gratuita de corrección de paladar fisurado. ¿Cuál de las
arterias palatinas podría haberse lesionado durante esa cirugía?

Mayor

Dentro de las anomalías congénitas se puede presentar un tejido pancreático accesorio ¿Cuál es la
ubicación más común de este tejido?

Estómago
Paciente con insuficiencia mitral moderada a severa, con aumento de volumen de la aurícula
izquierda; esta condición tendrá como consecuencia a nivel del sistema digestivo:

la disfagia a sólidos

El inicio de la fase faríngea de la deglución se debe a estímulos sensitivos que viajan por el nervio
craneal:

V (nervio maxilar)

El mesocolon transverso se origina en:

la pared posterior del abdomen

La contracción del músculo ………………………… permite la eliminación de gases (flatos) sin salida
de material fecal ; es el mismo músculo cuya relajación, sobretodo en cuclillas, permite el paso del
contenido fecal con menor esfuerzo durante la defecación

puborrectal

Paciente mujer de 54 años se presenta con náuseas, vómitos, estreñimiento, y es diagnosticada de


abdomen agudo quirúrgico; en la cirugía encuentran un vólvulo de ciego. Esta anomalía puede
explicarse por:

Falta de fusión del mesenterio

Paciente mujer de 23 años con faringitis aguda , toma para el dolor una tableta de paracetamol con
un poco de agua. Durante la deglución, se relaja su esfínter esofágico inferior y el fondo del
estómago, mientras el bolo está aún en el esófago. ¿Qué sustancia provocará con mayor
probabilidad la relajación del esfínter esofágico inferior y el fondo del estómago en esta mujer?

Óxido nítrico

Luego de tres horas dando exámenes, un alumno de medicina comienza a sentir hambre. Esta
situación es probable que sea mediada por la _____ que es sintetizada por el _____

grelina / estómago

Varón de 72 años, con antecedente de diabetes mellitus tipo 2, que presenta entropía diabética
caracterizada por estreñimiento. Este problema puede estar asociado a:

deficiencia de óxido nítrico


Varón de 54 años con diabetes mellitus tipo 2, es diagnosticado de gastroparesia debido a que
presenta sensación de llenura precoz al comer, y reflujo gastroesofágico. Esta alteración en la
relajación receptiva y en el vaciamiento gástrico lo más probable es que se deba a una alteración en:

el nervio vago

Varón de 67 años con tos y disminución de peso asociado a tabaquismo pesado, presenta
actualmente disfagia progresiva a alimentos sólidos. Se considera la presencia de un carcinoma de
bronquio izquierdo y por esta razón le realizan una endoscopia esofágica para descartar la posibilidad
de una compresión esofágica por el tumor. Se espera revisar el esófago en la _____ estrechez, que
está a nivel de la vértebra _____.

Tercera estrechez - T6

En un niño menor de dos años con divertículo intestinal, este divertículo tiene su origen en una falla
en la obliteración de:

Conducto vitelino

Mujer de 43 años sufre un grave accidente de tránsito y está hospitalizada en coma, es alimentada
por vía intravenosa durante varias semanas. Producto de este tipo de alimentación, se encuentra en
la endoscopia atrofia de la mucosa gastrointestinal. La causa más probable de esta atrofia son los
bajos niveles séricos de la hormona:

Gastrina

Una mujer de 30 años llega al consultorio porque se queja de dificultades para deglutir, la cual se
agravan cada vez más. Se realiza un estudio manométrico para examinar la generación de presión a
lo largo del esófago. Esta prueba revela que las contracciones como respuesta a la deglución están
mal sincronizadas y que la presión en el esfínter esofágico inferior permanece elevada. El diagnóstico
más probable es _____ producido por niveles bajos de _____:

acalasia / óxido nítrico

Paciente de 2 años, llega a emergencia por haber ingerido una moneda con la que estaba jugando. El
lugar más probable donde puede haberse quedado suspendido este objeto es a nivel del
estrechamiento producido a nivel del:

músculo cricofaríngeo

En una apendicectomía, al realizar la incisión de McBurney en la fosa iliaca derecha, es necesario


cortar los siguientes músculos, de afuera hacia adentro:

Oblicuo externo - Oblicuo interno - Transverso


Un varón de 90 años que se encuentra postrado en cama, es referido del asilo para endoscopia por
dificultad para deglutir luego de tomar un medicamento para aliviar el dolor de la noche anterior. La
endoscopia revela que la píldora se alojó en el esófago y causó una reacción inflamatoria. Lo más
probable es que esto haya sido por la producción de múltiples ondas:

secundarias

Mujer de 23 años es diagnosticada de bulimia, al examen físico se observa ulceraciones en el


segundo y tercer dedo de la mano derecha. Esto se puede deber al uso continuo de estos dedos para
inducir el vómito, mediante la estimulación del par craneal:

IX

Varón de 52 años se presenta por diarrea persistente de seis semanas de duración. En la


colonoscopia se observa un pólipo a nivel del íleon distal. El patólogo informa que se trata de un
tumor neuroendocrino, probablemente originado por las células enterocromafines del intestino. La
sustancia que más probablemente esté produciendo este tumor es:

Serotonina

La fase oclusal de la masticación se realiza con la contracción de los músculos:

masetero y temporal

Al tomar su café en Starbucks, un estudiante de medicina sufre una quemadura de primer grado en el
tercio anterior de la superficie dorsal de la lengua. La información de dolor es transmitida por el
nervio:

Lingual

Paciente es evaluado por faringitis aguda en consultorio externo. El médico de familia le solicita que
abra la boca y saque la lengua Para realizar la acción de sacar la lengua, es necesario que se
contraiga el músculo:

geniogloso

Paciente con síndrome de Sjogren, presenta “boca seca” (disminución de la producción de saliva) y
caries dental, asociada a la pérdida de la función de tampón de la saliva. Esta desmineralización del
diente puede comprometer a las prolongaciones citoplasmáticas ubicadas en los tubos huecos de la
estructura señalada con la letra:

SHADICK PARCIAL

1. Cuando el istmo de las fauces se cierra, se evita que el alimento pase hacia la orofaringe y
permite respirar mientras se mastica. Este cierre se debe a la contracción y aproximación de
los músculos:
- Palatofaríngeos
- Palatoglosos
- Estiloglosos
- Estilofaríngeos
2. Las siguientes alternativas son ciertas sobre la actividad eléctrica del músculo
gastrointestinal, EXCEPTO:
- La despolarización lenta se debe principalmente al ingreso de Na+.
- Las ondas lentas son más frecuentes en el duodeno.
- Si el potencial de membrana es más positivo, habrá mayor frecuencia de espigas.
- Las ondas lentas no son potenciales de acción.
3. Durante el paso del bolo hacia la orofaringe, se desencadena una serie de contracciones
musculares que estrechan la cavidad faringea. Estas contracciones están mediadas por el
nervio craneal:
- IX
- XII
- XI
- X
4. ¿Cuál de las siguientes alternativas detallan los músculos que ayudan a empujar el bolo
hacia la orofaringe?
- Estilogloso y palatogloso
- Estilogloso y geniogloso
- Hiogloso y geniogloso
- Geniogloso y palatofaringeo
5. En un paciente es derivado por lesión del nervio vago derecho. Al evaluar el velo del paladar,
se solicita al paciente que diga ahh, entonces se puede observar que la úvula:
- Se eleva en el centro
- Se desvía a la derecha
- Se desvía a la izquierda
- Permanece sin movimiento
6. ¿Cuál de las siguientes estructuras tiene inervación somática?
- Estómago
- Mesosigmoides
- Peritoneo parietal
- Peritoneo Visceral
7. Estas diseñando un proyecto de investigación sobre los niveles de colesterol que se
absorben luego de una comida grasosa y deseas cuantificar la cantidad de colesterol que es
absorbido por el intestino antes que el hígado lo metabolice ¿de cual de los siguientes vasos
obtendrías la muestra para tu análisis?
- Vena porta
- Vena cava superior
- Vena hemiácigos accesoria
- Conducto torácico
8. La masticación es básicamente:
- Un movimiento reflejo
- Importante para la digestión sobretodo de carnes
- Un ralentizador del vaciamiento gástrico
- Una actividad consciente
9. En la estructura dentaria, se observa qué hay una composición muy similar a la del hueso en
la capa denominada:
- Predestina
- Esmalte
- Dentina
- Cemento
10. El nervio palatino menor inerva un área del paladar que está recubierta por epitelio:
- Estratificado plano no queratinizado
- Estratificado plano queratinizado
- Simple cilíndrico
- Simple plano
11. Mujer de 30 años llega a emergencia con dolor en hipogástrico. Al examen físico presenta
una masa palpable de 10cm de diámetro en la misma región, usted sospecharía de las
siguientes condiciones, excepto:
- Tumor renal
- Cáncer de recto superior
- Embarazo
- Tumor uterino
12. Un familiar le comenta que tiene úlcera gástrica por exceso de producción de ácido; con sus
conocimientos del sistema digestivo, usted le recomendaría que reduzca el consumo de:
- Agua
- Aminoácidos
- Carbohidratos
- Vitaminas
13. Con respecto de la regulación del pH del estómago; al utilizar un bloqueador de histamina,
usted espera que el pH del estómago:
- Disminuya
- Se mantenga sin cambio
- Se neutralice por acción de bicarbonato
- Aumente
14. Las siguientes alternativas son ciertas sobre las contracciones tónicas del músculo
gastrointestinal, excepto:
- Se encuentran principalmente es esfínteres
- Tienen relación con el ingreso persistente de iones sodio
- Obedece a una mayor frecuencia de potenciales en espiga
- Tienen regulación hormonal
15. ¿Cuál de las siguientes condiciones considera que es un trastorno de la musculatura lisa
esofágica?
- Asinergia faringoesfinteriana
- Hipotonía de los constrictores faríngeos
- Acalasia
- Hipertonía del esfínter esofágico superior
16. ¿Cuál de las siguientes alternativas es correcta sobre la motilidad esofágica?
- Las ondas primarias no son propulsoras y siempre van precedidas de deglución
- Las ondas primarias son propulsoras y pueden no ser precedidas por deglución
- Las ondas secundarias son propulsoras y no van precedidas de deglución
- Las ondas secundarias son propulsoras y siempre van precedidas de deglución
17. Respecto al peristaltismo intestinal, para cumplir la ley del intestino, usted espera que a nivel
distal del quimo se libere:
- Sustancia P
- Péptido liberador de gastrina (GRP)
- Acetilcolina
- Péptido intestinal vasoactivo
18. El peristaltismo depende que a nivel distal del bolo se secrete:
- Noradrenalina secretada por las fibras del sistema simpático
- Acetilcolina por las neuronas provenientes del nervio vago
- Péptido intestinal vasoactivo por neuronas
- Óxido nítrico por células endoteliales locales
19. La relajación receptiva gástrica, se produce principalmente por la acción de:
- El péptido intestinal vasoactivo
- La sustancia P
- La acetilcolina
- La bombesina
20. El peristaltismo intestinal se produce gracias a un reflejo que:
- Llega a los ganglios pre vertebrales
- Llega a la médula espinal
- Se origina dentro de la pared intestinal
- Llega al sistema nervioso central
21. ¿Cuál de las siguientes alternativas estimula las ondas de motilidad gastrointestinal
denominadas complejos migratorios interdigestivos?
- Alimentos
- Metoclopramida
- ERITROMICINA
- Colecistoquinina (CKK)
22. ¿Cuál de las siguientes alternativas es correcta sobre el control del peristaltismo?
- El peristaltismo intestinal aumenta por efecto de la colecistoquinina (CKK)
- La secretina aumenta la motilidad del intestino delgado
- La serotonina no tiene efecto sobre la motilidad digestiva
- El reflejo gastroentérico evita el avance del quimo
23. En un paciente con shock hipovolémico, la peristaltismo intestinal se encuentra:
- Aumentada
- Sin cambios
- Disminuida
- Invertida
24. Es un derivado del mesenterio dorsal:

- D
- B
- C
- A
25. En un paciente con falla en la fusión de los conductos de las yemas central y dorsal del
páncreas, usted esperaría encontrar:
- Drenaje de la mayor parte del jugo pancreático en la papila menor
- Drenaje adecuado del jugo pancreático
- Drenaje de la mayor parte del jugo pancreático en la papila mayor
- Estenosis del duodeno
Shiro Parcial

1. La relajación receptiva gástrica, se produce principalmente por la acción de:


- La sustancia P
- El péptido intestinal vasoactivo
- La bombesina
- La acetilcolina

2. ¿Cuál de las siguientes alternativas es correcta sobre el movimiento


peristáltico?
- Es independiente del plexo mientérico
- El contenido intestinal avanza sólo 5-10 cm
- Es un reflejo largo que depende de la integración con el tronco encefálico
- Se dirige en sentido distal siempre, nunca en sentido proximal

3. Con respecto a la regulación del peristaltismo, al aplicarle atropina


(antagonista colinérgico) a un paciente, es de esperarse que el peristaltismo:
- aumente
- se mantenga sin alteración
- estimule la acción de los receptores dopaminérgicos
- disminuya

4. El estímulo habitual para el movimiento peristáltico es:


- contracción de la musculatura circular Interna
- distensión local
- acción de la sustancia P
- estimulación vago-vagal

5. El peristaltismo depende que a nivel distal del bolo se secrete:


- péptido intestinal vasoactivo por neuronas
- noradrenalina secretada por las fibras del sistema simpático
- óxido nítrico por células endoteliales locales
- acetilcolina por las neuronas provenientes del nervio vago

6. La hormona responsable de los complejos migratorios interdigestivos tiene las


siguientes características, EXCEPTO:
- es inhibida por el alimento
- cumple funciones de aumentar la motilidad y secreción gástrica e intestinal
- se produce en el estómago y el duodeno
- se libera de forma cíclica
7. Respecto al peristaltismo intestinal, para cumplir la ley del intestino , usted
espera que a nivel distal del quimo se libere:
- péptido liberador de gastrina (GRP)
- acetilcolina
- sustancia P
- péptido intestinal vasoactivo

8. La distención del yeyuno provoca que se:


- aumente la frecuencia de las ondas lentas
- disminuya el número de espigas
- despolarice el potencial de reposo de membrana
- produzca una contracción tónica

9. Un familiar le comenta que tiene úlcera gástrica por exceso de producción de


ácido; con sus conocimientos del sistema digestivo, usted le recomendaría
que reduzca el consumo de:
- carbohidratos
- agua
- aminoácidos
- vitaminas

10. El pH óptimo para la digestión a nivel duodenal es ____________ y está


regulado principalmente por la liberación de _________________
- 6-8 / secretina
- 5-6 / colecistoquinina
- 5-6 / gastrina
- 8-10 / gastrina

11. El tubo digestivo posee glándulas, las glándulas submucosas se encuentran


en el:
- íleon y esófago
- esófago y duodeno
- estómago y duodeno
- duodeno y recto

12. En una cirugía abierta (laparotomía), el cirujano al abrir la cavidad peritoneal


por la parte anterior (línea media), lo primero que observa es:
- Colon sigmoides
- Estomago
- Duodeno
- Epiplón mayor

13. Sobre el control de la peristalsis del tubo digestivo, ________________ es un


mediador neural que induce la relajación durante la peristalsis
- la acetilcolina
- el péptido intestinal vasoactivo
- la serotonina
- la somatostatina

14. ¿Cuál de las siguientes alternativas es correcta sobre el control del


peristaltismo?
- El peristaltismo intestinal aumenta por efecto de la colecistoquinina (CCK)
- El reflejo gastroentérico evita el avance del quimo
- La serotonina no tiene efecto sobre la motilidad digestiva
- La secretina aumenta la motilidad del intestino delgado

15. El frenillo de los labios se encuentra en:


- la cavidad oral
- el piso de la boca
- el dorso de la lengua
- la cavidad vestibular

16. Las siguientes alternativas son ciertas sobre la actividad eléctrica del músculo
gastrointestinal, EXCEPTO:
- Si el potencial de membrana es más positivo, habrá mayor frecuencia de
espigas
- La despolarización lenta se debe principalmente al ingreso de Na+
- Las ondas lentas no son potenciales de acción
- Las ondas lentas son más frecuentes en el duodeno

17. Durante una cena, una gestante inspira por la boca profundamente de manera
frecuente; sin embargo, el organismo evita que el aire ingrese al esófago por la
acción:
- del istmo de las fauces
- del esfínter esofágico inferior
- del músculo cricofaríngeo
- de la epiglotis

18. El músculo liso gastrointestinal funciona como un sincitio debido a:


- el plexo mientérico de Auerbach
- el calcio
- las fibras musculares más largas
- las uniones en hendidura

19. Al ingresar líquidos o sólidos en la cavidad oral, un mecanismo que permite


que una persona respire mientras mastica es:
- la depresión del paladar blando
- el movimiento hacia afuera de los pliegues palatogloso y palatofaríngeo
- la depresión de la parte posterior de la lengua
- la elevación del paladar blando

20. Al rozar agua caliente en la punta de la lengua, usted esperaría que el estímulo
viaje a través del nervio:
- cuerda del tímpano
- lingual
- glosofaríngeo
- hipogloso

21. Las siguientes alternativas son correctas sobre la deglución, EXCEPTO:


- Consta de 2 fases
- En la fase voluntaria, el bolo es impulsado hacia arriba y hacia atrás por la
lengua
- El centro de la respiración es inhibido a nivel bulbar
- Para impedir el paso del alimento a la nasofaringe, el paladar blando se eleva

22. Paciente adulto con reflujo gastroesofágico es más probable que presente:
- desfluorización de los dientes
- destrucción de ameloblastos
- remodelación del esmalte
- descalcificación del esmalte

23. Considerando sus conocimientos en embriología, la disposición del intestino


delgado final tras la retracción de asas intestinales, se distribuye de la
siguiente manera:
- asas yeyunales en hipogastrio
- asas yeyunales en cuadrante inferior derecho
- asas ileales en cuadrante inferior derecho
- duodeno en hipocondrio izquierdo

24. Señale cuál de las estructuras que en el embrión se


encuentra comunicada con el saco vitelino por medio del
conducto onfalomesentérico:
- C
- A
- D
- B

25. Con seguridad, usted puede decir que la siguiente


cicatriz postapendicectomía se encuentra en la región denominada
- fosa iliaca derecha
- flanco izquierdo
- flanco derecho
- hipocondrio derecho

Parcial - José G.
1. Niña de 6 años se asusta por que se le ha aflojado un diente deciduo. Este
fenómeno se produce por:
a. laxitud del ligamento periodóntico
b. fractura del cemento
c. desmineralización del esmalte dental
d. aumento anómalo de la predentina

2. ¿Cuál de las siguientes estructuras deriva del intestino anterior?


a. A
b. B
c. C
d. D

3. En un paciente de 3 semanas de edad, con vómitos en proyectil, y nódulo epigástrico


reptante, usted esperaría encontrar:
a. engrosamiento de la circular interna pilórica
b. distensibilidad disminuida de la región oral del estómago
c. colecistoquinina aumentada
d. vómitos biliosos e intolerancia a los ácidos grasos

4. Sobre el control de la peristalsis del tubo digestivo, ________________ es un


mediador neural que induce la relajación durante la peristalsis.
a. el péptido intestinal vasoactivo
b. la serotonina
c. la acetilcolina
d. la somatostatina

5. En el plexo mientérico, el origen de los impulsos eferentes está en:


a. el plexo de Meissner
b. los ganglios paravertebrales
c. las células intersticiales de Cajal
d. el plexo de Aurbach

6. Un familiar le comenta que tiene úlcera gástrica por exceso de producción de ácido;
con sus conocimientos del sistema digestivo, usted le recomendaría que reduzca el
consumo de:
a. aminoácidos
b. vitaminas
c. carbohidratos
d. agua

7. Las siguientes alternativas son ciertas en relación al mesenterio, EXCEPTO:


a. Los omentos cumplen una función principal de irrigación visceral
b. El omento menor deriva del mesenterio ventral
c. El ligamento esplenorrenal une al bazo con el riñón izquierdo
d. Los mesenterios cumplen la función de sostén y suspensión de órganos

8. ¿Cuál de las siguientes alternativas es una característica de la estructura del


esófago?
a. Contiene glándulas submucosas principalmente en su tercio distal
b. Aumenta la presión intra esofágica durante la inspiración
c. Contiene músculo estriado en casi toda su longitud
d. El esfínter esofágico inferior es un esfínter anatómico

9. En una cirugía abierta (laparotomía), el cirujano al abrir la cavidad peritoneal por la


parte anterior (línea media), lo primero que observa es:
a. Epiplón mayor
b. Estómago
c. Colon sigmoides
d. Duodeno

10. El tubo digestivo posee glándulas, las glándulas submucosas se encuentran en el:
a. esófago y duodeno
b. duodeno y recto
c. íleon y esófago
d. estómago y duodeno

11. ¿Cuál de las siguientes alternativas detallan los músculos que ayudan a empujar el
bolo hacia la orofaringe?
a. Estilogloso y palatogloso
b. Geniogloso y palatofaríngeo
c. Hiogloso y geniogloso
d. Estilogloso y geniogloso

12. Al ingresar líquidos o sólidos en la cavidad oral, un mecanismo que permite que una
persona respire mientras mastica es:
a. la depresión del paladar blando
b. la elevación del paladar blando
c. el movimiento hacia afuera de los pliegues palatogloso y palatofaríngeo
d. la depresión de la parte posterior de la lengua

13. Durante el paso del bolo hacia la orofaringe, se desencadena una serie de
contracciones musculares que estrechan la cavidad faríngea. Estas contracciones
están mediadas por el nervio craneal:
a. X
b. IX
c. XI
d. XII
14. Las siguientes alternativas son factores que determinan la patencia y función
adecuada del esfínter esofágico inferior, EXCEPTO:
a. Hipertrofia de la circular interna
b. Canales lentos de calcio
c. Plicatura diafragmática
d. Angulación con el estómago

15. Durante una cena, una gestante inspira por la boca profundamente de manera
frecuente; sin embargo, el organismo evita que el aire ingrese al esófago por la
acción:
a. del músculo cricofaríngeo
b. del istmo de las fauces
c. del esfínter esofágico inferior
d. de la epiglotis

16. En un estudiante de medicina que está rindiendo un examen parcial, lo más probable
es que en ese momento su tránsito intestinal se encuentre:
a. muy lento
b. estimulado por acción de la sustancia P
c. muy acelerado
d. sin alteraciones

17. El principal gobernante sobre todos los movimientos gastrointestinales es el sistema


nervioso:
a. mientérico
b. parasimpático
c. somático
d. simpático

18. ¿Cuál de las siguientes condiciones considera que es un trastorno de la musculatura


lisa esofágica?
a. Acalasia
b. Hipotonía de los constrictores faríngeos
c. Hipertonía del esfínter esofágico superior
d. Asinergia faringoesfinteriana

19. El estímulo habitual para el movimiento peristáltico es:


a. distensión local
b. acción de la sustancia P
c. contracción de la musculatura circular Interna
d. estimulación vago-vagal

20. La hormona responsable de los complejos migratorios interdigestivos tiene las


siguientes características, EXCEPTO:
a. cumple funciones de aumentar la motilidad y secreción gástrica e intestinal
b. se libera de forma cíclica
c. se produce en el estómago y el duodeno
d. es inhibida por el alimento
21. Respecto al peristaltismo intestinal, para cumplir la ley del intestino , usted espera
que a nivel distal del quimo se libere:
a. péptido intestinal vasoactivo
b. acetilcolina
c. péptido liberador de gastrina (GRP)
d. sustancia P

22. ¿Cuál de las siguientes alternativas es correcta sobre el control autónomo del
aparato gastrointestinal?
a. La estimulación simpática estimula a la muscularis mucosae
b. El plexo mientérico de Auerbach cumple funciones inhibitorias
c. Las terminaciones nerviosas simpáticas liberan mayor cantidad de adrenalina
que noradrenalina
d. Al seccionar el vago, la inervación parasimpática del colon sigmoides
disminuye

23. La distención del yeyuno provoca que se:


a. despolarice el potencial de reposo de membrana
b. produzca una contracción tónica
c. disminuya el número de espigas
d. aumente la frecuencia de las ondas lentas

24. El frenillo de los labios se encuentra en:


a. la cavidad vestibular
b. el dorso de la lengua
c. el piso de la boca
d. la cavidad oral

25. Con seguridad, usted puede decir que la siguiente cicatriz postapendicectomía se
encuentra en la región denominada:

a. flanco derecho
b. hipocondrio derecho
c. flanco izquierdo
d. fosa iliaca derecha

EXAMEN PARCIAL - PEQUE

1. La masticación es básicamente:
● un movimiento reflejo
● un ralentizador del vaciamiento gástrico
● una actividad consciente
● importante para la digestión sobretodo de carnes
2. En el plexo mientérico, el origen de los impulsos eferentes está en:
● el plexo de Meissner
● el plexo de Aurbach
● las células intersticiales de Cajal
● los ganglios paravertebrales
3. ¿Cuál de las siguientes alternativas es correcta sobre el control del peristaltismo?
● La secretina aumenta la motilidad del intestino delgado
● El reflejo gastroentérico evita el avance del quimo
● La serotonina no tiene efecto sobre la motilidad digestiva
● El peristaltismo intestinal aumenta por efecto de la colecistoquinina (CCK)
4. Señale cuál de las estructuras que en el embrión se encuentra comunicada con el
saco vitelino por medio del conducto onfalomesentérico:

● D
● A
● C
● B
5. En un paciente de 3 semanas de edad, con vómitos en proyectil, y nódulo epigástrico
reptante, usted esperaría encontrar:
● vómitos biliosos e intolerancia a los ácidos grasos
● distensibilidad disminuida de la región oral del estómago
● engrosamiento de la circular interna pilórica
● colecistoquinina aumentada
6. Durante el paso del bolo hacia la orofaringe, se desencadena una serie de
contracciones musculares que estrechan la cavidad faríngea. Estas contracciones
están mediadas por el nervio craneal:
● IX
● XII
● X
● XI
7. Cuando el istmo de las fauces se cierra, se evita que el alimento pase hacia la
orofarínge y permite respirar mientras se mastica. Este cierre se debe a la
contracción y aproximación de los músculos:
● estilofaríngeos
● palatofaríngeos
● estiloglosos
● palatoglosos
8. Las siguientes alternativas son factores que determinan la patencia y función
adecuada del esfínter esofágico inferior, EXCEPTO:
● Hipertrofia de la circular interna
● Canales lentos de calcio
● Plicatura diafragmática
● Angulación con el estómago
9. El tubo digestivo posee glándulas, las glándulas submucosas se encuentran en el:
● duodeno y recto
● esófago y duodeno
● íleon y esófago
● estómago y duodeno
10. ¿Cuál de las siguientes estructuras tiene inervación somática?
● Mesosigmoides
● Peritoneo visceral
● Estómago
● Peritoneo parietal
11. Mujer de 30 años llega a emergencia con dolor en hipogástrico. Al examen físico
presenta una masa palpable de 10 cm de diámetro a en la misma región, usted
sospecharía de las siguientes condiciones, EXCEPTO:
● Embarazo
● Tumor renal
● Cáncer de recto superior
● Tumor uterino
12. Las siguientes alternativas son ciertas en relación al mesenterio, EXCEPTO:
● Los omentos cumplen una función principal de irrigación visceral
● Los mesenterios cumplen la función de sostén y suspensión de órganos
● El omento menor deriva del mesenterio ventra
● El ligamento esplenorrenal une al bazo con el riñón izquierdo
13. Al realizar una vagotomía por úlcera péptica, usted esperaría una disminución de
producción de gastrina debido a la:
● menor distensión de las paredes gástricas
● ausencia de acetilcolina vagal
● ausencia de bombesina vagal
● ausencia de histamina
14. Al ingresar líquidos o sólidos en la cavidad oral, un mecanismo que permite que una
persona respire mientras mastica es:
● la elevación del paladar blando
● la depresión de la parte posterior de la lengu
● el movimiento hacia afuera de los pliegues palatogloso y palatofaríngeo
● la depresión del paladar blando
15. ¿Cuál de las siguientes alternativas detallan los músculos que ayudan a empujar el
bolo hacia la orofaringe?
● Estilogloso y geniogloso
● Geniogloso y palatofaríngeo
● Hiogloso y geniogloso
● Estilogloso y palatogloso
16. La información sensitiva aferente del sistema gastrointestinal pasa por las siguientes
estructuras, EXCEPTO:
● Ganglios prevertebrales
● Médula espinal
● Tronco encefálico
● Tálamo
17. Los reflejos enterogástricos son desencadenados por las siguientes circunstancias,
EXCEPTO:
● Alta carga de carbohidratos en duodeno
● Distensión de pared intestina
● Quimo con alta osmolaridad
● Disminución de pH duodenal
18. Las siguientes alternativas son ciertas sobre las contracciones tónicas del músculo
gastrointestinal, EXCEPTO:
● Obedece a una mayor frecuencia de potenciales en espiga
● Tienen regulación hormonal
● Se encuentran principalmente en esfínteres
● Tienen relación con el ingreso persistente de iones sodio
19. Respecto al peristaltismo intestinal, para cumplir la ley del intestino , usted espera
que a nivel distal del quimo se libere:
● sustancia P
● péptido intestinal vasoactivo
● péptido liberador de gastrina (GRP)
● acetilcolina
20. El principal gobernante sobre todos los movimientos gastrointestinales es el sistema
nervioso:
● mientérico
● parasimpático
● somático
● simpático
21. La relajación receptiva gástrica, se produce principalmente por la acción de:
● la bombesina
● la sustancia P
● la acetilcolina
● el péptido intestinal vasoactivo
22. Con respecto a la regulación del peristaltismo, al aplicarle atropina (antagonista
colinérgico) a un paciente, es de esperarse que el peristaltismo:
● disminuya
● se mantenga sin alteración
● aumente
● estimule la acción de los receptores dopaminérgicos
23. ¿Cuál de las siguientes alternativas es correcta sobre la motilidad esofágica?
● Las ondas primarias son propulsoras y pueden no ser precedidas por
deglució
● Las ondas secundarias son propulsoras y siempre van precedidas de
deglución
● Las ondas primarias no son propulsoras y siempre van precedidas de
deglución
● Las ondas secundarias son propulsoras y no van precedidas de deglución
24. El nervio palatino menor inerva un área del paladar que está recubierta por epitelio:
● simple plano
● estratificado plano no queratinizado
● estratificado plano queratinizado
● simple cilíndrico
25. Paciente adulto con reflujo gastroesofágico es más probable que presente:
● descalcificación del esmalte
● remodelación del esmalte
● destrucción de ameloblastos
● desfluorización de los dientes

ALEJANDRO HERRERA

1. Al rozar agua caliente en la punta de la lengua, usted esperaría que el estímulo viaje
a través del nervio:

a) Lingual

b) Cuerda del timpano

c) Glosofaríngeo

d) Hipogloso

2. Al ingresar líquidos o sólidos en la cavidad oral, un mecanismo que permite que una
persona respire mientras mastica es:

a) El movimiento hacia afuera de los pliegues palatogloso y Palatofaríngeo

b) La elevación del paladar blando


c) La depresión de la parte posterior de la lengua

d) La depresión del paladar blando

3. Paciente adulto con reflujo gastroesofágico es más probable que presente:

a) Descalcificación del esmalte

b) Destrucción de ameloblastos

c) Remodelación del esmalte

d) Desfluorización de los dientes

4. La masticación es básicamente:

a) Importante para la digestión sobretodo de carnes

b) Un ralentizador del vaciamiento gástrico

c) Una actividad consciente

d) Un movimiento reflejo

5. Paciente de 34 años es víctima de asalto con arma de fuego, recibiendo un impacto


directo en el abdomen. En base a la radiografía, usted puede registrar en la historia
clínica que el proyectil se encuentra topográficamente en el:

a) Flanco derecho

b) Mesogastrio

c) Flanco izquierdo
d) Hipocondrio izquierdo

6. Señale cuál de las estructuras que en el embrión se encuentra comunicada con el


saco vitelino por medio del conducto onfalomesentérico:

7. En un paciente de 3 semanas de edad, con vómitos en proyectil, y nódulo


epigástrico reptante, usted esperaría encontrar:

a) colecistoquinina aumentada

b) Vómitos biliosos e intolerancia a los ácidos grasos

c) Distensibilidad disminuida de la región oral del estómago

d) Engrosamiento de la circular interna pilórica

8. ¿Cuál de las siguientes estructuras tiene inervación somática?

a) Estómago

b) Peritoneo visceral
c) Mesosigmoides

d) Peritoneo parietal

9. Estas diseñando un proyecto de investigación sobre los niveles de colesterol que se


absorben luego de una comida grasosa y deseas cuantificar la cantidad de colesterol
que es absorbido por el intestino antes que el hígado lo metabolice ¿de cuál de los
siguientes vasos obtendrías la muestra para tu análisis?

a) Vena porta

b) Vena cava superior

c) Conducto torácico

d) Vena hemiácigos accesoria

10. Durante el paso del bolo hacia la orofaringe, se desencadena una serie de
contracciones musculares que estrechan la cavidad faríngea. Estas contracciones están
mediadas por el nervio craneal:

a) X

b) XI

c) XII

d) IX

11. Cuando el istmo de las fauces se cierra, se evita que el alimento pase hacia la
orofarínge y permite respirar mientras se mastica. Este cierre se debe a la contracción y
aproximación de los músculos:

a) Palatoglosos

b) Palatofaríngeos

c) Estiloglosos

d) Estilofaríngeos

12. Las siguientes alternativas son factores que determinan la patencia y función
adecuada del esfínter esofágico inferior, EXCEPTO:

a) Plicatura diafragmática
b) Hipertrofia de la circular interna

c) Angulación con el estómago

d) Canales lentos de calcio

13. Respecto al peristaltismo intestinal, para cumplir la ley del intestino, usted espera
que a nivel distal del quimo se libere:

a) Péptido liberador de gastrina (GRP)

b) Acetilcolina

c) Péptido intestinal vasoactivo

d) Sustancia P

14. El estímulo habitual para el movimiento peristáltico es:

a) Acción de la sustancia P

b) Contracción de la musculatura circular Interna

c) Estimulación vago-vagal

d) Distensión local

15. ¿Cuál de las siguientes condiciones considera que es un trastorno de la


musculatura lisa esofágica?

a) Acalasia

b) Asinergia faringoesfinteriana

c) Hipotonía de los constrictores faríngeos

d) Hipertonía del esfínter esofágico superior

16. ¿Cuál de las siguientes alternativas es correcta sobre la motilidad esofágica?

a) Las ondas primarias son propulsoras y pueden no ser precedidas por


deglución

b) Las ondas primarias no son propulsoras y siempre van precedidas de


deglución
c) Las ondas secundarias son propulsoras y siempre van precedidas de
deglución

d) Las ondas secundarias son propulsoras y no van precedidas de


deglución

17. El peristaltismo depende que a nivel distal del bolo se secrete:

a) Péptido intestinal vasoactivo por neuronas

b) Noradrenalina secretada por las fibras del sistema simpático

c) Acetilcolina por las neuronas provenientes del nervio vago

d) Óxido nítrico por células endoteliales locales

18. ¿Cuál de las siguientes alternativas es correcta sobre el movimiento peristáltico?

a) Es un reflejo largo que depende de la integración con el tronco


encefálico

b) Se dirige en sentido distal siempre, nunca en sentido proximal

c) El contenido intestinal avanza sólo 5-10 cm

d) Es independiente del plexo mientérico

19. En un estudiante de medicina que está rindiendo un examen parcial, lo más


probable es que en ese momento su tránsito intestinal se encuentre:

a) Muy acelerado

b) Estimulado por acción de la sustancia p

c) Sin alteraciones

d) Muy lento

20. Las siguientes alternativas son ciertas sobre las contracciones tónicas del músculo
gastrointestinal, EXCEPTO:

a) Tienen relación con el ingreso persistente de iones sodio

b) Se encuentran principalmente en esfínteres


c) Tienen regulación hormonal

d) Obedece a una mayor frecuencia de potenciales en espiga

21. Con respecto de la regulación del pH del estómago; al utilizar un bloqueador de


histamina, usted espera que el pH del estómago:

a) Aumente

b) Disminuya

c) Se mantenga sin cambio

d) Se neutralice por acción de bicarbonato

22. Las siguientes hormonas disminuyen el vaciamiento gástrico, EXCEPTO:

a) Gastrina

b) Péptido insulinotrópico dependiente de glucosa

c) Colecistoquinina

d) Secretina

23. El frenillo de los labios se encuentra en:

a) El piso de la boca

b) La cavidad oral

c) La cavidad vestibular

d) El dorso de la lengua

24. En un paciente con shock hipovolémico, la peristalsis intestinal se encuentra:

a) Aumentada

b) Sin cambios

c) Invertida

d) Disminuida
25. Sobre el control de la peristalsis del tubo digestivo, ________________ es un
mediador neural que induce la relajación durante la peristalsis.

a) La somatostatina

b) El péptido intestinal vasoactivo

c) La acetilcolina

d) La serotonina

26. Las siguientes funciones son inhibidas por la hormona somatostatina, EXCEPTO:
a) motilidad intestinal
b) secreción de enzimas pancreaticas
c) Secrecion gastrica de HCl
d) Liberacion de gastrina

27. ¿Cuál de las siguientes alternativas es correcta sobre el control del peristaltismo?

A) El reflejo gastroentérico evita el avance del quimo


B) La secretina aumenta la motilidad del intestino delgado
C) El peristaltismo intestinal aumenta por efecto de la colecistoquinina (CCK)
D) La serotonina no tiene efecto sobre la motilidad digestiva

28. En el plexo mientérico, el origen de los impulsos eferentes está en:

a) el plexo de Auerbach
b) los ganglios paravertebrales
c) Las células intersticiales de cajal
d) plexo de meissner

29. Los corpúsculos gustativos se encuentran en la lengua, pero además se les puede
encontrar en:

a) el dorso de la lengua
b) el paladar blando
c) el paladar duro
d) las encías

30. ¿Cuál de las siguientes condiciones considera que es un trastorno de la musculatura lisa
esofágica?

a) Hipertonía del esfínter esofágico superior


b) Acalasia
c) Hipotonía de los constrictores faríngeos
d) Asinergia faringoesfinteriana
Dayanne Cruz

1. ¿Cuál de las siguientes alternativas detallan los músculos que ayudan a empujar el
bolo hacia la orofaringe?
a. Geniogloso y palatofaríngeo
b. Estilogloso y geniogloso
c. Hiogloso y geniogloso
d. Estilogloso y palatoglo
2. Paciente con lesión del hipogloso del lado izquierdo. Para evaluarlo se le pide al
paciente que saque la lengua, la cual se espera que la punta de la lengua se dirija
hacia:
a. abajo
b. el lado izquierdo
c. adelante
d. el lado derecho
3. Cuando el istmo de las fauces se cierra, se evita que el alimento pase hacia la
orofarínge y permite respirar mientras se mastica. Este cierre se debe a la
contracción y aproximación de los músculos
a. estiloglosos
b. palatofaríngeos
c. palatoglosos
d. estilofaríngeos
4. Las siguientes alternativas son ciertas sobre la actividad eléctrica del músculo
gastrointestinal, EXCEPTO:
a. Si el potencial de membrana es más positivo, habrá mayor frecuencia de
espigas
b. Las ondas lentas son más frecuentes en el duodeno
c. La despolarización lenta se debe principalmente al ingreso de Na+
d. Las ondas lentas no son potenciales de acción
5. Durante la deglución, el bolo es impedido de regresar a la cavidad oral gracias a la
acción de diversos músculos, entre ellos el músculo:
a. estilofaríngeo
b. palatofaríngeo
c. cricofaríngeo
d. salpingofaríngeo
6. En una cirugía abierta (laparotomía), el cirujano al abrir la cavidad peritoneal por la
parte anterior (línea media), lo primero que observa es
a. Estomago
b. Duodeno
c. Colon sigmoides
d. Epiplón mayor
7. Estas diseñando un proyecto de investigación sobre los niveles de colesterol que se
absorben luego de una comida grasosa y deseas cuantificar la cantidad de colesterol
que es absorbido por el intestino antes que el hígado lo metabolice ¿de cuál de los
siguientes vasos obtendrías la muestra para tu análisis?
a. Vena cava superior
b. Conducto torácico
c. Vena hemiácigos accesoria
d. Vena porta
8. Paciente de 34 años es víctima de asalto con arma de fuego, recibiendo un impacto
directo en el abdomen. En base a la radiografía, usted puede registrar en la historia
clínica que el proyectil se encuentra topográficamente en el:

a. flanco izquierdo
b. hipocondrio izquierdo
c. flanco derecho
d. mesogastrio

9. El nervio palatino menor inerva un área del paladar que está recubierta por
epitelio:

a. estratificado plano queratinizado


b. estratificado plano no queratinizado
c. simple plano
d. simple cilíndrico

10. ¿Cuál de las siguientes estructuras deriva del intestino anterior?

a. C
b. B
c. A
d. D

11. ¿Cuál de las siguientes atresias/fístulas traqueo esofágicas considera usted que es
incompatible con la vida (de no recibir tratamiento)?

a. Tipo E
b. Tipo B
c. Tipo C
d. Tipo A

12. Las siguientes hormonas disminuyen el vaciamiento gástrico, EXCEPTO:

a. Péptido insulinotrópico dependiente de glucosa


b. Secretina
c. Colecistoquinina
d. Gastrina

13. Las siguientes alternativas son ciertas en relación al mesenterio, EXCEPTO

a. El omento menor deriva del mesenterio ventral


b. El ligamento esplenorrenal une al bazo con el riñón izquierdo
c. Los mesenterios cumplen la función de sostén y suspensión de órganos
d. Los omentos cumplen una función principal de irrigación viscera

14. ¿Cuál de las siguientes alternativas es una característica de la estructura del esófago?

a. Contiene músculo estriado en casi toda su longitud


b. El esfínter esofágico inferior es un esfínter anatómico
c. Aumenta la presión intra esofágica durante la inspiración
d. Contiene glándulas submucosas principalmente en su tercio distal

15. ¿Cuál de las siguientes alternativas es correcta sobre el control del peristaltismo?

a. La serotonina no tiene efecto sobre la motilidad digestiva


b. El reflejo gastroentérico evita el avance del quimo
c. La secretina aumenta la motilidad del intestino delgado
d. El peristaltismo intestinal aumenta por efecto de la colecistoquinina (CCK)

16. En el plexo mientérico, el origen de los impulsos eferentes está en:

a. el plexo de Meissner
b. las células intersticiales de Cajal
c. el plexo de Aurbach
d. los ganglios paravertebrales

17. El peristaltismo depende que a nivel distal del bolo se secrete:

a. noradrenalina secretada por las fibras del sistema simpático


b. óxido nítrico por células endoteliales locales
c. acetilcolina por las neuronas provenientes del nervio vago
d. péptido intestinal vasoactivo por neuronas

18. En un estudiante de medicina que está rindiendo un examen parcial, lo más probable es
que

a. muy lento
b. sin alteraciones
c. estimulado por acción de la sustancia P
d. muy acelerado en ese momento su tránsito intestinal se encuentre:

19. Los reflejos enterogástricos son desencadenados por las siguientes circunstancias,
EXCEPTO:

a. Distensión de pared intestinal


b. Disminución de pH duodenal
c. Quimo con alta osmolaridad
d. Alta carga de carbohidratos en duodeno

20. La distención del yeyuno provoca que se:


a. disminuya el número de espigas
b. despolarice el potencial de reposo de membrana
c. produzca una contracción tónica
d. aumente la frecuencia de las ondas lentas

21. Respecto al peristaltismo intestinal, para cumplir la ley del intestino , usted espera que a
nivel distal del quimo se libere:
a. péptido intestinal vasoactivo
b. sustancia P
c. péptido liberador de gastrina (GRP)
d. acetilcolina
22. ¿Cuál de las siguientes alternativas es correcta sobre el movimiento peristáltico?
a. Es un reflejo largo que depende de la integración con el tronco encefálico
b. El contenido intestinal avanza sólo 5-10 cm
c. Es independiente del plexo mientérico
d. Se dirige en sentido distal siempre, nunca en sentido proximal
23. ¿Cuál de las siguientes alternativas estimula las ondas de motilidad gastrointestinal
denominadas complejos migratorios interdigestivos?
a. Metoclopramida
b. Alimentos
c. Eritromicina
d. Colecistoquinina (CCK)
24. ¿Cuál de las siguientes condiciones considera que es un trastorno de la musculatura lisa
esofágica?
a. Hipertonía del esfínter esofágico superior
b. Asinergia faringoesfinteriana
c. Acalasia
d. Hipotonía de los constrictores faríngeos
25. En la estructura dentaria, se observa que hay una composición muy similar a la del
hueso en la capa denominada
a. dentina
b. predentina.
c. esmalte
d. cemento

1. ¿Cuál de las siguientes alternativas detallan los músculos que ayudan a empujar el bolo
hacia la orofaringe?

- Estilogloso y palatogloso
- Hiogloso y geniogloso
- Geniogloso y palatofaríngeo

2. Paciente con lesión del hipogloso del lado izquierdo. Para evaluarlo se le pide al paciente que
saque la lengua, la cual se espera que la punta de la lengua se dirija hacia
- El lado izquierdo
- Adelante
- Abajo
- El lado derecho

3. Niña de 6 años se asusta porque se le ha aflojado un diente deciduo. Este fenómeno se


produce por:
- Laxitud del ligamento periodontal
- Aumento anómalo de la predentina
- Fractura del cemento
- Desmineralización del esmalte dental

4. Sobre el control de la peristalsis del tubo digestivo, ________________ es un mediador


neutral que induce la relajación durante la peristalsis
- El péptido intestinal vasoactivo
- Somatostatina
- Serotonina
- Acetilcolina

5. En el plexo mientérico , el origen de los impulsos eferentes esta en


- El plexo de Auerbach
- Los ganglios paravertebrales
- Plexo de Meissner
- Células intersticiales de Cajal
6. Los corpúsculos gustativos se encuentran en la lengua, pero además se les puede encontrar
en:
- El dorso de la lengua
- El paladar duro
- El paladar blando
- Las encías

7. La masticación es básicamente:
- Un movimiento reflejo
- Una actividad consciente
- Un ralentizador del vaciamiento gástrico
- Importante para la digestión sobre todo de carnes

8. Con respecto de la regulación del pH del estómago; al utilizar un bloqueador de histamina,


usted espera que el pH del estómago:
- Aumente
- Disminuya
- Neutraliza por acción de bicarbonato
- Se mantenga sin cambio

9. Al realizar una vagotomía por úlcera péptica, usted esperaría una disminución de producción
de gastrina debido a la:
- Ausencia de histamina
- Menor distensión de las paredes gástricas
- ausencia de bombesina vagal
- ausencia de acetilcolina vagal

10. Mujer de 30 años llega a emergencia con dolor en hipogástrico. Al examen físico presenta
una masa palpable de 10 cm de diámetro a en la misma región, usted sospecharía de las
siguientes condiciones, EXCEPTO:
- Tumor renal
- Cáncer de recto superior
- Embarazo
- Tumor uterino

11. En un estudiante de medicina que está rindiendo un examen parcial, lo más probable es que
en ese momento su tránsito intestinal se encuentre:
- Muy lento
- Estimulado por acción de la sustancia P
- muy acelerado
- sin alteraciones

12. ¿Cuál de las siguientes alternativas estimula las ondas de motilidad gastrointestinal
denominadas complejos migratorios interdigestivos?
- Eritromicina
- Colecistoquinina
- Alimentos
- Metoclopramida
13. Respecto al peristaltismo intestinal, para cumplir la ley del intestino , usted espera que a nivel
distal del quimo se libere:
- Péptido intestinal vasoactivo
- acetilcolina
- sustancia p
- péptido liberador de gastrina
14. ¿ Cual de las siguientes alternativas es correcta sobre el control autonomo del aparato
gastrointestinal?
- La estimulación simpática estimula a la muscularis mucosae
- Las terminaciones nerviosas simpáticas liberan mayor cantidad de adrenalina que
noradrenalina
- al seccionar al vago , la inervación parasimpática del colon sigmoides disminuye
- el plexo mientérico de auerbach cumple funciones inhibitorias
15. La distensión del yeyuno provoca que
- Aumente la frecuencia de ondas lentas
- disminuya el número de espigas
- produzca contracción tónica
- despolariza el potencial de reposo de membrana
Francisco Parcial
Cuando una persona coloca en su boca una sustancia con alta concentración de carbohidratos, lo
que debería pasar es que:
- aumente las concentraciones séricas de gastrina
- disminuya la sensación de hambre
- aumenta la motilidad gástrica
- se dilate el esfínter esofágico inferior

ANGELA PARCIAL:
1. Las carnes deben su sabor especialmente delicioso debido a que presentan en su
composición:
- glutamato
2. En un paciente es derivado por lesión del nervio vago derecho. Al evaluar el velo del
paladar, se solicita al paciente que diga ahh , entonces se puede observar que la úvula:
- se desvía a la izquierda
3. Al rozar agua caliente en la punta de la lengua, usted esperaría que el estímulo viaje a
través del nervio:
- lingual
4. Considerando sus conocimientos en embriología, la disposición del intestino delgado
final tras la retracción de asas intestinales, se distribuye de la siguiente manera:
- asas ileales en cuadrante inferior derecho
5. En un paciente de 3 semanas de edad, con vómitos en proyectil, y nódulo epigástrico
reptante, usted esperaría encontrar:
- engrosamiento de la circular interna pilórica
6. La masticación es básicamente:
- un movimiento reflejo
7. Un familiar le comenta que tiene úlcera gástrica por exceso de producción de ácido;
con sus conocimientos del sistema digestivo, usted le recomendaría que reduzca el
consumo de:
- aminoácidos
8. Con respecto de la regulación del pH del estómago; al utilizar un bloqueador de
histamina, usted espera que el pH del estómago:
- aumente
9. Paciente adulto con reflujo gastroesofágico es más probable que presente:
- descalcificación del esmalte
10. Sobre el control de la peristalsis del tubo digestivo, ________________ es un mediador
neural que induce la relajación durante la peristalsis.
- el péptido intestinal vasoactivo
11. En el plexo mientérico, el origen de los impulsos eferentes está en:
- el plexo de Meissner
12. Mujer de 30 años llega a emergencia con dolor en hipogástrico. Al examen físico
presenta una masa palpable de 10 cm de diámetro a en la misma región, usted
sospecharía de las siguientes condiciones, EXCEPTO:
- Tumor renal
13. Los reflejos enterogástricos son desencadenados por las siguientes circunstancias,
EXCEPTO:
- Alta carga de carbohidratos en duodeno
14. La hormona responsable de los complejos migratorios interdigestivos tiene las
siguientes características, EXCEPTO:
- cumple funciones de aumentar la motilidad y secreción gástrica e intestinal
15. La relajación receptiva gástrica, se produce principalmente por la acción de:
- el péptido intestinal vasoactivo
16. ¿Cuál de las siguientes alternativas estimula las ondas de motilidad gastrointestinal
denominadas complejos migratorios interdigestivos?
- Eritromicina
17. En un estudiante de medicina que está rindiendo un examen parcial, lo más probable
es que en ese momento su tránsito intestinal se encuentre:
- muy lento
18. El estímulo habitual para el movimiento peristáltico es:
- distensión local
19. La distención del yeyuno provoca que se:
- despolarice el potencial de reposo de membrana
20. El principal gobernante sobre todos los movimientos gastrointestinales es el sistema
nervioso:
- mientérico
21. Estas diseñando un proyecto de investigación sobre los niveles de colesterol que se
absorben luego de una comida grasosa y deseas cuantificar la cantidad de colesterol
que es absorbido por el intestino antes que el hígado lo metabolice ¿de cuál de los
siguientes vasos obtendrías la muestra para tu análisis?
- Conducto torácico
22. El tubo digestivo posee glándulas, las glándulas submucosas se encuentran en el:
- esófago y duodeno
23. Durante la deglución, el bolo es impedido de regresar a la cavidad oral gracias a la
acción de diversos músculos, entre ellos el músculo:
- palatofaríngeo
24. Cuando el istmo de las fauces se cierra, se evita que el alimento pase hacia la
orofarínge y permite respirar mientras se mastica. Este cierre se debe a la contracción
y aproximación de los músculos:
- palatoglosos
25. El músculo liso gastrointestinal funciona como un sincitio debido a:
- las uniones en hendidura

En el esófago, el plexo mientérico o de Auerbach interviene en el _______ de los impulsos ______

a. fin / aferentes
b. fin / eferentes
c. inicio / eferentes
d. inicio / aferentes

Pregunta 1
El nervio palatino menor inerva un área del paladar que está recubierta por epitelio:
● estratificado plano no queratinizado
● simple plano
● estratificado plano queratinizado
● simple cilíndrico
Pregunta 2
¿Cuál de las siguientes atresias/fístulas traqueo esofágicas considera usted que es
incompatible con la vida (de no recibir tratamiento)?
● Tipo A
● Tipo B
● Tipo C
● Tipo E
Pregunta 3
¿Cuál de las siguientes estructuras deriva del intestino anterior?
● A
● D
● C
● B
Pregunta 4
El tubo digestivo posee glándulas, las glándulas submucosas se encuentran en el:
● esófago y duodeno
● duodeno y recto
● íleon y esófago
● estómago y duodeno
Pregunta 5
Estas diseñando un proyecto de investigación sobre los niveles de colesterol que se
absorben luego de una comida grasosa y deseas cuantificar la cantidad de colesterol que es
absorbido por el intestino antes que el hígado lo metabolice ¿de cuál de los siguientes vasos
obtendrías la muestra para tu análisis?
● Vena hemiácigos accesoria
● Vena porta
● Conducto torácico
● Vena cava superior
Pregunta 6
Un familiar le comenta que tiene úlcera gástrica por exceso de producción de ácido; con sus
conocimientos del sistema digestivo, usted le recomendaría que reduzca el consumo de:
● vitaminas
● agua
● carbohidratos
● aminoácidos
Pregunta 7
Con respecto de la regulación del pH del estómago; al utilizar un bloqueador de histamina,
usted espera que el pH del estómago:
● se neutralice por acción de bicarbonato
● se mantenga sin cambio
● disminuya
● aumente
Pregunta 8
¿Cuál de las siguientes alternativas es una característica de la estructura del esófago?
● El esfínter esofágico inferior es un esfínter anatómico
● Contiene músculo estriado en casi toda su longitud
● Contiene glándulas submucosas principalmente en su tercio distal
● Aumenta la presión intra esofágica durante la inspiración
Pregunta 9
¿Cuál de las siguientes alternativas detallan los músculos que ayudan a empujar el bolo
hacia la orofaringe?
● Hiogloso y geniogloso
● Geniogloso y palatofaríngeo
● Estilogloso y geniogloso
● Estilogloso y palatogloso
Pregunta 10
Al ingresar líquidos o sólidos en la cavidad oral, un mecanismo que permite que una
persona respire mientras mastica es:
● la depresión del paladar blando
● la elevación del paladar blando
● el movimiento hacia afuera de los pliegues palatogloso y palatofaríngeo
● la depresión de la parte posterior de la lengua
Pregunta 11
Niña de 6 años se asusta por que se le ha aflojado un diente deciduo. Este fenómeno se
produce por:
● laxitud del ligamento periodóntico
● desmineralización del esmalte dental
● aumento anómalo de la predentina
● fractura del cemento
Pregunta 12
Con seguridad, usted puede decir que la siguiente cicatriz postapendicectomía se encuentra
en la región denominada:
● flanco derecho
● hipocondrio derecho
● fosa iliaca derecha
● flanco izquierdo
Pregunta 13
Cuando el istmo de las fauces se cierra, se evita que el alimento pase hacia la orofarínge y
permite respirar mientras se mastica. Este cierre se debe a la contracción y aproximación de
los músculos:
● estiloglosos
● palatofaríngeos
● estilofaríngeos
● palatoglosos
Pregunta 14
Durante el paso del bolo hacia la orofaringe, se desencadena una serie de contracciones
musculares que estrechan la cavidad faríngea. Estas contracciones están mediadas por el
nervio craneal:
● X
● IX
● XII
● XI
Pregunta 15
Durante la deglución, el bolo es impedido de regresar a la cavidad oral gracias a la acción
de diversos músculos, entre ellos el músculo:
● palatofaríngeo
● estilofaríngeo
● salpingofaríngeo
● cricofaríngeo
Pregunta 16
En el plexo mientérico, el origen de los impulsos eferentes está en:
● el plexo de Aurbach
● el plexo de Meissner
● las células intersticiales de Cajal
● los ganglios paravertebrales
Pregunta 17
¿Cuál de las siguientes alternativas es correcta sobre el control del peristaltismo?
● La serotonina no tiene efecto sobre la motilidad digestiva
● El reflejo gastroentérico evita el avance del quimo
● La secretina aumenta la motilidad del intestino delgado
● El peristaltismo intestinal aumenta por efecto de la colecistoquinina (CCK)
Pregunta 18
En un estudiante de medicina que está rindiendo un examen parcial, lo más probable es que
en ese momento su tránsito intestinal se encuentre:
● estimulado por acción de la sustancia P
● sin alteraciones
● muy acelerado
● muy lento
Pregunta 19
El peristaltismo intestinal se produce gracias a un reflejo que:
● llega a los ganglios pre vertebrales
● llega a la médula espinal
● se origina dentro de la pared intestinal
● llega al sistema nervioso central
Pregunta 20
El estímulo habitual para el movimiento peristáltico es:
● distensión local
● acción de la sustancia P
● contracción de la musculatura circular Interna
● estimulación vago-vagal
Pregunta 21
¿Cuál de las siguientes alternativas es correcta sobre el control autónomo del aparato
gastrointestinal?
● La estimulación simpática estimula a la muscularis mucosae
● El plexo mientérico de Auerbach cumple funciones inhibitorias
● Las terminaciones nerviosas simpáticas liberan mayor cantidad de adrenalina que
noradrenalina
● Al seccionar el vago, la inervación parasimpática del colon sigmoides disminuye
Pregunta 22
El principal gobernante sobre todos los movimientos gastrointestinales es el sistema
nervioso:
● simpático
● somático
● mientérico
● parasimpático
Pregunta 23
¿Cuál de las siguientes alternativas estimula las ondas de motilidad gastrointestinal
denominadas complejos migratorios interdigestivos?
● Eritromicina
● Colecistoquinina (CCK)
● Alimentos
● Metoclopramida
Pregunta 24
Con respecto a la regulación del peristaltismo, al aplicarle atropina (antagonista colinérgico)
a un paciente, es de esperarse que el peristaltismo:
● disminuya
● estimule la acción de los receptores dopaminérgicos
● se mantenga sin alteración
● aumente
Pregunta 25
¿Cuál de las siguientes alternativas es correcta sobre el movimiento peristáltico?
● Es independiente del plexo mientérico
● Es un reflejo largo que depende de la integración con el tronco encefálico
● El contenido intestinal avanza sólo 5-10 cm
● Se dirige en sentido distal siempre, nunca en sentido proximal

¿Cuál de las siguientes alternativas es una característica de la estructura del esófago?

- Contiene glándulas submucosas principalmente en su tercio distal

- Aumenta la presión intra esofágica durante la inspiración

- El esfínter esofágico inferior es un esfínter anatómico

Las siguientes hormonas disminuyen el vaciamiento gástrico, EXCEPTO:

- Gastrina

- Péptido insulinotrópico dependiente de glucosa

- Secretina

- Colecistoquinina

Luego de una comida rica en proteínas, se espera que los valores de secretina en sangre:

- sólo se altere si hay un aumento de colecistoquinina (CCK)

- se mantenga sin cambio

- aumenten

- disminuyan

¿Cuál de las siguientes estructuras tiene inervación somática?

- Peritoneo parietal

- Mesosigmoides

- Peritoneo visceraL

- Estómago
Estas diseñando un proyecto de investigación sobre los niveles de colesterol que se
absorben luego de una comida grasosa y deseas cuantificar la cantidad de colesterol que es
absorbido por el intestino antes que el hígado lo metabolice ¿de cuál de los siguientes vasos
obtendrías la muestra para tu análisis?

- Vena porta

- Vena hemiácigos accesoria

- Vena cava superior

- Conducto torácico

Durante el paso del bolo hacia la orofaringe, se desencadena una serie de contracciones
musculares que estrechan la cavidad faríngea. Estas contracciones están mediadas por el
nervio craneal:

- X

- XII

- IX

- XI

Las siguientes alternativas son factores que determinan la patencia y función adecuada del
esfínter esofágico inferior, EXCEPTO:

- Hipertrofia de la circular interna

- Angulación con el estómago

- Canales lentos de calcio

- Canales lentos de calcio

Cuando el istmo de las fauces se cierra, se evita que el alimento pase hacia la orofarínge y
permite respirar mientras se mastica. Este cierre se debe a la contracción y aproximación de
los músculos:

- palatofaríngeos

- estilofaríngeos

- estiloglosos

- Palatogloso

En la estructura dentaria, se observa que hay una composición muy similar a la del hueso
en la capa denominada:

- dentina
- predentina

- cemento

- esmalte

El frenillo de los labios se encuentra en:

- el dorso de la lengua

- la cavidad vestibular

- el piso de la boca

- la cavidad oral

Es un derivado del mesenterio dorsal:

En un paciente con falla en la fusión de los conductos de las yemas ventral y dorsal del
páncreas, usted esperaría encontrar:

- drenaje de la mayor parte del jugo pancreático en la papila menor

- drenaje adecuado del jugo pancreático

- estenosis del duodeno

- drenaje de la mayor parte del jugo pancreático en la papila mayor

Mujer de 30 años llega a emergencia con dolor en hipogástrico. Al examen físico presenta
una masa palpable de 10 cm de diámetro a en la misma región, usted sospecharía de las
siguientes condiciones, EXCEPTO:

- Tumor renal

- Tumor uterino

- Cáncer de recto superior

- Embarazo

El principal gobernante sobre todos los movimientos gastrointestinales es el sistema


nervioso:

- Mientérico
- Simpático

- Parasimpático

- Somático

La distención del yeyuno provoca que se:

- despolarice el potencial de reposo de membrana

- disminuya el número de espigas

- produzca una contracción tónica

- aumente la frecuencia de las ondas lentas

La información sensitiva aferente del sistema gastrointestinal pasa por las siguientes
estructuras, EXCEPTO:

- Tronco encefálico

- Ganglios prevertebrales

- Médula espinal

- Tálamo

¿Cuál de las siguientes alternativas es correcta sobre la motilidad esofágica?

- Las ondas secundarias son propulsoras y no van precedidas de deglución

- Las ondas primarias no son propulsoras y siempre van precedidas de deglución

- Las ondas primarias son propulsoras y pueden no ser precedidas por deglución

- Las ondas secundarias son propulsoras y siempre van precedidas de deglución

¿Cuál de las siguientes alternativas es correcta sobre el control autónomo del aparato
gastrointestinal?

- La estimulación simpática estimula a la muscularis mucosae

- Las terminaciones nerviosas simpáticas liberan mayor cantidad de adrenalina que


noradrenalina

- Al seccionar el vago, la inervación parasimpática del colon sigmoides disminuye

- El plexo mientérico de Auerbach cumple funciones inhibitorias

El peristaltismo intestinal se produce gracias a un reflejo que:

● - llega al sistema nervioso central


● - se origina dentro de la pared intestinal
● - llega a los ganglios prevertebrales
● - llega a la médula espinal

¿Cuál de las siguientes alternativas estimula las ondas de motilidad gastrointestinal


denominadas complejos migratorios interdigestivos?

- Metoclopramida

- Colecistoquinina (CCK)

- Eritromicina

- Alimentos

El peristaltismo depende que a nivel distal del bolo se secrete:

- acetilcolina por las neuronas provenientes del nervio vago

- óxido nítrico por células endoteliales locales

- péptido intestinal vasoactivo por neuronas

- noradrenalina secretada por las fibras del sistema simpático

Sobre el control de la peristalsis del tubo digestivo, ________________ es un mediador


neural que induce la relajación durante la peristalsis.

- el péptido intestinal vasoactivo

- la somatostatina

- la serotonina

- la acetilcolina

En el esófago, el plexo mientérico o de Auerbach interviene en el _______ de los impulsos


______

- inicio / eferentes

- fin / eferentes

- inicio / aferentes

- fin / aferentes

Al rozar agua caliente en la punta de la lengua, usted esperaría que el estímulo viaje a
través del nervio:

- cuerda del tímpano

- lingual
- hipogloso

- glosofaríngeo

¿Cuál de las siguientes alternativas detallan los músculos que ayudan a empujar el bolo
hacia la orofaringe?

- Estilogloso y palatogloso

- Estilogloso y geniogloso

- Hiogloso y geniogloso

- Geniogloso y palatofarínge
CI3 - Francisco

Los pliegues gástricos gruesos son prácticamente inexistentes a nivel de:

el fondo

En un paciente con xerostomía presenta las siguientes condiciones, EXCEPTO:

Infecciones del oído a repetición

En fases iniciales, la infección por Helicobacter pylori genera disminución del pH gástrico
debido a:

la disminución de somatostatina

La fase intestinal de la secreción gástrica se debe básicamente a la participación de las


células:

G del duodeno

Los siguientes mecanismos determinan la patencia del esfínter esofágico inferior y ayudan a
impedir la enfermedad por reflujo gastroesofágico, EXCEPTO:

Longitud mayor del esófago

A raíz de la infección por Helicobacter pilory, la destrucción de las células ____________


ocasiona que aumente la producción de ácido en el estómago y por ello se produzca úlceras
____________

D / duodenales

En caso se produzca la perforación de la cara anterior del estómago, esta perforación


ocasionará _________ producto de la peritonitis química.

íleo intestinal

Al usar un parasimpaticomimético (agonista colinérgico), usted espera que la saliva


presente:

una mayor cantidad de sodio

La saliva siempre será hipotónica debido a:

la impermeabilidad de los conductos al agua

En la producción de HCl, la acción de la somatostatina disminuye la accion de:

la gastrina
CI3 DE DIGESTIVO (LES)

En una gestante de 11 semanas con antecedente de esofagitis eosinofílica, con hiperémesis


gravídica, que acude a emergencia por hematemesis leve, y presenta súbitamente disnea y
dolor torácico. Usted sospecharía de:

a) síndrome de Mallory Weiss


b) neumonía
c) esofagitis erosiva severa por reflujo
d) síndrome de Boerhaav

La fase intestinal de la secreción gástrica se debe básicamente a la participación de las


células:

a) G del duodeno
b) I del yeyuno
c) D del estómago
d) S del íleon

La vena porta se forma gracias a la unión de la vena mesentérica superior con la vena:

a) esplénica
b) mesentérica inferior
c) celiaca
d) gástrica izquierda

Aquellas células que producen mayor cantidad de moco en el epitelio gástrico son las:

a) mucosas superficiales
b) enterocromafines
c) mucosas del cuello
d) mucosas del fondo

La saliva siempre será hipotónica debido a:

a) la mayor permeabilidad al sodio en los conductos


b) el estímulo del sistema parasimpática
c) la impermeabilidad de los conductos al agua
d) la ausencia de conductor estriado

Para determinar que un paciente tiene esófago de Barrett, debemos encontrar ___________
en la biopsia de esófago.

a) células caliciformes
b) displasia
c) epitelio gástrico
d) glándulas cardiales

Paciente con acalasia es sometido a tratamiento endoscópico o quirúrgico, usted le ha


informado al paciente previamente que es posible que una complicación de este tratamiento
es que quede con cierto grado de:

a) reflujo gastroesofágico
b) úlceras gástricas
c) odinofagia
d) gastritis

¿Cuál de las siguientes alternativas es una causa de reflujo gastroesofágico de contenido


ácido?

a) Acalasia
b) Divertículo de Zenker
c) Hernia Hiatal
d) Anillo de Schatzki

En caso se produzca la perforación de la cara anterior del estómago, esta perforación


ocasionará _________ producto de la peritonitis química.

a) mayor secreción de colecistoquinina


b) íleo intestinal
c) esófago de Barret
d) mayor secreción de colecistoquinina
e) úlcera duodenal

La sangre que lleva la vena porta es tipo:

a) venosa
b) mixta
c) arterial

1. Al evaluar a un paciente con parotiditis purulenta (con absceso), usted buscaría el orificio
terminal del conducto de _____________ a la altura de _____________ → Stenon / la
segunda molar superior

2. Para bloquear la principal fuerza promotora de producción de ácido clorhídrico, lo ideal es un


medicamento que sea bloqueador de la bomba: de hidrógeno-potasio

3. En fases iniciales, la infección por Helicobacter pylori genera disminución del pH gástrico debido a →
la disminución de somatostatina
4. La comunicación entre la irrigación gástrica y la esofágica depende de una rama de la
arteria:tronco celiaco
5. Los pliegues gástricos gruesos son prácticamente inexistentes a nivel de → el fondo
6. En pacientes con hernia hiatal, la constante inflamación del esófago, puede aparecer
_______________ como complicación debida a la fibrosis por inflamación crónica. → anillo
de Schatzki
7. En un paciente con xerostomía presenta las siguientes condiciones, EXCEPTO:
Infecciones del oído a repetición

ANGELA CI-1:
1) Paciente de 24 años con dolor abdominal tipo cólico intenso en mesogastrio. Según
sus conocimientos de macroestructura, el origen del dolor puede ser el
___________:
Respuestas:
a) Colon
b) Íleon
c) Esófago
d) Estómago

2) Paciente se queja de dolor en hipocondrio derecho, pero superficialmente. El dermatoma


relacionado es (marque la mejor respuesta):
Respuestas:
a) T9
b) T11
c) T12
d) T10

3) Señale la respuesta correcta:


Respuestas:
a) El esófago sólo tiene adventicia
b) El colon ascendente sólo tiene serosa
c) El apéndice cecal sólo tiene serosa
d) El páncreas sólo tiene adventicia

4) Cuál de las siguientes estructuras no tiene vasos sanguíneos:


Respuestas:
a) Ligamento
b) Mesenterio
c) Epitelio intestinal
d) Omento

5) Paciente tiene una úlcera sangrante en el segundo tercio del Yeyuno. La arteria de la cual
proviene la sangre arterial para dicha zona es la arteria:
Respuestas:
a) Tronco celíaco
b) Mesentérica inferior

c) Mesentérica superior
d) Gástrica izquierda
e) Iliaca común

6) Al retirar completamente el mesenterio de un órgano, el mismo se vería afectado


principalmente en su:
Respuestas:
a) Inervación
b) Irrigación
c) Tamaño
d) No se afecta en absoluto

7) Respecto a la anatomía del estómago, marque lo correcto:


Respuestas:
a) El píloro se encuentra en el cuerpo gástrico
b) La incisura angularis puede estar en la curvatura mayor.
c) La porción más distal del estómago es el cardias
d) El fondo gástrico forma la curvatura mayor

8) Al examinar a un paciente, usted encuentra dolor localizado en fosa iliaca derecha y


diagnostica apendicitis. En este paciente, usted puede inferir:
Respuestas:
a) Hay inflamación de todo el peritoneo parietal (peritonitis)
b) El peritoneo parietal regional está afectado
c) El peritoneo visceral regional está principalmente afectado
d) El diagnóstico está errado por no corresponder a la región abdominal adecuada

9) Marque el órgano que se considera retroperitoneal:


Respuestas:
a) Lóbulo izquierdo del hígado
b) Vesícula biliar
c) Parte de la vía biliar
d) Sigmoides

10) La peristalsis o peristaltismo hace referencia a:


Respuestas:
a) Motilidad para mezclado de alimentos.
b) No es parte de la motilidad
c) Motilidad para fraccionamiento de alimentos.
d) Motilidad para movilizar el alimento de proximal a distal.

ANGELA CI-3:

1. Al usar atropina en una paciente, usted esperaría:


a) la disminución de gastrina
b) la disminución de secreción gástrica por bloqueo de M3
c) la hipersalivación
d) el aumento de histamina

2. Para determinar que un paciente tiene esófago de Barret, debemos encontrar


______________ en la biopsia de esófago.
a) células caliciformes
b) displasia
c) glándulas cardiales
d) epitelio gástrico

3. Paciente con acalasia es sometido a tratamiento endoscópico o quirurgico, usted le


ha informado al paciente previamente que es posible que una complicación de este
tratamiento es que quede con cierto grado de:
a) reflujo gastroesofágico
b) gastritis
c) odinofagia
d) úlceras gástricas

4. En un paciente con xerostomía presenta las siguientes condiciones, EXCEPTO:


a) Caries
b) Infecciones del oído a repetición
c) Disfagia
d) Erosiones de la mucosa oral

5. A raíz de la infección por Helicobacter pilory, la destrucción de las células


____________ ocasiona que aumente la producción de ácido en el estómago y por
eso se produzca úlceras _____________
a) S / pancreáticas
b) D / duodenales
c) parietales / gástricas
d) G / gástricas

6. Cuando un paciente recibe estímulo autónomo mixto (simpático y parasimpático), el


flujo de saliva:
a) es mayor que frente a un estímulo parasimpático aislado
b) aumenta en relación al basal
c) es menor que frente a un estímulo simpático aislado
d) disminuye en relación al basal

7. El conducto de Stenon, para entrar a la cavidad vestibular, debe atravesar el


músculo:

a) genihioideo
b) buccinador
c) masetero
d) milohioideo

8. Al evaluar a un paciente con parotiditis purulenta (con absceso), usted buscaría el


orificio terminal del conducto de _____________ a la altura de _____________

a) Stenon / la segunda molar superior


b) Stenon / las carúnculas
c) Wharton / el frenillo sublingual
d) Wharton / el piso de la boca

9. Los siguientes mecanismos determinan la patencia del esfínter esofágico inferior y


ayudan a impedir la enfermedad por reflujo gastroesofágico, EXCEPTO:

a) Canales lentos de calcio


b) Plicatura diafragmática
c) Ángulo de Hiss
d) Longitud mayor del esófago
10. La sangre que lleva la vena porta es tipo:

a) venosa
b) mixta
c) arterial
ANGELA- CI4:

1) Con respecto a la microestructura del hígado ¿Cuál de las siguientes alternativas es


correcta?

Respuestas:

a) Los hepatocitos están interconectados por uniones herméticas


b) La célula de Kupffer se encuentra fuera del sinusoide y fagocita células
c) La célula de Ito se encuentra en el espacio de Disse y reserva glucógeno
d) El sinusoide es un capilar fenestrado

2) La pancreatitis aguda puede producirse debido a una obstrucción en el flujo del conducto
de Wirsung, debido a un cálculo biliar que obstruye:

Respuestas:

a) el conducto de Wirsung
b) la ampolla de Vater
c) la papila menor
d) el tercio medio del colédoco

3) En un paciente con una estenosis severa del colédoco por una complicación quirúrgica,
usted esperaría que desarrolle:

Respuestas:

a) el aumento de transaminasas mayor que el de fosfatasa alcalina


b) hiperbilirrubinemia no conjugada
c) la cabeza de Medusa
d) el aumento del tiempo de protrombina

4) Marque lo correcto respecto al acino hepático:

Respuestas:

a) La zona 1 es la más que recibe más sangre


b) La zona 3 se afecta en menos en una deshidratación severa
c) Dos vértices del rombo acinar se constituyen por triadas portales
d) El eje menor lo constituye un eje imaginario entre dos venas centrolobulillares

5) En un paciente cirrótico con encefalopatía hepática ¿cuál de las siguientes alternativas


sustenta la reducción de carnes rojas en la dieta?

Respuestas:

a) Hasta el 80% del colesterol se transforma en sales biliares


b) El factor de crecimiento hepatocitario fomenta la regeneración del hepatocito
c) El amonio se produce principalmente en el intestino
d) La fructosa y la galactosa se convierten en glucosa en el hígado
6) La relajación del esfínter de Oddi se produce directamente por acción de:

Respuestas:

a) la acetilcolina
b) la colecistoquinina (CCK)
c) la sustancia P
d) el péptido vasoactivo intestinal (VIP)

7) Para aumentar la cantidad de bicarbonato en el flujo pancreático es necesario:

Respuestas:

a) inhibir al nervio vago


b) estimular las células S
c) ingerir aminoácidos como triptófano o fenilalanina
d) estimular la secreción de péptido liberador de colecistoquinina (CCK)

8) La bilis que sale de la vesícula biliar tiene como componente principal:

Respuestas:

a) a la bilirrubina
b) al colesterol
c) a los fosfolípidos
d) a los ácidos biliares

9) La presión parcial de oxígeno en la Zona 1 del sinusoide hepático de debe ser


___________ mmHg

Respuestas:

a) 100
b) entre 95 y 45
c) 40
d) menor de 40

10) Los hepatocitos tienen una gran capacidad regenerativa, en parte gracias a las células
madre hepáticas que se localizan en los:

Respuestas:

a) canales de Herring
b) sinusoides hepáticos
c) espacios de Mall
d) espacios de Disse
CI3 - José G

1. Para bloquear la principal fuerza promotora de producción de ácido clorhídrico, lo


ideal es un medicamento que sea bloqueador de la bomba:
a. de potasio
b. de hidrógeno-potasio
c. de cloro asociado a fibrosis quística
d. sodio-potasio

2. Al usar un parasimpaticomimético (agonista colinérgico), usted espera que la saliva


presente:
a. una menor cantidad de cloro
b. una menor cantidad de bicarbonatos
c. una mayor cantidad de sodio
d. Hipertonicidad

3. En la glándula fúndica, se evidencia células madre o stem cells a nivel de:


a. la foveola
b. el fondo
c. el istmo
d. el cuello

4. La vena porta se forma gracias a la unión de la vena mesentérica superior con la


vena:
a. mesentérica inferior
b. gástrica izquierda
c. esplénica
d. celiaca

5. En la producción de HCl, la acción de la somatostatina disminuye la accion de:


a. la gastrina
b. las prostaglandinas
c. la histamina
d. la acetilcolina

6. ¿Cuál de las siguientes alternativas es una causa de reflujo gastroesofágico de


contenido ácido?
a. Anillo de Schatzki
b. Diertículo de Zenker
c. Hernia Hiatal
d. Acalasia
7. En caso se produzca la perforación de la cara anterior del estómago, esta
perforación ocasionará _________ producto de la peritonitis química.
a. íleo intestinal
b. mayor secreción de colecistoquinina
c. esófago de Barret
d. úlcera duodenal

8. Dentro de las patologías que producen sangrado en el esófago, la que sangra más
es:
a. la esofagitis por reflujo gastroesofágico
b. el divertículo de Zenker
c. el síndrome de Mallory Weiss
d. el síndrome de Boerhaave
9. La fase intestinal de la secreción gástrica se debe básicamente a la participación de
las células:
a. I del yeyuno
b. S del íleon
c. G del duodeno
d. D del estómago

10. Las células enteroendocrinas en el estómago se localizan en la glándula oxíntica, al


mismo nivel que las células:
a. parietales
b. absortivas
c. principales
d. mucosas

CI 3: io

En las glándulas salivales, el principal lugar de intercambio iónico se da a nivel del:

A. conducto excretor
B. conducto estriado
C. conducto intercalar
D. acino

Al usar atropina en un paciente, usted esperaría:

A. la disminución de gastrina
B. la disminución de secreción gástrica por bloqueo de M3
C. el aumento de histamina
D. la hipersalivación
En un paciente con acalasia, se presentará una disminución de ___________________ en
el esfínter esofágico inferior.

A. acetilcolina
B. sustancia P
C. péptido intestinal vasoactivo
D. colecistoquinina

En cuanto a las sustancias secretadas por el estómago ¿Cuál de las siguientes sustancias estimula la
liberación de pepsinógeno?

A. Pepsinógeno
B. Secretina
C. Colecistoqunina
D. Gastrina

Las células enteroendocrinas en el estómago se localizan en la glándula oxíntica, al mismo nivel que
las células

A. parietales
B. principales
C. mucosas
D. absortivas

La fase intestinal de la secreción gástrica se debe básicamente a la participación de las


células

A. G del duodeno
B. I del yeyuno
C. S del íleon
D. D del estómago

El conducto de Stenon, para entrar a la cavidad vestibular, debe atravesar el músculo:

A. genihioideo
B. buccinador
C. milohioideo
D. masetero

Paciente con acalasia es sometido a tratamiento endoscópico o quirúrgico, usted le ha


informado al paciente previamente que es posible que una complicación de este tratamiento
es que quede con cierto grado de:

A. reflujo gastroesofágico
B. gastritis
C. odinofagia
D. úlceras gástricas

La sangre que lleva la vena porta es tipo:


A. mixta
B. arterial
C. venosa

Los pliegues gástricos gruesos son prácticamente inexistentes a nivel de:

A. el cuerpo
B. el fondo
C. la incisura angularis
D. el antro

En la producción de HCl, la acción de la somatostatina disminuye la accion de:

A. las prostaglandinas
B. la histamina
C. la acetilcolina
D. la gastrina

En el estómago se secretan las siguientes sustancias, EXCEPTO:

A. grelina
B. motilina
C. somatostatina
D. gastrina

La vena porta se forma gracias a la unión de la vena mesentérica superior con la vena:

A. mesentérica inferior
B. esplénica
C. celiaca
D. gástrica izquierda

CI3 - PEQUE

La saliva siempre será hipotónica debido a:

A. la mayor permeabilidad al sodio en los conductos


B. el estímulo del sistema parasimpático
C. la impermeabilidad de los conductos al agua
D. la ausencia de conductor estriado

En la producción de HCl, la acción de la somatostatina disminuye la acción de:

A. la histamina
B. las prostaglandinas
C. la gastrina
D. la acetilcolina

Un paciente con polimiositis posee alteración en la regulación del mecanismo de la


deglución; por eso hay que considerar la deglución de la saliva, pues a diario se produce
_______mL
A. 300
B. 500
C. 100
D. 1000

Al evaluar a un paciente con parotiditis purulenta (con absceso), usted buscaría el orificio
terminal del conducto de _____________ a la altura de _____________

A. Wharton / el piso de la boca


B. Stenon / la segunda molar superior
C. Wharton / el frenillo sublingual
D. Stenon / las carúnculas

En un paciente con acalasia, se presentará una disminución de ___________________ en


el esfínter esofágico inferior.

A. sustancia P
B. acetilcolina
C. colecistoquinina
D. péptido intestinal vasoactivo

En pacientes con hernia hiatal, la constante inflamación del esófago, puede aparecer
_______________ como complicación debida a la fibrosis por inflamación crónica.

A. anillo de Schatzki
B. esófago de Barret
C. divertículo de Zenker
D. reflujo gastroesofágico

Los pliegues gástricos gruesos son prácticamente inexistentes a nivel de:

A. el cuerpo
B. la incisura angularis
C. el fondo
D. el antro

Paciente con acalasia es sometido a tratamiento endoscópico o quirúrgico, usted le ha


informado al paciente previamente que es posible que una complicación de este tratamiento
es que quede con cierto grado de:

A. gastritis
B. úlceras gástricas
C. odinofagia
D. reflujo gastroesofágico

¿Cuál de las siguientes alternativas es una causa de reflujo gastroesofágico de contenido


ácido?

A. Acalasia
B. Diertículo de Zenke
C. Anillo de Schatzki
D. Hernia Hiatal

En las glándulas salivales, el principal lugar de intercambio iónico se da a nivel del:

A. conducto intercalar
B. conducto excretor
C. conducto estriado
D. acino

CI4 - Francisco

En un paciente con cirrosis hepática avanzada se encuentra tendencia a:

la hipoglicemia

En un paciente con infección por SARS-CoV-2 con compromiso severo e ingresado en la


unidad de cuidados intensivos (UCI), debido a la tormenta de citoquinas y a la desregulación
inmune ¿Cuál de las siguientes alternativas estará elevada en sangre al evaluar el perfil
hepático?

transaminasas

Para poder absorber al torrente sanguíneo todos los carbohidratos del azúcar común (de
mesa) es necesario usar únicamente los transportadores:

GLUT2, SGLT1, GLUT5

En la regulación de la glucosa sérica participa el hígado ¿Cuál de las siguientes alternativas


evalúa mejor esta función hepática?

Glucosa sérica en ayunas

En un paciente con intoxicación por órganos fosforados, la acción de la colecistoquinina


(CCK) está bloqueada a nivel de:

el esfínter de Oddi

En el hígado, el aumento de la resistencia vascular en los sinusoides hepáticos ocasionará:

salida de plasma hacia el intersticio

En un paciente cirrótico con encefalopatía hepática ¿cuál de las siguientes alternativas


sustenta la reducción de carnes rojas en la dieta?

El amonio se produce principalmente en el intestino

¿Cuál de las siguientes alternativas es correcta sobre la estructura hepática?

El flujo biliar en el lobulillo hepático es centrífugo

Las siguientes sustancias son secretadas por el páncreas, EXCEPTO:

Tripsina
Un paso importante para la poder absorber los lípidos es la emulsificación de las grasas.
¿Cuál de las siguientes alternativas es el principal factor encargado de dicha emulsificación?

Ácidos biliares

CI 4 : Clau A.

● Para aumentar la cantidad de bicarbonato en el flujo pancreático es necesario:

estimular las células S

ingerir aminoácidos como triptófano o fenilalanina

inhibir al nervio vago

estimular la secreción de péptido liberador de colecistoquinina (CCK)

● Marque la respuesta correcta:

El hígado recibe principalmente sangre venosa

Funcionalmente, contiene dos lóbulos.

Tiene una función endocrina pero no exocrina

Pesa aproximadamente 2,5 kg

● En un paciente con intoxicación por órganos fosforados, la acción de la


colecistoquinina (CCK) está bloqueada a nivel de:

la vesícula biliar

la célula parietal

el esfínter de Oddi

el sistema nervioso central

● En el síndrome de Mirizzi, el paciente tiene cálculos en la vesícula biliar; pero se


obstruye el conducto hepático común debido a que un cálculo se ubica y crece de
tamaño en

el colédoco

el conducto accesorio de Lushka

la válvula espiral de Heister

la bolsa de Hartmann

● ¿Cuál de las siguientes alternativas es correcta sobre la estructura hepática?

El flujo biliar en el lobulillo hepático es centrífugo

Los colangiocitos producen bilis


En la triada portal, se encuentra la vena derivada de la suprahepática

El flujo sinusoidal en el lobulillo hepático es de adentro hacia afuera

● En un recién nacido menor de 24 horas con atresia biliar, se encuentra elevación de la:

bilirrubina directa

alanina aminotransferasa (ALT)

bilirrubina indirecta

hemoglobina

● Un paso importante para la poder absorber los lípidos es la emulsificación de las


grasas. ¿Cuál de las siguientes alternativas es el principal factor encargado de dicha
emulsificación?

Fosfolípidos

Colesterol

Ácidos biliares

Bilirrubina

● En un paciente cirrótico con encefalopatía hepática ¿cuál de las siguientes alternativas


sustenta la reducción de carnes rojas en la dieta?

La fructosa y la galactosa se convierten en glucosa en el hígado

El factor de crecimiento hepatocitario fomenta la regeneración del hepatocito

El amonio se produce principalmente en el intestino

Hasta el 80% del colesterol se transforma en sales biliares

● En un paciente con cirrosis hepática la cabeza de medusa que aparece en la pared


abdominal, podría desaparecer si al paciente se le:

esclerosa las venas hemorroides internas

oblitera el ligamento redondo

administra antiandrógenos

oblitera la arteria gástrica izquierda

● Paciente con Lupus Eritematosos que desarrolla hipertensión portal debido a


trombosis portal, es probable que desarrolle várices a nivel de:

recto superior

hemorroides externas

recto inferior
canal anal

CI 4: Sifu

La presión parcial de oxígeno en la Zona 1 del sinusoide hepático de debe ser


___________ mmHg

● entre 95 y 45
● 40
● 100
● menor de 40

La bilis que sale de la vesícula biliar tiene como componente principal:

● a los ácidos biliares


● al colesterol
● a los fosfolípidos
● a la bilirrubina

En un paciente con carcinoma de páncreas, el tumor ha invadido la unión entre la


venas esplénica y mesentérica superior; eso quiere decir que estamos seguros que
el tumor se encuentra a nivel del ________ del páncreas.

● cola
● cabeza
● cuello
● cuerpo

En un paciente con una estenosis severa del colédoco por una complicación
quirúrgica, usted esperaría que desarrolle:

Respuestas:

● el aumento de transaminasas mayor que el de fosfatasa alcalina


● el aumento del tiempo de protrombina
● la cabeza de Medusa
● hiperbilirrubinemia no conjugada

Para aumentar la cantidad de bicarbonato en el flujo pancreático es necesario:

estimular las células S

ingerir aminoácidos como triptófano o fenilalanina

inhibir al nervio vago


estimular la secreción de péptido liberador de colecistoquinina (CCK)

Con respecto a la microestructura del hígado ¿Cuál de las siguientes alternativas es


correcta?

La célula de Kupffer se encuentra fuera del sinusoide y fagocita células

Los hepatocitos están interconectados por uniones hermética

El sinusoide es un capilar fenestrado

La célula de Ito se encuentra en el espacio de Disse y reserva glucógeno

En el hígado, el aumento de la resistencia vascular en los sinusoides hepáticos


ocasionará:

aumento del flujo hacia la vena porta

aumento de la presión de llenado vesicular

aumento del flujo hacia la vena cava superior

salida de plasma hacia el intersticio

En el hígado, el aumento de la resistencia vascular en los sinusoides hepáticos


ocasionará:

● salida de plasma hacia el intersticio


● aumento del flujo hacia la vena porta
● aumento de la presión de llenado vesicular
● aumento del flujo hacia la vena cava superior

En un recién nacido menor de 24 horas con atresia biliar, se encuentra elevación de


la:

● bilirrubina directa
● hemoglobina
● bilirrubina indirecta
● alanina aminotransferasa (ALT)

En un paciente con intoxicación por órganos fosforados, la acción de la


colecistoquinina (CCK) está bloqueada a nivel de:
● el esfínter de Oddi
● la célula parietal
● la vesícula biliar
● el sistema nervioso central

¿Cuál de las siguientes alternativas es una comunicación entre el tejido hepático y la


vesícula biliar?

Válvula espiral de Heister

Conducto de Lushka

Senos de Rokitansky-Aschoff

Divertículos vesiculares

CI4 - Francisco

En un paciente con cirrosis hepática avanzada se encuentra tendencia a:

● la disminución de bilirrubina hidrosoluble


● la hiperglucemia
● la hipoglucemia
● el acortamiento del tiempo de protrombina

En un paciente con infección por SARS-CoV-2 con compromiso severo e ingresado


en la unidad de cuidados intensivos (UCI), debido a la tormenta de citoquinas y a la
desregulación inmune ¿Cuál de las siguientes alternativas estará elevada en sangre
al evaluar el perfil hepático?

● transaminasas
● amilasas
● bilirrubinas
● fosfatasas

Para poder absorber al torrente sanguíneo todos los carbohidratos del azúcar común
(de mesa) es necesario usar únicamente los transportadores:

● GLUT5, SGLT1
● GLUT2, GLUT5
● SGLT1, GLUT2
● GLUT2, SGLT1, GLUT5

En la regulación de la glucosa sérica participa el hígado ¿Cuál de las siguientes


alternativas evalúa mejor esta función hepática?
● Péptido C en sangre
● Glucosa sérica en ayunas
● Glucosa en orina
● Tolerancia oral a la glucosa

En un paciente cirrótico con encefalopatía hepática ¿cuál de las siguientes


alternativas sustenta la reducción de carnes rojas en la dieta?

● La fructosa y la galactosa se convierten en glucosa en el hígado


● Hasta el 80% del colesterol se transforma en sales biliares
● El factor de crecimiento hepatocitario fomenta la regeneración del hepatocito
● El amonio se produce principalmente en el intestino

¿Cuál de las siguientes alternativas es correcta sobre la estructura hepática?

● El flujo biliar en el lobulillo hepático es centrífugo


● Los colangiocitos producen bilis
● El flujo sinusoidal en el lobulillo hepático es de adentro hacia afuera
● En la triada portal, se encuentra la vena derivada de la suprahepática

Las siguientes sustancias son secretadas por el páncreas, EXCEPTO:

● Lipasa
● Péptido Monitor
● Nucleasas
● Tripsina

Un paso importante para la poder absorber los lípidos es la emulsificación de las


grasas. ¿Cuál de las siguientes alternativas es el principal factor encargado de dicha
emulsificación?

Colesterol

Fosfolípidos

Ácidos biliares

Bilirrubina

CI4 - HEYDY
1. Un paso importante para la poder absorber los lípidos es la emulsificación de las
grasas. ¿Cuál de las siguientes alternativas es el principal factor encargado de dicha
emulsificación?
a. Ácidos biliares
b. bilirrubina
c. colesterol
d. fosfolípidos
e. Ácidos biliares
2. En un paciente con cirrosis hepática la cabeza de medusa que aparece en la pared
abdominal, podría desaparecer si al paciente se le:
a. oblitera la arteria gástrica izquierda
b. administra antiandrógenos
c. oblitera la arteria gástrica izquierda
d. oblitera el ligamento redondo
e. esclerosa las venas hemorroides internas
3. La bilis que sale de la vesícula biliar tiene como componente principal:
a. a los ácidos biliares
b. a la bilirrubina
c. al colesterol
d. a los fosfolípidos
4. Los hepatocitos tienen una gran capacidad regenerativa, en parte gracias a las células
madre hepáticas que se localizan en los:
a. espacios de Disse
b. canales de Herring
c. sinusoides hepáticos
d. espacios de mall
5. Las siguientes sustancias son secretadas por el páncreas, EXCEPTO:
a. tripsina
b. péptido monitor
c. lipasa
d. nucleasa
6. Marque lo correcto respecto al acino hepático:
a. La zona 1 es la más que recibe más sangre
b. El eje menor lo constituye un eje imaginario entre dos venas centrolobulillares
c. Dos vértices del rombo acinar se constituyen por triadas portales
d. La zona 1 es la más que recibe más sangre
e. La zona 3 se afecta en menos en una deshidratación severa
7. La vía biliar extrahepática se encuentra ubicada dentro del ligamento:
a. Hepatoduodenal
b. Falciforme
c. Hepatogástrico
d. Coronario anterior
8. En un paciente con cáncer de páncreas y que desarrolla ictericia, la localización más
probable del tumor es en:
a. el cuello del páncreas
b. la cola del páncreas
c. la cabeza del páncreas
d. la vesícula por metástasis
9. El efecto de un medicamento colerético se evidencia por:
a. el aumento de secreción biliar
b. la mayor producción de colesterol en la bilis
c. el aumento de formación de micelas
d. la disminución de absorción de sales biliares
10. La presión parcial de oxígeno en la Zona 1 del sinusoide hepático de debe ser
___________ mmHg
a. entre 95 y 45
b. 40
c. 100
d. menor de 40

En un paciente con una estenosis severa del colédoco por una complicación quirúrgica, usted
esperaría que desarrolle --- aumento de tiempo de protrombina

La vía biliar extrahepática se encuentra ubicada dentro del ligamento --- hepatoduodenal

Un recién nacido con enfermedad congénita, tiene una mutación de la proteína MRP2 encargada del
transporte de la bilirrubina conjugada hacia el interior del canalículo. Antes de las 24 horas de nacido
presenta ictericia y además presentará con mayor probabilidad ----- coluria

La bilis que sale de la vesícula biliar tiene como componente principal ---- a los ácidos biliares

Para aumentar la cantidad de bicarbonato en el flujo pancreático es necesario ----- estimular las
células s

Los hepatocitos tienen una gran capacidad regenerativa, en parte gracias a las células madre
hepáticas que se localizan en los ---- canales de Herring

En el síndrome de Mirizzi, el paciente tiene cálculos en la vesícula biliar; pero se obstruye el conducto
hepático común debido a que un cálculo se ubica y crece de tamaño en ----- La bolsa de Hartmann

¿Cuál de las siguientes alternativas es correcta sobre la estructura hepática? ---- El flujo biliar en el
lobulillo hepático es centrífugo

La relajación del esfínter de Oddi se produce directamente por acción de ----- VIP

En la anemia perniciosa, con atrofia gástrica marcada, usted esperaría encontrar disminución en la
absorción de ___________ a nivel de ________ ----- vitamina B12 / íleon distal

Dentro de las patologías que producen sangrado en el esófago, la que sangra más es ----- el
síndrome de Boerhaave

En la glándula fúndica, se evidencia células madre o stem cells a nivel de ----- el istmo

Al usar atropina en un paciente, usted esperaría --- la disminución de secreción gástrica por bloqueo
de M3

La comunicación entre la irrigación gástrica y la esofágica depende de una rama de la arteria ----
Tronco celíaco

En fases iniciales, la infección por Helicobacter pylori genera disminución del pH gástrico debido a ----
- la disminución de somatostatina

En un paciente que usa AINEs a altas dosis por artritis reumatoide, usted esperaría que presente
erosiones y úlceras gástricas debido a ----- la disminución de irrigación en la mucosa

A raíz de la infección por Helicobacter pilory, la destrucción de las células ____________ ocasiona
que aumente la producción de ácido en el estómago y por ello se produzca úlceras ____________ ---
-- D/duodenales
1. El agua se absorbe en yeyuno, íleon y colon y se excreta en las
heces. Señale en orden descendente la cantidad de agua absorbida
o excretada en los siguientes elementos.
A) Colon, yeyuno, íleon, heces.
B) Heces, colon, íleon, yeyuno.
C) Yeyuno, íleon, colon, heces.
D) Colon, íleon, yeyuno, heces.
E) Heces, yeyuno, íleon, colon.

2. Después de un desastre natural en Haití, surgió un brote de cólera


en personas desplazadas que vivían en tiendas de campamentos.
Los sujetos afectados presentaron graves síntomas diarreicos:
¿por cuál de los siguientes cambios del transporte intestinal
aparecieron?
A) Aumentan el cotransporte de sodio-potasio en el intestino
delgado.
B) Incrementan la secreción de potasio en el colon.
C) Inhiben la absorción de potasio en las criptas de Lieberkühn.
D) Aumentan la absorción de sodio en el intestino delgado.
E) Incrementan la secreción de cloruro hacia la luz intestinal.

3. Un varón de 50 años acudió a su médico y le señaló que tenía


dolor epigástrico intenso, pirosis frecuentes y pérdida identifi cada
de 9.5 kg en un lapso de 6 meses. Según él, no tuvo alivio alguno
con productos que se adquirían sin receta como antihistamínicos
contra el receptor H2. Fue referido a un gastroenterólogo y en la
endoscopia gastroduodenal se identifi caron erosiones y úlceras en
la porción proximal del duodeno y una mayor producción de
ácido gástrico con el sujeto en ayunas. El paciente muy
probablemente tiene un tumor que secreta: ¿cuáles de las
hormonas siguientes?
A) Secretina.
B) Somatostatina.
C) Motilina.
D) Gastrina.
E) Colecistocinina.

4. ¿Cuál de los siguientes tiene el pH más alto?


A) Jugo gástrico.
B) Contenido de la luz colónica.
C) Jugo pancreático.
D) Saliva.
E) Contenido de las glándulas intestinales.

5. Una mujer de 60 años es sometida a pancreatectomía total, por la


presencia de una neoplasia. De los resultados siguiente: ¿cuál no se
esperaría después de que se recuperara de la operación?
A) Esteatorrea.
B) Hiperglucemia.
C) Acidosis metabólica.
D) Incremento de peso.
E) Menor absorción de aminoácidos.

1 La absorción máxima de ácidos grasos de cadena corta producidos


por las bacterias ocurre en:
A) estómago.
B) duodeno.
C) yeyuno.
D) íleon.
E) colon.

2. Una mujer premenopáusica físicamente activa solicitó orientación


de su médico familiar en cuanto a las medidas a tomar para
asegurar la disponibilidad adecuada de calcio de alimentos, para
afianzar su salud ósea en etapas ulteriores de la vida. De los
siguientes componentes de la alimentación: ¿cuál debe
incrementar la captación de calcio?
A) Proteínas.
B) Oxalatos.
C) Hierro.
D) Vitamina D.
E) Sodio.

3. Un niño que manifiesta ausencia congénita de enterocinasa


presentaría una disminución habitual en:
A) frecuencia de pancreatitis.
B) absorción de glucosa.
C) reabsorción de ácidos biliares.
D) pH gástrico.
E) asimilación de proteínas.

4. En la enfermedad de Hartnup (un defecto del transporte de


aminoácidos neutrales), los pacientes no presentan deficiencia de
estos aminoácidos gracias a la actividad de
A) PepT1.
B) peptidasas del borde en cepillo.
C) Na, K-ATPasa.
D) regulador de la conductancia transmembrana de la fibrosis
quística (CFTR).
E) tripsina.

5. Un recién nacido es llevado al pediatra a causa de diarrea grave


que se agrava al alimentarse. Los síntomas disminuyen cuando los
nutrientes se administran por vía intravenosa. Lo más probable es
que el niño tenga una mutación en cuál de los siguientes
transportadores intestinales:
A) Na, K-ATPasa.
B) NHE3.
C) SGLT1.
D) H+, K+-ATPasa.
E) NKCC1.

1. En los lactantes, la defecación suele ocurrir después de una comida.


La causa de las contracciones colónicas en esta situación es
A) la histamina.
B) el aumento de las concentraciones de CCK en la circulación
sanguínea.
C) el reflejo gastrocólico.
D) el aumento de las concentraciones de somatostatina en la
circulación sanguínea.
E) el reflejo enterogástrico.

2. Los síntomas del síndrome de vaciamiento rápido (molestia después


de las comidas en los pacientes con derivaciones intestinales como
la anastomosis gastroyeyunal) son causados en parte por:
A) aumento de la presión arterial.
B) aumento de la secreción de glucagon.
C) aumento de la secreción de CCK.
D) hipoglucemia.
E) hiperglucemia.

3. Las presiones gástricas raras veces aumentan por arriba de los


niveles que abren el esfínter esofágico inferior, aun cuando el estómago esté lleno de comida. ¿A
cuál de los siguientes procesos
se debe este fenómeno?
A) Peristaltismo.
B) Reflejo gastroileal.
C) Segmentación.
D) Estimulación del centro del vómito.
E) Relajación receptiva.

4. ¿Cuál de las siguientes sustancias detona el complejo motor


migratorio?
A) Motilina.
B) NO.
C) CCK.
D) Somatostatina.
E) Secretina.
5. Un paciente es referido al gastroenterólogo porque tiene dificultad
persistente para la deglución. En el estudio endoscópico se
advierte que no se abre totalmente el esfínter esofágico inferior
conforme el bolo llega a él, por lo que se hace el diagnóstico de
acalasia.
En la exploración o en las biopsias estudiadas de muestras de la
región esfinteriana: ¿en cuál de los elementos siguientes cabría
esperar disminución?
A) Peristaltismo esofágico.
B) Expresión de óxido nítrico sintasa neuronal.
C) Receptores de acetilcolina.
D) Liberación de sustancia P.
E) Contracción del diafragma crural.

1. Un paciente que sufre colitis ulcerosa grave es sometido a


colectomía total con elaboración de un estoma. Después de la
recuperación completa de la operación y al comparar su estado
posoperatorio con el que tenía antes de la operación: ¿cuál de los
siguientes factores cabría esperar que disminuya?
A) Capacidad de absorber lípidos.
B) Capacidad de coagular la sangre.
C) Concentraciones circulantes de ácidos biliares conjugados.
D) Urea en orina.
E) Urobilinógeno en orina.
2. Una cirujana estudia nuevos métodos de trasplante de hígado y
realiza una hepatectomía completa en un animal de
experimentación. Antes de injertar el hígado donado cabría
esperar incremento en la concentración sanguínea de:
A) glucosa.
B) fibrinógeno.
C) 25-hidroxicolecalciferol.
D) bilirrubina conjugada.
E) estrógenos.

3. ¿Cuál de los siguientes tipos de células protege contra la septicemia


consecutiva a la translocación de bacterias intestinales?
A) Célula estrellada hepática.
B) Colangiocito.
C) Célula de Kupffer.
D) Hepatocito.
E) Célula epitelial de la vesícula biliar.

4. El citocromo P450 (CYP) se expresa de manera extraordinaria en


los hepatocitos. ¿En cuál de las siguientes no desempeña una
función importante?
A) Formación de ácidos biliares.
B) Carcinogénesis.
C) Formación de hormonas esteroideas.
D) Desintoxicación de fármacos.
E) Síntesis de glucógeno.

5. Una mujer de 40 años acude con su médico familiar y le señala que


ha tenido dolor abdominal episódico e intenso, en particular
fuerte después de ingerir una comida grasosa. Un estudio
imagenológico señala que su vesícula biliar muestra dilatación
aguda y se hace el diagnóstico de colelitiasis. ¿En qué sitio
anatómico un cálculo vesicular agravaría el riesgo de
pancreatitis?
A) Conducto hepático izquierdo.
B) Conducto hepático derecho.
C) Conducto cístico.
D) Colédoco.
E) Esfínter de Oddi.

6. La bilis de la vesícula biliar, en comparación con la que está en el


hígado, contiene una menor concentración de:
A) Ácidos biliares.
B) Iones de sodio.
C) Protones.
D) Glucosa.
E) Protones.

1. Un niño de 2 años es llevado a la consulta por diarrea persistente y edema de las


extremidades, además falta de crecimiento y desarrollo en relación a su edad. Los análisis de
sangre revelan que tiene concentración plasmática baja de proteínas (hipoproteinemia).
Durante la endoscopía duodenal, se coloca colecistokinina (CCK) endovenosa y se recoge
muestras del líquido duodenal; el resultado del líquido confirma incapacidad para hidrolizar
proteínas a un pH neutro, esta situación mejora al añadir una pequeña cantidad de tripsina.
El paciente probablemente esté sufriendo la falta congénita de

d. Enterocinasa

2. Experimentalmente se incrementa la velocidad de la secreción salival con una sustancia, el


análisis de la composición de esta saliva obtenida se espera encontrar
c. Disminución de concentración de potasio

3. Paciente varón de 46 años soltero, consulta por odinofagia y bajo de peso, tiene antecedente
de tuberculosis desde hace 3 meses y es fumador crónico (10 cigarrillos por día); al evaluar
la cavidad oral se identifica lesión blanquecina en el dorso de la lengua y paladar blando, las
lesiones se desprenden con el baja lengua dejando una base eritematosa. Esta lesión
corresponde probablemente a
b. Candidiasis oral

4. Minero de 32 años de edad, que acude a centro de salud por presentar de forma progresiva
desde hace 1 año dificultad para ingerir alimentos sólidos y luego líquidos; refiere
regurgitaciones alimentarias y marcada pérdida de peso (15 kilos). Radiografia baritada de
esófago como se muestra en la figura. El presente caso se explica por
Relajación incompleta del esfínter esofágico inferior

5. Paciente mujer de 35 años acude a consulta por sensación de sequedad y lesiones en


cavidad oral. Al examen se observa atrofia de la mucosa, fisuras y úlceras; nota además
sequedad e irritación de la córnea y aumento del tamaño de las glándulas parotídeas. Su
diagnóstico más probable es artritis reumatoide; el hallazgo más probable en una biopsia de
glándula parótida es
b. Gran infiltración de linfocitos y células plasmáticas
6. Un paciente con anemia acude con su médico quejándose de episodios frecuentes de
gastroenteritis. Un análisis de sangre revela anticuerpos circulantes dirigidos contra células
parietales gástricas. Su anemia es atribuible a la hiposecreción de
Factor intrínseco

7. Un niño de cuatro años de edad es llevado a la consulta por cuadros diarreicos frecuentes
caracterizados por heces pálidas, voluminosas y fétidas, presenta bajo peso y talla. Se mide
la concentración de cloruro en el sudor y se encuentra que sus valores son muy elevados. La
alteración más importante a nivel de células ductales del páncreas tiene relación directa con
la conductancia de
d. Cloro
8. Una mujer de 50 años de edad que sufrió durante varios años resequedad de los ojos debida
a producción inadecuada de lágrimas es enviada con un gastroenterólogo para evaluación de
pirosis crónica. El examen endoscópico revela erosiones y tejido cicatrizal en la parte distal
del esófago justo por arriba del esfínter esofágico inferior. Las lesiones pueden atribuirse a la
disminución de uno de los siguientes componentes salivales:
a. Bicarbonato
9. En el reflejo peristáltico del intestino delgado, uno de los siguientes eventos sucede en la
porción oral del bolo alimenticio
d. Acción de acetilcolina en el músculo circular
10. Experimentalmente se coloca una dosis alta de secretina en la luz intestinal duodenal; como
consecuencia de esto, en el jugo pancreático de la misma luz intestinal se observa la
disminución de la concentración de
b. Cl-
11. Un varón de 58 años de edad con enfermedad de Crohn severo fue sometido a una
resección ileal. Después de la cirugía este paciente padecerá de esteatorrea, esto se explica
porque
La micelas no pueden formarse
12. La toxina del Vibrio cholerae causa diarrea debido a
b. El Incremento de la secreción de cloro por las células de la cripta intestinal
13. ¿Cuál de las siguientes alternativas es una característica de la secreción exocrina del
páncreas?
a. Tiene una baja concentración de Cl- respecto al plasma
14. Una madre lleva a su hijo de dos años de edad a la sala de urgencias, estresada porque el
niño deglutió una moneda de 10 céntimos mientras la familia cenaba en un restaurante. El
médico observa mediante fluoroscopía que la moneda se halla en el estómago y asegura a la
madre que la moneda se eliminará con las heces. El médico recomienda utilizar la respuesta
fisiológica que permitirá la evacuación de la moneda del estómago al intestino
b. Son los movimientos de mezcla y trituración
c. Es provocada por el ayuno

15. La composición de la bilis es modificada conforme fluye por los conductillos biliares. Durante
este tránsito se espera que aumente la concentración de
a. Ig A
c. Monómeros de ácido biliar
16. Se mide experimentalmente el contenido gástrico de dos personas. La persona “A” tiene alto
contenido de grasa y la persona “B” tiene un contenido hipertónico ¿Cuál de las siguientes es
correcto respecto al vaciamiento gástrico?
Hay ralentización del vaciado gástrico en ambos casos
17. Un paciente varón de 18 años de edad acude al médico para sus exámenes de rutina. Sus
resultados de laboratorio muestran un valor de bilirrubina sérica de 4 mg/dl y una bilirrubina
directa de 0,3 mg/dl. Las pruebas de función hepática son normales. La alteración que
explica mejor este caso es por la deficiencia de
b. Glucuronil transferasa
18. Un hombre de 57 años de edad es llevado a urgencias con hematemesis masiva rojo
brillante, a su llegada se halla inconciente con PA: 80/40 mm Hg y FC: 124 lat/min. Luce
ictérico con presencia de “arañas vasculares en el tórax anterior y extremidades”, abdomen
distendido con signo de oleada positiva. Se encuentra esplenomegalia y pérdida de la masa
muscular en extremidades. La anastomosis vascular responsable del sangrado en este
paciente es
b. Vena gástrica izquierda y vena ácigos
19. Un estudiante de medicina está comiendo un plato de comida a base de champiñones,
espárrago y salsa de soya. El sabor umami contenido en todos estos alimentos actúa a nivel
de los botones gustativos estimulando
Un receptor acoplado a proteína G
20. Un hombre de 22 años de edad se presenta al médico con una historia de 1 año de
evolución caracterizado por dolor recurrente en fosa iliaca derecha y diarrea. Manifiesta
además pérdida de peso de 8 kg durante este periodo. La colonoscopía revela múltiples
lesiones en el ileon terminal y colon. La biopsia de estas lesiones revela engrosamiento,
inflamación y ulceración de la mucosa. El diagnóstico más probable en este caso es
b. Enfermedad de Crohn
21. Una de las funciones del músculo señalado es
d. Deprime la mandíbula cuando el hioides está fijo

22. Varón de 61 años que consulta por dolor retro esternal intenso desde hace 6 horas y
después de vómitos intensos y repetidos; al examen se observa disnea, cianosis, hipotensión
y signos clínicos de shock. La radiografía simple de tórax muestra neumomediastino. El
líquido en el espacio pleural aspirado tiene alta concentración de amilasa. ¿Cuál de las
siguientes alternativas puede explicar este cuadro clínico?
b. Rotura espontánea de esófago
23. La secreción del ácido en la célula parietal gástrica se lleva a cabo por una ATPasa
especifica que intercambia hidrogeniones (H+) del citosol por
d. K+
24. En condiciones normales el ingreso de 600 ml de líquido es el estómago provoca un
aumento de presión intragástrica de unos 12 cm de H2O. Después de una vagotomía (corte
del nervio vago) es de esperar que el ingreso del mismo volumen de líquido provoque lo
siguiente:
Un aumento mayor de la presión
25. Una paciente de 30 años de edad es sometida a una cirugía en oído medio derecho por un
problema de otoesclerosis. Luego de la cirugía refiere alteración en la percepción de sabores.
Al evaluar el caso usted esperaría encontrar
b. Alteración en la sensación del gusto en los dos tercios anteriores de la lengua
d. Sensación del dolor, tacto y temperatura conservada en toda la lengua

26. ¿Cuál de las siguientes alterativas es correcta?


c. Las sales biliares desconjugadas son absorbidas preferentemente en el colon
27. En un paciente de 45 años de edad con colestasis biliar, se encuentra una elevación de los
niveles sanguíneos de fosfatasa alcalina hasta 3 veces la cifra normal. ¿Cuál de las
siguientes alternativas estará también elevada como evidencia del daño de la vía biliar?
Gamma glutamil transpeptidasa
28. Revisando la angiografía de un hombre de 70 años en estudio por aneurisma de aorta
abdominal el radiólogo informa de la presencia de una oclusión completa de la arteria
mesentérica inferior. El paciente se encuentra completamente asintomático. ¿Cuál de las
siguientes arterias se anastomosa a la sistema arterial de la mesentérica inferior?
Cólica media
29. Un varón de 75 años ingresa al consultorio por presentar ictericia marcada de piel y las
escleras. El estudio del paciente mostró que presentaba un tumor que obstruía la totalidad
del conducto hepático común. ¿Cuál de las siguientes estructuras se encontrará dilatada en
este paciente?
Conductos de Hering
30. Respecto a la siguiente imagen que representa una estructura de la mucosa gástrica, la
estructura con número ……….. produce

2 / pepsinógeno
PREGUNTAS PA TIPEAR

Secreta grandes cantidades de bicarbonato y agua para neutralizar la acidez del quimo:

● Páncreas

La sacarosa se desdobla en:

● Glucosa y fructosa

Enfermedad en que no hay relajación del cardias como respuesta de deglución y hay falta de
peristaltismo en el esófago:

● Acalasia

Histológicamente la mucosa del tubo digestivo presenta las siguientes regiones excepto

● Muscular
● Epitelio
● Corion
● Lámina propia
● Muscularis mucosae

Es la porción principal y de mayor tamaño del estómago, en la cual se forma el quimo:

● Cuerpo

Entre las capas musculares se encuentran grupos de células ganglionares y haces de fibras
nerviosas amielínicas, que en conjunto representan:

● Plexo mientérico de auerbach

A nivel del íleon, se encuentran agregaciones permanentes de nódulos linfoides llamados:

● Placas de Peyer

En relación al intestino grueso, la capa mucosa está constituida por tejido conectivo laxo y mesotelio,
presenta depósitos de grasa llamados:

● Apéndices epiploicos

En que porción del tubo digestivo se sintetiza vitamina K y B

● Colon

Contienen enzimas, excepto:

● Saliva
● Bilis
● Jugo Pancreático
● Jugo intestinal
● Jugo gástrico

Producen el HCl en el estómago

● Células parietales
No es parte del duodeno:

● Descendente
● Ileal
● Ascendente
● Horizontal
● Superior

Enzima proteolítica contenida en el jugo pancreático

● Tripsina

Conducto de drenaje de la glándula parótida en la boca

● Stenon

La unidad anatomofisiológica del hígado es

● Estroma
● Hepatocito
● Lobulillo hepático
● Hilio del hígado
● Parénquima

Los ameloblastos se localizan en:

● Esmalte

Contiene glucoproteínas que permite la lubricación de la boca:

● Secreción mucosa de la saliva

El bolo alimenticio pasa hacia la faringe a través de:

● Istmo de las fauces

El conducto colédoco se forma por la convergencia de los conductos:

A. Cístico - Wirsung
B. Santorini - hepático
C. Cístico - Santorini
D. Cístico - hepático
E. Cardiaco - hepático

Las glándulas parótidas y submaxilares respectivamente presentan como conductos:

A. Rivinus y Stenon
B. Wharton y Stenon
C. Stenon y Rivinus
D. Stenon y Wharton
E. Wharton y Rivinus

Membrana epitelial que sostiene a casi todas las vísceras del abdomen

A. Mesenterio
B. Fundus
C. Peritoneo
D. Epitelio
E. Mucosa

En el mecanismo de formación de HCl intervienen los siguientes elementos químicos:

A. H2CO3
B. CO2
C. H2O
D. H+
E. T.A.

La gingivorragia se produce por deficiencia de vitamina:

A. B
B. D
C. E
D. C
E. A

Fase de secreción del HCl que se activa por saborear

A. Vagal
B. Intestinal
C. Gástrica
D. Neurógena
E. AyD

Producen colecistoquinina:

A. Células Paneth
B. Células argentafines
C. Células APUD
D. Brunner
E. N.A.

El tripsinógeno es activado por:

A. Enterocinasa
B. Quimiotripsina
C. Tripsina
D. Elastasa
E. A+B

Con respecto a la microestructura del hígado ¿Cuál de las siguientes alternativas es correcta?
- Los hepatocitos están interconectados por uniones herméticas
- La célula de Ito se encuentra en el espacio de Disse y reserva glucógeno
- El sinusoide es un capilar fenestrado
- La célula de Kupffer se encuentra fuera del sinusoide y fagocita células

En un paciente con una estenosis severa del colédoco por una complicación quirúrgica, usted esperaría
que desarrolle:

- la cabeza de Medusa
- el aumento de transaminasas mayor que el de fosfatasa alcalina
- hiperbilirrubinemia no conjugada
- el aumento del tiempo de protrombina

Marque lo correcto respecto al acino hepático:

- Dos vértices del rombo acinar se constituyen por triadas portales


- El eje menor lo constituye un eje imaginario entre dos venas centrolobulillares
- La zona 1 es la más que recibe más sangre
- La zona 3 se afecta en menos en una deshidratación severa

Los hepatocitos tienen una gran capacidad regenerativa, en parte gracias a las células madre hepáticas
que se localizan en los:

- espacios de Disse
- sinusoides hepáticos
- canales de Herring
- espacios de Mall

En un paciente con infección por SARS-CoV-2 con compromiso severo e ingresado en la unidad de
cuidados intensivos (UCI), debido a la tormenta de citoquinas y a la desregulación inmune ¿Cuál de
las siguientes alternativas estará elevada en sangre al evaluar el perfil hepático?

- transaminasas
- fosfatasas
- amilasas
- bilirrubinas

La presión parcial de oxígeno en la Zona 1 del sinusoide hepático de debe ser ___________ mmHg

- menor de 40
- 40
- 100

- entre 95 y 45

En un paciente cirrótico con encefalopatía hepática ¿cuál de las siguientes alternativas sustenta la
reducción de carnes rojas en la dieta?

- El amonio se produce principalmente en el intestino


- Hasta el 80% del colesterol se transforma en sales biliares
- La fructosa y la galactosa se convierten en glucosa en el hígado
- El factor de crecimiento hepatocitario fomenta la regeneración del hepatocito

En el hígado, el aumento de la resistencia vascular en los sinusoides hepáticos ocasionará:


- aumento del flujo hacia la vena porta
- aumento de la presión de llenado vesicular
- aumento del flujo hacia la vena cava superior
- salida de plasma hacia el intersticio

En un paciente con cirrosis hepática se desarrollará ascitis debido a los siguientes mecanismos,
EXCEPTO:

- Hipoalbuminemia
- Estrechamiento de la porta
- Hiperflujo portal
- Aumento de reabsorción renal de sodio

El efecto de un medicamento colerético se evidencia por:

- el aumento de formación de micelas


- el aumento de secreción biliar
- la mayor producción de colesterol en la bilis
- la disminución de absorción de sales biliares

CI4 - PEQUE

Paciente con Lupus Eritematosos que desarrolla hipertensión portal debido a trombosis portal, es
probable que desarrolle várices a nivel de

- canal anal
- recto inferior
- recto superior
- hemorroides externas

¿Cuál de las siguientes alternativas es una comunicación entre el tejido hepático y la vesícula biliar?

- Válvula espiral de Heister


- Conducto de Lushka
- Divertículos vesiculares
- Senos de Rokitansky-Aschoff

En un paciente con cirrosis hepática terminal, qué hormona aumenta la absorción de agua en el colon

- Disminuida
- No influye
- Aumentada
- Normal

En un paciente con cirrosis hepática avanzada se encuentra tendencia a

- la hiperglicemia
- el acortamiento del tiempo de protrombina
- la disminución de bilirrubina hidrosoluble
- la hipoglicemia

Un paso importante para la poder absorber los lípidos es la emulsificación de las grasas. ¿Cuál de las
siguientes alternativas es el principal factor encargado de dicha emulsificación?

- Ácidos biliares
- Bilirrubina
- Colesterol
- Fosfolípidos

Para aumentar la cantidad de bicarbonato en el flujo pancreático es necesario

- inhibir al nervio vago


- estimular la secreción de péptido liberador de colecistoquinina (CCK
- ingerir aminoácidos como triptófano o fenilalanina
- estimular las células S

En un paciente con cirrosis hepática se desarrollará ascitis debido a los siguientes mecanismos,
EXCEPTO:

- Hipoalbuminemia
- Aumento de reabsorción renal de sodio
- Estrechamiento de la porta
- Hiperflujo portal

En el síndrome de Mirizzi, el paciente tiene cálculos en la vesícula biliar; pero se obstruye el conducto
hepático común debido a que un cálculo se ubica y crece de tamaño en:

- la bolsa de Hartmann
- el conducto accesorio de Lushka
- el colédoco
- la válvula espiral de Heister

En un paciente con intoxicación por órganos fosforados, la acción de la colecistoquinina (CCK) está
bloqueada a nivel de:

- el sistema nervioso central


- el esfínter de Oddi
- la vesícula biliar
- la célula parietal

Con respecto a la microestructura del hígado ¿Cuál de las siguientes alternativas es correcta?

- Los hepatocitos están interconectados por uniones herméticas


- El sinusoide es un capilar fenestrado
- La célula de Ito se encuentra en el espacio de Disse y reserva glucógeno
- La célula de Kupffer se encuentra fuera del sinusoide y fagocita células

CI 4 + lo que me mandaron a tiepar - Shiro


En un paciente con cirrosis hepática la cabeza de medusa que aparece en la pared
abdominal, podría desaparecer si al paciente se le:
- esclerosa las venas hemorroides internas
- oblitera el ligamento redondo
- administra antiandrógenos
- oblitera la arteria gástrica izquierda

Para poder absorber al torrente sanguíneo todos los carbohidratos de la lactosa, es


necesario usar únicamente los transportadores:
- SGLT1, GLUT2
- GLUT5, SGLT1
- GLUT2, SGLT1, GLUT5
- GLUT2, GLUT5

Interviene en la digestión intestinal, excepto:


- jugo pancreático
- bilis
- jugo intestinal
- jugo gástrico
- tripsina

De las siguientes enzimas digestivas;


I) Ptialina
II) Pepsina
III) Tripsina
IV) Enteroquinasa
V) Aminopolipeptidasa
son producidas por el estomago:

- I, II, III
- III, IV, V
- I, III, IV
- II, IV
- Solo II

La secretina, hormona secretada por la mucosa duodenal, desempeña varias funciones,


indique la que corresponde:
- Activa la secreción gástrica
- Provoca contracción de las vesícula biliar
- Inhibe la secreción gástrica
- Activa la secreción pancreática
- Inhibe la secreción pancreática

Los sinusoides hepáticos, contienen células que pertenecen al RES con actividad fagocítica,
a estas células se les conoce con el nombre de:
- Yuxta globemerulares
- Kupffer
- Remack
- Podocitos
- Falciformes

El tubo digestivo esta formado por excepto:


- Boca
- intestino grueso
- hígado
- estómago
- intestino delgado

El plexo de Auerbach se encuentra en la capa


- Musculares mucoseae
- Subucosa
- Mucosa
- Muscular
- Serosa

Cual no es función del hígado


- hemocateresis
- secreción de bilis
- hematopoyesis
- anticoagulante
- almacenamiento principalmente de vitaminas A, D y B12

El estómago presenta epitelio


- estratificado plano sin queratina
- cúbico simple
- cilindrico simple no modificado
- cilindrico simple con ribete en cepillo
- escamoso plano

La digestion de los almidones se inicia en _______, por la enzima _______


- estomago - amilasa gastrica
- intestino delgado - amilasa pancreatica
- esofago - alfa amilasa salival
- boca - alfa amilasa salival
- intestino delgado - colon

Respecto a glándulas salivales es cierto, excepto:


- Parótida presenta el conducto de stenon
- Submaxilares secretan el 70% del volumen desaliva
- Sublinguales posee el conducto de Rivinus
- Submaxilares tienen secreción seromucosa
- Parótidas presenta secreción mucosa

El padre de rosa es alcohólico desde hace 20 años y ahora cura el médico por presentar la
piel y escleras amarillentas (ictéricas) el médico le ha dicho que su hígado se encuentra
muy dañado, Entonces qué enfermedad podría presentar
- hepatitis B
- Apendicitis
- gastritis
- cirrosis hepática
- N.A

Una de las características de la bilirrubina, es que tiene un color:


amarillento
Las placas de Peyer, se encuentran en:
- duodeno
- yeyuno
- ileon
- colón
- recto

La función de la bilis es:


- emulsificar proteínas
- emulsificar las grasas
- emulsificar carbohidratos
- facilitar la digestión de los aminoácidos
- degradar las proteínas

No es Constituyente del jugo pancreático


- tripsinógeno
- enterocinasa
- amilasa
- carboxipeptidasa
- bicarbonato

Órgano del aparato digestivo que realiza la absorción de aproximadamente el 90% de


sustancias:
- Esofago
- estómago

ALEJO

CI 1

1. La motilidad intestinal es estimulada principalmente por el:

- Sistema simpático

- Sistema piramidal

- Plexo de Auerbach

- Sistema parasimpático

2. La peristalsis o peristaltismo hace referencia a:

- Motilidad para movilizar el alimento de proximal a distal.

- No es parte de la motilidad

- Motilidad para mezclado de alimentos.

- Motilidad para fraccionamiento de alimentos.


3. Marque el órgano que se encuentra más distal en el tubo digestivo.

- Estomago

- Ciego

- Íleon

- Duodeno

4. Marque la respuesta incorrecta:

- La mucosa consta de epitelio, lámina propia y muscularis mucosae.

- En todo el tubo digestivo, se observa dos capas de muscular propia:


circular interna y longitudinal externa

- El colon contiene tenias

- Fuera de la cavidad abdominal, el esófago presenta capa adventicia.

5. Al iniciar la digestión, aumenta el consumo de oxígeno por la mucosa. Esto conlleva


a una hipoxia local, lo cual hace que se libere _____________, el cual produce
vasodilatación:

- Colecistoquinina

- Adenosina

- Histamina

- Noradrenalina

6. Cuál de las siguientes estructuras no tiene vasos sanguíneos:

- Epitelio intestinal

- Ligamento

- Omento

- Mesenterio

7. Paciente tiene una úlcera sangrante en el segundo tercio del Yeyuno. La arteria de
la cual proviene la sangre arterial para dicha zona es la arteria:

- Mesentérica superior

- Tronco celíaco

- Mesentérica inferior

- Iliaca común
- Gástrica izquierda

8. Paciente de 24 años con dolor abdominal tipo cólico intenso en mesogastrio. Según
sus conocimientos de macroestructura, el origen del dolor puede ser el ___________:

- Íleon

- Colon

- Estómago

- Esófago

9. Dentro de las funciones del abdomen, se encuentra la defecación y micción, en las


cuales la presión intra abdominal debe:

- Aumentar

- No tiene relación el abdomen con dichas funciones

- Mantenerse igual

- Disminuir

10. En la inspiración, la pared abdominal debe ____________ para ____________:

- Contraerse aumentar presión intra abdominal

- Relajarse disminuir presión intra torácica

- Relajarse aumentar presión intra abdominal

- Contraerse aumentar presión intra torácica

CI 2

1. Respecto a los péptidos gastrointestinales, marque lo correcto.

- No existe sustancia neurocrina que tenga efecto en la motilidad del tubo


digestivo

- Las sustancias paracrinas pueden viajar a través de vasos sanguíneos

- Las sustancias neurocrinas son peptidos que hacen su efecto en


distancias cortas

- Las sustancias paracrinas atraviesan la circulación portal

2. Al disminuir el pH duodenal por el HCl gástrico, se libera principalmente una


hormona cuya célula diana es:
- Acinos pancreáticos

- Células ductales del colédoco

- Célula ductal del Wirsung

- Células S del intestino

3. En un paciente con gastroparesia (motilidad lenta del estómago), que presenta


distensión abdominal después de comer, usted le recomendaría que evite el consumo
de lípidos y aminoácidos para disminuir la acción de:

- Secretina

- CCK

- Somatostatina

- Gastrina

4. La razón por la que el potencial de acción viaja rápidamente en sentido longitudinal


por el musculo liso gastrointestinal es la presencia de uniones en hendidura, además de
la presencia de:

- La presencia del plexo submucoso de Meissner

- Varicosidades

- Mayor cantidad de ACh

- Las fibras musculares no se disponen en haces musculares

5. Paciente obeso con Covid-19 es intubado por interno inexperto, quien al solicitar
que bombeen aire dentro del tubo endotraqueal, nota que el epigastrio se distiende. Al
sospechar que ha introducido el tubo en el estómago, también es cierto que:

- Disminuiría el tono del píloro

- Aumenta la frecuencia de ondas lentas

- Aumenta el pH gástrico

- Disminuye el pH gástrico

6. En un paciente con diarrea por hipermotilidad, usted sospecharía en el posible


aumento de las siguientes sustancias, excepto:

- Sustancia P

- ACh

- Péptido intestinal vasoactivo (VIP)


- Motilina

7. El ecografista sabe que para poder visualizar el nacimiento de la arteria


mesentérica superior, debe colocar el transductor sobre la piel de la siguiente región
abdominal:

- Epigastrio

- Hipocondrio derecho

- Hipogastrio

- Mesogastrio

8. Al consumir un pan con mantequilla, la sensación de hambre disminuye debido a la


acción de:

- Grelina

- Somatostatina

- Colecistoquinina (CCK)

- Leptina

9. Una de las siguientes sustancias no comparte con las otras la misma acción sobre
la producción de ácido gástrico:

- Péptido insulinotrópico dependiente de glucosa (GIP)

- Colecistoquinina

- Somatostatina

- Secretina

10. Al ingerir grandes cantidades de dulces, con la subsecuente estimulación de


incretinas, usted esperaría que el apetito ______________, debido a
__________________

- disminuya insulina

- aumente CCK

- disminuya CCK

- aumente grelina

PARCIAL

1. Al rozar agua caliente en la punta de la lengua, usted esperaría que el estímulo


viaje a través del nervio:
- lingual

- cuerda del tímpano

- glosofaríngeo

- hipogloso

2. Al ingresar líquidos o sólidos en la cavidad oral, un mecanismo que permite que una
persona respire mientras mastica es:

- el movimiento hacia afuera de los pliegues palatogloso y palatofaríngeo

- la elevación del paladar blando

- la depresión de la parte posterior de la lengua

- la depresión del paladar blando

3. Paciente adulto con reflujo gastroesofágico es más probable que presente:

- descalcificación del esmalte

- destrucción de ameloblastos

- remodelación del esmalte

- desfluorización de los dientes

4. La masticación es básicamente:

- importante para la digestión sobretodo de carnes

- un ralentizador del vaciamiento gástrico

- una actividad consciente

- un movimiento reflejo

5. Paciente de 34 años es víctima de asalto con arma de fuego, recibiendo un impacto


directo en el abdomen. En base a la radiografía, usted puede registrar en la historia
clínica que el proyectil se encuentra topográficamente en el:
- flanco derecho

- mesogastrio

- flanco izquierdo

- hipocondrio izquierdo

6. Señale cuál de las estructuras que en el embrión se encuentra comunicada con el


saco vitelino por medio del conducto onfalomesentérico:
- D

- B

- C

- A

7. En un paciente de 3 semanas de edad, con vómitos en proyectil, y nódulo


epigástrico reptante, usted esperaría encontrar:

- colecistoquinina aumentada

- vómitos biliosos e intolerancia a los ácidos grasos

- distensibilidad disminuida de la región oral del estómago

- engrosamiento de la circular interna pilórica

8. ¿Cuál de las siguientes estructuras tiene inervación somática?

- Estómago

- Peritoneo visceral

- Mesosigmoides

- Peritoneo parietal

9. Estas diseñando un proyecto de investigación sobre los niveles de colesterol que se


absorben luego de una comida grasosa y deseas cuantificar la cantidad de colesterol
que es absorbido por el intestino antes que el hígado lo metabolice ¿de cuál de los
siguientes vasos obtendrías la muestra para tu análisis?
- Vena porta

- Vena cava superior

- Conducto torácico

- Vena hemiácigos accesoria

10. Durante el paso del bolo hacia la orofaringe, se desencadena una serie de
contracciones musculares que estrechan la cavidad faríngea. Estas contracciones están
mediadas por el nervio craneal:

- X

- XI

- XII

- IX

11. Cuando el istmo de las fauces se cierra, se evita que el alimento pase hacia la
orofarínge y permite respirar mientras se mastica. Este cierre se debe a la contracción y
aproximación de los músculos:

- Palatoglosos

- Palatofaríngeos

- Estiloglosos

- Estilofaríngeos

12. Las siguientes alternativas son factores que determinan la patencia y función
adecuada del esfínter esofágico inferior, EXCEPTO:

- Plicatura diafragmática

- Hipertrofia de la circular interna

- Angulación con el estómago

- Canales lentos de calcio

13. Respecto al peristaltismo intestinal, para cumplir la ley del intestino , usted espera
que a nivel distal del quimo se libere:

- péptido liberador de gastrina (GRP)

- acetilcolina

- péptido intestinal vasoactivo

- sustancia P
14. El estímulo habitual para el movimiento peristáltico es:

- acción de la sustancia P

- contracción de la musculatura circular Interna

- estimulación vago-vagal

- distensión local

15. ¿Cuál de las siguientes condiciones considera que es un trastorno de la


musculatura lisa esofágica?

- Acalasia

- Asinergia faringoesfinteriana

- Hipotonía de los constrictores faríngeos

- Hipertonía del esfínter esofágico superior

16. ¿Cuál de las siguientes alternativas es correcta sobre la motilidad esofágica?

- Las ondas primarias son propulsoras y pueden no ser precedidas por


deglución

- Las ondas primarias no son propulsoras y siempre van precedidas de


deglución

- Las ondas secundarias son propulsoras y siempre van precedidas de


deglución

- Las ondas secundarias son propulsoras y no van precedidas de


deglución

17. El peristaltismo depende que a nivel distal del bolo se secrete:

- noradrenalina secretada por las fibras del sistema simpático

- acetilcolina por las neuronas provenientes del nervio vago

- óxido nítrico por células endoteliales locales

- péptido intestinal vasoactivo por neuronas

18. ¿Cuál de las siguientes alternativas es correcta sobre el movimiento peristáltico?

- Es un reflejo largo que depende de la integración con el tronco


encefálico

- Se dirige en sentido distal siempre, nunca en sentido proximal

- El contenido intestinal avanza sólo 5-10 cm


- Es independiente del plexo mientérico

19. En un estudiante de medicina que está rindiendo un examen parcial, lo más


probable es que en ese momento su tránsito intestinal se encuentre:

- muy lento

- muy acelerado

- estimulado por acción de la sustancia P

- sin alteraciones

20. Las siguientes alternativas son ciertas sobre las contracciones tónicas del músculo
gastrointestinal, EXCEPTO:

- Tienen relación con el ingreso persistente de iones sodio

- Se encuentran principalmente en esfínteres

- Tienen regulación hormonal

- Obedece a una mayor frecuencia de potenciales en espiga

21. Con respecto de la regulación del pH del estómago; al utilizar un bloqueador de


histamina, usted espera que el pH del estómago:

- aumente

- disminuya

- se mantenga sin cambio

- se neutralice por acción de bicarbonato

22. Las siguientes hormonas disminuyen el vaciamiento gástrico, EXCEPTO:

- Gastrina

- Péptido insulinotrópico dependiente de glucosa

- Colecistoquinina

- Secretina

23. El frenillo de los labios se encuentra en:

- el piso de la boca

- la cavidad oral

- la cavidad vestibular
- el dorso de la lengua

24. En un paciente con shock hipovolémico, la peristalsis intestinal se encuentra:

- aumentada

- sin cambios

- invertida

- disminuida

25. Sobre el control de la peristalsis del tubo digestivo, ________________ es un


mediador neural que induce la relajación durante la peristalsis.

- la somatostatina

- el péptido intestinal vasoactivo

- la acetilcolina

- la serotonina

CI 3

1. En cuanto a las sustancias secretadas por el estómago ¿Cuál de las siguientes


sustancias estimula la liberación de pepsinógeno?

- Pepsinógeno

- Secretina

- Colecistoqunina

- Gastrina

2. Las células enteroendocrinas en el estómago se localizan en la glándula oxíntica, al


mismo nivel que las células:

- parietales

- principales

- mucosas

- absortivas

3. La fase intestinal de la secreción gástrica se debe básicamente a la participación de


las células:

- G del duodeno

- I del yeyuno
- S del íleon

- D del estómago

4. El conducto de Stenon, para entrar a la cavidad vestibular, debe atravesar el


músculo:

- genihioideo

- buccinador

- milohioideo

- masetero

5. Paciente con acalasia es sometido a tratamiento endoscópico o quirúrgico, usted le


ha informado al paciente previamente que es posible que una complicación de este
tratamiento es que quede con cierto grado de:

- reflujo gastroesofágico

- gastritis

- odinofagia

- úlceras gástricas

6. La sangre que lleva la vena porta es tipo:

- mixta

- arterial

- venosa

7. Los pliegues gástricos gruesos son prácticamente inexistentes a nivel de:

- el cuerpo

- el fondo

- la incisura angularis

- el antro

8. En la producción de HCl, la acción de la somatostatina disminuye la accion de:

- las prostaglandinas

- la histamina

- la acetilcolina
- la gastrina

9. En el estómago se secretan las siguientes sustancias, EXCEPTO:

- grelina

- Correcto

- motilina

- somatostatina

- gastrina

10. La vena porta se forma gracias a la unión de la vena mesentérica superior con la
vena:

- mesentérica inferior

- esplénica

- celiaca

- gástrica izquierda

CI 4

1. En el hígado, el aumento de la resistencia vascular en los sinusoides hepáticos


ocasionará:

- aumento de la presión de llenado vesicular

- salida de plasma hacia el intersticio

- aumento del flujo hacia la vena porta

- aumento del flujo hacia la vena cava superior

2. En un paciente con carcinoma de páncreas, el tumor ha invadido la unión entre la


venas esplénica y mesentérica superior; eso quiere decir que estamos seguros que el
tumor se encuentra a nivel del ________ del páncreas.

- cuerpo

- cola

- cabeza

- cuello

3. En un paciente con cirrosis hepática la cabeza de medusa que aparece en la pared


abdominal, podría desaparecer si al paciente se le:
- esclerosa las venas hemorroides internas

- oblitera el ligamento redondo

- administra antiandrógenos

- oblitera la arteria gástrica izquierda

4. En un paciente con intoxicación por órganos fosforados, la acción de la


colecistoquinina (CCK) está bloqueada a nivel de:

- el esfínter de Oddi

- la vesícula biliar

- la célula parietal

- el sistema nervioso central

5. Con respecto a la microestructura del hígado ¿Cuál de las siguientes alternativas es


correcta?

- La célula de Ito se encuentra en el espacio de Disse y reserva glucógeno

- Los hepatocitos están interconectados por uniones herméticas

- La célula de Kupffer se encuentra fuera del sinusoide y fagocita células

- El sinusoide es un capilar fenestrado

6. Paciente con Lupus Eritematosos que desarrolla hipertensión portal debido a


trombosis portal, es probable que desarrolle várices a nivel de:

- recto inferior

- recto superior

- hemorroides externas

- canal anal

7. El efecto de un medicamento colerético se evidencia por:

- la mayor producción de colesterol en la bilis

- la disminución de absorción de sales biliares

- el aumento de secreción biliar

- el aumento de formación de micelas

8. En un paciente con cáncer de páncreas y que desarrolla ictericia, la localización


más probable del tumor es en:
- la vesícula por metástasis

- el cuello del páncreas

- la cabeza del páncreas

- la cola del páncreas

9. En un recién nacido menor de 24 horas con atresia biliar, se encuentra elevación de


la:

- bilirrubina directa

- bilirrubina indirecta

- alanina aminotransferasa (ALT)

- hemoglobina

10. ¿Cuál de las siguientes alternativas es correcta sobre la estructura hepática?

- Los colangiocitos producen bilis

- En la triada portal, se encuentra la vena derivada de la suprahepática

- El flujo sinusoidal en el lobulillo hepático es de adentro hacia afuera

- El flujo biliar en el lobulillo hepático es centrífugo

1. Después de la ligadura del cordón umbilical la vena umbilical se oblitera quedando


como vestigio fibroso, el:

a) ligamento coronario

b) ligamento falciforme

c) ligamento de Arancio

d) ligamento triangular

e) ligamento redondo

1. Unidades funcionales del hígado :

a)Lobulillo hepático

b) Lobulillo portal

c) acino hepático
d) Espacio porta

e) Espacio de Kiernan

1. Plexo nervioso del sistema nervioso vegetativo o autónomo que controla


principalmente la secreción glandular gastrointestinal:

a) Plexo de Auerbach

b) Plexo lumbar

d) Plexo cervical

c) Plexo sacro

e) Plexo de Meissner

1. La digestión de las proteinas se inicia en el estómago mediante y termina en el


intestino delgado mediante enzimas provenientes del...

a) renina - Jugo intestinal

b) pepsina - jugo intestinal

c) amilasa - Jugo pancreático

d) pepsina - jugo pancreático

e) cuajo - Jugo pancreático y bilis

1. El reflejo de defecación envia impulsos nerviosos a la medula sacra, que origina


impulsos motores que llegan al colon sigmoideo, colon descendente, recto y ano,a
través de:

a) Sistema nervioso central

b) Encéfalo

c) Sistema nervioso simpático

d) Sistema nervioso somático

e) Sistema nervioso parasimpático


1. La mucosa gástrica presenta epitelio de tipo:

a) Poliestratificado cilindrico

b) Simple cúbico modificado

c) Polimorfo

d) Simple cilindrico no modificado

e) Simple cubico no modificado

1. Extensión amplia del peritoneo que recubre y suspende al yeyuno e ileon a la pared
abdominal posterior:

a) Mesocolon transverso

b) Epiplón menor

c) Mesenterio

d) Epiplón mayor

e) Ligamento hepatoduodenal

1. En relación a las glándulas salivales; lo incorrecto es:

a) La parótida presenta conducto de Stenon.

b) La secreción salival de la submaxilar es seromucosa

c) El conducto de Rivinus corresponde a lasublingual

d) El conducto de Wharton corresponde a la submaxilar

e) La secreción salival de la parótida es serosa y secreta el 70% del total de la saliva

1. El proceso de degradación de glucógeno en glucosa, recibe el nombre de:

a) Glucogénesis

b) Gluconeogénesis

c) Glucogenólisis

d) Glucólisis

e) Glicoilsis
1. Cuando los riñones dejan de producir orina debido a obstrucción de los nefrones de
uno o ambos riñones, a esta anomalía se le denomina:

a) Enuresis

c) Oliguria

e) Disuria

b) Anuria

d) Cistitis

1. Se caracteriza por la disminución progresiva y por lo común irreversible de la


velocidad de filtración glomerular. Con efectos tales como edema, acidosis, uremia,
anemia, aumento de niveles de potasio osteomalacia Denominándose a este
trastorno:

a) Insuficiencia renal

b) Nefrosis

c) Insuficiencia renal crónica

d) Enfermedad de Britaht

e) Pielonefritis

Preguntas (cami)

1. En un paciente con cáncer de páncreas y que desarrolla ictericia, la localización más


probable del tumor es en:
- la cola del pancreas
- la cabeza del pancreas
- la vesicula por metastasis
- el cuello del pancreas

José G - CI 4

1. Marque lo correcto respecto al acino hepático:


a. Dos vértices del rombo acinar se constituyen por triadas portales
2. En un paciente con infección por SARS-CoV-2 con compromiso severo e ingresado en la
unidad de cuidados intensivos (UCI), debido a la tormenta de citoquinas y a la desregulación
inmune ¿Cuál de las siguientes alternativas estará elevada en sangre al evaluar el perfil
hepático?
a. Transaminasas
3. La relajación del esfínter de Oddi se produce directamente por acción de:
a. el péptido vasoactivo intestinal (VIP)
4. La presión parcial de oxígeno en la Zona 1 del sinusoide hepático de debe ser _____ mmHg
a. entre 95 y 45
5. En un recién nacido menor de 24 horas con atresia biliar, se encuentra elevación de la:
a. bilirrubina directa
6. En un paciente con una estenosis severa del colédoco por una complicación quirúrgica, usted
esperaría que desarrolle:
a. el aumento del tiempo de protrombina
7. En un paciente con cirrosis hepática la cabeza de medusa que aparece en la pared abdominal,
podría desaparecer si al paciente se le:
a. oblitera el ligamento redondo
8. En el síndrome de Mirizzi, el paciente tiene cálculos en la vesícula biliar; pero se obstruye el
conducto hepático común debido a que un cálculo se ubica y crece de tamaño en:
a. la bolsa de Hartmann
9. Con respecto a la microestructura del hígado ¿Cuál de las siguientes alternativas es correcta?
a. Los hepatocitos están interconectados por uniones herméticas
10. En un paciente con intoxicación por órganos fosforados, la acción de la colecistoquinina (CCK)
está bloqueada a nivel de:
a. el esfínter de Oddi

1. Dentro de las funciones del abdomen, se encuentran la defecación y micción, en las cuales la
presión intra abdominal debe:

a. aumentar

2. Paciente se queja de dolor en hipocondrio derecho, pero superficialmente. El dermatoma


relacionado es (marque la mejor respuesta):

a. T9

3. En la evaluación de una tomografía abdominal, el interno observa un aneurisma en una arteria que
se dirige al riñón derecho. Con seguridad se puede afirmar que está a nivel de la vértebra:

a. L2

4. Al evaluar una tomografía abdominal, el médico asistente le pide al interno de la UPC que
encuentre la imagen con el corte a nivel de L1. El interno sabiamente busca el ________ para
ubicar la vértebra L1.

a. cuello del páncreas

5. Paciente de 24 años con dolor abdominal tipo cólico intenso en mesogastrio. Según sus
conocimientos de macroestructura, el origen del dolor puede ser el ___________:

a. Colon
6. Al examinar a un paciente, usted encuentra dolor localizado en fosa iliaca derecha y diagnostica
apendicitis. En este paciente, usted puede inferir:

a. el peritoneo parietal regional está afectado

7. Respecto a la anatomía del estómago, marque lo correcto

a. el fondo gástrico forma la curva mayor

8. Marque el órgano que se encuentra más distal en el tubo digestivo.

a. ciego

9. La peristalsis o peristaltismo hace referencia a

a. Motilidad para movilizar el alimento de proximal a distal

10. Al retirar completamente el mesenterio de un órgano, el mismo se vería afectado principalmente


en su

a. irrigación

1.Paciente con apendicitis aguda, que debuta con dolor en mesogastrio. Este dolor se debe a
estimulación de receptores del dolor cuyas fibras van a viajar a la médula espinal a través de
a. nervios simpáticos
2. Paciente con intoxicación por órganos fosforados (inhibidores de acetilcolinesterasa), se espera
que el tránsito intestinal se encuentre
a. aumentado

3. Al seccionar el nervio facial a nivel timpánico, usted esperaría


a. disminución del gusto en la punta de la lengua

4. Al ingerir rápidamente un litro de agua, usted esperaría que la gastrina aumente por efecto de
a. ACh del sistema mientérico

5.Marque lo correcto en relación al divertículo de Meckel


a. Se encuentra usualmente 60 cm de la VIC

6. usted encuentra músculo estriado en el siguiente segmento


a. esfinter anal externo

7. Al introducir una solución azucarada directamente al estómago mediante una gastrostomía


(comunicación entre la piel abdominal y el estómago), la sustancia que provocará que aumenten los
niveles séricos de insulina es
a. Péptido tipo glucagón 1 (GLP - 1)
8. Marque lo correcto respecto a la siguiente imagen
a. se produjo por falta de fusión de los ductos dorsal y ventral

9. es un ligamento derivado del mesenterio dorsal


a. gastrocólico

10. es un paciente con diarrea por hipermotilidad, usted sospecharía en el posible aumento de las
siguientes sustancias, excepto
a. péptido intestinal vasoactivo

1. En una gestante de 11 semanas con antecedente de esofagitis eosinofílica, con hiperémesis


gravídica, que acude a emergencia por hematemesis leve, y presenta súbitamente disnea y
dolor torácico. Usted sospecharía de
a. sindrome de Mallory Weiss

2. En un paciente que usa AINEs a altas dosis por artritis reumatoide, usted esperaría que presente
erosiones y úlceras gástricas debido a
a. disminución de irrigación en la mucosa

3. la saliva siempre será hipotónica debido a


a. la impermeabilidad de los conductos al agua

4. en el estómago se secretan las siguientes sustancias, EXCEPTO


a. motilina

5. Cuando un paciente recibe estímulo autónomo mixto (simpático y parasimpático), el flujo de saliva
a. aumenta en relación al basal
6. La fase intestinal de la secreción gástrica se debe básicamente a la participación de las células
a. G del duodeno

7. la inhibición de secreción gástrica es secundaria a:


a. el reflejo enterogástrico

8. aquellas células que producen mayor cantidad de moco en el epitelio gástrico son:
a. mucosas superficiales

9. ¿cuál de las siguientes alternativas es causa de reflujo gastroesofágico de contenido ácido?


a. hernia hiatal
10. para determinar que un paciente tiene esófago de Barrett, debemos encontrar…………. en la
biopsia de esófago
a. células calciformes

1. ¿ cuál de las siguientes alternativas es la correcta sobre la estructura hepática?


a. el flujo biliar en el lobulillo hepático es centrífugo
2. en un paciente cirrótico con encefalopatía hepática ¿cuál de las siguientes alternativas sustenta la
reducción de carnes rojas en la dieta?
a. el amonio se produce principalmente en el intestino
3. para poner absorber al torrente sanguíneo todos los carbohidratos del azúcar común (de mesa) es
necesario usar únicamente los transportadores:
a. GLUT2, SGLT1, GLUT5

4. En un paciente con una estenosis severa del colédoco por una complicación quirúrgica, usted
esperaria que desarrolle
a. el aumento del tiempo de protrombina

5. en el síndrome de Mirizzi, el paciente tiene cálculos en la vesícula biliar, pero se obstruye el


conducto hepático común debido a que un cálculo se ubica y crece de tamaño en:
a. bolsa de Hartmann

6. en un paciente con carcinoma de páncreas, el tumor ha invadido la unión entre las venas esplénica
y mesentérica superior: esto quiere decir que estamos seguros que el tumor se encuentra a nivel
del….. del páncreas
a. cuello

7. las siguientes sustancias son secretadas por el páncreas, EXCEPTO:


a. tripsina

en un paciente con intoxicación por organofosforados, la acción de la colecistoquinina (CCK) está


bloqueada a nivel de:
a. el esfínter de Oddi

9. en el hígado, el aumento de la resistencia vascular de los sinusoides hepáticos ocasionará:


a. aumento del flujo hacia la vena porta
b. salida de plasma hacia el intersticio
10. en un paciente con cáncer de páncreas y que desarrolla ictericia, la localización más probable del
tumor es en:
a. la cabeza del páncreas

LORENA FERNÁNDEZ
1 . El conducto colédoco se forma por la convergencia de los conductos
Cístico - hepático

2 . Las glándulas parótidas y submaxilares respectivamente presentan como conductos


Stenon y Wharton

3 . Membrana epitelial que sostiene a casi todas las vísceras del abdomen
Peritoneo

4 . En el mecanismo de formación de HCl intervienen los siguientes elementos químicos


H2O, CO2, H, H2CO3

5 . La gingivorragia se produce por deficiencia de vitamina


C

6 . Fase de secreción de HCl que se activa para saborear


Vagal y Neurógena

7. Producen colecistoquinina
Células APUD

8. El tripsinógeno es activado por


Enterocinasa

9 . Secreta grandes cantidades de bicarbonato y agua, para neutralizar la acidez del quimo
Páncreas

10 . La sacarosa se desdobla en
Glucosa y fructosa

11. Enfermedad en que no hay relajación del cardias como respuesta de deglución y hay falta de
peristaltismo en el esofago
Acalasia

12 . Histológicamente la mucosa del tubo digestivo presenta las siguientes regiones, excepto
muscular

13 . Es la porción principal y de mayor tamaño del estómago, en la cual se forma el quimo


Cuerpo

14 . Entre las capas musculares se encuentran células ganglionares y haces de fibras nerviosas
amielínicas que en conjunto representan
Plexo mientérico de Auerbach

15 . A nivel del íleon se encuentran agregaciones permanentes de nódulos linfoides llamados


Placas de Payer

Un neonato con vómitos biliosos, persistentes y con baja ponderal, usted sospecharia:
Atresia duodenal

Paciente ha sido puesto en “nacida por vía oral” (nil per os, NPO) durante 24 horas, la sensación de
hambre en este paciente se debe la par esencia en el hipotálamo de la sustancia:
Neuropéptido Y

La absorción de ácido fólico se produce gracias a:


cotransporte dependiente de Na

Paciente con deficiencia congénita de procolipasa, sufre de esteatorrea cada vez que come comidas
ricas en grasa. Los lípidos que no son hidrolizados debido a esta deficiencia son
Triglicéridos

En paciente con hipertensión portal, la causa más probable para una hematemesis es
varices esofágicas
Varón de 73 años es traído a urgencias con dolor abdominal intenso. En la tomografía se observa un
aneurisma de la aorta abdominal que afecta al origen de la arteria mesentérica superior. Basado en
sus conocimientos, usted sabe que el órgano que se encontrará más afectado será:
Yeyuno

En los neonatos sanos, el color verde oscuro del meconio (primera deposición) se debe a
pigmentos biliares

¿cual de las siguientes alternativas es correcta sobre el control del peristaltismo?


el peristaltismo intestinal aumenta por efecto de colecistoquinina

En un paciente con falla en la fusion de los conductos de las yemas ventral y dorsal del pancreas,
usted esperaria encontrar:
drenaje de la mayor parte del jugo pancreatico en la papila menor

En un paciente alcohólico con esplenomegalia, bilirrubina elevada y tiempo de protrombina elevado,


que acude por melena, usted sospecharía que esta se debe a
Várices esofágicas

En una cirugia abierta(laparotomia), el cirujano al abrir la cavidad peritoneal por la parte anterior (linea
media), lo primero que observa es:
epiplon mayor

El tubo digestivo posee glandulas, las glandulas submucosas se encuentran en el:


esofago y duodeno

Los reflejos enterogastricos son desencadenados por las siguientes circunstancias,excepto:


alta carga de carbohidrato en duodeno

Las siguientes alternativas son ciertas en relacion al mesenterio,excepto:


los omentos cumplen una funcion principal de irrigacion visceral

Las carnes deben su sabor especialmente delicioso debido a que presentan en su composicion:
glutamato

Con respecto de la regulacion del pH del estomago; al utilizar un bloqueador de histamina, usted
espera que el pH del estomago:
aumente

Al hacer una endoscopia, usted puede inferir los segmentos del esofago, gracias a algunas
estrecheces, excepto:
bronquio derecho

Cuando el itsmo de las fauces se cierra, se evita que le alimento pase hacia la orofaring y permite
respirar mientras se mastica. Este cierre se debe a la contraccion y aproximacion de los musculos:
palatoglosos

En el plexo mienterico, el origen de los impulsos eferentes esta en:


el plexo de meissner
Las siguientes alternativas son factores que determinan la patencia y funcion adecuada del esfinter
esofagico inferior, excepto:
hipertrofia de la circular interna

La rotacion en sentido longitudinal del estomago en el desarrollo embriologico condiciona que el


nervio vago derecho quede a nivel:
posterior

Paciente de 64 años de edad con tumor en cabeza del páncreas que compromete a una arteria que
discurre entre la cabeza del páncreas y el proceso uncinado. Dicha arteria es:
Mesentérica superior

Los quilomicrones llegan a los hepatocitos por:


Sinusoides

Con respecto a la regulacion del peristaltismo, al aplicarle atropina (antagonista colinergico) a un


paciente, es de esperarse que el peristaltismo:
disminuya

En la estructura dentaria, se observa que hay una composicion muy similar a la del hueso en la capa
denominada:
cemento

En un paciente con carcinoma de pancreas, el tumor ha invadido la union entre la venas esplenica y
mesentérica superior, eso quiere decir que estamos seguros que el tumor se encuentra a nivel del ----
----pancreas:
cuello
Marque lo correcto respecto al acino hepatico:
Dos vertices del rombo acinar se contituyen por triada portales

En un paciente con intoxicacion por organos fosforados, la accion de la colecistoquinina (CCK) esta
bloqueada a nivel de:
el esfinter de Oddi

Las siguientes sustancias son secretadas por el pancreas, excepto:


tripsina

En un paciente con cirrosis hepatica se desarrollara ascitis debido a los siguientes mecanismos,
excepto:
estrechamiento de la porta

La via biliar extrahepatica se encuentra ubicada dentro del ligamento:


hepatoduodenal

En un paciente con cancer de pancreas y que desarrolla ictericia, la localizacion mas probable del
tumor es:
la cabeza del páncreas

En el sindrome de Mizzi, el paciente tiene calculos en la vesicula biliar; pero se obstruye el conducto
hepatico comun debido a que un calculo se ubica y crece de tamaño en:
la bolsa de hartmann
En un paciente con una estenosis severa del coledoco por una complicacion quirurgica, usted
esperaria que desarrolle:
el aumento del tiempo de protrombina

En la anemia perniciosa, con atrofia gastrica marcada, usted esperaria encontrar dsiminucion en la
absorcion de -----------a nivel de------------
vitamina B12 / ileon distal

En un paciente es derivado por lesion del nervio vago derecho.Al evaluar el velo del paladar, e solicita
al paciente que diga ahh, entonces se puede observar que la uvula:
se desvia a la izquierda

Paciente con lesion del hipogloso del lado izquierdo.Para evaluarlo se le pide al paciente que saque
la lengua, la cual se espera que la punta de la lengua se dirija hacia:
el lado izquierdo

En cuanto a las sustancias secretadas por el estomago. ¿cual de las siguientes sustancias estimula
la liberacion de pepsinogeno?
secretina

La sangre que lleva la vena porta es tipo:


venosa

El conducto de stennon, para entrar a la cavidad vestibular, debe atravesar el músculo:


buccinador

En la produccion de HCL, la acción de la somatostatina disminuye la accion de:


la gastrina

En un paciente con xerostomía presenta las siguientes condiciones, excepto:


infeccione del oido a repeticion

Las siguientes alternativas son correctas sobre la deglucion, excepto:


consta de 2 fases

Aquellas celulas que producen mayor cantidad de moco en el epitelio gastrico son las:
mucosas superficiales

La fase intestinal de la secrecion gastrica se debe basicamente a la participacion de las celulas:


G del duodeno

La distension del yeyuno provoca que se:


despolarice el potencial de reposo de membrana

La hormona responsable de los complejos migratorios interdigestivos tiene las siguientes


caracteristicas, excepto:
cumple funciones de aumentar la motilidad y secrecion gastrica e intestinal

El peristaltismo intestinal se produce gracias a un reflejo que:


se origina dentro de la pared intestinal
paciente de 17 años, con diarrea crónica sin moco ni sangre, asociado a dolor abdominal, con
antecedente de masticar de 6 a 10 tableta de chicle trident que contiene sorbitol.El mecanismo de
este tipo de diarrea es:
osmotica

El peristaltismo depende que a nivel distal del bolo se secrete:


peptido intestinal vasoactivo por neuronas

Las heces tienen olor característico, cuyo responsable es:


mercaptanos

Con respecto de la regulacion del pH del estomago; al utilizar un bloqueador de histamina, usted
espera que el pH del estomago:
aumente

Las siguientes alternativas son cierta en relacion al mesenterio,excepto:


los omentos cumplen una funcion principal de irrigacion viscera

En un estudiante de medicina que esta rindiendo un examne parcial, lo mas probable es que en ese
momento su transito intestinal se encuentre:
muy lento

¿cual de las siguientes alternativas es correcta sobre la motilidad esofagica?


la ondas son propulsoras y no van precedidas de deglucion

¿cual de los siguientes requiere de la formación de micelas para su absorción intestinal?


vitamina K

paciente con volvulo del colon sigmoides.La necrosis de este segmento del colon se produce por una
alteración en la irrigación de la arteria:
mesentérica inferior

Además de acidosis metabólica, las diarreas se acompañan de:


Hipokalemia

En los pacientes con diarrea asociada al consumo de pastas o pan, y con anticuerpos antigliadina, la
malabsorción se produce básicamente por:
Atrofia de las vellosidades

En un paciente con cáncer de páncreas y que desarrolla ictericia, la localización más probable del
tumor es en

Cabeza del páncreas

Paciente de 62 años con vólvulo de intestino delgado e isquemia intestinal. ¿Qué estructura se utiliza
como punto de referencia para determinar la posición de la unión duodenoyeyunal?

Ligamento suspensorio del duodeno (Treitz)

La acción de la secretina sobre las células ductales del páncreas, es potenciada por:

Acetilcolina
¿Cuál de las siguientes sustancias tiene una mayor concentración en la saliva comparado con su
concentración plasmática?

potasio

Paciente con mutación de la enzima UDP glucuronil transferasa que la hace menos activa, al hacerle
un análisi de sangre, encontraremos valores elevados de (marque la mejor respuesta):

Bilirrubina indirecta

La localización de la vesícula biliar con respecto al lóbulo cuadrado es:

Lateral

En un paciente con un cálculo en la ampolla de Váter, usted esperaría encontrar en sangre


principalmente:

Fosfatasa alcalina elevada

El conducto colédoco recibe dicho nombre a partir de

La unión del conducto cístico con el hepático común

Paciente de 2 años, llega a emergencia por haber ingerido una moneda con la que estaba jugando,.
El lugar más probable donde puede haber quedado este objeto es sobre el estrechamiento a nivel del
músculo:

Cricofaríngeo

¿Cual de las siguientes alternativas es una comunicación entre el tejido hepático y la vesícula biliar?

Conducto de Luschka

NO es una característica del proceso llamado relajación receptiva del estómago:

Se produce posteriormente al cierre del esfínter esofágico inferior

Calificación Individual 01 - 202002


1. Al examinar a un paciente, usted encuentra dolor localizado en fosa iliaca derecha y
diagnostica apendicitis. En este paciente, usted puede inferir:
a. El peritoneo visceral regional está principalmente afectado
b. El peritoneo parietal regional está afectado
c. Hay inflamación de todo el peritoneo parietal (peritonitis)
d. El diagnóstico está errado por no corresponder a la región abdominal adecuada

2. Marque el órgano que se considera retroperitoneal:


a. Lóbulo izquierdo del hígado
b. Vesícula biliar
c. Sigmoides
d. Parte de la vía biliar

3. La peristalsis o peristaltismo hace referencia a:


a. Motilidad para movilizar el alimento de proximal a distal.
b. Motilidad para mezclado de alimentos.
c. No es parte de la motilidad
d. Motilidad para fraccionamiento de alimentos.

4. La motilidad intestinal es estimulada principalmente por el:


a. Sistema parasimpático
b. Plexo de Auerbach
c. Sistema piramidal
d. Sistema simpático

5. La estructura que fija órganos principalmente a la pared posterior abdominal se denomina:


a. Omento
b. Ligamento
c. Mesenterio
d. Fascia transversalis

6. Paciente con herida por proyectil por arma de fuego, con herida de ingreso en región
paraumbilical. Entre las estructuras que usted está seguro que debe haberse lesionado es:
a. Omento menor
b. Ligamento de Treitz
c. Mesosigmoides
d. Omento mayor

7. Paciente joven es traído a emergencia con abdomen agudo quirúrgico debido a herida contuso
penetrante por verduguillo (alambre grueso con punta aguzada) recibida en una pelea después de
un partido de futbol. Se observa herida en Hipocondrio Izquierdo. El órgano que debe estar
sangrando y produciendo hemoperitoneo es (marque la mejor respuesta):
a. Colon ascendente
b. Colon sigmoides
c. Hígado
d. Bazo

8. Al evaluar una tomografía abdominal, el médico asistente le pide al interno de la UPC que
encuentre la imagen con el corte a nivel de L1. El interno sabiamente busca el ________ para
ubicar la vértebra L1.
a. Nacimiento de la vena mesentérica superior
b. Nacimiento de la arteria mesentérica inferior
c. Cuello del páncreas
d. Cruce entre la Aorta y la Vena Porta

9. El ligamento hepatogástrico une el _______________ con el _______________ y forma la


entrada al ___________:
a. Hígado Estómago Orificio esofágico
b. Hígado Estómago Orificio omental
c. Estómago Hígado Orificio anal
d. Estómago Hígado Orificio gastrointestinal

10. En la inspiración, la pared abdominal debe ____________ para ____________:


a. Contraerse aumentar presión intra torácica
b. Relajarse aumentar presión intra abdominal
c. Contraerse aumentar presión intra abdominal
d. Relajarse disminuir presión intra torácica

Calificación Individual 02 - 202002


1. Al seccionar el nervio facial a nivel timpánico, usted esperaría:
a. Imposibilidad para protruir la lengua
b. Disminución del gusto en la punta de la lengua
c. Imposibilidad para el cierre del istmo de las fauces.
d. Ausencia de termoalgesia en la lengua
2. Marque lo correcto en relación al divertículo de Meckel.
a. Se encuentra usualmente a 60 cm de la VIC
b. Contiene mucosa esofágica en algunas ocasiones
c. Se relaciona a un defecto en el desarrollo del intestino posterior
d. Se produce en el lado mesentérico del íleon

3. Paciente obeso con Covid-19 es intubado por interno inexperto, quien al solicitar que bombeen
aire dentro del tubo endotraqueal, nota que el epigastrio se distiende. Al sospechar que ha
introducido el tubo en el estómago, también es cierto que:
a. Disminuye el pH gástrico
b. Aumenta el pH gástrico
c. Disminuiría el tono del píloro
d. Aumenta la frecuencia de ondas lentas

4. En un experimento, con una sonda nasogástrica se instila por goteo en el estómago una
sustancia líquida, y se obtiene como respuesta una dramática disminución del pH del estómago.
Dicha sustancia debe contener:
a. Lípidos
b. Carbohidratos
c. Aminoácidos
d. Secretina

5. Al introducir una solución azucarada directamente al estómago mediante una gastrostomía


(comunicación entre la piel abdominal y el estómago), la sustancia que provocará que aumenten
los niveles séricos de insulina es:
a. Péptido tipo glucagón 1 (GLP-1)
b. Enteroglucagon
c. Glucagon
d. Somatostatina

6. De los diferentes reflejos gastrointestinales, hay uno que produce movimiento del contenido
hacia la región distal, y se llama reflejo:
a. Gastrocólico
b. Entero-gástrico
c. Cólico-ileal
d. Vómito

7. En un paciente con gastroparesia (motilidad lenta del estómago), que presenta distensión
abdominal después de comer, usted le recomendaría que evite el consumo de lípidos y
aminoácidos para disminuir la acción de:
a. Somatostatina
b. Secretina
c. CCK
d. Gastrina

8. Una de las siguientes sustancias reguladoras, puede actuar de forma paracrina y como
hormona. Marque la correcta:
a. Somatostatina
b. Acetilcolina
c. GRP
d. Péptido insulinotrópico dependiente de glucosa.
9. Paciente con apendicitis aguda, que debuta con dolor en mesogastrio. Este dolor se debe a
estimulación de receptores del dolor cuyas fibras van a viajar a la médula espinal a través de:
a. Nervios simpáticos
b. Plexo hipogástrico
c. Nervio esplácnico pélvico
d. Nervio vago

10. En un paciente con hiperestimulación simpática se espera que las ondas lentas tengan un
ritmo:
a. Mayor en íleon que en duodeno
b. Menor en íleon terminal que en el duodeno
c. Mayor en estómago que en duodeno
d. Mayor en estómago que en íleon terminal

Calificación Individual 03 - 202002


1. La comunicación entre la irrigación gástrica y la esofágica depende de una rama de la arteria:
a. mesentérica superior
b. hepática común
c. mesentérica inferior
d. tronco celiaco

2. En un paciente que usa AINEs a altas dosis por artritis reumatoide, usted esperaría que
presente erosiones y úlceras gástricas debido a:
a. el aumento de prostaglandinas
b. la disminución de irrigación en la mucosa
c. el aumento de histamina
d. la inhibición de secreción de secretina

3. Las células enteroendocrinas en el estómago se localizan en la glándula oxíntica, al mismo


nivel que las células:
a. Principales
b. Parietales
c. Absortivas
d. Mucosas

4. En la anemia perniciosa, con atrofia gástrica marcada, usted esperaría encontrar disminución
en la absorción de ___________ a nivel de ________
a. vitamina B12 / íleon distal
b. proteínas / yeyuno
c. hierro / duodeno
d. ácido fólico / yeyuno

5. La saliva siempre será hipotónica debido a:


a. la mayor permeabilidad al sodio en los conductos
b. el estímulo del sistema parasimpático
c. la ausencia de conductor estriado
d. la impermeabilidad de los conductos al agua

6. Un paciente con polimiositis posee alteración en la regulación del mecanismo de la deglución;


por eso hay que considerar la deglución de la saliva, pues a diario se produce _______mL
a. 500
b. 1000
c. 100
d. 300
7. La inhibición de secreción gástrica es secundaria a:
a. la acción de la bombesina
b. el reflejo enterogástrico
c. la estimulación de células G
d. la activación de la bomba de protones

8. Al usar atropina en un paciente, usted esperaría:


a. la hipersalivación
b. la disminución de gastrina
c. el aumento de histamina
d. la disminución de secreción gástrica por bloqueo de M3

9. En una gestante de 11 semanas con antecedente de esofagitis eosinofílica, con hiperémesis


gravídica, que acude a emergencia por hematemesis leve, y presenta súbitamente disnea y dolor
torácico. Usted sospecharía de:
a. Neumonía
b. síndrome de Mallory Weiss
c. esofagitis erosiva severa por reflujo
d. síndrome de Boerhaave

10. Dentro de las patologías que producen sangrado en el esófago, la que sangra más es:
a. el divertículo de Zenker
b. la esofagitis por reflujo gastroesofágico
c. el síndrome de Mallory Weiss
d. el síndrome de Boerhaave

Calificación Individual 04 - 202002


1. En un paciente con infección por SARS-CoV-2 con compromiso severo e ingresado en la
unidad de cuidados intensivos (UCI), debido a la tormenta de citoquinas y a la desregulación
inmune ¿Cuál de las siguientes alternativas estará elevada en sangre al evaluar el perfil hepático?
a. Transaminasas
b. Bilirrubinas
c. Amilasas
d. Fosfatasas

2. En el hígado, el aumento de la resistencia vascular en los sinusoides hepáticos ocasionará:


a. aumento del flujo hacia la vena cava superior
b. aumento del flujo hacia la vena porta
c. salida de plasma hacia el intersticio
d. aumento de la presión de llenado vesicular

3. En un paciente con intoxicación por órganos fosforados, la acción de la colecistoquinina (CCK)


está bloqueada a nivel de:
a. la célula parietal
b. el esfínter de Oddi
c. la vesícula biliar
d. el sistema nervioso central

4. ¿Cuál de las siguientes alternativas es correcta sobre la estructura hepática?


a. El flujo sinusoidal en el lobulillo hepático es de adentro hacia afuera
b. En la triada portal, se encuentra la vena derivada de la suprahepática
c. El flujo biliar en el lobulillo hepático es centrífugo
d. Los colangiocitos producen bilis
5. Paciente con Lupus Eritematosos que desarrolla hipertensión portal debido a trombosis portal,
es probable que desarrolle várices a nivel de:
a. recto inferior
b. recto superior
c. hemorroides externas
d. canal anal

6. La bilis que sale de la vesícula biliar tiene como componente principal:


a. a la bilirrubina
b. a los fosfolípidos
c. a los ácidos biliares
d. al colesterol

7. Marque lo correcto respecto al acino hepático:


a. Dos vértices del rombo acinar se constituyen por triadas portales
b. La zona 3 se afecta en menos en una deshidratación severa
c. El eje menor lo constituye un eje imaginario entre dos venas centrolobulillares
d. La zona 1 es la más que recibe más sangre

8. En un paciente con cirrosis hepática la cabeza de medusa que aparece en la pared abdominal,
podría desaparecer si al paciente se le:
a. administra antiandrógenos
b. oblitera el ligamento redondo
c. esclerosa las venas hemorroides internas
d. oblitera la arteria gástrica izquierda

9. Con respecto a la microestructura del hígado ¿Cuál de las siguientes alternativas es correcta?
a. La célula de Kupffer se encuentra fuera del sinusoide y fagocita células
b. El sinusoide es un capilar fenestrado
c. La célula de Ito se encuentra en el espacio de Disse y reserva glucógeno
d. Los hepatocitos están interconectados por uniones herméticas

10. En un paciente con cirrosis hepática se desarrollará ascitis debido a los siguientes
mecanismos, EXCEPTO:
a. Aumento de reabsorción renal de sodio
b. Estrechamiento de la porta
c. Hipoalbuminemia
d. Hiperflujo portal

1. Al examinar a un paciente, usted encuentra dolor localizado en fosa iliaca derecha y diagnostica
apendicitis. En este paciente, usted puede inferir:
El peritoneo parietal regional está afectado.

2. La motilidad intestinal es estimulada principalmente por el:


Plexo de Auerbach

3. Durante el vómito, ¿el contenido gástrico tiene que pasar necesariamente por cuál estructura para
llegar al esófago? Marque la mejor respuesta:
Cardias

4. Respecto a la anatomía del estómago, marque lo correcto:


El fondo gástrico forma la curvatura mayor
5. Marque la respuesta incorrecta:
En todo el tubo digestivo, se observa dos capas de muscular propia: circular interna y longitudinal
externa

6. Paciente se queja de dolor en hipocondrio derecho, pero superficialmente. El dermatoma relacionado es


(marque la mejor respuesta):
T9

7. Dentro de las funciones del abdomen, se encuentra la defecación y micción, en las cuales la presión intra
abdominal debe:
Aumentar

8. Un alumno de medicina decide hacerse un piercing en el ombligo. Al realizarle el procedimiento,


sangra ligeramente. Esta sangre proviene de la arteria (marque la mejor respuesta) Epigástrica inferior

9. Señale la respuesta correcta:


El apéndice cecal sólo tiene serosa

10. Paciente mujer es traída a emergencia por sufrir una herida contuso penetrante por cuchillo realizada
por su esposo en un ataque de celos. Se observa herida en flanco izquierdo. Esta solución de continuidad
ha comprometido varios músculos de la pared abdominal, excepto: Recto abdominal

11. Paciente con herida por proyectil por arma de fuego, con herida de ingreso en región paraumbilical.
Entre las estructuras que usted está seguro que debe haberse lesionado es:
Omento mayor

12. En la evaluación de una tomografía abdominal, el interno observa un aneurisma en una arteria que se
dirige al riñón derecho. Con seguridad se puede afirmar que está a nivel de la vértebra: L1

13. Paciente joven es traído a emergencia con abdomen agudo quirúrgico debido a herida contuso
penetrante por verduguillo (alambre grueso con punta aguzada) recibida en una pelea después de un
partido de fútbol. Se observa herida en Hipocondrio Izquierdo. El órgano que debe estar sangrando y
produciendo hemoperitoneo es (marque la mejor respuesta):
Bazo

14. Marque el órgano que se encuentra más distal en el tubo digestivo.


Ciego

15. La peristalsis o peristaltismo hace referencia a:


Motilidad para movilizar el alimento de proximal a distal.

16. Marque la respuesta incorrecta:


En todo el tubo digestivo, se observa dos capas de muscular propia: circular interna y longitudinal
externa

17. Paciente con vólvulo del colon sigmoides. La necrosis de ese segmento del colon se produce por una
alteración en la irrigación de la arteria: Mesentérica inferior

18. Marque el órgano que se considera retroperitoneal: Parte de la vía biliar

19. Un alumno de medicina decide hacerse un piercing en el ombligo. Al realizarle el procedimiento, sangra
ligeramente. Esta sangre proviene de la arteria (marque la mejor respuesta): Epigástrica inferior

20. Paciente tiene una úlcera sangrante en el segundo tercio del Yeyuno. La arteria de la cual proviene la
sangre arterial para dicha zona es la arteria: Mesentérica superior

21. Es inervado por aferentes somáticas: Peritoneo parietal


22. Paciente de 24 años con dolor abdominal tipo cólico intenso en mesogastrio. Según sus
conocimientos de macroestructura, el origen del dolor puede ser ….: Íleon

23. Paciente con hipoglucemia secundaria a un insulinoma (tumor neuroendócrino productor de


insulina). El órgano donde mayor probabilidad ha crecido este tumor es: retroperitoneal
24. Al evaluar una tomografía abdominal, el médico asistente le pide al interno de la UPC que encuentre la
imagen con el corte a nivel de L1. El interno sabiamente busca el …….. para ubicar la vértebra L1: Cuello del
páncreas

25. En la inspiración, la pared abdominal debe …. para ….. : relajarse disminuir presión intra torácica

26. Ligamento hepatogástrico une el ….. con el …… y forma la entrada al …… : Hígado Estómago Orificio
omental

27. Al retirar completamente el mesenterio de un órgano, el mismo se vería afectado principalmente en su:
Irrigación

28. La estructura que fija órganos principalmente a la pared posterior abdominal se denomina:
Mesenterio

29. Cuál de las siguientes estructuras no tiene vasos sanguíneos: Epitelio intestinal

30. Al iniciar la digestión, aumenta el consumo de oxígeno por la mucosa. Esto conlleva a una hipoxia
local, lo cual hace que se libere …., el cual produce vasodilatación: adenosina

31. Sustancia que inhibe la secreción y la motilidad del estómago prolongando el tiempo de digestión:
péptido insulinotrópico dependiente de la glucosa (GIP)

32. Marque lo correcto: La hernia fisiológica se produce en la sexta semana y es la salida temporal de asas
intestinales a través del colon umbilical

33. Marque la respuesta correcta en relación a la gastrina: Las células G son las productoras y se
encuentran principalmente en el antro gástrico.

34. El consumir caramelos indirectamente activa la vía: POMC/CART

35. ¿En qué capa se encuentra la alteración principal en el Hirschsprung o megacolon agangliónico?:
Muscular propia

36. Con respecto a las ondas lentas, marque la afirmación correcta: Son contracciones rítmicas
espontáneas

37. El uso de Ranitidina bloquea el receptor H2 de la histamina en las células parietales. La histamina llega
a estas células por: Difusión

38. La triada sintomática: vómitos explosivos post-prandiales, movimientos peristálticos epigástricos


visibles de izquierda a derecha y nódulos palpable epigástrico subcostal derecho, pertenecen a: Estenosis
congénita hipertrófica del píloro

39. Durante una cirugía oncológica, ¿la extirpación de cuál de los siguientes órganos se vería
comprometida por la presencia de adventicia?: Recto

40. En cuanto a los reflejos gastrointestinales, un reflejo que estimula el tránsito intestinal es el reflejo:
Gastrocólico

41. El ligamento falciforme divide al hígado en dos lóbulos derecho e izquierdo. Embriológicamente
deriva del: Mesenterio ventral
42. La presencia de atresias y estenosis duodenales se deben básicamente a una: Falta de recanalización
43. Estudiante de medicina de 20 años, se ha amanecido estudiando para su examen de Sistema Digestivo.
No ha probado alimento desde la cena, por lo que se puede afirmar que la motilidad de esta persona está
siendo regulada por: Motilina

44. Paciente con disminución del apetito marcada asociada a cáncer terminal, para promover la ingesta de
alimentos se podría usar análogos de: Endorfinas

45. Las ondas lentas se producen por la apertura cíclica de canales de: Calcio

46. La forma más común de atresia esofágica contiene: Estenosis proximal del esófago más fístula
traqueoesofágica distal

47. Al deglutir un bolo alimenticio, es lógico suponer que al pasar por el esofago haya un mayor
consumo de oxigeno en la pared del tercio: Proximal

48. Paciente que come entera una pizza familiar de chorizo y queso. Es posible esperar que debido a la
cantidad de alimento ingerida, las ondas lentas hayan: sufrido ninguna alteración en su frecuencia.

49. La hernia fisiológica se produce dentro de: cordón umbilical

50. El crecimiento de un adenocarcinoma de páncreas compromete la pared gástrica por contigüidad


¿Que parte del estómago se esperaría esté comprometido?: Pared posterior del antro

51. Estimula la producción de saliva: Vasodilatación periglandular

52. Durante la secreción de saliva, es de esperarse que las concentraciones de ….. y …… disminuyan al
disminuir el flujo: Sodio Bicarbonato

53. Con respecto a la secreción gástrica de HCI: a mayor secreción de HCI en el lumen gástrico, mayor pH en la
sangre venosa gástrica.

54. Respecto a las enfermedades del esófago, marque lo correcto: el diagnóstico diferencial de la
acalasia es la enfermedad de Chagas esofágica

55. Con respecto a las lesiones y enfermedades de la boca, marque lo correcto: la eritroplasia debe ser
biopsiada
56. Respecto a las glándulas salivales, marque lo incorrecto:
a. la glándula sublingual tiene forma de garfio
b. la glándula sublingual drena a través del conducto de wharton
c. la glándula parótida produce secreción serosa
d. la glandula parotida drena a través del conducto de Stenon

57. El omeprazol actúa sobre la membrana ….. de la célula ….. : apical/parietal

58. Durante el sueño, la concentración de bicarbonato en la saliva: Disminuye

59. Durante el ataque con gas sarín (bloqueador de la acetilcolinesterasa) en el metro de Tokio, en 1995, el
personal de salud noto que los pacientes afectados presentaban: Hipersalivación

60. La célula mucosa del cuello gástrico produce: Moco

Parcial 2020-01 (1)


1. Al evaluar la orofaringe de un paciente, el médico le solicita que abra la boca, saque la lengua y diga a. Al
hacer esta maniobra, nota que el paladar se desvía hacia la derecha, lo cual le hace sospechar que el paciente
sufre de una lesión del nervio craneal: X contralateral
2. Un bolo alimenticio grande y poco masticado se atasca en el esofago, esto ocasiona una sensación de
dolor que es transmitida por los nervios: esplácnicos

3. Para realizar el movimiento mecánico de abrir la boca, primero se necesita: fijar el hueso hioides

4. ¿Cual de las siguientes alternativas se define como la protrusión directa del contenido abdominal a la
cavidad amniótica por un defecto de la pared corporal?: Gastrosquisis

5. Un paciente requiere que se le coloque una sonda de alimentación directamente al estómago


(gastrostomía), el cirujano deberá hacer una incisión en la piel del abdomen ¿cuál de las siguientes
raíces nerviosas debe ser anestesiada para este procedimiento? T8

6. En un paciente de 43 años con tumor carcinoide de páncreas productor de gastrina (Síndrome de


Zollinger-Ellison) se puede esperar encontrar una potenciación del reflejo: gastrocólico

7. El mecanismo de la defecación incluye la participación de diversas estructuras ¿cuál de las siguientes


alternativas es correcta?: Puede ser mediado por un reflejo intrínseco

8. Cuando el contenido del estómago ingresa al duodeno, uno de los reflejos que inhiben el vaciamiento
gástrico es a través del: Sistema nervioso mientérico

9. Durante la masticación, gran parte del proceso masticatorio se debe a: El reflejo masticatorio

10. Las glándulas salivales tienen conductos para la excreción de la saliva, las glándulas …… drenan en las
carúnculas sublinguales. RPTA: Sublinguales

11. Los diferentes segmentos del tubo digestivo son susceptibles de reflejos y movimientos según su
contenido. Si colocamos mediante una sonda un bolo alimenticio directamente en el tercio medio del
esofago: se producira ondas secundarias

12. En una persona si enfermedad se espera que el tránsito intestinal se vea disminuido cuando se
presenta el reflejo: Doloroso

13. El divertículo de Meckel es una anomalía congénita que ocurre por la persistencia del conducto
vitelino y da origen a una estructura sacular, el cual se encuentra en el: borde antimesentérico
14. Si al intubar a un paciente, por error se ingresa el tubo endotraqueal en el esofago y se insufla el
manguito endotraqueal (globo TET), la dilatación de este manguito generará: múltiples ondas
secundarias

15. El orificio omental, o hiato de Winslow, se encuentra limitado por el ligamento: hepatoduodenal

16. Paciente de 24 años acude a consulta externa por presentar una fístula oronasal (comunicación entre la
cavidad oral y la cavidad nasal). Esta fístula es una consecuencia tardía de la lesión de un vaso sanguíneo por
el antecedente de haber sido operado de paladar hendido en los primeros años de vida, aparentemente en
una campaña gratuita de corrección de paladar fisurado. ¿Cual de las arterias palatinas podría haberse
lesionado durante esa cirugía?: Mayor

17. Dentro de las anomalías congénitas se puede presentar un tejido pancreático accesorio ¿cuál es la
ubicación más común de este tejido?: Estómago

18. Una recién nacida es evaluada por el neonatólogo y evidencia que el canal anal está completamente
cerrado. Este problema se debe probablemente a una anomalía en el desarrollo de: la membrana cloacal

19. En la digestión de los alimentos, la hormona __________ se libera frente a la presencia de


péptidos y monoglicéridos, y tiene un efecto marcado en la disminución del vaciamiento gástrico →
colecistoquinina
20. Dentro de las anomalías congénitas se puede presentar un tejido pancreático accesorio ¿ Cuál
es la ubicación más común de este tejido? → Estómago

21. Los catadores de vino tienen una habilidad increíble al momento de separar los sabores. Este aumento
de la sensibilidad gustativa debido a una mayor cantidad de papilas linguales y de corpúsculos gustativos se
conoce como: hipergeusia

22. En muchos países se usa el suplemento de flúor en el agua potable o los dentífricos, con el fin de
hacer el esmalte más resistente a la desmineralización inducida directamente por: el ácido

23. Durante el desarrollo de la región cloacal, una cuña de mesodermo ubicado entre el alantoides y el
intestino posterior vendrá a formar el: tabique urorrectal

24. En una persona sana, el momento adecuado para encontrar los mayores niveles de grelina en sangre
sería: antes de comer

25. El duodeno está constituido por el segmento terminal del intestino anterior y el segmento proximal del
intestino medio ¿Cuál de las siguientes alternativas describe mejor este lugar de unión entre los dos
intestinos?

Distal al origen de la yema hepática


26. En una persona sana, el uso de atropina producirá a nivel del estómago: Aumentará el pH del
estómago

27. En una persona sana, el consumo de leche produce indirectamente → Inhibición del vaciamiento
gástrico
28. La motilidad del colon es importante y lenta comparada con la del intestino delgado. Los
movimientos en masa ocasionan la: distensión rectal
29. Con respecto a la motilidad gástrica, los potenciales de acción disminuyen en frecuencia por efecto de:
el péptido insulinotrópico dependiente de glucosa

30. La sensación del gusto depende de la presencia de papilas gustativas en la lengua, de las cuales, algunas
de ellas tienen un surco terminal por donde drenan unas glándulas salivales linguales (llamadas glándulas de
von Ebner). Esta descripción se refiere a las papilas: circunvaladas

31. Dentro de la estructura de los dientes, la parte del diente cubierta por esmalte y que se puede
ver mediante la inspección visual de la boca se denomina → corona clínica

32. En el conducto anal se encuentra la unión entre las regiones del endodermo y el ectodermo, esta
unión se evidencia al observar: la línea pectínea

33. La sensación del gusto depende de la presencia de papilas gustativas en la lengua, las cuales tienen
corpúsculos gustativos conteniendo células neuroepiteliales sensoriales. Estas células neuroepiteliales
pueden ser dañadas fácilmente, por suerte, su tiempo de recambio es de alrededor de: 10 días

34. En un varón de 47 años con sección medular a nivel de T6 debido a un accidente automovilístico, sus
terapeutas han desarrollado un mecanismo para distender el recto e iniciar el reflejo rectoesfinteriano, lo
cual producirá la contracción de: la pared del recto

35. Durante un experimento, se insufla rápidamente dos litros de agua en un globo colocado
dentro del estómago de un voluntario. ¿cuál de las siguientes situaciones del
músculo liso será consecuencia directa de este cambio de volumen en el estómago? →
Despolarización
36. Paciente de 56 años con accidente cerebrovascular reciente. En la resonancia se observa daño de los
núcleos laterales del hipotálamo. Por este motivo es muy probable que el paciente sufra de: inanición

37. Paciente con insuficiencia mitral moderada a severa, con aumento de volumen de la aurícula
izquierda, esta condición tendrá como consecuencia a nivel del sistema digestivo: la disfagia a sólidos

38. El inicio de la fase faríngea de la deglución se debe a estímulos sensitivos que viajan por el nervio
craneal: V

39. El mesocolon transverso se origina en: la pared posterior del abdomen

40. El esofago en su microestructura tiene basicamente adventicia, a excepción de la región distal,


donde tiene serosa, específicamente a partir del nivel de: T10

41. En un paciente con arcadas, se debe considerar que durante la ocurrencia de dichas arcadas,
debemos encontrar contenido gástrico en: Tórax

42. La reabsorción de Sodio y Cloro en las glándulas salivales se da principalmente en el: Conducto
estriado

43. Para que se puedan digerir las grasas, es preferible que primero sean emulsificadas. La hormona que
estimula la liberación de las sustancias emulsificadoras es: CCK
44. Paciente de 13 días de vida con vómitos explosivos a las 2 horas después de lactar. Al examen físico se
palpa la oliva pilórica ¿cual es el nervio cuyos filetes dan inervación eferente a la estructura afectada?: Vago

45. La localización de la vesícula biliar con respecto al lóbulo cuadrado es: Lateral

46. La triada portal está constituida por el conducto biliar y la arteria hepática y una pequeña rama de la
vena: Porta

47. Al ingerir una cantidad de glucosa por vía oral, esta es interiorizada en las células del organismo más rápido
que si esa misma cantidad hubiese sido administrada por vía endovenosa. Este fenómeno sucede gracias a una
sustancia secretada por las células: K

48. Paciente de 62 años con vólvulo de intestino delgado e isquemia intestinal. ¿Qué estructura se utiliza
como punto de referencia para determinar la posición de la unión duodenoyeyunal? : Ligamento
suspensorio del duodeno (de Treitz)

49. En un paciente con Zollinger Ellison, usted esperaría encontrar: Esteatorrea 50.

La motilidad intestinal es estimulada por: Colecistoquinina y gastrina

51. La onda peristática secundaria del esofago en la deglución, es producida por: Plexo mientérico
esofágico

52. Paciente con parálisis bilateral del nervio hipogloso, el unico musculo de la lengua que conservará su
movimiento es el: palatogloso

53. ¿Cual de las siguientes sustancias tiene mayor concentración de la saliva comparado con su
concentración plasmática? Potasio

54. ¿Cual de las siguientes estimula la secreción enzimática exocrina del páncreas?: Colecistoquinina

55. Al comer un pollo a la brasa, con papas fritas y ensalada, la sustancia que estimula la liberación de HCI
en el estómago es: Bombesina

56. Marque lo correcto con respecto a Esófago de Barret: Se relaciona con reflujo gastroesofágico
ECU 1
Estudiante de medicina de la UPC de 21 años sufre de gastritis aguda ocasionada por comer en lugares poco
higiénicos. Suele consumir caramelos ( chupar ) mientras está en clase hasta la tarde. Toma gaseosas
regularmente (carbohidratos 46%, sodio 53%). También toma regular cantidad de leche (grasa 35%, lactosa
35%, proteínas 30%), pues le calma un poco el dolor el ardor que siente por la gastritis. Incluso, cuando
puede, se toma dos vasos de agua fría para calmar las molestias. Ha decidido ir al médico para tratarse pues
ya no soporta el dolor, el cual está seguro que los síntomas se deben a una elevada producción de ácido
clorhídrico en el estómago, y por ello le ha recetado Ranitidina (antihistamínico), con lo que siente mejoría.
El uso de atropina en este paciente:
- Aumentará el pH del estómago
Entre las sustancias cerebrales que producen ansiedad está la serotonina, la cual también tiene acción: -
Anorexigénica
El consumo de dos vasos de agua seguidos agua generará indirectamente un aumento en la liberación de:
- Ácido clorhídrico
En este paciente con gastritis aguda debida a una alta producción de ácido clorhídrico, sería lógico
esperar que el píloro tenga un tono muscular:
- Aumentado
El consumo de leche produce directamente un aumento de los niveles séricos de la hormona: -
Colecistoquinina (CCK)
El consumo de leche produce directamente un aumento de los niveles séricos de la hormona: -
Gastrina

ECU 2
Niño de sexo masculino de 2 años de edad, sufre de estreñimiento desde el nacimiento (1 deposición cada 3-
4 días). Madre menciona que le estimula la defecación con un termómetro rectal, y continuo uso de enemas
y laxantes. Desde hace 6 meses comienza con vómitos postprandiales. Los síntomas aumentan en frecuencia
y magnitud y están en relación con los episodios de estreñimiento. No refiere fiebre, tos, diarrea ni lesiones
cutáneas. Al examen físico presenta regular estado general, luce deshidratado. Abdomen distendido, blando,
depresible e indoloro. No se palpan masas abdominales. Se permeabiliza el canal anal con termómetro rectal,
encontrando cierta resistencia. Salida de material fecal mal oliente en regular cantidad. Exámenes de
laboratorio: hemograma normal. Signos inflamatorios de fase aguda negativos. Alcalosis metabólica leve en
sangre venosa.
Radiografía con enema baritado muestra recto y colon sigmoides dilatados (megacolon). Biopsia
profunda: ausencia de células ganglionares en la muestra enviada. Se realiza cirugía correctiva. El
contenido fecal se detiene en la zona inmediatamente proximal a la zona donde hay una menor
presencia de:
- Péptido intestinal vasoactivo
En cuanto a los reflejos gastrocólico y gastroduodenal en este paciente, indique lo correcto: - Se
pueden considerar reflejos vago-vagales
En este paciente se considera que está abolido el reflejo:
- Rectoesfinteriano
Debido al acúmulo de material fecal en todo el marco colónico, y a la irritación química asociada, el
peristaltismo del íleon distal se debe encontrar:
- Inhibido
Es un reflejo propio de la pared intestinal:
- Peristaltismo
A diferencia de las arcadas, los vómitos presentan apertura de:
- Esfínter esofágico superior

1. Un niño de 2 años es llevado a la consulta por diarrea persistente y edema de las


extremidades, además falta de crecimiento y desarrollo en relación a su edad. Los análisis
de sangre revelan que tiene concentración plasmática baja de proteínas (hipoproteinemia).
Durante la endoscopía duodenal, se coloca colecistocinina (CCK) endovenosa y se recoge
muestras del líquido duodenal; el resultado del líquido confirma incapacidad para hidrolizar
proteínas a un pH neutro, esta situación mejora al añadir una pequeña cantidad de tripsina.
El paciente probablemente esté sufriendo la falta congénita de
-Enterocinasa

2. Experimentalmente se incrementa la velocidad de la secreción salival con una


sustancia, el análisis de la composición de esta saliva obtenida se espera
encontrar…………..

-Disminución de concentración de potasio

3. Paciente varón de 46 años soltero, consulta por odinofagia y bajo de peso, tiene
antecedente de tuberculosis desde hace 3 meses y es fumador crónico (10 cigarrillos por
día); al evaluar la cavidad oral se identifica lesión blanquecina en el dorso de la lengua y
paladar blando, las lesiones se desprenden con el baja lengua dejando una base
eritematosa. Esta lesión corresponde probablemente a

……………………….…..
-Candidiasis oral

4. Paciente mujer de 35 años acude a consulta por sensación de sequedad y lesiones en


cavidad oral. Al examen se observa atrofia de la mucosa, fisuras y úlceras; nota además
sequedad e irritación de la córnea y aumento del tamaño de las glándulas parotídeas. Su
diagnóstico más probable es artritis reumatoide; el hallazgo más probable en una biopsia
de glándula parótida es……..….

-Gran infiltración de linfocitos y células plasmáticas

5. Un paciente con anemia acude con su médico quejándose de episodios frecuentes de


gastroenteritis. Un análisis de sangre revela anticuerpos circulantes dirigidos contra células
parietales gástricas. Su anemia es atribuible a la hiposecreción de
-Factor intrínseco

6. Dos estudiantes deciden tomar un receso para comer una hamburguesa a la hora del
almuerzo. Antes de llegar a la cafetería, impulsos nerviosos provenientes del
complejo vagal dorsal iniciarán la secreción de ácido gástrico por la liberación de

…………………….. desde el sistema nervioso entérico.


-GRP

7. Un niño de cuatro años de edad es llevado a la consulta por cuadros diarreicos


frecuentes caracterizados por heces pálidas, voluminosas y fétidas, presenta bajo peso y
talla. Se mide la concentración de cloruro en el sudor y se encuentra que sus valores son
muy elevados. La alteración más importante a nivel de células ductales del páncreas tiene
relación directa con la conductancia de…………

-Cloro

8. Una mujer de 50 años de edad que sufrió durante varios años resequedad de los ojos
debida a producción inadecuada de lágrimas es enviada con un gastroenterólogo para
evaluación de pirosis crónica. El examen endoscópico revela erosiones y tejido cicatrizal
en la parte distal del esófago justo por arriba del esfínter esofágico inferior. Las lesiones
pueden atribuirse a la disminución de uno de los siguientes componentes salivales:

-Bicarbonato

9. Se evalúa los valores séricos de las siguientes sustancias a un paciente con


enfermedad hepática terminal; en este paciente se espera encontrar la combinación con
la letra …………
-disminuida, aumentada, disminuida

10. Una mujer de 35 años de edad HIV positiva, se presenta al médico con dolor
abdominal en cuadrante superior derecho e ictericia. La paciente refiere haber tenido
múltiples episodios de ictericia durante los últimos 10 años. Los exámenes para
determinar hepatitis viral, dieron positivos para Hepatitis B, siendo catalogado el caso
como hepatitis crónica con alteración funcional. En un examen de sangre ¿cuál de los
siguientes parámetros está disminuido?

-Albúmina

11. En el reflejo peristáltico del intestino delgado, uno de los siguientes eventos
sucede en la porción oral del bolo alimenticio…………...

-Acción de acetilcolina en el músculo circular

12. Experimentalmente se coloca una dosis alta de secretina en la luz intestinal


duodenal; como consecuencia de esto, en el jugo pancreático de la misma luz
intestinal se observa la disminución de la concentración de …..………..

-Cl

13. Un varón de 58 años de edad con enfermedad de Crohn severo fue sometido a una
resección ileal. Después de la cirugía este paciente padecerá de esteatorrea, esto se
explica porque …..………..

- La micelas no pueden formarse


14. En un experimento se inserta un balón en el estómago de un voluntario, se infla poco
a poco mientras que se vigilan las presiones intraluminales. Aunque el volumen del balón
aumenta considerablemente, las presiones permanecen constantes. Esta relación
volumen-presión se explica por la liberación local de …………..

-Óxido nítrico y péptido inhibidor vasoactivo

15. La toxina de Vibrio cholerae causa diarrea debido a…….

-El Incremento de la secreción de cloro por las células de la cripta intestinal

16. ¿Cuál de las siguientes alternativas es una característica de la secreción exocrina del
páncreas?

-Tiene una baja concentración de Cl- respecto al plasma

17. Una madre lleva a su hijo de dos años de edad a la sala de urgencias, estresada porque
el niño deglutió una moneda de 10 céntimos mientras la familia cenaba en un restaurante.
El médico observa mediante fluoroscopía que la moneda se halla en el
estómago y asegura a la madre que la moneda se eliminará con las heces. El médico
recomienda utilizar la respuesta fisiológica que permitirá la evacuación de la moneda del
estómago al intestino ………….…..

-Son los movimientos de mezcla y trituración


-. Es provocada por el ayuno

18. Las estructuras en el hígado que permite que los productos metabólicos unidos a
proteínas tengan acceso a las membranas basolaterales de los hepatocitos, son….. - Las
fenestras sinusoidales
19. La composición de la bilis es modificada conforme fluye por los conductillos
biliares. Durante este tránsito se espera que aumente la concentración de…….

-Monómeros de ácido biliar


-Ig A

20. Se mide experimentalmente el contenido gástrico de dos personas. La persona “A”


tiene alto contenido de grasa y la persona “B” tiene un contenido hipertónico ¿Cuál de las
siguientes es correcto respecto al vaciamiento gástrico?
- Hay ralentización del vaciado gástrico en ambos casos

21. El examen endoscópico de un paciente con hipertensión portal grave revela venas
tortuosas que sobresalen hacia la luz del esófago. El paciente recibe tratamiento quirúrgico
mediante la colocación de una derivación que conecta la vena porta a la vena cava.
Después de la operación el riesgo de encefalopatía y el

riesgo de sangrado de várices ……………..


-Aumentará/disminuirá

22. Un paciente varón de 18 años de edad acude al médico para sus exámenes de rutina.
Sus resultados de laboratorio muestran un valor de bilirrubina sérica de 4 mg/dl y una
bilirrubina directa de 0,3 mg/dl. Las pruebas de función hepática son normales. La
alteración que explica mejor este caso es por la deficiencia de

………………..
-Glucuronil transferasa

23. Un hombre de 57 años de edad es llevado a urgencias con hematemesis masiva rojo
brillante, a su llegada se halla inconciente con PA: 80/40 mm Hg y FC: 124 lat/min. Luce
ictérico con presencia de “arañas vasculares en el tórax anterior y extremidades”, abdomen
distendido con signo de oleada positiva. Se encuentra esplenomegalia y pérdida de la
masa muscular en extremidades. La anastomosis vascular responsable del sangrado en
este paciente es

-Vena gástrica izquierda y vena ácigos

24. Un estudiante de medicina está comiendo un plato de comida a base de


champiñones, espárrago y salsa de soya. El sabor umami contenido en todos estos
alimentos actúa a nivel de los botones gustativos estimulando ………………..

-Un receptor acoplado a proteína G


25. Un hombre de 22 años de edad se presenta al médico con una historia de 1 año de
evolución caracterizado por dolor recurrente en fosa iliaca derecha y diarrea. Manifiesta
además pérdida de peso de 8 kg durante este periodo. La colonoscopía revela múltiples
lesiones en el ileon terminal y colon. La biopsia de estas lesiones revela engrosamiento,
inflamación y ulceración de la mucosa. El diagnóstico más probable en este caso
es…….

-Enfermedad de Crohn

26. Varón de 61 años que consulta por dolor retro esternal intenso desde hace 6 horas y
después de vómitos intensos y repetidos; al examen se observa disnea, cianosis,
hipotensión y signos clínicos de shock. La radiografía simple de tórax muestra
neumomediastino. El líquido en el espacio pleural aspirado tiene alta concentración de
amilasa. ¿Cuál de las siguientes alternativas puede explicar este cuadro clínico? –

Rotura espontánea de esófago


27. La secreción del ácido en la célula parietal gástrica se lleva a cabo por una
ATPasa especifica que intercambia hidrogeniones (H+) del citosol por…..
-K +

28. En condiciones normales el ingreso de 600 ml de líquido es el estómago provoca un


aumento de presión intragástrica de unos 12 cm de H2O. Después de una vagotomía
(corte del nervio vago) es de esperar que el ingreso del mismo volumen de líquido
provoque lo siguiente: …………………………………

-Un aumento mayor de la presión

29. Una paciente de 30 años de edad es sometida a una cirugía en oído medio derecho
por un problema de otoesclerosis. Luego de la cirugía refiere alteración en la percepción
de sabores. Al evaluar el caso usted esperaría encontrar……….

-Alteración en la sensación del gusto en los dos tercios anteriores de la


lengua
-Sensación del dolor, tacto y temperatura conservada en toda la lengua 30.

¿Cuál de las siguientes alternativas es correcta?

-Las sales biliares desconjugadas son absorbidas preferentemente en el colon

31. En un paciente de 45 años de edad con colestasis biliar, se encuentra una elevación
de los niveles sanguíneos de fosfatasa alcalina hasta 3 veces la cifra normal. ¿Cuál de
las siguientes alternativas estará también elevada como evidencia del daño de la vía
biliar?

-Gamma glutamil transpeptidasa

32. Revisando la angiografía de un hombre de 70 años en estudio por aneurisma de aorta


abdominal el radiólogo informa de la presencia de una oclusión completa de la arteria
mesentérica inferior. El paciente se encuentra completamente asintomático.

¿Cuál de las siguientes arterias se anastomosa a la sistema arterial de la mesentérica


inferior?
-Cólica media

33. Lactante de 3 meses de vida es atendido por presentar diarrea, se administra una
solución de glucosa y electrólitos por vía oral. La proteína de membrana apical que
explica la capacidad de esta solución para proporcionar aporte de glucosa e hidratación
es ………..

-SGLT-1

34. Paciente ha sufrido herida de bala en el abdomen, se le ha tenido que extirpar el


segmento medio y distal del ileon. En este caso la síntesis hepática de sales biliares estará
…..…..

-Incrementada por estímulo de la enzima colesterol 7 alfa hidroxilasa

35. Un varón de 75 años ingresa al consultorio por presentar ictericia marcada de piel y las
escleras. El estudio del paciente mostró que presentaba un tumor que obstruía la totalidad
del conducto hepático común. ¿Cuál de las siguientes estructuras se encontrará dilatada
en este paciente?
-Conductos de Hering
36. En un paciente con insuficiencia renal crónica, el déficit en la absorción de calcio a
nivel del enterocito se debe a lo siguiente:

-No se convierte la 25 hidroxicolecalciferol a 1,25 dihidroxicolecalciferol

37. Varón de 30 años es traído a emergencia por agresión abdominal con arma de fuego
(pistola) y es sometido a laparotomía exploratoria, observándose isquemia del colon
ascendente y parte del colon trasverso ¿la lesión de cuál de las siguientes arterias explicaría
esta isquemia?

d. Mesentérica superior

38. Respecto a las sustancias gastrointestinales que regulan la secreción pancreática;


marque la afirmación correcta:

b. La acetilcolina es capaz de estimular la secreción enzimática y de bicarbonato del


páncreas
39. Ante una lesión del X par craneal, ¿cuál de los siguientes músculos mantiene
conservada su función?:

b. Tensor del velo del paladar

40. Experimentalmente se utiliza atropina (anticolinérgico) para inhibir la secreción de gastrina,


sin embargo la secreción de esta hormona se sigue dando ante estímulos vagales. Esta
situación se explica porque la atropina:

d. No bloquea la acción del péptido GRP

41. Un varón de 50 años es sometido a extirpación del duodeno y parte proximal del
yeyuno. Esta situación ocasionaría la pérdida de las células ……….. , productoras de
………………… que estimula la secreción de bicarbonato por el páncreas.

“S” / secretina

42. Se evalúa la expresión de la proteína Agrp en una persona con alteración del apetito; lo
correcto respecto a esta proteína es…..

La mutación del gen que la codifica produce adelgazamiento

43. Juana cae de la bicicleta y se fractura la región anterior del hueso maxilar superior con
compromiso de la fosa incisiva. Al examen físico de la región esperaría encontrar alteración en
la sensibilidad de la encía …………………
palatina anterior

44. Recién nacido es atendido por el neonatólogo y luego entregado a su madre para dar de
lactar; la madre al dar de lactar observa coloración azulada de labios, acompañado de tos
persistente, dificultad respiratoria y distención abdominal. Se le intenta colocar una sonda
nasogástrica pero esta retorna a la cavidad oral en todos los intentos. ¿Cuál de las siguientes
anomalías del desarrollo es el más probable en este caso?

b. Atresia esofágica proximal con fístula traqueoesofágica distal l


45. ¿Cuál de los siguientes mecanismos ocurre durante la defecación? En

la posición de “cuclillas” el músculo puborrectal se halla relajado

46. Un paciente luego de un accidente sufre lesión del piso de la boca, se constata daño del
nervio “cuerda del tímpano”, en este caso se esperaría en

47. contrar disminución de la………………………….… de la lengua

Sensación del gusto en los dos tercios anteriores

48. ¿Cuál de las siguientes afirmaciones es la correcta sobre la gastrina?


Actúa en la célula diana mediante su receptor CCk tipo B

49. Al recibir un paciente con signos de hipovolemia y antecedente de trauma en abdomen por
accidente de tránsito, usted identifica radiológicamente: lesión de primera vértebra lumbar y
signos de lesión en páncreas; durante la cirugía se observó pobre irrigación de asas intestinales.
El vaso afectado es la arteria ……..

c. mesentérica inferior

50. Un paciente sufre de daño a nivel del cuello con lesión muscular en la región de la faringe.
En el examen físico se determina dificultad para la elevación de la faringe y para el cierre del
itsmo de las fauces. En este caso, probablemente esté afectado el músculo:

c. palatofaringeo

51. Varón de 50 años a quien le realizan la curación de la segunda molar de la arcada superior
derecha. En un momento determinado, el paciente acusa de intenso dolor de la pieza dentaria
en tratamiento. La vía aferente del dolor viaja a través del nervio …………
a. trigémino V2

52. La distención gástrica por los alimentos produce incremento de secreción de HCl
mediante la producción de ………….. que estimula a las célulasvía proteína

………..
a. gastrina / parietal / Gq

53. Un niño de tres años llega a emergencia con disfagia (dificultad para tragar), dolor retro
esternal, salivación y llanto. Se sospecha de ingesta de cuerpo extraño (moneda) en el
esófago; al ser evaluado se constata en una radiografía presencia de cuerpo extraño a nivel
de C6 (6° vértebra cervical). El cuerpo extraño estará suspendido a nivel del estrechamiento
producido por………..

c. el músculo cricofaríngeo

54. La triada portal (arteria hepática, vena portal y conducto biliar común) está contenida en
el ligamento …….……… y derivan embriológicamente del ……

a. hepato duodenal / mesenterio ventral


55. Un paciente refiere no percibir algunos sabores, al examen físico se constata alteración en
la percepción de sabores y del dolor en el tercio posterior de la lengua ¿Cuál de los siguientes
nervios estará alterada en su función?

c. Glosofaríngeo (IX par)

56. En el caso de un paciente con gastrinoma (tumor productor de gastrina), la presencia de


úlceras duodenales y erosión de la mucosa gástrica, se debe principalmente a……. c. el
exceso de HCl por estímulo de receptores CCK-B en la célula parietal

56. El reflejo entero gástrico se caracterizan por:


d. originarse debido a la distensión duodenal y presencia del quimo ácido

57. Mauricio tiene dificultad para deprimir el paladar y elevar la parte posterior de la
lengua. En este caso estará afectado un músculo, específicamente el músculo
…………….
b. extrínseco – palatogloso

58. En condiciones normales, el ingreso de 600 ml de líquido es el estómago provoca un


aumento de presión intragástrica de unos 12 cm de H2O. Después de una vagotomía (corte
del nervio vago) es de esperar que el ingreso del mismo volumen de líquido ocasione
………………………………… de la presión intragástrica.
c. un aumento mayor

59. La explicación fisiológica de presentar somnolencia de 30 minutos a 1 hora después de


ingerir alimentos, se explica por: a. Aumento del cloro intraluminal
e. Aumento de la alcalinidad sanguínea
60.Se presenta un paciente, el cual presenta un antecedente de tuberculosis intestinal, por lo
cual, se le resecó 80 cm de íleon distal. Desde el punto de vista fisiológico, el paciente puede
presentar una de las siguientes alteraciones: a. Disminución de la secreción de Vitamina B12
e. Disminución de la absorción de ácido glicocólico

61. Un paciente es sometido experimentalmente a un fármaco que modifica el flujo salival,


obteniéndose un volumen de saliva de 288 ml en 6 horas. En este caso las concentraciones de
electrolitos y bicarbonato en la saliva obtenida varían de la siguiente manera: a. ↑ Na+,
↓ K+, ↑ Cl-, ↑ HCO3-
1. b. ↓ Na+, ↓ Cl-, ↑ K+, ↓ HCO3-

62. Uno de los siguientes elementos debería hallarse con más probabilidad en el esófago de
un paciente que sufre de reflujo gastro esofágico…
a. Pepsina

63. Un paciente de 40 años cursa con anemia de 8g/dl, aqueja además de astenia y sensación
de hormigueo bilateral en los miembros inferiores, al examen se halla alteración de la
sensibilidad a la vibración y camina con ampliación de la base de sustentación. Uno de los
siguientes procedimientos sería de ayuda para el diagnóstico de este paciente: a. Tomografía
cerebral
b. Biopsia de la mucosa gástrica

64.Paciente de 60 años ingresa por caída hace 1 hora y pequeño hematoma en cuero
cabelludo, al examen físico ampliado se observa ictericia de piel y mucosas generalizada,
abdomen blando, se palpa estructura quística no dolorosa en hipocondrio derecho que
corresponde a vesícula biliar (signo de Courvoisier), en los exámenes de laboratorio se halla
niveles bajos en la formación de estercobilinógeno y urobilinógeno en heces, incremento de
la bilirrubina conjugada en la orina, elevación de fosfatasa alcalina y gamma glutamil
transpeptidasa séricas. El presente cuadro puede ser explicado por: a. Reabsorción de
hematoma
c.Carcinoma de la cabeza de páncreas

65. Un recién nacido presenta vómitos biliosos poco tiempo después de cada alimento. Al
preguntar a la madre sobre antecedentes, ella recuerda que tuvo polihidramnios durante la
gestación, pero un análisis de cariotipo fue normal. Una de las siguientes es la causa más
probable de estos hallazgos en el recién nacido: a. Enfermedad de Hirschprung e. Malrotación
de la yema pancreática ventral

66.En un estudio de la secreción de hormonas gastrointestinales, sus concentraciones en la


vena porta se midieron durante perfusión luminal del intestino delgado con soluciones de
diversas magnitudes de pH. ¿Qué hormona aumentará en el plasma de la vena porta durante
perfusión a través del intestino con una solución de pH 3?
a. CCK
e. secretina

67.Paciente de 30 años que ingresa a causa de un traumatismo abdominal cerrado. En la


exploración se aprecia discreta palidez de piel y mucosas, auscultación pulmonar normal,
taquicardia de 120 /min. Discreta distensión abdominal y matidez en flancos; el hematocrito,
que era prácticamente normal al ingreso, disminuye a 30% a las tres horas. En la Rx de tórax
se objetiva fractura de las costillas 10-11 izquierdas. La causa más probable de la anemización
en este paciente es: a. traumatismo renal con hemorragia retroperitoneal.
c. rotura de bazo con hemoperitoneo.
68. Revisando la angiotomografía de un hombre de 70 años en estudio por aneurisma de
aorta abdominal, el radiólogo le informa de la presencia de una oclusión completa de la
arteria mesentérica inferior. El paciente se encuentra completamente asintomático. La
oclusión de la arteria mesentérica inferior cursa de manera asintomática en muchas
ocasiones ya que el territorio que irriga puede recibir flujo proveniente de la arteria: a. cólica
derecha
e. cólica media

69. En las patologías de esófago es importante conocer bien la anatomía esofágica. ¿Cuál de
las siguientes afirmaciones es correcta? a. El esófago tiene capa mucosa, muscular y serosa
c. El esófago torácico pasa por detrás del cayado aórtico

70. A pesar de que pueda haber variaciones anatómicas, lo habitual es que el ciego sea
irrigado por una rama arterial que proviene de unas de las siguientes arterias: a. Iliaca
derecha
d. Mesentérica superior

71. Ante un paciente con una cirugía abdominal urgente, el informe operatorio señala que se
ha realizado una resección de todo el duodeno y del tercio proximal del yeyuno manteniendo
íntegros el estómago y todo el íleon, así como los dos tercios distales del yeyuno. En el
seguimiento nutricional del paciente ¿Qué vitamina o mineral presentará con menor
probabilidad una disminución de su absorción?
a. Cianocobalamina

72. ¿Cuál de las siguientes sustancias forma parte de la secreción biliar? a. Tripsina
Lecitina

73.¿De qué musculo forma parte el ligamento inguinal?


-Oblicuo externo del abdomen

74.¿Cuál de las siguientes enzimas está localizada en el borde en cepillo y juega un rol en la
digestión de proteínas?
e. Carboxipeptidasa A.

75. Una de los siguientes sustancias, NO sirve como un buen agente emulsificante: a.
Colesterol

76. La sustancia que estimula el crecimiento de la mucosa gástrica es: a.


Secretina
d. Gastrina

77.¿Cuál de las siguientes alternativas es una función de la colecistokinina? a.


Relajación de la vesícula para la salida de bilis
d. Secreción de enzimas pancreáticas

78.Con respecto a la anatomía del tronco celiaco, señale lo correcto a. El tronco celiaco se
origina de la cara posterior de la aorta abdominal
d. La hepática común que es una de sus ramas, participa en la irrigación del estómago.

79. Con respecto a la anatomía del duodeno, marque la respuesta correcta: a. Tiene una
distribución en forma de “C”, que rodea la cola del páncreas
b. La 3ra porción duodenal está contenida en la pinza vascular aortomesentérica

80. En el íleon se absorbe aproximadamente el 95% dea través de la circulación


enterohepática.
a. agua
c. sales biliares

81. La estimula el mecanismo paracrino de la secreción de ácido clorhídrico. a.


histamina

82.En la digestión de proteinas,es el principal estímulo para convertir el


pepsinógeno en pepsina. a. la gastrina
b. el pH ácido

83. Con respecto a la somatostatina, marque lo correcto:


a. Es secretada por las células S del intestino
Interviene en la fase intestinal de la secreción gástrica

84. En pecten anal, es una estructura comprendida entre: a. la línea pectínea y los senos
anales
d. la línea anocutánea y la línea pectínea

85.¿Cuál de las siguientes alternativas es una proenzima pancreática? a. Tripsina 1. b.


Elastasa
2. c. Quimotripsinógeno
3. d. Amilasa
4. e. Procarboxipeptidasa C

86. Eneste paciente, el bloqueo farmacológico de los receptores H2 en la mucosa


gástrica:
a) No tiene efecto sobre la secreción de ácido inducida por el vago b)
Evita la activación de adenilciclasa por gastrina
c) Inhibe la secreción de ácido inducida por gastrina y mediada por el vago d) Causa un
aumento en el transporte de potasio por las células parietales gástricas Se validó la A :)

87. Si se considerara una gastrectomía total para curar la gastritis del paciente, cuál de las
siguientes sustancias ya no se produciría:
a) Gastrina
b) Quimiotripsina
c) Amilasa

d) Pepsinógeno

88. Un paciente hipertenso está tomando un medicamento bloqueador de receptores alfa


1 adrenérgicos (prazosina) y como efecto secundario se queja de: d) Lo escaso que es el
medicamento
e) No tiene problemas en la salivación

c) Hiposalivación
d) Hipersalivación

89. Con respecto a las lesiones y enfermedades de la boca, marque lo correcto: f)


La leucoplasia se desprende al roce

b) la eritroplasia puede degenerar en adenocarcinoma


c) El muguet oral es una enfermedad bacteriana en inmunodeprimidos d)
la eritroplasia debe ser biopsiada
NOTA: fue validada la opción B ya que no es motivo del curso que sepamos el puto
cáncer.

90. En este paciente, se puede asumir que la pancreatitis ha sido ocasionada por una
disminución en el efecto de:
g) Amilasa
h) Lipasa

c) Inhibidor de la tripsina
d) Entercinasa

91.Un efecto secundario en el estómago por la acción de la secretina es:


i) Disminución en la liberación de pepsinógeno

b) Menor actividad de la pepsina


c) Mayor paso de bicarbonato a sangre periférica
d) Aumento en la producción de factor intrínseco
92. Dentro de los factores protectores de la mucosa gástrica se pueden mencionar
múltiples protagonistas. Uno de ellos es:
j) CCK
k) Gastrina

c) Receptor muscarínico
d) Pepsina

93. La saliva puede tener una variedad de electrolitos en su composición. Entre ellos el
cloro, respecto al cual se puede afirmar:
l) Su mayor concentración se consigue con el flujo bajo

b) Su concentración no llega a ser tan alta como en el plasma


c) Con flujo alto su concentración es mayor que la del plasma
d) Su menor concentración se alcanza con flujo alto

94. En el síndrome de boca seca o síndrome de Sjogren, una de las complicaciones


asociadas es:
a) caries
b) Disminución de la acidez gástrica
c) Aumento en de la producción de saliva
d) Aumento del pH bucal

95. Estimula la producción de saliva:


a) Vasodilatación periglandular
b) Atropina
c) Fatiga o cansancio
d) Expresión de miedo

96. El omeprazol actúa sobre la membrana de la célula


m) Basolateral/principal
n) apical/principal
o) Basolateral/parietal

d) apical/parietal

97. Para protegerse del entorno ácido, el Helicobacter pylori se autogenera un entorno de
pH menos ácido alrededor suyo, gracias a una enzima que alcaliniza su entorno local
mediante la conversión de:
a) urea en NH3
b) H2O y CO2 en ácido carbónico
c) NH3 en urea
d) H2CO3 en bicarbonato
98. La anemia perniciosa destruye las células:
p) mucosas del cuello
b) oxínticas
c) principales
d) mucosas superficiales

99. La célula mucosa del cuello gástrico produce:


a) Moco
b) ácido clorhídrico
c) pepsinógeno
d) Factor intrínseco

100.El aumento en la acidez del estómago producido principalmente por la infección de


Helicobacter pylori se debe a la disminución de:
a) Somatostatina
b) Bicarbonato por las glándulas de Brunner
c) Secretina
d) Colecistoquinina

101. De las siguientes sustancias secretadas por los órganos de este paciente, la más
alcalina es la secreción:
a) pancreática
b) Esofágica
c) Yeyunal
d) Salival

102.En cuanto a la gastritis de este paciente, se encontró que era producida por la
bacteria Helicobacter pylori. Esta bacteria sobrevive en el medio ácido del estómago
gracias a:
a) ácido clorhídrico
b) Toxina CagA

c) Ureasa
d) Jugo pancreático

103.La lengua está recubierta por epitelio:


c) pseudoestratificado columnar no queratinizado

b) plano estratificado no queratinizado


c) pseudoestratificado columnar ciliado
d) plano estratificado queratinizado

104.El esfínter anal interno tiene musculatura …….. y tiene


control ……..
d) lisa / voluntario

b) lisa / involuntario
c) esquelética / simpático
d) esquelética / parasimpático

19) La arteria Aorta proporciona la irrigación al tubo digestivo ¿cuál de las siguientes
arterias proporciona la irrigación al ángulo cólico derecho?

a) mesentérica superior
b) mesentérica inferior
c) frénica inferior
d) tronco celiaco

20) Paciente de 26 años que le cuenta en su historia clínica que cada vez que almuerza a
los 20 min tiene deseo de defecar, le comenta que su hijo de 1 mes le pasa lo mismo pero
más intenso. Esto se explica por el reflejo …….., el cual está …… en el paciente
a) colicoileal / normal
b) colicoileal / alterado
c) gastrocólico / normal
d) gastrocólico / alterado

21) La región del estómago que se comunica con el duodeno es la a)


pilórica
b) cardias
c) cuerpo
d) fórnix

22) Acude a consulta un px que fue diagnosticado de úlcera péptica 3 días antes. Luego
de múltiples pruebas diagnósticas se concluye que el paciente presenta un tumor
secretor de gastrina, ¿cual de las siguientes situaciones estará incrementada? a)
distensión gástrica
b) inhibición del vaciado gástrico
c) secrecion de acido clorhidrico
d) inhibición de la secreción de pepsinógeno
23) En el sistema digestivo, el control del apetito está dado por un complejo sistema de
sustancias y órganos integradores, los cuales regulan la ingesta de alimentos. La …… es
una sustancia orexígena y es sintetizada por el ……
a) leptina / estómago
b) felina / intestino
c) leptina / estómago

d) grelina / estómago

24) Con respecto a la actividad eléctrica del sistema digestivo, marque la alternativa
correcta
a) corresponden a potenciales de acción que están presentes de forma continua y le dan
capacidad de peristalsis autónoma al sistema digestivo

b) la frecuencia de las ondas lentas NO se ve influenciada por la actividad neural ni las


hormonas gastrointestinales
c) en el estómago las ondas lentas se dan en una frecuencia de 6 x min d) las
ondas lentas son cambios lentos y ondulantes del potencial en reposo e) la
frecuencia de las ondas lentas va de 6 a 12 ondas por minuto

25) Ante una lesión del IX pc, el músculo …… se altera en su función a)


palatogloso

b) estilofaríngeo
c) palatofaríngeo
d) constrictor superior

26) Un varón de 50 años es sometido a extirpación del duodeno y parte proximal del
yeyuno. La pérdida de estímulo hormonal en el páncreas para la secreción enzimática se
explica por la pérdida de células.

a) parietales, productoras de factor intrínseco.


b) K productoras de factor intrínseco
c) M productora de CCK

d) I productora de CCK

27) Marque la respuesta correcta:


A. El bronquio derecho constituye una de las estrecheces del esófago B.
Todos los órganos del sistema digestivo tienen capa serosa

C. La pared gástrica en el fondo es más delgada que en el cuerpo y antro D.


El esfínter de Oddi rodea a la papila menor duodenal

28) Marque la respuesta correcta en relación a la gastrina:


A. Al distenderse el estómago, se inhibe su producción.
B. Se estimula por la liberación de noradrenalina

C. Las células G son las productoras y se encuentran principalmente en el antro


gástrico
D. Las células G se encuentran principalmente en el fondo gástrico 29) Para

poder morder una manzana, es necesario usar el siguiente músculo:

A. Milohiodeo
B. Tensor del paladar C.
Orbicular de los labios D.
Buccinador
30) Sustancia que inhibe la secreción y la motilidad del estómago prolongando el
tiempo de digestión:
A. Enteroglucagon.
B. Polipéptido pancreático
C. Péptido 1 similar al glucagón (GLP-1).

D. Péptido insulinotrópico dependiente de la glucosa (GIP).

31) El nacimiento de la arteria mesentérica superior se puede encontrar en cuál de los


cuadrantes abdominales:
A. Hipocondrio derecho
B. Hipogastrio

C. Epigastrio
D. Mesogastrio

32) Entre las múltiples causas de la Enfermedad por Reflujo Gastroesofágico, se


puede considerar también a una alteración en las del esfínter esofágico inferior:

A. Ondas secundarias
B. Contracciones tónicas
C. Ondas lentas
D. Glándulas subesofágicas

33) Producto de la alimentación, se producen diversas sustancias peptídicas, cininas y


bradicininas, las cuales permiten que:
A. Se produzca neovascularización en los territorios de las arterias abdominales B.
La acción de la lipasa pancreática se vea incrementada
C. El consumo de O2 del intestino aumente ligeramente

D. El flujo sanguíneo intestinal aumente hasta 8 veces


34) El dolor periumbilical o epigástrico en el inicio de una apendicitis aguda se debe a: A.
Estímulo del nervio vago.
B. Íleo secundario.
C. Irritación del peritoneo parietal.

D. Estímulo del sistema simpático.

35) El aumento en la actividad motora de la pared gástrica genera un aumento en los


niveles locales de qué sustancia en la microvasculatura:

A.Adenosina
B. Colecistoquinina CCK
C. Endotelina
D. Gastrina

36) ¿Cuál de los siguientes péptidos inhibe el vaciamiento gástrico?

A. Colecistoquinina
B. Péptido inhibidor gástrico
C. Motilina
D. Gastrina
37) Los músculos de la masticación que producen la retropulsión de la mandíbula son:

A. temporales [mas seguro]


B. maseteros
C. milohioideos
D. pterigoideos
38) En relación a la fisiología gástrica, marque lo correcto:

A. la cimetidina actúa en la región basolateral de la célula parietal


B. la marea alcalina se debe al paso de bicarbonato través de la membrana apical de la
célula principal
C. el cloro difunde hacia el exterior por la la región basolateral de la célula parietal D.
la salida de hidrogeniones a la luz es por difusión facilitada

39) Durante el sueño, la concentración de bicarbonato en la saliva: A.


Se eleva a niveles mayores que los del plasma
B. Aumenta
C. No tiene efecto

D. Disminuye

40) La secreción de saliva es importante en la fisiología digestiva. Su concentración de


potasio llega a ser menor que la del plasma cuando su secreción tiene un flujo:

A. Intermedio
B. Nunca
C. Bajo
D. Alto

42) Respecto a las glándulas salivales, marque lo incorrecto:


A. la glándula parótida produce secreción serosa
B. la glándula sublingual drena a través de conducto de Wharton
C. La glándula parótida drena a través del conducto de Stenon

D. la glándula sublingual tiene forma de garfio

43) Con respecto a la saliva, marque la respuesta correcta:


A. será hipertónica cuando el flujo es bajo
B. a mayor flujo, menor concentración de Na
C. a mayor flujo, menor concentración de cloro

D. el sistema simpático estimula su secreción

CI 3

44) En relación a la circulación hepática, marque lo correcto:

a) Los sinusoides hepáticos transportan sangre mixta


b) La vena porta proporciona el 50% de sangre al hígado
c) La vena porta se forma a partir de la vena esplénica y la mesentérica inferior

d) La arteria hepática deriva de la mesentérica superior

45) Dentro de las funciones de las células de Ito, marque lo incorrecto: a)


Sintetizan colágeno
b) Almacenan vitamina A
c) Se les llama células estrelladas

d) Pueden fagocitar patógenos y actúan como presentadoras de antígeno

46) Paciente con tumor neuroendocrino productor de secretina, debido a lo cual se puede
esperar que su secreción pancreática, comparada con la de una persona sana de bajo
flujo, tenga una concentración de:
a) Sodio aumentada
b) Igual
c) Bicarbonato aumentada
d) Potasio disminuida
47) El GALT se localiza en:

a) Lámina propia
b) submucosa
c) borde en cepillo
d) superficie de criptas de Lieberkühn

48) En relación a la histología hepática, marque lo correcto:


a) la zona 1 se afecta rápidamente en estados de hipovolemia y shock b)
La zona 1 se encuentra cercana a la vena central lobulillar

c) La zona 3 se encuentra más cerca a la vena central lobulillar


d) La zona 3 se encuentra más cerca al eje menor del acino hepático 49)

El acino pancreático difere con el de las glándulas salivales en:

a) Contiene células centroacinares


b) No produce secreción serosa
c) El páncreas produce principalmente secreción mucosa
d) No tienen diferencias

50) Durante la digestión de las grasa, para que la lipasa actúe adecuadamente se
requiere que el pH aumento en la luz intestinal, lo cual es logrado, entre otros, por la
secreción de las células:

a) Del conducto interlobulillar


b) Centroacinares
c) Acinares
d) Alfa

51) La secreción de la colecistoquinina (CCK) se produce en la fase:

a) intestinal
b) En las 3 por igual
c) gástrica
d) Cefálica

52) ¿Por cuál de las siguientes células es secretada principalmente la pro enzima
procarboxipeptidasa?

a) Acinares del páncreas


b) Epiteliales del duodeno
c) Ductales del páncreas
d) Centro Acinares del páncreas

53) Una mujer de 43 años dolor en hipocondrio derecho e icterica. En la ecografía se


evidencia cálculos biliares. Estos cálculos lo más probable es que se encuentren
localizados en:

a) conducto colédoco
b) Conducto cístico
c) Vesícula biliar
d) Conducto pancreático secundario

ECU 1:
Estudiante de 21 años sufre de gastritis aguda ocasionada por comer en lugares poco
higenicos. Suele consumir caramelos (“chupar”) mientras esta en base hasta la tarde.
También toma regular cantidad de leche (grasa, lactora, proteinas), pues le calma el dolor y
el ardor que sitnete por la gastritis (tiene dispepsia y cuando toma la leche se le pasa).
Incluso cuando puede, se toma dos vasos de agua frita y le calma la molestia. Ha decido
ir al medico para tratarse, pues ya no soporta el dolor, el cual esta seguro que los
síntomas se producen por elevada producion de HCl en el estomago, y por ello le ha
recetado ranitidina
1.1) El consumir caramelos eleva los niveles en sangre de una hormona cuya función es
la estimulación de las células.
- Beta del páncreas por GIP el cual es una incretina y por consiguiente estimula las
células pancreáticas

1.2) Consumir caramelos indirectamente actica la via:

-POMP/ CART saciedad

1.3) Consumo de leche produce indirectamente


- CCK inhibición del vaciamiento gástrico mayor tonicidad del esfínter pilórico

1.4) Cuando el px toma dos vasos de agua, genera indirectamente un aumento en la


liberación de:
- vaso de agua distención → g astrina → secreción de HCl

1.5) El uso de ranitidina bloquea el receptor H2 de la histamina en las células


parietales, la histamina llega a estas células por:
- histamina es una hormona paracrina por → difusión

**endocrina es por via hematógena y si fuera neuroendorina es por un NTs

1.6) Aumenta la secreción salival:


- noradrenalina a través de los receptores Beta 2

1.7) En este paciente con gastritis aguda debida a una alta producción de ácido
clorhídrico, sería lógico esperar que el píloro tenga un tono muscular: - primero la
secretina
- luego CCK

**ambas reguladores del HCl, Gatritis aguda debido a una alta producción de HCL
piloro estará aumentado (por la CCK)

1.8) Debido al uso de ranitidina, los valores de somatoestina en sangre: -


ranitidina disminuye acción de gastrina se quiere secretar mas no actúan los
inhibidores como la somatoestina somatoestina disminuye

1.9) El uso de atropina en este paciente:


- Inhibirá la acción de las prostaglandinas
- Aumentará la producción de ácido clorhídrico
- Disminuirá la acción del receptor CCK-B

-Aumentará el pH del estómago


ECU 2:
Niño de sexo masculino de 2 años de edad, sufre de estreñimiento desde el nacimiento
(1 deposición cada 3-4 días). Madre menciona que le estimula la defecación con un
termómetro rectal, y continuo uso de enemas y laxantes. Desde hace 6 meses
comienza con vómitos postprandiales. Los síntomas aumentan en frecuencia y
magnitud y están en relación con los episodios de estreñimiento. No refiere fiebre, tos,
diarrea ni lesiones cutáneas. Al examen físico presenta regular
estado general, luce deshidratado. Abdomen distendido, blando, depresible e indoloro. No
se palpan masas abdominales. Se permeabiliza el canal anal con termómetro rectal,
encontrando cierta resistencia. Salida de material fecal mal oliente en regular cantidad.
Exámenes de laboratorio: hemograma normal. Signos inflamatorios de fase aguda
negativos. Alcalosis metabólica leve en sangre
venosa. Radiografía con enema baritado muestra recto y colon sigmoides dilatados
(megacolon). Biopsia profunda: ausencia de células ganglionares en la muestra
enviada. Se realiza cirugía correctiva.

2.1) Durante la fase esofágica de la deglución, para un bolo alimenticio determinado, a


medida que avanza el bolo la fuerza de la contracción se hace más: - hiperpolarizado
- fuerte
- dependiente de Ach
- debil

2.2) Cuando este paciente ingiera sus alimentos, se espera que al momento de pasar el
bolo alimenticio por el esfínter esofágico superior, la presión intraesofágica disminuya en:
- la porción proximal al bolo
- el tercio medio del esófago

-el cardias
- el lugar donde se contraiga la muscular propia

2.3) Al examinar la orofaringe del paciente, uno puede hallar fácilmente la amígdala
palatina, pues esta se encuentra inmediatamente detrás del músculo:

-Palatogloso
- Palatofaringeo
- Hiogloso
- Elevador del velo del paladar

2.4) Con respecto a la defecación en este caso, marque la respuesta correcta: - En


posición de cuclillas, el músculo puborectal genera un ángulo más agudo en el recto
- El sigmoides y el recto están inervados por el nervio vago
- La sensación de defecar sólo se da cuando el recto es ocupado por heces y
alcanzado el 80% de su capacidad

-El esfínter anal comprometido tiene inervación autónoma

2.5) En este paciente [hirschsprung] se considera que está abolido el reflejo: -


Coloileal

-Rectoesfinteriano
- Gastrocólico
- Relajación receptiva

2.6) No se espera que sea causa del vómito:

-Ayuno prolongado
- Estimulación faríngea y del glosofaríngeo
- Irritación de la mucosa gástrica
- Dolor intenso

ECU 3:
Paciente de 54 años con antecedentes de alcoholismo, gastritis crónica, tabaquismo
pesado, obesidad, cálculos biliares y cirrosis, es llevado a la emergencia por dolor
abdominal en epigastrio irradiado a la espalda y trastorno del sensorio. Al examen físico:
presión arterial 85/50 mmHg, frecuencia cardíaca 100 latidos/min, frecuencia respiratoria
18 x minuto, temperatura axilar 36°C.
Conjuntivas pálidas, escleras ictéricas nevus arácnidos en tronco, distensión
abdominal marcada, cabeza de medusa, matidez desplazable en ambos flancos e
hipogastrio, dolor a la palpación de abdomen.
Tiempo de protrombina: 24 seg (testigo: 13 seg); TPT: 38 seg, glicemia: 165 mg/dL,
uremia: 20 mg/dL, ASAT: 76 UI/L, ALAT: 22 UI/L, albumina: 2,5 g/dL, bilirrubina total: 2,6
mg/dL, bilirrubina directa: 1,4 mg/dL, amilasa sérica 4000 U/L.

3.1) En esta paciente, al aumento de la amilasa sérica, se debe directamente a una


lesión de:

a) páncreas
b) vesícula y árbol biliar
c) estómago
d) hígado

3.2) Considerando que el paciente sufre de gastritis, se puede decir que la secreción de
ácido por la mucosa gástrica

a) involucra transporte activo de hidrogeniones


b) es realizada principalmente por células principales
c) es inhibida por antihistaminas tomadas por pacientes con rinitis alérgica d)
involucra la liberación de HCl de los gránulos zimógenos

3.3) El paciente tiene hemorragia digestiva alta por várices sangrantes como
complicación. Llegando a estar en shock hipovolémico por hemorragia masiva, se
encontrara necrosis hepática en:
a) zona 1
b) no se afectan los lobulillos hepáticos en hemorragia

c) zona 3
d) zona 2

3.4) El misoprostol, análogo de las prostaglandinas está mejor indicado en: a)


cicatrización de úlcera péptica duodenal
b) erradica el helicobacter pylori
c) tratar el sind de Zollinger ellison

d) prevenir daño por AINES

3.5) De las siguientes sustancias secretadas por los órganos de este paciente, la más
alcalina es la secreción:
- Esofágica
- Salival
- Yeyunal
-Pancreática
3.6) En este paciente, se puede asumir que la pancreatitis ha sido ocasionada por una
disminución en el efecto de:
- Lipasa
- Enterocinasa
- Amilasa
-Inhibidor de la tripsina
3.7) ¿Cuál de las siguientes sustancias es segregada por el páncreas? -

Amilasa

- Pepsina
- Quimiotripsina
- Tripsina
3.8) Cada vez que este paciente toma alcohol, la acidificación de la luz del duodeno:

-Disminuye el vaciamiento gástrico


- Aumenta la contracción del esfínter de Oddi
- Aumenta la secreción del ácido gástrico
- Disminuye la secreción pancreática del bicarbonato
SISTEMA DIGESTIVO (ME 154) EXAMEN FINAL
Ciclo 2018-01
1. Un niño de 2 años es llevado a la consulta por diarrea persistente y edema de las extremidades, además
falta de crecimiento y desarrollo en relación a su edad. Los análisis de sangre revelan que tiene
concentración plasmática baja de proteínas (hipoproteinemia). Durante la endoscopía duodenal, se coloca
colecistokinina (CCK) endovenosa y se recoge muestras del líquido duodenal; el resultado del líquido
confirma incapacidad para hidrolizar proteínas a un pH neutro, esta situación mejora al añadir una pequeña
cantidad de tripsina. El paciente probablemente esté sufriendo la falta congénita de

………….
(Unidad 4, sesión 26, logro 2: Explicar la Digestión y absorción de las proteínas y sus alteraciones) a.
Pepsinógeno
b. PEPT-1
c. Carboxipeptidasas
d. Enterocinasa

2. Experimentalmente se incrementa la velocidad de la secreción salival con una sustancia, el análisis de la


composición de esta saliva obtenida se espera encontrar…………..

(Unidad 3, sesión 17, logro 5 : Explica la Influencia de la velocidad del flujo salival en la composición de la
saliva)
a. Elevación de concentración de bicarbonato, sodio y potasio
b. Elevación de concentración de cloro, sodio y potasio
c. Disminución de concentración de potasio
d. Disminución de concentración de potasio y bicarbonato

3. Paciente varón de 46 años soltero, consulta por odinofagia y bajo de peso, tiene antecedente de
tuberculosis desde hace 3 meses y es fumador crónico (10 cigarrillos por día); al evaluar la cavidad oral se
identifica lesión blanquecina en el dorso de la lengua y paladar blando, las lesiones se desprenden con el
baja lengua dejando una base eritematosa. Esta lesión corresponde probablemente a ……………………….…..

( Unidad 3, sesión18, logro 1-2 : Describe las enfermedades inflamatorias, infecciosas y proliferativas de la
cavidad oral)
a. Eritroplaquia
b. Candidiasis oral
c. Leucoplaquia vellosa
d. Fibroma en cavidad oral

4. Minero de 32 años de edad, que acude a centro de salud por presentar de forma progresiva desde hace 1
año dificultad para ingerir alimentos sólidos y luego líquidos; refiere regurgitaciones alimentarias y marcada
pérdida de peso (15 kilos). Radiografia baritada de
esófago como se muestra en la figura. El presente caso se
explica por……………….

(Unidad 2, sesión 12, logro 4: Identificar y describir la función


de los esfínteres esofágicos)
a. Contracción incompleta del esfínter esofágico inferior
b. Dificultad para el inicio de la deglución c. Relajación

incompleta del esfínter pilórico d. Relajación incompleta

del esfínter esofágico inferior

5. Paciente mujer de 35 años acude a consulta por sensación de sequedad y lesiones en cavidad oral. Al
examen se observa atrofia de la mucosa, fisuras y úlceras; nota además sequedad e irritación de la córnea y
aumento del tamaño de las glándulas parotídeas. Su diagnóstico más probable es artritis reumatoide; el
hallazgo más probable en una biopsia de glándula parótida es……..….

(Unidad 3, sesión 18, logro 3: Describe las enfermedades más frecuentes de las glándulas salivales) a.
Hiperplasia de acinos glandulares serosos
b. Gran infiltración de linfocitos y células plasmáticas
c. Gran infiltrado de linfocitos y macrófagos
d. Presencia de acinos normales con hiperplasia de células ductales
6. Un paciente con anemia acude con su médico quejándose de episodios frecuentes de gastroenteritis. Un
análisis de sangre revela anticuerpos circulantes dirigidos contra células parietales gástricas. Su anemia es
atribuible a la hiposecreción de

………………………
(Unidad 3, sesión 20, logro 5: Gastritis crónica. Tipos de gastritis)
a. Factor intrínseco
b. Proteina R (haptocorrina)
c. Pepsinógeno
d. Ácido clorhídrico

7. Dos estudiantes deciden tomar un receso para comer una hamburguesa a la hora del almuerzo. Antes de
llegar a la cafetería, impulsos nerviosos provenientes del complejo vagal dorsal iniciarán la secreción de ácido
gástrico por la liberación dedesde el sistema nervioso entérico.

(Unidad 3, sesión 20, logro 2: Regulación de la secreción gástrica: estimulación, fases de la secreción) a.
Serotonina
b. Óxido nítrico
c. GRP (péptido liberador de gastrina)
d. Péptido intestinal vaso activo

8. Un niño de cuatro años de edad es llevado a la consulta por cuadros diarreicos frecuentes
caracterizados por heces pálidas, voluminosas y fétidas, presenta bajo peso y talla. Se mide la
concentración de cloruro en el sudor y se encuentra que sus valores son muy elevados. La alteración más
importante a nivel de células ductales del páncreas tiene relación directa con la conductancia de…………

(Unidad 3, sesión 23, logro 5 Explica la Secreción pancreática: formación del jugo pancreático, influencia de la
velocidad de flujo y regulación)
a. Potasio
b. Bicarbonato
c. Sodio
d. Cloro

9. Una mujer de 50 años de edad que sufrió durante varios años resequedad de los ojos debida a
producción inadecuada de lágrimas es enviada con un gastroenterólogo para evaluación de pirosis crónica.
El examen endoscópico revela erosiones y tejido cicatrizal en la parte distal del esófago justo por arriba del
esfínter esofágico inferior. Las lesiones pueden atribuirse a la disminución de uno de los siguientes
componentes salivales:

(Unidad 3, sesión 17, logro 4: Explicar la Formación de la saliva y cuáles son sus componentes) a.
Bicarbonato
b. Lactoferrina
c. Ig A
d. Amilasa
10. Se evalúa los valores séricos de las siguientes sustancias a un paciente con enfermedad hepática
terminal; en este paciente se espera encontrar la combinación con la letra …………
hepática, encefalopatía hepática e
(Unidad 3, sesión 22, logro 5: Describe las hipertensión portal) Glucosa
Pruebas de función hepática, la Insuficiencia Amoniaco Albúmina

a. Aumentada Disminuida Disminuida b. Disminuida Aumentada Aumentada c. Aumentada


Aumentada Aumentada d. Disminuida Aumentada Disminuida

11. Una mujer de 35 años de edad HIV positiva, se presenta al médico con dolor abdominal en
cuadrante superior derecho e ictericia. La paciente refiere haber tenido múltiples episodios de ictericia
durante los últimos 10 años. Los exámenes para determinar hepatitis viral, dieron positivos para
Hepatitis B, siendo catalogado el caso como hepatitis crónica con alteración funcional. En un examen de
sangre ¿cuál de los siguientes parámetros está disminuido?
(unidad 3, sesión 22, logro 5: Pruebas de función hepática, Insuficiencia hepática, encefalopatía hepática
e hipertensión portal)
a. Fosfatasa alcalina
b. Albumina
c. Bilirrubina
d. Tiempo de protrombina

12. En el reflejo peristáltico del intestino delgado, uno de los siguientes eventos sucede en la porción
oral del bolo alimenticio…………...

(Unidad 2, sesión 13, logro 4: Explicar la Motilidad del intestino delgado: Contracciones segmentarias y
peristálticas)
a. Disminución de 5 hidroxitriptamina desde las neuronas IPAN
b. Contracción del músculo longitudinal
c. Acción del péptido intestinal vasoactivo (VIP) en el músculo circular
d. Acción de acetilcolina en el músculo circular

13. Experimentalmente se coloca una dosis alta de secretina en la luz intestinal duodenal; como
consecuencia de esto, en el jugo pancreático de la misma luz intestinal se observa la disminución de la
concentración de …..………..

(Unidad 3, sesión 23, logro 5: Explica la Secreción pancreática: formación del jugo pancreático, influencia
de la velocidad de flujo y regulación)
a. Na+
b. Cl
c. K+
d. HCO3-
14. Un varón de 58 años de edad con enfermedad de Crohn severo fue sometido a una resección ileal.
Después de la cirugía este paciente padecerá de esteatorrea, esto se explica porque …..……….. (unidad
4, sesión 26, logro 4: Explica las alteraciones en la Absorción de lípidos) a. El pool de ácidos biliares se
incrementa
b. Los quilomicrones no pueden formarse en el lumen intestinal
c. La micelas no pueden formarse
d. El páncreas no secreta lipasa

15. En un experimento se inserta un balón en el estómago de un voluntario, se infla poco a poco


mientras que se vigilan las presiones intraluminales. Aunque el volumen del balón aumenta
considerablemente, las presiones permanecen constantes. Esta relación volumen-presión se explica por
la liberación local de …………..
(Unidad 2, sesión 13, logro 1 Explica la Motilidad gástrica: relajación receptiva)
a. Acetil colina y gastrina
b. Colecistoquinina y óxido nítrico
c. Óxido nítrico y péptido inhibidor vasoactivo
d. Norepinefrina y óxido nítrico
16. La toxina del Vibrio cholerae causa diarrea debido a…….
(Unidad 4, sesión 27, logro 6: Explica el transporte hidroelectrolítico intestinal, toxina colérica)
a. La fosforilación del canal CFTR de los enterocitos de las vellosidades intestinales b. El
Incremento de la secreción de cloro por las células de la cripta intestinal
c. La inhibición de la producción de AMPc por las células epitelailes
d. El incremento de la absorción de agua y sodio a través de las uniones estrechas 17. ¿Cuál de las

siguientes alternativas es una característica de la secreción exocrina del páncreas?

(Unidad 3, sesión 23, logro 5: Secreción pancreática: formación del jugo pancreático, influencia de la
velocidad de flujo y regulación)
a. Tiene una baja concentración de Cl-respecto al plasma
b. Es estimulada por la presencia de bicarbonato en el duodeno
c. La secreción enzimática es estimulada principalmente por la gastrina
d. Es hipotónica respecto al plasma

18. Una madre lleva a su hijo de dos años de edad a la sala de urgencias, estresada porque el niño
deglutió una moneda de 10 céntimos mientras la familia cenaba en un restaurante. El médico observa
mediante fluoroscopía que la moneda se halla en el estómago y asegura a la madre que la moneda se
eliminará con las heces. El médico recomienda utilizar la respuesta fisiológica que permitirá la
evacuación de la moneda del estómago al intestino ………….…..

(Unidad 2, sesión 13, logro 2: Explica la Motilidad gástrica: mezclado y vaciamiento)


a. Es por la relajación receptiva
b. Son los movimientos de mezcla y trituración
c. Es provocada por el ayuno
d. Es por la relajación del esfínter esofágico superior

19. Las estructuras en el hígado que permite que los productos metabólicos unidos a proteínas tengan
acceso a las membranas basolaterales de los hepatocitos, son…..
(Unidad 3, sesión 21, logro 4-5: Explica la Organización micro estructural del hígado)
a. Los Canalículos
b. Las fenestras sinusoidales
c. Las uniones intercelulares herméticas
d. Las células de Ito

20. La composición de la bilis es modificada conforme fluye por los conductillos biliares. Durante este
tránsito se espera que aumente la concentración de…….

(Unidad 3, sesión 22, logro 2: Describe la Secreción biliar, visión general del sistema biliar extrahepático y
composición de la bilis)
a. Ig A
b. Glucosa
c. Monómeros de ácido biliar
d. Vitamina A

21. Se mide experimentalmente el contenido gástrico de dos personas. La persona “A” tiene alto
contenido de grasa y la persona “B” tiene un contenido hipertónico ¿Cuál de las siguientes es correcto
respecto al vaciamiento gástrico? (Unidad 2, sesión 13, logro 2: Describe la Motilidad y vaciamiento
gástrico)
a. Hay ralentización del vaciado gástrico solo en “A”
b. El vaciamiento gástrico es más rápido en ambos
c. En ambos casos hay incremento de la motilina
d. Hay ralentización del vaciado gástrico en ambos casos
22. El examen endoscópico de un paciente con hipertensión portal grave revela venas tortuosas que
sobresalen hacia la luz del esófago. El paciente recibe tratamiento quirúrgico mediante la colocación de
una derivación que conecta la vena porta a la vena cava. Después de la operación el riesgo de
encefalopatía ………………….. y el riesgo de sangrado de várices ……………..

(Unidad 3, sesión 22, logro 5: Describe la Insuficiencia hepática, encefalopatía hepática e hipertensión
portal)
a. Aumentará/disminuirá
b. Disminuirá/disminuirá

c. Aumentará/aumentará
d. Disminuirá/aumentará

23. Un paciente varón de 18 años de edad acude al médico para sus exámenes de rutina. Sus resultados
de laboratorio muestran un valor de bilirrubina sérica de 4 mg/dl y una bilirrubina directa de 0,3 mg/dl.
Las pruebas de función hepática son normales. La alteración que explica mejor este caso es por la
deficiencia de ………………..
(Unidad 3, sesión 22, logro 3: Explica la Producción y excreción de bilirrubina. Tipos de bilirrubina e
ictericia)
a. Transaminasas
b. Glucuronil transferasa
c. Hemo oxigenasa
d. La 7 alfa hidroxilasa

24. Un hombre de 57 años de edad es llevado a urgencias con hematemesis masiva rojo brillante, a su
llegada se halla inconciente con PA: 80/40 mm Hg y FC: 124 lat/min. Luce ictérico con presencia de
“arañas vasculares en el tórax anterior y extremidades”, abdomen distendido con signo de oleada
positiva. Se encuentra esplenomegalia y pérdida de la masa muscular en extremidades. La anastomosis
vascular responsable del sangrado en este paciente es ………….…..

(Unidad 3, sesión 21, logro 2: Describe las anastomosis porto sistémicas)


a. Arteria gástrica izquierda y vena ácigos
b. Vena gástrica izquierda y vena ácigos
c. Vena paraumbilical y vena epigástrica inferior
d. Vena gástrica izquierda y vena esofágica superior

25. Un estudiante de medicina está comiendo un plato de comida a base de champiñones, espárrago y
salsa de soya. El sabor umami contenido en todos estos alimentos actúa a nivel de los botones
gustativos estimulando ………………..
(Unidad 2, sesión 10, logro 5: Describe los tipos y mecanismos moleculares para la detección de los
sabores)
a. El ingreso de sodio
b. Un receptor acoplado a proteína G
c. Su receptor específico T1R3
d. El ingreso de hidrógeno

26. Un hombre de 22 años de edad se presenta al médico con una historia de 1 año de evolución
caracterizado por dolor recurrente en fosa iliaca derecha y diarrea. Manifiesta además pérdida de peso
de 8 kg durante este periodo. La colonoscopía revela múltiples lesiones en el ileon terminal y colon. La
biopsia de estas lesiones revela engrosamiento, inflamación y ulceración de la mucosa. El diagnóstico
más probable en este caso es…….
(Unidad 4, sesión 28, logro 5: Describe la Enfermedad inflamatoria intestinal. Generalidades, morfología
y características)
a. Sprue celiaco
b. Enfermedad de Crohn
c. Sindrome de colon irritable
d. Colitis ulcerativa

27. Una de las funciones del músculo señalado es:

(Unidad 2, sesión 8, logro 3: Describir el Piso de la boca: estructuras blandas que la conforman)
a. Eleva el paladar blando
b. Recibe inervación del nervio maxilar
c. Deprime el hioides cuando la mandíbula está fija
d. Deprime la mandíbula cuando el hioides está fijo

28. Varón de 61 años que consulta por dolor retro esternal intenso desde hace 6 horas y después de
vómitos intensos y repetidos; al examen se observa disnea, cianosis, hipotensión y signos clínicos de
shock. La radiografía simple de tórax muestra neumomediastino. El líquido en el espacio pleural
aspirado tiene alta concentración de amilasa. ¿Cuál de las siguientes alternativas puede explicar este
cuadro clínico?
(Unidad 3, sesión 18, logro 6: Describe algunas Enfermedades del esófago)
a. Sindrome de Mallory Weiss
b. Rotura espontánea de esófago
c. Neumotórax por probable herida penetrante
d. Perforación de ulcera gástrica de cara posterior, con complicación torácica

29. La secreción del ácido en la célula parietal gástrica se lleva a cabo por una ATPasa especifica que
intercambia hidrogeniones (H+) del citosol por…..
(Unidad 3, sesión 20, logro 1: Explica la Secreción del HCl y sustancias que la alteran)
a. Cl
b. HCO3-
c. Na +
d. K+

30. En condiciones normales el ingreso de 600 ml de líquido es el estómago provoca un aumento de


presión intragástrica de unos 12 cm de H2O. Después de una vagotomía (corte del nervio vago) es de
esperar que el ingreso del mismo volumen de líquido provoque lo siguiente: …………………………………

(Unidad 2, sesión 13, logro 1: Describe la Motilidad gástrica: relajación receptiva)


a. Un aumento igual de la presión
b. Que no aumente la presión
c. Un aumento mayor de la presión
d. Una disminución de la presión

31. Una paciente de 30 años de edad es sometida a una cirugía en oído medio derecho por un problema
de otoesclerosis. Luego de la cirugía refiere alteración en la percepción de sabores. Al evaluar el caso
usted esperaría encontrar……….
(Unidad 2, sesión 10, logro 5: Describe la Irrigación e inervación de la lengua)
a. Alteración en la sensación del dolor y temperatura en el tercio posterior de la lengua
b. Alteración en la sensación del gusto en los dos tercios anteriores de la lengua c.
Alteración en la sensación del gusto en la punta de la lengua
d. Sensación del dolor, tacto y temperatura conservada en toda la lengua

32. ¿Cuál de las siguientes alterativas es correcta?

(Unidad 4, sesión 26 : Explica la digestión y absorción de nutrientes y sus alteraciones) a. En el


borde luminal, en cepillo, del intestino delgado, la absorción de sodio únicamente se realiza
asociada a la de glucosa.
b. El lugar principal para la absorción del hierro es el ileon
c. Las sales biliares desconjugadas son absorbidas preferentemente en el colon
d. El proceso de digestión y absorción de la vitamina B12 no se altera en insuficiencia pancreática.

33. En un paciente de 45 años de edad con colestasis biliar, se encuentra una elevación de los niveles
sanguíneos de fosfatasa alcalina hasta 3 veces la cifra normal. ¿Cuál de las siguientes alternativas estará
también elevada como evidencia del daño de la vía biliar?

(Unidad 3, sesión 22, logro 5: Pruebas de función hepática, Insuficiencia hepática, encefalopatía hepática
e hipertensión portal)
a. Tiempo de protrombina y albúmina sérica
b. Transaminasas hepáticas (ALT y AST)
c. Glucoronil transferasa
d. Gamma glutamil transpeptidasa
34. Revisando la angiografía de un hombre de 70 años en estudio por aneurisma de aorta abdominal el
radiólogo informa de la presencia de una oclusión completa de la arteria mesentérica inferior. El
paciente se encuentra completamente asintomático. ¿Cuál de las siguientes arterias se anastomosa a la
sistema arterial de la mesentérica inferior?

(Unidad 4, sesión 25, logro 1: Identifica la Arteria mesentérica superior e inferior, ramas y anastomosis)
a. Ileal
b. Cólica media
c. Sigmoideas
d. Cólica izquierda

35. Lactante de 3 meses de vida es atendido por presentar diarrea, se administra una solución de
glucosa y electrólitos por vía oral. La proteína de membrana apical que explica la capacidad de esta
solución para proporcionar aporte de glucosa e hidratación es ………..
(Unidad 4, sesión 26, logro 1: Explica la Digestión y Absorción de los hidratos de carbono. Alteraciones)
a. GLUT-5
b. SGLT-1
c. CFTR
d. GLUT-2

36. Paciente ha sufrido herida de bala en el abdomen, se le ha tenido que extirpar el segmento medio y
distal del ileon. En este caso la síntesis hepática de sales biliares estará …..…..
(Unidad 3, sesión 22, logro 4: Explica la formación, función y Circulación entero hepática de lasa sales
biliares)
a. Disminuida por inhibición de la colesterol 7 alfa hidroxilasa
b. Incrementada por estímulo de la enzima colesterol 7 alfa hidroxilasa
c. Incrementada por inhibición de la colesterol 7 alfa hidroxilasa
d. Sin cambios en el ritmo de síntesis
37. Un varón de 75 años ingresa al consultorio por presentar ictericia marcada de piel y las escleras. El
estudio del paciente mostró que presentaba un tumor que obstruía la totalidad del conducto hepático
común. ¿Cuál de las siguientes estructuras se encontrará dilatada en este paciente?

(Unidad 3, sesión 21, logro 6: Describir el árbol biliar intrahepático)


a. Conducto de Wirsung
b. Conductos de Hering
c. Conducto colédoco
d. Conducto cístico

38. Correlaciones las dos columnas y marque la fórmula correcta:

(Unidad 4, sesión 28, logro 1: Diarrea: definición, mecanismos: osmótica, secretoria y exudativa)
1. Enfermedad Hirschsprung( ) heces con moco y sangre
2. Diarrea osmótica( ) intolerancia a lactosa
3. Diarrea secretoria( ) aganglionosis congénita
4. Diarrea exudativa( ) canales de Cl- en las células de la cripta

a.- 4231
b.- 1234
c.- 2143
d.- 4213

39. Respecto a la siguiente imagen que representa una estructura de la mucosa gástrica, la estructura
con número ………..

produce ……………………..
(Unidad 3, sesión 19, logro 4: La glándula fúndica. Funciones y tipos de células con sus características)
a. 3 / pepsina
b. 1 / Pepsinógeno
c. 4 / HCl y factor extrínseco
d. 2 / pepsinógeno

40. En un paciente con insuficiencia renal crónica, el déficit en la absorción de calcio a nivel del
enterocito se debe a lo siguiente:
(Unidad 4, sesión 26, logro 6: Explica la Absorción de calcio y hierro)
a. No se convierte la 25 hidroxicolecalciferol a 1,25 dihidroxicolecalciferol
b. No se convierte la 1,25 dihidroxicolecalciferol a 25 hidroxicolecalciferol
c. Se incrementa la producción de Calbindina
d. Existe un descenso de la alfa 25 hidroxilasa renal
CLAVES EXAMEN PARCIAL DE SISTEMA DIGESTIVO 2019 – 00

1. Varón de 30 años es traído a emergencia por agresión abdominal con arma de fuego (pistola) y es
sometido a laparotomía exploratoria, observándose isquemia del colon ascendente y parte del colon
trasverso ¿la lesión de cuál de las siguientes arterias explicaría esta isquemia? (unidad 1, sesión 2,
logro 6: (Describe la irrigación visceral: arterias de tronco celiaco, arteria mesentérica superior e
inferior, topografía de superficie, órganos por cuadrante) a. Celiaca
b. Colónica derecha
c. Mesentérica inferior
d. Mesentérica superior

2. Respecto a las sustancias gastrointestinales que regulan la secreción pancreática; marque la


afirmación correcta:
(unidad 1, sesión 3, logros 2 y 3: describir las hormonas gastrointestinales: estímulos y funciones) a. La
Secretina, es la hormona más importante para la secreción de bicarbonato por las células acinares del
páncreas
b. La acetilcolina es capaz de estimular la secreción enzimática y de bicarbonato del
páncreas c. La gastrina, es la hormona más importante para la secreción de enzimas
pancreáticas d. La colecistoquinina (CCK) estimula al páncreas solo para secreción
enzimática

3. Ante una lesión del X par craneal, ¿cuál de los siguientes músculos mantiene conservada su función?:

(unidad 2, sesión 08, logro 4: Paladar blando: componentes musculares)


a. Elevador del velo del paladar
b. Tensor del velo del paladar
c. Palatofaríngeo
d. Glosofaríngeo

4. Experimentalmente se utiliza atropina (anticolinérgico) para inhibir la secreción de gastrina, sin


embargo la secreción de esta hormona se sigue dando ante estímulos vagales. Esta situación se explica
porque la atropina:

(unidad 1, sesión 3, logro 3 : describir las hormonas gastrointestinales: estímulo y funciones de la


gastrina y colecistoquinina)
a. Bloquea parcialmente la bomba de protones en la célula G
b. Inhibe la acción de acetilcolina e histamina en la célula G
c. Solo inhibe la acción del péptido GRP en la célula G
d. No bloquea la acción del péptido GRP

5. Un varón de 50 años es sometido a extirpación del duodeno y parte proximal del yeyuno. Esta
situación ocasionaría la pérdida de las células ……….. , productoras deque estimula la secreción de

bicarbonato por el páncreas.


(unidad 1, sesión 3, logro 3: describir las hormonas gastrointestianles: estímulos y funciones de la
secretina y péptido insulinotrópico dependiente de glucosa)
a. “S” / secretina
b. Parietales / secretina
c. “I” / colecistoquinina
d. “S” / colecistoquinina
6. Recién nacido que presenta tumoración abdominal a nivel del cordón umbilical (fotografía). ¿cuál de
las siguientes afirmaciones es correcta respecto a este defecto en el desarrollo embriológico del
intestino?: (unidad 1, sesión 5, logro 2: identificar las anomalías del desarrollo del intestino medio)

a. Corresponde a una Gastrosquisis


b. Las vísceras se hallan cubiertas por piel
c. No está asociado a otras malformaciones
d. Se asocia a malformaciones cardiacas y del tubo
neural

7. Varón de 35 años acude a la emergencia por trauma abdominal y se decida realizar una laparoscopía
exploratoria. El cirujano observa la disposición de los órganos abdominales como se representa en el
siguiente esquema. Esta disposición de órganos se explica por la rotación(SMA=arteria mesentérica
superior)
(unidad 1, sesión 5, logro 3: identificar las anomalías del desarrollo del intestino medio: defectos de
rotación, estenosis y atresias)
a. anti horaria del intestino medio, en sólo 90°
b. incompleta del intestino medio (270°)
c. horaria del intestino medio
d. horaria del estómago

8. Se evalúa la expresión de la proteína Agrp en una persona con alteración del apetito; lo correcto
respecto a esta proteína es…..
(unidad 1, sesión 3, logro 4: Explica los mecanismos de control del apetito y saciedad )
a. Esta proteína es un potente anorexigénico
b. La mutación del gen que la codifica produce adelgazamiento
c. La sobre producción de la proteína lleva a obesidad por agonismo de receptores MC3 y MC4
d. La sobre producción de la proteína disminuye el apetito por antagonismo de receptores MC4

9. Juana cae de la bicicleta y se fractura la región anterior del hueso maxilar superior con compromiso
de la fosa incisiva. Al examen físico de la región esperaría encontrar alteración en la sensibilidad de la
encía …………………

(unidad 2, sesión 8, logro5: paladar: paladar duro y blando: irrigación e inervación)


a. bucal posterior
b. Lingual anterior
c. palatina anterior
d. palatina posterior
10. Recién nacido es atendido por el neonatólogo y luego entregado a su madre para dar de lactar; la
madre al dar de lactar observa coloración azulada de labios, acompañado de tos persistente, dificultad
respiratoria y distención abdominal. Se le intenta colocar una sonda nasogástrica pero esta retorna a la
cavidad oral en todos los intentos. ¿Cuál de las siguientes anomalías del desarrollo es el más probable
en este caso? (unidad 1, sesión 4, logro 3: identificar las anomalías en el desarrollo del esófago: atresia
y/o fístula traqueo esofágica)

a. Estenosis esofágica proximal con Fístula traqueo esofágica distal


b. Atresia esofágica proximal con fístula traqueoesofágica distal
c. Atresia esofágica distal con fístula traqueoesofágica proximal
d. Fístula traqueoesofágica proximal y distal

11. ¿Cuál de los siguientes mecanismos ocurre durante la defecación?


(unidad 2, sesión 13, logro 6: motilidad del intestino grueso: contracciones segmentarias, movimientos
en masa, defecación y reflejo gastrocólico)
a. Contracción refleja del esfínter anal interno
b. En la posición de “cuclillas” el músculo puborectal se halla relajado
c. Relajación del esfínter anal externo por efectos del VIP y óxido nítrico
d. La materia fecal en el recto estimula la contracción del sigmoides por los nervios pudendos

12. La estructura número 4 (gráfico) corresponde a


……….… y está ………..
(unidad 2, sesión 9, logro 2: Partes de un diente. Capas del diente: Esmalte: características y células que
lo producen)
a. el cemento / mineralizado en 90%
b. la dentina / formada por ameloblastos
c. el esmalte / formado por células derivadas del mesénquima
d. la dentina / formado por células derivadas de la cresta neural

13. Un paciente luego de un accidente sufre lesión del piso de la boca, se constata daño del nervio
“cuerda del tímpano”, en este caso se esperaría encontrar disminución de lade la lengua (unidad
2, sesión 10, logro 3: Irrigación e inervación de la lengua)
a. Motilidad en los dos tercios anteriores
b. Sensación del gusto en el tercio posterior
c. Sensación del gusto en los dos tercios anteriores
d. Sensibilidad al tacto en los dos tercios anteriores
14. ¿Cuál de las siguientes afirmaciones es la correcta sobre la gastrina?

(unidad 1, sesión 3, logro 1: reconocer las características de las sustancias reguladoras


gastrointestinales: hormonas, sustancias paracrinas y neurocrinas)
a. Produce atrofia de la mucosa gástrica
b. Es producida por la célula G del cuerpo gástrico
c. Es estimulada por la distensión gástrica y el Ph bajo
d. Actúa en la célula diana mediante su receptor CCk tipo B

15. Al recibir un paciente con signos de hipovolemia y antecedente de trauma en abdomen por
accidente de tránsito, usted identifica radiológicamente: lesión de primera vértebra lumbar y signos de
lesión en páncreas; durante la cirugía se observó pobre irrigación de asas intestinales. El vaso afectado
es la arteria ……..
(unidad 1, sesión 1, logro 6: reconocer las estructuras a nivel de L1, nivel de los principales vasos
sanguíneos)
a. esplénica
b. hepática común
c. mesentérica inferior
d. mesentérica superior

16. Un paciente sufre de daño a nivel del cuello con lesión muscular en la región de la faringe. En el
examen físico se determina dificultad para la elevación de la faringe y para el cierre del itsmo de las
fauces. En este caso, probablemente esté afectado el músculo:

(unidad 2, sesión 11, logro 2: Músculos de la faringe: identificación, constrictores y longitudinales)


a. palatogloso
b. estilofarinfeo
c. palatofaringeo
d. constrictor inferior

17. Varón de 50 años a quien le realizan la curación de la segunda molar de la arcada superior derecha.
En un momento determinado, el paciente acusa de intenso dolor de la pieza dentaria en tratamiento. La
vía aferente del dolor viaja a través del nervio …………

(unidad 2, sesión 9, logro 6: Inervación de los dientes)


a. trigémino V2
b. trigémino V3
c. naso palatino
d. palatino menor

18. La distención gástrica por los alimentos produce incremento de secreción de HCl mediante la
producción de

………….. que estimula a las células ……………. vía proteína ………..


(Unidad 1, sesión 3, logro 2: Describe las hormonas gastrointestinales: Estímulo y funciones de la
gastrina y colecistoquinina)
a. gastrina / parietal / Gq
b. gastrina / principal / Gs
c. acetilcolina / parietal /Gi
d. acetilcolina / principal / Gi

19. Un niño de tres años llega a emergencia con disfagia (dificultad para tragar), dolor retro esternal,
salivación y llanto. Se sospecha de ingesta de cuerpo extraño (moneda) en el esófago; al ser evaluado se
constata en una radiografía presencia de cuerpo extraño a nivel de C6 (6° vértebra cervical). El cuerpo
extraño estará suspendido a nivel del estrechamiento producido por………..

(unidad 2, sesión 11, logro4: Esófago, características anatómicas, relación con órganos vecinos y
estrecheces)
a. el cayado aórtico
b. el hiato esofágico
c. el músculo cricofaríngeo
d. el bronquio principal izquierdo

20. La triada portal (arteria hepática, vena portal y conducto biliar común) está contenida en el
ligamento

…….……… y derivan embriológicamente del ……


(Unidad 1, sesión 1, logro 4: Identifica el peritoneo, mesenterio, omento y ligamentos, retroperitoneo.)
a. hepato duodenal / mesenterio ventral

b. gastro esplénico / mesenterio dorsal


c. hepato gástrico / omento menor
d. falciforme / omento menor

21. En relación al movimiento de peristaltismo del tubo digestivo: en la flecha negra del gráfico se
produce la liberación de ……………… a nivel del músculo ………..
(unidad 2, sesión 7, logro 6: Control hormonal y tipos de movimiento)
a. noradrenalina, sustancia P y neuropéptido “ Y” / circular
b. acetilcolina y sustancia P / longitudinal
c. óxido nítrico y PIV / longitudinal
d. óxido nítrico y PIV / circular

22. Un paciente refiere no percibir algunos sabores, al examen físico se constata alteración en la
percepción de sabores y del dolor en el tercio posterior de la lengua ¿Cuál de los siguientes nervios
estará alterada en su función?
(unidad 2, sesión 10, logro 5: Sabores, tipos y mecanismos moleculares para su detección)
a. Lingual (rama del V par)
b. Cuerda del tímpano (VII par)
c. Glosofaríngeo (IX par)
d. Hipogloso (XII par)
23. El gráfico detalla la estructura de la pared del tubo digestivo
intestinal ¿Cuál de las siguientes asociaciones es correcta?
(unidad 2, sesión 7, logro 1: La pared y músculo liso gastrointestinal )
a. “1” – peristaltismo
b. “2” – secreción enzimática
c. “3” – deriva del mesodermo
d. “4” – doble hoja de tejido graso

24. En el caso de un paciente con gastrinoma (tumor productor de gastrina), la presencia de úlceras
duodenales y erosión de la mucosa gástrica, se debe principalmente a…….
(unidad 1, sesión 3, logro 2: describir las hormonas gastrointestinales: estímulo y funciones de la gastrina
y colecistoquinina)
a. la acción directa de la gastrina sobre la célula principal
b. la sobre expresión de los receptores “G” en la célula parietal
c. el exceso de HCl por estímulo de receptores CCK-B en la célula parietal
d. el exceso de HCl por estímulo directo de receptores de acetilcolina en la célula parietal

25. El reflejo entero gástrico se caracterizan por:

(unidad 2, sesión 13, logro 6: Motilidad del intestino grueso: contracciones segmentarias, movimientos
en masa defecación y reflejo gastrocólico)
a. favorecer la motilidad gástrica gracias a la CCk
b. inhibir la motilidad gástrica y estimular la secreción ácida
c. movilizar grandes volúmenes desde el estómago al duodeno
d. originarse debido a la distensión duodenal y presencia del quimo ácido

26. Mauricio tiene dificultad para deprimir el paladar y elevar la parte posterior de la lengua. En este
caso estará afectado un músculo ………………., específicamente el músculo ……………. (Unidad 2, sesión
10, logro 2: Músculos de la lengua: clasificación, identificación y sus funciones) a. intrínseco –
longitudinal inferior
b. extrínseco – palatogloso
c. extrínseco – transverso
d. extrínseco – estilogloso

27. Una de las funciones del músculo señalado es:

(Unidad 2, sesión 8, logro 3: Describir el Piso de la boca:


estructuras blandas que la conforman)
a. deprimir la lengua
b. elevar el paladar blando
c. deprimir el hioides cuando la mandíbula está fija
d. deprimir la mandíbula cuando el hioides está fijo
28. Paciente varón de 30 años es evaluado por probable enfermedad de Chagas, cursa con problemas de
motilidad del colon; los estudios de biopsia determinan ausencia de células ganglionares. Según el
gráfico
¿cuál es la capa en la que se determina la ausencia de dichas células?
(unidad 1, sesión 2, logro 1: describir las generalidades de la estructura del tubo digestivo: esófago,
estómago intestino delgado y grueso)
a. Mucosa - 1
b. Muscular propia – 1
c. Muscular de la mucosa - 2
d. Muscular propia - 3

29. Paciente varón de 32 años, que acude a centro de salud por presentar de forma progresiva desde
hace 1 año dificultad para ingerir alimentos sólidos y luego líquidos; refiere regurgitaciones alimentarias
y marcada pérdida de peso (15 kilos). Radiografía baritada (sustancia de
contraste) de esófago se muestra en la figura. El presente caso se explica
por……………….
(Unidad 2, sesión 12, logro 4: Identificar y describir la función de los esfínteres
esofágicos)
a. aumento de la peristalsis esofágica
b. relajación incompleta del esfínter pilórico
c. relajación incompleta del esfínter esofágico inferior
d. perdida de producción de PIV y óxido nítrico en el esfínter esofágico
superior

30. En condiciones normales, el ingreso de 600 ml de líquido es el estómago provoca un aumento de


presión intragástrica de unos 12 cm de H2O. Después de una vagotomía (corte del nervio vago) es de
esperar que el ingreso del mismo volumen de líquido ocasionede la presión intragástrica. (Unidad 2,
sesión 13, logro 1: Describe la Motilidad gástrica: relajación receptiva) a. la disminución
b. la no variación
c. un aumento mayor
d. un aumento similar o igual
SISTEMA DIGESTIVO (ME 154) CLAVES EXAMEN FINAL
Ciclo 201900

1. Un niño de 2 años es llevado a la consulta por diarrea persistente, edema de las extremidades y falta
de crecimiento en relación a su edad. Los análisis de sangre revelan que tiene concentración plasmática
baja de proteínas (hipoproteinemia). Como parte del estudio se coloca colecistokinina (CCK) endovenosa
y se recoge muestras del líquido duodenal por endoscopía; el resultado del líquido confirma incapacidad
para hidrolizar proteínas a un pH neutro, esta situación mejora al añadir una pequeña cantidad de
tripsina. El paciente probablemente esté sufriendo la falta congénita de …………. (Unidad 4, sesión 26,
logro 2: Explicar la Digestión y absorción de las proteínas y sus alteraciones) a. PEPT-1
b. pepsinógeno
c. enterocinasa
d. carboxipeptidasas

2. Paciente mujer de 35 años acude a consulta por sensación de sequedad y lesiones en cavidad oral. Al
examen se observa atrofia de la mucosa, fisuras y úlceras; nota además sequedad e irritación de la
córnea y aumento del tamaño de las glándulas parotídas. Su diagnóstico más probable es artritis
reumatoide; el hallazgo más probable en una biopsia de glándula parótida es……..….

(Unidad 3, sesión 18, logro 3: Describe las enfermedades más frecuentes de las glándulas salivales)
a. Presencia de acinos normales con hiperplasia de células ductales
b. Gran infiltración de linfocitos y células plasmáticas
c. Hiperplasia de acinos glandulares serosos
d. Gran infiltrado de linfocitos y neutrófilos

3. Un hombre de 42 años de edad se presenta al médico con una historia de 1 año de evolución,
caracterizado por dolor abdominal bajo y diarreas con crisis sanguinolentas. Manifiesta además pérdida
de peso de 8 kg durante este periodo. La colonoscopía revela lesión difusa en el colon con afectación del
recto. La biopsia de estas lesiones revela adelgazamiento de la pared, inflamación y ulceración de la
mucosa y sub mucosa. El diagnóstico más probable en este caso es:

(Unidad 4, sesión 28, logro 5: Describe la Enfermedad inflamatoria intestinal. Generalidades, morfología
y características)
a. sindrome de colon irritable
b. enfermedad de Crohn
c. colitis ulcerativa
d. sprue celiaco

4. Dos estudiantes deciden tomar un receso para comer una hamburguesa a la hora del almuerzo. Antes
de llegar a la cafetería, impulsos nerviosos provenientes del complejo vagal dorsal iniciarán la secreción
de ácido gástrico por la liberación dedesde el sistema nervioso entérico.

(Unidad 3, sesión 20, logro 2: Regulación de la secreción gástrica: estimulación, fases de la secreción)
a. Serotonina
b. Colecistoquinina
c. Péptido inhibidor vaso activo
d. GRP (péptido liberador de gastrina)

5. Un niño de cuatro años de edad es llevado a la consulta por cuadros diarreicos frecuentes
caracterizados por heces pálidas, voluminosas y fétidas; al examen físico presenta bajo peso y talla para
la edad. Se mide la concentración de cloruro en el sudor y se encuentra que sus valores son muy
elevados. La alteración más

importante a nivel de células ductales del páncreas tiene relación directa con la conductancia de…………
(Unidad 3, sesión 23, logro 5 Explica la Secreción pancreática: formación del jugo pancreático, influencia
de la velocidad de flujo y regulación)
a. Bicarbonato
b. Potasio
c. Sodio
d. Cloro

6. Se evalúa los valores séricos de las siguientes sustancias a un paciente con enfermedad hepática
terminal; en este paciente se espera encontrar la combinación con la letra ………… (Unidad 3,
sesión 22, logro 5: Describe las Pruebas de función hepática, la Insuficiencia hepática,
encefalopatía hepática e hipertensión portal)

Glucosa Amoniaco Albúmina


a. Aumentada Disminuida Disminuida
b. Disminuida Aumentada Aumentada
c. Aumentada Aumentada Aumentada
d. Disminuida Aumentada Disminuida

7. Una mujer de 35 años de edad HIV positiva, se presenta al médico con dolor abdominal en cuadrante
superior derecho e ictericia. La paciente refiere haber tenido múltiples episodios de ictericia durante los
últimos 10 años. Los exámenes para determinar hepatitis viral, dieron positivos para Hepatitis B, siendo
catalogado el caso como hepatitis crónica con alteración funcional. En un examen de sangre ¿cuál de los
siguientes parámetros está disminuido?
(unidad 3, sesión 22, logro 5: Pruebas de función hepática, Insuficiencia hepática, encefalopatía hepática
e hipertensión portal)
a. Albumina
b. Bilirrubina
c. Fosfatasa alcalina
d. Tiempo de protrombina

8. En el reflejo peristáltico del intestino delgado ¿Cuál de los siguientes eventos sucede en la porción
caudal del bolo alimenticio?

(Unidad 2, sesión 13, logro 4: Explicar la Motilidad del intestino delgado: Contracciones segmentarias y
peristálticas)
a. Acción del péptido inhibidor vasoactivo (VIP) en el músculo circular
b. Acción del NO (óxido nítrico) en el músculo longitudinal
c. Contracción del músculo longitudinal interno
d. Acción de acetilcolina en el músculo circular

9. Un varón de 58 años de edad con enfermedad de Crohn severo fue sometido a una resección ileal.
Después de la cirugía este paciente padecerá de esteatorrea, esto se explica porque …..………..

(unidad 4, sesión 26, logro 4: Explica las alteraciones en la Absorción de lípidos)


a. se inhibe la acción de la 7 alfa hidroxilasa
b. el pool de ácidos biliares se incrementa
c. hay mala absorción de ácidos biliares
d. el páncreas no secreta lipasa

10. En un experimento se inserta un balón en el estómago de un voluntario, se infla poco a poco


mientras que se vigilan las presiones intraluminales. Aunque el volumen del balón aumenta
considerablemente, las presiones permanecen constantes. Esta relación volumen-presión se explica por
la liberación local de …………..

(Unidad 2, sesión 13, logro 1 Explica la Motilidad gástrica: relajación receptiva)


a. acetil colina y gastrina
b. norepinefrina y óxido nítrico
c. colecistoquinina y óxido nítrico
d. óxido nítrico y péptido inhibidor vasoactivo

11. ¿Cuál de las siguientes alternativas es una característica de la secreción exocrina del páncreas?

(Unidad 3, sesión 23, logro 5: Secreción pancreática: formación del jugo pancreático, influencia de la
velocidad de flujo y regulación)
a. Es hipotónica respecto al plasma
b. Su mayor estímulo se da en la fase intestinal
c. Es estimulada por la presencia de bicarbonato en el duodeno
d. La secreción enzimática es estimulada principalmente por la secretina

12. Las estructuras en el hígado que permite que los productos metabólicos unidos a proteínas tengan
acceso a las membranas basolaterales de los hepatocitos, son…..
(Unidad 3, sesión 21, logro 4-5: Explica la Organización micro estructural del hígado)
a. los canalículos
b. las células de Ito
c. las fenestras sinusoidales
d. las uniones intercelulares herméticas

13. La composición de la bilis es modificada conforme fluye por los conductillos biliares. Durante este
tránsito se espera que aumente la concentración de…….

(Unidad 3, sesión 22, logro 2: Describe la Secreción biliar, visión general del sistema biliar extrahepático y
composición de la bilis)
a. Ig A
b. Glucosa
c. Protones
d. Vitamina A

14. Se mide experimentalmente el contenido gástrico de dos personas. La persona “A” tiene alto
contenido de grasa y la persona “B” tiene un contenido isotónico ¿Cuál de las siguientes es correcto
respecto al vaciamiento gástrico? (Unidad 2, sesión 13, logro 2: Describe la Motilidad y vaciamiento
gástrico)
a. Hay ralentización del vaciado gástrico solo en “A”
b. El vaciamiento gástrico es más rápido en ambos
c. Hay ralentización del vaciado gástrico solo en “B”
d. Hay ralentización del vaciado gástrico en ambos casos

15. El examen endoscópico de un paciente con hipertensión portal grave revela venas tortuosas que
sobresalen hacia la luz del esófago. El paciente recibe tratamiento quirúrgico mediante la colocación de
una derivación que conecta la vena porta a la vena cava. Después de la operación el riesgo de
encefalopatíay el riesgo

de sangrado de várices ……………..


(Unidad 3, sesión 22, logro 5: Describe la Insuficiencia hepática, encefalopatía hepática e hipertensión
portal)
a. disminuirá / disminuirá
b. disminuirá / aumentará
c. aumentará / disminuirá
d. aumentará / aumentará
16. Un paciente varón de 18 años de edad acude al médico para sus exámenes de rutina. Sus resultados
de laboratorio muestran un valor de bilirrubina sérica de 4 mg/dl y una bilirrubina directa de 0,3 mg/dl.
Las pruebas de función hepática son normales. La alteración que explica mejor este caso es por la
deficiencia de ………………..

(Unidad 3, sesión 22, logro 3: Explica la Producción y excreción de bilirrubina. Tipos de bilirrubina e
ictericia)
a. transaminasas
b. hemo oxigenasa
c. la 7 alfa hidroxilasa
d. glucuronil transferasa

17. Un hombre de 57 años de edad es llevado a urgencias con hematemesis masiva rojo brillante, a su
llegada se halla inconsciente con PA: 80/40 mm Hg y FC: 124 lat/min. Luce ictérico con presencia de
“arañas vasculares en el tórax anterior y extremidades”, abdomen distendido con signo de oleada
positiva. Se encuentra esplenomegalia y pérdida de la masa muscular en extremidades. La anastomosis
vascular responsable del sangrado en este paciente es ………….…..

(Unidad 3, sesión 21, logro 2: Describe las anastomosis porto sistémicas)


a. vena gástrica izquierda y vena ácigos
b. arteria gástrica izquierda y vena ácigos
c. vena paraumbilical y vena epigástrica inferior
d. vena gástrica izquierda y vena esofágica superior
18. Un estudiante de medicina está comiendo un plato de comida a base de champiñones, espárrago y
salsa de soya. El estímulo del sabor umami contenido en todos estos alimentos viaja a través del
nervio………………..

(Unidad 2, sesión 10, logro 3: Describe la irrigación e inervación de la lengua)


a. Lingual
b. Hipogloso
c. Glosofaringeo
d. Cuerda del tímpano

19. Una paciente de 30 años de edad es sometida a una cirugía en oído medio derecho por un problema
de otoesclerosis. Luego de la cirugía refiere alteración sensitiva de la lengua. Al evaluar el caso usted
esperaría encontrar……….
(Unidad 2, sesión 10, logro 5: Describe la Irrigación e inervación de la lengua)
a. Alteración en la sensación del dolor y temperatura en el tercio posterior de la lengua
b. Alteración en la sensación del dolor en los dos tercios anteriores de la lengua c.
Alteración en la sensación del gusto en el tercio posterior de la lengua
d. Sensación del dolor, tacto y temperatura conservadas

20. En un paciente de 45 años de edad con colestasis biliar, se encuentra una elevación de los niveles
sanguíneos de fosfatasa alcalina hasta 3 veces la cifra normal. ¿Cuál de las siguientes alternativas estará
también elevada como evidencia del daño de la vía biliar?

(Unidad 3, sesión 22, logro 5: Pruebas de función hepática, Insuficiencia hepática, encefalopatía hepática
e hipertensión portal)
a. Tiempo de protrombina y albúmina sérica
b. Transaminasas hepáticas (ALT y AST)
c. Gamma glutamil transpeptidasa
d. Glucoronil transferasa

21. Experimentalmente se incrementa la velocidad de la secreción salival con una sustancia, en el


análisis de la composición de esta saliva obtenida se espera encontrar…………..

(Unidad 3, sesión 17, logro 5 : Explica la Influencia de la velocidad del flujo salival en la composición de la
saliva)
a. disminución de la concentración de bicarbonato que supera la concentración plasmática
b. aumento de la concentración de cloro y sodio que supera la concentración plasmática c.
aumento de la concentración de bicarbonato que supera la concentración plasmática d.
disminución de concentración de potasio y bicarbonato

22. Lactante de 3 meses de vida es atendido por presentar diarrea, se administra una solución de
glucosa y electrólitos por vía oral. La proteína de membrana apical que explica la capacidad de esta
solución para proporcionar aporte de glucosa e hidratación es ………..

(Unidad 4, sesión 26, logro 1: Explica la Digestión y Absorción de los hidratos de carbono. Alteraciones)
a. CFTR
b. SGLT-1
c. GLUT-2
d. GLUT-5

23. Paciente ha sufrido herida de bala en el abdomen, se le ha tenido que extirpar el segmento medio y
distal del ileon. En este caso la síntesis hepática de sales biliares estará …..…..

(Unidad 3, sesión 22, logro 4: Explica la formación, función y Circulación entero hepática de lasa sales
biliares)
a. Sin cambios en el ritmo de síntesis
b. Disminuida por inhibición de la enzima colesterol 7 alfa hidroxilasa
c. Incrementada por estímulo de la enzima colesterol 7 alfa hidroxilasa
d. Incrementada por inhibición de la enzima colesterol 7 alfa hidroxilasa
24. Un varón de 75 años ingresa al consultorio por presentar ictericia marcada de piel y las escleras. El
estudio del paciente mostró que presentaba un tumor que obstruía la totalidad del conducto hepático
común. ¿Cuál de los siguientes conductos se encontrará dilatado en este paciente?

(Unidad 3, sesión 21, logro 6: Describir el árbol biliar intrahepático)


a. de Wirsung
b. de Hering
c. colédoco
d. cístico

25. Correlaciones las dos columnas y marque la fórmula correcta:


(Unidad 4, sesión 28, logro 1: Diarrea: definición, mecanismos: osmótica, secretoria y exudativa)
1. Enfermedad Hirschsprung( ) heces con moco y sangre
2. Diarrea osmótica( ) intolerancia a lactosa
3. Diarrea secretoria( ) aganglionosis congénita
4. Diarrea exudativa( ) canales de Cl- en las células de la cripta

a.- 4231
b.- 1234
c.- 2143
d.- 4213
26. La vena umbilical obliterada del hígado después del nacimiento se transforma en el ligamento:

(Unidad 3, sesión 21, logro 1: Hígado: relación con la pared abdominal, caras, lóbulos, ligamentos , hilio
hepático)
a. cruzado
b. redondo
c. coronario
d. falciforme

27. Llega a su guardia nocturna una madre que trae a su RN masculino de 2 semanas de vida con mal
estado general y sequedad de mucosas. Usted observa que lacta ávidamente, pero a las 2 horas
presenta vómito postprandial no bilioso en proyectil. Al realizar la historia clínica, descubre que el
lactante recibió profilaxis con macrólidos para tos ferina. Usted sospecha principalmente en:

(Unidad 1, sesión 4, logro 4: Desarrollo y anomalías del intestino anterior)


a. estenosis pilórica hipertrófica congénita
b. fistula traqueo esofágica
c. estenosis duodenal
d. atresia duodenal

28. En la regulación del apetito y la saciedad, la estimulación experimental crónica del núcleo ventro
medial del hipotálamo producirá:

(Unidad 1, sesión 3, logro 4: explica los mecanismos de control del apetito y saciedad)
a. afagia
b. obesidad
c. hiperfagia
d. activación de neuronas relacionadas a NPY

29. Paciente mujer de 25 años acude por dolor en fosa ilíaca derecha que empeora al toser o caminar,
asociada a náuseas y vómitos por lo cual acude a emergencia. Dos días después de realizarle una
apendicectomía, la paciente desarrolla fiebre alta (39 °C), está hipotensa y presenta dolor abdominal. La
laparotomía exploratoria muestra un gran volumen de sangre en la cavidad peritoneal por lesión de un
vaso producida durante la apendicectomía.
¿Cuál de las siguientes arterias debe ligarse para detener la hemorragia?
(Unidad 4, sesión 27, logro 4: Irrigación arterial del colon, recto y conducto anal)
a. cólica derecha y arteria rectal superior.
b. ileocólica y arteria cólica media.
c. mesentérica superior.
d. ileocólica.

30. La onda peristáltica secundaria del esófago se caracteriza por ser originada ………

(unidad 2, sesión 12, logro 3: Motilidad esofágica: fases y características)


a. por el plexo de meissner del esófago
b. por el plexo mientérico del esófago
c. por el reflejo de la deglución
d. durante la masticación

31. ¿Cuál de los siguientes es una causa de ictericia con bilirrubina conjugada aumentada?

(Unidad 3, sesión 22, logro 3: Producción y excreción de bilirrubina. Tipos de bilirrubina, ictericia)
a. Ictericia del recién nacido
b. Obstrucción del colédoco
c. Anemia hemolítica
d. Gran hematoma

32. En relación a la absorción de nutrientes, la absorción de dipéptidos y tripéptidos a nivel de las células
epiteliales del intestino delgado, se da principalmente debido a:
(Unidad 4, sesión 26, logro 2: Digestión y absorción de las proteínas. Alteraciones)
a. el incremento de los canales de Cl- en la membrana apical
b. la gradiente de bicarbonato en la membrana basal
c. la gradiente de iones H+ en la membrana apical
d. la gradiente de Na+ en la membrana apical

33. Paciente de 20 años es traído a la emergencia por presentar diarreas desde hace 2 días. Familiar
refiere que las deposiciones son líquidas y abundantes, al examen luce deshidratado y se plantea que la
diarrea es producida por una toxina que estimula la transformación de ATP a AMPc con apertura de
canales de Cl- y pérdida de agua. El tipo de diarrea más probable es:
(Unidad 4, sesión 28, logro 1: Diarrea: definición , mecanismos: osmótica, secretoria y supurativa)
a. osmótica
b. exudativa
c. secretoria
d. por intolerancia a lactosa

34. Un niño fue operado por una obstrucción intestinal, observándose la presencia de divertículo de
Meckel. Según lo referido, marque lo correcto:

(Unidad 1, sesión 5, logro 2: identifica las anomalías del desarrollo del intestino medio: onfalocele y
gastrosquisis (diferencias), Divertículo de Meckel)
a. el 50% de la población lo presenta
b. se localiza en el íleon muy cerca al yeyuno
c. puede poseer tejido gástrico o pancreático
d. se produce por una mala rotación de los intestinos

35. Marque la alternativa correcta respecto a la estructura marcada en el gráfico:


(Unidad 3, sesión 22, logro 2: Secreción biliar. Visión general del sistema biliar extrahepático y
composición de la bilis)
a. Se halla a 2 centímetros debajo de la papila duodenal mayor
b. Llega el conducto colédoco y pancreático principal
c. Llega el conducto hepático común y pancreático principal
d. Llega el conducto pancreático accesorio

36. ¿Cuál de las siguientes moléculas se encontrará aumentada en el citoplasma de las células parietales
de un paciente con sindrome de Zollinguer Ellison?

(Unidad 3, sesión 20, logro 4: Enfermedad ulcerosa péptica: úlcera gástrica, duodenal. síndrome de
Zollinger – Ellison)
a. Péptido liberador de gastrina (GRP)
b. Proteína G estimulante (GS)
c. Inositol Trifosfato (IP3)
d. AMP cíclico (AMPc)

37. Los fármacos inhibidores de la bomba de protones, actúan bloqueando la ………..……..

(Unidad 3, sesión 20, logro 3: Regulación de la secreción gástrica: inhibición, Secreción de pepsinógeno y
factor intrínseco)
a. anhidrasa carbónica
b. ATPasa H+/K+ en la membrana luminal
c. ATPasa H+/K+ en la membrana basolateral
d. ATPasa Na+/K+ en la membrana basolateral

38. Un paciente fue diagnosticado de gastritis autoinmune, ¿cuál de las siguientes alternativas es FALSA
respecto a esta enfermedad?
(Unidad 3, sesión 20, logro 5: Gastritis crónica: helicobacter pylori, autoinmune. Tipos de gastritis)
a. Afecta principalmente el fondo y cuerpo gástrico
b. Se produce hiperplasia de células G secundaria a la aclorhidria
c. El propio sistema inmune destruye principalmente las células parietales
d. Se produce atrofia de la mucosa, aclorhidria, hipergastrinemia y déficit de vitamina

B6 39. Marque la correlación correcta:

(Unidad 3, sesión:18, logros:1 y 2: Describe las enfermedades inflamatorias/infecciosas y proliferativas


de la cavidad oral)
1. Herpes virus( ) En relación al abuso de antibióticos
2. Candidiasis oral( ) Lesiones vesiculares como racimo de uvas
3. Eritroplaquia( ) Mega esófago
4. Enfermedad de Chagas( ) Lesión pre cancerígena

a.- 2431
b.- 1234
c.- 4123
d.- 2143
40. En un paciente con insuficiencia renal crónica, el déficit en la absorción de calcio a nivel del
enterocito se debe a lo siguiente:

(Unidad 4, sesión 26, logro 6: Explica la Absorción de calcio y hierro)


a. No se convierte la 25 hidroxicolecalciferol a 1,25 dihidroxicolecalciferol
b. No se convierte la 1,25 dihidroxicolecalciferol a 25 hidroxicolecalciferol
c. Existe un descenso de la alfa 25 hidroxilasa renal
d. Se incrementa la producción de Calbindina
SISTEMA DIGESTIVO (ME154) EXAMEN FINAL 2019 01

1. La explicación fisiológica de presentar somnolencia de 30 minutos a 1 hora después de ingerir


alimentos, se explica por:
a. Aumento del cloro intraluminal
b. Aumento del bicarbonato intraluminal
c. Disminución de ácido carbónico en la célula parietal
d. Disminución de la actividad de la anhidrasa carbónica
e. Aumento de la alcalinidad sanguínea

2. Con respecto a la irrigación arterial del colon, a que arteria corresponde la señalada con la flecha
a. Cólica derecha
b. Cólica media
c. Cólica izquierda
d. Ileobisecoapendículocólica
e. Arco de Riolano

3. Si un paciente presentara dentro del punto de vista fisiológico, una disminución de enterocinasa,
entonces esto originaría una disminución de la actividad de:
a. la pepsina
b.la lipasa
c. la quimotripsina
d. el peptido insulinotropo dependiente de glucosa
e. la amilasa
Se valida la opción b debido a su relación con la colipasa.

4. Con respecto a la anatomía del hígado, señale a que estructura pertenece la marcada por el número
1.
a. Ligamento falciforme
b. Línea de Cantlie
c. Ligamento triangular
d. Ligamento coronario
e. Ligamento teres
Se valida la opción a debido a la ubicación del número 1 en
donde se unen el ligamento falciforme y ligamento
coronario.
5. Se presenta un paciente, el cual presenta un antecedente de tuberculosis intestinal, por lo cual, se le
resecó 80 cm de íleon distal. Desde el punto de vista fisiológico, el paciente puede presentar una de las
siguientes alteraciones:
a. Disminución de la secreción de Vitamina B12
b. Aumento indiscriminado de absorción de ácido fólico
c. Disminución de la absorción de hierro
d. Aumento de la secreción de bicarbonato
e. Disminución de la absorción de ácido glicocólico

6. Un paciente es sometido experimentalmente a un fármaco que modifica el flujo salival, obteniéndose


un volumen de saliva de 288 ml en 6 horas. En este caso las concentraciones de electrolitos y
bicarbonato en la saliva obtenida varían de la siguiente manera:
a.↑ Na+, ↓ K+, ↑ Cl-, ↑ HCO3-
b.↓ Na+, ↓ Cl-, ↑ K+, ↓ HCO3-
c.↑ Na+, ↑ Cl-, ↓ K+, ↓ HCO3-
d.↑ Na+, ↑ Cl-, ↑ K+, ↑ HCO3-
e.↓ Na+, ↓ Cl-, ↓ K+, ↓ HCO3-
Se valida la opción a debido a que se puede considerar como un aumento del flujo de

saliva. 7. La siguiente imagen histológica corresponde a la glándula

…………… y la estructura señalada produce ………


a. salival sublingual / mucopolisacáridos
b. oxíntica / pepsinógeno
c. salival submaxilar / ptialina
d. salival parótida / amilasas
e. antrales / gastrina

8. Paciente varón de 65 años con antecedente de hipercolesterolemia, hipertensión arterial, fibrilación


auricular y dos infartos al miocardio previos, aqueja de dolor abdominal intenso de inicio súbito,
distensión abdominal, se decide cirugía con resección de 1,5 metros de intestino delgado terminal y
colon ascendente. Como consecuencia de la resección el paciente tendrá deficiencia de: a. Vitamina C
b. Tiamina
c. Vitamina A
d. Vitamina B1
e. Vitamina B6
Se valida esta opción debido a que su absorción está relacionada al íleon.

9. Uno de los siguientes elementos debería hallarse con más probabilidad en el esófago de un paciente
que sufre de
reflujo gastro esofágico…
a. Pepsina
b. Tripsina
c. Quimiotripsina
d. Carboxipeptidasa
e. Ácidos biliares

10. Un paciente de 40 años cursa con anemia de 8g/dl, aqueja además de astenia y sensación de
hormigueo bilateral en los miembros inferiores, al examen se halla alteración de la sensibilidad a la
vibración y camina con ampliación de la base de sustentación. Uno de los siguientes procedimientos
sería de ayuda para el diagnóstico de este paciente:
a. Tomografía cerebral
b. Biopsia de la mucosa gástrica
1. Un niño de 2 años es llevado a la consulta por diarrea persistente y edema de las
extremidades, además falta de crecimiento y desarrollo en relación a su edad. Los
análisis de sangre revelan que tiene concentración plasmática baja de proteínas
(hipoproteinemia). Durante la endoscopía duodenal, se coloca colecistocinina (CCK)
endovenosa y se recoge muestras del líquido duodenal; el resultado del líquido
confirma incapacidad para hidrolizar proteínas a un pH neutro, esta situación mejora
al añadir una pequeña cantidad de tripsina. El paciente probablemente esté
sufriendo la falta congénita de
​-Enterocinasa
2. Experimentalmente se incrementa la velocidad de la secreción salival con una
sustancia, el análisis de la composición de esta saliva obtenida se espera
encontrar…………..
-Disminución de concentración de potasio
3. Paciente varón de 46 años soltero, consulta por odinofagia y bajo de peso, tiene
antecedente de tuberculosis desde hace 3 meses y es fumador crónico (10
cigarrillos por día); al evaluar la cavidad oral se identifica lesión blanquecina en el
dorso de la lengua y paladar blando, las lesiones se desprenden con el baja lengua
dejando una base eritematosa. Esta lesión corresponde probablemente a
……………………….…..
-​ Candidiasis oral
4. Paciente mujer de 35 años acude a consulta por sensación de sequedad y lesiones
en cavidad oral. Al examen se observa atrofia de la mucosa, fisuras y úlceras; nota
además sequedad e irritación de la córnea y aumento del tamaño de las glándulas
parotídeas. Su diagnóstico más probable es artritis reumatoide; el hallazgo más
probable en una biopsia de glándula parótida es……..….
​-Gran infiltración de linfocitos y células plasmáticas
5. Un paciente con anemia acude con su médico quejándose de episodios frecuentes
de gastroenteritis. Un análisis de sangre revela anticuerpos circulantes dirigidos
contra células parietales gástricas. Su anemia es atribuible a la hiposecreción de
​-Factor intrínseco
6. Dos estudiantes deciden tomar un receso para comer una hamburguesa a la hora
del almuerzo. Antes de llegar a la cafetería, impulsos nerviosos provenientes del
complejo vagal dorsal iniciarán la secreción de ácido gástrico por la liberación de
…………………….. desde el sistema nervioso entérico.
-GRP
7. Un niño de cuatro años de edad es llevado a la consulta por cuadros diarreicos
frecuentes caracterizados por heces pálidas, voluminosas y fétidas, presenta bajo
peso y talla. Se mide la concentración de cloruro en el sudor y se encuentra que sus
valores son muy elevados. La alteración más importante a nivel de células ductales
del páncreas tiene relación directa con la conductancia de…………
​-Cloro
8. Una mujer de 50 años de edad que sufrió durante varios años resequedad de los
ojos debida a producción inadecuada de lágrimas es enviada con un
gastroenterólogo para evaluación de pirosis crónica. El examen endoscópico
revela erosiones y tejido cicatrizal en la parte distal del esófago justo por arriba
del esfínter esofágico inferior. Las lesiones pueden atribuirse a la disminución de
uno de los siguientes componentes salivales:
​-Bicarbonato
9. Se evalúa los valores séricos de las siguientes sustancias a un paciente con
enfermedad hepática terminal; en este paciente se espera encontrar la combinación
con la letra …………
​-disminuida, aumentada, disminuida
10. Una mujer de 35 años de edad HIV positiva, se presenta al médico con dolor
abdominal en cuadrante superior derecho e ictericia. La paciente refiere haber tenido
múltiples episodios de ictericia durante los últimos 10 años. Los exámenes para
determinar hepatitis viral, dieron positivos para Hepatitis B, siendo catalogado el caso
como hepatitis crónica con alteración funcional. En un examen de sangre ¿cuál de los
siguientes parámetros está disminuido?
​-Albúmina
11. En el reflejo peristáltico del intestino delgado, uno de los siguientes eventos sucede
en la porción oral del bolo alimenticio…………...
​-Acción de acetilcolina en el músculo circular
12. Experimentalmente se coloca una dosis alta de secretina en la luz intestinal
duodenal; como consecuencia de esto, en el jugo pancreático de la misma luz
intestinal se observa la disminución de la concentración de …..………..
​-Cl
13. Un varón de 58 años de edad con enfermedad de Crohn severo fue sometido a una
resección ileal. Después de la cirugía este paciente padecerá de esteatorrea, esto se
explica porque …..………..
- ​La micelas no pueden formarse
14. En un experimento se inserta un balón en el estómago de un voluntario, se infla poco
a poco mientras que se vigilan las presiones intraluminales. Aunque el volumen del
balón aumenta considerablemente, las presiones permanecen constantes. Esta
relación volumen-presión se explica por la liberación local de …………..
​-Óxido nítrico y péptido inhibidor vasoactivo
15. La toxina de Vibrio cholerae causa diarrea debido a…….
​-El Incremento de la secreción de cloro por las células de la cripta intestinal
16. ¿Cuál de las siguientes alternativas es una característica de la secreción exocrina
del páncreas?
​-Tiene una baja concentración de Cl- respecto al plasma
17. Una madre lleva a su hijo de dos años de edad a la sala de urgencias, estresada
porque el niño deglutió una moneda de 10 céntimos mientras la familia cenaba en un
restaurante. El médico observa mediante fluoroscopía que la moneda se halla en el
estómago y asegura a la madre que la moneda se eliminará con las heces. El médico
recomienda utilizar la respuesta fisiológica que permitirá la evacuación de la moneda
del estómago al intestino ………….…..
​-Son los movimientos de mezcla y trituración
-​ . Es provocada por el ayuno
18. Las estructuras en el hígado que permite que los productos metabólicos unidos a
proteínas tengan acceso a las membranas basolaterales de los hepatocitos, son…..
-Las fenestras sinusoidales
19. La composición de la bilis es modificada conforme fluye por los conductillos biliares.
Durante este tránsito se espera que aumente la concentración de…….
​-Monómeros de ácido biliar
-Ig A
20. Se mide experimentalmente el contenido gástrico de dos personas. La persona “A”
tiene alto contenido de grasa y la persona “B” tiene un contenido hipertónico ¿Cuál
de las siguientes es correcto respecto al vaciamiento gástrico?
- Hay ralentización del vaciado gástrico en ambos casos
21. El examen endoscópico de un paciente con hipertensión portal grave revela venas
tortuosas que sobresalen hacia la luz del esófago. El paciente recibe tratamiento
quirúrgico mediante la colocación de una derivación que conecta la vena porta a la
vena cava. Después de la operación el riesgo de encefalopatía y el
riesgo de sangrado de várices ……………..
​-Aumentará/disminuirá
22. Un paciente varón de 18 años de edad acude al médico para sus exámenes de
rutina. Sus resultados de laboratorio muestran un valor de bilirrubina sérica de 4
mg/dl y una bilirrubina directa de 0,3 mg/dl. Las pruebas de función hepática son
normales. La alteración que explica mejor este caso es por la deficiencia de
………………..
-Glucuronil transferasa
23. Un hombre de 57 años de edad es llevado a urgencias con hematemesis masiva
rojo brillante, a su llegada se halla inconciente con PA: 80/40 mm Hg y FC: 124
lat/min. Luce ictérico con presencia de “arañas vasculares en el tórax anterior y
extremidades”, abdomen distendido con signo de oleada positiva. Se encuentra
esplenomegalia y pérdida de la masa muscular en extremidades. La anastomosis
vascular responsable del sangrado en este paciente es
​-Vena gástrica izquierda y vena ácigos
24. Un estudiante de medicina está comiendo un plato de comida a base de
champiñones, espárrago y salsa de soya. El sabor umami contenido en todos estos
alimentos actúa a nivel de los botones gustativos estimulando ………………..
​-Un receptor acoplado a proteína G
25. Un hombre de 22 años de edad se presenta al médico con una historia de 1 año de
evolución caracterizado por dolor recurrente en fosa iliaca derecha y diarrea.
Manifiesta además pérdida de peso de 8 kg durante este periodo. La colonoscopía
revela múltiples lesiones en el ileon terminal y colon. La biopsia de estas lesiones
revela engrosamiento, inflamación y ulceración de la mucosa. El diagnóstico más
probable en este caso es…….
​-Enfermedad de Crohn
26. Varón de 61 años que consulta por dolor retro esternal intenso desde hace 6 horas y
después de vómitos intensos y repetidos; al examen se observa disnea, cianosis,
hipotensión y signos clínicos de shock. La radiografía simple de tórax muestra
neumomediastino. El líquido en el espacio pleural aspirado tiene alta concentración
de amilasa. ¿Cuál de las siguientes alternativas puede explicar este cuadro clínico?
-​Rotura espontánea de esófago
27. La secreción del ácido en la célula parietal gástrica se lleva a cabo por una ATPasa
especifica que intercambia hidrogeniones (H+) del citosol por…..
​-K +
28. En condiciones normales el ingreso de 600 ml de líquido es el estómago provoca un
aumento de presión intragástrica de unos 12 cm de H2O. Después de una
vagotomía (corte del nervio vago) es de esperar que el ingreso del mismo volumen
de líquido provoque lo siguiente: …………………………………
​-Un aumento mayor de la presión
29. Una paciente de 30 años de edad es sometida a una cirugía en oído medio derecho
por un problema de otoesclerosis. Luego de la cirugía refiere alteración en la
percepción de sabores. Al evaluar el caso usted esperaría encontrar……….
​-Alteración en la sensación del gusto en los dos tercios anteriores de la
l​ engua
​-Sensación del dolor, tacto y temperatura conservada en toda la lengua
30. ¿Cuál de las siguientes alternativas es correcta?
​-Las sales biliares desconjugadas son absorbidas preferentemente en el colon
31. En un paciente de 45 años de edad con colestasis biliar, se encuentra una elevación
de los niveles sanguíneos de fosfatasa alcalina hasta 3 veces la cifra normal. ¿Cuál
de las siguientes alternativas estará también elevada como evidencia del daño de la
vía biliar?
​-Gamma glutamil transpeptidasa
32. Revisando la angiografía de un hombre de 70 años en estudio por aneurisma de
aorta abdominal el radiólogo informa de la presencia de una oclusión completa de la
arteria mesentérica inferior. El paciente se encuentra completamente asintomático.
¿Cuál de las siguientes arterias se anastomosa a la sistema arterial de la
mesentérica inferior?
​-Cólica media
33. Lactante de 3 meses de vida es atendido por presentar diarrea, se administra una
solución de glucosa y electrólitos por vía oral. La proteína de membrana apical que
explica la capacidad de esta solución para proporcionar aporte de glucosa e
hidratación es ………..
​-SGLT-1
34. Paciente ha sufrido herida de bala en el abdomen, se le ha tenido que extirpar el
segmento medio y distal del ileon. En este caso la síntesis hepática de sales biliares
estará …..…..
​-Incrementada por estímulo de la enzima colesterol 7 alfa hidroxilasa
35. Un varón de 75 años ingresa al consultorio por presentar ictericia marcada de piel y
las escleras. El estudio del paciente mostró que presentaba un tumor que obstruía la
totalidad del conducto hepático común. ¿Cuál de las siguientes estructuras se
encontrará dilatada en este paciente?
​-Conductos de Hering
36. En un paciente con insuficiencia renal crónica, el déficit en la absorción de calcio a
nivel del enterocito se debe a lo siguiente:
​-No se convierte la 25 hidroxicolecalciferol a 1,25 dihidroxicolecalciferol
37. Varón de 30 años es traído a emergencia por agresión abdominal con arma de fuego
(pistola) y es sometido a laparotomía exploratoria, observándose isquemia del colon
ascendente y parte del colon trasverso ¿la lesión de cuál de las siguientes arterias
explicaría esta isquemia?
d. Mesentérica superior
38. Respecto a las sustancias gastrointestinales que regulan la secreción pancreática;
marque la afirmación correcta:
b. La acetilcolina es capaz de estimular la secreción enzimática y de bicarbonato del
páncreas
39. Ante una lesión del X par craneal, ¿cuál de los siguientes músculos mantiene
conservada su función?:
b. Tensor del velo del paladar
40. Experimentalmente se utiliza atropina (anticolinérgico) para inhibir la secreción de
gastrina, sin embargo la secreción de esta hormona se sigue dando ante estímulos
vagales. Esta situación se explica porque la atropina:
d. No bloquea la acción del péptido GRP
41. Un varón de 50 años es sometido a extirpación del duodeno y parte proximal del
yeyuno. Esta situación ocasionaría la pérdida de las células ​………​.. , productoras de
………………… ​que estimula la secreción de bicarbonato por el páncreas.
“S” / secretina
42. Se evalúa la expresión de la proteína Agrp en una persona con alteración del apetito; lo
correcto respecto a esta proteína es​…​..
La mutación del gen que la codifica produce adelgazamiento
43. Juana cae de la bicicleta y se fractura la región anterior del hueso maxilar superior con
compromiso de la fosa incisiva. Al examen físico de la región esperaría encontrar
alteración en la sensibilidad de la encía ​…………………
palatina anterior
44. Recién nacido es atendido por el neonatólogo y luego entregado a su madre para dar
de lactar; la madre al dar de lactar observa coloración azulada de labios, acompañado
de tos persistente, dificultad respiratoria y distención abdominal. Se le intenta colocar
una sonda nasogástrica pero esta retorna a la cavidad oral en todos los intentos.
¿Cuál de las siguientes anomalías del desarrollo es el más probable en este caso?
b. Atresia esofágica proximal con fístula traqueoesofágica distal l
45. ¿Cuál de los siguientes mecanismos ocurre durante la defecación?
En la posición de “cuclillas” el músculo puborrectal se halla relajado
46. Un paciente luego de un accidente sufre lesión del piso de la boca, se constata daño
del nervio “cuerda del tímpano”, en este caso se esperaría encontrar disminución de
la​…………………………​.​… ​de la lengua
Sensación del gusto en los dos tercios anteriores
47. ¿Cuál de las siguientes afirmaciones es la correcta sobre la gastrina?
Actúa en la célula diana mediante su receptor CCk tipo B
48. Al recibir un paciente con signos de hipovolemia y antecedente de trauma en
abdomen por accidente de tránsito, usted identifica radiológicamente: lesión de
primera vértebra lumbar y signos de lesión en páncreas; durante la cirugía se observó
pobre irrigación de asas intestinales. El vaso afectado es la arteria ​……​..
c. mesentérica inferior
49. Un paciente sufre de daño a nivel del cuello con lesión muscular en la región de la
faringe. En el examen físico se determina dificultad para la elevación de la faringe y
para el cierre del itsmo de las fauces. En este caso, probablemente esté afectado el
músculo:
c. palatofaringeo
50. Varón de 50 años a quien le realizan la curación de la segunda molar de la arcada
superior derecha. En un momento determinado, el paciente acusa de intenso
dolor de la pieza dentaria en tratamiento. La vía aferente del dolor viaja a través
del nervio ​…………
a. trigémino V2
51. La distención gástrica por los alimentos produce incremento de secreción de HCl
mediante la producción de ​…………​.. que estimula a las célulasvía proteína
………​..
a. gastrina / parietal / Gq
52. Un niño de tres años llega a emergencia con disfagia (dificultad para tragar), dolor
retro esternal, salivación y llanto. Se sospecha de ingesta de cuerpo extraño
(moneda) en el esófago; al ser evaluado se constata en una radiografía presencia de
cuerpo extraño a nivel de C6 (6° vértebra cervical). El cuerpo extraño estará
suspendido a nivel del estrechamiento producido por​………​..
c. el músculo cricofaríngeo
53. La triada portal (arteria hepática, vena portal y conducto biliar común) está contenida
en el ligamento ​……​.​……… ​y derivan embriológicamente del ​……
a. hepato duodenal / mesenterio ventral
54. Un paciente refiere no percibir algunos sabores, al examen físico se constata
alteración en la percepción de sabores y del dolor en el tercio posterior de la lengua
¿Cuál de los siguientes nervios estará alterada en su función?
c. Glosofaríngeo (IX par)
55. En el caso de un paciente con gastrinoma (tumor productor de gastrina), la presencia
de úlceras duodenales y erosión de la mucosa gástrica, se debe principalmente
a​……​.
c. el exceso de HCl por estímulo de receptores CCK-B en la célula parietal
56. El reflejo entero gástrico se caracterizan por:
d. originarse debido a la distensión duodenal y presencia del quimo ácido
57. Mauricio tiene dificultad para deprimir el paladar y elevar la parte posterior de la lengua.
En este caso estará afectado un músculo, específicamente el músculo
……………​.
b. extrínseco – palatogloso
58. En condiciones normales, el ingreso de 600 ml de líquido es el estómago provoca un
aumento de presión intragástrica de unos 12 cm de H2O. Después de una vagotomía
(corte del nervio vago) es de esperar que el ingreso del mismo volumen de líquido
ocasione
………………………………… ​de la presión
intragástrica.
c. un aumento mayor
59. La explicación fisiológica de presentar somnolencia de 30 minutos a 1 hora
después de ingerir alimentos, se explica por: a. Aumento del cloro intraluminal
e. Aumento de la alcalinidad sanguínea
60.Se presenta un paciente, el cual presenta un antecedente de tuberculosis intestinal,
por lo cual, se le resecó 80 cm de íleon distal. Desde el punto de vista fisiológico, el
paciente puede presentar una de las siguientes alteraciones: a. Disminución de la
secreción de Vitamina B12

e. Disminución de la absorción de ácido glicocólico


61. Un paciente es sometido experimentalmente a un fármaco que modifica el flujo salival,
obteniéndose un volumen de saliva de 288 ml en 6 horas. En este caso las concentraciones
de electrolitos y bicarbonato en la saliva obtenida varían de la siguiente manera: a. ​↑
Na+,
↓ K ​+, ​↑ ​Cl-, ​↑ ​HCO3-

1. b. ​↓ N
​ a+, ​↓ ​Cl-, ​↑ ​K+, ​↓ ​HCO3-

62. Uno de los siguientes elementos debería hallarse con más probabilidad en el esófago
de un paciente que sufre de reflujo gastro esofágico​…
a. Pepsina

63. Un paciente de 40 años cursa con anemia de 8g/dl, aqueja además de astenia y sensación
de hormigueo bilateral en los miembros inferiores, al examen se halla alteración de la
sensibilidad a la vibración y camina con ampliación de la base de sustentación. Uno de los
siguientes procedimientos sería de ayuda para el diagnóstico de este paciente:
a. Tomografía cerebral
b. Biopsia de la mucosa gástrica

64.Paciente de 60 años ingresa por caída hace 1 hora y pequeño hematoma en cuero
cabelludo, al examen físico ampliado se observa ictericia de piel y mucosas generalizada,
abdomen blando, se palpa estructura quística no dolorosa en hipocondrio derecho que
corresponde a vesícula biliar (signo de Courvoisier), en los exámenes de laboratorio se halla
niveles bajos en la formación de estercobilinógeno y urobilinógeno en heces, incremento de
la bilirrubina conjugada en la orina, elevación de fosfatasa alcalina y gamma glutamil
transpeptidasa séricas. El presente cuadro puede ser explicado por: a. Reabsorción de
hematoma
c.Carcinoma de la cabeza de páncreas
65. Un recién nacido presenta vómitos biliosos poco tiempo después de cada alimento. Al
preguntar a la madre sobre antecedentes, ella recuerda que tuvo polihidramnios durante la
gestación, pero un análisis de cariotipo fue normal. Una de las siguientes es la causa más
probable de estos hallazgos en el recién nacido: a. Enfermedad de Hirschprung
e. Malrotación de la yema pancreática ventral
66.En un estudio de la secreción de hormonas gastrointestinales, sus concentraciones en la
vena porta se midieron durante perfusión luminal del intestino delgado con soluciones de
diversas magnitudes de pH. ¿Qué hormona aumentará en el plasma de la vena porta durante
perfusión a través del intestino con una solución de pH 3?
a. CCK
e. secretina
67.​Paciente de 30 años que ingresa a causa de un traumatismo abdominal cerrado. En la
exploración se aprecia discreta palidez de piel y mucosas, auscultación pulmonar normal,
taquicardia de 120 /min. Discreta distensión abdominal y matidez en flancos; el hematocrito,
que era prácticamente normal al ingreso, disminuye a 30% a las tres horas. En la Rx de tórax
se objetiva fractura de las costillas 10-11 izquierdas. La causa más probable de la
anemización en este paciente es: a. traumatismo renal con hemorragia retroperitoneal.
c. rotura de bazo con hemoperitoneo.
68. Revisando la angiotomografía de un hombre de 70 años en estudio por aneurisma de
aorta abdominal, el radiólogo le informa de la presencia de una oclusión completa de la
arteria mesentérica inferior. El paciente se encuentra completamente asintomático. La
oclusión de la arteria mesentérica inferior cursa de manera asintomática en muchas
ocasiones ya que el territorio que irriga puede recibir flujo proveniente de la arteria:
a. cólica derecha
e. cólica media

69. En las patologías de esófago es importante conocer bien la anatomía esofágica. ¿Cuál de
las siguientes afirmaciones es correcta? a. El esófago tiene capa mucosa, muscular y serosa

c. El esófago torácico pasa por detrás del cayado aórtico

70. A pesar de que pueda haber variaciones anatómicas, lo habitual es que el ciego sea
irrigado por una rama arterial que proviene de unas de las siguientes arterias: a. Iliaca
derecha

d. Mesentérica superior

71. Ante un paciente con una cirugía abdominal urgente, el informe operatorio señala que
se ha realizado una resección de todo el duodeno y del tercio proximal del yeyuno
manteniendo íntegros el estómago y todo el íleon, así como los dos tercios distales del
yeyuno. En el seguimiento nutricional del paciente ¿Qué vitamina o mineral presentará con
menor probabilidad una disminución de su absorción?
a. Cianocobalamina

72. ¿Cuál de las siguientes sustancias forma parte de la secreción biliar? a. Tripsina
Lecitina

73.¿De qué musculo forma parte el ligamento inguinal?


-Oblicuo externo del abdomen
74.¿Cuál de las siguientes enzimas está localizada en el borde en cepillo y juega un rol en la
digestión de proteínas?
e. ​Carboxipeptidasa A.
75. Una de los siguientes sustancias, ​NO ​sirve como un buen agente emulsificante:
a. Colesterol
76. ​La sustancia que estimula el crecimiento de la mucosa gástrica es:

a. Secretina

d. Gastrina

77.¿Cuál de las siguientes alternativas es una función de la colecistokinina?

a. Relajación de la vesícula para la salida de bilis

d. Secreción de enzimas pancreáticas

78.​Con respecto a la anatomía del tronco celiaco, señale lo correcto a. El tronco celiaco se
origina de la cara posterior de la aorta abdominal
d. La hepática común que es una de sus ramas, participa en la irrigación del
estómago.
79. ​Con respecto a la anatomía del duodeno, marque la respuesta correcta: a. Tiene una
distribución en forma de “C”, que rodea la cola del páncreas
b. La 3ra porción duodenal está contenida en la pinza vascular aortomesentérica
80. ​En el íleon se absorbe aproximadamente el 95% dea través de la circulación
enterohepática.
a. agua
c. sales biliares
81. La estimula el mecanismo paracrino de la secreción de ácido clorhídrico.
a. histamina
82.​En la digestión de proteinas,es el principal estímulo para convertir el
pepsinógeno en pepsina. a. la gastrina
b. el pH ácido
83. ​Con respecto a la somatostatina, marque lo correcto:
a. Es secretada por las células S del intestino
Interviene en la fase intestinal de la secreción gástrica
84. ​En pecten anal, es una estructura comprendida entre: a. la línea pectínea y los senos
anales
d. la línea anocutánea y la línea pectínea

85.¿Cuál de las siguientes alternativas es una proenzima pancreática? a. Tripsina

1. b. Elastasa
2. c. Quimotripsinógeno
3. d. Amilasa
4. e. Procarboxipeptidasa C
86. ​En este paciente, el bloqueo farmacológico de los receptores H2 en la mucosa gástrica:
​a) No tiene efecto sobre la secreción de ácido inducida por el vago
b) Evita la activación de adenilciclasa por gastrina
​c) Inhibe la secreción de ácido inducida por gastrina y mediada por el vago
d) Causa un aumento en el transporte de potasio por las células parietales gástricas
Se validó la A :)
87. Si se considerara una gastrectomía total para curar la gastritis del paciente, cuál de
las siguientes sustancias ya no se produciría:
a) Gastrina
b) Quimiotripsina
c) Amilasa
​d) Pepsinógeno

88. Un paciente hipertenso está tomando un medicamento bloqueador de receptores alfa


1 adrenérgicos (prazosina) y como efecto secundario se queja de:
d) Lo escaso que es el medicamento
e) No tiene problemas en la salivación
​c) Hiposalivación
d) Hipersalivación

89. Con respecto a las lesiones y enfermedades de la boca, marque lo correcto:


f) La leucoplasia se desprende al roce
​b) la eritroplasia puede degenerar en adenocarcinoma
c) El muguet oral es una enfermedad bacteriana en inmunodeprimidos
​d) la eritroplasia debe ser biopsiada
​NOTA: fue validada la opción B ya que no es motivo del curso que sepamos el puto cáncer.

90. En este paciente, se puede asumir que la pancreatitis ha sido ocasionada por
una disminución en el efecto de:
g) Amilasa
h) Lipasa
​c) Inhibidor de la tripsina
d) Entercinasa
91.Un efecto secundario en el estómago por la acción de la secretina es:
i) Disminución en la liberación de pepsinógeno
​b) Menor actividad de la pepsina
c) Mayor paso de bicarbonato a sangre periférica
d) Aumento en la producción de factor intrínseco
92. Dentro de los factores protectores de la mucosa gástrica se pueden mencionar
múltiples protagonistas. Uno de ellos es:
j) CCK
k) Gastrina
c​ ) Receptor muscarínico
d) Pepsina
93. La saliva puede tener una variedad de electrolitos en su composición. Entre ellos el
cloro, respecto al cual se puede afirmar:
l) Su mayor concentración se consigue con el flujo bajo
​ ) Su concentración no llega a ser tan alta como en el plasma
b
c) Con flujo alto su concentración es mayor que la del plasma
d) Su menor concentración se alcanza con flujo alto
94. En el síndrome de boca seca o síndrome de Sjogren, una de las
complicaciones asociadas es:
​a) caries
b) Disminución de la acidez gástrica
c) Aumento en de la producción de saliva
d) Aumento del pH bucal
95. Estimula la producción de saliva:
​ ) Vasodilatación periglandular
a
b) Atropina
c) Fatiga o cansancio
d) Expresión de miedo
96. El omeprazol actúa sobre la membrana​ ​de la célula
m) Basolateral/principal
n) apical/principal
o) Basolateral/parietal
​d) apical/parietal
97. Para protegerse del entorno ácido, el Helicobacter pylori se autogenera un entorno de
pH menos ácido alrededor suyo, gracias a una enzima que alcaliniza su entorno local
mediante la conversión de:
​a) urea en NH3
b) H2O y CO2 en ácido carbónico
c) NH3 en urea
d) H2CO3 en bicarbonato
98. La anemia perniciosa destruye las células:
p) mucosas del cuello
b) oxínticas
c) principales
d) mucosas superficiales
99. La célula mucosa del cuello gástrico produce:
a) Moco
b) ácido clorhídrico
c) pepsinógeno
d) Factor intrínseco
100.El aumento en la acidez del estómago producido principalmente por la infección de
Helicobacter pylori se debe a la disminución de:
a) Somatostatina
b) Bicarbonato por las glándulas de Brunner
c) Secretina
d) Colecistoquinina
101. De las siguientes sustancias secretadas por los órganos de este paciente, la más
alcalina es la secreción:
a) pancreática
b) Esofágica
c) Yeyunal
d) Salival
102.En cuanto a la gastritis de este paciente, se encontró que era producida por la bacteria
Helicobacter pylori. Esta bacteria sobrevive en el medio ácido del estómago gracias a:
a) ácido clorhídrico
b) Toxina CagA
c) Ureasa
d) Jugo pancreático
103.La lengua está recubierta por epitelio:
c) pseudoestratificado columnar no queratinizado
b) plano estratificado no queratinizado
c) pseudoestratificado columnar ciliado
d) plano estratificado queratinizado
104.El esfínter anal interno tiene musculatura …….. y tiene control ……..
d) lisa / voluntario
b) lisa / involuntario
c) esquelética / simpático
d) esquelética / parasimpático
105.La arteria Aorta proporciona la irrigación al tubo digestivo ¿cuál de las siguientes
arterias proporciona la irrigación al ángulo cólico derecho?
a) mesentérica superior
b) mesentérica inferior
c) frénica inferior
d) tronco celiaco
106.Paciente de 26 años que le cuenta en su historia clínica que cada vez que almuerza a
los 20 min tiene deseo de defecar, le comenta que su hijo de 1 mes le pasa lo mismo pero
más intenso. Esto se explica por el reflejo …….., el cual está …… en el paciente
e) colicoileal / normal
f) colicoileal / alterado
c) gastrocólico / normal
d) gastrocólico / alterado
107.La región del estómago que se comunica con el duodeno es la
a) pilórica
b) cardias
c) cuerpo
d) fórnix
108.Acude a consulta un px que fue diagnosticado de úlcera péptica 3 días antes. Luego de
múltiples pruebas diagnósticas se concluye que el paciente presenta un tumor secretor de
gastrina, ¿cual de las siguientes situaciones estará incrementada?
g) distensión gástrica
h) inhibición del vaciado gástrico
c) secrecion de acido clorhidrico
d) inhibición de la secreción de pepsinógeno
109.En el sistema digestivo, el control del apetito está dado por un complejo sistema de
sustancias y órganos integradores, los cuales regulan la ingesta de alimentos. La …… es una
sustancia orexígena y es sintetizada por el ……
i) leptina / estómago
j) felina / intestino
k) leptina / estómago
d) grelina / estómago
110. Con respecto a la actividad eléctrica del sistema digestivo, marque la alternativa
correcta
l) corresponden a potenciales de acción que están presentes de forma continua y le
dan capacidad de peristalsis autónoma al sistema digestivo
b) la frecuencia de las ondas lentas NO se ve influenciada por la actividad neural ni las
hormonas gastrointestinales
c) en el estómago las ondas lentas se dan en una frecuencia de 6 x min
d) las ondas lentas son cambios lentos y ondulantes del potencial en reposo
e) la frecuencia de las ondas lentas va de 6 a 12 ondas por minuto
111. Ante una lesión del IX pc, el músculo …… se altera en su función
m) palatogloso
b) estilofaríngeo
c) palatofaríngeo
d) constrictor superior
112. Un varón de 50 años es sometido a extirpación del duodeno y parte proximal del
yeyuno. La pérdida de estímulo hormonal en el páncreas para la secreción enzimática se
explica por la pérdida de células
n) parietales, productoras de factor intrínseco
o) K productoras de factor intrínseco
p) M productora de CCK
d) I productora de CCK
113.Marque la respuesta correcta:
A. El bronquio derecho constituye una de las estrecheces del esófago
B. Todos los órganos del sistema digestivo tienen capa serosa
C. La pared gástrica en el fondo es más delgada que en el cuerpo y antro
D. El esfínter de Oddi rodea a la papila menor duodenal
114. Marque la respuesta correcta en relación a la gastrina:
A. Al distenderse el estómago, se inhibe su producción.
B. Se estimula por la liberación de noradrenalina
C. Las células G son las productoras y se encuentran principalmente en el antro
gástrico
D. Las células G se encuentran principalmente en el fondo gástrico
115. Para poder morder una manzana, es necesario usar el siguiente músculo:
A. Milohiodeo
B. Tensor del paladar
C. Orbicular de los labios
D. Buccinador
116. Sustancia que inhibe la secreción y la motilidad del estómago prolongando el tiempo de
digestión:
A. Enteroglucagon.
B. Polipéptido pancreático
C. Péptido 1 similar al glucagón (GLP-1).
D. Péptido insulinotrópico dependiente de la glucosa (GIP).
117. El nacimiento de la arteria mesentérica superior se puede encontrar en cuál de los
cuadrantes abdominales:
A. Hipocondrio derecho
B. Hipogastrio
C. Epigastrio
D. Mesogastrio
​ ntre las múltiples causas de la Enfermedad por Reflujo Gastroesofágico, se puede
118. E
considerar también a una alteración en las​ ​del esfínter esofágico inferior:
A. Ondas secundarias
​ . Contracciones tónicas
B
C. Ondas lentas
D. Glándulas subesofágicas
119.Producto de la alimentación, se producen diversas sustancias peptídicas, cininas y
bradicininas, las cuales permiten que:
A. Se produzca neovascularización en los territorios de las arterias abdominales
B. La acción de la lipasa pancreática se vea incrementada
C. El consumo de O2 del intestino aumente ligeramente
​D. El flujo sanguíneo intestinal aumente hasta 8 veces
120.El dolor periumbilical o epigástrico en el inicio de una apendicitis aguda se debe a:
A. Estímulo del nervio vago.
B. Íleo secundario.
C. Irritación del peritoneo parietal.
​D. Estímulo del sistema simpático.
121.El aumento en la actividad motora de la pared gástrica genera un aumento en los
niveles locales de qué sustancia en la microvasculatura:
​A.Adenosina
B. Colecistoquinina CCK
C. Endotelina
D. Gastrina
122.¿Cuál de los siguientes péptidos inhibe el vaciamiento gástrico?
​A. Colecist​oquinina
B. Péptido inhibidor gástrico
C. Motilina
D. Gastrina
123. Los músculos de la masticación que producen la retropulsión de la mandíbula son:
​A. temporales [mas seguro]
​B. maseteros
C. milohioideos
D. pterigoideos
124. En relación a la fisiología gástrica, marque lo correcto:
​A. la cimetidina actúa en la región basolateral de la célula parietal
B. la marea alcalina se debe al paso de bicarbonato través de la membrana apical de
la célula principal
C. el cloro difunde hacia el exterior por la la región basolateral de la célula parietal
D. la salida de hidrogeniones a la luz es por difusión facilitada
125. Durante el sueño, la concentración de bicarbonato en la saliva:
A. Se eleva a niveles mayores que los del plasma
B. Aumenta
C. No tiene efecto
​D. Disminuye
126. La secreción de saliva es importante en la fisiología digestiva. Su concentración
de potasio llega a ser menor que la del plasma cuando su secreción tiene un flujo:
A. Intermedio
​B. Nunca
C. Bajo
D. Alto
127. Respecto a las glándulas salivales, marque lo incorrecto:
A. la glándula parótida produce secreción serosa
B. la glándula sublingual drena a través de conducto de Wharton
C. La glándula parótida drena a través del conducto de Stenon
​D. la glándula sublingual tiene forma de garfio
128. ​Con respecto a la saliva, marque la respuesta correcta:
D. será hipertónica cuando el flujo es bajo
E. a mayor flujo, menor concentración de Na
F. a mayor flujo, menor concentración de cloro
​D. el sistema simpático estimula su secreción
CI 3
129. En relación a la circulación hepática, marque lo correcto:
​a) Los sinusoides hepáticos transportan sangre mixta
b) La vena porta proporciona el 50% de sangre al hígado
c) La vena porta se forma a partir de la vena esplénica y la mesentérica inferior
d) La arteria hepática deriva de la mesentérica superior
130. Dentro de las funciones de las células de Ito, marque lo incorrecto:
a) Sintetizan colágeno
b) Almacenan vitamina A
c) Se les llama células estrelladas
​d) Pueden fagocitar patógenos y actúan como presentadoras de antígeno
131.Paciente con tumor neuroendocrino productor de secretina, debido a lo cual se puede
esperar que su secreción pancreática, comparada con la de una persona sana de bajo flujo,
tenga una concentración de:
a) Sodio aumentada
b) Igual
​c) Bicarbonato aumentada
d) Potasio disminuida
132.El GALT se localiza en:
​a) Lámina propia
b) submucosa
c) borde en cepillo
d) superficie de criptas de Lieberkühn
133. En relación a la histología hepática, marque lo correcto:
a) la zona 1 se afecta rápidamente en estados de hipovolemia y shock
b) La zona 1 se encuentra cercana a la vena central lobulillar
​c) La zona 3 se encuentra más cerca a la vena central lobulillar
d) La zona 3 se encuentra más cerca al eje menor del acino hepático
134.El acino pancreático difere con el de las glándulas salivales en:
​ ) Contiene células centroacinares
a
b) No produce secreción serosa
c) El páncreas produce principalmente secreción mucosa
d) No tienen diferencias
135.Durante la digestión de las grasa, para que la lipasa actúe adecuadamente se requiere
que el pH aumento en la luz intestinal, lo cual es logrado, entre otros, por la secreción de las
células:
a) Del conducto interlobulillar
​ ) Centroacinares
b
c) Acinares
d) Alfa
136.La secreción de la colecistoquinina (CCK) se produce en la fase:
​ ) intestinal
a
b) En las 3 por igual
c) gástrica
d) Cefálica
137. ¿Por cuál de las siguientes células es secretada principalmente la pro
enzima procarboxipeptidasa?
​a) Acinares del páncreas
b) Epiteliales del duodeno
c) Ductales del páncreas
d) Centro Acinares del páncreas
138.Una mujer de 43 años dolor en hipocondrio derecho e icterica. En la ecografía se
evidencia cálculos biliares. Estos cálculos lo más probable es que se encuentren localizados
en:
​a) conducto colédoco
b) Conducto cístico
c) Vesícula biliar
d) Conducto pancreático secundario
139. ​Una mujer de 49 años ingresa en el hospital con dolor en epigástrico que migra hacia el lado
derecho y atrás hacia la escápula, sin ictericia. La ecografía muestra un gran cálculo biliar. ¿En cuál
de las siguientes estructuras es más probable que se localice el cálculo biliar?
→ bolsa de Hartmann

140. Una mujer de 38 años ingresa en el hospital con signos de colecistitis y cálculos biliares. Durante
la colangiografía, se inserta el catéter en la vesícula biliar con mucha dificultad. ¿Cuál de las
estructuras interfiere con más probabilidad con el paso del catéter por el conducto cístico?
→ válvula espiral (de Heister)

141. La internalización de proteínas o fragmentos de proteínas hacia el intracelular, es característico


de:
→ células M
→ se validó enterocitos

ECU 1:
Estudiante de 21 años sufre de gastritis aguda ocasionada por comer en lugares poco
higenicos. Suele consumir caramelos (“chupar”) mientras esta en base hasta la tarde.
También toma regular cantidad de leche (grasa, lactora, proteinas), pues le calma el dolor y
el ardor que sitnete por la gastritis ​(tiene dispepsia y cuando toma la leche se le pasa).
Incluso cuando puede, se toma dos vasos de agua frita y le calma la molestia. Ha decido ir
al medico para tratarse, pues ya no soporta el dolor, el cual esta seguro que los síntomas
se producen por elevada producion de HCl en el estomago, y por ello le ha recetado
ranitidina
139. El consumir caramelos eleva los niveles en sangre de una hormona cuya función es
la estimulación de las células.
- ​Beta del páncreas por GIP​ el cual es una incretina y por consiguiente estimula
las células pancreáticas
140. Consumir caramelos indirectamente actica la via:
-​ POMP/ CART saciedad
141. Consumo de leche produce indirectamente
- ​CCK​ inhibición del vaciamiento gástrico mayor tonicidad del esfínter pilórico
142.Cuando el px toma dos vasos de agua, genera indirectamente un aumento en la
liberación de:
- vaso de agua distención → ​g​ ​astrina​ → secreción de HCl
143.El uso de ranitidina bloquea el receptor H2 de la histamina en las células parietales, la
histamina llega a estas células por:
- histamina es una hormona paracrina por →​ difusión
**endocrina es por via hematógena y si fuera neuroendorina es por un NTs
144. Aumenta la secreción salival:
- ​noradrenalina​ a través de los receptores Beta 2
145.En este paciente con gastritis aguda debida a una alta producción de ácido clorhídrico,
sería lógico esperar que el píloro tenga un tono muscular:
- primero la​ secretina
- luego CCK
**ambas reguladores del HCl, Gatritis aguda debido a una alta producción de HCL piloro
estará aumentado (por la CCK)
146.Debido al uso de ranitidina, los valores de somatoestina en sangre:
- ranitidina disminuye acción de gastrina se quiere secretar mas no actúan
los inhibidores como la somatoestina somatoestina ​disminuye
147.El uso de atropina en este paciente:
- Inhibirá la acción de las prostaglandinas
- Aumentará la producción de ácido clorhídrico
- Disminuirá la acción del receptor CCK-B
​-Aumentará el pH del estómago
ECU 2:
Niño de sexo masculino de 2 años de edad, sufre de estreñimiento desde el nacimiento (1
deposición cada 3-4 días). Madre menciona que le estimula la defecación con un
termómetro rectal, y continuo uso de enemas y laxantes. Desde hace 6 meses comienza
con vómitos postprandiales. Los síntomas aumentan en frecuencia y magnitud y están en
relación con los episodios de estreñimiento. No refiere fiebre, tos, diarrea ni lesiones
cutáneas. Al examen físico presenta regular estado general, luce deshidratado. Abdomen
distendido, blando, depresible e indoloro. No se palpan masas abdominales. Se
permeabiliza el canal anal con termómetro rectal, encontrando cierta resistencia. Salida de
material fecal mal oliente en regular cantidad. Exámenes de laboratorio: hemograma
normal. Signos inflamatorios de fase aguda negativos. Alcalosis metabólica leve en sangre
venosa. Radiografía con enema baritado muestra recto y colon sigmoides dilatados
(megacolon). Biopsia profunda: ausencia de células ganglionares en la muestra enviada.
Se realiza cirugía correctiva.
148. ​Durante la fase esofágica de la deglución, para un bolo alimenticio determinado,
a medida que avanza el bolo la fuerza de la contracción se hace más:
- hiperpolarizado
- fuerte
- dependiente de Ach
- debil
149. Cuando este paciente ingiera sus alimentos, se espera que al momento de pasar el
bolo alimenticio por el esfínter esofágico superior, la presión intraesofágica disminuya en:
- la porción proximal al bolo
- el tercio medio del esófago
​-el cardias
- el lugar donde se contraiga la muscular propia
150. Al examinar la orofaringe del paciente, uno puede hallar fácilmente la amígdala
palatina, pues esta se encuentra inmediatamente detrás del músculo:
​-Palatogloso
- Palatofaringeo
- Hiogloso
- Elevador del velo del paladar
151. Con respecto a la defecación en este caso, marque la respuesta correcta:
- En posición de cuclillas, el músculo puborectal genera un ángulo más agudo en
el recto
- El sigmoides y el recto están inervados por el nervio vago
- La sensación de defecar sólo se da cuando el recto es ocupado por heces
y alcanzado el 80% de su capacidad
​-El esfínter anal comprometido tiene inervación autónoma
152. En este paciente [hirschsprung] se considera que está abolido el reflejo:
- Coloileal
​-Rectoesfinteriano
- Gastrocólico
- Relajación receptiva
153.No se espera que sea causa del vómito:
-​ Ayuno prolongado
- Estimulación faríngea y del glosofaríngeo
- Irritación de la mucosa gástrica
- Dolor intenso

ECU 3:
Paciente de 54 años con antecedentes de alcoholismo, gastritis crónica, tabaquismo
pesado, obesidad, cálculos biliares y cirrosis, es llevado a la emergencia por dolor
abdominal en epigastrio irradiado a la espalda y trastorno del sensorio.
Al examen físico: presión arterial 85/50 mmHg, frecuencia cardíaca 100 latidos/min,
frecuencia respiratoria 18 x minuto, temperatura axilar 36°C.
Conjuntivas pálidas, escleras ictéricas nevus arácnidos en tronco, distensión abdominal
marcada, cabeza de medusa, matidez desplazable en ambos flancos e hipogastrio, dolor a
la palpación de abdomen.
Tiempo de protrombina: 24 seg (testigo: 13 seg); TPT: 38 seg, glicemia: 165 mg/dL, uremia:
20 mg/dL, ASAT: 76 UI/L, ALAT: 22 UI/L, albumina: 2,5 g/dL, bilirrubina total: 2,6 mg/dL,
bilirrubina directa: 1,4 mg/dL, amilasa sérica 4000 U/L.
154. ​En esta paciente, al aumento de la amilasa sérica, se debe directamente a una lesión
de:
​a) páncreas
b) vesícula y árbol biliar
c) estómago
d) hígado
155. Considerando que el paciente sufre de gastritis, se puede decir que la secreción de
ácido por la mucosa gástrica
​a) involucra transporte activo de hidrogeniones
b) es realizada principalmente por células principales
c) es inhibida por antihistaminas tomadas por pacientes con rinitis alérgica
d) involucra la liberación de HCl de los gránulos zimógenos
156. El paciente tiene hemorragia digestiva alta por várices sangrantes como complicación.
Llegando a estar en shock hipovolémico por hemorragia masiva, se encontrara necrosis
hepática en:
a) zona 1
b) no se afectan los lobulillos hepáticos en hemorragia
​c) zona 3
d) zona 2
157. El misoprostol, análogo de las prostaglandinas está mejor indicado en:
c) cicatrización de úlcera péptica duodenal
d) erradica el helicobacter pylori
e) tratar el sind de Zollinger ellison
​d) prevenir daño por AINES
158. De las siguientes sustancias secretadas por los órganos de este paciente, la
más alcalina es la secreción:
- Esofágica
- Salival
- Yeyunal
​-Pancreática
159. En este paciente, se puede asumir que la pancreatitis ha sido ocasionada por
una disminución en el efecto de:
- Lipasa
- Enterocinasa
- Amilasa
​-Inhibidor de la tripsina
160. ¿Cuál de las siguientes sustancias es segregada por el páncreas?
​-Amilasa
-Pepsina
- Quimiotripsina
- Tripsina
161. Cada vez que este paciente toma alcohol, la acidificación de la luz del duodeno:
-​ Disminuye el vaciamiento gástrico
- Aumenta la contracción del esfínter de Oddi
- Aumenta la secreción del ácido gástrico
- Disminuye la secreción pancreática del bicarbonato

162. ​Un niño de 2 años es llevado a la consulta por diarrea persistente y edema de las extremidades, además
falta de crecimiento y desarrollo en relación a su edad. Los análisis de sangre revelan que tiene concentración
plasmática baja de proteínas (hipoproteinemia). Durante la endoscopía duodenal, se coloca colecistokinina
(CCK) endovenosa y se recoge muestras del líquido duodenal; el resultado del líquido confirma incapacidad para
hidrolizar proteínas a un pH neutro, esta situación mejora al añadir una pequeña cantidad de tripsina. El
paciente probablemente esté sufriendo la falta congénita de
………….
(Unidad 4, sesión 26, logro 2: Explicar la Digestión y absorción de las proteínas y sus alteraciones)
a. Pepsinógeno
b. PEPT-1
c. Carboxipeptidasas
d. Enterocinasa
163. ​Experimentalmente se incrementa la velocidad de la secreción salival con una sustancia, el análisis de
la composición de esta saliva obtenida se espera encontrar…………..
(Unidad 3, sesión 17, logro 5 : Explica la Influencia de la velocidad del flujo salival en la composición de la
saliva)
e. Elevación de concentración de bicarbonato, sodio y potasio
f. Elevación de concentración de cloro, sodio y potasio
g. Disminución de concentración de potasio
h. Disminución de concentración de potasio y bicarbonato
164. ​Paciente varón de 46 años soltero, consulta por odinofagia y bajo de peso, tiene antecedente de
tuberculosis desde hace 3 meses y es fumador crónico (10 cigarrillos por día); al evaluar la cavidad oral se
identifica lesión blanquecina en el dorso de la lengua y paladar blando, las lesiones se desprenden con el baja
lengua dejando una base eritematosa. Esta lesión corresponde probablemente a ……………………….…..
( Unidad 3, sesión18, logro 1-2 : Describe las enfermedades inflamatorias, infecciosas y proliferativas de la
cavidad oral)
i. Eritroplaquia
j. Candidiasis oral
k. Leucoplaquia vellosa
l. Fibroma en cavidad oral
165. Minero de 32 años de edad, que acude a centro de
salud por presentar de forma progresiva desde hace 1
año dificultad para ingerir alimentos sólidos y luego
líquidos; refiere regurgitaciones alimentarias y marcada
pérdida de peso (15 kilos). Radiografia baritada de
esófago como se muestra en la figura. El presente caso
se explica por……………….
(Unidad 2, sesión 12, logro 4: Identificar y
describir la función de los esfínteres esofágicos)
m. Contracción incompleta del esfínter esofágico inferior
n. Dificultad para el inicio de la deglución
o. Relajación incompleta del esfínter pilórico
p. Relajación incompleta del esfínter esofágico inferior
166. Paciente mujer de 35 años acude a consulta por sensación de sequedad y lesiones en cavidad oral. Al examen se observa
atrofia de la mucosa, fisuras y úlceras; nota además sequedad e irritación de la córnea y aumento del tamaño de las glándulas
parotídeas. Su diagnóstico más probable es artritis reumatoide; el hallazgo más probable en una biopsia de glándula parótida
es……..….
(Unidad 3, sesión 18, logro 3: Describe las enfermedades más frecuentes de las glándulas salivales)
q. Hiperplasia de acinos glandulares serosos
r. Gran infiltración de linfocitos y células plasmáticas
s. Gran infiltrado de linfocitos y macrófagos
t. Presencia de acinos normales con hiperplasia de células ductales

167. Un paciente con anemia acude con su médico quejándose de episodios frecuentes de gastroenteritis. Un análisis de sangre
revela anticuerpos circulantes dirigidos contra células parietales gástricas. Su anemia es atribuible a la hiposecreción de
………………………
(Unidad 3, sesión 20, logro 5: Gastritis crónica. Tipos de gastritis)
u. Factor intrínseco
v. Proteina R (haptocorrina)
w. Pepsinógeno
x. Ácido clorhídrico

168. ​Dos estudiantes deciden tomar un receso para comer una hamburguesa a la hora del almuerzo. Antes de llegar a la cafetería,
impulsos nerviosos provenientes del complejo vagal dorsal iniciarán la secreción de ácido gástrico por la liberación dedesde el
sistema nervioso entérico.
(Unidad 3, sesión 20, logro 2: Regulación de la secreción gástrica: estimulación, fases de la secreción)
y. Serotonina
z. Óxido nítrico
aa. GRP (péptido liberador de gastrina)
bb. Péptido intestinal vaso activo

169. Un niño de cuatro años de edad es llevado a la consulta por cuadros diarreicos frecuentes caracterizados por heces pálidas,
voluminosas y fétidas, presenta bajo peso y talla. Se mide la concentración de cloruro en el sudor y se encuentra que sus valores
son muy elevados. La alteración más importante a nivel de células ductales del páncreas tiene relación directa con la
conductancia de…………
(Unidad 3, sesión 23, logro 5 Explica la Secreción pancreática: formación del jugo pancreático, influencia de la velocidad de
flujo y regulación)
cc. Potasio
dd. Bicarbonato
ee. Sodio
ff. Cloro

170.Una mujer de 50 años de edad que sufrió durante varios años resequedad de los ojos debida a producción inadecuada de
lágrimas es enviada con un gastroenterólogo para evaluación de pirosis crónica. El examen endoscópico revela erosiones y tejido
cicatrizal en la parte distal del esófago justo por arriba del esfínter esofágico inferior. Las lesiones pueden atribuirse a la
disminución de uno de los siguientes componentes salivales:
(Unidad 3, sesión 17, logro 4: Explicar la Formación de la saliva y cuáles son sus componentes)
gg. Bicarbonato
hh. Lactoferrina
ii. Ig A
jj. Amilasa
171. Se evalúa los valores séricos de las siguientes sustancias a un paciente con enfermedad hepática terminal; en este paciente se
espera encontrar la combinación con la letra …………
(Unidad 3, sesión 22, logro 5: Describe las Pruebas de función hepática, la Insuficiencia hepática, encefalopatía hepática e
hipertensión portal)

Glucosa Amoniaco Albúmina


a. Aumenta Disminuida Disminuida
da
b. Disminui Aumentada Aumentada
da
c. Aumenta Aumentada Aumentada
da
d. Disminui Aumentada Disminuida
da
172. Una mujer de 35 años de edad HIV positiva, se presenta al médico con dolor abdominal en cuadrante superior derecho e
ictericia. La paciente refiere haber tenido múltiples episodios de ictericia durante los últimos 10 años. Los exámenes para
determinar hepatitis viral, dieron positivos para Hepatitis B, siendo catalogado el caso como hepatitis crónica con alteración
funcional. En un examen de sangre ¿cuál de los siguientes parámetros está disminuido?
(unidad 3, sesión 22, logro 5: Pruebas de función hepática, Insuficiencia hepática, encefalopatía hepática e hipertensión
portal)
kk. Fosfatasa alcalina
ll. Albumina
mm. Bilirrubina
nn. Tiempo de protrombina
173. En el reflejo peristáltico del intestino delgado, uno de los siguientes eventos sucede en la porción oral del bolo
alimenticio…………...
(Unidad 2, sesión 13, logro 4: Explicar la Motilidad del intestino delgado: Contracciones segmentarias y peristálticas)
oo. Disminución de 5 hidroxitriptamina desde las neuronas IPAN
pp. Contracción del músculo longitudinal
qq. Acción del péptido intestinal vasoactivo (VIP) en el músculo circular
rr. Acción de acetilcolina en el músculo circular

174. Experimentalmente se coloca una dosis alta de secretina en la luz intestinal duodenal; como consecuencia de esto, en el
jugo pancreático de la misma luz intestinal se observa la disminución de la concentración de …..………..
(Unidad 3, sesión 23, logro 5: Explica la Secreción pancreática: formación del jugo pancreático, influencia de la velocidad de
flujo y regulación)
ss. Na​+
tt. Cl​-
uu. K​+
vv. HCO3​-

175. Un varón de 58 años de edad con enfermedad de Crohn severo fue sometido a una resección ileal. Después de la cirugía
este paciente padecerá de esteatorrea, esto se explica porque …..………..
(unidad 4, sesión 26, logro 4: Explica las alteraciones en la Absorción de lípidos)
ww. El pool de ácidos biliares se incrementa
xx. Los quilomicrones no pueden formarse en el lumen intestinal
yy. La micelas no pueden formarse
zz. El páncreas no secreta lipasa

176. En un experimento se inserta un balón en el estómago de un voluntario, se infla poco a poco mientras que se vigilan las
presiones intraluminales. Aunque el volumen del balón aumenta considerablemente, las presiones permanecen constantes. Esta
relación volumen-presión se explica por la liberación local de …………..
(Unidad 2, sesión 13, logro 1 Explica la Motilidad gástrica: relajación receptiva)
aaa. Acetil colina y gastrina
bbb. Colecistoquinina y óxido nítrico
ccc. Óxido nítrico y péptido inhibidor vasoactivo
ddd. Norepinefrina y óxido nítrico
177. La toxina del Vibrio cholerae causa diarrea debido a…….
(Unidad 4, sesión 27, logro 6: Explica el transporte hidroelectrolítico intestinal, toxina colérica)
eee.La fosforilación del canal CFTR de los enterocitos de las vellosidades intestinales
fff. El Incremento de la secreción de cloro por las células de la cripta intestinal
ggg. La inhibición de la producción de AMPc por las células epitelailes
hhh. El incremento de la absorción de agua y sodio a través de las uniones estrechas

178. ¿Cuál de las siguientes alternativas es una característica de la secreción exocrina del páncreas?
(Unidad 3, sesión 23, logro 5: Secreción pancreática: formación del jugo pancreático, influencia de la velocidad de flujo y
regulación)
iii. Tiene una baja concentración de Cl​-​ respecto al plasma
jjj. Es estimulada por la presencia de bicarbonato en el duodeno
kkk. La secreción enzimática es estimulada principalmente por la gastrina
lll. Es hipotónica respecto al plasma

179. Una madre lleva a su hijo de dos años de edad a la sala de urgencias, estresada porque el niño deglutió una moneda de 10
céntimos mientras la familia cenaba en un restaurante. El médico observa mediante fluoroscopía que la moneda se halla en el
estómago y asegura a la madre que la moneda se eliminará con las heces. El médico recomienda utilizar la respuesta fisiológica
que permitirá la evacuación de la moneda del estómago al intestino ………….…..
(Unidad 2, sesión 13, logro 2: Explica la Motilidad gástrica: mezclado y vaciamiento)
mmm. Es por la relajación receptiva
nnn. Son los movimientos de mezcla y trituración
ooo. Es provocada por el ayuno
ppp. Es por la relajación del esfínter esofágico superior

180. Las estructuras en el hígado que permite que los productos metabólicos unidos a proteínas tengan acceso a las membranas
basolaterales de los hepatocitos, son…..
(Unidad 3, sesión 21, logro 4-5: Explica la Organización micro estructural del hígado)
qqq. Los Canalículos
rrr. Las fenestras sinusoidales
sss. Las uniones intercelulares herméticas
ttt. Las células de Ito

181. La composición de la bilis es modificada conforme fluye por los conductillos biliares. Durante este tránsito se espera que
aumente la concentración de…….
(Unidad 3, sesión 22, logro 2: Describe la Secreción biliar, visión general del sistema biliar extrahepático y composición de la
bilis)
uuu. Ig A
vvv. Glucosa
www. Monómeros de ácido biliar
xxx. Vitamina A

181. Se mide experimentalmente el contenido gástrico de dos personas. La persona “A” tiene alto contenido de grasa y la
persona “B” tiene un contenido hipertónico ¿Cuál de las siguientes es correcto respecto al vaciamiento gástrico? ​(Unidad
2, sesión 13, logro 2: Describe la Motilidad y vaciamiento gástrico)
yyy. Hay ralentización del vaciado gástrico solo en “A”
zzz. El vaciamiento gástrico es más rápido en ambos
aaaa. En ambos casos hay incremento de la motilina
bbbb. Hay ralentización del vaciado gástrico en ambos casos

182. El examen endoscópico de un paciente con hipertensión portal grave revela venas tortuosas que sobresalen hacia la luz del
esófago. El paciente recibe tratamiento quirúrgico mediante la colocación de una derivación que conecta la vena porta a la vena
cava. Después de la operación el riesgo de encefalopatía ………………….. y el riesgo de sangrado de várices ……………..
(Unidad 3, sesión 22, logro 5: Describe la Insuficiencia hepática, encefalopatía hepática e hipertensión portal)
cccc. Aumentará/disminuirá
dddd. Disminuirá/disminuirá
eeee. Aumentará/aumentará
ffff. Disminuirá/aumentará
183. Un paciente varón de 18 años de edad acude al médico para sus exámenes de rutina. Sus resultados de laboratorio
muestran un valor de bilirrubina sérica de 4 mg/dl y una bilirrubina directa de 0,3 mg/dl. Las pruebas de función hepática son
normales. La alteración que explica mejor este caso es por la deficiencia de ………………..
(Unidad 3, sesión 22, logro 3: Explica la Producción y excreción de bilirrubina. Tipos de bilirrubina e ictericia)
gggg. Transaminasas
hhhh. Glucuronil transferasa
iiii. Hemo oxigenasa
jjjj. La 7 alfa hidroxilasa

184. Un hombre de 57 años de edad es llevado a urgencias con hematemesis masiva rojo brillante, a su llegada se halla
inconciente con PA: 80/40 mm Hg y FC: 124 lat/min. Luce ictérico con presencia de “arañas vasculares en el tórax anterior y
extremidades”, abdomen distendido con signo de oleada positiva. Se encuentra esplenomegalia y pérdida de la masa muscular
en extremidades. La anastomosis vascular responsable del sangrado en este paciente es ………….…..
(Unidad 3, sesión 21, logro 2: Describe las anastomosis porto sistémicas)
kkkk. Arteria gástrica izquierda y vena ácigos
llll. Vena gástrica izquierda y vena ácigos
mmmm. Vena paraumbilical y vena epigástrica inferior
nnnn. Vena gástrica izquierda y vena esofágica superior

185. Un estudiante de medicina está comiendo un plato de comida a base de champiñones, espárrago y salsa de soya. El sabor
umami contenido en todos estos alimentos actúa a nivel de los botones gustativos estimulando ………………..
(Unidad 2, sesión 10, logro 5: Describe los tipos y mecanismos moleculares para la detección de los sabores)
oooo. El ingreso de sodio
pppp. Un receptor acoplado a proteína G
qqqq. Su receptor específico T1R3
rrrr. El ingreso de hidrógeno

186. Un hombre de 22 años de edad se presenta al médico con una historia de 1 año de evolución caracterizado por dolor
recurrente en fosa iliaca derecha y diarrea. Manifiesta además pérdida de peso de 8 kg durante este periodo. La
colonoscopía revela múltiples lesiones en el ileon terminal y colon. La biopsia de estas lesiones revela engrosamiento,
inflamación y ulceración de la mucosa. El diagnóstico más probable en este caso es…….
(Unidad 4, sesión 28, logro 5: Describe la Enfermedad inflamatoria intestinal. Generalidades, morfología y características)
ssss. Sprue celiaco
tttt. Enfermedad de Crohn
uuuu. Sindrome de colon irritable
vvvv. Colitis ulcerativa

187. Una de las funciones del músculo señalado es:


(Unidad 2, sesión 8, logro 3: Describir el Piso de la boca: estructuras
blandas que la conforman)
wwww. Eleva el paladar blando
xxxx. Recibe inervación del nervio maxilar
yyyy. Deprime el hioides cuando la mandíbula está fija
zzzz. Deprime la mandíbula cuando el hioides está fijo

188. Varón de 61 años que consulta por dolor retro esternal intenso desde hace 6 horas y después de vómitos intensos y
repetidos; al examen se observa disnea, cianosis, hipotensión y signos clínicos de shock. La radiografía simple de tórax
muestra neumomediastino. El líquido en el espacio pleural aspirado tiene alta concentración de amilasa. ¿Cuál de las
siguientes alternativas puede explicar este cuadro clínico?
(Unidad 3, sesión 18, logro 6: Describe algunas Enfermedades del esófago)
aaaaa. Sindrome de Mallory Weiss
bbbbb. Rotura espontánea de esófago
ccccc. Neumotórax por probable herida penetrante
ddddd. Perforación de ulcera gástrica de cara posterior, con complicación torácica
189. La secreción del ácido en la célula parietal gástrica se lleva a cabo por una ATPasa especifica que intercambia hidrogeniones
(H+) del citosol por…..
(Unidad 3, sesión 20, logro 1: Explica la Secreción del HCl y sustancias que la alteran)
eeeee. Cl-
fffff.HCO3-
ggggg. Na +
hhhhh. K+

190. ​En condiciones normales el ingreso de 600 ml de líquido es el estómago provoca un aumento de presión intragástrica de
unos 12 cm de H2O. Después de una vagotomía (corte del nervio vago) es de esperar que el ingreso del mismo volumen de
líquido provoque lo siguiente: …………………………………
(Unidad 2, sesión 13, logro 1: Describe la Motilidad gástrica: relajación receptiva)
iiiii. Un aumento igual de la presión
jjjjj. Que no aumente la presión
kkkkk. Un aumento mayor de la presión
lllll. Una disminución de la presión

191.Una paciente de 30 años de edad es sometida a una cirugía en oído medio derecho por un problema de otoesclerosis. Luego
de la cirugía refiere alteración en la percepción de sabores. Al evaluar el caso usted esperaría encontrar……….
(Unidad 2, sesión 10, logro 5: Describe la Irrigación e inervación de la lengua)
mmmmm. Alteración en la sensación del dolor y temperatura en el tercio posterior de la lengua
nnnnn. Alteración en la sensación del gusto en los dos tercios anteriores de la lengua
ooooo. Alteración en la sensación del gusto en la punta de la lengua
ppppp. Sensación del dolor, tacto y temperatura conservada en toda la lengua

192. ¿Cuál de las siguientes alterativas es correcta?


(Unidad 4, sesión 26 : Explica la digestión y absorción de nutrientes y sus alteraciones)
qqqqq. En el borde luminal, en cepillo, del intestino delgado, la absorción de sodio únicamente se realiza asociada a
la de glucosa.
rrrrr. El lugar principal para la absorción del hierro es el ileon
sssss. Las sales biliares desconjugadas son absorbidas preferentemente en el colon
ttttt. El proceso de digestión y absorción de la vitamina B12 no se altera en insuficiencia pancreática.

193. En un paciente de 45 años de edad con colestasis biliar, se encuentra una elevación de los niveles sanguíneos de fosfatasa
alcalina hasta 3 veces la cifra normal. ¿Cuál de las siguientes alternativas estará también elevada como evidencia del daño de la
vía biliar?
(Unidad 3, sesión 22, logro 5: Pruebas de función hepática, Insuficiencia hepática, encefalopatía hepática e
hipertensión portal)
uuuuu. Tiempo de protrombina y albúmina sérica
vvvvv. Transaminasas hepáticas (ALT y AST)
wwwww. Glucoronil transferasa
xxxxx. Gamma glutamil transpeptidasa

194. Revisando la angiografía de un hombre de 70 años en estudio por aneurisma de aorta abdominal el radiólogo informa de la
presencia de una oclusión completa de la arteria mesentérica inferior. El paciente se encuentra completamente asintomático.
¿Cuál de las siguientes arterias se anastomosa a la sistema arterial de la mesentérica inferior?
(Unidad 4, sesión 25, logro 1: Identifica la Arteria mesentérica superior e inferior, ramas y anastomosis)
yyyyy. Ileal
zzzzz. Cólica media
aaaaaa. Sigmoideas
bbbbbb. Cólica izquierda

195. ​Lactante de 3 meses de vida es atendido por presentar diarrea, se administra una solución de glucosa y electrólitos por vía
oral. La proteína de membrana apical que explica la capacidad de esta solución para proporcionar aporte de glucosa e
hidratación es ………..
(Unidad 4, sesión 26, logro 1: Explica la Digestión y Absorción de los hidratos de carbono. Alteraciones)
cccccc. GLUT-5
dddddd. SGLT-1
eeeeee. CFTR
ffffff.GLUT-2
196. Paciente ha sufrido herida de bala en el abdomen, se le ha tenido que extirpar el segmento medio y distal del ileon. En este
caso la síntesis hepática de sales biliares estará …..…..
(Unidad 3, sesión 22, logro 4: Explica la formación, función y Circulación entero hepática de lasa sales biliares)
gggggg. Disminuida por inhibición de la colesterol 7 alfa hidroxilasa
hhhhhh. Incrementada por estímulo de la enzima colesterol 7 alfa hidroxilasa
iiiiii. Incrementada por inhibición de la colesterol 7 alfa hidroxilasa
jjjjjj. Sin cambios en el ritmo de síntesis

197. ​Un varón de 75 años ingresa al consultorio por presentar ictericia marcada de piel y las escleras. El estudio del paciente
mostró que presentaba un tumor que obstruía la totalidad del conducto hepático común. ¿Cuál de las siguientes estructuras se
encontrará dilatada en este paciente?
(Unidad 3, sesión 21, logro 6: Describir el árbol biliar intrahepático)
kkkkkk. Conducto de Wirsung
llllll. Conductos de Hering
mmmmmm. Conducto colédoco
nnnnnn. Conducto cístico

198.Correlaciones las dos columnas y marque la fórmula correcta:


(Unidad 4, sesión 28, logro 1: Diarrea: definición, mecanismos: osmótica, secretoria y exudativa)
1. Enfermedad Hirschsprung(4) heces con moco y sangre
2. Diarrea osmótica(2) intolerancia a lactosa
3. Diarrea secretoria(1 ) aganglionosis congénita
4. Diarrea exudativa(3) canales de Cl- en las células de la cripta

a.- 4231 b.- 1234 c.- 2143 ​d.- 4213


199. Respecto a la siguiente imagen que representa una estructura de la mucosa gástrica, la estructura con número ………..
produce ……………………..
(Unidad 3, sesión 19, logro 4: La glándula fúndica. Funciones y tipos de células con sus características)
oooooo. 3 / pepsina
pppppp. 1 / Pepsinógeno
qqqqqq. 4 / HCl y factor extrínseco
rrrrrr. 2 / pepsinógeno

200. En un paciente con insuficiencia renal crónica, el déficit en la absorción de calcio a nivel del enterocito se debe a lo
siguiente:
(Unidad 4, sesión 26, logro 6: Explica la Absorción de calcio y hierro)
ssssss. No se convierte la 25 hidroxicolecalciferol a 1,25 dihidroxicolecalciferol
tttttt. No se convierte la 1,25 dihidroxicolecalciferol a 25 hidroxicolecalciferol
uuuuuu. Se incrementa la producción de Calbindina
vvvvvv. Existe un descenso de la alfa 25 hidroxilasa renal
201. Varón de 30 años es traído a emergencia por agresión abdominal con arma de fuego (pistola) y es sometido a
laparotomía exploratoria, observándose isquemia del colon ascendente y parte del colon trasverso ¿la lesión de cuál de las
siguientes arterias explicaría esta isquemia?
(unidad 1, sesión 2, logro 6: (D​escribe la irrigación visceral: arterias de tronco celiaco, arteria mesentérica
superior e inferior​, topografía de superficie, órganos por cuadrante)

a. Celiaca
b. Colónica derecha
c. Mesentérica inferior
d. Mesentérica superior
202. Respecto a las sustancias gastrointestinales que regulan la secreción pancreática; marque la afirmación correcta:
(unidad 1, sesión 3, logros 2 y 3: describir las hormonas gastrointestinales: estímulos y funciones)

a. La Secretina, es la hormona más importante para la secreción de bicarbonato por las células acinares del
páncreas
b. La acetilcolina es capaz de estimular la secreción enzimática y de bicarbonato del páncreas
c. La gastrina, es la hormona más importante para la secreción de enzimas pancreáticas
d. La colecistoquinina (CCK) estimula al páncreas solo para secreción enzimática

203. Ante una lesión del X par craneal, ¿cuál de los siguientes músculos mantiene conservada su función?:
(unidad 2, sesión 08, logro 4: Paladar blando: componentes musculares)

e. Elevador del velo del paladar


f. Tensor del velo del paladar
g. Palatofaríngeo
h. Glosofaríngeo
204. Experimentalmente se utiliza atropina (anticolinérgico) para inhibir la secreción de gastrina, sin embargo la secreción
de esta hormona se sigue dando ante estímulos vagales. Esta situación se explica porque la atropina:
(unidad 1, sesión 3, logro 3 : describir las hormonas gastrointestinales: estímulo y funciones de la gastrina y
colecistoquinina)

a. Bloquea parcialmente la bomba de protones en la célula G


b. Inhibe la acción de acetilcolina e histamina en la célula G
c. Solo inhibe la acción del péptido GRP en la célula G
d. No bloquea la acción del péptido GRP
205. ​Un varón de 50 años es sometido a extirpación del duodeno y parte proximal del yeyuno. Esta situación
ocasionaría la pérdida de las células ……….. , productoras deque estimula la secreción de
bicarbonato por el páncreas.
(unidad 1, sesión 3, logro 3: describir las hormonas gastrointestianles: estímulos y funciones de la secretina y
péptido insulinotrópico dependiente de glucosa)

a. “S” / secretina
b. Parietales / secretina
c. “I” / colecistoquinina
d. “S” / colecistoquinina

206.Recién nacido que presenta tumoración abdominal a nivel del cordón umbilical (fotografía). ¿cuál de las siguientes
afirmaciones es correcta respecto a este defecto en el desarrollo embriológico del intestino?: ​(unidad 1, sesión 5, logro 2:
identificar las anomalías del desarrollo del intestino medio)

a. Corresponde a una Gastrosquisis


b. Las vísceras se hallan cubiertas por piel
c. No está asociado a otras malformaciones
d. Se asocia a malformaciones cardiacas y
del tubo neural

207. Varón de 35 años acude a la emergencia por trauma abdominal y se decida


realizar una laparoscopía exploratoria. El cirujano observa la disposición de los
órganos abdominales como se representa en el siguiente esquema. Esta
disposición de órganos se explica por la rotación(SMA=arteria mesentérica
superior)
(unidad 1, sesión 5, logro 3: identificar las anomalías del desarrollo del
intestino medio: defectos de rotación, estenosis y atresias)

a. anti horaria del intestino medio, en sólo 90°


b. incompleta del intestino medio (270°)
c. horaria del intestino medio
d. horaria del estómago

208. Se evalúa la expresión de la proteína Agrp en una persona con alteración del apetito; lo correcto respecto a esta
proteína es…..
(unidad 1, sesión 3, logro 4: E​ xplica los mecanismos de control del apetito y saciedad ​)

a. Esta proteína es un potente anorexigénico


b. La mutación del gen que la codifica produce adelgazamiento
c. La sobre producción de la proteína lleva a obesidad por agonismo de receptores MC3 y MC4
d. La sobre producción de la proteína disminuye el apetito por antagonismo de receptores MC4

209. Juana cae de la bicicleta y se fractura la región anterior del hueso maxilar superior con compromiso de la fosa incisiva.
Al examen físico de la región esperaría encontrar alteración en la sensibilidad de la encía …………………
(unidad 2, sesión 8, logro5: paladar: paladar duro y blando: irrigación e inervación)

a. bucal posterior
b. Lingual anterior
c. palatina anterior
d. palatina posterior
210. Recién nacido es atendido por el neonatólogo y luego entregado a su madre para dar de lactar; la madre al dar de lactar
observa coloración azulada de labios, acompañado de tos persistente, dificultad respiratoria y distención abdominal. Se le intenta
colocar una sonda nasogástrica pero esta retorna a la cavidad oral en todos los intentos. ¿Cuál de las siguientes anomalías del
desarrollo es el más probable en este caso? ​(unidad 1, sesión 4, logro 3: identificar las anomalías en el desarrollo del esófago:
atresia y/o fístula traqueo esofágica)

a. Estenosis esofágica proximal con Fístula traqueo esofágica distal


b. Atresia esofágica proximal con fístula traqueoesofágica distal
c. Atresia esofágica distal con fístula traqueoesofágica proximal
d. Fístula traqueoesofágica proximal y distal

211. ¿Cuál de los siguientes mecanismos ocurre durante la defecación?


(unidad 2, sesión 13, logro 6: motilidad del intestino grueso: contracciones segmentarias, movimientos en
masa, defecación y reflejo gastrocólico)

i. Contracción refleja del esfínter anal interno


j. En la posición de “cuclillas” el músculo puborectal se halla relajado
k. Relajación del esfínter anal externo por efectos del VIP y óxido nítrico
l. La materia fecal en el recto estimula la contracción del sigmoides por los nervios pudendos

212. La estructura número 4 (gráfico) corresponde a


……….… y está ………..
(unidad 2, sesión 9, logro 2: Partes de un diente.
Capas del diente: Esmalte: características y células
que lo producen)

m. el cemento / mineralizado en 90%


n. la dentina / formada por ameloblastos
o. el esmalte / formado por células derivadas
del mesénquima
p. la dentina / formado por células derivadas de la
cresta neural

213. Un paciente luego de un accidente sufre lesión del piso de la boca, se constata daño del nervio “cuerda del
tímpano”, en este caso se esperaría encontrar disminución de lade la lengua
(unidad 2, sesión 10, logro 3: Irrigación e inervación de la lengua)

a. Motilidad en los dos tercios anteriores


b. Sensación del gusto en el tercio posterior
c. Sensación del gusto en los dos tercios anteriores
d. Sensibilidad al tacto en los dos tercios anteriores

214. ¿Cuál de las siguientes afirmaciones es la correcta sobre la gastrina?


(unidad 1, sesión 3, logro 1: reconocer las características de las sustancias reguladoras gastrointestinales:
hormonas, sustancias paracrinas y neurocrinas)

q. Produce atrofia de la mucosa gástrica


r. Es producida por la célula G del cuerpo gástrico
s. Es estimulada por la distensión gástrica y el Ph bajo
t. Actúa en la célula diana mediante su receptor CCk tipo B
215. Al recibir un paciente con signos de hipovolemia y antecedente de trauma en abdomen por accidente de tránsito, usted
identifica radiológicamente: lesión de primera vértebra lumbar y signos de lesión en páncreas; durante la cirugía se observó pobre
irrigación de asas intestinales. El vaso afectado es la arteria ……..
(unidad 1, sesión 1, logro 6: reconocer las estructuras a nivel de L1, nivel de los principales vasos sanguíneos)

a. esplénica
b. hepática común
c. mesentérica inferior
d. mesentérica superior

216. Un paciente sufre de daño a nivel del cuello con lesión muscular en la región de la faringe. En el examen físico se
determina dificultad para la elevación de la faringe y para el cierre del itsmo de las fauces. En este caso, probablemente esté
afectado el músculo:
(unidad 2, sesión 11, logro 2: Músculos de la faringe: identificación, constrictores y longitudinales)

a. palatogloso
b. estilofarinfeo
c. palatofaringeo
d. constrictor inferior

217. Varón de 50 años a quien le realizan la curación de la segunda molar de la arcada superior derecha. En un
momento determinado, el paciente acusa de intenso dolor de la pieza dentaria en tratamiento. La vía aferente del dolor
viaja a través del nervio …………
(unidad 2, sesión 9, logro 6: Inervación de los dientes)

a. trigémino V2
b. trigémino V3
c. naso palatino
d. palatino menor

218. La distención gástrica por los alimentos produce incremento de secreción de HCl mediante la producción de
………….. que estimula a las células ……………. vía proteína ………..
(Unidad 1, sesión 3, logro 2: Describe las hormonas gastrointestinales: Estímulo y funciones de la gastrina y
colecistoquinina)

u. gastrina / parietal / Gq
v. gastrina / principal / Gs
w. acetilcolina / parietal /Gi
x. acetilcolina / principal / Gi

218. Un niño de tres años llega a emergencia con disfagia (dificultad para tragar), dolor retro esternal, salivación y llanto. Se
sospecha de ingesta de cuerpo extraño (moneda) en el esófago; al ser evaluado se constata en una radiografía presencia de
cuerpo extraño a nivel de C6 (6° vértebra cervical). El cuerpo extraño estará suspendido a nivel del estrechamiento
producido por………..
(unidad 2, sesión 11, logro4: Esófago, características anatómicas, relación con órganos vecinos y estrecheces)

a. el cayado aórtico
b. el hiato esofágico
c. el músculo cricofaríngeo
d. el bronquio principal izquierdo

219. La triada portal (arteria hepática, vena portal y conducto biliar común) está contenida en el ligamento
…….……… y derivan embriológicamente del ……
(Unidad 1, sesión 1, logro 4: Identifica el peritoneo, mesenterio, omento y ligamentos, retroperitoneo.)

y. hepato duodenal / mesenterio ventral


z. gastro esplénico / mesenterio dorsal
aa. hepato gástrico / omento menor
bb. falciforme / omento menor

220. En relación al movimiento de peristaltismo del tubo digestivo:


en la flecha negra del gráfico se produce la liberación de
……………… a nivel del músculo ………..
(unidad 2, sesión 7, logro 6: Control hormonal y tipos de
movimiento)

a. noradrenalina, sustancia P y neuropéptido “ Y” / circular


b. acetilcolina y sustancia P / longitudinal
c. óxido nítrico y PIV / longitudinal
d. óxido nítrico y PIV / circular

221. Un paciente refiere no percibir algunos sabores, al examen físico se constata alteración en la percepción de sabores y
del dolor en el tercio posterior de la lengua ¿Cuál de los siguientes nervios estará alterada en su función?
(unidad 2, sesión 10, logro 5: Sabores, tipos y mecanismos moleculares para su detección)

a. Lingual (rama del V par)


b. Cuerda del tímpano (VII par)
c. Glosofaríngeo (IX par)
d. Hipogloso (XII par)

222. El gráfico detalla la estructura de la pared del tubo digestivo


intestinal ¿Cuál de las siguientes asociaciones es correcta?
(unidad 2, sesión 7, logro 1: La pared y músculo liso
gastrointestinal )

a. “1” – peristaltismo
b. “2” – secreción enzimática
c. “3” – deriva del mesodermo
d. “4” – doble hoja de tejido graso
223. En el caso de un paciente con gastrinoma (tumor productor de gastrina), la presencia de úlceras duodenales y erosión
de la mucosa gástrica, se debe principalmente a…….
(unidad 1, sesión 3, logro 2: describir las hormonas gastrointestinales: estímulo y funciones de la gastrina y
colecistoquinina)

a. la acción directa de la gastrina sobre la célula principal


b. la sobre expresión de los receptores “G” en la célula parietal
c. el exceso de HCl por estímulo de receptores CCK-B en la célula parietal
d. el exceso de HCl por estímulo directo de receptores de acetilcolina en la célula parietal

224. El reflejo entero gástrico se caracterizan por:


(unidad 2, sesión 13, logro 6: Motilidad del intestino grueso: contracciones segmentarias, movimientos en
masa defecación y reflejo gastrocólico)

cc. favorecer la motilidad gástrica gracias a la CCk


dd. inhibir la motilidad gástrica y estimular la secreción ácida
ee. movilizar grandes volúmenes desde el estómago al duodeno
ff. originarse debido a la distensión duodenal y presencia del quimo ácido
225. Mauricio tiene dificultad para deprimir el paladar y elevar la parte posterior de la lengua. En este caso estará afectado un
músculo ………………., específicamente el músculo …………….
(Unidad 2, sesión 10, logro 2: Músculos de la lengua: clasificación, identificación y sus funciones)

a. intrínseco – longitudinal inferior


b. extrínseco – palatogloso
c. extrínseco – transverso
d. extrínseco – estilogloso

226. Una de las funciones del músculo señalado es:


(Unidad 2, sesión 8, logro 3: Describir el Piso de la boca:
estructuras blandas que la conforman)

gg. deprimir la lengua


hh. elevar el paladar blando
ii. deprimir el hioides cuando la mandíbula está fija
jj. deprimir la mandíbula cuando el hioides está fijo

227. Paciente varón de 30 años es evaluado por probable enfermedad de Chagas, cursa con problemas de motilidad del
colon; los estudios de biopsia determinan ausencia de células ganglionares. Según el
gráfico
¿cuál es la capa en la que se determina la ausencia de dichas células?
(unidad 1, sesión 2, logro 1: describir las generalidades de la estructura del
tubo digestivo: esófago, estómago intestino delgado y grueso)

a. Mucosa - 1
b. Muscular propia – 1
c. Muscular de la mucosa - 2
d. Muscular propia - 3

228. ​Paciente varón de 32 años, que acude a centro de salud por presentar de forma progresiva desde hace 1 año dificultad
para ingerir alimentos sólidos y luego líquidos; refiere regurgitaciones alimentarias y marcada pérdida de peso (15 kilos).
Radiografía baritada (sustancia de contraste​) ​de esófago se muestra en la figura. El presente caso se explica
por……………….
(Unidad 2, sesión 12, logro 4: Identificar y describir la función de los esfínteres esofágicos)

a. aumento de la peristalsis esofágica


b. relajación incompleta del esfínter pilórico
c. relajación incompleta del esfínter esofágico inferior
d. perdida de producción de PIV y óxido nítrico en el esfínter
esofágico superior
229. En condiciones normales, el ingreso de 600 ml de líquido es el estómago provoca un aumento de presión
intragástrica de unos 12 cm de H​2​O. Después de una vagotomía (corte del nervio vago) es de esperar que el ingreso del
mismo volumen de líquido ocasionede la presión intragástrica.
(Unidad 2, sesión 13, logro 1: Describe la Motilidad gástrica: relajación receptiva)

a. la disminución
b. la no variación
c. un aumento mayor
d. un aumento similar o igual

230. Un niño de 2 años es llevado a la consulta por diarrea persistente, edema de las extremidades y falta de crecimiento en
relación a su edad. Los análisis de sangre revelan que tiene concentración plasmática baja de proteínas (hipoproteinemia).
Como parte del estudio se coloca colecistokinina (CCK) endovenosa y se recoge muestras del líquido duodenal por
endoscopía; el resultado del líquido confirma incapacidad para hidrolizar proteínas a un pH neutro, esta situación mejora al
añadir una pequeña cantidad de tripsina. El paciente probablemente esté sufriendo la falta congénita de ………….
(Unidad 4, sesión 26, logro 2: Explicar la Digestión y absorción de las proteínas y sus alteraciones)
a. PEPT-1
b. pepsinógeno
c. enterocinasa
d. carboxipeptidasas
231. Paciente mujer de 35 años acude a consulta por sensación de sequedad y lesiones en cavidad oral. Al examen se
observa atrofia de la mucosa, fisuras y úlceras; nota además sequedad e irritación de la córnea y aumento del tamaño de
las glándulas parotídas. Su diagnóstico más probable es artritis reumatoide; el hallazgo más probable en una biopsia de
glándula parótida es……..….
(Unidad 3, sesión 18, logro 3: Describe las enfermedades más frecuentes de las glándulas salivales)
e. Presencia de acinos normales con hiperplasia de células ductales
f. Gran infiltración de linfocitos y células plasmáticas
g. Hiperplasia de acinos glandulares serosos
h. Gran infiltrado de linfocitos y neutrófilos
232. Un hombre de 42 años de edad se presenta al médico con una historia de 1 año de evolución, caracterizado por dolor
abdominal bajo y diarreas con crisis sanguinolentas. Manifiesta además pérdida de peso de 8 kg durante este periodo. La
colonoscopía revela lesión difusa en el colon con afectación del recto. La biopsia de estas lesiones revela adelgazamiento de
la pared, inflamación y ulceración de la mucosa y sub mucosa. El diagnóstico más probable en este caso es:
(Unidad 4, sesión 28, logro 5: Describe la Enfermedad inflamatoria intestinal. Generalidades, morfología y
características)
i. sindrome de colon irritable
j. enfermedad de Crohn
k. colitis ulcerativa
l. sprue celiaco
233. ​Dos estudiantes deciden tomar un receso para comer una hamburguesa a la hora del almuerzo. Antes de llegar a la
cafetería, impulsos nerviosos provenientes del complejo vagal dorsal iniciarán la secreción de ácido gástrico por la liberación
dedesde el sistema nervioso entérico.
(Unidad 3, sesión 20, logro 2: Regulación de la secreción gástrica: estimulación, fases de la secreción)
m. Serotonina
n. Colecistoquinina
o. Péptido inhibidor vaso activo
p. GRP (péptido liberador de gastrina)

234. Un niño de cuatro años de edad es llevado a la consulta por cuadros diarreicos frecuentes caracterizados por
heces pálidas, voluminosas y fétidas; al examen físico presenta bajo peso y talla para la edad. Se mide la concentración
de cloruro en el sudor y se encuentra que sus valores son muy elevados. La alteración más importante a nivel de
células ductales del páncreas tiene relación directa con la conductancia de………… ​(Unidad 3, sesión 23, logro 5
Explica la Secreción pancreática: formación del jugo pancreático, influencia de la velocidad de flujo y regulación)
q. Bicarbonato
r. Potasio
s. Sodio
t. Cloro
235. Se evalúa los valores séricos de las siguientes sustancias a un paciente con enfermedad hepática terminal; en este
paciente se espera encontrar la combinación con la letra …………
(Unidad 3, sesión 22, logro 5: Describe las Pruebas de función hepática, la Insuficiencia hepática, encefalopatía
hepática e hipertensión portal)
Glucosa Amoniaco Albúmin
a
a. Aumenta Disminuida Disminui
da da
b. Disminui Aumentada Aumenta
da da
c. Aumenta Aumentada Aumenta
da da
d. Disminui Aumentada Disminui
da da

236. Una mujer de 35 años de edad HIV positiva, se presenta al médico con dolor abdominal en cuadrante superior
derecho e ictericia. La paciente refiere haber tenido múltiples episodios de ictericia durante los últimos 10 años. Los
exámenes para determinar hepatitis viral, dieron positivos para Hepatitis B, siendo catalogado el caso como hepatitis
crónica con alteración funcional. En un examen de sangre ¿cuál de los siguientes parámetros está disminuido?
(unidad 3, sesión 22, logro 5: Pruebas de función hepática, Insuficiencia hepática, encefalopatía hepática e
hipertensión portal)
u. Albumina
v. Bilirrubina
w. Fosfatasa alcalina
x. Tiempo de protrombina

237. En el reflejo peristáltico del intestino delgado ¿Cuál de los siguientes eventos sucede en la porción caudal del bolo
alimenticio?
(Unidad 2, sesión 13, logro 4: Explicar la Motilidad del intestino delgado: Contracciones segmentarias y peristálticas)
y. Acción del péptido inhibidor vasoactivo (VIP) en el músculo circular
z. Acción del NO (óxido nítrico) en el músculo longitudinal
aa. Contracción del músculo longitudinal interno
bb. Acción de acetilcolina en el músculo circular

238. Un varón de 58 años de edad con enfermedad de Crohn severo fue sometido a una resección ileal. Después de la
cirugía este paciente padecerá de esteatorrea, esto se explica porque …..………..
(unidad 4, sesión 26, logro 4: Explica las alteraciones en la Absorción de lípidos)
cc. se inhibe la acción de la 7 alfa hidroxilasa
dd. el pool de ácidos biliares se incrementa
ee. hay mala absorción de ácidos biliares
ff. el páncreas no secreta lipasa

239. En un experimento se inserta un balón en el estómago de un voluntario, se infla poco a poco mientras que se
vigilan las presiones intraluminales. Aunque el volumen del balón aumenta considerablemente, las presiones
permanecen constantes. Esta relación volumen-presión se explica por la liberación local de …………..
(Unidad 2, sesión 13, logro 1 Explica la Motilidad gástrica: relajación receptiva)
gg. acetil colina y gastrina
hh. norepinefrina y óxido nítrico
ii. colecistoquinina y óxido nítrico
jj. óxido nítrico y péptido inhibidor vasoactivo

240. ¿Cuál de las siguientes alternativas es una característica de la secreción exocrina del páncreas?
(Unidad 3, sesión 23, logro 5: Secreción pancreática: formación del jugo pancreático, influencia de la velocidad de
flujo y regulación)
kk. Es hipotónica respecto al plasma
ll. Su mayor estímulo se da en la fase intestinal
mm. Es estimulada por la presencia de bicarbonato en el duodeno
nn. La secreción enzimática es estimulada principalmente por la secretina
241. Las estructuras en el hígado que permite que los productos metabólicos unidos a proteínas tengan acceso a las
membranas basolaterales de los hepatocitos, son…..
(Unidad 3, sesión 21, logro 4-5: Explica la Organización micro estructural del hígado)
oo. los canalículos
pp. las células de Ito
qq. las fenestras sinusoidales
rr. las uniones intercelulares herméticas

242. La composición de la bilis es modificada conforme fluye por los conductillos biliares. Durante este tránsito se
espera que aumente la concentración de……. ​(Unidad 3, sesión 22, logro 2: Describe la Secreción biliar, visión general
del sistema biliar extrahepático y composición de la bilis)
ss. Ig A
tt. Glucosa
uu. Protones
vv. Vitamina A

243. Se mide experimentalmente el contenido gástrico de dos personas. La persona “A” tiene alto contenido de grasa y la
persona “B” tiene un contenido isotónico ¿Cuál de las siguientes es correcto respecto al vaciamiento gástrico? ​(Unidad 2,
sesión 13, logro 2: Describe la Motilidad y vaciamiento gástrico)
ww. Hay ralentización del vaciado gástrico solo en “A”
xx. El vaciamiento gástrico es más rápido en ambos
yy. Hay ralentización del vaciado gástrico solo en “B”
zz. Hay ralentización del vaciado gástrico en ambos casos

244. El examen endoscópico de un paciente con hipertensión portal grave revela venas tortuosas que sobresalen hacia la luz
del esófago. El paciente recibe tratamiento quirúrgico mediante la colocación de una derivación que conecta la vena porta a
la vena cava. Después de la operación el riesgo de encefalopatíay el riesgo
de sangrado de várices ……………..​(Unidad 3, sesión 22, logro 5: Describe la Insuficiencia hepática, encefalopatía hepática e
hipertensión portal)
aaa. disminuirá / disminuirá
bbb. disminuirá / aumentará
ccc. aumentará / disminuirá
ddd. aumentará / aumentará

245. Un paciente varón de 18 años de edad acude al médico para sus exámenes de rutina. Sus resultados de laboratorio
muestran un valor de bilirrubina sérica de 4 mg/dl y una bilirrubina directa de 0,3 mg/dl. Las pruebas de función hepática
son normales. La alteración que explica mejor este caso es por la deficiencia de ………………..
(Unidad 3, sesión 22, logro 3: Explica la Producción y excreción de bilirrubina. Tipos de bilirrubina e ictericia)
eee.transaminasas
fff. hemo oxigenasa
ggg. la 7 alfa hidroxilasa
hhh. glucuronil transferasa

246. Un hombre de 57 años de edad es llevado a urgencias con hematemesis masiva rojo brillante, a su llegada se halla
inconsciente con PA: 80/40 mm Hg y FC: 124 lat/min. Luce ictérico con presencia de “arañas vasculares en el tórax anterior y
extremidades”, abdomen distendido con signo de oleada positiva. Se encuentra esplenomegalia y pérdida de la masa
muscular en extremidades. La anastomosis vascular responsable del sangrado en este paciente es ………….…..
(Unidad 3, sesión 21, logro 2: Describe las anastomosis porto sistémicas)
iii. vena gástrica izquierda y vena ácigos
jjj. arteria gástrica izquierda y vena ácigos
kkk. vena paraumbilical y vena epigástrica inferior
lll. vena gástrica izquierda y vena esofágica superior
247. Un estudiante de medicina está comiendo un plato de comida a base de champiñones, espárrago y salsa de soya. El
estímulo del sabor umami contenido en todos estos alimentos viaja a través del nervio………………..
(Unidad 2, sesión 10, logro 3: Describe la irrigación e inervación de la lengua)
mmm. Lingual
nnn. Hipogloso
ooo. Glosofaringeo
ppp. Cuerda del tímpano
248. Una paciente de 30 años de edad es sometida a una cirugía en oído medio derecho por un problema de
otoesclerosis. Luego de la cirugía refiere alteración sensitiva de la lengua. Al evaluar el caso usted esperaría
encontrar……….
(Unidad 2, sesión 10, logro 5: Describe la Irrigación e inervación de la lengua)
qqq. Alteración en la sensación del dolor y temperatura en el tercio posterior de la lengua
rrr. Alteración en la sensación del dolor en los dos tercios anteriores de la lengua
sss. Alteración en la sensación del gusto en el tercio posterior de la lengua
ttt. Sensación del dolor, tacto y temperatura conservadas

249. En un paciente de 45 años de edad con colestasis biliar, se encuentra una elevación de los niveles sanguíneos de
fosfatasa alcalina hasta 3 veces la cifra normal. ¿Cuál de las siguientes alternativas estará también elevada como evidencia
del daño de la vía biliar?
(Unidad 3, sesión 22, logro 5: Pruebas de función hepática, Insuficiencia hepática, encefalopatía hepática e
hipertensión portal)
uuu. Tiempo de protrombina y albúmina sérica
vvv. Transaminasas hepáticas (ALT y AST)
www. Gamma glutamil transpeptidasa
xxx. Glucoronil transferasa

250. Experimentalmente se incrementa la velocidad de la secreción salival con una sustancia, en el análisis de la
composición de esta saliva obtenida se espera encontrar…………..
(Unidad 3, sesión 17, logro 5 : Explica la Influencia de la velocidad del flujo salival en la composición de la saliva)
yyy. disminución de la concentración de bicarbonato que supera la concentración plasmática
zzz. aumento de la concentración de cloro y sodio que supera la concentración plasmática
aaaa. aumento de la concentración de bicarbonato que supera la concentración plasmática
bbbb. disminución de concentración de potasio y bicarbonato

251. Lactante de 3 meses de vida es atendido por presentar diarrea, se administra una solución de glucosa y
electrólitos por vía oral. La proteína de membrana apical que explica la capacidad de esta solución para
proporcionar aporte de glucosa e hidratación es ………..
(Unidad 4, sesión 26, logro 1: Explica la Digestión y Absorción de los hidratos de carbono. Alteraciones)
cccc. CFTR
dddd. SGLT-1
eeee. GLUT-2
ffff. GLUT-5

252. Paciente ha sufrido herida de bala en el abdomen, se le ha tenido que extirpar el segmento medio y distal del
ileon. En este caso la síntesis hepática de sales biliares estará …..…..
(Unidad 3, sesión 22, logro 4: Explica la formación, función y Circulación entero hepática de lasa sales biliares)
gggg. Sin cambios en el ritmo de síntesis
hhhh. Disminuida por inhibición de la enzima colesterol 7 alfa hidroxilasa
iiii. Incrementada por estímulo de la enzima colesterol 7 alfa hidroxilasa
jjjj. Incrementada por inhibición de la enzima colesterol 7 alfa hidroxilasa

253.Un varón de 75 años ingresa al consultorio por presentar ictericia marcada de piel y las escleras. El estudio del
paciente mostró que presentaba un tumor que obstruía la totalidad del conducto hepático común. ¿Cuál de los siguientes
conductos se encontrará dilatado en este paciente?
(Unidad 3, sesión 21, logro 6: Describir el árbol biliar intrahepático)
kkkk. de Wirsung
llll. de Hering
mmmm. colédoco
nnnn. cístico
254. Correlaciones las dos columnas y marque la fórmula correcta: ​(Unidad 4, sesión 28, logro 1: Diarrea: definición, mecanismos:
osmótica, secretoria y exudativa)

1. Enfermedad Hirschsprung( ) heces con moco y sangre


2. Diarrea osmótica( ) intolerancia a lactosa
3. Diarrea secretoria( ) aganglionosis congénita
4. Diarrea exudativa( ) canales de Cl- en las células de la cripta a.- 4231b.- 1234c.- 2143​d.-

4213

255. La vena umbilical obliterada del hígado después del nacimiento se transforma en el ligamento:
(Unidad 3, sesión 21, logro 1: Hígado: relación con la pared abdominal, caras, lóbulos, ligamentos , hilio hepático)
oooo. cruzado
pppp. redondo
qqqq. coronario
rrrr. falciforme

256. Llega a su guardia nocturna una madre que trae a su RN masculino de 2 semanas de vida con mal estado general y
sequedad de mucosas. Usted observa que lacta ávidamente, pero a las 2 horas presenta vómito postprandial no bilioso en
proyectil. Al realizar la historia clínica, descubre que el lactante recibió profilaxis con macrólidos para tos ferina. Usted
sospecha principalmente en:
(Unidad 1, sesión 4, logro 4: Desarrollo y anomalías del intestino anterior)
ssss. estenosis pilórica hipertrófica congénita
tttt. fistula traqueo esofágica
uuuu. estenosis duodenal
vvvv. atresia duodenal

257. En la regulación del apetito y la saciedad, la estimulación experimental crónica del núcleo ventro medial del
hipotálamo producirá:
(Unidad 1, sesión 3, logro 4: explica los mecanismos de control del apetito y saciedad)
wwww. afagia
xxxx. obesidad
yyyy. hiperfagia
zzzz. activación de neuronas relacionadas a NPY

258. Paciente mujer de 25 años acude por dolor en fosa ilíaca derecha que empeora al toser o caminar, asociada a náuseas y
vómitos por lo cual acude a emergencia. Dos días después de realizarle una apendicectomía, la paciente desarrolla fiebre
alta (39 °C), está hipotensa y presenta dolor abdominal. La laparotomía exploratoria muestra un gran volumen de sangre en
la cavidad peritoneal por lesión de un vaso producida durante la apendicectomía.
¿Cuál de las siguientes arterias debe ligarse para detener la hemorragia?
(Unidad 4, sesión 27, logro 4: Irrigación arterial del colon, recto y conducto anal)
aaaaa. cólica derecha y arteria rectal superior.
bbbbb. ileocólica y arteria cólica media.
ccccc. mesentérica superior.
ddddd. ileocólica.

289. La onda peristáltica secundaria del esófago se caracteriza por ser originada ………
(unidad 2, sesión 12, logro 3: Motilidad esofágica: fases y características)
eeeee. por el plexo de meissner del esófago
fffff. por el plexo mientérico del esófago
ggggg. por el reflejo de la deglución
hhhhh. durante la masticación

290. ¿Cuál de los siguientes es una causa de ictericia con bilirrubina conjugada aumentada?
(Unidad 3, sesión 22, logro 3: Producción y excreción de bilirrubina. Tipos de bilirrubina, ictericia)
iiiii.Ictericia del recién nacido
jjjjj. Obstrucción del colédoco
kkkkk. Anemia hemolítica
lllll.Gran hematoma
291. En relación a la absorción de nutrientes, la absorción de dipéptidos y tripéptidos a nivel de las células epiteliales del
intestino delgado, se da principalmente debido a:
(Unidad 4, sesión 26, logro 2: Digestión y absorción de las proteínas. Alteraciones)
mmmmm. el incremento de los canales de Cl- en la membrana apical
nnnnn. la gradiente de bicarbonato en la membrana basal
ooooo. la gradiente de iones H+ en la membrana apical
ppppp. la gradiente de Na+ en la membrana apical

292. Paciente de 20 años es traído a la emergencia por presentar diarreas desde hace 2 días. Familiar refiere que las
deposiciones son líquidas y abundantes, al examen luce deshidratado y se plantea que la diarrea es producida por una
toxina que estimula la transformación de ATP a AMPc con apertura de canales de Cl- y pérdida de agua. El tipo de diarrea
más probable es:
(Unidad 4, sesión 28, logro 1: Diarrea: definición , mecanismos: osmótica, secretoria y supurativa)
qqqqq. osmótica
rrrrr. exudativa
sssss. secretoria
ttttt. por intolerancia a lactosa
293. Un niño fue operado por una obstrucción intestinal, observándose la presencia de divertículo de Meckel. Según lo
referido, marque lo correcto:
(Unidad 1, sesión 5, logro 2: identifica las anomalías del desarrollo del intestino medio: onfalocele y gastrosquisis
(diferencias), Divertículo de Meckel)
uuuuu. el 50% de la población lo presenta
vvvvv. se localiza en el íleon muy cerca al yeyuno
wwwww. puede poseer tejido gástrico o pancreático
xxxxx. se produce por una mala rotación de los intestinos

294. Marque la alternativa correcta respecto a la


estructura marcada en el gráfico:
(Unidad 3, sesión 22, logro 2: Secreción biliar.
Visión general del sistema biliar extrahepático y
composición de la bilis)

yyyyy. Se halla a 2 centímetros debajo


de la papila duodenal mayor
zzzzz. Llega el conducto colédoco y
pancreático principal
aaaaaa. Llega el conducto hepático
común y pancreático principal
bbbbbb. Llega el conducto pancreático accesorio

295. ¿Cuál de las siguientes moléculas se encontrará aumentada en el citoplasma de las células parietales de un
paciente con sindrome de Zollinguer Ellison? ​(Unidad 3, sesión 20, logro 4: Enfermedad ulcerosa péptica: úlcera
gástrica, duodenal. síndrome de Zollinger – Ellison)
cccccc. Péptido liberador de gastrina (GRP)
dddddd. Proteína G estimulante (GS)
eeeeee. Inositol Trifosfato (IP3)
ffffff. AMP cíclico (AMPc)

296. Los fármacos inhibidores de la bomba de protones, actúan bloqueando la ………..…….. ​(Unidad 3, sesión 20, logro 3:
Regulación de la secreción gástrica: inhibición, Secreción de pepsinógeno y factor intrínseco)
gggggg. anhidrasa carbónica
hhhhhh. ATPasa H+/K+ en la membrana luminal
iiiiii. ATPasa H+/K+ en la membrana basolateral
jjjjjj. ATPasa Na+/K+ en la membrana basolateral
297. Un paciente fue diagnosticado de gastritis autoinmune, ¿cuál de las siguientes alternativas es ​FALSA ​respecto a esta
enfermedad? ​(Unidad 3, sesión 20, logro 5: Gastritis crónica: helicobacter pylori, autoinmune. Tipos de gastritis)
kkkkkk. Afecta principalmente el fondo y cuerpo gástrico
llllll. Se produce hiperplasia de células G secundaria a la aclorhidria
mmmmmm. El propio sistema inmune destruye principalmente las células parietales
nnnnnn. Se produce atrofia de la mucosa, aclorhidria, hipergastrinemia y déficit de vitamina B6
298.​Marque la correlación correcta: ​(Unidad 3, sesión:18, logros:1 y 2: Describe las enfermedades inflamatorias/infecciosas y
proliferativas de la cavidad oral)

1. Herpes virus( ) En relación al abuso de antibióticos


2. Candidiasis oral( ) Lesiones vesiculares como racimo de uvas
3. Eritroplaquia( ) Mega esófago
4. Enfermedad de Chagas( ) Lesión pre cancerígena

a.- 2431b.- 1234c.- 4123 ​d.- 2143

299. En un paciente con insuficiencia renal crónica, el déficit en la absorción de calcio a nivel del enterocito se debe a lo
siguiente: ​(Unidad 4, sesión 26, logro 6: Explica la Absorción de calcio y hierro)
oooooo. No se convierte la 25 hidroxicolecalciferol a 1,25 dihidroxicolecalciferol
pppppp. No se convierte la 1,25 dihidroxicolecalciferol a 25 hidroxicolecalciferol
qqqqqq. Existe un descenso de la alfa 25 hidroxilasa renal
rrrrrr. Se incrementa la producción de Calbindina
300. ¿Cuál de las glándulas salivales es responsable del mayor porcentaje del volumen de la saliva en condiciones basales?
a. Parótida
b. Sub Palatinas c.
Sublinguales d.
Submaxilares

301. La lengua está recubierta por epitelio:


c. pseudoestratificado columnar no queratinizado
d. plano estratificado no queratinizado
e. pseudoestratificado columnar ciliado
f. plano estratificado queratinizado

302. El esfínter anal interno tiene musculatura …………… y tiene control ………………….
g. lisa / voluntario
h. lisa / involuntario
i. esquelética / simpático
j. esquelética / parasimpático

303. La arteria aorta proporciona la irrigación al tubo digestivo ¿cuál de las siguientes arterias proporciona la irrigación al ángulo
cólico derecho?
k. Mesentérica superior
l. Mesentérica inferior
m. Frénica inferior
n. Tronco celiaco

304. Paciente de 26 años que le cuenta en su historia clínica que cada vez que almuerza, a los 20 minutos tiene deseo de
defecar. Le comenta que su hijo de 1 mes le pasa lo mismo pero más intenso. Esto se explica por el reflejo
…………………, el cual estáen el paciente.
o. colicoileal / normal
p. colicoileal / alterado
q. gastrocolico / normal
r. gastrocolico / alterado
305. La región del estómago que se comunica con el duodeno se denomina:
s. pilórica
t. cardias
u. cuerpo
v. fórnix
306. Acude a consulta un paciente que fue diagnosticado de ulcera péptica 3 días antes. Luego de múltiples pruebas diagnósticas,
se concluye que el paciente presenta un tumor secretor de gastrina ¿Cuál de las siguientes situaciones estará incrementada?
w. Distención gástrica
x. Inhibición del vaciado gástrico
y. Secreción de ácido clorhídrico (HCl)
z. Inhibición de la secreción de pepsinógeno
307. En el sistema digestivo, el control del apetito esta dado por un complejo sistema de sustancias y órganos integradores los
cuales regulan la ingesta de alimentos. Laes una sustancia oroxígena y es sintetizada
por el ……………………..
aa. leptina / intestino
bb. grelina / intestino
cc. leptina / estómago
dd. grelina / estómago
308. Sobre el control autónomo del sistema digestivo, marque la alternativa correcta:
ee. La inervación dada por el sistema simpático es de tipo preganglionar.
ff. El sistema parasimpático usa como neurotransmisores a la acetilcolina y la noradrenalina.
gg. El nervio vago (par craneal X) le da inervación simpática a la mayoría del sistema digestivo.
hh. En el sistema simpático, los nervios responsables hacen una primera sinapsis en ganglios próximos al órgano a
inervar.
ii. En la inervación de tipo parasimpático, solo interviene el plexo submucoso, sin embargo, en la de tipo
simpático intervienen tanto el submucoso como el mientérico.
309. Con respecto a la actividad eléctrica del sistema digestivo, marque la alternativa correcta
jj. Corresponden a potenciales de acción que están presentes de forma continua y le dan la capacidad de
perístasis autónoma al sistema digestivo.
kk. La frecuencia de las ondas lentas no se ve influenciada por la actividad neural ni las hormas
gastrointestinales.
ll. En el estómago las ondas lentas se dan en una frecuencia de 6 por minuto.
mm. Las ondas lentas son cambios lentos y ondulantes del potencial en reposo.
nn. La frecuencia de las ondas lentas va de 6 a 12 ondas por minuto.

310. Ante una lesión del IX par craneal, el músculo…se altera en su función.
oo. palatogloso
pp. estilofaríngeo
qq. palatofaríngeo
rr. constrictor superior

311. Un varón de 50 años es sometido a extirpación de duodeno y parte proximal de yeyuno. La pérdida de estímulo hormonal
en el páncreas para la secreción enzimática se explica por la pérdida de las células ……………………
ss. Parietales, productoras de factor intrínseco
tt. “K” productoras de factor intrínseco
uu. “M” productoras de CCK
vv. “I” productoras de CCK
312. ​Respecto al mecanismo de la defecación ¿Cuál de las siguientes afirmaciones es correcta?
ww. Se produce contracción refleja del esfínter anal interno
xx. Se produce contracción o relajación del esfínter anal externo por señales de la corteza cerebral
yy. La presencia de materia fecal en el recto estimula la contracción del sigmoides por los nervios pélvicos
simpáticos
zz. En la posición de “cuclillas” el músculo puborectal se halla contraído favoreciendo la evacuación de la materia
fecal

313. Un niño de tres años llega a emergencia con disfagia (dificultad para tragar), salivación y llanto. Se sospecha de ingesta de
cuerpo extraño: moneda en el esófago; al ser evaluado se constata en una radiografía presencia de cuerpo extraño a nivel de C6 y
C7 (6° y 7° vértebra cervical). El cuerpo extraño estará suspendido a nivel del estrechamiento producido por el ………..
aaa. cayado aórtico
bbb. hiato esofágico
ccc. músculo cricofaríngeo
ddd. bronquio principal izquierdo

314. En el caso de un paciente con un tumor productor de gastrina, la presencia de úlceras duodenales y erosión de la mucosa
gástrica se debe principalmente a…….
eee.la acción paracrina de la gastrina sobre la célula parietal
fff. el exceso de HCl por estímulo de receptores CCK-B en la célula parietal
ggg. la sobre expresión de los receptores “G” para gastrina en la célula parietal
hhh. el exceso de HCl por estímulo directo de receptores “H” en la célula parietal

315. La onda peristáltica secundaria del esófago se caracteriza por ser originada ………
iii. por el plexo de submucoso del esófago
jjj. por el plexo mientérico del esófago
kkk. por el reflejo de la deglución
lll. durante la masticación

316. Marque lo correcto sobre las ondas lentas en el tubo digestivo


mmm. No son despolarizaciones
nnn. Son potenciales de acción subumbrales
ooo. Se constituyen de despolarizaciones y repolarizaciones
ppp. Son rítmicas y generadas por el sistema nerviosos autónomo

317. Recién nacido que presenta protrusión de contenidos abdominales los cuales no están cubiertos por peritoneo y salen de la
cavidad abdominal a través de un defecto de la pared. ¿Cómo se denomina a la afección que presenta este paciente?
qqq. Onfalocele
rrr. Atresia biliar
sss. Gastrosquisis
ttt. Divertículo de Meckel

318. Experimentalmente se utiliza atropina (anticolinérgico) para inhibir la secreción de gastrina, sin embargo, la secreción de
esta hormona se sigue dando ante estímulos vagales. Esta situación se explica porque la atropina:
uuu. no bloquea la acción del péptido GRP
vvv. solo inhibe la acción del péptido GRP en la célula G
www. inhibe la acción de acetilcolina e histamina en la célula G
xxx. bloquea parcialmente la bomba de protones en la célula G
319. Niña de 4 días es llevada a la emergencia pediátrica por presentar llanto constante, la madre refiere coloración azulada de
labios al momento de lactar, acompañado de tos persistente y dificultad respiratoria así como distención abdominal. Se le coloca
sonda nasogástrica para alimentación notando que retorna a la cavidad oral en todos los intentos. ¿Cuál es la anomalía del
desarrollo en este caso?
yyy. Solo fístula traqueo esofágica
zzz. Fístula traqueo esofágica proximal y distal
aaaa. Atresia esofágica proximal con fístula traqueo esofágica distal
bbbb. Atresia esofágica distal con fístula traqueo esofágica proximal
320. Paciente varón de 36 años es traído a la emergencia luego de sufrir un accidente de tránsito, presenta traumatismos múltiples
en cabeza y tronco. Al examen físico se evidencia hematoma en hemicara izquierda, ligera protrusión y caída del lado izquierdo del
maxilar inferior, por lo que se realiza una tomografía donde se halla una fractura de la apófisis coronoides del maxilar inferior.
¿Qué músculo está relacionada directamente con esta situación?
cccc. Masetero
dddd. Temporal
eeee. Buccinador
ffff. Pterigoideo medial

321. Un paciente refiere no percibir algunos sabores. Al examen físico constata alteración del sabor dulce y umami.
¿Cuál de los siguientes nervios estará alterada su función?
gggg. Cuerda del tímpano (VII par)
hhhh. Lingual (rama del V par)
iiii. Glosofaríngeo (IX par)
jjjj. Hipogloso (XII par)

322. A los pocos días de nacido, regresa a neonatología un niño con problemas de motilidad del colon; los estudios determinan
ausencia congénita de células ganglionares. Según el gráfico ¿cuál es la capa en la que se determina la ausencia de dichas células?
kkkk.Mucosa - 2
llll. Muscular propia - 2
mmmm.Muscular propia - 3
nnnn. Muscular de la mucosa - 3

323. Con respecto al control autonómico en el tracto gastrointestinal y en relación a su fisiología. ¿Cuál es la función del
sistema nervioso parasimpático en el tracto gastrointestinal?
oooo. Inhiben la contracción muscular y estimulan la secreción de sustancias a nivel de la submucosa
pppp. Estimulan la contracción muscular y estimulan la secreción de sustancias a nivel de la mucosa
qqqq. Inhiben la contracción muscular e inhiben la secreción de sustancias a nivel de la submucosa
rrrr. Estimulan la contracción muscular e inhiben la secreción de sustancias a nivel de la mucosa

324. Un estudiante que está preocupado por su examen parcial, no ha desayunado ni almorzado; cuando al fin ingiere alimentos,
esto le provoca el aumento de los movimientos musculares del tracto gastrointestinal y la sensación de defecar. ¿Qué reflejo se
ha activado?
ssss. Entero-gástrico
tttt. Gastro-cólico
uuuu. Cólico-ileal
vvvv. Ileo-ileal
325. ¿De qué par craneal es rama el nervio palatino mayor?
wwww. Vago
xxxx. Hipogloso
yyyy. Trigémino
zzzz. Palatogloso

326. ¿En cuál de las fases de la deglución la epiglotis separa la vía respiratoria de la digestiva?
aaaaa. oral
bbbbb. laríngea
ccccc. faríngea
ddddd. esofágica

327. Los péptidos intestinales se pueden clasificar como sustancias endocrinas, neurocrinas y paracrinas, dentro de las
paracrinas se encuentran la somastotatina e histamina. Marque la respuesta correcta
eeeee. La somastotatina es sintetizada por las células B de la mucosa gástrica
fffff.La histamina actúa estimulando su receptor tipo H1 en la mucosa gástrica
ggggg. La histamina es sintetizada por células de tipo paracrino de las glándulas gástricas
hhhhh. La somatostatina presenta dentro de sus funciones la estimulación de la secreción de H​+

328. En relación a los órganos intraabdominales y sus estructuras de fijación, elija la alternativa correcta a.El
mesenterio permite la suspensión e irrigación de los órganos retroperitoneales
b. Tanto el hígado como la vesícula biliar se encuentran ubicados a nivel del flanco derecho
c. El colon, el duodeno y el resto de intestino delgado son órganos considerados netamente como
peritoneales
d. Los ligamentos que encontramos dentro de la cavidad abdominal son el esplenorenal y el gastrofrénico e.Los
omentos van desde el estómago y la segunda porción del duodeno a otras estructuras
intraabdominales y existen dos: el omento mayor y el omento menor

329. Paciente varón de 27 años es llevado por bomberos a emergencia luego de ser asaltado y, tras resistirse, es cortado con el
pico de una botella a nivel abdominal. Al examen físico usted observa que a través de la herida se puede observar la protrusión
de asas intestinales. En relación con las capas de la pared abdominal, marque la alternativa correcta.
iiiii. La fascia de Scarpa está constituida principalmente por tejido adiposo
jjjjj. La pared abdominal está formada por piel, huesos, músculos, fascias y peritoneo parietal
kkkkk. La fascia de Camper es una estructura fibrosa que carece de grasa y su grosor es constante en toda la
pared abdominal
lllll. El músculo oblicuo externo discurre en dirección súpero-interna y se inserta en el borde inferior de las ultimas 3
a 4 costillas
mmmmm. El músculo recto del abdomen tiene como funciones comprimir el contenido del abdomen,
tensar la pared del abdomen y flexionar la columna

330. La explicación fisiológica de presentar somnolencia de 30 minutos a 1 hora después de ingerir alimentos, se explica
por:
a. Aumento del cloro intraluminal
b. Aumento del bicarbonato intraluminal
c. Disminución de ácido carbónico en la célula parietal
d. Disminución de la actividad de la anhidrasa carbónica
e. Aumento de la alcalinidad sanguínea

331. Con respecto a la irrigación arterial del colon, a que arteria corresponde la señalada con la
flecha
f. Cólica derecha
g. Cólica media
h. Cólica izquierda
i. Ileobisecoapendículocólica
j. Arco de Riolano

332. Si un paciente presentara dentro del punto de vista fisiológico, una disminución de
enterocinasa, entonces esto originaría una disminución de la actividad de:
k. la pepsina​ b.la lipasa
c. la quimotripsina
d. el peptido insulinotropo dependiente de glucosa
e. la amilasa

Se valida la opción b debido a su relación con la colipasa.


333. Con respecto a la anatomía del hígado, señale a que estructura pertenece
la marcada por el número 1.
l. Ligamento falciforme
m. Línea de Cantlie
n. Ligamento triangular
o. Ligamento coronario
p. Ligamento teres

Se valida la opción a debido a la ubicación del número 1 en donde se unen el


ligamento falciforme y ligamento coronario.

334. Se presenta un paciente, el cual presenta un antecedente de tuberculosis intestinal, por lo cual, se le resecó 80
cm de íleon distal. Desde el punto de vista fisiológico, el paciente puede presentar una de las siguientes alteraciones:
q. Disminución de la secreción de Vitamina B12
r. Aumento indiscriminado de absorción de ácido fólico
s. Disminución de la absorción de hierro
t. Aumento de la secreción de bicarbonato
u. Disminución de la absorción de ácido glicocólico

335. Un paciente es sometido experimentalmente a un fármaco que modifica el flujo salival, obteniéndose un volumen
de saliva de 288 ml en 6 horas. En este caso las concentraciones de electrolitos y bicarbonato en la saliva obtenida
varían de la siguiente manera:
a.↑ Na+, ↓ K+, ↑ Cl-, ↑ HCO3-
b.↓ Na+, ↓ Cl-, ↑ K+, ↓ HCO3-
c.↑ Na+, ↑ Cl-, ↓ K+, ↓ HCO3-
d.↑ Na+, ↑ Cl-, ↑ K+, ↑ HCO3-
e.↓ Na+, ↓ Cl-, ↓ K+, ↓ HCO3-

Se valida la opción a debido a que se puede considerar como un aumento del flujo de saliva.

336. La siguiente imagen histológica corresponde a la glándula


…………… y la estructura señalada produce ………
v. salival sublingual / mucopolisacáridos
w. oxíntica / pepsinógeno
x. salival submaxilar / ptialina
y. salival parótida / amilasas
z. antrales / gastrina

337. Paciente varón de 65 años con antecedente de hipercolesterolemia, hipertensión arterial, fibrilación auricular y
dos infartos al miocardio previos, aqueja de dolor abdominal intenso de inicio súbito, distensión abdominal, se decide
cirugía con resección de 1,5 metros de intestino delgado terminal y colon ascendente. Como consecuencia de la
resección el paciente tendrá deficiencia de:
aa. Vitamina C
bb. Tiamina
cc. Vitamina A
dd. Vitamina B1
ee. Vitamina B6

Se valida esta opción debido a que su absorción está relacionada al íleon.


338. Uno de los siguientes elementos debería hallarse con más probabilidad en el esófago de un paciente que sufre de
reflujo gastro esofágico…
ff. Pepsina
gg. Tripsina
hh. Quimiotripsina
ii. Carboxipeptidasa
jj. Ácidos biliares

339. Un paciente de 40 años cursa con anemia de 8g/dl, aqueja además de astenia y sensación de hormigueo bilateral
en los miembros inferiores, al examen se halla alteración de la sensibilidad a la vibración y camina con ampliación de la
base de sustentación. Uno de los siguientes procedimientos sería de ayuda para el diagnóstico de este paciente:
kk. Tomografía cerebral
ll. Biopsia de la mucosa gástrica
mm. Biopsia de hígado
nn. Examen de sangre oculta en heces
oo. Biopsia de Ileon proximal

340. Paciente de 60 años ingresa por caída hace 1 hora y pequeño hematoma en cuero cabelludo, al examen físico
ampliado se observa ictericia de piel y mucosas generalizada, abdomen blando, se palpa estructura quística no
dolorosa en hipocondrio derecho que corresponde a vesícula biliar (signo de Courvoisier), en los exámenes de
laboratorio se halla niveles bajos en la formación de estercobilinógeno y urobilinógeno en heces, incremento de la
bilirrubina conjugada en la orina, elevación de fosfatasa alcalina y gamma glutamil transpeptidasa séricas. El presente
cuadro puede ser explicado por:
pp. Reabsorción de hematoma
qq. Litiasis vesicular
rr. Carcinoma de la cabeza de páncreas
ss. Carcinoma con estenosis del conducto hepático común
tt. Anemia hemolítica

341. Paciente varón de 58 años con antecedente de alcoholismo crónico es diagnosticado y recibe tratamiento por
cirrosis hepática. Hace 2 días refiere familiar que tuvo cambio de conducta y no reconoce a algunos familiares. Al
examen físico, se halla ascitis, circulación colateral en abdomen, telangiectasias, en el examen de sistema nervioso:
rigidez de extremidades, ROT incrementados, desorientación en el espacio y asterixis. ¿cuál de las siguientes
circunstancias, explicaría el cuadro en este paciente?
uu. Uso de diuréticos ahorradores de potasio
vv. Incremento de actividad de ureasa bacteriana duodenal
ww. Hemorragia gastrointestinal
xx. Disminución de la producción de NH3+ en el colon
yy. Dieta normo proteica

Se valida la opción e debido al efecto sobre la encefalopatía.


Con respecto de la opción b es incorrecta debido a que hace referencia al duodeno, debería indicar colon.

342. Un recién nacido presenta vómitos biliosos poco tiempo después de cada alimento. Al preguntar a la madre sobre
antecedentes, ella recuerda que tuvo polihidramnios durante la gestación, pero un análisis de cariotipo fue normal. Una
de las siguientes es la causa más probable de estos hallazgos en el recién nacido:
zz. Enfermedad de Hirschprung
aaa. Fístula tráqueo esofágica
bbb. Divertículo ileal
ccc. Estenosis pilórica
ddd. Malrotación de la yema pancreática ventral

343. Un lobulillo hepático se puede dividir en tres zonas como se muestra en el gráfico. ¿Cuál de las siguientes
afirmaciones sobre las tres zonas es verdadera?
eee.La zona 1 tiene los menores depósitos de glucógeno
fff. La zona 3 es la primera en afectarse en una colestasis extra hepática
ggg. La zona 2 es más susceptible a la injuria por isquemia que la zona periportal d.La
zona 2 tiene la mayor capacidad de regeneración
e.La zona 1 es la que tiene menos actividad metabólica.

La pregunta 14 ha sido anulada, sin embargo, ningún estudiante se verá afectado negativamente en su
puntaje debido a esta anulación.

344. En un estudio de la secreción de hormonas gastrointestinales, sus concentraciones en la vena porta se midieron
durante perfusión luminal del intestino delgado con soluciones de diversas magnitudes de pH. ¿Qué hormona
aumentará en el plasma de la vena porta durante perfusión a través del intestino con una solución de pH 3?
hhh. CCK
iii. gastrina
jjj. GIP
kkk. motilina
lll. secretina

345. Paciente de 30 años que ingresa a causa de un traumatismo abdominal cerrado. En la exploración se aprecia
discreta palidez de piel y mucosas, auscultación pulmonar normal, taquicardia de 120 /min. Discreta distensión
abdominal y matidez en flancos; el hematocrito, que era prácticamente normal al ingreso, disminuye a 30% a las tres
horas. En la Rx de tórax se objetiva fractura de las costillas 10-11 izquierdas. La causa más probable de la anemización
en este paciente es:
mmm. traumatismo renal con hemorragia retroperitoneal.
nnn. rotura de hígado con hemoperitoneo.
ooo. rotura de bazo con hemoperitoneo​.
ppp. rotura de mesos con hemoperitoneo.
qqq. traumatismo pancreático con pancreatitis traumática.

346. Mujer de 65 años. Consulta por síndrome constitucional asociado a


dolor abdominal epigástrico progresivo irradiado a espalda, de dos meses de
evolución. El diagnostico de sospecha de adenocarcinoma de páncreas se
confirma por biopsia. Se realiza examen de imagen de abdomen para
evaluación de estructuras vasculares próximas al tumor pancreático. ¿Cuál
es el nombre de la vena señalada que está ausente, trombosada por
infiltración tumoral, condicionando circulación colateral en la pared
gástrica?

rrr. Mesentérica superior


sss. Coronaria estomaquica
ttt. Esplénica
uuu. Porta
vvv. Renal izquierda
347. Revisando la angiotomografía de un hombre de 70 años en estudio por aneurisma de aorta abdominal, el
radiólogo le informa de la presencia de una oclusión completa de la arteria mesentérica inferior. El paciente se
encuentra completamente asintomático. La oclusión de la arteria mesentérica inferior cursa de manera
asintomática en muchas ocasiones ya que el territorio que irriga puede recibir flujo proveniente de la arteria:
www. cólica derecha
xxx. gastroduodenal
yyy. Epigástrica inferior izquierda
zzz. esplénica
aaaa. cólica media

348. En las patologías de esófago es importante conocer bien la anatomía esofágica. ¿Cuál de las siguientes
afirmaciones es correcta?
bbbb. El esófago tiene capa mucosa, muscular y serosa
cccc. El esófago abdominal es más largo que el cervical
dddd. El esófago torácico pasa por detrás del cayado aórtico
eeee. El epitelio esofágico normal es ​de tipo ​cilíndrico.
ffff. El esófago abdominal es discretamente más largo que el torácico

349. A pesar de que pueda haber variaciones anatómicas, lo habitual es que el ciego sea irrigado por una rama arterial
que proviene de unas de las siguientes arterias:
gggg. Iliaca derecha
hhhh. Mesentérica inferior
iiii. Hepática derecha
jjjj. Mesentérica superior
kkkk. Iliaca izquierda

350. Ante un paciente con una cirugía abdominal urgente, el informe operatorio señala que se ha realizado una
resección de todo el duodeno y del tercio proximal del yeyuno manteniendo íntegros el estómago y todo el íleon, así
como los dos tercios distales del yeyuno. En el seguimiento nutricional del paciente ¿Qué vitamina o mineral
presentará con menor probabilidad una disminución de su absorción?
llll. Cianocobalamina
mmmm. Calcio
nnnn. Hierro
oooo. Transcobalamina
pppp. Transferrina

351. ¿Cuál de las siguientes alternativas detallan las venas que confluyen y forman la vena señalada?
qqqq. mesentérica superior, gástrica izquierda y
gastroepiploica izquierda
rrrr. mesentérica inferior, gástrica izquierda y renal
ssss. esplénica, mesentérica superior y mesentérica inferior
tttt. esplénica, pancreatoduodenal y omental izquierda
uuuu. gástrica izquierda, esplénica y hepática común

352. ¿Cuál de las siguientes sustancias forma parte de la secreción biliar?


vvvv. Tripsina
wwww. Lecitina
xxxx. Elastasa
yyyy. Quimotripsina
zzzz. Pepsina
353. El tubo digestivo contiene diferentes tipos de epitelios y glándulas. La estructura señalada es unay
está localizada en el …………...
aaaaa. glándula de Brunner / intestino
grueso​ b.cripta de Lieberkuhn / colon
c. cripta de Lieberkuhn / intestino delgado
d. glándula oxintica / estomago
e. célula parietal / estómago

Aunque las criptas de Lieberkuhn están


presentes en el intestino delgado, la
microfotografía es de epitelio de
colon.

354. ¿De qué musculo forma parte el ligamento inguinal?


bbbbb. Oblicuo externo del abdomen
ccccc. Oblicuo interno del abdomen
ddddd. Transverso del abdomen
eeeee. Psoas
fffff.Dorsal ancho

355. Señale cuál de las siguientes afirmaciones ​NO ​se relaciona a la siguiente glándula anexa del tubo digestivo
mostrada en la imagen:
ggggg. Es una glándula exocrina compuesta exclusivamente por acinos serosos
hhhhh. Su inervación está dada por el nervio auricular mayor (ramo posterior C2), que inerva la vaina
de la glándula así como la piel por encima de esta.
iiiii. Esta glándula produce una secreción mucinosa acuosa, llamada mucoserosa, a través del conducto de
Wharton.
jjjjj. Su inflamación puede ser causada por un virus de los Paramyxoviridae, que provocan una
enfermedad muy frecuentemente en niños y adolescentes
kkkkk. Es una glándula endocrina y probablemente sea de origen pancreático

Se valida la opción e debido a que no está relacionada


con la imagen.

356. ¿Cuál de las siguientes enzimas está localizada en el borde en cepillo y juega un rol en la digestión de proteínas?
lllll. Alfa dextrinasa
mmmmm. Pepsina
nnnnn. Enterocinasa
ooooo. Lactasa
ppppp. Carboxipeptidasa A​.

Se valida la opción c debido a que es correcta en relación a la pregunta.


357. Una de los siguientes sustancias, ​NO s​ irve como un buen agente emulsificante:
qqqqq. Colesterol
rrrrr. Ácidos grasos
sssss. Sales biliares
ttttt. Lecitina
uuuuu. Proteínas de la dieta

Se valida la opción e debido a que es correcta en relación a la pregunta.

358. La sustancia que estimula el crecimiento de la mucosa gástrica es:


vvvvv. Secretina
wwwww. Motilina
xxxxx. Péptido estimulante de la mucosa gástrica
yyyyy. Gastrina
zzzzz. Histamina

359. ¿Cuál de las siguientes alternativas es una función de la colecistokinina?


aaaaaa. Relajación de la vesícula para la salida de
bilis​ b.Secreción de ácidos biliares
c. Contracción del esfinter de Oddi
d. Secreción de enzimas pancreáticas
e. Contracción del duodeno

Se valida la opción b debido al efecto de la CCK sobre la vesicula biliar.

360. Con respecto a la anatomía del tronco celiaco, señale lo correcto


bbbbbb. El tronco celiaco se origina de la cara posterior de la aorta abdominal
cccccc. Es una arteria delgada que tiene un calibre entre 2 y 3 mm
dddddd. Una de sus ramas es la arteria gástrica derecha
eeeeee. La hepática común que es una de sus ramas, participa en la irrigación del estómago​.

361. Con respecto a la anatomía del duodeno, marque la respuesta correcta:


ffffff. Tiene una distribución en forma de “C”, que rodea la cola del páncreas
gggggg. La 3ra porción duodenal está contenida en la pinza vascular aortomesentérica
hhhhhh. Entre la 1ra y 2da porción se forma un ángulo, conocido como el ángulo de Treitz
iiiiii. La 4ta porción se dirige a la izquierda, hacia abajo y hacia atrás.
jjjjjj. En la tercera porción desemboca el conducto colédoco.

362. El hígado está ampliamente tapizado por peritoneo, la estructura que conecta la cara diafragmática del hígado
precisamente con el diafragma es el ligamento:
kkkkkk. teres
llllll. falciforme
c.triangular
d.hepático común
e.coronario

Se validan la opción c y e debido a que forman parte de los ligamentos que fijan el hígado al diafragma.

363. En el íleon se absorbe aproximadamente el 95% dea través de la circulación enterohepática.


mmmmmm. agua
nnnnnn. colesterol
oooooo. sales biliares
pppppp. hidróxicobalamina
qqqqqq. factor intrínseca
364. Laestimula el mecanismo paracrino de la secreción de ácido clorhídrico.
rrrrrr. histamina
ssssss. acetilcolina
tttttt. gastrina
uuuuuu. secretina
vvvvvv. somatostatina

365. En la digestión de proteinas,es el principal estímulo para convertir el pepsinógeno en pepsina.


wwwwww. la gastrina
xxxxxx. el pH ácido
yyyyyy. la acetilcolina
zzzzzz. la ptialina
aaaaaaa. la somatostatina

366. Con respecto a la somatostatina, marque lo correcto:


bbbbbbb. Es secretada por las células S del intestino
ccccccc. Induce a la producción de VIP
ddddddd. Interviene en la fase intestinal de la secreción gástrica
eeeeeee. Produce acetilcolina para estimular a la célula parietal
fffffff. No interviene en la regulación de la secreción de ácido clorhídico

367. En pecten anal, es una estructura comprendida entre:


ggggggg.la línea pectínea y los senos anales
hhhhhhh. la línea blanca y la apertura anal
iiiiiii. el esfínter anal interno y el externo
jjjjjjj. la línea anocutánea y la línea pectínea
kkkkkkk. la línea blanca y columnas anales

368. ¿Cuál de las siguientes alternativas es una proenzima pancreática?


lllllll. Tripsina
mmmmmmm. Elastasa
nnnnnnn. Quimotripsinógeno
ooooooo. Amilasa
ppppppp. Procarboxipepitidasa C.

369. En la segmentación hepática de Coinaud, el segmento hepático señalado con la flecha, corresponde a :
En la segmentación hepática de Coinaud, la flecha señala el segmentohepático.
qqqqqqq. IV
rrrrrrr. V
sssssss. VI
ttttttt. VII
uuuuuuu. VIII
370. La contracción del músculopermite la eliminación de gases (flatos) sin salida de material fecal;
es el mismo músculo cuya relajación, sobretodo en cuclillas, permite el paso del contenido fecal con menor
esfuerzo durante la defecación.
a) Isquirectal​ b)
Puborrectal
c) Esfínter anal externo
d) Esfínter anal interno

371. Paciente mujer de 54 años se presenta con náuseas, vómitos, estreñimiento, y es diagnosticada de abdomen
agudo quirúrgico; en la cirugía encuentran un vólvulo de ciego. Esta anomalía puede explicarse por::
b) Falta de rotación intestinal
c) Falta de fusión del mesenterio
d) Defecto en la formación de la cloaca
e) Falta de formación del omento mayor

372. Paciente mujer de 23 años con faringitis aguda, toma para el dolor una tableta de paracetamol con un poco de
agua. Durante la deglución, se relaja su esfínter esofágico inferior y el fondo del estómago, mientras el bolo está aún
en el esófago. ¿Qué sustancia provocara con mayor probabilidad la relajación del esfínter esofágico inferior y el fondo
del estómago en esta mujer?
f) Óxido nítrico
g) Sustancia P
h) Histamina
i) Motilina

373. Luego de tres horas dando exámenes, un alumno de medicina comienza a sentir hambre. Esta situación es
probable que sea mediada por la​ ​que es sintetizada por el​ ​:
j) leptina / intestino
k) leptina / estómago
l) grelina​ /​ estómago
m) grelina / tejido adiposo

374. Varón de 72 años, con antecedente de diabetes mellitus tipo 2, que presenta enteropatía diabética caracterizada
por estreñimiento. Este problema puede estar asociado a:
n) deficiencia de óxido nítrico
o) aumento del reflejo gastrocólico
p) disminución de la secreción de colecistocinina (CCK)
q) aumento de la secreción del péptido intestinal vasoactivo (PIV)

375. Varón de 54 años con Diabetes Mellitus tipo 2, es diagnosticado de gastroparesia debido a que presenta sensación
de llenura precoz al comer, y reflujo gastroesofágico. Esta alteración en la relajación receptiva y en el vaciamiento
gástrico lo más probable es que se deba a una alteración en:
r) el nervio vago
s) el ganglio celíaco
t) plexo submucoso
u) nervio hipogástrico

376. Varón de 67 años con tos y disminución de peso asociado a tabaquismo pesado, presenta actualmente disfagia
progresiva a alimentos sólidos. Se considera la presencia de un carcinoma de bronquio izquierdo y por esta razón le
realizan una endoscopía esofágica para descartar la posibilidad de una compresión esofágica por el tumor. Se espera
revisar el esófago en la​ ​estrechez, que está a nivel de la vértebra
v) Tercera estrechez -T6
w) Segunda estrechez - C6
x) Segunda estrechez - T4
y) Tercera estrechez -T10

377. Varón de 34 años con dolor abdominal agudo en flanco derecho que se irradia a fosa ilíaca derecha, es operado y
se encuentra un divertículo intestinal inflamado, ubicado a 93 cm de la válvula ileocecal. El origen de este divertículo
es una falla en la obliteración de:
z) Conducto vitelino
aa) Alantoides
bb) Cloaca
cc) Conducto anorectal
dd) Uraco

378. En un niño menor de dos años con divertículo intestinal, este divertículo tiene su origen en una falla en la
obliteración de:
a)Conducto anorectal
b) Conducto vitelino
c) Alantoides
d) Cloaca
e) Uraco

379. Mujer de 43 años sufre un grave accidente de tránsito y está hospitalizada en coma, es alimentada por vía
intravenosa durante varias semanas. Producto de este tipo de alimentación, se encuentra en la endoscopía atrofia de la
mucosa gastrointestinal. La causa más probable de esta atrofia son los bajos niveles séricos de la hormona:
ee) Colecistocinina
ff) Secretina
c)Gastrina
d) PIV

380. Una mujer de 30 años llega al consultorio porque se queja de dificultades para deglutir, la cual se agravan cada vez
más. Se realiza un estudio manométrico para examinar la generación de presión a lo largo del esófago. Esta prueba
revela que las contracciones como respuesta a la deglución están mal sincronizadas y que la presión en el esfínter
esofágico inferior permanece elevada. El diagnóstico más probable es​ ​producido por niveles bajos de
gg) acalasia / sustancia P​ b)
acalasia / óxido nítrico
c) enfermedad por reflujo gastrointestinal / acetilcolina
d) enfermedad por reflujo gastrointestinal / óxido nítrico

381. Paciente de 2 años, llega a emergencia por haber ingerido una moneda con la que estaba jugando. El lugar más
probable donde puede haberse quedado suspendido este objeto es a nivel del estrechamiento producido a nivel del:
hh) músculo milohiodeo
ii) músculo aritenoideo
jj) músculo cricofaríngeo
kk) constrictor superior de la faringe

382. En una apendicectomía, al realizar la incisión de McBurney en la fosa iliaca derecha, es necesario cortar los
siguientes músculos, de afuera hacia adentro:
ll) Recto – Oblicuo externo – Transverso
mm) Recto – Oblicuo externo – Oblicuo interno
nn) Oblicuo externo – Oblicuo interno – Recto
oo) Oblicuo externo – Oblicuo interno – Transverso

383. Un varón de 90 años que se encuentra postrado en cama, es referido del asilo para endoscopia por dificultad para
deglutir luego de tomar un medicamento para aliviar el dolor la noche anterior. La endoscopía revela que la píldora se
alojó en el esófago y causó una reacción inflamatoria. Lo más probable es que esto haya sido por la producción de
múltiples ondas:
pp) secundarias
qq) primarias
rr) lentas
ss) segmentarias

384. Mujer de 23 años es diagnosticada de bulimia, al examen físico se observa ulceraciones en el segundo y tercero
dedo de la mano derecha. Esto se puede deber al uso continuo de estos dedos para inducir el vómito, mediante la
estimulación del par craneal:
tt) V​ b)
IX
c) X
d) XI

385. Varón de 52 años se presenta por diarrea persistente de seis semanas de duración. En la colonoscopia se observa
un pólipo a nivel del íleon distal. El patólogo informa que se trata de un tumor neuroendócrino, probablemente
originado por las células enterocromafines del intestino. La sustancia que más probablemente esté produciendo este
tumor es:
uu) Serotonina
vv) Insulina
ww)CCK
xx) GIP

386. La fase oclusal de la masticación se realiza con la contracción de los músculos:


yy) digástricos
zz) masetero y temporal
aaa)orbicular y buccinador
bbb) pterigoideo lateral y digástrico

387. Al tomar su café en Starbucks, un estudiante de medicina sufre una quemadura de primer grado en el tercio
anterior de la superficie dorsal de la lengua. La información de dolor es transmitida por el nervio:
ccc) cuerda del tímpano
ddd) glosofarínge
o​ c)lingual
d) facial

388. Paciente es evaluado por faringitis aguda en consultorio externo. El médico de familia le solicita que abra la boca
y saque la lengua. Para realizar la acción de sacar la lengua, es necesario que se contraiga el músculo:
eee)estilogloso​ b)
geniogloso
c) palatogloso
d) transverso de la lengua

389. Paciente con síndrome de


Sjögren, presenta “boca seca”
(disminución de la producción de
saliva) y caries dental, asociada a la
pérdida de la función de tampón de
la saliva. Esta desminerilización del
diente puede comprometer a las
prolongaciones citoplasmáticas
ubicadas en los tubos huecos de la
estructura señalada con la letra:
fff) B
ggg)A
hhh) E
iii) C

390. Mujer de 32 años acude a


consulta por presentar disfagia de
progresión lenta, reflujo
gastroesofágico y vómitos desde hace
3 meses de evolución progresiva. Se le
realiza un estudio radiológico con
contraste en el que se observa
estrechamiento del esfínter esofágico
inferior (imagen). Según sus
conocimientos, este paciente se
beneficiaría con el uso de:
jjj) agonista beta adrenérgico
kkk) agonista alfa adrenérgico
c)análogo de óxido nítrico
d) análogo de Sustancia P
391. Paciente de sexo masculino de 82 años de edad ingresa a emergencia con dolor abdominal agudo y diarreas. Se le
realiza una arteriografía en la que se observa que la arteria aorta tiene un trombo ocluyendo el 95% del flujo, a nivel
del nacimiento de la arteria mesentérica inferior. ¿Cuál de las siguientes arterias podría contribuir a la irrigación
colateral del colon descendente?
a) cólica media
b) sigmoidea
c) rectal superior
d) ileocólica

392. Niño de 5 años presenta dolor esofágico y hematemesis (vómitos hemorrágicos) luego de tragarse una espina de
pescado. En la endoscopía se observa perforación del esófago distal a la cuarta estrechez esofágica. ¿Las ramas de cuál
de las siguientes arterias estarán lesionada con mayor probabilidad?
e) Gástrica izquierda
f) Bronquiales
g) Frénica inferior
h) Tiroidea inferior

393. Al ingerir una cucharada de mantequilla es muy probable que se disminuya la sensación de hambre por medio de
la activación de la vía POMC/CART (POMC=proopiomelanocortina y CART=transcripción regulada de cocaína y
anfetamina), activada directamente por la hormona:
i) colecistoquinina (CCK)
j) insulina
k) grelina
l) secretina

394. Al comer unas papitas fritas con mayonesa, el vaciamiento gástrico disminuye por efecto directo de la hormona:
a)colecistoquinina (CCK)
b) bombesina
c) motilina
d) gastrina

395. Recién nacido de dos horas es diagnosticado de hernia umbilical de 1,5 cm de diámetro; el cirujano pediatra
solicita una tomografía abdominal en donde se evidencia que la hernia umbilical está ocupada por una porción del
tracto gastrointestinal. ¿Qué porción del tracto gastrointestinal estaría ocupando esta hernia con mayor
probabilidad?
m) Íleon
n) Colon sigmoides
o) Duodeno
p) Colon transverso
396. Recién nacido de 7 horas, de parto por cesárea debido a polihidramnios (aumento del volumen del líquido
amniótico), con regurgitación de la leche materna y artificial, y no ha presentado meconio. Se le realiza una tomografía
donde se evidencia aire en el estómago y una malformación del desarrollo esofágico. Con respecto a esta malformación
lo más probable es que se pueda tratar de una atresia esofágica:
q) proximal con fístula traqueoesofágica distal
r) distal con fístula traqueoesofágica proximal
s) proximal y distal
t) sin fístula

397. Lactante de 6 meses de edad que es traído a consulta por presentar vómitos no biliosos a repetición y retraso en
el crecimiento. En la radiografía de abdomen simple se observa nivel hidroaéreo en estómago y en primera porción de
duodeno (doble burbuja). ¿Cuál de las siguientes alternativas puede explicar la condición del lactante?
u) Páncreas anular
v) Atresia duodenal en la tercera porción
w) Atresia yeyunal
x) Hipertrofia del píloro

398. En ausencia o deficiencia de la secreción de la hormona motilina, se producirá:


a)sobrecrecimiento bacteriano
b) diarrea
c) aumento del vaciamiento gástrico
d) hipertrofia del píloro
399. La estimulación parasimpática aumenta la motilidad intestinal, mientras que la estimulación simpática la
disminuye. ¿Sobre cuál de las siguientes alternativas el sistema nervioso autónomo actúa para el control de la
motilidad intestinal?
y) Potencial de membrana en el plexo mientérico (de Auerbach)
z) Frecuencia de ondas lentas
aa) Secreción de secretina
bb) Nivel de IP3 en el plexo submucoso (de Meissner)

400.En un recién nacido con protrusión de contenidos abdominales y cubiertas por amnios o peritoneo, es cierto que:
a)Se presenta por un defecto en el cierre de la pared
b) Se acompaña de otras malformaciones congénitas
c) Se debe al no retorno de la hernia fisiológica
d) Se produce a través del ombligo

401. Lactante de 20 días con estreñimiento, distención abdominal progresiva, acompañada ocasionalmente de vómitos
biliosos. Como antecedente, el meconio lo eliminó por primera vez a las 72 horas de nacido. Su mamá menciona que
ayuda a la evacuación con ayuda de un termómetro rectal. Se sospecha de megacolon agangliónico (Enfermedad de
Hirschsprung). ¿Cuál de las siguientes alternativas explica el caso?
cc) Se presenta contracciones tónicas en la región ano rectal
dd) Se presenta dilatación de tracto gastrointestinal afectado
ee) Las células ganglionares sólo han migrado al ano recto
ff) La zona que más se afecta es inervada por fibras del nervio esplácnico menor

402. ¿Cuál de los siguientes reflejos disminuye el tránsito gastrointestinal?


a)Doloroso
b) Gastrocólico
c) De defecación
d) Colicoileal

403. Al ingerir una sustancia ácida como el vino (pH 3), se estimula la motilidad gástrica por acción de la hormona:
a)motilina
b) secretina
c) colecistoquinina (CCK)
d) bombesina

404.Paciente de 24 años acude a consulta externa por presentar una fístula oronasal (comunicación entre la cavidad oral
y la cavidad nasal). Está fístula está asociada al antecedente de haber sido operada de paladar hendido a los dos años de
edad, durante una campaña gratuita extranjera de corrección de paladar fisurado. ¿Cuál de las arterias palatinas podría
haberse lesionado durante esa cirugía?
gg) Mayor
hh) Menor
ii) Ascendente
jj) Rama palatina de la faríngea ascendente

405.Paciente de sexo masculino de 52 años con úlcera péptica gástrica de 14 años de evolución, con cuadro de
hemorragia digestiva alta hace 4 meses, sin cicatrización de la úlcera. Entre las opciones quirúrgicas se considera
realizarle un vaguectomía troncal (sección del nervio vago) a nivel del hiato esofágico. ¿Cuál de las siguientes
complicaciones podría esperarse producto de la pérdida de inervación parasimpática?
kk) Menor inervación del colon ascendente
ll) Se perderá el reflejo de defecación
mm) Se perderá el reflejo de micción
nn) Impotencia sexual

406.Paciente de 23 años con bulimia es traída a la emergencia deshidratada, semiconsciente y con alcalosis
metabólica. Los vómitos autoinfligidos por esta paciente se producen por estimulación de receptores en la base de la
lengua que mandan información directamente al:
oo) núcleo del tracto solitario
pp) centro del vómito en el tallo encefálico
qq) zona quimiorreceptora gatillo
rr) cerebelo

407.Niño de 3 años es traído a emergencia por madre quien manifiesta que hace 10 horas deglutió una pila pequeña
de reloj de bordes romos. El niño está asintomático. Usted la tranquiliza diciéndole es un cuerpo extraño tan pequeño
de seguro que va a seguir el tránsito intestinal como lo haría un bolo alimenticio, y que lo más probable es que en ese
momento se encuentre en:
ss) colon
tt) estómago
uu) yeyuno
vv) recto

408. Los movimientos en masa son un tipo de movimiento muy importante, una de las consecuencias de estos
movimientos es:
ww)la distensión rectal
xx) el peristaltismo del intestino delgado
yy) la retropulsión gástrica
zz) la contracción del esfínter anal interno

409.Durante la deglución, al momento que el bolo alimenticio pasa por el esfínter esofágico superior, se espera que la
presión intraesofágica:
aaa)disminuya en el cardias
bbb) disminuya en el tercio medio del esófago
ccc) aumente en la porción distal al bolo
ddd) aumente en el tercio medio del esófago

410.Paciente con enfermedad de Chagas que presenta disfagia a sólidos. ¿Cuál de las siguientes puede ser la causa de
esta complicación?
eee)Disminución de células ganglionares en el esfínter esofágico inferior
fff) Aumento en la liberación de óxido nítrico en el esfínter esofágico inferior
ggg)Disminución de las neuronas que liberan péptido intestinal vasoactivo
hhh) Aumento de la actividad de la motilina en el esófago distal

QUIZIZZ
1. Los vasos mesentéricos superiores se hallan a nivel de:
a) Cuello del páncreas

2. El nivel en el que se encuentra el píloro y el páncreas se puede determinar usando el


A) plano transpilórico

3. El dolor de estómago asociado a gastritis se suele ubicar en


a) epigastrio

4. La colecistoquinina inhibe el
a) vaciamiento gástrico

5. Paciente con disfasia ( dificultad para pasar alimentos) con to y disminución de peso. Con antecedente de
tabaquismo pesado. La sospecha es que tenga una disminución del diámetro esófago a nivel de la
a) tercera estrechez

6. La digestión de las proteínas se inicia en:


a) estómago

7. El estómago recibe información simpática proveniente del:


a) ganglio celíaco

8. El reflujo gastroesofagico tiene múltiples etiologías, una de ellas tiene que ver con alteración a nivel de:
a) primera estrechez
b) segunda estrechez
c) tercera estrechez
d) cuarta estrechez

9. La fístula retroperitoneal es causada por una falla en el desarrollo de:


a) tabique urorrectal

10. El nervio vago inerva el:


a) músculo estriado del esófago

11. El divertículo de Meckel es un rezago de:


a) conducto vitelino

12. La presencia de orina que sale por el ombligo de un recién nacido casa vez que llora, es posible que se deba a
un defecto en el desarrollo del
a) seno urogenital

13. La fístula retroperitoneal es causada por una falla en el desarrollo de


a) tabique urorrectal

14. Enfermedad asociada con un error en el desarrollo de las células de Cajal:


a) enfermedad de Hirschsprung

15. Aproximadamente en la semana 6 del desarrollo embrionario, el intestino medio gira 90 herniandose a nivel
del
a) cordón umbilical

16. El conducto biliar deriva del


a) endodermo

17. Paciente mujer con 54 años con nauseas y vómitos y abdomen agudo quirúrgico, se ingresa a sala de
operaciones donde se encuentra vólvulo de ciego, esto se debe a
a) Falta de fusión del mesenterio

18. Paciente con cirrosis hepática con hipertensión portal, en el que es posible encontrar que los vasos
umbilicales están permeables dentro de
a) ligamento redondo

19. Es normal encontrar glándulas submucosas en


a) esófago medio
b) esófago proximal
c) esófago distal
d) estómago

20. Cual de los siguientes órganos son intraperitoneales


a) estómago, vesícula biliar, íleon, hígado
CASO 1

Estudiante de medicina de la UPC de 21 años sufre de gastritis aguda ocasionada por comer en lugares
poco higiénicos. Suele consumir caramelos ( chupar ) mientras está en clase hasta la tarde. Toma gaseosas
regularmente (carbohidratos 46%, sodio 53%). También toma regular cantidad de leche (grasa 35%, lactosa
35%, proteínas 30%), pues le calma un poco el dolor el ardor que siente por la gastritis. Incluso, cuando
puede, se toma dos vasos de agua fría para calmar las molestias. Ha decidido ir al médico para tratarse pues
ya no soporta el dolor, el cual está seguro que los síntomas se deben a una elevada producción de ácido
clorhídrico en el estómago, y por ello le ha recetado Ranitidina (antihistamínico), con lo que siente mejoría.

● Para reducir la secreción de HCl en esta paciente se podría usar sustancias similares a:

-​ ​Péptido insulinotrópico dependiente de la glucosa (GIP)

● Si se usara atropina en esta paciente, se esperaría que disminuya la liberación de:


- Enzimas pancreáticas

● El consumir caramelos eleva los niveles en sangre de una hormona cuya función es la
estimulación de las células:
- ​ ​Beta del páncreas

● En este paciente con gastritis aguda debida a una alta producción de ácido clorhídrico, si se le
hiciera un examen de sangre, se encontraría elevados los niveles de:
- Colecistoquinina

● El consumo de una pequeña cantidad de gaseosa aumentará directamente la concentración


sérica de cuál de las siguientes hormonas:

- Péptido 1 similar al glucagón (GLP-1)

● El consumo rápido de 500 mL de gaseosa aumentará directamente la concentración sérica de


cuál de las siguientes hormonas:

- Gastrina

● Estimulan la secreción ácida gástrica

- Proteínas

● Con respecto a las ondas lentas, marque la afirmación correcta:}

- Son contracciones rítmicas espontáneas

● El uso de Ranitidina bloquea el receptor H2 de la histamina en las células parietales. La


histamina llega a estas células por:

- ​Difusión

● El consumir caramelos indirectamente activa la vía:

- POMC/CART
● ¿Cuál de los siguientes péptidos inhibe el vaciamiento gástrico?

- Colecistoquinina

● Para poder morder una manzana, es necesario usar el siguiente músculo:

- Milohiodeo

● El crecimiento de un adenocarcinoma de cuello de páncreas puede comprometer la pared


gástrica por continuidad. ¿Qué parte del estómago estaría comprometido con mayor
probabilidad?

- Pared posterior del antro

● El nacimiento de la arteria mesentérica superior se puede encontrar en cuál de los cuadrantes


abdominales:

- Epigastrio

● En cuanto a la colecistoquinina, marque la respuesta correcta:

- Potencia la acción de buffer con bicarbonato

● El aumento en la actividad motora de la pared gástrica genera un aumento en los


niveles locales de qué sustancia en la microvasculatura:

- Adenosina

CASO 2

Niño de sexo masculino de 2 años de edad, sufre de estreñimiento desde el nacimiento (1 deposición
cada 3-4 días). Madre menciona que le estimula la defecación con un termómetro rectal, y continuo uso
de enemas y laxantes. Desde hace 6 meses comienza con vómitos postprandiales. Los síntomas
aumentan en frecuencia y magnitud y están en relación con los episodios de estreñimiento. No refiere
fiebre, tos, diarrea ni lesiones cutáneas. Al examen físico presenta regular estado general, luce
deshidratado. Abdomen distendido, blando, depresible e indoloro. No se palpan masas abdominales.
Se permeabiliza el canal anal con termómetro rectal, encontrando cierta resistencia. Salida de material
fecal mal oliente en regular cantidad. Exámenes de laboratorio: hemograma normal. Signos
inflamatorios de fase aguda negativos. Alcalosis metabólica leve en sangre venosa. Radiografía con
enema baritado muestra recto y colon sigmoides dilatados (megacolon). Biopsia profunda: ausencia de
células ganglionares en la muestra enviada. Se realiza cirugía correctiva.

● La percepción de la pirosis (sensación de dolor o quemazón en el esófago) asociado al reflujo


gastroesofágico, puede aparecer o exacerbarse debido a:

- Ejercicio
● Considerando que este paciente está sometido a estrés por el agravamiento de su enfermedad,
es posible afirmar que sus ondas lentas están:

- Hiperpolarizadas

● En cuanto a los reflejos gastrocólico y gastroduodenal en este paciente, indique lo correcto:

- Se pueden considerar reflejos vago-vagales

● diferencia de las arcadas, los vómitos presentan apertura de:


- Esfínter esofágico superior

● En la Enfermedad de Hirschprung, el patólogo debe declarar un área como sana o libre de


enfermedad si es que encuentra:
- Células intersticiales en el plexo mioentérico
● Durante la fase faríngea de la deglución es espera que ocurra el siguiente fenómeno:
- El paladar blando cierra la entrada a la nasofaringe
● En este paciente se considera que está abolido el reflejo:
- Rectoesfinteriano
● Debido al acúmulo de material fecal en todo el marco colónico, y a la irritación química
asociada, el peristaltismo del íleon distal se debe encontrar:
- Inhibido

--------------------------------------------------------------------------------------------------------------------------------------------------
● Con respecto a la defecación señale el enunciado correcto:

- Es estimulado por un llenado de la cuarta parte del volumen rectal

● El contenido fecal se detiene en la zona inmediatamente proximal a la zona donde hay una
menor presencia de:

- Péptido intestinal vasoactivo

● La presencia de atresias y estenosis duodenales se deben básicamente a una:

- Falta de recanalización

● El ligamento de Treitz característicamente:

- Suspende el ángulo de Treitz

● Al deglutir un bolo alimenticio, es lógico suponer que al pasar por el esófago haya un mayor
consumo de oxígeno en la pared del tercio:

- Proxima​l

● Estudiante de medicina de 20 años, se ha amanecido estudiando para su examen de Sistema


Digestivo. No ha probado alimento desde la cena, por lo que se puede afirmar que la motilidad
de esta persona está siendo regulada por:

- Motilina

● La hernia fisiológica se produce dentro de:

- Cordón umbilical

● El crecimiento de un adenocarcinoma de páncreas compromete la pared gástrica por


contigüidad. ¿Qué parte del estómago se esperaría esté comprometido?

- Pared posterior del antro

● Al comer unas papitas fritas con mayonesa, el vaciamiento gástrico disminuye por efecto
directo de la hormona:

- colecistoquinina (CCK)

● Paciente que come entera una pizza familiar de chorizo y queso. Es posible esperar que debido
a la cantidad de alimento ingerida, las ondas lentas hayan:

- Sufrido ninguna alteración en su frecuencia

● En este caso se puede afirmar con seguridad que se presenta:

- contracciones tónicas en la región ano rectal

● La forma más común de atresia esofágica contiene:


- Estenosis proximal del esófago más fístula traqueoesofágica distal

● Con respecto a la saliva, marque la respuesta correcta:

- el sistema simpático estimula su secreción

● Respecto a las enfermedades del esófago, marque lo correcto:

- el diagnóstico diferencial de la acalasia es la enfermedad de Chagas esofágica

● En relación a la fisilogía gástrica, marque lo correcto:

- la cimetidina actúa en la región basolateral de la célula parietal

● La célula mucosa del cuello gástrico produce:

- Moco

● La saliva puede tener una variedad de electrolitos en su composición. Entre ellos el cloro,
respecto al cual se puede afirmar:

- Su concentración no llega a ser tan alta como en el plasma

● Los músculos de la masticación que producen la retropulsión de la mandíbula son:

- temporales

● Respecto a las glándulas salivales, marque lo incorrecto:

- la glándula sublingual tiene forma de garfio

● Respecto a la anatomía del estómago, marque lo correcto:

- la arteria gástrica derecha nace de la arteria hepática común

● En cuanto a la saliva, marque lo correcto:

- La amilasa cumple función digestiva

● Durante el ataque con gas sarín (bloqueador de la acetilcolinesterasa) en el metro de Tokio, en


1995, el personal de salud notó que los pacientes afectados presentaban:

- Hipersalivación

● Para estimular la motilidad intestinal se podría usar un análogo de: ​CCK

● No se espera que sea causa del vómito: ​AYUNO PROLONGADO

● En la Enfermedad de Hirschprung, el patólogo debe declarar un área como sana o libre de enfermedad
si es que encuentra: ​CÉLULAS INTERSTICIALES EN EL PLEXO MIOENTÉRICO.

● En la zona afectada por esta enfermedad, se espera que las ondas lentas estén: ​ABOLIDAS

● Si este paciente tuviera arcadas, entonces se debe considerar que durante la ocurrencia de dichas
arcadas, debemos encontrar contenido gástrico en: ​ESÓFAGO​.

● Cuando este paciente ingiera sus alimentos, se espera que al momento de pasar el bolo alimenticio por
el esfínter esofágico superior, la presión intraesofágica disminuya en: ​EL CARDIAS.
CASO 3

Paciente de 54 años con antecedentes de alcoholismo, gastritis crónica, tabaquismo pesado,


obesidad, cálculos biliares y cirrosis, es llevado a la emergencia por dolor abdominal en
epigastrio irradiado a la espalda y trastorno del sensorio.
Al examen físico: presión arterial 85/50 mmHg, frecuencia cardíaca 100 latidos/min, frecuencia
respiratoria 18 x minuto, temperatura axilar 36°C.
Conjuntivas pálidas, escleras ictéricas nevus arácnidos en tronco, distensión abdominal
marcada, cabeza de medusa, matidez desplazable en ambos flancos e hipogastrio, dolor a la
palpación de abdomen.
Tiempo de protrombina: 24 seg (testigo: 13 seg); TPT: 38 seg, glicemia: 165 mg/dL, uremia: 20
mg/dL, ASAT: 76 UI/L, ALAT: 22 UI/L, albumina: 2,5 g/dL, bilirrubina total: 2,6 mg/dL,
bilirrubina directa: 1,4 mg/dL, amilasa sérica 4000 U/L.

● Un efecto secundario en el estómago por la acción de la secretina es:

- Menor actividad de la pepsina

● Considerando que el paciente sufre de gastritis, se puede decir que la secreción de ácido por la
mucosa gástrica:

- Involucra transporte activo de Hidrogeniones

● En cuanto a la gastritis de este paciente, se encontró que era producida por la bacteria
Helicobacter pylori. Esta bacteria sobrevive en el medio ácido del estómago gracias a:

- Ureasa

● La bilirrubina directa aumentada en cirrosis hepática se excreta en la orina debido a:

- Ser hidrosoluble

● Paciente de 42 años con adenocarcinoma ductular. La TC ha demostrado claramente que el


tumor está en el cuello del páncreas y que hay un gran vaso ocluido. ¿Cuál de los siguientes
vasos estaría más probablemente obstruido?

- Vena porta.

● En relación a la histología hepática, marque lo correcto:


- La zona 3 se encuentra más cerca a la vena central lobulillar
CASO 4

Mujer de 83 años acude a emergencia por dolor abdominal desde hace 4 días,
localizado en epigastrio, irradiado a ambos hipocondrios, nauseas, vómitos y
distensión abdominal; tiene antecedente de cardiopatía hipertensiva, diabetes
mellitus tipo II y fibrilación auricular. Refiere deposiciones diarreicas muco
sanguinolentas hace 1 día. Los exámenes iniciales muestran PA: 110/60 mmHg, FC:
110/mn, leucocitos: 17800, neutrófilos de 93%. TAC abdómino pélvica se observa
oclusión completa de arteria mesentérica superior por trombo asociado a placa
ateromatosa.
● En esta paciente, ¿cuál de las siguientes sustancias no tendrá una considerable
disminución en su absorción? (marque la mejor respuesta):

- Calcio

● El mecanismo de la diarrea muco sanguinolenta que presenta la paciente, puede mejor definido
como de tipo (marque la mejor respuesta):}

- Exudativa

● Considerando que se ha comprometido el íleon distal, entre otras áreas, la atrofia o


descamación del epitelio de superficie explicaría cuál de los signos o síntomas de la paciente
(marque la mejor respuesta):

- Diarreas mucosanguinolentas

● Producto de esta isquemia, la expresión de cuál de las siguientes enzimas se vería


notablemente disminuida. Marque la mejor respuesta:

- Enteroquinasa

● Durante la cirugía, el cirujano observó que además la paciente tenía divertículos en el sigma. Se
sabe que estos divertículos:

- Se pueden asociar a estreñimiento crónico

● Después de remover quirúrgicamente el territorio gastrointestinal afectado, estaremos seguros


que no quedará en la paciente la siguiente estructura:
- Placas de peyer
● Durante la cirugía, se observa que el colon transverso no está muy afectado por la isquemia,
esto se debe a la anastomosis de las arterias:
- Cólica media e izquierda
● Durante la cirugía, el cirujano observó que además la paciente tenía divertículos en el sigma. Se
sabe que estos divertículos:
- Se pueden asociar a estreñimiento crónico
● En la paciente, el flujo sanguíneo de cuál de las arterias pancreáticas estará afectada:
- Pancreaticoduodenal posteroinferior

● Una mujer de 43 años dolor en hipocondrio derecho e ictericia. En la ecografía se evidencia cálculos
biliares. Estos cálculos lo más probable es que se encuentren localizados en: ​CONDUCTO
COLÉDOCO.

● Si existe un fármaco de acción colerética, se asume que se refiere a que potencia o estimula la:
RECIRCULACIÓN DE SALES BILIARES.

● Paciente con tumor neutro endocrino productor de secretina, debido o cual se puede esperar que su
secreción pancreática, comparada con la de una persona sana en estado de bajo fluido, tenga una
concentración de: ​BICARBONATO AUMENTADA
● Por cuál de las siguientes células es secretada principalmente la pro enzima procarboxipeptidasa:
ACINARES DEL PÁNCREAS.

● En relación a la secreción biliar y su composición, marque lo correcto: ​LAS SALES BILIARES


CORRESPONDEN AL 50% DE SU COMPOSICIÓN .

● El triángulo hepatocístico es importante de conocer en una colecistectomía, debido que tiene:


ARTERIA CÍSTICA.

● Durante una endoscopia, se ingresa en la papila mayor en la segunda porción del duodeno, y dentro de
la vía biliar extrahepatica, se toma una muestra de secreción biliar. Dicha muestra debe contener ácido:
QUENODESOXICÓLICO​.

● La internalización de proteínas o fragmentos de proteínas hacia el intracelular, es característico de :


CÉLULAS M
● Las ramas más pequeñas del árbol biliar son: ​CANALÍCULOS BILIARES.
● Con respecto a las lesiones y enfermedades de la boca, marque lo correcto: ​LA ERITROPLASIA DEBE
SER BIOPSIADA.
● Respecto a la motilidad del colon, marque lo correcto: ​Con la distención del estómago, suelen aparecer
movimientos en masa
● En un paciente hipertenso con tratamiento a base de beta bloqueadores, la producción de saliva se
espera que: ​DISMINUYA
● La saliva puede tener una variedad de electrolitos en su composición. Entre ellos el cloro, respecto al
cual se puede afirmar: ​SU CONCENTRACIÓN NO LLEGA A SER TAN ALTA O EN EL PLASMA.
● ESTIMULA LA PRODUCCIÓN DE SALIVA: ​VASODILATACIÓN PERIGLANDULAR.
● DURANTE EL SUEÑO LA COCENTRACIÓN DE BICARBONATO EN LA SALIVA: ​DISMINUYE​ .
● Un paciente hipertenso está tomando un medicamento bloqueador de receptores alfa-1 adrenérgicos
(prazosina), y como efecto secundario se queja de: ​HIPOSALIVACIÓN​.
● La atropina es el tratamiento para la intoxicación por órganos fosforados. Se administra hasta alcanzar
niveles terapéuticos. Un buen indicador que la atropina ya está en niveles terapéuticos es: ​SEQUEDAD
DE LA MUCOSA ORAL.
● En cuanto a la saliva, marque lo correcto: ​LA AMILASA CUMPLE FUNCIÓN DIGESTIVA.
● Con respecto a la saliva: ​EL SISTEMA SIMPÁTICO ESTIMULA SU SECRECIÓN.
● La secreción de agua y bicarbonato por el páncreas exocrino se da básicamente en la fase:
- Intestinal
● En relación a la secreción biliar y su composición, marque lo correcto:
- Las sales biliares corresponden al 50% de su composición
● Si existe un fármaco con acción colerética, se asume que se refiere a que potencia o estimula la:
- Recirculación de sales biliares
● Hombre de 65 años con isquemia intestinal leve por oclusión aterosclerótica de la arteria mesentérica
superior, pero la irrigación sanguínea colateral ha retrasado el inicio de la necrosis. ¿Qué vasos
ofrecen colaterales entre el tronco celíaco y la arteria mesentérica superior?
- Pancreaticoduodenal superior e inferior.
● Una mujer de 43 años dolor en hipocondrio derecho e ictericia. En la ecografía se evidencia cálculos
biliares. Estos cálculos lo más probable es que se encuentren localizados en:
- Conducto colédoco
● Cuando el alimento se encuentra en el estómago, se produce la liberación de enzimas pancreáticas
básicamente debido a la acción de:
- Vago
● Paciente con tumor neuroendocrino productor de secretina, debido a lo cual se puede esperar que su
secreción pancreática, comparada con la de una persona sana en estado de bajo flujo, tenga una
concentración de:
- bicarbonato aumentada
● ¿Por cuál de las siguientes células es secretada principalmente la pro enzima procarboxipeptidasa?
- Acinares del páncreas
● Dentro de las funciones de las células de Ito, marque lo incorrecto:
- Pueden fagocitar patógenos y actúan como presentadoras de antígeno
● Paciente de 42 años con adenocarcinoma ductular. La TC ha demostrado claramente que el tumor está
en el cuello del páncreas y que hay un gran vaso ocluido. ¿Cuál de los siguientes vasos estaría más
probablemente obstruido?
- Vena porta.
● Respecto a la anatomía del estómago, marque lo correcto:
- la arteria gástrica derecha nace de la arteria hepática común
● Con respecto a las lesiones y enfermedades de la boca, marque lo correcto:
- a eritroplasia debe ser biopsiada
● La secreción de saliva es importante en la fisiología digestiva. Su concentración de potasio llega a ser
menor que la del plasma cuando su secreción tiene un flujo:
- Nunca
● Para protegerse del entorno ácido, el Helicobacter pylori se autogenera un entorno de pH menos ácido
alrededor s​uyo, gracias a una enzima que alcaliniza su entorno local mediante la conversión de:
- Urea en NH3
● Durante el sueño, la concentración de bicarbonato en la saliva:
- Disminuye
● Respecto a la motilidad del colon, marque lo correcto:
- Con la distensión del estómago, suelen aparecer movimientos en masa
● Respecto a las enfermedades del esófago, marque lo correcto:
- el diagnóstico diferencial de la acalasia es la enfermedad de Chagas esofágica
● El omeprazol actúa sobre la membrana _____________ de la célula ____________
- apical / parietal
● Durante el ataque con gas sarín (bloqueador de la acetilcolinesterasa) en el metro de Tokio, en 1995, el
personal de salud notó que los pacientes afectados presentaban:
- Hipersalivación
● Un paciente hipertenso está tomando un medicamento bloqueador de receptores alfa-1 adrenérgicos
(prazosina), y como efecto secundario se queja de:
- Hiposalivación
● La arteria palatina menor irriga un área de la boca cuyo epitelio es:
- Escamoso no queratinizado
● Producto de la alimentación, se producen diversas sustancias peptídicas, cininas y bradicininas, las
cuales permiten que:
- El flujo sanguíneo intestinal aumente hasta 8 veces
● En relación a los principios de motilidad, marque lo incorrecto:
- Las dos terceras partes proximales del esófago están conformadas por músculo
esquelético
● El crecimiento de un adenocarcinoma de cuello de páncreas puede comprometer la pared gástrica por
continuidad. ¿Qué parte del estómago estaría comprometido con mayor probabilidad
- Pared posterior del antro
En un paciente con apendicitis aguda, la sensación de dolor producido por esta inflamación es llevada
por el nervio:
- Esplácnico menor
Para poder morder una manzana, es necesario usar el siguiente músculo:
- Milohiodeo
El dolor periumbilical o epigástrico en el inicio de una apendicitis aguda se debe a:
- Estímulo del sistema simpático.
El nacimiento de la arteria mesentérica superior se puede encontrar en cuál de los cuadrantes
abdominales:
- Epigastrio
El uso de Ranitidina bloquea el receptor H2 de la histamina en las células parietales. La
histamina llega a estas células por:
- Difusión
Marque la respuesta correcta en relación a la gastrina:
- Las células G son las productoras y se encuentran principalmente en el antro gástrico
HOLA PUTAS YA SABEN CONTROL F
Sistema
Digestivo
1. Al examinar a un paciente, usted encuentra dolor localizado en fosa iliaca derecha y diagnostica apendicitis.
En este paciente, usted puede inferir:
El peritoneo parietal regional está afectado
2. La motilidad intestinal es estimulada principalmente por el:
Plexo de Auerbach
3. Durante el vómito, ¿el contenido gástrico tiene que pasar necesariamente por cuál estructura para llegar al
esófago? Marque la mejor respuesta:
Cardias
4. Respecto a la anatomía del estómago, marque lo correcto:
El fondo gástrico forma la curvatura mayor
5. Marque la respuesta incorrecta:
En todo el tubo digestivo, se observa dos capas de muscular propia: circular interna y longitudinal externa
6. Paciente se queja de dolor en hipocondrio derecho, pero superficialmente. El dermatoma relacionado es

A
(marque la mejor respuesta):
T9
7. Dentro de las funciones del abdomen, se encuentra la defecación y micción, en las cuales la presión intra

EK
abdominal debe:
Aumentar
8. Un alumno de medicina decide hacerse un piercing en el ombligo. Al realizarle el procedimiento, sangra
ligeramente. Esta sangre proviene de la arteria (marque la mejor respuesta)
Epigástrica inferior
OT
9. Señale la respuesta correcta:
El apéndice cecal sólo tiene serosa
10. Paciente mujer es traída a emergencia por sufrir una herida contuso penetrante por cuchillo realizada por
su esposo en un ataque de celos. Se observa herida en flanco izquierdo. Esta solución de continuidad ha
comprometido varios músculos de la pared abdominal, excepto:
NC

Recto abdominal
11. Paciente con herida por proyectil por arma de fuego, con herida de ingreso en región paraumbilical. Entre
las estructuras que usted está seguro que debe haberse lesionado es:
Omento mayor
12. En la evaluación de una tomografía abdominal, el interno observa un aneurisma en una arteria que se dirige
BA

al riñón derecho. Con seguridad se puede afirmar que está a nivel de la vértebra:
L1
13. Paciente joven es traído a emergencia con abdomen agudo quirúrgico debido a herida contuso penetrante
por verduguillo (alambre grueso con punta aguzada) recibida en una pelea después de un partido de fútbol.
Se observa herida en Hipocondrio Izquierdo. El órgano que debe estar sangrando y produciendo
hemoperitoneo es (marque la mejor respuesta):
Bazo
14. Marque el órgano que se encuentra más distal en el tubo digestivo.
Ciego
15. La peristalsis o peristaltismo hace referencia a:
Motilidad para movilizar el alimento de proximal a distal.
16. Marque la respuesta incorrecta:
En todo el tubo digestivo, se observa dos capas de muscular propia: circular interna y longitudinal externa
17. Paciente con vólvulo del colon sigmoides. La necrosis de ese segmento del colon se produce por una
alteración en la irrigación de la arteria: Mesentérica inferior
18. Marque el órgano que se considera retroperitoneal: Parte de la vía biliar
19. Un alumno de medicina decide hacerse un piercing en el ombligo. Al realizarle el procedimiento, sangra
ligeramente. Esta sangre proviene de la arteria (marque la mejor respuesta): Epigástrica inferior
20. Paciente tiene una úlcera sangrante en el segundo tercio del Yeyuno. La arteria de la cual proviene la sangre
arterial para dicha zona es la arteria: Mesentérica superior
21. Es inervado por aferentes somáticas: Peritoneo parietal
22. Paciente de 24 años con dolor abdominal tipo cólico intenso en mesogastrio. Según sus conocimientos de
macroestructura, el origen del dolor puede ser ….: Íleon
23. Paciente con hipoglucemia secundaria a un insulinoma (tumor neuroendócrino productor de insulina). El
órgano donde mayor probabilidad ha crecido este tumor es: retroperitoneal
24. Al evaluar una tomografía abdominal, el médico asistente le pide al interno de la UPC que encuentre la
imagen con el corte a nivel de L1. El interno sabiamente busca el …….. para ubicar la vértebra L1: Cuello del
páncreas
25. En la inspiración, la pared abdominal debe …. para ….. : relajarse disminuir presión intra torácica
26. Ligamento hepatogástrico une el ….. con el …… y forma la entrada al …… : Hígado Estómago Orificio omental
27. Al retirar completamente el mesenterio de un órgano, el mismo se vería afectado principalmente en su:
Irrigación
28. La estructura que fija órganos principalmente a la pared posterior abdominal se denomina: Mesenterio
29. Cuál de las siguientes estructuras no tiene vasos sanguíneos: Epitelio intestinal
30. Al iniciar la digestión, aumenta el consumo de oxígeno por la mucosa. Esto conlleva a una hipoxia local, lo
cual hace que se libere …., el cual produce vasodilatación: adenosina
31. Sustancia que inhibe la secreción y la motilidad del estómago prolongando el tiempo de digestión: péptido
insulinotrópico dependiente de la glucosa (GIP)
32. Marque lo correcto: La hernia fisiológica se produce en la sexta semana y es la salida temporal de asas
intestinales a través del colon umbilical
33. Marque la respuesta correcta en relación a la gastrina: Las células G son las productoras y se encuentran

A
principalmente en el antro gástrico.
34. El consumir caramelos indirectamente activa la vía: POMC/CART
35. ¿En qué capa se encuentra la alteración principal en el Hirschsprung o megacolon agangliónico?: Muscular

EK
propia
36. Con respecto a las ondas lentas, marque la afirmación correcta: Son contracciones rítmicas espontáneas
37. El uso de Ranitidina bloquea el receptor H2 de la histamina en las células parietales. La histamina llega a
estas células por: Difusión
OT
38. La triada sintomática: vómitos explosivos post-prandiales, movimientos peristálticos epigástricos visibles
de izquierda a derecha y nódulos palpable epigástrico subcostal derecho, pertenecen a: Estenosis congénita
hipertrófica del píloro
39. Durante una cirugía oncológica, ¿la extirpación de cuál de los siguientes órganos se vería comprometida
por la presencia de adventicia?: Recto
NC

40. En cuanto a los reflejos gastrointestinales, un reflejo que estimula el tránsito intestinal es el reflejo:
Gastrocólico
41. El ligamento falciforme divide al hígado en dos lóbulos derecho e izquierdo. Embriológicamente deriva del:
Mesenterio ventral
42. La presencia de atresias y estenosis duodenales se deben básicamente a una: Falta de recanalización
BA

43. Estudiante de medicina de 20 años, se ha amanecido estudiando para su examen de Sistema Digestivo. No
ha probado alimento desde la cena, por lo que se puede afirmar que la motilidad de esta persona está
siendo regulada por: Motilina
44. Paciente con disminución del apetito marcada asociada a cáncer terminal, para promover la ingesta de
alimentos se podría usar análogos de: Endorfinas
45. Las ondas lentas se producen por la apertura cíclica de canales de: Calcio
46. La forma más común de atresia esofágica contiene: Estenosis proximal del esófago más fístula
traqueoesofágica distal
47. Al deglutir un bolo alimenticio, es lógico suponer que al pasar por el esofago haya un mayor consumo de
oxigeno en la pared del tercio: Proximal
48. Paciente que come entera una pizza familiar de chorizo y queso. Es posible esperar que debido a la cantidad
de alimento ingerida, las ondas lentas hayan: sufrido ninguna alteración en su frecuencia.
49. La hernia fisiológica se produce dentro de: cordón umbilical
50. El crecimiento de un adenocarcinoma de páncreas compromete la pared gástrica por contigüidad ¿Que
parte del estómago se esperaría esté comprometido?: Pared posterior del antro
51. Estimula la producción de saliva: Vasodilatación periglandular
52. Durante la secreción de saliva, es de esperarse que las concentraciones de ….. y …… disminuyan al disminuir
el flujo: Sodio Bicarbonato
53. Con respecto a la secreción gástrica de HCI: a mayor secreción de HCI en el lumen gástrico, mayor pH en la
sangre venosa gástrica.
54. Respecto a las enfermedades del esófago, marque lo correcto: el diagnóstico diferencial de la acalasia es la
enfermedad de Chagas esofágica
55. Con respecto a las lesiones y enfermedades de la boca, marque lo correcto: la eritroplasia debe ser
biopsiada
56. Respecto a las glándulas salivales, marque lo incorrecto:
a. la glándula sublingual tiene forma de garfio
b. la glándula sublingual drena a través del conducto de wharton
c. la glándula parótida produce secreción serosa
d. la glandula parotida drena a través del conducto de Stenon
57. El omeprazol actúa sobre la membrana ….. de la célula ….. : apical/parietal
58. Durante el sueño, la concentración de bicarbonato en la saliva: Disminuye
59. Durante el ataque con gas sarín (bloqueador de la acetilcolinesterasa) en el metro de Tokio, en 1995, el
personal de salud noto que los pacientes afectados presentaban: Hipersalivación
60. La célula mucosa del cuello gástrico produce: Moco

Parcial 2020-01 (1)

1. Al evaluar la orofaringe de un paciente, el médico le solicita que abra la boca, saque la lengua y diga a. Al
hacer esta maniobra, nota que el paladar se desvía hacia la derecha, lo cual le hace sospechar que el
paciente sufre de una lesión del nervio craneal: X contralateral
2. Un bolo alimenticio grande y poco masticado se atasca en el esofago, esto ocasiona una sensación de

A
dolor que es transmitida por los nervios: esplácnicos
3. Para realizar el movimiento mecánico de abrir la boca, primero se necesita: fijar el hueso hioides

EK
4. ¿Cual de las siguientes alternativas se define como la protrusión directa del contenido abdominal a la
cavidad amniótica por un defecto de la pared corporal?: Gastrosquisis
5. Un paciente requiere que se le coloque una sonda de alimentación directamente al estómago
(gastrostomía), el cirujano deberá hacer una incisión en la piel del abdomen ¿cuál de las siguientes raíces
nerviosas debe ser anestesiada para este procedimiento? T8
OT
6. En un paciente de 43 años con tumor carcinoide de páncreas productor de gastrina (Síndrome de
Zollinger-Ellison) se puede esperar encontrar una potenciación del reflejo: gastrocólico
7. El mecanismo de la defecación incluye la participación de diversas estructuras ¿cuál de las siguientes
alternativas es correcta?: Puede ser mediado por un reflejo intrínseco
8. Cuando el contenido del estómago ingresa al duodeno, uno de los reflejos que inhiben el vaciamiento
NC

gástrico es a través del: Sistema nervioso mientérico


9. Durante la masticación, gran parte del proceso masticatorio se debe a: El reflejo masticatorio
10. Las glándulas salivales tienen conductos para la excreción de la saliva, las glándulas …… drenan en las
carúnculas sublinguales. RPTA: Sublinguales
11. Los diferentes segmentos del tubo digestivo son susceptibles de reflejos y movimientos según su
BA

contenido. Si colocamos mediante una sonda un bolo alimenticio directamente en el tercio medio del
esofago: se producira ondas secundarias
12. En una persona si enfermedad se espera que el tránsito intestinal se vea disminuido cuando se presenta el
reflejo: Doloroso
13. El divertículo de Meckel es una anomalía congénita que ocurre por la persistencia del conducto vitelino y
da origen a una estructura sacular, el cual se encuentra en el: borde antimesentérico
14. Si al intubar a un paciente, por error se ingresa el tubo endotraqueal en el esofago y se insufla el
manguito endotraqueal (globo TET), la dilatación de este manguito generará: múltiples ondas secundarias
15. El orificio omental, o hiato de Winslow, se encuentra limitado por el ligamento: hepatoduodenal
16. Paciente de 24 años acude a consulta externa por presentar una fístula oronasal (comunicación entre la
cavidad oral y la cavidad nasal). Esta fístula es una consecuencia tardía de la lesión de un vaso sanguíneo
por el antecedente de haber sido operado de paladar hendido en los primeros años de vida,
aparentemente en una campaña gratuita de corrección de paladar fisurado. ¿Cual de las arterias palatinas
podría haberse lesionado durante esa cirugía?: Mayor
17. Dentro de las anomalías congénitas se puede presentar un tejido pancreático accesorio ¿cuál es la
ubicación más común de este tejido?: Estómago
18. Una recién nacida es evaluada por el neonatólogo y evidencia que el canal anal está completamente
cerrado. Este problema se debe probablemente a una anomalía en el desarrollo de: la membrana cloacal
19. En la digestión de los alimentos, la hormona __________ se libera frente a la presencia de péptidos y
monoglicéridos, y tiene un efecto marcado en la disminución del vaciamiento gástrico →
colecistoquinina
20. Dentro de las anomalías congénitas se puede presentar un tejido pancreático accesorio ¿Cuál es la
ubicación más común de este tejido? → Estómago
21. Los catadores de vino tienen una habilidad increíble al momento de separar los sabores. Este aumento de
la sensibilidad gustativa debido a una mayor cantidad de papilas linguales y de corpúsculos gustativos se
conoce como: hipergeusia
22. En muchos países se usa el suplemento de flúor en el agua potable o los dentífricos, con el fin de hacer el
esmalte más resistente a la desmineralización inducida directamente por: el ácido
23. Durante el desarrollo de la región cloacal, una cuña de mesodermo ubicado entre el alantoides y el
intestino posterior vendrá a formar el: tabique urorrectal
24. En una persona sana, el momento adecuado para encontrar los mayores niveles de grelina en sangre
sería: antes de comer
25. El duodeno está constituido por el segmento terminal del intestino anterior y el segmento proximal del
intestino medio ¿Cuál de las siguientes alternativas describe mejor este lugar de unión entre los dos
intestinos?
Distal al origen de la yema hepática

A
26. En una persona sana, el uso de atropina producirá a nivel del estómago: Aumentará el pH del estómago
27. En una persona sana, el consumo de leche produce indirectamente → Inhibición del vaciamiento

EK
gástrico
28. La motilidad del colon es importante y lenta comparada con la del intestino delgado. Los movimientos en
masa ocasionan la: distensión rectal
29. Con respecto a la motilidad gástrica, los potenciales de acción disminuyen en frecuencia por efecto de:
el péptido insulinotrópico dependiente de glucosa
30. La sensación del gusto depende de la presencia de papilas gustativas en la lengua, de las cuales, algunas
OT
de ellas tienen un surco terminal por donde drenan unas glándulas salivales linguales (llamadas glándulas
de von Ebner). Esta descripción se refiere a las papilas: circunvaladas
31. Dentro de la estructura de los dientes, la parte del diente cubierta por esmalte y que se puede ver
mediante la inspección visual de la boca se denomina → corona clínica
NC

32. En el conducto anal se encuentra la unión entre las regiones del endodermo y el ectodermo, esta unión se
evidencia al observar: la línea pectínea
33. La sensación del gusto depende de la presencia de papilas gustativas en la lengua, las cuales tienen
corpúsculos gustativos conteniendo células neuroepiteliales sensoriales. Estas células neuroepiteliales
pueden ser dañadas fácilmente, por suerte, su tiempo de recambio es de alrededor de: 10 días
BA

34. En un varón de 47 años con sección medular a nivel de T6 debido a un accidente automovilístico, sus
terapeutas han desarrollado un mecanismo para distender el recto e iniciar el reflejo rectoesfinteriano, lo
cual producirá la contracción de: la pared del recto
35. Durante un experimento, se insufla rápidamente dos litros de agua en un globo colocado dentro
del estómago de un voluntario. ¿cuál de las siguientes situaciones del músculo liso será
consecuencia directa de este cambio de volumen en el estómago? → Despolarización
36. Paciente de 56 años con accidente cerebrovascular reciente. En la resonancia se observa daño de los
núcleos laterales del hipotálamo. Por este motivo es muy probable que el paciente sufra de: inanición
37. Paciente con insuficiencia mitral moderada a severa, con aumento de volumen de la aurícula izquierda,
esta condición tendrá como consecuencia a nivel del sistema digestivo: la disfagia a sólidos
38. El inicio de la fase faríngea de la deglución se debe a estímulos sensitivos que viajan por el nervio craneal:
V
39. El mesocolon transverso se origina en: la pared posterior del abdomen
40. El esofago en su microestructura tiene basicamente adventicia, a excepción de la región distal, donde
tiene serosa, específicamente a partir del nivel de: T10
41. En un paciente con arcadas, se debe considerar que durante la ocurrencia de dichas arcadas, debemos
encontrar contenido gástrico en: Tórax
42. La reabsorción de Sodio y Cloro en las glándulas salivales se da principalmente en el: Conducto estriado
43. Para que se puedan digerir las grasas, es preferible que primero sean emulsificadas. La hormona que
estimula la liberación de las sustancias emulsificadoras es: CCK
44. Paciente de 13 días de vida con vómitos explosivos a las 2 horas después de lactar. Al examen físico se
palpa la oliva pilórica ¿cual es el nervio cuyos filetes dan inervación eferente a la estructura afectada?:
Vago
45. La localización de la vesícula biliar con respecto al lóbulo cuadrado es: Lateral
46. La triada portal está constituida por el conducto biliar y la arteria hepática y una pequeña rama de la
vena: Porta
47. Al ingerir una cantidad de glucosa por vía oral, esta es interiorizada en las células del organismo más
rápido que si esa misma cantidad hubiese sido administrada por vía endovenosa. Este fenómeno sucede
gracias a una sustancia secretada por las células: K
48. Paciente de 62 años con vólvulo de intestino delgado e isquemia intestinal. ¿Qué estructura se utiliza
como punto de referencia para determinar la posición de la unión duodenoyeyunal? : Ligamento
suspensorio del duodeno (de Treitz)
49. En un paciente con Zollinger Ellison, usted esperaría encontrar: Esteatorrea
50. La motilidad intestinal es estimulada por: Colecistoquinina y gastrina
51. La onda peristática secundaria del esofago en la deglución, es producida por: Plexo mientérico esofágico
52. Paciente con parálisis bilateral del nervio hipogloso, el unico musculo de la lengua que conservará su

A
movimiento es el: palatogloso
53. ¿Cual de las siguientes sustancias tiene mayor concentración de la saliva comparado con su concentración
plasmática? Potasio

EK
54. ¿Cual de las siguientes estimula la secreción enzimática exocrina del páncreas?: Colecistoquinina
55. Al comer un pollo a la brasa, con papas fritas y ensalada, la sustancia que estimula la liberación de HCI en
el estómago es: Bombesina
56. Marque lo correcto con respecto a Esófago de Barret: Se relaciona con reflujo gastroesofágico
OT
ECU 1
Estudiante de medicina de la UPC de 21 años sufre de gastritis aguda ocasionada por comer en lugares poco
higiénicos. Suele consumir caramelos ( chupar ) mientras está en clase hasta la tarde. Toma gaseosas regularmente
(carbohidratos 46%, sodio 53%). También toma regular cantidad de leche (grasa 35%, lactosa 35%, proteínas 30%),
NC

pues le calma un poco el dolor el ardor que siente por la gastritis. Incluso, cuando puede, se toma dos vasos de
agua fría para calmar las molestias. Ha decidido ir al médico para tratarse pues ya no soporta el dolor, el cual está
seguro que los síntomas se deben a una elevada producción de ácido clorhídrico en el estómago, y por ello le ha
recetado Ranitidina (antihistamínico), con lo que siente mejoría.

El uso de atropina en este paciente:


BA

- Aumentará el pH del estómago

Entre las sustancias cerebrales que producen ansiedad está la serotonina, la cual también tiene acción:
- Anorexigénica

El consumo de dos vasos de agua seguidos agua generará indirectamente un aumento en la liberación de:
- Ácido clorhídrico

En este paciente con gastritis aguda debida a una alta producción de ácido clorhídrico, sería lógico esperar que el
píloro tenga un tono muscular:
- Aumentado

El consumo de leche produce directamente un aumento de los niveles séricos de la hormona:


- Colecistoquinina (CCK)

El consumo de leche produce directamente un aumento de los niveles séricos de la hormona:


- Gastrina

ECU 2
Niño de sexo masculino de 2 años de edad, sufre de estreñimiento desde el nacimiento (1 deposición cada 3-4
días). Madre menciona que le estimula la defecación con un termómetro rectal, y continuo uso de enemas y
laxantes. Desde hace 6 meses comienza con vómitos postprandiales. Los síntomas aumentan en frecuencia y
magnitud y están en relación con los episodios de estreñimiento. No refiere fiebre, tos, diarrea ni lesiones
cutáneas. Al examen físico presenta regular estado general, luce deshidratado. Abdomen distendido, blando,
depresible e indoloro. No se palpan masas abdominales. Se permeabiliza el canal anal con termómetro rectal,
encontrando cierta resistencia. Salida de material fecal mal oliente en regular cantidad. Exámenes de laboratorio:
hemograma normal. Signos inflamatorios de fase aguda negativos. Alcalosis metabólica leve en sangre venosa.
Radiografía con enema baritado muestra recto y colon sigmoides dilatados (megacolon). Biopsia profunda:
ausencia de células ganglionares en la muestra enviada. Se realiza cirugía correctiva.

El contenido fecal se detiene en la zona inmediatamente proximal a la zona donde hay una menor presencia de:
- Péptido intestinal vasoactivo

En cuanto a los reflejos gastrocólico y gastroduodenal en este paciente, indique lo correcto:


- Se pueden considerar reflejos vago-vagales

En este paciente se considera que está abolido el reflejo:


- Rectoesfinteriano

Debido al acúmulo de material fecal en todo el marco colónico, y a la irritación química asociada, el peristaltismo
del íleon distal se debe encontrar:
- Inhibido

Es un reflejo propio de la pared intestinal:


- Peristaltismo

A
A diferencia de las arcadas, los vómitos presentan apertura de:
- Esfínter esofágico superior

EK
1. Un niño de 2 años es llevado a la consulta por diarrea persistente y edema de las
OT
extremidades, además falta de crecimiento y desarrollo en relación a su edad. Los
análisis de sangre revelan que tiene concentración plasmática baja de proteínas
(hipoproteinemia). Durante la endoscopía duodenal, se coloca colecistocinina (CCK)
endovenosa y se recoge muestras del líquido duodenal; el resultado del líquido
NC

confirma incapacidad para hidrolizar proteínas a un pH neutro, esta situación mejora


al añadir una pequeña cantidad de tripsina. El paciente probablemente esté sufriendo
la falta congénita de
-Enterocinasa
2. Experimentalmente se incrementa la velocidad de la secreción salival con una
BA

sustancia, el análisis de la composición de esta saliva obtenida se espera


encontrar…………..
-Disminución de concentración de potasio
3. Paciente varón de 46 años soltero, consulta por odinofagia y bajo de peso, tiene
antecedente de tuberculosis desde hace 3 meses y es fumador crónico (10 cigarrillos
por día); al evaluar la cavidad oral se identifica lesión blanquecina en el dorso de la
lengua y paladar blando, las lesiones se desprenden con el baja lengua dejando una
base eritematosa. Esta lesión corresponde probablemente a
……………………….…..
-Candidiasis oral
4. Paciente mujer de 35 años acude a consulta por sensación de sequedad y lesiones
en cavidad oral. Al examen se observa atrofia de la mucosa, fisuras y úlceras; nota
además sequedad e irritación de la córnea y aumento del tamaño de las glándulas
parotídeas. Su diagnóstico más probable es artritis reumatoide; el hallazgo más
probable en una biopsia de glándula parótida es……..….
-Gran infiltración de linfocitos y células plasmáticas
5. Un paciente con anemia acude con su médico quejándose de episodios frecuentes
de gastroenteritis. Un análisis de sangre revela anticuerpos circulantes dirigidos
contra células parietales gástricas. Su anemia es atribuible a la hiposecreción de
-Factor intrínseco
6. Dos estudiantes deciden tomar un receso para comer una hamburguesa a la hora
del almuerzo. Antes de llegar a la cafetería, impulsos nerviosos provenientes del
complejo vagal dorsal iniciarán la secreción de ácido gástrico por la liberación de
…………………….. desde el sistema nervioso entérico.
-GRP
7. Un niño de cuatro años de edad es llevado a la consulta por cuadros diarreicos
frecuentes caracterizados por heces pálidas, voluminosas y fétidas, presenta bajo
peso y talla. Se mide la concentración de cloruro en el sudor y se encuentra que sus
valores son muy elevados. La alteración más importante a nivel de células ductales
del páncreas tiene relación directa con la conductancia de…………
-Cloro
8. Una mujer de 50 años de edad que sufrió durante varios años resequedad de los
ojos debida a producción inadecuada de lágrimas es enviada con un
gastroenterólogo para evaluación de pirosis crónica. El examen endoscópico
revela erosiones y tejido cicatrizal en la parte distal del esófago justo por arriba del
esfínter esofágico inferior. Las lesiones pueden atribuirse a la disminución de uno
de los siguientes componentes salivales:

A
-Bicarbonato

EK
9. Se evalúa los valores séricos de las siguientes sustancias a un paciente con
enfermedad hepática terminal; en este paciente se espera encontrar la combinación
con la letra …………
-disminuida, aumentada, disminuida
OT
10. Una mujer de 35 años de edad HIV positiva, se presenta al médico con dolor
abdominal en cuadrante superior derecho e ictericia. La paciente refiere haber tenido
múltiples episodios de ictericia durante los últimos 10 años. Los exámenes para
determinar hepatitis viral, dieron positivos para Hepatitis B, siendo catalogado el caso
como hepatitis crónica con alteración funcional. En un examen de sangre ¿cuál de los
NC

siguientes parámetros está disminuido?


-Albúmina
11. En el reflejo peristáltico del intestino delgado, uno de los siguientes eventos sucede
en la porción oral del bolo alimenticio…………...
BA

-Acción de acetilcolina en el músculo circular


12. Experimentalmente se coloca una dosis alta de secretina en la luz intestinal
duodenal; como consecuencia de esto, en el jugo pancreático de la misma luz
intestinal se observa la disminución de la concentración de …..………..
-Cl
13. Un varón de 58 años de edad con enfermedad de Crohn severo fue sometido a una
resección ileal. Después de la cirugía este paciente padecerá de esteatorrea, esto se
explica porque …..………..
- La micelas no pueden formarse
14. En un experimento se inserta un balón en el estómago de un voluntario, se infla poco a
poco mientras que se vigilan las presiones intraluminales. Aunque el volumen del
balón aumenta considerablemente, las presiones permanecen constantes. Esta
relación volumen-presión se explica por la liberación local de …………..
-Óxido nítrico y péptido inhibidor vasoactivo
15. La toxina de Vibrio cholerae causa diarrea debido a…….
-El Incremento de la secreción de cloro por las células de la cripta intestinal
16. ¿Cuál de las siguientes alternativas es una característica de la secreción exocrina
del páncreas?
-Tiene una baja concentración de Cl- respecto al plasma
17. Una madre lleva a su hijo de dos años de edad a la sala de urgencias, estresada
porque el niño deglutió una moneda de 10 céntimos mientras la familia cenaba en un
restaurante. El médico observa mediante fluoroscopía que la moneda se halla en el
estómago y asegura a la madre que la moneda se eliminará con las heces. El médico
recomienda utilizar la respuesta fisiológica que permitirá la evacuación de la moneda
del estómago al intestino ………….…..
-Son los movimientos de mezcla y trituración
-. Es provocada por el ayuno
18. Las estructuras en el hígado que permite que los productos metabólicos unidos a
proteínas tengan acceso a las membranas basolaterales de los hepatocitos, son….. -
Las fenestras sinusoidales
19. La composición de la bilis es modificada conforme fluye por los conductillos biliares.
Durante este tránsito se espera que aumente la concentración de…….
-Monómeros de ácido biliar
-Ig A
20. Se mide experimentalmente el contenido gástrico de dos personas. La persona “A”
tiene alto contenido de grasa y la persona “B” tiene un contenido hipertónico ¿Cuál de
las siguientes es correcto respecto al vaciamiento gástrico?
- Hay ralentización del vaciado gástrico en ambos casos

A
21. El examen endoscópico de un paciente con hipertensión portal grave revela venas
tortuosas que sobresalen hacia la luz del esófago. El paciente recibe tratamiento

EK
quirúrgico mediante la colocación de una derivación que conecta la vena porta a la
vena cava. Después de la operación el riesgo de encefalopatía y el
riesgo de sangrado de várices ……………..
-Aumentará/disminuirá
22. Un paciente varón de 18 años de edad acude al médico para sus exámenes de
OT
rutina. Sus resultados de laboratorio muestran un valor de bilirrubina sérica de 4
mg/dl y una bilirrubina directa de 0,3 mg/dl. Las pruebas de función hepática son
normales. La alteración que explica mejor este caso es por la deficiencia de
………………..
NC

-Glucuronil transferasa
23. Un hombre de 57 años de edad es llevado a urgencias con hematemesis masiva
rojo brillante, a su llegada se halla inconciente con PA: 80/40 mm Hg y FC: 124
lat/min. Luce ictérico con presencia de “arañas vasculares en el tórax anterior y
extremidades”, abdomen distendido con signo de oleada positiva. Se encuentra
BA

esplenomegalia y pérdida de la masa muscular en extremidades. La anastomosis


vascular responsable del sangrado en este paciente es
-Vena gástrica izquierda y vena ácigos
24. Un estudiante de medicina está comiendo un plato de comida a base de
champiñones, espárrago y salsa de soya. El sabor umami contenido en todos estos
alimentos actúa a nivel de los botones gustativos estimulando ………………..
-Un receptor acoplado a proteína G
25. Un hombre de 22 años de edad se presenta al médico con una historia de 1 año de
evolución caracterizado por dolor recurrente en fosa iliaca derecha y diarrea.
Manifiesta además pérdida de peso de 8 kg durante este periodo. La colonoscopía
revela múltiples lesiones en el ileon terminal y colon. La biopsia de estas lesiones
revela engrosamiento, inflamación y ulceración de la mucosa. El diagnóstico más
probable en este caso es…….
-Enfermedad de Crohn
26. Varón de 61 años que consulta por dolor retro esternal intenso desde hace 6 horas y
después de vómitos intensos y repetidos; al examen se observa disnea, cianosis,
hipotensión y signos clínicos de shock. La radiografía simple de tórax muestra
neumomediastino. El líquido en el espacio pleural aspirado tiene alta concentración de
amilasa. ¿Cuál de las siguientes alternativas puede explicar este cuadro clínico? -
Rotura espontánea de esófago
27. La secreción del ácido en la célula parietal gástrica se lleva a cabo por una ATPasa
especifica que intercambia hidrogeniones (H+) del citosol por…..
-K +
28. En condiciones normales el ingreso de 600 ml de líquido es el estómago provoca un
aumento de presión intragástrica de unos 12 cm de H2O. Después de una vagotomía
(corte del nervio vago) es de esperar que el ingreso del mismo volumen de líquido
provoque lo siguiente: …………………………………
-Un aumento mayor de la presión
29. Una paciente de 30 años de edad es sometida a una cirugía en oído medio derecho
por un problema de otoesclerosis. Luego de la cirugía refiere alteración en la
percepción de sabores. Al evaluar el caso usted esperaría encontrar……….
-Alteración en la sensación del gusto en los dos tercios anteriores de la
lengua
-Sensación del dolor, tacto y temperatura conservada en toda la lengua

30. ¿Cuál de las siguientes alternativas es correcta?


-Las sales biliares desconjugadas son absorbidas preferentemente en el colon

A
31. En un paciente de 45 años de edad con colestasis biliar, se encuentra una elevación
de los niveles sanguíneos de fosfatasa alcalina hasta 3 veces la cifra normal. ¿Cuál de

EK
las siguientes alternativas estará también elevada como evidencia del daño de la vía
biliar?
-Gamma glutamil transpeptidasa
32. Revisando la angiografía de un hombre de 70 años en estudio por aneurisma de aorta
abdominal el radiólogo informa de la presencia de una oclusión completa de la arteria
OT
mesentérica inferior. El paciente se encuentra completamente asintomático.
¿Cuál de las siguientes arterias se anastomosa a la sistema arterial de la
mesentérica inferior?
-Cólica media
33. Lactante de 3 meses de vida es atendido por presentar diarrea, se administra una
NC

solución de glucosa y electrólitos por vía oral. La proteína de membrana apical que
explica la capacidad de esta solución para proporcionar aporte de glucosa e
hidratación es ………..
-SGLT-1
BA

34. Paciente ha sufrido herida de bala en el abdomen, se le ha tenido que extirpar el


segmento medio y distal del ileon. En este caso la síntesis hepática de sales biliares
estará …..…..
-Incrementada por estímulo de la enzima colesterol 7 alfa hidroxilasa
35. Un varón de 75 años ingresa al consultorio por presentar ictericia marcada de piel y
las escleras. El estudio del paciente mostró que presentaba un tumor que obstruía la
totalidad del conducto hepático común. ¿Cuál de las siguientes estructuras se
encontrará dilatada en este paciente?
-Conductos de Hering
36. En un paciente con insuficiencia renal crónica, el déficit en la absorción de calcio a
nivel del enterocito se debe a lo siguiente:
-No se convierte la 25 hidroxicolecalciferol a 1,25 dihidroxicolecalciferol
37. Varón de 30 años es traído a emergencia por agresión abdominal con arma de fuego
(pistola) y es sometido a laparotomía exploratoria, observándose isquemia del colon
ascendente y parte del colon trasverso ¿la lesión de cuál de las siguientes arterias
explicaría esta isquemia?
d. Mesentérica superior
38. Respecto a las sustancias gastrointestinales que regulan la secreción pancreática;
marque la afirmación correcta:
b. La acetilcolina es capaz de estimular la secreción enzimática y de bicarbonato del
páncreas
39. Ante una lesión del X par craneal, ¿cuál de los siguientes músculos mantiene
conservada su función?:
b. Tensor del velo del paladar
40. Experimentalmente se utiliza atropina (anticolinérgico) para inhibir la secreción de
gastrina, sin embargo la secreción de esta hormona se sigue dando ante estímulos
vagales. Esta situación se explica porque la atropina:
d. No bloquea la acción del péptido GRP

41. Un varón de 50 años es sometido a extirpación del duodeno y parte proximal del
yeyuno. Esta situación ocasionaría la pérdida de las células ……….. , productoras de
………………… que estimula la secreción de bicarbonato por el páncreas.

A
“S” / secretina
42. Se evalúa la expresión de la proteína Agrp en una persona con alteración del apetito; lo

EK
correcto respecto a esta proteína es…..
La mutación del gen que la codifica produce adelgazamiento
43. Juana cae de la bicicleta y se fractura la región anterior del hueso maxilar superior con
compromiso de la fosa incisiva. Al examen físico de la región esperaría encontrar
alteración en la sensibilidad de la encía …………………
OT
palatina anterior
44. Recién nacido es atendido por el neonatólogo y luego entregado a su madre para dar
de lactar; la madre al dar de lactar observa coloración azulada de labios, acompañado
de tos persistente, dificultad respiratoria y distención abdominal. Se le intenta colocar
NC

una sonda nasogástrica pero esta retorna a la cavidad oral en todos los intentos. ¿Cuál
de las siguientes anomalías del desarrollo es el más probable en este caso?
b. Atresia esofágica proximal con fístula traqueoesofágica distal l
45. ¿Cuál de los siguientes mecanismos ocurre durante la defecación?
BA

En la posición de “cuclillas” el músculo puborrectal se halla relajado


46. Un paciente luego de un accidente sufre lesión del piso de la boca, se constata daño
del nervio “cuerda del tímpano”, en este caso se esperaría en
47. contrar disminución de la………………………….… de la lengua
Sensación del gusto en los dos tercios anteriores
48. ¿Cuál de las siguientes afirmaciones es la correcta sobre la gastrina?
Actúa en la célula diana mediante su receptor CCk tipo B
49. Al recibir un paciente con signos de hipovolemia y antecedente de trauma en
abdomen por accidente de tránsito, usted identifica radiológicamente: lesión de
primera vértebra lumbar y signos de lesión en páncreas; durante la cirugía se observó
pobre irrigación de asas intestinales. El vaso afectado es la arteria ……..
c. mesentérica inferior
50. Un paciente sufre de daño a nivel del cuello con lesión muscular en la región de la
faringe. En el examen físico se determina dificultad para la elevación de la faringe y para
el cierre del itsmo de las fauces. En este caso, probablemente esté afectado el músculo:
c. palatofaringeo
51. Varón de 50 años a quien le realizan la curación de la segunda molar de la arcada
superior derecha. En un momento determinado, el paciente acusa de intenso dolor
de la pieza dentaria en tratamiento. La vía aferente del dolor viaja a través del
nervio …………
a. trigémino V2
52. La distención gástrica por los alimentos produce incremento de secreción de HCl
mediante la producción de ………….. que estimula a las célulasvía proteína
………..
a. gastrina / parietal / Gq

53. Un niño de tres años llega a emergencia con disfagia (dificultad para tragar), dolor
retro esternal, salivación y llanto. Se sospecha de ingesta de cuerpo extraño (moneda)
en el esófago; al ser evaluado se constata en una radiografía presencia de cuerpo
extraño a nivel de C6 (6° vértebra cervical). El cuerpo extraño estará suspendido a

A
nivel del estrechamiento producido por………..
c. el músculo cricofaríngeo

EK
54. La triada portal (arteria hepática, vena portal y conducto biliar común) está contenida
en el ligamento …….……… y derivan embriológicamente del ……
a. hepato duodenal / mesenterio ventral
55. Un paciente refiere no percibir algunos sabores, al examen físico se constata
OT
alteración en la percepción de sabores y del dolor en el tercio posterior de la lengua
¿Cuál de los siguientes nervios estará alterada en su función?
c. Glosofaríngeo (IX par)
56. En el caso de un paciente con gastrinoma (tumor productor de gastrina), la presencia
NC

de úlceras duodenales y erosión de la mucosa gástrica, se debe principalmente a…….


c. el exceso de HCl por estímulo de receptores CCK-B en la célula parietal
56. El reflejo entero gástrico se caracterizan por:
d. originarse debido a la distensión duodenal y presencia del quimo ácido
BA

57. Mauricio tiene dificultad para deprimir el paladar y elevar la parte posterior de la lengua.
En este caso estará afectado un músculo, específicamente el músculo
…………….
b. extrínseco – palatogloso
58. En condiciones normales, el ingreso de 600 ml de líquido es el estómago provoca un
aumento de presión intragástrica de unos 12 cm de H2O. Después de una vagotomía (corte
del nervio vago) es de esperar que el ingreso del mismo volumen de líquido ocasione
………………………………… de la presión
intragástrica.
c. un aumento mayor
59. La explicación fisiológica de presentar somnolencia de 30 minutos a 1 hora después
de ingerir alimentos, se explica por: a. Aumento del cloro intraluminal
e. Aumento de la alcalinidad sanguínea
60.Se presenta un paciente, el cual presenta un antecedente de tuberculosis intestinal, por
lo cual, se le resecó 80 cm de íleon distal. Desde el punto de vista fisiológico, el paciente
puede presentar una de las siguientes alteraciones: a. Disminución de la secreción de
Vitamina B12

e. Disminución de la absorción de ácido glicocólico


61. Un paciente es sometido experimentalmente a un fármaco que modifica el flujo salival,
obteniéndose un volumen de saliva de 288 ml en 6 horas. En este caso las concentraciones
de electrolitos y bicarbonato en la saliva obtenida varían de la siguiente manera: a. ↑ Na+,
↓ K+, ↑ Cl-, ↑ HCO3-

1. b. ↓ Na+, ↓ Cl-, ↑ K+, ↓ HCO3-

62. Uno de los siguientes elementos debería hallarse con más probabilidad en el esófago de
un paciente que sufre de reflujo gastro esofágico…
a. Pepsina

63. Un paciente de 40 años cursa con anemia de 8g/dl, aqueja además de astenia y sensación
de hormigueo bilateral en los miembros inferiores, al examen se halla alteración de la
sensibilidad a la vibración y camina con ampliación de la base de sustentación. Uno de los
siguientes procedimientos sería de ayuda para el diagnóstico de este paciente:
a. Tomografía cerebral

A
b. Biopsia de la mucosa gástrica

EK
64.Paciente de 60 años ingresa por caída hace 1 hora y pequeño hematoma en cuero
cabelludo, al examen físico ampliado se observa ictericia de piel y mucosas generalizada,
abdomen blando, se palpa estructura quística no dolorosa en hipocondrio derecho que
corresponde a vesícula biliar (signo de Courvoisier), en los exámenes de laboratorio se halla
OT
niveles bajos en la formación de estercobilinógeno y urobilinógeno en heces, incremento de la
bilirrubina conjugada en la orina, elevación de fosfatasa alcalina y gamma glutamil
transpeptidasa séricas. El presente cuadro puede ser explicado por: a. Reabsorción de
hematoma
NC

c.Carcinoma de la cabeza de páncreas


65. Un recién nacido presenta vómitos biliosos poco tiempo después de cada alimento. Al
preguntar a la madre sobre antecedentes, ella recuerda que tuvo polihidramnios durante la
gestación, pero un análisis de cariotipo fue normal. Una de las siguientes es la causa más
BA

probable de estos hallazgos en el recién nacido: a. Enfermedad de Hirschprung


e. Malrotación de la yema pancreática ventral
66.En un estudio de la secreción de hormonas gastrointestinales, sus concentraciones en la
vena porta se midieron durante perfusión luminal del intestino delgado con soluciones de
diversas magnitudes de pH. ¿Qué hormona aumentará en el plasma de la vena porta durante
perfusión a través del intestino con una solución de pH 3?
a. CCK
e. secretina
67.Paciente de 30 años que ingresa a causa de un traumatismo abdominal cerrado. En la
exploración se aprecia discreta palidez de piel y mucosas, auscultación pulmonar normal,
taquicardia de 120 /min. Discreta distensión abdominal y matidez en flancos; el hematocrito,
que era prácticamente normal al ingreso, disminuye a 30% a las tres horas. En la Rx de tórax
se objetiva fractura de las costillas 10-11 izquierdas. La causa más probable de la anemización
en este paciente es: a. traumatismo renal con hemorragia retroperitoneal.
c. rotura de bazo con hemoperitoneo.
68. Revisando la angiotomografía de un hombre de 70 años en estudio por aneurisma de
aorta abdominal, el radiólogo le informa de la presencia de una oclusión completa de la
arteria mesentérica inferior. El paciente se encuentra completamente asintomático. La
oclusión de la arteria mesentérica inferior cursa de manera asintomática en muchas
ocasiones ya que el territorio que irriga puede recibir flujo proveniente de la arteria:
a. cólica derecha
e. cólica media

69. En las patologías de esófago es importante conocer bien la anatomía esofágica. ¿Cuál de
las siguientes afirmaciones es correcta? a. El esófago tiene capa mucosa, muscular y serosa

c. El esófago torácico pasa por detrás del cayado aórtico

70. A pesar de que pueda haber variaciones anatómicas, lo habitual es que el ciego sea
irrigado por una rama arterial que proviene de unas de las siguientes arterias: a. Iliaca
derecha

d. Mesentérica superior

71. Ante un paciente con una cirugía abdominal urgente, el informe operatorio señala que se
ha realizado una resección de todo el duodeno y del tercio proximal del yeyuno manteniendo

A
íntegros el estómago y todo el íleon, así como los dos tercios distales del yeyuno. En el
seguimiento nutricional del paciente ¿Qué vitamina o mineral presentará con menor

EK
probabilidad una disminución de su absorción?
a. Cianocobalamina

72. ¿Cuál de las siguientes sustancias forma parte de la secreción biliar? a. Tripsina
OT
Lecitina

73.¿De qué musculo forma parte el ligamento inguinal?


-Oblicuo externo del abdomen
74.¿Cuál de las siguientes enzimas está localizada en el borde en cepillo y juega un rol en la
NC

digestión de proteínas?
e. Carboxipeptidasa A.
75. Una de los siguientes sustancias, NO sirve como un buen agente emulsificante:
a. Colesterol
76. La sustancia que estimula el crecimiento de la mucosa gástrica es:
BA

a. Secretina

d. Gastrina

77.¿Cuál de las siguientes alternativas es una función de la colecistokinina?

a. Relajación de la vesícula para la salida de bilis

d. Secreción de enzimas pancreáticas

78.Con respecto a la anatomía del tronco celiaco, señale lo correcto a. El tronco celiaco se
origina de la cara posterior de la aorta abdominal
d. La hepática común que es una de sus ramas, participa en la irrigación del
estómago.
79. Con respecto a la anatomía del duodeno, marque la respuesta correcta: a. Tiene una
distribución en forma de “C”, que rodea la cola del páncreas
b. La 3ra porción duodenal está contenida en la pinza vascular aortomesentérica
80. En el íleon se absorbe aproximadamente el 95% dea través de la circulación
enterohepática.
a. agua
c. sales biliares
81. La estimula el mecanismo paracrino de la secreción de ácido clorhídrico.
a. histamina
82.En la digestión de proteinas,es el principal estímulo para convertir el
pepsinógeno en pepsina. a. la gastrina
b. el pH ácido
83. Con respecto a la somatostatina, marque lo correcto:
a. Es secretada por las células S del intestino
Interviene en la fase intestinal de la secreción gástrica
84. En pecten anal, es una estructura comprendida entre: a. la línea pectínea y los senos
anales
d. la línea anocutánea y la línea pectínea

85.¿Cuál de las siguientes alternativas es una proenzima pancreática? a. Tripsina

A
1. b. Elastasa
2. c. Quimotripsinógeno

EK
3. d. Amilasa
4. e. Procarboxipeptidasa C
86. En este paciente, el bloqueo farmacológico de los receptores H2 en la mucosa gástrica:
a) No tiene efecto sobre la secreción de ácido inducida por el vago
OT
b) Evita la activación de adenilciclasa por gastrina
c) Inhibe la secreción de ácido inducida por gastrina y mediada por el vago
d) Causa un aumento en el transporte de potasio por las células parietales gástricas
Se validó la A :)
87. Si se considerara una gastrectomía total para curar la gastritis del paciente, cuál de las
NC

siguientes sustancias ya no se produciría:


a) Gastrina
b) Quimiotripsina
c) Amilasa
BA

d) Pepsinógeno

88. Un paciente hipertenso está tomando un medicamento bloqueador de receptores alfa 1


adrenérgicos (prazosina) y como efecto secundario se queja de:
d) Lo escaso que es el medicamento
e) No tiene problemas en la salivación
c) Hiposalivación
d) Hipersalivación

89. Con respecto a las lesiones y enfermedades de la boca, marque lo correcto:


f) La leucoplasia se desprende al roce
b) la eritroplasia puede degenerar en adenocarcinoma
c) El muguet oral es una enfermedad bacteriana en inmunodeprimidos
d) la eritroplasia debe ser biopsiada
NOTA: fue validada la opción B ya que no es motivo del curso que sepamos el puto cáncer.

90. En este paciente, se puede asumir que la pancreatitis ha sido ocasionada por una
disminución en el efecto de:
g) Amilasa
h) Lipasa
c) Inhibidor de la tripsina
d) Entercinasa
91.Un efecto secundario en el estómago por la acción de la secretina es:
i) Disminución en la liberación de pepsinógeno
b) Menor actividad de la pepsina
c) Mayor paso de bicarbonato a sangre periférica
d) Aumento en la producción de factor intrínseco
92. Dentro de los factores protectores de la mucosa gástrica se pueden mencionar múltiples
protagonistas. Uno de ellos es:
j) CCK
k) Gastrina
c) Receptor muscarínico
d) Pepsina
93. La saliva puede tener una variedad de electrolitos en su composición. Entre ellos el cloro,
respecto al cual se puede afirmar:
l) Su mayor concentración se consigue con el flujo bajo
b) Su concentración no llega a ser tan alta como en el plasma
c) Con flujo alto su concentración es mayor que la del plasma

A
d) Su menor concentración se alcanza con flujo alto
94. En el síndrome de boca seca o síndrome de Sjogren, una de las complicaciones

EK
asociadas es:
a) caries
b) Disminución de la acidez gástrica
c) Aumento en de la producción de saliva
d) Aumento del pH bucal
OT
95. Estimula la producción de saliva:
a) Vasodilatación periglandular
b) Atropina
c) Fatiga o cansancio
d) Expresión de miedo
NC

96. El omeprazol actúa sobre la membrana de la célula


m) Basolateral/principal
n) apical/principal
o) Basolateral/parietal
d) apical/parietal
BA

97. Para protegerse del entorno ácido, el Helicobacter pylori se autogenera un entorno de
pH menos ácido alrededor suyo, gracias a una enzima que alcaliniza su entorno local
mediante la conversión de:
a) urea en NH3
b) H2O y CO2 en ácido carbónico
c) NH3 en urea
d) H2CO3 en bicarbonato
98. La anemia perniciosa destruye las células:
p) mucosas del cuello
b) oxínticas
c) principales
d) mucosas superficiales
99. La célula mucosa del cuello gástrico produce:
a) Moco
b) ácido clorhídrico
c) pepsinógeno
d) Factor intrínseco
100.El aumento en la acidez del estómago producido principalmente por la infección de
Helicobacter pylori se debe a la disminución de:
a) Somatostatina
b) Bicarbonato por las glándulas de Brunner
c) Secretina
d) Colecistoquinina
101. De las siguientes sustancias secretadas por los órganos de este paciente, la más
alcalina es la secreción:
a) pancreática
b) Esofágica
c) Yeyunal
d) Salival
102.En cuanto a la gastritis de este paciente, se encontró que era producida por la bacteria
Helicobacter pylori. Esta bacteria sobrevive en el medio ácido del estómago gracias a:
a) ácido clorhídrico
b) Toxina CagA
c) Ureasa

A
d) Jugo pancreático
103.La lengua está recubierta por epitelio:

EK
c) pseudoestratificado columnar no queratinizado
b) plano estratificado no queratinizado
c) pseudoestratificado columnar ciliado
d) plano estratificado queratinizado
104.El esfínter anal interno tiene musculatura …….. y tiene control ……..
OT
d) lisa / voluntario
b) lisa / involuntario
c) esquelética / simpático
d) esquelética / parasimpático
NC

19) La arteria Aorta proporciona la irrigación al tubo digestivo ¿cuál de las siguientes
arterias proporciona la irrigación al ángulo cólico derecho?
a) mesentérica superior
b) mesentérica inferior
BA

c) frénica inferior
d) tronco celiaco

20) Paciente de 26 años que le cuenta en su historia clínica que cada vez que almuerza a los
20 min tiene deseo de defecar, le comenta que su hijo de 1 mes le pasa lo mismo pero más
intenso. Esto se explica por el reflejo …….., el cual está …… en el paciente
a) colicoileal / normal
b) colicoileal / alterado
c) gastrocólico / normal
d) gastrocólico / alterado

21) La región del estómago que se comunica con el duodeno es la


a) pilórica
b) cardias
c) cuerpo
d) fórnix

22) Acude a consulta un px que fue diagnosticado de úlcera péptica 3 días antes. Luego de
múltiples pruebas diagnósticas se concluye que el paciente presenta un tumor secretor de
gastrina, ¿cual de las siguientes situaciones estará incrementada?
a) distensión gástrica
b) inhibición del vaciado gástrico
c) secrecion de acido clorhidrico
d) inhibición de la secreción de pepsinógeno

A
EK
OT
NC
BA
23) En el sistema digestivo, el control del apetito está dado por un complejo sistema de
sustancias y órganos integradores, los cuales regulan la ingesta de alimentos. La …… es
una sustancia orexígena y es sintetizada por el ……
a) leptina / estómago
b) felina / intestino
c) leptina / estómago
d) grelina / estómago

24) Con respecto a la actividad eléctrica del sistema digestivo, marque la alternativa
correcta
a) corresponden a potenciales de acción que están presentes de forma continua y le
dan capacidad de peristalsis autónoma al sistema digestivo
b) la frecuencia de las ondas lentas NO se ve influenciada por la actividad neural ni las
hormonas gastrointestinales
c) en el estómago las ondas lentas se dan en una frecuencia de 6 x min

A
d) las ondas lentas son cambios lentos y ondulantes del potencial en reposo
e) la frecuencia de las ondas lentas va de 6 a 12 ondas por minuto

EK
25) Ante una lesión del IX pc, el músculo …… se altera en su función
a) palatogloso
b) estilofaríngeo
c) palatofaríngeo
OT
d) constrictor superior

26) Un varón de 50 años es sometido a extirpación del duodeno y parte proximal del yeyuno.
La pérdida de estímulo hormonal en el páncreas para la secreción enzimática se explica
NC

por la pérdida de células


a) parietales, productoras de factor intrínseco
b) K productoras de factor intrínseco
c) M productora de CCK
d) I productora de CCK
BA

27) Marque la respuesta correcta:


A. El bronquio derecho constituye una de las estrecheces del esófago
B. Todos los órganos del sistema digestivo tienen capa serosa
C. La pared gástrica en el fondo es más delgada que en el cuerpo y antro
D. El esfínter de Oddi rodea a la papila menor duodenal

28) Marque la respuesta correcta en relación a la gastrina:


A. Al distenderse el estómago, se inhibe su producción.
B. Se estimula por la liberación de noradrenalina
C. Las células G son las productoras y se encuentran principalmente en el antro
gástrico
D. Las células G se encuentran principalmente en el fondo gástrico

29) Para poder morder una manzana, es necesario usar el siguiente músculo:
A. Milohiodeo
B. Tensor del paladar
C. Orbicular de los labios
D. Buccinador
30) Sustancia que inhibe la secreción y la motilidad del estómago prolongando el tiempo de
digestión:
A. Enteroglucagon.
B. Polipéptido pancreático
C. Péptido 1 similar al glucagón (GLP-1).
D. Péptido insulinotrópico dependiente de la glucosa (GIP).

31) El nacimiento de la arteria mesentérica superior se puede encontrar en cuál de los


cuadrantes abdominales:
A. Hipocondrio derecho
B. Hipogastrio
C. Epigastrio
D. Mesogastrio
32) Entre las múltiples causas de la Enfermedad por Reflujo Gastroesofágico, se puede
considerar también a una alteración en las del esfínter esofágico inferior:

A
A. Ondas secundarias
B. Contracciones tónicas

EK
C. Ondas lentas
D. Glándulas subesofágicas

33) Producto de la alimentación, se producen diversas sustancias peptídicas, cininas y


OT
bradicininas, las cuales permiten que:
A. Se produzca neovascularización en los territorios de las arterias abdominales
B. La acción de la lipasa pancreática se vea incrementada
C. El consumo de O2 del intestino aumente ligeramente
D. El flujo sanguíneo intestinal aumente hasta 8 veces
NC

34) El dolor periumbilical o epigástrico en el inicio de una apendicitis aguda se debe a:


A. Estímulo del nervio vago.
B. Íleo secundario.
C. Irritación del peritoneo parietal.
BA

D. Estímulo del sistema simpático.

35) El aumento en la actividad motora de la pared gástrica genera un aumento en los


niveles locales de qué sustancia en la microvasculatura:
A.Adenosina
B. Colecistoquinina CCK
C. Endotelina
D. Gastrina

36) ¿Cuál de los siguientes péptidos inhibe el vaciamiento gástrico?


A. Colecistoquinina
B. Péptido inhibidor gástrico
C. Motilina
D. Gastrina
37) Los músculos de la masticación que producen la retropulsión de la mandíbula son:
A. temporales [mas seguro]
B. maseteros
C. milohioideos
D. pterigoideos
38) En relación a la fisiología gástrica, marque lo correcto:
A. la cimetidina actúa en la región basolateral de la célula parietal
B. la marea alcalina se debe al paso de bicarbonato través de la membrana apical de la
célula principal
C. el cloro difunde hacia el exterior por la la región basolateral de la célula parietal
D. la salida de hidrogeniones a la luz es por difusión facilitada

39) Durante el sueño, la concentración de bicarbonato en la saliva:


A. Se eleva a niveles mayores que los del plasma
B. Aumenta
C. No tiene efecto
D. Disminuye

40) La secreción de saliva es importante en la fisiología digestiva. Su concentración de


potasio llega a ser menor que la del plasma cuando su secreción tiene un flujo:

A
A. Intermedio
B. Nunca

EK
C. Bajo
D. Alto

42) Respecto a las glándulas salivales, marque lo incorrecto:


A. la glándula parótida produce secreción serosa
OT
B. la glándula sublingual drena a través de conducto de Wharton
C. La glándula parótida drena a través del conducto de Stenon
D. la glándula sublingual tiene forma de garfio
NC

43) Con respecto a la saliva, marque la respuesta correcta:


A. será hipertónica cuando el flujo es bajo
B. a mayor flujo, menor concentración de Na
C. a mayor flujo, menor concentración de cloro
D. el sistema simpático estimula su secreción
BA

CI 3
44) En relación a la circulación hepática, marque lo correcto:
a) Los sinusoides hepáticos transportan sangre mixta
b) La vena porta proporciona el 50% de sangre al hígado
c) La vena porta se forma a partir de la vena esplénica y la mesentérica inferior
d) La arteria hepática deriva de la mesentérica superior

45) Dentro de las funciones de las células de Ito, marque lo incorrecto:


a) Sintetizan colágeno
b) Almacenan vitamina A
c) Se les llama células estrelladas
d) Pueden fagocitar patógenos y actúan como presentadoras de antígeno
46) Paciente con tumor neuroendocrino productor de secretina, debido a lo cual se puede
esperar que su secreción pancreática, comparada con la de una persona sana de bajo flujo,
tenga una concentración de:
a) Sodio aumentada
b) Igual
c) Bicarbonato aumentada
d) Potasio disminuida
47) El GALT se localiza en:
a) Lámina propia
b) submucosa
c) borde en cepillo
d) superficie de criptas de Lieberkühn

48) En relación a la histología hepática, marque lo correcto:


a) la zona 1 se afecta rápidamente en estados de hipovolemia y shock
b) La zona 1 se encuentra cercana a la vena central lobulillar
c) La zona 3 se encuentra más cerca a la vena central lobulillar
d) La zona 3 se encuentra más cerca al eje menor del acino hepático
49) El acino pancreático difere con el de las glándulas salivales en:
a) Contiene células centroacinares
b) No produce secreción serosa
c) El páncreas produce principalmente secreción mucosa

A
d) No tienen diferencias
50) Durante la digestión de las grasa, para que la lipasa actúe adecuadamente se requiere que el

EK
pH aumento en la luz intestinal, lo cual es logrado, entre otros, por la secreción de las
células:
a) Del conducto interlobulillar
b) Centroacinares
c) Acinares
OT
d) Alfa
51) La secreción de la colecistoquinina (CCK) se produce en la fase:
a) intestinal
b) En las 3 por igual
NC

c) gástrica
d) Cefálica
52) ¿Por cuál de las siguientes células es secretada principalmente la pro enzima
procarboxipeptidasa?
a) Acinares del páncreas
BA

b) Epiteliales del duodeno


c) Ductales del páncreas
d) Centro Acinares del páncreas
53) Una mujer de 43 años dolor en hipocondrio derecho e icterica. En la ecografía se evidencia
cálculos biliares. Estos cálculos lo más probable es que se encuentren localizados en:
a) conducto colédoco
b) Conducto cístico
c) Vesícula biliar
d) Conducto pancreático secundario
ECU 1:
Estudiante de 21 años sufre de gastritis aguda ocasionada por comer en lugares poco
higenicos. Suele consumir caramelos (“chupar”) mientras esta en base hasta la tarde.
También toma regular cantidad de leche (grasa, lactora, proteinas), pues le calma el dolor y
el ardor que sitnete por la gastritis (tiene dispepsia y cuando toma la leche se le pasa).
Incluso cuando puede, se toma dos vasos de agua frita y le calma la molestia. Ha decido ir al
medico para tratarse, pues ya no soporta el dolor, el cual esta seguro que los síntomas se
producen por elevada producion de HCl en el estomago, y por ello le ha recetado ranitidina
1.1) El consumir caramelos eleva los niveles en sangre de una hormona cuya función es la
estimulación de las células.
- Beta del páncreas por GIP el cual es una incretina y por consiguiente estimula las
células pancreáticas

1.2) Consumir caramelos indirectamente actica la via:


-POMP/ CART saciedad

1.3) Consumo de leche produce indirectamente


- CCK inhibición del vaciamiento gástrico mayor tonicidad del esfínter pilórico

1.4) Cuando el px toma dos vasos de agua, genera indirectamente un aumento en la


liberación de:
vaso de agua distención → g astrina → secreción de HCl

A
-

1.5) El uso de ranitidina bloquea el receptor H2 de la histamina en las células parietales, la

EK
histamina llega a estas células por:
- histamina es una hormona paracrina por → difusión
**endocrina es por via hematógena y si fuera neuroendorina es por un NTs
OT
1.6) Aumenta la secreción salival:
- noradrenalina a través de los receptores Beta 2
1.7) En este paciente con gastritis aguda debida a una alta producción de ácido clorhídrico, sería
lógico esperar que el píloro tenga un tono muscular:
NC

- primero la secretina
- luego CCK
**ambas reguladores del HCl, Gatritis aguda debido a una alta producción de HCL piloro
estará aumentado (por la CCK)
1.8) Debido al uso de ranitidina, los valores de somatoestina en sangre:
BA

- ranitidina disminuye acción de gastrina se quiere secretar mas no actúan los


inhibidores como la somatoestina somatoestina disminuye
1.9) El uso de atropina en este paciente:
- Inhibirá la acción de las prostaglandinas
- Aumentará la producción de ácido clorhídrico
- Disminuirá la acción del receptor CCK-B
-Aumentará el pH del estómago
ECU 2:
Niño de sexo masculino de 2 años de edad, sufre de estreñimiento desde el nacimiento (1
deposición cada 3-4 días). Madre menciona que le estimula la defecación con un
termómetro rectal, y continuo uso de enemas y laxantes. Desde hace 6 meses comienza
con vómitos postprandiales. Los síntomas aumentan en frecuencia y magnitud y están en
relación con los episodios de estreñimiento. No refiere fiebre, tos, diarrea ni lesiones
cutáneas. Al examen físico presenta regular estado general, luce deshidratado. Abdomen
distendido, blando, depresible e indoloro. No se palpan masas abdominales. Se
permeabiliza el canal anal con termómetro rectal, encontrando cierta resistencia. Salida de
material fecal mal oliente en regular cantidad. Exámenes de laboratorio: hemograma
normal. Signos inflamatorios de fase aguda negativos. Alcalosis metabólica leve en sangre
venosa. Radiografía con enema baritado muestra recto y colon sigmoides dilatados
(megacolon). Biopsia profunda: ausencia de células ganglionares en la muestra enviada. Se
realiza cirugía correctiva.

2.1) Durante la fase esofágica de la deglución, para un bolo alimenticio determinado, a


medida que avanza el bolo la fuerza de la contracción se hace más:
- hiperpolarizado
- fuerte
- dependiente de Ach
- debil
2.2) Cuando este paciente ingiera sus alimentos, se espera que al momento de pasar el bolo
alimenticio por el esfínter esofágico superior, la presión intraesofágica disminuya en:
- la porción proximal al bolo
- el tercio medio del esófago
-el cardias

A
- el lugar donde se contraiga la muscular propia
2.3) Al examinar la orofaringe del paciente, uno puede hallar fácilmente la amígdala

EK
palatina, pues esta se encuentra inmediatamente detrás del músculo:
-Palatogloso
- Palatofaringeo
- Hiogloso
- Elevador del velo del paladar
OT
2.4) Con respecto a la defecación en este caso, marque la respuesta correcta:
- En posición de cuclillas, el músculo puborectal genera un ángulo más agudo en el
recto
NC

- El sigmoides y el recto están inervados por el nervio vago


- La sensación de defecar sólo se da cuando el recto es ocupado por heces y
alcanzado el 80% de su capacidad
-El esfínter anal comprometido tiene inervación autónoma
BA

2.5) En este paciente [hirschsprung] se considera que está abolido el reflejo:


- Coloileal
-Rectoesfinteriano
- Gastrocólico
- Relajación receptiva

2.6) No se espera que sea causa del vómito:


-Ayuno prolongado
- Estimulación faríngea y del glosofaríngeo
- Irritación de la mucosa gástrica
- Dolor intenso

ECU 3:
Paciente de 54 años con antecedentes de alcoholismo, gastritis crónica, tabaquismo
pesado, obesidad, cálculos biliares y cirrosis, es llevado a la emergencia por dolor
abdominal en epigastrio irradiado a la espalda y trastorno del sensorio.
Al examen físico: presión arterial 85/50 mmHg, frecuencia cardíaca 100 latidos/min,
frecuencia respiratoria 18 x minuto, temperatura axilar 36°C.
Conjuntivas pálidas, escleras ictéricas nevus arácnidos en tronco, distensión abdominal
marcada, cabeza de medusa, matidez desplazable en ambos flancos e hipogastrio, dolor a
la palpación de abdomen.
Tiempo de protrombina: 24 seg (testigo: 13 seg); TPT: 38 seg, glicemia: 165 mg/dL, uremia:
20 mg/dL, ASAT: 76 UI/L, ALAT: 22 UI/L, albumina: 2,5 g/dL, bilirrubina total: 2,6 mg/dL,
bilirrubina directa: 1,4 mg/dL, amilasa sérica 4000 U/L.
3.1) En esta paciente, al aumento de la amilasa sérica, se debe directamente a una lesión de:
a) páncreas
b) vesícula y árbol biliar
c) estómago
d) hígado
3.2) Considerando que el paciente sufre de gastritis, se puede decir que la secreción de
ácido por la mucosa gástrica
a) involucra transporte activo de hidrogeniones
b) es realizada principalmente por células principales

A
c) es inhibida por antihistaminas tomadas por pacientes con rinitis alérgica
d) involucra la liberación de HCl de los gránulos zimógenos

EK
3.3) El paciente tiene hemorragia digestiva alta por várices sangrantes como complicación.
Llegando a estar en shock hipovolémico por hemorragia masiva, se encontrara necrosis
hepática en:
a) zona 1
b) no se afectan los lobulillos hepáticos en hemorragia
OT
c) zona 3
d) zona 2
3.4) El misoprostol, análogo de las prostaglandinas está mejor indicado en:
a) cicatrización de úlcera péptica duodenal
b) erradica el helicobacter pylori
NC

c) tratar el sind de Zollinger ellison


d) prevenir daño por AINES
3.5) De las siguientes sustancias secretadas por los órganos de este paciente, la más
alcalina es la secreción:
BA

- Esofágica
- Salival
- Yeyunal
-Pancreática
3.6) En este paciente, se puede asumir que la pancreatitis ha sido ocasionada por una
disminución en el efecto de:
- Lipasa
- Enterocinasa
- Amilasa
-Inhibidor de la tripsina
3.7) ¿Cuál de las siguientes sustancias es segregada por el páncreas?
-Amilasa
- Pepsina
- Quimiotripsina
- Tripsina
3.8) Cada vez que este paciente toma alcohol, la acidificación de la luz del duodeno:
-Disminuye el vaciamiento gástrico
- Aumenta la contracción del esfínter de Oddi
- Aumenta la secreción del ácido gástrico
- Disminuye la secreción pancreática del bicarbonato

A
EK
OT
NC
BA
SISTEMA
DIGESTIVO (ME
154) EXAMEN
FINAL
Ciclo 2018-01

1. Un niño de 2 años es llevado a la consulta por diarrea persistente y edema de las extremidades, además falta
de crecimiento y desarrollo en relación a su edad. Los análisis de sangre revelan que tiene concentración
plasmática baja de proteínas (hipoproteinemia). Durante la endoscopía duodenal, se coloca colecistokinina
(CCK) endovenosa y se recoge muestras del líquido duodenal; el resultado del líquido confirma incapacidad
para hidrolizar proteínas a un pH neutro, esta situación mejora al añadir una pequeña cantidad de tripsina.
El paciente probablemente esté sufriendo la falta congénita de
………….
(Unidad 4, sesión 26, logro 2: Explicar la Digestión y absorción de las proteínas y sus alteraciones)
a. Pepsinógeno

A
b. PEPT-1
c. Carboxipeptidasas
d. Enterocinasa

EK
2. Experimentalmente se incrementa la velocidad de la secreción salival con una sustancia, el análisis de
la composición de esta saliva obtenida se espera encontrar…………..
(Unidad 3, sesión 17, logro 5 : Explica la Influencia de la velocidad del flujo salival en la composición de la
saliva)
OT
a. Elevación de concentración de bicarbonato, sodio y potasio
b. Elevación de concentración de cloro, sodio y potasio
c. Disminución de concentración de potasio
d. Disminución de concentración de potasio y bicarbonato
NC

3. Paciente varón de 46 años soltero, consulta por odinofagia y bajo de peso, tiene antecedente de
tuberculosis desde hace 3 meses y es fumador crónico (10 cigarrillos por día); al evaluar la cavidad oral se
identifica lesión blanquecina en el dorso de la lengua y paladar blando, las lesiones se desprenden con el
baja lengua dejando una base eritematosa. Esta lesión corresponde probablemente a ……………………….…..
( Unidad 3, sesión18, logro 1-2 : Describe las enfermedades inflamatorias, infecciosas y proliferativas de la
BA

cavidad oral)
a. Eritroplaquia
b. Candidiasis oral
c. Leucoplaquia vellosa
d. Fibroma en cavidad oral

4. Minero de 32 años de edad, que acude a centro de


salud por presentar de forma progresiva desde hace
1 año dificultad para ingerir alimentos sólidos y
luego líquidos; refiere regurgitaciones alimentarias y
marcada pérdida de peso (15 kilos). Radiografia
baritada de esófago como se muestra en la figura. El
presente caso se explica por……………….
(Unidad 2, sesión 12, logro 4: Identificar y describir
la función de los esfínteres esofágicos)
a. Contracción incompleta del esfínter esofágico inferior
b. Dificultad para el inicio de la deglución
c. Relajación incompleta del esfínter pilórico
d. Relajación incompleta del esfínter esofágico inferior
5. Paciente mujer de 35 años acude a consulta por sensación de sequedad y lesiones en cavidad oral. Al
examen se observa atrofia de la mucosa, fisuras y úlceras; nota además sequedad e irritación de la
córnea y aumento del tamaño de las glándulas parotídeas. Su diagnóstico más probable es artritis
reumatoide; el hallazgo más probable en una biopsia de glándula parótida es……..….
(Unidad 3, sesión 18, logro 3: Describe las enfermedades más frecuentes de las glándulas salivales)
a. Hiperplasia de acinos glandulares serosos
b. Gran infiltración de linfocitos y células plasmáticas
c. Gran infiltrado de linfocitos y macrófagos
d. Presencia de acinos normales con hiperplasia de células ductales

6. Un paciente con anemia acude con su médico quejándose de episodios frecuentes de gastroenteritis. Un
análisis de sangre revela anticuerpos circulantes dirigidos contra células parietales gástricas. Su anemia es
atribuible a la hiposecreción de
………………………
(Unidad 3, sesión 20, logro 5: Gastritis crónica. Tipos de gastritis)
a. Factor intrínseco

A
b. Proteina R (haptocorrina)
c. Pepsinógeno
d. Ácido clorhídrico

EK
7. Dos estudiantes deciden tomar un receso para comer una hamburguesa a la hora del almuerzo. Antes de
llegar a la cafetería, impulsos nerviosos provenientes del complejo vagal dorsal iniciarán la secreción de
ácido gástrico por la liberación dedesde el sistema nervioso entérico.
(Unidad 3, sesión 20, logro 2: Regulación de la secreción gástrica: estimulación, fases de la secreción)
OT
a. Serotonina
b. Óxido nítrico
c. GRP (péptido liberador de gastrina)
d. Péptido intestinal vaso activo
NC

8. Un niño de cuatro años de edad es llevado a la consulta por cuadros diarreicos frecuentes caracterizados
por heces pálidas, voluminosas y fétidas, presenta bajo peso y talla. Se mide la concentración de cloruro en
el sudor y se encuentra que sus valores son muy elevados. La alteración más importante a nivel de células
ductales del páncreas tiene relación directa con la conductancia de…………
(Unidad 3, sesión 23, logro 5 Explica la Secreción pancreática: formación del jugo pancreático, influencia de la
BA

velocidad de flujo y regulación)


a. Potasio
b. Bicarbonato
c. Sodio
d. Cloro

9. Una mujer de 50 años de edad que sufrió durante varios años resequedad de los ojos debida a producción
inadecuada de lágrimas es enviada con un gastroenterólogo para evaluación de pirosis crónica. El examen
endoscópico revela erosiones y tejido cicatrizal en la parte distal del esófago justo por arriba del esfínter
esofágico inferior. Las lesiones pueden atribuirse a la disminución de uno de los siguientes componentes
salivales:
(Unidad 3, sesión 17, logro 4: Explicar la Formación de la saliva y cuáles son sus componentes)
a. Bicarbonato
b. Lactoferrina
c. Ig A
d. Amilasa
10. Se evalúa los valores séricos de las siguientes sustancias a un paciente con enfermedad hepática terminal;
en este paciente se espera encontrar la combinación con la letra …………
(Unidad 3, sesión 22, logro 5: Describe las Pruebas de función hepática, la Insuficiencia hepática,
encefalopatía hepática e hipertensión portal)

Glucosa Amoniaco Albúmina


a. Aumenta Disminuida Disminuida
da
b. Disminui Aumentada Aumentada
da
c. Aumenta Aumentada Aumentada
da
d. Disminui Aumentada Disminuida
da

A
11. Una mujer de 35 años de edad HIV positiva, se presenta al médico con dolor abdominal en cuadrante
superior derecho e ictericia. La paciente refiere haber tenido múltiples episodios de ictericia durante los

EK
últimos 10 años. Los exámenes para determinar hepatitis viral, dieron positivos para Hepatitis B, siendo
catalogado el caso como hepatitis crónica con alteración funcional. En un examen de sangre ¿cuál de los
siguientes parámetros está disminuido?
(unidad 3, sesión 22, logro 5: Pruebas de función hepática, Insuficiencia hepática, encefalopatía hepática
e hipertensión portal)
OT
a. Fosfatasa alcalina
b. Albumina
c. Bilirrubina
d. Tiempo de protrombina
NC

12. En el reflejo peristáltico del intestino delgado, uno de los siguientes eventos sucede en la
porción oral del bolo alimenticio…………...
(Unidad 2, sesión 13, logro 4: Explicar la Motilidad del intestino delgado: Contracciones segmentarias y
peristálticas)
a. Disminución de 5 hidroxitriptamina desde las neuronas IPAN
BA

b. Contracción del músculo longitudinal


c. Acción del péptido intestinal vasoactivo (VIP) en el músculo circular
d. Acción de acetilcolina en el músculo circular

13. Experimentalmente se coloca una dosis alta de secretina en la luz intestinal duodenal; como
consecuencia de esto, en el jugo pancreático de la misma luz intestinal se observa la disminución de la
concentración de …..………..
(Unidad 3, sesión 23, logro 5: Explica la Secreción pancreática: formación del jugo pancreático, influencia
de la velocidad de flujo y regulación)
a. Na+
b. Cl-
c. K+
d. HCO3-

14. Un varón de 58 años de edad con enfermedad de Crohn severo fue sometido a una resección ileal.
Después de la cirugía este paciente padecerá de esteatorrea, esto se explica porque …..………..
(unidad 4, sesión 26, logro 4: Explica las alteraciones en la Absorción de lípidos)
a. El pool de ácidos biliares se incrementa
b. Los quilomicrones no pueden formarse en el lumen intestinal
c. La micelas no pueden formarse
d. El páncreas no secreta lipasa

15. En un experimento se inserta un balón en el estómago de un voluntario, se infla poco a poco mientras que
se vigilan las presiones intraluminales. Aunque el volumen del balón aumenta considerablemente, las
presiones permanecen constantes. Esta relación volumen-presión se explica por la liberación local de
…………..
(Unidad 2, sesión 13, logro 1 Explica la Motilidad gástrica: relajación receptiva)
a. Acetil colina y gastrina
b. Colecistoquinina y óxido nítrico
c. Óxido nítrico y péptido inhibidor vasoactivo
d. Norepinefrina y óxido nítrico

A
EK
OT
NC
BA
16. La toxina del Vibrio cholerae causa diarrea debido a…….
(Unidad 4, sesión 27, logro 6: Explica el transporte hidroelectrolítico intestinal, toxina colérica)
a. La fosforilación del canal CFTR de los enterocitos de las vellosidades intestinales
b. El Incremento de la secreción de cloro por las células de la cripta intestinal
c. La inhibición de la producción de AMPc por las células epitelailes
d. El incremento de la absorción de agua y sodio a través de las uniones estrechas

17. ¿Cuál de las siguientes alternativas es una característica de la secreción exocrina del páncreas?
(Unidad 3, sesión 23, logro 5: Secreción pancreática: formación del jugo pancreático, influencia de la
velocidad de flujo y regulación)
a. Tiene una baja concentración de Cl- respecto al plasma
b. Es estimulada por la presencia de bicarbonato en el duodeno
c. La secreción enzimática es estimulada principalmente por la gastrina
d. Es hipotónica respecto al plasma

A
18. Una madre lleva a su hijo de dos años de edad a la sala de urgencias, estresada porque el niño deglutió una
moneda de 10 céntimos mientras la familia cenaba en un restaurante. El médico observa mediante
fluoroscopía que la moneda se halla en el estómago y asegura a la madre que la moneda se eliminará con

EK
las heces. El médico recomienda utilizar la respuesta fisiológica que permitirá la evacuación de la moneda
del estómago al intestino ………….…..
(Unidad 2, sesión 13, logro 2: Explica la Motilidad gástrica: mezclado y vaciamiento)
a. Es por la relajación receptiva
b. Son los movimientos de mezcla y trituración
c. Es provocada por el ayuno
OT
d. Es por la relajación del esfínter esofágico superior

19. Las estructuras en el hígado que permite que los productos metabólicos unidos a proteínas tengan acceso
a las membranas basolaterales de los hepatocitos, son…..
(Unidad 3, sesión 21, logro 4-5: Explica la Organización micro estructural del hígado)
NC

a. Los Canalículos
b. Las fenestras sinusoidales
c. Las uniones intercelulares herméticas
d. Las células de Ito
BA

20. La composición de la bilis es modificada conforme fluye por los conductillos biliares. Durante este
tránsito se espera que aumente la concentración de…….
(Unidad 3, sesión 22, logro 2: Describe la Secreción biliar, visión general del sistema biliar extrahepático y
composición de la bilis)
a. Ig A
b. Glucosa
c. Monómeros de ácido biliar
d. Vitamina A

21. Se mide experimentalmente el contenido gástrico de dos personas. La persona “A” tiene alto
contenido de grasa y la persona “B” tiene un contenido hipertónico ¿Cuál de las siguientes es
correcto respecto al vaciamiento gástrico? (Unidad 2, sesión 13, logro 2: Describe la Motilidad y
vaciamiento gástrico)
a. Hay ralentización del vaciado gástrico solo en “A”
b. El vaciamiento gástrico es más rápido en ambos
c. En ambos casos hay incremento de la motilina
d. Hay ralentización del vaciado gástrico en ambos casos

22. El examen endoscópico de un paciente con hipertensión portal grave revela venas tortuosas que sobresalen
hacia la luz del esófago. El paciente recibe tratamiento quirúrgico mediante la colocación de una derivación
que conecta la vena porta a la vena cava. Después de la operación el riesgo de encefalopatía …………………..
y el riesgo de sangrado de várices ……………..
(Unidad 3, sesión 22, logro 5: Describe la Insuficiencia hepática, encefalopatía hepática e hipertensión portal)
a. Aumentará/disminuirá
b. Disminuirá/disminuirá
c. Aumentará/aumentará
d. Disminuirá/aumentará

A
EK
OT
NC
BA
23. Un paciente varón de 18 años de edad acude al médico para sus exámenes de rutina. Sus resultados de
laboratorio muestran un valor de bilirrubina sérica de 4 mg/dl y una bilirrubina directa de 0,3 mg/dl. Las
pruebas de función hepática son normales. La alteración que explica mejor este caso es por la
deficiencia de ………………..
(Unidad 3, sesión 22, logro 3: Explica la Producción y excreción de bilirrubina. Tipos de bilirrubina e ictericia)
a. Transaminasas
b. Glucuronil transferasa
c. Hemo oxigenasa
d. La 7 alfa hidroxilasa

24. Un hombre de 57 años de edad es llevado a urgencias con hematemesis masiva rojo brillante, a su llegada
se halla inconciente con PA: 80/40 mm Hg y FC: 124 lat/min. Luce ictérico con presencia de “arañas
vasculares en el tórax anterior y extremidades”, abdomen distendido con signo de oleada positiva. Se
encuentra esplenomegalia y pérdida de la masa muscular en extremidades. La anastomosis vascular
responsable del sangrado en este paciente es ………….…..
(Unidad 3, sesión 21, logro 2: Describe las anastomosis porto sistémicas)

A
a. Arteria gástrica izquierda y vena ácigos
b. Vena gástrica izquierda y vena ácigos
c. Vena paraumbilical y vena epigástrica inferior

EK
d. Vena gástrica izquierda y vena esofágica superior

25. Un estudiante de medicina está comiendo un plato de comida a base de champiñones, espárrago y salsa
de soya. El sabor umami contenido en todos estos alimentos actúa a nivel de los botones gustativos
estimulando ………………..
OT
(Unidad 2, sesión 10, logro 5: Describe los tipos y mecanismos moleculares para la detección de los sabores)
a. El ingreso de sodio
b. Un receptor acoplado a proteína G
c. Su receptor específico T1R3
d. El ingreso de hidrógeno
NC

26. Un hombre de 22 años de edad se presenta al médico con una historia de 1 año de evolución
caracterizado por dolor recurrente en fosa iliaca derecha y diarrea. Manifiesta además pérdida de
peso de 8 kg durante este periodo. La colonoscopía revela múltiples lesiones en el ileon terminal y
colon. La biopsia de estas lesiones revela engrosamiento, inflamación y ulceración de la mucosa. El
diagnóstico más probable en este caso es…….
BA

(Unidad 4, sesión 28, logro 5: Describe la Enfermedad inflamatoria intestinal. Generalidades, morfología y
características)
a. Sprue celiaco
b. Enfermedad de Crohn
c. Sindrome de colon irritable
d. Colitis ulcerativa

27. Una de las funciones del músculo señalado es:


(Unidad 2, sesión 8, logro 3: Describir el Piso de la
boca: estructuras blandas que la conforman)
a. Eleva el paladar blando
b. Recibe inervación del nervio maxilar
c. Deprime el hioides cuando la mandíbula está fija
d. Deprime la mandíbula cuando el hioides está fijo

28. Varón de 61 años que consulta por dolor retro esternal intenso desde hace 6 horas y después de
vómitos intensos y repetidos; al examen se observa disnea, cianosis, hipotensión y signos clínicos de
shock. La radiografía simple de tórax muestra neumomediastino. El líquido en el espacio pleural
aspirado tiene alta concentración de amilasa. ¿Cuál de las siguientes alternativas puede explicar este
cuadro clínico?
(Unidad 3, sesión 18, logro 6: Describe algunas Enfermedades del esófago)
a. Sindrome de Mallory Weiss
b. Rotura espontánea de esófago
c. Neumotórax por probable herida penetrante
d. Perforación de ulcera gástrica de cara posterior, con complicación torácica

A
EK
OT
NC
BA
29. La secreción del ácido en la célula parietal gástrica se lleva a cabo por una ATPasa especifica que
intercambia hidrogeniones (H+) del citosol por…..
(Unidad 3, sesión 20, logro 1: Explica la Secreción del HCl y sustancias que la alteran)
a. Cl-
b. HCO3-
c. Na +
d. K+

30. En condiciones normales el ingreso de 600 ml de líquido es el estómago provoca un aumento de presión
intragástrica de unos 12 cm de H2O. Después de una vagotomía (corte del nervio vago) es de esperar que
el ingreso del mismo volumen de líquido provoque lo siguiente: …………………………………
(Unidad 2, sesión 13, logro 1: Describe la Motilidad gástrica: relajación receptiva)
a. Un aumento igual de la presión
b. Que no aumente la presión
c. Un aumento mayor de la presión
d. Una disminución de la presión

A
31. Una paciente de 30 años de edad es sometida a una cirugía en oído medio derecho por un problema de

EK
otoesclerosis. Luego de la cirugía refiere alteración en la percepción de sabores. Al evaluar el caso usted
esperaría encontrar……….
(Unidad 2, sesión 10, logro 5: Describe la Irrigación e inervación de la lengua)
a. Alteración en la sensación del dolor y temperatura en el tercio posterior de la lengua
b. Alteración en la sensación del gusto en los dos tercios anteriores de la lengua
c. Alteración en la sensación del gusto en la punta de la lengua
OT
d. Sensación del dolor, tacto y temperatura conservada en toda la lengua

32. ¿Cuál de las siguientes alterativas es correcta?


(Unidad 4, sesión 26 : Explica la digestión y absorción de nutrientes y sus alteraciones)
a. En el borde luminal, en cepillo, del intestino delgado, la absorción de sodio únicamente se realiza
NC

asociada a la de glucosa.
b. El lugar principal para la absorción del hierro es el ileon
c. Las sales biliares desconjugadas son absorbidas preferentemente en el colon
d. El proceso de digestión y absorción de la vitamina B12 no se altera en insuficiencia pancreática.

33. En un paciente de 45 años de edad con colestasis biliar, se encuentra una elevación de los niveles
BA

sanguíneos de fosfatasa alcalina hasta 3 veces la cifra normal. ¿Cuál de las siguientes alternativas estará
también elevada como evidencia del daño de la vía biliar?
(Unidad 3, sesión 22, logro 5: Pruebas de función hepática, Insuficiencia hepática, encefalopatía hepática
e hipertensión portal)
a. Tiempo de protrombina y albúmina sérica
b. Transaminasas hepáticas (ALT y AST)
c. Glucoronil transferasa
d. Gamma glutamil transpeptidasa

34. Revisando la angiografía de un hombre de 70 años en estudio por aneurisma de aorta abdominal el
radiólogo informa de la presencia de una oclusión completa de la arteria mesentérica inferior. El paciente
se encuentra completamente asintomático. ¿Cuál de las siguientes arterias se anastomosa a la sistema
arterial de la mesentérica inferior?
(Unidad 4, sesión 25, logro 1: Identifica la Arteria mesentérica superior e inferior, ramas y anastomosis)
a. Ileal
b. Cólica media
c. Sigmoideas
d. Cólica izquierda

35. Lactante de 3 meses de vida es atendido por presentar diarrea, se administra una solución de glucosa y
electrólitos por vía oral. La proteína de membrana apical que explica la capacidad de esta solución para
proporcionar aporte de glucosa e hidratación es ………..
(Unidad 4, sesión 26, logro 1: Explica la Digestión y Absorción de los hidratos de carbono. Alteraciones)
a. GLUT-5
b. SGLT-1
c. CFTR
d.
GLUT-2

BA
NC
OT
EK
A
36. Paciente ha sufrido herida de bala en el abdomen, se le ha tenido que extirpar el segmento medio y distal
del ileon. En este caso la síntesis hepática de sales biliares estará …..…..
(Unidad 3, sesión 22, logro 4: Explica la formación, función y Circulación entero hepática de lasa sales biliares)
a. Disminuida por inhibición de la colesterol 7 alfa hidroxilasa
b. Incrementada por estímulo de la enzima colesterol 7 alfa hidroxilasa
c. Incrementada por inhibición de la colesterol 7 alfa hidroxilasa
d. Sin cambios en el ritmo de síntesis

37. Un varón de 75 años ingresa al consultorio por presentar ictericia marcada de piel y las escleras. El estudio
del paciente mostró que presentaba un tumor que obstruía la totalidad del conducto hepático común.
¿Cuál de las siguientes estructuras se encontrará dilatada en este paciente?
(Unidad 3, sesión 21, logro 6: Describir el árbol biliar intrahepático)
a. Conducto de Wirsung
b. Conductos de Hering

A
c. Conducto colédoco
d. Conducto cístico

EK
38. Correlaciones las dos columnas y marque la fórmula correcta:
(Unidad 4, sesión 28, logro 1: Diarrea: definición, mecanismos: osmótica, secretoria y exudativa)
1. Enfermedad Hirschsprung( ) heces con moco y sangre
2. Diarrea osmótica( ) intolerancia a lactosa
3. Diarrea secretoria( ) aganglionosis congénita
OT
4. Diarrea exudativa( ) canales de Cl- en las células de la cripta

a.- 4231b.- 1234c.- 2143d.- 4213


NC

39. Respecto a la siguiente imagen que representa una estructura de la mucosa gástrica, la estructura con
número ………..
produce ……………………..
(Unidad 3, sesión 19, logro 4: La glándula fúndica. Funciones y tipos de células con sus características)
BA

a. 3 / pepsina
b. 1 / Pepsinógeno
c. 4 / HCl y factor extrínseco
d. 2 / pepsinógeno

2
40. En un paciente con insuficiencia renal crónica, el déficit en la absorción de calcio a nivel del
enterocito se debe a lo siguiente:
(Unidad 4, sesión 26, logro 6: Explica la Absorción de calcio y hierro)
a. No se convierte la 25 hidroxicolecalciferol a 1,25 dihidroxicolecalciferol
b. No se convierte la 1,25 dihidroxicolecalciferol a 25 hidroxicolecalciferol
c. Se incrementa la producción de Calbindina
d. Existe un descenso de la alfa 25 hidroxilasa renal

A
EK
OT
NC
BA
CLAVES EXAMEN
PARCIAL DE
SISTEMA
DIGESTIVO 2019 -
00

1. Varón de 30 años es traído a emergencia por agresión abdominal con arma de fuego (pistola)
y es sometido a laparotomía exploratoria, observándose isquemia del colon ascendente y
parte del colon trasverso ¿la lesión de cuál de las siguientes arterias explicaría esta isquemia?
(unidad 1, sesión 2, logro 6: (Describe la irrigación visceral: arterias de tronco celiaco,
arteria mesentérica superior e inferior, topografía de superficie, órganos por cuadrante)

A
a. Celiaca
b. Colónica derecha
c. Mesentérica inferior

EK
d. Mesentérica superior

2. Respecto a las sustancias gastrointestinales que regulan la secreción pancreática;


marque la afirmación correcta:
(unidad 1, sesión 3, logros 2 y 3: describir las hormonas gastrointestinales: estímulos y funciones)
OT
a. La Secretina, es la hormona más importante para la secreción de bicarbonato por las
células acinares del páncreas
b. La acetilcolina es capaz de estimular la secreción enzimática y de bicarbonato del páncreas
c. La gastrina, es la hormona más importante para la secreción de enzimas pancreáticas
d. La colecistoquinina (CCK) estimula al páncreas solo para secreción enzimática
NC

3. Ante una lesión del X par craneal, ¿cuál de los siguientes músculos mantiene conservada su
función?:
(unidad 2, sesión 08, logro 4: Paladar blando: componentes musculares)

a. Elevador del velo del paladar


BA

b. Tensor del velo del paladar


c. Palatofaríngeo
d. Glosofaríngeo

4. Experimentalmente se utiliza atropina (anticolinérgico) para inhibir la secreción de


gastrina, sin embargo la secreción de esta hormona se sigue dando ante estímulos
vagales. Esta situación se explica porque la atropina:
(unidad 1, sesión 3, logro 3 : describir las hormonas gastrointestinales: estímulo y funciones de la
gastrina y colecistoquinina)

a. Bloquea parcialmente la bomba de protones en la célula G


b. Inhibe la acción de acetilcolina e histamina en la célula G
c. Solo inhibe la acción del péptido GRP en la célula G
d. No bloquea la acción del péptido GRP

5. Un varón de 50 años es sometido a extirpación del duodeno y parte proximal del


yeyuno. Esta situación ocasionaría la pérdida de las células ……….. , productoras deque
estimula la secreción de
bicarbonato por el páncreas.
(unidad 1, sesión 3, logro 3: describir las hormonas gastrointestianles: estímulos y funciones
de la secretina y péptido insulinotrópico dependiente de glucosa)

a. “S” / secretina
b. Parietales / secretina
c. “I” / colecistoquinina
d. “S” / colecistoquinina
6. Recién nacido que presenta tumoración abdominal a nivel del cordón umbilical
(fotografía). ¿cuál de las siguientes afirmaciones es correcta respecto a este defecto en
el desarrollo embriológico del intestino?: (unidad 1, sesión 5, logro 2: identificar las
anomalías del desarrollo del intestino medio)

a. Corresponde a una Gastrosquisis


b. Las vísceras se hallan cubiertas por piel
c. No está asociado a otras malformaciones
d. Se asocia a
malformaciones cardiacas
y del tubo neural

A
7. Varón de 35 años acude a la emergencia por
trauma abdominal y se decida realizar una
laparoscopía exploratoria. El cirujano observa la
EK
OT
disposición de los órganos abdominales como se
representa en el siguiente esquema. Esta
disposición de órganos se explica por la
rotación(SMA=arteria mesentérica superior)
(unidad 1, sesión 5, logro 3: identificar las anomalías del
desarrollo del intestino medio: defectos de rotación,
NC

estenosis y atresias)

a. anti horaria del intestino medio, en sólo 90°


b. incompleta del intestino medio (270°)
c. horaria del intestino medio
BA

d. horaria del estómago

8. Se evalúa la expresión de la proteína Agrp en una persona con alteración del apetito; lo
correcto respecto a esta proteína es…..
(unidad 1, sesión 3, logro 4: Explica los mecanismos de control del apetito y saciedad )

a. Esta proteína es un potente anorexigénico


b. La mutación del gen que la codifica produce adelgazamiento
c. La sobre producción de la proteína lleva a obesidad por agonismo de receptores MC3 y MC4
d. La sobre producción de la proteína disminuye el apetito por antagonismo de receptores MC4

9. Juana cae de la bicicleta y se fractura la región anterior del hueso maxilar superior con
compromiso de la fosa incisiva. Al examen físico de la región esperaría encontrar alteración
en la sensibilidad de la encía …………………
(unidad 2, sesión 8, logro5: paladar: paladar duro y blando: irrigación e inervación)

a. bucal posterior
b. Lingual anterior
c. palatina anterior
d. palatina posterior
10. Recién nacido es atendido por el neonatólogo y luego entregado a su madre para dar de
lactar; la madre al dar de lactar observa coloración azulada de labios, acompañado de tos
persistente, dificultad respiratoria y distención abdominal. Se le intenta colocar una sonda
nasogástrica pero esta retorna a la cavidad oral en todos los intentos. ¿Cuál de las
siguientes anomalías del desarrollo es el más probable en este caso? (unidad 1, sesión 4,
logro 3: identificar las anomalías en el desarrollo del esófago: atresia y/o fístula traqueo
esofágica)

a. Estenosis esofágica proximal con Fístula traqueo esofágica distal


b. Atresia esofágica proximal con fístula traqueoesofágica distal
c. Atresia esofágica distal con fístula traqueoesofágica proximal
d. Fístula traqueoesofágica proximal y distal

11. ¿Cuál de los siguientes mecanismos ocurre durante la defecación?


(unidad 2, sesión 13, logro 6: motilidad del intestino grueso: contracciones segmentarias,
movimientos en masa, defecación y reflejo gastrocólico)

A
a. Contracción refleja del esfínter anal interno

EK
b. En la posición de “cuclillas” el músculo puborectal se halla relajado
c. Relajación del esfínter anal externo por efectos del VIP y óxido nítrico
d. La materia fecal en el recto estimula la contracción del sigmoides por los nervios pudendos

12. La estructura número 4 (gráfico) corresponde a


……….… y está ………..
OT
(unidad 2, sesión 9, logro 2: Partes
de un diente. Capas del diente:
Esmalte: características y células
que lo producen)

a. el cemento / mineralizado en 90%


NC

b. la dentina / formada por ameloblastos


c. el esmalte / formado por
células derivadas del
mesénquima
d. la dentina / formado por
BA

células derivadas de la cresta


neural

13. Un paciente luego de un accidente sufre lesión del piso de la boca, se constata daño del
nervio “cuerda del tímpano”, en este caso se esperaría encontrar disminución de lade la
lengua
(unidad 2, sesión 10, logro 3: Irrigación e inervación de la lengua)

a. Motilidad en los dos tercios anteriores


b. Sensación del gusto en el tercio posterior
c. Sensación del gusto en los dos tercios anteriores
d. Sensibilidad al tacto en los dos tercios anteriores

14. ¿Cuál de las siguientes afirmaciones es la correcta sobre la gastrina?


(unidad 1, sesión 3, logro 1: reconocer las características de las sustancias reguladoras
gastrointestinales: hormonas, sustancias paracrinas y neurocrinas)

a. Produce atrofia de la mucosa gástrica


b. Es producida por la célula G del cuerpo gástrico
c. Es estimulada por la distensión gástrica y el Ph bajo
d. Actúa en la célula diana mediante su receptor CCk tipo B
15. Al recibir un paciente con signos de hipovolemia y antecedente de trauma en abdomen por
accidente de tránsito, usted identifica radiológicamente: lesión de primera vértebra lumbar y
signos de lesión en páncreas; durante la cirugía se observó pobre irrigación de asas
intestinales. El vaso afectado es la arteria ……..
(unidad 1, sesión 1, logro 6: reconocer las estructuras a nivel de L1, nivel de los principales vasos
sanguíneos)

a. esplénica
b. hepática común
c. mesentérica inferior
d. mesentérica superior

16. Un paciente sufre de daño a nivel del cuello con lesión muscular en la región de la faringe. En
el examen físico se determina dificultad para la elevación de la faringe y para el cierre del
itsmo de las fauces. En este caso, probablemente esté afectado el músculo:
(unidad 2, sesión 11, logro 2: Músculos de la faringe: identificación, constrictores y longitudinales)

A
a. palatogloso

EK
b. estilofarinfeo
c. palatofaringeo
d. constrictor inferior

17. Varón de 50 años a quien le realizan la curación de la segunda molar de la arcada


superior derecha. En un momento determinado, el paciente acusa de intenso dolor de la
OT
pieza dentaria en tratamiento. La vía aferente del dolor viaja a través del nervio …………
(unidad 2, sesión 9, logro 6: Inervación de los dientes)

a. trigémino V2
b. trigémino V3
c. naso palatino
NC

d. palatino menor

18. La distención gástrica por los alimentos produce incremento de secreción de HCl mediante la
producción de
………….. que estimula a las células ……………. vía proteína ………..
BA

(Unidad 1, sesión 3, logro 2: Describe las hormonas gastrointestinales: Estímulo y funciones de la


gastrina y colecistoquinina)

a. gastrina / parietal / Gq
b. gastrina / principal / Gs
c. acetilcolina / parietal /Gi
d. acetilcolina / principal / Gi

19. Un niño de tres años llega a emergencia con disfagia (dificultad para tragar), dolor retro
esternal, salivación y llanto. Se sospecha de ingesta de cuerpo extraño (moneda) en el
esófago; al ser evaluado se constata en una radiografía presencia de cuerpo extraño a nivel
de C6 (6° vértebra cervical). El cuerpo extraño estará suspendido a nivel del estrechamiento
producido por………..
(unidad 2, sesión 11, logro4: Esófago, características anatómicas, relación con órganos vecinos y
estrecheces)

a. el cayado aórtico
b. el hiato esofágico
c. el músculo cricofaríngeo
d. el bronquio principal izquierdo

20. La triada portal (arteria hepática, vena portal y conducto biliar común) está contenida en el
ligamento
…….……… y derivan embriológicamente del ……
(Unidad 1, sesión 1, logro 4: Identifica el peritoneo, mesenterio, omento y ligamentos,
retroperitoneo.)

a. hepato duodenal / mesenterio ventral


b. gastro esplénico / mesenterio dorsal
c. hepato gástrico / omento menor
d. falciforme / omento menor

A
EK
OT
NC
BA
21. En relación al movimiento de
peristaltismo del tubo digestivo: en
la flecha negra del gráfico se
produce la liberación de ………………
a nivel del músculo ………..
(unidad 2, sesión 7, logro 6: Control
hormonal y tipos de movimiento)

a. noradrenalina, sustancia P y neuropéptido “ Y” / circular


b. acetilcolina y sustancia P / longitudinal
c. óxido nítrico y PIV / longitudinal
d. óxido nítrico y PIV / circular

A
EK
22. Un paciente refiere no percibir algunos sabores, al examen físico se constata alteración en
la percepción de sabores y del dolor en el tercio posterior de la lengua ¿Cuál de los
siguientes nervios estará alterada en su función?
(unidad 2, sesión 10, logro 5: Sabores, tipos y mecanismos moleculares para su detección)

a. Lingual (rama del V par)


OT
b. Cuerda del tímpano (VII par)
c. Glosofaríngeo (IX par)
d. Hipogloso (XII par)

23. El gráfico detalla la estructura de


la pared del tubo digestivo
NC

intestinal ¿Cuál de las siguientes


asociaciones es correcta?
(unidad 2, sesión 7, logro 1: La
pared y músculo liso
gastrointestinal )
BA

a. “1” – peristaltismo
b. “2” – secreción enzimática
c. “3” – deriva del mesodermo
d. “4” – doble hoja de tejido graso

24. En el caso de un paciente con gastrinoma (tumor productor de gastrina), la presencia de


úlceras duodenales y erosión de la mucosa gástrica, se debe principalmente a…….
(unidad 1, sesión 3, logro 2: describir las hormonas gastrointestinales: estímulo y funciones de la
gastrina y colecistoquinina)

a. la acción directa de la gastrina sobre la célula principal


b. la sobre expresión de los receptores “G” en la célula parietal
c. el exceso de HCl por estímulo de receptores CCK-B en la célula parietal
d. el exceso de HCl por estímulo directo de receptores de acetilcolina en la célula parietal

25. El reflejo entero gástrico se caracterizan por:


(unidad 2, sesión 13, logro 6: Motilidad del intestino grueso: contracciones segmentarias,
movimientos en masa defecación y reflejo gastrocólico)

a. favorecer la motilidad gástrica gracias a la CCk


b. inhibir la motilidad gástrica y estimular la secreción ácida
c. movilizar grandes volúmenes desde el estómago al duodeno
d. originarse debido a la distensión duodenal y presencia del quimo ácido
26. Mauricio tiene dificultad para deprimir el paladar y elevar la parte posterior de la lengua. En
este caso estará afectado un músculo ………………., específicamente el músculo …………….
(Unidad 2, sesión 10, logro 2: Músculos de la lengua: clasificación, identificación y sus funciones)

a. intrínseco – longitudinal inferior


b. extrínseco – palatogloso
c. extrínseco – transverso
d. extrínseco – estilogloso

27. Una de las funciones del músculo señalado es:


(Unidad 2, sesión 8, logro 3: Describir el Piso de la boca:
estructuras blandas que la conforman)

A
a. deprimir la lengua
b. elevar el paladar blando

EK
c. deprimir el hioides cuando la mandíbula está fija
d. deprimir la mandíbula cuando el hioides está fijo
OT
28. Paciente varón de 30 años es evaluado por probable enfermedad de Chagas, cursa con
problemas de motilidad del colon; los estudios de biopsia determinan ausencia de
células ganglionares. Según el gráfico
¿cuál es la capa en la que se determina la ausencia de dichas células?
(unidad 1, sesión 2, logro 1: describir las generalidades de la estructura del tubo digestivo: esófago,
NC

estómago intestino delgado y grueso)

a. Mucosa - 1
b. Muscular propia – 1
c. Muscular de la mucosa - 2
BA

d. Muscular propia - 3

1 2

3
29. Paciente varón de 32 años, que acude a centro de salud por presentar de forma progresiva
desde hace 1 año dificultad para ingerir alimentos sólidos y luego líquidos; refiere
regurgitaciones alimentarias y marcada pérdida de peso (15 kilos). Radiografía baritada
(sustancia de contraste) de esófago se muestra en la figura. El presente caso se explica
por……………….
(Unidad 2, sesión 12, logro 4: Identificar y describir la función de los esfínteres esofágicos)

a. aumento de la peristalsis esofágica


b. relajación incompleta del esfínter pilórico
c. relajación incompleta del esfínter esofágico inferior
d. perdida de producción de PIV y óxido

A
nítrico en el esfínter esofágico superior

EK
OT
30. En condiciones normales, el ingreso de 600 ml de líquido es el estómago provoca un
aumento de presión intragástrica de unos 12 cm de H2O. Después de una vagotomía (corte
del nervio vago) es de esperar que el ingreso del mismo volumen de líquido ocasionede la
NC

presión intragástrica.
(Unidad 2, sesión 13, logro 1: Describe la Motilidad gástrica: relajación receptiva)

a. la disminución
b. la no variación
BA

c. un aumento mayor
d. un aumento similar o igual
SIS
TEMA
DIGESTIVO
(ME 154)
CLAVES
EXAMEN
FINAL
Ciclo 201900

1. Un niño de 2 años es llevado a la consulta por diarrea persistente, edema de las extremidades y
falta de crecimiento en relación a su edad. Los análisis de sangre revelan que tiene concentración
plasmática baja de proteínas (hipoproteinemia). Como parte del estudio se coloca colecistokinina
(CCK) endovenosa y se recoge muestras del líquido duodenal por endoscopía; el resultado del
líquido confirma incapacidad para hidrolizar proteínas a un pH neutro, esta situación mejora al
añadir una pequeña cantidad de tripsina. El paciente probablemente esté sufriendo la falta

A
congénita de ………….
(Unidad 4, sesión 26, logro 2: Explicar la Digestión y absorción de las proteínas y sus alteraciones)
a. PEPT-1

EK
b. pepsinógeno
c. enterocinasa
d. carboxipeptidasas

2. Paciente mujer de 35 años acude a consulta por sensación de sequedad y lesiones en cavidad
OT
oral. Al examen se observa atrofia de la mucosa, fisuras y úlceras; nota además sequedad e
irritación de la córnea y aumento del tamaño de las glándulas parotídas. Su diagnóstico más
probable es artritis reumatoide; el hallazgo más probable en una biopsia de glándula parótida
es……..….
(Unidad 3, sesión 18, logro 3: Describe las enfermedades más frecuentes de las glándulas salivales)
a. Presencia de acinos normales con hiperplasia de células ductales
NC

b. Gran infiltración de linfocitos y células plasmáticas


c. Hiperplasia de acinos glandulares serosos
d. Gran infiltrado de linfocitos y neutrófilos

3. Un hombre de 42 años de edad se presenta al médico con una historia de 1 año de evolución,
caracterizado por dolor abdominal bajo y diarreas con crisis sanguinolentas. Manifiesta además
BA

pérdida de peso de 8 kg durante este periodo. La colonoscopía revela lesión difusa en el colon
con afectación del recto. La biopsia de estas lesiones revela adelgazamiento de la pared,
inflamación y ulceración de la mucosa y sub mucosa. El diagnóstico más probable en este caso es:
(Unidad 4, sesión 28, logro 5: Describe la Enfermedad inflamatoria intestinal. Generalidades,
morfología y características)
a. sindrome de colon irritable
b. enfermedad de Crohn
c. colitis ulcerativa
d. sprue celiaco

4. Dos estudiantes deciden tomar un receso para comer una hamburguesa a la hora del almuerzo.
Antes de llegar a la cafetería, impulsos nerviosos provenientes del complejo vagal dorsal iniciarán
la secreción de ácido gástrico por la liberación dedesde el sistema nervioso entérico.
(Unidad 3, sesión 20, logro 2: Regulación de la secreción gástrica: estimulación, fases de la secreción)
a. Serotonina
b. Colecistoquinina
c. Péptido inhibidor vaso activo
d. GRP (péptido liberador de gastrina)

5. Un niño de cuatro años de edad es llevado a la consulta por cuadros diarreicos frecuentes
caracterizados por heces pálidas, voluminosas y fétidas; al examen físico presenta bajo peso
y talla para la edad. Se mide la concentración de cloruro en el sudor y se encuentra que sus
valores son muy elevados. La alteración más
importante a nivel de células ductales del páncreas tiene relación directa con la conductancia de…………
(Unidad 3, sesión 23, logro 5 Explica la Secreción pancreática: formación del jugo
pancreático, influencia de la velocidad de flujo y regulación)
a. Bicarbonato
b. Potasio
c. Sodio
d. Cloro

A
EK
OT
NC
BA
6. Se evalúa los valores séricos de las siguientes sustancias a un paciente con enfermedad hepática
terminal; en este paciente se espera encontrar la combinación con la letra …………
(Unidad 3, sesión 22, logro 5: Describe las Pruebas de función hepática, la Insuficiencia
hepática, encefalopatía hepática e hipertensión portal)
Glucosa Amoniaco Albúmin
a
a. Aumenta Disminuida Disminui
da da
b. Disminui Aumentada Aumenta
da da
c. Aumenta Aumentada Aumenta
da da
d. Disminui Aumentada Disminui
da da

A
7. Una mujer de 35 años de edad HIV positiva, se presenta al médico con dolor abdominal en
cuadrante superior derecho e ictericia. La paciente refiere haber tenido múltiples episodios de

EK
ictericia durante los últimos 10 años. Los exámenes para determinar hepatitis viral, dieron
positivos para Hepatitis B, siendo catalogado el caso como hepatitis crónica con alteración
funcional. En un examen de sangre ¿cuál de los siguientes parámetros está disminuido?
(unidad 3, sesión 22, logro 5: Pruebas de función hepática, Insuficiencia hepática,
encefalopatía hepática e hipertensión portal)
a. Albumina
OT
b. Bilirrubina
c. Fosfatasa alcalina
d. Tiempo de protrombina

8. En el reflejo peristáltico del intestino delgado ¿Cuál de los siguientes eventos sucede en la
porción caudal del bolo alimenticio?
NC

(Unidad 2, sesión 13, logro 4: Explicar la Motilidad del intestino delgado: Contracciones
segmentarias y peristálticas)
a. Acción del péptido inhibidor vasoactivo (VIP) en el músculo circular
b. Acción del NO (óxido nítrico) en el músculo longitudinal
c. Contracción del músculo longitudinal interno
BA

d. Acción de acetilcolina en el músculo circular

9. Un varón de 58 años de edad con enfermedad de Crohn severo fue sometido a una resección
ileal. Después de la cirugía este paciente padecerá de esteatorrea, esto se explica porque
…..………..
(unidad 4, sesión 26, logro 4: Explica las alteraciones en la Absorción de lípidos)
a. se inhibe la acción de la 7 alfa hidroxilasa
b. el pool de ácidos biliares se incrementa
c. hay mala absorción de ácidos biliares
d. el páncreas no secreta lipasa

10. En un experimento se inserta un balón en el estómago de un voluntario, se infla poco a poco


mientras que se vigilan las presiones intraluminales. Aunque el volumen del balón aumenta
considerablemente, las presiones permanecen constantes. Esta relación volumen-presión se
explica por la liberación local de …………..
(Unidad 2, sesión 13, logro 1 Explica la Motilidad gástrica: relajación receptiva)
a. acetil colina y gastrina
b. norepinefrina y óxido nítrico
c. colecistoquinina y óxido nítrico
d. óxido nítrico y péptido inhibidor vasoactivo

11. ¿Cuál de las siguientes alternativas es una característica de la secreción exocrina del páncreas?
(Unidad 3, sesión 23, logro 5: Secreción pancreática: formación del jugo pancreático, influencia de la
velocidad de flujo y regulación)
a. Es hipotónica respecto al plasma
b. Su mayor estímulo se da en la fase intestinal
c. Es estimulada por la presencia de bicarbonato en el duodeno
d. La secreción enzimática es estimulada principalmente por la secretina
12. Las estructuras en el hígado que permite que los productos metabólicos unidos a proteínas
tengan acceso a las membranas basolaterales de los hepatocitos, son…..
(Unidad 3, sesión 21, logro 4-5: Explica la Organización micro estructural del hígado)
a. los canalículos
b. las células de Ito
c. las fenestras sinusoidales
d. las uniones intercelulares herméticas

13. La composición de la bilis es modificada conforme fluye por los conductillos biliares.
Durante este tránsito se espera que aumente la concentración de…….
(Unidad 3, sesión 22, logro 2: Describe la Secreción biliar, visión general del sistema biliar
extrahepático y composición de la bilis)
a. Ig A
b. Glucosa
c. Protones
d. Vitamina A

A
14. Se mide experimentalmente el contenido gástrico de dos personas. La persona “A” tiene alto
contenido de grasa y la persona “B” tiene un contenido isotónico ¿Cuál de las siguientes es
correcto respecto al vaciamiento gástrico? (Unidad 2, sesión 13, logro 2: Describe la Motilidad y

EK
vaciamiento gástrico)
a. Hay ralentización del vaciado gástrico solo en “A”
b. El vaciamiento gástrico es más rápido en ambos
c. Hay ralentización del vaciado gástrico solo en “B”
d. Hay ralentización del vaciado gástrico en ambos casos
OT
15. El examen endoscópico de un paciente con hipertensión portal grave revela venas tortuosas que
sobresalen hacia la luz del esófago. El paciente recibe tratamiento quirúrgico mediante la
colocación de una derivación que conecta la vena porta a la vena cava. Después de la operación
el riesgo de encefalopatíay el riesgo
de sangrado de várices ……………..
NC

(Unidad 3, sesión 22, logro 5: Describe la Insuficiencia hepática, encefalopatía hepática e hipertensión
portal)
a. disminuirá / disminuirá
b. disminuirá / aumentará
c. aumentará / disminuirá
BA

d. aumentará / aumentará

16. Un paciente varón de 18 años de edad acude al médico para sus exámenes de rutina. Sus
resultados de laboratorio muestran un valor de bilirrubina sérica de 4 mg/dl y una bilirrubina
directa de 0,3 mg/dl. Las pruebas de función hepática son normales. La alteración que explica
mejor este caso es por la deficiencia de ………………..
(Unidad 3, sesión 22, logro 3: Explica la Producción y excreción de bilirrubina. Tipos de bilirrubina e
ictericia)
a. transaminasas
b. hemo oxigenasa
c. la 7 alfa hidroxilasa
d. glucuronil transferasa

17. Un hombre de 57 años de edad es llevado a urgencias con hematemesis masiva rojo brillante, a
su llegada se halla inconsciente con PA: 80/40 mm Hg y FC: 124 lat/min. Luce ictérico con
presencia de “arañas vasculares en el tórax anterior y extremidades”, abdomen distendido con
signo de oleada positiva. Se encuentra esplenomegalia y pérdida de la masa muscular en
extremidades. La anastomosis vascular responsable del sangrado en este paciente es ………….…..
(Unidad 3, sesión 21, logro 2: Describe las anastomosis porto sistémicas)
a. vena gástrica izquierda y vena ácigos
b. arteria gástrica izquierda y vena ácigos
c. vena paraumbilical y vena epigástrica inferior
d. vena gástrica izquierda y vena esofágica superior
18. Un estudiante de medicina está comiendo un plato de comida a base de champiñones,
espárrago y salsa de soya. El estímulo del sabor umami contenido en todos estos alimentos viaja
a través del nervio………………..
(Unidad 2, sesión 10, logro 3: Describe la irrigación e inervación de la lengua)
a. Lingual
b. Hipogloso
c. Glosofaringeo
d. Cuerda del tímpano

A
EK
OT
NC
BA
19. Una paciente de 30 años de edad es sometida a una cirugía en oído medio derecho por un
problema de otoesclerosis. Luego de la cirugía refiere alteración sensitiva de la lengua. Al
evaluar el caso usted esperaría encontrar……….
(Unidad 2, sesión 10, logro 5: Describe la Irrigación e inervación de la lengua)
a. Alteración en la sensación del dolor y temperatura en el tercio posterior de la lengua
b. Alteración en la sensación del dolor en los dos tercios anteriores de la lengua
c. Alteración en la sensación del gusto en el tercio posterior de la lengua
d. Sensación del dolor, tacto y temperatura conservadas

20. En un paciente de 45 años de edad con colestasis biliar, se encuentra una elevación de los niveles
sanguíneos de fosfatasa alcalina hasta 3 veces la cifra normal. ¿Cuál de las siguientes alternativas
estará también elevada como evidencia del daño de la vía biliar?
(Unidad 3, sesión 22, logro 5: Pruebas de función hepática, Insuficiencia hepática,
encefalopatía hepática e hipertensión portal)
a. Tiempo de protrombina y albúmina sérica
b. Transaminasas hepáticas (ALT y AST)

A
c. Gamma glutamil transpeptidasa
d. Glucoronil transferasa

EK
21. Experimentalmente se incrementa la velocidad de la secreción salival con una sustancia,
en el análisis de la composición de esta saliva obtenida se espera encontrar…………..
(Unidad 3, sesión 17, logro 5 : Explica la Influencia de la velocidad del flujo salival en la composición de la
saliva)
a. disminución de la concentración de bicarbonato que supera la concentración plasmática
OT
b. aumento de la concentración de cloro y sodio que supera la concentración plasmática
c. aumento de la concentración de bicarbonato que supera la concentración plasmática
d. disminución de concentración de potasio y bicarbonato

22. Lactante de 3 meses de vida es atendido por presentar diarrea, se administra una
solución de glucosa y electrólitos por vía oral. La proteína de membrana apical que
NC

explica la capacidad de esta solución para proporcionar aporte de glucosa e hidratación


es ………..
(Unidad 4, sesión 26, logro 1: Explica la Digestión y Absorción de los hidratos de carbono. Alteraciones)
a. CFTR
b. SGLT-1
BA

c. GLUT-2
d. GLUT-5

23. Paciente ha sufrido herida de bala en el abdomen, se le ha tenido que extirpar el segmento
medio y distal del ileon. En este caso la síntesis hepática de sales biliares estará …..…..
(Unidad 3, sesión 22, logro 4: Explica la formación, función y Circulación entero hepática de lasa sales
biliares)
a. Sin cambios en el ritmo de síntesis
b. Disminuida por inhibición de la enzima colesterol 7 alfa hidroxilasa
c. Incrementada por estímulo de la enzima colesterol 7 alfa hidroxilasa
d. Incrementada por inhibición de la enzima colesterol 7 alfa hidroxilasa

24. Un varón de 75 años ingresa al consultorio por presentar ictericia marcada de piel y las
escleras. El estudio del paciente mostró que presentaba un tumor que obstruía la totalidad del
conducto hepático común. ¿Cuál de los siguientes conductos se encontrará dilatado en este
paciente?
(Unidad 3, sesión 21, logro 6: Describir el árbol biliar intrahepático)
a. de Wirsung
b. de Hering
c. colédoco
d. cístico
25. Correlaciones las dos columnas y marque la fórmula correcta:
(Unidad 4, sesión 28, logro 1: Diarrea: definición, mecanismos: osmótica, secretoria y exudativa)

1. Enfermedad Hirschsprung( ) heces con moco y sangre


2. Diarrea osmótica( ) intolerancia a lactosa
3. Diarrea secretoria( ) aganglionosis congénita
4. Diarrea exudativa( ) canales de Cl- en las células de la cripta a.- 4231b.-

1234c.- 2143d.- 4213

26. La vena umbilical obliterada del hígado después del nacimiento se transforma en el ligamento:
(Unidad 3, sesión 21, logro 1: Hígado: relación con la pared abdominal, caras, lóbulos, ligamentos , hilio
hepático)
a. cruzado
b. redondo
c. coronario
d. falciforme

A
27. Llega a su guardia nocturna una madre que trae a su RN masculino de 2 semanas de vida con mal
estado general y sequedad de mucosas. Usted observa que lacta ávidamente, pero a las 2 horas

EK
presenta vómito postprandial no bilioso en proyectil. Al realizar la historia clínica, descubre que el
lactante recibió profilaxis con macrólidos para tos ferina. Usted sospecha principalmente en:
(Unidad 1, sesión 4, logro 4: Desarrollo y anomalías del intestino anterior)
a. estenosis pilórica hipertrófica congénita
b. fistula traqueo esofágica
OT
c. estenosis duodenal
d. atresia duodenal

28. En la regulación del apetito y la saciedad, la estimulación experimental crónica del núcleo
ventro medial del hipotálamo producirá:
(Unidad 1, sesión 3, logro 4: explica los mecanismos de control del apetito y saciedad)
NC

a. afagia
b. obesidad
c. hiperfagia
d. activación de neuronas relacionadas a NPY

29. Paciente mujer de 25 años acude por dolor en fosa ilíaca derecha que empeora al toser o
BA

caminar, asociada a náuseas y vómitos por lo cual acude a emergencia. Dos días después de
realizarle una apendicectomía, la paciente desarrolla fiebre alta (39 °C), está hipotensa y presenta
dolor abdominal. La laparotomía exploratoria muestra un gran volumen de sangre en la cavidad
peritoneal por lesión de un vaso producida durante la apendicectomía.
¿Cuál de las siguientes arterias debe ligarse para detener la hemorragia?
(Unidad 4, sesión 27, logro 4: Irrigación arterial del colon, recto y conducto anal)
a. cólica derecha y arteria rectal superior.
b. ileocólica y arteria cólica media.
c. mesentérica superior.
d. ileocólica.

30. La onda peristáltica secundaria del esófago se caracteriza por ser originada ………
(unidad 2, sesión 12, logro 3: Motilidad esofágica: fases y características)
a. por el plexo de meissner del esófago
b. por el plexo mientérico del esófago
c. por el reflejo de la deglución
d. durante la masticación

31. ¿Cuál de los siguientes es una causa de ictericia con bilirrubina conjugada aumentada?
(Unidad 3, sesión 22, logro 3: Producción y excreción de bilirrubina. Tipos de bilirrubina, ictericia)
a. Ictericia del recién nacido
b. Obstrucción del colédoco
c. Anemia hemolítica
d. Gran hematoma
32. En relación a la absorción de nutrientes, la absorción de dipéptidos y tripéptidos a nivel de las
células epiteliales del intestino delgado, se da principalmente debido a:
(Unidad 4, sesión 26, logro 2: Digestión y absorción de las proteínas. Alteraciones)
a. el incremento de los canales de Cl- en la membrana apical
b. la gradiente de bicarbonato en la membrana basal
c. la gradiente de iones H+ en la membrana apical
d. la gradiente de Na+ en la membrana apical

33. Paciente de 20 años es traído a la emergencia por presentar diarreas desde hace 2 días. Familiar
refiere que las deposiciones son líquidas y abundantes, al examen luce deshidratado y se plantea
que la diarrea es producida por una toxina que estimula la transformación de ATP a AMPc con
apertura de canales de Cl- y pérdida de agua. El tipo de diarrea más probable es:
(Unidad 4, sesión 28, logro 1: Diarrea: definición , mecanismos: osmótica, secretoria y supurativa)
a. osmótica
b. exudativa
c. secretoria
d. por intolerancia a lactosa

A
34. Un niño fue operado por una obstrucción intestinal, observándose la presencia de divertículo de
Meckel. Según lo referido, marque lo correcto:

EK
(Unidad 1, sesión 5, logro 2: identifica las anomalías del desarrollo del intestino medio: onfalocele y
gastrosquisis (diferencias), Divertículo de Meckel)
a. el 50% de la población lo presenta
b. se localiza en el íleon muy cerca al yeyuno
c. puede poseer tejido gástrico o pancreático
OT
d. se produce por una mala rotación de los intestinos

35. Marque la alternativa


correcta respecto a la
estructura marcada en el
gráfico:
NC

(Unidad 3, sesión 22, logro 2:


Secreción biliar. Visión general
del sistema biliar extrahepático
y composición de la bilis)

a. Se halla a 2
BA

centímetros debajo
de la papila
duodenal mayor
b. Llega el conducto
colédoco y pancreático
principal
c. Llega el conducto
hepático común y
pancreático
principal
d. Llega el conducto pancreático accesorio

36. ¿Cuál de las siguientes moléculas se encontrará aumentada en el citoplasma de las células
parietales de un paciente con sindrome de Zollinguer Ellison?
(Unidad 3, sesión 20, logro 4: Enfermedad ulcerosa péptica: úlcera gástrica, duodenal.
síndrome de Zollinger – Ellison)
a. Péptido liberador de gastrina (GRP)
b. Proteína G estimulante (GS)
c. Inositol Trifosfato (IP3)
d. AMP cíclico (AMPc)

37. Los fármacos inhibidores de la bomba de protones, actúan bloqueando la ………..……..


(Unidad 3, sesión 20, logro 3: Regulación de la secreción gástrica: inhibición, Secreción de
pepsinógeno y factor intrínseco)
a. anhidrasa carbónica
b. ATPasa H+/K+ en la membrana luminal
c. ATPasa H+/K+ en la membrana basolateral
d. ATPasa Na+/K+ en la membrana basolateral

A
EK
OT
NC
BA
38. Un paciente fue diagnosticado de gastritis autoinmune, ¿cuál de las siguientes alternativas es
FALSA respecto a esta enfermedad?
(Unidad 3, sesión 20, logro 5: Gastritis crónica: helicobacter pylori, autoinmune. Tipos de gastritis)
a. Afecta principalmente el fondo y cuerpo gástrico
b. Se produce hiperplasia de células G secundaria a la aclorhidria
c. El propio sistema inmune destruye principalmente las células parietales
d. Se produce atrofia de la mucosa, aclorhidria, hipergastrinemia y déficit de vitamina B6

39. Marque la correlación correcta:


(Unidad 3, sesión:18, logros:1 y 2: Describe las enfermedades inflamatorias/infecciosas y
proliferativas de la cavidad oral)

1. Herpes virus( ) En relación al abuso de antibióticos


2. Candidiasis oral( ) Lesiones vesiculares como racimo de uvas
3. Eritroplaquia( ) Mega esófago
4. Enfermedad de Chagas( ) Lesión pre cancerígena

A
a.- 2431b.- 1234c.- 4123d.- 2143

EK
40. En un paciente con insuficiencia renal crónica, el déficit en la absorción de calcio a nivel del
enterocito se debe a lo siguiente:
(Unidad 4, sesión 26, logro 6: Explica la Absorción de calcio y hierro)
a. No se convierte la 25 hidroxicolecalciferol a 1,25 dihidroxicolecalciferol
b. No se convierte la 1,25 dihidroxicolecalciferol a 25 hidroxicolecalciferol
OT
c. Existe un descenso de la alfa 25 hidroxilasa renal
d. Se incrementa la producción de Calbindina
NC
BA
SISTEMA DIGESTIVO
(ME154) EXAMEN
PARCIAL 2019 01

Profesores : Alfaro Salazar, Herberth Romulo; Callata Caceres, Gunter; Cayo Quiñe, Alexandra Mariel; Correa
Borit, Jorge Mauricio; Cruz Cutty, Lourdes Marylin; Guzmán Calderón, Gerly Edson; Jáuregui
Farfán, Jorge Jesús; Mayor Zevallos, Otto Alberto; Montoya Suárez, José Luis; Palacios Bazan,
Enrique Elias; Robles Pino, Alexander Anibal; Wong Bravo, Juan Carlos
Sección : Todas las secciones
Duración : 50 minutos.
Indicaciones:
Lea atentamente cada pregunta antes de responder:
- Se prohíbe el uso del celular y cualquier dispositivo electrónico.

A
- Está prohibido intercambiar materiales.
- Coloque su código de alumno en la tarjeta de respuestas. Si su código contiene una letra reemplácela por
un valor numérico siguiendo la siguiente equivalencia: A=9, B=8, C=7, D=6, E=5, F=4, G=3.

EK
- Traslade sus respuestas a la tarjeta, llenando los círculos de manera completa con lapicero negro o azul.
Está prohibido el llenado con lápiz, lapicero de otro color o con lapicero de tinta borrable.
- Sea cuidadoso en el llenado de la tarjeta de respuestas, pues solo esta tiene validez para la calificación.
- Al terminar su examen avise al docente a cargo, no se levante de su sitio; debe entregar la hoja de
respuestas con la carátula del examen, este cuadernillo de preguntas se lo llevará cada estudiante.
OT
1. ¿Cuál de las glándulas salivales es responsable del mayor porcentaje del volumen de la saliva en
condiciones basales?
a. Parótida
b. Subpalatin
as c.
NC

Sublingula
es d.
Submaxilar
es

2. La lengua está recubierta por epitelio:


BA

a. pseudoestratificado columnar no queratinizado


b. plano estratificado no queratinizado
c. pseudoestratificado columnar ciliado
d. plano estratificado queratinizado

3. El esfínter anal interno tiene musculatura …………… y tiene control ………………….


a. lisa / voluntario
b. lisa / involuntario
c. esquelética / simpático
d. esquelética / parasimpático

4. La arteria aorta proporciona la irrigación al tubo digestivo ¿cuál de las siguientes arterias proporciona la
irrigación al ángulo cólico derecho?
a. Mesentérica superior
b. Mesentérica inferior
c. Frénica inferior
d. Tronco celiaco

5. Paciente de 26 años que le cuenta en su historia clínica que cada vez que almuerza, a los 20 minutos
tiene deseo de defecar. Le comenta que su hijo de 1 mes le pasa lo mismo pero más intenso. Esto se
explica por el reflejo
…………………, el cual estáen el paciente.
a. colicoileal / normal
b. colicoileal / alterado
c. gastrocolico / normal
d. gastrocolico / alterado
6. La región del estómago que se comunica con el duodeno se denomina:
a. pilórica
b. cardias
c. cuerpo
d. fórnix

7. Acude a consulta un paciente que fue diagnosticado de ulcera péptica 3 días antes. Luego de múltiples
pruebas diagnósticas, se concluye que el paciente presenta un tumor secretor de gastrina ¿Cuál de las
siguientes situaciones estará incrementada?
a. Distención gástrica
b. Inhibición del vaciado gástrico
c. Secreción de ácido clorhídrico (HCl)
d. Inhibición de la secreción de pepsinógeno

8. En el sistema digestivo, el control del apetito esta dado por un complejo sistema de sustancias y órganos

A
integradores los cuales regulan la ingesta de alimentos. Laes una sustancia oroxígena y es sintetizada
por el ……………………..
a. leptina / intestino

EK
b. grelina / intestino
c. leptina / estómago
d. grelina / estómago

9. Sobre el control autónomo del sistema digestivo, marque la alternativa correcta:


a. La inervación dada por el sistema simpático es de tipo preganglionar.
OT
b. El sistema parasimpático usa como neurotransmisores a la acetilcolina y la noradrenalina.
c. El nervio vago (par craneal X) le da inervación simpática a la mayoría del sistema digestivo.
d. En el sistema simpático, los nervios responsables hacen una primera sinapsis en ganglios
próximos al órgano a inervar.
e. En la inervación de tipo parasimpático, solo interviene el plexo submucoso, sin embargo, en la
de tipo simpático intervienen tanto el submucoso como el mientérico.
NC

10. Con respecto a la actividad eléctrica del sistema digestivo, marque la alternativa correcta
a. Corresponden a potenciales de acción que están presentes de forma continua y le dan la
capacidad de perístasis autónoma al sistema digestivo.
b. La frecuencia de las ondas lentas no se ve influenciada por la actividad neural ni las
BA

hormas gastrointestinales.
c. En el estómago las ondas lentas se dan en una frecuencia de 6 por minuto.
d. Las ondas lentas son cambios lentos y ondulantes del potencial en reposo.
e. La frecuencia de las ondas lentas va de 6 a 12 ondas por minuto.

11. Ante una lesión del IX par craneal, el músculo…se altera en su función.
a. palatogloso
b. estilofaríngeo
c. palatofaríngeo
d. constrictor superior

12. Un varón de 50 años es sometido a extirpación de duodeno y parte proximal de yeyuno. La pérdida de
estímulo hormonal en el páncreas para la secreción enzimática se explica por la pérdida de las células
……………………
a. Parietales, productoras de factor intrínseco
b. “K” productoras de factor intrínseco
c. “M” productoras de CCK
d. “I” productoras de CCK
13. Respecto al mecanismo de la defecación ¿Cuál de las siguientes afirmaciones es correcta?
a. Se produce contracción refleja del esfínter anal interno
b. Se produce contracción o relajación del esfínter anal externo por señales de la corteza cerebral
c. La presencia de materia fecal en el recto estimula la contracción del sigmoides por los nervios
pélvicos simpáticos
d. En la posición de “cuclillas” el músculo puborectal se halla contraído favoreciendo la
evacuación de la materia fecal

14. Un niño de tres años llega a emergencia con disfagia (dificultad para tragar), salivación y llanto. Se sospecha
de ingesta de cuerpo extraño: moneda en el esófago; al ser evaluado se constata en una radiografía presencia
de cuerpo extraño a nivel de C6 y C7 (6° y 7° vértebra cervical). El cuerpo extraño estará suspendido a nivel
del estrechamiento producido por el ………..
a. cayado aórtico
b. hiato esofágico
c. músculo cricofaríngeo
d. bronquio principal izquierdo

A
15. En el caso de un paciente con un tumor productor de gastrina, la presencia de úlceras duodenales y

EK
erosión de la mucosa gástrica se debe principalmente a…….
a. la acción paracrina de la gastrina sobre la célula parietal
b. el exceso de HCl por estímulo de receptores CCK-B en la célula parietal
c. la sobre expresión de los receptores “G” para gastrina en la célula parietal
d. el exceso de HCl por estímulo directo de receptores “H” en la célula parietal
OT
16. La onda peristáltica secundaria del esófago se caracteriza por ser originada ………
a. por el plexo de submucoso del esófago
b. por el plexo mientérico del esófago
c. por el reflejo de la deglución
d. durante la masticación
NC

17. Marque lo correcto sobre las ondas lentas en el tubo digestivo


a. No son despolarizaciones
b. Son potenciales de acción subumbrales
c. Se constituyen de despolarizaciones y repolarizaciones
d. Son rítmicas y generadas por el sistema nerviosos autónomo
BA

18. Recién nacido que presenta protrusión de contenidos abdominales los cuales no están cubiertos por
peritoneo y salen de la cavidad abdominal a través de un defecto de la pared. ¿Cómo se denomina a la
afección que presenta este paciente?
a. Onfalocele
b. Atresia biliar
c. Gastrosquisis
d. Divertículo de Meckel

19. Experimentalmente se utiliza atropina (anticolinérgico) para inhibir la secreción de gastrina, sin
embargo, la secreción de esta hormona se sigue dando ante estímulos vagales. Esta situación se explica
porque la atropina:
a. no bloquea la acción del péptido GRP
b. solo inhibe la acción del péptido GRP en la célula G
c. inhibe la acción de acetilcolina e histamina en la célula G
d. bloquea parcialmente la bomba de protones en la célula G

20. Niña de 4 días es llevada a la emergencia pediátrica por presentar llanto constante, la madre refiere
coloración azulada de labios al momento de lactar, acompañado de tos persistente y dificultad respiratoria
así como distención abdominal. Se le coloca sonda nasogástrica para alimentación notando que retorna a
la cavidad oral en todos los intentos. ¿Cuál es la anomalía del desarrollo en este caso?
a. Solo fístula traqueo esofágica
b. Fístula traqueo esofágica proximal y distal
c. Atresia esofágica proximal con fístula traqueo esofágica distal
d. Atresia esofágica distal con fístula traqueo esofágica proximal
21. Paciente varón de 36 años es traído a la emergencia luego de sufrir un accidente de tránsito, presenta
traumatismos múltiples en cabeza y tronco. Al examen físico se evidencia hematoma en hemicara izquierda,
ligera protrusión y caída del lado izquierdo del maxilar inferior, por lo que se realiza una tomografía donde
se halla una fractura de la apófisis coronoides del maxilar inferior. ¿Qué músculo está relacionada
directamente con esta situación?
a. Masetero
b. Temporal
c. Buccinador
d. Pterigoideo medial

22. Un paciente refiere no percibir algunos sabores. Al examen físico constata alteración del sabor dulce y
umami.
¿Cuál de los siguientes nervios estará alterada su función?
a. Cuerda del tímpano (VII par)

A
b. Lingual (rama del V par)
c. Glosofaríngeo (IX par)
d. Hipogloso (XII par)

EK
23. A los pocos días de nacido, regresa a neonatología un niño con problemas de motilidad del colon; los
estudios determinan ausencia congénita de células ganglionares. Según el gráfico ¿cuál es la capa en la que
se determina la ausencia de dichas células?
a. Mucosa - 2
b. Muscular propia - 2
OT
c. Muscular propia - 3
d. Muscular de la mucosa - 3
NC

1 2
BA

24. Con respecto al control autonómico en el tracto gastrointestinal y en relación a su fisiología. ¿Cuál es la
función del sistema nervioso parasimpático en el tracto gastrointestinal?
a. Inhiben la contracción muscular y estimulan la secreción de sustancias a nivel de la submucosa
b. Estimulan la contracción muscular y estimulan la secreción de sustancias a nivel de la mucosa
c. Inhiben la contracción muscular e inhiben la secreción de sustancias a nivel de la submucosa
d. Estimulan la contracción muscular e inhiben la secreción de sustancias a nivel de la mucosa

25. Un estudiante que está preocupado por su examen parcial, no ha desayunado ni almorzado; cuando al fin
ingiere alimentos, esto le provoca el aumento de los movimientos musculares del tracto gastrointestinal y
la sensación de defecar. ¿Qué reflejo se ha activado?
a. Entero-gástrico
b. Gastro-cólico
c. Cólico-ileal
d. Ileo-ileal
26. ¿De qué par craneal es rama el nervio palatino mayor?
a. Vago
b. Hipogloso
c. Trigémino
d. Palatogloso

27. ¿En cuál de las fases de la deglución la epiglotis separa la vía respiratoria de la digestiva?
a. oral
b. laríngea
c. faríngea
d. esofágica

28. Los péptidos intestinales se pueden clasificar como sustancias endocrinas, neurocrinas y paracrinas,
dentro de las paracrinas se encuentran la somastotatina e histamina. Marque la respuesta correcta
a. La somastotatina es sintetizada por las células B de la mucosa gástrica

A
b. La histamina actúa estimulando su receptor tipo H1 en la mucosa gástrica
c. La histamina es sintetizada por células de tipo paracrino de las glándulas gástricas
d. La somatostatina presenta dentro de sus funciones la estimulación de la secreción de H +

EK
29. En relación a los órganos intraabdominales y sus estructuras de fijación, elija la alternativa
correcta a.El mesenterio permite la suspensión e irrigación de los órganos
retroperitoneales
b. Tanto el hígado como la vesícula biliar se encuentran ubicados a nivel del flanco derecho
c. El colon, el duodeno y el resto de intestino delgado son órganos considerados
OT
netamente como peritoneales
d. Los ligamentos que encontramos dentro de la cavidad abdominal son el esplenorenal y el
gastrofrénico e.Los omentos van desde el estómago y la segunda porción del duodeno a otras
estructuras
intraabdominales y existen dos: el omento mayor y el omento menor
NC

30. Paciente varón de 27 años es llevado por bomberos a emergencia luego de ser asaltado y, tras resistirse, es
cortado con el pico de una botella a nivel abdominal. Al examen físico usted observa que a través de la herida
se puede observar la protrusión de asas intestinales. En relación con las capas de la pared abdominal, marque
la alternativa correcta.
a. La fascia de Scarpa está constituida principalmente por tejido adiposo
BA

b. La pared abdominal está formada por piel, huesos, músculos, fascias y peritoneo parietal
c. La fascia de Camper es una estructura fibrosa que carece de grasa y su grosor es constante en
toda la pared abdominal
d. El músculo oblicuo externo discurre en dirección súpero-interna y se inserta en el borde
inferior de las ultimas 3 a 4 costillas
e. El músculo recto del abdomen tiene como funciones comprimir el contenido del abdomen,
tensar la pared del abdomen y flexionar la columna
SISTEMA DIGESTIVO
(ME154) EXAMEN
FINAL 2019 01

Profesores : Alfaro Salazar, Herberth Romulo; Callata Caceres, Gunter; Cayo Quiñe, Alexandra Mariel; Correa
Borit, Jorge Mauricio; Cruz Cutty, Lourdes Marylin; Guzmán Calderón, Gerly Edson; Jáuregui
Farfán, Jorge Jesús; Mayor Zevallos, Otto Alberto; Montoya Suárez, José Luis; Palacios Bazan,
Enrique Elias; Robles Pino, Alexander Anibal; Wong Bravo, Juan Carlos
Sección : Todas las secciones
Duración : 70 minutos.
Indicaciones:
Lea atentamente cada pregunta antes de responder:
- Se prohíbe el uso del celular y cualquier dispositivo electrónico.
- Está prohibido intercambiar materiales.

A
- Coloque su código de alumno en la tarjeta de respuestas. Si su código contiene una letra reemplácela por
un valor numérico siguiendo la siguiente equivalencia: A=9, B=8, C=7, D=6, E=5, F=4, G=3.
- Traslade sus respuestas a la tarjeta, llenando los círculos de manera completa con lapicero negro o azul.

EK
Está prohibido el llenado con lápiz, lapicero de otro color o con lapicero de tinta borrable.
- Sea cuidadoso en el llenado de la tarjeta de respuestas, pues solo esta tiene validez para la calificación.
- Al terminar su examen avise al docente a cargo, no se levante de su sitio; debe entregar la hoja de
respuestas con la carátula del examen, este cuadernillo de preguntas se lo llevará cada estudiante.
OT
1. La explicación fisiológica de presentar somnolencia de 30 minutos a 1 hora después de ingerir
alimentos, se explica por:
a. Aumento del cloro intraluminal
b. Aumento del bicarbonato intraluminal
c. Disminución de ácido carbónico en la célula parietal
NC

d. Disminución de la actividad de la anhidrasa carbónica


e. Aumento de la alcalinidad sanguínea

2. Con respecto a la irrigación arterial del colon, a que arteria corresponde la


señalada con la flecha
a. Cólica derecha
BA

b. Cólica media
c. Cólica izquierda
d. Ileobisecoapendículocólica
e. Arco de Riolano

3. Si un paciente presentara dentro del punto de vista fisiológico, una


disminución de enterocinasa, entonces esto originaría una disminución de
la actividad de:
a. la
pepsina
b.la
lipasa
c. la quimotripsina
d. el peptido insulinotropo dependiente de glucosa
e. la amilasa

Se valida la opción b debido a su relación con la colipasa.


4. Con respecto a la anatomía del hígado, señale a que estructura
pertenece la marcada por el número 1.
a. Ligamento falciforme
b. Línea de Cantlie
c. Ligamento triangular
d. Ligamento coronario
e. Ligamento teres

Se valida la opción a debido a la ubicación del número 1 en donde se


unen el ligamento falciforme y ligamento coronario.

5. Se presenta un paciente, el cual presenta un antecedente de tuberculosis intestinal, por lo cual, se le


resecó 80 cm de íleon distal. Desde el punto de vista fisiológico, el paciente puede presentar una de las
siguientes alteraciones:
a. Disminución de la secreción de Vitamina B12

A
b. Aumento indiscriminado de absorción de ácido fólico
c. Disminución de la absorción de hierro

EK
d. Aumento de la secreción de bicarbonato
e. Disminución de la absorción de ácido glicocólico

6. Un paciente es sometido experimentalmente a un fármaco que modifica el flujo salival, obteniéndose un


volumen de saliva de 288 ml en 6 horas. En este caso las concentraciones de electrolitos y bicarbonato en la
saliva obtenida varían de la siguiente manera:
OT
a.↑ Na+, ↓ K+, ↑ Cl-, ↑
HCO3- b.↓ Na+, ↓ Cl-, ↑
K+, ↓ HCO3-
c.↑ Na+, ↑ Cl-, ↓ K+, ↓ HCO3-
d.↑ Na+, ↑ Cl-, ↑ K+, ↑ HCO3-
e.↓ Na+, ↓ Cl-, ↓ K+, ↓ HCO3-
NC

Se valida la opción a debido a que se puede considerar como un aumento del flujo de saliva.

7. La siguiente imagen histológica corresponde a la glándula


…………… y la estructura señalada produce ………
BA

a. salival sublingual / mucopolisacáridos


b. oxíntica / pepsinógeno
c. salival submaxilar / ptialina
d. salival parótida / amilasas
e. antrales / gastrina

8. Paciente varón de 65 años con antecedente de hipercolesterolemia, hipertensión arterial, fibrilación


auricular y dos infartos al miocardio previos, aqueja de dolor abdominal intenso de inicio súbito,
distensión abdominal, se decide cirugía con resección de 1,5 metros de intestino delgado terminal y colon
ascendente. Como consecuencia de la resección el paciente tendrá deficiencia de:
a. Vitamina C
b. Tiamina
c. Vitamina A
d. Vitamina B1
e. Vitamina B6

Se valida esta opción debido a que su absorción está relacionada al íleon.


9. Uno de los siguientes elementos debería hallarse con más probabilidad en el esófago de un paciente que
sufre de
reflujo gastro esofágico…
a. Pepsina
b. Tripsina
c. Quimiotripsina
d. Carboxipeptidasa
e. Ácidos biliares

10. Un paciente de 40 años cursa con anemia de 8g/dl, aqueja además de astenia y sensación de hormigueo
bilateral en los miembros inferiores, al examen se halla alteración de la sensibilidad a la vibración y camina
con ampliación de la base de sustentación. Uno de los siguientes procedimientos sería de ayuda para el
diagnóstico de este paciente:
a. Tomografía cerebral
b. Biopsia de la mucosa gástrica
c. Biopsia de hígado

A
d. Examen de sangre oculta en heces
e. Biopsia de Ileon proximal

EK
11. Paciente de 60 años ingresa por caída hace 1 hora y pequeño hematoma en cuero cabelludo, al examen
físico ampliado se observa ictericia de piel y mucosas generalizada, abdomen blando, se palpa estructura
quística no dolorosa en hipocondrio derecho que corresponde a vesícula biliar (signo de Courvoisier), en
los exámenes de laboratorio se halla niveles bajos en la formación de estercobilinógeno y urobilinógeno
en heces, incremento de la bilirrubina conjugada en la orina, elevación de fosfatasa alcalina y gamma
OT
glutamil transpeptidasa séricas. El presente cuadro puede ser explicado por:
a. Reabsorción de hematoma
b. Litiasis vesicular
c. Carcinoma de la cabeza de páncreas
d. Carcinoma con estenosis del conducto hepático común
e. Anemia hemolítica
NC

12. Paciente varón de 58 años con antecedente de alcoholismo crónico es diagnosticado y recibe tratamiento
por cirrosis hepática. Hace 2 días refiere familiar que tuvo cambio de conducta y no reconoce a algunos
familiares. Al examen físico, se halla ascitis, circulación colateral en abdomen, telangiectasias, en el
examen de sistema nervioso: rigidez de extremidades, ROT incrementados, desorientación en el espacio y
BA

asterixis. ¿cuál de las siguientes circunstancias, explicaría el cuadro en este paciente?


a. Uso de diuréticos ahorradores de potasio
b. Incremento de actividad de ureasa bacteriana duodenal
c. Hemorragia gastrointestinal
d. Disminución de la producción de NH3+ en el colon
e. Dieta normo proteica

Se valida la opción e debido al efecto sobre la encefalopatía.


Con respecto de la opción b es incorrecta debido a que hace referencia al duodeno, debería indicar colon.

13. Un recién nacido presenta vómitos biliosos poco tiempo después de cada alimento. Al preguntar a la
madre sobre antecedentes, ella recuerda que tuvo polihidramnios durante la gestación, pero un análisis de
cariotipo fue normal. Una de las siguientes es la causa más probable de estos hallazgos en el recién nacido:
a. Enfermedad de Hirschprung
b. Fístula tráqueo esofágica
c. Divertículo ileal
d. Estenosis pilórica
e. Malrotación de la yema pancreática ventral

14. Un lobulillo hepático se puede dividir en tres zonas como se muestra en el gráfico. ¿Cuál de las
siguientes afirmaciones sobre las tres zonas es verdadera?
a. La zona 1 tiene los menores depósitos de glucógeno
b. La zona 3 es la primera en afectarse en una colestasis extra hepática
c. La zona 2 es más susceptible a la injuria por isquemia que la zona
periportal d.La zona 2 tiene la mayor capacidad de regeneración
e.La zona 1 es la que tiene menos actividad metabólica.

A
La pregunta 14 ha sido anulada, sin embargo, ningún estudiante se verá afectado
negativamente en su puntaje debido a esta anulación.

EK
15. En un estudio de la secreción de hormonas gastrointestinales, sus concentraciones en la vena porta se
midieron durante perfusión luminal del intestino delgado con soluciones de diversas magnitudes de pH.
¿Qué hormona aumentará en el plasma de la vena porta durante perfusión a través del intestino con
una solución de pH 3?
a. CCK
OT
b. gastrina
c. GIP
d. motilina
e. secretina

16. Paciente de 30 años que ingresa a causa de un traumatismo abdominal cerrado. En la exploración se
NC

aprecia discreta palidez de piel y mucosas, auscultación pulmonar normal, taquicardia de 120 /min.
Discreta distensión abdominal y matidez en flancos; el hematocrito, que era prácticamente normal al
ingreso, disminuye a 30% a las tres horas. En la Rx de tórax se objetiva fractura de las costillas 10-11
izquierdas. La causa más probable de la anemización en este paciente es:
a. traumatismo renal con hemorragia retroperitoneal.
BA

b. rotura de hígado con hemoperitoneo.


c. rotura de bazo con hemoperitoneo.
d. rotura de mesos con hemoperitoneo.
e. traumatismo pancreático con pancreatitis traumática.

17. Mujer de 65 años. Consulta por síndrome constitucional


asociado a dolor abdominal epigástrico progresivo irradiado a
espalda, de dos meses de evolución. El diagnostico de sospecha
de adenocarcinoma de páncreas se confirma por biopsia. Se
realiza examen de imagen de abdomen para evaluación de
estructuras vasculares próximas al tumor pancreático. ¿Cuál es
el nombre de la vena señalada que está ausente, trombosada
por infiltración tumoral, condicionando circulación colateral en
la pared gástrica?

a. Mesentérica superior
b. Coronaria estomaquica
c. Esplénica
d. Porta
e. Renal izquierda
18. Revisando la angiotomografía de un hombre de 70 años en estudio por aneurisma de aorta abdominal,
el radiólogo le informa de la presencia de una oclusión completa de la arteria mesentérica inferior. El
paciente se encuentra completamente asintomático. La oclusión de la arteria mesentérica inferior
cursa de manera asintomática en muchas ocasiones ya que el territorio que irriga puede recibir flujo
proveniente de la arteria:
a. cólica derecha
b. gastroduodenal
c. Epigástrica inferior izquierda
d. esplénica
e. cólica media

19. En las patologías de esófago es importante conocer bien la anatomía esofágica. ¿Cuál de las
siguientes afirmaciones es correcta?
a. El esófago tiene capa mucosa, muscular y serosa
b. El esófago abdominal es más largo que el cervical
c. El esófago torácico pasa por detrás del cayado aórtico

A
d. El epitelio esofágico normal es de tipo cilíndrico.
e. El esófago abdominal es discretamente más largo que el torácico

EK
20. A pesar de que pueda haber variaciones anatómicas, lo habitual es que el ciego sea irrigado por una rama
arterial que proviene de unas de las siguientes arterias:
a. Iliaca derecha
b. Mesentérica inferior
c. Hepática derecha
OT
d. Mesentérica superior
e. Iliaca izquierda

21. Ante un paciente con una cirugía abdominal urgente, el informe operatorio señala que se ha realizado una
resección de todo el duodeno y del tercio proximal del yeyuno manteniendo íntegros el estómago y todo
el íleon, así como los dos tercios distales del yeyuno. En el seguimiento nutricional del paciente ¿Qué
NC

vitamina o mineral presentará con menor probabilidad una disminución de su absorción?


a. Cianocobalamina
b. Calcio
c. Hierro
d. Transcobalamina
BA

e. Transferrina

22. ¿Cuál de las siguientes alternativas detallan las venas que confluyen y forman la vena señalada?
a. mesentérica superior, gástrica izquierda y
gastroepiploica izquierda
b. mesentérica inferior, gástrica izquierda y renal
c. esplénica, mesentérica superior y mesentérica inferior
d. esplénica, pancreatoduodenal y omental izquierda
e. gástrica izquierda, esplénica y hepática común

23. ¿Cuál de las siguientes sustancias forma parte de la secreción biliar?


a. Tripsina
b. Lecitina
c. Elastasa
d. Quimotripsina
e. Pepsina
24. El tubo digestivo contiene diferentes tipos de epitelios y glándulas. La estructura señalada es unay
está localizada en el …………...
a. glándula de Brunner / intestino
grueso b.cripta de Lieberkuhn /
colon
c. cripta de Lieberkuhn / intestino delgado
d. glándula oxintica / estomago
e. célula parietal / estómago

Aunque las criptas de


Lieberkuhn están presentes en el

A
intestino delgado, la
microfotografía es de epitelio de

EK
colon.

25. ¿De qué musculo forma parte el ligamento inguinal?


a. Oblicuo externo del abdomen
b. Oblicuo interno del abdomen
OT
c. Transverso del abdomen
d. Psoas
e. Dorsal ancho

26. Señale cuál de las siguientes afirmaciones NO se relaciona a la siguiente glándula anexa del tubo
digestivo mostrada en la imagen:
NC

a. Es una glándula exocrina compuesta exclusivamente por acinos serosos


b. Su inervación está dada por el nervio auricular mayor (ramo posterior C2), que inerva la
vaina de la glándula así como la piel por encima de esta.
c. Esta glándula produce una secreción mucinosa acuosa, llamada mucoserosa, a través del
conducto de Wharton.
BA

d. Su inflamación puede ser causada por un virus de los Paramyxoviridae, que provocan una
enfermedad muy frecuentemente en niños y adolescentes
e. Es una glándula endocrina y probablemente sea de origen pancreático

Se valida la opción e debido a que no está


relacionada con la imagen.

27. ¿Cuál de las siguientes enzimas está localizada en el borde en cepillo y juega un rol en la digestión de
proteínas?
a. Alfa dextrinasa
b. Pepsina
c. Enterocinasa
d. Lactasa
e. Carboxipeptidasa A.

Se valida la opción c debido a que es correcta en relación a la pregunta.


28. Una de los siguientes sustancias, NO sirve como un buen agente emulsificante:
a. Colesterol
b. Ácidos grasos
c. Sales biliares
d. Lecitina
e. Proteínas de la dieta

Se valida la opción e debido a que es correcta en relación a la pregunta.

29. La sustancia que estimula el crecimiento de la mucosa gástrica es:


a. Secretina
b. Motilina
c. Péptido estimulante de la mucosa gástrica
d. Gastrina
e. Histamina

A
30. ¿Cuál de las siguientes alternativas es una función de la colecistokinina?
a. Relajación de la vesícula para la salida de

EK
bilis b.Secreción de ácidos biliares
c. Contracción del esfinter de Oddi
d. Secreción de enzimas pancreáticas
e. Contracción del duodeno

Se valida la opción b debido al efecto de la CCK sobre la vesicula biliar.


OT
31. Con respecto a la anatomía del tronco celiaco, señale lo correcto
a. El tronco celiaco se origina de la cara posterior de la aorta abdominal
b. Es una arteria delgada que tiene un calibre entre 2 y 3 mm
c. Una de sus ramas es la arteria gástrica derecha
d. La hepática común que es una de sus ramas, participa en la irrigación del estómago.
NC

32. Con respecto a la anatomía del duodeno, marque la respuesta correcta:


a. Tiene una distribución en forma de “C”, que rodea la cola del páncreas
b. La 3ra porción duodenal está contenida en la pinza vascular aortomesentérica
c. Entre la 1ra y 2da porción se forma un ángulo, conocido como el ángulo de Treitz
BA

d. La 4ta porción se dirige a la izquierda, hacia abajo y hacia atrás.


e. En la tercera porción desemboca el conducto colédoco.

33. El hígado está ampliamente tapizado por peritoneo, la estructura que conecta la cara diafragmática del
hígado precisamente con el diafragma es el ligamento:
a. teres
b. falcifor
me
c.triang
ular
d.hepático
común
e.coronario

Se validan la opción c y e debido a que forman parte de los ligamentos que fijan el hígado al diafragma.

34. En el íleon se absorbe aproximadamente el 95% dea través de la circulación enterohepática.


a. agua
b. colesterol
c. sales biliares
d. hidróxicobalamina
e. factor intrínseca
35. Laestimula el mecanismo paracrino de la secreción de ácido clorhídrico.
a. histamina
b. acetilcolina
c. gastrina
d. secretina
e. somatostatina

36. En la digestión de proteinas,es el principal estímulo para convertir el pepsinógeno en pepsina.


a. la gastrina
b. el pH ácido
c. la acetilcolina
d. la ptialina
e. la somatostatina

37. Con respecto a la somatostatina, marque lo correcto:


a. Es secretada por las células S del intestino

A
b. Induce a la producción de VIP
c. Interviene en la fase intestinal de la secreción gástrica

EK
d. Produce acetilcolina para estimular a la célula parietal
e. No interviene en la regulación de la secreción de ácido clorhídico

38. En pecten anal, es una estructura comprendida entre:


a. la línea pectínea y los senos anales
b. la línea blanca y la apertura anal
OT
c. el esfínter anal interno y el externo
d. la línea anocutánea y la línea pectínea
e. la línea blanca y columnas anales

39. ¿Cuál de las siguientes alternativas es una proenzima pancreática?


a. Tripsina
NC

b. Elastasa
c. Quimotripsinógeno
d. Amilasa
e. Procarboxipepitidasa C.
BA

40. En la segmentación hepática de Coinaud, el segmento hepático señalado con la flecha,


corresponde a : En la segmentación hepática de Coinaud, la flecha señala el segmentohepático.
a. IV
b. V
c. VI
d. VII
e. VIII
EXAMEN
PARCIAL SISTEMA
DIGESTIVO (ME154)
Ciclo 2019-02

Sección:Todas
Profesores:Alfaro Salazar, Herberth Romulo; Alva Muñoz, Jose Carlos; Mayor Zevallos, Otto Alberto;
Duración:30 minutos.
Indicaciones:
- Lea atentamente cada pregunta antes de responder.
- Se prohíbe el uso del celular y cualquier dispositivo electrónico.
- Está prohibido intercambiar materiales.
- Coloque su código de alumno en la tarjeta de respuestas. Si su código contiene una letra reemplácela por
un valor numérico siguiendo la siguiente equivalencia: A=9, B=8, C=7, D=6, E=5, F=4 y G=3.
- Traslade sus respuestas a la tarjeta, llenando los círculos de manera completa con lapicero negro o azul.

A
Está prohibido el llenado con lápiz, lapicero de otro color o con lapicero de tinta borrable.
- Sea cuidadoso en el llenado de la tarjeta de respuestas, pues solo esta tiene validez para la calificación.
- Al terminar su examen avise al docente a cargo, no se levante de su sitio; debe entregar la hoja de

EK
respuestas con la carátula del examen, este cuadernillo de preguntas se lo llevará cada estudiante.

1. La contracción del músculopermite la eliminación de gases (flatos) sin salida de material fecal;
es el mismo músculo cuya relajación, sobretodo en cuclillas, permite el paso del contenido fecal con
OT
menor esfuerzo durante la defecación.
a) Isquirect
al b)
Puborrec
tal
c) Esfínter anal externo
NC

d) Esfínter anal interno

2. Paciente mujer de 54 años se presenta con náuseas, vómitos, estreñimiento, y es diagnosticada de


abdomen agudo quirúrgico; en la cirugía encuentran un vólvulo de ciego. Esta anomalía puede
explicarse por::
BA

a) Falta de rotación intestinal


b) Falta de fusión del mesenterio
c) Defecto en la formación de la cloaca
d) Falta de formación del omento mayor

3. Paciente mujer de 23 años con faringitis aguda, toma para el dolor una tableta de paracetamol con un
poco de agua. Durante la deglución, se relaja su esfínter esofágico inferior y el fondo del estómago,
mientras el bolo está aún en el esófago. ¿Qué sustancia provocara con mayor probabilidad la relajación
del esfínter esofágico inferior y el fondo del estómago en esta mujer?
a) Óxido nítrico
b) Sustancia P
c) Histamina
d) Motilina

4. Luego de tres horas dando exámenes, un alumno de medicina comienza a sentir hambre. Esta
situación es probable que sea mediada por la que es sintetizada por el :
a) leptina / intestino
b) leptina / estómago
c) grelina / estómago
d) grelina / tejido adiposo

5. Varón de 72 años, con antecedente de diabetes mellitus tipo 2, que presenta enteropatía diabética
caracterizada por estreñimiento. Este problema puede estar asociado a:
a) deficiencia de óxido nítrico
b) aumento del reflejo gastrocólico
c) disminución de la secreción de colecistocinina (CCK)
d) aumento de la secreción del péptido intestinal vasoactivo (PIV)
6. Varón de 54 años con Diabetes Mellitus tipo 2, es diagnosticado de gastroparesia debido a que presenta
sensación de llenura precoz al comer, y reflujo gastroesofágico. Esta alteración en la relajación receptiva
y en el vaciamiento gástrico lo más probable es que se deba a una alteración en:
a) el nervio vago
b) el ganglio celíaco
c) plexo submucoso
d) nervio hipogástrico

7. Varón de 67 años con tos y disminución de peso asociado a tabaquismo pesado, presenta actualmente
disfagia progresiva a alimentos sólidos. Se considera la presencia de un carcinoma de bronquio
izquierdo y por esta razón le realizan una endoscopía esofágica para descartar la posibilidad de una
compresión esofágica por el tumor. Se espera revisar el esófago en la estrechez, que está a nivel de la
vértebra
a) Tercera estrechez -T6
b) Segunda estrechez - C6
c) Segunda estrechez - T4

A
d) Tercera estrechez -T10

EK
8. Varón de 34 años con dolor abdominal agudo en flanco derecho que se irradia a fosa ilíaca derecha, es
operado y se encuentra un divertículo intestinal inflamado, ubicado a 93 cm de la válvula ileocecal. El
origen de este divertículo es una falla en la obliteración de:
a) Conducto vitelino
b) Alantoides
c) Cloaca
OT
d) Conducto anorectal
e) Uraco

En un niño menor de dos años con divertículo intestinal, este divertículo tiene su origen en una
falla en la obliteración de:
NC

a)Conducto
anorectal b)
Conducto
vitelino
c) Alantoides
BA

d) Cloaca
e) Uraco

9. Mujer de 43 años sufre un grave accidente de tránsito y está hospitalizada en coma, es alimentada por vía
intravenosa durante varias semanas. Producto de este tipo de alimentación, se encuentra en la endoscopía
atrofia de la mucosa gastrointestinal. La causa más probable de esta atrofia son los bajos niveles séricos de
la hormona:
a) Colecistocinina
b) Secreti
na
c)Gast
rina
d) PIV

10. Una mujer de 30 años llega al consultorio porque se queja de dificultades para deglutir, la cual se agravan
cada vez más. Se realiza un estudio manométrico para examinar la generación de presión a lo largo del
esófago. Esta prueba revela que las contracciones como respuesta a la deglución están mal sincronizadas y
que la presión en el esfínter esofágico inferior permanece elevada. El diagnóstico más probable es
producido por niveles bajos de
a) acalasia / sustancia
P b) acalasia / óxido
nítrico
c) enfermedad por reflujo gastrointestinal / acetilcolina
d) enfermedad por reflujo gastrointestinal / óxido nítrico

11. Paciente de 2 años, llega a emergencia por haber ingerido una moneda con la que estaba jugando. El
lugar más probable donde puede haberse quedado suspendido este objeto es a nivel del estrechamiento
producido a nivel del:
a) músculo milohiodeo
b) músculo aritenoideo

A
EK
OT
NC
BA
c) músculo cricofaríngeo
d) constrictor superior de la faringe

12. En una apendicectomía, al realizar la incisión de McBurney en la fosa iliaca derecha, es necesario
cortar los siguientes músculos, de afuera hacia adentro:
a) Recto – Oblicuo externo – Transverso
b) Recto – Oblicuo externo – Oblicuo interno
c) Oblicuo externo – Oblicuo interno – Recto
d) Oblicuo externo – Oblicuo interno – Transverso

13. Un varón de 90 años que se encuentra postrado en cama, es referido del asilo para endoscopia por
dificultad para deglutir luego de tomar un medicamento para aliviar el dolor la noche anterior. La
endoscopía revela que la píldora se alojó en el esófago y causó una reacción inflamatoria. Lo más
probable es que esto haya sido por la producción de múltiples ondas:
a) secundarias
b) primarias

A
c) lentas
d) segmentarias

EK
14. Mujer de 23 años es diagnosticada de bulimia, al examen físico se observa ulceraciones en el segundo y
tercero dedo de la mano derecha. Esto se puede deber al uso continuo de estos dedos para inducir el
vómito, mediante la estimulación del par craneal:
a) V
b
OT
)
I
X
c) X
d) XI
NC

15. Varón de 52 años se presenta por diarrea persistente de seis semanas de duración. En la colonoscopia se
observa un pólipo a nivel del íleon distal. El patólogo informa que se trata de un tumor neuroendócrino,
probablemente originado por las células enterocromafines del intestino. La sustancia que más
probablemente esté produciendo este tumor es:
a) Serotonina
BA

b) Insulina
c) CCK
d) GIP

16. La fase oclusal de la masticación se realiza con la contracción de los músculos:


a) digástricos
b) masetero y temporal
c) orbicular y buccinador
d) pterigoideo lateral y digástrico

17. Al tomar su café en Starbucks, un estudiante de medicina sufre una quemadura de primer grado en
el tercio anterior de la superficie dorsal de la lengua. La información de dolor es transmitida por el
nervio:
a) cuerda del tímpano
b) glosofaríng
eo
c)lingual
d) facial

18. Paciente es evaluado por faringitis aguda en consultorio externo. El médico de familia le solicita que abra
la boca y saque la lengua. Para realizar la acción de sacar la lengua, es necesario que se contraiga el
músculo:
a) estiloglo
so b)
geniogl
oso
c) palatogloso
d) transverso de la lengua

A
EK
OT
NC
BA
19. Paciente con síndrome de Sjögren, presenta “boca seca”
(disminución de la producción de saliva) y caries dental, asociada
a la pérdida de la función de tampón de la saliva. Esta
desminerilización del diente puede comprometer a las
prolongaciones citoplasmáticas ubicadas en los tubos huecos de
la estructura señalada con la letra:
a) B
b) A
c) E
d) C

A
20. Mujer de 32 años acude a consulta por presentar disfagia de
progresión lenta, reflujo gastroesofágico y vómitos desde hace 3

EK
meses de evolución progresiva. Se le realiza un estudio radiológico
con contraste en el que se observa estrechamiento del esfínter
esofágico inferior (imagen). Según sus conocimientos, este paciente
se beneficiaría con el uso de:
a) agonista beta adrenérgico
b) agonista alfa
OT
adrenérgico c)análogo
de óxido nítrico
d) análogo de Sustancia P
NC
BA
EXAMEN
PARCIAL SISTEMA
DIGESTIVO (ME154)
Ciclo
202000

Sección:Todas
Profesores:Alva Muñoz, Jose Carlos
Duración:35 minutos.
Indicaciones:
- Lea atentamente cada pregunta antes de responder.
- Se prohíbe el uso del celular y cualquier dispositivo electrónico.
- Está prohibido intercambiar materiales.
- Coloque su código de alumno en la tarjeta de respuestas. Si su código contiene una letra reemplácela por
un valor numérico siguiendo la siguiente equivalencia: A=9, B=8, C=7, D=6, E=5, F=4 y G=3.

A
- Traslade sus respuestas a la tarjeta, llenando los círculos de manera completa con lapicero negro o azul.
Está prohibido el llenado con lápiz, lapicero de otro color o con lapicero de tinta borrable.
- Sea cuidadoso en el llenado de la tarjeta de respuestas, pues solo esta tiene validez para la calificación.

EK
- Al terminar su examen avise al docente a cargo, no se levante de su sitio; debe entregar la hoja de
respuestas con la carátula del examen, este cuadernillo de preguntas se lo llevará cada estudiante.

1. Paciente de sexo masculino de 82 años de edad ingresa a emergencia con dolor abdominal agudo y
diarreas. Se le realiza una arteriografía en la que se observa que la arteria aorta tiene un trombo
OT
ocluyendo el 95% del flujo, a nivel del nacimiento de la arteria mesentérica inferior. ¿Cuál de las siguientes
arterias podría contribuir a la irrigación colateral del colon descendente?
a) cólica media
b) sigmoidea
c) rectal superior
d) ileocólica
NC

2. Niño de 5 años presenta dolor esofágico y hematemesis (vómitos hemorrágicos) luego de tragarse una
espina de pescado. En la endoscopía se observa perforación del esófago distal a la cuarta estrechez
esofágica. ¿Las ramas de cuál de las siguientes arterias estarán lesionada con mayor probabilidad?
a) Gástrica izquierda
b) Bronquiales
BA

c) Frénica inferior
d) Tiroidea inferior

3. Al ingerir una cucharada de mantequilla es muy probable que se disminuya la sensación de hambre por
medio de la activación de la vía POMC/CART (POMC=proopiomelanocortina y CART=transcripción
regulada de cocaína y anfetamina), activada directamente por la hormona:
a) colecistoquinina (CCK)
b) insulina
c) grelina
d) secretina

4. Al comer unas papitas fritas con mayonesa, el vaciamiento gástrico disminuye por efecto directo de la
hormona: a)colecistoquinina (CCK)
b) bombesina
c) motilina
d) gastrina

5. Recién nacido de dos horas es diagnosticado de hernia umbilical de 1,5 cm de diámetro; el cirujano
pediatra solicita una tomografía abdominal en donde se evidencia que la hernia umbilical está ocupada
por una porción del tracto gastrointestinal. ¿Qué porción del tracto gastrointestinal estaría ocupando
esta hernia con mayor probabilidad?
a) Íleon
b) Colon sigmoides
c) Duodeno
d) Colon transverso
6. Recién nacido de 7 horas, de parto por cesárea debido a polihidramnios (aumento del volumen del
líquido amniótico), con regurgitación de la leche materna y artificial, y no ha presentado meconio. Se le
realiza una tomografía donde se evidencia aire en el estómago y una malformación del desarrollo
esofágico. Con respecto a esta malformación lo más probable es que se pueda tratar de una atresia
esofágica:
a) proximal con fístula traqueoesofágica distal
b) distal con fístula traqueoesofágica proximal
c) proximal y distal
d) sin fístula

7. Lactante de 6 meses de edad que es traído a consulta por presentar vómitos no biliosos a repetición y
retraso en el crecimiento. En la radiografía de abdomen simple se observa nivel hidroaéreo en estómago y
en primera porción de duodeno (doble burbuja). ¿Cuál de las siguientes alternativas puede explicar la
condición del lactante?
a) Páncreas anular
b) Atresia duodenal en la tercera porción

A
c) Atresia yeyunal
d) Hipertrofia del píloro

EK
8. En ausencia o deficiencia de la secreción de la hormona motilina, se
producirá: a)sobrecrecimiento bacteriano
b) diarrea
c) aumento del vaciamiento gástrico
d) hipertrofia del píloro
OT
9. La estimulación parasimpática aumenta la motilidad intestinal, mientras que la estimulación simpática
la disminuye. ¿Sobre cuál de las siguientes alternativas el sistema nervioso autónomo actúa para el
control de la motilidad intestinal?
a) Potencial de membrana en el plexo mientérico (de Auerbach)
b) Frecuencia de ondas lentas
NC

c) Secreción de secretina
d) Nivel de IP3 en el plexo submucoso (de Meissner)

10. En un recién nacido con protrusión de contenidos abdominales y cubiertas por amnios o peritoneo, es
cierto que: a)Se presenta por un defecto en el cierre de la pared
BA

b) Se acompaña de otras malformaciones congénitas


c) Se debe al no retorno de la hernia fisiológica
d) Se produce a través del ombligo

11. Lactante de 20 días con estreñimiento, distención abdominal progresiva, acompañada ocasionalmente de
vómitos biliosos. Como antecedente, el meconio lo eliminó por primera vez a las 72 horas de nacido. Su
mamá menciona que ayuda a la evacuación con ayuda de un termómetro rectal. Se sospecha de
megacolon agangliónico (Enfermedad de Hirschsprung). ¿Cuál de las siguientes alternativas explica el caso?
a) Se presenta contracciones tónicas en la región ano rectal
b) Se presenta dilatación de tracto gastrointestinal afectado
c) Las células ganglionares sólo han migrado al ano recto
d) La zona que más se afecta es inervada por fibras del nervio esplácnico menor

12. ¿Cuál de los siguientes reflejos disminuye el tránsito


gastrointestinal? a)Doloroso
b) Gastrocólico
c) De defecación
d) Colicoileal

13. Al ingerir una sustancia ácida como el vino (pH 3), se estimula la motilidad gástrica por acción de la
hormona: a)motilina
b) secretina
c) colecistoquinina (CCK)
d) bombesina
14. Paciente de 24 años acude a consulta externa por presentar una fístula oronasal (comunicación entre la
cavidad oral y la cavidad nasal). Está fístula está asociada al antecedente de haber sido operada de
paladar hendido a los dos años de edad, durante una campaña gratuita extranjera de corrección de
paladar fisurado. ¿Cuál de las arterias palatinas podría haberse lesionado durante esa cirugía?
a) Mayor
b) Menor
c) Ascendente
d) Rama palatina de la faríngea ascendente

15. Paciente de sexo masculino de 52 años con úlcera péptica gástrica de 14 años de evolución, con cuadro
de hemorragia digestiva alta hace 4 meses, sin cicatrización de la úlcera. Entre las opciones quirúrgicas se
considera realizarle un vaguectomía troncal (sección del nervio vago) a nivel del hiato esofágico. ¿Cuál de
las siguientes complicaciones podría esperarse producto de la pérdida de inervación parasimpática?
a) Menor inervación del colon ascendente
b) Se perderá el reflejo de defecación
c) Se perderá el reflejo de micción

A
d) Impotencia sexual

EK
16. Paciente de 23 años con bulimia es traída a la emergencia deshidratada, semiconsciente y con alcalosis
metabólica. Los vómitos autoinfligidos por esta paciente se producen por estimulación de receptores en
la base de la lengua que mandan información directamente al:
a) núcleo del tracto solitario
b) centro del vómito en el tallo encefálico
c) zona quimiorreceptora gatillo
OT
d) cerebelo

17. Niño de 3 años es traído a emergencia por madre quien manifiesta que hace 10 horas deglutió una pila
pequeña de reloj de bordes romos. El niño está asintomático. Usted la tranquiliza diciéndole es un cuerpo
extraño tan pequeño de seguro que va a seguir el tránsito intestinal como lo haría un bolo alimenticio, y
que lo más probable es que en ese momento se encuentre en:
NC

a) colon
b) estómago
c) yeyuno
d) recto
BA

18. Los movimientos en masa son un tipo de movimiento muy importante, una de las consecuencias
de estos movimientos es:
a) la distensión rectal
b) el peristaltismo del intestino delgado
c) la retropulsión gástrica
d) la contracción del esfínter anal interno

19. Durante la deglución, al momento que el bolo alimenticio pasa por el esfínter esofágico superior, se
espera que la presión intraesofágica:
a) disminuya en el cardias
b) disminuya en el tercio medio del esófago
c) aumente en la porción distal al bolo
d) aumente en el tercio medio del esófago

20. Paciente con enfermedad de Chagas que presenta disfagia a sólidos. ¿Cuál de las siguientes puede ser la
causa de esta complicación?
a) Disminución de células ganglionares en el esfínter esofágico inferior
b) Aumento en la liberación de óxido nítrico en el esfínter esofágico inferior
c) Disminución de las neuronas que liberan péptido intestinal vasoactivo
d) Aumento de la actividad de la motilina en el esófago distal
QUIZIZZ
1. Los vasos mesentéricos superiores se hallan a nivel de:
a) Cuello del páncreas

2. El nivel en el que se encuentra el píloro y el páncreas se puede determinar usando el


A) plano transpilórico

3. El dolor de estómago asociado a gastritis se suele ubicar en


a) epigastrio

4. La colecistoquinina inhibe el
a) vaciamiento gástrico

A
5. Paciente con disfasia ( dificultad para pasar alimentos) con to y disminución de peso. Con antecedente

EK
de tabaquismo pesado. La sospecha es que tenga una disminución del diámetro esófago a nivel de la
a) tercera estrechez

6. La digestión de las proteínas se inicia en:


OT
a) estómago

7. El estómago recibe información simpática proveniente del:


a) ganglio celíaco
NC

8. El reflujo gastroesofagico tiene múltiples etiologías, una de ellas tiene que ver con alteración a nivel de:
a) primera estrechez
b) segunda estrechez
BA

c) tercera estrechez
d) cuarta estrechez

9. La fístula retroperitoneal es causada por una falla en el desarrollo de:


a) tabique urorrectal

10. El nervio vago inerva el:


a) músculo estriado del esófago

11. El divertículo de Meckel es un rezago de:


a) conducto vitelino

12. La presencia de orina que sale por el ombligo de un recién nacido casa vez que llora, es posible que se
deba a un defecto en el desarrollo del
a) seno urogenital

13. La fístula retroperitoneal es causada por una falla en el desarrollo de


a) tabique urorrectal

14. Enfermedad asociada con un error en el desarrollo de las células de Cajal:


a) enfermedad de Hirschsprung

15. Aproximadamente en la semana 6 del desarrollo embrionario, el intestino medio gira 90 herniandose a
nivel del
a) cordón umbilical

16. El conducto biliar deriva del


a) endodermo

17. Paciente mujer con 54 años con nauseas y vómitos y abdomen agudo quirúrgico, se ingresa a sala de
operaciones donde se encuentra vólvulo de ciego, esto se debe a
a) Falta de fusión del mesenterio

A
18. Paciente con cirrosis hepática con hipertensión portal, en el que es posible encontrar que los vasos

EK
umbilicales están permeables dentro de
a) ligamento redondo

CUAL
OT
19. Es normal encontrar glándulas submucosas en

ES
NC

a) esófago medio
b) esófago proximal
BA

c) esófago distal
d) estómago

20. Cual de los siguientes órganos son intraperitoneales


a) estómago, vesícula biliar, íleon, hígado

Estudiante de medicina de la UPC de 21 años sufre de gastritis aguda ocasionada por comer en lugares
poco higiénicos. Suele consumir caramelos ( chupar ) mientras está en clase hasta la tarde. Toma gaseosas
regularmente (carbohidratos 46%, sodio 53%). También toma regular cantidad de leche (grasa 35%, lactosa
35%, proteínas 30%), pues le calma un poco el dolor el ardor que siente por la gastritis. Incluso, cuando
puede, se toma dos vasos de agua fría para calmar las molestias. Ha decidido ir al médico para tratarse pues
ya no soporta el dolor, el cual está seguro que los síntomas se deben a una elevada producción de ácido
clorhídrico en el estómago, y por ello le ha recetado Ranitidina (antihistamínico), con lo que siente mejoría.

● Para reducir la secreción de HCl en esta paciente se podría usar sustancias similares a:

- Péptido insulinotrópico dependiente de la glucosa (GIP)

● Si se usara atropina en esta paciente, se esperaría que disminuya la liberación de:


- Enzimas pancreáticas
● El consumir caramelos eleva los niveles en sangre de una hormona cuya función es la
estimulación de las células:
- Beta del páncreas

● En este paciente con gastritis aguda debida a una alta producción de ácido clorhídrico, si
se le hiciera un examen de sangre, se encontraría elevados los niveles de:
- Colecistoquinina

● El consumo de una pequeña cantidad de gaseosa aumentará directamente la


concentración sérica de cuál de las siguientes hormonas:

- Péptido 1 similar al glucagón (GLP-1)

● El consumo rápido de 500 mL de gaseosa aumentará directamente la concentración sérica


de cuál de las siguientes hormonas:

A
- Gastrina

EK
● Estimulan la secreción ácida gástrica

- Proteínas
OT
● Con respecto a las ondas lentas, marque la afirmación correcta:}

- Son contracciones rítmicas espontáneas

● El uso de Ranitidina bloquea el receptor H2 de la histamina en las células


NC

parietales. La histamina llega a estas células por:

- Difusión

● El consumir caramelos indirectamente activa la vía:


BA

- POMC/CART

● ¿Cuál de los siguientes péptidos inhibe el vaciamiento gástrico?

- Colecistoquinina

● Para poder morder una manzana, es necesario usar el siguiente músculo:

- Milohiodeo

● El crecimiento de un adenocarcinoma de cuello de páncreas puede comprometer la pared


gástrica por continuidad. ¿Qué parte del estómago estaría comprometido con mayor
probabilidad?

- Pared posterior del antro

● El nacimiento de la arteria mesentérica superior se puede encontrar en cuál de los


cuadrantes abdominales:

- Epigastrio

● En cuanto a la colecistoquinina, marque la respuesta correcta:

- Potencia la acción de buffer con bicarbonato

● El aumento en la actividad motora de la pared gástrica genera un aumento en los


niveles locales de qué sustancia en la microvasculatura:

- Adenosina

Niño de sexo masculino de 2 años de edad, sufre de estreñimiento desde el nacimiento (1


deposición cada 3-4 días). Madre menciona que le estimula la defecación con un termómetro
rectal, y continuo uso de enemas y laxantes. Desde hace 6 meses comienza con vómitos
postprandiales. Los síntomas aumentan en frecuencia y magnitud y están en relación con los
episodios de estreñimiento. No refiere fiebre, tos, diarrea ni lesiones cutáneas. Al examen físico
presenta regular estado general, luce deshidratado. Abdomen distendido, blando, depresible e
indoloro. No se palpan masas abdominales. Se permeabiliza el canal anal con termómetro rectal,
encontrando cierta resistencia. Salida de material fecal mal oliente en regular cantidad. Exámenes
de laboratorio: hemograma normal. Signos inflamatorios de fase aguda negativos. Alcalosis
metabólica leve en sangre venosa. Radiografía con enema baritado muestra recto y colon
sigmoides dilatados (megacolon). Biopsia profunda: ausencia de células ganglionares en la
muestra enviada. Se realiza cirugía correctiva.

A
● La percepción de la pirosis (sensación de dolor o quemazón en el esófago) asociado al
reflujo gastroesofágico, puede aparecer o exacerbarse debido a:

EK
- Ejercicio

● Considerando que este paciente está sometido a estrés por el agravamiento de su


enfermedad, es posible afirmar que sus ondas lentas están:
OT
- Hiperpolarizadas

● Debido al acúmulo de material fecal en todo el marco colónico, y a la irritación química


asociada, el peristaltismo del íleon distal se debe encontrar:

- Inhibido
NC

● En cuanto a los reflejos gastrocólico y gastroduodenal en este paciente, indique lo


correcto:

- Se pueden considerar reflejos vago-vagales


BA

● Con respecto a la defecación señale el enunciado correcto:

- Es estimulado por un llenado de la cuarta parte del volumen rectal

● El contenido fecal se detiene en la zona inmediatamente proximal a la zona donde hay una
menor presencia de:

- Péptido intestinal vasoactivo

● La presencia de atresias y estenosis duodenales se deben básicamente a una:

- Falta de recanalización

● El ligamento de Treitz característicamente:

- Suspende el ángulo de Treitz

● Al deglutir un bolo alimenticio, es lógico suponer que al pasar por el esófago haya un
mayor consumo de oxígeno en la pared del tercio:

- Proximal

● Estudiante de medicina de 20 años, se ha amanecido estudiando para su examen de


Sistema Digestivo. No ha probado alimento desde la cena, por lo que se puede afirmar que
la motilidad de esta persona está siendo regulada por:

- Motilina

● La hernia fisiológica se produce dentro de:


- Cordón umbilical

● El crecimiento de un adenocarcinoma de páncreas compromete la pared gástrica por


contigüidad. ¿Qué parte del estómago se esperaría esté comprometido?

- Pared posterior del antro

● Al comer unas papitas fritas con mayonesa, el vaciamiento gástrico disminuye por efecto
directo de la hormona:

- colecistoquinina (CCK)

● Paciente que come entera una pizza familiar de chorizo y queso. Es posible esperar que
debido a la cantidad de alimento ingerida, las ondas lentas hayan:

A
- Sufrido ninguna alteración en su frecuencia

● En este caso se puede afirmar con seguridad que se presenta:

EK
- contracciones tónicas en la región ano rectal

● La forma más común de atresia esofágica contiene:

- Estenosis proximal del esófago más fístula traqueoesofágica distal


OT
● Con respecto a la saliva, marque la respuesta correcta:

- el sistema simpático estimula su secreción

● Respecto a las enfermedades del esófago, marque lo correcto:


NC

- el diagnóstico diferencial de la acalasia es la enfermedad de Chagas esofágica

● En relación a la fisilogía gástrica, marque lo correcto:

- la cimetidina actúa en la región basolateral de la célula parietal


BA

● La célula mucosa del cuello gástrico produce:

- Moco

● La saliva puede tener una variedad de electrolitos en su composición. Entre ellos el cloro,
respecto al cual se puede afirmar:

- Su concentración no llega a ser tan alta como en el plasma

● Los músculos de la masticación que producen la retropulsión de la mandíbula son:

- temporales

● Respecto a las glándulas salivales, marque lo incorrecto:

- la glándula sublingual tiene forma de garfio

● Respecto a la anatomía del estómago, marque lo correcto:

- la arteria gástrica derecha nace de la arteria hepática común

● En cuanto a la saliva, marque lo correcto:

- La amilasa cumple función digestiva

● Durante el ataque con gas sarín (bloqueador de la acetilcolinesterasa) en el metro de


Tokio, en 1995, el personal de salud notó que los pacientes afectados presentaban:

- Hipersalivación

Paciente de 54 años con antecedentes de alcoholismo, gastritis crónica, tabaquismo


pesado, obesidad, cálculos biliares y cirrosis, es llevado a la emergencia por dolor
abdominal en epigastrio irradiado a la espalda y trastorno del sensorio.
Al examen físico: presión arterial 85/50 mmHg, frecuencia cardíaca 100 latidos/min,
frecuencia respiratoria 18 x minuto, temperatura axilar 36°C.
Conjuntivas pálidas, escleras ictéricas nevus arácnidos en tronco, distensión abdominal
marcada, cabeza de medusa, matidez desplazable en ambos flancos e hipogastrio, dolor
a la palpación de abdomen.
Tiempo de protrombina: 24 seg (testigo: 13 seg); TPT: 38 seg, glicemia: 165 mg/dL,

A
uremia: 20 mg/dL, ASAT: 76 UI/L, ALAT: 22 UI/L, albumina: 2,5 g/dL, bilirrubina total: 2,6
mg/dL, bilirrubina directa: 1,4 mg/dL, amilasa sérica 4000 U/L.

EK
● Un efecto secundario en el estómago por la acción de la secretina es:

- Menor actividad de la pepsina

● Considerando que el paciente sufre de gastritis, se puede decir que la secreción de ácido
por la mucosa gástrica:
OT
- Involucra transporte activo de Hidrogeniones

● En cuanto a la gastritis de este paciente, se encontró que era producida por la bacteria
Helicobacter pylori. Esta bacteria sobrevive en el medio ácido del estómago gracias a:
NC

- Ureasa

● La bilirrubina directa aumentada en cirrosis hepática se excreta en la orina debido a:


BA

- Ser hidrosoluble

● Paciente de 42 años con adenocarcinoma ductular. La TC ha demostrado claramente que el


tumor está en el cuello del páncreas y que hay un gran vaso ocluido. ¿Cuál de los siguientes
vasos estaría más probablemente obstruido?

- Vena porta.

● En relación a la histología hepática, marque lo correcto:


- La zona 3 se encuentra más cerca a la vena central lobulillar

Mujer de 83 años acude a emergencia por dolor abdominal desde hace 4 días,
localizado en epigastrio, irradiado a ambos hipocondrios, nauseas, vómitos y
distensión abdominal; tiene antecedente de cardiopatía hipertensiva, diabetes
mellitus tipo II y fibrilación auricular. Refiere deposiciones diarreicas muco
sanguinolentas hace 1 día. Los exámenes iniciales muestran PA: 110/60
mmHg, FC: 110/mn, leucocitos: 17800, neutrófilos de 93%. TAC abdómino
pélvica se observa oclusión completa de arteria mesentérica superior por
trombo asociado a placa ateromatosa.
● En esta paciente, ¿cuál de las siguientes sustancias no tendrá una considerable
disminución en su absorción? (marque la mejor respuesta):

- Calcio

● El mecanismo de la diarrea muco sanguinolenta que presenta la paciente, puede mejor


definido como de tipo (marque la mejor respuesta):}

- Exudativa

● Considerando que se ha comprometido el íleon distal, entre otras áreas, la atrofia o


descamación del epitelio de superficie explicaría cuál de los signos o síntomas de la
paciente (marque la mejor respuesta):

A
- Diarreas mucosanguinolentas

● Producto de esta isquemia, la expresión de cuál de las siguientes enzimas se vería

EK
notablemente disminuida. Marque la mejor respuesta:

- Enteroquinasa

● Durante la cirugía, el cirujano observó que además la paciente tenía divertículos en el


sigma. Se sabe que estos divertículos:
OT
- Se pueden asociar a estreñimiento crónico

Autoevaluación 1:
NC

Un paciente varón de 68 años consulta por dolor y abultamiento en la región inguinal derecha, que
aparece tras la realización de un esfuerzo físico. En la exploración en bipedestación presencia de una
tumoración blanda, depresible, que aumenta con la tos. El presente caso describe una hernia:
Inguinal directa.
Umbilical.
BA

Femoral.
Inguinal indirecta.

La Grelina es sintetizada por………….. y activa las neuronas relacionadas con


……………………….en el núcleo arqueado del hipotálamo}

El páncreas/AGRP-NPY
El intestino delgado/POMC-CART
El estómago/AGRP-CART
El estómago/AGRP-NPY

Cuál de los siguientes órganos son intraperitoneales:

Estómago, Vesicula biliar, Ileón, Higado


Páncreas, Colon descendente, Hígado, Vesícula biliar
Recto, Hígado, Colon transverso, Yeyuno
Estómago, Yeyuno, Duodeno, Páncreas

La distención gástrica por los alimentos produce incremento de secreción de HCl mediante el siguiente
mecanismo:

Producción de Gastrina que desencadena su cascada de señalización en la célula parietal vía proteína Gi
Producción de Acetilcolina que desencadena su cascada de señalización en la célula parietal vía proteína
Gs
Producción de Gastrina que desencadena su cascada de señalización en la célula principal vía proteína
Gs
Producción de Gastrina que desencadena su cascada de señalización en la célula parietal vía proteína Gq

La inervación de la piel del abdomen debajo del ombligo hasta la región púbica está dada por:

T7, T9
T11, T12, L1
T7, T9, L2
L2, L3, S1

Paciente de 38 años que tras riña durante partido de futbol sufre un traumatismo con arma blanca en

A
cuadrante inferior izquierdo del abdomen. En la tomografía de urgencias se evidencia gran
hematoma de pared y ausencia de neumoperitoneo. Desde la piel hacia al peritoneo, en ¿qué orden

EK
se atravesó la pared abdominal?

Oblicuo externo, oblicuo interno, musculo transverso, TCSC


TCSC, oblicuo externo, oblicuo interno, musculo transverso
TCSC, oblicuo interno, oblicuo externo, musculo transverso
OT
TCSC, musculo transverso, oblicuo externo, oblicuo interno
Oblicuo externo, musculo transverso, oblicuo interno, TCSC

Alrededor de 90% de los pacientes afectados por el síndrome de Zollinger-Ellison desarrollan ulcera
NC

péptica. La causa es:

Gastritis atrófica
Hipersecreción ectópica de gastrina
Reflujo gastroesofágico
BA

Gastropatía hipertrófica

Un hombre de 22 años sufre traumatismo en el flanco izquierdo superior del abdomen al tratar de
defenderse del robo de su vehículo. Un amigo lo traslada a un centro hospitalario. El paciente
presenta signos de hipovolemia con taquicardia e hipotensión. Se queja de dolor en el sitio de lesión
que se irradia al hombro izquierdo. Elñ mórgano probablemente lesionado es

Hígado
Bazo
Estómago
Colon

Aproximadamente en la semana 6 del desarrollo embrionario, el intestino medio gira 90° herniándose
a nivel del:

Borde inferior del bazo


Caudal al borde hepático derecho
Cordón umbilical
Lado derecho del abdomen
La triada portal (arteria hepatica, vena portal y conducto biliar común) está contenida en el:

Ligamento Gstroesplenico
Ligamento gastrohepático
Ligamento hepatoduodenal
Ligamento Falciforme

En el siguiente gráfico marque la relación correcta:

A
EK
1
4
OT
NC
BA

3 2

Omento mayor – 4
Plexo de Meissner – 1
Muscular de la mucosa – 3
Glándula de la mucosa

¿Cuál de las siguientes sustancias es liberada por neuronas en el tracto GI,


Participa en la regulación hidro electrolíticay produce relajación del musculo liso?

Gastrina
Secretina
CCK
VIP
La colecistoquinina inhibe:

El Vaciamiento gástrico
La Secrecion pancreatica de HCO3- La Contraccion de la
vesicular biliar
La Relajación del esfínter de oddi

¿La secreción de cuál de las siguientes es inhibida por un pH bajo?

Secretina
Gastrina
CCK
VIP

A
El conducto biliar deriva del:

EK
Ectodermo.
Endodermo.
Mesodermo.
Mesotelio.
OT
Respecto a la imagen mostrada la estructura señalada con el número 1 corresponde a:

Glándula parótida
NC

Glándula sublingual
Glándula submaxilar
Glándula lingual
BA

3
Los vasos mesentéricos superiores se hallan a nivel de:

El fondo gástrico
El cuello del páncreas
La cabeza del páncreas
El hilio hepático

La imagen mostrada representa un defecto de ……………… y recibe el nombre de: ……………………….

A
EK
OT
NC
BA

La pared abdominal/onfalocele
La pared abdominal/divertículo de Meckel
Malrotación intestinal/gastrosquisis
La pared abdominal/gastrosquisis

Actividad extra sem 1:

1. Existen 3 perforaciones de la membrana pleuro-parietal, denominados hiatos, causados por la vena


cava inferior, el esófago y la aorta. Mencione a que nivel torácico (vertebras torácicas) se encuentran
estos hiatos.
• VCI: T8
• ESOFAGO: T10
• AORTA: T12

2. Mencione a nivel de que dermatoma se encuentra el ombligo.


• T10
3. Mencione en orden, de adentro hacia afuera, los componentes de la pared abdominal anterior.
a) Peritoneo
b) Fascia extraperitoneal
c) Fascia transversalis
d) Músculos con sus aponeurosis
e) Fascia de Scarpa
f) Fascia de Camper
g) Grasa
h) Piel

4. ¿Cuál es el plexo nervioso que se encuentra entre la capa muscular circular interna y longitudinal
externa?
• Plexo mientérico de Auerbach
5. ¿Qué célula es considerada el marcapaso intestinal, ya que se encarga de las contracciones fácicas y
tonicas del sistema gastrointestinal?
• Célula intersticial de Cajal

A
EXAMEN PARCIAL 2019-01
1. ¿Cuál de las glándulas salivales es responsable del mayor porcentaje del volumen de la saliva en

EK
condiciones basales?
a. Parótida
b. Subpalatinas
c. Sublingulaes
d. Submaxilares
OT
2. La lengua está recubierta por epitelio:
a. pseudoestratificado columnar no queratinizado
b. plano estratificado no queratinizado
NC

c. pseudoestratificado columnar ciliado


d. plano estratificado queratinizado

3. El esfínter anal interno tiene musculatura …………… y tiene control …………………. a. lisa / voluntario
b. lisa / involuntario
BA

c. esquelética / simpático
d. esquelética / parasimpático

4. La arteria aorta proporciona la irrigación al tubo digestivo ¿cuál de las siguientes arterias
proporciona la irrigación al ángulo cólico derecho?
a. Mesentérica superior
b. Mesentérica inferior
c. Frénica inferior
d. Tronco celiaco

5. Paciente de 26 años que le cuenta en su historia clínica que cada vez que almuerza, a los 20 minutos
tiene deseo de defecar. Le comenta que su hijo de 1 mes le pasa lo mismo pero más intenso. Esto se
explica por el reflejo …………………, el cual está ……………..en el paciente.
a. colicoileal / normal
b. colicoileal / alterado
c. gastrocolico / normal
d. gastrocolico / alterado

6. La región del estómago que se comunica con el duodeno se denomina:


a. pilórica
b. cardias
c. cuerpo
d. fórnix

7. Acude a consulta un paciente que fue diagnosticado de ulcera péptica 3 días antes. Luego de
múltiples pruebas diagnósticas, se concluye que el paciente presenta un tumor secretor de gastrina
¿Cuál de las siguientes situaciones estará incrementada?
a. Distención gástrica
b. Inhibición del vaciado gástrico
c. Secreción de ácido clorhídrico (HCl)
d. Inhibición de la secreción de pepsinógeno

8. En el sistema digestivo, el control del apetito esta dado por un complejo sistema de sustancias y

A
órganos integradores los cuales regulan la ingesta de alimentos. La ……………… es una sustancia
oroxígena y es sintetizada por el ……………………..
a. leptina / intestino

EK
b. grelina / intestino
c. leptina / estómago
d. grelina / estómago
OT
9. Sobre el control autónomo del sistema digestivo, marque la alternativa correcta:
a. La inervación dada por el sistema simpático es de tipo preganglionar.
b. El sistema parasimpático usa como neurotransmisores a la acetilcolina y la noradrenalina.
c. El nervio vago (par craneal X) le da inervación simpática a la mayoría del sistema digestivo.
d. En el sistema simpático, los nervios responsables hacen una primera sinapsis en ganglios
NC

próximos al órgano a inervar.


e. En la inervación de tipo parasimpático, solo interviene el plexo submucoso, sin embargo, en
la de tipo simpático intervienen tanto el submucoso como el mientérico.
BA

10. Con respecto a la actividad eléctrica del sistema digestivo, marque la alternativa correcta
a. Corresponden a potenciales de acción que están presentes de forma continua y le dan la
capacidad de perístasis autónoma al sistema digestivo.
b. La frecuencia de las ondas lentas no se ve influenciada por la actividad neural ni las
hormas gastrointestinales.
c. En el estómago las ondas lentas se dan en una frecuencia de 6 por minuto.
d. Las ondas lentas son cambios lentos y ondulantes del potencial en reposo.
e. La frecuencia de las ondas lentas va de 6 a 12 ondas por minuto.

11. Ante una lesión del IX par craneal, el músculo……………….. se altera en su función.
a. palatogloso
b. estilofaríngeo
c. palatofaríngeo
d. constrictor superior

12. Un varón de 50 años es sometido a extirpación de duodeno y parte proximal de yeyuno. La pérdida
de estímulo hormonal en el páncreas para la secreción enzimática se explica por la pérdida de las
células …………………… a. Parietales, productoras de factor intrínseco
b. “K” productoras de factor intrínseco
c. “M” productoras de CCK
d. “I” productoras de CCK
13. Respecto al mecanismo de la defecación ¿Cuál de las siguientes afirmaciones es correcta?
a. Se produce contracción refleja del esfínter anal interno
b. Se produce contracción o relajación del esfínter anal externo por señales de la corteza
cerebral
c. La presencia de materia fecal en el recto estimula la contracción del sigmoides por los
nervios pélvicos simpáticos
d. En la posición de “cuclillas” el músculo puborectal se halla contraído favoreciendo la
evacuación de la materia fecal

14. Un niño de tres años llega a emergencia con disfagia (dificultad para tragar), salivación y llanto. Se
sospecha de ingesta de cuerpo extraño: moneda en el esófago; al ser evaluado se constata en una
radiografía presencia de cuerpo extraño a nivel de C6 y C7 (6° y 7° vértebra cervical). El cuerpo
extraño estará suspendido a nivel del estrechamiento producido por el ………..

A
a. cayado aórtico
b. hiato esofágico
c. músculo cricofaríngeo

EK
d. bronquio principal izquierdo

15. En el caso de un paciente con un tumor productor de gastrina, la presencia de úlceras duodenales y
erosión de la mucosa gástrica se debe principalmente a…….
OT
a. la acción paracrina de la gastrina sobre la célula parietal
b. el exceso de HCl por estímulo de receptores CCK-B en la célula parietal
c. la sobre expresión de los receptores “G” para gastrina en la célula parietal
d. el exceso de HCl por estímulo directo de receptores “H” en la célula parietal
NC

16. La onda peristáltica secundaria del esófago se caracteriza por ser originada ………
a. por el plexo de submucoso del esófago
b. por el plexo mientérico del esófago
c. por el reflejo de la deglución
d. durante la masticación
BA

17. Marque lo correcto sobre las ondas lentas en el tubo digestivo


a. No son despolarizaciones
b. Son potenciales de acción subumbrales
c. Se constituyen de despolarizaciones y repolarizaciones
d. Son rítmicas y generadas por el sistema nerviosos autónomo

18. Recién nacido que presenta protrusión de contenidos abdominales los cuales no están cubiertos por
peritoneo y salen de la cavidad abdominal a través de un defecto de la pared. ¿Cómo se denomina a
la afección que presenta este paciente?
a. Onfalocele
b. Atresia biliar
c. Gastrosquisis
d. Divertículo de Meckel

19. Experimentalmente se utiliza atropina (anticolinérgico) para inhibir la secreción de gastrina, sin
embargo, la secreción de esta hormona se sigue dando ante estímulos vagales. Esta situación se
explica porque la atropina:
a. no bloquea la acción del péptido GRP
b. solo inhibe la acción del péptido GRP en la célula G
c. inhibe la acción de acetilcolina e histamina en la célula G
d. bloquea parcialmente la bomba de protones en la célula G
20. Niña de 4 días es llevada a la emergencia pediátrica por presentar llanto constante, la madre refiere
coloración azulada de labios al momento de lactar, acompañado de tos persistente y dificultad
respiratoria así como distención abdominal. Se le coloca sonda nasogástrica para alimentación
notando que retorna a la cavidad oral en todos los intentos. ¿Cuál es la anomalía del desarrollo en
este caso?
a. Solo fístula traqueo esofágica
b. Fístula traqueo esofágica proximal y distal
c. Atresia esofágica proximal con fístula traqueo esofágica distal
d. Atresia esofágica distal con fístula traqueo esofágica proximal

21. Paciente varón de 36 años es traído a la emergencia luego de sufrir un accidente de tránsito,
presenta traumatismos múltiples en cabeza y tronco. Al examen físico se evidencia hematoma en

A
hemicara izquierda, ligera protrusión y caída del lado izquierdo del maxilar inferior, por lo que se
realiza una tomografía donde se halla una fractura de la apófisis coronoides del maxilar inferior.
¿Qué músculo está relacionada directamente con esta situación?

EK
a. Masetero
b. Temporal
c. Buccinador
d. Pterigoideo medial
OT
22. Un paciente refiere no percibir algunos sabores. Al examen físico constata alteración del sabor dulce
y umami. ¿Cuál de los siguientes nervios estará alterada su función?
a. Cuerda del tímpano (VII par)
b. Lingual (rama del V par)
NC

c. Glosofaríngeo (IX par)


d. Hipogloso (XII par)

23. A los pocos días de nacido, regresa a neonatología un niño con problemas de motilidad del colon; los
estudios determinan ausencia congénita de células ganglionares. Según el gráfico ¿cuál es la capa en
BA

la que se determina la ausencia de dichas células?


a. Mucosa - 2
b. Muscular propia - 2
c. Muscular propia- 3
d. Muscular de la mucosa -3

1 2

24. Con respecto al control autonómico en el tracto gastrointestinal y en relación a su fisiología. ¿Cuál
es la función del sistema nervioso parasimpático en el tracto gastrointestinal?
a. Inhiben la contracción muscular y estimulan la secreción de sustancias a nivel de la
submucosa
b. Estimulan la contracción muscular y estimulan la secreción de sustancias a nivel de la
mucosa
c. Inhiben la contracción muscular e inhiben la secreción de sustancias a nivel de la submucosa
d. Estimulan la contracción muscular e inhiben la secreción de sustancias a nivel de la mucosa

25. Un estudiante que está preocupado por su examen parcial, no ha desayunado ni almorzado; cuando
al fin ingiere alimentos, esto le provoca el aumento de los movimientos musculares del tracto
gastrointestinal y la sensación de defecar. ¿Qué reflejo se ha activado?
a. Entero-gástrico
b. Gastro-cólico
c. Cólico-ileal
d. Ileo-ileal

A
26. ¿De qué par craneal es rama el nervio palatino mayor?
a. Vago

EK
b. Hipogloso
c. Trigémino
d. Palatogloso

27. ¿En cuál de las fases de la deglución la epiglotis separa la vía respiratoria de la digestiva? a. oral
OT
b. laríngea
c. faríngea
d. esofágica

28. Los péptidos intestinales se pueden clasificar como sustancias endocrinas, neurocrinas y paracrinas,
NC

dentro de las paracrinas se encuentran la somastotatina e histamina. Marque la respuesta correcta


a. La somastotatina es sintetizada por las células B de la mucosa gástrica
b. La histamina actúa estimulando su receptor tipo H1 en la mucosa gástrica
c. La histamina es sintetizada por células de tipo paracrino de las glándulas gástricas
BA

d. La somatostatina presenta dentro de sus funciones la estimulación de la secreción de H +

29. En relación a los órganos intraabdominales y sus estructuras de fijación, elija la alternativa correcta
FFF
a. El mesenterio permite la suspensión e irrigación de los órganos retroperitoneales
b. Tanto el hígado como la vesícula biliar se encuentran ubicados a nivel del flanco derecho
c. El colon, el duodeno y el resto de intestino delgado son órganos considerados netamente
como peritoneales
d. Los ligamentos que encontramos dentro de la cavidad abdominal son el esplenorenal y el
gastrofrénico
e. Los omentos van desde el estómago y la segunda porción del duodeno a otras estructuras
intraabdominales y existen dos: el omento mayor y el omento menor

30. Paciente varón de 27 años es llevado por bomberos a emergencia luego de ser asaltado y, tras
resistirse, es cortado con el pico de una botella a nivel abdominal. Al examen físico usted observa
que a través de la herida se puede observar la protrusión de asas intestinales. En relación con las
capas de la pared abdominal, marque la alternativa correcta.
a. La fascia de Scarpa está constituida principalmente por tejido adiposo
b. La pared abdominal está formada por piel, huesos, músculos, fascias y peritoneo parietal
c. La fascia de Camper es una estructura fibrosa que carece de grasa y su grosor es constante
en toda la pared abdominal
d. El músculo oblicuo externo discurre en dirección súpero-interna y se inserta en el borde
inferior de las ultimas 3 a 4 costillas
e. El músculo recto del abdomen tiene como funciones comprimir el contenido del abdomen,
tensar la pared del abdomen y flexionar la columna

Final 2019-01

1. La explicación fisiológica de presentar somnolencia de 30 minutos a 1 hora después de ingerir


alimentos, se explica por:
a. Aumento del cloro intraluminal
b. Aumento del bicarbonato intraluminal
c. Disminución de ácido carbónico en la célula parietal
d. Disminución de la actividad de la anhidrasa carbónica

A
e. Aumento de la alcalinidad sanguínea

EK
2. Con respecto a la irrigación arterial del colon, a que arteria corresponde la
señalada con la flecha
a. Cólica derecha
b. Cólica media
c. Cólica izquierda
OT
d. Ileobisecoapendículocólica
e. Arco de Riolano

3. Si un paciente presentara dentro del punto de vista fisiológico, una disminución de enterocinasa,
NC

entonces esto originaría una disminución de la actividad de:


a. la pepsina
b. la lipasa
c. la quimotripsina
d. el peptido insulinotropo dependiente de glucosa
BA

e. la amilasa

Se valida la opción b debido a su relación con la colipasa.

Se valida la opción a debido a la ubicación del número 1 en


donde se unen el
ligamento falciforme y ligamento
coronario.
4. Con respecto a la anatomía del hígado, señale a que estructura
pertenece la marcada por el número 1.
a. Ligamento falciforme
b. Línea de Cantlie
c. Ligamento triangular
d. Ligamento coronario
e. Ligamento teres

5. Se presenta un paciente, el cual presenta un antecedente de tuberculosis intestinal, por lo cual, se


le resecó 80 cm de íleon distal. Desde el punto de vista fisiológico, el paciente puede presentar una
de las siguientes alteraciones:
a. Disminución de la secreción de Vitamina B12
b. Aumento indiscriminado de absorción de ácido fólico
c. Disminución de la absorción de hierro
d. Aumento de la secreción de bicarbonato
e. Disminución de la absorción de ácido glicocólico

6. Un paciente es sometido experimentalmente a un fármaco que modifica el flujo salival,


obteniéndose un volumen de saliva de 288 ml en 6 horas. En este caso las concentraciones de
electrolitos y bicarbonato en la saliva obtenida varían de la siguiente manera:
a. ↑ Na+, ↓ K+, ↑ Cl-, ↑ HCO3-
b. ↓ Na+, ↓ Cl-, ↑ K+, ↓ HCO3-
c. ↑ Na+, ↑ Cl-, ↓ K+, ↓ HCO3-
d. ↑ Na+, ↑ Cl-, ↑ K+, ↑ HCO3-

A
e. ↓ Na+, ↓ Cl-, ↓ K+, ↓ HCO3-

EK
Se valida la opción a debido a que se puede considerar como un aumento del flujo de saliva.

7. La siguiente imagen histológica corresponde a la


glándula
…………… y la estructura señalada produce ………
OT
a. salival sublingual / mucopolisacáridos
b. oxíntica / pepsinógeno
c. salival submaxilar / ptialina
d. salival parótida / amilasas
e. antrales / gastrina
NC
BA

8. Paciente varón de 65 años con antecedente de hipercolesterolemia, hipertensión arterial,


fibrilación auricular y dos infartos al miocardio previos, aqueja de dolor abdominal intenso de inicio
súbito, distensión abdominal, se decide cirugía con resección de 1,5 metros de intestino delgado
terminal y colon ascendente. Como consecuencia de la resección el paciente tendrá deficiencia de:
a. Vitamina C
b. Tiamina
c. Vitamina A
d. Vitamina B1
e. Vitamina B6

Se valida esta opción debido a que su absorción está relacionada al íleon.

9. Uno de los siguientes elementos debería hallarse con más probabilidad en el esófago de un
paciente que sufre de reflujo gastro esofágico…
a. Pepsina
b. Tripsina
c. Quimiotripsina
d. Carboxipeptidasa
e. Ácidos biliares
10. Un paciente de 40 años cursa con anemia de 8g/dl, aqueja además de astenia y sensación de
hormigueo bilateral en los miembros inferiores, al examen se halla alteración de la sensibilidad a la
vibración y camina con ampliación de la base de sustentación. Uno de los siguientes
procedimientos sería de ayuda para el diagnóstico de este paciente:
a. Tomografía cerebral
b. Biopsia de la mucosa gástrica
c. Biopsia de hígado
d. Examen de sangre oculta en heces
e. Biopsia de Ileon proximal

11. Paciente de 60 años ingresa por caída hace 1 hora y pequeño hematoma en cuero cabelludo, al
examen físico ampliado se observa ictericia de piel y mucosas generalizada, abdomen blando, se
palpa estructura quística no dolorosa en hipocondrio derecho que corresponde a vesícula biliar

A
(signo de Courvoisier), en los exámenes de laboratorio se halla niveles bajos en la formación de
estercobilinógeno y urobilinógeno en heces, incremento de la bilirrubina conjugada en la orina,
elevación de fosfatasa alcalina y gamma glutamil transpeptidasa séricas. El presente cuadro puede

EK
ser explicado por:
a. Reabsorción de hematoma
b. Litiasis vesicular
c. Carcinoma de la cabeza de páncreas
OT
d. Carcinoma con estenosis del conducto hepático común
e. Anemia hemolítica

12. Paciente varón de 58 años con antecedente de alcoholismo crónico es diagnosticado y recibe
tratamiento por cirrosis hepática. Hace 2 días refiere familiar que tuvo cambio de conducta y no
NC

reconoce a algunos familiares. Al examen físico, se halla ascitis, circulación colateral en abdomen,
telangiectasias, en el examen de sistema nervioso: rigidez de extremidades, ROT incrementados,
desorientación en el espacio y asterixis. ¿cuál de las siguientes circunstancias, explicaría el cuadro
en este paciente?
a. Uso de diuréticos ahorradores de potasio
BA

b. Incremento de actividad de ureasa bacteriana duodenal


c. Hemorragia gastrointestinal
d. Disminución de la producción de NH3+ en el colon
e. Dieta normo proteica

Se valida la opción e debido al efecto sobre la


encefalopatía.
Con respecto de la opción b es incorrecta debido a que hace referencia al duodeno,
debería indicar colon.

13. Un recién nacido presenta vómitos biliosos poco tiempo después de cada alimento. Al preguntar a
la madre sobre antecedentes, ella recuerda que tuvo polihidramnios durante la gestación, pero un
análisis de cariotipo fue normal. Una de las siguientes es la causa más probable de estos hallazgos
en el recién nacido:
a. Enfermedad de Hirschprung
b. Fístula tráqueo esofágica
c. Divertículo ileal
d. Estenosis pilórica
e. Malrotación de la yema pancreática ventral

14. Un lobulillo hepático se puede dividir en tres zonas como se muestra en el gráfico. ¿Cuál de las
siguientes afirmaciones sobre las tres zonas es verdadera?
a. La zona 1 tiene los menores depósitos de glucógeno
b. La zona 3 es la primera en afectarse en una colestasis extra hepática
c. La zona 2 es más susceptible a la injuria por isquemia que la zona periportal
d. La zona 2 tiene la mayor capacidad de regeneración
e. La zona 1 es la que tiene menos actividad metabólica.

La pregunta 14 ha sido anulada, sin embargo, ningún estudiante se verá afectado


negativamente en su
puntaje debido a esta
anulación.

15. En un estudio de la secreción de hormonas gastrointestinales, sus concentraciones en la vena porta


se midieron durante perfusión luminal del intestino delgado con soluciones de diversas magnitudes
de pH. ¿Qué hormona aumentará en el plasma de la vena porta durante perfusión a través del
intestino con una solución de pH 3? a. CCK
b. gastrina
c. GIP
d. motilina
e. secretina

16. Paciente de 30 años que ingresa a causa de un traumatismo abdominal cerrado. En la exploración
se aprecia discreta palidez de piel y mucosas, auscultación pulmonar normal, taquicardia de 120
/min. Discreta distensión abdominal y matidez en flancos; el hematocrito, que era prácticamente
normal al ingreso, disminuye a 30% a las tres horas. En la Rx de tórax se objetiva fractura de las
costillas 10-11 izquierdas. La causa más probable de la anemización en este paciente es:
a. traumatismo renal con hemorragia retroperitoneal.
b. rotura de hígado con hemoperitoneo.
c. rotura de bazo con hemoperitoneo.
d. rotura de mesos con hemoperitoneo.
e. traumatismo pancreático con pancreatitis traumática.

17. Mujer de 65 años. Consulta por síndrome constitucional asociado a dolor abdominal epigástrico
progresivo irradiado a espalda, de dos meses de evolución. El diagnostico de sospecha de
adenocarcinoma de páncreas se confirma por biopsia. Se realiza examen de imagen de abdomen
para evaluación de estructuras vasculares próximas al tumor pancreático. ¿Cuál es el nombre de la
vena señalada que está ausente, trombosada por infiltración
tumoral, condicionando circulación colateral en la pared
gástrica?

a. Mesentérica superior
b. Coronaria estomaquica
c. Esplénica
d. Porta
e. Renal izquierda

18. Revisando la angiotomografía de un hombre de 70 años en estudio por aneurisma de aorta


abdominal, el radiólogo le informa de la presencia de una oclusión completa de la arteria
mesentérica inferior. El paciente se encuentra completamente asintomático. La oclusión de la
arteria mesentérica inferior cursa de manera asintomática en muchas ocasiones ya que el territorio
que irriga puede recibir flujo proveniente de la arteria: a. cólica derecha
b. gastroduodenal
c. Epigástrica inferior izquierda
d. esplénica
e. cólica media

19. En las patologías de esófago es importante conocer bien la anatomía esofágica. ¿Cuál de las
siguientes afirmaciones es correcta?
a. El esófago tiene capa mucosa, muscular y serosa
b. El esófago abdominal es más largo que el cervical
c. El esófago torácico pasa por detrás del cayado aórtico
d. El epitelio esofágico normal es de tipo cilíndrico.
e. El esófago abdominal es discretamente más largo que el torácico

20. A pesar de que pueda haber variaciones anatómicas, lo habitual es que el ciego sea irrigado por una
rama arterial que proviene de unas de las siguientes arterias:
a. Iliaca derecha
b. Mesentérica inferior
c. Hepática derecha
d. Mesentérica superior
e. Iliaca izquierda

21. Ante un paciente con una cirugía abdominal urgente, el informe operatorio señala que se ha
realizado una resección de todo el duodeno y del tercio proximal del yeyuno manteniendo íntegros
el estómago y todo el íleon, así como los dos tercios distales del yeyuno. En el seguimiento
nutricional del paciente ¿Qué vitamina o mineral presentará con menor probabilidad una
disminución de su absorción?
a. Cianocobalamina
b. Calcio
c. Hierro
d. Transcobalamina
e. Transferrina
22. ¿Cuál de las siguientes alternativas detallan las venas que
confluyen y forman la vena señalada?
a. mesentérica superior, gástrica izquierda y gastroepiploica
izquierda
b. mesentérica inferior, gástrica izquierda y renal
c. esplénica, mesentérica superior y mesentérica inferior
d. esplénica, pancreatoduodenal y omental izquierda
e. gástrica izquierda, esplénica y hepática común

23. ¿Cuál de las siguientes sustancias forma parte de la secreción biliar?


a. Tripsina
b. Lecitina
c. Elastasa
d. Quimotripsina
e. Pepsina

24. El tubo digestivo contiene diferentes tipos de epitelios y glándulas. La estructura señalada es una
………………. y está localizada en el …………...
a. glándula de Brunner / intestino grueso
b. cripta de Lieberkuhn / colon
c. cripta de Lieberkuhn / intestino delgado
d. glándula oxintica / estomago
Aunque las criptas de Lieberkuhn
están
presentes en el intestino
delgado, la
microfotografía es de epitelio de
colon.
e. célula parietal / estómago

25. ¿De qué musculo forma parte el ligamento inguinal?


a. Oblicuo externo del abdomen
b. Oblicuo interno del abdomen
c. Transverso del abdomen
d. Psoas
e. Dorsal ancho

26. Señale cuál de las siguientes afirmaciones NO se relaciona a la siguiente glándula anexa del tubo
digestivo mostrada en la imagen:
a. Es una glándula exocrina compuesta exclusivamente por acinos serosos
b. Su inervación está dada por el nervio auricular mayor (ramo posterior C2), que inerva la
vaina de la glándula así como la piel por encima de esta.
c. Esta glándula produce una secreción mucinosa acuosa, llamada mucoserosa, a través del
conducto de Wharton.
d. Su inflamación puede ser causada por un virus de los Paramyxoviridae, que provocan una
enfermedad muy frecuentemente en niños y adolescentes
e. Es una glándula endocrina y probablemente sea de origen pancreático

Se valida la opción e debido a que no está


relacionada
con la
imagen.

27. ¿Cuál de las siguientes enzimas está localizada en el borde en cepillo y juega un rol en la digestión
de proteínas? a. Alfa dextrinasa
b. Pepsina
c. Enterocinasa
d. Lactasa
e. Carboxipeptidasa A.

Se valida la opción c debido a que es correcta en relación a la pregunta.

28. Una de los siguientes sustancias, NO sirve como un buen agente emulsificante: a. Colesterol
b. Ácidos grasos
c. Sales biliares
d. Lecitina
e. Proteínas de la dieta

Se valida la opción e debido a que es correcta en relación a la pregunta.

29. La sustancia que estimula el crecimiento de la mucosa gástrica es:


a. Secretina
b. Motilina
c. Péptido estimulante de la mucosa gástrica
d. Gastrina
e. Histamina

30. ¿Cuál de las siguientes alternativas es una función de la colecistokinina?


a. Relajación de la vesícula para la salida de bilis
b. Secreción de ácidos biliares
c. Contracción del esfinter de Oddi
d. Secreción de enzimas pancreáticas
e. Contracción del duodeno

Se valida la opción b debido al efecto de la CCK sobre la vesicula biliar.

31. Con respecto a la anatomía del tronco celiaco, señale lo correcto


a. El tronco celiaco se origina de la cara posterior de la aorta abdominal
b. Es una arteria delgada que tiene un calibre entre 2 y 3 mm
c. Una de sus ramas es la arteria gástrica derecha
d. La hepática común que es una de sus ramas, participa en la irrigación del estómago.

32. Con respecto a la anatomía del duodeno, marque la respuesta correcta:


a. Tiene una distribución en forma de “C”, que rodea la cola del páncreas
b. La 3ra porción duodenal está contenida en la pinza vascular aortomesentérica
c. Entre la 1ra y 2da porción se forma un ángulo, conocido como el ángulo de Treitz
d. La 4ta porción se dirige a la izquierda, hacia abajo y hacia atrás.
e. En la tercera porción desemboca el conducto colédoco.

33. El hígado está ampliamente tapizado por peritoneo, la estructura que conecta la cara diafragmática
del hígado precisamente con el diafragma es el ligamento:
a. teres
b. falciforme
c. triangular
d. hepático común
e. coronario

Se validan la opción c y e debido a que forman parte de los ligamentos que fijan el hígado al diafragma.

34. En el íleon se absorbe aproximadamente el 95% de …………………. a través de la circulación


enterohepática. a. agua
b. colesterol
c. sales biliares
d. hidróxicobalamina
e. factor intrínseca

35. La ……………. estimula el mecanismo paracrino de la secreción de ácido clorhídrico.


a. histamina
b. acetilcolina
c. gastrina
d. secretina
e. somatostatina

36. En la digestión de proteinas, ……………. es el principal estímulo para convertir el pepsinógeno en


pepsina. a. la gastrina
b. el pH ácido
c. la acetilcolina
d. la ptialina
e. la somatostatina
37. Con respecto a la somatostatina, marque lo correcto:
a. Es secretada por las células S del intestino
b. Induce a la producción de VIP
c. Interviene en la fase intestinal de la secreción gástrica
d. Produce acetilcolina para estimular a la célula parietal
e. No interviene en la regulación de la secreción de ácido clorhídico

38. En pecten anal, es una estructura comprendida entre:


a. la línea pectínea y los senos anales
b. la línea blanca y la apertura anal
c. el esfínter anal interno y el externo
d. la línea anocutánea y la línea pectínea
e. la línea blanca y columnas anales

39. ¿Cuál de las siguientes alternativas es una proenzima pancreática?


a. Tripsina
b. Elastasa
c. Quimotripsinógeno
d. Amilasa
e. Procarboxipepitidasa C.

40. En la segmentación hepática de Coinaud, el segmento hepático señalado con la flecha, corresponde
a : En la segmentación hepática de Coinaud, la flecha señala el segmento ………….. hepático.
a. IV b. V

c. VI
d. VII
e. VIII

Parcial 2019-02

1. La contracción del músculo ………………………… permite la eliminación de gases (flatos) sin salida de
material fecal; es el mismo músculo cuya relajación, sobretodo en cuclillas, permite el paso del
contenido fecal con menor esfuerzo durante la defecación.
a) Isquirectal
b) Puborrectal
c) Esfínter anal externo
d) Esfínter anal interno
2. Paciente mujer de 54 años se presenta con náuseas, vómitos, estreñimiento, y es diagnosticada de
abdomen agudo quirúrgico; en la cirugía encuentran un vólvulo de ciego. Esta anomalía puede
explicarse por:: a) Falta de rotación intestinal
b) Falta de fusión del mesenterio
c) Defecto en la formación de la cloaca
d) Falta de formación del omento mayor

3. Paciente mujer de 23 años con faringitis aguda, toma para el dolor una tableta de paracetamol con
un poco de agua. Durante la deglución, se relaja su esfínter esofágico inferior y el fondo del
estómago, mientras el bolo está aún en el esófago. ¿Qué sustancia provocara con mayor
probabilidad la relajación del esfínter esofágico inferior y el fondo del estómago en esta mujer?
a) Óxido nítrico
b) Sustancia P
c) Histamina
d) Motilina

4. Luego de tres horas dando exámenes, un alumno de medicina comienza a sentir hambre. Esta
situación es probable que sea mediada por la ______________ que es sintetizada por el
___________________: a) leptina / intestino
b) leptina / estómago
c) grelina / estómago
d) grelina / tejido adiposo

5. Varón de 72 años, con antecedente de diabetes mellitus tipo 2, que presenta enteropatía diabética
caracterizada por estreñimiento. Este problema puede estar asociado a:
a) deficiencia de óxido nítrico
b) aumento del reflejo gastrocólico
c) disminución de la secreción de colecistocinina (CCK)
d) aumento de la secreción del péptido intestinal vasoactivo (PIV)

6. Varón de 54 años con Diabetes Mellitus tipo 2, es diagnosticado de gastroparesia debido a que
presenta sensación de llenura precoz al comer, y reflujo gastroesofágico. Esta alteración en la
relajación receptiva y en el vaciamiento gástrico lo más probable es que se deba a una alteración en:
a) el nervio vago
b) el ganglio celíaco
c) plexo submucoso
d) nervio hipogástrico

7. Varón de 67 años con tos y disminución de peso asociado a tabaquismo pesado, presenta
actualmente disfagia progresiva a alimentos sólidos. Se considera la presencia de un carcinoma de
bronquio izquierdo y por esta razón le realizan una endoscopía esofágica para descartar la
posibilidad de una compresión esofágica por el tumor. Se espera revisar el esófago en la
__________________ estrechez, que está a nivel de la vértebra ______________ a) Tercera
estrechez -T6
b) Segunda estrechez - C6
c) Segunda estrechez - T4
d) Tercera estrechez -T10

8. Varón de 34 años con dolor abdominal agudo en flanco derecho que se irradia a fosa ilíaca derecha,
es operado y se encuentra un divertículo intestinal inflamado, ubicado a 93 cm de la válvula
ileocecal. El origen de este divertículo es una falla en la obliteración de:
a) Conducto vitelino
b) Alantoides
c) Cloaca
d) Conducto anorectal
e) Uraco

En un niño menor de dos años con divertículo intestinal, este divertículo tiene su origen en una falla
en la obliteración de:
a) Conducto anorectal
b) Conducto vitelino
c) Alantoides
d) Cloaca
e) Uraco

9. Mujer de 43 años sufre un grave accidente de tránsito y está hospitalizada en coma, es alimentada
por vía intravenosa durante varias semanas. Producto de este tipo de alimentación, se encuentra en
la endoscopía atrofia de la mucosa gastrointestinal. La causa más probable de esta atrofia son los
bajos niveles séricos de la hormona: a) Colecistocinina
b) Secretina
c) Gastrina
d) PIV

10. Una mujer de 30 años llega al consultorio porque se queja de dificultades para deglutir, la cual se
agravan cada vez más. Se realiza un estudio manométrico para examinar la generación de presión a
lo largo del esófago. Esta prueba revela que las contracciones como respuesta a la deglución están
mal sincronizadas y que la presión en el esfínter esofágico inferior permanece elevada. El
diagnóstico más probable es ____________________ producido por niveles bajos de
____________________ a) acalasia / sustancia P
b) acalasia / óxido nítrico
c) enfermedad por reflujo gastrointestinal / acetilcolina
d) enfermedad por reflujo gastrointestinal / óxido nítrico

11. Paciente de 2 años, llega a emergencia por haber ingerido una moneda con la que estaba jugando.
El lugar más probable donde puede haberse quedado suspendido este objeto es a nivel del
estrechamiento producido a nivel del:
a) músculo milohiodeo
b) músculo aritenoideo
c) músculo cricofaríngeo
d) constrictor superior de la faringe

12. En una apendicectomía, al realizar la incisión de McBurney en la fosa iliaca derecha, es necesario
cortar los siguientes músculos, de afuera hacia adentro:
a) Recto – Oblicuo externo – Transverso
b) Recto – Oblicuo externo – Oblicuo interno
c) Oblicuo externo – Oblicuo interno – Recto
d) Oblicuo externo – Oblicuo interno – Transverso

13. Un varón de 90 años que se encuentra postrado en cama, es referido del asilo para endoscopia por
dificultad para deglutir luego de tomar un medicamento para aliviar el dolor la noche anterior. La
endoscopía revela que la píldora se alojó en el esófago y causó una reacción inflamatoria. Lo más
probable es que esto haya sido por la producción de múltiples ondas:
a) secundarias
b) primarias
c) lentas
d) segmentarias

14. Mujer de 23 años es diagnosticada de bulimia, al examen físico se observa ulceraciones en el


segundo y tercero dedo de la mano derecha. Esto se puede deber al uso continuo de estos dedos
para inducir el vómito, mediante la estimulación del par craneal:
a) V
b) IX
c) X
d) XI

15. Varón de 52 años se presenta por diarrea persistente de seis semanas de duración. En la
colonoscopia se observa un pólipo a nivel del íleon distal. El patólogo informa que se trata de un
tumor neuroendócrino, probablemente originado por las células enterocromafines del intestino. La
sustancia que más probablemente esté produciendo este tumor es:
a) Serotonina
b) Insulina
c) CCK
d) GIP

16. La fase oclusal de la masticación se realiza con la contracción de los músculos: a) digástricos
b) masetero y temporal
c) orbicular y buccinador
d) pterigoideo lateral y digástrico

17. Al tomar su café en Starbucks, un estudiante de medicina sufre una quemadura de primer grado en
el tercio anterior de la superficie dorsal de la lengua. La información de dolor es transmitida por el
nervio: a) cuerda del tímpano
b) glosofaríngeo
c) lingual
d) facial

18. Paciente es evaluado por faringitis aguda en consultorio externo. El médico de familia le solicita que
abra la boca y saque la lengua. Para realizar la acción de sacar la lengua, es necesario que se
contraiga el músculo: a) estilogloso
b) geniogloso
c) palatogloso
d) transverso de la lengua

19. Paciente con síndrome de Sjögren, presenta “boca seca” (disminución de


la producción de saliva) y caries dental, asociada a la pérdida de la función
de tampón de la saliva. Esta desminerilización del diente puede
comprometer a las prolongaciones citoplasmáticas ubicadas en los tubos
huecos de la estructura señalada con la letra:
a) B
b) A
c) E
d) C

20. Mujer de 32 años acude a consulta por presentar disfagia de progresión


lenta, reflujo gastroesofágico y vómitos desde hace 3 meses de evolución
progresiva. Se le realiza un estudio radiológico con contraste en el que se
observa estrechamiento del esfínter esofágico inferior (imagen). Según sus
conocimientos, este paciente se beneficiaría con el uso de: a) agonista
beta adrenérgico
b) agonista alfa adrenérgico
c) análogo de óxido nítrico
d) análogo de Sustancia P

Final 2019-02

1) La reabsorción de sodio y cloro en las glándulas salivales se da principalmente en el:


a) conducto intercalado
b) conducto excretor
c) conducto estriado
d) acino glandular

2) Paciente de 35 años con dolor en hipocondrio derecho irradiado a la punta de la escápula. Se observa
en la ecografía abdominal cálculos en la vesícula biliar. Es operado de emergencia realizando una
incisión a lo largo del reborde costal derecho. La información de dolor de esta zona es inervada por
las raíces nerviosas: a) T5 – T9
b) T6 – T7
c) T7 – T8
d) T9 – T10

3) Paciente de 23 años con dolor intenso periumbilical de inicio agudo. Es ingresado a sala de
operaciones por abdomen agudo, el cirujano encuentra sangrado a unos 60 cm proximal a la válvula
ileocecal. La estructura que está sangrando muy probablemente es un derivado embriológico del:
a) uraco
b) ligamento Hepatoduodenal
c) alantoides
d) conducto vitelino

4) Paciente de 64 años de edad con dolor en epigastrio que se distribuye en banda hacia la espalda. En
la tomografía se encuentra tumor en cabeza de páncreas que compromete a un vaso que discurre
entre la cabeza del páncreas y el proceso uncinado. Debido a esto, este cáncer se considera
irresecable porque compromete a la arteria: a) mesentérica superior
b) mesentérica inferior
c) tronco celíaco
d) aorta

5) Paciente con carcinoma gástrico avanzado, en preoperatorio para gastrectotomía total. ¿Cuál de los
siguientes ganglios linfáticos recibirá primero células metastásticas con mayor probabilidad: a)
celíaco
b) cisterna del quilo
c) esplénico
d) gástrico posterior
6) Paciente de 77 años con dolor abdominal difuso de 3 horas de evolución. En los estudios de imágenes
se observa oclusión probablemente aterosclerótica de la arteria mesentérica superior; no se observa
necrosis, lo cual puede ser explicado por la irrigación sanguínea colateral. ¿Qué vasos ofrecen
colaterales entre el tronco celíaco y la arteria mesentérica superior?
a) Gástrica izquierda y hepática
b) Cística y duodenal
c) Gastroomental derecha e izquierda
d) Pancreaticoduodenal superior e inferior

7) Paciente de 62 años con vólvulo de intestino delgado e isquemia intestinal. Se realiza laparotomía
exploratoria para liberar la obstrucción. ¿Cuál estructura se utilizará como punto de referencia para
determinar la posición de la unión duodenoyeyunal?
a) Vasos rectos
b) Ligamento suspensorio del duodeno (de Treitz)
c) Ligamento frenocólico
d) Nacimiento de la Arteria mesentérica superior

8) Paciente de 42 años con dolor abdominal intenso y hematemesis. En la endoscopía se observa una
úlcera duodenal posterior perforada con hemorragia intrabdominal. ¿Cuál de las siguientes arterias
estará comprometida? a) Gástrica izquierda
b) Mesentérica superior
c) Gástrica derecha
d) Pancreaticoduodenal posterosuperior

9) Paciente de 51 años con antecedente de enfermedad diverticular acude a emergencia por sangrado
profuso y dolor en cuadrantes inferiores. ¿Cuál es el origen más probable de la sangre que pierde en
la hemorragia? a) Cólica media
b) Mesentérica inferior
c) Rectal superior
d) Rectal inferior

10) Paciente de 13 días de vida con vómitos explosivos a las dos horas después de lactar. Al examen físico
se palpa la oliva pilórica. ¿Cuál es el nervió cuyos filetes dan inervación eferente a la estructura
afectada? a) Vago
b) Esplácnico torácico mayor
c) Mesentérico superior
d) Esplácnico torácico menor

11) Paciente de 80 años con dolor abdominal intenso y antecedente de estreñimiento crónico. En la
colonoscopía se observa divertículos con áreas ulceradas difusas en colon sigmoides y descendente.
El cirujano programa una cirugía de resección y planifica que para realizar esta resección tendrá que
cortar los siguientes vasos y nervios: a) plexo mesentérico superior y arteria rectal.
b) nervio esplácnico torácico mayor y arteria cólica media.
c) ramas del nervio vago y arteria ileocólica.
d) nervio esplácnico pélvico y artera cólica izquierda.

12) Paciente de 46 años ingresa a emergencia con dolor en cuadrante superior derecho e ictericia. En la
ecografía se observa cálculos en la vesícula biliar. ¿Cuál de los siguientes nervios transmite el dolor
de la colecistitis?
a) Filetes aferentes del nervio vago derecho, referido al ángulo inferior de la escápula
b) Filetes nerviosos de las raíces T1 a T4, con referencia al hombro izquierdo
c) Fibras aferentes simpáticas del nervio esplácnico torácico mayor, con referencia a los
dermatomas T6 a T8
d) Fibras aferentes de los ramos primarios dorsales de los nervios raquídeos T8 a T10 con
referencia al epigastrio

13) Un niño de 8 años es alimentado por sus padres con un Cheeseburguer, papitas fritas y un vaso de
Coca Cola. ¿La presencia de lípidos, carbohidratos y proteínas en el duodeno estimulan la liberación
de cuál de las siguientes hormonas en la mucosa duodenal?
a) Colecistoquinina (CCK)
b) Péptido inhibidor gástrico (GIP)
c) Secretina
d) Insulina

14) La fase cefálica del control de la secreción gástrica corresponde a cerca del 30% de la secreción ácida
y es consecuencia de un reflejo. ¿Cuál de las siguientes alternativas puede eliminar totalmente la
fase cefálica de la secreción gástrica?
a) Vaguectomía
b) Uso de atropina
c) Uso de bloqueador de receptor H2 de histamina
d) Uso de antiácidos

15) Recién nacido de 24 horas con llanto, vómitos y sin eliminación de meconio. Luego de los estudios
auxiliares se diagnostica la Enfermedad de Hirshprung, esta enfermedad se caracteriza por la
ausencia de:
a) las células de Cajal en plexo mioentérico
b) el plexo mioentérico en el recto y colon sigmoides
c) las células mioepiteliales en el recto distal
d) el plexo de Meissner en el recto distal y colon sigmoides

16) Los complejos motores migratorios aparecen aproximadamente cada 90 minutos entre las comidas,
y se considera que son estimulados por la hormona motilina. La ausencia de estos complejos
migratorios podría producir un aumento en:
a) la motilidad duodenal
b) el vaciamiento gástrico
c) la deglución
d) las bacterias intestinales

17) ¿Cuál de las siguientes es una consecuencia probable de la resección del íleon? a) Gastritis atrófica
b) Deficiencia de vitamina B12
c) Esteatorrea
d) Úlcera péptica

18) Los movimientos en masa son importantes en la fisiología intestinal. Estos movimientos en masa
ocasionan: a) la sensación de defecar
b) el peristaltismo duodenal
c) la retropulsión gástrica
d) la contracción del esfínter anal interno

19) La toxina colérica hace que aumente los niveles de AMPc intracelular, y este aumento hace que se
mantenga abierto un canal en las células de la cripta de Lieberkuhn. En condiciones fisiológicas, en
una persona sana ¿Qué sustancia puede promover que el canal quede abierto también?
a) Somatostatina
b) Óxido nítrico (NO)
c) Péptido intestinal vasoactivo (VIP)
d) Péptido similar al glucagón 1 (GLP1)

20) Paciente de 64 años con tumor abdominal que comprime la cisterna del quilo. En la biopsia de
duodeno tomada como parte del estudio, el patólogo puede observar
a) Dilatación del vaso quilífero central
b) Contracción de las venas de las vellosidades
c) Vellosidades intestinales más largas
d) Engrosamiento de la lámina basal
21) En la enfermedad de Crohn es posible encontrar células de Paneth en el colon. Esto se puede deber
a la especial función de estas células en:
a) la activación de la inmunidad adquirida
b) mantener la inmunidad innata
c) producir hormonas
d) producir Ig A

22) En 1967 se descubrió que la epidemia de Kuru, una enfermedad por priones, en el distrito de Okapa
en Papua Nueva Guinea, era causada por la costumbre de comer la carne de los muertos. Ahora se
sabe que las proteínas priónicas ingresan al organismo a través de:
a) los enterocitos
b) la transmigración
c) las células de Paneth
d) las células M

23) Paciente de 48 años con alteraciones en el tránsito intestinal por diabetes mellitus tipo 2; se presenta
con esteatorrea, flatulencia y malabsorción de grasas. Las pruebas de función hepática y biliar están
dentro de rangos normales. Una causa de la disminución de sales biliares puede ser:
a) el sobrecrecimiento bacteriano
b) la deficiencia de pepsina
c) la deficiencia de elastasa
d) la hiperestimulación del GLUT5
24) Al usar azúcar de mesa (sacarosa) para endulzar su café, el estudiante de medicina sabe que lo más
probable es que para su absorción tendrá utilizar el/los transportadore(s) ________________ que se
encuentran en la membrana apical de los enterocitos.
a) SGLT-1
b) GLUT2 y GLUT5
c) SGLT-1 y GLUT5
d) SGLT-1 y GLUT2

25) Una persona con la producción normal de lactasa; cada vez que toma leche, los productos de la
degradación de la lactosa por parte de la lactasa ingresarán al enterocito usando el/los
transportador(es) _____________: a) SGLT-1
b) GLUT2 y GLUT5
c) SGLT-1 y GLUT5
d) SGLT1 y GLUT2

26) En una persona sana, luego de una comida basada en carnes rojas cocinadas con baja cantidad de
sal, al enterocito pueden ingresar solamente:
a) aminoácidos
b) aminoácidos y dipéptidos
c) aminoácidos, dipéptidos y tripéptidos
d) aminoácidos, dipéptidos, tripéptidos y tetrapéptidos

27) Paciente con deficiencia congénita de procolipasa, sufre de esteatorrea cada vez que come comidas
ricas en grasas. En el estudio de composición de las heces, uno espera encontrar una cantidad
aumentada de: a) colesterol
b) triglicéridos
c) fosfolípidos
d) lisolecitina

28) Al tomar su café en Starbucks, un estudiante de medicina sufre una quemadura de primer grado en
el tercio anterior de la superficie dorsal de la lengua. La información de dolor es transmitida por el
nervio: a) cuerda del tímpano
b) glosofaríngeo
c) lingual
d) facial
29) Paciente de 32 años con esteatosis hepática no alcohólica. Se le realiza una biopsia hepática que
confirma la esteatosis; en el tejido se observan depósitos de lípidos en los hepatocitos, los cuales
contienen principalmente: a) triglicéridos
b) colesterol
c) ácidos grasos libre
d) acil-carnitina

30) Paciente de 21 años que es estudiante de medicina, llega a consulta refiriendo que en épocas de
exámenes su piel se vuelve amarillenta. Le realizan el diagnóstico de Síndrome de Gilbert asociado a
mutación del gen UGT1A1, luego de unas semanas acude a la consulta por ictericia asociada a resfrío.
Al hacerle un análisis de sangre, usted sabe que encontrará valores elevados de:
a) bilirrubina directa
b) bilirrubina indirecta
c) bilirrubina tipo delta
d) fosfatasa alcalina
31) Se realizó un experimento en el cual se inyectó tinta china en el peritoneo de ratas de laboratorio. Al
realizarse una biopsia hepática de dichos animales, se encontró que el tinte negro estaba depositado
en: a) las células de Ito
b) los hepatocitos
c) las células de Kupffer
d) las células de Disse

32) Paciente de 74 años de edad con shock hipovolémico asociado a deshidratación aguda severa. En
este paciente es posible encontrar hipoxia en zona …… del lobulillo hepático e infarto ……………… del
intestino. a) 1 / mucoso
b) 1 / transmural
c) 3 / transmural
d) 3 / mucoso

33) Niño de 5 años con historia de tres días de evolución caracterizado por fiebre, malestar general,
odinofagia, hiporexia, e irritabilidad. Al examen se observa lesiones ulcerativas de 4 mm de diámetro
en mucosa yugal, con fondo blanquecino y eritema periférico. El diagnóstico más probable es:
a) candidiasis oral
b) leucoplasia
c) aftas
d) herpes

34) Paciente de 52 años con enfermedad por reflujo gastroesofágico de 30 años de evolución. Se realiza
endoscopía en la que se encuentra mucosa eritematosa proximal a la línea Z. Para corroborar la
presencia de lesión preneoplásica, se toma una biopsia de esa zona, en la que se espera encontrar:
a) metaplasia gástrica
b) metaplasia intestinal
c) displasia gástrica
d) adenocarcinoma

35) Un estudiante de medicina termina su último examen final. Al abrir la boca para comer una
hamburguesa con papas fritas, la activación de los receptores muscarínicos de las células acinar y
ductal estimularan un mayor flujo de saliva, con lo cual disminuirá la concentración salival de:
a) potasio
b) bicarbonato
c) sodio
d) cloro
36) Un estudiante toma su desayuno consistente en un pan con mantequilla y queso. Antes que se
puedan digerir las grasas, es necesario que sean emulsificadas. La hormona ………………………. estimula
la liberación de las sustancias emulsificadoras.
a) colecistoquinina (CCK)
b) secretina
c) lipasa pancreática
d) gastrina

37) Al comer un pollo a la brasa entero, con papitas fritas y ensalada, la sustancia que estimulará la
liberación de HCl en el estómago es:
a) el neuropeptido Y
b) la secretina
c) la bombesina
d) la colecistoquinina (CCK)
38) Paciente con disminución marcada del apetito asociado a cáncer terminal, se podría utilizar análogos
de ……………..
para promover la ingesta de alimentos.
a) el péptido similar al glucagón (GLP)
b) la serotonina
c) la secretina
d) la endorfina

39) Un hombre de 51 años presenta de forma súbita vómitos masivos de sangre roja brillante. Tiene
como antecedente hepatitis viral B hace 23 años. En la exploración física: FC 103 latidos/min, PA
85/50 mmHg, se palapa la punta del bazo y niega vómitos. Su hematocrito es 21%, la prueba
serológica de HBsAg es positiva. En la ecografía se observa hígado con nodulaciones. ¿Cuál es la causa
más probable para la hematemesis?
a) Esófago de Barrett
b) Síndrome de Mallory Weiss
c) Varices esofágicas
d) Esofagitis por reflujo

40) Durante el reflejo del vómito, uno de los primeros sucesos es:
a) el peristaltismo inverso
b) la contracción del píloro
c) la apertura de la glotis
d) el esfínter esofágico inferior abierto

41) Paciente de 34 años que acude por diarrea desde hace 4 días asociado a comer papa rellena con ají
en el Estadio Nacional, se exacerba cuando toma lácteos o come grasas. Se acompaña de flatulencia
y episodios de tenesmo. Este cuadro de diarrea e intolerancia a la lactosa de inicio agudo se explica
por la:
a) enfermedad de Crohn
b) infección por V. cholera
c) colitis ulcerativa
d) giardiasis

42) En cuanto a la secreción pancreática, mientras mayor es el flujo, mayor es la concentración de: a)
potasio
b) bicarbonato
c) cloro
d) sodio

43) Paciente de 10 años con diarrea crónica, distensión abdominal, anorexia. Se le ha encontrado
anticuerpos antigliadina y antiendomisio. Es más probable que la diarrea se correlacione con el
hallazgo histológico de: a) adelgazamiento de las criptas
b) linfocitos intramusculares
c) atrofia de las vellosidades
d) úlceras duodenales

44) La glándula parótida está inervada por el par craneal:


a) VII
b) IX
c) X
d) XII
45) Las glándulas de Brunner se encuentran en la:
a) mucosa del esófago
b) submucosa del íleon
c) mucosa del yeyuno
d) submucosa del duodeno

46) ¿Cuál de las siguientes afirmaciones describe correctamente la función de la inervación


parasimpática del tracto gastrointestinal?
a) La norepinefrina es el principal neurotransmisor excitatorio.
b) La actividad parasimpática produce la relajación de los esfínteres
c) La actividad parasimpática excesiva puede provocar un trastorno llamado ileo paralítico
(parálisis del músculo liso intestinal)
d) El pH luminal, la osmolaridad y la distensión muscular son detectados por fibras
parasimpáticas eferentes

47) Al ingerir una cantidad de glucosa por vía oral, esta es interiorizada en las células del organismo más
rápido que si esa misma cantidad de glucosa hubiese sido administrada por vía endovenosa. Este
fenómeno sucede gracias a la acción de la sustancia secretada por las células:
a) G
b) I
c) K
d) S

48) A …………… secreción de HCl en el lumen gástrico, …………….. pH en la sangre venosa gástrica a) mayor
/mayor
b) mayor / menor
c) menor / mayor
d) mayor / igual

49) ¿En cuál de las siguientes situaciones hay un menor flujo de secreción salival?
a) Masticar goma de mascar
b) Imaginarse ser sometido a un examen dental
c) Exposición a olor nauseabundo
d) Sueño
50) Al seccionar el nervio facial a nivel timpánico, usted esperaría:
A. Disminución del gusto en la punta de la lengua
B. Imposibilidad para el cierre del istmo de las fauces
C. Ausencia de la termoalgesia de la lengua
D. Imposibilidad para protruir la lengua
51) En un paciente con hiperestimulación simpática se espera que las ondas lentas tengan un ritmo:
A. Menor en íleon terminal que en el duodeno
B. Mayor en el íleon que el duodeno
C. Mayor en el estómago que el duodeno
D. Mayor en el estómago que en íleon terminal
52) Marque lo correcto en relación al divertículo de Meckel.
A. Se encuentra usualmente a 60 cm de la VIC
B. Contiene mucosa esofágica en algunas ocasiones
C. Se produce en el lado del íleon
D. Se relaciona un defecto en el desarrollo del intestino posterior
53) Al disminuir el pH duodenal por el HCl gástrico, se libera principalmente una hormona cuya célula
diana es :
A. Células S del intestino
B. Células ductales del colédoco
C. Acino pancreáticos
D. Células ductal del Wirsung
54) El ecografista sabe que para poder visualizar el nacimiento de la arteria mesentérica superior, debe
colocar el transductor sobre la piel de la siguiente región abdominal:
A. Epigastrio
B. Hipogastrio
C. Hipocondrio derecho
D. Mesogastrio
55) Durante una cirugía oncológica, el cirujano observa que los órganos abdominales tienen libre
movimiento dentro de la cavidad abdominal, excepto:
A. Vesícula biliar
B. Yeyuno
C. Estómago
D. Colon ascendente
56) Paciente obeso con Covid-19 es intubado por interno inexperto, quien al solicitar que bombeen aire
dentro del tubo endotraqueal, nota que el epigastrio se ditiende. Al sospecha que ha introducido el
tubo en el estómago, también es cierto que:
A. Aumenta la frecuencia de ondas lentas
B. Disminuye el pH gástrico
C. Disminuiría el tono del píloro
D. Aumenta el pH gástrico
57) Paciente con apendicitis aguda, que debuta con dolor en mesogastrio. ese dolor se debea
estimulación de receptores del dolor cuyas fibras van a viajar a la médula espinal a través de:
A. Nervios simpáticos
B. Plexo hipogástrico
C. Nervio esplácnico pélvico
D. Nervio vago
58) La rotación en sentido longitudinal del estómago en el desarrollo embriológico condiciona que el
nervio vago derecho quede a nivel:
A. Izquierdo
B. Oblicuo
C. Anterior
D. Posterior
59) Una de las siguientes sustancias reguladoras, puede actuar de forma paracrina y como hormona.
Marque la correcta:
A. Péptido insulinotrópico dependiente de glucosa
B. GRP
C. Acetilcolina
D. Somatostatina
60) El alcohol y la cafeína estimulan la fase ______________ de la producción de ácido clorhídrico. a)
cefálica
b) gástrica
c) intestinal
d) gástrica y cefálica
ECU 1
1) Con respecto a la saliva, marque la respuesta correcta:

Respuesta seleccionada:
el sistema simpático estimula su secreción
● Pregunta 2
2 de 2 puntos

Respecto a las enfermedades del esófago, marque lo correcto:

Respuesta
seleccionada: el diagnóstico diferencial de la acalasia es la enfermedad de Chagas
esofágica
● Pregunta 3
0 de 2 puntos

En relación a la fisilogía gástrica, marque lo correcto:

la cimetidina actúa en la región basolateral de la célula parietal


● Pregunta 4
2 de 2 puntos

La célula mucosa del cuello gástrico produce:

Respuesta seleccionada:
Moc
o
● Pregunta 5
0 de 2 puntos

La saliva puede tener una variedad de electrolitos en su composición. Entre ellos el cloro,
respecto al cual se puede afirmar:

Respuesta seleccionada:
Respuestas:

Su concentración no llega a ser tan alta como en el plasma

● Pregunta 6
2 de 2 puntos
Los músculos de la masticación que producen la retropulsión de la mandíbula son:

Respuesta seleccionada:
temporales
● Pregunta 7
0 de 2 puntos

Respecto a las glándulas salivales, marque lo incorrecto:

Respuesta seleccionada:
la glándula sublingual tiene forma de garfio

● Pregunta 8
2 de 2 puntos

Respecto a la anatomía del estómago, marque lo correcto:

Respuesta seleccionada:
la arteria gástrica derecha nace de la arteria hepática común
● Pregunta 9
2 de 2 puntos

En cuanto a la saliva, marque lo correcto:

Respuesta seleccionada:
La amilasa cumple función digestiva
● Pregunta 10
2 de 2 puntos

Durante el ataque con gas sarín (bloqueador de la acetilcolinesterasa) en el metro de Tokio, en


1995, el personal de salud notó que los pacientes afectados presentaban:

Respuesta seleccionada:
HipersalivacióN

CI1
● Pregunta 1

La presencia de atresias y estenosis duodenales se deben


básicamente a una:

Falta de recanalización


Pregunta 2

● 2 de 2 puntos

El ligamento de Treitz característicamente:

Respuesta
seleccionada: Suspende el ángulo de Treitz

Respuestas: Está adherido a la unión yeyuno-ileal

Se encuentra a nivel de hipogastrio


derecho
Suspende el ángulo de Treitz

Recibe irrigación de la arteria mesentérica


inferior


Pregunta 3

● 0 de 2 puntos

Al deglutir un bolo alimenticio, es lógico suponer que al pasar por


el esófago haya un mayor consumo de oxígeno en la pared
del tercio:

Respuesta seleccionada:
Distal

Respuestas: No hay diferencia

Medio

Proximal

Distal


Pregunta 4

● 0 de 2 puntos


Estudiante de medicina de 20 años, se ha amanecido estudiando
para su examen de Sistema Digestivo. No ha probado
alimento desde la cena, por lo que se puede afirmar que la
motilidad de esta persona está siendo regulada por:

Respuesta seleccionada:
CCK

Respuestas: VIP

Motilina

CCK

Adrenalina


Pregunta 5

● 2 de 2 puntos

La hernia fisiológica se produce dentro de:

Respuesta seleccionada:
Cordón umbilical

Respuestas: Alantoides

Saco Vitelino

Saco Amniótico

Cordón umbilical

Pregunta 6

● 0 de 2 puntos

El crecimiento de un adenocarcinoma de páncreas compromete


la pared gástrica por contigüidad. ¿Qué parte del estómago
se esperaría esté comprometido?

Respuesta seleccionada:
Pared anterior del píloro

Respuestas: Pared anterior del píloro

Pared posterior del fondo

Pared anterior del cardias

Pared posterior del antro


Pregunta 7

● 2 de 2 puntos

Al comer unas papitas fritas con mayonesa, el vaciamiento


gástrico disminuye por efecto directo de la hormona:

Respuesta seleccionada:
colecistoquinina (CCK)

Respuestas: motilina
somatostatina

secretina

colecistoquinina (CCK)


Pregunta 8

● 0 de 2 puntos

Paciente que come entera una pizza familiar de chorizo y queso.


Es posible esperar que debido a la cantidad de alimento
ingerida, las ondas lentas hayan:

Respuesta
seleccionada: Aumentado su frecuencia por estímulo
parasimpático

Respuestas: Disminuido su frecuencia por estímulo


parasimpático

Sufrido ninguna alteración en su frecuencia

Aumentado su frecuencia por estímulo


parasimpático

Aumentado su frecuencia por estímulo


simpático


Pregunta 9

● 2 de 2 puntos


En este caso se puede afirmar con seguridad que se presenta:

Respuesta
seleccionada: contracciones tónicas en la región ano
rectal

Respuestas: pérdida de inervación por el nervio


esplácnico menor

dilatación de tracto gastrointestinal


afectado

contracciones tónicas en la región ano


rectal

células ganglionares sólo en el ano recto


Pregunta 10

● 2 de 2 puntos

La forma más común de atresia esofágica contiene:

Respuesta
selecciona Estenosis proximal del esófago más fístula
da: traqueoesofágica distal

Respuestas: Estenosis distal del esófago más fístula


traqueoesofágica proximal

Estenosis proximal y distal del esófago más


fístula traqueoesofágica distal
Estenosis distal del esófago más fístula
traqueoesofágica distal

Estenosis proximal del esófago más fístula


traqueoesofágica distal


Niño de sexo masculino de 2 años de edad, sufre de
estreñimiento desde el nacimiento (1 deposición cada 3-4
días). Madre menciona que le estimula la defecación con
un termómetro rectal, y continuo uso de enemas y
laxantes. Desde hace 6 meses comienza con vómitos
postprandiales. Los síntomas aumentan en frecuencia y
magnitud y están en relación con los episodios de
estreñimiento. No refiere fiebre, tos, diarrea ni lesiones
cutáneas. Al examen físico presenta regular estado
general, luce deshidratado. Abdomen distendido, blando,
depresible e indoloro. No se palpan masas abdominales.
Se permeabiliza el canal anal con termómetro rectal,
encontrando cierta resistencia. Salida de material fecal mal
oliente en regular cantidad. Exámenes de laboratorio:
hemograma normal. Signos inflamatorios de fase aguda
negativos. Alcalosis metabólica leve en sangre venosa.
Radiografía con enema baritado muestra recto y colon
sigmoides dilatados (megacolon). Biopsia profunda:
ausencia de células ganglionares en la muestra enviada.
Se realiza cirugía correctiva.
La percepción de la pirosis (sensación de dolor o
quemazón en el esófago) asociado al reflujo
gastroesofágico, puede aparecer o exacerbarse debido
a:

Respuesta
seleccionada: Ejercicio

Respuestas: Somatostatina

Uso de antiácidos

Ejercicio

Bipedestación


Pregunta 2
● 0 de 3,3334 puntos

Niño de sexo masculino de 2 años de edad,


sufre de estreñimiento desde el nacimiento
(1 deposición cada 3-4 días). Madre
menciona que le estimula la defecación con
un termómetro rectal, y continuo uso de
enemas y laxantes. Desde hace 6 meses
comienza con vómitos postprandiales. Los
síntomas aumentan en frecuencia y
magnitud y están en relación con los
episodios de estreñimiento. No refiere fiebre,
tos, diarrea ni lesiones cutáneas. Al examen
físico presenta regular estado general, luce
deshidratado. Abdomen distendido, blando,
depresible e indoloro. No se palpan masas
abdominales. Se permeabiliza el canal anal
con termómetro rectal, encontrando cierta
resistencia. Salida de material fecal mal
oliente en regular cantidad. Exámenes de
laboratorio: hemograma normal. Signos
inflamatorios de fase aguda negativos.
Alcalosis metabólica leve en sangre venosa.
Radiografía con enema baritado muestra
recto y colon sigmoides dilatados
(megacolon). Biopsia profunda: ausencia de
células ganglionares en la muestra enviada.
Se realiza cirugía correctiva.
Considerando que este paciente está
sometido a estrés por el agravamiento de
su enfermedad, es posible afirmar que
sus ondas lentas están:

Respuesta seleccionada:
Hipopolarizadas

Respuestas: Hipopolarizadas

Hiperpolarizadas

Desmotivadas

Despolarizadas

Pregunta 3

● 3,3334 de 3,3334 puntos

Niño de sexo masculino de 2 años de edad,


sufre de estreñimiento desde el nacimiento
(1 deposición cada 3-4 días). Madre
menciona que le estimula la defecación con
un termómetro rectal, y continuo uso de
enemas y laxantes. Desde hace 6 meses
comienza con vómitos postprandiales. Los
síntomas aumentan en frecuencia y
magnitud y están en relación con los
episodios de estreñimiento. No refiere fiebre,
tos, diarrea ni lesiones cutáneas. Al examen
físico presenta regular estado general, luce
deshidratado. Abdomen distendido, blando,
depresible e indoloro. No se palpan masas
abdominales. Se permeabiliza el canal anal
con termómetro rectal, encontrando cierta
resistencia. Salida de material fecal mal
oliente en regular cantidad. Exámenes de
laboratorio: hemograma normal. Signos
inflamatorios de fase aguda negativos.
Alcalosis metabólica leve en sangre venosa.
Radiografía con enema baritado muestra
recto y colon sigmoides dilatados
(megacolon). Biopsia profunda: ausencia de
células ganglionares en la muestra enviada.
Se realiza cirugía correctiva.
Debido al acúmulo de material fecal en todo
el marco colónico, y a la irritación
química asociada, el peristaltismo del
íleon distal se debe encontrar:

Respuesta
seleccionada: Inhibido

Respuestas: Afectado por un reflejo


vago-vagal

No sufre alteraciones

Estimulado
Inhibido


Pregunta 4

● 0 de 3,3334 puntos

Niño de sexo masculino de 2 años de edad,


sufre de estreñimiento desde el nacimiento
(1 deposición cada 3-4 días). Madre
menciona que le estimula la defecación con
un termómetro rectal, y continuo uso de
enemas y laxantes. Desde hace 6 meses
comienza con vómitos postprandiales. Los
síntomas aumentan en frecuencia y
magnitud y están en relación con los
episodios de estreñimiento. No refiere fiebre,
tos, diarrea ni lesiones cutáneas. Al examen
físico presenta regular estado general, luce
deshidratado. Abdomen distendido, blando,
depresible e indoloro. No se palpan masas
abdominales. Se permeabiliza el canal anal
con termómetro rectal, encontrando cierta
resistencia. Salida de material fecal mal
oliente en regular cantidad. Exámenes de
laboratorio: hemograma normal. Signos
inflamatorios de fase aguda negativos.
Alcalosis metabólica leve en sangre venosa.
Radiografía con enema baritado muestra
recto y colon sigmoides dilatados
(megacolon). Biopsia profunda: ausencia de
células ganglionares en la muestra enviada.
Se realiza cirugía correctiva.
En cuanto a los reflejos gastrocólico y
gastroduodenal en este paciente, indique
lo correcto:
Respuesta
selecci Se dan por nervios intrínsecos
onada: del sistema entérico

Respuestas El control del nervio vago sobre


: el recto se ha abolido

El reflejo gastrocólico es más


marcado en adultos que en
niños

Se dan por nervios intrínsecos


del sistema entérico

Se pueden considerar reflejos


vago-vagales


Pregunta 5

● 3,3334 de 3,3334 puntos

Niño de sexo masculino de 2 años de edad,


sufre de estreñimiento desde el nacimiento
(1 deposición cada 3-4 días). Madre
menciona que le estimula la defecación con
un termómetro rectal, y continuo uso de
enemas y laxantes. Desde hace 6 meses
comienza con vómitos postprandiales. Los
síntomas aumentan en frecuencia y
magnitud y están en relación con los
episodios de estreñimiento. No refiere fiebre,
tos, diarrea ni lesiones cutáneas. Al examen
físico presenta regular estado general, luce
deshidratado. Abdomen distendido, blando,
depresible e indoloro. No se palpan masas
abdominales. Se permeabiliza el canal anal
con termómetro rectal, encontrando cierta
resistencia. Salida de material fecal mal
oliente en regular cantidad. Exámenes de
laboratorio: hemograma normal. Signos
inflamatorios de fase aguda negativos.
Alcalosis metabólica leve en sangre venosa.
Radiografía con enema baritado muestra
recto y colon sigmoides dilatados
(megacolon). Biopsia profunda: ausencia de
células ganglionares en la muestra enviada.
Se realiza cirugía correctiva.
Con respecto a la defecación señale el
enunciado correcto:

Respuesta
selecci Es estimulado por un llenado de
onada: la cuarta parte del volumen
rectal

Respuestas Es completamente voluntario y


: mediado por el nervio
pudendo

La aferencia parasimpática es
transmitida por vía del nervio
vago

Es un reflejo netamente local

Es estimulado por un llenado de


la cuarta parte del volumen
rectal


Pregunta 6

● 0 de 3,3334 puntos


Niño de sexo masculino de 2 años de edad,
sufre de estreñimiento desde el nacimiento
(1 deposición cada 3-4 días). Madre
menciona que le estimula la defecación con
un termómetro rectal, y continuo uso de
enemas y laxantes. Desde hace 6 meses
comienza con vómitos postprandiales. Los
síntomas aumentan en frecuencia y
magnitud y están en relación con los
episodios de estreñimiento. No refiere fiebre,
tos, diarrea ni lesiones cutáneas. Al examen
físico presenta regular estado general, luce
deshidratado. Abdomen distendido, blando,
depresible e indoloro. No se palpan masas
abdominales. Se permeabiliza el canal anal
con termómetro rectal, encontrando cierta
resistencia. Salida de material fecal mal
oliente en regular cantidad. Exámenes de
laboratorio: hemograma normal. Signos
inflamatorios de fase aguda negativos.
Alcalosis metabólica leve en sangre venosa.
Radiografía con enema baritado muestra
recto y colon sigmoides dilatados
(megacolon). Biopsia profunda: ausencia de
células ganglionares en la muestra enviada.
Se realiza cirugía correctiva.
El contenido fecal se detiene en la zona
inmediatamente proximal a la zona donde
hay una menor presencia de:

Respuesta
seleccionada: Neuropéptido Y

Respuestas: Enteroquinasa

Péptido intestinal
vasoactivo

Neuropéptido Y

Acetilcolina


Estudiante de medicina de la UPC de 21 años sufre de
gastritis aguda ocasionada por comer en lugares poco
higiénicos. Suele consumir caramelos ( chupar ) mientras
está en clase hasta la tarde. Toma gaseosas regularmente
(carbohidratos 46%, sodio 53%). También toma regular
cantidad de leche (grasa 35%, lactosa 35%, proteínas
30%), pues le calma un poco el dolor el ardor que siente
por la gastritis. Incluso, cuando puede, se toma dos vasos
de agua fría para calmar las molestias. Ha decidido ir al
médico para tratarse pues ya no soporta el dolor, el cual
está seguro que los síntomas se deben a una elevada
producción de ácido clorhídrico en el estómago, y por ello
le ha recetado Ranitidina (antihistamínico), con lo que
siente mejoría.
Para reducir la secreción de HCl en esta paciente se
podría usar sustancias similares a:

Respuesta
seleccio Péptido insulinotrópico
nada: dependiente de la glucosa
(GIP)

Respuestas: Bombesina

Péptido insulinotrópico
dependiente de la glucosa
(GIP)

Acetilcolina

Histamina


Pregunta 2

● 0 de 3,3333 puntos


Estudiante de medicina de la UPC de 21 años
sufre de gastritis aguda ocasionada por
comer en lugares poco higiénicos. Suele
consumir caramelos ( chupar ) mientras está
en clase hasta la tarde. Toma gaseosas
regularmente (carbohidratos 46%, sodio
53%). También toma regular cantidad de
leche (grasa 35%, lactosa 35%, proteínas
30%), pues le calma un poco el dolor el
ardor que siente por la gastritis. Incluso,
cuando puede, se toma dos vasos de agua
fría para calmar las molestias. Ha decidido ir
al médico para tratarse pues ya no soporta
el dolor, el cual está seguro que los
síntomas se deben a una elevada
producción de ácido clorhídrico en el
estómago, y por ello le ha recetado
Ranitidina (antihistamínico), con lo que
siente mejoría.
Si se usara atropina en esta paciente, se
esperaría que disminuya la liberación de:

Respuesta
seleccionada: Bombesina

Respuestas: GIP

Bombesina

Secretina

Enzimas pancreáticas


Pregunta 3

● 0 de 3,3333 puntos


Estudiante de medicina de la UPC de 21 años
sufre de gastritis aguda ocasionada por
comer en lugares poco higiénicos. Suele
consumir caramelos ( chupar ) mientras está
en clase hasta la tarde. Toma gaseosas
regularmente (carbohidratos 46%, sodio
53%). También toma regular cantidad de
leche (grasa 35%, lactosa 35%, proteínas
30%), pues le calma un poco el dolor el
ardor que siente por la gastritis. Incluso,
cuando puede, se toma dos vasos de agua
fría para calmar las molestias. Ha decidido ir
al médico para tratarse pues ya no soporta
el dolor, el cual está seguro que los
síntomas se deben a una elevada
producción de ácido clorhídrico en el
estómago, y por ello le ha recetado
Ranitidina (antihistamínico), con lo que
siente mejoría.
El consumir caramelos eleva los niveles en
sangre de una hormona cuya función es
la estimulación de las células:

Respuesta
seleccionada: K del duodeno

Respuestas: Alfa del páncreas

K del duodeno

Beta del páncreas

G del antro


Pregunta 4

● 0 de 3,3333 puntos


Estudiante de medicina de la UPC de 21 años
sufre de gastritis aguda ocasionada por
comer en lugares poco higiénicos. Suele
consumir caramelos ( chupar ) mientras está
en clase hasta la tarde. Toma gaseosas
regularmente (carbohidratos 46%, sodio
53%). También toma regular cantidad de
leche (grasa 35%, lactosa 35%, proteínas
30%), pues le calma un poco el dolor el
ardor que siente por la gastritis. Incluso,
cuando puede, se toma dos vasos de agua
fría para calmar las molestias. Ha decidido ir
al médico para tratarse pues ya no soporta
el dolor, el cual está seguro que los
síntomas se deben a una elevada
producción de ácido clorhídrico en el
estómago, y por ello le ha recetado
Ranitidina (antihistamínico), con lo que
siente mejoría.
En este paciente con gastritis aguda debida a
una alta producción de ácido clorhídrico,
si se le hiciera un examen de sangre, se
encontraría elevados los niveles de:

Respuesta
seleccio Histamina
nada:

Respuestas: Sustancia P

Colecistoquinina

Péptido insulinotrópico
dependiente de la glucosa
(GIP)

Histamina


Pregunta 5

● 3,3333 de 3,3333 puntos


Estudiante de medicina de la UPC de 21 años
sufre de gastritis aguda ocasionada por
comer en lugares poco higiénicos. Suele
consumir caramelos ( chupar ) mientras está
en clase hasta la tarde. Toma gaseosas
regularmente (carbohidratos 46%, sodio
53%). También toma regular cantidad de
leche (grasa 35%, lactosa 35%, proteínas
30%), pues le calma un poco el dolor el
ardor que siente por la gastritis. Incluso,
cuando puede, se toma dos vasos de agua
fría para calmar las molestias. Ha decidido ir
al médico para tratarse pues ya no soporta
el dolor, el cual está seguro que los
síntomas se deben a una elevada
producción de ácido clorhídrico en el
estómago, y por ello le ha recetado
Ranitidina (antihistamínico), con lo que
siente mejoría.
El consumo de una pequeña cantidad de
gaseosa aumentará directamente la
concentración sérica de cuál de las
siguientes hormonas:

Respuesta
seleccionad Péptido 1 similar al
a: glucagón (GLP-1)

Respuestas: Colecistoquinina (CCK)

Motilina

Secretina

Péptido 1 similar al
glucagón (GLP-1)


Pregunta 6

● 3,3333 de 3,3333 puntos


Estudiante de medicina de la UPC de 21 años
sufre de gastritis aguda ocasionada por
comer en lugares poco higiénicos. Suele
consumir caramelos ( chupar ) mientras está
en clase hasta la tarde. Toma gaseosas
regularmente (carbohidratos 46%, sodio
53%). También toma regular cantidad de
leche (grasa 35%, lactosa 35%, proteínas
30%), pues le calma un poco el dolor el
ardor que siente por la gastritis. Incluso,
cuando puede, se toma dos vasos de agua
fría para calmar las molestias. Ha decidido ir
al médico para tratarse pues ya no soporta
el dolor, el cual está seguro que los
síntomas se deben a una elevada
producción de ácido clorhídrico en el
estómago, y por ello le ha recetado
Ranitidina (antihistamínico), con lo que
siente mejoría.
El consumo rápido de 500 mL de gaseosa
aumentará directamente la concentración
sérica de cuál de las siguientes
hormonas:

Respuesta
seleccionada: Gastrina

Respuestas: Secretina

Colecistoquinina
(CCK)

Gastrina

Neuropéptido Y


Cuando el tubo digestivo está en reposo, los complejos
mioeléctricos migratorios son desencadenados por:

Respuesta
seleccionad la serotonina
a:

Respuestas: la sustancia P
la serotonina

la motilina

el péptido intestinal
vasoactivo (VIP)


Pregunta 2

● 1 de 1 puntos

El orificio omental, o hiato de Winslow, se


encuentra limitado por el ligamento:

Respuesta seleccionada:
hepatoduodenal

Respuestas:
hepatoduodenal

gastroduodenal

gastroeesplénico

esplenorrenal


Pregunta 3

● 1 de 1 puntos


En un paciente de 43 años con tumor carcinoide
de páncreas productor de gastrina
(Síndrome de Zollinger-Ellison) se puede
esperar encontrar una potenciación del
reflejo:

Respuesta
seleccionada: gastrocólico

Respuestas: del vómito

de relajación receptiva

gastrocólico

ileocólico


Pregunta 4

● 1 de 1 puntos

La mucosa irrigada por la arteria palatina mayor


está recubierta por un epitelio:

Respuesta
seleccionada: plano escamoso
queratinizado

Respuestas:
plano escamoso
queratinizado

plano simple

cilíndrico simple no ciliado

plano escamoso no
queratinizado

Pregunta 5

● 1 de 1 puntos

En un paciente con enfermedad de Hirschprung


(megacolon agangliónico) con dilatación de
los dos tercios proximales del recto, la
dilatación de este segmento se ha producido
con mayor probabilidad debido a los
movimientos:

Respuesta seleccionada:
en masa

Respuestas: peristálticos

en masa

de retropulsión

segmentarios


Pregunta 6

● 1 de 1 puntos

En el sistema digestivo la liberación hormonal se


presenta ante diversos factores o estímulos.
La hormona ____________ es estimulada
por la presencia de alimentos en el bulbo
duodenal a predominio de ácidos grasos y
triglicéridos, por estimulación vagal y por la
hormona secretina.
Respuesta
seleccionada: colecistocinina (CCK)

Respuestas: lipasa pancreática

gastrina

colecistocinina (CCK)

motilina


Pregunta 7

● 1 de 1 puntos

¿Cuál de las siguientes estructuras se la conoce


como el vigilante del abdomen , por su
capacidad de desplazarse y adherirse a
cualquier zona inflamada o envolver el
órgano para frenar la inflamación?

Respuesta
seleccionada: Omento mayor

Respuestas: Omento menor

Omento mayor

Mesogastrio anterior

Intestino delgado


Pregunta 8
● 1 de 1 puntos

¿Cuál de las siguientes sustancias disminuye la


fuerza de las contracciones de
segmentación?

Respuesta
seleccionada: Atropina

Respuestas:
Atropina

Acetilcolina

Colecistocinina (CCK)

Insulina


Pregunta 9

● 1 de 1 puntos

Durante la deglución, inmediatamente luego que


el bolo alimenticio pasa por el esfínter
esofágico superior, se espera que la presión
intraluminal:

Respuesta
seleccionad disminuya en el cardias
a:

Respuestas: aumente en el tercio medio


del esófago

disminuya en el tercio medio


del esófago
aumente en la porción distal
al bolo

disminuya en el cardias


Pregunta 10

● 1 de 1 puntos

Durante el desarrollo del tubo digestivo en la


vida intrauterina, cuando el asa intestinal
rota 90° en el sentido de las manecillas del
reloj entonces:

Respuesta
selecciona el colon pasa por detrás del
da: duodeno

Respuestas: se forma el colon izquierdo

el colon pasa por detrás del


duodeno

se realiza el plegamiento
normal de los intestinos

se desarrolla un situs inversus


Pregunta 11

● 1 de 1 puntos


La irritación del peritoneo intestinal, como la que
sucede en una peritonitis, producirá:

Respuesta
seleccion la inhibición del peristaltismo
ada:

Respuestas: la estimulación del


peristaltismo

la inhibición del peristaltismo

la destrucción de las células


intersticiales de Cajal

el aumento de las
contracciones tónicas


Pregunta 12

● 1 de 1 puntos

Si al intubar a un paciente, por error se ingresa


el tubo endotraqueal en el esófago y se
insufla el manguito endotraqueal (globo
TET), la dilatación de este manguito
generará:

Respuesta
seleccio múltiples ondas secundarias
nada:

Respuestas: múltiples ondas peristálticas


primarias

múltiples ondas secundarias

una onda peristáltica primaria


una onda peristáltica secundaria
seguida de una terciaria


Pregunta 13

● 1 de 1 puntos

Paciente de 24 años acude a consulta externa


por presentar una fístula oronasal
(comunicación entre la cavidad oral y la
cavidad nasal). Está fístula es una
consecuencia tardía de la lesión de un vaso
sanguíneo por el antecedente de haber sido
operado de paladar hendido en los primeros
años de vida, aparentemente en una
campaña gratuita de corrección de paladar
fisurado. ¿Cuál de las arterias palatinas
podría haberse lesionado durante esa
cirugía?

Respuesta
seleccionad Mayor
a:

Respuestas:
Mayor

Ascendente

Menor

Rama palatina de la faríngea


ascendente

Pregunta 14

● 0 de 1 puntos

El omento mayor deriva del mesenterio


__________ y se inserta en el ___________

Respuesta
seleccionada: medio / colon

Respuestas: ventral / duodeno

dorsal / estómago

ventral / estómago

medio / colon


Pregunta 15

● 0 de 1 puntos

Para realizar el movimiento mecánico de abrir la


boca, primero se necesita:
Respuesta
selecciona la contracción de los
da: músculos buccinadores

Respuestas:
fijar el hueso hioides

la retropulsión de la lengua

elevar la laringe

la contracción de los
músculos buccinadores


Pregunta 16

● 0 de 1 puntos

El mesocolon transverso se origina en:

Respuesta
seleccionada: la curvatura mayor del
estómago

Respuestas: el colon transverso

la curvatura mayor del


estómago

la curvatura menor del


estómago

la pared posterior del


abdomen

Pregunta 17

● 0 de 1 puntos

Paciente con insuficiencia mitral moderada a


severa, con aumento de volumen de la
aurícula izquierda; esta condición tendrá
como consecuencia a nivel del sistema
digestivo:

Respuesta
selecciona la acalasia
da:

Respuestas:
la disfagia a sólidos

alteraciones de la fase
faríngea de la deglución

la acalasia

la estenosis hipertrófica del


cardias


Pregunta 18

● 0 de 1 puntos

¿Cuál de las siguientes alternativas se define


como la herniación de las vísceras
abdominales por un anillo umbilical
agrandado?
Respuesta seleccionada:
Gastrosquisis

Respuestas: Hernia femoral

Onfalocele

Hernia inguinal

Gastrosquisis


Pregunta 19

● 1 de 1 puntos

Durante el desarrollo intrauterino del páncreas,


cuando la porción izquierda de la yema
pancreática ventral migra en dirección
opuesta, se forma el páncreas:

Respuesta seleccionada:
anular

Respuestas: ectópico

anular

divisum

derecho


Pregunta 20
● 1 de 1 puntos

Al deglutir algo, durante la fase esofágica, las


ondas lentas del esófago se:

Respuesta
seleccionada: hacen menos negativas

Respuestas: hiperpolarizan

hacen menos negativas

asocian a menor actividad


motora

estabilizan
ECU 1
Estudiante de medicina de la UPC de 21 años sufre de gastritis aguda ocasionada por comer
en lugares poco higiénicos. Suele consumir caramelos ( chupar ) mientras está en clase hasta
la tarde. Toma gaseosas regularmente (carbohidratos 46%, sodio 53%). También toma
regular cantidad de leche (grasa 35%, lactosa 35%, proteínas 30%), pues le calma un poco el
dolor el ardor que siente por la gastritis. Incluso, cuando puede, se toma dos vasos de agua
fría para calmar las molestias. Ha decidido ir al médico para tratarse pues ya no soporta el
dolor, el cual está seguro que los síntomas se deben a una elevada producción de ácido
clorhídrico en el estómago, y por ello le ha recetado Ranitidina (antihistamínico), con lo que
siente mejoría.

El uso de atropina en este paciente:


- Aumentará el pH del estómago

Entre las sustancias cerebrales que producen ansiedad está la serotonina, la cual también
tiene acción:
- Anorexigénica

El consumo de dos vasos de agua seguidos agua generará indirectamente un aumento en la


liberación de:
- Ácido clorhídrico

En este paciente con gastritis aguda debida a una alta producción de ácido clorhídrico, sería
lógico esperar que el píloro tenga un tono muscular:
- Aumentado

El consumo de leche produce directamente un aumento de los niveles séricos de la


hormona:
- Colecistoquinina (CCK)

El consumo de leche produce directamente un aumento de los niveles séricos de la


hormona:
- Gastrina

ECU 2
Niño de sexo masculino de 2 años de edad, sufre de estreñimiento desde el nacimiento (1
deposición cada 3-4 días). Madre menciona que le estimula la defecación con un
termómetro rectal, y continuo uso de enemas y laxantes. Desde hace 6 meses comienza con
vómitos postprandiales. Los síntomas aumentan en frecuencia y magnitud y están en
relación con los episodios de estreñimiento. No refiere fiebre, tos, diarrea ni lesiones
cutáneas. Al examen físico presenta regular estado general, luce deshidratado. Abdomen
distendido, blando, depresible e indoloro. No se palpan masas abdominales. Se permeabiliza
el canal anal con termómetro rectal, encontrando cierta resistencia. Salida de material fecal
mal oliente en regular cantidad. Exámenes de laboratorio: hemograma normal. Signos
inflamatorios de fase aguda negativos. Alcalosis metabólica leve en sangre venosa.
Radiografía con enema baritado muestra recto y colon sigmoides dilatados (megacolon).
Biopsia profunda: ausencia de células ganglionares en la muestra enviada. Se realiza cirugía
correctiva.

El contenido fecal se detiene en la zona inmediatamente proximal a la zona donde hay una
menor presencia de:
- Péptido intestinal vasoactivo

En cuanto a los reflejos gastrocólico y gastroduodenal en este paciente, indique lo correcto:


- Se pueden considerar reflejos vago-vagales

En este paciente se considera que está abolido el reflejo:


- Rectoesfinteriano

Debido al acúmulo de material fecal en todo el marco colónico, y a la irritación química


asociada, el peristaltismo del íleon distal se debe encontrar:
- Inhibido

Es un reflejo propio de la pared intestinal:


- Peristaltismo

A diferencia de las arcadas, los vómitos presentan apertura de:


- Esfínter esofágico superior

CI 1
Sustancia que inhibe la secreción y la motilidad del estómago prolongando el tiempo de
digestión:
- Péptido insulinotrópico dependiente de la glucosa (GIP)

Marque lo correcto:
- La hernia fisiológica se produce en la sexta semana y es la salida temporal de
asas intestinales a través del cordón umbilical

Marque la respuesta correcta en relación a la gastrina:


- Las células G son las productoras y se encuentran principalmente en el antro
gástrico
El consumir caramelos indirectamente activa la vía:
- POMC/CART

¿En qué capa se encuentra la alteración principal en el Hirschsprung o megacolon


agangliónico?:
- Muscular propia

Con respecto a las ondas lentas, marque la afirmación correcta:


- Son contracciones rítmicas espontáneas

El uso de Ranitidina bloquea el receptor H2 de la histamina en las células parietales. La


histamina llega a estas células por:
- Difusión

La triada sintomática: vómitos explosivos post-prandiales, movimientos peristálticos


epigástricos visibles de izquierda a derecha y nódulo palpable epigástrico subcostal derecho,
pertenecen a:
- Estenosis congénita hipertrófica del píloro.

Durante una cirugía oncológica, ¿la extirpación de cuál de los siguientes órganos se vería
comprometida por la presencia de adventicia?:
- Recto

En cuanto a los reflejos gastrointestinales, un reflejo que estimula el tránsito intestinal es el


reflejo:
- Gastrocólico

El ligamento falciforme divide al hígado en dos lóbulos derecho e izquierdo.


Embriológicamente deriva del:
- Mesenterio ventral

La presencia de atresias y estenosis duodenales se deben básicamente a una:


- Falta de recanalización

Estudiante de medicina de 20 años, se ha amanecido estudiando para su examen de Sistema


Digestivo. No ha probado alimento desde la cena, por lo que se puede afirmar que la
motilidad de esta persona está siendo regulada por:
- Motilina

Paciente con disminución del apetito marcada asociada a cáncer terminal, para promover la
ingesta de alimentos se podría usar análogos de:
- Endorfinas

Las ondas lentas se producen por la apertura cíclica de canales de:


- Calcio

La forma más común de atresia esofágica contiene:


- Estenosis proximal del esófago más fístula traqueoesofágica distal

Al deglutir un bolo alimenticio, es lógico suponer que al pasar por el esófago haya un mayor
consumo de oxígeno en la pared del tercio:
- Proximal

Paciente que come entera una pizza familiar de chorizo y queso. Es posible esperar que
debido a la cantidad de alimento ingerida, las ondas lentas hayan:
- Sufrido ninguna alteración en su frecuencia

La hernia fisiológica se produce dentro de:


- Cordón umbilical

El crecimiento de un adenocarcinoma de páncreas compromete la pared gástrica por


contigüidad. ¿Qué parte del estómago se esperaría esté comprometido?
- Pared posterior del antro

CI 2
Estimula la producción de saliva:
- Vasodilatación periglandular

Durante la secreción de saliva, es de esperarse que las concentraciones de ________ y


______ disminuyan al disminuir el flujo:
- Sodio Bicarbonato

Con respecto a la secreción gástrica de HCl:


- a mayor secreción de HCl en el lumen gástrico, mayor pH en la sangre venosa
gástrica

Respecto a las enfermedades del esófago, marque lo correcto:


- el diagnóstico diferencial de la acalasia es la enfermedad de Chagas esofágica

Con respecto a las lesiones y enfermedades de la boca, marque lo correcto:


- la eritroplasia debe ser biopsiada
Respecto a las glándulas salivales, marque lo incorrecto:
- la glándula sublingual tiene forma de garfio

El omeprazol actúa sobre la membrana _____________ de la célula ____________


- apical / parietal

Durante el sueño, la concentración de bicarbonato en la saliva:


- Disminuye

Durante el ataque con gas sarín (bloqueador de la acetilcolinesterasa) en el metro de Tokio,


en 1995, el personal de salud notó que los pacientes afectados presentaban:
- Hipersalivación

La célula mucosa del cuello gástrico produce:


- Moco

ECU 3
El caso:
Paciente de 54 años con antecedentes de alcoholismo, gastritis crónica, tabaquismo pesado,
obesidad, cálculos biliares y cirrosis, es llevado a la emergencia por dolor abdominal en
epigastrio irradiado a la espalda y trastorno del sensorio.
Al examen físico: presión arterial 85/50 mmHg, frecuencia cardíaca 100 latidos/min,
frecuencia respiratoria 18 x minuto, temperatura axilar 36°C.
Conjuntivas pálidas, escleras ictéricas nevus arácnidos en tronco, distensión abdominal
marcada, cabeza de medusa, matidez desplazable en ambos flancos e hipogastrio, dolor a la
palpación de abdomen.
Tiempo de protrombina: 24 seg (testigo: 13 seg); TPT: 38 seg, glicemia: 165 mg/dL, uremia:
20 mg/dL, ASAT: 76 UI/L, ALAT: 22 UI/L, albumina: 2,5 g/dL, bilirrubina total: 2,6 mg/dL,
bilirrubina directa: 1,4 mg/dL, amilasa sérica 4000 U/L.

En todas las preguntas marque la mejor respuesta.


De las siguientes sustancias secretadas por los órganos de este paciente, la más alcalina es la
secreción:
- Pancreática

En esta paciente, el aumento de la Amilasa sérica se debe directamente a una lesión de


- Páncreas

En cuanto a la gastritis de este paciente, se encontró que era producida por la bacteria
Helicobacter pylori. Esta bacteria sobrevive en el medio ácido del estómago gracias a:
- Ureasa
Para evaluar la función hepática podemos medir los niveles séricos de:
- Tiempo de protrombina

El misoprostol, análogo de las prostaglandinas, está mejor indicado en:


- Prevenir daño por AINES

El paciente llegó con trastorno del sensorio posiblemente debida a encefalopatía hepática
producida por déficit en el metabolismo del/la:
- Amoniaco

CI 3
En cuanto al urobilinógeno, se puede decir que es cierto:
- Recircula hacia el hígado

Marque lo correcto:
- Las venas sublobulillares desembocan en las venas hepáticas

En cuanto a la secreción de iones y agua en los conductos biliares es correcto lo siguiente:


- Es estimulada por la secretina

Paciente con tumor neuroendocrino productor de secretina, debido a lo cual se puede


esperar que su secreción pancreática, comparada con la de una persona sana en estado de
bajo flujo, tenga una concentración de:
- bicarbonato aumentada

El efecto colerético de las sales biliares se refiere a:


- Estimulo de secreción biliar

Paciente de 42 años con adenocarcinoma ductular. La TC ha demostrado claramente que el


tumor está en el cuello del páncreas y que hay un gran vaso ocluido. ¿Cuál de los siguientes
vasos estaría más probablemente obstruido?
- Vena porta

Las ramas más pequeñas del árbol biliar son:


- Canalículos biliares

La secreción de agua y bicarbonato por el páncreas exocrino se da básicamente en la fase:


- Intestinal
Cuando el alimento se encuentra en el estómago, se produce la liberación de enzimas
pancreáticas básicamente debido a la acción de:
- Vago

La bilirrubina directa aumentada en cirrosis hepática se excreta en la orina debido a:


- Ser hidrosoluble

ECU 4
Caso:
Mujer de 83 años acude a emergencia por dolor abdominal desde hace 4 días, localizado en
epigastrio, irradiado a ambos hipocondrios, nauseas, vómitos y distensión abdominal; tiene
antecedente de cardiopatía hipertensiva, diabetes mellitus tipo II y fibrilación auricular.
Refiere deposiciones diarreicas muco sanguinolentas hace 1 día. Los exámenes iniciales
muestran PA: 110/60 mmHg, FC: 110/mn, leucocitos: 17800, neutrófilos de 93%. TAC
abdómino pélvica se observa oclusión completa de arteria mesentérica superior por trombo
asociado a placa ateromatosa.

Considerando que se ha comprometido el íleon distal, entre otras áreas, la atrofia o


descamación del epitelio de superficie explicaría cuál de los signos o síntomas de la paciente
(marque la mejor respuesta):
- Diarreas mucosanguinolentas

Después de remover quirúrgicamente el territorio gastrointestinal afectado, estaremos


seguros que no quedará en la paciente la siguiente estructura:
- Placas de Peyer

El mecanismo de la diarrea muco sanguinolenta que presenta la paciente, puede mejor


definido como de tipo (marque la mejor respuesta):
- Exudativa

Durante la cirugía, el cirujano observó que además la paciente tenía divertículos en el sigma.
Se sabe que estos divertículos:
- Se pueden asociar a estreñimiento crónico

En esta paciente, ¿cuál de las siguientes sustancias no tendrá una considerable disminución
en su absorción? (marque la mejor respuesta):
- Calcio

PARCIAL
Al evaluar la orofaringe de un paciente, el médico le solicita que abra la boca, saque la
lengua y diga a . Al hacer esta maniobra, nota que el paladar se desvía hacia la derecha, lo
cual le hace sospechar que el paciente sufre de una lesión del nervio craneal:
- X contralateral

Un bolo alimenticio grande y poco masticado se atasca en el esófago, esto ocasiona una
sensación de dolor que es transmitida por los nervios:
- esplácnicos

Para realizar el movimiento mecánico de abrir la boca, primero se necesita:


- fijar el hueso hioides

¿Cuál de las siguientes alternativas se define como la protrusión directa del contenido
abdominal a la cavidad amniótica por un defecto de la pared corporal?
- Gastrosquisis

Un paciente requiere que se le coloque una sonda de alimentación directamente al


estómago (gastrostomía), el cirujano deberá hacer una incisión en la piel del abdomen ¿cuál
de las siguientes raíces nerviosas debe ser anestesiada para este procedimiento?
- T8

En un paciente de 43 años con tumor carcinoide de páncreas productor de gastrina


(Síndrome de Zollinger-Ellison) se puede esperar encontrar una potenciación del reflejo:
- gastrocólico

El mecanismo de la defecación incluye la participación de diversas estructuras ¿Cuál de las


siguientes alternativas es correcta?
- Puede ser mediado por un reflejo intrínseco

Cuando el contenido del estómago ingresa al duodeno, uno de los reflejos que inhiben el
vaciamiento gástrico es a través del:
- sistema nervioso mientérico

Durante la masticación, gran parte del proceso masticatorio se debe a:


- el reflejo masticatorio

Las glándulas salivales tienes conductos para la excreción de la saliva; las glándulas
____________ drenan en las carúnculas sublinguales.
- sublinguales
Los diferentes segmentos del tubo digestivo son susceptibles de reflejos y movimientos
según su contenido. Si colocáramos mediante una sonda un bolo alimenticio directamente
en el tercio medio del esófago:
- se producirá ondas secundarias

En una persona si enfermedad se espera que el tránsito intestinal se vea disminuido cuando
se presenta el reflejo:
- doloroso

El divertículo de Meckel es una anomalía congénita que ocurre por la persistencia del
conducto vitelino y da origen a una estructura sacular, el cual se encuentra en el:
- borde antimesentérico

Si al intubar a un paciente, por error se ingresa el tubo endotraqueal en el esófago y se


insufla el manguito endotraqueal (globo TET), la dilatación de este manguito generará:
- múltiples ondas secundarias

El orificio omental, o hiato de Winslow, se encuentra limitado por el ligamento:


- hepatoduodenal

Paciente de 24 años acude a consulta externa por presentar una fístula oronasal
(comunicación entre la cavidad oral y la cavidad nasal). Está fístula es una consecuencia
tardía de la lesión de un vaso sanguíneo por el antecedente de haber sido operado de
paladar hendido en los primeros años de vida, aparentemente en una campaña gratuita de
corrección de paladar fisurado. ¿Cuál de las arterias palatinas podría haberse lesionado
durante esa cirugía?
- Mayor

Dentro de las anomalías congénitas se puede presentar un tejido pancreático accesorio


¿Cuál es la ubicación más común de este tejido?
- Estómago

Paciente con insuficiencia mitral moderada a severa, con aumento de volumen de la


aurícula izquierda; esta condición tendrá como consecuencia a nivel del sistema digestivo:
- la disfagia a sólidos

El inicio de la fase faríngea de la deglución se debe a estímulos sensitivos que viajan por el
nervio craneal:
- V

El mesocolon transverso se origina en:


- la pared posterior del abdomen

En la digestión de los alimentos, la hormona __________ se libera frente a la presencia de


péptidos y monoglicéridos, y tiene un efecto marcado en la disminución del vaciamiento
gástrico.
- colecistoquinina

En un varón de 47 años con sección medular a nivel de T6 debido a un accidente


automovilístico, sus terapeutas han desarrollado un mecanismo para distender el recto e
iniciar el reflejo rectoesfinteriano, lo cual producirá la contracción de:
- la pared del recto

Los catadores de vino tienen una habilidad increíble al momento de separar los sabores.
Este aumento de la sensibilidad gustativa debido a una mayor cantidad de papilas linguales
y de corpúsculos gustativos se conoce como:
- hipergeusia

Una recién nacida es evaluada por el neonatólogo y evidencia que el canal anal está
completamente cerrado. Este problema se debe probablemente a una anomalía en el
desarrollo de:
- la membrana cloacal

Durante un experimento, se insufla rápidamente dos litros de agua en un globo colocado


dentro del estómago de un voluntario. ¿cuál de las siguientes situaciones del músculo liso
será consecuencia directa de este cambio de volumen en el estómago?
- Despolarización

En muchos países se usa el suplemento de fluor en el agua potable o los dentríficos, con el
fin de hacer el esmalte más resistente a la desmineralización inducida directamente por:
- el ácido

La sensación del gusto depende de la presencia de papilas gustativas en la lengua, las cuales
tienen corpúsculos gustativos conteniendo células neuroepiteliales sensoriales. Estas células
neuroepiteliales pueden ser dañadas fácilmente, por suerte, su tiempo de recambio es de
alrededor de:
- 10 días

Paciente de 56 años con accidente cerebrovascular reciente. En la resonancia se observa


daño de los núcleos laterales del hipotálamo. Por este motivo es muy probable que el
paciente sufra de:
- Inanición
Durante el desarrollo de la región cloacal, una cuña de mesodermo ubicado entre el
alantoides y el intestino posterior vendrá a formar el:
- tabique urorrectal

En el conducto anal se encuentra la unión entre las regiones del endodermo y el ectodermo,
esta unión se evidencia al observar:
- la línea pectínea

En una persona sana, el momento adecuado para encontrar los mayores niveles de grelina
en sangre sería:
- antes de come

Dentro de la estructura de los dientes, la parte del diente cubierta por esmalte y que se
puede ver mediante la inspección visual de la boca se denomina
- corona clínica

El esófago en su microestructura tiene básicamente adventicia, a excepción de la región


distal, donde tiene serosa, específicamente a partir del nivel de:
- T10

En un paciente con arcadas, se debe considerar que durante la ocurrencia de dichas arcadas,
debemos encontrar contenido gástrico en:
- tórax

El duodeno está constituido por el segmento terminal del intestino anterior y el segmento
proximal del intestino medio ¿Cuál de las siguientes alternativas describe mejor este lugar
de unión entre los dos intestinos?
- Distal al origen de la yema hepática

En una persona sana, el uso de atropina producirá a nivel del estómago:


- Aumentará el pH del estómago

En una persona sana, el consumo de leche produce indirectamente


- Inhibición del vaciamiento gástrico

La motilidad del colon es importante y lenta comparada con la del intestino delgado. Los
movimientos en masa ocasionan la:
- distensión rectal
Con respecto a la motilidad gástrica, los potenciales de acción disminuyen en frecuencia por
efecto de:
- el péptido insulinotrópico dependiente de glucosa

La sensación del gusto depende de la presencia de papilas gustativas en la lengua, de las


cuales, algunas de ellas tienen un surco terminal por donde drenan unas glándulas salivales
linguales (llamadas glándulas de von Ebner). Esta descripción se refiere a las papilas:
- circunvaladas
1. Un niño de 2 años es llevado a la consulta por diarrea persistente y edema de
las extremidades, además falta de crecimiento y desarrollo en relación a su
edad. Los análisis de sangre revelan que tiene concentración plasmática baja de
proteínas (hipoproteinemia). Durante la endoscopía duodenal, se coloca
colecistocinina (CCK) endovenosa y se recoge muestras del líquido duodenal; el
resultado del líquido confirma incapacidad para hidrolizar proteínas a un pH
neutro, esta situación mejora al añadir una pequeña cantidad de tripsina. El
paciente probablemente esté sufriendo la falta congénita de
-Enterocinasa
2. Experimentalmente se incrementa la velocidad de la secreción salival con
una sustancia, el análisis de la composición de esta saliva obtenida se
espera encontrar…………..
-Disminución de concentración de potasio
3. Paciente varón de 46 años soltero, consulta por odinofagia y bajo de peso, tiene
antecedente de tuberculosis desde hace 3 meses y es fumador crónico (10
cigarrillos por día); al evaluar la cavidad oral se identifica lesión blanquecina en
el dorso de la lengua y paladar blando, las lesiones se desprenden con el baja
lengua dejando una base eritematosa. Esta lesión corresponde probablemente
a
……………………….
…..
-Candidiasis oral
4. Paciente mujer de 35 años acude a consulta por sensación de sequedad y
lesiones en cavidad oral. Al examen se observa atrofia de la mucosa, fisuras y
úlceras; nota además sequedad e irritación de la córnea y aumento del tamaño
de las glándulas parotídeas. Su diagnóstico más probable es artritis reumatoide;
el hallazgo más probable en una biopsia de glándula parótida es……..….
-Gran infiltración de linfocitos y células plasmáticas
5. Un paciente con anemia acude con su médico quejándose de episodios
frecuentes de gastroenteritis. Un análisis de sangre revela anticuerpos
circulantes dirigidos contra células parietales gástricas. Su anemia es atribuible
a la hiposecreción de
-Factor intrínseco
6. Dos estudiantes deciden tomar un receso para comer una hamburguesa a la
hora del almuerzo. Antes de llegar a la cafetería, impulsos nerviosos
provenientes del complejo vagal dorsal iniciarán la secreción de ácido gástrico
por la liberación de
…………………….. desde el sistema nervioso entérico.
-GRP
7. Un niño de cuatro años de edad es llevado a la consulta por cuadros diarreicos
frecuentes caracterizados por heces pálidas, voluminosas y fétidas, presenta
bajo peso y talla. Se mide la concentración de cloruro en el sudor y se encuentra
que sus valores son muy elevados. La alteración más importante a nivel de
células ductales del páncreas tiene relación directa con la conductancia
de…………
-Cloro
8. Una mujer de 50 años de edad que sufrió durante varios años resequedad de
los ojos debida a producción inadecuada de lágrimas es enviada con un
gastroenterólogo para evaluación de pirosis crónica. El examen endoscópico
revela erosiones y tejido cicatrizal en la parte distal del esófago justo por
arriba del esfínter esofágico inferior. Las lesiones pueden atribuirse a la
disminución de uno de los siguientes componentes salivales:
-Bicarbonato

9. Se evalúa los valores séricos de las siguientes sustancias a un paciente con


enfermedad hepática terminal; en este paciente se espera encontrar la
combinación con la letra …………
-disminuida, aumentada, disminuida
10. Una mujer de 35 años de edad HIV positiva, se presenta al médico con dolor
abdominal en cuadrante superior derecho e ictericia. La paciente refiere haber
tenido múltiples episodios de ictericia durante los últimos 10 años. Los exámenes
para determinar hepatitis viral, dieron positivos para Hepatitis B, siendo
catalogado el caso como hepatitis crónica con alteración funcional. En un
examen de sangre ¿cuál de los siguientes parámetros está disminuido?
-Albúmina
11. En el reflejo peristáltico del intestino delgado, uno de los siguientes eventos
sucede en la porción oral del bolo alimenticio…………...
-Acción de acetilcolina en el músculo circular
12. Experimentalmente se coloca una dosis alta de secretina en la luz
intestinal duodenal; como consecuencia de esto, en el jugo pancreático de
la misma luz intestinal se observa la disminución de la concentración de
…..………..
-Cl
13. Un varón de 58 años de edad con enfermedad de Crohn severo fue sometido a
una resección ileal. Después de la cirugía este paciente padecerá de esteatorrea,
esto se explica porque …..………..
- La micelas no pueden formarse
14. En un experimento se inserta un balón en el estómago de un voluntario, se infla
poco a poco mientras que se vigilan las presiones intraluminales. Aunque el
volumen del balón aumenta considerablemente, las presiones permanecen
constantes. Esta relación volumen-presión se explica por la liberación local de
…………..
-Óxido nítrico y péptido inhibidor vasoactivo
15. La toxina de Vibrio cholerae causa diarrea debido a…….
-El Incremento de la secreción de cloro por las células de la cripta intestinal
16. ¿Cuál de las siguientes alternativas es una característica de la secreción
exocrina del páncreas?
-Tiene una baja concentración de Cl- respecto al plasma
17. Una madre lleva a su hijo de dos años de edad a la sala de urgencias, estresada
porque el niño deglutió una moneda de 10 céntimos mientras la familia cenaba en
un restaurante. El médico observa mediante fluoroscopía que la moneda se halla
en el estómago y asegura a la madre que la moneda se eliminará con las heces.
El médico recomienda utilizar la respuesta fisiológica que permitirá la evacuación
de la moneda del estómago al intestino ………….…..
-Son los movimientos de mezcla y trituración
-. Es provocada por el ayuno
18. Las estructuras en el hígado que permite que los productos metabólicos unidos a
proteínas tengan acceso a las membranas basolaterales de los hepatocitos,
son….. -Las fenestras sinusoidales
19. La composición de la bilis es modificada conforme fluye por los conductillos
biliares. Durante este tránsito se espera que aumente la concentración de…….
-Monómeros de ácido biliar
-Ig A
20. Se mide experimentalmente el contenido gástrico de dos personas. La persona
“A” tiene alto contenido de grasa y la persona “B” tiene un contenido hipertónico
¿Cuál de las siguientes es correcto respecto al vaciamiento gástrico?
- Hay ralentización del vaciado gástrico en ambos casos
21. El examen endoscópico de un paciente con hipertensión portal grave revela
venas tortuosas que sobresalen hacia la luz del esófago. El paciente recibe
tratamiento quirúrgico mediante la colocación de una derivación que conecta la
vena porta a la vena cava. Después de la operación el riesgo de encefalopatía y
el
riesgo de sangrado de várices ……………..
-Aumentará/disminuirá
22. Un paciente varón de 18 años de edad acude al médico para sus exámenes
de rutina. Sus resultados de laboratorio muestran un valor de bilirrubina
sérica de 4 mg/dl y una bilirrubina directa de 0,3 mg/dl. Las pruebas de
función hepática son normales. La alteración que explica mejor este caso es
por la deficiencia de
………………..
-Glucuronil transferasa
23. Un hombre de 57 años de edad es llevado a urgencias con hematemesis
masiva rojo brillante, a su llegada se halla inconciente con PA: 80/40 mm Hg y
FC: 124 lat/min. Luce ictérico con presencia de “arañas vasculares en el tórax
anterior y extremidades”, abdomen distendido con signo de oleada positiva.
Se encuentra esplenomegalia y pérdida de la masa muscular en
extremidades. La anastomosis vascular responsable del sangrado en este
paciente es
-Vena gástrica izquierda y vena ácigos
24. Un estudiante de medicina está comiendo un plato de comida a base de
champiñones, espárrago y salsa de soya. El sabor umami contenido en todos
estos alimentos actúa a nivel de los botones gustativos estimulando
………………..
-Un receptor acoplado a proteína G
25. Un hombre de 22 años de edad se presenta al médico con una historia de 1 año
de evolución caracterizado por dolor recurrente en fosa iliaca derecha y diarrea.
Manifiesta además pérdida de peso de 8 kg durante este periodo. La
colonoscopía revela múltiples lesiones en el ileon terminal y colon. La biopsia
de estas lesiones revela engrosamiento, inflamación y ulceración de la mucosa.
El diagnóstico más probable en este caso es…….
-Enfermedad de Crohn
26. Varón de 61 años que consulta por dolor retro esternal intenso desde hace 6
horas y después de vómitos intensos y repetidos; al examen se observa disnea,
cianosis, hipotensión y signos clínicos de shock. La radiografía simple de tórax
muestra neumomediastino. El líquido en el espacio pleural aspirado tiene alta
concentración de amilasa. ¿Cuál de las siguientes alternativas puede explicar
este cuadro clínico? -Rotura espontánea de esófago
27. La secreción del ácido en la célula parietal gástrica se lleva a cabo por una
ATPasa especifica que intercambia hidrogeniones (H+) del citosol por…..
-K +
28. En condiciones normales el ingreso de 600 ml de líquido es el estómago provoca
un aumento de presión intragástrica de unos 12 cm de H2O. Después de una
vagotomía (corte del nervio vago) es de esperar que el ingreso del mismo
volumen de líquido provoque lo siguiente: …………………………………
-Un aumento mayor de la presión
29. Una paciente de 30 años de edad es sometida a una cirugía en oído medio
derecho por un problema de otoesclerosis. Luego de la cirugía refiere alteración
en la percepción de sabores. Al evaluar el caso usted esperaría
encontrar……….
-Alteración en la sensación del gusto en los dos tercios anteriores de la
lengua
-Sensación del dolor, tacto y temperatura conservada en toda la lengua

30. ¿Cuál de las siguientes alternativas es correcta?


-Las sales biliares desconjugadas son absorbidas preferentemente en el
colon
31. En un paciente de 45 años de edad con colestasis biliar, se encuentra una
elevación de los niveles sanguíneos de fosfatasa alcalina hasta 3 veces la cifra
normal. ¿Cuál de las siguientes alternativas estará también elevada como
evidencia del daño de la vía biliar?
-Gamma glutamil transpeptidasa
32. Revisando la angiografía de un hombre de 70 años en estudio por aneurisma de
aorta abdominal el radiólogo informa de la presencia de una oclusión completa
de la arteria mesentérica inferior. El paciente se encuentra completamente
asintomático.
¿Cuál de las siguientes arterias se anastomosa a la sistema arterial
de la mesentérica inferior?
-Cólica media
33. Lactante de 3 meses de vida es atendido por presentar diarrea, se administra
una solución de glucosa y electrólitos por vía oral. La proteína de membrana
apical que explica la capacidad de esta solución para proporcionar aporte de
glucosa e hidratación es ………..
-SGLT-1
34. Paciente ha sufrido herida de bala en el abdomen, se le ha tenido que extirpar el
segmento medio y distal del ileon. En este caso la síntesis hepática de sales
biliares estará …..…..
-Incrementada por estímulo de la enzima colesterol 7 alfa hidroxilasa
35. Un varón de 75 años ingresa al consultorio por presentar ictericia marcada de
piel y las escleras. El estudio del paciente mostró que presentaba un tumor que
obstruía la totalidad del conducto hepático común. ¿Cuál de las siguientes
estructuras se encontrará dilatada en este paciente?
-Conductos de Hering
36. En un paciente con insuficiencia renal crónica, el déficit en la absorción de
calcio a nivel del enterocito se debe a lo siguiente:
-No se convierte la 25 hidroxicolecalciferol a 1,25 dihidroxicolecalciferol
37. Varón de 30 años es traído a emergencia por agresión abdominal con arma de
fuego (pistola) y es sometido a laparotomía exploratoria, observándose isquemia
del colon ascendente y parte del colon trasverso ¿la lesión de cuál de las
siguientes arterias explicaría esta isquemia?
d. Mesentérica superior
38. Respecto a las sustancias gastrointestinales que regulan la secreción pancreática;
marque la afirmación correcta:
b. La acetilcolina es capaz de estimular la secreción enzimática y de bicarbonato
del páncreas
39. Ante una lesión del X par craneal, ¿cuál de los siguientes músculos mantiene
conservada su función?:
b. Tensor del velo del paladar
40. Experimentalmente se utiliza atropina (anticolinérgico) para inhibir la secreción
de gastrina, sin embargo la secreción de esta hormona se sigue dando ante
estímulos vagales. Esta situación se explica porque la atropina:
d. No bloquea la acción del péptido GRP

41. Un varón de 50 años es sometido a extirpación del duodeno y parte proximal del
yeyuno. Esta situación ocasionaría la pérdida de las células ……….. ,
productoras de ………………… que estimula la secreción de bicarbonato por el
páncreas.
“S” / secretina
42. Se evalúa la expresión de la proteína Agrp en una persona con alteración del
apetito; lo correcto respecto a esta proteína es…..
La mutación del gen que la codifica produce adelgazamiento
43. Juana cae de la bicicleta y se fractura la región anterior del hueso maxilar superior
con compromiso de la fosa incisiva. Al examen físico de la región esperaría
encontrar alteración en la sensibilidad de la encía …………………
palatina anterior
44. Recién nacido es atendido por el neonatólogo y luego entregado a su madre para
dar de lactar; la madre al dar de lactar observa coloración azulada de labios,
acompañado de tos persistente, dificultad respiratoria y distención abdominal. Se
le intenta colocar una sonda nasogástrica pero esta retorna a la cavidad oral en
todos los intentos. ¿Cuál de las siguientes anomalías del desarrollo es el más
probable en este caso?
b. Atresia esofágica proximal con fístula traqueoesofágica distal l
45. ¿Cuál de los siguientes mecanismos ocurre durante la defecación?
En la posición de “cuclillas” el músculo puborrectal se halla relajado
46. Un paciente luego de un accidente sufre lesión del piso de la boca, se constata
daño del nervio “cuerda del tímpano”, en este caso se esperaría en
47. contrar disminución de la………………………….… de la lengua
Sensación del gusto en los dos tercios anteriores
48. ¿Cuál de las siguientes afirmaciones es la correcta sobre la gastrina?
Actúa en la célula diana mediante su receptor CCk tipo B
49. Al recibir un paciente con signos de hipovolemia y antecedente de trauma en
abdomen por accidente de tránsito, usted identifica radiológicamente: lesión de
primera vértebra lumbar y signos de lesión en páncreas; durante la cirugía se
observó pobre irrigación de asas intestinales. El vaso afectado es la arteria ……..
c. mesentérica inferior
50. Un paciente sufre de daño a nivel del cuello con lesión muscular en la región de la
faringe. En el examen físico se determina dificultad para la elevación de la faringe y
para el cierre del itsmo de las fauces. En este caso, probablemente esté afectado el
músculo:
c. palatofaringeo
51. Varón de 50 años a quien le realizan la curación de la segunda molar de la
arcada superior derecha. En un momento determinado, el paciente acusa de
intenso dolor de la pieza dentaria en tratamiento. La vía aferente del dolor
viaja a través del nervio …………
a. trigémino V2
52. La distención gástrica por los alimentos produce incremento de secreción de
HCl mediante la producción de ………….. que estimula a las célulasvía
proteína
………..
a. gastrina / parietal / Gq

53. Un niño de tres años llega a emergencia con disfagia (dificultad para tragar),
dolor retro esternal, salivación y llanto. Se sospecha de ingesta de cuerpo
extraño (moneda) en el esófago; al ser evaluado se constata en una radiografía
presencia de cuerpo extraño a nivel de C6 (6° vértebra cervical). El cuerpo
extraño estará suspendido a nivel del estrechamiento producido por………..
c. el músculo cricofaríngeo
54. La triada portal (arteria hepática, vena portal y conducto biliar común) está
contenida en el ligamento …….……… y derivan embriológicamente del ……
a. hepato duodenal / mesenterio ventral
55. Un paciente refiere no percibir algunos sabores, al examen físico se constata
alteración en la percepción de sabores y del dolor en el tercio posterior de la
lengua ¿Cuál de los siguientes nervios estará alterada en su función?
c. Glosofaríngeo (IX par)
56. En el caso de un paciente con gastrinoma (tumor productor de gastrina), la
presencia de úlceras duodenales y erosión de la mucosa gástrica, se debe
principalmente a…….
c. el exceso de HCl por estímulo de receptores CCK-B en la célula parietal
56. El reflejo entero gástrico se caracterizan por:
d. originarse debido a la distensión duodenal y presencia del quimo ácido
57. Mauricio tiene dificultad para deprimir el paladar y elevar la parte posterior de la
lengua. En este caso estará afectado un músculo, específicamente el músculo
…………….
b. extrínseco – palatogloso
58. En condiciones normales, el ingreso de 600 ml de líquido es el estómago provoca
un aumento de presión intragástrica de unos 12 cm de H2O. Después de una
vagotomía (corte del nervio vago) es de esperar que el ingreso del mismo volumen de
líquido ocasione
………………………………… de la presión
intragástrica.
c. un aumento mayor
59. La explicación fisiológica de presentar somnolencia de 30 minutos a 1 hora
después de ingerir alimentos, se explica por: a. Aumento del cloro intraluminal
e. Aumento de la alcalinidad sanguínea
60.Se presenta un paciente, el cual presenta un antecedente de tuberculosis
intestinal, por lo cual, se le resecó 80 cm de íleon distal. Desde el punto de vista
fisiológico, el paciente puede presentar una de las siguientes alteraciones: a.
Disminución de la secreción de Vitamina B12

e. Disminución de la absorción de ácido glicocólico


61. Un paciente es sometido experimentalmente a un fármaco que modifica el flujo
salival, obteniéndose un volumen de saliva de 288 ml en 6 horas. En este caso las
concentraciones de electrolitos y bicarbonato en la saliva obtenida varían de la
siguiente manera: a. ↑ Na+,
↓ K+, ↑ Cl-, ↑ HCO3-

1. b. ↓ Na+, ↓ Cl-, ↑ K+, ↓ HCO3-

62. Uno de los siguientes elementos debería hallarse con más probabilidad en el
esófago de un paciente que sufre de reflujo gastro esofágico…
a. Pepsina

63. Un paciente de 40 años cursa con anemia de 8g/dl, aqueja además de astenia y
sensación de hormigueo bilateral en los miembros inferiores, al examen se halla alteración
de la sensibilidad a la vibración y camina con ampliación de la base de sustentación. Uno
de los siguientes procedimientos sería de ayuda para el diagnóstico de este paciente:
a. Tomografía cerebral
b. Biopsia de la mucosa gástrica

64.Paciente de 60 años ingresa por caída hace 1 hora y pequeño hematoma en cuero
cabelludo, al examen físico ampliado se observa ictericia de piel y mucosas generalizada,
abdomen blando, se palpa estructura quística no dolorosa en hipocondrio derecho que
corresponde a vesícula biliar (signo de Courvoisier), en los exámenes de laboratorio se
halla niveles bajos en la formación de estercobilinógeno y urobilinógeno en heces,
incremento de la bilirrubina conjugada en la orina, elevación de fosfatasa alcalina y
gamma glutamil transpeptidasa séricas. El presente cuadro puede ser explicado por: a.
Reabsorción de hematoma
c.Carcinoma de la cabeza de páncreas
65. Un recién nacido presenta vómitos biliosos poco tiempo después de cada alimento.
Al preguntar a la madre sobre antecedentes, ella recuerda que tuvo polihidramnios
durante la gestación, pero un análisis de cariotipo fue normal. Una de las siguientes es la
causa más probable de estos hallazgos en el recién nacido: a. Enfermedad de
Hirschprung
e. Malrotación de la yema pancreática ventral
66.En un estudio de la secreción de hormonas gastrointestinales, sus concentraciones en
la vena porta se midieron durante perfusión luminal del intestino delgado con soluciones
de diversas magnitudes de pH. ¿Qué hormona aumentará en el plasma de la vena porta
durante perfusión a través del intestino con una solución de pH 3?
a. CCK
e. secretina
67.Paciente de 30 años que ingresa a causa de un traumatismo abdominal cerrado. En la
exploración se aprecia discreta palidez de piel y mucosas, auscultación pulmonar normal,
taquicardia de 120 /min. Discreta distensión abdominal y matidez en flancos; el
hematocrito, que era prácticamente normal al ingreso, disminuye a 30% a las tres horas.
En la Rx de tórax se objetiva fractura de las costillas 10-11 izquierdas. La causa más
probable de la anemización en este paciente es: a. traumatismo renal con hemorragia
retroperitoneal.
c. rotura de bazo con hemoperitoneo.
68. Revisando la angiotomografía de un hombre de 70 años en estudio por aneurisma
de aorta abdominal, el radiólogo le informa de la presencia de una oclusión completa
de la arteria mesentérica inferior. El paciente se encuentra completamente
asintomático. La oclusión de la arteria mesentérica inferior cursa de manera
asintomática en muchas ocasiones ya que el territorio que irriga puede recibir flujo
proveniente de la arteria:
a. cólica derecha
e. cólica media

69. En las patologías de esófago es importante conocer bien la anatomía esofágica. ¿Cuál
de las siguientes afirmaciones es correcta? a. El esófago tiene capa mucosa, muscular y
serosa

c. El esófago torácico pasa por detrás del cayado aórtico

70. A pesar de que pueda haber variaciones anatómicas, lo habitual es que el ciego
sea irrigado por una rama arterial que proviene de unas de las siguientes arterias:
a. Iliaca derecha

d. Mesentérica superior

71. Ante un paciente con una cirugía abdominal urgente, el informe operatorio señala
que se ha realizado una resección de todo el duodeno y del tercio proximal del yeyuno
manteniendo íntegros el estómago y todo el íleon, así como los dos tercios distales del
yeyuno. En el seguimiento nutricional del paciente ¿Qué vitamina o mineral presentará
con menor probabilidad una disminución de su absorción?
a. Cianocobalamina

72. ¿Cuál de las siguientes sustancias forma parte de la secreción biliar? a. Tripsina
Lecitina

73.¿De qué musculo forma parte el ligamento inguinal?


-Oblicuo externo del abdomen
74.¿Cuál de las siguientes enzimas está localizada en el borde en cepillo y juega un rol
en la digestión de proteínas?
e. Carboxipeptidasa A.
75. Una de los siguientes sustancias, NO sirve como un buen agente emulsificante:
a. Colesterol
76. La sustancia que estimula el crecimiento de la mucosa gástrica es:

a. Secretina

d. Gastrina

77.¿Cuál de las siguientes alternativas es una función de la colecistokinina?

a. Relajación de la vesícula para la salida de bilis

d. Secreción de enzimas pancreáticas

78.Con respecto a la anatomía del tronco celiaco, señale lo correcto a. El tronco celiaco
se origina de la cara posterior de la aorta abdominal
d. La hepática común que es una de sus ramas, participa en la irrigación del
estómago.
79. Con respecto a la anatomía del duodeno, marque la respuesta correcta: a. Tiene
una distribución en forma de “C”, que rodea la cola del páncreas
b. La 3ra porción duodenal está contenida en la pinza vascular aortomesentérica
80. En el íleon se absorbe aproximadamente el 95% dea través de la circulación
enterohepática.
a. agua
c. sales biliares
81. La estimula el mecanismo paracrino de la secreción de ácido clorhídrico.
a. histamina
82.En la digestión de proteinas,es el principal estímulo para convertir el
pepsinógeno en pepsina. a. la gastrina
b. el pH ácido
83. Con respecto a la somatostatina, marque lo correcto:
a. Es secretada por las células S del intestino
Interviene en la fase intestinal de la secreción gástrica
84. En pecten anal, es una estructura comprendida entre: a. la línea pectínea y los
senos anales
d. la línea anocutánea y la línea pectínea

85.¿Cuál de las siguientes alternativas es una proenzima pancreática? a. Tripsina

1. b. Elastasa
2. c. Quimotripsinógeno
3. d. Amilasa
4. e. Procarboxipeptidasa C
86. En este paciente, el bloqueo farmacológico de los receptores H2 en la mucosa gástrica:
a) No tiene efecto sobre la secreción de ácido inducida por el vago
b) Evita la activación de adenilciclasa por gastrina
c) Inhibe la secreción de ácido inducida por gastrina y mediada por el vago
d) Causa un aumento en el transporte de potasio por las células parietales gástricas
Se validó la A :)
87. Si se considerara una gastrectomía total para curar la gastritis del paciente, cuál
de las siguientes sustancias ya no se produciría:
a) Gastrina
b) Quimiotripsina
c) Amilasa
d) Pepsinógeno

88. Un paciente hipertenso está tomando un medicamento bloqueador de receptores


alfa 1 adrenérgicos (prazosina) y como efecto secundario se queja de:
d) Lo escaso que es el medicamento
e) No tiene problemas en la salivación
c) Hiposalivación
d) Hipersalivación

89. Con respecto a las lesiones y enfermedades de la boca, marque lo correcto:


f) La leucoplasia se desprende al roce
b) la eritroplasia puede degenerar en adenocarcinoma
c) El muguet oral es una enfermedad bacteriana en inmunodeprimidos
d) la eritroplasia debe ser biopsiada
NOTA: fue validada la opción B ya que no es motivo del curso que sepamos el puto
cáncer.

90. En este paciente, se puede asumir que la pancreatitis ha sido ocasionada por
una disminución en el efecto de:
g) Amilasa
h) Lipasa
c) Inhibidor de la tripsina
d) Entercinasa
91.Un efecto secundario en el estómago por la acción de la secretina es:
i) Disminución en la liberación de pepsinógeno
b) Menor actividad de la pepsina
c) Mayor paso de bicarbonato a sangre periférica
d) Aumento en la producción de factor intrínseco
92. Dentro de los factores protectores de la mucosa gástrica se pueden mencionar
múltiples protagonistas. Uno de ellos es:
j) CCK
k) Gastrina
c) Receptor muscarínico
d) Pepsina
93. La saliva puede tener una variedad de electrolitos en su composición. Entre ellos el
cloro, respecto al cual se puede afirmar:
l) Su mayor concentración se consigue con el flujo bajo
b) Su concentración no llega a ser tan alta como en el plasma
c) Con flujo alto su concentración es mayor que la del plasma
d) Su menor concentración se alcanza con flujo alto
94. En el síndrome de boca seca o síndrome de Sjogren, una de las
complicaciones asociadas es:
a) caries
b) Disminución de la acidez gástrica
c) Aumento en de la producción de saliva
d) Aumento del pH bucal
95. Estimula la producción de
saliva:
a) Vasodilatación periglandular
b) Atropina
c) Fatiga o cansancio
d) Expresión de miedo
96. El omeprazol actúa sobre la membrana de la célula
m) Basolateral/principal
n) apical/principal
o) Basolateral/parietal
d) apical/parietal
97. Para protegerse del entorno ácido, el Helicobacter pylori se autogenera un entorno
de pH menos ácido alrededor suyo, gracias a una enzima que alcaliniza su entorno
local mediante la conversión de:
a) urea en NH3
b) H2O y CO2 en ácido carbónico
c) NH3 en urea
d) H2CO3 en bicarbonato
98. La anemia perniciosa destruye las células:
p) mucosas del cuello
b) oxínticas
c) principales
d) mucosas superficiales
99. La célula mucosa del cuello gástrico produce:
a) Moco
b) ácido clorhídrico
c) pepsinógeno
d) Factor intrínseco
100.El aumento en la acidez del estómago producido principalmente por la
infección de Helicobacter pylori se debe a la disminución de:
a) Somatostatina
b) Bicarbonato por las glándulas de Brunner
c) Secretina
d) Colecistoquinina
101. De las siguientes sustancias secretadas por los órganos de este paciente, la
más alcalina es la secreción:
a) pancreática
b) Esofágica
c) Yeyunal
d) Salival
102.En cuanto a la gastritis de este paciente, se encontró que era producida por la
bacteria Helicobacter pylori. Esta bacteria sobrevive en el medio ácido del estómago
gracias a:
a) ácido clorhídrico
b) Toxina CagA
c) Ureasa
d) Jugo pancreático
103.La lengua está recubierta por epitelio:
c) pseudoestratificado columnar no queratinizado
b) plano estratificado no queratinizado
c) pseudoestratificado columnar ciliado
d) plano estratificado queratinizado
104.El esfínter anal interno tiene musculatura …….. y tiene control ……..
d) lisa / voluntario
b) lisa / involuntario
c) esquelética / simpático
d) esquelética / parasimpático

19) La arteria Aorta proporciona la irrigación al tubo digestivo ¿cuál de las


siguientes arterias proporciona la irrigación al ángulo cólico derecho?
a) mesentérica superior
b) mesentérica inferior
c) frénica inferior
d) tronco celiaco

20) Paciente de 26 años que le cuenta en su historia clínica que cada vez que almuerza a
los 20 min tiene deseo de defecar, le comenta que su hijo de 1 mes le pasa lo mismo
pero más intenso. Esto se explica por el reflejo …….., el cual está …… en el paciente
a) colicoileal / normal
b) colicoileal / alterado
c) gastrocólico / normal
d) gastrocólico / alterado

21) La región del estómago que se comunica con el duodeno es la


a) pilórica
b) cardias
c) cuerpo
d) fórnix

22) Acude a consulta un px que fue diagnosticado de úlcera péptica 3 días antes. Luego
de múltiples pruebas diagnósticas se concluye que el paciente presenta un tumor
secretor de gastrina, ¿cual de las siguientes situaciones estará incrementada?
a) distensión gástrica
b) inhibición del vaciado gástrico
c) secrecion de acido clorhidrico
d) inhibición de la secreción de pepsinógeno
23) En el sistema digestivo, el control del apetito está dado por un complejo sistema de
sustancias y órganos integradores, los cuales regulan la ingesta de alimentos. La
…… es una sustancia orexígena y es sintetizada por el ……
a) leptina / estómago
b) felina / intestino
c) leptina / estómago
d) grelina / estómago

24) Con respecto a la actividad eléctrica del sistema digestivo, marque la


alternativa correcta
a) corresponden a potenciales de acción que están presentes de forma continua
y le dan capacidad de peristalsis autónoma al sistema digestivo
b) la frecuencia de las ondas lentas NO se ve influenciada por la actividad neural ni
las
hormonas gastrointestinales
c) en el estómago las ondas lentas se dan en una frecuencia de 6 x min
d) las ondas lentas son cambios lentos y ondulantes del potencial en reposo
e) la frecuencia de las ondas lentas va de 6 a 12 ondas por minuto

25) Ante una lesión del IX pc, el músculo …… se altera en su función


a) palatogloso
b) estilofaríngeo
c) palatofaríngeo
d) constrictor superior

26) Un varón de 50 años es sometido a extirpación del duodeno y parte proximal del
yeyuno. La pérdida de estímulo hormonal en el páncreas para la secreción
enzimática se explica por la pérdida de células
a) parietales, productoras de factor intrínseco
b) K productoras de factor intrínseco
c) M productora de CCK
d) I productora de CCK

27) Marque la respuesta correcta:


A. El bronquio derecho constituye una de las estrecheces del esófago
B. Todos los órganos del sistema digestivo tienen capa serosa
C. La pared gástrica en el fondo es más delgada que en el cuerpo y antro
D. El esfínter de Oddi rodea a la papila menor duodenal

28) Marque la respuesta correcta en relación a la gastrina:


A. Al distenderse el estómago, se inhibe su producción.
B. Se estimula por la liberación de noradrenalina
C. Las células G son las productoras y se encuentran principalmente en el antro
gástrico
D. Las células G se encuentran principalmente en el fondo gástrico

29) Para poder morder una manzana, es necesario usar el siguiente músculo:
A. Milohiodeo
B. Tensor del paladar
C. Orbicular de los labios
D. Buccinador
30) Sustancia que inhibe la secreción y la motilidad del estómago prolongando el tiempo
de digestión:
A. Enteroglucagon.
B. Polipéptido pancreático
C. Péptido 1 similar al glucagón (GLP-1).
D. Péptido insulinotrópico dependiente de la glucosa (GIP).

31) El nacimiento de la arteria mesentérica superior se puede encontrar en cuál de


los cuadrantes abdominales:
A. Hipocondrio derecho
B. Hipogastrio
C. Epigastrio
D. Mesogastrio
32) Entre las múltiples causas de la Enfermedad por Reflujo Gastroesofágico, se puede
considerar también a una alteración en las del esfínter esofágico inferior:
A. Ondas secundarias
B. Contracciones tónicas
C. Ondas lentas
D. Glándulas subesofágicas

33) Producto de la alimentación, se producen diversas sustancias peptídicas, cininas


y bradicininas, las cuales permiten que:
A. Se produzca neovascularización en los territorios de las arterias abdominales
B. La acción de la lipasa pancreática se vea incrementada
C. El consumo de O2 del intestino aumente ligeramente
D. El flujo sanguíneo intestinal aumente hasta 8 veces

34) El dolor periumbilical o epigástrico en el inicio de una apendicitis aguda se debe a:


A. Estímulo del nervio vago.
B. Íleo secundario.
C. Irritación del peritoneo parietal.
D. Estímulo del sistema simpático.

35) El aumento en la actividad motora de la pared gástrica genera un aumento en


los niveles locales de qué sustancia en la microvasculatura:
A.Adenosina
B. Colecistoquinina CCK
C. Endotelina
D. Gastrina

36) ¿Cuál de los siguientes péptidos inhibe el vaciamiento gástrico?


A. Colecistoquinina
B. Péptido inhibidor gástrico
C. Motilina
D. Gastrina
37) Los músculos de la masticación que producen la retropulsión de la mandíbula son:
A. temporales [mas seguro]
B. maseteros
C. milohioideos
D. pterigoideos
38) En relación a la fisiología gástrica, marque lo correcto:
A. la cimetidina actúa en la región basolateral de la célula parietal
B. la marea alcalina se debe al paso de bicarbonato través de la membrana apical
de la célula principal
C. el cloro difunde hacia el exterior por la la región basolateral de la célula parietal
D. la salida de hidrogeniones a la luz es por difusión facilitada

39) Durante el sueño, la concentración de bicarbonato en la saliva:


A. Se eleva a niveles mayores que los del plasma
B. Aumenta
C. No tiene efecto
D. Disminuye

40) La secreción de saliva es importante en la fisiología digestiva. Su concentración


de potasio llega a ser menor que la del plasma cuando su secreción tiene un flujo:
A. Intermedio
B. Nunca
C. Bajo
D. Alto

42) Respecto a las glándulas salivales, marque lo incorrecto:


A. la glándula parótida produce secreción serosa
B. la glándula sublingual drena a través de conducto de Wharton
C. La glándula parótida drena a través del conducto de Stenon
D. la glándula sublingual tiene forma de garfio

43) Con respecto a la saliva, marque la respuesta correcta:


A. será hipertónica cuando el flujo es bajo
B. a mayor flujo, menor concentración de Na
C. a mayor flujo, menor concentración de cloro
D. el sistema simpático estimula su secreción
CI 3
44) En relación a la circulación hepática, marque lo correcto:
a) Los sinusoides hepáticos transportan sangre mixta
b) La vena porta proporciona el 50% de sangre al hígado
c) La vena porta se forma a partir de la vena esplénica y la mesentérica inferior
d) La arteria hepática deriva de la mesentérica superior

45) Dentro de las funciones de las células de Ito, marque lo incorrecto:


a) Sintetizan colágeno
b) Almacenan vitamina A
c) Se les llama células estrelladas
d) Pueden fagocitar patógenos y actúan como presentadoras de antígeno
46) Paciente con tumor neuroendocrino productor de secretina, debido a lo cual se puede
esperar que su secreción pancreática, comparada con la de una persona sana de bajo
flujo, tenga una concentración de:
a) Sodio aumentada
b) Igual
c) Bicarbonato aumentada
d) Potasio disminuida
47) El GALT se localiza en:
a) Lámina propia
b) submucosa
c) borde en cepillo
d) superficie de criptas de Lieberkühn

48) En relación a la histología hepática, marque lo correcto:


a) la zona 1 se afecta rápidamente en estados de hipovolemia y shock
b) La zona 1 se encuentra cercana a la vena central lobulillar
c) La zona 3 se encuentra más cerca a la vena central lobulillar
d) La zona 3 se encuentra más cerca al eje menor del acino hepático
49) El acino pancreático difere con el de las glándulas salivales en:
a) Contiene células centroacinares
b) No produce secreción serosa
c) El páncreas produce principalmente secreción mucosa
d) No tienen diferencias
50) Durante la digestión de las grasa, para que la lipasa actúe adecuadamente se requiere
que el pH aumento en la luz intestinal, lo cual es logrado, entre otros, por la secreción
de las células:
a) Del conducto interlobulillar
b) Centroacinares
c) Acinares
d) Alfa
51) La secreción de la colecistoquinina (CCK) se produce en la fase:
a) intestinal
b) En las 3 por igual
c) gástrica
d) Cefálica
52) ¿Por cuál de las siguientes células es secretada principalmente la pro
enzima procarboxipeptidasa?
a) Acinares del páncreas
b) Epiteliales del duodeno
c) Ductales del páncreas
d) Centro Acinares del páncreas
53) Una mujer de 43 años dolor en hipocondrio derecho e icterica. En la ecografía se
evidencia cálculos biliares. Estos cálculos lo más probable es que se encuentren
localizados en:
a) conducto colédoco
b) Conducto cístico
c) Vesícula biliar
d) Conducto pancreático secundario
ECU 1:
Estudiante de 21 años sufre de gastritis aguda ocasionada por comer en lugares poco
higenicos. Suele consumir caramelos (“chupar”) mientras esta en base hasta la tarde.
También toma regular cantidad de leche (grasa, lactora, proteinas), pues le calma el
dolor y el ardor que sitnete por la gastritis (tiene dispepsia y cuando toma la leche se le
pasa).
Incluso cuando puede, se toma dos vasos de agua frita y le calma la molestia. Ha
decido ir al medico para tratarse, pues ya no soporta el dolor, el cual esta seguro que
los síntomas se producen por elevada producion de HCl en el estomago, y por ello le ha
recetado ranitidina
1.1) El consumir caramelos eleva los niveles en sangre de una hormona cuya función es la
estimulación de las células.
- Beta del páncreas por GIP el cual es una incretina y por consiguiente
estimula las células pancreáticas

1.2) Consumir caramelos indirectamente actica la via:


-POMP/ CART saciedad

1.3) Consumo de leche produce indirectamente


- CCK inhibición del vaciamiento gástrico mayor tonicidad del esfínter pilórico

1.4) Cuando el px toma dos vasos de agua, genera indirectamente un aumento en la


liberación de:
- vaso de agua distención → g astrina → secreción de HCl

1.5) El uso de ranitidina bloquea el receptor H2 de la histamina en las células parietales, la


histamina llega a estas células por:
- histamina es una hormona paracrina por → difusión
**endocrina es por via hematógena y si fuera neuroendorina es por un NTs

1.6) Aumenta la secreción salival:


- noradrenalina a través de los receptores Beta 2
1.7) En este paciente con gastritis aguda debida a una alta producción de ácido clorhídrico,
sería lógico esperar que el píloro tenga un tono muscular:
- primero la secretina
- luego CCK
**ambas reguladores del HCl, Gatritis aguda debido a una alta producción de HCL
piloro estará aumentado (por la CCK)
1.8) Debido al uso de ranitidina, los valores de somatoestina en sangre:
- ranitidina disminuye acción de gastrina se quiere secretar mas no
actúan los inhibidores como la somatoestina somatoestina disminuye
1.9) El uso de atropina en este paciente:
- Inhibirá la acción de las prostaglandinas
- Aumentará la producción de ácido clorhídrico
- Disminuirá la acción del receptor CCK-B
-Aumentará el pH del estómago
ECU 2:
Niño de sexo masculino de 2 años de edad, sufre de estreñimiento desde el
nacimiento (1 deposición cada 3-4 días). Madre menciona que le estimula la
defecación con un termómetro rectal, y continuo uso de enemas y laxantes. Desde
hace 6 meses comienza con vómitos postprandiales. Los síntomas aumentan en
frecuencia y magnitud y están en relación con los episodios de estreñimiento. No
refiere fiebre, tos, diarrea ni lesiones cutáneas. Al examen físico presenta regular
estado general, luce deshidratado. Abdomen distendido, blando, depresible e indoloro.
No se palpan masas abdominales. Se permeabiliza el canal anal con termómetro
rectal, encontrando cierta resistencia. Salida de material fecal mal oliente en regular
cantidad. Exámenes de laboratorio: hemograma normal. Signos inflamatorios de fase
aguda negativos. Alcalosis metabólica leve en sangre
venosa. Radiografía con enema baritado muestra recto y colon sigmoides dilatados
(megacolon). Biopsia profunda: ausencia de células ganglionares en la muestra
enviada. Se realiza cirugía correctiva.

2.1) Durante la fase esofágica de la deglución, para un bolo alimenticio determinado, a


medida que avanza el bolo la fuerza de la contracción se hace más:
- hiperpolarizado
- fuerte
- dependiente de Ach
- debil
2.2) Cuando este paciente ingiera sus alimentos, se espera que al momento de pasar el
bolo alimenticio por el esfínter esofágico superior, la presión intraesofágica
disminuya en:
- la porción proximal al bolo
- el tercio medio del esófago
-el cardias
- el lugar donde se contraiga la muscular propia
2.3) Al examinar la orofaringe del paciente, uno puede hallar fácilmente la amígdala
palatina, pues esta se encuentra inmediatamente detrás del músculo:
-Palatogloso
- Palatofaringeo
- Hiogloso
- Elevador del velo del paladar

2.4) Con respecto a la defecación en este caso, marque la respuesta correcta:


- En posición de cuclillas, el músculo puborectal genera un ángulo más agudo
en el recto
- El sigmoides y el recto están inervados por el nervio vago
- La sensación de defecar sólo se da cuando el recto es ocupado por
heces y alcanzado el 80% de su capacidad
-El esfínter anal comprometido tiene inervación autónoma

2.5) En este paciente [hirschsprung] se considera que está abolido el reflejo:


- Coloileal
-Rectoesfinteriano
- Gastrocólico
- Relajación receptiva

2.6) No se espera que sea causa del vómito:


-Ayuno prolongado
- Estimulación faríngea y del glosofaríngeo
- Irritación de la mucosa gástrica
- Dolor intenso

ECU 3:
Paciente de 54 años con antecedentes de alcoholismo, gastritis crónica,
tabaquismo pesado, obesidad, cálculos biliares y cirrosis, es llevado a la
emergencia por dolor abdominal en epigastrio irradiado a la espalda y trastorno
del sensorio.
Al examen físico: presión arterial 85/50 mmHg, frecuencia cardíaca 100
latidos/min, frecuencia respiratoria 18 x minuto, temperatura axilar 36°C.
Conjuntivas pálidas, escleras ictéricas nevus arácnidos en tronco, distensión
abdominal marcada, cabeza de medusa, matidez desplazable en ambos flancos e
hipogastrio, dolor a la palpación de abdomen.
Tiempo de protrombina: 24 seg (testigo: 13 seg); TPT: 38 seg, glicemia: 165 mg/dL,
uremia: 20 mg/dL, ASAT: 76 UI/L, ALAT: 22 UI/L, albumina: 2,5 g/dL, bilirrubina total:
2,6 mg/dL, bilirrubina directa: 1,4 mg/dL, amilasa sérica 4000 U/L.
3.1) En esta paciente, al aumento de la amilasa sérica, se debe directamente a una lesión
de:
a) páncreas
b) vesícula y árbol biliar
c) estómago
d) hígado
3.2) Considerando que el paciente sufre de gastritis, se puede decir que la secreción de
ácido por la mucosa gástrica
a) involucra transporte activo de hidrogeniones
b) es realizada principalmente por células principales
c) es inhibida por antihistaminas tomadas por pacientes con rinitis alérgica
d) involucra la liberación de HCl de los gránulos zimógenos
3.3) El paciente tiene hemorragia digestiva alta por várices sangrantes como complicación.
Llegando a estar en shock hipovolémico por hemorragia masiva, se encontrara
necrosis hepática en:
a) zona 1
b) no se afectan los lobulillos hepáticos en hemorragia
c) zona 3
d) zona 2
3.4) El misoprostol, análogo de las prostaglandinas está mejor indicado en:
a) cicatrización de úlcera péptica duodenal
b) erradica el helicobacter pylori
c) tratar el sind de Zollinger ellison
d) prevenir daño por AINES
3.5) De las siguientes sustancias secretadas por los órganos de este paciente, la más
alcalina es la secreción:
- Esofágica
- Salival
- Yeyunal
-Pancreática
3.6) En este paciente, se puede asumir que la pancreatitis ha sido ocasionada por una
disminución en el efecto de:
- Lipasa
- Enterocinasa
- Amilasa
-Inhibidor de la tripsina
3.7) ¿Cuál de las siguientes sustancias es segregada por el páncreas?
-Amilasa
- Pepsina
- Quimiotripsina
- Tripsina
3.8) Cada vez que este paciente toma alcohol, la acidificación de la luz del duodeno:
-Disminuye el vaciamiento gástrico
- Aumenta la contracción del esfínter de Oddi
- Aumenta la secreción del ácido gástrico
- Disminuye la secreción pancreática del bicarbonato
SISTEMA
DIGESTIVO
(ME 154)
EXAMEN
FINAL
Ciclo 2018-01

1. Un niño de 2 años es llevado a la consulta por diarrea persistente y edema de las extremidades, además
falta de crecimiento y desarrollo en relación a su edad. Los análisis de sangre revelan que tiene
concentración plasmática baja de proteínas (hipoproteinemia). Durante la endoscopía duodenal, se
coloca colecistokinina (CCK) endovenosa y se recoge muestras del líquido duodenal; el resultado del
líquido confirma incapacidad para hidrolizar proteínas a un pH neutro, esta situación mejora al añadir
una pequeña cantidad de tripsina. El paciente probablemente esté sufriendo la falta congénita de
………….
(Unidad 4, sesión 26, logro 2: Explicar la Digestión y absorción de las proteínas y sus alteraciones)
a. Pepsinógeno
b. PEPT-1
c. Carboxipeptidasas
d. Enterocinasa

2. Experimentalmente se incrementa la velocidad de la secreción salival con una sustancia, el


análisis de la composición de esta saliva obtenida se espera encontrar…………..
(Unidad 3, sesión 17, logro 5 : Explica la Influencia de la velocidad del flujo salival en la composición de
la saliva)
a. Elevación de concentración de bicarbonato, sodio y potasio
b. Elevación de concentración de cloro, sodio y potasio
c. Disminución de concentración de potasio
d. Disminución de concentración de potasio y bicarbonato

3. Paciente varón de 46 años soltero, consulta por odinofagia y bajo de peso, tiene antecedente de
tuberculosis desde hace 3 meses y es fumador crónico (10 cigarrillos por día); al evaluar la cavidad
oral se identifica lesión blanquecina en el dorso de la lengua y paladar blando, las lesiones se
desprenden con el baja lengua dejando una base eritematosa. Esta lesión corresponde
probablemente a ……………………….…..
( Unidad 3, sesión18, logro 1-2 : Describe las enfermedades inflamatorias, infecciosas y proliferativas
de la cavidad oral)
a. Eritroplaquia
b. Candidiasis oral
c. Leucoplaquia vellosa
d. Fibroma en cavidad oral

4. Minero de 32 años de edad, que acude a


centro de salud por presentar de forma
progresiva desde hace 1 año dificultad para
ingerir alimentos sólidos y luego líquidos;
refiere regurgitaciones alimentarias y marcada
pérdida de peso (15 kilos). Radiografia
baritada de esófago como se muestra en la
figura. El presente caso se explica
por……………….
(Unidad 2, sesión 12, logro 4: Identificar y
describir la función de los esfínteres
esofágicos)
a. Contracción incompleta del esfínter esofágico inferior
b. Dificultad para el inicio de la deglución
c. Relajación incompleta del esfínter pilórico
d. Relajación incompleta del esfínter esofágico inferior
5. Paciente mujer de 35 años acude a consulta por sensación de sequedad y lesiones en cavidad
oral. Al examen se observa atrofia de la mucosa, fisuras y úlceras; nota además sequedad e
irritación de la córnea y aumento del tamaño de las glándulas parotídeas. Su diagnóstico más
probable es artritis reumatoide; el hallazgo más probable en una biopsia de glándula parótida
es……..….
(Unidad 3, sesión 18, logro 3: Describe las enfermedades más frecuentes de las glándulas salivales)
a. Hiperplasia de acinos glandulares serosos
b. Gran infiltración de linfocitos y células plasmáticas
c. Gran infiltrado de linfocitos y macrófagos
d. Presencia de acinos normales con hiperplasia de células ductales

6. Un paciente con anemia acude con su médico quejándose de episodios frecuentes de


gastroenteritis. Un análisis de sangre revela anticuerpos circulantes dirigidos contra células
parietales gástricas. Su anemia es atribuible a la hiposecreción de
………………………
(Unidad 3, sesión 20, logro 5: Gastritis crónica. Tipos de gastritis)
a. Factor intrínseco
b. Proteina R (haptocorrina)
c. Pepsinógeno
d. Ácido clorhídrico

7. Dos estudiantes deciden tomar un receso para comer una hamburguesa a la hora del almuerzo.
Antes de llegar a la cafetería, impulsos nerviosos provenientes del complejo vagal dorsal iniciarán la
secreción de ácido gástrico por la liberación dedesde el sistema nervioso entérico.
(Unidad 3, sesión 20, logro 2: Regulación de la secreción gástrica: estimulación, fases de la secreción)
a. Serotonina
b. Óxido nítrico
c. GRP (péptido liberador de gastrina)
d. Péptido intestinal vaso activo

8. Un niño de cuatro años de edad es llevado a la consulta por cuadros diarreicos frecuentes
caracterizados por heces pálidas, voluminosas y fétidas, presenta bajo peso y talla. Se mide la
concentración de cloruro en el sudor y se encuentra que sus valores son muy elevados. La alteración
más importante a nivel de células ductales del páncreas tiene relación directa con la conductancia
de…………
(Unidad 3, sesión 23, logro 5 Explica la Secreción pancreática: formación del jugo pancreático,
influencia de la velocidad de flujo y regulación)
a. Potasio
b. Bicarbonato
c. Sodio
d. Cloro

9. Una mujer de 50 años de edad que sufrió durante varios años resequedad de los ojos debida a
producción inadecuada de lágrimas es enviada con un gastroenterólogo para evaluación de pirosis
crónica. El examen endoscópico revela erosiones y tejido cicatrizal en la parte distal del esófago
justo por arriba del esfínter esofágico inferior. Las lesiones pueden atribuirse a la disminución de
uno de los siguientes componentes salivales:
(Unidad 3, sesión 17, logro 4: Explicar la Formación de la saliva y cuáles son sus componentes)
a. Bicarbonato
b. Lactoferrina
c. Ig A
d. Amilasa
10. Se evalúa los valores séricos de las siguientes sustancias a un paciente con enfermedad hepática
terminal; en este paciente se espera encontrar la combinación con la letra …………
(Unidad 3, sesión 22, logro 5: Describe las Pruebas de función hepática, la Insuficiencia hepática,
encefalopatía hepática e hipertensión portal)

Glucosa Amoniaco Albúmina


a. Aumenta Disminuida Disminuida
da
b. Disminui Aumentada Aumentada
da
c. Aumenta Aumentada Aumentada
da
d. Disminui Aumentada Disminuida
da

11. Una mujer de 35 años de edad HIV positiva, se presenta al médico con dolor abdominal en
cuadrante superior derecho e ictericia. La paciente refiere haber tenido múltiples episodios de
ictericia durante los últimos 10 años. Los exámenes para determinar hepatitis viral, dieron positivos
para Hepatitis B, siendo catalogado el caso como hepatitis crónica con alteración funcional. En un
examen de sangre ¿cuál de los siguientes parámetros está disminuido?
(unidad 3, sesión 22, logro 5: Pruebas de función hepática, Insuficiencia hepática, encefalopatía
hepática e hipertensión portal)
a. Fosfatasa alcalina
b. Albumina
c. Bilirrubina
d. Tiempo de protrombina

12. En el reflejo peristáltico del intestino delgado, uno de los siguientes eventos sucede en
la porción oral del bolo alimenticio…………...
(Unidad 2, sesión 13, logro 4: Explicar la Motilidad del intestino delgado: Contracciones segmentarias y
peristálticas)
a. Disminución de 5 hidroxitriptamina desde las neuronas IPAN
b. Contracción del músculo longitudinal
c. Acción del péptido intestinal vasoactivo (VIP) en el músculo circular
d. Acción de acetilcolina en el músculo circular

13. Experimentalmente se coloca una dosis alta de secretina en la luz intestinal duodenal; como
consecuencia de esto, en el jugo pancreático de la misma luz intestinal se observa la disminución
de la concentración de …..………..
(Unidad 3, sesión 23, logro 5: Explica la Secreción pancreática: formación del jugo pancreático,
influencia de la velocidad de flujo y regulación)
a. Na+
b. Cl-
c. K+
d. HCO3-

14. Un varón de 58 años de edad con enfermedad de Crohn severo fue sometido a una resección
ileal. Después de la cirugía este paciente padecerá de esteatorrea, esto se explica porque
…..………..
(unidad 4, sesión 26, logro 4: Explica las alteraciones en la Absorción de lípidos)
a. El pool de ácidos biliares se incrementa
b. Los quilomicrones no pueden formarse en el lumen intestinal
c. La micelas no pueden formarse
d. El páncreas no secreta lipasa

15. En un experimento se inserta un balón en el estómago de un voluntario, se infla poco a poco


mientras que se vigilan las presiones intraluminales. Aunque el volumen del balón aumenta
considerablemente, las presiones permanecen constantes. Esta relación volumen-presión se explica
por la liberación local de …………..
(Unidad 2, sesión 13, logro 1 Explica la Motilidad gástrica: relajación receptiva)
a. Acetil colina y gastrina
b. Colecistoquinina y óxido nítrico
c. Óxido nítrico y péptido inhibidor vasoactivo
d. Norepinefrina y óxido nítrico
16. La toxina del Vibrio cholerae causa diarrea debido a…….
(Unidad 4, sesión 27, logro 6: Explica el transporte hidroelectrolítico intestinal, toxina colérica)
a. La fosforilación del canal CFTR de los enterocitos de las vellosidades intestinales
b. El Incremento de la secreción de cloro por las células de la cripta intestinal
c. La inhibición de la producción de AMPc por las células epitelailes
d. El incremento de la absorción de agua y sodio a través de las uniones estrechas

17. ¿Cuál de las siguientes alternativas es una característica de la secreción exocrina del páncreas?
(Unidad 3, sesión 23, logro 5: Secreción pancreática: formación del jugo pancreático, influencia de la
velocidad de flujo y regulación)
a. Tiene una baja concentración de Cl- respecto al plasma
b. Es estimulada por la presencia de bicarbonato en el duodeno
c. La secreción enzimática es estimulada principalmente por la gastrina
d. Es hipotónica respecto al plasma

18. Una madre lleva a su hijo de dos años de edad a la sala de urgencias, estresada porque el niño
deglutió una moneda de 10 céntimos mientras la familia cenaba en un restaurante. El médico
observa mediante fluoroscopía que la moneda se halla en el estómago y asegura a la madre que la
moneda se eliminará con las heces. El médico recomienda utilizar la respuesta fisiológica que
permitirá la evacuación de la moneda del estómago al intestino ………….…..
(Unidad 2, sesión 13, logro 2: Explica la Motilidad gástrica: mezclado y vaciamiento)
a. Es por la relajación receptiva
b. Son los movimientos de mezcla y trituración
c. Es provocada por el ayuno
d. Es por la relajación del esfínter esofágico superior

19. Las estructuras en el hígado que permite que los productos metabólicos unidos a proteínas tengan
acceso a las membranas basolaterales de los hepatocitos, son…..
(Unidad 3, sesión 21, logro 4-5: Explica la Organización micro estructural del hígado)
a. Los Canalículos
b. Las fenestras sinusoidales
c. Las uniones intercelulares herméticas
d. Las células de Ito

20. La composición de la bilis es modificada conforme fluye por los conductillos biliares. Durante
este tránsito se espera que aumente la concentración de…….
(Unidad 3, sesión 22, logro 2: Describe la Secreción biliar, visión general del sistema biliar
extrahepático y composición de la bilis)
a. Ig A
b. Glucosa
c. Monómeros de ácido biliar
d. Vitamina A

21. Se mide experimentalmente el contenido gástrico de dos personas. La persona “A” tiene alto
contenido de grasa y la persona “B” tiene un contenido hipertónico ¿Cuál de las siguientes es
correcto respecto al vaciamiento gástrico? (Unidad 2, sesión 13, logro 2: Describe la Motilidad
y vaciamiento gástrico)
a. Hay ralentización del vaciado gástrico solo en “A”
b. El vaciamiento gástrico es más rápido en ambos
c. En ambos casos hay incremento de la motilina
d. Hay ralentización del vaciado gástrico en ambos casos

22. El examen endoscópico de un paciente con hipertensión portal grave revela venas tortuosas que
sobresalen hacia la luz del esófago. El paciente recibe tratamiento quirúrgico mediante la colocación
de una derivación que conecta la vena porta a la vena cava. Después de la operación el riesgo de
encefalopatía ………………….. y el riesgo de sangrado de várices ……………..
(Unidad 3, sesión 22, logro 5: Describe la Insuficiencia hepática, encefalopatía hepática e hipertensión
portal)
a. Aumentará/disminuirá
b. Disminuirá/disminuirá
c. Aumentará/aumentará
d. Disminuirá/aumentará
23. Un paciente varón de 18 años de edad acude al médico para sus exámenes de rutina. Sus
resultados de laboratorio muestran un valor de bilirrubina sérica de 4 mg/dl y una bilirrubina
directa de 0,3 mg/dl. Las pruebas de función hepática son normales. La alteración que explica
mejor este caso es por la deficiencia de ………………..
(Unidad 3, sesión 22, logro 3: Explica la Producción y excreción de bilirrubina. Tipos de bilirrubina e
ictericia)
a. Transaminasas
b. Glucuronil transferasa
c. Hemo oxigenasa
d. La 7 alfa hidroxilasa

24. Un hombre de 57 años de edad es llevado a urgencias con hematemesis masiva rojo brillante, a su
llegada se halla inconciente con PA: 80/40 mm Hg y FC: 124 lat/min. Luce ictérico con presencia de
“arañas vasculares en el tórax anterior y extremidades”, abdomen distendido con signo de oleada
positiva. Se encuentra esplenomegalia y pérdida de la masa muscular en extremidades. La
anastomosis vascular responsable del sangrado en este paciente es ………….…..
(Unidad 3, sesión 21, logro 2: Describe las anastomosis porto sistémicas)
a. Arteria gástrica izquierda y vena ácigos
b. Vena gástrica izquierda y vena ácigos
c. Vena paraumbilical y vena epigástrica inferior
d. Vena gástrica izquierda y vena esofágica superior

25. Un estudiante de medicina está comiendo un plato de comida a base de champiñones, espárrago
y salsa de soya. El sabor umami contenido en todos estos alimentos actúa a nivel de los botones
gustativos estimulando ………………..
(Unidad 2, sesión 10, logro 5: Describe los tipos y mecanismos moleculares para la detección de los
sabores)
a. El ingreso de sodio
b. Un receptor acoplado a proteína G
c. Su receptor específico T1R3
d. El ingreso de hidrógeno

26. Un hombre de 22 años de edad se presenta al médico con una historia de 1 año de evolución
caracterizado por dolor recurrente en fosa iliaca derecha y diarrea. Manifiesta además pérdida
de peso de 8 kg durante este periodo. La colonoscopía revela múltiples lesiones en el ileon
terminal y colon. La biopsia de estas lesiones revela engrosamiento, inflamación y ulceración
de la mucosa. El diagnóstico más probable en este caso es…….
(Unidad 4, sesión 28, logro 5: Describe la Enfermedad inflamatoria intestinal. Generalidades,
morfología y características)
a. Sprue celiaco
b. Enfermedad de Crohn
c. Sindrome de colon irritable
d. Colitis ulcerativa

27. Una de las funciones del músculo señalado es:


(Unidad 2, sesión 8, logro 3: Describir el Piso
de la boca: estructuras blandas que la
conforman)
a. Eleva el paladar blando
b. Recibe inervación del nervio maxilar
c. Deprime el hioides cuando la mandíbula está fija
d. Deprime la mandíbula cuando el hioides está fijo

28. Varón de 61 años que consulta por dolor retro esternal intenso desde hace 6 horas y después
de vómitos intensos y repetidos; al examen se observa disnea, cianosis, hipotensión y signos
clínicos de shock. La radiografía simple de tórax muestra neumomediastino. El líquido en el
espacio pleural aspirado tiene alta concentración de amilasa. ¿Cuál de las siguientes
alternativas puede explicar este cuadro clínico?
(Unidad 3, sesión 18, logro 6: Describe algunas Enfermedades del esófago)
a. Sindrome de Mallory Weiss
b. Rotura espontánea de esófago
c. Neumotórax por probable herida penetrante
d. Perforación de ulcera gástrica de cara posterior, con complicación torácica
29. La secreción del ácido en la célula parietal gástrica se lleva a cabo por una ATPasa especifica que
intercambia hidrogeniones (H+) del citosol por…..
(Unidad 3, sesión 20, logro 1: Explica la Secreción del HCl y sustancias que la alteran)
a. Cl-
b. HCO3-
c. Na +
d. K+

30. En condiciones normales el ingreso de 600 ml de líquido es el estómago provoca un aumento de


presión intragástrica de unos 12 cm de H2O. Después de una vagotomía (corte del nervio vago) es
de esperar que el ingreso del mismo volumen de líquido provoque lo siguiente:
…………………………………
(Unidad 2, sesión 13, logro 1: Describe la Motilidad gástrica: relajación receptiva)
a. Un aumento igual de la presión
b. Que no aumente la presión
c. Un aumento mayor de la presión
d. Una disminución de la presión

31. Una paciente de 30 años de edad es sometida a una cirugía en oído medio derecho por un problema
de otoesclerosis. Luego de la cirugía refiere alteración en la percepción de sabores. Al evaluar el caso
usted esperaría encontrar……….
(Unidad 2, sesión 10, logro 5: Describe la Irrigación e inervación de la lengua)
a. Alteración en la sensación del dolor y temperatura en el tercio posterior de la lengua
b. Alteración en la sensación del gusto en los dos tercios anteriores de la lengua
c. Alteración en la sensación del gusto en la punta de la lengua
d. Sensación del dolor, tacto y temperatura conservada en toda la lengua

32. ¿Cuál de las siguientes alterativas es correcta?


(Unidad 4, sesión 26 : Explica la digestión y absorción de nutrientes y sus alteraciones)
a. En el borde luminal, en cepillo, del intestino delgado, la absorción de sodio únicamente se
realiza asociada a la de glucosa.
b. El lugar principal para la absorción del hierro es el ileon
c. Las sales biliares desconjugadas son absorbidas preferentemente en el colon
d. El proceso de digestión y absorción de la vitamina B12 no se altera en insuficiencia
pancreática.

33. En un paciente de 45 años de edad con colestasis biliar, se encuentra una elevación de los niveles
sanguíneos de fosfatasa alcalina hasta 3 veces la cifra normal. ¿Cuál de las siguientes alternativas
estará también elevada como evidencia del daño de la vía biliar?
(Unidad 3, sesión 22, logro 5: Pruebas de función hepática, Insuficiencia hepática, encefalopatía
hepática e hipertensión portal)
a. Tiempo de protrombina y albúmina sérica
b. Transaminasas hepáticas (ALT y AST)
c. Glucoronil transferasa
d. Gamma glutamil transpeptidasa

34. Revisando la angiografía de un hombre de 70 años en estudio por aneurisma de aorta abdominal el
radiólogo informa de la presencia de una oclusión completa de la arteria mesentérica inferior. El
paciente se encuentra completamente asintomático. ¿Cuál de las siguientes arterias se anastomosa
a la sistema arterial de la mesentérica inferior?
(Unidad 4, sesión 25, logro 1: Identifica la Arteria mesentérica superior e inferior, ramas y anastomosis)
a. Ileal
b. Cólica media
c. Sigmoideas
d. Cólica izquierda

35. Lactante de 3 meses de vida es atendido por presentar diarrea, se administra una solución de
glucosa y electrólitos por vía oral. La proteína de membrana apical que explica la capacidad de esta
solución para proporcionar aporte de glucosa e hidratación es ………..
(Unidad 4, sesión 26, logro 1: Explica la Digestión y Absorción de los hidratos de carbono. Alteraciones)
a. GLUT-5
b. SGLT-1
c. CFTR
d. GLUT-2
36. Paciente ha sufrido herida de bala en el abdomen, se le ha tenido que extirpar el segmento medio y
distal del ileon. En este caso la síntesis hepática de sales biliares estará …..…..
(Unidad 3, sesión 22, logro 4: Explica la formación, función y Circulación entero hepática de lasa sales
biliares)
a. Disminuida por inhibición de la colesterol 7 alfa hidroxilasa
b. Incrementada por estímulo de la enzima colesterol 7 alfa hidroxilasa
c. Incrementada por inhibición de la colesterol 7 alfa hidroxilasa
d. Sin cambios en el ritmo de síntesis

37. Un varón de 75 años ingresa al consultorio por presentar ictericia marcada de piel y las escleras. El
estudio del paciente mostró que presentaba un tumor que obstruía la totalidad del conducto
hepático común. ¿Cuál de las siguientes estructuras se encontrará dilatada en este paciente?
(Unidad 3, sesión 21, logro 6: Describir el árbol biliar intrahepático)
a. Conducto de Wirsung
b. Conductos de Hering
c. Conducto colédoco
d. Conducto cístico

38. Correlaciones las dos columnas y marque la fórmula correcta:


(Unidad 4, sesión 28, logro 1: Diarrea: definición, mecanismos: osmótica, secretoria y exudativa)
1. Enfermedad Hirschsprung( ) heces con moco y sangre
2. Diarrea osmótica( ) intolerancia a lactosa
3. Diarrea secretoria( ) aganglionosis congénita
4. Diarrea exudativa( ) canales de Cl- en las células de la cripta

a.- 4231b.- 1234c.- 2143d.- 4213

39. Respecto a la siguiente imagen que representa una estructura de la mucosa gástrica, la estructura con
número ………..
produce ……………………..
(Unidad 3, sesión 19, logro 4: La glándula fúndica. Funciones y tipos de células con sus características)
a. 3 / pepsina
b. 1 / Pepsinógeno
c. 4 / HCl y factor extrínseco
d. 2 / pepsinógeno

2
40. En un paciente con insuficiencia renal crónica, el déficit en la absorción de calcio a nivel del
enterocito se debe a lo siguiente:
(Unidad 4, sesión 26, logro 6: Explica la Absorción de calcio y hierro)
a. No se convierte la 25 hidroxicolecalciferol a 1,25 dihidroxicolecalciferol
b. No se convierte la 1,25 dihidroxicolecalciferol a 25 hidroxicolecalciferol
c. Se incrementa la producción de Calbindina
d. Existe un descenso de la alfa 25 hidroxilasa renal
CLAVES EXAMEN PARCIAL DE SISTEMA
DIGESTIVO 2019 - 00

1. Varón de 30 años es traído a emergencia por agresión abdominal con arma de fuego
(pistola) y es sometido a laparotomía exploratoria, observándose isquemia del colon
ascendente y parte del colon trasverso ¿la lesión de cuál de las siguientes arterias
explicaría esta isquemia?
(unidad 1, sesión 2, logro 6: (Describe la irrigación visceral: arterias de tronco
celiaco, arteria mesentérica superior e inferior, topografía de superficie, órganos
por cuadrante)

a. Celiaca
b. Colónica derecha
c. Mesentérica inferior
d. Mesentérica superior

2. Respecto a las sustancias gastrointestinales que regulan la secreción pancreática;


marque la afirmación correcta:
(unidad 1, sesión 3, logros 2 y 3: describir las hormonas gastrointestinales: estímulos y
funciones)

a. La Secretina, es la hormona más importante para la secreción de bicarbonato por


las células acinares del páncreas
b. La acetilcolina es capaz de estimular la secreción enzimática y de bicarbonato del páncreas
c. La gastrina, es la hormona más importante para la secreción de enzimas pancreáticas
d. La colecistoquinina (CCK) estimula al páncreas solo para secreción enzimática

3. Ante una lesión del X par craneal, ¿cuál de los siguientes músculos mantiene conservada su
función?:
(unidad 2, sesión 08, logro 4: Paladar blando: componentes musculares)

a. Elevador del velo del paladar


b. Tensor del velo del paladar
c. Palatofaríngeo
d. Glosofaríngeo

4. Experimentalmente se utiliza atropina (anticolinérgico) para inhibir la secreción de


gastrina, sin embargo la secreción de esta hormona se sigue dando ante estímulos
vagales. Esta situación se explica porque la atropina:
(unidad 1, sesión 3, logro 3 : describir las hormonas gastrointestinales: estímulo y funciones
de la gastrina y colecistoquinina)

a. Bloquea parcialmente la bomba de protones en la célula G


b. Inhibe la acción de acetilcolina e histamina en la célula G
c. Solo inhibe la acción del péptido GRP en la célula G
d. No bloquea la acción del péptido GRP

5. Un varón de 50 años es sometido a extirpación del duodeno y parte proximal


del yeyuno. Esta situación ocasionaría la pérdida de las células ……….. ,
productoras deque estimula la secreción de
bicarbonato por el páncreas.
(unidad 1, sesión 3, logro 3: describir las hormonas gastrointestianles: estímulos y
funciones de la secretina y péptido insulinotrópico dependiente de glucosa)

a. “S” / secretina
b. Parietales / secretina
c. “I” / colecistoquinina
d. “S” / colecistoquinina
6. Recién nacido que presenta tumoración abdominal a nivel del cordón umbilical
(fotografía). ¿cuál de las siguientes afirmaciones es correcta respecto a este
defecto en el desarrollo embriológico del intestino?: (unidad 1, sesión 5, logro 2:
identificar las anomalías del desarrollo del intestino medio)

a. Corresponde a una Gastrosquisis


b. Las vísceras se hallan cubiertas por piel
c. No está asociado a otras malformaciones
d. Se asocia a
malformaciones
cardiacas y del tubo
neural

7. Varón de 35 años acude a la emergencia por


trauma abdominal y se decida realizar una
laparoscopía exploratoria. El cirujano
observa la disposición de los órganos
abdominales como se representa en el
siguiente esquema. Esta disposición de
órganos se explica por la
rotación(SMA=arteria mesentérica superior)
(unidad 1, sesión 5, logro 3: identificar las anomalías
del desarrollo del intestino medio: defectos de
rotación, estenosis y atresias)

a. anti horaria del intestino medio, en sólo 90°


b. incompleta del intestino medio (270°)
c. horaria del intestino medio
d. horaria del estómago

8. Se evalúa la expresión de la proteína Agrp en una persona con alteración del


apetito; lo correcto respecto a esta proteína es…..
(unidad 1, sesión 3, logro 4: Explica los mecanismos de control del apetito y saciedad )

a. Esta proteína es un potente anorexigénico


b. La mutación del gen que la codifica produce adelgazamiento
c. La sobre producción de la proteína lleva a obesidad por agonismo de receptores MC3 y MC4
d. La sobre producción de la proteína disminuye el apetito por antagonismo de receptores MC4

9. Juana cae de la bicicleta y se fractura la región anterior del hueso maxilar superior con
compromiso de la fosa incisiva. Al examen físico de la región esperaría encontrar
alteración en la sensibilidad de la encía …………………
(unidad 2, sesión 8, logro5: paladar: paladar duro y blando: irrigación e inervación)

a. bucal posterior
b. Lingual anterior
c. palatina anterior
d. palatina posterior
10. Recién nacido es atendido por el neonatólogo y luego entregado a su madre para
dar de lactar; la madre al dar de lactar observa coloración azulada de labios,
acompañado de tos persistente, dificultad respiratoria y distención abdominal. Se le
intenta colocar una sonda nasogástrica pero esta retorna a la cavidad oral en todos
los intentos. ¿Cuál de las siguientes anomalías del desarrollo es el más probable en
este caso? (unidad 1, sesión 4, logro 3: identificar las anomalías en el desarrollo del
esófago: atresia y/o fístula traqueo esofágica)

a. Estenosis esofágica proximal con Fístula traqueo esofágica distal


b. Atresia esofágica proximal con fístula traqueoesofágica distal
c. Atresia esofágica distal con fístula traqueoesofágica proximal
d. Fístula traqueoesofágica proximal y distal

11. ¿Cuál de los siguientes mecanismos ocurre durante la defecación?


(unidad 2, sesión 13, logro 6: motilidad del intestino grueso: contracciones
segmentarias, movimientos en masa, defecación y reflejo gastrocólico)

a. Contracción refleja del esfínter anal interno


b. En la posición de “cuclillas” el músculo puborectal se halla relajado
c. Relajación del esfínter anal externo por efectos del VIP y óxido nítrico
d. La materia fecal en el recto estimula la contracción del sigmoides por los nervios pudendos

12. La estructura número 4 (gráfico) corresponde a


……….… y está ………..
(unidad 2, sesión 9, logro 2:
Partes de un diente. Capas
del diente: Esmalte:
características y células que
lo producen)

a. el cemento / mineralizado en 90%


b. la dentina / formada por ameloblastos
c. el esmalte / formado
por células derivadas
del mesénquima
d. la dentina / formado por
células derivadas de la
cresta neural

13. Un paciente luego de un accidente sufre lesión del piso de la boca, se constata
daño del nervio “cuerda del tímpano”, en este caso se esperaría encontrar
disminución de lade la lengua
(unidad 2, sesión 10, logro 3: Irrigación e inervación de la lengua)

a. Motilidad en los dos tercios anteriores


b. Sensación del gusto en el tercio posterior
c. Sensación del gusto en los dos tercios anteriores
d. Sensibilidad al tacto en los dos tercios anteriores

14. ¿Cuál de las siguientes afirmaciones es la correcta sobre la gastrina?


(unidad 1, sesión 3, logro 1: reconocer las características de las sustancias
reguladoras gastrointestinales: hormonas, sustancias paracrinas y neurocrinas)

a. Produce atrofia de la mucosa gástrica


b. Es producida por la célula G del cuerpo gástrico
c. Es estimulada por la distensión gástrica y el Ph bajo
d. Actúa en la célula diana mediante su receptor CCk tipo B
15. Al recibir un paciente con signos de hipovolemia y antecedente de trauma en
abdomen por accidente de tránsito, usted identifica radiológicamente: lesión de
primera vértebra lumbar y signos de lesión en páncreas; durante la cirugía se observó
pobre irrigación de asas intestinales. El vaso afectado es la arteria ……..
(unidad 1, sesión 1, logro 6: reconocer las estructuras a nivel de L1, nivel de los principales
vasos sanguíneos)

a. esplénica
b. hepática común
c. mesentérica inferior
d. mesentérica superior

16. Un paciente sufre de daño a nivel del cuello con lesión muscular en la región de la
faringe. En el examen físico se determina dificultad para la elevación de la faringe y
para el cierre del itsmo de las fauces. En este caso, probablemente esté afectado el
músculo:
(unidad 2, sesión 11, logro 2: Músculos de la faringe: identificación, constrictores y
longitudinales)

a. palatogloso
b. estilofarinfeo
c. palatofaringeo
d. constrictor inferior

17. Varón de 50 años a quien le realizan la curación de la segunda molar de la arcada


superior derecha. En un momento determinado, el paciente acusa de intenso
dolor de la pieza dentaria en tratamiento. La vía aferente del dolor viaja a través
del nervio …………
(unidad 2, sesión 9, logro 6: Inervación de los dientes)

a. trigémino V2
b. trigémino V3
c. naso palatino
d. palatino menor

18. La distención gástrica por los alimentos produce incremento de secreción de HCl mediante la
producción de
………….. que estimula a las células ……………. vía proteína ………..
(Unidad 1, sesión 3, logro 2: Describe las hormonas gastrointestinales: Estímulo y funciones de
la gastrina y colecistoquinina)

a. gastrina / parietal / Gq
b. gastrina / principal / Gs
c. acetilcolina / parietal /Gi
d. acetilcolina / principal / Gi

19. Un niño de tres años llega a emergencia con disfagia (dificultad para tragar), dolor
retro esternal, salivación y llanto. Se sospecha de ingesta de cuerpo extraño
(moneda) en el esófago; al ser evaluado se constata en una radiografía presencia de
cuerpo extraño a nivel de C6 (6° vértebra cervical). El cuerpo extraño estará
suspendido a nivel del estrechamiento producido por………..
(unidad 2, sesión 11, logro4: Esófago, características anatómicas, relación con órganos
vecinos y estrecheces)

a. el cayado aórtico
b. el hiato esofágico
c. el músculo cricofaríngeo
d. el bronquio principal izquierdo

20. La triada portal (arteria hepática, vena portal y conducto biliar común) está contenida en el
ligamento
…….……… y derivan embriológicamente del ……
(Unidad 1, sesión 1, logro 4: Identifica el peritoneo, mesenterio, omento y ligamentos,
retroperitoneo.)

a. hepato duodenal / mesenterio ventral


b. gastro esplénico / mesenterio dorsal
c. hepato gástrico / omento menor
d. falciforme / omento menor
21. En relación al movimiento de
peristaltismo del tubo
digestivo: en la flecha negra
del gráfico se produce la
liberación de ……………… a
nivel del músculo ………..
(unidad 2, sesión 7, logro 6:
Control hormonal y tipos de
movimiento)

a. noradrenalina, sustancia P y neuropéptido “ Y” / circular


b. acetilcolina y sustancia P / longitudinal
c. óxido nítrico y PIV / longitudinal
d. óxido nítrico y PIV / circular

22. Un paciente refiere no percibir algunos sabores, al examen físico se constata


alteración en la percepción de sabores y del dolor en el tercio posterior de la lengua
¿Cuál de los siguientes nervios estará alterada en su función?
(unidad 2, sesión 10, logro 5: Sabores, tipos y mecanismos moleculares para su detección)

a. Lingual (rama del V par)


b. Cuerda del tímpano (VII par)
c. Glosofaríngeo (IX par)
d. Hipogloso (XII par)

23. El gráfico detalla la


estructura de la pared del
tubo digestivo intestinal
¿Cuál de las siguientes
asociaciones es correcta?
(unidad 2, sesión 7, logro 1:
La pared y músculo liso
gastrointestinal )

a. “1” – peristaltismo
b. “2” – secreción enzimática
c. “3” – deriva del mesodermo
d. “4” – doble hoja de tejido graso

24. En el caso de un paciente con gastrinoma (tumor productor de gastrina), la presencia


de úlceras duodenales y erosión de la mucosa gástrica, se debe principalmente a…….
(unidad 1, sesión 3, logro 2: describir las hormonas gastrointestinales: estímulo y funciones de
la gastrina y colecistoquinina)

a. la acción directa de la gastrina sobre la célula principal


b. la sobre expresión de los receptores “G” en la célula parietal
c. el exceso de HCl por estímulo de receptores CCK-B en la célula parietal
d. el exceso de HCl por estímulo directo de receptores de acetilcolina en la célula parietal

25. El reflejo entero gástrico se caracterizan por:


(unidad 2, sesión 13, logro 6: Motilidad del intestino grueso: contracciones
segmentarias, movimientos en masa defecación y reflejo gastrocólico)

a. favorecer la motilidad gástrica gracias a la CCk


b. inhibir la motilidad gástrica y estimular la secreción ácida
c. movilizar grandes volúmenes desde el estómago al duodeno
d. originarse debido a la distensión duodenal y presencia del quimo ácido
26. Mauricio tiene dificultad para deprimir el paladar y elevar la parte posterior de la
lengua. En este caso estará afectado un músculo ………………., específicamente el
músculo …………….
(Unidad 2, sesión 10, logro 2: Músculos de la lengua: clasificación, identificación y sus
funciones)

a. intrínseco – longitudinal inferior


b. extrínseco – palatogloso
c. extrínseco – transverso
d. extrínseco – estilogloso

27. Una de las funciones del músculo señalado es:


(Unidad 2, sesión 8, logro 3: Describir el Piso de la boca:
estructuras blandas que la conforman)

a. deprimir la lengua
b. elevar el paladar blando
c. deprimir el hioides cuando la mandíbula está fija
d. deprimir la mandíbula cuando el hioides está fijo

28. Paciente varón de 30 años es evaluado por probable enfermedad de Chagas, cursa
con problemas de motilidad del colon; los estudios de biopsia determinan
ausencia de células ganglionares. Según el gráfico
¿cuál es la capa en la que se determina la ausencia de dichas células?
(unidad 1, sesión 2, logro 1: describir las generalidades de la estructura del tubo digestivo:
esófago, estómago intestino delgado y grueso)

a. Mucosa - 1
b. Muscular propia – 1
c. Muscular de la mucosa - 2
d. Muscular propia - 3

1 2

3
29. Paciente varón de 32 años, que acude a centro de salud por presentar de forma
progresiva desde hace 1 año dificultad para ingerir alimentos sólidos y luego líquidos;
refiere regurgitaciones alimentarias y marcada pérdida de peso (15 kilos). Radiografía
baritada (sustancia de contraste) de esófago se muestra en la figura. El presente caso
se explica por……………….
(Unidad 2, sesión 12, logro 4: Identificar y describir la función de los esfínteres esofágicos)

a. aumento de la peristalsis esofágica


b. relajación incompleta del esfínter pilórico
c. relajación incompleta del esfínter esofágico inferior
d. perdida de producción de PIV y
óxido nítrico en el esfínter
esofágico superior

30. En condiciones normales, el ingreso de 600 ml de líquido es el estómago provoca un


aumento de presión intragástrica de unos 12 cm de H 2O. Después de una vagotomía
(corte del nervio vago) es de esperar que el ingreso del mismo volumen de líquido
ocasionede la presión intragástrica.
(Unidad 2, sesión 13, logro 1: Describe la Motilidad gástrica: relajación receptiva)

a. la disminución
b. la no variación
c. un aumento mayor
d. un aumento similar o igual
SISTEMA DIGESTIVO (ME 154) CLAVES
EXAMEN FINAL
Ciclo 201900

1. Un niño de 2 años es llevado a la consulta por diarrea persistente, edema de las


extremidades y falta de crecimiento en relación a su edad. Los análisis de sangre revelan que
tiene concentración plasmática baja de proteínas (hipoproteinemia). Como parte del estudio
se coloca colecistokinina (CCK) endovenosa y se recoge muestras del líquido duodenal por
endoscopía; el resultado del líquido confirma incapacidad para hidrolizar proteínas a un pH
neutro, esta situación mejora al añadir una pequeña cantidad de tripsina. El paciente
probablemente esté sufriendo la falta congénita de ………….
(Unidad 4, sesión 26, logro 2: Explicar la Digestión y absorción de las proteínas y sus alteraciones)
a. PEPT-1
b. pepsinógeno
c. enterocinasa
d. carboxipeptidasas

2. Paciente mujer de 35 años acude a consulta por sensación de sequedad y lesiones en


cavidad oral. Al examen se observa atrofia de la mucosa, fisuras y úlceras; nota además
sequedad e irritación de la córnea y aumento del tamaño de las glándulas parotídas. Su
diagnóstico más probable es artritis reumatoide; el hallazgo más probable en una biopsia
de glándula parótida es……..….
(Unidad 3, sesión 18, logro 3: Describe las enfermedades más frecuentes de las glándulas
salivales)
a. Presencia de acinos normales con hiperplasia de células ductales
b. Gran infiltración de linfocitos y células plasmáticas
c. Hiperplasia de acinos glandulares serosos
d. Gran infiltrado de linfocitos y neutrófilos

3. Un hombre de 42 años de edad se presenta al médico con una historia de 1 año de


evolución, caracterizado por dolor abdominal bajo y diarreas con crisis sanguinolentas.
Manifiesta además pérdida de peso de 8 kg durante este periodo. La colonoscopía revela
lesión difusa en el colon con afectación del recto. La biopsia de estas lesiones revela
adelgazamiento de la pared, inflamación y ulceración de la mucosa y sub mucosa. El
diagnóstico más probable en este caso es:
(Unidad 4, sesión 28, logro 5: Describe la Enfermedad inflamatoria intestinal.
Generalidades, morfología y características)
a. sindrome de colon irritable
b. enfermedad de Crohn
c. colitis ulcerativa
d. sprue celiaco

4. Dos estudiantes deciden tomar un receso para comer una hamburguesa a la hora del
almuerzo. Antes de llegar a la cafetería, impulsos nerviosos provenientes del complejo
vagal dorsal iniciarán la secreción de ácido gástrico por la liberación dedesde el sistema
nervioso entérico.
(Unidad 3, sesión 20, logro 2: Regulación de la secreción gástrica: estimulación, fases de la
secreción)
a. Serotonina
b. Colecistoquinina
c. Péptido inhibidor vaso activo
d. GRP (péptido liberador de gastrina)

5. Un niño de cuatro años de edad es llevado a la consulta por cuadros diarreicos


frecuentes caracterizados por heces pálidas, voluminosas y fétidas; al examen físico
presenta bajo peso y talla para la edad. Se mide la concentración de cloruro en el
sudor y se encuentra que sus valores son muy elevados. La alteración más
importante a nivel de células ductales del páncreas tiene relación directa con la conductancia
de…………
(Unidad 3, sesión 23, logro 5 Explica la Secreción pancreática: formación del jugo
pancreático, influencia de la velocidad de flujo y regulación)
a. Bicarbonato
b. Potasio
c. Sodio
d. Cloro
6. Se evalúa los valores séricos de las siguientes sustancias a un paciente con enfermedad
hepática terminal; en este paciente se espera encontrar la combinación con la letra …………
(Unidad 3, sesión 22, logro 5: Describe las Pruebas de función hepática, la Insuficiencia
hepática, encefalopatía hepática e hipertensión portal)
Glucosa Amoniaco Albúmin
a
a. Aumenta Disminuida Disminui
da da
b. Disminui Aumentada Aumenta
da da
c. Aumenta Aumentada Aumenta
da da
d. Disminui Aumentada Disminui
da da

7. Una mujer de 35 años de edad HIV positiva, se presenta al médico con dolor abdominal
en cuadrante superior derecho e ictericia. La paciente refiere haber tenido múltiples
episodios de ictericia durante los últimos 10 años. Los exámenes para determinar
hepatitis viral, dieron positivos para Hepatitis B, siendo catalogado el caso como
hepatitis crónica con alteración funcional. En un examen de sangre ¿cuál de los
siguientes parámetros está disminuido?
(unidad 3, sesión 22, logro 5: Pruebas de función hepática, Insuficiencia hepática,
encefalopatía hepática e hipertensión portal)
a. Albumina
b. Bilirrubina
c. Fosfatasa alcalina
d. Tiempo de protrombina

8. En el reflejo peristáltico del intestino delgado ¿Cuál de los siguientes eventos sucede en la
porción caudal del bolo alimenticio?
(Unidad 2, sesión 13, logro 4: Explicar la Motilidad del intestino delgado: Contracciones
segmentarias y peristálticas)
a. Acción del péptido inhibidor vasoactivo (VIP) en el músculo circular
b. Acción del NO (óxido nítrico) en el músculo longitudinal
c. Contracción del músculo longitudinal interno
d. Acción de acetilcolina en el músculo circular

9. Un varón de 58 años de edad con enfermedad de Crohn severo fue sometido a una
resección ileal. Después de la cirugía este paciente padecerá de esteatorrea, esto se
explica porque …..………..
(unidad 4, sesión 26, logro 4: Explica las alteraciones en la Absorción de lípidos)
a. se inhibe la acción de la 7 alfa hidroxilasa
b. el pool de ácidos biliares se incrementa
c. hay mala absorción de ácidos biliares
d. el páncreas no secreta lipasa

10. En un experimento se inserta un balón en el estómago de un voluntario, se infla poco


a poco mientras que se vigilan las presiones intraluminales. Aunque el volumen del
balón aumenta considerablemente, las presiones permanecen constantes. Esta
relación volumen-presión se explica por la liberación local de …………..
(Unidad 2, sesión 13, logro 1 Explica la Motilidad gástrica: relajación receptiva)
a. acetil colina y gastrina
b. norepinefrina y óxido nítrico
c. colecistoquinina y óxido nítrico
d. óxido nítrico y péptido inhibidor vasoactivo

11. ¿Cuál de las siguientes alternativas es una característica de la secreción exocrina del páncreas?
(Unidad 3, sesión 23, logro 5: Secreción pancreática: formación del jugo pancreático,
influencia de la velocidad de flujo y regulación)
a. Es hipotónica respecto al plasma
b. Su mayor estímulo se da en la fase intestinal
c. Es estimulada por la presencia de bicarbonato en el duodeno
d. La secreción enzimática es estimulada principalmente por la secretina
12. Las estructuras en el hígado que permite que los productos metabólicos unidos a
proteínas tengan acceso a las membranas basolaterales de los hepatocitos, son…..
(Unidad 3, sesión 21, logro 4-5: Explica la Organización micro estructural del hígado)
a. los canalículos
b. las células de Ito
c. las fenestras sinusoidales
d. las uniones intercelulares herméticas

13. La composición de la bilis es modificada conforme fluye por los conductillos biliares.
Durante este tránsito se espera que aumente la concentración de…….
(Unidad 3, sesión 22, logro 2: Describe la Secreción biliar, visión general del sistema biliar
extrahepático y composición de la bilis)
a. Ig A
b. Glucosa
c. Protones
d. Vitamina A

14. Se mide experimentalmente el contenido gástrico de dos personas. La persona “A” tiene
alto contenido de grasa y la persona “B” tiene un contenido isotónico ¿Cuál de las
siguientes es correcto respecto al vaciamiento gástrico? (Unidad 2, sesión 13, logro 2:
Describe la Motilidad y vaciamiento gástrico)
a. Hay ralentización del vaciado gástrico solo en “A”
b. El vaciamiento gástrico es más rápido en ambos
c. Hay ralentización del vaciado gástrico solo en “B”
d. Hay ralentización del vaciado gástrico en ambos casos

15. El examen endoscópico de un paciente con hipertensión portal grave revela venas
tortuosas que sobresalen hacia la luz del esófago. El paciente recibe tratamiento
quirúrgico mediante la colocación de una derivación que conecta la vena porta a la vena
cava. Después de la operación el riesgo de encefalopatíay el riesgo
de sangrado de várices ……………..
(Unidad 3, sesión 22, logro 5: Describe la Insuficiencia hepática, encefalopatía hepática e
hipertensión portal)
a. disminuirá / disminuirá
b. disminuirá / aumentará
c. aumentará / disminuirá
d. aumentará / aumentará

16. Un paciente varón de 18 años de edad acude al médico para sus exámenes de rutina. Sus
resultados de laboratorio muestran un valor de bilirrubina sérica de 4 mg/dl y una
bilirrubina directa de 0,3 mg/dl. Las pruebas de función hepática son normales. La
alteración que explica mejor este caso es por la deficiencia de ………………..
(Unidad 3, sesión 22, logro 3: Explica la Producción y excreción de bilirrubina. Tipos de bilirrubina e
ictericia)
a. transaminasas
b. hemo oxigenasa
c. la 7 alfa hidroxilasa
d. glucuronil transferasa

17. Un hombre de 57 años de edad es llevado a urgencias con hematemesis masiva rojo
brillante, a su llegada se halla inconsciente con PA: 80/40 mm Hg y FC: 124 lat/min. Luce
ictérico con presencia de “arañas vasculares en el tórax anterior y extremidades”,
abdomen distendido con signo de oleada positiva. Se encuentra esplenomegalia y pérdida
de la masa muscular en extremidades. La anastomosis vascular responsable del sangrado
en este paciente es ………….…..
(Unidad 3, sesión 21, logro 2: Describe las anastomosis porto sistémicas)
a. vena gástrica izquierda y vena ácigos
b. arteria gástrica izquierda y vena ácigos
c. vena paraumbilical y vena epigástrica inferior
d. vena gástrica izquierda y vena esofágica superior
18. Un estudiante de medicina está comiendo un plato de comida a base de champiñones,
espárrago y salsa de soya. El estímulo del sabor umami contenido en todos estos
alimentos viaja a través del nervio………………..
(Unidad 2, sesión 10, logro 3: Describe la irrigación e inervación de la lengua)
a. Lingual
b. Hipogloso
c. Glosofaringeo
d. Cuerda del tímpano
19. Una paciente de 30 años de edad es sometida a una cirugía en oído medio derecho
por un problema de otoesclerosis. Luego de la cirugía refiere alteración sensitiva de
la lengua. Al evaluar el caso usted esperaría encontrar……….
(Unidad 2, sesión 10, logro 5: Describe la Irrigación e inervación de la lengua)
a. Alteración en la sensación del dolor y temperatura en el tercio posterior de la lengua
b. Alteración en la sensación del dolor en los dos tercios anteriores de la lengua
c. Alteración en la sensación del gusto en el tercio posterior de la lengua
d. Sensación del dolor, tacto y temperatura conservadas

20. En un paciente de 45 años de edad con colestasis biliar, se encuentra una elevación de los
niveles sanguíneos de fosfatasa alcalina hasta 3 veces la cifra normal. ¿Cuál de las
siguientes alternativas estará también elevada como evidencia del daño de la vía biliar?
(Unidad 3, sesión 22, logro 5: Pruebas de función hepática, Insuficiencia hepática,
encefalopatía hepática e hipertensión portal)
a. Tiempo de protrombina y albúmina sérica
b. Transaminasas hepáticas (ALT y AST)
c. Gamma glutamil transpeptidasa
d. Glucoronil transferasa

21. Experimentalmente se incrementa la velocidad de la secreción salival con una


sustancia, en el análisis de la composición de esta saliva obtenida se espera
encontrar…………..
(Unidad 3, sesión 17, logro 5 : Explica la Influencia de la velocidad del flujo salival en la
composición de la saliva)
a. disminución de la concentración de bicarbonato que supera la concentración plasmática
b. aumento de la concentración de cloro y sodio que supera la concentración plasmática
c. aumento de la concentración de bicarbonato que supera la concentración plasmática
d. disminución de concentración de potasio y bicarbonato

22. Lactante de 3 meses de vida es atendido por presentar diarrea, se administra una
solución de glucosa y electrólitos por vía oral. La proteína de membrana apical
que explica la capacidad de esta solución para proporcionar aporte de glucosa e
hidratación es ………..
(Unidad 4, sesión 26, logro 1: Explica la Digestión y Absorción de los hidratos de carbono.
Alteraciones)
a. CFTR
b. SGLT-1
c. GLUT-2
d. GLUT-5

23. Paciente ha sufrido herida de bala en el abdomen, se le ha tenido que extirpar el


segmento medio y distal del ileon. En este caso la síntesis hepática de sales biliares
estará …..…..
(Unidad 3, sesión 22, logro 4: Explica la formación, función y Circulación entero hepática de lasa
sales biliares)
a. Sin cambios en el ritmo de síntesis
b. Disminuida por inhibición de la enzima colesterol 7 alfa hidroxilasa
c. Incrementada por estímulo de la enzima colesterol 7 alfa hidroxilasa
d. Incrementada por inhibición de la enzima colesterol 7 alfa hidroxilasa

24. Un varón de 75 años ingresa al consultorio por presentar ictericia marcada de piel y las
escleras. El estudio del paciente mostró que presentaba un tumor que obstruía la
totalidad del conducto hepático común. ¿Cuál de los siguientes conductos se encontrará
dilatado en este paciente?
(Unidad 3, sesión 21, logro 6: Describir el árbol biliar intrahepático)
a. de Wirsung
b. de Hering
c. colédoco
d. cístico
25. Correlaciones las dos columnas y marque la fórmula correcta:
(Unidad 4, sesión 28, logro 1: Diarrea: definición, mecanismos: osmótica, secretoria y exudativa)

1. Enfermedad Hirschsprung( ) heces con moco y sangre


2. Diarrea osmótica( ) intolerancia a lactosa
3. Diarrea secretoria( ) aganglionosis congénita
4. Diarrea exudativa( ) canales de Cl- en las células de la cripta a.-

4231b.- 1234c.- 2143d.- 4213

26. La vena umbilical obliterada del hígado después del nacimiento se transforma en el ligamento:
(Unidad 3, sesión 21, logro 1: Hígado: relación con la pared abdominal, caras, lóbulos, ligamentos
, hilio hepático)
a. cruzado
b. redondo
c. coronario
d. falciforme

27. Llega a su guardia nocturna una madre que trae a su RN masculino de 2 semanas de vida
con mal estado general y sequedad de mucosas. Usted observa que lacta ávidamente, pero
a las 2 horas presenta vómito postprandial no bilioso en proyectil. Al realizar la historia
clínica, descubre que el lactante recibió profilaxis con macrólidos para tos ferina. Usted
sospecha principalmente en:
(Unidad 1, sesión 4, logro 4: Desarrollo y anomalías del intestino anterior)
a. estenosis pilórica hipertrófica congénita
b. fistula traqueo esofágica
c. estenosis duodenal
d. atresia duodenal

28. En la regulación del apetito y la saciedad, la estimulación experimental crónica del


núcleo ventro medial del hipotálamo producirá:
(Unidad 1, sesión 3, logro 4: explica los mecanismos de control del apetito y saciedad)
a. afagia
b. obesidad
c. hiperfagia
d. activación de neuronas relacionadas a NPY

29. Paciente mujer de 25 años acude por dolor en fosa ilíaca derecha que empeora al toser o
caminar, asociada a náuseas y vómitos por lo cual acude a emergencia. Dos días después
de realizarle una apendicectomía, la paciente desarrolla fiebre alta (39 °C), está hipotensa y
presenta dolor abdominal. La laparotomía exploratoria muestra un gran volumen de
sangre en la cavidad peritoneal por lesión de un vaso producida durante la
apendicectomía.
¿Cuál de las siguientes arterias debe ligarse para detener la hemorragia?
(Unidad 4, sesión 27, logro 4: Irrigación arterial del colon, recto y conducto anal)
a. cólica derecha y arteria rectal superior.
b. ileocólica y arteria cólica media.
c. mesentérica superior.
d. ileocólica.

30. La onda peristáltica secundaria del esófago se caracteriza por ser originada ………
(unidad 2, sesión 12, logro 3: Motilidad esofágica: fases y características)
a. por el plexo de meissner del esófago
b. por el plexo mientérico del esófago
c. por el reflejo de la deglución
d. durante la masticación

31. ¿Cuál de los siguientes es una causa de ictericia con bilirrubina conjugada aumentada?
(Unidad 3, sesión 22, logro 3: Producción y excreción de bilirrubina. Tipos de bilirrubina, ictericia)
a. Ictericia del recién nacido
b. Obstrucción del colédoco
c. Anemia hemolítica
d. Gran hematoma
32. En relación a la absorción de nutrientes, la absorción de dipéptidos y tripéptidos a nivel
de las células epiteliales del intestino delgado, se da principalmente debido a:
(Unidad 4, sesión 26, logro 2: Digestión y absorción de las proteínas. Alteraciones)
a. el incremento de los canales de Cl- en la membrana apical
b. la gradiente de bicarbonato en la membrana basal
c. la gradiente de iones H+ en la membrana apical
d. la gradiente de Na+ en la membrana apical

33. Paciente de 20 años es traído a la emergencia por presentar diarreas desde hace 2 días.
Familiar refiere que las deposiciones son líquidas y abundantes, al examen luce
deshidratado y se plantea que la diarrea es producida por una toxina que estimula la
transformación de ATP a AMPc con apertura de canales de Cl- y pérdida de agua. El tipo de
diarrea más probable es:
(Unidad 4, sesión 28, logro 1: Diarrea: definición , mecanismos: osmótica, secretoria y supurativa)
a. osmótica
b. exudativa
c. secretoria
d. por intolerancia a lactosa

34. Un niño fue operado por una obstrucción intestinal, observándose la presencia de
divertículo de Meckel. Según lo referido, marque lo correcto:
(Unidad 1, sesión 5, logro 2: identifica las anomalías del desarrollo del intestino medio: onfalocele
y gastrosquisis (diferencias), Divertículo de Meckel)
a. el 50% de la población lo presenta
b. se localiza en el íleon muy cerca al yeyuno
c. puede poseer tejido gástrico o pancreático
d. se produce por una mala rotación de los intestinos

35. Marque la
alternativa correcta
respecto a la
estructura marcada
en el gráfico:
(Unidad 3, sesión 22,
logro 2: Secreción biliar.
Visión general del
sistema biliar
extrahepático y
composición de la bilis)

a. Se halla a 2
centímetros
debajo de la
papila
duodenal
mayor
b. Llega el conducto
colédoco y
pancreático
principal
c. Llega el
conducto
hepático
común y
pancreático
principal
d. Llega el conducto pancreático accesorio

36. ¿Cuál de las siguientes moléculas se encontrará aumentada en el citoplasma de las


células parietales de un paciente con sindrome de Zollinguer Ellison?
(Unidad 3, sesión 20, logro 4: Enfermedad ulcerosa péptica: úlcera gástrica, duodenal.
síndrome de Zollinger – Ellison)
a. Péptido liberador de gastrina (GRP)
b. Proteína G estimulante (GS)
c. Inositol Trifosfato (IP3)
d. AMP cíclico (AMPc)

37. Los fármacos inhibidores de la bomba de protones, actúan bloqueando la ………..……..


(Unidad 3, sesión 20, logro 3: Regulación de la secreción gástrica: inhibición, Secreción de
pepsinógeno y factor intrínseco)
a. anhidrasa carbónica
b. ATPasa H+/K+ en la membrana luminal
c. ATPasa H+/K+ en la membrana basolateral
d. ATPasa Na+/K+ en la membrana basolateral
38. Un paciente fue diagnosticado de gastritis autoinmune, ¿cuál de las siguientes
alternativas es FALSA respecto a esta enfermedad?
(Unidad 3, sesión 20, logro 5: Gastritis crónica: helicobacter pylori, autoinmune. Tipos de gastritis)
a. Afecta principalmente el fondo y cuerpo gástrico
b. Se produce hiperplasia de células G secundaria a la aclorhidria
c. El propio sistema inmune destruye principalmente las células parietales
d. Se produce atrofia de la mucosa, aclorhidria, hipergastrinemia y déficit de vitamina B6

39. Marque la correlación correcta:


(Unidad 3, sesión:18, logros:1 y 2: Describe las enfermedades inflamatorias/infecciosas y
proliferativas de la cavidad oral)

1. Herpes virus( ) En relación al abuso de antibióticos


2. Candidiasis oral( ) Lesiones vesiculares como racimo de uvas
3. Eritroplaquia( ) Mega esófago
4. Enfermedad de Chagas( ) Lesión pre cancerígena

a.- 2431b.- 1234c.- 4123d.- 2143

40. En un paciente con insuficiencia renal crónica, el déficit en la absorción de calcio a nivel
del enterocito se debe a lo siguiente:
(Unidad 4, sesión 26, logro 6: Explica la Absorción de calcio y hierro)
a. No se convierte la 25 hidroxicolecalciferol a 1,25 dihidroxicolecalciferol
b. No se convierte la 1,25 dihidroxicolecalciferol a 25 hidroxicolecalciferol
c. Existe un descenso de la alfa 25 hidroxilasa renal
d. Se incrementa la producción de Calbindina
SISTEMA DIGESTIVO (ME154) EXAMEN PARCIAL 2019 01

Profesores : Alfaro Salazar, Herberth Romulo; Callata Caceres, Gunter; Cayo Quiñe, Alexandra Mariel;
Correa Borit, Jorge Mauricio; Cruz Cutty, Lourdes Marylin; Guzmán Calderón, Gerly Edson;
Jáuregui Farfán, Jorge Jesús; Mayor Zevallos, Otto Alberto; Montoya Suárez, José Luis;
Palacios Bazan, Enrique Elias; Robles Pino, Alexander Anibal; Wong Bravo, Juan Carlos
Sección : Todas las secciones
Duración : 50 minutos.
Indicaciones:
Lea atentamente cada pregunta antes de responder:
- Se prohíbe el uso del celular y cualquier dispositivo electrónico.
- Está prohibido intercambiar materiales.
- Coloque su código de alumno en la tarjeta de respuestas. Si su código contiene una letra
reemplácela por un valor numérico siguiendo la siguiente equivalencia: A=9, B=8, C=7, D=6, E=5, F=4,
G=3.
- Traslade sus respuestas a la tarjeta, llenando los círculos de manera completa con lapicero negro o
azul. Está prohibido el llenado con lápiz, lapicero de otro color o con lapicero de tinta borrable.
- Sea cuidadoso en el llenado de la tarjeta de respuestas, pues solo esta tiene validez para la
calificación.
- Al terminar su examen avise al docente a cargo, no se levante de su sitio; debe entregar la hoja de
respuestas con la carátula del examen, este cuadernillo de preguntas se lo llevará cada estudiante.

1. ¿Cuál de las glándulas salivales es responsable del mayor porcentaje del volumen de la saliva en
condiciones basales?
a. Parótida
b. Sub
pala
tina
s c.
Subl
ingu
laes
d.
Sub
max
ilare
s

2. La lengua está recubierta por epitelio:


a. pseudoestratificado columnar no queratinizado
b. plano estratificado no queratinizado
c. pseudoestratificado columnar ciliado
d. plano estratificado queratinizado

3. El esfínter anal interno tiene musculatura …………… y tiene control ………………….


a. lisa / voluntario
b. lisa / involuntario
c. esquelética / simpático
d. esquelética / parasimpático

4. La arteria aorta proporciona la irrigación al tubo digestivo ¿cuál de las siguientes arterias
proporciona la irrigación al ángulo cólico derecho?
a. Mesentérica superior
b. Mesentérica inferior
c. Frénica inferior
d. Tronco celiaco

5. Paciente de 26 años que le cuenta en su historia clínica que cada vez que almuerza, a los 20
minutos tiene deseo de defecar. Le comenta que su hijo de 1 mes le pasa lo mismo pero más
intenso. Esto se explica por el reflejo
…………………, el cual estáen el paciente.
a. colicoileal / normal
b. colicoileal / alterado
c. gastrocolico / normal
d. gastrocolico / alterado
6. La región del estómago que se comunica con el duodeno se denomina:
a. pilórica
b. cardias
c. cuerpo
d. fórnix

7. Acude a consulta un paciente que fue diagnosticado de ulcera péptica 3 días antes. Luego de múltiples
pruebas diagnósticas, se concluye que el paciente presenta un tumor secretor de gastrina ¿Cuál de las
siguientes situaciones estará incrementada?
a. Distención gástrica
b. Inhibición del vaciado gástrico
c. Secreción de ácido clorhídrico (HCl)
d. Inhibición de la secreción de pepsinógeno

8. En el sistema digestivo, el control del apetito esta dado por un complejo sistema de sustancias y
órganos integradores los cuales regulan la ingesta de alimentos. Laes una sustancia oroxígena y es
sintetizada
por el ……………………..
a. leptina / intestino
b. grelina / intestino
c. leptina / estómago
d. grelina / estómago

9. Sobre el control autónomo del sistema digestivo, marque la alternativa correcta:


a. La inervación dada por el sistema simpático es de tipo preganglionar.
b. El sistema parasimpático usa como neurotransmisores a la acetilcolina y la noradrenalina.
c. El nervio vago (par craneal X) le da inervación simpática a la mayoría del sistema digestivo.
d. En el sistema simpático, los nervios responsables hacen una primera sinapsis en
ganglios próximos al órgano a inervar.
e. En la inervación de tipo parasimpático, solo interviene el plexo submucoso, sin
embargo, en la de tipo simpático intervienen tanto el submucoso como el mientérico.

10. Con respecto a la actividad eléctrica del sistema digestivo, marque la alternativa correcta
a. Corresponden a potenciales de acción que están presentes de forma continua y le dan la
capacidad de perístasis autónoma al sistema digestivo.
b. La frecuencia de las ondas lentas no se ve influenciada por la actividad neural
ni las hormas gastrointestinales.
c. En el estómago las ondas lentas se dan en una frecuencia de 6 por minuto.
d. Las ondas lentas son cambios lentos y ondulantes del potencial en reposo.
e. La frecuencia de las ondas lentas va de 6 a 12 ondas por minuto.

11. Ante una lesión del IX par craneal, el músculo…se altera en su función.
a. palatogloso
b. estilofaríngeo
c. palatofaríngeo
d. constrictor superior

12. Un varón de 50 años es sometido a extirpación de duodeno y parte proximal de yeyuno. La


pérdida de estímulo hormonal en el páncreas para la secreción enzimática se explica por la
pérdida de las células ……………………
a. Parietales, productoras de factor intrínseco
b. “K” productoras de factor intrínseco
c. “M” productoras de CCK
d. “I” productoras de CCK
13. Respecto al mecanismo de la defecación ¿Cuál de las siguientes afirmaciones es correcta?
a. Se produce contracción refleja del esfínter anal interno
b. Se produce contracción o relajación del esfínter anal externo por señales de la corteza
cerebral
c. La presencia de materia fecal en el recto estimula la contracción del sigmoides por los
nervios pélvicos simpáticos
d. En la posición de “cuclillas” el músculo puborectal se halla contraído favoreciendo la
evacuación de la materia fecal

14. Un niño de tres años llega a emergencia con disfagia (dificultad para tragar), salivación y llanto. Se
sospecha de ingesta de cuerpo extraño: moneda en el esófago; al ser evaluado se constata en una
radiografía presencia de cuerpo extraño a nivel de C6 y C7 (6° y 7° vértebra cervical). El cuerpo extraño
estará suspendido a nivel del estrechamiento producido por el ………..
a. cayado aórtico
b. hiato esofágico
c. músculo cricofaríngeo
d. bronquio principal izquierdo

15. En el caso de un paciente con un tumor productor de gastrina, la presencia de úlceras duodenales y
erosión de la mucosa gástrica se debe principalmente a…….
a. la acción paracrina de la gastrina sobre la célula parietal
b. el exceso de HCl por estímulo de receptores CCK-B en la célula parietal
c. la sobre expresión de los receptores “G” para gastrina en la célula parietal
d. el exceso de HCl por estímulo directo de receptores “H” en la célula parietal

16. La onda peristáltica secundaria del esófago se caracteriza por ser originada ………
a. por el plexo de submucoso del esófago
b. por el plexo mientérico del esófago
c. por el reflejo de la deglución
d. durante la masticación

17. Marque lo correcto sobre las ondas lentas en el tubo digestivo


a. No son despolarizaciones
b. Son potenciales de acción subumbrales
c. Se constituyen de despolarizaciones y repolarizaciones
d. Son rítmicas y generadas por el sistema nerviosos autónomo

18. Recién nacido que presenta protrusión de contenidos abdominales los cuales no están cubiertos
por peritoneo y salen de la cavidad abdominal a través de un defecto de la pared. ¿Cómo se
denomina a la afección que presenta este paciente?
a. Onfalocele
b. Atresia biliar
c. Gastrosquisis
d. Divertículo de Meckel

19. Experimentalmente se utiliza atropina (anticolinérgico) para inhibir la secreción de gastrina, sin
embargo, la secreción de esta hormona se sigue dando ante estímulos vagales. Esta situación se
explica porque la atropina:
a. no bloquea la acción del péptido GRP
b. solo inhibe la acción del péptido GRP en la célula G
c. inhibe la acción de acetilcolina e histamina en la célula G
d. bloquea parcialmente la bomba de protones en la célula G

20. Niña de 4 días es llevada a la emergencia pediátrica por presentar llanto constante, la madre refiere
coloración azulada de labios al momento de lactar, acompañado de tos persistente y dificultad
respiratoria así como distención abdominal. Se le coloca sonda nasogástrica para alimentación
notando que retorna a la cavidad oral en todos los intentos. ¿Cuál es la anomalía del desarrollo en
este caso?
a. Solo fístula traqueo esofágica
b. Fístula traqueo esofágica proximal y distal
c. Atresia esofágica proximal con fístula traqueo esofágica distal
d. Atresia esofágica distal con fístula traqueo esofágica proximal
21. Paciente varón de 36 años es traído a la emergencia luego de sufrir un accidente de tránsito, presenta
traumatismos múltiples en cabeza y tronco. Al examen físico se evidencia hematoma en hemicara
izquierda, ligera protrusión y caída del lado izquierdo del maxilar inferior, por lo que se realiza una
tomografía donde se halla una fractura de la apófisis coronoides del maxilar inferior. ¿Qué músculo
está relacionada directamente con esta situación?
a. Masetero
b. Temporal
c. Buccinador
d. Pterigoideo medial

22. Un paciente refiere no percibir algunos sabores. Al examen físico constata alteración del sabor dulce y
umami.
¿Cuál de los siguientes nervios estará alterada su función?
a. Cuerda del tímpano (VII par)
b. Lingual (rama del V par)
c. Glosofaríngeo (IX par)
d. Hipogloso (XII par)

23. A los pocos días de nacido, regresa a neonatología un niño con problemas de motilidad del colon; los
estudios determinan ausencia congénita de células ganglionares. Según el gráfico ¿cuál es la capa en
la que se determina la ausencia de dichas células?
a. Mucosa - 2
b. Muscular propia - 2
c. Muscular propia - 3
d. Muscular de la mucosa - 3

1 2

24. Con respecto al control autonómico en el tracto gastrointestinal y en relación a su fisiología. ¿Cuál
es la función del sistema nervioso parasimpático en el tracto gastrointestinal?
a. Inhiben la contracción muscular y estimulan la secreción de sustancias a nivel de la
submucosa
b. Estimulan la contracción muscular y estimulan la secreción de sustancias a nivel de la
mucosa
c. Inhiben la contracción muscular e inhiben la secreción de sustancias a nivel de la submucosa
d. Estimulan la contracción muscular e inhiben la secreción de sustancias a nivel de la mucosa

25. Un estudiante que está preocupado por su examen parcial, no ha desayunado ni almorzado; cuando
al fin ingiere alimentos, esto le provoca el aumento de los movimientos musculares del tracto
gastrointestinal y la sensación de defecar. ¿Qué reflejo se ha activado?
a. Entero-gástrico
b. Gastro-cólico
c. Cólico-ileal
d. Ileo-ileal
26. ¿De qué par craneal es rama el nervio palatino mayor?
a. Vago
b. Hipogloso
c. Trigémino
d. Palatogloso

27. ¿En cuál de las fases de la deglución la epiglotis separa la vía respiratoria de la digestiva?
a. oral
b. laríngea
c. faríngea
d. esofágica

28. Los péptidos intestinales se pueden clasificar como sustancias endocrinas, neurocrinas y paracrinas,
dentro de las paracrinas se encuentran la somastotatina e histamina. Marque la respuesta correcta
a. La somastotatina es sintetizada por las células B de la mucosa gástrica
b. La histamina actúa estimulando su receptor tipo H1 en la mucosa gástrica
c. La histamina es sintetizada por células de tipo paracrino de las glándulas gástricas
d. La somatostatina presenta dentro de sus funciones la estimulación de la secreción de H +

29. En relación a los órganos intraabdominales y sus estructuras de fijación, elija la


alternativa correcta a.El mesenterio permite la suspensión e irrigación de los
órganos retroperitoneales
b. Tanto el hígado como la vesícula biliar se encuentran ubicados a nivel del flanco derecho
c. El colon, el duodeno y el resto de intestino delgado son órganos considerados
netamente como peritoneales
d. Los ligamentos que encontramos dentro de la cavidad abdominal son el esplenorenal y el
gastrofrénico e.Los omentos van desde el estómago y la segunda porción del duodeno a otras
estructuras
intraabdominales y existen dos: el omento mayor y el omento menor

30. Paciente varón de 27 años es llevado por bomberos a emergencia luego de ser asaltado y, tras
resistirse, es cortado con el pico de una botella a nivel abdominal. Al examen físico usted observa que
a través de la herida se puede observar la protrusión de asas intestinales. En relación con las capas de
la pared abdominal, marque la alternativa correcta.
a. La fascia de Scarpa está constituida principalmente por tejido adiposo
b. La pared abdominal está formada por piel, huesos, músculos, fascias y peritoneo parietal
c. La fascia de Camper es una estructura fibrosa que carece de grasa y su grosor es
constante en toda la pared abdominal
d. El músculo oblicuo externo discurre en dirección súpero-interna y se inserta en el borde
inferior de las ultimas 3 a 4 costillas
e. El músculo recto del abdomen tiene como funciones comprimir el contenido del
abdomen, tensar la pared del abdomen y flexionar la columna
SISTEMA
DIGESTIVO
(ME154)
EXAMEN FINAL
2019 01

Profesores : Alfaro Salazar, Herberth Romulo; Callata Caceres, Gunter; Cayo Quiñe, Alexandra Mariel;
Correa Borit, Jorge Mauricio; Cruz Cutty, Lourdes Marylin; Guzmán Calderón, Gerly Edson;
Jáuregui Farfán, Jorge Jesús; Mayor Zevallos, Otto Alberto; Montoya Suárez, José Luis;
Palacios Bazan, Enrique Elias; Robles Pino, Alexander Anibal; Wong Bravo, Juan Carlos
Sección : Todas las secciones
Duración : 70 minutos.
Indicaciones:
Lea atentamente cada pregunta antes de responder:
- Se prohíbe el uso del celular y cualquier dispositivo electrónico.
- Está prohibido intercambiar materiales.
- Coloque su código de alumno en la tarjeta de respuestas. Si su código contiene una letra
reemplácela por un valor numérico siguiendo la siguiente equivalencia: A=9, B=8, C=7, D=6, E=5, F=4,
G=3.
- Traslade sus respuestas a la tarjeta, llenando los círculos de manera completa con lapicero negro o
azul. Está prohibido el llenado con lápiz, lapicero de otro color o con lapicero de tinta borrable.
- Sea cuidadoso en el llenado de la tarjeta de respuestas, pues solo esta tiene validez para la
calificación.
- Al terminar su examen avise al docente a cargo, no se levante de su sitio; debe entregar la hoja de
respuestas con la carátula del examen, este cuadernillo de preguntas se lo llevará cada estudiante.

1. La explicación fisiológica de presentar somnolencia de 30 minutos a 1 hora después de ingerir


alimentos, se explica por:
a. Aumento del cloro intraluminal
b. Aumento del bicarbonato intraluminal
c. Disminución de ácido carbónico en la célula parietal
d. Disminución de la actividad de la anhidrasa carbónica
e. Aumento de la alcalinidad sanguínea

2. Con respecto a la irrigación arterial del colon, a que arteria


corresponde la señalada con la flecha
a. Cólica derecha
b. Cólica media
c. Cólica izquierda
d. Ileobisecoapendículocólica
e. Arco de Riolano

3. Si un paciente presentara dentro del punto de vista fisiológico, una


disminución de enterocinasa, entonces esto originaría una
disminución de la actividad de:
a. L
a
p
e
p
s
i
n
a
b
.l
a
li
p
a
s
a
c. la quimotripsina
d. el peptido insulinotropo dependiente de glucosa
e. la amilasa

Se valida la opción b debido a su relación con la colipasa.


4. Con respecto a la anatomía del hígado, señale a que
estructura pertenece la marcada por el número 1.
a. Ligamento falciforme
b. Línea de Cantlie
c. Ligamento triangular
d. Ligamento coronario
e. Ligamento teres

Se valida la opción a debido a la ubicación del número 1 en


donde se unen el ligamento falciforme y ligamento coronario.

5. Se presenta un paciente, el cual presenta un antecedente de tuberculosis intestinal, por lo cual,


se le resecó 80 cm de íleon distal. Desde el punto de vista fisiológico, el paciente puede presentar
una de las siguientes alteraciones:
a. Disminución de la secreción de Vitamina B12
b. Aumento indiscriminado de absorción de ácido fólico
c. Disminución de la absorción de hierro
d. Aumento de la secreción de bicarbonato
e. Disminución de la absorción de ácido glicocólico

6. Un paciente es sometido experimentalmente a un fármaco que modifica el flujo salival, obteniéndose


un volumen de saliva de 288 ml en 6 horas. En este caso las concentraciones de electrolitos y
bicarbonato en la saliva obtenida varían de la siguiente manera:
a.↑ Na+, ↓ K+, ↑
Cl-, ↑ HCO3- b.↓
Na+, ↓ Cl-, ↑ K+, ↓
HCO3-
c.↑ Na+, ↑ Cl-, ↓ K+, ↓ HCO3-
d.↑ Na+, ↑ Cl-, ↑ K+, ↑ HCO3-
e.↓ Na+, ↓ Cl-, ↓ K+, ↓ HCO3-

Se valida la opción a debido a que se puede considerar como un aumento del flujo de saliva.

7. La siguiente imagen histológica corresponde a la glándula


…………… y la estructura señalada produce ………
a. salival sublingual / mucopolisacáridos
b. oxíntica / pepsinógeno
c. salival submaxilar / ptialina
d. salival parótida / amilasas
e. antrales / gastrina

8. Paciente varón de 65 años con antecedente de hipercolesterolemia, hipertensión arterial, fibrilación


auricular y dos infartos al miocardio previos, aqueja de dolor abdominal intenso de inicio súbito,
distensión abdominal, se decide cirugía con resección de 1,5 metros de intestino delgado terminal y
colon ascendente. Como consecuencia de la resección el paciente tendrá deficiencia de:
a. Vitamina C
b. Tiamina
c. Vitamina A
d. Vitamina B1
e. Vitamina B6

Se valida esta opción debido a que su absorción está relacionada al íleon.


9. Uno de los siguientes elementos debería hallarse con más probabilidad en el esófago de un paciente
que sufre de
reflujo gastro esofágico…
a. Pepsina
b. Tripsina
c. Quimiotripsina
d. Carboxipeptidasa
e. Ácidos biliares

10. Un paciente de 40 años cursa con anemia de 8g/dl, aqueja además de astenia y sensación de
hormigueo bilateral en los miembros inferiores, al examen se halla alteración de la sensibilidad a la
vibración y camina con ampliación de la base de sustentación. Uno de los siguientes procedimientos
sería de ayuda para el diagnóstico de este paciente:
a. Tomografía cerebral
b. Biopsia de la mucosa gástrica
c. Biopsia de hígado
d. Examen de sangre oculta en heces
e. Biopsia de Ileon proximal

11. Paciente de 60 años ingresa por caída hace 1 hora y pequeño hematoma en cuero cabelludo, al
examen físico ampliado se observa ictericia de piel y mucosas generalizada, abdomen blando, se
palpa estructura quística no dolorosa en hipocondrio derecho que corresponde a vesícula biliar
(signo de Courvoisier), en los exámenes de laboratorio se halla niveles bajos en la formación de
estercobilinógeno y urobilinógeno en heces, incremento de la bilirrubina conjugada en la orina,
elevación de fosfatasa alcalina y gamma glutamil transpeptidasa séricas. El presente cuadro puede
ser explicado por:
a. Reabsorción de hematoma
b. Litiasis vesicular
c. Carcinoma de la cabeza de páncreas
d. Carcinoma con estenosis del conducto hepático común
e. Anemia hemolítica

12. Paciente varón de 58 años con antecedente de alcoholismo crónico es diagnosticado y recibe
tratamiento por cirrosis hepática. Hace 2 días refiere familiar que tuvo cambio de conducta y no
reconoce a algunos familiares. Al examen físico, se halla ascitis, circulación colateral en abdomen,
telangiectasias, en el examen de sistema nervioso: rigidez de extremidades, ROT incrementados,
desorientación en el espacio y asterixis. ¿cuál de las siguientes circunstancias, explicaría el cuadro
en este paciente?
a. Uso de diuréticos ahorradores de potasio
b. Incremento de actividad de ureasa bacteriana duodenal
c. Hemorragia gastrointestinal
d. Disminución de la producción de NH3+ en el colon
e. Dieta normo proteica

Se valida la opción e debido al efecto sobre la encefalopatía.


Con respecto de la opción b es incorrecta debido a que hace referencia al duodeno, debería indicar colon.

13. Un recién nacido presenta vómitos biliosos poco tiempo después de cada alimento. Al preguntar a la
madre sobre antecedentes, ella recuerda que tuvo polihidramnios durante la gestación, pero un
análisis de cariotipo fue normal. Una de las siguientes es la causa más probable de estos hallazgos
en el recién nacido:
a. Enfermedad de Hirschprung
b. Fístula tráqueo esofágica
c. Divertículo ileal
d. Estenosis pilórica
e. Malrotación de la yema pancreática ventral

14. Un lobulillo hepático se puede dividir en tres zonas como se muestra en el gráfico. ¿Cuál de
las siguientes afirmaciones sobre las tres zonas es verdadera?
a. La zona 1 tiene los menores depósitos de glucógeno
b. La zona 3 es la primera en afectarse en una colestasis extra hepática
c. La zona 2 es más susceptible a la injuria por isquemia que la zona
periportal d.La zona 2 tiene la mayor capacidad de regeneración
e.La zona 1 es la que tiene menos actividad metabólica.

La pregunta 14 ha sido anulada, sin embargo, ningún estudiante se verá afectado


negativamente en su puntaje debido a esta anulación.

15. En un estudio de la secreción de hormonas gastrointestinales, sus concentraciones en la vena


porta se midieron durante perfusión luminal del intestino delgado con soluciones de diversas
magnitudes de pH. ¿Qué hormona aumentará en el plasma de la vena porta durante perfusión a
través del intestino con una solución de pH 3?
a. CCK
b. gastrina
c. GIP
d. motilina
e. secretina

16. Paciente de 30 años que ingresa a causa de un traumatismo abdominal cerrado. En la exploración
se aprecia discreta palidez de piel y mucosas, auscultación pulmonar normal, taquicardia de 120
/min. Discreta distensión abdominal y matidez en flancos; el hematocrito, que era prácticamente
normal al ingreso, disminuye a 30% a las tres horas. En la Rx de tórax se objetiva fractura de las
costillas 10-11 izquierdas. La causa más probable de la anemización en este paciente es:
a. traumatismo renal con hemorragia retroperitoneal.
b. rotura de hígado con hemoperitoneo.
c. rotura de bazo con hemoperitoneo.
d. rotura de mesos con hemoperitoneo.
e. traumatismo pancreático con pancreatitis traumática.

17. Mujer de 65 años. Consulta por síndrome constitucional


asociado a dolor abdominal epigástrico progresivo
irradiado a espalda, de dos meses de evolución. El
diagnostico de sospecha de adenocarcinoma de páncreas
se confirma por biopsia. Se realiza examen de imagen de
abdomen para evaluación de estructuras vasculares
próximas al tumor pancreático. ¿Cuál es el nombre de la
vena señalada que está ausente, trombosada por
infiltración tumoral, condicionando circulación colateral
en la pared gástrica?

a. Mesentérica superior
b. Coronaria estomaquica
c. Esplénica
d. Porta
e. Renal izquierda
18. Revisando la angiotomografía de un hombre de 70 años en estudio por aneurisma de aorta
abdominal, el radiólogo le informa de la presencia de una oclusión completa de la arteria
mesentérica inferior. El paciente se encuentra completamente asintomático. La oclusión de la
arteria mesentérica inferior cursa de manera asintomática en muchas ocasiones ya que el
territorio que irriga puede recibir flujo proveniente de la arteria:
a. cólica derecha
b. gastroduodenal
c. Epigástrica inferior izquierda
d. esplénica
e. cólica media

19. En las patologías de esófago es importante conocer bien la anatomía esofágica. ¿Cuál de
las siguientes afirmaciones es correcta?
a. El esófago tiene capa mucosa, muscular y serosa
b. El esófago abdominal es más largo que el cervical
c. El esófago torácico pasa por detrás del cayado aórtico
d. El epitelio esofágico normal es de tipo cilíndrico.
e. El esófago abdominal es discretamente más largo que el torácico

20. A pesar de que pueda haber variaciones anatómicas, lo habitual es que el ciego sea irrigado por una
rama arterial que proviene de unas de las siguientes arterias:
a. Iliaca derecha
b. Mesentérica inferior
c. Hepática derecha
d. Mesentérica superior
e. Iliaca izquierda

21. Ante un paciente con una cirugía abdominal urgente, el informe operatorio señala que se ha
realizado una resección de todo el duodeno y del tercio proximal del yeyuno manteniendo íntegros
el estómago y todo el íleon, así como los dos tercios distales del yeyuno. En el seguimiento
nutricional del paciente ¿Qué vitamina o mineral presentará con menor probabilidad una
disminución de su absorción?
a. Cianocobalamina
b. Calcio
c. Hierro
d. Transcobalamina
e. Transferrina

22. ¿Cuál de las siguientes alternativas detallan las venas que confluyen y forman la vena señalada?
a. mesentérica superior, gástrica izquierda y
gastroepiploica izquierda
b. mesentérica inferior, gástrica izquierda y renal
c. esplénica, mesentérica superior y mesentérica inferior
d. esplénica, pancreatoduodenal y omental izquierda
e. gástrica izquierda, esplénica y hepática común

23. ¿Cuál de las siguientes sustancias forma parte de la secreción biliar?


a. Tripsina
b. Lecitina
c. Elastasa
d. Quimotripsina
e. Pepsina
24. El tubo digestivo contiene diferentes tipos de epitelios y glándulas. La estructura señalada es unay
está localizada en el …………...
a. glándula de Brunner /
intestino grueso b.cripta de
Lieberkuhn / colon
c. cripta de Lieberkuhn / intestino delgado
d. glándula oxintica / estomago
e. célula parietal / estómago

Aunque las criptas de


Lieberkuhn están
presentes en el intestino
delgado, la
microfotografía es de
epitelio de colon.

25. ¿De qué musculo forma parte el ligamento inguinal?


a. Oblicuo externo del abdomen
b. Oblicuo interno del abdomen
c. Transverso del abdomen
d. Psoas
e. Dorsal ancho

26. Señale cuál de las siguientes afirmaciones NO se relaciona a la siguiente glándula anexa del
tubo digestivo mostrada en la imagen:
a. Es una glándula exocrina compuesta exclusivamente por acinos serosos
b. Su inervación está dada por el nervio auricular mayor (ramo posterior C2), que inerva
la vaina de la glándula así como la piel por encima de esta.
c. Esta glándula produce una secreción mucinosa acuosa, llamada mucoserosa, a través
del conducto de Wharton.
d. Su inflamación puede ser causada por un virus de los Paramyxoviridae, que provocan
una enfermedad muy frecuentemente en niños y adolescentes
e. Es una glándula endocrina y probablemente sea de origen pancreático

Se valida la opción e debido a que no


está relacionada con la imagen.

27. ¿Cuál de las siguientes enzimas está localizada en el borde en cepillo y juega un rol en la digestión de
proteínas?
a. Alfa dextrinasa
b. Pepsina
c. Enterocinasa
d. Lactasa
e. Carboxipeptidasa A.

Se valida la opción c debido a que es correcta en relación a la pregunta.


28. Una de los siguientes sustancias, NO sirve como un buen agente emulsificante:
a. Colesterol
b. Ácidos grasos
c. Sales biliares
d. Lecitina
e. Proteínas de la dieta

Se valida la opción e debido a que es correcta en relación a la pregunta.

29. La sustancia que estimula el crecimiento de la mucosa gástrica es:


a. Secretina
b. Motilina
c. Péptido estimulante de la mucosa gástrica
d. Gastrina
e. Histamina

30. ¿Cuál de las siguientes alternativas es una función de la colecistokinina?


a. Relajación de la vesícula para la
salida de bilis b.Secreción de ácidos
biliares
c. Contracción del esfinter de Oddi
d. Secreción de enzimas pancreáticas
e. Contracción del duodeno

Se valida la opción b debido al efecto de la CCK sobre la vesicula biliar.

31. Con respecto a la anatomía del tronco celiaco, señale lo correcto


a. El tronco celiaco se origina de la cara posterior de la aorta abdominal
b. Es una arteria delgada que tiene un calibre entre 2 y 3 mm
c. Una de sus ramas es la arteria gástrica derecha
d. La hepática común que es una de sus ramas, participa en la irrigación del estómago.

32. Con respecto a la anatomía del duodeno, marque la respuesta correcta:


a. Tiene una distribución en forma de “C”, que rodea la cola del páncreas
b. La 3ra porción duodenal está contenida en la pinza vascular aortomesentérica
c. Entre la 1ra y 2da porción se forma un ángulo, conocido como el ángulo de Treitz
d. La 4ta porción se dirige a la izquierda, hacia abajo y hacia atrás.
e. En la tercera porción desemboca el conducto colédoco.

33. El hígado está ampliamente tapizado por peritoneo, la estructura que conecta la cara
diafragmática del hígado precisamente con el diafragma es el ligamento:
a. teres
b. f
a
l
c
if
o
r
m
e
c
.t
ri
a
n
g
u
l
a
r
d.hepá
tico
común
e.coro
nario

Se validan la opción c y e debido a que forman parte de los ligamentos que fijan el hígado al diafragma.

34. En el íleon se absorbe aproximadamente el 95% dea través de la circulación enterohepática.


a. agua
b. colesterol
c. sales biliares
d. hidróxicobalamina
e. factor intrínseca
35. Laestimula el mecanismo paracrino de la secreción de ácido clorhídrico.
a. histamina
b. acetilcolina
c. gastrina
d. secretina
e. somatostatina

36. En la digestión de proteinas,es el principal estímulo para convertir el pepsinógeno en pepsina.


a. la gastrina
b. el pH ácido
c. la acetilcolina
d. la ptialina
e. la somatostatina

37. Con respecto a la somatostatina, marque lo correcto:


a. Es secretada por las células S del intestino
b. Induce a la producción de VIP
c. Interviene en la fase intestinal de la secreción gástrica
d. Produce acetilcolina para estimular a la célula parietal
e. No interviene en la regulación de la secreción de ácido clorhídico

38. En pecten anal, es una estructura comprendida entre:


a. la línea pectínea y los senos anales
b. la línea blanca y la apertura anal
c. el esfínter anal interno y el externo
d. la línea anocutánea y la línea pectínea
e. la línea blanca y columnas anales

39. ¿Cuál de las siguientes alternativas es una proenzima pancreática?


a. Tripsina
b. Elastasa
c. Quimotripsinógeno
d. Amilasa
e. Procarboxipepitidasa C.

40. En la segmentación hepática de Coinaud, el segmento hepático señalado con la flecha,


corresponde a : En la segmentación hepática de Coinaud, la flecha señala el
segmentohepático.
a. IV
b. V
c. VI
d. VII
e. VIII
EXAMEN
PARCIAL
SISTEMA
DIGESTIVO
(ME154)
Ciclo 2019-02

Sección:Todas
Profesores:Alfaro Salazar, Herberth Romulo; Alva Muñoz, Jose Carlos; Mayor Zevallos, Otto Alberto;
Duración:30 minutos.
Indicaciones:
- Lea atentamente cada pregunta antes de responder.
- Se prohíbe el uso del celular y cualquier dispositivo electrónico.
- Está prohibido intercambiar materiales.
- Coloque su código de alumno en la tarjeta de respuestas. Si su código contiene una letra
reemplácela por un valor numérico siguiendo la siguiente equivalencia: A=9, B=8, C=7, D=6, E=5, F=4
y G=3.
- Traslade sus respuestas a la tarjeta, llenando los círculos de manera completa con lapicero negro o
azul. Está prohibido el llenado con lápiz, lapicero de otro color o con lapicero de tinta borrable.
- Sea cuidadoso en el llenado de la tarjeta de respuestas, pues solo esta tiene validez para la
calificación.
- Al terminar su examen avise al docente a cargo, no se levante de su sitio; debe entregar la hoja de
respuestas con la carátula del examen, este cuadernillo de preguntas se lo llevará cada estudiante.

1. La contracción del músculo………. permite la eliminación de gases (flatos) sin salida de material fecal;
es el mismo músculo cuya relajación, sobretodo en cuclillas, permite el paso del contenido
fecal con menor esfuerzo durante la defecación.
a) Isq
uir
ec
tal
b)
Pu
bo
rre
ct
al
c) Esfínter anal externo
d) Esfínter anal interno

2. Paciente mujer de 54 años se presenta con náuseas, vómitos, estreñimiento, y es


diagnosticada de abdomen agudo quirúrgico; en la cirugía encuentran un vólvulo de ciego. Esta
anomalía puede explicarse por::
a) Falta de rotación intestinal
b) Falta de fusión del mesenterio
c) Defecto en la formación de la cloaca
d) Falta de formación del omento mayor

3. Paciente mujer de 23 años con faringitis aguda, toma para el dolor una tableta de paracetamol con
un poco de agua. Durante la deglución, se relaja su esfínter esofágico inferior y el fondo del
estómago, mientras el bolo está aún en el esófago. ¿Qué sustancia provocara con mayor
probabilidad la relajación del esfínter esofágico inferior y el fondo del estómago en esta mujer?
a) Óxido nítrico
b) Sustancia P
c) Histamina
d) Motilina

4. Luego de tres horas dando exámenes, un alumno de medicina comienza a sentir hambre.
Esta situación es probable que sea mediada por la que es sintetizada por el :
a) leptina / intestino
b) leptina / estómago
c) grelina / estómago
d) grelina / tejido adiposo

5. Varón de 72 años, con antecedente de diabetes mellitus tipo 2, que presenta enteropatía diabética
caracterizada por estreñimiento. Este problema puede estar asociado a:
a) deficiencia de óxido nítrico
b) aumento del reflejo gastrocólico
c) disminución de la secreción de colecistocinina (CCK)
d) aumento de la secreción del péptido intestinal vasoactivo (PIV)
6. Varón de 54 años con Diabetes Mellitus tipo 2, es diagnosticado de gastroparesia debido a que
presenta sensación de llenura precoz al comer, y reflujo gastroesofágico. Esta alteración en la
relajación receptiva y en el vaciamiento gástrico lo más probable es que se deba a una alteración
en:
a) el nervio vago
b) el ganglio celíaco
c) plexo submucoso
d) nervio hipogástrico

7. Varón de 67 años con tos y disminución de peso asociado a tabaquismo pesado, presenta
actualmente disfagia progresiva a alimentos sólidos. Se considera la presencia de un carcinoma
de bronquio izquierdo y por esta razón le realizan una endoscopía esofágica para descartar la
posibilidad de una compresión esofágica por el tumor. Se espera revisar el esófago en la
estrechez, que está a nivel de la vértebra
a) Tercera estrechez -T6
b) Segunda estrechez - C6
c) Segunda estrechez - T4
d) Tercera estrechez -T10

8. Varón de 34 años con dolor abdominal agudo en flanco derecho que se irradia a fosa ilíaca derecha,
es operado y se encuentra un divertículo intestinal inflamado, ubicado a 93 cm de la válvula
ileocecal. El origen de este divertículo es una falla en la obliteración de:
a) Conducto vitelino
b) Alantoides
c) Cloaca
d) Conducto anorectal
e) Uraco

En un niño menor de dos años con divertículo intestinal, este divertículo tiene su origen en
una falla en la obliteración de:
a)Conduct
o
anorectal
b)
Conducto
vitelino
c) Alantoides
d) Cloaca
e) Uraco

9. Mujer de 43 años sufre un grave accidente de tránsito y está hospitalizada en coma, es alimentada
por vía intravenosa durante varias semanas. Producto de este tipo de alimentación, se encuentra en
la endoscopía atrofia de la mucosa gastrointestinal. La causa más probable de esta atrofia son los
bajos niveles séricos de la hormona:
a) Colecistocinina
b) S
e
c
r
e
t
i
n
a
c
)
G
a
s
t
r
i
n
a
d) PIV

10. Una mujer de 30 años llega al consultorio porque se queja de dificultades para deglutir, la cual se
agravan cada vez más. Se realiza un estudio manométrico para examinar la generación de presión a
lo largo del esófago. Esta prueba revela que las contracciones como respuesta a la deglución están
mal sincronizadas y que la presión en el esfínter esofágico inferior permanece elevada. El
diagnóstico más probable es producido por niveles bajos de
a) acalasia /
sustancia P
b) acalasia /
óxido nítrico
c) enfermedad por reflujo gastrointestinal / acetilcolina
d) enfermedad por reflujo gastrointestinal / óxido nítrico

11. Paciente de 2 años, llega a emergencia por haber ingerido una moneda con la que estaba jugando.
El lugar más probable donde puede haberse quedado suspendido este objeto es a nivel del
estrechamiento producido a nivel del:
a) músculo milohiodeo
b) músculo aritenoideo
c) músculo cricofaríngeo
d) constrictor superior de la faringe

12. En una apendicectomía, al realizar la incisión de McBurney en la fosa iliaca derecha, es


necesario cortar los siguientes músculos, de afuera hacia adentro:
a) Recto – Oblicuo externo – Transverso
b) Recto – Oblicuo externo – Oblicuo interno
c) Oblicuo externo – Oblicuo interno – Recto
d) Oblicuo externo – Oblicuo interno – Transverso

13. Un varón de 90 años que se encuentra postrado en cama, es referido del asilo para endoscopia
por dificultad para deglutir luego de tomar un medicamento para aliviar el dolor la noche
anterior. La endoscopía revela que la píldora se alojó en el esófago y causó una reacción
inflamatoria. Lo más probable es que esto haya sido por la producción de múltiples ondas:
a) secundarias
b) primarias
c) lentas
d) segmentarias

14. Mujer de 23 años es diagnosticada de bulimia, al examen físico se observa ulceraciones en el


segundo y tercero dedo de la mano derecha. Esto se puede deber al uso continuo de estos dedos
para inducir el vómito, mediante la estimulación del par craneal:
a) V
b
)
I
X
c) X
d) XI

15. Varón de 52 años se presenta por diarrea persistente de seis semanas de duración. En la
colonoscopia se observa un pólipo a nivel del íleon distal. El patólogo informa que se trata de un
tumor neuroendócrino, probablemente originado por las células enterocromafines del intestino. La
sustancia que más probablemente esté produciendo este tumor es:
a) Serotonina
b) Insulina
c) CCK
d) GIP

16. La fase oclusal de la masticación se realiza con la contracción de los músculos:


a) digástricos
b) masetero y temporal
c) orbicular y buccinador
d) pterigoideo lateral y digástrico

17. Al tomar su café en Starbucks, un estudiante de medicina sufre una quemadura de primer
grado en el tercio anterior de la superficie dorsal de la lengua. La información de dolor es
transmitida por el nervio:
a) cuerda del tímpano
b) glos
ofarí
nge
o
c)lin
gual
d) facial

18. Paciente es evaluado por faringitis aguda en consultorio externo. El médico de familia le solicita
que abra la boca y saque la lengua. Para realizar la acción de sacar la lengua, es necesario que se
contraiga el músculo:
a) e
st
il
o
gl
o
s
o
b)
g
e
ni
o
gl
o
s
o
c) palatogloso
d) transverso de la lengua
19. Paciente con síndrome de Sjögren,
presenta “boca seca” (disminución de la
producción de saliva) y caries dental,
asociada a la pérdida de la función de
tampón de la saliva. Esta
desminerilización del diente puede
comprometer a las prolongaciones
citoplasmáticas ubicadas en los tubos
huecos de la estructura señalada con la
letra:
a) B
b) A
c) E
d) C

20. Mujer de 32 años acude a consulta por


presentar disfagia de progresión lenta, reflujo
gastroesofágico y vómitos desde hace 3
meses de evolución progresiva. Se le realiza
un estudio radiológico con contraste en el que
se observa estrechamiento del esfínter
esofágico inferior (imagen). Según sus
conocimientos, este paciente se beneficiaría
con el uso de:
a) agonista beta adrenérgico
b) agonista alfa
adrenérgico
c)análogo de
óxido nítrico
d) análogo de Sustancia P
EXAMEN PARCIAL SISTEMA DIGESTIVO
(ME154)
Ciclo 202000

Sección:Todas
Profesores:Alva Muñoz, Jose Carlos
Duración:35 minutos.
Indicaciones:
- Lea atentamente cada pregunta antes de responder.
- Se prohíbe el uso del celular y cualquier dispositivo electrónico.
- Está prohibido intercambiar materiales.
- Coloque su código de alumno en la tarjeta de respuestas. Si su código contiene una letra
reemplácela por un valor numérico siguiendo la siguiente equivalencia: A=9, B=8, C=7, D=6, E=5, F=4
y G=3.
- Traslade sus respuestas a la tarjeta, llenando los círculos de manera completa con lapicero negro o
azul. Está prohibido el llenado con lápiz, lapicero de otro color o con lapicero de tinta borrable.
- Sea cuidadoso en el llenado de la tarjeta de respuestas, pues solo esta tiene validez para la
calificación.
- Al terminar su examen avise al docente a cargo, no se levante de su sitio; debe entregar la hoja de
respuestas con la carátula del examen, este cuadernillo de preguntas se lo llevará cada estudiante.

1. Paciente de sexo masculino de 82 años de edad ingresa a emergencia con dolor abdominal agudo y
diarreas. Se le realiza una arteriografía en la que se observa que la arteria aorta tiene un trombo
ocluyendo el 95% del flujo, a nivel del nacimiento de la arteria mesentérica inferior. ¿Cuál de las
siguientes arterias podría contribuir a la irrigación colateral del colon descendente?
a) cólica media
b) sigmoidea
c) rectal superior
d) ileocólica

2. Niño de 5 años presenta dolor esofágico y hematemesis (vómitos hemorrágicos) luego de tragarse
una espina de pescado. En la endoscopía se observa perforación del esófago distal a la cuarta
estrechez esofágica. ¿Las ramas de cuál de las siguientes arterias estarán lesionada con mayor
probabilidad?
a) Gástrica izquierda
b) Bronquiales
c) Frénica inferior
d) Tiroidea inferior

3. Al ingerir una cucharada de mantequilla es muy probable que se disminuya la sensación de hambre
por medio de la activación de la vía POMC/CART (POMC=proopiomelanocortina y
CART=transcripción regulada de cocaína y anfetamina), activada directamente por la hormona:
a) colecistoquinina (CCK)
b) insulina
c) grelina
d) secretina

4. Al comer unas papitas fritas con mayonesa, el vaciamiento gástrico disminuye por efecto directo
de la hormona: a)colecistoquinina (CCK)
b) bombesina
c) motilina
d) gastrina

5. Recién nacido de dos horas es diagnosticado de hernia umbilical de 1,5 cm de diámetro; el


cirujano pediatra solicita una tomografía abdominal en donde se evidencia que la hernia
umbilical está ocupada por una porción del tracto gastrointestinal. ¿Qué porción del tracto
gastrointestinal estaría ocupando esta hernia con mayor probabilidad?
a) Íleon
b) Colon sigmoides
c) Duodeno
d) Colon transverso
6. Recién nacido de 7 horas, de parto por cesárea debido a polihidramnios (aumento del volumen del
líquido amniótico), con regurgitación de la leche materna y artificial, y no ha presentado meconio.
Se le realiza una tomografía donde se evidencia aire en el estómago y una malformación del
desarrollo esofágico. Con respecto a esta malformación lo más probable es que se pueda tratar de
una atresia esofágica:
a) proximal con fístula traqueoesofágica distal
b) distal con fístula traqueoesofágica proximal
c) proximal y distal
d) sin fístula

7. Lactante de 6 meses de edad que es traído a consulta por presentar vómitos no biliosos a repetición
y retraso en el crecimiento. En la radiografía de abdomen simple se observa nivel hidroaéreo en
estómago y en primera porción de duodeno (doble burbuja). ¿Cuál de las siguientes alternativas
puede explicar la condición del lactante?
a) Páncreas anular
b) Atresia duodenal en la tercera porción
c) Atresia yeyunal
d) Hipertrofia del píloro

8. En ausencia o deficiencia de la secreción de la hormona motilina,


se producirá: a)sobrecrecimiento bacteriano
b) diarrea
c) aumento del vaciamiento gástrico
d) hipertrofia del píloro

9. La estimulación parasimpática aumenta la motilidad intestinal, mientras que la estimulación


simpática la disminuye. ¿Sobre cuál de las siguientes alternativas el sistema nervioso autónomo
actúa para el control de la motilidad intestinal?
a) Potencial de membrana en el plexo mientérico (de Auerbach)
b) Frecuencia de ondas lentas
c) Secreción de secretina
d) Nivel de IP3 en el plexo submucoso (de Meissner)

10. En un recién nacido con protrusión de contenidos abdominales y cubiertas por amnios o peritoneo,
es cierto que: a)Se presenta por un defecto en el cierre de la pared
b) Se acompaña de otras malformaciones congénitas
c) Se debe al no retorno de la hernia fisiológica
d) Se produce a través del ombligo

11. Lactante de 20 días con estreñimiento, distención abdominal progresiva, acompañada


ocasionalmente de vómitos biliosos. Como antecedente, el meconio lo eliminó por primera vez a las
72 horas de nacido. Su mamá menciona que ayuda a la evacuación con ayuda de un termómetro
rectal. Se sospecha de megacolon agangliónico (Enfermedad de Hirschsprung). ¿Cuál de las
siguientes alternativas explica el caso?
a) Se presenta contracciones tónicas en la región ano rectal
b) Se presenta dilatación de tracto gastrointestinal afectado
c) Las células ganglionares sólo han migrado al ano recto
d) La zona que más se afecta es inervada por fibras del nervio esplácnico menor

12. ¿Cuál de los siguientes reflejos disminuye el tránsito


gastrointestinal?
a)Doloroso
b) Gastrocólico
c) De defecación
d) Colicoileal

13. Al ingerir una sustancia ácida como el vino (pH 3), se estimula la motilidad gástrica por acción
de la hormona:
a)motilina
b) secretina
c) colecistoquinina (CCK)
d) bombesina
14. Paciente de 24 años acude a consulta externa por presentar una fístula oronasal (comunicación
entre la cavidad oral y la cavidad nasal). Está fístula está asociada al antecedente de haber sido
operada de paladar hendido a los dos años de edad, durante una campaña gratuita extranjera de
corrección de paladar fisurado. ¿Cuál de las arterias palatinas podría haberse lesionado durante
esa cirugía?
a) Mayor
b) Menor
c) Ascendente
d) Rama palatina de la faríngea ascendente

15. Paciente de sexo masculino de 52 años con úlcera péptica gástrica de 14 años de evolución, con
cuadro de hemorragia digestiva alta hace 4 meses, sin cicatrización de la úlcera. Entre las opciones
quirúrgicas se considera realizarle un vaguectomía troncal (sección del nervio vago) a nivel del
hiato esofágico. ¿Cuál de las siguientes complicaciones podría esperarse producto de la pérdida de
inervación parasimpática?
a) Menor inervación del colon ascendente
b) Se perderá el reflejo de defecación
c) Se perderá el reflejo de micción
d) Impotencia sexual

16. Paciente de 23 años con bulimia es traída a la emergencia deshidratada, semiconsciente y con
alcalosis metabólica. Los vómitos autoinfligidos por esta paciente se producen por estimulación
de receptores en la base de la lengua que mandan información directamente al:
a) núcleo del tracto solitario
b) centro del vómito en el tallo encefálico
c) zona quimiorreceptora gatillo
d) cerebelo

17. Niño de 3 años es traído a emergencia por madre quien manifiesta que hace 10 horas deglutió una
pila pequeña de reloj de bordes romos. El niño está asintomático. Usted la tranquiliza diciéndole es
un cuerpo extraño tan pequeño de seguro que va a seguir el tránsito intestinal como lo haría un
bolo alimenticio, y que lo más probable es que en ese momento se encuentre en:
a) colon
b) estómago
c) yeyuno
d) recto

18. Los movimientos en masa son un tipo de movimiento muy importante, una de las
consecuencias de estos movimientos es:
a) la distensión rectal
b) el peristaltismo del intestino delgado
c) la retropulsión gástrica
d) la contracción del esfínter anal interno

19. Durante la deglución, al momento que el bolo alimenticio pasa por el esfínter esofágico superior, se
espera que la presión intraesofágica:
a) disminuya en el cardias
b) disminuya en el tercio medio del esófago
c) aumente en la porción distal al bolo
d) aumente en el tercio medio del esófago

20. Paciente con enfermedad de Chagas que presenta disfagia a sólidos. ¿Cuál de las siguientes puede
ser la causa de esta complicación?
a) Disminución de células ganglionares en el esfínter esofágico inferior
b) Aumento en la liberación de óxido nítrico en el esfínter esofágico inferior
c) Disminución de las neuronas que liberan péptido intestinal vasoactivo
d) Aumento de la actividad de la motilina en el esófago distal
QUIZIZZ
1. Los vasos mesentéricos superiores se hallan a nivel de:
a) Cuello del páncreas

2. El nivel en el que se encuentra el píloro y el páncreas se puede determinar usando el


A) plano transpilórico

3. El dolor de estómago asociado a gastritis se suele ubicar en


a) epigastrio

4. La colecistoquinina inhibe el
a) vaciamiento gástrico

5. Paciente con disfasia ( dificultad para pasar alimentos) con to y disminución de peso. Con
antecedente de tabaquismo pesado. La sospecha es que tenga una disminución del diámetro
esófago a nivel de la
a) tercera estrechez

6. La digestión de las proteínas se inicia en:


a) estómago

7. El estómago recibe información simpática proveniente del:


a) ganglio celíaco

8. El reflujo gastroesofagico tiene múltiples etiologías, una de ellas tiene que ver con alteración a
nivel de:
a) primera estrechez
b) segunda estrechez
c) tercera estrechez
d) cuarta estrechez

9. La fístula retroperitoneal es causada por una falla en el desarrollo de:


a) tabique urorrectal

10. El nervio vago inerva el:


a) músculo estriado del esófago

11. El divertículo de Meckel es un rezago de:


a) conducto vitelino

12. La presencia de orina que sale por el ombligo de un recién nacido casa vez que llora, es posible
que se deba a un defecto en el desarrollo del
a) seno urogenital

13. La fístula retroperitoneal es causada por una falla en el desarrollo de


a) tabique urorrectal

14. Enfermedad asociada con un error en el desarrollo de las células de Cajal:


a) enfermedad de Hirschsprung

15. Aproximadamente en la semana 6 del desarrollo embrionario, el intestino medio gira 90


herniandose a nivel del
a) cordón umbilical

16. El conducto biliar deriva del


a) endodermo

17. Paciente mujer con 54 años con nauseas y vómitos y abdomen agudo quirúrgico, se ingresa a sala
de operaciones donde se encuentra vólvulo de ciego, esto se debe a
a) Falta de fusión del mesenterio

18. Paciente con cirrosis hepática con hipertensión portal, en el que es posible encontrar que los
vasos umbilicales están permeables dentro de
a) ligamento redondo

19. Es normal encontrar glándulas submucosas en CUAL


ES
a) esófago medio
b) esófago proximal
c) esófago distal
d) estómago

20. Cual de los siguientes órganos son intraperitoneales


a) estómago, vesícula biliar, íleon, hígado

Estudiante de medicina de la UPC de 21 años sufre de gastritis aguda ocasionada por comer en lugares
poco higiénicos. Suele consumir caramelos ( chupar ) mientras está en clase hasta la tarde. Toma gaseosas
regularmente (carbohidratos 46%, sodio 53%). También toma regular cantidad de leche (grasa 35%, lactosa
35%, proteínas 30%), pues le calma un poco el dolor el ardor que siente por la gastritis. Incluso, cuando
puede, se toma dos vasos de agua fría para calmar las molestias. Ha decidido ir al médico para tratarse pues
ya no soporta el dolor, el cual está seguro que los síntomas se deben a una elevada producción de ácido
clorhídrico en el estómago, y por ello le ha recetado Ranitidina (antihistamínico), con lo que siente mejoría.

● Para reducir la secreción de HCl en esta paciente se podría usar sustancias similares a:

- Péptido insulinotrópico dependiente de la glucosa (GIP)


● Si se usara atropina en esta paciente, se esperaría que disminuya la liberación de:
- Enzimas pancreáticas

● El consumir caramelos eleva los niveles en sangre de una hormona cuya función es
la estimulación de las células:
- Beta del páncreas

● En este paciente con gastritis aguda debida a una alta producción de ácido
clorhídrico, si se le hiciera un examen de sangre, se encontraría elevados los niveles
de:
- Colecistoquinina

● El consumo de una pequeña cantidad de gaseosa aumentará directamente la


concentración sérica de cuál de las siguientes hormonas:

- Péptido 1 similar al glucagón (GLP-1)

● El consumo rápido de 500 mL de gaseosa aumentará directamente la concentración


sérica de cuál de las siguientes hormonas:

- Gastrina

● Estimulan la secreción ácida gástrica

- Proteínas

● Con respecto a las ondas lentas, marque la afirmación correcta:}

- Son contracciones rítmicas espontáneas

● El uso de Ranitidina bloquea el receptor H2 de la histamina en las células


parietales. La histamina llega a estas células por:

- Difusión

● El consumir caramelos indirectamente activa la vía:

- POMC/CART

● ¿Cuál de los siguientes péptidos inhibe el vaciamiento gástrico?

- Colecistoquinina

● Para poder morder una manzana, es necesario usar el siguiente músculo:

- Milohiodeo

● El crecimiento de un adenocarcinoma de cuello de páncreas puede comprometer la


pared gástrica por continuidad. ¿Qué parte del estómago estaría comprometido con
mayor probabilidad?

- Pared posterior del antro

● El nacimiento de la arteria mesentérica superior se puede encontrar en cuál de los


cuadrantes abdominales:

- Epigastrio
● En cuanto a la colecistoquinina, marque la respuesta correcta:

- Potencia la acción de buffer con bicarbonato

● El aumento en la actividad motora de la pared gástrica genera un aumento en


los niveles locales de qué sustancia en la microvasculatura:

- Adenosina

Niño de sexo masculino de 2 años de edad, sufre de estreñimiento desde el nacimiento (1


deposición cada 3-4 días). Madre menciona que le estimula la defecación con un termómetro
rectal, y continuo uso de enemas y laxantes. Desde hace 6 meses comienza con vómitos
postprandiales. Los síntomas aumentan en frecuencia y magnitud y están en relación con
los episodios de estreñimiento. No refiere fiebre, tos, diarrea ni lesiones cutáneas. Al
examen físico presenta regular estado general, luce deshidratado. Abdomen distendido,
blando, depresible e indoloro. No se palpan masas abdominales. Se permeabiliza el canal
anal con termómetro rectal, encontrando cierta resistencia. Salida de material fecal mal
oliente en regular cantidad. Exámenes de laboratorio: hemograma normal. Signos
inflamatorios de fase aguda negativos. Alcalosis metabólica leve en sangre venosa.
Radiografía con enema baritado muestra recto y colon sigmoides dilatados (megacolon).
Biopsia profunda: ausencia de células ganglionares en la muestra enviada. Se realiza cirugía
correctiva.

● La percepción de la pirosis (sensación de dolor o quemazón en el esófago) asociado


al reflujo gastroesofágico, puede aparecer o exacerbarse debido a:

- Ejercicio

● Considerando que este paciente está sometido a estrés por el agravamiento de su


enfermedad, es posible afirmar que sus ondas lentas están:

- Hiperpolarizadas

● Debido al acúmulo de material fecal en todo el marco colónico, y a la irritación


química asociada, el peristaltismo del íleon distal se debe encontrar:

- Inhibido

● En cuanto a los reflejos gastrocólico y gastroduodenal en este paciente, indique lo


correcto:

- Se pueden considerar reflejos vago-vagales

● Con respecto a la defecación señale el enunciado correcto:

- Es estimulado por un llenado de la cuarta parte del volumen rectal

● El contenido fecal se detiene en la zona inmediatamente proximal a la zona donde


hay una menor presencia de:

- Péptido intestinal vasoactivo

● La presencia de atresias y estenosis duodenales se deben básicamente a una:

- Falta de recanalización

● El ligamento de Treitz característicamente:

- Suspende el ángulo de Treitz

● Al deglutir un bolo alimenticio, es lógico suponer que al pasar por el esófago haya
un mayor consumo de oxígeno en la pared del tercio:

- Proximal
● Estudiante de medicina de 20 años, se ha amanecido estudiando para su examen de
Sistema Digestivo. No ha probado alimento desde la cena, por lo que se puede
afirmar que la motilidad de esta persona está siendo regulada por:

- Motilina

● La hernia fisiológica se produce dentro de:

- Cordón umbilical

● El crecimiento de un adenocarcinoma de páncreas compromete la pared gástrica por


contigüidad. ¿Qué parte del estómago se esperaría esté comprometido?

- Pared posterior del antro

● Al comer unas papitas fritas con mayonesa, el vaciamiento gástrico disminuye por
efecto directo de la hormona:

- colecistoquinina (CCK)

● Paciente que come entera una pizza familiar de chorizo y queso. Es posible esperar
que debido a la cantidad de alimento ingerida, las ondas lentas hayan:

- Sufrido ninguna alteración en su frecuencia

● En este caso se puede afirmar con seguridad que se presenta:

- contracciones tónicas en la región ano rectal

● La forma más común de atresia esofágica contiene:

- Estenosis proximal del esófago más fístula traqueoesofágica distal

● Con respecto a la saliva, marque la respuesta correcta:

- el sistema simpático estimula su secreción

● Respecto a las enfermedades del esófago, marque lo correcto:

- el diagnóstico diferencial de la acalasia es la enfermedad de Chagas esofágica

● En relación a la fisilogía gástrica, marque lo correcto:

- la cimetidina actúa en la región basolateral de la célula parietal

● La célula mucosa del cuello gástrico produce:

- Moco

● La saliva puede tener una variedad de electrolitos en su composición. Entre ellos el cloro,
respecto al cual se puede afirmar:

- Su concentración no llega a ser tan alta como en el plasma

● Los músculos de la masticación que producen la retropulsión de la mandíbula son:

- temporales

● Respecto a las glándulas salivales, marque lo incorrecto:

- la glándula sublingual tiene forma de garfio

● Respecto a la anatomía del estómago, marque lo correcto:


- la arteria gástrica derecha nace de la arteria hepática común

● En cuanto a la saliva, marque lo correcto:

- La amilasa cumple función digestiva

● Durante el ataque con gas sarín (bloqueador de la acetilcolinesterasa) en el metro de


Tokio, en 1995, el personal de salud notó que los pacientes afectados presentaban:

- Hipersalivación

Paciente de 54 años con antecedentes de alcoholismo, gastritis crónica,


tabaquismo pesado, obesidad, cálculos biliares y cirrosis, es llevado a la
emergencia por dolor abdominal en epigastrio irradiado a la espalda y trastorno del
sensorio.
Al examen físico: presión arterial 85/50 mmHg, frecuencia cardíaca 100 latidos/min,
frecuencia respiratoria 18 x minuto, temperatura axilar 36°C.
Conjuntivas pálidas, escleras ictéricas nevus arácnidos en tronco, distensión
abdominal marcada, cabeza de medusa, matidez desplazable en ambos flancos e
hipogastrio, dolor a la palpación de abdomen.
Tiempo de protrombina: 24 seg (testigo: 13 seg); TPT: 38 seg, glicemia: 165 mg/dL,
uremia: 20 mg/dL, ASAT: 76 UI/L, ALAT: 22 UI/L, albumina: 2,5 g/dL, bilirrubina total:
2,6 mg/dL, bilirrubina directa: 1,4 mg/dL, amilasa sérica 4000 U/L.

● Un efecto secundario en el estómago por la acción de la secretina es:

- Menor actividad de la pepsina

● Considerando que el paciente sufre de gastritis, se puede decir que la secreción de


ácido por la mucosa gástrica:

- Involucra transporte activo de Hidrogeniones

● En cuanto a la gastritis de este paciente, se encontró que era producida por la


bacteria Helicobacter pylori. Esta bacteria sobrevive en el medio ácido del estómago
gracias a:

- Ureasa

● La bilirrubina directa aumentada en cirrosis hepática se excreta en la orina debido a:

- Ser hidrosoluble

● Paciente de 42 años con adenocarcinoma ductular. La TC ha demostrado claramente que el


tumor está en el cuello del páncreas y que hay un gran vaso ocluido. ¿Cuál de los siguientes
vasos estaría más probablemente obstruido?

- Vena porta.

● En relación a la histología hepática, marque lo correcto:


- La zona 3 se encuentra más cerca a la vena central lobulillar
Mujer de 83 años acude a emergencia por dolor abdominal desde hace 4
días, localizado en epigastrio, irradiado a ambos hipocondrios, nauseas,
vómitos y distensión abdominal; tiene antecedente de cardiopatía
hipertensiva, diabetes mellitus tipo II y fibrilación auricular. Refiere
deposiciones diarreicas muco sanguinolentas hace 1 día. Los exámenes
iniciales muestran PA: 110/60 mmHg, FC: 110/mn, leucocitos: 17800,
neutrófilos de 93%. TAC abdómino pélvica se observa oclusión completa
de arteria mesentérica superior por trombo asociado a placa ateromatosa.

● En esta paciente, ¿cuál de las siguientes sustancias no tendrá una


considerable disminución en su absorción? (marque la mejor respuesta):

- Calcio

● El mecanismo de la diarrea muco sanguinolenta que presenta la paciente, puede


mejor definido como de tipo (marque la mejor respuesta):}

- Exudativa

● Considerando que se ha comprometido el íleon distal, entre otras áreas, la atrofia o


descamación del epitelio de superficie explicaría cuál de los signos o síntomas de la
paciente (marque la mejor respuesta):

- Diarreas mucosanguinolentas

● Producto de esta isquemia, la expresión de cuál de las siguientes enzimas se vería


notablemente disminuida. Marque la mejor respuesta:

- Enteroquinasa

● Durante la cirugía, el cirujano observó que además la paciente tenía divertículos en


el sigma. Se sabe que estos divertículos:

- Se pueden asociar a estreñimiento crónico

1) La glandula parotida tiene principalmente acinos de tipo:


Tipo seroso
2) Presenta movimientos en masa:
Colon
3) A mayor flujo de saliva, disminuye la concentración de:
potasio
4) Es rama de la arteria mesentérica superior:
Arteria cólica media
5) En el intestino delgado se absorben los carbohidratos en forma de:
Fructosa
6) El dolor asociado a apendicitis clásicamente se ubica en:
Fosa iliaca derecho
7) La glandula submandibular recibe inervación traída por el nervio:
Cuerda del tímpano
8) La línea alba se encuentra:
Entre los rectos abdominales
9) la digestión de los carbohidratos se inician en:
Cavidad oral
10) Permite la suspensión e irrigación de los órganos peritoneales:
Mesenterio
11) Los nervios esplácnicos pélvicos (S2-S4) llevan información de tipo:
Parasimpática
12) Paciente con cirrosis hepática que tiene hipertensión portal con varices esofágicas y
actualmente presenta varices en estómago distal. Estas varices están relacionadas a
aumento en la presión de las venas:
Gástrica derecha
13) Es un órgano peritoneal:
Hígado
14) Al realizar un piercing en el ombligo, la sensación de dolor se transmite por:
T10
15) A mayor flujo de saliva disminuye la concentración de:
Potasio
16) La colecistoquinina inhibe:
El vaciamiento gástrico
17) El dolor del estómago asociado a gastritis se suele ubicar en:
Epigastrio
18) Es considerada una lesión preneoplásica:
Leucoplasia
19) Los sinusoides hepáticos son en realidad capilares de tipo:
Fenestrados
20) Paciente con cirrosis hepática y presión de vena cava de 15 mmHg, lo mas probable
es que el paciente presente:
Ascitis
21) La glándula submaxilar le hace un gancho al:
Músculo milohioideo
22) La glandula parotida esta inervada por el par craneal:
IX pc
23) Los nervios esplácnicos lumbares (L1-L2) llevan información de tipo:
Simpático
24) Los 9 cuadrantes del abdomen se delinean usando el plano subcostal, las líneas
medioclaviculares y:
Plano intertubercular
25) El estómago recibe información simpática proveniente del:
Ganglio celiaco
26) Es un órgano retroperitoneal:
Páncreas
27) La digestión de los lípidos se inicia en:
Intestino delgado
28) La digestión de las proteínas se inicia en:
Estómago
29) La arteria esplénica proviene de la aorta, y la vena esplénica desemboca en la vena:
Porta
30) Enfermedad asociado con un error en el desarrollo de las células de Cajal:
Enfermedad de Hirschsprung
31) El ácido acetilsalicílico actúa en la membrana:
Basolateral de la célula parietal
32) g
33) g
34) g
35) g
36) g
37) g
38) g
39) g
40) g
41) g
42) g
43) g
44) g
45) g
46) g
47) g
48) g
49) g
50) g
51) g
52) g
SISTEMA DIGESTIVO
EXAMEN PARCIAL
201802

Sección: Todas
Profesores: Choque Chávez, Fernando Diego; Damián Bastidas, Narda Lucía; Irribarren Gamarra, Maria Patricia;
Stapleton Valdivia, Mauricio Juan Jose.
Duración: 50 minutos.
Indicaciones:
- Lea atentamente cada pregunta antes de responder.
- Se prohíbe el uso del celular y cualquier dispositivo electrónico.
- Está prohibido intercambiar materiales.
- Coloque su código de alumno en la tarjeta de respuestas. Si su código contiene una letra reemplácela por un valor
numérico siguiendo la siguiente equivalencia: A=9, B=8, C=7, D=6, E=5.
- Traslade sus respuestas a la tarjeta, llenando los círculos de manera completa con lapicero negro o azul. Está
prohibido el llenado con lápiz, lapicero de otro color o con lapicero de tinta borrable.
- Sea cuidadoso en el llenado de la tarjeta de respuestas, pues solo esta tiene validez para la calificación.
- Al terminar su examen avise al docente a cargo, no se levante de su sitio; debe entregar la hoja de respuestas con
la carátula del examen, este cuadernillo de preguntas se lo llevará cada estudiante.

Chorrillos, octubre de 2018

1. En todo el sistema gastrointestinal se encuentra diferentes tipos de músculos ¿En cuales estructuras
encontraríamos músculo estriado?
A. Faringe, esfinter esofágico inferior, colon descendente.
B. Esfínter esofágico superior, laringe, esfínter anal externo.
C. Faringe, esfínter esofágico superior, esfínter anal externo.
D. Esfínter esofágico superior, esfínter esofágico inferior, colon.

2. Durante la fase …..……………….. el paladar blando es traccionado hacia arriba.


A. oral
B. gástrica
C. faríngea
D. esofágica

3. El tubo digestivo a nivel del ………………………….. está revestido internamente por epitelio ………….
A. Esófago / plano simple
B. Estómago / cilíndrico simple
C. Intestino grueso / plano estratificado
D. Intestino delgado / plano de transición

4. Señale la estructura del tubo digestivo que presenta tres capas musculares en su pared.
A. Esófago
B. Estómago
C. Intestino grueso
D. Intestino delgado

5. Paciente de 47 años con sobrepeso, acude a consulta por presentar dolor tipo cólico en el cuadrante superior
derecho del abdomen. El dolor aumenta luego de ingesta de comidas con abundante grasa. Esta situación se explica
por el aumento en la secreción de …………………………. que estimula la contracción de la vesícula biliar.
A. Gastrina
B. Colecistoquinina
C. Péptido inhibidor gástrico
D. Péptido intestinal vasoactivo
6. Si hay un aumenta de la hormona grelina a nivel del hipotálamo ¿Cuál es la consecuencia en el organismo?
A. Hambre
B. Saciedad
C. Aumento de somatostatina
D. Disminución del tránsito intestinal

7. Señale el péptido gastrointestinal que produce relajación del músculo liso gastro-intestinal.
A. Péptido intestinal vasoactivo (PIV)
B. Gastrina vasoactiva (GV)
C. Colecistoquinina (CCK).
D. Acetilcolina (Ach)

8. ¿Cuál de las siguientes alternativas inhibiría la relajación receptiva a nivel del estómago?
A. Histamina
B. Bloqueo del nervio vago (X)
C. Péptido inhibir gástrico (GIP)
D. Péptido intestinal vasoactivo (VIP)

9. Una mujer de 28 años, con diagnóstico de Diabetes Mellitus Tipo 1, acude por presentar desde hace 10 años
estreñimiento y distensión abdominal. Se realiza estudio y se determina que la paciente presenta un retraso del
vaciamiento gástrico debido a gastroparesia diabética. ¿Cuál de los siguientes hechos aumentaría el tiempo del
vaciamiento gástrico?
A. Aumento de gastrina
B. Estimulación parasimpática
C. Ácidos grasos en el duodeno
D. Quimo isotónico en el duodeno

10. Experimentalmente, se aplica un inhibidor selectivo del péptido intestinal vasoactivo (PIV) durante la contracción
peristáltica del intestino delgado. ¿Cuál es el efecto de este inhibidor en la motilidad del intestino delgado?
A. Parálisis del movimiento anterogrado
B. Disminución del tránsito intestinal
C. Aumento del tránsito intestinal
D. Movimiento retrogrado

11. ¿Cuál de los siguientes eventos ocurre durante la defecación?


A. Relajación del esfínter anal externo
B. Relajación del músculo liso del recto
C. Contracción del esfínter anal interno
D. Disminución de la presión intrabdominal

12. Señale el péptido que cumple la función de disminuir la ingesta de alimentos.


A. Neuropéptido Y
B. Ghrelina Gastrica
C. Proteína relacionada a agouti
D. Proopiomelanocortina (POMC)

13. Indique el lugar de secreción del péptido YY


A. Estómago
B. Duodeno
C. Yeyuno
D. Ileon

14. La grelina es secretada en el estómago y estimula a las neuronas del núcleo ………………….. para la estimulación de
la secreción de……………..
A. arqueado / neuropéptido Y.
B. arqueado / melanocortinas.
C. paraventricular / neuropéptido Y.
D. lateral del hipotálamo / hormonas orexigénicas.
15. Considerando el desarrollo embriológico del intestino medio. ¿Qué evento de gran importancia se produce en la
sexta semana?
A. Retracción de asas intestinales primitivas
B. Aparición del primordio hepático y pancreático
C. Inicio de la secreción de insulina por el páncreas
D. Salida temporal de asas intestinales a través de cordón umbilical

16. En el ……………………………. se presentan los movimiento de …………………….


A. esófago / retropropulsión y mezcla
B. estómago / segmentación y ondas lentas
C. intestino grueso / propulsión y ondas en espiga
D. intestino delgado / segmentación y peristaltismo

17. Marita sufre una parálisis del músculo masetero. ¿Qué limitaciones se producirán a nivel del movimiento de la
mandíbula?
A. Lateralización
B. Propulsión
C. Elevación
D. Ninguna

18. Señale el movimiento que se produce a nivel del ciego y colon proximal que tiene como finalidad favorecer la
absorción de agua y sales.
A. De masa
B. De mezcla
C. Propulsivo
D. Peristáltico

19. La enfermedad de Hirchsprung se caracteriza por una dilatación anormal de colon y disminución de los
movimientos propulsivos que traerá como principal consecuencia …………
A. diarrea acuosa.
B. estreñimiento crónico.
C. dilatación de la válvula ileocecal.
D. disminución de la flora intestinal.

20. ¿Cuál de los siguientes factores estimulan el vaciamiento gástrico?


A. Colecistoquinina (CCK)
B. Neuropéptido Y
C. Secretina
D. Gastrina

21. La secreción de ………………………. estimula la motilidad gástrica.


A. Colecistoquinina
B. Secretina
C. Gastrina
D. Motilina

22. La hormona ……………………….. es secretada por las células “I” del …………..
A. Colecistoquinina / duodeno y yeyuno
B. Gastrina / duodeno y yeyuno
C. Colecistoquinina / estómago
D. Gastrina / estómago
GLUT2, SGLT1, GLUT5

CI 1

1. La motilidad intestinal es estimulada principalmente por el:


- Sistema simpático
- Sistema piramidal
- Plexo de Auerbach
- Sistema parasimpático
2. La peristalsis o peristaltismo hace referencia a:
- Motilidad para movilizar el alimento de proximal a distal.
- No es parte de la motilidad
- Motilidad para mezclado de alimentos.
- Motilidad para fraccionamiento de alimentos.
3. Marque el órgano que se encuentra más distal en el tubo digestivo.
- Estomago
- Ciego
- Íleon
- Duodeno
4. Marque la respuesta incorrecta:
- La mucosa consta de epitelio, lámina propia y muscularis mucosae.
- En todo el tubo digestivo, se observa dos capas de muscular propia: circular
interna y longitudinal externa
- El colon contiene tenias
- Fuera de la cavidad abdominal, el esófago presenta capa adventicia.
5. Al iniciar la digestión, aumenta el consumo de oxígeno por la mucosa. Esto conlleva a
una hipoxia local, lo cual hace que se libere _____________, el cual produce
vasodilatación:
- Colecistoquinina
- Adenosina
- Histamina
- Noradrenalina
6. Cuál de las siguientes estructuras no tiene vasos sanguíneos:
- Epitelio intestinal
- Ligamento
- Omento
- Mesenterio
7. Paciente tiene una úlcera sangrante en el segundo tercio del Yeyuno. La arteria de la
cual proviene la sangre arterial para dicha zona es la arteria:
- Mesentérica superior
- Tronco celíaco
- Mesentérica inferior
- Iliaca común
- Gástrica izquierda
8. Paciente de 24 años con dolor abdominal tipo cólico intenso en mesogastrio. Según
sus conocimientos de macroestructura, el origen del dolor puede ser el ___________:
- Íleon
- Colon
- Estómago
- Esófago
9. Dentro de las funciones del abdomen, se encuentra la defecación y micción, en las
cuales la presión intra abdominal debe:
- Aumentar
- No tiene relación el abdomen con dichas funciones
- Mantenerse igual
- Disminuir
10. En la inspiración, la pared abdominal debe ____________ para ____________:
- Contraerse aumentar presión intra abdominal
- Relajarse disminuir presión intra torácica
- Relajarse aumentar presión intra abdominal
- Contraerse aumentar presión intra torácica

CI 2

1. Respecto a los péptidos gastrointestinales, marque lo correcto.


- No existe sustancia neurocrina que tenga efecto en la motilidad del tubo
digestivo
- Las sustancias paracrinas pueden viajar a través de vasos sanguíneos
- Las sustancias neurocrinas son peptidos que hacen su efecto en distancias
cortas
- Las sustancias paracrinas atraviesan la circulación portal
2. Al disminuir el pH duodenal por el HCl gástrico, se libera principalmente una hormona
cuya célula diana es:
- Acinos pancreáticos
- Células ductales del colédoco
- Célula ductal del Wirsung
- Células S del intestino
3. En un paciente con gastroparesia (motilidad lenta del estómago), que presenta
distensión abdominal después de comer, usted le recomendaría que evite el consumo
de lípidos y aminoácidos para disminuir la acción de:
- Secretina
- CCK
- Somatostatina
- Gastrina
4. La razón por la que el potencial de acción viaja rápidamente en sentido longitudinal
por el musculo liso gastrointestinal es la presencia de uniones en hendidura, además
de la presencia de:
- La presencia del plexo submucoso de Meissner
- Varicosidades
- Mayor cantidad de ACh
- Las fibras musculares no se disponen en haces musculares
5. Paciente obeso con Covid-19 es intubado por interno inexperto, quien al solicitar que
bombeen aire dentro del tubo endotraqueal, nota que el epigastrio se distiende. Al
sospechar que ha introducido el tubo en el estómago, también es cierto que:
- Disminuiría el tono del píloro
- Aumenta la frecuencia de ondas lentas
- Aumenta el pH gástrico
- Disminuye el pH gástrico
6. En un paciente con diarrea por hipermotilidad, usted sospecharía en el posible
aumento de las siguientes sustancias, excepto:
- Sustancia P
- ACh
- Péptido intestinal vasoactivo (VIP)
- Motilina
7. El ecografista sabe que para poder visualizar el nacimiento de la arteria mesentérica
superior, debe colocar el transductor sobre la piel de la siguiente región abdominal:
- Epigastrio
- Hipocondrio derecho
- Hipogastrio
- Mesogastrio
8. Al consumir un pan con mantequilla, la sensación de hambre disminuye debido a la
acción de:
- Grelina
- Somatostatina
- Colecistoquinina (CCK)
- Leptina
9. Una de las siguientes sustancias no comparte con las otras la misma acción sobre la
producción de ácido gástrico:
- Péptido insulinotrópico dependiente de glucosa (GIP)
- Colecistoquinina
- Somatostatina
- Secretina
10. Al ingerir grandes cantidades de dulces, con la subsecuente estimulación de incretinas,
usted esperaría que el apetito ______________, debido a __________________
- disminuya insulina
- aumente CCK
- disminuya CCK
- aumente grelina

PARCIAL

1. Al rozar agua caliente en la punta de la lengua, usted esperaría que el estímulo viaje a
través del nervio:
- lingual
- cuerda del tímpano
- glosofaríngeo
- hipogloso
2. Al ingresar líquidos o sólidos en la cavidad oral, un mecanismo que permite que una
persona respire mientras mastica es:
- el movimiento hacia afuera de los pliegues palatogloso y palatofaríngeo
- la elevación del paladar blando
- la depresión de la parte posterior de la lengua
- la depresión del paladar blando
3. Paciente adulto con reflujo gastroesofágico es más probable que presente:
- descalcificación del esmalte
- destrucción de ameloblastos
- remodelación del esmalte
- desfluorización de los dientes
4. La masticación es básicamente:
- importante para la digestión sobretodo de carnes
- un ralentizador del vaciamiento gástrico
- una actividad consciente
- un movimiento reflejo
5. Paciente de 34 años es víctima de asalto con arma de fuego, recibiendo un impacto
directo en el abdomen. En base a la radiografía, usted puede registrar en la historia
clínica que el proyectil se encuentra topográficamente en el:

- flanco derecho
- mesogastrio
- flanco izquierdo
- hipocondrio izquierdo
6. Señale cuál de las estructuras que en el embrión se encuentra comunicada con el saco
vitelino por medio del conducto onfalomesentérico:
- D
- B
- C
- A
7. En un paciente de 3 semanas de edad, con vómitos en proyectil, y nódulo epigástrico
reptante, usted esperaría encontrar:
- colecistoquinina aumentada
- vómitos biliosos e intolerancia a los ácidos grasos
- distensibilidad disminuida de la región oral del estómago
- engrosamiento de la circular interna pilórica
8. ¿Cuál de las siguientes estructuras tiene inervación somática?
- Estómago
- Peritoneo visceral
- Mesosigmoides
- Peritoneo parietal
9. Estas diseñando un proyecto de investigación sobre los niveles de colesterol que se
absorben luego de una comida grasosa y deseas cuantificar la cantidad de colesterol
que es absorbido por el intestino antes que el hígado lo metabolice ¿de cuál de los
siguientes vasos obtendrías la muestra para tu análisis?
- Vena porta
- Vena cava superior
- Conducto torácico
- Vena hemiácigos accesoria
10. Durante el paso del bolo hacia la orofaringe, se desencadena una serie de
contracciones musculares que estrechan la cavidad faríngea. Estas contracciones están
mediadas por el nervio craneal:
- X
- XI
- XII
- IX
11. Cuando el istmo de las fauces se cierra, se evita que el alimento pase hacia la
orofarínge y permite respirar mientras se mastica. Este cierre se debe a la contracción
y aproximación de los músculos:
- Palatoglosos
- Palatofaríngeos
- Estiloglosos
- Estilofaríngeos
12. Las siguientes alternativas son factores que determinan la patencia y función adecuada
del esfínter esofágico inferior, EXCEPTO:
- Plicatura diafragmática
- Hipertrofia de la circular interna
- Angulación con el estómago
- Canales lentos de calcio
13. Respecto al peristaltismo intestinal, para cumplir la ley del intestino , usted espera que
a nivel distal del quimo se libere:
- péptido liberador de gastrina (GRP)
- acetilcolina
- péptido intestinal vasoactivo
- sustancia P
14. El estímulo habitual para el movimiento peristáltico es:
- acción de la sustancia P
- contracción de la musculatura circular Interna
- estimulación vago-vagal
- distensión local
15. ¿Cuál de las siguientes condiciones considera que es un trastorno de la musculatura
lisa esofágica?
- Acalasia
- Asinergia faringoesfinteriana
- Hipotonía de los constrictores faríngeos
- Hipertonía del esfínter esofágico superior
16. ¿Cuál de las siguientes alternativas es correcta sobre la motilidad esofágica?
- Las ondas primarias son propulsoras y pueden no ser precedidas por deglución
- Las ondas primarias no son propulsoras y siempre van precedidas de deglución
- Las ondas secundarias son propulsoras y siempre van precedidas de deglución
- Las ondas secundarias son propulsoras y no van precedidas de deglución
17. El peristaltismo depende que a nivel distal del bolo se secrete:
- noradrenalina secretada por las fibras del sistema simpático
- acetilcolina por las neuronas provenientes del nervio vago
- óxido nítrico por células endoteliales locales
- péptido intestinal vasoactivo por neuronas
18. ¿Cuál de las siguientes alternativas es correcta sobre el movimiento peristáltico?
- Es un reflejo largo que depende de la integración con el tronco encefálico
- Se dirige en sentido distal siempre, nunca en sentido proximal
- El contenido intestinal avanza sólo 5-10 cm
- Es independiente del plexo mientérico
19. En un estudiante de medicina que está rindiendo un examen parcial, lo más probable
es que en ese momento su tránsito intestinal se encuentre:
- muy lento
- muy acelerado
- estimulado por acción de la sustancia P
- sin alteraciones
20. Las siguientes alternativas son ciertas sobre las contracciones tónicas del músculo
gastrointestinal, EXCEPTO:
- Tienen relación con el ingreso persistente de iones sodio
- Se encuentran principalmente en esfínteres
- Tienen regulación hormonal
- Obedece a una mayor frecuencia de potenciales en espiga
21. Con respecto de la regulación del pH del estómago; al utilizar un bloqueador de
histamina, usted espera que el pH del estómago:
- aumente
- disminuya
- se mantenga sin cambio
- se neutralice por acción de bicarbonato
22. Las siguientes hormonas disminuyen el vaciamiento gástrico, EXCEPTO:
- Gastrina
- Péptido insulinotrópico dependiente de glucosa
- Colecistoquinina
- Secretina
23. El frenillo de los labios se encuentra en:
- el piso de la boca
- la cavidad oral
- la cavidad vestibular
- el dorso de la lengua
24. En un paciente con shock hipovolémico, la peristalsis intestinal se encuentra:
- aumentada
- sin cambios
- invertida
- disminuida
25. Sobre el control de la peristalsis del tubo digestivo, ________________ es un
mediador neural que induce la relajación durante la peristalsis.
- la somatostatina
- el péptido intestinal vasoactivo
- la acetilcolina
- la serotonina

CI 3

1. En cuanto a las sustancias secretadas por el estómago ¿Cuál de las siguientes


sustancias estimula la liberación de pepsinógeno?
- Pepsinógeno
- Secretina
- Colecistoqunina
- Gastrina
2. Las células enteroendocrinas en el estómago se localizan en la glándula oxíntica, al
mismo nivel que las células:
- parietales
- principales
- mucosas
- absortivas
3. La fase intestinal de la secreción gástrica se debe básicamente a la participación de las
células:
- G del duodeno
- I del yeyuno
- S del íleon
- D del estómago
4. El conducto de Stenon, para entrar a la cavidad vestibular, debe atravesar el músculo:
- genihioideo
- buccinador
- milohioideo
- masetero
5. Paciente con acalasia es sometido a tratamiento endoscópico o quirúrgico, usted le ha
informado al paciente previamente que es posible que una complicación de este
tratamiento es que quede con cierto grado de:
- reflujo gastroesofágico
- gastritis
- odinofagia
- úlceras gástricas
6. La sangre que lleva la vena porta es tipo:
- mixta
- arterial
- venosa
7. Los pliegues gástricos gruesos son prácticamente inexistentes a nivel de:
- el cuerpo
- el fondo
- la incisura angularis
- el antro
8. En la producción de HCl, la acción de la somatostatina disminuye la accion de:
- las prostaglandinas
- la histamina
- la acetilcolina
- la gastrina
9. En el estómago se secretan las siguientes sustancias, EXCEPTO:
- grelina
- Correcto
- motilina
- somatostatina
- gastrina
10. La vena porta se forma gracias a la unión de la vena mesentérica superior con la vena:
- mesentérica inferior
- esplénica
- celiaca
- gástrica izquierda

CI 4

1. En el hígado, el aumento de la resistencia vascular en los sinusoides hepáticos


ocasionará:
- aumento de la presión de llenado vesicular
- salida de plasma hacia el intersticio
- aumento del flujo hacia la vena porta
- aumento del flujo hacia la vena cava superior
2. En un paciente con carcinoma de páncreas, el tumor ha invadido la unión entre la
venas esplénica y mesentérica superior; eso quiere decir que estamos seguros que el
tumor se encuentra a nivel del ________ del páncreas.
- cuerpo
- cola
- cabeza
- cuello
3. En un paciente con cirrosis hepática la cabeza de medusa que aparece en la pared
abdominal, podría desaparecer si al paciente se le:
- esclerosa las venas hemorroides internas
- oblitera el ligamento redondo
- administra antiandrógenos
- oblitera la arteria gástrica izquierda
4. En un paciente con intoxicación por órganos fosforados, la acción de la
colecistoquinina (CCK) está bloqueada a nivel de:
- el esfínter de Oddi
- la vesícula biliar
- la célula parietal
- el sistema nervioso central
5. Con respecto a la microestructura del hígado ¿Cuál de las siguientes alternativas es
correcta?
- La célula de Ito se encuentra en el espacio de Disse y reserva glucógeno
- Los hepatocitos están interconectados por uniones herméticas
- La célula de Kupffer se encuentra fuera del sinusoide y fagocita células
- El sinusoide es un capilar fenestrado
6. Paciente con Lupus Eritematosos que desarrolla hipertensión portal debido a
trombosis portal, es probable que desarrolle várices a nivel de:
- recto inferior
- recto superior
- hemorroides externas
- canal anal
7. El efecto de un medicamento colerético se evidencia por:
- la mayor producción de colesterol en la bilis
- la disminución de absorción de sales biliares
- el aumento de secreción biliar
- el aumento de formación de micelas
8. En un paciente con cáncer de páncreas y que desarrolla ictericia, la localización más
probable del tumor es en:
- la vesícula por metástasis
- el cuello del páncreas
- la cabeza del páncreas
- la cola del páncreas
9. En un recién nacido menor de 24 horas con atresia biliar, se encuentra elevación de la:
- bilirrubina directa
- bilirrubina indirecta
- alanina aminotransferasa (ALT)
- hemoglobina
10. ¿Cuál de las siguientes alternativas es correcta sobre la estructura hepática?
- Los colangiocitos producen bilis
- En la triada portal, se encuentra la vena derivada de la suprahepática
- El flujo sinusoidal en el lobulillo hepático es de adentro hacia afuera
- El flujo biliar en el lobulillo hepático es centrífugo
EXAMEN PARCIAL 2020-02

Pregunta 1

Niña de 6 años se asusta por que se le ha aflojado un diente deciduo. Este fenómenose produce
por:

laxitud del ligamento periodóntico

fractura del cemento

desmineralización del esmalte dental

aumento anómalo de la predentina

2.- ¿Cuál de las siguientes estructuras deriva del intestino anterior?


B

3.- En un paciente de 3 semanas de edad, con vómitos en proyectil, y nódulo epigástrico reptante,
usted esperaría encontrar:

engrosamiento de la circular interna pilórica

distensibilidad disminuida de la región oral del estómago

colecistoquinina aumentada

vómitos biliosos e intolerancia a los ácidos grasos

4.- Sobre el control de la peristalsis del tubo digestivo, ________________ es un mediador neural
que induce la relajación durante la peristalsis.

el péptido intestinal vasoactivo

la serotonina

la acetilcolina

la somatostatina

5.- En el plexo mientérico, el origen de los impulsos eferentes está en:

los ganglios paravertebrales

las células intersticiales de Cajal

el plexo de Meissner

el plexo de Aurbach
6.- Un familiar le comenta que tiene úlcera gástrica por exceso de producción de ácido; con sus
conocimientos del sistema digestivo, usted le recomendaría que reduzca el consumo de:

vitaminas

carbohidratos

agua

aminoácidos

7.- Las siguientes alternativas son ciertas en relación al mesenterio, EXCEPTO:

El omento menor deriva del mesenterio ventral

El ligamento esplenorrenal une al bazo con el riñón izquierdo

Los mesenterios cumplen la función de sostén y suspensión de órganos

Los omentos cumplen una función principal de irrigación visceral

8.- ¿Cuál de las siguientes alternativas es una característica de la estructura del esófago?

Aumenta la presión intra esofágica durante la inspiración

Contiene músculo estriado en casi toda su longitud

Contiene glándulas submucosas principalmente en su tercio distal

El esfínter esofágico inferior es un esfínter anatómico

9.- En una cirugía abierta (laparotomía), el cirujano al abrir la cavidad peritoneal por la parte anterior
(línea media), lo primero que observa es:

Estomago

Colon sigmoides

Duodeno

Epiplón mayor
10.- El tubo digestivo posee glándulas, las glándulas submucosas se encuentran en el:

duodeno y recto

íleon y esófago

estómago y duodeno

esófago y duodeno

11.- ¿Cuál de las siguientes alternativas detallan los músculos que ayudan a empujar el bolo hacia
la orofaringe?

Geniogloso y palatofaríngeo

Estilogloso y palatogloso

Hiogloso y geniogloso

Estilogloso y geniogloso

12.- Al ingresar líquidos o sólidos en la cavidad oral, un mecanismo que permite que una persona
respire mientras mastica es:

la elevación del paladar blando

el movimiento hacia afuera de los pliegues palatogloso y palatofaríngeo

la depresión de la parte posterior de la lengua

la depresión del paladar blando

13.- Durante el paso del bolo hacia la orofaringe, se desencadena una serie de contracciones
musculares que estrechan la cavidad faríngea. Estas contracciones están mediadas por el nervio
craneal:

IX

XI

XII
14.- Las siguientes alternativas son factores que determinan la patencia y función adecuada del
esfínter esofágico inferior, EXCEPTO:

Canales lentos de calcio

Plicatura diafragmática

Hipertrofia de la circular interna

Angulación con el estómago

15.- Durante una cena, una gestante inspira por la boca profundamente de manera frecuente; sin
embargo, el organismo evita que el aire ingrese al esófago por la acción:

del músculo cricofaríngeo

del istmo de las fauces

del esfínter esofágico inferior

de la epiglotis

16.- En un estudiante de medicina que está rindiendo un examen parcial, lo más probable es que en
ese momento su tránsito intestinal se encuentre:

muy lento

estimulado por acción de la sustancia P

muy acelerado

sin alteraciones

17.- El principal gobernante sobre todos los movimientos gastrointestinales es el sistema nervioso:
parasimpático

somático

mientérico

simpático
18.- ¿Cuál de las siguientes condiciones considera que es un trastorno de la musculatura lisa
esofágica?

Hipotonía de los constrictores faríngeos

Hipertonía del esfínter esofágico superior

Asinergia faringoesfinteriana

Acalasia

19.- El estímulo habitual para el movimiento peristáltico es:


acción de la sustancia P

contracción de la musculatura circular Interna

estimulación vago-vagal

distensión local

20.- La hormona responsable de los complejos migratorios interdigestivos tiene las siguientes
características, EXCEPTO:

se libera de forma cíclica

se produce en el estómago y el duodeno

cumple funciones de aumentar la motilidad y secreción gástrica e intestinal

es inhibida por el alimento

21.- Respecto al peristaltismo intestinal, para cumplir la ley del intestino , usted espera que a nivel
distal del quimo se libere:

acetilcolina

péptido liberador de gastrina (GRP)

sustancia P

péptido intestinal vasoactivo


22.- ¿Cuál de las siguientes alternativas es correcta sobre el control autónomo del aparato
gastrointestinal?

El plexo mientérico de Auerbach cumple funciones inhibitorias

Las terminaciones nerviosas simpáticas liberan mayor cantidad de adrenalina que noradrenalina

La estimulación simpática estimula a la muscularis mucosae

Al seccionar el vago, la inervación parasimpática del colon sigmoides disminuye

23.- La distención del yeyuno provoca que se:

despolarice el potencial de reposo de membrana

produzca una contracción tónica

disminuya el número de espigas

aumente la frecuencia de las ondas lentas

24.- El frenillo de los labios se encuentra en:

el dorso de la lengua

el piso de la boca

la cavidad oral

la cavidad vestibular

25.- Con seguridad, usted puede decir que la siguiente cicatriz postapendicectomía se encuentra en
la región denominada:

flanco derecho

hipocondrio derecho

flanco izquierdo

fosa iliaca derecha


Banqueo Digestivo

1. Dentro de las funciones del abdomen, se encuentra la defecació n y micció n, en las cuales la
presió n intra abdominal debe:
Aumentar

2. Paciente de 24 añ os con dolor abdominal tipo có lico intenso en mesogastrio. Segú n sus
conocimientos de macroestructura, el origen del dolor puede ser el ___________ :
Ileon

3. En la evaluació n de una tomografía abdominal, el interno observa un aneurisma en una arteria


que se dirige al riñ ó n derecho. Con seguridad se puede afirmar que está a nivel de la vé rtebra:
L1

4. Es inervado por aferentes somaticas


Peritoneo parietal

5. Paciente se queja de dolor en hipocondrio derecho, pero superficialmente. El dermatoma


relacionado es (marque la mejor respuesta):
T9

6. Al iniciar la digestió n, aumenta el consumo de oxígeno por la mucosa. Esto conlleva a una
hipoxia local, lo cual hace que se libere _____________, el cual produce vasodilatació n:
Adenosina

7. Respecto a la anatomía del estó mago, marque lo correcto:


El fondo gá strico forma la curvatura mayor

8. La motilidad intestinal es estimulada principalmente por el:


Plexo de Auerbach

9. Paciente con vó lvulo del colon sigmoides. La necrosis de este segmento del colon se produce
por una alteració n en la irrigació n de la arteria:
Mesenterica inferior

10. Marque la respuesta incorrecta


En todo el tubo digestivo, se observa dos capas de muscular propia: circular interna y
longitudinal externa

11. Al disminuir el pH duodenal por el HCl gá strico, se libera principalmente una hormona cuya
cé lula diana es:
Cé lula ductal del Wirsung

12. Al consumir un pan con mantequilla, la sensació n de hambre disminuye debido a la acció n de:
Colecistoquinina (CCK)
13. La rotació n en sentido longitudinal del estó mago en el desarrollo embrioló gico condiciona que
el nervio vago derecho quede a nivel:
Posterior

14. Una de las siguientes sustancias reguladoras, puede actuar de forma paracrina y como
hormona. Marque la correcta:
Somatostatina

15. De los diferentes reflejos gastrointestinales, hay uno que produce movimiento del contenido
hacia la regió n distal, y se llama reflejo:
Gastrocó lico

16. Todos los mú sculos motores de la lengua está n inervados por el XII par, excepto:
palatogloso

17. Al ingerir rá pidamente un litro de agua, usted esperaría que la gastrina aumente por efecto de:
ACh del sistema mienté rico

18. En un paciente con hiperestimulació n simpá tica se espera que las ondas lentas tengan un
ritmo:
Menor en íleon terminal que en el duodeno

19. Paciente con apendicitis aguda, que debuta con dolor en mesogastrio. Este dolor se debe a
estimulació n de receptores del dolor cuyas fibras van a viajar a la mé dula espinal a travé s de:
Nervios simpá ticos

20. Paciente obeso con Covid-19 es intubado por interno inexperto, quien al solicitar que bombeen
aire dentro del tubo endotraqueal, nota que el epigastrio se distiende. Al sospechar que ha
introducido el tubo en el estó mago, tambié n es cierto que:
Disminuye el pH gá strico

21. Cuando una persona coloca en su boca una sustancia con alta concentració n de carbohidratos,
lo que debería pasar es que:
disminuya la sensació n de hambre

22. El pH ó ptimo para la digestió n a nivel duodenal es ____________ y está regulado principalmente
por la liberació n de _________________
6-8 / secretina

23. Los corpú sculos gustativos se encuentran en la lengua, pero ademá s se les puede encontrar en:
el paladar blando

24. Al ingresar líquidos o só lidos en la cavidad oral, un mecanismo que permite que una persona
respire mientras mastica es:
la depresió n del paladar blando

25. Al rozar agua caliente en la punta de la lengua, usted esperaría que el estímulo viaje a travé s del
nervio:
lingual

26. Sobre el control de la peristalsis del tubo digestivo, ________________ es un mediador neural que
induce la relajació n durante la peristalsis.
el pé ptido intestinal vasoactivo
27. ¿Cuá l de las siguientes alternativas es correcta sobre el control del peristaltismo?
El peristaltismo intestinal aumenta por efecto de la colecistoquinina (CCK)

28. ¿Cuá l de las siguientes alternativas es una característica de la estructura del esó fago?
Contiene glá ndulas submucosas principalmente en su tercio distal

29. En la estructura dentaria, se observa que hay una composició n muy similar a la del hueso en la
capa denominada:
cemento

30. El tubo digestivo posee glá ndulas, las glá ndulas submucosas se encuentran en el:
esó fago y duodeno

31. Estas diseñ ando un proyecto de investigació n sobre los niveles de colesterol que se absorben
luego de una comida grasosa y deseas cuantificar la cantidad de colesterol que es absorbido por
el intestino antes que el hígado lo metabolice ¿de cuá l de los siguientes vasos obtendrías la
muestra para tu aná lisis?
Conducto torá cico

32. Mujer de 30 añ os llega a emergencia con dolor en hipogá strico. Al examen físico presenta una
masa palpable de 10 cm de diá metro a en la misma regió n, usted sospecharía de las siguientes
condiciones, EXCEPTO:
Tumor renal

33. Las siguientes alternativas son factores que determinan la patencia y funció n adecuada del
esfínter esofá gico inferior, EXCEPTO:
Hipertrofia de la circular interna

34. Cuando el istmo de las fauces se cierra, se evita que el alimento pase hacia la orofarínge y
permite respirar mientras se mastica. Este cierre se debe a la contracció n y aproximació n de los
mú sculos:
palatoglosos

35. Las siguientes alternativas son ciertas sobre la actividad elé ctrica del mú sculo gastrointestinal,
EXCEPTO:
La despolarizació n lenta se debe principalmente al ingreso de Na+

36. ¿Cuá l de las siguientes alternativas es correcta sobre el movimiento peristá ltico?
El contenido intestinal avanza só lo 5-10 cm

37. La informació n sensitiva aferente del sistema gastrointestinal pasa por las siguientes
estructuras, EXCEPTO:
Tá lamo

38. Con respecto a la regulació n del peristaltismo, al aplicarle atropina (antagonista coliné rgico) a
un paciente, es de esperarse que el peristaltismo:
disminuya

39. ¿Cuá l de las siguientes alternativas es correcta sobre el control autó nomo del aparato
gastrointestinal?
La estimulació n simpá tica estimula a la muscularis mucosae
40. Respecto al peristaltismo intestinal, para cumplir la ley del intestino , usted espera que a nivel
distal del quimo se libere:
pé ptido intestinal vasoactivo

41. ¿Cuá l de las siguientes alternativas estimula las ondas de motilidad gastrointestinal
denominadas complejos migratorios interdigestivos?
Eritromicina

42. Las siguientes alternativas son ciertas sobre las contracciones tó nicas del mú sculo
gastrointestinal, EXCEPTO:
Tienen relació n con el ingreso persistente de iones sodio

43. El peristaltismo depende que a nivel distal del bolo se secrete:


pé ptido intestinal vasoactivo por neuronas

44. Señ ale cuá l de las estructuras que en el embrió n se encuentra comunicada con el saco vitelino
por medio del conducto onfalomesenté rico:
A
45. Es un derivado del mesenterio dorsal:
D

46. El ecografista sabe que pude visualizar el nacimiento de la arteria


mesenterica superior, debe colocar el transductor sobre la piel de la sgte region abdominal:
Epigastrio

47. Durante una cirugia oncologica, el cirujano observa que los organos abdominales tienen libre
movimiento dentro de la cavidad abdominal, excepto:
Colon ascendente

48. Al introducir una solucion azucarada directamente al estomago mediante una gastrometria
(comunicación entre la piel abdominal y el estomago), la sustancia que provocara que
aumente los niveles sericos de insulina es:
Peptido tipo glucagon (GLP-1)

49. Al ingerir grandes cantidades de dulces, con la subsecuente estimulacion de incretinas, usted
esperaria que el apetito ___________ debido a ___________
Disminuya / insulina

50. En un paciente con hiperestimulacion simpatica se espera que las ondas lentas tengan un
ritmo:
Menor en ileon terminal que en el duodeno

51. En un paciente con schok distributivo, usted decide iniciar noradrenalina por un cateter CVC,
consiguiendo aumentar la PA. ¿Qué efecto sobre la motilidad intestinal espera encontrar?
El potencial de reposo de las fibras musculares se hace mas negativo

52. La hormona que tiene un efecto sinergico con la secretina para optimizar el pH duodenal y la
digestion es:
CCK

53. Paciente joven que es traido a emergencia con abdomen agudo quirurgico debido a herida
contusopenetrante por verduguillo (alambre grueso con punta aguzada) recibida en una
pelea despues de un partido de futbol. Se observa herida en hipocondrio izquierdo. El organo
que debe estar sangrando y produciendo hemoperitoneo es:
Bazo

54. Paciente se queja de dolor en hipocondrio derecho, pero superficialmente. El dermatoma


relacionado es:
T9
55. Paciente de 24 años con dolor abdominal tipo colico intenso en mesogastrio. Según sus
conocimientos de macroestructura, el origen del dolor puede ser el _____
Ileon

56. Paciente tiene una ulcera sangrante en el segundo tercio del yeyuno. La arteria de la cual
proviene la sangre arterial para dicha zona es la art.:
Mesenterica superior

57. Durante el vomito, ¿el contenido gastrico tiene que pasar necesariamente por cual estructura
para llegar al esofago?
Cardias

58. La peristalsis o peristaltismo hace referencia a:


Motilidad para movilizar el alimento proximal a distal

59. Respecto a la anatomia del estomago, marque lo correcto:


El fondo gastrico forma la curvatura mayor

60. Al iniciar la digestion, aumenta el consumo del oxigeno por la mucosa. Esto conlleva a una
hipoxis local, lo cual hace que se libere ______, el cual produce:
Adenosina

61. La motilidad intestinal es estimulada parcialmente por el:


Plexo de Auerbach

62. En un paciente con falla en la fusion de los conductos de las yema ventral y dorsal del
pancreas, usted esperaria encontrar:
Drenaje de la mayor parte del jugo pancreatico en la papila menor

63. ¿Cuál de los sgtes alternaticas detallan los músculos que ayudan a empujar el bolo hacia la
orofaringe?
Estilogloso y palatogloso
64. En el esofago, el plexo mienterico o de Auerbach interviene en el _________ de los impulsos
_________
Fin / eferentes

65. Sobre el control de la peristalsis del tubo digestivo, ____________ es un indicador neural que
induce la relajacion durante la peristalsis.
El peptido intestinal vasoactivo

66. Las sgtes alternativas son correctas sobre la deglucion, EXCEPTO:


Consta de 2 fases

67. Paciente adulto con reflujo gastroesofagico es mas probable que presente:
Descalcificacion del esmalte

68. En una cirugia abierta (laparotomia), el cirujano al abrir la cavidad peritoneal por la parte
anterior (linea media), lo primero que observa es:
Epiplon mayor

69. Durante la deglucion, el bolo es impedido de ingresar a la cavidad oral gracias a la accion de
diversos músculos, entre ellos el musculo:
Palatofaringeo

70. Durante una cena, una gestante inspira por la boca profundamente de manera frecuente, sin
embargo, el organismo evita que el aire ingrese al esofago por la accion:
Del musculo cricofaringeo

71. Las sgtes alternativas son factores que determinan la patencia y funcion adecuada del esfinter
esofagico inferior, EXCEPTO:
Hipertrofia de la circular interna

72. La distencion del yeyuno provoca que se:


Despolarice el potencial de reposo de la membrana

73. El estimulo habitual para el movimiento peristaltico es:


Distencion local
74. La hormona responsable de los complejos migratorios interdigestivos tiene las siguientes
caracteristicas, EXCEPTO:
Cumple funciones de aumentar la motilidad y secrecion gastrica e intestinal

75. Con respecto a la regulación del peristaltismo, al aplicarle atropina (antagonista colinergico) a
un paciente, es de esperarse que el peristaltismo:
Disminuya
76. ¿Cuá l de las siguientes alternativas es correcta sobre la motilidad esofagica?
Las ondas secundarias son propulsoras y no van precedidas de deglucion

77. El principal gobernante sobre todos los movimientos gastrointestinales en el sistema


nervioso:
Mienterico

78. El peristaltismo intestinal se produce gracias a un reflejo que:


Se origina dentro de la pared intestinal

79. Un familiar le comenta que tiene ulcera gastrica por exceso de produccion de acido, con sus
conocimientos del sistema disgestivo, usted le recomendaria que reduzca el consumo de:
Aminoácidos

80. Un paciente con polimiositis posee alteración en la regulación del mecanismo de la deglución;
por eso hay que considerar la deglución de la saliva, pues a diario se produce _______mL
1000

81. En la producción de HCl, la acción de la somatostatina disminuye la accion de:


La gastrina

82. Al usar un parasimpaticomimético (agonista colinérgico), usted espera que la saliva presente:
Una mayor cantidad de sodio

83. Al evaluar a un paciente con parotiditis purulenta (con absceso), usted buscaría el orificio
terminal del conducto de _____________ a la altura de _____________
Stenon / la segunda molar superior

84. En cuanto a las sustancias secretadas por el estómago ¿Cuál de las siguientes sustancias
estimula la liberación de pepsinógeno?
Secretina

85. Para determinar que un paciente tiene esófago de Barrett, debemos encontrar ___________ en la
biopsia de esófago.
Células caliciformes

86. Al usar atropina en un paciente, usted esperaría:


la disminución de secreción gástrica por bloqueo de M3

87. El conducto de Stenon, para entrar a la cavidad vestibular, debe atravesar el músculo:
Buccinador

88. Las células enteroendocrinas en el estómago se localizan en la glándula oxíntica, al mismo


nivel que las células:
Principales
89. La inhibición de secreción gástrica es secundaria a:
El reflejo enterogastrico

90. En un paciente con infección por SARS-CoV-2 con compromiso severo e ingresado en la
unidad de cuidados intensivos (UCI), debido a la tormenta de citoquinas y a la desregulación
inmune ¿Cuál de las siguientes alternativas estará elevada en sangre al evaluar el perfil
hepático?
Transaminasas

91. Un recién nacido con enfermedad congénita, tiene una mutación de la proteína MRP2
encargada del transporte de la bilirrubina conjugada hacia el interior del canalículo. Antes de
las 24 horas de nacido presenta ictericia y además presentará con mayor probabilidad:
coluria

92. El efecto de un medicamento colerético se evidencia por:


el aumento de secreción biliar

1. En un paciente cirrótico con encefalopatía hepática ¿cuál de las siguientes alternativas sustenta
la reducción de carnes rojas en la dieta?
El amonio se produce principalmente en el intestino

2. En un paciente con cirrosis hepática la cabeza de medusa que aparece en la pared abdominal,
podría desaparecer si al paciente se le:
oblitera el ligamento redondo

3. En un paciente con cirrosis hepática se desarrollará ascitis debido a los siguientes


mecanismos, EXCEPTO:
Estrechamiento de la porta

4. Para poder absorber al torrente sanguíneo todos los carbohidratos de la lactosa, es necesario
usar únicamente los transportadores:
SGLT1, GLUT2

5. ¿Cuál de las siguientes alternativas es una comunicación entre el tejido hepático y la vesícula
biliar?
Conducto de Luschka

6. La bilis que sale de la vesícula biliar tiene como componente principal:


Acidos biliares

7. La relajación del esfínter de Oddi se produce directamente por acción de:


Peptido vasoactivo intestinal (VIP)

8. En un paciente con xerostomia presenta las sgtes condiciones, EXCEPTO:


Infecciones del oido a repeticion

9. Los pliegues gastricos gruesos son practicamente inexistente a nivel de:


Fonfo

10. La vena porta de forma gracias a la union de la vena mesenterica superior con la vena:
Esplenica
11. En caso se produzca la perforacion de la cara anteiror del estomago, esta perforacion
ocasionara _____ producto de la peritonitis quimica.
Ileo intestinal

12. Cuando un paciente recibe estimulo autonomo mixto (simpatico y parasimpatico), el flujo de
saliva:
Aumenta en relacion basal

13. En una gestante de 11 semanas con antecedentes de esofagitis eosinofilica, con hiperemesis
gravidica, que acude a emergencia por hematemesis leve, y presenta subitamente disnea y
dolor toracico. Usted sospecharia de:
Síndrome de Boerhaave

14. Sangre q lleva la vena porta es tipo:


Venosa

15. Aquellas cells q producen mayor cantidad de moco en el epitelio gastrico son las:
Mucosas superficiales

16. Dentro de las patologias q producen sangrado en el esofago, la que sangra mas es:
Síndrome de Mallory Weiss

17. En las glandulas salivales, el principal lugar de intercambio ionico se da a nivel del:
Conducto estriado

18. Via biliar extrahepatica se encuentra ubicada dentro del ligamento:


Hepatoduodenal

19. En un paicente con intoxicacion de organso fosforados, la accion de la colecistoquinina (CCK)


esta bloqueada a nivel de:
Esfinter de Oddi
93. En un paciente con carcinoma de pancreas, el tumor ha invadido la union entre la vena
esplenica y mesenterica superior, eso quiere decir que estamos seguros q el tumor se
encuentra a nivel del _____ del pancreas.
Cuello

94. ¿Cuál de las sgtes alternaticas es correcta sobre la estructura hepatica?


El flujo biliar en el lobulillo hepatico es centrifugo

95. En una paciente con una estenosis severe del coledoco por una complicacion quirurgica,
usted esperaria q desarrolle:
Aumento del tiempo de protrombina

96. Los hepatocitos tienen gran capacidad regenerativa, en parte gracias a las cells madre
hepaticas q se localizan en los:
Canales de Herning
97. Paciente con Lupus eritematosos q desarrolla hipertension portal debido a trombosis portal,
es probable que desarrolle varices a nivel de:
Recto superior

1. En todo el sistema gastrointestinal se encuentra diferentes tipos de músculos ¿En cuales estructuras
encontraríamos músculo estriado?
Faringe, esfínter esofágico superior, esfínter anal externo.

2. Durante la fase …..……………….. el paladar blando es traccionado hacia arriba.


faríngea

3. El tubo digestivo a nivel del ………………………….. está revestido internamente por epitelio ………….
Estómago / cilíndrico simple

4. Señale la estructura del tubo digestivo que presenta tres capas musculares en su pared.
Estómago

5. Paciente de 47 años con sobrepeso, acude a consulta por presentar dolor tipo cólico en el cuadrante
superior derecho del abdomen. El dolor aumenta luego de ingesta de comidas con abundante grasa.
Esta situación se explica por el aumento en la secreción de …………………………. que estimula la
contracción de la vesícula biliar.
Colecistoquinina

6. Si hay un aumenta de la hormona grelina a nivel del hipotálamo ¿Cuál es la consecuencia en el


organismo?
Hambre

7. Señale el péptido gastrointestinal que produce relajación del músculo liso gastro-intestinal.
Péptido intestinal vasoactivo (PIV)

8. ¿Cuál de las siguientes alternativas inhibiría la relajación receptiva a nivel del estómago?
Bloqueo del nervio vago (X)

9. Una mujer de 28 años, con diagnóstico de Diabetes Mellitus Tipo 1, acude por presentar desde hace
10 años estreñimiento y distensión abdominal. Se realiza estudio y se determina que la paciente
presenta un retraso del vaciamiento gástrico debido a gastroparesia diabética. ¿Cuál de los
siguientes hechos aumentaría el tiempo del vaciamiento gástrico?
Ácidos grasos en el duodeno

10. Experimentalmente, se aplica un inhibidor selectivo del péptido intestinal vasoactivo (PIV) durante
la contracción peristáltica del intestino delgado. ¿Cuál es el efecto de este inhibidor en la motilidad
del intestino delgado?
Disminución del tránsito intestinal
11. ¿Cuál de los siguientes eventos ocurre durante la defecación?
Relajación del esfínter anal externo

12. Señale el péptido que cumple la función de disminuir la ingesta de alimentos.


Proopiomelanocortina (POMC)

13. Indique el lugar de secreción del péptido YY


Estómago

14. La grelina es secretada en el estómago y estimula a las neuronas del núcleo ………………….. para la
estimulación de la secreción de……………..
arqueado / neuropéptido Y.

15. Considerando el desarrollo embriológico del intestino medio. ¿Qué evento de gran importancia se
produce en la sexta semana?
Salida temporal de asas intestinales a través de cordón umbilical

16. En el ……………………………. se presentan los movimiento de …………………….


intestino delgado / segmentación y peristaltismo

17. Marita sufre una parálisis del músculo masetero. ¿Qué limitaciones se producirán a nivel del
movimiento de la mandíbula?
Elevación

18. Señale el movimiento que se produce a nivel del ciego y colon proximal que tiene como finalidad
favorecer la absorción de agua y sales.
De mezcla

19. La enfermedad de Hirchsprung se caracteriza por una dilatación anormal de colon y disminución de
los movimientos propulsivos que traerá como principal consecuencia …………
estreñimiento crónico.

20. ¿Cuál de los siguientes factores estimulan el vaciamiento gástrico?


Secretina

21. La secreción de ………………………. estimula la motilidad gástrica.


Motilina

22. La hormona ……………………….. es secretada por las células “I” del …………..
Colecistoquinina / duodeno y yeyuno
23. El reflejo enterogástrico aumenta ……………………… debido a la…………………..
el tono del esfínter pilórico / distensión del duodeno.

24. ¿Cuál de las siguientes sustancias disminuye el vaciado gástrico?


Colecistocinina.

25. El plexo mientérico se encuentra ubicado en………………. y funciona controlando la ………………


entre las capas musculares / motilidad

26. ¿Cuál de las siguientes glándulas está conformada por glándulas serosas y secreta la mayor
cantidad de amilasa en la boca?
Parótidas

27. ¿Cuál de los siguientes factores es más importante para percepción mediante las papilas
gustativas?
Agua

28. Paciente obeso, fumador, consumidor de alcohol que acude por presentar molestias como disfagia,
ardor retroesternal, alteración del sueño y sensación de boca amarga en las mañanas. ¿Cuál de las
siguientes recomendaciones podría darle a este paciente?
Elevar la cabecera de la cama al dormir.

29. En la enfermedad por reflujo se encuentra alterada la función del esfínter …………………….
esofágico inferior

30. En la diarrea psicógena hay una estimulación del ……………….. que …………………….. la motilidad
gastrointestinal.
Sistema nervioso parasimpático / aumenta

Producto de la hipertrofia pilorica uno espera encontrar q los niveles de Gastrina se encuentren:

Elevados

Al realizar una pilorotomia (corte del piloro para descomprimir) ¿Q hormonas se comezara a
liberar rapidamente en respuesta a este tratamiento?

Colecistoquinina

Debido al signo de onda peristaltica, se debe asumir q como consecuencia hay un aumento en la
liberacion de _____ en la pared del estomago:
Adenosina

La oliva pilorica se logra palpar a nivel de:

L1

El uso de atropina se espera q disminuya la liberacion de:

Enzimas pancreaticas

El consumo de una pequena cantidad de gaseosa aumentara directamente la concentración serica


de cual de las sgtes hormonas:

Peptido 1 similar al glucagon (GLP-1)

La colecistoquininca inhibe: el vaciamiento gastrico

Paciente con Lupus Eritematosos que desarrolla hipertensión portal debido a trombosis portal, es
probable que desarrolle várices a nivel de: → recto superior

En el síndrome de Mirizzi, el paciente tiene cálculos en la vesícula biliar; pero se obstruye el


conducto hepático común debido a que un cálculo se ubica y crece de tamaño en: → la bolsa de
Hartmann

En un recién nacido menor de 24 horas con atresia biliar, se encuentra elevación de la: →
bilirrubina directa

En el hígado, el aumento de la resistencia vascular en los sinusoides hepáticos ocasionará → salida


de plasma hacia el intersticio

Un paso importante para la poder absorber los lípidos es la emulsificación de las grasas. ¿Cuál de
las siguientes alternativas es el principal factor encargado de dicha emulsificación? → Ácidos
biliares

La vía biliar extrahepática se encuentra ubicada dentro del ligamento: → Hepatoduodenal

¿Cuál de las siguientes alternativas es correcta sobre la estructura hepática? → El flujo biliar en el
lobulillo hepático es centrífugo

En un paciente cirrótico con encefalopatía hepática ¿cuál de las siguientes alternativas sustenta la
reducción de carnes rojas en la dieta? → El amonio se produce principalmente en el intestino
La relajación del esfínter de Oddi se produce directamente por acción de: → el péptido vasoactivo
intestinal (VIP)

C! 1
1) El dolor periumbilical o epigástrico en el inicio de una apendicitis aguda se debe a:
• - Irritación del peritoneo parietal
• - Estímulo del sistema simpático
• - Íleo secundario
• - Estímulo del nervio vago

2) ¿Cuál de los siguientes péptidos inhibe el vaciamiento gástrico?


• - Péptido inhibidor gástrico
• - motilina
• - Gastrina
• - Colecistoquinina

3) El uso de Ranitidina bloquea el receptor H2 de la histamina en las células


parietales. La histamina llega a estas células por:
• - difusión
• - vía hematógena
• - la luz gástrica

4) El consumir caramelos indirectamente activa la vía:


• - POMC/CART
• - AGRP/NPY
• - grelina
• - MCR-4

5) Entre las múltiples causas de la Enfermedad por Reflujo Gastroesofágico, se puede


considerar también a una alteración en las ______________ del esfínter esofágico
inferior:
• - ondas secundarias
• - contracciones tónica
• - ondas lentas

6) El aumento en la actividad motora de la pared gástrica genera un aumento en los


niveles locales de qué sustancia en la microvasculatura:
• - adenosina
• - CCK
• - endotelina
• - gastrina

7) Durante una cirugía oncológica, ¿la extirpación de cuál de los siguientes órganos se
vería comprometida por la presencia de adventicia?:
• - yeyuno
• - estómago
- recto
- vesícula biliar

8) En un paciente con apendicitis aguda, la sensación de dolor producido por esta


inflamación es llevada por el nervio:
• - vago
• - esplácnico menor
• - esplácnico mayor
• - pélvico

9) ¿Cuál de los siguientes líquidos corporales tiene el pH más alto?:


• - saliva
• - jugo gástrico
• - jugo pancreático
• - bilis en la vesícula biliar

10) Marque la respuesta correcta en relación a la gastrina:


• - Al distender el estómago, se inhibe su producción.
• - Las células G son las productoras y se encuentran principalmente en el antro
gástrico
• - Se estimula por la liberación de noradrenalina
• - Las células G se encuentran principalmente en el fondo gástrico

C2
1) Respecto a la anatomía del estómago, marque lo correcto:
• - la arteria gástrica derecha nace de la arteria hepática común
• - No contiene muscular de la mucosa
• - la arteria gástrica izquierda irriga la curvatura mayor del estómago
• - Tiene una capacidad de 3 litros en la mayoría de personas

2) Respecto a las enfermedades del esófago, marque lo correcto:


• - el diagnóstico diferencial de la acalasia es la enfermedad de Chagas
esofágica
• - el síndrome de Boeerhave tiene mejor pronóstico que el síndrome de Mallory
Weiss
• - el síndrome de Mallory Weiss raramente se presenta con hematemesis
• - un agente infeccioso común del esófago es el Streptococo beta hemolítico

3) Estimula la producción de saliva:


• - atropina
• - vasodilatación periglandular
• - expresión de miedo
• - fatiga o cansancio

4) Respecto a la motilidad del colon, marque lo correcto:


• - Con la distención del estómago, suelen aparecer movimientos en masa
• - Se producen contracciones segmentarias principalmente en el colon
izquierdo
• - Los movimientos de masa se encargaran del mezclado de las heces
• - Las haustras del colon contribuyen al reflejo de defecación

5) En cuanto a la saliva, marque lo correcto:


• - La amilasa cumple función digestiva
• - el sistema simpático disminuye su secreción
• - cuando hay un estímulo de mayor secreción, se vuelve hipertónica
• - se producen 200 m diarios en condiciones normales
• - La amilasa cumple función digestiva

6) La saliva puede tener una variedad de electrolitos en su composición. Entre ellos el


cloro, respecto al cual se puede afirmar:
• - Su concentración no llega a ser tan alta como en el plasma
• - Su menor concentración se alcanza con flujo alto
• - Su mayor concentración se consigue con flujo bajo
• - Con flujo alta, su concentración es mayor que la del plasma

7) En un paciente hipertenso con tratamiento a base de beta bloqueadores, la


producción de saliva se espera que:
• - disminuya
• - no se afecte
• - aumente
• - disminuya
• - sólo se afecta si los receptores de acetilcolina están activos

8) Durante la secreción de saliva, es de esperarse que las concentraciones de


________ y ______ disminuyan al disminuir el flujo:
• - potasio de cloro
• - potasio sodio
• - bicarbonato potasio
• - sodio bicarbonato

9) El omeprazol actúa sobre la membrana _____________ de la célula ____________


• - basolateral / apical
• - basolateral / parietal
• - apical / principal
- apical / parietal

10) Dentro de los factores protectores de la mucosa gástrica se pueden mencionar


múltiples protagonistas. Uno de ellos es:
• - CCK

• - gastrina

• - receptor muscarínico
• - pepsina

C3
1) Si existe un fármaco con acción colerética, se asume que se refiere a que potencia
o estimula la:
• - Secreción de enzimas biliares
• - Contracción de la vesícula biliar
• - Excreción fecal de sales biliares
• - Recirculación de sales biliares

2) Una mujer de 38 años ingresa en el hospital con signos de colecistitis y cálculos


biliares. Durante la colangiografía, se inserta el catéter en la vesícula biliar con mucha
dificultad. ¿Cuál de las estructuras interfiere con más probabilidad con el paso del
catéter por el conducto cístico?
• - Válvula espiral (de Heister)
• - Compresión de la vena porta del conducto cístico
• - Adherencias del ligamento hepatoduodenal
• - Compresión del conducto cístico por una arteria hepática.

3) El acino pancreático difiere con el de las glándulas salivales en:


• - Contiene celulares centroacinares
• - No produce secreción serosa
• - El páncreas produce principalmente secreción mucosa
• - no tiene diferencias
4) En relación a la circulación hepática, marque lo correcto:
• - Los sinusoides hepáticos transportan sangre mixta
• - La vena porta proporciona el 50% e sangre al hígado
• - La vena porta se forma a partir de la vena esplénica y la mesentérica inferior
• - La arteria hepática deriva de la mesentérica superior

5) Hombre de 65 años con isquemia intestinal leve por oclusión aterosclerótica de la


arteria mesentérica superior, pero la irrigación sanguínea colateral ha retrasado el
inicio de la necrosis. ¿Qué vasos ofrecen colaterales entre el tronco celíaco y la arteria
mesentérica superior?
• - Gastroomental derecha e izquierda
• - Gástrica izquierda y hepática.
• - Pancreaticoduodenal superior e inferior.
• - Cística y gastroduodenal

6) El GALT se localiza en:


• - borde en cepillo
• - lamina propia
• - submucosa
• - superficie de criptas de Lieberkun

7) ¿Por cuál de las siguientes células es secretada principalmente la pro enzima


procarboxipeptidasa?
• - acinares del páncreas
• - ductales del páncreas
• - centro acinares del páncreas
• - epiteliales del duodeno

8) Marque lo correcto:
• - Las venas sublobulillares desembocan en las venas hepáticas
• - Las venas hepáticas contienen válvulas
• - Usualmente, son dos grandes venas suprahepáticas
• - Ninguna es correcta

9) Una mujer de 49 años ingresa en el hospital con dolor en epigástrico que migra
hacia el lado derecho y atrás hacia la escápula, sin ictericia. La ecografía muestra un
gran cálculo biliar. ¿En cuál de las siguientes estructuras es más probable que se
localice el cálculo biliar?
• - conducto hepático derecho
• - conducto colédoco
• - bolsa de Hartmann
• - conducto hepático izquierdo

10) Cuando el alimento se encuentra en el estómago, se produce la liberación de


enzimas pancreáticas básicamente debido a la acción de:
• - vago
• - bombesina
• - CCK
• - secretina

C4
1) Paciente de 21 años con dolor intenso en arcada dentaria superior debido a
emergencia de la tercera molar. Frecuencia cardíaca 108 latidos por minuto,
PA: 140/80 mmHg, agitado, pálido, son sudoración fría, y boca con saliva
espesa. En este paciente se espera que la secreción exocrina pancreática se
encuentre:
o - disminuida
o - aumentada
o - extremadamente elevada
o - sin cambios

2) Las sales biliares son absorbidas a la sangre en :


o - íleon
o - yeyuno
o - colon proximal
o - duodeno

3) Un signo característico en pacientes con encefalopatía hepática es:


o - convulsiones tónicas
o - reflejo de Babinsky
o - Onicomicosis
o - Asterixis

4) El efecto colerético de las sales biliares se refiere a:


o - estimulo de secreción biliar
o - inhibición de secreción biliar
o - aumento de la bilirrubina indirecta
o - ninguna de las anteriores

5) Paciente de 42 años con adenocarcinoma ductular. La TC ha demostrado


claramente que el tumor está en el cuello del páncreas y que hay un gran vaso
ocluido. ¿Cuál de los siguientes vasos estaría más probablemente obstruido?
o - arteria mesentérica superior
o - arteria pancreática magna
o - vena mesentérica inferior
o - vena porta

6) La secreción de agua y bicarbonato por el páncreas exocrino se da


básicamente en la fase:
o - intestinal
o - gástrica
o - en las tres por igual
o - oral

7) En relación a la secreción biliar y su composición, marque lo correcto:


o - las sales biliares corresponde al 50% de su composición
o - se producen alrededor de 3 litros diarios
o - los fosfolípidos excretados son anfipáticos, a diferencia de las sales
biliares
o - participan principalmente en la digestión de carbohidratos

8) Paciente con tumor neuroendocrino productor de secretina, debido a lo cual se


puede esperar que su secreción pancreática, comparada con la de una
persona sana en estado de bajo flujo, tenga una concentración de:
o - sodio aumentada
o - potasio disminuida
o - bicarbonato aumentada
o - igual

9) La vena central terminal desemboca en:


o - venas sublobulillares
o - vena porta
o - sinusoides hepáticos
o - espacio de Disee

10) Las invaginaciones del epitelio de la vesícula biliar que se extiende incluso
hasta la muscular se denomina:
o - conductos de Lushka
o - senos de Rokitansky-Aschoff
o - apéndices epiploicos
o - divertículos biliares

PARCIAL 2020-02

1.- Con respecto a la regulación del peristaltismo, al aplicarle atropina (antagonista colinérgico) a un
paciente, es de esperarse que el peristaltismo:

estimule la acción de los receptores dopaminérgicos

disminuya

se mantenga sin alteración

aumente

2.- Respecto al peristaltismo intestinal, para cumplir la ley del intestino , usted espera que a nivel
distal del quimo se libere:

péptido liberador de gastrina (GRP)

sustancia P

acetilcolina

péptido intestinal vasoactivo

3.- El estímulo habitual para el movimiento peristáltico es:

acción de la sustancia P

estimulación vago-vagal

contracción de la musculatura circular Interna


distensión local

4.- ¿Cuál de las siguientes condiciones considera que es un trastorno de la musculatura


lisaesofágica?

Hipotonía de los constrictores faríngeos

Asinergia faringoesfinteriana

Hipertonía del esfínter esofágico superior

Acalasia

5.- En un estudiante de medicina que está rindiendo un examen parcial, lo más probable esque en
ese momento su tránsito intestinal se encuentre:

sin alteraciones

estimulado por acción de la sustancia P

muy lento

muy acelerado

6.- El peristaltismo intestinal se produce gracias a un reflejo que:

llega a los ganglios pre vertebrales

llega al sistema nervioso central

se origina dentro de la pared intestinal

llega a la médula espinal

7.- Las siguientes alternativas son ciertas sobre las contracciones tónicas del músculo
gastrointestinal, EXCEPTO:

Tienen relación con el ingreso persistente de iones sodio

Se encuentran principalmente en esfínteres

Obedece a una mayor frecuencia de potenciales en espiga

Tienen regulación hormonal


8.- La información sensitiva aferente del sistema gastrointestinal pasa por las siguientesestructuras,
EXCEPTO:

Tálamo

Médula espinal

Ganglios prevertebrales

Tronco encefálico

9.- Al realizar una vagotomía por úlcera péptica, usted esperaría una disminución deproducción de
gastrina debido a la:

menor distensión de las paredes gástricas

ausencia de acetilcolina vagal

ausencia de histamina

ausencia de bombesina vagal

10.- Las siguientes funciones son inhibidas por la hormona somatostatina, EXCEPTO:

Secreción de enzimas pancreáticas

Secreción gástrica de HCl

Liberación de gastrina

Motilidad intestinal

11.- En una cirugía abierta (laparotomía), el cirujano al abrir la cavidad peritoneal por la parteanterior
(línea media), lo primero que observa es:

Epiplón mayor

Estomago

Duodeno

Colon sigmoides

12.- ¿Cuál de las siguientes estructuras tiene inervación somática?

Estómago
Peritoneo parietal

Peritoneo visceral

Mesosigmoides

13.- En el plexo mientérico, el origen de los impulsos eferentes está en:

el plexo de Meissner

las células intersticiales de Cajal

los ganglios paravertebrales

el plexo de Aurbach

14.- En el esófago, el plexo mientérico o de Auerbach interviene en el _______ de losimpulsos


______

inicio / aferentes

fin / eferentes

fin / aferentes

inicio / eferentes

15.- Paciente con lesión del hipogloso del lado izquierdo. Para evaluarlo se le pide al pacienteque
saque la lengua, la cual se espera que la punta de la lengua se dirija hacia:

abajo

el lado izquierdo

adelante

el lado derecho

16.- Al ingresar líquidos o sólidos en la cavidad oral, un mecanismo que permite que unapersona
respire mientras mastica es:

la elevación del paladar blando

la depresión de la parte posterior de la lengua

el movimiento hacia afuera de los pliegues palatogloso ypalatofaríngeo


la depresión del paladar blando

17.- Paciente adulto con reflujo gastroesofágico es más probable que presente:
destrucción de ameloblastos

remodelación del esmalte

descalcificación del esmalte

desfluorización de los dientes

18.- En un paciente de 3 semanas de edad, con vómitos en proyectil, y nódulo epigástricoreptante,


usted esperaría encontrar:

engrosamiento de la circular interna pilórica

colecistoquinina aumentada

distensibilidad disminuida de la región oral del estómago

vómitos biliosos e intolerancia a los ácidos grasos

19.- ¿Cuál de las siguientes atresias/fístulas traqueo esofágicas considera usted que esincompatible
con la vida (de no recibir tratamiento)?

Tipo E

Tipo B

Tipo C

Tipo A

20.- El músculo liso gastrointestinal funciona como un sincitio debido a:


el calcio

las uniones en hendidura

las fibras musculares más largas

el plexo mientérico de Auerbach

21.- Durante el paso del bolo hacia la orofaringe, se desencadena una serie de
contraccionesmusculares que estrechan la cavidad faríngea. Estas contracciones están mediadas
porel nervio craneal:
XII

IX

XI

22.- Las siguientes alternativas son ciertas sobre la actividad eléctrica del músculo gastrointestinal,
EXCEPTO:

Si el potencial de membrana es más positivo, habrá mayorfrecuencia de espigas

Las ondas lentas no son potenciales de acción

Las ondas lentas son más frecuentes en el duodeno

La despolarización lenta se debe principalmente al ingreso de Na+

23.- Los corpúsculos gustativos se encuentran en la lengua, pero además se les puede encontrar
en:

el dorso de la lengua

las encías

el paladar blando

el paladar duro

24.- Mujer de 30 años llega a emergencia con dolor en hipogástrico. Al examen físico presentauna
masa palpable de 10 cm de diámetro a en la misma región, usted sospecharía de lassiguientes
condiciones, EXCEPTO:

Tumor renal

Embarazo

Tumor uterino

Cáncer de recto superior

25.- La masticación es básicamente:


importante para la digestión sobretodo de carnes

una actividad consciente


un movimiento reflejo

un ralentizador del vaciamiento gástrico


1. Al examinar a un paciente, usted encuentra dolor localizado en fosa iliaca derecha y diagnostica
apendicitis. En este paciente, usted puede inferir:
El peritoneo parietal regional está afectado
2. La motilidad intestinal es estimulada principalmente por el:
Plexo de Auerbach
3. Durante el vómito, ¿el contenido gástrico tiene que pasar necesariamente por cuál estructura para
llegar al esófago? Marque la mejor respuesta:
Cardias
4. Respecto a la anatomía del estómago, marque lo correcto:
El fondo gástrico forma la curvatura mayor
5. Marque la respuesta incorrecta:
En todo el tubo digestivo, se observa dos capas de muscular propia: circular interna y
longitudinal externa
6. Paciente se queja de dolor en hipocondrio derecho, pero superficialmente. El dermatoma relacionado
es (marque la mejor respuesta):
T9
7. Dentro de las funciones del abdomen, se encuentra la defecación y micción, en las cuales la presión
intra abdominal debe:
Aumentar
8. Un alumno de medicina decide hacerse un piercing en el ombligo. Al realizarle el procedimiento,
sangra ligeramente. Esta sangre proviene de la arteria (marque la mejor respuesta)
Epigástrica inferior
9. Señale la respuesta correcta:
El apéndice cecal sólo tiene serosa
10. Paciente mujer es traída a emergencia por sufrir una herida contuso penetrante por cuchillo realizada
por su esposo en un ataque de celos. Se observa herida en flanco izquierdo. Esta solución de
continuidad ha comprometido varios músculos de la pared abdominal, excepto:
Recto abdominal
11. Paciente con herida por proyectil por arma de fuego, con herida de ingreso en región paraumbilical.
Entre las estructuras que usted está seguro que debe haberse lesionado es:
Omento mayor
12. En la evaluación de una tomografía abdominal, el interno observa un aneurisma en una arteria que
se dirige al riñón derecho. Con seguridad se puede afirmar que está a nivel de la vértebra:
L1
13. Paciente joven es traído a emergencia con abdomen agudo quirúrgico debido a herida contuso
penetrante por verduguillo (alambre grueso con punta aguzada) recibida en una pelea después de un
partido de fútbol. Se observa herida en Hipocondrio Izquierdo. El órgano que debe estar sangrando y
produciendo hemoperitoneo es (marque la mejor respuesta):
Bazo
14. Marque el órgano que se encuentra más distal en el tubo digestivo.
Ciego
15. La peristalsis o peristaltismo hace referencia a:
Motilidad para movilizar el alimento de proximal a distal.
16. Marque la respuesta incorrecta:
En todo el tubo digestivo, se observa dos capas de muscular propia: circular interna y
longitudinal externa
17. Paciente con vólvulo del colon sigmoides. La necrosis de ese segmento del colon se produce por una
alteración en la irrigación de la arteria:
Mesentérica inferior
18. Marque el órgano que se considera retroperitoneal:
Parte de la vía biliar
19. Un alumno de medicina decide hacerse un piercing en el ombligo. Al realizarle el procedimiento,
sangra ligeramente. Esta sangre proviene de la arteria (marque la mejor respuesta):
Epigástrica inferior
20. Paciente tiene una úlcera sangrante en el segundo tercio del Yeyuno. La arteria de la cual proviene la
sangre arterial para dicha zona es la arteria:
Mesentérica superior
21. Es inervado por aferentes somáticas:
Peritoneo parietal
22. Paciente de 24 años con dolor abdominal tipo cólico intenso en mesogastrio. Según sus
conocimientos de macroestructura, el origen del dolor puede ser _____:
Íleon
23. Paciente con hipoglucemia secundaria a un insulinoma (tumor neuroendócrino productor de insulina).
El órgano donde mayor probabilidad ha crecido este tumor es:
retroperitoneal
24. Al evaluar una tomografía abdominal, el médico asistente le pide al interno de la UPC que encuentre
la imagen con el corte a nivel de L1. El interno sabiamente busca el para ubicar la vértebra L1:
Cuello del páncreas
25. En la inspiración, la pared abdominal debe relajarse para disminuir presión intra torácica
26. Ligamento hepatogástrico une el ….. con el …… y forma la entrada al …… :
Hígado Estómago Orificio omental
27. Al retirar completamente el mesenterio de un órgano, el mismo se vería afectado principalmente en
su:
Irrigación
28. La estructura que fija órganos principalmente a la pared posterior abdominal se denomina:
Mesenterio
29. Cuál de las siguientes estructuras no tiene vasos sanguíneos:
Epitelio intestinal
30. Al iniciar la digestión, aumenta el consumo de oxígeno por la mucosa. Esto conlleva a una hipoxia
local, lo cual hace que se libere …., el cual produce vasodilatación:
adenosina
31. Sustancia que inhibe la secreción y la motilidad del estómago prolongando el tiempo de digestión:
péptido insulinotrópico dependiente de la glucosa (GIP)
32. Marque lo correcto:
La hernia fisiológica se produce en la sexta semana y es la salida temporal de asas
intestinales a través del colon umbilical
33. Marque la respuesta correcta en relación a la gastrina:
Las células G son las productoras y se encuentran principalmente en el antro gástrico.
34. El consumir caramelos indirectamente activa la vía:
POMC/CART
35. ¿En qué capa se encuentra la alteración principal en el Hirschsprung o megacolon agangliónico?:
Muscular propia
36. Con respecto a las ondas lentas, marque la afirmación correcta:
Son contracciones rítmicas espontáneas
37. El uso de Ranitidina bloquea el receptor H2 de la histamina en las células parietales. La histamina
llega a estas células por:
Difusión
38. La triada sintomática: vómitos explosivos post-prandiales, movimientos peristálticos epigástricos
visibles de izquierda a derecha y nódulos palpable epigástrico subcostal derecho, pertenecen a:
Estenosis congénita hipertrófica del píloro
39. Durante una cirugía oncológica, ¿la extirpación de cuál de los siguientes órganos se vería
comprometida por la presencia de adventicia?:
Recto
40. En cuanto a los reflejos gastrointestinales, un reflejo que estimula el tránsito intestinal es el reflejo:
Gastrocólico
41. El ligamento falciforme divide al hígado en dos lóbulos derecho e izquierdo. Embriológicamente
deriva del:
Mesenterio ventral
42. La presencia de atresias y estenosis duodenales se deben básicamente a una:
Falta de recanalización
43. Estudiante de medicina de 20 años, se ha amanecido estudiando para su examen de Sistema
Digestivo. No ha probado alimento desde la cena, por lo que se puede afirmar que la motilidad de
esta persona está siendo regulada por:
Motilina
44. Paciente con disminución del apetito marcada asociada a cáncer terminal, para promover la ingesta
de alimentos se podría usar análogos de:
Endorfinas
45. Las ondas lentas se producen por la apertura cíclica de canales de:
Calcio
46. La forma más común de atresia esofágica contiene:
Estenosis proximal del esófago más fístula traqueoesofágica distal
47. Al deglutir un bolo alimenticio, es lógico suponer que al pasar por el esofago haya un mayor consumo
de oxigeno en la pared del tercio:
Proximal
48. Paciente que come entera una pizza familiar de chorizo y queso. Es posible esperar que debido a la
cantidad de alimento ingerida, las ondas lentas hayan:
sufrido ninguna alteración en su frecuencia.
49. La hernia fisiológica se produce dentro de:
cordón umbilical
50. El crecimiento de un adenocarcinoma de páncreas compromete la pared gástrica por contigüidad
¿Que parte del estómago se esperaría esté comprometido?:
Pared posterior del antro
51. Estimula la producción de saliva:
Vasodilatación periglandular
52. Durante la secreción de saliva, es de esperarse que las concentraciones de ….. y …… disminuyan al
disminuir el flujo:
Sodio, Bicarbonato
53. Con respecto a la secreción gástrica de HCI:
a mayor secreción de HCI en el lumen gástrico, mayor pH en la sangre venosa gástrica.
54. Respecto a las enfermedades del esófago, marque lo correcto: el diagnóstico diferencial de la
acalasia es la enfermedad de Chagas esofágica
55. Con respecto a las lesiones y enfermedades de la boca, marque lo correcto: la eritroplasia debe ser
biopsiada
56. Respecto a las glándulas salivales, marque lo incorrecto:
la glándula sublingual tiene forma de garfio
57. El omeprazol actúa sobre la membrana ….. de la célula ….. :
apical/parietal
58. Durante el sueño, la concentración de bicarbonato en la saliva:
Disminuye
59. Durante el ataque con gas sarín (bloqueador de la acetilcolinesterasa) en el metro de Tokio, en 1995,
el personal de salud noto que los pacientes afectados presentaban:
Hipersalivación
60. La célula mucosa del cuello gástrico produce:
Moco

Parcia de 2020-01
61. Al evaluar la orofaringe de un paciente, el médico le solicita que abra la boca, saque la lengua y diga
a. Al hacer esta maniobra, nota que el paladar se desvía hacia la derecha, lo cual le hace sospechar
que el paciente sufre de una lesión del nervio craneal:
X contralateral
62. Un bolo alimenticio grande y poco masticado se atasca en el esofago, esto ocasiona una sensación
de dolor que es transmitida por los nervios:
esplácnicos
63. Para realizar el movimiento mecánico de abrir la boca, primero se necesita:
fijar el hueso hioides
64. ¿Cual de las siguientes alternativas se define como la protrusión directa del contenido abdominal a la
cavidad amniótica por un defecto de la pared corporal?:
Gastrosquisis
65. Un paciente requiere que se le coloque una sonda de alimentación directamente al estómago
(gastrostomía), el cirujano deberá hacer una incisión en la piel del abdomen ¿cuál de las siguientes
raíces nerviosas debe ser anestesiada para este procedimiento?
T8
66. En un paciente de 43 años con tumor carcinoide de páncreas productor de gastrina (Síndrome de
Zollinger-Ellison) se puede esperar encontrar una potenciación del reflejo:
gastrocólico
67. El mecanismo de la defecación incluye la participación de diversas estructuras ¿cuál de las
siguientes alternativas es correcta?:
Puede ser mediado por un reflejo intrínseco
68. Cuando el contenido del estómago ingresa al duodeno, uno de los reflejos que inhiben el vaciamiento
gástrico es a través del:
Sistema nervioso mientérico
69. Durante la masticación, gran parte del proceso masticatorio se debe a:
El reflejo masticatorio
70. Las glándulas salivales tienen conductos para la excreción de la saliva, las glándulas …… drenan en
las carúnculas sublinguales.
Sublinguales
71. Los diferentes segmentos del tubo digestivo son susceptibles de reflejos y movimientos según su
contenido. Si colocamos mediante una sonda un bolo alimenticio directamente en el tercio medio del
esofago:
se producira ondas secundarias
72. En una persona si enfermedad se espera que el tránsito intestinal se vea disminuido cuando se
presenta el reflejo:
Doloroso
73. El divertículo de Meckel es una anomalía congénita que ocurre por la persistencia del conducto
vitelino y da origen a una estructura sacular, el cual se encuentra en el:
borde antimesentérico
74. Si al intubar a un paciente, por error se ingresa el tubo endotraqueal en el esofago y se insufla el
manguito endotraqueal (globo TET), la dilatación de este manguito generará:
múltiples ondas secundarias
75. El orificio omental, o hiato de Winslow, se encuentra limitado por el ligamento:
hepatoduodenal
76. Paciente de 24 años acude a consulta externa por presentar una fístula oronasal (comunicación entre
la cavidad oral y la cavidad nasal). Esta fístula es una consecuencia tardía de la lesión de un vaso
sanguíneo por el antecedente de haber sido operado de paladar hendido en los primeros años de
vida, aparentemente en una campaña gratuita de corrección de paladar fisurado. ¿Cual de las
arterias palatinas podría haberse lesionado durante esa cirugía?:
Mayor
77. Dentro de las anomalías congénitas se puede presentar un tejido pancreático accesorio ¿cuál es la
ubicación más común de este tejido?:
Estómago
78. Una recién nacida es evaluada por el neonatólogo y evidencia que el canal anal está completamente
cerrado. Este problema se debe probablemente a una anomalía en el desarrollo de:
la membrana cloacal
79. En la digestión de los alimentos, la hormona se libera frente a la presencia de péptidos y
monoglicéridos, y tiene un efecto marcado en la disminución del vaciamiento gástrico
colecistoquinina
80. Dentro de las anomalías congénitas se puede presentar un tejido pancreático accesorio ¿Cuál es la
ubicación más común de este tejido?
Estómago
81. Los catadores de vino tienen una habilidad increíble al momento de separar los sabores. Este
aumento de la sensibilidad gustativa debido a una mayor cantidad de papilas linguales y de
corpúsculos gustativos se conoce como:
hipergeusia
82. En muchos países se usa el suplemento de flúor en el agua potable o los dentífricos, con el fin de
hacer el esmalte más resistente a la desmineralización inducida directamente por:
el ácido
83. Durante el desarrollo de la región cloacal, una cuña de mesodermo ubicado entre el alantoides y el
intestino posterior vendrá a formar el:
tabique urorrectal
84. En una persona sana, el momento adecuado para encontrar los mayores niveles de grelina en
sangre sería:
antes de comer
85. El duodeno está constituido por el segmento terminal del intestino anterior y el segmento
proximal del intestino medio ¿Cuál de las siguientes alternativas describe mejor este lugar de unión
entre los dos intestinos?
Distal al origen de la yema hepática
86. En una persona sana, el uso de atropina producirá a nivel del estómago:
Aumentará el pH del estómago
87. En una persona sana, el consumo de leche produce indirectamente
Inhibición del vaciamiento gástrico
88. La motilidad del colon es importante y lenta comparada con la del intestino delgado. Los movimientos
en masa ocasionan la:
distensión rectal
89. Con respecto a la motilidad gástrica, los potenciales de acción disminuyen en frecuencia por efecto
de:
el péptido insulinotrópico dependiente de glucosa
90. La sensación del gusto depende de la presencia de papilas gustativas en la lengua, de las cuales,
algunas de ellas tienen un surco terminal por donde drenan unas glándulas salivales linguales
(llamadas glándulas de von Ebner). Esta descripción se refiere a las papilas:
circunvaladas
91. Dentro de la estructura de los dientes, la parte del diente cubierta por esmalte y que se puede ver
mediante la inspección visual de la boca se denomina
corona clínica
92. En el conducto anal se encuentra la unión entre las regiones del endodermo y el ectodermo, esta
unión se evidencia al observar:
la línea pectínea
93. La sensación del gusto depende de la presencia de papilas gustativas en la lengua, las cuales tienen
corpúsculos gustativos conteniendo células neuroepiteliales sensoriales. Estas células
neuroepiteliales pueden ser dañadas fácilmente, por suerte, su tiempo de recambio es de alrededor
de:
10 días
94. En un varón de 47 años con sección medular a nivel de T6 debido a un accidente automovilístico, sus
terapeutas han desarrollado un mecanismo para distender el recto e iniciar el reflejo
rectoesfinteriano, lo cual producirá la contracción de:
la pared del recto
95. Durante un experimento, se insufla rápidamente dos litros de agua en un globo colocado dentro del
estómago de un voluntario. ¿cuál de las siguientes situaciones del músculo liso será consecuencia
directa de este cambio de volumen en el estómago?
Despolarización
96. Paciente de 56 años con accidente cerebrovascular reciente. En la resonancia se observa daño de
los núcleos laterales del hipotálamo. Por este motivo es muy probable que el paciente sufra de:
inanición
97. Paciente con insuficiencia mitral moderada a severa, con aumento de volumen de la aurícula
izquierda, esta condición tendrá como consecuencia a nivel del sistema digestivo:
la disfagia a sólidos
98. El inicio de la fase faríngea de la deglución se debe a estímulos sensitivos que viajan por el nervio
craneal: V
99. El mesocolon transverso se origina en:
la pared posterior del abdomen
100. El esofago en su microestructura tiene basicamente adventicia, a excepción de la región distal,
donde tiene serosa, específicamente a partir del nivel de:
T10
101. En un paciente con arcadas, se debe considerar que durante la ocurrencia de dichas arcadas,
debemos encontrar contenido gástrico en:
Tórax
102. La reabsorción de Sodio y Cloro en las glándulas salivales se da principalmente en el: Conducto
estriado
103. Para que se puedan digerir las grasas, es preferible que primero sean emulsificadas. La hormona
que estimula la liberación de las sustancias emulsificadoras es:
CCK
104. Paciente de 13 días de vida con vómitos explosivos a las 2 horas después de lactar. Al examen
físico se palpa la oliva pilórica ¿cual es el nervio cuyos filetes dan inervación eferente a la estructura
afectada?:
Vago
105. La localización de la vesícula biliar con respecto al lóbulo cuadrado es:
Lateral
106. La triada portal está constituida por el conducto biliar y la arteria hepática y una pequeña rama de
la vena:
Porta
107. Al ingerir una cantidad de glucosa por vía oral, esta es interiorizada en las células del organismo
más rápido que si esa misma cantidad hubiese sido administrada por vía endovenosa. Este
fenómeno sucede gracias a una sustancia secretada por las células:
K
108. Paciente de 62 años con vólvulo de intestino delgado e isquemia intestinal. ¿Qué estructura se
utiliza como punto de referencia para determinar la posición de la unión duodenoyeyunal? :
Ligamento suspensorio del duodeno (de Treitz)
109. En un paciente con Zollinger Ellison, usted esperaría encontrar:
Esteatorrea
110. La motilidad intestinal es estimulada por:
Colecistoquinina y gastrina
111.La onda peristática secundaria del esofago en la deglución, es producida por:
Plexo mientérico esofágico
112. Paciente con parálisis bilateral del nervio hipogloso, el unico musculo de la lengua que
conservará su movimiento es el:
palatogloso
113. ¿Cual de las siguientes sustancias tiene mayor concentración de la saliva comparado con su
concentración plasmática?
Potasio
114. ¿Cual de las siguientes estimula la secreción enzimática exocrina del páncreas?:
Colecistoquinina
115. Al comer un pollo a la brasa, con papas fritas y ensalada, la sustancia que estimula la liberación
de HCI en el estómago es:
Bombesina
116. Marque lo correcto con respecto a Esófago de Barret:
Se relaciona con reflujo gastroesofágico
117. Un niño de 2 años es llevado a la consulta por diarrea persistente y edema de las extremidades,
además falta de crecimiento y desarrollo en relación a su edad. Los análisis de sangre revelan que
tiene concentración plasmática baja de proteínas (hipoproteinemia). Durante la endoscopía duodenal,
se coloca colecistocinina (CCK) endovenosa y se recoge muestras del líquido duodenal; el resultado
del líquido confirma incapacidad para hidrolizar proteínas a un pH neutro, esta situación mejora al
añadir una pequeña cantidad de tripsina. El paciente probablemente esté sufriendo la falta congénita
de
-Enterocinasa
118. Experimentalmente se incrementa la velocidad de la secreción salival con una sustancia, el
análisis de la composición de esta saliva obtenida se espera encontrar…………..
-Disminución de concentración de potasio
119. Paciente varón de 46 años soltero, consulta por odinofagia y bajo de peso, tiene antecedente de
tuberculosis desde hace 3 meses y es fumador crónico (10 cigarrillos por día); al evaluar la cavidad
oral se identifica lesión blanquecina en el dorso de la lengua y paladar blando, las lesiones se
desprenden con el baja lengua dejando una base eritematosa. Esta lesión corresponde
probablemente a ……………………….…..
-Candidiasis oral
120. Paciente mujer de 35 años acude a consulta por sensación de sequedad y lesiones en cavidad
oral. Al examen se observa atrofia de la mucosa, fisuras y úlceras; nota además sequedad e irritación
de la córnea y aumento del tamaño de las glándulas parotídeas. Su diagnóstico más probable es
artritis reumatoide; el hallazgo más probable en una biopsia de glándula parótida es……..….
-Gran infiltración de linfocitos y células plasmáticas
121. Un paciente con anemia acude con su médico quejándose de episodios frecuentes de
gastroenteritis. Un análisis de sangre revela anticuerpos circulantes dirigidos contra células parietales
gástricas. Su anemia es atribuible a la hiposecreción de
-Factor intrínseco
122. Dos estudiantes deciden tomar un receso para comer una hamburguesa a la hora del almuerzo.
Antes de llegar a la cafetería, impulsos nerviosos provenientes del complejo vagal dorsal iniciarán la
secreción de ácido gástrico por la liberación de …………………….. desde el sistema nervioso
entérico.
-GRP
123. Un niño de cuatro años de edad es llevado a la consulta por cuadros diarreicos frecuentes
caracterizados por heces pálidas, voluminosas y fétidas, presenta bajo peso y talla. Se mide la
concentración de cloruro en el sudor y se encuentra que sus valores son muy elevados. La alteración
más importante a nivel de células ductales del páncreas tiene relación directa con la conductancia
de…………
-Cloro
124. Una mujer de 50 años de edad que sufrió durante varios años resequedad de los ojos debida a
producción inadecuada de lágrimas es enviada con un gastroenterólogo para evaluación de pirosis
crónica. El examen endoscópico revela erosiones y tejido cicatrizal en la parte distal del esófago justo
por arriba del esfínter esofágico inferior. Las lesiones pueden atribuirse a la disminución de uno de
los siguientes componentes salivales:
-Bicarbonato
125. Se evalúa los valores séricos de las siguientes sustancias a un paciente con enfermedad
hepática terminal; en este paciente se espera encontrar la combinación con la letra …………
-disminuida, aumentada, disminuida
126. Una mujer de 35 años de edad HIV positiva, se presenta al médico con dolor abdominal en
cuadrante superior derecho e ictericia. La paciente refiere haber tenido múltiples episodios de
ictericia durante los últimos 10 años. Los exámenes para determinar hepatitis viral, dieron positivos
para Hepatitis B, siendo catalogado el caso como hepatitis crónica con alteración funcional. En un
examen de sangre ¿cuál de los siguientes parámetros está disminuido?
-Albúmina
127. En el reflejo peristáltico del intestino delgado, uno de los siguientes eventos sucede en la porción
oral del bolo alimenticio…………...
-Acción de acetilcolina en el músculo circular
128. Experimentalmente se coloca una dosis alta de secretina en la luz intestinal duodenal; como
consecuencia de esto, en el jugo pancreático de la misma luz intestinal se observa la disminución de
la concentración de …..………..
-Cl
129. Un varón de 58 años de edad con enfermedad de Crohn severo fue sometido a una resección
ileal. Después de la cirugía este paciente padecerá de esteatorrea, esto se explica porque
…..………..
- La micelas no pueden formarse
130. En un experimento se inserta un balón en el estómago de un voluntario, se infla poco a poco
mientras que se vigilan las presiones intraluminales. Aunque el volumen del balón aumenta
considerablemente, las presiones permanecen constantes. Esta relación volumen-presión se explica
por la liberación local de …………..
-Óxido nítrico y péptido inhibidor vasoactivo
131. La toxina de Vibrio cholerae causa diarrea debido a…….
-El Incremento de la secreción de cloro por las células de la cripta intestinal
132. ¿Cuál de las siguientes alternativas es una característica de la secreción exocrina del páncreas?
-Tiene una baja concentración de Cl- respecto al plasma
133. Una madre lleva a su hijo de dos años de edad a la sala de urgencias, estresada porque el niño
deglutió una moneda de 10 céntimos mientras la familia cenaba en un restaurante. El médico
observa mediante fluoroscopía que la moneda se halla en el estómago y asegura a la madre que la
moneda se eliminará con las heces. El médico recomienda utilizar la respuesta fisiológica que
permitirá la evacuación de la moneda del estómago al intestino ………….…..
-Son los movimientos de mezcla y trituración
-. Es provocada por el ayuno
134. Las estructuras en el hígado que permite que los productos metabólicos unidos a proteínas
tengan acceso a las membranas basolaterales de los hepatocitos, son…..
- Las fenestras sinusoidales
135. La composición de la bilis es modificada conforme fluye por los conductillos biliares. Durante este
tránsito se espera que aumente la concentración de……
-Ig A
136. Se mide experimentalmente el contenido gástrico de dos personas. La persona “A” tiene alto
contenido de grasa y la persona “B” tiene un contenido hipertónico ¿Cuál de las siguientes es
correcto respecto al vaciamiento gástrico?
- Hay ralentización del vaciado gástrico en ambos casos
137. El examen endoscópico de un paciente con hipertensión portal grave revela venas tortuosas que
sobresalen hacia la luz del esófago. El paciente recibe tratamiento quirúrgico mediante la colocación
de una derivación que conecta la vena porta a la vena cava. Después de la operación el riesgo de
encefalopatía y el riesgo de sangrado de várices ……………..
-Aumentará/disminuirá
138. Un paciente varón de 18 años de edad acude al médico para sus exámenes de rutina. Sus
resultados de laboratorio muestran un valor de bilirrubina sérica de 4 mg/dl y una bilirrubina directa
de 0,3 mg/dl. Las pruebas de función hepática son normales. La alteración que explica mejor este
caso es por la deficiencia de ………………..
-Glucuronil transferasa
139. Un hombre de 57 años de edad es llevado a urgencias con hematemesis masiva rojo brillante, a
su llegada se halla inconciente con PA: 80/40 mm Hg y FC: 124 lat/min. Luce ictérico con presencia
de “arañas vasculares en el tórax anterior y extremidades”, abdomen distendido con signo de oleada
positiva. Se encuentra esplenomegalia y pérdida de la masa muscular en extremidades. La
anastomosis vascular responsable del sangrado en este paciente es
-Vena gástrica izquierda y vena ácigos
140. Un estudiante de medicina está comiendo un plato de comida a base de champiñones, espárrago
y salsa de soya. El sabor umami contenido en todos estos alimentos actúa a nivel de los botones
gustativos estimulando ………………..
-Un receptor acoplado a proteína G
141. Un hombre de 22 años de edad se presenta al médico con una historia de 1 año de evolución
caracterizado por dolor recurrente en fosa iliaca derecha y diarrea. Manifiesta además pérdida de
peso de 8 kg durante este periodo. La colonoscopía revela múltiples lesiones en el ileon terminal y
colon. La biopsia de estas lesiones revela engrosamiento, inflamación y ulceración de la mucosa. El
diagnóstico más probable en este caso es…….
-Enfermedad de Crohn
142. Varón de 61 años que consulta por dolor retro esternal intenso desde hace 6 horas y después de
vómitos intensos y repetidos; al examen se observa disnea, cianosis, hipotensión y signos clínicos de
shock. La radiografía simple de tórax muestra neumomediastino. El líquido en el espacio pleural
aspirado tiene alta concentración de amilasa. ¿Cuál de las siguientes alternativas puede explicar este
cuadro clínico?
-Rotura espontánea de esófago
143. La secreción del ácido en la célula parietal gástrica se lleva a cabo por una ATPasa especifica
que intercambia hidrogeniones (H+) del citosol por….. -K +
144. En condiciones normales el ingreso de 600 ml de líquido es el estómago provoca un aumento de
presión intragástrica de unos 12 cm de H2O. Después de una vagotomía (corte del nervio vago) es
de esperar que el ingreso del mismo volumen de líquido provoque lo siguiente:
…………………………………
-Un aumento mayor de la presión
145. Una paciente de 30 años de edad es sometida a una cirugía en oído medio derecho por un
problema de otoesclerosis. Luego de la cirugía refiere alteración en la percepción de sabores. Al
evaluar el caso usted esperaría encontrar……….
-Alteración en la sensación del gusto en los dos tercios anteriores de la lengua
-Sensación del dolor, tacto y temperatura conservada en toda la lengua
146. ¿Cuál de las siguientes alternativas es correcta?
-Las sales biliares desconjugadas son absorbidas preferentemente en el colon
147. En un paciente de 45 años de edad con colestasis biliar, se encuentra una elevación de los
niveles sanguíneos de fosfatasa alcalina hasta 3 veces la cifra normal. ¿Cuál de las siguientes
alternativas estará también elevada como evidencia del daño de la vía biliar?
-Gamma glutamil transpeptidasa
148. Revisando la angiografía de un hombre de 70 años en estudio por aneurisma de aorta abdominal
el radiólogo informa de la presencia de una oclusión completa de la arteria mesentérica inferior. El
paciente se encuentra completamente asintomático. ¿Cuál de las siguientes arterias se anastomosa
a la sistema arterial de la mesentérica inferior?
-Cólica media
149. Lactante de 3 meses de vida es atendido por presentar diarrea, se administra una solución de
glucosa y electrólitos por vía oral. La proteína de membrana apical que explica la capacidad de esta
solución para proporcionar aporte de glucosa e hidratación es ………..
-SGLT-1
150. Paciente ha sufrido herida de bala en el abdomen, se le ha tenido que extirpar el segmento
medio y distal del ileon. En este caso la síntesis hepática de sales biliares estará …..…..
-Incrementada por estímulo de la enzima colesterol 7 alfa hidroxilasa
151. Un varón de 75 años ingresa al consultorio por presentar ictericia marcada de piel y las escleras.
El estudio del paciente mostró que presentaba un tumor que obstruía la totalidad del conducto
hepático común. ¿Cuál de las siguientes estructuras se encontrará dilatada en este paciente?
-Conductos de Hering
152. En un paciente con insuficiencia renal crónica, el déficit en la absorción de calcio a nivel del
enterocito se debe a lo siguiente:
-No se convierte la 25 hidroxicolecalciferol a 1,25 dihidroxicolecalciferol
153. Varón de 30 años es traído a emergencia por agresión abdominal con arma de fuego (pistola) y
es sometido a laparotomía exploratoria, observándose isquemia del colon ascendente y parte del
colon trasverso ¿la lesión de cuál de las siguientes arterias explicaría esta isquemia?
- Mesentérica superior
154. Respecto a las sustancias gastrointestinales que regulan la secreción pancreática; marque la
afirmación correcta:
- La acetilcolina es capaz de estimular la secreción enzimática y de bicarbonato del páncreas
155. Ante una lesión del X par craneal, ¿cuál de los siguientes músculos mantiene conservada su
función?:
- Tensor del velo del paladar
156. Experimentalmente se utiliza atropina (anticolinérgico) para inhibir la secreción de gastrina, sin
embargo la secreción de esta hormona se sigue dando ante estímulos vagales. Esta situación se
explica porque la atropina:
- No bloquea la acción del péptido GRP
157. Un varón de 50 años es sometido a extirpación del duodeno y parte proximal del yeyuno. Esta
situación ocasionaría la pérdida de las células, productoras de………………… que estimula la
secreción de bicarbonato por el páncreas.
-“S” / secretina
158. Se evalúa la expresión de la proteína Agrp en una persona con alteración del apetito; lo correcto
respecto a esta proteína es…..
- La mutación del gen que la codifica produce adelgazamiento
159. Juana cae de la bicicleta y se fractura la región anterior del hueso maxilar superior con
compromiso de la fosa incisiva. Al examen físico de la región esperaría encontrar alteración en la
sensibilidad de la encía …………………
- Palatina anterior
160. Recién nacido es atendido por el neonatólogo y luego entregado a su madre para dar de lactar;
la madre al dar de lactar observa coloración azulada de labios, acompañado de tos persistente,
dificultad respiratoria y distención abdominal. Se le intenta colocar una sonda nasogástrica pero esta
retorna a la cavidad oral en todos los intentos. ¿Cuál de las siguientes anomalías del desarrollo es el
más probable en este caso?
Atresia esofágica proximal con fístula traqueoesofágica distal
161. ¿Cuál de los siguientes mecanismos ocurre durante la defecación?
- En la posición de “cuclillas” el músculo puborrectal se halla relajado
162. Un paciente luego de un accidente sufre lesión del piso de la boca, se constata daño del nervio
“cuerda del tímpano”, en este caso se esperaría encontrar disminución de la lengua
-Sensación del gusto en los dos tercios anteriores
163. las siguientes afirmaciones es la correcta sobre la gastrina?
- Actúa en la célula diana mediante su receptor CCk tipo B
164. Al recibir un paciente con signos de hipovolemia y antecedente de trauma en abdomen por
accidente de tránsito, usted identifica radiológicamente: lesión de primera vértebra lumbar y signos
de lesión en páncreas; durante la cirugía se observó pobre irrigación de asas intestinales. El vaso
afectado es la arteria ……..
- mesentérica inferior
165. Un paciente sufre de daño a nivel del cuello con lesión muscular en la región de la faringe. En el
examen físico se determina dificultad para la elevación de la faringe y para el cierre del itsmo de las
fauces. En este caso, probablemente esté afectado el músculo:
- Palatofaringeo
166. Varón de 50 años a quien le realizan la curación de la segunda molar de la arcada superior
derecha. En un momento determinado, el paciente acusa de intenso dolor de la pieza dentaria en
tratamiento. La vía aferente del dolor viaja a través del nervio …………
- trigémino V2
167. La distención gástrica por los alimentos produce incremento de secreción de HCl mediante la
producción de que estimula a las células vía proteína
- gastrina / parietal / Gq
168. Un niño de tres años llega a emergencia con disfagia (dificultad para tragar), dolor retro esternal,
salivación y llanto. Se sospecha de ingesta de cuerpo extraño (moneda) en el esófago; al ser
evaluado se constata en una radiografía presencia de cuerpo extraño a nivel de C6 (6° vértebra
cervical). El cuerpo extraño estará suspendido a nivel del estrechamiento producido por………..
- el músculo cricofaríngeo
169. La triada portal (arteria hepática, vena portal y conducto biliar común) está contenida en el
ligamento …….……… y derivan embriológicamente del ……
- hepato duodenal / mesenterio ventral
170. Un paciente refiere no percibir algunos sabores, al examen físico se constata alteración en la
percepción de sabores y del dolor en el tercio posterior de la lengua ¿Cuál de los siguientes nervios
estará alterada en su función?
- Glosofaríngeo (IX par)
171. En el caso de un paciente con gastrinoma (tumor productor de gastrina), la presencia de úlceras
duodenales y erosión de la mucosa gástrica, se debe principalmente a…….
- El exceso de HCl por estímulo de receptores CCK-B en la célula parietal
172. El reflejo enterogástrico se caracterizan por:
- Originarse debido a la distensión duodenal y presencia del quimo ácido
173. Mauricio tiene dificultad para deprimir el paladar y elevar la parte posterior de la lengua. En este
caso estará afectado un músculo, específicamente el músculo…………….
- Extrínseco – palatogloso
174. En condiciones normales, el ingreso de 600 ml de líquido es el estómago provoca un aumento de
presión intragástrica de unos 12 cm de H2O. Después de una vagotomía (corte del nervio vago) es
de esperar que el ingreso del mismo volumen de líquido ocasione ………………………………… de la
presión intragástrica.
- un aumento mayor
175. La explicación fisiológica de presentar somnolencia de 30 minutos a 1 hora después de ingerir
alimentos, se explica por:
- Aumento de la alcalinidad sanguínea
176. Se presenta un paciente, el cual presenta un antecedente de tuberculosis intestinal, por lo cual,
se le resecó 80 cm de íleon distal. Desde el punto de vista fisiológico, el paciente puede presentar
una de las siguientes alteraciones: a. Disminución de la secreción de Vitamina B12
- Disminución de la absorción de ácido glicocólico
177. Un paciente es sometido experimentalmente a un fármaco que modifica el flujo
salival,obteniéndose un volumen de saliva de 288 ml en 6 horas. En este caso las concentraciones
de electrolitos y bicarbonato en la saliva obtenida varían de la siguiente manera:
- ↓ Na+, ↓ Cl-, ↑ K+, ↓ HCO3--
178. Uno de los siguientes elementos debería hallarse con más probabilidad en el esófago de un
paciente que sufre de reflujo gastro esofágico…
- Pepsina
179. Un paciente de 40 años cursa con anemia de 8g/dl, aqueja además de astenia y sensación de
hormigueo bilateral en los miembros inferiores, al examen se halla alteración de la sensibilidad a la
vibración y camina con ampliación de la base de sustentación. Uno de los siguientes procedimientos
sería de ayuda para el diagnóstico de este paciente:
- Biopsia de la mucosa gástrica
180. Paciente de 60 años ingresa por caída hace 1 hora y pequeño hematoma en cuero cabelludo, al
examen físico ampliado se observa ictericia de piel y mucosas generalizada, abdomen blando, se
palpa estructura quística no dolorosa en hipocondrio derecho que corresponde a vesícula biliar (signo
de Courvoisier), en los exámenes de laboratorio se halla niveles bajos en la formación de
estercobilinógeno y urobilinógeno en heces, incremento de la bilirrubina conjugada en la orina,
elevación de fosfatasa alcalina y gamma glutamil transpeptidasa séricas. El presente cuadro puede
ser explicado por
- Carcinoma de la cabeza de páncreas
181. Un recién nacido presenta vómitos biliosos poco tiempo después de cada alimento. Al preguntar
a la madre sobre antecedentes, ella recuerda que tuvo polihidramnios durante la gestación, pero un
análisis de cariotipo fue normal. Una de las siguientes es la causa más probable de estos hallazgos
en el recién nacido:
- Malrotación de la yema pancreática ventral
182. En un estudio de la secreción de hormonas gastrointestinales, sus concentraciones en la vena
porta se midieron durante perfusión luminal del intestino delgado con soluciones de diversas
magnitudes de pH. ¿Qué hormona aumentará en el plasma de la vena porta durante perfusión a
través del intestino con una solución de pH 3?
- secretina
183. Paciente de 30 años que ingresa a causa de un traumatismo abdominal cerrado. En la
exploración se aprecia discreta palidez de piel y mucosas, auscultación pulmonar normal, taquicardia
de 120 /min. Discreta distensión abdominal y matidez en flancos; el hematocrito, que era
prácticamente normal al ingreso, disminuye a 30% a las tres horas. En la Rx de tórax se objetiva
fractura de las costillas 10-11 izquierdas. La causa más probable de la anemización en este paciente
es: a. traumatismo renal con hemorragia retroperitoneal.
- rotura de bazo con hemoperitoneo.
184. Revisando la angiotomografía de un hombre de 70 años en estudio por aneurisma de aorta
abdominal, el radiólogo le informa de la presencia de una oclusión completa de la arteria mesentérica
inferior. El paciente se encuentra completamente asintomático. La oclusión de la arteria mesentérica
inferior cursa de manera asintomática en muchas ocasiones ya que el territorio que irriga puede
recibir flujo proveniente de la arteria:
- cólica media
185. En las patologías de esófago es importante conocer bien la anatomía esofágica. ¿Cuál de las
siguientes afirmaciones es correcta?
- El esófago torácico pasa por detrás del cayado aórtico
186. A pesar de que pueda haber variaciones anatómicas, lo habitual es que el ciego sea irrigado por
una rama arterial que proviene de unas de las siguientes arterias: a. Iliaca derecha
- Mesentérica superior
187. Ante un paciente con una cirugía abdominal urgente, el informe operatorio señala que se ha
realizado una resección de todo el duodeno y del tercio proximal del yeyuno manteniendo íntegros el
estómago y todo el íleon, así como los dos tercios distales del yeyuno. En el seguimiento nutricional
del paciente ¿Qué vitamina o mineral presentará con menor probabilidad una disminución de su
absorción?
- Cianocobalamina
188. ¿Cuál de las siguientes sustancias forma parte de la secreción biliar?
- Lecitina
189. ¿De qué músculo forma parte el ligamento inguinal?
-Oblicuo externo del abdomen
190. ¿Cuál de las siguientes enzimas está localizada en el borde en cepillo y juega un rol en la
digestión de proteínas?
- Carboxipeptidasa A.
191. Una de los siguientes sustancias, NO sirve como un buen agente emulsificante:
- Colesterol
192. La sustancia que estimula el crecimiento de la mucosa gástrica es:
- Gastrina
193. ¿Cuál de las siguientes alternativas es una función de la colecistokinina?
- Secreción de enzimas pancreáticas
194. Con respecto a la anatomía del tronco celiaco, señale lo correcto
- La hepática común que es una de sus ramas, participa en la irrigación del estómago.
195. Con respecto a la anatomía del duodeno, marque la respuesta correcta: a. Tiene una distribución
en forma de “C”, que rodea la cola del páncreas
-La 3ra porción duodenal está contenida en la pinza vascular aortomesentérica
196. En el íleon se absorbe aproximadamente el 95% de la circulación enterohepática.
- Sales biliares
197. La estimula el mecanismo paracrino de la secreción de ácido clorhídrico.
- histamina
198. En la digestión de proteínas,es el principal estímulo para convertir el pepsinógeno en pepsina.
- el pH ácido
199. Con respecto a la somatostatina, marque lo correcto:
- Interviene en la fase intestinal de la secreción gástrica
200. En pecten anal, es una estructura comprendida entre: a. la línea pectínea y los senos anales
- la línea anocutánea y la línea pectínea
201. Se traza un plano a nivel de la mitad de la distancia entre la horquilla esternal y la sínfisis
del pubis, a ese nivel se espera encontrar:
Cuello del páncreas
202. El mesocolon transverso se origina en:
Pared posterior del abdomen
203. El orificio omental, o hiato de Winslow, se encuentra limitado por el ligamento:
hepatoduodenal
204. Se la conoce como el vigilante del abdomen , por su capacidad de desplazarse y
adherirse a cualquier zona inflamada, y envolviendo el órgano para frenar la inflamación:
Omento mayor
205. Marque lo correcto con respecto a lo que sucede al contraerse el diafragma:
Se aplana el diafragma
206. Las glándulas submucosas se encuentran en mayor cantidad a nivel:
proximal a la unión esofagogástrica
207. En cuanto a la distribución del tubo digestivo en la cavidad abdominal, se puede afirmar
que los órganos intraabdominales están suspendidos por:
Mesenterio
208. Al palpar una masa pétrea en hipocondrio derecho, es probable que se trate de un tumor
de:
vesícula biliar
209. Al momento de espirar profundamente, ocurre lo siguiente:
aumento de la presión intraabdominal
210. La estrechez esofágica más proximal es:
cricofaríngeo
211. El peritoneo parietal está separado del peritoneo visceral por la fascia de _______ que
está fijada a la pared posterior del abdomen, y es importante porque por esta vía se pueden
diseminar las infecciones peritoneales:
Todlt
212. Marque la afirmación incorrecta:
los omentos principalmente irrigan órganos intraabdominales
213. Los dobleces de peritoneo que comunican un órgano con la superficie interna de la
pared abdominal posterior antiguamente se denominaban ligamentos , sin embargo,
actualmente se denominan:
Repliegues
214. Señale el órgano del sistema digestivo que no contiene adventicia:
Vesícula biliar
215. Es considerado un órgano sólo inicialmente intraperitoneal:
Páncreas
216. Marque el órgano que se encuentra fuera de la cavidad peritoneal:
recto
217. Para colocarle una sonda de alimentación directamente al estómago (gastrostomía), el
cirujano deberá hacer una incisión en la piel del abdomen inervada por las raíces de:
T8
218. El íleon se encuentra principalmente en:
Cuadrante inferior derecho
219. Tiene inervación somática:
Peritoneo parietal
SISTEMA DIGESTIVO
EXAMEN PARCIAL
201802

Sección: Todas
Profesores: Choque Chávez, Fernando Diego; Damián Bastidas, Narda Lucía; Irribarren Gamarra, Maria Patricia;
Stapleton Valdivia, Mauricio Juan Jose.
Duración: 50 minutos.
Indicaciones:
- Lea atentamente cada pregunta antes de responder.
- Se prohíbe el uso del celular y cualquier dispositivo electrónico.
- Está prohibido intercambiar materiales.
- Coloque su código de alumno en la tarjeta de respuestas. Si su código contiene una letra reemplácela por un valor
numérico siguiendo la siguiente equivalencia: A=9, B=8, C=7, D=6, E=5.
- Traslade sus respuestas a la tarjeta, llenando los círculos de manera completa con lapicero negro o azul. Está
prohibido el llenado con lápiz, lapicero de otro color o con lapicero de tinta borrable.
- Sea cuidadoso en el llenado de la tarjeta de respuestas, pues solo esta tiene validez para la calificación.
- Al terminar su examen avise al docente a cargo, no se levante de su sitio; debe entregar la hoja de respuestas con
la carátula del examen, este cuadernillo de preguntas se lo llevará cada estudiante.

Chorrillos, octubre de 2018

1. En todo el sistema gastrointestinal se encuentra diferentes tipos de músculos ¿En cuales estructuras
encontraríamos músculo estriado?
A. Faringe, esfinter esofágico inferior, colon descendente.
B. Esfínter esofágico superior, laringe, esfínter anal externo.
C. Faringe, esfínter esofágico superior, esfínter anal externo.
D. Esfínter esofágico superior, esfínter esofágico inferior, colon.

2. Durante la fase …..……………….. el paladar blando es traccionado hacia arriba.


A. oral
B. gástrica
C. faríngea
D. esofágica

3. El tubo digestivo a nivel del ………………………….. está revestido internamente por epitelio ………….
A. Esófago / plano simple
B. Estómago / cilíndrico simple
C. Intestino grueso / plano estratificado
D. Intestino delgado / plano de transición

4. Señale la estructura del tubo digestivo que presenta tres capas musculares en su pared.
A. Esófago
B. Estómago
C. Intestino grueso
D. Intestino delgado

5. Paciente de 47 años con sobrepeso, acude a consulta por presentar dolor tipo cólico en el cuadrante superior
derecho del abdomen. El dolor aumenta luego de ingesta de comidas con abundante grasa. Esta situación se explica
por el aumento en la secreción de …………………………. que estimula la contracción de la vesícula biliar.
A. Gastrina
B. Colecistoquinina
C. Péptido inhibidor gástrico
D. Péptido intestinal vasoactivo
6. Si hay un aumenta de la hormona grelina a nivel del hipotálamo ¿Cuál es la consecuencia en el organismo?
A. Hambre
B. Saciedad
C. Aumento de somatostatina
D. Disminución del tránsito intestinal

7. Señale el péptido gastrointestinal que produce relajación del músculo liso gastro-intestinal.
A. Péptido intestinal vasoactivo (PIV)
B. Gastrina vasoactiva (GV)
C. Colecistoquinina (CCK).
D. Acetilcolina (Ach)

8. ¿Cuál de las siguientes alternativas inhibiría la relajación receptiva a nivel del estómago?
A. Histamina
B. Bloqueo del nervio vago (X)
C. Péptido inhibir gástrico (GIP)
D. Péptido intestinal vasoactivo (VIP)

9. Una mujer de 28 años, con diagnóstico de Diabetes Mellitus Tipo 1, acude por presentar desde hace 10 años
estreñimiento y distensión abdominal. Se realiza estudio y se determina que la paciente presenta un retraso del
vaciamiento gástrico debido a gastroparesia diabética. ¿Cuál de los siguientes hechos aumentaría el tiempo del
vaciamiento gástrico?
A. Aumento de gastrina
B. Estimulación parasimpática
C. Ácidos grasos en el duodeno
D. Quimo isotónico en el duodeno

10. Experimentalmente, se aplica un inhibidor selectivo del péptido intestinal vasoactivo (PIV) durante la contracción
peristáltica del intestino delgado. ¿Cuál es el efecto de este inhibidor en la motilidad del intestino delgado?
A. Parálisis del movimiento anterogrado
B. Disminución del tránsito intestinal
C. Aumento del tránsito intestinal
D. Movimiento retrogrado

11. ¿Cuál de los siguientes eventos ocurre durante la defecación?


A. Relajación del esfínter anal externo
B. Relajación del músculo liso del recto
C. Contracción del esfínter anal interno
D. Disminución de la presión intrabdominal

12. Señale el péptido que cumple la función de disminuir la ingesta de alimentos.


A. Neuropéptido Y
B. Ghrelina Gastrica
C. Proteína relacionada a agouti
D. Proopiomelanocortina (POMC)

13. Indique el lugar de secreción del péptido YY


A. Estómago
B. Duodeno
C. Yeyuno
D. Ileon

14. La grelina es secretada en el estómago y estimula a las neuronas del núcleo ………………….. para la estimulación de
la secreción de……………..
A. arqueado / neuropéptido Y.
B. arqueado / melanocortinas.
C. paraventricular / neuropéptido Y.
D. lateral del hipotálamo / hormonas orexigénicas.
15. Considerando el desarrollo embriológico del intestino medio. ¿Qué evento de gran importancia se produce en la
sexta semana?
A. Retracción de asas intestinales primitivas
B. Aparición del primordio hepático y pancreático
C. Inicio de la secreción de insulina por el páncreas
D. Salida temporal de asas intestinales a través de cordón umbilical

16. En el ……………………………. se presentan los movimiento de …………………….


A. esófago / retropropulsión y mezcla
B. estómago / segmentación y ondas lentas
C. intestino grueso / propulsión y ondas en espiga
D. intestino delgado / segmentación y peristaltismo

17. Marita sufre una parálisis del músculo masetero. ¿Qué limitaciones se producirán a nivel del movimiento de la
mandíbula?
A. Lateralización
B. Propulsión
C. Elevación
D. Ninguna

18. Señale el movimiento que se produce a nivel del ciego y colon proximal que tiene como finalidad favorecer la
absorción de agua y sales.
A. De masa
B. De mezcla
C. Propulsivo
D. Peristáltico

19. La enfermedad de Hirchsprung se caracteriza por una dilatación anormal de colon y disminución de los
movimientos propulsivos que traerá como principal consecuencia …………
A. diarrea acuosa.
B. estreñimiento crónico.
C. dilatación de la válvula ileocecal.
D. disminución de la flora intestinal.

20. ¿Cuál de los siguientes factores estimulan el vaciamiento gástrico?


A. Colecistoquinina (CCK)
B. Neuropéptido Y
C. Secretina
D. Gastrina

21. La secreción de ………………………. estimula la motilidad gástrica.


A. Colecistoquinina
B. Secretina
C. Gastrina
D. Motilina

22. La hormona ……………………….. es secretada por las células “I” del …………..
A. Colecistoquinina / duodeno y yeyuno
B. Gastrina / duodeno y yeyuno
C. Colecistoquinina / estómago
D. Gastrina / estómago
GLUT2, SGLT1, GLUT5

CI 1

1. La motilidad intestinal es estimulada principalmente por el:


- Sistema simpático
- Sistema piramidal
- Plexo de Auerbach
- Sistema parasimpático
2. La peristalsis o peristaltismo hace referencia a:
- Motilidad para movilizar el alimento de proximal a distal.
- No es parte de la motilidad
- Motilidad para mezclado de alimentos.
- Motilidad para fraccionamiento de alimentos.
3. Marque el órgano que se encuentra más distal en el tubo digestivo.
- Estomago
- Ciego
- Íleon
- Duodeno
4. Marque la respuesta incorrecta:
- La mucosa consta de epitelio, lámina propia y muscularis mucosae.
- En todo el tubo digestivo, se observa dos capas de muscular propia: circular
interna y longitudinal externa
- El colon contiene tenias
- Fuera de la cavidad abdominal, el esófago presenta capa adventicia.
5. Al iniciar la digestión, aumenta el consumo de oxígeno por la mucosa. Esto conlleva a
una hipoxia local, lo cual hace que se libere _____________, el cual produce
vasodilatación:
- Colecistoquinina
- Adenosina
- Histamina
- Noradrenalina
6. Cuál de las siguientes estructuras no tiene vasos sanguíneos:
- Epitelio intestinal
- Ligamento
- Omento
- Mesenterio
7. Paciente tiene una úlcera sangrante en el segundo tercio del Yeyuno. La arteria de la
cual proviene la sangre arterial para dicha zona es la arteria:
- Mesentérica superior
- Tronco celíaco
- Mesentérica inferior
- Iliaca común
- Gástrica izquierda
8. Paciente de 24 años con dolor abdominal tipo cólico intenso en mesogastrio. Según
sus conocimientos de macroestructura, el origen del dolor puede ser el ___________:
- Íleon
- Colon
- Estómago
- Esófago
9. Dentro de las funciones del abdomen, se encuentra la defecación y micción, en las
cuales la presión intra abdominal debe:
- Aumentar
- No tiene relación el abdomen con dichas funciones
- Mantenerse igual
- Disminuir
10. En la inspiración, la pared abdominal debe ____________ para ____________:
- Contraerse aumentar presión intra abdominal
- Relajarse disminuir presión intra torácica
- Relajarse aumentar presión intra abdominal
- Contraerse aumentar presión intra torácica

CI 2

1. Respecto a los péptidos gastrointestinales, marque lo correcto.


- No existe sustancia neurocrina que tenga efecto en la motilidad del tubo
digestivo
- Las sustancias paracrinas pueden viajar a través de vasos sanguíneos
- Las sustancias neurocrinas son peptidos que hacen su efecto en distancias
cortas
- Las sustancias paracrinas atraviesan la circulación portal
2. Al disminuir el pH duodenal por el HCl gástrico, se libera principalmente una hormona
cuya célula diana es:
- Acinos pancreáticos
- Células ductales del colédoco
- Célula ductal del Wirsung
- Células S del intestino
3. En un paciente con gastroparesia (motilidad lenta del estómago), que presenta
distensión abdominal después de comer, usted le recomendaría que evite el consumo
de lípidos y aminoácidos para disminuir la acción de:
- Secretina
- CCK
- Somatostatina
- Gastrina
4. La razón por la que el potencial de acción viaja rápidamente en sentido longitudinal
por el musculo liso gastrointestinal es la presencia de uniones en hendidura, además
de la presencia de:
- La presencia del plexo submucoso de Meissner
- Varicosidades
- Mayor cantidad de ACh
- Las fibras musculares no se disponen en haces musculares
5. Paciente obeso con Covid-19 es intubado por interno inexperto, quien al solicitar que
bombeen aire dentro del tubo endotraqueal, nota que el epigastrio se distiende. Al
sospechar que ha introducido el tubo en el estómago, también es cierto que:
- Disminuiría el tono del píloro
- Aumenta la frecuencia de ondas lentas
- Aumenta el pH gástrico
- Disminuye el pH gástrico
6. En un paciente con diarrea por hipermotilidad, usted sospecharía en el posible
aumento de las siguientes sustancias, excepto:
- Sustancia P
- ACh
- Péptido intestinal vasoactivo (VIP)
- Motilina
7. El ecografista sabe que para poder visualizar el nacimiento de la arteria mesentérica
superior, debe colocar el transductor sobre la piel de la siguiente región abdominal:
- Epigastrio
- Hipocondrio derecho
- Hipogastrio
- Mesogastrio
8. Al consumir un pan con mantequilla, la sensación de hambre disminuye debido a la
acción de:
- Grelina
- Somatostatina
- Colecistoquinina (CCK)
- Leptina
9. Una de las siguientes sustancias no comparte con las otras la misma acción sobre la
producción de ácido gástrico:
- Péptido insulinotrópico dependiente de glucosa (GIP)
- Colecistoquinina
- Somatostatina
- Secretina
10. Al ingerir grandes cantidades de dulces, con la subsecuente estimulación de incretinas,
usted esperaría que el apetito ______________, debido a __________________
- disminuya insulina
- aumente CCK
- disminuya CCK
- aumente grelina

PARCIAL

1. Al rozar agua caliente en la punta de la lengua, usted esperaría que el estímulo viaje a
través del nervio:
- lingual
- cuerda del tímpano
- glosofaríngeo
- hipogloso
2. Al ingresar líquidos o sólidos en la cavidad oral, un mecanismo que permite que una
persona respire mientras mastica es:
- el movimiento hacia afuera de los pliegues palatogloso y palatofaríngeo
- la elevación del paladar blando
- la depresión de la parte posterior de la lengua
- la depresión del paladar blando
3. Paciente adulto con reflujo gastroesofágico es más probable que presente:
- descalcificación del esmalte
- destrucción de ameloblastos
- remodelación del esmalte
- desfluorización de los dientes
4. La masticación es básicamente:
- importante para la digestión sobretodo de carnes
- un ralentizador del vaciamiento gástrico
- una actividad consciente
- un movimiento reflejo
5. Paciente de 34 años es víctima de asalto con arma de fuego, recibiendo un impacto
directo en el abdomen. En base a la radiografía, usted puede registrar en la historia
clínica que el proyectil se encuentra topográficamente en el:

- flanco derecho
- mesogastrio
- flanco izquierdo
- hipocondrio izquierdo
6. Señale cuál de las estructuras que en el embrión se encuentra comunicada con el saco
vitelino por medio del conducto onfalomesentérico:
- D
- B
- C
- A
7. En un paciente de 3 semanas de edad, con vómitos en proyectil, y nódulo epigástrico
reptante, usted esperaría encontrar:
- colecistoquinina aumentada
- vómitos biliosos e intolerancia a los ácidos grasos
- distensibilidad disminuida de la región oral del estómago
- engrosamiento de la circular interna pilórica
8. ¿Cuál de las siguientes estructuras tiene inervación somática?
- Estómago
- Peritoneo visceral
- Mesosigmoides
- Peritoneo parietal
9. Estas diseñando un proyecto de investigación sobre los niveles de colesterol que se
absorben luego de una comida grasosa y deseas cuantificar la cantidad de colesterol
que es absorbido por el intestino antes que el hígado lo metabolice ¿de cuál de los
siguientes vasos obtendrías la muestra para tu análisis?
- Vena porta
- Vena cava superior
- Conducto torácico
- Vena hemiácigos accesoria
10. Durante el paso del bolo hacia la orofaringe, se desencadena una serie de
contracciones musculares que estrechan la cavidad faríngea. Estas contracciones están
mediadas por el nervio craneal:
- X
- XI
- XII
- IX
11. Cuando el istmo de las fauces se cierra, se evita que el alimento pase hacia la
orofarínge y permite respirar mientras se mastica. Este cierre se debe a la contracción
y aproximación de los músculos:
- Palatoglosos
- Palatofaríngeos
- Estiloglosos
- Estilofaríngeos
12. Las siguientes alternativas son factores que determinan la patencia y función adecuada
del esfínter esofágico inferior, EXCEPTO:
- Plicatura diafragmática
- Hipertrofia de la circular interna
- Angulación con el estómago
- Canales lentos de calcio
13. Respecto al peristaltismo intestinal, para cumplir la ley del intestino , usted espera que
a nivel distal del quimo se libere:
- péptido liberador de gastrina (GRP)
- acetilcolina
- péptido intestinal vasoactivo
- sustancia P
14. El estímulo habitual para el movimiento peristáltico es:
- acción de la sustancia P
- contracción de la musculatura circular Interna
- estimulación vago-vagal
- distensión local
15. ¿Cuál de las siguientes condiciones considera que es un trastorno de la musculatura
lisa esofágica?
- Acalasia
- Asinergia faringoesfinteriana
- Hipotonía de los constrictores faríngeos
- Hipertonía del esfínter esofágico superior
16. ¿Cuál de las siguientes alternativas es correcta sobre la motilidad esofágica?
- Las ondas primarias son propulsoras y pueden no ser precedidas por deglución
- Las ondas primarias no son propulsoras y siempre van precedidas de deglución
- Las ondas secundarias son propulsoras y siempre van precedidas de deglución
- Las ondas secundarias son propulsoras y no van precedidas de deglución
17. El peristaltismo depende que a nivel distal del bolo se secrete:
- noradrenalina secretada por las fibras del sistema simpático
- acetilcolina por las neuronas provenientes del nervio vago
- óxido nítrico por células endoteliales locales
- péptido intestinal vasoactivo por neuronas
18. ¿Cuál de las siguientes alternativas es correcta sobre el movimiento peristáltico?
- Es un reflejo largo que depende de la integración con el tronco encefálico
- Se dirige en sentido distal siempre, nunca en sentido proximal
- El contenido intestinal avanza sólo 5-10 cm
- Es independiente del plexo mientérico
19. En un estudiante de medicina que está rindiendo un examen parcial, lo más probable
es que en ese momento su tránsito intestinal se encuentre:
- muy lento
- muy acelerado
- estimulado por acción de la sustancia P
- sin alteraciones
20. Las siguientes alternativas son ciertas sobre las contracciones tónicas del músculo
gastrointestinal, EXCEPTO:
- Tienen relación con el ingreso persistente de iones sodio
- Se encuentran principalmente en esfínteres
- Tienen regulación hormonal
- Obedece a una mayor frecuencia de potenciales en espiga
21. Con respecto de la regulación del pH del estómago; al utilizar un bloqueador de
histamina, usted espera que el pH del estómago:
- aumente
- disminuya
- se mantenga sin cambio
- se neutralice por acción de bicarbonato
22. Las siguientes hormonas disminuyen el vaciamiento gástrico, EXCEPTO:
- Gastrina
- Péptido insulinotrópico dependiente de glucosa
- Colecistoquinina
- Secretina
23. El frenillo de los labios se encuentra en:
- el piso de la boca
- la cavidad oral
- la cavidad vestibular
- el dorso de la lengua
24. En un paciente con shock hipovolémico, la peristalsis intestinal se encuentra:
- aumentada
- sin cambios
- invertida
- disminuida
25. Sobre el control de la peristalsis del tubo digestivo, ________________ es un
mediador neural que induce la relajación durante la peristalsis.
- la somatostatina
- el péptido intestinal vasoactivo
- la acetilcolina
- la serotonina

CI 3

1. En cuanto a las sustancias secretadas por el estómago ¿Cuál de las siguientes


sustancias estimula la liberación de pepsinógeno?
- Pepsinógeno
- Secretina
- Colecistoqunina
- Gastrina
2. Las células enteroendocrinas en el estómago se localizan en la glándula oxíntica, al
mismo nivel que las células:
- parietales
- principales
- mucosas
- absortivas
3. La fase intestinal de la secreción gástrica se debe básicamente a la participación de las
células:
- G del duodeno
- I del yeyuno
- S del íleon
- D del estómago
4. El conducto de Stenon, para entrar a la cavidad vestibular, debe atravesar el músculo:
- genihioideo
- buccinador
- milohioideo
- masetero
5. Paciente con acalasia es sometido a tratamiento endoscópico o quirúrgico, usted le ha
informado al paciente previamente que es posible que una complicación de este
tratamiento es que quede con cierto grado de:
- reflujo gastroesofágico
- gastritis
- odinofagia
- úlceras gástricas
6. La sangre que lleva la vena porta es tipo:
- mixta
- arterial
- venosa
7. Los pliegues gástricos gruesos son prácticamente inexistentes a nivel de:
- el cuerpo
- el fondo
- la incisura angularis
- el antro
8. En la producción de HCl, la acción de la somatostatina disminuye la accion de:
- las prostaglandinas
- la histamina
- la acetilcolina
- la gastrina
9. En el estómago se secretan las siguientes sustancias, EXCEPTO:
- grelina
- Correcto
- motilina
- somatostatina
- gastrina
10. La vena porta se forma gracias a la unión de la vena mesentérica superior con la vena:
- mesentérica inferior
- esplénica
- celiaca
- gástrica izquierda

CI 4

1. En el hígado, el aumento de la resistencia vascular en los sinusoides hepáticos


ocasionará:
- aumento de la presión de llenado vesicular
- salida de plasma hacia el intersticio
- aumento del flujo hacia la vena porta
- aumento del flujo hacia la vena cava superior
2. En un paciente con carcinoma de páncreas, el tumor ha invadido la unión entre la
venas esplénica y mesentérica superior; eso quiere decir que estamos seguros que el
tumor se encuentra a nivel del ________ del páncreas.
- cuerpo
- cola
- cabeza
- cuello
3. En un paciente con cirrosis hepática la cabeza de medusa que aparece en la pared
abdominal, podría desaparecer si al paciente se le:
- esclerosa las venas hemorroides internas
- oblitera el ligamento redondo
- administra antiandrógenos
- oblitera la arteria gástrica izquierda
4. En un paciente con intoxicación por órganos fosforados, la acción de la
colecistoquinina (CCK) está bloqueada a nivel de:
- el esfínter de Oddi
- la vesícula biliar
- la célula parietal
- el sistema nervioso central
5. Con respecto a la microestructura del hígado ¿Cuál de las siguientes alternativas es
correcta?
- La célula de Ito se encuentra en el espacio de Disse y reserva glucógeno
- Los hepatocitos están interconectados por uniones herméticas
- La célula de Kupffer se encuentra fuera del sinusoide y fagocita células
- El sinusoide es un capilar fenestrado
6. Paciente con Lupus Eritematosos que desarrolla hipertensión portal debido a
trombosis portal, es probable que desarrolle várices a nivel de:
- recto inferior
- recto superior
- hemorroides externas
- canal anal
7. El efecto de un medicamento colerético se evidencia por:
- la mayor producción de colesterol en la bilis
- la disminución de absorción de sales biliares
- el aumento de secreción biliar
- el aumento de formación de micelas
8. En un paciente con cáncer de páncreas y que desarrolla ictericia, la localización más
probable del tumor es en:
- la vesícula por metástasis
- el cuello del páncreas
- la cabeza del páncreas
- la cola del páncreas
9. En un recién nacido menor de 24 horas con atresia biliar, se encuentra elevación de la:
- bilirrubina directa
- bilirrubina indirecta
- alanina aminotransferasa (ALT)
- hemoglobina
10. ¿Cuál de las siguientes alternativas es correcta sobre la estructura hepática?
- Los colangiocitos producen bilis
- En la triada portal, se encuentra la vena derivada de la suprahepática
- El flujo sinusoidal en el lobulillo hepático es de adentro hacia afuera
- El flujo biliar en el lobulillo hepático es centrífugo
EXAMEN PARCIAL SISTEMA DIGESTIVO
(ME154)
Ciclo 202000

Sección:Todas
Profesores:Alva Muñoz, Jose Carlos
Duración:35 minutos.
Indicaciones:
- Lea atentamente cada pregunta antes de responder.
- Se prohíbe el uso del celular y cualquier dispositivo electrónico.
- Está prohibido intercambiar materiales.
- Coloque su código de alumno en la tarjeta de respuestas. Si su código contiene una letra
reemplácela por un valor numérico siguiendo la siguiente equivalencia: A=9, B=8, C=7, D=6, E=5, F=4
y G=3.
- Traslade sus respuestas a la tarjeta, llenando los círculos de manera completa con lapicero negro o
azul. Está prohibido el llenado con lápiz, lapicero de otro color o con lapicero de tinta borrable.
- Sea cuidadoso en el llenado de la tarjeta de respuestas, pues solo esta tiene validez para la
calificación.
- Al terminar su examen avise al docente a cargo, no se levante de su sitio; debe entregar la hoja de
respuestas con la carátula del examen, este cuadernillo de preguntas se lo llevará cada estudiante.

1. Paciente de sexo masculino de 82 años de edad ingresa a emergencia con dolor abdominal agudo y
diarreas. Se le realiza una arteriografía en la que se observa que la arteria aorta tiene un trombo
ocluyendo el 95% del flujo, a nivel del nacimiento de la arteria mesentérica inferior. ¿Cuál de las
siguientes arterias podría contribuir a la irrigación colateral del colon descendente?
a) cólica media
b) sigmoidea
c) rectal superior
d) ileocólica

2. Niño de 5 años presenta dolor esofágico y hematemesis (vómitos hemorrágicos) luego de tragarse
una espina de pescado. En la endoscopía se observa perforación del esófago distal a la cuarta
estrechez esofágica. ¿Las ramas de cuál de las siguientes arterias estarán lesionada con mayor
probabilidad?
a) Gástrica izquierda
b) Bronquiales
c) Frénica inferior
d) Tiroidea inferior

3. Al ingerir una cucharada de mantequilla es muy probable que se disminuya la sensación de hambre
por medio de la activación de la vía POMC/CART (POMC=proopiomelanocortina y
CART=transcripción regulada de cocaína y anfetamina), activada directamente por la hormona:
a) colecistoquinina (CCK)
b) insulina
c) grelina
d) secretina

4. Al comer unas papitas fritas con mayonesa, el vaciamiento gástrico disminuye por efecto directo
de la hormona: a)colecistoquinina (CCK)
b) bombesina
c) motilina
d) gastrina

5. Recién nacido de dos horas es diagnosticado de hernia umbilical de 1,5 cm de diámetro; el


cirujano pediatra solicita una tomografía abdominal en donde se evidencia que la hernia
umbilical está ocupada por una porción del tracto gastrointestinal. ¿Qué porción del tracto
gastrointestinal estaría ocupando esta hernia con mayor probabilidad?
a) Íleon
b) Colon sigmoides
c) Duodeno
d) Colon transverso
6. Recién nacido de 7 horas, de parto por cesárea debido a polihidramnios (aumento del volumen del
líquido amniótico), con regurgitación de la leche materna y artificial, y no ha presentado meconio.
Se le realiza una tomografía donde se evidencia aire en el estómago y una malformación del
desarrollo esofágico. Con respecto a esta malformación lo más probable es que se pueda tratar de
una atresia esofágica:
a) proximal con fístula traqueoesofágica distal
b) distal con fístula traqueoesofágica proximal
c) proximal y distal
d) sin fístula

7. Lactante de 6 meses de edad que es traído a consulta por presentar vómitos no biliosos a repetición
y retraso en el crecimiento. En la radiografía de abdomen simple se observa nivel hidroaéreo en
estómago y en primera porción de duodeno (doble burbuja). ¿Cuál de las siguientes alternativas
puede explicar la condición del lactante?
a) Páncreas anular
b) Atresia duodenal en la tercera porción
c) Atresia yeyunal
d) Hipertrofia del píloro

8. En ausencia o deficiencia de la secreción de la hormona motilina,


se producirá: a)sobrecrecimiento bacteriano
b) diarrea
c) aumento del vaciamiento gástrico
d) hipertrofia del píloro

9. La estimulación parasimpática aumenta la motilidad intestinal, mientras que la estimulación


simpática la disminuye. ¿Sobre cuál de las siguientes alternativas el sistema nervioso autónomo
actúa para el control de la motilidad intestinal?
a) Potencial de membrana en el plexo mientérico (de Auerbach)
b) Frecuencia de ondas lentas
c) Secreción de secretina
d) Nivel de IP3 en el plexo submucoso (de Meissner)

10. En un recién nacido con protrusión de contenidos abdominales y cubiertas por amnios o peritoneo,
es cierto que: a)Se presenta por un defecto en el cierre de la pared
b) Se acompaña de otras malformaciones congénitas
c) Se debe al no retorno de la hernia fisiológica
d) Se produce a través del ombligo

11. Lactante de 20 días con estreñimiento, distención abdominal progresiva, acompañada


ocasionalmente de vómitos biliosos. Como antecedente, el meconio lo eliminó por primera vez a las
72 horas de nacido. Su mamá menciona que ayuda a la evacuación con ayuda de un termómetro
rectal. Se sospecha de megacolon agangliónico (Enfermedad de Hirschsprung). ¿Cuál de las
siguientes alternativas explica el caso?
a) Se presenta contracciones tónicas en la región ano rectal
b) Se presenta dilatación de tracto gastrointestinal afectado
c) Las células ganglionares sólo han migrado al ano recto
d) La zona que más se afecta es inervada por fibras del nervio esplácnico menor

12. ¿Cuál de los siguientes reflejos disminuye el tránsito


gastrointestinal?
a)Doloroso
b) Gastrocólico
c) De defecación
d) Colicoileal

13. Al ingerir una sustancia ácida como el vino (pH 3), se estimula la motilidad gástrica por acción
de la hormona:
a)motilina
b) secretina
c) colecistoquinina (CCK)
d) bombesina
14. Paciente de 24 años acude a consulta externa por presentar una fístula oronasal (comunicación
entre la cavidad oral y la cavidad nasal). Está fístula está asociada al antecedente de haber sido
operada de paladar hendido a los dos años de edad, durante una campaña gratuita extranjera de
corrección de paladar fisurado. ¿Cuál de las arterias palatinas podría haberse lesionado durante
esa cirugía?
a) Mayor
b) Menor
c) Ascendente
d) Rama palatina de la faríngea ascendente

15. Paciente de sexo masculino de 52 años con úlcera péptica gástrica de 14 años de evolución, con
cuadro de hemorragia digestiva alta hace 4 meses, sin cicatrización de la úlcera. Entre las opciones
quirúrgicas se considera realizarle un vaguectomía troncal (sección del nervio vago) a nivel del
hiato esofágico. ¿Cuál de las siguientes complicaciones podría esperarse producto de la pérdida de
inervación parasimpática?
a) Menor inervación del colon ascendente
b) Se perderá el reflejo de defecación
c) Se perderá el reflejo de micción
d) Impotencia sexual

16. Paciente de 23 años con bulimia es traída a la emergencia deshidratada, semiconsciente y con
alcalosis metabólica. Los vómitos autoinfligidos por esta paciente se producen por estimulación
de receptores en la base de la lengua que mandan información directamente al:
a) núcleo del tracto solitario
b) centro del vómito en el tallo encefálico
c) zona quimiorreceptora gatillo
d) cerebelo

17. Niño de 3 años es traído a emergencia por madre quien manifiesta que hace 10 horas deglutió una
pila pequeña de reloj de bordes romos. El niño está asintomático. Usted la tranquiliza diciéndole es
un cuerpo extraño tan pequeño de seguro que va a seguir el tránsito intestinal como lo haría un
bolo alimenticio, y que lo más probable es que en ese momento se encuentre en:
a) colon
b) estómago
c) yeyuno
d) recto

18. Los movimientos en masa son un tipo de movimiento muy importante, una de las
consecuencias de estos movimientos es:
a) la distensión rectal
b) el peristaltismo del intestino delgado
c) la retropulsión gástrica
d) la contracción del esfínter anal interno

19. Durante la deglución, al momento que el bolo alimenticio pasa por el esfínter esofágico superior, se
espera que la presión intraesofágica:
a) disminuya en el cardias
b) disminuya en el tercio medio del esófago
c) aumente en la porción distal al bolo
d) aumente en el tercio medio del esófago

20. Paciente con enfermedad de Chagas que presenta disfagia a sólidos. ¿Cuál de las siguientes puede
ser la causa de esta complicación?
a) Disminución de células ganglionares en el esfínter esofágico inferior
b) Aumento en la liberación de óxido nítrico en el esfínter esofágico inferior
c) Disminución de las neuronas que liberan péptido intestinal vasoactivo
d) Aumento de la actividad de la motilina en el esófago distal
EXAMEN PARCIAL 2020-02

Pregunta 1

Niña de 6 años se asusta por que se le ha aflojado un diente deciduo. Este fenómenose produce
por:

laxitud del ligamento periodóntico

fractura del cemento

desmineralización del esmalte dental

aumento anómalo de la predentina

2.- ¿Cuál de las siguientes estructuras deriva del intestino anterior?


B

3.- En un paciente de 3 semanas de edad, con vómitos en proyectil, y nódulo epigástrico reptante,
usted esperaría encontrar:

engrosamiento de la circular interna pilórica

distensibilidad disminuida de la región oral del estómago

colecistoquinina aumentada

vómitos biliosos e intolerancia a los ácidos grasos

4.- Sobre el control de la peristalsis del tubo digestivo, ________________ es un mediador neural
que induce la relajación durante la peristalsis.

el péptido intestinal vasoactivo

la serotonina

la acetilcolina

la somatostatina

5.- En el plexo mientérico, el origen de los impulsos eferentes está en:

los ganglios paravertebrales

las células intersticiales de Cajal

el plexo de Meissner

el plexo de Aurbach
6.- Un familiar le comenta que tiene úlcera gástrica por exceso de producción de ácido; con sus
conocimientos del sistema digestivo, usted le recomendaría que reduzca el consumo de:

vitaminas

carbohidratos

agua

aminoácidos

7.- Las siguientes alternativas son ciertas en relación al mesenterio, EXCEPTO:

El omento menor deriva del mesenterio ventral

El ligamento esplenorrenal une al bazo con el riñón izquierdo

Los mesenterios cumplen la función de sostén y suspensión de órganos

Los omentos cumplen una función principal de irrigación visceral

8.- ¿Cuál de las siguientes alternativas es una característica de la estructura del esófago?

Aumenta la presión intra esofágica durante la inspiración

Contiene músculo estriado en casi toda su longitud

Contiene glándulas submucosas principalmente en su tercio distal

El esfínter esofágico inferior es un esfínter anatómico

9.- En una cirugía abierta (laparotomía), el cirujano al abrir la cavidad peritoneal por la parte anterior
(línea media), lo primero que observa es:

Estomago

Colon sigmoides

Duodeno

Epiplón mayor
10.- El tubo digestivo posee glándulas, las glándulas submucosas se encuentran en el:

duodeno y recto

íleon y esófago

estómago y duodeno

esófago y duodeno

11.- ¿Cuál de las siguientes alternativas detallan los músculos que ayudan a empujar el bolo hacia
la orofaringe?

Geniogloso y palatofaríngeo

Estilogloso y palatogloso

Hiogloso y geniogloso

Estilogloso y geniogloso

12.- Al ingresar líquidos o sólidos en la cavidad oral, un mecanismo que permite que una persona
respire mientras mastica es:

la elevación del paladar blando

el movimiento hacia afuera de los pliegues palatogloso y palatofaríngeo

la depresión de la parte posterior de la lengua

la depresión del paladar blando

13.- Durante el paso del bolo hacia la orofaringe, se desencadena una serie de contracciones
musculares que estrechan la cavidad faríngea. Estas contracciones están mediadas por el nervio
craneal:

IX

XI

XII
14.- Las siguientes alternativas son factores que determinan la patencia y función adecuada del
esfínter esofágico inferior, EXCEPTO:

Canales lentos de calcio

Plicatura diafragmática

Hipertrofia de la circular interna

Angulación con el estómago

15.- Durante una cena, una gestante inspira por la boca profundamente de manera frecuente; sin
embargo, el organismo evita que el aire ingrese al esófago por la acción:

del músculo cricofaríngeo

del istmo de las fauces

del esfínter esofágico inferior

de la epiglotis

16.- En un estudiante de medicina que está rindiendo un examen parcial, lo más probable es que en
ese momento su tránsito intestinal se encuentre:

muy lento

estimulado por acción de la sustancia P

muy acelerado

sin alteraciones

17.- El principal gobernante sobre todos los movimientos gastrointestinales es el sistema nervioso:
parasimpático

somático

mientérico

simpático
18.- ¿Cuál de las siguientes condiciones considera que es un trastorno de la musculatura lisa
esofágica?

Hipotonía de los constrictores faríngeos

Hipertonía del esfínter esofágico superior

Asinergia faringoesfinteriana

Acalasia

19.- El estímulo habitual para el movimiento peristáltico es:


acción de la sustancia P

contracción de la musculatura circular Interna

estimulación vago-vagal

distensión local

20.- La hormona responsable de los complejos migratorios interdigestivos tiene las siguientes
características, EXCEPTO:

se libera de forma cíclica

se produce en el estómago y el duodeno

cumple funciones de aumentar la motilidad y secreción gástrica e intestinal

es inhibida por el alimento

21.- Respecto al peristaltismo intestinal, para cumplir la ley del intestino , usted espera que a nivel
distal del quimo se libere:

acetilcolina

péptido liberador de gastrina (GRP)

sustancia P

péptido intestinal vasoactivo


22.- ¿Cuál de las siguientes alternativas es correcta sobre el control autónomo del aparato
gastrointestinal?

El plexo mientérico de Auerbach cumple funciones inhibitorias

Las terminaciones nerviosas simpáticas liberan mayor cantidad de adrenalina que noradrenalina

La estimulación simpática estimula a la muscularis mucosae

Al seccionar el vago, la inervación parasimpática del colon sigmoides disminuye

23.- La distención del yeyuno provoca que se:

despolarice el potencial de reposo de membrana

produzca una contracción tónica

disminuya el número de espigas

aumente la frecuencia de las ondas lentas

24.- El frenillo de los labios se encuentra en:

el dorso de la lengua

el piso de la boca

la cavidad oral

la cavidad vestibular

25.- Con seguridad, usted puede decir que la siguiente cicatriz postapendicectomía se encuentra en
la región denominada:

flanco derecho

hipocondrio derecho

flanco izquierdo

fosa iliaca derecha


CLAVES EXAMEN PARCIAL DE SISTEMA
DIGESTIVO 2019 - 00

1. Varón de 30 años es traído a emergencia por agresión abdominal con arma de fuego
(pistola) y es sometido a laparotomía exploratoria, observándose isquemia del colon
ascendente y parte del colon trasverso ¿la lesión de cuál de las siguientes arterias
explicaría esta isquemia?
(unidad 1, sesión 2, logro 6: (Describe la irrigación visceral: arterias de tronco
celiaco, arteria mesentérica superior e inferior, topografía de superficie, órganos
por cuadrante)

a. Celiaca
b. Colónica derecha
c. Mesentérica inferior
d. Mesentérica superior

2. Respecto a las sustancias gastrointestinales que regulan la secreción pancreática;


marque la afirmación correcta:
(unidad 1, sesión 3, logros 2 y 3: describir las hormonas gastrointestinales: estímulos y
funciones)

a. La Secretina, es la hormona más importante para la secreción de bicarbonato por


las células acinares del páncreas
b. La acetilcolina es capaz de estimular la secreción enzimática y de bicarbonato del páncreas
c. La gastrina, es la hormona más importante para la secreción de enzimas pancreáticas
d. La colecistoquinina (CCK) estimula al páncreas solo para secreción enzimática

3. Ante una lesión del X par craneal, ¿cuál de los siguientes músculos mantiene conservada su
función?:
(unidad 2, sesión 08, logro 4: Paladar blando: componentes musculares)

a. Elevador del velo del paladar


b. Tensor del velo del paladar
c. Palatofaríngeo
d. Glosofaríngeo

4. Experimentalmente se utiliza atropina (anticolinérgico) para inhibir la secreción de


gastrina, sin embargo la secreción de esta hormona se sigue dando ante estímulos
vagales. Esta situación se explica porque la atropina:
(unidad 1, sesión 3, logro 3 : describir las hormonas gastrointestinales: estímulo y funciones
de la gastrina y colecistoquinina)

a. Bloquea parcialmente la bomba de protones en la célula G


b. Inhibe la acción de acetilcolina e histamina en la célula G
c. Solo inhibe la acción del péptido GRP en la célula G
d. No bloquea la acción del péptido GRP

5. Un varón de 50 años es sometido a extirpación del duodeno y parte proximal


del yeyuno. Esta situación ocasionaría la pérdida de las células ……….. ,
productoras deque estimula la secreción de
bicarbonato por el páncreas.
(unidad 1, sesión 3, logro 3: describir las hormonas gastrointestianles: estímulos y
funciones de la secretina y péptido insulinotrópico dependiente de glucosa)

a. “S” / secretina
b. Parietales / secretina
c. “I” / colecistoquinina
d. “S” / colecistoquinina
6. Recién nacido que presenta tumoración abdominal a nivel del cordón umbilical
(fotografía). ¿cuál de las siguientes afirmaciones es correcta respecto a este
defecto en el desarrollo embriológico del intestino?: (unidad 1, sesión 5, logro 2:
identificar las anomalías del desarrollo del intestino medio)

a. Corresponde a una Gastrosquisis


b. Las vísceras se hallan cubiertas por piel
c. No está asociado a otras malformaciones
d. Se asocia a
malformaciones
cardiacas y del tubo
neural

7. Varón de 35 años acude a la emergencia por


trauma abdominal y se decida realizar una
laparoscopía exploratoria. El cirujano
observa la disposición de los órganos
abdominales como se representa en el
siguiente esquema. Esta disposición de
órganos se explica por la
rotación(SMA=arteria mesentérica superior)
(unidad 1, sesión 5, logro 3: identificar las anomalías
del desarrollo del intestino medio: defectos de
rotación, estenosis y atresias)

a. anti horaria del intestino medio, en sólo 90°


b. incompleta del intestino medio (270°)
c. horaria del intestino medio
d. horaria del estómago

8. Se evalúa la expresión de la proteína Agrp en una persona con alteración del


apetito; lo correcto respecto a esta proteína es…..
(unidad 1, sesión 3, logro 4: Explica los mecanismos de control del apetito y saciedad )

a. Esta proteína es un potente anorexigénico


b. La mutación del gen que la codifica produce adelgazamiento
c. La sobre producción de la proteína lleva a obesidad por agonismo de receptores MC3 y MC4
d. La sobre producción de la proteína disminuye el apetito por antagonismo de receptores MC4

9. Juana cae de la bicicleta y se fractura la región anterior del hueso maxilar superior con
compromiso de la fosa incisiva. Al examen físico de la región esperaría encontrar
alteración en la sensibilidad de la encía …………………
(unidad 2, sesión 8, logro5: paladar: paladar duro y blando: irrigación e inervación)

a. bucal posterior
b. Lingual anterior
c. palatina anterior
d. palatina posterior
10. Recién nacido es atendido por el neonatólogo y luego entregado a su madre para
dar de lactar; la madre al dar de lactar observa coloración azulada de labios,
acompañado de tos persistente, dificultad respiratoria y distención abdominal. Se le
intenta colocar una sonda nasogástrica pero esta retorna a la cavidad oral en todos
los intentos. ¿Cuál de las siguientes anomalías del desarrollo es el más probable en
este caso? (unidad 1, sesión 4, logro 3: identificar las anomalías en el desarrollo del
esófago: atresia y/o fístula traqueo esofágica)

a. Estenosis esofágica proximal con Fístula traqueo esofágica distal


b. Atresia esofágica proximal con fístula traqueoesofágica distal
c. Atresia esofágica distal con fístula traqueoesofágica proximal
d. Fístula traqueoesofágica proximal y distal

11. ¿Cuál de los siguientes mecanismos ocurre durante la defecación?


(unidad 2, sesión 13, logro 6: motilidad del intestino grueso: contracciones
segmentarias, movimientos en masa, defecación y reflejo gastrocólico)

a. Contracción refleja del esfínter anal interno


b. En la posición de “cuclillas” el músculo puborectal se halla relajado
c. Relajación del esfínter anal externo por efectos del VIP y óxido nítrico
d. La materia fecal en el recto estimula la contracción del sigmoides por los nervios pudendos

12. La estructura número 4 (gráfico) corresponde a


……….… y está ………..
(unidad 2, sesión 9, logro 2:
Partes de un diente. Capas
del diente: Esmalte:
características y células que
lo producen)

a. el cemento / mineralizado en 90%


b. la dentina / formada por ameloblastos
c. el esmalte / formado
por células derivadas
del mesénquima
d. la dentina / formado por
células derivadas de la
cresta neural

13. Un paciente luego de un accidente sufre lesión del piso de la boca, se constata
daño del nervio “cuerda del tímpano”, en este caso se esperaría encontrar
disminución de lade la lengua
(unidad 2, sesión 10, logro 3: Irrigación e inervación de la lengua)

a. Motilidad en los dos tercios anteriores


b. Sensación del gusto en el tercio posterior
c. Sensación del gusto en los dos tercios anteriores
d. Sensibilidad al tacto en los dos tercios anteriores

14. ¿Cuál de las siguientes afirmaciones es la correcta sobre la gastrina?


(unidad 1, sesión 3, logro 1: reconocer las características de las sustancias
reguladoras gastrointestinales: hormonas, sustancias paracrinas y neurocrinas)

a. Produce atrofia de la mucosa gástrica


b. Es producida por la célula G del cuerpo gástrico
c. Es estimulada por la distensión gástrica y el Ph bajo
d. Actúa en la célula diana mediante su receptor CCk tipo B
15. Al recibir un paciente con signos de hipovolemia y antecedente de trauma en
abdomen por accidente de tránsito, usted identifica radiológicamente: lesión de
primera vértebra lumbar y signos de lesión en páncreas; durante la cirugía se observó
pobre irrigación de asas intestinales. El vaso afectado es la arteria ……..
(unidad 1, sesión 1, logro 6: reconocer las estructuras a nivel de L1, nivel de los principales
vasos sanguíneos)

a. esplénica
b. hepática común
c. mesentérica inferior
d. mesentérica superior

16. Un paciente sufre de daño a nivel del cuello con lesión muscular en la región de la
faringe. En el examen físico se determina dificultad para la elevación de la faringe y
para el cierre del itsmo de las fauces. En este caso, probablemente esté afectado el
músculo:
(unidad 2, sesión 11, logro 2: Músculos de la faringe: identificación, constrictores y
longitudinales)

a. palatogloso
b. estilofarinfeo
c. palatofaringeo
d. constrictor inferior

17. Varón de 50 años a quien le realizan la curación de la segunda molar de la arcada


superior derecha. En un momento determinado, el paciente acusa de intenso
dolor de la pieza dentaria en tratamiento. La vía aferente del dolor viaja a través
del nervio …………
(unidad 2, sesión 9, logro 6: Inervación de los dientes)

a. trigémino V2
b. trigémino V3
c. naso palatino
d. palatino menor

18. La distención gástrica por los alimentos produce incremento de secreción de HCl mediante la
producción de
………….. que estimula a las células ……………. vía proteína ………..
(Unidad 1, sesión 3, logro 2: Describe las hormonas gastrointestinales: Estímulo y funciones de
la gastrina y colecistoquinina)

a. gastrina / parietal / Gq
b. gastrina / principal / Gs
c. acetilcolina / parietal /Gi
d. acetilcolina / principal / Gi

19. Un niño de tres años llega a emergencia con disfagia (dificultad para tragar), dolor
retro esternal, salivación y llanto. Se sospecha de ingesta de cuerpo extraño
(moneda) en el esófago; al ser evaluado se constata en una radiografía presencia de
cuerpo extraño a nivel de C6 (6° vértebra cervical). El cuerpo extraño estará
suspendido a nivel del estrechamiento producido por………..
(unidad 2, sesión 11, logro4: Esófago, características anatómicas, relación con órganos
vecinos y estrecheces)

a. el cayado aórtico
b. el hiato esofágico
c. el músculo cricofaríngeo
d. el bronquio principal izquierdo

20. La triada portal (arteria hepática, vena portal y conducto biliar común) está contenida en el
ligamento
…….……… y derivan embriológicamente del ……
(Unidad 1, sesión 1, logro 4: Identifica el peritoneo, mesenterio, omento y ligamentos,
retroperitoneo.)

a. hepato duodenal / mesenterio ventral


b. gastro esplénico / mesenterio dorsal
c. hepato gástrico / omento menor
d. falciforme / omento menor
21. En relación al movimiento de
peristaltismo del tubo
digestivo: en la flecha negra
del gráfico se produce la
liberación de ……………… a
nivel del músculo ………..
(unidad 2, sesión 7, logro 6:
Control hormonal y tipos de
movimiento)

a. noradrenalina, sustancia P y neuropéptido “ Y” / circular


b. acetilcolina y sustancia P / longitudinal
c. óxido nítrico y PIV / longitudinal
d. óxido nítrico y PIV / circular

22. Un paciente refiere no percibir algunos sabores, al examen físico se constata


alteración en la percepción de sabores y del dolor en el tercio posterior de la lengua
¿Cuál de los siguientes nervios estará alterada en su función?
(unidad 2, sesión 10, logro 5: Sabores, tipos y mecanismos moleculares para su detección)

a. Lingual (rama del V par)


b. Cuerda del tímpano (VII par)
c. Glosofaríngeo (IX par)
d. Hipogloso (XII par)

23. El gráfico detalla la


estructura de la pared del
tubo digestivo intestinal
¿Cuál de las siguientes
asociaciones es correcta?
(unidad 2, sesión 7, logro 1:
La pared y músculo liso
gastrointestinal )

a. “1” – peristaltismo
b. “2” – secreción enzimática
c. “3” – deriva del mesodermo
d. “4” – doble hoja de tejido graso

24. En el caso de un paciente con gastrinoma (tumor productor de gastrina), la presencia


de úlceras duodenales y erosión de la mucosa gástrica, se debe principalmente a…….
(unidad 1, sesión 3, logro 2: describir las hormonas gastrointestinales: estímulo y funciones de
la gastrina y colecistoquinina)

a. la acción directa de la gastrina sobre la célula principal


b. la sobre expresión de los receptores “G” en la célula parietal
c. el exceso de HCl por estímulo de receptores CCK-B en la célula parietal
d. el exceso de HCl por estímulo directo de receptores de acetilcolina en la célula parietal

25. El reflejo entero gástrico se caracterizan por:


(unidad 2, sesión 13, logro 6: Motilidad del intestino grueso: contracciones
segmentarias, movimientos en masa defecación y reflejo gastrocólico)

a. favorecer la motilidad gástrica gracias a la CCk


b. inhibir la motilidad gástrica y estimular la secreción ácida
c. movilizar grandes volúmenes desde el estómago al duodeno
d. originarse debido a la distensión duodenal y presencia del quimo ácido
26. Mauricio tiene dificultad para deprimir el paladar y elevar la parte posterior de la
lengua. En este caso estará afectado un músculo ………………., específicamente el
músculo …………….
(Unidad 2, sesión 10, logro 2: Músculos de la lengua: clasificación, identificación y sus
funciones)

a. intrínseco – longitudinal inferior


b. extrínseco – palatogloso
c. extrínseco – transverso
d. extrínseco – estilogloso

27. Una de las funciones del músculo señalado es:


(Unidad 2, sesión 8, logro 3: Describir el Piso de la boca:
estructuras blandas que la conforman)

a. deprimir la lengua
b. elevar el paladar blando
c. deprimir el hioides cuando la mandíbula está fija
d. deprimir la mandíbula cuando el hioides está fijo

28. Paciente varón de 30 años es evaluado por probable enfermedad de Chagas, cursa
con problemas de motilidad del colon; los estudios de biopsia determinan
ausencia de células ganglionares. Según el gráfico
¿cuál es la capa en la que se determina la ausencia de dichas células?
(unidad 1, sesión 2, logro 1: describir las generalidades de la estructura del tubo digestivo:
esófago, estómago intestino delgado y grueso)

a. Mucosa - 1
b. Muscular propia – 1
c. Muscular de la mucosa - 2
d. Muscular propia - 3

1 2

3
29. Paciente varón de 32 años, que acude a centro de salud por presentar de forma
progresiva desde hace 1 año dificultad para ingerir alimentos sólidos y luego líquidos;
refiere regurgitaciones alimentarias y marcada pérdida de peso (15 kilos). Radiografía
baritada (sustancia de contraste) de esófago se muestra en la figura. El presente caso
se explica por……………….
(Unidad 2, sesión 12, logro 4: Identificar y describir la función de los esfínteres esofágicos)

a. aumento de la peristalsis esofágica


b. relajación incompleta del esfínter pilórico
c. relajación incompleta del esfínter esofágico inferior
d. perdida de producción de PIV y
óxido nítrico en el esfínter
esofágico superior

30. En condiciones normales, el ingreso de 600 ml de líquido es el estómago provoca un


aumento de presión intragástrica de unos 12 cm de H 2O. Después de una vagotomía
(corte del nervio vago) es de esperar que el ingreso del mismo volumen de líquido
ocasionede la presión intragástrica.
(Unidad 2, sesión 13, logro 1: Describe la Motilidad gástrica: relajación receptiva)

a. la disminución
b. la no variación
c. un aumento mayor
d. un aumento similar o igual
SISTEMA
DIGESTIVO
(ME154)
EXAMEN FINAL
2019 01

Profesores : Alfaro Salazar, Herberth Romulo; Callata Caceres, Gunter; Cayo Quiñe, Alexandra Mariel;
Correa Borit, Jorge Mauricio; Cruz Cutty, Lourdes Marylin; Guzmán Calderón, Gerly Edson;
Jáuregui Farfán, Jorge Jesús; Mayor Zevallos, Otto Alberto; Montoya Suárez, José Luis;
Palacios Bazan, Enrique Elias; Robles Pino, Alexander Anibal; Wong Bravo, Juan Carlos
Sección : Todas las secciones
Duración : 70 minutos.
Indicaciones:
Lea atentamente cada pregunta antes de responder:
- Se prohíbe el uso del celular y cualquier dispositivo electrónico.
- Está prohibido intercambiar materiales.
- Coloque su código de alumno en la tarjeta de respuestas. Si su código contiene una letra
reemplácela por un valor numérico siguiendo la siguiente equivalencia: A=9, B=8, C=7, D=6, E=5, F=4,
G=3.
- Traslade sus respuestas a la tarjeta, llenando los círculos de manera completa con lapicero negro o
azul. Está prohibido el llenado con lápiz, lapicero de otro color o con lapicero de tinta borrable.
- Sea cuidadoso en el llenado de la tarjeta de respuestas, pues solo esta tiene validez para la
calificación.
- Al terminar su examen avise al docente a cargo, no se levante de su sitio; debe entregar la hoja de
respuestas con la carátula del examen, este cuadernillo de preguntas se lo llevará cada estudiante.

1. La explicación fisiológica de presentar somnolencia de 30 minutos a 1 hora después de ingerir


alimentos, se explica por:
a. Aumento del cloro intraluminal
b. Aumento del bicarbonato intraluminal
c. Disminución de ácido carbónico en la célula parietal
d. Disminución de la actividad de la anhidrasa carbónica
e. Aumento de la alcalinidad sanguínea

2. Con respecto a la irrigación arterial del colon, a que arteria


corresponde la señalada con la flecha
a. Cólica derecha
b. Cólica media
c. Cólica izquierda
d. Ileobisecoapendículocólica
e. Arco de Riolano

3. Si un paciente presentara dentro del punto de vista fisiológico, una


disminución de enterocinasa, entonces esto originaría una
disminución de la actividad de:

Á
a. L
A. La
a pepsina
B. La
p lipasa
C. La
e quimiotripsina
D. El
p péptido insulinotropo dependiente de glucosa
s amilasa
E. La
i
n
a
b
.l
a
li
p
a
4. Con respecto a la anatomía del hígado, señale a que
estructura pertenece la marcada por el número 1.
a. Ligamento falciforme
b. Línea de Cantlie
c. Ligamento triangular
d. Ligamento coronario
e. Ligamento teres

jefe
Se valida la opción a debido a la ubicación del número 1 en
donde se unen el ligamento falciforme y ligamento coronario.

5. Se presenta un paciente, el cual presenta un antecedente de tuberculosis intestinal, por lo cual,


se le resecó 80 cm de íleon distal. Desde el punto de vista fisiológico, el paciente puede presentar
una de las siguientes alteraciones:
a. Disminución de la secreción de Vitamina B12
b. Aumento indiscriminado de absorción de ácido fólico
c. Disminución de la absorción de hierro
d. Aumento de la secreción de bicarbonato
e. Disminución de la absorción de ácido glicocólico

6. Un paciente es sometido experimentalmente a un fármaco que modifica el flujo salival, obteniéndose


un volumen de saliva de 288 ml en 6 horas. En este caso las concentraciones de electrolitos y
bicarbonato en la saliva obtenida varían de la siguiente manera:
a.↑ Na+, ↓ K+, ↑
Cl-, ↑ HCO3- b.↓
Na+, ↓ Cl-, ↑ K+, ↓
HCO3-
c.↑ Na+, ↑ Cl-, ↓ K+, ↓ HCO3-
d.↑ Na+, ↑ Cl-, ↑ K+, ↑ HCO3-
e.↓ Na+, ↓ Cl-, ↓ K+, ↓ HCO3-

Se valida la opción a debido a que se puede considerar como un aumento del flujo de saliva.

7. La siguiente imagen histológica corresponde a la glándula


…………… y la estructura señalada produce ………
a. salival sublingual / mucopolisacáridos
b. oxíntica / pepsinógeno
c. salival submaxilar / ptialina
d. salival parótida / amilasas
e. antrales / gastrina

8. Paciente varón de 65 años con antecedente de hipercolesterolemia, hipertensión arterial, fibrilación


auricular y dos infartos al miocardio previos, aqueja de dolor abdominal intenso de inicio súbito,
distensión abdominal, se decide cirugía con resección de 1,5 metros de intestino delgado terminal y
colon ascendente. Como consecuencia de la resección el paciente tendrá deficiencia de:
a. Vitamina C
b. Tiamina
c. Vitamina A
d. Vitamina B1
e. Vitamina B6

Se valida esta opción debido a que su absorción está relacionada al íleon.


9. Uno de los siguientes elementos debería hallarse con más probabilidad en el esófago de un paciente
que sufre de
reflujo gastro esofágico…
a. Pepsina
b. Tripsina
c. Quimiotripsina
d. Carboxipeptidasa
e. Ácidos biliares

10. Un paciente de 40 años cursa con anemia de 8g/dl, aqueja además de astenia y sensación de
hormigueo bilateral en los miembros inferiores, al examen se halla alteración de la sensibilidad a la
vibración y camina con ampliación de la base de sustentación. Uno de los siguientes procedimientos
sería de ayuda para el diagnóstico de este paciente:
a. Tomografía cerebral
b. Biopsia de la mucosa gástrica
c. Biopsia de hígado
d. Examen de sangre oculta en heces
e. Biopsia de Ileon proximal

11. Paciente de 60 años ingresa por caída hace 1 hora y pequeño hematoma en cuero cabelludo, al
examen físico ampliado se observa ictericia de piel y mucosas generalizada, abdomen blando, se
palpa estructura quística no dolorosa en hipocondrio derecho que corresponde a vesícula biliar
(signo de Courvoisier), en los exámenes de laboratorio se halla niveles bajos en la formación de
estercobilinógeno y urobilinógeno en heces, incremento de la bilirrubina conjugada en la orina,
elevación de fosfatasa alcalina y gamma glutamil transpeptidasa séricas. El presente cuadro puede
ser explicado por:
a. Reabsorción de hematoma
b. Litiasis vesicular
c. Carcinoma de la cabeza de páncreas
d. Carcinoma con estenosis del conducto hepático común
e. Anemia hemolítica

12. Paciente varón de 58 años con antecedente de alcoholismo crónico es diagnosticado y recibe
tratamiento por cirrosis hepática. Hace 2 días refiere familiar que tuvo cambio de conducta y no
reconoce a algunos familiares. Al examen físico, se halla ascitis, circulación colateral en abdomen,
telangiectasias, en el examen de sistema nervioso: rigidez de extremidades, ROT incrementados,
desorientación en el espacio y asterixis. ¿cuál de las siguientes circunstancias, explicaría el cuadro
en este paciente?
a. Uso de diuréticos ahorradores de potasio
b. Incremento de actividad de ureasa bacteriana duodenal
c. Hemorragia gastrointestinal
d. Disminución de la producción de NH3+ en el colon
e. Dieta normo proteica

Se valida la opción e debido al efecto sobre la encefalopatía.


Con respecto de la opción b es incorrecta debido a que hace referencia al duodeno, debería indicar colon.

13. Un recién nacido presenta vómitos biliosos poco tiempo después de cada alimento. Al preguntar a la
madre sobre antecedentes, ella recuerda que tuvo polihidramnios durante la gestación, pero un
análisis de cariotipo fue normal. Una de las siguientes es la causa más probable de estos hallazgos
en el recién nacido:
a. Enfermedad de Hirschprung
b. Fístula tráqueo esofágica
c. Divertículo ileal
d. Estenosis pilórica
e. Malrotación de la yema pancreática ventral

14. Un lobulillo hepático se puede dividir en tres zonas como se muestra en el gráfico. ¿Cuál de
las siguientes afirmaciones sobre las tres zonas es verdadera?
a. La zona 1 tiene los menores depósitos de glucógeno
b. La zona 3 es la primera en afectarse en una colestasis extra hepática
c. La zona 2 es más susceptible a la injuria por isquemia que la zona
periportal d.La zona 2 tiene la mayor capacidad de regeneración
e.La zona 1 es la que tiene menos actividad metabólica.

La pregunta 14 ha sido anulada, sin embargo, ningún estudiante se verá afectado


negativamente en su puntaje debido a esta anulación.

15. En un estudio de la secreción de hormonas gastrointestinales, sus concentraciones en la vena


porta se midieron durante perfusión luminal del intestino delgado con soluciones de diversas
magnitudes de pH. ¿Qué hormona aumentará en el plasma de la vena porta durante perfusión a
través del intestino con una solución de pH 3?
a. CCK
b. gastrina
c. GIP
d. motilina
e. secretina

16. Paciente de 30 años que ingresa a causa de un traumatismo abdominal cerrado. En la exploración
se aprecia discreta palidez de piel y mucosas, auscultación pulmonar normal, taquicardia de 120
/min. Discreta distensión abdominal y matidez en flancos; el hematocrito, que era prácticamente
normal al ingreso, disminuye a 30% a las tres horas. En la Rx de tórax se objetiva fractura de las
costillas 10-11 izquierdas. La causa más probable de la anemización en este paciente es:
a. traumatismo renal con hemorragia retroperitoneal.
b. rotura de hígado con hemoperitoneo.
c. rotura de bazo con hemoperitoneo.
d. rotura de mesos con hemoperitoneo.
e. traumatismo pancreático con pancreatitis traumática.

17. Mujer de 65 años. Consulta por síndrome constitucional


asociado a dolor abdominal epigástrico progresivo
irradiado a espalda, de dos meses de evolución. El
diagnostico de sospecha de adenocarcinoma de páncreas
se confirma por biopsia. Se realiza examen de imagen de
abdomen para evaluación de estructuras vasculares
próximas al tumor pancreático. ¿Cuál es el nombre de la
vena señalada que está ausente, trombosada por
infiltración tumoral, condicionando circulación colateral
en la pared gástrica?

a. Mesentérica superior
b. Coronaria estomaquica
c. Esplénica
d. Porta
e. Renal izquierda
18. Revisando la angiotomografía de un hombre de 70 años en estudio por aneurisma de aorta
abdominal, el radiólogo le informa de la presencia de una oclusión completa de la arteria
mesentérica inferior. El paciente se encuentra completamente asintomático. La oclusión de la
arteria mesentérica inferior cursa de manera asintomática en muchas ocasiones ya que el
territorio que irriga puede recibir flujo proveniente de la arteria:
a. cólica derecha
b. gastroduodenal
c. Epigástrica inferior izquierda
d. esplénica
e. cólica media

19. En las patologías de esófago es importante conocer bien la anatomía esofágica. ¿Cuál de
las siguientes afirmaciones es correcta?
a. El esófago tiene capa mucosa, muscular y serosa
b. El esófago abdominal es más largo que el cervical
c. El esófago torácico pasa por detrás del cayado aórtico
d. El epitelio esofágico normal es de tipo cilíndrico.
e. El esófago abdominal es discretamente más largo que el torácico

20. A pesar de que pueda haber variaciones anatómicas, lo habitual es que el ciego sea irrigado por una
rama arterial que proviene de unas de las siguientes arterias:
a. Iliaca derecha
b. Mesentérica inferior
c. Hepática derecha
d. Mesentérica superior
e. Iliaca izquierda

21. Ante un paciente con una cirugía abdominal urgente, el informe operatorio señala que se ha
realizado una resección de todo el duodeno y del tercio proximal del yeyuno manteniendo íntegros
el estómago y todo el íleon, así como los dos tercios distales del yeyuno. En el seguimiento
nutricional del paciente ¿Qué vitamina o mineral presentará con menor probabilidad una
disminución de su absorción?
a. Cianocobalamina
b. Calcio
c. Hierro
d. Transcobalamina
e. Transferrina

22. ¿Cuál de las siguientes alternativas detallan las venas que confluyen y forman la vena señalada?
a. mesentérica superior, gástrica izquierda y
gastroepiploica izquierda
b. mesentérica inferior, gástrica izquierda y renal
c. esplénica, mesentérica superior y mesentérica inferior
d. esplénica, pancreatoduodenal y omental izquierda
e. gástrica izquierda, esplénica y hepática común

23. ¿Cuál de las siguientes sustancias forma parte de la secreción biliar?


a. Tripsina
b. Lecitina
c. Elastasa
d. Quimotripsina
e. Pepsina
24. El tubo digestivo contiene diferentes tipos de epitelios y glándulas. La estructura señalada es unay
está localizada en el …………...
a. glándula de Brunner /
intestino grueso b.cripta de
Lieberkuhn / colon
c. cripta de Lieberkuhn / intestino delgado
d. glándula oxintica / estomago
e. célula parietal / estómago

Aunque las criptas de


Lieberkuhn están
presentes en el intestino
delgado, la
microfotografía es de
epitelio de colon.

25. ¿De qué musculo forma parte el ligamento inguinal?


a. Oblicuo externo del abdomen
b. Oblicuo interno del abdomen
c. Transverso del abdomen
d. Psoas
e. Dorsal ancho

26. Señale cuál de las siguientes afirmaciones NO se relaciona a la siguiente glándula anexa del
tubo digestivo mostrada en la imagen:
a. Es una glándula exocrina compuesta exclusivamente por acinos serosos
b. Su inervación está dada por el nervio auricular mayor (ramo posterior C2), que inerva
la vaina de la glándula así como la piel por encima de esta.
c. Esta glándula produce una secreción mucinosa acuosa, llamada mucoserosa, a través
del conducto de Wharton.
d. Su inflamación puede ser causada por un virus de los Paramyxoviridae, que provocan
una enfermedad muy frecuentemente en niños y adolescentes
e. Es una glándula endocrina y probablemente sea de origen pancreático

Se valida la opción e debido a que no


está relacionada con la imagen.

27. ¿Cuál de las siguientes enzimas está localizada en el borde en cepillo y juega un rol en la digestión de
proteínas?
a. Alfa dextrinasa
b. Pepsina
c. Enterocinasa
d. Lactasa
e. Carboxipeptidasa A.

Se valida la opción c debido a que es correcta en relación a la pregunta.


e. Dorsal ancho

26. Señale cuál de las siguientes afirmaciones NO se relaciona a la siguiente glándula anexa del tubo
digestivo mostrada en la imagen:
a. Es una glándula exocrina compuesta exclusivamente por acinos serosos
b. Su inervación está dada por el nervio auricular mayor (ramo posterior C2), que inerva la
vaina de la glándula así como la piel por encima de esta.
c. Esta glándula produce una secreción mucinosa acuosa, llamada mucoserosa, a través del
conducto de Wharton.
d. Su inflamación puede ser causada por un virus de los Paramyxoviridae, que provocan una
enfermedad muy frecuentemente en niños y adolescentes
e. Es una glándula endocrina y probablemente sea de origen pancreático

Se valida la opción e debido a que no está


relacionada
con la
imagen.

27. ¿Cuál de las siguientes enzimas está localizada en el borde en cepillo y juega un rol en la digestión
de proteínas? a. Alfa dextrinasa
b. Pepsina
c. Enterocinasa
d. Lactasa
e. Carboxipeptidasa A.

Se valida la opción c debido a que es correcta en relación a la pregunta.

28. Una de los siguientes sustancias, NO sirve como un buen agente emulsificante: a. Colesterol
b. Ácidos grasos
c. Sales biliares
d. Lecitina
e. Proteínas de la dieta

Se valida la opción e debido a que es correcta en relación a la pregunta.

29. La sustancia que estimula el crecimiento de la mucosa gástrica es:


a. Secretina
b. Motilina
c. Péptido estimulante de la mucosa gástrica
d. Gastrina
e. Histamina

30. ¿Cuál de las siguientes alternativas es una función de la colecistokinina?


a. Relajación de la vesícula para la salida de bilis
b. Secreción de ácidos biliares
c. Contracción del esfinter de Oddi
d. Secreción de enzimas pancreáticas
e. Contracción del duodeno

Se valida la opción b debido al efecto de la CCK sobre la vesicula biliar.

31. Con respecto a la anatomía del tronco celiaco, señale lo correcto


a. El tronco celiaco se origina de la cara posterior de la aorta abdominal
b. Es una arteria delgada que tiene un calibre entre 2 y 3 mm
c. Una de sus ramas es la arteria gástrica derecha
d. La hepática común que es una de sus ramas, participa en la irrigación del estómago.

32. Con respecto a la anatomía del duodeno, marque la respuesta correcta:


a. Tiene una distribución en forma de “C”, que rodea la cola del páncreas
b. La 3ra porción duodenal está contenida en la pinza vascular aortomesentérica
c. Entre la 1ra y 2da porción se forma un ángulo, conocido como el ángulo de Treitz
d. La 4ta porción se dirige a la izquierda, hacia abajo y hacia atrás.
e. En la tercera porción desemboca el conducto colédoco.

33. El hígado está ampliamente tapizado por peritoneo, la estructura que conecta la cara diafragmática
del hígado precisamente con el diafragma es el ligamento:
a. teres
b. falciforme
c. triangular
d. hepático común
e. coronario

Se validan la opción c y e debido a que forman parte de los ligamentos que fijan el hígado al diafragma.

34. En el íleon se absorbe aproximadamente el 95% de …………………. a través de la circulación


enterohepática. a. agua
b. colesterol
c. sales biliares
d. hidróxicobalamina
e. factor intrínseca

35. La ……………. estimula el mecanismo paracrino de la secreción de ácido clorhídrico.


a. histamina
b. acetilcolina
c. gastrina
d. secretina
e. somatostatina

36. En la digestión de proteinas, ……………. es el principal estímulo para convertir el pepsinógeno en


pepsina. a. la gastrina
b. el pH ácido
c. la acetilcolina
d. la ptialina
e. la somatostatina
37. Con respecto a la somatostatina, marque lo correcto:
a. Es secretada por las células S del intestino
b. Induce a la producción de VIP
c. Interviene en la fase intestinal de la secreción gástrica
d. Produce acetilcolina para estimular a la célula parietal
e. No interviene en la regulación de la secreción de ácido clorhídico

38. En pecten anal, es una estructura comprendida entre:


a. la línea pectínea y los senos anales
b. la línea blanca y la apertura anal
c. el esfínter anal interno y el externo
d. la línea anocutánea y la línea pectínea
e. la línea blanca y columnas anales

39. ¿Cuál de las siguientes alternativas es una proenzima pancreática?


a. Tripsina
b. Elastasa
c. Quimotripsinógeno
d. Amilasa
e. Procarboxipepitidasa C.

40. En la segmentación hepática de Coinaud, el segmento hepático señalado con la flecha, corresponde
a : En la segmentación hepática de Coinaud, la flecha señala el segmento ………….. hepático.
a. IV b. V

c. VI
d. VII
e. VIII

Parcial 2019-02

1. La contracción del músculo ………………………… permite la eliminación de gases (flatos) sin salida de
material fecal; es el mismo músculo cuya relajación, sobretodo en cuclillas, permite el paso del
contenido fecal con menor esfuerzo durante la defecación.
a) Isquirectal
b) Puborrectal
c) Esfínter anal externo
d) Esfínter anal interno
2. Paciente mujer de 54 años se presenta con náuseas, vómitos, estreñimiento, y es diagnosticada de
abdomen agudo quirúrgico; en la cirugía encuentran un vólvulo de ciego. Esta anomalía puede
explicarse por:: a) Falta de rotación intestinal
b) Falta de fusión del mesenterio
c) Defecto en la formación de la cloaca
d) Falta de formación del omento mayor

3. Paciente mujer de 23 años con faringitis aguda, toma para el dolor una tableta de paracetamol con
un poco de agua. Durante la deglución, se relaja su esfínter esofágico inferior y el fondo del
estómago, mientras el bolo está aún en el esófago. ¿Qué sustancia provocara con mayor
probabilidad la relajación del esfínter esofágico inferior y el fondo del estómago en esta mujer?
a) Óxido nítrico
b) Sustancia P
c) Histamina
d) Motilina

4. Luego de tres horas dando exámenes, un alumno de medicina comienza a sentir hambre. Esta
situación es probable que sea mediada por la ______________ que es sintetizada por el
___________________: a) leptina / intestino
b) leptina / estómago
c) grelina / estómago
d) grelina / tejido adiposo

5. Varón de 72 años, con antecedente de diabetes mellitus tipo 2, que presenta enteropatía diabética
caracterizada por estreñimiento. Este problema puede estar asociado a:
a) deficiencia de óxido nítrico
b) aumento del reflejo gastrocólico
c) disminución de la secreción de colecistocinina (CCK)
d) aumento de la secreción del péptido intestinal vasoactivo (PIV)

6. Varón de 54 años con Diabetes Mellitus tipo 2, es diagnosticado de gastroparesia debido a que
presenta sensación de llenura precoz al comer, y reflujo gastroesofágico. Esta alteración en la
relajación receptiva y en el vaciamiento gástrico lo más probable es que se deba a una alteración en:
a) el nervio vago
b) el ganglio celíaco
c) plexo submucoso
d) nervio hipogástrico

7. Varón de 67 años con tos y disminución de peso asociado a tabaquismo pesado, presenta
actualmente disfagia progresiva a alimentos sólidos. Se considera la presencia de un carcinoma de
bronquio izquierdo y por esta razón le realizan una endoscopía esofágica para descartar la
posibilidad de una compresión esofágica por el tumor. Se espera revisar el esófago en la
__________________ estrechez, que está a nivel de la vértebra ______________ a) Tercera
estrechez -T6
b) Segunda estrechez - C6
c) Segunda estrechez - T4
d) Tercera estrechez -T10

8. Varón de 34 años con dolor abdominal agudo en flanco derecho que se irradia a fosa ilíaca derecha,
es operado y se encuentra un divertículo intestinal inflamado, ubicado a 93 cm de la válvula
ileocecal. El origen de este divertículo es una falla en la obliteración de:
a) Conducto vitelino
b) Alantoides
c) Cloaca
d) Conducto anorectal
e) Uraco

En un niño menor de dos años con divertículo intestinal, este divertículo tiene su origen en una falla
en la obliteración de:
a) Conducto anorectal
b) Conducto vitelino
c) Alantoides
d) Cloaca
e) Uraco

9. Mujer de 43 años sufre un grave accidente de tránsito y está hospitalizada en coma, es alimentada
por vía intravenosa durante varias semanas. Producto de este tipo de alimentación, se encuentra en
la endoscopía atrofia de la mucosa gastrointestinal. La causa más probable de esta atrofia son los
bajos niveles séricos de la hormona: a) Colecistocinina
b) Secretina
c) Gastrina
d) PIV

10. Una mujer de 30 años llega al consultorio porque se queja de dificultades para deglutir, la cual se
agravan cada vez más. Se realiza un estudio manométrico para examinar la generación de presión a
lo largo del esófago. Esta prueba revela que las contracciones como respuesta a la deglución están
mal sincronizadas y que la presión en el esfínter esofágico inferior permanece elevada. El
diagnóstico más probable es ____________________ producido por niveles bajos de
____________________ a) acalasia / sustancia P
b) acalasia / óxido nítrico
c) enfermedad por reflujo gastrointestinal / acetilcolina
d) enfermedad por reflujo gastrointestinal / óxido nítrico

11. Paciente de 2 años, llega a emergencia por haber ingerido una moneda con la que estaba jugando.
El lugar más probable donde puede haberse quedado suspendido este objeto es a nivel del
estrechamiento producido a nivel del:
a) músculo milohiodeo
b) músculo aritenoideo
c) músculo cricofaríngeo
d) constrictor superior de la faringe

12. En una apendicectomía, al realizar la incisión de McBurney en la fosa iliaca derecha, es necesario
cortar los siguientes músculos, de afuera hacia adentro:
a) Recto – Oblicuo externo – Transverso
b) Recto – Oblicuo externo – Oblicuo interno
c) Oblicuo externo – Oblicuo interno – Recto
d) Oblicuo externo – Oblicuo interno – Transverso

13. Un varón de 90 años que se encuentra postrado en cama, es referido del asilo para endoscopia por
dificultad para deglutir luego de tomar un medicamento para aliviar el dolor la noche anterior. La
endoscopía revela que la píldora se alojó en el esófago y causó una reacción inflamatoria. Lo más
probable es que esto haya sido por la producción de múltiples ondas:
a) secundarias
b) primarias
c) lentas
d) segmentarias

14. Mujer de 23 años es diagnosticada de bulimia, al examen físico se observa ulceraciones en el


segundo y tercero dedo de la mano derecha. Esto se puede deber al uso continuo de estos dedos
para inducir el vómito, mediante la estimulación del par craneal:
a) V
b) IX
c) X
d) XI

15. Varón de 52 años se presenta por diarrea persistente de seis semanas de duración. En la
colonoscopia se observa un pólipo a nivel del íleon distal. El patólogo informa que se trata de un
tumor neuroendócrino, probablemente originado por las células enterocromafines del intestino. La
sustancia que más probablemente esté produciendo este tumor es:
a) Serotonina
b) Insulina
c) CCK
d) GIP

16. La fase oclusal de la masticación se realiza con la contracción de los músculos: a) digástricos
b) masetero y temporal
c) orbicular y buccinador
d) pterigoideo lateral y digástrico

17. Al tomar su café en Starbucks, un estudiante de medicina sufre una quemadura de primer grado en
el tercio anterior de la superficie dorsal de la lengua. La información de dolor es transmitida por el
nervio: a) cuerda del tímpano
b) glosofaríngeo
c) lingual
d) facial

18. Paciente es evaluado por faringitis aguda en consultorio externo. El médico de familia le solicita que
abra la boca y saque la lengua. Para realizar la acción de sacar la lengua, es necesario que se
contraiga el músculo: a) estilogloso
b) geniogloso
c) palatogloso
d) transverso de la lengua

19. Paciente con síndrome de Sjögren, presenta “boca seca” (disminución de


la producción de saliva) y caries dental, asociada a la pérdida de la función
de tampón de la saliva. Esta desminerilización del diente puede
comprometer a las prolongaciones citoplasmáticas ubicadas en los tubos
huecos de la estructura señalada con la letra:
a) B
b) A
c) E
d) C

20. Mujer de 32 años acude a consulta por presentar disfagia de progresión


lenta, reflujo gastroesofágico y vómitos desde hace 3 meses de evolución
progresiva. Se le realiza un estudio radiológico con contraste en el que se
observa estrechamiento del esfínter esofágico inferior (imagen). Según sus
conocimientos, este paciente se beneficiaría con el uso de: a) agonista
beta adrenérgico
b) agonista alfa adrenérgico
c) análogo de óxido nítrico
d) análogo de Sustancia P

Final 2019-02

1) La reabsorción de sodio y cloro en las glándulas salivales se da principalmente en el:


a) conducto intercalado
b) conducto excretor
c) conducto estriado
d) acino glandular

2) Paciente de 35 años con dolor en hipocondrio derecho irradiado a la punta de la escápula. Se observa
en la ecografía abdominal cálculos en la vesícula biliar. Es operado de emergencia realizando una
incisión a lo largo del reborde costal derecho. La información de dolor de esta zona es inervada por
las raíces nerviosas: a) T5 – T9
b) T6 – T7
c) T7 – T8
d) T9 – T10

3) Paciente de 23 años con dolor intenso periumbilical de inicio agudo. Es ingresado a sala de
operaciones por abdomen agudo, el cirujano encuentra sangrado a unos 60 cm proximal a la válvula
ileocecal. La estructura que está sangrando muy probablemente es un derivado embriológico del:
a) uraco
b) ligamento Hepatoduodenal
c) alantoides
d) conducto vitelino

4) Paciente de 64 años de edad con dolor en epigastrio que se distribuye en banda hacia la espalda. En
la tomografía se encuentra tumor en cabeza de páncreas que compromete a un vaso que discurre
entre la cabeza del páncreas y el proceso uncinado. Debido a esto, este cáncer se considera
irresecable porque compromete a la arteria: a) mesentérica superior
b) mesentérica inferior
c) tronco celíaco
d) aorta

5) Paciente con carcinoma gástrico avanzado, en preoperatorio para gastrectotomía total. ¿Cuál de los
siguientes ganglios linfáticos recibirá primero células metastásticas con mayor probabilidad: a)
celíaco
b) cisterna del quilo
c) esplénico
d) gástrico posterior
6) Paciente de 77 años con dolor abdominal difuso de 3 horas de evolución. En los estudios de imágenes
se observa oclusión probablemente aterosclerótica de la arteria mesentérica superior; no se observa
necrosis, lo cual puede ser explicado por la irrigación sanguínea colateral. ¿Qué vasos ofrecen
colaterales entre el tronco celíaco y la arteria mesentérica superior?
a) Gástrica izquierda y hepática
b) Cística y duodenal
c) Gastroomental derecha e izquierda
d) Pancreaticoduodenal superior e inferior

7) Paciente de 62 años con vólvulo de intestino delgado e isquemia intestinal. Se realiza laparotomía
exploratoria para liberar la obstrucción. ¿Cuál estructura se utilizará como punto de referencia para
determinar la posición de la unión duodenoyeyunal?
a) Vasos rectos
b) Ligamento suspensorio del duodeno (de Treitz)
c) Ligamento frenocólico
d) Nacimiento de la Arteria mesentérica superior

8) Paciente de 42 años con dolor abdominal intenso y hematemesis. En la endoscopía se observa una
úlcera duodenal posterior perforada con hemorragia intrabdominal. ¿Cuál de las siguientes arterias
estará comprometida? a) Gástrica izquierda
b) Mesentérica superior
c) Gástrica derecha
d) Pancreaticoduodenal posterosuperior

9) Paciente de 51 años con antecedente de enfermedad diverticular acude a emergencia por sangrado
profuso y dolor en cuadrantes inferiores. ¿Cuál es el origen más probable de la sangre que pierde en
la hemorragia? a) Cólica media
b) Mesentérica inferior
c) Rectal superior
d) Rectal inferior

10) Paciente de 13 días de vida con vómitos explosivos a las dos horas después de lactar. Al examen físico
se palpa la oliva pilórica. ¿Cuál es el nervió cuyos filetes dan inervación eferente a la estructura
afectada? a) Vago
b) Esplácnico torácico mayor
c) Mesentérico superior
d) Esplácnico torácico menor

11) Paciente de 80 años con dolor abdominal intenso y antecedente de estreñimiento crónico. En la
colonoscopía se observa divertículos con áreas ulceradas difusas en colon sigmoides y descendente.
El cirujano programa una cirugía de resección y planifica que para realizar esta resección tendrá que
cortar los siguientes vasos y nervios: a) plexo mesentérico superior y arteria rectal.
b) nervio esplácnico torácico mayor y arteria cólica media.
c) ramas del nervio vago y arteria ileocólica.
d) nervio esplácnico pélvico y artera cólica izquierda.

12) Paciente de 46 años ingresa a emergencia con dolor en cuadrante superior derecho e ictericia. En la
ecografía se observa cálculos en la vesícula biliar. ¿Cuál de los siguientes nervios transmite el dolor
de la colecistitis?
a) Filetes aferentes del nervio vago derecho, referido al ángulo inferior de la escápula
b) Filetes nerviosos de las raíces T1 a T4, con referencia al hombro izquierdo
c) Fibras aferentes simpáticas del nervio esplácnico torácico mayor, con referencia a los
dermatomas T6 a T8
d) Fibras aferentes de los ramos primarios dorsales de los nervios raquídeos T8 a T10 con
referencia al epigastrio

13) Un niño de 8 años es alimentado por sus padres con un Cheeseburguer, papitas fritas y un vaso de
Coca Cola. ¿La presencia de lípidos, carbohidratos y proteínas en el duodeno estimulan la liberación
de cuál de las siguientes hormonas en la mucosa duodenal?
a) Colecistoquinina (CCK)
b) Péptido inhibidor gástrico (GIP)
c) Secretina
d) Insulina

14) La fase cefálica del control de la secreción gástrica corresponde a cerca del 30% de la secreción ácida
y es consecuencia de un reflejo. ¿Cuál de las siguientes alternativas puede eliminar totalmente la
fase cefálica de la secreción gástrica?
a) Vaguectomía
b) Uso de atropina
c) Uso de bloqueador de receptor H2 de histamina
d) Uso de antiácidos

15) Recién nacido de 24 horas con llanto, vómitos y sin eliminación de meconio. Luego de los estudios
auxiliares se diagnostica la Enfermedad de Hirshprung, esta enfermedad se caracteriza por la
ausencia de:
a) las células de Cajal en plexo mioentérico
b) el plexo mioentérico en el recto y colon sigmoides
c) las células mioepiteliales en el recto distal
d) el plexo de Meissner en el recto distal y colon sigmoides

16) Los complejos motores migratorios aparecen aproximadamente cada 90 minutos entre las comidas,
y se considera que son estimulados por la hormona motilina. La ausencia de estos complejos
migratorios podría producir un aumento en:
a) la motilidad duodenal
b) el vaciamiento gástrico
c) la deglución
d) las bacterias intestinales

17) ¿Cuál de las siguientes es una consecuencia probable de la resección del íleon? a) Gastritis atrófica
b) Deficiencia de vitamina B12
c) Esteatorrea
d) Úlcera péptica

18) Los movimientos en masa son importantes en la fisiología intestinal. Estos movimientos en masa
ocasionan: a) la sensación de defecar
b) el peristaltismo duodenal
c) la retropulsión gástrica
d) la contracción del esfínter anal interno

19) La toxina colérica hace que aumente los niveles de AMPc intracelular, y este aumento hace que se
mantenga abierto un canal en las células de la cripta de Lieberkuhn. En condiciones fisiológicas, en
una persona sana ¿Qué sustancia puede promover que el canal quede abierto también?
a) Somatostatina
b) Óxido nítrico (NO)
c) Péptido intestinal vasoactivo (VIP)
d) Péptido similar al glucagón 1 (GLP1)

20) Paciente de 64 años con tumor abdominal que comprime la cisterna del quilo. En la biopsia de
duodeno tomada como parte del estudio, el patólogo puede observar
a) Dilatación del vaso quilífero central
b) Contracción de las venas de las vellosidades
c) Vellosidades intestinales más largas
d) Engrosamiento de la lámina basal
21) En la enfermedad de Crohn es posible encontrar células de Paneth en el colon. Esto se puede deber
a la especial función de estas células en:
a) la activación de la inmunidad adquirida
b) mantener la inmunidad innata
c) producir hormonas
d) producir Ig A

22) En 1967 se descubrió que la epidemia de Kuru, una enfermedad por priones, en el distrito de Okapa
en Papua Nueva Guinea, era causada por la costumbre de comer la carne de los muertos. Ahora se
sabe que las proteínas priónicas ingresan al organismo a través de:
a) los enterocitos
b) la transmigración
c) las células de Paneth
d) las células M

23) Paciente de 48 años con alteraciones en el tránsito intestinal por diabetes mellitus tipo 2; se presenta
con esteatorrea, flatulencia y malabsorción de grasas. Las pruebas de función hepática y biliar están
dentro de rangos normales. Una causa de la disminución de sales biliares puede ser:
a) el sobrecrecimiento bacteriano
b) la deficiencia de pepsina
c) la deficiencia de elastasa
d) la hiperestimulación del GLUT5
24) Al usar azúcar de mesa (sacarosa) para endulzar su café, el estudiante de medicina sabe que lo más
probable es que para su absorción tendrá utilizar el/los transportadore(s) ________________ que se
encuentran en la membrana apical de los enterocitos.
a) SGLT-1
b) GLUT2 y GLUT5
c) SGLT-1 y GLUT5
d) SGLT-1 y GLUT2

25) Una persona con la producción normal de lactasa; cada vez que toma leche, los productos de la
degradación de la lactosa por parte de la lactasa ingresarán al enterocito usando el/los
transportador(es) _____________: a) SGLT-1
b) GLUT2 y GLUT5
c) SGLT-1 y GLUT5
d) SGLT1 y GLUT2

26) En una persona sana, luego de una comida basada en carnes rojas cocinadas con baja cantidad de
sal, al enterocito pueden ingresar solamente:
a) aminoácidos
b) aminoácidos y dipéptidos
c) aminoácidos, dipéptidos y tripéptidos
d) aminoácidos, dipéptidos, tripéptidos y tetrapéptidos

27) Paciente con deficiencia congénita de procolipasa, sufre de esteatorrea cada vez que come comidas
ricas en grasas. En el estudio de composición de las heces, uno espera encontrar una cantidad
aumentada de: a) colesterol
b) triglicéridos
c) fosfolípidos
d) lisolecitina

28) Al tomar su café en Starbucks, un estudiante de medicina sufre una quemadura de primer grado en
el tercio anterior de la superficie dorsal de la lengua. La información de dolor es transmitida por el
nervio: a) cuerda del tímpano
b) glosofaríngeo
c) lingual
d) facial
29) Paciente de 32 años con esteatosis hepática no alcohólica. Se le realiza una biopsia hepática que
confirma la esteatosis; en el tejido se observan depósitos de lípidos en los hepatocitos, los cuales
contienen principalmente: a) triglicéridos
b) colesterol
c) ácidos grasos libre
d) acil-carnitina

30) Paciente de 21 años que es estudiante de medicina, llega a consulta refiriendo que en épocas de
exámenes su piel se vuelve amarillenta. Le realizan el diagnóstico de Síndrome de Gilbert asociado a
mutación del gen UGT1A1, luego de unas semanas acude a la consulta por ictericia asociada a resfrío.
Al hacerle un análisis de sangre, usted sabe que encontrará valores elevados de:
a) bilirrubina directa
b) bilirrubina indirecta
c) bilirrubina tipo delta
d) fosfatasa alcalina
31) Se realizó un experimento en el cual se inyectó tinta china en el peritoneo de ratas de laboratorio. Al
realizarse una biopsia hepática de dichos animales, se encontró que el tinte negro estaba depositado
en: a) las células de Ito
b) los hepatocitos
c) las células de Kupffer
d) las células de Disse

32) Paciente de 74 años de edad con shock hipovolémico asociado a deshidratación aguda severa. En
este paciente es posible encontrar hipoxia en zona …… del lobulillo hepático e infarto ……………… del
intestino. a) 1 / mucoso
b) 1 / transmural
c) 3 / transmural
d) 3 / mucoso

33) Niño de 5 años con historia de tres días de evolución caracterizado por fiebre, malestar general,
odinofagia, hiporexia, e irritabilidad. Al examen se observa lesiones ulcerativas de 4 mm de diámetro
en mucosa yugal, con fondo blanquecino y eritema periférico. El diagnóstico más probable es:
a) candidiasis oral
b) leucoplasia
c) aftas
d) herpes

34) Paciente de 52 años con enfermedad por reflujo gastroesofágico de 30 años de evolución. Se realiza
endoscopía en la que se encuentra mucosa eritematosa proximal a la línea Z. Para corroborar la
presencia de lesión preneoplásica, se toma una biopsia de esa zona, en la que se espera encontrar:
a) metaplasia gástrica
b) metaplasia intestinal
c) displasia gástrica
d) adenocarcinoma

35) Un estudiante de medicina termina su último examen final. Al abrir la boca para comer una
hamburguesa con papas fritas, la activación de los receptores muscarínicos de las células acinar y
ductal estimularan un mayor flujo de saliva, con lo cual disminuirá la concentración salival de:
a) potasio
b) bicarbonato
c) sodio
d) cloro
36) Un estudiante toma su desayuno consistente en un pan con mantequilla y queso. Antes que se
puedan digerir las grasas, es necesario que sean emulsificadas. La hormona ………………………. estimula
la liberación de las sustancias emulsificadoras.
a) colecistoquinina (CCK)
b) secretina
c) lipasa pancreática
d) gastrina

37) Al comer un pollo a la brasa entero, con papitas fritas y ensalada, la sustancia que estimulará la
liberación de HCl en el estómago es:
a) el neuropeptido Y
b) la secretina
c) la bombesina
d) la colecistoquinina (CCK)
38) Paciente con disminución marcada del apetito asociado a cáncer terminal, se podría utilizar análogos
de ……………..
para promover la ingesta de alimentos.
a) el péptido similar al glucagón (GLP)
b) la serotonina
c) la secretina
d) la endorfina

39) Un hombre de 51 años presenta de forma súbita vómitos masivos de sangre roja brillante. Tiene
como antecedente hepatitis viral B hace 23 años. En la exploración física: FC 103 latidos/min, PA
85/50 mmHg, se palapa la punta del bazo y niega vómitos. Su hematocrito es 21%, la prueba
serológica de HBsAg es positiva. En la ecografía se observa hígado con nodulaciones. ¿Cuál es la causa
más probable para la hematemesis?
a) Esófago de Barrett
b) Síndrome de Mallory Weiss
c) Varices esofágicas
d) Esofagitis por reflujo

40) Durante el reflejo del vómito, uno de los primeros sucesos es:
a) el peristaltismo inverso
b) la contracción del píloro
c) la apertura de la glotis
d) el esfínter esofágico inferior abierto

41) Paciente de 34 años que acude por diarrea desde hace 4 días asociado a comer papa rellena con ají
en el Estadio Nacional, se exacerba cuando toma lácteos o come grasas. Se acompaña de flatulencia
y episodios de tenesmo. Este cuadro de diarrea e intolerancia a la lactosa de inicio agudo se explica
por la:
a) enfermedad de Crohn
b) infección por V. cholera
c) colitis ulcerativa
d) giardiasis

42) En cuanto a la secreción pancreática, mientras mayor es el flujo, mayor es la concentración de: a)
potasio
b) bicarbonato
c) cloro
d) sodio

43) Paciente de 10 años con diarrea crónica, distensión abdominal, anorexia. Se le ha encontrado
anticuerpos antigliadina y antiendomisio. Es más probable que la diarrea se correlacione con el
hallazgo histológico de: a) adelgazamiento de las criptas
b) linfocitos intramusculares
c) atrofia de las vellosidades
d) úlceras duodenales

44) La glándula parótida está inervada por el par craneal:


a) VII
b) IX
c) X
d) XII
45) Las glándulas de Brunner se encuentran en la:
a) mucosa del esófago
b) submucosa del íleon
c) mucosa del yeyuno
d) submucosa del duodeno

46) ¿Cuál de las siguientes afirmaciones describe correctamente la función de la inervación


parasimpática del tracto gastrointestinal?
a) La norepinefrina es el principal neurotransmisor excitatorio.
b) La actividad parasimpática produce la relajación de los esfínteres
c) La actividad parasimpática excesiva puede provocar un trastorno llamado ileo paralítico
(parálisis del músculo liso intestinal)
d) El pH luminal, la osmolaridad y la distensión muscular son detectados por fibras
parasimpáticas eferentes

47) Al ingerir una cantidad de glucosa por vía oral, esta es interiorizada en las células del organismo más
rápido que si esa misma cantidad de glucosa hubiese sido administrada por vía endovenosa. Este
fenómeno sucede gracias a la acción de la sustancia secretada por las células:
a) G
b) I
c) K
d) S

48) A …………… secreción de HCl en el lumen gástrico, …………….. pH en la sangre venosa gástrica a) mayor
/mayor
b) mayor / menor
c) menor / mayor
d) mayor / igual

49) ¿En cuál de las siguientes situaciones hay un menor flujo de secreción salival?
a) Masticar goma de mascar
b) Imaginarse ser sometido a un examen dental
c) Exposición a olor nauseabundo
d) Sueño
50) Al seccionar el nervio facial a nivel timpánico, usted esperaría:
A. Disminución del gusto en la punta de la lengua
B. Imposibilidad para el cierre del istmo de las fauces
C. Ausencia de la termoalgesia de la lengua
D. Imposibilidad para protruir la lengua
51) En un paciente con hiperestimulación simpática se espera que las ondas lentas tengan un ritmo:
A. Menor en íleon terminal que en el duodeno
B. Mayor en el íleon que el duodeno
C. Mayor en el estómago que el duodeno
D. Mayor en el estómago que en íleon terminal
52) Marque lo correcto en relación al divertículo de Meckel.
A. Se encuentra usualmente a 60 cm de la VIC
B. Contiene mucosa esofágica en algunas ocasiones
C. Se produce en el lado del íleon
D. Se relaciona un defecto en el desarrollo del intestino posterior
53) Al disminuir el pH duodenal por el HCl gástrico, se libera principalmente una hormona cuya célula
diana es :
A. Células S del intestino
B. Células ductales del colédoco
C. Acino pancreáticos
D. Células ductal del Wirsung
54) El ecografista sabe que para poder visualizar el nacimiento de la arteria mesentérica superior, debe
colocar el transductor sobre la piel de la siguiente región abdominal:
A. Epigastrio
B. Hipogastrio
C. Hipocondrio derecho
D. Mesogastrio
55) Durante una cirugía oncológica, el cirujano observa que los órganos abdominales tienen libre
movimiento dentro de la cavidad abdominal, excepto:
A. Vesícula biliar
B. Yeyuno
C. Estómago
D. Colon ascendente
56) Paciente obeso con Covid-19 es intubado por interno inexperto, quien al solicitar que bombeen aire
dentro del tubo endotraqueal, nota que el epigastrio se ditiende. Al sospecha que ha introducido el
tubo en el estómago, también es cierto que:
A. Aumenta la frecuencia de ondas lentas
B. Disminuye el pH gástrico
C. Disminuiría el tono del píloro
D. Aumenta el pH gástrico
57) Paciente con apendicitis aguda, que debuta con dolor en mesogastrio. ese dolor se debea
estimulación de receptores del dolor cuyas fibras van a viajar a la médula espinal a través de:
A. Nervios simpáticos
B. Plexo hipogástrico
C. Nervio esplácnico pélvico
D. Nervio vago
58) La rotación en sentido longitudinal del estómago en el desarrollo embriológico condiciona que el
nervio vago derecho quede a nivel:
A. Izquierdo
B. Oblicuo
C. Anterior
D. Posterior
59) Una de las siguientes sustancias reguladoras, puede actuar de forma paracrina y como hormona.
Marque la correcta:
A. Péptido insulinotrópico dependiente de glucosa
B. GRP
C. Acetilcolina
D. Somatostatina
60) El alcohol y la cafeína estimulan la fase ______________ de la producción de ácido clorhídrico. a)
cefálica
b) gástrica
c) intestinal
d) gástrica y cefálica
ECU 1
Estudiante de medicina de la UPC de 21 años sufre de gastritis aguda ocasionada por comer
en lugares poco higiénicos. Suele consumir caramelos ( chupar ) mientras está en clase hasta
la tarde. Toma gaseosas regularmente (carbohidratos 46%, sodio 53%). También toma
regular cantidad de leche (grasa 35%, lactosa 35%, proteínas 30%), pues le calma un poco el
dolor el ardor que siente por la gastritis. Incluso, cuando puede, se toma dos vasos de agua
fría para calmar las molestias. Ha decidido ir al médico para tratarse pues ya no soporta el
dolor, el cual está seguro que los síntomas se deben a una elevada producción de ácido
clorhídrico en el estómago, y por ello le ha recetado Ranitidina (antihistamínico), con lo que
siente mejoría.

El uso de atropina en este paciente:


- Aumentará el pH del estómago

Entre las sustancias cerebrales que producen ansiedad está la serotonina, la cual también
tiene acción:
- Anorexigénica

El consumo de dos vasos de agua seguidos agua generará indirectamente un aumento en la


liberación de:
- Ácido clorhídrico

En este paciente con gastritis aguda debida a una alta producción de ácido clorhídrico, sería
lógico esperar que el píloro tenga un tono muscular:
- Aumentado

El consumo de leche produce directamente un aumento de los niveles séricos de la


hormona:
- Colecistoquinina (CCK)

El consumo de leche produce directamente un aumento de los niveles séricos de la


hormona:
- Gastrina

ECU 2
Niño de sexo masculino de 2 años de edad, sufre de estreñimiento desde el nacimiento (1
deposición cada 3-4 días). Madre menciona que le estimula la defecación con un
termómetro rectal, y continuo uso de enemas y laxantes. Desde hace 6 meses comienza con
vómitos postprandiales. Los síntomas aumentan en frecuencia y magnitud y están en
relación con los episodios de estreñimiento. No refiere fiebre, tos, diarrea ni lesiones
cutáneas. Al examen físico presenta regular estado general, luce deshidratado. Abdomen
distendido, blando, depresible e indoloro. No se palpan masas abdominales. Se permeabiliza
el canal anal con termómetro rectal, encontrando cierta resistencia. Salida de material fecal
mal oliente en regular cantidad. Exámenes de laboratorio: hemograma normal. Signos
inflamatorios de fase aguda negativos. Alcalosis metabólica leve en sangre venosa.
Radiografía con enema baritado muestra recto y colon sigmoides dilatados (megacolon).
Biopsia profunda: ausencia de células ganglionares en la muestra enviada. Se realiza cirugía
correctiva.

El contenido fecal se detiene en la zona inmediatamente proximal a la zona donde hay una
menor presencia de:
- Péptido intestinal vasoactivo

En cuanto a los reflejos gastrocólico y gastroduodenal en este paciente, indique lo correcto:


- Se pueden considerar reflejos vago-vagales

En este paciente se considera que está abolido el reflejo:


- Rectoesfinteriano

Debido al acúmulo de material fecal en todo el marco colónico, y a la irritación química


asociada, el peristaltismo del íleon distal se debe encontrar:
- Inhibido

Es un reflejo propio de la pared intestinal:


- Peristaltismo

A diferencia de las arcadas, los vómitos presentan apertura de:


- Esfínter esofágico superior

CI 1
Sustancia que inhibe la secreción y la motilidad del estómago prolongando el tiempo de
digestión:
- Péptido insulinotrópico dependiente de la glucosa (GIP)

Marque lo correcto:
- La hernia fisiológica se produce en la sexta semana y es la salida temporal de
asas intestinales a través del cordón umbilical

Marque la respuesta correcta en relación a la gastrina:


- Las células G son las productoras y se encuentran principalmente en el antro
gástrico
El consumir caramelos indirectamente activa la vía:
- POMC/CART

¿En qué capa se encuentra la alteración principal en el Hirschsprung o megacolon


agangliónico?:
- Muscular propia

Con respecto a las ondas lentas, marque la afirmación correcta:


- Son contracciones rítmicas espontáneas

El uso de Ranitidina bloquea el receptor H2 de la histamina en las células parietales. La


histamina llega a estas células por:
- Difusión

La triada sintomática: vómitos explosivos post-prandiales, movimientos peristálticos


epigástricos visibles de izquierda a derecha y nódulo palpable epigástrico subcostal derecho,
pertenecen a:
- Estenosis congénita hipertrófica del píloro.

Durante una cirugía oncológica, ¿la extirpación de cuál de los siguientes órganos se vería
comprometida por la presencia de adventicia?:
- Recto

En cuanto a los reflejos gastrointestinales, un reflejo que estimula el tránsito intestinal es el


reflejo:
- Gastrocólico

El ligamento falciforme divide al hígado en dos lóbulos derecho e izquierdo.


Embriológicamente deriva del:
- Mesenterio ventral

La presencia de atresias y estenosis duodenales se deben básicamente a una:


- Falta de recanalización

Estudiante de medicina de 20 años, se ha amanecido estudiando para su examen de Sistema


Digestivo. No ha probado alimento desde la cena, por lo que se puede afirmar que la
motilidad de esta persona está siendo regulada por:
- Motilina

Paciente con disminución del apetito marcada asociada a cáncer terminal, para promover la
ingesta de alimentos se podría usar análogos de:
- Endorfinas

Las ondas lentas se producen por la apertura cíclica de canales de:


- Calcio

La forma más común de atresia esofágica contiene:


- Estenosis proximal del esófago más fístula traqueoesofágica distal

Al deglutir un bolo alimenticio, es lógico suponer que al pasar por el esófago haya un mayor
consumo de oxígeno en la pared del tercio:
- Proximal

Paciente que come entera una pizza familiar de chorizo y queso. Es posible esperar que
debido a la cantidad de alimento ingerida, las ondas lentas hayan:
- Sufrido ninguna alteración en su frecuencia

La hernia fisiológica se produce dentro de:


- Cordón umbilical

El crecimiento de un adenocarcinoma de páncreas compromete la pared gástrica por


contigüidad. ¿Qué parte del estómago se esperaría esté comprometido?
- Pared posterior del antro

CI 2
Estimula la producción de saliva:
- Vasodilatación periglandular

Durante la secreción de saliva, es de esperarse que las concentraciones de ________ y


______ disminuyan al disminuir el flujo:
- Sodio Bicarbonato

Con respecto a la secreción gástrica de HCl:


- a mayor secreción de HCl en el lumen gástrico, mayor pH en la sangre venosa
gástrica

Respecto a las enfermedades del esófago, marque lo correcto:


- el diagnóstico diferencial de la acalasia es la enfermedad de Chagas esofágica

Con respecto a las lesiones y enfermedades de la boca, marque lo correcto:


- la eritroplasia debe ser biopsiada
Respecto a las glándulas salivales, marque lo incorrecto:
- la glándula sublingual tiene forma de garfio

El omeprazol actúa sobre la membrana _____________ de la célula ____________


- apical / parietal

Durante el sueño, la concentración de bicarbonato en la saliva:


- Disminuye

Durante el ataque con gas sarín (bloqueador de la acetilcolinesterasa) en el metro de Tokio,


en 1995, el personal de salud notó que los pacientes afectados presentaban:
- Hipersalivación

La célula mucosa del cuello gástrico produce:


- Moco

ECU 3
El caso:
Paciente de 54 años con antecedentes de alcoholismo, gastritis crónica, tabaquismo pesado,
obesidad, cálculos biliares y cirrosis, es llevado a la emergencia por dolor abdominal en
epigastrio irradiado a la espalda y trastorno del sensorio.
Al examen físico: presión arterial 85/50 mmHg, frecuencia cardíaca 100 latidos/min,
frecuencia respiratoria 18 x minuto, temperatura axilar 36°C.
Conjuntivas pálidas, escleras ictéricas nevus arácnidos en tronco, distensión abdominal
marcada, cabeza de medusa, matidez desplazable en ambos flancos e hipogastrio, dolor a la
palpación de abdomen.
Tiempo de protrombina: 24 seg (testigo: 13 seg); TPT: 38 seg, glicemia: 165 mg/dL, uremia:
20 mg/dL, ASAT: 76 UI/L, ALAT: 22 UI/L, albumina: 2,5 g/dL, bilirrubina total: 2,6 mg/dL,
bilirrubina directa: 1,4 mg/dL, amilasa sérica 4000 U/L.

En todas las preguntas marque la mejor respuesta.


De las siguientes sustancias secretadas por los órganos de este paciente, la más alcalina es la
secreción:
- Pancreática

En esta paciente, el aumento de la Amilasa sérica se debe directamente a una lesión de


- Páncreas

En cuanto a la gastritis de este paciente, se encontró que era producida por la bacteria
Helicobacter pylori. Esta bacteria sobrevive en el medio ácido del estómago gracias a:
- Ureasa
Para evaluar la función hepática podemos medir los niveles séricos de:
- Tiempo de protrombina

El misoprostol, análogo de las prostaglandinas, está mejor indicado en:


- Prevenir daño por AINES

El paciente llegó con trastorno del sensorio posiblemente debida a encefalopatía hepática
producida por déficit en el metabolismo del/la:
- Amoniaco

CI 3
En cuanto al urobilinógeno, se puede decir que es cierto:
- Recircula hacia el hígado

Marque lo correcto:
- Las venas sublobulillares desembocan en las venas hepáticas

En cuanto a la secreción de iones y agua en los conductos biliares es correcto lo siguiente:


- Es estimulada por la secretina

Paciente con tumor neuroendocrino productor de secretina, debido a lo cual se puede


esperar que su secreción pancreática, comparada con la de una persona sana en estado de
bajo flujo, tenga una concentración de:
- bicarbonato aumentada

El efecto colerético de las sales biliares se refiere a:


- Estimulo de secreción biliar

Paciente de 42 años con adenocarcinoma ductular. La TC ha demostrado claramente que el


tumor está en el cuello del páncreas y que hay un gran vaso ocluido. ¿Cuál de los siguientes
vasos estaría más probablemente obstruido?
- Vena porta

Las ramas más pequeñas del árbol biliar son:


- Canalículos biliares

La secreción de agua y bicarbonato por el páncreas exocrino se da básicamente en la fase:


- Intestinal
Cuando el alimento se encuentra en el estómago, se produce la liberación de enzimas
pancreáticas básicamente debido a la acción de:
- Vago

La bilirrubina directa aumentada en cirrosis hepática se excreta en la orina debido a:


- Ser hidrosoluble

ECU 4
Caso:
Mujer de 83 años acude a emergencia por dolor abdominal desde hace 4 días, localizado en
epigastrio, irradiado a ambos hipocondrios, nauseas, vómitos y distensión abdominal; tiene
antecedente de cardiopatía hipertensiva, diabetes mellitus tipo II y fibrilación auricular.
Refiere deposiciones diarreicas muco sanguinolentas hace 1 día. Los exámenes iniciales
muestran PA: 110/60 mmHg, FC: 110/mn, leucocitos: 17800, neutrófilos de 93%. TAC
abdómino pélvica se observa oclusión completa de arteria mesentérica superior por trombo
asociado a placa ateromatosa.

Considerando que se ha comprometido el íleon distal, entre otras áreas, la atrofia o


descamación del epitelio de superficie explicaría cuál de los signos o síntomas de la paciente
(marque la mejor respuesta):
- Diarreas mucosanguinolentas

Después de remover quirúrgicamente el territorio gastrointestinal afectado, estaremos


seguros que no quedará en la paciente la siguiente estructura:
- Placas de Peyer

El mecanismo de la diarrea muco sanguinolenta que presenta la paciente, puede mejor


definido como de tipo (marque la mejor respuesta):
- Exudativa

Durante la cirugía, el cirujano observó que además la paciente tenía divertículos en el sigma.
Se sabe que estos divertículos:
- Se pueden asociar a estreñimiento crónico

En esta paciente, ¿cuál de las siguientes sustancias no tendrá una considerable disminución
en su absorción? (marque la mejor respuesta):
- Calcio

PARCIAL
Al evaluar la orofaringe de un paciente, el médico le solicita que abra la boca, saque la
lengua y diga a . Al hacer esta maniobra, nota que el paladar se desvía hacia la derecha, lo
cual le hace sospechar que el paciente sufre de una lesión del nervio craneal:
- X contralateral

Un bolo alimenticio grande y poco masticado se atasca en el esófago, esto ocasiona una
sensación de dolor que es transmitida por los nervios:
- esplácnicos

Para realizar el movimiento mecánico de abrir la boca, primero se necesita:


- fijar el hueso hioides

¿Cuál de las siguientes alternativas se define como la protrusión directa del contenido
abdominal a la cavidad amniótica por un defecto de la pared corporal?
- Gastrosquisis

Un paciente requiere que se le coloque una sonda de alimentación directamente al


estómago (gastrostomía), el cirujano deberá hacer una incisión en la piel del abdomen ¿cuál
de las siguientes raíces nerviosas debe ser anestesiada para este procedimiento?
- T8

En un paciente de 43 años con tumor carcinoide de páncreas productor de gastrina


(Síndrome de Zollinger-Ellison) se puede esperar encontrar una potenciación del reflejo:
- gastrocólico

El mecanismo de la defecación incluye la participación de diversas estructuras ¿Cuál de las


siguientes alternativas es correcta?
- Puede ser mediado por un reflejo intrínseco

Cuando el contenido del estómago ingresa al duodeno, uno de los reflejos que inhiben el
vaciamiento gástrico es a través del:
- sistema nervioso mientérico

Durante la masticación, gran parte del proceso masticatorio se debe a:


- el reflejo masticatorio

Las glándulas salivales tienes conductos para la excreción de la saliva; las glándulas
____________ drenan en las carúnculas sublinguales.
- sublinguales
Los diferentes segmentos del tubo digestivo son susceptibles de reflejos y movimientos
según su contenido. Si colocáramos mediante una sonda un bolo alimenticio directamente
en el tercio medio del esófago:
- se producirá ondas secundarias

En una persona si enfermedad se espera que el tránsito intestinal se vea disminuido cuando
se presenta el reflejo:
- doloroso

El divertículo de Meckel es una anomalía congénita que ocurre por la persistencia del
conducto vitelino y da origen a una estructura sacular, el cual se encuentra en el:
- borde antimesentérico

Si al intubar a un paciente, por error se ingresa el tubo endotraqueal en el esófago y se


insufla el manguito endotraqueal (globo TET), la dilatación de este manguito generará:
- múltiples ondas secundarias

El orificio omental, o hiato de Winslow, se encuentra limitado por el ligamento:


- hepatoduodenal

Paciente de 24 años acude a consulta externa por presentar una fístula oronasal
(comunicación entre la cavidad oral y la cavidad nasal). Está fístula es una consecuencia
tardía de la lesión de un vaso sanguíneo por el antecedente de haber sido operado de
paladar hendido en los primeros años de vida, aparentemente en una campaña gratuita de
corrección de paladar fisurado. ¿Cuál de las arterias palatinas podría haberse lesionado
durante esa cirugía?
- Mayor

Dentro de las anomalías congénitas se puede presentar un tejido pancreático accesorio


¿Cuál es la ubicación más común de este tejido?
- Estómago

Paciente con insuficiencia mitral moderada a severa, con aumento de volumen de la


aurícula izquierda; esta condición tendrá como consecuencia a nivel del sistema digestivo:
- la disfagia a sólidos

El inicio de la fase faríngea de la deglución se debe a estímulos sensitivos que viajan por el
nervio craneal:
- V

El mesocolon transverso se origina en:


- la pared posterior del abdomen

En la digestión de los alimentos, la hormona __________ se libera frente a la presencia de


péptidos y monoglicéridos, y tiene un efecto marcado en la disminución del vaciamiento
gástrico.
- colecistoquinina

En un varón de 47 años con sección medular a nivel de T6 debido a un accidente


automovilístico, sus terapeutas han desarrollado un mecanismo para distender el recto e
iniciar el reflejo rectoesfinteriano, lo cual producirá la contracción de:
- la pared del recto

Los catadores de vino tienen una habilidad increíble al momento de separar los sabores.
Este aumento de la sensibilidad gustativa debido a una mayor cantidad de papilas linguales
y de corpúsculos gustativos se conoce como:
- hipergeusia

Una recién nacida es evaluada por el neonatólogo y evidencia que el canal anal está
completamente cerrado. Este problema se debe probablemente a una anomalía en el
desarrollo de:
- la membrana cloacal

Durante un experimento, se insufla rápidamente dos litros de agua en un globo colocado


dentro del estómago de un voluntario. ¿cuál de las siguientes situaciones del músculo liso
será consecuencia directa de este cambio de volumen en el estómago?
- Despolarización

En muchos países se usa el suplemento de fluor en el agua potable o los dentríficos, con el
fin de hacer el esmalte más resistente a la desmineralización inducida directamente por:
- el ácido

La sensación del gusto depende de la presencia de papilas gustativas en la lengua, las cuales
tienen corpúsculos gustativos conteniendo células neuroepiteliales sensoriales. Estas células
neuroepiteliales pueden ser dañadas fácilmente, por suerte, su tiempo de recambio es de
alrededor de:
- 10 días

Paciente de 56 años con accidente cerebrovascular reciente. En la resonancia se observa


daño de los núcleos laterales del hipotálamo. Por este motivo es muy probable que el
paciente sufra de:
- Inanición
Durante el desarrollo de la región cloacal, una cuña de mesodermo ubicado entre el
alantoides y el intestino posterior vendrá a formar el:
- tabique urorrectal

En el conducto anal se encuentra la unión entre las regiones del endodermo y el ectodermo,
esta unión se evidencia al observar:
- la línea pectínea

En una persona sana, el momento adecuado para encontrar los mayores niveles de grelina
en sangre sería:
- antes de come

Dentro de la estructura de los dientes, la parte del diente cubierta por esmalte y que se
puede ver mediante la inspección visual de la boca se denomina
- corona clínica

El esófago en su microestructura tiene básicamente adventicia, a excepción de la región


distal, donde tiene serosa, específicamente a partir del nivel de:
- T10

En un paciente con arcadas, se debe considerar que durante la ocurrencia de dichas arcadas,
debemos encontrar contenido gástrico en:
- tórax

El duodeno está constituido por el segmento terminal del intestino anterior y el segmento
proximal del intestino medio ¿Cuál de las siguientes alternativas describe mejor este lugar
de unión entre los dos intestinos?
- Distal al origen de la yema hepática

En una persona sana, el uso de atropina producirá a nivel del estómago:


- Aumentará el pH del estómago

En una persona sana, el consumo de leche produce indirectamente


- Inhibición del vaciamiento gástrico

La motilidad del colon es importante y lenta comparada con la del intestino delgado. Los
movimientos en masa ocasionan la:
- distensión rectal
Con respecto a la motilidad gástrica, los potenciales de acción disminuyen en frecuencia por
efecto de:
- el péptido insulinotrópico dependiente de glucosa

La sensación del gusto depende de la presencia de papilas gustativas en la lengua, de las


cuales, algunas de ellas tienen un surco terminal por donde drenan unas glándulas salivales
linguales (llamadas glándulas de von Ebner). Esta descripción se refiere a las papilas:
- circunvaladas
1. Un niño de 2 años es llevado a la consulta por diarrea persistente y edema de
las extremidades, además falta de crecimiento y desarrollo en relación a su
edad. Los análisis de sangre revelan que tiene concentración plasmática baja de
proteínas (hipoproteinemia). Durante la endoscopía duodenal, se coloca
colecistocinina (CCK) endovenosa y se recoge muestras del líquido duodenal; el
resultado del líquido confirma incapacidad para hidrolizar proteínas a un pH
neutro, esta situación mejora al añadir una pequeña cantidad de tripsina. El
paciente probablemente esté sufriendo la falta congénita de
-Enterocinasa
2. Experimentalmente se incrementa la velocidad de la secreción salival con
una sustancia, el análisis de la composición de esta saliva obtenida se
espera encontrar…………..
-Disminución de concentración de potasio
3. Paciente varón de 46 años soltero, consulta por odinofagia y bajo de peso, tiene
antecedente de tuberculosis desde hace 3 meses y es fumador crónico (10
cigarrillos por día); al evaluar la cavidad oral se identifica lesión blanquecina en
el dorso de la lengua y paladar blando, las lesiones se desprenden con el baja
lengua dejando una base eritematosa. Esta lesión corresponde probablemente
a
……………………….
…..
-Candidiasis oral
4. Paciente mujer de 35 años acude a consulta por sensación de sequedad y
lesiones en cavidad oral. Al examen se observa atrofia de la mucosa, fisuras y
úlceras; nota además sequedad e irritación de la córnea y aumento del tamaño
de las glándulas parotídeas. Su diagnóstico más probable es artritis reumatoide;
el hallazgo más probable en una biopsia de glándula parótida es……..….
-Gran infiltración de linfocitos y células plasmáticas
5. Un paciente con anemia acude con su médico quejándose de episodios
frecuentes de gastroenteritis. Un análisis de sangre revela anticuerpos
circulantes dirigidos contra células parietales gástricas. Su anemia es atribuible
a la hiposecreción de
-Factor intrínseco
6. Dos estudiantes deciden tomar un receso para comer una hamburguesa a la
hora del almuerzo. Antes de llegar a la cafetería, impulsos nerviosos
provenientes del complejo vagal dorsal iniciarán la secreción de ácido gástrico
por la liberación de
…………………….. desde el sistema nervioso entérico.
-GRP
7. Un niño de cuatro años de edad es llevado a la consulta por cuadros diarreicos
frecuentes caracterizados por heces pálidas, voluminosas y fétidas, presenta
bajo peso y talla. Se mide la concentración de cloruro en el sudor y se encuentra
que sus valores son muy elevados. La alteración más importante a nivel de
células ductales del páncreas tiene relación directa con la conductancia
de…………
-Cloro
8. Una mujer de 50 años de edad que sufrió durante varios años resequedad de
los ojos debida a producción inadecuada de lágrimas es enviada con un
gastroenterólogo para evaluación de pirosis crónica. El examen endoscópico
revela erosiones y tejido cicatrizal en la parte distal del esófago justo por
arriba del esfínter esofágico inferior. Las lesiones pueden atribuirse a la
disminución de uno de los siguientes componentes salivales:
-Bicarbonato

9. Se evalúa los valores séricos de las siguientes sustancias a un paciente con


enfermedad hepática terminal; en este paciente se espera encontrar la
combinación con la letra …………
-disminuida, aumentada, disminuida
10. Una mujer de 35 años de edad HIV positiva, se presenta al médico con dolor
abdominal en cuadrante superior derecho e ictericia. La paciente refiere haber
tenido múltiples episodios de ictericia durante los últimos 10 años. Los exámenes
para determinar hepatitis viral, dieron positivos para Hepatitis B, siendo
catalogado el caso como hepatitis crónica con alteración funcional. En un
examen de sangre ¿cuál de los siguientes parámetros está disminuido?
-Albúmina
11. En el reflejo peristáltico del intestino delgado, uno de los siguientes eventos
sucede en la porción oral del bolo alimenticio…………...
-Acción de acetilcolina en el músculo circular
12. Experimentalmente se coloca una dosis alta de secretina en la luz
intestinal duodenal; como consecuencia de esto, en el jugo pancreático de
la misma luz intestinal se observa la disminución de la concentración de
…..………..
-Cl
13. Un varón de 58 años de edad con enfermedad de Crohn severo fue sometido a
una resección ileal. Después de la cirugía este paciente padecerá de esteatorrea,
esto se explica porque …..………..
- La micelas no pueden formarse
14. En un experimento se inserta un balón en el estómago de un voluntario, se infla
poco a poco mientras que se vigilan las presiones intraluminales. Aunque el
volumen del balón aumenta considerablemente, las presiones permanecen
constantes. Esta relación volumen-presión se explica por la liberación local de
…………..
-Óxido nítrico y péptido inhibidor vasoactivo
15. La toxina de Vibrio cholerae causa diarrea debido a…….
-El Incremento de la secreción de cloro por las células de la cripta intestinal
16. ¿Cuál de las siguientes alternativas es una característica de la secreción
exocrina del páncreas?
-Tiene una baja concentración de Cl- respecto al plasma
17. Una madre lleva a su hijo de dos años de edad a la sala de urgencias, estresada
porque el niño deglutió una moneda de 10 céntimos mientras la familia cenaba en
un restaurante. El médico observa mediante fluoroscopía que la moneda se halla
en el estómago y asegura a la madre que la moneda se eliminará con las heces.
El médico recomienda utilizar la respuesta fisiológica que permitirá la evacuación
de la moneda del estómago al intestino ………….…..
-Son los movimientos de mezcla y trituración
-. Es provocada por el ayuno
18. Las estructuras en el hígado que permite que los productos metabólicos unidos a
proteínas tengan acceso a las membranas basolaterales de los hepatocitos,
son….. -Las fenestras sinusoidales
19. La composición de la bilis es modificada conforme fluye por los conductillos
biliares. Durante este tránsito se espera que aumente la concentración de…….
-Monómeros de ácido biliar
-Ig A
20. Se mide experimentalmente el contenido gástrico de dos personas. La persona
“A” tiene alto contenido de grasa y la persona “B” tiene un contenido hipertónico
¿Cuál de las siguientes es correcto respecto al vaciamiento gástrico?
- Hay ralentización del vaciado gástrico en ambos casos
21. El examen endoscópico de un paciente con hipertensión portal grave revela
venas tortuosas que sobresalen hacia la luz del esófago. El paciente recibe
tratamiento quirúrgico mediante la colocación de una derivación que conecta la
vena porta a la vena cava. Después de la operación el riesgo de encefalopatía y
el
riesgo de sangrado de várices ……………..
-Aumentará/disminuirá
22. Un paciente varón de 18 años de edad acude al médico para sus exámenes
de rutina. Sus resultados de laboratorio muestran un valor de bilirrubina
sérica de 4 mg/dl y una bilirrubina directa de 0,3 mg/dl. Las pruebas de
función hepática son normales. La alteración que explica mejor este caso es
por la deficiencia de
………………..
-Glucuronil transferasa
23. Un hombre de 57 años de edad es llevado a urgencias con hematemesis
masiva rojo brillante, a su llegada se halla inconciente con PA: 80/40 mm Hg y
FC: 124 lat/min. Luce ictérico con presencia de “arañas vasculares en el tórax
anterior y extremidades”, abdomen distendido con signo de oleada positiva.
Se encuentra esplenomegalia y pérdida de la masa muscular en
extremidades. La anastomosis vascular responsable del sangrado en este
paciente es
-Vena gástrica izquierda y vena ácigos
24. Un estudiante de medicina está comiendo un plato de comida a base de
champiñones, espárrago y salsa de soya. El sabor umami contenido en todos
estos alimentos actúa a nivel de los botones gustativos estimulando
………………..
-Un receptor acoplado a proteína G
25. Un hombre de 22 años de edad se presenta al médico con una historia de 1 año
de evolución caracterizado por dolor recurrente en fosa iliaca derecha y diarrea.
Manifiesta además pérdida de peso de 8 kg durante este periodo. La
colonoscopía revela múltiples lesiones en el ileon terminal y colon. La biopsia
de estas lesiones revela engrosamiento, inflamación y ulceración de la mucosa.
El diagnóstico más probable en este caso es…….
-Enfermedad de Crohn
26. Varón de 61 años que consulta por dolor retro esternal intenso desde hace 6
horas y después de vómitos intensos y repetidos; al examen se observa disnea,
cianosis, hipotensión y signos clínicos de shock. La radiografía simple de tórax
muestra neumomediastino. El líquido en el espacio pleural aspirado tiene alta
concentración de amilasa. ¿Cuál de las siguientes alternativas puede explicar
este cuadro clínico? -Rotura espontánea de esófago
27. La secreción del ácido en la célula parietal gástrica se lleva a cabo por una
ATPasa especifica que intercambia hidrogeniones (H+) del citosol por…..
-K +
28. En condiciones normales el ingreso de 600 ml de líquido es el estómago provoca
un aumento de presión intragástrica de unos 12 cm de H2O. Después de una
vagotomía (corte del nervio vago) es de esperar que el ingreso del mismo
volumen de líquido provoque lo siguiente: …………………………………
-Un aumento mayor de la presión
29. Una paciente de 30 años de edad es sometida a una cirugía en oído medio
derecho por un problema de otoesclerosis. Luego de la cirugía refiere alteración
en la percepción de sabores. Al evaluar el caso usted esperaría
encontrar……….
-Alteración en la sensación del gusto en los dos tercios anteriores de la
lengua
-Sensación del dolor, tacto y temperatura conservada en toda la lengua

30. ¿Cuál de las siguientes alternativas es correcta?


-Las sales biliares desconjugadas son absorbidas preferentemente en el
colon
31. En un paciente de 45 años de edad con colestasis biliar, se encuentra una
elevación de los niveles sanguíneos de fosfatasa alcalina hasta 3 veces la cifra
normal. ¿Cuál de las siguientes alternativas estará también elevada como
evidencia del daño de la vía biliar?
-Gamma glutamil transpeptidasa
32. Revisando la angiografía de un hombre de 70 años en estudio por aneurisma de
aorta abdominal el radiólogo informa de la presencia de una oclusión completa
de la arteria mesentérica inferior. El paciente se encuentra completamente
asintomático.
¿Cuál de las siguientes arterias se anastomosa a la sistema arterial
de la mesentérica inferior?
-Cólica media
33. Lactante de 3 meses de vida es atendido por presentar diarrea, se administra
una solución de glucosa y electrólitos por vía oral. La proteína de membrana
apical que explica la capacidad de esta solución para proporcionar aporte de
glucosa e hidratación es ………..
-SGLT-1
34. Paciente ha sufrido herida de bala en el abdomen, se le ha tenido que extirpar el
segmento medio y distal del ileon. En este caso la síntesis hepática de sales
biliares estará …..…..
-Incrementada por estímulo de la enzima colesterol 7 alfa hidroxilasa
35. Un varón de 75 años ingresa al consultorio por presentar ictericia marcada de
piel y las escleras. El estudio del paciente mostró que presentaba un tumor que
obstruía la totalidad del conducto hepático común. ¿Cuál de las siguientes
estructuras se encontrará dilatada en este paciente?
-Conductos de Hering
36. En un paciente con insuficiencia renal crónica, el déficit en la absorción de
calcio a nivel del enterocito se debe a lo siguiente:
-No se convierte la 25 hidroxicolecalciferol a 1,25 dihidroxicolecalciferol
37. Varón de 30 años es traído a emergencia por agresión abdominal con arma de
fuego (pistola) y es sometido a laparotomía exploratoria, observándose isquemia
del colon ascendente y parte del colon trasverso ¿la lesión de cuál de las
siguientes arterias explicaría esta isquemia?
d. Mesentérica superior
38. Respecto a las sustancias gastrointestinales que regulan la secreción pancreática;
marque la afirmación correcta:
b. La acetilcolina es capaz de estimular la secreción enzimática y de bicarbonato
del páncreas
39. Ante una lesión del X par craneal, ¿cuál de los siguientes músculos mantiene
conservada su función?:
b. Tensor del velo del paladar
40. Experimentalmente se utiliza atropina (anticolinérgico) para inhibir la secreción
de gastrina, sin embargo la secreción de esta hormona se sigue dando ante
estímulos vagales. Esta situación se explica porque la atropina:
d. No bloquea la acción del péptido GRP

41. Un varón de 50 años es sometido a extirpación del duodeno y parte proximal del
yeyuno. Esta situación ocasionaría la pérdida de las células ……….. ,
productoras de ………………… que estimula la secreción de bicarbonato por el
páncreas.
“S” / secretina
42. Se evalúa la expresión de la proteína Agrp en una persona con alteración del
apetito; lo correcto respecto a esta proteína es…..
La mutación del gen que la codifica produce adelgazamiento
43. Juana cae de la bicicleta y se fractura la región anterior del hueso maxilar superior
con compromiso de la fosa incisiva. Al examen físico de la región esperaría
encontrar alteración en la sensibilidad de la encía …………………
palatina anterior
44. Recién nacido es atendido por el neonatólogo y luego entregado a su madre para
dar de lactar; la madre al dar de lactar observa coloración azulada de labios,
acompañado de tos persistente, dificultad respiratoria y distención abdominal. Se
le intenta colocar una sonda nasogástrica pero esta retorna a la cavidad oral en
todos los intentos. ¿Cuál de las siguientes anomalías del desarrollo es el más
probable en este caso?
b. Atresia esofágica proximal con fístula traqueoesofágica distal l
45. ¿Cuál de los siguientes mecanismos ocurre durante la defecación?
En la posición de “cuclillas” el músculo puborrectal se halla relajado
46. Un paciente luego de un accidente sufre lesión del piso de la boca, se constata
daño del nervio “cuerda del tímpano”, en este caso se esperaría en
47. contrar disminución de la………………………….… de la lengua
Sensación del gusto en los dos tercios anteriores
48. ¿Cuál de las siguientes afirmaciones es la correcta sobre la gastrina?
Actúa en la célula diana mediante su receptor CCk tipo B
49. Al recibir un paciente con signos de hipovolemia y antecedente de trauma en
abdomen por accidente de tránsito, usted identifica radiológicamente: lesión de
primera vértebra lumbar y signos de lesión en páncreas; durante la cirugía se
observó pobre irrigación de asas intestinales. El vaso afectado es la arteria ……..
c. mesentérica inferior
50. Un paciente sufre de daño a nivel del cuello con lesión muscular en la región de la
faringe. En el examen físico se determina dificultad para la elevación de la faringe y
para el cierre del itsmo de las fauces. En este caso, probablemente esté afectado el
músculo:
c. palatofaringeo
51. Varón de 50 años a quien le realizan la curación de la segunda molar de la
arcada superior derecha. En un momento determinado, el paciente acusa de
intenso dolor de la pieza dentaria en tratamiento. La vía aferente del dolor
viaja a través del nervio …………
a. trigémino V2
52. La distención gástrica por los alimentos produce incremento de secreción de
HCl mediante la producción de ………….. que estimula a las célulasvía
proteína
………..
a. gastrina / parietal / Gq

53. Un niño de tres años llega a emergencia con disfagia (dificultad para tragar),
dolor retro esternal, salivación y llanto. Se sospecha de ingesta de cuerpo
extraño (moneda) en el esófago; al ser evaluado se constata en una radiografía
presencia de cuerpo extraño a nivel de C6 (6° vértebra cervical). El cuerpo
extraño estará suspendido a nivel del estrechamiento producido por………..
c. el músculo cricofaríngeo
54. La triada portal (arteria hepática, vena portal y conducto biliar común) está
contenida en el ligamento …….……… y derivan embriológicamente del ……
a. hepato duodenal / mesenterio ventral
55. Un paciente refiere no percibir algunos sabores, al examen físico se constata
alteración en la percepción de sabores y del dolor en el tercio posterior de la
lengua ¿Cuál de los siguientes nervios estará alterada en su función?
c. Glosofaríngeo (IX par)
56. En el caso de un paciente con gastrinoma (tumor productor de gastrina), la
presencia de úlceras duodenales y erosión de la mucosa gástrica, se debe
principalmente a…….
c. el exceso de HCl por estímulo de receptores CCK-B en la célula parietal
56. El reflejo entero gástrico se caracterizan por:
d. originarse debido a la distensión duodenal y presencia del quimo ácido
57. Mauricio tiene dificultad para deprimir el paladar y elevar la parte posterior de la
lengua. En este caso estará afectado un músculo, específicamente el músculo
…………….
b. extrínseco – palatogloso
58. En condiciones normales, el ingreso de 600 ml de líquido es el estómago provoca
un aumento de presión intragástrica de unos 12 cm de H2O. Después de una
vagotomía (corte del nervio vago) es de esperar que el ingreso del mismo volumen de
líquido ocasione
………………………………… de la presión
intragástrica.
c. un aumento mayor
59. La explicación fisiológica de presentar somnolencia de 30 minutos a 1 hora
después de ingerir alimentos, se explica por: a. Aumento del cloro intraluminal
e. Aumento de la alcalinidad sanguínea
60.Se presenta un paciente, el cual presenta un antecedente de tuberculosis
intestinal, por lo cual, se le resecó 80 cm de íleon distal. Desde el punto de vista
fisiológico, el paciente puede presentar una de las siguientes alteraciones: a.
Disminución de la secreción de Vitamina B12

e. Disminución de la absorción de ácido glicocólico


61. Un paciente es sometido experimentalmente a un fármaco que modifica el flujo
salival, obteniéndose un volumen de saliva de 288 ml en 6 horas. En este caso las
concentraciones de electrolitos y bicarbonato en la saliva obtenida varían de la
siguiente manera: a. ↑ Na+,
↓ K+, ↑ Cl-, ↑ HCO3-

1. b. ↓ Na+, ↓ Cl-, ↑ K+, ↓ HCO3-

62. Uno de los siguientes elementos debería hallarse con más probabilidad en el
esófago de un paciente que sufre de reflujo gastro esofágico…
a. Pepsina

63. Un paciente de 40 años cursa con anemia de 8g/dl, aqueja además de astenia y
sensación de hormigueo bilateral en los miembros inferiores, al examen se halla alteración
de la sensibilidad a la vibración y camina con ampliación de la base de sustentación. Uno
de los siguientes procedimientos sería de ayuda para el diagnóstico de este paciente:
a. Tomografía cerebral
b. Biopsia de la mucosa gástrica

64.Paciente de 60 años ingresa por caída hace 1 hora y pequeño hematoma en cuero
cabelludo, al examen físico ampliado se observa ictericia de piel y mucosas generalizada,
abdomen blando, se palpa estructura quística no dolorosa en hipocondrio derecho que
corresponde a vesícula biliar (signo de Courvoisier), en los exámenes de laboratorio se
halla niveles bajos en la formación de estercobilinógeno y urobilinógeno en heces,
incremento de la bilirrubina conjugada en la orina, elevación de fosfatasa alcalina y
gamma glutamil transpeptidasa séricas. El presente cuadro puede ser explicado por: a.
Reabsorción de hematoma
c.Carcinoma de la cabeza de páncreas
65. Un recién nacido presenta vómitos biliosos poco tiempo después de cada alimento.
Al preguntar a la madre sobre antecedentes, ella recuerda que tuvo polihidramnios
durante la gestación, pero un análisis de cariotipo fue normal. Una de las siguientes es la
causa más probable de estos hallazgos en el recién nacido: a. Enfermedad de
Hirschprung
e. Malrotación de la yema pancreática ventral
66.En un estudio de la secreción de hormonas gastrointestinales, sus concentraciones en
la vena porta se midieron durante perfusión luminal del intestino delgado con soluciones
de diversas magnitudes de pH. ¿Qué hormona aumentará en el plasma de la vena porta
durante perfusión a través del intestino con una solución de pH 3?
a. CCK
e. secretina
67.Paciente de 30 años que ingresa a causa de un traumatismo abdominal cerrado. En la
exploración se aprecia discreta palidez de piel y mucosas, auscultación pulmonar normal,
taquicardia de 120 /min. Discreta distensión abdominal y matidez en flancos; el
hematocrito, que era prácticamente normal al ingreso, disminuye a 30% a las tres horas.
En la Rx de tórax se objetiva fractura de las costillas 10-11 izquierdas. La causa más
probable de la anemización en este paciente es: a. traumatismo renal con hemorragia
retroperitoneal.
c. rotura de bazo con hemoperitoneo.
68. Revisando la angiotomografía de un hombre de 70 años en estudio por aneurisma
de aorta abdominal, el radiólogo le informa de la presencia de una oclusión completa
de la arteria mesentérica inferior. El paciente se encuentra completamente
asintomático. La oclusión de la arteria mesentérica inferior cursa de manera
asintomática en muchas ocasiones ya que el territorio que irriga puede recibir flujo
proveniente de la arteria:
a. cólica derecha
e. cólica media

69. En las patologías de esófago es importante conocer bien la anatomía esofágica. ¿Cuál
de las siguientes afirmaciones es correcta? a. El esófago tiene capa mucosa, muscular y
serosa

c. El esófago torácico pasa por detrás del cayado aórtico

70. A pesar de que pueda haber variaciones anatómicas, lo habitual es que el ciego
sea irrigado por una rama arterial que proviene de unas de las siguientes arterias:
a. Iliaca derecha

d. Mesentérica superior

71. Ante un paciente con una cirugía abdominal urgente, el informe operatorio señala
que se ha realizado una resección de todo el duodeno y del tercio proximal del yeyuno
manteniendo íntegros el estómago y todo el íleon, así como los dos tercios distales del
yeyuno. En el seguimiento nutricional del paciente ¿Qué vitamina o mineral presentará
con menor probabilidad una disminución de su absorción?
a. Cianocobalamina

72. ¿Cuál de las siguientes sustancias forma parte de la secreción biliar? a. Tripsina
Lecitina

73.¿De qué musculo forma parte el ligamento inguinal?


-Oblicuo externo del abdomen
74.¿Cuál de las siguientes enzimas está localizada en el borde en cepillo y juega un rol
en la digestión de proteínas?
e. Carboxipeptidasa A.
75. Una de los siguientes sustancias, NO sirve como un buen agente emulsificante:
a. Colesterol
76. La sustancia que estimula el crecimiento de la mucosa gástrica es:

a. Secretina

d. Gastrina

77.¿Cuál de las siguientes alternativas es una función de la colecistokinina?

a. Relajación de la vesícula para la salida de bilis

d. Secreción de enzimas pancreáticas

78.Con respecto a la anatomía del tronco celiaco, señale lo correcto a. El tronco celiaco
se origina de la cara posterior de la aorta abdominal
d. La hepática común que es una de sus ramas, participa en la irrigación del
estómago.
79. Con respecto a la anatomía del duodeno, marque la respuesta correcta: a. Tiene
una distribución en forma de “C”, que rodea la cola del páncreas
b. La 3ra porción duodenal está contenida en la pinza vascular aortomesentérica
80. En el íleon se absorbe aproximadamente el 95% dea través de la circulación
enterohepática.
a. agua
c. sales biliares
81. La estimula el mecanismo paracrino de la secreción de ácido clorhídrico.
a. histamina
82.En la digestión de proteinas,es el principal estímulo para convertir el
pepsinógeno en pepsina. a. la gastrina
b. el pH ácido
83. Con respecto a la somatostatina, marque lo correcto:
a. Es secretada por las células S del intestino
Interviene en la fase intestinal de la secreción gástrica
84. En pecten anal, es una estructura comprendida entre: a. la línea pectínea y los
senos anales
d. la línea anocutánea y la línea pectínea

85.¿Cuál de las siguientes alternativas es una proenzima pancreática? a. Tripsina

1. b. Elastasa
2. c. Quimotripsinógeno
3. d. Amilasa
4. e. Procarboxipeptidasa C
86. En este paciente, el bloqueo farmacológico de los receptores H2 en la mucosa gástrica:
a) No tiene efecto sobre la secreción de ácido inducida por el vago
b) Evita la activación de adenilciclasa por gastrina
c) Inhibe la secreción de ácido inducida por gastrina y mediada por el vago
d) Causa un aumento en el transporte de potasio por las células parietales gástricas
Se validó la A :)
87. Si se considerara una gastrectomía total para curar la gastritis del paciente, cuál
de las siguientes sustancias ya no se produciría:
a) Gastrina
b) Quimiotripsina
c) Amilasa
d) Pepsinógeno

88. Un paciente hipertenso está tomando un medicamento bloqueador de receptores


alfa 1 adrenérgicos (prazosina) y como efecto secundario se queja de:
d) Lo escaso que es el medicamento
e) No tiene problemas en la salivación
c) Hiposalivación
d) Hipersalivación

89. Con respecto a las lesiones y enfermedades de la boca, marque lo correcto:


f) La leucoplasia se desprende al roce
b) la eritroplasia puede degenerar en adenocarcinoma
c) El muguet oral es una enfermedad bacteriana en inmunodeprimidos
d) la eritroplasia debe ser biopsiada
NOTA: fue validada la opción B ya que no es motivo del curso que sepamos el puto
cáncer.

90. En este paciente, se puede asumir que la pancreatitis ha sido ocasionada por
una disminución en el efecto de:
g) Amilasa
h) Lipasa
c) Inhibidor de la tripsina
d) Entercinasa
91.Un efecto secundario en el estómago por la acción de la secretina es:
i) Disminución en la liberación de pepsinógeno
b) Menor actividad de la pepsina
c) Mayor paso de bicarbonato a sangre periférica
d) Aumento en la producción de factor intrínseco
92. Dentro de los factores protectores de la mucosa gástrica se pueden mencionar
múltiples protagonistas. Uno de ellos es:
j) CCK
k) Gastrina
c) Receptor muscarínico
d) Pepsina
93. La saliva puede tener una variedad de electrolitos en su composición. Entre ellos el
cloro, respecto al cual se puede afirmar:
l) Su mayor concentración se consigue con el flujo bajo
b) Su concentración no llega a ser tan alta como en el plasma
c) Con flujo alto su concentración es mayor que la del plasma
d) Su menor concentración se alcanza con flujo alto
94. En el síndrome de boca seca o síndrome de Sjogren, una de las
complicaciones asociadas es:
a) caries
b) Disminución de la acidez gástrica
c) Aumento en de la producción de saliva
d) Aumento del pH bucal
95. Estimula la producción de
saliva:
a) Vasodilatación periglandular
b) Atropina
c) Fatiga o cansancio
d) Expresión de miedo
96. El omeprazol actúa sobre la membrana de la célula
m) Basolateral/principal
n) apical/principal
o) Basolateral/parietal
d) apical/parietal
97. Para protegerse del entorno ácido, el Helicobacter pylori se autogenera un entorno
de pH menos ácido alrededor suyo, gracias a una enzima que alcaliniza su entorno
local mediante la conversión de:
a) urea en NH3
b) H2O y CO2 en ácido carbónico
c) NH3 en urea
d) H2CO3 en bicarbonato
98. La anemia perniciosa destruye las células:
p) mucosas del cuello
b) oxínticas
c) principales
d) mucosas superficiales
99. La célula mucosa del cuello gástrico produce:
a) Moco
b) ácido clorhídrico
c) pepsinógeno
d) Factor intrínseco
100.El aumento en la acidez del estómago producido principalmente por la
infección de Helicobacter pylori se debe a la disminución de:
a) Somatostatina
b) Bicarbonato por las glándulas de Brunner
c) Secretina
d) Colecistoquinina
101. De las siguientes sustancias secretadas por los órganos de este paciente, la
más alcalina es la secreción:
a) pancreática
b) Esofágica
c) Yeyunal
d) Salival
102.En cuanto a la gastritis de este paciente, se encontró que era producida por la
bacteria Helicobacter pylori. Esta bacteria sobrevive en el medio ácido del estómago
gracias a:
a) ácido clorhídrico
b) Toxina CagA
c) Ureasa
d) Jugo pancreático
103.La lengua está recubierta por epitelio:
c) pseudoestratificado columnar no queratinizado
b) plano estratificado no queratinizado
c) pseudoestratificado columnar ciliado
d) plano estratificado queratinizado
104.El esfínter anal interno tiene musculatura …….. y tiene control ……..
d) lisa / voluntario
b) lisa / involuntario
c) esquelética / simpático
d) esquelética / parasimpático

19) La arteria Aorta proporciona la irrigación al tubo digestivo ¿cuál de las


siguientes arterias proporciona la irrigación al ángulo cólico derecho?
a) mesentérica superior
b) mesentérica inferior
c) frénica inferior
d) tronco celiaco

20) Paciente de 26 años que le cuenta en su historia clínica que cada vez que almuerza a
los 20 min tiene deseo de defecar, le comenta que su hijo de 1 mes le pasa lo mismo
pero más intenso. Esto se explica por el reflejo …….., el cual está …… en el paciente
a) colicoileal / normal
b) colicoileal / alterado
c) gastrocólico / normal
d) gastrocólico / alterado

21) La región del estómago que se comunica con el duodeno es la


a) pilórica
b) cardias
c) cuerpo
d) fórnix

22) Acude a consulta un px que fue diagnosticado de úlcera péptica 3 días antes. Luego
de múltiples pruebas diagnósticas se concluye que el paciente presenta un tumor
secretor de gastrina, ¿cual de las siguientes situaciones estará incrementada?
a) distensión gástrica
b) inhibición del vaciado gástrico
c) secrecion de acido clorhidrico
d) inhibición de la secreción de pepsinógeno
23) En el sistema digestivo, el control del apetito está dado por un complejo sistema de
sustancias y órganos integradores, los cuales regulan la ingesta de alimentos. La
…… es una sustancia orexígena y es sintetizada por el ……
a) leptina / estómago
b) felina / intestino
c) leptina / estómago
d) grelina / estómago

24) Con respecto a la actividad eléctrica del sistema digestivo, marque la


alternativa correcta
a) corresponden a potenciales de acción que están presentes de forma continua
y le dan capacidad de peristalsis autónoma al sistema digestivo
b) la frecuencia de las ondas lentas NO se ve influenciada por la actividad neural ni
las
hormonas gastrointestinales
c) en el estómago las ondas lentas se dan en una frecuencia de 6 x min
d) las ondas lentas son cambios lentos y ondulantes del potencial en reposo
e) la frecuencia de las ondas lentas va de 6 a 12 ondas por minuto

25) Ante una lesión del IX pc, el músculo …… se altera en su función


a) palatogloso
b) estilofaríngeo
c) palatofaríngeo
d) constrictor superior

26) Un varón de 50 años es sometido a extirpación del duodeno y parte proximal del
yeyuno. La pérdida de estímulo hormonal en el páncreas para la secreción
enzimática se explica por la pérdida de células
a) parietales, productoras de factor intrínseco
b) K productoras de factor intrínseco
c) M productora de CCK
d) I productora de CCK

27) Marque la respuesta correcta:


A. El bronquio derecho constituye una de las estrecheces del esófago
B. Todos los órganos del sistema digestivo tienen capa serosa
C. La pared gástrica en el fondo es más delgada que en el cuerpo y antro
D. El esfínter de Oddi rodea a la papila menor duodenal

28) Marque la respuesta correcta en relación a la gastrina:


A. Al distenderse el estómago, se inhibe su producción.
B. Se estimula por la liberación de noradrenalina
C. Las células G son las productoras y se encuentran principalmente en el antro
gástrico
D. Las células G se encuentran principalmente en el fondo gástrico

29) Para poder morder una manzana, es necesario usar el siguiente músculo:
A. Milohiodeo
B. Tensor del paladar
C. Orbicular de los labios
D. Buccinador
30) Sustancia que inhibe la secreción y la motilidad del estómago prolongando el tiempo
de digestión:
A. Enteroglucagon.
B. Polipéptido pancreático
C. Péptido 1 similar al glucagón (GLP-1).
D. Péptido insulinotrópico dependiente de la glucosa (GIP).

31) El nacimiento de la arteria mesentérica superior se puede encontrar en cuál de


los cuadrantes abdominales:
A. Hipocondrio derecho
B. Hipogastrio
C. Epigastrio
D. Mesogastrio
32) Entre las múltiples causas de la Enfermedad por Reflujo Gastroesofágico, se puede
considerar también a una alteración en las del esfínter esofágico inferior:
A. Ondas secundarias
B. Contracciones tónicas
C. Ondas lentas
D. Glándulas subesofágicas

33) Producto de la alimentación, se producen diversas sustancias peptídicas, cininas


y bradicininas, las cuales permiten que:
A. Se produzca neovascularización en los territorios de las arterias abdominales
B. La acción de la lipasa pancreática se vea incrementada
C. El consumo de O2 del intestino aumente ligeramente
D. El flujo sanguíneo intestinal aumente hasta 8 veces

34) El dolor periumbilical o epigástrico en el inicio de una apendicitis aguda se debe a:


A. Estímulo del nervio vago.
B. Íleo secundario.
C. Irritación del peritoneo parietal.
D. Estímulo del sistema simpático.

35) El aumento en la actividad motora de la pared gástrica genera un aumento en


los niveles locales de qué sustancia en la microvasculatura:
A.Adenosina
B. Colecistoquinina CCK
C. Endotelina
D. Gastrina

36) ¿Cuál de los siguientes péptidos inhibe el vaciamiento gástrico?


A. Colecistoquinina
B. Péptido inhibidor gástrico
C. Motilina
D. Gastrina
37) Los músculos de la masticación que producen la retropulsión de la mandíbula son:
A. temporales [mas seguro]
B. maseteros
C. milohioideos
D. pterigoideos
38) En relación a la fisiología gástrica, marque lo correcto:
A. la cimetidina actúa en la región basolateral de la célula parietal
B. la marea alcalina se debe al paso de bicarbonato través de la membrana apical
de la célula principal
C. el cloro difunde hacia el exterior por la la región basolateral de la célula parietal
D. la salida de hidrogeniones a la luz es por difusión facilitada

39) Durante el sueño, la concentración de bicarbonato en la saliva:


A. Se eleva a niveles mayores que los del plasma
B. Aumenta
C. No tiene efecto
D. Disminuye

40) La secreción de saliva es importante en la fisiología digestiva. Su concentración


de potasio llega a ser menor que la del plasma cuando su secreción tiene un flujo:
A. Intermedio
B. Nunca
C. Bajo
D. Alto

42) Respecto a las glándulas salivales, marque lo incorrecto:


A. la glándula parótida produce secreción serosa
B. la glándula sublingual drena a través de conducto de Wharton
C. La glándula parótida drena a través del conducto de Stenon
D. la glándula sublingual tiene forma de garfio

43) Con respecto a la saliva, marque la respuesta correcta:


A. será hipertónica cuando el flujo es bajo
B. a mayor flujo, menor concentración de Na
C. a mayor flujo, menor concentración de cloro
D. el sistema simpático estimula su secreción
CI 3
44) En relación a la circulación hepática, marque lo correcto:
a) Los sinusoides hepáticos transportan sangre mixta
b) La vena porta proporciona el 50% de sangre al hígado
c) La vena porta se forma a partir de la vena esplénica y la mesentérica inferior
d) La arteria hepática deriva de la mesentérica superior

45) Dentro de las funciones de las células de Ito, marque lo incorrecto:


a) Sintetizan colágeno
b) Almacenan vitamina A
c) Se les llama células estrelladas
d) Pueden fagocitar patógenos y actúan como presentadoras de antígeno
46) Paciente con tumor neuroendocrino productor de secretina, debido a lo cual se puede
esperar que su secreción pancreática, comparada con la de una persona sana de bajo
flujo, tenga una concentración de:
a) Sodio aumentada
b) Igual
c) Bicarbonato aumentada
d) Potasio disminuida
47) El GALT se localiza en:
a) Lámina propia
b) submucosa
c) borde en cepillo
d) superficie de criptas de Lieberkühn

48) En relación a la histología hepática, marque lo correcto:


a) la zona 1 se afecta rápidamente en estados de hipovolemia y shock
b) La zona 1 se encuentra cercana a la vena central lobulillar
c) La zona 3 se encuentra más cerca a la vena central lobulillar
d) La zona 3 se encuentra más cerca al eje menor del acino hepático
49) El acino pancreático difere con el de las glándulas salivales en:
a) Contiene células centroacinares
b) No produce secreción serosa
c) El páncreas produce principalmente secreción mucosa
d) No tienen diferencias
50) Durante la digestión de las grasa, para que la lipasa actúe adecuadamente se requiere
que el pH aumento en la luz intestinal, lo cual es logrado, entre otros, por la secreción
de las células:
a) Del conducto interlobulillar
b) Centroacinares
c) Acinares
d) Alfa
51) La secreción de la colecistoquinina (CCK) se produce en la fase:
a) intestinal
b) En las 3 por igual
c) gástrica
d) Cefálica
52) ¿Por cuál de las siguientes células es secretada principalmente la pro
enzima procarboxipeptidasa?
a) Acinares del páncreas
b) Epiteliales del duodeno
c) Ductales del páncreas
d) Centro Acinares del páncreas
53) Una mujer de 43 años dolor en hipocondrio derecho e icterica. En la ecografía se
evidencia cálculos biliares. Estos cálculos lo más probable es que se encuentren
localizados en:
a) conducto colédoco
b) Conducto cístico
c) Vesícula biliar
d) Conducto pancreático secundario
ECU 1:
Estudiante de 21 años sufre de gastritis aguda ocasionada por comer en lugares poco
higenicos. Suele consumir caramelos (“chupar”) mientras esta en base hasta la tarde.
También toma regular cantidad de leche (grasa, lactora, proteinas), pues le calma el
dolor y el ardor que sitnete por la gastritis (tiene dispepsia y cuando toma la leche se le
pasa).
Incluso cuando puede, se toma dos vasos de agua frita y le calma la molestia. Ha
decido ir al medico para tratarse, pues ya no soporta el dolor, el cual esta seguro que
los síntomas se producen por elevada producion de HCl en el estomago, y por ello le ha
recetado ranitidina
1.1) El consumir caramelos eleva los niveles en sangre de una hormona cuya función es la
estimulación de las células.
- Beta del páncreas por GIP el cual es una incretina y por consiguiente
estimula las células pancreáticas

1.2) Consumir caramelos indirectamente actica la via:


-POMP/ CART saciedad

1.3) Consumo de leche produce indirectamente


- CCK inhibición del vaciamiento gástrico mayor tonicidad del esfínter pilórico

1.4) Cuando el px toma dos vasos de agua, genera indirectamente un aumento en la


liberación de:
- vaso de agua distención → g astrina → secreción de HCl

1.5) El uso de ranitidina bloquea el receptor H2 de la histamina en las células parietales, la


histamina llega a estas células por:
- histamina es una hormona paracrina por → difusión
**endocrina es por via hematógena y si fuera neuroendorina es por un NTs

1.6) Aumenta la secreción salival:


- noradrenalina a través de los receptores Beta 2
1.7) En este paciente con gastritis aguda debida a una alta producción de ácido clorhídrico,
sería lógico esperar que el píloro tenga un tono muscular:
- primero la secretina
- luego CCK
**ambas reguladores del HCl, Gatritis aguda debido a una alta producción de HCL
piloro estará aumentado (por la CCK)
1.8) Debido al uso de ranitidina, los valores de somatoestina en sangre:
- ranitidina disminuye acción de gastrina se quiere secretar mas no
actúan los inhibidores como la somatoestina somatoestina disminuye
1.9) El uso de atropina en este paciente:
- Inhibirá la acción de las prostaglandinas
- Aumentará la producción de ácido clorhídrico
- Disminuirá la acción del receptor CCK-B
-Aumentará el pH del estómago
ECU 2:
Niño de sexo masculino de 2 años de edad, sufre de estreñimiento desde el
nacimiento (1 deposición cada 3-4 días). Madre menciona que le estimula la
defecación con un termómetro rectal, y continuo uso de enemas y laxantes. Desde
hace 6 meses comienza con vómitos postprandiales. Los síntomas aumentan en
frecuencia y magnitud y están en relación con los episodios de estreñimiento. No
refiere fiebre, tos, diarrea ni lesiones cutáneas. Al examen físico presenta regular
estado general, luce deshidratado. Abdomen distendido, blando, depresible e indoloro.
No se palpan masas abdominales. Se permeabiliza el canal anal con termómetro
rectal, encontrando cierta resistencia. Salida de material fecal mal oliente en regular
cantidad. Exámenes de laboratorio: hemograma normal. Signos inflamatorios de fase
aguda negativos. Alcalosis metabólica leve en sangre
venosa. Radiografía con enema baritado muestra recto y colon sigmoides dilatados
(megacolon). Biopsia profunda: ausencia de células ganglionares en la muestra
enviada. Se realiza cirugía correctiva.

2.1) Durante la fase esofágica de la deglución, para un bolo alimenticio determinado, a


medida que avanza el bolo la fuerza de la contracción se hace más:
- hiperpolarizado
- fuerte
- dependiente de Ach
- debil
2.2) Cuando este paciente ingiera sus alimentos, se espera que al momento de pasar el
bolo alimenticio por el esfínter esofágico superior, la presión intraesofágica
disminuya en:
- la porción proximal al bolo
- el tercio medio del esófago
-el cardias
- el lugar donde se contraiga la muscular propia
2.3) Al examinar la orofaringe del paciente, uno puede hallar fácilmente la amígdala
palatina, pues esta se encuentra inmediatamente detrás del músculo:
-Palatogloso
- Palatofaringeo
- Hiogloso
- Elevador del velo del paladar

2.4) Con respecto a la defecación en este caso, marque la respuesta correcta:


- En posición de cuclillas, el músculo puborectal genera un ángulo más agudo
en el recto
- El sigmoides y el recto están inervados por el nervio vago
- La sensación de defecar sólo se da cuando el recto es ocupado por
heces y alcanzado el 80% de su capacidad
-El esfínter anal comprometido tiene inervación autónoma

2.5) En este paciente [hirschsprung] se considera que está abolido el reflejo:


- Coloileal
-Rectoesfinteriano
- Gastrocólico
- Relajación receptiva

2.6) No se espera que sea causa del vómito:


-Ayuno prolongado
- Estimulación faríngea y del glosofaríngeo
- Irritación de la mucosa gástrica
- Dolor intenso

ECU 3:
Paciente de 54 años con antecedentes de alcoholismo, gastritis crónica,
tabaquismo

También podría gustarte